Vous êtes sur la page 1sur 1745

AIR 2008 SUPREME COURT 399 "Lucknow Development Authority v.

Krishna Gopal
Lahori"
(From : Allahabad)
Coram : 2 Dr. A. PASAYAT AND LOKESHWAR SINGH PANTA, JJ.
Civil Appeal No. 5112 of 2007 (arising out of SLP (C) No. 12446 of 2005), D/- 2 -11
-2007.
Lucknow Development Authority v. Krishna Gopal Lahori and Ors. @page-SC400
(A) Land Acquisition Act (1 of 1894), S.23 - ACQUISITION OF LAND - Acquisition
compensation - Large area acquired - Rates fixed for small plots - No absolute rule that
they be kept out of consideration.
Where large area is the subject-matter of acquisition, rate at which small plots are sold
cannot be said to be a safe criteria. It cannot, however, be laid down as an absolute
proposition that in such cases, the rates fixed for the small plots cannot be the basis for
fixation of the rate. For example, where there is no other material it may in appropriate
cases be open to the adjudicating Court to make comparison of the prices paid for small
plots of land. However, in such cases necessary deductions/ adjustments have to be made
while determining the prices. (Paras 15, 16)
(B) Land Acquisition Act (1 of 1894), S.23 - ACQUISITION OF LAND - SALE -
Acquisition compensation - Determination - Comparable sale - Essentials for
constituting.
For sale to be considered as comparable sale it is essential that the sale is within a
reasonable time of the date of notification u/S. 4(1); it should be a bona fide transaction;
it should be of the land acquired or of the land adjacent to the land acquired ; and it
should possess similar advantages. (Para 18)
(C) Land Acquisition Act (1 of 1894), S.23 - ACQUISITION OF LAND - Acquisition
compensation - Deductions towards development charges - For agricultural land or
undeveloped land which has potential value for housing or commercial purposes -
Normally the deduction is 1/3rd amount of compensation - It may vary depending on its
nature, location, expenditure involved etc. - Fact that an area is developed or adjacent to a
developed area - Does not ipso facto make every land situated in area also developed,
particularly, when vast tracts are acquired. (Para 22)
Cases Referred : Chronological Paras
2004 AIR SCW 75 : AIR 2004 SC 1185 (Ref.) (Pt.C) 24
2004 AIR SCW 2089 : AIR 2004 SC 2006 : 2004 All LJ 1438 (Ref.) (Pt. C) 24
(2004) 12 SCC 425 (Ref.) (Pt. C) 24
2003 AIR SCW 1491 : AIR 2003 SC 1987 (Ref.) (Pt. B) 20
2002 AIR SCW 4644 : AIR 2003 SC 202 (Ref.) (Pt. C) 23
AIR 1989 SC 1222 17
AIR 1984 SC 892 15
AIR 1977 SC 1560 15
AIR 1971 SC 2015 15
AIR 1959 SC 429 (Rel. on) (Pt. B) 19
AIR 1939 PC 98 17
Umesh Chandra Sr. Advocate, Krishna Chandra, S. A. Kashif and Shakil Ahmed Syed,
for Appellant; Jayant Bhushan Sr. Advocate Ashwini Garg and Vijay Kumar, for
Respondents.
* F. A. No. 54 of 1998, D/- 9-2-2005 (All).
Judgement
1. Dr. ARIJIT PASAYAT, J.:-Leave granted.
2. Challenge in this appeal is to the judgment of a Division Bench of the Allahabad High
Court, Lucknow Bench dismissing the appeal filed by the appellant under Section 54 of
the Land Acquisition Act, 1894 (in short the 'Act') read with Section 96 of the Code of
Civil Procedure, 1908 (in short 'CPC').
3. In the First Appeal challenge was to the award dated 18.2.1998 passed by the Presiding
Officer, Nagar Mahapalika Tribunal, Lucknow in a reference under Section 18 of the Act
in Land Case No.746 of 1991 titled Krishna Gopal Lahoti v. State of U.P.
4. The factual background in a nutshell is as follows:
A large area of land measuring 194 bigha 19 biswa 14 biswansi and 14 kachwansi
situated in village Purania and Mahibullapur was sought to be acquired by appellant-
Lucknow Development Authority under the Housing and Development Scheme known as
"Timber Nagar Avasiya Yojana". Khasra Plot No. 379 measuring 8 bigha, and Khasra
Plot No. 394 measuring 2 bigha, 8 biswa 15 biswansi situated at village Mahibullapur and
belonging to the claimants Krishna Gopal Lahoti, Sharad Kumar Lahoti, Sunil Kumar
Lahoti and Sudhir Kumar Lahoti were also acquired under the said scheme. The relevant
notification under Section 4 was issued on 26.3.1986. The notification under Section 6 of
the Act was published on 28.5.1986. The possession of the acquired land was taken on
17.12.1986 and Award under Section 11 of the Act was made on 27.5.1988 by the Special
Land Acquisition Officer. The Special Land Acquisition Officer in his Award under
Section 11 of the Act determined the market value of the land in question at the rate of
Rs. 2.20 per sq. ft.
@page-SC401
5. Aggrieved by the aforesaid Award, reference under Section 18 was preferred by the
land owners, inter alia, stating that adjoining to the land in question, there is Lucknow-
Sitapur Highway and nearby the acquired land there are number of colonies such as
Aliganj Colony, Kapurthala Complex, P and T Colony, Arif Complex, Public Service
Commission and Office of Geological Survey of India.
6. According to the landowners, the land in question has great potential value and the
market value as determined by the Special Land Acquisition Officer is quite inadequate.
The market value of the land at the rate of Rs. 60/- per sq.ft. was claimed by the
respondents besides statutory benefit under Act 68 of 1984.
7. The Lucknow Development Authority and the State of U.P. filed written statements
separately. It was stated that the compensation as determined and awarded by the Special
Land Acquisition Officer is quite adequate and the claimants are not entitled to the
benefits of the provisions of Act 68 of 1984. It was stated that claim petition is barred by
time. It is also barred by the provisions of the Urban Land Ceiling Act, 1976 (in short
'ULC' Act) and by the provisions of Section 31 of the Act.
8. Both the parties led oral and documentary evidence.
9. The learned Tribunal could not find any substance in the pleas raised by the appellants
regarding claim being barred under various heads as alleged in the written statements and
all the issues were decided in negative against the appellant. The Tribunal further found
that the claimants are entitled to the benefit of provisions of Act of 68 of 1984 and on the
basis of the evidence on record, the Tribunal determined the market value of the land at
Rs. 6/- per sq. ft. and accordingly compensation was awarded by the impugned Award.
10. Against the Award, the First Appeal was filed before the High Court. Primarily, it was
contended before the High Court that the Tribunal had not properly evaluated the
evidence on record and wrongly placed reliance on a sale deed relating to a small piece of
land. It was also submitted that without any proper appreciation of materials on record
the compensation was enhanced.
11. Stand of the respondents before the High Court was that there was no illegality in the
Award passed by the reference court. It was submitted that the land was situated near
densely populated area having great potential value and the appellate authority is selling
the same land at the rate of Rs. 300/- per sq. ft. The reference court on the basis of oral
and documentary evidence has awarded compensation at the rate of Rs. 6/- per sq. ft.
along with other benefits as provided under the Act. The High Court found that the
claimants had filed number of sale deeds of varying rates ranging between Rs.10/- per sq.
ft. to Rs. 5/- per sq. ft. but the sale deed relating to the Plot No.166 situated at
Mahibullahpur was relied upon by the Tribunal and the reasons for enhancing the
compensation were assigned which according to the High Court did not call for any
interference. The High Court did not find any substance in the plea of the appellant that
the sale deed (Ex.C-38) was unduly relied upon by the Tribunal. It was pointed out that
the sale deed is related to a very small piece of land as against the large area of more than
10 bighas involved in the present case. The High Court referred to certain decisions of
this Court to hold that while determining the market value of the land, the potentiality of
the land is a very material consideration and several factors like location of the land, its
surroundings, available facilities thereon in the vicinity, nature of the land have to be
taken into account. The High Court also found that there was no similarity between the
land which was the subject matter of dispute in Land Acquisition Case No. 204 of 1992
where the rate fixed was Rs.1.85 per sq. ft.
12. The High Court further found that two sale deeds (Ga 26 and Ga 27) reflected that the
rate was Rs. 3/- per sq. ft. However, instances were referred to in holding that the market
value is much higher. After granting deduction of 25% on account of expenses to be
incurred towards plotting and development charges, the rate was fixed at Rs. 6/- per sq. ft.
Therefore, the High Court did not find any substance in the stand that the deduction
should be at least 40% and not 25% as done. Accordingly, appeal as noted above was
dismissed.
13. In support of the appeal, learned counsel for the appellant reiterated the stand taken
before the High Court.
14. In response, learned counsel for the respondents submitted that three sale deeds
namely, C-38, 39 and 40 clearly show that
@page-SC402
rate is much higher. It was pointed out that this Court has depending on the facts of the
case, allowed deductions ranging between 20% to 33%. That cannot be a hard and fast
rule and in fact it would depend upon various factors.
15. Where large area is the subject-matter of acquisition, rate at which small plots are
sold cannot be said to be a safe criteria. Reference in this context may be made to three
decisions of this Court in The Collector of Lakhimpur v. Bhuban Chandra Dutta (AIR
1971 SC 2015); Prithvi Raj Taneja (dead) by LRs. v. The State of Madhya Pradesh and
Anr. (AIR 1977 SC 1560) and Smt. Kausalya Devi Bogra and Ors. etc. v. Land
Acquisition Officer, Aurangabad and Anr. (AIR 1984 SC 892).
16. It cannot, however, be laid down as an absolute proposition that the rates fixed for the
small plots cannot be the basis for fixation of the rate. For example, where there is no
other material it may in appropriate cases be open to the adjudicating Court to make
comparison of the prices paid for small plots of land. However, in such cases necessary
deductions/adjustments have to be made while determining the prices.
17

. In the case of Suresh Kumar v. Town Improvement Trust, Bhopal (1989 (1) SVLR (C)
399) in a case under the Madhya Pradesh Town Improvement Trust Act, 1960 this Court
held that the rates paid for small parcels of land do not provide a useful guide for
determining the market value of the land acquired. While determining the market value of
the land acquired it has to be correctly determined and paid so that there is neither unjust
enrichment on the part of the acquirer nor undue deprivation on the part of the owner. It is
an accepted principle as laid down in the case of Vyricherla Narayana Gajapatiraju v.
Revenue Divisional Officer, Vizagapatam (AIR 1939 PC 98) that the compensation must
be determined by reference to the price which a willing vendor might reasonably expect
to receive from the willing purchaser. While considering the market value, disinclination
of the vendor to part with his land and the urgent necessity of the purchaser to buy it must
alike be disregarded, neither must be considered as acting under any compulsion. The
value of the land is not to be estimated as its value to the purchaser. But similarly this
does not mean that the fact that some particular purchaser might desire the land more
than others is to be disregarded. The wish of a particular purchaser, though not his
compulsion may always be taken into consideration for what it is worth. Section 23 of the
Act enumerates the matters to be considered in determining compensation. The first
criterion to be taken into consideration is the market value of the land on the date of the
publication of the notification under Section 4(1). Similarly, Section 24 of the Act
enumerates the matters which the Court shall not take into consideration in determining
the compensation. A safeguard is provided in Section 25 of the Act that the amount of
compensation to be awarded by the Court shall not be less than the amount awarded by
the Collector under Section 11. Value of the potentiality is to be determined on such
materials as are available and without indulgence in any fits of imagination.
Impracticability of determining the potential value is writ large in almost all cases. There
is bound to be some amount of guess work involved while determining the potentiality.
AIR 1989 SC 1222

18. It can be broadly stated that the element of speculation is reduced to minimum if the
underlying principles of fixation of market value with reference to comparable sales are
made:
(i) when sale is within a reasonable time of the date of notification under Section4(1);
(ii) it should be a bona fide transaction;
(iii) it should be of the land acquired or of the land adjacent to the land acquired; and
(iv) it should possess similar advantages.
19. It is only when these factors are present, it can merit a consideration as a comparable
case (See The Special Land Acquisition Officer, Bangalore v. T. Adinarayan Setty (AIR
1959 SC 429).
20

. These aspects have been highlighted in Ravinder Narain and Anr. v. Union of India
(2003 (4) SCC 481). 2003 AIR SCW 1491

21. The deduction to be made towards development charges cannot be proved in any
strait-jacket formula. It would depend upon the facts of each case.
22. It is well settled that in respect of agricultural land or undeveloped land which has
potential value for housing or commercial purposes, normally 1/3rd amount of
compensation has to be deducted out of the amount of compensation payable on the
@page-SC403
acquired land subject to certain variations depending on its nature, location, extent of
expenditure involved for development and the area required for roads and other civic
amenities to develop the land so as to make the plots for residential or commercial
purposes. A land may be plain or uneven, the soil of the land may be soft or hard bearing
on the foundation for the purpose of making construction; may be the land is situated in
the midst of a developed area all around but that land may have a hillock or may be low-
lying or may be having deep ditches. So the amount of expenses that may be incurred in
developing the area also varies. A claimant who claims that his land is fully developed
and nothing more is required to be done for developmental purposes, must show on the
basis of evidence that it is such a land and it is so located. In the absence of such
evidence, merely saying that the area adjoining his land is a developed area, is not
enough particularly when the extent of the acquired land is large and even if a small
portion of the land is abutting the main road in the developed area, does not give the land
the character of a developed area. In 84 acres of land acquired even if one portion on one
side abuts the main road, the remaining large area where planned development is
required, needs laying of internal roads, drainage, sewer, water, electricity lines,
providing civil amenities etc. However, in cases of some land where there are certain
advantages by virtue of the developed area around, it may help in reducing the percentage
of cut to be applied, as the developmental charges required may be less on that account.
There may be various factual factors which may have to be taken into consideration while
applying the cut in payment of compensation towards developmental charges, may be in
some cases it is more than 1/3rd and in some cases less than 1/3rd. It must be
remembered that there is difference between a developed area and an area having
potential value, which is yet to be developed. The fact that an area is developed or
adjacent to a developed area will not ipso facto make every land situated in the area also
developed to be valued as a building site or plot, particularly, when vast tracts are
acquired, as in this case, for development purpose.
23

. The aforesaid aspects were highlighted in Kasturi and Ors. v. State of Haryana (2003 (1)
SCC 354). 2002 AIR SCW 4644
24

. A reference may also be made to what has been stated in Kiran Tandon v. Allahabad
Development Authority and Anr. (2004 (10) SCC 745); State of West Bengal v.
Kedarnath Rajgarhia Charitable Trust Estate (2004 (12) SCC 425) and V. Hanumantha
Reddy (dead) by LRs. v. Land Acquisition Officer and Mandal R. Officer (2003 (12)
SCC 642). 2004 AIR SCW 2089
2004 AIR SCW 75

25. Keeping in view the general principles and the factual scenario as evident from the
materials brought on record, we sustain the market value fixed (i.e. Rs.8/- sq.ft.) but
instead of 25% development charges one-third has to be deducted. The entitlements shall
be worked out on that basis.
26. The appeal is allowed to the aforesaid extent with no order as to costs.
Appeal allowed.
AIR 2008 SUPREME COURT 403 "Oriental Insurance Co. Ltd. v. Raj Kumari"
(From : Punjab and Haryana)*
Coram : 2 Dr. A. PASAYAT AND LOKESHWAR SINGH PANTA, JJ.
Civil Appeal No. 5209 of 2007 (arising out of SLP (C) No. 2511 of 2006),D/- 14 -11
-2007.
Oriental Insurance Co. Ltd. v. Smt. Raj Kumari and Ors.
(A) Constitution of India, Art.141 - PRECEDENT - Precedent - Principle on which
question before Court is decided - Constitutes precedent - Not every observations made
in judgment.
Every decision contains three basic postulates : (a) findings of material facts, direct and
inferential. An inferential finding of facts is the inference which the Judge draws from the
direct, or perceptible facts; (b) statements of the principles of law applicable to the legal
problems disclosed by the facts; and (c) judgment based on the combined effect of the
above. A decision is an authority for what it actually decides. What is of the essence in a
decision is its ratio and not every observation found therein nor what logically flows from
the various observations made in the judgment. The enunciation of the reason or principle
on which a question
@page-SC404
before a Court has been decided is alone binding as a precedent. Observations of courts
are neither to be read as Euclid's Theorems nor as provisions of the statute and that too
taken out of their context. (Paras 11, 12)
(B) Motor Vehicles Act (59 of 1988), S.147 - MOTOR VEHICLES - INSURANCE -
Liability of insurer - Limited to certain sum - Insured a transport company - No evidence
adduced to show that claimant would have difficulty in recovering sum awarded from
insured - Insurer cannot be directed to pay entire compensation and subsequently recover
excess paid from insured. (Para 16)
Cases Referred : Chronological Paras
2002 AIR SCW 259 : AIR 2002 SC 651 6
1998 AIR SCW 1327 : AIR 1998 SC 1433 7
1996 AIR SCW 4020 (Rel. on, Pnt A) 11
1995 AIR SCW 1142 : AIR 1995 SC 1113 7
AIR 1988 SC 719 7
(1972) 2 WLR 537 (Ref., Pnt A) 13
(1971) 1 WLR 1062 (Ref., Pnt A) 13
(1970) 2 All ER 294 (Ref., Pnt A) 13
AIR 1968 SC 647 (Ref. on, Pnt A) 11
1951 AC 737 : (1951) 2 All ER 1 (Ref. Pnt A) 12
1901 AC 495 (HL) (Ref.) 11
M. K. Dua and Kishore Rawat, for Appellant; Dinesh Chander Yadav and Dr. Kailash
Chand, for Respondents.
* F. A. F. O. No. 1029 of 1986, D/- 13-7-2005 (PandH).
Judgement
1. Dr. ARIJIT PASAYAT, J.:- Leave granted.
2. Challenge in this appeal is to the order passed by a learned Single Judge of the Punjab
and Haryana High Court. By the impugned judgment, the High Court held that though the
liability of the appellant (hereinafter referred to as the insurer) was limited to Rs. 50,000/-
yet it was to first pay the amount awarded to the claimants and recover amount in excess
of Rs. 50,000/- from the owner and driver of the offending vehicle.
2A. Factual position in a nutshell is as follows:
One Karan Singh, conductor of the bus No. DEP-3514 lost his life in an accident which
took place on 14.7.1984. The bus belonged to M/s. Mewat Transport Company Private
Limited (hereinafter referred to as the insured). The bus was driven by deceased Karan
Singh and it dashed in a tanker No. HRG-2852. The impact was so intense and severe
that several persons sitting in the bus died, while many others sustained injuries. The
widow, minor children and parents of aforesaid Karan Singh lodged claim petition
claiming compensation of Rs.1,40,000/-. The Tribunal took several claim petitions
together and in respect of the claim under consideration awarded compensation of Rs.
57,600/- along with 12% interest p.a. from the date of institution of the claim petition. It
was, however, held that liability of the insurer was limited to Rs. 50,000/-.
3. The claimants filed appeal before the Punjab and Haryana High Court. By the
impugned order the High Court enhanced claim of compensation to Rs.1,25,200/-. It was
held, as was done by the Tribunal, that the liability of the insurer was limited to Rs.
50,000/- in terms of the insurance policy. However, it was held that the entire amount was
to be paid by the insurer to the claimants and it was entitled to recover the amount in
excess of Rs. 50,000/- from the owner and the driver of the vehicle.
4. In support of the appeal, learned counsel for the appellant submitted that having held
that the liability of the insurance company was limited to Rs. 50,000/-, the High Court
was not justified in directing payment of the entire amount by it and to recover the
differential amount.
5. There is no appearance on behalf of the respondents.
6

. It would be appropriate to take a note of what was held by the Constitution Bench of
this Court in New India Assurance Co. Ltd. v. C.M. Jaya and Ors. (2002 (2) SCC 278). In
that case it was held, inter alia, as follows: 2002 AIR SCW 259, Para 13
"In the circumstances, we hold that the liability of the appellant-insurance company is
limited to Rs. 50,000/-, as held by the Tribunal. In the view we have taken, it is
unnecessary to go into the question relating to either maintainability of cross-objections
before the High Court against the appellant alone or as to the enhancement of
compensation when the owner and driver have not filed appeal against the impugned
judgment."
7. The questions that were considered by the Constitution Bench are as follows:

"The question involved in these appeals 1995 AIR SCW 1142


1998 AIR SCW 1327

@page-SC405
is whether in a case of insurance policy not taking any higher liability by accepting a
higher premium, in case of payment of compensation to a third party, the insurer would
be liable to the extent limited under Section 95(2) or the insurer would be liable to pay
the entire amount and he may ultimately recover from the insured. On this question, there
appears to be some apparent conflict in the two three-Judge Bench decision of this Court-
(1) New India Assurance Co. Ltd. v. Shanti Bai (1995 (2) SCC 539) and (2) Amrit Lal
Sood v. Kaushalya Devi Thapar (1998 (3) SCC 744).

2. In the latter decision, unfortunately the decision in New India Assurance case (supra)
has not been noticed though reference has been made to the decision of this Court in
National Insurance Co. Ltd. v. Jugal Kishore [(1988) 1 SCC 626], which was relied upon
in the earlier three-Judge Bench Judgment. In view of the apparent conflict in these two
three-Judge Bench decisions, we think it appropriate that the records of this case may be
placed before my Lord, the Chief Justice of India to constitute a larger Bench for
resolving the conflict. We accordingly so direct. The record may now be placed before
the Hon'ble the Chief Justice of India." AIR 1988 SC 719

8. It would be evident from the conclusions of this Court the liability of the insurance
company would in the instant case be limited to quantum which was to be indemnified in
terms of the policy. The Tribunal and the High Court have held accordingly.
9. In Oriental Insurance Co. Ltd. vs. Shakuntala Garg and Ors. (Civil Appeal No. 104 of
2000, disposed of on 10.1.2003), it was held as follows:
"Learned counsel for the appellant at this stage expressed an apprehension that by virtue
of the terms of the Award, the appellant may be required to pay the entire amount and
recover it from the owner. In the light of the modification of the impugned Award, such
question does not arise."
10. It is true that in certain cases this Court has, after looking into the fact situation,
directed the insurance company to make payment with liberty to recover the amount in
excess of the liability from the insured. Those decisions were given on the facts-situation
of the cases concerned.
11

. Reliance on the decision without looking into the factual background of the case before
it is clearly impermissible. A decision is a precedent on its own facts. Each case presents
its own features. It is not everything said by a Judge while giving a judgment that
constitutes a precedent. The only thing in a Judges decision binding a party is the
principle upon which the case is decided and for this reason it is important to analyse a
decision and isolate from it the ratio decidendi. According to the well-settled theory of
precedents, every decision contains three basic postulates (i)findings of material facts,
direct and inferential. An inferential finding of facts is the inference which the Judge
draws from the direct, or perceptible facts; (ii) statements of the principles of law
applicable to the legal problems disclosed by the facts; and (iii) judgment based on the
combined effect of the above. A decision is an authority for what it actually decides.
What is of the essence in a decision is its ratio and not every observation found therein
nor what logically flows from the various observations made in the judgment. The
enunciation of the reason or principle on which a question before a Court has been
decided is alone binding as a precedent. (See: State of Orissa v. Sudhansu Sekhar Misra
and Ors. (AIR 1968 SC 647) and Union of India and Ors. v. Dhanwanti Devi and Ors.
(1996 (6) SCC 44). A case is a precedent and binding for what it explicitly decides and no
more. The words used by Judges in their judgments are not to be read as if they are words
in Act of Parliament. In Quinn v. Leathem (1901) AC 495 (H.L.), Earl of Halsbury LC
observed that every judgment must be read as applicable to the particular facts proved or
assumed to be proved, since the generality of the expressions which are found there are
not intended to be exposition of the whole law but governed and qualified by the
particular facts of the case in which such expressions are found and a case is only an
authority for what it actually decides.1996 AIR SCW 4020

12. Courts should not place reliance on decisions without discussing as to how the factual
situation fits in with the fact-situation of the decision on which reliance is placed.
Observations of Courts are neither to be read as Euclid's Theorems nor as provisions of
the statute and that too taken out of their context. These observations must be read in the
context in which they appear to have been stated. Judgments of Courts
@page-SC406
are not to be construed as statutes. To interpret words, phrases and provisions of a statute,
it may become necessary for Judges to embark into lengthy discussions but the discussion
is meant to explain and not to define. Judges interpret statutes, they do not interpret
judgments. They interpret words of statutes; their words are not to be interpreted as
statutes. In London Graving Dock Co. Ltd. V. Horton (1951 AC 737 at p.761), Lord Mac
Dermot observed:
"The matter cannot, of course, be settled merely by treating the ipsissima verba of Willes,
J as though they were part of an Act of Parliament and applying the rules of interpretation
appropriate thereto. This is not to detract from the great weight to be given to the
language actually used by that most distinguished judge."
13. In Home Office v. Dorset Yacht Co. (1970 (2) All ER 294) Lord Reid said, "Lord
Atkins speech.....is not to be treated as if it was a statute definition. It will require
qualification in new circumstances." Megarry, J in (1971) 1 WLR 1062 observed: "One
must not, of course, construe even a reserved judgment of Russell L.J. as if it were an Act
of Parliament." And, in Herrington v. British Railways Board (1972 (2) WLR 537) Lord
Morris said:
"There is always peril in treating the words of a speech or judgment as though they are
words in a legislative enactment, and it is to be remembered that judicial utterances made
in the setting of the facts of a particular case."
14. Circumstantial flexibility, one additional or different fact may make a world of
difference between conclusions in two cases. Disposal of cases by blindly placing
reliance on a decision is not proper.
15. The following words of Lord Denning in the matter of applying precedents have
become locus classicus:
"Each case depends on its own facts and a close similarity between one case and another
is not enough because even a single significant detail may alter the entire aspect, in
deciding such cases, one should avoid the temptation to decide cases (as said by
Cordozo) by matching the colour of one case against the colour of another. To decide
therefore, on which side of the line a case falls, the broad resemblance to another case is
not at all decisive."
*** *** ***
"Precedent should be followed only so far as it marks the path of justice, but you must cut
the dead wood and trim off the side branches else you will find yourself lost in thickets
and branches. My plea is to keep the path to justice clear of obstructions which could
impede it."
16. In the instant case the insurer was a private limited company doing transport business.
There was no material placed before the High Court to show that the claimants would
have any difficulty in recovering the awarded amount from it. That being so, the High
Court's order is modified to the extent that the insurer shall pay an amount of Rs.50,000/-
with interest awarded to claimants. The balance has to be paid by the insured.
17. Another point urged before this Court in support of the appeal was that the rate of
interest is high. The liability of the insurance company is limited to Rs.50,000/? with
interest @ 9% p.a. from the date of the application. The rate is being fixed considering
the date of accident. The insured shall forthwith make payment of the balance amount
with interest to the claimants and in any event not later than 3 months from the date of
this order.
18. The appeal is allowed to the aforesaid extent with no order as to costs.
Order accordingly.
AIR 2008 SUPREME COURT 406 "Thankachan v. State of Kerala"
(From : 2005 Cri LJ 2385 (Kerala))
Coram : 2 Dr. A. PASAYAT AND P. SATHASIVAM, JJ.
Criminal Appeal No. 1535 of 2007 (arising out of SLP (Cri.) No. 3646 of 2006), D/- 13
-11 -2007.
Thankachan and Anr. v. State of Kerala.
(A) Penal Code (45 of 1860), S.300, Exception 1 and Exception 4 - MURDER - Murder
or culpable homicide not murder - Exception 4 deals with a case of prosecution not
covered by Exception 1 - Exceptions 1 and 4 are founded upon same principle namely
absence of premeditation - But, while in the case of Exception 1 there is total deprivation
of self-control, in case of Exception 4, there is only that heat of passion which clouds
men's sober reason and urges them to deeds which they would not otherwise do - There is
provocation
@page-SC407
in Exception 4 as in Exception 1; but injury done is not direct consequence of that
provocation.
2006 AIR SCW 1678, Rel. on. (Para 10)
(B) Penal Code (45 of 1860), S.300, S.304, Part I - MURDER - CULPABLE HOMICIDE
- Murder or culpable homicide - Sudden quarrel - Accused persons allegedly inflicted
injuries on deceased with chopper - Prosecution version itself showed that deceased first
assaulted one of accused with broken bottle causing injury - Accused had given knife
blow to deceased in sudden quarrel in heat of passion - Conviction of accused would be,
therefore, under S. 304, Part I and not under S. 302.
2005 Cri LJ 2385 (Ker.), Reversed. (Paras 10, 12)
Cases Referred : Chronological Paras
2006 AIR SCW 1678 : 2006 Cri LJ 2111, (Rel. on) 11
C. N. Sree Kumar, P. R. Nayak and Harshad V. Hameed, Advocates for Appellants; G.
Prakash, Advocate for Respondent.
Judgement
Dr. ARIJIT PASAYAT, J.:-Leave granted.
2. Challenge in this appeal is to the order passed by a Division Bench of the Kerala High
Court, dismissing the appeal filed by the appellants who were described as A2 and A3
indicating their position before the trial court, while allowing the appeals filed by the two
other accused persons (A1 and A4).
3. The conviction of the appellants for offence punishable under Section 302 read with
Section 34 of the Indian Penal Code, 1860 (in short the 'IPC') and sentence of
imprisonment for life and a fine of Rs.20,000/- with default stipulation was upheld.
4. The prosecution version in a nutshell is as follows:
On 7.2.1997 at or about 6.45 p.m. at Ayamkudy Kara in Muttuchira Village of Vaikom
Taluk in Kotayam District, the 4th accused came driving his goods autorickshaw (pick-u-
auto) along with A1 to A3 in the said goods carrier and pulled up in front of Marangattil
House of Sathyadevan @ Sahadevan @ Sahadi (hereinafter referred to as the 'deceased').
The deceased was the driver of a mini lorry. A2 straightway went over to the deceased
who was sitting along with PW2 in the veranda of his house. A2 caught hold of the
deceased by the tuck of his dhoti and dragged him on to the Ezhumanthuruthi Kapoola
road in front. The deceased picked up a soda bottle from the parapet of his house. Seeing
this A2 went and picked a soda bottle from the adjacent grocery shop run by Rajamma
(PW 7), the wife of the deceased and came on to the road. From the southern mud road
(road margin) in front of the aforesaid grocery shop, A2 struck the deceased on the head
with the soda bottle. Then the deceased also hit A2 on the head with the soda bottle in his
hand and inflicted an injury. Seeing this A2 sprinkled chilly powder on the eyes of the
deceased. The chilly powder got into the eyes of the deceased who stood there with both
hands held against his face and rubbing his eyes. A1 then exhorted his companions to cut
Sahadevan to death. Thereupon A2 drew a chopper from inside his shirt and cut the
deceased on his head inflicting injuries. A3 stabbed the deceased on his right arm with a
knife inflicting injury. A4 then cut the deceased on the back of his head with a chopper.
The deceased fell on the road and was taken by PWs.1, 2 and 8 to the Kottayam Medical
College Hospital. The deceased who had become unconscious on account of the injuries
sustained by him succumbed to the same at about 2.10 p.m. on 8.2.1997. Since the
aforesaid acts were done by A1 to A4 in prosecution of their common intention to do so,
the accused persons were charged for having committed the offence of murder punishable
under Section 302 read with Section 34 IPC.
On the accused pleading not guilty to the charge framed against them by the court below
for the aforementioned offence, the prosecution was permitted to adduce evidence in
support of its case. The prosecution examined 16 witnesses as PWs 1 to 16 and got
marked 17 documents as Exts. P1 to P17 and 8 material objects as MOs. 1 to8.
After the closure of the prosecution's evidence the accused were questioned under Section
313(1) of the Code of Criminal Procedure, 1973 (in short 'Cr.P.C.') with regard to the
incriminating circumstances appearing against them in the evidence for the prosecution.
They denied those circumstances and maintained their innocence. They admitted that
Exts. P16 and P17 are the wound certificates pertaining to A2 and A3 respectively.
When called upon to enter on their
@page-SC408
defence, the accused examined the Secretary of the Ayamkudy Branch of KPMS as DW1.
5. Placing reliance on the evidence of PWs 2, 3, 7 and 8 the trial Court recorded
conviction. As noted above, appeal was preferred before the High Court by all the four
accused persons, and the appeal filed by the present appellants was dismissed while that
of co-accused was allowed.
6. In support of the appeal learned counsel for the appellant submitted even if prosecution
version accepted in toto offence under Section 302 IPC is not made out. As a matter of
fact it is the prosecution version that the deceased first assaulted appellant No.1 with a
broken bottle and caused several injuries.
7. Learned counsel for the respondent on the other hand submitted that the trial Court and
the High Court have rightly found the accused persons guilty of offence punishable under
Section 302 IPC.
8. In essence the stand of learned counsel for the appellant is that Exception IV to Section
304 IPC would apply to the facts of the case.
9. For bringing in operation of Exception 4 to Section 300 IPC, it has to be established
that the act was committed without premeditation, in a sudden fight in the heat of passion
upon a sudden quarrel without the offender having taken undue advantage and not having
acted in a cruel or unusual manner.
10. The Fourth Exception to Section 300 IPC covers acts done in a sudden fight. The said
Exception deals with a case of prosecution not covered by the First Exception, after
which its place would have been more appropriate. The Exception is founded upon the
same principle, for in both there is absence of premeditation. But, while in the case of
Exception 1 there is total deprivation of self-control, in case of Exception 4, there is only
that heat of passion which clouds men's sober reason and urges them to deeds which they
would not otherwise do. There is provocation in Exception 4 as in Exception 1; but the
injury done is not the direct consequence of that provocation. In fact Exception 4 deals
with cases in which notwithstanding that a blow may have been struck, or some
provocation given in the origin of the dispute or in whatever way the quarrel may have
originated, yet the subsequent conduct of both parties puts them in respect of guilt upon
equal footing. A "sudden fight" implies mutual provocation and blows on each side. The
homicide committed is then clearly not traceable to unilateral provocation, nor in such
cases could the whole blame be placed on one side. For if it were so, the Exception more
appropriately applicable would be Exception 1. There is no previous deliberation or
determination to fight. A fight suddenly takes place, for which both parties are more or
less to be blamed. It may be that one of them starts it, but if the other had not aggravated
it by his own conduct it would not have taken the serious turn it did. There is then mutual
provocation and aggravation, and it is difficult to apportion the share of blame which
attaches to each fighter. The help of Exception 4 can be invoked if death is caused (a)
without premeditation; (b) in a sudden fight; (c) without the offender having taken undue
advantage or acted in a cruel or unusual manner; and (d) the fight must have been with
the person killed. To bring a case within Exception 4 all the ingredients mentioned in it
must be found. It is to be noted that the "fight" occurring in Exception 4 to Section 300
IPC is not defined in IPC. It takes two to make a fight. Heat of passion requires that there
must be no time for the passions to cool down and in this case, the parties have worked
themselves into a fury on account of the verbal altercation in the beginning. A fight is a
combat between two or more persons whether with or without weapons. It is not possible
to enunciate any general rule as to what shall be deemed to be a sudden quarrel. It is a
question of fact and whether a quarrel is sudden or not must necessarily depend upon the
proved facts of each case. For the application of Exception 4, it is not sufficient to show
that there was a sudden quarrel and that there was no premeditation. It must further be
shown that the offender has not taken undue advantage or acted in cruel or unusual
manner. The expression "undue advantage" as used in the provision means "unfair
advantage".
11

. The above position is highlighted in Sandhya Jadhav v. State of Maharashtra (2006) 4


SCC 653). 2006 AIR SCW 1678

12. Considering the background facts, appropriate conviction would be under Section
304 Part I IPC and not Section 302 IPC. The conviction is accordingly altered. Custodial
sentence of ten years would suffice.
@page-SC409
Fine amount is reduced to Rs.5,000/-. In case fine is not paid, default sentence would be
two years.
13. Appeal is allowed to the aforesaid extent.
Appeal allowed.
AIR 2008 SUPREME COURT 409 "Antram v. State of Maharashtra"
(From : Bombay)*
Coram : 2 Dr. A. PASAYAT AND P. SATHASIVAM, JJ.
Criminal Appeal No. 1529 of 2007 (arising out of SLP (Cri.) No. 376 of 2007), D/- 12
-11 -2007.
Antram v. State of Maharashtra.
Penal Code (45 of 1860), S.300, S.299, Exception 2 - Evidence Act (1 of 1872), S.32 -
MURDER - DYING DECLARATION - EVIDENCE - ASSAULT - Murder -
Circumstantial evidence - Accused allegedy assaulted his wife with axe - Relation
between accused and deceased were not cordial - Authenticity of dying declarations of
deceased was established - Dying declarations, both oral and written, did not suffer from
any infirmity - And were sufficient to fasten guilt of accused - Plea that deceased could
have survived with proper medical care and that death is not referable to cause of death in
ordinary course of nature due to ante mortem injuries - Not tenable - Plea clearly
overlooks Exception 2 to S. 299 - Conviction of accused is proper.
AIR 1996 SC 2962 and AIR 1974 SC 2328, Relied on. (Paras 10, 14)
Cases Referred : Chronological Paras
1996 AIR SCW 1174 : AIR 1996 SC 2962 : 1996 Cri LJ 1872, (Rel. on) 12
AIR 1974 SC 2328 : 1975 Cri LJ 16, (Rel. on) 13
Mrs. S. Usha Reddy, for Appellant; R. K. Adsure, for Respondent.
* Cri. Appeal No. 218 of 2005, D/- 24-6-2005 (Bom).
Judgement
Dr. ARIJIT PASAYAT, J.:-Leave granted.
2. Challenge in this appeal is to the judgment of a Division Bench of the Bombay High
Court, Aurangabad Bench dismissing the appeal filed by the appellant upholding his
conviction for offence punishable under Section 302 of the Indian Penal Code, 1860 (in
short the 'IPC')and the sentence of imprisonment for life and fine of Rs.200/- with default
stipulation. The judgment impugned before the High Court was delivered by learned
Second Additional Sessions Judge, Latur in Sessions case No. 24 of 2004.
3. Prosecution version in a nutshell is as follows:
On 28.11.2003 at about 9:30 a.m., appellant Antram caused death of his wife Shobha by
brutally assaulting her with an axe. Shobha was married to the accused long back. Two
sons and the daughter were the children born from said wedlock. About 3 years prior to
alleged incident, Shobha had started residing at village Kamkheda with the children,
which is place of her parents. Accused belongs to village Zari Khurd. He used to
intermittently visit Shobha and children at village Kamkheda. The relations between
husband and wife had not remained cordial because accused used to object to Shobha
undertaking any job. This was because he used to suspect her character.
Accused had been to village Kamkheda about 8 days prior to alleged incident. There is no
eye witness to the incident. Prosecution case rests on circumstantial evidence. Accused
and deceased were in the house at the time of incident. A quarrel took place between the
couple during which accused brutally assaulted Shobha with an axe and thereafter ran
away. Sangeeta (PW-5), daughter of deceased and accused, was about to proceed to
school when the quarrel had started. It was Friday and there was Saraswati Puja in the
school. At the suggestion of teacher, Sangeeta returned home for bringing some flowers.
Since she noticed that house was locked from outside she enquired from the neighbour
about her mother. Ultimately, she returned home, opened the door to find mother Shobha
lying on the floor in injured condition with bleeding injuries on her head, face etc. A
blood stained axe was also lying there. She enquired from her mother as to what had
happened. She disclosed that she was assaulted by Sangeeta's father i.e., accused.
Sangeeta reported the matter to her maternal uncle Tukaram (PW-2), who arrived at the
spot and enquired from Shobha when she repeated that she was assaulted by accused.
Tukaram (P.W.2) and his uncle Ganpat took Shobha to Hospital at Renapur in an auto
rickshaw. At the advice of Medical Officer, Renapur, she was
@page-SC410
shifted to Civil Hospital, Latur. The Medical Officer at Civil Hospital, Latur, advised
them to take her to S.R.T. Medical College, Ambajogal. The relatives, however, took her
to Dr. Sham Agroya (P.W.6) a private medical practitioner and neuro surgeon at Latur.
Tukaram went to Police Station, Renapur and reported the matter to Police. His report
was reduced to writing and the same was treated as FIR (Exh.24), which set law into
motion.
4. Although prosecution does not have any direct evidence about the incident on record, it
relies upon dying declaration of deceased Shobha on more than one occasions. Apart
from oral narration to daughter Sangeeta and cousin brother Tukaram, the dying
declaration has also come on record in the form of history of incident as recorded by
Medical Officer Dr. Warad (P.W.4), who was then attached to Primary Health Centre,
Renapur, where injured Shobha was taken immediately after incident. As it was a medico
legal case, Dr. Agroya, while admitting Shobha, gave intimation of the same to Police
Station by a written letter, whereupon Police Inspector visited the hospital on 29.11.2003
and in presence of Dr. Agroya, he recorded statement of injured Shobha. Shobha was
subjected to surgical treatment by Dr. Agroya on 29.11.2003 and subsequently she was
discharged from the hospital on 09.12.2003 as cured patient. However, Shobha expired
on 10.12.2003.
5. Consequently, the offence, which was initially registered under Section 307 of IPC was
converted to one under Section 302 of IPC. On completion of the investigation, charge
sheet was filed in the Court of competent Magistrate and upon committal, Sessions Judge
recorded conviction and sentence as described hereinabove at the conclusion of Sessions
trial.
6. The accused in his statement, recorded under Section 313 of the Code of Criminal
Procedure, 1973 (in short the 'Code'), pleaded innocence. Before the trial court ten
witnesses were examined to further prosecution version.
7. Before the High Court the stand of the appellant essentially was that the dying
declarations are not reliable. As a matter of fact there were four dying declarations
recorded; two were oral and two were recorded. Apart from oral narration to Sangeeta
(PW-5) daughter of the deceased , Tukaram (PW2) cousin brother's statement was
recorded as dying declarations by PW.4 Dr. Vilas Warad, Medical Officer, Primary Health
Centre, Renapur who initially examined the injured and recorded her statement. Dr.
Agroya (PW-6) while admitting the deceased gave intimation to the Police station . It was
also submitted that the offences were not covered under Section 302 IPC, 304 Part I and
304 Part II IPC. The trial court did not accept this stand and recorded conviction and
sentence as noted above. The accused and the State reiterated the respective stand before
the High Court. High Court found that the dying declarations were reliable and there was
not much variation in the version. However, the dying declaration as contained in Exh. 36
was kept out of consideration, and the dying declarations before the doctor and the cousin
brother were accepted. Coming to the plea that the case was not covered under Section
302 IPC, the High Court referred to the factual aspects, the injuries sustained and came to
the conclusion that case was clearly covered under Section 302 IPC.
8. The stand taken by the appellant before the High Court was reiterated.
9. Learned counsel for the State supported the judgment of the courts below.
10. So far as the dying declarations are concerned as rightly observed by the High Court
even if Exhibit 36 is kept out of consideration, the dying declarations both oral and
written were sufficient to fasten the guilt of the accused. The High Court has elaborately
dealt with the authenticity of the dying declarations and had rightly come to the
conclusion that they did not suffer from any infirmity.
11. Coming to the question as to applicability of Section 302 IPC, great emphasis was
laid on the evidence of Dr. S.K. Shinde (PW-7). It was contended that the death was due
to medical negligence and therefore accused could not have been convicted under Section
302 IPC. It was submitted that had the patient been given proper care, there was a
possibility of removing thick mucus and food particles from trachea and bronchi by using
certain instruments and with proper
@page-SC411
medicines, she could have survived. The High Court noted that the throwing out the
vomit by the deceased was not a natural course but it was a result of two injuries i.e.
injuries Nos. 3 and 4. The High Court found that the presence of mucus and food
particles in the trachea and bronchi cannot be totally delinked from the injuries inflicted
by the accused. It was the stand of the accused that the death was due to Septicaemia and
therefore, it is not referable to cause of death in the ordinary course of nature due to ante
mortem injuries.
12

. In State of Haryana v. Pala and Ors. (AIR 1996 SC 2962) it was noted as follows.
1996 AIR SCW 1174, (Para 3)

"In answering the question whether a wound is dangerous to life, the danger must be
assessed on the probable primary effects of the injury. Such possibilities as the
occurrence of tetanus or septicaemia, later on, are not to be taken into consideration."
13. In Sudershan Kumar v. State of Delhi (AIR 1974 SC 2328) it was noted as follows:
"The fact that the deceased lingered for about 12 days would not show that the death was
not the direct result of the act of the accused in throwing acid on her. So also the fact that
the deceased developed symptoms of malaena and respiratory failure and they also
contributed to her death could not in any way affect the conclusion that the injuries
caused by the acid burns were the direct cause of her death."
14. As noted above it was emphasized by learned counsel for the appellant that with
proper medical care the deceased could have survived and therefore Section 302 IPC has
no application. The plea clearly overlooks Exception 2 to Section 299 IPC, which reads
as follows:
"Explanation 2.- Where death is caused by bodily injury, the person who causes such
bodily injury shall be deemed to have caused the death, although by resorting to proper
remedies and skilful treatment the death might have been prevented."
15. When the background facts are examined on the touchstone of the principles of law
highlighted, the inevitable result is that the appeal is without merit, deserves dismissal,
which we direct.
Appeal dismissed.
AIR 2008 SUPREME COURT 411 "State of U. P. v. Atar Singh"
(From : Allahabad)*
Coram : 2 Dr. A. PASAYAT AND D. K. JAIN, JJ.
Criminal Appeal No. 54 of 2001, D/- 12 -11 -2007.
State of U.P. v. Atar Singh and Ors.
(A) Penal Code (45 of 1860), S.300 - MURDER - Murder - Proof - Non-explanation of
injuries on accused by prosecution - Not by itself sufficient to reject other probable,
consistent and creditworthy evidence - Moreso when injuries are simple or superficial in
nature.
It is not an invariable rule that the prosecution has to explain the injuries sustained by the
accused in the same occurrence. If the witnesses examined on behalf of the prosecution
are believed by the Court in proof of guilt of the accused beyond reasonable doubt,
question of obligation of prosecution to explain injuries sustained by the accused will not
arise. When the prosecution comes with a definite case that the offence has been
committed by the accused and proves it's case beyond any reasonable doubt, it becomes
hardly necessary for the prosecution to again explain how and under what circumstances
injuries have been inflicted on the person of the accused. It is more so when the injuries
are simple or superficial in nature. In the case at hand, trifle and superficial injuries on
accused are of little assistance to them to throw doubt on veracity of prosecution case.
AIR 1990 SC 1459, AIR 1972 SC 2593, 2003 AIR SCW 6905, Rel. on.
(Paras 12, 13)
(B) Penal Code (45 of 1860), S.300 - MURDER - ASSAULT - WITNESS -
INVESTIGATION - Murder - Proof - Accused persons assaulted deceased - No
immediate motive proved - No corroboration to prosecution version by any independent
witnesses - Presence of alleged eye- witness on spot not established - Persons whose
name noted in FIR as witnesses, not examined - Statement of deceased recorded by
investigating officer under S. 161 of Cr. P. C. cannot be treated to be dying declaration -
Non- recording of dying declaration in presence of Magistrate not explained - Held in
@page-SC412
circumstances acquittal of accused, cannot be interfered with. (Para 14)
Cases Referred : Chronological Paras
2006 AIR SCW 3568 : AIR 2006 SC 2667 : 2006 Cri LJ 3634 10
2003 AIR SCW 4065 : AIR 2003 SC 3609 : 2003 Cri LJ 3892 10
2003 AIR SCW 5044 : AIR 2003 SC 4407 : 2003 Cri LJ 5010 10
2003 AIR SCW 5095 : AIR 2003 SC 4664 : 2003 Cri LJ 5040 10
2003 AIR SCW 6905 : AIR 2004 SC 742 (Rel. on) 13
2002 AIR SCW 1532 : AIR 2002 SC 1621 : 2002 Cri LJ 2024 10
2000 AIR SCW 1430 : AIR 2000 SC 1833 : 2000 Cri LJ 2212 10
1996 AIR SCW 2438 : AIR 1996 SC 2035 : 1996 Cri LJ 2867 10
AIR 1990 SC 1459 : 1990 Cri LJ 1510 (Rel. on) 12
AIR 1988 SC 863 : 1988 Cri LJ 925 13
AIR 1976 SC 2263 : 1976 Cri LJ 1736 12
AIR 1974 SC 2165 : 1974 Cri LJ 1486 5
AIR 1973 SC 2622 : 1973 Cri LJ 1783 10
AIR 1972 SC 2593 : 1973 Cri LJ 44 (Rel. on) 13
AIR 1968 SC 1281 : 1968 Cri LJ 1479 12
Ratnakar Dash, Sr. Advocate, Sanjay Singh, Anuvrat Sharma, for Appellant; Ms. Kusum
Chaudhary, for Respondents.
* Cri. A. No. 2124 of 1980, D/- 13-4-2000, reported in 2000 All LJ 2012 : 2000 Cri LJ
4933 (All.)
Judgement
Dr. ARIJIT PASAYAT, J.:- Challenge in this appeal is to the judgment rendered by a
Division Bench of the Allahabad High Court which by the impugned judgment acquitted
the respondents and set aside the conviction recorded by the learned Additional Sessions
Judge in .Sessions Trial No.316 of 1979. Each of the accused had been convicted by the
trial court and sentenced to life imprisonment under Section 302 of the Indian Penal
Code, 1860 (in short the 'IPC') read with Section 149 IPC, three months RI under Section
323 read with Section 149 IPC, six months RI under Section 324 IPC read with Section
149 IPC and two years RI under Section 452 IPC. Accused Jai Singh, Atar Singh, Mohan
Singh, Beer Singh and Baburam were further convicted under Section 147 IPC and
sentenced to nine months RI. Accused Ramesh and Lal Singh were however convicted
under Section 148 IPC and sentenced to one year's RI. All the sentences were directed to
run concurrently. The High Court reversed the judgment and directed acquittal in the
appeal filed by the accused persons.
2. Prosecution version as unfolded during trial is as follows:
One Ram Murti (hereinafter referred to as 'deceased') lost his life in the incident whereas
three others namely, Shyam Pal (PW 1), Sohan Pal (PW 3) and Katori Devi sustained
injuries. The incident took place on 4.5.1979 at about 6.30 P.M. in village Balli Nagla,
Police Station Qadarchowk, District Budaun. The report of the incident was lodged by
Shyam Pal (PW 1) on 5.5.1979 at 3.15 A.M. The distance of police station from the place
of occurrence is 8 kms. The accused-respondents Lal Singh and Ramesh were allegedly
armed with spears whereas rest had lathis. The accused-respondents Jai Singh, Atar
Singh, Lal Singh, Mohar Singh and Beer Singh are the sons of Dallu who also allegedly
participated in the incident but died after few days of the incident. About 6 months before
this incident, Durgapal-brother-in-law of Shyam Pal (PW 1) had abducted Dhika daughter
of Dallu. Accused-respondents began to bear ill will against him and his family members
on this account. On 4-5-79 at about 6.30 P.M., exchange of hot words and abuses took
place between Shyam Pal (PW 1) and Dallu at the Chaupal of Nek Ram in connection
with abduction of Dhika. Some persons intervened in the matter and Shyam Pal went to
his home. A little later, all the accused-respondents along with Dallu entered the house of
Shyam Pal. As mentioned earlier, Lal Singh and Ramesh were armed with spears whereas
rest had lathis. Dallu asked the other accused persons to teach a lesson to Shyam Pal and
his family members for defaming him. All the accused-respondents then started
assaulting Shyam Pal (PW 1) and his brothers Sonpal and Ram Murti who were present
there. When their mother Katori Devi came to their rescue, she was also beaten up. Nathu
Singh (PW 2), Ulnfat Irfan, Prem Pal and others also arrived there. Shyam Pal (PW 1),
Ram Murti, Sohan Pal (PW 3) and their mother Katori Devi sustained injuries. Shyam Pal
(PW 2) with his nephew Prempal went to the police station and lodged a report by oral
narration on 5.5.1979 at 3.15 A.M. which was taken down by head constable Baburam
(PW 4). Investigation was undertaken and on completion thereof, charge sheet was filed.
Accused persons pleaded innocence. In order to
@page-SC413
further accusations, prosecution examined eleven witnesses. Learned trial Judge recorded
conviction primarily relying on the evidence of injured witnesses.
3. It was firstly noticed by the High Court that the motive assigned by the prosecution
against the accused respondents did not stand the test of logic. The incident of kidnapping
and abduction of Dhika daughter of Dallu by Durgapal-brother-in-law of Shyam Pal
(PW-1) had taken place about six months before. Even no FIR had been lodged against
Durgapal from the side of accused persons regarding that incident. It was admitted by
PW-1 that even no Panchayat was convened. Further Shyam Pal (PW-1) had admitted
that at the time of exchange of hot words with Dallu at the Chaupal of Nek Ram, two
persons namely, Nek Ram and Urman Singh were there who had intervened. None of
them was produced by the prosecution to indicate the origin of the incident. Dallu himself
was a T.B. patient and the High Court found it hard to believe that after alleged exchange
of hot words at the Chaupal of Nek Ram, he with all his sons, brother and nephew would
have appeared in the house of PW-1 to assault him and his family members. Accordingly,
it was held that even there was no immediate motive for the alleged occurrence.
4. It was also noted that there was no corroboration to the prosecution version by any
independent witnesses. Nathu Singh (PW-2) was resident of another village who claimed
to be present at the place of occurence. He stated that he had come to the village to meet
his relative. According to him the house of Rajpal was situated at a distance of 15-16
paces from the place of incident. The High Court noted that the existence of Rajpal's
house in the vicinity of place of occurrence had not been shown in the site plan. The High
Court found that some parts of his statement could not be reconciled with other parts eg.
that he had reached the village of incident at 6.30 a.m. and was present at the time of
incident which took place about 12 hours later. His statement was to the effect that he had
gone to his son- in-law Rajpal as the latter was about to go to his father-in-law's house
and he wanted to send some cows to his father-in-law. He wanted to send this information
to his father-in-law but his cousin-in-law was not available. He also stated that after some
time he had returned to his village. The High Court found his presence to be not
established. The High Court also noted that Sohan Pal (PW-3) who claimed to be an eye
witness was the brother of PW-1.
5. The High Court noted that even though in the FIR names of some other persons have
been noted as witnesses, none of them had been examined. The High Court was of the
view that statement of the deceased recorded by the investigating officer under Section
161 of the Code of Criminal Procedure, 1973 (in short the 'Cr.P.C.') cannot be treated to
be the dying declaration. The investigating officer (PW-11) noted that when he reached
the spot in the morning of 5.5.1979 subsequent to the lodging of the FIR at about 3.15
a.m. he had found the deceased, Sohan Pal and Katori to be lying there in injured
condition. He recorded the statement of the deceased (Exh.Ka. 20). The High Court
referred to the bed head ticket of the deceased in which it was stated that his general
condition was noted low when he was admitted in the hospital on 5.5.1979. The High
Court also noted the admitted position that the investigating officer did not follow the
instructions contained in Rule 115 of the U.P. Police Regulations relating to recording of
dying declaration. Reference was made to a decision of this Court in Palak Ram v. State
of U.P. ( AIR 1974 SC 2165) wherein it was noted that it would not be prudent to base
conviction on a dying declaration made to the investigating officer which is not signed by
the persons making it and has not been taken in the presence of two witnesses.
6. The High Court also noted that there was no explanation offered as to why the dying
declaration was not recorded in the presence of the Magistrate which is the usual course,
though he died on 7.5.1979 at about 4.00 p.m. Therefore, the High Court treated the same
to be a statement recorded in terms of Section 161 of Cr.P.C. which cannot be treated to
be a dying declaration.
7. The High Court also noted another factor which according to it was significant, i.e. the
presence of large number of injuries on accused Mohar Singh for which no explanation
was offered. This according to the High Court cast a genuine doubt about the actual time,
place, number of assailants and weapons for the injuries. The High Court noted that
injuries on accused Mohar Singh were not superficial and some of them were even
incised wounds. The investigating officer had admitted that Mohar Singh was
@page-SC414
arrested on 6.5.1979. The High Court found it rather unusual that he was produced for
medical examination before a Doctor Shiv Kumar Saxena (PW?5) on 5.5.1979 at 5.20
p.m. by a constable of the Police Station. Therefore, the High Court noted that if there
was no explanation offered as to why he was not arrested on 5.5.1979, the FIR was
claimed to have been lodged at 3.15 a.m. on that day. The High Court noted that though
PW-1 and PW-3 were stated to be injured witnesses in the background facts the
prosecution version was highly improbabilised. The evidence of PW-2 was found to be
not truthful. As a cumulative result of the discussions the High Court found that the
prosecution has not been able to substantiate its version.
8. As noted above, the State has questioned correctness of the conclusions recorded by
the High Court. With reference to the evidence of injured witnesses, PW-1 and PW-3 it is
stated that they are injured witnesses and their version was to be taken as credible and
cogent. There was no reason as to why the injured person would falsely implicate the
innocent person.
9. None appeared for the respondents when the matter was called.
10

. There is no embargo on the appellate Court reviewing the evidence upon which an order
of acquittal is based. Generally, the order of acquittal shall not be interfered with because
the presumption of innocence of the accused is further strengthened by acquittal. The
golden thread which runs through the web of administration of justice in criminal cases is
that if two views are possible on the evidence adduced in the case, one pointing to the
guilt of the accused and the other to his innocence, the view which is favourable to the
accused should be adopted. The paramount consideration of the Court is to ensure that
miscarriage of justice is prevented. A miscarriage of justice which may arise from
acquittal of the guilty is no less than from the conviction of an innocent. In a case where
admissible evidence is ignored, a duty is cast upon the appellate Court to re-appreciate
the evidence where the accused has been acquitted, for the purpose of ascertaining as to
whether any of the accused really committed any offence or not. [See Bhagwan Singh
and Ors. v. State of Madhya Pradesh (2002 (2) Supreme 567)]. The principle to be
followed by appellate Court considering the appeal against the judgment of acquittal is to
interfere only when there are compelling and substantial reasons for doing so. If the
impugned judgment is clearly unreasonable and relevant and convincing materials have
been unjustifiably eliminated in the process, it is a compelling reason for interference.
These aspects were highlighted by this Court in Shivaji Sahabrao Bobade and Anr. v.
State of Maharashtra (AIR 1973 SC 2622), Ramesh Babulal Doshi v. State of Gujarat
(1996 (4) Supreme 167), Jaswant Singh v. State of Haryana (2000 (3) Supreme 320), Raj
Kishore Jha v. State of Bihar and Ors. (2003 (7) Supreme 152), State of Punjab v. Karnail
Singh (2003 (5) Supreme 508 and State of Punjab v. Pohla Singh and Anr. (2003 (7)
Supreme 17) and V.N. Ratheesh v. State of Kerala (2006(10) SCC 617). 2002 AIR
SCW 1532
1996 AIR SCW 2438
2000 AIR SCW 1430
2003 AIR SCW 5095
2003 AIR SCW 4065
2003 AIR SCW 5044
2006 AIR SCW 3568

11. As is rightly contended by learned counsel for the appellant-State in isolation the
circumstances highlighted by the High Court may not be sufficient to direct acquittal.
Two important factors which have been noted by the High Court are (i) non explanation
of injuries on accused Mohar Singh and (ii) the reason for his non arrest on 5.5.1979
when he had appeared before the police officers and had been sent for medical
examination.
12

. We shall first deal with the question regarding non-explanation of injuries on the
accused. Issue is if there is no such explanation what would be its effect? We are not
prepared to agree with the learned counsel for the defence that in each and every case
where prosecution fails to explain the injuries found on some of the accused, the
prosecution case should automatically be rejected, without any further probe. In Mohar
Rai and Bharath Rai v. The State of Bihar (1968 (3) SCR 525), it was observed: AIR
1968 SC 1281, Para 6

"...In our judgment, the failure of the prosecution to offer any explanation in that regard
shows that evidence of the prosecution witnesses relating to the incident is not true or at
any rate not wholly true. Further those injuries probabilise the plea taken by
@page-SC415
the appellants."

In another important case Lakshmi Singh and Ors. v. State of Bihar (1976 (4) SCC 394),
after referring to the ratio laid down in Mohar Rai's case (supra), this Court observed:
AIR 1976 SC 2263

"Where the prosecution fails to explain the injuries on the accused, two results follow:
(1) that the evidence of the prosecution witnesses is untrue; and (2) that the injuries
probabilise the plea taken by the appellants."
It was further observed that:
"In a murder case, the non-explanation of the injuries sustained by the accused at about
the time of the occurrence or in the course of altercation is a very important circumstance
from which the Court can draw the following inferences:
(1) that the prosecution has suppressed the genesis and the origin of the occurrence and
has thus not presented the true version;
(2) that the witnesses who have denied the presence of the injuries on the person of the
accused are lying on a most material point and, therefore, their evidence is unreliable;
(3) that in case there is a defence version which explains the injuries on the person of the
accused assumes much greater importance where the evidence consists of interested or
inimical witnesses or where the defence gives a version which competes in probability
with that of the prosecution one."

In Mohar Rai's case (supra) it is made clear that failure of the prosecution to offer any
explanation regarding the injuries found on the accused may show that the evidence
related to the incident is not true or at any rate not wholly true. Likewise in Lakshmi
Singh's case (supra) it is observed that any non-explanation of the injuries on the accused
by the prosecution may affect the prosecution case. But such a non-explanation may
assume greater importance where the defence gives a version which competes in
probability with that of the prosecution. But where the evidence is clear, cogent and
creditworthy and where the Court can distinguish the truth from falsehood the mere fact
that the injuries are not explained by the prosecution cannot by itself be a sole basis to
reject such evidence, and consequently the whole case. Much depends on the facts and
circumstances of each case. These aspects were highlighted by this Court in Vijayee
Singh and Ors. v. State of U.P. (AIR 1990 SC 1459). AIR 1968 SC 1281
AIR 1976 SC 2263

13

. Non-explanation of injuries by the prosecution will not affect prosecution case where
injuries sustained by the accused are minor and superficial or where the evidence is so
clear and cogent, so independent and disinterested, so probable, consistent and
creditworthy, that it outweighs the effect of the omission on the part of prosecution to
explain the injuries. As observed by this Court in Ramlagan Singh v. State of Bihar (AIR
1972 SC 2593) prosecution is not called upon in all cases to explain the injuries received
by the accused persons. It is for the defence to put questions to the prosecution witnesses
regarding the injuries of the accused persons. When that is not done, there is no occasion
for the prosecution witnesses to explain any injury on the person of an accused. In Hare
Krishna Singh and Ors. v. State of Bihar (AIR 1988 SC 863), it was observed that the
obligation of the prosecution to explain the injuries sustained by the accused in the same
occurrence may not arise in each and every case. In other words, it is not an invariable
rule that the prosecution has to explain the injuries sustained by the accused in the same
occurrence. If the witnesses examined on behalf of the prosecution are believed by the
Court in proof of guilt of the accused beyond reasonable doubt, question of obligation of
prosecution to explain injuries sustained by the accused will not arise. When the
prosecution comes with a definite case that the offence has been committed by the
accused and proves it's case beyond any reasonable doubt, it becomes hardly necessary
for the prosecution to again explain how and under what circumstances injuries have
been inflicted on the person of the accused. It is more so when the injuries are simple or
superficial in nature. In the case at hand, trifle and superficial injuries on accused are of
little assistance to them to throw doubt on veracity of prosecution case. (See Surendra
Paswan v. State of Jharkhand (2003) 8 Supreme 476). 2003 AIR SCW 6905

14. Considering the cumulative effect of circumstances which have weighed with the
@page-SC416
High Court to direct acquittal, it cannot be said that the view taken by the High Court is
not a plausible view. That being so, we are not inclined to interfere with the order of
acquittal. The appeal deserves to be dismissed which we direct.
Appeal dismissed.
AIR 2008 SUPREME COURT 416 "P. Vaikunta Shenoy and Co., M/s. v. P. Hari Sharma"
(From : 2001 (3) Kant LJ 357)
Coram : 2 A. K. MATHUR AND MARKANDEY KATJU, JJ.
Civil Appeal No. 5540 of 2001, D/- 31 -10 -2007.
M/s. P. Vaikunta Shenoy and Co. v. P. Hari Sharma.
Karnataka Money Lenders Act (12 of 1962), S.2(10) - MONEY LENDERS - WORDS
AND PHRASES - Money lender - Definition has to be given purposive interpretation -
Person advancing money to supplier of goods to ensure regular supply - Not a money
lender.
2001 (3) Kant LJ 357, Reversed.
In business various methods are adopted by a businessman for ensuring the smooth
running of his business. Very often, one of the methods is that the businessman advances
money to his supplier of goods to ensure that the supplies are regular and are made to him
rather than being diverted to other parties. There is nothing illegal in this practice and it is
widespread. In such cases the object of advancing the loan is not to earn interest thereon
but to ensure the regular supply of goods. Though, no doubt, interest is charged on these
loans yet that was not the principal object of advancing the loan. Such a person cannot be
said to be a money lender. The purpose of the Act is to prevent the malpractice of
oppression by money lenders to take advantage of peoples' poverty. Keeping in view the
object of Act a purposive interpretation has to be given to the definition of money
lenders.
2001 (3) Kant LJ 357, Reversed. (Paras 7, 8, 9, 10, 13)
Cases Referred : Chronological Paras
AIR 1988 SC 2239 (Ref.) 12
AIR 1963 SC 1207 (Foll.) 11
V. B. Joshi, Kailash Pandey, for Appellant; G. V. Chandrashekhar, P. P. Singh, for
Respondent.
Judgement
1. MARKANDEY KATJU, J.:-This appeal has been filed against the impugned judgment
of the Karnataka High Court dated 25.03.2000 in R.F.A. No. 531 of 1997. We have heard
learned counsel for the parties and perused the record.
2. The plaintiff-appellant has alleged that he was carrying the business of commission
agent. The defendant was having an areca nut (supari) garden and he used to supply the
areca nuts to the plaintiff. The defendant used to receive money from the plaintiff off and
on, which the plaintiff used to advance him to secure regular supply of the areca nuts. It
was alleged by the plaintiff that defendant had borrowed Rs.72,044.43 paise as per the
ledger account regularly maintained by the plaintiff. Hence the plaintiff filed a suit for
recovery of this amount with interest at the rate of 18 per cent per annum.
3. The defendant denied the plaintiffs case and advanced the plea that plaintiff was a
money-lender and he did not have a licence as required by the Karnataka Money Lenders
Act, 1961. Consequently, the defendant alleged that the suit was not maintainable as the
plaintiff had not taken a licence under the aforesaid Act.
4. The Trial Court decreed the suit of the plaintiff but the said decree was set aside by the
High Court. Hence this appeal.
5. Learned counsel for the plaintiff-appellant has submitted that the plaintiff was not a
money-lender as defined in Section 2 (10) of the Karnataka Money Lenders Act. The
aforesaid Section 2 (10) states that a money-lender is one who "carries on the business of
money lending in the State."
Section 2 (2) defines the business of money lending as follows:-
'Business of money lending means business of advancing loan whether or not in
connection with or in addition to any other business.'
6. Learned counsel for the respondent submits that in view of the aforesaid definitions the
appellant was clearly a money-lender. We do not agree.
7. It may be mentioned that the purpose of the Act was to prevent the malpractice of
oppression by money-lenders to take advantage of people's poverty.
8. In the money lending business the object of the money-lender is to earn interest on the
loan he has advanced. In the
@page-SC417
present case the object of advancing the loan by the appellant was not to earn interest
thereon but to ensure the regular supply of areca nuts. Though, no doubt, interest at the
rate of 18 per cent per annum was charged on these loans yet that was not the principal
object of advancing the loan.
9. In business various methods are adopted by a businessman for ensuring the smooth
running of his business. Very often, one of the methods is that the businessman advances
money to his supplier of goods to ensure that the supplies are regular and are made to him
rather than being diverted to other parties. There is nothing illegal in this practice and it is
widespread.
10. When we construe the provisions of the Karnataka Money Lenders Act we must see
the object for which it was made and we have to adopt the purposive construction.
11. As observed by this Court in New India Sugar Mills v. Commissioner of Sales Tax
[AIR 1963 SC 1207, p. 1213 : 1963 Supp (2) SCR 459]:-
"It is a recognized rule of interpretation of statutes that expressions used therein should
ordinarily be understood in a sense in which they best harmonize with the object of the
statute, and which effectuate the object of the legislature." (See also the decisions
mentioned in G.P. Singh's "Principles of Statutory Interpretation: 9th Edition 2004 at
Page 110).
12. To give an example, under the U.P. Bhoodan Yagna Act, 1953 the lands which were
donated by large landholders could be allotted to landless persons. It was held by this
Court in U.P. Bhoodan Yagna Samiti v. Braj Kishore [AIR 1988 SC 2239: 1988 (4) SCC
274] that the expression landless persons should be interpreted to mean landless peasants
and not landless businessman. If a literal meaning was given to the expression 'landless
persons' then even a very rich businessman who possessed hundreds of crores of rupees
can claim allotment of a piece of land on the ground that he was a landless person as he
owns no land. That could not possibly be the object of the Act. The object of the Act was
to give land to landless peasants only.
13. In view of the above discussion we are of the opinion that a purposive interpretation
has to be given to the definition of money-lenders. From this angle the appellant could
not be said to be a money-lender as he was not really doing the business of money
lending in the strict sense but was only advancing loans to secure the regular supply of
areca nuts.
14. In view of the above this appeal is allowed, impugned judgment of the High Court is
set aside and the judgment of the trial court is restored. No order as to costs.
Appeal allowed.
AIR 2008 SUPREME COURT 417 "State of Haryana v. Navneet Verma"
(From : 2004 (5) Serv LR 242 (Punj and Hary))
Coram : 2 TARUN CHATTERJEE AND P. SATHASIVAM, JJ.
Civil Appeal No. 5064 of 2007 (arising out of SLP (C) No. 23499 of 2004), D/- 31 -10
-2007.
State of Haryana and Ors. v. Navneet Verma.
(A) Constitution of India, Art.16, Art.226 - EQUALITY IN PUBLIC EMPLOYMENT -
WRITS - Abolition of post - Powers of Govt. - Scope of interference by Court - Limited
to cases where abolition is activated by mala fide.
The power of Govt. to abolish posts and the scope of power of Courts to interfere can be
summarized as follows :-
(a) The power to create or abolish a post rests with the Government;
(b) Whether a particular post is necessary is a matter depending upon the exigencies of
the situation and administrative necessity;
(c) creation and abolition of posts is a matter of Government policy and every sovereign
Government has this power in the interest and necessity of internal administration;
(d) creation, continuance and abolition of posts are all decided by the Government in the
interest of administration and general public;
(e) the Court would be the least competent in the face of scanty material to decide
whether the Government acted honestly in creating a post or refusing to create a post or
its decision suffers from mala fide, legal or factual;
(f) as long as the decision to abolish the post is taken in good faith in the absence of
material, interference by the Court is not warranted. (Para 11)
@page-SC418
(B) Constitution of India, Art.16 - EQUALITY IN PUBLIC EMPLOYMENT -
CONSTITUTIONALITY OF AN ACT - SERVICE MATTERS - Abolition of post -
Validity - Posts of Account Executive in State Bureau of public enterprise - Posts purely
temporary - Respondent appointed as Accounts Executive - Abolition of posts challenged
by him as being means to weed him out - Misunderstanding between him and his superior
and enquiry held against him pleaded in support - Record, however, showing that
abolition was made after assessment of work load and on basis of report of Committee -
Many more posts in Bureau have been abolished subsequently in effort to control non-
planned expenditure - Abolition of posts of Accounts Executive cannot be said to be mala
fide.
2004 (5) Serv LR 242 (Pand H), Reversed. (Paras 15, 16)
Cases Referred : Chronological Paras
2006 AIR SCW 1797 : AIR 2006 SC 3413 (Ref.) 10
AIR 1980 SC 1255 : 1980 Lab IC 710 (Ref.) 10
AIR 1976 SC 1199 : 1976 Lab IC 849 (Ref.) 10
AIR 1973 SC 2641 : 1973 Lab IC 1593 (Ref.) 10
AIR 1972 SC 873 : 1972 Lab IC 433 (Ref.) 10
Manjit Singh, AAG., P. N. Misra, Sr. Advocate, Hari Kesh Singh and T. V. George, with
them for Appellants; M. K. Dua and Satbir Singh, for Respondent.
Judgement
P. SATHASIVAM, J.:-Leave granted.
2. Whether the abolition of the post has been done in good faith or whether it is a
camouflage to cover up and conceal the real intention of weeding out the respondent from
service is the only question to be decided in this appeal.
BRIEF FACTS:
3. The respondent-herein was appointed as Accounts Executive in the Haryana Bureau of
Public Enterprises (in short the HBPE) on 16.07.1993. While continuing so, his services
were terminated on 31.12.1994 on the ground that the post of Accounts Executive has
been abolished. According to the respondent-herein, the third appellant applied for the
post of Financial Adviser. Even though she did not fulfill the prescribed requirement of
three years experience after doing chartered accountancy, she had been appointed as
Financial Adviser and the respondent-herein who was appointed as Accounts Executive
was required to report to her due to which she wanted him to work as her personal staff.
He did not carry out petty directions. When he brought these facts to the notice of the
Member Secretary, HBPE, he wanted him to resign the job. The Member Secretary and
the Financial Adviser seemed to have different attitude and started planning to eliminate
him. Having no other option, on 26.07.1994, he made a representation to the then Chief
Minister of Haryana. In the meanwhile, the Financial Adviser-third appellant herein
lodged a false and baseless complaint against him to the Member Secretary. Based on the
complaint of the Financial Adviser, a preliminary inquiry was conducted. However, no
action was taken against him on the basis of the report of the Inquiry Officer. But instead
of taking any action, in accordance with law, his services have been dispensed with
consequent upon the abolition of the posts of Accounts Executives.
4. According to the respondent, there is no justification for abolition of the posts of
Accounts Executive and it was done with a mala fide intention to dispense with his
services without any basis. Though two posts of Accounts Executives were advertised,
only one post was filled up by appointing him. Thus, according to him, the post was not
abolished in good faith, but this was a device to weed him out from service. Therefore,
the order of termination on the ground of abolition of post is liable to be set aside.
5. It is the case of Haryana Government and HBPE that the abolition of posts of Accounts
Executives was not done with mala fide intention or extraneous reasons. According to
them, in January, 1994, it was felt that the work of Bureau was not being performed as
per official requirements and it required restructuring of the staff vis-a-vis the work load
of the Bureau and it was observed that the contribution of two posts of Accounts
Executives, especially, when there are two posts of Accountants were not result oriented
and the work could smoothly be carried out on even without the two posts of Accounts
Executives. It was further stated that the Accountants can send their case directly to the
Financial Advisor, who was the head of the financial wing in giving advice regarding
financial offers. Consequently, two posts of Accounts Executives were abolished and
services of the respondent
@page-SC419
were terminated as he was no longer required.
6. Though a complaint was received from the Financial Adviser regarding mis-behaviour
by the respondent-herein, preliminary inquiry was conducted into the allegations made,
but no action was taken and the preliminary inquiry has no relevance with regard to the
decision taken on the abolition of the posts. The said decision was taken to abolish the
posts of Accounts Executives after obtaining permission of the government.
7. The Learned Single Judge of the Punjab and Haryana High Court, after finding that the
abolition of posts of Accounts Executives have not been done in good faith but only
intended to get rid of the respondent-writ petitioner herein, quashed the order dated
30.12.1994 abolishing the posts of Accounts Executive and consequential termination of
services of the writ petitioner. In the same order, the learned Judge issued direction for
reinstatement with all consequential benefits. The said order was challenged by the
Government of Haryana, HBPE as well as Smt. Kiran Lekha Walia, Financial Adviser,
HBPE in Letters Patent Appeal No. 163 of 1999. The Division Bench, by impugned
order, accepted the conclusion arrived at by the learned Single Judge and finding no merit
in the appeal dismissed the same with no order as to costs. Questioning the order of the
High Court, the appellants have filed the above appeal by way of special leave.
8. We have heard Mr. P.N.Misra, learned senior counsel, appearing for the appellants and
Mr. M.K.Dua, learned counsel, appearing for the respondent.
9. As observed earlier, we have to find out whether the abolition of posts of Accounts
Executive has been done in good faith or whether it is a camouflage to cover up and send
out the respondent-herein from service.
10. Before proceeding to ascertain the answer for the above question, it is useful to refer
to the appointment order of the Government of Haryana dated 13.07.1993 wherein the
respondent-herein was appointed as Accounts Executive in HBPE. Among the other
terms, clause 2 of the said order is relevant which reads as under:-
"This offer of appointment is purely against temporary post which is liable to be
abolished at any time and carries no promise of subsequent permanent employment. No
offer of permanent vacancy can be made to him at present. Consequently his services can
be terminated without notice whenever there is no vacancy against which he can be
retained."
It is clear that the respondent-herein was appointed purely against temporary post and it is
liable to be abolished at any time. The said clause makes it clear that the post has no
assurance or promise for a permanent employment. It also makes it clear that his services
can be terminated without notice whenever there is no vacancy against which he can be
retained. Now, with this background, let us consider the law laid down by this Court with
regard to power of the Government in abolishing temporary/permanent post.

i) M. Ramanatha Pillai v. The State of Kerala and Another, (1973) 2 SCC 650, a
Constitution Bench of this Court held as under: AIR 1973 SC 2641

"23. A post may be abolished in good faith. The order abolishing the post may lose its
effective character if it is established to have been made arbitrarily, mala fide or as a
mask of some penal action within the meaning of Article 311(2)."

ii) Shri Kedar Nath Bahl v. The State of Punjab and Others, (1974) 3 SCC 21, a three-
Judge Bench of this Court held in para 11 as under: AIR 1972 SC 873
"... ... ....If, in the interest of the Administration, the temporary post is abolished, the
question as to what were the personal relations between the appellant and his superiors
was irrelevant. Moreover, all that the appellant has been able to say is that his immediate
superiors in the Department were hostile to him. But here we are concerned not with the
action of his immediate superiors but the action of the Government. The decision to
discontinue the post was the decision of the Government and it is not alleged in the Writ
Petition that in taking this decision the Government acted mala fide. We, therefore, agree
with the High Court that there is no substance in the allegation that the post was
discontinued or abolished in order to punish the appellant."

iii) State of Haryana v. Shri Des Raj Sangar and Another, (1976) 2 SCC 844, this Court,
in para 8, has held: AIR 1976 SC 1199, Para 7

@page-SC420
"... ... ... The fact that the post to be abolished is held by a person who is confirmed in that
post and the post which is not abolished is held by a person who is not permanent would
not affect the legality of the decision to abolish the former post as long as the decision to
abolish the post is taken in good faith.

iv) Dr. N.C. Singhal v. Union of India and Others, (1980) 3 SCC 29. Similar issue was
considered in detail. Accepting the stand of the Government of India in abolishing the
post, this Court held thus: AIR 1980 SC 1255

"18. . ... ... ... The need for the post of the requirements of the hospital, or the need for an
ad hoc or additional appointment is a matter which the Government is competent to
decide and in the absence of requisite material the Court cannot interpose its own
decision on the necessity of creation or abolition of posts. Whether a particular post is
necessary is a matter depending upon the exigencies of the situation and administrative
necessity. The Government is a better Judge of the interests of the general public for
whose service the hospitals are set up. And whether a hospital catering to the needs of
general public providing medical relief in different specialities has need for a particular
post in a particular speciality would be better judged by the Government running the
hospital. If Government is a better judge it must have the power to create or abolish the
posts depending upon the needs of the hospital and the requirements of general public.
Creation and abolition of posts is a matter of Government policy and every sovereign
Government has this power in the interest and necessity of internal administration. The
creation or abolition of post is dictated by policy decision, exigencies of circumstances
and administrative necessity. The creation, the continuance and the abolition of post are
all decided by the Government in the interest of administration and general public (see
M.Ram-anatha Pillai v. State of Kerala.) [1974] 1 S.C.R. 515 at 520. The Court would be
the least competent in the face of scanty material to decide whether the Government acted
honestly in creating a post or refusing to create a post or its decision suffers from mala
fide, legal or factual." AIR 1973 SC 2641
v) In the recent decision - Avas Vikas Sansthan and Another v. Avas Vikas Sansthan
Engineers Assn. and Others, (2006) 4 SCC 132, analyzing all earlier decisions, this Court
has concluded:2006 AIR SCW 1797, Para 56

"59. It is well settled that the power to abolish a post which may result in the holder
thereof ceasing to be a Government Servant has got to be recognized. The measure of
economy and the need for streamlining the administration to make it more efficient may
induce any State Government to make alterations in the staffing pattern of the civil
services necessitating either the increase or the decrease in the number of posts or abolish
the post. In such an event, a Department which was abolished or abandoned wholly or
partially for want of funds, the Court cannot, by a writ of mandamus, direct the employer
to continue employing such employees as have been dislodged."
11. We summarize the power of government in abolishing a post and role of the court for
interference:
a) the power to create or abolish a post rests with the government;
b) whether a particular post is necessary is a matter depending upon the exigencies of the
situation and administrative necessity;
c) creation and abolition of posts is a matter of government policy and every sovereign
government has this power in the interest and necessity of internal administration;
d) creation, continuance and abolition of posts are all decided by the government in the
interest of administration and general public;
e) the court would be the least competent in the face of scanty material to decide whether
the government acted honestly in creating a post or refusing to create a post or its
decision suffers from mala fide, legal or factual;
f) as long as the decision to abolish the post is taken in good faith in the absence of
material, interference by the court is not warranted.
12. With the above principles, let us consider whether the abolition of the posts of
Accounts Executives are justified and consequential order of termination terminating the
respondent-herein from the said post is sustainable.
13. The main grievance of the respondent herein was that since he was also
@page-SC421
equally qualified as that of Financial Adviser, appellant No.3 herein, she was not in
favour of continuing him in HBPE as Accounts Executive. The other grievance was that
appellant No.3 used to humiliate by insulting him. According to him, instead of taking
any action against him under the service Rules, his services have been dispensed with by
abolishing the post of Accounts Executive. In other words, it is his specific case that the
abolition of post of Accounts Executive was done with a mala fide intention to dispense
with his service without any basis. The materials placed before the High Court as well as
this Court show that HBPE as well as the Financial Adviser denied those allegations. In
the light of the complaint, allegations and counter allegations, we verified the relevant
records which are available in the paper book. The first complaint (Annexure R1) dated
15.09.1993 shows that there was some misunderstanding between the Accounts
Executive and Financial Adviser, however, scrutiny of the said complaint clearly shows
that only Personal Assistant to Financial Adviser was responsible for the alleged
misbehaviour. In the complaint, Accounts Executive has specifically stated, The P.A.
attached to the F.A. grossly misbehaves and uses foul and abusive language on the
instigation of the F.A. We also perused the letter of the Member Secretary, HBPE dated
21.09.1993 addressed to the Accounts Executive and subsequent correspondence which
are available in the appeal paper book. As observed earlier, though certain
misunderstanding arose between the Accounts Executive and the Financial Adviser, it is
impossible to believe that for this reason the Government abolished the post of Accounts
Executive and consequently terminated the service of the respondent herein.
14. Learned senior counsel appearing for the appellants, by placing relevant materials,
submitted that taking note of lesser activities of HBPE and after assessing the work load
as well as sanctioned staff and after full deliberations by responsible officers, it was
recommended to the Government for abolition of two posts of Accounts Executives. On
accepting the recommendations of the Committee constituted by HBPE, the Government
passed an order according sanction to abolish two temporary posts of Accounts
Executives sanctioned for HBPE from 31.12.1994 (A/N) vide order dated 30.12.1994
(Annexure R-13). The said Government Order was issued in the name of Governor of
Haryana and with the concurrence of Finance Department. Based on the said decision of
the Government and in view of Condition No.2 of the appointment letter of the
respondent bearing No. 2/8/88-Estt-1 dated 13.07.1993, his services has been terminated
from 31.12.1994 (AN). In paragraph supra, we already referred to Condition No.2 of the
appointment order which makes it clear that the appointment of the respondent was fully
against temporary post and the same is liable to be abolished at any time and without
notice. In such circumstances, we are unable to accept the claim of the respondent that
there was no bona fide in terminating his service and the High Court has committed an
error in accepting the said claim.
15. It is also relevant to point out that by office order dated 05.08.1994 HBPE has
earmarked various types of works/subjects to be handled by Financial Adviser, 3rd
appellant herein, and Accounts Executive, respondent herein. A perusal of the said office
order, filed as Annexure R-7 in the appeal paper book, clearly demarcates various types
of work to be handled by the Financial Advisor and the Accounts Executive. Learned
senior counsel appearing for the appellants has also brought to our notice various minutes
of the meetings and the ultimate decision by HBPE for pruning their staff considering
less work load and present position of the staff sanctioned. Proceedings dated 12.01.1994
(Annexure R-3) shows that in order to assess the work load and present position of the
staff sanctioned for the HBPE a meeting of the officers of the Bureau was held by the
Chairman, HBPE on 11.09.1994 in which the Member Secretary, Personnel Adviser,
Financial Adviser and Assistant Research Officer of HBPE were participated. In the said
meeting, the Chairman felt that the work of the Bureau was not being performed as per its
mandate and suggested some restructuring of staff vis-a-vis the workload of the Bureau.
He further observed that when there are three professional posts to handle the financial
affairs, Management Affairs and Personnel Affairs of all the State Public Enterprises then
there was no necessity of having two posts of Accounts Executives having the similar
qualifications and experience as the Financial Adviser. It was also suggested that in the
next officers level meeting, the complete restructuring of
@page-SC422
staff vis-a-vis officers oriented nature of work of the Bureau and sanctioned staff may be
put up with details. Thereafter on 11.04.1994 (Annexure R-4), a meeting was held. The
minutes of the meeting shows that the details of the sanctioned staff, work load and re-
structuring was discussed in detail. It further shows that the instructions on economy in
expenditure issued by the State Government from time to time were also brought to the
notice of the Chairman for which he desired that these should also be made applicable for
the staff of the Bureau. The Chairman had also made clear that the Bureau should not
have excess staff as compared to the work load. In the said meeting while assessing the
work load and staff in the Financial Wing, it was observed that the contribution of two
posts of Accounts Executives especially when there are two posts of Accountants was not
result oriented and the work can smoothly be carried on even without the two posts of
Accounts Executives. The Chairman desired that MS/HBPE should assess the work load
and the staff strength of Finance and Accounts Wing and discuss in detail in the next
officers level meeting. It is also relevant to point out that before taking final decision; the
respondent herein by letter dated 26.07.1994 made a complaint to Hon'ble the Chief
Minister of Haryana about his difficulty in working as an Accounts Executive with HBPE
which is annexed as Annexure R-5. The complaint makes it clear that the Accounts
Executive had some grievance against the Member Secretary and the Financial Advisor
regarding the distribution of work. Further materials placed show that on the basis of the
complaint, a preliminary enquiry was conducted and ultimately no action was taken
based on the same. Finally, as stated earlier, after getting all the required details regarding
the work load and sanctioned staff of HBPE and on the basis of the report of the
Committee, a decision was taken by the Chairman that there was no requirement for two
posts of Accounts Executives and can be abolished without affecting the work of the
Bureau. Based on the said decision, the Government while accepting the same accorded
sanction to abolish two temporary posts of Accounts Executives sanctioned earlier. In
those circumstances, it is impossible to accept the stand of the respondent herein that the
posts of Account Executives were abolished in order to terminate him from the said
service. We are of the view that the High Court has committed an error in accepting the
claim of the respondent herein.
16. Rejoinder affidavit filed on behalf of the HBPE before this Court clearly shows the
various steps taken by the Committee and the ultimate decision of the Chairman for
reduction of the staff strength. It is specifically stated in the reply that in accordance with
the instructions of the Government, the requirement of posts for HBPE has been reviewed
from time to time and various posts have been abolished even after the decision to
abolish two posts of Accounts Executives. The information about the abolition of the
various posts given in the rejoinder affidavit dated 20.07.2005 is relevant which reads as
under:

S.No. Name of the Post No. of posts Date of abolition


1. Accounts Executive 2 31.12.1994
2. Senior Research Officer 1 29.6.1999
3. Assistant Research Officer 1 28.8.2000
4. Asstt. District Attorney 1 1.3.2000
5. Computer Operator 1 1.6.2001
6. JSS/Steno Typist 4 31.12.94/22.10.2001
7. Clerk 1 22.10.2001
8. Peon 4 31.12.94/22.10.2001
9. Personnel Advisor 1 14.5.2004
10. Accountant 1 14.5.2004

@page-SC423
It is clear from the materials placed in the rejoinder affidavit that the Government of
Haryana has been making earnest efforts to control its non-planned expenditure. The
rejoinder affidavit also shows that due to various efforts including the action taken by the
HBPE non-planned expenditure has been substantially reduced. In the light of the
particulars furnished, we are of the opinion that the decision to abolish the posts of
Accounts Executives was taken on the basis of the overall assessment of the work load
and staff requirement of the Bureau and the same was finally approved and sanctioned by
the Government and consequent to the said decision, the service of the respondent herein
was terminated. We hold that the entire action was taken in good faith and there is no
substantial material to arrive at a conclusion that the abolition of the post was due to
revenge against the respondent herein.
17. In the light of the above discussion, we are unable to accept the conclusion arrived at
by the High Court and we are in agreement with the decision taken by the Bureau and the
Government. Consequently, the orders passed by the High Court in L.P.A. No. 163 of
1999 dated 16.03.2004 and in C.W.P. No. 442 of 1995 dated 29.01.1999 are set aside. The
appeal is, accordingly, allowed. There shall be no order as to costs.
Appeal allowed.
AIR 2008 SUPREME COURT 423 "I. Laxma Reddy v. A. P. S. R. T. C."
(From : Andhra Pradesh)*
Coram : 2 Dr. A. PASAYAT AND P. SATHASIVAM, JJ.
Civil Appeal No. 4511 of 2005, D/- 22 -11 -2007.
I. Laxma Reddy v. A. P. S. R. T. C. and Ors.
Industrial Disputes Act (14 of 1947), Sch.2, Item 6 - INDUSTRIAL DISPUTE -
REINSTATEMENT - Reinstatement in service - Refixation of pay - National increments
earned during period of his absence - Cannot be take into consideration.
2003 (2) SCC 212, 2005 AIR SCW 3809, Rel. on. (Paras 6, 7)
Cases Referred : Chronological Paras
2005 AIR SCW 3809 : AIR 2005 SC 3791 (Rel. on) 7
(2003) 2 SCC 212 (Rel. on.) 3, 6, 7
2001 (4) Andh LD 568 2, 3
V. Sridhar Reddy and Abhijit Sengupta for Appellant; D. Mahesh Babu for Respondents.
* W. A. No. 1092 of 2003, D/- 10-7-2003 (AP).
Judgement
Dr. ARIJIT PASAYAT, J. :- Challenge in this appeal is to the order passed by a Division
Bench of the Andhra Pradesh High Court allowing the writ appeal filed by the
respondent-Andhra Pradesh State Road Transport Corporation (in short the Corporation)
and its functionaries.
2. A writ petition was filed by the appellant claiming that since an award was passed by
the Labour Court directing his re-instatement, his pay has to be fixed after taking into
consideration the notional increments. Learned Single Judge relied on a Division Bench's
decision in APSRTC Khammam Region and Anr. v. P. Nageswara Rao (2001 (4) ALD
568 (DB)) and allowed the writ petition.
3. Present respondents filed a writ appeal before the High Court questioning correctness
of the judgment. The High Court noticed that the view expressed by the Division Bench
in P. Nageswara Rao's case (supra) was dis-approved by this Court in A.P.S.R.T.C. v. S.
Narsagoud (2003 (2) SCC 212) and, therefore, allowed the writ appeal directing dismissal
of the writ petition.
4. In support of the appeal, learned counsel for the appellant submitted that when an order
of re-instatement is passed for all practical purposes there will be continuity in service
and, when the re-instatement is done the pay has to be fixed after taking into
consideration the notional increments which would have otherwise accrued.
5. Learned counsel for the respondents on the other hand supported the order passed by
the High Court.
6. The principles of law on the point are no more res integra. This Court in S.
Narsagoud's case (supra) succinctly crystallized principle of law in para 9 of the
judgment :
"We find merit in the submission so made. There is a difference between an order of
reinstatement accompanied by a simple direction for continuity of service and a direction
where reinstatement is accompanied by a specific direction that the employee shall be
entitled to all the consequential benefits, which necessarily flow from reinstatement or
accompanied by a specific direction that
@page-SC424
the employee shall be entitled to the benefit of the increments earned during the period of
absence. In our opinion, the employee after having been held guilty of unauthorized
absence from duty cannot claim the benefit of increments notionally earned during the
period of unauthorized absence in the absence of a specific direction in that regard and
merely because he has been directed to be reinstated with the benefit of continuity in
service.
7

. The position was re-iterated in A.P. State Road Transport Corporation and Ors. v. Abdul
Kareem (2005 (6) SCC 36). In view of what has been stated by this Court in S.
Narsagoud and Abdul Kareem cases (supra), there is no merit in this appeal which is
accordingly dismissed. There will be no order as to costs. 2005 AIR SCW 3809

Appeal dismissed.
AIR 2008 SUPREME COURT 424 "P. J. Chacko v. Chairman, L.I.C. of India"
(From : Kerala)
Coram : 2 S. B. SINHA AND H. S. BEDI, JJ.
Civil Appeal No. 5322 of 2007 (arising out of SLP (C) No. 23951 of 2005), D/- 20 -11
-2007.
P. J. Chacko and Anr. v. Chairman, Life Insurance Corporation of India and Ors.
(A) Insurance Act (4 of 1938), S.45 - INSURANCE - CONTRACT - Insurance policy -
Repudiation - Deliberate wrong answer given by insured having a great bearing on
contract of insurance - Policy may be repudiated.
Contract Act (9 of 1872), S.126.
A deliberate wrong answer which has a great bearing on the contract of insurance, if
discovered, may lead to the policy being vitiated in law. The purpose for taking a policy
of insurance is not very material. It may serve the purpose of social security but then the
same should not be obtained with a fraudulent act by the insured. Proposal can be
repudiated if a fraudulent act is discovered. The proposer must show that his intention
was bona fide. It must appear from the face of record. (Para 16)
In a case where insured although had in fact undergone an operation for Adenoma
thyroid, in the application form for obtaining policy answer 'no' was given to the
question whether insured had been operated, the insured died within 6 months of taking
policy and the policy was obtained through his brother who was an authorised agent of
L.I.C., it was not necessary for the insurer to establish that the suppression was
fraudulently made by the policy holder or that he must have been aware at the time of
making the statement that the same was false or that the fact was suppressed which was
material to disclose. The repudiation of policy was held proper. (Paras 16, 21)
It is true that Life Insurance Corporation being a State within the meaning of Aricle 12 of
the Constitution of India, its action must be fair, just and equitable but the same would
not mean that it shall be asked to make a charity of public money, although the contract
of insurance is found to be vitiated by reason of an act of the insured. (Para 20)
(B) Evidence Act (1 of 1872), S.115 - ESTOPPEL - CONTRACT - INSURANCE -
Estoppel - Contract of life insurance - Person making a wrong statement with knowledge
of consequence therefor - Estopped from pleading that even if such a fact had been
disclosed, it would not have made any material change.
Insurance Act (4 of 1938), Pre. (Para 15)
Cases Referred : Chronological Paras
2001 AIR SCW 161 : AIR 2001 SC 549 (Disting.) 21
AIR 1962 SC 814 9, 13
AIR 1960 Mad 484 (Disting. and Explan.) 8, 17, 19
AIR 1959 Pat 413 24
AIR 1938 Cal 641 22
R. Sathish, for Appellants; Paramjit Singh Patwalia, Sr. Advocate, S. Rajappa, for
Respondents.
Judgement
S. B. SINHA, J. :- Leave granted.
2. Application of Section 45 of the Insurance Act, 1938 is in question in this appeal which
arises out of a judgment and order dated 17th December, 2004 passed by a Division
Bench of the High Court of Kerala at Ernakulam in A.F.A. No. 18 of 2000 setting aside
the judgment and order of a learned Single Judge dated 23rd September, 2000 passed in
Appeal Suit No.633 of 1993 confirming the judgment and decree passed by the
Subordinate Judge of Kozhikode in OS No. 240 of 1990 dated 27th February, 1993.
@page-SC425
3. Plaintiffs in the suit are the appellants herein. They filed the said suit inter alia for
recovery of the amount of insurance on the death of one Chackochan (hereinafter referred
to as 'the insured'). The insured took an insurance policy on 21st February, 1987. He
died on 6th July, 1987. On his death, the appellants herein claimed the insured amount.
On the premise that the insured suppressed material facts, the policy had been repudiated
by the respondent on 10th February, 1989. Non-disclosure and mis-statement in the
proposal form to the various questions to which answers were given by the insured is said
to be the reason for the aforementioned repudiation of the contract of insurance.
4. It now stands admitted that the insured had undergone an operation for Adenoma
Thyroid. The particulars furnished by him while filling up the application form for
obtaining the said policy were as under :-
(a) Did you ever have any operation, accident or injury? The answer was "No". (b)
Have your remained absent from place of your work on ground of health during the last 5
years ? To which answer was "No". (c) What has been your state of health? The answer
was good.
The fact that the said answers were incorrect is not in dispute. The suit filed by the
appellants, however, was decreed.
5. On an appeal preferred by the respondents, on the premise that despite such wrong
answers, as the injured died on account of "polyneuritis", a learned Single Judge of the
High Court opined that there was nothing to indicate that if the injured had disclosed the
factum of previous operation, the appellant-Corporation might not have inclined to insure
and insisted on a higher premium and thus there was no material to show that the non-
disclosure was of a material fact justifying repudiation of the policy by the Corporation.
6. On an intra court appeal, the Division Bench of the High Court, however, by reason of
the impugned judgment opined that the parties are bound by the warranty clause
contained in the agreement which is also clear from the declaration signed by the insured
and the non-disclosure related to a material fact which was required to be answered
correctly under question No.22(a).
7. Mr. R. Sathish, learned counsel appearing on behalf of the appellants would submit
that a clear finding of fact having been arrived at by the trial court that despite
undergoing Adenoma Thyroid operation four years prior to the date of proposal of policy,
the cause of insured's death being 'polyneuritis' which had no connection with the
operation and the judgment of the trial court having been affirmed by the learned Single
Judge, should not have been interfered with by the Division Bench. Our attention was
further drawn to the fact that the medical officer had noted a black mole on lower aspect
of left side of neck and from Ext. A1 wherefrom it appeared that there had been no past
history suggestive of allergies, injuries, operations, diseases like rheumatic fever, syphilis
etc. and the deceased having no other complaint due to operation, the impugned judgment
cannot be sustained.
8. Life Insurance policy, it was submitted is a requirement of social security. In that view
of the matter, a suppression could not have been led to repudiation of policy, particularly
when the doctor who examined the insured was appointed by the respondent-Corporation
itself. Our attention in this behalf has been drawn to the decision of the Madras High
Court in All India General Insurance Co. Ltd. and another v. S.P. Maheshwari : AIR
1960 Madras 484 for the proposition that there exists a distinction between a
'representation' and a 'warranty'.
9

. Mr. Patwalia, learned Senior Counsel, appearing on behalf of the respondents, on the
other hand, submitted that having regard to the provisions contained in Section 45 of the
Insurance Act and the policy having been repudiated within a period of 2 years, the
impugned judgment should not be interfered with. It was submitted that undergoing of an
operation having a direct nexus with the health of the insured, suppression thereof has
rightly been considered with all seriousness by the Corporation. It was argued that the
operation underwent by the insured being a major one, was a material fact which ought to
have been disclosed. Not only the insured had given wrong answers to the questions, his
brother himself being a Life Insurance Corporation's agent and furthermore in view of the
fact that a declaration was given by the insured that no untrue averment was made
therein, the contract of insurance was null and void and all monies which had been paid
in respect thereof would stand forfeited to the AIR 1962 SC 814

@page-SC426
Corporation. Learned counsel for the Corporation has placed strong reliance on Mithoolal
Nayak v. Life Insurance Corporation of India : 1962 Suppl (2) SCR 571.
10. The basic fact of the matter is not in dispute. The insured had undergone an operation
for Adenoma Thyroid. It was a major operation. Although the said operation was
undergone by him four years prior to the date of the proposal made by him, he did not
disclose thereabout prior to obtaining the insurance policy. We may notice that he died
within six months from the date of taking of the policy i.e. on 6th July, 1987, policy
having taken on 21st February, 1987.
11. Section 45 of the Insurance Act reads as under :-
45. Policy not to be called in question on ground of mis-statement after two years,-
No policy of life insurance effected before the commencement of this Act shall after the
expiry of two years from the date of commencement of this Act and no policy of life
insurance effected after the coming into force of this Act shall after the expiry of two
years from the date on which it was effected, be called in question by an insurer on the
ground that a statement made in the proposal for insurance or in any report of a medical
officer, or referee, or friend of the insured, or in any other document leading to the issue
of the policy, was inaccurate or false, unless the insurer shows that such statement was on
a material matter or suppressed facts which it was material to disclose and that it was
fraudulently made by the policy-holder and that the policy-holder knew at the time of
making it that the statement was false or that it suppressed facts which it was material to
disclose :
Provided that nothing in this section shall prevent the insurer from calling for proof of
age at any time if he is entitled to do so, and no policy shall be deemed to be called in
question merely because the terms of the policy are adjusted on subsequent proof that the
age of the life insured was incorrectly stated in the proposal.
12. Section 45 postulates repudiation of such policy within a period of two years. By
reason of the aforementioned provision, a period of limitation of two years had, thus,
been specified and on the expiry thereof the policy was not capable of being called in
question, inter alia on the ground that certain facts have been suppressed which were
material to disclose or that it was fraudulently been made by the policy holder or that the
policy holder knew at the time of making it that the statement was false. Statute,
therefore, itself provides for the limitation for valid repudiation of an insurance policy. It
takes into account the social security aspect of the matter
13. There are three conditions for application of second part of Section 45 of the
Insurance Act which are :-
(a) the statement must be on a material matter or must suppress facts which it was
material to disclose;
(b) the suppression must be fraudulently made by the policy-holder; and
(c) the policy-holder must have known at the time of making the statement that it was
false or that it suppressed facts which it was material to disclose.

[See Mithoolal Nayak (supra)] AIR 1962 SC 814

14. The insured's brother was an agent of the Life Corporation of India. It was he, who
had asked the insured to take the insurance policy. He, being an authorized agent of the
Life Insurance Corporation, presumably knew the effect of misstatement of facts.
Misstatement by itself, however, was not material for repudiation of the policy unless the
same is material in nature.
15. The insured furthermore was aware of the consequence of making a misstatement of
fact. If a person makes a wrong statement with knowledge of consequence therefor, he
would ordinarily be estopped from pleading that even if such a fact had been disclosed, it
would not have made any material change.
16. The purpose for taking a policy of insurance is not, in our opinion, very material. It
may serve the purpose of social security but then the same should not be obtained with a
fraudulent act by the insured. Proposal can be repudiated if a fraudulent act is discovered.
The proposer must show that his intention was bona fide. It must appear from the face of
the record. In a case of this nature it was not necessary for the insurer to establish that the
suppression was fraudulently made by the policy holder or that he must have been aware
at the time of making the statement that the same was false or that the fact was
suppressed which was material to disclose. A deliberate wrong answer which has a great
bearing on the
@page-SC427
contract of insurance, if discovered may lead to the police being vitiated in law.
17

. It is no doubt true that there exists a distinction between a 'representation' and a


'warranty'. A Division Bench of the Madras High Court in S.P. Maheshwari (supra) upon
taking into consideration the history of insurance laws in United States of America, in
England and in India stated :- AIR 1962 SC 814

"(10) One great principle of insurance law is that a contract of insurance is based upon
utmost good faith Uberrima fides; in fact it is the fundamental basis upon which all
contracts of insurance are made. In this respect there is no difference between one
contract of insurance and another. Whether it be life or fire or marine the understanding is
that the contract is uberrima fides and though there may be certain circumstances from
the peculiar nature of marine insurance which require to be disclosed, and which do not
apply to other contracts of insurance, that is rather an illustration of the application of the
principle than a distinction in principle. From the very fact that the contract involves a
risk and that it purports to shift the risk from one party to the other, each one is required
to be absolutely innocent of every circumstance which goes to influence the judgment of
the other while entering into the transaction."
18. While the parties entered into a contract of insurance the same shall, subject to
statutory interdict, be governed by the ordinary law of contract. The insurer may not rely
upon the disclosures made by the insured. It may gather information from other sources.
The Madras High Court, although in our opinion, has rightly issued a note of caution to
construe a representation and warranty as a general proposition which may operate
harshly against the policy holders, itself noticed :-
"(12) The principles underlying the doctrine of disclosure and the rule of good faith
oblige the proposer to answer every question put to him with complete honesty. Honesty
implies truthfulness. But it happens that no man can do more than say what he believes to
be the truth."
19. Whether in a given case the court should take judicial notice of practice followed in
such cases or not would depend upon the facts and circumstances of each case. If it is
found that the agent himself was interested in getting the policy executed by the Life
Insurance Corporation, such common knowledge takes a back seat.

In S.P. Maheshwari (supra), it was stated : AIR 1960 SC 484

"(27) This brings us on finally to the topics of non-disclosure or misrepresentation which


are practically the positive and negative aspects of the same thing. The effect of
misrepresentation on the contract is precisely the same as that of non-disclosure; it
affords the aggrieved party a ground for avoiding the contract. There are a number of
dicta and one decision to the effect that life insurance is an exception to the general rule
that innocent misrepresentation may afford grounds for avoiding a policy and that the
misrepresentation must be fraudulent to have this effect upon a policy of life insurance.
But in order to give the insurer grounds for avoidance both under non-disclosure as well
as misrepresentations, both must relate only to material information."
The said decision, therefore, is of no assistance to the appellants herein.
20. We are not unmindful of the fact that Life Insurance Corporation being a State within
the meaning of Article 12 of the Constitution of India, its action must be fair, just and
equitable but the same would not mean that it shall be asked to make a charity of public
money, although the contract of insurance is found to be vitiated by reason of an act of
the insured. This is not a case where the contract of insurance or a clause thereof is
unreasonable, unfair or irrational which could make the court carried the bargaining
powers of the contracting parties. It is also not the case of the appellants that in framing
the aforesaid questionnaire in the application/proposal form, the respondents had acted
unjustifiably or the conditions imposed are unconstitutional.
21

. In Life Insurance Corpn. Of India and Ors. v. Asha Goel (Smt) and Anr. [(2001) SCC
160], whereupon reliance has been placed by Mr. Sathish, it was held : 2001 AIR
SCW 161, (Para 12)

The contracts of insurance including the contract of life assurance are contracts uberrima
fides and every fact of material (sic material fact) must be disclosed, otherwise, there is
good ground for rescission of the contract. The duty to disclose material facts continues
right up to the conclusion of the
@page-SC428
contract and also implies any material alteration in the character of the risk which may
take place between the proposal and its acceptance. If there are any misstatements or
suppression of material facts, the policy can be called into question. For determination of
the question whether there has been suppression of any material facts it may be necessary
to also examine whether the suppression relates to a fact which is in the exclusive
knowledge of the person intending to take the policy and it could not be ascertained by
reasonable enquiry by a prudent person.
It has not been shown in this case that repudiation of the contract of insurance was not
done by the respondent with extreme care and caution or was otherwise invalid in law.
The Division Bench of the High Court has taken all the aspects of the matter in
consideration and, in our opinion arrived at a just decision.
22. Strong reliance has been placed by the learned counsel for the appellants on Allianz
Und Stuttgarter Life Insurance Bank Ltd. v. Hemanta Kumar Das [AIR 1938 Cal 641]
wherein in regard to some purported statements made by the proposor in regard to his age
was not found to be material as would appear from the following :
It is to be borne in mind that this was an insurance by a man who admittedly was, at any
rate, at the age of over forty-five years. He himself stated that he was fifty four.
Therefore, the transaction came within the category of those proposals which require at
the outset the furnishing by the "proponents" of proof of their age. Noot Behari Das was
required to furnish proof of his age. He produced a horoscope. The horoscope was
accepted by the company as being sufficient. Therefore, we may take that the company
issued the policy upon the footing that they were insuring the life of a man whose age
was fifty four. This is not a case where the proposer says that his age was fifty four and
the Company merely accepted that statement at its face value and proceeded to issue a
policy on that footing and subsequently, either shortly afterwards or a long time
afterwards, admitted the age as stated in the policy in accordance with the provisions of
Cl.9(2) thereof. This was a case where the whole transaction from the very beginning
proceeded upon the basis that the company had satisfied themselves that the proposer was
of the age of fifty four and then issued the policy accordingly. In my view therefore the
admission contained in the endorsement at page 3 of the policy is of such a character that
the defendants when the policy matured could not be heard to say that the age of the
insured was anything different from what he himself had stated it to be in February 1934.
It is not necessary that one should apply in terms of the principle of estoppel, because that
is merely a rule of evidence. In my view, this matter goes far deeper than that. The
question of the age of the deceased was a definite and determining factor in the
transaction from the very outset.
23. It is not a case where the company had further enquired into the matter in regard to
the question as to whether the proposor was operated upon or not.
24. In Ratan Lal and Anr. v. Metropolitan Insurance Co. Ltd. [AIR 1959 Pat 413], a
distinction was made between as to what is material and what is not material. In regard
to the disclosure of facts in that case itself, it was opined :
The well-settled law in the field of insurance is that contracts of insurance including the
contracts of life assurance are contracts uberrima fides and every fact of materiality must
be disclosed otherwise there is good ground for rescission. And this duty to disclose
continues up to the conclusion of the contract and covers any material alteration in the
character of the risk which may take place between proposal and acceptance.
25. Ratio of the said decision, therefore, instead of assisting the case of appellants, runs
counter to his contention.
26. Keeping in view the facts and circumstances of the case, we are of the opinion that no
case has been made out for our interference with the impugned judgment. The appeal
fails and is accordingly dismissed. No costs.
Appeal dismissed.
@page-SC429
AIR 2008 SUPREME COURT 429 "Manohar Shankar Nale v. Jaipalsing Shivlalsing
Rajput"
(From : Bombay)
Coram : 2 S. B. SINHA AND H. S. BEDI, JJ.
Civil Appeal No. 5323 of 2007 (arising out of SLP (C) No. 25959 of 2005), D/- 20 -11
-2007.
Manohar Shankar Nale and Ors. v. Jaipalsing Shivlalsing Rajput and Ors.
(A) Civil P.C. (5 of 1908), S.114, O.47, R.1 - REVIEW - DOCTRINES - Review -
Doctrine of merger - Does not apply when review petition is dismissed.
Where a review petition is dismissed, the doctrine of merger will have no application
whatsoever. It is one thing to say that the judgment debtor was entitled to file an
application for review in terms of S. 114 read with O. 47, R. 1 of Civil P.C. but it is
another thing to say that the decree passed in favour of the decree holder merged with the
order dismissing the review application. Matter might have been different, if the review
application had been allowed either wholly or in part in terms whereof an application for
execution of the decree could have been filed only in terms of the modified decree.
(Para 11)
(B) Civil P.C. (5 of 1908), O.21, R.35, O.47, R.1 - EXECUTION - LIMITATION -
POSSESSION - APPEAL - Execution - Limitation - Decree for possession and mesne
profits - Review petition against, dismissed - Appeal to S. C. against dismissal of review
petition - Stay granted by S. C. to order dismissing review petition - Does not bar
execution of decree - Further direction given by S. C. that computation of mesne profit
would go on and the same would be deposited by the appellant - Does not have effect of
staying operation of decree - Neither order of dismissal of review petition nor stay
granted by S. C. to that order have effect of postponing starting point of limitation or
execution.
Limitation Act (36 of 1963), Art.136.
W. P. No. 5927 of 2004, D/- 26-7-2005 (Bom), Reversed. (Paras 11, 15, 16, 17)
(C) Civil P.C. (5 of 1908), S.2(2) - DECREE - POSSESSION - Decree - Composite
decree - Decree for possession and for computation of mesne profits - Not composite
decree. (Para 10)
Cases Referred : Chronological Paras
2007 AIR SCW 1011 : AIR 2007 SC 1077 (Ref.) 13
2006 AIR SCW 6446 : 2007 CLC 3 (Ref.) 14
2005 AIR SCW 1381 : AIR 2005 SC 1944 (Disting) 17
1999 AIR SCW 3401 : AIR 1999 SC 3421 (Ref.) 13
AIR 1923 Cal 288 (Ref.) 7
Satyajit A. Desai, Mrs. Anangha S. Desai, Amol N. Suryawanshi and Venkateswara Rao
Anumolu for Appellants; Makarand D. Adkar, Vijay Kumar and Vishwajit Singh for
Respondents.
Judgement
S. B. SINHA, J. :- Leave granted.
2. Applicability of the provisions of Article 136 as contained in the Schedule appended to
the Limitation Act, 1963 is in question in this appeal which arises out of a judgment and
order dated 26.7.2005 passed by a learned Single Judge of the Bombay High Court,
Nagpur Bench at Nagpur in Writ Petition No. 5927 of 2004 affirming an order dated
26.10.2004 passed by the Civil Judge, Junior Division in Regular Darkhast No. 32 of
2001.
3. Shivlalsing, predecessor-in-interest of the respondents filed a suit in the Court of Civil
Judge, Junior Division, Malkapur. The said suit was marked as Regular Civil Suit No.
250/1965. It was dismissed on 24.12.1968. An appeal preferred thereagainst was also
dismissed. However, a second appeal filed by the plaintiff-decree holder was allowed
upon setting aside the judgment and decree of the Courts below, the operative portion
whereof reads as under :
"For the reasons stated in the accompanying judgment, the court allows the appeal, sets
aside the decrees of both the courts below dismissing the suit and instead. The Court
orders that the plaintiffs suit for possession of 32 gunthas area, as shown in the copy of
the map Exh. 30, from out of Survey No. 59/1 area 3 acres 12 gunthas, shall stand
decreed with costs throughout. The appellant-plaintiff is also entitled to an enquiry under
the provisions of the Order XX Rule 12(1) C.P.C. for mesne profit in respect of the suit
land from the date of the suit till the actual delivery of possession.
4. An application for review thereof was filed by the predecessor-in-interest of the
appellants herein. The said review petition was dismissed by an Order dated 12.8.1985.
Appellants preferred a special leave petition thereagainst before this Court and leave
having been granted, the matter was marked
@page-SC430
as Civil Appeal No. 1836 of 1986.
5. By an Order dated 21.3.1988, this Court passed an order of stay in the following
terms :
The Application for Stay above mentioned being called on for hearing before this Court
on the 21st day of March, 1988 upon hearing Counsel for the parties herein. This Court
Doth Order that the order of this Court dated the 8th May, 1986 passed in Civil
Miscellaneous Petition No. 10447 of 1986 be and is hereby confirmed and that pending
the hearing and final disposal by this Court of the appeal above mentioned, the operation
of the Order dated the 1st July, 1985 of the High Court of Judicature at Bombay (Nagpur
Bench) Nagpur in Miscellaneous Civil Application No. 134 of 1984 in Second Appeal
No. 158 of 1972 be and is hereby stayed and the Civil Judge, Junior Division, Malkapur,
Maharashtra be and is hereby directed to ascertain the amount of mesne profits which
shall be deposited by the appellant herein.
6. The said appeal, however, came to be dismissed as being incompetent by this Court
opining that no appeal lay in terms of Order 47 Rule 7 of the Code of Civil Procedure
rejecting a review application.
7. Admittedly, an application for executing the said decree was filed by the decree holder
only on 10.12.2001. Appellant, having been noticed in the said execution proceeding,
inter alia, raised a contention about maintainability thereof on the premise that the same
was barred by limitation. The Executing Court by an Order dated 26.10.2004 rejected the
said contention holding :
"It is only a disputed point as to whether the decree was stayed by Hon'ble Apex Court. If
it was stayed, the Darkhast would be within limitation and if not, it would not be Record
of original darkhast is called for copy of writ in respect of stay is on the said record, from
Hon'ble Apex Court stayed the effect and operation of the order passed by Hon'ble High
Court while deciding review petition against the order passed in Second Appeal. It is
submitted by Shri Tarkase, learned counsel for the J.D. that the original decree was not
stayed. It is submitted by Shri Rawant, learned counsel for the D.H. that the said order
had the effect of staying the original decree as it merged into the orders passed by
Hon'ble High Court. I am in agreement with this submission of Shri Rawant. Shri
Tarkase, learned counsel for the J.D. placed his reliance on the decision in the case of
Ram Ratan Choudhary Vs- Upendra Chondru Das, A.I.R. 1923 Cal. 288. But, there was
to stay is that case (sic) and hence the ratio is not applicable to the facts of the present
case. Hence, there is no substance in the submission that the execution is not within
limitation. Hence, I hold that the execution is well within limitation as an order of stay to
the order of Hon'ble High Court was operating.
8. As noticed hereinbefore, a learned Single Judge of the Bombay High Court, Nagpur
Bench, Nagpur by reason of the impugned judgment dated 26.7.2005 dismissed the writ
petition preferred by the appellant thereagainst stating;
"The learned trial court while rejecting the application has found that the judgment and
decree dated 02/09/1983 had merged into the order of this Court while rejecting review
dated 01/07/1985. He, therefore, submits that the effect of the stay granted by the Apex
Court was to stay the execution of the decree and as such the execution of the decree
cannot be proceeded till vacation of the stay by the Apex Court. The execution of
proceedings have been filed within a period of 12 days from the date on which the Apex
Court had vacated the stay.
In that view of the matter, I do not find any perversity or infirmity in the jurisdiction
exercised by the learned trial court so as to warrant any interference of this court in the
extraordinary jurisdiction under Article 227 of the Constitution of India. Hence the
petition is rejected.
9. Mr. Satyajit A. Desai, learned counsel appearing on behalf of the appellant in support
of the appeal would submit that the learned Courts below acted illegally and without
jurisdiction in passing the impugned judgment insofar as they failed to take into
consideration that no order having been passed by this Court staying execution of the
proceedings or operation of the judgment and decree, the impugned judgment must be set
aside being wholly unsustainable in law.
Mr. Makarand D. Adkar, learned counsel appearing on behalf of the respondents, on the
other hand, would submit that a composite decree having been passed by the High Court
and furthermore in view of the fact that this Court in its order dated
@page-SC431
21.3.1988 clearly directed computation of mesne profit, the execution case was not
barred by limitation.
It was further submitted that the remedy of review being a statutory remedy available to
an aggrieved party, an Order passed in such proceedings would merge with the main
judgment and decree.
10. The decree passed by the High Court in the aforementioned Second Appeal No. 158
of 1972 is in two parts. The Court granted a decree for possession. In respect of an area
measuring 32 gunthas, as delineated in the map Exh. 30, out of Survey No. 59/1
measuring 3 acres 12 gunthas. Respondent was also found to be entitled to an enquiry in
terms of Order 20 Rule 12 of the Code of Civil Procedure in regard to computation of
mesne profit from the date of the institution of the suit, till the date of the actual delivery
of possession. It is therefore, not correct to contend that the decree was a composite one.
The proceeding for computation of mesne profit required to be undertaken in terms of
Order 20 Rule 12 of the Code of Civil Procedure was subject to institution of a
proceeding but, by reason thereof, the execution of the decree in regard to the possession
of 30 gunthas of land was not required to be awaited till the outcome.
11. It is also incorrect to contend that in a case of this nature, namely where a review
petition was dismissed, the doctrine of merger will have any application whatsoever.
It is one thing to say that the respondent was entitled to file an application for review in
terms of Section 114 read with Order 47 Rule 1 of the Code of Civil Procedure, but it is
another thing to say that the decree passed in favour of the respondent merged with the
order dismissing the review application. Matter might have been different, if the review
application had been allowed either wholly or in part in terms whereof an application for
execution of the decree could have been filed only in terms of the modified decree.
It is not the contention of the respondent that any order of stay was passed in the review
application. There was, thus, no bar in proceeding with the execution case.
12. Keeping in view the aforenoticed factual backdrop, we may notice the law applicable
herein.
Article 136 of the Limitation Act reads as under:

Description of application Period of limitation Time from which period


begins to run
136 For the execution of any decree (other than a decree granting a mandatory
injunction) or order of any civil Court. Twelve years When the decree or order be
comes enforceable or where the decree or any subsequent order directs any payment of
money or the delivery of any property to be made at a cer-tain date or at recurring
periods,when default in making the pay-ment or delivery in respect of which execution is
sought,takes place :
Provided that an application for the enforcement or execution of a decree granting a
perpetual injunction shall not be subject to any period of limitation.

13

. The decree of possession passed in favour of the respondent became enforceable


immediately on its passing. An execution petition therefor was required to be filed within
a period of 12 years. In W.B. Essential Commodities Supply Corpn. v Swadesh Agro
Farming and Storage Pvt. Ltd., 1999 AIR SCW 3401, Paras 8, 10 and 11)

@page-SC432
[(1999) 8 SCC 315], this Court opined;
"(1) A decree or order is said to be enforceable when it is executable. For a decree to be
executable, it must be in existence. A decree would be deemed to come into existence
immediately on the pronouncement of the judgment. But it is a fact of which judicial
notice may be taken of that drawing up and signing of the decree takes some time after
the pronouncement of the judgment; the Code of Civil Procedure itself enjoins that the
decree shall be drawn up expeditiously and in any case within 15 days from the date of
the judgment. If the decree were to bear the date when it is actually drawn up and signed
then that date will be incompatible with the date of the judgment. This incongruity is
taken care of by Order 20 Rule 7 CPC which, inter alia, provides that the decree shall
bear the date and the day on which the judgment was pronounced."
(2) "Rule 6-A enjoins that the last paragraph of the judgment shall state in precise terms
the relief which has been granted by such judgment. It has fixed the outer time-limit of 15
days from the date of pronouncement of the judgment within which the decree must be
drawn up. In the event of the decree not so drawn Rule 1 of Order 41 CPC without filing
a copy of the decree appealed against and for that purpose the last paragraph of the
judgment shall be treated as a decree. For the purpose of execution also, provision is
made in clause (b) of the said sub-rule which says that so long as the decree is not drawn
up, the last paragraph of the judgment shall be deemed to be a decree. Clause (b) has
thus enabled the party interested in executing the decree before it is drawn up to apply for
a copy of the last paragraph only, without being required to apply for a copy of the whole
of the judgment. It further lays down that the last paragraph of the judgment shall cease
to have the effect of the decree for purposes of execution or for any other purposes when
the decree has been drawn up."
(3) "It follows that the decree became enforceable the moment the judgment is delivered
and merely because there will be delay in drawing up of the decree, it cannot be said that
the decree is not enforceable till it is prepared. This is so because an enforceable decree
in one form or the other is available to a decree-holder from the date of the judgment till
the expiry of the period of limitation under Article 136 of the Limitation Act."

See also Hasham Abbas Sayyad Vs. Usman Abbas Sayyad and Ors. [(2007) 2 SCC 355]
2007 AIR SCW 1011

14

. In Raghunath Rai Bareja and Anr. v. Punjab National Bank And Others [(2006) 13
SCALE 511], this Court opined; 2006 AIR SCW 6446

"9. Under Article 136 to the Schedule of the Limitation Act, 1963 the period for applying
for execution of any decree is 12 years from the date when the decree becomes
enforceable. Since in the present case the final decree was passed and became
enforceable on 15.1.1987, the period of limitation for filing an execution application
expired on 15.1.1999".
15. The order of purported stay passed by this Court in terms of its Order dated 21.3.1988
is also of no assistance to the plaintiff-decree holder. The Special Leave Petition was
filed only against the Order dated 1.7.1985 refusing to review its judgment and decree
dated 2.9.1983. The stay of operation of the Order dated 1.7.1985 for all intent and
purport was meaningless as the review petition already stood dismissed.
16. Further direction of this Court that computation of mesne profit would go on and the
same would be deposited by the appellant is of no consequence inasmuch as by reason
thereof neither proceeding was stayed nor the operation of the judgment and decree had
been stayed. In fact, it was an order passed in favour of the decree holder. The said
direction did not come in his way to execute the decree for possession.
17. We, therefore, do not see any reason to hold that the said order could be construed to
be one passed in favour of the decree holder-respondent as was contended or otherwise.

Mr. Adkar relies upon the decision of this Court in Rekha Mukherjee v. Ashis Kumar Das
and Others [(2005) 3 SCC 427] which has no application in the present case. 2005
AIR SCW 1381

18. We, therefore, are of the opinion that the Executing Court as well as the High Court
committed a manifest error in opining that the Execution Petition was not barred by
limitation. The appeal is allowed. The impugned judgment is set aside with costs.
Counsel fee quantified at Rs. 5,000/-.
Appeal allowed.
@page-SC433
AIR 2008 SUPREME COURT 433 "M. P. State Electricity Board v. Grasim Industries
Ltd."
(From : Madhya Pradesh)*
Coram : 2 Dr. A. PASAYAT AND R. V. RAVEENDRAN, JJ.
Civil Appeal Nos. 1019 with 1026-1034 and 3223 etc. of 2006, D/- 12 -11 -2007.
M. P. State Electricity Board and Anr. v. Grasim Industries Ltd.
(A) Electricity (Supply) Act (54 of 1948), S.49, S.79 - ELECTRICITY - SALE - Sale of
electricity - Security deposit - Board not statutorily obligated to pay interest thereon -
Condition in agreement as to payment of interest - Can be deleted depending on factual
scenario. (Paras 13, 15)
(B) Electricity (Supply) Act (54 of 1948), S.49, S.79 - ELECTRICITY - SALE - Sale of
electricity - Requirement as to security deposit - Object of.
The deposit though called security deposit is really an adjustable advance payment of
consumption charges. The payment is in terms of the agreement interpreting the
conditions of supply. This security deposit is revisable from time to time on the basis of
average consumption charges depending upon the actual consumption over a period.
(Para 11)
Cases Referred : Chronological Paras
1993 AIR SCW 2025 : AIR 1993 SC 2005 (Foll.) 2, 6, 7, 14
M. L. Verma, S. S. Vaidyanathan, M. L. Jaiswal, A. K. Chitale and Kailash Vasdev. Sr.
Advocate Prakash Shrivastava, Akshat Shrivastava, Ms. Pradipti Shrivastava, Miten
Mahapatra, Sakesh Kr. Rohit Singh, Dharmendra Kumar Sinha, M. R. Vij, Niraj Sharma,
Praveen Kumar, K. V. Vishwa-nathan, Vikram Bajaj, Sanjeev Kumar, Vishal Gupta (for
M/s. Khaitan and Co.), S. K. Verma, Sushil Kumar Jain, Sanjay Grover and Nikilesh R.
(for M/s. Temple Law Firm), with them for Appellant.
* W. P. No. 751 of 1996 (107/99), D/- 9-10-2003 (MP) (Indore Bench).
Judgement
Dr. ARIJIT PASAYAT, J. :- In each of the appeals challenge is to the order passed by a
Division Bench of the Madhya Pradesh High Court, Indore Bench, in Letters Patent
Appeals/writ petitions filed by the respondents in each case. CA Nos.1033 and 1034 of
2006 have been filed with leave to file special leave petition. It is to be noted that while
allowing the writ petitions filed, the High Court placed reliance on the judgment rendered
in the Letters Patent Appeal filed under clause 10 of the Letters Patent by Grasim
Cement, Raipur, i.e. LPA 20207 of 1997. In the cases where the Letters Patent Appeals
were filed, learned Single Judge had decided in favour of the appellant-Board.
2

. Challenge in the writ petitions filed, which were decided related to the illegality of
action taken by the appellant-Board in deleting Clauses 21(f) and 21(g) of the Board's
General Conditions for Supply of Electrical Energy and The Sale of Miscellaneous and
General Charges. These related to agreement for payment of interest on security deposits.
The notification is dated 24.1.1996. Learned Single Judge in the cases which were subject
matter of the Letters Patent Appeal held that such a course was permissible. Reliance for
the purpose was placed on a decision of this Court in Ferro Alloys Corpn. Ltd. V. A.P.
State Electricity Board and Anr. (1993 Supp (4) SCC 136). While deciding the appeals
and the writ petitions, the Division Bench held that the view of the learned Single Judge
is not correct and for the purpose relied on paragraph 158 of the judgment in Ferro Alloys
case (supra). 1993 AIR SCW 2025

3. Mr. C.S. Vaidyanathan, learned senior counsel for the appellant-Board submitted that
the Division Bench read only a part of paragraph 158 of the judgment and not the
relevant part which empowers the Board to delete such a condition.
4. It is submitted that notification dated 24/1/1996 was issued in exercise of powers
conferred under Section 49 of the Electricity (Supply) Act, 1948 (in short the 'Supply
Act').
5. Learned counsel for the respondent, on the other hand, observed that this Court
categorically in paragraph 158 noted the lack of power to delete the condition relating to
payability of interest on security deposits.
6

. It is to be noticed that in Ferroy Alloys case (supra), this Court was dealing with two
categories of consumers in different States. One category related to Boards' regulations
for the States of Andhra Pradesh, Uttar Pradesh and Bihar, where there was provision for
payment of interest. In respect of some other States such as, Rajasthan and Orissa, there
1993 AIR SCW 2025

@page-SC434
was no such provision. This Court in paragraphs 143 and 145 held that where there is no
provision for payment of interest, the same is not illegal. We are not concerned with that
category of cases.
7

. Since the fate of these appeals primarily depends upon the view expressed by this Court
in Ferro Alloys case (supra) at paragraph 158, this paragraph needs to be noticed. The
same reads as follows: 1993 AIR SCW 2025, Para 135
"In view of the above finding, upholding the clause relating to non-payment of interest,
for example, Rajasthan and Orissa, what is to happen to such of those cases where
interest is provided like Andhra Pradesh, Uttar Pradesh and Bihar? In all those cases
wherever the Electricity Boards have framed a provision for payment of interest after
adjusting its finances at a stated rate they cannot be allowed to delete such a clause. The
provision for interest has been made by the various Boards having regard to the overall
budgetary and financial position and further, keeping in view the quantum and mode of
security deposit and billing and recovery practice. Nor again, could the Board withhold
payment of interest on the basis of this judgment. However, if there is any change in the
circumstances affecting the budgetary and financial position, the Board can examine the
case and decide the future course of action. But any change resulting in non-payment or
reduction of interest will have to be justified by cogent reasons and materials having a
bearing on the financial position of each Board and facts and circumstances of each
case."
(Underlined for emphasis)
8. Indisputably a bare reading of paragraph 158 quoted above shows that it is permissible
for the Board to take a decision relating to the desirability for payment of interest on
security deposits or otherwise.
9. Each of the Electricity Boards before us is a State within the meaning of Article 12 of
the Constitution of India. The Boards are different from licensees. Each of the Boards has
framed its own terms and conditions of supply. One such condition relates to security
deposits. Such a deposit varies from Board to Board. For example, under the terms and
conditions notified by Andhra Pradesh Electricity Board under Condition No. 28.1.1, the
consumer is required to deposit with the Board a sum in cash equivalent to estimated
three months consumption charges. In the case of Rajasthan, the security is in the form of
cash for one month and bank or insurance guarantee for two months.
10. The legislative sanction behind the power of the Board to direct a consumer to furnish
security may be examined. It has already been seen that the Supply Act is complementary
to the Electricity Act, 1910. Section 26 of the Supply Act states that the Board shall have
all the powers and obligations of a licensee under the Electricity Act. And this shall be
deemed to be a licence of the Board for the purpose of the Act. Under the regulations
framed by the Board in exercise of powers of Section 49 read with Section 79(j) the
consumer is only entitled and the Board has an obligation to supply energy to the
consumer upon such terms and conditions as laid down in the regulations. If, therefore,
the regulations prescribed a security deposit that will have to be complied with. It also
requires to be noticed under Clause (6) of Schedule II of the Electricity Act that the
requisition for supply of energy by the Board is to be made under proviso (a) after a
written contract is duly executed with sufficient security. This, together with the
regulations stated above, would be enough to clothe it with legal sanction. In cases where
regulations have not been made Rule 27 of the Rules made under the Electricity Act
enables the adoption of model form of draft conditions of supply. Annexure VI in Clause
14 states that the licensee may require any consumer to deposit security for the payment
of his monthly bills for energy supplied and for the value of the meter and other apparatus
installed in his premises. Thus, the Board has the power to make regulations to demand
security from the consumers.
11. The next question will be: what is the object in demanding security? The deposit
though called security deposit is really an adjustable advance payment of consumption
charges. The payment is in terms of the agreement interpreting the conditions of supply.
This security deposit is revisable from time to time on the basis of average consumption
charges depending upon the actual consumption over a period. This is the position under
the terms of supply of energy with reference to all the Boards.
12. For supply of electricity the Board needs finance for production, supply and
@page-SC435
other charges necessary for supply of electricity. For this purpose, it takes loans from
various financial institutions. This is best illustrated if one looks at the transactions of
Punjab Electricity Board where electric energy is generated through hydro as well as
thermal plants for ultimate sale to the consumers. Of the total power generated about 50
per cent is through hydro plants. The remaining energy is generated through thermal
power plants which are operated on coal/oil. Due to limited hydro resources within the
State of Punjab the dependency on power on thermal plants is on the increase. The
present requirement for working of thermal plants is more than 52 lakh tonnes of coal per
annum. In addition, 60 thousand kilo litre of furnace oil is required. The coal
companies/Coal India Limited together with major suppliers or power plant like M/s.
BHEL demand cost of coal/spares/ projects in advance for the supply of material. The
Board is also required to purchase power from Central projects N.T.P.C., N.H.P.C. in
order to meet the demand for power by the consumers. For purchase of such power again
advance payments are made by the Board. On such advances the Board is not paid
interest. The effect is, the Board is obliged to bear the liability of hundreds of crores of
rupees per annum. It has no option but to pay the charges and deposits in order to keep
the power available at a level to meet with the demand of the consumers. It is the case of
the Board that it has opened letters of credit by making advance deposits in favour of
National Thermal Power Corporation and the suppliers. Coal India Limited has also
asked the Board to open revolving letters of credit in favour of coal companies/Coal India
Limited. Despatch of coal is only against the letter of credit.
13. In the above premises, it follows that there is nothing to indicate under the scheme of
the Electricity Act or Schedule VI of the Supply Act that interest must be paid on the
security deposit.
14

. These aspects have been highlighted in Ferro Alloys case (supra). 1993 AIR SCW 2025

15. Obviously, the Division Bench of the High Court has not considered the effect of the
underlined observations of this Court regarding the permissibility to delete provisions for
payment on security deposits, as noted in the said paragraph 158. This has to be decided
on the factual position of each case. We find that in the order of the learned Single Judge
which formed the subject matter of challenge in the LPAs, there are certain factual
conclusions arrived at by learned Single Judge. The Division Bench has not dealt with the
acceptability or otherwise of the view and has only referred to paragraph 158 to hold that
it cannot be done, overlooking the underlined portion relating to the permissibility for
such a course to be adopted.
16. In the aforesaid circumstances, we deem it proper to set aside the impugned judgment
in each case and remit the matter to the High Court for a fresh consideration in the light
of what has been stated in paragraph 158 so far as it relates to the Boards' powers to
delete provision relating to payment of interest on security deposits on the factual
scenario. We make it clear that we have not expressed any opinion on the merits of the
case.
17. The appeals are disposed of accordingly with no orders as to costs.
Order accordingly.
AIR 2008 SUPREME COURT 435 "Union of India v. Mahajabeen Akhtar"
(From : Delhi)
Coram : 2 S. B. SINHA AND H. S. BEDI, JJ.
Civil Appeal No. 5087 of 2007 (arising out of SLP (C) No. 6635 of 2005), D/- 1 -11
-2007.
Union of India and Anr. v. Mahajabeen Akhtar.
Constitution of India, Art.39(d), Art.16 - DIRECTIVE PRINCIPLES - EQUALITY IN
PUBLIC EMPLOYMENT - PAY PARITY - Equal pay for equal work - Applicability -
Persons have to be similarly situated in all respects - Equivalence in pay depends on
many factors - Research Asst. (Urdu) - Held not entitled to pay parity with Research Asst.
in other languages.
A large number of factors, namely, educational qualifications, nature of duty, nature of
responsibility, nature of method of recruitment etc. will be relevant for determining
equivalence in the matter of fixation of scale of pay. Art. 39(d) as also Art. 14 of the
Constitution of India must be applied, inter alia, on the premise that equality clause
should be invoked in respect of the people who are similarly situated in all respects.
(Paras 19, 25)
@page-SC436
The respondent was working as Research Assistant (Urdu) in Bureau of Promotion of
Urdu language. The Central Govt. constituted National Council for Promotion of Urdu
Language (NCPUL) in place of the Bureau of Promotion of Urdu Language. NCPUL
started functioning from 1-4-1996. Employees of the Bureau were given an option either
to continue to work in the Govt. Department or get themselves transferred to NCPUL.
Respondent opted for Govt. service. She was redeployed as Librarian in National Gallery
of Modern Art and designated as Assistant Librarian and Information Assistant. The scale
of pay of Research Assistants was revised by 5th Pay Commission. The respondent
claimed pay parity with Research Assistants in Central Hindi Directorate of the Ministry
of Human Resources Development, Department of Education, Central Institute of Indian
Language, commission for Scientific and Technical Terminology, Ministry of Human
Resources Development, Department of Education and Bureau for Promotion on ground
that respondent had been in the job of the Bureau of Promotion of Urdu Language as on
1-1-1996 from which date the recommendations of the Fifth Pay Commission came to be
implemented.
Held when the requisite criteria in regard to appointment (essential qualifications),
promotion, transfer as well as the nature of duties required to be performed by the
incumbents of posts vis-a-vis that of Research Assistant (Urdu) and those of Research
Assistants in other languages is different. Not only that, the nature of duties is also
different. The respondent cannot claim pay parity. Moreover respondents pay has been
revised keeping in view educational qualification and other relevant factors by an expert
body, no exception thereto can be taken. (Paras 17, 18, 24, 26)
Cases Referred : Chronological Paras
2006 AIR SCW 3013 : AIR 2006 SC 2296 : 2006 (4) AIR Kar R 652 : 2006 (4) ALJ 590:
2006 (3) AIR Jhar R 537 (Rel on) 21
2006 AIR SCW 6035 : AIR 2007 SC 262 : 2007 (1) AIR Kar R 555 (Ref) 27
2005 AIR SCW 5632 : AIR 2006 SC 161 : 2005 Lab IC 4322 : 2006 (1) AIR Kar R 80:
2006 (1) AIR Bom R 158 : 2006 (1) AIR Jhar 238 (Rel on) 22
(2004)1 SCC 347 (Rel on) 20
2003 AIR SCW 2278 : AIR 2003 SC 2305 : 2003 Lab IC 1749 : 2003 All LJ 1435
(Disting) 25
2002 AIR SCW 2896 : AIR 2002 SC 2589 : 2002 Lab IC 2630 (Ref) 19, 27
(2000)8 SCC 580 (Ref) 19
1999 AIR SCW 1153 : AIR 1999 SC 1356 : 1999 Lab IC 1639 (Disting) 23
1992 AIR SCW 1071 : AIR 1992 SC 1203 : 1992 Lab IC 1047 (Ref) 19
AIR 1991 SC 1367 : 1991 Lab IC 1120 (Disting) 23
AIR 1989 SC 19 : 1989 Lab IC 1146 19
AIR 1988 SC 1663 :L 1989 Lab IC 1314 (Disting) 25
A. Sharan, ASG, Ms. Sunita Sharma, Ms. Sushma Suri, for Appellants; Sudhir
Kulshreshtha, for Respondent.
Judgement
S. B. SINHA, J. :-Leave granted.
2. Applicability of the doctrine of 'equal pay for equal work' is in question in this appeal
which arises out of a judgment and order dated 19.8.2004 passed by a Division Bench of
the High Court of Delhi in Civil Writ Petition No.3719 of 2002 dismissing the writ
petition filed by the appellant questioning an order dated 11.9.2000 passed in Original
Application No.52 of 2000 by the Central Administrative Tribunal directing to consider
the question of grant of replacement pay-scale of Rs.6500-10500 to the respondent, with
consequential benefits in her favour.
3. Basic fact of the matter is not in dispute.
4. Respondent herein was appointed as Technical Assistant of Urdu Language in the
Bureau of Promotion of Urdu Language. She was placed in the pay scale of Rs.425-700.
She was promoted as Research Assistant in the scale of pay of Rs.550-900. The said scale
of pay was revised to Rs.1640-2900 on the recommendations of the Fourth Pay Revision
Commission.
5. The Central Government constituted National Council for Promotion of Urdu
Language (NCPUL) in place of the Bureau of Promotion of Urdu Language. NCPUL
started functioning from 1.4.1996. Employees of the Bureau were given an option either
to continue to work in the Government Department or get themselves transferred to
NCPUL. Respondent opted for Government service. Her name was, therefore, referred to
surplus cell for redeployment. She was redeployed as Librarian in National Gallery of
Modern Art and designated as Assistant Librarian and Information Assistant. Her pay was
upgraded in the scale of Rs.6500-10500.
@page-SC437
6. Indisputably, the scale of pay of Rs.1640-2900 was revised to Rs.5500-9000.
7. Consequent upon the recommendations of the Fifth Pay Commission, Respondent filed
a representation for upgradation of her pay-scale which was not acceded to. She
thereafter filed an application before the Central Administrative Tribunal. By reason of an
order dated 11.9.2000, the learned Tribunal allowed the said application opining :
"In the above view of the matter the application succeeds and is accordingly allowed. The
respondents are directed to consider the grant of the replacement scale of Rs.6500-
10500/- to the applicant, keeping in view the similarity in essential qualification,
functions in responsibilities with those in CHD, CIIL, CSIT w.e.f 1.1.96, with
consequential benefits. This should be done within four months from the receipt of this
order. Parties to bear their own costs (sic manner)."
In arriving at the said conclusion, the Tribunal held :
"All the institutes including BPU were functioning on 1.1.1996 when the
recommendations of the 5th Pay Commission were implemented. BPU came to be
abolished only on 31.3.1996, and, therefore, there is no reason why the Research
Assistant in BPU should have been treated in a different matter."
8. A writ petition filed by the appellant herein against the said order has been dismissed
by the High Court by reason of the impugned judgment stating :
"The case of the respondent in her OA was that the post of Research Assistant in the
Bureau of Urdu and also in the other sister departments was in the pay scale of Rs.1640-
2900 upto 31.12.1995 and that qualifications required for the incumbents also were the
same and that duties, functions performed were also similar in nature and, therefore, if
post of Research Assistant was placed in the pay scale of Rs.6500-10500 in those
Department under the same Education Department, she was also entitled to the same pay
scale on the principle of equality. We find that there is no specific denial or rebuttal to this
by the petitioners in their reply to the OA. Their stands seems to be couched in general
terms. They also seem to be suffering from some misconception that since the post of
Research Assistant was abolished in the Bureau of Urdu (NCPUL) and, therefore, the
analogy of the pay scale granted to Research Assistant in other sister Departments could
not be applied to her case. What is missed is that respondent was asking for the revised
pay scale at par with the Research Assistants in other offices under the Education
Department on the basis of similarity in the nature of discharging of duties etc. which was
not controverted by the petitioner and to which she was entitled in the absence of any
denial in this regard. Therefore, it can't be said the Tribunal has gone wrong in directing
petitioner to consider this respondent for grant of pay scale of Rs.6500-10500 from
1.1.1996 on the analogy of the scale granted to Research Assistant in other Offices in the
Education Department, in view of the similarity in qualifications functions and
responsibilities of the post of Research Assistant in the Bureau on one hand and in the
CHD, CSTT, CIIL on the other. The Tribunal order is accordingly affirmed and petition is
disposed of."
9. Mr. Amrendra Sharan, learned Additional Solicitor General of India appearing on
behalf of the appellants, submitted that the Tribunal and consequently the High Court
committed a serious error in arriving at the aforementioned conclusion in so far as they
failed to take into consideration the fact that the nature of qualification and other relevant
factors clearly point out that the post of Librarian is not equivalent to that of the post of
Research Assistant in other regional languages.
10. Mr. Kulshreshtha, learned counsel appearing on behalf of the respondent, on the other
hand, would submit that as the respondent had been in the job of the Bureau of Promotion
of Urdu Language as on 1.1.1996 from which date the recommendations of the Fifth Pay
Commission came to be implemented, the impugned judgment and order should not be
interfered with.
11. Promotion of regional languages is undertaken by various bodies including Central
Hindi Directorate of the Ministry of Human Resources Development, Department of
Education, Central Institute of Indian Language, Commission for Scientific and Technical
Terminology, Ministry of Human Resources Development, Department of Education and
Bureau for Promotion.
12. So far as the educational and other
@page-SC438
qualifications required by direct recruits for promotion of the Urdu language are
concerned, following are stated to be the essential qualifications :
"(i) Masters Degree of a recognized University or equivalent.
(ii) Must have taken Urdu as optional subject at the graduation level for 3 years/2 years
degree course in the case of M.As. or must have taken Urdu as a second language upto
2nd years of 3 years degree graduation in case of MA/M.Sc. M.Com or must have taken
Urdu at High School/Higher Secondary School level in the case of M.Sc/M.Com where
offering Urdu as a second language at degree level is not furnished.
(iii) One year's experience of teaching or terminological and/or translation/editing work
in Urdu
Note 1 : Qualifications are relaxable at the discretion of the Union Public Service
Commission in case of candidates otherwise well qualified.
Note 2 : The qualification regarding experience is relaxable at the discretion of the Union
Public Service Commission in the case of candidates belonging to the Scheduled Castes
and Schedules Tribes if, at any stage of selection, the Union Public Service Commission
is of the opinion that sufficient number of candidates from these communities possessing
the requisite experience are not likely to be available to fill up the vacancies reserved for
them.
Desirable : Working knowledge of one or more, modern Indian languages other than
Urdu."
13. However, in respect of Hindi language, the qualifications prescribed are as under :
"(i) For post of Research Assistant (Hindi) : Master's Degree in Hindi or Sanskrit with
Hindi as an elective subject at Degree stage from a recognized university or equivalent
and should have studied English as a compulsory/optional subject at degree level.
(ii) For Post of Research Assistant (Regional Language) Master's Degree in Hindi with
knowledge of regional language concerned and English at Secondary School level or
Master's Degree in the regional language concerned with Hindi and English as
compulsory/optional subject at secondary school examination level. (Regional language
includes only those languages which have been specified in the Eighth Schedule of the
Constitution of India, as amended from time to time, baring Hindi and Sanskrit)
(iii) For post of requiring knowledge of Medicine : Degree in Integrated System of Indian
Medicine Bachelor of Indian Medicine and Surgery/Bachelor of Ayurvedic Medicine and
Surgery or Ayurveda/Pharmacy or equivalent from a recognized university or board with
Hindi and English as compulsory/optional subject at secondary school examination level.
(iv) For post requiring knowledge of Engineering : (Civil, Mechanical, Electrical,
Electronics, Computer Science, Textile, Mineral Leather Technology) : Diploma of a
recognized Institution/University or equivalent in the subject concerned with Hindi and
English as compulsory/optional subject as secondary school examination level.
(v) For post of Research Assistant (Management)/Research Assistant (Public
Administration) : Post-graduate diploma in Management/Public Administration
respectively from a recognized university or equivalent with knowledge of English and
Hindi as compulsory/optional subject at secondary school examination level or
equivalent.
(vi) For post of Research Assistant (Journalism): Masters degree in Hindi with Diploma
in Journalism/Mass Communication with English as compulsory/optional subject at
secondary school examination level.
(vii) For posts in any subject other than these mentioned above : Masters Degree of
recognized University or equivalent in the subject concerned with English and Hindi as
compulsory/optional subject at Secondary School Examination level.
Note 1 : Qualifications are relaxable at the discretion of the Union Public Service
Commission in case of candidates otherwise well qualified.
Note 2 : Selected candidates will have to complete a departmental training programme
during their probation.
Desirable : Only for posts of Research Assistant (Hindi) : Certificate/Diploma from a
recognized Institute in Translation or Applied Linguistics or Functional Hindi."
14. The essential qualifications required for other languages in CIIL are stated to be as
under :
"(i) Master's Degree in
@page-SC439
Linguistic/Comparative Philology/Indian Language and Literature/
Psychology/Education/Sociology/ Anthropology/Folklore/Statutics from recognized
University or equivalent.
(ii) One year's research/teaching experience.
(iii) Proficiency in any Indian Language as a subject at the Secondary School Level in the
case of Master of Arts in Linguistics or Comparative Philology or as a subject at the
degree level in the case of Master of Arts in Subject other than Linguistics and
Comparative philology.
Note 1 : Specific requirement will be indicated at the time of recruitment.
15. We may also note that in the case of recruitment by promotion, deputation, transfer
and grades from which promotion or deputation or transfer to be made, the following are
the requisite qualifications :
Urdu
Promotion :
Technical Assistant (Urdu) working in the Bureau for Promotion of Urdu with 5 years
regular service in the grade.
Transfer or deputation :
(a) Officers under the Central Government/State Government :
(i) holding analogous posts; or
(ii) with 5 years service in posts in the scale of pay of Rs.425-700 or equivalent; and
(b) Possessing the Education qualifications and experience prescribed for direct recruits
under column-7. Period of deputation including period of deputation in another ex-cadre
post held immediately preceding this appointment in the same organization/department
shall ordinarily not exceed 3 years."
Hindi
Transfer on deputation/transfer :
Officers under the Central Government
(a) (i) Holding analogous posts in regular posts on regular basis, or
(ii) With 5 years regular service in post in the scale of pay of Rs.1400-2300/2600 or
(iii) with 15 years regular service in post in the scale of Rs.950-1500 or equivalent.
(b) possessing the educational qualifications and experience prescribed for direct
recruitment under column 8. (period of deputation including period of deputation in
another ex cadre post held immediately preceding this appointment in the same or some
other organization/department of the Central Government shall ordinarily not to exceed 3
years. The maximum age limit for appointment by transfer on deputation including
transfer shall be not exceeding 56 years, as on the closing date of receipt of applications."
16. We may now consider different nature of duties required to be performed by these
categories of officers :
"Urdu
To assist the officer with whom they are attached in implementing the publication
programme BPU at various stages. This includes organizing of subjects panel melting,
implementing their decisions, checking and editing mss, organizing Terminology
committee meeting and preparing of glossary of technical terms maintenance of record of
all the above mentioned activity and programming the duty allotted from time to time in
furtherance of the activity of BPU.
Hindi
To assist in the implementation of schemes relating to periodicals, preparation of
Dictionaries-Lingual Bilingual, Trilingual and Multilingual, preparation of Dictionaries in
Foreign Languages under Cultural Exchange Programme.
Other Regional Languages
To assist in Linguistic and in material production in various Indian Languages including
the non-scheduled languages."
17. The requisite criteria in regard to such appointment, promotion, transfer as well as the
nature of duties required to be performed by the incumbents of posts vis-a-vis that of
Research Assistant (Urdu) therefor, are different. Knowledge of English for Research
Assistant (Urdu) is not necessary whereas for the Research Assistant (Hindi) and other
regional languages, the same is essential.
18. So far as the Research Assistant for CIIL is concerned, the essential qualifications
therefore are absolutely different. So far as the educational qualifications required for
promotion to the said post by the incumbents of the Research Assistant to Research
Assistant (Hindi) is concerned, therefore also different educational qualifications are
required. Not only that, the nature of duties is also different. Whereas the Research
Assistants in respect of Urdu language are required to assist the officer with whom they
@page-SC440
are attached, the Research Assistants in Hindi and Research Assistants of CIIL are
required to assist implementation of the scheme. The Tribunal and consequently the High
Court might not, thus, be correct in opining that the educational qualifications as also the
nature of duty being the same, respondent was entitled to the benefit of the said scale of
pay.
19

. The question came to be considered in a large number of decisions of this Court wherein
it unhesitantly came to the conclusion that a large number of factors, namely, educational
qualifications, nature of duty, nature of responsibility, nature of method of recruitment
etc. will be relevant for determining equivalence in the matter of fixation of scale of pay.
{See Secretary, Finance Department and Ors. v. West Bengal Registration Service
Association and Ors. [1993 Supp.(1) SCC 153]; State of U.P. and Ors. v. J.P. Chaurasia
and Ors. [(1989) 1 SCC 121]; Union of India and Ors. v. Pradip Kumar Dey [(2000) 8
SCC 580] and State of Haryana and Anr. v. Haryana Civil Secretariat Personal Staff
Association [(2002) 6 SCC 72]}. 1992 AIR SCW 1071
AIR 1989 SC 19
2002 AIR SCW 2896

20. In Government of West Bengal v. Traun K. Roy and Ors. [(2004 (1) SCC 347], this
Court held as under :
"Question of violation of Article 14 of the Constitution of India on the part of the State
would arise only if the persons are similarly placed. Equality clause contained in Article
14, in other words, will have no application where the persons are not similarly situated
or when there is a valid classification based on a reasonable differentia."
21

. In U.P. State Sugar Corporation Ltd. and Anr. v. Sant Raj Singh and Ors. [(2006) 9
SCC 82], this Court opined : 2006 AIR SCW 3013

"The doctrine of equal pay for equal work, as adumbrated under Article 39(d) of the
Constitution of India read with Article 14 thereof, cannot be applied in a vaccum. The
constitutional scheme postulates equal pay for equal work for those who are equally
placed in all respects. Possession of a higher qualification has all along been treated by
this Court to be a valid basis for classification of two categories of employees."
22

. Same principle was reiterated by a Three Judge Bench of this Court in State of Haryana
and Ors. v. Charanjit Singh and Ors. [(2006) 9 SCC 321]. 2005 AIR SCW 5632

23

. We are not oblivious of some decisions of this Court wherein salary on the basis of
revised pay scales has been directed to be paid on the premise that no change in the duties
and functions of employees similarly situated had taken place although the concerned
employees were working in the different public sector undertakings {See The Employees
of Tennery and Footwear Corporation of India Ltd. and Anr. v. Union of India and Ors.
[1991 Supp.(2) SCC 565]} or where scale of pay is to be fixed for the judicial officers
posted in the State cadre vis-a-vis Union Territory Cadre {[Alvaro Noronha Ferriera and
Anr. v. Union of India and Ors. [(1999) 4 SCC 408]} but such a question does not arise
herein, as different scale of pay was recommended by an expert body having regard to the
nature of duties and functions. It is not a case where discrimination is sought to be made
on the basis of territory or posting in public sector undertaking. AIR 1991 SC 1367
1999 AIR SCW 1153

24. On the facts obtaining in this case, therefore, we are of the opinion that the doctrine of
equal pay for equal work has no application. The matter may have been different, had the
scales of pay have been determined on the basis of educational qualification, nature of
duties and other relevant factors. We are also not oblivious of the fact that ordinarily the
scales of pay of employees working in different departments should be treated to be at par
and the same scale of pay shall be recommended. Respondent did not opt for her services
to be placed on deputation. She opted to stay in the Government service as a surplus. She
was placed in list as Librarian in National Gallery of Modern Art. She was designated as
Assistant Librarian and Information Assistant. Her pay scale was determined at Rs.6500-
10500 which was the revised scale of pay. Her case has admittedly not been considered
by the Fifth Pay Revision Commission. If a scale of pay in a higher category has been
refixed keeping in view the educational qualifications and other relevant factors by an
expert body, no exception thereto can be taken. Concededly it was for the Union of India
to assign good reasons for placing her in a different scale of pay. It has been done. We
have noticed hereinbefore that not only the essential educational qualifications are
different but the nature of duties is also different.
@page-SC441
Article 39(d) as also Article 14 of the Constitution of India must be applied, inter alia, on
the premise that equality clause should be invoked in respect of the people who are
similarly situated in all respects.
25

. Mr. Kulshreshtha has placed strong reliance on State of U.P. and Ors. v. U.P. Sales Tax
Officers Grade II Association 2003 (6) SCC 250]. In that case the Pay Revision
Commission did not consider cases of a group of employees. On the aforementioned
premise, they were held to be entitled to the scale of pay which had been granted to the
persons similarly situated. We are not concerned with such an issue herein as the case of
the respondent has been considered and she has been given the benefit of a revised scale.
It was not necessary for the Government which had the requisite jurisdiction to remove
anomaly as has been held by this Court in Haryana State Adhyapak Sangh and Ors. v.
State of Haryana and Ors. [(1988) 4 SCC 571], whereupon Mr. Kul-shreshtha relied on.
As the Union of India has already applied its mind and revised the respondents pay in the
scale of pay of Rs.5500-9000, it was for the respondent to show that she had been
discriminated against. We have noticed hereinbefore that neither in fact nor in law, any
case of discrimination has been made out. 2003 AIR SCW 2278
AIR 1988 SC 1663

26. Our attention has been drawn to the findings of the Tribunal that the incumbents to
the post of Research Assistants in the Bureau and Institutions like Central Hindu
Directorate and Central Institution of Indian Languages etc. are similarly qualified and
they have been performing similar functions. There was no factual foundation for arriving
at the same finding. Consequently, the said conclusion was wrongly drawn by the
Tribunal. Furthermore, no formula having mathematical exactitude can be pressed into
service in a situation of this nature. The Tribunal and consequently the High Court, in our
opinion, therefore, was not correct in arriving at the said decision.
27

. Another aspect of the matter, however, cannot be ignored. Respondent has been paid the
amount by way of difference in the scale of pay only for a short period. She has been held
to be entitled only for a sum of Rs.7,000/- and odd. We are, therefore, of the opinion that
this Court, in exercise of its jurisdiction under Article 142 of the Constitution, should
direct that the amount already paid need not be recovered. Similar direction has been
passed by this Court in Haryana Civil Secretariat Personal Staff Association (supra)
stating : 2002 AIR SCW 2896
(Para 10)

"The courts should approach such matters with restraint and interfere only when they are
satisfied that the decision of the Government is patently irrational, unjust and prejudicial
to a section of employees and the Government while taking the decision has ignored
factors which are material and relevant for a decision in the matter. Even in a case where
the court holds the order passed by the Government to be unsustainable then ordinarily a
direction should be given to the State Government or the authority taking the decision to
reconsider the matter and pass a proper order. The court should avoid giving a declaration
granting a particular scale of pay and compelling the Government to implement the same.

{[See also Punjab National Bank and Ors. v. Manjeet Singh and Anr. [(2006) 8 SCC
647]} 2006 AIR SCW 6035

28. We, therefore, although agree with the submissions of learned Additional Solicitor
General, in the facts and circumstances of this case, decline to grant any relief in favour
of the appellant. The appeal is dismissed in view of our observations aforementioned.
There shall, however, be no order as to costs.
Appeal dismissed.
AIR 2008 SUPREME COURT 441 "Paramjit Singh v. State of Punjab"
(From : Punjab and Haryana)*
Coram : 2 PRAKASH PRABHAKAR NAOLEKAR AND B. SUDERSHAN REDDY,
JJ.
Criminal Appeal No. 1474 of 2005, D/- 31 -10 -2007
Paramjit Singh @ Mithu Singh v. State of Punjab, Through Secretary (Home).
(A) Criminal P.C. (2 of 1974), S.154 - Punjab Police Rules (1934), R.2 - FIR - POLICE
OFFICERS - F. I. R. - Recording - Punjab Police Rules do not override provisions of
Criminal P. C. - Plea that under Rules information must be reduced to writing and be
entered in Police Station daily diary and only thereafter FIR is to be issued - Not tenable -
Further truth and veracity of contents of
@page-SC442
FIR cannot in all cases be tested with reference to entries in Police Station daily diary.
A bare reading of the relevant rule under Punjab Police Rules makes it clear that every
information relating to the commission of a cognizable offence, if given orally to an
officer-in-charge of a police station shall be reduced to writing and the substance thereof
shall be entered in a book to be kept in such form as may be prescribed and only
thereafter in the Police Station diary. The procedure prescribed under Criminal P. C. is
required to be followed scrupulously by the Officer-in-charge of the Police Station. The
Punjab Police Rules do not in any manner override the provisions of the Code of
Criminal Procedure. The said rules are meant for the guidance of the Police Officers in
the State and supplement the provisions of the Code of Criminal Procedure but not
supplant them. The truth and veracity of contents of the FIR cannot in all cases be tested
with a reference to the entries made in the Police Station daily diary which is maintained
under the Punjab Police Rules. (Para 13)
The plea that under the Punjab Police Rules information must be reduced to writing and
be entered in the police station daily diary and only thereafter the First Information
Report is to be issued is absolutely untenable. In the instant case, the original of the DDR
has not been filed into the Court and what has been filed was only a photocopy and that
too at the instance of the accused after four years of the closure of evidence. It is clearly
evident from the statement of prosecution witness that the First Information Report was
recorded first and the DDR thereafter. He further stated that the DDR had been recorded
on the basis of the facts recorded in the First Information Report. Thus, it would be
difficult to comprehend as to how to- tally a different version is found in DDR which is
said to be a photocopy of DDR. Thus, the timing and contents of FIR based on the entries
made in DDR cannot be doubted when there are serious doubts about the genuineness of
the very document DDR. (Paras 12, 14)
(B) Criminal P.C. (2 of 1974), S.156 - INVESTIGATION - Investigation - Defect or
procedural irregularity - By itself cannot vitiate and nullify trial based on such erroneous
investigation. (Para 13)
(C) Penal Code (45 of 1860), S.300, S.34 - MURDER - COMMON INTENTION -
EVIDENCE - Murder - Accused were armed with deadly weapons and standing outside
the house of deceased - Deceased and eye-witness going towards their houses - Accused
persons assaulted deceased - Said eye-witness raised alarm which attracted son of
deceased to spot who also witnessed the incident - Thus, presence of both the eye-
witnesses cannot be doubted on ground that they have not made any attempt to rescue
deceased - Moreso, when their presence at the scene of offence is evident from FIR
which was lodged by one of them.
Evidence Act (1 of 1872), S.3. (Paras 15, 16)
(D) Evidence Act (1 of 1872), S.3 - EVIDENCE - MURDER - Murder - Eye-witness, son
of deceased - Testimony of - Credibility of - Fact that he could not spell out accurately
situs of injuries on dead body - Would not make his presence at spot doubtful. (Para
16)
(E) Penal Code (45 of 1860), S.300, S.34 - MURDER - COMMON INTENTION -
WITNESS - Murder - Common intention - Accused persons attacked deceased and
caused injuries - Common intention to kill is evident - Consistent evidence of eye-
witnesses, the direct witnesses - Fact that appellant accused inflicted injuries with deadly
weapon itself shows that he had also shared common intention - Sharing of common
intention by appellant-accused is evident from fact that he was armed with deadly
weapon and inflicted two injuries on victim - Thus, in circumstances whether those
injuries were sufficient in ordinary course to cause death pales into insignificance -
Conviction of accused under S. 302 read with S. 34 - No interference. (Para 18)

Jana Kalyan Das and Avijeet Bhujabal, for Appellant; Kuldip Singh, for Respondent.
* Cri. Appeal No. 25-DBA of 1995, D/- 27-9-2004 (PandH)
Judgement
B. SUDERSHAN REDDY, J. :- The appellant has preferred this appeal under Section
379 of the Code of Criminal Procedure read with provisions of the Supreme Court
(Enlargement of Criminal Appellate Jurisdiction) Act, 1970 impugning the judgment and
order of the High Court of Punjab and Haryana in Criminal Appeal No. 25-DBA of
@page-SC443
1995 whereby the High Court reversed the judgment of acquittal against the appellant,
who was tried along with three other co-accused, recorded by the Additional Sessions
Judge, Sangrur in Sessions Case No. 44 of 1989. The High Court accordingly convicted
the appellant for the offence punishable under Section 302 read with 34 of the Indian
Penal Code (IPC) and sentenced to undergo imprisonment for life and to pay a fine of Rs.
5,000/-, in default of payment, to undergo rigorous imprisonment for six months.

2. The prosecution case, in brief, is that the accused Mukhtiar Singh (A-1) and Gurdial
Singh (A-2) and deceased Harnek Singh were real brothers. The family consists of eight
brothers altogether. Deceased Harnek Singh along with his wife Tej Kaur and son
Gurmail Singh (PW-4) were living jointly with one of his brother Amar Singh (PW-3).
Gurdev Singh and Dalbara Singh, two other brothers were residing together whereas the
others were residing separately in their respective houses located in their agricultural
lands. Dalip Singh, father of Gurcharan Singh (A-3) and Mithu Singh (A-4) were residing
separately. Dalip Singh is stated to have entered into an agreement to sell his house to
Babu Singh, Balak Singh and their sons but the same could not be fructified into regular
sale as Harnek Singh had interfered in the deal. The accused accordingly developed
grudge as against Harnek Singh over his unwarranted interference in the sale transaction.
3. On 6.5.1989 about 7.00 p.m. Amar Singh (PW-3) and deceased Harnek Singh were
going towards their houses in the fields whereas Tej Kaur and Gurmail Singh (PW-4)
were already present in the house. The deceased Harnek Singh and Amar Singh (PW-3)
saw all the four accused standing outside the house of Mukhtiar Singh(A-1). Mukhtiar
Singh(A-1) was armed with a Sumewali Dang, both Gurcharan Singh @ Charna (A-
3)and the appellant were armed with a gandasa each. Gurdial Singh (A-2) shouted a
lalkara that Harnek Singh should be taught a lesson for interfering in Dalip Singh's
property deal and he should be killed, on which the rest of the three accused inflicted
several injuries on Harnek Singh. Amar Singh (PW-3) raised an alarm, which attracted
Tej Kaur and Gurmail Singh to the spot and they too witnessed the incident. The accused
ran away from the scene of occurrence. Harnek Singh was first removed to his farm
house and then to the Civil Hospital, Longowal by Amar Singh (PW-3). It was about 9.10
p.m. Dr. Rakesh Jain (PW-6) having noticed the critical condition of Harnek Singh
immediately referred him to the Civil Hospital, Sangrur. Dr. Rakesh Jain (PW-6) sent
information to SHO, Police Station, Longowal at about 9.25 p.m. Harnek Singh,
however, died soon after reaching the Civil Hospital, Sangrur. Amar Singh (PW-3) along
with his brother Gurdev Singh left the hospital and reached the police station, Longowal
at about 1.15 a.m. on 7.5.1989 and lodged First Information Report. Gurmail Singh (PW-
4) was at the hospital near the dead body. The special report sent to the Illaqa Magistrate,
Sangrur reached at 5.00 a.m. The police on the completion of the investigation filed
charge sheet against Gurcharan Singh @ Charna (A-3) for the offence punishable under
Section 302 of the IPC whereas the other accused were charged under Section 302/34 of
the IPC. The accused pleaded not guilty.
4. The prosecution in order to establish its case relied on the evidence of Dr. K.S. Raikhy
(PW-1), who performed the post-mortem examination on the dead body and found six
injuries thereon, three incised and three lacerated; Amar Singh (PW-3) and Gurmail
Singh (PW-4), the two eye witnesses; ASI, Malikat Singh (PW-5), the Investigating
Officer and Dr. Rakesh Jain (PW-6) who first received the injured at Civil Hospital,
Longowal.
5. The trial court upon appreciation of evidence available on record acquitted all the
accused of the charges. The trial court recorded finding that the presence of the eye-
witnesses Amar Singh (PW-3) and Gurmail Singh (PW-4) was unlikely as they were not
stamped witnesses and had not intervened at the time when Harnek Singh was being
belaboured. The court also found that Gurmail Singh (PW-4) had apparently not been
present at the spot as his ocular version did not correspond with the medical evidence
with regard to the number and situs of the injuries on the dead body. The trial court also
referred to the contents in the DDR (Exh. DX/1) in which it is stated that the accused
were armed with Sotis and there was no reference to any of the accused armed with
gandasa and accordingly held this important circumstance itself nullify the prosecution
story. The trial court also held
@page-SC444
that there was delay in lodging the First Information Report. The trial court accordingly
acquitted all the accused giving them the benefit of doubt.
6. The High Court upon re-appreciation of evidence found that there was no delay in
lodging the First Information Report. The High Court also reversed the finding recorded
by the trial court as regards the presence of the eye-witnesses as it came to the conclusion
that there is no reason to doubt the presence of Amar Singh (PW-3) and Gurmail Singh
(PW-4) at the scene of occurrence. The High Court took the view that the presence of
eye-witnesses was absolutely natural and they had good reason for being present at the
scene of offence. The High Court relied upon the First Information Report in which it has
been mentioned that the accused were armed with gandasas and dangs. The High Court
found the DDR (Exh. DX/1) is the photocopy of the original which was not produced in
the court. Be it noted that the prosecution had closed its evidence on 22.9.1993 and the
statements of all the accused under Section 313 Cr.P.C. had thereafter been recorded and
it is only thereafter the accused moved an application to recall Amar Singh (PW-3) and
Iqbal Rai (PW-7) who had recorded the DDR (Exh. DX/1) which was allowed by the trial
court. The application was ordered about 4 years after the closure of evidence. The High
Court upon appreciation of the evidence, however, found that the so-called entry made in
the DDR (Exh. DX/1) by itself may not make any difference to the prosecution case
inasmuch as the evidence of Iqbal Rai (PW-7) clearly reveals that the First Information
Report had been recorded first and entries in the DDR were made thereafter. The High
Court also found that the injuries found on the body of the deceased were inflicted by
sharp cutting weapons which finds support from the medical evidence. The High Court in
conclusion held:
"As per the ocular version, injuries 1,3 and 4 are incised wounds, which had allegedly
been caused by Gurcharan Singh and Mithu accused and injury No. 2 by Gurcharan
Singh whereas injury No. 5 had been attributed to Mukhtiar Singh accused. There is no
clear cut evidence as to who had caused injury No. 6, which had been detected by Dr.
K.S. Raikhy (PW-1) at the time of post-mortem examination. We also observe that the
Gandasa is a cutting weapon with a Lathi attached to it. It is, therefore, possible that a
Gandasa could have been used Lathiwise as well while causing the lacerated injuries.
Mukhtiar Singh, who was armed with a Dang, has been attributed one simple lacerated
wound 3 cm x 3 cm in dimension. He is, therefore, entitled to claim some benefit in an
appeal against acquittal for an incident, which happened in the year 1989. Gurdial Singh
was unarmed and only a Lalkara has been attributed to him. To be on the safe side, he too
must be dealt with in the same manner as Mukhtiar Singh.
We accordingly dismiss the appeal qua Mukhtiar Singh and Gurdial Singh. We, however,
find that case against Gurcharan Singh and Mithu stands proved beyond doubt. The
appeal qua them is allowed. Gurcharan Singh is held guilty for an offence punishable
under Section 302 of the Indian Penal Code whereas Mithu Singh is held guilty for the
offence punishable under Section 302/34 thereof. They are sentenced to undergo
imprisonment for life and to pay a fine of Rs. 5,000/- each and in default of payment of
fine, to undergo rigorous imprisonment for six months each. The fine, if paid, shall be
paid to Tej Kaur, the widow of the deceased."
7. This appeal has been preferred by Mithu Singh (A-4) alone.
8. The learned counsel appearing for the appellant submitted that the entries made in the
DDR in which it has been mentioned that the accused were armed with Sotis completely
falsify the prosecution story. The learned counsel made an attempt to contend that the
entries were first made in the DDR based on the First Information Report made by Amar
Singh (PW-3) and only thereafter the First Information Report has been recorded making
improvements to implicate the accused in the case. The learned counsel also contended
that the presence of Amar Singh (PW-3) and Gurmail Singh (PW-4) at the scene of
occurrence is highly doubtful for they did not intervene when the deceased was being
attacked. It was also contended that two injuries were simple in nature out of which one
is alleged to have been caused by the appellant herein and, therefore, there is no evidence
of any common intention to kill the deceased.
9. The learned counsel for the State of Punjab submitted that the common intention is
evident from the fact that the appellant was armed with deadly weapon and it
@page-SC445
is immaterial as to the nature of the injuries inflicted by the appellant on the body of the
deceased. The learned counsel supported the findings of the High Court.
10. We have considered the submissions made during the course of hearing of the appeal
and perused the evidence available on record.
11. We shall first deal with the contention with regard to delay in lodging the First
Information Report. The evidence available on record reveals that the incident took place
on 6.5.1989 at 7.00 p.m in village Longowal. The distance between village and police
station is about 3 kms. It is in the evidence of Amar Singh (PW-3) and Gurcharan Singh
(PW-4) that they had immediately removed critically injured Harnek Singh to their farm
house and thereafter to the Primary Health Center, Longowal in a bullock cart and
reached there at 9.10 p.m. Dr. Rakesh Jain (PW-6) who attended the injured sent the ruqa
(Exh. PN) to the Police Station, Longowal at 9.25 p.m. Having regard to the grievous
nature of injuries and condition of the victim Dr. Rakesh Jain (PW-6) referred the injured
to the Civil Hospital, Sangrur. The evidence of Dr. Rakesh Jain (PW-6) in this regard
remains unimpeached and there is absolutely no reason to disbelieve any portion of his
evidence. It is Amar Singh (PW-3) who took the injured to the Civil Hospital at
Longowal and thereafter to the Hospital at Sangrur where the injured succumbed to
injuries. It is only thereafter Amar Singh (PW-3) went to police station which is at a
distance of about 9-10 kms. from the Hospital and lodged First Information Report. Amar
Singh (PW-3) was present not only at the scene of offence but accompanied the injured to
Civil Hospital, Longowal and thereafter to the Hospital at Sangrur. It is only after Harnek
Singh died in the Hospital Amar Singh (PW-3) left to police station to lodge First
Information Report at 1.15 a.m on 7.5.1989. The special report sent by the police reached
the Illaqa Magistrate at 5.00 a.m. In order to determine whether the FIR was lodged at the
time it is alleged to have been recorded, the courts normally look for certain external
checks. One of the checks is the receipt of the copy of the FIR, called a special report, by
the Illaqa Magistrate. In this case, the report has been received by the Illaqa Magistrate in
time. The second external check equally important is the sending of the copy of the FIR
along with the dead body and its reference in the inquest report. This requirement is also
complied with in the present case. The inquest report clearly refers to the lodging of the
First Information Report by Amar Singh (PW-3) at 1.15 a.m. on 7.5.1989 in Police
Station, Longowal and it also refers to the registration of the First Information Report and
dispatch of special report for their delivery to the concerned authorities. Thereafter,
Malkiat Singh, ASI (PW-5) along with some constables and Amar Singh (PW-3) rushed
to the Civil Hospital, Sangrur where the inquest report has been prepared. The sequence
of events clearly reveals that there was any unexplained and unreasonable delay in
lodging the FIR. In the circumstances, it cannot be said that the FIR was ante-timed and
brought into existence after some deliberations.
12. We do not find any merit in the contention that the entries made in the DDR (Exh.
DX/1) in which it has been mentioned that the accused were armed with Sotis falsify the
First Information Report lodged by Amar Singh (PW-3). We have already noted that the
original of the DDR has not been filed into the court and what has been filed was only a
photocopy and that too at the instance of the accused after four years of the closure of
evidence. It is clearly evident from the statement of Iqbal Rai (PW-7) that the First
Information Report was recorded first and the DDR thereafter. He further stated that the
DDR had been recorded on the basis of the facts recorded in the First Information Report.
We find it difficult to comprehend as to how totally a different version is found in DDR
which is said to be a photocopy of DDR. There is obviously something more than meets
the eye. The contention that under the Punjab Police Rules information must be reduced
to writing and be entered in the police station daily diary and only thereafter the First
Information Report is to be issued is absolutely untenable. The relevant rule says:
"Every information covered by Section 154, Criminal Procedure Code must be reduced to
writing as provided in that Section and the substance thereof must be entered in the police
station daily diary which is the book provided for the purpose."
A bare reading of the rule makes it clear that every information relating to the
@page-SC446
commission of a cognizable offence, if given orally to an officer-in-charge of a police
station shall be reduced to writing and the substance thereof shall be entered in a book to
be kept in such form as may be prescribed and only thereafter in the Police Station diary.
13. Chapter XII of the Code of Criminal Procedure, 1973 deals with information to the
police and their powers to investigate. Investigation into allegations relating to
commission of a cognizable offence starts on information given to an Officer-in-charge of
a Police Station and recorded under Section 154 of the Code. If from information so
received or otherwise, the Officer-in-charge of the Police Station, if satisfied that such
information discloses the commission of a cognizable offence, shall either investigate the
case himself or direct the investigation to any police officer subordinate to him, in the
manner provided by the Code. The procedure as regards the registration of information
relating to the commission of a cognizable offence and the procedure for investigation is
structured and regulated by Chapter XII of the Code. The procedure prescribed is
required to be followed scrupulously by the Officer-in-charge of the Police Station. The
Punjab Police Rules do not in any manner override the provisions of the Code of
Criminal Procedure. The said rules are meant for the guidance of the Police Officers in
the State and supplement the provisions of the Code of Criminal Procedure but not
supplant them. In our considered opinion the truth and veracity of contents of the FIR
cannot in all cases be tested with a reference to the entries made in the police station daily
diary which is maintained under the Punjab Police Rules. This avoidable controversy
need not detain us any further since it is well settled that even a defect, if any, found in
investigation, however, serious has no direct bearing on the competence or the procedure
relating to the cognizance or the trial. A defect or procedural irregularity, if any, in
investigation itself cannot vitiate and nullify the trial based on such erroneous
investigation.
14. Amar Singh (PW-3) clearly and categorically stated that the entries in the DDR had
been recorded on the basis of the facts given in the First Information Report. It is difficult
to place any reliance upon the photocopy of the DDR that was produced before the court
after four years of the closure of evidence. There is no explanation as to the fate of
original DDR. It is not possible to doubt the timing and contents of FIR based on the
entries made in DDR. We have serious doubts about the genuineness of the very
document DDR. We wish to say no more on this aspect of the matter.
15. In our considered opinion there is no basis to contend that Amar Singh (PW-3) and
Gurmail Singh (PW-4) were not present at the scene of offence and did not witness the
incident. The contention was that Amar Singh (PW-3) and Gurmail Singh (PW-4) had not
intervened to save Harnek Singh when he was being attacked by the accused. The
evidence available on record reveals that Amar Singh (PW-3) and deceased Harnek Singh
had almost reached their house when they had been waylaid by the accused. The
appellant and Gurcharan Singh (A-3) were armed with gandasas and Mukhtiar Singh (A-
1) with a dang. As has been rightly observed by the High Court that it would be well nigh
impossible to apply a universal yardstick as to how a person would react to a given
situation. The presence of Amar Singh (PW-3) and Gurmail Singh (PW-4) cannot be
doubted on the ground that they have not made any attempt to rescue the deceased. We
cannot ignore the fact that the accused were armed with deadly weapons and the same
may have deterred PW-3 and PW-4 in making any attempt to rescue the victim when he
was under attack.
16. It is true that Gurmail Singh (PW-4) had not been able to spell out accurately the situs
of the injuries on the dead body but the same would not make his presence doubtful. The
victim was under attack from a group of persons armed with deadly weapons. He must
have made attempts to save himself from the attack and in the process may have not
remained static without moving one way or the other. One cannot expect that in such a
situation the witness would graphically describe the nature of injuries and spell out
accurately the situs of the injuries on the body of the victim. Their presence at the scene
of offence is evident from the First Information Report itself which was lodged by Amar
Singh (PW-3) himself. The fact remains Harnek Singh had been waylaid by the four
accused and thereafter inflicted several blows with the gandasas and dang.
@page-SC447
17. It is required to notice that Dr. K. S. Raikhy (PW-1) had found six injuries on the dead
body at the time of post-mortem examination. The injuries found on the body were:
1. Incised wound 12 cms x 5 cms x bone deep on the right parieto-temporal region,
wound placed obliquely bone cut and brain matter and mengis protruding through the
wound; dark clotted blood was present on the wound. On dissection underlying bone was
cut, mengis cut. The brain matter protruding through the wound haematoma was present.
2. Lacerated wound 3 cms x 2 cms on the left clavicular region. The wound was skin
deep. On dissection underlying bone was intact. Haematoma was present.
3.Incised wound 3 cms x 2 cms x 1 cm on the posterior-lateral aspect of left forearm 2
cms above the wrist joint. Dark blood clot was present. On dissection the underlying bone
was intact haematoma was present.
4. Incised wound 10 cms x 3 cms x bone deep on the antero-lateral aspect of left leg,
wound placed obliquely 3 cms below the knee joint. Dark blood clot was present in the
wound. On dissection the underlying tibia bone was cut. Haematoma was present.
5. Lacerated wound 3 cms x 3 cms x skin deep on the anterior aspect of left leg 10 cms
below the tibial tuberosity. Dark blood clot was present. On dissection the underlying
bone was intact.
6. Lacerated wound 4 cms x 2 cms x bone deep on the antero-lateral aspect of right leg 3
cms below the tibial tuberosity. Dark blood clot was present. On dissection the
underlying bone was intact.
It is in the evidence of PW-3 that Gurcharan Singh inflicted injury No. 1 by giving a
gandasa blow on the head of the victim and whereas the appellant herein inflicted injury
Nos. 3 and 4 gandasa blows on his right leg below the knee and another blow on the left
side of the chest using the reverse side of the gandasa. The appellant was armed with
deadly weapon namely gandasa. Dr. K. S. Raikhy (PW-1) stated in his evidence that the
cause of death was shock and haemorrhage and all the injuries were ante-mortem in
nature. It is further stated by him that injury No. 1 itself was sufficient to cause death in
the ordinary course of nature.
18. The learned counsel for the appellants further submitted that the injuries inflicted by
the appellant were not sufficient to cause the death of the victim and, therefore, the
common intention to kill is not evident and, therefore, he cannot be convicted for the
offence punishable under Section 302 read with Section 34 of the IPC. We are unable to
agree. The evidence of PWs-3 and 4 the direct witnesses is consistent and they had
deposed that the appellant inflicted injuries with gandasa to kill the deceased. The fact
that the appellant inflicted injuries with the deadly weapon itself shows that he had also
shared the common intention. In order to convict the person vicariously under Section 34,
it is not necessary to prove that each and every one of them had indulged in such overt act
inflicting deadly injuries. It is enough if the material available on record discloses that the
overt act of one or more of the accused was or were done in furtherance of common
intention. The common intention shared by the appellant is evident from the fact that he
was armed with deadly weapon and inflicted two injuries on the victim. All the accused
attacked the deceased and caused injuries in furtherance of the common intention to
murder the deceased. In such a situation the nature of injuries inflicted by the appellant
on the victim and whether those injuries were sufficient in the ordinary course to cause
death pales into insignificance. The appellant was not a curious onlooker and had not
accompanied the assailant who gave a deadly blow out of any ideal curiosity. Each one of
them is liable for that act of murder as if the act of murder was done by each one of them.
It is true that if the High Court had adopted this reasoning even Mukhtiar Singh (A-1) and
Gurdial Singh (A-2) could not have escaped from conviction. However, we do not
propose to express any firm opinion on that aspect of the matter since there is no appeal
by the State against their acquittal.
19. For the aforesaid reasons we find no merit in this appeal. The appeal shall accordingly
stand dismissed.
Appeal dismissed.
@page-SC448
AIR 2008 SUPREME COURT 448 "Subhash v. State of Haryana"
(From : Punjab and Haryana)*
Coram : 2 S. H. KAPADIA AND B. SUDERSHAN REDDY, JJ.
Criminal Appeal No. 1107 of 2007 (arising out of SLP (Cri.) No. 3992 of 2006), D/- 21
-8 -2007.
Subhash v. State of Haryana.
Penal Code (45 of 1860), S.392, S.397, S.300, S.304, Part II - ROBBERY - MURDER -
CULPABLE HOMICIDE - Robbery and murder - Proof - Accused person allegedly
threw deceased and other person into canal after committing robbery of money and
tractor - Nothing on record to doubt presence of said person at scene of occurrence -
Circumstances of recovery of dead body of deceased, recovery of tractor from possession
of accused and 'Barma' from possession of co-accused proved guilt - Sequence of events
revealed reasons for delay in lodging F. I. R. - Prosecution had established guilt of
appellant beyond all reasonable doubt - However, there was no intention of causing death
of deceased - Appellant is therefore liable to be convicted under Ss. 392, 397 and 304,
Part II - Sentence of 7½ years R. I. already undergone, is sufficient.(Paras 10 , 11, 12, 13,
14, 15, 16, 17)

D. P. Singh, Sanjay Jain, Rajat Singh and Ms. Pranika Singh, for Appellant; Sandeep
Sharma, Rajeev Gaur 'Naseem' and T. V. George, for Respondent.
* Cri. Appeal No. 183 D.B. of 1997, D/- 28-2-2006 (PandH).
Judgement
B. SUDERSHAN REDDY, J.:-Leave granted.
2. This appeal by special leave is directed against the judgment of the Punjab and
Haryana High Court confirming the conviction of the appellant under Section 392 read
with Section 397 IPC and the sentence of rigorous imprisonment for a period of 7 years
and under section 302 read with Section 34, IPC, imprisonment for life and a fine of Rs.
10,000/-, in default of payment of fine to undergo further two years rigorous
imprisonment. Both the sentences were directed to run concurrently. The appellant was
charged with the offences for having robbed tractor and caused the death of Raghbir
Singh-deceased.
3. The prosecution story, briefly stated, is that on 9.10.1991 Raghbir Singh(deceased)
along with Mane Ram (PW-8) after collecting 'Barma' (an instrument for drawing out
water) from Nand Lal (PW-5) resident of village Kailana left for their village- Mandi at
about 12 noon on a Swaraj make tractor. On the way when they reached at village
Pugthala they purchased half bottle of liquor from a liquor vend. Raghbir Singh
(deceased) purchased a nip of liquor separately and carried it with him. Subhash
(appellant-accused) along with son of Basu Sardar and Jai Kumar met Raghbir Singh
(deceased) and Mane Rame (PW -8) near the village Chamrara. Son of Basu Sardar took
the nip of liquor from Raghbir Singh (deceased) and consumed the same. Thereafter all
accused persons went to the house of son of Basu Sardar and again consumed illicit
liquor. After consuming illicit liquor they came back at the place where they had met
Mane Ram (PW-8) and Raghbir Singh (deceased). An altercation took place among son
of Basu Sardar, Subash (appellant-accused) and Jai Kumar wherein the son of Basu
Sardar inflicted injury upon the head of Subhash (appellant-accused) and fled away.
Subhash (appellant-accused) was brought to the village Pugthala in order to get his
wound dressed up from a doctor on the tractor of Amar Singh being driven by Joginder
Singh (accused). Thereafter Joginder (accused) took the tractor towards village Bajana on
the bank of canal. When they reached near the bridge of canal in the village Kasandi
Joginder accused stopped the tractor on which the accused persons started robbing of
money from the pocket of Raghbir Singh (deceased). It was about 8.00 P.M. Enraged by
the intervention of Mane Ram (PW-8) all the accused persons caught hold of Mane Ram
and pressed his neck and threw him into the canal. Thereafter accused persons caught
hold of Raghbir Singh (deceased) by his legs and also threw him into the same canal.
Raghbir Singh (deceased) tried to catch hold of the grass grown on the bank of the canal.
Jai Kumar (accused) and Joginder Singh (accused) gave leg blows on the face of Raghbir
Singh and again thrown into the water and he did not come out of the canal. Mane Ram
(PW-8) knew swimming and came out of the canal after having covered a distance of
three acres and narrated the incident to Jai Singh a resident of village Kasandi. Thereafter,
Mane Ram (PW-8) along with Jai Singh searched for Raghbir Singh (deceased) and the
tractor
@page-SC449
belonging to Amar Singh (PW- 6) but they could not find them. Then Mane Ram (PW-8)
and Jai Singh went to village Mandi. It was about mid-night. They narrated the
occurrence to father of Raghbir Singh (deceased). They went to the city police station,
Gohana. Police party along with Mane Ram (PW-8) and Jai Singh reached at the place of
occurrence at about 10.00 A.M on 10.10.1991 and recorded the statement of Mane Ram
(PW-8). Based on the statement of Mane Ram (PW-8) the Police Station Gohana issued
first information report and registered a P.S. Case No. 259 on 10.10.1991 under Section
392 read with Sections 397 and 302 read with Section 34, IPC against the appellant.
4. After completion of the investigation, the police filed charge sheet under Section 392
read with 397 IPC and under Section 302 read with 34 IPC against all the accused
including the appellant. The prosecution in all examined 11 witnesses (PW?1 to PW-11)
and got marked various documents in evidence. The statement of the accused appellant
under Section 313 Cr.P.C. was recorded in which he took the stand that he was innocent
and falsely implicated of the charge levelled against him
5. The learned Sessions Judge upon appreciation of evidence available on record found
the appellant guilty of the offence punishable under Section 392 read with 397 and 302
read with 34, IPC and the same was affirmed by the High Court.
6. Hence this appeal by special leave.
7. We have heard Shri D.P. Singh, learned counsel appearing on behalf of the appellant-
accused and Shri Sandeep Sharma, learned counsel appearing for the State.
8. In order to consider as to whether the prosecution established the charge against the
appellant for the offence punishable under Section 392 read with 397 and 302 read with
34 IPC beyond reasonable doubt it is just and necessary to appreciate the evidence
available on record.
9. In the facts and circumstances of the case, it is not necessary to consider in detail the
statements of all the prosecution witnesses except PW-8. In our view, the prosecution
case entirely rests on the testimony of Mane Ram ( PW-8) who is an independent and
impartial witness. He deposed that the accused persons threw him and Raghbir Singh
(deceased) into the canal after robbing money and the tractor. The relevant portion of his
testimony is as under:
"The accused started robbing of money from the pocket of Raghbir (deceased). On my
intervention, all the accused caught hold of me and pressed by neck and threw me in
canal. It was about 8 P.M. After throwing me in the canal, the accused caught hold of
Raghbir deceased by the legs and also threw him in the canal. Raghbir deceased tried to
catch hold of the grass grown on the bank of the canal. Thereafter, accused Joginder and
Jai kumar gave leg blows on the face of Raghbir deceased. Thereafter Raghbir was again
thrown in the water and did not come out whereas I kept on swimming with the flow of
water and came out of the canal after having covered a distance of about three acres.
After having out of the water I went to V. Kasandi. I had not seen the tractor on the bank
of the canal."
10. The learned counsel for the appellant urged before us that the prosecution fabricated
the evidence to falsely implicate the appellant. The learned counsel for the appellant
further contended that the presence of PW-8 at the scene of occurrence is highly doubtful.
This theory which is now sought to be propounded is contrary to the very defence set up
by the appellant in the sessions case as is evident from the cross-examination of PW-8. It
was suggested to PW-8 that he along with Raghbir (deceased) in a drunken condition was
present at the Kasandi bridge (scene of occurrence) and they were unable to control
themselves and Raghbir laid on the bank of the canal and accidentally fell into the water
and drowned. This suggestion made to PW-8 that PW-8 along with Raghbir Singh
(deceased) was very much present at the scene of occurrence completely negatives the
submission now made by the appellant's counsel. There is nothing on record to doubt the
presence of PW-8 at the scene of occurrence. Nothing has been suggested to PW-8 as to
why he should have given false evidence against the appellant.
11. We also do not found any merit in the contention of the learned counsel for the
appellant that no specific overt act has such been attributed to the appellant and,
therefore, he is entitled to acquittal of the offence alleged against him. PW-8, in clear and
categorical terms in his evidence stated that Joginder Singh all of a sudden stopped the
tractor when it had reached at the spot in
@page-SC450
between two canals near the bridge of canal in village Kasandi. That all the accused
started robbing of money from the pocket of Raghbir (deceased) and when he intervened
all the accused caught hold of him and pressed his neck and he was thrown into the canal.
That after throwing PW-8 into the canal all the accused caught hold of Raghbir
(deceased) by his legs and threw him into the canal. Raghbir (deceased) tried to catch
hold of the grass grown on the bank of the canal but the accused gave blows on the face
of Raghbir (deceased) and was again thrown into the water. Raghbir (deceased) did not
come out from the water.
12. The evidence and material available on record further reveal circumstances to prove
the guilt of the appellant: (1) The first circumstance is the recovery of the dead body of
Raghbir Singh (deceased) from the place of occurrence. (2) The sub-Inspector Man Singh
(PW-11) recovered the Tractor No. HR-06-8501 from the possession of the accused
persons which was the same tractor robbed by the accused. (3) The third circumstance is
recovery of 'Barma' by Sub-Inspector Man Singh (PW-11) from the possession of
Joginder Singh (co-accused) in pursuance of his disclosure statement.
13. The learned counsel vehemently contended that there is no proper explanation
forthcoming from the prosecution for the delay in lodging the first information report and
the same casts a serious doubt on the case set up by the prosecution against the appellant.
We are not impressed by the submission as the evidence available on record reveals the
sequence of events leading to filing of first information report. The distance between the
Kasandi bridge and Mandi is about 10-12 kms. It is in the evidence of PW-8 that after his
coming out of the canal he went to village Kasandi and met Jai Singh and narrated the
whole story to him; thereafter along with Jai Singh he returned to the scene of occurrence
and searched for Raghbir (deceased) and the tractor but could not find them. Thereafter
along with Jai Singh he went to Mandi on his tractor and narrated the story to the father
of the deceased about the occurrence who reported the matter to the Sarpanch of the
village. Thereafter he along with Jai Singh went to police post Israna who refused to
lodge the report but informed the Gohana police. It is true that the distance between the
Kasandi bridge and the police station Gohana is 5 kms. but PW-8 and his evidence had
narrated the sequence of events which reveals the reasons for the delay, if any, in lodging
the first information report at police station Gohana. There is nothing unnatural on the
part of PW-8 in his first going to Kasandi village nearby to the scene of occurrence and
informing Jai Singh about the incident and thereafter returning to Kasandi bridge along
with Jai Singh. Having found the deceased-Raghbir Singh and the tractor were missing he
along with Jai Singh went to Mandi village to inform the father of the deceased. One does
not expect PW-8 to straightway first go to the nearest police station and lodge the first
information report even without informing the near relatives of the deceased.
14. Be that as it may, it is not the case of the appellant that after occurrence of the
incident some deliberations took place in order to falsely implicate the appellant in the
case. No suggestion of any enmity between the appellant and PW-8 has been made. There
is no reason to disbelieve the sequence of events narrated by PW-8. In such view of the
matter mere delay in lodging the first information report, in the facts and circumstances
of the case cannot be held to be fatal to the prosecution case.
15. For all the aforesaid reasons we hold that the prosecution has been able to establish
the guilt of the appellant beyond all reasonable doubt for conviction under Section 392
read with 397 IPC for having robbed money and tractor. The sentence of rigorous
imprisonment for a period of 7 years for each of the offence under Section 392 read with
397 IPC is accordingly upheld.
16. The question that falls for our consideration is whether the facts and circumstances
and the evidence available on record justify the conviction of the appellant under Section
302 read with 34 IPC for having caused death of Raghbir (deceased)? The evidence
available on record does not suggest that there has been any intention of causing the
death of Raghbir (deceased).The case falls under Part II of Section 304 IPC. The
appellant committed the offence of culpable homicide not amounting to murder. The
appellant is accordingly convicted under Part II of Section 304, IPC and sentenced to
undergo rigorous imprisonment for 7 years. The sentences to run concurrently.
17. It is brought to our notice that the appellant has already served the sentence
@page-SC451
of 7½ years rigorous imprisonment. Sentence already undergone is sufficient to meet the
ends of justice. He is accordingly directed to be released forthwith, unless required in
connection with any other case.
18. The appeal is accordingly allowed in part.
Appeal partly allowed.
AIR 2008 SUPREME COURT 451 "Zenith Steel Tubes and Industries Ltd. v. Sicom
Limited"
(From : Bombay)*
Coram : 2 ALTAMAS KABIR AND B. SUDERSHAN REDDY, JJ.
Civil Appeal No. 5347 of 2007(@ SLP (C) No. 8486 of 2007), D/- 21 -11 -2007.
Zenith Steel Tubes and Industries Ltd. and Anr. v. Sicom Limited.
Sick Industrial Companies (Special Provisions) Act (1 of 1986), S.22 - SICK
INDUSTRIAL UNDERTAKING - LARGER BENCH - Term "proceedings" or "suits" in
S. 22 - Interpretation of - Divergent views expressed in decisions by Coordinate Benches
in 2006 AIR SCW 5718 and 2003 AIR SCW 1358 - Matter referred to larger Bench.
(Para 25)
Cases Referred : Chronological Paras
2006 AIR SCW 5718 : AIR 2007 SC 168 : 2006 CLC 1655 9, 11, 22, 24
2003 AIR SCW 1358 : AIR 2003 SC 1886 : 2003 CLC 339 10, 11, 17, 18, 19, 21, 22, 23,
24
2001 (106) Comp Cas 587 (Bom) 19
2000 AIR SCW 521 : AIR 2000 SC 926 : 2000 CLC 453 20
2000 AIR SCW 2740 : AIR 2000 SC 2553 : 2000 CLC 1492 8, 10, 17
AIR 1998 Bom 247 8
1993 AIR SCW 991 7, 10
AIR 1989 SC 2240 15
AIR 1978 SC 1765 21
AIR 1970 SC 196 : 1970 Lab IC 256 14
AIR 1940 PC 105 15
Shekhar Naphade, Sr. Adv., Shivaji M. Jadhav, Himanshu Gupta, Brij Kishor Sah and
Rahul Joshi, for Appellants; Jay Savla and Ms. Meenashi, for Respondent.
* Appeal No. 1 of 2007 in Misc. Petn. No. 64 of 2000, D/- 29-1-2007 (Bom)
Judgement
ALTAMAS KABIR, J. :- Leave granted.
2. The appellant No.1 company was carrying on business of manufacturing galvanised
pipes. In November, 1992 the appellant-company approached the respondent for financial
assistance amounting to Rs.1,42,000/- to meet a part of the cost for setting up a factory in
village Madap, Taluq Kolhapur in the District of Raigarh for the manufacture of
galvanised pipes. The said amount was duly sanctioned and the said sum of
Rs.1,42,000/- was advanced by the respondent to the appellant company. An agreement
was entered into for a term loan and the appellant-company also created a security for
repayment of the amount by hypothecating its plant and machinery and creating an
equitable mortgage of its factory premises situated in the above mentioned village. A loan
agreement was executed on 30.3.1993 for repayment of the loan in various instalments.
On the same day, the second appellant executed a personal guarantee for repayment of
the loan amount in case of default by the appellant-company.
3. The appellant-company committed several defaults in repayment of the loan amount
compelling the respondent to issue a notice on 16.10.98 calling upon the appellant-
company to pay the overdue amount within a stipulated period. Despite such notice, the
appellant-company failed to make payment and accordingly, by a further notice dated
10.1.1999 the respondent called upon the appellant-company to repay the entire amount
due and payable to the respondent by 3.2.1999 failing which the possession of the assets
of the appellant-company would be taken on 5.2.1999.
4. Since, despite such notice the appellant-company failed and neglected to pay the entire
amount as demanded, the respondent issued a notice to the appellant No.2 on 13.6.2000
calling upon him to pay the entire amount by invoking the personal guarantee given by
the second appellant. As in the case of the appellant-company, the second appellant did
not also make the payment as demanded, and consequently, the respondent filed a
petition against the second appellant under Section 31(1)(aa) of the State Financial
Corporations Act, 1951, on 10.10.2000 for enforcing the personal guarantee given by the
said appellant.
5. In the meantime, the appellant-company applied to the Board for Industrial and
Financial Reconstruction (BIFR) and was declared a sick company by the BIFR under the
provisions of the Sick Industrial Companies (Special Provisions) Act, 1985, (SICA), and
the company is still under the said
@page-SC452
Board.
6. Taking advantage of the aforesaid position, the second appellant contended before the
single Judge of the Bombay High Court that in view of Section 22 of the aforesaid Act,
the personal guarantee given by the second appellant could not be invoked. It was also
contended that the respondent could not enforce the guarantee till such time as the assets
which had been mortgaged in its favour had not been realised. Both the said contentions
were rejected by the learned single Judge upon holding that the liability of the guarantor
was independent of that of the principal debtor, and accordingly, the guarantee could be
invoked and the amount could be recovered from the guarantor. The guarantor was
directed to pay Rs.1,67,89,225/- with further interest on the principal amount of Rs.92
lakhs from the date of the petition till payment at the rate of 12%. The said decision of the
learned single Judge was challenged by the appellants herein before the Division Bench
of the Bombay High Court in Appeal No.1/2007. The Division Bench on consideration
of the different decisions of this Court came to the conclusion that the provisions of
Section 22 of SICA, as amended in 1994, did not prohibit any proceeding, other than a
suit for enforcement of any security against the guarantor. On such finding and also upon
holding that the liability of the guarantor was co-extensive with the principal debtor and
that the creditor was not required to exercise his right as a mortgagee before proceeding
against the guarantor, the Division Bench dismissed the appeal with costs on 29.1.2007.
It is the decision both of the learned single Judge as also the Division Bench of the High
Court which is the subject matter of this appeal.
7

. Appearing in support of the appeal, Mr. Shekhar Naphade, learned Senior Advocate,
submitted that both the learned single Judge and the Division Bench of the High Court
had erred in giving a narrow meaning to the word 'suit' as used in Section 22 of SICA. He
submitted that the context in which the expression 'suit' had been used in Section 22 of
the aforesaid Act made such expression all pervasive to include other proceedings as well
before a court or other authority empowered to recover debts and other dues against the
company. It was urged that in the case of Maharashtra Tubes Limited v. State Industrial
Corporation of Maharashtra Ltd. (1993) 2 SCC 144, it had been held that the expression
"proceedings" in Section 22(1) of SICA must be widely construed and could not be
confined to legal proceedings understood in the narrow sense of proceeding in a court of
law or a legal tribunal for attachment and sale of the debtor's property. However, since
the said decision could be applied to companies only and not to guarantors, the legislature
amended the provisions of Section 22(1) so as to extend the protection given to
companies to guarantors also so that they too were given the protection of Section 22 of
the Act. Mr. Naphade submitted that the object with which the 1985 Act was enacted was
primarily to assist sick industries which had failed to meet their financial obligations. It
was urged that in certain cases it was the Directors of the company who themselves stood
guarantee for the loans advanced to the company and the enforcement of such guarantee
against the Directors would cause obstructions in the way of the BIFR to revive the said
company, which was also one of the objects of the 1985 Act. 1993 AIR SCW 991

. In this regard, Mr. Naphade also referred to the decision of this Court in Patheja Bros.
Forgings and Stamping and Anr. v. ICICI Ltd. and others, 2000 (6) SCC 545, where the
question involved was whether Section 22 of the SICA would cover a suit against a
guarantor of a loan or advance that had been granted to an industrial company. Mr.
Naphade pointed out that upon holding that the words of Section 22 were crystal clear
and there was no ambiguity therein, this Court had held that no suit for enforcement of a
guarantee in respect of a loan or advance granted to the industrial company concerned
would lie or could be proceeded with without the consent of the Board or the Appellate
Authority under the Act. Mr. Naphade also submitted that while dealing with the
aforesaid question, this Court had overruled the decision of the Bombay High Court in
Madalsa International Ltd. v. Central Bank of India, AIR 1998 Bom 247, wherein it had
been held that Section 22 would apply to companies only and not to guarantors who
would be affected personally and the words "of any guarantee in respect of any loan or
advance granted to the industrial company" would have to be read as the guarantee given
by the industrial 2000 AIR SCW 2740

@page-SC453
company itself and none else.
9

. To emphasise his aforesaid submission Mr. Naphade laid particular emphasis on the
decision of this Court in Paramjit Singh Patheja v. ICDS Ltd. (JT 2006 (10) SC 41) where
in connection with the enforcement of an arbitral award and the issuance of an insolvency
notice under Section 9(2) of the Presidency Towns Insolvency Act, 1909 this Court, inter
alia, held that it is a well established rule that a provision must be construed in a manner
which would give effect to its purpose and to cure the mischief in the light of which it
was enacted. It was further observed that the object of Section 22 of SICA in protecting
guarantors from legal proceedings pending a reference to BIFR by the principal debtor
was to ensure that a scheme for rehabilitation would not be defeated by isolated
proceedings adopted against the guarantors of a sick company. In order to achieve such
purpose, it was imperative that the expression "suit" in Section 22 be given its plain
meaning, namely, any proceedings adopted for realisation of a right vested in a party by
law. 2006 AIR SCW 5718

10

. Mr. Naphade then submitted that the Bombay High Court had wrongly relied upon the
decision of this Court in Kailash Nath Agrawal and Ors. v. Pradeshiya Industrial and
Investment Corporation of U.P. Ltd. and Anr. (2003 (4) SCC 305), wherein the decision
rendered by this Court in the Maharashtra Tubes' case (supra) as also in Patheja Bros.
Forging case (supra) were distinguished and it was held that in both the cases while
considering the effects of the amendment to Section 22(1) of SICA, the Courts were
concerned with 'suits' which had been dealt with in the case of Patheja Bros, and not with
'proceedings' indicated in the first part of Section 22(1) of the 1985 Act. Mr. Naphade
added that the decision in the Maharashtra Tubes' case (supra) had been rendered prior to
the amendment of Section 22(1) of SICA, where as Patheja's case, as also the case of
Paramjit Patheja were rendered after the amendment was effected, to extend the
protection of Section 22 to guarantors as well. 2003 AIR SCW 1358
1993 AIR SCW 991
2000 AIR SCW 2740
11

. Mr. Naphade submitted that the decision in Kailash Nath Agrawal's case had been
rendered by this Court in the context of interpretation of the expressions 'suit' and
'proceedings' used in Section 22(1) of SICA, 1985. In construing the said two expressions
this Court was of the view that while the expression 'proceedings' used in Section 22(1)
would have to be confined to companies alone, the expression 'suit' had been introduced
by amendment to extend the protective cover of Section 22 to guarantors as well. It was
submitted that the purpose for which such amendment had been effected, namely, to
extend the protective cover of Section 22 to guarantors also, would be rendered
meaningless if coercive action continued to be taken against guarantors who could even
be the Directors of the company in question. It was urged that the continuing ambiguity
was sought to be explained in the Paramjit Singh Patheja case (supra) wherein it was
explained that the expression 'suit' would have to be understood in a larger context to
include other proceedings as well before a legal forum. 2003 AIR SCW 1358
2006 AIR SCW 5718

12. Mr. Naphade submitted that, in any event, the liability of the appellant No. 2 under
the guarantee given could be enforced under Section 31((1)(aa) of the State Financial
Corporations Act, 1951, only if and when the appellant made a default in repayment of
the loan. Having regard to the fact that the appellant No.1 had made a reference to the
BIFR under Section 15 of the 1985 Act, the liability of the appellant-company stood
suspended under Section 22 of the said Act. As the liability of the appellant-company
stood suspended, there could be no question of any default having been committed by the
appellant company towards repayment of the loan. According to Mr. Naphade, since the
respondent had filed an application under Section 31(1)(aa) of the above Act making only
a monetary claim against the appellant No.2, on a true construction of the above
provisions the said Section permits enforcement only of the security given by the
guarantor and since in the instant case the respondent had filed an application not for
enforcement of any security but for claiming only the amount of guarantee the same
could not be enforced against the appellant No. 2. According to Mr. Naphade the
appellant No. 2 has not given any other security which could be proceeded against by the
respondent.
13. Mr. Naphade submitted that the Bombay High Court had no jurisdiction to
@page-SC454
entertain the application made under Section 31(1)(aa) of the Act and the order passed
there above was a nullity.
14

. It was also submitted that I.A. No.1 of 2007 was filed in the special leave petition for
leave to place on record additional grounds as set out in the application and prayed that
the same be allowed to be placed on record by way of additional grounds. Inasmuch as,
such prayer was objected to on behalf of the respondent, Mr. Naphade referred to the
decision of this Court in the Management of State Bank of Hyderabad v. Vasudev Anant
Bhide etc., 1969 (2) SCC 491, wherein while considering as to whether a claim was
barred under Article 137 of the Limitation Act, an objection was taken that such ground
had not been raised either before the Labour Court or even in the special leave petition
filed in this Court. In the said case, on an application made to permit the appellant to raise
the question of limitation based upon Article 137 of the Limitation Act, this Court
permitted the appellant to raise such plea as no fresh facts were required to be
investigated and the matter could be dealt with as a pure question of law. AIR 1970 SC
196

15

. Mr. Naphade also referred to the decision of this Court in Pandurang Ramchandra
Mandlik v. Shantibai Ramchandra Ghatge and Ors. (1989 Supp (2) SCC 627) which was
a case dealing with ousting of jurisdiction of the Civil Court with regard to the provisions
of Section 80 and Section 85 of the Bombay Tenancy and Agricultural Lands Act, 1948.
Referring to the decision of the Judicial Committee in Secretary of State v. Mask and
Company (AIR 1940 PC 105), where it was observed that the exclusion of the
jurisdiction of the Civil Court was not to be readily inferred, but that such exclusion must
either be explicitly expressed or clearly implied, it was held that there was nothing in the
language or context of Section 80 or Section 85 of the above Act to suggest that the
jurisdiction of the Civil Court was expressly or by necessary implication barred with
regard to the question as to whether the defendants have become statutory owners of the
land. AIR 1989 SC 2240

16. Mr. Naphade concluded his submissions by urging that both the learned single Judge
and the Division Bench of the Bombay High Court had misconstrued the provisions of
Section 22 of the 1985 Act, as amended, in holding that the amended provisions granting
protection to guarantors in suits for enforcement, could not be stretched to include
proceedings for enforcement as well.
17

. Appearing for the respondent, Mr. Jay Savla, learned advocate, contended that the
controversy regarding the protection given by Section 22 of SICA to guarantors had been
set at rest by this Court in Kailash Nath Agrawal's case (supra). He submitted that while
in the case of Patheja Bros. Forgings and Stamping case (supra) this court had to
consider whether a suit against a guarantor would be covered by the protection provided
under Section 22(1) of SICA, the question in Kailash Nath Agrawal's case this Court was
concerned not with a "suit" but a proceeding for recovery of dues and in those
circumstances this Court had examined the use of the expressions "proceeding" and "suit"
used in different parts of Section 22(1) of SICA. It was in that context that this Court
distinguished the earlier decision in Patheja Bros. Forgings and Stampings case and upon
holding that since the legislature had expressly chosen to make a distinction between
suits for recovery of money and enforcement of guarantees and proceedings for the
recovery of money, such distinction had to be given effect to. It was held that even under
the amended provisions only a limited protection had been afforded to guarantors with
regard to the recovery of dues by way of suit, but not by way of proceedings, and,
accordingly, a proceeding for recovery of money against a guarantor would stand outside
the protection afforded under Section 22(1) of the 1985 Act. 2003 AIR SCW 1358
2000 AIR SCW 2740

18

. It was urged that in the instant case, a situation similar to that in Kailash Nath Agrawal's
case had arisen, since the proceeding had been initiated against the guarantor under the
relevant provisions of the State Financial Corporations Act, 1951, which stood outside the
purview of Section 22(1) of SICA. 2003 AIR SCW 1358

19. Mr. Savla submitted that although the decision in Kailash Nath Agrawal's case was
not referred to by the Division Bench of the Bombay High Court, a similar decision
rendered by the Division Bench of the Bombay High Court in Dewal Singhal v. State of
Maharashtra (2001 (106) Company Cases 587) was relied upon. In the said decision it
was held that the protection conferred on
@page-SC455
guarantors under Section 22 of SICA is a limited protection and the bar is restricted only
to a suit and did not apply to any other proceedings.
20

. Mr. Savla referred to the decision of this Court in BSI Ltd. and Anr. v Gift Holdings Pvt.
Ltd. and Another, (2000 (2) SCC 737), which was rendered in a slightly different
situation involving a fine imposed on a company in a criminal case against the company
and its Directors under Section 138 of the Negotiable Instruments Act, 1881. It was held
in that case that the ban envisaged in Section 22(1) of SICA would not be attracted in
case of punishment of fine imposed on the company for such offence if it was with the
consent of the BIFR. Furthermore, the ban imposed under the said provision of SICA
against maintainability of a suit for recovery of money would not cover prosecution
proceedings for an offence under Section 138 of the Negotiable Instruments Act. This
Court observed that as the ambit of "suit" has been clearly delineated in Section 22(1)
itself, it could not be stretched by employing the maxim that contemporaneous exposition
is the best and strongest in law. 2000 AIR SCW 521

21

. Mr. Savla urged that a proceeding under the State Financial Corporations Act could not
be equated with a suit as had been held by this Court in Gujarat State Financial
Corporation v. M/s. Natson Manufacturing Co.(P) Ltd. (1979 (1) SCR 372) and having
regard to the decision in Kailash Nath Agrawal's case (supra) such a proceeding would
not be entitled to the protection envisaged under Section 22(1) of SICA. AIR 1978 SC
1765
2003 AIR SCW 1358

22

. As to the second limb of Mr. Naphade's submission regarding the right of the respondent
to proceed against the guarantor before realising its securities, Mr. Savla reiterated the
High Court's view that the claim against the guarantor was against him personally and
was independent of the sureties given in mortgage by the Principal Debtor. Mr. Savla
submitted that the decision rendered in Kailash Nath Agrawal's case does not appear to
have been brought to the notice of the Hon'ble Judges deciding the Paramjeet Singh
Patheja case (supra) and same was decided on other earlier decisions of this Court which
dealt essentially with suits for recovery of dues. 2003 AIR SCW 1358
2006 AIR SCW 5718

23. It was submitted that since the Division Bench of the High Court took a view which
finds support in Kailash Nath Agrawal's case, no case had been made out for interference
with the same.
24

. In the decisions of this Court cited before us, two divergent views have been expressed
in respect of the same issue involved in this appeal. In the other decisions, this Court had
no occasion to go into the said issue which involved the interpretation of the Section
22(1) of the SICA in respect of either proceedings or 'suits' respectively. In Kailash Nath
Agrawal's case (supra) this Court has taken the view that the legislature appears to have
knowingly used two different expressions in Section 22(1) of SICA, namely, 'proceeding'
in the first part and the expression 'suit' in the second part and the protection of Section
22 extended to guarantors in respect of suits alone and the use of the expression
'proceeding' could not be extended to include suits as well nor could the expression 'suit'
be extended to include the expression 'proceeding' also. On the other hand, in Paramjeet
Singh Pathejas case (supra) it was held that the expression 'suit' which extends the
protection of Section 22(1) to guarantors, would have to be interpreted to include
'proceeding' also, in view of the intention of the legislature to protect sick industrial
companies where references were pending before the BIFR. It is also evident from the
decision in Paramjeet Singh Patheja's case (supra) that the views expressed in Kailash
Nath Agrawal's case (supra) had not been brought to the notice of the learned Judges who
decided the matter. Even if we are inclined to agree with one of the two interpretations,
the anomalous situation will continue since the decisions are that of coordinate Benches.
2003 AIR SCW 1358
2006 AIR SCW 5718

25. In such circumstances, we consider it fit and proper that the matter should be referred
to a larger Bench to resolve the existing anomaly resulting from the different views
expressed in the two above-mentioned cases.
26. Accordingly, the Registry is directed to place this matter before the Hon'ble the Chief
Justice of India for appropriate orders in the light of what has been stated hereinbefore.
Order accordingly.
@page-SC456
AIR 2008 SUPREME COURT 456 "O. N. G. C. Ltd. v. Garware Shipping Corporation
Ltd."
(From : Bombay)*
Coram : 2 Dr. A. PASAYAT AND S. H. KAPADIA, JJ.
Civil Appeal No. 5210 of 2007 (arising out of SLP (C) No. 15036 of 2005), D/- 14 -11
-2007.
O. N. G. C. Ltd. v. Garware Shipping Corporation Ltd.
(A) Arbitration and Conciliation Act (26 of 1996), S.34 - ARBITRATION AND
CONCILIATION - Award - Setting aside - Charter of ships by ONGC - Dispute over
computation of cost of repairs and maintenance (R and M) of vessels to be paid to
lessors - Arbitrator passing award on basis that year of operation of vessel would be
relevant factor for cost of R and M - Terms of agreement fixing a normative value as per
SCI vessels, escalation @ 9.5% per year and reimbursement of difference between
normative amount given and the actual R and M expense of SCI's ships of 5400 BHP -
Stress, therefore, is on reimbursement - Year of operation is thus immaterial - Award
passed on year of operation basis liable to be set aside. (Para 27)
(B) Arbitration and Conciliation Act (26 of 1996), S.34 - ARBITRATION AND
CONCILIATION - Setting aside of award - Award based on wrong basis and perverse
conclusions - Liable to be set aside - No proposition that Courts could be slow to interfere
with arbitrator's Award, even in such cases. (Para 28)

Ashok Desai, Sr. Advocate, Gopal Jain, Vipin Nair, P. B. Suresh, Ms. Mridul (for M/s.
Temple Law Firm), for Appellant; R. F. Nariman, Mukul Rohatgi, Sr. Advocates, Biren
Saraf, Santosh Paul, Abhishek K. Rao, Praji K. J., M. J. Paul, for Respondent.
* Appeal No. 57 of 2005 in A. P. No. 329 of 2004, D/- 1-3-2005 (Bom).
Judgement
Dr. ARIJIT PASAYAT, J. :-Leave granted.
2. Challenge in this appeal is to the judgment rendered by a Division Bench of the
Bombay High Court dealing with an appeal questioning the correctness of the order
passed by a learned Single Judge who dismissed the appellant's appeal under Section 34
of the Arbitration and Conciliation Act, 1996 (in short the Act) questioning the Arbitrators
award.
3. The controversy lies within a very narrow compass.
4. The factual background is almost undisputed and is essentially as follows:
The appellant required off shore vessels (in short OSVs) inter alia, for supplying material
from its onshore bases to its offshore installations. After initially meeting its requirements
by chartering foreign OSVs, the appellant decided to develop a fleet of Indian Flag
vessels. Various Indian companies including the respondent and the Shipping Corporation
of India (in short SCI) acquired OSVs, with a view to chartering them to the appellant.
The respondent acquired five vessels (named Garware I to Garware V) which were
handed over to the appellant in the months of November and December, 1983 and
January and March, 1984.
The dispute pertains to the cost of repairs and maintenance of the respondent's OSVs for
the eleventh to the sixteenth year of their operation. Even though there is no dispute
regarding the first two terms of five years each, reference to the manner in which the
rates for the same were arrived at is necessary. A working group under the Director
General of Shipping was constituted by the Ministry of Petroleum to determine the floor
day rate in respect of the vessels keeping two objects in mind, i.e. (a) long term
availability of the OSVs for the appellant and (b) economic viability to ensure the
respondents survival in the business. The report was submitted by the working group on
8.3.1984 suggesting the day rate which comprised of two components, i.e. (a) capital
recovery factor and (b) operating expenses. Contracts were accordingly entered into for
the first five year period beginning from 1983-84. The Government of India by an order
dated 18.8.1984 approved the report in certain respects only. There is no dispute between
the parties regarding the payments of operating costs for the first five years. The charter
was extended by another five years. A committee presided over by Dr. A.N.Saxena was
formed to review the operating costs payable for the extended term. The Government
approved the report of the Committee on 5.8.1993. There is no dispute between the
parties in respect of the payments regarding the second five year period also.
5. The present dispute relates to the period beyond ten years so far as relates to
@page-SC457
the basis for computing the rates for repairs and maintenance. By an order dated
29.4.1993 the charter was extended by a further six years. By an order dated 16.3.1995
and as modified by an order dated 14.9.1995, a committee also presided over by Dr. A.N.
Saxena was formed to recommend a suitable formula for the charter rate for the further
extended period.
6. The committee submitted its report on 14.9.1997. This committee made
recommendations inter alia in respect of repair and maintenance expenses. The reference
to arbitration was confined only to the payment of these repairs and maintenance
expenses.
7. The Government of India by a letter dated 15.6.1998 accepted the recommendations of
the second Dr. A.N. Saxena Committee only partially. Representations were thereafter
made by the Indian Shipping Companies including the respondent for reconsideration of
the recommendations. Pursuant thereto, the Government of India appointed a High Level
Working Group presided over by Mr. Naresh Narad for considering the outstanding
pending issues. The following recommendations of the High Power Committee are
relevant:

?Pending Issues.
1. Determination of year from which R and M expenses are to be actualized. a) 1 to
5 years as per payments already made. Settled cases Not to be reopened.
b) 6 to 10 years as per norms fixed by Dr. Saxena Committee of 1995-77.
c) 11 and 12 years to actualized on the basis of S.C.Is OSVs as recommended by Dr.
Saxena Committee of 1995-77.?

8. Disputes and differences arose between the appellant and inter alia the respondent and
others regarding the method to the adopted for calculating rates payable with reference to
the eleventh to the sixteenth years. The respondent, therefore, filed Writ Petition No.2788
of 2001 for various reliefs.
9. By an order dated 7.12.2001 a Division Bench of the High Court recorded that the Writ
Petition involved certain contractual disputes and that both the parties had agreed to refer
the disputes raised in the Writ Petition to the sole arbitration of Mr. Justice M.L.Pendse (a
former Judge of the Bombay High Court and the former Chief Justice of the Karnataka
High Court). The order which is a short one, reads as follows:
"This writ petition involves certain contractual disputes relating to repairs and
maintenance expenses etc. contract between the parties contain an arbitration clause.
Both the parties agree to refer the disputes raised in the writ petition to sole arbitration of
Justice M.L. Pendse (Retd.). Parties further agree that in case Justice Pendse is not in a
position to take up the arbitration, Justice D.R. Rege (Retd.) shall be the arbitrator for the
disputes between the parties. Arbitrator is requested to dispose of the arbitration as
expeditiously as possible.
Petition is disposed of."
10. The learned arbitrator noted the respondents submission that while calculating the
payments due for the 11th to 16th years of the operation of the OSVs of the respondent,
the appellant has overlooked the important fact that the OSVs of the respondent were
taken on charter one year prior to the appellant obtaining the OSVs of the SCI. The
respondent, therefore, contended that it was not correct that the corresponding years of
SCI should be taken into account while determining amounts payable to the respondent.
The learned arbitrator rejected the appellants contention. He held that the committee
nowhere recommended that irrespective of the period of induction,
@page-SC458
the years should be calculated of that of the SCI. He held that the respondent had not
questioned the recommendations made by the High Level Working Group Report and the
second Saxena Committee Report but merely contended that the mode of implementation
thereof was incorrect. The arbitrator further observed and accepted that it was not open
for him to go behind the report and the only area of enquiry is whether or not the report
was correctly implemented. He came to the conclusion that on a close scrutiny of the
reports, it was clear that neither of the committees examined whether the entitlement of
each OSV is to be determined with reference to the years of actual user or only with
reference to the calendar years. He also came to the conclusion that for the computation
of repairs and maintenance expenses, it was necessary to take into consideration the years
of operation and not the calendar years. It was held that the 12th year of operation of
SCIs OSVs should be equated with the 13th year of operation of the OSVs of the
respondent and so on. He also held that the interpretation suggested by the appellant
would lead to great injustice. For instance, the OSV of the respondent would complete 11
years of operation while the OSVs of the SCI would have operated only for 10 years.
11. Appellant questioned correctness of learned Arbitrators conclusion by filing an appeal
under Section 34 of the Act. Learned Single Judge dismissed the appeal holding that the
conclusion was rational. An appeal filed was also dismissed.
12. According to the Division Bench, the learned arbitrator has considered and construed
the reports while arriving at his conclusions. The entire dispute in the Writ Petition and
before the learned arbitrator centered around this issue. The basis of the calculation
adopted by the learned arbitrator was, not only logical but just and fair. The provisions of
the said reports are not such that they required no interpretation and were merely to be
applied without anything more. They called for a proper interpretation and construction
before being applied to the facts of the case. The learned arbitrator did so.
13. The learned Single Judge held that undoubtedly there was no reference so far as the
period of 13 to 16 years are concerned to the learned Arbitrator. But the prayers and the
writ petitions clearly indicated that even for that period an issue was raised.
14. The Division Bench was of the view that even if the mode of calculation as applied
by the arbitrator is not very appropriate in its effect, that could not be a ground for
exercise of power under Section 34.
15. It noted that the reference in fact did not include the 13th to the 16th year to inspect
that the arbitrator thought it improper to open the same. The High Court was of the view
that a narrow technical reading of the Award cannot be made.
16. In support of the appeal, learned counsel for the appellant submitted that both learned
Single Judge and the Division Bench failed to notice that the Award made by the
Arbitrator was beyond the reference made. The arbitrators view that the corresponding
year could be a more appropriate factor is without foundation. The Bench mark of SCI in
a particular year could not be departed from. There was no scope for shifting of figures.
There is no rule of universal application that the cost of maintenance would be more
when the vehicle becomes older. The normative figure for third period remained constant.
The order of operation is the operating order and the financial order is defining. Though
in the Writ Petition there was challenge to 13 to 16 years, a bare reading of the writ
petition shows that it did not relate to the said period.
17. In response, learned counsel for the respondent submitted that two views are possible
and, therefore, High Court's view should not have interfered. Arbitrator had accepted one
view which is possible. No one says that it is dehors the Committees report. It is a case
where no interference is called for under Article 136 of the Constitution of India, 1950 (in
short the Constitution) as substantive justice had been done, even though the order may
be wrong on some parts.
18. Some of relevant parts of the Report of HLWG need to be noted:
" This High Level Working Group therefore, concludes that Rand M expenses are to be
actualized with effect from the 11th year of operation."
It was further noted as follows:
"This Committee was seized of the anomaly of lower rates being paid to those owners
who exercised greater management effectiveness by ensuring lower capital costs,
@page-SC459
lower interest rates and lower debt equity ratios."
19. The High Level Working Group, therefore, felt that though it is now not possible to
correct any anomalies that may have crept in during the first twelve years, at least for the
last term of four years the formula should reflect, as far as practicable, the principle of
equal pay for equal work.
20. It is to be noted that the anomalies referred to in the subsequent paragraphs relate to
the anomaly of lower rates being paid to owners to exercise greater management
effectiveness by ensuring lower capital because of lower interest rate and lower rate
equality ratio.
21. The recommendations of the HLWG are as follows:

1. Determination of year from which R and M expenses are to be actualized a.1st to


5th year as per payments already made. Settled cases not to be reopened.
b. 6th to 10th year as per norms fixed by the Dr. Saxena Committee of 1995-97.
c.11th and 12th years to be actualised on the basis of SCIs OSVs as recommended by the
Dr. Saxena Committee of 1995-97.
5. Ceiling rates for A type Vessels only pertaining to the period beyond 12 years of
operation a. From 1st to 5th year ceiling rates as already paid by ONGC.
b. From 6th to 10th year floor rates to be paid by ONGC.
c. For the 11th and 12th years ceiling rates to be paid by ONGC
6. Compensation in lieu of CRF a. The Operating Expenses (including Crew Salary
and Wages covering agreements between INSA and MUI/ NUSI) as determined on the
last day of the 12th year of operation for each vessel, (as per recommendations of the Dr.
Saxena Committee and further modified by this Working Group) to be fixed and made
applicable for the next four year i.e. from the 13th to the 16th year.

22.Operating costs are to be calculated on the basis of actual expenditure incurred by SCI
in operating SCIs OSVs for full (first) one year period.
23. The committee observed that the actual expenses of SCI have not followed any
uniform pattern. The scale to be suggested by the committee needed to be based on some
normative amount for a base year over which an escalation of 9.5% per annum may be
considered for a block of five years and for subsequent block of five years the base may
be changed in the same pattern as that of victualling cost. The committee observed
@page-SC460
that the total cost of repair and maintenance for the block of five years, that is, 1988-89 to
1992-93 of SCI's OSVs is Rs.106.482 lacs per OSV as against the recommended amount
of Rs.97.618 lacs given in the JS and FA committee report. The committee considered
that the SCIs audited statement of Rand M expenses may be considered as appropriate
amount for reimbursement to shipowners. The difference between SCIs audited Rand M
expenses and the normative amount was to be reimbursed to the shipowners on receipt of
SCIs audit statement from time to time in proportion to the BHP of the respective OSVs.
24. As the concept of reimbursement is the measure fixed, the year of operation can vary
is an irrelevant factor. The repair and maintenance expenses have also been dealt by the
Committee.
25. In accordance with the deliberation of the Committee on this at para 3.5.5, the
committee recommended the normative Rand M expenses of OSV of 5400 BHP for the
year 1988-89 to 1993-94 to be same as given in the JSand FA Committee report. The
Rand M expenses for the subsequent years were recommended at the rate of 9.5%
escalation per annum on the expense of the year 1993-94 upto 1998-99 and the amount
for of subsequent years @ 9.5% escalation per annum. The difference between the
recommended normative amount given and the actual Rand M expense of SCI's OSVs
of 5400 BHP (audited statements) were to be reimbursed on year to year basis after
receipt of the audited statement from SCI additional reimbursement of corresponding
overhead expenses in the ratio of 15:85 of the differential amount will also be made. The
differential amounts for other OSVs were recommended to be calculated pro rata basis of
the BHP of the respective OSVs w.r.t. above differential amount for OSVs of 5400 BHP.
26. Though there was some controversy as to whether the year referred to is the financial
year as reimbursement was on year to year basis after receipt of the auditors statement
from SCI the norms obviously relate to financial year.
27. A few factual aspects need to be noted. So far as Essar is concerned, the year is same
as SCI. In case of Bann, there was one time settlement and it is only JESCO which
challenged the report. SCIs first year of operation was 1984-85. The figures for that year
provide some material for rationalization. It is to be noted that stress is on re-
imbursement. Thus the measure is fixed and, therefore, year of operation is immaterial. It
needs no reiteration that claim was for 11th and 12th years and the award also covered
from 13th to 16th year. It is also to be noted that the HLWG referred to certain anomalies.
But they related to the previous years. The Bench Mark is the figure of SCI of particular
year. So when entry to business was made is irrelevant.
28. There is no proposition that the Courts could be slow to interfere with the arbitrators
Award, even if the conclusions are perverse, and even when the very basis of the
Arbitrators award is wrong. In any case this is a case where interference is warranted and
we set aside the norms prescribed by the Arbitrator as upheld by the learned Single Judge
and the Division Bench.
29. The appeal is allowed to the aforesaid extent with no order as to costs.
Order accordingly.
AIR 2008 SUPREME COURT 460 "United India Insurance Co. Ltd. v. Serjerao"
(From : Bombay)*
Coram : 2 A. PASAYAT AND P. SATHASIVAM, JJ.
Civil Appeal No. 5201 with 5202 to 5205, 5207 and 5208 of 2007 (arising out of SLP (C)
No. 9417 with 9418 to 9423 of 2005), D/- 14 -11 -2007.
United India Insurance Co. Ltd. v. Serjerao and Ors.
(A) Motor Vehicles Act (59 of 1988), S.147 - MOTOR VEHICLES - INSURANCE -
Liability of insurer - Labourers travelling in tractor trollies - Insurance Company has no
liability. (Para 8)
(B) Motor Vehicles Act (59 of 1988), S.140, S.173 - MOTOR VEHICLES - APPEAL -
No fault liability - Order passed on basis of - Appealable u/S. 173. (Para 6)
Cases Referred : Chronological Paras
2007 AIR SCW 3734 : AIR 2007 SC 1971 (Rel. on, Pnt. A) 8
2007 AIR SCW 4590 : AIR 2007 SC 2582 (Foll., Pnt. B) 6, 8
Sudhir Kumar Gupta, for Appellant.
* W. P. No. 4187 of 2003, D/- 20-4-2004 (Bom) (Aurangabad Bench)
Judgement
Dr. ARIJIT PASAYAT, J. :- Leave granted.
@page-SC461
2. Challenge in these appeals is to the order passed by a learned Single Judge of the
Bombay High Court, Aurangabad Bench dismissing the writ petitions filed by the
appellant (described hereinafter as 'the Insurance Company'). The controversy lies within
a very narrow compass.
3. The respondents were travelling in the trolly attached to a tractor as labourers. They
claimed to have suffered injuries because the tractor with the trolly in each case met with
an accident. Petitions claiming compensation under the Motor Vehicles Act, 1988 (in
short 'the Act') were filed along with application under Section 140 of the Act. Order was
passed by the learned Additional District Judge and Ex-officio Member, Motor Accident
Claims Tribunal, Latur (in short 'the MACT') on the principle of no fault liability. The
Insurance Company took the stand that it had no liability in respect of the persons
travelling in the Trolly and the owner of the Tractor is liable to pay compensation. This
plea was rejected by the MACT. Appeal in terms of Section 173 of the Act in each case
was preferred before the High Court. Learned Single Judge, prima facie, was of the view
that the appeal was not maintainable. Nevertheless, he referred the matter to the Division
Bench, which, it appears referred it to a Full Bench. While the matter was pending
consideration by the Full Bench, execution proceedings were filed. Therefore, writ
petitions were filed before the High Court. The High Court, by the impugned order in
each case, dismissed the writ petitions holding that though arguable questions were
involved, the writ petitions did not deserve consideration.
4. In support of the appeals, learned counsel for the appellant-Insurance Company
submitted that the appeals in terms of Section 173 of the Act were maintainable and in
any event, the Insurance Company has no liability in respect of the persons travelling in
trollies attached to the tractors.
5. There is no appearance on behalf of the respondents when the matter was called.
6

. So far as the question of maintainability aspect is concerned, the issue is concluded by a


judgment of this Court in Smt. Yallwwa and Ors. v. National Insurance Co. Ltd. and Anr.
(2007 (8) SCALE 77). 2007 AIR SCW 4590

7. In paragraphs 16 to 19 of the judgment, it was observed as follows:


"16. The question which is required to be considered is what would be the meaning of the
term 'award' when such a contention is raised. Although in a given situation having regard
to the liability of the owner of the vehicle, a Claim Tribunal need not go into the question
as to whether the owner of the vehicle in question was at fault or not, but determination
of the liability of the Insurance Company, in our opinion, stands on a different footing.
When a statutory liability has been imposed upon the owner, in our opinion, the same
cannot extend the liability of an insurer to indemnify the owner, although in terms of the
insurance policy or under the Act, it would not be liable therefor.
17. In a given case, the statutory liability of an Insurance Company, therefore, either may
be nil or a sum lower than the amount specified under Section 140 of the Act. Thus, when
a separate application is filed in terms of Section 140 of the Act, in terms of Section 168
thereof, an insurer has to be given a notice in which event, it goes without saying, it
would be open to the Insurance Company to plead and prove that it is not liable at all.
18. Furthermore, it is not in dispute that there can be more than one award particularly
when a sum paid may have to be adjusted from the final award. Keeping in view the
provisions of Section 168 of the Act, there cannot be any doubt whatsoever that an award
for enforcing the right under Section 140 of the Act is also required to be passed under
Section 168 only after the parties concerned have filed their pleadings and have been
given a reasonable opportunity of being heard. A Claims Tribunal, thus, must be satisfied
that the conditions precedent specified in Section 140 of the Act have been substantiated,
which is the basis for making an award.
19. Furthermore, evidently, the amount directed to be paid even in terms of Chapter-X of
the Act must as of necessity, in the event of non-compliance of directions has to be
recovered in terms of Section 174 of the Act. There is no other provision in the Act
which takes care of such a situation. We, therefore, are of the opinion that even when
objections are raised by the Insurance Company in regard to its liability, the Tribunal is
required to render a decision upon the issue, which would attain finality and,
@page-SC462
thus, the same would be an award within the meaning of Section 173 of the Act."
8

. So far as the question of liability regarding labourers travelling in trollies is concerned,


the matter was considered by this Court in Oriental Insurance Company Ltd. v. Brij
Mohan and Ors. (2007 (7) SCALE 753) and it was held that the Insurance Company has
no liability. In view of the aforesaid two decisions of this Court, we set aside the
impugned order in each case and remit the matters to the High Court to consider the
matters afresh in the light of what has been stated by this Court in Smt. Yallwwas case
(supra) and Brij Mohans case (supra). 2007 AIR SCW 3734
2007 AIR SCW 4590

9. The appeals are accordingly disposed of with no order as to costs.


Order accordingly.
AIR 2008 SUPREME COURT 462 "Gali Venkataiah v. State of A. P."
(From : Andhra Pradesh)*
Coram : 2 Dr. A. PASAYAT AND P. SATHASIVAM, JJ.
Criminal Appeal No. 1533 of 2007 (arising out of SLP (Cri.) No. 5907 of 2006), D/- 12
-11 -2007.
Gali Venkataiah v. State of A.P.
(A) Evidence Act (1 of 1872), S.3 - EVIDENCE - WITNESS - PLEA - FALSE
IMPLICATION - Interested witness - Relationship - Does not affect credibility - If plea
of false implication is made foundation therefor has to be laid.
AIR 1953 SC 364, Rel. on. (Para 7)
(B) Penal Code (45 of 1860), S.300, Exception 4 - MURDER - CULPABLE HOMICIDE
- Murder - Sudden fight - Accused and deceased brothers - Relationship between them
strained - Incident starting with exchange of hot words, then quarrel between the two and
culminating in accused inflicting knife blow on chest of deceased - Accused entitled to
benefit of Exception 4 to S. 300 - Conviction liable to be altered to one under S. 304, Part
I. (Paras 14, 16, 18)
(C) Penal Code (45 of 1860), S.300, Exception 4 - MURDER - Sudden fight - Benefit of
Exception 4 to S. 300 when available - Scope of explanation unfolded. (Para 11)
Cases Referred : Chronological Paras
(2007) 1 SCC 699 (Ref.) (Pt. A) 13
2006 AIR SCW 1678 : 2006 Cri LJ 2111 : 2006 (3) AIR Bom R 321 : 2006 (2) AIR Jhar
R 666 (Ref.) (Pt. B) 17
2004 AIR SCW 7376 : AIR 2005 SC 1460 (Ref.) (Pt. A) 13
2002 AIR SCW 4271 : AIR 2002 SC 3633 : 2003 Cri LJ 41 12
(2002) 3 SCC 76 12
AIR 1974 SC 276 : 1974 Cri LJ 331 (Ref.) (Pt. A) 9
AIR 1973 SC 2407 : 1973 Cri LJ 1589 12
AIR 1965 SC 202 : 1965 (1) Cri LJ 226 (Ref.) (Pt. A) 11
AIR 1957 SC 614 : 1957 Cri LJ 1000 (Ref.) (Pt. A) 9
AIR 1953 SC 364 : 1953 Cri LJ 1465 (Rel. on) 8, 10
AIR 1952 SC 54 : 1952 Cri LJ 547 10
Sidharth Luthra, Sr. Advocate, Sameer Parekh, Lalit Chauhan, Ms. Ranjeet Rohtagi and
Ms. Diksha Rai (for M/s. Parekh and Co. ), for Appellant; Mrs. D. Bharathi Reddy, for
Respondent.
* Cri. Appeal No. 1105 of 2003, D/- 31-10-2005 (AP)
Judgement
1. Dr. ARIJIT PASAYAT, J. :- Leave granted.
2. Challenge in this appeal is to the order passed by a Division Bench of the Andhra
Pradesh High Court upholding the conviction of appellant for offence punishable under
Section 302 of the Indian Penal Code, 1860 (in short the 'IPC') and sentence of
imprisonment for life and fine of Rs.1,000/- with default stipulation in terms of the
judgment of learned 4th Additional Sessions Judge, Nellore.
3. Background facts in a nutshell are as follows:
One Gali Krishnaiah (hereinafter referred to as the 'deceased') Gali Seethaiah and the
appellant are brothers and the relation between them was strained. Prior to the incident,
the appellant threatened the deceased that he would kill him. While so, on 13.09.1999, at
about 8.30 a.m. the appellant with an intent to kill the deceased, armed with a knife, went
to him, pulled him and stabbed on his left side of the chest and caused vital stab injury,
besides causing another cut injury over middle of the left forearm. The knife pierced into
the chest of the deceased and struck. When the sons of the deceased raised hue and cry,
the
@page-SC463
appellant left the spot leaving the knife there itself. On the way to the hospital, the
deceased succumbed to the injuries sustained by him. Based on the complaint presented
by the wife of the deceased (PW1), a case in Crime No. 161 of 1999 on the file of the II
Town (L and O) P.S., Nellore was registered and the same was investigated into. After
completion of investigation, charge-sheet was filed. Accused denied the charges and
claimed false implication. During trial, twelve witnesses were examined to further
prosecution version. Placing reliance on the evidence of eye-witnesses PWs 1 to 3,
conviction as noted above, was recorded and sentence imposed.
4. Challenging correctness of the judgment rendered by the trial court an appeal was
preferred before the High Court. The primary stand was that the witnesses PWs 1 to 3
were the wife and the sons of the deceased and were, therefore, interested witnesses.
Further the other witnesses who were independent did not support the prosecution
version. In any event it was submitted that an offence under Section 302, IPC is not
made out.
5. The prosecution supported the judgment of the trial court. The High Court noticed that
the evidence of P.Ws. 1 to 3 is clear, cogent and credible and therefore the conviction
cannot be faulted. It was also noticed that the evidence of PW6 was to the effect that he
found the appellant and the deceased struggling with each other and therefore it was of
the view that the conviction as recorded by the trial court did not suffer from any
infirmity.
6. In support of the appeal learned counsel for the appellant submitted that the evidence
of P.Ws. 1 to 3 should not have been relied upon as they were related to the deceased.
Further the evidence of P.Ws. 4 and 6 who did not support the prosecution version in its
entirety should not have been acted upon. In any event, it was contented that the assault
was made in course of sudden quarrel.
7. We shall first deal with the contention regarding interestedness of the witnesses for
furthering prosecution version. Relationship is not a factor to affect credibility of a
witness. It is more often than not that a relation would not conceal actual culprit and
make allegations against an innocent person. Foundation has to be laid if plea of false
implication is made. In such cases, the court has to adopt a careful approach and analyse
evidence to find out whether it is cogent and credible.
8

. In Dalip Singh and Ors. v. The State of Punjab (AIR 1953 SC 364) it has been laid
down as under: (Para 26)

"A witness is normally to be considered independent unless he or she springs from


sources which are likely to be tainted and that usually means unless the witness has
cause, such as enmity against the accused, to wish to implicate him falsely. Ordinarily a
close relation would be the last to screen the real culprit and falsely implicate an innocent
person. It is true, when feelings run high and there is personal cause for enmity, that
there is a tendency to drag in an innocent person against whom a witness has a grudge
along with the guilty, but foundation must be laid for such a criticism and the mere fact of
relationship far from being a foundation is often a sure guarantee of truth. However, we
are not attempting any sweeping generalization. Each case must be judged on its own
facts. Our observations are only made to combat what is so often put forward in cases
before us as a general rule of prudence. There is no such general rule. Each case must be
limited to and be governed by its own facts."
9

. The above decision has since been followed in Guli Chand and Ors. v. State of
Rajasthan (1974 (3) SCC 698) in which Vadivelu Thevar v. State of Madras (AIR 1957
SC 614) was also relied upon. AIR 1974 SC 276

10

. We may also observe that the ground that the witness being a close relative and
consequently being a partisan witness, should not be relied upon, has no substance. This
theory was repelled by this Court as early as in Dalip Singh's case (supra) in which
surprise was expressed over the impression which prevailed in the minds of the Members
of the Bar that relatives were not independent witnesses. Speaking through Vivian Bose,
J. it was observed: AIR 1953 SC 364, (Para 25)

"We are unable to agree with the learned Judges of the High Court that the testimony of
the two eye-witnesses requires corroboration. If the foundation for such an observation is
based on the fact that the witnesses are women and that the fate of seven men hangs on
their testimony, we know of no
@page-SC464
such rule. If it is grounded on the reason that they are closely related to the deceased we
are unable to concur. This is a fallacy common to many criminal cases and one which
another Bench of this Court endeavoured to dispel in 'Rameshwar v. State of Rajasthan'
(AIR 1952 SC 54 at p. 59). We find, however, that it unfortunately still persists, if not in
the judgments of the Courts, at any rate in the arguments of counsel."
11. Again in Masalti and Ors. v. State of U.P. (AIR 1965 SC 202) this Court observed
(pp. 209-210 para 14):
"But it would, we think, be unreasonable to contend that evidence given by witnesses
should be discarded only on the ground that it is evidence of partisan or interested
witnesses.......The mechanical rejection of such evidence on the sole ground that it is
partisan would invariably lead to failure of justice. No hard and fast rule can be laid
down as to how much evidence should be appreciated. Judicial approach has to be
cautious in dealing with such evidence; but the plea that such evidence should be rejected
because it is partisan cannot be accepted as correct."
12

. To the same effect is the decision in State of Punjab v. Jagir Singh (AIR 1973 SC 2407),
Lehna v. State of Haryana (2002 (3) SCC 76) and Gangadhar Behera and Ors. v. State of
Orissa (2002 (8) SCC 381). 2002 AIR SCW 4271

13

. The above position was highlighted in Babulal Bhagwan Khandare and Anr. v. State of
Maharashtra [2005(10) SCC 404] and in Salim Saheb v. State of M.P. (2007(1) SCC
699). 2004 AIR SCW 7376

14. It appears from the evidence of the witnesses that the relationship between the
appellant and the deceased was strained and much before the assault was made, there was
exchange of hot words between the accused and the deceased and they were quarrelling
with each other.
15. For bringing in operation of Exception 4 to Section 300, IPC, it has to be established
that the act was committed without premeditation, in a sudden fight in the heat of passion
upon a sudden quarrel without the offender having taken undue advantage and not having
acted in a cruel or unusual manner.
16. The Fourth Exception to Section 300, IPC covers acts done in a sudden fight. The
said Exception deals with a case of prosecution not covered by the First Exception, after
which its place would have been more appropriate. The Exception is founded upon the
same principle, for, in both there is absence of premeditation. But, while in the case of
Exception 1 there is total deprivation of self-control, in case of Exception 4, there is only
that heat of passion which clouds men's sober reasons and urges them to deeds which
they would not otherwise do. There is provocation in Exception 4 as in Exception 1; but
the injury done is not the direct consequence of that provocation. In fact Exception 4
deals with cases in which notwithstanding that a blow may have been struck, or some
provocation given in the origin of the dispute or in whatever way the quarrel may have
originated, yet the subsequent conduct of both parties puts them in respect of guilt upon
equal footing. A "sudden fight" implies mutual provocation and blows on each side. The
homicide committed is then clearly not traceable to unilateral provocation, nor in such
cases could the whole blame be placed on one side. For, if it were so, the Exception more
appropriately applicable would be Exception 1. There is no previous deliberation or
determination to fight. A fight suddenly takes place, for which both parties are more or
less to be blamed. It may be that one of them starts it, but if the other had not aggravated
it by his own conduct it would not have taken the serious turn it did. There is then mutual
provocation and aggravation, and it is difficult to apportion the share of blame which
attaches to each fighter. The help of Exception 4 can be invoked if death is caused. (a)
without premeditation; (b) in a sudden fight; (c) without the offender having taken undue
advantage or acted in a cruel or unusual manner; and (d) the fight must have been with
the person killed. To bring a case within Exception 4 all the ingredients mentioned in it
must be found. It is to be noted that the "fight" occurring in Exception 4 to Section 300,
IPC is not defined in IPC. It takes two to make a fight. Heat of passion requires that there
must be no time for the passions to cool down and in this case, the parties have worked
themselves into a fury on account of the verbal altercation in the beginning. A fight is a
combat between two or more persons whether with or without weapons. It is not possible
to enunciate any general rule as to what shall be deemed to be a sudden quarrel. It is a
question of fact
@page-SC465
and whether a quarrel is sudden or not must necessarily depend upon the proved facts of
each case. For the application of Exception 4, it is not sufficient to show that there was a
sudden quarrel and there was no premeditation. It must further be shown that the offender
has not taken undue advantage or acted in cruel or unusual manner. The expression
"undue advantage" as used in the provision means "unfair advantage".
17

. The above position is highlighted in Sandhya Jadhav v. State of Maharashtra, (2006) 4


SCC 653. 2006 AIR SCW 1678

18. Considering the factual background we are of the view that the appropriate conviction
would be in terms of Section 304, Part I, IPC, custodial sentence of ten years would meet
the ends of justice. The appeal is allowed to the aforesaid extent.
Order accordingly.
AIR 2008 SUPREME COURT 465 "Larsen and Toubro Ltd. v. Fertilizer and Chemicals
Travancore Ltd."
(From : Kerala)*
Coram : 2 Dr. A. PASAYAT AND LOKESHWAR SINGH PANTA, JJ.
Civil Appeal No. 5159 with 5160 of 2007 (arising out of SLP (C) Nos. 4014 with 4015 of
2006), D/- 12 -11 -2007.
Larsen and Toubro Ltd. v. Fertilizer and Chemicals Travancore Ltd.
Arbitration and Conciliation Act (26 of 1996), S.11 - ARBITRATION AND
CONCILIATION - APPOINTMENT - AMENDMENT - Appointment of Arbitrator -
Dispute as to withholding of certain amount in breach of conditions of purchase order -
Purchase order contained Special Conditions, Standard Terms and Conditions - Articles
25 and 26 of Standard Conditions contained provision for appointment of named
Arbitrator - Amendments incorporated to Articles 25 and 26 by Special Conditions only
provided that provisions of relevant Arbitration Act will be applicable - And that venue of
arbitration shall be Cochin - It cannot be said Special Conditions superseded Standard
Conditions - Apprehension that named Arbitrator may not act fairly is without any
foundation.
AIR 1988 SC 1099, Relied on. (Paras 8, 11, 12)
Cases Referred : Chronological Paras
AIR 1988 SC 1099 (Rel. on.) 10
AIR 1988 SC 2232 (Rel. on) 9
V. A. Mohta, Sr. Advocate, Nilkanta Nayak, Ms. Shweta Bharti, Ms. Aditi Mohan, Ms.
Neelam and Ms. Niranjana Singh, for Appellant; C. N. Sree Kumar, for the Respondent.
* A. R. No. 29 of 1999, D/- 18-12-1999 (Ker)
Judgement
Dr. ARIJIT PASAYAT, J. :-Leave granted.
2. Challenge in these appeals is to the order passed by the learned Single Judge of the
Kerala High Court disposing of arbitration request filed before him.
3. Background facts in a nutshell are as follows:-
Appellant and the respondent entered into a contract vide purchase order No. 3020/02-
2701/016/1018, dated 7.1.1995. Alleging that in breach of the terms and conditions of the
purchase order certain amounts were withheld, the appellant invoked the arbitration
agreement purportedly in terms of new Article 26 of the Special Conditions and
suggested three names for appointment of an independent sole arbitrator and called upon
the respondent to name one out of the three names. The respondent took the stand that it
is only the Managing Director of the respondent who can be appointed as a named
arbitrator as per Article 26 of the Standard Conditions and refused to appoint a sole
independent arbitrator. The High Court of Kerala was moved seeking appointment of an
arbitrator by Arbitration Request 29/99. Learned Single Judge declined the arbitration
request on the ground that terms and conditions of the purchase order provides for
arbitration by the Chairman and Managing Director of the respondent. A writ petition was
filed under Article 226 of the Constitution of India, 1950 (in short the 'Constitution').
During pendency of the said writ petition this Court in CA Nos. 3777, 4168 and 4169 of
2003 held that the order passed under Section 11 of the Arbitration and Conciliation Act,
1996 (in short the 'Act') is a judicial order and writ petition challenging the said order
under Article 226 of the Constitution is not maintainable. Therefore, this appeal has been
filed.
4. In support of the appeals, learned counsel for the appellant submitted that the provision
contained in Article 26 of the Standard Conditions is not the actual provision for
arbitration. The same is contained in the Special Terms and Conditions attached to the
purchase order and the work order
@page-SC466
respectively. The purchase order and the work order contained Special Conditions,
Standard Terms and Conditions. By Article 16 of the Special Terms and Conditions of
the purchase, there is amendment to Article 25 of the Standard Conditions. There is
similar amendment to Article 26 of the standard terms so far as it related to
commissioning. The provisions contained in the Standard Conditions in both the cases, it
was submitted by learned counsel for the appellant, is not actual provision for
arbitration.The general condition stated that all disputes and differences are required to be
referred to the Chairman and Managing Director of the respondent-company for his
decision and it will be binding on the parties. It was further contended that the provisions
contained in the Special Conditions by themselves do not have any provision for
arbitration. It does not have any clause that disputes and differences shall be settled by
arbitration. In both the cases, the Special Conditions specifically state that it is by way of
amendment of general condition only and not in supersession of that provision. The
Chairman and the Managing Director of the respondent-company cannot be treated as
independent person to be appointed as arbitrator. This was essentially the stand which
did not find acceptance. It is submitted by learned counsel for the appellant that certain
changes were suggested by the respondent.
5. Learned counsel for the respondent on the other hand submitted that the High Court
view is unexceptionable.
6. At this juncture it would be necessary to take note of the few conditions :
Clause 16 of the Work Order reads as follows:-
"16. Work Order Conditions:
The order shall be governed by the above conditions as well as by the conditions
stipulated in Attachment I, II and III of this Work Order, except the following:
The order shall be governed by the present Special Conditions of work (W.O. Attachment
III) as well as by the conditions stipulated in Attachment I, and II of this Work Order,
except the following:
Spec. No.3020/CS/04: Standard Terms and Conditions of Erection and Commissioning.
Art. 4.0.0 Taxes, Duties and Levies (Comment)
Taxes shall be as per Article 4.0.0. However, at present conditions; tax on this Work
Order is not applicable.
Art. 13.0.0 Termination (Comment)
FACT can terminate the Work Order without giving any reason provided that reasonable
cost for termination and actual out-of-pocket expenses will be reimbursed.
Art. 15.0.0 Changes (Amendment)
FACT shall issue amendment orders which provide for changes in the scope of work
required by FACT under the Work Order, and for equitable adjustment in the price and
delivery/completion time, if any, hereunder.
Art. 21.0.0 Tests on Completion and Taking Over (New Article Added).
Art. 21.5.0 (New Article)
The Primary Reformer Package under the scope of this Work Order shall be deemed to be
taken over by FACT immediately after satisfactory pre-commissioning is over within 10
days of Contractor's notice to Owner for commencement of commissioning after pre-
commissioning, whichever is earlier. In case taking over is delayed due to no fault of
Contractor, after the notice given by Contractor in this regard about the Completion, the
entire Primary Reformer Package is deemed to be taken over by FACT.
Art. 24.0.0 Indemnification
Secondary liability such as indemnification for loss caused by stoppage of plant of like
will be excluded from Contractor's liabilities under the Work Order.
Art. 26.0.0 Applicable Law and Settlement of Disputes (Amendment)
The provisions of the Indian Arbitration Act, 1940 and the rules thereunder, any statutory,
modifications there for the time being in force will be applied.
The venue for the arbitration shall be Cochin, and the language of the proceedings shall
be the English language.
During the arbitration proceedings, both parties shall continue to discharge their
obligations under the Work Order."
7. There was addition and not substitution of condition. Without amendment there was
arbitration clause and if there was no amendment the only substitution, then that there
was no arbitration clause. In the arbitration request in the statement of facts it
@page-SC467
has been clearly stated that Article 26 of the Standard Terms and Conditions of purchase
form part of the Work Order. The same read as follows:
"Article 26: Work Order shall be subject to and shall in all respects be governed by Indian
law. Any dispute or difference connected with or arising out of WORK ORDER which
cannot be settled by mutual agreement of the parties shall be referred to the Chairman
and Managing Director of FACT, and his decision will be binding on the parties. Any
legal proceeding relating to this WORK ORDER shall be limited to Courts of law under
the jurisdiction of the Kerala High Court at Ernakulam District, Kerala State, India."
8. The stand of the learned counsel for the appellant that the Special Conditions of the
work order superseded the Standard Terms and Conditions, is not correct. The mere fact
that the arbitrator was named does not render the arbitration proceedings invalid.
9

. In Secretary to Government, Transport Deptt., Madras v. Munuswamy Mudliar and Anr.


(1988 Suppl SCC 651) it was noted as follow: AIR 1988 SC 2232

"7. Pursuant to this the Superintending Engineer of that Circle, at the relevant time, was
previously appointed as arbitrator. There was succession to that office by another
incumbent and the succeeding Superintending Engineer wanted to continue the
arbitration proceedings but before that an application was made under Section 5 of the
Arbitration Act, 1940 (hereinafter called 'the Act') for removal of the arbitrator, before the
learned Judge of the City Civil Court, Madras."
10. Again in paras 11 to 13 it was noted as follows:
"11. This is a case of removal of a named arbitrator under Section 5 of the Act which
gives jurisdiction to the court to revoke the authority of the arbitrator. When the parties
entered into the contract, the parties knew the terms of the contract including arbitration
clause. The parties knew the scheme and the fact that the Chief Engineer is superior and
the Superintending Engineer is subordinate to the Chief Engineer of the particular Circle.
In spite of that the parties agreed and entered into arbitration and indeed submitted to the
jurisdiction of the Superintending Engineer at that time to begin with, who, however,
could not complete the arbitration because he was transferred and succeeded by a
successor. In those circumstances on the facts stated no bias can reasonably be
apprehended and made a ground for removal of a named arbitrator. In our opinion this
cannot be, at all, a good or valid legal ground. Unless there is allegation against the
named arbitrator either against his honesty or capacity or mala fide or interest in the
subject-matter or reasonable apprehension of the bias, a named and agreed arbitrator
cannot and shou1d not be removed in exercise of a discretion vested in the Court under
Section 5 of the Act.
12. Reasonable apprehension of bias in the mind of a reasonable man can be a ground for
removal of the arbitrator. A predisposition to decide for or against one party, without
proper regard to the true merits of the dispute is bias. There must be reasonable
apprehension of that predisposition. The reasonable apprehension must be based on
cogent materials. See the observations of Mustill and Boyd, Commercial Arbitration,
1982 edn., page 214. Halsbury's Laws of England, 4th edn., Volume 2, para 551, page
282 describe that the test for bias is whether a reasonable intelligent man, fully apprised
of all the circumstances, would feel a serious apprehension of bias.

13. This Court in International Airports Authority of India v. K. D. Bali (1988 (2) SCC
360) held that there must be reasonable evidence to satisfy that there was a real likelihood
of bias. Vague suspicions of whimsical, capricious and unreasonable people should not be
made the standard to regulate normal human conduct. In this country in numerous
contracts with the Government, clauses requiring the Superintending Engineer or some
official of the Government to be the arbitrator are there. It cannot be said that the
Superintending Engineer, as such cannot be entrusted with the work of arbitration and
that an apprehension, simpliciter in the mind of the contractor without any tangible
ground, would be a justification for removal. No other ground for the alleged
apprehension was indicated in the pleadings before the learned Judge or the decision of
the learned Judge. There was, in our opinion, no ground for removal of the arbitrator.
Mere imagination of a ground cannot be an excuse for apprehending bias AIR 1988 SC
1099

@page-SC468
in the mind of the chosen arbitrator."
11. The apprehension that named arbitrator may not act fairly is without any foundation.
The High Court has rightly held that by Article 16 of the Special Terms and Conditions of
purchase there was an amendment to Article 25 which reads as follows:-
"The provisions of the Indian Arbitration Act, 1940, and the rules thereunder, any
statutory modifications thereof of the time being in force will be applied. The venue of
the arbitration shall be Cochin, and the language of the proceedings shall be the English
Language. During the arbitration proceedings, both parties shall continue to discharge
their obligations under the Purchase Order."
12. Similar was the amendment to Article 26 of the Standard Terms and Conditions for
erection and commissioning in Article 16 of the Special Conditions of work attached to
the Work Order. The Special Conditions themselves show that Articles 25 and 26
contained provisions for arbitration. The amendments incorporated by the Special
Conditions only provide that the provisions of the relevant Arbitration Act and the Rules
made thereunder and any statutory modifications thereof for the time being in force will
be applicable and the venue of arbitration and language of the proceedings.
13. The appeals are sans merit, deserve dismissal, which we direct.
Appeals dismissed.
AIR 2008 SUPREME COURT 468 "Jaswant Talkies, M/s. v. Commercial Taxes Officer,
Bhilwara"
(From : Rajasthan)*
Coram : 2 Dr. A. PASAYAT AND D. K. JAIN, JJ.
Civil Appeal No. 5161 of 2007 (arising out of SLP (C) No. 7053 of 2006), D/- 12 -11
-2007.
M/s. Jaswant Talkies v. Commercial Taxes Officer, Bhilwara.
Rajasthan Sales Tax Act (22 of 1995), S.65 - Rajasthan Entertainment and Advertisement
Tax Act (24 of 1957), S.10 (as amended in 1998) - SALES TAX - ENTERTAINMENT
TAX - AMENDMENT - Penalty - Levy of - Ticketless viewers found in Cinema Hall of
assessee much before amendment - Quantum of penalty provided in S. 10 is Rs. 500/- per
contravention - To make provision stringent, shift was made by Amendment Act to per
viewer - Amendment is neither clarificatory nor retrospective - Levy of penalty @ of Rs.
500 per viewer not proper.
S. B. Civil (Sales Tax) Revn. No. 173 of 2000, D/- 3-2-2006 (Raj), Reversed. (Paras
9, 10)
Cases Referred : Chronological Paras
2004 (1) STT 237 (Raj) 4
2003 (1) WLC (Raj) 306 4
Dr. Manish Singhvi, P. V. Yogeswaran, for Appellant; Aruneshwar Gupta, AAG, Naveen
Kumar Singh, Mukul Sood and Shashwat Gupta, for Respondent.
* S. B. Civil (Sales Tax) Revn. No. 173 of 2000, D/- 3-3-2006 (Raj), (at Jodhpur)
Judgement
1. Dr. ARIJIT PASAYAT, J. :-Leave granted.
2. Challenge in this appeal is to the order passed by a learned Single Judge of the
Rajasthan High Court at Jodhpur, allowing the revision filed by the respondent. The said
revision petition was filed under Section 86 of the Rajasthan Sales Tax Act, 1994 (in
short the 'Act').
3. Factual position which is almost undisputed is as follows:
The appellant is an exhibitor of cinematograph films. On 3.2.1996 cinema hall of the
appellant was inspected by the Commercial Tax Inspectors. At that time a movie
"Alladdin" was being shown in the morning show. At the time of inspection, 878 viewers
were found watching the movie without tickets. It was found that the daily collection
register maintained by the appellant was not properly maintained. The inspectors put their
signatures after drawing a line in the register so that no entry can be made thereafter.
Alleging that the appellant admitted 878 viewers without tickets, a show cause notice was
issued under Section 10 of the Rajasthan Entertainment and Advertisement Tax Act, 1957
(in short the 'Entertainment Act') prima facie being of the view that offence under
Sections 6(1) and 6(2) of the Entertainment Act has been committed. The appellant
submitted its reply and stated that girl students of a school had gone to watch the movie
which was meant for children and in any event there was no scope for imposition of
penalty of Rs.500/- in respect of each viewer. The Commercial Tax Officer, Bhilwara was
of the view that penalty at the rate of Rs. 500/- per viewer was to be imposed and in
addition penalty of Rs. 500/- under Section 10(3)(b) was also to be
@page-SC469
imposed. The officer was of the view that mere fact that children of Mahila Aashram
were being shown the picture and the tickets were to be handed over to the school
authorities after counting the number of children was not relevant.
The inspection was done at 10.00 a.m. when the school children had just entered the Hall
and, therefore, even before the show started the inspection was done. The order of
assessment was challenged before the Commissioner (Appeals). The said authority found
that all the tickets were with the Manager of the Cinema, but since each viewer did not
possess a ticket there was contravention. Therefore, penalty under Section 10(3)(a) of the
Entertainment Act was upheld while setting aside the penalty imposed under Section
10(3)(b)(iii).
The appellant preferred an appeal against the said order before the Rajasthan Taxation
Board, Ajmer (in short the 'Taxation Board'). Before the Taxation Board, it was contended
that the total cost of the tickets was Rs. 3006/- and penalty of Rs. 4,39,000/- at the rate of
Rs. 500/- per viewer was unconscionable. The Taxation Board found that the penalty that
was imposed was not imposable at the rate of Rs. 500/- per viewer and the maximum
penalty imposable was Rs. 500/-.
Revenue filed a revision petition as noted above and by the impugned order penalty
imposed by the Assessing Officer was restored.
4. In support of the appeal, learned counsel for the appellant submitted that the High
Court has relied on a decision of its own Court in Maharana Talkies, Bhilwara v. State of
Rajasthan and Ors. (2004 (1) STT 237 (Raj.HC)) ignoring the view expressed in State of
Rajasthan and Ors. v. RTT and Ors. (2003 WLC (Raj.)) which held that no penalty can be
levied per person. The High Court also, it is pointed out, held in that case that the
subsequent amendment was not clarificatory.
5. Learned counsel for the State on the other hand supported the order.
6. The legislative history of the statutory provision needs to be noted.
Section 10 of the Rajasthan Entertainment and Advertisement Act, 1957 existing pre-
1982
10. Offence and penalties.- (1) Notwithstanding anything contained in any law for the
time being in force, a ticket for admission to an entertainment shall not be resold for
profit by the holder thereof.
(2) Whoever re-sells any ticket for admission in contravention to the provisions of sub-
section (1) shall, on conviction before a Magistrate, be liable to pay fine which may
extend to two hundred rupees.
(3)(a) The proprietor of any entertainment or any person employed by him in any place of
entertainment, who admits any person to any place of entertainment in contravention of
the provisions of sub-section (1) or sub-section (2) of Section 6, or
(b) The proprietor of an entertainment who-
(i) fails to pay the tax due from him under this Act within the prescribed time, or
(ii) fraudulently evades the payment of tax due from him under this Act,
(iii) contravenes any of the provisions of this Act or the rules framed thereunder for
which no other penalty has been provided under this Act.
shall be liable to pay by way of penalty, in addition to the amount of tax payable by him a
sum not exceeding Rs. 500/-.
Section 10 of the Rajasthan Entertainment and Advertisement Act, 1957 from 1982
10. Offence and penalties.- (1) Notwithstanding anything contained in any law for the
time being in force, a ticket for admission to an entertainment shall not be resold for
profit by the holder thereof.
(2) Whoever re-sells any ticket for admission in contravention to the provisions of sub-
section (1) shall, on conviction before a Magistrate, be liable to pay fine which may
extend to two hundred rupees.
(3)(a) The proprietor of an entertainment or any person employed by him in any place of
entertainment, who admits any person to any place of entertainment in contravention of
the provisions of sub-section (1) or sub-section (2) of Section 6, or
(b) The proprietor of an entertainment who-
(i) fails to pay the tax due from him under this Act within the prescribed time, or
(ii) fraudulently evades the payment of tax due from him under this Act, or
(iii) contravenes any of the provisions of
@page-SC470
this Act or the rules framed thereunder for which no other penalty has been provided
under this Act,
shall be liable to pay by way of penalty-
(i) in respect of cases referred to in clause (a) and sub-clauses (i) and (iii) of clause (b) in
addition to the amount of tax payable by him, a sum not exceeding Rs. 500/- and
(ii) in respect of cases referred to in sub-clause (ii) of clause (b) in addition to the amount
of tax payable by him a sum not exceeding rupees five hundred or double the amount of
tax evaded whichever is higher.
(4) The prescribed authority not below the rank of an Assistant Commercial Taxes Officer
may, after affording a reasonable opportunity of being heard to the person affected,
impose the penalty mentioned in sub-section (3).
(5) The person affected may, within one month of the communication of the order
directing payment of any sum by way of penalty under sub-section (3) appeal to the
prescribed authority.
Amended Section 10 of the Rajasthan Entertainment and Advertisement Act, 1957 with
effect from 31.7.1998
10. Offence and penalties- (1) Notwithstanding anything contained in any law for the
time being in force a ticket for admission to an entertainment shall not be resold for profit
by the holder thereof.
(2) Whoever re-sells any ticket for admission in contravention to the provisions of sub-
section (1) shall, on conviction before a Magistrate, be liable to pay fine which may
extend to two hundred rupees.
(3)(a) The proprietor of an entertainment or any person employed by him in any place of
entertainment, who admits any person to any place of entertainment in contravention of
the provisions of sub-section (1) or sub-section (2) of Section 6, or
(b) the proprietor of an entertainment who-
(i) fails to pay the tax due from him under this Act within the prescribed time, or
(ii) fraudulently evades the payment of tax due from him under this Act, or
(iii) contravenes any of the provisions of this Act or the rules framed thereunder for
which no other penalty has been provided under this Act shall be liable to pay by way of
penalty-
(i) in respect of cases referred to in clause (a) and sub-clause (1) of clause (b) regarding
entertainment tax, in addition to the amount of tax payable by him, a sum not exceeding
Rs.100/- per person;
(ii) in respect of cases referred to in sub-clause (1) of clause (b) regarding advertisement
tax and in respect of cases referred to in sub-clause (iii) of clause (b) in addition to the
amount of tax payable by him, a sum not exceeding Rs.500; and
(iii)in respect of cases referred to in sub-clause (ii) of clause (b) in addition to the amount
of tax payable by him a sum not exceeding Rupees five hundred or double the amount of
tax evaded whichever is higher.
(4) The prescribed authority not below the rank of an Assistant Commercial Taxes Officer
may, after affording a reasonable opportunity of being heard to the person affected,
impose the penalty mentioned in sub-section (3).
7. It is to be noted that Section 10 of the Entertainment Act has been amended w.e.f. 31st
July, 1998. The amended provision permits imposition of penalty per person. Section
10(3)(b) contemplates two types of penalties. The first relates to cases covered by clause
(a) and sub-clauses (i) and (ii) of Clause (b). The second relates to sub-clause (ii) of
Clause (b) of sub-section 3 of Section 10. The case at hand relates to clause (a) of sub-
section (3) of Section 10. If the position was clear as contended by learned counsel for
the State that the penalty is to be at the rate of Rs. 500/- per person, there was no reason
for the amendment which specifically provided for imposition of penalty per person.
8. The Statement of Objects and Reasons for the amendment also throw considerable
light on this position. The same reads as follows:
B. Amendment in the Rajasthan Entertainment and Advertisements Tax Act, 1957
Certain amendments in the Rajasthan Entertainment and Advertisements Tax Act, 1957
have been considered necessary and were under consideration for quite some time past.
Owing to repeal of the Rajasthan Sales Tax Act, 1954 and coming into force of new Act
of 1994, definition of the term 'Sales Tax Act', and due to proposed induction of certain
new provisions in the 'Act of 1957', definitions of the terms 'appellate authority', 'Tax
Board' and 'Tribunal' are proposed to be inserted vide clause 9 of the Bill.
@page-SC471
To provide for limitation for assessments, new section 5BB, to ensure effective recovery
of the outstanding entertainment tax by providing special mode of recovery, substitution
of section 9, to enable payment of outstanding dues in installments, new section 5BB, to
enable the Commissioner to reduce or waive interest and penalty under the Act in case of
genuine hardship, new section 9C are proposed to be inserted, substituted or inserted, as
the case may be, vide clauses 12, 13 or 14 of the Bill respectively.
Moreover to make the penalty provision more practicable, section 10 (3) (b) is proposed
to be substituted vide clause 15 of the Bill.
Besides above, to provide for statutory remedy of first appeal, new section 13-A, to
provide for appeal against order of appellate authority, new section 13-B, to provide for
revision to tribunal, new section 13-C provide for powers of revision to Commissioner in
case of an order passed by the prescribed authority being erroneous or prejudicial to the
interest of State revenue, new section 13D, and to provide for rectification of mistakes
apparent from the record, new section 13-E; are proposed to be inserted vide clause 19 of
the Bill.
Some of the consequential or minor amendments are also proposed in various sections of
the Act of 1957 vide clauses 10, 11, 17, 20 and 21 of the Bill.
Extract of the Rajasthan Finance Bill, 1998, Relevant portion:
15. Amendment of Section l0, Rajasthan Act No. 24 of 1956:
In section 10 of the principal Act,-
(a) for existing clause (b) of sub-section (3), the following clause shall be substituted,
namely:
"(b) the proprietor of any entertainment who-
(i) fails to pay the tax due from him under this Act within the prescribed time, or
(ii) fraudulently evades the payment of tax due from him under this Act, or
(iii) contravenes any of the provisions of this Act or the rules framed thereunder, for
which no other penalty has been provided under this Act, shall be liable to pay by way of
penalty:
(i) in respect of cases referred to in clause (a) and sub-clause (i) of clause (b) regarding
entertainment tax, in addition to the amount of tax payable by him, a sum not exceeding
Rs.100/- per person.
(ii) in respect of cases referred to in sub-clause (i) of clause (b) regarding advertisement
tax and in respect of cases referred to in sub-clause (iii) of clause (b), in addition to the
amount of tax payable by him, a sum not exceeding Rs. 500; and
(iii) in respect of cases referred to in sub-clause (ii) of clause (b) in addition to the
amount of tax payable by him a sum not exceeding Rupees five hundred or double the
amount of tax evaded whichever is higher."
(b) sub-section (5) shall be deleted.
9. It is also irrational that as against a tax liability of Rs. 3006/-, penalty of Rs. 4,39,000/-
was to be imposed. Though in all cases quantum would not be a relevant factor, but on
analyzing the scheme of the Statute it is clear that the stress is on the contravention. The
contravention essentially is of admitting persons without valid ticket and at the relevant
point of time, had no nexus with the number of persons. To make the provision stringent
the shift has been made to per person. But the amendment has no retrospective effect and
as noted above is not clarificatory.
10. As noted above, the amendment does not appear to be clarificatory in nature as
contended by learned counsel for the respondent-State. The view of the High Court,
therefore, is not correct. The conclusion of the Taxation Board was the correct view.
11. The appeal deserves to be allowed which we direct. There shall be no order as to
costs.
Appeal allowed.
AIR 2008 SUPREME COURT 471 "Mohan Kumar Rayana v. Komal Mohan Rayana"
(From : Bombay)*
Coram : 2 C. K. THAKKER AND ALTAMAS KABIR, JJ.
Civil Appeal Nos. 5088-5097 of 2007 (arising out of SLP (C) Nos. 15167-15176 of
2007), D/- 1 -11 -2007.
Mohan Kumar Rayana v. Komal Mohan Rayana. @page-SC472
Hindu Minority and Guardianship Act (32 of 1956), S.6 read with S.7 - MINORITY
AND GUARDIANSHIP - HIGH COURT - APPEAL - Custody of minor daughter -
Visitation rights of farther - Directions given by High Court pending appeal - Not
observed strictly by both parties - Order granting access to father to meet minor kept in
abeyance by High Court pending appeal till receipt of report of Psychiatrist - Appeal
against by father - Court after meeting parties as well as minor in chamber forusing view
that father should not be denied complete access to child pending disposal of appeals by
High Court - Further directions as to visitation time passed - Since welfare of minor is
involved, directions also given to dispose of matter expeditiously. (Paras 15, 16, 17)
Dr. A. M. Singhvi, Sr. Advocate, Sudhanshu Batra, S. Jayaram, Pramit Saxena, Amit
Yadav, Dr. Kevic Setalwad and S. V. Deshpande, for Appellant; Ms. Indu Malhotra, Sr.
Advocate, R. I. Lalwani, Ms. S. I. Jayakar (Lalwani), Ms. Sunieta Ozha, Nitin Ramesh
and Ms. Anitha Shenoy, for Respondent.
* From the interim Judgments and Orders dated 12-7-2007, 19-7-2007, 27-7-2007, 6-8-
2007 of the High Court of Bombay in F. C. A. Nos. 61 and 29, C. A. No. 81 with 39 with
169 in F. C. A. No. 29 of 2007.
Judgement
1. ALTAMAS KABIR, J. :-Leave granted.
2. Since both the parties to the special leave petitions are before us, Notice of the Appeals
is waived on behalf of the respondent, Komal Mohan Rayana.
3. The appeals arise out of circumstances wherein owing to disputes and differences
between a married couple, the child born of the wedlock has become the object of a tussle
for custody between the two parents.
4. The subject-matter of these appeals are four orders passed by the Bombay High Court
on 12th July, 2007, 19th July, 2007, 27th July, 2007 and 6th August, 2007 in two appeals
from a Petition No.D-65/2005 before the Family Court. In order to appreciate the
circumstances in which these orders came to be passed, it will be necessary to state a few
facts leading to the commencement of the proceedings before the Family Court.
5. Admittedly, the appellant herein, who is the husband of the respondent, married the
respondent on 2nd March, 2002. A daughter was born to them and she was named
Anisha. Initially there were no disputes as such between the parties but after the
daughter's birth, the atmosphere in the marital home began to change. We shall not go
into the causes as alleged by the respondent since such allegations are not relevant for our
purpose, but we can only observe that one of the reasons given by the respondent for the
changed circumstances was the change in behaviour of the appellant towards her, on
account of addiction to alcohol in the company of his friends.
6. In any event, there appears to have been some marital discord, which resulted in the
respondent leaving the matrimonial house in July 2004 with her minor daughter and
seeking shelter with her parents at Bandra. According to the respondent, during the said
period she continued to send Anisha to the Kinder Campus School at Chembur, the area
where the appellant was residing and permitted him on occasions to keep back Anisha at
his residence. The respondent has alleged that in October 2005, taking advantage of such
a situation, the appellant kept Anisha back with him and did not return her to the
respondent's custody. This compelled the respondent to meet her daughter in the school
campus, but since this arrangement did not also work out, in the last week of November
2005, she approached the Chembur police and with their help got back the custody of her
daughter. A series of allegations were thereafter made that on 30th November, 2005 the
appellant, with the help of some of his associates, forcibly removed Anisha from the
respondent's custody and made her completely inaccessible to the respondent. It is in such
compelling circumstances that she moved the Family Court seeking custody of her minor
daughter under Section 6 of the Hindu Minority and Guardianship Act, 1956 read with
Ss. 7 and 25 of the Guardians and Wards Act, 1890.
7. The appellant herein also filed a Custody Petition, being D-66 of 2005, and both the
applications were taken up for hearing together by the learned Family Court. By its
judgment dated 2nd February, 2007 the Family Court dismissed the appellant's
application for custody and allowed the application filed by the respdondent by passing
the following order :
ORDER
The Respondent/Mohan Kumar Rayana is directed to hand over custody of the minor
daughter Anisha to the petitioner/mother Komal Rayana immediately after completion of
her final terms of the current
@page-SC473
academic session 2006-2007.
The Respondent/father shall take all the steps to provide all facilities to the minor
daughter to enjoy her extra-curricular activities and studies.
After the child Anisha goes to the custody of the mother as ordered above, the
Respondent/father would be at liberty and privilege to avail her access every alternate
weekends, meet her at school at any time and share 50% of her school vacations, as per
mutual arrangement with the petitioner/mother.
The petitioner/mother should in consultation with the Respondent/father decide the
question of her further academic education and she should not move the child out of the
jurisdiction of the Court without its prior permission and of course after due intimation to
the Respondent/father.
The father/respondent shall meet all the expenses for the education, food and clothes etc.
of the minor daughter Anisha and the Petitioner/mother of her own accord may contribute
to the same for the child and she should not be prohibited by the respondent/father from
giving the child Anisha anything for her own comfort and pleasant living. This
arrangement for custody is made on the basis of the prior consideration for the welfare of
the minor Anisha and in the event of change of circumstances either of the parents shall
be at liberty and privilege to approach this Court for fresh direction on the basis of
changed circumstances.
The custody petition D-65/05 moved by the Respondent/father Mohan Kumar Rayana
stands dismissed with visitation and access rights as ordered above."
8. Aggrieved by the said Judgment and order of the Family Court, the appellant filed
Family Court Appeal No. 29/2007 before the Bombay High Court on 23.2.2007 and the
same was admitted on 7th March, 2007 and was said to have been per-emptorily fixed for
final hearing on 26th March, 2007. On 26th March, 2007 the respondent also filed an
appeal, being Family Court Appeal No.61/2007, challenging the operation of the
judgment of the Family Court dated 2.2.2007 granting access to the appellant to meet
Anisha. The said appeal was also admitted on 3rd May, 2007. On the same day, the
directions contained in the order of the Family Court dated 2.2.07 regarding access to the
appellant to meet Anisha, were modified by the High Court by directing that the minor
child would be available to the appellant as and when he was physically present in
Bombay at his house. It was also stipulated that whenever the appellant was not available
in Bombay the child should remain with the respondent. It was specifically mentioned
that the child should not be removed by the appellant out of Bombay for any reason
whatsoever, except in the circumstances mentioned in the order.
9. A Special Leave Petition was filed by the appellant against the order of the High Court
dated 3.5.07 and the same was disposed of on 18.6.07 with a direction upon the High
Court to hear the Family Court appeal expeditiously.
10. Certain circumstances intervened which prompted the Division Bench of the Bombay
High Court to modify its order dated 3.5.07 on 12.7.07 by reducing the access granted to
the appellant and limited such access only to the day time on the ensuing Saturday and
Sunday. The said order passed in the two above-mentioned appeals is one of the orders
forming the subject matter of the appeals before us.
11. Subsequently, after interviewing the parties and the minor child, the High Court
passed a further order on 19.7.07 directing the appellant and the respondent to visit a
psychiatrist with the child and to obtain a report from him. The access granted to the
appellant on Saturdays and Sundays from 9 A.M. to 9 P.M. was continued. The said order
passed in application No.81/2007 filed by the respondent herein in Family Court Appeal
No.61/2007, is one of the other orders which form the subject matter of the present
appeals before us.
12. A third order was passed by the Bombay High Court on 27.7.07 directing the
appellant and the respondent to seek appointment with a psychiatrist within a week, and
he was also directed to submit his report within 2 weeks after the parties were examined.
The interim arrangement made earlier was directed to continue. The said order is the third
order which is impugned in the present appeals. The fourth order impugned in these
appeals was passed on 6.8.07 in the pending Civil Application No.81/2007, whereby, in
view of the intervening circumstances, the High Court passed the following order.
@page-SC474
"IN THE HIGH COURT OF JUDICATURE AT BOMBAY CIVIL APPELLATE
JURISDICTION FAMILY COURT APPEAL NO.61 OF 2007 ALONGWITH
CIVIL APPLICATION NO.81 OF 2007 ALONGWITH
FAMILY COURT APPEAL NO.29 OF 2007
Mr. R.T. Lalwani, Advocate for the applicant/wife
Mr. Kevic Settalwad Advocate i/b D.H. Law and Associates for Respondent/husband
CORAM : J.N. PATEL AND A.S. SAYED, JJ
DATE : AUGUST 6, 2007 P.C. (Per J.N. Patel, J.):
Heard. We find from the conduct of the parties that the parties are repeatedly moving this
Court in the matter on one pretext or the other. It is highly impossible for the Court to
monitor each and everything. This being matrimonial matter relating to access of the
child, the Court has issued directions from time to time and it is expected that both the
parties shall comply with the directions of this Court and facilitate each other and
cooperate with each other in the matter. But it appears that the parties are trying to
interpret the order in the manner they want, without being concerned about the welfare of
the child, which is of paramount importance. This Court has suggested to the parties to
go for counselling and already a psychiatric of J.J. Hospital is appointed for the same.
Recent development is represented by the counsel for the parties shows that on the last
date of access there was some quarrel between the parties, which lead to hospitalisation
of the wife, for injuries suffered by her and she is presently admitted in Lilawati Hospital
and likely to be discharged today or tomorrow.
2. In our considered opinion the respondent/wife deserves an opportunity to place her
affidavit on record.
3. In view of the recent development as brought to our notice, we are left with no option,
but hold all our interim orders/relief to grant access to father, in abeyance till this Court
receives report of the psychiatrist. We make it clear that the parties, if fail to cooperate
with the Court in resolving the issue, this Court would remove the matter from its board.
It is not expected from the parties to resolve their domestic quarrel in the court and ask
the Court to adjudicate each and every issue, whether minor or major, relevant or
irrelevant. We hope that the parties would maintain some discipline in observing the
orders of the Court and cooperate.
4. Parties are at liberty to mention the matter only after they comply with the orders of
this Court and report of the psychiatrist is received. Thereafter this Court proposes to pass
the further orders. The matter stands adjourned for 4 weeks. We make it clear that on the
mean time we would not entertain any application for interim relief, or for permitting the
parties to meet the child, or to take matter on board, which has led this Court to hold all
orders passed earlier in abeyance.
(A.A.SAYED, J.) (J.N. PATEL, J.)
TRUE COPY"
13. By the aforesaid order all access to the appellant was kept in abeyance till the Court
received the report of the psychiatrist. The main grievance of the appellant is that by the
order of 6.8.07 he was completely denied any access to the minor child. He was also
aggrieved by the reduction of access time by the other orders as well.
14. Since these appeals have been preferred against the interim orders passed by the
Bombay High Court in the two pending Family Court Appeals, learned counsel for the
appellant, submitted that in these appeals the only grievance of the appellant was with
regard to denial of complete access to his child. He prayed that the visitation rights
which had been granted by the Family Court be restored during the pendency of the two
appeals in the Bombay High Court.
15. Since we are only called upon to decide the said issue, we are not required to go into
any other question relating to the appeals pending before the Bombay High Court. We
have met the appellant, the respondent and also the minor child, Anisha, separately, in
chamber, to ascertain what each had to say regarding the making of interim arrangements
to allow the appellant to have access to Anisha.
16. After having looked through the materials on record and after considering the views
of the parties and the minor girl, we are of the view that the appellant should not be
denied complete access to his minor child, even if there has been a default in complying
with the directions of the High Court and that pending the disposal of the appeals he
@page-SC475
should be allowed to have access to his minor child, at least to some extent.
17. We, accordingly, dispose of these appeals with the following directions :-
i) Since the welfare of a minor child is involved, the High Court is requested to try and
dispose of the pending appeals as expeditiously as possible, but preferably within three
months from the date of communication of this order;
ii) The appellant/father of the minor, will be entitled to have access to Anisha on
weekends on Saturdays and Sundays and will be entitled, if the child is willing, to keep
her with him on Saturday night. For the said purpose, the appellant shall receive the child
from the respondent at 10.00 a.m. on Saturday from her residence at Bandra or from a
mutually agreed upon venue and shall return the child to the respondent on Sunday by
2.00 p.m. In the event Anisha is unwilling to stay with the appellant overnight, the
appellant will then make her over to the respondent on Saturday itself by 9.00 p.m.; in
that case, the appellant will be entitled to take Anisha out on Sunday also between 9.00
a.m. to 5.00 p.m.;
iii) Both the appellant as well as the respondent must co-operate with each other in
making the aforesaid arrangements work. The respondent shall not prevent the appellant
from having access to Anisha in the manner indicated above. Likewise, once Anisha is
handed over to the appellant he too must honour the aforesaid arrangements and not keep
Anisha with him beyond the time stipulated. In the event of either of the parties violating
the aforesaid arrangement, the other party would be at liberty to pray for appropriate
orders before the Bombay High Court in the pending appeals;
iv) The aforesaid arrangement is being made so that the appellant can have access to his
minor daughter and also to ensure that the childs education does not suffer in any way
during the week.
18. The appeals are, accordingly, disposed of with the aforesaid modifications of the
interim orders passed by the High Court and save as aforesaid, all the other interim
directions shall continue to remain operative.
19. Since, in terms of our earlier directions, the expenses of the respondent and Anisha
for coming from Bombay to Delhi and other litigation expenses is said to have been
deposited by the appellant with the Registry of this Court, the respondent shall be entitled
to withdraw the same. There shall be no further order as to costs in these appeals.
Order accordingly.
AIR 2008 SUPREME COURT 475 "Haryana Urban Development Authority v. Om Pal"
(From : Punjab and Haryana)*
Coram : 2 S. B. SINHA AND MARKANDEY KATJU, JJ.
Civil Appeal No. 1869 of 2007 (arising out of SLP (C) No. 23386 of 2004), D/- 10 -4
-2007.
Haryana Urban Development Authority v. Om Pal.
(A) Industrial Disputes Act (14 of 1947), S.25F, S.25B - INDUSTRIAL DISPUTE -
RETRENCHMENT - DAILY-WAGE WORKERS - Retrenchment - Reckoning
continuity of service - Period during which workman, daily wager, was working in one
establishment - Would not enure to his benefit when he was recruited separately in
another establishment - Moreso, when he was not transferred from one division to the
other.
In the instant case the workman, daily wager worked for a period of 145 days in Sub-
Division No. 2. He, however, subsequently worked in Sub-Division No. 3 for a period of
90 days. His services were terminated. It has not been denied or disputed that the two
Sub-Divisions constituted two different establishments. Only because there is one
Controlling Authority, the same by itself would not mean that the establishments were not
separate. The workman was not shown to be appointed in both the establishments by the
same authority. Thus, once two establishments are held to be separate and distinct having
different cadre strength of the workmen, if any, the period during which the workman
was working in one establishment would not enure to his benefit when he was recruited
separately in another establishment, particularly when he was not transferred from one
Sub-Division to the other. Therefore, the order directing the reinstatement of workman
with full back wages and continuity of service would be liable to be set aside.
2006 AIR SCW 5630, Ref. (Paras 5, 8)
@page-SC476
(B) Industrial Disputes Act (14 of 1947), S.11A - INDUSTRIAL DISPUTE -
TRIBUNALS - REINSTATEMENT - BACK WAGES - Powers of Tribunal - Workman
worked for short period as daily wager in 1994-95 - Award directing his reinstatement
with full back wages passed in year 2003 - Illegal. (Para 8)
Cases Referred : Chronological Paras
2006 AIR SCW 5630 (Ref) 6
Satinder Gulati, Kamaldeep Narang and Dr. Kailash Chand, for Appellant; Sanjay Bansal,
Reepak Kansal and G. K. Bansal, for Respondent.
* C. W. P. No. 5948 of 2004, D/- 8-4-2004 (PandH).
Judgement
S. B. SINHA, J. :-Leave granted.
2. Respondent herein was appointed as a daily-wager. From October, 1994 to February,
1995, he worked for a period of 145 days in Sub-Division No.2, Panipat. He, however,
worked in Sub-Division No.3 for a period of 90 days from March 1995 to July, 1995. His
services were terminated. An industrial dispute was raised questioning validity of the said
order of termination. The said industrial dispute was referred by the Appropriate
Government to the Industrial Tribunal-cum-Labour Court, Panipat, for its determination.
It was registered as Reference No.59 of 1999. By an award dated 28.2.2003, the
Industrial Court on the premise that the services rendered by the respondent in both the
Sub-Divisions should be counted for the purpose of Section 25F read with Section 25B of
the Industrial Disputes Act, 1947, directed his reinstatement with continuity of service
and full back-wages from the date of demand notice i.e. 14.9.1995. A writ petition filed
thereagainst by the appellant herein was dismissed. The appellant has, therefore, filed
this appeal by special leave.
3. The short question which arises for consideration by us in this appeal is as to whether
in the aforementioned fact situation, the Industrial Tribunal-cum-Labour Court was
justified in directing reinstatement of the respondent with full back-wages and continuity
of service. It has not been denied or disputed that the two Sub-Divisions constituted two
different establishments. Only because there is one Controlling Authority, the same by
itself would not mean that the establishments were not separate.
4. Respondent did not produce before the Industrial Tribunal-cum-Labour Court his
offers of appointment. If offers of appointment had been issued in his favour by the two
Sub-Divisions separately, the same ipso facto would lead to the conclusion that they were
separate and distinct. If his appointment was only on the basis of entry in the muster
roll(s), the designation of the authority who was authorised to appoint him as a daily-
wager would be the determinative factor. It is not the case of the respondent that he was
appointed in both the establishments by the same authority.
5. The Industrial Tribunal-cum-Labour Court unfortunately did not go into the said
question at all. If both the establishments are treated to be one establishment, for the
purpose of reckoning continuity of service within the meaning of Section 25B of the Act,
as was held by the Tribunal, a person working at different point of time in different
establishments of the statutory authority, would be entitled to claim reinstatement on the
basis thereof. However, in that event, one establishment even may not know that the
workman had worked in another establishment. In absence of such a knowledge, the
authority retrenching the workman concerned would not be able to comply with the
statutory provisions contained in Section 25F of the Act. Thus, once two establishments
are held to be separate and distinct having different cadre strength of the workmen, if
any, we are of the opinion that the period during which the workman was working in one
establishment would not enure to his benefit when he was recruited separately in another
establishment, particularly when he was not transferred from one Sub-Division to the
other. In this case he was appointed merely on daily wages.
6

. In Union of India and Others v. Jummasha Diwan [(2006) 8 SCC 544], this Court
opined : 2006 AIR SCW 5630, Para 8

".........There are several establishments of the Railway Administration. If a workman


voluntarily gives up his job in one of the establishments and joins another, the same
would not amount to his being in continuous service. When a casual employee is
employed in different establishments, may be under the same employer, e.g., the Railway
Administration of India as a whole, having different administrative set-ups, different
requirements and different projects, the concept of continuous service cannot be applied
and it cannot be said that even in such a situation he would be entitled to a higher status
being in continuous service.
@page-SC477
It is not in dispute that the establishment of Appellant 3 herein had started a project. His
recruitment in the said establishment would, therefore, constitute a fresh employment. In
a case of this nature, the Respondent would not be entitled to his seniority. If the project
came to a close, the requirements of Section 25-N of the Act were not required to be
complied with."
7. Moreover, it is now also well-settled that despite a wide discretionary power conferred
upon the Industrial Courts under Section 11A of the 1947 Act, the relief of reinstatement
with full back-wages should not be granted automatically only because it would be lawful
to do so. Grant of relief would depend on the fact situation obtaining in each case. It will
depend upon several factors; one of which would be as to whether the recruitment was
effected in terms of the statutory provisions operating in the field, if any.
8. Respondent worked for a very short period. He only worked, as noticed hereinbefore,
in 1994-95. The Industrial Tribunal-cum-Labour Court, therefore, in our opinion
committed an illegality, while passing an award in the year 2003, directing the
reinstatement of the respondent with full back-wages. Although we are of the opinion that
the respondent was not entitled to any relief, whatsoever, we direct the appellant to pay
him a sum of Rs.25,000/-.
9. This appeal is allowed to the aforementioned extent. However, in the facts and
circumstances of this case, there shall be no order as to costs.
Order accordingly.
AIR 2008 SUPREME COURT 477 "D. S. Lahoriya v. State of Rajasthan"
(From : Rajasthan)
Coram : 2 C. K. THAKKER AND P. K. BALASUBRAMANYAN, JJ.
Criminal Appeal No. 728 of 2007 (arising out of SLP (Cri.) No. 4570 of 2006), D/- 14 -5
-2007.
D. S. Lahoriya @ Rajeev Sudan @ Vinay Kr. v. State of Rajasthan.
(A) Extradition Act (34 of 1962), S.3, S.12, S.21 - Penal Code (45 of 1860), S.468 -
Explosive Substances Act (6 of 1908), S.4 - EXTRADITION - FORGERY -
EXPLOSIVE - SUPREME COURT - Extradition - Appellant was allegedly involved in
conspiracy of kidnapping to exert pressure on Government to release an alleged
Khalistani terrorist - Plea that appellant could not have been prosecuted in Indian Court
considering Extradition Treaty between America and Great Britain and extradition order
passed by American Court - Supreme Court had dealt with all contentions relating to
extradition in cognate matter - And allowed extradition of appellant for trial under
Explosive Substances Act - Prosecution, conviction and sentence of appellant under said
Act is, therefore, proper. (Paras 13, 14)
(B) Constitution of India, Art.134 - Penal Code (45 of 1860), S.364A, S.420 - Arms Act
(54 of 1959), S.4 - APPEAL - KIDNAPPING - CHEATING - FIRE ARMS - Infructuous
appeal - Kidnapping to press demand for release of terrorist - Appellant convicted under
S. 364A, I. P. C. - Order to undergo life imprisonment upheld by Supreme Court -
Appellant challenged his conviction and sentence whereby he was ordered to undergo
imprisonment for seven years - He had already undergone said sentence - No useful
purpose will be served by entertaining into merits of matter - Appeal having become
infructuous, therefore, disposed of. (Paras 12, 15, 16)
Cases Referred : Chronological Paras
2007 AIR SCW 5013 : AIR 2007 SC 2774 : 2007 Cri LJ 4080 3
2001 AIR SCW 1731 : AIR 2001 SC 1716 : 2001 Cri LJ 2188 5
Sushil Kumar, Sr. Advocate, Ms. Kamini Jaiswal, Dasvir Singh Dalee, Ms. Shomila
Bakshi, Ms. Sunita Dwivedi, Vinay Arora and Sudarshan Singh Ravat, with him for
Appellant; Milind Kumar (for Aruneshwar Gupta, Addl. Advocate General), for
Respondent.
Judgement
C. K. THAKKER, J.:-Leave granted.
2. The present appeal is filed against the judgment and order dated December 6, 2005
passed by a Single Judge of the High Court of Judicature for Rajasthan (Jaipur Bench) in
S.B. Criminal Appeal No. 332 of 2005. By the said order, a Single Judge of the High
Court dismissed the appeal filed by the appellant herein who had been convicted by the
Additional Sessions Judge (Fast Track Court) No.1, Jaipur City, Jaipur in Sessions Case
No. 27 of 2003.
3

. It is not necessary to set out facts in detail since we have stated all the facts in
reported in 2007 AIR SCW 5013

@page-SC478
Criminal Appeal No. 867 of 2006 (Suman Sood @ Kamal Jeet Kaur v. State of
Rajasthan) and a cognate matter which we have decided today. Suffice it to state that a
complaint being First Information Report (FIR) No. 84 of 1995 was registered at Malviya
Nagar Police Station, Jaipur on February 26, 1995 against Daya Singh, appellant herein,
Suman Sood @ Kamal Jeet Kaur (accused No.2) and one Harnek Singh @ Surender
Verma (absconding) for offences punishable under Sections 353, 420, 468, 471, 472, 473,
474 read with Section 120B Indian Penal Code ('IPC' for short), for offences punishable
under Sections 4 and 5 of the Explosive Substances Act, 1908, Sections 7 and 25 of Arms
Act, 1959 and Section 18 of TADA etc.
4. The case of the prosecution was that the appellant herein along with his wife Suman
Sood @ Kamal Jeet Kaur fabricated Registration Certificate for purchasing several
vehicles in order to carry out conspiracy of kidnapping and abducting one Rajender
Mirdha, son of Shri Ram Niwas Mirdha to exert pressure on the Government of India to
release one Devendra Singh Bhullar, an alleged Khalistani terrorist who was being held
in custody by the police. It was also alleged that the appellant was found to be in
possession of prohibited arms and ammunition allegedly recovered from House No. B-
117, Model Town, Ashok Nagar during police raid where the appellant was staying.
5

. Initially, prosecution was launched in the Designated Court at Ajmer since the
provisions of TADA were also invoked. The appellant herein, however, challenged his
prosecution under TADA. In Daya Singh Lahoria v. Union of India and Ors., (2001) 4
SCC 516, this Court upheld the challenge since the prosecution of the accused could only
be maintained in accordance with the Extradition Treaty and the Decree of Extradition
under which the accused were extradited by the United States of America to India.2001
AIR SCW 1731

6. Thereafter, the case was registered as Sessions Case No. 27 of 2003 under the Indian
Penal Code and also under the Explosive Substances Act, 1908. The trial Court, after
considering the evidence on record, convicted accused No.1 (appellant herein) as under:
Under Section 420 IPC : to 7 years R.I. and a fine of Rs.500/-, in default of payment of
fine, to further undergo 6 months S.I.
Under Section 468 IPC : to 7 years R.I. and a fine of Rs.500/-, in default of payment of
fine, to further undergo 6 months S.I.
Under Section 471 IPC : to 2 years R.I.
Under Section 4 of the Explosive Substances Act: to 7 years R.I. and a fine of Rs.500/-,
in default of payment of fine, to further undergo 6 months S.I.
7. So far as accused No.2 (Suman Sood) is concerned, she was acquitted by the Court
observing that the prosecution was unable to prove the case beyond doubt against her.
8. The appellant, being aggrieved by the order of conviction and sentence, preferred an
appeal in the High Court of Rajasthan. Likewise, the State of Rajasthan, being aggrieved
by an order of acquittal passed by the trial Court against Suman Sood instituted an
application for leave to appeal against acquittal.
9. A Single Judge of the High Court refused to grant leave against accused No.2 (Suman
Sood) holding that the trial Court was right in recording an order of acquittal against her
and no case had been made out to grant leave. Leave was accordingly refused.

10. So far as the appellant is concerned, at the time of hearing of appeal, it was stated by
the learned counsel for the appellant that maximum sentence awarded to the appellant
was of seven years and all sentences were ordered to run concurrently. The appellant had
already remained in jail for seven years and thus he had already undergone the sentence
of imprisonment. He, therefore, did not press the appeal. The High Court disposed of the
appeal and observed :
"At the very outset, the learned counsel Mr. G. S. Fauzdar for the accused-appellant Daya
Singh contended that maximum sentence in the present case was seven years and all the
sentences were ordered to run concurrently and appellant has already completed his
sentence of imprisonment of seven years, therefore, in these circumstances, he does not
press the appeal filed on behalf of Daya Singh, challenging his order of conviction and
sentence passed by the trial Court as mentioned above. In view of the above statement of
the learned counsel for the appellant-Daya Singh @ Vinay Kumar,
@page-SC479
the S.B. Cr. Appeal No. 332/05 filed by Daya Singh is hereby dismissed as not pressed."
11. Ms. Kamini Jaiswal, appearing for the appellant, challenged the order of conviction
and sentence. When her attention was invited by the Court to the above paragraph, she
submitted that she did not dispute that such a statement was made on behalf of the
accused in the High Court, but submitted that, she be permitted to argue the appeal,
particularly when in other Special Leave Petitions, leave was granted and appeals were
heard on merits. She also submitted that the impugned judgment and order of conviction
and sentence are ex facie, illegal, unlawful and liable to be set aside. She also submitted
that considering the Extradition Treaty of 1931 between United States of America and
Great Britain and the Extradition Order passed by the American Court on June 11, 1997,
the appellant could not have been prosecuted in Indian Court and the trial of the appellant
was without authority of law. On merits also, no case had been made out by the
prosecution. The other accused (Suman Sood) was acquitted on the same evidence and
leave to appeal against an order of acquittal was refused by the High Court. The
conviction recorded by the trial Court against the appellant on the same evidence is also
vitiated and deserves to be set aside.
12. We would have considered the prayer of the learned counsel particularly when the
case relates to administration of criminal justice and other matters were pending. In the
facts and circumstances, however, we are of the considered opinion that no useful
purpose would be served in entering into the merits of the matter.
13. So far as extradition of the appellant is concerned, we have already dealt with all
contentions relating to Extradition Treaty as well as Extradition Order exhaustively in the
other matter. There we have noted that extradition of the appellant was also allowed for
trial of offences punishable under the Explosive Substances Act, 1908.
14. In our opinion, therefore, prosecution, conviction and sentence of the appellant for
offences punishable under Explosive Substances Act, 1908 cannot be said to be without
jurisdiction or in excess of authority of law. The said contention, therefore, has no force
and must be negatived.
15. So far as the other contention is concerned, we have dismissed the appeal filed by the
appellant against his conviction for an offence punishable under Section 364A, IPC
wherein the appellant-accused has been ordered to undergo imprisonment for life. No
useful purpose, therefore, will be served by entering into the merits of the matter as the
maximum punishment awarded by the trial Court and confirmed by the High Court in the
present appeal was of seven years for the offences said to have been committed by the
appellant and the appellant had already undergone the said sentence. The counsel
appearing for the appellant in the High Court appears to have kept in view the above
position and did not press the appeal. In the light of the finding recorded by us in the
cognate matter, this appeal is, more or less, academic and has become infructuous in view
of the following circumstances;
(i) the appellant has been convicted for an offence punishable under Section 364A, IPC
and has been ordered to undergo sentence of imprisonment for life and we have upheld
the said order; and
(ii) in the present appeal, the appellant has challenged his conviction and sentence
whereby he has been ordered to undergo imprisonment for seven years. The appellant has
remained in jail for seven years and the said period is over.
16. For the foregoing reasons, the appeal deserves to be disposed of and is accordingly
disposed of without entering into merits of the case.
Order accordingly.
AIR 2008 SUPREME COURT 479 "Amarsingh Munnasingh Suryawanshi v. State of
Maharashtra"
(From : Bombay)
Coram : 2 S. B. SINHA AND H. S. BEDI, JJ.
Criminal Appeal No. 97 of 2006, D/- 27 -9 -2007.
Amarsingh Munnasingh Suryawanshi v. State of Maharashtra.
(A) Evidence Act (1 of 1872), S.32 - DYING DECLARATION - Dying declaration -
Credibility - Thumb impression of deceased taken - Endorsement of Medical Officer also
obtained to the effect that deceased had all along been conscious when her dying
declaration was recorded - Contradiction in deposition of doctor making endorsement -
On first day of examination he stated that he had made endorsement after dying
declration was recorded - But after he was recalled
@page-SC480
for re-examination and cross-examination he stated that he had made his signature on
margin of blank paper on which dying declaration was recorded - Such contradiction
liable to be ignored because doctor was examined after more than six years after date of
making endorsement. (Paras 19, 20)
(B) Evidence Act (1 of 1872), S.32 - DYING DECLARATION - Dying declaration -
Credibility - Two dying declarations - Both recorded in different languages - Deceased
proficient in both languages - Dying declarations cannot be challenged. (Para 21)
(C) Evidence Act (1 of 1872), S.32 - DYING DECLARATION - POLICE OFFICERS -
MAGISTRATE - Dying declaration - Credibility - First dying declaration recorded by
Police Officer after consulting doctor and after satisfying himself on basis of advice by
doctor that deceased was in fit mental condition to give statement - Second dying
declaration recorded by Special Judicial Magistrate when Medical Officer was present at
that place - Dying declarations cannot be challenged on ground that it was obligatory on
part of Police Officer or Special Judicial Magistrate to put question to deceased about
whether she was in mental condition to make her statement.(Paras 32, 33)
(D) Evidence Act (1 of 1872), S.32 - DYING DECLARATION - WITNESS - Dying
declaration - Credibility - Witness who recorded dying declaration categorically stating
that he had been asking questions to which deceased was giving replies - Fact that dying
declaration was not in question and answer form - Not material. (Para 15)
(E) Penal Code (45 of 1860), S.300 - MURDER - WITNESS - Murder of wife by
husband - Proof - Son of deceased residing with relatives of accused-husband for six
years before he came to Court for deposition - Fact that he went back from statement
made by him under S. 161, Cr. P. C. and did not support prosecution case - Would not
lead to conclusion that prosecution failed to prove its case.
Evidence Act (1 of 1872), S.154. (Para 34)
(F) Penal Code (45 of 1860), S.300 - MURDER - PLEA - Murder - Proof - Murder of
wife by husband - Plea of alibi not proved by accused - No explanation offered by him as
to why he was absconding for about a month - Husband and wife with their children
living in one room and besides their children only husband and wife were present in
house at time of occurrence - Fact that kerosene was put on wife's body and fire was lit
was proved - Husband must be held to be aware that such act was likely to cause death in
ordinary course of nature - No evidence to show that death was result of accident -
Conviction of accused for murder upheld. (Paras 35 , 36, 37)
Cases Referred : Chronological Paras
2007 AIR SCW 295 : 2007 (2) AIR Jhar R 177 : 2007 Cri LJ 1174 (Foll.) Pt.G) 35
2007 AIR SCW 4820 : AIR 2007 SC 2666 : 2007 Cri LJ 3737 : 2007 (5) AIR Kar R 275
31
2007 AIR SCW 5995 : 2007 Cri LJ 4722 31
2007 AIR SCW 6021 31
2006 AIR SCW 1037 : AIR 2006 SC 1448 : 2006 Cri LJ 1625 : 2006 (2) AIR Jhar R 325
(Foll.) 28
2006 AIR SCW 1053 : AIR 2006 SC 1319 : 2006 Cri LJ 1629 : 2006 (2) AIR Jhar R 292
(Disting.) 29
AIR 1976 SC 1994 : 1976 Cri LJ 1548 30
AIR 1958 SC 465 : 1958 Cri LJ 818 37
S. D. Singh, Vijay Kumar, Vishwajit Singh and Makarand D. Adkar, for Appellant;
Chinmoy Khaladkar and Ravindra Keshavrao Adsure, for Respondent.
Judgement
S. B. SINHA, J.:-Appellant is before us being aggrieved by and dissatisfied with the
judgment dated 19.7.2005 passed by a Division Bench of the Bombay High Court,
Aurangabad Bench, in Criminal Appeal No. 576 of 1996 dismissing the appeal preferred
by him against the judgment and order of conviction and sentence dated 7.9.1996 of the
learned Additional Sessions Judge, Jalna under Section 302, I.P.C. in Sessions Case No.
24 of 1991.
2. Kamlabai, the deceased, was married to the appellant 15 years prior to the incident
which took place at about 9.00 p.m. on 7.6.1990. The couple had five children - two
daughters and three sons - all being minor at the material time. Appellant was not doing
any work. He was said to be addicted to liquor. Deceased used to work as a labourer in
Bajrang Dal Mill.
3. It is the case of the prosecution that she had all along been ill-treated by her
@page-SC481
husband. On the fateful day of 7.6.1990, she came from work at about 6.30 p.m. She had
prepared food. At about 9 p.m. when she was serving food to the appellant, he took out
kerosene from the lamp, poured it on her and set her ablaze. He, thereafter, fled away
from the place.
4. On hearing her cries, the neighbours came and extinguished the fire. But she was left at
lurch. It appears that somebody informed her uncle-Laxmansingh who visited her next
day morning. He took her to the Civil Hospital, Jalna.
5. The said incident having been reported to the police authorities, the Officer In-charge
of the Police Station asked the Head Constable Sitaram to visit the hospital. He visited
the hospital at about 8.45 p.m. and recorded the dying declaration of the deceased which
was marked as Exhibit 37 before the learned trial Judge. On the said basis, a First
Information Report was lodged. Another dying declaration was recorded by the Special
Judicial Magistrate at about 3.15 p.m. on the same day. The said dying declaration was
marked as Exhibit 43 before the learned Sessions Judge.
6. We may notice that the deceased breathed her last on 22.6.1990, i.e., about 15 days
after the incident.
7. Despite the appellant having been named in the First Information Report, in the
aforementioned two dying declarations, he could not be arrested as he was absconding.
He surrendered only on 5.7.1990.
8. Before the learned trial Judge twelve witnesses were examined on behalf of
prosecution. Apart from P.W.2-Vinod son of the deceased, no other witness was examined
by the prosecution to prove the act of commission of the offence. He, however, turned
hostile. It also appears that other prosecution witnesses also did not support the
prosecution case.
9. We may, however, notice that the defence of the appellant was that at the relevant time
he was in Bombay. In support of the said defence, however, he did not examine any
witness.
10. Learned trial Judge upon consideration of the materials on record, recorded a
judgment of conviction against the appellant. He was sentenced to undergo rigorous
imprisonment for life. Appeal preferred by him has been dismissed by the High Court by
reason of the impugned judgment.
11. Learned counsel appearing on behalf of the appellant would submit;
(i) none of the two dying declarations Exhibits 37 and 43 being reliable, the learned trial
Judge as also the High Court committed a serious error in recording a judgment of
conviction against the appellant;
(ii) no certificate having been obtained to the effect that the deceased was in a fit mental
condition to make the said statement, the dying declaration should not have been relied
upon;
(iii) one of the dying declarations having been recorded in Marathi language and another
in Hindi, a doubt is created as to whether the deceased understood the implication of the
statements made by her;
(iv) P.W.2-Vinod was the only eye-witness to the incident and he having not supported
the prosecution case, the impugned judgment is wholly unsustainable;
(v) The learned trial Judge as also the High Court failed to apply their mind in regard to
various infirmities in the prosecution case.
12. Mr. Chinmoy Khaladkar, learned counsel appearing for the State, on the other hand,
supported the impugned judgment.
13. Indisputably, apart from the two dying declarations, no substantive evidence has been
brought on record to prove the prosecution case. We will at the outset consider as to
whether the said two dying declarations are trustworthy in nature. The first dying
declaration was recorded by P.W.6-Head Constable Sitaram. He before recording the
dying declaration took the opinion of the Medical Officer. He was told that the deceased
was conscious and fit to make a statement. Only thereafter the statement of Kamlabai was
recorded.
14. We have been taken through the deposition of Sitaram. It may be true that on the body
of dying declaration, he did not mention as to whether the left thumb or the right thumb
impression had been taken, but as has been noticed by the learned trial Judge that her left
hand fingers being burnt and there being only some burn injuries on the right hand, only
inference which could be drawn was that in the absence of any material having been on
record to show that the thumb impression of the deceased was a forged one, it was the
right thumb impression.
@page-SC482
15. The said witness categorically stated that he did take down the statement of the
deceased in Marathi language although the mother tongue of the deceased was Hindi.
Both the witnesses knew Hindi. They also knew Marathi. Before P.W.6- Head Constable
Sitaram the deceased gave her statement in Marathi, which was recorded in the said
language. The first dying declaration was not in the question and answer form, but it was
not necessary. The witness had categorically stated that he had been asking questions to
which the deceased was giving replies.
16. The other dying declaration, as noticed, hereinbefore, was recorded by P.W.8-Sharad
who was the Special Judicial Magistrate. He was asked by P.W.6 to record the dying
declaration of the deceased. Before recording the dying declaration, this witness took the
necessary precaution of obtaining the opinion of the medical officer as to whether she
was in a fit mental condition to give her statement and having received the answer in the
affirmative, he had recorded the statement of the deceased in question and answer form.
17. In the said dying declaration (Exhibit 43), not only the thumb impression of the
deceased was taken but endorsement of the medical officer was also obtained to the effect
that she had all along been conscious when her dying declaration was recorded and an
endorsement to that effect was also made by Dr. Shantilal who examined himself as
P.W.12.
18. From the evidence of Dr. Shantilal, it appears that the deceased had suffered
superficial to deep burns being to the extent of 43-45%. She died of septicaemia with
shock due to extensive burns with cardio-respiratory failure.
19. The only contradiction which appears to have been brought on record in his
deposition is that on the first day of the examination he stated that he had made the
endorsement after the dying declaration was recorded, but after he was recalled for re-
examination and cross-examination he stated that he had made his signature on the
margin of blank paper on which the dying declaration was recorded.
20. Learned trial Judge, in our opinion, has rightly ignored the aforementioned minor
contradiction having regard to the fact that he was examined after more than six years
after the date of making the aforementioned endorsement.
21. So far as the submission of learned counsel that the first dying declaration was
recorded in Marathi and the second dying declaration in Hindi is concerned, the same, in
our view, does not make any difference as she was proficient in both the languages. Her
mother tongue being Hindi she evidently knew that language, but the same would not
mean that she being resident of Maharasthra for a long time would not know the Marathi
language. Admittedly, the appellant (her husband) is a Marathi.
22. The deceased, as noticed hereinbefore, suffered superficial to deep burn injuries and
as such injuries were on the organs such as hands, fingers, legs, etc. She did not suffer
any burn injury on the upper part of the body. She suffered agony and pain throughout the
night of 7.6.1990.
23. We have noticed hereinbefore that on both the occasions the medical officer certified
that she was in a fit mental state to give her dying declaration.
24. Both the witnesses who recorded her dying declarations satisfied themselves that the
deceased answered the questions after understanding the effect thereof.
25. Learned counsel submitted that in the dying declaration dated 8.6.1990, the deceased
in answer to the question as to who was at home, disclosed that besides she and her
husband nobody else was there, although according to the prosecution case, P.W.2 Vinod
was present.
26. It may be noticed that while answering the question which was posed to her, namely,
when the incident happened who was at home? In answer thereto she had stated "I and
my husband were there nobody else was there" which evidently would mean that no
outsider was present. It is not the case of the appellant that children on the date of
occurrence were residing somewhere else.
27. It is now a well settled principle of law that a conviction can be based on the dying
declaration alone, subject of course, to the satisfaction of the Court that the same is
trustworthy.
28

. In Ravi Kumar alias Kutti Ravi vs. State of Tamil Nadu, (2006) 9 SCC 240, this Court
held as under: 2006 AIR SCW 1037, (Para 4)

"5. ... The dying declaration is admissible


@page-SC483
upon consideration that the declarant has made it in extremity, when the maker is at the
point of death and when every hope of this world is gone, when every motive to the
falsehood is silenced and the mind is induced by the most powerful consideration to
speak the truth. Notwithstanding the same, care and caution must be exercised in
considering the weight to be given to these species of evidence on account of the
existence of many circumstances which may affect their truth. The court has always to be
on guard to see that the statement of the deceased was not the result of either tutoring or
prompting or a product of imagination. The court has also to see and ensure that the
deceased was in a fit state of mind and had the opportunity to observe and identify the
assailant. Normally, therefore, the court in order to satisfy itself that the deceased was in
fit mental condition to make the dying declaration, has to look for the medical opinion.
Once the court is satisfied that the declaration was true and voluntary, it undoubtedly, can
base its conviction on dying declaration without any further corroboration. It cannot be
laid down as an absolute rule of law that the dying declaration cannot form the sole basis
of conviction unless it is corroborated. The rule requiring corroboration is merely the rule
of prudence."
29
. Learned counsel appearing on behalf of the appellant, however, has strongly relied upon
a decision of this Court in P. Mani vs. State of Tamil Nadu, (2006) 3 SCC 161. In that
case this Court noticed the factual matrix thus: 2006 AIR SCW 1053

"12. ...........The incident admittedly took place inside a small room. It had two doors. The
prosecution witnesses knocked on both the doors. Their call to the deceased to open the
door remained unanswered and only then they took recourse to breaking open the door.
According to them, not only the appellant herein was with them at that point of time, but
also he took part in dousing the flames. Indisputably, he took the deceased to the hospital.
If the version of the deceased in her dying declaration is accepted as correct, the
witnesses and in particular the neighbours would have lodged a First Information Report
and in any event would not have permitted the appellant to take her to the hospital."
"14. Indisputably conviction can be recorded on the basis of dying declaration alone but
therefor the same must be wholly reliable.
(Emphasis supplied)
The said decision, therefore, is of no assistance to the appellant as apart from the fact that
factual matrix of the matter was different, as a proposition of law it has clearly been laid
down that conviction can be based on dying declaration alone.
30

. In K. Ramachandra Reddy and another vs. The Public Prosecutor, (1976) 3 SCC 618,
whereupon reliance has been placed this Court came to the conclusion that the
prosecution case was not reliable at all. A large number of discrepancies were also found
in the evidence of the doctor vis-a-vis the Magistrate who had recorded the said dying
declaration. It was noticed: AIR 1976 SC 1994

"11.....The Magistrate P.W. 11 who recorded the dying declaration has admitted that the
injured was suffering from pain and he was not in a position to sign and so his thumb
impression was taken. The Magistrate further admitted that the injured was taking time to
answer the questions. The Magistrate further admitted that the injured was very much
suffering with pain. In spite of these facts the Magistrate appears to have committed a
serious irregularity in not putting a direct question to the injured whether he was capable
mentally to make any statement."
31

. In Mehiboobsab Abbasabi Nadaf Vs. State of Karnataka (2007 (9) SCALE 473, this
Court held: 2007 AIR SCW 4820

"6. Conviction can indisputably be based on a dying declaration. But, before it can be
acted upon, the same must be held to have been rendered voluntarily and truthfully.
Consistency in the dying declaration is the relevant factor for placing full reliance
thereupon. In this case, the deceased herself had taken contradictory and inconsistent
stand in different dying declarations. They, therefore, should not be accepted on their face
value. Caution, in this behalf, is required to be applied.
Recently, in State of Rajasthan vs. Parthu (2007 (11) SCALE 460), it was held: 2007
AIR SCW 5995

"(12) It is now a well settled principles of law that a judgment of conviction can be
recorded on the basis of the dying declaration alone subject of course to the
@page-SC484
satisfaction of the Court that the same was true and voluntary. For the purpose of
ascertaining truth or voluntariness of the dying declaration, the Court may look to the
other circumstances. Apart from the fact, as noticed hereinbefore, that the homicidal
nature of death was not disputed by the respondent herein and furthermore as he in his
statement under Section 313 had raised a positive defence that she died of an accident, we
are of the opinion the High Court adopted a wrong approach. It is not disputed that the
deceased and the appellant were living separately from their family. It has also not been
disputed that at the time when the incident occurred, the respondent was in his house
together with the deceased. It is furthermore not in dispute that after the incident took
place, the respondent was not to be found. He was arrested only on 20-6-1995. If the
deceased and the respondent were together in their house at the time when the incident
took place which was at about 10 O'clock in the night, it was for the respondent to show
as to how the death of the deceased took place."

(See also Nallapati Sivaiah vs. Sub-Divisional Officer, Guntur, A.P.-2007 (11) SCALE
477). 2007 AIR SCW 6021

32. Learned counsel would submit that it was obligatory on the part of both P.W.6 Head
Constable Sitaram and P.W.8 Special Judicial Magistrate-Sharad to satisfy themselves
that the deceased was in a mental condition to make her statement and to prove the said
fact it was obligatory on their part to put a question in that behalf to the deceased.
33. In this case as noticed hereinbefore, when the dying declaration was recorded by
P.W.6 Head Constable Sitaram, he consulted the doctor and recorded the dying
declaration only after he satisfied himself on the basis of the advice given to him that the
patient was in a fit mental condition to give her statement. So far as dying declaration
recorded by P.W.8 Special Judicial Magistrate-Sharad is concerned, we have noticed
hereinbefore that he took all the precautions and in fact a medical officer was present
when the said dying declaration was being recorded.
34. P.W.2 Vinod was the son of the appellant. He had remained with the relatives of the
appellant for six years before he came to the court for deposition. If he had gone back
from his statement made by him under Section 161, Cr.P.C., and did not support the
prosecution case, the same, in our opinion, by itself would not lead to the conclusion that
the prosecution has failed to prove its case.
35

. There cannot be any doubt whatsoever that the appellant had not been able to prove his
alibi. He did not examine any witness to support his case. He did not offer any
explanation whatsoever as to why for about a month he was absconding. In a situation of
this nature where admittedly the husband, wife and children were residing in one room,
the prosecution having been able to prove that apart from the minor children, at the time
of occurrence it was he and the deceased alone who were residing in the house, it was for
the appellant to prove that how the deceased had met her death. This aspect of the matter
was considered by this Court in Raj Kumar Prasad Tamarkar vs. State of Bihar and
Another,(2007) 1 SCALE 19. 2007 AIR SCW 295

36. The cause of death is not in dispute. What was contended by the appellant was that
the death was not homicidal in nature but it was a result of an accident. Yet again, nothing
has been brought on record to show that the death was a result of any accident.
37

. The fact remains that the kerosene was put on her body and fire was lit. We, thus, cannot
accept the plea of the learned counsel appearing on behalf of the appellant that there was
no intention on the part of the accused to cause her death. He must be held to be aware
that such an act was likely to cause death in the ordinary course of nature. (See Virsa
Singh vs. State of Punjab, 1958 SCR 1495). AIR 1958 SC 465

38. For the reasons aforementioned we do not find any merit in this appeal. The appeal is
dismissed accordingly.
Appeal dismissed.
AIR 2008 SUPREME COURT 484 "National Insurance Co. Ltd. v. Cholleti
Bharatamma"
(From : Andhra Pradesh)*
Coram : 2 S. B. SINHA AND H. S. BEDI, JJ.
Civil Appeal Nos. 4845-4847 with 4848-4850, 4851, 4852-54 of 2007 (arising out of SLP
(C) Nos. 7237-7239 with 7241-7243, 7248 and 7288-7290 of 2003), D/- 12 -10 -2007.
National Insurance Co. Ltd. v. Cholleti Bharatamma and Ors.
(A) Motor Vehicles Act (59 of 1988), S.147 (before amendment in year 1994) - MOTOR
VEHICLES - CARRIAGE OF GOODS - INSURANCE - - Goods carriage - Carrying
passengers
@page-SC485
Accident - Insurance Company would be liable for death and injury of said passengers.
A. A. O. Nos. 1387, 1901 and 1923 of 1998, D/- 13-11-2002 (AP), Partly Reversed.
(Paras 12, 14)
(B) Motor Vehicles Act (59 of 1988), S.147 - MOTOR VEHICLES - CARRIAGE OF
GOODS - INSURANCE - Liability of insurer - Goods carriage - No proof that deceased
was travelling in lorry along with driver or cleaner, as owner of goods - Travelling with
goods itself does not entitle anyone to protection under S. 147 - Deceased held, not
entitled to protection under S. 147. (Para 18)
(C) Motor Vehicles Act (59 of 1988), S.147 - MOTOR VEHICLES - INSURANCE -
CARRIAGE OF GOODS - Liability of insurer - Goods carriage - Carrying 20 persons -
Premium paid only for one person travelling as owner - Claimants of deceased who was
travelling as owner of goods, held entitled to compensation.(Paras 23, 24)
(D) Motor Vehicles Act (59 of 1988), S.147 - MOTOR VEHICLES - INSURANCE -
CARRIAGE OF GOODS - Liability of insurer - Goods carriage carrying passengers -
Accident - Evidence of one of passengers showing that he was travelling in vehicle with
his goods as owner and not the deceased - Deceased thus was gratuitous passenger and
policy would not cover his life - Insurance Company not liable.
A. A. O. Nos. 1387, 1901 and 1923 of 1998, D/- 13-11-2002 (AP), Reversed. (Para
31)
Cases Referred : Chronological Paras
2007 AIR SCW 1505 : AIR 2007 SC 1334 : 2007 (3) ALJ 193 : 2007 (2) AIR Jhar R 772
(Foll.) (Pt. A) 11
2007 AIR SCW 3734 : AIR 2007 SC 1971 11
(2005) 12 SCC 243 (Foll.) Pt. A) 10
2004 AIR SCW 212 : AIR 2004 SC 1340 (Foll.) (Pt.A) 10
2002 AIR SCW 5259 : AIR 2003 SC 607 (Foll.) (Pt.A) 9, 10, 29
2001 AIR SCW 3291 : AIR 2001 SC 3363 7
1999 AIR SCW 255 : AIR 1999 SC 589 9
1999 AIR SCW 4337 : AIR 2000 SC 235 5, 6, 9, 13, 16, 23, 29, 30
1996 AIR SCW 2466 : AIR 1996 SC 2054 15
Dr. Laxmi Shashtri, Ms. Kiran Suri, Kishore Rawat, M. K. Dua, Pramod Dayal, Vishnu
Mehra and B.K. Satija, for Appellant; Dr. Sushil Balwada, Vidya K. Sagar, B. D. Sharma,
Ms. Asha Gopalan Nair, A. V. Rao, Prabhakar Parnam and Venkateswara Rao Anumolu,
for Respondents.
* A. A. O. Nos. 1387, 1901 and 1923 of 1998, D/- 13-11-2002 (AP).
Judgement
1. S. B. SINHA, J.:-Leave granted in all the Special Leave Petitions.
2. The question involved in these appeals centres around the liability of the insurance
company to indemnity the owner of the vehicle in respect of death of passengers
travelling in goods carriage. The dates of accident being different, different provisions
would apply. We would notice the law operating in the field at the outset and apply the
same in each case separately.
3. The relevant portion of Section 147 of the Motor Vehicles Act (for short "the Act"),
prior to its amendment, reads as under:
"Requirements of policies and limits of liability.- (1) In order to comply with the
requirements of this Chapter, a policy of insurance may be a policy which-
(a) ***
(b) insures the person or classes of persons specified in the policy to the extent specified
in sub-section (2)-
(i) against any liability which may be incurred by him in respect of the death of or bodily
injury to any person or damage to any property of a third party caused by a or arising out
of the use of the vehicle in a public place;
(ii) ***
Provided that a policy shall not be required-
(i) to cover liability in respect of the death, arising out of and in the course of his
employment, of the employees insured by the policy or in respect of bodily injury
sustained by such an employee arising out of and in the course of his employment other
than a liability arising under the Workmen's Compensation Act, 1923, in respect of the
death of, or bodily injury to, any such employee-
(a) ***
(b) ***
(c) if it is a goods vehicle, being carried in the vehicle; or
(ii) except where the vehicle is a vehicle in which passengers are carried for hire or
reward or by reason of or in pursuance of contract of employment, to cover liability in
respect of the death of or bodily injury to persons being carried in or upon or
@page-SC486
entering or mounting or alighting from the vehicle at the time of the occurrence of the
event out of which a claim arises..."
4. The said provision underwent an amendment in the year 1994 by Motor Vehicles
Amendment Act, 1994 which reads as under:
"147 - Requirements of policies and limits of liability
(1) In order to comply with the requirements of this Chapter, a policy of insurance must
be a policy which-
(a) ***
(b) insures the person or classes of persons specified in the policy to the extent specified
in sub- section (2)-
(i) against any liability which may be incurred by him in respect of the death of or bodily
[injury to any person, including owner of the goods or his authorised representative
carried in the vehicle] or damage to any property of a third party caused by or arising out
of the use of the vehicle in a public place;
(ii) ***
5

. In New India Insurance Co. v. Satpal Singh and Ors. [(2000) 1 SCC 237], this Court
proceeded on an assumption that the provisions of 1939 Act and the provisions of 1988
Act are in pari materia. 1999 AIR SCW 4337

6. In Satpal Singh (supra), interpreting the provisions contained in Sections 147 and 149
of the Motor Vehicles Act, this Court held:
"...Hence, under sub-section (2), there is no upper limitation for the insurer regarding the
amount of compensation awarded in respect of death or bodily injury of a victim of the
accident. It is, therefore, apparent that the limit contained in the old Act has been
removed and the policy should insure the liability incurred and cover injury to any person
including owner of the goods or his authorised representative carried in the vehicle . The
legislature has also taken care of even the policies which were in force on the date of
commencement of the Act by specifically providing that any policy of insurance
containing any limit regarding the insurers liability shall continue to be effective for a
period of four months from commencement of the Act or till the date of expiry of such
policy, whichever is earlier. This means, after the said period of four months, a new
insurance policy consistent with the new Act is required to be obtained.
11. The result is that under the new Act an insurance policy covering third-party risk is
not required to exclude gratuitous passengers in a vehicle, no matter that the vehicle is of
any type or class. Hence the decisions rendered under the old Act vis-a-vis gratuitous
passengers are of no avail while considering the liability of the insurance company in
respect of any accident which occurred or would occur after the new Act came into
force."
7
. In Ramesh Kumar v. National Insurance Co. Ltd. and Anr. [(2001) 6 SCC 713], this
Court categorized the cases arise out of the Motor Vehicles Act, 1939, stating: 2001
AIR SCW 3291

"The first category of cases arise out of the Motor Vehicles Act, 1939 (hereinafter
referred to as "the old Act"). The question raised for this category is:
'Whether the insurance company is liable to pay the compensation on account of the
death or bodily injury of the gratuitous passengers including the owner of the goods or
his representative, travelling in a goods vehicle under Section 95 of the said Act?'

The second category of cases arise out of the Motor Vehciles Act, 1988 (hereinafter
referred to as "the new Act") prior to its amendment in 1994. In this category also a
similar question is raised. The third category of cases also arises under the new Act but
after its amendment by Act 54 of 1994. In this category also the same question is raised."
8. The Act does not contemplate that a goods carriage shall carry a large number of
passengers with small percentage of goods as considerably the insurance policy covers
the death or injuries either of the owner of the goods or his authorized representative.
9

. Correctness of the decision in Satpal Singh (supra) came up for consideration before a
three-Judge Bench of this Court in New India Assurance Co. Ltd. v. Asha Rani and others
[(2003) 2 SCC 223]. 1999 AIR SCW 4337
2002 AIR SCW 5259

In Asha Rani (supra), having regard to various definitions involving the legal question, it
was held:

"23. The applicability of the decision of this Court in Mallawwa v. Oriental Insurance Co.
Ltd., in this case must be considered 1999 AIR SCW 255

@page-SC487
keeping that aspect in view. Section2(35) of the 1988 Act does not include passengers in
goods carriage whereas Section2(25) of the 1939 Act did as even passengers could be
carried in a goods vehicle. The difference in the definitions of "goods vehicle" in the
1939 Act and "goods carriage" in the 1988 Act is significant. By reason of the change in
the definitions of the terminology, the legislature intended that a goods vehicle could not
carry any passenger, as the words "in addition to passengers" occurring in the definition
of goods vehicle in the 1939 Act were omitted. Furthermore, it categorically states that
"goods carriage" would mean a motor vehicle constructed or adapted for use "solely for
the carriage of goods". Carrying of passengers in a "goods carriage", thus, is not
contemplated under the 1988 Act.
24. We have further noticed that Section 147 of the 1988 Act prescribing the requirements
of an insurance policy does not contain a provision similar to clause (ii) of the proviso
appended to Section 95 of the 1939 Act. The decision of this Court in Mallawwa case
must be held to have been rendered having regard to the aforementioned provisions.
25. Section 147 of the 1988 Act, inter alia, prescribes compulsory coverage against the
death of or bodily injury to any passenger of "public service vehicle". Proviso appended
thereto categorically states that compulsory coverage in respect of drivers and conductors
of public service vehicle and employees carried in a goods vehicle would be limited to
the liability under the Workmen's Compensation Act. It does not speak of any passenger
in a "goods carriage."
26. In view of the changes in the relevant provisions in the 1988 Act vis-a-vis the 1939
Act, we are of the opinion that the meaning of the words "any person" must also be
attributed having regard to the context in which they have been used i.e. "a third party".
Keeping in view the provisions of the 1988 Act, we are of the opinion that as the
provisions thereof do not enjoin any statutory liability on the owner of a vehicle to get his
vehicle insured for any passenger travelling in a goods vehicle, the insurers would not be
liable therefor.
27. Furthermore, sub-clause (i) of clause (b) of sub-section (1) of Section 147 speaks of
liability which may be incurred by the owner of a vehicle in respect of death of or bodily
injury to any person or damage to any property of a third party caused by or arising out of
the use of the vehicle in a public place, whereas sub-clause (ii) thereof deals with liability
which may be incurred by the owner of a vehicle against the death of or bodily injury to
any passenger of a public service vehicle caused by or arising out of the use of the
vehicle in a public place.

28. An owner of a passenger-carrying vehicle must pay premium for covering the risks of
the passengers. If a liability other than the limited liability provided for under the Act is
to be enhanced under an insurance policy, additional premium is required to be paid. But
if the ratio of this Court's decision in New India Assurance Co. v. Satpal Singh, is taken to
its logical conclusion, although for such passengers, the owner of a goods carriage need
not take out an insurance policy, they would be deemed to have been covered under the
policy wherefor even no premium is required to be paid." 1999 AIR SCW 4337

10

. The effect of 1994 amendment came up for consideration in National Insurance Co. Ltd.
v. Baljit Kaur and Ors. [(2004) 2 SCC 1], wherein this court following Asha Rani (supra)
opined that the words "injury to any person" would only mean a third party and not a
passenger travelling on a goods carriage whether gratuitous or otherwise. The question
came up for consideration again in National Insurance Co. Ltd. v. Bommithi
Subbhayamma and Ors. [(2005) 12 SCC 243] wherein upon taking into consideration a
large number of decisions, the said view was reiterated. 2004 AIR SCW 212

11

. Yet again in New India Assurance Co. Ltd. v. Vedwati and Ors. [(2007) 3 SCALE 397]
this Court held: 2007 AIR SCW 1505

"13. The difference in the language of "goods vehicle" as appear in the old Act and
"goods carriage" in the Act is of significance. A bare reading of the provisions makes it
clear that the legislative intent was to prohibit goods vehicle from carrying any passenger.
This is clear from the expression "in addition to passengers" as contained in definition of
"goods vehicle" in the old Act. The position becomes further clear because the expression
used is "goods carriage" is solely for the carriage of goods. Carrying of passengers in a
goods carriage is not contemplated in the Act. There is no provision
@page-SC488
similar to Clause (ii) of the proviso appended to Section 95 of the old Act prescribing
requirement of insurance policy. Even Section 147 of the Act mandates compulsory
coverage against death of or bodily injury to any passenger of "public service vehicle".
The proviso makes it further clear that compulsory coverage in respect of drivers and
conductors of public service vehicle and employees carried in goods vehicle would be
limited to liability under the Workmen's Compensation Act, 1923 (in short 'WC Act").
There is no reference to any passenger in "goods carriage".
14. The inevitable conclusion, therefore, is that provisions of the Act do not enjoin any
statutory liability on the owner of a vehicle to get his vehicle insured for any passenger
travelling in a goods carriage and the insurer would have no liability therefor."

[See also Oriental Insurance Co. Ltd. v. Brij Mohan and Ors. 2007 (7) SCALE 753]
2007 AIR SCW 3734

CA @ SLP (C) Nos.7237-7239 of 2003


12. The claimants were travelling in a lorry. It was a goods carriage carrying goods like
rice, tent hours articles, chairs, utensils and vegetables required on the occasions of
marriage. The marriage of one D. Bhaskar was to take place. When the lorry reached Ali
Nagar cross road at about 8.30 a.m., as a result of rash and negligent driving of the driver,
the accident took place as the lorry struck to a stationary truck. Several people suffered
injuries. Two of them died. Several claim applications were filed on behalf of the injured
as also the dependents of the deceased.
The date of accident being 16.12.1993, the amendment carried out in the year 1994 in
Section 147 of the Motor Vehicles Act would not be applicable.
13. The Motor Accident Claims Tribunal, Nalgonda, by a judgment and award dated
13.11.1997 awarded various sums overruling the defence of the appellant herein that they
were unauthorized passengers. The High Court, however, by reason of the impugned
judgment, relying on or on the basis of a decision of this Court in Satpal Singh (supra)
directed as under:

"The learned counsel for the Insurance Company submitted that the issue involved in
these appeals is squarely covered by the decision of the Supreme Court in the case
reported in New Indian Assurance Company Ltd. v. Satpal Singh, 2000 ACJ 1, wherein
their Lordships held that under the Motor Vehicles Act, 1988 all insurance policies
covering third party risks are not required to exclude gratuitous passengers in the Vehicle
though Vehicle is of any type or class. 1999 AIR SCW 4337

In view of the proposition of law laid down by the Supreme Court in the decision stated
supra, these appeals are dismissed. No costs."
14. Following the aforementioned principles, the impugned judgment cannot be sustained
which is set aside. The appeals are allowed accordingly.
CA @ SLP (C) No.7241-7243/03
15. In the aforementioned case, accident took place on 24.12.1993. Respondents herein
filed a claim petition claiming compensation for the death of one Kota Venkatarao who
had allegedly paid a sum of Rs. 20/- for travelling in the lorry. The Tribunal held:

"In the absence of rebuttal evidence from the deceased and some others travelled in the
said vehicle in the capacity of owner of the luggage which was carried by them at the
time of accident. It cannot be said that it is a violation of the policy, since it is not
fundamental breach so as to afford to the insurer to eschew the liability altogether as per
the decision reported in AIR 1996 Supreme Court 2054." 1996 AIR SCW 2466

16. The High Court, however, relying upon Satpal Singh (supra) opined:

"This issue raised in this appeal is covered by the decision of the Supreme Court in New
India Assustance Co. Ltd. v. Satpal Singh [(2000) 1 SCC 237] wherein their Lordships
held that under the Motor Vehicles Act, 1988 all Insurance policies covering third party
risks are not required to exclude gratuitous passengers in the vehicles though the vehicle
is of any type or class. Following the same, the appeal is dismissed. No order as to costs."
1999 AIR SCW 4337

17. It is now well settled that the owner of the goods means only the person who travels
in the cabin of the vehicle.
18. In this case, the High Court had proceeded on the basis that they were gratuitous
passengers. The admitted plea of the respondents themselves was that the
@page-SC489
deceased had boarded the lorry and paid an amount of Rs.20/- as transport charges. It has
not been proved that the deceased was travelling in the lorry along with the driver or the
cleaner as the owner of the goods. Travelling with the goods itself does not entitle anyone
to protection under Section 147 of the Motor Vehicles Act.
19. For the reasons aforementioned, this appeal is dismissed.
CA @ SLP (C) No.7248/03
20. The accident in this case took place on 3.1.1991. Twenty persons were travelling in
the truck. The policy covered the risk only of the owner of the goods. Before the learned
Tribunal, it was contended that the risk of the owners of the goods is covered by the
policy. It was held:
"On a careful consideration of the various authorities cited by the learned counsels for
both the parties, Section 147, 149; Rule 277(3) and 252 of Rules framed under M.V. Act I
have no hesitation to conclude that the risk of the owner of the goods is also covered by
the policy issued by the insurance companies, from the evidence of R.W.1 who is no
other than the employee of R-2 as well as terms of Ex.B-2 Policy, it is obvious that the
risk of the owner of the goods is covered, but it is restricted only to one person as owner
of the goods. Thus, there can be no doubt that the owner of the goods can travel in the
goods vehicle and if they are involved in the accident, their risk is covered subject to the
terms and conditions of the policy issued by the insurance companies.
21. The learned Tribunal, however, noticed:
"...Thus, the claim form corroborate the testimony of the petitioners that deceased or the
injured as the case may be travelled in the vehicle as owner of goods. But it is mentioned
in Ex.B-3 claim form as well as in Ex.B1 permit that the seating capacity of the lorry is
only '3' including driver and cleaner which would go to show that only one passenger can
travel in it..."
22. Upon considering the evidences on record, it was held:
"As the permitted seating capacity of the lorry is only '3' including the driver and cleaner
and as only one non-fare paying passenger as owner of goods can travel in the cabin and
as the deceased has admittedly travelled in the cabin beyond seating capacity and
contrary to the terms of the permit as well as Rule 252(2) of the Motor Vehicles Rules. I
am of the view that R-2 cannot be fastened with the liability to pay compensation along
with R-1 to all the injured and legal representatives of deceased. At best it is liable to pay
compensation jointly and severally along with R-1 only in respect of one non-fare paying
passengers, who is the owner of the goods. As per the endorsement I.M.T. 14(b) unless
additional premium is paid for the number of persons who travelled in the lorry, as
owners. I am of the view that R-2 cannot be fastened with liability. Further all the
petitioners and deceased cannot be deemed to have travelled as owners of the paddy as
the paddy is said to be in bags and orally kept in loose in the lorry and it is enough if any
one of them have travelled in the lorry on behalf of all, as owner of the lorry. Rule 277(3)
of A.P. Motor Vehicles Rules, clearly shows that no person shall be carried in the goods
vehicle except as provided in the Rule under the statute and as the only person, who are
permitted to carry in goods vehicles are the owner of hirer or bona fide employee of
owner of hirer and total number of such persons, who could be carried in goods vehicles
is not more than seven including the driver. As per Rule 252(2) person shall be carried in
the cab of the vehicle beyond the seating capacity as per clause (2). No person shall be
carried on the load or otherwise. Rule 4 empowers the R.T.A. to allow large number of
persons to be carried. As the seating capacity of the lorry is only '3' as per Ex.B1 and B3
and as the risk of only owner of goods is covered by Ex.B2 policy, whereas about 40 to
42 persons travelled in the lorry by sitting on the load, which is not permitted and as there
is no material to show that R.T.A. permitted carriage of more than seating capacity but on
the other hand the permit is cancelled. I am in agreement with the contention of the
learned counsel for the respondent that it cannot be fastened with the liability for
compensation."
23

. The High Court, however, dismissed the appeals preferred by the respondents relying
upon Satpal Singh (supra). Submission of the learned counsel appearing on behalf of the
respondent is that within the aforementioned twenty persons, it is the respondents having
preferred an appeal, this Court 1999 AIR SCW 4337

@page-SC490
should hold that at least the claimants-respondents are entitled to compensation as the
deceased was travelling as owner of the goods. The learned Tribunal discussed the matter
in great details. It is not in dispute that premium has been paid only for one person.
24. In the facts and circumstances of this case, we are of the opinion that the contention
of the respondent should be accepted. This appeal is, thus, dismissed.
CA @ SLP (C) Nos.7288-7290/03
25. In this case, the accident took place on 1.5.1997. Indisputably, the respondent was
travelling as a passenger. The Tribunal, while determining the issue as to whether the
accident took place due to rash and negligent driving of the first respondent- driver of
lorry AEW 5199, held:
"...The lorry was overturned and caused the instantaneous death of four passengers. He
received small injuries. He also deposed that the accident took place due to the
negligence of the driver of the said lorry. On perusing his evidence I am satisfied that he
is a truthful witness. He was travelling in the crime vehicle along with deceased along
with his goods as per his evidence..."
26. The learned counsel appearing for the respondent, submitted that from the
aforementioned finding, it is evident that the respondent was travelling as the owner of
the goods. We do not think that the said submission is correct. PW-2, in his evidence,
stated:
"I am doing Tamarind business. I witnessed the accident which took place about 3 years
back at about 6 a.m. at Borrampalem junction beyond Talluru. At the time of the accident
I was in the crime lorry by the side of the driver. Myself and 6 others were carrying
tamarind in that lorry belonging to us. We boarded the lorry along with our load of
tamarind at Dharamavaram to go to Rajanagaram. We were selling the Tamarind at
Rajanagaram in retail by taking the tamarind there in our lorry from our village of
Dharmavaram."
27. The Tribunal, therefore, correctly recorded that according to PW-2, he was travelling
with his goods as owner thereof and not the deceased.
28. Shaik Shabbeer Pasha and Shaik Nazeer Pasha are the Driver and owner respectively
of the lorry which was travelling to Rajahmundry from Visakhapatnam. At Borrampalem,
while trying to overtake another speeding lorry, the same turned turtle. Three persons
who were travelling in the vehicle had been killed. Claim for compensation were filed
before the Motor Accident Claims Tribunal. The appellants therein opposed the claim.
Tribunal awarded compensation to the legal heirs of the deceased.
29

. Challenging the legality of the award of the Tribunal, learned counsel for the appellant
contended that the deceased were gratuitous passengers and the policy did not cover their
lives. Learned counsel also submitted that the decision in Satpal Singh's case (supra)
being referred to a Larger Bench in Asha Rani's case (supra), the same was not a binding
authority. 1999 AIR SCW 4337
2002 AIR SCW 5259

30. While stating that the submissions of learned counsel for insurance company could
not be sustained, the High Court dismissed the appeal of the insurance company
following Satpal Singh (supra).
31. In view of the nature of evidence available before us, we have no other option but to
set aside the judgment. These appeals are, therefore, allowed accordingly. There shall be
no order as to costs in each case.
Order accordingly.
AIR 2008 SUPREME COURT 490 "New India Assurance Co. Ltd. v. Kendra Devi"
(From : Uttaranchal)
Coram : 2 TARUN CHATTERJEE AND P. SATHASIVAM, JJ.
Civil Appeal No. 5067 of 2007 (arising out of SLP (C) No. 7686 of 2006), D/- 31 -10
-2007.
New India Assurance Co. Ltd. v. Kendra Devi and Ors.
Motor Vehicles Act (59 of 1988), S.147 - Constitution of India, Art.133 - MOTOR
VEHICLES - APPEAL - INSURANCE - Liability of insurer - Accident of taxi - Owner
driving it died - Insurance policy covering only risk of paid driver - Considering fact that
deceased was only bread earner of family - S. C. refused to interfere with award. (Para
7)
Cases Referred : Chronological Paras
(2006) 9 SCC 174 (Ref.) 6
Sanjiv Sharma, Harinder Singh, Ms. Manjusha Jha, Arun K. Sinha, Rakesh Singh,
Mukesh Kumar Sinha, for Appellant;
@page-SC491
A. S. Rawat, K. S. Rana, for Respondents.
Judgement
P. SATHASIVAM, J.:-Leave granted.
2. Questioning the orders of the High Court of Uttaranchal at Nainital dated 24.08.2004
in A.O. No. 436 of 2001 and dated 27.10.2005 in R.A. No.8 of 2005, New India
Assurance Company Ltd. through its Regional Manager, New Delhi has filed the above
appeal.

3. Brief facts are as follows: Smt. Kendra Devi, respondent No.1 herein, filed Claim
Petition - M.A.C. No.4 of 1994, before the Motor Accident Claims Tribunal, Uttarkashi,
claiming compensation of Rs.4,67,000/- on account of death of her husband in a motor
vehicle accident. According to respondent No.1 herein, her husband, Prakash Singh
Parmar, was the owner and driver of Taxi No. UMX 491. On 10.11.1993 while her
husband was going from Matali to Uttarkashi, he lost control over the vehicle and met
with an accident due to which the vehicle rolled into the river Bhagirathi near Barrthi.
Her husband sustained fatal injuries in the accident and succumbed to the injuries at the
spot. According to respondent No.1, at the time of the accident her husband was earning
Rs.3000/- per month.
4. Before the Tribunal, the appellant, New India Assurance Co. Ltd. contended that the
vehicle was insured only for five persons whereas it was carrying eight passengers,
therefore, the owner and driver of the vehicle violated the terms and conditions of the
Insurance Policy, in view of the same, they are not liable to pay compensation to the
claimants. The Tribunal, based on the materials before it, after finding that the driver of
the vehicle had valid licence and taking note of the age of the deceased i.e., 50 years, by
applying multiplier of ten, awarded a total compensation of Rs.1,25,000/- along with
interest @ 12%. Aggrieved by the said award, the Insurance Company filed an appeal
before the High Court. The High Court, after finding no infirmity and illegality in the
order passed by the Motor Accident Claims Tribunal, dismissed the appeal and the review
application filed by the Insurance Company was also dismissed. Against the said orders
of the High Court, the New India Assurance Company Ltd. filed the present appeal.
5. Heard learned counsel appearing for the appellant-New India Assurance Company as
well as the respondents-claimants.
6. The only contention of the learned counsel for the appellant-Insurance Company, is
that inasmuch as the insurance policy was issued for paid driver and not for owner who
also happened to drive the vehicle himself at the time of the accident. In support of his
contention, learned counsel drew our attention to the Insurance Policy(Annexure P-3).
Perusal of the Schedule of Premium mentioned in the Insurance Policy, shows that apart
from liability to public risk, the owner has paid premium only for paid driver and/or
conductor. By contending that in the case on hand, the deceased being the owner-cum-
driver and without additional premium/coverage for owner-cum-driver, the insurance
company is not liable to pay any compensation for death of the deceased who was owner-
cum-driver and not paid driver as mentioned in the Schedule of Premium. In support of
his contention, learned counsel for the appellant heavily relied on Section 147 of the
Motor Vehicles Act, 1988 which speaks about the statutory liabilities and a decision of
this Court in New India Assurance Co. Ltd. Vs. Meera Bai and Ors., (2006) 9 SCC 174.
7. We have carefully considered the above provision as well the decision of this Court.
Taking note of the peculiar fact that the claimants have lost their only breadwinner, we
are not inclined to interfere with the concurrent orders of the Tribunal as well as the High
Court. Consequently, the appeal fails and the same is dismissed. No cost.
Appeal dismissed.
AIR 2008 SUPREME COURT 491 "Girja Kumar v. State of H. P."
(From : 2000 AIHC 4274 (H.P.))
Coram : 2 TARUN CHATTERJEE AND P. SATHASIVAM, JJ.
Civil Appeal No. 6616 of 2000, D/- 31 -10 -2007.
Girja Kumar and Ors. v. State of H.P. and Anr.
Specific Relief Act (47 of 1963), S.34 - Limitation Act (36 of 1963), Art.64, Art.65 -
DECLARATION OF TITLE - LIMITATION - ADVERSE POSSESSION - REMAND
OF MATTER - Suit for declaration of title on basis of adverse possession - Plaintiff
claiming possession for 40 years - No defence raised by defendants either before trial
Court or even before High Court that plaintiff was ejected by Settlement Officer and suit
was barred by limitation - Plaintiff claiming that order was passed by
@page-SC492
Settlement Officer without notice and without hearing - No material produced by
defendants that order was validly passed - Dismissal of suit on limitation improper -
Matter remanded. (Para 5)

Himinder Lal, for Appellants; J. S. Attri, for Respondents.


Judgement
1. P. SATHASIVAM, J.:-The plaintiffs who succeeded before the trial Court, lower
appellate Court and lost their suit before the High Court filed the above appeal.
2. Brief facts required for the disposal of this appeal are as follows:
According to the plaintiffs-appellants, they were in possession of the suit land for more
than 40 years. The suit was filed in 1989 for declaration of title on the ground of adverse
possession and for injunction. The trial Court decreed the suit and the appeal filed by the
defendants was dismissed by the first appellate Court. When the second appeal was filed
before the High Court, the High Court, after finding that there is no evidence, remanded
the matter to the trial Court for taking further evidence. The said order of the High Court
was challenged before this Court by way of C.A. No. 1348 of 1999. By judgment dated
8.3.1999, this Court, after recording a finding that the High Court was in error and not
justified in sending the matter back to cure any lacuna in the evidence, set aside the order,
restored Second Appeal No. 304 of 1992 to the file of the High Court and directed it to
dispose of the same afresh on the available evidence.
3. Pursuant to the said direction, the High Court formulated two substantial questions of
law which are as under:
"1. Whether the suit was barred in view of the provisions of Section 163(3) of the H.P.
Land Revenue Act, 1953?
2. Whether the suit of the plaintiff, in the facts of the case that on 2nd April, 1970, an
order of ejectment was passed ordering the ejectment of the plaintiff under Section 163 of
the H.P. Land Revenue Act, 1954, can be said to be within the period of limitation?
While considering the first question, the High Court concluded that inasmuch as an order
of ejectment of the plaintiffs from the land in dispute under Section 163 of the H.P. Land
Revenue Act, 1953 was passed on 2.4.1970 by the Settlement Officer, the suit having
been filed about 19 years after such order is barred by limitation. After arriving at such
conclusion on question No.1, the High Court, without going into the second question or
adverting to the case of the plaintiffs i.e., adverse possession, by order dated 04.01.2000,
allowed the second appeal, set aside the judgments and decrees passed by the trial Court
as well as by the lower appellate Court and dismissed the suit of the plaintiffs as being
time barred. Challenging the said order, the plaintiffs have filed the present appeal.
4. Heard Mr. Himinder Lal, learned counsel appearing for the appellants and Mr. Vivek
Singh Attri, learned counsel appearing for the respondents.
5. We have already extracted the first substantial question of law which relates to
limitation. The High Court proceeded the said issue on the ground that though the order
of ejectment of the plaintiffs from the land in dispute came to be passed by the Settlement
Officer on 2.4.1970 however, the plaintiffs filed the Suit No. 41 of 1989 only on
23.2.1989 before the Senior Sub-Judge, Mandi which was barred by limitation. The High
Court, in arriving at such a conclusion, relied on Section 163 of the H.P. Land Revenue
(Amendment) Act No.15 of 1989. Learned counsel for the respondents herein pointed out
that the order said to have been passed on 2.4.1970 by the Settlement Officer was not
communicated to them. It is also their claim that they were not issued notice or afforded
an opportunity to put forth their case before making an entry/passing an order by the
Settlement Officer. In fact, neither before the Courts below nor before the High Court, the
proceedings dated 2.4.1970 was produced by the defendants. In fact, there is no specific
plea in the written statement as to the limitation and no issue was framed by the trial
Court and no point was determined by the lower appellate Court. We verify the defence
stated in the written statement, issues framed by the trial Court and the points determined
by the lower appellate Court. There is no such plea and issue as to the limitation. Though
it would be open to the parties to the suit to raise the plea of limitation before the High
Court as pointed out earlier, the defendants have not taken any effort to place the alleged
proceedings dated 2.4.1970 of the Settlement Officer. It is not clear how the High Court
arrived at a specific conclusion that suit filed by the plaintiffs was barred by limitation.
Even according to the
@page-SC493
defendants, the entire records relating to the said proceedings were lost. We have already
referred to the fact that such issue was not raised and argued before the Courts below. On
the other hand, the plaintiffs concentrated that they are entitled to decree in respect of the
suit property based on the continuous and uninterrupted possession for over a period of
40 years, that too, to the knowledge of the defendants. In view of the assertion of the
plaintiffs that they were neither heard nor afforded an opportunity in the alleged
proceedings dated 2.4.1970, the onus is on the defendants/Department to place those
relevant record to show that there was valid order by the competent authority. Admittedly,
such record was not called for and no material was placed before the Courts below
including the High Court. On the other hand, based on the acceptable oral and
documentary evidence regarding the claim of adverse possession, the trial Court decreed
the suit which was affirmed by the lower Court. Inasmuch as the High Court dismissed
the suit only on the ground of limitation and not gone into the claim of adverse
possession by plaintiffs, in view of our conclusion and disagreement with the High
Courts decision, we have no other option except to remand the matter once again to the
High Court for disposal of the second appeal afresh in respect of other issue i.e., adverse
possession.
6. Under these circumstances, the judgment and decree dated 4.1.2000 passed by the
High Court of Himachal Pradesh in R.S.A. No. 304 of 1992 is set aside and the matter is
remitted to the High Court to decide the issue relating to adverse possession. However, it
is made clear that except the finding relating to limitation, we have not expressed any
opinion on the conclusion of the courts below relating to the other issues; hence it is for
the High Court to decide the same on merits one way or the other with the available
materials. Inasmuch as the plaintiffs suit is of the year 1989 and even the RSA is of the
year 1992, we request the High Court to dispose of the matter within a period of four
months from the date of receipt of this judgment.
7. The appeal is allowed to the extent mentioned above. No order as to costs.
Order accordingly.
AIR 2008 SUPREME COURT 493 "A. Lewis v. M. T. Ramamurthy"
(From : Karnataka)
Coram : 2 TARUN CHATTERJEE AND P. SATHASIVAM, JJ.
Civil Appeal No. 5066 of 2007 (arising out of SLP (C) No. 19240 of 2006), D/- 31 -10
-2007.
A. Lewis and Anr. v. M. T. Ramamurthy and Ors.
Transfer of Property Act (4 of 1882), S.53A - CONTRACT - DECLARATION OF
TITLE - POSSESSION - Part performance - Benefit of protection under S. 53-A - Not
available to transferee who remains passive - Suit for declaration of title and possession
by purchaser of property - Defendants claiming to have been put in possession in part
performance of earlier agreement for sale - Not intimating their intention to perform their
part of contract - No evidence to show that plaintiff had notice of earlier agreement -
Defendants not entitled to protection of S. 53-A. (Para 6)
S. Balaji, for Appellants; K. K. Mani, C. K. R. Lenin Sekar, K. Narayanan, Mayur R.
Shah, for Respondents.
Judgement
P. SATHASIVAM, J.:-Leave granted.
2. This appeal is directed against the final judgment dated 13.07.2006 passed by the High
Court of Karnataka at Bangalore in R.F.A. Nos. 827 and 718 of 2000 whereby the High
Court dismissed the appeals preferred by the appellants.
BRIEF FACTS:
3. The appellants filed the above appeal seeking declaration of ownership over the suit
property with recovery of possession and mesne profits. The suit property in question is
in respect of two different portions of premises bearing No. 26, Nissan Huts, Austin
Town, Bangalore which originally belonged to Muniyappa, respondent No.3 herein (since
deceased). On 23.12.1982, a registered sale deed was executed by respondent No.3 herein
in favour of respondent No.1. Respondent No.1 issued a notice to respondent No.3 and
the other occupants of the suit property for handing over possession of the suit property.
A reply was sent by counsel on behalf of Defendant No.1 in each suit claiming that the
sale deed pleaded by respondent No. 1 was not genuine and contending that respondent
No.3 had entered into an agreement of sale on 04.10.1982 in respect of the said suit
@page-SC494
property in favour of the appellants herein for a sale consideration of Rs.14,000/-. It was
also stated that an amount of Rs.10,000/- had already been paid as part of sale
consideration and actual possession was also delivered to the said purchasers in part
performance of the agreement to sell. Therefore, Respondent No.1 herein filed two suits
bearing O.S. No. 10607 of 1985 and O.S. No. 10609 of 1985 on the file of the XXVIII
Additional City Civil and Sessions Judge, Mayo Hall at Bangalore claiming that he is the
absolute owner of the suit schedule property and for possession from first defendant,
respondent No.2 herein, along with mesne profits. By common judgment, the learned trial
Judge decreed the suits declaring respondent No.1 herein, as the owner of the suit
schedule property and directed the appellants herein to deliver possession of the suit
property to the plaintiff within six months from the date of the receipt of the order and
also directed that the plaintiff is entitled to mesne profits from 10.7.1985 and a further
direction was also given to initiate an enquiry for determination of mesne profits under
Order XX Rule 12 C.P.C. Challenging the said judgment, defendant Nos. 3 and 4,
appellants herein, filed R.F.A. Nos. 827 and 718 of 2000 and defendant No.2, respondent
No.3 herein, filed R.F.A. Nos. 730 and 830 of 2000 before the High Court. The High
Court dismissed all the four appeals with costs and directed defendant Nos. 1, 3 and 4 to
hand over vacant possession of the suit property within six months. Aggrieved by the
judgment in R.F.A. Nos. 827 and 718 of 2000, this appeal has been preferred by way of
special leave before this Court.
4. We heard Mr. S. Balaji, learned counsel for the appellants and Mr. K.K. Mani, learned
counsel for the respondents and perused all the relevant materials and records filed in this
Court.
5. Learned counsel for the appellants mainly contended that the High Court committed an
error in dismissing their appeals on the ground that Ex.D-1 Agreement to sell dated
04.10.1982 is antedated. According to him, in view of the fact that it was not the case of
the contesting respondents herein, the High Court ought not to have reversed the finding
of the fact in the appeal. On the other hand, learned counsel appearing for the contesting
respondents, after taking us through the entire materials including the pleadings of both
the parties, conclusion of the trial Judge and the decision arrived at by the High Court,
submitted that there is no valid ground for interference. In view of the assertion of the
counsel for the appellants, we have carefully perused the judgment of the High Court as
well as the other materials. It is true that the High Court in para 21 of its judgment has
concluded "considering these factors, I am of the opinion that this document, Ex.D-1
must have come into existence subsequent to the sale deed to defeat the rights of the
plaintiff and, therefore, Additional issue No.5 in each case has to be answered in the
negative." First of all, there is no specific finding either by the trial Court or by the High
Court to the effect that Ex.D-1 Agreement to sell is antedated. On the other hand, the trial
Court as well as the High Court considered the claim of the plaintiffs based on Ex.P-1
dated 23.12.1982 as well as the defence of the contesting defendants based on agreement
of sale dated 04.10.1982.
6. As rightly pointed out by the High Court, the existence of right to claim protection
under Section 53-A of the Transfer of Property Act would not be available if the
transferee just kept quiet and remained passive without taking effective steps. Further, he
must also perform his part of the contract and convey his willingness. On the other hand,
the factual finding is that there was no intimation by defendant Nos. 3 and 4 to perform
their part of contract to claim protection of Section 53-A of the Transfer of Property Act.
Likewise, as rightly concluded by the courts below, there is no material to show that the
plaintiff had notice of agreement of sale Ex.D-1 in favour of defendant Nos. 3 and 4. The
conclusion of the High Court that defendant Nos. 3 and 4 or even defendant No.1 who
claims through them are not entitled to protection of Section 53-A of the Transfer of
Property Act is acceptable and the argument contrary to the said conclusion is liable to be
rejected.
7. As rightly pointed out, the sale deed executed by the second defendant in favour of the
plaintiff is not only a registered document but the plaintiff apart from offering his
evidence, has also examined the scribe of the document and both the courts below rightly
found that Ex.P-1 has been proved in accordance with law. It is not in dispute that Ex.D-1
is an unregistered document. It is also not in dispute that the said document has not been
produced until
@page-SC495
defendant Nos. 3 and 4 were impleaded. Considering the evidence and of the claim that
second defendant was in need of money for discharging his antecedent debts and for
family maintenance etc., both the Courts disbelieved his version and rightly concluded it
was highly doubtful that the agreement of sale had been executed by him prior to the sale
deed. Only after analyzing all the above-mentioned material aspects, the High Court has
rightly concluded that Ex.D-1 must have come into existence subsequent to the sale deed
in order to defeat the rights of the plaintiff. If the High Court dismissed the appeal based
on the only reason as argued by learned counsel for the appellants, it would be
appropriate to remit it to the High Court for fresh disposal on all aspects. However, as
observed earlier, the High Court as the first appellate court analyzed the entire evidence
and concluded that the plaintiff had proved that the sale deed had been executed in his
favour by Defendant No.2 under Ex.P-1, consequently confirmed the decision of the trial
Court in decreeing the suits as prayed for. We are in entire agreement with the said
conclusion and unable to accept the argument of learned counsel for the appellants.
8. In the light of the above discussion, the appeal fails and is dismissed. No costs.
Appeal dismissed.
AIR 2008 SUPREME COURT 495 "People's Union For Civil Liberties v. Union of
India"
Coram : 2 Dr. A. PASAYAT AND S. H. KAPADIA, JJ.
I. A. Nos. 34, 35, 37, 40, 49, 54, 58-62, 77 in W. P. (C) No. 1966 of 2001, D/- 20 -11
-2007.
People's Union For Civil Liberties v. Union of India and Ors.
Maternity Benefit Act (53 of 1961), S.28 - MATERNITY BENEFIT - SUPREME
COURT - EQUALITY IN PUBLIC EMPLOYMENT - National Maternity Benefit
Scheme - Modification of and introduction of new Scheme called Janani Suraksha Yojna
- Centre providing 100% funds - Some States not utilising such funds to fullest extent -
Supreme Court directed Central Govt. to ensure that such funds is not utilised for any
other purpose etc. - Central Govt. directed to make amendments to ensure that benefit of
scheme is not given against concept of family planning and to mothers who have married
despite being below prohibited age.
Constitution of India, Art.16. (Paras 14, 15)

Mohan Parasaran, ASG, Mrs. Jayshree Anand, Addl. AG, Colin Gonsalves, T. S. Doabia,
Jaideep Gupta, Anil Diwan, Sr. Advocates, Vipin M. Benjamin, Jai Singh, Ms. Jyoti
Mendiratta, J. S. Attri, Vivek Singh Attri, D. S. Mahra, Ms. Sushma Suri, B. K. Sood, R.
C. Kathia, Kh. Nobin Singh, Tarun Jamwal, Dr. Kailash Chand, Venkateshwara Rao
Anumolu, Ravindra Keshavrao Adsure, Gautam Godara, K. K. Mahalik, Ms. Sunita
Sharma, Jana Kalyan Das, Ms. Hemantika Wahi, Ms. Shivangi, B. V. Balaram Das, Ms.
Indra Sawhney, Aruneshwar Gupta, Naveen Kumar Singh, Shashwat Gupta, Riku Sarma
(for M/s. Corporate Law Group), Ms. Rachana Srivastava, S. V. Deshpande, Manoj
Saxena, Rajnish Kr. Singh, Rahul Shukla, T. V. George, Ms. Kamini Jaiswal, Ranjan
Mukherjee, S. C. Ghosh, V. G. Pragasam, S. Joseph Aristotle, S. Prabu
Ramasubramanian, Jatinder Kumar Bhatia, Ravi Prakash Mehrotra, Gopal Singh, B. S.
Banthia, Tara Chandra Sharma, Ms. Neelam Sharma, B. B. Singh, Kumar Rajesh Singh,
Anil Shrivastava, Ritu Raj, Gopal Prasad, Prakash Shrivastava, Ramesh Babu M. R., Mrs.
D. Bharathi Reddy, Amit Kr. Chawla, Sanjay R. Hegde, U. Hazarika, Satya Mitra, Ms.
Sumita Hazarika, R. K. Maheshwari, Ms. A. Subhashini, Uday B. Dube, Kuldip Singh,
Prashant Kumar, Vishwajit Singh, Anis Suhrawardy, Ashok Bhan, S. W. A. Qadri, Jubair
Ahmed Khan, K. V. Mohan, Manish Sharma, Ms. Suparna Srivastava, Ms. Nidhi
Minocha, Rajesh Srivastava, Anuvrat Sharma, K. N. Madhusoodhanan, R. Sathish, R. C.
Kaushik, Pradeep Misra, G. Prakash, Ms. Malvika Trivedi, Anil Kumar Jha, Gopal Singh,
Sib Shankar Mishra, A. Mariarputham, Mrs. Aruna Mathur (for Arputham Aruna and
Co.), for appearing parties.
Judgement
1. Dr. ARIJIT PASAYAT, J. :-By this order two IAs. No.37 of 2004 and No.54 of 2005
stand disposed of. IA No.37 of 2004 is an application by the Union of India for
permission to modify the National Maternity Benefit Scheme (in short NMBS) and to
introduce a new scheme called the Janani Suraksha Yojana (in short JSY). IA No.54 of
2005 is an application by the petitioner questioning legality of the discontinuation of the
benefit under the NMBS due to introduction of JSY. By order dated 27.4.2004 this Court
directed as follows:
@page-SC496
"No Scheme...in particular........... National Maternity Benefit Scheme shall be
discontinued or restricted in any way without prior approval of the Court."
2. Again by order dated 9.5.2005 this Court directed as follows:
"By LA 37, permission is sought to modify The National Maternity Benefit Scheme
(NMBC) and to introduce a new scheme namely Janani Suraksha Yojana (JSY). Whereas
in IA 54, the prayer is that the Scheme should not be modified by reducing, abridging or
qualifying in any way the social assistance entitlements created under the original scheme
of NMBS for expecting BPL mothers, including rash entitlement of Rs.500/- provided
therein. We have requested learned Additional Solicitor General to place on record further
material in the form of affidavit to effectively implement the new Scheme sought to be
introduced. The further material shall include the approximate distance of Public Health
Centre from the residential complexes and the facility of transportation etc. The
Commissioner shall also examine the matter in depth and file a report. The response to
the application may be filed within eight weeks. Meanwhile, the existing National
Maternity Benefit Scheme will continue."
3. The Government set a numerical ceiling of 57.5 lakh beneficiaries as the annual target
for NMBS. However, the number of beneficiaries under JSY in 2006-07 was only 26.2
lakh i.e. 45.5% and in the year 2005-06 this was as low as 5.7 lakh i.e. 10%. While there
has been an improvement in the last one year, the coverage under this scheme is still way
below the target number of women to be covered by the NMBS.
4. According to the Union of India the JSY was introduced to put a premium on the
willingness of poor women to go in for institutional delivery instead of home delivery.
But it was recognized that in States with lower institutional delivery rates, one of the
reasons for low performance have been lesser availabilities of facilities in the Health
Centres, which act as disincentive for the poor illiterate women to seek the services.
5. Pursuant to the order of this Court dated 9.5.2005 the Commissioner had prepared a
report.
6. After discussions with the Commissioner appointed by this Court, senior officials, the
Central Government took a decision to modify the JSY Scheme to continue benefits of
NMBS and also to improve upon such benefits for non-institutional delivery, where the
woman chooses to deliver her baby at home. In this connection, a letter dated 13.7.2006
was written to the Commissioner by the Secretary, Health and Family Welfare under the
amended JSY. The Low Performing States and High Performing States were defined as
follows:
"4.1 The scheme focuses on the poor pregnant woman with special dispensation for
States having low institutional delivery rates namely the States of Uttar Pradesh,
Uttaranchal, Bihar, Jharkhand, Madhya Pradesh, Chhattisgarh, Assam, Rajasthan, Orissa
and Jammu and Kashmir. While these States have been named as Low Performing States
(LPS), the remaining States have been named as High Performing States (HPS)."
7. The table below gives details of the number of beneficiaries under JSY (all these would
have received Rs.500/- under NMBS irrespective of place of delivery) vis-a-vis the
annual targets set by the Government of India for NMBS.
Percentage of Eligible Beneficiaries Covered Under NMBS

@page-SC497

8. The scheme as the details above go to show has virtually not taken off in many States.
Delhi has given the benefit under the NMBS to only 20 women in 2006-07, while in
Chandigarh the number of beneficiaries is 0. In Sikkim, Meghalaya, Himachal Pradesh,
Uttar Pradesh, Haryana and Dadra and Nagar Haveli less than even 10% of the eligible
beneficiaries have been covered under the NMBS. Except for the States of Andhra
Pradesh, Jammu and Kashmir, Rajasthan, Madhya Pradesh, Assam, Orissa and Mizoram
where more than 75% of the eligible beneficiaries seem to have been reached out to, the
performance of this scheme has been very poor in all other States.
Indicated below are percentage of Home delivery figures
@page-SC498

9. In the States of Madhya Pradesh, Haryana, Manipur and Delhi there are almost no JSY
beneficiaries who had a home delivery. This indicates that in these States the scheme's
focus continues to be only on institutional deliveries and not all deliveries. Even in the
States of Assam, Rajasthan, Meghalaya, Orissa and Chhattisgarh the JSY has been
disproportionately given to only those who have had institutional deliveries.
10. At this juncture, the financial performance needs to be noted.
11. The Janani Suraksha Yojana is a centrally-sponsored scheme with the Centre
providing 100% of the funds. Some States e.g. Andhra Pradesh make their own
contribution thereby increasing the amount of cash assistance for institutional deliveries.
Tamil Nadu has introduced a separate scheme for providing mothers with Rs.1000/? per
month for six months i.e. three months prior to the delivery and three months after. Given
below are the details of allocation and utilization of the funds provided by the Central
Government.
12. Out of the funds provided for JSY for 2006-07, about 71.2% of the funds allocated
have been utilized in the year 2006-07.
Utilization of funds allocated by JSY

13. Looking at the State-wise break-up it is seen that States like Delhi, Nagaland and
Arunachal Pradesh, and Union Territories of Chandigarh and Daman and Diu have not at
all utilized the funds allocated to them for the purpose of JSY. Among other States,
Manipur, Jharkhand and Haryana utilized less than 20% of the funds released to them.
Only 10 States spent more than 70% of the funds allocated to them under JSY.
14. At the time of hearing of the applications, learned counsel for the petitioner and the
Union of India highlighted various
@page-SC499
aspects. Considering the submissions and the material data placed on record we direct as
follows:-
(a) The Union of India and all the State Governments and the Union Territories shall (i)
continue with the NMBS and (ii) ensure
@page-SC500
that all BPL pregnant women get cash assistance 8-12 weeks prior to the delivery.
(b) The amount shall be Rs.500/- per birth irrespective of number of children and the age
of the woman.
(c) The Union of India, State Governments and the Union Territories shall file affidavits
within 8 weeks from today indicating the total number of births in the State, number of
eligible BPL women who have received the benefits, number of BPL women who had
home/non-institutional deliveries and have received the benefit, number of BPL women
who had institutional deliveries and have received the benefit.
(d) The total number of resources allocated and utilized for the period 2000-2006.
(e) All concerned Governments are directed to regularly advertise the revised scheme so
that the intended beneficiaries can become aware of the scheme.
(f) The Central Government shall ensure that the money earmarked for the scheme is not
utilized for any other purpose. The mere insistence on utilization certificate may not yield
the expected result.
(g) It shall be the duty of all the concerned to ensure that the benefits of the scheme reach
the intended beneficiaries. In case it is noticed that there is any diversion of the funds
allocated for the scheme, such stringent action as is called for shall be taken against the
erring officials responsible for diversion of the funds.
15. At this juncture it would be necessary to take note of certain connected issues which
have relevance. It seems from the scheme that irrespective of number of children, the
beneficiaries are given the benefit. This in a way goes against the concept of family
planning which is intended to curb the population growth. Further the age of the mother
is a relevant factor because women below a particular age are prohibited from legally
getting married. The Union of India shall consider this aspect while considering the
desirability of the continuation of the scheme in the present form. After considering the
aforesaid aspects and if need be, necessary amendments may be made.
16. The IAs are accordingly disposed of.
Order accordingly.
AIR 2008 SUPREME COURT 500 "Santosh v. Saraswathibai"
(From : AIR 2006 Karnataka 85)
Coram : 2 S. B. SINHA AND H. S. BEDI, JJ.
Civil Appeal No. 5321 of 2007 (arising out of S.L.P. (C) No. 5437 of 2006), D/- 20 -11
-2007.
Santosh and Ors. v. Saraswathibai and Anr.
Hindu Succession Act (30 of 1956), S.14(1) - SUCCESSION - POSSESSION -
Enlargement of estate of female Hindu - Property given to wife by her deceased-husband
for maintenance - Her possession to suit property accepted by defendants, other heir of
deceased in consent decree passed in previous suit - Pre-existing right of wife for
maintenance getting thus crystallised by consent decree - She cannot be divested of her
absolute ownership by invoking S. 14(2). (Paras 13, 17)
Cases Referred : Chronological Paras
2006 (13) Scale 408 12
2005 AIR SCW 2203 12
1996 AIR SCW 229 : AIR 1996 SC 855 (Foll.) 16
1995 AIR SCW 3885 : AIR 1996 SC 172 16
AIR 1977 SC 1944 (Foll.) 15, 16
AIR 1959 SC 577 (Rel. on) 9, 12
AIR 1957 Cal 557 12
C. G. Solshe and Vinesh Solshe, for Appellants; Basava Prabhu S. Patil, V. N.
Raghupathy, B.Subrahmanya Prasad, Narayan P. Kengasur and B. B. Patil, for
Respondents.
Judgement
1. S. B. SINHA, J. :-Leave granted
2

. Application of Section 14(1) of the Hindu Succession Act, 1956 (hereinafter referred to
as 'the Act') calls for consideration in this appeal which arises out of a judgment and order
dated 15th November, 2005 passed by a learned Single Judge of the Karnataka High
Court at Bangalore in Regular Second Appeal No. 276 of 2003. Reported in AIR 2006
Kant 85

3. The short factual matrix involved may be noticed at the threshold.


4. For the said purpose, we may notice the genealogy of the family.

5. Original Suit bearing No. 34 of 1964 was filed by Sundrabai, first wife of Trimukhrao
(respondent No.1 herein) against Mallikarjun (since deceased) and Kashibai (defendant
No.4 therein). A
@page-SC501
consent decree was passed in that suit, the relevant clauses whereof were as under :-
"1. The plaintiff and the defendant No.2 are the wives of deceased Trimbakrao. The
defendant No.1 is the son of Trimbakrao. The plaintiff Sundrabai and Mallikarjun and his
mother Kashibai resides separately.
2. That as per compromise the land Sy. No.73 Kh. No.145 to the extent of measuring 6
acres 33 guntas. R.A. Rs.9.00 situated at Khandala, on north side bounded in the east the
land of Shivalingappa Biradar in the west public way. In the north the land of
Shankarappa in the south remain land Sy. No.73, was given to the plaintiff for her
maintenance. She is in possession of that portion of land since 5/6 years. The defendants
will not interfere and obstruct in the peaceful possession of that portion of land which
was given to the plaintiff for her maintenance, till her death. The defendant Nos.1 and 2
will remain in possession of remaining half portion of land Sy.No.73 on south side as
owner. The plaintiff will not alienate (through gift or sale) land which was given for her
maintenance.
3. That after the death of the plaintiff the land given to her will be reversed to the
defendant Nos.1 and 2. The parties will bear their costs of the cost.""
6. Mallikarjun died in the year 1987. Sundrabai died in 1992. Respondents herein who
are heirs and legal representatives of Sundrabai filed Original Suit No.210 of 1993
claiming the said land admeasuring 6 Acres 33 guntas from the Northern side of Survey
No. 73 on the ground that she had become owner thereof in terms of Section 14(1) of the
Act. Appellants who were arrayed in the suit as defendants inter alia contended that
Sundrabai was not the absolute owner of the said property having been allotted to her by
reason of the consent decree. It was furthermore contended that the land in suit was
allotted to her by way of maintenance which she could keep in possession only during her
lifetime and, therefore, Section 14(2) of the Act and not Section 14(1) of the Act would
apply.
7. The said suit was dismissed on the premise that the plaintiffs were not the owners and
were not in possession of the suit property.
8. On an appeal having been preferred thereagainst, the Principal District Judge Bidar by
his judgment and order dated 2nd January, 2003 allowed the said appeal opining that
Section 14(1) of the Act being
@page-SC502
applicable, the plaintiffs became the owners of the suit property. By reason of the
impugned judgment the second appeal preferred by the appellants has been dismissed.
9

. Mr. Solashe, learned counsel appearing on behalf of the appellants, in support of this
appeal inter alia would submit that the High Court committed a serious error of law in so
far as it failed to take into consideration that Sundrabai on partition could not have been
allotted 6 Acres 33 guntas of land and in that view of the matter, although the land which
could fall in her share would be covered by Section 14(1) of the Act, but according to the
learned counsel the share of Sundrabai in the joint family being 7/24 in the total land of
joint family being 12 acres 33 guntas, share of Sundrabai would come only to 3 acres 29
guntas. It was submitted that the word "possessed" contained in Section 14 (1) of the Act
must be construed to mean ownership as has been held by this Court in [1959] Supp 1
SCR 968, Gummala-pura Taggina Matada Kotturuswami vs. Setra Veeravva and others.
AIR 1959 SC 577

10. Mr. Patil, learned counsel appearing on behalf of the respondents, on the other hand,
would support the judgment. It was moreover contended that there is nothing on record to
show that the total agricultural lands belonging to the joint family was only to the extent
of 12 acres 33 guntas. In fact, Mr. Patil would urge that there were other properties of the
joint family besides, the suit land.
11. Sub-sections (1) and (2) of Section 14 of the Act reads as under:-
"14. Property of a female Hindu to be her absolute property- (1) Any property possessed
by a female Hindu, whether acquired before or after the commencement of this Act, shall
be held by her as full owner thereof and not as a limited owner.
Explanation.- In this sub-section, property includes both movable and immovable
property acquired by a female Hindu by inheritance or devise, or at a partition, or in lieu
of maintenance or arrears of maintenance, or by gift from any person, whether a relative
or not, before, at or after the marriage, or by her own skill or exertion, or by purchase or
by prescription, or in any other manner whatsoever, and also any such property held by
her as stridhana immediately before the commencement of this Act.
(2) Nothing contained in sub-section (1) shall apply to any property acquired by way of
gift or under a will or any other instrument or under a decree or order of a civil Court or
under an award where the terms of the gift, will or other instrument or the decree, order
or award prescribe a restricted estate in such property."
12
. In Gummalapura Taggina Matada (supra) on which reliance has been placed by the
learned counsel, this Court held that Section 14(1) of the Act has wide application. It not
only takes within its sweep a land which was not only in possession of the female Hindu
but also covers the land over which she has a right to possess stating:-- AIR 1959 SC
577, Para 11

"Of course, possession referred to in section 14 need not be actual physical possession or
personal occupation of the property by the Hindu female but may be possession in law.
The possession of a licensee, lessee or a mortgagee from the female owner or the
possession of a guardian or a trustee or an agent of the female owner would be her
possession for the purpose of section 14. The word "possessed" is used in section 14 in a
broad sense and in the context possession means the state of owning or having in one's
hands or power. It includes possession by receipt of rents and profits. The learned Judges
expressed the view that even if a trespasser were in possession of the land belonging to a
female owner, it might conceivably be regarded as being in possession of the female
owner, provided the trespasser had not perfected his title. We do not think that it is
necessary in the present case to go to the extent to which the learned Judges went. It is
sufficient to say that "possessed" in section 14 is used in a broad sense and in the context
means the state of owning or having in one's hand or power. In the case of Gostha Behari
v. Haridas Samanta, P. N. Mookherjee, J., expressed his opinion as to the meaning of the
words "any property possessed by a female Hindu" in the following words :- AIR
1957 Cal 557 at P. 559

"The opening words in "property possessed by a female Hindu" obviously mean that to
come within the purview of the section the property must be in possession of the female
concerned at the date of the commencement of the Act. They clearly contemplate the
female's possession when the Act came into force. That possession might have
@page-SC503
been either actual or constructive or in any form recognised by law, but unless the female
Hindu, whose limited estate in the disputed property is claimed to have been transformed
into absolute estate under this particular section, was at least in such possession, taking
the word "possession" in its widest connotation, when the Act came into force, the section
would not apply."

(See also (2005) 5 SCC 390, Shakuntla Devi vs. Kamla and others and 2006 (13) SCALE
408, Chandrika Singh (D) by L.Rs. vs. Sarjug Singh and another. 2005 AIR SCW 2203

13. Sundrabai's possession in respect of 6 acres 33 guntas of land even prior to the
institution of the suit has been accepted in the said consent decree. Appellants herein
undertook not to interfere in her peaceful possession thereover. Admittedly after the death
of Trimukhrao who died after coming into force of the Act, Sundrabai became one of the
co-owners of the property being one of his wives who had half share in the joint
properties. Succession thereof was governed by Sections 6, 8 and 12 of the Act. It is,
therefore, not a case where she had no right to possess the said land. If she had a right to
possess the said land as a co-owner, the question of divesting her of the said right by
invoking sub-section (2) of Section 14 of the Act would not arise.
14. The stipulations made in the consent decree dated 20th July, 1964 must thus be
construed having regard to the well settled legal position. It is now a well settled principle
of law that in considering a deed, the legal position obtaining in this behalf should be
kept in mind.
15

. Legal position in regard to the right of a female Hindu was laid down by this Court in
(1977) 3 SCC 99, V. Tulasamma and others vs. Sesha Reddy (dead) by L.Rs., wherein
the legal consequences were summarized as under :-AIR 1977 SC 1944

"(1) The Hindu female's right to maintenance is not an empty formality or an illusory
claim being conceded as a matter of grace and generosity, but is a tangible right against
property which flows from the spiritual relationship between the husband and the wife
and is recognised and enjoined by pure Shastric Hindu Law and has been strongly
stressed even by the earlier Hindu jurists starting from Yajnavalkya to Manu. Such a right
may not be a right to property but it is a right against property and the husband has a
personal obligation to maintain his wife and if he or the family has property, the female
has the legal right to be maintained therefrom. If a charge is created for the maintenance
of a female, the said right becomes a legally enforceable one. At any rate, even without a
charge the claim for maintenance is doubtless a pre-existing right so that any transfer
declaring or recognising such a right does not confer any new title but merely endorses or
confirms the pre-existing rights.
(2) Section 14(1) and the Explanation thereto have been couched in the widest possible
terms and must be liberally construed in favour of the females so as to advance the object
of the 1956 Act and promote the socio-economic ends sought to be achieved by this long
needed legislation.
(3) Sub-section (2) of Section 14 is in the nature of a proviso and has a field of its own
without interfering with the operation of Section 14(1) materially. The proviso should not
be construed in a manner so as to destroy the effect of the main provision or the
protection granted by Section 14(1) or in a way so as to become totally inconsistent with
the main provision.
(4) Sub-section (2) of Section 14 applies to instruments, decrees, awards, gifts, etc. which
create independent and new titles in favour of the females for the first time and has no
application where the instrument concerned merely seeks to confirm, endorse, declare or
recognise pre-existing rights. In such cases a restricted estate in favour of a female is
legally permissible and Section 14(1) will not operate in this sphere. Where, however, an
instrument merely declares or recognises a pre-existing right, such as a claim to
maintenance or partition or share to which the female is entitled, the sub-section has
absolutely no application and the females limited interest would automatically be
enlarged into an absolute one by force of Section 14(1) and the restrictions placed, if any,
under the document would have to be ignored. Thus where a property is allotted or
transferred to a female in lieu of maintenance or a share at partition, the instrument is
taken out of the ambit of sub-section (2) and would be governed by Section 14(1) despite
any restrictions placed on the powers of the transferee.
(5) The use of express terms like
@page-SC504
'property acquired by a female Hindu at a partition', 'or in lieu of maintenance', or 'arrears
of maintenance', etc. in the Explanation to Section 14(1) clearly makes sub-section (2)
inapplicable to these categories which have been expressly excepted from the operation
of sub-section (2).
(6) The words 'possessed by' used by the Legislature in Section 14(1) are of the widest
possible amplitude and include the state of owning a property even though the owner is
not in actual or physical possession of the same. Thus, where a widow gets a share in the
property under a preliminary decree before or at the time when the 1956 Act had been
passed but had not been given actual possession under a final decree, the property would
be deemed to be possessed by her and by force of Section 14(1) she would get absolute
interest in the property. It is equally well settled that the possession of the widow,
however, must be under some vestige of a claim, right or title, because the section does
not contemplate the possession of any rank trespasser without any right or title.
(7) That the words 'restricted estate' used in Section 14(2) are wider than limited interest
as indicated in Section 14(1) and they include not only limited interest, but also any other
kind of limitation that may be placed on the transferee."
16

. In Nazar Singh and others vs. Jagjit Kaur and others : (1996) 1 SCC 35, this Court
following Tulasamma (supra) held as under :- 1996 AIR SCW 229, Para 8
AIR 1977 SC 1944

"The principles enunciated in this decision have been reiterated in a number of decisions
later but have never been departed from. According to this decision, sub-section (2) is
confined to cases where property is acquired by a female Hindu for the first time as a
grant without any pre-existing right under a gift, will, instrument, decree, order or award,
the terms of which prescribe a restricted estate in the property. It has also been held that
where the property is acquired by a Hindu female in lieu of right of maintenance inter
alia, it is in virtue of a pre-existing right and such an acquisition would not be within the
scope and ambit of sub-section (2) even if the instrument, decree, order or award allotting
the property to her prescribes a restricted estate in the property. Applying this principle, it
must be held that the suit lands, which were given to Harmel Kaur by Gurdial Singh in
lieu of her maintenance, were held by Harmel Kaur as full owner thereof and not as a
limited owner notwithstanding the several restrictive covenants accompanying the grant.
(Also see the recent decision of this Court in Mangat Mal v. Punni Devi, where a right to
residence in a house property was held to attract sub-section (1) of Section 14
notwithstanding the fact that the grant expressly conferred only a limited estate upon her.)
According to sub-section (1), where any property is given to a female Hindu in lieu of her
maintenance before the commencement of the Hindu Succession Act, such property
becomes the absolute property of such female Hindu on the commencement of the Act
provided the said property was "possessed" by her. Where, however, the property is given
to a female Hindu towards her maintenance after the commencement of the Act, she
becomes the absolute owner thereof the moment she is placed in possession of the said
property (unless, of course, she is already in possession) notwithstanding the limitations
and restrictions contained in the instrument, grant or award whereunder the property is
given to her. This proposition follows from the words in sub-section (1), which insofar as
is relevant read : "Any property possessed by a female Hindu....after the commencement
of this Act shall be held by her as full owner and not as a limited owner". In other words,
though the instrument, grant, award or deed creates a limited estate or a restricted estate,
as the case may be, it stands transformed into an absolute estate provided such property is
given to a female Hindu in lieu of maintenance and is placed in her possession. So far as
the expression "possessed" is concerned, it too has been the subject-matter of
interpretation by several decisions of this Court to which it is not necessary to refer for
the purpose of this case. 1995 AIR SCW 3885

(Emphasis supplied)
17. In view of the aforementioned binding authoritative pronouncements of this Court,
we are of the opinion that the pre-existing right of Sundrabai was crystallized by reason
of the said consent decree. Furthermore there is nothing on record to show that 12 acres
33 guntas of land was the only property belonging to the joint family and thus, she had
been granted more lands to which she was not entitled to.
18. In view of the matter, we do not find
@page-SC505
any infirmity in the impugned judgment. The appeal is accordingly dismissed with costs.
Counsels fees assessed at Rs.10,000/-.
Appeal dismissed.
AIR 2008 SUPREME COURT 505 "D. Sailu v. State of Andhra Pradesh"
(From : Andhra Pradesh)*
Coram : 2 Dr. A. PASAYAT AND AFTAB ALAM, JJ.
Criminal Appeal No. 1592 of 2007(arising out of SLP (Cri.) No. 3627 of 2006), D/- 20
-11 -2007.
D. Sailu v. State of A.P.
(A) Evidence Act (1 of 1872), S.3 - EVIDENCE - WITNESS - Interested witness -
Testimony of - Reliability - Relationship is not a factor to affect credibility of a witness -
It is more often than not that a relation would not conceal actual culprit and make
allegations against an innocent person - Plea that witness being a close relative and
partisan witness should not be relied upon - Is, therefore, untenable.
2002 AIR SCW 4271; 2004 AIR SCW 7376; 2007 (1) SCC 699; AIR 1953 SC 364; AIR
1974 SC 276; AIR 1957 SC 614; AIR 1965 SC 202; AIR 1973 SC 2407; 2002 (3) SCC
76, Relied on. (Paras 11, 14)
(B) Evidence Act (1 of 1872), S.60, S.45 - EVIDENCE - WITNESS - Ocular evidence
vis-a-vis medical evidence - Evaluation of - Medical evidence at variance with ocular
evidence - It would be erroneous to accord undue primacy to hypothetical answers of
medical witnesses to exclude eye-witnesses' account which has to be tested
independently.
2003 AIR SCW 3688, Relied on. (Para 19)
(C) Penal Code (45 of 1860), S.300, Exception 1 and Exception 4 - MURDER - Murder
or culpable homicide - Distinction between First and Fourth Exception to S. 300, stated.
The Fourth Exception to Section 300, IPC covers acts done in a sudden fight. The said
Exception deals with a case of prosecution not covered by the First Exception, after
which its place would have been more appropriate. The Exception is founded upon the
same principle, for, in both there is absence of premeditation. But, while in the case of
Exception 1 there is total deprivation of self-control, in case of Exception 4, there is only
that heat of passion which clouds men's sober reason and urges them to deeds which they
would not otherwise do. There is provocation in Exception 4 as in Exception 1; but the
injury done is not the direct consequence of that provocation. For the application of
Exception 4, it is not sufficient to show that there was a sudden quarrel and that there was
no premeditation. It must further be shown that the offender has not taken undue
advantage or acted in cruel or unusual manner. The expression "undue advantage" as used
in the provision means "unfair advantage".
2006 AIR SCW 1678, Relied on.
(Para 26)
(D) Penal Code (45 of 1860), S.300, S.304, Part I - MURDER - CULPABLE
HOMICIDE - Murder or culpable homicide - Proof - Sudden quarrel - Accused allegedly
stabbed deceased with knife - There was evidence of eye-witnesses - Medical evidence
was not contrary to ocular evidence as claimed - Doctor had clearly stated as to under
what circumstances lacerated injury can be caused by knife - Occurrence took place in
course of sudden quarrel and verbal altercation - Conviction of accused under S. 300,
therefore, altered to one under S. 304, Part I.
Cri. A. No. 287 of 2003, D/- 15-7-2005 (A. P.), Reversed. (Paras 22, 28)
Cases Referred : Chronological Paras
(2007) 1 SCC 699 (Rel. on) 17
2006 AIR SCW 1678 : 2006 Cri LJ 2111 (Rel. on) 27
2004 AIR SCW 7376 : AIR 2005 SC 1460 (Rel. on) 17
2003 AIR SCW 3688 : AIR 2003 SC 2978 : 2003 Cri LJ 3705 (Rel. on) 21
2002 AIR SCW 4271 : AIR 2002 SC 3633 : 2003 Cri LJ 41 (Rel. on) 6
(2002) 3 SCC 76 (Rel. on) 16
AIR 1974 SC 276 : 1974 Cri LJ 331 (Rel. on) 13
AIR 1973 SC 2407 : 1973 Cri LJ 1589 (Rel. on) 16
AIR 1965 SC 202 : 1965 (1) Cri LJ 226 (Rel. on) 15
AIR 1957 SC 614 : 1957 Cri LJ 1000 (Rel. on) 13
AIR 1953 SC 364 (Rel. on) 12, 14
AIR 1952 SC 54 : 1952 Cri LJ 547 14
@page-SC506

Ajay Sharma, for Appellant; Mrs. D. Bharathi Reddy, for Respondent.


* Cri. A. No. 287 of 2003, D/- 15-7-2005 (A. P.)
Judgement
1. Dr. ARIJIT PASAYAT, J.:- Leave granted.
2. Challenge in this appeal is to the order passed by a Division Bench of the Andhra
Pradesh High Court upholding the conviction of the appellant (hereinafter referred to as
'Accused No.1') for offence punishable under Section 302 of the Indian Penal Code, 1860
(in short the 'IPC') and sentence of imprisonment for life and fine of Rs.200/- with default
stipulation.
3. Background facts as projected by the prosecution are as follows:
Accused persons D. Sailu, Ramaiah, D. Pentamma and Yadaiah are described as A-1, A-
2, A-3 and A-4. Samuel (hereinafter referred to as the deceased) was the husband of
Shantamma (P.W.1). A-1 is the son of the elder brother of the deceased, A-2 is the father
of A-1, A-3 is the wife of A-2 and A-4 is the younger brother of A?1. The deceased and
the accused were not on good terms as they quarrelled with each other over bore water
for the fields. Fifteen days prior to the date of incident, the accused and the deceased
quarrelled with each other. On the date of incident i.e. 24.11.1999 at about 8.00 p.m., A-1
asked the deceased as to why he (deceased) scolded the mother of A-1. The deceased told
him that he did not scold his mother. Then P.W.1, the wife of the deceased, caught hold of
the hands of A-1. A-4, the younger brother of A-1, came and attempted to beat the
deceased. P.W.1 pushed the younger brother of A-1. A-2 beat P.W.1 with hands and A-1
stabbed the deceased at the instigation of A-2 with a knife on the left side of the stomach.
As a result, the deceased fell down. A-3 also came there along with A-2 and beat P.W.1.
Thereafter, the deceased was taken to the Sangareddy Hospital in an auto.
4. The Village Administrative Officer gave Ex.P8 report to P.W.14, who registered the
case in Cr.No. 82 of 1999 under Section 302 read with 34, IPC against Al to A4. P.W.15
took up investigation, visited the scene of offence and conducted scene of offence
panchanama in the presence of P.W.10 and another and seized controlled earth from the
scene. Thereafter, he proceeded to Government Hospital and held inquest on the dead
body of the deceased in the presence of P.W.12 and others. He seized blood-stained
clothes from the body of the deceased. On inquest it was found that the deceased died as
a result of the injuries sustained by him. P.W.8 is the Doctor, who conducted the autopsy,
opined that the deceased died due to shock and hemorrhage due to injury to vital organ.
On 13.11.1999, A-1 to A-4 were arrested by the Sub-Inspector of Police, Kondapur and
produced before P.W.15. P.W.15 interrogated A-1 and A-1 gave confessional statement in
Ex.P6 and in pursuance of the confessional statement, a knife was recovered under
Ex.P7. As A-1 also sustained injuries, he was referred to hospital and examined by the
Doctor and Ex.P.10, wound certificate was issued. After receipt of the Forensic Sciences
Laboratory Report, he filed the charge sheet against A-1 and A-3 for the offence under
Section 302 read with 34, IPC. As A-4 was juvenile, he was produced before the Judicial
First Class Magistrate, Nizamabad, which is a juvenile Court. A-2 was absconding.
5. The learned Additional Judicial First Class Magistrate, Modak at Sangareddy, after
considering the material on record, came to the conclusion that the offence alleged
against the accused is exclusively triable by the Court of Session and, therefore, he
committed the case to the Court of Session. The learned Sessions Judge took the case on
file in S.C. No. 129 of 2001 and after hearing the prosecution and the defence and after
considering the material on record, charge for commission of offence punishable under
Section 302 read with 34, IPC was framed against A-1 and A-3. As A-1 and A-3 denied
the charge levelled against them, the prosecution examined P.Ws. 1 to 15 and marked
Exs. P1 to P12 besides marking of M.O.1 to prove its case. PWs. 1 to 4 were stated to be
eye-witnesses to the occurrence.
6. The stand of the appellant before the trial court was that the evidence of PWs 1 to 4
cannot be believed particularly when they are related to deceased and the presence of A2
and A3 at the time of incident is very much doubtful as they belong to some other village.
It was also contended that the medical evidence corroded credibility of ocular testimony
of PWs 1 to 4 as the injuries noticed were lacerated injuries which could not been caused
by a knife. P.Ws. 1 to 4 falsely implicated the accused. The trial court found the evidence
of PWs 1 to 4 to be credible and cogent and therefore convicted the accused-appellant. It
did not accept the plea of the accused that offence under
@page-SC507
Section 302, IPC is not made out.
7. The learned Sessions Judge accepting the evidence of P.Ws. 1 to 4, to be cogent and
credible came to the conclusion that A-1 caused injuries to the deceased and therefore he
was convicted and sentenced as stated above. Benefit of doubt was given to A-3 and
accordingly he was acquitted.
8. The judgment of the trial court was challenged before the High Court and the pleas
canvassed before the trial court were reiterated. The High Court as noted above did not
find any substance in the appeal and upheld the conviction and sentence imposed.
9. It was submitted by learned counsel for the appellant in support of the present appeal,
that PWs. 1 to 4 were related to the deceased and therefore their version is tainted. The
medical evidence rendered the ocular version improbable.
10. Learned counsel for the respondent-State supported the judgments of lower Court and
High Court.
11. We shall first deal with the contention regarding interestedness of the witnesses for
furthering prosecution version. Relationship is not a factor to affect credibility of a
witness. It is more often than not that a relation would not conceal actual culprit and
make allegations against an innocent person. Foundation has to be laid if plea of false
implication is made. In such cases, the court has to adopt a careful approach and analyse
evidence to find out whether it is cogent and credible.
12

. In Dalip Singh and Ors. v. The State of Punjab (AIR 1953 SC 364) it has been laid down
as under:- Para 26 of AIR

"A witness is normally to be considered independent unless he or she springs from


sources which are likely to be tainted and that usually means unless the witness has
cause, such as enmity against the accused, to wish to implicate him falsely. Ordinarily a
close relation would be the last to screen the real culprit and falsely implicate an innocent
person. It is true, when feelings run high and there is personal cause for enmity, that there
is a tendency to drag in an innocent person against whom a witness has a grudge along
with the guilty, but foundation must be laid for such a criticism and the mere fact of
relationship far from being a foundation is often a sure guarantee of truth. However, we
are not attempting any sweeping generalization. Each case must be judged on its own
facts. Our observations are only made to combat what is so often put forward in cases
before us as a general rule of prudence. There is no such general rule. Each case must be
limited to and be governed by its own facts."
13

. The above decision has since been followed in Guli Chand and Ors. v. State of
Rajasthan (1974 (3) SCC 698) in which Vadivelu Thevar v. State of Madras (AIR 1957
SC 614) was also relied upon. AIR 1974 SC 276
14

. We may also observe that the ground that the witness being a close relative and
consequently being a partisan witness, should not be relied upon, has no substance. This
theory was repelled by this Court as early as in Dalip Singh's case (supra) in which
surprise was expressed over the impression which prevailed in the minds of the Members
of the Bar that relatives were not independent witnesses. Speaking through Vivian Bose,
J. it was observed: AIR 1953 SC 364, Para 25

"We are unable to agree with the learned Judges of the High Court that the testimony of
the two eye-witnesses requires corroboration. If the foundation for such an observation is
based on the fact that the witnesses are women and that the fate of seven men hangs on
their testimony, we know of no such rule. If it is grounded on the reason that they are
closely related to the deceased we are unable to concur. This is a fallacy common to many
criminal cases and one which another Bench of this Court endeavoured to dispel in
Rameshwar v. State of Rajasthan (AIR 1952 SC 54 at p.59). We find, however, that it
unfortunately still persists, if not in the judgments of the Courts, at any rate in the
arguments of counsel."
15. Again in Masalti and Ors. v. State of U.P., (AIR 1965 SC 202) this Court observed
(p. 209-210 para 14):
"But it would, we think, be unreasonable to contend that evidence given by witnesses
should be discarded only on the ground that it is evidence of partisan or interested
witnesses.......The mechanical rejection of such evidence on the sole ground that it is
partisan would invariably lead to failure of justice. No hard and fast rule can be laid down
as to how much evidence should be appreciated. Judicial approach has to be
@page-SC508
cautious in dealing with such evidence; but the plea that such evidence should be rejected
because it is partisan cannot be accepted as correct."
16

. To the same effect is the decision in State of Punjab v. Jagir Singh (AIR 1973 SC 2407);
Lehna v. State of Haryana (2002 (3) SCC 76) and Gangadhar Behera and Ors. v. State of
Orissa (2002 (8) SCC 381). 2002 AIR SCW 4271

17

. The above position was highlighted in Babulal Bhagwan Khandare and Anr. v. State of
Maharashtra [2005 (10) SCC 404] and in Salim Saheb v. State of M.P. (2007 (1) SCC
699). 2004 AIR SCW 7376

18. The further plea related to primacy of medical evidence. The ocular testimonies has
been analysed in great detail and has been rightly held to be cogent.
19. Coming to the plea that the medical evidence is at variance with ocular evidence, it
has to be noted that it would be erroneous to accord undue primacy to the hypothetical
answers of medical witnesses to exclude the eye-witnesses' account which had to be
tested independently and not treated as the "variable" keeping the medical evidence as the
"constant".
20. It is trite that where the eye-witnesses' account is found credible and trustworthy,
medical opinion pointing to alternative possibilities is not accepted as conclusive.
Witnesses, as Bentham said, are the eyes and ears of justice. Hence the importance and
primacy of the quality of the trial process. Eye-witnesses' account would require a careful
independent assessment and evaluation for its credibility which should not be adversely
prejudged making any other evidence, including medical evidence, as the sole touchstone
for the test of such credibility. The evidence must be tested for its inherent consistency
and the inherent probability of the story; consistency with the account of other witnesses
held to be creditworthy; consistency with the undisputed facts, the "credit" of the
witnesses; their performance in the witness box; their power of observation etc. Then the
probative value of such evidence becomes eligible to be put into the scales for a
cumulative evaluation.
21

. The above position was reiterated in Krishan and another v. State represented by
Inspector of Police [(2003) 7 SCC 56]. 2003 AIR SCW 3688

22. Even otherwise, factually also the medical evidence is not contrary to ocular evidence
as claimed. On the contrary the doctor (PW 8) has clearly stated as to under what
circumstances lacerated injury can be caused by a knife.
23. Learned counsel for the appellant submitted that the occurrence took place in course
of sudden quarrel and, therefore, the trial court and the High Court were not justified in
holding the accused-appellant guilty of offence punishable under Section 302, IPC.
24. In essence the stand of learned counsel for the appellant is that Exception IV to
Section 304, IPC would apply to the facts of the case.
25. For bringing in operation of Exception 4 to Section 300, IPC, it has to be established
that the act was committed without premeditation, in a sudden fight in the heat of passion
upon a sudden quarrel without the offender having taken undue advantage and not having
acted in a cruel or unusual manner.
26. The Fourth Exception to Section 300, IPC covers acts done in a sudden fight. The
said Exception deals with a case of prosecution not covered by the First Exception, after
which its place would have been more appropriate. The Exception is founded upon the
same principle, for, in both there is absence of premeditation. But, while in the case of
Exception 1 there is total deprivation of self-control, in case of Exception 4, there is only
that heat of passion which clouds men's sober reason and urges them to deeds which they
would not otherwise do. There is provocation in Exception 4 as in Exception 1; but the
injury done is not the direct consequence of that provocation. In fact Exception 4 deals
with cases in which notwithstanding that a blow may have been struck, or some
provocation given in the origin of the dispute or in whatever way the quarrel may have
originated, yet the subsequent conduct of both parties puts them in respect of guilt upon
equal footing. A "sudden fight" implies mutual provocation and blows on each side. The
homicide committed is then clearly not traceable to unilateral provocation, nor in such
cases could the whole blame be placed on one side. For if it were so, the Exception more
appropriately applicable would be Exception 1. There is no previous deliberation or
determination to fight. A fight suddenly takes place, for which both parties are more or
less to be
@page-SC509
blamed. It may be that one of them starts it, but if the other had not aggravated it by his
own conduct it would not have taken the serious turn it did. There is then mutual
provocation and aggravation, and it is difficult to apportion the share of blame which
attaches to each fighter. The help of Exception 4 can be invoked if death is caused (a)
without premeditation; (b) in a sudden fight; (c) without the offender having taken undue
advantage or acted in a cruel or unusual manner; and (d) the fight must have been with
the person killed. To bring a case within Exception 4 all the ingredients mentioned in it
must be found. It is to be noted that the "fight" occurring in Exception 4 to Section 300
IPC is not defined in IPC. It takes two to make a fight. Heat of passion requires that there
must be no time for the passions to cool down and in this case, the parties have worked
themselves into a fury on account of the verbal altercation in the beginning. A fight is a
combat between two or more persons whether with or without weapons. It is not possible
to enunciate any general rule as to what shall be deemed to be a sudden quarrel. It is a
question of fact and whether a quarrel is sudden or not must necessarily depend upon the
proved facts of each case. For the application of Exception 4, it is not sufficient to show
that there was a sudden quarrel and that there was no premeditation. It must further be
shown that the offender has not taken undue advantage or acted in cruel or unusual
manner. The expression "undue advantage" as used in the provision means "unfair
advantage".
27

. The above position is highlighted in Sandhya Jadhav v. State of Maharashtra (2006) 4


SCC 653). 2006 AIR SCW 1678

28. Considering the background facts, appropriate conviction would be under Section 304
Part I IPC and not Section 302 IPC. The conviction is accordingly altered. Custodial
sentence of ten years would suffice.
29. Appeal is allowed to the aforesaid extent.
Appeal allowed.
AIR 2008 SUPREME COURT 509 "State of Rajasthan v. M/s. Khandaka Jain Jewellers"
(From : Rajasthan)
Coram : 2 A. K. MATHUR AND MARKANDEY KATJU, JJ.
Civil Appeal No. 5273 of 2007 (arising out of SLP (C) No. 19439 of 2006), D/- 16 -11
-2007.
State of Rajasthan and Ors. v. M/s. Khandaka Jain Jewellers.
(A) Stamp Act (2 of 1899), S.17, S.2(12) - Rajasthan Stamp Law (Adaptation) Act (7 of
1952), S.47A(1) (as inserted by Raj. Act 10 of 1982) - STAMP DUTY - SALE DEED -
EXECUTION - Stamp duty - Assessment - Relevant date for determining market value -
Stamp duty on sale deed - Current market value at time of execution has to be seen - Fact
that purchaser had to litigate for getting sale deed executed - Immaterial.
AIR 1998 AP 252, Overruled.
D. B. Civil Spl. Appeal Writ No. 427 of 2002, D/- 23-11-2005 (Raj), Reversed.
The valuation of property mentioned in instrument has to be done at current market value
at time of its execution. Fact that purchaser had to litigate for long to get the instrument
(Sale Deed) registered is immaterial. Section 17 stipulates that all the instruments
chargeable with duty and executed by person of India shall be stamped before or "at the
time of execution". The word "execution" has been defined in Section 2(12) which says
that "Execution" used with reference to the instruments, mean "signed" and "signature". It
shows that the document which is sought to be registered has to be signed by both the
parties. Till that time the document does not become an instrument for registration. A
reading of Section 2(12) with Section 17 clearly contemplates that the document should
be complete in all respects when both the parties should have signed it with regard to the
transfer of the immovable property. The expression "execution" read with Section 17
leaves no manner of doubt that the current valuation is to be seen when the instrument is
sought to be registered. Therefore, the market value of the instrument has to be seen at
the time of the execution of the sale deed, and not at the time when agreement to sale was
entered into. An agreement to sell is not a sale. An agreement to sale becomes a sale after
both the parties signed the sale deed. A taxing statute has to be construed strictly and
considerations of hardship or equity have
@page-SC510
no role to play in its construction.
AIR 1998 AP 252, Overruled.
D. B. Civil Spl. Appeal Writ No. 427 of 2002, D/- 23-11-2005 (Raj), Reversed.
(Paras 10, 13, 14)
(B) INTERPRETATION OF STATUTES - PRINCIPLES - Interpretation of Statutes -
Taxing statute - To be construed strictly principle of "Actus curie neminem gravabit" -
Does not affect the Principles of interpretation of a taxing statute. (Para 14)
Cases Referred : Chronological Paras
AIR 1998 Andh Pra 252, (Overruled)3, 14, 15
AIR 1957 SC 657 (Rel.on) 11
V. Madhukar, Sumit Ghosh and Aruneshwar Gupta, for Appellants; Dr. Manish Singhvi
and P. V. Yogeswaran, for Respondent.
Judgement
1. A. K. MATHUR, J. :- Leave granted.
2. This appeal is directed against the judgment dated 23.11.2005 passed by the Division
Bench of the High Court of Judicature for Rajasthan at Jaipur Bench, Jaipur in SBCWP
No. 133/1997 and DBCSA No. 427/2002 whereby the division bench has affirmed the
order of the learned Single Judge.
3. Brief facts which are necessary for the disposal of this appeal are as under:
The S.B. Civil Writ Petition No. 133/97 was filed by M/s. Khandaka Jain Jewellers,
petitioner (respondent herein) in the High Court of Judicature for Rajasthan, Jaipur
Bench, Jaipur who prayed that a direction may be issued to the respondent Nos. 2and 3
to register the sale deeds sent by the Court of additional district Judge No. 1, Jaipur city
in execution application No. 15/94 and 16/94 and to send back the same to the Court
immediately after registration. It was also prayed that the respondents may be directed to
register the sale deeds on the stamps on which it is executed by the executing court and
not to charge more stamp duty from respondent (herein). It was further prayed to quash
and set aside the proceedings taken under Section 47A(2) of the Stamps Act, 1952 in case
No. 442/95 and 443/95 on 4th March, 1997 for determination of the valuation of the sale
deed for registration.
The respondent is a registered firm and it entered into two agreements for purchase of
properties with Shri Prem Chand Ajmera, resident of 2148, Haldiyon Ka Rasta Jaipur by
one agreement dated 20th October, 1983. The property was agreed to be purchased for a
sum of Rs. 1,41,000/- out of which Rs. 20,000/- were paid at the time of the agreement.
As the vendor failed to comply with the terms of the agreement, the respondent vendee
filed a suit for specific performance of the contract in the Court of district Judge, Jaipur
city which was later on transferred to the Court of additional district Judge No.1, Jaipur
city under registration No. 216/86. The suit was decreed by the Judgment and decree
dated 2nd February,1994. In pursuance of the said decree, the respondent firm deposited
an amount of Rs. 1,21,000/- in the Court on 9th May, 1994. Since the vendor did not
execute the sale deed, therefore, the respondent firm filed the execution application No.
16/90 before the Court of additional district Judge No. 1, Jaipur city.
In another agreement dated 20th October, 1983 the vendor Premchand agreed to sell a
portion of property for a sum of Rs. 50,000/- out of which Rs. 10,000/- was paid at the
time of agreement. The respondent firm purchased the stamp papers and got the sale deed
typed. In this case also the vendor failed to fulfill the condition of agreement and to
execute the sale deed. Consequently, the respondent firm filed another suit for specific
performance of the contract in the Court of district Judge, Jaipur city. It was also
transferred to the court of additional district Judge No. 1, Jaipur city under registration
No. 151/91. The suit was decreed vide judgment and decree dated 2nd February, 1994
and the respondent firm was directed to deposit the remaining amount of Rs. 40,000/- and
the judgment debtor would execute the sale deed. If the judgment debtor fails to comply
with the decree, the decree holder would be entitled to get the sale deed registered and to
get the possession. In compliance of the judgment and decree passed by the Court, the
respondent firm deposited an amount of Rs. 40,000/- in the court but the judgment debtor
did not execute the sale deed. The execution application No. 15/94 was filed before the
Court of additional district Judge No. 1, Jaipur city. Both these applications No. 15/94
and 16/94 were taken up by the executing court and the respondent firm was directed to
submit the stamp papers for the execution of the two sale deeds. The stamp papers for a
sum of Rs.14,100/- and
@page-SC511
Rs. 5,000/- for execution of the sale deeds in respect of properties purchased for a sum of
Rs. 1,41,000/- and Rs. 50,000/- respectively, were submitted by the respondent firm.
The learned executing court executed the sale deeds and sent the same on 17th March,
1995 for registration before the Sub-Registrar, Registration Department, Collectorate
Bani Park, Jaipur. The Sub-Registrar exercising its powers under Section 47A(1) of the
Stamp Act sent these two sale deeds to Collector (Stamps) Jaipur for determining the
market value and to assess the charge of the stamp duty. The Collector (stamps)
registered these two cases No. 442/95 and 443/95 of the respondent firm and passed the
order dated 5th March, 1997. In case No. 442/95 he assessed value of the property as Rs.
5,60,000/- and deficient stamp duty was raised to the extent of Rs. 41,900/- and deficient
registration fees as Rs 1500/- and he also levied the penalty of Rs. 1000/-. Thus, the total
amount against the respondent firm raised was Rs. 44,400/-. In the second case No.
443/95 he assessed value of the property as Rs. 3,87,580/- and deficient stamp duty to the
extent of Rs. 33,758/- and deficient registration fees as Rs. 1500/- and the penalty of Rs.
1000/-. Thus the total amount directed to be recovered from the respondent firm was Rs.
36,258/-. The respondent firm filed writ petition challenging both these orders and the
contention of the respondent firm was that the valuation of the property should be taken
when the agreement of sale deed was executed, and not at the time of the registration of
the sale deed. The learned Single Judge relying on the judgment in the case of Sub
Registrat, Kodad Town and Mandal v. Amaranaini China Venkat Rao and Others
reported in AIR 1998 Andhra Pradesh 252 allowed the writ petition and observed that
since the vendor backed out and did not execute the sale deed of the property in
pursuance of the agreement on 20th October, 1983 therefore, the respondent firm filed a
suit for specific performance of contract in 1986 and the suit was decreed. The
respondent firm was ready and willing to pay the amount, and therefore, it was not his
fault. The same was the position regarding the second suit which was filed in 1991. The
learned Judge after considering the matter directed to set aside both the orders and held
that for the purpose of charging stamp duty, etc, the relevant date for assessment of the
market value shall be the date on which the suit for specific performance of the
agreement to sale was filed. Consequently the order dated 4th March, 1997 (Annexures 5
and 6) was quashed and the authorities were directed to pass a fresh order regarding the
market value of the property in question for the purpose of levy of the stamp duty as on
the date of filing of the suit and also directed to undertake this exercise keeping in view
the observation of the judgment within a period of one month from the date of receipt of
the certified copy of the order after notice to respondent firm.
4. Aggrieved against this order, an appeal was preferred before the Division Bench of the
Rajasthan High Court at Jaipur Bench and the Division Bench affirmed the order of the
learned single Judge. Aggrieved against the order of the Division Bench, the present
appeal was preferred by the State of Rajasthan and Ors., appellants herein.
5. We have heard learned counsel for the parties and perused the records.
6. The question is whether the valuation should be assessed on the market rate prevailing
at the time of registration of the sale deed or when the parties entered into agreement to
sell.
7. Learned counsel for the State has submitted that the Stamp Act is a taxing statute and a
taxing statute has to be construed strictly. Whatsoever may have been the consideration
for the vendor not to get the sale deed executed is a matter between both the parties, but
when the matter is before the registering Authority the registering Authority has to see the
valuation of the property at the market rate at the time of the registration as per Section
17 of the Act. Therefore, a notice under Section 47A of the (Rajasthan Amendment)
Stamp Duty Act was given and proper valuation was determined for registration. As
against this, the learned counsel for the respondent submitted that Section 3 of the Act is
a charging section. The registering authority has to see the instrument and the
consideration mentioned therein for payment of duty as per Section 27 of the Act. If he
finds it undervalued then he can hold an inquiry with regard to market value which was
prevailing at the time of agreement to sell.
8. In order to appreciate the controversy involved in the matter, it is necessary to
@page-SC512
reproduce the relevant provisions of the Stamp Act which are as under:
Section 2(12) of the Act reads as under:
"(12) "Executed", and "execution", used with reference to instruments, mean "signed"
and "signature"."
Section 3 of the Act reads as under:
"3. Instruments chargeable with duty - Subject to the provisions of this Act and the
exemptions contained in Schedule I, the following instruments shall be chargeable with
duty of the amount indicated in that Schedule as the proper duty therefor, respectively,
that is to say-
(a) every instrument mentioned in that Schedule which, not having been previously
executed by any person, is executed in (India) on or after the first day of July, 1899;
(b) every bill of exchange payable otherwise than on demand or promissory note drawn
or made out of India on or after that day and accepted or paid, or presented for acceptance
or payment, or endorsed, transferred or otherwise negotiated, in India; and
(c) every instrument (other than a bill of exchange or promissory note) mentioned in that
Schedule, which, not having been previously executed by any person, is executed out of
India on or after that day relates to any property situate, or to any matter or thing done or
to be done, in India and is received in India:
Provided that no duty shall be chargeable in respect of-
(1) any instrument executed by, or on behalf of, or in favour of, the Government in cases
where, but for this exemption, the Government would be liable to pay the duty chargeable
in respect of such instrument;
(2) any instrument for the sale, transfer or other disposition, either absolutely or by way
of mortgage or otherwise, of any ship or vessel, or any part, interest, share or property of
or in any ship or vessel, registered under the Merchant Shipping Act, 1894, or under Act
19 of 1938, or the Indian Registration of Ships Act, 1841 (10 of 1841) as amended by
subsequent Acts.
(3) Any instrument executed, by or on behalf of, or in favour of the Developer, or Unit or
in connection with the carrying out of purposes of the Special Economic Zone.
...
..."
Section 17 of the Act reads as under:
"17. Instruments executed in India - All instrument chargeable with duty and executed by
any person in India shall be stamped before or at the time of execution."
Section 27 of the Act reads as under:
"27. Facts affecting duty to be set forth in instrument.- The consideration (if any) and all
other facts and circumstances affecting the chargeability of any instrument with duty, or
the amount of the duty with which it is chargeable, shall be fully and truly set forth
therein."
Section 47-A inserted by Rajasthan(Amendment) State Stamp Act reads as under:
"S.47-A. Instruments under-valued, how to be valued.- (1) Notwithstanding anything
contained in the Registration Act, 1908 (Central Act XVI of 1908) and the rules made
thereunder as in force in Rajasthan where in the case of any instrument relating to an
immovable property chargeable with an ad valorem duty on the market value of the
property as set forth in the instrument, the registering officer has, while registering the
instruments, reason to believe that the market value of the property has not been truly set
forth in the instrument, he may either before or after registering the instrument, send it in
original to the Collector for determination of the market-value and to assess and charge
the duty in conformity with such determination together with a penalty not exceeding ten-
times the deficient stamp duty chargeable and surcharge, if any, payable on such
instrument.
(2) On receipt of the instrument under sub-section(1), the Collector shall, after giving the
parties a reasonable opportunity of being heard and after holding an enquiry in the
prescribed manner determine the market-value and the duty including penalty and
surcharge, if any, payable thereon; and if the amount of duty including penalty and
surcharge, if any, already paid, is deficient, the deficient amount shall be payable by the
person liable to pay the duty including penalty and surcharge, if any.
(2-A) Where it appears to a person having by law or consent of parties authority to
receive evidence or a person in charge of a public office, during the course of inspection
or otherwise, except an officer of a police, that an instrument is undervalued, such person
shall forthwith make a
@page-SC513
reference to the Collector in that matter.
(3) The Collector may, suo motu, or on a reference made under sub-section (2-A) call for
and examine any instrument not referred to him under sub-section (1), from any person
referred to in sub-section (2-A) or the executant or any other person for the purpose of
satisfying himself as to the correctness of the market-value of such property has not been
truly set forth in the instrument, he may determine in accordance with the procedure
provided in sub-section (2), the market-value and the amount of stamp duty together with
a penalty not exceeding ten times the deficient stamp duty chargeable on it, which shall
be payable by the person liable to pay the stamp duty and penalty.
(4) Where for any reason the original document called for by the Collector under sub-
section (3) is not produced or cannot be produced, the Collector may after recording the
reasons for its non-production call for a certified copy of the entries of the document
from the registering officer concerned and exercise the powers conferred on him under
sub-section (3).
(5) For the purpose of enquiries under this section, the Collector shall have power to
summon and enforce the attendance of witnesses including the parties to the instrument
or any of them, and to complete the production of documents by the same means, and so
far as may be in the same manner, as is provided in the case of Civil Court under Code of
Civil Procedure, 1908 (Central Act V of 1908)."
9. The contention of the learned counsel for the State that as per Section 17 of the Act, the
market value has to be taken into consideration because Section 17 stipulates that all the
instruments chargeable with duty and executed by person of India shall be stamped
before or "at the time of execution". The word "execution" has been defined in Section
2(12) of the Act which says that "Execution" used with reference to the instruments,
mean "signed" and "signature". Therefore, it shows that the document which is sought to
be registered has to be signed by both the parties. Till that time the document does not
become an instrument for registration. A reading of Section 2(12) with Section 17 clearly
contemplates that the document should be complete in all respects when both the parties
should have signed it with regard to the transfer of the immovable property. It is
irrelevant whether the matter had gone in for litigation.
10. It may be mentioned that there is a difference between an agreement to sell and a sale.
Stamp duty on a sale has to be assessed on the market value of the property at the time of
the sale, and not at the time of the prior agreement to sell, nor at the time of filing of the
suit. This is evident from section 17 of the Act. It is true that as per Section 3, the
instrument is to be registered on the basis of the valuation disclosed therein. But Section
47-A of the Rajasthan(Amendment) Stamp Duty Act contemplates that in case it is found
that properties are under-valued then it is open for the Collector (Stamps) to assess the
correct market value. Therefore, in the present case when the registering authority found
that valuation of the property was not correct as mentioned in the instrument, it sent the
document to the Collector for ascertaining the correct market value of the property. The
expression "execution" read with Section 17 leaves no manner of doubt that the current
valuation is to be seen when the instrument is sought to be registered. The Stamp Act is in
the nature of a taxing statute, and a taxing statute is not dependant on any contingency.
Since the word "execution" read with Section 17 clearly says that the instrument has to be
seen at the time when it is sought to be registered and in that if it is found that the
instrument has been undervalued then it is open for the registering authority to enquire
into its correct market value. The learned single Judge as well as the Division Bench in
the present case had taken into consideration that the agreement to sell was entered into
but it was not executed. Therefore, the incumbent had to file a suit for seeking a decree
for execution of the agreement and that took a long time. Therefore, the Courts below
concluded that the valuation which was in the instrument should be taken into account. In
our opinion this is not a correct approach. Even the valuation at the time of the decree is
also not relevant. What is relevant in fact is the actual valuation of the property at the
time of the sale. The crucial expression used in Section 17 is "at the time of execution".
Therefore, the market value of the instrument has to be seen at the time of the execution
of the sale deed, and not at the time when agreement to sale was entered into. An
agreement to sell is not a sale. An
@page-SC514
agreement to sell becomes a sale after both the parties signed the sale deed. A taxing
statute is not contingent on the inconvenience of the parties. It is needless to emphasize
that a taxing statute has to be construed strictly and considerations of hardship or equity
have no role to play in its construction. VISCOUNT SIMON quoted with approval a
passage from ROWLATT, J. expressing the principle in the following words
"In a taxing Act one has to look merely at what is clearly said. There is no room for any
intendment. There is no equity about a tax. There is no presumption as to tax. Nothing is
to be read in, nothing is to be implied. One can only look fairly at the language used."
11. The same view was expressed by Hon'ble Bhagwati J. in the case of A.V. Fernandez v.
State of Kerala reported in AIR 1957 SC 657. The principle is as follows:
"In construing fiscal statutes and in determining the liability of a subject to tax one must
have regard to the strict letter of the law. If the revenue satisfies the court that the case
falls strictly within the provisions of the law, the subject can be taxed. If on the other
hand, the case is not covered within the four corners of the provisions of the taxing
statute, no tax can be imposed by inference or by analogy or by trying to probe into the
intention of the Legislature and by considering what was the substance of the matter."
Hon'ble Shah J. has formulated the principle thus: "In interpreting a taxing statute,
equitable considerations are entirely out of place. Nor can taxing statutes be interpreted
on any presumptions or assumptions. The court must look squarely at the words of the
statute and interpret them. It must interpret a taxing statute in the light of what is clearly
expressed; it cannot imply anything which is not expressed; it cannot import provisions in
the statute so as to supply any assumed deficiency."
Therefore, a taxing statute has to be read as it is. In other words, the literal rule of
interpretation applies to it.
12. In this back-ground, if we construe Section 17 read with Section 2(12) then there is
no manner of doubt that at the time of registration, the Registering Authority is under an
obligation to ascertain the correct market value at that time, and should not go by the
value mentioned in the instrument.
13. Learned counsel for the respondent submitted that if we construe Section 3 read with
Section 27 of the Act then the Registering Authority is under an obligation to only see the
value mentioned in the instrument. In our opinion Section 3 which is the charging section
cannot be read in isolation but has to be read along with Section 17 of the Act. From a
composite reading of Sections 3,17 and 27, it becomes abundantly clear that the valuation
given in an instrument is not conclusive. If any doubt arises in the mind of the
Registering Authority that the instrument is under-valued then as per Section 47-A of the
Rajasthan (Amendment) the instrument can be sent to the Collector for determination of
the correct market value. Under Section 47-A read with Sections 3,17 and 27, it becomes
clear that the Registering Authority has to ascertain the correct valuation given in the
instrument regarding market value of the property at the time of the sale.
14

. Learned Counsel for the respondent strenuously urged before us that in fact when the
agreement to sell was not executed by the vendor, the respondent had no option but to file
a suit and a long time was taken for obtaining a decree for execution of the agreement. He
was not at fault and as such the valuation given in the instrument should be taken into
consideration because during the litigation the valuation of the property has shot up. In
this connection, learned counsel has invited our attention to the principle "Actus curie
neminem gravabit" meaning thereby that no person shall suffer on account of litigation.
Hence learned counsel submitted that since the matter had been in the litigation for a long
time, the respondent cannot be made to suffer. He invited our attention to the decision of
the Andhra Pradesh High Court Sub-Registrar, Kodad Town and Mandal (supra). It is
true that no one should suffer on account of the pendency of the matter but this
consideration does not affect the Principles of interpretation of a taxing statute. A taxing
statute has to be construed as it is all these contingencies that the matter was under
litigation and the value of the property by that time shot up cannot be taken into account
for interpreting the provisions of a taxing statute. As already mentioned above a taxing
statute has to be construed strictly and if it is construed strictly then the plea that AIR
1998 AP 252

@page-SC515
the incumbent took a long time to get a decree for execution against the vendor that
consideration cannot weigh with the Court for interpreting the provisions of the taxing
statutes. Therefore, simply because the matter have been in the litigation for a long time
that cannot be a consideration to accept the market value of the instrument when the
agreement to sale was entered. As per Section 17, it clearly says at the time when
registration is made, the valuation is to be seen on that basis.
15. In the case of Sub-Registrar, Kodad Town and Mandal (Supra), the learned single
Judge of the Andhra Pradesh High Court felt persuaded on account of 30 years' long
litigation and therefore, declined to send the papers back to the Collector for valuation at
the market value. With great respect, the view taken by the learned single Judge is against
the principles of interpretation of a taxing statute. Therefore, we are of the opinion that
the view taken by the learned single Judge of the Andhra Pradesh High Court is not
correct.
16. Accordingly, we are of the opinion that the view taken by the learned single Judge as
well as by the Division Bench cannot be sustained and the same is set aside. The
Collector shall determine was the valuation of the instrument on the basis of the market
value of the property at the date when the document was tendered by the respondent for
registration, and the respondent shall pay the stamp duty charges and surcharge, if any, as
assessed by the Collector as per the provisions of the Act. The appeal of the State is
allowed. No order as to costs.
Appeal allowed.
AIR 2008 SUPREME COURT 515 "Mahmood v. State of U. P."
(From : Allahabad)*
Coram : 2 ALTAMAS KABIR AND B. SUDERSHAN REDDY, JJ.
Criminal Appeal No. 402 of 2006, D/- 15 -11 -2007.
Mahmood and Anr. v. State of U.P.
(A) Criminal P.C. (2 of 1974), S.157 - INVESTIGATION - MAGISTRATE - Sending
special report to Magistrate - Time required by Station House Officer after recording FIR
- No universal rule as to, can be laid down - Each case turns on its own facts.
2002 AIR SCW 4290, AIR 1975 SC 1962, AIR 1975 SC 1193, Rel. on. (Para 12)
(B) Criminal P.C. (2 of 1974), S.154 - FIR - FIR - Informant alone rushed to police
station 9 Kms. away from place of occurrence by bicycle and lodged written FIR within 1
1/2 hours of incident - His evidence showing that he took about 15-20 minutes to prepare
his report and nobody advised him in its preparation - Said version cannot be disbelieved
- Stating details of incident in written FIR nor unnatural or unusual - Nothing artificial in
FIR - Cannot be said to be contrived one brought into existence after due deliberations -
Moreover Inquest Report prepared later on refers to lodging of FIR by informant - Mere
fact that crime number is not mentioned in it is of no significance - Further arrest of one
person from place of occurrence at 3.00 P.M. by investigating officer in connection with
another case - Would not militate against time of FIR as shown in police papers - Held,
FIR is not ante time and ante dated. (Paras 16, 17, 18, 21)
(C) Penal Code (45 of 1860), S.300, S.149 - MURDER - UNLAWFUL ASSEMBLY -
ASSAULT - Murder - Unlawful assembly - Accused persons assaulted deceased with fire
arms and lathi - Son of deceased proved to be present in his fields situated nearby place
of occurrence - He gave detailed version as to manner of assault and role played by each
of accused - His presence cannot be doubted merely because he made no attempt to save
his father - Names of other witnesses also mentioned as eye- witnesses in FIR itself
-Thus said witnesses were not chance witnesses and their presence cannot be disbelieved
on basis of medical evidence - Their testimony cannot be disbelieved - Specific overt act
of accused need not be established as membership of unlawful assembly already
established - Conviction of accused - No interference. (Paras 22, 23, 26, 27)
(D) Evidence Act (1 of 1872), S.45 - EVIDENCE - Expert evidence - Medical Officer - Is
not ballistic expert - He was not expected to answer whether injury in question could
have been caused by bullet alone.
The Medical Officer is not ballistic expert. He was not expected to answer as to whether
injury in question could have been caused
@page-SC516
by bullet alone. His opinion to that extent is of no consequence. It is well settled that
medical evidence is only an evidence of opinion and it is not conclusive and when oral
evidence is found to be inconsistent with the medical opinion, the question of relying
upon one or the other would depend upon the facts and circumstances of each case. No
hard and fast rule can be laid down therefor. The ocular evidence if otherwise is
acceptable has to be given importance over medical opinion. However, where the mdical
evidence totally improbabilises the ocular version the same can be taken to be a factor to
affect credibility of the prosecution version. Thus no reliance can be placed upon the
opinion of the Medical Officer that the injury in question could have been caused only
with bullet since he is not a ballistic expert. This part of the evidence of the Medical
Officer cannot be considered to be the opinion of an expert and the same has no
evidentiary value.
(Para 25)
(E) Penal Code (45 of 1860), S.149 - UNLAWFUL ASSEMBLY - Unlawful assembly -
Its membership once established, prosecution need not establish any specific overt act to
any of the accused for fastening of liability with aid of S. 149. (Para 26)
Cases Referred : Chronological Paras
2006 AIR SCW 2686 : AIR 2006 SC 2500 : 2006 Cri LJ 2886 14
2002 AIR SCW 4290 : AIR 2002 SC 3648 : 2003 Cri LJ 17 (Rel. on) 11
2001 AIR SCW 2833 : AIR 2001 SC 3173 : 2001 Cri LJ 3969 9
1994 AIR SCW 2210 : 1995 Cri LJ 427 8
AIR 1975 SC 1193 : 1974 Cri LJ 908 (Rel. on) 13
AIR 1975 SC 1962 : 1975 Cri LJ 1734 (Rel. on) 13
Harjinder Singh, Sr. Advocate, R. C. Kohli, Ms. Shikha Tyagi, Ms. Seema Juneja, for
Appellants; Shail Kumar Dwivedi, AAG, A. K. Jha, Prashant Choudhary, Ms. Vandana
Mishra, Manoj Dwivedi, for Respondent.
* Cri. A. No. 367 of 1980, D/- 17-5-2005 (All) (LB).
Judgement
B. SUDERSHAN REDDY, J. :- This is an appeal by special leave preferred by the
appellants-Mahmood and Khaliq. The appellant-Mahmood has been convicted for the
offence punishable under Section 302 read with Section 149 of IPC and sentenced to
imprisonment for life. He has been also convicted under Section 148 of IPC and
sentenced to undergo rigorous imprisonment for 1½ year. The second appellant has been
convicted for the offence punishable under Section 302 read with Section 149 and
sentenced to undergo life imprisonment. He has been further convicted under Section 147
of IPC and sentenced to undergo one year rigorous imprisonment and further convicted
under Section 379 of IPC and sentenced to undergo rigorous imprisonment for a period of
two years.
2. Put briefly the prosecution case is as follows :
On 19th February, 1977 at about 4.45 p.m. the accused Ram Samujh and Mahmood
appellant No.1 both armed with guns, Khalid-appellant No.2, Bajrang and one
unidentified person armed with lathi assaulted deceased Ram Singh at Galiyara near the
fields of Ram Sewak Ahir, while he was returning to his village Badipur on his
motorcycle. It was alleged that the accused Ram Samujh and Mahmood fired four shots,
as a result of which the deceased fell down injured and thereafter Khaliq snatched the
licensed revolver belonging to the deceased and all the five fled away from the scene.
Ram Singh died on the spot. The incident of murderous attack was witnessed by Jaikirat
Singh (P.W.1) who is none other than the son of deceased-Ram Singh, Ram Ratan
(P.W.2), resident of village Sujerpur hamlet of Bodipur and Ram Adhar (P.W.3). P.W.1
lodged written First Information Report Ext.Ka.1 on the same day at 4.45 p.m. naming all
the accused and the manner in which the murderous attack on the deceased had taken
place. Jagdamba Prasad Dwivedi (P.W.7) the office in-charge of Police station, Kothi
rushed to the scene of offence at about 6.00 p.m. and found the dead body of Ram Singh
and his motorcycle in galiyara near the fields of Ram Sewak Ahir. The broken pieces of
the skull of the deceased and broken three teeth were seized from the place of occurrence.
The discharged cartridge and tickli were also seized from the spot. P.W.7 after preparing
the Inquest Report (Ext. Ka.7) sent the dead body for conducting post-mortem. Dr. R.S.
Katiyar P.W.5 performed the autopsy on the dead body on 20th February, 1977 at about
9.45 a.m. and found as many as five ante-mortem gun shot wounds. A cap of cartridge
was extricated from the brain of the deceased. Scalp bones were found fractured. It was
found that vital organs like peritoneum, liver, kidneys
@page-SC517
were badly ruptured. In the opinion of the doctor, the cause of death was due to shock and
hemorrhage resulting from ante-mortem injuries. The investigation of the case was
transferred in the first week of March, 1977 to CBCID. Inspector M.L. Gautam having
completed rest of the investigation submitted charge-sheet against the appellants and
other accused.
3. The accused have denied the charges framed against them and took the plea that they
have been falsely implicated due to enmity. The accused were accordingly put on trial.
The prosecution in order to establish its case in altogether examined 8 witnesses and got
marked 39 documents as Exts. Ka.1-39. Amongst the witnesses examined by the
prosecution, Jaikirath Singh, Ram Ratan and Ram Adhar (P.Ws. 1,2 and 3) respectively
were eye-witnesses to the murderous attack on the deceased. The accused also led
evidence and examined Virendra Singh DW 1, Laxmi Narain Sinha DW 2 and Bindra
Charan DW 3.
4. The learned Sessions Judge upon appreciation of the oral evidence and material on
record found all the accused guilty of the charges framed against them and sentenced
them to various terms of imprisonment. On appeal the High Court of Allahabad
confirmed the conviction and sentences imposed by the learned Sessions Judge. The
appellants who are accused Nos.2 and 3 respectively alone have preferred this appeal by
special leave, challenging their conviction and sentence.
5. We have elaborately heard the learned senior counsel Shri Harjinder Singh and Shri
R.C. Kohli as well as Shri Shail Kumar Dwivedi, learned Additional Advocate General
for the State.
6. The learned senior counsel Shri Harjinder Singh mainly contended that the FIR lodged
by P.W.1 Jaikirath Singh was ante-timed and ante-dated and brought into existence after
due deliberations and consultations with the police.
7. According to the learned senior counsel, the special report required to be sent to the
superior authorities and a copy of check FIR to the Illaqua Magistrate as required under
Section 157 of the Code of Criminal Procedure was not sent by the police. That apart
arrest of Maiku Bhujwa before 3.40 p.m. and his detention in the police station at 5.30
p.m. and also the fact that some seizure memos, prepared by Investigating Officer on the
same day which do not bear any crime number, are more than sufficient to doubt the
timings of FIR Ext.Ka.1.
8

. There is no doubt that FIR in a criminal case and particularly in murder case is a vital
and valuable piece of evidence for the purpose of appreciating evidence led by the
prosecution at the trial. FIR is the earliest information regarding the circumstances under
which the crime was committed, including the names of the actual culprits and the part
played by them, the weapons, if any, used as also the names of the eye-witnesses, if any.
Delay in lodging the FIR may result in embellishment, which is a creature of an after
thought. This court in Meharaj Singh vs. State of U.P.1observed that with a view to
determine whether the FIR was lodged at the time it is alleged to have been recorded, the
courts generally look for certain external checks. One of the check is the receipt of the
copy of the FIR, called as a Special Report in a murder case, by the local Magistrate. "If
this report is received by the Magistrate late it can give rise to an inference that the FIR
was not lodged at the time it is alleged to have been recorded, unless, of course, the
prosecution can offer a satisfactory explanation for the delay in despatching or receipt of
the copy of the FIR by the local Magistrate. The second external check equally important
is sending of copy of the FIR along with the dead body and its reference in the Inquest
Report". 1994 AIR SCW 2210

1 (1994) 5 SCC 188.


9

. This court while construing Section 157 of the Code of Criminal Procedure in Anil Rai
vs. State of Bihar2observed that the said provision is designed to keep the Magistrate
informed of the investigation of such cognizable offence so as to be able to control the
investigation and if necessary to give appropriate direction under Section 159 of the
Code. "But where the FIR is shown to have actually been recorded without delay and
investigation started on the basis of the FIR, the delay in sending the copy of the report to
the Magistrate cannot by itself justify the conclusion that the investigation was tainted
and the prosecution insupportable." 2001 AIR SCW 2833

2 (2001) 7 SC 318
10. This court further took the view that the delay contemplated under Section 157
@page-SC518
of the Code for doubting the authenticity of the FIR is not every delay but only extra-
ordinary and unexplained delay. We do not propose to burden this short judgment of ours
with various authoritative pronouncements on the subject since the law is so well settled
that delay in despatch of FIR by itself is not a circumstance which can throw out the
prosecutions case in its entirety, particularly in cases where the prosecution provides
cogent and reasonable explanation for the delay in despatch of the FIR.
11

. The same principle has been reiterated by this court in Alla China Apparao and Ors. v.
State of A.P.3wherein this court while construing the expression "forthwith" in Section
157 (1) of Code of Criminal Procedure observed that "it is a matter of common
experience that there has been tremendous rise in the crime resulting into enormous
volume of work, but increase in the police force has not been made in the same
proportion. In view of the aforesaid factors, the expression "forthwith" within the
meaning of Section 157(1) obviously cannot mean that the prosecution is required to
explain every hours delay in sending the first information report to the magistrate, of
course, the same has to be sent with reasonable despatch, which would obviously mean
within a reasonable possible time in the circumstances prevailing. Therefore, in our view,
the first information report was sent to the magistrate with reasonable promptitude and no
delay at all was caused in forwarding the same to the magistrate. In any view of the
matter, even if magistrates court was closed by and the first information report reached
him within six hours from the time of its lodgment, in view of the increase in work load,
we have no hesitation in saying that even in such a case it cannot be said that there was
any delay at all in forwarding the first information report to the magistrate." 2002
AIR SCW 4290

3 2002 (8) JT (SC) 167


12. It is not possible to lay down any universal rule as to within what time the special
report is required to be despatched by the Station House officer after recording the FIR.
Each case turns on its own facts.
13

. The learned senior counsel invited our attention to the judgments of this court in Balaka
Singh and ors. vs. State of Punjab4and Datar Singh vs. The State of Punjab5in which this
court highlighted the importance of despatch of special report to the Illaqua Magistrate.
There is no dispute with the proposition that it is the duty of the Station House Officer to
despatch Special Report to the Illaqua Magistrate as is required under Section 157(2) of
the Code of Criminal Procedure. But there may be variety of factors and circumstances
for the delay in despatch of the FIR and its receipt by the local Magistrate. The existence
of FIR and its time may become doubtful in cases where there is no satisfactory and
proper explanation from the investigating agencies. AIR 1975 SC 1193

4 AIR 1975 SC 1962


5 (1975) 4 SCC 272
14

. In Budh Singh and Ors. vs. State of UP6, this court while making reference of the
regulations made by the State of U.P. in terms of the U.P. Police Act held the regulations
to be statutory in nature. The regulations provide the procedure as to how and in what
form the information relating to commission of a cognizable offence when given to an
officer in-charge of a police station is to be recorded and sent to superior officers. The
regulations are procedural in nature which are meant for the guidance of the police. The
regulations do not supplant but supplement the provisions of Code of Criminal
Procedure. 2006 AIR SCW 2686

6 2006 (11) JT (SC) 503


15. We shall now consider the facts of the present case and apply the law declared by this
court in more than one decision.
16. It is in the evidence of Jaikirath Singh (P.W.1) that he rushed to the police station by a
bicycle and lodged written FIR Ext.Ka.1 within 1½ hours of the incident. The distance
between the place of occurrence and the police station is about 9 kms. It is in his evidence
that he took about 15-20 minutes to prepare his report and nobody advised him in
preparation of the report. He went to the police station all alone. We do not find any
reason whatsoever to disbelieve this version given by PW 1. There is nothing unnatural
and unusual in PW 1 stating the details of the incident in his written FIR Ext.Ka.1. The
behavioural pattern and response of individuals in a given situation may differ from
person to person. From a bare reading of the FIR Ext.Ka.1 we do not find anything
artificial in it. It cannot be said to be a contrived one brought into existence after due
deliberations as contended by the counsel for the appellant.
17. Be it noted, Jagdamba Prasad
@page-SC519
Dwivedi, PW 7, the officer in-charge of police station, Kothi having received the relevant
papers in village Sethmau, rushed to the place of occurrence and reached there at about
6.00 p.m. where he found the dead body of Ram Singh. The inquest report Ext.Ka.7 was
prepared on the spot and the body was sent for post-mortem examination. The Inquest
Report Ext.Ka.7 specifically refers to the lodging of FIR by PW 1 at 4.45 p.m. on
19.02.1977. The mere fact that crime number is not mentioned in the Inquest Report is of
no significance.
18. The sequence of events, namely, that Jagdamba Prasad Dwivedi -PW 7 reached the
scene of offence at 6.00 p.m. and prepared Inquest Report duly mentioning about lodging
of the FIR by PW 1 at 4.45 p.m. on 19th February, 1977 followed by despatch of the dead
body to the hospital which reached the hospital by 9.30 p.m. and the post-mortem
examination at 9.30 a.m. on 20th February, 1977 in clear and unequivocal terms reveal
that the FIR was lodged at the time it is stated to have been recorded. It cannot be treated
as an ante-timed and ante-dated one. It is required to note that 20th February, 1977 being
Sunday, the Illaqua Magistrate received special report on 21st February, 1977. The
special report was despatched by dak.
19. Arrest of one Maiku Bhujwa on 19th February, 1977 at 3.00 p.m. in Crime No.17
under Section 147 etc. and his being lodged in police station at about 5.30 p.m. by two
constables Ram Naresh and Ram Tool Misra as shown in Exts. Ka. 3 and 4 has been used
as a sheet anchor to challenge the time of FIR Ext.Ka.1 by saying that if the two
constables were summoned by Station Officer, on reaching the place of occurrence, then
in all probability Station Officer reached the place of occurrence by 3.00 p.m. even before
the FIR was issued.
20. The High Court adverting to this aspect of the matter observed "the investigating
officer Sri Dwivedi does not say that he arrested Maiku Bhujwa. Moreover, arrest of
Maiku was not in connection with the murder in question, but was in connection with
another case. Most importantly, what could have been the object behind delaying the time
of occurrence of reaching Sri Dwivedi, on the spot, has not been made clear by Sri
Kidwai. We are of the view that arrest of Maiku at about 3.00 p.m. and his lodging in
Hawalat at 5.30 p.m. by two constables, does not militate against the time of FIR Ext.Ka-
1 as shown in police papers. It is also possible that some manipulation was made in the
context of the arrest of Maiku, to make the case against him more sound".
21. We do not find any fallacy or error in the reasoning of the High Court. For the
aforesaid reasons we do not find any substance in the submission made by the learned
senior counsel about the ante-time and ante-dating of the FIR. The findings in this regard
as recorded by Sessions Judge as well as the High Court are supported by acceptable
evidence and there is no reason to take a different view. It is well settled that this court
normally does not reappreciate the evidence unless it is shown that the findings are
patently erroneous or perverse in nature. However, in order to satisfy ourselves we have
looked into the evidence of PWs 1,2,3 and 7 and we are satisfied that the FIR was lodged
on the date and time as stated by the prosecution.
22. The prosecution story entirely rests upon the direct evidence of PW Nos. 1, 2 and 3.
PW-1 is none other than the son of deceased Ram Singh. He was present in his fields
situated nearby the place of occurrence where his father was attacked. Jaikirat (PW-1) no
doubt was doing his part time G.N.S. in plantation at Lucknow but that itself would not
make his presence doubtful at the scene of offence on the fateful day. The defence did not
elicit anything in the cross-examination casting any doubt about the presence of PW-1 at
the scene of offence. There is nothing unnatural about the conduct of PW-1 at the scene
of occurrence. He gave detailed version as to the manner of assault and the role played by
each of the accused. The names of PW-2 and PW-3 were also mentioned as eye-witnesses
in the First Information Report itself. In the circumstances, PW-2 and PW-3 cannot be
treated as chance witnesses. The Trial Court and as well as the High Court did not
commit any error in relying on the testimony of PW-2 and PW-3 as eye-witnesses of the
occurrence which fully stands corroborated with the testimony of PW-1. Be that as it
may, there was not even a suggestion to PW-2 and PW-3 that they had animosity towards
the accused persons. They are independent witnesses and there is no reason for them to
speak against the accused.
23. However, it was strenuously urged that the presence of Jaikirat (PW-1) at the
@page-SC520
scene of offence is highly doubtful as he made no attempt whatsoever to save his father
from being further assaulted. We find no substance in this contention. It is in the evidence
of Jaikirat (PW-1) that all the four shots were fired in quick succession and at that
moment PW-1 was at some distance from the actual place of attack. Be it noted that at
least 2 accused were armed with fire-arms and one with lathi and they were using the
weapons with all impunity. In such circumstances, Jaikirat (PW-1) may not have
mustered his courage to jump into the fray and risk his own life. It is very difficult to
predict or express any opinion as to what could have been normal or natural conduct of a
person in such a situation. Response of individuals in such situations may differ from
person to person. It is not possible to reject the evidence or doubt the presence of PW-1
on that ground.
24. The post-mortem examination of the deceased Ram Singh was performed by
Dr.R.S.Katiyar (PW-5). The post-mortem report is exhibit Ka-4. The Medical Officer
found the following ante-mortem injuries on the person of the deceased:
1. A gun shot wound (wound of entry) 3 cm x 1 cm. over left side of face just above the
left side of the lower lip. Wound of Ext. 3 cm x 2 cm. over the right parietal bone, 7 cm.
Above the right ear.
2. A gun shot wound 2.5 cm x 1 cm. over the right side of face below max. prominence.
3. Multiple gun shot wounds in an area of 13 cm x 11 cm. over the right side of back
below the inferior angle of scapula.
4. A gun shot wound (wound of entry) 2 cm x 2 cm over the right side of the back 2 cm.
Right to 12th thoracic vertebra.
5. Multiple gun shot wounds in an area of 9 cm. X 4 cm. over the back and middle of
right arm.
25. Relying on his evidence the learned counsel for the appellant contended that the oral
account as given by PW-1, 2 and 3 is at variance with medical evidence available on
record. It is contended that while according to the eye-witnesses all the four shots were
fired from the gun, from right side of the victim, wound No.1 (wound of entry) was on
the left side of the face and caused by bullet and this evidence belies the claim of eye
witnesses that they saw the assault on Ram Singh. It is true that to a pointed query in
cross-examination as regards the nature of injury No. 1, the Medical Officer stated that
the said injury was caused by bullet only. The learned counsel contended that weapons in
the hands of the accused even according to PW-1 were of 12 bore guns and not any
pistols or revolvers. No bullet injury could have been caused with the fire-arms that were
alleged to be in the hands of the assailants. We find no substance in this submission. The
Medical Officer is not ballistic expert. He was not expected to answer as to whether
injury No. 1 could have been caused by bullet alone. His opinion to that extent is of no
consequence. It is well settled that medical evidence is only an evidence of opinion and it
is not conclusive and when oral evidence is found to be inconsistent with the medical
opinion, the question of relying upon one or the other would depend upon the facts and
circumstances of each case. No hard and fast rule can be laid down therefor. The ocular
evidence if otherwise is acceptable has to be given importance over medical opinion.
However, where the medical evidence totally improbabilises the ocular version the same
can be taken to be a factor to affect credibility of the prosecution version. We are not
inclined to place any reliance upon the opinion of the Medical Officer that the injury
No.1 could have been caused only with bullet since he is not a ballistic expert. This part
of the evidence of the Medical Officer cannot be considered to be the opinion of an
expert and the same has no evidentiary value. It is not possible to disbelieve the evidence
of PWs-1, 2 and 3 and their presence at the scene of occurrence based on the medical
evidence. The High Court rightly observed that the controversy as regards injury No. 1
and whether the same could have been caused by bullet or pellet to be without any basis.
26. The learned counsel for the State rightly contended that in case of attack by members
of un-lawful assembly on the victim in furtherance of common object, it is not necessary
for the prosecution to establish overt act done by each accused. It is required to be
noticed that Ram Smujh (A-1) who had fired two shots, convicted by the Sessions Court,
did not even challenge his conviction in the High Court. The appellants have been rightly
convicted under Section 302 read with aid of Section 149 of IPC. PW-5 in his evidence
stated that all the injuries sustained by the deceased were from gun. It is further stated
that from the body of
@page-SC521
deceased one bullet, one cover tikli, two dat and 40 'chare' shots were taken out, put in
packet and sealed........" It is also stated in his evidence that injuries caused on the body of
the deceased were sufficient in the normal course to cause death. This part of the medical
evidence if juxtaposed with the oral evidence of PWs-1, 2 and 3 it becomes unnecessary
to go into the question as to which accused caused what injury and which was a fatal one.
Once a membership of an unlawful assembly is established, it is not incumbent on the
prosecution to establish any specific overt act to any of the accused for fastening of
liability with the aid of section 149 of the IPC. Commission of overt act by each member
of the unlawful assembly is not necessary. The common object of the unlawful assembly
of the accused in the present case is evident from the fact that some of them were armed
with deadly weapons. None of them were curious onlookers or spectators to the macabre
drama that was enacted on 19.2.1977 at 3.30 p.m. at galiyara, village Badipur.
27. For the aforesaid reasons, we find no merit in this appeal. The appeal is accordingly
dismissed.
Appeal dismissed.
AIR 2008 SUPREME COURT 521 "Union of India v. State of U. P."
(From : 2001 All LJ 611)
Coram : 2 A. K. MATHUR AND MARKANDEY KATJU, JJ.
Civil Appeal No. 2549 of 2001, D/- 1 -11 -2007.
Union of India and Ors. v. State of U.P. and Ors.
Constitution of India, Art.285 - CESS - PROPERTY TAX - WATER TAX - Exemption of
property of Union from State taxation - Service charges for supply of water and
maintenance of sewerage system of Railway colonies as provided by Jal Sansthan - Is fee
and not tax - Not violative of Art. 285.
U.P. Water Supply and Sewerage Act (43 of 1975), S.52.
Service charges for supply of water and maintenance of sewarage system of Railway
colonies, as provided by Jal Sansthan cannot be said to be tax on the property of the
Union and it is fee. Hence it is not violative of Art. 285. So far as supply of water and
maintenance of sewerage is concerned, the Jal Sansthan is to maintain it and it is they
who bear all the expenses for the maintenance of sewerage and supply of water. It has to
create its own funds and therefore, levy under the Act is a must. In order to supply water
and maintain sewerage system, the Jal Sansthan has to incur the expenditure for the same.
It is in fact a service which is being rendered by the Jal Sansthan to the Railways, and the
Railways cannot take this service from the Jal Sansthan without paying the charges for
the same. Though the expression 'tax' has been used in the Act of 1975 but in fact it is in
the nature of a fee for the services rendered by the Jal Sansthan. Moreover what is
contemplated under Art. 285 is taxation on the property of the Union. It is a plain and
simple charge for service rendered by the Jal Sansthan for which the Jal Sansthan has to
maintain staff for regular supply of water as well as for sewerage system of the effluent
discharge by the railway over their platform or from their staff quarters.
(Paras 9, 23)
Cases Referred : Chronological Paras
2006 AIR SCW 4031 : AIR 2006 SC 2897 (Ref.) 20
2005 AIR SCW 256 : AIR 2005 SC 635 (Ref.) 19
2004 AIR SCW 466 : AIR 2004 SC 2912 (Foll.) 17
2004 AIR SCW 4665 : AIR 2004 SC 4499 (Ref.) 21
1997 AIR SCW 2851 : AIR 1997 SC 2847 (Disting.) 17, 22
(1996)7 SCC 542 (Disting.) 4, 16
1992 AIR SCW 1737 : AIR 1992 SC 1597 (Disting.) 4, 14, 16
AIR 1963 SC 1760 (Disting.) 10, 21
Shail Kumar Dwivedi, AAG (UP), T. S. Doabia, Sr. Advocates, Ms. Kiran Bhardwaj, Ms.
Asha G. Nair, Vishal Nayar, Manpreet Singh Doabia, R. C. Kathia, D. S. Mahra, Mrs.
Anil Katiyar, Rajesh, Wasim Ahmad Qadri, Manoj Kumar Dwivedi, G. Venkateswara
Rao, Ms. Chitra Markandaya, Vivek Vishnoi, Mukesh Verma, Manish Shanker, M. R.
Shamshad, Yash Pal Dhingra, for appearing parties.
Judgement
1. A. K. MATHUR, J. :- This appeal is directed against the judgment dated 12.12.2000
passed by the Division Bench of the Allahabad High Court whereby the Division Bench
has dismissed the writ petition filed by the appellants and upheld the recovery
proceedings initiated against the appellants for the demand raised by the Jal
@page-SC522
Sansthan, Allahabad as water and sewer charges.
2. The Union of India and two others filed a writ petition before the High Court of
Allahabad challenging the orders of recovery dated 1.7.1999 and 20.12.1999 issued by
the Executive Engineer, Jal Sansthan, Khusru Bagh, Allahabad on account of service
charges on Railway properties situated at Allahabad for the period from October, 1994 to
March, 1999. The appellants also challenged the recovery certificate issued by the
Tahasildar, Sadar, Allahabad for recovery of a sum of Rs.26,23,360/- from the appellant
No.2 i.e. the Divisional Railway Manager, Northern Railway, Allahabad. It was alleged
by the Jal Sansthan that the appellants were liable to pay the sewerage charges for 3125
seats at the rates notified under Allahabad Jal Sansthan Notification published in U.P.
Gazette dated 19.11.1994. The plea of the appellants was that they were holding the
property of the Central Government for which the service charges were not payable under
Article 285 of the Constitution of India as such charges were in the nature of a tax. It was
submitted that in view of the policy taken by the Ministry of Railways, Government of
India such charges cannot be recovered as this was totally exempted but the respondent
Jal Sansthan did not heed to it and they moved the Tahasildar, Sadar, Allahabad for
effecting recovery. Therefore, the appellants were constrained to file the present writ
petition before the High Court of Allahabad.
3. The writ petition was contested by the respondents and they filed their reply and
pointed out that in view of various circulars of the Ministry of Railways, the appellants
have been paying the service charges to the Jal Sansthan and in that connection it was
pointed out that other Central Government Offices situated in Allahabad i.e. Telephone
Department; Post Offices; Accountant General Office; Central Excise Department;
Income Tax Offices were all making regular payment of service charge and sewerage
charge to the Jal Sansthan, Allahabad. It was also pointed out that earlier the demand of
service charges was being paid by the Railway Administration to the Allahabad Nagar
Mahapalika but with the establishment of Allahabad Jal Sansthan under the U.P. Water
Supply and Sewerage Act, 1975 (hereinafter to be referred to as 'the Act') the aforesaid
charges were being levied and realized by the Allahabad Jal Sansthan.
4

. On the basis of these pleadings the question that came up before the Division Bench of
the High Court was whether such demand raised by Allahabad Jal Sansthan for the
services rendered by it to the Railway colonies was sustainable or not. The short question
was whether Article 285 of the Constitution of India will exempt the Railway
Administration from paying the water and sewerage charges under the Act of 1975. In
this connection, reference was specially made to two decisions of this Court i.e. Union of
India v. Purna Municipal Council and Ors. [(1992) 1 SCC 100] and Union of India and
Anr. v. Ranchi Municipal Corporation, Ranchi and Ors. [(1996) 7 SCC 542]. There is no
dispute that the bulk of water is supplied by the Jal Sansthan for maintenance of the
railway platforms as well as railway colonies and the Jal Sansthan is catering to the need
of maintaining the sewerage system not only at the railway stations but in the adjoining
areas and also the residential quarters, offices, godowns, shades are being maintained by
the Union of India through the Railways. The contention of the appellants in the writ
petition was that in view of the aforesaid two decisions of this Court the question is no
more res integra and the Jal Sansthan cannot charge for the supply of water and
maintenance of sewerage system. In this connection, Section 184 of the Railways Act,
1989 was also referred to which lays down that the railway administration shall not be
liable to pay any tax in aid of the funds of any authority unless the Central Government
by notification declares the railway administration to be liable to pay the tax specified in
such notification. In this connection, Clause (I) of Article 289 of the Constitution was
also pressed into service. But the High Court did not dwell on this aspect in absence of
the material placed in support thereof and did not permit to raise this plea. 1992 AIR
SCW 1737

5. As against this, it was contended on behalf of the respondents that the writ petitioner-
appellants herein were paying its predecessors the amount for water and sewerage
charges and there was no reason why they should discontinue the payment for the same.
However, it was contended by the appellants that merely because they were paying the
charges that does not become law or a vested right accrued in favour of the
@page-SC523
respondents to continue with the charges.
6. It was contended by the Jal Sansthan that the so called water and sewer charges is not a
tax and it is a fee for the services rendered by the Jal Sansthan. Hence the exemption
granted to the property of the Union from the State taxation under Article 285 of the
Constitution has no relevance to the present case as the property of the Union of India
was not being subjected to any tax. It was only a fee which has been charged for the
services rendered and this has been the practice which is prevalent since long as other
departments of the Central Government have been paying the same. In this background,
the Division Bench of the High Court after exhaustively dealing with several cases on the
subject came to the conclusion that in view of the provisions of the Act of 1975 and with
reference to Article 285 and Article 289 of the Constitution of India, consumption charges
on water or such services which are rendered under the statutory obligation for which the
Jal Sansthan is to maintain its own funds is a fee and not tax. Hence, the writ petitioners
were liable to pay such charges and they must honour the bills which have been served
upon them. It was also observed that the appellants have been uninterruptedly paying
such bills as a contractual obligation. It was also pointed out that the railway is not being
charged with any tax but what is being charged is a fee for the service rendered by the Jal
Sansthan. Aggrieved against this order passed by the Division Bench of the High Court,
the present appeal was filed by the appellants.
7. We have heard learned counsel for the parties and perused the record. One thing is very
clear from the facts, namely, that the Jal Sansthan which has been established under the
Act of 1975, has taken over certain duties of the Municipality i.e. supply of water and
maintenance of sewer. It is also not in dispute that prior to this, the railways were paying
for the services like water and sewer to the then Municipality and likewise other
departments of the Central Government are also paying the same charges. Therefore, the
question is whether the service charges like supply of water and sewerage can be said to
be a tax on the properties of the Railways.
8. Article 285 exempts the property of the Union from State taxation. Article 285 of the
Constitution reads as under :
"285. Exemption of property of the Union from State taxation.- (1) The property of the
Union shall, save in so far as Parliament may by law otherwise provide, be exempt from
all taxes imposed by a State or by any authority within a State.
(2) Nothing in clause (1) shall, until Parliament by law otherwise provides, prevent any
authority within a State from levying any tax on any property of the Union to which such
property was immediately before the commencement of this Constitution liable or treated
as liable, so long as that tax continues to be levied in that State."
9. From a perusal of Article 285 it is clear that no property of the Union of India shall be
subject to tax imposed by the State, save as Parliament may otherwise provide. The
question is whether 'the charges for' supply of water and maintenance of sewerage is in
the nature of a tax or a fee for the services rendered by the Jal Sansthan. There is a
distinction between a tax and a fee, and hence one has to see the nature of the levy
whether it is in the nature of tax or whether it is in the nature of fee for the services
rendered by any instrumentality of the State like the Jal Sansthan. There is no two
opinion in the matter that so far as supply of water and maintenance of sewerage is
concerned, the Jal Sansthan is to maintain it and it is they who bear all the expenses for
the maintenance of sewerage and supply of water. It has to create its own funds and
therefore, levy under the Act is a must. In order to supply water and maintain sewerage
system, the Jal Sansthan has to incur the expenditure for the same. It is in fact a service
which is being rendered by the Jal Sansthan to the Railways, and the Railways cannot
take this service from the Jal Sansthan without paying the charges for the same. Though
the expression tax has been used in the Act of 1975 but in fact it is in the nature of a fee
for the services rendered by the Jal Sansthan. What is contemplated under Article 285 is
taxation on the property of the Union. In our opinion the Jal Sansthan is not charging any
tax on the property of the Union; what is being charged is a fee for services rendered to
the Union through the Railways. Therefore, it is a plain and simple charge for service
rendered by the Jal Sansthan for which the Jal Sansthan has to maintain staff for regular
supply of water as well as for sewerage system of the effluent discharge by the railway
over their
@page-SC524
platform or from their staff quarters. It is in the nature of a fee for service rendered and
not any tax on the property of the Railways.
10. The distinction has to be kept in mind between a tax and a fee. Exemption under
Article 285 is on the levy of any tax on the property of the Union by the State, and
exemption is not for charges for the services rendered by the State or its instrumentality
which in reality amounts to a fee. In this connection, a reference was made to the decision
of this Court in Re : Sea Customs Act (1878), S.20(2) [AIR 1963 SC 1760]. This was a
case in which a reference was made by the President of India with regard to levy of
custom and excise duties on the State under Article 289 of the Constitution of India
wherein Sinha, CJ., Gajendragadkar, Wanchoo and Shah,JJ. answered the question at
paragraph 31 as follows :
"(31) For the reasons given above, it must be held that the immunity granted to the States
in respect of Union Taxation does not extend to duties of customs including export duties
or duties of excise. The answer to the three questions referred to us must, therefore, be in
the negative.
11. But a contrary view was taken by S.K.Das, Sarkar and Das Gupta,JJ. They concluded
in paragraph 71 as follows:
"(71) For the reasons given above our opinion is that the answers to the three questions
referred to this court must be in the affirmative and against the stand taken by the Union."
12. Hidayatullah, J. answering the question in paragraph 121, held as follows :
"(121) My answers to the questions are:
(1) The provisions of the Art. 289 of the Constitution preclude the Union from imposing
or authorizing the imposition of, customs duties on the import or export of the property of
a State used for purpose other than those specified in cl. (2) of that Article, if the
imposition is to raise revenue but not to regulate external trade.
(2) The provisions of Art. 289 of the Constitution of India preclude the Union from
imposing, or authorizing the imposition of excise duties on the production or manufacture
in India of the property of a State used for purposes other than those specified in cl.(2) of
that Article."
13. Ayyangar,J. has also expressed a separate opinion concurring with the Chief Justice.
This decision on reference of the President of India only dealt with the question of Article
289 of the Constitution and we are not concerned in the present case with the effect of
Article 289 which is, so far as the present controversy is concerned, of no useful
assistance.
14

. Learned counsel for the appellant has relied on the decision of this Court in Union of
India v Purna Municipal Council (supra). In this case, the Railways challenged the notice
of demand issued by Purna Municipal Council claiming Rs.28,400/- by way of 'service
charges' due for the period from 1954 to 1960. The Union of India made a reference to
Article 285 of the Constitution of India read with Section 135 of the Indian Railways Act,
1890. It is not clear from this decision whether the service charge demanded by the Purna
Municipal Council was in reality a tax on the property of the Union or a charge for some
service rendered, rather the decision proceeded on the assumption that it was a tax and
not a fee. The Court disposed of the matter holding as follows: 1992 AIR SCW 1737,
Para 5

"The interplay of the constitutional and legal provisions being well cut and well defined
requires no marked elaboration to stress the point. Accordingly, we allow this appeal, set
aside the judgment and order of the High Court and issue the writ and direction asked for
in favour of the Union of India restraining the respondent council from raising demands
on the railway in regard to service charges. We make it clear that the rights of the local
authority as flowing under Section 135 of the Indian Railways Act, 1890 stand preserved
in the event of the Central Government moving into the matter, if not already moved. In
the circumstances of the case, however, there will be no order as to costs."
15. From this it is not clear whether the impugned demand was a charge for some service
rendered, such as that which is involved in the present case with regard to water supply or
with regard to sewerage. As already pointed out, what is prohibited by Article 285 is
taxation on the property of the Railways and it does not prohibit charge of a fee on
account of some service rendered by the local bodies or instrumentality of the State like
supply of water or maintenance of sewerage. Such a charge would be in the nature of a
fee and not a tax.
16

. The other decision which has been 1992 AIR SCW 1737

@page-SC525
heavily relied on by the appellants in Ranchi Municipal Corporation, Ranchi and Ors.
(supra). In this case, their Lordships merely followed the decision in Purna Municipal
Council (supra) and disposed of the matter. Again the question is what was the nature of
the demand raised by the State against the Railways. In this case, their Lordships after
following the judgment in Purna Municipal Council (supra) observed as follows :
"Therefore, it cannot be construed that there is any contract between the Union of India
and the Municipality. In view of the fact that the Municipality has no right to demand
service charges from the Union of India, the demand made by the Municipality is clearly
ultra vires its power. It is true that earlier WP No.2844 of 1992 was filed and was
dismissed by the High Court and the special leave was refused by this Court on the
ground of gross delay."
It was also observed at paragraph 5 as follows :
"It is now settled law that the summary dismissal does not constitute res judicata for
deciding the controversy. Moreover, this being a recurring liability which is ultra vires the
power, earlier summary dismissal of the case does not operate as a res judicata."
17

. Therefore, from the perusal of these two decisions what emerges is that no property of
the Union of India can be subjected to State taxation, but these decisions do not deal with
a charge for services rendered by any State or an instrumentality of the State. In this
connection, our attention was invited to a decision of this Court in New Delhi Municipal
Council v. State of Punjab and Ors. [ (1997) 7 SCC 339]. This was also a case where
Articles 289, 246(4), 245(1) and 1(2), 3(b) and 285 came up for consideration. As per the
majority it was held that levy of property tax on such lands/buildings which are not used
or occupied for the purpose of any trade or business carried on by the State Government
with profit motive was invalid and incompetent by virtue of Article 289(1). But if the
levy is on lands/ buildings used or occupied for any trade or business carried on by or on
behalf of the State Governments, then by virtue of Article 289(2), the levy would be
valid. It was also observed that it was for the authorities under the enactments to
determine with notice to the affected State Governments, which land or building is used
or occupied for the purposes of any trade or business carried on by or on behalf of the
State Government. As against this, the minority view was that the States are entitled to
exemption from levy of property tax on their lands/ buildings situated within NCT
including those occupied for trade or business purposes. This case also does not throw
any light on the question whether the services which are being given by the State
Government or its instrumentality or the local bodies like supply of water and
maintenance of sewerage will have the exemption under Article 285 of the Constitution ?
This was also a case with regard to levy on the property of the State. So far as we are
concerned in the present case, there is no levy on the property of the Union of India.
Therefore, this case also does not provide us any useful assistance. As against this, our
attention was invited to a subsequent decision of this Court in Municipal Corporation,
Amritsar v. Senior Superintendent of Post Offices, Amritsar Division and Anr. [(2004) 3
SCC 92]. In this case, their Lordships were directly dealing with charges for the water
supply, street light, drainage services being rendered to P and T Department's buildings
situated within the Municipal limits. In that context, their Lordships held as follows :
1997 AIR SCW 2851
2004 AIR SCW 466, Para 7

"The demand so made was with regard to the services rendered to the respondents'
Department, like water supply, street-lighting, drainage and approach roads to the land
and buildings. In the counter, the respondents averred that they are paying for the services
rendered by the appellant Corporation by way of water and sewerage charges and power
charges separately. It is also categorically averred that no other specific services are being
provided to the respondents for which the tax in the shape of service charges can be
levied and realized from the respondents. There is no provision in the Municipal
Corporation Act for levying service charges. The only provision is by way of tax.
Undisputedly, the appellant Corporation is collecting the tax from general public for
water supply, street-lighting and approach roads etc. Thus, the "tax" was sought to be
imposed in the garb of "service charges". The interplay of the constitutional and legal
provisions being well cut and well defined, it was clearly not within the competence of
the Corporation to impose tax on the property of the Union of India, the same
@page-SC526
being violative of Article 285(1) of the Constitution."
18. In this case, what is clear is that in fact the P and T Department was paying for
water supply and sewerage separately and it was over and above that some service
charges were levied under the garb of service charges which was exempted by the
Constitution. In the present case, what is being charged is in fact water supply and
sewerage. Therefore, so far as this part is concerned, it is affirmed by this Court in the
aforesaid decision. But what is not accepted was that over and above the charges for
supply of water and sewerage and power charges, the Municipal Corporation was levying
service charges which were not contemplated under the Municipal Corporation Act for
levying such service charges. Therefore, indirectly so far as demand for water supply,
sewerage was concerned, it was accepted by the P and T Department and they were
paying the same to the Municipal Corporation.
19

. Our attention was invited to another decision of this Court in Sona Chandi Oal
Committee and Ors. v. State of Maharashtra [(2005) 2 SCC 345]. In this case, the
question was whether levy of inspection fee for renewal of moneylender's licence was
valid or not. Their Lordships held that fee charged is regulatory in nature to further the
objects of the Act and it has nexus with services rendered to moneylenders. However, it
was observed that service to be rendered is not a condition precedent and there should be
reasonable relationship between levy of fee and services rendered and in that context,
their Lordships affirmed the validity of levy of fee under the Bombay Money-Lenders
Act, 1946. 2005 AIR SCW 256

20

. Our attention was also invited to a decision of this Court in Vijayalashmi Rice Mill and
Ors. v. Commercial Tax Officers, Palakol and Ors. [(2006) 6 SCC 763]. In this case, their
Lordships considered the distinction between fee, cesses and taxes. Their Lordships held
that ordinarily a tax generates general revenue not for any service rendered. However, the
nomenclature is not important. Sometimes a 'tax' may be in reality a fee, depending upon
its nature. It was observed that the earlier concept of fee has undergone a sea change and
rendering of some specific service to a particular payer of fee is no longer considered
necessary to sustain the levy of fee provided there is a broad and general correlationship
between the totality of the fee imposed and the totality of the expenses on the service
rendered. This discussion makes it clear that the distinction between a tax and a fee
remains, even though the concept of a fee has undergone a sea change. 2006 AIR
SCW 4031

21

. A reference was also made to another decision of this Court in Karya Palak Engineer,
CPWD, Bikaner v. Rajasthan Taxation Board, Ajmer and Ors. [(2004) 7 SCC 195]. In
this case, a three Judge Bench held that Article 285 which contemplates exemption of
Union property from State tax, does not extent to exemption from levy of indirect tax. In
this case, the question was exemption of sales tax in a works contract for erection of
barbed wire. CPWD in terms of the contract supplied the construction materials after
purchasing the same on payment of consideration and was adjusting the value of the
materials in the final bills of the contractor. The question was whether there was
immunity for the property of the Union from the State taxation under Article 285. Their
Lordships held that from the case law it is clear that the Union is not exempted from the
levy of indirect tax under Article 285. Their Lordships after examining the decision in
Re : Sea Customs Act (1878), S.20(2) (supra) in reference by a nine Judge Bench
observed that Article 285 is a mandate and not indirect tax such as sales tax. Their
Lordships concluded with reference to sales tax which reads as follows : 2004 AIR
SCW 4665, Para 15
AIR 1963 SC 1760

"We may in this connection contrast sales tax which is also imposed with reference to
goods sold, where the taxable event is the act of sale. Therefore, though both excise duty
and sales tax are levied with reference to goods, the two are very different imposts; in one
case the imposition is on the act of manufacture or production while in the other it is on
the fact of sale. In neither case therefore can it be said that the excise duty or sales tax is a
tax directly on the goods for in that event they will really become the same tax."
22

. The aforesaid decision came up for consideration in New Delhi Municipal Council
(supra). Their Lordships concluded at paragraph 16 as follows : 1997 AIR SCW 2851

@page-SC527
"From the above judgment of this Court, it is clear that the Union is not exempted from
the levy of indirect tax under Article 285 of the Constitution. The above discussion also
shows that reliance placed on the judgment of this Court in the case of New Delhi
Municipal Council by one of the learned counsel for the appellants is wholly
misconceived and is opposed to his contention with reference to Article 285 of the
Constitution."
23. Though these observations were in reference to Sales Tax Act but the reasoning
equally applies in this case also. In this case what is being charged is for service rendered
by the Jal Sansthan i.e. an instrumentality of the State under the Act of 1975. Section 52
of the Act states that the Jal Sansthan can levy tax, fee and charge for water supply and
for sewerage services rendered by it as water tax and sewerage tax at the rates mentioned
therein. Though the charge was loosely termed as 'tax' but as already mentioned before,
nomenclature is not important. In substance what is being charged is fee for the supply of
water as well as maintenance of the sewerage system. Therefore, in our opinion, such
service charges are a fee and cannot be said to be hit by Article 285 of the Constitution. In
this context it is to be made clear that what is exempted by Article 285 is a tax on the
property of the Union of India but not a charge for services which are being rendered in
the nature of water supply, for maintenance of sewerage system. Therefore, in our
opinion, the view taken by the Division Bench of the Allahabad High Court is correct that
the charge is a fee, being service charges for supply of water and maintenance of
sewerage system, which cannot be said to be tax on the property of the Union. Hence it is
not violative of the provisions of Article 285 of the Constitution.
24. As a result of our above discussion, we do not find any merit in this appeal and the
same is dismissed. There will be no order as to costs.
Appeal dismissed.
AIR 2008 SUPREME COURT 527 "Didigam Bikshapathi v. State of A. P."
(From : Andhra Pradesh)*
Coram : 2 Dr. A. PASAYAT AND TARUN CHATTERJEE, JJ.
Criminal Appeal No. 1643 of 2007 (arising out of SLP (Cri.) No. 2205 of 2006), D/- 29
-11 -2007.
Didigam Bikshapathi and Anr. v. State of A.P.
(A) Criminal P.C. (2 of 1974), S.482 - INHERENT POWERS - Inherent powers - Courts
have all such powers as are necessary to do right and to undo wrong in course of
administration of justice on principle that when the law gives a person anything it gives
him that without which it cannot exist. (Para 7)
(B) Criminal P.C. (2 of 1974), S.482 - INHERENT POWERS - HIGH COURT - Inherent
powers - Powers possessed by High Court are very wide - Requires great caution in its
exercise - Inherent power should not be exercised to stifle a legitimate prosecution.
(2005) 13 SCC 540, 2006 AIR SCW 2330, AIR 1992 SC 604, Relied on. (Paras 9, 10)
(C) Penal Code (45 of 1860), S.306 - Criminal P.C. (2 of 1974), S.482 - ABETMENT TO
SUICIDE - INHERENT POWERS - Abetment to commit suicide - Proceedings for -
Quashing of - Victim committed suicide having been insulted and humiliated by
utterances made by accused - Suicide note clearly referred to background in which victim
took extreme step of taking away his own life - Further, acts of accused-appellants and
roles played by them were also referred in suicide note - Prayer to quash proceedings is
liable to be rejected.
2005 AIR SCW 1326, Disting. (Paras 11, 12)
Cases Referred : Chronological Paras
2006 AIR SCW 2330 : AIR 2006 SC 1937 : 2006 Cri LJ 2468 (Rel. on) 10
2005 AIR SCW 1326 : AIR 2005 SC 1775 : 2005 Cri LJ 1737 (Disting Pnt. C) 5, 11
(2005) 13 SCC 540 (Rel. on) 10
AIR 1992 SC 604 : 1992 Cri LJ 527 (Rel. on) 9
AIR 1960 SC 866 : 1960 Cri LJ 1239 (Rel. on, Pnt. B) 8
G. V. Choudhari and K. Shivraj Choudhuri, for Appellants; P. Vinay Kumar and Mrs. D.
Bharathi Reddy, for Respondent.
@page-SC528

* Cri. Petn. No. 2758 of 2002, D/- 29-3-2006 (AP).


Judgement
1. Dr. ARIJIT PASAYAT, J.:-Leave granted.
2. Challenge in this appeal is to the judgment rendered by a learned Single Judge of the
Andhra Pradesh High Court, dismissing the petition filed by the appellants under Section
482 of the Code of Criminal Procedure, 1973 (for short 'the Code'). Prayer was to quash
the proceedings in SC No.498 of 2001 on the file of VII Additional Metropolitan
Sessions Judge, Hyderabad, initiated against them for commission of offence punishable
under Section 306 of the Indian Penal Code, 1860 (in short 'IPC').
3. Accusations which led to the institution of the proceedings are essentially are as
follows:
Budida Krishnamurthy (hereinafter referred to as the 'deceased') had close friendship with
the appellant (A1). About four years back he appointed deceased and others as field
officers in his finance firm namely; Uma Hire Purchase and Finance. While so, the
appellant No.1 joined as a partner in Kanaka Mahalaxmi Real Estate Ventures run by
Mekala Ravi and Mekala Venu. The deceased and two other field officers namely; Budida
Laxmaiah (L.W.7) and Thandra Mallaiah (L.W.8) sold about 15 plots in that group to
Kommaipalli villagers and collected various amounts from them and handed over the
same to the appellant No.1. As he did not pay the money to the Kanaka Mahalaxmi Real
Estate Ventures, the other partners did not register the plots in favour of the persons, who
paid the money to the deceased. Since the deceased demanded for registration of the plots
in favour of the prospective purchasers, he (appellant No.1) escaped with his family from
Jangaon and was staying at his in-laws house. The deceased went there and demanded
registration of the plots, but the appellants abused him in filthy language and the accused
neither registered the plots nor returned the amount. Due to the mental harassment and
unable to bear the pressure from the purchasers of the plots, the deceased committed
suicide by falling under an un-known train in the night of 17.4.2001 leaving a suicide
note narrating the reasons for his committing suicide.
4. Before the High Court the stand was that the ingredients necessary to constitute
offence under Section 306 IPC are absent. There is no element of abetment. The High
Court did not accept the contention taking note of the statement made in the suicide note.
The High Court felt that this was not a fit case where the jurisdiction under Section 482
of the Code is to be exercised.
5. In support of the appeal learned counsel for the appellant submitted that there was no
question of abetment. Merely because the person committed suicide having been insulted
and humiliated due to the comments or utterances made by the accused, that does not
constitute an offence punishable under Section 306 IPC. Therefore, the High Court ought
to have quashed the proceedings. Strong reliance was placed on a decision of this Court
in Netai Dutta v. State of West Bengal (2005 AIR SCW 1326). Further it was submitted
that there was only a vague reference to appellant No.2 wife of appellant No.1, and on
that score, the appeal deserves to be allowed so far as she is concerned.
6. In response, learned counsel for the respondent submitted that the suicide note clearly
refers to various acts of the appellants due to which the unfortunate step of committing
suicide was taken by the victim and in any event it is not a fit case where jurisdiction
under Section 482 is to be exercised.
7. Section 482 does not confer any new powers on the High Court. It only saves the
inherent power which the Court possessed before the enactment of the Code. It envisages
three circumstances under which the inherent jurisdiction may be exercised, namely, (i) to
give effect to an order under the Code (ii) to prevent abuse of the process of court, and
(iii) to otherwise secure the ends of justice. It is neither possible nor desirable to lay down
any inflexible rule which would govern the exercise of inherent jurisdiction. No
legislative enactment dealing with procedure can provide for all cases that may possibly
arise. Courts, therefore, have inherent powers apart from express provisions of law which
are necessary for proper discharge of functions and duties imposed upon them by law.
That is the doctrine which finds expression in the section which merely recognizes and
preserves inherent powers of the High Courts. All courts, whether civil or criminal
possess, in the absence of any express provision, as inherent in their constitution, all such
@page-SC529
powers as are necessary to do the right and to undo a wrong in course of administration
of justice on the principle "quando lex aliauid alicui concedit, concedere videtur et id sine
guo res ipsae esse non potest" (when the law gives a person anything it gives him that
without which it cannot exist). While exercising powers under the section, the court does
not function as a court of appeal or revision. Inherent jurisdiction under the section
though wide has to be exercised sparingly, carefully and with caution and only when such
exercise is justified by the tests specifically laid down in the section itself. It is to be
exercised ex debito justitiae to do real and substantial justice for the administration of
which alone courts exist. Authority of the court exists for advancement of justice and if
any attempt is made to abuse that authority so as to produce injustice, the court has power
to prevent abuse. It would be an abuse of process of the court, to allow any action which
would result in injustice and prevent promotion of justice, in exercise of the powers court
would be justified to quash any proceeding if it finds that initiation/continuance of it
amounts to 'abuse of the process of court or quashing of these proceedings would
otherwise serve the ends of justice. When no offence is disclosed by the report, the court
may examine the question of fact. When a report is sought to be quashed, it is permissible
to look into the materials to assess what the report has alleged and whether any offence is
made out even if the allegations are accepted in toto.
8. In R.P. Kapur v. State of Punjab AIR 1960 SC 866 this Court summarized some
categories of cases where inherent power can and should be exercised to quash the
proceedings.
(i) where it manifestly appears that there is a legal bar against the institution or
continuance e.g. want of sanction;
(ii) where the allegations in the first information report or complaint taken at its face
value and accepted in their entirety do not constitute the offence alleged;
(iii) where the allegations constitute an offence, but there is no legal evidence adduced or
the evidence adduced clearly or manifestly fails to prove the charge.
9

. In dealing with the last category, it is important to bear in mind the distinction between a
case where there is no legal evidence or where there is evidence which is clearly
inconsistent with the accusations made, and a case where there is legal evidence which,
on appreciation, may or may not support the accusations. When exercising jurisdiction
under Section 482 of the Code the High Court would not ordinarily embark upon an
enquiry whether the evidence in question is reliable or not or whether on a reasonable
appreciation of it accusation would not be sustained. That is the function of the trial
Judge. Judicial process should not be an instrument of oppression, or, needless
harassment. Court should be circumspect and judicious in exercising discretion and
should take all relevant facts and circumstances into consideration before issuing process,
lest it would be an instrument in the hands of a private complainant to unleash vendetta to
harass any person needlessly. At the same time the section is not an instrument handed
over to an accused to short-circuit a prosecution and bring about its sudden death. The
scope of exercise of power under Section 482 of the Code. and the categories of cases
where the High Court may exercise its power under it relating to cognizable offences to
prevent abuse of process of any court or otherwise to secure the ends of justice were set
out in some detail by this Court in State of Haryana v. Bhajan Lal (1992 Supp (1) SCC
335). A note of caution was, however, added that the power should be exercised sparingly
and that too in rarest of rare cases. The illustrative categories indicated by this Court are
as follows: AIR 1992 SC 604
"(1) Where the allegations made in the first information report or the complaint, even if
they are taken at their face value and accepted in their entirety do not prima facie
constitute any offence or make out a case against the accused.
(2) Where the allegations in the first information report and other materials, if any,
accompanying the FIR do not disclose a cognizable offence, justifying an investigation
by police officers under Section 156 (1) of the Code except under an order of a
Magistrate within the purview of Section 155 (2) of the Code.
(3) Where the uncontroverted allegations made in the FIR or complaint and the evidence
collected in support of the same do not disclose the commission of any offence and make
out a case against the accused.
(4) Where the allegations in the FIR do
@page-SC530
not constitute a cognizable offence but constitute only a non-cognizable offence, no
investigation is permitted by a police officer without an order of a Magistrate as
contemplated under Section 155 (2) of the Code.
(5) Where the allegations made in the FIR or "complaint are so absurd and inherently
improbable on the basis of which no prudent person can ever reach a just conclusion that
there is sufficient ground for proceeding against the accused.
(6) Where there is an express legal bar engrafted in any of the provisions of the Code or
the Act concerned (under which a criminal proceeding is instituted) to the institution and
continuance of the proceedings and/or where there is a specific provision in the Code or
Act concerned, providing efficacious redress for the grievance of the aggrieved party.
(7) Where a criminal proceeding is manifestly attended with mala fide and/or where the
proceeding is maliciously instituted with an ulterior motive for wreaking vengeance on
the accused and with a view to spite him due to private and personal grudge."
10

. As noted above, the powers possessed by the High Court under Section 482 of the Code
are very wide and the very plenitude of the power requires great caution in its exercise.
Court must be careful to see that its decision in exercise of this power is based on sound
principles. The inherent power should not be exercised to stifle a legitimate prosecution.
The High Court being the highest court of a State should normally refrain from giving a
prima facie decision in a case where the entire facts are incomplete and hazy, more so
when the evidence has not been collected and produced before the Court and the issues
involved, whether factual or legal, are of magnitude and cannot be seen in their true
perspective without sufficient material. Of course, no hard and fast rule can be laid down
in regard to cases in which the High Court will exercise its extraordinary jurisdiction of
quashing the proceeding at any stage.( See State of Orissa v. Saroj Kumar Sahoo (2005)
13 SCC 540 and Minu Kumari v. State of Bihar AIR 2006 SC 1937). 2006 AIR
SCW 2330

11

. The suicide note clearly refers to the background in which the victim took the extreme
step of taking away his own life by committing suicide. It is not a case where there is no
reference to any act by the accused. In Netai Dutta's case (supra) para 6 it was observed
as follows: 2005 AIR SCW 1326

"6. In the suicide note, except referring to the name of the appellant at two places, there is
no reference of any act or incidence whereby the appellant herein is alleged to have
committed any wilful act or omission or intentionally aided or instigated the deceased
Pranab Kumar Nag in committing the act of suicide. There is no case that the appellant
has played any part or any role in any conspiracy, which ultimately instigated or resulted
in the commission of suicide by deceased Pranab Kumar Nag."
12. In the instant case the suicide note clearly refers to the acts of the accused-appellants
and the roles played by them. Therefore, the High Court rightly rejected the prayer of
exercise of power under Section 482 of the Code. We make it clear that any observation
made by the High Court and by us while dismissing of the present appeal shall be
construed to be determinative factor in the trial.
Appeal dismissed.
AIR 2008 SUPREME COURT 530 "Chaturbhuj v. Sita Bai"
(From : Madhya Pradesh)*
Coram : 2 Dr. A. PASAYAT AND AFTAB ALAM, JJ.
Criminal Appeal No. 1627 of 2007 (arising out of SLP (Cri.) No. 4379 of 2006), D/- 27
-11 -2007.
Chaturbhuj v. Sita Bai.
Criminal P.C. (2 of 1974), S.125 - MAINTENANCE - WORDS AND PHRASES -
Maintenance - Claim by deserted wife - Wife earning some income - Does not disentitle
her - Phrase "unable to maintain herself" - Means unable to maintain herself in way she
was living with her husband.
The respondent had filed an application under Section 125 of Cr.P.C. claiming
maintenance from the appellant. The appellant and the respondent had entered into
marital knot about four decades back and for more than two decades they were living
separately. In the application it was claimed by respondent that she was unemployed and
unable to maintain herself. The object of the maintenance proceedings is not to punish a
person for his past neglect, but to prevent
@page-SC531
vagrancy by compelling those who can provide support to those who are unable to
support themselves and who have a moral claim to support. The phrase "unable to
maintain herself" would mean that means available to the deserted wife while she was
living with her husband and would not take within itself the efforts made by the wife after
desertion to survive somehow. Under the law the burden is placed in the first place upon
the wife to show that the means of her husband are sufficient. There is no dispute that the
appellant has the requisite means. But there is an inseparable condition which has also to
be satisfied that the wife was unable to maintain herself. These two conditions are in
addition to the requirement that the husband must have neglected or refused to maintain
his wife. The appellant has placed material to show that the respondent-wife was earning
some income. That is not sufficient to rule out application of Section 125. It has to be
established that with the amount she earned the respondent-wife was able to maintain
herself. Whether the deserted wife was unable to maintain herself, has to be decided on
the basis of the material placed on record. Where the personal income of the wife is
insufficient, she can claim maintenance under Section 125. The test is whether the wife is
in a position to maintain herself in the way she was used to in the place of her husband.
The conclusions of Courts that respondent-wife unable to maintain herself was factual
and cannot be interfered with in absence of perversity.
AIR 1978 SC 1807, 2005 AIR SCW 1601, AIR 1975 SC 83, Relied on.
(Paras 5, 7, 8, 9)
Cases Referred : Chronological Paras
2005 AIR SCW 1601 : AIR 2005 SC 1809 : 2005 Cri LJ 1241 (Rel. on) 5
AIR 1978 SC 1807 : 1979 Cri LJ 3 (Rel. on) 5
AIR 1975 SC 83 : 1975 Cri LJ 40 (Rel. on) 8
Shashindra Tripathi, Mrs. Sharad Tripathi and Debasis Misra, for Appellant; Shashi
Bhushan Kumar, for Respondent.
* Misc. Cri. Case No. 1385 of 2006, D/- 31-3-2006 (MP), (Indore Bench).
Judgement
Dr. ARIJIT PASAYAT, J.:- Leave granted.
2. Challenge in this appeal is to the order passed by a learned Single Judge of the
Madhya Pradesh High Court, Indore Bench, dismissing the revision petition filed by the
appellant in terms of Section 482 of the Code of Criminal Procedure, 1973 (in short
'Cr.P.C.'). The challenge before the High Court was to the order passed by learned
Judicial Magistrate, First Class, Neemuch, M.P. as affirmed by the learned Additional
Sessions Judge, Neemuch, M.P. The respondent had filed an application under Section
125 of Cr.P.C. claiming maintenance from the appellant. Undisputedly, the appellant and
the respondent had entered into marital knot about four decades back and for more than
two decades they were living separately. In the application it was claimed that she was
unemployed and unable to maintain herself. Appellant had retired from the post of
Assistant Director of Agriculture and was getting about Rs.8,000/- as pension and a
similar amount as house rent. Besides this, he was lending money to people on interest.
The appellant claimed Rs.10,000/- as maintenance. The stand of the appellant was that
the applicant was living in the house constructed by the present appellant who had
purchased 7 bighas of land in Ratlam in the name of the applicant. She let out the house
on rent and since 1979 was residing with one of their sons. The applicant sold the
agricultural land on 13.3.2003. The sale proceeds were still with the applicant. The
appellant was getting pension of about Rs.5,700/- p.m. and was not getting any house rent
regularly. He was getting 2-3 thousand rupees per month. The plea that the appellant had
married another lady was denied. It was further submitted that the applicant at the
relevant point of time was staying in the house of the appellant and electricity and water
dues were being paid by him. The applicant can maintain herself from the money
received from the sale of agricultural land and rent. Considering the evidence on record,
the trial Court found that the applicant-respondent did not have sufficient means to
maintain herself.
3. Revision petition was filed by the present appellant. Challenge was to the direction to
pay Rs.1500/- p.m. by the trial Court. The stand was that the applicant was able to
maintain herself from her income was reiterated. The revisional court analysed the
evidence and held that the appellant's monthly income was more than Rs.10,000/- and the
amount received as rent by the respondent-claimant was not sufficient to maintain herself.
The revision was accordingly dismissed. The matter was further
@page-SC532
carried before the High Court by filing an application in terms of Section 482 Cr.P.C. The
High Court noticed that the conclusions have been arrived at on appreciation of evidence
and, therefore, there is no scope for any interference.
4. Section 125 Cr.P.C. reads as follows:
"125. (1) If any person having sufficient means neglects or refuses to maintain-
(a) his wife, unable to maintain herself, or
(b) his legitimate or illegitimate minor child, whether married or not, unable to maintain
itself, or
(c) his legitimate or illegitimate child (not being a married daughter) who has attained
majority, where such child is, by reason of any physical or mental abnormality or injury
unable to maintain itself, or
(d) his father or mother, unable to maintain himself or herself,
a Magistrate of the First Class may, upon proof of such neglect or refusal, order such
person to make a monthly allowance for the maintenance of his wife or such child, father
or mother, at such monthly rate not exceeding five hundred rupees in the whole, as such
Magistrate thinks fit, and to pay the same to such person as the Magistrate may from time
to time direct:
Provided that the Magistrate may order the father of a minor female child referred to in
clause (b) to make such allowance, until she attains her majority, if the Magistrate is
satisfied that the husband of such minor female child, if married, is not possessed of
sufficient means.
Explanation.- For the purposes of this Chapter,
(a) 'minor' means a person who, under the provisions of the Indian Majority Act, 1875 (9
of 1875), is deemed not to have attained his majority;
(b) 'wife' includes a woman who has been divorced by, or has obtained a divorce from,
her husband and has not remarried."
["(2) Any such allowance for the maintenance or interim maintenance and expenses of
proceeding shall be payable from the date of the order, or, if so ordered, from the date of
the application for maintenance or interim maintenance and expenses of proceeding, as
the case may be.";]
(3) If any person so ordered fails without sufficient cause to comply with the order, any
such Magistrate may, for every breach of the order, issue a warrant for levying the
amount due in the manner provided for levying fines, and may sentence such person, for
the whole, or any part of each month's allowance4 [allowance for the maintenance or the
interim maintenance and expenses of proceeding, as the case may be] remaining unpaid
after the execution of the warrant, to imprisonment for a term which may extend to one
month or until payment if sooner made:
Provided that no warrant shall be issued for the recovery of any amount due under this
section unless application be made to the Court to levy such amount within a period of
one year from the date on which it became due:
Provided further that if such person offers to maintain his wife on condition of her living
with him, and she refuses to live with him, such Magistrate may consider any grounds of
refusal stated by her, and may make an order under this section notwithstanding such
offer, if he is satisfied that there is just ground for so doing.
Explanation.- If a husband has contracted marriage with another woman or keeps a
mistress, it shall be considered to be just ground for his wife's refusal to live with him.
(4) No wife shall be entitled to receive an4 [allowance for the maintenance or the interim
maintenance and expenses of proceeding , as the case may be] from her husband under
this section if she is living in adultery, or if, without any sufficient reason, she refuses to
live with her, husband, or if they are living separately by mutual consent.
(5) On proof that any wife in whose favour an order has been made under this section is
living in adultery, or that without sufficient reason she refuses to live with her husband, or
that they are living separately by mutual consent, the Magistrate shall cancel the order."
5

. The object of the maintenance proceedings is not to punish a person for his past neglect,
but to prevent vagrancy by compelling those who can provide support to those who are
unable to support themselves and who have a moral claim to support. The phrase "unable
to maintain herself" in the instant case would mean that means available to the deserted
wife while she was living with her husband and would 2005 AIR SCW 1601

@page-SC533
not take within itself the efforts made by the wife after desertion to survive somehow.
Section 125 Cr.P.C. is a measure of social justice and is specially enacted to protect
women and children and as noted by this Court in Captain Ramesh Chander Kaushal v.
Mrs. Veena Kaushal and Ors. (AIR 1978 SC 1807) falls within constitutional sweep of
Article 15(3) reinforced by Article 39 of the Constitution of India, 1950 (in short the
'Constitution'). It is meant to achieve a social purpose. The object is to prevent vagrancy
and destitution. It provides a speedy remedy for the supply of food, clothing and shelter
to the deserted wife. It gives effect to fundamental rights and natural duties of a man to
maintain his wife, children and parents when they are unable to maintain themselves. The
aforesaid position was highlighted in Savitaben Somabhai Bhatiya v. State of Gujarat and
Ors. (2005 (2) Supreme 503).
6. Under the law the burden is placed in the first place upon the wife to show that the
means of her husband are sufficient. In the instant case there is no dispute that the
appellant has the requisite means.
7. But there is an inseparable condition which has also to be satisfied that the wife was
unable to maintain herself. These two conditions are in addition to the requirement that
the husband must have neglected or refused to maintain his wife. It has to be established
that the wife was unable to maintain herself. The appellant has placed material to show
that the respondent-wife was earning some income. That is not sufficient to rule out
application of Section 125 Cr.P.C. It has to be established that with the amount she earned
the respondent-wife was able to maintain herself.
8. In an illustrative case where wife was surviving by begging, would not amount to her
ability to maintain herself. It can also be not said that the wife has been capable of
earning but she was not making an effort to earn. Whether the deserted wife was unable
to maintain herself, has to be decided on the basis of the material placed on record.
Where the personal income of the wife is insufficient she can claim maintenance under
Section 125 Cr.P.C. The test is whether the wife is in a position to maintain herself in the
way she was used to in the place of her husband. In Bhagwan v. Kamla Devi (AIR 1975
SC 83) it was observed that the wife should be in a position to maintain standard of living
which is neither luxurious nor penurious but what is consistent with status of a family.
The expression "unable to maintain herself" does not mean that the wife must be
absolutely destitute before she can apply for maintenance under Section 125 Cr.P.C.
9. In the instant case the trial Court, the Revisional Court and the High Court have
analysed the evidence and held that the respondent wife was unable to maintain herself.
The conclusions are essentially factual and they are not perverse. That being so there is
no scope for interference in this appeal which is dismissed.
Appeal dismissed.
AIR 2008 SUPREME COURT 533 "Kapildeo Mandal v. State of Bihar"
(From : 2004 (22) East Cri C 345 Patna)
Coram : 2 PRAKASH PRABHAKAR NAOLEKAR AND D. K. JAIN, JJ.
Criminal Appeal Nos. 432-433 of 2005, D/- 29 -11 -2007.
Kapildeo Mandal and Ors. v. State of Bihar.
(A) Evidence Act (1 of 1872), S.3 - EVIDENCE - WITNESS - Eye-witness - Credibility
- Not to be judged merely on basis of his relationship with deceased and strained relation
with accused. (Para 8)
(B) Penal Code (45 of 1860), S.300 - Evidence Act (1 of 1872), S.3 -MURDER -
EVIDENCE - WITNESS - Murder - Allegation that accused persons caused fire arm
injuries to deceased and other family members - Eye-witnesses relatives of deceased -
Their relation with accused strained - Testimony that accused were identified in torch and
lantern light - But no lantern or torch seized by I. O. - Evidence of identification is thus
doubtful - Non-recovery of cartridges /pellets from house - Doctor also did not find any
fire arm injury on body of deceased - Medical evidence thus totally inconsistent with
ocular evidence - Evidence of eye-witnesses doubtful - Accused entitled to benefit of
doubt.
2004 (2) East Cri C 345 (Pat.), Reversed. (Paras 9, 10, 12)
(C) Evidence Act (1 of 1872), S.3 - EVIDENCE - Ocular evidence vis-a-vis medical
evidence - In case of variance between two ocular evidence to get primacy - But in case
of total inconsistency - Medical
@page-SC534
evidence assumes importance. (Para 11)
Cases Referred : Chronological Paras
2004 AIR SCW 5754 : AIR 2004 SC 5056 : 2004 Cri LJ 4973 (Rel. on, Pt. A) 8
2003 AIR SCW 2837 : AIR 2003 SC 3975 : 20033 Cri LJ 300 (Rel. on. Pt. C) 11
2003 AIR SCW 6731 : AIR 2004 SC 1053 : 2004 Cri LJ 640 (Rel. on, Pt. A) 8, 11
2002 AIR SCW 3442 : AIR 2002 SC 2945 : 2002 Cri LJ 4081 (Rel. on, Pt. A) 8
1994 AIR SCW 2298 : 1994 Cri LJ 3848 (Rel. on, Pt. C) 11
AIR 1988 SC 2154 : 1989 Cri LJ 288 (Rel. on, Pt. C) 11
AIR 1979 SC 1194 : 1979 Cri LJ 939 (Rel. on, Pt. C) 11
AIR 1965 SC 202 (Rel. on, Pt. A) 8
AIR 1953 SC 415 : 1953 Cri LJ 1761 (Rel. on, Pt. C) 11
S. B. Sanyal, Sunil Kumar, Sr. Advocates, Ranjan Mukherjee, Ms. Anita Kanungo,
Awanish Sinha and Himanshu Shekhar, with them for Appellants; Anukul Raj, Gopal
Singh and Rituraj Biswas, for Respondent.
Judgement
1. P. P. NAOLEKAR, J.:-These appeals are directed against the judgment and order dated
16th April, 2004 of the High Court of Judicature at Patna passed in Criminal Appeal Nos.
646 of 1987 and 32 of 1988, whereby the appeals of the appellants were dismissed by the
High Court and their conviction and sentence was maintained.
2. Criminal Appeal No. 432 of 2005 by special leave was filed by accused No. 1
Kapildeo Mandal (A-1) and accused No. 5 Milan Mandal (A-5), whereas Criminal
Appeal No. 433 of 2005 by special leave was filed by accused No. 2 Dip Narain Mandal
(A-2), accused No. 3 Subhit Mandal (A-3) and accused No. 4 Pratap Mandal (A-4). After
the case was reserved for judgment by this Court, it was informed by the Registry of this
Court that A-3 Subhit Mandal S/o Chedi Mandal, fell seriously ill and was sent for
treatment to Jawaharlal Nehru Medical College and Hospital, Bhagalpur, and during the
course of treatment he died on 6th February, 2007. The appeal filed by A-3 is, therefore,
rendered infructuous.
3. All the accused persons were convicted by the 3rd Additional Sessions Judge,
Bhagalpur in Sessions Trial No. 34 of 1983 under Sections 302 read with Section149,
IPC and sentenced for imprisonment for life for having committed the offence of murder
of deceased Sitaram Mandal. The accused were also convicted under Sections 452 and
148, IPC. A-1 and A-4 were further convicted under Section 323, IPC. Two appeals
preferred by the accused against their conviction and sentence were dismissed by the
High Court and thus they are before us by special leave.
4. The incident took place in the night between 14th and 15th July, 1979. As per the
prosecution case as reported in the FIR by PW-9 Ramanand Mandal, at about 11.00 p.m.
he woke up after hearing the sound of barking dogs. A lantern was burning in the
verandah of his house. He saw persons, namely, A-1 and his younger brother A?5
entering from the inner courtyard from the roof of his house. One of them went to the
southern side and opened the window from that side. A-2 and A-4 entered the house
along with some other persons. One person opened the main door on the eastern side. A-3
and 5-6 other persons entered from that door. A-3 was carrying gun, whereas A-1, A-2
and A-5 were carrying country-made pistols. Other persons were carrying swords and
lathis. They assaulted the inmate of the house. A-2 fired at Sitaram Mandal as a result of
which he was badly injured. A-3 Subhit Mandal ordered to kill PW-9 Ramanand Mandal,
upon which A-1 fired upon PW-9. A-1 assaulted PW-9 with the butt of a country-made
pistol on the head. One of the miscreants sprinkled kerosene oil of two bottles upon the
body of PW-6 Brahmadeo Mandal and made search of a match-box to set fire upon his
body. PW-5 Mahesh Mandal was assaulted with a lathi. While leaving, the miscreants
took away some articles from the house. On hearing a hue and cry, some villagers
reached the spot. The occurrence and the assault was due to a land dispute between the
parties. In the incident, because of the assault made, Sitaram Mandal died.
5. The prosecution examined PW-1 Madan Mandal who is not an eye-witness. He
reached the place of incident after the incident was over. He stated that when he reached
the place of incident PW-9 Ramanand Mandal informed him that the accused appellants
were the persons involved in the assault made on the deceased Sitaram Mandal and him.
He admitted that all this happened because of a land dispute between the family of
Ramanand and family of accused Dip Narain Mandal. His
@page-SC535
statement was recorded by the police after two days of the incident. PW-2 Jiten Rabidas
also reached the spot after the occurrence. He stated that when he enquired from PW-9
Ramanand Mandal, his brother and female members about the incident, they told that
they did not identify any person. They told him that after committing dacoity dacoits fled
away. PW-3 Adhiklal Mandal also reached the spot after the incident happened. He
admitted that there was enmity between the two families since before the occurrence as
they were litigating. PW-4 Tej Narayan Mandal reached the place of incident afterwards.
He stated that he was informed of the names of the accused appellants. He is a relation of
the complainant party. PW-5 Mahesh Mandal is one of the sons of the deceased. He
deposed that he saw the incident in the flash of a torch-light. Besides the torch light, a
lantern was also burning in the verandah. He identified the accused appellants and stated
that Subhit Mandal (A-3) was armed with a double barrel gun and Dip Narain Mandal
(A-2) with a country-made pistol. A-3 shot with a double barrel gun at his father Sitaram
Mandal. A-2 also fired with the pistol at his father. Pratap Mandal (A-4) hit him on the
head with a knife. He saw the miscreants injuring Ramanand Mandal (PW-9). Thereafter,
after collecting ornaments and clothes, they fled away. As per this witness, the deceased
Sitaram Mandal had received firearm injury and he received injury by a knife. His
statement was recorded after four days of the incident and he admitted that before giving
statement to the police he took advice from the family members about the occurrence and
then made a statement to the police. The witness stated that Subhit Mandal (A-3) put the
gun on the chest of his father and fired at him and Dip Narain Mandal (A-2) put the pistol
near the mouth of his father and fired with the pistol. The witness admitted that there was
a litigation between the families. The witness also stated that he did not give the torch to
the S.I. of Police nor did he produce that torch in the court. PW-6 Brahmadeo Mandal,
another son of the deceased, identified the accused appellants to be the miscreants. He
stated that Subhit Mandal (A-3) was armed with a double barrel gun; A-1, A-2 and A-5
were armed with pistols and A-4 was armed with a knife and a lathi. A-5 poured kerosene
oil on his body and asked for a match-box. It was stated by this witness that the
properties, ornaments, clothes, etc. were looted and taken away by the miscreants. He
saw that his uncle Ramanand had sustained injury. This witness admitted in cross-
examination that he did not see who assaulted his father Sitaram Mandal. The witness
admitted that there was a land dispute between the accused persons and the complainant
party. PW-7 Bimla Devi is wife of PW-9 Ramanand Mandal. She identified the accused
persons in the light of a lantern. She admitted that the accused persons whom she had
identified had covered their faces with gamochha but they had not tied turbans. PW-9
Ramanand Mandal identified all the accused persons and stated that A-3 was armed with
a gun, and A-1, A-2 and A-5 were armed with country-made pistols. A-2 shot at the
deceased. A-3 ordered A-1 to assault him and A-1 shot at him which missed.
6. CW-2 Shreedhar Choubey is the Investigating Officer who recorded the statements of
the witnesses examined by the prosecution. He stated that none of the witnesses produced
any blood-stained clothes before him. PW-5 Mahesh Mandal, son of the deceased, had
not given any list of articles stolen from his house: Neither PW-6 Brahmadeo Mandal told
him that A-1 took away ornaments and clothes. The pouring of kerosene oil on
Brahmadeo was not told to him by PW-7 Bimla Devi nor did she tell him about the
assault made on her. PW-7 did not inform him that she identified the accused persons in
the light of a lantern. During the investigation, he did not find any empty cartridges, burnt
cotton, burnt papers, wads or pellets inside the house or in the outer verandah.
7. CW-1 Dr. Ambroj Kumar Choudhury stated that on 16th July, 1979 he conducted post-
mortem examination on the body of Sitaram Mandal and found the following ante-
mortem injuries:
(i) Abraison 2 ½" x 1 ½" on just below the left eye.
(ii) One stitched wound on frontal bone. On cutting the stitches the dimension of the
wound was found to be ½" x ½" x bone deep. The margins were lacerated. On dissection
underlying tissues were infiltrated with blood and blot clots. On further dissection
fracture of frontal bone was detected.
(iii) One stitched wound just below the left ear. On cutting the stitches the dimension of
the wound was found to be
@page-SC536
1 ½" x ½" x bone deep. The margins were lacerated and the laceration of external pine of
left ear.
(iv) One stitched wound on the left side chin. On cutting the stitches the dimension of the
wound was found to be 1" x ½" x deep to the mouth cavity. On further dissection the
laceration of muscle and fracture of the left ramus of mandible was detected.
(v) One stitched wound on the right side of the chest in between 10th and 12th ribs. On
cutting the stitches the dimension of the wound was found to be ½" x ¼" x deep upto
abdominal cavity. The wound was incised and penetrating, the weapon after passing
through the skin, intercostal muscle adjoining nerve vessel entered into the right lobe of
the liver via right side of the diaphyram, right side of the peritoneal cavity was filled with
blood and the blood clots.
According to the doctor, injuries Nos. (i) and (iii) were simple and injuries Nos. (ii), (iv)
and (v) were grievous in nature. As per the doctor's evidence, injuries Nos. (i) to (iv) were
caused by hard blunt weapon and injury No. (v) was caused by sharp penetrating weapon.
As per the doctor's evidence, the death occurred due to shock and haemorrage on account
of the said injuries. In the cross-examination, the doctor admitted that he did not find any
indication of any firearm injury on the person of the deceased.
8. From the evidence of the witnesses examined by the prosecution, it is clear that there
was animosity between the side of the complainant and the accused persons. There was a
litigation between the parties and they did not have good relations. The witnesses Mahesh
Mandal (PW-5), Brahmadeo Mandal (PW-6), Bimla Devi (PW-7) (wife of the informant)
and the informant Ramanand Mandal (PW-9), are closely related to the deceased. At the
same time, their presence in the house where the incident took place at 11.00 o'clock at
night cannot be doubted. Other witnesses who were examined by the prosecution had
reached the spot after the incident had already taken place and they were not the eye-
witnesses to the incident. Now it is well settled by series of decisions of this Court that
while appreciating the evidence of the witnesses related to the deceased, having strained
relations with the accused party, their evidence cannot be discarded solely on that basis,
but the court is required to carefully scrutinize it and find out if there is scope for taking
view whereby the court can reach to the conclusion that it is a case of false implication.
The credibility of a witness cannot be judged merely on the basis of his close relation
with the deceased and as such cannot be a ground to discard his testimony, if it otherwise
inspires confidence and, particularly so, when it is corroborated by the evidence of
independent and injured witnesses. Speaking for a 5-Judge Bench in a celebrated
judgment, viz., Masalti and Ors. v. The State of Uttar Pradesh, AIR 1965 SC 202 (in para
14), P.B. Gajendragadkar, C.J. said:
"... There is no doubt that when a criminal Court has to appreciate evidence given by
witnesses who are partisan or interested, it has to be very careful in weighing such
evidence. Whether or not there are discrepancies in the evidence; whether or not evidence
strikes the Court as genuine; whether or not the story disclosed by the evidence is
probable, are all matters which must be taken into account. But it would, we think, be
unreasonable to contend that evidence given by witnesses should be discarded only on
the ground that it is evidence of partisan or interested witnesses. Often enough, where
factions prevail in villages and murders are committed as a result of enmity between such
factions, criminal Courts have to deal with evidence of a partisan type. The mechanical
rejection of such evidence on the sole ground that it is a partisan would invariably lead to
failure of justice. No hard and fast rule can be laid down as to how much evidence should
be appreciated. Judicial approach has to be cautious in dealing with such evidence; but
the plea that such evidence should be rejected because it is partisan cannot be accepted as
correct."

In Nallabothu Venkaiah v. State of A.P., (2002) 7 SCC 117 (in para 13), this Court held:
2002 AIR SCW 3442, Para 12

"... The test, in such circumstances, as correctly adopted by the trial court, is that if the
witnesses are interested, the same must be scrutinized with due care and caution in the
light of the medical evidence and other surrounding circumstances. Animosity is double-
edged sword and it can cut both sides. It can be a ground for false implication. It can also
be a ground for assault. ..."

In Ramanand Yadav v. Prabhunath Jha and Ors., (2003) 12 SCC 606 (in para 15), this
Court held :- 2003 AIR SCW 6731, Para 15

@page-SC537
"... But at the same time if the relatives or interested witnesses are examined, the court
has a duty to analyse the evidence with deeper scrutiny and then come to a conclusion as
to whether it has a ring of truth or there is reason for holding that the evidence is biased.
Whenever a plea is taken that the witness is partisan or had any hostility towards the
accused, foundation for the same has to be laid. ...".

In State of Himachal Pradesh v. Mast Ram, AIR 2004 SC 5056 (in para 11), this Court
said:- 2004 AIR SCW 5754

"... The law on the point is well settled that the testimony of the relative witnesses cannot
be disbelieved on the ground of relationship. The only main requirement is to examine
their testimony with caution. . Their testimony was thrown out at the threshold on the
ground of animosity and relationship. This is not the requirement of Law....".
9. In the present case, we find from the evidence of the witnesses examined by the
prosecution as already noticed that the witnesses are related and their relations were
strained with the appellants on account of the litigation. The incident happened at 11.00
o'clock in the night. The witnesses have stated that they have seen the incident and
recognised the appellants either in the torch-light or in the lantern-light which was
burning at their house. It has come in evidence of the witnesses as well as the
Investigating Officer that neither the torch or the lantern was seized by the I.O. during the
course of investigation nor was it produced before the court. In the circumstances, it is
difficult to believe that the appellants have been identified in the torch-light or in the
lantern-light. One of the witnesses Jiten Rabidas (PW-2), who is related to the deceased
and reached the place of occurrence immediately after the incident of dacoity, said that
when he made enquiries from Ramanand Mandal (PW-9), who lodged the FIR, his
brother, and other female members, they specifically told him that they did not identify
the persons who had committed the dacoity in the house. Family members told that after
committing dacoity, they fled away. PW-7 Bimla Devi, wife of Ramanand Mandal, has
stated that the persons who had committed dacoity at their residence had tied gamochha
on their faces. All the eye-witnesses have categorically stated that guns and country-made
pistols were used by the accused-appellants in commission of the crime. Shreedhar
Choubey (CW-2), who was Investigating Officer, has deposed that he did not find any
empty cartridge, burnt cotton, burnt paper, pellets inside the house or in the outer
verandah and so long he was investigating the case, no bullets or pellets were received at
the police station from the hospital. Therefore, it is clear that he has not seized any
pellets, cartridges or bullets from the place of incident. There is no evidence on record
that either the gun or the country-made pistols were recovered from the accused-
appellants by the I.O. The statement of Dr. A.K. Choudhury (CW-1) indicates that the
doctor did not find any pellet or cartridge from the body of the deceased in post-mortem.
That apart, it is the case of the prosecution that Ramanand Mandal (PW-9) received
injury on the head. He was examined by the doctor but no medical evidence was
produced by the prosecution to prove the injury on the person of PW-9.
10. On the face of the evidence led by the prosecution, the medical evidence of the
injuries sustained by the deceased in this case assumes significant importance. All the
eye-witnesses have categorically stated that the deceased was injured by the use of
firearm, whereas the medical evidence given by Dr. A.K. Choudhury (CW-1) specifically
indicates that no firearm injuries were found on the person of the deceased. The doctor
has stated: "I did not find any indication of any firearm injury on the person of the
deceased. No pellets, bullets or any cartridge were found by me in any of the wounds
found by me."
11

. It is now well settled by series of decisions of this Court that while appreciating
variance between medical evidence and ocular evidence, oral evidence of eye-witness has
to get primacy as medical evidence is basically opinionative. [See Mange v. State of
Haryana (1979) 4 SCC 349 (conviction based on sole testimony of eye-witness); State of
U.P. v. Krishna Gopal and Anr., (1988) 4 SCC 302 (in para 24); and Ramanand Yadav v.
Prabhu Nath Jha and Ors., (2003) 12 SCC 606 (in para 17)]. But when the court finds
inconsistency in the evidence given by the eye-witnesses which is totally inconsistent to
that given by the medical experts, then evidence is appreciated in different perspective by
the AIR 1979 SC 1194
AIR 1988 SC 2154
2003 AIR SCW 6731, Para 17
AIR 1953 SC 415, Para 10

@page-SC538
courts. In Mohinder Singh v. The State, (1950) SCR 821 (at page 828), this Court said:-
"... In a case where death is due to injuries or wounds caused by a lethal weapon, it has
always been considered to be the duty of the prosecution to prove by expert evidence that
it was likely or at least possible for the injuries to have been caused with the weapon with
which and in the manner in which they are alleged to have been caused. It is elementary
that where the prosecution has a definite or positive case, it must prove the whole of that
case. In the present case, it is doubtful whether the injuries which are attributed to the
appellant were caused by a gun or by a rifle. Indeed, it seems more likely that they were
caused by a rifle than by a gun, and yet the case for the prosecution is that the appellant
was armed with a gun and, in his examination, it was definitely put to him that he was
armed with the gun P.16. It is only by the evidence of a duly qualified expert that it could
have been ascertained whether the injuries attributed to the appellant were caused by a
gun or by a rifle and such evidence alone could settle the controversy as to whether they
could possibly have been caused by a firearm being used at such a close range as is
suggested in the evidence. ...."

In Mani Ram and Ors. v. State of U.P., 1994 Supp(2) SCC 289 (in para 9), this Court
held: 1994 AIR SCW 2298, Para 10

"... It is well settled by long series of decisions of this Court that where the direct
evidence is not supported by the expert evidence then the evidence is wanting in the most
material part of the prosecution case and, therefore, it would be difficult to convict the
accused on the basis of such evidence. If the evidence of the prosecution witnesses is
totally inconsistent with the medical evidence this is a most fundamental defect in the
prosecution case and unless this inconsistency is reasonably explained it is sufficient not
only to discredit the evidence but the entire case. ...".

In another case of Thaman Kumar v. State of Union Territory of Chandigarh, AIR 2003
SC 3975 (in para 16), this Court held: 2003 AIR SCW 2837

"The conflict between oral testimony and medical evidence can be of varied dimensions
and shapes. There may be a case where there is total absence of injuries which are
normally caused by a particular weapon. There is another category where though the
injuries found on the victim are of the type which are possible by the weapon of assault,
but the size and dimension of the injuries do not exactly tally with the size and dimension
of the weapon. The third category can be where the injuries found on the victim are such
which are normally caused by the weapon of assault but they are not found on that
portion of the body where they are deposed to have been caused by the eye-witnesses.
The same kind of inference cannot be drawn in the three categories of apparent conflict in
oral and medical evidence enumerated above. In the first category it may legitimately be
inferred that the oral evidence regarding assault having been made from a particular
weapon is not truthful. However, in the second and third category no such inference can
straightway be drawn. The manner and method of assault, the position of the victim, the
resistance offered by him, the opportunity available to the witnesses to see the occurrence
like their distance, presence of light and many other similar factors will have to be taken
into consideration in judging the reliability of ocular testimony."
12. In the present case, the medical evidence is to the effect that there were no firearm
injuries on the body of the deceased, whereas the eye-witnesses' version is that the
accused-appellants were carrying firearms and the injuries were caused by the firearms.
In such a situation and circumstance, the medical evidence will assume importance while
appreciating the evidence led by the prosecution, by the court and will have priority over
the ocular version and can be used to repel the testimony of the eye-witnesses as it goes
to the root of the matter having an effect to repel conclusively the eye-witnesses' version
to be true. The medical evidence when specifically rules out the injury claimed to have
been inflicted as per the eye-witnesses' version, then the court can draw adverse inference
to the effect that the prosecution version as being put forth before the court, is not
trustworthy. In the present case, the medical evidence completely rules out the
prosecution version of the injuries being caused by firearms, coupled with the fact that no
evidence has been produced by the prosecution of any pellet or bullet being recovered
from the
@page-SC539
place of incident or from the body of the deceased in post-mortem. In the light of the fact
that there was a previous enmity between the parties and the eye-witnesses examined are
related to the deceased and are interested witnesses; and that in absence of the lantern or
the torch, in the light of which the incident was said to have been witnessed, the
prosecution case as placed before the court is full of doubts, and as such the accused-
appellants are entitled for benefit of doubt.
13. For the aforesaid reasons, the appeals are allowed. The judgment of the High Court
and that of the trial court are set aside. The accused-appellants are directed to be set at
liberty if they are not required in any other case.
Appeal allowed
AIR 2008 SUPREME COURT 539 "K. N. Anantharaja Gupta v. D. V. Usha Vijaykumar"
(From : Karnataka)*
Coram : 2 TARUN CHATTERJEE AND P. SATHASIVAM, JJ.
Civil Appeal No. 5547 of 2007 (arising out of SLP (C) No. 285 of 2007), D/- 30 -11
-2007.
K. N. Anantharaja Gupta v. D. V. Usha Vijaykumar.
Karnataka Rent Act (34 of 2001), S.27(2)(r) - HOUSES AND RENTS - EVICTION -
Eviction - Ground - Self occupation after reconstruction - Landlord alleging that she with
her children are residing in house of her father-in-law - Nothing to show that there was
any threat of eviction - Eviction order passed without considering whether landlord could
be said to be not in possession of reasonably suitable accommodation as also condition of
suit house - Liable to be set aside.
H. R. P. No. 366 of 2004, D/- 19-10-2006 (Kant), Reversed.(Para 5)

S. N. Bhat for Appellant; K. Maruthi Rao, Ms. K. Radha and Ms. Anjani Aiyagari for
Respondent.
* H. R. R. P. No. 366 of 2004, D/- 19-10-2006 (Kant)
Judgement
1. TARUN CHATTERJEE, J. :-Leave granted.
2. An eviction petition being HRC No. 233 of 2002 was filed before the Chief Judge,
Small Cause Court, Bangalore for eviction of the appellant from the residential premises
bearing No. 100, Surveyor Street, Bangalore-4 (in short "the suit premises") under
Section 27(2)(r) read with Section 31 of the Karnataka Rent Act (in short "the Act") on
the ground that since the suit premises is old and in a dilapidated condition, the same was
required to be demolished in order to put up a new construction and that the respondent
required the suit premises for use and occupation by herself and her children after
demolition and reconstruction of the same as she and her children were staying in her
father-in-law's house. It was also the case of the respondent that the appellant had been
residing in the suit premises for more than 20 years and therefore, he should find his own
suitable accommodation and accordingly, he was liable to be evicted.
3. A written statement was filed by the appellant in which the allegations made in the
eviction petition were denied and it was stated that the respondent was not entitled to
evict the appellant as she did not require the suit premises for her bona fide use and
occupation. It was further alleged in the written statement that since the respondent was
not the sole owner of the suit premises, the eviction petition filed at her instance only was
not maintainable and therefore, the same was liable to be dismissed. It was also alleged
that the condition of the suit premises was not so dilapidated for which demolition and
reconstruction was necessary. The chief Judge of the Small Cause Court, Bangalore by
his order dated 1st of April, 2004 dismissed the eviction petition of the respondent.
Aggrieved by the aforesaid order of the Chief Judge of the Small Cause Court at
Bangalore, the respondent filed a revision petition before the High Court of Karnataka at
Bangalore being H. R. R. P. No. 366 of 2004. The High Court by it's order dated 19th of
October, 2006, had set aside the order of the Chief Judge of the Small Cause Court
thereby allowing the revision petition and directing eviction of the appellant from the suit
premises but granted six months time to vacate and handover the possession of the same
to the respondent. It is this order of the High Court, which is now under challenge in this
Court by way of a special leave petition in respect of which leave has already been
granted.
@page-SC540
4. Heard the learned counsel for the parties and examined the impugned order of the High
Court as well as the order of the Small Cause Court and the other materials on record. In
our view, the High Court was not justified in reversing the judgment of the Small Cause
Court without being satisfied whether the respondent had fulfilled the conditions required
for eviction of the appellant as laid down under Section 27(2)(r) of the Act. Chapter 6 of
the Act deals with regulation of eviction. Section 27 of the Act deals with protection of
tenants against eviction. Sub-section (1) of Section 27 clearly says that notwitstanding
anything to the contrary contained in any other law or contract, no order or decree for the
recovery of possession of any premises shall be made in favour of the landlord save as
provided in sub-section (2) of Section 27. Sub-section (2) of Section 27 empowers the
Court, on an application made to it in the prescribed manner, to make an order for the
recovery of possession of the premises on one or more of the grounds enumerated
therein. Clause (r) of sub-section (2) of Section 27 being one such ground and involved in
present case runs as under :
"(r) that the premises let are required, whether in the same form or after re-construction
or re-building, by the landlord for occupation for himself or for any member of his family
if he is the owner thereof, or for any person for whose benefit the premises are held and
that the landlord or such person has no other reasonably suitable accommodation....."
We have examined this provision viz., Section 27(2)(r) of the Act in detail. After a careful
examination of this provision, we summarize as follows :
No order or decree for the recovery of possession of any premises shall be made by the
Court against the tenant, save as provided in Section 27(2). A plain reading of Section
27(2)(r) would clearly show that a decree for eviction or an order for recovery of
possession can be passed by a Court if the premises let is required, whether in the same
form or after reconstruction or rebuilding by the landlord for occupation for himself or
for any member of his family if:
(i) he is the owner of the said premises and
(ii) the landlord or such person has no other reasonably suitable accommodation. It is
only when the aforesaid conditions are satisfied the Court can pass an order or decree of
possession of the suit premises against the tenant. We have already noted that the eviction
petition of the respondent was dismissed by the Chief Judge, Small Cause Court,
Bangalore on the ground that the respondent had failed to prove that the suit premises
was required for use and occupation by herself and her children after demolition and
reconstruction and that the respondent had failed to prove that she and her children had
no other reasonably suitable accommodation. This finding as to bona fide requirement of
the respondent was reversed by the High Court in revision. Let us, therefore, examine
whether the High Court was justified in reversing the finding of the Chief Judge, Small
Cause Court, Bangalore and whether the conditions as required under Section 27(2)(r) of
the Act have been satisfied so as to evict the appellant from the suit premises. While
reversing the finding of the Chief Judge, Small Cause Court, Bangalore, so far as the
requirements of Section 27(2)(r) are concerned, the High Court made the following
findings:-
"It is also emerged on the face of it that the petitioner needs the accommodation for her
and her children and she needs to demolish and take up a construction and obtain plan
from the authority. This aspect of the matter has been overlooked by the Trial Court.
Therefore, I am of the considered view that the petitioner has made out a case. The
premises is required for her occupation to take up the construction and to give the same
for personal use by her children as the claim is bona fide."
Having found as quoted hereinabove, the High Court reversed the order of the Chief
Judge, Small Cause Court, Bangalore and held that the respondent was entitled to an
order of eviction under Section 27(2)(r) of the Act. As noted hereinabove, before an order
or decree for eviction is passed, the Court must be satisfied that the premises let is
required by the landlord for occupation for himself or for any member of his family, if he
is the owner of the same and the landlord or such person has no other reasonably suitable
accommodation. In the present case, the respondent is, admittedly, a co-owner of the suit
premises. It is well settled that a co-owner is entitled to evict a tenant on the ground of
bona fide requirement. However, this aspect need not be gone
@page-SC541
into in detail in view of the fact that the High Court had not recorded any finding on the
question whether the respondent was an owner or co-owner in respect of the suit
premises. Now, the question is whether the respondent and her children are in possession
of a reasonably suitable accommodation. According to the respondent, she has been
living with her children in the residence of her father-in-law. The question would,
therefore, be whether this accommodation could be said to be reasonably suitable
accommodation. Admittedly, from the record, it does not appear that there has been any
threat of eviction of the respondent and her children by her father-in-law from the house
in which they are presently residing. This aspect of the matter, we are afraid, was not
taken into consideration by the High Court. Before passing any order of eviction, it was
the duty of the High Court to come to a finding that the respondent was not in possession
of a reasonably suitable accommodation, which is the mandatory requirement under
Section 27(2)(r) of the Act.
5. That apart, there is another aspect of this matter. As noted hereinabove, the eviction of
the tenant was sought under Section 27(2)(r) of the Act by alleging that the suit premises
was required by the respondent and her children for their own use and occupation after
demolition and reconstruction of the building already existing. In order to satisfy this
condition, as enumerated in Section 27(2)(r) of the Act, it is essential that the Court
should also find that the premises let needs to be demolished and that the same would be
reconstructed after demolition. It is only after this that the question of user of the same
after reconstruction would be taken into consideration. From the order of the High court
passed in revision, it would be evident that the only ground on which the order of the
Chief Judge, Small Cause Court, Bangalore was reversed was that the respondent needed
the suit premises to demolish the same and to take up new construction and obtain plans
from the authority. In our view, before granting a decree for eviction on the ground of
demolition and reconstruction and then for use of the same for occupation, the Court
must be satisfied that :-
(i) the suit premises is so dilapidated that it needs demolition;
(ii) the landlord has the capacity to reconstruct the suit premises after demolition;
(iii) the sanctioned plan has to be taken from the concerned authority.
The High court proceeded only on the ground that the respondent required the suit
premises for occupation by herself and her children and needed to demolish and take up a
new construction on the same. In our view, this would not satisfy the requirements
envisaged in Section 27(2)(r) of the Act. The Court, as noted herein earlier, must be
satisfied that all the conditions, as enumerated above, have been satisfied by the landlord
by production of cogent evidence in respect of the same. Only an expression of desire
would not entitle the landlord to get a decree for eviction under Section 27(2)(r) of the
Act.
6. Another aspect involved in this case needs to be stated because the eviction petition
was filed not only under Section 27(2)(r) of the Act but also under Section 31 of the Act,
recourse to which is available to a widow only once. We, however, need not go into this
question at all. In any view of the matter, the High Court, while reversing the order of the
chief Judge, Small Cause Court, Bangalore had also not adhered to this aspect of the
matter and therefore, it is also not necessary for us to go into this question in this appeal.
7. For the reasons aforesaid, we are unable to sustain the order of the High Court and
accordingly, the impugned judgment of the High Court is set aside and the matter is
remitted back to the High Court for a decision in the light of the findings made
hereinabove. While deciding the revision petition, it will be open to the High Court either
to permit the parties to lead evidence in the High Court or to frame the questions and
direct the Chief Judge, Small Cause Court, Bangalore to take evidence and to make a
finding on the same, which may then be transmitted to the High Court and thereafter, the
High Court will decide the revision petition in the light of the finding, the evidence
adduced and the evidence already on record within a period of six months from the date
of supply of a copy of this order to it without granting any unnecessary adjournment to
either of the parties.
8. For the reasons aforesaid, the appeal is thus allowed to the extent indicated above.
There will be no order as to costs.
Appeal allowed.
@page-SC542
AIR 2008 SUPREME COURT 542 "Janak Raj v. Pardeep Kumar"
(From : Jammu and Kashmir)
Coram : 2 A. K. MATHUR AND MARKANDEY KATJU, JJ.
Civil Appeal No. 7080 of 2001, D/- 27 -11 -2007.
Janak Raj v. Pardeep Kumar.
J. and K. Houses and Shops Rent Control Act (34 of 1966), S.11(1)(i), S.12(3), Proviso -
HOUSES AND RENTS - EVICTION - APPLICABILITY OF AN ACT - TENANCY -
Eviction - Default in payment of rent - Protection to defaulting tenant not defaulting
thrice in 18 months - Applicability - Tenant defaulting thrice in 18 months - Depositing
certain sum thereafter - Sum deposited has to be adjusted chronologically - Cannot be
adjusted first to satisfy 2nd and 3rd default - Sum deposited by tenant falling short of rent
due towards first default - Tenant thus, has committed 3 defaults - Not entitled to
protection of S. 12 (3) Proviso.
Civil S. A. No. 4 of 1995, D/- 9-11-2000 (J. and K.), Reversed. (Paras 7, 8, 9, 10)

P. S. Patwalia, Sr. Advocate, Purnima Bhat, for Appellant; Subramonium Prasad, for
Respondent.
Judgement
JUDGMENT :- We have heard learned counsel for the parties.
2. This appeal by special leave is directed against the impugned judgment and order of
the learned single Judge of the High Court of Jammu and Kashmir at Jammu dated 9th
November, 2000 in Civil Second Appeal No. 4 of 1995 whereby the learned single Judge
upheld the finding of the First Appellate Court and dismissed the suit of the plaintiff
(appellant herein).
3. The brief facts which are necessary for the disposal of the appeal are that a suit was
filed by the landlord (appellant herein) for eviction of the tenant respondent. The landlord
claimed the rent for January, 1984 to January, 1985 by sending a notice dated 27-2-1985
to the tenant. Thereafter the tenant committed a second default of payment of rent for
February, 1985 and March, 1985. The third default was committed in April, 1985 and
May, 1985. The total amount deposited by the tenant on 25-5-1995 was Rs. 6,000/-.
4. Section 11(1)(i) of the Jammu and Kashmir Houses and Shops Rent Control Act, 1966
provides that if two months rent is not paid by the tenant within the period specified
therein the tenant will be liable for eviction, provided that the landlord has served a notice
on the tenant calling upon him to pay the arrears of rent, and the arrears are not paid
within 30 days of service of the notice.
5. Section 11(1)(i) is however subject to Section 12 of the Act which reads as under:-
"12. When a tenant can get the benefit of protection against eviction. (1) If in a suit for
recovery of possession of any house or shop from the tenant the landlord would not get a
decree for possession but for clause (i) of the proviso to sub-section (1) of Section 11, the
court shall determine the amount of rent legally payable by the tenant and which is in
arrears taking into consideration any order made under sub-section (4) and effect thereof
up to the date of the order mentioned hereafter, as also the amount of interest on such
arrears of rent calculated at the rate of nine and three eighths per centum per annum from
the day when the rent became arrears up to such date, together with the amount of such
cost of the suit as if fairly allowable to the plaintiff landlord, and shall make an order on
the tenant for paying the aggregate of the amounts (specifying in the order such aggregate
sum) on or before a date fixed in the order.
(2) Such date fixed for payment shall be the fifteenth day from the date of the order,
excluding the day of the order.
(3) If, within the time fixed in the order under Sub-section (1), the tenant deposits in the
Court the sum specified in the said order, the suit, so far as it is a suit for recovery of
possession of the house or shop, shall be dismissed by the Court. In default of such
payment the Court shall proceed with the hearing of the suit:
Provided that the tenant shall not be entitled to the benefit of protection against eviction
under this section, if, notwithstanding the receipt of notice under proviso to clause (i) of
the proviso to sub-section (1) of Section 11, he makes a default in the payment of the rent
referred to in clause (i) of the proviso to sub-section (1) of Section 11 on three occasions
within a period of eighteen months.
6. In our opinion, in view of the proviso to Section 12(3) referred to above, the
respondent tenant cannot get the benefit of
@page-SC543
Section 12.
7. In the present case, it is the admitted position that the rent was Rs. 500/- per month.
The respondent tenant was in default in not paying the rent from January, 1984 to
January, 1985 (inclusive) but he is alleged to have deposited a sum of Rs. 6,000/- on 25-
5-1985 whereas the rent for this period was Rs. 6,500/- i.e. the rent paid was short by Rs.
500/-. As such, in our opinion he is a defaulter for this period. Part payment of rent in our
opinion makes the tenant a defaulter, for he has to make full payment. The second default
committed by the respondent tenant was in February, 1985 and March, 1985, and the
third default committed by him was in April, 1985 and May, 1985. Therefore, he is a
defaulter three times during the period of eighteen months. As such, he is not entitled to
the protection of Section 12.
8. Unfortunately, the courts below have taken the amount of Rs. 6,000/- deposited by the
respondent tenant as rent towards February, 1985 and March, 1985; and April, 1985 and
May, 1985 to take him out of the defaulters clause but this view taken by the Courts
below cannot be countenanced. As a matter of fact he had deposited a sum of Rs. 6,000/-
for the aforesaid three defaults. That sum cannot be adjusted against the second and third
defaults as has been done by the Courts below.
9. The tenant is firstly supposed to clear the first default i.e. from 1984 to 1985, then the
second default of February and March, 1985 and then the third default of April and May,
1985. One has to proceed chronologically in the matter. The sum of Rs. 6,000/- has to be
first treated as part payment against the rent from January, 1984 to January, 1985.
However, for the rent from January, 1984 to January, 1985 the total amount deposited by
the tenant was Rs. 6,000/- which is short of the total rent due for this period by Rs. 500/-.
Therefore, it will also be treated to be a default. Consequently, this default, coupled with
the defaults of February and March, 1985 and April, 1985 and May, 1985 will constitute
three defaults. As such the view taken by both the Courts below cannot be countenanced.
10. Consequently, the judgment and order of the High Court as also the order of the First
Appellate Court are set aside and the decree granted by the Trial Court for eviction is
maintained.
11. The appeal is accordingly, allowed.
12. However, the respondent tenant is directed to handover the vacant possession of the
premises in question to the appellant-landlord by 31st August, 2008. The respondent shall
file the usual undertaking to this effect within four weeks from today in this Court.
13. It is submitted by learned counsel for the respondent that the entire arrears of rent
have already been paid. He shall continue to pay the rent till 31st August, 2008.
Appeal allowed.
AIR 2008 SUPREME COURT 543 "Binapani Paul v. Pratima Ghosh"
(From : 2004 (1) Cal HN 185)
Coram : 2 S. B. SINHA AND MARKANDEY KATJU, JJ.
Civil Appeal No. 8098 of 2004, D/- 27 -4 -2007.
Binapani Paul v. Pratima Ghosh and Ors.
(A) Transfer of Property Act (4 of 1882), S.41 - IMMOVABLE PROPERTY - Purchase
of property by husband in name of his wife - Practice prevalent in respect thereto at
relevant time - Not of much importance - A Court of law is required to determine such a
question - Without anything more, it cannot determine same on basis of such alleged
practice only. (Para 12)
(B) Transfer of Property Act (4 of 1882), S.41 - IMMOVABLE PROPERTY - BENAMI
TRANSACTION - POWER OF ATTORNEY - RIGHT TO PROPERTY - Benami
transaction - When can be inferred - Property purchased in the name of wife through
power of attorney executed by wife - Husband being attesting witness to power of
attorney showing that he knew transaction executed by wife - Insurance policy also
purchased in name of wife - Property mutated in name of wife immediately - Purchase of
property in year 1935 when Hindu Women's Right to Property Act did not come into
force - Couple having a son and seven minor daughters - Intention of husband in such a
case to purchase property for benefit and security of wife and daughters, can be inferred
in such case - Held that, it was not benami transaction.
2004 (1) Cal HN 185, Reversed.
Where in year 1935 the property was purchased by husband in the name of wife through
a power of attorney which was
@page-SC544
executed by wife in favour of husband's brother and husband himself attested the power
of attorney and name of wife was mutated in land records, it was held that it was not a
benami transaction but the property was purchased by husband for the benefit of his wife
as he had seven minor daughters and a son and he wanted to secure the future of his wife
and daughters. Moreover, in 1935, Hindu Women's Right to Property Act, 1937 did not
come into force. He, therefore, might have been of the opinion that in case of his early
death, something should be kept apart for his wife and daughters. When a person
develops such an intention, it would be opposed to the essential characteristics of a
benami transaction. He furthermore was not a debtor. He was not required to avoid any
liability. He had no apparent motive for entering into a benami transaction. Further, in the
power of attorney she was not described as his wife but dauther of 'B'. Therefore, he
being in the position of husband and if he intended to have a benami transaction,
ordinarily, he would not get his wife described as daughter of somebody instead of his
own wife. Such unusual step on the part of husband would lead to one conclusion that he
intended to purchase the property for the benefit of his wife. Moreover, in the power of
attorney it was categorically stated that it was wife who had decided to purchase the said
property and it was she who was appointing her husband's brother as her attorney.
Further, the fact that an insurance was also made in her name is also a pointer to show
that husband intended to provide sufficient money at the hands of his wife. It was more so
when after the death of husband and wife, their son maltreated the daughters who were
unmarried at that time and those daughters mutated the property in their name and the
fact that the son allowed the order of mutation to attain finality, thus, would also be a
pointer to suggest that despite such bitter relationship between the parties he accepted the
same; moreso, when mutation of one's name in the Municipal Corporation confers upon
him a variety of rights and obligations.
2004 (1) Cal HN 185, Reversed.
(Paras 13, 15, 19, 35, 51)
(C) Limitation Act (36 of 1963), Art.64, Art.65 - LIMITATION - PLEA - Plea of ouster -
If pleaded, title has to be acknowledged - Once such plea is taken, irrespective of fact that
as to whether any other plea is raised or not, conduct of parties would be material - If,
therefore, plea of ouster is not established, a fortiori the title of other co-sharers must be
held to have been accepted. (Para 40)
(D) HINDU LAW - GIFT - Hindu Law - Dayabhaga School - Does not prohibit gift of
immovable property in favour of his wife by her husband - It merely says that
Dayabhaga did not recognize it to be her stridhan - It was only for the purpose of
inheritance and succession - Same has nothing to do with Benami transaction of Property
and to determine nature of transaction. (Para 49)
Cases Referred : Chronological Paras
2007 AIR SCW 2897 : AIR 2007 SC 1753 : 2007 (4) AIR Kar R 227 (Ref) 43
2006 AIR SCW 4368 : 2006 (6) AIR Kar R 13 (Rel on) 41
2006 AIR SCW 4905 : AIR 2006 SC 3359 (Rel on) 38
2006 AIR SCW 5794 : AIR 2007 SC 204 (Ref) 43
2004 AIR SCW 4948 : AIR 2004 SC 4187 (Rel on) 50
AIR 1999 Cal 86 38
(1996)4 SCC 490 50
1994 Supp (1) SCC 734 50
AIR 1980 SC 727 (Ref) 27, 50
AIR 1979 SC 553 (Rel on) 44
AIR 1977 SC 796 : 1977 Cri LJ 566 50
AIR 1974 SC 171 (Rel on) 29, 50
1974 Cal LJ 370 16
AIR 1965 SC 271 (Ref) 22
(1956)60 Cal WN 88631
AIR 1940 Cal 356 32, 48
AIR 1939 Pat 462 (Disting) 34
AIR 1938 Mad 8 26
AIR 1934 Mad 671 30
AIR 1932 PC 13 25
AIR 1931 Bom 97 38
AIR 1927 PC 230 38
AIR 1927 Mad 194 31
AIR 1925 PC 181 12
(1877) ILR 1 Mad 281 48
(1854)6 Moo Ind App 53 12
S. B. Sanyal, Sr. Advocate, Rauf Rahim, Mohd. Iqbal, for Appellant; Bhaskar P. Gupta,
Sr. Advocate, Tara Chandra Sharma, Ms. Neelam Sharma, Devadatt Kamat, Abhijat P.
Medh, for Respondents.
Judgement
S. B. SINHA, J.:-One Dr. Ashutosh Ghosh (Dr. Ghosh), a Physician practising at
Rangoon was a prosperous person. He
@page-SC545
purchased two immovable properties in Calcutta in the year 1927 situate at 79/3-A and
79/3-B, Lower Circular Road, Calcutta, in his own name. Suprovabala was his wife. They
at the relevant time had seven daughters, including the appellant herein and a son named,
Amal. Respondent Nos. 1 and 2 are his wife and daughter. Suprovabala intended to
purchase the premises situate at No. 24, Convent Road, Calcutta belonging to the estate
of Late Edwin St. Clair Vallente. She executed a power of attorney in favour of one Atul
Chandra Ghosh, brother of Dr. Ghosh, the relevant portion whereof reads as under :
?...Whereas I have decided to purchase premises No.24, Convent Road, Calcutta,
belonging to the Estate of Late Mr. Edwin St. Chair Vallente at the price of Rs.26000/-
(Rupees Twenty Six thousand only) but the agreement for sale has not yet been entered
into with the Administration General of Bengal as Administrator to the Estate of Edwin
St. Clair Vallente now therefore know. Yet that I hereby appoint Atul Chandra Ghosh of
79/3-A, Lower Circular Road, Calcutta my attorney to do and execute for me and in my
name and all acts, matters and things that may be necessary in order to complete the said
purchase and particularly the following : ...
In witness whereof I set and subscribe my hand and seal at Rangoon this 23rd day of
September 1935 in the presence of
Date: 23.09.1935
No.1986
Date of Registry : 17.10.1935
Sd/- Smt. Supravabla Ghosh
Sd/- K.N. Ganguli
Advocate High Court and Councilor
Corporation of Rangoon
Sd/- S.N. Ganduly, Advocate, High Court
Sd/- Ashutosh Ghosh M.B. (Cal)
Medical Practioner...?
2. The said power of attorney, however, was preceded and followed by two telegrams of
Dr. Ghosh addressed to his brother in relation to execution thereof as also purchase of the
said property. The said power of attorney was executed before a Magistrate at Rangoon.
Dr. Ghosh was an attesting witness therein. Interestingly, Suprovabala described herself
as daughter of Babu Rangalal Ghosh and not the wife of Dr. Ghosh therein. A registered
indenture was executed on 16.11.1935 by the Administrator General of Bengal to the
estate of Edurn St. Clair Vallentine in favour of Suprovabala for a sum of Rs. 26,000/-.
Indisputably, during the life time of Dr. Ghosh, the name of Suprovabala was mutated.
She had all along been in possession of the said property. Dr. Ghosh died in Rangoon in
the year 1940. Suprovabala continued to reside in the suit premises. She died on
26.05.1942 leaving, as indicated hereinbefore, seven daughters and son Amal. Amal was
married to Respondent No. 1 herein in 1946.
3. In the year 1958, the daughters of Suprovabala got their names mutated in place of
their mother. Amal objected thereto, but his objection was rejected. Marriage of four
sisters of Amal took place in the suit premises during the period 1944 to 1970. Although
initially all the sisters and the brother were living together in the said house, inter alia,
after their marriage the daughters of Suprovabala started living at their respective
husbands? places. However, three sisters allegedly continued to live in the said house till
May, 1958 but they had to leave it because of ill-treatment of Amal and his wife. It
appears that in the year 1964, two unmarried daughters of Suprovabala who had been
living there were also compelled to leave the house. They filed a suit for maintenance
with liberty to claim their right to take appropriate legal action to recover their share of
the said premises at an appropriate time, which was allowed by the High Court. Three out
of the seven daughters of Dr. Ghosh filed a suit for partition against Amal on 19.09.1973
claiming 3/7th share of the property of their mother, a final decree for partition as also a
decree for accounts.
4. Amal in his written statement filed in the suit inter alia contended that Suprovabala was
benamidar of Dr. Ghosh. Suprovabala, therefore, had only a limited interest under the
Hindu Women?s Right to Property Act, 1937 and on her death Amal became the absolute
owner. Amal died during pendency of the suit whereupon Respondent Nos. 1 and 2 were
substituted in his place.
5. Before the learned Trial Judge, plaintiff - Binapani examined herself as PW-3. A
common relation of the parties being Chandi Charan Ghosh examined himself as PW-4.
Respondent No. 1 did not examine herself. Putul Ghosh, daughter of Amal who was born
only in 1954 examined herself as DW-1.
@page-SC546
6. The learned Trial Judge decreed the suit holding that Dr. Ghosh intended to purchase
the said property for the benefit of his wife. The Trial Court in its judgment opined that if
Dr. Ghosh wanted to purchase the property for himself, there was no necessity for
execution of power of attorney by Suprovabala in favour of Atul Chandra Ghosh. It was
noticed that the power of attorney had been attested by Dr. Ghosh which is a pointer to
show that the property was purchased by him for the benefit of his wife. Circumstances
surrounding the same, it was held, also led to the said conclusion. It was, therefore, not
held to be a case of benami transaction. A first appeal was preferred thereagainst before
the High Court by Respondent Nos. 1 and 2. A Division Bench of the High Court
although completed hearing of the appeal on 25.01.2002, delivered judgment after 19
months, i.e., on 29.07.2003.
7. The High Court opined that - :
(i) it was for the plaintiff to prove that Dr. Ghosh purchased the property for the benefit of
his wife;
(ii) purchase by Suprovabala through an attorney does not negative the nature of
transaction being a benami one;
(iii) mutation of names of all the heirs of Suprovabala was of no consequence.
(iv) Dr. Ghosh could not have gifted the property in favour of his wife being
impermissible under the Dayabhaga School of Hindu Law.
8. Mr. S. B. Sanyal, learned senior counsel appearing on behalf of the appellant,
submitted that the High Court committed a manifest error in passing the impugned
judgment insofar:
(i) the onus of proof had wrongly been placed upon the plaintiff;
(ii) the defendant had not been able to show any motive for the benami purchase.
(iii) the presumption that an apparent state of affairs is the real state of affairs has not
been rebutted by aduction of any cogent evidence.
(iv) contribution of purchase money is only one of the factors for proving benami
transaction but intention also plays a significant role in relation thereto which was
required to be determined having regard to the surrounding circumstances, the
relationship of the parties, the motive governing their action and the subsequent conduct
of the parties.
(v) Putul Ghosh (DW-1) cannot be said to have any knowledge about the transaction and
there was no reason as to why her mother Pratima Ghosh did not examine herself as a
witness.
9. Mr. Devadatt Kamat, learned counsel appearing on behalf of Respondent Nos. 4 to 7
supplemented the argument of Mr. Sanyal stating that the High Court cursorily dealt with
the question of intention in relation to the transaction in question. Our attention has also
been drawn to Section 5 of the Power of Attorney Act, 1882.
10. Mr. Bhaskar P. Gupta, learned senior counsel appearing on behalf of Respondent Nos.
1 and 2, on the other hand, would submit that:
(i) the suit property having been acquired in the year 1935, as purchases of property in
the benami name of wives being prevalent at the relevant time, the case was required to
be considered from that angle.
(ii) a transaction in benami may be entered into for no apparent reason.
(iii) doctrine of advancement has no application in India.
(iv) Benami Transactions (Prohibition) Act, 1988 has no retrospective effect. The source
of money being an important factor for determining benami nature of transaction, the
onus lay on the plaintiffs.
(v) the parties being governed by the Dayabhaga School of Hindu Law, Dr. Ghosh could
not have made a gift of immovable property in favour of his wife.
11. Before embarking upon the rival contentions of the parties, we may also notice that
Dr. Ghosh had a life insurance. Suprovabala was his nominee and after his death, the
entire amount of insurance was received by her.
12. A question as to whether a transaction evidences a benami nature thereof is always
difficult to answer. It is a case where despite some evidence brought on records by the
plaintiffs that Suprovabala paid the consideration amount or at least a part of it, we may
proceed to determine the issues between the parties on the premise that the amount of
consideration was provided by Dr. Ghosh. A person may for various reasons intend to
purchase a property in the name of his wife. It may be for one reason or the other. There
may or may not be a practice
@page-SC547
in respect thereto. A purported prevalent practice in this behalf, as was observed by the
Judicial Committee, in Sura Lakshmiah Chetty and Others v. Kothandarama Pillai [AIR
1925 PC 121] and Gopeekrist Gosain v. Gungapersaud Gosain [(1854) 6 Moore's Indian
Appeals 53], is in our opinion not of much importance. A court of law is required to
determine such a question. Without anything more, it cannot determine the same on the
basis of such an alleged practice only.
13. Dr. Ghosh was a prosperous person. He must be a medical practitioner of repute. He
had purchased two very valuable properties in Calcutta in quick succession being situate
at 79/3-A and 79/3-B, Lower Circular Road, Calcutta, which is a very prime area in the
town of Calcutta. The property in question was purchased in 1935. Admittedly,
renovations were made in the year 1938. He died in the year 1940 at Rangoon. At that
point of time, none of his children was married. He had seven daughters. In 1935, Hindu
Women's Right to Property Act, 1937 did not come into force. He, therefore, might have
been of the opinion that in case of his early death, which appears to have been his
premonition, something should be kept apart for his wife and daughters. When a person
develops such an intention, it would be opposed to the essential characteristics of a
benami transaction. He furthermore was not a debtor. He was not required to avoid any
liability. He had no apparent motive for entering into a benami transaction. The plaintiffs'
case that he had done so for the benefit of his wife, therefore, must be considered from
that angle.
14. Amal appears to be the eldest amongst the children. When a son is the eldest amongst
the children, expectation of a father will always be that on his death, he would look after
his mother and sisters. Son would perform his duties not only by providing maintenance
to the daughters, to which they were otherwise entitled to, but also they were to be
married. Dr Ghosh's eagerness to purchase the property is evidenced by two telegrams
dated 20th and 24th September, 1935.
15. Mr. Gupta's submission that the said telegrams are relevant to show Dr. Ghosh's
personal involvement in the transaction may not be of much significance. They were at
Rangoon. Negotiations for purchase were to be held with the Administrator General of
Bengal. Earnest money was to be deposited. The deed was to be drawn up. In those days,
a Hindu wife was supposed to maintain some 'purdah'. We do not know whether she
knew English or not. She, therefore, was not expected to draft a telegram and go to post
office for the purpose of transmission thereof. But, the power of attorney executed by her
plays an important role. The power of attorney must have also been drafted at the behest
of Dr. Ghosh. Ordinarily, Suprovabala would be described as the wife of Dr. Ghosh. She
was not. She was described as the daughter of Babu Rangalal Ghosh. Dr. Ghosh himself
was an attesting witness. He being in the position of husband and if we accept the case of
the defendants-respondents that he intended to have a benami transaction, ordinarily, he
would not get his wife described as daughter of somebody instead of his own wife. Such
unusual step on the part of Dr. Ghosh leads to one conclusion that he intended to
purchase the property for the benefit of his wife. The recitals made in the power of
attorney are also of much significance. It was categorically stated that it was Suprovabala
who had decided to purchase the said property and it was she who was appointing her
husband's brother as her attorney.
16. In Tara Sundari Sen v. Pasupati Kumar Banerjee and Ors. [1974 CLJ 370], it was
observed - :
"...The only purpose of Nagendra Nath Ganguly having been a signatory to the said
document must have been to represent to the world at large that the property was being
acquired by Sm. Shantabala as her absolute property and that her husband had no right,
title or interest in the same..."
17. It was further observed therein - :
"The significance and value of these indisputable facts have to be carefully assessed. It is
common case that the ultimate source of the money was the income and savings of
Nagendra Nath Ganguly. The plaintiff contends that Nagendra Nath made a gift of the
money to his wife Shantabala to enable her to acquire the properties. If that be so, the
properties were Shantabala's Ajoutuka Stridhana. That Nagendra made gift out of his
funds does not in any way prejudice the plaintiff's case. Once the gift was made, if it was
made at all, the money belonged absolutely to Shantabala and the properties she
purchased were hers and hers alone. That Nagendra engaged a contractor or a supervisor
for construction of a structure
@page-SC548
on the land purchased by Shantabala or that he made payments to the contractor or the
supervisor will not by itself be any evidence of his ownership. The husband of a Hindu
lady living in a common matrimonial home usually manages and maintains her
properties. The Court can and ought to take judicial notice of the fact that ordinarily in a
Hindu household the husband deals with strangers and trademen. Therefore, the fact that
payments were made by Nagendra Nath Ganguly is not inconsistent with the case that the
premises belonged to Shantabala absolutely."
18. In a given situation, execution of a power of attorney may not be of importance but
then the backdrop of events and the manner in which the power of attorney was drafted as
well as the very fact that Dr. Ghosh himself became an attesting witness thereto, the same
plays very significant role. If in the light of the so-called practice as then existed, i.e., to
purchase property in the name of his wife, Dr. Ghosh intended to enter into a benami
transaction, his intention, therefor, would have been clear and unambiguous or in any
event, the same would have been explicit from the surrounding circumstances. They were
not. Moreover, immediately after the purchase, the name of Suprovabala was mutated.
She started paying tax. There is no evidence to show that Dr. Ghosh took an active role
except providing for the amount in regard to the construction of the house. Evidence on
records clearly show that Suprovabala had also been looking after the constructions of the
house along with Chandi Charan Ghosh (PW-4).
19. The fact, which we have noticed hereinbefore, viz., that an insurance was also made
in her name is also a pointer to show that Dr. Ghosh intended to provide sufficient money
at the hands of his wife. [See Ext. A (13)] Ordinarily, a son would be made a nominee.
We must place on record the social condition as thence prevailing, viz., a son under the
law was bound to maintain his family and, therefore, the entire property at the disposal of
the father would be given to the son.
20. We do not have any direct evidence of conclusive nature in this regard before us. We
must, therefore, deal with the matter on reasonable probabilities and legal inferences.
21. Dr. Ghosh indisputably was a person having a superior knowledge and understanding.
He was holding a responsible position in the society. He was in a noble profession. When
he made attestation of the deed of the power of attorney keeping in view the fact that he
was the husband there cannot be any doubt that he fully understood in regard to the
nature of the transaction as also the contents and merits thereof.
22. We may at this juncture also notice a Constitution Bench decision of this Court in
Kanakarathanammal v. V.S. Loganatha Mudaliar [AIR 1965 SC 271 : (1964) 6 SCR 1]
wherein this Court had an occasion to deal with the question of providing money to the
wife, the purpose for purchase of the property vis-a-vis a transaction which was benami
in nature. For the purpose of inferring acknowledgement and/ or admission by husband
that the property was purchased by his wife, this Court, upon taking into consideration
the provisions of Mysore Hindu Law Women's Rights Act (10 of 1933), opined - :
"12. We have carefully considered the arguments thus presented to us by the respective
parties and we are satisfied that it would be straining the language of Section 10(2)(b) to
hold that the property purchased in the name of the wife with the money gifted to her by
her husband should be taken to amount to a property gifted under Section 10(2)(b). The
argument about the substance of the transaction is of no assistance in the present case,
because the requirement of Section 10(2)(b) is that the property which is the subject-
matter of devolution must itself be a gift from the husband to the wife. Can we say that
the property purchased under the sale deed was such a gift from the husband to his wife?
The answer to this question must clearly be in the negative. With what funds the property
is purchased by the female is irrelevant for the purpose of Section 10(2)(d); so too the
source, the title to the fund with which the said property was purchased. All that is
relevant to enquire is: has the property been purchased by the female, or has it been gifted
to her by her husband? Now, it seems clear that in deciding under which class of
properties specified by clauses (b) and(d) of Section 10(2) the present property falls, it
would not be possible to entertain the argument that we must treat the gift of the money
and the purchase of the property as one transaction and hold on that basis that the
property itself has been gifted by the husband to his
@page-SC549
wife. The obvious question to ask in this connection is, has the property been gifted by
the husband to his wife, and quite clearly a gift of immovable property worth more than
Rs. 100 can be made only by registered deed. The enquiry as to whether the property was
purchased with the money given by the husband to the wife would in that sense be
foreign to Section 10 (2)(d) gift of money which would fall under Section 10(2)(b) if
converted into another kind of property would not help to take the property under the
same clause, because the converted property assumes a different character and falls under
Section 10(2)(d). Take a case where the husband gifts a house to his wife, and later, the
wife sells the house and purchases land with the proceeds realised from the said sale. It is,
we think, difficult to accede to the argument that the land purchased with the sale-
proceeds of the house should, like the house itself, be treated as a gift from the husband
to the wife; but that is exactly what the appellants argument; will inevitably mean. The
gift that is contemplated by Section 10(2)(b) must be a gift of the very property in specie
made by the husband or other relations therein mentioned. Therefore, we are satisfied that
the trial court was right in coming to the conclusion that even if the property belonged to
the appellants mother, her failure to implead her brothers who would inherit the property
along with her makes the suit incompetent. It is true that this question had not been
considered by the High Court, but since it is a pure point of law depending upon the
construction of Section 10 of the Act, we do not think it necessary to remand the case for
that purpose to the High Court..."
23. Mr. Gupta made an endeavour to distinguish the said decision on fact of the matter
submitting that therein the father wrote a large number of letters which included a
discussion of the wife's will where he had acknowledged the wife's title to the property,
but we have to consider the crux of the matter to understand the underlying principle laid
down therein.
24. Acceptance of acknowledgement of title comes in various forms. It may be before the
transaction is entered into and may be subsequent thereto. The court has to gather the
intention of the concerned parties on the basis of the circumstances surrounding the
transaction and not from the conduct of the parties only at a subsequent stage. It may be
true that ipso jure acknowledgement of title would mean the same should be only after
the title is acquired, but, whether addressing ourselves to a question of this nature, viz., as
to whether Dr. Ghosh intended to enter into a benami transaction in the name of his wife,
either surrounding circumstances leading to the inference that he had no such intention
must be gathered from the totality of the circumstances both preceding and subsequent to
the transaction in question or if the intention of the person providing for the fund for
purchasing the property has a major role to play, how it was given also assumes some
significance. Apart from the fact that Dr. Ghosh himself was keen to see that the property
is purchased for the benefit of his wife, we must notice that it was also mutated in her
name. When a mutation takes place with the knowledge of the husband, although not
conclusive, would provide for a link in the chain.
25. To decipher the intention of the parties, this Court must go back to the societal
situation as was prevailing in 1935. Dr. Ghosh as a man of ordinary prudence wanted to
make provision to protect and insure the welfare of his seven daughters and wife. In a
case of this nature, the answer to such a question has to be in the affirmative. Question of
intention is always relatable and peculiar to the facts of each case. [See Nawab Mirza
Mohammad Sadiq Ali Khan and Others v. Nawab Fakr Jahan Begam and Another AIR
1932 PC 13]
26. In Chittaluri Sitamma and another v. Saphar Sitapatirao and others [AIR 1938 Madras
8], it was held - :
"...The mere suspicion that the purchases might not have wholly been made with the
lady's money will certainly not suffice to establish that the purchases were benami, nor
even the suspicion that moneys belonging to Jagannadha Rao whether in a smaller
measure or a larger measure, must have also contributed to these purchases. Even in cases
where there is positive evidence that money had been contributed by the husband and not
by the wife, that circumstance is not conclusive in favour of the benami character of the
transaction though it is an important character..."
27
. The learned counsel for both the parties have relied on a decision of this Court in
Thakur Bhim Singh (Dead) By LRs and Another v. AIR 1980 SC 727

@page-SC550
Thakur Kan Singh [(1980) 3 SCC 72] wherein it has been held that the true character of a
transaction is governed by the intention of the person who contributed the purchase
money and the question as to what his intention was, has to be decided by - :
(a) Surrounding circumstances
(b) Relationship of the parties
(c) Motives governing their action in bringing about the transaction and
(d) Their subsequent conduct.
28. All the four factors stated may have to be considered cumulatively. The relationship
between the parties was husband and wife. Primary motive of the transaction was security
for the wife and seven minor daughters as they were not protected by the law as then
prevailing. The legal position obtaining at the relevant time may be considered to be a
relevant factor for proving peculiar circumstances existing and the conduct of Dr. Ghosh
which is demonstrated by his having signed the registered power of attorney.
29

. This aspect of the matter has been considered by this Court in Jaydayal Poddar
(Deceased) Through L.Rs. and Another v. Mst. Bibi Hazira and Others [(1974) 1 SCC 3],
wherein this Court held - : AIR 1974 SC 171, (Para 6)

"...The essence of a benami is the intention of the party or parties concerned; and not
unoften, such intention is shrouded in a thick veil which cannot be easily pierced through.
But such difficulties do not relieve the person asserting the transaction to be benami of
any part of the serious onus that rests on him; nor justify the acceptance of mere
conjectures or surmises, as a substitute for proof. The reason is that a deed is a solemn
document prepared and executed after considerable deliberation, and the person expressly
shown as the purchaser or transferee in the deed, starts with the initial presumption in his
favour that the apparent state of affairs is the real state of affairs. Though the question,
whether a particular sale is benami or not, is largely one of fact, and for determining this
question, no absolute formulae or acid test, uniformly applicable in all situations, can be
laid down; yet in weighing the probabilities and for gathering the relevant indicia, the
Courts are usually guided by these circumstances: (1) the source from which the purchase
money came; (2) the nature and possession of the property, after the purchase; (3) motive,
if any, for giving the transaction a benami colour; (4) the position of the parties and the
relationship, if any, between the claimant and the alleged benamidar; (5) the custody of
the title-deeds after the sale and (6) the conduct of the parties concerned in dealing with
the property after the sale."
30. Source of money had never been the sole consideration. It is merely one of the
relevant considerations but not determinative in character. [See Thulasi Ammal v. Official
Receiver, Coimbator AIR 1934 Madras 671].
31. In Protimarani Debi and Anr. v. Patitpaban Mukherjee and Ors. [60 CWN 886], the
Calcutta High Court observed - :
"The correct proposition was stated in Official Assignee of Madras vs. Natesha Gramani
(AIR 1927 Madras 194). There is no presumption that when a property stands in the
name of a female the Court will immediately jump to the conclusion without any proof
that it really belongs to the husband of the female. Before such a presumption is raised or
attracted it is necessary for the person who wants to make out that the property is not the
property of the female, in whose name the document stands, to establish the fact that the
consideration money for the purpose had come from the husband."
32. It will be useful at this juncture to notice a judgment of the Calcutta High Court in
K.K. Das, Receiver and others v. Sm. Amina Khatun Bibi and another [AIR 1940 Cal
356], wherein it was held that where a husband provides for the money for construction
of a building on a land which is in the name of his wife, he did not intend to reserve any
right in the structures raised therein.
33. In 1935, the appellant herein was a minor. Whether she was aged 9 years or 14 years,
thus, is immaterial. She, however, had the occasion to know something about the property
from her mother or father. Dr. Ghosh expired only in 1940 and Suprovabala died in 1942.
If the children had no knowledge about the title of her mother, there would not have been
any occasion for them to make any application for mutation of their names. Amal was
married in 1946. Allegedly, he and his wife started maltreating the sisters. Three of them,
as noticed hereinbefore, were yet to be
@page-SC551
married. The dispute between the parties rose to such a pass that three of the sisters had to
leave the house. They had to seek for a shelter somewhere else. So long as the
relationship between the parties was good, evidently, no problem arose. The mutation in
the name of the daughters, therefore, assumes considerable significance. It is not a
coincidence that three daughters had to leave the house and an application for mutation
was filed in the year 1958. Amal objected thereto and it would not be a matter beyond
anybody's comprehension that he had fought out the same bitterly. He must have done it
and despite the same mutation was done in the name of all. Only a suggestion was given
to PW-4 that the name of all the co-sharers was mutated only because husband of one of
the sisters was in Calcutta Municipal Corporation. If that be so, it was expected of Amal
to prefer an appeal thereagainst. It was expected that he would file a suit for declaration
to assert his own title as he did in the suit.
34. Mr. Gupta has relied upon a decision of the Patna High Court in Shahdeo Karan
Singh and others v. Usman Ali Khan [AIR 1939 Patna 462] wherein it was held that
obtaining mutation of names do not establish a gift. This may be so. But, however, in this
case, we are concerned with the conduct of the parties.
35. The fact that Amal allowed the order of mutation to attain finality, thus, would also be
a pointer to suggest that despite such bitter relationship between the parties he accepted
the same; more so, when mutation of one's name in the Municipal Corporation confers
upon him a variety of rights and obligations. He had rights and obligations in relation
thereto because, according to him, in relation to the said property vis-a-vis Calcutta
Municipal Corporation, he was residing with his wife, he allegedly inducted tenants and
had been realizing rent from them.
36. Tenants could have denied his title. He would not have been given permission to
make any additions or alterations. He, in absence of an order of mutation, might not be
given other amenities, if he had filed such an application in his own name. He, therefore,
knew that mutation of names of all the parties in the Calcutta Municipal Corporation may
bring forth to him many obstacles in future in the enjoyment of the property. At least he
could have taken such a step even after the suit filed by two of the sisters for
maintenance. The suit was decreed. Even in the said suit, the right to claim partition in
the properties had been kept reserved.
37. We have seen hereinbefore that the appellant examined herself as a witness. The wife
of Amal even did not do so. An adverse inference should be drawn against her.
38

. In Tulsi and Others v. Chandrika Prasad and Others [(2006) 8 SCC 322], this Court
observed - : 2006 AIR SCW 4905, (Paras 25 and 26)

"Before the courts below, the Appellant No. 1 did not examine herself. The Respondents
categorically averred in the plaint that the mortgage amount was tendered to her as also to
her husband. Having regard to the peculiar facts and circumstances of this case, we are of
the opinion that she should have examined herself to deny such tender.

In Sardar Gurbakhsh Singh v. Gurdial Singh and Another [AIR 1927 PC 230], the Privy
Council emphasized the need of examination of the parties as witnesses. [See also
Martand Pandharinath v. Radhabai, AIR 1931 Bom 97 and Sri Sudhir Ranjan Paul v. Sri
Chhatter Singh Baid and Anr., Cal LT 1999(3) HC 261]" AIR 1999 Cal 86

39. Daughter of Respondent No. 1 (Respondent No. 2) who was born in 1954 examined
herself as DW-1. She evidently had no knowledge about the transaction. She could not
have any. At least it was expected that Respondent No. 1 might have gathered some
knowledge keeping in view the conduct of her husband vis-a-vis the sisters in relation to
the property. Even otherwise, she was a party to the suit. No evidence, worth the name,
therefore, had been adduced on behalf of Respondent No. - 1.
40. Interestingly, Amal pleaded ouster. If ouster is to be pleaded, the title has to be
acknowledged. Once such a plea is taken, irrespective of the fact that as to whether any
other plea is raised or not, conduct of the parties would be material. If, therefore, plea of
ouster is not established, a' fortiori the title of other co-sharers must be held to have been
accepted.
41

. In T. Anjanappa and Others v. Somalingappa and Another [(2006) 7 SCC 570], it was
held - : 2006 AIR SCW 4368

"12. The concept of adverse possession contemplates a hostile possession i.e. a


possession which is expressly or impliedly in denial of the title of the true owner.
@page-SC552
Possession to be adverse must be possession by a person who does not acknowledge the
other's rights but denies them. The principle of law is firmly established that a person
who bases his title on adverse possession must show by clear and unequivocal evidence
that his possession was hostile to the real owner and amounted to denial of his title to the
property claimed. For deciding whether the alleged acts of a person constituted adverse
possession, the animus of the person doing those acts is the most crucial factor. Adverse
possession is commenced in wrong and is aimed against right. A person is said to hold the
property adversely to the real owner when that person in denial of the owner's right
excluded him from the enjoyment of his property."
42. It was further held - :
"21. The High Court has erred in holding that even if the defendants claim adverse
possession, they do not have to prove who is the true owner and even if they had believed
that the Government was the true owner and not the plaintiffs, the same was
inconsequential. Obviously, the requirements of proving adverse possession have not
been established. If the defendants are not sure who is the true owner the question of their
being in hostile possession and the question of denying title of the true owner do not
arise..."
43

. [See also Govindammal v. R. Perumal Chettiar and Ors., (2006) 11 SCC 600 and P.T.
Munichikkanna Reddy and Ors. v. Revamma and Ors., Civil Appeal No. 7062 of 2000
decided on 24th April, 2007]. 2006 AIR SCW 5794
reported in 2007 AIR SCW 2897

44

. Amal, therefore, could not have turned round and challenged the title of the appellant
and other respondents. [See Syed Abdul Khader v. Rami Reddy and Others (1979) 2 SCC
601]. AIR 1979 SC 553

45. PW-3 in her evidence made three significant statements - :


(i) The property was purchased for the benefit of the mother without keeping any
financial interest;
(ii) During the lifetime of her father, her mother used to exercise right, title and interest of
the property and she continued to do so even after her father's death.
(iii) Her mother used to say that the property belonged to her.
46. PW-4 Chandi Charan Ghosh is a common relation. According to him, Dr. Ghosh
acknowledged the title of his wife before him. We may not rely on his evidence in its
entirety but we intend to emphasise that at least some evidence has been adduced on
behalf of the appellant whereas no evidence, worth the name, has been adduced on behalf
of the defendants respondents. DW-1, as noticed hereinbefore, having born in 1954, could
not have any personal knowledge either in regard to the transaction or in regard to the
management of the property by Suprovabala whatsoever. She was even only four years
old when the name of all co-sharers was mutated in the records of the Calcutta Municipal
Corporation. She, however, admitted that there are two other houses standing in the name
of Dr. Ghosh. She even could not say anything about the power of attorney. She accepted
that the suit house was in the name of Suprovabala till 1958. She accepted that her father
objected to the mutation but the same was granted and no further step had been taken.
Although she claimed that she had been looking after the affairs, she could not give any
details about the purported litigations as against the tenants initiated by her father.
47. Reliance placed by Mr. Gupta on Hindu Women's Right to Property Act, 1937 is
misplaced as the property was purchased in the year 1935. The said Act had no
application at that point of time. There, however, cannot be any doubt whatsoever in
regard to the legal position that in respect of other properties of Dr. Ghosh, she had a
limited interest.
48

. Reliance by the High Court upon Mulla's Hindu Law for the proposition that husband
could not give immovable property as stridhan to his wife, in our opinion, is wholly
misplaced. Mulla has relied upon a decision of the Madras High Court in Venkata Rama
Rau v. Venkata Suriya Rau and Another [ILR (1877) Madras 281 at 286]. What Mulla in
fact says is that any gift or immovable property under Dayabhaga law would not become
wife's stridhan. It is, however, not in dispute that the amount necessary for purchasing an
immovable property can be a subject-matter of gift by a person in favour of his wife. [See
K.K. Das (supra)] AIR 1940 Cal 356

@page-SC553
49. We are also really not concerned with such a situation as the situation had undergone
a sea-change after coming into force of the Transfer of Property Act. The Transfer of
Property Act prescribes that any clog on transfer of property right to transfer would be
void. Dayabhaga does not prohibit gift of immovable property in favour of his wife by
her husband. It merely says that Dayabhaga did not recognize it to be her stridhan. It was
only for the purpose of inheritance and succession. The same has nothing to do with the
Benami Transaction of the Property and to determine the nature of transaction.
50

. Burden of proof as regards the benami nature of transaction was also on the respondent.
This aspect of the matter has been considered by this Court in Valliammal (D) By LRS. v.
Subramaniam and Others [(2004) 7 SCC 233] wherein a Division Bench of this Court
held - : 2004 AIR SCW 4948

"13. This Court in a number of judgments has held that it is well established that burden
of proving that a particular sale is benami lies on the person who alleges the transaction
to be a benami. The essence of a benami transaction is the intention of the party or parties
concerned and often, such intention is shrouded in a thick veil which cannot be easily
pierced through. But such difficulties do not relieve the person asserting the transaction to
be benami of any part of the serious onus that rests on him, nor justify the acceptance of
mere conjectures or surmises, as a substitute for proof. Refer to Jaydayal Poddar v. Bibi
Hazra, Krishnanand Agnihotri v. State of M.P., Thakur Bhim Singh v. Thakur Kan Singh,
Pratap Singh v. Sarojini Devi and Heirs of Vrajlal J. Ganatra v. Heirs of Parshottam S.
Shah. It has been held in the judgments referred to above that the question whether a
particular sale is a benami or not, is largely one of fact, and for determining the question
no absolute formulas or acid test, uniformly applicable in all situations can be laid. After
saying so, this Court spelt out the following six circumstances which can be taken as a
guide to determine the nature of the transaction - : AIR 1974 SC 171
AIR 1977 SC 796
AIR 1980 SC 727

(1) the source from which the purchase money came;


(2) the nature and possession of the property, after the purchase;
(3) motive, if any, for giving the transaction a benami colour;
(4) the position of the parties and the relationship, if any, between the claimant and the
alleged benamidar;
(5) the custody of the title deeds after the sale; and
(6) the conduct of the parties concerned in dealing with the property after the sale.
(Jaydayal Poddar v. Bibi Hazra1, SCC p. 7, para 6)
14. The above indicia are not exhaustive and their efficacy varies according to the facts of
each case. Nevertheless, the source from where the purchase money came and the motive
why the property was purchased benami are by far the most important tests for
determining whether the sale standing in the name of one person, is in reality for the
benefit of another. We would examine the present transaction on the touchstone of the
above two indicia.
*** *** ***
18. It is well settled that intention of the parties is the essence of the benami transaction
and the money must have been provided by the party invoking the doctrine of benami.
The evidence shows clearly that the original plaintiff did not have any justification for
purchasing the property in the name of Ramayee Ammal. The reason given by him is not
at all acceptable. The source of money is not at all traceable to the plaintiff. No person
named in the plaint or anyone else was examined as a witness. The failure of the plaintiff
to examine the relevant witnesses completely demolishes his case."
51. For the reasons aforementioned, the impugned judgment cannot be sustained which is
set aside accordingly. The judgment of the Trial Court is restored. The appeal is allowed.
In the peculiar facts and circumstances of this case, however, there shall be no order as to
costs.
Appeal allowed.
AIR 2008 SUPREME COURT 553 "Indian Overseas Bank, Anna Salai v. P. Ganesan"
(From : 2006 Lab IC 1519 (Mad))
Coram : 2 S. B. SINHA AND H. S. BEDI, JJ.
Civil Appeal No. 5369 of 2007 (arising out of SLP (C) No. 8683 of 2006), D/- 23 -11
-2007.
Indian Overseas Bank, Anna Salai and Anr. v. P. Ganesan and Ors. @page-SC554
Industrial Disputes Act (14 of 1947), Sch.2, Item 3 - INDUSTRIAL DISPUTE -
DEPARTMENTAL PROCEEDINGS - WRITS - DISCIPLINARY PROCEEDINGS -
Departmental proceedings - Stay of, on ground of pendency of criminal proceedings
against delinquent - Delinquent officers without filing reply to show cause notice, moved
High Court - Nothing shown that any complicated question of law, arose for
determination in criminal case - Large number of witnesses examined on behalf of
management - Delinquents did not cross examine them - Disciplinary proceedings
proceeded to great extent - No finding recorded that delinquents would be prejudiced if
departmental proceedings, not stayed - In facts and circumstances stay of departmental
proceedings without analyzing and applying principle of law evolved in binding
precedent - Liable to be set aside.
2006 Lab IC 1519 (Mad), Reversed.
2006 AIR SCW 2709, 2004 AIR SCW 7290 and 2007 AIR SCW 304, Disting.
Constitution of India, Art.226. (Paras 20, 22, 26, 27)
Cases Referred : Chronological Paras
2007 AIR SCW 304 (Disting.) 13, 25
2006 AIR SCW 2709 : AIR 2006 SC 2129 : 2006 (4) AIR Kar R 641 : 2006 (3) AIR Jhar
R 312 (Disting.) 13, 24
2004 AIR SCW 4558 : AIR 2004 SC 4127 : 2004 Lab IC 3226 (Ref.) 12, 19
2004 AIR SCW 7290 : AIR 2005 SC 1406 : 2005 Lab IC 1624 (Disting.) 13, 23
1999 AIR SCW 1098 : AIR 1999 SC 1416 : 1999 Lab IC 1565 (Ref.) 18, 19, 22
1996 AIR SCW 4160 : AIR 1997 SC 13 : 1996 Lab IC 2750 (Ref.) 10, 17, 19
AIR 1965 SC 155 (Rel. on) 16
AIR 1960 SC 806 (Rel. on) 16
Altaf Ahmed, Sr. Advocate, Vishal Chauhan, Ambrish Kumar, for Appellants; V. Prakash,
Sr. Advocate, Amit Sharma, Anupam Lal Das, S. Nanda Kumar, Satish Kumar,
Anandselvam, Ms. K. Indira, V. N. Raghupathy, for Respondents.
Judgement
S. B. SINHA, J. - :- Leave granted.
2. Whether pendency of a criminal case by itself would be a sufficient ground for stay of
the departmental proceedings is the principal question which arises for our consideration
in this case.
3. The basic fact of the matter is not in dispute.
4. Respondents 1 to 4 herein are the office bearers of the All India Indian Overseas Bank
SC/ST Employees Welfare Association. Indisputably another association was also
operating in the establishment of the appellant known as All India Indian Overseas Bank
Employees Union of which respondent No.5 is the President. Registration of similar
names gave rise to a dispute between the parties.
5. Indisputably a first information report was lodged on 27th January, 2005 with the
"Thousand Lights Police Station, Chennai" alleging that L. Balasubramanian, Respondent
No.5, herein was assaulted on 27th January, 2005 at about 12.35 p.m. within the bank
premises by Respondents Nos. 1 to 4 as well as by Mr. P. Rajalingam, the Deputy Chief
Officer working in the Regional Office of the appellant-bank. All the respondents are
indisputably employees of the bank. Their designations are as under - :-
(a) Respondent No.1 (P. Ganesan) is employed as Assistant Manager, Thiruvottiyur
Market Branch. He is also the General Secretary of All India Indian Overseas Bank
SC/ST Employees' Welfare Association.
(b) Respondent No.2 (S.Vijayakumar) is employed as a clerk in the Central Clearing
Office at Chennai. He is also the Vice President of the All India Indian Overseas Bank
SC/ST Employees' Welfare Association.
(c) Respondent No.3 (R Amalraj) is employed as a Messenger in the Inspection
Department, Central Office at Chennai. He is also the Organizing Secretary of the All
India Indian Overseas Bank SC/ST Employees' Welfare Association.
(d) Respondent No. 4 (A. Dakshina-moorthy) is employed as an Assistant Manager in the
Walltax Road Branch, Chennai. He is also the Deputy General Secretary of the All India
Indian Overseas Bank SC/ST Employees' Welfare Association.
(e) Respondent No.5 (L. Balasubra-manian) is employed as "Special Assistant" in
Foreign Exchange Department, Central Office at Chennai. He is also the President of All
India Indian Overseas Bank Employees Union (AIOBEU) as well as the President of the
National Confederation of Bank Employees (NCBE)."
6. An enquiry was conducted leading to discovery of certain additional facts. It may not
be necessary for us to take note of the same at this stage. Respondent No.5 was admitted
in the National Hospital for
@page-SC555
medical treatment. He remained confined therein for a period of twelve days. A charge-
sheet was filed by the Investigating Officer in the criminal case against Respondents Nos.
1 to 4 under Sections 341, 323, 324, 427, 307 and 506 (ii) of the Indian Penal Code.
Respondent Nos. 1 and 3 were arrested. They were, however, released on bail on 15th
February, 2005. Respondents Nos. 2 and 4, however, absconded. On or about 15th
February, 2005 they obtained anticipatory bail. Another first information was lodged by
Respondent Nos. - 1 to 4 against respondent No.5 in the said Thousand Lights Police
Station, Chennai.
7. Respondent Nos. 1 to 4 were placed under suspension by the appellant-bank by an
order dated 28th January, 2005. Charge-sheets dated 21st February, 2005 were also
served upon them. They were asked to show cause as to why disciplinary action be not
taken against them for their acts of indiscipline, insubordination, unruly and disorderly
behaviour, use of filthy language and most shameful abuses as well as murderous assault
on respondent No.5 within the office premises of the bank as also causing damage to
valuable properties and serious impairment to the banks prestige and reputation in the
estimation of public at large. Requests were made by the said respondents to revoke the
order of suspension by an application dated 9th March, 2005 assuring the authority that
they would fully co-operate with the authorities of the bank in the disciplinary
proceedings initiated against them. They, furthermore, requested the bank to grant them
one month's time to submit their reply to the show cause. Orders of suspension were
revoked on 16th April, 2005. No reply to the show cause notice, however, was filed by
them despite opportunities granted. Departmental enquiry was held against them on 18th
May, 2005 ; 3rd June, 2005, 17th June, 2005, 28th June, 2005, 8th July, 2005, 19th July,
2005, 29th July, 2005 and 4th August, 2005, the details whereof, as stated by the
appellants in their affidavit before the High Court, are as under - :-
"(a) ......The Enquiry was fixed for 18.5.2005 in respect of which the Respondents had
been duly intimated fairly in advance. However, the said Enquiry was adjourned to
3.6.2005 acceding to the written request dated 16.5.2005 of the respondents.
(b) The Enquiry was further adjourned to 17.6.2005 in response to the Respondents'
request for adjournment.
(c) The Enquiry was then fixed for 8.7.2005 in view of the Respondents' written request
dated 25.6.2005. However, the Respondents remained absent on 8.7.2005, and therefore
they were set ex parte, while adjourning the Enquiry to 19.7.2005.
(d) Meanwhile, the list of Management Witnesses was mailed to the Respondents Nos. 1
to 4 who admitted to have received the same.
(e) On 19.7.2005, only Respondents Nos.2 and 4 attended the Enquiry Proceedings, and
categorically admitted to have received the List of Management Witnesses, whereas
Respondents Nos.1 and 3 remained absent.
(f) The Enquiry was then adjourned to 29.7.2005 and again adjourned to 4.8.2005 when
except Respondent No.3 (R. Amalraj), the other Respondents Nos.1, 2 and 4 attended the
proceedings, and requested for further postponement, while representing that on the next
date of the proceedings, they would either bring their "Defence Representative" or they
would themselves conduct their "defence" without seeking any further postponement
thereof. Accordingly, the Enquiry was finally adjourned to 19.8.2005 to be held "on day-
to-day basis until conclusion".
8. While on the one hand the respondents were seeking adjournments and taking time
from the Enquiry Officer in the said disciplinary proceedings, they, on the other hand,
moved the High Court of Madras by filing writ petitions under Article 226 of the
Constitution of India. The said writ petitions were marked as W.P. Nos. 26176, 26177,
26178 and 26179 of 2005. Interlocutory applications were also filed therein praying for
stay of proceedings in the departmental enquiry pursuant to the said charge-sheet dated
21st February, 2005 on the premise that on identical facts criminal cases had been filed
against them. An ad interim order of stay was granted by the High Court by an order of
16th August, 2005 stating - :-
"Though this Court generally did not entertain Writ Petitions relating to Charge-memos
on the ground that Criminal proceedings are pending, the question as to whether the
Departmental Proceedings and the Criminal case are based on identical and similar set of
facts and whether the Charge in the Criminal case is of the grave nature
@page-SC556
which involves complicated questions of law and facts are the factors to be examined in
the Writ Petitions."
The said interim order of stay was produced before the Enquiry Officer. As the interim
order of stay was granted only for a period of four weeks and the same having not been
extended the enquiry proceedings continued. One witness being MW1 was examined on
21st October, 2005. The said departmental enquiry also proceeded on 22nd October, 2005
but the respondents did not participate therein on which date MW2 was examined. Yet
again on 24th October, 2005, MW3 and MW4 were examined and the matter was
adjourned to 25th October, 2005 when MW5 and MW6 were examined. On 26th
October, 2005, MW7 and MW8 were examined-in-chief and the enquiry was adjourned
to 27th October, 2005. Yet again on 28th October, 2005, MW9 and MW10 were
examined and the enquiry was adjourned to 29th October, 2005 on which date MW11
was examined. It is stated that the respondents attended the enquiry on 29th October,
2005 and nominated their Defence Representative to defend them. A prayer for
adjournment made on their behalf, however, was declined by the Enquiry Officer. MW11
was examined-in-chief on that date. The enquiry was adjourned to 31st October, 2005 on
which date MW12 and MW13 were examined-in-chief. It was adjourned to 9th
December, 2005 for cross-examination of the Management Witnesses.
9. The application for vacating the stay filed by the appellants on 6th September, 2005
was dismissed by a learned Single Judge of the High Court by an order dated 7th
December, 2005 stating - :-
"Once the Learned Single Judge has indicated the reasons which weighed with him in
exercising the extra-ordinary jurisdiction under Article 226 of the Constitution of India
against the impugned Charge-Memo, except to expedite the writ petitions, it may not be
proper for this Court to vacate the Interim Stay at this stage. Accordingly, the vacate stay
petitions, viz. WPMPs Nos. 2047 to 2050 of 2005 are dismissed. The Interim stay granted
by this Court on 16.8.2005 is made absolute.
Expedite the Writ Petitions and post the same for final hearing in the second week of
February, 2006."
10. Writ appeals preferred by the appellants against that order were disposed of by a
Division Bench of the Court by reason of the impugned judgment opining - :-
"14. In the instant case, there is no dispute that the criminal action and the disciplinary
proceedings are founded upon the same set of facts. In fact, the disciplinary proceedings
are solely based upon the criminal complaint lodged by the president of a rival union,
who is also facing prosecution with regard to the same incident. It has been conceded
before us that the bank had not conducted any independent enquiry before initiating the
impugned departmental proceedings.
15. In our opinion, in the peculiar facts and circumstances of the case on hand, fair play
requires that postponing of the departmental proceedings till the criminal cases are
decided. We are, therefore, of the view that the prayer made by the petitioners for
deferring the departmental proceedings till the conclusion of the criminal trial has to be
accepted and it is ordered accordingly."
11. The appellants are thus before us.
12

. Mr. Altaf Ahmed, learned senior counsel appearing on behalf of the appellants in
support of the appeal would, inter alia, submit that the High Court committed a serious
error in passing the impugned judgment in so far as it failed to take into consideration
that as the enquiry proceedings proceeded to a great extent the same should not have been
stayed. Reliance in this behalf has been placed on Kendriya Vidyalaya Sangathan and
others vs. T. Srinivas : (2004) 7 SCC 442. 2004 AIR SCW 4558

13

. Mr. G. Prakash, learned counsel appearing on behalf of the respondents, on the other
hand, submitted that the High Court having exercised its discretionary jurisdiction upon
application of law operating in this behalf, this Court should not exercise its jurisdiction
under Article 136 of the Constitution of India. Learned counsel urged that in a matter of
this nature where rival parties had clashed with each other and case and counter case have
been instituted one investigated by the Assistant Commissioner of Police and another by
an Inspector of Police, the respondents would be highly prejudiced if the departmental
proceedings are allowed to continue; particularly when the officers of the appellant-bank
have been proceeding with a bias. It was contended that the question as to whether there
exists any 2006 AIR SCW 2709
2004 AIR SCW 7290
2007 AIR SCW 304
@page-SC557
complicated question of law must be judged from the employees point of view, they
being belonging to the weaker section. Learned counsel would, in support of his
contention, strongly relied upon G.M. Tank vs. State of Gujarat and others : (2006) 5
SCC 446 ; Hindustan Petroleum Corporation Ltd. and others vs. Sarvesh Berry : (2005)
10 SCC 471 and Sathi Vijay Kumar vs. Tota Singh and others : 2006 (14) Scale 199.
14. Before embarking upon the rival contentions of the parties we may notice that
Respondent Nos. 1 to 4 have filed an application for quashing the order taking
cognizance against them before the High Court under Section 482 of the Code of
Criminal Procedure which was marked as Crl. O.P. No. 18163 of 2006 and by an order
dated 17th July, 2006 further proceedings in the criminal case have been stayed.
15. Legal position operating in the field is no longer res integra. A departmental
proceedings pending a criminal proceedings does not warrant an automatic stay. The
superior courts before exercising its discretionary jurisdiction in this regard must take
into consideration the fact as to whether the charges as also the evidence in both the
proceedings are common and as to whether any complicated question of law is involved
in the matter.
16

. In Delhi Cloth and General Mills Ltd. vs. Kushal Bhan : AIR 1960 SC 806 this Court
while holding that the employer should not wait for the decision of the criminal court
before taking any disciplinary action against the employee and such an action on the part
of the employer does not violate the principle of natural justice, observed - :- (Para
3)

"We may, however, add that if the case is of a grave nature or involves questions of fact
or law, which are not simple, it would be advisable for the employer to wait the decision
of the trial court, so that the defence of the employee in the criminal case may not be
prejudiced."
The same principle was reiterated in Tata Oil Mills Co. Ltd. vs. The Workmen : AIR 1965
SC 155.
17

. In State of Rajasthan vs. B.K. Meena and others : (1996) 6 SCC 417 this Court held - :-
1996 AIR SCW 4160, (Para 14)

"The staying of disciplinary proceedings, it is emphasised, is a matter to be determined


having regard to the facts and circumstances of a given case and that no hard and fast
rules can be enunciated in that behalf. The only ground suggested in the above decisions
as constituting a valid ground for staying the disciplinary proceedings is "that the defence
of the employee in the criminal case may not be prejudiced." This ground has, however,
been hedged in by providing further that this may be done in cases of grave nature
involving questions of fact and law. In our respectful opinion, it means that not only the
charges must be grave but that the case must involve complicated questions of law and
fact. Moreover, 'advisability', 'desirability' or 'propriety', as the case may be, has to be
determined in each case taking into consideration all the facts and circumstances of the
case."
18

. Capt. M. Paul Anthony v. Bharat Gold Mines Ltd. and another : (1999) 3 SCC 679 also
deserves to be noticed. This Court therein held that the departmental proceedings need
not be stayed during pendency of the criminal case save and except for cogent reasons.
The Court summarized its findings as under - :- 1999 AIR SCW 1098

"(i) Departmental proceedings and proceedings in a criminal case can proceed


simultaneously as there is no bar in their being conducted simultaneously, though
separately.
(ii) If the departmental proceedings and the criminal case are based on identical and
similar set of facts and the charge in the criminal case against the delinquent employee is
of a grave nature which involves complicated questions of law and fact, it would be
desirable to stay the departmental proceedings till the conclusion of the criminal case.
(iii) Whether the nature of a charge in a criminal case is grave and whether complicated
questions of fact and law are involved in that case, will depend upon the nature of
offence, the nature of the case launched against the employee on the basis of evidence
and material collected against him during investigation or as reflected in the charge sheet.
(iv) The factors mentioned at (ii) and (iii) above cannot be considered in isolation to stay
the Departmental proceedings but due regard has to be given to the fact that the
@page-SC558
departmental proceedings cannot be unduly delayed.
(v) If the criminal case does not proceed or its disposal is being unduly delayed, the
departmental proceedings, even if they were stayed on account of the pendency of the
criminal case, can be resumed and proceeded with so as to conclude them at an early
date, so that if the employee is found not guilty his honour may be vindicated and in case
he is found guilty, the administration may get rid of him at the earliest."
19

. The issue came up for consideration yet again in T. Srinivas (supra) where this Court
while analyzing B.K. Meena (supra) and Capt. M. Paul Anthony (supra) held that - :-
2004 AIR SCW 4558, (Para 11)
1996 AIR SCW 4160
1999 AIR SCW 1098

"From the above, it is clear that the advisability, desirability or propriety, as the case may
be, in regard to a departmental enquiry has to be determined in each case taking into
consideration all facts and circumstances of the case. This judgment also lays down that
the stay of departmental proceedings cannot be and should not be a matter of course."
20. The High Court, unfortunately, although noticed some of the binding precedents of
the Court failed to apply the law in its proper perspective. The High Court was not correct
in its view in concluding that the stay of the departmental proceedings should be granted
in the peculiar facts and circumstances of the case without analyzing and applying the
principle of law evolved in the aforementioned decisions. It, therefore, misdirected itself
in law. What was necessary to be noticed by the High Court was not only existence of
identical facts and the evidence in the matter, it was also required to take into
consideration the question as to whether the charges levelled against the delinquent
officers, both in the criminal case as also the disciplinary proceedings, were same.
Furthermore it was obligatory on the part of the High Court to arrive at a finding that the
non stayed of the disciplinary proceedings shall not only prejudice the delinquent officers
but the matter also involves a complicated question of law.
21. The standard of proof in a disciplinary proceedings and that in a criminal trial is
different. If there are additional charges against the delinquent officers including the
charges of damaging the property belonging to the bank which was not the subject matter
of allegations in a criminal case, the departmental proceedings should not have been
stayed.
22. Furthermore Respondent Nos. 1 to 4 have now moved the High Court for quashing of
the order taking cognizance of offence against them in the criminal proceedings. The
criminal proceedings have been stayed. Thus, even applying the principle laid down in
Capt. M. Paul Anthony (supra) the impugned judgment cannot be sustained. Before the
High Court no contention was raised that because Respondent Nos. 1 to 4 are office
bearers of a trade union, the authorities were biased against them. Nothing has been
shown that any complicated question of law arose for determination in the criminal case.
23

. Reliance placed by Mr. Prakash on Hindustan Petroleum Corporation Ltd. (supra) is not
apposite. There were certain special features which were noticed by this Court. In that
case itself it was held - :- 2004 AIR SCW 7290, (Para 10)

"There can be no strait-jacket formula as to in which case the departmental proceedings


are to be stayed. There may be cases where the trial of the case get prolonged by the
dilatory method adopted by the delinquent official. He cannot be permitted to, on one
hand, prolong the criminal case and at the same time contend that the departmental
proceedings should be stayed on the ground that the criminal case is pending.
(Emphasis supplied)
Therein the departmental proceedings were allowed to continue despite the fact that the
delinquent officer therein had been charged for commission of an offence under Section
13(1)(e) read with Section 13(2) of the Prevention of Corruption Act, 1988.
24

. In G.M. Tank (supra) this Court was dealing with a case where the delinquent officer
was acquitted. The said decision has no application in the instant case. 2006 AIR
SCW 2709

25

. Sathi Vijay Kumar (supra) pertains to a case involving election dispute. The question
which arose therein was as to whether despite the fact that there was no provision in the
Representation of the People 2007 AIR SCW 304
@page-SC559
Act, 1961 for striking out the pleadings, the Tribunal had the power to do so. We are not
concerned with such a question in this matter.
26. Furthermore the discretionary writ jurisdiction under Article 226 of the Constitution
of India should be exercised keeping in view the conduct of the parties. Respondents
made a representation that in the event the order of suspension is revoked, they would co-
operate with the Enquiry Officer. They kept on filing applications for extension of time
which were allowed. They took benefit thereof. Without, however filing show cause, they
moved the High Court. Furthermore before the Enquiry Officer also, as noticed
hereinbefore, although they had appointed the defence counsel, did not cross-examine the
witnesses examined on behalf of the Management. A large number of witnesses had
already been examined on behalf of the appellants. The disciplinary proceedings, as we
have noticed hereinbefore, have proceeded to a great extent. In such a situation we are of
the firm view that the discretionary jurisdiction should not have been exercised in favour
of Respondents 1 to 4 by the High Court.
27. For the reasons abovementioned the impugned judgment cannot be sustained which is
hereby set aside. The appeal is accordingly allowed.
28. We would, however, like to observe that in the event any prayer is made by the
respondents to cross-examine the witnesses examined on behalf of the appellants, the
Enquiry Officer may consider the same in accordance with law. Keeping in view the
conduct of the respondents they are directed to bear the costs of the appellants both
before the High Court as also before us. Counsel's fee assessed at Rs.25,000/-.
Appeal allowed.
AIR 2008 SUPREME COURT 559 "Dir. S. C. T. I. for Med. Sci. and Tech. v. M.
Pushkaran"
(From : Kerala)*
Coram : 2 S. B. SINHA AND H. S. BEDI, JJ.
Civil Appeal No. 5368 of 2007 (arising out of SLP (C) No. 6619 of 2007), D/- 23 -11
-2007.
Dir. S. C. T. I. for Med. Sci. and Tech. and Anr. v. M. Pushkaran.
(A) Constitution of India, Art.16 - EQUALITY IN PUBLIC EMPLOYMENT -
APPOINTMENT - SERVICE MATTERS - Appointment - Select listed candidates - Do
not have vested right to appointment - Unless mala fide of employer is proved - No
direction for appointment would be issued. (Para 16)
(B) Constitution of India, Art.16 - EQUALITY IN PUBLIC EMPLOYMENT -
APPOINTMENT - SERVICE MATTERS - POLICY DECISION - Appointment -
Vacancies of posts of Security Guards advertised - Select list prepared - Appointment
made to two of three vacant posts - Subsequent policy decision to contract out services of
Guards - Refusal to appoint person in select list on third vacant post in view of new
policy - Improper - Selected person was to be offered appointment at a point of time
when no policy decision was taken or was even in contemplation - There was thus no
reason to refuse appointment. (Para 18)
Cases Referred : Chronological Paras
2007 AIR SCW 5729 (Ref.) 17
2007 AIR SCW 5989 (Ref.) 17
2007 AIR SCW 6967 (Ref.) 17
2006 AIR SCW 4930 : 2006 Lab IC 4195 (Ref.) 17
2005 AIR SCW 2120 : AIR 2005 SC 2775 (Rel. on) (Pt.A) 15
2004 AIR SCW 5768 : AIR 2004 SC 5061 9, 17
2001 AIR SCW 1720 : AIR 2001 SC 1851 : 2001 Lab IC 1726 (Rel. on) Pt. A) 17
1999 AIR SCW 4300 : AIR 2000 SC 205 (Ref.) 14
1996 AIR SCW 691 : AIR 1996 SC 1145 : 1996 Lab IC 957 9
1995 AIR SCW 2044 : 1995 Lab IC 1775 (Rel. on) (Pt-A) 13
1993 AIR SCW 2314 (Rel. on) (Pt. A) 14
1991 AIR SCW 1583 : AIR 1991 SC 1612 : 1991 Lab IC 1460 (Rel. on) Pt. A) 9, 12,
17
AIR 1987 SC 169 : 1987 Lab IC 34 12
AIR 1987 SC 331 : 1987 Lab IC 447 14
AIR 1984 SC 1850 12
AIR 1977 SC 276 : 1977 Lab IC 52 14
AIR 1973 SC 2216 : 1973 Lab IC 1212 12, 14
L. Nageshwara Rao, Sr. Advocate, Ragenth Basant, Ms. Liz Mathew, Senthil Jagadeesan,
for Appellants; P. S. Narasimha, M. Gireesh Kumar, Khwairakpam Nobin Singh, for
Respondent.
* W. A. No. 2075 of 2006, D/- 4-12-2006 (Ker).
Judgement
S. B. SINHA, J. - :-Leave granted.
2. A short question which arises for
@page-SC560
consideration in this appeal is as to whether the respondent herein had any legal right for
being appointed against the post of three security guards advertised by the appellant
institute.
3. The basic fact of the matter is not in dispute. An advertisement was issued for
appointment to the post of security guards. There were three permanent posts. The select
list contained names of five candidates. The name of the respondent appeared at Sl. No. 4
therein. It was finalized on 11.04.2005. It had a validity period of one year i.e. upto
10.04.2006. Whereas two candidates were offered appointments on 13.04.2005 and
5.05.2005, the third candidate was offered appointment on 13.06.2005. He declined the
same. Respondent, however, for reasons best known to the appellant, was not offered any
appointment. He filed a writ petition questioning his non-appointment on 12.12.2005.
4. On or about 13.07.2005, however, a purported policy decision was taken to contract
out some of the services in a phased manner to make the administration efficient and cost
effective in the following terms - :
"After detailed deliberations, it was resolved that (i) a copy of the request sent to the
Employment Exchange, Thiruvanantha-puram may simultaneously be circulated/ posted
by the Institute to all the Employment Exchanges in Kerala especially in case of direct
recruitment of Group D posts specifying the number of candidates to be sponsored for
each post so as to achieve a wider coverage; (ii) in the case of Group C and B Direct
recruitment posts, paper advertisement shall, continue to be resorted in one or two
leading newspapers and (iii) for temporary vacancies/leave vacancies of Cleaning
Attendants/Security Guards, the external contract system prevalent in BMT Wing may be
extended to the Hospital Wing also in a phased manner."
5. A resolution was adopted by the Governing Body in a meeting held at the Institute on
29.12.2005 in the following terms - :
"We have been deliberating for quite sometime on contracting out some of the services on
a phased manner to make it more efficient and cost effective. It is noted that the security
at BMT Wing, Poojappura that was contracted out on a trial basis has been found
successful.
It was noted that at present there are 2 permanent vacancies of Security Guards and 2
permanent posts of Drivers that are lying vacant.
It was resolved to abolish these vacant posts and services may be contracted out/ hired
and ratify the decision of the Director not to fill the two vacant posts of Security Guards
and Drivers on permanent basis."
6. A learned Single Judge of the High Court by a judgment and order dated 20.09.2006
inter alia opined - :
"5. I do not think that the petitioner has made out a case for interference. No doubt, the
petitioner approached this Court on 12.12.2005. Ext. R1(b) decision is dated 29.12.2005.
But, I do not think that that is sufficient to overturn the decision of the management. The
question as to which are the posts to be filled up, is all a management decision.
Ordinarily, it is not for this Court to veto the wisdom of the employer in regard to the
posts which are to be retained and posts which are to be abolished. A decision to abolish a
post cannot be attacked by a person figuring in a rank list, unless, no doubt, an
extraordinarily case of malice or per se arbitrary action is established. Apparently, the
respondents felt that the post need not be retained, having regard to the advantages that
would flow from contracting of these services as also the pecuniary loss that would
otherwise flow. It is hard to characterize such a decision as arbitrary, as sought to be
shown in the Reply Affidavit. It is settled law that a person in the rank list has no legal
right to command the employer to appoint him. This is not a case where after having
taken a decision to fill up the posts, the respondent is not offering appointment to the
petitioner. Ext. R1(b) is not challenged by petitioner. In such circumstances, the Writ
petition fails and it is dismissed."
7. On an intra-court appeal preferred by the respondent herein from the said judgment
and order, the Division Bench, however, reversed the same, inter alia, holding - :
"...If the vacancy was abolished necessarily there was no question of appointment either
on substantial or on temporary basis. There is a decision to fill it up on temporary basis.
Thus, contract appointment reveal the existence of the vacancy. The 3rd among the
vacancies notified was one really intended to be filled up even on 13.6.2005 when the 3rd
rank holder in the list had been offered appointment. The decision contained in
@page-SC561
Ext. - R1(b) is the decision taken by the Governing Body. The petitioner/appellant need
not challenge the decision taken by the Governing Body, when there is no decision in
Ext. R1(b) to abolish the post but only to fill up the permanent posts on contract basis.
Then, the next person included in the list for regular appointment has to be considered..."
8. Appellants are, thus, before us - :
9. Mr. L. Nageshwara Rao, learned senior counsel appearing on behalf of the appellants,
submitted that the Division Bench of the High Court committed a serious error in holding
that there was a vacancy on a temporary basis.
It was urged that keeping in view a number of decisions of this Court, the impugned
judgment is wholly unsustainable. Reliance in this behalf has been placed on
Shankarasan Dash v. Union of India [(1991) 3 SCC 47], State of Bihar and Others v. Md.
Kalimuddin and Others [(1996) 2 SCC 7] and Punjab State Electricity Board and Others
v. Malkiat Singh[(2005) 9 SCC 22]. 1991 AIR SCW 1583
1996 AIR SCW 691
2004 AIR SCW 5768

10. Mr. P.S. Narasimha, learned counsel appearing on behalf of the respondent, on the
other hand, would submit that the institution had four departments. In some of the
departments a policy decision to contract out the services was taken; but, so far as the
department in which the respondent was to be appointed, no policy decision had been
adopted for contracting out the job of the security persons and in that view of the matter
the respondent had a legitimate expectation of his being appointed.
11. The law operating in the field in this behalf is neither in doubt nor in dispute. Only
because the name of a person appears in the select list, the same by itself may not be a
ground for offering him an appointment. A person in the select list does not have any
legal right in this behalf.
The selectees do not have any legal right of appointment subject, inter alia, to bona fide
action on the part of the State. We may notice some of the precedents operating in the
field.
12

. In Shankarsan Dash v. Union of India [(1991) 3 SCC 47], this Court held - : 1991
AIR SCW 1583

"7 . It is not correct to say that if a number of vacancies are notified for appointment and
adequate number of candidates are found fit, the successful candidates acquire an
indefeasible right to be appointed which cannot be legitimately denied. Ordinarily the
notification merely amounts to an invitation to qualified candidates to apply for
recruitment and on their selection they do not acquire any right to the post. Unless the
relevant recruitment rules so indicate, the State is under no legal duty to fill up all or any
of the vacancies. However, it does not mean that the State has the licence of acting in an
arbitrary manner. The decision not to fill up the vacancies has to be taken bona fide for
appropriate reasons. And if the vacancies or any of them are filled up, the State is bound
to respect the comparative merit of the candidates, as reflected at the recruitment test, and
no discrimination can be permitted. This correct position has been consistently followed
by this Court, and we do not find any discordant note in the decisions in State of Haryana
v. Subhash Chander Marwaha, Neelima Shangla v. State of Haryana, or Jatendra Kumar
v. State of Punjab." AIR 1973 SC 2216
AIR 1987 SC 169
AIR 1984 SC 1850

13
. Yet again in R.S. Mittal v. Union of India [1995 Supp (2) SCC 230], this Court held - :
1995 AIR SCW 2044

"It is no doubt correct that a person on the select panel has no vested right to be appointed
to the post for which he has been selected. He has a right to be considered for
appointment. But at the same time, the appointing authority cannot ignore the select panel
or decline to make the appointment on its whims. When a person has been selected by the
Selection Board and there is a vacancy which can be offered to him, keeping in view his
merit position, then, ordinarily, there is no justification to ignore him for appointment.
There has to be a justifiable reason to decline to appoint a person who is on the select
panel. In the present case, there has been a mere inaction on the part of the Government.
No reason whatsoever, not to talk of a justifiable reason, was given as to why the
appointments were not offered to the candidates expeditiously and in accordance with
law. The appointment should have been offered to Mr Murgad within a reasonable time
of availability of the vacancy and thereafter to the next
@page-SC562
candidate. The Central Government's approach in this case was wholly unjustified."
(Emphasis supplied)
14

. In Asha Kaul (Mrs.) and Another v. State of Jammu and Kashmir [(1993) 2 SCC 573],
this Court held - : 1993 AIR SCW 2314

"8. It is true that mere inclusion in the select list does not confer upon the candidates
included therein an indefeasible right to appointment (State of Haryana v. Subhash
Chander Marwaha; Mani Subrat Jain v. State of Haryana; State of Kerala v. A.
Lakshmikutty) but that is only one aspect of the matter. The other aspect is the obligation
of the Government to act fairly. The whole exercise cannot be reduced to a farce. Having
sent a requisition/request to the Commission to select a particular number of candidates
for a particular category, in pursuance of which the Commission issues a notification,
holds a written test, conducts interviews, prepares a select list and then communicates to
the Government - the Government cannot quietly and without good and valid reasons
nullify the whole exercise and tell the candidates when they complain that they have no
legal right to appointment. We do not think that any Government can adopt such a stand
with any justification today..." AIR 1973 SC 2216
AIR 1977 SC 276
AIR 1987 SC 331

{[See also A.P. Aggarwal v. Govt. of NCT of Delhi and Another [(2000) 1 SCC 600]}.
1999 AIR SCW 4300

15
. In Food Corpn. Of India and Others v. Bhanu Lodh and Others [(2005) 3 SCC 618], this
Court held - : 2005 AIR SCW 2120

"14. Merely because vacancies are notified, the State is not obliged to fill up all the
vacancies unless there is some provision to the contrary in the applicable rules. However,
there is no doubt that the decision not to fill up the vacancies, has to be taken bona fide
and must pass the test of reasonableness so as not to fail on the touchstone of Article 14
of the Constitution. Again, if the vacancies are proposed to be filled, then the State is
obliged to fill them in accordance with merit from the list of the selected candidates.
Whether to fill up or not to fill up a post, is a policy decision, and unless it is infected
with the vice of arbitrariness, there is no scope for interference in judicial review."
16. It is, therefore, evident that whereas the selectee as such has no legal right and the
superior court in exercise of its power of judicial review would not ordinarily direct
issuance of any writ in absence of any pleading and proof of mala fide or arbitrariness on
the part of the employer. Each case, therefore, must be considered on its own merit.
17

. In All India SC and ST Employees Association and Another v. A. Arthur Jeen and
Others [(2001) 6 SCC 380], it was opined - : 2001 AIR SCW 1720

"10. Merely because the names of the candidates were included in the panel indicating
their provisional selection, they did not acquire any indefeasible right for appointment
even against the existing vacancies and the State is under no legal duty to fill up all or
any of the vacancies as laid down by the Constitution Bench of this Court, after referring
to earlier cases in Shankarsan Dash v. Union of India. 1991 AIR SCW 1583

[See also Malkiat Singh (supra), Pitta Naveen Kumar and Others v. Raja Narasaiah
Zangiti and Others (2006) 10 SCC 261, State of Rajasthan and Ors. v. Jagdish Chopra
2007 (10) SCALE 470, Union of India and Others v. S. Vinodh Kumar and Others, 2007
(11) SCALE 257 and State of M.P. and Ors. v. Sanjay Kumar Pathak and Ors. 2007 (12)
SCALE 72]. 2004 AIR SCW 5768
2006 AIR SCW 4930
2007 AIR SCW 5729
2007 AIR SCW 5989
2007 AIR SCW 6967

18. The application of law would, therefore, depend upon the fact situation obtaining in
each case. The judgment of the High Court in view of the aforementioned authoritative
pronouncements cannot be said to be perverse. The respondent was to be offered with the
appointment at a point of time when no policy decision was taken. There was, thus, no
reason not to offer any appointment in his favour. Why the select panel was ignored has
not been explained. Even the purported policy decision was not in their contemplation.
We, therefore, do not see any reason to interfere with the impugned judgment.
19. Furthermore, the respondent is an ex-serviceman. He in ordinary case should have
been offered appointment particularly when three posts were vacant. The decision to
abolish posts was not taken at a point of
@page-SC563
time when he had filed the writ petition. It was expected that on 16.06.2005 when the
third candidate refused to join the post, he should have been offered the same.
20. The policy decision to abolish the posts as also contracting out the security services
was taken by the appellant much thereafter, viz., on or about 29.12.2005. We are,
therefore, of the opinion that it is not a fit case where we should interfere with the
impugned judgment. The appeal is dismissed. No costs.
Appeal dismissed.
AIR 2008 SUPREME COURT 563 "T. Vijendradas v. M. Subramanian"
(From : AIR 2006 Mad 288)
Coram : 2 S. B. SINHA AND H. S. BEDI, JJ.
Civil Appeal No. 4727 of 2007 (arising out of SLP (C) No. 17549 of 2006), D/- 9 -10
-2007.
T. Vijendradas and Anr. v. M. Subramanian and Ors.
Civil P.C. (5 of 1908), O.21, R.92(4), O.34, R.1 - AUCTION SALE - MORTGAGE -
OBJECT OF AN ACT - MUNICIPALITIES - SUPREME COURT - Object of - Suit for
recovery of statutory charge by Municipality against owner of property - Owner not
disclosing that he had already sold property and allowing ex parte decree to be passed -
Auction sale - Upset price reduced twice without intimation to judgment-debtor -
Property purchased in auction by wife of judgment-debtor vendor - Suit by first purchaser
for declaration and possession against vendor and his wife - Municipality not made party
- Supreme Court in its jurisdiction under Art. 142 of Constitution, considering object of
O. 21, R. 92(4) and (5) directed plaintiff-first purchaser to pay amount which Court paid
to Municipality out of auction amount and also pay amount of property tax to
Municipality.
Constitution of India, Art.142.
T.N. District Municipalities Act (5 of 1920), S.85.
One of the objects sought to be achieved in amending O. 21, R. 92 was to do complete
justice to the parties so as to enable the auction-purchaser to get back the amount from
the decree-holder and revive the execution proceedings so that the decree-holder may
proceed against the judgment-debtor for realisation of the decretal amount.(Para 28)
In a suit for recovery of a meagre amount filed by the Municipalities against the property
owner, the property owner did not disclose that he had already sold the property and ex
parte decree was passed. In execution of decree, property was sold in auction. While
holding auction upset price was reduced twice without intimation to judgment-debtor.
Property was purchased in auction by the wife of judgment- debtor, without disclosing
the relationship. Having learnt the fact of auction sale, the first purchaser filed a suit for
declaration and possession against her vendor and his wife i.e. the auction-purchaser.
Municipality, however, was not impleaded in the suit. Supreme Court in such case
observed that as the defendants were guilty of "suppressio veri" and in the event of any
conflicting interest, Supreme Court in exercise of its equity jurisdiction under Art. 142 of
the Constitution of India should weigh the effect of a fraud and the consequence of non-
impleadment of a necessary party. It was, therefore, held that the scale of justice weighs
in favour of the person who is a victim of fraud and, thus, any relief should not be refused
in his favour, only because he might have been wrongly advised. The purport and object
for which O. - 21, R. 92(5) was enacted furthermore would be better subserved if it is
directed that the plaintiff shall pay amount which Court paid to Municipality out of
amount of auction. It was further observed by the Supreme Court that the plaintiffs had
not claimed any relief against the Municipality. The Municipality's right to realise the
amount of property tax together with interest, if any, was not in dispute. Although the
liability of owner in terms of the 1920 Act to pay the property tax continued, the plaintiffs
was also liable to pay the amount of property tax after the date of sale. In a case of this
nature, therefore, the plaintiffs can be directed to pay the amount of property tax by way
of redemption of mortgage in favour of the Municipality and that if any amount is
available with Court out of amount received from auction sale, same may be paid to the
defendants. (Paras 27, 28, 29, 31)
Cases Referred : Chronological Paras
2007 AIR SCW 2212 : AIR 2007 SC 1546 (Ref.) 15
2007 AIR SCW 3734 : AIR 2007 SC 1971 23
2007 (3) Scale 349 24
@page-SC564

2006 AIR SCW 4905 : AIR 2006 SC 3359 (Ref.) 14


2004 AIR SCW 4462 : AIR 2004 SC 4504 : 2004 Lab IC 3206 (Ref.) 21
2003 AIR SCW 3518 : AIR 2003 SC 2889 23
(2003) 8 SCC 319 14
2002 AIR SCW 2658 : AIR 2002 SC 2426 : 2002 Lab IC 2324 (Ref.) 21
1994 AIR SCW 243 : AIR 1994 SC 853 14
AIR 1979 SC 1682 22
AIR 1967 SC 1390 (Ref.) 25
AIR 1963 SC 786 (Disting.) 22
K. V. Viswanathan, K. V. Venkataramani, B. Ragunath and R. Nedumaran, for Appellants;
K. Parasaran, Sr. Advocate Mrs. V. Mohana, Ms. Hari Priya and Anirudh Sharma, for
Respondents.
Judgement
1. S. B. SINHA, J. - :-Leave granted.
2. A short but interesting question in regard to interpretation of Order XXI, Rule 92(4) of
the Code of Civil Procedure, 1908 (for short, 'CPC') is involved in this appeal.
Facts :
3. R. Venugopal (since deceased), original defendant No. 3, was the owner of the suit
property consisting of 8 cents and 116 sq. ft. of land situated within the Coimbatore
Municipal Corporation (for short, 'the Municipality'). He transferred his right, title and
interest therein by reason of a registered deed of sale in favour of one Sakunthala, the
original plaintiff No. 1 in the suit and mother of Respondent Nos. 1 and 2 herein. It is,
however, not in dispute that the factum of sale was not intimated to the authorities of the
Municipality either by the vendor or by the vendee thereof. The vendee's name was not
mutated in the records of the Municipality. Indisputably, property tax in respect of
premises in question had not been paid for the period from 01.04.1970 to 31.03.1973.
The property tax for two quarters, thus, was to be paid by Venugopal whereas the rest was
to be paid by the vendee. The matter relating to payment of property tax is governed by
the Tamil Nadu District Municipalities Act (Tamil Nadu Act V of 1920) (for the sake of
brevity, hereinafter called and referred to as 'the 1920 Act').
4. With a view to enforce a statutory charge as laid down in the 1920 Act, the
Municipality filed a suit, which was marked as O.S. No. 986 of 1973. Sakunthala was not
a party therein. Venugopal although entered his appearance in the suit but at the
subsequent stages, did not appear. Yet again he did not inform the court about the fact that
he had sold the property in favour of Sakunthala. He allegedly sent an information to
Sakunthala in regard thereto, but admittedly, the latter was sent at a wrong address.
The said suit was decreed in 1978. The Municipality filed an application for execution of
the said decree, which was marked as E.P. No. 2620 of 1978. The property in question
was put up on auction sale, the upset price wherefor was fixed at Rs. - 20,000/- by an
order dated 19.03.1979. However, as allegedly no buyer was available, an application for
reduction of upset price was filed being E.A. No. 284 of 1979 for bringing it down from
Rs. 20,000/- to Rs. 5,000/-. It was, however, directed to be reduced to Rs. 10,000/-, but
therefor no notice was issued to the judgment- debtor, as is required in terms of Order
XXI, Rule 66, CPC. Yet again without any other order being passed for further reduction
of the upset price, the suit property was sold on auction for a sum of Rs. 8,010/- in favour
of one Manickam, original defendant No. 1, on 06.08.1979.
5. It will be appropriate to place on record that although at one point of time there existed
a dispute as to whether the said Manickam was the wife of Venugopal or not, it stands
accepted that a relationship of husband and wife had been existing by and between them.
The said auction sale was not only confirmed by an order dated 11.10.1979, but a sale
certificate was also issued on or about 04.12.1979. An application for withdrawal of the
said auction amount was filed by the Municipality, which had been allowed and a cheque
was directed to be issued in its favour. The said order was complied with on 20.02.1981.
6. Manickam allegedly sold the said property in favour of one M/s. Ramans for a sum of
Rs.41,066/- by a registered deed of sale dated 22.08.1981. Having learnt that the property
in question had been sold in auction, Sakunthala filed a suit for declaration and
possession, against her vendor, his mother and wife on or about 27.08.1981. The plaintiff
then allegedly had no knowledge in regard to the sale of the said property in favour of
one M/s. Ramans. In her written statement, defendant No. 1 denied that she was the wife
of the judgment-debtor and disclosed that she had sold the
@page-SC565
property during the pendency of the said suit. M/s. Ramans despite knowledge of the
pendency of the suit sold the said property in favour of the appellants herein. M/s.
Ramans and the appellants herein thereafter on an application made in that behalf were
impleaded as defendants in the said suit. Indisputably, in their written statement, the
appellants raised a plea that the Municipality was a necessary party. The said suit was
decreed by a judgment and decree dated 19.12.1996. An appeal thereagainst preferred by
the appellants has been dismissed by a judgment and order dated 12.01.1999. A Second
Appeal preferred by the appellants has also been dismissed.
Submissions - :
7. The principal contentions raised in this appeal by Mr. K.V. Viswanathan, learned
counsel for the appellants are - :
(i) In view of the mandatory provisions contained in sub-rule (4) of Rule 92 of Order
XXI, CPC, the decree-holder was a necessary party and in its absence the judgment and
decree passed by the courts below are nullities.
(ii) In terms of Order I, Rule 9, CPC, non-impleadment of a necessary party would render
a suit not maintainable.
(iii) Sakunthala having questioned the title of Venugopal as on the date of holding of the
auction, in terms of the aforementioned provision, she was bound to implead the
Municipality as a party defendant in the suit.
(iv) The finding of commission of fraud as alleged by the plaintiff either on the part of the
Municipality or on the part of the appellants and his predecessors, would not render the
auction sale void.
(v) Had the Municipality been impleaded as a party, it could have shown that no fraud
had been committed on the court in the matter of holding of the auction.
(vi) The plaintiff having not examined herself, and only an auditor having been examined
on her behalf, an adverse inference should have been drawn in that regard by the courts
below.
8. Mr. K. Parasaran, learned Senior Counsel appearing on behalf of the respondents, on
the other hand, would submit - :
(i) A specific plea of fraud having been raised on the part of the auction purchaser, who
was wife of the judgment-debtor and the defendant in the suit, and a finding of fact in that
behalf having been arrived at by the trial court as also by the appellate court, the
Municipality cannot be said to be a necessary party.
(ii) The plaintiffs-respondents had rightly been held to have not raised any contention in
regard to commission of fraud on the part of the Municipality, as has been held by the
learned Trial Judge as also the Appellate Court, it was not a necessary party to the suit.
(iii) The suit as framed did not attract the provisions of Order XXI, Rule 92, CPC, as a
decree passed in favour of the judgment-debtor Venugopal was a mortgage decree, as
contemplated under Order XXXIV, Rule 1, CPC vis-a-vis the Transfer of Property Act,

(iv) Order XXI, Rule 92, in any event, must be given a contextual meaning.
(v) The principle of caveat emptor will be applicable in this case.
(vi) Appellants being purchasers pendent lite are bound by the decree passed by the
learned Trial Judge.
(vii) The position of the judgment-debtor 'Venugopal' being that of a trustee, as envisaged
under Section 88 of the Indian Trusts Act, 1882 it was obligatory on his part to protect the
interest of Sakunthala.
Statutory Provisions - :
9. Sections 85 and 88 of the 1920 Act, which are relevant for the purpose of adjudication
of this case, read as under - :
"85. Property tax a first charge on property.- The property tax on buildings and lands
shall, subject to the prior payment of land revenue, if any, due to the Government
thereon, be a first charge upon the said buildings or lands and upon the movable property,
if any, found within or upon the same and belonging to the person liable to such tax."
"88. Obligation of transferor and transferee to give notice of transfer.- (1) Whenever the
title of any person primarily liable to the payment of property tax on any premises to or
over such premises is transferred, the person whose title is transferred, and the person to
whom the same shall be transferred shall within three months after the execution of the
instrument of transfer or after its registration if it be registered or after the transfer is
effected, if no instrument be executed, give notice of such
@page-SC566
transfer to the Executive authority.
(2) In the event of death of any person primarily liable as aforesaid the person to whom
the title of the deceased shall be transferred, as heir or otherwise shall give written notice
of such transfer to the executive authority within one year from the death of the deceased.
(3) The notice to be given under this section shall be in such form as the executive
authority may direct and the transferee or the person to whom the title passes, as the case
may be, shall, if so required, be bound to produce before the executive authority any
documents evidencing the transfer or succession.
(4) Every person who makes a transfer as aforesaid without giving such notice to the
executive authority shall in addition to any other liability which he incurs through such
neglect, continue to be liable for the payment of property tax assessed on the premises
transferred until he gives notice or until the transfer shall have been recorded in the
municipal registers but nothing in this section shall be held to affect-
(a) the liability of the transferee for the payment of the said tax, or
(b) the prior claim of the municipal council under section 85".
Fraud - :
10. Relationship between Venugopal and Manickam is not disputed. A contention has,
however, been raised by the respondents themselves that Sakunthala was a national of
Malaysian origin. Venugopal and Manickam, as noticed hereinbefore, denied and
disputed their relationship of husband and wife. Despite such a stand taken at least before
the High Court, the same was given up. Keeping that aspect in view we may notice the
findings of the learned Trial Judge - :
"...The sale deed Ex.A.1 has been suppressed and an ex parte decree has been obtained in
that suit. It is a fraudulent act of Venugopal.........The suit property has been brought in
auction in E.P. No. 2620 of 1978 suppressing the already existing sale in favour of the
plaintiff Sakuntala. This Court therefore, considers that such an act is unjust and
fraudulent."
The First Appellate Court also found as under :
"...It is also clear that the 3rd respondent did not inform the plaintiffs/appellants that for
the arrears of property tax in respect of the property sold to them, a decree has been
obtained and that the 3rd respondent did not appear in court and inform the Court that he
had sold the property to the plaintiffs and hence the plaintiffs also should be impleaded as
parties to the suit. D.W.1, the 5th respondent examined on the side of the respondents, has
stated in his evidence that on 2.8.1978 Venugopal appeared in Court in the case filed by
the Corporation, that Venugopal had means to pay the arrears of tax of Rs. 406/- and that
even after the decree in the said suit, Venugopal did not pay the arrears of property tax of
Rs. 406/-. It is therefore, clear that the 3rd respondent has acted fraudulently by not
paying the arrears of tax even after the sale in favour of the plaintiffs, though he had
means to pay and the plaintiffs were also not informed about the same.........Even when
the property was brought for auction, the 3rd respondent did not contest it. Though the
property tax arrears is a small amount of Rs. 406/-, he has not chosen to pay the same.
It was further found - :
"The 3rd respondent in his written statement has stated that the 1st respondent is not his
wife, that the 1st respondent in the written statement has stated that the 3rd respondent is
not her husband and that she is not married........The fraudulent conduct is very clear from
their statement in their written statement that the 1st respondent is not the wife of the 3rd
respondent. There is no doubt that Corporation has filed a suit for recovery of a small
amount of Rs. 406.76 towards arrears of property tax and obtained a decree, which was
followed by execution proceedings, which was also not contested, that wantonly allowing
all the proceedings to go on, the 3rd res-pondent, through his wife, fraudulently took the
property in auction for Rs. 8,010/-..."
The Court held - :
"...The 3rd respondent in his written statement has stated that though it is not necessary
for him to inform anything about the proceedings to anybody regarding the auction sale,
he informed the plaintiff by registered post and that under those circumstances there is no
chance to say that he acted fraudulently. It was stated by the learned counsel for the
appellants that on the side of the respondents, in support of the above, Ex. B.1 has been
filed. A perusal
@page-SC567
of Ex. B.1 shows that the registered post has been returned stating that there is no such
addressee. In the above registered post, the address found is M. Sakunthala, wife of
Muthyya Chettiar, Thisoolpadam, Thirupathur Post, Ramanathapuram District. But it is
clear from the plaint documents that the place of plaintiff-Sakunthala is "Siruoodalpatti
village, Tirupathur Taluk, Sivaganga District. There is no doubt that the 3rd respondent
want only sent the registered post giving a false address, got it returned and filing it into
Court is clearly a continuation of his fraudulent action".
11. Although not very relevant, but we may notice that the vendor had never paid the
arrears of property tax to the Municipality. He had other properties and the quantum of
tax for which the suit was filed was only Rs. 406.76. A further finding of fact had been
arrived at that despite knowledge, no notice was sent to Sakunthala at her known address
i.e. Sirukoodalpatti Village which was the address disclosed in the sale deed, but sent to
another address. In the suit only the registered cover had been filed but the contents
thereof had not been disclosed. It was obligatory on the part of the appellant to bring the
office copy of the said notice on the record.
On the basis of the aforementioned findings, the courts below had arrived at a conclusion
that Venugopal had not been acting bona fide and an attempt had been made to create
evidence to cover up his fraud.
Statutory application - :
12. Section 55(1)(g) of the Transfer of Property Act, 1882 envisages payment of taxes in
respect of the property by the vendor up to the date of sale. The liability of the vendee to
pay the property tax arises only from the date of sale. However, Sections 85 and 88 of the
1920 Act provide that in the event, the factum of sale is not communicated, the liability of
the vendor shall continue. Consequently a statutory charge is created on the property. A
person having an interest in the property, therefore, might have a right of redemption. A
suit for realisation of the dues in respect of a property in respect whereof a statutory
charge is created, a suit could also be filed by the Municipality, apart from taking
recourse to the procedure provided for realisation of the said dues as land revenue.
13. A decree passed in terms of Order XXXIV, Rule 1, CPC is somewhat different from a
decree passed in an ordinary money suit. If in terms of the provisions of the statute,
plaintiffs-respondents are found to be liable to pay the property tax, at an appropriate
stage, they can redeem the charge. The plaintiffs-respondents did not claim any relief
against the Municipality. Its right, title and interest over the property having regard to the
statutory charge thereover has not been denied or disputed. It is also not a case where
fraud was alleged to have been committed by the Municipality either in conspiracy with
the judgment-debtor Venugopal or otherwise. What was contended was that having
regard to the fact that upset price was brought down without following the procedure as
contained in Order XXI, Rule 66, CPC, an irregularity or fraud was practised in the
context of the conduct of the auction sale. Irregularity committed in conducting the
auction sale and commission of fraud either on court or on a party to the suit stand
completely on different footings.
14. Mr. Viswanathan's contention that in effect and substance the plaint contained
allegation of fraud on Municipality, in our opinion, cannot be accepted. As is well-
known, the pleadings must be read in their entirety for the purpose of proper construction
thereof. What had been alleged in the plaint is the commission of fraud by Venugopal.
His conduct throughout the proceedings has been taken into consideration by the courts
below; viz., despite transferring the property he did not intimate thereabout to the
Municipality. Even when the suit was filed, he did not make any attempt to raise a plea
which was expected of any reasonable and prudent man that he ceased to be liable to pay
the property tax as he had already transferred the property. The purported intimation
given to the plaintiffs was also found to be an act of fraud on his part inasmuch as the
purported notice to the plaintiff had deliberately been sent at a wrong address. The
contents of the notice had also not been brought on record. Despite having sufficient
means he voluntarily suffered an ex parte decree. He never objected to reduction of upset
price. He despite the fact did not contest the suit, participated in the auction sale and
purchased the property in the name of his wife. Not only that the auction-purchaser sold
the property to a third party, who again despite the knowledge of pendency of the suit
@page-SC568
transferred the property in favour of the appellants.

The effect of commission of such fraud either on court or on a party is no longer res
integra. [See S.P. Chengalvaraya Naidu (Dead) by LRs. v. Jagannath (Dead) by L.Rs. and
others [(1994) 1 SCC 1]; Ram Chandra Singh v. Savitri Devi [(2003) 8 SCC 319] and
Tulsi and others v. Chandrika Prasad and others [(2006) 8 SCC 322]. 1994 AIR
SCW 243
2006 AIR SCW 4905

15

. In A.V. Papayya Sastry and others v. Govt. of A.P. and others [(2007) 4 SCC 221], it was
held - : 2007 AIR SCW 2212
"21. Now, it is well settled principle of law that if any judgment or order is obtained by
fraud, it cannot be said to be a judgment or order in law. Before three centuries, Chief
Justice Edward Coke proclaimed - :
"Fraud avoids all judicial acts, ecclesiastical or temporal."
22. It is thus settled proposition of law that a judgment, decree or order obtained by
playing fraud on the Court, Tribunal or Authority is a nullity and non est in the eye of
law. Such a judgment, decree or order - by the first Court or by the final Court - has to be
treated as nullity by every Court, superior or inferior. It can be challenged in any Court, at
any time, in appeal, revision, writ or even in collateral proceedings."
Non-impleadment of the Municipality in a suit - :
16. Keeping in view the principles of law, we may notice the effect of non-impleadment
of the Municipality, as envisaged under Order XXI, Rule 92(4) and Rule 92(5), CPC,
which reads as under - :
"92. Sale when to become absolute or be set aside-
(4) Where a third party challenges the judgment-debtor's title by filing a suit against the
auction-purchaser, the decree-holder and the judgment-debtor shall be necessary parties
to the suit.
(5) If the suit referred to in sub-rule (4) is decreed, the Court shall direct the decree-
holder to refund the money to the auction-purchaser, and where such an order is passed
the execution proceeding in which the sale had been held shall, unless the Court
otherwise directs, be revived at the stage at which the sale was ordered."
17. The purported allegations of the fraud on the Municipality in the plaint reads as under
-:
"IX The property originally proclaimed for sale at Rs. 20,000/-. The upset price was
allowed to be reduced on application without notice being issued to other side. The entire
execution is not only fraud but also irregular, illegal and void."
... ... ... ...
XI The property originally proclaimed for sale at Rs. 20,000. Due to irregularities in the
procedure and want of proper publication and publicity the sale did not take place and
there were series of application to reduce and upset price and the property ultimately sold
on the upset price fixed at Rs. 8000/-..."
The findings of the Trial Court on the said issue are as under - :
"...Ex A-9 is the copy of the interim application filed in the execution petition No. - 2620
of 1978 by the corporation. That application has been filed to reduce the upset price from
Rs. 20,000/- to 5,000/-. Further it has been stated in that petition that reduction of upset
price is prayed for on account of the fact that the house constructed in the suit property is
in a dilapidated condition. For the purpose of reduction of price, false details have been
furnished..........With the intention of suppressing the real facts from the eyes of the court,
it has been stated that there is a building in the suit property in a dilapidated
condition.......Further in that application it is stated that 12 years have lapsed after the
judgment in OS No. 986/73. But the judgment has been delivered in 1973. Execution
petition has been filed in 1978, 5 years after the judgment. While so, it is clear that the
said application contains allegations against the truth..."
18. It may be true that an observation had been made that the Municipality did so at the
instance of Venugopal, but there was no warrant for the following finding - :-
"...No notice has been given to the respondents in the petition filed for reduction of upset
price. Because of these defects in brining the property for auction in the execution
petition, it is clear that that there are legal flaws...."
It was also found - :
"It was argued on the side of the defendants that since Coimbatore Corporation
@page-SC569
has not been made a party to this suit, this suit has to be dismissed. This suit has been
filed for the relief of declaration that the suit property absolutely belongs to the plaintiff
on the basis of Ex. A.1. For deciding the right with regard to the suit property,
Corporation is not a necessary party. The powers to decide as to which property belongs
to whom has not been granted to the Corporation. Only a power to collect property tax is
vested with the Corporation. The Court, therefore, holds that in the suit filed to decide as
to whether the suit property belongs to the plaintiff or the defendants, the Corporation is
an unnecessary party. It is not necessary to implead Corporation as a party to decide the
disputes arising between the individuals. Further in the suit and in the execution
proceedings by the Corporation, Plaintiff Sakunthala is not a party. While so, this Court
holds that it is not necessary to add Corporation as a party to the suit. The Corporation
has also not sent any notice at any time stating that the suit property did not belong to the
plaintiffs. Hence this Court holds that so far as this case is concerned, the Corporation
need not be impleaded as a party. Hence this issue is answered against the defendants."
Such findings had been affirmed by the Appellate Court, stating - :
"A perusal of the above shows that the Corporation need not be impleaded as a party nor
any case to be filed to set aside the auction proceedings and this will not affect the
plaintiff's case in any way.
From the above, it is clear that the 3rd respondent has executed Ex. A.1 sale deed in
favour of the first plaintiff for valid consideration, which has not been denied by the 3rd
respondent, that the 1st respondent became entitled to the property so purchased later
under the court auction sale, which has been held to be invalid under law, this court hold
that the first plaintiff is entitled to a declaration that the suit property purchased under
Ex.A.1 belongs to the first plaintiff and after her plaintiffs 2 and 3 are entitled to the suit
property..."
19. We have noticed hereinbefore that a suit filed in terms of Order XXXIV, Rule 1, CPC
stands on a different footing. Non-joinder of a proper party in terms of Order I, Rule 9
would not render a suit not maintainable. We are, however, not oblivious of the purport
and object in amendment of Order XXI, Rule 92 CPC. The Law Commission in its 54th
Report recommended - :
"21.48D. Whatever be the correct view on the existing language, it appears to us that
something should be done to improve the position. No doubt, to permit the auction-
purchaser to sue for refund from the decree-holder, is to add to the troubles of the decree-
holder, and thus to delay execution. But that seems to be the only possible alternative. As
between the decree-holder and the auction-purchaser, if some one has to suffer, the
former should suffer.
It may not be feasible for the court to inquire into the title of the judgment-debtor (at the
time of the proclamation), in an elaborate manner; but that does not answer the basic
question, namely, when a sale held by a Court and culminating in a certificate issued by
the court is held to be a nullity for want of title, by reason of a defect discovered after
expiry of the period for making objections under rule 91 etc., is it justice to dispose of the
purchaser's grievance by saying that the purchaser purchased the property at his peril?
The decree-holder should re-imburse him for the loss suffered by him, because it is the
decree-holder at whose instance the sale was held. The abstract principle that there is no
warranty at court sales fails to yield a just result in this case.
The auction-purchaser should have a right to sue the decree-holder. Where a third party
challenges the judgment-debtor's title by filing a suit against the auction-purchaser, the
decree-holder and judgment-debtor should be necessary parties, and in that suit the court
shall direct the decree-holder to refund the money to the auction-purchaser.
If such a decree is passed, the original execution proceedings shall be revived at the stage
where the sale was ordered, unless the court otherwise directs. This provision is necessary
to avoid complications as to limitation."
Recommendation
"21.49. We, therefore, recommend that the following sub-rules should be added to Order
21, rule 92 - :-
"(5) Where a third party challenges the judgment-debtor's title by filing a suit against the
auction-purchaser, the decree-holder and the judgment-debtor shall be necessary parties
to the suit;
@page-SC570
(6) If the suit referred to in sub-rule (5) is decreed, the court shall, direct the decree-
holder to refund the money to the auction-purchaser, and, where such an order is passed,
the execution proceedings in which the sale had been held shall, unless the court
otherwise directs, be revived at the stage at which the sale was ordered."
20. The Statement of Objects and Reasons also lead to the same inference, wherein it
was, inter alia, stated - :
"Rule 92 is being amended to provide that where a third party challenges the judgment-
debtor's title by filing a suit against the auction-purchaser, the decree holder and the
judgment-debtor should be necessary parties to that suit and if the suit is decreed, the
Court shall direct the decree-holder to refund the money to the auction-purchaser. With a
view to avoiding complications with regard to limitation, the rule further provides that
where a decree is passed in favour of such third party, the original execution proceeding
will become revived at the stage where the sale was ordered unless the Court otherwise
directs."
21

. It is true the purpose of impleadment of a necessary party is to see as to whether without


it no order can be made effectively. If an effective order can be made, the suit will not be
defeated. A decree which is passed in terms of Order XXI, Rule 92(4) does not take into
consideration the effect of a statutory charge on a property, vis-a-vis the statutory right of
any persons having interest in the property to redeem or sell the same at any point of
time. When a fraud is practised on a court, the same is rendered a nullity. In a case of
nullity, even the principles of natural justice are not required to be complied with.
[Kendriya Vidyalaya Sangathan and Others v. Ajay Kumar Das and Others (2002) 4 SCC
503 and A. Umarani v. Registrar, Cooperative Societies and Others (2004) 7 SCC 112-
para 65]. 2002 AIR SCW 2658
2004 AIR SCW 4462
22

. Once it is held that by reason of commission of a fraud, a decree is rendered to be void


rendering all subsequent proceedings taken pursuant thereto also nullity, in our opinion, it
would be wholly inequitable to confer a benefit on a party, who is a beneficiary
thereunder. The decisions rendered in Udit Narain Singh Malpaharia v. Additional
Member, Board of Revenue, Bihar [(1963) Supp. 1 SCR 676] and Profulla Chrone
Requitte and Others v. Satya Chorone Requitte [(1979) 3 SCC 409] whereupon reliance
has been placed by Mr. Viswanathan, may not have any application in a case of this
nature in view of the fact that the principal question which, in our opinion, would be
more pertinent is as to whether even in a situation of this nature, the discretionary
jurisdiction under Article 136 of the Constitution of India should be invoked particularly
when the party raising the said question has been impleaded as a party. AIR 1963 SC
786
AIR 1979 SC 1682

23

. We would assume that the courts below proceeded on a wrong premise that Order XXI,
Rule 92(4) is not attracted, but the question as regards fraud committed by the judgment-
debtor has been gone into a great details. We are satisfied that the findings arrived at by
the learned Trial Judge and affirmed by the First Appellate Court also by the High Court
are equitable. It is in a situation of this nature, we are of the opinion that this Court in
exercise of its jurisdiction under Article 142 of the Constitution of India can pass an
appropriate order with a view to do complete justice to the parties. [Chandra Singh v.
State of Rajasthan and Another (2003) 6 SCC 545 Oriental Insurance Co. Ltd. v. Brij
Mohan and Ors. -2007 (7) SCALE 753 - para 14]. 2003 AIR SCW 3518
2007 AIR SCW 3734

Are the Appellants bound by the decree ?


24. In this case the appellants as also the aforementioned M/s Ramans purchased the
property pendente lite. They would be deemed to have notice of the sale made by
Venugopal in favour of the plaintiff-respondents. Section 3 of the Transfer of Property
Act provides that a person is said to have notice of the fact when he actually knows that
fact, where any transaction relating to immovable property is required by law to be and
has been effected by a registered instrument. [See Lachhman Dass v. Jagat Ram and
Others 2007 (3) SCALE 349]. They have purchased the property with notice, apart from
the fact that the transfer made in their favour was hit by Section 52 of the Transfer of
Property Act. The decree obtained by the Municipality had been passed under Order
XXXIV CPC. Respondents had a subsisting right of redemption. Order XXXIV, Rule 15
CPC provides that all the provisions contained therein shall, as far as may be, apply to a
mortgage by deposit of title-deeds within
@page-SC571
the meaning of Section 58, and to a charge within the meaning of Section 100 of the
Transfer of Property Act. The charge created under Section 85 of the 1920 Act would be
one covered by Section 100 of the Transfer of Property Act. Section 100 of the Transfer
of Property Act reads as under - :
"S. 100 Charges- Where immoveable property of one person is by act of parties or
operation of law made security for the payment of money to another, and the transaction
does not amount to a mortgage, the latter person is said to have a charge on the property;
and all the provisions hereinbefore contained which apply to a simple mortgage shall, so
far as may be, apply to such charge.
Nothing in this section applies to the charge of a trustee on the trust-property for expenses
properly incurred in the execution of his trust, and, save as otherwise expressly provided
by any law for the time being in force, no charge shall be enforced against any property
in the hands of a person to whom such property has been transferred for consideration
and without notice of the charge."
25

. In Mangru Mahto and Others v. Shri Thakur Taraknathji Tarake-shwar Math and Others
[1967 (3) SCR 125], this Court held - : AIR 1967 SC 1390

"A lease granted by the mortgagor, out of the ordinary course of management, though not
binding on the mortgagee, is binding as between the mortgagor and the lessee. Such a
lessee acquires an interest in the right of redemption and is entitled to redeem. If such a
lease is created before the institution of a suit relating to the mortgage, the lessee must be
joined as a party to the suit under Order 34 Rule 1 CPC; otherwise he will not be bound
by the decree passed in the suit and will continue to retain his right of redemption. But in
view of Section 52 of the Transfer of Property Act, if the mortgagor grants such a lease
during the pendency of a suit for sale by the mortgagee, the lessee is bound by the result
of the litigation. If the property is sold in execution of the decree passed in the suit, the
lessee cannot resist a claim for possession by the auction-purchaser. The lessee could
apply for being joined as a party to the suit and ask for an opportunity to redeem the
property. But if he allows the property to be sold in execution of the mortgage decree and
they have now lost the present case, the lessees allowed the suit lands to be sold in
execution of the mortgage decree and they have now lost the right of redemption. They
cannot resist the claim of the auction purchaser of recovery of possession of the lands."
26. Materials have been brought on record to show that a preliminary decree and a final
decree in terms of Order XXXIV have been passed. The learned Trial Judge also found
so. It is also appropriate to notice the following findings of the learned Trial Judge in
regard to issuance of the two encumbrances certificates - :
"...Ex. A.17 is the encumbrance certificate. Thiru E. Ayyasami filed an application and
obtained that encumbrance certificate. That encumbrance certificate has been issued on
28.6.1983 from the office of the Sub Registrar. In that the sale deed dated 12.11.70 in
favour of the plaintiff is shown. Similarly in that encumbrance certificate, the sale deed
dated 4.12.79 in respect of the suit property and another sale deed dated 22.8.81 in favour
of Manickam find a place. Ex. A. 18 is the questionnaire regarding family card. In that it
is mentioned that Manickam is the wife of Venugopal. But in the written statement filed
by the defendant it is stated that Manickam is not the wife of Venugopal. Ex. A. 16 is the
voters list issued to the family of Venugopal. In that document also it is stated that
Manickam is the wife of Venugopal. Ex. A. 20 is the copy of the complaint filed in the
criminal Court in STC No. 2119/94. That complaint has been filed by advocate Thiru N.
Sundaravadivelu and advocate Thiru S. Krishnamurthy. This document has been fled to
show that the defendants Venugopal and Manickam together engaged those two lawyers
and were conducting the case.
32. Ex. B. 4 is the encumbrance certificate obtained on the application by Thiru James.
That certificate has been issued on 24.7.80 by the Sub Registrar's Office. In that the sale
in favour of Sakunthala do not find a place. But in Ex. A. 17 the encumbrance certificate
obtained on 28.6.1983, the sale deed in favour of the plaintiff Sakunthala has been
mentioned. In an encumbrance certificate issued three years before Ex. B4 encumbrance
certificate was issued, the sale deed in favour of the plaintiff Sakunthala finds a place. In
an encumbrance certificate obtained thereafter that sale does not find a place. Hence it is
clear that because of the arrangements made by the defendant
@page-SC572
Venugopal, the sale in favour of the plaintiff Sakunthala does not find a place there.
Similarly Ex. B. 5 is the encumbrance certificate obtained by Thiru M.P. Ramakrishnan
on his application issued by the office of the Sub Registrar on 26.4.84 containing no
encumbrance. Hence it is clear that the matters contained in that encumbrance certificate
are false. Ex. B. 6 is the encumbrance certificate obtained by Mr. M.P. Ramakrishnan on
12.4.84. In that, the sale dated 22.8.1981 in favour of Manickam finds a place. Therefore
is clear that the encumbrance certificates marked on the side of the defendants contained
details contrary to truth. This court therefore hold that those encumbrance certificates
have been issued only in connivance with Venugopal. This Court hold that by selling the
property to 5th defendant to get more profit, the sale in favour of Sakunthala in the
encumbrance certificate was suppressed.
Conclusion :
27. Appellants and their predecessors, therefore, are also guilty of suppressio veri.
Ordinarily a statute shall prevail over the common law principle. However, in a case of
this nature, in the event of any conflicting interest, this Court in exercise of its equity
jurisdiction under Article 142 of the Constitution of India is to weigh the effect of a fraud
and the consequence of non-impleadment of a necessary party. We would hold that the
scale of justice weighs in favour of the person who is a victim of fraud and, thus, we
should not refuse any relief in his favour, only because he might have been wrongly
advised. The purport and object for which Order XXI, Rule 92(5) was enacted
furthermore would be better subserved if it is directed that the respondents shall pay the
amount which the Court paid to the Municipality out of the amount of auction.
28. We have noticed hereinbefore that one of the objects sought to be achieved in
amending Order XXI, Rule 92 was to do complete justice to the parties so as to enable
the auction purchaser to get back the amount from the decree-holder and revive the
execution proceedings so that the decree-holder may proceed against the judgment-debtor
for realisation of the decretal amount. In this case, the plaintiffs-respondents had not
claimed any relief against the Municipality. The Municipality's right to realise the amount
of property tax together with interest, if any, is not in dispute. Although the liability of
Venugopal in terms of the 1920 Act to pay the property tax continued, it has been
accepted at the Bar that the plaintiffs-respondents was also liable to pay the amount of
property tax after the date of sale. In a case of this nature, therefore, the plaintiffs-
respondents can be directed to pay the amount of property tax by way of redemption of
mortgage in favour of the Municipality.
29. If any amount is available with the court out of the amount received from the auction
sale, the same may be paid to the appellants. Appellants would also be otherwise entitled
to file an appropriate suit as against Manickam and others.
30. We, therefore, are of the opinion that subject to the terms aforementioned, the
appellants should not be granted any relief.
31. For the views we have taken, it is not necessary for us to go into the other contentions
raised by the learned counsel for the parties. We, therefore, direct the plaintiffs-
respondents to deposit the amount paid to the Municipality out of the auction amount by
the order of the executing court within six months from date and on such payment, the
appeal shall stand dismissed. However, in the facts and circumstances of the case, there
shall be no order as to costs.
Order accordingly.
AIR 2008 SUPREME COURT 572 "Rajendra Datta Zarekar v. State of Goa"
(From : Bombay)*
Coram : 2 G. P. MATHUR AND G. S. SINGHVI, JJ.
Criminal Appeal No. 32 of 2007, D/- 4 -12 -2007.
Rajendra Datta Zarekar v. State of Goa.
(A) Penal Code (45 of 1860), S.375 - RAPE - FALSE IMPLICATION - Rape - False
implication of accused - Normally improbable - Rape leaves a permanent scar and has a
serious psychological impact on victim and also her family members - No one would
normally concoct a story of rape just to falsely implicate a person. (Para 13)
(B) Penal Code (45 of 1860), S.375 - RAPE - Rape - Rupture of Hymen - Not essential to
constitute rape. (Para 14)
@page-SC573

(C) Penal Code (45 of 1860), S.376 - RAPE - PENOLOGY - Rape - Sentence - Victim a
child of 6 years - Accused a grown up person of 20 years - No adequate or special reasons
to impose less sentence made out - Sentence less than 10 years cannot be awarded in
view of S. 376(2)(f) - Fine of Rs. 10,000/- imposed on accused, however, reduced to Rs.
1,000/-. (Para 15)
Cases Referred : Chronological Paras
2006 AIR SCW 4550 : AIR 2006 SC 3098 : 2006 Cri LJ 4594 (Rel. on) 14
2004 AIR SCW 2486 : AIR 2004 SC 2636 : 2004 Cri LJ 2109 (Disting.) 15
1992 AIR SCW 1480 14
Vinay Navare and Naresh Kumar, for Appellant; Ms. A. Subhashini, for Respondent.
* Cri. Appeal No. 4 of 2005, D/- 16-8-2006 (Bom).
Judgement
G. P. MATHUR, J. :- This appeal has been preferred under S. 2(A) of the Supreme Court
(Enlargement of Criminal Appellate Jurisdiction) Act, 1970 against the judgment and
order dated 16-8-2006 of Goa Bench of Bombay High Court by which the appeal filed by
the State was allowed and judgment and order dated 28-7-2004 of First Ad hoc Assistant
Sessions Judge, Panaji in Sessions Case No. 1 of 2004 acquitting the accused was set
aside. The High Court convicted the appellant-Rajendra under Ss. 376(2)(f) and 342, IPC
and sentenced him to ten years R.I. and a fine of Rs. 10,000/- under the first count and
one month's R.I. and a fine of Rs. 1,000/- under the second count.
2. The case of the prosecution, in brief, is that P.W. 2 Satyam Ahire along with his wife
P.W. 1 Pushpa and three children was residing in Usgao in Ponda. He was working as a
security officer in Bethora Industrial Estate. In the evening of 14-10-2003 his eldest
daughter Supriya was studying at her home while the second daughter P.W. 8 Sonia, aged
about six years, was playing in the courtyard of his house. P.W. 4 Mohandas Goankar,
uncle of Pushpa, owned some rooms close by which were given on rent to some boys
who were working in the Nestle factory. The accused-Rajendra was living on rent in one
of these rooms. At about 5.00 p.m. on 14-10-2003 P.W. 1 Pushpa heard cries of her
daughter Sonia from inside the room, which was in occupation of the accused-Rajendra.
She went to the room and found it closed from inside and, therefore, she knocked at the
door. After some time the accused-Rajendra opened the door of the room. She enquired
from the accused as to what he was doing inside the room along with her daughter but he
kept quiet. On enquiry Sonia told her mother that the accused-Rajendra took her inside
his room while she was playing in the courtyard. He closed the door, switched off the
light and forced her to lie down on the mat. Thereafter the accused removed her panty
and also removed his own pant and lied down over her. He inserted his private part in the
private part of Sonia and did some movement. Pushpa then brought Sonia to her own
house and called her sister and other family members. After Satyam Ahire (father of
Sonia) had come back, they went to the Police Station, Ponda where they reached at
about 9.00 p.m. The PSI of Ponda Police Station sent them to Medical College where
Sonia was medically examined and the doctor confirmed that she had been subjected to
rape. Thereafter a formal FIR was registered at 11.45 p.m. on 14-10-2003 at P.S. Ponda.
3. After the case had been registered at the police station the same was investigated by
P.W. 11 Shivram Vaigankar, PSI. He recorded statements of witnesses under S. 161, Cr.
P.C. The accused-Rajendra was arrested at 5.00 a.m. on 15-10-2003 and the clothes
which he was wearing were taken into custody. He also took in custody the clothes of the
girl Sonia. He prepared a site plan and panchnama of the scene of occurrence in presence
of two witnesses and also seized a bed sheet and nylon mat. A photograph of the room
was also taken. After completing investigation he submitted charge-sheet under Ss. 342
and 376, I.P.C. against the accused-Rajendra.
4. The prosecution in support of its case examined 11 witnesses before the trial Court and
filed some documentary evidence. The learned Assistant Sessions Judge, after appraisal
of the evidence, came to the conclusion that the case of the prosecution was not
established beyond doubt against the accused-Rajendra and he accordingly acquitted him
by the judgment and order dated 28-7-2004. Feeling aggrieved by the judgment and order
of the learned Assistant Sessions Judge the State preferred an appeal before the High
Court. The High Court, after a detailed consideration of the evidence, allowed the appeal
and reversed the judgment of acquittal recorded by the Assistant Sessions Judge. The
High Court convicted
@page-SC574
the accused under S. 376(2)(f), I.P.C. and sentenced him to ten years R.I. and a fine of Rs.
10,000/- and in default to undergo six month's S.I. He was further convicted under S. 342,
I.P.C. and was sentenced to undergo one month's R.I. and a fine of Rs. 1,000/- and in
default to undergo 15 days' S.I.
5. We have heard Mr. Vinay Navare, learned counsel for the appellant-Rajendra and Ms.
A. Subhashini, learned counsel for the State of Goa, and have perused the record.
6. P.W. 4 Mohandas Gaonkar has deposed that he owns three houses and has four rooms
in one of his houses out of which one room had been given on rent to some boys who
were working in Nestle factory. The accused-Rajendra, who was employed in Nestle
factory, had been given one room on rent and his duty hours were from 8.00 a.m. to 4.30
p.m. On 14-10-2003 he returned from the market at about 6.30 p.m. when P.W. 1 Pushpa,
her sister and some others were present there in the house. Pushpa told him about the
incident regarding commission of rape by the accused-Rajendra on Sonia inside the
room.
7. P.W. 2 Satyam Ahire deposed that Mohandas Gaonkar is his wife's uncle and is a close
neighbour. He had given one room in his house to boys working in Nestle factory and the
accused-Rajendra was a tenant in one such room. On 14-10-2003 he had gone to the
market and returned from there after 5.00 p.m. When he returned home he found his
daughter Sonia weating and was in a very bad condition. She was lying in bed. His wife
P.W. 1 Pushpa informed him that the accused-Rajendra had pulled Sonia inside his room
and had committed rape upon her. He has further deposed that he then went to police
station by rikshaw and from there he was sent to medical college for medical examination
of Sonia.
8. P.W. 1 Pushpa has given a detailed version of the incident. She has deposed that she
has three children and Sonia, who is aged about six years, is her second daughter. Her
uncle Mohandas Gaonkar lives nearby and he had let out one of the rooms in his house to
the accused-Rajendra. In the evening of 14-10-2003 her elder daughter was studying at
home while Sonia was playing in the courtyard near the house. While she was preparing
some eatables she heard the cries of Sonia coming from the side of the house of her uncle
Mohandas Gaonkar. She immediately rushed there and found that the room which was
under the tenancy of the accused-Rajendra was closed from inside. She also heard the
cries of Sonia coming from inside the room. She then knocked at the door which was
opened by the accused after about five minutes. She enquired from the accused as to what
had happened but he kept quiet. Sonia, who had rushed to her mother, informed her that
she was taken inside the room by the accused and thereafter he closed the room from
inside and switched off the light. The accused forced her to lie down on the mat and after
removing her panty also removed his pant and lied down over her. He inserted his private
part inside her private part inside her private part and did some movement. Pushpa then
came to her own house and called her family members. After her husband had come back
from the market she went to the police station to lodge the report. The FIR was lodged by
her which is Exhibit 8. The police referred her daughter Sonia to Goa Medical College
for her medical examination. She went there where Sonia was medically examined. She
was cross-examined at length but nothing material has come out in the same which may
throw any doubt regarding the prosecution version of the incident. In cross-examination
she has said that her husband was not present and had gone to market and he returned
from the market at about 5.30 p.m. Sonia was crying loudly and was having great
difficulty in passing urine. She has further deposed that one day before her statement in
Court Sandesh, brother of the accused-Rajendra, and his mother had come to her house
pleading for mercy. She denied the defence suggestion that she had some kind of
relations with the accused and used to borrow money from him and had falsely
implicated him as she wanted some more money from him.
9. The prosecutrix Sonia was examined as P.W. 8. The learned Assistant Sessions Judge
put questions to her in order to ascertain whether she was in a position to give statement
in Court. After being satisfied about her mental capacity, her statement was recorded. The
learned Assistant Sessions Judge has noted that she wanted to be near her mother at the
time of recording her statement and that the accused had been sent little away with the
consent of his
@page-SC575
advocate so that the witness may be comfortable. Sonia stated that the accused-Rajendra
was residing in the room near her house. When she was near the house of her aunty the
accused came near her and pulled her inside his room and closed it from inside. He
removed her panty and his own pant and made her lie down on the mat. He lied over her,
inserted his private part in her private part and did some movement. She cried out of pain.
After few minutes her mother came and knocked at the door. After couple of minutes the
accused opened the door. She then narrated the entire incident to her mother. She
identified the panty and frock, which she was wearing at the time of the incident and
were seized by police, during the course of her statement in Court. She further said that
she was taken to the hospital for her medical examination.
10. P.W. 10 Dr. E.J. Rodrigues, Associated Professor in Forensic Medicine, Medical
College, Goa, examined Sonia at about 11.45 p.m. on 14-10-2003 in the presence of Dr.
Mrinalini, lecturer in the Medical College. He has deposed that the girl Sonia was of thin
built having a height of 97 cms. and weighing 26 kgs. The gait of Sonia was slightly
painful. Her genital development was of infant type nature. Pubic hairs were not erupted
and there were no injuries on inner aspects of thighs. There was a bruise reddish 2 x 1.5
cms. area on right labia majora and right labia minora, which was tender to touch. There
was a laceration of 5 mm. x 2 mm. on right labia minora near the clitoris which was
tender to touch. The hymen was intact. There were no fresh or old tears to hymen.
Hymnal opening admits tip of little finger. Hymnal border was bruised, edematous and
tender to touch. The vaginal contents and vaginal walls were normal. He opined as
under :-
"I certify that on physical genetical examination there is evidence of recent penetration.
Vaginal swabs and smear slides were retained for serological examination."
11. The same doctor also examined the accused-Rajendra at 12.15 p.m. on 15-10-2003 at
the request of police of Ponda Police Station in the presence of Dr. Girish Kamat. There
were no injuries on his body. His genital development was good. His pubic hairs were
black and not matted with blood or semen (preserved). On physical and genital
examination there was nothing to suggest that Rajendra was incapable of sexual
intercourse.
12. P.W. 3 Raju Sunktankar is the photographer who took photographs of the room and he
has proved the same. P.W. 5 Tarun Kumar is a panch witness of seizure of clothes of P.W.
8 Sonia and of accused-Rajendra. P.W. 6 Narayan is the panch witness of seizure of
matters and bed sheet. P.W. 9 Sanjay had examined the blood group of Sonia and of
accused-Rajendra. P.W. 11 Shivram Vaigankar, PSI of Ponda Police Station has deposed
about the lodging of FIR by Pushpa and registration of the case after he had received the
medical examination report of Sonia. He has deposed about the various steps taken by
him during the course of investigation of the case.
13. We have given above the gist of the evidence adduced by the prosecution. The
evidence shows that the accused-Rajendra was living as tenant in a room in the house of
P.W. 4 Mohandas Gaonkar, who is uncle of P.W. 1 Pushpa. The room in which the
accused was living is close to the house of the victim.In the evening of 14-10-2003 when
Sonia was playing outside the courtyard of her house the accused pulled her and took her
to his own room, bolted it from inside and after removing the clothes of Sonia and his
own pant committed rape upon her. The cries of Sonia attracted her mother Pushpa who
came there, knocked at the door and after some time the accused opened the same. Sonia
was crying loudly and she narrated the incident to her mother. Pushpa went to the police
station after her husband P.W. 2 Satyam Ahire and some other relations had come there.
The police sent Sonia to Medical College where she was medically examined by P.W. 10
Dr. Rodrigues who certified that she had been subjected to rape. Thereafter the FIR of the
incident was registered at the police station at 11.45 p.m. the same night. In spite of fairly
lengthy cross-examination nothing has come out in the statements of Pushpa and Sonia
which may throw even a slightest doubt on the prosecution version of the incident. There
is no enmity of any kind between Pushpa and the accused-Rajendra which may impel her
to falsely implicate the accused. Though a suggestion regarding taking of some money by
Pushpa from the accused has been made and a further suggestion has been made that she
wanted to have some kind of relationship with the accused but the same has not at all
been made probable
@page-SC576
much established by any evidence. The rape leaves a permanent scar and has a serious
psychological impact on the victim and also her family members and, therefore, no one
would normally concoct a story of rape just to falsely implicate a person. In the present
case there was not even an iota of evidence to show that P.W. 1 Pushpa or her husband
Satyam Ahire had any reason whatsoever to falsely implicate the accused-Rajendra. We
have carefully gone through the evidence and in our opinion the evidence lead by the
prosecution fully establishes the case against the accused-Rajendra beyond any shadow
of doubt.
14

. Learned counsel for the appellant has next submitted that the doctor had found that the
hymen of Sonia was intact and, therefore, the charge for rape under S. 376, I.P.C. as
defined in S. 375, I.P.C. has not been made out. An identical question was considered by
a Bench of this Court in Santosh Kumar v. State of M.P., 2006 (8) JT SC 171, and para 10
of the report is reproduced below :- 2006 AIR SCW 4550, Para 7

"10. The question, which arises for consideration, is whether the proved facts establish
the offence of rape. It is not necessary for us to refer to various authorities as the said
question has been examined in considerable detail in Madan Gopal Kakkad v. Naval
Dubey, 1992 (3) JT (SC) 270 and paras 37 to 39 of the said judgment are being
reproduced below : 1992 AIR SCW 1480
"37. We feel that it would be quite appropriate, in this context, to reproduce the opinion
expressed by Modi in Medical Jurisprudence and Toxicology (Twenty First Edition) at
page 369 which reads thus :
"Thus to constitute the offence of rape it is not necessary that there should be complete
penetration of penis with emission of semen and rupture of hymen. Partial penetration of
the penis within the labia majora or the vulva or pudenda with or without emission of
semen or even an attempt at penetration is quite sufficient for the purpose of the law. It is
therefore quite possible to commit legally the offence of rape without producing any
injury to the genitals or leaving any seminal stains. In such a case the medical officer
should mention the negative facts in his report, but should not give his opinion that no
rape had been committed. Rape is crime and not a medical condition. Rape is a legal term
and not a diagnosis to be made by the medical officer treating the victim. The only
statement that can be made by the medical officer is that there is evidence of recent
sexual activity. Whether the rape has occurred or not is a legal conclusion, not a medical
one."
38. In Parikh's Textbook of Medical Jurisprudence and Toxicology, the following passage
is found :
"Sexual intercourse.- In law, this term is held to mean the slightest degree of penetration
of the vulva by the penis with or without emission of semen. It is therefore quite possible
to commit legally the offence of rape without producing any injury to the genitals or
leaving any seminal stains."
39. In Encyclopedia of Crime and Justice (Vol. 4) at page 1356, it is stated :
". . . . . .even slight penetration is sufficient and emission is unnecessary."
Therefore, the absence of injuries on the private parts of a victim specially a married lady
cannot, ipso facto, lead to an inference that no rape has been committed."
Here the victim was a very young girl of six years of age and it is quite likely that full
penetration did not take place as the accused is a grown up person of over 20 years of
age. The injuries clearly indicate that rape, as defined in S. 375, I.P.C. did take place.
15

. Learned counsel for the appellant further submitted that the sentence of ten years R.I.
awarded by the High Court is very severe and the same may be reduced. It may be
mentioned here that S. 376(2)(f), I.P.C. specifically provides that where the victim is less
than 12 years of age the sentence awarded shall not be less than 10 years but it may be for
life and the accused shall also be liable to fine. The proviso, no doubt, says that the Court
may for adequate and special reasons to be mentioned in the judgment, impose a sentence
of imprisonment for a term of less than ten years. Here the victim P.W. 8 Sonia was aged
about six years and, therefore, the case is fully covered by Cl. (f) of sub-section (2) of S.
376, I.P.C. and the sentence awarded cannot be less than ten years unless there are
adequate and special reasons for doing so. We do not find any adequate or special reasons
for imposing a sentence of less than ten years. Learned counsel for the appellant placed
reliance on State of Chhattisgarh v. Derha (2004) 9 SCC 699, for reducing the sentence.
In the 2004 AIR SCW 2486

@page-SC577
authority cited what weighed with the Court was that the accused was hardly eighteen
years of age and had already served about six and half years' imprisonment. He was
married and had a family. In these circumstances the Court considered it proper to reduce
the sentence to seven years. Such is not the case here and, therefore, we are legally bound
to award a sentence of ten years. R.I. However, we feel that the fine of Rs. 10,000/-
awarded under S. 376(2)(f), I.P.C. is excessive and the same is reduced to Rs. 1,000/- and
the fine of Rs. 1,000/- awarded under S. 342, I.P.C. is set aside.
16. In the result the appeal is dismissed with the modification that the fine of Rs. 10,000/-
imposed under S. 376(2)(f), I.P.C. is reduced to Rs. 1,000/- and the fine of Rs. 1,000/-
imposed under S. 342, I.P.C. is set aside. The substantive sentence of ten years R.I.
awarded under S. 376(2)(f), I.P.C. and one month R.I. under S. 342, I.P.C. are maintained.
Appeal dismissed.
AIR 2008 SUPREME COURT 577 "Dayanand V. V. S. Samiti v. Education Inspector,
Greater Bombay"
(From : Bombay)
Coram : 2 A. K. MATHUR AND MARKANDEY KATJU, JJ.
Civil Appeal No. 5979 of 2001, D/- 25 -10 -2007.
Dayanand Vedic Vidhalaya Sanchalak Samiti v. Education Inspector, Greater Bombay
and Anr.
Constitution of India, Art.226 - Bombay Primary Education Act (61 of 1947), S.63 -
Bombay Primary Education Rules (1949), R.2 - WRITS - EDUCATION - HIGH COURT
- Interim order - Powers of High Court - Order of Education Inspector - Rejecting
approval for appointments of teachers allegedly made in violation of Rules - Writ petition
against - Interim order by High Court directing grant of ad hoc approval to some teachers
- Illegal, as there is no provision in Rules for grant of ad hoc approval - Practice of
granting such interim orders which really amounts to grant of final relief - Deprecated.
(Paras 3, 4)

A. T. M. Rangaramanujam, Sr. Advocate, M. A. Chinnasamy, Vimal Wadhawani, K.


Krishna Kumar and Ms. Ruby Singh Ahuja, for Appellant; Chirag M. Shroff, Mukesh
Kumar, Ms. Mahima C. Shroff, S.S. Shinde and Ms. Asha Gopalan Nair, for
Respondents.
Judgement
JUDGMENT :-We have heard learned counsel for the parties.
2. This appeal by special leave is directed against the impugned interlocutory order dated
23rd November, 2000 in Writ Petition No. 2209 of 2000 passed by the High Court of
Bombay. The prayer in that writ petition was for quashing the order of the Education
Inspector, Greater Mumbai dated 30-9-2000. The order dated 30-9-2000 states that since
certain appointments of teachers were made by the Dayanand Vedic Vidyalaya, Mumbai
in violation of the Rules, approval for such appointments is rejected.
3. In order opinion in such a petition there was no scope for grant of any interim order,
and instead the writ petition should have been decided finally at an early date. However,
what the High Court has done by the impugned interim order is to direct grant of ad hoc
approval to some teachers.
4. This is a strange order passed by the High Court. There is no provision in the rules for
grant of ad hoc approval. Either approval is granted, or the prayer is rejected. There is no
scope for a half way measure like that adopted by the High Court. We cannot appreciate
such kind of interim orders which really amount to grant of final relief. This kind of
practice of granting such interim orders should be discontinued and should not be
encouraged.
5. However, in this appeal against the order of the High Court, this Court on 29th January,
2001 observed that "the operation of the order under challenge shall remain stayed unless
the order under challenge has already given effect to." The writ petition is still pending
before the High Court of Bombay. We request the High Court to dispose off the matter
finally, if not disposed off so far, preferably within a period of three months from the date
on which a copy of this order is received by it.
6. The interim order dated 29th January, 2001 passed by this Court is made absolute, but
shall only continue till the final disposal of the writ petition by the High Court.
7. The Civil Appeal is accordingly, disposed of.
Order accordingly.
@page-SC578
AIR 2008 SUPREME COURT 578 "R. Radhakrishnan v. Director General of Police"
(From : Madras)
Coram : 2 S. B. SINHA AND H. S. BEDI, JJ.
Civil Appeal Nos. 4874 with 4875 of 2007 (arising out of SLP (C) Nos. 17394 with
17395 of 2006), D/- 12 -10 -2007.
R. Radhakrishnan v. Director General of Police and Ors.
Constitution of India, Art.16 - EQUALITY IN PUBLIC EMPLOYMENT -
APPOINTMENT - OBSCENITY - Appointment - Suppression of material fact - Post of
Fireman - Applicant in his application not disclosing material fact as to his involvement
in criminal case that too a cognizable offence under S. 294(b) of IPC - Non-selection of
applicant - No interference.
(1996) 11 SCC 605, Foll.
1988 Supp SCC 795, Disting. (Para 13)
Cases Referred : Chronological Paras
(1996) 11 SCC 605 (Foll.) 11
1988 Supp SCC 795 (Disting) 12
V. Prabhakar, V. Subramani, Mrs. Ravathy Raghavan and Ramjee Prasad, for Appellant;
R. - Venkatramani, Sr. Advocate, V. G. Pragasam, S. Joseph Aristotle and S. Prabhu
Ramasubramanian, for Respondents.
Judgement
S. B. SINHA, J. - :-Leave granted.
2. Appellant, aggrieved by and dissatisfied with judgments and orders dated 21.01.2004
and 27.04.2006 passed by the High Court of Judicature at Madras in Writ Petition No.
13357 of 2002 and R.A. No. 68 of 2005 respectively, is before us.
3. Pursuant to or in furtherance of an advertisement dated 29.12.1999 having been issued
in that behalf, the appellant filed an application for appointment to the post of Fireman on
05.01.2000. He was provisionally selected whereafter he submitted a verification roll, the
relevant part whereof reads as under - :
"I realize that if I am enlisted and my statement which has been made by me is found to
be false, I shall render myself liable to be dismissed for obtaining service under false
pretences.
*** *** ***

15. Have you ever been concerned in any criminal case as accused? No
16. Have you ever been arrested or convicted and sentenced to undergo imprisonment
or pay a fine in any criminal or other offence - ? If so, give details with C.C. No. and
Court. No
*** *** ***
18. Are there any civil or criminal cases pending against you - ? If so, details. No

4. It now stands admitted that he, however, was involved in an incident which occurred
on 15.04.2000, and was proceeded against under Section 294(b) of the Indian Penal
Code.He was arrested but was released on bail.He, however, was acquitted of the said
charge on 25.09.2000.Inter alia on the premise that he had made false statement in his
verification roll, in regard to the pendency of the aforementioned case, he was not
selected.
5. He filed an original application before the Tamil Nadu Administrative Tribunal.The
learned Tribunal by reason of a judgment and order dated 4.03.2002 opined that as he had
been acquitted in the criminal case, there did not exist any reason as to why he should be
denied an appointment to the post of Fireman.A writ petition preferred thereagainst by the
respondent herein was allowed by reason of the impugned judgment.
6. Mr. V. Prabhakar, learned counsel appearing on behalf of the appellant, in support of
this appeal, raised a short question, viz., having regard to the fact that the appellant
signed the application prior to the date when the alleged accident took place and also
stood acquitted when he filled up the verification roll, he cannot be said to have wilfully
suppressed any material fact warranting denial from appointment in service.
7. The learned counsel would contend that in a case of this nature, the High Court ought
to have taken a sympathetic view and should not have allowed the writ petition of the
respondent only on the ground that he had suppressed the factum of his involvement in a
criminal case.
8. Mr. R. Venkatramani, learned senior counsel appearing on behalf of the respondent, on
the other hand, would submit that bona fide or otherwise on the part of the appellant
cannot be a criteria for determining the issue.The learned counsel
@page-SC579
submitted that had the relevant fact, viz., involvement in a criminal case and that too a
cognizable offence under Section 294(b) of the Indian Penal Code, been disclosed, the
appointing authority could have verified his character and suitability for appointment.It
was pointed out that the persons similarly situated against whom criminal cases had been
instituted had not been selected.
9. The learned counsel furthermore submitted that in view of the fact that the appellant
knew that he would be liable to be dismissed in service if the statement made in the
verification roll was found to be false cannot now be heard to say that he omitted to
mention the pendency of the criminal case under a bona fide belief or otherwise.
10. Indisputably, Appellant intended to obtain appointment in a uniformed service.The
standard expected of a person intended to serve in such a service is different from the one
of a person who intended to serve other services.Application for appointment and the
verification roll were both in Hindi as also in English. He, therefore, knew and
understood the implication of his statement or omission to disclose a vital
information.The fact that in the event such a disclosure had been made, the authority
could have verified his character as also suitability of the appointment is not in dispute.It
is also not in dispute that the persons who had not made such disclosures and were, thus,
similarly situated had not been appointed.
11. The question came up for consideration before this Court in Delhi Administration
through its Chief Secretary and Others v. Sushil Kumar [(1996) 11 SCC 605] wherein it
was categorically held :
"3...The Tribunal in the impugned order allowed the application on the ground that since
the respondent had been discharged and/or acquitted of the offence punishable under
Section 304 IPC, under Section 324 read with Section 34 IPC and under Section 324 IPC,
he cannot be denied the right of appointment to the post under the State. The question is
whether the view taken by the Tribunal is correct in law? It is seen that verification of the
character and antecedents is one of the important criteria to test whether the selected
candidate is suitable to a post under the State. Though he was found physically fit, passed
the written test and interview and was provisionally selected, on account of his
antecedent record, the appointing authority found it not desirable to appoint a person of
such record as a Constable to the disciplined force. The view taken by the appointing
authority in the background of the case cannot be said to be unwarranted...."
12. Mr. Prabhakar has relied upon a decision of this Court in T. S. Vasudavan Nair v.
Director of Vikram Sarabhai Space Centre and Others [1988 Supp SCC 795].The said
decision has been rendered, as would be evident from the judgment itself, on special facts
and circumstances of the said case and cannot be treated to be a binding precedent.
13. In the instant case, indisputably, the appellant had suppressed a material fact. In a
case of this nature, we are of the opinion that question of exercising an equitable
jurisdiction in his favour would not arise.
14. For the reasons aforementioned, there is no merit in these appeals which are
dismissed accordingly. However, in the facts and circumstances of this case, there shall
be no order as to costs.
Appeals dismissed.
AIR 2008 SUPREME COURT 579 "K. R. Mohan Reddy v. M/s. Net Work Inc. Rep. Tr.
M. D."
(From : AIR 2007 (NOC) 148 : 2007 (2) AIR Jhar R (NOC) 470 (AP))
Coram : 2 S. B. SINHA AND H. S. BEDI, JJ.
Civil Appeal No. 4588 of 2007 (arising out of SLP (C) No. 3540 of 2007), D/- 26 -9
-2007.
K. R. Mohan Reddy v. M/s. Net Work Inc. Rep. Tr. M. D.
Civil P.C. (5 of 1908), O.41, R.27(1)(aa), R.27(1)(b) - APPEAL - EVIDENCE -
Additional evidence - Adduction at appellate stage - Application made under O. 41, R.
27(1), Cl. - (aa) - Cannot be proceeded with as if it is one under Cl. (b) of O. 41, R. 27(1)
- Conditions precedent for application of Cl. (aa) and Cl. (b) are different.
Clauses (a), (aa) and (b) of sub-rule (1) of Rule 27 of Order 41 refer to three different
situations. Power of the appellate Court to pass any order thereunder is limited. For
exercising its jurisdiction thereunder, the appellate Court must arrive at a finding that one
or the other conditions enumerated thereunder is satisfied. A good reason must also be
shown as to why the evidence was not produced in the trial Court. The conditions
precedent for application of Clause (aa)
@page-SC580
of sub-rule (1) of Rule 27 of Order XLI is different from that of Clause (b). In the event
the former is to be applied, it would be for the applicant to show that the ingredients or
conditions precedent mentioned therein are satisfied. On the other hand Clause (b) to sub-
rule (1) of Rule 27 of Order XLI of CPC is to be taken recourse to, the appellate Court
was bound to consider the entire evidence on record and come to an independent finding
for arriving at a just decision; adduction of additional evidence as has been prayed by the
appellant was necessary.
(Paras 15, 18)
Cases Referred : Chronological Paras
2006 AIR SCW 2169 : AIR 2006 SC 1864 20
AIR 1965 SC 1008 20
Rakesh Dewedi, Sr. Advocate, Annam D. N. Rao and Ms. Fita Gupta, for Appellant; Udai
U. Lalit, Sr. Advocate, P. S. Narsimha, Somiran Sharma, L. Rashmani, Mrs. Mandakini
and Aribam Guneshwar Sharma, for Respondent.
Judgement
S. B. SINHA, J. - :-Leave granted.
1

. This appeal is directed against the judgment and order dated 20/9/2006 passed by a
Division Bench of the Andhra Pradesh High Court in C.C.C.A.No.253/2004 and the
application for adduction of additional evidence marked as City Civil Court Appeal Misc.
Petition No. 239 of 2006, whereby and whereunder the application filed by the
respondent herein, purported to be under Order 41 Rule 27 of the Code of Civil
Procedure was allowed. reported in AIR 2007 (NOC) 148 : 2007 (2) AIR Jhar R
(NOC) 470 (AP)

2. The parties entered into a partnership. The said partnership was reconstituted on
1.7.1994. Plaintiff -respondent contended that pursuant to the reconstituted partnership
deed, the appellant had handed over certain works to the respondent-firm for its
execution. It is also not in dispute that the appellant herein retired as a partner from the
said Firm. However, it has been contended that despite his retirement, the appellant had
requested the respondent to continue the work allotted in his name so as to protect his
turnover and continuation of his registration as a special class contractor. Further, the case
of the respondent was that the appellant was to hand over the payment which he is
supposed to receive in lieu of the Khammam Project. According to it upon settlement of
accounts of Khammam Project, an amount of Rs. 34,82,000/- was found due and payable
by the appellant to the respondent Firm.
3. The contention of the respondent, on the other hand, is that in respect of construction
of Minister's quarter at Hyderabad, the appellant had paid. Rs. 8,00,000/- and Rs.
5,25,316/-. It claimed that a sum of Rs. 8,03,350/- was owing to it by the appellant in
respect of the Vijayawada work.
4. According to the respondent, the appellant issued a cheque of Rs. 34,82,000/- in favour
of the Firm in respect of Khammam Project along with a covering letter wherein the
appellant assured the respondent that he would settle the accounts pertaining to the other
two projects after finalizing the accounts with the department. The said cheque was
dishonoured.
5. On the aforementioned premise, on or about 21.1.2002 a suit for recovery of Rs.
50,74,109/- along with the interest @ 24% per annum was filed by the respondent herein.
6. Appellant in his written statement, while denying and disputing the aforementioned
contentions of the plaintiff-respondent, inter alia, asserted that the cheque had been
obtained by it by fraud,forgery and with the connivance of one Shri K. Ramesh Reddy
and Mr. Y.S.Subramaniam, Managing Partner of the Respondent. It was also asserted by
the appellant that he had retired from the partnership on 12.1.2000 and all the accounts
between the parties were comprehensively settled which was also recorded in the deed of
retirement.
7. The trial Court framed a general issue as to whether the plaintiffs are entitled to any
relief and did not frame a specific issue with regard to the case of the appellant herein
that the said cheque was an outcome of fraud and forgery. The learned trial Judge,
however, dismissed the suit holding as under - :
"i) Defendant admits assignment of Khammam and Vijayawada Projects to the petitioner.
ii) Clause 8 of the retirement deed does not mention about pending work with the
respondent and future dues payable .
iii) Plaintiff did not file its accounts to prove that the plaintiff has executed the entire
work at Khammam Hospital.
@page-SC581
iv) Pending disposal of the suit filed by the defendant against the plaintiff for recovery of
the amounts paid to the plaintiff after retirement (7.6.2000 and 13.7.2000) on the ground
that the same was by way of loan, it is difficult to take a view that the defendant has paid
any money after his retirement towards the dues payable under the aforesaid transaction.
v) There is a possibility that the cheque was dishonestly obtained."
8. Respondent preferred an appeal thereagainst. Indisputably, an application under Order
XLI Rule 27 of Code of Civil Procedure was filed on 22.3.2006.
9. It, however, appears that no notice was issued in relation thereto. The said application
purported to have come up for consideration along with the hearing of the appeal.
10. With the consent of the parties, the main appeal itself was to be heard.
11. By reason of the impugned judgment, although, the High Court noticed the findings
of the learned trial Judge and various decisions operating in the field, inter alia, held that
the application for adduction of additional evidence filed by the respondent herein should
be allowed, the same being a requirement of Court and/or was otherwise for substantial
cause.
12. Mr.Rakesh Dwivedi, learned senior counsel appearing on behalf of the appellant, inter
alia, would submit that the High Court prior to passing of the said order did not give an
opportunity to the appellant to file an objection in regard to the maintainability of the said
application.
13. In any event, the learned counsel would contend that the respondent's application
being based on clause (aa) of sub-rule (1) of Rule 27 of order XLI of C.P.C., the High
Court committed a serious error in relying upon Clause (b) thereof.
14. Mr.Uday Umesh Lalit, learned senior counsel appearing on behalf of the respondent,
on the other hand, would submit that if the case of the respondent as set out in his plaint
vis-a-vis the findings of learned trial Judge are to be considered in their entirety, the High
Court was correct in its view.
15. The High Court, in our opinion, failed to apply the provisions of Order 41 Rule 27 of
CPC in its correct perspective. Clauses (a), (aa) and (b) of sub-rule (1) of Rule 27 of
Order XLI refer to three different situations. Power of the appellate court to pass any
order thereunder is limited. For exercising its jurisdiction thereunder, the appellate Court
must arrive at a finding that one or the other conditions enumerated thereunder is
satisfied. A good reason must also be shown as to why the evidence was not produced in
the trial Court.
16. Respondent in its application categorically stated that the books of accounts had been
misplaced and the same were discovered a few days prior to the filing of the said
application while the office was being shifted.
17. The High Court, unfortunately did not enter into the said questions at all . As
indicated hereinbefore, the High Court proceeded on the basis as if clause(b) of sub-rule
(1) of Rule 27 of Order XLI of CPC was applicable.
18. It is now a trite law that the conditions precedent for application of clause (aa) of sub-
rule (1) of Rule 27 of Order XLI is different from that of clause(b). In the event the
former is to be applied, it would be for the applicant to show that the ingredients or
conditions precedent mentioned therein are satisfied. On the other hand clause(b) to sub-
rule (1) of Rule 27 of Order XLI of CPC is to be taken recourse to, the appellate Court
was bound to consider the entire evidences on record and come to an independent finding
for arriving at a just decision; adduction of additional evidence as has been prayed by the
appellant was necessary.
19. The fact that the High Court failed to do so, in our opinion, amounts to misdirection
in law. Furthermore, if the High Court is correct in its view that the plaintiff-respondent
had proceeded on the basis that the suit in its entirely based on a cheque, wherefor, it was
not necessary for it to file the books of accounts before the trial Court, finding contrary
thereto could not have been arrived at that the same was in fact required to be proved so
as to enable the appellate Court to arrive at a just conclusion.
20

. The Supreme Court in State of Gujarat v. Mahendrakumar Parshottambhai Desai(dead)


by L.Rs-. (2006) 9 SCC 772 relying upon Municipal Corporation of Greater Bombay v.
Lal Pancham and Ors., 2006 AIR SCW 2169, (Para 10)
AIR 1965 SC 1008

@page-SC582
held as under - :
"Though the appellate Court has the power to allow a document to be produced and a
witness to be examined under Order XLI Rule 27 CPC, the requirement of the said Court
must be limited to those cases where it found it necessary to obtain such evidence for
enabling it to pronounce judgment. This provision did not entitle the appellate court to let
in fresh evidence at the appellate stage where even without such evidence it can
pronounce judgment in the case. It does not entitle the appellate court to let in fresh
evidence only for purposes of pronouncement of judgment in a particular way."
21. Appellate Court should not pass an order so as to patch up the weakness of the
evidence of the unsuccessful party before the trial Court, but it will be different if the
Court itself require the evidence to do justice between the parties. The ability to
pronounce judgment is to be understood as the ability to pronounce judgment
satisfactorily to the mind of the Court. But mere difficulty is not sufficient to issue such
direction. While saying so, however, we do not mean that the Court at an appropriate
stage would be precluded from considering the applicability of clause (b).
22. We are, therefore, of the opinion that the impugned judgment cannot be sustained. It
is set aside accordingly. The respondent may file additional affidavit in support of its
application under Order XLI Rule 27 CPC within two weeks from the date of receipt of
copy of this order. The appellant may file his response both to the Original Application as
also the additional affidavit, if any, within four weeks thereafter.
23. We would request the High Court to consider the entire matter in accordance with law
afresh on merits.
24. The appeal is disposed of with the aforementioned observations.
Order accordingly.
AIR 2008 SUPREME COURT 582 "State of M.P. v. Babulal"
(From : Madhya Pradesh)*
Coram : 2 C. K. THAKKER AND ALTAMAS KABIR, JJ.
Criminal Appeal No. 1658 of 2007 (arising out of S.L.P. (Cri.) No. 5974 of 2005), D/- 3
-12 -2007.
State of M.P. v. Babulal.
(A) Penal Code (45 of 1860), S.376 - RAPE - COMPLAINT - EVIDENCE - FALSE
IMPLICATION - Rape - Complaint filed by husband of victim - No unexplained delay -
Evidence of victim was "straightforward" and believable - Plea of false implication -
Ground of failure to return advance money taken from accused by husband of victim -
Found not tenable - Conviction of accused - Not illegal. (Paras 13, 17)
(B) Penal Code (45 of 1860), S.376 - RAPE - SENTENCE IMPOSITION - Rape -
Sentence - Imposition of - Undeserved indulgence or liberal attitude in not awarding
adequate sentence - Not proper.
The Courts are, expected to try and decide cases of sexual crime against women with
utmost sensitivity. Such cases need to be dealt with sternly and severely. A socially
sensitized Judge is a better armour in cases of crime against women than long clauses of
penal provisions, containing complex exceptions and complicated provisos. Once a
person is convicted for an offence of rape, he should be treated with a heavy hand. An
undeserved indulgence or liberal attitude in not awarding adequate sentence in such cases
would amount to allowing or even to encouraging 'potential criminals.' The society can
no longer endure under such serious threats. Courts must hear the loud cry for justice by
society in cases of heinous crime of rape and impose adequate sentence. Public
abhorrence of the crime needs reflection through imposition of appropriate sentence by
the Court.
2006 AIR SCW 1123, Relied on.
(Paras 25, 26)
(C) Penal Code (45 of 1860), S.376 - RAPE - SENTENCE IMPOSITION - Rape -
Imposition of sentence less than minimum - Recording of "adequate" and "special"
reasons is sine qua non - Reasons that accused was "illiterate agriculturist" from rural
area" and "amount of fine of Rs. 2500/- was imposed on him" - Can neither be said to be
'special' nor 'adequate' - Thus, order reducing sentence imposed on accused to "period
already undergone" which was only two months and three days and much less than
minimum required to be imposed - Liable to be set aside.
Criminal Appeal No. 298 of 2003, D/- 21-3-2003 (MP), Reversed. (Paras 30, 31, 32)
@page-SC583
Cases Referred : Chronological Paras
2006 AIR SCW 1123 : AIR 2006 SC 1267 : 2006 Cri LJ 1679 (Rel. on) 26
1992 AIR SCW 1480 (Ref.) 2
1992 AIR SCW 2365 : AIR 1992 SC 2004 : 1992 Cri LJ 3655 (Ref.) 14
1983 SC 753 : 1983 Cri LJ 1096 (Ref.) 14
AIR 1973 SC 1457 : 1974 Cri LJ 243 (Ref.) 22
D. K. Singh, C. D. Singh, Merusagar Samantary and Vairagya Vardhan, for Appellant;
Anish Kumar Gupta, Umesh Babu Chaurasia, Ms. Deepshikha Bharati and Mrs. Rita
Gupta, for Respondent.
* Cri. A. No. 298 of 2003, D/- 21-8-2003 (MP).
Judgement
1. C. K. THAKKER, J. :-Leave granted.
2

. The present appeal reminds us observations of Hon'ble Mr. Justice S. Ratnavel Pandian
in Madan Gopal Kakkad v. Naval Dubey and Anr., (1992) 3 SCC 204 that "offenders of
sexual assault who are menace to the civilized society should be mercilessly and
inexorably punished in the severest terms". Dealing with a case of sexual assault, His
Lordship emphasized on Courts of Law their duty to handle offenders of such crimes
with a heavy hand. His Lordship concluded: 1992 AIR SCW 1480

"We feel that Judges who bear the Sword of Justice should not hesitate to use that sword
with the utmost severity, to the full and to the end if the gravity of the offences so
demand".
3. The case on hand, in our considered view, exhibits not only casual, indifferent and
perfunctory approach but insensitive attitude adopted by the High Court in awarding
sentence on an offender who perpetrated a heinous crime of committing rape on a
married woman in broad daylight. The case of the prosecution was that respondent
Babulal was residing at village Daulatpur, Tehsil Ikchavaar, District Sehore in Madhya
Pradesh. On July 23, 2002, at about 12.00 noon in his own tapri, he criminally
intimidated the prosecutrix-PW5, aged about 22 years, a married lady (hereinafter
referred to as 'PW5-X') and committed rape on her. According to the prosecution, PW5-X
was living with her husband in the house of the accused. On the day of the incident, she
was washing a drum on tapri when the accused caught her from behind and threw her on
the ground. The prosecutrix-PW5 shouted and resisted, but the accused threatened her
with knife and committed rape on her. Even thereafter, he threatened to kill her if she
reported the incident to anyone else. In the evening, PW5-X told about sexual assault to
her husband and her mother-in-law Dallubai, a blind lady. PW8-Ramcharan, who was the
employer of PW7-Shiv Narayan-husband of PW5 was also informed who assured that he
would talk to the accused and PW5 should not leave the place due to fear. On the next
day, i.e. July 24, 2002, when the elder brother of Shiv Narayan arrived, the prosecutrix
(PW5-X) and her husband (PW7) went to the police station, Ikchavaar and lodged a
complaint. PW5-X was then sent for medical examination, site plan was prepared and
statements of witnesses were recorded. PW5 was medically examined. The accused was
also sent for medical examination. It was found that he was absolutely competent to
commit sexual intercourse. After completion of usual investigation, charge-sheet was
submitted for offences punishable under Section 376 read with Section 506, Part II,
Indian Penal Code (IPC). The accused denied the charge. In his statement under Section
313 of the Code of Criminal Procedure, 1973, he contended that in order to avoid
repayment of loan taken from Ramcharan-PW8, the prosecutrix (PW5-X) had falsely
implicated him in the case.
4. The trial Court considered the evidence adduced by the prosecution and particularly
sworn testimony of PW5-prosecutrix, PW7-Shiv Narayan-husband of prosecutrix and
PW9-Dr. Madhu Sharma, immediate Assistant Surgeon, Public Health Centre, Ikchavaar
and held that it was proved beyond reasonable doubt that the accused had committed the
offence of rape. So far as PW8-Ramcharan is concerned, he did not support the
prosecution and was declared 'hostile'. The trial Court, however, acquitted the accused of
the charge under Section 506, II IPC.
5. On sentence, the trial Court heard the accused who prayed for grant of probation
which, in our opinion, was rightly refused by the Court. In the light of mandate in sub-
section (1) of Section 376, IPC, the trial Court imposed minimum sentence of seven
years' rigorous imprisonment and to pay fine of Rs.2,500/- (two thousand five hundred).
In default of payment of fine, the accused was ordered to undergo rigorous imprisonment
for six months more. The amount of fine was ordered to be paid to the prosecutrix X.
6. The aggrieved accused preferred an appeal before the High Court of Madhya
@page-SC584
Pradesh. The learned counsel for the accused did not challenge the finding of conviction
but prayed for mercy and leniency in sentence. The learned Judge of the High Court
upheld the argument of the learned counsel for the appellant and observed that the
accused was initially in custody from September 11, 2002 to October 10, 2002 and again
after the pronouncement of the judgment, he was sent to jail on January 23, 2003 till he
was enlarged on bail on February 26, 2003. The learned Judge also observed that the
accused was an 'illiterate agriculturist from rural area' and fine of Rs.2,500/- was also
imposed on him. According to the learned Judge, on the facts of the case, the
imprisonment for two months and three days which had already undergone by the
accused could be said to be 'just and proper' and accordingly the appeal was partly
allowed.
7. Aggrieved by the said order passed by the High Court, the State has approached this
Court.
8. On November 21, 2005, notice as also bailable warrant was issued against the
respondent which was duly served upon him. The respondent also appeared through an
advocate. On March 19, 2007 when the matter was called out, the advocate appearing for
the respondent-accused stated that he had no papers. The Court, therefore, ordered that
papers be given to the learned counsel appearing for the respondent by the counsel for the
State. The matter was then called out for final hearing.
9. We have heard learned counsel appearing for the parties.
10. The learned counsel for the State contended that the High Court had committed a
serious error of law in reducing the sentence imposed by the trial Court. He submitted
that sub-section (1) of Section 376, IPC provides minimum sentence of rigorous
imprisonment for seven years which was imposed by the trial Court and there was no
reason for the High Court to interfere with the said order. Maximum imprisonment
imposable on the offender under the said provision is ten years. The High Court was,
therefore, not right in reducing the sentence and that too when the accused had undergone
only for two months and three days. It was also submitted that no 'adequate and special
reasons' were recorded by the High Court for reducing the sentence and even on that
ground also the order is vulnerable. The counsel submitted that the High Court ought to
have appreciated the fact that the offence was committed in broad daylight. He, therefore,
submitted that the order passed by the High Court deserves to be set aside by restoring
the order of the trial Court.
11. The learned counsel for the respondent-accused submitted that the discretion
exercised by the High Court considering the position of the accused, cannot be said to be
illegal and deserves no interference.
12. Having heard the learned counsel for the parties, in our opinion, the High Court had
manifestly erred in allowing the appeal and in reducing the sentence imposed on the
offender to the period 'already undergone'.
13. So far as conviction of the respondent is concerned, we find no infirmity in the
reasons recorded and the conclusion arrived at by the trial Court. The trial Court rightly
held that on the fateful day, at 12.00 noon, the accused committed the crime. In her
testimony on oath, prosecutrix narrated the incident and stated that when she was
washing the kothi on tapri, the accused came from the behind, caught her, pulled her
down on the earth and committed rape on her. The trial Court rightly observed that the
prosecutrix informed her husband about the incident, who in turn contacted PW8-
Ramcharan-employer, but Ramcharan-PW8 did nothing. The matter was also reported by
prosecutrix to her mother-in-law Dallubai who was blind. PW7-Shiv Narayan-husband of
the prosecutrix intimated his elder brother about the incident when he came next day and
thereafter First Information Report (FIR) was lodged. The trial Court rightly held that
there was no unexplained delay in filing the complaint. The 'straightforward' evidence of
prosecutrix-PW5 was believed by the Court and accordingly the accused was convicted.
We are fully satisfied that in recording a finding of guilt against the respondent, the trial
Court had not committed any error, either of fact or of law.
14

. As held by this Court in several cases, if a Court of law finds evidence of prosecutrix
truthful, trustworthy and reliable, conviction can be recorded solely on the basis of her
testimony and no further corro-boration is necessary. In this connection, we may refer to
only two leading decisions of this Court in Bharwada Bhoginbhai Hirjibhai v. State of
Gujarat, (1983) 3 SCC 217 and AIR 1983 SC 753
1992 AIR SCW 2365

@page-SC585
State of Rajasthan v. Narayan, (1992) 3 SCC 615.
15. In the first case, this Court, speaking through M.P. Thakkar, J. stated:
"9. In the Indian setting, refusal to act on the testimony of a victim of sexual assault in the
absence of corroboration as a rule, is adding insult to injury. Why should the evidence of
the girl or the woman who complains of rape or sexual molestation be viewed with the
aid of spectacles fitted with lenses tinged with doubt, disbelief or suspicion? To do so is
to justify the charge of male chauvinism in a male dominated society. We must analyze
the argument in support of the need for corroboration and subject it to relentless and
remorseless cross-examination. And we must do so with a logical, and not an
opinionated, eye in the light of probabilities with our feet firmly planted on the soil of
India and with our eyes focused on the Indian horizon. We must not be swept off the feet
by the approach made in the western world which has its own social milieu, its own
social mores, its own permissive values, and its own code of life. Corroboration may be
considered essential to establish a sexual offence in the backdrop of the social ecology of
the western world. It is wholly unnecessary to import the said concept on a turnkey basis
and to transplant it on the Indian soil regardless of the altogether different atmosphere,
attitudes, mores, responses of the Indian society, and its profile. The identities of the two
worlds are different. The solution of problems cannot therefore be identical. It is
conceivable in the western society that a female may level false accusation as regards
sexual molestation against a male for several reasons such as :
(1) The female may be a 'gold digger' and may well have an economic motive to extract
money by holding out the gun of prosecution or public exposure.
(2) She may be suffering from psychological neurosis and may seek an escape from the
neurotic prison by phantasizing or imagining a situation where she is desired, wanted,
and chased by males.
(3) She may want to wreak vengeance on the male for real or imaginary wrongs. She may
have a grudge against a particular male, or males in general, and may have the design to
square the account.
(4) She may have been induced to do so in consideration of economic rewards, by a
person interested in placing the accused in a compromising or embarrassing position, on
account of personal or political vendetta.
(5) She may do so to gain notoriety or publicity or to appease her own ego or to satisfy
her feeling of self-importance in the context of her inferiority complex.
(6) She may do so on account of jealousy.
(7) She may do so to win sympathy of others.
(8) She may do so upon being repulsed".
16. In the second case, which was also of rape, there was delay of three days in lodging
FIR. This Court held that it was not a factor causing doubt on the story of the prosecution
in view of the generally known fact that the rape victim or her husband would hesitate to
approach the police. It was also held that unless the evidence discloses that she and her
husband had strong reasons to falsely implicate the accused, ordinarily the Court should
have no hesitation in accepting her version regarding the incident.
17. In the case on hand, the defence put forward by the respondent-accused was that the
husband of the prosecutrix had taken advance money from PW8-Ramcharan-employer
towards labour charges and since he had no intention to return the said amount, the
prosecutrix falsely implicated the accused in the case. In our considered opinion, the trial
Court rightly rejected the defence. Hence, in our opinion, the order of conviction
recorded by the trial Court and confirmed by the High Court cannot be said to be faulty
and conviction of the respondent-accused cannot be said to be illegal.
18. The next question relates to adequacy of sentence. Let us consider it on principle as
well as in practice, in the light of statutory provisions.
19. Punishment is the sanction imposed on the offender for the infringement of law
committed by him. Once a person is tried for commission of an offence and found guilty
by a competent Court, it is the duty of the Court to impose on him such sentence as is
prescribed by law. The award of sentence is consequential on and incidental to
conviction. The law does not envisage a person being convicted for an offence without a
sentence being imposed therefor.
20. The object of punishment has been succinctly stated in Halsbury's Laws of
@page-SC586
England, (4th Edition; Vol.II; para 482) thus;
"The aims of punishment are now considered to be retribution, justice, deterrence,
reformation and protection and modern sentencing policy reflects a combination of
several or all of these aims. The retributive element is intended to show public revulsion
to the offence and to punish the offender for his wrong conduct. The concept of justice as
an aim of punishment means both that the punishment should fit the offence and also that
like offences should receive similar punishments. An increasingly important aspect of
punishment is deterrence and sentences are aimed at deterring not only the actual
offender from further offences but also potential offenders from breaking the law. The
importance of reformation of the offender is shown by the growing emphasis laid upon it
by much modern legislation, but judicial opinion towards this particular aim is varied and
rehabilitation will not usually be accorded precedence over deterrence. The main aim of
punishment in judicial thought, however, is still the protection of society and the other
objects frequently receive only secondary consideration when sentences are being
decided".
(Emphasis supplied)
21. In justice-delivery system, sentencing is indeed a difficult and complex question.
Every Court must be conscious and mindful of proportion between an offence committed
and penalty imposed as also its impact on society in general and the victim of the crime
in particular.
22

. In B. G. Goswami v. Delhi Administration, (1974) 3 SCC 85, this Court stated: AIR
1973 SC 1457

"Now the question of sentence is always a difficult question, requiring as it does, proper
adjustment and balancing of various considerations which weigh with a judicial mind in
determining its appropriate quantum in a given case. The main purpose of the sentence
broadly stated is that the accused must realise that he has committed an act which is not
only harmful to the society of which he forms an integral part but is also harmful to his
own future, both as an individual and as a member of the society. Punishment is designed
to protect society by deterring potential offenders as also by preventing the guilty party
from repeating the offence; it is also designed to reform the offender and re-claim him as
a law abiding citizen for the good of the society as a whole. Reformatory, deterrent and
punitive aspects of punishment thus play their due part in judicial thinking while
determining this question. In modern civilized societies, however, reformatory aspect is
being given somewhat greater importance. Too lenient as well as too harsh sentences both
lose their efficaciousness. One does not deter and the other may frustrate thereby making
the offender a hardened criminal".
(Emphasis supplied)
[see also Salmond on Jurisprudence, (2004); p.94]
23. Penal laws, by and large, adhere to the doctrine of proportionality in prescribing
sentences according to culpability of criminal conduct. Judges in principle agree that
sentence ought always to commensurate with the crime. In practice, however, sentences
are determined on other relevant and germane considerations. Sometimes it is the
correctional need that justifies lesser sentence. Sometimes the circumstances under which
the offence is committed play an important role. Sometimes it is the degree of
deliberation shown by the offender in committing a crime which is material. Sentencing
is thus a delicate task which requires skill, talent and consideration of several factors,
such as, the nature of offence, circumstances extenuating or aggravating - in which it was
committed, prior criminal record of the offender, if any, age and background of the
criminal with reference to education, home life, social adjustment, emotional and mental
condition, prospects of his reformation and rehabilitation, etc. All these and similar other
considerations can, hopefully and legitimately, tilt the scale on the propriety of sentence.
24. Moreover, social impact of the crime, particularly where it relates to offences against
women, cannot be lost sight of and per se require exemplary treatment. Any liberal
attitude of imposition of meager sentence or too sympathetic view may be counter
productive in the long run and against social interest which needs to be cared for,
protected and strengthened by string of deterrence inbuilt in the sentencing system.
25. Sexual violence apart from being a dehumanizing act is also an unlawful intrusion of
the right to privacy and sanctity of a female. It is a serious blow to her supreme honour
and offends her self-esteem and dignity. It degrades and humiliates the victim
@page-SC587
and leaves behind a traumatic experience. It has been rightly said that whereas a murderer
destroys the physical frame of a victim, a rapist degrades and defiles the soul of a
helpless female. The Courts are, therefore, expected to try and decide cases of sexual
crime against women with utmost sensitivity. Such cases need to be dealt with sternly and
severely. A socially sensitized Judge is a better armour in cases of crime against women
than long clauses of penal provisions, containing complex exceptions and complicated
provisos.
26

. Once a person is convicted for an offence of rape, he should be treated with a heavy
hand. An undeserved indulgence or liberal attitude in not awarding adequate sentence in
such cases would amount to allowing or even to encouraging 'potential criminals'. The
society can no longer endure under such serious threats. Courts must hear the loud cry for
justice by society in cases of heinous crime of rape and impose adequate sentence. Public
abhorrence of the crime needs reflection through imposition of appropriate sentence by
the Court [Dinesh v. State of Rajasthan, (2006) 3 SCC 771]. 2006 AIR SCW 1123
27. Now, let us consider the legal position in the light of statutory provisions and
amendments made. The Law Commission took note of various decisions rendered by this
Court from time to time wherein it was observed that considering the rise in crime and
the growing menace to sexual abuse, necessary change should be made. The Law
Commission, therefore, in its 84th Report stated:
"It is often stated that a woman who is raped undergoes two crises - the rape and the
subsequent trial. While the first seriously wounds her dignity, curbs her individual,
destroys her sense of security and may often ruin her physically, the second is no less
potent of mischief, inasmuch as it not only forces her to relive through the traumatic
experience, but also does so in the glare of publicity in a totally alien atmosphere, with
the whole apparatus and paraphernalia of the criminal justice system focused upon her.
In particular, it is now well established that sexual activities with young girls of immature
age have a traumatic effect which often persists through life, leading subsequently to
disorders, unless there are counter-balancing factors in family life and in social attitudes
which could act as a cushion against such traumatic effects.
Rape is the 'ultimate violation of the self'. It is a humiliating event in a woman's life
which reads to fear for existence and a sense of powerlessness. The victim needs empathy
and safety and a sense of re-assurance. In the absence of public sensitivity to these needs,
the experience of figuring in a report of the offence may itself become another assault.
Forcible rape is unique among crimes, in the manner in which its victims are dealt with
by the criminal justice system. Raped women have to undergo certain tribulations. These
begin with their treatment by the police and continue through a male-dominated criminal
justice system. Acquittal of many de facto guilty rapists adds to the sense of injustice.
In effect, the focus of the law upon corroboration, consent and character of the
prosecutrix and a standard of proof of guilt going beyond reasonable doubt have resulted
in an increasing alienation of the general public from the legal system, who find the law
and legal language difficult to understand and who think that the Courts are not run so
well as one would expect.
28. Pursuant to the Law Commission's Report, Parliament amended Sections 375 and
376, IPC by the Criminal Law (Amendment) Act, 1983. (Act 43 of 1983). Sub-section (1)
of Section 376 now prescribes minimum sentence of rigorous imprisonment of seven
years on the person convicted under Section 376(1) unless the case is covered by proviso.
Sub-section (1) read with proviso is material which reads thus:
376. Punishment for rape
(1) Whoever, except in the cases provided for by sub-section (2), commits rape shall be
punished with imprisonment of either description for a term which shall not be less than
seven years but which may be for life or for a term which may extend to ten years and
shall also be liable to fine unless the woman raped is his own wife and is not under
twelve years of age, in which cases, he shall be punished with imprisonment of either
description for a term which may extend to two years or with fine or with both:
Provided that the Court may, for adequate and special reasons to be mentioned in the
judgment, impose a sentence of imprisonment for a term of less than seven years.
(Emphasis supplied)
@page-SC588
29. The proviso to sub-section (1) of Section 376, IPC thus enjoins the Court if it imposes
less than the minimum sentence of seven years rigorous imprisonment on an offender of
rape to record 'adequate and special reasons' in the judgment. Recording of reasons is,
therefore, sine qua non or condition precedent for imposing sentence less than the
minimum required by law. Moreover, such reasons must be both (i) 'adequate' and (ii)
'special'. What is 'adequate' and 'special' would depend upon several factors and no strait-
jacket formula can be laid down as a rule of law of universal application.
30. In the instant case, 'special' and 'adequate' reasons according to the learned Judge of
the High Court were; (i) the respondent was an 'illiterate agriculturist from rural area' and
(ii) an amount of fine of Rs.2,500/- was imposed on him. No other reason whatsoever has
been mentioned in the judgment, nor is found from the record of the case. With respect to
the learned Judge, in our considered opinion, the so called reasons can neither be said to
be 'special' nor 'adequate'. On the contrary, in the Special Leave Petition seeking leave to
appeal, the applicant-State has averred that the learned Judge was in the habit of passing
such orders by reducing sentence to the period 'already undergone' in serious offences
punishable under Sections 304, 307, 376, etc. A list is also given of some of the matters
decided by him. Our attention was also invited by the learned Government Advocate that
in several cases, this Court has set aside the decisions rendered by the same learned
Judge.
31. In our judgment, by passing the order impugned in the present appeal and by reducing
the sentence imposed on the respondent by the trial Court to the 'period already
undergone' which was only two months and three days, the learned Judge of the High
Court has committed grave illegality which had resulted in 'miscarriage of justice'. There
were no reasons much less 'adequate' and 'special' reasons to reduce the sentence less than
the minimum required to be imposed under sub-section (1) of Section 376, IPC. The
order is, therefore, liable to be set aside. On the facts and in the circumstances of the case,
in our opinion, the trial Court was wholly right and fully justified in awarding rigorous
imprisonment for seven years as envisaged by sub-section (1) of Section 376, IPC and
there was no earthly reason to interfere with the said order by the High Court. The appeal,
therefore, deserves to be allowed.
32. For the foregoing reasons, the appeal filed by the State is allowed. The order of
conviction recorded by the trial Court and confirmed by the High Court is upheld. The
High Court was, however, wrong in reducing the sentence and the trial Court rightly
imposed rigorous imprisonment of seven years on the respondent-accused. We, therefore,
restore that part of the order of the trial Court directing the respondent to suffer rigorous
imprisonment for seven years. It goes without saying that the period of sentence already
undergone by the respondent-accused will be given set off.
33. Ordered accordingly.
Appeal allowed.
AIR 2008 SUPREME COURT 588 "Dadan Ram v. State of Bihar"
(From : Patna)
Coram : 2 TARUN CHATTERJEE AND P. SATHASIVAM, JJ.
Civil Appeal No. 26 of 2001, D/- 23 -11 -2007.
Dadan Ram and Ors. v. State of Bihar and Ors.
Bihar Land Reforms (Fixation of Ceiling Area and Acquisition of Surplus Land) Act (12
of 1962), S.45B - LAND REFORMS - LAND CEILING - NATURAL JUSTICE -
Reopening of proceedings - Notice to all parties including person(s) in possession of land
in question - Mandatory - Fact that person(s) in possession had knowledge of order of
status quo passed by District Collector - Would not be substantial compliance of rules of
natural justice.
(2003) 10 SCC 239, Rel. on. (Paras 11, 12, 13)
Cases Referred : Chronological Paras
(2003) 10 SCC 239 (Rel. on) 11
S. B. Sanyal, Sr. Advocate, Ranjan Mukherjee, for Appellants; P. S. Mishra, Sr. Advocate,
Dhruv Kumar Jha, Upendra Mishra, Ravi Chandra Prakash, Himanshu Shekhar, Gopal
Singh, Anukul Raj, Manish Kumar, for Respondents.
Judgement
P. SATHASIVAM, J. :-Whether the parcha holders, who are in possession of the land in
question, have any right to be heard in a proceeding arising out of Section 45-B of the
Bihar Land Reforms (Fixation of
@page-SC589
Ceiling Area and Acquisition of Surplus Land) Act, 1961 (hereinafter referred to as the
Act) is the only question to be decided in this appeal.
2. This appeal is directed against the impugned final judgment and order dated 14.9.1999
passed by the Division Bench of the High Court of Judicature at Patna in L.P.A. No. 1545
of 1997 whereby the High Court dismissed the L.P.A. filed by the appellants herein
against the judgment and order dated 24.11.1997 passed by the learned single Judge of
the High Court in C.W.J.C. No. 12036 of 1996 arising from a proceeding under Section
45-B of the Act.
3. The Ceiling (surplus) proceedings bearing Ceiling Case No. 149 of 1973-74 were
initiated against Nand Kishore Tiwari, respondent No.8 herein and a notice to the said
effect was issued to him under Section 6(1) of the Act in Form LC-1 as prescribed under
Rule 5 under the said Act and respondent No.8 submitted a return under the said Act in
respect of his entire land, total 19 acres 71 decimals. The aforesaid land was found to be
owned and possessed by the family as defined in Section 2(ee) of the said Act i.e. land
holder, respondent No.8, his wife Sumitra Devi and their two minor children as on
appointed day i.e., 9.9.1970 and ultimately the authority concerned by holding 15 acres of
class I lands was permitted to be retained by respondent No.8 under Section 5(1) and the
remaining 4.64 acres of land were declared as surplus. Final publication of draft
statement under Section 11(1) of the said Act was accordingly made and subsequent to
that a notification under Section 15(1) was also issued on 15.1.1993. Against the said
notification, respondent No.8 filed an appeal under Section 30(1)(b) of the Act before the
Commissioner, Patna Division, Patna bearing Ceiling Appeal No. 160 of 1994. The
Commissioner dismissed the appeal on merits. Dissatisfied therewith, a Ceiling Revision
was filed before the Member, Board of Revenue, Bihar, Patna which was also dismissed.
Writ Petition filed for quashing the earlier orders was dismissed by the High Court.
Subsequently another writ application which was filed in the High Court by respondent
No.8 herein and the same was disposed of with a direction to raise the matter before the
Collector of the District within two weeks from the date of the receipt of the order.
However, no such application was filed by respondent No.8 before the District Collector,
therefore, after the expiry of said such period, the High Courts order became infructuous.
The wife of respondent No.8, namely, Sumitra Devi filed an application before the
District Collector for re-opening the case under Section 95-B of the Ceiling Act and the
same was dismissed. Challenging the order of the District Collector as well as the orders
of the appellate Court which was passed in the appeal filed by respondent No.8 and the
order passed in Revision application, an application under Section 32 of the Act was also
filed before the Member, Board of Revenue, Bihar and the same was finally disposed of
on 12.7.1995 with a direction that the Collector shall ascertain the allegation. After final
publication under Section 15(1) of the Ceiling Act, the aforesaid excess land i.e. 4 acres
64 decimals was distributed to 8 down-trodden people of the village and separate parchas
were issued in name of the aforesaid eight persons and the possession was also delivered
to them. The District Collector transferred the case to the Court of Additional Collector
who re-opened the case, conducted the impugned proceedings in question and held that
the land holder has no excess land. Accordingly, he set aside the notification issued under
Section 15(1) of the Ceiling Act. Respondent No.5 before disposal of the application
under Section 45-B of the Ceiling Act, did not issue notice nor opportunity was given to
the appellants with whom the aforesaid lands were in possession.
4. Being aggrieved, the appellants filed C.W.J.C. No. 12036 of 1996 before the High
Court. Learned single Judge of the High Court dismissed the same. Dissatisfied
therewith, L.P.A. was filed before the Division Bench of the High Court of Patna. The
Division Bench of the High Court dismissed the same affirming the order passed by the
learned single Judge. Aggrieved by the said order, the appellants filed the present appeal
by way of special leave petition
5. We heard Mr. S.B. Sanyal, learned senior counsel for the appellants and Mr. P.S.
Mishra, learned senior counsel for the contesting private respondent Nos. 8 and 9 and Mr.
Gopal Singh, learned counsel for the State of Bihar.
6. After taking us through the entire proceedings including the orders passed by the
authorities under the Act as well as the High Court, Mr. S.B. Sanyal, learned senior
counsel, for the appellants mainly submitted that
@page-SC590
inasmuch as the appellants-parcha holders who were in lawful possession of the land in
question and continuing the same even today are entitled to notice and opportunity of
being heard in a proceeding arising out of Section 45 of the Act. He also contended that
in view of abuse of process by the contesting private respondent Nos. 8 and 9 who are
none else than husband and wife, all the orders are liable to be quashed. On the other
hand, Mr. P.S.Mishra, learned senior counsel for respondent Nos. 8 and 9 submitted that
in view of order of status quo which was passed in the presence of both parties even in
the absence of the separate notice in a proceeding under Section 45-B of the Act in the
facts and circumstances, the rules of natural justice were substantially complied with and
hence there is no ground for interference by this Court under Article 146 of the
Constitution of India.
7. We have carefully perused the annexures and relevant materials and considered the
rival contentions with reference to the pleadings.
8. If we accept the first contention of the learned senior counsel for the appellants,
namely, notice or opportunity of being heard in a proceeding under Section 45-B of the
Act, there is no need to consider the other contentions. It is seen from the materials
placed, after proper notice, the land holder and respondent No.8, and his wife - Sumitra
Devi and their two minor children on the appointed day, i.e., 09.09.1970 were permitted
to retain 15 acres of class-I lands and the remaining 4.64 acres of land were declared as
surplus. Based on the same, a notification under Section 15(1) was issued on 15.01.1993.
The appeal as well as the revision filed against the same was dismissed. The writ petition
filed by the 8th respondent was also dismissed. When special leave petition was filed
against the order of the High Court, the same was disposed of with a permission to the
applicant to raise the matter before the Collector of the concerned District. It is brought to
our notice that though no such petition was filed by the 8th respondent, subsequently his
wife - Sumitra Devi - 9th respondent herein has filed an application before the District
Collector, Bhojpur 4th respondent-herein for re-opening the case under Section 45-B of
the Act. The said application was dismissed by the District Collector and subsequent to
that an application under Section 32 of the said Act was filed before the Member, Board
of Revenue, Bihar challenging the entire order of the District Collector. The same was
finally disposed of on 12.07.1995 with a direction to the Collector to consider the
grievance of Sumitra Devi. It is further seen that the District Collector, Bhojpur
transferred the case to the Court of Addl. District Collector who re-opened the case and
found that the land holder has no excess land and notification issued under Section 15(1)
of the Act was to be set aside. Questioning the said proceedings, the appellants filed
CWJC No. 12036 of 1996 before the High Court, Patna. Learned Single Judge, who
heard the matter, dismissed the same by order dated 24.11.1997. Dissatisfied with the said
order, an appeal was filed before the Division Bench in L.P.A. No. 1545 of 1997 which
was also dismissed affirming the order dated 24.11.1997 passed by the learned Single
Judge.
9. At this juncture, it is relevant to mention that based on the earlier proceedings holding
that the 8th respondent herein was having excess land of 4.64 acres, the authority
concerned, after following the procedure, and after proper verification assigned the
excess lands in favour of the appellants. According to them, from that date onwards, they
are in possession of the assigned lands and they are the parcha-holders.
10. Since initially at the instance of 8th respondent and thereafter his wife - Sumitra Devi
9th respondent-herein, the case was re-opened and found no excess land available, it is
useful to refer the relevant provision, i.e., Section 45-B of the Act. The said provision was
inserted by Bihar Act 22 of 1976. The Section reads as under:-
"45-B. State Government to call for and examine records.- The State Government *[or
the Collector of the district who may be authorized in this behalf] may, at any time, call
for and examine any record of any proceeding disposed of by a Collector under the Act
and may, if it thinks fit, direct that the case be reopened and disposed of afresh in
accordance with the provisions of the Act."
* Deleted by Act 8 of 1997
11. It is not in dispute that prior to the aforesaid amendment, there was no such power
enabling the Collector, Member, Board of Revenue or State Government to re-open the
case for fresh disposal which had been
@page-SC591
concluded. By the aforesaid Section 45-B, power has been vested in the State
Government or in the Collector of the District (since deleted by Act 8 of 1997) re-
opening of cases which had been disposed of so that they may be heard afresh in
accordance with the provisions of the Act. Though the amended provision contains very
wide and extra-ordinary power, admittedly no guidelines have been provided as to when
such power is to be exercised. In fact, no period of limitation has been fixed, the result
whereof may be that a proceeding which had been initiated under the provisions of the
Act and has been concluded by final orders passed by the original, appellate and the
revisional authority can be re-opened after lapse of several years. The amended provision
also makes it clear that while exercising powers under the said provision, no one can act
as an appellate or revisional Court. It is an extra-ordinary power which can be invoked
only if earlier order is found to have been passed not in accordance with the Act. The
proceedings under the amended section are quasi judicial, the right to get opportunity of
hearing cannot be denied in such proceedings. Under this section initially both the State
Government/Collector has the jurisdiction, but by the amendment Act 8 of 1997, State
alone is empowered to re-open such matters for valid reasons. The proceedings are quasi
judicial in nature. Considering the fact that the State Government and previously the
District Collector were authorized to re-open the issue which was concluded, we are of
the view that prior to re-opening, issue of notice and opportunity of hearing of the land
holder or person in possession of the land are mandatory. In the instant case, from the
materials it is clear that the appellants as parcha holders, though the issue was re-opened
they were not issued notice or given an opportunity to put-forth their case. Though the
High Court has concluded that in view of the order of status quo which was passed in the
presence of both parties including the present appellants, the rules of natural justice were
substantially complied with in view of the power conferred on the State Government to
re-open a case that too even after final notification, the person/persons who are in
possession of the land in question or parcha holders are entitled opportunity of notice and
they must be heard before final decision being taken. In this regard, it is useful to refer to
the decision of this Court in Baban Paswan and Another vs. Pratima Devi and Others,
(2003) 10 SCC 239. The case relates to determination of the ceiling area in respect of the
family of Prabal Pratap Singh and Dinesh Prasad Singh and it was then worked out that
43.26 acres was excess land. The respondent 1 - Pratima Devi being the sister of the
aforesaid two persons raised some dispute stating that she was not heard in the matter. In
the meanwhile the surplus land was distributed to different persons and the appellants
came into possession of some areas of that surplus land pursuant to the allotment made in
their favour in 1985. Thereafter, Prabal Pratap Singh and Dinesh Prasad Singh filed a writ
petition challenging the aforesaid determination of the excess land and also the
distribution in favour of the appellants. Though the appellants were made parties in the
said writ petition, the High Court ultimately dismissed their writ petition and the LPA
filed by those two persons was withdrawn subsequently. Thereafter, the 1st respondent
Pratima Devi filed CWJC No. 323 of 1999 before the High Court contending that she was
not heard and she was vitally interested in the matter before determining the ceiling area
applicable to the family of Pratima Devi, Prabal Pratap Singh and Dinesh Prasad Singh.
In that writ petition, the appellants in this Court were not made parties though the land
was in the lawful possession of the appellants. The writ petition was allowed by the High
Court by ordering certain areas claimed by the 1st respondent to be excluded from the
ceiling limit of the aforesaid two persons (Prabal Pratap Singh and Dinesh Prasad Singh).
When the appellants came to know about the said verdict of the High Court in the writ
petition they filed LPA by obtaining permission. But the LPA was dismissed by a
Division Bench of the High Court holding that the appellants/parcha-holders cannot
acquire any right merely because parcha has been issued in their favour and since the
acquisition has been held to be invalid they have no option but to walk out. Questioning
the said order, the appellants have filed special leave petition before this Court.
Considering the issue which is similar to our case, this Court held as under:
"5. We are not inclined to take the view that the appellants are not entitled to be heard in
the writ petition filed by the 1st respondent Pratima Devi merely because the
determination of the ceiling area had taken
@page-SC592
place at a time when the appellants had no right in the land. The appellants have been put
in possession of the land way back in 1985 by holding that it was a surplus area
pertaining to the family. They being in the enjoyment of the land on the strength of the
said allotment, they must have the right to substantiate that the allotment has been rightly
made in their favour and the area was rightfully held to be surplus area."
After holding so, this Court set aside the judgment passed by the learned Single Judge
and the Division Bench of the High Court and remitted to the High Court for disposal
after affording opportunity to the appellants. The decision therein is directly applicable to
the case on hand particularly in the light of the language used in amended Section 45-B
of the Act.
12. In view of the same, we are unable to agree with the observation of the Division
Bench since the appellants had the knowledge of the order of status quo passed by the
District Collector on 24.8.2005, the rules of natural justice were substantially complied
with. We have already held that prior to re-opening, notice to all the parties including
person(s) in possession was mandatory. It is not in dispute that the case was re-opened
and earlier decision was reversed holding that there was no excess land without issuing
notice to the appellants. Section 45-B empowers the State Government to re-open the
case which was already been disposed of by the Collector under the Act. After re-opening
the case, the State Government is to dispose of the matter afresh in accordance with law.
It is, therefore, clear that before passing any order in a concluded issue, the authority is
expected to satisfy the minimum requirement of principles of natural justice by issuance
of notice and hearing. Further, the said power to re-open has to be exercised sparingly
and for adequate reasons and the proceeding concluded earlier cannot be re-opened
merely for verification whether the orders were correctly passed. The order of re-opening
should be passed after hearing the parties concerned and where an order of re-opening the
case had been passed without hearing the party against whom it was passed, the order
suffers with legal infirmity and liable to be quashed. The reason behind in issuing a show
cause notice is precisely very clear in view of the fact that a proceeding once concluded
after a regular hearing should not be ordered to be re-opened suo motu by the authorities
concerned in a capricious manner and reasonableness requires that parties to be affected
by the same should be heard.
13. In this view of the matter, we are of the view that the orders impugned suffer from the
infirmity of not giving reasonable opportunity to the appellants before reopening the
proceedings. The order, therefore, is liable to be set aside. Under these circumstances, the
order of the High Court both learned Single Judge and the Division Bench are set aside.
However, the State Government is free to pass fresh order if they so desire under Section
45-B of the Act after affording opportunity to all the parties including the appellants
herein. Civil appeal is allowed to this extent. No costs.
Order accordingly.
AIR 2008 SUPREME COURT 592 "Amit Kumar v. State of U. P."
(From : Allahabad)*
Coram : 2 C. K. THAKKER AND ALTAMAS KABIR, JJ.
Civil Appeal No. 5455 of 2007 (arising out of SLP (C) No. 7731 of 2005), D/- 28 -11
-2007.
Amit Kumar v. State of U. P. and Ors.
U.P. Entertainment and Betting Tax Act (28 of 1979), S.3, S.11(3) - ENTERTAINMENT
TAX - Entertainment tax - Exemption as educational charity show - Fashion show -
Alleged to be charity show held with object of settings of Institute of Art, fashion
designing and modelling - Advertisement issued however showing that object of show
was to invite people to come and watch new world of glamour and modelling - Show
cannot be said to be educative for prospective students - Liable to levy of entertainment
tax - Show was held with full knowledge that tax was payable - Issuance only of
invitation cards and not tickets was only a subterfuge to evade tax. (Para 15)

Pramod Kumar Yadav, Viresh Kumar Yadav and Rameshwar Prasad Goyal, for Appellant;
S. K. Dwiwedi, Vinha Dwivedi, G. V. Rao, Raj Kumar Gupta, Ms. Vandana Mishra,
Kamlendra Mishra and Raj Kumar Mehta, for Respondents.
@page-SC593

* C. M. W. P. No. 2166 of 2002, D/- 22-2-2005 (All).


Judgement
ALTAMAS KABIR, J. :-Leave granted.
2. This appeal by way of Special Leave involves the question as to whether entertainment
tax was payable by the appellant in respect of a fashion show held at Gorakhpur in Uttar
Pradesh on 9th July, 2000 at St. Andrews Inter College for the selection of "Mr.
Gorakhpur" and "Miss Gorakhpur".
3. As it would appear from the materials on record, the appellant was found to be the
organiser of the aforesaid fashion show which had been held without the permission of
the District Magistrate. On the basis of enquiry, it was found that entertainment tax had
not been paid for performing the aforesaid fashion show and accordingly a show cause
notice was issued to the appellant which was replied to by the appellant. Not being
satisfied with the explanation given, the District Magistrate assessed a sum of
Rs.43,270.00 by way of entertainment tax for the programme and a further sum of
Rs.20,000.00 by way of penalty which was imposed upon the Cambridge Intertia Group
under whose banner the appellant is said to have arranged the fashion show.
4. In his reply to the show cause notice dated 11th July, 2000 under Section 12 of the
Uttar Pradesh Entertainment and Betting Tax Act, 1979 (hereinafter referred to as 'the
1979 Act'), the appellant contended that he was only a choreographer of Cambridge
Intertia Group which arranged the programme. The appellant contended that the
programme, as arranged, did not attract the provisions of the aforesaid Act and that the
show cause notice was without jurisdiction. A specific stand was also taken that Section 5
of the 1979 Act provided that any programme relating to entertainment could not be held
without prior permission but that since the programme was not entertainment within the
meaning of the Act, the same had been held by the Institution with prior intimation to the
office of the District Magistrate. It was reiterated that the programme was of a
competitive nature and there was no element of entertainment involved. Furthermore,
neither was any cultural, music and dance programme conducted nor was any amount
collected from the spectators by way of entry fee. According to the appellant, the show
was organised as a charity show with the specific purpose of publicising the event,
inasmuch as, there was a proposal initiated by Ms. Neetu Nathaniel (respondent No. 7
herein) for establishing an Institute of Art, Fashion, Designing and Modelling at
Gorakhpur in collaboration with the respondent No. 8, Smt. S. Mishra, proprietor of the
Cambridge Intertia Group.
5. Another stand taken by the appellant in the reply to the show cause notice was that
Miss Neetu Nathaniel was the Director and Smt. S. Mishra was the Convener of the show
and that the entire programme had been conducted under the direction of Miss Neetu
Nathaniel. In his reply, the appellant requested the District Magistrate to issue notice to
Miss Neetu Nathaniel who could enlighten him as to the alleged collection of money
against tickets sold and funds collected from the organisers. Since the appellant was only
a choreographer and his main function was to provide information about the candidates
participating in the programme, he denied that he had been involved with the holding of
the programme other than as a choreographer for the show.
6. As mentioned hereinabove, by his order dated 24th July, 2000 the District Magistrate,
Gorakhpur did not accept the explanation offered by the appellant and also the contention
that Miss Neetu Nathaniel was, in fact, the Director of the programme with Smt. S.
Mishra as the Convener. The District Magistrate chose to rely on the report submitted by
his department as to the collection of entry fee from the spectators and funds from the
organisers. Reference was also made to other shows of similar nature held in Gorakhpur
where fashion shows had been held after depositing the entertainment tax payable in
respect thereof and after obtaining the permission of the District Magistrate. Rejecting the
explanation offered by the appellant, the District Magistrate came to the conclusion that
the appellant had collected a total sum of Rs.1,62,500.00 from the spectators and a further
sum of Rs.25,000.00 from the five organisers at the rate of Rs.5,000/- from each
organiser. It was on that basis that a demand was raised by way of entertainment tax for
Rs.43,270.00 at the rate of 30% on the total collected amount of Rs.1,87,500.00.
7. The appellant challenged the said order of the District Magistrate by way of Civil
Misc. Writ Petition No. 2166/2002 in the Allahabad High Court. The same was taken up
for disposal on 22nd February, 2005 and
@page-SC594
on behalf of the writ-petitioner/appellant herein, it was sought to be reiterated that the
writ-petitioner/appellant was only the choreographer and had no function in holding the
fashion show. It was also urged that the programme in its entirety was charitable in nature
and being for an educational purpose, was exempted under Section 11(3) of the 1979 Act.
8. Negating the claim of the writ-petitioner/appellant, the High Court held that a fashion
show could not be said to be in aid of education and was only meant to entertain the
public. On the said finding, the Allahabad High Court dismissed the writ petition against
which the appellant filed SLP(C) No. 7731 of 2005 wherein leave has been granted.
9. Mr. Rameshwar Prasad Goyal, learned Counsel appearing for the appellant, reiterated
the submission which had earlier been made before the District Magistrate and the High
Court and submitted that the fashion show being merely competitive in nature and being
organised for the sake of publicity in connection with the proposed establishment of an
Institute of Art, Fashion Designing and Modelling by the Cambridge Intertia Group, the
provisions of the 1979 Act were not at all attracted and the show cause notice which had
been issued by the District Magistrate was without jurisdiction or in excess of the
jurisdiction vested in him under the Act.
10. Mr. Goyal urged that both the District Magistrate as also the High Court had wrongly
arrived at the conclusion that the appellant was responsible for organising and holding the
fashion show. He reiterated the submission made earlier before the other authorities that it
was the Respondent Nos. 7 and 8 who were the real organisers and convener of the
fashion show and the liability of entertainment tax, if any, had been wrongly foisted upon
him.
11. On behalf of Respondent Nos. 1 to 6, it was, however, submitted by Mr. S.K.
Dviwedi, the Additional Advocate General for the State of U.P., that the submissions
made on behalf of the appellant would be disproved on a perusal of the advertisement
which was published on the occasion, being Annexure CA-2 of the Counter Affidavit
filed on behalf of the Respondent Nos. 1 to 5. Mr. Dviwedi submitted that from the said
advertisement it would be clear that while Cambridge Intertia Group as an Institute of
Art, Fashion Designing and Modelling was presenting the fashion show, the Respondent
No. 7 was the Director, the Respondent No. 8 was the Programme Manager and the
appellant was the Programme Director and Choreographer on the occasion which was to
be attended by a Minister of the State Government together with various persons shown
as the sponsors of the programme.
12. Mr. Dviwedi also submitted that since the Minister was the Chief Guest at the fashion
show, various arrangements had been made so that the programme could be conducted
safely and without any disturbance. Furthermore, one of the sponsors shown in the
advertisement, namely, Mr. Pradeep Tekriwal, had specifically written to the authorities
informing them that he was neither a sponsor of the programme nor did he have anything
to do with the programme including making any monetary contribution.
13. Mr. Dviwedi submitted that both the District Magistrate and the High Court had
rightly held that the stand taken on behalf of the appellant that the fashion show was
merely a charity show was not tenable and it had been rightly held that the same was for
the purpose of entertainment and that large sums of money had been collected from the
spectators on the occasion. Mr. Dviwedi also submitted that despite the efforts of the
appellant to shift the liability of payment of the entertainment tax demanded by the
Respondent Nos. 7 and 8, it had been established that it was the appellant who had
master-minded the show with the full knowledge that the same was being held for the
purpose of entertainment and that entertainment tax was payable in respect thereof under
the 1979 Act.
14. Mr. Dviwedi submitted that no case had been made out on the appellant's behalf for
interference with the orders passed by the District Magistrate which were upheld by the
High Court.
15. We have carefully considered the submissions made on behalf of the respective
parties and we are inclined to agree with Mr. Dviwedi that the fashion show was held
with full knowledge that entertainment tax was payable in respect thereof and that though
tickets may not have been issued in respect of the programme and only invitation cards
had been issued, the same was merely a subterfuge for the purpose of evading and/or
avoiding payment of entertainment tax.
@page-SC595
It is difficult to believe that the fashion show was held with the object of educating
prospective students who would be interested in joining the Institute of Art, Fashion
Designing and Modelling and was, therefore, exempt under Section 11(3) of the 1979
Act. As the advertisement referred to above indicates the object of the show was to invite
people to come and watch the new world of glamour and modelling and to see the world
of exotic fashion in Gorakhpur itself.
16. We, therefore, see no reason to interfere with the order passed by the District
Magistrate, Gorakhpur and the High Court and we have no hesitation in dismissing the
appeal, but there will be no order as to costs.
17. While parting with the appeal, however, we cannot but remark upon some of the
statements made in the Writ Petition filed by the appellant before the High Court,
particularly those made in paragraphs 11 and 13 which, in our view, had little or no
relevance to the issue involved in the present appeal.
Appeal dismissed.
AIR 2008 SUPREME COURT 595 "B. S. Council of Ayurvedic and Unani Medicine v.
State of Bihar"
(From : Patna)*
Coram : 2 B. N. AGRAWAL AND PRAKASH PRABHAKAR NAOLEKAR, JJ.
Civil Appeal No. 4643 with 4644-4645 and 4646 of 2003, D/- 1 -11 -2007.
Bihar State Council of Ayurvedic and Unani Medicine v. State of Bihar and Ors.

WITH
Dr. Sudhir Kumar Singh and Ors. v. State of Bihar and Ors.

AND
Ashok Kumar and Ors. v. State of Bihar and Ors.
(A) Indian Medicine Central Council Act (48 of 1970), S.14, Sch.2 - Bihar Development
of Ayurvedic and Unani Systems of Medicine Act (31 of 1951), S.1 - MEDICAL
COUNCIL - CONSTITUTIONALITY OF AN ACT - Validity - 1951 State Act and 1970
Central Act are complementary to each other - Second Schedule of 1970 Act permits
grant of GAMS degree (Graduate of Ayurvedic Medicine and Surgery) - Introduction of
B. A. M. S. degree by 1970 Act - It cannot be said that GAMS degree issued by faculty
was derecognised or not in operation thereafter.
The Bihar Development of Ayurvedic and Unani Systems of Medicine Act (1951) is
consistent with the Indian Medicine Central Council Act (1970) in regard to granting of
the GAMS degree, as the degree granted under the 1951 State Act is still recognized
under the 1970 Central Act. The 1951 State Act and the 1970 Central Act are
complementary to each other. The faculty comes under the definition of 'medical
institution' under Section 2(f) of the 1970 Central Act and GAMS degree awarded by the
Faculty is a recognised medical qualification under Section 14 of the 1970 Central Act.
The Second Schedule of the 1970 Act grants authority to the Faculty to grant GAMS
degree. The High Court has, therefore, clearly committed an error in holding that after the
BAMS degree has been introduced, GAMS degree issued by the Faculty was
derecognised or not in operation after the 1970 Act came into force.
(Paras 11, 12)
(B) Bihar Indigenous Medical Educational Institution (Regulation and Control) Act (20
of 1982), S.5 - Indian Medicine Central Council Act (48 of 1970), S.14, Sch.2 - Bihar
Development of Ayurvedic and Unani Systems of Medicine Act (31 of 1951), S.54 -
MEDICAL COUNCIL - EDUCATION - Recognition of degree - Degree already granted
to students, recognised under 1951 Act - Accepted to be recognised degree under 1970
Act - Would not be ipso facto illegal on ground that permission to colleges of said
students was not granted by State Govt. as required under 1982 Act - State Govt.,
however, directed to take appropriate steps under the 1982 Act if any body, agency,
college or institution is/are functioning without permission as required under 1982 Act.
(Para 20)
(C) Indian Medicine Central Council Act (48 of 1970) (as amended by Amendment Act
2003), S.13A, S.13B, S.13C - MEDICAL COUNCIL - AMENDMENT - EDUCATION -
Degree conferred on students prior to commencement of Amending Act , 2003 - Is to be
treated as recognised degree - Although concerned medical college has not sought
permission of Central Govt. within period of three years from commencement of
Amending Act, 2003.
L. P. A. No. 463 of 2000, D/- 12-12-2001 (Pat.), Reversed.
@page-SC596

The amendment brought about in the Indian Medicine Central Council Act, 1970, in 2003
by introduction of Ss. 13A, 13B and 13C are the provisions for continuance of the
institution which has not obtained prior permission of the Central Government and,
therefore, time limit of three years has been provided under S. 13C to regularize the
institution's affairs as required under the Act by seeking permission of the Central
Government. Insertion of Section 13A in the 1970 Central Act in the year 2003 has
regulated the opening of an indigenous medical college. The non-obstante clause clearly
indicates that a medical institution cannot be established except with the prior permission
of the Central Government. Under S. 13B, any medical qualification granted by the
colleges established without the prior permission of the Central Government is not a
recognized medical qualification. The reasonable reading of S. 13C(1) puts the existing
colleges at par with the new colleges as both of them are required to seek permission
within three years from the commencement of the Amending Act. The phrase 'on or
before' has made it clear that the existing colleges are also required to seek permission
and there is no exemption. S. 13C(2) further provides that the medical qualification
granted by existing colleges whose establishment has not been recognized by the Central
Government, the medical qualification would not be a recognized qualification. Similar
requirement is to be fulfilled by the new medical colleges opened, i.e., to seek permission
of the Central Government for the medical qualification to be recognized qualification.
Thus, new colleges or existing colleges cannot any more grant a recognized qualification
without the sanction of the Central Government. S. 13C(2) does not say that the effect of
non-permission by the Central Government to the existing colleges after the Amending
Act came into force would render the medical qualifications already granted by the
existing colleges before the insertion of Ss. 13A, 13B and 13C in 2003, un-recognized.
The whole spectrum of the amendment brought about by introducing Ss. 13A, 13B and
13C indicates that it has an application from the date they have been introduced by an
amendment in the 1970 Central Act. The effect of the amendment brought about is that
all the medical colleges which are in existence or the medical colleges which have to be
established should compulsorily seek permission of the Central Government within the
period provided and on failure to get the permission of the Central Government the
medical qualification granted to any student of such medical college shall not be a
recognized medical qualification for the purposes of the 1970 Act. The established
colleges are also required to seek permission of the Central Government for the medical
qualification to be recognized medical qualification but it would not mean that the
already conferred medical qualification of the students studied in such previously
established medical colleges would not be a recognised medical qualification under the
1970 Act. When a degree has been legally conferred on the students prior to the
commencement of the Amending Act of 2003, it shall be treated as a recognized degree
although the medical college has not sought permission of the Central Government within
a period of three years from the commencement of the Amending Act of 2003. The
provisions of S. 13B whereby the qualification granted to any student of a medical
college would not be deemed to be a recognized medical qualification would not apply.
Therefore, the GAMS degree conferred by unrecognized colleges on the students shall be
treated as a recognized degree for the purposes of taking admission to the higher courses
of study and also for the purposes of employment.
L. P. A. No. 463 of 2000, D/- 12-12-2001 (Pat.), Reversed. (Paras 27, 28, 29)
(D) INTERPRETATION OF STATUTES - Interpretation of Statutes - Words used by
legislature - Construction - Causing injustice or even hardship or inconvenience - Not to
be put unless such was intention of legislature.
Where the legislature has used words in an Act which if generally construed, must lead to
palpable injustice and consequences revolting to the mind of any reasonable man, the
Court will always endeavour to place on such words a reasonable limitation, on the
ground that the legislature could not have intended such consequence to ensue, unless the
express language in the Act or binding authority prevents such limitation being
interpolated into the Act. In construing an Act, a construction ought not be put that would
work injustice, or even hardship or inconvenience, unless it is clear that such was the
intention of the legislature. It is also settled that where the language of the legislature
admits of two constructions and
@page-SC597
if construction in one way would lead to obvious injustice, the Courts act upon the view
that such a result could not have been intended, unless the intention had been manifested
in express words. Out of the two interpretations, that language of the statute should be
preferred to that interpretation which would frustrate it. It is a cardinal rule governing the
interpretation of the statutes that when the language of the legislature admits of two
constructions, the Court should not adopt the construction which would lead to an
absurdity or obvious injustice. It is equally well settled that within two constructions that
alternative is to be chosen which would be consistent with the smooth working of the
system which the statute purported to be regulating and that alternative is to be rejected
which will introduce uncertainty, friction or confusion with the working of the system.
AIR 1961 SC 1549, AIR 1973 SC 1461, Ref. (Para 24)
(E) INTERPRETATION OF STATUTES - Interpretation of Statutes - Court must always
lean to interpretation which is reasonable one - Literal interpretation which does not fit in
with scheme of Act under consideration should be discarded.
1996 AIR SCW 2120, AIR 1986 SC 137, Ref. (Para 25)
Cases Referred : Chronological Paras
1996 AIR SCW 2120 : AIR 1996 SC 1826 (Ref.) 26
AIR 1986 SC 137 (Ref.) 26
AIR 1973 SC 1461 (Ref.) 24
AIR 1961 SC 1549 : 1961 (2) Cri LJ 720 (Ref.) 24
AIR 1959 Punj 497 26
S. B. Sanyal, Sr. Advocate, Akhilesh Kumar Pandey, Sudhanshu Saran, Ms. Ranjana
Narayan, Ms. Shefali Jain, Ranjan Mukherjee, S. C. Ghosh, M. Qamaruddin, Ms. M.
Qamaruddin, Ambar Qamaruddin, Anukul Raj, Gopal Singh, Rituraj Biswas, Shrish
Kumar Misra and Navin Prakash, for the appearing parties.
* L. P. A. No. 463 of 2000, D/- 12-12-2001 (Pat).
Judgement
P. P. NAOLEKAR, J. : The brief facts of the case are that six petitioners in CWJC No.
7253 of 1998 before the Patna High Court who had obtained GAMS (Graduate of
Ayurvedic Medicine and Surgery) degree from the State Faculty of Ayurvedic and Unani
Medicines (for short "the Faculty") established under Section 17 of the Bihar
Development of Ayurvedic and Unani Systems of Medicine Act, 1951 (for short "the
1951 Act") were not permitted to appear in the examination for admission in Post
Graduate Course in Ayurved leading to award of Degree of Doctor of Medicine in
Ayurved. It was the case of the petitioners that they had passed the GAMS examination
conducted by the Faculty under the 1951 Act and were conferred GAMS degree by the
Faculty and, thus, they were qualified to appear in the examination for obtaining the
Degree of Doctor of Medicine in Ayurved. After service of notice, the respondents
entered appearance and the State filed reply wherein the stand taken by the State was that
GAMS Degree obtained by the petitioners in 1997 was not valid and recognized degree
because according to the letter dated 4.7.1998 sent by the Secretary, Central Council of
Indian Medicine (for short "CCIM"), GAMS course was no longer recognized by the
CCIM. The respondent-CCIM alleged that in accordance with the requirements of the
Indian Medicine Central Council Act, 1970 (for short "the 1970 Act"), CCIM had
prescribed regulations providing for BAMS (Bachelor of Ayurvedic Medicine and
Surgery) course at graduate level and MD(Ay.) course at post-graduate level, and only the
course prescribed by CCIM is to be conducted by the universities and the prescribed
degree can only be awarded by them as per the 1970 Act. It was also the case of the
respondents that after the Bihar Indigenous Medical Educational Institution (Regulation
and Control) Act, 1982 (for short "the 1982 Act"), the GAMS degree could only be
recognized if it is conferred on the students who had studied from the colleges recognized
under the 1982 Act.
2. On the pleadings of the parties, the High Court considered the case on the aspect
whether the Faculty under the 1951 Act has unqualified right to grant affiliation to such
institutions or colleges which are not following the BAMS course prescribed by CCIM
through regulations under the 1970 Act and further whether the provisions of the 1982
Act which seek to regulate institutions imparting training in Ayurvedic and Unani
Systems of Medicine shall cover and regulate even those institutions which have been
granted affiliation by the Faculty. The High Court held that the system of course for
GAMS had come to an end for quite some time and BAMS course has been
@page-SC598
followed as per the regulations of CCIM; hence, only on the basis of a continued entry in
the Second Schedule of the 1970 Act which recognized GAMS degree, which is in the
view of the High Court is archaic, no right can be found in the person or institution to
ignore the course validly prescribed by the competent authority-CCIM. The High Court
further held that the 1982 Act aims at curing a rampant evil in concerned colleges in the
State of Bihar and hence the State Government was given control in the matter of making
queries into the standard of educational institutions teaching Indian system of medicine,
and thereafter proceeding for recognition of the institution under the 1982 Act. It was
held that when the petitioners who obtained GAMS degrees had studied in the
educational institutions which have not followed course prescribed by CCIM, the
statutory central authority, and further when such institutions have been run in total
contravention and violation of the 1982 Act, they are not entitled to for issuance of any
writ from the court.
3. Another writ petition being CWJC No. 825 of 1998 filed by Pramila Kumari and Ors.
in the Patna High Court challenged the order whereby they had not been allowed to
compete in the selection for appointment to the post of Ayurvedic Medical Officer on the
basis that they were the holders of GAMS degree from the Bihar State Faculty, which
was claimed to be a recognized degree by the CCIM. The petitioners sought relief that
they be permitted to fill up the forms and to take part in the examination and further for
declaration that GAMS degree granted by the Faculty was equivalent to BAMS degree
granted by a recognized University of the State.
4. The learned Single Judge differed with the view taken by the court in CWJC No. 7253
of 1998 and held that Faculty has been created under the 1951 Act, much prior to the
promulgation of the 1982 Act, the powers under the 1951 Act of granting GAMS degree
by the Faculty is also recognized under the 1970 Central Act as per Second Schedule
thereof. The court was also concerned with the fate of the students who had been
conferred GAMS degree by a body created under the 1951 Act and the degree has been
saved by recognizing it under the 1970 Central Act. In this view, the matter was directed
to be placed before a Division Bench after necessary orders of Hon'ble the Chief Justice.
5. The judgment of the learned Single Judge in CWJC No. 7253 of 1998 was challenged
by filing LPA No. 451 of 2000 by only one petitioner, namely, Dr. Sudhir Kumar Singh
and other petitioners were impleaded in the case as respondents. Also the Bihar State
Council of Ayuyrvedic and Unani Medicine aggrieved by the judgment in CWJC No.
7253 of 1998, filed another letters patent appeal which was registered as LPA No. 463 of
2000. CWJC No. 825 of 1998 was placed along with the LPAs before the Division Bench
for decision. The writ petitioners re-asserted their submissions before the Division Bench
that they had completed the course of GAMS degree and passed examination conducted
by the Faculty under the 1951 Act. As per Second Schedule of the 1970 Act, a central
Act, which contains State-wise entries, entries Nos. 6 to 9A relate to the
institutions/universities of Bihar which recognize GAMS degree under entry No. 6 from
1953 onwards. It was submitted that as the degree conferred on the writ petitioners is a
recognized degree on the basis of the said entry in the 1970 Act, they were entitled to
appear for entrance test to the post-graduate course and also for consideration for
appointment to the post of Ayurvedic Medical Officers on the basis of GAMS degree
which they were holding.
6. The Division Bench agreed with the reasoning adopted by the learned Single Judge in
CWJC No. 7253 of 1990 and held that under the scheme of the 1970 Act as well as the
Bihar Indigenous Medical Educational Institution (Regulation and Control) Ordinance,
1981 which was replaced by Bihar Act 20 of 1982, the CCIM was authorised to prescribe
the course of studies in the system of medicine so that the Indian system of medicine may
maintain uniformity and standard of teaching all over the country, which has been sought
to be achieved by the regulations framed under the 1970 Act. The Division Bench also
agreed with the learned Single Judge that the course of study of GAMS had come to an
end and had been replaced by BAMS course, much before the writ petitioners acquired
their GAMS degree. The Court approved the decision of the learned Single Judge
whereby he had come to the conclusion that the 1982 Act has been enacted to regulate the
indiscriminate opening of indigenous medical institutions in the State by persons or
@page-SC599
bodies registered under the Societies Registration Act, 1960 and had in fact
commercialized the system of education in indigenous medicine; therefore, the
institutions which are not recognized by the State under the 1982 Act could not impart the
study in Ayurvedic medicine. It was held that the State authorities under the 1982 Act
have rightly taken the follow-up action. On these findings being arrived at by the
Division Bench, no merit was found in the LPAs and the writ petition, which were
dismissed by the Division Bench. Aggrieved by the order dated 12.12.2001 passed by the
Division Bench in the LPAs and the WP, the appellants, namely, Bihar State Council of
Ayurvedic and Unani Medicine (in Civil Appeal No.4643/2003), Dr. Sudhir Kumar Singh
and Ors. (in Civil Appeal Nos. 4644-46 of 2003) and Ashok Kumar Singh and Ors. (in
Civil Appeal No. 4646 of 2003) are before us by special leave.
7. It is an admitted fact before us that the writ petitioners have studied from Ramjee
Prasad, Ram Kumari Devi @ Marni Devi Ayurvedic Medical College and Hospital,
Fatuha and Shrihari Shakuntalayam Ayurvedic Medical College, Muzaffarpur, Bihar. The
said colleges were granted affiliation by the Faculty on 19.8.1995 with retrospective
effect from the session commenced in 1992 and they are recognized under the 1951 Act.
8. The Bihar Development of Ayurvedic and Unani Systems of Medicine Act, 1951
received the assent of the President on 12.9.1951 and the assent was first published in the
Bihar Gazette, Extraordinary, dated 17.10.1951. This Act was enacted to provide for the
development of the Ayurvedic and Unani systems of medicine, to regulate their teaching
and practice, and to control the sale of indigenous medicinal herbs and drugs in the State
of Bihar. In exercise of powers under Section 3, the State Government shall, by
notification, constitute a Council to be called the Bihar State Council of Ayurvedic and
Unani Medicines, which shall consist of a President and the Members mentioned in
clauses (a) to (n) of Section 3(1). Under Section 17 of this Act, the Council shall establish
a State Faculty of Ayurvedic and Unani Medicines for the purposes of the Act which shall
consist of a Chairman and the Members enumerated in clauses (a) to (d) of Section 17(1).
Under clause (d) of sub-section (2) of Section 17, it shall be the duty of the Faculty to
recognize educational or instructional institutions of the Ayurvedic and Unani systems of
medicine for purposes of affiliation. Under clause (b) of Section 17(2), the Faculty is
authorized to hold examination and grant certificates to, and confer degrees or diplomas
on, persons who shall have pursued a course of study in the institutions affiliated to the
Faculty. Section 37 of this Act authorizes the Council to establish educational institutions,
prescribe courses of study, etc. subject to the rules as may be prescribed by the State
Government in this behalf. Section 37 clothes the Council with power to establish its own
educational or instructional institutions for the purpose of conducting courses of
Ayurvedic and Unani systems of medicine. Under Section 54, the Council is authorized to
make regulations subject to the provisions of the Act and the rules made by the State
Government.
9. Looking into the aforesaid provisions, it is clear to us that the Council constituted by
the State Government under the 1951 Act shall establish a State Faculty under Section 17
which shall have the authority to recognize educational or instructional institutions of
Ayurvedic and Unani systems of medicine, to conduct examinations of the persons
studying in such affiliated institutions, and to grant certificates and confer degrees or
diplomas.
10. Under Section 54 of the 1951 Act, the Council has framed regulations called the
Bihar Development of Ayurvedic and Unani Systems of Medicines Regulations, 1959.
Regulation 16 thereof provides for courses of study for the Degree (Graduate of
Ayurvedic Medicine and Surgery) (GAMS). Thus, the Faculty established by the Council
under the 1951 Act has been authorized to recognize the educational institutions or
instructional institutions of Ayurvedic and Unani Systems of Medicine and affiliate them
to the Faculty. The Faculty is also authorized to conduct examinations and confer degree
of GAMS.
11. The Indian Medicine Central Council Act, 1970 (Central Act) provides for
constitution of a Central Council of Indian Medicine (CCIM) and the maintenance of a
Central Register of Indian Medicine and for matters connected therewith. This Act was
enacted by the Parliament and came into force on 21.12.1970. Introduction to this Act
reads as under:
@page-SC600
"To consider problems relating to the Indian system of medicine and Homoeopathy a
number of Committees were appointed by the Government of India, which had
recommended that a statutory Central Council on the lines of the Medicinal Council of
India for modern system of medicine should be established for the proper development of
these systems of medicine (Ayurveda, Siddha and Unani). In June, 1966 the Central
Council of Health at its 13th meeting, while discussing the policy on Ayurvedic
education, recommended the setting up of a Central Council for Indian system of
medicine to lay down and regulate standards of education and examinations,
qualifications and practice in these systems. On the basis of the above recommendations
the Indian Medicine Central Council Bill was introduced in the Parliament."
Sections 13A, 13B and 13C with their sub-sections have been substituted by the Indian
Medicine Central Council (Amendment) Act, 2003 (No. 58 of 2003) w.e.f. 7.11.2003,
which prescribe for the permission for establishment of new medical colleges, new
courses of study, etc.; non-recognition of medical qualifications in certain cases; and time
for seeking permission of the Central Government for certain existing or new medical
colleges. We shall deal with these Sections in detail when we take up the submissions of
the counsel of the effect of these Sections on the GAMS degree conferred on the students
prior to coming into force of Amending Act 58 of 2003. Section 14 falling in Chapter III
of the 1970 Central Act provides for recognition of the medical qualifications granted by
any university, board or other medical institution in India which are included in the
Second Schedule. The Second Schedule provides for the recognized medical
qualifications, i.e. degrees/diplomas, awarded by the States/Boards/Faculties/Universities
before the constitution of the Central Council of Indian Medicine. Under the 1970 Act,
the CCIM is competent to prescribe the minimum standard of education including
curriculam and syllabi as well as other requirements like hospital, library, students hostel,
staff for college, staff for hospital, library, herbal garden, requirements of various
departments of colleges, etc. The Second Schedule prescribes the institutions/colleges
and the medical qualifications which are recognized under the Act for the different States.
For the State of Bihar, item No. 6 of the Second Schedule reads as under :

Name of Universities,Board or Medical Institution Recognised Medical Qualifications


Abbreviation for Registration Remarks

xxx xxx xxx


Bihar
6. State Faculty of Ayur-vedic and Unani Medi-cines, Patna, Bihar. Graduate in
Ayurvedic Medicine and Surgery G.A.M.S. From 1953 onwards.
xxx xxx xxx

Under the 1970 Act, the State Faculty established under the Bihar State Council of
Ayurvedic and Unani Medicines (appellant in LPA No. 463 of 2000 and appellant in Civil
Appeal No. 4643 of 2000) is empowered to confer a degree of Graduate in Ayurvedic
Medicine and Surgery (GAMS) from 1953 onwards. It is an admitted fact that the 1951
Act has not been repealed by the 1970 Central Act nor it is the submission of any counsel
appearing for respective parties that the provisions of the 1951 Act, in regard to
conferment of GAMS degree, are repugnant to the provisions of the 1970 Act. The
Second Schedule in the 1970 Act itself recognizes the GAMS degree given by the State
Faculty of Ayurvedic and Unani Medicines, Patna, Bihar from 1953 onwards and thus it
cannot be said that the course prescribed by the Faculty for conferment of a degree of
GAMS is de-recognised under the 1970 Act. The 1970 Act read with regulations made
thereunder prescribes course for conferment
@page-SC601
of a degree of BAMS by a University, whereas the 1951 Act prescribes course for
conferment of a GAMS degree by State Faculty. Degree conferred by a university and
degree conferred by a faculty are different for which separate courses have been
prescribed. The 1951 Act having not been repealed by the 1970 Act, or till the Second
Schedule is not amended de-recognising the degree of GAMS, the degree of GAMS
given by the State Faculty will remain intact. No amendment has been brought about till
today whereby the degree of GAMS given by the State Faculty is de-recognised under the
1970 Act. The 1951 State Act with its rules and regulations, is a complete code for
recognizing and granting affiliation to indigenous medical institutions by the Faculty,
provide the course of study in the institutions, and regulate the functioning of the
institutions affiliated to the Faculty. The Faculty while exercising its powers has to abide
by the conditions laid down in the rules and regulations.
12. The 1951 State Act is consistent with the 1970 Central Act in regard to granting of the
GAMS degree, as the degree granted under the 1951 State Act is still recognized under
the 1970 Central Act. The 1951 State Act and the 1970 Central Act are complementary to
each other. The Faculty comes under the definition of 'medical institution' under Section
2(f) of the 1970 Central Act and GAMS degree awarded by the Faculty is a recognised
medical qualification under Section 14 of the 1970 Central Act. The Second Schedule of
the 1970 Act grants authority to the Faculty to grant GAMS degree. The High Court has,
therefore, clearly committed an error in holding that after the BAMS degree has been
introduced, GAMS degree issued by the Faculty was de-recognised or not in operation
after the 1970 Act came into force.
13. The question, however, is whether with the introduction of the Bihar Indigenous
Medical Educational Institution (Regulation and Control) Act, 1982, the students who
have studied in the colleges which were not recognized under the said 1982 Act could be
conferred with GAMS degree by the Faculty, and if such degrees are conferred what shall
be the fate of the degrees conferred on such students? We would also be required to
consider the effect of the Indian Medicine Central Council (Amendment) Act, 2003,
particularly Sections 13A, 13B and 13C which have been substituted by way of
amendment in the 1970 Act and came into force on 7.11.2003, on the degrees conferred
on the students who have studied in the colleges which have not sought or have not been
given permission as required under Section 13C of the 1970 Act to open the college or
continue the college, by the Central Government.
14. It is urged by the learned counsel for the appellants that the colleges which are
affiliated to the Faculty under the 1951 Act do not require any approval from the State
Government to start or to continue the educational institution or to run the courses of
study in indigenous system of medicine leading to the degree, diploma etc., as included in
Second Schedule of the 1970 Act, as the 1951 Act is a self-contained code. Whereas, it is
the submission of the learned counsel for the respondents that after the Ordinance of 1981
and the Act of 1982 came into force, all colleges which are affiliated to the Faculty or
which have to be opened after the Ordinance of 1981 and the Act of 1982 came into
force, require permission of the State Government for opening or continuing the colleges
or institution running the colleges, imparting education in indigenous system of medicine.
If any college or the educational institution running the college continues the educational
facility, imparting education in indigenous system of medicine leading to the degree,
diploma etc., as included in the Act of 1982 without permission, would run the risk to
their students of not being conferred with a recognized degree and penalties provided
under the 1982 Act. The counsel further submits that after the introduction of 1982 Act
the power of the Faculty to grant affiliation is circumscribed by the requirement of the
State Government's permission to open the college imparting education in Ayurvedic and
Unani systems of medicine.
15. Under the 1951 Act, Section 17 provides for the establishment of the Faculty. Sub-
section (2) of Section 17 provides : it shall be the duty of the Faculty to prescribe the
course of study and curricula for general instructions, or special refresher courses, in
institutions affiliated to the Faculty. By virtue of clause (d) of sub-section (2) of Section
17, the Faculty is to recognize educational or instructional institutions of the Ayurvedic
and Unani systems of medicine for purposes of affiliation. The manner in which the
affiliation is to be given is
@page-SC602
provided in Chapter II of the 1959 Regulations whereunder an application for affiliation
of an institution shall be made to the Registrar, State Council of Ayurvedic and Unani
Medicines, Bihar. After the application is received for affiliation, the Faculty will
scrutinize the application and if it is satisfied on the basis of the material supplied in the
application or otherwise that the institution proposed to be affiliated has nearly fulfilled
or is likely to fulfil all the conditions imposed by the Council established under the Act
and is likely to run efficiently, it would depute an Inspector to visit the institution, make
inquiry and report back to the Faculty. After the completion of the inquiry and submission
of the inspection report, the Faculty shall give recognition to the institution either
permanently or provisionally for a limited period or may reject it. The decision of the
Faculty shall be communicated to the institution concerned as soon as possible. It is clear
from the aforesaid provisions that the Faculty under the 1951 Act has been empowered
with the power to affiliate institutions which are imparting education in Ayurvedic and
Unani systems of medicine.
16. The Bihar Indigenous Medical Educational Institution (Regulation and Control)
Ordinance, 1981 which provides for regulation and control of educational institutions of
indigenous system of medicine in the State of Bihar was promulgated on 16th November,
1981. Preamble to the Ordinance reads as under :-
"Whereas, the Legislature of the State of Bihar is not in session ;
And, whereas, the Governor of Bihar is satisfied that in spite of repeated warnings from
Government through Press Notes and Notices unregulated and indiscriminate opening of
Indigenous Medical Educational Institutions in this State by persons or bodies registered
under the Societies Registration Act, 1960 or otherwise without providing for adequate
teaching facilities is hampering the cause of Indigenous Medical Education and is highly
detrimental to the interest of students, admitted to such institutions after charging heavy
capitation fee or donation and as such the circumstances exist which render it necessary
to prescribe for regulation and control on the opening of College or Institutions of
Indigenous System of Medicine in the State of Bihar - ;......"
17. The Ordinance was later replaced by introduction of the Act, viz., the Bihar
Indigneous Medical Educational Institution (Regulation and Control) Act, 1981 (Act 20
of 1982) which came into force on 21st January, 1982. Section 3 of the Act requires the
Governing Body or Organizing Committee or any body or institution intending to start
medical course of study of indigenous system of medicine, along with requisite
information regarding the study, to apply to the State Government in the Health
Department.

18. Section 5 contemplates that on receipt of an application for permission to open the
medical course of study of indigenous system of medicine, the State Government would
cause the inspection of the body, agency, college or institution by the Central Council of
Indian Medicine (CCIM) or Inspector appointed by the State Government to see whether
the conditions laid down by the CCIM constituted under Section 3 of the 1970 Act are
fulfilled or not. Section 6 further provides that on completion of the inspection the State
Government in the Health Department will seek permission of the Government of India
and the CCIM of India for granting permission to the starting of the course of medical
studies in indigenous system of medicine by the applicant. Section 7 postulates that to all
private medical colleges and medical institutions in indigenous system of medicine,
preparing or intending to prepare students for study in indigenous system of medicine
leading to degree, diploma, etc. and which have not been permanently affiliated to any
University in the State of Bihar, the provisions of the 1982 Act shall apply. The 1982 Act
has been made applicable to all private medical colleges and medical institutions which
are not permanently affiliated to any University in the State of Bihar. By virtue of Section
9, the institutions which have been functioning without prior permission or approval of
the State Government are required to apply for such permission within a period of one
month from the date of coming into force of the 1982 Act. This Section prohibits
admission of the students in such institutions till the grant of permission by the
Government. It also provides that in case the application is not moved within the
stipulated period or the State Government refuses permission, they will be deemed to
have been established in contravention of the provisions of the Act. Section 10 provides
for penalty and a person contravening any of the provisions of
@page-SC603
the Act is made liable for punishment with a fine which may extend to Rs. 10,000/- and
imprisonment for a term which may extend upto three years. In case of continuing
contravention, such person shall be liable to pay a further fine which may extend to
Rs.1,000/- per day after the date of the first conviction for the period during which he is
proved to have persisted in such contravention. The offence is made non-bailable and
cognizable. As per Section 11, if the application moved for permission to start medical
course of study of indigenous system of medicine either under Section 3 or Section 9 is
refused as the institution or college is not found eligible or does not qualify for
permission, it is incumbent on the organizer of such institution to close it down within a
period of three months of refusal of permission. Section 15 gives authority to the State
Government to seize the accounts of an institution contravening the provisions of the Act.
From these provisions, it is apparent that after introduction of the 1981 Ordinance and the
1982 Act, the Governing Body or the Organizing Committee or any body or institution
intending to start any course of study in indigenous system of medicine is required to
seek permission of the State Government to open a private medical college or medical
institution for admitting the students to be conferred with a degree, diploma, etc., as
included in the Second Schedule of the 1970 Act. It is only the Governing Body or the
Organising Committee or any body or institution which has been permanently affiliated
to any University in the State of Bihar is exempted from the provisions of the 1981
Ordinance or the 1982 Act. Institutions already imparting education in indigenous system
of medicine are required to take permission after coming into force of the 1982 Act.
19. The Act provides for imposition of the fine and imprisonment for any person who
contravenes any of the provisions of the 1981 Ordinance or the 1982 Act. If the
permission is refused, the institution will be closed down. Section 13 of the Act further
authorizes the State Government to authorize any officer to enter into the premises of the
institution contravening the provisions of the 1981 Ordinance or the 1982 Act for the
purposes of inspection and carrying into effect the provisions of the Ordinance or the Act.
Such officer may be empowered to close down the institution and to lock and seal it. The
Act also provides provision for seizure of the accounts by the State Government of an
institution contravening the provisions of the Ordinance or the Act. The Act arms the
State Government with various powers including the penal powers. Although the colleges
were opened in the year 1992 without the authority or the permission of the State
Government as required under the Act, no steps have been taken by the State of Bihar,
and the students admitted in the two institutions which were affiliated with the Faculty
were conferred with the GAMS Degree. After reading the provisions of the Act, it is
apparent to us that the 1982 Act is supplementary to the 1951 Act. The 1951 Act although
provides for the inspection of the institutions which have to be affiliated to the Faculty,
does not lay down that the conditions laid down by the CCIM are to be followed and
adhered to. That has been provided under the 1982 Act. So the colleges or the institutions
which want to impart education in the indigenous system of medicine have not only to
follow the conditions laid down by the Faculty or the Council under the 1951 Act, but
also under the 1982 Act. The college or the institution after the Act came into force
cannot continue without the permission of the State Government as contemplated in the
1982 Act.
20. We have examined the provisions of the 1982 Act. The counsel for the State or the
University could not point out as to what shall happen to the degrees given to the students
who studied in the colleges which have been affiliated with the Faculty but without
permission under the 1982 Act. We do not find any provision in the 1982 Act which takes
away the degree already granted to the students conferred by the Faculty, recognized
under the 1951 Act, and is being accepted to be a recognized degree under the 1970 Act.
Therefore, by virtue of introduction of the 1982 Act, it cannot be said that the degrees
conferred on the students who have studied in the colleges which have not been granted
permission by the State Government as required under the 1982 Act, will be ipso facto
illegal and could not be given effect to. However, we make it clear that any body, agency,
college or institution which has not sought permission from the State Government would
not be granted affiliation by the Faculty under the 1951 Act and the State Government
shall take appropriate steps under the 1982 Act if any body,
@page-SC604
agency, college or institution is/are functioning without the permission of the State
Government as required under the 1982 Act.
21. It is then contended by the learned counsel for the State that after the coming into
force of the Indian Medicine Central Council (Amendment) Act, 2003 (for short "the
Amending Act") on 7th November, 2003, if any medical college established on or before
the commencement of the Amending Act does not seek permission of the Central
Government within the period of three years from the said commencement, the medical
qualification granted to any student of such medical college shall not be deemed to be a
recognized medical qualification for the purposes of the 1970 Act. It is submitted that the
two colleges from where the appellant-students were educated having not sought
permission from the Central Government under the 1970 Act, the GAMS degree
conferred on them shall not be a recognized medical qualification for the purposes of the
1970 Act, as a result whereof they are not eligible for admission for higher course of
study or for employment on the basis of the GAMS degree conferred on them which is
not a recognized medical qualification. For this proposition, the learned counsel for the
State has relied upon the provisions of Sections 13A, 13B and 13C which have been
introduced by Amending Act of 2003. For a better understanding of the contentions, the
relevant portions of the Sections are reproduced hereunder:
"13A. Permission for establishment of new medical college, new course of study, etc.- (1)
Notwithstanding anything contained in this Act or any other law for the time being in
force,-
(a) no person shall establish a medical college; or
(b) no medical college shall-
(i) open a new or higher course of study or training, including a post-graduate course of
study or training, which would enable a student of such course or training to qualify
himself for the award of any recognised medical qualification; or
(ii) increase its admission capacity in any course of study or training including a post-
graduate course of study or training,
except with the previous permission of the Central Government obtained in accordance
with the provisions of this section.
Explanation 1.-For the purposes of this section, "person" includes any University or a
trust, but does not include the Central Government.
Explanation 2.-For the purposes of this section, "admission capacity", in relation to any
course of study or training, including post-graduate course of study or training, in a
medical college, means the maximum number of students as may be fixed by the Central
Government from time to time for being admitted to such course or training.
xxx xxx xxx
13B. Non-recognition of medical qualifications in certain cases.-(1) Where any medical
college is established without the previous permission of the Central Government in
accordance with the provisions of section 13A, medical qualification granted to any
student of such medical college shall not be deemed to be a recognised medical
qualification for the purposes of this Act.
(2) Where any medical college opens a new or higher course of study or training
including a post-graduate course of study or training without the previous permission of
the Central Government in accordance with the provisions of section 13A, medical
qualification granted to any student of such medical college on the basis of such study or
training shall not be deemed to be a recognised medical qualification for the purposes of
this Act.
(3) Where any medical college increases its admission capacity in any course of study or
training without the previous permission of the Central Government in accordance with
the provisions of section 13A, medical qualification granted to any student of such
medical college on the basis of the increase in its admission capacity shall not be deemed
to be a recognised medical qualification for the purposes of this Act.
13C. Time for seeking permission for certain existing medical colleges.- (1) If any person
has established a medical college or any medical college has opened a new or higher
course of study or training or increased the admission capacity on or before the
commencement of the Indian Medicine Central Council (Amendment) Act, 2003, such
person or medical college, as the case may be, shall seek, within a period of three years
from the said commencement, permission of the Central Government in accordance with
the provisions of section 13A.
(2) If any person or medical college, as
@page-SC605
the case may be, fails to seek permission under sub-section (1), the provisions of section
13B shall apply, so far as may be, as if permission of the Central Government under
section 13A has been refused."
22. For the purposes of the 1970 Act, 'Indian medicine' is a system of Indian medicine
commonly known as Ashtang Ayurveda, Siddha or Unani Tibb. Section 2(ea) of the 1970
Act defines 'medical college' to mean a college of Indian medicine where a person
undergoes a course of study or training which will qualify him for the award of a
recognized medical qualification. Section 13A(1) prohibits any person to establish a
medical college; and a medical college to open a new or higher course of study or
training including a post-graduate course of study or training, which would enable the
students of that medical college for the award of any recognised medical qualification or
to increase its admission capacity except with the previous permission of the Central
Government obtained in accordance with the provisions of Section 13A. Sub-sections (2),
(3), (4), (5), (6), (7), (8) and (9) of Section 13A lay down the manner in which the Central
Government is to be approached for establishment of a new medical college or for
opening of a new higher course of study or increasing admission capacity and how it
would be dealt with. Section 13B postulates that where any medical college is established
or an established medical college opens a new higher course of study or training or where
any medical college increases its admission capacity in any course of study or training
without the permission of the Central Government, the medical qualification granted to
any student of such medical college or the higher course of study or training or admission
in the increased capacity in any course of study, would not be a recognized medical
qualification for the purposes of the Act. Section 13C, however, provides a breathing time
to the medical colleges which have been established on or before the commencement of
the Amending Act of 2003 without the permission of the Central Government to get such
permission within a period of three years from the commencement of the Act. Therefore,
the colleges or the institutions which have not obtained the permission of the Central
Government may do so within a period of three years from the commencement of the Act
to save the medical qualification conferred on the students of such medical colleges from
the rigour of Section 13B of the 1970 Act. However, as per sub-section (2) of Section
13C, if any person or medical college fails to seek permission within three years of
commencement of the Act, qualification granted to any student of such medical
institution shall not be a recognised medical qualification and it shall be deemed that
permission to open or start a new course or increase strength of students was refused by
the Central Government. Medical colleges opened on or before the coming into force of
the Amending Act of 2003 are necessarily required to take permission within three years
to save the recognized medical qualification of the students. On their failure, the medical
qualification conferred on the students shall come to naught. Under Section 13A, a
person who establishes a medical college or a medical college opens a new higher course
of study or increases the admission capacity is required to move an application for
permission of the Central Government. For obtaining permission as required under
Section 13A, every person or medical college is required to submit a scheme in such form
with requisite fee, containing such particulars as provided under sub-section (3) of
Section 13A. The Central Government on receipt of such application may require the
applicant to submit such other particulars as may be considered necessary. The Central
Government after considering the scheme and recommendations of the Central Council
and after obtaining such other particulars as felt necessary, may approve the scheme with
such conditions which are considered necessary. The Central Government may also
disapprove the scheme. Sub-section (6) of Section 13A provides that where within a
period of one year from the date of submission of scheme to the Central Government, no
order is communicated by the Central Government to a person or medical college, such
scheme shall be deemed to have been approved by the Central Government in the form in
which it was submitted. From the aforesaid provisions, it is apparent that an application
seeking permission for opening a medical college has to be moved by a person which
also includes the university or a trust or a medical college or those which are already
running the medical college when the Amending Act came into force. Section 13A
nowhere provides that the students who have studied in the medical
@page-SC606
colleges would be eligible to seek permission of the Central Government under that
Section. Section 13A or Section 13B or Section 13C nowhere contemplates moving of an
application by the students to take steps under Section 13A. In such a situation, the
question arises what shall happen to the degrees conferred on the students who have
studied in the medical colleges established prior to the commencement of the Amending
Act where the Governing Body or Organising Committee or any body or institution does
not take any step for seeking permission of the Central Government and the period
prescribed under Section 13C of three years has expired or where the institution has been
closed down immediately after the commencement of the Amending Act of 2003 and,
therefore, no body is interested in seeking permission of the Central Government.
23. The provisions of Sections 13A, 13B and 13C of the 1970 Act as introduced by the
Amending Act of 2003, if given retrospective operation, the medical qualification
acquired from the study in the medical colleges which have been opened prior to the
commencement of the Amending Act of 2003 and conferred medical qualification on the
students who studied in such medical colleges, the degrees so conferred in the absence of
the permission of the Central Government would be non est though there is no fault on
the part of the students who have studied in the institutions which are recognized and
affiliated to the Faculty under the 1951 Act.
24

. In our opinion, where the legislature has used words in an Act which if generally
construed, must lead to palpable injustice and consequences revolting to the mind of any
reasonable man, the court will always endeavour to place on such words a reasonable
limitation, on the ground that the legislature could not have intended such consequence to
ensue, unless the express language in the Act or binding authority prevents such
limitation being interpolated into the Act. In construing an Act, a construction ought not
be put that would work injustice, or even hardship or inconvenience, unless it is clear that
such was the intention of the legislature. It is also settled that where the language of the
legislature admits of two constructions and if construction in one way would lead to
obvious injustice, the courts act upon the view that such a result could not have been
intended, unless the intention had been manifested in express words. Out of the two
interpretations, that language of the statute should be preferred to that interpretation
which would frustrate it. It is a cardinal rule governing the interpretation of the statutes
that when the language of the legislature admits of two constructions, the court should
not adopt the construction which would lead to an absurdity or obvious injustice. It is
equally well settled that within two constructions that alternative is to be chosen which
would be consistent with the smooth working of the system which the statute purported to
be regulating and that alternative is to be rejected which will introduce uncertainty,
friction or confusion with the working of the system. [See Collector of Customs v.
Digvijaysinhji Spinning and Weaving Mills Ltd. (1962) 1 SCR 896, at page 899 and His
Holiness Kesvananda Bharati v. State of Kerala, AIR 1973 SC 1461]. AIR 1961 SC
1549

25. The court must always lean to the interpretation which is a reasonable one, and
discard the literal interpretation which does not fit in with the scheme of the Act under
consideration.
26

. In series of judgments of this Court, these exceptional situations have been provided
for. In Narashimaha Murthy v. Susheelabai, (1996) 3 SCC 644 (at page 647), it was held
that: 1996 AIR SCW 2120, Para 20

".........The purpose of law is to prevent brooding sense of injustice. It is not the words of
the law but the spirit and eternal sense of it that makes the law meaningful.............."
In the case of American Home Products Corporation v. Mac Laboratories Pvt. Ltd. and
Another, AIR 1986 SC 137 (at page 166, para 66), it was held that:
".... It is a well-known principle of interpretation of statutes that a construction should not
be put upon a statutory provision which would lead to manifest absurdity or futility,
palpable injustice, or absurd inconvenience or anomaly........."
Further, in the case of State of Punjab v. Sat Ram Das, AIR 1959 Punj. 497, the Punjab
High Court held that:
"To avoid absurdity or incongruity, grammatical and ordinary sense of the words can, in
certain circumstances, be avoided."
@page-SC607
27. The amendment brought about in the Indian Medicine Central Council Act, 1970, in
2003 by introduction of Sections 13A, 13B and 13C are the provisions for continuance of
the institution which has not obtained prior permission of the Central Government and,
therefore, time limit of three years has been provided under Section 13C to regularize the
institution's affairs as required under the Act by seeking permission of the Central
Government. Insertion of Section 13A in the 1970 Central Act in the year 2003 has
regulated the opening of an indigenous medical college. The non obstante clause clearly
indicates that a medical institution cannot be established except with the prior permission
of the Central Government. Under Section 13B, any medical qualification granted by the
colleges established without the prior permission of the Central Government is not a
recognized medical qualification. The reasonable reading of Section 13C(1) puts the
existing colleges at par with the new colleges as both of them are required to seek
permission within three years from the commencement of the Amending Act. The phrase
'on or before' has made it clear that the existing colleges are also required to seek
permission and there is no exemption. Section 13C(2) further provides that the medical
qualification granted by existing colleges whose establishment has not been recognized
by the Central Government, the medical qualification would not be a recognized
qualification. Similar requirement is to be fulfilled by the new medical colleges opened,
i.e., to seek permission of the Central Government for the medical qualification to be
recognized qualification. Thus, new colleges or existing colleges cannot any more grant a
recognized qualification without the sanction of the Central Government. Section 13C(2)
does not say that the effect of non-permission by the Central Government to the existing
colleges after the Amending Act came into force would render the medical qualifications
already granted by the existing colleges before the insertion of Sections 13A, 13B and
13C in 2003, un-recognised. The whole spectrum of the amendment brought about by
introducing Sections 13A, 13B and 13C indicates that it has an application from the date
they have been introduced by an amendment in the 1970 Central Act. The effect of the
amendment brought about is clear to us that all the medical colleges which are in
existence or the medical colleges which have to be established should compulsorily seek
permission of the Central Government within the period provided and on failure to get
the permission of the Central Government the medical qualification granted to any
student of such medical college shall not be a recognized medical qualification for the
purposes of the 1970 Act. The established colleges are also required to seek permission of
the Central Government for the medical qualification to be recognized medical
qualification but it would not mean that the already conferred medical qualification of the
students studied in such previously established medical colleges would not be a
recognised medical qualification under the 1970 Act.
28. On a reasonable construction of these Sections, we hold that the provisions of Section
13B whereby the qualification granted to any student of a medical college would not be
deemed to be a recognized medical qualification would not apply. When a degree has
been legally conferred on the students prior to the commencement of the Amending Act
of 2003, it shall be treated as a recognized degree although the medical college has not
sought permission of the Central Government within a period of three years from the
commencement of the Amending Act of 2003.
29. For the reasons aforesaid, the appeals are allowed. The judgment of the High Court is
set aside and we hold that the GAMS degree conferred on the appellant-students shall be
treated as a recognized degree for the purposes of taking admission to the higher courses
of study and also for the purposes of employment.
30. There shall be no order as to costs.
Appeals allowed.
AIR 2008 SUPREME COURT 607 "Abdul Gafur v. State of Assam"
(From : Gauhati)*
Coram : 2 Dr. A. PASAYAT AND D. K. JAIN, JJ.
Criminal Appeal No. 1675 of 2007 (arising out of SLP (Cri.) No. 6635 of 2006), D/- 6
-12 -2007.
Abdul Gafur and Ors. v. State of Assam.
Penal Code (45 of 1860), S.395, S.397 - DACOITY - EVIDENCE - FIR - Dacoity -
Appreciation of evidence - Accused neighbours of victims
@page-SC608
Telephonic message given to police by inmate of house however describing accused as
strangers - Regular FIR lodged after much delay - Allegations of rape on young girl of
victim's family - Not substantiated by any material - Animosity between accused and
informant family admitted - Prosecution case unacceptable - Accused liable to be
acquitted.
Crl. Appeal No. 201 of 1998, D/- 12-5-2006 (Gau.), Reversed. (Paras 10, 11)

H. L. Agrawal, Sr. Advocate, Azim H. Laskar, Anand, Abhijit Sengupta, for Appellants;
Avijit Roy (for M/s. Corporate Law Group), for Respondent.
* Cri. Appeal No. 201 of 1998, 12-5-2006 (Gau.).
Judgement
1. Dr. ARIJIT PASAYAT, J. :- Leave granted.
2. Challenge in this appeal is to the order passed by a learned Single Judge of the Gauhati
High Court dismissing the appeal filed by the appellant.
3. Background facts according to the prosecution in a nutshell are as follows:
On the night of 11.4.88 at about 6.30 P.M. the accused Abdul Gafur, Hokoi Mian, Najir
Ali, Sayed Ali, Latif Ali, Aklas Mian, Ashu Mian and Tabai Mian being armed with
deadly weapons namely, dao, lathis, dagger etc. surrounded the house of Satyendra Nath
Gupta at village Brahrnanshashan, assaulted him, his wife Smt. Hemamalini Gupta, his
son Subhendu Gupta, his eldest daughter Anjali Gupta and his relatives Sushil Chanda
causing grievous injuries to them, tied them up and then looted gold ornaments, namely,
chains, bangles, ear rings etc. valued at Rs.42,950.00 from the possession of the female
inmates of the house, namely, Hemamalini Gupta, Anjali Gupta, Mitra Gupta, Rubi Gupta
and Nell Gupta. That apart two of the accused persons, namely, Hokoi Mian and Aklas
Uddin committed rape respectively on Mitra Gupta and Rubi Gupta and decamped with
the looted booties. During the course of occurrence Sushil Chandra Gupta the son of
Satyendra Nath Gupta informed police over telephone that decoity was being committed
in the house of Satyendra Nath Gupta and that Satyendra Nath Gupta and his wife were
assaulted by the decoits causing grievous injuries to them. At the Nilambazar out post a
general diary vide entry No.212 at 8.15 p.m. on the night of 11.4.1988 was recorded and
on the basis of such information enquiry was launched. Thereafter Satyendra Nath Gupta
also lodged a written Ejahar with police of Nilambazar out post. The Officer Incharge of
Nilambazar out post sent the written ejahar to the Officer Incharge of Karimganj P.S.
whereupon the Officer Incharge of Karimganj P.S. registered a case under Section
395/397/376 of the Indian Penal Code, 1860 (in short the 'IPC'). S.I. of police T.C.
Bailong after completion of enquiry/investigation submitted charge sheet against the
accused Abdul Gafur, Hokoi Mian, Boloi Mian, Sayed Ali, Aklas Uddin, Najir Ali, Latif
Ali, Ashu Mian and Tabai Mian for alleged commission of offences punishable under
Sections 395 and 397 IPC.
The charges against the accused Najir Ali, Ashu Mian and Tabai Mian were proven and
they were declared to be proclaimed absconders. The case against other six accused
persons, namely Abdul Gafur, Hokoi Mian, Boloi Mian, Sayed Ali, Alas Uddin and Latif
Ali was committed to the Court of Session by learned Judicial Magistrate, 1st Class,
Karimganj.
4. During trial nine witnesses were examined to further the prosecution version.
5. Placing reliance on the evidence of witnesses-PWs. 1, 2, 3, 5 and 8, the trial court
found that accused appellant 1,2,3,5and6 guilty of offence punishable under Section 395
read with Section 397 IPC and accused appellant Nos. 3 and 5 were guilty of offence
punishable under Section 354 IPC. For the offence relatable to Section 395 read with
Section 397, each was sentenced to undergo rigorous imprisonment for seven years and
to pay a fine of Rs.2,000/- with default stipulation. For the offence relatable to Section
354 IPC they were sentenced to rigorous imprisonment of one year each.
6. The convicted accused persons preferred an appeal before the High Court. As afore-
stated the High Court dismissed the appeal and affirmed the conviction and sentence.
7. In support of the appeal learned counsel for the appellant submitted that the High Court
has disposed of the appeal cryptically without even discussing the various submissions
made. There are also several infirmities in the conclusions arrived at.
8. Learned counsel for the respondent-State on the other hand supported the judgment of
the trial court and the High Court.
@page-SC609
9. The High Court has noted as if a telephonic message was given by a stranger regarding
decoity. However, in evidence it has given that the information was given not by a
stranger but by Sushil Chandra Gupta, PW 1. In the information given it was stated that
some stranger had committed dacoity.
10. The accused persons are not strangers and were practically neighbours of the
informant and his family. The High Court noted that there was no intention to falsely
implicate accused persons because of enmity and there was no reason as to why dignity
of two young girls would be put at stake by alleging rape. It is to be noted that in fact
rape was alleged but the Trial Court found that there was no material to substantiate the
plea of rape. The evidence is totally inconsistent and lacks credence. The High Court's
observations were clearly based on surmises and contrary to the factual scenario. The
High Court has noted that the evidence of PWs. 1,2,3,5 and 8 stand fully corroborated by
the medical evidence. Significantly, on consideration of the evidence of PW 4, it is clear
that the evidence of this witness is clearly contrary to the medical evidence. To add to the
confusion, it is noted that the High Court recorded as finding that appellant Abdul Gafur
was absconding. As a matter of fact the evidence of Investigating Officer (in short the
'I.O') shows that he had arrested Abdul Gafur on the date the First Information Report (in
short the 'FIR') was lodged. Unfortunately the High Court has merely referred to certain
conclusions of the Trial court without analyzing the evidence and various submissions
made by the appellants. To add to the vulnerability of the prosecution version, the FIR
was lodged long after the incident and in fact law was already set on motion after the
telephonic message had been received.
11. The aforesaid infirmities in the background of admitted animosity between the parties
renders the prosecution version unacceptable. The Trial Court and the High Court did not
analyse the evidence correctly and acted on mere surmises and conjectures. That being
so, the appellants deserve to be acquitted, which we direct.
12. Appeal is allowed. The appellants are acquitted of the charges. They be set forth at
liberty if not required in any other case.
Appeal allowed.
AIR 2008 SUPREME COURT 609 "State of Punjab v. Raninder Singh"
(From : Punjab and Haryana)
Coram : 2 A. K. MATHUR AND MARKANDEY KATJU, JJ.
Criminal Appeal Nos. 1608 with 1607 of 2007 (@ out of SLP (Cri.) Nos. 3433 with 3514
of 2007), D/- 19 -11 -2007.
State of Punjab v. Raninder Singh and Anr.
WITH
State of Punjab v. Jagjit Singh.
(A) Criminal P.C. (2 of 1974), S.378 - ANTICIPATORY BAIL - APPEARANCE -
INTERROGATION - Anticipatory bail - Condition that accused must appear before
investigating officer for interrogation when required Breach of - Bail can be cancelled.
(Para 5)
(B) Contempt of Courts Act (70 of 1971), S.2(c), S.14 - CONTEMPT OF COURT -
POLICE OFFICERS - HIGH COURT - Contempt of Court - Use of improper language
-Police Officer using loose expressions in his affidavit and application - Held, though was
not contempt, language used should have been in consonance with dignity of Court and
facts stated should be correct - Notice for contempt issued by High Court liable to be set
aside. (Para 9)

K. K. Khurana, A. A. G., K. K. Venugopal, Ashwani Kumar Mata, Ravi Shankar Prasad,


U. U. Lalit, L. N. Rao, Sr. Advocates, Kuldip Singh, Ajay Pal, Vaibhav Dang, Amrendra
Kumar Mehta, Mrs. Rashi Khurana, Atul Nanda, Rajesh Kumar, Sandeep Bajaj (for M/s.
Law Associates and Co.), Arun Monga, Ms. Rameeza and Ms. Naresh Bakshi, with them
for the appearing parties.
Criminal Appeal No. 1608 of 2007 @ Slp (Crl.)) No. 3433 of 2007
Judgement
JUDGMENT:-We have heard learned counsel for the parties.
2. Leave granted.
3. We have perused the impugned order dated 24th May, 2007 granting anticipatory bail
passed by the learned single Judge of the Punjab and Haryana High Court in Crl. Misc.
No. 33867-M of 2007. After hearing learned counsel for the parties, we are of the opinion
that no interference is called for by this Court. The appeal is accordingly, dismissed.
4. However, we make it clear that in case the respondents do not cooperate with the
investigation, then it is always open for the
@page-SC610
State to move an application before the High Court for cancellation of the bail, which will
be decided in accordance with law.
5. It may be mentioned here that Section 438 (2)(i) of the Code of Criminal Procedure is
very clear that while granting anticipatory bail the Court can lay down a condition that
the accused shall make himself available for interrogation by a police officer as and when
required. The purpose of such a provision is that anticipatory bail cannot be permitted to
be abused. It is therefore, implicit that whenever the Court imposes such a condition in its
order, and the accused called for interrogation or for certain investigation does not appear
before the investigating officer then it will be open for the State to move the High Court
for cancellation of bail.
6. We make it clear that this order is confined only to the F. I. R. in the present case.
Criminal Appeal No. 1607/2007 @ SLP (Crl.) No. 3514/2007
7. We have heard learned counsel for the parties.
8. Leave granted.
9. This appeal by special leave is directed against the orders dated 15-5-2007 and 31-5-
2007 passed by the learned single Judge of the Punjab and Haryana High Court in Crl.
Misc. No. 27116-M of 2007 and CRM No. 36313 of 2007 whereby a contempt notice
was issued to the Senior Superintendent of Police, Vigilance Bureau, Ludhiana who has
sworn the affidavit along with the application. Normally the Courts should not be
oversensitive and should not take very serious note of any loose expressions in the
application. Contempt jurisdiction is to be sparingly exercised in very exceptional cases,
as one of us (Markandey Katju, J.) has observed in an article 'Contempt of Court : The
Need for a Fresh Look' published in the Journal Section of AIR 2007 (March Part), and
we agree with the views expressed therein. However, the applicant should use proper
language and state correct facts in his application. Although it is not contempt, proper
decorum should be maintained. Be that as it may, we are of the opinion that the learned
Judge should not have issued contempt notice in the matter. The S. S. P. had sworn the
affidavit but the counsel who has prepared the application should have been more careful
while drafting such an application. They should not make incorrect statements. The
language used by them should be in consonance with the dignity of the Court.
10. Having regard to the facts and circumstances of the case, we do not think it to be a
proper case where contempt notice ought to have been issued.
11. In the facts and circumstances of the case, while maintaining the order dated 15-5-
2007 we set aside the direction in the last paragraph of the learned single Judge's Order
dated 31st May, 2007 issuing notice to the S. S. P., Vigilance Bureau, Ludhiana.
12. The appeal is accordingly, disposed of.
Order accordingly.
AIR 2008 SUPREME COURT 610 "Shaik China Brahamam v. State of A. P."
(From : Andhra Pradesh)*
Coram : 2 G. P. MATHUR AND D. K. JAIN, JJ.
Criminal Appeal No. 927 of 2006, D/- 29 -11 -2007.
Shaik China Brahamam v. State of A. P.
(A) Penal Code (45 of 1860), S.300 - MURDER - WITNESS - Murder - Proof - Accused
with one other alleged to have assaulted deceased with multiple knife injuries resulting in
death - Exchange of hot words between accused and deceased two days prior to incident
over borrowing of money - Proves motive - Eye-witness not inimical towards accused -
His narration of incident corroborated by medical evidence - Accused and co-accused
liable to be convicted for murder. (Para 11)
(B) Penal Code (45 of 1860), S.34, S.300 - MURDER - COMMON INTENTION -
EVIDENCE - PLEA - Common intention - Murder case - Accused armed with knife and
appellant armed with iron rod going together and assaulting deceased - Medical evidence
showing that knife injuries were fatal - Plea by appellant that he has not caused any fatal
injury - Could not, therefore, be convicted for murder with aid of S. 34 - Not tenable -
When criminal act is done with common intention all are liable in some manner as
principal offender - Moreso, in instant case appellant had played an active role. (Para
12)

I. V. Narayana, Sr. Advocate, T. N. Rao,


@page-SC611
Ms. Manjeet Kirpal and Paramjeet Singh, with him for Appellant; Ms. Altaf Fathima and
Ms. D. Bharathi Reddy, for Respondent.
* Cri. A. No. 116 of 2004, D/- 2-3-2006 (A.P.)
Judgement
1. G. P. MATHUR, J. :- This appeal under Section 2 of the Supreme Court (Enlargement
of Criminal Appellate Jurisdiction) Act, 1970 has been preferred against the judgment
and order dated 2.3.2006 of Andhra Pradesh High Court, by which the appeal filed by the
State was allowed and the judgment and order dated 24.9.2002 of the learned Sessions
Judge, Guntur acquitting the two accused in Sessions Case No.466 of 2000 was set aside.
The High Court by the impugned judgment and order convicted both the accused Shaik
Khasim Saida (A-1) and Shaik China Brahmam (A?2) under Section 302 read with
Section 34 IPC and sentenced them to imprisonment for life and a fine of Rs.200/- each.
2. The case of the prosecution, in brief, is that Shaik Khasim Saida (A-1) had borrowed
Rs.300/- from the deceased Shaik Masthan Vali some time back, but he did not repay the
amount due to which their relations became strained. At about 4.00 p.m. on 6.4.1999, the
deceased Shaik Masthan Vali and his cousin Shaik Baba Vali (PW.1) were returning to the
village from northern side of Chandravanka rivulet after attending the call of nature. Both
the accused suddenly appeared on the spot. A-1 stabbed the deceased Shaik Masthan Vali
repeatedly with a knife which he has carrying and A-2 caused injuries to the deceased
with iron pipe. After receiving injuries the deceased fell down dead on the spot. PW.1
Shaik Baba Vali lodged an FIR at P.S. Macherla at 6.30 p.m. on the same day. On the
basis of the FIR a crime was registered as Case Crime No.55 of 1999 under Section 302
read with Section 34 IPC at the police station.
3. After the case had been registered at the police station, PW.8 K. Babu Rao, Inspector of
Police, P.S. Macherla commenced investigation of the case. He arrested A-1 and A-2 and
went to the scene of occurrence and prepared a site plan. After recording statement of
witnesses, he submitted charge-sheet against both the accused A-1 and A-2. The
prosecution in order to establish its case examined nine witnesses and filed some
documentary evidence. The learned Sessions Judge, Guntur, by the judgment and order
dated 24.9.2002 acquitted both the accused A-1 and A-2. Feeling aggrieved by the order
of the learned Sessions Judge, the State filed appeal in the High Court, which was
allowed and accused were convicted under Section 302 read with Section 34 IPC and
were sentenced to imprisonment for life and a fine of Rs.200/- each. The present appeal
has been filed only by Shaik China Brahmam (A-2). It appears that Shaik Khasim Saida
(A-1) has not preferred any appeal against his conviction and sentence.
4. We have heard Mr. I.V. Narayana, learned counsel for the appellant and Ms. Altaf
Fathima, learned counsel for the State of Andhra Pradesh and have perused the record.
5. The case basically rests on the testimony of PW.1 Shaik Baba Vali. He has deposed that
his house is situate near Chennakesava Swamy Temple in Macherla and both the accused
viz. A-1 and A-2 are also residents of the same place. The deceased Shaik Masthan Vali
was also resident of Macherla. The deceased had informed him that A-1 had borrowed
money from him and had not returned the same and due to this their relations had become
strained. At about 4.00 p.m. on 6.4.1999, he and deceased Shaik Masthan Vali had gone
to the field by the side of Chandravanka rivulet for answering the call of nature.
Thereafter, they were returning home and the deceased was little behind him. Suddenly
he saw that A-1 had caught hold of the deceased by putting his arm around his neck and
then he started giving him repeated blows by a knife. A-2 also assaulted the deceased
with an iron pipe. The deceased raised an alarm. When PW.1 tried to save the deceased,
both the accused threatened him that they would also assault him. The deceased Shaik
Masthan Vali snatched the knife from the hands of A-1, but A-1 again snatched back the
knife from the deceased and gave him several blows. In the process of snatching the
knife, the hands of A-1 also got cut injuries. PW.1 then went to the house of the deceased
Shaik Masthan Vali and informed his wife and other relations about the incident.
Thereafter, he went to P.S. Macherla and presented a written report. He got the report
scribed by a person who was sitting outside the police station. He identified the knife
M.O. 1 and the iron pipe M.O. 2, which were shown to him in Court. He also identified
the clothes, which the deceased was wearing viz. M.O. 3 the blood
@page-SC612
stained shirt, M.O. 4 the blood stained banian, M.O. 5 the blood stained dhoti, M.O. 6 the
chappal which the deceased was wearing and M.O. 7 the towel which the deceased was
having on his body. In his cross-examination, he has stated that his house and that of the
deceased were situate in the same ward and in side-by-side streets. The accused A-1 and
A-2 were staying in a parallel streets. The distance between the house of the deceased and
the police station is about 3 furlongs. He clarified that M.O. 2 is an iron pipe.
6. PW.2 Sk. Masthan Bee is the wife of the deceased Shaik Masthan Vali. She deposed
that her house is by the side of Chennakesava Swamy Temple in Macherla and the house
of PW.1 Shaik Baba Vali was near her house and the houses of the accused were situate at
some distance. She further deposed that PW.1 Shaik Baba Vali came to her house and
informed that A-1 and A-2 had killed her husband. There was some dispute between her
husband and the accused on account of borrowing of Rs.300/- and two days prior to the
incident an altercation had taken place between them at the tea stall of Achari, which she
had also seen. At that time A-1 had said loudly that he would kill her husband. After
learning about the incident from PW.1 she rushed to the scene of occurrence and saw the
dead body of her husband lying there. She denied the defence suggestion that A-1 had not
loudly said two days back that he would kill her husband or that PW.1 had not informed
her that A-1 and A-2 had killed her husband.
7. PW.5 Dr. P. Rajasekhara Reddy was working as Civil Assistant Surgeon in Community
Health Centre, Macherla from October 1997 to 4.1.2001. He conducted postmortem
examination over the body of Shaik Masthan Vali from 11.00 a.m. onwards on 7.4.1999
and found the following injuries on the same :-
"1. A cut injury over the anterior aspect of neck, cutting the trachea and carotids, 14 cm x
5 cm.
2. A cut injury over the right temple, 3 cm x 1 cm, dark brown in colour.
3. A cut injury over the nape, 10 cm x 4 cm.
4. Two cut injuries over the occipital area, each 3 cm x 1 cm side by side.
5. Another cut injury over the occipital area, 2" above the wound No.4 semi circle, 7 cm x
3 cm, exposing the brain matters.
6. A cut injury over the left parietal area above the left ear, 3 cm x 1 cm.
7. A cut injury behind the left ear, 4 cm x 1 cm.
8. A cut injury below the left ear, 2 cm x 1 cm.
9. A cut injury on the right ear, 5 cm in length.
10. A cut injury on the right thumb 2 cm x 2 mm.
11. A cut injury on the right little finger, 1 cm x 2 mm.
12. A cut injury over the right wrist on flexor side, 4 cm x 2 cm.
13. A cut injury over the right scapular area, 2 cm x ½ cm.
14. A contusion over the left scapular area, 3 cm in diameter, brown in colour.
15. A contusion on the right axilla, 4 cm x 1 cm, brown in colour.
16. A cut injury over the left thumb, circling the both surfaces, 3 cm x 1 mm.
17. Two small cut injuries on the left index finger, each 1 cm x 1 mm 1 cm apart.
18. A contusion on the left forearm 3 cm x 4 cm.
19. Scrotum was swollen.
20. On opening the body all viscera are normal and pale.
Patient died about 18 to 22 hours prior to postmortem examination."
In the opinion of the doctor, the deceased had died on account of shock and haemorrhage
due to multiple injuries and cardio-pulmonary arrest. Injury No.1 caused on the anterior
aspect of the neck cutting the trachea and carotids and injury No.5 i.e. the injury on the
occipital area were fatal. He further opined that injury No.1 and injury No.5 were
sufficient to cause instantaneous death.
8. PW.9 Dr. S. Sakunthala was Civil Assistant Surgeon at Government Hospital,
Macherla. She examined Shaik Khasim Saida (A-1) at 7.55 p.m. on 6.4.1999 and found
the following injuries on his body :
"1. Incised wound 4 x ½ cm, over right palm, bleeding present, red.
2. Incised wound 3 x ½ cm, over right palm, 1 cm. below No. 1 injury, bleeding present,
red.
@page-SC613
3. Incised wound ½ x ¼ cm, over left index, ring, middle, little fingers except thumb,
over palm inner side, red."
In the opinion of the doctor, the injuries were simple in nature and were caused due to a
sharp object. The duration of injuries was 3 to 4 hours.
9. On the same day, i.e., on 6.4.1999 at 7.30 p.m. PW.9 Dr. S. Sakunthala also examined
Shaik China Brahmam (A-2) and found the following injuries on his body :-
"Incised wound 1 cm x ¼ cm over right index finger, over palm side."
The doctor opined that the injury was simple in nature and was caused due to a sharp
object. The duration of injury was 3 to 4 hours.
10. PW.7 T.A. Rambabu was Head Constable, P.S. Macherla. He has deposed that on
6.4.1999, when he was attending to his duties in the police station, he received
information that a murder had taken place near Chandravanka Vagu (rivulet). He went to
the office of Inspector of Police and informed him about the same. At about 6.30 p.m.
PW.1 Shaik Baba Vali came to the police station and presented a written report Ex.P1. He
registered the same as Case Crime No.55 of 1999 under Section 302 IPC and sent copies
of the FIR to all concerned. Thereafter, the Police Inspector came to the police station at
6.40 p.m. along with two persons who were having injuries on their hands. He sent them
for medical examination. PW.8, K. Babu Rao was Inspector of Police, P.S. Macherla. He
deposed that about 5.30 p.m. on 6.4.1999, he heard a rumour that a murder had taken
place near Chandravanka Vagu. He immediately left for the scene of occurrence along
with some police personnel which was between Macherla and Jammalamadaka near the
field of one Pathu Sahab and saw a dead body there. He also received information about
the presence of the accused near Jasmine Garden. He proceeded there with his staff. He
saw A-1 and A-2 and arrested them. They had injuries on their hands. They were taken to
the police station from where they were sent for medical examination. As it had become
dark, he did not conduct any further investigation which he commenced on the next day
at 7.00 a.m. He prepared a site plan. He also seized the knife M.O. 1 and the iron pipe
M.O.2. He also seized M.O. 6 and M.O. 7 belonging to the deceased. He held inquest
over the body of the deceased at 8.00 a.m. After recording statements of the witnesses
under Section 161 Cr.P.C. and preparing other relevant papers, he submitted charge-sheet
against the two accused viz. A-1 and A-2.
11. We have given above the gist of the evidence adduced by the prosecution. The
testimony of PW.2, Sk. Masthan Bee wife of deceased Shaik Masthan Vali establishes
that there was dispute regarding borrowing of Rs.300/- between A-1 and the deceased.
Two days prior to the occurrence, exchange of hot words had taken place between them
at the tea stall of Achari, where A-2 was also present and at that time A-1 had declared
that he would kill the deceased Shaik Masthan Vali. This shows that there was motive on
the part of the accused to assault the deceased. PW.1 Shaik Baba Vali had deposed that at
about 4.00 p.m. on 6.4.1999 he had gone along with deceased across Chandravanka Vagu
for answering the call of nature. While returning A-1 armed with knife and A-2 armed
with rod started assaulting the deceased Shaik Masthan Vali. He has also deposed that the
deceased had snatched the knife from A-1, but A-1 snatched it back and again caused
injuries from the same to the deceased. P.W.1 gave immediate information about the
occurrence to the family members including the wife of the deceased, P.W. 2 Sk. Masthan
Bee, which is established from her testimony. The most important feature of the case is
that P.W. 8K. Babu Rao, Inspector of P.S. Macherla, received information at about 5.30
p.m. that a murder had taken place near Chandravanka Vagu. He left for the scene of
occurrence and found the dead body there. He also received information about the
presence of the accused near Jasmine Garden and arrested them at about 6.40 p.m. Both
the accused were sent for medical examination and three incised wounds were found on
the right and left palm of A-1 and one incised wound was found on the right palm of A-2.
The injuries on the palm of A-1 and A-2 completely corroborate the version given by
P.W.1 Shaik Baba Vali that the deceased had snatched the knife from the hands of A-1,
but accused again snatched it back. The multiple injuries found on the body of the
deceased, which are mostly cut injuries besides contusions, also corroborate the eye-
witness account given by P.W.1 Shaik Baba Vali. It may be
@page-SC614
mentioned here that there is no evidence on record to show that there was any enmity
between P.W.1 Shaik Baba Vali and the accused, on account of which he may falsely
implicate them. In fact, the defence has given no suggestion in his cross-examination that
he had any reason to falsely implicate the accused. Thus, from the evidence on record, the
case of the prosecution is fully established. The High Court, therefore, rightly convicted
both the accused. The learned Sessions Judge had committed manifest error of law in
giving too much weight to some minor and insignificant contradictions. The learned
Sessions Judge had also given undue importance to the timings deposed to by P.W.1 in
going from the place of occurrence to the house of the deceased, then going from there to
the police station and getting the FIR scribed. He is not a highly educated person and he
was not expected to have a very accurate idea of timings. The learned Sessions Judge
disbelieved the prosecution case on grounds which were not even worth taking notice of
and were completely divorced from reality. The view taken by the learned Sessions Judge
being wholly perverse, was rightly set aside by the High Court and it was perfectly
justified in convicting both the accused A-1 and A-2.
12. Learned counsel for the appellant has next submitted that the appellant herein viz.
Shaik China Brahmam (A-2) cannot be held liable under section 302 read with Section
34, IPC as the main injuries were given by Shaik Khasim Saida (A-1), who was armed
with a knife and he was responsible for injuries to trachea and occipital region which
proved fatal. He has submitted that the appellant-Shaik China Brahmam (A-2) was armed
with an iron pipe and he did not cause any fatal injury. We are unable to accept the
submission made. It has come in evidence that the pipe with which A-2 was armed was in
the shape of an iron rod and iron rod can also cause fatal injuries. When a criminal act is
done by several persons in furtherance of common intention of all, the other offenders are
liable for that act in the same manner as the principal offender as if the act was done by
such offenders also. In this case, both the accused went jointly to a place where the
deceased had gone for attending the call of nature and they jointly assaulted him. The fact
that A-2 also received injuries in his palm shows that he took active part in snatching the
knife from the hands of the deceased when he had succeeded in snatching it from A-1.
This clearly shows that A-2 shared the common intention with A-1 to cause injuries to the
deceased. The essential conditions for the application of Section 34, IPC are common
intention to commit an offence and participation by all the accused in doing act or acts in
furtherance of that common intention. If these two ingredients are established, all the
accused shall be liable for the said offence. We have no doubt that in the present case
both the ingredients are fully established and, therefore, A-2 is also liable for commission
of the offence. We are, therefore, clearly of the opinion that A-2 is guilty of the offence
under Section 302 read with Section 34, IPC and the High Court rightly convicted and
sentenced him for the said offence.
13. In the result, the appeal fails and is hereby dismissed.
Appeal dismissed.
AIR 2008 SUPREME COURT 614 "New India Assurance Co. Ltd. v. Prabhu Lal"
Coram : 2 C. K. THAKKER AND TARUN CHATTERJEE, JJ.
Civil Appeal Nos. 5539 with 5540-5541 of 2007* (arising out of SLP (C) Nos. 7370,
17794 of 2004, 7618 of 2005), D/- 30 -11 -2007.
New India Assurance Co. Ltd. v. Prabhu Lal.
Motor Vehicles Act (59 of 1988), S.147, S.2(47), S.3 - MOTOR VEHICLES -
INSURANCE - LICENSE - Liability of insurer - Defence of validity of driver's licence -
Vehicle involved in accident driven by driver holding L. M. V. licence - No endorsement
on licence to drive transport vehicle - Permit issued by Transport Authority clearly
showing that vehicle in question was a 'transport vehicle' - Yet holding vehicle to be L.
M. V. on basis of unladen weight and fact that it was not carrying goods at relevant time -
And saddling liability on insurance company - Improper. (Paras 23, 24, 28, 29, 42)
Cases Referred : Chronological Paras
2006 AIR SCW 1649 : AIR 2006 SC 3440 : 2006 (2) AIR Jhar R 662 : 2006 (3) AIR Kar
R 215 35
@page-SC615

2004 AIR SCW 663 : AIR 2004 SC 1531 (Disting.) 41


2003 AIR SCW 3739 : AIR 2003 SC 4161 40
AIR 2000 HP 91 (Approved) 36
1999 AIR SCW 3142 : AIR 1999 SC 3181 (Disting.) 9, 10, 14, 15, 23, 24, 30, 31,
32, 33, 36
1996 AIR SCW 2466 : AIR 1996 SC 2054 (Disting.) 39
AIR 1987 SC 1184 (Disting.) 37, 38
Kishore Rawat and M. K. Dua, for Appellant; Sangram Singh Solanki, Pankaj Kumar
Singh, J. P. N. Gupta, K. L. Janjani, Jagjit Singh Chhabra, Shekhar G. Devasa, B. V. Pinto
and D. K. Garg, for Respondent.
* From Judgment of National Consumer Disputes Redressal Commission, New Delhi in
Revn. Petn. No. 880 of 2002, D/- 17-10-2003.
Judgement
1. C. K. THAKKER, J. :-Leave granted.
2. In all these appeals, a common question of law has been raised by the parties. It is,
therefore, appropriate if we deal with and decide all the appeals by a common judgment.
In all the three appeals, the claim of the claimant has been upheld finally by the National
Consumer Disputes Redressal Commission, New Delhi ('National Commission' for short)
which has been challenged by the Insurance Company in this Court.
3. To appreciate the controversy, it would be appropriate if we narrate the facts in the first
case i.e. New India Assurance Co. Ltd. v. Prabhu Lal.
4. A complaint was filed by the complainant Prabhu Lal under Section 12 of the
Consumer Protection Act, 1986 before the District Consumer Disputes Redressal Forum,
Kota (Rajasthan) ('District Forum' for short) claiming compensation from the respondent-
Insurance Company as also from Tata Finance Limited, Jaipur. The case of the
complainant was that he purchased a vehicle-Tata 709 with Registration No. RJ-20G-
2828 from Tata Finance Limited, Jaipur. The insurance was taken from New India
Assurance Company effective from October 17, 1997 to October 16, 1998. Premium
amount of Rs.8235/- was duly paid. It was the case of the complainant that on April 17,
1998, the vehicle of the complainant was being driven by Mohd. Julfikar to Indore for
getting Chilly. At about 4.30 a.m. in the early morning, the driver of Roadways Bus No.
MP 13-C-3935 drove the bus with very high speed in rash and negligent manner which
resulted in an accident at Yashwant Nagar. Due to said accident, Ram Narain-brother of
the complainant who was sitting with Mohd. Julfikar, sustained injuries. Mohd. Julfikar
immediately ran away leaving the vehicle but as Ram Narain received serious injuries, he
could not come out of the vehicle. The complainant lodged First Information Report
(FIR) No. 131 of 1998 with the Manpur Police Station, Yashwant Nagar, District Indore
under Sections 279 and 337 of the Indian Penal Code (IPC) against driver Kalu of M.P.
Roadways Bus. Vehicle of the complainant was then inspected by Tatas, estimate was
prepared and claim was submitted in the prescribed form by the complainant to the
Insurance Company on June 12, 1998. The amount of the claim was, however, not paid to
the complainant. The complainant, therefore, moved the District Forum praying for an
award of Rs. - 4,70,000/? towards the claim of vehicle, Rs.15,000/- towards mental
agony, Rs. - 5,000/- towards driving charges of the vehicle from Indore to Kota and Rs. -
25,000/- for survey fee.
5. The Insurance Company filed its reply refuting the claim of the complainant.
According to the Company, it had not committed any deficiency in rendering 'service'. It
was also the case of the Company that it had fulfilled all contractual obligations as to
claim. The Company informed the complainant about its decision on December 21, 1999
stating that the claim was not allowable and the amount was not payable. The Insurance
Company, therefore, prayed for the dismissal of the complaint.
6. According to the District Forum, the main question was whether the Insurance
Company was deficient in rendering service and wrongly disallowed insurance claim of
the complainant. The Forum considered the question and heard the parties. According to
the complainant, at the time of accident, vehicle was driven by Mohd. Julfikar who was
having a licence to drive Light Motor Vehicle (LMV) as also Heavy Motor Vehicle
(HMV). In spite of it, the Insurance Company disallowed the insurance claim of the
complainant on the ground that the driver was not having valid driving licence to drive
the vehicle in question. It was also the contention of the complainant that certain
documents produced by the Insurance Company were not genuine. The complainant was
not an educated man and he knew only how to sign. If the officials of Insurance Company
had obtained signatures of the complainant on certain documents without reading over
@page-SC616
to him and making him properly understood, the complainant should not suffer.
According to the complainant, Insurance Company wrongly presumed and proceeded on
the basis that the vehicle was driven by Ram Narain at the time of accident, who was
having a valid driving licence to drive only Light Motor Vehicle and negatived the claim.
It was, therefore, prayed that an award be passed in favour of the complainant.
7. The case of the Insurance Company, on the other hand, was that the vehicle in
question, at the time of accident, was driven by Ram Narain, brother of the complainant.
Admittedly, Ram Narain was possessing licence to drive Light Motor Vehicle and not
Heavy Motor Vehicle. He, therefore, could not have driven Transport Vehicle in absence
of necessary endorsement as required and the Insurance Company could not be held
liable. In this connection, Insurance Company relied on the permit issued by Transport
Authority, the Form submitted by the complainant, licence issued and other documents.
The Insurance Company also relied upon FIR filed at Police Station, Manpur, wherein it
was stated that the vehicle was driven by Ram Narain. Moreover, when the officers of the
Insurance Company approached the complainant, they were informed by the complainant
that the vehicle was driven by Ram Narain. As an afterthought, only with a view to get
the amount of compensation, it was asserted and a case had been put forward before the
Consumer Forum that the vehicle was driven by Mohd. Julfikar. It was contended that the
complainant realized belatedly that if true facts would be placed before the Forum, in
view of legal position, he would not be able to get any amount from the Insurance
Company. It was, therefore, asserted that Mohd. Julfikar was driving the vehicle but it
was not true. The Insurance Company, hence, submitted that there was no deficiency in
rendering service by the Company and the claim was liable to be dismissed.
8. The Tata Finance Limited, Jaipur in its reply stated that the complainant had purchased
the vehicle on the basis of Hire-Purchase Agreement and the amount was to be paid in
instalments. At the time of incident, Rs. 3,65,026/- were due and payable to the Company.
Until the full amount was paid, the Financer was to remain owner of the vehicle. It was
also stated that though Tata Finance Company requested the Insurance Company several
times to make payment of the balance hire-purchase amount, it was not done.
9
. The District Forum, after considering the rival contentions of the parties and referring to
the case law on the point, particularly a decision of this Court in Ashok Gangadhar
Maratha v. Oriental Insurance Co. Ltd., (1999) 6 SCC 620, held that the complainant was
not entitled to compensation. According to the District Forum, in Ashok Gangadhar, this
Court held that if the driver was having effective driving licence to ply Light Motor
Vehicle (LMV), he could not have plied Heavy Motor Vehicle (HMV) or Transport
Vehicle. The District Forum observed that from the evidence on record, it was proved that
at the time of accident, Ram Narain was plying the vehicle in question and not Mohd.
Julfikar as asserted. Ram Narain was having valid and effective driving licence to ply
Light Motor Vehicle and as such he could not have plied the transport vehicle. The claim
was, therefore, not tenable and accordingly the complaint was dismissed. 1999 AIR
SCW 3142

10. Being aggrieved by the order passed by the District Forum, the claimant approached
the Consumer Disputes Redressal Commission of Rajasthan, Jaipur ('State Commission'
for short). The State Commission held that the principle laid down in Ashok Gangadhar
would apply. But according to the State Commission, the District Forum was not right in
dismissing the claim observing that the said decision was against the complainant. In fact,
the point was decided in favour of the complainant and the complainant-claimant would
be entitled to the benefit of the judgment and the Insurance Company must be held liable.
Accordingly, the appeal was allowed. The order passed by the District Forum was set
aside and the Insurance Company was ordered to pay the amount mentioned in the
operative part of the judgment along with interest at the rate of 15% p.a.
11. Aggrieved Insurance Company approached National Forum against the order passed
by the State Commission but the National Commission also dismissed the Revision and
confirmed the order passed by the State Commission. It is this order which is challenged
in this Court.
12. On April 23, 2004, notice was issued
@page-SC617
by the Court. It appears that meanwhile in other matters, a similar question came up
before this Court and hence all the matters were ordered to be placed for hearing together.
13. We have heard learned counsel for the parties.
14

. The learned counsel for the appellant-Insurance Company contended that the State
Forum as well as National Forum had committed an error of law in holding the appellant-
Insurance Company liable and directing it to pay compensation. It was submitted that
there was no deficiency on the part of the appellant-Company in rendering service to the
complainant and hence Consumer Forum had no jurisdiction to entertain, deal with and
decide the dispute. It was also submitted that it was clearly established from the relevant
documents on record that at the time of accident, Ram Narain was plying the vehicle and
not Mohd. Julfikar. Admittedly, Ram Narain was having valid driving licence to ply Light
Motor Vehicle. The vehicle in question was a transport vehicle and hence it could not
have been plied by Ram Narain. In absence of valid licence to drive the said vehicle, the
complainant could not claim compensation from the Insurance Company and no direction
could be issued to the Company to pay compensation to the complainant. The District
Forum was, therefore, fully justified in dismissing complaint of the respondent-
complainant and both, State Commission as well as National Commission - were in error
in granting the prayer of the complainant and the orders passed by them are liable to be
set aside. It was also submitted by the learned counsel that State Commission as also
National Commission, misunderstood Ashok Gangadhar. It is no doubt true that in Ashok
Gangadhar, the claim of the complainant was upheld by this Court. But it was because the
relevant documentary evidence was not placed before the Authorities. This Court,
therefore, held that since material documents were not produced by the Company, the
complainant should not suffer and in absence of such evidence, the Insurance Company
cannot be absolved of liability. But the ratio laid down in Ashok Gangadhar supports the
case of the Insurance Company that if necessary documents are on record and they go to
show that the licence issued in favour of the driver to ply a particular type of vehicle, he
could not have plied other vehicle and the Insurance Company could not be held liable if
there was breach of that condition. In the case on hand, all the documents were on record,
contention was raised by the Insurance Company from the very beginning that the vehicle
was a transport vehicle, which was driven by Ram Narain who was holding licence to ply
only Light Motor Vehicle. Hence, he could not have plied the vehicle in question, a
finding was recorded in favour of the Insurance Company by the District Forum which
had not been disturbed by the State Commission or by the National Commission and
hence the complaint ought to have been dismissed. 1999 AIR SCW 3142

15. The learned counsel for the respondent submitted that it was the case of the
complainant before District Forum that the vehicle was driven by Mohd. Julfikar who
possessed valid licence to ply the vehicle but as soon as the accident took place, he fled
away since he was scared that passengers in the bus might not spare him and he might be
beaten. As Ram Narain sustained several injuries, he could not go away. Unfortunately,
the District Forum dismissed the complaint which necessitated challenging the decision
and the complainant succeeded before the State Forum and National Forum. As to Ashok
Gangadhar, the counsel submitted that the said decision helps the complainant and both
the Commissions were right in following it and in directing the Insurance Company to
pay compensation to the complainant. He, therefore, submitted that the appeal deserves to
be dismissed.
16. Before we deal with contentions raised by the parties on merits, it would be
appropriate to examine the relevant provisions of the Motor Vehicles Act, 1988
(hereinafter referred to as 'the Act'). By the Act of 1988, the Motor Vehicles Act, 1939
(old Act) had been repealed. The new Act has been enacted with a view 'to consolidate
and amend the law relating to motor vehicles'. Section 2 is a 'legislative dictionary' and
defines various terms. Relevant clauses of the said section are Clauses (10), (14), (21),
(28) and (47) which define 'driving licence', 'goods carriage', 'light motor vehicle', 'motor
vehicle' and 'transport vehicle' respectively. They read as under:
2. Definitions.- In this Act, unless the context otherwise requires,
(10) "driving licence" means the licence
@page-SC618
issued by a competent authority under Chapter II authorising the person specified therein
to drive, otherwise than as a learner, a motor vehicle or a motor vehicle of any specified
class or description;
(14) "goods carriage" means any motor vehicle constructed or adapted for use solely for
the carriage of goods, or any motor vehicle not so constructed or adapted when used for
the carriage of goods;
(21) "light motor vehicle" means a transport vehicle or omnibus the gross vehicle weight
of either of which or a motor car or tractor or road-roller the unladen weight of any of
which, does not exceed 7,500 kilograms;
(28) "motor vehicle" or "vehicle" means any mechanically propelled vehicle adapted for
use upon roads whether the power of 1 Subs. and ins. by Act. 580 propulsion is
transmitted thereto from an external or internal source and includes a chassis to which a
body has not been attached and a trailer; but does not include a vehicle running upon
fixed rails or a vehicle of a special type adapted for use only in a factory or in any other
enclosed premises or a vehicle having less than four wheels fitted with engine capacity of
not exceeding thirty-five cubic centimetres;
(47) "transport vehicle" means a public service vehicle, a goods carriage, an educational
institution bus or a private service vehicle;
17. Section 3(1) of the Act requires holding of driving licence which is material and reads
thus:
3. Necessity for driving licence.- (1) No person shall drive a motor vehicle in any public
place unless he holds an effective driving licence issued to him authorising him to drive
the vehicle; and no person shall so drive a transport vehicle other than a motor cab hired
for his own use or rented under any scheme made under sub- section (2) of section 75
unless his driving licence specifically entitles him so to do.
(Emphasis supplied)
18. Section 5 declares that no owner or person in charge of a motor vehicle shall cause or
permit any person which does not satisfy the provisions of Section 3 to drive the vehicle.
Section 10 deals with form and contents of licences. It enacts that every driving licence
(except a driving licence issued under Section 18 which provides for driving motor
vehicles belonged to the Central Government) shall be in such form and shall contain
such information as may be prescribed by the Central Government. It also states that a
driving licence shall be expressed as entitling the driver to drive a motor vehicle of one or
more of the types of motor vehicles specified in sub-section (2). Section 15 provides for
'renewal of driving licences'. Section 27 empowers the Central Government to make rules
in respect of matters enumerated therein. Section 66 prohibits an owner of motor vehicle
to use or to permit the use of motor vehicle as a transport vehicle in any public place save
in accordance with the conditions of permit granted by an appropriate authority. Whereas
Section 147 deals with requirements of policies and limits of liability, Section 149
imposes duty on insurers to satisfy judgments and awards against persons insured in
respect of third party risks.
19. The Central Government has framed rules known as the Central Motor Vehicles
Rules, 1989 (hereinafter referred to as 'the Rules').
20. Rule 16 of the Rules prescribes the form in which driving licence is issued. The form
provides that the holder of a licence can drive any vehicle of the description mentioned
therein. Where authorization is granted to drive transport vehicle, it is expressly so
provided by making an endorsement to that effect.
21. Now, it is the case of the Insurance Company that the vehicle of the complainant
which met with an accident was a 'transport vehicle'. It was submitted that the insured
vehicle was a 'goods carriage' and was thus a 'transport vehicle'. The vehicle was driven
by Ram Narain, who was authorized to drive Light Motor Vehicle and not a transport
vehicle. Since the driver had no licence to drive transport vehicle in absence of necessary
endorsement in his licence to that effect, he could not have driven Tata 709 and when that
vehicle met with an accident, Insurance Company could not be made liable to pay
compensation.
22. Now, let us consider both these points. As far as vehicle is concerned, it is clear from
the record that it was Tata 709, registration No.RJ-20G-2828. The permit in respect of the
said vehicle is on record issued by the Transport Authority, Kota. From the registration, it
is clear that it was registered as a truck, a goods carrier and
@page-SC619
was described as public carrier. Load carrying capacity was shown to be 4100.00 Kgs.
The permit was valid up to November 11, 2002.
23. The District Forum held that the documents clearly mentioned that the vehicle was a
'goods carriage' as defined in Section 2(14) covered by the category of 'transport vehicle'
under Section 2(47) of the Act. The State Commission held that since the gross weight of
the vehicle was only 6800 Kgs. and did not exceed permissible limits (7500 Kgs.) nor it
was carrying goods at the time of accident, it was a Light Motor Vehicle. For coming to
that conclusion, the State Commission relied upon Ashok Gangadhar.

24. In our considered view, the State Commission was wrong in reversing the finding
recorded by the District Forum. So far as Ashok Gangadhar is concerned, we will deal
with the said decision little later but from the documentary evidence on record and
particularly, from the permit issued by the Transport Authority, it is amply clear that the
vehicle was a 'goods carrier' [Section 2(14)]. If it is so, obviously, it was a 'transport
vehicle' falling under clause (47) of Section 2 of the Act. The District Forum was,
therefore, right in considering the question of liability of the Insurance Company on the
basis that Tata 709 which met with an accident was 'transport vehicle'.
25. The second question is as to who was driving the vehicle which collided with M.P.
Roadways Bus on April 17, 1998. In this connection, it may be stated that it was the case
of the complainant that the vehicle (Tata 709) was driven by Mohd. Julfikar to Indore.
Because of rash and negligent driving by Kalu, driver of other vehicle i.e. M.P. Roadways
bus, there was an accident and Ram Narain, brother of the complainant, sustained serious
injuries. Mohd. Julfikar was having valid licence to drive Light Motor Vehicle (LMV) as
well as Heavy Motor Vehicle (HMV) and hence the complainant was entitled to
compensation from the Insurance Company.
26. The contention of the Insurance Company, on the other hand, was that it conducted an
inquiry which revealed that at the time of accident it was not Mohd. Julfikar who was
driving the vehicle, but it was Ram Narain who was driving it. Ram Narain was having
licence to drive Light Motor Vehicle only and since the vehicle in question was a
transport vehicle, he could not have driven the said vehicle in absence of an endorsement
as required by law and hence the complainant was not entitled to any amount from the
Insurance Company and the Insurance Company could not be held liable.
27. The District Forum, as observed earlier, considered the assertion of the complainant
and the defence of the Insurance Company as to who was driving Tata 709 and on the
basis of overall evidence adduced before it, held that it was Ram Narain who was driving
the vehicle that met with an accident. The said Ram Narain was not having licence to
drive transport vehicle and as such, Insurance Company was not liable. The District
Forum noted that in the FIR lodged in respect of the accident, Ram Narain was shown to
be the driver of the vehicle. Not only that but the evidence adduced before the District
Forum also went to show that at the time of accident, Ram Narain was the driver of the
insured vehicle. The argument of the complainant that the officials of the Insurance
Company obtained his signatures on some documents without reading them over and
making the claimant to understand the contents thereof was negatived. The assertion of
the complainant that he was 'illiterate' and was knowing only how to put his signature
was also not believed by the District Forum. The said finding of fact has not been set
aside either by the State Commission or by the National Commission. Even otherwise,
from the evidence on record, we are satisfied that it was Ram Narain who was driving the
vehicle at the time of accident. We have, therefore, to proceed to consider whether the
complainant was entitled to claim compensation from the Insurance Company in such an
eventuality.
28. The argument of the Insurance Company is that at the time of accident, Ram Narain
had no valid and effective licence to drive Tata 709. Indisputably, Ram Narain was
having a licence to drive Light Motor Vehicle. The learned counsel for the Insurance
Company, referring to various provisions of the Act submitted that if a person is having
licence to drive Light Motor Vehicle, he cannot drive a transport vehicle unless his
driving licence specifically entitles him so to do (Section 3). Clauses (14), (21), (28) and
(47) of Section 2 make it clear that
@page-SC620
if a vehicle is 'Light Motor Vehicle', but falls under the category of Transport Vehicle, the
driving licence has to be duly endorsed under Section 3 of the Act. If it is not done, a
person holding driving licence to ply Light Motor Vehicle cannot ply transport vehicle. It
is not in dispute that in the instant case, Ram Narain was having licence to drive Light
Motor Vehicle. The licence was not endorsed as required and hence, he could not have
driven Tata 709 in absence of requisite endorsement and Insurance Company could not be
held liable.
29. We find considerable force in the submission of the learned counsel for the Insurance
Company. We also find that the District Forum considered the question in its proper
perspective and held that the vehicle driven by Ram Narain was covered by the category
of transport vehicle under Clause (47) of Section 2 of the Act. Section 3, therefore,
required the driver to have an endorsement which would entitle him to ply such vehicle.
It is not even the case of the complainant that there was such endorsement and Ram
Narain was allowed to ply transport vehicle. On the contrary, the case of the complainant
was that it was Mohd. Julfikar who was driving the vehicle. To us, therefore, the District
Forum was right in holding that Ram Narain could not have driven the vehicle in
question.
30

. The learned counsel for the complainant, however, heavily relied upon Ashok
Gangadhar. In that case, the appellant was the owner of a truck, Light Motor Vehicle,
which was insured with the respondent Insurance Company. The vehicle met with an
accident and a claim was lodged by the complainant before the Consumer Commission. It
was contended by the Insurance Company that the truck was a goods carriage or a
transport vehicle and since the driver of the truck was holding a driving licence issued in
Form No. 6 to drive light motor vehicle only, he was not authorized to drive transport
vehicle as there was no endorsement on his driving licence authorizing him to drive such
transport vehicle. The aggrieved complainant approached this Court. Allowing the appeal
and setting aside the order passed by the Commission, this Court held that the driver of
the vehicle was holding a valid driving licence for driving a Light Motor Vehicle and
there was no material on record to show that he was disqualified from holding an
effective valid licence at the time of accident. In view of those facts, the Court held that
the policy did not insist on the driver to have a licence to drive a transport vehicle by
obtaining a specific endorsement. Considering the definition of 'Light Motor Vehicle' as
given in Clause (21) of Section 2 of the Act, this Court held that such Light Motor
Vehicle (LMV) cannot always mean a light goods carriage. A Light Motor Vehicle
(LMV) can be a non-transport vehicle as well. The Court proceeded to observe that since
there was neither a pleading nor a permit produced on record, the vehicle remained as a
Light Motor Vehicle. And though it can be said to have been designed to use as a
transport vehicle or a goods carriage, it could not be held on account of statutory
prohibition contained in Section 66 of the Act to be a transport vehicle. It was, therefore,
held that the Commission was not right in rejecting the claim of the claimant.
Accordingly this Court set aside the order passed by the Commission and directed the
Insurance Company to pay compensation to the complainant. 1999 AIR SCW 3142

31. It is no doubt true that in Ashok Gangadhar, in spite of the fact that the driver was
holding valid driving licence to ply Light Motor Vehicle (LMV), this Court upheld the
claim and ordered the Insurance Company to pay compensation. But, in our considered
opinion, the learned counsel for the Insurance Company is right in submitting that it was
because of the fact that there was neither pleading nor proof as regards the permit issued
by the Transport Authority. In absence of pleading and proof, this Court held that, it could
not be said that the driver had no valid licence to ply the vehicle which met with an
accident and he could not be deprived of the compensation. This is clear if one reads
paragraph 11 of the judgment, which reads thus:
"11. To reiterate, since a vehicle cannot be used as transport vehicle on a public road
unless there is a permit issued by the Regional Transport Authority for that purpose, and
since in the instant case there is neither a pleading to that effect by any party nor is there
any permit on record, the vehicle in question would remain a light motor vehicle. The
respondent also does not say that any permit was granted to the appellant for plying the
vehicle as a transport vehicle under Section 66 of the Act, Moreover, on the date of
accident, the vehicle was
@page-SC621
not carrying any goods, and though it could be said to have been designed to be used as a
transport vehicle or goods-carrier, it cannot be so held on account of the statutory
prohibition contained in Section 66 of the Act."
(Emphasis supplied)
32. In our judgment, Ashok Gangadhar did not lay down that the driver holding licence to
drive a Light Motor Vehicle need not have an endorsement to drive transport vehicle and
yet he can drive such vehicle. It was on the peculiar facts of the case, as the Insurance
Company neither pleaded nor proved that the vehicle was transport vehicle by placing on
record the permit issued by the Transport Authority that the Insurance Company was held
liable.
33

. In the present case, all the facts were before the District Forum. It considered the
assertion of the complainant and defence of the Insurance Company in the light of the
relevant documentary evidence and held that it was established that the vehicle which
met with an accident was a 'transport vehicle'. Ram Narain was having a licence to drive
Light Motor Vehicle only and there was no endorsement as required by Section 3 of the
Act read with Rule 16 of the Rules and Form No. 6. In view of necessary documents on
record, the Insurance Company was right in submitting that Ashok Gangadhar does not
apply to the case on hand and the Insurance Company was not liable. 1999 AIR
SCW 3142

34. The matter can be looked from another angle also. Section 14 referred to above,
provides for currency of licence to drive motor vehicles. Sub-section (2) thereof
expressly enacts that a driving licence issued or renewed under the Act shall, "in the case
of a licence to drive a transport vehicle, be effective for a period of three years". It also
states that "in the case of any other licence, if the person obtaining the licence, either
originally or on renewal thereof, had not attained the age of fifty years on the date of
issue or, as the case may be, renewal thereof, be effective for a period of twenty years
from the date of such issue or renewal". It is thus clear that if a licence is issued or
renewed in respect of a transport vehicle, it can be done only for a period of three years.
But, in case of any other vehicle, such issuance or renewal can be for twenty years
provided the person in whose favour licence issued or renewed had not attained the age
of 50 years. In the present case, the licence was renewed on November 17, 1995 up to
November 16, 2015 i.e. for a period of twenty years. From this fact also, it is clear that
the licence was in respect of 'a motor vehicle other than the transport vehicle'.
35

. The learned counsel for the Insurance Company also referred to a decision of this Court
in National Insurance Company v. Kusum Rai and Ors., (2006) 4 SCC 250, wherein this
Court held that if the vehicle is a taxi which is being driven by a driver holding licence
for driving Light Motor Vehicle only without there being any endorsement for driving
transport vehicle, the Insurance Company cannot be ordered to pay compensation. 2006
AIR SCW 1649

36

. We may also refer to a decision of the High Court of Himachal Pradesh in New India
Assurance Co. Ltd., Shimla v. Suraj Prakash and Ors., AIR 2000 HP 91. There the vehicle
involved in an accident was taxi, a public service vehicle. But the licence issued in favour
of the driver was to ply light motor vehicle and there was no endorsement to drive
transport vehicle. It was, therefore, held by the High Court that the Insurance Company
cannot be saddled with the liability to pay compensation to the claimant. There too, the
claimant placed reliance on Ashok Gangadhar. The Court, however, distinguished it
observing that "there was neither any evidence therein nor was there any claim for insurer
that the vehicle concerned therein was having a permit for goods carriage or that it had a
permit or authorization for plying the vehicle as a transport vehicle". In our considered
view, the High Court was right in taking the above view. 1999 AIR SCW 3142

37

. The learned counsel for the complainant invited our attention to certain decisions of this
Court. In Skandia Insurance Co. Ltd. v. Kokilaben Chandravadan and Ors., (1987) 2 SCC
654, it was held that if a truck driver leaves the truck with engine in motion after handing
over the truck to cleaner who was not a duly licensed person who drives the truck which
causes an accident, it cannot be contended by the Insurance Company that it would not be
liable to pay compensation to a third party who sustains injury because of the accident.
AIR 1987 SC 1184

38. The ratio laid down in Skandia Insurance Co. Ltd., in our considered
@page-SC622
opinion, does not apply to the case on hand as it was in respect of 'third party' that the
Court held that the Insurance Company must pay compensation. This is clear from
paragraph 13 of the judgment in which the Court stated:
"13. In order to divine (sic derive) the intention of the legislature in the course of
interpretation of the relevant provisions there can scarcely be a better test than that of
probing into the motive and philosophy of the relevant provisions keeping in mind the
goals to be achieved by enacting the same. Ordinarily it is not the concern of the
legislature whether the owner of the vehicle insures his vehicle or not. If the vehicle is
not insured any legal liability arising on account of third party risk will have to be borne
by the owner of the vehicle. Why then has the legislature insisted on a person using a
motor vehicle in a public place to insure against third party risk by enacting Section 94.
Surely the obligation has not been imposed in order to promote the business of the
insurers engaged in the business of automobile insurance. The provision has been inserted
in order to protect the members of the Community travelling in vehicles or using the
roads from the risk attendant upon the user of motor vehicles on the roads. The law may
provide for compensation to victims of the accidents who sustain injuries in the course of
an automobile accident or compensation to the dependents of the victims in the case of a
fatal accident. However, such protection would remain a protection on paper unless there
is a guarantee that the compensation awarded by the Courts would be recoverable from
the persons held liable for the consequences of the accident. A Court can only pass an
award or a decree. It cannot ensure that such an award or decree results in the amount
awarded being actually recovered, from the person held liable who may not have the
resources. The exercise undertaken by the law Courts would then be an exercise in
futility. And the outcome of the legal proceedings which by the very nature of things
involve the time cost and money cost invested from the scarce resources of the
Community would make a mockery of the injured victim, or the dependents of the
deceased victim of the accident, who themselves are obliged to incur not inconsiderable
expenditure of time, money and energy in litigation. To overcome this ugly situation the
legislature has made it obligatory that no motor vehicle shall be used unless a third party
insurance is in force. To use the vehicle without the requisite third party insurance being
in force is a penal offence (Section 94 of the Motor Vehicles Act). The legislature was
also faced with another problem. The insurance policy might provide for liability walled
in by conditions which may be specified in the contract of policy. In order to make the
protection real, the legislature has also provided that the judgment obtained shall not be
defeated by the incorporation of exclusion clauses other than those authorised by Section
96 and by providing that except and save to the extent permitted by Section 96 it will be
the obligation of the Insurance Company to satisfy the judgment obtained against the
persons insured against third party risks. (vide Section 96). In other words, the legislature
has insisted and made it incumbent on the user of a motor vehicle to be armed with an
insurance policy covering third party risks which is in conformity with the provisions
enacted by the legislature. It is so provided in order to ensure that the injured victims of
automobile accidents or the dependents of the victims of fatal accidents are really
compensated in terms of money and not in terms of promise. Such a benign provision
enacted by the legislature having regard to the fact that in the modern age the use of
motor vehicles notwithstanding the attendant hazards, has become an inescapable fact of
life, has to be interpreted in a meaningful manner which serves rather than defeats the
purpose of the legislation. The provision has, therefore, to be interpreted in the twilight of
the aforesaid perspective."
39

. Similar is the reasoning and conclusion in B. V. Nagaraju v. M/s. Oriental Insurance Co.
Ltd., (1996) 4 SCC 647. In that case, there was breach of condition as to carry passengers
in a goods vehicle more than the number permitted in terms of insurance policy. The
Court there held that the breach of the said provision could not be said to be such a
fundamental character so as to afford ground to the insurer to deny indemnification unless
there were some factors which contributed to the causing of the accident. The Court held
that exclusionary provision in the insurance policy must be retained so as to serve the
main purpose of the policy which was to indemnify the damage caused 1996 AIR
SCW 2466

@page-SC623
to the vehicle.
40

. In Jitendra Kumar v. Oriental Insurance Co. Ltd. and Anr. (2003) 6 SCC 420, the Court
held that if the vehicle was damaged due to accidental fire, the fact that the driver was not
holding valid driving licence at the time of incident would not empower the Insurance
Company to repudiate the claim and it could not be put forward as a ground to deny the
liability of the Insurance Company that the driver did not have valid licence at the time of
accident in question. 2003 AIR SCW 3739

41
. Finally, a reference was made to National Insurance Co. Ltd. v. Swaran Singh and Ors.
(2004) 3 SCC 297. That case also related to third party victims of motor vehicle accidents
and to us the ratio in Swaran Singh does not carry the case of the claimant further. 2004
AIR SCW 663

42. For the aforesaid reasons, in our opinion, the conclusion arrived at by the District
Forum cannot be said to be faulty and it was right in holding that on the basis of the
evidence adduced by the Insurance Company, the complainant was not entitled to claim
any compensation from the Insurance Company and Insurance Company cannot be held
liable. The decision could not have been interfered with by the State Commission or by
the National Commission and hence the orders of the State Commission and National
Commission are liable to be set aside by restoring the order passed by the District Forum.
we do accordingly.
43. The appeal is, therefore, allowed. The orders passed by the State Commission and
National Commission are set aside and the order passed by the District Forum is restored.
44. In the matter of Nasir Ahmed (SLP No. 7618 of 2005), the vehicle was a luxury taxi -
passenger carrying commercial vehicle. There also the driving licence issued in favour of
the driver was to ply Light Motor Vehicle (LMV) and hence the driver could not have
driven the vehicle in question. In that case too, the licence was renewed for a period of
twenty years i.e. from February 5, 2000 to February 4, 2020. Again, there was no
endorsement as required by Section 3 of the Act. A specific plea was taken by the
Insurance Company but the Authorities held the Insurance Company liable which could
not have been done. The reasoning and conclusion arrived at by us in the matter of
Prabhu Lal (SLP No. 7370 of 2004) would apply to the case of Nasir Ahmed. That appeal
is, therefore, allowed.
45. In Chandra Prakash Saxena (SLP No. 17794 of 2004), the vehicle involved in
accident was a Jeep Commander made by Mahindra and Mahindra, a passenger carrying
commercial vehicle, and in view of the fact that the driver was holding licence to drive
Light Motor Vehicle (LMV), he could not have plied the vehicle in question. For the
reasons recorded hereinabove in the main matter of Prabhu Lal i.e. SLP(C) No. 7370 of
2004, the Insurance Company could not have been held liable and that appeal also
deserves to be allowed.
46. For the foregoing reasons, all the three appeals are allowed and the orders passed
against the Insurance Company are set aside holding that the Insurance Company cannot
be held liable. There shall, however, be no order as to costs.
Appeals allowed.
AIR 2008 SUPREME COURT 623 "State of Rajasthan v. Pareshar Soni"
(From : AIR 2005 Rajasthan 271)
Coram : 2 G. P. MATHUR AND ALTAMAS KABIR, JJ.
Petition for Special Leave to Appeal (C) No. 24912 of 2005, D/- 20 -11 -2007.
State of Rajasthan and Ors. v. Pareshar Soni.
Rajasthan Municipalities Act (38 of 1959), S.173A (Before its amendment in 1999) -
MUNICIPALITIES - SALE DEED - AMENDMENT - LAND - Change of user -
Charges, for - Respondent acquiring title to land in question by registered sale deed in
year 1987 - Predecessor-in-interest of respondent acquiring ownership and possession of
property pursuant to decree passed by High Court - Plea by respondent that land was
never allotted to her predecessor by Municipality or State Govt., remaining unrebutted -
Amended S. 173-A thus, restricting user of land for purpose other than that for which it
was originally allotted or sold by Municipality or Corporation - Would not apply -
Consequently, demand by Municipality for conversion charges from respondent - Cannot
be made. (Paras 14, 15, 16, 18, 19)

Ms. Madhurima Tatia and Aruneshwar Gupta, for Appellants; Rajendra Singhvi and
Ashok Kumar Singh, for Respondent.
@page-SC624

Judgement
1. ALTAMAS KABIR, J. :-Since the issue to be decided in this special leave petition is
limited to the interpretation of Section 173-A of the Rajasthan Municipalities Act, 1959
(hereinafter referred to as 'the Act'), it was decided to hear out the special leave petition at
the admission stage itself.
2. The facts involved in the special leave petition, in brief, are as follows:
3. The respondent herein, Pareshar Soni, purchased an old house situated at Moti Chowk,
Jodhpur, from one Smt. Shanti Kumari Lodha by a registered sale deed dated 15.5.1987.
It appears from the judgment forming the subject-matter of the special leave petition that
Smt. Shanti Kumari acquired the ownership and possession of the said property pursuant
to a judgment and decree passed by the Calcutta High Court on 6.6.1972 in Civil Suit No.
867 of 1934. After purchasing the property the said respondent filed an application before
the Municipal Corporation, Jodhpur, under Section 170(1) of the Act seeking permission
to construct a residential house and a few shops on the said property. Inasmuch as, the
Municipal Corporation did not take any step on the said application, a notice was sent on
behalf of the respondent herein to the Corporation under Section 170(8) of the Act
providing for a response within 15 days of receipt of the notice. It appears that the
Corporation did not pay any heed even to the said notice, and accordingly, by invoking
the deeming clause the respondent started construction work on the said property.
4. On 24th May, 1997 the Municipal Corporation raised a demand upon the respondent
for a sum of Rs.1,66,874/- by way of conversion charges and compounding fees. The
respondent objected to the said demand and filed a representation challenging its validity
on the ground that the property in question is nearly 200 years old and reference thereto
had been made in a Patta of the year 1865 pertaining to a neighbouring house, showing
the property to be a free-hold property. Consequently, according to the respondent, no
commercial charges could be levied thereupon. The respondent, however, deposited a
sum of Rs.1,619 towards construction fees. As the representation filed by the respondent
did not yield any result, the respondent was compelled to file a writ petition before the
Jodhpur Bench of the Rajasthan High Court, being SBC Writ Petition No. 2159 of 1997.
The Municipal Corporation, Jodhpur, which is the petitioner No. 2 in the present special
leave petition filed its reply contending that Section 173-A of the Act permits the State
Government to allow any person to use the land for the purposes other than for which it
was originally allotted. It was also contended that the regulation of the areas, markets etc.
were all within the compass of the Act and the 'Patta' said to have been issued by the
earlier State would also remain subject to the Act. One other stand which was taken
before the learned Single Judge was that the respondent/writ petitioner had failed to
produce a copy of the Patta for the property in question to indicate the rights and the
conditions of usage of the land provided therein. Upon hearing the parties the learned
Single Judge by his judgment and order dated 19th July, 1999 dismissed the writ petition
solely on the ground of non-production of the Patta by the respondent herein.
5. The respondent thereupon filed a Special Appeal before the Division Bench of the
aforesaid High Court, being DB (Civil) Special Appeal No.1109/1999. Before the
Division Bench the main contention advanced on behalf of the respondent herein was that
the property in dispute was a free-hold Hindu Undivided Family (HUF) property which
was nearly 2 centuries old and at the relevant point of time the erstwhile State had
allotted the same on a free-hold basis. Consequently, the provisions of Section 173-A of
the Act would not apply in the instant case.
6. It was also contended on behalf of the respondent that the impugned demand notice
was illegal and without any legal validity because the said property had neither been
allotted nor sold to the respondent or her predecessor-in-interest by the Municipal
Corporation or the State Government, as contemplated in Section 173-A of the Act, and
as such no conversion charges or compounding fees could be levied and demanded by the
Municipal Authorities.
7. While deciding the appeal, the Division Bench on a bare reading of Section 173-A of
the Act observed that in order that the said section should apply, two conditions were
necessary : (i) that the land has been allotted or sold to any person by the Municipal
Corporation or the State Government, and (ii) that the allotment or sale of
@page-SC625
the land by the Municipal Corporation or the State Government was subject to the
condition of limiting its use for a particular purpose. The Division Bench also observed
that if either of the two conditions were not fulfilled the provisions of Section 173-A
would not apply. Basing its decision on the contention advanced on behalf of the
respondent that the said property had never been sold or allotted to them being a HUF
property, the Division Bench affirmed the view taken by the Single Judge and, after going
through the registered sale deed, came to the conclusion that the property had neither
been allotted nor sold by the Municipal Corporation or the State Government and that too
for specific purpose. The Division Bench also came to a finding that there was no
evidence on record to suggest that the land in question had come into the hands of
respondents herein with certain conditions limiting its use. Noting that the averments in
the writ petition stood unrebutted the Division Bench also concluded that it had been
established that the respondent herein acquired the ownership and possession over the
land by the registered sale deed without any condition or restraint on its use. On the basis
of its aforesaid finding, the Division Bench allowed the appeal filed by the respondent
herein and set aside the order of the single Judge. Consequentially, the writ petition filed
by the respondent herein was allowed.
8. On behalf of the petitioners it was submitted that the only point which required an
answer in the instant special leave petition was whether the provisions of sub-section (4)
of Section 173-A of the Act which had been introduced by amendment in 1999 would
apply to the case of the respondent when admittedly she had acquired title to the property
on 15th May, 1987 long prior to the said amendment. It was submitted that since the
amendment had been effected, the Municipal Corporation was entitled to raise a demand
on the basis thereof on account of the change of user of the land by the respondent.
9. It was also submitted that the single Judge had not committed any error in dismissing
the writ application on the ground that the writ petitioner/respondent herein had failed to
produce the 'Patta' of the property which would have clearly indicated the use to which
the said property could be applied. In the absence of any evidence to the contrary,
produced by the writ petitioner/respondent herein, the single Judge had very rightly held
that it was not possible to accept the bald statement made on behalf of the writ
petitioner/respondent herein that the property had not been allotted or sold to her
predecessor-in-interest either by the Municipal Corporation or the State Government.
10. The submissions made on behalf of the appellant were controverted on behalf of the
respondent and it was urged that the registered sale deed by which the respondent herein
had acquired the property had not indicated any condition relating to specific user of the
property in question. Apart from the above, reference had been made to a 'Patta' of 1965
issued in favour of the owner of the adjoining property which indicated that the said
property was free-hold on which there could be no levy of commercial charges. The said
statement of fact had not been rebutted by the petitioner and the representation filed by
the respondent remained unattended to and unanswered by the petitioner. It was
submitted on behalf of the respondent that in the absence of any evidence produced on
behalf of the Municipal Corporation, the Division Bench had not committed any error in
coming to the conclusion that the respondent had been able to establish that she had
acquired ownership and possession over the property by a registered sale deed without
any condition placing any limitation on its use.
11. On the question of application of sub-section (4) of Section 173-A, as introduced in
the Act by the amendment of 1999, it was submitted that, in any event, the same could
have no application since the respondent by raising a residential house and shops had not
violated any of the provisions of the Act in force at the relevant time, since there was no
restriction on the user of the land when it was acquired by the respondent.
12. In view of the difference in the unamended provisions of Section 173-A and the
amendments introduced therein by the Rajasthan Municipalities (Amendment) Act, 1999,
it would be appropriate at this stage to set out the unamended provisions of Section 173-
A as well as the amendments effected therein by the aforesaid Amending Act of 1999.
13. Section 173-A of the Act prior to its amendment was as follows:
@page-SC626
173-A. Power of the State Government to allow change in the use of land-
(1) Notwithstanding anything contained in this Act, where any land has been allotted or
sold to any person by a municipality or the State Government subject to the condition of
restraining its use for a particular purpose, the State Government may, if it is satisfied so
to do in public interest, allow the owner or holder of such land to use it for any other
purpose other than the purpose for which it was originally allotted or sold, on payment of
such conversion charges as may be prescribed :
Provided that the rates of conversion charges may be different for different areas and for
different purposes.
(2) The conversion charges so realised shall be credited to the Consolidated Fund of the
State or to the fund of the Municipality as may be determined by the State Government.
(3) Such charges shall be the first charge on the interest of the person liable in the land
the use of which has been changed and shall be recoverable as arrears of land revenue.
Section 173-A of the Act as amended by the Amending Act of 1999 reads as follows:
173-A. Restriction on change of use of land and power of the State Government to allow
change of use of land-
(1) No person shall use or permit the use of any land situated in any municipal area, for
the purpose other than that for which such land was originally allotted or sold to any
person by the State Government, any Municipality, any other local authority in
accordance with any law for the time being inforce or, otherwise than as specified under a
Master Plan, wherever it is in operation.
(2) In the case of any land not allotted or sold as aforesaid and not covered under sub-
section (1), no person shall use or permit the use of any such land situated in a municipal
area for the purpose other than that for which such land use was or is permissible, in
accordance with the Master Plan, wherever it is in operation, or under any law for the
time being in force.
(3) Notwithstanding anything contained in sub-section (1) or sub-section (2), the State
Government or any authority authorised by it by notification in the Official Gazette, may
allow the owner or holder of any such land to have change of use thereof, if it is satisfied
so to do in public interest, on payment of conversion charges at such rates and in such
manner as may be prescribed with respect to the following changes in use :-
(i) From residential to commercial or any other purpose; or
(ii) From commercial to any other purpose; or
(iii) From industrial to commercial or any other purpose; or
(iv) From cinema to commercial or any other purpose:
Provided that rates of conversion charges may be, different for different areas and for
different purposes.
(4) Any person who has already changed the use of land in violation of the provisions of
this Act in force at the time of change of use, shall apply to the State Government or any
authority authorised by it under sub-section (3), within six months from the date of
commencement of the Rajasthan Municipalities (Amendment) Act, 1999 (Act No.19 of
1999) for regularisation of said use and upon regularisation of the change of use of land
he shall deposit the amount contemplated under sub-section (3).
14. From the submissions made on behalf of the respective parties and the materials on
record, admittedly the respondent herein acquired title to the property in question by a
registered sale deed dated 15.5.1987. It is also not denied that her predecessor-in-interest
acquired ownership and possession of the property pursuant to a judgment and decree
passed by the Calcutta High Court on 6.6.1972 in Civil Suit No. 867 of 1934.
15. There is also no dispute that the respondent filed an application under Section 170(1)
of the Act on 5.1.1996 seeking permission to construct a residential house and shops on
the property in question and not having received any response thereto, a notice was given
on her behalf under Section 170(8) of the Act, which was also not replied to and
consequently on the basis of the deeming provision the respondent started construction
invoking such deeming clause contained in sub-section (8) of Section 170 itself.
16. While it is true that the respondent
@page-SC627
had not produced a copy of the 'Patta' for the property in question she had all along
contended that the property in question had never been allotted to her predecessor-in-
interest either by the Municipal Corporation or by the State Government, which stand
stood unrebutted on behalf of the petitioner. There is also no denial by the petitioner that
the property had been acquired by the predecessor-in-interest of the respondent by virtue
of a decree passed by the Calcutta High Court in respect of the ancestral properties of the
parties to the suit.
17. In such circumstances, we do not think it will be proper for this Court in the special
leave petition to once again embark on an inquiry, without any evidence on record, as to
whether the property had been allotted either by the Municipal Corporation or the State
Government. In either case, some record would have been maintained either by the
Municipal Corporation or the State Government, which was not produced either before
the Court in the writ proceedings or before us.
18. We, therefore, have to accept the conclusion of the Division Bench that the property
had neither been allotted by the Municipal Corporation or by the State Government or
that any restriction had been placed on its user. Consequently, the question of demanding
conversion charges for change of user would also not arise and the amended provisions of
sub-section (4) of Section 173-A would also have no application to the facts of the case,
since it is controlled by the very opening words that no person shall use or permit the use
of any land situated in any municipal area, for the purpose other than that for which such
land was originally allotted or sold to any person by the State Government. If the basis on
which sub-section (4) of Section 173-A could be applied, is not available to the petitioner
the demand raised by it towards conversion charges also is not maintainable.
19. We, therefore, have no hesitation in holding that Section 173-A as amended in 1999
would not apply to the case of the respondent and the Division Bench of the Rajasthan
High Court at Jodhpur had correctly allowed the appeal by the respondent.
20. In that view of the matter, the instant special leave petition is dismissed, but there
shall be no order as to costs.
Petition dismissed.
AIR 2008 SUPREME COURT 627 "Murugan v. State"
(From : Madras)
Coram : 2 S. B. SINHA AND H. S. BEDI, JJ.
Criminal Appeal No. 1276 of 2005, D/- 4 -12 -2007.
Murugan and Ors. v. State.
Penal Code (45 of 1860), S.300 - MURDER - Murder - Infliction of single injury - By
itself not relevant factor to hold that accused had no intention to cause murder of
deceased.
Infliction of a single injury by itself is not a relevant factor to hold that the assailant had
no intention to cause murder of the deceased. What is important in a case of this nature is
to consider the entire circumstances to arrive at one conclusion or the other. When a
group of people come with an intention to assault particular person(s), with dangerous
weapon, the same would attract to principles laid down in AIR 1956 SC 465. Injured
prosecution witnesses testified in regard to the intention of the accused to cause the death
of the deceased. Thus, conviction of the accused for murder could not be interfered with.
(Paras 13, 14, 17)
Cases Referred : Chronological Paras
2007 AIR SCW 5230 : 2007 Cri LJ 4294 (Ref.) 13
AIR 1958 SC 465 : 1958 Cri LJ 818 (Rel. on) 11, 13
Shekhar Naphade, Sr. Advocate, T. Raja, Gopal Singh, Atulesh Kumar and Vishal
Jogdand, for Appellants; V. Kanakaraj, Sr. Advocate, V. G. Pragasam, S. Joseph Aristotle
and S. Prabhu Ramasubramanian, for Respondent.
Judgement
1. S. B. SINHA, J. :- This appeal is directed against a judgment and order dated
14.10.2004 passed by a Division Bench of the Madras High Court, Madurai Bench in
Criminal Appeal No.115 of 1997 whereby and whereunder the appeal preferred by the
appellants herein against a judgment of conviction and sentence dated 10.2.1997 passed
by the Additional District and Sessions Judge-cum-Chief Judicial Magistrate,
Ramanathpuram convicting the accused No. 7 under Section 302 of the Indian Penal
Code and sentencing him to undergo life imprisonment and accused Nos. 8, 9 and 10
under Section 324 of the Indian Penal Code and sentencing them to undergo one year's
rigorous imprisonment was dismissed.
@page-SC628
2. A quarrel ensued between the prosecution party and the accused on 10.7.1973 in regard
to drawing of water from a well. Accused were said to have been bearing grudge towards
the deceased as a criminal case was instituted against them at Kumuthi Police Station. On
4.8.1993, at about 6.15 p.m., a procession of villagers was taken out for celebrating a
festival known as "Mulaipari" festival. It started at village Keelamathupatti. When the
procession reached near the village known as Gandaru, with a view to commit murder of
PW-5, Nagarajan and other persons, the accused persons who were 11 in number
allegedly formed an unlawful assembly with weapons like knife, cycle chains and sticks
in their hands. Accused No.1, Subramanian, and accused No. 3, Selvaraj, are said to have
instigated others to commit murder of PW-5 Nagarajan consequent whereupon accused
No. 4, Ramu, attacked him with a stick causing a fracture on his right hand. When
deceased Kannan tried to obstruct him from doing so, accused No. 4, Ramu, instructed
accused No.2, Selvaraj, to kill him also. Consequently, accused No.2, Selvaraj, caught
hold of the hands of Kannan, whereas accused No.6 caught hold of his shoulders. Taking
advantage of the said situation, the appellant No.1 herein (accused No.7 - Murugan) is
said to have stabbed him with a knife on his chest. Appellant No.1 is said to have caused
an injury on PW-1, Chithiraichamy, also with his knife, whereas accused No.8 attacked
PW-1 with cycle chain and caused an injury on his head thereby. Appellant No.3-
Selvaraj, (accused No.9) is said to have attacked PW-3, Kathirvel, with a knife on his
nose and head whereas accused No.10, Boomi, attacked PW-3 with a cycle chain and
caused injuries to him. PW-2, Murugesan was also attacked with a stick.
3. A First Information Report was lodged immediately after the said occurrence. Out of
11 accused, however, only accused Nos. 7 and 8 to 10, were convicted and sentenced in
the manner, as noticed hereinbefore, whereas others were acquitted. An appeal preferred
thereagainst by the appellants herein has been dismissed by reason of the impugned
judgment.
4. Mr. Naphade, learned senior counsel, in support of the appeal, would, inter alia, submit
:
(1) Appellant No.1 having inflicted only one blow with a knife on the deceased, the
offence, if any, committed by him falls under Section 304, Part II of the Indian Penal
Code and not under Section 302 thereof.
(2) Appellant No.2 (accused No.8), having examined defence witnesses to prove his plea
of alibi, the learned Sessions Judge, as also the High Court, committed a serious error in
recording a judgment of conviction against him.
In this connection our attention has been drawn to the following purported findings of the
learned Sessions Judge :
"The 8th accused has not committed the offence punishable under Section 324, IPC."
(3) All the other accused having caused only simple injuries, sentence of one year's
rigorous imprisonment is on the higher side.
5. Mr. V. Kanakaraj, learned senior counsel appearing on behalf of the State, on the other
hand, supported the impugned judgment.
6. The prosecution, in support of its case, examined twenty one witnesses.
The nature of evidence of the eye-witnesses to the occurrence whereupon strong reliance
has been placed by both the learned Sessions Judge as also the High Court being identical
in nature, we would notice the deposition of PW-1, Chithiraichamy, only. He spoke about
the incident which took place on 4th August, 1993. A complaint was made to him by a
woman named Malathi. She was reprimanded by him stating that there should not quarrel
over drawing of water. She was sent back to her home. Malathi was the daughter of the
accused No.2. While the deceased and the injured persons including the said PW-1 had
been participating in the said procession, accused Nos.1 to 3 came and gave an
exhortation to cut and kill Nagarajan. Accused No.4, Ramu, is said to have assaulted in
his right forearm. The deceased Kannan intervened. He asked them not to do so,
whereupon accused No.2 gave an exhortation to kill him, pursuant whereto accused No.6,
Challaiah caught hold of both his shoulders as a result whereof, he could not make a
move. Appellant No.1, Murugan, stabbed him on the left side of his chest. While PW-1
asked the assailants not to do so, Appellant No.1 attempted to stab him also on his neck
with a knife but as he turned his neck on the right side, a laceration was caused on the
right side of his neck. Accused No.8 also
@page-SC629
attacked him with a cycle chain on his head.
7. He wanted to cause an injury to appellant No.2 (accused No.8) with a small knife but
he was prevented from doing so by one Palaniammal as a result whereof Palaniammal
suffered a laceration in his right hand. Appellant No.3, Sathiah (accused No.9) stabbed
Kathirvelu in his hand with a knife as a result whereof he received an injury. Accused
No.10, Boomi, assaulted Kathirvelu with a cycle chain on his head resulting in causing
bleeding injury on his head and the right side nose. Accused No.11 Ramamoorthi
(accused No.4) assaulted witness Murugasan with a stick on his right upper arm
whereupon all persons shouted whereafter the accused ran away.
8. He, not only, thus, described the incident in great details but also identified the material
objects which were seized at the instance of the accused persons being the weapons of
offence. Evidence of PW-2 and other witnesses are also on the same vein.

9. Appellants had a motive. They came in a group. All of them were armed. Both parties
are related to each other. An occurrence had taken place which took place on 10th July,
1993 giving rise to initiation of a criminal case. They, with a view to take revenge, caused
murder of Nagarajan and injuries on some of the prosecution witnesses. The prosecution
witnesses were injured witnesses.
10. No cogent argument has been advanced by Mr. Naphade so as to enable him to hold
that the said witnesses are not trustworthy. They have been believed by the courts below.
Nothing has been pointed out to us as to why we should differ with the said findings.
11. Intention on the part of a person to commit murder must be gathered from the
backdrop of events and the circumstances attending thereto.
A similar question came up for consideration before this Court in Virsa Singh v. State of
Punjab [1958 AIR SC 465], wherein this Court, upon a detailed analysis of the provisions
of Sections 299 and 300 of the Indian Penal Code opined that in order to attract "thirdly"
contained in Section 300 of the Indian Penal Code, it must be established :
"To put it shortly, the prosecution must prove the following facts before it can bring a
case under Section 300 "thirdly";
First, it must establish, quite objectively, that a bodily injury is present;
Secondly, the nature of the injury must be proved; These are purely objective
investigations.
Thirdly, it must be proved that there was an intention to inflict that particular bodily
injury, that is to say, that it was not accidental or unintentional, or that some other kind of
injury was intended.
Once these three elements are proved to be present, the enquiry proceeds further and,
Fourthly, it must be proved that the injury of the type just described made up of the three
elements set out above is sufficient to cause death in the ordinary course of nature. This
part of the enquiry is purely objective and inferential and has nothing to do with the
intention of the offender."
Once the aforementioned factors are established, absence of any knowledge that an act of
that kind would likely to cause death become immaterial. The intention to cause the
bodily injury, if proved, the rest of the enquiry would be purely objective and the only
question is whether as a matter of purely objective inference, the injury is sufficient in the
ordinary course of nature to cause death.
12. The medical evidence, emanating from the deposition of PW-12, Dr.
Meenakshisundaram, reveals :
"I found the following external injuries on the body. (1) A cut injury measuring 1 x - x 3"
on the left side chest. It was in the outer aspect of the 6th left side rib bone between the
middle of the collar bone. On opening the wound, the wound has injured the intercostals
muscles and blood vessels in the middle rib bones. It has punctured the left ventricle. The
injury was slanting and upwards and forwards. It has pierced the left ventricle. There was
1½ litres of colour changed blood in the thoracic cavity. The internal injuries were as
follows :
There was no fracture of the head and rib bones. Heart was empty and pale and was
weighing 250 grams. The left ventricle was punctured. The lungs was pale and was
weighing 500 grams, and was wet when pressed. Abdomen was normal, stomach was
containing digested food. Liver weighed 1450 grams and was pale and was wet when
pressed. Spleen was also wet when pressed and weighed 165 grams, and was pale.
@page-SC630
Kidneys were pale and moist on pressure. Intestines empty. Bladder empty. There was no
fracture on the pelvis. Hyoid bone was intact and the skull was also intact. Skull
membranes were intact. Brain was pale and weighed 1450 grams. There was no intra-
cranial haemorrhage in the brain and was weighing 1250 grams. There was no fracture in
the spinal cord."
13. Infliction of a single injury by itself is not a relevant factor to hold that the assailant
had no intention to cause murder of the deceased. What is important in a case of this
nature is to consider the entire circumstances to arrive at one conclusion or the other.
When a group of people come with an intention to assault particular person(s), with
dangerous weapon, the same would attract to principles laid down in Virsa Singh (supra).
Prosecution witnesses testified in regard to their intention.

Virsa Singh's case has been followed by this Court in Anil v. State of Haryana [2007 (7)
SCALE 56]. AIR 1958 SC 465
2007 AIR SCW 5230

14. We, therefore, are unable to accede to the submissions of Mr. Naphade that appellant
No.1 had no intention to cause death of Nagarajan.
Although, in this case, there was enough material to convict other appellants for
commission of an offence under Section 302/34 of the Indian Penal Code, unfortunately,
the appellants, other than the first appellant, have been only convicted under Section 324
of the Indian Penal Code. As the State did not prefer any appeal thereagainst, this Court is
unable to alter the judgment of conviction and sentence.
15. The learned Trial Judge appears to have committed more than one mistake in his
judgment. He, while analysing the evidence of DW-1, who was examined on behalf of
appellant No.2 (accused No.8) to prove alibi on his part, although came to the conclusion
that the said evidence was not acceptable, as indicated hereinbefore, arrived at the finding
that the 8th accused has not committed the offence under Section 324 of the Indian Penal
Code. Evidently, a typographical error has crept in as otherwise he could not have
convicted him along with appellants No.3 and 4 for commission of an offence under
Section 324 of the Indian Penal Code. We may also notice that whereas in the beginning
of the judgment, he recorded the sentence imposed upon appellants No.2 to 4 to undergo
rigorous imprisonment for three years, in the operative part of the judgment, he directed
that they were sentenced to undergo rigorous imprisonment for one year only. We have,
however, no option but to hold that the later part of the judgment should be taken to be
correct.
16. So far as the purported plea of alibi of appellant No.2 is concerned, a clear finding has
been arrived at that DW-1 has manipulated Exhibit D-3 just to help the said accused. The
High Court also agreed with the said view.
We do not see any reason to differ with the views of the learned Sessions Judge as also
the High Court.
17. For the aforementioned reasons, we do not find any merit in this appeal. It is
dismissed accordingly.
Appeal dismissed.
AIR 2008 SUPREME COURT 630 "Gurdev Singh v. Narain Singh"
(From : Punjab and Haryana)*
Coram : 2 S. B. SINHA AND H. S. BEDI, JJ.
Civil Appeal No. 5237 of 2007 (arising out of SLP (C) No. 5476 of 2006), D/- 12 -11
-2007.
Gurdev Singh v. Narain Singh.
Civil P.C. (5 of 1908), S.47 - EXECUTION - DECREE - INJUNCTION - Powers of
executing Court - Decree for permanent injunction - Restraining appellant from planting
any tree on land in question - Decree did not speak of removal of any tree which had
already been planted - Interpreting said decree by executing Court and holding that there
should not be any tree within two karams on either side of common boundary of parties -
Not in consonance with tenor of decree - Decree holder cannot seek execution of decree
by way of removal of trees - Same could not also be directed by executing Court.
C. R. No. 4526 of 2004, D/- 23-2-2006 (PandH), Reversed. (Paras 8, 9)

Sarup Singh, Sr. Advocate, Yash Pal Dhingra, Kuldip Singh, for Appellant; G. S. Punia,
Davender Mohan Verma, Ms. Minakshi Vij, for Respondent.
* C. R. No. 4526 of 2004, D/- 23-2-2006 (PandH)
Judgement
JUDGMENT :-Leave granted.
@page-SC631
2. Respondent herein filed a suit for permanent injunction against the appellant. The suit
was marked as Civil Suit No. 226 of 1987. A decree for permanent injunction was passed
by the learned trial Judge on 19-1-1989, the operative portion whereof reads as under :
"This suit comes today before me (Balbir Singh PCS, Addl. Senior Sub Judge Jagraon)
for final disposal, in the presence of the counsel for the parties, it is ordered that :-
(illegible) the suit of the plaintiff is decreed against the defendant for a permanent
injunction restraining the one side and khasra No. 218/1 and 17/1 on the other side situate
in the area of village Abbupura Tehsil, Jagraon, District Ludhiana peculiar circumstances
of the case, the parties are left to their own costs."
3. The decree holder filed an application for execution of the decree praying, inter alia,
for removal of the tree from the lands in question. A Commissioner was appointed. He
submitted a report stating as under :
(i) I compared the site plan with the situation of the disputed property where tree of
Bohar exists.
(ii) I measured the distance of Bohar tree from the common Butt of Khasra Nos. 17/2 and
218/1 with the help of a measurement tape. The centre point of Butt upto the centre point
of radius of Bohar tree is 11 feet i.e. 2 karams.
(iii) The branches of Bohar tree comes across the common Butt into khasra No. 218/1
approximately 6/7 feet.
(iv) I prepared a rough site plan at the spot, which is also attached herewith. according to
my observation the half portion of the Bohar tree falls within two karams from the
common Butt of properties of the parties."
4. The learned Commissioner in his report did not state that the Bohar tree was planted
after passing of the decree.
5. The executing Court relying on or the basis of the said report as also some decisions of
this Court while holding that the executing Court has the requisite jurisdiction to construe
a decree, opined as under :
"Now applying ratio of the above said cases this Court has to see what was spirit of the
decree which was under execution and clear that the plain meaning of the decree is that
there should be no tree within two karams on either side of the common boundary of the
parties and if it is there, the executing Court can very well order its removal in order to
give effect to the spirit of the decree. In the present case, it has been reported by the Local
Commissioner that the tree is within 2 karams. Moreover, J. D. does not plead any claim
over that tree. Rather he pleads it to be a naturally grown tree. so, in these circumstances
direction is given to the J. D. to remove the trees which is standing within 2 karams of the
common butt within a period of one month from today."
6. By reason of the impugned judgment the High Court has affirmed the said order.
7. Mr. Swarup Singh, learned senior counsel appearing on behalf of the appellant would
submit that the executing Court as also the High Court committed a manifest error in
interpreting the decree.
8. We agree with the said contention. A bare perusal of the decree in question would
clearly demonstrate that the appellant herein was restrained by a permanent injunction
from planting any tree on khasra Nos. 17/2 on the one side and khasra No. 218/1 and 17/1
on the other side. The decree did not speak of removal of any tree which had already been
planted. The executing Court, as noticed hereinbefore, while interpreting the said decree
proceeded completely on a wrong premise to hold that there should not be any tree within
two Karams on either side of the common boundary of the parties. Such an interpretation
evidently is not in consonance with the tenor of the decree. A jurisdictional error, thus,
has been committed by the High0 Court.
9. It is well stated that executing Court cannot go behind the decree. As the decree did not
clothe the decree holder to pray for execution of the decree by way of removal of the
trees, the same could not have been directed by the learned executing Court in the name
of construing the spirit of the decree under execution.
10. We, therefore, set aside the impugned judgment and remit the matter to the executing
Court for determination of the question as to whether the Bohar tree in question was in
existence prior to passing of the decree or not. The executing Court thereafter may
proceed with the matter in accordance with law.
11. The appeal is allowed. No costs.
Appeal allowed.
@page-SC632
AIR 2008 SUPREME COURT 632 "Desh Raj v. Bodh Raj"
(From : Himachal Pradesh)
Coram : 2 K. G. BALAKRISHNAN AND R. V. RAVEENDRAN, J.
Civil Appeal No. 4676 of 2005, D/- 30 -11 -2007.
Desh Raj v. Bodh Raj.
(A) Evidence Act (1 of 1872), S.35 - EVIDENCE - DOCUMENTS - Documentary
evidence - Caste of candidate, respondent contesting election on reserved seat - Proof -
Documents like application made by his father for his admission to school, birth register
extract and village Pariwar Register extracts produced to establish caste of respondent -
Said documentary evidence cannot be said to have no evidentiary value - No
irreconcilable difference between said documents - Cannot be rejected. (Paras 20, 21)
(B) Evidence Act (1 of 1872), S.90 - DOCUMENTS - ELECTION - Caste - Proof -
Dispute as to caste of candidate contesting election - Application for admission to school
by father showing caste of candidate, produced - Said application submitted by father
containing his thumb mark - It is a document more than 30 years old - Attracts
presumption under S. 90 - Merely because there was difficulty in reading one figure in
the date in said application - Cannot be a ground to reject it.(Para 21)
(C) Representation of the People Act (43 of 1951), S.100 - ELECTION - SCHEDULED
CASTES AND SCHEDULED TRIBES - DOCUMENTS - RESERVATION - Election of
MLA - Respondent contested on seat reserved for Scheduled Caste - Documentary
evidence showing that respondent, candidate's father and his family members including
respondent held out to be and accepted as belonging to "Tarkhan Caste" which is not
Scheduled Caste - Only after 1990 respondent attempting to show that he belonged to
"Lohar" Caste which is Scheduled Caste - Caste Certificate showing his caste as "Lohar",
a Scheduled Caste issued by Executive Magistrate in 1991 - Cannot be a proof of caste of
respondent - Thus, respondent held did not belong to "Scheduled Caste" - Not qualified to
be chosen to fill seat in constitutency reserved for Scheduled Castes - His election liable
to be set aside.
Election Petition 1 of 2003, D/- 7-6-2005 (HP), Reversed.
AIR 1984 SC 905 and 1994 AIR SCW 4116, Relied on. (Paras 26, 27, 28)
Cases Referred : Chronological Paras
2005 AIR SCW 1476 : AIR 2005 SC 1868 (Ref.) 20
1994 AIR SCW 4116 : AIR 1995 SC 94 (Ref.) 20, 26
AIR 1988 SC 1796 (Ref.) 19, 20
AIR 1984 SC 905 (Rel. on) 26
AIR 1982 SC 1057 : 1982 Cri LJ 994 (Ref.) 20
Mahabir Singh, Sr. Advocate, Ajay Pal, Nikhil Jain, Gagan Deep Sharma, Ms. Preeti
Singh and Dilbag Singh (for Sunil Kumar Jain), for Appellant; Anoop C. Choudhari, Ms.
June Choudhari, Sr. Advocates, J. S. Attri, for Respondent.
Judgement
K. G. BALAKRISHNAN, C.J.I. :-This statutory appeal under Section 116A of the
Representation of People Act 1951, is filed by an Election Petitioner against the judgment
dated 7.6.2005 of the Himachal Pradesh High Court dismissing his Election Petition No.1
of 2003 challenging the election of the respondent (Bodh Raj) as Member of Legislative
Assembly from 35-Gangath (SC) Assembly Constituency.
2. The case of the appellant in brief is that 35-Gangath Assembly Constituency is
reserved for scheduled castes, that he and the respondent, among others were candidates
for election from the said constituency. In the said election held on 26.2.2003, the
respondent secured the highest number of votes namely 24499 and was declared as
elected. The respondent had in his nomination paper declared that he belongs to a
scheduled caste (Lohar) and in support of his claim, had produced a caste certificate
dated 16.12.1991 issued by the Executive Magistrate, Indora, District Kangra certifying
that he belonged to scheduled caste of Lohar. Only a few days before the polling, the
appellant learnt that respondent does not belong to Lohar caste but belongs to 'Tarkhan'
caste which is not a scheduled caste in the State of Himachal Pradesh. According to
appellant, the respondent was disqualified to contest the election in the Assembly
Constituency reserved for scheduled caste and therefore, the election of the respondent
was void.
3. The respondent resisted the said election petition. In his written statement, he
@page-SC633
asserted that he belonged to Lohar caste (a Scheduled Caste) and was eligible and
qualified to contest as a candidate for the reserved Assembly Constituency (35-Gangath).
He also contended that he was not served a complete and attested copy of election
petition and therefore, the petition was liable to be rejected.
4. Issues 1 to 3 framed by the High Court (relating to the respondent's contention that he
was not served a complete and attested true copy of the election petition) were treated
and tried as the preliminary issues and held against the respondent by order dated
26.9.2003. Thereafter, evidence was led in regard to the issues (4) to (6) which read thus:
(4) Whether the respondent is not a member of Lohar Caste (SC) and was not qualified
on the date of his election to fill the seat in the Assembly, from reserved Constituency for
SC?
(5) Whether nomination paper of respondent has wrongly and improperly been accepted?
(6) Relief
After appreciating the oral and documentary evidence, the learned Single Judge of the
High Court by judgment dated 7.6.2005 held that the appellant failed to prove that
respondent did not belong to a schedule caste (Lohar) and was not qualified to contest the
election to the assembly seat reserved for scheduled caste. As a consequence, he
dismissed the petition. The said judgment is under challenge in this appeal.
5. It is not in dispute that a person who does not belong to a scheduled caste, cannot offer
himself as a candidate for election to a reserved constituency. Article 173 of the
Constitution prescribes the qualification for membership of the State Legislature and
provides that a person shall not be qualified to be chosen to fill a seat in the legislature of
a State unless he is a citizen of India, not less than 25 years of age, and possesses such
other qualifications as may be prescribed in that behalf by or under any law made by
Parliament. Section 5 of the Representation of People Act, 1951 ('Act' for short) made by
the Parliament prescribes the qualification for membership of a Legislative Assembly. It
provides that a person shall not be qualified to be chosen to fill a seat in the Legislative
Assembly of a State, reserved for the scheduled castes of that State, unless he is a
member of any of those scheduled castes and he is an elector for any Assembly
Constituency in that State. Section 100 of the Act enumerates the grounds for declaring
an election to be void. Clause (a) of sub-section (1) thereof provides that if the High
Court is of the opinion that on the date of his election, a return candidate was not
qualified, or was disqualified, to be chosen to fill the seat under the Constitution or under
the Act, the High Court shall declare the election of the returned candidate to be void.
Thus, if a candidate who contests the election, representing himself by belonging to a
schedule caste, is shown in a proceeding contesting his election, as not belonging to a
schedule caste of the State, his election is liable to be declared as void. Therefore, the
only question that arises for our consideration is whether the appellant had proved that
the respondent does not belong to 'Lohar Caste' - a scheduled caste of the State of
Himachal Pradesh.
6. The appellant had let in oral evidence by examining some residents of the respondent's
village Mohtli - Jagdish Raj (PW7), Satpal (PW8), Joginder Singh (PW9) and Mohal Lal
(PW10) to show that the respondent belonged to 'Tarkhan' caste. He also let in
documentary evidence in the nature of school records (Ex.PW-2/A, Exs.PW-3/A and
3/B), birth register extracts (Exs.PW-6/A, 6/B, and 6/C) and Pariwar Register maintained
by the Gram Sabha (Exs.PW-4/A, 4/B, 4/C and 4/D) to show that the caste of respondent
was 'Tarkhan' and that after 1990 respondent had attempted to represent that his caste as
Lohar. We will first consider the oral evidence.
7. Jagdish Raj (PW7) stated that the respondent's mother and his father were cousins, that
he and respondent belong to Tarkhan caste and are residents of Mohtli village. According
to him, the village has three Mohallas. About ten families of Tarkhan caste and those
belonging to the Rajput caste reside in Jaildar Mohalla. Persons belonging to the
scheduled castes of Chamar, Mahashay, Batwal and Bazigar reside in the Harijan
Mohalla. Brahmins reside in the Brahmin Mohalla. He stated that no one belonging to
'Lohar' caste resided in the village. He also states that respondent's parents were Milkhi
Ram and Giano Devi. He also gave the names of other Tarkhan families in the village
who were the relatives of the respondent. His evidence was rejected
@page-SC634
by the High Court on the ground that the witness had admitted that his grandfather had
worked as a blacksmith and a person who worked as a blacksmith was called as a Lohar
and a person who worked as carpenter was called as Tarkhan and on the ground that he
was not in a position to say the degree of relationship between his father and respondent's
mother, when he claimed that they were cousins.
8. Satpal (PW8), another resident of Mohtli village, stated that respondent was a Tarkhan
by caste, that he knew respondent's father Milkhi Ram as also his relatives who all
belonged to Tarkhan caste and who were displaced persons who had come from Pakistan
and settled in the village Mohtli. He also stated that Mohtli village is divided into three
areas namely Brahman Abadi where Brahamins lived, Jaildar Mohalla where Tarkhan and
Rajput families resided and a separate Mohalla where people belonging to scheduled
castes - Chamar, Mahashay, Batwal and Bazigar resided and that there was no Lohar
family in the village Mohtli. He also stated that except respondent, there was no other
person known as Bodh Raj son of Milkhi Ram in the village Mohtli. The evidence of this
witness was rejected by the High Court on the ground that the witness was able to state
the number of issues of each son and daughter of Milkhi Ram.
9. Joginder Singh (PW9), another resident of Mohtli village, stated that respondent
belonged to Tarkhan caste, that he also knew the respondent's father Milkhi Ram as also
their relatives who all belonged to Tarkhan caste. He also stated that Milkhi Ram had five
children namely three sons (Sat Pal, Yash Pal and Bodh Raj) and two daughters (Satya
Devi and Raj Rani) and the respondent was youngest among the five issues of Milkhi
Ram. He also stated that Tarkhans and Rajputs reside in Jaildar Mohalla, that persons
belonging to scheduled castes of Chamar, Bazigar, Batwal and Mahashay resided in a
separate Mohalla, and Brahmins resided in another separate Mohalla. The evidence of
this witness was rejected on the ground that he was a sympathizer towards BJP party to
which the appellant belonged and therefore, inimical towards respondent who belonged
to Congress Party.
10. Mohan Lal (PW10) who is also a resident of the Mohtli, stated that respondent
belonged to Tarkhan caste and that there was no person other than respondent in the
village who is known as Bodh Raj, son of Milkhi Ram and that no Lohar family resides
in the village Mohtli. His evidence was rejected by the High Court on the ground that he
did not know respondent's father Milkhi Ram and had not stated the occupation of the
respondent's family members.
11. The appellant Desh Raj gave evidence as PW-11. He stated that respondent belonged
to Tarkhan caste and was not qualified to contest the election for a seat reserved for
scheduled castes. He stated that only 4 to 5 days before the polling, he came to know
from his workers that respondent belonged to a backward caste (BC) and not a scheduled
caste. His evidence by the High Court was rejected as he had no personal knowledge
about the caste of the respondent.
12. We may also refer to the evidence of the respondent's witnesses hailing from the
village Mohtli. RW-1 Yash Pal, respondent's elder brother, examined as RW-1 stated that
he and respondent are Lohars by occupation. He also stated that Basaba Ram and Nasib
Chand who are related to him were also Lohars by 'occupation'. In his cross-examination,
he stated that his father Milkhi Ram had five children (three sons and two daughters), that
the respondent was the youngest, that his grandfather's name was Gopi, that he and
respondent studied in the village school, and that respondent was carrying on the business
of scooter repairs. Tilak Raj examined as RW-4 stated that respondent, and his relations
Khazana Ram and Basaba Ram were Lohar by caste as they were doing the job of
Lohars. Ved Prakash (RW-6) stated that respondent and his brothers as also Basaba Ram
and Khazana Ram worked as Lohars and were, therefore, belonged to Lohar caste. He
admitted that he was elected as the Pradhan and respondent was elected as Up Pradhan of
Mohtli Gram Panchayat in the year 1990 that both belonged to congress party. He also
admitted that gram panchayat maintained a register known as Pariwar Register, that
Pradhan of the gram panchayat was the overall custodian of all records and that the
details of all families residing in the panchayat areas including names, age, address, caste
etc., are recorded in the said register. He admitted that in the Ex.PW4/A, the Pariwar
Register relating to the year 1976, the caste of respondent and his family had been shown
as Tarkhan and that then corrected as
@page-SC635
'Lohar'. Ram Singh (RW-7) stated that he knew respondent's father Milkhi Ram and his
three sons including respondent were belonged to Lohar caste. He also stated that he had
seen the members of the respondent's family working as Lohar and therefore, he stated
that he belonged to the caste of Lohar. Bua Butta (RW-8) another resident of Mohtli
village stated that he knew the respondent, that respondent was a Lohar by caste.
According to him, because he used to get agricultural iron implements prepared and
repaired by him, the respondent belonged to Lohar caste. He asserted that except
respondent there is no other Bodh Raj, son of Milkhi Ram in the Mohtli village. Maggai
Singh (RW-11) who is a resident of a neighbouring village of Surajpur stated that he used
to get Lohar's job done from Milkhi Ram, Khazana Ram and Chaina Ram and that 'since
they were working as Lohars, they were Lohars by caste. He also clarified that he had not
enquired about their castes and that it is possible that respondent and his family may be
Tarkhans.
13. What emerges from the aforesaid oral evidence is that while the witnesses examined
by the appellant (PWs 7, 8, 9 and 10), who all belonging to Mohtli village to which
respondent belonged, stated that they knew him and his family well and that he belonged
to Tarkhan caste. The cross-examination of these witnesses (PWs 7, 8, 9 and 10) has not
brought out anything significant to disbelieve their evidence. On the other hand, the
evidence of the witnesses of respondent (RWs 1, 4, 6, 7, 8 and 11) has been to highlight
the occupation of respondent and his relatives. They have all stated that because the
respondent's family and relatives were doing the job of Lohars, they belong to the caste
of 'Lohar'. In fact, the evidence of appellant's elder brother Yash Pal in his short
examination-in-chief, extracted below, is significant :
"I know the respondent. He is my brother. We are Lohars by occupation. Name of my
father is Milkhi Ram. I know Basawa Ram and Nasib Chand also. They are related to me.
They are also Lohars by occupation."
14. We will next consider the documentary evidence. Ex.PW-2/A is the admission and
withdrawal Register of Government Primary School, Mohtli for the relevant period.
Entry at Sl. No.1739 in the said Register shows that Bodh Raj, son of Milkhi Ram,
Labourer (date of birth 2.5.1956; caste: Tarkhan) was admitted on 16.4.1962 to the First
Standard and his name was struck off due to lack of attendance on 11.2.1964. There is
another entry relating to Bodh Raj, son of Milkhi Ram of Mohtli village at Sl. No. 1959.
This entry shows that Bodh Raj, son of Milkhi Ram (date of birth 2.5.1956, caste :
Tarkhan) was admitted to the second standard. The portion of the sheet where the date of
admission was noted is torn and it is however, evident from the other entries in the sheet
that the admission for the second time was made in April, 1964. The entry also shows that
he studied up to 5th standard and completed his education in the school on 31.3.1967.
15. Ex.PW-3/A is the application form for admission given to the Government
Secondary School by Milkhi Ram. It gives the name of the student as Bodh Raj, father's
name as Milkhi Ram, date of birth as 2.5.1956 and the caste as Tarkhan. It contains the
thumb mark of Milkhi Ram. Ex.PW-3/B is the Admission Register of Mohtli Government
Middle School for the period 1962 to 1969. Entry No.778 relates to Bodh Raj son of
Milkhi Ram, Mazdoor, caste Tarkhan. The admission was noted in a page at the top of
which was the date 11.9.1967. As the next page starts with the date 4.4.1968, it is to be
inferred that the admission to the middle school was in the year 1967-68.
16. Ex.PW-6/A is the extract of the Birth Register maintained by the Indora Police Station
(page 376 entry no.27) whose limits include Mohtli village. Ex.PW-6/B is the true
English translation of Ex. PW/6A which is in Urdu. Ex. PW6/C is the certificate of birth.
They relate to the birth of the fifth child of Milkhi Ram (son of Gopi Ram) and Smt.
Giano, on 2.5.1956. The place of residence of the parents is shown as Mohtli and their
caste is shown as Tarkhan. The name of the male child is shown as Bodh Ram. The
registration was made on 16.5.1956, on the report of the Chowkidar.
17. Ex.PW-4/A, Ex.PW-4/B, Ex.PW-4/C and Ex.PW-4/D are the Pariwar Register of
Mohtli Village for the years 1976, 1977, 1982-89 and 1990 onwards. The said register is
maintained as required by the relevant rules relating to Gram Sabhas. In Ex.PW-4/A
relating to the year 1976, the family of Yash Pal is shown as consisting of Yash Pal, his
wife Prem Lata, daughter Guddi and brother Bodh Raj. Under the column 'whether
scheduled caste or scheduled tribe',
@page-SC636
the caste is entered as 'Tarkhan', which is struck off and substituted by the word 'Lohar'
without any attestation regarding correction. In Ex.PW-4/B is the pariwar register relating
to the year 1977, the entry relating to Sat Pal and his family shows that his family
consisted of himself, his wife Kamlesh, his brothers Yash Pal and Bodh Raj and his
children Asha, Nirasha and Sushil Kumar. Under the column 'whether scheduled caste or
scheduled tribe', the caste is entered as 'Tarkhan' which is struck off and substituted by the
words 'Lohar' without any attestation regarding correction. In Ex.PW-4/C which is the
Pariwar register for the year 1982-1989, the entry regarding the family of Sat Pal shows
the family as consisting of himself, his wife Kamlesh, his children Asha, Nirasha and
Sushil Kumar, his brother Yash Pal and his wife Prem Lata and child Guddi and another
brother Bodh Raj. Here again, under the column 'whether scheduled caste or scheduled
tribe', the entry is 'Tarkhan' which is struck off and substituted by the word 'Lohar'
without any attestation regarding the correction. Ex.PW-4/D is the Pariwar register for the
year 1990 onwards and in this register, the family of Bodh Raj is shown as consisting of
himself, his wife Kunti Devi and children Rajiv Kumar and Pankaj Kumar and under the
column 'whether scheduled caste or schedule tribe', the caste is shown as 'Lohar'.
18. The High Court has rejected all these documents as either not proved or not of any
evidentiary value. We may now consider whether they were properly proved.
19. Ex.PW-2/A (admission and withdrawal register of the Government primary school,
Mohtli) was produced by PW-2 (Kamla Kumari) employed in the Government primary
school, Mohtli, in response, summons issued to the said school to produce the said
register. She also gave evidence regarding entries Nos. 1739 and 1959 relating to Bodh
Raj and gave the particulars entered in regard to Bodh Raj under the said two entries. In
her cross-examination, she stated that she has been posted in the said school for the last
two years and that she had not made the said entries. The High Court has rejected the said
School Register on the ground that the said register Ex.PW-2/A and the entries therein
relating to Bodh Raj merely on the ground that PW-2 was not the author of the entries
and she has no personal knowledge about the entries. The High Court relied on the
decision of this Court in Birad Mal Singhvi v. Anand Purohit [AIR 1988 SC 1796].
20

. Section 35 of the Evidence Act provides that an entry in any public or other official
book or register or record, stating a fact in issue or relevant fact and made by a public
servant in the discharge of his official duty or by any other person in performance of a
duty specifically enjoined by law of the country in which such book or register is kept, is
itself a relevant fact. Having regard to the provisions of Section 35, entries in school
admission registers in regard to age, caste etc., have always been considered as relevant
and admissible. [See - : Umesh Chandra vs. State of Rajasthan - 1982 (2) SCC 202 and
State of Punjab v. Mohinder Singh - 2005 (3) SCC 702]. In Kumari Madhuri Patil v.
Addl. Commissioner [1994 (6) SCC 241], this Court observed that caste is reflected in
relevant entries in the public records or school or college admission register at the
relevant time and certificates are issued on its basis. In Birad Mal Singhvi (supra), this
Court after referring to the ingredients of Section 35 held thus : AIR 1982 SC 1057
2005 AIR SCW 1476
1994 AIR SCW 4116

"An entry relating to date of birth made in the school register is relevant and admissible
under Section 35 of the Act, but the entry regarding to the age of a person in a school
register is of not much evidentiary value to prove the age of the person in the absence of
material on which the age was recorded. The entries regarding dates of birth contained in
the scholar's register and the secondary school examination have no probative value, as
no person on whose information the dates of birth of the aforesaid candidates was
mentioned in the school record, was examined. In the absence of the connecting
evidence, the documents produced by the respondent, to prove the age of the aforesaid
two candidates have no evidentiary value."
This Court further held unless the parents, or persons conversant with their date of birth
were examined, the entry in the school register by itself will not have much evidentiary
value. In this case, we are concerned with the 'caste' and not the date of birth. The
residents of a village have more familiarity with the 'caste' of a co-villager, than the date
of birth of the co-villager. Several villagers who knew the respondent and
@page-SC637
their father, including a cousin of the respondent has been examined and they have stated
the caste of the respondent. Appellant has also produced other documentary evidence
which clinch the issue, namely the application made by the respondent's father for
admission of respondent to school, birth register extract and village Pariwar Register
extracts to establish the caste of the respondent. Further the said entries in the school
register were made nearly forty years prior to the election petition. When read with other
oral and documentary evidence, it cannot be said that Ex.PW-2/A has no evidentiary
value even by applying the strict standards mentioned in Birad Mal Sanghvi.
21. We will next refer to Ex. PW3/A and Ex. PW3/B produced by PW-3 Smt. Indersh
Bala, Principal of the Mohtli Senior Secondary School in response to a summons issued
by the High Court. Ex. PW3A is application for admission submitted to the School by
Milkhi Ram, father of Bodh Raj, registered as Sl.No.478. Ex.PW-3/B is the School
Admission Register and entry 778 showed that Bodh Raj son of Milkhi Ram, caste
Tarkhan, was admitted to middle school and had passed 8th standard from the school.
PW-3 stated that the particulars mentioned in the entry were that he was the son of Milkhi
Ram, resident of village Mohtli and that his caste was Tarkhan. In her cross-examination,
she stated that she was working in the said school for the last about one and half years
and has no personal knowledge about the entries made therein. The High Court rejected
both Ex.PW-3/A and PW-3/B on the ground that the date of Ex.PW-3/A was not clear and
can be read as 22.4.1996 or 23.4.1968 and neither of those dates correlated to Ex.PW-3/B
as that showed that admission must have been made between 11.9.1967 and 4.4.1968. It
is evident from Ex.PW-2/A that Bodh Raj left the primary school on 31.3.1967. The date
on which the application for admission was registered was seen as '22.4.196__'. Only
regarding the last figure in the 'year' the Court had a doubt whether it was '6' or '7' or '8' as
that would make the year 1966, or 1967, or 1968. Merely because there was difficulty in
reading one figure in the date cannot be a ground to refuse to accept Ex.PW-3/A. The said
application submitted by Milkhi Ram, containing his thumb mark, being a document
more than 30 years old attracts the presumption under Section 90 of Evidence Act. As
Ex.PW-3/A gives the caste as 'Tarkhan', it has to be treated as clinching evidence. Ex.PW-
3/B which was also produced from proper custody in pursuance of summons issued from
the Court showed that Bodh Raj, son of Milkhi Ram, Tarkhan caste, belonging to Mohtli
village studied upto 8th standard. Here again it should be noticed that the evidence of the
witnesses of both appellant and respondent is that there is only one Bodh Raj, son of
Milkhi Ram in Mohtli village. Therefore, there was no justification to hold that there
were some irreconcilable difference between Ex.PW-3/A and Ex.PW-3/B and rejected
both the documents. Another reason given by the High Court to reject the said evidence is
that Ex.PW-3/B showed that the Bodh Raj had passed the 8th Standard and whereas he
had stated in his cross examination that his qualification is under 'middle'. The High
Court interpreted this as having failed in 8th standard, and considered the said statement
as a contradiction and therefore, an additional ground for rejecting Ex.PW-3/B. The
respondent had been evasive in his evidence about his date of birth and particulars of his
relatives in the village, to avoid being linked to the caste mentioned in the school records.
Therefore, his statement that he was under 'middle' was not a ground to reject the
correction of Ex.PW-3/B. Insofar as the evidentiary value of Ex.PW-3/B, our observation
with reference to Ex.PW-2/A equally apply to Ex.PW-3/B also.
22. We are of the view that the High Court committed an error in ignoring the entries in
the admission and withdrawal registers of the Government primary and middle schools,
Mohtli (Ex. PW-2/A and Ex. PW-3/B). We have already noticed the evidence (of PW8
and RW8) that there is only one Bodh Raj, son of Milkhi Ram in the village of Mohtli.
Respondent does not claim that there was any other Bodh Raj, son of Milkhi Ram in the
village of Mohtli. Respondent, who was examined as RW-5, specifically admits that he
studied in the Government primary school, Mohtli. He gives his age as 48 years in 2004
which corresponds with the age that is entered in the said register. When he was put a
specific question about his date of birth that is 2.5.1956 (which was the date entered in
the said registers), the respondent gave an evasive answer stating that he did not know
whether his date of birth was
@page-SC638
2.5.1956. What is significant is that he did not deny that his date of birth was 2.5.1956. In
fact RW-9 examined by respondent admitted that date of birth of respondent is 2.5.1956.
The admission of the respondent that he was born around 1956 and was a resident of
Mohtli village and studied in the Government primary school, Mohtli, when read with the
school records, prove beyond doubt that the entries in Ex. PW2/A and Ex. PW3/B
referred to above relating to Bodh Raj, son of Milkhi Ram of Mohtli village, Tarkhan
caste, refers to respondent.
23. In response of summons issued by the High Court, PW-6 Naresh Sood working as
Projectionist in the office of CMO, Dharmashala, brought the birth register and
maintained by the Indora Police Station. The relevant entry relating to birth of the fifth
child of Milkhi Ram and Giano of Mohtli village of Tarkhan caste on 2.5.1956 was
marked as Ex.PW-6/A. An English translation of the Urdu extracts was Ex.PW-6/B, and
the certificate as Ex.PW-6/C. The said register and the extract showed the name of the
child as 'Bodhu Ram'. It also shows that the entry was made on 16.5.1956 on the
information given by the, Chowkidar. The High Court rejected the said evidence merely
on the ground that the name of the child was mentioned as 'Buddu Ram' and not as Bodh
Raj. This again is a public record relating to births maintained as per Rules in the usual
course of discharge of official functions. The Punjab Police Rules, 1934 (applicable to
Himachal Pradesh) require maintenance of a Register of Births and Deaths at the Police
Station (vide Rule 22.45 in Chapter XXII relating to Police Station. Rule 22.66 gives the
manner of maintaining such Register. Clause (5) states that birth and death registers shall
be retained at the Police Station for one year after the date of last entry and shall be sent
to the Civil Surgeon for record. The Rule requires the village watchman should diligently
report births and deaths of his village diligently. Therefore the said birth records ought to
have been accepted by the High Court. The High Court has rejected the Birth Extract and
certificate as they relate to Buddu Ram and not Bodh Raj. It is quite possible that the
person who gave information mentioned the name as Buddu Ram instead of Bodh Raj or
that the child was also known as Buddu Ram initially. But what is relevant is that fifth
child of Milkhi Ram and Giano of Mohtli village who belonged to Tarkhan caste was
born on 2.5.1956. It is nobody's case that Milkhi Ram and Giano of Mohtli village had
some other fifth child born on 2.5.1956.
24. In pursuance of summons issued by the Court, Chunni Lal, the Panchayat Secretary
of Gram Panchayat, Mohtli (PW-4) produced the Pariwar register prepared and
maintained as required under the Rules relating to Gram Sabhas. The pariwar registers for
the years 1976, 1977, 1982 to 1989 and 1990 onwards were produced as Exs.PW-4/A,
PW-4/B, PW-4/C and PW-4/D. In Ex.PW-4/A, Bodh Raj was shown as family member of
elder brother Yash Pal. In Exs.PW-4/B and PW-4/C, he was shown as a family member of
elder brother Sat Pal. In all these registers, the family was shown as of Tarkhan caste.
Against the column 'whether scheduled caste or scheduled tribe', the entry was 'Tarkhan'
which was struck off and substituted by the entry Lohar. The correction was not attested.
On the other hand, Ex.PW-4/D relating to the period of 1990 onwards showed the
respondent himself as the head of his family and his caste as Lohar. RW-6, Ved Prakash,
was the Pradhan of the Mohtli Gram Panchayat elected for two terms in 1985 and 1990.
He also had admitted that the Gram Sabha was maintaining a pariwar register containing
the details of all families residing in the panchayat area including their ages, occupations,
castes etc. The suggestion put by respondent (RW-5) and Ved Prakash [RW6] (Pradhan
during 1985-1995 and elected in 1990) was that when respondent became the Up-
Pradhan of the Mohtli Gram Panchayat in 1990, he managed to get the entries in Exs.PW-
4/A, PW-4/B, and PW-4/C, relating to caste namely 'Tarkhan' struck off and substituted
the word 'Lohar'. The suggestion of course was denied. If the substitution was with
reference to the entry in only one register, it could have been explained away as a
mistake. But it is significant that the registers of the years 1976, 1977 and 1982-1989 all
show the caste of the family as 'Tarkhan' and all the entries are struck off and substituted
by the word 'Lohar'. The High Court has refused to rely on Ex. PW4/A, B, C only on the
ground that the entries in the register contained some other corrections and that the
manner in which they were maintained raised a doubt about the probative value of the
document. We are of the view that in the absence of any satisfactory
@page-SC639
explanation of the caste 'Tarkhan' being struck off and substituted by 'Lohar', the
conclusion is that they were all done subsequent to 1990 when respondent became the
Up-Pradhan.
25. The evidence let in by appellant clearly establish the following :
(a) Respondent was born in and is a resident of Mohtli village. His date of birth is
2.5.1956.
(b) Respondent is the last and fifth child of his parents are Milki Ram and Giano.
Respondent is the only 'Bodh Raj', son of Milkhi Ram in Mohtli village.
(c) Respondent was a student of Government primary and middle schools, Mohtli. The
school records show that respondent is the son of Milkhi Ram of Mohtli and his caste was
Tarkhan on the basis of particulars furnished by his father.
(d) In the birth register maintained in the jurisdictional Police Station as per the Punjab
Police Rules, his date of birth was registered as 2.5.1956 and the caste of his parents was
shown as Tarkhan;
(e) That in the Pariwar Registers maintained by the Gram Sabha between 1976 and 1989,
the caste of his family was shown as 'Tarkhan' and that sometime thereafter, it was struck
off and shown as 'Lohar'.
The evidence of the residents of Mohtli village (PWs.7 to 10) support the same. There is
nothing in the cross-examination of PWs.7 to 10 to disbelieve their statements that the
respondent belonged to Tarkhan caste. However, even if we exclude the entire oral
evidence, the documentary evidence produced by the appellant, to which we have
adverted to above, clearly demonstrate that the respondent's father and his family
members including respondent had always held out to be and accepted as persons
belonging to Tarkhan caste. It was only after 1990, the respondent tried to show that he
belonged to Lohar caste.
26

. The learned counsel for the respondent submitted that in view of Ex. PW4/D and
Ex.RW-5A, he should be considered as having established that he belongs to Lohar caste.
Ex. PW-4/D is the Pariwar Register extract for the year 1990 onwards. The same no
doubt shows the caste of respondent as Lohar. But when Ex.PW-4/D is read in
conjunction with PW-4/A, PW-4/B and PW-4/C which are the Pariwar Register extracts
relating to the previous years (1976, 1977 and 1982-1989) where his caste was shown as
Tarkhan and later altered as 'Lohar', the entry in Ex.PW-4/D becomes a self serving
statement. The respondent was elected as the Upa-pradhan of Mohtli Gram Panchayat in
the year 1990 (RW-6, Ved Prakash, belonging to his party was elected as Pradhan). In his
capacity as Upa-pradhan he had access to the records of the Panchayat, and it is obvious
that with the intention of representing himself as belonging to a Scheduled Caste of
Lohar, had ensured that his caste was shown as Lohar in PW-4/D. The alteration of the
entries relating to caste in Exs.PW4/A, 4/B and 4/C, from 'Tarkhan' to 'Lohar' should be
looked at in this background, particularly when it is seen that the correction of caste by
striking out 'Tarkhan' is not only in regard to the family of respondent but also in the case
of some of the relatives of the respondent. In so far as the caste certificate Ex.RW-5/A
issued by the Executive Magistrate, Indora, relied on by respondent, it has to be observed
that such caste certificates are not given after a thorough investigation. When the caste of
respondent is in issue and when primary evidence regarding caste is led by appellant, and
the attempt of respondent to claim to be a 'Lohar' from 1990 is evident, the caste
certificate issued by the Executive Magistrate on 1.12.1991 cannot be taken as evidence
to prove the caste of the respondent. The decision of this Court in R. - Palanimuthu v.
Returning Officer [1984 (Supp.) SCC 77], supports this position. In Madhuri Patil
(supra), this court observed that when the school records show a particular caste, the caste
certificates issued to the candidates and his relatives by the Executive Magistrate showing
a different caste should be ignored. Reference was also made to the caste certificate of
two relatives. But they are also of the period subsequent to 1990 when respondent started
showing that he belonged to Lohar caste. They have to be ignored as observed by this
Court in Madhuri Patil (supra). AIR 1984 SC 905
1994 AIR SCW 4116

27. In view of the above, we are of the view that the appellant has clearly established that
the respondent and his family belong to Tarkhan caste which is not a scheduled caste in
Himachal Pradesh. It is also clear that from around 1990, the respondent has made efforts
to show his caste
@page-SC640
as 'Lohar', a scheduled caste. Consequently, we hold that the respondent who did not
belong to a Scheduled Caste, was not qualified to be chosen to fill a seat in the
Legislative Assembly reserved for Scheduled Castes.
28. Therefore, we allow this appeal, set aside the judgment of the High Court and declare
the election of the returned candidate (Bodh Raj) from 35-Gangath Assembly
Constituency in the 2003 Election, to be void. Parties to bear their respective costs.
Appeal allowed.
AIR 2008 SUPREME COURT 640 "Y. N. Gangadhara Setty v. J. P. Reddy"
Coram : 2 TARUN CHATTERJEE AND DALVEER BHANDARI, JJ.
Contempt Petn. (C) No. 222 of 2006 In Spl. Leave Petn. (C) No. 24199 of 2005, D/- 2
-11 -2007.
Y. N. Gangadhara Setty and Ors. v. Jaya Prakash Reddy, M. D., Karnataka Co-operative
Milk Producers Federation.
Contempt of Courts Act (70 of 1971), S.14, S.2 - CONTEMPT OF COURT -
ACQUISITION OF LAND - SUPREME COURT - SPECIAL LEAVE PETITION -
COMPROMISE - Wilful disobedience of order of Court - Acquisition of land belonging
to Company - By Government Order sanction was accorded to reconvey disputed land in
favour of Company - Special Land Acquisition Officer entered into a compromise and
agreed to reconvey 2 acres 5 guntas of land in their favour before Supreme Court -
However, terms of compromise were not complied with by State Govt. and on contrary
buildings were constructed on the said land - Company filed various litigations for
repossession of land but in vain - In Special Leave Petition filed by contemner Supreme
Court again directed compliance of compromise - Said order was also not complied with
by contemner - In instant Contempt Petition Supreme Court again directed respondent to
hand-over possession without any loss of time - Cost of reconstruction of existing
buildings directed to be valued by an independent Government approved valuer. (Paras
23, 24)

Arun Jaitley, Mukul Rohatgi, Sr. Advocates, Gopal Jain and Ms. Ruby Singh Ahuja, for
Appellants Uday Holla, Sr. Advocate, Varun Thakur, Hage Lampu and A. S. Bhasme, for
Respondent.
Judgement
1. DALVEER BHANDARI, J. :-This case has a chequered history. It is alleged that
despite orders of this Court in Civil Misc. Petition No.5513 of 1972 in Civil Appeal
No.514 of 1971 dated 30th August, 1972, (about 35 years ago) there has been a wilful
disobedience and defiance of the order of this Court. It is further alleged that there is also
non-compliance of order dated 5th December, 2005 passed by this Court in SLP (C)
No.24199 of 2005.
2. Brief facts which are relevant to dispose of this contempt application are recapitulated
as under:-
The applicants M/s. Y. S. Setty and Sons were the owners of land admeasuring 20 acres
and 3 guntas which was acquired by the State of Karnataka on 19.1.1961 under the Land
Acquisition Act. The disputed land bearing Survey No.76/2 measuring 2 acres and 5
guntas being part of the total land.
3. The applicants filed several representations in the year 1962-63 before the Government
of Karnataka for recovery of only 2 acres 5 guntas in survey No.76/2 in favour of Y. S.
Setty and Sons out of the total land acquired by the contemnor. On 15.5.1962, the
applicants M/s. Y. S. Setty and Sons received compensation of land. An appeal was filed
before the High Court seeking enhancement of compensation. The High Court allowed
the appeal in part. The Special Land Acquisition Officer challenged the order of the High
Court before this Court by way of a special leave petition.
4. On 15.9.1967 vide Government Order No. RD 74 LGB 67, sanction was accorded to
reconvey the disputed land in favour of the applicants Y. S. Setty and Sons by the State
Government subject to fulfilling of certain conditions.
5. On 14.10.1969, the Land Acquisition Officer wrote to the State Government informing
that the disputed land has been reconveyed in the name of Y. S. Setty and Sons and the
final order was awaited from the State Government.
6. The Special Land Acquisition Officer entered into a compromise in Civil Appeal
No.514 of 1971 with M/s. Y. S. Setty and Sons and agreed to reconvey 2 acres 5 guntas of
land in their favour. Pursuant to the compromise reached between the parties, this Court
decreed the appeal in terms of compromise between the parties. The relevant portion of
the order dated 30th August, 1972 reads as under:-
@page-SC641
"The Application for recording compromise above-mentioned being called on for hearing
before this Court on the 30th day of August, 1972. Upon perusing the said application
and the terms of Compromise and upon hearing Mr. R.B. Datar, counsel for the petitioner
residing appellant herein and Mr. K.N. Bhatt, counsel for the respondent, the Court took
on record the said terms of compromise and by and with the consent of the parties herein,
the court decreed the appeal in respect of 2 acres and 5 guntas of land out of 15 acres and
18 guntas of the land in Survey No.76/2 Adugodi Village in terms of the said terms of
compromise annexed thereto as Schedule 'A'."
7. It is alleged that despite the orders passed by this Court in a compromise petition, the
possession of the land was not handed over to the applicants Y. S. Setty and Sons or their
predecessors-in-title.
8. The firm Y. S. Setty and Sons was dissolved on 31.12.1973. The legal heirs and
successors-in-interest of Y. S. Setty and Sons made various representations before the
State Government to reconvey the disputed land in their favour but the same were of no
avail as the Government has not handed over the possession till date. Instead of giving
the possession, the contemnor herein made construction over the disputed land.
9. The erstwhile partners and successors-in-interest of the applicants Y. S. Setty and Sons
filed a writ petition bearing Writ Petition No.4276 of 1995 before the High Court seeking
directions to the contemnor herein to comply with the order dated 30.8.1972 passed by
this court and to direct the State Government to comply with the government order dated
15.9.1967.
10. The applicants herein filed suit OS No.6969/1999 before the City Civil Court at
Bangalore for reconveyance of the scheduled/disputed property. The court granted
interim injunction in favour of plaintiff on 15.2.2000.
11. The contemnor herein challenged the said order before the High Court in MFA
No.1189/2000. The High Court vide order dated 11.4.2000 vacated the interim injunction
and directed the trial court to dispose of the suit untrammeled with the observations made
in the judgment.
12. The applicants herein filed a special leave petition against the order dated 11.4.2000
and filed an interim application in Civil Appeal No.514 of 1971 seeking reconveyance
and delivery of the disputed land. This court vide common order dated 03.12.2001
dismissed the special leave petition and directed the trial court to expedite the hearing of
the suit, if possible within six months. This Court further disposed of the interlocutory
application filed and recorded the statement made by the State Government that "......the
government has executed the conveyance deed. "
13. The State Government executed reconveyance deed dated 31.5.2001 in favour of the
applicants herein by the Deputy Commissioner, Bangalore for the disputed land. In spite
of the execution of the reconveyance deed, the State Government did not hand over
possession of the land to the applicants herein. This attitude of the State Government
compelled the applicants to file a writ petition before the High Court bearing No.18166
of 2002.
14. The learned Single Judge vide Order dated 31.3.2004 allowed the writ petition and
directed as under:-
"Direction is issued to the KMF to restore possession of the schedule land measuring 2
acres 5 guntas in Survey No.76/2 of Audigodo Village, Bangalore, South Taluk, bounded
on the east by Sy.No.76/1, West by 76/2, north by 76/3 and south by Bangalore Hosur
Road, within three months from today."
15. The learned Single Judge had recorded that the contemnor herein, i.e., Karnataka
Milk Federation have, ".....no right whatsoever in respect of the scheduled property, has
no right to resist delivery of possession pursuant to the deed of sale executed by the State
Government in favour of the petitioners." The Court further held that
"The KMF being an instrumentality of the State shall not come in the way of the State
Government in honouring its commitment made in the compromise petition filed before
the Supreme Court, much before the KDDC came into existence."
16. The Court also recorded that, The KMF proceeded to put up construction contrary to
the directions issued by the State Government to deliver possession of the scheduled land
to the petitioners. Further, the KMF being aware of all the above narrated facts proceeded
to put up construction. This Court in the MFA (1189/2000)
@page-SC642
referred to above has observed that the KMF cannot plead any equity in its favour in the
event it has been proceeded to put up construction. The suit filed by the petitioners is also
disposed of since the State Government itself has reconveyed the property under the
registered deed dated 31.5.2001. This Court also disposed of the SLP (C) No.9309/2001
and the application filed in SLP No.514/71 taking into consideration the subsequent
development of the Government reconveying the property in favour of the petitioners
(applicants herein)........ "Therefore, in considered view of the Court the petitioners were
entitled for possession of the scheduled land from the KMF."
17. The contemnor aggrieved by the order dated 31.3.2004 filed a writ appeal before the
Division Bench of the High Court. The Division Bench vide order dated 13.7.2005
dismissed the appeal and held as under:-
"Even though several contentions were raised before the learned Single Judge, the
learned Single Judge considering the fact that a compromise was entered into between the
parties before this Court, the appellant cannot go back from the same. It was contended
before the learned Single Judge that the State Government had no authority to enter into a
compromise because the land when once acquired and handed over to Bangalore Dairy, it
cannot be de-notified and given back to the owners. That contention has been rightly
rejected by the learned Single Judge. The Government of then State of Mysore acquired
the land for the Bangalore Dairy which was a wing of the State Government at that time.
The appellant Corporation had not come into existence at that time. The acquisition was
therefore for itself. Therefore, the Government had rightly agreed for the compromise
before this Court. Hence, there was nothing wrong in the Government agreeing to de-
notify the land which was acquired earlier. The present appellant which came into
existence later on in place of the earlier Bangalore Dairy cannot question what had been
done by the State Government in their own interest before this Court. The Division Bench
therefore did not see any error committed by the learned Single Judge in rejecting the
contention of the appellant."
18. The contemnor herein filed a special leave petition against the order dated 13.7.2005
passed by the learned Division Bench. The special leave petition was dismissed by this
Court vide order dated 5.12.2005. This Court held as under :
"We do not find any ground to interfere with the order. The SLP is dismissed
accordingly."
19. Thereafter, the applicants through their advocates sent a legal notice dated 17.1.2006
and called upon the contemnor herein to comply with the orders dated 30.8.1972 and
5.12.2005 passed by this Court. The contemnor addressed a letter dated 25.1.2006 to the
applicants herein and has merely made an offer to give possession of the land which is
neither the scheduled land nor adjoining to the scheduled land.
20. The applicants thereafter had sent another letter dated 6.2.2006 through their advocate
and again called upon the contemnor to comply with the orders dated 30.8.1972 and
5.12.2005 passed by this Court and to deliver possession of 2 acres and 5 guntas of land
as per the compromise reached between the parties before this Court. The contemnor-
federation herein through its advocate addressed another letter dated 14.2.2006 to the
advocate of the applicants herein and reiterated the offer made in the letter dated
25.1.2006.
21. In the contempt petition, it is alleged that the contemnor had made all attempts to
avoid compliance of the orders of this Court and made every efforts to circumvent and
over-reach the orders passed by this court.
22. It may be pertinent to mention that the Map which has been annexed with this petition
shows that initially the contemnor had agreed to hand-over 2 acres 5 guntas of land from
the portion marked as "A". The contemnor-federation herein showed its inability to give
that portion, therefore, the applicants herein YS Setty and Sons had agreed to take 2 acres
5 guntas of land, as per Map shown in Schedule B which the contemnor agreed to give to
the applicants.
23. We have heard learned counsel for the parties at length and we find considerable
merit in this application. On consideration of the totality of the facts and circumstances,
we direct the respondent-federation to hand-over possession of 2 acres 5 guntas, as per
annexed Map shown as Scheduled "B" without any loss of time.
24. We further direct that the cost of the reconstruction of the existing buildings shall
@page-SC643
be valued by an independent Government approved valuer within two weeks from today.
The re-construction cost so fixed shall be deposited within a week from the report of the
valuer.
25. The respondent-federation will be entitled to demolish the buildings within 8 weeks
from the date of deposit and shall hand over vacant possession of the area of 2 acres 5
guntas as per Schedule "B". The original re-conveyance deed shall be rectified within
four weeks of handing over vacant, peaceful and physical possession by the respondent-
federation.
26. In case there is non-compliance of the orders of this Court, the court will take very
serious view of the matter and pass appropriate orders.
27. The contempt petition is disposed of accordingly.
Order accordingly.
AIR 2008 SUPREME COURT 643 "Prabha Arora v. Brij Mohini Anand"
(From : 2007 (4) ALJ 404 (Utr)
Coram : 2 A. K. MATHUR AND MARKANDEY KATJU, JJ.
Civil Appeal No. 2371 of 2007, D/- 31 -10 -2007.
Prabha Arora and Anr. v. Brij Mohini Anand and Ors.
U.P. Urban Buildings (Regulation of Letting, Rent and Eviction) Act (13 of 1972),
S.21(1)(a) - EVICTION - TENANCY - APPEAL - TRUST - Eviction of tenant - Sought
by landlady, a retired teacher getting meager pension to open tutorial center - Application
for release of premises allowed - Appeal - During its pendency Trust was created in
respect of property in question - Purpose mentioned in application for release under S. 21
was not for doing charitable work - Held, need for which eviction of tenant was sought
totally disappeared.
2007 (2) ALJ 404 (Utr), Reversed.
AIR 1961 SC 1023, and AIR 1981 SC 1113, Foll. (Paras 4, 8, 9)
Cases Referred : Chronological Paras
2004 AIR SCW 5789 : 2004 All LJ 3892 5
AIR 1981 SC 1113 (Foll.) 7
AIR 1981 SC 1711 5
AIR 1961 SC 1023 (Foll.) 6
Dinesh Dwivedi, Sr. Advocate, P. N. Gupta, for Appellants; M. N. Krishnamani, S. C.
Maheshwari, Sr. Advocates, Ms. J. Quddisi, Vipul Maheshwari, M. P. S. Tomer, Ms.
Sandhya Goswami, for Respondents.
Judgement
1

. MARKANDEY KATJU, J.:- This appeal has been filed against the judgment dated
09.10.2006 passed by the Uttaranchal High Court in Writ Petition No. 337 of 2004 (M/S).
Heard learned counsel for the parties and perused the records. reported in 2007 (2)
ALJ 404

2. The appellants before us are the tenants of the premises in dispute while the
respondents are the landlords. The landlady filed the petition under Section 21 (1) (a) of
the U.P. Urban Buildings (Regulation of Letting, Rent and Eviction) Act, 1972. The
grounds mentioned in the release application of the landlady was that she is a retired
teacher getting only a pension of Rs. 538/- per month which is insufficient for her needs.
Hence to augment her income she wants to run tuition/coaching classes in the premises in
question. The said petition was rejected by the Prescribed Authority, but in appeal the
Appellate Authority (ADJ Dehradun) by his judgment dated 16.03.2004 reversed the
order of the Prescribed Authority and allowed the release application.The said judgment
dated 16.03.2004 has been upheld by the High Court by the impugned judgment dated
09.10.2006. Hence this appeal.
3. It may be mentioned that during the pendency of the appeal before the Appellate
Authority a Trust was created in respect of the property in question vide trust deed dated
04.08.2003 (copy of which is annexed as Annexure P-8 to this appeal).
4. Learned counsel for the appellants, Shri Dinesh Dwivedi, submitted that in view of the
aforesaid trust deed dated 04.08.2003 the very purpose for which eviction was sought of
the tenants through the release application has disappeared. We are in agreement with this
submission.The trust deed nowhere mentions that any income of the Trust will be given
to the petitioner who filed the release application. In fact, Section 51 of the Trust Act
debars a trustee from using the trust property for his own profit.

. In Kedar Nath Agrawal (dead) and Anr. v. Dhanraji Devi (dead) by LRs. and Anr. [2004
(8) SCC 76] this Court held that the court has to consider the changed circumstances
2004 AIR SCW 5789

@page-SC644
during the pendency of the litigation. This decision relied on the earlier decision of this
Court in Hasmat Rai v. Raghunath Prasad [1981 (3) SCC 103: AIR 1981 SC 1711] in
which it was observed that where possession is sought for personal requirement, the said
requirement must not only exist on the date of the filing of the petition but must also
subsist till the final decree for an order for eviction is made. If, in the meantime, events
crop up which would show that the landlord's requirement no longer subsists then the
action must fail.
6. In Tulsidas Kilachand and Ors. v. Commissioner of Income Tax [AIR 1961 SC 1023] it
was held that on creation of a Trust the property passes to the trustees. Hence, in our
opinion, rent is now to be paid to the trustees who will collect it on behalf of the Trust.
7

. In M.M. Quasim v. Manohar Lal Sharma and Ors. [1981 (3) SCC 36] this Court held
that on transfer of property to a person who is not a party to the proceedings the suit for
eviction will fail. AIR 1981 SC 1113

8. Learned counsel for the respondents, Shri M.N. Krishnamani, submitted that the
petition under Section 21 for eviction was filed on the ground that the petitioner wanted
to do charitable work, and after creation of the Trust also the purpose remains the same.
We do not agree. In the petition under Section 21 it is stated in paragraph Nos. 3-7 of the
petition that the petitioner has a monthly pension of Rs. 538/- only and she wants to
augment her income as it is difficult for her to survive on the meager pension. Hence she
wants to open a tutorial centre in the premises in dispute to earn some money. The
purpose mentioned in the petition under Section 21 was not for doing charitable work.
However, after the execution of the trust deed the premises in dispute now belongs to the
Trust. The need mentioned in the petition under Section 21 has totally disappeared.
9. In this view of the matter, the impugned judgment of the High Court dated 09.10.2006
and of the Appellate Authority dated 16.03.2006 cannot be sustained and are set aside.
10. Appeal is allowed, no order as to costs
Appeal allowed.
AIR 2008 SUPREME COURT 644 "L. P. Shashi Kumar v. Industrial Development Bank
of India Ltd."
(From : Andhra Pradesh)
Coram : 2 Dr. A. R. LAKSHMANAN AND ALTAMAS KABIR, JJ.
Civil Appeal No. 644 of 2007 (arising out of SLP (C) No. 9949 of 2006), D/- 8 -2 -2007.
L. P. Shashi Kumar v. Industrial Development Bank of India Ltd. and Ors.
Constitution of India, Art.133 - APPEAL - LOAN - SUPREME COURT - BANKING -
Appeal - Directions by Court - Settlement of term loan - Guarantor deposited amount
with Bank as directed by Supreme Court towards full and final settlement of dues of
Bank - Bank directed to release securities to guarantor - Bank also directed to withdraw
writ petition filed before High Court as also the guarantor to withdraw case filed before
State Consumer Commission.(Para 6)

G. Bikshapathy, Sr. Advocate, Ms. T. Anamika and A. Chandra Mohan, for Appellant;
Sanjay Bhatt and Amit Kumar, for Respondents.
Judgement
Dr. AR. LAKSHMANAN, J. :-Leave granted.
2. Heard Mr. G. Bikshapathy, learned senior counsel appearing on behalf of the appellant
and Mr.Sanjay Bhatt, learned counsel appearing on behalf of the contesting respondent
No.1 (Industrial Dev. Bank of India Ltd.).
3. This appeal was filed against the order passed by the High Court of Andhra Pradesh
dated 27.04.2006 in W.P.No.7951 of 2006 and W.P.M.P.No.10172 of 2006. The High
Court was approached by the first respondent herein against the direction given by the
A.P. State Consumer Disputes Redressal Commission directing release of security to the
appellant herein. The High Court stayed the operation of the order dated 06.04.2006
passed by the A.P. State Consumer Disputes Redressal Commission, Hyderabad in CDIA
No.33/2006 (CD No.1/2006). When the matter came up for admission before this Court
on 19.06.2006, this Court passed the following Order :-
"Issue notice returnable within six weeks.
In the meanwhile, the operation of the impugned order passed by the High Court in
WPMP No. 10172/2006 in WP No.7951/2006 dated 27.04.2006 shall remain stayed.
In the meantime, either the principal
@page-SC645
debtor or the guarantor are at liberty to deposit the money with the bank so that this Court
may consider the prayer for return of share certificates and title documents to the
petitioner-guarantor. This direction is issued since the GTL is ready to pay the
outstanding principal amount of Rs.4.80 crores along with 9% interest per annum from
December, 2003."
4. By Order dated 31.10.2006, the main Special Leave Petition itself was directed to be
listed for final hearing. Today, when the matter is taken up for hearing, learned senior
counsel appearing on behalf of the appellant placed before us a communication dated
01.02.2007 addressed to the Deputy General Manager, Industrial Development Bank of
India Limited, Hyderabad. The said communication reads thus :-
"Date : 01-02-2007
To
The Deputy General Manager,
Industrial Development Bank of India Limited,
Main Branch,
Hyderabad - 500 001
Dear Sir,
Sub.:-Full and Final Settlement of Term Loan of Rs.1000 lacs.
Ref.:- 1) Our Term Loan account GOLT0101100,
GOLT0101101 and GOLT0101102
2) - Your letter bearing No. IDBI(H) No.5278/CFD(GTL)
With reference to the above-cited subject, please find enclosed Cheque bearing
No.672421 for Rs.54,22,237.00, drawn on M/s. UCO Bank, Abid Circle Branch, towards
full and final settlement of Term Loan referred in (1) above.
We request yourselves to acknowledge the receipt of the abovesaid payment, issue letter
towards full and final settlement of the said Term Loan and release all charges created on
the assets of our company, personal guarantees executed by the management of our
company in favour of IDBI, release pledge of 15,00,000 equity shares of Goldstone
Technologies Ltd., held by Goldstone Exports Ltd. and any other securities/sureties
executed in this regard.
Thanking you
Yours truly,
For Goldstone Technologies Ltd.
Sd/-
D.P. Sreenivas
Executive Director"
5. The learned senior counsel for the appellant has also placed before us a communication
by way of reply to the communication dated 01.02.2007 sent to the Managing Director,
Goldstone Technologies Ltd. The said letter reads thus :-
"IDBI(H) No./CFD(GTL) February 3, 2007
The Managing Director,
Goldstone Technologies Ltd.,
Amarchand Sharma Complex,
S.D.Road, Secunderabad - 500 003.
Dear Sir,
Rupee term loan of Rs.1000 lakh - Settlement of dues
Please refer to your letter dated February 1, 2007 forwarding a cheque for Rs. 54,22,237/-
(Rupees fifty four lakh twenty two thousand two hundred thirty seven only) towards full
and final settlement of dues of IDBI. In this connection, we advise that on receipt of the
aforesaid payment, there are no dues from the company to IDBI and the term loan
account of Rs.1000 lakh stands fully repaid. We, therefore, advise you to arrange to
withdraw the cases pending before the A.P. State Consumer Disputes Redressal
Commission, Hyderabad and Hon'ble Supreme Court of India to enable us to release the
securities etc. at the earliest.
Yours - faithfully,
Sd/-
(V. V. Rao)
Dy. General Manager"
6. It is thus seen that in compliance of the Order of this Court dated 19.06.2006, the
guarantor deposited the money with the first respondent. Since the sum of Rs. 54,22,237/-
(Rupees Fifty four lakh twenty two thousand two hundred thirty seven only) has been
paid by the appellant to the first respondent-bank towards full and final settlement of the
dues of IDBI, the IDBI has requested the appellant herein to withdraw the cases pending
before the A.P. State Consumer Disputes Redressal Commission, Hyderabad and in this
Court to enable them to release the securities at the earliest. In view of the payment of Rs.
54,22,237/- made by the appellant towards full and final settlement of the dues of IDBI,
the said bank is directed to release the securities to respondent No.3 (Goldstone
Technologies Ltd.) forthwith. In view of the final payment now received by IDBI, there is
nothing further
@page-SC646
survives for the bank to pursue the Writ Petition No.7951/2006 filed by them before the
High Court. Therefore, the IDBI is directed to withdraw the said Writ Petition from the
High Court. Likewise, the appellant is also directed to withdraw the case filed by them
before the A.P. State Consumer Disputes Redressal Commission, Hyderabad
immediately. The zerox copies of the letter dated 01.02.2007 and 03.02.2007 are placed
on record and will form part of this Order.
7. We make it clear that the deposit of sum of Rs. 54,22,237/- now made by the appellant
is in addition to the sum of Rs. 89,96,636/- deposited by the guarantor pursuant to our
interim order dated 19.06.2006 in full and final settlement of the bank dues.

8. The appeal stands disposed of accordingly.


9. No costs.
Order accordingly.
AIR 2008 SUPREME COURT 646 "Rabindra Kumar Shaw v. Manick Lal Shaw"
(From : Calcutta)*
Coram : 2 Dr. A. PASAYAT AND LOKESHWAR SINGH PANTA, JJ.
Civil Appeal No. 4926 of 2007 (arising out of SLP (C) Nos. 19179 and 19180 of 2005),
D/- 22 -10 -2007.
Rabindra Kumar Shaw (D) by L.Rs. v. Manick Lal Shaw.
Civil P.C. (5 of 1908), O.39, R.1, R.2 - DECLARATION OF TITLE - INJUNCTION -
POSSESSION - Suit for declaration of title and permanent injunction - Application by
plaintiff for interim injunction restraining defendant from interfering with possession of
plaintiff and from taking forcible possession - Rejection of, on ground of non-
impleadment of three sons of defendant in suit - Subsequently on application by plaintiff,
three sons of defendant impleaded in suit - In view of changed circumstances, matter
remitted to trial Court for fresh consideration. (Para 5)

Tapash Ray, Sr. Advocate, Bijan Kumar Ghosh and S. K. Poddar, for Appellants; Jaydeep
Gupta, Satyajit Salia, Mrs. V. D. Khanna and Rajkumar Laholi, for Respondent.
* F.M.A. No. 1471 of 2005, D/- 23-3-2005 (Cal)
Judgement
1. Dr. ARIJIT PASAYAT, J. :- Leave granted.
2. Challenge in these appeals is to the order passed by a Division Bench of the Calcutta
High Court allowing the appeal filed by the respondent-Manick Lal Shaw. The appeal
was filed by the respondent who was the defendant in the suit for declaration of title and
permanent injunction. The same was directed against the order dated 4th December, 2004
passed by learned Judge, 10th BENCH, City Civil Court at Calcutta in Title Suit No. 815
of 2000 thereby rejecting the application under Order 39, Rule 4 of the Code of Civil
Procedure, 1908 (in short the 'CPC') filed by the defendant and allowing the application
under Order 39, Rules 1 and 2 filed by the plaintiffs.
3. During the pendency of the suit, application in terms of Order 39, Rules 1 and 2, CPC
was filed praying for an order of injunction and restraining the defendant from interfering
with the possession of the plaintiff in the suit property and from taking forcible
possession by breaking open the padlock in the suit property. On such application,
learned trial Judge granted ad interim order of status quo. Against such order the
defendant filed an appeal before the High Court which was heard by a Division Bench
and the said Division Bench did not interfere with the order as the main application for
injunction was yet to be decided on merits. Plaintiff filed an application under Section
151 of CPC for enforcing the said ad interim order of status quo with the help of police
and the learned trial Judge allowed the application. The defendant filed a revision before
the High Court but the High Court did not interfere with the said order on the ground that
so long as the ad interim order was subsisting there was no reason for interference with
the order for implementation of the order. Subsequently, an application under Order 39,
Rule 4, CPC was filed by the defendant for vacating the earlier interim order. The High
Court noted that it would have normally remitted the matter to learned trial Judge for
consideration of the application under Order 39, Rule 4, CPC and the written objection
filed to the original
@page-SC647
application for injunction on merits. But it was pointed out that in the suit, plaintiff had
not impleaded the three sons of the defendant who had admittedly become co-owners of
the property along with the defendant and as such no effective order of injunction can be
passed in the suit in the absence of all co-owners of the property. The High Court,
therefore, held that in the circumstances it was a fit case where application for injunction
filed by the plaintiff was to be dismissed in the absence of necessary parties to the suit
and on that ground alone the application was dismissed. The High Court noted that it had
not gone into the merits of the case and only on the technical ground as noted above, the
application for temporary injunction was rejected. In view of the dismissal of the appeal
the application No. CAN 1209/2005 had become infructuous.
4. During hearing of the appeal, learned counsel for the appellants who are the legal heirs
of Rabindra Kumar Shaw, the original plaintiff submitted that the High Court had not
decided the case on merits and had passed the impugned order only on the technical
ground that the three sons of the defendant who are co-owners had not been impleaded.
As a matter of fact subsequently an application in terms of Order 1, Rule 10(2) read with
Section 151, CPC was filed by the plaintiff on 8-11-2005 for impleading the three sons of
the defendant. The prayer was accepted by the trial Judge by order dated 19-4-2005.
5. As the basic objection as to the maintainability of the application no longer survives in
view of the impleadment of the three sons of the defendant, the matters need to be heard
afresh. As noted above, the High Court noted that it had not gone into the merits of the
case and except on the technical ground of non-impleadment of the three sons of the
defendant, the application for temporary injunction was rejected. In view of the changed
circumstances we remit the matter to the trial Court to consider the matter afresh. The
effect of the impleadment of the three sons of the defendant, needless to say, shall be
considered by the trial Court.
6. The appeals are accordingly disposed of. There will be no order as to costs.
Order accordingly.
AIR 2008 SUPREME COURT 647 "State of Tamil Nadu v. Seshachalam"
(From : Madras)*
Coram : 2 S. B. SINHA AND H. S. BEDI, JJ.
Civil Appeal Nos. 1938 with 1940 to 1942, 1944, 1946, 1947, 1949, 1950, 1952, 1954,
1955, 1957, 1958 and 1960 of 2007, D/- 18 -9 -2007.
State of T.N. v. Seshachalam.
Constitution of India, Art.226, Art.14, Art.16 - WRITS - EQUALITY IN PUBLIC
EMPLOYMENT - SERVICE MATTERS - Delay and laches - 'One unit' system for all
departments of Secretariat of State Govt. - Finance and Law Departments, however,
excluded from 'one unit' system - No challenge as to, filed by any aggrieved employee
from 'one unit' - Notification No. 126, D/- 29-5-1998 providing for up-gradation of post
and stepping up pay of employees of 'one unit' accordingly, on par with their juniors in
Finance Department - Applications by some employees who were retired long back prior
to Notification of 1998, for stepping up their pay in view of Notification - Liable to be
dismissed on ground of delay.
W. P. No. 7971 of 2004, D/- 21-4-2006 (Mad), Reversed.
Delay or laches is a relevant factor for a Court of law to determine the question as to
whether the claim made by an applicant deserves consideration. Delay and/or laches on
the part of a Govt. Servant may deprive him of the benefit, which had been given to
others. Art. 14 would not, in a situation of that nature, be attracted, as it is well known
that law leans in favour of those who are alert and vigilant. Moreover, legitimate
expectation is a part of the principles of natural justice. No fresh right can be created by
invoking the doctrine of legitimate expectation. By reason thereof only the existing right
is saved subject, of course, to the provisions of the statute. (Para 11)
Where there existed 'one unit' system for promotion of employees in all departments of
Secretariat, however, the Finance and Law Departments were excluded from such 'one
unit' system and such different treatment meted out to Finance and Law Departments
where employees junior to those serving in other department got promotion on fortuitous
circumstances, was never questioned
@page-SC648
by employees of other departments, by filing representations or by filing applications
before Administrative Tribunal and for the first time after Notification GOMS 126, D/-
29-5-1998 providing for up-gradation of pay of senior employees revised on or before
28-1-1994 in 'one unit', on par with their juniors in Finance Department, application for
the up-gradation of their pay accordingly, was filed by the employees who were retired
long back prior to Notification D/- 29-5- 1998, such applications were liable to be
dismissed on ground of delay in filing same. In such a case, only because a cut- off date
was fixed, the same per se cannot be said to be arbitrary as some date was required to be
fixed for that purpose.
W. P. No. 7971 of 2004, D/- 21-4-2006 (Mad), Reversed.
Moreover, the notification envisaged a personal pay by way of stepping up of pay. It was
given the prospective effect. No arrear of pay was to be paid. The upgradation sanctioned
was to lapse in the event of retirement of the individuals or their promotion to the
upgraded post. The said up-gradations were to be subject to the terms and conditions
contained in Cl. 12 of the said order, a reading whereof would clearly, lead to only one
conclusion that it was meant to be applied to the existing employees. It would, therefore,
be incorrect to construe that the notification applied to all who had been recruited to the
Tamil Nadu Secretariat Service on or before 28-1-1994. The Court should not give a
strained or extended meaning thereto. While construing such a notification, the financial
impact thereof is also required to be taken into consideration. (Paras 14, 15)
Cases Referred : Chronological Paras
2007 AIR SCW 77 : 2007 Lab IC 991 : 2007 (2) AIR Kar R 34 (Foll.) 12
2007 AIR SCW 672 : AIR 2007 SC 924 : 2007 (2) ALJ 242 (Foll.) 12
2007 AIR SCW 1705 : AIR 2007 SC 1365 (Foll.) 12
2007 AIR SCW 3752 : AIR 2007 SC 1984 (Foll.) 13
2006 AIR SCW 2676 : AIR 2006 SC 2145 : 2006 (4) AIR Kar R 194 (Disting.) 20
2006 AIR SCW 3889 : AIR 2006 SC 2844 : 2006 Lab IC 3301 : 2006 (5) AIR Kar R 434
15
2005 AIR SCW 6045 : AIR 2006 SC 407 : 2006 Lab IC 137 (Foll.) 15
(2005) 1 SCC 625 (Foll.) 13
(2004) 1 SCC 347 (Foll.) 12
1998 AIR SCW 186 : AIR 1998 SC 591 13
1997 AIR SCW 3839 : AIR 1997 SC 3910 : 1997 All LJ 2202 13
(1997) 3 SCC 398 13
1992 AIR SCW 1247 : AIR 1992 SC 1277 : 1992 Lab IC 1371 (Foll.) 11
1991 AIR SCW 704 12
(1987) 4 SCC 31 16
AIR 1983 SC 130 : 1983 Lab IC 1 12
AIR 1974 SC 1 : 1974 Lab IC 1 (Disting.) 19
AIR 1952 SC 75 : 1952 Cri LJ 510 (Disting.) 17
(1896) 165 US 150 17
M. S. Ganesh, Sr. Advocate, V. G. Pragasam, S. Joseph Aristotle and S. Prabu Rama-
subramanian, for Appellant; Dr. A. E. Chelliah, R. Venkatramani, Sr. Advocates, Ms.
Vasanthakumari Chelliah, K. Ramanujam, Ms. A. Annapurai, Dinesh Kumar Garg, R.
Anand Padmanabhan, Pramod Dayal, Mrs. S. Lesi and Namachivayam (for P. Nara-
simhan), for Respondent; Caveator-in-person.
* W.P. No. 7971 of 2004, D/- 21-4-2006 (Mad).
Judgement
1. S. B. SINHA, J. These appeals involving identical questions of law and fact were taken
up for hearing together and are being disposed of by this common judgment.
2. Respondents herein have been working in the Secretariat of the Government of Tamil
Nadu. Each and every department in the Government Secretariat prior to 1961 had a
separate unit for appointment, promotion etc. The State, however, amended the Special
Rules in the year 1961 whereby all the departments in the Secretariat were made the "one
unit" for the purpose of appointment and promotion. Appointments in the Secretariat at
all entry level posts, i.e., Junior Assistants (subsequently re-designated as Assistants),
Assistants (subsequently re-designated as Assistant Section Officers), Typist/Personal
Clerks were to be made from the common list of candidates selected by the Tamil Nadu
Public Service Commission. Promotion to different higher posts in different departments
was also being made from amongst those employees. The Government of Tamil Nadu,
however, by issuing G.O.Ms. No.1290, dated 05.06.1970 excluded the Finance and Law
Departments from the "one unit" system. Whereas posts in the cadre of Assistants,
Assistant Section Officers, Typists/Personal Clerks continued to be filled up from the
common list of candidates, but in Finance and Law Departments, further promotions
@page-SC649
were effected from amongst the employees allotted thereto only. Appointments to
Finance Department, however, were made at random and probably in terms of the option
exercised by any particular candidate. Many persons, who have, thus, been ranking
higher were employed in "one unit" departments whereas some of the candidates ranking
lower were employed under fortuitous circumstances in the Finance Department. The
employees working in the Finance Department, therefore, obtained promotions much
ahead of their peers or even seniors who were discharging their duties in other
departments coming within the "one unit".
3. G.O.Ms. No.3288 (Public Services Department) was thereafter issued on 29.10.1971
specifying Finance and Law Departments as separate units from the level of
Superintendent (Section Officer) and above. Admittedly, however, Rule 4 of the Special
Rules of the Tamil Nadu Secretariat Service was amended in that behalf. The said policy,
however, is said to have been implemented. Two employees, S. Kalaiselvan and S.
Sivasubramanian, filed an Original Application before the Tamil Nadu Administrative
Tribunal in the year 1990 claiming promotion and scale of pay at par with those who
were working in the Finance Department and who were said to be juniors to them but had
been promoted to higher posts in Finance Department. The said Original Application was
allowed by the Tribunal by an order dated 16.4.1993 opining that there existed no
guidelines to allot any employee to the Finance Department, vis-a-vis, other departments
and, thus, the employees working in other departments could not have been deprived of
the benefit of promotion. It was furthermore pointed out that even Rule 4 of the Special
Rules for the Tamil Nadu Secretariat Service had not been amended by the said GOMs
No.1290 dated 05.06.1970.
4. The Government of Tamil Nadu thereafter amended the Service Rules with
retrospective effect from 05.06.1970 by issuing G.O.Ms. No.30 Personnel and
Administrative Reforms (D) Department, dated 28.1.1994. Upon issuance of the said
Government Order, an application for review was filed but the same was dismissed by the
Tribunal by an order dated 30.1.1995. The Government was thereafter advised to
implement the order of the Tribunal by giving promotion to the concerned employees
with retrospective effect from the date on which their juniors had been promoted as
Assistant Section Officers in the Finance Department. Sanction was also accorded for
creation of two supernumerary posts, namely, posts of Assistant Section Officers in the
respective departments. Several representations thereafter were made by persons said to
be similarly situated claiming promotion and parity in the scale of pay as compared to
their counterparts in the Finance Department. A large number of Original Applications
were also filed before the Tamil Nadu Administrative Tribunal. Upon consideration of
various pros and cons, the Government of Tamil Nadu issued a GOM bearing No.126,
dated 29.5.1998, relevant paragraphs whereof read as under :
"10. The Government accordingly direct that :-
(i) the pay of the seniors in One Unit who have been recruited to the Tamil Nadu
Secretariat Service on or before 28.1.1994, shall be stepped up on par with their juniors in
the Finance unit by upgrading the posts held by them to the scale of pay applicable to the
juniors with immediate effect.
(ii) The stepping up of their pay on par with the juniors in the Finance Unit by upgrading
the posts held by them to the scale of pay applicable to the junior ordered in sub-para (1)
above is purely a person-oriented upgradation and no new posts will be created for this
purpose.
(iii) The upgradation sanctioned for the seniors will lapse in the event of the retirement of
the individuals concerned or their promotion to the upgraded post in their normal turn.
(iv) The pay of the other seniors in the One Unit in the same cadre will be stepped up on
par with immediate juniors in the Finance Unit, with effect from the date of issue of this
order.
(v) In respect of the Typists/Personal Clerks/Personal Assistants, in One Unit who have
not relinquished their right for promotion as Assistant Section Officer, and are still
awaiting their turn for promotion as Assistant Section Officer, their pay shall be upgraded
to Assistant Section Officer scale on par with their immediate junior in the Finance Unit
who got his promotion as Assistant Section Officer.
11. The benefits of upgradation of pay of the seniors on par with their juniors as per
@page-SC650
Commission's Seniority list ordered in sub-paras (i) to (iv) of Para 10 above, shall also be
extended to those seniors in the Finance Unit who were recruited before 28.1.1994 and/or
drawing less pay than their juniors in One Unit.
12. The upgradation ordered above is subject to the following terms and conditions :
(1) The upgradation ordered will involve only stepping up of pay of the senior on par
with his junior in the upgraded scale of pay.
(2) It does not entitle him to any claim for arrears of pay.
xxx xxx xxx
These orders shall come into force with effect from the date of issue of the orders.
13 ...
14. The Departments of Secretariat concerned shall issue necessary orders for
upgradation of posts and for stepping-up of the pay of the Seniors in One Unit in the
upgraded scales ordered in para 10 above, after obtaining necessary individual
undertaking in the format enclosed from the seniors concerned to the effect that they
accept the terms and conditions of this order."
5. The said Government Order further stipulated that undertaking should be given by the
seniors getting upgradation of their pay with their juniors in the Finance Department in
the format enclosed to the effect that they accept the terms and conditions thereto.
Respondents before us, save and except R. Ragothaman in CA No.1955 of 2007
indisputably had retired much prior to issuance of the said Government Order dated
29.5.1998. They also made representations before the appellant demanding fixation of
their pay at par with their juniors in the Finance Department. As the said request was not
acceded to, a large number of original applications were filed before the Tamil Nadu
Administrative Tribunal. By a common judgment pronounced on 20.1.2004, the Tribunal
dismissed the said applications opining that the same were barred by limitation. It was
held that the applicants having retired long back and having filed applications between
1998 to 2003 and the promoters having retired as Under Secretaries, Deputy Secretaries
and Joint Secretaries and in some cases as Additional Secretaries, they should have raised
the dispute long back when their juniors had been given promotions in the Finance
Department and as the original applications were filed after 20 years, the same could not
be entertained.
6. Aggrieved by the said order of the Tribunal, respondents filed writ petitions before the
High Court of Judicature at Madras. By reason of the impugned judgment dated
21.4.2006, a Division Bench of the High Court, inter alia, held that the cause of action for
filing the original application arose only upon issuance of GOMs No.126, dated
29.5.1998 and in that view of the matter it cannot be said that the original applications
filed by the respondents suffered from delay and laches and/or otherwise barred by
limitation as GOMs No.126 applied also in respect of those who had retired before
29.5.1998. It was also opined that the respondents who had not been in service on or
before 28.1.1994 came within the scope and ambit of the said GOMs. Although GOMs
126 provided for operation with prospective effect and by reason thereof past benefits
were not made available, the same should be construed in consonance with the provisions
contained in Article 14 of the Constitution of India, holding :
"There is no specific clause in G.O.Ms. No.126 excluding the applicability of this G.O. to
the persons who had retired before 29.5.1998. The G.O. itself recites that the Government
wanted to provide a solution to the long standing problem and had decided to take a
sympathetic view to effect lasting and equitable solution to the long standing issue so as
to redress the grievances of the seniors in the One Unit by upgrading the pay of the
seniors in One Unit on par with their immediate juniors in the Finance Unit.
Keeping in view the explicit intention of the Government, it is apparent that the G.O. had
been issued as a beneficial measure and the provisions in such G.O. are to be liberally
construed so as to benefit the employees for whose benefit the G.O. was avowedly
issued. It is not disputed that the petitioners were in service on and before 28.1.1994 and,
therefore, they fall within the scope and ambit of the impugned G.O. Once they are
covered under the said G.O., the benefit of the said G.O. will flow automatically."
It was further held that the said Government Order applied not only to the existing staff
but also to the retired employees and as such a beneficial interpretation to the said
@page-SC651
Government Order should be given as the said provisions have to be read in consonance
with Article 14 of the Constitution of India.
7. Mr. M. S. Ganesh, learned senior counsel appearing on behalf of the appellants, would
submit that the High Court committed a serious error in passing the impugned judgment
in so far as it failed to take into consideration that :
(i) no explanation was offered by the respondents for not preferring the claim petitions
prior to or immediately after the announcement of the order dated 16.04.1993;
(ii) the High Court committed serious error in interfering with the finding of fact arrived
at by the Tribunal;
(iii) doctrine of legitimate expectation does not postulate conferment of any right which
has been lost for any reason whatsoever; and
(iv) as in terms of Articles 14 and 16(1) of the Constitution, no employee has any
fundamental right of promotion, upgradation, allocation of any particular department or
to receive any benefit after superannuation the impugned judgment is unsustainable.
8. Mr. Venkataramani, Dr. A.E. Chelliah, senior counsel appearing on behalf of
respondents and caveator-in-person on the other hand contended that :
(i) from the perusal of the order of Tribunal dated 16.04.1993, it would appear that
observations made therein were not confined only to the two employees who had filed the
original application but covered the cases of all others similarly situated;
(ii) having regard to the fact that the State was required to respond comprehensively to
the said observations and that the review application filed by the State in view of the said
order has been dismissed, before issuing the GOMs No.126, dated 29.5.1998, the State
must be held to have considered the ground realities as also the plight of those employees
who had suffered discrimination, irrespective of the fact as to whether they were in
service or had retired.
(iii) GOMs No.126, dated 29.5.1998 must be given effect to for stepping up the scale of
pay of the employees to bring them at par with their juniors in the Finance Department
with the object of treating all the employees equally;
(iv) no new right having been created by GOMs No.126, dated 29.5.1998, any mini
classification or micro classification would offend Article 14 of the Constitution of India
as there was no rational object behind the same and it is not possible to segregate the
cases of the employees in service vis-a-vis the retired employees.
9. The employees of the Finance and Law Departments were being treated differently
from a long time. The respondents herein never questioned the purported different
treatment meted out to them by the State either by making representations or by filing
any application before the Central Administrative Tribunal. Only two of the employees
did. Their applications were allowed, inter alia, on the premise that posting of employees
in the Finance and Law Departments took place by way of fortuitous circumstances and
were not supported by any rationality. The State, we have noticed hereinbefore, amended
the Rules with retrospective effect. The said Rule is still in force. Validity of the said Rule
has not been questioned by the respondents. Different treatments meted out to the
employees of the Finance and Law Departments vis-a-vis other department is now
covered by Rules, but despite the same, the State intended to assuage the feelings of the
employees by issuance of the said GOMs No.126, dated 29.5.1998. The said notification
was issued upon considering various factors including pendency of a large number of
matters before the Administrative Tribunal on the said issue. The State intended to lay
down a policy for providing financial benefits with prospective effect. Various pros and
cons therefor were examined. Avenues available to the State were taken to into
consideration. Only thereafter it was directed :
"The Government, after careful consideration of all these points and also the related
issues involved, have decided to take a sympathetic view and attempt a lasting and
equitable solution to this long standing issue, so as to redress the grievances of the seniors
in the One Unit, by upgrading the pay of the seniors in One Unit on par with their
immediate juniors in the Finance Unit."
10. It is one thing to say that the State had come up with a policy decision which is
beneficial to all the employees irrespective of the fact as to whether they had reached the
age of superannuation or not, the only criteria being that they were recruited to the
@page-SC652
Tamil Nadu Secretariat Service on or before 28.1.1994 but it is another thing to say that
the claim petitions filed by the responders were based on the success of their colleagues
before the Administrative Tribunal in the year 1994. The employees working in the
Finance Department had been promoted long back. We have noticed hereinbefore that
some of them retired as Additional Secretaries whereas the respondents retired as merely
Assistants. Presumably, promotions to the employees of the Finance Department were
given systematically over a long period of time but no such grievance was made nor any
application was filed before the appropriate forum. Such grievance, in our opinion,
should have been raised or proper application before the Tribunal should have been filed
long long back. It was in the aforementioned situation, the Tribunal was of the opinion
that their applications were barred by limitation. Assuming that the cause of action for
filing such applications arose in view of the observations made by the Tribunal in its
order dated 16.4.1993 passed in Original Application No.166 of 1990, but then in terms
of the Act and the Rules, the respondents were required to file a proper application within
a period of one year only. It is borne out from the records that, in fact, 62 such
applications were already pending when GOMs No.126 was issued.
11

. Some of the respondents might have filed representations but filing of representations
alone would not save the period of limitation. Delay or laches is a relevant factor for a
court of law to determine the question as to whether the claim made by an applicant
deserves consideration. Delay and/or laches on the part of a Government servant may
deprive him of the benefit which had been given to others. Article 14 of the Constitution
of India would not, in a situation of that nature, be attracted as it is well known that law
leans in favour of those who are alert and vigilant. Opinion of the High Court that GOMs
No.126, dated 29.5.1998 gave a fresh lease of life having regard to the legitimate
expectation, in our opinion, is based on a wrong premise. Legitimate expectation is a part
of the principles of natural justice. No fresh right can be created by invoking the doctrine
of legitimate expectation. By reason thereof only the existing right is saved subject, of
course, to the provisions of the statute. {See State of Himachal Pradesh and Anr. v.
Kailash Chand Mahajan and Ors. [1992 Supp (2) SCC 351]}. 1992 AIR SCW 1247

12. We may notice that in Government of West Bengal v. Tarun K. Roy and Ors. [(2004)
1 SCC 347], this Court held :
"The respondents furthermore are not even entitled to any relief on the ground of gross
delay and laches on their part in filing the writ petition. The first two writ petitions were
filed in the year 1976 wherein the respondents herein approached the High Court in 1992.
In between 1976 and 1992 not only two writ petitions had been decided but one way or
the other, even the matter had been considered by this Court in Debdas Kumar (supra).
The plea of delay, which Mr. Krishnamani states, should be a ground for denying the
relief to the other persons similarly situated would operate against the respondents.
Furthermore, the other employees not being before this Court although they are
ventilating their grievances before appropriate courts of law no order should be passed
which would prejudice their cause. In such a situation, we are not prepared to make any
observation only for the purpose of grant of some relief to the respondents to which they
are not legally entitled to so as to deprive others therefrom who may be found to be
entitled thereto by a court of law." 1991 AIR SCW 704

See also Chairman, U.P. Jal Nigam and Anr. v. Jaswant Singh and Anr. [2006 (12)
SCALE 347] and New Delhi Municipal Council v. Pan Singh and Ors. [2007 (4) SCALE
204]. 2007 AIR SCW 672
2007 AIR SCW 1705

Only because a cut-off date has been fixed, the same per se cannot be said to be arbitrary
as some date is required to be fixed for that purpose.

Recently, this Court in K.S. Krishnaswamy etc. v. Union of India and Anr. [2006 (12)
SCALE 307] held : 2007 AIR SCW 77, (Para 21)
AIR 1983 SC 130

"Nakara's case (supra) was a case of revision of pensionary benefits and classification of
pensioners into two groups by drawing a cut-off line and granting the revised pensionary
benefits to employees retiring on or after the cut- off date. The criterion made applicable
was "being in service and retiring subsequent to the specified date". This Court held that
for being eligible for
@page-SC653
liberalised pension scheme, application of such a criterion is violative of Article 14 of the
Constitution, as it was both arbitrary and discriminatory in nature. It was further held that
the employees who retired prior to a specified date, and those who retired thereafter
formed one class of pensioners. The attempt to classify them into separate classes/groups
for the purpose of pensionary benefits was not founded on any intelligible differentia,
which had a rational nexus with the object sought to be achieved. The facts of Nakara's
case (supra) are not available in the facts of the present case. In other words, the facts in
Nakara's case are clearly distinguishable."
13. In Bannari Amman Sugars Ltd. v. Commercial Tax Officer and Ors. [(2005) 1 SCC
625], a Division Bench of this Court, as regards applicability of doctrine of promissory
estoppel, opined :
"In order to invoke the doctrine of promissory estoppel clear, sound and positive
foundation must be laid in the petition itself by the party invoking the doctrine and bald
expressions without any supporting material to the effect that the doctrine is attracted
because the party invoking the doctrine has altered its position relying on the assurance of
the Government would not be sufficient to press into aid the doctrine. The Courts are
bound to consider all aspects including the results sought to be achieved and the public
good at large, because while considering the applicability of the doctrine, the Courts have
to do equity and the fundamental principles of equity must for ever be present in the mind
of the Court.

20. In Shrijee Sales Coporation and Anr. v. Union of India (1997 (3) SCC 398) it was
observed that once public interest is accepted as the superior equity which can override
individual equity the principle would be applicable even in cases where a period has been
indicated for operation of the promise. If there is a supervening public equity, the
Government would be allowed to change its stand and has the power to withdraw from
representation made by it which induced persons to take certain steps which may have
gone adverse to the interest of such persons on account of such withdrawal. Moreover,
the Government is competent to rescind from the promise even if there is no manifest
public interest involved, provided no one is put in any adverse situation which cannot be
rectified. Similar view was expressed in Pawan Alloys and Casting Pvt. Ltd., Meerut etc.
etc. v. U.P. State Electricity Board and Ors. (AIR 1997 SC 3810 ) and in Sales Tax
Officer and Anr. v. Shree Durga Oil Mills and Anr. (1998 (1) SCC 573) and it was further
held that the Government could change its industrial policy if the situation so warranted
and merely because the resolution was announced for a particular period, it did not mean
that the Government could not amend and change the policy under any circumstances. If
the party claiming application of the doctrine acted on the basis of a notification it should
have known that such notification was liable to be amended or rescinded at any point of
time, if the Government felt that it was necessary to do so in public interest." 1997
AIR SCW 3839
1998 AIR SCW 186

{See also Southern Petrochemical Industries Co. Ltd. v. Electricity Inspector and E.T.I.O.
and Ors. [(2007) 5 SCC 447]. 2007 AIR SCW 3752

14. Interpretation of GOMs No.126 would, no doubt, depend upon the backdrop of the
events in which it was made but it is trite that the intention of the maker of the policy
must be drawn from the language used therein. For the said purpose, the entire document
should be read in its entirety. Original Application No.166 of 1990 was filed by two
serving employees. The State could in obedience to the Tribunal's order create two
supplementary posts and promote them thereto so as to treat them at par with their juniors
working in the Finance Department. The Notification envisages a personal pay by way of
stepping up of pay. It was given the prospective effect. No arrear of pay was to be paid.
The upgradation sanctioned was to lapse in the event of retirement of the individuals or
their promotion to the upgraded post. The said upgradation were to be subject to the
terms and conditions contained in clause 12 of the said order, a reading whereof would
clearly, in our opinion, lead to only one conclusion that it was meant to be applied to the
existing employees. By reason thereof, on upgradation, the seniors were required to
continue to perform the duties attached to the existing post till they get their normal
promotion to the next higher category. Upgradation of their posts was further dependant
on the fact as to whether they had been promoted in their normal course only. It was
meant to be a
@page-SC654
one time affair. In respect of some categories of employees, the question of upgradation
was deferred as specified in paragraph 12(6).
15

. It would, in our opinion, therefore, be incorrect to construe that the notification applied
to all who had been recruited to the Tamil Nadu Secretariat Service on or before
28.1.1994. Additional benefits have been accorded by reason of the said notification. A
person who fulfills the conditions, thus, would be entitled to the benefits provided for
therein. Those who had not fulfilled the same could not claim any benefit thereunder. For
the said purpose, the Court, in our view, should not give a strained or extended meaning
thereto. While construing such a notification, the financial impact thereof is also required
to be taken into consideration. {See State of A.P. and Anr. v. A.P. Pensioners' Association
and Ors. [(2005) 13 SCC 161] and Union of India and Anr. v. Manik Lal Banerjee
[(2006) 9 SCC 643]}. 2005 AIR SCW 6045
2006 AIR SCW 3889

16. Reliance placed by the learned counsel on R.L. Marwaha v. Union of India and Ors.
[(1987) 4 SCC 31] is misplaced. This Court in the said decision was considering validity
of a subordinate legislation whereby retrospective effect was granted. It was not a case
where pensionary benefit was granted to a class of employees. The benefit was meant to
be accorded to the existing employees only.
17

. Reliance has been placed by Mr. Venkataramani on the following passage of The State
of West Bengal v. Anwar Ali Sarkar [(1952) 3 SCR 284] : AIR 1952 SC 75

"The learned Attorney-General, appearing in support of these appeals, however, contends


that while a reasonable classification of the kind mentioned above may be a test of the
validity of a particular piece of legislation, it may not be the only test which will cover all
cases and that there may be other tests also. In answer to the query of the Court he
formulates an alternative test in the following words : If there is in fact inequality of
treatment and such inequality is not made with a special intention of prejudicing any
particular person or persons but is made in the general interest of administration, there is
no infringement of Article 14. It is at once obvious that, according to the test thus
formulated, the validity of State action, legislative or executive, is made entirely
dependent on the state of mind of the authority. This test will permit even flagrantly
discriminatory State action on the specious plea of good faith and of the subjective view
of the executive authority as to the existence of a supposed general interest of
administration. This test, if accepted, will amount to adding at the end of Article 14 the
words "except in good faith and in the general interest of administration." This is clearly
not permissible for the Court to do. Further, it is obvious that the addition of these words
will, in the language of Brewer, J., in Gulf, Colorado and Santa Fe Railway Co. v. W. H.
Ellis (165 US 150), make the protecting clause a mere rope of sand, in no manner
restraining State action. I am not, therefore, prepared to accept the proposition
propounded by the learned Attorney-General, unsupported as it is by any judicial
decision, as a sound test for determining the validity of State action."
This Court therein was dealing with the provisions of the West Bengal Special Courts
Act. The said decision, in our opinion, has no application with the facts and
circumstances of this case, particularly, when in the said decision itself, it has been
pointed out that Article 14 does not insist that every piece of legislation must have
universal application and it does not take away from the State the power to classify
person for the purpose of legislation.
18. As to what, therefore, is necessary for this purpose is that classification must be
rational and in order to pass the test : (1) the classification must be founded on an
intelligible differentia, and (2) the differentia must have a rational relation to the object
sought to be achieved by the Act.
19

. Equally misplaced is the decision of this Court in The State of Jammu and Kashmir v.
Shri Triloki Nath Khosa and Ors. [(1974) 1 SCC 19], wherein this Court, inter alia, held
that educational qualification can be held to be a criteria for valid classification for
different scales of pay. Justice V. R. Krishna Iyer, held : AIR 1974 SC 1

"The social meaning of Articles 14 to 16 is neither dull uniformity nor specious


'talentism'. It is a process of producing quality out of larger areas of equality extending
better facilities to the latent capabilities of the lowly. It is not a methodology of
substitution of pervasive and slovenly mediocrity for activist and intelligent - but not
@page-SC655
snobbish and uncommitted - cadres. However, if the State uses classification casuistically
for salvaging status and elitism, the point of no return is reached for Articles 14 to 16 and
the Court's jurisdiction awakens to deaden such manoeuvres. The soul of Article 16 is the
promotion of the common man's capabilities, over-powering environmental adversities
and opening up full opportunities to develop in official life without succumbing to the
sophistic argument of the elite that talent is the privilege of the few and they must rule,
wriggling out of the democratic imperative of Articles 14 and 16 by the theory of
classified equality which at its worst degenerates into class domination."
20. Reference has also been made by Mr. Venkataramani to a decision of this Court in
U.P. Raghavendra Acharya and Ors. v. State of Karnataka and Ors. [2006 (6) SCALE 23]
wherein it was held that pension is not a bounty and it is a deferred salary. This Court is
not concerned herein with such a situation. In the said decision, this Court was concerned
with a case where an employee retiring on a particular date was to receive 50% of the
pension on the enhanced salary. In the fact situation obtaining therein that as the revision
of pay and consequent revision in pension had come into force and by reason of a
notification, the modality of computing the pension was required to be determined, those
who had fulfilled the conditions laid down therein were held to be entitled to the benefits
provided for thereunder holding that the concerned employees had a vested right therein.
21. For the reasons aforementioned, we regret to express our inability to agree with the
view of the High Court. The impugned order of the High Court is, therefore, set aside.
The appeals are allowed. In the facts and circumstances of the case, however, there shall
be no orders as to costs.
Appeals allowed.
AIR 2008 SUPREME COURT 655 "Election Commission of India v. St. Mary's School"
(From : Delhi)
Coram : 2 S. B. SINHA AND H. S. BEDI, JJ.
Civil Appeal No. 5659 of 2007 (arising out of SLP (C) No. 21963 of 2004), D/- 6 -12
-2007.
Election Commission of India v. St. Mary's School and Ors.
(A) Constitution of India, Art.324 - Representation of the People Act (43 of 1950), S.29 -
Representation of the People Act (43 of 1951), S.159 - ELECTION - SUPREME COURT
- Holding of election - Utilising services of teachers in school - Supreme Court directed
that all teaching staff shall be put on duties of roll revisions and election works on
holidays and non-teaching days - Teachers should not ordinarily be put on duty on
teaching days and within teaching hours - Nonteaching staff, however, may be put on
such duties on any day or at any time, if permissible in law. (Para 32)
(B) Constitution of India, Art.324, Art.21A - Representation of the People Act (43 of
1950), S.29 - Representation of the People Act (43 of 1951), S.159 - ELECTION -
Holding of election - Utilising services of teaching staff in school during school timings -
Provisions of 1950 and 1951 Acts though enacted in terms of Art. 324 - Must be given
restricted meaning - Holding election is of paramount importance - But for this purpose
education of children cannot be neglected - Balance between two is to be maintained.
The provisions of the 1950 and 1951 Acts although were enacted in terms of Article 324
of the Constitution of India, the same must be given restricted meaning. Holding of an
election is no doubt of paramount importance. But for the said purpose the education of
the children cannot be neglected. Therefore, it is necessary to maintain the balance
between the two. With an advent of technology requisitioning of a large number of people
for carrying out the election may not be necessary. The Election Commission has
different roles to play. Preparation of an electoral rolls, revision of electoral rolls, when
objections are filed, hearing the parties and determining the objections, enumeration of
the voter list and to hold elections as and when due. The Election Commission and its
officers can formulate an effective scheme to see that the services of a large number of
teachers are not required. The State admittedly is not in a position to perform its
sovereign function of imparting education. Such functions necessarily are required to be
performed by the private sectors. Those students who are in a position to get admission in
the public schools presumably would also be in a position to appoint tutors whereas those
students who are admitted to the Government schools

@page-SC656
ordinarily would be from the middle or lower middle class or poor families. The state of
primary education in India is in deplorable condition. There admittedly is a heavy drop
outs from the schools particularly from amongst the girl schools. If right to exercise
franchise is an important one, right to education is also no less important being a
fundamental right. (Paras 27, 28)
(C) Constitution of India, Art.133 - APPEAL - Appeal - Question in regard to application
of constitutional right and in particular fundamental right - Cannot be thwarted only by
reason of a concession made by a counsel. (Para 31)
Cases Referred : Chronological Paras
1995 AIR SCW 1095 : AIR 1995 SC 1078 (Ref.) 20
1993 AIR SCW 863 : AIR 1993 SC 2178 (Ref.) 23, 24
1992 AIR SCW 2100 : AIR 1992 SC 1858 (Ref.) 22, 23
AIR 1984 SC 802 23
(1954) 98 Law Ed 873 : 347 US 483 26
K. K. Venugopal, Sr. Advocate, Ms. Meenakshi Arora, S. K. Mendiratta, S. Gupta and
Ankur Talwar, with him, for Appellant; R. K. Shukla, Sr. Advocate, Ms. Reena George,
Prashant Bhushan, D. S. Mahra, Ms. Rashmi Malhotra, Ms. Indira Sawhney, Mrs. Anil
Katiyar, Sanjiv Sen and Praveen Swarup, with him, for Respondents.
Judgement
1. S. B. SINHA, J. :-Leave granted.
2. A short but interesting question, as to how conflict in two constitutional rights should
be balanced, is involved in this appeal which arises out of a judgment and order dated
11.08.2004 passed by a Division Bench of the Delhi High Court in Writ Petition (Civil)
No. 1076 of 2003.
3. Respondent No.1 is an unaided school. It is governed by the provisions of the Delhi
School Education Act, 1973 (for short, 'the Act') and the rules framed thereunder. It filed
a writ petition in public interest, questioning the action of the appellant and the
respondent Nos. 2 to 5 herein as regards utilizing the services of the teachers of the
Government schools for various purposes during school timings, as a result whereof the
students reading in the said schools are deprived of obtaining instructions from their
teachers during such period. In the writ petition it was pointed out that the absence of
teachers occur due to their deployment for non-educational purposes; and as the teaching
and administrative staff of these schools have been used by the State agencies as well as
the appellant herein for various other duties outside school during school hours including:
lPolling duties to general election to Lok Sabha
lPolling duties to general election to Delhi Legislative Assembly
l Polling duties to MCD elections
lGurudwara election
lRevision of polling lists
lPulse polio drive
lPreparation of census lists
lSurveys on malaria, pollution etc.
4. The Act and the Rules framed thereunder which govern the field mandate that all the
schools in Delhi have to function for a minimum of 210 days in a year. It was pointed out
that although the extent of the period differed, the teachers were asked to perform polling
duties for a few months and also for census duties for considerable period. The writ
petition highlighted that absence of teachers from the school for a long time resulted in
unfinished courses, high drop out rates, poor results and inability to compete in open
examinations, such as medicine, engineering etc. and/or to get admission in other
prestigious or professional colleges. It was contended that an informal survey conducted
by the petitioner therein demonstrated that about 9,00,000 students had enrolled
themselves in class 1 of the schools run by the Municipal Corporation of Delhi, but only
50,000 of them appeared in the secondary examination.
5. The Municipal Corporation of Delhi (MCD) in its counter-affidavit contended that
absence of the teachers and other administrative staff of schools for performing duties
allocated by the officers of the Election Commission is in national interest. In the
counter-affidavit, it was, stated:
"3. That the main work assigned to teachers is in relation to teaching work. However, in
the larger national interest, some of the teachers are called upon to do some other
Government work relating to public interests like polio vaccination, preparation of voter
list, etc.
4. That only during the work of census in the year 2001, a slightly large number
@page-SC657
of teachers were required for the purpose. Even for this purpose, to take care that the
teaching activities are not hampered in any manner, letter No. F-4/12/2000, dated
04.02.01, referring to the Order of Lt. Governor of the NCT of Delhi having been passed,
whereby the enumerators were directed to perform their census enumeration duties
before/after their normal school hours. The census work is conducted only once in a
decade.
5. That it is reiterated that it is seen that the government work relating to public interest
done by the teachers does not hamper the teaching activities. The additional government
public work is some time assigned to some of the teachers in national interest."
6. The New Delhi Municipal Committee (NDMC) in its additional affidavit filed before
the High Court, stated :
"2. That the total number of sanctioned posts of teachers in NDMC Schools are about
1200. For polling duty, almost 90% of the teaching staff is deployed on duty. For census
work in the year 2000 and enumeration work in the year 2001, 90% of the staff were
assigned duty. For Revision of electoral rolls, almost 50% of the teaching staff is put on
duty.
3. For polling duties, the Election Commission deploys the teachers of the NDMC
Schools for the purposes of holding Parliamentary Elections, Delhi Assembly Elections
and even Municipal Corporation Elections. Generally the elections are held on Sundays,
but before the actual polling takes places, the teaching staff is called for three working
days for the purposes of training, collection of election material etc. In the year 1998-99,
all the elections were held i.e. Parliamentary, Delhi Assembly and Corporation Elections
and hence the teachers were not on duty for 9 working days.
4. That in the year 2001, when the work of Special Revision of Electoral Roll was
required to be conducted, the then Chief Electoral Officer informed the Respondent
NDMC that the said work shall be conducted by the teachers between 13.9.2001 to
12.10.2001 by the teachers. Initially it was supposed to be a part time job, however, it
was later on converted into a full time work so as to complete the assignment within the
prescribed time."
7. During the pendency of the writ petition before the High Court, the Government of
NCT of Delhi issued a circular letter, relevant portion whereof is as under - :
"This duty binds the State-Governments to provide the requisite number of staff to the
Election Commission for conducting elections and taking into consideration the
insufficient number of staff available in generalist cadre, the government proposes to
utilize the services of teaching staff for the following duties :
(a) Election Duty
(b) Revision of Electoral Rolls
(b) Revision of Electoral Rolls : For the intensive revision of electoral rolls, we require
approximately 55,000 employees whereas the Cadre strength of the General cadre of the
Govt. of NCT of Delhi is only around 9000. In view of this, the services of teachers are
indispensable for intensive Revision of Election Rolls. This intensive revision of electoral
rolls is not a regular process and this takes place with a periodicity of 4 to 5 years as per
instructions of the Election Commission of India.
The Summary/Special Revision of Electoral Rolls takes place once in a year and the
services of teachers are normally not utilized and this is managed within the cadre
strength. However, in some of the institutions that are declared as designated locations,
the services of the Head of the Institution are taken to receive the Form and they are
declared as Designated Officers. Normally, these officials are not engaged in teaching and
handle the work in addition to their administrative duties. They are assisted by non-
teaching staff.
The Staff thus deployed function under the superintendence and control of the Chief
Electoral Officer and remain under the administrative control of Chief Electoral Officer
during this period.
Elections being a sovereign function of the State, the work of conduct of elections cannot
be delegated to persons who are not employees of government or to any non-
governmental agency, parastatal organizations etc.
The Government draws staff from every Govt. department and it is not true that only
teachers are deployed for election work. Staff from other categories from almost every
department is engaged for election duty.
However, with a view to ensure that
@page-SC658
election work does not come in conflict with the interest of education, the respondent
Government would like to take the following steps to ensure that teaching work is least
affected :
(i) As far as possible the revision of Electoral roll shall be taken up during holidays or
teachers would be deployed to perform the work on holidays.
(ii) The teachers who are on non-teaching posts i.e. Physical Education Teachers,
Drawing Teachers, Librarian and Lab Assistants, Yoga Instructors etc. would be deployed
for election work.
(iii) The teaching hour loss, if any, shall be compensated by holding extra classes, so that
the minimum prescribed teaching hours are completed.
(iv) The Principals/Heads of Institutions will be directed to make internal adjustment of
time tables and reschedule the classes for making up any possible teaching losses."
8. Learned counsel appearing for the parties, however, as it appears from the impugned
judgment, accepted before the High Court that the services of the teachers should be
utilized for non-teaching purposes only on a day which is not a working day for the
students.
9. The Election Commission is, thus, before us.
10. Mr. K.K. Venugopal, learned Senior Counsel appearing on behalf of the appellant,
would, inter alia, submit :
(i) Holding of an elections is a sovereign function.
(ii) The Election Commission having regard to the provisions contained in the
Constitution of India as also the Representation of the People Act, 1951 is required to
conduct elections for the purpose of upholding democracy.
(iii) Democracy being a basic feature of the Constitution of India, it is obligatory on the
part of the Election Commission to ensure that the citizens who are entitled to vote are
not deprived of their right and those who are not entitled thereto are not permitted to do
so.
(iv) In terms of the provisions of clauses (1) and (6) of Article 324 of the Constitution of
India, it is mandated that whenever the Election Commission asks for deployment of staff
for the purpose of conducting elections, it is obligatory on the part of the President of
India or the Governor of the State to make such number of staff made available to it, and
with a view to fulfill the said constitutional object, the Parliament amended Section 159
of the 1951 Act so as to provide :
"159. Staff of certain authorities to be made available for election work.-
(1) The authorities specified in sub-section (2) shall, when so requested by a Regional
Commissioner appointed under clause (4) of Article 324 or the Chief Electoral Officer of
the State, make available to any Returning Officer such staff as may be necessary for the
performance of any duties in connection with an election.
(2) The following shall be the authorities for the purposes of sub-section (1), namely:-
(i) every local authority;
(ii) every University established or incorporated by or under a Central, Provincial or State
Act;
(iii) a Government company as defined in section 617 of the Companies Act, 1956 (1 of
1956);
(iv) any other institution, concern or undertaking which is established by or under a
Central, Provincial or State Act or which is controlled, or financed wholly or substantially
by funds provided, directly or indirectly, by the Central Government or a State
Government."
(v) The High Court, in that view of the matter could not have issued any direction which
may for all intent and purport interfere with the electoral process.
11. Learned counsel appearing on behalf of the Municipal Corporation of Delhi, New
Delhi Municipal Committee and Union of India adopted the submissions of Mr.
Venugopal.
12. Ms. Reena George, learned counsel appearing on behalf of the writ petitioner-
respondent, on the other hand, submit that - :
(a) the impugned order having been passed in terms of consent of the parties, this Court
should not exercise its jurisdiction under Article 136 of the Constitution of India.
(b) Right to education being a fundamental right having regard to Article 21A of the
Constitution of India, it is obligatory on the part of the State to ensure that the students
are not deprived thereof.
@page-SC659
(c) In the affidavit filed on behalf of the NDMC, it was clearly demonstrated that in some
schools where teaching or instructions are imparted for Class IX or X students, no teacher
was available for a period of two months.
(d) The purpose for which the education is imparted in the schools is to see that the
Government in the municipal schools must ensure that they compete with the standard
maintained by the private schools.
13. Indisputably, for upholding the democracy and the democratic values, holding of
elections is imperative. There cannot also be any doubt or dispute that keeping in view
the constitutional mandate provided for under clauses (1) and (6) of Article 324 of the
Constitution of India, the President of India or the Governor of a State i.e. the Central
Government as also the State Government have a duty to make available to the Election
Commission, or to a Regional Commissioner such staff, as may be necessary for the
discharge of functions conferred on the Election Commission by clause (1) in terms
whereof a power of superintendence, direction and control of elections is to be vested in
the Election Commission, if request in this regard is made. Article 327 of the Constitution
of India empowers the Parliaments to make laws with respect to all matters relating to, or
in connection with, elections to either House of the Parliament or to the House or either
House of the Legislature of a State including the preparation of the electoral rolls, the
delimitation of constituencies and all other matters necessary for securing the due
constitution of such House or Houses.
14. The Parliament with a view to give effect to the said constitutional functions enacted
the Representation of the People Act, 1950 (1950 Act) and the Representation of the
People Act, 1951 (1951 Act).
15. We may notice certain provisions of the said Acts.
1950 Act :
Section 13-A of 1950 Act provides for the designation or nomination of the Chief
Electoral Officers, by the Election Commissioner; whereas Section 13-AA provides for
designation or nomination of a District Election Officer, who is to be an officer of
Government.
Sections 13B, 13CC and Section 29 of the 1950 Act read as under :
"13B. - Electoral registration officers.- (1) The electoral roll for each parliamentary
constituency in the State of Jammu and Kashmir or in a Union territory not having a
Legislative Assembly, each assembly constituency and each Council constituency shall
be prepared and revised by an electoral registration officer who shall be such officer of
Government or of a local authority as the Election Commission may, in consultation with
the Government of the State in which the constituency is situated, designate or nominate
in this behalf.
(2) An electoral registration officer may, subject to any prescribed restrictions, employ
such persons as he thinks fit for the preparation and revision of the electoral roll for the
constituency."
"13CC. Chief Electoral Officers, District Election Officers, etc., deemed to be on
deputation to Election Commission.- The officers referred to in this Part and any other
officer or staff employed in connection with the preparation, revision and correction of
the electoral rolls for, and the conduct of, all elections shall be deemed to be on
deputation to the Election Commission for the period during which they are so employed
and such officers and staff shall, during that period, be subject to the control,
superintendence and discipline of the Election Commission."
"29. Staff of local authorities to be made available.- Every local authority in a State shall,
when so requested by the Chief Electoral Officer of the State, make available to any
Electoral Registration Officer such staff as may be necessary for the performance of any
duties in connection with the preparation and revision of electoral rolls."
1951 Act :
Sections 2(1)(bb) and 2(1)(cc) of 1951 Act provide for the meanings of the terms "Chief
Electoral Officer" and the "District Election Officer", who would be an officer appointed
under Sections 13-A and 13-AA of the 1950 Act.
Part IV of 1951 Act, does not lay down any procedure for requisitioning of a person for
being appointed as the Returning Officer, an Assistant Returning Officer; or Presiding
Officer, or Polling Officer.
However, it may be noticed that after Section 22, which provides for appointment of
Assistant Returning Officers a proviso was added by reason of Act No. 47 of 1966 in
@page-SC660
terms whereof the words "an Officer of Government or of a local authority" had been
inserted.
Sections 26, 28A, 151 and Section 159 read as under :
"26 Appointment of presiding officers for polling stations.- (1) The district election
officer shall appoint a presiding officer for each polling station and such polling officer or
officers as he thinks necessary, but he shall not appoint any person who has been
employed by or on behalf of, or has been otherwise working for, a candidate in or about
the election:
Provided that if a polling officer is absent from the polling station, the presiding officer
may appoint any person who is present at the polling station other than a person who has
been employed by or on behalf of, or has been otherwise working for, a candidate in or
about the election, to be the polling officer during the absence of the former officer, and
inform the district election officer accordingly:
Provided further that nothing in this sub-section shall prevent the district election officer
from appointing the same person to be the presiding officer for more than one polling
station in the same premises.
(2) A polling officer shall, if so directed by the presiding officer, perform all or any of the
functions of a presiding officer under this Act or any rules or orders made thereunder,
(3) If the presiding officer, owing to illness or other unavoidable cause, is obliged to
absent himself from the polling station, his functions shall be performed by such polling
officer as has been previously authorised by the district election officer to perform such
functions during any such absence.
(4) References in this Act to the presiding officer shall, unless the context otherwise
requires, be deemed to include any person performing any function which he is
authorised to perform under sub-section (2) or sub-section (3), as the case may be."
"28A. Returning officer, presiding officer, etc., deemed to be on deputation to Election
Commission.- The returning officer, assistant returning officer, presiding officer, polling
officer and any other officer appointed under this Part, and any police officer designated
for the time being by the State Government, for the conduct of any election shall be
deemed to be on deputation to the Election Commission for the period commencing on
and from the date of the notification calling for such election and ending with the date of
declaration of the results of such election and accordingly, such officers shall, during that
period, be subject to the control, superintendence and discipline of the Election
Commission."
"151. Casual vacancies in the State Legislative Councils.- When before the expiration of
the term of office of a member elected to the Legislative Council of a State, his seat
becomes vacant or is declared vacant or his election to the Legislative Council is declared
void, the Election Commission shall, by a notification in the Official Gazette, call upon
the Council constituency concerned or the members of the Legislative Assembly of the
State, as the case may be, to elect a person for the purpose of filling the vacancy so
caused, before such date as may be specified in the notification, and the provisions of this
Act and of the rules and orders made thereunder shall apply, as far as may be, in relation
to the election of a member to fill such vacancy."
"159. Staff of certain authorities to be made available for election work.- (1) The
authorities specified in sub-section (2) shall, when so requested by a Regional
Commissioner appointed under clause (4) of Article 324 or the Chief Electoral Officer of
the State, make available to any returning officer such staff as may be necessary for the
performance of any duties in connection with an election.
(2) The following shall be the authorities for the purposes of sub-section (1), namely:-
(i) every local authority;
(ii) every University established or incorporated by or under a Central, Provincial or State
Act;
(iii) a Government company as defined in section 617 of the Companies Act, 1956 (1 of
1956);
(iv) any other institution, concern or undertaking which is established by or under a
Central, Provincial or State Act or which is controlled, or financed wholly or substantially
by funds provided, directly or indirectly, by the Central Government or a State
Government."
16-19. We may, however, notice that prior to enactment of Act No. 12 of 1998 in
@page-SC661
terms of Section 159 of the 1951 Act the obligation to make available to any Returning
Officer such staff as may be necessary in connection with an election was only confined
to the local authority.
20

. The question as to whether the staff of the State Bank of India could be requisitioned for
the purpose came up for consideration before this Court in Election Commission of India
v. State Bank of India Staff Association, Local Head Office Unit, Patna and Others
[(1995) Supp 2 SCC 13], wherein this Court upheld the judgment and order of the
Division Bench of the Patna High Court, opining that the officers of the State Bank of
India cannot be requisitioned in terms of Section 26 of the 1951 Act or otherwise. 1995
AIR SCW 1095

21. The constitutional and statutory scheme would lead to a realistic conclusion when
emphasis was laid that it is for the Central Government and the State Governments alone
to provide for the requisite staff. How would they do it is one thing. It may be by fresh
recruitment for the purposes for which the staff are requisitioned or for deployment or by
way of deputation. Indisputably, there are certain functions which may be performed only
by the Government staff. For the said purposes they may be sent on deputation e.g.
Sections 21 and 22 of the 1951 Act provides for the Returning Officers and Assistant
Returning Officers who must be an officer of Government or of a local authority.
Therefore, their services can be requisitioned under clause (6) of Article 324 of the
Constitution of India as also Section 159 of the 1951 Act. The Election Commission or
the Regional Commissioner, as the case may be, is also entitled to request for
requisitioning the services of the persons in the employment of the Government or the
local authority and others who may not be officers of the Government or the local
authority. The services of other employees who are not officers may also be requisitioned.
The Parliament was aware that in an election, requisition of services of the employees of
the Central Government or the State Governments may prove to be insufficient and, thus,
a direction for appointment of the staff from amongst the officers of the local authority
and others have been made.
22

. On the other hand, however, right to education is held to be a fundamental right. It was
so stated in Mohini Jain v. State of Karnataka [(1992) 3 SCC 666] in the following
terms : 1992 AIR SCW 2100

"12. "Right to life" is the compendious expression for all those rights which the courts
must enforce because they are basic to the dignified enjoyment of life. It extends to the
full range of conduct which the individual is free to pursue. The right to education flows
directly from right to life. The right to life under Article 21 and the dignity of an
individual cannot be assured unless it is accompanied by the right to education. The State
Government is under an obligation to make endeavour to provide educational facilities at
all levels to its citizens."
23

. The aforementioned ratio has been affirmed with certain modification by this Court in
Unni Krishnan, J.P. and others v. State of Andhra Pradesh and others [(1993) 1 SCC 645],
expressly stating : 1993 AIR SCW 863, Para 142

"......Having regard to the fundamental significance of education to the life of an


individual and the nation, and adopting the reasoning and logic adopted in the earlier
decisions of this Court referred to hereinbefore, we hold, agreeing with the statement in
Bandhua Mukti Morcha that right to education is implicit in and flows from the right to
life guaranteed by Article 21. That the right to education has been treated as one of
transcendental importance in the life of an individual has been recognised not only in this
country since thousands of years, but all over the world. In Mohini Jain the importance of
education has been duly and rightly stressed. The relevant observations have already been
set out in para 7 hereinbefore. In particular, we agree with the observation that without
education being provided to the citizens of this country, the objectives set forth in the
Preamble to the Constitution cannot be achieved. The Constitution would fail........"
AIR 1984 SC 802
1992 AIR SCW 2100

24

. Article 45 is the only provision in our Constitution which fixes a time limit during
which the State is to provide for free and compulsory education for children until they
complete the age of 14 years. The Constitution has been amended keeping in view the
aforementioned provisions as also the decision of this Court in Unni Krishnan (supra) by
inserting Article 21A of the Constitution of India, which reads as under : 1993 AIR
SCW 863

"The right to education which flows from


@page-SC662
Article 21 is not an absolute right. It must be construed in the light of directive principles.
A true democracy is one where education is universal, where people understand what is
good for them and the nation and the right to education have to be determined. Right to
education, understood in the context of Articles 45 and 41, means that every child/citizen
of this country has a right to free education until he completes the age of fourteen years,
and (b) after child/citizen completes 14 years, his right to education is circumscribed by
the limits of the economic capacity of the State and its development. It is significant that
among the several Articles in Part IV, only Article 45 speaks of a time limit; no other
Article does. It is not a mere pious wish and the State cannot flout the said direction even
after 44 years on the ground that the Article merely calls upon it to "endeavour to
provide" the same and on the further ground that the said Article is not enforceable by
virtue of the declaration in Article 37. The passage of 44 years more than four time the
period stipulated in Article 45 has converted the obligation created by the Article into an
enforceable right. At least now the State should honour the command of Article 45. It
must be made a reality."
25. Sixty years of independence, however, has not brought about the desired result of
imparting compulsory education to all the children. Education is one of the most
important functions of the State. The State has a basic responsibility in regard thereto.
26. In Brown v. Board of Education [(98 LEd 873 : 347 US 483 (1954)], Earl Warren, CJ,
speaking for the US Supreme Court emphasized the right to education in the following
terms :
"Today, education is the most important function of the State and local
Governments.......... It is required in the performance of our most basic responsibility,
even services in the armed forces. It is the very foundation of good citizenship. Today it
is the principal instrument in awakening the child to cultural values, in preparing him for
later professional training, and in helping him to adjust normally to his environment. In
these days it is doubtful any child may reasonably be expected to succeed in life if he is
denied the opportunity of an education."
27. The provisions of the 1950 and 1951 Acts although were enacted in terms of Article
324 of the Constitution of India, the same must be given restricted meaning. Holding of
an election is no doubt of paramount importance. But for the said purpose the education
of the children cannot be neglected. Therefore, it is necessary to maintain the balance
between the two.
28. With an advent of technology requisitioning of a large number of people for carrying
out the election may not be necessary. We may notice that the Election Commission has
different roles to play. Preparation of an electoral rolls, revision of electoral rolls, when
objections are filed, hearing the parties and determining the objections, enumeration of
the voter list and to hold elections as and when due. The Election Commission and its
officers, in our opinion, can formulate an effective scheme to see that the services of a
large number of teachers are not required. The State admittedly is not in a position to
perform its sovereign function of imparting education. Such functions necessarily are
required to be performed by the private actors. Those students who are in a position to get
admission in the public schools presumably would also be in a position to appoint tutors
whereas those students who are admitted to the Government schools ordinarily would be
from the middle or lower middle class or poor families. The state of primary education in
India is in deplorable condition. There admittedly is a heavy drop outs from the schools
particularly from amongst the girl schools. The question if right to exercise franchise
whereupon the emphasis is laid by Mr. Venugopal is an important one, right to education
is also no less important being a fundamental right.
29. The Human Rights Conventions have imposed a duty on the Contracting States to set
up institutions of higher education which would lead to the conclusion that the citizens
thereof should be afforded and an effective right of access to them. In a democratic
society, a right to education is indispensable in the interpretation of right to development
as a human right. [See Leyla Sahin v. Turkey, decided by the European Court of Human
Rights on 10th November, 2005]. Thus, right to development is also considered to be a
basic human right.
30. It is probably with that end in view the counsel appearing for the Election
Commission had also joined the other counsel appearing for the respondents, to suggest
@page-SC663
the court that the services of the teachers may not be requisitioned on the days on which
the schools are open. Submission of Mr. Venugopal that such a contention had not been
made by the learned counsel appearing on behalf of the Election Commission cannot be
accepted.
31. We have, however, considered the matter at some details as the question in regard to
the application of the constitutional right and in particular fundamental right cannot be
thwarted only by reason of a concession made by a counsel.
32. We would, however, notice that the Election Commission before us also categorically
stated that as far as possible teachers would be put on electoral roll revision works on
holidays, non-teaching days and non-teaching hours; whereas non-teaching staff be put
on duty any time. We, therefore, direct that all teaching staff shall be put on the duties of
roll revisions and election works on holidays and non-teaching days. Teachers should not
ordinarily be put on duty on teaching days and within teaching hours. Non-teaching staff,
however, may be put on such duties on any day or at any time, if permissible in law.
33. Subject to the aforementioned modifications, this appeal is dismissed. However, in
the facts and circumstances of this case, there shall be no order as to costs.
Appeal dismissed.
AIR 2008 SUPREME COURT 663 "Anuj Garg v. Hotel Association of India"
(From : 2006 (86) DRJ 668 (Delhi))
Coram : 2 S. B. SINHA AND H. S. BEDI, JJ.
Civil Appeal Nos. 5657 with 5658 of 2007(arising out of SLP (C) Nos. 12781 with 16127
of 2006), D/- 6 -12 -2007.
Anuj Garg and Ors. v. Hotel Association of India and Ors.
(A) Constitution of India, Art.245, Art.13 - LEGISLATURE - LAW -
CONSTITUTIONALITY OF AN ACT - Law - Constitutional validity - To be decided as
per societal conditions prevalent at relevant time - Changed social psyche and
expectations are important factors to be considered in upkeep of law. (Paras 7, 8)
(B) Constitution of India, Art.14 - EQUALITY - Gender equality - Establishment -
Relevance of international treaties. (Para 9)
(C) Punjab Excise Act (1 of 1914), S.30 - Constitution of India, Art.14, Art.16, Art.21 -
STATE EXCISE - EQUALITY IN PUBLIC EMPLOYMENT - RIGHT TO LIFE -
Prohibition on employment of women in Hotels and Bars serving liquor - Violates gender
equality - Personal freedom is a fundamental tenet - Cannot be compromised on ground
of security of women - Parens patriae power of State - Not beyond judicial scrutiny.
Section 30 of Punjab Act prohibiting employment of women in any part of premises in
which liquor or intoxicating drug is consumed by the public results in an invidious
discrimination. (Para 53)
Right to Self-Determination is an important offshoot of Gender Justice discourse. At the
same time, security and protection to carry out such choice or option specifically, and
state of violence-free being generally is another tenet of the same movement. In fact, the
latter is apparently a more basic value in comparison to right to options in the feminist
matrix. Privacy rights prescribe autonomy to choose profession whereas security
concerns texture methodology of delivery of this assurance. But it is a reasonable
proposition that the measures to safeguard such a guarantee of autonomy should not be so
strong that the essence of the guarantee is lost. State protection must not translate into
censorship. It is to be borne in mind that legislations with pronounced 'protective
discrimination' aims, such as this one, potentially serve as double edged swords. Strict
scrutiny test should be employed while assessing the implications of this variety of
legislations. Legislation should not be only assessed on its proposed aims but rather on
the implications and the effects. Section - 30 of Punjab Act suffers from incurable
fixations of stereotype morality and conception of sexual role. The perspective thus
arrived at is outmoded in content and stifling in means. (Paras 33, 34, 44, 45)
Instead of prohibiting women employment in the bars altogether the State should focus
on factoring in ways through which unequal consequences of sex differences can be
eliminated. It is State's duty to ensure circumstances of safety which inspire confidence in
women to discharge the duty freely in accordance to the requirements of the profession
they choose to follow. Any other policy inference (such as the one embodied under S. 30)
from societal conditions would
@page-SC664
be oppressive on the women and against the privacy rights. (Para 41)
(D) Constitution of India, Art.14, Art.226, Art.32 - EQUALITY - WRITS - Gender
equality - Protective discrimination statute - Standard of judicial scrutiny.
The test to review a Protective Discrimination Statute would entail a two pronged
scrutiny : (a) The legislative interference should be justified in principle, (b) the same
should be proportionate in measure. The Court's task is to determine whether the
measures furthered by the State in form of legislative mandate, to augment the legitimate
aim of protecting the interests of women are proportionate to the other bulk of well
settled gender norms such as autonomy, equality of opportunity, right to privacy et al. The
bottom-line in this behalf would be a functioning modern democratic society which
ensures freedom to pursue varied opportunities and options without discriminating on the
basis of sex, race, caste or any other like basis. In fine, there should be a reasonable
relationship of proportionality between the means used and the aim pursued. (Para
49)
(E) Punjab Excise act (1 of 1914), S.30 - STATE EXCISE - RIGHT TO LIFE -
EQUALITY IN PUBLIC EMPLOYMENT - PROHIBITION - Employment in Hotels
and Bars serving liquor - Prohibition on employment of men below 25 years of age -
Violates right to livelihood.
Constitution of India, Art.21, Art.16. (Para 54)
(F) Punjab Excise Act (1 of 1914), S.30 - STATE EXCISE - Hotels and Bars serving
liquor - Prohibition on employment of women - Amounts to invidious discrimination on
ground of sex - Mere possibility of untoward incidents - Cannot be ground to make it
intra vires. (Para 51)
(G) Constitution of India, Art.133 - APPEAL - PLEA - HIGH COURT - JUDGMENT -
New plea - Plea not raised in High Court - Judgment allowed to attain finality - Party
estopped from challenging correctness of judgment in appeal filed by third party. (Para
52)

(H) Constitution of India, Pre., Art.16, Art.21, Art.14, Art.19(1)(g) - STATE EXCISE -
RIGHT TO LIFE - FREEDOM OF TRADE - DOCTRINES - Trade in liquor - Doctrine
of res extra commercium - Cannot be extended to prohibit employment of women in
Hotels and Bars serving liquor.
Punjab Excise Act (1 of 1914), S.30. (Para 25)
Cases Referred : Chronological Paras
2006 AIR SCW 1869 : AIR 2006 SC 3480 (Rel. on, Pnt H) 26
(2004) 9 SCC 512 (Ref., Pnt B) 16
(2004)18 BHRC 52 21
2003 AIR SCW 3536 : AIR 2003 SC 2902 (Rel. on, Pnt-A) 7
2000 AIR SCW 969 : AIR 2000 SC 1274 : 2000 Lab IC 1033 (Ref., Pnt-B) 12
1999 AIR SCW 811 : AIR 1999 SC 1149 (Ref., Pnt B) 10
(1998) ECHR 21 38
1997 AIR SCW 3043 : AIR 1997 SC 3011 : 1997 Lab IC 2890 (Ref., Pnt B) 14
(1997) ECHR 6 38
1996 AIR SCW 2178 : AIR 1996 SC 1864 (Ref., Pnt B) 13
(1996) 58 US 515 50
(1993) ECHR 29 38
(1985) 473 US 432 30
(1985) ECHR 7 38
AIR 1982 SC 879 : 1982 Lab IC 806 (Ref .Pnt B) 15
AIR 1981 SC 1829 : 1981 Lab IC 1313 (Ref. Pnt B) 11
(1973) 411 US 677 : 36 L ED 2d 583 42
433 US 321 43
Rajiv Dutta, Sr. Advocate, M. P. Shorawala, Ms. Jyoti Saxena, Vipin K. Saxena, Ms.
Shashi Kiran, for Appellants; Arun Jaitley, Nagender Rai, Sr. Advocates, Ravi Sikri, Mrs.
Saket Sikri, Ms. Madhu Sikri, Vikas Sharma, Ms. Sweta Garg and D. S. Mahra, for
Respondents.
Judgement
1. S. B. SINHA, J. :-Leave granted.
Introduction
2. Constitutional validity of Section 30 of the Punjab Excise Act, 1914 (for short "the
Act") prohibiting employment of "any man under the age of 25 years" or "any woman" in
any part of such premises in which liquor or intoxicating drug is consumed by the public
is the question involved in this appeal which arises out of a judgment and order dated
12.01.2006 passed by the High Court of Delhi in CWP No. 4692 of 1999.
Background Facts
3. First Respondent is the Hotel Association of India. Its members carry on business in
hotels. Liquor is served in the hotels not only in the bar but also in the restaurant. Liquor
is also served in rooms as
@page-SC665
part of room service. First Respondent with four others filed a writ petition before the
Delhi High Court questioning the validity of the said provision. By reason of the
impugned judgment, Section 30 of the Act has been declared to be ultra vires Articles
19(1)(g), 14 and 15 of the Constitution of India to the extent it prohibits employment of
any woman in any part of such premises, in which liquor or intoxicating drugs are
consumed by the public.
4. National Capital Territory of Delhi appears to have accepted the said judgment. But as
a respondent, it seeks to support the impugned statutory provision, although no Special
Leave Petition has been filed by it. Appellants herein, who are a few citizens of Delhi, are
before us.
A special leave petition has been filed by the First Respondent questioning that part of the
order whereby restrictions had been put on employment of any man below the age of 25
years.
Submissions
5. Mr. Rajiv Dutta, learned senior counsel appearing on behalf of the appellants, in
support of this appeal, submitted that as nobody has any fundamental right to deal in
liquor, being 'res extra commercium', the State had the right to make a law and/or
continue the old law imposing reasonable restrictions on the nature of employment
therein.
6. Mr. Arun Jaitley, learned senior counsel appearing on behalf of the respondents, on the
other hand, supported the impugned judgment.
Constitutional Backdrop
7. The Act is a pre-constitutional legislation. Although it is saved in terms of Article 372
of the Constitution, challenge to its validity on the touchstone of Articles 14, 15 and 19 of
the Constitution of India, is permissible in law. While embarking on the questions raised,
it may be pertinent to know that a statute although could have been held to be a valid
piece of legislation keeping in view the societal condition of those times, but with the
changes occurring therein both in the domestic as also international arena, such a law can
also be declared invalid.

In John Vallamattom and Anr. v. Union of India (2003) 6 SCC 611], this Court, while
referring to an amendment made in UK in relation to a provision which was in pari
materia with Section 118 of Indian Succession Act, observed : 2003 AIR SCW 3536,
Para 28

"The constitutionality of a provision, it is trite, will have to be judged keeping in view the
interpretative changes of the statute affected by passage of time."
Referring to the changing legal scenario and having regard to the Declaration on the
Right to Development adopted by the World Conference on Human Rights as also Article
18 of the United Nations Covenant on Civil and Political Rights, 1966, it was held :
"33. It is trite that having regard to Article 13(1) of the Constitution, the constitutionality
of the impugned legislation is required to be considered on the basis of laws existing on
26-1-1950, but while doing so the court is not precluded from taking into consideration
the subsequent events which have taken place thereafter. It is further trite that the law
although may be constitutional when enacted but with passage of time the same may be
held to be unconstitutional in view of the changed situation."

8. Changed social psyche and expectations are important factors to be considered in the
upkeep of law. Decision on relevance will be more often a function of time we are
operating in. Primacy to such transformation in constitutional rights analysis would not
be out of place. It will be in fitness of the discussion to refer to the following text from
"Habits of the Heart: Individualism and Commitment in American Life" by R. Bellah, R.
Madsen, W. Sullivan, A. Swidler and S. Tipton, 1985, page 286 which suggests factoring
in of such social changes.
"The transformation of our culture and our society would have to happen at a number of
levels. If it occurred only in the minds of individuals (as to some degree it already has) it
would be powerless. If it came only from the initiative of the State, it would be
tyrannical. Personal transformation among large numbers is essential, and it must not
only be a transformation of consciousness but must also involve individual action. But
individuals need the nurture of crops that carry a moral tradition reinforcing their own
aspirations.
These are commitments that require a new social ecology and a social movement
dedicated to the idea of such a transformation."
International Treaties
9. International treaties vis-a-vis the
@page-SC666
rights of women was noticed by this Court in a large number of judgments, some of
which we may notice at this stage.
10

. In Githa Hariharan v. Reserve Bank of India [(1999) 2 SCC 228], this Court was faced
with construing Section 6(a) of Hindu Minority and Guardianship Act, 1956 and Section
19(b) of Guardians and Wards Act, 1890. The sections were challenged as violative of the
equality clause of the Constitution, inasmuch as the mother of the minor is relegated to an
inferior position on ground of sex alone since her right, as a natural guardian of the
minor, is made cognizable only 'after' the father. The court relied upon the Convention on
the Elimination of All Forms of Discrimination Against Women, 1979 ("CEDAW") and
the Beijing Declaration, which directs all State parties to take appropriate measures to
prevent discrimination of all forms against women is quite clear. It was held by the court
that the domestic courts are under an obligation to give due regard to International
Conventions and Norms for construing domestic laws when there is no inconsistency
between them. 1999 AIR SCW 811

11

. In Air India v. Nergesh Meerza [(1981) 4 SCC 335], this Court was faced with the
constitutional validity of Regulation 46(i)(c) of Air India Employees' Service
Regulations, it was provided that the services of the Air Hostesses would stand
terminated on first pregnancy. The Court after considering various US Supreme Court
judgments regarding pregnant women held that the observations made therein would
apply to the domestic cases. AIR 1981 SC 1829

12

. In Municipal Corporation of Delhi v. Female Workers (Muster Roll) and Anr. [(2000) 3
SCC 224], the short question which was to be decided by this Court was whether having
regard to the provisions contained in Maternity Benefit Act, 1961, women engaged on
casual basis or on muster roll basis on daily wages and not only those in regular
employment were eligible for maternity leave. The Court while upholding the right of the
female workers to get maternity leave relied upon the doctrine of social justice as
embodied in Universal Declaration of Human Rights Act, 1948 and Article 11 of the
Convention on the Elimination of All Forms of Discrimination Against Women held that
the provisions of the same must be read into the service contracts of Municipal
Corporation. 2000 AIR SCW 969

13

. In Madhu Kishwar and Ors. v. State of Bihar and Ors. [(1996) 5 SCC 125], challenge
was made to certain provisions of Chotanagpur Tenancy Act, 1908 providing succession
to property in the male line in favour of the male on the premise that the provisions are
discriminatory and unfair against women and, therefore, ultra vires the equality clause in
the Constitution. The Court while upholding the fundamental right of the Tribal women
to the right to livelihood held that the State was under an obligation to enforce the
provisions of the Vienna Convention on the Elimination of All Forms of Discrimination
Against Women (CEDAW) which provided that discrimination against women violated
the principles of equality of rights and respects for human dignity. 1996 AIR SCW 2178

14
. In Vishaka and Ors. v. State of Rajasthan and Ors. [(1997) 6 SCC 241], the writ petition
was filed for the enforcement of the fundamental rights of working women under Articles
14, 19 and 21 of the Constitution of India with the aim of finding suitable methods for
realization of the true concept of "gender equality"; and preventing sexual harassment of
working women in all work places through judicial process to fill the vacuum in existing
legislation. This Court while framing the guidelines and norms to be observed by the
employers in work places to ensure the prevention of sexual harassment of women, inter
alia, relied on the provisions in the Convention on the Elimination of All Forms of
Discrimination Against Women as also the general recommendations of CEDAW for
construing the nature and ambit of constitutional guarantee of gender equality in our
Constitution. 1997 AIR SCW 3043

15

. In Randhir Singh v. Union of India and Ors. [(1982) 1 SCC 618], this Court while
holding that non-observance of the principle of 'equal pay for equal work' for both men
and women under Article 39(d) of the Constitution amounted to violation of Articles 14
and 16, recognized that the principle was expressly recognized by all socialist systems of
law including the Preamble to the Constitution of the International Labour Organization.
AIR 1982 SC 879

16. In Liverpool and London S.P. and I. Association Ltd. v. M.V. Sea Success I and Anr.
[(2004) 9 SCC 512], this Court had to
@page-SC667
interpret the meaning and import of the word 'necessaries' used in Section 5 of the
Admiralty Court Act, 1861. The Court while importing the meaning of the same through
Foreign (American) Court decisions, opined - :
"It is true that this Court is not bound by the American decisions. The American decisions
have merely a persuasive value but this Court would not hesitate in borrowing the
principles if the same is in consonance with the scheme of Indian law keeping in view the
changing global scenario. Global changes and outlook in trade and commerce could be a
relevant factor. With the change of time, from narrow and pedantic approach, the court
may resort to broad and liberal interpretation. What was not considered to be a necessity
a century back, may be held to be so now."
Setting of the Debate
17. In the instant matter, we are in the thick of debate relating to Individual Rights of
women. The classical counter to individual rights is the community orientation of rights.
There is no such shade to the current matter. Here the individual rights are challenged by
a problem of practical import of enforcement and security.
18. Therefore, the important jurisprudential tenet involved in the matter is not the
prioritization of rights inter se but practical implementation issues competing with a right.
It is one thing when two norms falling in the same category (for instance Individual
Rights versus Community Orientation of Rights) compete and quite another when two
norms with unequal hierarchical status come in conflict with each other.
19. At the very outset we want to define the contours of the discussion which is going to
ensue. Firstly, the issue floated by the State is very significant, nonetheless does not fall
in the same class as that of rights which it comes in conflict with, ontologically. Secondly,
the issue at hand has no social spillovers. The rights of women as individuals rest beyond
doubts in this age. If we consider (various strands of) feminist jurisprudence as also
identity politics it is clear that time has come that we take leave of the theme
encapsulated under Section 30. And thirdly we will also focus our attention on the
interplay of doctrines of self-determination and an individual's best interests.
Equality
20. When the original Act was enacted, the concept of equality between two sexes was
unknown. The makers of the Constitution intended to apply equality amongst men and
women in all spheres of life. In framing Articles 14 and 15 of the Constitution, the
constitutional goal in that behalf was sought to be achieved. Although the same would not
mean that under no circumstance, classification, inter alia, on the ground of sex would be
wholly impermissible but it is trite that when the validity of a legislation is tested on the
anvil of equality clauses contained in Articles 14 and 15, the burden therefor would be on
the State. While considering validity of a legislation of this nature, the court was to take
notice of the other provisions of the Constitution including those contained in Part IV A
of the Constitution.
21. In Bhe and Ors. v. The Magistrate, Khayelisha and Ors. [(2004) 18 BHRC 52], the
South African Constitutional Court was required to consider the constitutionality of the
Black Administration Act, 1927 (South Africa) and the Regulations of the Administration
and Distribution of the Estates of Deceased Blacks (South Africa). This scheme was
purporting to give effect to the customary law of succession where principle of male
primogeniture is central to customary law of succession.
It was held by the majority that the rule of male primogeniture as it applied in customary
law to the inheritance of property was inconsistent with the Constitution and invalid to
the extent that it excluded or hindered women and extra-marital children from inheriting
property. The rules of succession in customary law had not been given the space to adapt
and to keep pace with changing social conditions and values. Instead, they had overtime
become increasingly out of step with the real values and circumstances of the society they
were meant to serve. The application of the customary law rules of succession in
circumstances vastly different from their traditional setting caused much hardship. Thus
the official rules of customary law of succession were no longer universally observed.
The exclusion of women from inheritance on the grounds of gender was a clear violation
of the constitutional prohibition against unfair discrimination.
Further, the principle of primogeniture
@page-SC668
also violated the right of women to human dignity as it implied that women were not fit
or competent to own and administer property. Its effect was to subject those women to a
status of perpetual minority, placing them automatically under the control of male heirs,
simply by virtue of gender differentiation.
Remark on changing realities
22. We may now look into the ground reality. In India, hospitality industry has grown by
leaps and bounds. As noticed hereinbefore, liquor, in the hospitality industry, is being
served not only in the bar but also in the restaurant. Service of liquor is permissible also
in the rooms of a hotel.
23. The impugned provision provides for wide restrictions. It prohibits employment of
any woman in any part of the premises where liquor is being served. It would prohibit
employment of women and men below 25 years in any of the restaurants. As liquor is
permitted to be served even in rooms, the restriction would also operate in any of the
services including housekeeping where a woman has to enter into a room; the logical
corollary of such a wide restriction would be that even if service of liquor is made
permissible in the flight, the employment of women as air-hostesses may be held to be
prohibited.
24. Hotel Management has opened up a viesta of young men and women for
employment. A large number of them are taking hotel management graduation courses.
They pass their examinations at a very young age. If prohibition in employment of
women and men below 25 years is to be implemented in its letter and spirit, a large
section of young graduates who have spent a lot of time, money and energy in obtaining
the degree or diploma in hotel management would be deprived of their right of
employment. Right to be considered for employment subject to just exceptions is
recognized by Article 16 of the Constitution. Right of employment itself may not be a
fundamental right but in terms of both Articles 14 and 16 of the Constitution of India,
each person similarly situated has a fundamental right to be considered therefor. When a
discrimination is sought to be made on the purported ground of classification, such
classification must be founded on a rational criteria. The criteria which in absence of any
constitutional provision and, it will bear repetition to state, having regard to the societal
conditions as they prevailed in early 20th century, may not be a rational criteria in the
21st century. In the early 20th century, the hospitality sector was not open to women in
general. In the last 60 years, women in India have gained entry in all spheres of public
life. They have also been representing people at grass root democracy. They are now
employed as drivers of heavy transport vehicles, conductors of service carriage, pilots et.
al. Women can be seen to be occupying Class IV posts to the post of a Chief Executive
Officer of a Multinational Company. They are now widely accepted both in police as also
army services.
Res Extra Commercium Issue
25. Occupation/service in the management of hotel industry is a specialized job. It
requires specialized skill. To deprive a large section of successful young men and women
from obtaining any job for which they have duly been trained, in our opinion, would be
wholly unjust. The State cannot invoke the doctrine of 'res extra commercium' in the
matter of appointment of eligible persons. The said principle could have been invoked if
the State intended to adopt a policy of prohibition. It is one thing to say that the trade in
liquor is regulated but it is another thing to say that such regulations which are principally
in the area of manufacture, sale, export and import of intoxicants should be allowed to
operate in other fields also.
26

. In Kerala Samsthana Chethu Thozhilali Union v. State of Kerala and others [(2006) 4
SCC 327], this Court held: 2006 AIR SCW 1869, Paras 55 and 57
"When an employer gives employment to a person, a contract of employment is entered
into. The right of the citizens to enter into any contract, unless it is expressly prohibited
by law or is opposed to public policy, cannot be restricted. Such a power to enter into a
contract is within the realm of the Indian Contract Act. It has not been and could not be
contended that a contract of employment in the toddy shops would be hit by Section 23 of
the Indian Contract Act. So long as the contract of employment in a particular trade is not
prohibited either in terms of the statutory or constitutional scheme, the State's
intervention would be unwarranted unless there exists a statutory interdict. Even to what
extent such a legislative power can be exercised would be the subject-matter of debate
but in a case of this
@page-SC669
nature there cannot be any doubt that the impugned rules are also contrary to the
provisions of the Indian Contract Act as also the Specific Relief Act, 1963."
It was further observed:
"Furthermore, a person may not have any fundamental right to trade or do business in
liquor, but the person's right to grant employment or seek employment, when a business
is carried on in terms of the provisions of the licence, is not regulated."
Parens Patriae Power of State
27. One important justification to Section 30 of the Act is parens patriae power of State. It
is a considered fact that use of parens patriae power is not entirely beyond the pale of
judicial scrutiny.
28. Parens Patriae power has only been able to gain definitive legalist orientation as it
shifted its underpinning from being merely moralist to a more objective grounding i.e.
utility.
29. The subject matter of the Parens Patriae power can be adjudged on two counts:
(i) in terms of its necessity, and
(ii) assessment of any tradeoff or adverse impact, if any.
30. This inquiry gives the doctrine an objective orientation and therefore prevents it from
falling foul of due process challenge. (See City of Cleburne v. Cleburne Living Center,
473 US 432, 439-41 (1985)). Parens Patriae power is subject to constitutional challenge
on the ground of Right to Privacy also. Young men and women know what would be the
best offer for them in the service sector. In the age of internet, they would know all pros
and cons of a profession. It is their life; subject to constitutional, statutory and social
interdicts - a citizen of India should be allowed to live her life on her own terms.
31. Let us understand various standards which objectify Parens Patriae. Best interests
standard is one test in US jurisdiction in Child Custody matters. Similarly other standards
have evolved amongst which right to self-determination holds an important place.
Right to employment vis-a-vis Security: Competing Values
32. The instant matter involves a fundamental tension between right to employment and
security.
33. The fundamental tension between autonomy and security is difficult to resolve. It is
also a tricky jurisprudential issue. Right to Self-Determination is an important offshoot of
Gender Justice discourse. At the same time, security and protection to carry out such
choice or option specifically, and state of violence-free being generally is another tenet of
the same movement. In fact, the latter is apparently a more basic value in comparison to
right to options in the feminist matrix.
34. Privacy rights prescribe autonomy to choose profession whereas security concerns
texture methodology of delivery of this assurance. But it is a reasonable proposition that
that the measures to safeguard such a guarantee of autonomy should not be so strong that
the essence of the guarantee is lost. State protection must not translate into censorship.
35. At the same time we do not intend to further the rhetoric of empty rights. Women
would be as vulnerable without State protection as by the loss of freedom because of
impugned Act. The present law ends up victimizing its subject in the name of protection.
In that regard the interference prescribed by State for pursuing the ends of protection
should be proportionate to the legitimate aims. The standard for judging the
proportionality should be a standard capable of being called reasonable in a modern
democratic society.
36. Instead of putting curbs on women's freedom, empowerment would be a more tenable
and socially wise approach. This empowerment should reflect in the law enforcement
strategies of the State as well as law modelling done in this behalf.
37. Also with the advent of modern State, new models of security must be developed.
There can be a setting where the cost of security in the establishment can be distributed
between the State and the employer.
38. Gender equality today is recognized by the European Court as one of the key
principles underlying the Convention and a goal to be achieved by member States of the
Council of Europe.
In the case of Abdulaziz, Cabales and Balkandali v. United Kingdom, [1985] ECHR 7,
the Court held:
"As to the present matter, it can be said that the advancement of the equality of the sexes
is today a major goal in the member
@page-SC670
States of the Council of Europe. This means that very weighty reasons would have to be
advanced before a difference of treatment on the ground of sex could be regarded as
compatible with the Convention."
Following Abdulaziz (supra) the European Court of Human Rights once again observed
in Van Raalte v. The Netherlands, [1997] ECHR 6:
"In the applicant's submission, differences in treatment based on sex were already
unacceptable when section 25 of the General Child Care Benefits Act was enacted in
1962. The wording of Article 14 of the Convention (Art. 14) showed that such had been
the prevailing view as early as 1950. Moreover, legal and social developments showed a
clear trend towards equality between men and women. The applicant drew attention to,
inter alia, the Court's Abdulaziz, Cabales and Balkandali v. the United Kingdom
judgment of 28 May 1985 (Series A No. 94), which stated explicitly that "the
advancement of the equality of the sexes is today a major goal in the member States of
the Council of Europe" and that "very weighty reasons would have to be advanced before
a difference of treatment on the ground of sex could be regarded as compatible with the
Convention" (loc. cit., p. 38, para. 78)."
(Emphasis supplied)
(See also Schuler-Zgraggen v. Swizerland, [1993] ECHR 29; and Petrovic v. Austria,
[1998] ECHR 21)
Stereotype Roles and Right to Options
39. Professor Williams in "The Equality Crisis: Some Reflections on Culture, Courts, and
Feminism" published in 7 WOMEN'S RTS. L. REP. 175 (1982) notes issues arising
where biological distinction between sexes is assessed in the backdrop of cultural norms
and stereotypes. She characterizes them as "hard cases". In hard cases, the issue of
biological difference between sexes gathers an overtone of societal conditions so much so
that the real differences are pronounced by the oppressive cultural norms of the time. This
combination of biological and social determinants may find expression in popular
legislative mandate. Such legislations definitely deserve deeper judicial scrutiny. It is for
the court to review that the majoritarian impulses rooted in moralistic tradition do not
impinge upon individual autonomy. This is the backdrop of deeper judicial scrutiny of
such legislations world over.
40. Therefore, one issue of immediate relevance in such cases is the effect of the
traditional cultural norms as also the state of general ambience in the society which
women have to face while opting for an employment which is otherwise completely
innocuous for the male counterpart. In such circumstances the question revolves around
the approach of State.
41. Instead of prohibiting women employment in the bars altogether the State should
focus on factoring in ways through which unequal consequences of sex differences can be
eliminated. It is State's duty to ensure circumstances of safety which inspire confidence in
women to discharge the duty freely in accordance to the requirements of the profession
they choose to follow. Any other policy inference (such as the one embodied under
section 30) from societal conditions would be oppressive on the women and against the
privacy rights.
42. The description of the notion of "romantic paternalism" by the US Supreme Court in
Frontiero v. Richardson (411 U.S. 677, 93 S.Ct. 1764) makes for an interesting reading. It
is not to say that Indian society is similarly situated and suffers from the same degree of
troublesome legislative past but nevertheless the tenor and context are not to be missed.
The court noted in this case of military service:
"There can be no doubt that our Nation has had a long and unfortunate history of sex
discrimination. Traditionally, such discrimination was rationalized by an attitude of
'romantic paternalism' which, in practical effect, put women, not on a pedestal, but in a
cage........
As a result of notions such as these, our statute books gradually became laden with gross,
stereotyped distinctions between the sexes......."
The court also maintained the strict scrutiny standard for review and repelled the
administrative convenience argument in the following terms:
"In any case, our prior decisions make clear that, although efficacious administration of
governmental programs is not without some importance, 'the Constitution recognizes
higher values than speed and efficiency.' And when we enter the realm of 'strict judicial
scrutiny,' there can be no
@page-SC671
doubt that 'administrative convenience' is not a shibboleth, the mere recitation of which
dictates constitutionality.
On the contrary, any statutory scheme which draws a sharp line between the sexes, solely
for the purpose of achieving administrative convenience, necessarily commands
'dissimilar treatment for men and women who are similarly situated,' and therefore
involves the 'very kind of arbitrary legislative choice' forbidden by the (Constitution). We
therefore conclude that, by according differential treatment to male and female members
of the uniformed services for the sole purpose of achieving administrative convenience,
the challenged statutes violate the Due Process Clause of the Fifth Amendment."
43. In another similar case wherein there was an effective bar on females for the position
of guards or correctional counsellors in the Alabama State penitentiary system. The
prison facility housed sexual offenders and the majority opinion on this basis inter alia
upheld the bar. Justice Marshall's dissent captures the ranges of issues within a
progressive paradigm. Dissent in Dothard v. Rawlinson (433 U.S. 321, 97 S.Ct. 2720)
serves as useful advice in the following terms:
"It appears that the real disqualifying factor in the Court's view is 'the employee's very
womanhood.' The Court refers to the large number of sex offenders in Alabama prisons,
and to 'the likelihood that inmates would assault a woman because she was a woman.' In
short, the fundamental justification for the decision is that women as guards will generate
sexual assaults. With all respect, this rationale regrettably perpetuates one of the most
insidious of the old myths about women that women, wittingly or not, are seductive
sexual objects. The effect of the decision, made, I am sure with the best of intentions, is
to punish women because their very presence might provoke sexual assaults. It is women
who are made to pay the price in lost job opportunities for the threat of depraved conduct
by prison inmates. Once again, 'the pedestal upon which women have been placed has
upon closer inspection, been revealed as a cage.' It is particularly ironic that the cage is
erected here in response to feared misbehavior by imprisoned criminals."
He also notes the nature of protective discrimination (as garb) in the following terms:
"The Court points to no evidence in the record to support the asserted 'likelihood that
inmates would assault a woman because she was a woman.' Perhaps the Court relies upon
common sense, or 'innate recognition'. But the danger in this emotionally laden context is
that common sense will be used to mask the "romantic paternalism" and persisting
discriminatory attitudes that the Court properly eschews. To me, the only matter of innate
recognition is that the incidence of sexually motivated attacks on guards will be minute
compared to the 'likelihood that inmates will assault' a guard because he or she is a guard.
The proper response to inevitable attacks on both female and male guards is not to limit
the employment opportunities of law-abiding women who wish to contribute to their
community, but to take swift and sure punitive action against the inmate offenders.
Presumably, one of the goals of the Alabama prison system is the eradication of inmates'
antisocial behavior patterns so that prisoners will be able to live one day in free society.
Sex offenders can begin this process by learning to relate to women guards in a socially
acceptable manner. To deprive women of job opportunities because of the threatened
behavior of convicted criminals is to turn our social priorities upside down."
The Standard of Judicial Scrutiny
44. It is to be borne in mind that legislations with pronounced "protective discrimination"
aims, such as this one, potentially serve as double edged swords. Strict scrutiny test
should be employed while assessing the implications of this variety of legislations.
Legislation should not be only assessed on its proposed aims but rather on the
implications and the effects. The impugned legislation suffers from incurable fixations of
stereotype morality and conception of sexual role. The perspective thus arrived at is
outmoded in content and stifling in means.
45. No law in its ultimate effect should end up perpetuating the oppression of women.
Personal freedom is a fundamental tenet which cannot be compromised in the name of
expediency until unless there is a compelling State purpose. Heightened level of scrutiny
is the normative threshold for judicial review in such cases.
46. Professor Christine A. Littleton in her widely quoted article RECONSTRUCTING
@page-SC672
SEXUAL EQUALITY, 75 CALR 1279, July 1987 makes a useful observation in this
regard:
"The difference between human beings, whether perceived or real, and whether
biologically or socially based, should not be permitted to make a difference in the lived-
out equality of those persons. I call this the model of 'equality as acceptance.' To achieve
this form of sexual equality, male and female 'differences' must be costless relative to
each other."
47. Having regard to the scope of Section 30 of the Act and the impugned legislation
generally the Court has to reach to a finding as to whether the legislative interference to
the autonomy in employment opportunities for women is justified as a legitimate aim and
proportionate to the aim pursued. In this behalf it would be relevant to understand the
approach of European Court of Human Rights which has very often dealt with matters of
competing public interests and tuned new legal devices for the same. Doctrine of
Proportionality and Incompatibility would definitely find mention in such a discussion.
48. The test to review such a Protective Discrimination Statute would entail a two
pronged scrutiny:
(a) the legislative interference (induced by sex discriminatory legalisation in the instant
case) should be justified in principle,
(b) the same should be proportionate in measure.
49. The Court's task is to determine whether the measures furthered by the State in form
of legislative mandate, to augment the legitimate aim of protecting the interests of women
are proportionate to the other bulk of well-settled gender norms such as autonomy,
equality of opportunity, right to privacy et al. The bottom-line in this behalf would be a
functioning modern democratic society which ensures freedom to pursue varied
opportunities and options without discriminating on the basis of sex, race, caste or any
other like basis. In fine, there should be a reasonable relationship of proportionality
between the means used and the aim pursued.
50. In United States v. Virginia (518 U.S. 515, 532-33 (1996)) Justice Ginsburg notes
with particular emphasis the need for an intrusive multi-stage review in sex
discrimination statutes. The court observed :
"The heightened review standard our precedent establishes does not make sex a
proscribed classification. Supposed "inherent differences" are no longer accepted as a
ground for race or national origin classifications. Physical differences between men and
women, however, are enduring. "Inherent differences" between men and women, we have
come to appreciate, remain cause for celebration, but not for denigration of the members
of either sex or for artificial constraints on an individual's opportunity. Sex classifications
may be used to compensate women "for particular economic disabilities [they have]
suffered," to "promote equal employment opportunity," to advance full development of
the talent and capacities of our Nation's people. But such classifications may not be used,
as they once were, to create or perpetuate the legal, social, and economic inferiority of
women." (internal citations omitted)
Changing Stand of the Government of NCT Delhi
51. The Government of NCT Delhi, although did not challenge the impugned judgment
of the Delhi High Court, seeks to enter into the fray through a side door. It, on the one
hand, challenges the locus of the appellant which objection, if upheld, would make the
appeal liable to be dismissed at the threshold, on the other, seeks to justify the validity of
Section 30 of the Act. It cites examples of Jessica Lal and BMW to highlight dangerous
consequences of allowing sale and consumption of liquor by young men below the age of
25 years and vulnerability of women while working in bars. When the restrictions were in
force, they could not prevent such occurrences. If the restriction goes, some such
incidents may again happen. But only on a pre-supposition that there is a possibility of
some incident happening, we cannot declare a law intra vires which is ex facie ultra vires.
52. We, furthermore, deprecate this practice of the Government of NCT to raise a
contention of the aforementioned nature which not only had not been raised before the
High Court but in an appeal filed by a few citizens maintainability whereof is in question.
It, having allowed the judgment of High Court to attain finality, is estopped by records to
question the correctness of the impugned judgment.
@page-SC673
Conclusion
53. In the instant case the end result is an invidious discrimination perpetrating sexual
differences.
54. Young men who take a degree or diploma in Hotel Management enter into service at
the age of 22 years or 23 years. It, thus, cannot prohibit employment of men below 25
years. Such a restriction keeping in view a citizen's right to be considered for
employment, which is a facet of the right to livelihood do not stand judicial scrutiny.
55. For the reasons aforementioned, we do not find any infirmity in the impugned
decision of the High Court. The appeal is accordingly dismissed. Cross-appeal filed by
the respondents is allowed. There shall be no order as to costs.
Appeal dismissed.
AIR 2008 SUPREME COURT 673 "Nopany Investments (P) Ltd., M/s. v. Santokh Singh
(HUF)"
(From : Delhi)*
Coram : 2 TARUN CHATTERJEE AND P. SATHASIVAM, JJ.
Civil Appeal No. 5761 of 2007 (arising out of SLP (C) No. 9963 of 2007), D/- 10 -12
-2007.
M/s. Nopany Investments (P.) Ltd. v. Santokh Singh (HUF).
HINDU LAW - EVICTION - TENANCY - ESTOPPEL - Hindu Law - HUF - Karta -
Junior member of joint Hindu Family - Can act as Karta of family - Said member all
along been realising rent from tenant - Suit for eviction of tenant from HUF Property
filed by said member - Tenant estopped from raising question of maintainability of suit at
instance of said member of HUF.
Evidence Act (1 of 1872), S.115.
If the Karta of the HUF was away in a remote place, (in this case in a foreign country)
and his return within a reasonable time was unlikely, a junior member of HUF could act
as the Karta of the family. In the instant case, the elder brother 'D' who is permanently
staying in United Kingdom was /is not in a position to handle the joint family property
for which reason he has himself executed a Power of Attorney in favour of younger
brother 'J', member of HUF. Furthermore, there has been no protest, either by 'D' or by
any member of the HUF to the filing of the suit of eviction of tenant from HUF property
by 'J'. That apart it would not be open to the tenant to raise the question of maintainability
of the suit at the instance of 'J', as the record shows that 'J' has all along been realizing the
rent from the tenant and for this reason, the tenant is now estopped from raising any such
question. Therefore, the suit is maintainable at the instance of 'J' claiming himself to be
the Karta of the HUF. (Paras 6, 7)
(B) Civil P.C. (5 of 1908), S.100, O.41, R.31 - APPEAL - APPELLATE COURT - TRIAL
COURT - Second appeal - Finding arrived at by first Appellate Court affirming judgment
of Trial Court - Same found by High Court as neither very cryptic nor based on non-
consideration of arguments advanced by parties before it - And was in compliance with
O. - 41, C. P. C. - Not liable to be set aside. (Paras 8, 9)
(C) Delhi Rent Control Act (59 of 1958), S.6A, S.15 - HOUSES AND RENTS -
TENANCY - EVICTION - Rent - Increase in - Landlord can raise rent by 10% every
three years - Irrespective of fact that eviction proceedings is pending and order under S.
15 has been passed by Additional Rent Controller - However, notice of increase of rent
under S. 8 has to be served on tenant intimating him intention to make increase. (Para
10)
(D) Delhi Rent Control Act (59 of 1958), S.3(c), S.14 - HOUSES AND RENTS -
EVICTION - TENANCY - LEASE - Eviction - Suit pending against tenant - Landlord
increasing rent of tenanted premises after giving notice - Said notice was also notice to
quit - On said increase suit premises brought out of purview of Act in view of S. 3(c) -
Suit for eviction of tenant withdrawn by landlord - Filing of fresh suit for eviction of
tenant under general law - Issuance of notice to quit to tenant - Not necessary under S.
106, T. P. Act.
Transfer of Property Act (4 of 1882), S.106. (Paras 12, 13, 14)
Cases Referred : Chronological Paras
2006 AIR SCW 1098 : AIR 2006 SC 1471 (Rel. on) 7
2001 AIR SCW 723 : AIR 2001 SC 965 (Disting.) 8, 9
2001 AIR SCW 1804 : AIR 2001 SC 2171 : 2001 AIR Kant - HCR 2672 (Disting.)
9
2001 AIR SCW 3326 : AIR 2001 SC 3580 (Disting.) 13
@page-SC674

1994 AIR SCW 5011 (Ref.) 11


1991 AIR SCW 1467 : AIR 1991 SC 1538 (Foll.) 6, 7
AIR 1988 SC 576 (Disting.) 6, 7
AIR 1979 SC 1745 (Rel. on) 12
AIR 1976 SC 1953 (Rel. on) 7
Bhaskar P. Gupta, Sr. Advocate, Rana Mukherjee, Subodh K. Pathak, Ms. Geeta Luthra,
Shashi Ranjan, B. Patnaik, Ms. Pinky Anand and D. N. Gobardhan, for Appellant; Rajiv
Dutta, Sr. Advocate, Rajesh Goyal, Ms. Mandeep Kaur and V. P. Singh, for Respondent.
* RSA No. 209 of 2005, D/- 19-4-2007 (Del).
Judgement
1. TARUN CHATTERJEE, J. - :- Leave granted.
2. This appeal has been preferred before us, assailing the judgment and decree dated 19th
of April, 2007, passed by the High Court of Delhi, whereby, the High Court had
dismissed the appeal of the appellant, thereby affirming the judgments of the courts
below decreeing the eviction suit filed at the instance of the respondent against the
appellant.
3. The facts leading to the filing of this appeal may be stated as follows :
4. On 16th of July, 1980, the appellant entered into a lease with Dr. Santokh Singh HUF
for a period of 4 years, with respect to the property situated at N-112, Panchsheel Park,
New Delhi (for short "the suit premises"), at a monthly rent of Rs. - 3500/-. Accordingly,
at the expiry of the aforesaid period of 4 years, a notice of eviction dated 5th of April,
1984 was issued which was followed by filing an Eviction Petition No. 432 of 1984
before the Additional Rent Controller by Jasraj Singh, claiming himself to be the Karta of
Dr. Santokh Singh HUF. The Additional Rent Controller passed an order directing the
appellant for payment of rent at the rate of Rs. 3500/-. After coming into force of Section
6A of the Delhi Rent Control Act, a notice dated 9th of January, 1992 was sent by Jasraj
Singh, in the above capacity, to the appellant for enhancement of rent by 10 per cent and
also termination of tenancy of the appellant. In reply to this notice, the appellant denied
the right of the respondent to enhance the rent. Another notice dated 31st of March, 1992
was sent afresh by the respondent notifying the appellant that the rent stood enhanced by
10 per cent while the tenancy stood terminated w.e.f. 16/17th of July, 1992. The aforesaid
Eviction Petition No. 432 of 1984 was withdrawn on 20th of August, 1992 by Jasraj
Singh. Thereafter, a notice dated 3rd of September, 1992 was sent by Jasraj Singh asking
the appellant to vacate the suit property to which the appellant did not concede and
refused to vacate the same by a reply dated 24th of September, 1992. On 6th of February,
1993, Dr. Santokh Singh HUF, through Jasraj Singh, claiming himself to be the Karta of
the HUF, instituted a suit seeking eviction of the appellant from the suit premises. The
trial court decreed the respondent's suit for possession, against which an appeal was
preferred before the Additional District Judge, Delhi. The first appellate court dismissed
the appeal summarily. Against this order of the first appellate court, a second appeal,
being R.S.A. No. 146 of 2003, was preferred before the High Court of Delhi, which
remanded the matter to the first appellate court for fresh consideration. In pursuance of
this direction of the High Court, the first appellate court, after fresh consideration of the
matter, affirmed the judgment passed by the Trial Court thereby dismissing the appeal of
the appellant herein. Being aggrieved and dissatisfied with the order of the first appellate
court, the appellant preferred a second appeal, being R.S.A. No. 209 of 2005, before the
High Court of Delhi, which, however, was also dismissed. It is this decision of the High
Court of Delhi, which is impugned in this appeal and in respect of which leave has
already been granted.
5. The pivotal questions, inter alia, in the facts and circumstances of this case, which
warrant our determination are as follows - :
(i) Whether Jasraj Singh could file the suit for eviction, in the capacity of the Karta of Dr.
Santokh Singh HUF, when, admittedly, an elder member of the aforesaid HUF was alive -
?
(ii) Whether the High Court was right in concluding that the first appellate court had duly
dealt with all the issues involved and re-appreciated evidence as provided under O.41,
R.31 of the Code of Civil Procedure (in short "the CPC") - ?
(iii) Whether the contractual tenancy between the landlord and tenant came to an end
merely by filing an Eviction Petition and whether the landlord could seek enhancement of
rent simultaneously or post-termination of tenancy - ?
@page-SC675
(iv) Whether the landlord could issue a notice under Section 6A of the Delhi Rent Control
Act, 1958 (in short "the Act") for increase of rent without seeking leave of the Rent
Controller during the pendency of an order under Section 15 of the Act directing the
tenant to deposit rent on a month to month basis - ?
6

. We have heard the learned counsel for the parties. As regards the first issue, as noted
hereinabove, the learned senior counsel Mr. Gupta appearing on behalf of the appellant
had questioned the maintainability of the suit filed at the instance of Jasraj Singh,
claiming himself to be the Karta of Dr. Santokh Singh HUF. The learned counsel Mr.
Gupta strongly argued before us that in view of the settled principle of law that the junior
member in a joint family cannot deal with the joint family property as Karta so long as
the elder brother is available, the respondent herein, who is admittedly a junior member
of the family, could not have instituted the eviction suit, claiming himself to be the Karta
of the family. In support of this argument, the learned senior counsel Mr. Gupta has
placed reliance on the decisions of this court in Sunil Kumar and another v. Ram Prakash
and others [(1988) 2 SCC 77] and Tribhovan Das Haribhai Tamboli v. Gujarat Revenue
Tribunal and others [(1991) 3 SCC 442]. Before we look at the views expressed by the
High Court on this question, it would be pertinent to note the ratios of the two authorities
cited before us. In Sunil Kumar and another v. Ram Prakash and others [supra], this court
held as follows - :- AIR 1988 SC 576
1991 AIR SCW 1467
AIR 1988 SC 576, (Para 21)

"In a Hindu family, the Karta or Manager occupies a unique position. It is not as if
anybody could become Manager of a joint Hindu family. As a general rule, the father of a
family, if alive, and in his absence the senior member of the family, is alone entitled to
manage the joint family property."
From a reading of the aforesaid observation of this court in Sunil Kumar and another v.
Ram Prakash and others [supra], we are unable to accept that a younger brother of a joint
Hindu family would not at all be entitled to manage the joint family property as the Karta
of the family. This decision only lays down a general rule that the father of a family, if
alive, and in his absence the senior member of the family would be entitled to manage the
joint family property. Apart from that, this decision was rendered on the question whether
a suit for permanent injunction, filed by co-parcerners for restraining the Karta of a joint
Hindu family from alienating the joint family property in pursuance of a sale agreement
with a third party, was maintainable or not. While considering that aspect of the matter,
this court considered as to when could the alienation of joint family property by the Karta
be permitted. Accordingly, it is difficult for us to agree with Mr. Gupta, learned senior
counsel appearing for the appellant, that the decision in Sunil Kumar and another v. Ram
Prakash and others [supra] would be applicable in the present case which, in our view,
does not at all hold that when the elder member of a joint Hindu family is alive, the
younger member would not at all be entitled to act as a Manager or Karta of the joint
family property.

In Tribhovandas's case [supra], this court held as follows - : 1991 AIR SCW 1467, (Para
13)

"The managership of the joint family property goes to a person by birth and is regulated
by seniority and the Karta or the Manager occupies a position superior to that of the other
members. A junior member cannot, therefore, deal with the joint family property as
manager so long as the Karta is available except where the Karta relinquishes his right
expressly or by necessary implication or in the absence of the Manager in exceptional and
extraordinary circumstances such as distress or calamity affecting the whole family and
for supporting the family or in the absence of the father whose whereabouts were not
known or who was away in remote place due to compelling circumstances and that his
return within the reasonable time was unlikely or not anticipated."
(Emphasis supplied)
From a careful reading of the observation of this court in Tribhovandas's case [supra], it
would be evident that a younger member of the joint Hindu family can deal with the joint
family property as Manager in the following circumstances - :-
(i) if the senior member or the Karta is not available;
(ii) where the Karta relinquishes his right expressly or by necessary implication;
@page-SC676
(iii) in the absence of the manager in exceptional and extraordinary circumstances such as
distress or calamity affecting the whole family and for supporting the family;
(iv) in the absence of the father - :-
(a) whose whereabouts were not known, or
(b) who was away in a remote place due to compelling circumstances and his return
within a reasonable time was unlikely or not anticipated.
Therefore, in Tribhovandas's case [supra], it has been made clear that under the aforesaid
circumstances, a junior member of the joint Hindu family can deal with the joint family
property as manager or act as the Karta of the same.
7. From the above observations of this court in the aforesaid two decisions, we can come
to this conclusion that it is usually the Father of the family, if he is alive, and in his
absence the senior member of the family, who is entitled to manage the joint family
property. In order to satisfy ourselves whether the conditions enumerated in
Tribhovandas's case [supra] have been satisfied in the present case, we may note the
findings arrived at by the High Court, which are as follows - :-
(i) Jasraj Singh, in his cross-examination before the trial court had explained that his
eldest brother Dhuman Raj Singh (supposed to be the Karta of the HUF) has been living
in United Kingdom for a long time. Therefore, the trial court had rightly presumed that
Dhuman Raj Singh was not in a position to discharge his duties as Karta of the HUF, due
to his absence from the country.
(ii) The respondent produced the Xerox copy of the power of attorney given by Dhuman
Raj Singh to Jasraj Singh.
(iii) The trial court relied upon the law discussed in the books namely, "Principles of
Hindu Law" by Mulla and Mulla and "Shri S.V. Gupta on Hindu Law", wherein it has
been observed that ordinarily, the right to act as the Karta of HUF is vested in the senior-
most male member but in his absence, the junior members can also act as Karta.
(Emphasis supplied)
(iv) There was no protest by any member of the joint Hindu family to the filing of the suit
by Jasraj Singh claiming himself to be the Karta of the HUF. There was also no whisper
or protest by Dhuman Raj Singh against the acting of Jasraj Singh as the Karta of the
HUF.

It may also be noted that the High Court relied on the decision of this court in
Narendrakumar J. Modi v. Commissioner of Income Tax, Gujarat II, Ahmedabad [(AIR)
1976 SC 1953], wherein it was held that so long as the members of a family remain
undivided, the senior member of the family is entitled to manage the family
properties ................. and is presumed to be manager until contrary is shown, but the
senior member may give up his right of management, and a junior member may be
appointed manager. Another decision in Mohinder Prasad Jain v. Manohar Lal Jain [2006
II AD (SC) 520], was also relied upon by the High Court wherein it has been held at
paragraph 10 as follows - : 2006 AIR SCW 1098, (Para 11)

"10. A suit filed by a co-owner, thus, is maintainable in law. It is not necessary for the co-
owner to show before initiating the eviction proceeding before the Rent Controller that he
had taken option or consent of the other co-owners. However, in the event, a co-owner
objects thereto, the same may be a relevant fact. In the instant case, nothing has been
brought on record to show that the co-owners of the respondent had objected to eviction
proceedings initiated by the respondent herein."
Having relied on the aforesaid decisions of this Court and a catena of other decisions and
the findings arrived at by it, as noted hereinabove, the High Court rejected the argument
of the appellant that Jasraj Singh could not have acted as the Karta of the family as his
elder brother, namely, Dhuman Raj Singh, being the senior most member of the HUF,
was alive. In view of our discussions made herein-earlier and considering the principles
laid down in Tribhovandas's case [supra] and Sunil Kumar's case [supra], we neither find
any infirmity nor do we find any reason to differ with the findings arrived at by the High
Court in the impugned judgment. It is true that in view of the decisions of this court in
Sunil Kumar's case [supra] and Tribhovandas's case [supra], it is only in exceptional
circumstances, as noted herein earlier, that a junior member can act as the Karta of the
family. But we venture to mention here that Dhuman Raj Singh, the senior member of the
HUF, admittedly, has been staying permanently in the United Kingdom for a long time. In
@page-SC677
Tribhovandas's case [supra] itself, it was held that if the Karta of the HUF was away in a
remote place, (in this case in a foreign country) and his return within a reasonable time
was unlikely, a junior member could act as the Karta of the family. In the present case,
the elder brother Dhuman Raj Singh, who is permanently staying in United Kingdom
was/is not in a position to handle the joint family property for which reason he has
himself executed a power of attorney in favour of Jasraj Singh. Furthermore, there has
been no protest, either by Dhuman Raj Singh or by any member of the HUF to the filing
of the suit by Jasraj Singh. That apart, in our view, it would not be open to the tenant to
raise the question of maintainability of the suit at the instance of Jasraj Singh as we find
from the record that Jasraj Singh has all along been realizing the rent from the tenant and
for this reason, the tenant is now estopped from raising any such question. In view of the
discussions made hereinabove, we are, therefore, of the view that the High Court was
fully justified in holding that the suit was maintainable at the instance of Jasraj Singh,
claiming himself to be the Karta of the HUF.
8. This takes us to the next issue namely, whether the High Court was right in concluding
that the first appellate court had duly dealt with all the issues involved and re-appreciated
the evidence as provided under O.41, R.31 of the CPC. The learned senior counsel for the
appellant Mr. Gupta sought to argue that the High Court had erred in holding that the first
appellate court had acted in due compliance with O.41 of the CPC. It may be noted that
the High Court, while concluding as aforesaid, came to the following findings - :-
1) The first appellate court has passed a speaking order and it is apparent that it has
applied its mind.
2) The First appellate court had to deal with the arguments which were advanced before
it. It had rightly given the short shrift to all those arguments which did not inject some
coherence.
3) The learned counsel for the appellant had failed to point out the issues regarding which
the First Appellate Court had not given its own conclusion.

4) The learned counsel for the appellant had also failed to show as to how the authority
cited viz., Santosh Hazari vs. Purushottam Tiwari (dead) by L.Rs. [AIR 2001 SC 965]
was applicable to the facts of the case. 2001 AIR SCW 723

. In our view, it is difficult for us to set aside the findings of the High Court on the
question whether the first appellate court, while deciding the questions of fact and law,
had complied with the requirements under O.41 of the CPC. We are in agreement with
the findings of the High Court as on a perusal of the judgment of the first appellate court,
it does not appear to us that the findings arrived at by the first appellate court affirming
the judgment of the trial court on any issue were either very cryptic or based on non-
consideration of the arguments advanced by the parties before it. In support of this
contention, before the High Court, the appellant had relied on a decision of this court in
the case of Santosh Hazari [supra], but in this appeal, the learned senior counsel for the
appellant Mr. Gupta has strongly relied on a decision of this court in the case of
Madhukar and Ors. vs. Sangram and Ors. [(2001) 4 SCC 756] and contended that since
the judgment of the first appellate court was cryptic in nature and the first appellate court
had not dealt with the issues involved in the appeal, the same was liable to be set aside
and the matter was liable to be sent back to the first appellate court for rehearing. We are
unable to accept this contention of the learned senior counsel for the appellant. Before we
consider the findings of the first appellate court as well as the High Court on this issue,
we must keep on record that in Madhukar and Ors. vs. Sangram and Ors. [supra], this
court had to reverse the findings of the High Court because the High Court erred in
allowing the plaintiff/respondent's first appeal without even considering the grounds on
which the trial court had dismissed the suit and without discussing the evidence on
record. On the same lines, the decision of this court in Santosh Hazari's case [supra] was
based. In our view, the aforesaid two decisions of this court are distinguishable on facts
with the present case. A perusal of the judgment of the first appellate court after remand
would clearly indicate that the same was neither cryptic nor based on non-consideration
of the issues involved in the appeal. Apart from that, it has to be kept in mind that the
decisions of this court in Madhukar and Ors. vs. Sangram and Ors. [supra] and Santosh
Hazari's case 2001 AIR SCW 1804
2001 AIR SCW 723, (Para 15)

@page-SC678
[supra], were considering the reversal of the findings of fact of the trial court. In the
present case, the first appellate court had affirmed the findings of the trial court, which
were based on total consideration of the material evidence - documentary and oral on
record. It is well settled that in the case of reversal, the first appellate court ought to give
some reason for reversing the findings of the trial court whereas in the case of
affirmation, the first appellate court accepts the reasons and findings of the trial court. In
any view of the matter, from a perusal of the judgment of the first appellate court, it is
clear that it reflects conscious application of mind and has recorded the findings
supported by reason on all the issues arising along with the contentions put forward by
the parties. In Santosh Hazari's case [supra], this court observed:-
"The task of an appellate court affirming the findings of the trial court is an easier one.
The appellate court agreeing with the view of the trial court need not restate the effect of
the evidence or reiterate the reasons given by the trial court; expression of general
agreement with the reasons given by the court, decision of which is under appeal, would
ordinarily suffice."
(Emphasis supplied).
Again, in Madhukar and Ors. vs. Sangram and Ors. [supra], this court had to set aside the
judgment of the High Court because the first appellate court was singularly silent as to
any discussion, either of the documentary or the oral evidence. In addition, this court in
that decision was of the view that the findings of the first appellate court were so cryptic
that none of the relevant aspects were noticed. In this background, this court at paragraph
8 observed as follows - :-
"Our careful perusal of the judgment in the first appeal shows that it hopelessly falls short
of considerations which are expected from the court of first appeal. We, accordingly set
aside the impugned judgment and decree of the High Court and remand the first appeal to
the High Court for its fresh disposal in accordance with law."
In view of our discussions made hereinabove, we are, therefore, unable to agree with the
learned senior counsel for the appellant Mr. Gupta that the High Court was not justified in
holding that the findings of the first appellate court were in compliance with O.41 of the
CPC. That apart, the learned senior counsel for the appellant Mr. Gupta could not satisfy
us or even point out the specific issues which, in his opinion, had been left to be
addressed by the first appellate court. In view of the discussions made hereinabove, we
are, therefore, of the view that no ground was made out by the appellant to set aside the
judgment of the High Court on the question whether the judgment of the first appellate
court was liable to be set aside for non-compliance with the mandatory provisions of
O.41 of the CPC.
10. Let us now deal with Issue Nos. 3 and 4. Since both these issues are interlinked, we
shall deal with these two issues together. Let us first consider whether the respondent
landlord could issue a notice under Section 6A of the Act for increase of rent when the
petition for eviction of the appellant was pending before the Additional Rent Controller
and when there had been an order to the tenant for deposit of rent on a month to month
basis under Section 15 of the Act. In our view, the first appellate court as well as the High
Court were fully justified in holding that it was open to a landlord to increase the rent of
the suit premises by 10% after giving a notice under section 6A of the Act. In this
connection, it would be appropriate to reproduce Section 6A of the Act which talks about
revision of rent and Section 8 of the Act which contemplates notice of increase of rent.
Section 6A runs as under - :-
"6A. Revision of rent - Notwithstanding anything contained in this Act, the standard rent,
or, where no standard rent is fixed under the provisions of this Act in respect of any
premises, the rent agreed upon between the landlord and the tenant, may be increased by
ten per cent every three years."
From a bare perusal of this provision under Section 6A of the Act, it is evident that by
this statutory provision, the standard rent and in cases where no standard rent is fixed
under the Act in respect of any premises, the rent agreed upon between the landlord and
the tenant, may be increased by 10% every three years. It is, therefore, open to the
landlord under Section 6A of the Act to increase the rent agreed upon between him and
the tenant by 10 % every three years, irrespective of the fact that an eviction proceeding
is pending and an order under Section 15 of the Act has been passed by the Additional
Rent Controller except that when a landlord wishes to so increase the rent of
@page-SC679
any premises, a notice of increase of rent, as provided under Section 8 of the Act, has to
be served on the tenant thereby intimating the tenant his intention to make the increase.
Section 8 of the Act runs as under:-
"Notice of increase of rent - (1) Where a landlord wishes to increase the rent of any
premises, he shall give the tenant notice of his intention to make the increase and in so far
as such increase is lawful under this Act, it shall be due and recoverable only in respect of
the period of the tenancy after the expiry of thirty days from the date on which the notice
is given.
(2) Every notice under sub-section (1) shall be in writing signed by or on behalf of the
landlord and given in the manner provided in section 106 of the Transfer of Property Act,
1882 (4 of 1882)."
Therefore, if the landlord wishes to increase the rent of any premises at any time, only a
notice expressing his intention to make such increase is required to be given to the tenant
and Section 6A of the Act, as noted herein earlier, clearly permits the landlord to increase
the rent by 10% every three years. In this view of the matter, after the completion of three
years, it was open to the landlord at any point even during the pendency of an eviction
petition to increase the rent of the suit premises after giving the prescribed notice to the
tenant.
11. At this stage, we may also consider Section 3(c) of the Act, which bars the application
of the Act to the premises whose monthly rent exceeds Rs. 3500/-. Section 3(c) of the Act
runs as under - :-
"Act not to apply to certain premises - Nothing in this Act shall apply-
(a) ..................
(b) ................
(c) to any premises, whether residential or not, whose monthly rent exceeds three
thousand and five hundred rupees;"

The Delhi Rent Control Act, 1958 was amended by Act No. 52 of 1988, which came into
effect from 1st of December, 1988. By this amendment of the 1958 Act on 1st of
December, 1988, Section 3(c) with other amendments was brought into force. Section
3(c) of the amended act provides that the provisions of the Act will not apply to any
premises whose monthly rent exceeds Rs. 3500/- from the date of coming into operation
of this Act. In D.C. Bhatia and others Vs. Union of India and another [(1995) 1 SCC
104], while considering the parent Act and the amending Act, this court held that the
objects of the amending Act are quite different from the objects of the parent Act. It
observed that one of the objects of the amending Act was to rationalize the rent control
law by bringing about a balance between the interest of landlords and tenants. It was also
observed that the object was not merely to protect the weaker section of the community.
The Rent Act had brought to a halt house-building activity for letting out. This court also
made an observation that many people with accommodation to spare did not let out the
accommodation for the fear of losing the accommodation. As a result of all these, there
was acute shortage of accommodation which caused hardship to the rich and the poor
alike and that in the background of this experience, the amending Act of 1988 was
passed. In paragraph 28 of the aforesaid decision in D.C. Bhatia's case [supra], this court
observed as follows - :- 1994 AIR SCW 5011

"In order to strike a balance between the interests of the landlords and also the tenants
and for giving a boost to house-building activity, the legislature in its wisdom has decided
to restrict the protection of the Rent Act only to those premises for which rent is payable
up to the sum of Rs 3500 per month and has decided not to extend this statutory
protection to the premises constructed on or after the date of coming into operation of the
Amending Act for a period of ten years. This is a matter of legislative policy. The
legislature could have repealed the Rent Act altogether. It can also repeal it step by step.
It has decided to confine the statutory protection to the existing tenancies whose monthly
rent did not exceed Rs. 3500."
Considering the aforesaid reasons which led to the amending Act of 1988, it is clear that
the legislature intended to strike a balance by allowing the landlords to evict a tenant,
who could pay more than Rs. 3500/- per month, from the tenanted premises.
12. In the present case, after serving a notice under Section 6A read with Section 8 of the
Act, the protection of the tenant under the Act automatically ceased to exist as the rent of
the tenanted premises exceeded Rs. 3500/- and the bar of Section 3(c) came into play. At
the risk of repetition, since, in the present case, the increase of rent by 10% on the rent
agreed upon between the
@page-SC680
appellant and the respondent brought the suit premises out of the purview of the Act in
view of Section 3(c) of the Act, it was not necessary to take leave of the rent controller
and the suit, as noted herein above, could be filed by the landlord under the general law.
The landlord was only required to serve a notice on the tenant expressing his intention to
make such increase. When the eviction petition was pending before the Additional Rent
Controller and the order passed by him under Section 15 of the Act directing the
appellant to deposit rent at the rate of Rs. 3500/- was also subsisting, the notice dated 9th
of January, 1992 was sent by the respondent to the appellant intimating him that he
wished to increase the rent by 10 per cent. Subsequent to this notice, another notice dated
31st of March, 1992 was sent by the respondent intimating the appellant that by virtue of
the notice dated 9th of January, 1992 and in view of Section 6A of the Act, the rent stood
enhanced by 10 per cent i.e. from Rs. 3500/- to Rs. 3850/?. It is an admitted position that
the tenancy of the appellant was terminated by a further notice dated 16/17th of July,
1992. Subsequent to this, the eviction petition No. 432 of 1984 was withdrawn by the
respondent on 20th of August, 1992 and the suit for eviction, out of which the present
appeal has arisen, was filed on 6th of February, 1993. That being the factual position, it
cannot at all be said that the suit could not be filed without the leave of the Additional
Rent Controller when, admittedly, at the time of filing of the said suit, the eviction
petition before the Additional Rent Controller had already been withdrawn nor can it be
said that the notice of increase of rent and termination of tenancy could not be given
simultaneously, when, in fact, the notice dated 16/17th of July, 1992 was also a notice to
quit and the notice intending increase of rent in terms of Section 6A of the Act was earlier
in date than the notice dated 16/17th of July, 1992. In any view of the matter, it is well
settled that filing of an eviction suit under the general law itself is a notice to quit on the
tenant. Therefore, we have no hesitation to hold that no notice to quit was necessary
under Section 106 of the Transfer of Property Act in order to enable the respondent to get
a decree of eviction against the appellant. This view has also been expressed in the
decision of this court in V. Dhanapal Chettiar Vs. Yesodai Ammal [AIR 1979 SC 1745].
13

. Before parting with this judgment, we may deal with a decision of this court in the case
of Ambalal Sarabhai Enterprises Ltd. Vs. Amrit Lal and Co. and another [(2001) 8 SCC
397] on which the learned senior counsel for the appellant Mr. Gupta placed strong
reliance. Relying on this decision, Mr. Gupta sought to argue that the amendment of the
Act being not retrospective in operation, in view of Section 6 of the General Clauses Act,
it would not affect the pending eviction proceeding, which would continue as if the Act
had not been amended and therefore, the suit filed by the respondent for eviction under
the general law without taking leave from the Additional Rent Controller could not be
said to be maintainable. In our view, the decision of this court in Ambalal Sarabhai's case
[supra] does not support the appellant but it supports the respondent. In that decision, this
court held that the vested right of the landlord under the general law continues so long it
is not abridged by the protective legislation, namely, the Rent Act, but the moment this
protection is withdrawn, the vested right of the landlord reappears which can be enforced
by him. Such being the position, we are, therefore, of the view that since the eviction
petition filed by the respondent before the Additional Rent Controller was withdrawn and
the tenancy was terminated by a fresh notice to quit and in view of the increase of rent
wished by the landlord in compliance with Section 6A read with Section 8 of the Act,
there cannot be any difficulty to hold that the suit in fact was maintainable under the
general law. That being the position, the decision of this court in Ambalal Sarabhai's case
[supra] cannot at all be applicable in favour of the appellant and which, in view of our
discussions made hereinabove, can only be applicable in favour of the respondent. 2001
AIR SCW 3326

14. For the reasons aforesaid, none of the grounds urged by the learned senior counsel for
the appellant Mr. Gupta can be accepted by us to interfere with the impugned judgment of
the High Court. Accordingly, the appeal fails and is hereby dismissed. However,
considering the facts and circumstances of the case, we grant time to the appellant to
vacate the premises in question by 29th of February, 2008 provided the appellant files an
usual undertaking in this regard in this court within a fortnight from this date. In default,
it will be open
@page-SC681
to the respondent to proceed to execute the decree for eviction of the appellant from the
suit premises in accordance with law. There will be no order as to costs.
Appeal dismissed.
AIR 2008 SUPREME COURT 681 "Magna Publishing Co. Ltd. v. Shilpa S. Shetty"
(From : Bombay)*
Coram : 2 Dr. A. PASAYAT AND P. SATHASIVAM, JJ.
Civil Appeal No. 344 of 2002, D/- 14 -12 -2007.
Magna Publishing Co. Ltd. and Ors. v. Shilpa S. Shetty.
Constitution of India, Art.133 - Civil P.C. (5 of 1908), O.39, R.1 - APPEAL -
INJUNCTION - DEFAMATION - Appeal - Against interim order - Interim order passed
in suit for defamation against magazine - Publisher restrained from re-publishing articles
in question or writing any defamatory article in future - Court refused to interfere - Suit
directed to be disposed of early. (Para 11)

E. C. Agrawala, Mahesh Agarwal, Rishi Agrawala, Amit Kumar Sharma, for Appellants;
S. K. Bhattacharya, for Respondent.
* Appeal No. 128 of 2001 in notice of Motion No. 25 of 2001 in Suit No. 36 of 2001,D/-
16-7-2001 (Bom).
Judgement
1. Dr. ARIJIT PASAYAT, J. - :-Heard learned counsel for the parties.
2. It appears that vide an interim order dated 12.1.2001, the High Court granted ad
interim injunction and a Division Bench of the Bombay High Court refused to interfere.
3. A brief reference to the factual aspects would suffice - :
4. The respondent filed a suit claiming that she is a film actress of good standing. Certain
articles were published in the magazine published by the appellants called Stardust. A suit
for damages of Rs.20 crores alleging that the articles are defamatory in nature and would
affect her career and for injunction restraining the appellants from publishing defamatory
articles was filed. Notice of motion for interim injunction was taken out. Learned Single
Judge was of the prima facie view that the articles deal with the personal life and are
defamatory in nature and granted interim injunction. The interim injunction reads as
follows - :
"Therefore, as directed in the case of Indian Express Newspapers (supra), a modified
injunction is hereby granted restraining the defendants from republishing the three
articles and/or from writing and publishing any defamatory article in the nature of the
three articles alleging that the plaintiff is having relationship with other actors or a
married man, which will operate till the disposal of the suit."
5. The said order dated 12.01.2001, as noted above, was challenged in appeal.
6. Before the Division Bench, the stand was that the interim injunction granted was
beyond the prayer made in the notice of motion. The High Court noted that in notice of
motion, the prayer was in the following terms - :
"That pending the hearing and final disposal of the suit, this Hon'ble Court be pleased to
issue an order and injunction restraining the defendants from in any way or manner
carrying out defamatory, allegations and imputations in future against the plaintiff".
7. The Division Bench was of the further view that the Learned Single Judge had not
granted interim protection beyond what was prayed and was covered by the prayer.
8. The other stand before the Division Bench was that moment justification is pleaded,
there can be no interim protection. This plea was also rejected stating that a person cannot
be defamed by allowing such publications in future. Justification shall be required to be
established at the time of hearing of the suit by leading evidence.
9. There were certain other stands relating to lack of pleadings about the reputation and
character. The Division Bench found that also to be without substance. The appeal was
accordingly dismissed.
10. Learned counsel for the appellant reiterated the stand taken before the Division
Bench. Mr. Bhattacharya, learned counsel for the respondent supported the order.
11. We find that the matter relates to an interim order and while granting leave, the prayer
for grant of interim relief was refused. In other words, interim order passed by learned
Single Judge as upheld by the Division Bench continues to be operative. Therefore,
without expressing any opinion on the merits of the case, we think it proper to dispose of
the appeal without interference. We,
@page-SC682
however, request the High Court to explore the possibility of early disposal of the suit
No.36/2001.
12. The appeal is disposed of accordingly.
Order accordingly.
AIR 2008 SUPREME COURT 682 "Sewa Ram v. State of U. P."
(From : 2005 All LJ 2830)
Coram : 2 Dr. A. PASAYAT AND AFTAB ALAM, JJ.
Criminal Appeal No. 1695 of 2007 (arising out of SLP (Cri.) No. 5400 of 2005), D/- 11
-12 -2007
Sewa Ram and Anr. v. State of U.P.
(A) Penal Code (45 of 1860), S.34 - COMMON INTENTION - Common intention - S.
34 has been enacted on principle of joint liability in doing of a criminal act - Section is
only a rule of evidence and does not create a substantive offence - Distinctive feature of
section is element of participation in action. (Para 12)
(B) Penal Code (45 of 1860), S.34 - COMMON INTENTION - Common intention -
Direct proof of common intention is seldom available - Such intention can only be
inferred from circumstances appearing from proved facts of case. (Para 12)
(C) Penal Code (45 of 1860), S.34 - COMMON INTENTION - Common intention - True
concept of section is that if two or more persons intentionally do an act jointly, position in
law is just the same as if each of them has done it individually by himself.
AIR 1977 SC 109, Relied on. (Para 12)
(D) Penal Code (45 of 1860), S.34 - COMMON INTENTION - Common intention -
Section does not say "the common intentions of all", nor does it say "an intention
common to all" - Under provisions of S. 34, essence of liability is to be found in
existence of a common intention animating accused leading to doing of a criminal act in
furtherance of such intention. (Para 13)
(E) Penal Code (45 of 1860), S.34 - COMMON INTENTION - Common intention -
Provision is intended to meet a case in which it may be difficult to distinguish between
acts of individual members of a party who act in furtherance of common intention of all
or to prove exactly what part was taken by each of them.
1993 AIR SCW 1843 and 2004 AIR SCW 810, Relied on. (Para 13)
(F) Penal Code (45 of 1860), S.300, S.34 - COMMON INTENTION - MURDER -
EVIDENCE - Murder case - Common intention - Application of principles enunciated in
S. 34 - Accused persons allegedly assaulted deceased with lathis and kanta - Evidence of
eye-witnesses suffered from no infirmity - Accused had common intention to murder
deceased due to enmity arising out of litigation pending between them - Conviction of
accused in terms of S. 302 r/w S. 34 is proper. (Paras 10, 15)
Cases Referred : Chronological Paras
2004 AIR SCW 810 : AIR 2004 SC 1808 : 2004 Cri LJ 1388 : 2004 All LJ 683 (Rel. on)
14
1993 AIR SCW 1843 : AIR 1993 SC 1899 : 1993 Cri LJ 2246 (Rel on) 13
AIR 1977 SC 109 : 1977 Cri LJ 164 (Rel. on) 12
Balraj Dewan, for Appellants; Sahdev Singh, Sandeep Singh and Anuvrat Sharma, for
Respondent.
Judgement
Dr. ARIJIT PASAYAT, J. - :-Leave granted.
2. Challenge in this appeal is to the judgment rendered by a Division Bench of the
Allahabad High Court dismissing the appeal filed by the appellants. Before the High
Court three persons had filed the appeal. During the pendency of the appeal, appellant
No.2 Ram Prasad died. Therefore, the appeal was held to have abetted so far he is
concerned.
3. The appellants were found guilty of having committed an offence punishable under
Section 302 read with Section 34 of the Indian Penal Code, 1860 (in short 'IPC') and each
was sentenced to undergo imprisonment for life. Appellant-Sewa Ram and the deceased-
accused Ram Prasad were further convicted for offence punishable under Section 323
read with Section 34 IPC and each was sentenced to undergo RI for six months and to
pay a fine of Rs.500/- with default stipulation.
@page-SC683
4. The conviction was recorded by IV learned Additional Sessions Judge, Pillibhit, in
Sessions Trial No. 249 of 1980.
5. Prosecution version as unfolded during trial is as follows - :
The complainant Shaukat Ali, son of Nathu Bux, resident of Barhara, P.S. Bisalpur, was
doing service at the house of Jagan Nath Prasad resident of village Chandpura who was
related to Smt. Genda Devi, widow of Jwala Prasad Kurmi, resident of Naugamia, P.S.
Bilsanda, who is hereinafter referred to as the 'deceased'. Litigation was going on
between Smt. Genda Devi and her step daughter Smt. Savitri Devi, who was living as
wife of Ram Prasad of village Naugamia. On 22.8.1980, the complainant Shaukat Ali
along with Jagan Nath and Smt. Genda Devi had gone to Tehsil Bisalpur in connection
with the litigation and they were returning from Tehsil to village Chandpura at about 4
O'clock and when they reached the outskirts of village Kangawan near the sugar cane
field of Babuji at 6 O'clock Smt. Genda Devi was going ahead; behind her was the
complainant and behind him was Jagan Nath. Suddenly accused Ram Prasad, Sewa Ram
and Parmeshwari having 'Lathis' in their hands and accused Sunder Lal having 'Kanta' in
his hand suddenly came out from the sugar cane field and began to beat Smt. Genda Devi
on which the complainant and Jagan Nath cried for help. Accused Ram Prasad and Sewa
Ram then beat the complainant with 'Lathis'. The complainant and Jagan Nath ran away
towards the village Kangavan and the accused ran away towards east. The complainant
and Jagan Nath saw Smt. Genda Devi and found that she had died in the paddy field. The
accused persons had murdered Smt. Genda Devi due to enmity of litigation. Thereafter
the complainant informed the residents of village Chandpura and the 'Chaukidar',
Pradhan and other people of the village, who came with the complainant to the spot. It
had fallen dark and due to fear, he at once did not come to the police station to lodge the
F.I.R. and remained sitting the whole night looking after the dead body. In the next
morning on 23.8.1980, the complainant lodged the F.I.R.(Ex. Ka. 3) at the Bisalpur police
station. The crime was registered as crime No. 247 under Sections 302/323 I.P.C. at the
Bisalpur police station and the S.O. Ram Lakhan Singh was entrusted with the
investigation of the case. The details were entered in the G.D., a copy of which is Ex.Ka.
4. The I.O. along with the S.I. Sahabdin arrived at the spot and prepared inquest report
(Ex. Ka. 8) of the dead-body of Smt. Genda Devi. The dead-body was sealed and sample
seal was preserved, which is Ex. Ka. 11. The I.O. made spot inspection and prepared the
site-plan Ex. Ka. 5. The post mortem of the dead-body of Smt. Genda Devi was
conducted by Dr. V.P. Agarwal. The complainant Shaukat Ali who received injuries was
also examined at the P.H.C. Bisalpur. His injury report is Ex. Ka.1. After completion of
the investigation, the I.O. submitted charge-sheet against the accused persons.
Finding a prima facie case against the accused persons, they were charged under Section
302 read with Section 34 I.P.C. and Section 323 read with Section 34 I.P.C. The charges
were read over and explained to the accused persons who pleaded not guilty and claimed
to be tried.
In support of the prosecution version Shaukat Ali, (P.W.1), Dr. C.K. Chaturvedi (PW2)
who conducted the medical examination of Shaukat Ali and Jagan Nath (P.W.3), Dr. V.P.
Agarwal, (PW 4) who conducted post mortem of the deceased Smt. Genda Devi, A.C.
Pancham Singh (PW 5), Constable Rampal Sharma, (PW 6) and S.I. Ramlakhan Singh
(PW 7) who conducted investigation were produced. The accused were examined who
denied the allegations and contended that they have been falsely implicated in this case
due to enmity.
6. Shaukat Ali the informant (PW1) and Jagan Nath (PW-3) claimed to be eye-witnesses.
The trial Court relying on the version of the eye-witnesses recorded conviction and
imposed sentence as aforesaid. Before the High Court the stand was that there was
inordinate delay in dispatching special report to the Magistrate. In addition, it was
submitted that the doctor who conducted the post-mortem noticed seven injuries and out
of them three were incised wounds and injury 3, 4, 5, and 7 were lacerated wounds. It
was submitted that the three accused persons who preferred appeal before the High Court
were holding lathies and the Kanthi was held by Sewa Ram who had been acquitted.
Therefore, it was submitted that offence under Section 302 IPC was not made out.
7. The case of the prosecution was that in view of the factual background offence
@page-SC684
punishable under Section 302 read with Section 34 IPC is made out. The High Court, as
noted above, dismissed the appeal filed by the appellants.
8. In support of the appeal learned counsel for the appellant submitted that even if
prosecution version is accepted in totality, offence under Section 302 is not made out,
much less by application of Section 34 IPC.
9. Learned counsel for the respondent-State supported the judgment of the High Court.
10. As rightly held by the High Court the evidence of the eye-witnesses PWs 1 and 3
suffered from no infirmity. The trial Court was, therefore, justified in convicting and
holding appellants guilty.
11. So far as the question as to whether Section 302 will be applied so far as appellants
are concerned, it is to be noted that the trial Court and the High Court considered their
cases in the background of Section 34, IPC.
12

. Section 34 has been enacted on the principle of joint liability in the doing of a criminal
act. The section is only a rule of evidence and does not create a substantive offence. The
distinctive feature of the section is the element of participation in action. The liability of
one person for an offence committed by another in the course of criminal act perpetrated
by several persons arises under Section 34 if such criminal act is done in furtherance of a
common intention of the persons who join in committing the crime. Direct proof of
common intention is seldom available and, therefore, such intention can only be inferred
from the circumstances appearing from the proved facts of the case and the proved
circumstances. In order to bring home the charge of common intention, the prosecution
has to establish by evidence, whether direct or circumstantial, that there was plan or
meeting of minds of all the accused persons to commit the offence for which they are
charged with the aid of Section 34, be it pre-arranged or on the spur of the moment; but it
must necessarily be before the commission of the crime. The true concept of the section
is that if two or more persons intentionally do an act jointly, the position in law is just the
same as if each of them has done it individually by himself. As observed in Ashok Kumar
v. State of Punjab [1977(1) SCC 746] the existence of a common intention amongst the
participants in a crime is the essential element for application of this section. It is not
necessary that the acts of the several persons charged with commission of an offence
jointly must be the same or identically similar. The acts may be different in character, but
must have been actuated by one and the same common intention in order to attract the
provision. AIR 1977 SC 109

13
. The section does not say "the common intentions of all", nor does it say "an intention
common to all". Under the provisions of Section 34 the essence of the liability is to be
found in the existence of a common intention animating the accused leading to the doing
of a criminal act in furtherance of such intention. As a result of the application of
principles enunciated in Section 34, when an accused is convicted under Section 302 read
with Section 34, in law it means that the accused is liable for the act which caused death
of the deceased in the same manner as if it was done by him alone. The provision is
intended to meet a case in which it may be difficult to distinguish between acts of
individual members of a party who act in furtherance of the common intention of all or to
prove exactly what part was taken by each of them. As was observed in Chinta Pulla
Reddy v. State of A.P. [1993 Supp (3) SCC 134]. Section 34 is applicable even if no
injury has been caused by the particular accused himself. For applying Section 34 it is not
necessary to show some overt act on the part of the accused. 1993 AIR SCW 1843

14

. The above position was highlighted in Girija Shankar v. State of U.P. [2004 (3) SCC
793]. 2004 AIR SCW 810

15. When the factual background is considered in the light of the principles highlighted
above, the inevitable conclusion is that the appellants have been rightly convicted in
terms of Section 302 read with Section 34, IPC.
16. The appeal is without merit and is dismissed.
Appeal dismissed.
@page-SC685
AIR 2008 SUPREME COURT 685 "Laxman Prasad v. Prodigy Electronics Ltd."
(From : Delhi)
Coram : 2 C. K. THAKKER AND ALTAMAS KABIR, JJ.
Civil Appeal No. 5751 of 2007 (arising out of SLP (C) No. 12405 of 2006), D/- 10 -12
-2007.
Laxman Prasad v. Prodigy Electronics Ltd. and Anr.
(A) Civil P.C. (5 of 1908), S.20 - CIVIL COURT - Territorial jurisdiction - Purpose that
S. - 20 intends to achieve.
Section 20 has been designed to secure that justice might be brought as near as possible
to every man's hearthstone and that the defendant should not be put to the trouble and
expense of travelling long distances in order to defend himself. (Para 21)
(B) Civil P.C. (5 of 1908), S.20 - CIVIL COURT - CONTRACT - PLEA - Territorial
jurisdiction - Cause of action - Suit for damages for breach of confidentiality clause in
service contract - Service contract entered into outside India - Breach of confidentiality
clause taking place at place 'D' in India - Court of place 'D' would have jurisdiction to try
suit - Plea that Court at place where agreement was entered or place where defendant
resides only had jurisdiction - Not tenable. (Paras 22)
(C) Civil P.C. (5 of 1908), S.20 - CIVIL COURT - CONTRACT - Territorial jurisdiction -
Cause of action - Different from law that would govern - Service contract with
confidentiality clause - Contract stipulating that Hong Kong Law would apply - Does not
mean that only Courts in Hong Kong would have jurisdiction - Breach of confidentiality
clause taking place in India - Indian Court would have jurisdiction as part of cause of
action arose in India - Law applicable would be as stipulated in contract. (Paras 15, 31)
(D) Civil P.C. (5 of 1908), S.35 - Constitution of India, Art.133 - COSTS - APPEAL -
APPELLATE COURT - Costs - Imposition of costs is in discretion of Court - When
Court, in light of facts have imposed costs - Appellate Court would not interfere. (Para
32)
Cases Referred : Chronological Paras
(2005)5 SCC 465 (Disting.) 29
1993 AIR SCW 131 : AIR 1993 SC 998 (Disting.) 28
(1990) 3 SCC 481 (Disting.) 24, 26
AIR 1989 SC 1239 (Ref.) 17
(1873) 8 CP 107 : 42 LJ CP 98 16
Manoj Swarup, Ajay Kumar and Arun Kumar Beriwal, for Appellant; Nikhil Nayyar,
Ankit Singhal and TVSR Sreyaj, for Respondents.
Judgement
1. C. K. THAKKER, J. - :- Leave granted.
2. The present appeal is directed against the judgment and order passed by the High Court
of Delhi on April 26, 2006 in I.A. No. 9562 of 2005 in Civil Suit (OS) No. 819 of 2005.
By the said order, the High Court dismissed an application filed by the appellant herein
under Order VII, Rules 10 and 11 of the Code of Civil Procedure, 1908 (hereafter
referred to as 'the Code').
3. Shortly stated the facts are that 'Prodigy Electronics Ltd.'-plaintiff (respondent No.1
herein) ('the Company' for short) was formed and incorporated under the laws of Hong
Kong and is engaged in the business of trading electronic goods under the name and style
'Prodigy Electronics', Hong Kong. The main area of business of the Company is Printed
Circuit Board (PCB). The business carried on by the Company involves keen
understanding of the requirements of the customers in order that the products may be
manufactured to the specific needs of the customers and they may be made available at
competitive prices. The Company is, therefore, required to take care of the consumer-
complaints, if any. It also involves a reach into the market identifying the potential
consumers of the products which involves substantial investment of time, effort and
finance. According to the Company, it has developed solid reputation in India under the
trade name and trade mark 'Prodigy Electronics' in the field of electronics generally and
PCBs particularly.
4. According to the plaintiff-Company, on July 22, 2002, the defendant (appellant herein)
joined Prodigy Electronics in India as a representative for marketing PCB products of the
Company in India. An employment contract was entered into on October 2, 2003 between
the defendant-employee and the plaintiff-Company. Under the said contract, the
defendant was given full-time employment in the Company at Hong Kong in the capacity
of 'International Business Development Manager'. He was given job
@page-SC686
profile of conducting all business of the Company in India. Subsequently, at the request
of the defendant who put forward personal reasons, he was relocated to India and a new
employment agreement dated September 13, 2004 was signed by the parties. It was
agreed that the job location of the appellant would be India. The defendant's tenure in
India started from October 1, 2004. The Company was to bear all expenditure, including
travel-expenses of the defendant. According to the Company, however, after relocation to
India in October, 2004, the defendant tendered his resignation by e-mail on the ground of
personal problems. It was also stated that he would decide his next course of action later
on. Though the Company gave assurance to the defendant to support in his personal
problems, the defendant did not withdraw the resignation and thus his employment came
to be terminated on December 20, 2004. According to the Company, the defendant sent
another e-mail on December 20, 2004 giving assurance to the Company that though he
would continue to be involved in marketing of PCB products, he would be associating
himself with manufacturers other than the customers and suppliers of the Company.
5. It is the allegation of the Company that the defendant contacted potential customers of
the Company and informed them that he was representing 'Prodigy'. He also submitted
quotes for PCB products. It was also the case of the Company that the defendant
participated in the Trade Fair in Delhi (Componex/Electronic India, 2005) which was
held between February 1, 2005 to February 4, 2005 at Pragati Maidan, New Delhi. In the
said Trade Fair, the defendant used the goodwill and passed on the trade name of the
plaintiff-Company. In the course of inquiries about the data generated about PCB
customers, the organizers of the Trade Fair informed the Company that Multi Circuit
Board (CHINA) Ltd., Hong Kong had participated in the fair and information could be
obtained from them. It was averred that the Company was shocked to receive the said
information since Multi Circuit Board (CHINA) Ltd. was the manufacturer from whom
the Company used to source its products for its Indian customers. The further inquiry by
the Company revealed that the said Multi Circuit Board (CHINA) Ltd. had a
representative in India and he was none else but the defendant who operated under the
name and style of "Prodigy Circuit Boards". On being contacted, Multi Circuit Board
(CHINA) Ltd. also confirmed that it had executed a contract with the defendant. It also
came to the notice of the Company that the defendant had registered a deceptively similar
domain name 'www.prodigycircuits.com' as far back as on October 5, 2004 while he was
still in the employment of the Company. The Company was thus convinced that the
defendant had not resigned on account of personal reasons or family problems but he
wanted to misuse confidential information which he had received from the Company and
he wanted to take undue advantage in spite of the agreement entered into with the
Company. The Company also discovered that the defendant had incorporated a Company
under the name and style of 'Canton Treasure Corporation Ltd.' on July 16, 2004 when he
was stationed in Hong Kong and was still serving with the Company. Thus, obvious
breach of employment contract was committed by the appellant.
6. In view of the above facts and breach of contract by the defendant, the plaintiff-
Company on May 28, 2005, filed a suit being Civil Suit (OS) No. 819 of 2005 in the
High Court of Delhi at New Delhi (Original Jurisdiction) for permanent and mandatory
injunction against the defendant as also for damages by ordering rendition of accounts.
Along with the plaint, the Company filed an application under Order XXXIX, Rules 1
and 2 read with Section 151 of the Code for interim injunction restraining the defendant
from using the name 'Prodigy', 'Prodigy Circuit' or any other identical or deceptively
similar name or from passing off any such identical or deceptively similar trade mark or
trade name.
7. Notice was issued to defendant. He filed his written statement on November 10, 2005.
The defendant also filed an application under Order VII, Rules 10 and 11 of the Code
praying for rejection/return of plaint for presentation to proper Court. It was contended by
the defendant that the plaint disclosed no cause of action and was liable to be rejected. It
was further stated that no requisite Court fee had been paid within the time granted by the
Court and on that ground also, the plaint deserved to be rejected. It was also asserted that
there was an agreement between the plaintiff-Company and the defendant by which
@page-SC687
exclusive jurisdiction was granted to Courts in Hong Kong and jurisdiction of all other
Courts had been ousted and on that ground also Delhi Court had no jurisdiction in the
matter.
8. The High Court, as observed earlier, considered the application of the defendant and by
the impugned order, dismissed it holding that the agreement did not take away
jurisdiction of the Court as contended by the defendant and the application had been filed
only with a view to delay the progress of the suit which was liable to be dismissed and it
was accordingly dismissed with costs of Rs.4,000/-.
9. The appellant-defendant has challenged the said order by filing the present appeal. On
August 7, 2006, notice was issued by this Court and in the meantime further proceedings
in the suit were stayed.
10. We have heard learned counsel for the parties.
11. The learned counsel for the appellant contended that the High Court was wholly
wrong in holding that Courts in India could entertain a Civil Suit and the application filed
by the defendant-appellant was liable to be rejected. According to the learned counsel, the
agreement entered into between the parties made it expressly clear that the law
applicable, in case of dispute between the parties would be law of Hong Kong Special
Administrative Region and, hence, Indian Courts have no jurisdiction to entertain, deal
with and decide such question. It was also submitted by the learned counsel that in the
light of the agreement between the parties, only remedy available to the plaintiff-
Company was to take appropriate proceedings in accordance with law in a competent
Court in Hong Kong and no Indian Court could have jurisdiction inasmuch as jurisdiction
of all Courts in India is barred by necessary implication. The counsel also contended that
the High Court committed an error in holding that Delhi High Court had jurisdiction as
the defendant was residing in Delhi. In the plaint itself, the plaintiff gave the address of
the defendant of Ghaziabad which is not in Delhi but in Uttar Pradesh (U.P.). The counsel
also made grievance that the High Court was not right in observing that the defendant
wanted to delay the proceedings and was not justified in imposing costs of Rs.4,000/-. It
was, therefore, submitted that the appeal deserves to be allowed by setting aside the order
passed by the High Court.
12. The learned counsel for the respondent-Company, on the other hand, supported the
order passed by the High Court. He submitted that the only thing which was relevant in
the agreement was as to applicability of laws. As per settled legal position, a suit could be
instituted in Delhi as part of cause of action had arisen within the territorial jurisdiction of
that Court. The High Court was right in observing that applicability of law had nothing to
do with situs of a suit and since the defendant had used the trade mark/trade name of the
plaintiff in Delhi in Trade Fair, it was open to the plaintiff Company to institute a suit in
Delhi. It was submitted that it is really surprising that though Hong Kong based Company
institutes a suit in Delhi where the defendant had used the trade mark/trade name, where
he resides and thus it is much more convenient to him to defend the suit, yet he objects to
the proceedings which clearly goes to show that the only intention on the part of the
defendant is to delay the proceedings. The High Court was, therefore, right in dismissing
the application and in ordering payment of costs by him. It was, therefore, submitted that
the appeal deserves to be dismissed.
13. Having heard the learned counsel for the parties, in our opinion, no case has been
made out by the appellant from which it can be said that Delhi Court had no jurisdiction.
Both the learned counsel referred to the agreement dated September 13, 2004 entered into
between the parties. Clause 10 of the agreement relates to "Resignation and Termination
of Service". In accordance with the said clause, the appellant herein left the plaintiff-
Company. Clause 10 stipulates that in the event of resignation or termination for any
reason, the employee would not engage himself in a similar or competitive business for a
period of two years, nor he would contact or solicit any customer or supplier with whom
the employer conducted business during the employment. Clause 14 provides for
'Conflict of Interest'. Clause 15 deals with 'Confidentiality'. It recites that upon accepting
employment with Prodigy, the defendant would maintain confidentiality which would
mean that he would not disclose any 'Prodigy confidential information' either during or
after his employment to anyone outside the Company, nor would use it for personal
benefit. Clause 18 is material
@page-SC688
for the purpose of controversy and may be reproduced - :
18. The terms and conditions as stipulated above shall be interpreted in accordance to the
laws of the Hong Kong Special Administrative Region.
(Emphasis supplied)
14. It is this Clause (Clause 18), which requires interpretation. According to the appellant,
since the terms and conditions in the agreement have to be interpreted in accordance with
the laws of Hong Kong, no Court in any country other than a Court in Hong Kong shall
have jurisdiction to entertain a suit, petition, application or any other proceeding. The
submission of the respondent-Company, on the other hand, is that what is agreed upon is
not territorial jurisdiction of a Court but applicability of laws. Clause 18 deals with the
second eventuality and declares that terms and conditions of the agreement would be
interpreted in accordance with the laws of Hong Kong.
15. We find considerable force in the submission of the learned counsel for the
respondent Company. In our view, 'cause of action' and 'applicability of law' are two
distinct, different and independent things and one cannot be confused with the other. The
expression 'cause of action' has not been defined in the Code. It is however settled law
that every suit presupposes the existence of a cause of action. If there is no cause of
action, the plaint has to be rejected [Rule 11(a) of Order VII). Stated simply, 'cause of
action' means a right to sue. It consists of material facts which are imperative for the
plaintiff to allege and prove to succeed in the suit. The classic definition of the expression
('cause of action') is found in the observations of Lord Brett in Cooke v. Gill, (1873) 8 CP
107 : 42 LJ CP 98.
16. His Lordship stated;
"Cause of action means every fact which it would be necessary for the plaintiff to prove,
if traversed, in order to support his right to the judgment of the court".
17
. In A.B.C. Laminart Pvt. Ltd. v. A.P. Agencies (1989) 2 SCC 163, this Court said; AIR
1989 SC 1239, (Para 27)

"A cause of action means every fact, which, if traversed, it would be necessary for the
plaintiff to prove in order to support his right to a judgment of the Court. In other words,
it is a bundle of facts which taken with the law applicable to them gives the plaintiff a
right to relief against the defendant. It must include some act done by the defendant since
in the absence of such an act no cause of action can possibly accrue. It is not limited to
the actual infringement of the right sued on but includes all the material facts on which it
is founded. It does not comprise evidence necessary to prove such facts, but every fact
necessary for the plaintiff to prove to enable him to obtain a decree. Everything which if
not proved would give the defendant a right to immediate judgment must be part of the
cause of action. But it has no relation whatever to the defence which may be set up by the
defendant nor does it depend upon the character of the relief prayed for by the plaintiff".
(Emphasis supplied)
18. Now, Sections 16 to 20 of the Code deal with territorial jurisdiction of a Court (place
of suing). Whereas Sections 16 to 18 relate to immovable property, suits for
compensation for wrongs to persons or movables have been dealt with under Section 19.
Section 20 of the Code is a residuary provision and covers all cases not falling under
Sections 16 to 19.
19. The relevant part of Section 20 reads thus;
20. Other suits to be instituted where defendants reside or cause of action arises.- Subject
to the limitations aforesaid, every suit shall be instituted in a Court within the local limits
of whose jurisdiction-
(a) the defendant, or each of the defendants where there are more than one, at the time of
the commencement of the suit, actually and voluntarily resides, or carries on business, or
personally works for gain; or
(b) any of the defendants, where there are more than one, at the time of the
commencement of the suit, actually and voluntarily resides, or carries on business, or
personally works for gain, provided that in such case either the leave of the Court is
given, or the defendants who do not reside, or carry on business, or personally work for
gain, as aforesaid, acquiesce in such institution; or
(c) the cause of action, wholly or in part, arises.
(Emphasis supplied)
20. Bare reading of Clause (c) leaves no room for doubt that a suit would lie in a court
within the local limits of whose jurisdiction the cause of action has arisen, wholly or
partly.
@page-SC689
21. Section 20 has been designed to secure that justice might be brought as near as
possible to every man's hearthstone and that the defendant should not be put to the
trouble and expense of travelling long distances in order to defend himself.
22. According to the plaintiff-Company, a suit instituted on the Original Side of the High
Court of Delhi is maintainable since a part of cause of action had accrued within the
territorial jurisdiction of Delhi Court (breach of agreement by defendant). The argument
of the defendant that the agreement was executed in Hong Kong and hence suit could
have been instituted only in that country is, in our opinion, not well founded. It is no
doubt true that the suit could have been instituted in Hong Kong as well. That, however,
does not take away the jurisdiction of Delhi Court where a part of cause of action has
arisen. In the plaint, it was specifically alleged by the plaintiff Company that the
defendant committed breach of terms and conditions of agreement during the Trade Fair
in February, 2005 held in Pragati Maidan, Delhi. It was, therefore, open to the plaintiff
Company to institute a suit in a competent Court within the jurisdiction of Delhi and that
is how the suit is filed in the High Court on its Original Side. In our considered opinion,
therefore, the contention of the appellant-defendant that the agreement was executed in a
foreign country or the defendant was a resident of Ghaziabad (Uttar Pradesh) cannot take
away, exclude or oust the jurisdiction of Delhi Court in view of the averment made in the
plaint that a part of cause of action had arisen within the local limits of Delhi.
23. It was submitted by the learned counsel for the appellant that once there is an
agreement as to choice of Court or forum, the parties are bound by it. For the said
proposition, our attention has been invited to several decisions rendered by this Court. We
do not intend to burden our judgment on that point as the law is well settled and the
learned counsel for the respondent-Company has not disputed the proposition. What was
contended was that Clause 18 does not take away the jurisdiction of a competent Court
and the agreement did not exclude territorial jurisdiction of any Court.
24. Learned counsel for the appellant relied on a decision of this Court in British India
Steam Navigation Co. Ltd. v. Shanmughavilas Cashew Industries and Ors., (1990) 3 SCC
481. In that case, the plaintiff purchased from the defendant-Company raw cashew nuts
which were shipped in a vessel chartered by the Company incorporated in England.
Clause 3 of the Bill of Lading dealt with jurisdiction of the Court. The said clause read as
under - :
3. JURISDICTION - : The contract evidenced by this bill of lading shall be governed by
English law and disputes determined in England or, at the option of the Carrier, at the
port of destination according to English law to the exclusion of the jurisdiction of the
Courts of any other country.
25. Though the above clause made it clear that the dispute should be determined in
England, this Court held that the objection as to territorial jurisdiction had been waived
by the defendant. So far as the law is concerned, it was held that proper law to govern the
contract was English law.
26. The learned counsel for the appellant submitted that the ratio laid down in British
India Steam Navigation Co. applies to the case on hand and the High Court of Delhi
committed an error of law in not upholding the objection of the defendant that Indian
Court had no jurisdiction to deal with the matter.
27. We are unable to agree. Clause 3, as extracted hereinabove, clearly provided that the
contract would be governed by English law. The High Court was, therefore, right in
observing that the case is not relevant so far as the question raised in the present matter.
28

. The counsel also referred to National Thermal Power Corporation v. Singer Company
and Ors., (1992) 3 SCC 551. The parties in that case by an agreement had chosen the
jurisdiction of one Court to the exclusion of the other. Likewise, they also agreed as to the
applicability of law. In the light of the fact situation, the Court held that the parties are
bound by such Agreement and it has to approach a Court in consonance with the
agreement. This judgment also does not help the appellant in the instant case. 1993
AIR SCW 131

29. Our attention was also invited to Technip S.A. v. S.M.S. Holding (P) Ltd. and Ors.,
(2005) 5 SCC 465. Even that case also does not help the appellant. What was held there
was as to the law applicable to the dispute and not the territorial jurisdiction of the Court.
On the contrary, para 23 of the
@page-SC690
said decision goes to show that territorial jurisdiction of the Court and applicability of
law are two different things and even if a matter is decided in the country other than the
country where the agreement has been executed, the law which would apply would be the
law agreed by the parties.
30. The Court stated;
"23. The relationship of Technip to Coflexip whether one of control or not is really a
question of their status. The applicable law would therefore be the law of their domicil,
namely, French law. Having determined their status according to French Law, the next
question as to their obligation under the Indian Law vis-a-vis SEAMEC would have to be
governed exclusively by Indian law (in this case the Act and the Regulations). SATs error
lay in not differentiating between the two issues of status and the obligation by reason of
the status and in seeking to cover both under a single system of law". (Emphasis
supplied)
31. In the case on hand, we have referred to the relevant clauses of the agreement. Clause
18 provides for applicability of law and it specifically declares that the terms and
conditions of the agreement shall be interpreted in accordance with 'the laws of Hong
Kong Special Administrative Region'. That, in our judgment, does not mean that a suit
can be instituted only in Hong Kong and not in any other country. Territorial jurisdiction
of a Court, when the plaintiff intends to invoke jurisdiction of any Court in India, has to
be ascertained on the basis of the principles laid down in the Code of Civil Procedure.
Since a part of 'cause of action' has arisen within the local limits of Delhi as averred in
the plaint by the plaintiff Company, the question has to be considered on the basis of such
averment. Since it is alleged that the appellant-defendant had committed breach of
agreement by using trade mark/trade name in Trade Fair, 2005 in Delhi, a part of cause of
action has arisen in Delhi. The plaintiff-Company, in the circumstances, could have filed
a suit in Delhi. So far as applicability of law is concerned, obviously as and when the suit
will come up for hearing, the Court will interpret the clause and take an appropriate
decision in accordance with law. It has, however, nothing to do with the local limits of the
jurisdiction of the Court. The High Court, in our opinion, was right in rejecting the
application and in overruling preliminary objection. Since prima facie the plaint disclosed
a cause of action as also territorial jurisdiction of the Court, the High Court rightly
rejected both the contentions and no error was committed by it in not rejecting plaint, nor
returning it for presentation to proper Court. 'Applicability of Hong Kong Law', 'entering
into an agreement in Hong Kong' or 'defendant residing in Ghaziabad (Uttar Pradesh)' or
any of them does not take away the jurisdiction of Delhi Court since a 'cause of action' at
least in part, can be said to have arisen in Delhi. We, therefore, see no substance in the
contention of the defendant-appellant.
32. So far as imposition of costs is concerned, normally it is in the discretion of the Court.
When the Court, in the light of the facts before it, satisfied that the defendant wanted to
delay the proceedings and ordered him to pay costs of Rs.4,000/-, it would not be
appropriate to interfere with that part of the order.
33. For the foregoing reasons, the appeal deserves to be dismissed and is accordingly
dismissed. However, in the facts and circumstance of the case, there shall be no order as
to costs.
Appeal dismissed.
AIR 2008 SUPREME COURT 690 "State of Rajasthan v. Ganeshi Lal"
(From : Rajasthan)*
Coram : 2 Dr. A. PASAYAT AND P. SATHASIVAM, JJ.
Civil Appeal No. 3021 of 2006, D/- 10 -12 -2007.
State of Rajasthan v. Ganeshi Lal.
(A) Constitution of India, Art.141 - PRECEDENT - Precedent - Decision is precedent on
its own facts - What is binding in a decision is the principle on which it is decided -
Reliance on a decision without looking into factual background of case before it is clearly
impermissible - Circumstantial flexibility, one additional or different fact may make a
world of difference between conclusions in two cases. (Paras 11, 14)
(B) Industrial Disputes Act (14 of 1947), S.2(s) - INDUSTRIAL DISPUTE - DAILY-
WAGE WORKERS - Workman - Daily wager, peon attached to Public Prosecutor - Not
@page-SC691
workman - As Law Department cannot by any stretch of imagination be considered as
industry.
D. B. C. Spl. A. No. 391 of 2004, D/- 21-5-2004, Reversed. (Paras 16, 17)
Cases Referred : Chronological Paras
1996 AIR SCW 4020 11
(1972) 2 WLR 537 13
(1971) 1 WLR 1062 13
(1970) 2 All ER 294 13
AIR 1968 SC 647 (Rel. on) 11
(1951) AC 737 12
(1901) AC 495 (HL) 11
Manish Kumar, Ansar Ahmad Chaudhary, for Appellant; Nikilesh Ramachandran, for
Respondent.
* DB Civil Special Appeal No. 391 of 2004, D/- 21-5-2004 (Raj.).
Judgement
1. Dr. ARIJIT PASAYAT, J. :- Challenge in this appeal is to the order passed by a
Division Bench of the Rajasthan High Court, Jodhpur, upholding the view taken by the
learned Single Judge. Before the High Court challenge was to the award of the Labour
Court, Bikaner.
2. Background facts are almost undisputed and are as follows :
Respondent was working as a peon attached to the Public Prosecutor. He was getting an
amount of Rs.1,000/- p.m. as a temporary employee on a contract basis. He was engaged
under the Joint Legal Remembrancer and Director, Litigation, Law Department, Jaipur.
His services were terminated by notice dated 5.12.1998 w.e.f. 7.12.1998, and according
to him, it was in violation of the provisions of Section 25-G of the Industrial Disputes
Act, 1947 (in short the 'Act'). Therefore, a dispute was raised. A reference was made to
the Labour Court, vide Notification No. F 1(1)(1145) L.F./2000 dated 31st July, 2000,
under Section 10 of the Act. The reference was of the following dispute :
"Whether the termination from service on 7.12.1998 of the applicant Shri Ganeshilal son
of Shri Noratmal Barber by the non-applicant (1) Additional Public Prosecutor, Rajgarh
District Churu (2) Joint Law Adviser and Director Litigation, Law Department, Rajasthan
Churu is proper and valid? If not then to what relief the applicant is entitled for?"
3. The claim was resisted by the present appellant on the ground that the Law department
is not an industry.
4. On a reference to the Labour Court the Presiding Officer, Labour Court, held that Law
department was an industry in view of what has been stated by this Court in relation to
various departments, hotel, school, public works department, irrigation department. This
view has been accepted by learned Single Judge who held that there was no scope for
interference under Article 226 of the Constitution of India, 1950 (in short 'the
Constitution').
5. The Division Bench after referring to Section 2(s) of the Act held that the view of the
Labour Court was correct.
6. Learned counsel for the appellant submitted that by no stretch of imagination the Law
department can be considered to be an industry. Learned counsel for the respondent on
the other hand submitted that the Labour Court and the High Court were justified in their
views.
7. Section 2(s) of the Act defines "workman" as follows :
"any person (including an apprentice) employed in any industry to do any manual,
unskilled, skilled, technical, operational, clerical or supervisory work for hire or reward,
whether the terms of employment be express or implied, and for the purposes of any
proceeding under this Act in relation to an Industrial Dispute, includes any such person
who has been dismissed, discharged or retrenched in connection with, or as a
consequence of, that dispute, or whose dismissal, discharge or retrenchment has led to
that dispute."
8. For bringing in application of Section 2(s) of the Act, the workman must be employed
in an industry. The Law department can, by no stretch of imagination, be considered as an
industry.
9. Learned counsel for the appellant submitted that whether any Government department
can be treated as industry is under consideration of a larger Bench of this Court.
10. The Labour Court and the High Court have not even indicated as to how the Law
department is an industry. Merely stating that in some cases Irrigation Department, Public
Works Department have been held to be covered by the expression "industry" in some
decisions.
11

. Reliance on the decision without looking into the factual background of the case before
it is clearly impermissible. A decision is a precedent on its own facts. Each case 1996
AIR SCW 4020

@page-SC692
presents its own features. It is not everything said by a Judge while giving a judgment
that constitutes a precedent. The only thing in a Judge's decision binding a party is the
principle upon which the case is decided and for this reason it is important to analyse a
decision and isolate from it the ratio decidendi. According to the well-settled theory of
precedents, every decision contains three basic postulates (i) findings of material facts,
direct and inferential. An inferential finding of facts is the inference which the Judge
draws from the direct, or perceptible facts; (ii) statements of the principles of law
applicable to the legal problems disclosed by the facts; and (iii) judgment based on the
combined effect of the above. A decision is an authority for what it actually decides.
What is of the essence in a decision is its ratio and not every observation found therein
nor what logically flows from the various observations made in the judgment. The
enunciation of the reason or principle on which a question before a Court has been
decided is alone binding as a precedent. (See - : State of Orissa v. Sudhansu Sekhar Misra
and Ors. (AIR 1968 SC 647) and Union of India and Ors. v. Dhanwanti Devi and Ors.
(1996 (6) SCC 44)). A case is a precedent and binding for what it explicitly decides and
no more. The words used by Judges in their judgments are not to be read as if they are
words in Act of Parliament. In Quinn v. Leathem (1901) AC 495 (HL), Earl of Halsbury
LC observed that every judgment must be read as applicable to the particular facts proved
or assumed to be proved, since the generality of the expressions which are found there are
not intended to be exposition of the whole law but governed and qualified by the
particular facts of the case in which such expressions are found and a case is only an
authority for what it actually decides.
12. Courts should not place reliance on decisions without discussing as to how the factual
situation fits in with the fact situation of the decision on which reliance is placed.
Observations of Courts are neither to be read as Euclid's theorems nor as provisions of
the statute and that too taken out of their context. These observations must be read in the
context in which they appear to have been stated. Judgments of Courts are not to be
construed as statutes. To interpret words, phrases and provisions of a statute, it may
become necessary for Judges to embark into lengthy discussions but the discussion is
meant to explain and not to define. Judges interpret statutes, they do not interpret
judgments. They interpret words of statutes; their words are not to be interpreted as
statutes. In London Graving Dock Co. Ltd. v. Horton (1951 AC 737 at p.761), Lord Mac
Dermot observed:
"The matter cannot, of course, be settled merely by treating the ipsissima vertra of Willes,
J as though they were part of an Act of Parliament and applying the rules of interpretation
appropriate thereto. This is not to detract from the great weight to be given to the
language actually used by that most distinguished Judge."
13. In Home Office v. Dorset Yacht Co. (1970 (2) All ER 294) Lord Reid said, "Lord
Atkin's speech.....is not to be treated as if it was a statute definition. It will require
qualification in new circumstances." Megarry, J in (1971) 1 WLR 1062 observed: "One
must not, of course, construe even a reserved judgment of Russell L.J. as if it were an Act
of Parliament." And, in Herrington v. British Railways Board (1972 (2) WLR 537) Lord
Morris said :
"There is always peril in treating the words of a speech or judgment as though they are
words in a legislative enactment, and it is to be remembered that judicial utterances made
in the setting of the facts of a particular case."
14. Circumstantial flexibility, one additional or different fact may make a world of
difference between conclusions in two cases. Disposal of cases by blindly placing
reliance on a decision is not proper.
15. The following words of Lord Denning in the matter of applying precedents have
become locus classicus :
"Each case depends on its own facts and a close similarity between one case and another
is not enough because even a single significant detail may alter the entire aspect, in
deciding such cases, one should avoid the temptation to decide cases (as said by
Cordozo) by matching the colour of one case against the colour of another. To decide
therefore, on which side of the line a case falls, the broad resemblance to another case is
not at all decisive."
*** *** ***
"Precedent should be followed only so far as it marks the path of justice, but you must cut
the dead wood and trim off the side
@page-SC693
branches else you will find yourself lost in thickets and branches. My plea is to keep the
path to justice clear of obstructions which could impede it."
16. As noted above, the accepted concept of an industry cannot be applied to the Law
department of the Government.
17. That being so, the view expressed by the Labour Court and the High Court is
indefensible. However, it appears that the respondent has been reinstated to the post he
was holding at the time of termination. In view of this fact, even though we have held
that the orders passed are clearly unsustainable. We leave it to the appellant to consider
whether the respondent can be continued, in view of the fact that he worked for some
years.
18. The appeal is allowed to the aforesaid extent without any order as to costs.
Order accordingly.
AIR 2008 SUPREME COURT 693 "U. P. Power Corpn. Ltd v. Sant Steels and Alloys (P)
Ltd."
(From : Allahabad)*
Coram : 2 A. K. MATHUR AND MARKANDEY KATJU, JJ.
Civil Appeal Nos. 1215-1216 of 2001, D/- 10 -12 -2007
U. P. Power Corpn. Ltd. and Anr. v. Sant Steels and Alloys (P) Ltd. and Ors.
Electricity (Supply) Act (54 of 1948), S.49 - ELECTRICITY - PROMISSORY
ESTOPPEL - PRINCIPLES - LEGISLATURE - Tariff - Development rebate - Grant of,
to new industrial Units in hill areas - Passing of notification by delegated authorities in
exercise of delegated authority - Revocation of - Has no flavour of statute - Not shown to
be issued in public interest - Principle of promissory estoppel would be attracted - Simply
because there was theft of energy - Cannot be ground to claim that revocation was in
public interest - Said benefits however, not being recognised by 1999 Act - Would not be
available to claimants from date of enactment of 1999 Act.
Constitution of India, Art.245.
Evidence Act (1 of 1872), S.115.
U.P. Electricity Reforms Regulation Act (24 of 1999), S.1.
Notification issued under S. 499 for giving the benefit of exemption for the new
industrial units in the hill areas was in the nature of delegated legislation and not an Act
framed by the State Legislature. Therefore, a distinction has to be made between the
delegated legislation and the primary legislation. So far as the primary legislation is
concerned, if the Act is passed by State Legislature and denies the benefit by the primary
legislation then no estoppel can be applied against that Act but so far as the case of
delegated legislation is concerned, where delegated authorities passes certain notification
in exercise of his delegated authority there is no contemplation mentioned in the Act itself
that it is capable of being revoked at any time. Then such acts cannot be treated at par
with the primary Act passed by the State Legislatutre. The State is fully competent to pass
an Act prospectively as well as retrospectively but restrospectivity to the extent of
aforesaid nature cannot stand. Therefore, this distinction has to be borne in mind. In the
instant case, the U. P. Electricity Reforms Act, 1999 came into force with effect from
2000. Therefore, if such benefit has not been extended then a different stand will follow
but so far as the delegated legislation is concerned, this kind of revocation cannot be
sustained. It is highly against the public morality that the incumbent who have felt
persuaded on account of the representation made by the State Government that they will
be given certain benefits and they acted on that representation, it does not behove on the
part of the appellant - Corporation to withdraw the said benefit before expiry of the
stipulated period by issuing the notification revoking the same which the respondents
were legitimately entitled to avail. The appellant Corporation which made a
representation and allowed the other party to act upon such representation could not resile
and leave the citizens in a lurch. In such a situation the principle of promissory estoppel
which has been evolved by the Courts which is based on public morality cannot permit
the State to act in such an arbitrary fashion. (Para 18)
This is not case in which serious public repercussion was involved. Simply because there
was theft of energy cannot be ground to hold that the revocation of such concession can
be said to be in public interest. Since the benefit was given to these units in the hill areas,
there should have been
@page-SC694
overwhelming evidence to show some mala fide on the part of these consumers which
have persuaded the Corporation to revoke it. If there was no misuse of the energy by
these units in the hill areas to whom the concession had been granted then in that case it
cannot be taken that there was really public interest involved which persuaded the
Corporation to revoke the same. No person can be permitted to misuse the concession or
benefit and invoke promissory estoppel. Promissory estoppel is not one sided affair, it is
rather two sided affair. If one party abuses the concession then it is always open to the
other party to revoke such concession but if one party avails the benefit and is acting on
the same representation made by the other party then the other party who has granted the
said benefit cannot revoke the same under the garb of public interest. (Para 18))
Whenever the State has made a representation to the public and the public has acted on
that representation and suffered economically or otherwise, then in that case the State
should be estopped from withdrawing such benefit to the detriment of the such people
except in public interest or against the Statute. So far as the public interest as involved in
the present case is concerned, there was no overwhelming evidence to revoke the benefit
granted to the industrial units in the hill areas. So far as the Statute is concerned, the
notification was issued under S. 49 and the same was revoked under S. 49 though there
was no such provision contained in S. 49 that it will be open to the Corporation to revoke
the same but could be possible by invoking the principle of General Clauses Act. But in
such delegated legislation such withdrawal could only be permitted if larger public
interest is involved or if the Act is passed by legislature. (Para 18)
Since such benefits have not been recognised by the Act of 1999, therefore, upto the date
of coming into force of the Act of 1999, all the benefits which were being given to the
respondent-entrepreneurs shall be protected by invoking the principle of promissory
estoppel but after coming into force of the Act of 1999, which is a primary legislation
enacted by the State Legislature the benefits from the date of the Act has come into force,
cannot be made available to the respondents.(Paras 19, 21)
Cases Referred : Chronological Paras
2006 AIR SCW 1500 (Ref) 16
2006 AIR SCW 5272 (Ref) 13
2004 AIR SCW 3703 : AIR 2004 SC 4559 (Ref) 15
1999 AIR SCW 2369 : AIR 1999 SC 2302 (Ref) 12
1998 AIR SCW 1866 : AIR 1998 SC 591 (Ref) 11, 14, 15, 18
1997 AIR SCW 3839 : AIR 1997 SC 3910 : 1997 All LJ 2202 (Rel. on) 2, 5, 7, 8, 14
(1997) 3 SCC 398 (Ref) 10, 14
1995 AIR SCW 680 : AIR 1995 SC 874 (Ref) 9, 10, 14
AIR 1979 SC 621 : 1979 All LJ 368 13
AIR 1974 SC 555 : 1974 Lab IC 427 (Ref) 13
AIR 1968 SC 718 13
Dr. A. M. Singhvi and Ratnakar Dash, Sr. Advocates, Pradeep Misra, Amit Bhandari,
Daleep Dhayani, Dinesh Kumar Garg and Anuvarat Sharma, for Appellants; Shanti
Bhushan, R. F. Nariman, S. Ganesh, M. L. Bhat, Sr. Advocates, Sudhir Kumar Gupta,
Anurag Pandey, Mihir Kumar Chaudhary, M. L. Lahoty, Anurag Pandey, Paban K.
Sharma, Ms. Poonam Lahoty, Ramesh Singh, Rajeev Sharma, R. Santa-nam, Ms.
Manjula Gupta, Irshad Ahmad, R. C. Verma and Pradeep Misra, for Respondents.
* W.P. Nos. 15292 and 15293 of 1999, D/- 25-5-2000 (All.)
Judgement
A. K. MATHUR, J. :- These appeals are directed against the order dated 25.5.2000 passed
by the Division Bench of the Allahabad High Court whereby the Division Bench has
allowed the writ petitions and Clause 9(a) of the notification dated 25.1.1999 (Annexure-
8 to the writ petition) and clause 8(a) of the notification dated 18.6.1998 (Annexure -7 to
the writ petition ) were struck down. It was further directed that the writ petitioners were
entitled to get hill development rebate of 33.33% on the total amount of the bill till the
period of 5 years from the date of commencement of supply of the electricity to them and
the appellant-Corporation was directed to issue electricity bills to the writ petitioners after
allowing 33.33% hill development rebate on the total amount of bill for the remaining
unexpired period of five years. Aggrieved against this order, the present appeals were
filed by U.P. Power Corporation Ltd.(hereinafter referred to as Corporation.)
@page-SC695
2

. In order to dispose of these appeals brief facts may be detailed below. Pursuant to
industrial policy of the State of Uttar Pradesh, U.P.State Electricity Board (now U.P.
Power Corporation Limited)[hereinafter to be referred to as the "Corporation"]- the
appellant herein framed its tariffs vide notifications dated 18.1.1992 and 15.7.1994. By
these notifications 33.33% hill development rebate was allowed to the new industrial
units for a period of five years from the date of commencement of the supply of the
electricity. The above concession was initially valid till 31.3.1995. It was later on
extended up to 31.3.1997. It was alleged that all the writ petitioners established industrial
units in the hill areas after huge investments and after executing agreement with the
appellant-Corporation. But subsequently, by notifications dated 18.6.1998 and 25.1.1999
the concession which was earlier given was reduced by the appellant-Corporation from
33.33% to 17% which is arbitrary and not permissible according to principle of
promissory estoppel and in that connection reliance was placed on a decision of this
Court in Pawan Alloys and Casting Pvt. Ltd., Meerut v. U.P.State Electricity Board and
Ors. (1997) 7 SCC 251. Written statement was filed by the appellant-Corporation and the
appellant took the stand that the impugned tariffs were new structured tariff in respect of
HV-1 category of consumers and it was empowered to frame tariff under the provisions
of Section 49 of the Electricity (Supply) Act, 1948 (hereinafter to be referred to as the Act
of 1948). It was also contended that this restructuring was necessitated in order to avoid
loss to the Corporation due to theft of electricity and it was done in the public inter est.
1997 AIR SCW 3839

3. In order to appreciate the controversy involved in the matter, it will be appropriate to


refer to the relevant tariff notification issued from time to time by the appellant-
Corporation. The first in point of time is the tariff vide notification dated 18.1.1992.
Relevant provisions of clauses read as under :
"4. Rate of Charge ( Energy Charges) :
All KWH consumed in 3 month 280 paise per
KWH.
5. Extra Charge or Rebate :
(i) In case of supply given at 400 volts, the consumer shall be required to pay an extra
charge of 10 per cent on the amount calculated at the rate of charge under item (4).
(ii) If supply is given at voltage more than 11KV, rebate mentioned below will be
admissible on the amount calculated at the rate of charge under item (4).
(a) Above 11 KV upto 66 KV 5%
(b) Above 66 KV upto 132 KV 7.5% )
(c) Above 132 KV 10%.
xx xx xx
xx xx xx
8. Concessions :
In respect of connections as may be located in any of the eight hill districts in U.P. whose
names are given below but excluding those existing at a height of less than 610 mts
(2,000feet) above M.S.L. in Dehradun and National districts a development rebate of 33
1/3% on the amount of the bill as computed under items 4 and 5 above will be given to
new connections for a period of five years from the date of commencement of supply.
This rebate will also be admissible for the unexpired period of five years to those existing
connections which have not completed five years from the date of commencement of
supply. This development rebate shall not be admissible to the
Departments/Corporations/Undertaking of State/Central Government and Local Bodies."
Name of eight Hill Districts :
1. Almora district
2. Chamoli district
3. Pauri Garhwal district
4. Pithoragarh district
5. Uttar Pradesh district
6. Tehri Garhwal district
7. Uttarkashi district
8. Dehradun district.
In respect of connections as may be located in Bundelkhand region, comprising Jhansi,
Lalitpur, Hamipur, Jalaun and Banda districts a development rebate of 50% on the
amount of the bill as computed under items 4 and 5 above will be given to new Industrial
units for a period of five years from the date of commencement of supply. This rebate
will also be admissible for the unexpired period of five years to those existing Industrial
units of the above district of Bundelkhand region who have not completed five years
from the date of commencement of supply. This
@page-SC696
development rebate shall however not be allowed to the
Department/Corporations/Undertakings of the State/ Central Government and Local
Bodies. " Therefore, this concession was extended to the entrepreneurs in the hill districts
including Dehradun who established their industries at the height of 610 metres (2000
feet) above M.S.L.for a period of five years. Then on 15.7.1994 another notification was
issued. Relevant provisions of Clauses 4,5 and 8 read as under :

"4. Rate of Charge ( Energy Charges):


All KWH consumed in 3 month 280 paise per KWH.
5. Extra Charge or Rebate :

(iii) In case of supply given at 400 volts, the consumer shall be required to pay an extra
charge of 10 per cent on the amount calculated at the rate of charge under item (4).
(iv) If supply is given at voltage more than 11KV, rebates mentioned below will be
admissible on the amount calculated at the rate of charge under item (4).

(a) Above 11 KV upto 66 KV 5%


(b) Above 66 KV upto 132 KV 7.5% )
(c) Above 132 KV 10%.

8. Concessions:- (a) In respect of connections as may be located in under mentioned areas


of the hill districts in U.P., a development rebate of 33 1/3 per cent on the amount of the
bill as computed under items 4 and 5 above will be given to new connections for a period
of five years from the date of commencement of supply. This rebate will also be
admissible for the unexpired period of five years to those existing connections which
have not completed five years from the date of commencement of supply. Provided that
the above development rebate shall not be admissible to the
Departments/Corporations/Undertakings of State/Central Government and local bodies.
Description of Area of Hill Districts :
1. Almora district
2. Pithoragarh district
3. Chamoli district
4. Uttarkashi district
5. Pauri Garhwal district excluding Nagarpalika area of Kotdwara.
6. Tehri Garhwal district excluding Muni Ki Reti and Dhalwala Blocks.
7. Nainital district excluding Haldwani, Rudrapur, Gadarpur, Kashipur, Bajpur, Ram
Nagar, Jaspur, Khatima and Sitarganj Block.
8. Dehradun district excluding Doiwala, Rampur, Sahaspur and Vikas Nagar Blocks.
(b) In respect of connections as may be located in Bundelkhand region, comprising
Jhansi, Lalitpur, Hamipur, Jalaun and Banda districts a development rebate of 50% on the
amount of the bill as computed under items 4 and 5 above will be given to new Industrial
units for a period of five years from the date of commencement of supply. This rebate
will also be admissible for the unexpired period of five years to those existing Industrial
units of the above district of Bundelkhand region who have not completed five years
from the date of commencement of supply. This development rebate of 50% in
Bundelkhand region shall, however, not be allowed to the Railways and Departments/
Corporations/ Undertakings of the State/ Central Government and Local Bodies. The
development rebates under this clause shall be allowed subject to the condition that the
net amount payable after allowing these rebates would not be less than the amount of
minimum consumption guarantee under item 6 above."
Meaning thereby that the energy charges were increased from 200 paise to 280 paise and
the concession granted to the hill areas continued. Thereafter, in supersession of earlier
notifications another notification was issued in which energy charges were increased
from 280 paise to 308 paise per KW. But the concession granted earlier continued.
Relevant provision reads as under :

"4. Rate of Charge ( Energy Charges):


All KWH consumed in one month 308 paise per KWh.
5. Extra Charge or Rebate :

(i) In case of supply given at 400 volts, the consumer shall be required to pay an extra
charge of 10 per cent on the amount calculated at the rate of charge under item (4).
(ii) If supply is given at voltage more than 11KV, rebate mentioned below will be
admissible on the amount calculated at the rate of charge under item (4).

(a) Above 11 KV upto 66 KV 5%


(b) Above 66 KV upto 132 KV 7.5% )
(c) Above 132 KV 10%.

xx xx xx
8. Concessions :
The concessions mentioned hereunder
@page-SC697
shall be applicable to consumers connected upto 31.3.97.
(a) In respect of connections as may be located in undermentioned areas of the hill
districts in U.P., a development rebate of 33 1/3 % on the amount of the bill as computed
under items 4 and 5 above will be given to new connections for a period of five years
from the date of commencement of supply. This rebate will also be admissible for the
unexpired period of five years to those existing connections which have not completed
five years from the date of commencement of supply.
Provided that the above development rebate shall not be admissible to the
Departments/Corporations/Undertakings of State/Central Government and local bodies.
Description of Area of Hill Districts :
1. Almora district
2. Pithoragarh district
3. Chamoli district
4. Uttarkashi district
5. Pauri Garhwal district excluding Nagarpalika area of Kotdwara.
6. Tehri Garhwal district excluding Muni Ki Reti town area and Dhalwala village under
Narendra Nagar Block.
7. Nainital district excluding Haldwani, Rudrapur, Gadarpur, Kashipur, Bajpur, Ram
Nagar, Jaspur, Khatima and Sitarganj Blocks.
8. Dehradun district excluding Doiwala, Rampur, Sahaspur and Vikas Nagar Blocks.
(b) In respect of connections as may be located in Bundelkhand region, comprising
Jhansi, Lalitpur, Hamipur, Jalaun and Banda districts a development rebate of 50% on the
amount of the bill as computed under items 4 and 5 above will be given to new Industrial
units for a period of five years from the date of commencement of supply. This rebate
will also be admissible for the unexpired period of five years to those existing Industrial
units of the above districts of Bundelkhand region who have not completed five years
from the date of commencement of supply. This development rebate of 50% in
Bundelkhand region shall, however, not be allowed to the Departments/ Corporations/
Undertakings of the State/ Central Government and Local Bodies.
The development rebates under this clause shall be allowed subject to the condition that
the net amount payable after allowing these rebates would not be less than the amount of
minimum consumption guarantee under item 6 above."
Thereafter, on 18.6.1998 a new notification came to be issued, which is relevant for our
purpose. By this notification the bills were divided into two parts, i.e. demand charge plus
energy charge. Relevant provisions of Clauses 4,5 and 8 read as under:
" 4. RATE OF CHARGE :
(A) Demand Charge

1. Induction Furnaces Rs.700/- per KVA/ month


2. ARC FurnacesRs.615/- per KVA/ month
3. Rolling/ Re-rolling Mills Rs.440/- per KVA/ month
(B) Plus Energy Charge
All KWH consumed in the month 100 paise per month

Notes :
(i) Any consumer availing the supply for more than one process of Induction Furnace,
ARC furnace or Rolling/ Re-rolling Mill, will be charged at the applicable rate of demand
charge whichever is higher. '
(ii) The recording of demand and energy shall be done through static Trivector Meters.
5. EXTRA CHARGE OR REBATE :
(i) In case of supply given at 400 volts, the consumer shall be required to pay an extra
charge of 10 per cent on the amount calculated at the rate of charge under item (4).
(ii) If supply is given at voltage more than 11 KV, rebate mentioned below will be
admissible on the amount calculated at the rate of charge under item (4).

(a) Above 11 KV upto 66 KV 5%


(b) Above 66 KV upto 132 KV 7.5% )
(c) Above 132 KV 10%.

xx xx xx
8. CONCESSION :
The concessions mentioned hereunder shall be applicable to consumers connected upto
31.03.1997.
(a) In respect of connections as may be located in undermentioned area of hill
@page-SC698
districts in U.P. a development rebate of 17% on the demand charges only as computed
under item (4) above will be given during the unexpired period of five years to those
existing connections which have not completed five years from the date of
commencement of supply.
Provided that the above development rebate shall not be available to the Department/
Corporations/ Undertaking of State/ Central Government and Local Bodies.
DESCRIPTION OF AREA OF HILL DISTRICTS :
1. Almora district
2. Pithoragarh district
3. Chamoli district
4. Pauri Garhwal district excluding Nagarpalika area of Kotdwara.
5. Uttarkashi district
6. Tehri Garhwal district excluding Muni Ki Reti town area and Dhalwala village under
Narendra Nagar Block.
7. Nainital district excluding Haldwani, Rudrapur, Gadarpur, Kashipur, Bajpur, Ram
Nagar, Jaspur, Khatima and Sitarganj Blocks.
8. Dehradun district excluding Doiwala, Rampur, Sahaspur and Vikas Nagar Blocks.
(b) In respect of connections as may be located in Bundelkhand region, comprising
Jhansi, Lalitpur, Hamipur, Jalaun and Banda districts a development rebate of 25% on the
demand charges only as computed under item 4 above will be given during the unexpired
period of five years to those existing industrial units of the above districts of
Bundelkhand region who have not completed five years from the date of commencement
of supply. This development rebate shall however not be allowed to the
Departments/Corporations/Undertakings of the State/Central Government and Local
Bodies.."
Similar is the notification dated 25.1.1999 which is identical to the notification dated
18.6.1998. But in this notification dated 25.1.1999 the concession was not in clause 8 but
the concession has been re-numbered from clause 8 to clause 9 which is identical and as
such need not be reproduced again. As a result of these two notifications i.e. notifications
dated 18.6.1998 and 25.1.1999 two significant things happened, that the tariff was
divided into two parts i.e. demand charge plus energy charge. The energy charge was
charged earlier at 308 paise per KV was reduced to 100 paise KVA per month but the
demand charge i.e. induction furnace, ARC furnace, rolling/re-rolling mills etc. which
were fixed charges, concession was given at the rate of 17% computed under item
No.4(A) i.e. induction furnace @ Rs.700/- per KVA/ month, ARC furnace @ Rs.615/- per
KVA/month and Rolling/ Re-rolling Mills @ Rs.440/- per KVA/ month. Therefore, as a
result of restructuring of tariff, the demand charges under item 4(A) were made fixed but
the energy charges were reduced from 308 paise to 100 paise per month. It is not the case
that the appellant has completely revoked the concession. It is the case that appellant-
Corporation has reduced the energy charges from 308 paise per KVA to 100 paise but the
demand charges have been fixed per KVA/ month and the concession has been re-
scheduled instead of giving them 33.33% the energy charges have been reduced which is
applicable to all but in the case of demand charges for hill areas it has been reduced to 17
% in respect of demand A charges and that was allowed to be continued for the unexpired
period of five years to its existing connections which have not completed five years from
the date of commencement of supply. At the same time the appellant- Corporation has
denied this benefit to the State Departments/Corporations, Undertakings of the
State/Central Government and Local Bodies. Therefore, so far as the private consumers
are concerned, this has been kept intact.
4. Now, in this factual controversy, we have to examine whether the concession in the
consumption of energy which has been given to the writ petitioners for establishing the
industries in the hill areas can be revoked or modified by the appellant- Corporation or
not. The High Court has taken the view that the appellant is bound on the principle of
promissory estoppel and it cannot revoke the benefit.
5. Dr.A.M.Singhvi, learned senior counsel for the appellant has given nine reasons that
this modification of the rebate is fully justified for the following reasons :
(i) That the notifications have been issued in exercise of the statutory provisions under
Section 49 of the Act of 1948, therefore, it has statutory flavour.
(ii) That there is complete change of tariff i.e. it has two parts, (a) demand charge and
@page-SC699
(b) energy charge.
(iii) That there has been reduction in the energy consumption charges i.e. from 308 paise
to 100 paise per unit.
(iv) That there was large scale theft of energy in the State of U.P.
(v) That units were closing on account of these concessions.
(vi) That there is no total withdrawal of the rebate but by restructuring concession at the
rate of 17% continues in the demand charges.
(vii) That the High Court has failed to consider the public interest which was specifically
pleaded by filing a detailed affidavit.
(viii) That no mala fide is attributed.
(ix) That actual cost of energy production has shot up to Rs.2.50.
Therefore, learned senior counsel for the appellant submitted that the appellant-
Corporation is fully within its right to modify the rebate and the principle of promissory
estoppel cannot estop. Dr. Singhvi also submitted that the Division Bench of the High
Court has relied on a decision in Pawan Alloys and Casting Pvt. Ltd. (supra) in which no
affidavit was filed. This was not appreciated by the High Court and therefore, the whole
situation has turned on that count. Dr. Singhvi has also raised the question of laches,
estoppel, waiver and acquiesence and submitted that the earlier writ petition was filed
challenging the notification dated 18.6.1998 and it was withdrawn with liberty and
thereafter on 4.11.1999 application to recall the order was filed which was rejected.
Again, another writ petition has been filed without permission of the High Court. Dr.
Singhvi submitted that by virtue of the U.P. Electricity Reforms Act, 1999, (hereinafter to
be referred to as the Act of 1999) now the new tariff has been fixed from August, 2000-
2001 by the Commission because now the power to determine the tariff has been given to
the Commission and no estoppel against the Statute can be pleaded after the Act of 1999
having come into force. Dr. Singhvi, learned senior counsel submitted that in view of the
affidavit filed by Shri C.R.Goswami, Executive Engineer, Electricity Distribution
Division, Kotdwar, Uttarakhand on behalf of the appellant and a comparative chart has
been annexed to indicate that in fact after introduction of two part tariff, energy
consumption of these units has considerably increased. The chart has been filed along
with the affidavit in respect of all the writ petitioners except Shree Sidhbali Steels Ltd.
1997 AIR SCW 3839

6. As against this, Mr. Shanti Bhusan, learned senior counsel for the respondent- writ
petitioners submitted that these concessions were given to the hill areas in pursuance to
the direction by the State Government in exercise of power under Section 78A of the Act
of 1948 and submitted that the State Government was fully competent to do so. The
State/Corporation has made a representation on which the private entrepreneurs have
made huge investments and therefore, the State Government/ Corporation cannot wriggle
out from it and the State Government/Corp. is estopped from withdrawing these
concessions. Mr. S. Ganesh, learned senior counsel appearing for some of the writ
petitioners has also submitted that the concession which has been given has a vested right
and it can only be revoked by the same Statute.
7

. Both the learned senior counsel appearing for the parties relied on number of decisions
of this Court on the subject. Since the High Court has relied primarily on the decision of
this Court in Pawan Alloys and Casting Pvt. Ltd. (supra), therefore, it would be profitable
to first examine the said decision. In this case, the U.P. State Electricity Board by
notifications issued in exercise of powers under Section 49 of the Act of 1948 held out
promises to the industrial units established in different parts of the State of U.P. and they
were given concession in the electricity charges to the extent of 10 per cent of rebate for a
period of three years for the first time and the same was prematurely withdrawn by
subsequent notification which gave rise to number of writ petitions being filed in the
High Court and the principle of promissory estoppel was invoked. In the writ petitions it
was contended that when rebate was given to the new industrial units for a period of three
years, the Board could not have arbitrarily withdrawn the same prior to the expiry of a
period of three years. It was contended that such withdrawal of concession is applicable
prospectively and cannot have retrospective effect to the earlier existing industrial units.
The Board contested the matter. The Allahabad High Court framed the following three
questions. (i) Whether the Board is estopped from withdrawing the said rebate before the
completion of the 3/5 year period, by virtue of the 1997 AIR SCW 3839, (Paras 31 and
32)

@page-SC700
doctrine of promissory estoppel ? (ii) Whether the agreement executed by the petitioners
bars them from questioning the impugned notification ? (iii) Whether the impugned
notification has no application to existing consumers and does it apply to only those
consumers who receive the supply on or after 1-8-1986 ? The High Court after hearing
the contesting parties came to the conclusion that the respondent-Board was estopped by
virtue of the doctrine of promissory estoppel from withdrawing the development rebate
before the completion of the period of three years. On second point, the High Court came
to the conclusion that the writ petitioners were barred from questioning the impugned
notification on the express terminology found in the agreements entered into by them
with the Board for supply of electricity and under those agreements the Board was given
full play to revise the tariff rates which included development rebate also from time to
time and consequently the impugned notification was not illegal. On the third issue, it
was held that the notification dated 31-7-1986 could not be said to be retrospective and
consequently, the High Court dismissed all the writ petitions. Aggrieved against this, the
matter came up before this Court by Pawan Alloys and Casting Pvt. Ltd. This Court after
review of all the earlier decisions observed as follows - :
"34. Consequently it must be held that relying upon the representations held out by the
Board in these earlier notifications assuring grant of incentive rebate of 10% on the total
bill of electricity consumption charges these new industries being assured that for three
years this concession will be available had burnt their boats and spent large amounts and
had established their industries in the area falling in the operative jurisdiction of the
Board in the State of U.P.
35. Under these circumstances when no public interest was sought to be pressed into
service by the Board for withdrawal of this incentive rebate, as seen earlier, the equity
which had arisen in favour of the appellants remained untouched and undisturbed by any
overwhelming and superior equity in favour of the Board entitling it to withdraw this
development rebate in a premature manner leaving these promises high and dry before
the requisite period of three years earlier guaranteed to them by way of development
rebate had got exhausted. This takes us to the consideration of the second aspect of the
matter."
8. Dr. Singhvi, learned senior counsel for the appellant-Corporation emphasized that in
fact the whole case turned on the question that no public interest was sought to be pressed
into service by the Board on the incentive rebate. But, in the present case, specific
affidavit was filed and all the detailed facts were disclosed pertaining to the public
interest but that was not dealt with by the High Court. Therefore, Pawan Alloys and
Casting Pvt. Ltd. (supra) case stands distinguished. Learned senior counsel submitted that
if proper public interest had been pleaded in Pawan Alloys and Casting Pvt. Ltd.(supra)
then perhaps the situation would have been different. In this connection, learned senior
counsel for the appellant-Corporation invited our attention to the question of public
interest which was pleaded before the High Court and which was not considered by the
High Court. Learned senior counsel for the appellant- Corporation submitted that all the
nine points which have been mentioned above were mentioned in the counter affidavit
filed by the appellant-Corporation before the High Court and in that connection, he
invited our attention to paragraphs 5, 6, 7, 10, 40, 42, 44, 48 of the counter affidavit and
specifically invited our attention to paragraph 53 that the Corporation is incurring a loss
of Rs.15 to 20 crores. Learned senior counsel also invited our attention to paragraphs 56,
58 and 60 of the counter affidavit filed before the High Court and submitted that it was
not in public interest to continue this benefit to these industries located in hill areas and
further submitted that the entire benefit was not withdrawn. This benefit has been
rationalized and as a result of this rationalization an affidavit was filed to show that the
energy consumption of these units has increased to manifold. Therefore, this restructuring
of the rebate has not proved disadvantageous to these industries but for the larger public
interest this was done and it not a case that the appellant has totally revoked the
concession but the concession still exists in modified form. Therefore, the whole exercise
was done in the public interest only. Learned senior counsel stressed that in fact all this
public interest was not disclosed in Pawan Alloys and Casting Pvt. Ltd. (supra).
Therefore, this turned against the
@page-SC701
Board on that count. In the present case all the nine points raised by him were raised
before the High Court of Allahabad but the High Court has totally ignored the same.
9

. Learned senior counsel for the appellant-Corporation also invited our attention to
another decision of this Court in Kasinka Trading and Anr. v. Union of India and Anr.
[ (1995) 1 SCC 274]. In this case, a notification was issued under Section 25 (1) of the
Customs Act in public interest exempting from basic duty and specific date to which it
will remain in force. Prior to expiry of that date another notification was issued in
exercise of same power in public interest withdrawing the exemption on excise duty on
the materials imported. Public interest was explained by the Government and in that
context, it was held that Government being satisfied about the public interest in
withdrawing the exemption no unequivocal representation or promise extended by merely
specifying the period of operation of the exemption notification so as to attract the
doctrine of promissory estoppel. It was pointed out that exemption under Section 25 was
not in the nature of any incentive and has the effect of only suspending levy and
collection of customs duty and can be revoked or withdrawn in public interest. It was
further observed that when exemption is granted in exercise of statutory powers, it is
implicit that it can also be rescinded or modified at any time in exercise of the same
power and it was observed that withdrawal of exemption is a matter of Government
policy with which the Court would not in the absence of any manifest injustice, mala
fides or fraud interfere. It was observed as follows : 1995 AIR SCW 680, (Paras 13 and
19)
"The doctrine of promissory estoppel is applicable against the Government also
particularly where it is necessary to prevent fraud or manifest injustice. The doctrine,
however, cannot be pressed into aid to compel the Government or the public authority "
to carry out a representation or promise which is contrary to law or which was outside the
authority or power of the officer of the Government or of the public authority to make".
To invoke the doctrine of promissory estoppel clear, sound and positive foundation must
be laid in the petition itself by the party invoking the doctrine. Bald expressions, without
any supporting material, to the effect that the doctrine is attracted because the party
invoking the doctrine has altered its position relying on the assurance of the Government
would not be sufficient to press into aid the doctrine. The doctrine of promissory estoppel
cannot be invoked in the abstract and the courts are bound to consider all aspects
including the results sought to be achieved and the public good at large, because while
considering the applicability of the doctrine, the courts have to do equity and the
fundamental principles of equity must for ever be present in the mind of the court, while
considering the applicability of the doctrine. The doctrine must yield when the equity so
demands if it can be shown having regard to the facts and circumstances of the case that
it would be inequitable to hold the Government or the public authority to its promise,
assurance or representation."
However, it was also observed as follows - :
"The reasons given by the Union of India justifying withdrawal of the exemption
notification are not irrelevant to the exercise of the power in "public interest", nor are the
same shown to be insufficient to support the exercise of that power. The exemption
notification was not issued as a potential source of extra profit for the importer. Again, at
the same time when the notification was withdrawn by the Government there was no
scope for any loss to be suffered by the importers. The exemption notification did not
hold out to the appellants any enforceable promise. Neither the notification was of an
executive character nor did it represent a scheme designed to achieve a particular
purpose. It was a notification issued in public interest and again withdrawn in public
interest."
10. Our attention was also invited to a decision of this Court in Shrijee Sales Corporation
and Anr. v. Union of India [(1997) 3 SCC 398]. In this case it was observed as follows :

"Moreover, the Government is competent to resile from a promise even if there is no


manifest public interest involved, provided, of course, no one is put in any adverse
situation which cannot be rectified. Even where there is no such overriding public
interest, it may still be within the competence of the Government to resile from the
promise on giving reasonable notice which need not be a formal notice, giving the
promisee a 1995 AIR SCW 680

@page-SC702
reasonable opportunity of resuming his position, provided, of course, it is possible for the
promisee to restore the status quo ante. If, however, the promisee cannot resume his
position, the promise would become final and irrevocable."
This case in turn followed Kasinka Trading (supra).
11
. Our attention was invited to a decision of this Court in Sales Tax Officer and Anr. v.
Shree Durga Oil Mills and Anr. [ (1998) 1 SCC 572]. In this case it was held that the
Government was competent to change its policy in public interest on the basis of resource
crunch and that would be sufficient for non-applicability of the rule of promissory
estoppel. Their Lordships held that public interest can override consideration of private
loss or gain. Any Industrial Policy Resolution (IPR) can be changed by the State looking
to its severe economic crunch and in this case the respondent sought to invoke this IPR
which was issued on 18.7.1979 and was effective for the period 1979-83. The respondent
established its industry on 28.11.1979. Therefore, on factual aspect also this Court found
that within four months of establishment of industry, the respondent was not likely to
suffer any loss. But at the same time, their Lordships observed as follows : 1998 AIR
SCW 186, (Para 17)

"Any IPR can be changed if there is an overriding public interest involved. In the instant
case, it has been stated on behalf of the State that various notifications granting sales tax
exemptions to the dealers resulted in severe resource crunch. On reconsideration of the
financial position, it was decided to limit the scope of the earlier exemption notifications
issued under Section 6 of the Orissa Sales Tax Act. Because of this new perception of the
economic scenario of the State, the scope of the earlier notifications had to be restricted.
Withdrawal of notification was done in public interest. The Court will not interfere with
any action taken by the Government in public interest. Public interest must override any
consideration of private loss or gain. Thus the plea of change of policy trade on the basis
of resource crunch should have been sufficient for dismissing the respondent's case based
on the doctrine of promissory estoppel."
12

. Our attention was invited to another decision of this Court in State of Rajasthan and Anr.
v. Mahaveer Oil Industries and Ors. [(1999) 4 SCC 357]. In this case also Government of
Rajasthan gave sales tax incentive scheme for industries in 1987 exempting new
industrial units from the tax on sale of goods manufactured by them for sale within the
State for a specified period i.e. from 5.3.1987 to 31.3.1997. Oil extraction and
manufacturing was one of the industries eligible to the benefit of the scheme but the same
was revoked. On facts it was found that the Scheme had failed to achieve its object and
had rather adversely affected the oil industry. In this situation, it was held that the
Government can in public interest revoke the policy and the doctrine of promissory
estoppel cannot preclude the Government from issuing such notification and on facts it
was found that the respondent had not taken any effective steps for starting a new unit
prior to the issuance of the notification. It was observed as follows : 1999 AIR
SCW 2369, (Para 14)

"Public interest requires that the State be held bound by the promise held out by it in such
a situation. But this does not preclude the State from withdrawing the benefit
prospectively even during the period of the Scheme, if public interest so requires. Even in
a case where a party has acted on the promise, if there is any supervening public interest
which requires that the benefit be withdrawn or the Scheme be modified, that
supervening pubic interest would prevail over any promissory estoppel."
13

. As against this, Mr.Shanti Bhushan, learned senior counsel appearing for the
respondents has submitted that in view of Section 78-A of the Act of 1948 a direction was
issued by the State Government for giving this development concession and the State was
competent to give such direction and in pursuance of that the hill development rebate was
given. Mr.Shanti Bhushan submitted that it will be arbitrary and unfair if those
entrepreneurs who have established their industries on the representation made by the
State that they will be given certain concessions and in pursuance of that they have made
huge investments and now that the concession has been withdrawn it will ruin those
entrepreneurs and therefore, the appellant-Corporation is estopped from going back from
their representation. In this connection, he principally relied on a decision of this Court in
M/s. Motilal Padampat Sugar Mills Co. Ltd. v. State of Uttar Pradesh and Ors. AIR
1979 SC 621

@page-SC703
[(1979) 2 SCC 409] and specially invited our attention to paragraph 24 of the judgment.
In paragraph 24, their Lordships have summed up the ratio of the earlier decisions given
by this Court as follows :

"Under our jurisprudence the Government is not exempt from liability to carry out the
representation made by it as to its future conduct and it cannot on some undefined and
undisclosed ground of necessity or expediency fail to carry out the promise solemnly
made by it, nor claim to be the judge of its own obligation to the citizen on an ex parte
appraisement of the circumstances in which the obligation has arisen. The law may,
therefore, now be taken to be settled as a result of this decision, that where the
Government makes a promise knowing or intending that it would be acted on by the
promisee and, in fact, the promisee, acting in reliance on it, alters his position, the
Government would be held bound by the promise and the promisee would be enforceable
against the Government at the instance of the promise, notwithstanding that there is no
consideration for the promise and the promise is not recorded in the form of a formal
contract as required by Article 299 of the Constitution. It is elementary that in a republic
governed by the rule of law, no one howsoever high or low, is above the law. Everyone is
subject to the law as fully and completely as any other and the Government is no
exception. It is indeed the prides of constitutional democracy and rule of law that the
Government stands on the same footing as a private individual so far as the obligation of
the law is concerned; the former is equally bound as the latter. It is indeed difficult to see
on what principle can a Government, committed to the rule of law, claim immunity from
the doctrine of promissory estoppel. Can the Government say that it is under no
obligation to act in a manner that is fair and just or that it is not bound by considerations
of "honesty and good faith"? Why should the Government not be held to a high standard
of rectangular rectitude while dealing with its citizens"? There was a time when the
doctrine of executive necessity was regarded as sufficient justification for the
Government to repudiate even its contractual obligations; but, let it be said to the eternal
glory of this Court, this doctrine was emphatically negatived in the Indo-Afghan
Agencies case and the supremacy of the rule of law was established. It was laid down by
this Court that the Government cannot claim to be immune from the applicability of the
rule of promissory estoppel and repudiate a promise made by it on the ground that such
promise may fetter its future executive action. If the Government does not want its
freedom of executive action to be hampered or restricted, the Government need not make
a promise knowing or intending that it would be acted on by the promisee and the
promisee would atter his position relying upon it. But if the Government makes such a
promisee and the promise acts in reliance upon it and alters his position, there is no
reason why the Government should not be compelled to make good such promise like
any other private individual. The law cannot acquire legitimacy and gain social
acceptance unless it accords with the moral values of the society and the constant
endeavour of the Courts and the legislature must, therefore, be to close the gap between
law and morality and bring about as near an approximation between the two as possible.
The doctrine of promissory estoppel is a significant judicial contribution in that direction.
But it is necessary to point out that since the doctrine of promissory estoppel is an
equitable doctrine, it must yield when the equity so requires.". AIR 1968 SC 718

Mr.Shanti Bhushan emphasized on the basis of this observation made in this case that
benevolent Government has to act with equity and the Court should yield in favour of the
equity whenever case arises of a citizen who has acted bona fidely on the basis of the
representation made by the Government or by the instrumentality of the State. Mr. Shanti
Bhushan submitted that since representation was made by the appellant-Corporation,
therefore, industries were established in the hill areas and now the appellant-Corporation
wanted to change the tariff that will be unconstitutional, unfair and arbitrary to the
citizens who have acted on the promise made by the appellant-Corporation. In this
connection, Mr. Shanti Bhushan also submitted that this is violative of Article 14 of the
Constitution as held in MRF Ltd., Kottayam v. Asstt. Commissioner (Assessment), Sales
Tax and Ors. [(2006) 8 SCC 702]. In that case, the Court held that revocation of such
notification is arbitrary and one of us (Hon'ble Katju, J.) was a party to the judgment. In
this case the concept of 2006 AIR SCW 5272

@page-SC704
doctrine of legitimate expectation was invoked. In this case, the State of Kerala issued
notification granting exemption for expansion in the manufacture of certain products
including rubber-based goods. The assessee manufacturer relying on that introduction of
exemption commenced commercial production after investing huge amount. This
concession was granted for a fixed period of seven years. But during the currency of the
period of exemption the State Government issued another notification excluding the
formation of a compound rubber from the definition of "manufacture" for the purpose of
the original exemption notification. Therefore, this premature deprivement of the
exemption to the assessee manufacturer was held by the Court arbitrary, unjust and
unreasonable. Their Lordships invoked the doctrine of legitimate expectation. It was
contended before the Court that the notification was a statutory one and no plea of
estoppel would lie against the statute. But their Lordships held that the principle of
underlying legitimate expectation was based on Article 14 of the Constitution and any
action taken by the State which went against the rule of fairness was liable to be struck
down. Finally this Court after review of the cases on the subject, invoked the principle of
promissory estoppel and also the legitimate expectation and found that the revocation of
the exemption granted for a period of seven years by the State Government was arbitrary,
unjust and unreasonable and was liable to be quashed. It was observed as follows :

"This Court in E. P. Royappa v. State of T.N. [(1974) 4 SCC 3] observed that where an act
is arbitrary, it is implicit in it that it is unequal both according to political logic and
constitutional law and is therefore violative of Article 14. Equity that arises in favour of a
party as a result of a representation made by the State is founded on the basic concept of "
justice and fair play". The attempt to take away the said benefit of exemption with effect
from 15-1-1998 and thereby deprive MRF of the benefit of exemption for more than 5
years out of a total period of 7 years, in our opinion, is highly arbitrary, unjust and
unreasonable and deserves to be quashed. .. .." AIR 1974 SC 555

14

. Mr. Shanti Bhushan, learned senior counsel invited our attention to paragraph 33 of the
judgment in Pawan Alloys and Casting Pvt. Ltd.(supra) and submitted that in fact an
argument was made at the Bar that the high-powered Tariff Realisation Committee
advised the Board for withdrawing this rebate and the Board acted in the light of the said
report submitted to it in the year 1986. It was submitted that the genesis of the
notification was the recommendation of the Tariff Realisation Committee. Therefore, the
Court concluded that the rebate was revoked not on the ground of general public interest
but solely on the ground of commercial interest of the Board. Therefore, it was observed
as follows : 1997 AIR SCW 3839, (Para 30)

"Consequently it must be held on the facts of these cases that the impugned withdrawal
notification was not backed up by any demands of public interest which would outweigh
the individual interests of the appellant-promisees who had acted upon the same."

Mr. Shanti Bhushan, learned senior counsel submitted that in the present case also, the
revocation is not on the basis of general public interest but it is only on account of losses
the Corporation trying to make up the losses revoked this concession. Therefore, learned
senior counsel submitted that it is not the consideration of general public interest but
based on the commercial angle. Learned senior counsel invited our attention to the
decision in Kasinka Trading and Anr. (supra), specially to paragraph 21 of the judgment
and submitted that, that case is distinguishable on the ground that it only suspended the
levy and collection of customs duty wholly or partially and there was no promise for
benefit to public at large. Thus, the exemption notification issued under Section 25(1) of
the Customs Act is an exercise of the statutory power of the State under the law itself and
the State can revoke the same as per General Clauses Act. Therefore, Mr. Shanti Bhushan
distinguished the case of Kasinka Trading (supra) that the said case was not the case in
which any promise was made and on which the assessee has acted and invoked certain
benefits. It was a general notification giving certain benefits and it was revoked back in
public interest. Learned senior counsel invited our attention to a decision of this Court in
Shrijee Sales Corporation and Anr. (supra) and submitted that it was not an inducement
but a case of promissory estoppel when a promise is made and 1995 AIR SCW 680
1995 AIR SCW 680
1998 AIR SCW 186

@page-SC705
citizen is induced to act on those representation, then in that case, once the party has
suffered on account of so called inducement, then in that case it cannot be revoked to the
disadvantage of the other party. Learned senior counsel submitted that in Shrijee Sales
Corporation and Anr. (supra) and Shree Durga Oil Mills and Anr.(supra) certain tax
exemption was given and subsequently it was revoked and learned senior counsel
submitted that those cases are distinguishable, that there were not the cases in which
inducement was made, and the party acted on that inducement. Those were the cases
where exemption was given on customs and sales tax but it was not in the nature of
inducement or any representation or promise on the part of the other party to encourage
the entrepreneurs to come and make their investments.
15

. Learned senior counsel invited our attention to a decision of this Court in State of
Punjab v. Nestle India Ltd. and Anr. [(2004) 6 SCC 465] in which a representation was
made by the Government in the manner de hors the Rules but a statement was made by
the Finance Minister in his Budget speech for 1996-97 making representation to the effect
that the State Government had abolished purchase tax on milk. The manufacturers of
milk products, therefore, were not paying the purchase tax on milk for the assessment
year 1996-97 and mentioned this fact in their returns. The taxing authority entertained
such returns. The manufacturers passed on the benefit of exemption to the dairy farmers
and milk producers. However, after expiry of the said assessment year, the Government
took a decision not to abolish purchase tax on milk and the taxing authority therefore
raised a demand for the assessment year 1996-97. On these facts, the Court held that in
absence of proof of any overriding public interest rendering the enforcement of estoppel
against the Government was inequitable, notwithstanding that no exemption notification
as required by the statute was issued. It was held that the State Government cannot resile
from its decision to exempt milk and raise a demand for the aforesaid assessment year.
However, the same principle of estoppel was not invoked after assessment year 1996-97.
The Court enforced the principle of estoppel. All the earlier cases on the subject were
reviewed by the Court and ultimately it was concluded as follows : 2004 AIR SCW 3703

"47. The appellant has been unable to establish any overriding public interest which
would make it inequitable to enforce the estoppel against the State Government. The
representation was made by the highest authorities including the Finance Minister in his
Budget speech after considering the financial implications of the grant of the exemption
to milk. It was found that the overall benefit to the State's economy and the public would
be greater if the exemption were allowed. The respondents have passed on the benefit of
that exemption by providing various facilities and concessions for the upliftment of the
milk producers. This has not been denied. It would, in the circumstances, be inequitable
to allow the State Government now to resile from its decision to exempt milk and
demand the purchase tax with retrospective effect from 1-4-1996 so that the respondents
cannot in any event readjust the expenditure already made. The High Court was also right
when it held that the operation of the estoppel would come to an end with the 1997
decision of the Cabinet."

Similarly, our attention was invited to paragraph 16 of the judgment in Shree Durga Oil
Mills and Anr.(supra). Mr.Shanti Bhushan submitted that in the aforesaid case Section 6
of the Orissa Sales Tax Act clearly contemplated that the State Government can grant
exemption from sales tax and likewise withdraw any such exemption. Learned senior
counsel submitted that so far as Section 49 of the Act of 1948 is concerned, there is no
such contemplation that it can also revoke the same. It is only because of the provisions
of the General Clauses Act it can be revoked but not once granted under Section 49(3) of
the Act of 1948, there is no provision for any revocation of the exemption granted to
certain class of persons having regard to the geographical condition of the area, the nature
of supply and the purpose for which supply is required and other relevant factors.
Mr.Shanti Bhushan also submitted that there is no allegation of theft in the hill area by
the persons to whom the power had been granted at a concessional rate. And all the
circumstances which have been taken into consideration for revocation of the exemption
notification show that there was no overwhelming consideration for 1998 AIR
SCW 186

@page-SC706
revoking such exemption in public interest.
16

. Mr. S. Ganesh, learned senior counsel appearing for some of the respondents invited our
attention to a decision of this Court in Mahabir Vegetable Oils (P) Ltd. and Anr. v. State
of Haryana and Ors. [(2006) 3 SCC 620]. In this case, the appellants were the owner of
solvent extraction plants. Industrial policy for the period 1.4.1988 to 31.3.1997 granted
incentive by way of sales tax exemption to the industries set up in backward areas in the
State. Solvent at that time was not included in the negative list in the Rules. In August,
1995 the appellants purchased land to set up a new unit and they made huge amount in
construction work, erection of plant and that investment constituted 45% of the total
investment. They started trial production on 26.3.1997 and commercial production on
29.3.1997 and then they applied for grant of exemption for payment of sales tax.
Meanwhile, the State Government notified its intention to amend the Haryana General
Sales Tax Rules and invited objections and thereafter they issued notification on
16.12.1996 which included solvent extraction plants in the negative list but Note 2
appended to that list provided that the industrial units which had made investment up to
25% of the anticipated cost of the project and which had been included in the negative list
for the first time would be entitled to the sales tax benefits related to the extent of
investment made up to 3.1.1996. On 28.5.1997 Note 2 was omitted. As a result of this,
the appellants were deprived of the benefit and consequently, the Department rejected the
application for exemption. This was challenged unsuccessfully before the High Court and
ultimately the matter reached this Court and this Court held that the incumbents had made
huge investment pursuant to and in furtherance of the representation made by the State
Government and the State Government without assigning any reason withdrew the
exemption with retrospective effect at the end of the operative period. The retrospective
withdrawal of the exemption was found to be bad in law. In this context, their Lordships
observed as follows: 2006 AIR SCW 1500

"Undisputedly, when the appellants started making investments, Rule 28-A was
operative. Representation indisputably was made in terms of the said Rules. The relevant
provisions of the Act and the Rules framed thereunder indisputably were made keeping in
view the industrial policy of the State."
Their Lordships held that the doctrine of promissory estoppel will operate even in the
legislative field. Learned senior counsel submitted that such concession which has been
granted cannot be revoked as the beneficiary acquired a vested right and the same can
only be revoked by the Statute.
17. In this background, in view of various decisions noticed above, it will appear that the
Court's approach in the matter of invoking the principle of promissory estoppel depends
on the facts of each case. But the general principle that emerges is that once a
representation has been made by one party and the other party acts on that representation
and makes investment and thereafter the other party resiles, such act cannot stated to be
fair and reasonable. When the State Government makes a representation and invites the
entrepreneurs by showing various benefits for encouraging to make investment by way of
industrial development of the backward areas or the hill areas, and thereafter the
entrepreneurs on the representations so made bona fidely make investment and thereafter
if the State Government resile from such benefits, then it certainly is an act of unfairness
and arbitrariness. Consideration of public interest and the fact that there cannot be any
estoppel against a Statute are exceptions.
18

. Learned senior counsel for the appellant has cited nine instances which can be loosely
categorised into two i.e. (i) that there cannot be any estoppel against the statute and (ii)
overriding public interest. So far as the first part is concerned i.e. the revocation has the
statute flavour i.e. the benefit which was extended under Section 49 of the Act of 1948
and the notification had been issued revoking the same benefit under Section 49 of the
Act of 1948 by invoking the provisions of the General Clauses Act that an authority
granting exemption has a right to revoke the same also. It is true that it has a right to
revoke the same but if the other party has suffered on that account then such
representation will be against the public policy and the morality. Notification issued
under Section 49 of the Act of 1948 for giving the benefit of exemption for the hill areas
was in the nature of delegated legislation and not an Act framed by the State Legislature.
Therefore, a distinction has to1998 AIR SCW 186

@page-SC707
be made between the delegated legislation and the primary legislation framed by the
Legislature. In Section 49 there is no specific stipulation that the notification issued under
Section 49 of the Act of 1948 can be revoked at any time as was in the case of Shree
Durga Oil Mills and Anr. (supra) where Section 6 of the Orissa Sales Tax Act itself
provided that the notification is capable of being revoked at any time. Therefore, a
distinction has to be made between the delegated legislation and the primary legislation.
So far as the primary legislation is concerned, if the Act is passed by State Legislature
and denies the benefit by the primary legislation then no estoppel can be applied against
that Act but so far as the case of delegated legislation is concerned, where delegated
authorities passes certain notification in exercise of his delegated authority there is no
contemplation mentioned in the Act itself that it is capable of being revoked at any time.
Then such Acts cannot be treated at par with the primary Act passed by the State
Legislature. The State is fully competent to pass an Act prospectively as well as
retrospectively but retrospectivity to the extent of aforesaid nature cannot stand.
Therefore, this distinction has to be borne in mind. In the present case, the U.P. Electricity
Reforms Act, 1999 came into force with effect from 2000. Therefore, if such benefit has
not been extended then a different stand will follow but so far as the delegated legislation
is concerned, this kind of revocation cannot be sustained. It is highly against the public
morality that the incumbent who have felt persuaded on account of the representation
made by the State Government that they will be given certain benefits and they acted on
that representation, it does not behove on the part of the appellant-Corporation to
withdraw the said benefit before expiry of the stipulated period by issuing the notification
revoking the same which the respondents were legitimately entitled to avail. We fail to
understand why the appellant-Corporation which made a representation and allowed the
other party to act upon such representation could resile and leave the citizens in a lurch.
In such a situation the principle of promissory estoppel which has been evolved by the
Courts which is based on public morality cannot permit the State to act in such an
arbitrary fashion. Other grounds for the purpose of public interest which have been
pleaded; namely that there are two methods of tariff provided by the amendment and the
actual consumption has been reduced based on the calculation of energy charges per KV
from 308 paise to 100 paise and there was large scale theft or that units were closing
down and there was no mala fide intention in the matter of revocation of the notification
and the cost of production of power has gone up to Rs.2.50 per unit, are considerations
which hardly involve any public interest. They were more of a nature of losses which has
been suffered by the Corporation and in order to make these losses, these methods were
evolved to reduce and to make good of the losses. Restructuring benefit to 17% of the
Tariff 4(A) (demand charges) are the factors which are aimed at to make the losses good
for the Corporation. This is not case in which serious public repercussion was involved.
These are not the factors which put together can constitute a public interest. Theft of the
energy if it was proved by cogent datas that as a result of giving this benefit to the
entrepreneurs in the hill areas, they were misusing it or there was theft of the energy at a
large scale by these persons to whom the concession had been given then of course such
factors, if all the datas were brought on record of course could have persuaded the Court
to take a different view of the matter. But simply because there was theft of energy allow
the State cannot persuade us to hold that the revocation of such concession can be said to
be in public interest. Since the benefit was given to these units in the hill areas, there
should have been overwhelming evidence to show some mala fide on the part of these
consumers which have persuaded the Corporation to revoke it. If there was no misuse of
the energy by these units in the hill areas to whom the concession had been granted then
in that case it cannot be taken that there was really public interest involved which
persuaded the Corporation to revoke the same. No person can be permitted to misuse the
concession or benefit and invoke promissory estoppel. Promissory estoppel is not one
sided affair, it is rather two sided affair. If one party abuses the concession then it is
always open to the other party to revoke such concession but if one party avails the
benefit and is acting on the same representation made by the other party then the other
@page-SC708
party who has granted the said benefit cannot revoke the same under the garb of public
interest. Therefore the grounds that the revocation notification was issued in public
interest and that same has the flavour of the statute, cannot persuade us to uphold it.
sustained. It is true that a detailed statement was given in various paragraphs of the
written statement filed by the appellant-Corporation before the Allahabad High Court and
unfortunately, the High Court did not advert to these details but we have examined all
these details and found that all the nine points raised by Dr. Singhvi does not persuade us
to take a contrary view from the view taken by the High Court. There is no gainsaying
that the public interest is paramount and the private interest has to be sacrificed for the
larger interest. But, after survey of all these cases on the subject, the judicial consensus
that emerges is that whenever the State has made a representation to the public and the
public has acted on that representation and suffered economically or otherwise, then in
that case the State should be estopped from withdrawing such benefit to the detriment of
the such people except in public interest or against the Statute. So far as the public
interest as involved in the present case is concerned, we have found that there was no
overwhelming evidence to revoke the benefit granted to the industrial units in the hill
areas. So far as the Statute is concerned, the notification was issued under Section 49 of
the Act of 1948 and the same was revoked under Section 49 of the Act of 1948 though
there was no such provision contained in Section 49 that it will be open to the
Corporation to revoke the same but could be possible by invoking the principle of
General Clauses Act. But in such delegated legislation such withdrawal could only be
permitted if larger public interest is involved or if the Act is passed by legislature.
19. Dr. Singhvi, learned senior counsel for the appellant-Corporation submitted that now
the Act of 1999 has come into force and that Act does not recognize the concessions
given to the hill areas and that this is a primary legislation i.e. Act passed by the State
Legislature. Therefore, to this extent we can accept the submission of Dr. Singhvi that
since the Act of 1999 does not recognize such hill developmental benefits, therefore,
from the date of passing of the Act of 1999 the said benefit cannot be accepted. We have
stated above that there cannot be estoppel against a statute. Since such benefits have not
been recognised by the Act of 1999, therefore, up to the date of coming into force of the
Act of 1999, all the benefits which were being given to the respondent- entrepreneurs
shall be protected by invoking the principle of promissory estoppel but after coming into
force of the Act of 1999, which is a primary legislation enacted by the State Legislature
the benefits from the date the Act has come into force, cannot be made available to the
respondents.
20. In this 21st century, when there is global economy, the question of faith is very
important. Government offers certain benefits to attract the entrepreneurs and the
entrepreneurs act on those beneficial offers. Thereafter, the Government withdraws those
benefits. This will seriously affect the credibility of the Government and would show the
shortsightedness of the governance. Therefore, in order to keep the faith of the people, the
Government or its instrumentality should abide by their commitments. In this context, the
action taken by the appellant-Corporation in revoking the benefits given to the
entrepreneurs in the hill areas will sadly reflect their credibility and people will not take
the word of the Government. That will shake the faith of the people in the governance.
Therefore, in order to keep the faith and maintain good governance it is necessary that
whatever representation is made by the Government or its instrumentality which induces
the other party to act, the Government should not be permitted to withdraw from that.
This is a matter of faith.
21. Therefore, as a result of our above discussion, we hold that the view taken by the
Allahabad High Court on revoking the principle of promissory estoppel is correct and the
respondent- units will be entitled to such benefits till the U.P. Electricity Reforms Act,
1999 came in to force. Since after coming into force the Act of 1999 no such concession
has been granted, therefore, the concession shall survive till the Act of 1999 came into
force. The appeals are accordingly disposed of with no order as to costs
Order accordingly.
@page-SC709
AIR 2008 SUPREME COURT 709 "Atma Singh v. State of Haryana"
(From : Punjab and Haryana)*
Coram : 2 G. P. MATHUR AND D. K. JAIN, JJ.
Civil Appeal Nos. 3148-3157 of 2000, D/- 7 -12 -2007.
Atma Singh (died) through L. Rs. and Ors. v. State of Haryana and Anr.
(A) Land Acquisition Act (1 of 1894), S.23 - ACQUISITION OF LAND - SALE -
Compensation - Determination - Comparable sales - Sale instances of small pieces of
land - Relevancy - Large tract of land acquired - Acquired land having potentiality for
being used as commercial, industrial and residential purposes - Exemplars of small plots -
Cannot be disregarded in such cases specially when exemplars of large pieces of land are
not available. (Para 8)
(B) Land Acquisition Act (1 of 1894), S.23 - ACQUISITION OF LAND - SALE -
Compensation - Determination - Large tract of land acquired - Determination of market
value on exemplars of sale of small plots - Percentage of cut to be applied - Depends
upon purpose for which acquisition is made - Land acquired for setting-up factory - Gives
good returns - Deduction from price exhibited by the exemplars even if they are of small
plots - Not justified - On facts deduction of 10% allowed in case of land acquired for
sugar factory.
The reasons given for the principle that price fetched for small plots cannot form safe
basis for valuation of large tracts of land, are that substantial area is used for development
of sites like laying out roads, drains, sewers, water and electricity lines and other civic
amenities. Expenses are also incurred in providing these basic amenities. That apart it
takes considerable period in carving out the roads, making sewers and drains and waiting
for the purchasers. Meanwhile the invested money is blocked up and the return on the
investment flows after a considerable period of time. In cases of acquisition made for
setting up factory, the factory would produce goods worth many crores in a year. An
industry established on acquired land, if run efficiently, earns money or makes profit
every year. The return from the land acquired for the purpose of Housing Colony, or
Offices, or Institution cannot even remotely be compared with the land which has been
acquired for the purpose of setting up a factory or industry. After all the factory cannot be
set up without land and if such land is giving substantial return, there is no justification
for making any deduction. (Paras 13, 14)
Cases Referred : Chronological Paras
2004 AIR SCW 797 : AIR 2004 SC 1800 12
2002 AIR SCW 4644 : AIR 2003 SC 202 9
(1996)11 SCC 542 12
1995 AIR SCW 3655 : AIR 1995 SC 2481 12
1992 AIR SCW 2791 : AIR 1992 SC 2298 9
AIR 1990 SC 2192 4
AIR 1989 SC 1222 5
AIR 1988 SC 943 4, 5, 10
AIR 1988 SC 1652 11
AIR 1984 SC 892 5
AIR 1979 SC 472 5
AIR 1977 SC 1560 4
AIR 1976 SC 2219 4
AIR 1969 SC 465 5
Manjit Singh, A. A. G., M. L. Varma, Mahendra Anand, Anoop G. Choudhary, Rakesh
Dwivedi, Brijender Chahar, Sr. Advocates, Rishi Malhotra, Prem Malhotra, Chander
Shekhar Ashri, T. V. George, Vinay Garg, Abhinav Jain, Mrs. Jyoti Chahar, Mrs. Deepam
Garg, Jagbir Singh Malik, Devendra Kumar Singh, S. K. Bansal, Mrs. Savitri Bansal,
Roopak Bansal and Dr. Kailash Chand, for the appearing parties.
* RFA Nos. 373-377, 690 to 693 and 378 of 1986, D/- 4-1-1989 (PandH).
Judgement
1. G. P. MATHUR, J. :- These appeals, by special leave, have been preferred against the
judgment and decree dated 4.1.1989 of High Court of Punjab and Haryana at Chandigarh,
by which 17 appeals preferred by claimant-appellants (landowners) against the common
judgment and award of the Additional District Judge, Kurukshetra, dated 31.8.1985 had
been decided. The claimant-appellants had sought enhancement of the amount of
compensation for acquisition of their land.
2. A notification under Section 4 of the Land Acquisition Act (hereinafter referred to as
'the Act') was issued for acquisition of 89 acres and 3 marlas of land for construction of a
co-operative sugar mill. The land was situate as one compact unit in four villages viz.
Kankar Shahbad, Chhapra, Jandheri and Jhambara and belonged to 17 families. In
response to the notice issued by the Collector under Section 9 of the Act, landowners
filed objections claiming compensation
@page-SC710
for their land which had been acquired. The Land Acquisition Collector, after holding an
enquiry, gave an award on 14.7.1983 under Section 11 of the Act. The Collector gave
award on the basis of quality of land, for which purpose he divided the acquired land in
seven categories and the market value was assessed at Rs.6,000/- to Rs.35,000/- per acre
for different types of lands. Feeling aggrieved by the award of the Collector, the
appellants herein (landowners) sought reference to the Court under Section 18 of the Act.
The learned Additional District Judge awarded compensation at a flat rate of Rs.43,000/-
per acre by placing reliance on Ex. R-6 and R-7, two instances of sale deeds of village
Chhapra. After taking average of these sale transactions, an addition of 25% was made
for fixing the market value of the land. Against the award made by the learned Additional
District Judge, the claimant-appellants (landowners) preferred 17 appeals before the High
Court. The High Court after appraisal of evidence on record held that the market value of
the land acquired was Rs.1,20,000/- per acre. It further held that the exemplars filed by
the appellants were of small pieces of land and, therefore, a deduction of 33% had to be
made and accordingly the market value of the land was assessed at Rs.80,000/- per acre.
Besides the market value, the appellants were also held entitled to statutory sums under
Sections 23(1-A), 23(2) and 28 of the Act. The State of Haryana had also filed appeals
against the award of the Additional District Judge, but the same were dismissed.
3. The appeals in this Court have only been filed by the landowners and the State of
Haryana has not filed any appeal challenging the judgment and decree of the High Court.
We have heard Shri M.L. Varma, learned Senior Advocate for the appellants and Shri
Rakesh Dwivedi, learned Senior Advocate for the Shahabad Cooperative Sugar Mills
Ltd., for whose benefit the land has been acquired.
4. In order to determine the compensation which the tenure-holders are entitled to get for
their land which has been acquired, the main question to be considered is what is the
market value of the land. Section 23(1) of the Act lays down what the Court has to take
into consideration while Section 24 lays down what the Court shall not take into
consideration and have to be neglected. The main object of the enquiry before the Court
is to determine the market value of the land acquired. The expression 'market value' has
been subject-matter of consideration by this Court in several cases. The market value is
the price that a willing purchaser would pay to a willing seller for the property having due
regard to its existing condition with all its existing advantages and its potential
possibilities when led out in most advantageous manner excluding any advantage due to
carrying out of the scheme for which the property is compulsorily acquired. In
considering market value disinclination of the vendor to part with his land and the urgent
necessity of the purchaser to buy should be disregarded. The guiding star would be the
conduct of hypothetical willing vendor who would offer the land and a purchaser in
normal human conduct would be willing to buy as a prudent man in normal market
conditions but not an anxious dealing at arms length nor facade of sale nor fictitious sale
brought about in quick succession or otherwise to inflate the market value. The
determination of market value is the prediction of an economic event viz., a price
outcome of hypothetical sale expressed in terms of probabilities. See Thakur Kanta
Prasad v. State of Bihar, AIR 1976 SC 2219; Prithvi Raj Taneja v. State of M. P., AIR
1977 SC 1560; Administrator General of West Bengal v. Collector, Varanasi, AIR 1988
SC 943 and Periyar v. State of Kerala, AIR 1990 SC 2192.
5. For ascertaining the market value of the land, the potentiality of the acquired land
should also be taken into consideration. Potentiality means capacity or possibility for
changing or developing into state of actuality. It is well settled that market value of a
property has to be determined having due regard to its existing condition with all its
existing advantages and its potential possibility when led out in its most advantageous
manner. The question whether a land has potential value or not, is primarily one of fact
depending upon its condition, situation, user to which it is put or is reasonably capable of
being put and proximity to residential, commercial or industrial areas or institutions. The
existing amenities like, water, electricity, possibility of their further extension, whether
near about Town is developing or has prospect of development have to be taken into
consideration. See Collector Raigarh v. Hari Singh Thakur, AIR 1979 SC 472, Raghubans
Narain v. State of U.P.,
@page-SC711
AIR 1969 SC 465 and Administrator General, W. B. v. Collector, Varanasi, AIR 1988 SC
943. It has been held in Kaushalya Devi v. L.A.O., Aurangabad, AIR 1984 SC 892 and
Suresh Kumar v. T.I. Trust, AIR 1980 SC 1222 that failing to consider potential value of
the acquired land is an error of principle.
6. As mentioned earlier, the learned Additional District Judge had awarded compensation
at a flat rate of Rs.43,000/- per acre by placing reliance on Ex. R-6 and R?7, two
instances of sale of village Chhapra. After taking an average of these two sale
transactions, an addition of 25% was made while fixing the market value of the land. The
High Court held that these two sale deeds were of 31.12.1980, while in the instant case,
the notification under Section 4 of the Act was published much later on 9.2.1983. That
apart, Ex.R-6 and R-7 were mutation orders and the corresponding sale deeds had not
been brought on the record. In fact, the learned Additional District Judge, in the earlier
part of the judgment, had himself discarded Ex. R-6 and R-7 as they were mutation
orders and were inadmissible in evidence. The High Court, therefore, rightly held that no
reliance could be placed upon Ex.R-6 and R-7 for determining the market value of the
land.
7. The claimant-appellants (landowners) had filed copies of four sale deeds which are
Exs.P-7, P-8, P-9 and P-10. In fact, Ex. P-7 is a copy of a sale deed by which Laxman
Singh bought some land in village Chhapra on 28.7.1982, which itself became subject-
matter of acquisition. Laxman Singh had deposed that he had bought the land for
construction of shops. All these four sale deeds related to sale transactions prior to the
issuance of the notification under Section 4 of the Act on 9.2.1983. The High Court
excluded Ex.P-8 from consideration as it related to a very small piece of land measuring
19 marlas only. The average price of the three sale deeds viz. Ex. P-7, P-9 and P-10 came
to little more than Rs.1,20,000/- per acre. Apart from these three sale deeds, no other
exemplars were filed either by the State of Haryana or by the landowners. The High
Court accepted the price exhibited by the aforesaid three sale transactions which came to
little more than Rs.1,20,000/- per acre. It thus recorded a finding that the market value of
the land was Rs.1,20,000/- per acre. In our opinion, there being no other documentary
evidence, the view taken by the High Court that the market value of the land was
Rs.1,20,000/- per acre is perfectly correct and calls for no interference.
8. Shri Rakesh Dwivedi, learned senior counsel for the sugar mill has submitted that the
exemplars filed by the appellants were of very small pieces of land and, therefore, they
are not safe guide to determine the market value of the land. It may be mentioned here
that while determining the market value, the potentiality of the land acquired has also to
be taken into consideration. The appellants have led evidence to show that the acquired
land had the potentiality to be used for commercial, industrial and residential purposes.
PW.1 Rakesh Kumar had prepared a site plan which showed that the acquired land was
adjacent to the abadi of Shahabad and abutted the Shahabad-Ladwa Road. The site plan
also shows that there existed rice shellers, cold storage, shops, godowns, a college and
houses etc. on both sides of Shahabad-Ladwa Road. PW.2 Baldev Singh was Patwari of
village Chhapra in the year 1983. He deposed that all the four villages viz. Kankar
Shahbad, Chhapra, Jandheri and Jhambara are adjacent to each other and the acquired
land abutted the Shahabad-Ladwa Road. He further deposed that the acquired land was 2
kilometer from G.T. Road and there were buildings, godowns, a cinema hall, factories on
both sides of the Shahabad-Ladwa Road. Therefore, there can be no manner of doubt that
the acquired land had the potentiality for being used for commercial, industrial and
residential purposes and there was fair possibility of increase in its market value in the
near future. Therefore, the fact that the exemplars filed by the appellants were of the
small pieces of land could not be a ground to discard them specially when exemplars of
large pieces of land were not available. They could, therefore, be used as a safe guide for
determining the market value of the land.
9

. Learned counsel for the appellants has seriously challenged the finding of the High
Court that the market value of the land determined on the basis of the exemplars filed by
the parties should be reduced by one-third on account of the fact that the exemplars relied
upon for ascertaining the market value related to sale of small pieces of land. According
to Shri M.L. Verma, learned senior counsel for the appellants, there is no uniform
principle that if a large area has 1992 AIR SCW 2791

@page-SC712
been acquired and the exemplars are of small pieces of land, the market value exhibited
by the exemplars must necessarily be reduced by one-third. Shri Verma has placed strong
reliance on Bhagwathula Samanna and Ors. v. Special Tehsildar and Land Acquisition
Officer, Visakha-patnam Municipality (1991) 4 SCC 506, wherein it was held as under :-
"In fixing the market value of a large property on the basis of a sale transaction for
smaller property, generally a deduction is given taking into consideration the expenses
required for development of the larger tract to make smaller plots within that area in order
to compare with the small plots dealt with under the sale transaction. However, in
applying this principle of deduction it is necessary to consider all relevant facts. It is not
the extent of the area covered under the acquisition which is the only relevant factor. If
smaller area within the large tract is already developed and situated in an advantageous
position suitable for building purposes and have all amenities such as roads, drainage,
electricity, communications etc. then the principle of deduction simply for the reason that
it is part of the large tract acquired, may not be justified.
In the present cases the lands covered by the acquisition are located by the side of the
National Highway and the Southern Railway Staff Quarters with the Town Planning Trust
road on the north. The neighbouring areas are already developed ones and houses have
been constructed, and the land has potential value for being used as building sites. Having
found that the land is to be valued only as building sites and having stated the
advantageous position in which the land in question lies though forming part of the larger
area, the High Court should not have applied the principles of deduction. It is not in every
case that such deduction is to be allowed. Therefore, the High Court erred in making a
deduction of one third of the value of the comparable sale and thus reducing the fair
market value of land from Rs. 10 per sq. yard to Rs.6.50 per sq. yard."

Shri Verma has also referred to Kasturi and Ors. v. State of Haryana (2003) 1 SCC 354,
wherein it was observed that in cases of those land where there are certain advantages by
virtue of the developed area around, it may help in reducing the percentage of cut to be
applied, as the development charges required may be less on that account. There may be
various factual factors which may have to be taken into consideration while applying the
cut in payment of compensation towards development charges, may be in some cases it is
more than 1/3rd and in some cases less than 1/3rd. Therefore, in this case taking into
consideration the potentiality of the acquired land for construction of residential and
commercial buildings, the deduction made was only 20%. 2002 AIR SCW 4644

10. Shri Rakesh Dwivedi, learned senior counsel for the sugar mill has, on the other hand,
strenuously urged that the evidence of market value shown by sale of small plots is not a
safe guide in valuing large areas of land and the prices fetched for small plots cannot be
directly adopted in valuing large extent of land as has been acquired in the present case.
He has thus contended that a deduction of 30% had rightly been made by the High Court
on account of acquisition of a large area. In support of his contention, Shri Dwivedi has
placed reliance upon several decisions of this Court. In order to appreciate the principle
laid down therein, it will be useful to refer to them in some detail. In Administrator
General of West Bengal v. Collector, Varanasi, AIR 1988 SC 943, it was held as follows
in para 6 of the report:-
"The principle requires that prices fetched for small developed plots cannot directly be
adopted in valuing large extents. However, if it is shown that the large extent to be valued
does admit of and is ripe for use for building purposes; that building lots that could be
laid out on the land would be good selling propositions and that valuation on the basis of
the method of a hypothetical layout could with justification be adopted, then in valuing
such small laid out sites the valuation indicated by sale of comparable small sites in the
area at or about the time of the notification would be relevant. In such a case, necessary
deductions for the extent of land required for the formation of roads and other civic
amenities; expenses of development of the sites by laying out roads, drains, sewers, water
and electricity lines, and the interest on the outlays for the period of deferment of the
realisation of the prices; the profits on the venture etc. are to be made."
@page-SC713
11. In Chimanlal v. Special Land Acquisition Officer, AIR 1988 SC 1652 it was held as
follows in para 4 (15) of the reports.
"Firstly while a smaller plot is within the reach of many, a large block of land will have to
be developed by preparing a lay out, carving out roads, leaving open space, plotting out
smaller plots, waiting for purchasers (meanwhile the invested money will be blocked up)
and the hazards of an entrepreneur. The factor can be discounted by making a deduction
by way of an allowance at an appropriate rate ranging approx, between 20% to 50% to
account for land required to be set apart for carving out lands and plotting out small plots.
The discounting will to some extent also depend on whether it is a rural area or urban
area, whether building activity is picking up, and whether waiting period during which
the capital of the entrepreneur would be locked up, will be longer or shorter and the
attendant hazards."
12

. Shri Dwivedi has also referred to Basant Kumar and Ors. v. Union of India and Ors.
(1996) 11 SCC 542, K. Vasundara Devi v. Revenue Divisional Officer (LAO) (1995) 5
SCC 426, H.P. Housing Board v. Bharat S. Negi and Ors. (2004) 2 SCC 184. In the first
cited case land was acquired for planned development of Delhi and in the other two cases
for Housing Boards and a deduction of 33% was applied. 1995 AIR SCW 3655
2004 AIR SCW 797

13. The reasons given for the principle that price fetched for small plots cannot form safe
basis for valuation of large tracts of land, according to cases referred to above, are that
substantial area is used for development of sites like laying out roads, drains, sewers,
water and electricity lines and other civic amenities. Expenses are also incurred in
providing these basic amenities. That apart it takes considerable period in carving out the
roads making sewers and drains and waiting for the purchasers. Meanwhile the invested
money is blocked up and the return on the investment flows after a considerable period of
time. In order to make up for the area of land which is used in providing civic amenities
and the waiting period during which the capital of the entrepreneur gets locked up a
deduction from 20% onward, depending upon the facts of each case, is made.
14. The question to be considered is whether in the present case those factors exist which
warrant a deduction by way of allowance from the price exhibited by the exemplars of
small plots which have been filed by the parties. The land has not been acquired for a
Housing Colony or Government Office or an Institution. The land has been acquired for
setting up a sugar factory. The factory would produce goods worth many crores in a year.
A sugar factory apart from producing sugar also produces many by-product in the same
process. One of the by-products is molasses, which is produced in huge quantity. Earlier,
it had no utility and its disposal used to be a big problem. But now molasses is used for
production of alcohol and ethanol which yield lot of revenue. Another by-product begasse
is now used for generation of power and press mud is utilized in manure. Therefore, the
profit from a sugar factory is substantial. Moreover, it is not confined to one year but will
accrue every year so long as the factory runs. A Housing Board does not run on business
lines. Once plots are carved out after acquisition of land and are sold to public, there is no
scope for earning any money in future. An industry established on acquired land, if run
efficiently, earns money or makes profit every year. The return from the land acquired for
the purpose of Housing Colony, or Offices, or Institution cannot even remotely be
compared with the land which has been acquired for the purpose of setting up a factory or
industry. After all the factory cannot be set up without land and if such land is giving
substantial return, there is no justification for making any deduction from the price
exhibited by the exemplars even if they are of small plots. It is possible that a part of the
acquired land might be used for construction of residential colony for the staff working in
the factory. Nevertheless where the remaining part of the acquired land is contributing to
production of goods yielding good profit, it would not be proper to make a deduction in
the price of land shown by the exemplars of small plots as the reasons for doing so
assigned in various decisions of this Court are not applicable in the case under
consideration.
15. Having regard to the entire facts and circumstances of the case, we are of the opinion
that a deduction of 10% from the market value of the land, which has been arrived at by
the High Court would meet the ends of justice. Therefore, the market value of the
acquired land for the purpose of payment of compensation to the land owners
@page-SC714
has to be assessed at Rs.1,08,000/- per acre.
16. In the result, the appeals are partly allowed. The claimant-appellants will be entitled
to compensation at the rate of Rs.1,08,000/- per acre. Besides the above amount, they will
also be entitled to the statutory sum in accordance with Section 23(1-A) and solatium at
the rate of 30% on the market value of the land in accordance with Section 23(2) of the
Act. They will also be entitled to interest as provided in Section 28 of the Act. The
appellants will be entitled to their costs.
Order accordingly.
AIR 2008 SUPREME COURT 714 "State of Chhattisgarh v. M/s. VTP Constructions"
(From : 2006(1) Cg. L. J. 308)
Coram : 2 Dr. A. PASAYAT AND AFTAB ALAM, JJ.
Civil Appeal No. 5679 of 2007 (arising out of SLP (C) No. 991 of 2007), D/- 7 -12
-2007.
State of Chhattisgarh and Ors.v. M/s. VTP Constructions.
M.P. Commercial Tax Act (5 of 1995), S.35 - Constitution of India, Art.245 -
LEGISLATIVE COMPETENCE - WORKS CONTRACT - INTER-STATE
TRADE/SALE - Works contract - Deduction at source of tax - S. 35 does not make any
provision for determination of value of goods supplied in course of inter-State trade
during execution of works contracts - Provision is beyond legislative competence of
State. (Para 10)
Cases Referred : Chronological Paras
2000 AIR SCW 708 : AIR 2000 SC 946 6, 7, 8, 10
2000 AIR SCW 958 : AIR 2000 SC 1268 6, 7, 8, 9, 10
(1996) 29 VKN 533 (MP) 5
AIR 1967 SC 1616 9
Rajesh Srivastava, for Appellants.
Judgement
1. Dr. ARIJIT PASAYAT, J. :-Leave granted.
2. Challenge in this appeal is to the judgment rendered by a Division Bench of the
Chhattisgarh High Court. Respondent filed a writ petition before the Chhattisgarh High
Court questioning constitutional validity of Section 35 of Chhattisgarh Vanijyik Kar
Adhiniyam, 1994 (hereinafter referred to as the 'Adhiniyam'). It corresponds to the
Chhattisgarh Commercial Tax Act, 1994 (in short the 'Act'). The respondent hereinafter is
described as the 'assessee'.
3. Background facts highlighted by the respondent are as follows:
The writ petitioner is a proprietary concern of one Shri Krishana Mudliar and it has been
executing works contracts for various Departments of the Chhattisgarh State Government
and others and was holding sales tax registration No.061/RDN/14, 2739/02. During the
assessment year 2001-2002, the writ petitioner had executed works contracts awarded by
Executive Engineer, P.W.D. (BandR), Division Khairagarh, for which it received payment
of Rs.1,27,115/- on which sales tax of Rs. - 2,545/-, being 2% of sum of Rs.1,27,115/-
was deducted at source towards the sales tax payable as provided under Section 35 of the
Adhiniyam. Certificate of tax deduction is dated 11/04/2001 made under Section 35 of
the Act.
4. In support of the writ petition, it was contended that Section 35 of the Act does not
make any provision for deduction and ascertainment of value and nature of goods
supplied during execution of work-contracts. Section 35 of the Act does not make any
provision for determination of value of goods supplied in the course of inter- State trade
during execution of works contracts.
5. The writ petition is opposed by the present appellants by filing reply/statement of
objections. In the reply statement, it was stated that the contractors who are engaged in
the construction of buildings, roads, bridges, dams etc. generally come from other States.
The process of assessment of sales tax is very lengthy and before the assessment is
completed, such contractors disappear from the scene after receiving full payment under
the contract. In such situation, it was very difficult for the Commercial Tax Department to
trace out such contractors and eventually sales tax payable by such contractors could not
be recovered at all thereby causing heavy financial loss to the Government. In order to
safeguard the interest of the State, Section 35 is enacted in the Act and that the State
Legislature has legislative competence to enact Section 35. It was stated that Section 35
is not a unique provision in the Act and similar provisions are enacted in the Sales Tax
Acts of other States, for example, Section 6-D of the Uttar Pradesh Trade Tax Act,
Section 6-E of the
@page-SC715
Bengal Finance (Sales Tax) Act, 1941; Section 25-A of the Bihar Finance Act, 1981;
Section 25-B of the Haryana General Sales Tax Act, 1973, Section 12-A of the Himachal
Pradesh General Sales Tax Act, 1968, Section 16-C of the Jammu and Kashmir General
Sales Tax Act, Section 19-A of the Karnataka Sales Tax Act and Section 13-AA of the
Orissa Sales Tax Act, 1947. It was also contended that the Constitutional validity of
Section 35 of the Adhiniyam was already considered and upheld by the Division Bench
of the Madhya Pradesh High Court in the case of Punj Lloyd Ltd. v. State of Madhya
Pradesh and Ors. (1996) 29 VKN 533.
6

. The High Court referred to decisions of this Court in Steel Authority of India Ltd. v.
State of Orissa and others [2000 (3) SCC 200) and M/s. Nathpa Jhakri Jt. Venture v. State
of Himachal Pradesh and Ors. [2000 (3) SCC 319] and declared the provision to be
unsustainable. 2000 AIR SCW 708
2000 AIR SCW 958

7. In support of the appeal learned counsel for the appellants submitted that the ambit and
scope of Section 35 of the Act were not kept in view by the High Court. The decision in
Steel Authority's case (supra) and Nathpa's case (supra) were dealing with different
provisions and, therefore, has no application to the facts of the case.
8

. Steel Authority's case (supra) related to Section 13AA of the Orissa Sales Tax Act, 1947
(hereinafter referred to as the 'Orissa Act'). In Nathpa Jhakri's case (supra) dispute related
to Section 12A of the Himachal Pradesh General Sales Tax Act, 1968 (in short the
'Himachal Pradesh Act'). While striking down Section 13AA of the Orissa Act, this Court
observed as follows in Steel Authority's case (supra) - : 2000 AIR SCW 708, (Para
13)
"There can be no doubt, upon a plain interpretation of Section 13AA, that it is enacted for
the purposes of deduction at source of the State sales tax that is payable by a contractor
on the value of a works contract. For the purpose of the deduction neither the owner nor
the Commissioner who issues to the contractor a certificate under Section 13AA(5) is
entitled to take into account the fact that the works contract involves transfer of property
in goods consequent upon of an inter-State sale, an outside sale or a sale in the course of
import. The owner is required by Section 13AA(1) to deposit towards the contractor's
liability to State sales tax four per cent of such amount as he credits or pays to the
contractor regardless of the fact that the value of the works contracts includes the value of
inter-State sales, outside sales or sales in the course of import. There is, in our view,
therefore, no doubt that the provisions of Section 13AA are beyond the powers of the
State Legislature for the State Legislature may make no law levying sales tax on inter-
State sales, outside sales or sales in the course of import."
9

. In Nathapa Jhakri's case (supra) this court held Section 12A of the Himachal Pradesh
Act to be unconstitutional and the relevant portion of the judgment reads as follows :
2000 AIR SCW 958, (Para 4)

"A bare perusal of the two provisions will make it clear that in either provision there is an
obligation to deduct from transactions relating to works contract on bills or invoices
raised by the works contractor an amount not exceeding 4 per cent or 2 per cent as the
case may be. Though the object of the provision is to meet the tax in respect of the
transactions on all works contract on the valuable consideration payable for the transfer
of property in goods involved in the execution of the works contract, the effect of the
provision is that, irrespective of whether the sales are inter-State sales or outside sales or
export sales which are outside the purview of the State Act and those transactions in
respect of which no tax can be levied even in terms of the enactment itself, such
deductions have to be made in the bills or invoices of the contractors. To say that if a
person is not liable for payment of tax inasmuch as on completion of the assessment
refund can be obtained at a later stage in no solace, as noticed in Bhawani Cotton Mills
Ltd. v. State of Punjab [(1967) 20 STC 290 (SC) : (1967) 3 SCR 577]. Further, there is no
provision for certification of the extent of the deduction that can be made by the
authority. Therefore, we must hold that arbitrary and uncanalised powers have been
conferred on the concerned person to deduct up to 4 per cent from the sum payable to the
works contract irrespective whether ultimately the transaction is liable for payment to any
sales tax at all. In that view of the matter, we have no hesitation in rejecting the
contention advanced on behalf of the State." AIR 1967 SC 1616

10. In view of what has been stated by this Court in Steel Authority's case and
@page-SC716
Nathpa's case (supra) the inevitable conclusion is that the High Court was right in holding
that Section 35 of the Act was constitutionally invalid. The direction for refund of the
amount collected from the respondent under the provisions of the said section had been
rightly directed to be refunded.
11. The appeal is sans merit and, therefore, deserves dismissal, which we direct. There
will be no order as to costs.
Appeal dismissed.
AIR 2008 SUPREME COURT 716 "Vinay Devanna Nayak v. Ryot Seva Sahakari Bank
Ltd."
(From : Karnataka)
Coram : 2 C. K. THAKKER AND MARKANDEY KATJU, JJ.
Criminal Appeal No. 1679 of 2007 (arising out of SLP (Cri.) No. 6908 of 2007), D/- 7
-12 -2007.
Vinay Devanna Nayak v. Ryot Seva Sahakari Bank Ltd.
Negotiable Instruments Act (26 of 1881), S.138, S.147 (as inserted by Negotiable
Instruments (Amendment and Miscellaneous Provisions) Act, 55 of 2002) - Criminal P.C.
(2 of 1974), S.320 - DISHONOUR OF CHEQUE - COMPOUNDING OF OFFENCE -
Dishonour of cheque - Compounding of offence - Technically provisions of S. 320, Cr. P.
C. do not apply to offences not covered by Indian Penal Code - However, considering
that object of S. 138 is to induce faith in efficacy of banking operations and credibility in
transacting business and also taking into account provision of S. 147, compounding of
offence should not normally be denied.
2000 AIR SCW 3483 (2000) 1 SCC 762; (2003) 9 SCC 214; (2004) 13 SCC 494; 2005
AIR SCW 3358; (2005) 10 SCC 632 and 1996 AIR SCW 840, Relied on.
Crl. R. P. No. 1003 of 2005, D/- 20-12-2005 (Kar.), Reversed. (Paras 15, 17, 18, 19)
Cases Referred : Chronological Paras
2005 AIR SCW 3358 : 2005 Cri LJ 4878 (Rel. on) 15
(2005) 10 SCC 632 (Rel. on) 15
(2004) 13 SCC 494 (Rel. on) 15
(2003) 9 SCC 214 (Rel. on) 15
2000 AIR SCW 3483 : AIR 2000 SC 3145 : 2000 Cri LJ 4673 (Rel. on) 14
(2000) 1 SCC 762 (Rel. on) 15
1996 AIR SCW 840 : AIR 1996 SC 2339 : 1996 Cri LJ 1692 (Rel. on) 17
S. Nanda Kumar, Ananda Selvam and V. N. Raghupathy, for Appellant; Ms. Renuga Devi
and P. Narashiman, for Respondent.
Judgement
1. C. K. THAKKER, J.:-Delay condoned. Leave granted.
2. The present appeal is filed against an order passed by the Court of Civil Judge (Jr.
Divn.) and Judicial Magistrate First Class, Ankola on April 12, 2004 in Criminal Case
No.73 of 2001, confirmed by the Sessions Judge, Fast Track Court-I, Karwar on March
24, 2005 in Criminal Appeal No. 50 of 2004 as also confirmed by the High Court of
Karnataka, Bangalore on December 20, 2005 in Criminal Revision Petition No.1003 of
2005.
3. Brief facts of the case are that the appellant herein was a member of Ryot Sewa
Sahakari Bank Ltd., Basgod, Taluka Ankola ('Complainant Bank' for short). He had
obtained a loan of Rs. 20,000/- from the Complainant-Bank on April 3, 1998 for business.
The amount was not paid by the appellant. The appellant issued a cheque of Rs. 24,000/-
on October 13, 2000 in favour of the Complainant-Bank and assured the Bank that it
would be honoured. But when the cheque was submitted for clearance, it was
dishonoured and returned to the drawer on December 22, 2000 with endorsement; "Loan
account due date is over and account is not in operation". The complainant, therefore,
issued a registered legal notice on December 26, 2000 which was duly served upon the
accused-loanee on December 30, 2000. In spite of the notice, no payment was made by
the accused and hence a criminal case was filed by the Bank against him under the
Negotiable Instruments Act, 1881 (hereinafter referred to as 'the Act'). A summons was
issued to the accused for an offence punishable under Section 138 of the Act. He pleaded
not guilty to the charge and claimed to be tried.
4. The Trial Court on the basis of evidence adduced by the complainant-Bank, held that
the accused had issued a cheque of Rs. 24,000/- which was dishonoured and even after
receipt of legal notice, he failed to pay the amount and thereby he had committed an
offence punishable under S. 138 of the Act. The Court, hence, convicted the accused and
ordered him to undergo imprisonment for a period of six months. The accused was also
ordered to pay a sum of Rs. 48,000/- as compensation within one month from the date of
the order. The Court
@page-SC717
ordered that out of the said amount of compensation, Rs. 43,000/- should be paid to the
complainant towards the compensation and Rs.5,000/- to be appropriated to the State. In
default of payment of compensation, the accused was ordered to undergo imprisonment
for a period of six months.
5. Being aggrieved by the order of conviction and sentence, the appellant preferred an
appeal. The Appellate Court confirmed the order of conviction and sentence recorded by
the Trial Court. It, however, reduced the amount of compensation from Rs. 43,000/- to
Rs. 30,000/- and fine from Rs. 5,000/- to Rs. 3,000/-. Order of default-sentence was
maintained.
6. The aggrieved accused invoked Revisional Jurisdiction of the High Court under
Section 401 read with Section 397 of the Code of Criminal Procedure, 1973. The High
Court on July 13, 2005, passed an interim order directing the accused to deposit the
compensation amount in the Court. But the accused failed to comply with the said order.
When the matter came up for hearing, the High Court, by the impugned order dated
December 20, 2005 dismissed the Revision Petition observing that there was no ground
to interfere with the order passed by the Trial Court and confirmed by the First Appellate
Court. It also observed that the petitioner-accused had not complied with the interim
order passed on July 13, 2005. Thus, there was no reason to admit the revision petition
and accordingly it was dismissed. The accused has challenged that order in this Court.
7. On November 12, 2007, the matter was placed for admission-hearing. It was stated by
the Learned Counsel for the appellant that the appellant intended to pay the amount.
Accordingly, notice was issued to the other side. Pursuant to the notice, the respondent-
Bank appeared and affidavit is filed by General Manager, Ryot Sewa Sahakari Bank
Limited, Basgod, wherein it was stated that the appellant had paid an amount of Rs.
45,000/- towards final settlement of the claim of the respondent-Bank on July 25, 2007
and the Bank had no other claim against the appellant and the matter has been settled
amicably.
8. We have heard the Learned Counsel for the parties. The Learned Counsel for the
appellant submitted that since the matter has been amicably settled between parties and
the amount of Rs. 45,000/- has been paid to the Bank towards 'full and final settlement'
and no further claim has remained, the compromise may be recorded, the appeal may be
allowed and appellant-accused may be ordered to be acquitted of the charge levelled and
conviction recorded against him by setting aside conviction as well as sentence.
9. The Learned Counsel for the respondent-bank admitted that there was a compromise
between the parties and an amount of Rs. 45,000/- had been accepted by the Bank
towards final settlement of the dues against the appellant and no further claim has been
put forward by the respondent-Bank.
10. In view of the fact that the matter has been settled and the amount of Rs. - 45,000/-
has been paid by the appellant and accepted by the Bank as 'full and final settlement' and
there are no further dues by the Bank, prima facie, there should be no objection to grant
the prayer of the accused and acquit him of the offence with which he was charged and
convicted by the Courts below.
11. It is no doubt true that every crime is considered to be an offence against the society
as a whole and not only against an individual even though an individual might have
suffered thereby. It is, therefore, the duty of the State to take appropriate action against
the offender. It is equally the duty of a Court of law administrating criminal justice to
punish a criminal. But there are offences and offences. Certain offences are very serious
in which compromise or settlement is not permissible. Some other offences, on the other
hand, are not so serious and the law may allow the parties to settle them by entering into
a compromise. The compounding of an offence signifies that the person against whom an
offence has been committed has received some gratification to an act as an inducement
for his abstaining from proceeding further with the case.
12. So far as the Code of Criminal Procedure is concerned Section 320 deals with
offences which are compoundable, either by the parties without the leave of the Court or
by the parties but only with the leave of the Court. Sub-section (1) of Section 320
enumerates the offences which are compoundable without the leave of the Court, while
sub-section (2) of the said section specifies the offences which are compoundable with
@page-SC718
the leave of the Court. Sub-section (9) of Section 320 declares; "No offence shall be
compounded except as provided by this section". It is thus clear that offences not referred
to in sub-sections (1) and (2) of Section 320 and not included in the Table are not
compoundable. Similarly, offences punishable under laws other than the Indian Penal
Code also cannot be compounded.
13. In the circumstances, a question may arise whether an offence punishable under
Section 138 of the Act which is a special law can be compounded. Whereas some High
Courts held that if the matter is settled between the parties, the offence can be
compounded, other High Courts took a contrary view.
14

. In Cranex Ltd. and Anr. v. Nagarjuna Finance Ltd. and Anr., (2000) 7 SCC 388, a
settlement had been entered between the parties during the pendency of appeal in
Sessions Court against an order of conviction and sentence recorded by the Magistrate
under section 138 of the Act. This Court directed the Appellate Court to consider the
settlement and to take appropriate action in accordance with law. 2000 AIR SCW 3483

15
. In O.P. Dholkia v. State of Haryana and Anr., (2000) 1 SCC 762, an order of conviction
recorded by the Trial Court was upheld by the Appellate as well as Revisional Court.
Thereafter, however, a compromise had been arrived at between the parties and the entire
amount was paid to the complainant. It was, therefore, submitted before this Court that
the accused may be acquitted. The Learned Counsel appearing for the State urged that
when the conviction and sentence had been maintained by all Courts, this Court need not
show any indulgence. Though the Court observed that there was 'some force' in the said
contention, taking into account the nature of offence and the fact that the complainant had
compromised the matter, permission was granted 'in the peculiar facts and circumstances'
of the case [see also Nambiram Veetil Pocker v. State of Kerala and Anr., (2003) 9 SCC
214]. It is thus clear that even though technically the provisions of Section 320 of the
Code of Criminal Procedure did not apply to offences not covered by the Indian Penal
Code, the fact as to compromise between the parties and payment of dues under Section
138 of the Act was considered a relevant fact and compounding was allowed by the Court
[vide Kishore Kumar v. J.K. Corporation Ltd., (2004) 13 SCC 494; Shailesh Shyam
Parsekar v. Baban @ Vishwanath, (2005) 4 SCC 162; K.J.B.L. Rama Reddy v.
Annapurna Seeds and Anr., (2005) 10 SCC 632]. 2005 AIR SCW 3358

16. Section 138 of the Act was inserted by the Banking, Public Financial Institutions and
Negotiable Instrument Law (Amendment) Act, 1988 (Act 66 of 1988) to regulate
financial promises in growing business, trade, commerce and industrial activities of the
country and the strict liability to promote greater vigilance in financial matters. The
incorporation of the provision is designed to safeguard the faith of the creditor in the
drawer of the cheque, which is essential to the economic life of a developing country like
India. The provision has been introduced with a view to curb cases of issuing cheques
indiscriminately by making stringent provisions and safeguarding interest of creditors.
17

. As observed by this Court in Electronic Trade and Technology Development


Corporation Ltd. v. Indian Technologists and Engineers, (1996) 2 SCC 739, the object of
bringing Section 138 in the statute book is to inculcate faith in the efficacy of banking
operations and credibility in transacting business on negotiable instruments. The
provision is intended to prevent dishonesty on the part of the drawer of negotiable
instruments in issuing cheques without sufficient funds or with a view to inducing the
payee or holder in due course to act upon it. It thus seeks to promote the efficacy of
banking operations and ensures credibility in transacting business through cheques. In
such matters, therefore, normally compounding of offences should not be denied.
Presumably, Parliament also realized this aspect and inserted Section 147 by the
Negotiable Instruments (Amendment and Miscellaneous Provisions) Act, 2002 (Act 55 of
2002). The said section reads thus: 1996 AIR SCW 840

S.147. Offences to be compoundable.- Notwithstanding anything contained in the Code


of Criminal Procedure, 1973 (2 of 1974), every offence punishable under this Act shall be
compoundable.
18. Taking into consideration even the said provision (Section 147) and the primary
object underlying Section 138, in our judgment, there is no reason to refuse compromise
between the parties. We, therefore,
@page-SC719
dispose of the appeal on the basis of the settlement arrived at between the appellant and
the respondent.
19. For the foregoing reasons the appeal deserves to be allowed and is accordingly
allowed by holding that since the matter has been compromised between the parties and
the amount of Rs. 45,000/- has been paid by the appellant towards full and final
settlement to the respondent-Bank towards its dues, the appellant is entitled to acquittal.
The order of conviction and sentence recorded by all courts is set aside and he is
acquitted of the charge levelled against him.
20. Ordered accordingly.
Appeal allowed.
AIR 2008 SUPREME COURT 719 "S. Bagirathi Ammal v. Palani Roman Catholic
Mission"
(From : 2001 AIHC 3826 (Mad.))
Coram : 3 B. N. AGRAWAL, PRAKASH PRABHAKAR NAOLEKAR AND P.
SATHASIVAM, JJ.
Civil Appeal Nos. 78-79 of 2002, D/- 6 -12 -2007
S. Bagirathi Ammal v. Palani Roman Catholic Mission.
(A) Madras City Tenants Protection Act (3 of 1922), S.1(3)(f) (as amended by Act 2 of
1996) - TENANCY - APPLICABILITY OF AN ACT - RELIGIOUS AND
CHARITABLE ENDOWMENTS - WORDS AND PHRASES - Applicability - Religious
institution - Palani Roman Catholic Mission - Is "religious institution" within meaning of
S. 1(3)(f) - Proceedings instituted against it - Would abate. (Paras 6, 10)
(B) Madras City Tenants Protection Act (3 of 1922), S.1(3)(f) (as amended by Act 2 of
1996) - ABATEMENT - EVICTION - AMENDMENT - SALE DEED - Eviction
proceedings - Tenant exercising option of purchasing sites from landlord - Landlord
falling in category of "religious institution" after enactment of Amendment Act - Thus
proceedings for purchase of property instituted by tenant would abate - Therefore,
execution of sale deed by court in favour of tenant much after enforcement of
Amendment Act - Would be nullity.
AIR 2006 (NOC) 965 : 2006 (5) Mad LJ 281 (FB), Overruled. (Paras 10, 11, 12)
(C) Civil P.C. (5 of 1908), O.47, R.1 - REVIEW - TENANCY - AMENDMENT - HIGH
COURT - Review - Apparent error - Proceedings for purchase of tenanted property by
tenant - Pendency - Benefits conferred by Amendment Act on landlord, religious
institutions during pendency of Proceedings - Order not giving benefits - Held, granting
benefits by High Court under amended Act by allowing review applications, was proper.
(Para 12)
Cases Referred : Chronological Paras
AIR 2006 (NOC) 965 : 2006 (2) Mad LJ 281 (FB) (Overruled) 11
2005 AIR SCW 6170 : AIR 2006 SC 523 (Ref.) 9
2000 AIR SCW 2608 : AIR 2000 SC 2587 (Ref.) 2
(2000) 3 Mad LJ 616 (FB) 9
M. N. Krishnamani, Sr. Advocate, B. Sri-dhar, Ms. I. Madhavi and K. Ram Kumar, for
Appellant; P. - P. Rao, Sr. Advocate, V. J. Francis, Anupam Mishra, Ms. Jessy Kurien, D.
S. Chadha, Purushottam T. S., Abhishek Gupta and Ms. Sahar Bakht, for Respondent.
Judgement

P. SATHASIVAM, J. :- Challenging the order dated 20.7.2001 passed by the High Court
of Judicature at Madras in Review Application Nos. 8 and 9 of 1997 filed by the
respondent herein whereby a learned single Judge of the High Court allowed the same,
the appellant has filed these appeals. reported in 2001 AIHC 3826

. The respondent herein was the owner of the suit vacant land in question. In 1959, the
suit land was leased out for five years by the respondent to the appellant herein. On
3.3.1965, the tenancy was renewed for another period of three years. After the expiry of
three years, the respondent wanted the appellant to vacate the premises. As the appellant
did not vacate, the respondent issued a notice on 28.8.1968 demanding possession for
which he sent a reply with false and frivolous allegations. In the year 1969, the
respondent filed O.S.No.218 of 1969 for recovery of land. The appellant also filed
O.S.No. 75 of 1970 as a counter-blast for getting a fresh lease document from the
respondent. On 14.12.1970, O.S.No. 218 of 1969 was compromised and O.S.No. 75 of
1970 was dismissed as not pressed. The appellant did not vacate the suit property in spite
of repeated demands by the respondent, therefore, the respondent filed a fresh Suit i.e.
O.S.No. 76 of 1977 for delivery of possession. On 27.7.1978, O.S. No.76 of 1977 was
decreed in favour of the respondent while O.P. No. 4 of 1977 filed by the appellant for
purchase of the land by her was erroneously dismissed and an order of eviction was
passed against the appellant 2000 AIR SCW 2608

@page-SC720
by the Court of District Munsif, Palani. The Madras City Tenants Protection Act, 1921
gives the option of purchasing the site from the landlord by the tenant in case a suit for
eviction is filed by the landlord where the tenant is the owner of the superstructure
standing thereon and if the tenant is not interested in buying the site then the landlord can
buy the superstructure or ask the tenant to remove the superstructure and seek delivery of
possession. The said Act was extended to the town of Palani in Tamil Nadu only in 1975,
therefore, the option of buying the site from the respondent became available to the
appellant as the owner of the superstructure. The appellant filed an application O.P. No. 4
of 1977 in Suit No. 76 of 1977 for purchase of land by him which was dismissed. Against
the said order, the appellant filed A.S.No. 121 of 1978 and another A.A.O. No. 94 of
1978 against the order in O.P. No. 4 of 1977. The appellate Court allowed the appeals of
the appellant directing the respondent to sell the land to the appellant for an amount of
Rs. 65,092.50. Aggrieved by the said order, the respondent filed S.A.No. 2149 of 1981
and C.R.P. No. 2204 of 1980 against the order allowing the petition of the appellant for
purchase of the suit property. The second appeal and the revision petition filed by the
respondent were dismissed by a learned single Judge of the High Court of Madras.
Against that order, the respondent filed S.L.P.(C) Nos. 5029 and 5030 of 1984 before this
Court which were dismissed. After the stay order operating from 1980 to 1985
continuously ceased to operate, the appellant deposited the full site value. With the
dismissal of the S.L.Ps. by this Court and the deposit of the full site value by the
appellant, the same became final. In the year 1985, the appellant filed an execution
petition being E.P. No. 257 of 1985 for execution of the sale deed of the land in his
favour by the respondent. On the other hand, the respondent filed an execution petition
being E.P. No. 79 of 1983 for executing the compromise decree in O.S.No.218 of 1969.
Both the petitions were taken up together for disposal. The Executing Court allowed E.P.
No. 257 of 1985 filed by the appellant for execution of the sale deed and dismissed E.P.
No. 79 of 1983 filed by the respondent. Dissatisfied therewith, the respondent filed C.R.P.
No. 1445 of 1988 against the order in E.P. No. 79 of 1983 and A.A.O. No. 767 of 1989
against the order in E.P. No. 257 of 1985. Both the petitions were heard together and the
same were dismissed by the High Court upholding the directions of the Execution Court
to execute the sale deed in favour of the appellant. On 28.10.1996, the sale deed in favour
of the appellant was executed by the Court of District Munsif, Palani. The sale deed was
registered as Document No. 1908 of 1996 in the Registrar's office. Against the order in
C.R.P. No. 1445 of 1988 and A.A. O. No. 767 of 1989, the respondent filed S.L.P.(C)
Nos. 22925 and 22926 of 1996 before this Court which were dismissed. After every thing
became final with the execution and registration of the sale deed in favour of the
appellant and the dismissal of the S.L.Ps. by this Court, the respondent filed review
applications being Review Application Nos. 8 and 9 of 1997 in the High Court against the
order dated 26.7.1997 passed by the High Court on the same grounds as were in S.L.Ps.
and the same were dismissed on the ground that they were not maintainable after the
dismissal of the S.L.Ps. by this Court. On 7.2.1997, the appellant filed an application
E.A.No. 820 of 1996 for return of the duly registered sale deed and the same was
allowed. The respondent was not a party to the said E.A. and he did not make any effort
to implead himself. Against the said order, the Registrar who was a party filed C.R.P. No.
1819 of 1997. In the said C.R.P., the respondent filed an application being C.M.P. No.
3005 of 1998 to implead himself which was dismissed by the High Court. The C.R.P.
filed by the Registrar was dismissed and the Registrar returned the sale deed to the
executing Court. Against the order dated 16.12.1998 in the R.A.Nos. 8 and 9 of 1997, the
respondent filed S.L.P.(C) Nos. 6097 and 6098 of 1999 before this Court. On 2.2.2001,
this Court passed an order of remand of the review applications in S.L.P.(C) Nos. 6097-
6098 of 1999 because of the decision of this Court in Kunhyammed v. State of Kerala,
(2000) 6 SCC 359, holding that the summary dismissal of a special leave petition does
not bar a review petition permissible under the law. The respondent filed an application in
the review applications for producing additional documents which was allowed by a
learned single Judge of the High Court. As a result of the order of the High Court, the
proceedings for return of the registered sale deed to the appellant was dismissed by the
executing Court. Aggrieved by the said order, these appeals have been preferred by the
appellant.
@page-SC721
3. Heard Mr. M.N. Krishnamani, learned senior counsel appearing for the appellant and
Mr. P. P. Rao, learned senior counsel appearing for the respondent.
4. The only point for consideration in these appeals is whether the High Court is justified
in allowing Review Application Nos. 8 and 9 of 1997 under Order XLVII, Rule 1, C.P.C.
5. Since we have already narrated the case of both the parties in the paragraphs supra,
there is no need to traverse the same once again. Before considering the rival claims
made by both the parties, it is useful to refer the provisions under Order XLVII, Rule 1,
C.P.C. relating to Review which read as under:
"1. Application for review of judgment:- (1) Any person considering himself aggrieved-
(a) by a decree or order from which an appeal is allowed, but from which no appeal has
been preferred,
(b) by a decree or order from which no appeal is allowed, or
(c) by a decision on a reference from a Court of Small Causes,
and who, from the discovery of new and important matter or evidence which, after the
exercise of due diligence was not within his knowledge or could not be produced by him
at the time when the decree was passed or order made, or on account of some mistake or
error apparent on the face of the record, or for any other sufficient reason, desires to
obtain a review of the decree passed or order made against him, may apply for a review
of judgment to the Court which passed the decree or made the order.
(2) A party who is not appealing from a decree or order may apply for a review of
judgment notwithstanding the pendency of an appeal by some other party except where
the ground of such appeal is common to the applicant and the appellant, or when, being
respondent, he can present to the Appellate Court the case on which he applies for the
review.
[Explanation- The fact that the decision on a question of law on which the judgment of
the Court is based has been reversed or modified by the subsequent decision of a superior
Court in any other case, shall not be a ground for the review of such judgment.]"
A reading of the above provision makes it clear that Review is permissible (a) from the
discovery of new and important matter or evidence which, after the exercise of due
diligence could not be produced by the party at the time when the decree was passed; (b)
on account of some mistake; (c) where error is apparent on the face of the record or is a
palpable wrong; (d) any other sufficient reason. If any of the conditions satisfy, the party
may apply for a review of the judgment or order of the Court which passed the decree or
order. The provision also makes it clear that an application for Review would be
maintainable not only upon discovery of a new and important piece of evidence or when
there exists an error apparent on the face of the record but also if the same is necessitated
on account of some mistake or for any other sufficient reason. An error contemplated
under the Rule must be such which is apparent on the face of the record and not an error
which has to be fished out and searched. In other words, it must be an error of
inadvertence. It should be something more than a mere error and it must be one which
must be manifest on the face of the record. When does an error cease to be mere error and
becomes an error apparent on the face of the record depends upon the materials placed
before the Court. If the error is so apparent that without further investigation or enquiry,
only one conclusion can be drawn in favour of the appellant, in such circumstances, the
review will lie. Under the guise of review, the parties are not entitled to re-hearing of the
same issue but the issue can be decided just by a perusal of the records and if it is
manifest can be set at right by reviewing the order. With this background, let us analyze
the impugned judgment of the High Court and find out whether it satisfy any of the tests
formulated above.
6. It is the claim of the respondent herein that it is a Roman Catholic Mission and is a
religious institution within the meaning of Amended provisions of The Tamil Nadu City
Tenants Protection Act, 1921 (hereinafter referred to as "the Principal Act). The "Roman
Catholic Mission" (hereinafter "Mission" in short), in support of the above claim, filed
several documents, namely, Ex. A-1 to A-15 and also let in evidence of PW-1 and PW-2
who were conversant with their activities. The High Court, on appreciation of those
materials, arrived at a factual conclusion that the same came under the Roman
@page-SC722
Catholic Diocese which has as its object and maintenance of churches and hence it is a
"religious institution". Though it was contended by the learned senior counsel appearing
for the appellant herein that only certificate of registration was produced by the Mission
to substantiate its case that it is a religious institution, in view of the categorical factual
finding by the High Court based on acceptable oral and documentary evidence, we reject
the said objection. It is relevant to point out that when the above appeals were heard on
19.01.2006 at length, this Court after finding that it would be just and expedient to call
for a finding from the trial Court as to whether the Palani Roman Catholic Mission is a
'religious institution' or 'institution of religious charity' belonging to Hindu, Muslim,
Christian or other religion within the meaning of Section 1(3)(f) of the Madras City
Tenants Protection Act, 1921, as amended by Act 2 of 1996, directed the trial Court to
record a finding on the said question after giving opportunity of adducing oral and
documentary evidence to the parties and thereupon remit its finding to this Court within a
period of six months from the date of receipt of copy of the said order. Pursuant to the
said direction, this Court received a report dated 09.08.1996 from the trial Court i.e.
District Munsiff, Palani and the same was handed over to learned counsel appearing on
behalf of the parties. They were given an opportunity to peruse the report and submit
their objection, if any. The report shows that the learned District Munsiff, after affording
opportunity to both parties and after recording evidence and relying on documents placed
by both parties, arrived at the following conclusion:-
"......... ... Thus, on cumulative appraisal of the evidence on record in the context of the
undisputed averments of the proof-affidavit of the P.W.1 and P.W.2 in particular P.W.2
with regard to the factum of conducting the religious ceremonies, prayers and masses in
the plaintiff-Mission, this Court feels that an inescapable and irresistible conclusion can
be drawn that the plaintiff-mission is a place of worship for the people who have faith in
the Christianity, particularly believers attached with the Roman Catholic denomination.
Therefore, in view of the finding as above, this Court hold that the Palani Roman
Catholic Mission is a religious institution in the context of the Section 1(f) of the Madras
City Tenants Protection Act, 1921 as amended by Act 2 of 1996.
In the result, on the basis of the evidence emerged on record, I hold that the Palani
Roman Catholic Mission is a religious institution within the meaning of Section 1(f) of
the Madras City Tenants Protection Act, 1921 as amended by the Act 2 of 1996."
In view of the conclusion of the High Court as well as the report of the trial Court holding
that Palani Roman Catholic Mission-respondent herein is a religious institution within the
meaning of Section 1 (f) of the principal Act as amended by Madras City Tenants
Protection Act as amended by Act 2 of 1996. We agree with the said conclusion.
7. Now we will consider the provisions of the Principal Act as well the provisions of the
Amendment Act i.e., the Madras City Tenants Protection (Amendment) Act, 1994 (Tamil
Nadu Act 2 of 1996) [hereinafter referred to as "the amended Act"]. The Statement of
Objects and Reasons of the Act shows that in many parts of the City of Madras (and other
Municipal towns) dwelling houses and other buildings have from time to time been
erected by tenants on lands belonging to others, in the expectation that subject to payment
of a fair ground rent they would be left in their undisturbed possession, in spite of any
agreement about duration of the tenancy and the terms on which the buildings were to be
leased. Attempts made or steps taken to evict a large number of such tenants had shown
that such expectations are likely to be defeated. The tenants, if they were evicted, can
remove the superstructure which can only be done by pulling down the building, or claim
compensation for the value of the building put up by them and the value of any tree
planted by them. As a result of such wholesale destruction, congested parts of the city
(municipal towns) would become more congested to the serious detriment of public
health. In the circumstances, it was though just and reasonable that the landlords when
they evict the tenants should pay for and take the buildings. There may however be cases
where the landlord is unwilling to eject a tenant, if he can get a fair rent for the land. The
Act provides for the payment of compensation to the tenant in case of ejectment for the
value of any buildings which may have been erected by him, or his predecessor-in-
interest. It also provides for the settlement of fair rent at the instance of the
@page-SC723
landlord, or tenant. Provision is also made to enable the tenant to purchase the land in his
occupation, subject to certain conditions.
8. Section 9 gives the right to the tenant, who has put up a superstructure to purchase
such part or extent of the land, be reasonably required for his enjoyment. Since we are
concerned about the Amended Act, there is no need to go into other provisions. The
Amended Act received the assent of the President on 5.1.1996 and published in the Tamil
Nadu Government Gazette Extraordinary Part IV, Section 2 dated 11.1.1996. By
Amendment Act, the Tamil Nadu legislature has amended Section 1 of the principal Act
and added certain provisions in sub-section (3). The amended provisions are as follows:
"Amendment of Section 1 - In Section 1 of the Madras City Tenants' Protection Act, 1921
(Tamil Nadu Act III of 1922), (hereinafter referred to as the principal Act), in sub-section
(3), in the first proviso; after clause (e), the following clause shall be added, namely:-
(f) by any religious institution or religious charity belonging to Hindu, Muslim, Christian
or other religion.
Explanation:- for the purpose of this clause-
(A) "religious institution" means any-
(i) temple
(ii) math;
(iii) mosque
(iv) church; or
(v) other place by whatever name known
which is dedicated to, or for the benefit of, or used as of right by, any community or
section thereof as a place of public religious worship;
(B) "religious charity" means a public charity associated with a religious festival or
observance of religious character (including a wakf associated with a religious festival or
observance of religious character), whether it be connected with any religious institution
or not."
3. Certain pending proceedings to abate- Every proceeding instituted by a tenant in
respect of any land owned by any religious charity belonging to Hindu, Muslim,
Christian or other religion and pending before any Court or other authority or officer on
the date of the publication of this Act in the Tamil Nadu Government Gazette, shall in so
far as the proceeding relates to any matter falling within the scope of the principal Act, as
amended by this Act, in respect of such land, abate and all rights and privileges which
may have accrued to that tenant in respect of any such land and subsisting immediately
before the said date shall in so far as such rights and privileges relate to any matter falling
within the scope of the principal Act, as amended by this Act, cease and determine and
shall not be enforceable:
Provided that nothing contained in this section shall be deemed to invalidate any suit or
proceeding in which a decree or order passed has been executed or satisfied in full before
the said date."
We have already mentioned that the amendment was published in the Gazette on
11.1.1996 and as per sub-section (2) of Section 1, it came into force on the date of
publication. In other words, from 11.1.1996 benefits conferred on the tenants under
Section 9 of the Principal Act have been deleted in respect of the lands belonging to
religious institution or religious charity of Hindu, Muslim, Christian or other religion. We
have already referred to the finding of the High Court holding that the respondent herein
is a Roman Catholic Mission which is a "religious institution" within the meaning of the
amended provision. The Amended Act has given the respondent herein a valuable right of
exemption from the provisions of the Principal Act.
9

. It is relevant to mention here that the Amendment Act No. 2 of 1996 has been upheld by
the Full Bench of the High Court in N. Sreedharan Nair vs. State of Tamil Nadu, (2000) 3
MLJ 616, and the said decision of the Full Bench has also been approved by this Court by
dismissing C.A.Nos. 4531 of 2003 etc.etc. titled Mylapore Club vs. State of T.N. and Anr.
2005 (5) CTC 494, filed against the same. 2005 AIR SCW 6170

10. Both before the High Court as well as before this Court, it was contended that in view
of the orders/decisions of various Courts including this Court, the issue cannot be agitated
once again by way of review application; hence, the impugned order of the High Court is
to be set aside. Mr. P.P. Rao, learned senior counsel appearing for the respondent, has
brought to our notice that in the earlier proceedings, this Court in Civil Appeal Nos.
1055-1056 of 2001 directed the High Court to consider the
@page-SC724
review applications afresh. In other words, by virtue of the said order, the High Court was
directed to decide the review applications on merits. In such circumstances, the High
Court was fully justified in analyzing the issue as directed by this Court and its ultimate
decision that Roman Catholic Mission is a "religious institution" cannot be faulted with
since it relied on acceptable materials in the form of oral and documentary evidence (vide
Ex. A-1 to A-15 and evidence of PW-1, PW-2). It was demonstrated that these religious
and charitable institutions were not only deprived of their legitimate income but also their
valuable properties. It was also their claim that because of the provision, namely, Section
9 of the Act, the tenants flourished and the landlord-institutions were crippled. It was
further pointed out that in those circumstances Act No. 2 of 1996 was enacted in order to
protect those religious institutions. We have already concluded that pleadings of the
respondent herein-review petitioners and various orders/judgments show that it is a
"religious institution". As rightly observed by the High Court, the claim that the
"Mission" is a "religious institution" is apparent from the materials without any further
investigation. In such circumstances, as per Section 1(f) of the amended Act, all
proceedings instituted by a tenant would abate. The amended Act came into force from
11.1.1996 and on the question whether on the date of coming into force of the amended
Act, giving certain benefits to the religious institutions and taking away the right of the
tenant under Section 9, the High Court concluded as under:
"........The Transfer C.M.A. which was a continuation of the application under Section 9
of the Tamil Nadu City Tenants Protection Act filed by the respondent who is the tenant
was still pending. The proceedings had not attained finality. Therefore, they terminated
and they became unenforceable. On the date when the first appeal and the C.M.A. were
disposed of, tenancy granted by religious institutions were still governed by the
provisions of the Act. Now, by the introduction of Act 2 of 1996, they cease to apply,
ergo, all proceedings instituted by the tenant shall abate. All rights and privileges that
may have accrued to her cease. They come to an end and they shall not be enforceable.
The jurisdiction of the Court to decide the tenant's claim ceased."
It is clear that on the date when the amended Act came into force, the application under
Section 9 of the principal Act filed by the tenant-appellant herein was still pending.
Though Mr. M.N. Krishnamani, learned senior counsel appearing for the appellant,
submitted that all formalities were completed before coming into force of the amended
Act, as pointed out earlier, pursuant to the order of the High Court, the sale deed was
executed only on 28.10.1996 whereas the amended Act (Act No.2 of 1996) came into
force on 11.1.1996 much earlier to the execution of the sale deed, hence, the contention
of learned senior counsel for the appellant is not acceptable and we are in agreement with
the conclusion arrived at by the High Court. As rightly concluded by the High Court, the
decree in O.P. No. 4 of 1977 became a nullity on and from 11.1.1996, the executing Court
committed an error in executing the sale deed after coming into force of amended Act.
Further as rightly observed by the High Court, unless the sale deed is executed either by
the Mission or by the Court, the fruits of the decree will not be realized by the tenants and
the proceedings will come to an end only upon execution of the sale deed. Therefore, the
tenant cannot be heard to say that the proviso applies to him and that the proceedings are
not invalidated. The High Court is right in holding that the decree not having been
executed by means of a sale deed, the proceedings are deemed to be pending and,
therefore, were determined with the coming into force of the amendment Act.
11

. Finally, Mr. M.N.Krishnamani placing reliance on the Full Bench decision of the
Madras High Court rendered in CRP(NPD) 2758 of 1996 titled Arulmigu Kasi
Viswanathaswamy Devasthanam v. Kasthuriammal, submitted that the moment tenant
deposited the amount the order is fully satisfied. He further pointed out that as per the
said decision the moment the order under Section 9 (3) (a) is passed, it shall be construed
that the proceedings got terminated and the suit stood dismissed as per Section 9 (3) (b)
of the Act. We are unable to accept the said proposition. The relevant provisions are as
follows:- reported in AIR 2006 (NOC) 965 (Mad)

"9. (3) (a) On payment of the price fixed under clause (b) of sub-section (1) the Court
shall pass an order directing the conveyance by the landlord to the tenant of the extent of
land for which the said price was fixed.
@page-SC725
The Court shall by the same order direct the tenant to put the landlord into possession of
the remaining extent of the land, if any, the stamp duty and registration fee in respect of
such conveyance shall be borne by the tenant.
(b) On the order referred to in clause (a) being made, the suit or proceeding shall stand
dismissed, and any decree or order in ejectment that may have been passed therein but
which has not been executed shall be vacated."
It is clear that if the tenant complies with the order passed under Section 9 (1) (b) and
deposits the amount within the time as fixed, the Court has to pass an order directing the
conveyance by the landlord to the tenant. It is true that as per Section 9 (3) (b) on passing
an order under clause (a) the suit or proceeding shall stand dismissed. In the light of the
language used in clause (a) i.e. "conveyance" to be made by the landlord to the tenant, till
the proper document conveying title to the tenant it is presumed that the proceeding is
kept pending. To put it clear that unless the sale deed is executed by the landlord in
favour of the tenant or in the alternative by the Court on behalf of the landlord the fruits
of the decree cannot be realized. The suit or proceeding will come to an end immediately
on execution of sale deed either by the landlord or by the Court on behalf of the landlord.
In our case, as said earlier, the sale deed was executed only on 28.10.1996, however, the
amended Act 2/96 came into force on 11.01.1996 much earlier to the execution of sale
deed. The view expressed in the Full Bench decision runs counter to the language used in
the statute and we are unable to accept the same.
12. From the materials, we are satisfied that the conclusion reached by the High Court
holding that the review petitioner/respondent herein is a "religious Mission"/ "institution"
within the meaning of amended provision and entitled to the benefits of amended Act.
Further if the same is not applied to the Mission, it would result in miscarriage of justice
and it had been rightly rectified by the High Court by the impugned judgment. The
benefit that has been bestowed upon the religious institution by the Legislature cannot be
ignored lightly merely because the issue was decided by way of review applications.
Inasmuch as at the relevant point of time, the Amended Act 2 of 1996 was not enacted
and not available for consideration before the Court and also of the fact that the
proceeding instituted by the tenant/appellant herein was pending and not reached finality
on the date of coming into force of the amended Act, we are satisfied that the High Court
is justified in granting the relief as provided under the amended Act (Act No.2 of 1996)
by allowing the review applications. As held earlier, if the judgment/order is vitiated by
an apparent error or it is a palpable wrong and if the error is self evident, review is
permissible and in this case the High Court has rightly applied the said principles as
provided under Order 47 Rule 1 C.P.C. In view of the same, we are unable to accept the
arguments of learned senior counsel appearing for the appellant, on the other hand, we
are in entire agreement with the view expressed by the High Court.
13. In the light of the above discussion and conclusion, the appeals fail and are
accordingly dismissed. No costs.
Appeals dismissed.
AIR 2008 SUPREME COURT 725 "Modi Tele Fibres Ltd. v. U. P. State Electricity
Board"
(From : Allahabad)
Coram : 2 R. V. RAVEENDRAN AND P. SATHASIVAM, JJ.
Civil Appeal No. 5976 of 2001, D/- 6 -12 -2007.
Modi Tele Fibres Ltd. v. U. P. State Electricity Board and Ors.
Electricity Act (9 of 1910), S.24 - ELECTRICITY - CONSTITUTIONALITY OF AN
ACT - Electricity bill - Demand for - Validity - Appellant Company having service
connection - Due to financial crunch it requested Board to provide electric supply directly
to residential colonies - Neither specific request was made for disconnection of service
connection nor requisite charges were paid for separate domestic connection - Board
continued to supply electricity through service connection - Demand for electricity bill
from Company even for residential colonies - Not improper - Further, appellant could
have switched off supply from their distributing mains which were in their custody and
possession could have disconnected supply themselves - Having not done so appellant
cannot blame respondents for not disconnecting supply and could not challenge demand
for electricity bill. (Paras 9, 10)
@page-SC726

Rajiv Dutta, Sr. Advocate, Ms. Surbhi Sharma, Milanka Chaudhury and M. A.
Chinnasamy, for Appellant; Pradeep Misra, for Respondents.
Judgement
JUDGMENT :-This appeal is directed against the final judgment and order dated
23.9.1999 passed by the Division Bench of the High Court of Judicature at Allahabad in
Civil Misc. Writ Petition No. 37862 of 1999, whereby the High Court dismissed the writ
petition preferred by the appellant-herein.
BACKGROUND FACTS:
2. The appellant-Modi Tele Fibres Ltd. was carrying on business of manufacturing
threads at Modinagar, Dist. Ghaziabad. However, the appellant-Company started
suffering huge losses on account of various factors such as fall in production, non-
availability of capital funds for meeting operational expenses etc. which were beyond the
control of the appellant. The appellant, on 16.06.1994, wrote a letter to respondent No.1-
U.P. State Electricity Board (hereinafter referred to as the 'UPSEB') to provide electric
supply directly to the residential colonies as the appellant was unable to continue the
payment directly on account of lack of funds. It is pertinent to mention here that
electricity to the residential colonies is fed through Modi Tele Fibres Ltd. Service
Connection No. 1008. The appellant-company entered into an agreement on 30.09.1994
in supersession of an earlier agreement dated 28.09.1983, with the UPSEB for supply of
electricity for 4000 KVA load of 11 KV voltage through the above-said Service
Connection. It is also pertinent to mention that an amount of Rs.67,46,700/- is lying with
the UPSEB as security, whereas the appellant has already been paying regularly the bills
for the electricity consumed by the company and the residential colonies. The appellant
wrote another letter on 30.06.1995 to the UPSEB informing that an application has been
made to the State Government for closing down of the unit and the UPSEB should
discontinue permanently the supply of electrical energy to the appellant vide S.C. No.
1008 reiterating its earlier request to provide separate domestic connection to residential
colonies. It was also reiterated that w.e.f. 01.08.1995, the appellant-company shall not be
liable for the supply made. Despite repeated requests, the UPSEB continued to supply
electricity through the service connection to the company as well as the residential
colonies at commercial rates. In reply, respondent No.2, vide letter dated 13.07.1995,
informed the appellant that only the person who had signed the agreement with the
UPSEB is empowered to apply for permanent disconnection and the request of the
appellant for permanent disconnection was not being considered. Thereafter, on
07.08.1995, the then Chairman of the appellant-Company who had signed the agreement
wrote a letter for permanent disconnection and to provide separate domestic connections
to the residential colonies reiterating that w.e.f. 06.09.1995, the Company shall not be
liable for the supply. Thereafter, on 04.09.1995, because of the heavy losses being
incurred, the appellant-company had to effect permanent closure and a notice of closure
dated 02.09.1995 was issued to all the employees. It is an admitted position that the
company w.e.f 04.09.1995 was not using any electric power for its factory, but electricity
was being given to the residential colonies through service connection No. 1008. The
appellant also brought to the notice of UPSEB that for realizing the electricity dues from
the residents of the colony, the High Court, in a similar case, passed an order in
pursuance of which bills directly were charged from the persons occupying the residential
quarters. Under these circumstances, the appellant again requested that it would hand
over all the infrastructure free of cost which is already used to provide separate domestic
connection to the residential colonies and asked to immediately discontinue electric
supply through the service connection. However, no heed was paid to the request of the
appellant and UPSEB kept on sending bills including the bills of electricity consumed by
the residential quarters. In the meantime, Punjab National Bank which extended financial
assistance to the appellant initiated recovery proceedings before the Debts Recovery
Tribunal. The Tribunal passed an interim order whereby the appellant was restrained from
leasing out the factory premises. Against that order, the appellant filed a petition under
Article 227 of the Constitution before the Delhi High Court, which vide order dated
08.03.1999 allowed the appellant to lease out the factory with a direction that 50% of the
rent amount shall be paid directly to the Punjab National Bank. Thereafter, 50% of the
rent is being received by the Bank and 50% rent by the appellant from the lessee.
@page-SC727
3. On 24.02.1999, UPSEB raised a bill demanding Rs.11,35,80,301/- from the appellant
for the period from April, 1995 to February, 1999 which includes electric supply to the
factory and to the residential quarters, surcharge, penalty etc. The appellant raised an
objection to the said bill on 24.04.1999 stating that it has repeatedly objected inasmuch as
firstly after closure of the factory on 04.09.1995 no electricity was being consumed and
was used by the factory and the bills pertain to consumption by the residential quarters
for which it had time and again requested for a separate connection.
4. On 24.07.1999 the Sub-Divisional Magistrate, Modinagar, Dist. Ghaziabad issued an
order to the lessee Lucky Tex Spinners Pvt. Ltd. directing that since an amount of
Rs.11,61,61,574.31 is due on the appellant as Government dues 50% of the rent amount
was attached and further directed to pay the same by pay order every month directly to
the Tehsildar. The UPSEB again issued a bill on 31.07.1999 for a sum of
Rs.13,40,42,018/-. In the meantime, the appellant made a reference to the BIFR under
Section 15 of the Sick Industrial Companies Act. On 20.08.1999, the appellant sent its
objection reiterating the stand that they were not liable to pay and returned the bills to the
UPSEB for cancellation. Being aggrieved by the order passed by the Sub Divisional
Magistrate, the appellant filed a writ petition in the High Court. The Division Bench of
the High Court by order dated 23.09.1999 dismissed the writ petition on the ground that
merely because the appellant had informed the UPSEB to provide separate domestic
connections to the residential colonies knowing fully well that they were already
consuming power through service connection No. 1008 in accordance with the terms of
the agreement, the liability will not cease. The High Court was of the view that while on
the one hand there was a prayer for disconnection but on the other hand regular
consumption not for a short period, but for years, the only conclusion was that the
consumer was enjoying the power supply and therefore the liability to pay for the power
consumed must be upheld. Dissatisfied with the order of the High Court the appellant
preferred the above appeal.
5. We heard Mr. Rajiv Dutta, learned counsel for the appellant and Mr. Pradeep Misra,
learned counsel for the respondents.
6. The grievance of the appellant is that even after the closure of their mill and in spite of
requests by way of letters and reminders for stopping the electrical supply to the
residential colony and for providing a separate metre connection to the residential
quarters of their employees, the respondent-UPSEB was unjustifiably claiming power
consumption charges from the appellant herein. Alternatively, it was submitted that it had
made payment upto March, 1995. However, if the bills for the period upto the date of
closure (i.e. upto 10.09.1995) are to be taken into account, then for the period from
01.04.1995 to 10.09.1995, the total amount of bills comes to Rs.1,14,10,734.00. Out of
the above, a sum of Rs.49,84,894/- is on account of supply of electricity to the residential
quarters which the appellant is not liable to pay as it had sent a notice in June, 1994. In
this regard, the admitted liability of the appellant is up to 10.09.1995 which comes to
Rs.64,25,840.00. The appellant had a security deposit of Rs.67,46,700/- with the UPSEB
and after adjusting the same, it is entitled to receive a sum of Rs.3,20,860/- from the
UPSEB.
7. It is not in dispute that the appellant was provided electric connection No. 1008 for
supply of electrical energy and an agreement had been executed on 30.09.1994 for supply
of 4000 KVA electric load. In the counter affidavit filed on behalf of respondent-UPSEB,
it has been specifically stated that the UPSEB, the predecessor in the interest of UP
Power Corporation, has no distributing means or any kind of control for contribution and
supply of electrical energy to the residential colonies of the workers of the appellant. In
fact in the counter affidavit the Board has stated that they were not aware about
arrangements made by the appellant for supply of power to their workers and the terms
and conditions for such supply as to whether it was free supply or whether the cost of
electricity consumed was being deducted from their wages. According to them, the
appellant was their consumer and bulk supply of 4000 KVA was being given to it and no
bifurcation in the connection as industrial or residential.
8. Learned counsel appearing for the appellant, by drawing our attention to various
clauses in the agreement and requests made by them in the form of letters seeking for dis-
connection of power supply to the
@page-SC728
residence of their employees and providing separate meter for their colonies, contended
that the respondents were not justified in demanding the amount as if arrears of power
consumed by them. We verified the requests made by the appellant. As rightly pointed out
by the respondents and in fact it was not disputed that electrical connection was provided
to the appellant-factory in service connection No 1008 for supply of electrical energy and
an agreement had been executed for the same on 30.09.1994 and supply to residential
colony was made by appellant under service connection No. 1008 of appellant. Therefore,
appellant cannot escape liability for electricity consumed in the residential colony. It
should also be noted that the requests for permanent disconnection made by appellant on
30.06.1995, 13.07.1995 and 07.08.1995 could not be acted upon as under the terms of the
supply agreement dated 30.09.1994, there could be no request for termination before the
end of two years. Significantly there was no letter for permanent disconnection after the
two year period, that is after 30.09.1996. Insofar as letter dated 16.06.1994 requesting for
electricity supply to residential quarters, it has to be ignored in view of the subsequent
agreement dated 30.09.1994 without separating supply to residential colony.
9. As rightly stated in para 15 of the additional affidavit filed on behalf of the UP Power
Corporation, in case the appellant did not want to supply the electricity to the residential
colonies of their workers they could have switched off the supply from their distributing
mains which were in their custody and possession. Admittedly, the appellant having such
a course available, did not do so because of their anticipation that law and order problem
would arise. Having failed to disconnect the electricity supply themselves, the appellant
cannot blame the respondents for not disconnecting the supply. It is true that pursuant to
the requests made by the appellant, the respondents/Board could have provided separate
connection for the residential connections in their colonies for the benefit of appellant's
employees. However, as pointed out in the additional affidavit necessary charges, namely,
costs and expenses for separate domestic connections were not paid. On the other hand,
the appellant was drawing power to their residential colonies in order to provide
uninterrupted supply to their employees. In those circumstances and in the light of the
specific information furnished in the additional affidavit particularly in paras 4, 8, 12 and
15, we are unable to accept the stand taken by the appellant.
10. With the materials placed before us, we are satisfied that the appellant being
consumer and consumed electricity through their service connection No. 1008 it has to
pay the amount for the same. We are also of the view that the appellant could have taken
effective steps for providing separate power connection to the residential colony of their
employees by approaching the respondents depositing necessary charges, cost and by
complying with the provisions of the Indian Electricity Act, the Electricity Supply Act,
rules and regulations made therein, which they failed to do. All the relevant aspects have
duly been considered and rightly rejected by the High Court. In regard to the alternative
contentions relating to excessive billing and non-adjustment of security deposit, these
factual aspects were not urged before the High Court and cannot be urged for the first
time before us. If there is any error in calculation of the amount shown as due, it is open
to the appellant to take up that issue separately with the respondents.
11. We do not find any ground for interference, consequently, the appeal fails and the
same is dismissed. However, there shall be no order as to costs.
Appeal dismissed.
AIR 2008 SUPREME COURT 728 "Shipping Corporation of India Ltd. v. M/s. Bharat
Earth Movers Ltd."
(From : Madras)
Coram : 2 S. B. SINHA AND G. S. SINGHVI, JJ.
Civil Appeal No. 5638 of 2007 (arising out of SLP (C) No. 7346 of 2005), D/- 5 -12
-2007.
Shipping Corporation of India Ltd. v. M/s. Bharat Earth Movers Ltd. and Anr.
(A) Carriage of Goods by Sea Act (26 of 1925), S.2 - Japanese Carriage of Goods by Sea
Act (1992), Art.1 - CARRIAGE OF GOODS - APPLICABILITY OF AN ACT -
Shipment of goods - Vessel sailed from port in Japan to its destination in India - Held,
Japanese Act and not Indian Act would be applicable.
O. S. A. No. 247 of 2000, D/- 2-12-2004 (Mad.), Reversed.
A bare perusal of S. 2 of the Indian Act 1925 would clearly demonstrate that the
@page-SC729
same applies to the carriage of goods by sea in ships carrying goods from any port in
India to any other port whether in or outside India which would mean that the Indian Act
shall apply only when the carriage of goods by sea in ships takes place from a port situate
within India and not a port outside India. The Japanese Act, on the other hand, applies in
a situation where carriage of goods by a ship is either from a loading Port or from a
discharging Port, either of which is located outside Japan. Therefore, Japanese Act will
clearly be applicable in the instant case, where goods were carried in vessel from Port in
Japan to its destination in India.
O. S. A. No. 247 of 2000, D/- 2-12-2004 (Mad.), Reversed.
(Para 17)
(B) Carriage of Goods by Sea Act (26 of 1925), S.4, Sch., Art.III, R.3, R.4 - CARRIAGE
OF GOODS - Damages - Shipment of 16 packages (cases) - Claimant confined its claim
of damages only for two cases - Liability of appellant would be limited in respect of two
cases - Cannot be calculated by taking into consideration the weight of 16 cases.
O. S. A. No. 247 of 2000, D/- 2-12-2004 (Mad.), Reversed. (Paras 18, 20)
(C) Carriage of Goods by Sea Act (26 of 1925), S.4, Sch., Art.III, R.3, R.4 - CARRIAGE
OF GOODS - Bill of lading - Invoice is not part of bill of lading - Value of goods is
required to be stated on bill of lading so as to enable shipping concern to calculate
quantum of freight - It cannot, in absence of any statutory provisions, be held to be
incorporated therein by necessary implication or otherwise. (Para 19)

C. A. Sundaram, Sr. Advocate, P. B. Suresh, Vipin Nair (for M/s. Temple Law Firm), for
Appellant; P. R. Sika, Chander Shekhar Ashri, for Respondents.
Judgement
S. B. SINHA, J. - :-Leave granted.
2. Application of the Indian Carriage of Goods by Sea Act, 1925 (for short "the Indian
Act") vis-a-vis the Japanese Carriage of Goods by Sea Act, 1992 (for short "the Japanese
Act") is in question in this appeal which arises out of a judgment and order dated
2.12.2004 passed by a Division Bench of the High Court of Judicature at Madras in OSA
No. 247 of 2000 affirming the judgment and decree dated 7.03.2000 passed by a learned
Single Judge thereof in CS No. 75 of 1996.
3. Appellant is an owner of a fleet of vessels. A consignment of six sets of Sub
Assemblies for PC 650 H.E. was entrusted by the respondent No. 1 for carriage thereof
from Kobe, Japan to Madras. It contained 16 packages. It arrived at the Port of Madras on
17.12.1994.
4. A part of the consignment was found in damaged condition. An inspection therefor was
made. Some damage was noticed in five cases. On the premise that the damage of short
delivery had been caused due to negligence on the part of the employees of the appellant,
a suit was filed on the original side of the Madras High Court. Claim of damage,
however, was therein confined to two cases only, viz., case Nos. 00002 and 0013. In the
said suit, the following relief was prayed for - :
"(a) A sum of Rs.16,72,143.87 with interest from the date of plaint till the date of
realization (interest of 18%) at 18% p.a as the transaction being commercial one under
Section 34 CPC."
5. In the written statement, the respondents inter alia pleaded 'limited liability' on their
part.
A learned Single Judge of the said Court held the appellant liable for payment of damages
being responsible for causing damage and loss to the consignment which had occurred at
a time when the cargo was in its charge.
6. In regard to the contention of the appellant that the contract of carriage having been
concluded in Japan, the Japanese Act shall apply and not the Indian Act, it was opined - :
"Another contention is raised on the side of the defendant that Indian Carriage of Goods
by Sea Act has been amended by the Multi Model Goods Transportation Act of 1993 and
the maximum liability of the carrier per package is not 100/- as contended by the plaintiff
and the maximum liability is 666.67 Special Drawing Rights per package or two special
drawing rights per kg of gross weight of the goods lost of damaged, whichever is higher.
Calculated thus, according to the defendant, the maximum liability of the defendant will
be only Rs. 1,31,471.11/- . Even assuming that the liability of the defendant has to be
calculated thus, the liability must be calculated taking into weight of 16 cases which are
governed
@page-SC730
by Ex.A-3 Bill of lading and in this case the liability will be more than what is claimed in
the plaint. Therefore, the defendant cannot resist the claim of the plaintiff on this ground
and the contract is governed by only Indian Carriage of Goods by Sea Act. Therefore, on
issue No. 6 and 7, I hold that the contract is governed by Indian Carriage of Goods by Sea
Act and the defendant is liable to the extent of the plaintiff's claim and these two issues
are therefore answered against the defendant."
7. The Division Bench of the High Court in an intra-court appeal preferred by the
appellant herein affirmed the said finding relying on or on the basis of Clause 6 of the
Bill of Lading, stating:
"On the basis of above clause 6, the submission of the learned counsel for the
appellant/defendant, that the Japanese Carriage of Goods by Sea Act is applicable to the
facts of the case, cannot be countenanced."
8. A notice was issued by this Court confined to the question as to whether the appellant
has a limited liability to the claim of the respondents.
9. Mr. C.A. Sundaram, learned senior counsel appearing on behalf of the appellant,
placed before us the relevant provisions of the Indian Act, Japanese Act as also the
International Convention for the Unification of Certain Rules of Law relating to Bills of
Lading (Hague Rules) to contend that as the price of the cargo had not been disclosed in
the Bill of Lading, the liability of the appellant must be held to be confined only to the
amount specified therein. It was urged that the High Court committed a serious error in
holding that the Indian Law would be applicable.
10. Mr. P.R. Sikka, learned counsel appearing on behalf of the respondents, however,
supported the impugned judgment.
11. Before embarking on the questions raised before us, we at the outset may observe that
the provisions of the Multimodal Transportation of Goods Act, 1993 whereto reference
has been made by the parties before the High Court are not applicable as admittedly the
mode of transport was by sea only and did not involve any multimodal transportation as
defined in Section 2(k) thereof.
12. The scope of the Japanese Act is stated in Article 1 thereof stating - :
"The provision of this Act (except article 20bis) shall apply to the carriage of goods by
ship from a loading port or to a discharging port, either of which is located outside Japan,
and Article 20bis shall apply to the carrier's and his servant's liability for damage to goods
caused by their tort."
Paragraph 4 of Article 2 defines "one unit of account" to mean "the amount equivalent to
one Special Drawing Right as defined in paragraph (1) of Article 3 of the International
Monetary Fund Agreement". Article 4 confers a liability upon the carrier stating that it
shall not be relieved therefrom unless exercise of due diligence under the said Article is
proved.
The provision regarding limited liability is contained in Article 13 of the Japanese Act,
which reads as under - :
"(1) The carrier's liability for a package or unit of the goods shall be the higher of the
following - :
1) An amount equivalent to 666.67 units of account.;
2) An amount equivalent to 2 units of account per kilo of gross weight of the goods lost,
damaged or delayed.
(2) The unit of account used in each item of the preceding paragraph shall be the final
publicized one at the date on which the carrier pays damages in respect of the goods.
(3) Where a container, pallet or similar article of transport (which as referred to as
"containers and etc." in this paragraph) is used for the transportation of the goods, the
number of containers and etc. or units shall be deemed to be the number of the packages
or units of the goods for the purpose of the preceding paragraph unless the goods' number
or volume or weight is enumerated in the bill of lading..."
13. Indian Act, however, in Section 2, provides for the application of Rules in the
following terms - :
"Subject to the provisions of this Act, the Rules set out in the Schedule (hereinafter
referred to as "the Rules") shall have effect in relation to and in connection with the
carriage of goods by sea in ships carrying goods from any port in India to any other port
whether in or outside India."
14. Schedule appended thereto provides for the Rules relating to Bills of Lading. Article
IV provides for rights and immunities, the relevant portion whereof reads as under - :
@page-SC731
"1. Neither the carrier nor the ship shall be liable for loss or damage arising or resulting
from unseaworthiness unless caused by want of due diligence on the part of the carrier to
make the ship seaworthy, and to secure that the ship is properly manned, equipped and
supplied, and to make the holds, refrigerating and cool chambers and all other parts of the
ship in which goods are carried fit and safe for their reception, carriage and preservation
in accordance with the provisions of paragraph - 1 of Article III.
Whenever loss or damage has resulted from unseaworthiness the burden of proving the
exercise of due diligence shall be on the carrier or other person claiming exemption under
this section."
Paragraph 5 of Article IV reads, thus - :
"5. Neither the carrier nor the ship shall in any event be or become liable for any loss or
damage to or in connection with goods in an amount exceeding 1001 per package or unit,
or the equivalent of that sum in other currency, unless the nature and value of such goods
have been declared by the shipper before shipment and inserted in the bill of lading......"
15. We may also notice that under the Special Drawing Rights as contained in the
International Monetary Fund Special Drawing Rights would mean 1.00XDR as
equivalent to 64.0948 INR and 666.67XDR as equivalent to 42,730.20 INR.
16. Clause 5 of the Hague Rules, to which both India and Japan are parties, reads as
under - :
"5. Neither the carrier nor the ship shall in any event be or become liable for any loss or
damage to or in connection with goods in an amount exceeding 100 pounds sterling per
package or unit, or the equivalent of that sum in other currency unless the nature and
value of such goods have been declared by the shipper before shipment and inserted in
the bill of lading.
This declaration if embodied in the bill of lading shall be prima facie evidence, but shall
not be binding or conclusive on the carrier."
17. Having noticed the relevant statutory provisions, we may also notice the relevant
terms and conditions of Bill of Lading which are as under:
"Clause 6 - : Liability for loss or damage where the stage is not known - :
When in accordance with the condition 4 hereof, the CTO is liable to pay compensation
in respect of loss or damage to the goods and the stage of transport where the loss or
damage occurred is not known, the liability of the CTO in respect of such loss or damage
shall not exceed the monetary limit indicated in this regard, in any international
convention or national law that would have applied, if the contract was for the carriage of
goods from a seaport in India and had been covered by a ocean bill of lading. However,
the CTO shall not in any case be liable for an amount greater than the actual loss to the
person entitled to make the claim...
Clause 7 - : Liability for loss or damage where the stage is known - :
(A) When in accordance with the condition 4 hereof, the CTO is liable to pay
compensation in respect of loss or damage to the goods and the stage of transport where
the loss or damage occurred is known, the liability of the CTO in respect of such loss or
damage shall be determined by the provisions contained in any International Convention
or National Law, which provisions would have applied if the claimant had made a
separate and direct contract with the CTO in respect of the particular stage of transport
where the loss or damage occurred and received as evidence thereof any particular
document which may be issued in order to make such International Convention or
National Law applicable..."
A bare perusal of Section 2 of the Indian Act would clearly demonstrate that the same
applies to the carriage of goods by sea in ships carrying goods from any port in India to
any other port whether in or outside India which would mean that the Indian Act shall
apply only when the carriage of goods by sea in ships takes place from a port situate
within India and not a port outside India. The Japanese Act, on the other hand, applies in
a situation where carriage of goods by a ship is either from a loading Port or from a
discharging Port, either of which is located outside Japan. Therefore, Japanese Act will
clearly be applicable in the instant case.
The High Court, as noticed hereinbefore, applied the provisions of the Indian law. We
may notice that Clause 6 of the Bill of Lading merely raises a legal fiction. It applies to a
case where the place of occurrence of loss
@page-SC732
or damage is not known. It merely provides that in such an event the quantum of loss
shall not exceed the monetary limit provided for in any international convention or
national law.
No reason has been assigned in support of its findings by the High Court. Clause 7 of the
Bill of Lading also should be read with Clause 6 thereof. In this case, the vessel sailed
from Japan; its destination being Chennai.
As the originating port is outside India, Section 2 of the Indian Act, as noticed
hereinbefore, will have no application. The High Court, in our opinion, misread the said
provision.
18. The provisions noticed hereinbefore, whether of the Japanese Act or the Indian Act or
the Hague Rules, provide for a limited liability. Contention of the appellant had been
rejected by the High Court inter alia on the premise that the plaintiff- respondent was
entitled to damages higher than the maximum liability provided for therein as the
quantum of damages was to be calculated upon taking into consideration the weight of all
the 16 cases and not only of two cases.
With respect, the approach of the High Court is wrong. If the plaintiff-respondent
confined its claim of damages only for two cases, there was no room for making the
observation that the liability must be calculated taking into consideration the weight of 16
cases. Even in support of the said conclusion, no reason has been assigned. The
discussions of the High Court end with the said finding which apparently is contrary to
the statutory provisions.
19. A contention has been raised before us for the first time that the value of the goods
had been declared in the Bill of Lading. It is based on the premise that Bill of Lading
refers to the invoice. We cannot accept the said contention. Invoice is not a part of the
Bill of Lading. The value of the goods is required to be stated on the Bill of Lading so as
to enable the shipping concern to calculate the quantum of freight. It cannot, in absence
of any statutory provisions, be held to be incorporated therein by necessary implication or
otherwise.
20. We, therefore, are of the opinion that the liability of the appellant being limited and
that too in respect of the two cases, the matter should be considered afresh in the light of
the observations made hereinbefore by the learned Single Judge. To the aforementioned
extent, the judgments and decrees of the High Court are set aside.
21. The appeal is allowed to the aforementioned extent. There shall, however, be no order
as to costs.
Order accordingly.
AIR 2008 SUPREME COURT 732 "General Manager, UCO Bank v. M. Venuranganath"
(From : Andhra Pradesh)*
Coram : 2 Dr. A. PASAYAT AND AFTAB ALAM, JJ.
Civil Appeal No. 5826 of 2007 (arising out of SLP (C) No. 23654 of 2004), D/- 12 -12
-2007.
General Manager, UCO Bank and Anr. v. M. Venuranganath.
Banking Companies (Acquisition and Transfer of Undertakings) Act (5 of 1970), S.19 -
United Commercial Bank (Conduct and Discipline and Appeal) Regulations (1976),
Regn.15(2), Regn.21(a) - BANKING - SALARIES - DISCIPLINARY ACTION -
SERVICE MATTERS - Manual on Disciplinary Action and Related Matters of UCO
Bank, Cl. 22(8) - Salary and allowances for period of suspension - Claim by officer of
UCO Bank, on being acquitted in criminal case - Maintainable under Cl. 22(8) of Manual
- Officer entitled to all benefits which he got while working - Cl. 22(8) is relatable to
Regn. 15 (2) which provides guidelines for operating Regn. 15(2) - Regn. 21(a) would
not be applicable since he is officer. (Paras 12, 13)

U. N. Bhachawat, Sr. Advocate, B. L. Anand, Alok Bhachawat and Ms. Pratibha Jain, for
Appellants; Ms. C. K. Sucharita, for Respondent.
* W. A. No. 685 of 2004, D/- 17-6-2004 (AP).
Judgement
Dr. ARIJIT PASAYAT, J. - :-Leave granted.
2. Challenge in this appeal is to the Judgment of a Division Bench of the Andhra Pradesh
High Court allowing the writ appeal filed by the respondent.
3. Background facts in a nutshell are as follows - :

The respondent, who, at the relevant point of time was Branch Manager of appellant
No.l-Bank and was posted at Nellore in Andhra Pradesh was charged and tried along with
one Shrinivasulu s/o. Chenchuramaiah reported in 2003 Lab IC 369 (AP)

@page-SC733
for offences punishable under Sections 120-B, 471 and 477 of Indian Penal Code 1860
(in short the 'the IPC') and Section 5(2) read with Section 5 (1)(d) of the Prevention of
Corruption Act, 1947 (in short the 'Prevention of Corruption Act'). Both the accused
persons were tried in the Court of Special Judge for CBI cases. They were acquitted by
judgment dated 11.12.2002 giving them benefit of doubt. The respondent was placed
under suspension from 15.06.1988 till he was reinstated on 04.05.1993. After his
reinstatement, departmental proceedings were initiated. The same were questioned by a
writ petition being writ Petition No.15797 of 1994 which was allowed by learned single
Judge. But in writ appeal No.884 of 1998, a Division Bench directed dismissal of the writ
petition. The departmental enquiry was concluded on 29.02.2003. The respondent was
found guilty. So far as payment of salary, allowances etc. are concerned, relevant portion
of the order read as follows - :
"In the light of the above punishment the undersigned further directs that Sri M. Venu
Ranganath will not be entitled to any salary and allowances and other attendant- benefits
including increment for the period spent by him under suspension, save and except the
Subsistence Allowance already paid to him during the said period."
3. The respondent filed writ petition being Writ Petition No.11615 of 1994 claiming pay
and allowances for the period of suspension which was dismissed by a learned Single
Judge. Respondent filed Writ Appeal No.685 of 2004 which was allowed by the
impugned order. It was inter-alia held by the Division Bench that C1ause-22 of the
applicable Manual i.e. A Manual on Disciplinary Action and Related Matters of UCO
Bank governed the case and not Regulation 15(2) of the United Commercial Bank
(Conduct and Discipline and Appeal) Regulation, 1976.
4. According to learned counsel for the appellants, the Division Bench was not justified in
holding that Clause 22(8) of the Manual was to operate and not Regulation 15(2) of the
Regulation. It is stated that the Manual is nothing but guidelines inducted and at the most,
may be termed as Executive Instructions. The Regulations are statutory in nature.
5. It is pointed out that acquittal in a criminal case has nothing to do with departmental
proceedings and law is clearly well settled. Notwithstanding acquittal in a criminal case,
departmental proceedings can be initiated and/or continued.
6. In response, learned counsel for the respondent submitted that the factual position
shows that the only time respondent was placed under suspension was because of the
criminal case under Regulation 12(1)(b). Even though departmental proceedings were
initiated, the respondent was never placed under suspension. According to her, the case
covered by Regulation 15(2) relates to cases not covered by sub-Regulation (1).
7. There can be no doubt that criminal proceedings and departmental proceedings operate
in different fields. Even though the person may have been acquitted in a criminal trial,
there is no embargo on his being departmentally proceeded against. But the question here
is slightly different. The relevant provisions need to be quoted - :
"11. Special procedure in certain cases - :
Notwithstanding anything contained in regulation 6 or regulation 7 or regulation 8 the
Disciplinary Authority may impose any of the penalties specified in regulation 4 if the
officer/employee has been convicted on a criminal charge, or on the strength of facts or
conclusions arrived at by a judicial trial."
Regulation 12 - : Suspension -:
(1) An officer employee may be placed under suspension by the competent authority-
(a) where a disciplinary proceeding against him is contemplated or is pending; or
(b) where a case against him in respect of any criminal offence is under investigation,
inquiry or trial.
(2) An officer employee shall be deemed to have been placed under suspension by an
order of the competent authority-
(a) with effect from the date of his detention, if he is detained in custody, whether on a
criminal charge or otherwise, for a period exceeding forty-eight hours;
(b) with effect from the date of conviction, if in the event of a conviction for an offence,
he is sentenced to a term of imprisonment exceeding forty-eight hours and is not
forthwith dismissed or removed or compulsorily retired consequent to such conviction.
Explanation:- The period of forty-eight
@page-SC734
hours referred to in clause (b) of this sub-regulation shall be computed from the
commencement of the imprisonment after the conviction and for this purpose,
intermittent periods of imprisonment, if any, shall be taken into account.
(3) Where a penalty of dismissal, removal or compulsory retirement from service
imposed upon an officer employee under suspension is set aside in appeal or on review
under these regulations and the case is remitted for further inquiry or action or with any
directions, the order of his suspension shall be deemed to have continued in force on and
from the date of the original order of dismissal, removal or compulsory retirement and
shall remain in force until further orders.
(4) Where a penalty of dismissal, removal or compulsorily retirement from service
imposed upon an officer employee under suspension is set aside or declared or rendered
void in consequence of or by a decision of a court of law, and the disciplinary authority,
on consideration of the circumstances of the case, decides to hold further inquiry against
him on the allegations on which the penalty of dismissal, removal or compulsory
retirement was originally imposed, the officer employee shall be deemed to have been
placed under suspension by the competent authority from the date of the original order of
dismissal, removal or compulsory retirement and shall continue to remain under
suspension until further orders.
(5) (a) An order of suspension made or deemed to have been made under this regulation
shall continue to remain in force until it is modified or revoked by the authority
competent to do so.
(b) An order of suspension made or deemed to have been made under this regulation may
at any time be modified or revoked by the authority which made or is deemed to have
made the order."
"Regulation 15 - : Pay allowances and treatment of service on termination of suspension -
:
(1) Where the competent authority holds that the officer employee has been fully
exonerated or that the suspension was unjustifiable, the officer employee concerned shall
be granted the full pay to which he would have been entitled had he not been suspended,
together with any allowance of which he was in receipt immediately prior to his
suspension, or may have been sanctioned subsequently and made applicable to all officer
employees.
(2) In all cases other than those referred to in sub-regulation (1), the officer employee
shall be granted such proportion of pay and allowances as the Competent Authority may
direct;
Provided that the payment of allowances under this sub-regulation shall be subject to all
other conditions to which such allowances are admissible - :
Provided further that the pay and allowances granted under this sub-regulation shall not
be less than the subsistence and other allowances admissible under regulation 14.
3(a) In a case falling under sub-regulation (1) the period of absence from duty shall, for
all purposes, be treated as a period spent on duty;
(b) In a case falling under sub-regulation (2), the period of absence from duty shall not be
treated as a period spent on duty unless the Competent Authority specifically directs, for
reason to be recorded in writing, that it shall be so treated for any specific purpose."
Clause 22.8 of the Manual "Where a suspended officer employee has been fully
exonerated in the departmental enquiry or acquittal by the court of law of the charges
levelled against him the competent authority holds that the suspension was unjustifiable,
he would be entitled to all benefits to which he would have been normally entitled, had
he been on duty. However, the employee in such a case would not be entitled to
accumulate leave beyond the permissible limit."
8. A bare reading of Regulation 12 shows that suspension can be directed under two
circumstances. The first is where a disciplinary proceeding against the concerned
employee is contemplated or is pending; and the second is where a case against him in
respect of any criminal offence is under investigation, inquiry or trial. Undisputedly, the
respondent was placed under suspension under Regulation 12(i)(b). Regulation 15 deals
with two types of situations. As the heading itself shows, it relates to pay and allowances
and termination of service on termination or suspension. Sub-Regulation (1) deals with
the power of competent authority on completion of the departmental
@page-SC735
enquiry. All other cases, except those covered by Sub-Regulation (1), the competent
authority has to direct as regards the proportion of pay and allowances to be granted.
9. Clause 22 of the Manual deals with two situations. One is full exoneration in the
departmental proceedings and other is acquittal by the court of law of the charges
levelled. Clause 22(8) specifically deals with acquittal by criminal court. It does not
exclude acquittal where accused has been given benefit of doubt. A close reading of sub-
regulation (1) of Regulation 15 would show that the same is relatable to departmental
proceedings. While other cases, meaning, cases not covered by departmental proceedings,
which obviously would include the criminal trial are covered by sub-regulation (2).
10. At this juncture, it would also be relevant to take note of Clause 21(9) of the
Regulation. It deals with entitlements for benefits after acquittal by a criminal court.
11. The same reads as follows - :
"Where a suspended employee has been fully exonerated in the departmental enquiry or
honourably acquitted by the courts of law of the charges levelled against him, he would
be entitled to all benefits to which he would have been normally entitled, had he been on
duty. However, the employee in such a case would not be entitled to all benefits to which
he would have been normally entitled, had he been on duty. However, the employee in
such a case would not be entitled to accumulate leave beyond the permissible limit.
However, if the employee is acquitted by being given the benefit of doubt he may be paid
such portion of pay and allowances as the management may deem proper and the period
of his suspension shall not be treated as period spent on duty unless the management so
direct."
12. It is to be noted that Regulation 21(9) does not relate to officers and the respondent
herein was an officer and, therefore, Regulation 21 has no relevance as it covers only the
award staff.
13. Clause 22(8) obviously is relatable to Clause 15(2), meaning that it provides
guidelines for operating sub-Regulation (2) of Regulation 15. The High Court was,
therefore, justified in holding that because of Clause 22(8), the respondent was entitled to
all benefits to which he would have been normally entitled, had he been on duty.
Therefore, no interference is called for.
14. The appeal is, accordingly, dismissed. There will be no order as to costs.
Appeal dismissed.
AIR 2008 SUPREME COURT 735 "National Insurance Co. v. Deepa Devi"
(From : AIR 2006 (NOC) 1011 : 2005(4) ACC 194) (HP))
Coram : 2 S. B. SINHA AND LOKESHWAR SINGH PANTA, JJ.
Civil Appeal No. 5796 of 2007 (arising out of SLP (C) No. 22778 of 2005), D/- 11 -12
-2007.
National Insurance Co. Ltd. v. Deepa Devi and Ors.
Motor Vehicles Act (59 of 1988), S.168, S.2(30) - Representation of the People Act (43 of
1951), S.160 - MOTOR VEHICLES - ELECTION - POSSESSION - Compensation -
Liability of registered owner - Car in question at time of accident was requisitioned by
Magistrate for assembly elections - Being requisitioned by statutory authority, owner had
no other alternative but to handover possession - Once possession is handed over, owner
cannot exercise any control thereupon - State shall be, therefore, liable to pay
compensation and not registered owner of car.
2004 AIR SCW 4974, 2006 AIR SCW 5871, Relied on.
2003 AIR SCW 780, Disting.
1989 AC J 596 (AP); AIR 1996 Guj. 51; and AIR 1988 Ori 229, Approved.
AIR 2006 (NOC) 1011 : 2005 (4) ACC 194 (HP), Reversed. (Paras 10, 13, 18)
Cases Referred : Chronological Paras
2006 AIR SCW 5871 : AIR 2007 SC 465 : 2007 CLC 26 : 2007 (1) AIR Jhar R 791 (Rel
on) 11
2004 AIR SCW 2586 : AIR 2004 SC 2258 11
2004 AIR SCW 4974 : AIR 2004 SC 4179 : 2004 Lab IC 3688 : 2004 All LJ 3425 (Rel
on) 11
2003 AIR SCW 780 : AIR 2003 SC 1446 (Disting) 14
2001 AIR SCW 5023 : AIR 2002 SC 302 11
1997 AIR SCW 3531 : AIR 1997 SC 3444 (Rel. on) 6, 15
AIR 1996 Guj 51 (Approved) 17
1989 ACJ 596 (AP) (Approved) 16
AIR 1988 SC 1332 (Disting) 7, 12
AIR 1988 Ori 229 (Approved) 16
@page-SC736

Parmanand Gaur, for Appellant; J. S. Attri and B. K. Satija, for Respondents.


Judgement
S. B. SINHA, J. - :-Leave granted.
2

. The short question involved in this appeal arising out of a judgment and order dated
17.05.2005 passed by the High Court of Himachal Pradesh in FAO (MVA) No. 208 of
1997 is as to whether in the event a car is requisitioned by the State for the purpose of
deploying the same in the election duty, who would be liable to pay compensation to the
victim of the accident in terms of the provisions of the Motor Vehicles Act, 1988 (for
short "the 1988 Act"). reported in AIR 2006 (NOC) 1011

3. Respondent No. 3 was the owner of a Maruti Gypsy bearing Registration No. HIS
6095. Appellant Company issued a policy of insurance in favour of Respondent No. 4 for
the said Maruti Gypsy for the period 10.06.1993 to 9.06.1994. In regard to limitation of
its use, the insurance policy provided - :
"For private car IXI and Motor Cycle/Scooter IYI.
Use only for social, domestic and pleasures and insured's own purpose"
4. The car in question was requisitioned during the Assembly Elections in the year 1993
by the Sub-Divisional Magistrate Rampur through the Deputy Commissioner, Shimla.
The said vehicle was in possession as also under the control of the said officer. On or
about 17.11.1993 while the Sub-Divisional Magistrate Rampur was travelling in the said
vehicle, an accident occurred as a result whereof a boy named Satish Kumar sustained
injuries. He later on expired.
5. Respondent No. 1 Deepa Devi and Joginder being the heirs and legal representatives of
the deceased filed an application for compensation in terms of Section 166 of the 1988
Act. The State of Himachal Pradesh as also the Sub-Divisional Magistrate Rampur were
impleaded therein. The Motor Accident Claims Tribunal in its judgment dated 28.09.1996
upheld the contention of the Insurance Company that under the terms of the insurance
policy, it was not liable to reimburse the owner of the vehicle as regards his liability to
pay compensation on account of said accident. A Division Bench of the High Court,
however, by reason of the impugned judgment, has set aside the said award of the
Tribunal, holding - :
"In view of the above discussion, the appeal is allowed and the award of the Tribunal is
modified and it is held that the owner of the vehicle, the State Government and the
Insurance Company are all jointly and severally liable to pay the compensation. Since the
vehicle was insured with the Insurance Company it shall deposit the amount payable to
the claimants..."
6

. Mr. Parmanand Gaur, learned counsel appearing on behalf of the appellant, submitted
that having regard to the definition of 'owner' as contained in Section - 2(30) of the 1988
Act and as the vehicle in question was not used for the purpose for which the contract of
insurance was entered into, the judgment of the High Court cannot be sustained. Strong
reliance in this behalf has been placed on Rajasthan State Road Transport Corporation v.
Kailash Nath Kothari and Others [(1997) 7 SCC 481]. 1997 AIR SCW 3531

. Mr. J. S. Attri, learned counsel appearing on behalf of Respondent Nos. 5 and 6, on the
other hand, would support the judgment contending that this Court in Guru Govekar v.
Miss Filomena F. Lobo and Others [(1988) 3 SCC 1] has categorically held that even if
the vehicle remains in possession of a third party, the registered owner of the vehicle shall
continue to be the owner within the meaning of the provisions of the 1988 Act and, thus,
would be liable for payment of damages to the victims of an accident. AIR 1988 SC
1332

8. The 1988 Act was enacted to consolidate and amend the law relating to motor vehicles.
It repeals and replaces the Motor Vehicles Act, 1939 (for short "the 1939 Act").
9. "Owner" has been defined in Section 2(19) of the 1939 Act to mean - :
"In this Act, unless the context otherwise requires,
*** *** ***
(19) "owner" means, where the person in possession of a motor vehicle is a minor, the
guardian of such minor, and in relation to a motor vehicle which is the subject of a higher
purchase agreement, the person in possession of the vehicle under that agreement;"
However, the said definition underwent a change by reason of Section 2(30) of the 1988
Act providing - :
"In this Act, unless the context otherwise requires,
*** *** ***
@page-SC737
(30) "owner" means a person in whose name a motor vehicle stands registered, and where
such person is a minor, the guardian of such minor, and in relation to a motor vehicle
which is the subject of a hire-purchase, agreement, or an agreement of lease or an
agreement of hypothecation, the person in possession of the vehicle under that
agreement;"
10. Parliament either under the 1939 Act or the 1988 Act did not take into consideration a
situation of this nature. No doubt, Respondent Nos. 3 and 4 continued to be the registered
owner of the vehicle despite the fact that the same was requisitioned by the District
Magistrate in exercise of its power conferred upon it under the Representation of People
Act. A vehicle is requisitioned by a statutory authority, pursuant to the provisions
contained in a statute. The owner of the vehicle cannot refuse to abide by the order of
requisition of the vehicle by the Deputy Commissioner. While the vehicle remains under
requisition, the owner does not exercise any control thereover. The driver may still be the
employee of the owner of the vehicle but he has to drive it as per the direction of the
officer of the State, who is put in-charge thereof. Save and except for legal ownership, for
all intent and purport, the registered owner of the vehicle loses entire control thereover.
He has no say as to whether the vehicle should be driven at a given point of time or not.
He cannot ask the driver not to drive a vehicle on a bad road. He or the driver could not
possibly say that the vehicle would not be driven in the night. The purpose of requisition
is to use the vehicle. For the period the vehicle remains under the control of the State and/
or its officers, the owner is only entitled to payment of compensation therefor in terms of
the Act but he cannot exercise any control thereupon. In a situation of this nature, this
Court must proceed on the presumption that the Parliament while enacting the 1988 Act
did not envisage such a situation. If in a given situation, the statutory definitions
contained in the 1988 Act cannot be given effect to in letter and spirit, the same should be
understood from the common sense point of view.
11

. In Mukesh K. Tripathi v. Senior Division Manager, LIC and Others [(2004) 8 SCC 387],
this Court observed - :2004 AIR SCW 4974, (Para 38)

"The interpretation clause contained in a statute although may deserve a broader meaning
having employed the word "includes" but therefor also it is necessary to keep in view the
scheme of the object and purport of the statute which takes him out of the said definition.
Furthermore, the interpretation section begins with the words "unless the context
otherwise requires".
In Ramesh Mehta v. Sanwal Chand Singhvi, it was noticed: (SCC p. 426, paras 27-28)
2004 AIR SCW 2586, (Paras 27-28)

"A definition is not to be read in isolation. It must be read in the context of the phrase
which would define it. It should not be vague or ambiguous. The definition of words
must be given a meaningful application; where the context makes the definition given in
the interpretation clause inapplicable, the same meaning cannot be assigned.

In State of Maharashtra v. Indian Medical Assn. one of us (V.N. Khare, C.J.) stated that
the definition given in the interpretation clause having regard to the contents would not
be applicable. It was stated: (SCC p. 598, para 8) 2001 AIR SCW 5023, (Para 8)

'A bare perusal of Section 2 of the Act shows that it starts with the words "in this Act,
unless the context otherwise requires...". Let us find out whether in the context of the
provisions of Section 64 of the Act the defined meaning of the expression "management"
can be assigned to the word "management" in Section 64 of the Act. In para 3 of the
Regulation, the Essentiality Certificate is required to be given by the State Government
and permission to establish a new medical college is to be given by the State Government
under Section 64 of the Act. If we give the defined meaning to the expression
"management" occurring in Section 64 of the Act, it would mean the State Government is
required to apply to itself for grant of permission to set up a government medical college
through the University. Similarly it would also mean the State Government applying to
itself for grant of Essentiality Certificate under para 3 of the Regulation. We are afraid the
defined meaning of the expression "management" cannot be assigned to the expression
"management" occurring in Section 64 of the Act. In the present case, the context does
not permit or requires to apply the defined meaning to the word "management" occurring
in
@page-SC738
Section 64 of the Act.'"

[See also Pandey and Co. Builders (P) Ltd. v. State of Bihar and Another (2007) 1 SCC
467] 2006 AIR SCW 5871

12

. In Guru Govekar (supra), this Court was considering the definition of 'owner' under the
1939 Act. Therein the car was handed over to a mechanic for carrying out certain
electrical repairs to the car, when the accident occurred. This Court in the said fact
situation held - : AIR 1988 SC 1332

"14. Thus on the facts of the case before us we are of the view that the insurer is liable to
pay the compensation found to be due to the claimant as a consequence of the injuries
suffered by her in a public place on account of the car colliding with her on account of
the negligence of the mechanic who had been engaged by the repairer who had
undertaken to repair the vehicle by virtue of the provisions contained in Section 94 of the
Act which provides that no person shall use except as a passenger or cause or allow any
other person to use a motor vehicle in a public place, unless there is in force in relation to
the use of the vehicle by that person or that other person, as the case may be, a policy of
insurance complying with the requirements of Chapter VIII of the Act. Any other view
will expose innocent third parties to go without compensation when they suffer injury on
account of such motor accidents and will defeat the very object of introducing the
necessity for taking out insurance policy under the Act."
13. It is not a case where the car was handed over to a person with consent of the owner
thereof. When a vehicle is requisitioned, the owner of the vehicle has no other alternative
but to handover the possession to statutory authority.
14

. We are not oblivious of another decision of this Court in Rikhi Ram and Another v.
Sukhrania (Smt) and Others [(2003) 3 SCC 97] wherein keeping in view the provisions of
Sections 94 and 95 of the 1939 Act, a plea taken by the owner of the car that he has
transferred the same in favour of another person and, thus, he had no liability for payment
of compensation was negatived, stating - : 2003 AIR SCW 780

"5. The aforesaid provision shows that it was intended to cover two legal objectives.
Firstly, that no one who was not a party to a contract would bring an action on a contract;
and secondly, that a person who has no interest in the subject-matter of an insurance can
claim the benefit of an insurance. Thus, once the vehicle is insured, the owner as well as
any other person can use the vehicle with the consent of the owner. Section 94 does not
provide that any person who will use the vehicle shall insure the vehicle in respect of his
separate use.
6. On an analysis of Sections 94 and 95, we further find that there are two third parties
when a vehicle is transferred by the owner to a purchaser. The purchaser is one of the
third parties to the contract and the other third party is for whose benefit the vehicle was
insured. So far, the transferee who is the third party in the contract, cannot get any
personal benefit under the policy unless there is a compliance with the provisions of the
Act. However, so far as third-party injured or victim is concerned, he can enforce liability
undertaken by the insurer."
We are also not concerned with such a situation.
15

. In Kailash Nath Kothari (supra), however, this Court in a case, where a bus was given
on lease by the owner of the vehicle Shri Sanjay Kumar in favour of the Rajasthan State
Road Transport Corporation, held that when an accident takes place when the bus was
plied under the control of the Corporation, it was the Corporation alone who would be
liable for payment of compensation, stating - : 1997 AIR SCW 3531, (Para 17)

"...Driver of the bus, even though an employee of the owner, was at the relevant time
performing his duties under the order and command of the conductor of RSRTC for
operation of the bus. So far as the passengers of the ill-fated bus are concerned, their
privity of contract was only with the RSRTC to whom they had paid the fare for
travelling in that bus and their safety therefore became the responsibility of the RSRTC
while travelling in the bus. They had no privity of contract with Shri Sanjay Kumar, the
owner of the bus at all. Had it been a case only of transfer of services of the driver and
not of transfer of control of the driver from the owner to RSRTC, the matter may have
been somewhat different. But on facts in this case and in view of Conditions 4 to 7 of the
agreement ( supra ), the RSRTC must be held to be vicariously liable for the tort
committed by the driver while plying the bus under contract of the RSRTC. The general
@page-SC739
proposition of law and the presumption arising therefrom that an employer , that is the
person who has the right to hire and fire the employee, is generally responsible
vicariously for the tort committed by the employee concerned during the course of his
employment and within the scope of his authority, is a rebuttable presumption. If the
original employer is able to establish that when the servant was lent, the effective control
over him was also transferred to the hirer, the original owner can avoid his liability and
the temporary employer or the hirer, as the case may be, must be held vicariously liable
for the tort committed by the employee concerned in the course of his employment while
under the command and control of the hirer notwithstanding the fact that the driver would
continue to be on the payroll of the original owner. The proposition based on the general
principle as noticed above is adequately rebutted in this case not only on the basis of the
evidence led by the parties but also on the basis of Conditions 6 and 7 ( supra ), which go
to show that the owner had not merely transferred the services of the driver to the RSRTC
but actual control and the driver was to act under the instructions, control and command
of the conductor and other officers of the RSRTC."
We may also notice at this stage certain judgments of some High Courts.
16

. In The National Insurance Co. Ltd. v. Durdadahya Kumar Samal and Others [1988 (2)
T.A.C. 25] where the vehicle was requisitioned by the Collector for election duty, the
High Court of Orissa held - : AIR 1988 Ori 229

"In a vehicle requisitioned, the driver remains under the control of the Collector and by
such driving the vehicle he can be accepted to have been employed by the Collector.
Thus, the Collector would be vicariously liable for the act of the driver in the present
case."
[See also New India Assurance Co. Ltd. v. S. Ramulamma and others 1989 ACJ 596]
17. In Chief Officer, Bhavnagar Municipality and another v. Bachubhai Arjanbhai and
others [AIR 1996 Gujarat 51], the High Court of Gujarat held - :
"7. The facts on record clearly indicate that the vehicle in question which belonged to the
State of Gujarat was entrusted to the Municipality for distribution of water to the citizens.
It was implicit in allowing the vehicle being used for such purpose that the State of
Gujarat which owned the vehicle also caused or allowed any driver of the Municipality
who was engaged in the work of distribution of water to the citizens, to use motor vehicle
for the purpose. Therefore, when the vehicle was driven by the driver of the Municipality
and the accident resulted due to his negligence, the insurer of the vehicle became liable to
pay the compensation under the provisions of the Act. It is, therefore, held that the State,
as the owner of the vehicle and the respondent Insurance Company as its insurer were
also liable to pay the compensation awarded by the Tribunal..."
18. We, therefore, are of the opinion that the State shall be liable to pay the amount of
compensation to the claimants and not the registered owner of the vehicle and
consequently the appellant herein.
19. For the reasons aforementioned, the impugned judgment cannot be upheld. It is set
aside accordingly. The appeal is allowed. No costs.
Appeal allowed.
AIR 2008 SUPREME COURT 739 "Sangam Spinners v. Regional Provident Fund
Commissioner-I"
(From : Rajasthan)*
Coram : 2 Dr. A. PASAYAT AND P. SATHASIVAM, JJ.
Civil Appeal No. 1785 of 2001, D/- 4 -12 -2007.
Sangam Spinners v. Regional Provident Fund Commissioner-I.
Employees' Provident Funds and Miscellaneous Provisions Act (19 of 1952), S.16(1)(d)
(prior to Amendment Act 10 of 1998) - PROVIDENT FUND - REPEAL AND SAVINGS
- Exemption to new establishment - Granted under S. 16(1)(d) for period of three years -
Employer would be entitled to protection for period of three years starting from date of
setting up of establishment - Irrespective of meanwhile repeal of provision for infancy
benefit.
D. B. Special Appeal No. 1150 of 2000, D/- 29-11-2000 (Raj.), Reversed. (Paras 16, 20)
Cases Referred : Chronological Paras
2006 AIR SCW 1025 : 2006 Lab IC 1284 : 2006 (2) AIR Jhar R 321 (Ref.)19
@page-SC740

1999 Lab IC 2197 (Bom) 14


1997 AIR SCW 3747 : AIR 1997 SC 3828 : 1998 Lab IC 100 (Ref.) 18
AIR 1989 SC 1933 17
AIR 1977 SC 552 : 1977 Tax LR 149 (Ref.) 13
AIR 1974 SC 1 : 1974 Lab IC 1 (Ref.) 18
AIR 1971 SC 1193 (Ref.) 12
AIR 1965 SC 1970 17
AIR 1951 SC 128 : 1951 Cri LJ 860 (Ref) 17
AIR 1927 PC 242 17
(1886) 31 Ch D 402 17
Dr. Manish Singhvi, P. V. Yogeswaran and Ashok K. Mahajan, for Appellant; S. Wasim A.
Qadri, D. - S. - Mahra and B. V. Balaram Das, for Respondent.
* D. B. Special Appeal No. 1150 of 2000, D/- 29-11-2000 (Raj.)
Judgement
Dr. ARIJIT PASAYAT, J. - :- Challenge in this appeal is to the judgment rendered by a
Division Bench of the Rajasthan High Court at Jodhpur dismissing the Special Appeal
filed by the appellant. Challenge in the Special Appeal was to the judgment of a learned
Single Judge whereby the writ petition filed by the appellant was dismissed upholding the
decision of the Regional Provident Fund Commissioner (in short the 'Commissioner'). It
was held that Section 16(1)(d) of the Employees' Provident Funds Act, 1952 (hereinafter
referred to as the 'Act') was omitted from the statute by Act No.10 of 1998 with
retrospective effect i.e. from 22.9.1997. In other words, it was held that the infancy
protection shall not be available to the appellant factory after 22.9.1997.
2. The factual scenario lies into a very narrow compass. Appellant started production on
1.9.1995 and according to it, it was entitled to benefit under Section 16(1)(d) of the Act
from that day. From August, 1998 the appellant started to comply with the provisions of
the Act as the three year fledging period as envisaged under Section 16(1)(d) of the Act
came to an end. On 26.3.1999 enquiry under Section 7A of the Act was initiated to secure
the compliance of the Act from September, 1995 to July, 1998. By order dated 27.7.2000
the Commissioner recorded a specific finding that the company was a new unit and was
eligible for exemption under Section 16(1)(d) of the Act but since it was effaced from the
statute from 22.9.1997 the benefit was available till that date and not thereafter. The writ
petition filed was dismissed by the learned Single Judge, and so was the special appeal.
3. In support of the appeal learned counsel for the appellant submitted that the view of the
High Court is untenable and even if retrospective effect was given the same was to not in
any way affect the entitlement of the appellant.
4. Learned counsel for the respondent on the other hand supported the orders of the
Commissioner and the High Court.
5. The position of Section 16 at different points of time can be noticed. Section - 16 as
originally enacted read as follows - :
"16. Act not to apply to factories belonging to Government or local authority and also to
infant factories.
This Act shall not apply to-
(a) any factory belonging to the government or a local authority, and
(b) any other factory established whether before or after the commencement, of this Act
unless three years have elapsed from its establishment.
6. Section 16 was amended by the Employees' Provident Funds (Amendment) Act, 1958
and sub-section (1) of Section 16 of the Principal Act was substituted as under:
"(1) This Act shall not apply to any establishment until the expiry of three years from the
date on which the establishment is, or has been set up.
Explanation - :- For the removal of doubts it is hereby declared that an establishment
shall not be deemed to be newly set up merely by reason of a change in its location."
7. Section 16(1) was once again amended by the Employees' Provident Funds
(Amendment) Act, 1960 and sub-section (1) of Section 16 was substituted as under:
"(1) This Act shall not apply - :
(a) to any establishment registered under the Co-operative Societies Act, 1912, or under
any other law for the time being in force in any State relating to Co-operative Societies,
employing less than fifty persons and working without the aid of power; or
(b) to any other establishment employing fifty or more persons or twenty or more but less
than fifty persons until the expiry of three years in the case of the former and five years in
the case of the latter, from the date on which the establishment is, or has been, set up.
Explanation - :- For the removal of doubts, it is hereby declared that an
@page-SC741
establishment shall not be deemed to be newly set up merely by reason of a change in its
location".
8. Section 16 was further amended by the Employees' Provident Funds and
Miscellaneous Provisions (Amendment) Act, 1988 with effect from 1.8.1988, and Clause
(b) of sub-section (1) of Section 16 was substituted by clauses (b), (c) and (d) and the said
amendment to Section 16 is as under - :
"(b) to any other establishment belonging to or under the control of the Central
Government or the State Government and whose employees are entitled to the benefit of
contributory provident fund or old age pension in accordance with any scheme or rule
framed by the Central Government or the State Government governing such benefit; or
(c) to any other establishment set up under any Central Provincial or State Act and whose
employees are entitled to the benefits of contributory provident fund or old age pension in
accordance with any scheme or rule framed under that Act governing such benefits; or
(d) to any other establishment newly set up, until the expiry of a period of three years
from the date on which such establishment is, or has been set up."
9. Thereafter, Section 16 was again amended by Employees' Provident Funds and
Miscellaneous Provisions (Amendment) Act, 1988, omitting clause (d) with explanation
in sub-section (1) of Section 16 with effect from 22.9.1997. (The said omission was
initially carried out by Ordinance No.17/1997 promulgated on 22.9.1997 followed by
Ordinance No.25/1997, dated 25.12.1997 and Ordinance No.8 of 1998, dated 23.4.1998
followed by Act 10 of 1998).
10. According to the appellants, the un-amended provisions as it stood after the
amendment in 1988 under clause (d), apply to their cases and they were entitled to the
protection regarding non-application of the Act for a period of 3 years from the date on
which such establishment was set up. According to the High Court, as clause (d) was
deleted with effect from 22.9.1997, the Act had application to every establishment and no
exemption or 'infancy period' whatsoever was available from 22.9.1997.
11. The crucial question, therefore, is the effect of the amendment on the existing rights.
12

. In Jayantilal Amratlal v. Union of India and others (AIR 1971 SC 1193), it has been laid
down as under - : (Para 8)

"In order to see whether the rights and liabilities under the repealed law have been put to
an end by the new enactment, the proper approach is not to enquire if the new enactment
has by its new provisions kept alive the rights and liabilities under the repealed law but
whether it has taken away those rights and liabilities. The absence of a saving clause in a
new enactment preserving the rights and liabilities under the repeated law is neither
material nor decisive of the question."
13

. In Govinddas and others v. Income Tax Officer and another (AIR 1977 SC 552), it was
laid down that - : (Para 10)

"Now it is well settled rule of interpretation hallowed by time and sanctified by judicial
decisions that unless the terms of a statute expressly so provide or necessarily require it,
retrospective operation should not be given to a statute so as to take away or impair an
existing right or create a new obligation or impose a new liability otherwise than as
regards matters of procedure. The general-rule as stated by HALSBURY in Vol. 36 of the
LAWS OF ENGLAND (3rd Edn,) and reiterated in several decisions of this Court as well
as English Courts is that all statutes other than those which are merely declaratory or
which relate only to matters of procedure or of evidence are prima facie prospective and
retrospective operation should not be given to a statute so as to affect, alter or destroy an
existing right or create a new liability or obligation unless that effect cannot be avoided
without doing violence to the language of the enactment. If the enactment is expressed in
language which is fairly capable of either interpretation, it ought to be construed as
prospective only."
14. A Division Bench of Bombay High Court while considering the earlier amendment to
Section 16(1)(d) curtailing the infancy period from 5 years to 3 years, held thus, in Magic
Wash Industries (P) Ltd. v. Assistant Provident Fund Commissioner, Panaji and Anr.
(1999 Lab IC 2197) - :
"There is no doubt that the vested rights or benefits under the legislation could be
retrospectively taken away by legislation, but then the statute taking away such rights or
benefits must expressly reflect its intention
@page-SC742
to that effect. The infancy period prior to the amended provision Section 16(1)(d) was
five years in the case of establishments employing 20 to 50 workers and in the event this
infancy benefit was to be withdrawn, it was necessary that the intention of the Legislature
should have been clearly reflected in the amended provision itself that the rights and
benefits which had already accrued stood withdrawn. The amended clause 16(1)(d) came
on the statute book on June 2, 1988, when it was assented by the President of India but
the amended Section 16 was put into operation only with effect from August 1, 1988,
which empowered the Central Government to appoint different dates for the coming into
force of different provisions of the Act. We find it difficult in the circumstances, to
conclude that the intention of the Legislature was to take away the benefit of infancy
period which had already accrued to the existing establishments and this benefit has not
been expressly taken away or by implication by the amended provision Section 16(1)(d).
In the circumstances, we are of the opinion that the infancy period benefit of the
petitioner for a period of five years with effect from May 26, 1986, is not taken away by
the amended provision Section 16(1)(d) of the Act; and the petitioner could continue to
enjoy the said infancy benefit for a period of five years till May, 1991. Therefore, the
demand made by respondent 1 for the period up to May, 1991, has to be quashed. The
petitioners are complying with the provisions of the Act with effect from June, 1991."
15. The matter can be looked at from another angle. Section 6 of the General Clauses Act,
1897 (in short 'General Clauses Act') deals with effect of repeal. The said provision so far
relevant reads as follows - :
"6. Effect of repeal.- Where this Act, or any (Central Act) or Regulation made after the
commencement of this Act, repeals any enactment hitherto made or hereafter to be made,
then, unless a different intention appears, the repeal shall not-
(a) revive anything not in force or existing at the time at which the repeal takes effect; or
(b) affect the previous operation of any enactment so repealed or anything duly done or
suffered thereunder; or
(c) affect any right, privilege, obligation or liability acquired, accrued or incurred under
any enactment so repealed; or
(d) affect any penalty, forfeiture or punishment incurred in respect of any offence
committed against any enactment so repealed; or
(e) affect any investigation, legal proceeding or remedy in respect of any such right,
privilege, obligation, liability, penalty, forfeiture or punishment as aforesaid;
and any such investigation, legal proceeding or remedy may be instituted, continued or
enforced, and any such penalty, forfeiture or punishment may be imposed as if the
repealing Act or Regulation had not been passed."
16. In terms of Clause (c) of Section 6 as quoted above, unless a different intention
appears the repeal shall not affect any right, privilege or liability acquired, accrued or
incurred under the enactment repealed. The effect of the amendment in the instant case is
the same.
17. It is a cardinal principle of construction that every statute is prima facie prospective
unless it is expressly or by necessary implication made to have retrospective operation
(See Keshvan Madhavan Memon v. State of Bombay AIR 1951 SC 128). But the rule in
general is applicable where the object of the statute is to affect vested rights or to impose
new burdens or to impair existing obligations. Unless there are words in the statute
sufficient to show the intention of the Legislature to affect existing rights, it is deemed to
be prospective only 'nova constitutio futuris formam imponere debet non praeteritis'. In
the words of LORD BLANESBURG, "provisions which touch a right in existence at the
passing of the statute are not to be applied retrospectively in the absence of express
enactment or necessary intendment." (See Delhi Cloth Mills and General Co. Ltd. v. CIT,
Delhi, AIR 1927 PC 242). "Every statute, it has been said", observed LOPES, L.J.,
"which takes away or impairs vested rights acquired under existing laws, or creates a new
obligation or imposes a new duty, or attaches a new disability in respect of transactions
already past, must be presumed to be intended not to have a retrospective effect."(See
Amireddi Raja Gopala Rao v. Amireddi Sitharamamma, AIR 1965 SC 1970). As a logical
corollary of the general rule, that retrospective operation is not taken to be intended
unless that intention is manifested
@page-SC743
by express words or necessary implication, there is a subordinate rule to the effect that a
statute or a section in it is not to be construed so as to have larger retrospective operation
than its language renders necessary. (See Reid v. Reid, (1886) 31 Ch D 402). In other
words close attention must be paid to the language of the statutory provision for
determining the scope of the retrospectivity intended by Parliament. (See Union of India
v. Raghubir Singh (AIR 1989 SC 1933). The above position has been highlighted in
"Principles of Statutory Interpretation" by Justice G.P. Singh. (Tenth Edition, 2006) at PP.
474 and 475)
18

. In The State of Jammu and Kashmir v. Shri Triloki Nath Khosa and others (1974 (1)
SCC 19) and in Chairman, Railway Board and Ors. v. C.R. Rangadhamaiah and Ors.
(1997 (6) SCC 623), this Court held that provision which operates to affect only the
future rights without affecting the benefits or rights which have already accrued or
enjoyed, till the deletion, is not retrospective in operation. AIR 1974 SC 1
1997 AIR SCW 3747

19

. The above position was highlighted by this court in S.L. Srinivasa Jute Twine Mills (P)
Ltd. v. Union of India and Anr. [2006 (2) SCC 740]. 2006 AIR SCW 1025
20. In view of the above position in law, the judgments of the Commissioner and the
High Court are indefensible and are set aside. The appellant shall be entitled to the
protection for the period of three years starting from the date the establishment was set up
irrespective of the repeal of the provision for such infancy protection.
21. Appeal is allowed. No costs.
Appeal allowed.
AIR 2008 SUPREME COURT 743 "Harpal Singh v. State of Punjab"
Coram : 2 G. P. MATHUR AND G. S. SINGHVI, JJ.
Criminal Appeal No. 548 of 2007, D/- 4 -12 -2007.
Harpal Singh v. State of Punjab.
Terrorist and Disruptive Activities (Prevention) Act (28 of 1987), S.12, S.18, S.20A (as
inserted by Act 43 of 1993) - Explosive Substances Act (6 of 1908), S.5 - TERRORIST
AND DISRUPTIVE ACTIVITIES - EXPLOSIVE - COGNIZANCE OF OFFENCE -
Cognizance of offence - Power of Designated Courts under TADA - Accused charge-
sheeted under TADA Act and Explosive Substances Act - Sanction of Competent
Authority under TADA Act was not obtained - Designated Court being debarred from
taking cognizance of offence under TADA Act, it could not have tried any offence under
any other Acts - Conviction and sentence of accused under Explosive Substances Act are,
therefore, liable to be set aside.
AIR 1990 SC 1962; 1997 AIR SCW 3565, Relied on. (Paras 12, 13)
Cases Referred : Chronological Paras
1997 AIR SCW 3565 : AIR 1997 SC 3475 : 1997 Cri LJ 4086 (Rel. on) 11
AIR 1990 SC 1962 : 1990 Cri LJ 1869 (Rel. on) 8
Sushil Kumar, Sr. Advocate, Sanjay Jain, Anmol Thakral, Vinay Arora, Mukesh Kumar
and Sudarshan Singh Rawat with him, for Appellant; Kuldip Singh, for Respondent.
Judgement
G. P. MATHUR, J. - :-This Appeal has been preferred under Section 19 of the Terrorist
and Disruptive Activities (Prevention) Act, 1987 against the judgment and order dated
16.3.2007 of Designated Court, Kapurthala at Jalandhar in TADA Sessions Case No. 4 of
2006 by which the appellant has been convicted under Section 5 of the Explosive
Substances Act, 1908 and has been sentenced to 5 years' R.I. and a fine of Rs.1,000/-.
2. The case of the prosecution, in brief, is that on 12.3.1992 Kamaljit Singh, SHO,
Santokh Singh, SI and some other police personnel were going from village Kukar Pind
to village Raipur in connection with investigation of a case bearing FIR No. 31 under
Section 302/34, IPC, Section 25 of Arms Act and Sections 3, 4 and 5 of Terrorist and
Disruptive Activities (Prevention) Act, 1987 (hereinafter referred to as 'TADA'). When
they reached on Byen Bridge in village Kukar Pind, they saw a person coming on foot.
On seeing the police party he tried to run away but he was apprehended after a chase and
on inquiry he disclosed his name as Harpal Singh (appellant in this appeal). On his
personal search explosive powder wrapped in a glazed paper was recovered from the
"jhola" (bag) which he was having in his right hand. The bag contained one kilogram
explosive powder which was taken into possession. A ruka was sent to the police station
on the basis of which FIR was registered against the appellant.
3. After investigation and receipt of the sanction and report of the Forensic Science
@page-SC744
Laboratory charge-sheet was submitted in the court on 24.2.1994 for prosecution of the
appellant under Sections 4, 5, and 9B(b) of the Explosives Act, 1884. There was no
mention of any offence under TADA or under the Explosive Substances Act, 1908. The
charge-sheet was submitted in the Designated Court which took cognizance of the
offence and proceeded with the trial of the appellant. Ultimately the Designated Court
acquitted the appellant for the offences under TADA and the Explosives Act, 1884, but
convicted him only under Section 5 of the Explosive Substances Act, 1908 and sentenced
him to 5 years' R.I. and a fine of Rs. - 1,000/?.
4. Shri Sushil Kumar, learned senior counsel for the appellant, has submitted that the case
against the appellant is not established from the evidence adduced by the prosecution but
instead of going into the facts and appraisal of evidence the appeal can be allowed on a
legal ground.
5. Part III of TADA deals with Designated Courts. Sub-section (1) of Section 9 of TADA
provides that the Central Government or a State Government may, by notification in the
Official Gazette, constitute one or more Designated Courts for such area or areas, or for
such case or class or group of case as may be specified in the notification. Sub-section (1)
of Section 11 of TADA lays down that notwithstanding anything contained in the Code of
Criminal Procedure, every offence punishable under any provision of this Act or any rule
made thereunder shall be triable only by the Designated Court within whose local
jurisdiction it was committed or, as the case may be, by the Designated Court constituted
for trying such offence under sub-section (1) of Section 9. Sections 12 and 18 of TADA
read as under - :-
"12. Power of Designated Courts with respect to other offences.- (1) When trying any
offence, a Designated Court may also try any other offence with which the accused may,
under the Code, be charged at the same trial if the offence is connected with such other
offence.
(2) If, in the course of any trial under this Act, of any offence, it is found that the accused
person has committed any other offence under this Act or any rule made thereunder or
under any other law, the Designated Court may convict such person of such other offence
and pass any sentence authorised by this Act or such rule or, as the case may be, such
other law, for the punishment thereof."
"18. Power to transfer cases to regular courts.- Where, after taking cognizance of any
offence, a Designated Court is of opinion that the offence is not triable by it, shall,
notwithstanding that it had no jurisdiction to try such offence, transfer the case for the
trial of such offence to any court having jurisdiction under the Code and the court to
which the case is transferred may proceed with the trial of the offence as if it had taken
cognizance of the offence."
By Act No. 43 of 1993 Section 20-A was introduced in TADA with effect from 22.5.1993
and the said section reads as under - :-
"20-A. Cognizance of offence.- (1) Notwithstanding anything contained in the Code, no
information about the commission of an offence under this Act shall be recorded by the
police without the prior approval of the District Superintendent of Police.
(2) No court shall take cognizance of any offence under this Act without the previous
sanction of the Inspector-General of Police, or as the case may be, the Commissioner of
Police."
Therefore, with effect from 22.5.1993, in view of sub-section (1) of Section 20-A, no FIR
about the commission of an offence under TADA can be recorded by the police without
prior approval of the District Superintendent of Police. Similarly on account of sub-
section (2) of Section 20-A no court can take cognizance of any offence under TADA
without the previous sanction of the Inspector General of Police, or as the case may be,
the Commissioner of Police. The effect of this amendment is that the Designated Court is
debarred from taking cognizance of any offence under TADA without the previous
sanction of the Inspector General of Police or, as the case may be, the Commissioner of
Police.
6. As mentioned earlier, the alleged recovery of the incriminating article from the
appellant took place on 12.3.1992 and the case was registered under Sections 4 and 5 of
the Explosives Act, 1884. The police, after investigation, submitted charge-sheet before
the Designated Court on 24.2.1994. In the charge-sheet there was no reference to any
case under TADA or under the Explosive Substances Act, 1908 against the appellant. The
appellant was granted bail
@page-SC745
in the case under the Explosives Act on 1.7.1992. Thereafter he went away to USA and
applied for political asylum. He was declared as absconder on 20.1.1995. The US
authorities deported him to India in 2006. Thereafter the police submitted supplementary
charge-sheet against the appellant on 29.5.2006 for his prosecution under TADA and the
Explosive Substances Act. The Designated Court thereafter tried the appellant for
offences under TADA besides Sections 4 and 5 of the Explosives Act and also under
Sections 4 and 5 of the Explosive Substances Act.
7. The important feature which is to be noted is that the prosecution did not obtain
sanction of the Inspector General of Police or of the Commissioner of Police for
prosecution of the appellant under TADA at any stage as is required by Section 20-A(2)
of TADA. The trial of the appellant before the Designated Court proceeded without the
sanction of the Inspector General of Police or the Commissioner of Police. In absence of
previous sanction the Designated Court had no jurisdiction to take cognizance of the
offence or to proceed with the trial of the appellant under TADA.
8

. The Designated Court, while trying an offence under TADA, is undoubtedly empowered
to try any other offence with which the accused may, under the Code of Criminal
Procedure, be charged at the same trial if the offence is connected with such other offence
in view of Section 12 of TADA and may convict such person of such other offence and
may pass any sentence authorized by TADA or by such other law for the punishment
thereof. But for application of Section 12 it is absolutely essential that the Designated
Court should be trying an offence under TADA. If the Designated Court is not trying an
offence under TADA it will have no jurisdiction to try any other offence. Section 18 also
points out the same situation which says that where, after taking cognizance of any
offence, a Designated Court is of opinion that the offence is not triable by it, shall,
notwithstanding that it had no jurisdiction to try such offence, transfer the case for the
trial of such offence to any court having jurisdiction under the Code of Criminal
Procedure. Thus the Designated Court gets the jurisdiction to try any other offence only if
it has the jurisdiction and is trying an offence under TADA. In Niranjan Singh Karam
Singh Punjabi vs. Jitendra Bhimraj Bijja and others, AIR 1990 SC 1962, it was observed
as under - :- Para 12 of AIR

"Section 12(1) no doubt empowers the Designated Court to try any offence punishable
under any other statute along with the offence punishable under the Act if the former is
connected with the latter. That, however, does not mean that even when the Designated
Court comes to the conclusion that there exists no sufficient ground for framing a charge
against the accused under S. 3(1) it must proceed to try the accused for the commission of
offences under other statutes. That would tantamount to usurping jurisdiction. Section 18,
therefore, in terms provides that where after taking cognizance of any offence the
Designated Court is of the opinion that the offence is not triable by it, it shall,
notwithstanding that it has no jurisdiction to try such offence, transfer the case for the
trial of such offence to any Court having jurisdiction under the Code."
9. At this stage it will be useful to refer to the dictionary meaning of the word
'Jurisdiction' - :-
Black's Law Dictionary : "Court's power to decide a case or issue a decree".
Words and Phrases - Legally defined - Third Edition (p.497) - : "By 'jurisdiction' is meant
the authority which a court has to decide matters that are litigated before it or to take
cognizance of matters presented in a formal way for its decision. The limits of this
authority are imposed by the statute, charter, or commission under which the court is
constituted, and may be extended or restricted by similar means. If no restriction or limit
is imposed the jurisdiction is said to be unlimited. A limitation may be either as to the
kind and nature of the actions and matters of which the particular court has cognizance,
or as to the area over which the jurisdiction extends."
Law Lexicon by P. Ramanatha Aiyar - 2nd Edn. Reprint 2000 : "An authority or power,
which a man hath to do justice in causes of complaint brought before him (Tomlin's Law
Dic.). The power to hear and determine the particular case involved; the power of a Court
or a judge to entertain an action, petition, or other proceeding; the legal power of hearing
and determining controversies. As applied to a particular claim or controversy,
jurisdiction is the power to
@page-SC746
hear and determine the controversy."
Jurisdiction, therefore, means the authority or power to entertain, hear and decide a case
and to do justice in the case and determine the controversy. In absence of jurisdiction the
court has no power to hear and decide the matter and the order passed by it would be a
nullity.
10. As mentioned earlier, in the first charge-sheet which was filed on 24.2.1994 there was
no mention of TADA at all. It was in the supplementary charge-sheet filed on 29.5.2006
that the prosecution introduced the offence under TADA. But there was no sanction of the
Inspector General of Police or of the Commissioner of Police as required under Section
20-A(2) of TADA and, therefore, the Designated Court had no jurisdiction to take
cognizance of the offence. Since the Designated Court lacked inherent jurisdiction to try
the offence under TADA it could not have tried the appellant even for offences under the
Explosive Substances Act, 1908 or the Explosives Act, 1884. Thus the conviction of the
appellant under Section 5 of the Explosive Substances Act, 1908 is illegal.
11
. The aforesaid view has also been taken by this Court in Rambhai Nathabhai Gadhvi and
others vs. State of Gujarat (1997) 7 SCC 744 and para 8 of the report is reproduced below
- :- 1997 AIR SCW 3565

"8. Taking cognizance is the act which the Designated Court has to perform and granting
sanction is an act which the sanctioning authority has to perform. Latter is a condition
precedent for the former. Sanction contemplated in the sub-section is the permission to
prosecute a particular person for the offence or offences under TADA. Sanction is not
granted to the Designated Court to take cognizance of the offence, but it is granted to the
prosecuting agency to approach the court concerned for enabling it to take cognizance of
the offence and to proceed to trial against the persons arraigned in the report. Thus a valid
sanction is sine qua non for enabling the prosecuting agency to approach the court in
order to enable the court to take cognizance of the offence under TADA as disclosed in
the report. The corollary is that, if there was no valid sanction the Designated Court gets
no jurisdiction to try a case against any person mentioned in the report as the court is
forbidden from taking cognizance of the offence without such sanction. If the Designated
Court has taken cognizance of the offence without a valid sanction, such action is without
jurisdiction and any proceedings adopted thereunder will also be without jurisdiction."
12. In view of the discussion made above there cannot be any escape from the conclusion
that the Designated Court had no jurisdiction to try and convict the appellant under the
Explosive Substances Act, 1908 in view of the fact that it could not have taken
cognizance of the offence under TADA for lack of sanction by the competent authority
under Section 20-A(2) of TADA. In view of the fact that the Designated Court could not
try the offence under TADA being debarred from taking cognizance thereof on account of
want of sanction by the competent authority under the mandatory provisions of Section
20-A(2), it could not try any offence under any other Act as well.
13. The appeal is accordingly allowed and the conviction of the appellant under Section 5
of the Explosive Substances Act and the sentence imposed thereunder are set aside. The
appellant shall be released forthwith unless wanted in some other case.
Appeal allowed.
AIR 2008 SUPREME COURT 746 "Anita Enterprises, M/s. v. Belfer Co-op. Housing
Society Ltd."
(From : Bombay)*
Coram : 2 B. N. AGRAWAL AND PRAKASH PRABHAKAR NAOLEKAR, JJ.
Civil Appeal Nos. 2990-2991 of 2005 with C. A. Nos. 2992-2995 of 2005, D/- 14 -11
-2007.
M/s. Anita Enterprises and Anr. v. Belfer Co-op. Housing Society Ltd. and Ors.
(A) Maharashtra Co-operative Societies Act (24 of 1961), S.29 - Maharashtra Co-
operative Societies Rules (1961), R.10(1)(b), R.28 - Bombay Rents, Hotel and Lodging
House Rates Control Act (57 of 1947), S.5(11) - CO-OPERATIVE SOCIETIES -
HOUSES AND RENTS - TENANCY - Tenant co-partnership housing society - Member
- Is not tenant of society - His status is higher than a tenant.
The status of a member in the case of tenant co-partnership housing society is not that of
a tenant within the meaning of S. 5(11) of the Rent Act 1947, there is therefore no
relationship of landlord and tenant
@page-SC747
between the Society and the member.(Para 28)
The status of a member in a tenant co-partnership housing society is very peculiar. The
ownership of the land and building both vests in the society and the member has, for all
practical purposes, right of occupation in perpetuity after the full value of the land and
building and interest accrued thereon have been paid by him. Although de jure he is not
owner of the flat allotted to him, but, in fact, he enjoys almost all the rights which an
owner enjoys, which includes right to transfer in case he fulfils the two pre-conditions,
namely, he occupies the property for a period of one year and the transfer is made in
favour of a person who is already a member or a person whose application for
membership has been accepted by the society or whose appeal under S. 23 of the
Societies Act 1961 has been allowed by the Registrar or to a person who is deemed to be
a member under sub-sec. (1A) of S. 23 of the Societies Act 1961. If it is held that the
society is a landlord and the member is a tenant within the meaning of the Rent Act, in
that event the society can evict the member by filing a suit for eviction if it requires the
premises for its bona fide need, but under the Societies Act, it can evict the member only
as a consequence of his expulsion from the membership and neither under the Societies
Act nor Rules framed thereunder nor Bye-Laws nor Regulations there is any provision
that a Society can evict a member in case it has got bona fide need of the same. The said
interpretation would thus be contrary to the object of the Societies Act. (Paras 27, 28)
(B) Maharashtra Co-operative Societies Act (24 of 1961), S.29 - CO-OPERATIVE
SOCIETIES - TENANCY - Tenant co-partnership housing society - Restriction on
transfer of property allotted to member for his own use - Provision is mandatory.
In a case of tenant co-partnership society the ownership of the land and the building both
vest in the society and the premises is allotted to the member for his occupation only and
not for the purpose of occupation of anybody else. That is the object of the Societies Act
1961 as would appear from bye-law 2, i.e., 'for use of the member', meaning thereby his
own use. Under bye-law 64(a) a member is restrained from parting with possession of the
flat without permission of the society which shall be granted upon filing application, by
an intending transferee, for grant of membership by admitting him as a nominal member.
In view of the language of S. - 29(2), which is in the nature of injunction upon the right
of a member to transfer unless the twin conditions are fulfilled, and the purpose for which
the house is allotted to a member, i.e., for his self occupation, it cannot be said in any
manner that the said provision is directory as giving such an interpretation would
frustrate object of the Societies Act whereunder a flat is allotted to a member for his self-
occupation as would appear from the Societies Act, Rules, bye-Laws and Regulations.
(Para 30)
(C) Maharashtra Co-operative Societies Act (24 of 1961), S.29 - Bombay Rents, Hotel
and Lodging House Rates Control Act (57 of 1947), S.28 - CO-OPERATIVE
SOCIETIES - HOUSES AND RENTS - TENANCY - Tenant co-partnership housing
society - Member inducting third person in his allotted accommodation - Such transfer
being against mandatory provisions of S. - 29 of 1961 Act - Relationship of landlord and
tenant does not get established between member and third person. (Para 33)
(D) Maharashtra Co-operative Societies Act (24 of 1961), S.91, S.29(2) - CO-
OPERATIVE SOCIETIES - TENANCY - ALLOTMENT OF PREMISES - Co-operative
court - Jurisdiction - Dispute touching business of society - Member of tenant co-
partnership housing society - Creating tenancy rights in respect of his allotted
accommodation - Question as to legality of such tenancy - Can be raised/decided by co-
operative Court.
The question regarding legality or otherwise of the creation of tenancy right by the
member of tenant co-partnership society which amounts to transfer of interest of a
member in the property of the Society, can be decided by raising a dispute before the Co-
operative Court. The induction by a member of non-member is prohibited by S. 29. The
transaction is not void but only voidable. Ordinarily in case of a transaction which is
voidable and not void, if an aggrieved party intends to avoid the same it is required to
obtain a decree from a competent civil Court by filing a properly constituted suit. But in a
case like the present one, if a party is first asked to obtain a decree from a competent civil
Court and only thereafter raise a dispute which is undisputedly touching upon the
business
@page-SC748
of the society under S. 91 of the Societies Act, the same would frustrate the provisions of
S. 91 and the intention of the Legislature in incorporating a cheap and expeditious
remedy by referring the same to a Court constituted under the Societies Act instead of
throwing a party to cumbersome procedure of moving a civil Court. Obtaining of
declaration from a competent civil court that relationship of landlord and tenant was not,
duly created and, therefore, the induction of a person by the member as tenant was
invalid, before raising a dispute under S. 91 of the Societies Act was not necessary.
(Paras 35, 36)
Cases Referred : Chronological Paras
AIR 1990 SC 1563 (Rel. on) (Pt. A) 25, 26, 28, 31
AIR 1986 SC 1194 (Disting) (Pt. D) 38
AIR 1975 SC 1470 (Disting.) 31, 32
D. M. Nargolkar, for Appellant; U. U. Lalit, Sr. Advocate, Ravindra Keshavrao Adsure,
Gaurav Agarwal, Prashant Kumar, V. N. Raghupathy, for Respondents.
* L.P.A. No. 217 of 2001 and W. P. No. 2253 of 2001, D/- 25-8-2004 and 5-7-2001
(Bom)
Judgement
1. B. N. AGRAWAL, J. :-These appeals by special leave have been filed against separate
orders rendered by a Division Bench of the Bombay High Court in Letters Patent Appeals
whereby the same have been dismissed as not maintainable, thereby confirming the
common judgment rendered by a learned Single Judge of the High Court in three writ
petitions filed under Article 227 of the Constitution of India [hereinafter referred to as
'the Constitution'].
2. The facts, in brief, are that the Belfer Co-operative Housing Society Limited, Bandra
[West], Mumbai, respondent No. 1 in Civil Appeal Nos. 2990-2991 of 2005, [hereinafter
referred to as 'the Society'], which was a tenant co-partnership housing society, held both
lands and flats constructed thereon and Dr. Gopal Mahadeo Dhadphale, respondent No. 2
in the said appeals [hereinafter referred to as 'the member'] was admitted as member of
the Society in the year 1962 and flat No. 4 on the ground floor was allotted to him. On
3.6.1982, the member inducted M/s. Anita Enterprises, appellant No. 1 in the said
appeals, in room No. 2 of the said flat on a monthly rental of Rs. 1000/- and on 3.10.1983
the appellant aforementioned was inducted in room No. 3 as well on a monthly rental of
Rs. 750/-. The member thereafter inducted M/s. Anita Medical Systems Pvt. Ltd.,
appellant No. 2 in the said appeals, in room No. 1 of the flat in question on a monthly
rental of Rs. 1000/- which was subsequently enhanced to Rs. 1500/- per month and both
the appellants were put in possession of the aforesaid premises. The appellants paid rent
upto the month of December, 1986 and as the member refused to accept the rental from
January, 1987, the rental was sent to him by cheques under registered post, but the same
was not accepted.
3. Thereupon, the appellants were asked to vacate the premises in question which
necessitated filing of two separate suits by them in the year 1987 before the Small Cause
Court for a declaration that they were tenants with regard to the aforesaid premises of
which they were in occupation and for perpetual injunction restraining the member from
interfering in any manner with their possession over the premises in question in which
suits only the member was made party and not the Society. The member in the said suits
contested the claim of the appellants and both the suits filed by the appellants were
dismissed by the trial court upon a finding that the appellants were not inducted as
tenants in the suit premises. But on appeal being preferred to the appellate bench of the
Small Cause Court, the same were decreed and it was held that the appellants were
inducted as tenants in the premises in their occupation.
4. In the meantime, the Society raised a dispute in the year 1989 before the Co-operative
Court under Section 91 of the Maharashtra Co-operative Societies Act, 1960 [hereinafter
referred to as 'the Societies Act'] praying therein that the appellants be evicted from the
premises in their occupation and the member be directed to occupy the same as,
according to the Society, the member had parted with possession of the premises in
question in favour of the appellants which was not permissible in law. The said case was
contested by the appellants in which the member and the appellants entered appearance
and all contested the claim of the Society. The Co-operative Court by its award decided
the dispute in favour of the Society, passed an order of eviction against the appellants and
directed the member to occupy the premises. The said order was upheld in appeal.
5. Thereafter, before the High Court three writ petitions were filed - one by the
@page-SC749
appellants against the aforesaid order passed by the appellate court upholding order
passed by the Co-operative Court and the other two writ petitions by the member against
the order passed by the appellate bench of the Small Cause Court whereby aforesaid
declaratory suits filed by the appellants were decreed. A learned Single Judge of the High
Court, by a common judgment, dismissed the writ petition filed by the appellants
whereby order passed by the Co-operative Court against the member and the appellants,
which was upheld in appeal, has been confirmed and allowed the writ petitions filed by
the member, set aside judgment and order passed by the appellate bench of Small Cause
Court and restored that of the trial court whereby declaratory suits filed by the appellants
were dismissed. The said judgment has been upheld by Division Bench of the High Court
by dismissing the Letters Patent Appeals on the ground that the same were not
maintainable in view of the fact that the writ petitions were filed under Article 227 of the
Constitution. Hence these appeals by special leave.
6. Undisputed facts are stated hereinafter. The Society was a tenant co-partnership
housing Society, the land and the structures standing thereon, which include the premises
in question, were held by it, respondent No. 2 was admitted as its member, allotted flat
No. 4 and put in possession thereof. The appellants are in occupation of the premises in
question since the date of their induction aforementioned and the member remained in
possession of the premises for a period of more than one year before induction of the
appellants therein. Induction of appellants as tenants by the member amounted to transfer
of interest by the member in the premises in question, which was property of the Society,
and the appellants were neither members of the Society nor can be said to be persons
whose application for membership had been accepted by the Society or persons whose
appeal under Section 23 of the Societies Act had been allowed by the Registrar or persons
who were deemed to be members under Section (1A) of Section 23 of the Societies Act.
The appellants were inducted without the consent of either the Society or its Managing
Committee and never admitted as nominal members of the Society.
7. Shri Shekhar Naphade, learned senior counsel appearing on behalf of the appellants in
support of the appeals, submitted that respondent No. 2 - in his capacity as member of the
tenant co-partnership housing society - has a possessory right in the premises in question
and the Society was only, by way of legal fiction, owner of the said premises. It was
further submitted that there was no relationship of landlord and tenant between the
Society and the member and there was such a relationship between the member and the
appellants, as such, the appellants were entitled to claim protection under the Bombay
Rents, Hotel and Lodging House Rates Control Act, 1947 [hereinafter referred to as 'the
Rent Act') and the proceeding under Section 91 of the Societies Act was not maintainable
in view of the bar incorporated under Section 28 of the Rent Act. According to him, the
Society was not justified in contending that there was relationship of landlord and tenant
between the Society and the member and consequently the appellants cannot be said to be
sub-tenants of the member as creation of sub-tenancy was forbidden under Section 15 of
the Rent Act unless there was contract to the contrary, which was not so in the case on
hand and, consequently such a sub-tenant cannot be treated to be a tenant within the
meaning of Section 5(11) of the Rent Act. It was also submitted that the tenancy right
could be created by the member as the transfer by him of his right in the premises was not
forbidden in law, therefore, the Society was not justified in contending that relationship
of landlord and tenant was not duly created inasmuch as even if there was restrictive right
of transfer and not absolute one if the tenancy was created in infraction of the same, the
transaction creating tenancy right in the appellants by the member cannot be said to be
void as such and if a party wanted to avoid the same, it was required to move a competent
civil court for a declaration that the same was invalid in law as the said transaction can, at
the highest, be said to be voidable and the said question cannot be examined by a Co-
operative Court purporting to act under Section 91 of the Societies Act.
8. On the other hand, Shri U.U. Lalit, learned senior counsel appearing on behalf of the
Society, submitted that the relationship between the Society and the member, as would
appear from the Bye-Laws of the Society as well as Regulations, was that of landlord and
tenant in respect of the
@page-SC750
premises held by the Society and the member purported to create right of a sub-tenant in
the appellants which was, in the absence of any contract to the contrary, forbidden by
Section 15 of the Rent Act, as such the appellants having not acquired the status of a
tenant within the meaning of the Rent Act, cannot claim protection thereunder from
eviction. Alternatively, it was submitted that even if it was treated that there existed no
relationship of landlord and tenant between the Society and the member and relationship
of landlord and tenant was created between the member and the appellants, the same was
not valid in law as it was not duly created in view of the fact that such a transaction being
in violation of the provisions of Section 29 of the Societies Act, was invalid as the
transfer made was, though entered into after completion of period of one year of
occupation of the member, to a non-member which was forbidden by law, as would
appear from the said provisions and the Bye-Laws of the Society and its legality or
otherwise could have been examined in a dispute raised under Section 91 of the Societies
Act. It was then submitted that asking the Society to first seek such a declaration from a
competent civil court and thereafter raise a dispute under Section 91 of the Societies Act
would frustrate the very object of the Societies Act. It was further submitted that in any
view of the matter, in the present case as the appellants had already filed suits before the
Small Cause Court for a declaration that their status was that of tenants under the Rent
Act, in which it was open to the Society to raise the question that the relationship of
landlord and tenant was not duly created, meaning thereby not in accordance with law but
contrary to law and for granting relief to the appellants therein the Court was called upon
to go into this question and decide the same. Learned counsel also submitted that as the
relationship of landlord and tenant was not duly created, the appellants could not claim
protection of the Rent Act and the petition under Section 91 of the Societies Act was
maintainable as the dispute raised was touching upon business of the Society.
9. In view of the rival submissions, the following questions fall for our consideration in
these appeals :-
1. Whether status of a member in a tenant co-partnership housing society is that of a
tenant or landlord within the meaning of the Rent Act and consequently there was any
relationship of landlord and tenant between the society and its member?
2. Whether purported status of the appellants, who were inducted by the member in the
premises in question was that of a tenant or sub-tenant within the meaning of Section
5(11) of the Rent Act and if it is held to be a tenant whether the relationship of landlord
and tenant between them was duly created so as to claim protection from eviction under
the Rent Act?
3. Whether the question regarding legality or otherwise of creation of tenancy right
between the appellants and the member of the Society could be adjudicated by the Small
Cause Court in suits filed by the appellants against member of the Society for declaration
that there was relationship of landlord and tenant between them and the High Court was
justified in restoring decree passed by the trial court to the effect that there was no
relationship of landlord and tenant between the appellants and member of the Society?
4. Whether the matter regarding legality or otherwise of creation of tenancy right between
the appellants and the member could be adjudicated by the Co-operative Court in dispute
raised under Section 91 of the Societies Act before the Co-operative Court or the Society
before raising any such dispute was required to obtain a declaratory decree from
competent civil court by filing a properly constituted suit before it?
10. In order to appreciate the points involved in these appeals, it would be useful to refer
to the relevant provisions of the Societies Act, Maharashtra Co-operative Societies Rules,
1961 [hereinafter referred to as 'the Rules'], Bye-Laws of the Society which were
registered with the Registrar, Co-operative Society, at the time of grant of registration to
it [hereinafter referred to as 'the Bye-Laws'], Regulations relating to tenancies to be
granted by the Society to members in respect of premises held by the Society contained
in Form A which are part of registered Bye-Laws of the Society [hereinafter referred to as
'the Regulations'] and the Rent Act which run thus :-
THE SOCIETIES ACT :
"Section 2. Definitions.- In this Act, unless the context otherwise requires,-
[(16) "housing society" "means a society, the object of which is to provide its
@page-SC751
members with open plots for housing, dwelling houses or flats; or if open plots, the
dwelling houses or flats are already acquired, to provide its members common amenities
and services];
(19)
(a.) "member" means a person joining in an application for the registration of a Co-
operative society which is subsequently registered, or a person duly admitted to
membership of a society after registration, and includes a nominal, associate or
sympathizer member;
* * * * *
(c.) "nominal member" means a person admitted to membership as such after registration
in accordance with the by-laws;
* * * * *"
"Section 23. Open membership.- (1) No society shall, without sufficient cause, refuse
admission to membership to any person duly qualified therefor under the provisions of
this Act and its by-laws.
[(1A) Where a society refuse to accept the application from an eligible person for
admission as a member, or the payment made by him in respect of membership, such
person may tender an application in such form as may be prescribed together with
payment in respect of membership, if any, to the Registrar, who shall forward the
application and the amount, if any so paid, to the society concerned within thirty days
from the date of receipt of such application and the amount; and thereupon if the society
fails to communicate any decision to the applicant within sixty days from the date of
receipt of such application and the amount by the society, the applicant shall be deemed
to have become a member of such society.] [If any question arises whether a person has
become a deemed member or otherwise, the same shall be decided by the Registrar after
giving a reasonable opportunity of being heard to all the concerned parties.]
(2) Any person aggrieved by the decision of a society, refusing him admission to its
membership, may appeal to the Registrar. [Every such appeal, as far as possible, be
disposed of by the Registrar within a period of three months from the date of its receipt - :
Provided that, where such appeal is not so disposed of within the said period of three
months, the Registrar shall record the reasons for the delay.]
(3) The decision of the Registrar in appeal, shall be final and the Registrar shall
communicate his decision to the parties within fifteen days from the date thereof.
[( 4) Without prejudice to the foregoing provisions of this section, in the case of agro-
processing societies or any other society for which a definite zone or an area of operation
is allotted by the State Government or the Registrar, it shall be obligatory on the part of
such society to admit, on an application made to it, every eligible person from that zone
or the area of operation, as the case may be, as a member of such society, unless such
person is already registered as a member of any other such society, in the same zone or
the area of operation.]"
"Section 29 - Restrictions on transfer or charge of share or interest .- (1) Subject to the
provisions of the last preceding section as to the maximum holding of shares and to any
rules made in this behalf, a transfer of, or charge on, the share or interest of a member in
the share capital of a society shall be subject to such conditions as may be prescribed.
(2) A member shall not transfer any share held by him or his interest in the capital or
property of any society, or any part thereof, unless-
(a) he has held such share or interest for not less than one year;
(b) the transfer is made to a member of the society or to a person whose application for
membership has been accepted [by the society, or to a person whose appeal under Section
23 of the Act has been allowed by the Registrar; or to a person who is deemed to be a
member under sub-section (1A) of-section 23.].
(3) Notwithstanding anything contained in sub-sections (1) and (2), where a member is
allowed to resign, or is expelled, or ceases to be a member on account of his being
disqualified by this Act or by the rules made thereunder or by the by-laws of the society,
the society may acquire the share or interest of such member in the share capital by
paying for it at the value determined in the manner prescribed provided that the total
payment of share capital of a society in any financial year for such purposes does not
exceed ten per cent of the paid-up share capital of the society on the last day of the
financial year immediately preceding.
Explanation -.[I]-The right to forfeit the
@page-SC752
share or interest of any expelled member in the share capital by virtue of any by-laws of
the society, shall not be affected by the aforesaid provision.
[Explanation II,-In this section, the expression "financial year" means the year ending on
the [31st day, of March] or, in the case of any society or class of societies the accounts of
which are with the previous sanction of the Registrar balanced on any other day, the year
ending on such day.]
(4) Where the State Government is a member of a society, the restrictions contained in
this section shall not apply to any transfer made by it of its share or interest in the capital
of the society; and that Government may, notwithstanding anything in this Act, withdraw
from the society its share capital at any time, after giving to the society notice thereof of
not less than three months".
"Section 31 - Share or interest not liable to attachment.- The share or interest of a
member in the capital of a society, or in the loan-stock issued by a housing society, or in
the funds raised by a society from its members by way of savings deposit, shall not be
liable to attachment or sale under any decree or order of a Court for or in respect of any
debt or liability incurred by the member; and accordingly, neither the Official Assignee
under the Presidency-Towns Insolvency Act, 1909, nor a Receiver under the Provincial
Insolvency Act, 1920, nor any such person or authority under any corresponding law for
the time being in force, shall be entitled to, or have any claim on, such share or interest."
"Section 47 - Prior claim of society.- (1) Notwithstanding anything in any other law for
the time being in force, but subject to any prior claim of Government in respect of land
revenue or any money recoverable as land revenue and to the provisions of sections 60
and 61 of the Code of Civil Procedure, 1908.
(a) any debt or outstanding demand, owing to a society by any member or past member
or deceased member, shall be a first charge,-
(i) upon the crops or other agricultural produce raised in whole or in part whether with or
without a loan taken from the society by such member or past member or deceased
member,-
(ii) upon any cattle, fodder for cattle, agricultural or industrial implements or machinery,
or raw materials for manufacture, or workshop, godown or place of business supplied, to
or purchased by such member or past member or deceased member, in whole or in part,
from any loan whether in money or goods made to him by the society, and
(iii) upon any movable property which may have been hypothecated, pledged or
otherwise mortgaged by a member with the society, and remaining in his custody; (b) any
outstanding demands or dues payable to a society by any member or past member or
deceased member, in respect of rent, shares, loans or purchase money or any other rights
or amounts payable to such society, shall be a first charge upon his interest in the
immovable property of the society,
Explanation.-The prior claim of Government in respect of dues other than land revenue,
shall be restricted for the purpose of sub-section (1) to the assets created by a member out
of the funds in respect of which the Government has a claim.
(2) No property or interest in property, which is subject to a charge under the foregoing
sub-section, shall be transferred in any manner without the previous permission of the
society; and such transfer shall be subject to such conditions, if any, as the society may
impose.
(3) Any transfer made in contravention of sub-section (2) shall be void.
(4) Notwithstanding anything contained in sub-sections (2) and (3), a society, which has
as one of its objects the disposal of the produce of its members, may provide in its bye-
laws, or may otherwise contract with its members,-
(a) that every such member shall dispose of his produce through the society, and
(b) that any member, who is found guilty of a breach of the bye-laws or of any such
contract, shall reimburse the society for any loss, determined in such manner as may be
specified in the bye-laws."
"Section 91 - Disputes .- (1) Notwithstanding [anything contained] in any other law for
the time being in force, any dispute touching the constitution, [elections of the committee
or its officers other than elections of committees of the specified societies including its
officer], conduct of general meetings, management or business of a society shall be
referred by any of the parties to the
@page-SC753
dispute, or by a federal society to which the society is affiliated or by a creditor of the
society, to the co-operative Court if both the parties thereto are one or other of the
following:-
(a) a society, its committee, any past committee, any past or present officer, any past or
present agent, any past or present servant or nominee, heir or legal representative of any
deceased officer, deceased agent or deceased servant of the society, or the Liquidator of
the society [or the official Assignee of a de-registered society].
(b) a member, past member of a person claiming through a member, past member of a
deceased member of society, or a society which is a member of the society [or a person
who claims to be a member of the society;]
(c) a person other than a member of the society, with whom the society, has any
transactions in respect of which any restrictions or regulations have been imposed, made
or prescribed under sections 43, 44 or 45, and any person claiming through such person;
(d) a surety of a member, past member or deceased member, or surety of a person other
than a member with whom the society has any transactions in respect of which
restrictions have been prescribed under section 45, whether such surety or person is or is
not a member of the society;
(e) any other society, or the Liquidator of such a society [or de-registered society or the
official Assignee of such a de-registered society].
[Provided that, an industrial dispute as defined in clause (k) of section 2 of the Industrial
Disputes Act, 1947 , or rejection of nomination paper at the election to a committee of
any society other than a notified society under section 73 -1 C or a society specified by or
under section 73-G, or refusal of admission to membership by a society to any person
qualified therefor [or any proceeding for the recovery of the amount as arrear of land
revenue on a certificate granted by the Registrar under sub-section (1) or (2) of section
101 or sub -section (1) of section 137 or the recovery proceeding of the Registrar or any
officer subordinate to him or an officer of society notified by the State Government, who
is empowered by the Registrar under sub-section (1) of section 156,] [or any orders,
decisions, awards and actions of the Registrar against which an appeal under section 152
or 152-A and revision under section 154 of the Act have been provided] shall not be
deemed to be a dispute for the purposes of this section.]
(3) Save as otherwise provided under [sub-section (2) to section 93], no Court shall have
jurisdiction to entertain any suit or other proceedings in respect of any dispute referred to
in sub-section (1).
Explanation 1.-A dispute between the Liquidator of a society [or an official Assignee of a
de-registered society] and [the members (including past members, or nominees, heirs or
legal representative or deceased members)] of the same society shall not be referred [to
the co-operative Court] under the provisions of sub-section (1).
Explanation 2.-For the purposes of this sub-section, a dispute shall include-
(i) a claim by or against a society for any debt or demand due to it from a member or due
from it to a member, past member or the nominee, heir or legal representative of a
deceased member, or servant for employee whether such a debt or demand be admitted or
not;
(ii) a claim by a surety for any sum or demand due to him from the principal borrower in
respect of a loan by a society and recovered from the surety owing to the default of the
principal borrower, whether such a sum or demand be admitted or not;
(iii) a claim by a society for any loss caused to it by a member, past member or deceased
member, by any officer, past officer or deceased officer, by any agent, past agent or
deceased agent, or by any servant, past servant, past servant or deceased servant, or by its
committee, past or present, whether such loss be admitted or not;
(iv) a refusal or failure by a member, past member or a nominee, heir or legal
representative of a deceased member, to deliver possession to a society of land or any
other asset resumed by it for breach of condition as the assignment."
"93. Transfer of disputes from one Co-operative Court to another and suspension of
proceedings in certain cases.-
* * * * *
(2) Notwithstanding anything contained in this Act, the Co-operative Court, on an
application made to it by any of the parties to the dispute, may, if it thinks fit suspend
@page-SC754
any proceedings in respect of any dispute, if the question at issue between a society and a
claimant or between different claimants, is one involving complicated questions of law
and fact, until the question has been tried by a regular suit instituted by one of the parties
or by the society. If any such suit is not instituted in a Civil Court within two months
from the date of the order of the Co-operative Court, shall continue the proceedings and
decide the dispute]"
[Emphasis Added]
THE RULES
"Rule 8. Matters in respect of which Registrar may direct society to make bye-laws or
society may make bye-laws.-
(1) The Registrar may require a society to make bye-laws in respect of all or any of the
following matters, that is to say.-
* * * * *
(c) the object of the society;
*****
(f) the privileges, rights, duties and liabilities of members including nominal, associate
and sympathizer members;
* * * * *
(m) the procedure for expulsion of members;
(2) A society may make bye-laws for all or any of the following matters, that is to say
* * * * *
(c) the conditions, if any, under which the transfer of share or interest of a member may
be permitted;"
"Rule 10. Classification and sub-classification of societies.- (1) After registration of a
society, the Registrar shall classify the society into one or other of the following classes
and sub-classes of societies prescribed below according to the principal object provided
in its bye-laws :

Class Sub-Class Examples of societies falling in the class or sub-class, as


the case may be
1 2 3
* * * *
5 Housing Society (a) Tenant Ownership Housing Society
(b) Tenant Co-partnership Housing Soci-ety
(c) Other Housing Societies Housing Societies where land is held either on leasehold or
free-hold basis by Societies and houses are owned or are to be owned by members.
Housing Societies which hold both land and buildings either on lease-hold or free-hold
basis and allot them to their members.
House Mortgage Societies and House Construction Societies.

"Rule 28. Expulsion of members.- Any member who has been persistently defaulting
payment of his dues or has been failing to comply with the provisions of the bye-laws
regarding sales of his produce through the society or other matters in connection with his
dealings with the society or who, in the opinion of the committee, has brought disrepute
to the society or has done other acts detrimental to the interest or proper working of the
society may, in accordance with the provisions of sub-section (1) of Section 35, be
expelled from the society. Expulsion from membership may involve forfeiture of shares
held by the member."
[Emphasis Added]
THE BYE-LAWS
"Bye-law 2.- The objects of the Society shall be :-
(a) To purchase plot No................... or to purchase building ..................................
constructed on Plot No. ..........................(referred to in the application for registration), or
any other plot or plots with the prior approval of the general meeting and of the Registrar
and to construct tenements on such plot or plots for the use of members;
*****
(f) To do all things necessary or expedient
@page-SC755
for the attainment of the objects specified in these bye-laws;"
"Bye-Law 7. All persons permanently residing in Bombay City and Suburban area or
who intend to settle down in Bombay City and Suburban area permanently and who have
signed the application for registration are original members. Other members shall be
admitted by the Committee. Every person on applying for membership shall deposit
Re.1/- as entrance fee and the value of at least five shares for which he shall receive a
copy of the bye-laws. Two adverse votes are sufficient to exclude an applicant. In case
where an application is refused, the deposit shall ordinarily be returned.
Note:- Resident is a person who resides in the house or tenement permanently and which
he does not leave for more than 4 months at a time."
"Bye-Law 12.- (1) A member shall be expelled from the Society by the vote of not less
than two-thirds of the members present and voting at a General Meeting of the Society on
a motion (which shall be final and conclusive) that in the opinion of the Meeting such
member has :
(a) been a persistent defaulter,
(b) willfully deceived the Society by false statements,
(c) been bankrupt or legally disabled,
(d) been criminally convicted of an offence involving moral turpitude,
(e) intentionally done any act likely to injure the credit of the Society,
(f) gravely misused the dwelling rented by him from the Society or habitually acted in it
in a disgraceful manner or in a manner which has caused serious offence to his
neighbours or
(g) without the previous written permission of the Managing Committee has let or sub-let
or given on caretaker or leave licence basis or used for accommodating paying guests or
disposed of in any other manner any portion of the dwelling
accommodation/shops/godowns/garages.
(h) failed to occupy his premises in the building of the society within a period not
exceeding six months from the date of the allotment of a flat."
"Bye-Law 64. No member shall be tenant of the Society unless he subscribes to such
number of shares as the Managing Committee prescribes."
"Bye-Law 64(a). A member to whom a tenement is allotted shall occupy it himself and
shall not assign, underlet, vacate or part with the possession of the tenement or any part
thereof without the previous consent in writing of the Managing Committee. Such
permission shall not be granted unless the member authorizes the society to recover rent
or compensation and taxes and other common charges from the sub-lettee, licensee or
caretaker. The society shall admit every such sub-lettee, licensee, or caretaker as a
nominal member of the Society".
[Emphasis Added]
THE REGULATIONS
"Regulation 4. No tenant shall assign, underlet, vacate or part with the possession of the
tenement or any part thereof without the previous consent in writing of the Society."
"Regulation 24. The rent shall be calculated as follows and shall be paid on the first day
of each calendar month: -
(a) A rent of 6¼ per cent per annum (which shall not be increased during the tenancy) on
the cost including the building, land, roads and other items, such cost to be certified by
the Committee whose decision shall be final and conclusive and to be paid by 12 equal
calendar monthly payments.
(b) A further rent during the term of 25 years of ...... Per cent per annum (which shall not
be increased during the said term of 25 years except for a new tenant) on the said cost
such rent to be applied to the share account of the tenants and to be paid by 12 equal
calendar monthly payments and it is anticipated that when all these payments are made
the dividend on the shares will be equal to the rent paid under clause 24(a) hereof.
(c) A further rent equal to the proportion (applicable to the tenement) of the expense
incurred from time to time in insurance against fire, tempest or flood or violence by an
army or mob or other irresistible force and in the management of the Society and the
maintenance and repair of the Society's Estate such expense and proportion thereof
payable by the tenant to be determined by the Certificate of the Committee whose
decision shall be final and conclusive such further rent to be paid on the 1st day of the
calendar month next following the date of the said certificate.
(d) A further rent equal to the proportion
@page-SC756
applicable to the tenement of the sum or sums from time to time paid by the Society in
respect of assessment and rates such proportion to be determined by the Certificate of the
Committee whose decision shall be final and conclusive such further rent to be paid on
the 1st day of the calendar month next following the date of the said certificate.
I agree to take the tenement known as....................... subject to the above regulations
which I agree to observe and perform and by which I agree to be bound"."
[Emphasis added]
THE RENT ACT
"Section 5. Definitions.- In this Act unless there is anything repugnant to the subject or
context,-
* * * * *
(3) "landlord" means any person who is for the time being, receiving, or entitled to
receive, rent in respect of any premises whether on his own account or on account, or on
behalf, or for the benefit of any other person or as a trustee, guardian, or receiver for any
other person or who would so receive the rent or be entitled to receive the rent if the
premises were let to a tenant; and includes any person not being a tenant who from time
to time derives title under a landlord; and further includes in respect of his sub-tenant, a
tenant who has sublet any premises; [and also includes in respect of a licensee deemed to
be a tenant by section 15A, licensor who has given such license, [and in respect of the
State Government, or as the case may be, the Government allottee referred to in sub-
clause (b) of clause (1A), deemed to be a tenant by section 15B, the person who was
entitled to receive the rent if the premises were let to a tenant immediately before the
coming into force of the Bombay Rents, Hotel and Lodging House Rates Control,
Bombay Land Requisition and Bombay Government Premises (Eviction) (Amendment)
Act, 1996]]."
*****
"(11). "tenant" means any person by whom or on whose account rent is payable for any
premises and includes,-
(a) such sub-tenants and other persons as have derived title under a tenant [before the 1st
day of February 1973;]
[(aa) any person to whom interest in premises, has been assigned or transferred as
permitted or deemed to be permitted, under section 15;]
(b) any person remaining after the determination of the lease, in possession, with or
without the assent of the landlord, of the premises leased to such person or his
predecessor who has derived title [before the first day of February 1973;]
[(bb) such licensees as shall deemed to be tenants for the purposes of this Act by section
15A]
[(bba) the State Government, or as the case may be, the "'Government allottee, referred to
in sub-clause (b) of clause (1A), deemed to be a tenant, for the purposes of this Act by
section 15B;].
[(c) (i) in relation to any premises let for residence, when the tenant dies, whether the
death has occurred before or after the commencement of the Bombay Rents, Hotel and
Lodging House Rates Control (Amendment) Act, 1978, any member of the tenant's
family residing with the tenant at the time of his death or, in the absence of such member,
any heir of the deceased tenant, as may be decided in default of agreement by the Court;
(ii) in relation to any permission let for the purposes of education, business, trade or
storage, when the tenant dies, whether the death has occurred before or after the
commencement of the said Act, any member of the tenant's family using the premises for
the purposes of education of carrying on business, trade or storage in the premises, with
the tenant at the time of his death, or, in the absence of such member, any heir of the
deceased tenant, as may be decided in default of agreement by the Court.
Explanation. The provisions of this clause for transmission of tenancy, shall not be
restricted to the death of the original tenant, but shall apply, and shall be deemed always
to have applied, even on the death of any subsequent tenant, who becomes tenant under
these provisions on the death of the last preceding tenant.".]
"Section 15. [In absence of contract to the contrary, tenant not to sub-let or transfer] [or to
give on licence].-
[(1)] Notwithstanding anything contained in any law [but subject to any contract to the
contrary,] it shall not be lawful after the coming into operation of this Act for any tenant
to sub-let the whole or any part of the
@page-SC757
premises let to him or to assign or transfer in any other manner his interest therein [and
after the date of commencement of the Bombay Rents, Hotel and Lodging House Rates
Control (Amendment) Act, 1973, for any tenant to give on licence the whole or part of
such premises]:
[Provided that the [State] Government may by notification in the Official Gazette, permit
in any area the transfer of interest in premises held under such [leases or class of leases
[or the giving on licence any premises or class of premises] and on such extent as may be
specified in the notification.]
[(2) The prohibition against the sub-letting of the whole or any part of the premises which
have been let to any tenant, and against the assignment or transfer in any other manner of
the interest of the tenant therein, contained in sub-section (1), shall, subject to the
provisions of this sub-section be deemed to have had no effect [before the 1st day of
February, 1973], in any area in which this Act was in operation before such
commencement; and accordingly, notwithstanding anything contained in any contract or
in the judgment, decree or order of a Court, any such sub-lease, assignment or transfer of
any such purported sub-lease, assignment or transfer in favour of any person who has
entered into possession, despite the prohibition in sub-section (1) as purported sub-lessee,
assignee or transferee and has continued in a possession [on the date aforesaid] shall be
deemed to be valid and effectual for all purposes, and any tenant who has sub-let any
premises or part thereof, assigned or transferred any interest therein, shall not be liable to
eviction under clause (e) of sub-section (1) of section 13.
The provisions aforesaid of this sub-section shall not affect in any manner the operation
of sub-section (1) after the [date aforesaid]".
"Section 15A. Certain licensees in occupation on 1st February, 1973 to become tenants.-
(1) Notwithstanding anything contained elsewhere in this Act or anything contrary in any
other law for the time being in force, or in any contract where any person is on the 1st
day of February, 1973 in occupation of any premises, or any part thereof which is not less
than a room, as a licensee he shall on that date be deemed to have become, for the
purpose of this Act, the tenant of the landlord, in respect of the premises or part thereof,
in his occupation.
(2) The provisions of sub-section (1) shall not affect in any manner the operation of sub-
section (1) of section 15 after the date aforesaid".
"Section 28. Jurisdiction of Courts.-
[(1)] Notwithstanding anything contained in any law and notwithstanding that by reason
of the amount of the claim or for any other reason, the suit or proceeding would not, but
for this provision, be within its jurisdiction.-
(a) in Greater Bombay, the Court of Small Causes, Bombay [*]
[(aa) in any area for which, a Court of Small Causes is established under the Provincial
Small Cause Courts Act, 1887, such Court, and]
(b) elsewhere, the Court of the Civil Judge (Junior Division) having jurisdiction in the
area in which the premises are situate or, if there is no such Civil Judge the Court of the
Civil Judge (Senior Division) having ordinary jurisdiction,
shall have jurisdiction to entertain and try any suit or proceeding between a landlord and
a tenant relating to the recovery of rent or possession of any premises to which any of the
provisions of this Part apply [or between a licensor and a licensee relating to the recovery
of the licence fee or charge] and to decide any application made under this Act and to
deal with any claim or question arising out of this Act or any of its provisions-and subject
to the provisions of sub-section (2) no other court shall have jurisdiction to entertain any
such suit, proceeding, or application or to deal with such claim or question.
[(2) (a) Notwithstanding anything contained in clause (aa) of sub-section (1), the District
Court may at any stage withdraw any such suit, proceeding or application pending in a
Court of Small Causes established for any area under the Provincial Small Cause Courts
Act, 1887, and transfer the same for trial or disposal to the Court of the Civil Judge
(Senior Division) having ordinary jurisdiction in such area.]
(b) Where any suit, proceeding or application has been withdrawn under clause (a), the
Court of the Civil Judge (Senior Division) which thereafter tries such suit,
@page-SC758
proceedings or application, as the case may be, may either re-try it or proceed from the
stage at which it was withdrawn.
(c) The Court of the Civil Judge trying any suit, proceeding or application withdrawn
under clause (a) from the Court of Small Causes, shall, for purposes of such suit,
proceeding or application, as the case may be, be deemed to be the Court of Small
Causes.
Explanation.- In this section "proceeding" does not include an execution proceeding
arising out of a decree passed before the coming into operation of this Act."
[Emphasis added]
11. In our country, for the first time, Co-operative Credit Societies Act, 1904 was passed,
to encourage thrift, self-help and co-operation among agriculturists, artisans and persons
of limited means, and for that purpose to provide for the constitution and control of co-
operative credit societies, which laid down the foundation of co-operative law. Under that
Act ten members could form a society and the object of the society was to raise funds
either from members or outsiders and give loans to the needy members out of the funds
so collected. The principle of limited liability was also recognized and the concept of
profit motive was given a go-by. The tendency towards concentration of wealth in a few
hands was discouraged by providing that no member could hold shares beyond a certain
limit. The said Act was replaced by the Co-operative Societies Act, 1912, which was
repealed by the Bombay Co-operative Societies Act, 1925. The aforementioned Act was
repealed by the Maharashtra Co-operative Societies Act, 1960 to consolidate and amend
the law relating to co-operative societies in the State of Maharashtra the objective of
which was to provide for the orderly development of the co-operative movement in the
State in accordance with the Directive Principles of State Policy enshrined in Part Four of
the Constitution of India.
12. 'Society' has been defined under Section 2(27) of the Societies Act to mean a co-
operative society registered or deemed to be registered under the Societies Act. Under
Section 2(5) 'bye-laws' means bye-laws registered under the Societies Act and for the
time being in force and includes registered amendments of such bye-laws. Under Section
2(16) 'housing society' has been defined to mean a society, the object of which is to
provide its members with open plots for housing, dwelling houses or flats; or if open
plots, the dwelling houses or flats are already acquired, to provide its members common
amenities and services. Section 2(19) defines 'member' to be a person joining in an
application for the registration of a co-operative society which is subsequently registered,
or a person duly admitted to membership of a Society after registration and includes an
associate, nominal or sympathiser member. Under Section 2(19)(c) 'nominal member' is a
person admitted to membership as such after registration in accordance with bye-laws.
Section 2(21) defines the expression 'prescribed' to mean prescribed by Rules. Under
Section 2(24) 'Registrar' has been defined to mean a person appointed as Registrar of Co-
operative Societies under the Societies Act. 'Rules' under Section 2(26) means Rules
made under the Societies Act.
13. Section 4 of the Societies Act lays down that a Society which has as its objects the
promotion of economic interests or general welfare of its members or of the public in
accordance with co-operative principles or a Society established with the object of
facilitating the operations of any such Society, may be registered under the Societies Act.
Under Section 9, upon the satisfaction of the Registrar, Co-operative Society and its bye-
laws are registered on an application made to that effect under Rule 8. Section 12 lays
down that the Registrar shall classify all societies into one or other of the classes of
societies defined in Section 2 and also into such sub-classes thereof, as may be
prescribed. Rule 10 prescribes such classification of the Societies and under Rule 10(1)
(5) three types of housing societies have been enumerated. First is under Rule 10(1)(5)(a)
which is a 'tenant ownership housing society' where land is held either on leasehold or
freehold basis by the Society and houses are owned or are to be owned by members. The
second type of housing society under Rule 10(1)(5)(b) is 'tenant co-partnership housing
society' which holds both land and buildings either on leasehold or freehold basis and
allots them to its members. The third type under Rule 10(1)(5)(c) is 'other housing
societies' which are called house mortgage and house construction societies and the same
do not come in any of the categories of housing societies referred to above.
@page-SC759
14. Section 22 of the Societies Act enumerates the persons who are eligible to be
admitted as members of a society. Section 23 lays down that no society shall, without
sufficient cause, refuse admission to membership to any person duly qualified therefor
under the provisions of the Societies Act and its bye-laws. Section 24 lays down that even
if any person does not fulfill the eligibility requirement enumerated in Section 22 of the
Societies Act, he can be admitted by the Society as a nominal, associate or sympathizer
member. According to Section 25 a person shall cease to be a member of the Society on
his resignation from the membership thereof being accepted or on the transfer of the
whole of his share or interest in the Society to another member or on his death or removal
or expulsion from the Society or where a firm, company, any other corporate body,
society or trust is its member, on its dissolution or ceasing to exist. Under Section 35 a
society is empowered by resolution passed by a majority of not less than three-fourths of
the members entitled to vote, who are present at its general meeting held for the purpose,
to expel a member for acts which are detrimental to the interest or proper working of the
Society. Under Rule 28 of the Rules any member who has been persistently defaulting
payment of dues or has failed to comply with the provisions of the bye-laws regarding
sales of his produce through the Society or other matters connected therewith in
connection with his dealings with the Society or who, in the opinion of the committee,
has brought disrepute to the Society or has done other acts detrimental to the interest or
proper working of the society may, in accordance with the provisions of sub-section (1)
of Section 35, be expelled from the society. Under bye-law 12 a member is required not
to let or sub-let or give on caretaker or leave and licence basis or use for accommodating
paying guests or dispose of in any manner any portion of the tenement without the
previous written permission of the Managing Committee of the Society and failure
thereof tantamounts to violation of the bye-laws within the meaning of Rule 28 and such
a member would be liable to be expelled thereunder.
15. Section 29(2) of the Societies Act provides that a member shall not transfer, inter alia,
any interest in the property of any society or any part thereof unless two conditions are
fulfilled, namely, the member has held such interest for not less than one year and the
transfer is made either to an existing member of the Society or to a person whose
application for membership has been accepted by the society or to a person whose appeal
under Section 23 of the Societies Act has been allowed by the Registrar or to a person
who is deemed to be a member under sub-section (1A) of Section 23 of the Societies Act.
16. Under Rule 8(2) of the Rules, a society may make bye-laws prescribing thereunder,
inter alia, conditions, if any, under which the transfer of share held by a member or
interest of a member in the tenement allotted to him may be permitted. Under bye-law
64(a) a member to whom a tenement is allotted shall be entitled to occupy the same
himself and is not permitted to assign, underlet, vacate or part with the possession of the
same or any part thereof without the previous consent in writing of the Managing
Committee of the Society, which permission shall not be granted unless the member
authorises the society to recover rent or compensation and taxes and other common
charges from the sub-lettee, licensee or caretaker who must agree to become its nominal
member. Regulation 4 of the Regulations, which every member at the time of his
admission to the membership of the Society is required to observe by giving an
undertaking in writing, provides that no tenant shall assign, underlet, vacate or part with
possession of the premises or any part thereof without the previous consent in writing of
the Society.
17. Bye-law 2(a) enumerates objects of the Society one of which would be to purchase
plot and to make constructions thereon for use of its members. The further objects of the
Society are to advance loans to its members, guarantee loans to its members for acquiring
building sites, constructing houses, to receive or guarantee repayments in lump sum or in
instalments and to do all things necessary or expedient for the attainment of the objects
specified in the bye-laws, meaning thereby that every member to whom a plot or flat is
allotted for his self-occupation and use shall not part with possession thereof unless
permitted by the Society, which, at the time of grant of permission, shall admit such
persons in whose favour the member intends to part with possession by admitting him as
a nominal
@page-SC760
member. Bye-law 7 lays down that all persons permanently residing in Bombay City and
Suburban areas or those who intend to settle down in Bombay City and Suburban areas
shall be eligible to be admitted as members by the Committee of the Society. Bye-law 64
requires that no member shall be a tenant of the society in respect of the tenement unless
he subscribes to such number of shares as the Managing Committee prescribes, which is
five paid-up shares in the society as per regulation 1 of the Regulations and upon
subscribing to such shares under bye-law 64 such a member shall be allotted a tenement
for his self-occupation who shall not part with possession thereof in any manner without
the previous consent in writing of the Managing Committee and while granting such
permission the person in whose favour the member intends to part with possession shall
be admitted as nominal member.
18. Under Regulation 2, the committee of the Society is required to maintain a register of
applicants for the tenements and the tenements would be offered to the members in the
order in which they appear in the register and in the event of two or more members
having made application on the same day, the one to whom the offer is to be made shall
be determined by lot. Regulation 3 permits the tenancy to continue as long as the tenant
or his successor in all respects observes and performs the obligations imposed upon him
under the Regulations as a tenant. Regulation 4 forbids a tenant to part with the
possession of the tenement without the previous consent in writing of the Society.
Regulation 24 enumerates the manner in which rent would be calculated. The cost of the
land, building, road and other items to be determined by the committee of the Society and
6-1/4% thereof shall be paid every year in 12 equal monthly instalments. In addition to
the aforesaid rent, the tenant is required to make certain other payments by way of rent as
required under sub-clauses (b) to (d) of Regulation 24. In the present case, the member in
whose favour tenement, i.e., the flat, has been allotted had undertaken in writing that he
agreed to take the tenement subject to the Regulations and agreed to observe the same.
19. Section 31 gives immunity to the share or interest of a member in the capital of a
Society or in the loan stock issued by Society or in the funds raised by a Society from its
members by way of savings deposit, from attachment or sale under any decree or order of
a court for or in respect of any debt or liability incurred by the member. According to the
provisions of Section 36 of the Societies Act, a Society, upon its registration, shall be
treated to be a body corporate, as such a juristic person. Section 47(2) of the Societies Act
lays down that interest of a member in those properties of the society whereupon a charge
has been created under sub-section (1) of Section 47 shall not be transferred in any
manner without the previous permission of the Society and for according permission the
Society may impose such conditions as it may deem fit and proper. According to Section
47(3) any transfer made in contravention of sub-section (2) of Section 47 shall be void.
Under Section 79AA Regulations can be framed by the Society for carrying on its trade
or business on the direction of the Registrar and the same are required to be approved by
him.
20. Section 91, which begins with a non obstante clause, lays down that if there is any
dispute, inter alia, touching the business of a society, the same shall be referred to
Cooperative Court by, inter alia, any of the parties to the dispute. Section 91(3) lays down
that, except as provided under sub-section (2) of Section 93 of the Societies Act, no court
shall have jurisdiction to entertain any suit or proceeding in respect of any dispute
referred to in sub-section (1) of Section 91 of the Societies Act. Section 93(2), which also
begins with a non obstante clause, provides that the Co-operative Court may, on an
application made to it by any of the parties to dispute, if it thinks fit, suspend any
proceedings in respect of any dispute, if the question at issue between a society and a
claimant or between different claimants is one involving complicated questions of law
and fact, until the question has been tried by a regular suit instituted by one of the parties
or by the Society, but if any such suit is not instituted in a Civil Court within two months
from the date of the order passed by the Co-operative Court, the proceedings under
Section 91(1) shall continue and dispute raised by the parties shall be decided by the Co-
operative Court. Section 94(1) lays down that the procedure for adjudication of the
aforesaid disputes raised by the parties shall, as far as possible, be the same as provided
in the case of Civil Court by the Code of Civil Procedure, 1908.
@page-SC761
Section 97 prescribes appeal before the Co-operative Appellate Court against award of
the Co-operative Court adjudicating the dispute raised under Section 91 of the Societies
Act.
21. According to Section 146(1)(a) if any member of a society makes any transfer of any
property or interest in property in contravention of sub-section (2) of Section 47 or any
persons knowingly acquires or abets in the acquisition of such property, the same, inter
alia, would be an offence under the Societies Act, punishment for which has been
provided under Section 147 of the Societies Act. Section 165 empowers the State
Government to make Rules for, inter alia, the conduct and regulation of business of
Society and carrying out the purposes of the Societies Act.
22. The Rent Act was enacted to amend and consolidate the law relating to the control of
rents and repair of certain premises, of rates of hotels and lodging house and of evictions
and also to control the charges for licence of premises etc. Section 5(3) of the Rent Act
defines a 'landlord' to be a person who, for the time being, is receiving or entitled to
receive rent of any premises from a tenant, which includes a licensee who is deemed to
be a tenant within the meaning of Section 15A of the Rent Act. Section 5(11) defines
'tenant' to mean a person by whom or on whose account rent is payable for any premises
which, inter alia, includes a licensee, who is a deemed tenant under Section 15A of the
Rent Act but shall not include such a sub-tenant to whom interest in the premises has
been purported to have been assigned or transferred in the absence of any contract to the
contrary as required under Section 15 of the Rent Act. Section 13 of the Rent Act
enumerates the grounds for eviction of a tenant which includes bona fide need of the
landlord. Section 15 of the Rent Act lays down that in the absence of any contract to the
contrary, it shall not be lawful for any tenant to sublet the whole or any part of the
premises in which he was inducted as a tenant. Section 15A lays down that if any licensee
who was in occupation of the premises, in which he was inducted as a licensee, before the
1st day of February, 1973 and the licence was subsisting on that date, he shall be deemed
to have become a tenant of the landlord in respect of the premises on that date i.e.,
1.2.1973 when Maharashtra Act 17 of 1973 came into force whereby Section 15A was
inserted in the Rent Act.
23. Section 28, which starts with a non obstante clause, lays down that suit or proceeding
between a landlord and a tenant relating to recovery of rent or possession of any premises
to which provisions of Rent Act apply shall be entertained by a court enumerated
thereunder and no other court shall have jurisdiction to entertain any such suit,
proceeding or application or to deal with such claim or question.
24. We now proceed to deal with the first question, i.e., whether, in the present case,
status of the member was that of a tenant or landlord within the meaning of the Rent Act
and consequently there was any relationship of landlord and tenant between them? Under
Rule 10(1)(5)(a) 'tenant ownership housing society' has been defined to mean housing
society where land is held either on leasehold or freehold basis by the societies and
houses are owned by its members, whereas under Rule 10(1)(5)(b) in case of 'tenant co-
partnership housing Society', the society holds both land and buildings either on
leasehold or freehold basis and allots them to its members. In the case of tenant co-
partnership housing society, it is clear from the Rules that the ownership of the land and
building both remains with the Society and member cannot be said to be co-owner, but in
the case of tenant ownership housing society, the ownership of the land remains with the
society, but ownership of the building/flat vests in the member. So far as tenant within the
meaning of Section - 5(11) of the Rent Act is concerned, he has a mere right to occupy.
He is entitled to the protection of the Statute so long as grounds for eviction are not made
out and can be evicted only by instituting a suit in a court enumerated under Section 28 of
the Rent Act.
25

. The concept of tenant co-partnership housing society was considered by this Court in
the case of Sanwarmal Kejriwal vs. Vishwa Co-operative Housing Society Ltd., (1990) 2
SCC 288, wherein it was noticed that the title to the property, i.e., the land and
building/flat both, vests in the Society. It has been further noted that cost of construction
of the dwelling is met from deposits and rent besides the share money. The rental is
usually determined on long term basis so calculated as to meet the cost of construction
AIR 1990 SC 1563
AIR 1990 SC 1563, (Para 12)

@page-SC762
and upkeep of the building and to guarantee perpetuity of occupation on repayment of the
whole value of the tenement or flat. At the end of the period the member is credited with
additional shares equal to the amount paid by him, the interest on these shares generally
matches the rental payable by him to the society. This Court has concluded that on full
payment, the member becomes entitled to occupy the tenement or flat free of charge as
the rental he has to pay to the society is almost met from the interest received from shares
held by him and, consequently, a member has more than a mere right to occupy the same.
In this regard, we may usefully refer to paragraph 13 of the case of Sanwarmal [supra]
which reads thus :-
"13. That takes us to the next question whether or not a member of a copartnership type
of a co-operative society has such interest in the premises allotted to him as would entitle
him to give the same on leave and licence basis to a non-member. In a tenant co-
partnership type of society the members are shareholders; but the title to the property
vests in the society which in turn rents the tenements or flats to its members. The cost of
construction of dwellings is met from deposits and loans besides the share money. The
rental is usually determined on long term basis so calculated as to meet the cost of
construction and upkeep of the building and to guarantee perpetuity of occupation on
repayment of the whole value of the tenement or flat. At the end of the period the member
is credited with additional shares equal to the amount paid by him; the interest on these
shares generally matches the rental payable by him to the society. Thus on full payment
the member becomes entitled to occupy the tenement of flat free of charge as the rental
he has to pay to the society is almost met from the interest received from shares held by
him. Thus a member has more than a mere right to occupy the flat."
[Emphasis added]
26. From a bare perusal of the aforesaid passage, it would be clear that after full payment
a member is entitled to continue to occupy the tenement free of charge and neither he is
liable to pay any rent to the society nor the society is entitled to receive any rent from the
member. According to the definition of 'landlord' under Section - 5(3) of the Rent Act,
landlord is a person who is for the time being receiving or entitled to receive rent and
under Section 5(11) a tenant is liable to pay rent, but in view of the observations of this
Court in Sanwarmal (supra), neither the Society is entitled to receive rent from the
member nor member is liable to pay any rent to the Society after the entire value of the
land and cost of construction of the building together with interest on its value has been
paid.
27. The matter may be examined from another angle. If it is held that the Society is a
landlord and the member is a tenant within the meaning of the Rent Act, in that event the
society can evict the member by filing a suit for eviction if it requires the premises for its
bona fide need, but under the Societies Act, it can evict the member only as a
consequence of his expulsion from the membership and neither under the Societies Act
nor Rules framed thereunder nor Bye-Laws nor Regulations there is any provision that a
Society can evict a member in case it has got bona fide need of the same. The said
interpretation would be contrary to the object of the Societies Act.
28. It appears to us that the status of a member in a tenant co-partnership housing Society
is very peculiar. The ownership of the land and building both vests in the Society and the
member has, for all practical purposes, right of occupation in perpetuity after the full
value of the land and building and interest accrued thereon have been paid by him.
Although de jure he is not owner of the flat allotted to him, but, in fact, he enjoys almost
all the rights which an owner enjoys, which includes right to transfer in case he fulfills
the two pre-conditions, namely, he occupies the property for a period of one year and the
transfer is made in favour of a person who is already a member or a person whose
application for membership has been accepted by the Society or whose appeal under
Section 23 of the Societies Act has been allowed by the Registrar or to a person who is
deemed to be a member under sub-section (1A) of Section 23 of the Societies Act. In case
any of these two conditions is not fulfilled, a member cannot be said to have any right of
transfer. Thus, we reiterate the law laid down by this Court in the case of Sanwarmal
(supra) that a member has more than a mere right to occupy the flat, meaning thereby
higher than tenant, which is not so in the case of a
@page-SC763
tenant within the meaning of Section 5(11) of the Rent Act. This being the position, we
have no difficulty in coming to the conclusion that the status of a member in the case of
tenant co-partnership housing society cannot be said to be that of a tenant within the
meaning of Section 5(11) of the Rent Act, as such there was no relationship of landlord
and tenant between the Society and the member.
29. We now turn to the second question, i.e., as to whether the status of the appellants was
that of tenant or sub-tenant. In view of our answer to question No. 1 that the status of the
member was higher than a tenant and although de jure he was not an owner but, for all
practical purposes, he was exercising almost all the rights of an owner, excepting absolute
right of transfer, he not being the tenant, there is no question of his creating sub-tenancy
in favour of the appellants. A member may not be an owner of the flat in the eye of law
but he may still be a landlord within the meaning of the Rent Act which does not
necessarily postulate a landlord to be an owner of the property, but if a person is entitled
to receive rent or receiving rent he may be treated to be a landlord within the meaning of
the Rent Act. The question arises as to whether such a member could create a tenancy
right under law, meaning thereby whether the relationship of landlord and tenant between
the member and the appellants was duly created so as to claim protection from eviction
under the Rent Act. The factum of letting out by the member to the appellants is not in
dispute. Purported creation of tenancy right in favour of the appellants was in infraction
of the provisions of Section 29(2) of the Societies Act whereunder there is a legislative
command to the member not to transfer his interest in the property of the Society unless
two conditions are fulfilled, firstly, the member has held the interest for a period of not
less than one year, meaning thereby that he has remained in occupation of the flat, which
was allotted to him by the Society, for a period of one year and, secondly, transfer was
made to a member of the Society or to a person whose application for membership has
been accepted by the Society or to a person whose appeal under Section 23 of the
Societies Act has been allowed by the Registrar and his application for membership has
been accepted by him or to a person who is deemed to be a member under sub-section
(1A) of Section 23. In the present case, the first pre-requisite is fulfilled, but so far as the
second one is concerned, the appellants who claim to be tenants were not existing
members of the Society nor they ever filed any application for membership of the
Society, much less its acceptance nor it has been claimed that they shall be deemed to
have become members of the Society under sub-section (1A) of Section 23 of the
Societies Act.
30. Undisputedly, in the present case, there is infraction of the provisions of Section 29(2)
of the Societies Act. Now it has to be seen whether sub-section (2) of Section 29 is
mandatory or directory. From the scheme of the Societies Act, the Rules, Bye-Laws and
Regulations it would be clear that in a case of tenant co-partnership Society the
ownership of the land and the building both vest in the Society and the premises is
allotted to the member for his occupation only and not for the purpose of occupation of
anybody else. That is the object of the Societies Act as would appear from bye-law 2, i.e.,
'for use of the member', meaning thereby his own use. According to bye-law 64, the flat is
allotted to a member for occupying it himself. Regulation 4 is also in line with bye-law
64 which says that no tenant shall part with possession of the premises without the
previous consent in writing of the Society. Under bye-law 64(a) also a member is
restrained from parting with possession of the flat without permission of the Society
which shall be granted upon filing application, by an intending transferee, for grant of
membership by admitting him as a nominal member. Keeping in mind the language of
Section 29(2), which is in the nature of injunction upon the right of a member to transfer
unless the twin conditions are fulfilled, and the purpose for which the house is allotted to
a member, i.e., for his self-occupation, it cannot be said in any manner that the said
provision is directory as giving such an interpretation would frustrate object of the
Societies Act whereunder a flat is allotted to a member for his self-occupation as would
appear from the Societies Act, Rules, Bye-Laws and Regulations. As such, we have no
option but to hold that the provisions of Section 29(2) are mandatory.
31

. The question that arises now is that if there is any infraction of the said provision,
whether the same would invalidate the AIR 1975 SC 1470
AIR 1990 SC 1563
@page-SC764
creation of relationship of landlord and tenant between the member and the appellants. It
has been submitted on behalf of the appellants that the member had absolute right of
transfer, as such relationship of landlord and tenant was duly created. In support of the
submission, reliance was placed upon two decisions of this Court in the cases of Ramesh
Himmatlal Shah vs. Harsukh Jadhavji Joshi, (1975) 2 SCC 105 and Sanwarmal [supra].
In Ramesh [supra], the question in issue was as to whether interest of a member in a flat
allotted to him by co-partnership housing society could be attached and sold in execution
of a decree passed against a member. This Court referred to the provisions of Sections
29(2), 31 and 47 of the Societies Act. The Court was considering matter in the light of the
provisions of Section 31 of the Societies Act which lays down that share or interest of a
member in the capital of a society or in the loan stock issued by a Society or in the funds
raised by a Society from its members shall not be liable to attachment or sale under any
decree or order of a court in respect of any debt or liability incurred by a member. Under
this provision, there was immunity from attachment/sale in execution of a decree to
certain types of interests of a member of the society, but the interests of a member in the
flat was not enumerated thereunder, as such there was no provision exempting interest of
a member in the flat allotted to him from being attached and sold in execution of a decree
against a member. Therefore, there was no legal bar in attachment and sale of interest of a
member in the flat allotted to him by the Society. Before this Court the question was
raised that the restriction under Section 29(2) put on the right of a member to transfer
should be equally applied in the case of auction sale in execution of decree against a
member. Court observed that "the only restrictions under Section 29(2) are that the
member may not transfer his interest in the property prior to one year and the transfer is
made to an existing member of the Society or to a person whose application for
membership has been accepted by the Society". Though the Court has considered the
provisions of Section 47 which lay down that interest of a member in such a property
which was allotted to him and upon which there is a charge, cannot be sold by a member
without permission of the society and if there is infraction thereof, such a transaction has
been declared by Section 47(3) of the Societies Act to be void, obviously the restrictions
under Section 29(2) providing thereunder restricted right of voluntary transfer cannot
apply to auction sale in execution of a decree against a member which is distress sale as
the same has not been specifically exempted under Section 31 of the Societies Act. Of
course, while dealing with the provisions of Section 29(2), the Court was also having in
mind that under Section 47(3) if the transfer is in infraction of the provisions of Section
47(2) the same has been declared, to be void, by the Statute itself whereas in a case of
infraction of Section 29(2) of the Societies Act, the Statute is silent. As a matter of fact,
the question as to what would be the effect of infraction of the provisions of Section
29(2) was not subject-matter of consideration before this Court in the case of Ramesh
(supra), as such the law laid down therein that there was no prohibition against transfer of
right to occupy a flat was in the light of the fact whether interest of the member in the flat
could be auctioned/sold in execution of a decree passed against the member, meaning
thereby distress sale. In the present case we are concerned with voluntary transfer which
is in contra-distinction to distress sale, as such the said case is clearly distinguishable.
32. Another decision upon which reliance was placed on behalf of the appellants is the
case of Sanwarmal [supra]. In that case, a petition was filed under Section 91 of the
Societies Act for passing an order of eviction of a licensee who was inducted as a licensee
by a member of the Society, though contrary to the provisions of Section 29(2) of the
Societies Act, but he became deemed tenant under Section 15A of the Rent Act on
1.2.1973, i.e., the date on which the amending Act came into force, as the licensee was in
occupation of the premises and licence was subsisting on that date, as such this Court
held that the petition under Section 91 of the Societies Act was not maintainable as the
licensee acquired the status of tenant even though there was no privity of contract
between the parties, but as the status of tenant was acquired, by legislative intervention,
under Section 15A of the Rent Act, as such he was entitled to claim protection under the
said Act. Though in that case the question whether right to occupy a flat in a case of
tenant co-partnership
@page-SC765
society is transferable or not was not in issue before the Court, but following the
judgment in the case of Ramesh [supra], this Court observed that the right of a member in
the flat is transferable. In our view, the aforesaid observation in the case of Sanwarmal
[supra) cannot be said to be ratio of the case but a mere obiter, as such the same can be of
no avail to the appellants.
33. It has been submitted that in case transfer has been made by a member in infraction of
the provisions of Section 47(2) of the Societies Act, according to the provisions of
Section 47(3) the same shall be void, but there is no such provision in case there is
infraction of the provisions of Section 29(2) which, we have already found, are
mandatory. In case there is infraction of a mandatory provision, in that event the
transaction cannot be said to be void but would obviously be voidable and once avoided,
the relationship of landlord and tenant, i.e., between the member and the appellants
cannot be said to have been duly created, meaning thereby in accordance with law. Thus,
we have no difficulty in holding that the relationship of landlord and tenant between the
appellants and the member was not duly created, as such the appellants would not be
entitled to claim protection under the Rent Act and the bar created under Section 28 of the
Rent Act would not operate.
34. This takes us to the next question whether legality or otherwise of the creation of
relationship of landlord and tenant between the member and the appellants could be
adjudicated in the suits filed by the appellants before the Court of Small Causes for
declaration that they were tenants in the premises in their respective occupation and there
was relationship of landlord and tenant between the member and the appellants. The said
suits were dismissed by the trial court after recording a finding that no relationship of
landlord and tenant existed between the member and the appellants, but on appeal being
taken the appellate Bench of the Court of Small Causes decreed the suits holding that
relationship of landlord and tenant did exist between the member and the appellants after
reversing decree passed by trial court. When the matter was taken to the High Court in
writ applications, the same have been allowed, decrees passed by the appellate Bench of
the Small Cause Court set aside and those of the trial court restored whereby it was held
that there was no relationship of landlord and tenant between the member and the
appellants. When suits were filed before the Small Cause Court by the appellants for a
declaration that there was relationship of landlord and tenant between them and the
member, it was open to the defendants to take a defence that no such relationship was
created either in fact or in law as creation of such a right was barred under Section 29(2)
of the Societies Act. In the present case the factum of creation of tenancy has not been
disputed, but what has been disputed is its legality. As the creation of tenancy was in
infraction of mandatory provisions of Section 29(2), it was voidable and invalid in law
although not void and the Small Cause Court was not only competent to decide the same
but obliged under law to go into the same before granting or refusing relief to the plaintiff
as the same was a point in issue in those suits. This being the position, we are of the view
that the High Court was justified in setting aside the decrees passed by the appellate
Bench of the Small Cause Court and restoring those of the Small Cause Court whereby
suits for declaration were dismissed after recording a finding that there was no
relationship of landlord and tenant between the member and the appellants who were
consequently not entitled to claim protection under the Rent Act and no interference by
this Court is called for.
35. The last question that falls for decision is as to whether the Society was required to
first obtain adjudication from a competent civil court by filing a properly constituted suit
for a declaration that relationship of landlord and tenant was not duly created and,
therefore, the induction of a person by the member as tenant was invalid, the same being
in infraction of mandatory provisions of Section 29(2) of the Societies Act before raising
a dispute under Section 91 of the Societies Act or the said question could be gone into in
a proceeding under Section 91 of the Societies Act before the Co-operative Court where a
dispute touching upon the business of the Society can be raised by the parties and in
deciding the said dispute was it permissible for the Court to go into the said question. It is
true that ordinarily in case of a transaction like the present one which is voidable and not
void, if an aggrieved party intends to avoid the same it is required to obtain a decree from
a competent civil court by filing a properly
@page-SC766
constituted suit. But in a case like the present one, if a party is first asked to obtain a
decree from a competent civil court and only thereafter raise a dispute which is
undisputedly touching upon the business of the Society under Section 91 of the Societies
Act, the same would frustrate the provisions of Section 91 and the intention of the
Legislature in incorporating a cheap and expeditious remedy by referring the same to a
court constituted under the Societies Act instead of throwing a party to cumbersome
procedure of moving a civil court.
36. The dispute raised in the present case, undoubtedly, touches upon business of the
Society which is a condition precedent for the applicability of Section 91 of the Societies
Act. The business of tenant co-partnership housing society is, after purchasing plots and
constructing houses/flats thereon, to allot the same to its members for their self-
occupation and for a period of one year they cannot part with possession of the same in
favour of anybody and on expiry of the said period can transfer the same in favour of
member of the Society or to a person whose application for membership has been
accepted by the Society or to a person whose appeal under Section 23 of the Societies Act
has been allowed by the Registrar or to a person who is deemed to be a member under
sub-section (1A) of Section 23 of the Societies Act. It is part of business of the Society to
see that the house/flat allotted to a member remains in his occupation or in occupation of
any other member and if any non-member intends a transfer in his favour, like the present
one, he is required to obtain previous consent in writing either of the Society or its
Managing Committee and in the event of consent being accorded, the Society shall admit
him as a nominal member in which eventuality only the transfer can be made in his
favour. In the present case, under Section 91 of the Societies Act, the Society was well
within its right to get a dispute adjudicated as to whether the member had, by inducting
the tenants in the flat, who were non-members, made a transfer in contravention of the
provisions of Section 29(2) of the Societies Act. Thus the question regarding legality or
otherwise of the creation of tenancy right by the member in favour of the appellants,
which amounts to transfer of interest of a member in the property of the Society, can be
decided by raising a dispute before the Co-operative Court.
37. In the present case, the only dispute raised before the Co-operative Court was as to
whether transfer made by a member in favour of a so-called transferee/tenant thereby
purporting to create a tenancy right in his favour was in infraction of the mandatory
provisions of Section 29(2) of the Societies Act, as such the same was touching upon
business of the Society. In case Co-operative Court decides such a dispute in favour of
the Society in that eventuality the so-called transferee/tenant would not be entitled to
claim any protection under the Rent Act, the bar provided under Section 28 of the Rent
Act would not operate and consequently the petition under Section 91 of the Societies Act
would be maintainable.
38

. Learned counsel appearing on behalf of the appellants submitted that even if the tenancy
is created in breach of law having statutory force, the same would not be void. Reliance
in this connection has been placed upon a decision of this Court in the case of Nanakram
vs. Kundalrai, (1986) 3 SCC 83, in which it was held that tenancy created in breach of
Rent Control Order, which was having a statutory force, was not void. In that case under
Clause 22 of the Central Provinces and Berar Letting of Houses and Rent Control Order,
1949 a landlord was required to report the matter to the Deputy Commissioner, upon
vacation of the premises, who alone was empowered to permit induction of any tenant,
but instead of adopting the said procedure, landlord himself inducted tenant which was
challenged by filing a suit before civil court for a declaration that creation of tenancy
right was invalid. The trial court decreed the suit and the said decree was confirmed in
appeal by the High Court. When the matter was brought to this Court, it was held that the
transaction was not void and the infraction alleged was not of mandatory provisions of
law which would obviously mean that the transaction was not even voidable, as such the
suit was liable to be dismissed. In our view, the case of Nanakram [supra] is quite
distinguishable and shall have no application to the present case as here there was
infraction of mandatory provisions of Section 29(2) of the Societies Act. Thus we hold
that the question regarding legality or otherwise of creation of relationship of landlord
and tenant between the member and AIR 1986 SC 1194

@page-SC767
the appellants could have been gone into by the Co-operative Court under Section 91 of
the Societies Act as it touches upon business of the Society and the High Court has not
committed any error in not interfering with the order passed by appellate court
confirming that rendered by the Co-operative Court.
39. For the foregoing reasons, we do not find any merit in these appeals which are
accordingly dismissed, but there shall be no order as to costs.
Appeal dismissed.
AIR 2008 SUPREME COURT 767 "Deddappa v. Branch Manager, National Insurance
Co. Ltd."
(From : Karnataka)
Coram : 2 S. B. SINHA AND H. S. BEDI, JJ.
Civil Appeal No. 5829 of 2007 (arising out of SLP (C) No. 7746 of 2006), D/- 12 -12
-2007.
Deddappa and Ors. v. The Branch Manager, National Insurance Co. Ltd.
Motor Vehicles Act (59 of 1988), S.147 - MOTOR VEHICLES - INSURANCE -
SUPREME COURT - Compensation - Liability of insurer - Cheque issued by insured
towards payment of premium was dishonoured - As a result policy of insurance was
cancelled - Insured was intimated about cancellation much before accident occurred -
Insurer not liable to pay compensation - However, since claimant hailed from lowest
strata of society, Court in exercise of powers under Art. 142, directed insurer to pay
compensation and recover it from owner.
Constitution of India, Art.142.
Insurance Act (4 of 1938), S.64VB.
The statutory liability of the Insurance Company vis-a-vis a third party in the context of
Sections 147 and 149 of the Act, arising under a contract of insurance would have to be
met if the contract is valid. If the contract of insurance has been cancelled and all
concerned have been intimated thereabout, the insurance company would not be liable to
satisfy the claim. A beneficial legislation as is well known should not be construed in
such a manner so as to bring within its ambit a benefit which was not contemplated by
the legislature to be given to the party.
Thus, where the cheque issued by insured towards premium was dishonoured and as a
result Policy of Insurance was cancelled and intimation of cancellation was given to
insured much prior to accident, the insured was not liable to pay compensation. However,
since claimant hailed from the lowest strata of society, Court in exercise of its powers
under Art. 142 directed insurer to pay compensation and recover it from owner.
(Paras 26, 28)
Cases Referred : Chronological Paras
2006 AIR SCW 2352 : AIR 2006 SC 1926 : 2006 Lab IC 2002 : 2006 (4) AIR Kar R 48 :
2006 (3) AIR Jhar R 127 (Ref.) 25
2001 AIR SCW 902 : AIR 2001 SC 1197 (Ref.) 8, 21
2000 AIR SCW 788 : AIR 2000 SC 182 (Ref.) 20
1998 AIR SCW 183 : AIR 1998 SC 588 (Ref.) 8, 16, 18
AIR 1985 SC 278 : 1985 Lab IC 544 (Ref.) 27
AIR 1976 SC 2229 (Ref.) 24
(1976)1 WLR 989 25
(1976) QB 44 25
C. M. Angadi and Rameshwar Prasad Goyal, for Appellants; Mrs. Pankaj Bala Verma,
Ms. Kiran Suri and Dr. Laxmi Shastri, for Respondent.
Judgement
1. S. B. SINHA, J. :- Leave granted.
2. This appeal is directed against the judgment and order dated 15.6.2005 passed by a
learned Single Judge of the High Court of Karnataka in M.F.A No. 5751 of 2002,
whereby and whereunder an appeal preferred by the respondent herein from the judgment
and order dated 12.06.2002 passed by the Motor Accidents Claims Tribunal in M.C.A.
No.113 of 2001 was allowed.
3. Shantamma, daughter of the appellant herein was sleeping in her hut. A tempo bearing
No.KA 37-2257 which was being rashly and negligently driven by Respondent No.2
herein ran over her. She died on the spot. Household articles of the appellant also were
damaged in the said accident.
4. An application for grant of compensation was filed by the appellants herein under
Section 166 of the Motor Vehicles Act, 1988 (for short "the Act") in the Court of C.J.
(SD) and Motor Accidents Claims Tribunal at Gangavati in the district of Koppal on
12.06.2006.
5. The said vehicle was insured with the National Insurance Company. A plea was taken
therein by the Insurance Company that although the vehicle in question was insured by
the owner for the period
@page-SC768
17.10.1997 and 16.10.1998, but the cheque issued therefor having been dishonoured, the
policy was cancelled and, thus, it was not liable therefor.
6. By an Award dated 12.06.2002, the learned Motor Vehicles Accidents Claims Tribunal
allowed the said claim application directing payment of compensation for a sum of
Rs.1,58,000/- with interest @ 12% per annum holding that the Insurer was liable to pay
the said awarded amount despite cancellation of the contract of insurance. As noticed
hereinbefore the High Court of Karnataka on an appeal preferred by the first respondent
herein allowed the same relying on the judgment of the Karnataka High Court in M.F.A.
No. 6430 of 2001.
7. Mr. C.M. Angadi, the learned counsel appearing on behalf of the appellant in support
of this appeal inter alia submitted that the High Court committed a serious error in
passing the impugned judgment in so far as it failed to take into consideration that when
the insurance cover was issued, the liability of the Insurance Company subsists despite
dishonour of cheque evidencing payment of the insurance premium.
8

. Strong reliance in this behalf has been placed on Oriental Insurance Co. Ltd. v. Inderjit
Kaur and Ors. [(1998) 1 SCC 371] and National Insurance Co. Ltd. v. Seema Malhotra
and Ors. [(2001) 3 SCC 151]. 1998 AIR SCW 183
2001 AIR SCW 902

9. Before embarking on the said question we may notice the admitted facts. Second
respondent who was driving the vehicle was also the owner thereof. The insurance policy
was to remain valid for the period 17.10.1997 to 16.10.1998. Respondent No.3 issued a
cheque on 15.10.1997. The said cheque was presented for encashment before the
Syndicate Bank. The Bank by its letter dated 21.10.1997 issued a 'Return Memo'
disclosing dishonour of the cheque with the remarks "fund insufficient". First Respondent
thereupon cancelled the policy of insurance. The said information was communicated to
Respondent No.2. An intimation thereabout was also given to the R.T.O. concerned.
10. Before the Motor Vehicle Accidents Claims Tribunal, the insurer has also examined
witnesses, inter alia, to prove cancellation of the policy of insurance, postal
acknowledgment showing intimation thereabout which was served to the insured and a
copy of the letter dated 6.11.1997 issued to the R.T.O. and the memo issued by the Bank
as regards dishonour of the cheque etc.
11. Indisputably, the accident had occurred on 6.2.1998 that is much after communication
of cancellation of the policy.
12. Keeping in view the aforementioned backdrop of all events, we may notice the legal
issues addressed before us by the learned counsel.
13. Section 147 of the Act obligates the owner of the motor vehicle to get the vehicle
insured insofar as the claim of third party is concerned. The Act does not deal with
contract of insurance as such. Contract of insurance is governed by the Insurance Act,
1938 (for short "the 1938 Act").
14. Section 64-VB of the 1938 Act provides that no risk is to be assumed unless premium
is received in advance in the following terms :-
"Section 64VB - No risk to be assumed unless premium is received in advance-(1) No
insurer shall assume any risk in India in respect of any insurance business on which
premium is not ordinarily payable outside India unless and until the premium payable is
received by him or is guaranteed to be paid by such person in such manner and within
such time as may be prescribed or unless and until deposit of such amount as may be
prescribed, is made in advance in the prescribed manner.
(2) For the purposes of this section, in the case of risks for which premium can be
ascertained in advance, the risk may be assumed not earlier than the date on which the
premium has been paid in cash or by cheque to the insurer.
Explanation.-Where the premium is tendered by postal money order or cheque sent by
post, the risk may be assumed on the date on which the money order is booked or the
cheque is posted, as the case may be.
(3) Any refund of premium which may become due to an insured on account of the
cancellation of a policy or alteration in its terms and conditions or otherwise shall be paid
by the insurer directly to the insured by a crossed or order cheque or by postal money
order and a proper receipt shall be obtained by the insurer from the insured, and such
refund shall in no case be credited to the account of the agent.
@page-SC769
(4) Where an insurance agent collects a premium on a policy of insurance on behalf of an
insurer, he shall deposit with, or despatch by post to, the insurer, the premium so
collected in full without deduction of his commission within twenty-four hours of the
collection excluding bank and postal holidays.
15. The said provision, therefore, in no unmistakable term provides for issuance of a valid
policy only on receipt of payment of the premium.
16. The question came up for consideration before this Court in Inderjit Kaur (supra),
wherein it was opined that a policy of insurance which is issued in public interest would
prevail over the interest of the insurance company. In that case a bus met with an
accident. The policy of insurance was issued on 30.11.1989. A letter stating that the
cheque had been dishonoured was sent by the Insurance Company to the insurer on
23.1.1990. The premium was paid in cash on 2.5.1990. The accident took place
19.4.1990. Despite noticing Section 64-VB of the 1938 Act, but having regard to the
underlying public policy behind the statutory scheme in respect of insurance as evidenced
by Sections 147 and Section 149 of the Act and in particular having regard to the fact that
policy of insurance to cover the bus without receiving the premium had already been
issued, this Court held that the Insurance Company was liable to indemnify the insured.
17. We may, however, notice that in terms of sub-section (5) of Section 147 and sub-
section (1) of Section 149 of the Act, the Insurance Company became liable to satisfy
awards of compensation in respect thereof, notwithstanding its entitlement to avoid or
cancel the policy for the reason that the cheque issued for payment of premium thereon
had not been honoured.
18

. The said question, however, was left open in Inderjit Kaur (supra). 1998 AIR
SCW 183

19. The said decision proceeded on the basis that it was the Insurance Company which
was responsible for placing itself in the said predicament as it had issued a policy of
insurance upon receipt only of a cheque towards the premium in contravention of the
provisions of Section 64-VB of the 1938 Act. The public interest in a situation of that
nature and applying the principle of estoppel, this Court held, would prevail over the
interest of the Insurance Company.
20

. The ratio of the said decision was, however, noticed by this Court in New India
Assurance Co. Ltd. v. Rula and Ors. [(2000) 3 SCC 195]. It was held that ordinarily a
liability under the contract of insurance would arise only on payment of premium, if such
payment was made a condition precedent for taking effect of the insurance policy but
such a condition which is intended for the benefit of the insurer can be waived by it.
2000 AIR SCW 788, (Para 11)

It was opined :-
"......If, on the date of accident, there was a policy of insurance in respect of the vehicle in
question, the third party would have a claim against the Insurance Company and the
owner of the vehicle would have to be indemnified in respect of the claim of that party.
Subsequent cancellation of the insurance policy on the ground of non-payment of
premium would not affect the rights already accrued in favour of the third party."
The dicta laid down therein clarifies that if on the date of accident the policy subsists,
then only the third party would be entitled to avail the benefit thereof.
21

. Almost an identical question again came up for consideration before this Court in
National Insurance Co. Ltd. v. Seema Malhotra and Ors. [(2001) 3 SCC 151], a Division
Bench noticed both the aforementioned decisions and analysed the same in the light of
Section 64-VB of the 1938 Act. It was held : 2001 AIR SCW 902

"17. In a contract of insurance when the insured gives a cheque towards payment of
premium or part of the premium, such a contract consists of reciprocal promise. The
drawer of the cheque promises the insurer that the cheque, on presentation, would yield
the amount in cash. It cannot be forgotten that a cheque is a bill of exchange drawn on a
specified banker. A bill of exchange is an instrument in writing containing an
unconditional order directing a certain person to pay a certain sum of money to a certain
person. It involves a promise that such money would be paid.
18. Thus, when the insured fails to pay the premium promised, or when the cheque issued
by him towards the premium is returned dishonoured by the bank concerned the insurer
need not perform his part of the
@page-SC770
promise. The corollary is that the insured cannot claim performance from the insurer in
such a situation.
19. Under Section 25 of the Contract Act an agreement made without consideration is
void. Section 65 of the Contract Act says that when a contract becomes void any person
who has received any advantage under such contract is bound to restore it to the person
from whom he received it. So, even if the insurer has disbursed the amount covered by
the policy to the insured before the cheque was returned dishonoured, the insurer is
entitled to get the money back.
20. However, if the insured makes up the premium even after the cheque was
dishonoured but before the date of accident it would be a different case as payment of
consideration can be treated as paid in the order in which the nature of transaction
required it. As such an event did not happen in this case, the Insurance Company is
legally justified in refusing to pay the amount claimed by the respondents."
22. A contract is based on reciprocal promise. Reciprocal promises by the parties are
condition precedents for a valid contract. A contract furthermore must be for
consideration.
23. In today's world payment made by cheque is ordinarily accepted as valid tender.
Section 64VB of the 1938 Act also provides for such a scheme.
24

. Payment by cheque, however, is subject to its encashment. In Damadilal and Ors. v.


Parashram and Ors. [(1976) 4 SCC 855], this Court observed : AIR 1976 SC 2229,
(Para 13)

"On the ground of default, it is not disputed that the defendants tendered the amount in
arrears by cheque within the prescribed time. The question is whether this was a lawful
tender. It is well-established that a cheque sent in payment of a debt on the request of the
creditor, unless dishonoured, operates as valid discharge of the debt and, if the cheque
was sent by post and was met on presentation, the date of payment is the date when the
cheque was posted..."
25

. Recently again in New India Assurance Co. Ltd. v. Harshadbhai Amrutbhai Modhiya
and Anr. [(2006) 5 SCC 192], although in the context of the Workmen's Compensation
Act, 1923, Balasubramanyan, J. opined : 2006 AIR SCW 2352, (Para 24)
"It is not brought to our notice that there is any other law enacted which stands in the way
of an insurance company and the insured entering into a contract confining the obligation
of the insurance company to indemnify to a particular head or to a particular amount
when it relates to a claim for compensation to a third party arising under the Workmen's
Compensation Act. In this situation, the obligation of the insurance company clearly
stands limited and the relevant proviso providing for exclusion of liability for interest or
penalty has to be given effect to. Unlike the scheme of the Motor Vehicles Act the
Workmen's Compensation Act does not confer a right on the claimant for compensation
under that Act to claim the payment of compensation in its entirety from the insurer
himself".

It was further observed :- (Para 23)

"The law relating to contracts of insurance is part of the general law of contract. So said
Roskill, L.J. in Cehave v. Bremer. This view was approved by Lord Wilberforce in
Reardon Smith v. Hansen-Tangen (All ER p. 576 h) wherein he said :
"It is desirable that the same legal principles should apply to the law of contract as a
whole and that different legal principles should not apply to different branches of that
law."
A contract of insurance is to be construed in the first place from the terms used in it,
which terms are themselves to be understood in their primary, natural, ordinary and
popular sense. (See Colinvaux's Law of Insurance , 7th Edn., para 2-01.) A policy of
insurance has therefore to be construed like any other contract. On a construction of the
contract in question it is clear that the insurer had not undertaken the liability for interest
and penalty, but had undertaken to indemnify the employer only to reimburse the
compensation the employer was liable to pay among other things under the Workmen's
Compensation Act. Unless one is in a position to void the exclusion clause concerning
liability for interest and penalty imposed on the insured on account of his failure to
comply with the requirements of the Workmen's Compensation Act of 1923, the insurer
cannot be made liable to the insured for those amounts.""
26. We are not oblivious of the distinction between the statutory liability of the Insurance
Company vis-a-vis a third party in the context of Sections 147 and 149 of the Act and its
liabilities in other cases. But
@page-SC771
the same liabilities arising under a contract of insurance would have to be met if the
contract is valid. If the contract of insurance has been cancelled and all concerned have
been intimated thereabout, we are of the opinion, the insurance company would not be
liable to satisfy the claim.
27

. A beneficial legislation as is well known should not be construed in such a manner so as


to bring within its ambit a benefit which was not contemplated by the legislature to be
given to the party. In Regional Director, Employees' State Insurance Corporation, Trichur
v. Ramanuja Match Industries [AIR 1985 SC 278], this Court held - : (Para 10)
"We do not doubt that beneficial legislations should have liberal construction with a view
to implementing the legislative intent but where such beneficial legislation has a scheme
of its own there is no warrant for the Court to travel beyond the scheme and extend the
scope of the statute on the pretext of extending the statutory benefit to those who are not
covered by the scheme."
We, therefore, agree with the opinion of the High Court.
28. However, as the appellant hails from the lowest strata of society, we are of the
opinion that in a case of this nature, we should, in exercise of our extra-ordinary
jurisdiction under Article 142 of the Constitution of India, direct the Respondent No.1 to
pay the amount of claim to the appellants herein and recover the same from the owner of
the vehicle viz., Respondent No.2, particularly in view of the fact that no appeal was
preferred by him. We direct accordingly.
29. We, therefore, allow the appeal with the aforementioned directions. In the facts and
circumstances of the case, however, there shall be no order as to costs.
Order accordingly.
AIR 2008 SUPREME COURT 771 "Quality Inn Southern Star, M/s. v. Regional Director,
Employees' State Insurance Corpn."
(From : Karnataka)*
Coram : 2 Dr. A. PASAYAT AND P. SATHASIVAM, JJ.
Civil Appeal No. 1250 of 2001, D/- 3 -12 -2007.
M/s. Quality Inn Southern Star v. The Regional Director, Employees' State Insurance
Corporation.
Employees' State Insurance Act (34 of 1948), S.2(22) - EMPLOYEES STATE
INSURANCE - WAGES - WORDS AND PHRASES - Memorandum issued by E. S. I.
Corporation No. P11113/97- Ins IV, dt. 6-11-2000 - Wages -Service charges collected by
hotel management from customers and distributed amongst employees - Not "wages"
within meaning of S. 2(22).
(2002) 2 Lab LJ 1002 (Mad), Approved.
M. F. A. 1497 of 1992, D/- 29-7-1999 (Kar), Reversed. (Para 10)
Cases Referred : Chronological Paras
2002 (2) Lab LJ 1002 (Mad.) (Approved) 5, 10
AIR 1976 SC 2303 : 1976 Lab IC 1474 (Ref.) 4
Shyam Divan, Sr. Advocate, Akhil Pal Chhabra, Ms. Sudha Malla and Rajan Narain, for
Appellant; C. S. Rajan, Sr. Advocate, V. J. Francis and Anupam Mishra, for Respondent.
* M.F.A. No. 1497 of 1992, D/- 29-7-1999 (Kar)
Judgement
1. Dr. ARIJIT PASAYAT, J. :- Challenge in this appeal is to the judgment of the learned
Single Judge of the Karnataka High Court dismissing the appeal filed by the appellant.
Challenge was to the order of the Employees' State Insurance Court (in short 'ESI Court')
in ESI application No.123/89. The appeal was filed under Section 82(2) of the
Employees' State Insurance Act, 1948 (in short the 'Act'). Order passed by the ESI Court
was on the petition filed under Section 75 of the Act.
2. Background facts are as follows:
A show-cause notice was issued by the respondent on the report of the ESI Inspector on
9.1.1981 calling upon the appellant to contribute premium for the period November, 1986
to November, 1987 in respect of service charges collected by it. Not being satisfied with
the explanation offered, order was passed under Section 45-A of the Act determining
amount of contribution payable. The order was challenged by the appellant by an
application under Section 75 of the Act. This application was contested by the respondent
and the ESI Court on consideration of the evidence brought before it and it came to hold
that the order under Section 45-A of the Act suffered from no infirmity.
3. According to the appellant, the basic question was whether the service charge collected
by the hotel management from the customers and distributed amongst the
@page-SC772
employees amounted to "wages" within the meaning of Section 2(22) of the Act.
According to the appellant this did not constitute wages. The respondent contended that
the appellant runs a three-star hotel and the establishment is covered under the Act.
Undisputedly, 10% of the total bill amount is compulsorily collected as services charges
and is included in the bills. The service charges so collected are distributed amongst the
employees of the appellant quarterly. The collection of service charges is essentially what
is called as "tips" and paid at the option of customers. The ESI Court held that looking at
the nature of the service charges, these are not directly paid by the customers to the
employees but form part of the bills which the customers are obliged to pay without any
option and this amount so collected is paid or distributed to the employees equally once
in three months. According to the ESI Court the appellant had total control and power of
distribution of the amount and this is distinguishable from "tips". This was treated in any
event covered by the expression "additional reimbursement". The High Court in appeal
upheld the view.
4

. Learned counsel for the appellant submitted that the payments were distributed equally
amongst all the employees periodically, once in three months. It was submitted that by a
circular it was clarified that service charges were outside the scope of wage as defined.
Reference was made to a decision of this Court in The Rambagh Palace Hotel, Jaipur v.
The Rajasthan Hotel Workers' Union, Jaipur (1976 (4) SCC 817). The High Court
distinguished the same holding that it related to "tips" and there was no consideration of
the aspect whether it was covered by the expression "reimbursement". AIR 1974 SC
2303

5. It was pointed out that the judgment of the High Court was delivered on 29.7.1999.
Subsequently, the memorandum was issued by the Corporation bearing No.P-1/13/97-
Ins.IV dated 6.11.2002 clearly stating that service charges of the nature involved in the
present dispute do not form part of the wages. It is also pointed out that the Madras High
Court in a decision in Sathianathan N. and Sons Pvt. Ltd. and Ors. v. E.S.I. Corporation
and Anr. (2002-II LLJ 1002) on 6.2.2002 took a different view.
6. Learned counsel for the respondent on the other hand supported orders of the ESI
Court and the High Court.
7. Section 2(22) defines wages as :
"Wages means all remuneration paid or payable, in cash to an employee or implied, were
fulfilled and includes (any payment to an employee in respect of any period of authorized
leave, lockout, strike which is not illegal or layoff and) other additional remuneration, if
any (paid at intervals not exceeding two months), but does not include
(a) any contribution paid by the employer to any pension fund or provident fund, or under
this act:
(b) any travelling allowance or the value of any travelling concession;
(c) any sum paid to the person employed to defray special expenses entailed on him by
the nature employment; or
(d) any gratuity payable on discharge"
8. The circular referred to by the learned counsel for the appellant reads as follows :
"E. Service charges cannot be included in "wages" for the following reasons-
(a) The Memorandum issued by the ESIC corporation number P11113/97-Ins.IV dated
6.11.2000 clearly states in paragraph 13 that:
"Service Charges are collected by management of the hotel on behalf of their employees
in lieu of direct tips and the same is paid to their employees at a later date. Such amount
collected as 'service charges' will not constitute wages under S. 2(22) of the ESI Act. In
the case of ESIC v. M/s. Rambagh Palace Hotel, Jaipur, the High Court of Jaipur has held
that 'service charges' are not wages under Section 2(22) of the ESI Act. This verdict of the
High Court of Jaipur was accepted in the ESIC and hence no contribution is payable on
'service charges'. (Earlier instructions were issued vide letter No. P. 12/11/4/79 Ins. Desk I
dtd.18.9.79)"
9. The introduction to the memorandum dated 6.11.2000 states that it has been issued
because :
"it is necessary that the instructions issued by this office from time to time are not only
consolidated but certain more items are included not only to clear the doubts of the what
constitutes part of wage under Section 2(22). Some of the instructions were issued long
back rather - as back as in 1967 and certain instructions are not even available in some of
the regions and it is difficult to keep a track on the old instructions. Keeping in view the
above aspects and
@page-SC773
consolidated instructions including some more items are as under:"
(b) In the present case, the amounts received by the employees were not in the nature of
"wages", as they were not given to the employees under the terms of the contract of
employment, either express or implied. The appointment letters expressly state that
employees are not entitled to any other remuneration. Thus the distribution of service
charges is expressly excluded from the wages."
10. In view of the above-said office memorandum and the view taken by the Madras
High Court in Sathianathan's case (supra) the orders of the ESI Court and the High Court
cannot be maintained and are accordingly set aside.
11. The appeal is allowed without any order as to costs.
Appeal allowed.
AIR 2008 SUPREME COURT 773 "Yadvendra Arya v. Mukesh Kumar Gupta"
(From : 2006 (3) All Rent Cas 572)
Coram : 2 Dr. A. PASAYAT AND LOKESHWAR SINGH PANTA , JJ.
Civil Appeal No. 5483 of 2007 (arising out of SLP (C) No. 19545 of 2006), D/- 28 -11
-2007.
Yadvendra Arya and Anr. v. Mukesh Kumar Gupta.
U.P. Urban Buildings (Regulation of Letting, Rent and Eviction) Act (13 of 1972),
S.21(1)(a) - SHOPS AND ESTABLISHMENT - TENANCY - Bona fide requirement -
Non-residential premises - Father of landlord doing his independent business - Plea by
tenant that landlord can be accommodated in business of his father - Shop in possession
of his father however not so elaborate as to accommodate landlord also - Landlord also
not wanting to join business along with his father but to do independent business in suit
premises - Order directing tenant to vacate shop - Not liable to be set aside - However, as
tenant was doing business in suit premises, time was granted to him to vacate premises.
(Para 17)
Cases Referred : Chronological Paras
2003 AIR SCW 5316 : AIR 2003 SC 4548 (Ref.) 16
2002 AIR SCW 2374 : AIR 2002 SC 2256 (Foll.) 13
2002 AIR SCW 5160 : AIR 2003 SC 532 : 2003 AIR Jhar HCR 150 (Foll.)11
2001 AIR SCW 4841 : AIR 2002 SC 200 : 2001 All LJ 2915 (Foll.) 14
2000 AIR SCW 66 : AIR 2000 SC 534 (Foll.) 12
1999 AIR SCW 2259 : AIR 1999 SC 2226 14
1999 AIR SCW 2666 : AIR 1999 SC 2507 13
1998 AIR SCW 202 : AIR 1998 SC 602 13
1998 AIR SCW 3345 : AIR 1999 SC 22 14
(1996) 5 SCC 353 12
AIR 1980 SC 161 13
AIR 1979 SC 272 (Foll.) 13, 15
Yunus Malik, Abhishek Vikas, Prashant Chaudhary, for Appellants; N. D. B. Raju, C. M.
Angadi, Rameshwar Prasad Goyal, for Respondent.
Judgement
1. Dr. ARIJIT PASAYAT, J. :-Leave granted.
2. Challenge in this appeal is to the order passed by a learned Single Judge of the
Uttranchal High Court allowing the writ petition filed by the respondent. Said respondent
undisputedly is the landlord of the premises which were let out to the present appellants.
3. An application under Section 21(1)(a) of U.P. Urban Buildings (Regulation of Letting,
Rent and Eviction) Act, 1972, (Act No.13 of 1972) (hereinafter referred to as the 'Act')
was filed by the respondent against the appellants praying for the release of the Shop
situated at Mohalla Bazar Ganj (Park Road), Kashipur, District Udham Singh Nagar,
which was under tenancy on the ground that the respondent has passed High School
Certificate Examination and is unemployed and he has no independent business to earn
his livelihood and, therefore, he wants to do the business of Electrical Goods, T.V.,
V.C.R., Music System, Cooking Range etc. in the said Shop.
4. It was, further, stated by the landlord in his release application that his father Sri
Mithilesh Kumar Gupta is doing the independent business in the name of Mithilesh
Kumar and Brothers of which his father is the sole owner and there is no possibility of
employing any other person, as the shop in possession of his father is not so elaborate so
as to accommodate the respondent also. It was also stated that he also does not want to
join the business along with his father, as he wants to do the independent business.
@page-SC774
It has further been stated in the release application that he has already been married in
1994 and is separate from his father and as such, the shop in dispute is required for his
own use and occupation for settling himself in the independent business.
5. A written statement was filed by the present appellants in which it was stated that the
landlord can be accommodated in the business of the father.
6. An affidavit was filed by the respondent who has deposed that he wants to run the
independent business and he cannot settle himself along with his father. So far as the
availability of the other shops are concerned, it was specifically stated that all other shops
are rented accommodation and the tenants are occupying the same.
7. The prescribed authority, Kashipur District Udham Singh Nagar allowed the
application of the respondent directing the appellants to vacate the shop within a period
of 30 days.
8. Being aggrieved the appellants preferred an appeal which was allowed by the appellate
authority. The respondent filed Writ Petition under Article 227 of the Constitution of
India, 1950 (in short the 'Constitution'). The High Court as noted above allowed the writ
petition of the respondent and directed the appellants to vacate the premises.
9. In support of the appeal, it was contented by learned counsel for the appellants that the
parameters relating to bona fide needs and comparative hardship have not been
considered in the proper perspective.
10. Learned counsel for the respondent on the other hand supported the judgment of the
High Court stating that the High Court has kept in view the factual scenario and applied
the appropriate and applicable principles and, therefore, no interference is called for.
11

. So far as the basic need concept is concerned in Akhileshwar Kumar and Others v.
Mustaqim and Others [AIR 2003 SC 532] it was inter alia held as follows : 2002
AIR SCW 5160, (Paras 3 and 4)

"In our opinion, the approach adopted by the High Court cannot be countenanced and has
occasioned a failure of justice. Overwhelming evidence is available to show that the
plaintiff No. 1 is sitting idle, without any adequate commercial activity available to him
so as to gainfully employ him. The plaintiff No. 1 and his father both have deposed to this
fact. Simply because the plaintiff No. 1 is provisionally assisting his father in their family
business, it does not mean that he should never start his own independent business. What
the High Court has overlooked is the evidence to the effect, relied on by the trial Court
too, that the husband of plaintiff No. 4, i.e. son-in-law of Ram Chandra Sao, was assisting
the latter in his business and there was little left to be done by the three sons.
4. So is the case with the availability of alternative accommodation, as opined by the
High Court. There is a shop in respect of which a suit for eviction was filed to satisfy the
need of plaintiff No. 2. The suit was compromised and the shop was got vacated. The
shop is meant for the business of plaintiff No. 2. There is yet another shop constructed by
the father of the plaintiffs which is situated over a septic tank but the same is almost
inaccessible inasmuch as there is a deep ditch in front of the shop and that is why it is
lying vacant and unutilized. Once it has been proved by a landlord that the suit
accommodation is required bona fide by him for his own purpose and such satisfaction
withstands the test of objective assessment by the Court of facts then choosing of the
accommodation which would be reasonable to satisfy such requirement has to be left to
the subjective choice of the needy. The Court cannot thrust upon its own choice on the
needy. Of course, the choice has to be exercised reasonably and not whimsically. The
alternative accommodation which have prevailed with the High Court are either not
available to the plaintiff No. 1 or not suitable in all respects as the suit accommodation is.
The approach of the High Court that an accommodation got vacated to satisfy the need of
plaintiff No. 2, who too is an educated unemployed should be diverted or can be
considered as relevant alternative accommodation to satisfy the requirement of plaintiff
No. 1, another educated unemployed brother, cannot be countenanced. So also
considering a shop situated over a septic tank and inaccessible on account of a ditch in
front of the shop and hence lying vacant cannot be considered a suitable alternative to the
suit shop which is situated in a marketing complex, is easily accessible and has been
purchased by the plaintiffs to satisfy the felt need of one of them."
@page-SC775
12

. In Ragavendra Kumar v. Firm Prem Machinery and Co. [2000(1) SCC 679] it was held
as follows : 2000 AIR SCW 66, (Para 10)

"It is settled position of law that the landlord is best judge of his requirement for
residential or business purpose and he has got complete freedom in the matter. (See:
Prativa Devi (Smt.) v. T.V. Krishnan, [(1996)5 SCC 353]. In the case in hand the plaintiff-
landlord wanted eviction of the tenant from the suit premises for starting his business as it
was suitable and it cannot be faulted."
13

. In Joginder Pal v. Naval Kishore Behal [(2002(5) SCC 397) it was held as follows :
2002 AIR SCW 2374, (Paras 5, 7, 8, 30, 31)

"In Malpe Vishwanath Acharya and Ors. v. State of Maharashtra and Anr. (1998 2 SCC 1)
this Court emphasized the need of social legislations like the Rent Control Act striking a
balance between rival interests so as to be just to law. "The law ought not to be unjust to
one and give a disproportionate benefit or protection to another section of the society".
While the shortage of accommodation makes it necessary to protect the tenants to save
them from exploitation but at the same time the need to protect tenant is coupled with an
obligation to ensure that the tenants are not conferred with a benefit disproportionately
larger than the one needed. Socially progressive legislation must have a holistic
perception and not a shortsighted parochial approach. Power to legislate socially
progressive legislation is coupled with a responsibility to avoid arbitrariness and
unreasonability. A legislation impregnated with tendency to give undue preference to one
section, at the cost of constraints by placing shackles on the other section, not only entails
miscarriage of justice but may also in constitutional invalidity. 1998 AIR SCW 202

xxx xxx xxx

The need for reasonable interpretation of rent control legislations was emphasized by this
Court in Bega Begum v. Abdul Ahad Khan (AIR 1979 SC 273). Speaking in the context
of reasonable requirement of landlord as a ground for eviction, the Court guarded against
any artificial extension entailing stretching or straining of language so as to make it
impossible or extremely difficult for the landlord to get a decree for eviction. The Court
warned that such a course would defeat the very purpose of the Act which affords the
facility of eviction of the tenant to the landlord on certain specified grounds. In Kewal
Singh v. Lajwanti (1980) 1 SCC 290) this Court has observed, while the rent control
legislation has given a number of facilities to the tenants, it should not be construed so as
to destroy the limited relief which it seeks to give to the landlord also. For instance, one
of the grounds for eviction which is contained in almost all the Rent Control Acts in the
country is the question of landlord's bona fide personal necessity. The concept of bona
fide necessity should be meaningfully construed so as to make the relief granted to the
landlord real and practical. Recently in Shiv Sarup Gupta v. Dr Mahesh Chand Gupta
(1999) 6 SCC 222) the Court has held that the concept of bona fide need or genuine
requirement needs a practical approach instructed by the realities of life. An approach
either too liberal or too conservative or pedantic must be guarded against. AIR 1980 SC
161
1999 AIR SCW 2666

9. The rent control legislations are heavily loaded in favour of the tenants treating them as
weaker sections of the society requiring legislative protection against exploitation and
unscrupulous devices of greedy landlords. The legislative intent has to be respected by
the courts while interpreting the laws. But it is being uncharitable to legislatures if they
are attributed with an intention that they lean only in favour of the tenants and while
being fair to the tenants, go to the extent of being unfair to the landlords. The legislature
is fair to the tenants and to the landlords - both. The courts have to adopt a reasonable and
balanced approach while interpreting rent control legislations starting with an assumption
that an equal treatment has been meted out to both the sections of the society. In spite of
the overall balance tilting in favour of the tenants, while interpreting such of the
provisions as take care of the interest of the landlord the court should not hesitate in
leaning in favour of the landlords. Such provisions are engrafted in rent control
legislations to take care of those situations where the landlords too are weak and feeble
and feel humble.
xxx xxx xxx
In providing key to the meaning of any word or expression the context in which it is set
has significance. Color and content emanating from context may permit sense
@page-SC776
being preferred to mere meaning depending on what is sought to be achieved and what is
sought to be prevented by the legislative scheme surrounding the expression.
Requirement of landlord for his own use, is an expression capable for attributing an
intention to the legislature that what was intended to be fulfilled is such requirement as
would persuade the landlord to have the premises vacated by the tenant, to forego the
rental income, and to put the premises to such use as the landlord would deem to be his
own use and in the given facts and circumstances of a case the Court too would hold it to
be so in contradistinction with a mere ruse to evict the tenant. The legislature intending to
protect the tenant also intends to lift the protection when it is the requirement of landlord
to put the accommodation to such use as he intends, away from leasing it out.
xxx xxx xxx
32. If we do not meaningly construe the concept of requirement the provision may suffer
from the risk of being branded as unreasonable, arbitrary or as placing uncalled for and
unreasonable restrictions on the right of the owner to hold and use his property. We
cannot place a construction on the expression 'for his own use' in such a way as to deny
the landlord a right to evict his tenant when he needs the accommodation for his own son
to settle himself well in his life. We have to give colour and content to the expression and
provide the skin of a living thought to the skeleton of the words, which the Legislature
has not itself chosen to define. The Indian society, its customs and requirements and the
context where the provision is set in the legislation are the guides leading to acceptance
of the meaning which we have chosen to assign to the words 'for his own use' in Section
13(3)(a)(ii) of the Act.
33.(1) In the present case, the requirement of landlord of the suit premises for user as
office of his chartered accountant son is the requirement of landlord 'for his own use'
within the meaning of Section 13(3)(a)(ii)."
14

. Again in G. C. Kapoor v. Nand Kumar Bhasin (AIR 2002 SC 200) it was noted as
follows : 2001 AIR SCW 4841

"It is settled position of law that bona fide requirement means that requirement must be
honest and not tainted with any oblique motive and is not a mere desire or wish. In
Dattatraya Laxman Kamble v. Abdul Rasul Moulali Kotkunde and Anr. (1999 (4) SCC 1)
this Court while considering the bona fide need of the landlord was of the view that when
a landlord says that he needs the building for his own occupation, he has to prove it but
there is no warrant for 'presuming that his need is not bona fide'. It was also held that
while deciding this question. Court would look into the broad aspects and if the Courts
feels any doubt about bona fide requirement, it is for the landlord to clear such doubt.
1999 AIR SCW 2259

10. In Raghunath G. Panhale (D) By LRs. v. Chaganlal Sundarji and Co. (1999 (8) SCC
1) his Court inter alia held that it was not necessary for landlord to prove that he had
money to invest in the new business contemplated nor that he had experience of it. It was
a case for eviction on the ground of bona fide requirement of the landlord for non-
residential purpose, as he wanted to start a grocery business in the suit premises to
improve his livelihood." 1998 AIR SCW 3345

15

. In Mst. Begam Begum and Ors. v. Abdul Ahad Khan (d) by LRs and Ors. (1979 (1)
SCC 273) this Court had occasion to deal in detail with the comparative hardships aspect
as follows: AIR 1979 SC 272

"Moreover Section 11(h) of the Act uses the words 'reasonable requirement' which
undoubtedly postulate that there must be an element of need as opposed to a mere desire
or wish. The distinction between desire and need should doubtless be kept in mind but
not so as to make even the genuine need as nothing but a desire as the High Court has
done in this case. It seems to us that the connotation of the term 'need' or 'requirement'
should not be artificially extended nor its language so unduly stretched or strained as to
make it impossible or extremely difficult for one landlord to get a decree for eviction.
Such a course would defeat the very purpose of the Act which affords the facility of
eviction of the tenant to the landlord on certain specified grounds. This appears to us to
be the general scheme of all the Rent Control Acts, prevalent in other State in the country.
This Court has considered the import of the word requirement and pointed out that it
merely connotes that there should be an element of need.
@page-SC777
In this connection our attention was drawn to the evidence led by the defendants that the
main source of their income is the hotel business carried on by them in the premises and
if they are thrown out they are likely to get any alternative accommodation. The High
Court has accepted the case of the defendants on this point, but does not appear to have
considered the natural consequences, which flow from a comparative assessment of the
advantages and disadvantages of the landlord and the tenant if a decree for eviction
follows. It is no doubt true that the tenant will have to be ousted from the house if a
decree for eviction is passed, but such an event would happen whenever a decree for
eviction is passed and was fully in contemplation of the legislature when Section 11(1)(h)
of the Act was introduced in the Act. This by itself would not be valid ground for refusing
the plaintiffs for eviction.
Thus, on careful comparison and assessment of the relative advantage and disadvantages
of the landlord and the tenant it seems to us that the scale is titled in favour of the
plaintiff. The inconvenience, loss and trouble resulting from denial of a decree for
eviction in favour of the plaintiffs far outweight the eviction from that point of view."
16

. It is to be noted that learned counsel for the appellants submitted that the matter should
have been remanded to the authorities for further consideration. Such a practice has been
deprecated by this Court in a large number of cases. [See: R.V.E. Venkatachala Gounder
v. Arulmigu Viswesaraswami and V.P. Temple and Anr. (2003 (8) SCC 752)]. 2003
AIR SCW 5316

17. Considering the factual background in the light of the principles as stated above, the
inevitable conclusion is that this appeal is without merit. Considering the fact that the
appellants are carrying on the business in the premises, time is granted to them to vacate
the premises in question by the end of June, 2008 subject to filing the usual undertaking
with the prescribed authority within a period of four weeks from today.
18. Appeal is dismissed but without any order as to costs.
Appeal dismissed.
AIR 2008 SUPREME COURT 777 "U.P. State Agro Industrial Corporation Ltd. v. Kisan
Upbhokta Parishad"
(From : Allahabad)
Coram : 2 A. K. MATHUR AND MARKANDEY KATJU, JJ.
Civil Appeal No. 7285 of 2001, D/- 7 -12 -2007.
U. P. State Agro Industrial Corporation Ltd. v. Kisan Upbhokta Parishad and Ors.
(A) INTERPRETATION OF STATUTES - Interpretation of Statutes - Word used - To be
given popular meaning - Unless statute/order has given it specific meaning - Language is
a tool of communication between human beings, and hence that meaning should be given
which helps communication between people. (Paras 11, 15)
(B) INTERPRETATION OF STATUTES - WORDS AND PHRASES - Interpretation of
Statutes - Words "agricultural implement" - Would not cover Animal driven vehicle
(A.D.V.) - Implement means tools - A. D. V. is not implement.
In common parlance implements are usually regarded as tools used by human beings with
their hands and sometimes with their legs, or driven by animal power. Thus, a plough
which is driven by oxen or horses would be regarded as an agricultural implement.
Similarly, a hoe or a spade would be agricultural implements. However, a bullock cart
which is used for carrying the agricultural produce from the farm to the market or the
sugar factory cannot, be regarded as an agricultural implement, because in common
parlance it would not be regarded by people as an implement. A bullock cart is surely not
a tool, though the plough which it pulls is certainly a tool and therefore, an agricultural
implement.
(Para 16)
(C) INTERPRETATION OF STATUTES - PRINCIPLES - Interpretation of Statutes -
Mimansa Principles of Interpretation can be used on appropriate occasion - Rare use of
the principles in our law Courts, regrettable. (Para 20)
Cases Referred : Chronological Paras
(2006) 12 JT (SC) 379 (Ref.) 19
1991 AIR SCW 2119 : AIR 1991 SC 1992 (Ref.) 17
(1892) ILR 14 All 67 (FB) 19
Rajesh, for Appellant; Vijay K. Jain, for Respondents.
Judgement
1. MARKANDEY KATJU, J. - :- This appeal has been filed against the impugned
@page-SC778
judgment of the Allahabad High Court dated 22.2.2000 in Writ Petition No.23662 of
1999.
2. Heard learned counsel for the parties and perused the record.
3. The respondent in this appeal, which is a Union of cane growers and looks after the
interest of sugarcane farmers in Meerut District, was the petitioner in the writ petition
before the Allahabad High Court. It was alleged in the writ petition that cane growers of
the area require implements and other equipments for agriculture. For this purpose it
purchases Animal Driven Vehicles (hereinafter called "ADV carts") in order to transport
the sugarcane from the agriculture fields to the sugar factories or other places where it is
required to be sent. The State Government from time to time has provided a subsidy on
the purchase of ADV carts and other agricultural implements.
4. It appears that the State Government issued an order dated 20.11.1996 stating that all
kinds of agricultural implements driven by hand operation or animal power should be
purchased from the U.P. State Agro Industrial Limited. The short question in the writ
petition before the High Court was whether the ADV carts are agricultural implements. If,
they are then in order to get subsidy, purchases had to be made only from the Corporation
and not from other parties.
5. The Cane Commissioner, U.P. issued a letter dated 5.3.1999, copy of which is
Annexure P-2 to this appeal, stating that in pursuance of the aforesaid Government order
dated 20.11.1996 of the U.P. Government, ADV carts can only be purchased from the
U.P. State Agro Industrial Limited. This order dated 5.3.1999 of the Cane Commissioner
was challenged in the writ petition on the ground that it was in conflict with the
Government order dated 20.11.1996.
6. The short question in this appeal is whether ADV carts are also agricultural
implements.
7. The Concise Oxford English Dictionary (Tenth Edn. Revised) defines implement "as a
tool, utensil or other piece of equipment used for a particular purpose". The same
dictionary defines 'tool' "as" a device or implement, typically hand-held, used to carry out
a particular function.
8. In Webster Comprehensive Dictionary (International Edn.) the word 'implement' has
been defined as "a thing used in work, especially in manual work; a utensil; tool". In the
same dictionary the word 'tool' has been defined as a "simple mechanism or implement,
as a hammer, saw, spade, or chisel, used chiefly in the direct manual working, moving,
shaping, or transforming of material".
9. In Advanced Law Lexicon by P. Ramanatha Aiyar (3rd edn. 2005) the word 'tool' has
been defined as "things designed to help the hand in work, especially in industrial
operations".
10. One word can have several meanings, and several words can have the same meaning
(synonyms). Thus, for example, the word 'ball' can mean the spherical object used in a
game, or it can also mean a dance; it can also mean having a nice time, etc. Similarly,
several words can have the same meaning e.g. the Sanskrit words 'pankaj', 'jalaj', 'kamal',
'padma', 'saroj', 'sarsij', etc. which all mean 'Lotus'.
11. No doubt the word 'implement' can have several dictionary meanings. However, in
interpretation it is well settled that ordinarily the meaning of the word or expression in
common parlance or in common use should be accepted, unless the statute or order in
which it is used has defined it with a specific meaning. There is no definition of the word
'implements' in the G.O. of the State Government dated 20.11.1996.
12. In the Mimansa Rules of Interpretation, which is our indigenous system of
interpretation, one of the principles is - :
(Vernacular Matter omitted-Ed.)
13. The above principle means the popular meaning overpowers the etymological
meaning".
14. For example, the word 'pankaja' literally means whatever grows in mud. The word
'panka' means 'mud', and the suffix 'ja' means 'which is born in'. Hence the etymological
meaning of the word 'pankaja' is 'that which is born in mud'. Thus literally there can be
several things which could mean 'pankaja' e.g. worms or insects born in mud, all kinds of
vegetation which are born and found in mud, etc. However, by popular usage the word
'pankaja' has acquired a particular meaning in common parlance i.e. lotus. This shows
that we should prefer the popular meaning or the meaning in common usage to the literal
meaning of a word.
15. The reason behind this principle is
@page-SC779
that language is a tool of communication between human beings, and hence that meaning
should be given to a word which helps communication between people. If the speaker of
a word uses it in one sense but the hearer understands it in another sense, there will be a
communication gap. Hence that meaning should be attributed to a word which everyone
would understand as it has acquired a special meaning in common parlance.
16. Keeping the above principle in mind we may now consider whether an Animal
Driven Vehicle can be said to be an agricultural implement. In our opinion it cannot, for
the obvious reasons that in common parlance implements are usually regarded as tools
used by human beings with their hands (and sometimes with their legs), or driven by
animal power. Thus, a plough which is driven by oxen or horses would be regarded as an
agricultural implement. Similarly, a hoe or a spade would be agricultural implements.
However, a bullock cart which is used for carrying the agricultural produce from the farm
to the market or the sugar factory cannot, in our opinion, be regarded as an agricultural
implement, because in common parlance it would not be regarded by people as an
implement. A bullock cart is surely not a tool, though the plough which it pulls (for
furrowing the land) is certainly a tool and therefore, an agricultural implement.
17

. Learned counsel for the respondent has relied on the decision of this Court in M/s. D.H.
Brothers Pvt. Ltd. vs. Commissioner of Sales Tax, U.P., AIR 1991 SC 1992, in which it
was held that sugarcane crushers are not agricultural implements. In that decision this
Court held that a sugarcane crusher is not used in the agricultural operation, rather it is
only when the agricultural operations have ended and the cane harvested and transported
to the cane crusher that the activity of the cane crusher begins. Learned counsel submitted
that in the present case also the ADV carts which are used for transporting the sugarcane
from the agricultural field to the sugar factory are not part of the agricultural operations,
as these ADV carts begin their activity of transportation only after the agricultural
operations are over. 1991 AIR SCW 2119

18. It is not necessary for us to deal with this submission because we have earlier held
that an ADV cart is not an agricultural implement since it is not a tool. In view of the
above we find no merit in this appeal and it is accordingly dismissed. No costs.
19. Before parting with this case, we would like to say that it is deeply regrettable that in
our Courts of law, lawyers quote Maxwell and Craies but nobody refers to the Mimansa
Principles of Interpretation. Today our so-called educated people are largely ignorant
about the great intellectual achievements of our ancestors and the intellectual treasury
they have bequeathed us. The Mimansa Principles of Interpretation is part of that
intellectual treasury, but it is distressing to note that apart from a reference to these
principles in the judgment of Sir John Edge, the then Chief Justice of Allahabad High
Court, in Beni Prasad vs. Hardai Devi, (1892) ILR 14 All 67 (FB), there has been almost
no utilization of these principles even in our own country (except by one of us, M. Katju,
J. in some of his judgments delivered at Allahabad High Court and in this Court vide M/s.
Ispat Industries Ltd. vs. Commissioner of Customs, Mumbai, JT 2006(12) SC 379.
20. It may be mentioned that the Mimansa Rules of Interpretation were our traditional
principles of interpretation laid down by Jaimini whose Sutras were explained by Shabar,
Kumarila Bhatta, Prabhakar, etc. These Mimansa Principles were regularly used by our
great jurists like Vijnaneshwar (author of Mitakshara), Jimutvahana (author of
Dayabhaga), Nanda Pandit (author of 'Dattak Mimansa') etc. whenever they found any
conflict between the various Smritis or any ambiguity or incongruity therein. There is no
reason why we cannot use these principles on appropriate occasions. However, it is a
matter of deep regret that these principles have rarely been used in our law Courts. It is
nowhere mentioned in our Constitution or any other law that only Maxwells Principles of
Interpretation can be used by the Court. We can use any system of interpretation which
helps us solve a difficulty. In certain situations Maxwells principles would be more
appropriate, while in other situations the Mimansa principles may be more suitable.
21. Since we have used a Mimansa principle in this judgment we thought it necessary to
briefly mention about the Mimansa principles of interpretation (the original works on
Mimansa are all in Sanskrit, but there is a very elucidating book in English
@page-SC780
on the subject by K.L. Sarkar called 'The Mimansa Rules of Interpretation' published in
the Tagore Law Lecture Series).
Appeal dismissed.
AIR 2008 SUPREME COURT 780 "Vineet Kumar Chauhan v. State of U. P."
(From : 2005 All LJ 3784)
Coram : 2 PRAKASH PRABHAKAR NAOLEKAR AND D. K. JAIN, JJ.
Criminal Appeal No. 35 of 2006, D/- 14 -12 -2007.
Vineet Kumar Chauhan v. State of U.P.
(A) Evidence Act (1 of 1872), S.45 - Penal Code (45 of 1860), S.300 - EVIDENCE -
MURDER - FIRE ARMS - Expert opinion - Necessity - Offence by use of fire-arm -
Ballistic Expert evidence - Absence of - Not always fatal to prosecution case.
It cannot be laid down as a general proposition that in every case where a firearm is
allegedly used by an accused person, the prosecution must lead the evidence of a Ballistic
Expert to prove the charge, irrespective of the quality of the direct evidence available on
record. It needs little emphasis that where direct evidence is of such an unimpeachable
character, and the nature of injuries, disclosed by post-mortem notes is consistent with the
direct evidence, the examination of Ballistic Expert may not be regarded as essential.
However, where direct evidence is not available or that there is some doubt as to whether
the injuries could or could not have been caused by a particular weapon, examination of
an expert would be desirable to cure an apparent inconsistency or for the purpose of
corroboration of oral evidence. (Para 10)
In the instant case when the ocular evidence is totally consistent with the opinion of
doctors who have given injury report and the post-mortem report and it was clear that the
bullet fired from revolver by accused had damaged the spinal cord of the victim leading
to paralysis of both lower limbs of victim and consequent death, the absence of Ballistic
Expert's evidence is not fatal to the case of prosecution, notwithstanding the fact that the
Forensic Science Laboratory, in its report had not expressed a difinite opinion about the
bullet recovered from the place of occurrence. (Para 12)
(B) Penal Code (45 of 1860), S.300, S.299 - MURDER - CULPABLE HOMICIDE -
Murder or culpable homicide not murder - Accused and victim neighbourers - Incident
preceded by altercation between accused and family member of victim - Accused
returning to his house in a huff - Taking revolver of his father - Firing indiscriminately
towards victim's house - Victim trying to close door getting hit by bullet which proved
fatal - Accused at best can be said to have knowledge that use of revolver was likely to
cause death - Liable to be convicted under S. - 299, Cl. (3) and not under S. 300. (Para
16)
Cases Referred : Chronological Paras
2001 AIR SCW 2441 : AIR 2001 SC 2408 : 2001 Cri LJ 3292 (Disting.) (Pt. A) 8
AIR 1977 SC 45 : 1977 Cri LJ 1 (Rel. on) 15
AIR 1966 SC 1874 : 1966 Cri LJ 1509 (Ref.) 15
AIR 1963 SC 340 : 1963 (1) Cri LJ 323 (Rel. on) (Pt. A) 10
AIR 1958 SC 465 : 1958 Cri LJ 818 ((Ref.) 15
AIR 1953 SC 415 : 1953 Cri LJ 1761 (Disting.) (Pt. A) 8, 11
Sushil Kumar, Sr. Advocate, Vinay Arora, Aditya Kumar, Sudarshan Singh Rawat, Anmol
Thakral and Sanjay Jani, for Appellant; Ratnakar Das, Sr. Advocate, T. N. Singh, Rajeev
Dubey and Kamlendra Mishra, for Respondent.
Judgement
1

. D. K. JAIN, J. :- This appeal under Section 2(a) of the Supreme Court (Enlargement of
Criminal Appellate Jurisdiction) Act, 1970 has been preferred against the judgment of the
High Court of Judicature at Allahabad in Government Appeal No. 415 of 2000. By the
impugned judgment, the appeal filed by the State of Uttar Pradesh has been allowed and
the appellant-Vineet Kumar Chauhan has been convicted under Section 302 of the Indian
Penal Code, ('IPC' for short) for causing the murder of Smt. Premwati. He has been
sentenced to suffer imprisonment for life. Reported in 2005 All LJ 3784

2. The genesis of the prosecution case, in brief, was that on 13.10.1993 at about 11.50
a.m., one Sri Krishna Sharma (P.W.1), husband of the deceased, lodged an F.I.R. with the
Police Station Majhola, District Moradabad to the effect that on that day, at
@page-SC781
about 9.45 a.m., when he along with his wife and children was watching television, the
appellant who was living opposite their house and was a cable operator along with his
servant Dharamveer, came to their house and tried to persuade his son-Ravindra Sharma
(P.W.2) to take a cable connection from them. Not being interested in the cable
connection, they declined the request of the appellant whereupon an altercation took
place between the appellant and P.W.2. The complainant and his wife intervened and
asked the appellant to leave their house. The appellant went to his house, brought out the
licensed revolver of his father and opened indiscriminate firing towards complainant's
house from the door of his house. Some bullets hit the door of the house of Sri Krishna
Sharma and while his wife, the victim, was closing the door, one of the bullets hit her in
the jaw. Sri Krishna Sharma brought his injured wife to the hospital for treatment and
thereafter lodged the F.I.R.
3. The victim was examined by Dr. Jagmal Singh, P.W.4. The following injuries were
found on her person :
1. Lacerated wound 1.5 cm. x .5 cm x not probed on face, left side over left mandible, 3
cm. below and outer to left angle of mouth. Advised X-ray of left side face and left side
neck.
2. Lacerated wound .5 cm x .5 cm x skin on left arm outer part, 4 cm. above left elbow.
4. Both the injuries were found to be fresh. Injury No.1 was alleged to have been caused
by firearm but final opinion was reserved to be given after the X-ray. Injury No.2 was
caused by a blunt object. On X-ray being taken, a radio opaque shadow elongated was
found in thoracic spine in dorsal region over T 5-6.
5. The victim remained under treatment and supervision of Dr. D.S. Ahlawat (P.W.5). On
15.10.1993, she was taken to Delhi for treatment. However, on 21.10.1993, she was again
admitted in Moradabad Hospital, where she developed bedsores. Smt. Premwati
ultimately died on 25.3.1994. As per the autopsy conducted by Dr. S.P. Singh (P.W.7) on
25.3.1994, the ante-mortem injuries were mainly deep bedsores on various parts of the
body and one old healed scar, size 1.2 cm x .5 cut, on the left face at the chin 2.5 cm.
away from medium plank thoracic spine. On internal examination, the doctor recovered a
metallic bullet from her spinal cord, which had caused extensive damage in thoracic spine
and paralysis in half of the body. The cause of death was opined to be septicemia and
toxemia due to bedsores. After investigations, charge-sheet under Sections 452 and 307
was filed against the appellant and his father. However, charges were framed against
them under Sections 302 and 307, IPC.
6. In support of the case, the prosecution examined seven witnesses, including Sri
Krishna Sharma (P.W.1) and Ravindra Sharma (P.W.2), who claimed to be the eye-
witnesses. As per testimony of P.W.5, the deceased had suffered paralysis in both her legs
due to bullet injury sustained in the spinal cord. The Trial Court found the evidence to be
insufficient to warrant conviction of both the accused. Doubting the presence of P.W.1-Sri
Krishna Sharma and P.W.2-Ravindra Sharma at the spot and inter alia, observing that
from the report of the Ballistic Expert it could not be established that the lead (from part
of the bullet) recovered from the spot pertained to a shot fired from revolver recovered
from the house of the accused-Vineet Kumar and that deceased had actually died of
septicemia and toxemia owing to bedsores, as she was not properly advised and attended
to while she was admitted in hospital and death was attributable to the negligence and
bedsore, the Trial Court directed their acquittal.
7. On appeal by the State, the High Court affirmed the acquittal of Dharamveer. Insofar
as the case of the appellant was concerned, the High Court found the ocular evidence qua
him to be perfectly in harmony with the medical evidence. Concluding that the appellant
did commit the offence of murder, as noted above, the High Court convicted him under
Section 302, I.P.C. It is this conviction and sentence which has been challenged in this
appeal.
8

. Mr. Sushil Kumar, learned senior counsel appearing on behalf of the appellant assailed
the conviction of the appellant mainly on the ground that apart from the fact that the
Ballistic Report casts a serious doubt that the distorted bullet allegedly recovered from
the spot came out of the seized revolver, it was also obligatory on the part of the
prosecution to send the bullet, allegedly recovered from the body of the deceased, for
being examined by the Ballistic Expert, so as AIR 1953 SC 415
2001 AIR SCW 2441

@page-SC782
to connect the recovered licensed revolver of the appellant's father with the crime. It was
submitted that since it was a positive case of the prosecution that the bullet which had hit
the deceased was fired from the seized revolver, omission to send the bullet for ballistic
examination is a serious infirmity in the prosecution case, which assumes still greater
significance because of Ballistic Report, which does not even establish that the remnants
of the bullet (lead), recovered from the place of incident, was of the bullet fired from the
revolver allegedly used by the appellant. In support, strong reliance is placed on the
decision of this Court in Mohinder Singh v. The State1, wherein it was observed that in a
case where death is due to injuries or wounds caused by a lethal weapon, it has always
been considered to be the duty of the prosecution to prove by expert evidence that it was
likely or at least possible for the injuries to have been caused with the weapon with which
and in the manner in which they are alleged to have been caused. Reference is also made
to another decision of this Court in State of M.P. v. Surpa2, expressing a similar view.
Learned counsel has also contended that all through the case of the prosecution was that
the accused was firing towards the house of the deceased without aiming at any person
and the bullet hit the deceased accidentally when she was closing the door of the house. It
is urged that in case the appellant had any intention to commit the murder of the deceased
or any member of her family, he would have gone to their house and shot them. It is
argued that even if the occurrence is admitted to have taken place in the manner alleged,
the appellant cannot be held guilty for the commission of offence punishable under
Section 302, IPC. It is asserted that the occurrence having taken place without pre-
meditation, in the heat of the passion upon a sudden quarrel, the appellant is entitled to
the benefit of Exception 4 of Section 300, IPC.
1 (1950) 1 SCR 821)
2 (2002) 9 SCC 447)
9. Learned counsel for the State, on the other hand, supported the view taken by the High
Court.
10

. It cannot be laid down as a general proposition that in every case where a firearm is
allegedly used by an accused person, the prosecution must lead the evidence of a Ballistic
Expert to prove the charge, irrespective of the quality of the direct evidence available on
record. It needs little emphasis that where direct evidence is of such an unimpeachable
character, and the nature of injuries, disclosed by post-mortem notes is consistent with the
direct evidence, the examination of Ballistic Expert may not be regarded as essential.
However, where direct evidence is not available or that there is some doubt as to whether
the injuries could or could not have been caused by a particular weapon, examination of
an expert would be desirable to cure an apparent inconsistency or for the purpose of
corroboration of oral evidence. (See: Gurcharan Singh v. State of Punjab3) AIR
1963 SC 340

3 (1963) 3 SCR 585)


11

. In Mohinder Singh's case (supra) on which strong reliance is placed on behalf of the
appellant, this Court has held that, where the prosecution case was that the accused shot
the deceased with a gun, but it appeared likely that the injuries on the deceased were
inflicted by a rifle and there was no evidence of a duly qualified expert to prove that the
injuries were caused by a gun, and the nature of the injuries was also such that the shots
must have been fired by more than one person and not by one person only, and the
prosecution had no evidence to show that another person also shot, and the oral evidence
was of witnesses who were not disinterested, the failure to examine an expert would be a
serious infirmity in the prosecution case. It is plain that these observations were made in a
case where the prosecution evidence was suffering from serious infirmities. Thus, in
determining the effect of these observations, the facts in respect of which these
observations came to be made cannot be lost sight of. The said case, therefore, cannot be
held to lay down an inflexible rule that in every case where an accused person is charged
with murder caused by a lethal weapon, the prosecution case can succeed in proving the
charge only if Ballistic Expert is examined. In what cases, the examination of a Ballistic
Expert is essential for the proof of the prosecution case, must depend upon the facts and
circumstances of each case. AIR 1953 SC 415

12. In the instant case, having regard to the ocular evidence adduced by the prosecution,
there is no reason to discard the
@page-SC783
prosecution theory that the injury as a result whereof Smt. Premwati suffered complete
paralysis of both the lower limbs etc. was caused by a bullet fired from a revolver. The
nature of the injury as proved by Dr. P. S. Ahlawat (P.W.5), under whose treatment the
deceased remained at Moradabad and Dr. S. P. Singh (P.W.7), who had conducted the
post-mortem examination is wholly consistent with the prosecution version. It is clear
that the bullet recovered by P.W.7 at the time of post-mortem of the victim had traversed
to thoracic spine through the neck from the face near the angle of the jaw, hitting the fifth
thoracic vertebra, badly damaging the underlying spinal cord. We are, therefore, of the
view that on the facts of the present case, the absence of Ballistic Expert's evidence is not
fatal to the case of the prosecution, notwithstanding the fact that the Forensic Science
Laboratory, in its report dated 18.2.1991, had not expressed a definite opinion about the
bullet recovered from the place of occurrence.

13. Insofar as the testimonies of P.W.1 and P.W.2, the two star witnesses of the
prosecution, are concerned, from the impugned judgment, it is manifest that the High
Court, on analysis of their statements, has found these to be trustworthy. The High Court
has observed that testimony of these two natural witnesses is of sterling character with no
holes whatsoever. Based on this evidence, the High Court has found that it was the
appellant who had opened fire from the revolver from his door, one of which had hit the
victim, who had come to close the main door of her house. Nothing has been shown to us
so as to warrant interference with the said finding recorded by the High Court. Therefore,
in the context of this unimpeachable evidence, it stands proved that the appellant had
gone to the house of the deceased; some unsavoury incident took place there; he returned
to his house in a huff; took out the revolver of his father and fired shots towards the house
of the deceased; one of the bullets hit the deceased and the same proved to be fatal.
Having bestowed our anxious consideration to the evidence on record, in particular the
testimony of P.W.1 and P.W. 2, we are of the opinion that the High Court was correct in
coming to the conclusion that the appellant was responsible for causing the fatal injury to
the deceased. We are also in agreement with the High Court that though as per the post-
mortem report the deceased died of septicemia and toxemia because of bedsores, the
basic cause of her death was the bullet injury caused to her by the appellant.
14. However, the next question for consideration is whether the offence established by
the prosecution against the appellant is "murder"- as held by the High Court or "culpable
homicide not amounting to murder"- as contended on behalf of the appellant?
15

. The academic distinction between "murder" and "culpable homicide not amounting to
murder" has been vividly brought out by this Court in State of Andhra Pradesh v.
Rayavarapu Punnayya and another4. It has been observed that the safest way of approach
to the interpretation and application of Sections 299 and 300, IPC is to keep in focus the
key words used in various clauses of the said Sections. Minutely comparing each of the
clauses of Sections 299 and 300, IPC and drawing support from the decisions of this
Court in Virsa Singh v. State of Punjab5and Rajwant v. State of Kerala6, speaking for the
Court, R. S. Sarkaria, J. neatly brought out the points of distinction between the two
offences, which have been time and again reiterated. Having done so, the court said that
whenever a Court is confronted with the question whether the offence is "murder" or
"culpable homicide not amounting to murder", on the facts of a case, it will be convenient
for it to approach the problem in three stages. The question to be considered at the first
stage would be, whether the accused has done an act by doing which he has caused the
death of another. Proof of such causal connection between the act of the accused and the
death, leads to the second stage for considering whether that act of the accused amounts
to "culpable homicide" as defined in Section 299. If the answer to this question is prima
facie found in the affirmative, the stage for considering the operation of Section 300,
Penal Code, is reached. This is the stage at which the court should determine whether the
facts proved by the prosecution bring the case within the ambit of any of the four clauses
of the definition of "murder" contained in Section 300. If the answer to this question is in
the AIR 1977 SC 45
AIR 1958 SC 465
AIR 1966 SC 1874

@page-SC784
negative the offence would be "culpable homicide not amounting to murder", punishable
under the first or the second part of Section 304, depending, respectively, on whether the
second or the third clause of Section 299 is applicable. If this question is found in the
positive, but the case comes within any of the Exceptions enumerated in Section 300, the
offence would still be "culpable homicide not amounting to murder", punishable under
the first part of Section 304, Penal Code. It was, however, clarified that these were only
broad guidelines to facilitate the task of the Court and not cast iron imperative.
4 (1976) 4 SCC 382
5 1958 SCR 1495
6 1966 Supp SCR 230
16. Reverting to the facts in hand, as noted above, it stands proved that there being a
direct causal connection between the hitting of the bullet, fired by the appellant, to the
deceased and her death, the death of the deceased was caused by the appellant. However,
having regard to the circumstances, briefly enumerated above, particularly the manner in
which the appellant fired the shots, in our view, the appellant could not be attributed the
mens rea requisite for bringing the case under clause (3) of Section 300, IPC.
Concededly, there was no enmity between the parties and there is no allegation of the
prosecution that before the occurrence, the appellant had pre-meditated the crime of
murder. We are inclined to think that having faced some sort of hostile attitude from the
family of the deceased over the cable connection, a sudden quarrel took place between
the appellant and the son of the deceased, on account of heat of passion, the appellant
went home; took out his father's revolver and started firing indiscriminately, and
unfortunately one of the bullets hit the deceased on her chin. At the most, it can be said
that he had the knowledge that the use of revolver was likely to cause death and, as such,
the present case would fall within the third clause of Section 299, IPC. Thus, in our
opinion, the offence committed by the appellant was only "culpable homicide not
amounting to murder". Under these circumstances, we are inclined to bring down the
offence from first degree "murder" to "culpable homicide not amounting to murder",
punishable under the Second Part of Section 304, IPC.
17. Consequently, we partly allow the appeal; set aside the conviction of the appellant
under Section 302, IPC and instead convict him under Section 304, Part II, IPC. The
sentence of rigorous imprisonment for five years would meet the ends of justice.
Order accordingly.
AIR 2008 SUPREME COURT 784 "Chinnathaman v. State, Rep. by Inspector of Police"
(From : Madras)
Coram : 2 C. K. THAKKER AND J. M. PANCHAL, JJ.
Criminal Appeal No. 79 of 2006, D/- 13 -12 -2007.
Chinnathaman v. State, Rep. by Inspector of Police.
Penal Code (45 of 1860), S.300, Exception I and S.304, Part II - MURDER -
CULPABLE HOMICIDE - Murder - Sudden provocation - Homicide due to blow on
neck by sickle - Injuries not found sufficient in ordinary course of nature to cause death -
Deceased found guilty of offering grave and sudden provation to accused who delivered
blow after altercation by deceased - No premeditation or pre-plan on part of accused to
cause death - Conviction u/S. 300, held, liable to be set aside - Accused liable to be
convicted u/S. 304, Part II in view of Exception to S. 300.
Criminal Appeal No. 648 of 1997, D/- 28-3-2005, (Mad), Partly Reversed. (Para 10)

K. V. Viswanathan, Atul Kumar Sinha, Anup Kumar and Devendra Singh, for Appellants;
V. Kanakaraj, Sr. Advocate, S. Joseph Aristotle, S. Prabhu Ramasubramanium and V. G.
Pragasam, for Respondent.
Judgement
1. J. M. PANCHAL, J. :- This appeal is directed against the judgment dated 28th March,
2005 rendered by the High Court of Judicature at Madras in Criminal Appeal No. 648 of
1997, by which, judgment dated February 14, 1997 passed by learned First Additional
Sessions Judge, Coimbatore in Sessions Case No. 63 of 1996, convicting the appellant
under Section 302 of the Indian Penal Code (for short "the Code") and sentencing him to
R.I. for life, is confirmed.
2. The facts emerging from the record of the case are as under :
The appellant is a resident of village Thirumalainaickenpalayam. The name of his
younger brother is Dorai @ Nataraj, who was also residing in the same village at the
relevant time. The elder daughter of Nataraj was going to Pioneer Mill School for studies.
She had an affair with one Kirshnamurthy, who was a teacher in the
@page-SC785
school. Therefore, Mr. Dorai vacated his house situated in village and shifted his family
to a house located in the garden. He also stopped Punitha from attending the school.
Punitha, however, eloped with her teacher and, therefore, a missing report was lodged by
Dorai @ Nataraj with Periyanaickenpalayam Police Station. The appellant came to know
that his uncle's grandson Maruthachalam and his sister's son Chandran had facilitated
elopement of Punitha with her teacher and, therefore, scolded both of them. The incident
in question took place on April 27, 1994. On the date of incident at about 10.00 a.m. the
appellant was repairing the leakage in the pipe fitted near the well situated in his field.
Maruthachalam with his brother Senthil Kumar approached the appellant and asked him
to give bitterguard. The appellant refused to give bitterguard saying that they had
defamed his family by helping Punitha to elope with her teacher. Thereupon a verbal
altercation took place. The appellant picked up aruval (sickle) lying on the ground and
caused injuries on the neck of Maruthachalam. Thereupon Senthil Kumar raised shouts as
a result of which Thiru Ramasamy, the father of Maruthachalam, who was working in his
field rushed at the place of incident. The appellant after causing injuries to
Maruthachalam left his field and went to Village Administrative Officer with the sickle.
The Village Administrative Officer recorded the statement of the appellant and took him
to Periyanaickenpalayam Police Station with sickle. At the said police station, Thiru
Jayabalan was discharging duties as Sub-Inspector. On the basis of the statement made by
the appellant before the Village Administrative Officer, the Sub-Inspector registered an
offence punishable under Section 302 of the Code against the appellant and commenced
investigation. The police officer went to the place of incident and seized incriminating
articles under a panchnama. The dead body of the deceased was sent to the hospital for
autopsy. The police officer also recorded the statements of those persons who were found
to be conversant with the facts of the case. The incriminating articles seized were sent to
forensic science laboratory for analysis. On completion of investigation the appellant was
charge-sheeted for commission of offence punishable under Section 302, IPC in the court
of learned Judicial Magistrate Court No.VI, Coimbatore. As the offence punishable under
Section 302, IPC is exclusively triable by Court of Session, the case was committed to
the court of learned Additional Sessions Judge, Coimbatore for trial.
3. The learned Sessions Judge framed charge against the appellant for commission of
offence punishable under Section 302, IPC. It was read over and explained to him. He
pleaded not guilty to the same and claimed to be tried. The prosecution, therefore,
examined 12 witnesses to prove its case against the appellant and also produced
necessary documentary evidence.
4. After recording of the evidence of prosecution witnesses was over, the learned Judge
explained to the appellant the circumstances appearing against him in the evidence of the
prosecution and recorded his statement under Section 313 of the Code of Criminal
Procedure. In his further statement, the case of the appellant was that of total denial. He
did not examine any witness in his defence.
5. On appreciation of evidence adduced by the prosecution the learned Judge held that it
was proved that the deceased had died a homicidal death. After placing reliance on the
testimony of eye-witnesses the learned Judge concluded that the appellant was the author
of injuries sustained by the deceased. The learned Judge thereafter considered the nature
of the offence committed by the appellant. After taking into consideration the facts of the
case and the provisions of Section 300, IPC, the learned Judge concluded that the
appellant had committed an offence punishable under Section 302, IPC. Therefore, he
convicted the appellant under Section 302, IPC and imposed sentence of R.I. for life vide
judgment dated February 14, 1997. Feeling aggrieved, the appellant preferred an appeal
before the High Court of Judicature at Madras. The Division Bench of the High Court has
dismissed the appeal by its Judgment dated March 28, 2005 giving rise to the instant
appeal by special leave.
6. This Court has heard the learned counsel for the parties and considered the evidence on
record.
7. The fact that the deceased died a homicidal death is not in dispute before this Court.
The medical officer who had performed autopsy on the dead body of the deceased has
mentioned in detail the injuries noticed by him, in his substantive evidence
@page-SC786
before the court. Eye-witness Senthil Kumar has stated in his testimony that the appellant
had caused injuries on the neck of the deceased by means of a sickle. The autopsy report
also mentions in detail the injuries sustained by the deceased. It is nobody's case that the
injuries found on the dead body of the deceased were self- inflicted. Therefore, the fact
that the deceased died a homicidal death stands proved beyond pale of doubt.
8. The testimony of eye-witness Senthil Kumar would indicate that on the day of the
incident the deceased in his company had gone to the field of the appellant and had
demanded bitterguard. According to the said witness because of the elopement of Punitha
with her teacher, the appellant was annoyed and had, therefore, refused to give
bitterguard saying that they had defamed his family. What is stated by the said witness is
that the deceased had thereupon asked the appellant to talk in a decent manner and had
hardly turned his back to leave the field of the appellant when the appellant had delivered
a blow with sickle on the neck of the deceased. Though this witness was cross-examined
at length nothing substantial could be brought on record which would cast a doubt on his
assertion that the appellant was the author of the injuries sustained by the deceased.
Similarly, another witness Thiru Ramasami has also deposed that on the day of incident
he had seen the deceased going towards the field of the appellant in the company of
Senthil Kumar and had asked the deceased as to where he was going. According to this
witness thereupon the deceased had informed the witness that he was going to the field of
the appellant to get bitterguard. What is asserted by the witness is that he had advised the
deceased not to go to the field of the appellant as the appellant was nurturing a feeling
that they had helped Punitha to elope with her teacher. The witness has further stated that
the deceased had stated that they would come back if the appellant was not inclined to
give bitterguard. The witness has also stated that he had heard altercation taking place
between the appellant and the deceased but was not able to follow the same as he was at a
little distance. What is claimed by the witness is that he had seen the appellant delivering
blow to the deceased and that he had gone to the field of the appellant to help his injured
son who was bleeding profusely. The witness has mentioned that the people who were in
the nearby fields, had gathered and thereafter he had gone to the police station where he
had learnt that the appellant had already lodged a complaint against himself.
9. This witness is also cross-examined in detail, but nothing could be brought on record
to impeach his credibility. This witness stands fully corroborated in material particulars
by the testimony of witness Senthil Kumar. This Court finds that the Sessions Court and
High Court were justified in holding that the appellant was the author of the injuries
sustained by the deceased.
10. This brings the court to consider the question as to which offence is committed by the
appellant. Admittedly, the incident had taken place in the field/garden belonging to the
appellant, where he was engaged in his farming activities. From the evidence led by the
prosecution it is evident that the deceased, in the company of witness Senthil Kumar had
gone to the field of the appellant to get bitterguard though they were warned not to do so
by the father of the deceased. In spite of knowing that the appellant was nurturing a
feeling that the deceased and his own sister's son had facilitated elopement of Punitha
with her teacher, the deceased in the company of Senthil Kumar had gone to the field of
the appellant on the pretext of getting bitterguard. The testimony of the father of the
deceased establishes that his deceased son, in the company of witness Senthil Kumar had
stayed in the field of the appellant for about 15 minutes and that there was an altercation
between the appellant and the deceased. The appellant never knew and anticipated that
the deceased would enter his field nor had prepared himself in advance to attack the
deceased. Thus there was no premeditation or pre-plan on the part of the appellant, to
cause the death of the deceased. Though the appellant is senior in age to the deceased, the
deceased had advised the appellant to behave nicely without rhyme or reason, when the
appellant had refused to part with bitterguard saying that the deceased and others had
disgraced his family by facilitating elopement of Punitha with her teacher. It is not the
case of the prosecution that on seeing that the deceased was entering his field in the
company of Senthil Kumar, the appellant had straightway
@page-SC787
attacked him. The evidence led by the prosecution clearly establishes that after verbal
duel, which had lasted for pretty long time, the appellant had picked up a sickle which is
an agricultural implement, lying on the ground and delivered a blow on the neck of the
deceased. By entering the field of the appellant on the pretext of getting bitterguard,
though he was knowing fully well that the appellant was nurturing a feeling that he had
played a role in the elopement of Punitha with her teacher as well as engaging himself in
an altercation with the appellant, and advising the appellant to behave the deceased had
offered grave and sudden provocation to the appellant as a result of which the appellant,
in the heat of the moment had delivered a blow with sickle to the deceased. The Medical
Officer who had performed autopsy on the dead body of the deceased has not stated that
the injuries sustained by the deceased were sufficient in the ordinary course of nature to
cause his death. It is not the case of the prosecution that the appellant had acted cruelly, in
the sense that he had delivered successive blows to the deceased. There was sufficient
time and opportunity to the appellant to give repeated blows. It is not the case of the
prosecution that the appellant wanted to deliver other blows and that he was prevented
from doing so, by any person. So, there is reasonable ground to believe that after giving
the blow, the appellant had stopped and not acted cruelly. As noticed earlier, the appellant
was doing his work and was not waiting for the deceased to come. On the facts and in the
circumstances of the case, this Court is of the opinion that Exception 1 to Section 300,
IPC would apply to the facts of the case and the offence committed by the appellant
would be one punishable under Section 304, IPC. There is nothing on record to indicate
that the appellant had committed culpable homicide amounting to murder by causing
death of the deceased with the intention of causing death of the deceased or of causing
such bodily injury as was likely to cause his death. Therefore, the provisions of Part II of
Sction 304, IPC would apply to the facts of the case on hand. Thus, the appeal will have
to be allowed by converting the conviction of the appellant under Section 302, IPC to one
punishable under Section 304 Part II, IPC. This Court has considered the submissions
advanced at the bar for the purpose of imposition of sentence on the appellant for
commission of offence punishable under Section 304, Part II, IPC. As held earlier there
was no pre-meditation or pre-plan on the part of the appellant to cause death of the
deceased, and the occurrence had taken place when the deceased, with another had
entered the field of the appellant and engaged himself in an altercation with the appellant
when the appellant had refused to part with bitterguard. Having regard to the attending
circumstances in which the incident had taken place, this Court is of the opinion that the
interest of justice would be served if the appellant is sentenced to rigorous imprisonment
for five years for commission of offence punishable under Section 304, Part II, IPC.
11. For the foregoing reasons, the appeal partly succeeds. The judgment of the High
Court of Judicature at Madras dated March 28, 2005 rendered in Criminal Appeal No.
648 of 1997, confirming the conviction of the appellant under Section 302, IPC and
sentence of life imprisonment recorded by the learned First Additional Sessions Judge,
Coimbatore vide judgment dated February 14, 1997, delivered in Sessions Case No. 63 of
1996, is set aside. Instead the appellant is convicted for commission of an offence
punishable under Section 304, Part II, IPC for the said offence. The appellant is sentenced
to undergo rigorous imprisonment for five years and a fine of Rs. 5,000/-, in default,
simple imprisonment for one year. The appeal accordingly stands disposed of.
Appeal partly allowed.
AIR 2008 SUPREME COURT 787 "Sanapareddy Maheedhar Seshagiri v. State of
Andhra Pradesh"
(From : 2007 (1) Andh LD (Cri) 526)
Coram : 2 S. B. SINHA AND G. S. SINGHVI, JJ.
Criminal Appeal No. 1708 of 2007 (arising out of SLP (Cri.) No. 6680 of 2006), D/- 13
-12 -2007.
Sanapareddy Maheedhar Seshagiri and Anr. v. State of A.P. and Anr.
(A) Criminal P.C. (2 of 1974), S.482 - INHERENT POWERS - HIGH COURT -
Quashing of proceedings - Exercise of powers by High Court - When permissible.
The High Court should be extremely cautious and slow to interfere with the investigation
and/or trial of criminal cases and should not stall the investigation and/or prosecution
except when it is convinced
@page-SC788
beyond any manner of doubt that the FIR does not disclose commission of any offence or
that the allegations contained in the FIR do not constitute any cognizable offence or that
the prosecution is barred by law or the High Court is convinced that it is necessary to
interfere to prevent abuse of the process of the Court. In dealing with such cases, the
High Court has to bear in mind that judicial intervention at the threshold of the legal
process initiated against a person accused of committing offence is highly detrimental to
the larger public and societal interest. The people and the society have a legitimate
expectation that those committing offences either against an individual or the society are
expeditiously brought to trial and, if found guilty, adquately punished. Therefore, while
deciding a petition filed for quashing the FIR or complaint or restraining the competent
authority from investigating the allegations contained in the FIR or complaint or for
stalling the trial of the case, the High Court should be extremely careful and circumspect.
If the allegations contained in the FIR or complaint discloses commission of some crime,
then the High Court must keep its hands off and allow the investigating agency to
complete the investigation without any fetter and also refrain from passing order which
may impede the trial. The High Court should not go into the merits and demerits of the
allegations simply because the petitioner alleges malus animus against the author of the
FIR or the complainant. The High Court must also refrain from making imaginary
journey in the realm of possible harassment which may be caused to the petitioner on
account of investigation of the FIR or complaint. Such a course will result in miscarriage
of justice and would encourage those accused of committing crimes to repeat the same.
However, if the High Court is satisfied that the complaint does not disclose commission
of any offence or prosecution is barred by limitation or that the proceedings of criminal
case would result in failure of justice, then it may exercise inherent power under S. 482,
Cr. P. C.
(Para 30)
(B) Criminal P.C. (2 of 1974), S.482 - INHERENT POWERS - CRUELTY BY
HUSBAND OR HIS RELATIVE - BREACH OF TRUST - DOWRY - Abuse of process
of Court - Proceedings against husband under S. 498A, 406, IPC and Ss. 4, 6, Dowry
Prohibition Act, 1961 - Continuation of, at belated stage - Found to be amounting to
harassment to husband - Also amounting to abuse of process of Court - Held, liable to be
quashed.
In the instant case after marriage the respondent-wife lived with appellant-husband for
less than one and a half months (eight days in India and about thirty days in Foreign
Country). The proceedings under Ss. 498A, 406 of IPC and Ss. 4, 6 of Dowry Prohibition
Act were initiated by respondent against appellant and his parents in year 2000. The High
Court quashed the said proceedings qua the parents of the appellant on the ground that
the Magistrate could not have taken cognizance after three years. Respondent is not
shown to have challenged the said order. Therefore, that order will be deemed to have
become final. Therefore, at this belated stage, there could be no justification for
continuation of the said proceedings against the husband. Rather, it would amount to
sheer harassment to the appellant and the respondent who are settled in USA, if they are
required to come to India for giving evidence in relation to an offence allegedly
committed in 1998-99. It is also extremely doubtful whether the Govt. of India will, after
lapse of such a long time, give sanction in terms of S. 188, Cr. P. C. Therefore, in the
peculiar facts the continuation of said proceedings against the husband would amount to
abuse of the process of the Court and thus liable to be set aside.
(Paras 33, 34)
Cases Referred : Chronological Paras
2005 AIR SCW 1319 : AIR 2005 SC 1989 : 2005 Cri LJ 1732 (Ref.) 22
2004 AIR SCW 6185 : AIR 2005 SC 9 : 2005 Cri LJ 92 : 2005 AIR-Jhar HCR 19 (Ref.)
12, 27
2003 AIR SCW 4062 : AIR 2003 SC 3635 : 2003 Cri LJ 3888 : 2003 AIR-Jhar HCR 1181
(Ref.) 12, 29
1999 AIR SCW 1793 : AIR 1999 SC 2071 : 1999 Cri LJ 3479 (Ref.) 19, 21, 22
1999 AIR SCW 4413 : AIR 2000 SC 297 : 2000 Cri LJ 485 (Ref.) 20
1993 AIR SCW 1866 : AIR 1993 SC 1637 : 1993 Cri LJ 2516 (Ref.) 14
1993 AIR SCW 3595 (Ref.) 17
1992 AIR SCW 237 : AIR 1992 SC 604 : 1992 Cri LJ 527 (Ref.) 26, 27
AIR 1982 SC 949 : 1982 Cri LJ 819 (Ref.) 26
AIR 1981 SC 1054 : 1981 Cri LJ 722 (Ref.) 16, 17
AIR 1980 SC 326 : 1980 Cri LJ 98 (Ref.) 26
@page-SC789

AIR 1960 SC 866 : 19660 Cri LJ 1239 (Ref.) 24, 26


Ms. Bina Madhavan and S. Udaya Kumar Sagar (for M/s. Lawyer's Knit and Co.), for
Appellants; I. V. Narayana, Sr. Advocate, T. N. Rao, Ms. Manjeet Kirpal, Paramjeet Singh
and L. D. Rajendar, for Respondents.
Judgement
G. S. SINGHVI, J. :- Leave granted.
2. This appeal is directed against the order dated 6.12.2006 passed by the learned Single
Judge of the Andhra Pradesh High Court whereby he dismissed the petition filed by the
appellants under Section 482 of the Criminal Procedure Code (for short Cr.P.C.) for
quashing the proceedings of CC No.240/2002 pending in the Court of XXII Metropolitan
Magistrate, Hyderabad in relation to offences under Sections 498A and 406, Indian Penal
Code read with Sections 4 and 6 of the Dowry Prohibition Act, 1961 (for short "the
Dowry Act").
3. Bhavani Shireesha, the eldest daughter of respondent No. 2 Shrimati D. Shaila, is a
doctor by profession. She was married to appellant No. 1 Sanapareddy Maheedhar
Seshagiri who is working as Software Engineer at New Jersey, USA on 22.04.1998 at
Hyderabad. Before marriage, the appellants and their parents demanded Rs. 5 lakh cash,
50 tola gold jewellery and Rs. 75,000/- towards Adapaduchu Katnam as dowry. They also
demanded transfer of the ground floor of the residential house belonging to respondent
No. 2 and her husband in favour of the parents of the appellants. Respondent No. 2 and
her husband agreed to pay Rs. 4 lakh cash, 60 tola gold jewellary and Rs. 75,000/-
towards Adapaduchu Katnam as dowry. They also agreed to bequeath the ground portion
of their house in the name of their daughter. The appellants and their parents accepted the
proposal and performed betrothal on 16.04.1998. Thereafter, the parents of the appellants
demanded Zen car and threatened to cancel the engagement unless the car is given. This
compelled the husband of respondent no. 2 to raise loan of Rs. 4 lakh and purchased the
car, which is said to have been kept at the disposal of the parents of the appellants. After
marriage, the appellants left for USA, but Shireesha Bhavani stayed back at Hyderabad
with their parents because she was undergoing training as House Surgeon. After
completing the training, Shireesha Bhavani went to USA along with the parents of the
appellants. She stayed at New Jersey from 1.11.1998 to 2.12.1998. During this period,
Shireesha Bhavani was subjected to cruelty and harassment by the appellants and their
parents on the ground that she did not bring enough dowry. On 3.12.1998 she went to
Maryland (U.S.A.) and stayed with her relatives. In April 1999, the parents of the
appellants returned to India. On 5.4.1999, appellant No.1 instituted divorce petition in
Superior Court at New Jersey and an ex parte decree was passed in his favour on
15.12.1999.
4. In the meanwhile, Shireesha Bhavani wrote letter dated 13.04.1999 to her parents
complaining of cruelty by the appellants and their parents. She disclosed that while she
was staying with the parents of the appellants at Hyderabad, the mother-in-law always
complained of lack of dowry and abused and criticized her and asked her to do menial
job. She further disclosed that appellant No. 1 and his brother harassed and also
pressurized her to bring additional money for purchase of a house at Hyderabad in the
name of the in-laws. She gave detailed account of the alleged harassment and torture
meted out by the appellants and their parents. Thereupon, respondent No. 2 filed
complaint dated 26.8.1999 in the Court of XXII Metropolitan Magistrate, Hyderabad
(hereinafter referred to as "the concerned Magistrate") detailing therein the facts relating
to demand of dowry by the appellants and their parents and the incidents of cruelty and
harassment to which her daughter was subjected at Hyderabad and New Jersey.
Respondent No. 2 also alleged that immediately after marriage, the appellants and their
parents complained about lack of dowry by saying that appellant No. 1 could have been
married for a dowry of Rs. 35 lakhs. Another allegation made by respondent No. 2 was
that her daughter was driven out of the house with an indication that she will be allowed
to return only after the demands of the accused-appellants and their parents are met. The
learned Magistrate referred the complaint for investigation under Section 156(3), Cr.P.C.
This led to registration of Crime No. 54/1999 at Women Police Station, CID, Hyderabad.
On 18.9.2000 the Inspector of Police, Women Protection Cell, C.I.D., Hyderabad
submitted final report with the prayer that the case may be treated as closed due to lack of
evidence. He mentioned that much progress could not be made due to non-availability of
de facto
@page-SC790
victim and other key witnesses in India and there was no immediate prospect of their
coming to India. He also mentioned that the accused party returned the personal
belongings including gold jewellery to the de facto victim in U.S.A. and that a decree of
divorce had been passed by the Superior Court of New Jersey, Chancery Division, Family
Part, Middlesex County. The Investigating Officer also made a reference to the direction
given by Additional Director General of Police, CID to close the case due to lack of
evidence.
5. By an order dated 1.11.2000, the concerned Magistrate rejected the final report and
directed the police to make further investigation. In the opinion of the learned Magistrate,
the investigation had not been done properly and the final report submitted under the
dictates of the Additional Director General of Police was not acceptable. While doing so,
the learned Magistrate made a reference to the letter addressed by Director General of
Police, CID, Andhra Pradesh to the Regional Passport Office, Hyderabad wherein it was
mentioned that Shrimati Bhavani Shireesha had been subjected to cruelty and a request
was made to cancel or impound the passport of the appellants.
6. In compliance of the direction given by the learned Magistrate the police conducted
further investigation and recorded statements of 18 persons. Notice was also issued to
Shrimati Shireesha Bhavani to appear before CID Police, Hyderabad. At that stage,
respondent No. 2 filed Criminal Petition No. 3912 of 2000 under Section 482, Cr.P.C. for
quashing the notice issued by the Inspector of Police, CID, Hyderabad for appearance of
her daughter in connection with the Crime No. 54 of 1999. The same was disposed of by
the learned Single Judge on 22.9.2000 with liberty to the petitioner to approach the
investigating agency and inform it about the efforts being made by her daughter to come
to India or to approach the concerned court for non-acceptance of final report, if any,
submitted by the police. Respondent No. 2 also filed Writ Petition No. 1173 of 2001 for
issue of a mandamus to the Regional Passport Officer, Secunderabad to impound the
passport of appellant no. 1 herein. That petition was disposed of by the learned Single
Judge on 26.9.2000 with a direction to the Regional Passport Officer to take appropriate
decision on the complaint made by respondent No. 2.
7. It is borne out from the record that on an application made by respondent No. - 2 the
concerned Magistrate issued warrant for search of the premises of the parents of the
appellants for recovery of the dowry articles and passport of her daughter. In the course
of search conducted by Sri P.Ventaka Rami Reddy, Inspector of Police (Women
Protection Cell) CID, Hyderabad on 19.7.2000 the parents of the appellants disclosed that
the passport has been sent to Shrimati B. Shireesha by Ordinary Post some time in
January/February, 1999, but they could not produce any evidence to substantiate the
same.
8. After disposal of Criminal Petition No. 3912 of 2000, Bhavani Shireesha obtained
duplicate passport and visa and came to India on 26.7.2002. She appeared before the
Investigating Officer on 27.7.2002 and gave statement under Section 161, Cr.P.C.
Thereafter, the police filed a charge-sheet under Sections 498A and 406, IPC read with
Sections 3, 4 and 6 of the Dowry Act. On 4.10.2002 the concerned Magistrate took
cognizance of the case and issued summons to the appellants and their parents.
9. It is also borne out from the record that without disclosing the fact that the concerned
Magistrate had already rejected the final report, the appellants and their parents filed Writ
Petition Nos. 6237 of 2001 and 2284 of 2001 with the prayer for quashing the
proceedings of Crime No. 54 of 1999 on the file of Women Protection Cell, CID,
Hyderabad. The learned Single Judge who heard the writ petitions made a reference to
order dated 26.9.2000 passed by another learned Single Judge in Criminal Petition No.
3912 of 2000 and disposed of both the petitions on 4.12.2001 by directing XXII
Metropolitan Magistrate, Hyderabad to pass appropriate order on the final report within a
period of two months of receipt of the copy of the order.
10. The parents of the appellants challenged the proceedings of CC No. 240 of 2002 in
Criminal Petition No. 1302 of 2003 filed under Section 482, Cr.P.C. They pleaded that in
view of the bar contained in Section 468, Cr.P.C. the concerned Magistrate did not have
the jurisdiction to take cognizance of the offences under Sections 498A and 406, IPC read
with Sections 3 and 4 of the Dowry Act. By an order dated 24.10.2006 the learned Single
Judge accepted their plea and quashed the proceedings of CC No. 240 of 2002. While
doing so,
@page-SC791
the learned Single Judge also expressed doubt regarding Bhavani Shireesha having come
to India for the purpose of making statement before the police.
11. Encouraged by the success of litigious venture undertaken by their parents, the
appellants filed Criminal Petition No. 4152 of 2006 for quashing the proceedings in CC
No. 240 of 2002. They pleaded that after the expiry of three years counted from the date
of filing the complaint, the learned Magistrate could not have taken cognizance of the
offences allegedly committed by them under Sections 498A and 406 read with Sections 4
and 6 of the Dowry Act. Another plea taken by them was that in the face of the decree of
divorce passed by the Superior Court at New Jersey, USA and the fact that Shrimati
Shireesha Bhavani had contracted marriage with one Mr. Venkat Puskar in the year 2000,
there was no warrants for initiation of criminal proceedings against them, and that the
offences allegedly committed by them outside India cannot be enquired into or tried
without obtaining prior sanction of the Central Government in terms of Section 188,
Cr.P.C.
12

. The learned Single Judge briefly referred to the parameters for exercise of power by the
High Court under Section 482, Cr.P.C., the ingredients of Sections 498A and 406, IPC
and Sections 3 and 4 of the Dowry Act and held that the proceedings in CC No. 240/2002
cannot be quashed because the learned Magistrate had taken cognizance within three
years. The learned Single Judge distinguished the judgments of this Court in M/s. Zandu
Pharmaceutical Works Ltd. v. Mohd. Sharaful Haque [2005 (1) SCC 122] and Ramesh
Chandra Sinha and Ors. v. State of Bihar and Ors. [2003 (7) SCC 254] by observing that
in those cases the Magistrate had taken cognizance long after three years. He then
observed that each act of cruelty could be a new starting point of limitation and,
therefore, the cognizance taken by the Magistrate cannot be treated as barred by time. As
regards the ex parte decree of divorce passed by the Court at New Jersey, the learned
Single Judge observed that the foreign judgment is not conclusive and that various facts
are required to be proved and established before the Criminal Court. The learned Single
Judge rejected the appellant's plea regarding lack of sanction of the Central Government
by observing that such sanction can be obtained even during the trial. 2004 AIR
SCW 6185
2003 AIR SCW 4062

13. Ms. Beena Madhavan, learned counsel for the appellants reiterated the contentions
raised on behalf of her clients before the High Court and argued that the learned Single
Judge committed an error by refusing to quash the proceedings of CC No.240 of 2002
ignoring the fact that the learned Magistrate had taken cognizance after almost four years
of the last act of alleged cruelty committed against Shireesha Bhavani. She submitted that
after dissolution of the marriage, Shrimati Shireesha Bhavani had taken back the Gold
and Silver jewellery and then contracted marriage with Mr. Venkat Puskar and this fact
ought to have been considered by the learned Single Judge while examining the
appellant's pleas that the proceedings of criminal case instituted against them amounts to
an abuse of the process of law. She then argued that in exercise of the power under
Section 482, Cr.P.C., the High Court is duty-bound to quash the proceedings which are
barred by time and protect the appellants against unwarranted persecution.
14
. Shri I.Venkata Narayana, learned Senior Advocate appearing for respondent No.2,
supported the order under challenge and argued that the learned Single Judge of the High
Court rightly declined to quash the proceedings of criminal case because the offences
committed by the appellants are continuing in nature. Shri Venkata Narayana further
argued that even though as on the date of taking cognizance of offences by the learned
Magistrate, a period of more than three years had elapsed, the proceedings of CC No.
240/2002 cannot be declared as barred by limitation because the appellants were not in
India and the period of their absence is liable to be excluded in terms of Section 470(4).
Shri Venkata Narayana relied on Section 472 and argued that offences of cruelty and
criminal breach of trust are continuing offences and prosecution launched against the
appellants cannot be treated as barred by time. He then submitted that the learned
Magistrate could also exercise power under Section 473 for extending the period of
limitation because the appellants and their parents did not co-operate in the investigation
and also prevented Smt. Shireesha Bhavani from coming to India to give her statement.
Lastly, the learned Senior Counsel relied on the 1993 AIR SCW 1866

@page-SC792
judgment of this Court in Ajay Agarwal v.Union of India [1993 (3) SCC 609] and argued
that the proceedings of the criminal case cannot be quashed only on the ground of lack of
sanction under Section 188, Cr.P.C.
15. We have considered the respective submissions and carefully scrutinised the record.
For deciding whether the learned Magistrate could take cognizance of offence under
Sections 498A and 406, IPC read with Sections 4 and 6 of the Dowry Act after expiry of
three years, it will be useful to notice the scheme of Chapter XXXVI of the Code of
Criminal Procedure. Section 468 which finds place in that Chapter creates a bar against
taking cognizance of an offence after lapse of the period of limitation. Sub-section (1)
thereof lays down that except as otherwise provided elsewhere in this Code, no Court,
shall take cognizance of an offence of the category specified in sub-section (2), after the
expiry of the period of limitation. Sub-section (2) specifies different periods of limitation
for different types of offences punishable with imprisonment for a term exceeding one
year but not exceeding three years, the period of limitation is three years. Section 469
specifies the point of time with reference to which the period of limitation is to be
counted. Section 470 provides for exclusion of time in certain cases. Sub-section (4)
thereof lays down that in computing the period of limitation, the time during which the
offender has been absent from India or from any territory outside India which is under the
administration of the Central Government or has avoided arrest by absconding or
concealing himself, shall be excluded. Section 472, which deals with continuing offence
declares that in case of a continuing offence, a fresh period of limitation shall begin to run
at every moment of the time during which the offence continues. Section 473, which
begins with non-obstante clause, empowers the Court to take cognizance of an offence
after the expiry of the period of limitation, if it is satisfied that the delay has been
properly explained and it is necessary so to do in the interest of justice.
16

. In State of Punjab v. Sarwan Singh [1981 (3) SCC 34], this Court noted that the object
of Section 468 Cr.P.C. is to create a bar against belated prosecutions and to prevent abuse
of the process of the court and observed that this is in consonance with the concept of
fairness of trial enshrined in Article 21 of the Constitution. AIR 1981 SC 1054

17

. In Venka Radhamanohari v. Vanka Venkata Reddy [1993 (3) SCC 4] this Court
considered the applicability of Section 468 to the cases involving matrimonial offences,
referred to the judgment in Sarwan Singhs case (supra) and observed : 1993 AIR
SCW 3593

"It is true that the object of introducing Section 468 was to put a bar of limitation on
prosecutions and to prevent the parties from filing cases after a long time, as it was
thought proper that after a long lapse of time, launching of prosecution may be vexatious,
because by that time even the evidence may disappear. This aspect has been mentioned in
the statement and object, for introducing a period of limitation, as well as by this Court in
the case of Sarwan Singh (supra). But, that consideration cannot be extended to
matrimonial offences, where the allegations are of cruelty, torture and assault by the
husband or other members of the family to the complainant. It is a matter of common
experience that victim is subjected to such cruelty repeatedly and it is more or less like a
continuing offence. It is only as a last resort that a wife openly comes before a court to
unfold and relate the day-to-day torture and cruelty faced by her, inside the house, which
many of such victims do not like to be made public. As such, courts while considering the
question of limitation for an offence under Section 498-A i.e. subjecting a woman to
cruelty by her husband or the relative of her husband, should judge that question, in the
light of Section 473 of the Code, which requires the Court, not only to examine as to
whether the delay has been properly explained, but as to whether it is necessary to do so
in the interests of justice.
[Emphasis added]
18. The court then compared Section 473, Cr.P.C. with Section 5 of the Limitation Act
and observed :
"For exercise of power under Section 5 of the Limitation Act, the onus is on the appellant
or the applicant to satisfy the court that there was sufficient cause for condonation of the
delay, whereas Section 473 enjoins a duty on the court to examine not only whether such
delay has been explained but as to whether it is the requirement of the justice to condone
or ignore such delay.
@page-SC793
As such, whenever the bar of Section 468 is applicable, the court has to apply its mind on
the question, whether it is necessary to condone such delay in the interests of justice.
While examining the question as to whether it is necessary to condone the delay in the
interest of justice, the Court has to take note of the nature of offence, the class to which
the victim belongs, including the background of the victim. If the power under Section
473 of the Code is to be exercised in the interests of justice, then while considering the
grievance by a lady, of torture, cruelty and inhuman treatment, by the husband and the
relatives of the husband, the interest of justice requires a deeper examination of such
grievances, instead of applying the rule of limitation and saying that with lapse of time
the cause of action itself has come to an end. The general rule of limitation is based on
the Latin maxim : v igilantibus, et non, dormientibus, jura subveniunt (the vigilant, and
not the sleepy, are assisted by the laws). That maxim cannot be applied in connection
with offences relating to cruelty against women. "
[Emphasis added]
19

. In Arun Vyas v. Anita Vyas [1999 (4) SCC 690 : 1999 SCC (Cri) 629] this Court again
considered the applicability of Section 473, Cr.P.C. in cases relating to matrimonial
offences and observed : 1999 AIR SCW 1793, Para 14

"The first limb confers power on every competent court to take cognizance of an offence
after the period of limitation if it is satisfied on the facts and in the circumstances of the
case that the delay has been properly explained and the second limb empowers such a
Court to take cognizance of an offence if it is satisfied on the facts and in the
circumstances of the case that it is necessary so to do in the interests of justice. It is true
that the expression in the interest of justice in Section 473 cannot be interpreted to mean
in the interest of prosecution. What the court has to see is interest of justice. The interest
of justice demands that the court should protect the oppressed and punish the
oppressor/offender. In complaints under Section 498-A the wife will invariably be
oppressed, having been subjected to cruelty by the husband and the in-laws. It is,
therefore, appropriate for the courts, in case of delayed complaints, to construe liberally
Section 473, Cr.P.C. in favour of a wife who is subjected to cruelty if on the facts and in
the circumstances of the case it is necessary so to do in the interests of justice. When the
conduct of the accused is such that applying the rule of limitation will give an unfair
advantage to him or result in miscarriage of justice, the court may take cognizance of an
offence after the expiry of the period of limitation in the interests of justice. This is only
illustrative, not exhaustive."
20

. In State of H.P. v. Tara Dutt [2000 (1) SCC 230] a three Judges Bench of this Court
considered whether there can be a presumption of condonation of delay under Section
473 Cr.P.C. and observed : 1999 AIR SCW 4413, (Para 7)

"Section 473 confers power on the Court taking cognizance after the expiry of the period
of limitation, if it is satisfied on the facts and in the circumstances of the case that the
delay has been properly explained and that it is necessary so to do in the interest of
justice. Obviously, therefore in respect of the offences for which a period of limitation
has been provided in Section 468, the power has been conferred on the court taking
cognizance to extend the said period of limitation where a proper and satisfactory
explanation of the delay is available and where the Court taking cognizance finds that it
would be in the interest of justice. This discretion conferred on the Court has to be
exercised judicially and on well-recognised principles. This being a discretion conferred
on the court taking cognizance, wherever the court exercises this discretion, the same
must be by a speaking order, indicating the satisfaction of the court that the delay was
satisfactorily explained and condonation of the same was in the interest of justice. In the
absence of a positive order to that effect it may not be permissible for a superior court to
come to the conclusion that the court must be deemed to have taken cognizance by
condoning the delay whenever the cognizance was barred and yet the court took
cognizance and proceeded with the trial of the offence. But the provisions are of no
application to the case in hand since for the offences charged, no period of limitation has
been provided in view of the imposable punishment thereunder. In this view of the matter
we have no hesitation to come to the conclusion that the High Court committed serious
error in holding that the conviction of the two respondents under Section 417 would be
barred as on the date of taking
@page-SC794
cognizance the Court could not have taken cognizance of the said offence. Needless to
mention, it is well settled by a catena of decisions of this Court that if an accused is
charged with a major offence but is not found guilty thereunder, he can be convicted of a
minor offence if the facts established indicate that such minor offence has been
committed."
21

. This Court then considered the earlier judgment in Arun Vyas v. Anita Vyas (supra) and
held : 1999 AIR SCW 1793

"The aforesaid observations made by this Court indicate that the order of the Magistrate
at the time of taking cognizance in case of an offence under Section - 498?A, should
indicate as to why the Magistrate does not think it sufficient in the interest of justice to
condone the delay inasmuch as an accused committing an offence under Section 498-A
should not be lightly let off. We have already indicated in the earlier part of this judgment
as to the true import and construction of Section 473 of the Code of Criminal Procedure.
The said provision being an enabling provision, whenever a Magistrate invokes the said
provision and condones the delay, the order of the Magistrate must indicate that he was
satisfied on the facts and circumstances of the case that the delay has been properly
explained and that it is necessary in the interest of justice to condone the delay. But
without such an order being there or in the absence of such positive order, it cannot (sic)
be said that the Magistrate has failed to exercise jurisdiction vested in law. It is no doubt
true that in view of the fact that an offence under Section 498-A is an offence against the
society and, therefore, in the matter of taking cognizance of the said offence, the
Magistrate must liberally construe the question of limitation but all the same the
Magistrate has to be satisfied, in case of period of limitation for taking cognizance under
Section 468(2)(c) having expired that the circumstances of the case require delay to be
condoned and further the same must be manifest in the order of the Magistrate itself. This
in our view is the correct interpretation of Section 473 of the Code of Criminal
Procedure.
22

. In Ramesh v. State of Tamil Nadu [2005 (3) SCC 507] this Court considered the issue of
limitation in taking cognizance of an offence under Section 498A and observed : 2005
AIR SCW 1319
1999 AIR SCW 1793
"On the point of limitation, we are of the view that the prosecution cannot be nullified at
the very threshold on the ground that the prescribed period of limitation had expired.
According to the learned counsel for the appellants, the alleged acts of cruelty giving rise
to the offence under Section 498-A ceased on the exit of the informant from the
matrimonial home on 2-10-1997 and no further acts of cruelty continued thereafter. The
outer limit of time for taking cognizance would therefore be 3-10-2000, it is contended.
However, at this juncture, we may clarify that there is an allegation in the FIR that on 13-
10-1998/14-10-1998, when the informants close relations met her in-laws at a hotel in
Chennai, they made it clear that she will not be allowed to live with her husband in
Mumbai unless she brought the demanded money and jewellery. Even going by this
statement, the taking of cognizance on 13-2-2002 pursuant to the charge-sheet filed on
28-12-2001 would be beyond the period of limitation. The commencement of limitation
could be taken as 2-10-1997 or at the most 14-10-1998. As pointed out by this Court in
Arun Vyas v. Anita Vyas (supra) the last act of cruelty would be the starting point of
limitation. The three-year period as per Section 468(2)(c) would expire by 14-10-2001
even if the latter date is taken into account. But that is not the end of the matter. We have
to still consider whether the benefit of extended period of limitation could be given to the
informant. True, the learned Magistrate should have paused to consider the question of
limitation before taking cognizance and he should have addressed himself to the question
whether there were grounds to extend the period of limitation. On account of failure to do
so, we would have, in the normal course, quashed the order of the Magistrate taking
cognizance and directed him to consider the question of applicability of Section 473.
However, having regard to the facts and circumstances of the case, we are not inclined to
exercise our jurisdiction under Article 136 of the Constitution to remit the matter to the
trial court for taking a decision on this aspect. The fact remains that the complaint was
lodged on 23-6-1999, that is to say, much before the expiry of the period of limitation and
the FIR was registered by the All-Women Police Station, Tiruchirapalli on that day. A
copy of the FIR was sent to the Magistrate's Court on the next day i.e. on 24-6-1999.
However, the process of investigation and
@page-SC795
filing of charge-sheet took its own time. The process of taking cognizance was
consequentially delayed. There is also the further fact that the appellants filed Writ
Petition (Crl.) No. 1719 of 2000 in the Bombay High Court for quashing the FIR or in the
alternative to direct its transfer to Mumbai. We are told that the High Court granted an ex
parte interim stay. On 20-8-2001, the writ petition was permitted to be withdrawn with
liberty to file a fresh petition. The charge-sheet was filed four months thereafter. It is in
this background that the delay has to be viewed."
23. The ratio of the above noted judgments is that while considering the applicability of
Section 468 to the complaints made by the victims of matrimonial offences, the Court can
invoke Section 473 and can take cognizance of an offence after expiry of the period of
limitation keeping in view the nature of allegations, the time taken by the police in
investigation and the fact that the offence of cruelty is a continuing offence and affects
the society at large. To put it differently, in cases involving matrimonial offences the
Court should not adopt a narrow and pedantic approach and should, in the interest of
justice, liberally exercise power under Section 473 for extending the period of limitation.
24. At this stage, we may also notice the parameters laid down by this Court for exercise
of power by the High Court under Section 482 Cr.P.C to give effect to any order made
under the Cr.P.C or to prevent abuse of the process of any court or otherwise to secure the
ends of justice. In R. P. Kapur v. State of Punjab [AIR 1960 SC 866] this Court
considered the question whether in exercise of its power under Section 561A of the Code
of Criminal Procedure, 1898 (Section 482 Cr.P.C. is pari materia to Section 561A of the
1898 Code), the High Court could quash criminal case registered against the appellant
who along with his mother-in-law was accused of committing offences under Sections
420, 109, 114 and 120B of the Indian Penal Code. The appellant unsuccessfully filed a
petition in the Punjab High Court for quashing the investigation of the First Information
Report (FIR) registered against him and then filed appeal before this Court. While
confirming the High Courts order this Court laid down the following proposition :-
"The inherent power of High Court under Section 561A, Criminal P.C. cannot be
exercised in regard to matters specifically covered by the other provisions of the Code.
The inherent jurisdiction of the High Court can be exercised to quash proceedings in a
proper case either to prevent the abuse of the process of any court or otherwise to secure
the ends of justice. Ordinarily criminal proceedings instituted against an accused person
must be tried under the provisions of the Code, and the High Court would be reluctant to
interfere with the said proceedings at an interlocutory stage. It is not possible, desirable or
expedient to lay down any inflexible rule which would govern the exercise of this
inherent jurisdiction."
25. This Court then carved out some exceptions to the above stated rule. These are :
(i) Where it manifestly appears that there is a legal bar against the institution or
continuance of the criminal proceedings in respect of the offences alleged. Absence of the
requisite sanction may, for instance, furnish cases under this category;
(ii) Where the allegations in the First Information Report or the complaint, even if they
are taken at their face value and accepted in their entirety, do not constitute the offence
alleged; in such cases no question of appreciating evidence arises; it is a matter merely of
looking at the complaint or the First Information Report to decide whether the offence
alleged is disclosed or not;
(iii) Where the allegations made against the accused person do constitute an offence
alleged but there is either no legal evidence adduced in support of the case or the
evidence adduced clearly or manifestly fails to prove the charge. In dealing with this class
of cases it is important to bear in mind the distinction between a case where there is no
legal evidence or where there is evidence which is manifestly and clearly inconsistent
with the accusation made and cases where there is legal evidence which on its
appreciation may or may not support the accusation in question. In exercising its
jurisdiction under Section 561-A the High Court would not embark upon an enquiry as to
whether the evidence in question is reliable or not. That is the function of the trial
magistrate, and ordinarily it would not be open to any party to invoke the High Courts
inherent jurisdiction and contend that on a reasonable appreciation of the evidence the
@page-SC796
accusation made against the accused would not be sustained.
26
. In State of Haryana v Bhajanlal [1992 Supp. (1) SCC 335] this Court considered the
scope of the High Courts power under Section 482 of Cr.P.C and Article 226 of the
Constitution to quash the FIR registered against the respondent, referred to several
judicial precedents including those of R. P. Kapoor v. State of Punjab (supra), State of
Bihar v. J.A.C. Saldanha [1980 (1) SCC 554] and State of West Bengal v. Swapan Kumar
Guha [1982 (1) SCC 561] and held that the High Court should not embark upon an
enquiry into the merits and demerits of the allegations and quash the proceedings without
allowing the investigating agency to complete its task. At the same time, the Court
identified the following cases in which the FIR or complaint can be quashed. 1992
AIR SCW 237
AIR 1960 SC 866
AIR 1980 SC 326
AIR 1982 SC 949

"(1) Where the allegations made in the first information report or the complaint, even if
they are taken at their face value and accepted in their entirety do not prima facie
constitute any offence or make out a case against the accused.
(2) Where the allegations in the first information report and other materials, if any,
accompanying the FIR do not disclose a cognizable offence, justifying an investigation
by police officers under Section 156(1) of the Code except under an order of a Magistrate
within the purview of Section 155(2) of the Code.
(3) Where the uncontroverted allegations made in the FIR or complaint and the evidence
collected in support of the same do not disclose the commission of any offence and make
out a case against the accused.
(4) Where the allegations in the FIR do not constitute a cognizable offence but constitute
only a non-cognizable offence, no investigation is permitted by a police officer without
an order of a Magistrate as contemplated under Section 155(2) of the Code.

(5) Where the allegations made in the FIR or complaint are so absurd and inherently
improbable on the basis of which no prudent person can ever reach a just conclusion that
there is sufficient ground for proceeding against the accused.
(6) Where there is an express legal bar engrafted in any of the provisions of the Code or
the Act concerned (under which a criminal proceeding is instituted) to the institution and
continuance of the proceedings and/or where there is a specific provision in the Code or
the Act concerned, providing efficacious redress for the grievance of the aggrieved party.
(7) Where a criminal proceeding is manifestly attended with mala fide and/or where the
proceeding is maliciously instituted with an ulterior motive for wreaking vengeance on
the accused and with a view to spite him due to private and personal grudge."
27

. The ratio of Bhajan Lal's case has been consistently followed in the subsequent
judgments. In M/s. Zandu Pharmaceutical Works Ltd. v. Mohd. Sharaful Haque (supra),
this Court referred to a large number of precedents on the subject and observed - : 1992
AIR SCW 237
2004 AIR SCW 6185
"The powers possessed by the High Court under Section 482 of the Code are very wide
and the very plenitude of the power requires great caution in its exercise. Court must be
careful to see that its decision in exercise of this power is based on sound principles. The
inherent power should not be exercised to stifle a legitimate prosecution. The High Court
being the highest court of a State should normally refrain from giving a prima facie
decision in a case where the entire facts are incomplete and hazy, more so when the
evidence has not been collected and produced before the court and the issues involved,
whether factual or legal, are of magnitude and cannot be seen in their true perspective
without sufficient material. Of course, no hard-and-fast rule can be laid down in regard to
cases in which the High Court will exercise its extraordinary jurisdiction of quashing the
proceeding at any stage. It would not be proper for the High Court to analyse the case of
the complainant in the light of all probabilities in order to determine whether a conviction
would be sustainable and on such premises arrive at a conclusion that the proceedings are
to be quashed. It would be erroneous to assess the material before it and conclude that the
complaint cannot be proceeded with. In a proceeding instituted on complaint, exercise of
the inherent powers to quash the proceedings is called for only in a case where the
complaint does not disclose any offence or is frivolous, vexatious or oppressive. If the
allegations set out in the complaint do not
@page-SC797
constitute the offence of which cognizance has been taken by the Magistrate, it is open to
the High Court to quash the same in exercise of the inherent powers under Section 482 of
the Code. It is not, however, necessary that there should be meticulous analysis of the
case before the trial to find out whether the case would end in conviction or acquittal. The
complaint has to be read as a whole. If it appears that on consideration of the allegations
in the light of the statement made on oath of the complainant that the ingredients of the
offence or offences are disclosed and there is no material to show that the complaint is
mala fide, frivolous or vexatious, in that even there would be no justification for
interference by the High Court. When an information is lodged at the police station and
an offence is registered, then the mala fides of the informant would be of secondary
importance. It is the material collected during the investigation and evidence led in court
which decides the fate of the accused person. The allegations of mala fides against the
informant are of no consequence and cannot by themselves be the basis for quashing the
proceedings."
28. In the aforementioned judgment, this Court set aside the order of the Patna High
Court and quashed the summons issued by the First Class Judicial Magistrate in
Complaint Case No.1613(C) of 2002 on the ground that the same was barred by
limitation prescribed under Section 468 (2)(C) Cr.P.C.
29

. In Ramesh Chand Sinha's case (supra) this Court quashed the decision of the Chief
Judicial Magistrate, Patna to take cognizance of the offence allegedly committed by the
appellants by observing that the same was barred by time and there were no valid
grounds to extend the period of limitation by invoking Section 473, Cr.P.C. 2003
AIR SCW 4062
30. A careful reading of the above noted judgments makes it clear that the High Court
should be extremely cautious and slow to interfere with the investigation and/or trial of
criminal cases and should not stall the investigation and/or prosecution except when it is
convinced beyond any manner of doubt that the FIR does not disclose commission of any
offence or that the allegations contained in the FIR do not constitute any cognizable
offence or that the prosecution is barred by law or the High Court is convinced that it is
necessary to interfere to prevent abuse of the process of the court. In dealing with such
cases, the High Court has to bear in mind that judicial intervention at the threshold of the
legal process initiated against a person accused of committing offence is highly
detrimental to the larger public and societal interest. The people and the society have a
legitimate expectation that those committing offences either against an individual or the
society are expeditiously brought to trial and, if found guilty, adequately punished.
Therefore, while deciding a petition filed for quashing the FIR or complaint or restraining
the competent authority from investigating the allegations contained in the FIR or
complaint or for stalling the trial of the case, the High Court should be extremely careful
and circumspect. If the allegations contained in the FIR or complaint discloses
commission of some crime, then the High Court must keep its hands off and allow the
investigating agency to complete the investigation without any fetter and also refrain
from passing order which may impede the trial. The High Court should not go into the
merits and demerits of the allegations simply because the petitioner alleges malus animus
against the author of the FIR or the complainant. The High Court must also refrain from
making imaginary journey in the realm of possible harassment which may be caused to
the petitioner on account of investigation of the FIR or complaint. Such a course will
result in miscarriage of justice and would encourage those accused of committing crimes
to repeat the same. However, if the High Court is satisfied that the complaint does not
disclose commission of any offence or prosecution is barred by limitation or that the
proceedings of criminal case would result in failure of justice, then it may exercise
inherent power under Section 482, Cr.P.C.
31. In the light of the above, we shall now consider whether the High Court committed an
error by refusing to quash the proceedings of CC No.240 of 2002.
32. Although, the learned Single Judge of High Court dealt with various points raised by
the appellants and negatived the same by recording the detailed order, his attention does
not appear to have been drawn to the order dated 24.10.2006 passed by the co-ordinate
bench in Criminal Petition No.1302/2003 whereby the proceedings of CC No.240/2002
were quashed qua the parents of the appellants on the ground that
@page-SC798
the learned Magistrate could not have taken cognizance after three years. Respondent
No.2 is not shown to have challenged the order passed in Criminal Petition
No.1302/2003. Therefore, that order will be deemed to have become final. We are sure
that if attention of the learned Single Judge, who decided Criminal Petition No.4152/2006
had been drawn to the order passed by another learned Single Judge in Criminal Petition
No.1302/2003, he may have, by taking note of the fact that the learned Magistrate did not
pass an order for condonation of delay or extension of the period of limitation in terms of
Section 473 Cr.P.C., quashed the proceedings of CC No.240/2002.
33. We are further of the view that in the peculiar facts of this case, continuation of
proceedings of CC No.240/2002 will amount to abuse of the process of the Court. It is
not in dispute that after marriage, Shireesha Bhavani lived with appellant No.1 for less
than one and a half months (eight days at Hyderabad and about thirty days at New
Jersey). It is also not in dispute that their marriage was dissolved by the Superior Court at
New Jersey vide decree dated 15.12.1999. Shireesha Bhavani is not shown to have
challenged the decree of divorce. As a matter of fact, she married Sri Venkat Puskar in
2000 and has two children from the second marriage. She also received all the articles of
dowry (including jewellery) by filing affidavit dated 28.12.1999 in the Superior Court at
New Jersey. As on today a period of almost nine years has elapsed of the marriage of
appellant No.1 and Shireesha Bhavani and seven years from her second marriage.
Therefore, at this belated stage, there does not appear to be any justification for
continuation of the proceedings in CC No.240/2002. Rather, it would amount to sheer
harassment to the appellant and Shireesha Bhavani who are settled in USA, if they are
required to come to India for giving evidence in relation to an offence allegedly
committed in 1998-99. It is also extremely doubtful whether the Government of India
will, after lapse of such a long time, give sanction in terms of Section 188, Cr.P.C.
34. For the reasons stated above, the appeal is allowed, the order of the learned Single
Judge of the High Court is set aside and the proceedings of CC No.240/2002, pending in
the Court of XXII Metropolitan Magistrate, Hyderabad, are quashed.
Appeal allowed.
AIR 2008 SUPREME COURT 798 "K.S.E.B., M/s. v. Commr. of Central Excise,
Thiruvananthapuram"
(From : 2006 (4) Ker LT 749)
Coram : 2 S. B. SINHA AND H. S. BEDI, JJ.
Civil Appeal No. 5832 of 2007 (arising out of SLP (C) No. 3724 of 2007), D/- 12 -12
-2007.
M/s. Kerala State Electricity Board v. Commr. of Central Excise, Thiruvananthapuram.
Finance Act (32 of 1994), S.65, S.69, S.75 - Service Tax Rules (1994), R.6 -
FINANCIAL PROVISIONS - SERVICE TAX - Liability to pay tax - Appellants entering
into agreement with foreign company for obtaining consultancy services for them -
Service provider did not have any independent office in India - Appellants, service
recipient, held, liable to pay service tax and interest on amount of tax due to such service
provider. (Para 21)
Cases Referred : Chronological Paras
2005 AIR SCW 613 : AIR 2005 SC 1103 : 2005 All LJ 559 19
2005 AIR SCW 2051 : AIR 2005 SC 3020 : 2006 Tax LR 75 (Ref.) 19
(2005)13 SCC 245 (Disting) 20
1999 AIR SCW 2771 : AIR 1999 SC 2596 : 1999 Tax LR 828 (Ref.) 19
AIR 1989 SC 516 19
AIR 1981 SC 774 : 1981 Tax LR 28919
AIR 1981 SC 1480 19
AIR 1975 SC 1758 19
AIR 1963 SC 1667 19
AIR 1962 SC 1281 (Ref.) 19
T. L. V. Iyer, Sr. Advocate, M. T. George, for Appellant; R. G. Padia, Sr. Advocate, Navin
Prakash, B. Krishna Prasad, for Respondent.
Judgement
S. B. SINHA, J. :- Leave granted.
A limited notice was issued to the effect as to whether the appellant Kerala State
Electricity Board, the service recipient, within the meaning of provisions of Finance Act,
1994, levying service tax, is liable to pay any interest on the amount of tax due to the
respondent.
2. The question involved in this appeal arises out of a judgment and order dated
25.7.2006 passed by a Division Bench of the High Court of Kerala at Ernakulam whereby
the appeal filed by the respondent herein from the judgment and order of the Customs
Excise and Service Tax Appellate Tribunal, Circuit Bench at Cochin in Final Order
No.477 of 2005, Appeal No.ST/36/2004 was allowed.
@page-SC799
3. The basic fact of the matter is not in dispute. Appellant herein entered into an
agreement with M/s. SNC Lavlin Inc. Montreal, Canada (Foreign company) in relation to
various projects for obtaining consultancy services from them.
The relevant clauses of the said agreement are as under :-
"16.1 - SNC Lavaline and all its expatriate personnel shall be responsible for timely and
prompt filing of all returns, estimates, accounts, information and details complete and
accurate in all respects as may be required under the applicable laws/regulations in India
before the appropriate authorities in India. In case SNC Lavaline or any of its expatriate
personnel do not comply with the above tax requirements, which results in any penalty,
interest or additional liability, the same shall be borne by SNC Lavaline.
16.2 - SNC Lavaline shall provide KSE Board the relevant orders/notices of demand,
invoices, appellate orders and other relevant information as the proof of the actual tax
liability to be borne by KSE Board, sufficiently in advance to enable KSEB to take
appropriate action in this connection.
16.3 - SNC Lavaline and its expatriate personnel, if required by KSEB, shall contest
appeals against any assessment/demand of an appropriate authority before such authority
at the request of and cost expenses of KSEB."
4. Despite the said contractual commitments, the appellant failed and/or neglected to pay
service tax on behalf of foreign company. It, on the other hand, raised a dispute that
having regard to the purported statutory obligations of the service provider as contained
in the Act and the Rules framed, it was not liable to pay any service tax.
5. By reason of the impugned judgment, the Division Bench of the Kerala High Court
construing the provisions of the Act in the light of the terms of the contract entered into
by and between the appellant and the foreign company opined that the liability in that
regard was on the appellant and not on the foreign company.
6. Mr. T.L.V. Iyer, learned senior counsel, in support of this appeal, inter alia, urged that
the liability to pay interest and penalty being statutory one, the service provider was
responsible therefor and not the service recipient.
7. Mr. R.G. Padia, learned senior counsel appearing on behalf of the respondent, on the
other hand, would support the impugned judgment.
8. The period for which the service tax was due is August 1998 to September 2002.
Under the agreement, indisputably, the appellant was responsible to make payment of the
service tax on behalf of the foreign company.
9. Section 65 of the Finance Act, 1994 provides for levy of service tax on the services
specified therein. Section 66 of the Act provides that the rate of tax shall be twelve per
cent of the value of taxable services specified therein and collected in such manner as
may be prescribed. Section 68 of the Act puts the burden of payment of tax on the service
provider.
Sections 68(2), 69(1), 71 and relevant parts of Sections 73 and 75 of the Finance Act,
1994 which are material for the purposes of this case, read as under :
"68. (2) Notwithstanding anything contained in sub-section (1), in respect of any taxable
service notified by the Central Government in the Official Gazette, the service tax
thereon shall be paid by such person and in such manner as may be prescribed at the rate
specified in section 66 and all the provisions of this Chapter shall apply to such person as
if he is the person liable for paying the service tax in relation to such service.
69. Registration.- (1) Every person liable to pay the service tax under this Chapter or the
rules made thereunder shall, within such time and in such manner and in such form as
may be prescribed, make an application for registration to the Superintendent of Central
Excise.
71. Verification of tax assessed by the assessee, etc.-
(1) The Superintendent of Central Excise may, on the basis of information contained in
the return filed by the assessee under section 70, verify the correctness of the tax assessed
by the assessee on the services provided.
(2) The Superintendent of Central Excise may require the assessee to produce any
accounts, documents or other evidence as he may deem necessary for such verification as
and when required.
(3) If on verification under sub-section (2),
@page-SC800
the Superintendent of Central Excise is of the opinion that service tax on any service
provided has escaped assessment or has been under-assessed, he may refer the matter to
the Assistant Commissioner of Central Excise or, as the case may be, the Deputy
Commissioner of Central Excise, who may pass such order of assessment as he thinks fit.
73. Recovery of Service Tax Not Levied or Paid or Short levied or Short-paid or
Erroneously Refunded.
(1) Where any service tax has not been levied or paid or has been short-levied or short-
paid or erroneously refunded, the Central Excise Officer may, within one year from the
relevant date, serve notice on the person chargeable with the service tax which has not
been levied or paid or which has been short-levied or short-paid or the person to whom
such tax refund has erroneously been made, requiring him to show cause why he should
not pay the amount specified in the notice:
xxx xxx xxx
(1A) Where any service tax has not been levied or paid or has been short-levied or short-
paid or erroneously refunded, by reason of fraud, collusion or any wilful mis-statement or
suppression of facts, or contravention of any of the provisions of this Chapter or the rules
made thereunder, with intent to evade payment of service tax, by such person or his
agent, to whom a notice is served under the proviso to sub-section (1) by the Central
Excise Officer, such person or agent may pay service tax in full or in part as may be
accepted by him, and the interest payable thereon under section 75 and penalty equal to
twenty-five per cent. of the service tax specified in the notice or the service tax so
accepted by such person within thirty days of the receipt of the notice.;
75. Interest on delayed payment of service tax
Every person, liable to pay the tax in accordance with the provisions of section 68 or
rules made thereunder, who fails to credit the tax or any part thereof to the account of the
Central Government within the period prescribed, shall pay simple interest at such rate
not below ten per cent and not exceeding thirty-six per cent. per annum, as is for the time
being fixed by the Central Government, by notification in the Official Gazette, for the
period by which such crediting of the tax or any part thereof is delayed."
10. The Central Government in exercise of its power conferred upon it by sub-section (1)
of Section 69 of the Finance Act, 1994 made Service Tax Rules, 1994 for the purpose of
assessment and collection of service tax. Service tax was imposed on Consultancy
Engineering Services w.e.f. 07.07.1997 by a Notification No.23 of 1997 dated
02.07.1997. Consulting Engineer as defined in Section 65(31) of the Finance Act, 1994 is
a professionally qualified or any body corporate or any other firm but that directly or
indirectly render any advice, consultancy or technical assistance in any manner to a client
in one or more disciplines of engineering.
11. Clause (g) of sub-section (105) of Section 65 of the Finance Act, 1994, as amended,
provides for the definition of taxable services rendered by a consulting engineer to mean
any service provided to a client by consulting engineer in relation to advice, consultancy
or technical service in any manner to client in one or more disciplines of engineering.
12. Sub-rule (1) of Rule 6 of Service Tax Rules, as applicable at the relevant time,
stipulated that in case of a person who was from outside India and did not have any office
in India, the service tax due on the service rendered by him should be paid by such person
or on his behalf by any other person authorized by him should submit to the
Commissioner of Central Excise in whose jurisdiction the taxable services have been
rendered by him a return containing specific details with necessary enclosures. Such
returns along with a demand draft have to be submitted within a period of 30 days from
the date of raising the bill on the client for the taxable services rendered.
13. We may furthermore notice that in terms of the proviso appended to sub-rule - (1) of
Rule 6 of Service Tax Rules, it is provided that in case of a person who was a non-
resident or was from outside India and who did not have any office in India, the service
tax due on the service rendered by him should be paid by such person or on his behalf by
another person authorized by him who should submit to the Commissioner of Central
Excise in whose jurisdiction the taxable services had been rendered, a return containing
specific details with necessary enclosures.
@page-SC801
14. The High Court has arrived at a finding of fact that the foreign company did not have
any office in India. It is not in dispute that the terms of the agreement entered into by and
between the appellant and foreign company at all material time, show that the
responsibility of meeting the service tax liability was on the service recipient and despite
the amendment of Rule 6 (1) w.e.f. 16.8.2002 agreement still held good as the service
recipient being the appellant had taken up the responsibility of meeting the liability of the
foreign company.
15. Clause 16.1 of the contract obligated the foreign company responsible only for filing
of returns, estimates, accounts, information and details complete and accurate in all
respects as may be required by any law or regulation. Only in the event the foreign
company did not comply with the said requirements resulting in imposition of any
penalty, interest or additional liability, the same shall be borne by it. Clause 16.1 did not
cast any obligation upon the foreign company to make the payment of tax; the same is
being the liability of the appellant.
16. Submissions of Mr. Iyer that the payment of interest was the statutory liability of the
service provider must be considered in the aforementioned context. If Appellant itself
was liable for payment of tax, it was also liable for payment of statutory interest
thereupon, if the same had not been deposited within the time stipulated by the statute.
The liability to pay tax was not on the foreign company. Only on default on the part of
the appellant the interest was leviable. Appellant was clearly liable therefor. In other
words, the liability being that of the appellant, it must accept the liability of payment of
interest leviable thereupon in terms of statute occasioned by the breach on its part to
deposit the amount of tax within the prescribed time.
17. Proviso appended to Rule 6 which has been inserted w.e.f. 28.2.1999 cast a liability
upon a person authorized by the foreign company to do it in that behalf. The details were
to be furnished by a person who was authorized. Clause (2) of the proviso provides for
submission of the demand draft within 30 days from the date of raising the bill. Appellant
being the person authorized to make payment of the service tax, Section 75 would come
into operation in the event of its failure to do so.
18. We may further notice that it was the appellant who had provided space and
accommodation to the personnel of M/s SNC Lavalin in their office premises and borne
expenditure related thereto. The service provider did not have any independent office.
19

. We may at this juncture notice the decisions cited by Mr. Iyer. In Laghu Udyog Bharati
and Anr. v. Union of India and Ors. [(1999) 6 SCC 418] this Court held that keeping in
view the statutory scheme as they existed in the amended rules providing for payment of
tax on the service recipient was illegal. The said provision, however, were amended with
retrospective effect. Challenge of the constitutional validity of the said amendment, came
up for consideration in Gujarat Ambuja Cements Ltd. and Anr. v. Union of India and Anr.
[(2005) 4 SCC 214] wherein a Division Bench categorically held that the basis of
reconsideration of the decisions in Laghu Udyog Bharati's case was taken away stating :
1999 AIR SCW 2771
2005 AIR SCW 2051, (Paras 23 to 30, 36, 37 and 39)

"22. As we have said, Rules 2(1)(d)(xii) and (xvii) had been held to be illegal in Laghu
Udyog Bharati only because the charging provisions of the Act provided otherwise. Now
that the charging section itself has been amended so as to make the provisions of the Act
and the Rules compatible, the criticism of the earlier law upheld by this Court can no
longer be availed of. There is thus no question of the Finance Act, 2000 overruling the
decision of this Court in Laghu Udyog Bharati as the law itself has been changed. A
legislature is competent to remove infirmities retrospectively and make any imposition of
tax declared invalid, valid. This has been the uniform approach of this Court. Such
exercise in validation must of course also be legislatively competent and legally
sustainable. Those issues are considered separately. On the first question, we hold that the
law must be taken as having always been as is now brought about by the Finance Act,
2000. The statutory foundation for the decision in Laghu Udyog Bharati has been
replaced and the decision has thereby ceased to be relevant for the purposes of construing
the present provisions [vide Ujagar Prints (II) v. Union of India]. Therefore subject to our
decision on the question of the legislative competence of Parliament to enact the law, and
assuming the amendments in 2003 to be legal for 1999 AIR SCW 2771
AIR 1989 SC 516

@page-SC802
the time being, we reject the submission of the writ petitioners that by the amendments
brought about by Sections 116 and 117 of the Finance Act, 2000, the decision in Laghu
Udyog Bharati has been legislatively overruled.

23. The next question is whether the levy of service tax on carriage of goods by transport
operators was legislatively competent. Laghu Udyog Bharati did not consider the
question of legislative competency. Before we consider the scope of the impugned Act, it
is necessary to determine the scope of the two legislative entries namely Entry 97 of List
I and Entry 56 of List II. It has been recognised in Godfrey Phillips that there is a
complete and careful demarcation of taxes in the Constitution and there is no overlapping
as far as the fields of taxation are concerned. This mutual exclusivity which has been
reflected in Article 246(1) means that taxing entries must be construed so as to maintain
exclusivity. Although generally speaking, a liberal interpretation must be given to taxing
entries, this would not bring within its purview a tax on subject-matter which a fair
reading of the entry does not cover. If in substance, the statute is not referable to a field
given to the State, the court will not by any principle of interpretation allow a statute not
covered by it to intrude upon this field. 2005 AIR SCW 613

24. Undisputedly, Chapter V of the Finance Act, 1994 was enacted with reference to the
residuary power defined in Entry 97 of List I. But as has been held in International
Tourist Corpn. v. State of Haryana : (SCC pp. 325-26, para 6-A) AIR 1981 SC 774,
(Para 7)

"Before exclusive legislative competence can be claimed for Parliament by resort to the
residuary power, the legislative incompetence of the State Legislature must be clearly
established. Entry 97 itself is specific in that a matter can be brought under that entry
only if it is not enumerated in List II or List III and in the case of a tax if it is not
mentioned in either of those lists."
25. In that case Section 3(3) of the Punjab Passengers and Goods Taxation Act, 1952 was
challenged by transport operators. The Act provided for the levy of the tax on passengers
and goods plying in the State of Haryana. According to the transport operators, the State
could not levy tax on passengers and goods carried by vehicles plying entirely along the
national highways. According to them this was solely within the power of the Centre
under Entry 23 read with Entry 97 of List I. The submission was held to be patently
fallacious by this Court. It was held that Entry 56 of List II did not exclude national
highways so that the passengers and goods carried on national highways would fall
directly and squarely within Entry 56 of List II. It was said that the State played a role in
the maintenance of the national highway and there was sufficient nexus between the tax
and passengers and goods carried on the national highway to justify the imposition.
26. The writ petitioners in this case have, relying on this judgment, argued that the Act
falls squarely within Entry 56 of List II and therefore could not be referred to Entry 97 of
List I. We do not agree.

27. There is a distinction between the object of tax, the incidence of tax and the
machinery for the collection of the tax. The distinction is important but is apt to be
confused. Legislative competence is to be determined with reference to the object of the
levy and not with reference to its incidence or machinery. There is a further distinction
between the objects of taxation in our constitutional scheme. The object of tax may be an
article or substance such as a tax on land and buildings under Entry 49 of List II, or a tax
on animals and boats under Entry 58, List II or on a taxable event such as manufacture of
goods under Entry 84 of List I, import or export of goods under Entry 83 of List I, entry
of goods under Entry 52 of List II or sale of goods under Entry 54, List II to name a few.
Theoretically, of course, as we have held in Godfrey Phillips India Ltd. v. State of U.P.
ultimately even a tax on goods will be on the taxable event of ownership or possession.
We need not go into this question except to emphasise that, broadly speaking the subject-
matter of taxation under Entry 56 of List II are goods and passengers. The phrase carried
by roads or natural waterways carves out the kind of goods or passengers which or who
can be subjected to tax under the entry. The ambit and purport of the entry has been dealt
with in Rai Ramkrishna v. State of Bihar where it was said in language which we cannot
better (SCR p. 908) : 2005 AIR SCW 613
AIR 1963 SC 1667, (Para 9)

"Entry 56 of the Second List refers to taxes on goods and passengers carried by road or
AIR 1981 SC 1480

@page-SC803
on inland waterways. It is clear that the State Legislatures are authorised to levy taxes on
goods and passengers by this entry. It is not on all goods and passengers that taxes can be
imposed under this entry; it is on goods and passengers carried by road or on inland
waterways that taxes can be imposed. The expression carried by road or on inland
waterways is an adjectival clause qualifying goods and passengers, that is to say, it is
goods and passengers of the said description that have to be taxed under this entry.
Nevertheless, it is obvious that the goods as such cannot pay taxes, and so taxes levied on
goods have to be recovered from some persons, and these persons must have an intimate
or direct connection or nexus with the goods before they can be called upon to pay the
taxes in respect of the carried goods. Similarly, passengers who are carried are taxed
under the entry. But, usually, it would be inexpedient, if not impossible, to recover the tax
directly from the passengers and so, it would be expedient and convenient to provide for
the recovery of the said tax from the owners of the vehicles themselves." (See also Sainik
Motors v. State of Rajasthan)
34. The point at which the collection of the tax is to be made is a question of legislative
convenience and part of the machinery for realisation and recovery of the tax. The
manner of the collection has been described as an accident of administration; it is not of
the essence of the duty. It will not change and does not affect the essential nature of the
tax. Subject to the legislative competence of the taxing authority a duty can be imposed at
the stage which the authority finds to be convenient and the most effective, whatever
stage it may be. The Central Government is therefore legally competent to evolve a
suitable machinery for collection of the service tax subject to the maintenance of a
rational connection between the tax and the person on whom it is imposed. By Sections
116 and 117 of the Finance Act, 2000, the tax is sought to be levied on the recipients of
the services. They cannot claim that they are not connected with the service since the
service is rendered to them.

35. In a similar fact situation under an Ordinance the Central Government was authorised
to levy and collect a duty of excise on all coal and coke dispatched from collieries. Rules
framed under the Ordinance provided for collection of the excise duty by the railway
administration by means of a surcharge on freight recoverable either from the consignor
or the consignee. The imposition of excise duty on the consignee was challenged on the
ground that the consignee had nothing to do with the manufacture or production of the
coal. Negativing this submission this Court in R. C. Jall v. Union of India, AIR at p. 1286
said : AIR 1962 SC 1281

"The argument confuses the incidence of taxation with the machinery provided for the
collection thereof."
36. In Rai Ramkrishna the tax under Entry 56 of List II was held to be competently levied
on the bus operators or bus owners even though the object of levy was passengers (which
they were not) because there was a direct connection between the object of the tax viz.
goods and passengers and the owners of the transport carrying the goods or passengers.
There is thus nothing inherently illegal or unconstitutional to provide for service tax to be
paid by the availer or user.

37. The writ petitioners have relying upon the decision in Dwarka Prasad v. Dwarka Das
Saraf contended that the amendment to Section 68 by the introduction of a proviso in
2003, was invalid. It is submitted that as the body of the section did not cover the subject-
matter, there was no question of creating an exception in respect thereto by a proviso.
According to the writ petitioners, the proviso cannot expand the body by creating a
separate charge. It is submitted that by merely amending the definition of the word
assessee it could not be understood to mean that thereby all customers of the services in
question were liable. 1975 SC 1758

38. The submission is misconceived for several reasons. Section 68 is a machinery


section in that it provides for the incidence of taxation and is not the charging section
which is Section 66. The amendments to Section 66 brought about in 2000 changed the
point of collection of tax from the provider of the service to such manner as may be
prescribed. Section 68(1-A) as it stood in 1997 provided for the collection and recovery
of service tax in respect of the services referred in sub-clauses (g) to (r) of Section
65(41), which included both the services with which we are concerned, from such person
and in such manner as may be prescribed. The 1998 Finance Act maintained this. Now
the Service Tax Rules, 1994
@page-SC804
provided for the collection and recovery of tax from the users or payers for the services.
This was the prescribed method. All that the proviso to Section 68(1-A) did was to
prescribe the procedure for collection with reference to services of goods transport
operators and clearing agents which services had already been expressly included under
the Finance Act, 2000 in the definition of taxable service."
20. Reliance placed by Mr. Iyer on Commissioner of Central Excise, Meerut II v. L.H.
Sugar Factories Ltd. and Ors. [(2005) 13 SCC 245] is also not of much assistance as the
decision was rendered in relation to the provisions of Income-tax Act holding that the
said Act also must be construed having regard to the charging provision.
21. We, therefore, are of the opinion that no case has been made out for interference with
the impugned judgment.
22. The appeal is dismissed with costs. Counsel's fee assessed at Rs.25,000/- (Rupees
twenty five thousand only).
Appeal dismissed.
AIR 2008 SUPREME COURT 804 "Ajay Mohan v. H. N. Rai"
(From : Bombay)
Coram : 2 S. B. SINHA AND H. S. BEDI, JJ.
Civil Appeal No. 5831 of 2007 (arising out of SLP (C) No. 13789 of 2007), D/- 12 -12
-2007
Ajay Mohan and Ors. v. H. N. Rai and Ors.
(A) Civil P.C. (5 of 1908), O.23, R.1 - WITHDRAWAL OF SUIT - APPEAL -
Withdrawal of appeal - Grant of permission - Court becomes functus officio thereafter -
Cannot grant further relief.
In the instant case, the order of the City Civil Court may be bad but then it was required
to be set aside by the Court of Appeal. An appeal had been preferred by the appellants
thereagainst but the same had been withdrawn. The said order of City Civil Court,
therefore, attained finality. The High Court, while allowing the appellant to withdraw the
appeal, no doubt, passed an order of status quo for a period of two weeks in terms of its
order but no reason therefor had been assigned. It ex facie had no jurisdiction to pass
such an interim order. Once the appeal was permitted to be withdrawn, the Court became
functus officio. It did not hear the parties on merit. It had not assigned any reason in
support thereof. Ordinarily, a Court, while allowing a party to withdraw an appeal, could
not have granted a further relief.
(2005) 11 SCC 509, Followed.
(Para 17)
(B) Civil P.C. (5 of 1908), O.39, R.1 - INJUNCTION - SUPREME COURT -
AMENDMENT - PLAINT - Application for injunction - Maintainability - Suit for
permanent injunction against dispossession - Application for interim injunction filed
therein - Order rejecting it - Becoming final on withdrawal of appeal against it - No
liberty granted to plaintiffs to file a fresh application for injunction - In subsequent
proceedings Supreme Court while permitting amendment of plaint directing defendants
not to create third party rights till disposal of chamber summons - Disposal of chamber
summons - Subsequent application for injunction - Rejection of - No infirmity - Only
because further prayer had been made in suit upon amending plaint, the same by itself did
not bring about situational change warranting application of mind afresh by Court.(Paras
19, 20, 21)
Cases Referred : Chronological Paras
(2007) SLP (C) No. 1218 of 2007, D/- 2-2-2007 (SC) (Ref.)11
2005 AIR SCW 270 : AIR 2005 SC 626 (Ref.) 15
2005 AIR SCW 3578 : AIR 2005 SC 3165 (Ref.) 15
(2005) 11 SCC 509 (Foll.) 17
2002 AIR SCW 5315 : AIR 2003 SC 649 (Ref.) 15
AIR 1964 SC 993 (Ref.) 15
AIR 1960 SC 941 (Ref.) 15
R. F. Nariman and Shyam Dewan, Sr. Advocates; Parimal Shroff, Mahesh Agarwal, Rishi
Agrawala, E. C. Agrawala, Gaurav Goel and Surya Kant Jadhav, for Appellants; Ashok
Desai and Mukul Rohtagi, Sr. Advocates, P. H. Parekh, Pradip Sancheti, Sameer Parekh,
Ajay K. Jha, Ms. N. Don, Parthiv Goswami, Kush Chaturvedi and Ms. Diksha Raj (for P.
H. Parekh and Co.), for Respondents.
Judgement
S. B. SINHA, J. :- Leave granted.
2. Appellants are aggrieved by and dissatisfied with the judgment and order dated
16.6.2007 passed in Appeal From Order No.320 of 2007 by a Division Bench of the
Bombay High Court whereby and whereunder an appeal from an order dated 12.4.2007
passed by City Civil Court, Bombay in
@page-SC805
Notice of Motion No.944 of 2007 rejecting an application for injunction filed by them
was dismissed.
3. Appellants are said to have become owners of the suit land by reason of a deed of gift,
which is said to have been executed by Mrs. Tara Sarup on 30.3.1968 in favour of the
first appellant. Indisputably, Respondents claim their right, title, interest and possession
on or over the land in suit in terms of an agreement of sale purported to have been
executed by the appellants herein in their favour on or about 23.10.1969.
4. Appellants' case in relation to the said agreement for sale are :
(a) It is a forged document.
(b) In any event, the plaintiff No.1 being minor on the date of execution of the agreement
(his date of birth being 8.3.1952), the same is void in law.
5. The claim of the respondents, on the other hand, is that out of the amount of
consideration mentioned in the said agreement, namely Rs.90,000/-, a sum of Rs.80,000/-
has already been paid and they were put in possession thereover in part performance
thereof, as envisaged under Section 53A of Transfer of Property Act.
6. Various proceedings appear to have initiated before the Revenue Courts in regard to
inclusion of the name of the respondents in the Revenue Records. It is further accepted
that the first appellant herein had executed three deeds of assignment in favour of the
second appellant herein on or about 29.6.1991.
A suit was filed by the appellants before the City Civil Court, Bombay which was marked
as Suit No. 4962 of 2006 claiming, inter alia, for a decree for permanent injunction
restraining the respondents from creating any right in or over the suit land on the basis of
revenue entries as also for a decree for permanent injunction restraining them from
interfering with their possession and occupation thereupon.
In the said suit, the appellants took out a notice of motion marked as Notice of Motion
No. 3551 of 2006 and by order dated 13.10.2006, learned Judge, City Civil Court,
Bombay refused to grant an order of injunction, inter alia, holding :
(i) The contentions advanced by the defendants are of much substance inasmuch as in
view of the execution of the agreement for sale, the onus was upon the plaintiffs to get
the said documents cancelled and treated as null and void. Such a prayer having not been
made, mere relief for injunction prayed for by the plaintiffs cannot give rise to existence
of prima facie case for grant of relief at the interlocutory stage.
(ii) The cardinal rule being that possession follows title, the plaintiff proceeded under the
assumption that he had assigned the suit property to plaintiff No.2 who is a builder and
developer and that plaintiff No.2 and plaintiff No.3 are said to be protecting the property.
(iii) The alleged threat of dispossession given by the defendants to the plaintiffs being
towards the end of May 2006, no details thereabout had been stated in the plaint and in
that view of the matter also the plaintiffs had failed to make out a prima facie case.
(iv) After a report was prepared by the Revenue Officer, allegedly the defendants were
found to be in possession. As the plaintiffs had not challenged the agreement of sale dated
23.10.1969 whereunder only the defendants had been claiming their right, validity thereof
or otherwise would be pre-judging the case at that stage.
It was also found that the plaintiffs had not approached the court with clean hands.
7. Appellants thereafter filed an application for amendment of plaint. They also preferred
an appeal against the said order dated 13.10.2006 in the High Court of Judicature at
Bombay. The said appeal, however, was withdrawn stating that they would move the trial
court for amendment of the plaint. While allowing the said prayer, an observation was
made that the trial court shall consider the question in regard to the amendment of plaint
without in any way being influenced by the observations made by the learned trial Judge
in the impugned order. Although, the High Court allowed the appellants to withdraw the
appeal, it directed the parties to maintain status quo for a period of two weeks.
8. A chamber summons thereafter was taken by the appellants on or about 1.12.2006
wherein not only amendment of the plaint was prayed for but an interim order of
injunction during the pendency of the said application was also prayed for. We may
notice the amendments sought for by the appellants in the said Notice of Motion :
"(a)(i) That it may be declared that the
@page-SC806
Defendants or any of them have no right, title or interest of any nature in respect of the
plots of land bearing at C.T.S. Nos.6A and 7/1A of village Powai, Taluka Kurla
admeasuring about 37,673 sq. mtrs. and C.T.S. Nos. 20 and 22 of village Tirandaz, Taluka
Kurla admeasuring about 27,582 sq. mtrs. or any part/s thereof by virtue of the alleged
Agreement for Sale dated 23rd October, 1969, being Exhibit "A10" hereto or otherwise
or at all;
(a)(ii) that the Defendants, their servants and agents may be permanently restrained by
an Order and injunction of this Hon'ble Court from claiming any right, title or interest of
any nature in respect of the plots of land bearing at C.T.S. Nos.A and 7/1A of village
Powai, Taluka Kurla admeasuring about 37,673 sq. mtrs. and C.T.S. Nos. 20 and 22 of
village Tirandaz, Taluka Kurla admeasuring about 27,582 sq. mtrs. or any part/s thereof
by virtue of the said alleged Agreement for Sale dated 23rd October, 1969 being Exhibit
"A-10" hereto or otherwise or at all.
11. Add in the prayer (a) in the Plaint after the words 'pass an order of injunction' add
"permanently".
12. Add in prayer (a) after the words 'Taluka: Kurla bearing ..' delete the words "CTS
No.22 (Approx.) admeasuring 18,083 sq. mts.' And instead add the following - :
"CTS Nos.20 and 22 admeasuring 27,582 sq. mts. and at village Powai Taluka Kurla
bearing CTS Nos. 6A and 7-1A admeasuring 37,673 sq. mts. And".
13. Add in the prayer (b) in the Plaint after the words 'pass an order of injunction' add
"permanently".
14. (i) Add in prayer (b) after the words 'Taluka : Kurla bearing ............' delete the words
"CTS No.22 (Approx.) admeasuring 18,083 sq. mts.' And instead add the following :
"CTS Nos. 20 and 22 admeasuring 27,582 sq. mts and at village Powai Taluka Kurla
bearing CTS Nos.6A and 7-1A admeasuring 37,673 sq. mts. And".
(ii) Add in the Fourth line of prayer (b) after the words land occupation of the plaintiffs'
the words "Nos.2 and 3".
15. In prayer clause (c) after the words prayer clauses add '(a)(i)'."
9. Prayer for interim relief was rejected by the learned judge, City Civil Court opining
that the earlier order dated 13.10.2006 became final.
10. Against the said order, the appellants again approached the High Court and by an
order dated 10.1.2007, a learned Single Judge noticing that the proposed amendment fell
short of relief of declaration that the suit agreement was null and void and to be set aside,
came to the conclusion - :
"This indicates that the Appellants were conscious that such relief will have to be pressed
in respect of the suit documents. Obviously, that perception is on account of the fact that
the said documents were made subject-matter of proceedings before the Revenue
Authorities indeed, the Respondents have stated on affidavit that the Original copy of the
said document has been lost in respect of which police complaint is already instituted. In
such a case, however, it is possible for the Respondents to establish the fact of existence
of such Agreement by relying on secondary evidence on fulfilling the required norms in
that behalf. Be that as it may, prima facie, it is seen from the record that the execution of
the suit documents has been disputed by the Appellants as back as in 1984, which stand
has been dealt with by the Authorities. Suffice it to observe that the Appellants would
succeed only if they were to challenge the subject Agreement, inasmuch as the
Defendants were asserting rights in respect of the suit land on the basis of the said
Agreement. The fact that the Original copy of the said Agreement is not in existence does
not alter the situation so as to absolve the Appellants from claiming relief that the said
Agreement is null and void and to set it aside."
On the said findings, the judgment and order of the City Civil Court was upheld.
11. A Special Leave Application was filed before this Court against the said order which
was marked as SLP (C) No.1218 of 2007. The same was disposed of by an order dated
2.2.2007, stating :
"Counsel for the respondent-defendants, on instructions, states that the defendants have
no intention to create third party rights till the disposal of the amendment application
filed by the petitioners before the concerned City Civil Court, which is coming up before
the said Court for disposal on 7th February, 2007. We direct the concerned Court to
dispose of the Chamber Summons on 7th February, 2007 and till then, as stated on
@page-SC807
behalf of the counsel for the respondents-defendants, no third party interest shall be
created. The Chamber Summons shall be decided uninfluenced by any observations made
by the High Court in the impugned order.
Counsel for the Petitioners submits that the prayers made in the Chamber Summons are
for amendment of the plaint as well as for interim reliefs.
The Court will consider all the reliefs prayed for in the Chamber Summons and pass
appropriate orders.
The Special Leave Petition is disposed of accordingly."
12. By an order dated 28.2.2007, the application for amendment was allowed. Keeping in
view the fact that the plaintiffs' prayer for grant of interim injunction was confined in the
earlier notice of motion till the disposal thereof, the plaintiff did not press for the second
prayer expressing his desire to take out a separate notice of motion.
A notice of motion for grant of injunction was again taken out which was dismissed by
reason of an order dated 12.3.2007 by the learned Judge, City Civil Court. In regard to
the order of this Court dated 2.2.2007, the learned Judge observed :
"I have read and re-read the order of the Supreme Court. The Apex Court has said that
this Court will consider all the reliefs prayed for in the Chamber Summons and pass
appropriate order. The word "interim" is defined in Black's Law Dictionary as "in the
meantime", "temporary" and "occurring in intervening time". The relevant meaning here
appears to be "occurring in intervening time". Therefore, I am of the view that the Apex
Court has referred to the period till the hearing and disposal of the Chamber Summons.
Now the chamber summon, is disposed of. It is allowed and all amendments are
incorporated in the plaint. Therefore, there is no stage, which is "occurring in intervening
time". In this view of the matter, I do not find any substance in this notice of motion. I,
therefore, pass the following order :
ORDER
Notice of motion stands dismissed. No order as to cost.
The same may be registered for statistical purpose."
By reason of the impugned judgment, the High Court has upheld the said order.
13. Mr. R.F. Nariman, learned senior counsel appearing on behalf of the appellant, inter
alia, would submit that the appellants have never been heard on merit of the matter. The
learned counsel argued that keeping in view the nature and purport of the order of this
Court dated 2.2.2007, the City Civil Court could not have relied upon its earlier order.
Consequently, the High Court had also committed a manifest error in applying the
principles of res judicata which have no application in the instant case. It was contended
that the purported finding of the learned Judge, City Civil Court to the effect that the
defendants had been found to be in actual physical possession of the suit property on the
date of institution of the suit was clearly erroneous inasmuch as no such finding had been
arrived at by the said court while passing its order dated 13.10.2006. It was urged that the
report of an officer appointed by the Revenue Minister found the appellant to be in
possession of the property and in that view of the matter, it was not necessary for them to
pray for a decree for cancellation and setting aside of the agreement for sale dated
23.10.1969. In any event, having regard to the observations made by the High Court, it
was obligatory on the part of the courts below to consider the merit of the matter afresh.
14. Mr. Ashok Desai, learned senior counsel appearing on behalf of the respondents, on
the other hand, contended that the learned City Civil Court rejected the application for
grant of injunction in favour of the appellants, inter alia, holding that - :
(a) Plaintiffs do not have any prima facie case;
(b) They had not approached the Court with clean hands;
(c) The delay in questioning the validity of the said agreement of sale disentitles the
plaintiffs from obtaining the order of injunction and they had not challenged the validity
of the said agreement in the suit.
It was contended that the very fact that the appellants had withdrawn the appeal without
reserving their liberty to move the Trial Judge again for injunction would clearly attract
the principles of res judicata, Mr. Desai would submit that although an opportunity had
been granted to the appellants to pray for a relief of cancellation of
@page-SC808
the said deed of sale, the same was not prayed for which would demonstrate speculative
nature of the litigation resorted to by the plaintiffs. A party to a suit, undoubtedly, may
file an application for injunction if a change in the situation has been brought about but
there being no said change, it was urged, the second application for injunction would not
be maintainable.
15

. It is a trite law that the principles of res judicata apply in different stages of the same
proceedings. [See Satyadhyan Ghosal and Ors. v. Smt. Deorajin Debi and Anr. [AIR 1960
SC 941], Arjun Singh v. Mohindra Kumar and Ors. [AIR 1964 SC 993] and C.V.
Rajendran and Anr. v. N.M. Muhammed Kunhi [(2002) 7 SCC 447], Ishwar Dutt v. Land
Acquisition Collector and Anr.[(2005)7 SCC 190] and Bhanu Kumar Jain v. Archana
Kumar and Anr. [(2005) 1 SCC 787]. 2002 AIR SCW 5315
2005 AIR SCW 3578
2005 AIR SCW 270

The entire claim of the plaintiff was based on their claim of possession of the lands in
suit. Defendants, on the other hand, claimed their right, title, interest and possession on
the basis of the purported agreement for sale. Whether possession had been delivered to
them in part performance of agreement of sale or not is essentially a question of fact.
Genuineness or otherwise of the said agreement also involves determination of a disputed
question.
16. Plaintiffs, while praying for the relief of interim injunction, were bound to establish a
prima facie case. They were also bound to show that the balance of convenience lay in
their favour and unless the prayer is granted, they will suffer an irreparable injury.
The learned Judge, City Civil Court clearly found that prima facie, the plaintiffs' suit was
not maintainable in absence of any prayer for cancellation and setting aside of the said
agreement for sale having been made for in the suit.
Appellants although had been contending that such a relief was not necessary as it was
merely a defence of the respondents, why they did not raise such a question in the
original suit is a matter of guess. We do not know as to why the plaintiffs, despite
opportunities having been given to them, failed to make such a prayer even while seeking
the Court's 'leave' to amend the plaint.
17. The order of the City Civil Court dated 13.10.2006 may be bad but then it was
required to be set aside by the Court of Appeal. An appeal had been preferred by the
appellants thereagainst but the same had been withdrawn. The said order dated
13.10.2006, therefore, attained finality. The High Court, while allowing the appellant to
withdraw the appeal, no doubt, passed an order of status quo for a period of two weeks in
terms of its order dated 23.11.2006 but no reason therefor had been assigned. It ex facie
had no jurisdiction to pass such an interim order. Once the appeal was permitted to be
withdrawn, the Court became functus officio. It did not hear the parties on merit. It had
not assigned any reason in support thereof. Ordinarily, a court, while allowing a party to
withdraw an appeal, could not have granted a further relief. [See G.E. Power Controls
India and Ors. v. S. Lakshmipathy and Ors. [(2005) 11 SCC 509].
18. Even then, the plaintiff preferred to file a fresh notice of motion. It did not file any
application for grant of injunction till the disposal of the suit. It, principally, in the said
notice of motion asked for amendment of the plaint. The second relief prayed for in the
said notice of motion was again withdrawn with liberty to file a fresh notice of motion.
Appellants, therefore, have been filing applications after applications without making
proper prayer therein at all stages.
19. So far as the order of this Court dated 2.2.2007 is concerned at the first blush, it
appears that this Court could not have granted any relief to reagitate the questions of
hearing the parties and interim relief once over again. Even if that be so, the said interim
relief having regard to the admitted facts was to be kept confined only for a short term,
namely, till the application for amendment is considered. This Court, therefore, did not
grant any liberty to the plaintiffs to file a fresh application for injunction. It could not
comprehend thereabout at that time. The Notice of Motion taken out for grant of
injunction was, therefore, required to be considered on its own merit. The plaintiffs had
not brought out any new circumstances warranting grant of any injunction in their favour.
Only because a further prayer had been made in the suit upon amending the plaint, the
same by itself did not bring about a situational change warranting application of mind
afresh by the learned Judge, City Civil Court. The only argument which is
@page-SC809
available to the appellants was that the suit, by reason of amendment made in the prayer,
has become maintainable. Maintainability of the suit itself does not give rise to a triable
issue. The issues which arose for consideration in the suit are the ones we would have
noticed hereinbefore, namely, inter alia, the validity of the agreement for sale and/or grant
of possession in favour of the defendants/respondents. How, by sheer amendment of the
plaint, the plaintiff could prove a prima facie case or show existence of a balance of
convenience in their favour, has not been demonstrated.
20. We are, therefore, of the opinion that although learned Judge, High Court, while
passing its order dated 13.10.2006 could have considered the merit of the application
filed by the appellant in regard to the relief for injunction, the same by itself, in our
opinion, did not warrant a direction to consider the matter afresh by the learned Judge,
City Civil Court.
We are, therefore, are of the opinion that the impugned judgment do not suffer from any
infirmity. We would, however, having regard to the peculiar facts and circumstances of
the case, request the learned Judge, City Civil Court to consider the desirability of
disposing of the suit as expeditiously as possible preferably within a period of six weeks
from the date of communication of this order. The parties are directed to render all
cooperation to the learned Judge in early disposal of the suit. If it is convenient to the
learned Judge, the hearing of the suit may be taken up on day to day basis.
21. This appeal is dismissed with costs. Counsel's fee quantified at Rs.25,000/- (Rupees
twenty five thousand only).
Appeal dismissed.
@page-SC809
AIR 2008 SUPREME COURT 809 "Eastern Book Company v. D. B. Modak"
(From : 2002 (101) DLT 205)
Coram : 2 B. N. AGRAWAL AND PRAKASH PRABHAKAR NAOLEKAR, JJ.
Civil Appeal No. 6472 of 2004, D/- 12 -12 -2007.
Eastern Book Company and Ors. v. D. B. Modak and Anr.
(A) Copyright Act (14 of 1957), S.13, S.52(1)(q)(iv) - COPYRIGHT - Copyright - Law
Journal - Copy-edited judgments - Claims for copyright - Merely establishing amount of
skill, labour and capital put in production of copy-edited judgment - Not sufficient to
claim copyright - Innovative thoughts and creativity necessary to claim copyright -
Derivative work produced must be some- thing different than original.
The judicial pronouncements of the Apex Court would be in the public domain and its
reproduction or publication would not infringe the copyright. That being the position, the
copy-edited judgments would not satisfy the copyright merely by establishing amount of
skill, labour and capital put in the inputs of the copy-edited judgments and the original or
innovative thoughts for the creativity are completely excluded. Accordingly, original or
innovative thoughts are necessary to establish copyright in the author's work. The
principle where there is common source the person relying on it must prove that he
actually went to the common source from where he borrowed the material, employing his
own skill, labour and brain and he did not copy, would not apply to the judgments of the
Courts because there is no copyright in the judgments of the Court, unless so made by the
Court itself. To secure a copyright for the judgments delivered by the Court, it is
necessary that the labour, skill and capital invested should be sufficient to communicate
or impart to the judgment printed in Law Journal some quality or character which the
original judgment does not possess and which differentiates the original judgment from
the printed one. The Copyright Act is not concerned with the original idea but with the
expression of thought. Copyright has nothing to do with originality or literary merit.
Copyrighted material is that what is created by the author by his own skill, labour and
investment of capital, may be it is a derivative work which gives a flavour of creativity.
The copyright work which comes into being should be original in the sense that by virtue
of selection, co-ordination or arrangement of pre-existing data contained in the work a
work somewhat different in character is produced by the author. To claim copyright in a
compilation, the author must produce the material with exercise of his skill and judgment
which may not be creativity in the sense that it is novel or non-obvious, but at the same
time it is not a product of merely labour and capital. The derivative work produced by the
author must have some distinguishable features and flavour to raw text of the
@page-SC810
judgments delivered by the Court. The tribial variation or inputs put in the judgment
would not satisfy the test of copyright of an author.
2004 (1) SCR 339 (Canada), Foll. (Para 38)
Adding, in the copyedited judgment cross-citations to the citation(s) already given in the
original text; adding names of cases and cross-citations where only the citation of the
case is given; adding citation and cross-citations where only name of the case is given;
inserting citation in case history where only the title and year of the impugned/earlier
order is given; presenting in their own style the cases when they are cited, repeated in the
judgment; providing previous references to the quoted matter in the judgment by giving
exact page and paragraph number as in the original case source /treatise /reference
material; adding margin headings to quoted extracts from statutes/ rules, etc., when they
are missing from the original text of the judgment; adding the number of the
Section/Rule/Article/paragraph to the extract quoted in the original text; adding the
names of Judges on whose behalf opinion is given; adding ellipsis "......" to indicate
breaks in quoted extract; supplying the matter inadvertently missed in quoted extracts in
the original text of the judgment; changing the text as per corrigenda issued, etc. etc.,
does not give the Law Journal copyright in the copy-edited judgment. (Paras 39, 40)
(B) Copyright Act (14 of 1957), S.13, S.52(1)(q)(iv) - COPYRIGHT - LAW REPORTS -
JUDGMENT - Copyright - Law Journal - Copy-edited judgment - Adding inputs by
segregating existing paragraphs in original text by breaking them into separate
paragraphs; adding internal paragraph numbering within a judgment after providing
uniform paragraph numbering to multiple judgments; and indicating in judgment the
Judges who have dissented or concurred - Require skill and judgment in great measure -
Such exercise and creation thereof has a flavour of minimum amount of creativity - Law
Journals have copyright in such part. (Para 42)
Cases Referred : Chronological Paras
2004 (1) SCR 339 (Canada) (Foll) 37
(2000) WLR 2416 (HL) 19
AIR 1991 SC 1473 14
AIR 1990 SC 2286 14
AIR 1988 SC 526 14
AIR 1986 SC 1999 14
AIR 1985 SC 1495 7, 18
AIR 1983 Raj 100 7
AIR 1982 MP 1 7
1982 PTC 85 28
AIR 1981 Guj 15 7
AIR 1978 SC 215 14
AIR 1978 SC 1296 : 1978 Cri LJ 1281 14
1976 (3) All ER 184 14
AIR 1975 SC 563 14
AIR 1975 SC 460 14
AIR 1974 SC 497 14
AIR 1973 SC 974 14
AIR 1973 SC 930 : 1973 Lab IC 565 7
AIR 1969 SC 1 7, 18
1969 (2) SCR 375 14
AIR 1968 SC 1379 18
AIR 1967 All 91 26
1964 (1)) WLR 273 (HL) 16
AIR 1961 Madras 111 25
AIR 1955 SC 287 7
AIR 1955 Mad 391 24
1952 ALJ 3322 18
AIR 1948 PC 82 7
AIR 1938 FC 43 7
AIR 1938 All 266 23
AIR 1924 PC 75 36
(1916)2 Ch 601 20, 36
(1900) AC 539 (HL) 18
ILR 13 Bom 358 27
(1901-1904) Cop Cas 49 22
(1866)1 R 1 Eq 697 21
158 F 3d 674 34
18 USPQ 2d 1275 33
IR 18 Eq 444 29
945 F 2d 509 35
ILR 17 Cal 952 29
Raju Ramachandran, Sr. Advocate, Sudeep Mallik, Ms. Anitha Shenoy, Saurab Sinha,
Rishad Ahmad Chowdhury, Raj Shekhar Rao, M. R. Vij, Nitin Ramesh, Samar Bansal, S.
K. Mohanty, Lokesh Kumar, M. K. Garg, Ms. Pratibha M. Singh, Gaurav Sharma,
Bishwajit Dubey, Ms. Shruti Kakker, Sumeet Bhatia and Maninder Singh, Advocates
with him for the Appearing Parties.
Judgement
1. P. P. NAOLEKAR, J. :-These appeals by special leave have been preferred against the
common judgment of a Division Bench of the High Court of Delhi involving the
analogous question and are, therefore, decided together by this judgment.
2. Appellant No. 1 Eastern Book Company is a registered partnership firm carrying on the
business of publishing law books.
@page-SC811
Appellant No. 2 EBC Publishing Pvt. Ltd. is a company incorporated and existing under
the Companies Act, 1956. The said appellants are involved in the printing and publishing
of various books relating to the field of law. One of the well-known publications of
appellant No. 1 Eastern Book Company is the law report Supreme Court Cases
(hereinafter called SCC). The appellant publishes all reportable judgments along with
non-reportable judgments of the Supreme Court of India. Yet another category included
in SCC is short judgments, orders, practice directions and record of proceedings. The law
report SCC was commenced in the year 1969 and has been in continuous publication ever
since. The name Supreme Court Cases has been coined by the appellants and they have
been using the same continuously, exclusively and extensively in relation to the law
reports published by them. For the purpose of publishing the judgments, orders and
proceedings of the Supreme Court, the copies of judgments, orders and proceedings are
procured from the office of the Registrar of the Supreme Court of India. After the initial
procurement of the judgments, orders and proceedings for publication, the appellants
make copy-editing wherein the judgments, orders and record of proceedings procured,
which is the raw source, are copy-edited by a team of assistant staff and various inputs
are put in the judgments and orders to make them user friendly by making an addition of
cross-references, standardization or formatting of the text, paragraph numbering,
verification and by putting other inputs. The appellants also prepare the headnotes
comprising of two portions, the short note consisting of catch/lead words written in bold;
and the long note, which is comprised of a brief discussion of the facts and the relevant
extracts from the judgments and orders of the Court. Headnotes are prepared by appellant
No. 3-Surendra Malik. As per the said appellant (plaintiff No. 3 in the suits filed in the
Delhi High Court), the preparation of the headnotes and putting the various inputs in the
raw text of the judgments and orders received from the Supreme Court Registry require
considerable amount of skill, labour and expertise and for the said work a substantial
amount of capital expenditure on the infrastructure, such as office, equipment, computers
and for maintaining extensive library, besides recurring expenditure on both the
management of human resources and infrastructural maintenance, is made by the
plaintiff-appellants. As per the appellants, SCC is a law report which carries case reports
comprising of the appellants version or presentation of those judgments and orders of the
Supreme Court after putting various inputs in the raw text and it constitutes an 'original
literary work' of the appellants in which copyright subsists under Section 13 of the
Copyright Act, 1957 (hereinafter referred to as the Act) and thus the appellants alone
have the exclusive right to make printed as well as electronic copies of the same under
Section 14 of the Act. Any scanning or copying or reproduction done of or from the
reports or pages or paragraphs or portions of any volume of SCC by any other person, is
an infringement of the copyright in SCC within the meaning of Section 51 of the Act.
3. The defendant-respondent No. 2 Spectrum Business Support Ltd. (in Civil Appeal No.
6472/2004) has brought out a software called "Grand Jurix" published on CD-ROMs and
the defendant-respondent No. 2 Regent Data Tech Pvt. Ltd. (in Civil Appeal No.
6905/2004) has brought out software package called "The Laws" published on CD-
ROMs. As per the appellants, all the modules in the defendant-respondents' software
packages have been lifted verbatim from the appellants' work; the respondents have
copied the appellants' sequencing, selection and arrangement of the cases coupled with
the entire text of copy-edited judgments as published in the plaintiff-appellants' law
report SCC, along with and including the style and formatting, the copy-editing
paragraph numbers, footnote numbers, cross-references, etc.; and such acts of the
defendant-respondents constitute infringement of the plaintiff-appellants' exclusive right
to the same.
4. The plaintiff-appellants herein moved the Court for temporary injunction by filing
applications in Suit No.758/2000 against Spectrum Business Support Ltd. and in Suit No.
624/2000 against Regent Data Tech Pvt. Ltd. before a learned Single Judge of the High
Court of Delhi. The interim orders of injunction were passed in the suits from time to
time. However, the defendant-respondents filed application for vacation of the stay order.
By a common judgment dated 17.1.2001, the Single Judge of the High Court dismissed
the appellants' applications for interim injunction and allowed the
@page-SC812
respondents' application for vacation of stay. However, before the Single Judge, the
respondents conceded that the appellants have copyright in the headnotes and as such
they undertook not to copy these headnotes in their CD-ROMs.
5. Aggrieved by the said order dated 17.1.2001 refusing to grant interim injunction, the
appellants preferred appeals before a Division Bench of the Delhi High Court and the
applications praying for interim relief were also filed in both the appeals. The
applications praying for the interim relief were disposed of by the Division Bench on
9.3.2001 directing that during the pendency of the appeals the respondents will be
entitled to sell their CD-ROMs with the text of the judgment of the Supreme Court along
with their own headnotes which should not in any way be a copy of the headnotes and the
text of the plaintiff-appellants.
6. The Division Bench of the Delhi High Court heard the matters finally and has held that
the appellants are not right in submitting that although the respondents have a right to
publish the raw judgments they could do so only after obtaining the same from the
original source, i.e. after obtaining certified copy of the judgment. The Division Bench
did not agree with the submission of the appellants that by making certain corrections in
the judgments or putting paragraph numbers or arranging the said judgments in a
particular manner while printing, the appellants can claim that the copy-edited judgments
become their 'original literary work'. If the right of a person like the appellants who are
merely reporting the judgments of the courts is stretched to this extent, then after a
judgment is reported by a particular journal, others would be barred from doing the same
and the very purpose of making these judgments in public domain, therefore, would be
frustrated. The Court has further held that the appellants are not the author of the
Supreme Court judgments and by merely making certain corrections therein or giving
paragraph numbers, the character of a judgment does not change and it does not become
materially different from the original judgment. Once a person has a right to obtain
certified copy of the judgment from the Registry of the Court and to publish it, it cannot
be said that he has no right to take text of the judgment from the journal where it is
already reported. The act of reproduction of any judgment or order of the Court, Tribunal
or any other judicial authority under Section 52(1)(q) of the Act, is not an infringement of
the copyright. Any person can, therefore, publish judgments of the Courts. The appellants
may have happened to have first published the judgments, but the same will not mean
that they can have a copyright therein. It is the considered opinion of the Division Bench
that no person can claim copyright in the text of the judgment by merely putting certain
inputs to make it user friendly. The appellants cannot claim copyright in the judgment of
the Court. But it has been held by the Court that reading the judgment and searching the
important portions thereof and collecting sentences from various places for the purposes
of making headnotes would involve labour and skill; and that there is originality and
creativity in preparation of the headnotes, but not when they are verbatim extracts from
the judgment and, therefore, there would be copyright in the headnotes to the judgments
prepared by the appellants. So far as footnotes and editorial notes are concerned, it cannot
be denied that these are the publisher's own creations and based on publisher's own
research and thus will have a copyright of the appellants. The Division Bench modified
the judgment of the Single Judge by directing the respondents that they shall be entitled
to sell their CD-ROMs with the text of the judgments of the Supreme Court along with
their own headnotes, editorial notes, if any, which should not in any way be copy of the
headnotes of the appellants. The respondents shall also not copy the footnotes and
editorial notes appearing in the journal of the appellants. Thus, the Court has not accepted
the case of the appellants that they have a copyright in the copy-edited judgments of the
Supreme Court. Aggrieved by the decision of the Division Bench of Delhi High Court,
the appellants have filed these appeals by special leave.
7. The appellants have claimed that the copyright subsists in SCC as a law report as a
whole based cumulatively and compendiously on all the substantial contributions of skill,
labour and capital in the creation of various parts of SCC, i.e., headnotes, editorial notes,
footnotes, the version of the copy-edited text of judgments as published in the appellants'
law report SCC, the selection of cases as published in SCC, the sequence
@page-SC813
and arrangement of cases as published in SCC and the index, table of cases, etc. which
are published in each volume of SCC, that give it the SCC volumes and thereby complete
SCC set, its character as a work as a whole. The appellants claim that the copyright
subsists in the copy-edited version. The appellants do not claim copyright in the raw text
of the judgments, certified copies of which are obtained from the Registry. The appellants
do not claim a monopoly in publishing judgments of the Supreme Court as they are being
published by other publishers also without copying from each other publication. The
appellants claim that their copyright is in the copy-edited version of the text of judgments
as published in SCC which is a creation of the appellants' skill, labour and capital and
there are contributions/inputs/ additions of the appellants in creating their version of the
text of judgments as published in SCC. The appellants placed before us the following
contributions, inputs and additions made by them to the text in the certified copies of the
judgments received by them from the Registry. The appellants assert that originality
inheres in the following aspects of its editorial process which are selected, coordinated
and arranged in such a way that the resulting work as a whole constitutes an original
work of the appellants.
MATTER ADDED PER SE TO THE RAW TEXT OF THE JUDGMENTS
1. Cross-citations are added to the citations(s) already given in the original text
For example,
a. SCC/AIR/LLJ citations added in addition to the SCR citation given in the text and
cross-citations separated by ?:?

Raw text obtained from Registry : SCC Page : Corresponding citations from SCC
Page :

R. Chitralakha and Anr. v. State of Mysore and Ors. 1964 (6) SCR 368 at 388 and Triloki
Nath v. J. and K State 1969 (1) SCR 103 at 105 and K. C. Vasanth Kumar v. Karnataka
1985 Supp (1) SCR 352 R. Chitralakha v. State of Mysore and Triloki Nath v. State
of J. and K.(II) and K. C. Vasanth Kumar v. State of Karnataka. (1964) 6 SCR 368,
388 : AIR 1964 SC 1823
1969) 1 SCR 103, 105 : AIR 1969 SC 1 : (1970) 1LLJ 629
1985 Supp SCC 714 : 1985 Supp 1 SCR 352
b. FCR, IA, Bom LR citations added in addition to the AIR citation given in raw text and
cross-citations separated by ? : ?

Raw text obtained from Registry : SCC Page : Corresponding citations from SCC
Page :

Dr. Hori Ram Singh v. Emperor (AIR 1938 FC 43), Gokulchand


Dwarkadas Morarka v. The King (AIR 1948 PC 82),
Shreekantiah Ramayya Munipalli v. State of Bombay (AIR 1955 SC 287) Hori Ram
Singh (Dr.) v. Emperor, GokulchandDwarkadas Morarka v. R.. Shreekantiah Ramayya
Munipalli v. State of Bombay. AIR 1939 FC 43 : 1939 FCR 159
AIR 1948 PC 82 : 75 IA 30
AIR 1955 SC 287 : 57 Bom LR 632

@page-SC814
2. (a) Names of cases and cross-citations are added where only the citation of the case is
given in the original text.
For example,
Citation alone given in text replaced with full case name: ?M.P. Oil Extraction (P) Ltd. v.
State of M.P." and Jab LJ cross-citation added to AIR citation already in raw text, and
separated by ? : ?

Raw text obtained from Registry : SCC Page : Corresponding citations from SCC
page :
The said decision has been reported in AIR 1982 MP 1. The said decision has been
reported in M.P. Oil Extraction (P) Ltd. v. State of M.P. AIR 1982 MP 1 : 1982 Jab LJ
795

2(b). Citations and cross-citations are added where only name of the case is given in the
original text.
For example
Name of case in text replaced with full case reference and cross-citations added as per
SCC style.

Raw text obtained from Registry : SCC Page : Corresponding citations from SCC
page :
Division Bench of this Court in Kishan Lal Sharma (supra) Division Bench of this Court
in Kishan Lal Sharma. Kishan Lal Sharma v. Prem Kishore, AIR 1983 Raj 100 :
1983 Raj LR 164
(d) Among the pensioners also, the above anomaly will prevail as pointed out in Janaki
Prasad.(d) Among the pensioners also, the above anomaly will prevail as pointed out in
Janaki Prasad. Janaki Prasad Parimoo v. State of J. and K., (1973) 1 SCC 420

(c). Citation inserted in case-history where only the title and year of the impugned/earlier
orders are given.
For example,
From the Judgment and Order dated June 17, 1980 of Gujarat High Court in Special Civil
Application No. 2711 of 1999: AIR 1981 Guj 15

3. SCC style of presenting (repeatedly) cited cases


For example,
Changes have been made in the name of the cited cases as per SCC style as ?Rattan
Singh?s case (supra)"; ?Mohammad?s case (supra)" and ?Range Forest Officer?s case" in
the raw text consecutively changed to ?Ratan Singh case"; ?Mohammed case and ?Range
Forest Officer case" in SCC.
@page-SC815

Raw text obtained from Registry : SCC Page:


In Rattan Singh?s case (supra), the High Court of Madhya Pradesh finding certain
illegalities in the prosecution relating to setting aside
In Mohammad?s case (supra), the observation of the Kerala High Court that ?if a clear
illegality or injustice comes to the notice of the High Court
In the third case relied on by Justice M.K. Chawla, namely, Range Forest Officer?s case,
a vehicle belonging to the respondent was confiscated. 140. In Ratan Singh case the
High Court of Madhya Pradesh finding certain illegalities in the prosecution relating to
setting aside
141. In Mohammed case, the observations of the Kerela High Court that if a clear
illegality or injustice comes to the notice of the High Court
142. In the third case relied on by Justice M.K. Chawla, namely, Range Forest Officer
case a vehicle belonging to the respondent was confiscated.
* The changes have been underlined.

4. Precise references to quoted matter are provided


For example,
a. The exact page and paragraph number as in the original case source is inserted.

Raw text obtained from Registry : SCC Page :


In Balaji it is stated :
?It seems fairly clear that the backward classes of citizens for whom special provision
After referring to the provisions of Articles 338(3), 340 (1), 341 and 342, the Court
proceeded to hold as follows:
?It would thus be seen that this provision contemplates that some Backward Classes may
by the Presidential order be included In Balaji it is stated: (SCR p. 458)
?It seems fairly clear that the backward classes of citizens for whom special
provisionAfter referring to the provisions of Articles 338(3), 340(1), 341 and 342, the
Court proceeded to hold as follows: (SCR p.458)
?It would thus be seen that this provision contemplates that some Backward Classes may
by the Presidential order be included
It may be appropriate to quote the relevant holding from the judgment:
?When Art. 15(4) refers to the special provision for the advancement of certain classes or
scheduled castes or scheduled tribes, it must not be ignored that the provision which is
authorised to be made It may be appropriate to quote the relevant holding from the
judgment: (SCR pp.467, 470)
?When Article 15(4) refers to the special provision for the advancement of certain classes
or Scheduled Castes and Scheduled Tribes, it must not be ignored that the provision
which is authorised to be made
The Privy Council observed: It may be well to add that their Lordships judgment does not
imply that every sum paid under mistake is recoverable The Privy Council observed:
(IA p.302, para 17)
?It may be well to add that their Lordships? judgment does not imply that every sum paid
under mistake is recoverable
* The changes have been highlighted

@page-SC816
b. The exact page and paragraph number as in the original treatises/reference material is
inserted.

Raw text obtained from Registry : SCC Page :


is very instructive,
?Supposing, for instance, reservations were made for a community or a collection of
communities, the total of which is very instructive: (CAD, Vol. 7, pp. 701-02)
?Supposing, for instance, reservations were made for a community or a collection of
communities, the total of which
is a community which is backward in the opinion of the Government.? is a
community which is backward in the opinion of the Government.? (CAD, Vol. 7, pp. 702)
* The changes have been highlighted.

5. Margin headings are added to quoted extracts from statutes/rules etc. when missing.
For example,
Section number and Margin Heading of the Section have been supplied.

Raw text obtained from Registry : SCC Page :


deals with sovereignty over, and limits of, territorial waters and says:
?(1) The sovereignty of India extends and has always extended to the territorial waters of
India (hereinafter referred to as the territorial waters) and to the seabed and subsoil
underlying, and the air space over such waters. deals with sovereignty over, and
limits of, territorial waters and says:
?3. Sovereignty over, and limits of, territorial waters.- (1) The sovereignty of India
extends and has always extended to the territorial waters of India (hereinafter referred to
as the territorial waters) and to the seabed and subsoil underlying, and the air space over
such waters.
It says: It says:

Raw text obtained from Registry : SCC Page:


(1) All lands, minerals and other things of value underlying the ocean within the
territorial waters, or the continental shelf, or the exclusive economic zone, of India shall
vest in the Union and be held for the purpose of the Union. 297. Things of value within
territorial waters or continental shelf and resources of the exclusive economic zone to
vest in the Union.- (1) All lands, minerals and other things of value underlying the ocean
within the territorial waters, or the continental shelf, or the exclusive economic zone, of
India shall vest in the Union and be held for the purpose of the Union.
That article reads as under:
?19(1) All citizens shall have the right That Article reads as under:
?19. Protection of certain rights regarding freedom of speech, etc.- (1) All citizens shall
have the right

@page-SC817
6. Number of the section/rule/article/paragraph is added to the extract quoted in the
original text
For example,
The sub-section numbers have been added to the text.

Raw text obtained from Registry: SCC Page:


The said provision reads as under:
?Where a landlord has acquired his interest in the premises by transfer, no suit for the
recovery of possession of the premises on any of the grounds mentioned in clause (f) or
clause (ff) of The said provision reads as under:
?13. (3-A) where a landlord has acquired his interest in the premises by transfer, no suit
for the recovery of possession of the premises on any of the grounds mentioned in clause
(f) or clause (ff) of
The said sub-section reads as under:
?If, in the course of any trial under this Act of any offence, it is found that the accused
person has committed any other offence under this Act or any rule made thereunder or
under any other law, he said sub-section reads as under:
?12. (2) If, in the course of any trial under this Act of any offence, it is found that the
accused person has committed any other offence under this Act or any rule made
thereunder or under any other law,
For convenience, we reproduce the sub-section here:
?Any person who is a member of a terrorists gang or a terrorists organization, which is
For convenience, we reproduce the sub-section here:
?3. (5) Any person who is a member of a terrorists? gang or a terrorists? organization,
which is
Sub-section (4) of Section 3 of TADA reads thus:
?whoever harbours or conceals, or attempts to harbour or conceal, any terrorist shall be
punishable with imprisonment for a term which shall not be less than five years but
Section 2 (1) (i) of the TADA which reads thus:-
?Words and expressions used but not defined in this Act and defined in the code shall
have the meanings respectively assigned to them in the Code?
Indian Penal Code by the following words in clause y of Section 2 of the Code:
?words and expressions used herein and not defined but defined in the Indian Penal Code
Sub-section (4) of Section 3 of TADA reads thus:
?3. (4) Whoever harbours or conceals, or attempts to harbour or conceal, any terrorist
shall be punishable with imprisonment for a term which shall not be less than five years
but
Section 2 (1) (i) of TADA which reads thus:
?2. (1) (i) words and expressions used but not defined in this Act and defined in the Code
shall have the meanings respectively assigned to them in the Code?
Indian Penal Code by the following words in clause ?y? of Section 2 of the Code:
?2. (y) words and expressions used herein and not defined but defined in the Indian Penal
Code

7. Phrases like concurring, partly concurring, partly dissenting, dissenting,


supplementing, majority expressing no opinion etc. are added to the original text.
For example,
Words like ?partly dissenting? and ?partly concurring? have been added as per the
application of Editors judgement regarding the opinions expressed by the Judges.
@page-SC818

Raw text obtained from Registry: SCC Page:


D.P. Wadhwa, J.
I agree that the appeal be dismissed.
However, I
S.C. AGRAWAL, J.
Special leave granted.
KOSHAL, J.
On a perusal of the judgment prepared by my learned brother, Krishna Iyer, J., I agree
respectfully with findings (2) to (11), (13) and (14) enumerated by him D. P.
WADHWA, J.- (partly concurring) I agree that the appeal be dismissed. However, I
AGRAWAL, J. (partly dissenting) Special leave granted.
Koshal, J. (partly dissenting) On a perusal of the judgment prepared by my learned
brother, Krishna Iyer, J., I agree respectfully with findings (2) to (11), (13) and (14)
enumerated by him

8. Judges on whose behalf opinion given: Expression such as for himself and Pathak,
C.J., or Fazal Ali and Ranganath Mishra, JJ. etc. are added to the original text.
For example,
A uniform style has been mentioned by SCC to take care of the fact that which judges
have signed the Judgment.

Raw text obtained from Registry: SCC Page:


RANGANATH MISHRA, J.
We have had the benefit of reading the judgment passed
RANGANATHAN, J.
The seeds of the present controversy were sown as early as in 1946. The
Judgments of the Court were delivered by RANGANATH MISHRA, J. (for himself and
Pathak, C.J.) (concurring)
The Judgments of the Court were delivered by
RANGANATHAN, J. (for himself and Ramaswami, J.) - The seeds of the present
controversy were sown as early as in 1946.

9. Existing paragraphs in the original text are broken up and separate paragraph numbers
are given.
For example,
Existing paragraph broken up into two paragraphs and separate paragraph number added
on application of editorial judgment

Raw text obtained from Registry: SCC Page:


but the risk involved in sacrificing efficiency of administration must always be borne in
mind when any State sets about making a provision for reservation of appointments of
posts. We see no justification to multiply the risk, which would be the consequence of
holding that reservation can be provided even in the matter of promotion.
weaker segments of We, the people of India. No other understanding can reconcile the
claim of a radical present and hangover of the unjust past.? A similar view was expressed
in Vasant Kumar by Chinnappa Reddy, J. The learned but the risk involved in
sacrificing efficiency of administration must always be borne in mind when any State sets
about making a provision for reservation of appointments or posts. (SCR p.606)
828. We see no justification to multiply ?the risk?, which would be the consequence of
holding that reservation can be provided even in the matter of promotion.
weaker segments of We, the people of India. No other understanding can reconcile the
claim of a radical present and hangover of the unjust past.?
833. A similar view was expressed in Vasanth Kumar by Chinnappa Reddy, J.

@page-SC819

Raw text obtained from Registry: SCC Page:


Judge said the mere securing of high marks at an examination may not necessarily mark
out a good administrator. The learned Judge said (SCC p. 739, para 36)
?[T]he mere securing of high marks at an examination may not necessarily mark out a
good administrator.

MATTER ADDED UPON VERIFICATION


10. Internal referencing: Use of paragraph numbering for internal referencing within a
judgment.
For example,
Internal paragraph numbering has been added after uniform paragraph numbering have
been provided to the multiple judgments. Para 86, 85, 89, 90, 91 and 92 have been
changed respectively to Paras 790-793, 794 and 797, 798, 799, 800 and 801 to 803.

Raw text obtained from Registry: SCC Page:


(d) ?Creamy layer? can be, and must be excluded. (Para 86)
(e) It is not correct to say that the backward class, social, educational and economic
backwardness are closely inter-twined in the Indian context. (Para 85)
(f) The adequacy of representation of a particular class in the services under the State is a
matter within the subjective satisfaction of the appropriate Government. The judicial
scrutiny in that behalf is the same as in other matters within the subjective satisfaction of
an authority. (Para 89)
(4) (a) A backward class of citizens cannot be identified only and exclusively with
reference to economic criteria. (Para 90)
(b) It is, of course, permissible for the Government or other authority to identify a
backward class of citizens on the basis of occupation-cum-income, without reference to
caste, if it is so advised. (Para 91)
(5) There is no constitutional bar to classify the backward classes of citizens into
backward and more backward categories. (Para 92) (d) ?Creamy layer? can be, and must
be excluded.(Paras 790-793)
(e) It is not necessary for a class to be designated as a backward class that it is situated
similarly to the Scheduled Castes/ Scheduled Tribes. (Paras 794 and 797)
(f) The adequacy of representation of a particular class in the services under the State is a
matter within the subjective satisfaction of the appropriate Government. The judicial
scrutiny in that behalf is the same as in other matters within the subjective satisfaction of
an authority. (Para 798)
(4) (a) A backward class of citizens cannot be identified only and exclusively with
reference to economic criteria. (Para 799)
(b) It is, of course, permissible for the Government or other authority to identify a
backward class of citizens on the basis of occupation-cum-income, without reference to
caste, if it is so advised. (Para 800)
(5) There is no constitutional bar to classify the backward classes of citizens into
backward and more backward categories. (Para 801 to 803)

11. Verification of first word of quoted extract and emphasis supplied on verification.
For example,

Raw text obtained from Registry: SCC Page:


The Rajasthan High Court in CIT v Rangnath Bangur opined:
?.........that once a reassessment pro The Rajasthan High Court in CIT v. Rangnath
Bangur opined: (p.498)
?[T]hat once a reassessment proceeding

@page-SC820

Raw text obtained from Registry: SCC Page:


ceeding is initiated, the original order of assessment is set aside or ceases to be operative.
The finality of such an assessment order is wiped out and a fresh order of assessment
would take the place of and completely substitute the initial order of assessment. It is,
therefore, clear that when is initiated, the original order of assessment is set aside or
ceases to be operative. The finality of such an assessment order is wiped out and a fresh
order of assessment would take the place of and completely substitute the initial order of
assessment. It is, therefore, clear that when
and said :
?reassessment proceedings cannot be contained only to such income which has escaped
assessment, but the entire assessmentand said : (P. 503)
?[R]eassessment proceedings cannot be confined only to such income which has escaped
assessment, but the entire assessment
Five Judges:
?the Constitution is the fundamental law of the land and it is wholly unnecessary to
provide in any law made by the legislature that anything done in disregard of the
Constitution is prohibited. Such a prohibition is to be read in every enactment. ?[T]he
Constitution is the fundamental law of the land and it is wholly unnecessary to provide in
any law made by the legislature that anything done in disregard of the Constitution is
prohibited. Such a prohibition is to be read in every enactment.? (emphasis supplied)

2. Ellipsis ?.......? is added to indicate breaks in quoted extract.For example,

Raw text obtained from Registry: SCC Page:


he has said that ?the word ?caste? appearing after ?scheduled? is really a misnomer and
has been used only for the purpose of identifying this 165), he has said that
?.......the word ?caste? appearing after ?scheduled? is really a misnomer and has been
used only for the purpose of identifying this
Gajendragadkar, J observed:
?Though castes in relation to Hindus may be a relevant factor to consider in determining
the social backwardness of groups or classes of citizens, it cannot be the sole or the
dominant test in that behalf. Gajendragadkar, J. observed:
?.........though castes in relation to Hindus may be a relevant factor to consider in
determining the social backwardness of groups or classes of citizens, it cannot be made
the sole or the dominant test in that behalf.
manner as may be prescribed duties of excise on all excisable goods which are produced
or manufactured in India as, and at the rates, set forth in the Schedule to the Central
Excise Tariff Act, 1985. manner as may be prescribed duties of excise on all
excisable goods.... which are produced or manufactured in.... India as, and at the rates, set
forth in the Schedule to the Central Excise Tariff Act, 1985.

13. Matter inadvertently missed in quoted extracts is supplied.


For example,
Incorporation of matter missing in quotations from cases.
@page-SC821

Raw text obtained from Registry: SCC Page:


Where there is no express exclusion the examination of the remedies and the scheme of
the particular Act to find out the intendment becomes necessary to see if the statute
creates a special right or a liability and provides for the determination of the right Where
there is no express exclusion the examination of the remedies and the scheme of the
particular Act to find out the intendment becomes necessary and the result of the inquiry
may be decisive. In the latter case
Mr Justice M. K. Chawla holding that parties have no locus standi. Mr Justice M.K.
Chawla holding that Mr. H.S. Chowdhary and other intervening parties have no locus
standi.
"38. State to secure a social order for the promotion of welfare of the people. (1) The
State shall strive to promote the welfare of the people by securing and protecting as
effectively as it may a social, economic and political, shall inform all the institutions of
the national life. "38. State to secure a social order for the promotion of welfare of
the people.- (1) The State shall strive to promote the welfare of the people by securing
and protecting as effectively as it may a social order in which justice, social, economic
and political, shall inform all the institutions of the national life.
The inputs of efficiency include a sense of belonging and of accountability (not
pejoratively used) if its composition takes in also the weaker segments of we, the people
of India. The inputs of efficiency include a sense of belonging and of accountability
which springs in the bosom of the bureaucracy (not pejoratively used) if its composition
takes in also the weaker segments of We, the people of India.
"It is no doubt true that the Act was amended by U.P. Act 26 of 1975 which came into
force on August 18, 1975 taking away the power of the Director to make an appointment
under Section 16 F (4) of the Act in the case of minority institutions. The amending Act
did not, however, provide proceedings under Section 16F of the Act. "It is no doubt
true that the Act was amended by U.P. Act 26 of 1975 which came into force on August
18, 1975 taking away the power of the Director to make an appointment under Section
16-F(4) of the Act in the case of minority institutions. The amending Act did not,
however, provide expressly that the amendment in question would apply to pending
proceedings under Section 16-F of the Act.
* The changes have been underlined.

14. Incomplete/incorrect case names or citations are completed/corrected.


For example,
Corrections in the case names.
@page-SC822

Raw text obtained from Registry SCC Page:


In R v. Greater London Council 1976 (3) ALL ER 184, one Albert Raymond Blackburn
73. In R v. Greater London Council, ex parte Blackburn, one Albert Raymond
Blackburn
Ray, C.J. in State of Uttar Pradesh v. Pradeep Tandon and Ors. 1975 (2) SCR 761 at 766
has gone to the extent of saying that: 47. Ray, CJ in State of U.P. v. Pradip Tandon has
gone to the extent of saying that: (SCC pp. 273-74, para 15)
Reference may be made to (1) Hindustan Zinc v. A.P. State Electricity Board, 1991 (3)
SCC 299; (2) Sitaram Sugars v. Union of India and Others, 1990 (3) SCC 223; (3)
D.C.M. v. S. Paramjit Singh, 1990 (4) SCC 723; (4) Minerva Talkies v. State of
Karnataka and Others, 1988 Suppl SCC 176; (5) State of Karnataka v. Ranganath Reddy,
1978 (1) SCR 641; (6) Kerala State Electricity Board v. S.N. Govind Prabhu, 1986 (4)
SCC; (7) Prag Ice Company v. Union of India and Others, 1978 (2) SCC 458; (8)
Sarawaswati Industries Syndicate Ltd. v. Union of India, 1975 (1) SCR 956; (9) Murti
Match Works v. Assistant Collector, Central Excise and Others, 1974 (3) SCR 121; (10)
T. Govindraja Mudaliar v. State of Tamil Nadu and Others, 1973 (3) SCR 222; and (11)
Narender Kumar v. Union of India and Others, 1969 (2) SCR 375. Reference may be
made to : (1) Hindustan Zinc Ltd. v. A.P. State Electricity Board; (2) Shri Sitaram Sugar
Co. Ltd. v. Union of India; (3) Delhi Cloth and General Mills Ltd. v. S. Paramjit Singh;
(4) Minerva Talkies v. State of Karnataka; (5) State of Karnataka v. Ranganath Reddy; (6)
Kerela State Electricity Board v. S.N.Govinda Prabhu and Bros.; (7) Prag Ice and Oil
Mills v. Union of India; (8) Saraswati Industries Syndicate Ltd. v. Union of India; (9)
Murthy Match Works v. Assistant Collector, Central Excise; (10) T. Govindaraja
Mudaliar v. State of T.N. and (11) Narender Kumar v. Union of India.
* The changes have been underlined.

15. Other corrections


For example,
a. Clauses numbered in terms of answers to questions framed by learned Judge have
been renumbered correctly in terms of questions framed, as (3)(e) actually has been found
to be answer to (3) (c) and vice-versa.
a1. Similarly, clause has been changed to sub-clause.

Raw text obtained from Registry: SCC Page:


(c) It is not necessary for a class to be designated as a backward class that it is situated
similarly to the Schedule Castes/Tribes. (Paras 87 and 88)
(d) Creamy layer can be, and must be excluded. (Para. 86)
(e) It is not correct to say that the back (c) It is not correct to say that the backward
class of citizen contemplated in Article 16 (4) is the same as the socially and
educationally backward classes referred to in Article 15(4). It is much wider. The accent
in Article 16(4) is on social backwardness. Of course, social, educational and economic
backwardness are closely inter-

@page-SC823

Raw text obtained from Registry: SCC Page:


ward class of citizen contemplated in Article 16 (4) is the same as the socially and
educationally backward classes referred to in Article 15(4). It is much wider. The accent
in Article 16(4) is on social backwardness. Of course, social, educational and economic
backwardness are closely inter-twined in the Indian context. twined in the Indian
context. (Paras 786-789)
(d) Creamy Layer can be, and must be excluded. (790-793)
(e) It is not necessary for a class to be designated as a backward class that it is situated
similarly to the Schedule Castes/ Schedule Tribes. (Paras 794 and 797)
that no better formula could be produced than the one that is embodied in clause (3) of
Article 10 of the Constitution; they will find that the view of those who believe and hold
that there shall be that no better formula could be produced than the one that is
embodied in sub-clause (3) of Article 10 of the Constitution; they will find that the view
of those who believe and hold that there shall be
16. Text has been changed as per corrigenda issued, which have been issued upon SCC
Editor?s request and suggestions.
For example,
SUPREME COURT OF INDIA
Corrigendum
This Courts order dated October 25, 1996 in CA 14553/96 @ SLP (C) No. 5570/93 in the
matter of Smt. Indira Sohan Lal (Dead) by LRs. v. Union of India

Page No. Line No. For Read

1 bottom line and deducted deducted


2 7-8 from bottom developed to bring on par with levelled land and huge
developed tobring them on levelled landand a huge
3 12-13 from bottom compelling material,nor the High Court refused to advert to
compelling material and High Court?s refusal to advert to it,

OTHER ADDITIONS/INSERTIONS MADE TO THE RAW TEXT


17. Compressing/simplification of information relating to case history.
For example

Raw text obtained from Registry: SCC Page:


CIVIL APPEAL NOS. 999-1005 OF 1997
[ARISING OUT OF S.L.P. (C) NOS. 18380-86 OF 1996] Civil Appeals Nos. 999 to
1316 of 1997 with I.A. No. 1 in C.As. arising out of SLPs. (C) Nos. 24224, 24285,
24315, 24320-22, 24325-26 and 24328-29 of 1996, decided on February 20, 1997.
WITH
CIVIL APPEAL NOS. 1006-1316 OF 1997 [ARISING OUT OF S.L.P. (C) NOS.
20293/96,

@page-SC824

Raw text obtained from Registry: SCC Page:


20662/96, 21726/96, 21824-26/96, 22224-502/96, 22771/96, 23196-97/96, 23199/96,
23700-703/96, 23744/96,23747-48/96, 23761/96, 23763/96, 23766/96, 23775-76/96,
24285/96,24315/96,24320-22/96, 24325-26/96, 24328-29/96 and 24224/96
WITH
INTERLOCUTORY APPLICATION NO.1
IN
CIVIL APPEALS
[ARISING OUT OF S.L.P.(C)NOS. 24224/96,24285/96,24315/96,24320-22/96, 24325-
26/96 and 24328-29/96.
passed by Madhya Pradesh High Court respectively in Misc. Petitions No. 1371 of 1992,
M.P. No. 1980 of 1992 and M.P. No. 2315 of 1992. All the said Misc. Petitions were filed
before the Madhya Pradesh High Court under Article 226 of the Constitution. passed
by Madhya Pradesh High Court respectively in Miscellaneous Petitions Nos. 1371, 1980
and 2315 of 1992. All the said miscellaneous petitions were filed before the Madhya
Pradesh High Court under Article 226 of the Constitution
* The changes have been underlined.

(SCC HAS UNIQUE STYLE)


18. There are certain norms followed at SCC for giving case names.
For example,

Raw text obtained from Registry: SCC Page:


Budh Prakash Jai Prakash v. Sales Tax Officer, Kanpur [1952 A.L.J. 332] Budh Prakash
Jai Prakash v. STO
Indian Aluminium Cables Limited vs. State of Haryana Indian Aluminium Cables
Ltd. v. State of Haryana
Trilok Nath Tiku and Another v. State of Jammu and Kashmir and Others Triloki Nath
Tiku v. State of J and K (I)
R. Chitralekha and Anr. v. State of Mysore and Ors. 1964 (6) SCR 368 at 388 and Triloki
Nath v. J and K State 1969 (1) SCR 103 at 105 and K.C. Vasanth Kumar v. Karnataka
1985 Supp. (1) SCR 352 R. Chitralekha v. State of Mysore and Triloki Nath v. State
of J and K (II) and K.C. Vasanth Kumar v. State of Karnataka
Minor P. Rajendran V. State of Madras and Ors. 1968 (2) SCR 786 at 790 P. Rajendran v.
State of Madras
State of Andhra Pradesh V. P. Sagar 1968 (3) SCR 595 State of A.P. v. P. Sagar
K.S. Venkataraman and Bharat Kala Bhandar Ltd. v. M.C. Dhamangaon K.S.
Venkataramanan and Bharat Kala Bhandar Ltd. v. Municipal Committee

19. Words like Section, Sec., Rule etc. are omitted, and only the number of the
Section/Rule is given at the beginning of the quoted extract.
@page-SC825

Raw text obtained from Registry: SCC Page:


Sec 2 (h). terrorist act has the meaning assigned to it in sub-section (1) of Section 3, and
the expression terrorist shall be construed accordingly;
?Rule 11. No refund of duties or charges erroneously paid, unless claimed within three
months ? No duties or charges which have been paid or have been adjusted in an account
current maintained with the Collector
?RULE 233B. Procedure to be followed to cases where duty is paid under protest.? (1)
Where an assessee desires to pay duty under protest he shall deliver to the proper officer
a letter to this ?2 (h) terrorist act has the meaning assigned to it in sub-section (1) of
Section 3, and the expression terrorist shall be construed accordingly;?
?11. No refund of duties or charges erroneously paid, unless claimed within three
months.? No duties or charges which have been paid or have been adjusted in an account
current maintained with the Collector
?233-B. Procedure to be followed in cases where duty is paid under protest. (1) Where an
assessee desires to pay duty under protest he shall deliver to the proper officer a letter to
this
20. Margin heading and the first clause/sub-section or initial matter of section/rule etc. is
made to run-on, instead of being let to start from a fresh line.

Raw text obtained from Registry: SCC Page:


?Liability of person to whom money is paid or thing delivered by mistake or under
coercion? 72. A person to whom money has been paid, or anything delivered, by mistake
or under coercion, must repay or return it. ?72. Liability of person to whom money is
paid or thing delivered, by mistake or under coercion.? A person to whom money has
been paid, or anything delivered, by mistake or under coercion, must repay or return it.
Sec 424. Refund of automobile accessories tax.
(a) No refund shall be made of any amount paid by or collected from any manufacturer,
producer, or importer in respect ?424. Refund of automobile accessories tax.? (a) No
refund shall be made of any amount paid by or collected from any manufacturer,
producer, or importer in respect
Section 3, which is the charging Section, reads:-3. Duties specified in the Schedule to the
Central Excise Tariff Act, 1985 to be levied.
(1) There shall be levied and collected in such manner as may be prescribed duties?175.
Section 3, which is the charging section, reads:
?3. Duties specified in the Schedule to the Central Excise Tariff Act, 1985 to be levied. -
(1) There shall be levied and collected in such manner as may be prescribed duties

21. Compressing of unquoted referends and use of *** for such parts.

Raw text obtained from Registry: SCC Page:


?six months?, the words ?five years? were substituted.
Explanation
(ii) ?relevant date- means,
(a) in the case of excisable goods on which duty of excise has not been levied or paid or
has been short-levied or short-paid ....
(c) in any other case, the date on which ?six months?, the words ?five years? were
substituted.
Explanation.?
(1)-(2) * * *
(3) (i) * * *
(ii) ?relevant date? means, ?
(a) in the case of excisable goods on which duty of excise has not been levied or paid or
has been short-levied or short-paid ....

@page-SC826

Raw text obtained from Registry: SCC Page:


the duty is to be paid under this Act or the rules made thereunder; (c) in any other case,
the date on which the duty is to be paid under this Act or the rules made thereunder,
(i).................. * *
(ii)................... * *
(iii) where the landlord of any building is
(1)a serving or retired Indian Soldier as defined in the Indian Soldiers (Litigation) Act,
1925 (IV of 1925) and such building was let out at any time before his retirement, or
(2) ................
and such landlord needs such building for occupation by himself or the members of his
family for residential purposes, (i)-(ii) * * *
(iii) where the landlord of any building is?
(1) a serving or retired Indian Soldier as defined in the Indian Soldiers (Litigation) Act,
1925 (IV of 1925), and such building was let out at any time before his retirement, or
(2) * * *
and such landlord needs such building for occupation by himself or the members of his
family for residential purposes,

22. Series of dots in the raw texts (i.e., ..........) are replaced with ellipsis (i.e.,....).

Raw text obtained from Registry: SCC Page:


so to say into the administration.............. .that no better formula could be produced than
the one that is embodied in clause (3) of Article 10 of the Constitution; they will find that
the view of those who believe and hold that there shall be equality of opportunity has
been embodied in sub-clause (1) of Article 10. It is a generic principle
................Supposing for instance, we are to concede in full the demand of those
communities who have not been so far employed in the public services to the fullest
extent, what would really happen is, we shall be completely destroying the first
proposition upon which we are all agreed, namely, that there shall be in an equality of
opportunity............ . I am sure they will agree that unless you use some such qualifying
so to say into the administration......... that no better formula could be produced
than the one that is embodied in sub-clause (3) of Article 10 of the Constitution; they will
find that the view of those who believe and hold that there shall be equality of
opportunity, has been embodied in sub-clause (1) of Article 10. It is a generic
principle......... Supposing for instance, we are to concede in full the demand of those
communities who have not been so far employed in the public services to the fullest
extent, what would really happen is, we shall be completely destroying the first
proposition upon which we are all agreed, namely, that there shall be in an equality of
opportunity..................... I am sure they will agree that unless you use some such
qualifying

23. Removal of abbreviations: sec., R. and cl. are substituted respectively with ?Section?,
?Rule? or ?clause?.

Raw text obtained from Registry: SCC Page:


Having regard to the object and language of s. 34 of the I.T. Act, 1922, s. 147 of the I.T.
Act, 1961, and s. 8 of the Surtax Act, 1964, the reopening of an assessment can only be
for the ben ?Having regard to the object and language of Section 34 of the I.T. Act,
1922, Section 147 of the I.T. Act, 1961, and Section 8 of the Surtax Act, 1964, the
reopening of an assessment can only be for the benefit of the Revenue subject to one
exception,
@page-SC827

Raw text obtained from Registry: SCC Page:


efit of the Revenue subject to one exception,
?...........it would not be in accordance either with cl. (1) of Art. 15 or cl. (2) of Art. 29 to
require the consideration of the castes of persons to be borne in mind for determining
what are socially and educationally backward classes. It is true that cl. (4) of Art. 15
contains a non-obstante clause with the result ?.............it would not be in
accordance either with clause (1) of Article 15 or clause (2) of Article 29 to require the
consideration of the castes of persons to be borne in mind for determining what are
socially and educationally backward classes. It is true that clause (4) of Article 15
contains a non-obstante clause with the result
* The changes have been underlined.

24. Hyphenation has been added after the section/rule numbers, which have alphabets,
suffixed to them.

Raw text obtained from Registry: SCC Page:


SCOPE OF SECTIONS 11B, 11D, 12A, 12B, 12C AND 12D OF THE CENTRAL
EXCISE ACT, 1944
Sections 11B and 11D in Chapter II and Sections 12A, 12B, 12C and 12D in Chapter II-A
are now to be considered :?
?11B. Claim for refund of duty
(1) Any person claiming refund of any duty of excise may make an application for refund
of such duty to the Assistant Commissioner of Central Excise before the Scope of
Sections 11-B, 11-D, 12-A, 12-B, 12-C and 12-D of The Central Excises and Salt Act,
1944
Sections 11-B and 11-D in Chapter II and Sections 12-A, 12-B, 12-C and 12-D in
Chapter II-A are now to be considered:
?11B. Claim for refund of duty.?
(1) Any person claiming refund of any duty of excise may make an application for refund
of such duty to the Assistant Collector of Central Excise before the
*The changes have been underlined.

25. Indentation
For example
SCC style of presentation of quoted extracts in separate indented paragraphs applied to
raw text.

Raw text obtained from Registry: SCC Page:


As Chinnappa Reddy, J. in Vasanth Kumar has rightly observed, Always one hears the
word efficiency as if it is sacrosanct and the sanctorum has to be fiercely guarded.
Efficiency is not a mantra which is whispered by the Guru in the Sishya?s ear.? 57. As
Chinnappa Reddy, J. in Vasanth Kumar has rightly observed: (SCC p.739, para 36)
?Always one hears the word efficiency as if it is sacrosanct and the sanctorum has to be
fiercely guarded. Efficiency is not a mantra which is whispered by the Guru in the
Sishya?s ear.?

26. Removal of full stops or removal of word ?No.?..


@page-SC828

Raw text obtained from Registry: SCC Page:


The appellant says that each of these R.S.Os. maintains an office, a stock yard and other
necessary paraphernalia for receiving, stocking, repairing and delivering motor vehicles
to their customers. The appellant says that almost seventy percent of its sales are to
parties other than State Transport Undertakings S.T.Us. The sales to S.T.Us., are in the
region of thirty percent of its production. The R.S.Os., the appellant says, contact the
local purchasers and the S.T.Us., book the order and also deliver the vehicles to them
pursuant to sales effected by them. The appellant always keeps the R.S.Os. well stocked
having regard to their requirements. By way of illustration, it is stated, the R.S.O. at
Hyderabad
All the three special leave petitions namely S.L.P. (Civil) No. 19279 of 1995, S.L.P.
(Civil ) No. 20137 of 1995 and S.L.P. (Civil ) No. 19796 of 1995 are directed against
common judgment dated 9.5.1995 The appellant says that each of these RSOs
maintains an office, a stock yard and other necessary paraphernalia for receiving,
stocking, repairing and delivering motor vehicles to their customers. The appellant says
almost seventy per cent of its sales are parties other than State Transport Undertakings
(STUs). The sales to STUs are in the region of thirty per cent of its production. The
RSOs, the appellant says, contact the local purchasers and the STUs book the orders and
also deliver the vehicles to them pursuant to sales effected by them. The appellant always
keeps the RSOs well stocked having regard to their requirements. By way of illustration,
it is stated, the RSO at Hyderabad
2. All the three special leave petititions namely SLP (Civil) No. 19729 of 1995, SLP
(Civil ) No. 20137 of 1995 and SLP (Civil ) No. 19796 of 1995 are directed against
common judgment dated 9-5-1995* The changes have been underlined.

27. Giving full forms of abbreviations to enhance readability and clarity.

Raw text obtained from Registry: SCC Page:


from legal consequences and therefore, they are also guilty of the offence u/s 201 IPC.
from legal consequences and therefore, they are also guilty of the offence under
Section 201, IPC.
* The changes have been underlined.

In addition to the above, capitalization and italicization is made wherever necessary in the
raw text; and punctuation, articles, spellings and compound words are also checked and
corrected, if required, in the original text.
8. The copyright protection finds its justification in fair play. When a person produces
something with his skill and labour, it normally belongs to him and the other person
would not be permitted to make a profit out of the skill and labour of the original author
and it is for this reason the Copyright Act, 1957 gives to the authors certain exclusive
rights in relation to the certain work referred in the Act. The object of the Act is to protect
the author of the copyright work from an unlawful reproduction or exploitation of his
work by others. Copyright is a right to stop others from exploiting the work without the
consent or assent of the owner of the copyright. A copyright law presents a balance
between the interests and rights of the author and that of the public in protecting the
public domain, or to claim the copyright and protect it under the copyright statute. One of
the key requirements is that of originality which contributes, and has a direct nexus, in
maintaining the interests of the author as well as that of public in protecting the matters in
public domain. It is a well-accepted principle of copyright law that there is no copyright
in the facts per se, as the facts are not created nor have they originated with the author of
any work which embodies these facts. The issue of copyright is closely connected to that
of commercial viability, and commercial con
@page-SC829
sequences and implications.
9. The development of copyright law in India is closely associated with the British
copyright law. Statute of Anne, the first Copyright Act in England, was passed in 17th
century which provided that the author of any book already printed will have the sole
right of printing such book for a term mentioned therein. Thereafter, came the Act of
1814, and then the Act of 1842 which repealed the two earlier Acts of 1709 and 1814.
The Copyright Act of 1911 in England had codified and consolidated the various earlier
Copyright Acts on different works. Then came the Copyright Act of 1956. In India, the
first Copyright Act was passed in 1914. This was nothing but a copy of the Copyright Act
of 1911 of United Kingdom with suitable modifications to make it applicable to the then
British India. The Copyright Act of 1957, which is the current statute, has followed and
adopted the principles and provisions contained in the U.K. Act of 1956 along with
introduction of many new provisions. Then came the Copyright (Amendment) Act, 1983
which made a number of amendments to the Act of 1957 and the Copyright
(Amendment) Act, 1984 which was mainly introduced with the object to discourage and
prevent the widespread piracy prevailing in video films and records. Thereafter, the
Copyright (Amendment) Act, 1994 has effected many major amendments in the
Copyright Act of 1957.
10. In the present case, the questions which require determination by the Court are : (1)
What shall be the standard of originality in the copy-edited judgments of the Supreme
Court which is a derivative work and what would be required in a derivative work to treat
it the original work of an author and thereby giving a protected right under the Copyright
Act, 1957 to the author of the derivative work ? and (2) Whether the entire version of the
copy-edited text of the judgments published in the appellants law report SCC would be
entitled for a copyright as an original literary work, the copy-edited judgments having
been claimed as a result of inextricable and inseparable admixture of the copy-editing
inputs and the raw text, taken together, as a result of insertion of all SCC copy-editing
inputs into the raw text, or whether the appellants would be entitled to the copyright in
some of the inputs which have been put in the raw text ?
11. Copyright is purely a creation of the statute under the 1957 Act. What rights the
author has in his work by virtue of his creation, are defined in Sections 14 and 17 of the
Act. These are exclusive rights, but subject to the other provisions of the Act. In the first
place, the work should qualify under the provisions of Section 13, for the subsistence of
copyright. Although the rights have been referred to as exclusive rights, there are various
exceptions to them which are listed in Section 52.
12. We are mainly concerned for the purpose of these appeals with Sections 2 [clauses
(k), (o), (y)], 13(1), 14(1)(a), 17, proviso (d) and 52(1)(q)(iv) of the Copyright Act, 1957.
The relevant provisions of these Sections are as under :
2. Interpretation.? In this Act, unless the context otherwise requires,
xxx xxx xxx
(k) ?Government work? means a work which is made or published by or under the
direction or control of
(i) the Government or any department of the Government;
ii) any Legislature in India;
(iii) any Court, Tribunal or other judicial authority in India;
xxx xxx xxx
(o) ?literary work? includes computer programmes, tables and compilations including
computer databases;
xxx xxxxxx
?(y) ?work? means any of the following works, namely :?
(i) a literary, dramatic, musical or artistic work;
(ii) a cinematograph film;
(iii) a sound recording;
"13. Works in which copyright subsists.? (1) Subject to the provisions of this section and
the other provisions of this Act, copyright shall subsist throughout India in the following
classes of works, that is to say,?
(a) original literary, dramatic, musical and artistic works;
(b) cinematograph films; and
(c) sound recording,
(2) Copyright shall not subsist in any work specified in sub-section (1), other than a work
to which the provisions of section 40 or section 41, apply, unless?
(i) in the case of a published work, the
@page-SC830
work is first published in India, or where the work is first published outside India, the
author is at the date of such publication, or in a case where the author was dead at that
date, was at the time of his death, a citizen of India;
(ii) in the case of an unpublished work other than a work of architecture, the author is at
the date of the making of the work a citizen of India or domiciled in India; and
(iii) in the case of a work of architecture, the work is located in India.
Explanation.? In the case of a work of joint authorship, the conditions conferring
copyright specified in this sub-section shall be satisfied by all the authors of the work.
(3)Copyright shall not subsist?
(a) in any cinematograph film if a substantial part of the film is an infringement of the
copyright in any other work;
(b) in any sound recording made in respect of a literary, dramatic or musical work, if in
making the sound recording, copyright in such work has been infringed.
xxx xxxxxx
14. Meaning of copyright.? (1) For the purposes of this Act, ?copyright? means the
exclusive right, subject to the provisions of this Act, to do or authorise the doing of any of
the following acts in respect of a work or any substantial part thereof, namely :?
(a) in the case of a literary, dramatic or musical work, not being a computer programme,?
(i) to reproduce the work in any material form including the storing of it in any medium
by electronic means;
(ii) to issue copies of the work to the public not being copies already in circulation;
(iii) to perform the work in public, or communicate it to the public;
(iv) to make any cinematograph film or sound recording in respect of the work;
(v) to make any translation of the work;
(vi) to make any adaptation of the work;
(vii) to do, in relation to a translation or an adaptation of the work, any of the acts
specified in relation to the work in sub-clauses (i) to (vi);
xxx xxxxxx?
17. First owner of copyright.? Subject to the provisions of this Act, the author of a work
shall be the first owner of the copy right therein :
Provided that?
xxx xxxxxx
(d) in the case of a Government work, Government shall, in the absence of any agreement
to the contrary, be the first owner of the copyright therein;
xxx xxxxxx?
?52. Certain acts not to be infringement of copyright.? (1) The following acts shall not
constitute an infringement of copyright, namely :?
(a) .....
xxx xxxxxx
(q) the reproduction or publication of?
(i) .........
xxx xxxxxx
(iv) any judgment or order of a Court, Tribunal or other judicial authority, unless the
reproduction or publication of such judgment or order is prohibited by the Court, the
Tribunal or other judicial authority, as the case may be;
xxx xxxxxx?
13. Subject to the provisions of Section 13 and the other provisions of the Act, there shall
be a copyright throughout India in original literary work, dramatic, musical and artistic
works, cinematograph films and sound recording, subject to the exceptions provided in
sub-sections (2) and (3) of Section 13. For copyright protection, all literary works have to
be original as per Section 13 of the Act. Broadly speaking, there would be two classes of
literary works : (a) primary or prior works: These are the literary works not based on
existing subject-matter and, therefore, would be called primary or prior works; and (b)
secondary or derivative works: These are literary works based on existing subject-matter.
Since such works are based on existing subject-matter, they are called derivative work or
secondary work. Work is defined in Section 2(y) which would be a literary, dramatic,
musical or artistic work; a cinematograph film; and a sound recording. Under Section
2(o), literary work would include computer programmes, tables and compilations
including computer databases. For the purposes of the Act, Section 14(1) enumerates
what shall be a copyright which is an exclusive right, subject to the provisions of the Act,
to do or authorize the doing of the acts provided in clauses (i) to (vii)
@page-SC831
in respect of a work or any substantial part thereof in the case of a literary, dramatic or
musical work, not being a computer programme. Section 2(k) defines the "government
work" which would be a work which is made or published by or under the direction or
control of, amongst others, any Court, Tribunal or other judicial authority in India. By
virtue of this definition, the judgments delivered by the Supreme Court would be a
government work. Under Section 17(d), the Government shall, in the absence of any
agreement to the contrary, be the first owner of the copyright in a government work. In
the absence of any agreement to the contrary, the government shall be the first owner of
the copyright in the judgments of the Supreme Court, the same being a government work
under Section 2(k). Section 52(1) expressly provides that certain acts enumerated therein
shall not constitute an infringement of copyright and sub-clause (iv) of clause (q)
excludes the reproduction or publication of any judgment or order of a Court, Tribunal or
other judicial authority, unless the reproduction or publication of such judgment or order
is prohibited by the Court, the Tribunal or other judicial authority from copyright. The
judicial pronouncements of the Apex Court would be in the public domain and its
reproduction or publication would not infringe the copyright. The reproduction or
publication of the judgments delivered by the Supreme Court by any number of persons
would not be infringement of a copyright of the first owner thereof, namely, the
Government, unless it is prohibited. The question, therefore, is whether by introducing
certain inputs in a judgment delivered by a court it becomes original copy-edited
judgment and the person or authority or company who did so could claim to have
embodied the originality in the said judgment and the judgment takes the colour of
original judgment having a copyright therein of its publisher.
14. In many cases, a work is derived from an existing work. Whether in such a derivative
work, a new copyright work is created, will depend on various factors, and would one of
them be only skill, capital and labour expended upon it to qualify for copyright protection
in a derivative literary work created from the pre-existing material in the public domain,
and the required exercise of independent skill, labour and capital in its creation by the
author would qualify him for the copyright protection in the derivative work. Or would it
be the creativity in a derivative work in which the final position will depend upon the
amount and value of the corrections and improvements, the independent skill and labour,
and the creativity in the end-product is such as to create a new copyright work to make
the creator of the derivative work the author of it; and if not, there will be no new
copyright work and then the original author will remain the author of the original work
and the creator of the derivative work will have been the author of the alterations or the
inputs put therein, for their nature will not have been such as to attract the protection
under the law of copyright.
15. It is submitted by Shri Raju Ramachandran, learned senior counsel for the appellants
that Section 52(1)(q)(iv) of the Act does not bar the recognization of copyright in the
copy-edited version of the text of judgments of the courts as published in law reports. The
Government is the first owner of copyright in the judgments of the courts as per Section
2(k) read with Section 17 and Section 52(1)(q)(iv) of the Act provides that any person
wanting to reproduce or publish judgments would not infringe the copyright of the
Government, but Section 52(1)(q)(iv) does not imply that in case a person has expended
independent skill, labour and capital on the judgments of the courts to create and publish
his version of the judgments, any other person is free to copy that persons version of the
judgments, substantially or in its entirety. Copyright subsists in the copy-edited version of
the text of judgments of the courts as published in law reports, which have been created
by the application of skill, labour and capital which is not trivial or negligible. The inputs
put in the copy-edited judgments in SCC, is a derivative literary work created from pre-
existing material of the judgments of the court which is in public domain. The exercise of
independent skill, labour and capital in its creation by the author of such work, and the
derivative literary work created by the expenditure of the independent skill, labour and
capital of the appellants gives them copyright in such creations. It is not necessary that
work created should have a literary merit. The courts can only evaluate whether the skill,
labour and capital actually employed, required in creating the work, is not trivial or
negligible. It is further urged
@page-SC832
by the learned senior counsel that in deciding whether a derivative work qualifies for
copyright protection, it must be considered as a whole, and it is not correct to dissect the
work into fragments and consider the copyrightability of each such fragment piecemeal
and individually apart from the whole. He submits that the respondents if wish to
reproduce or publish a work already in public domain is obliged to go to the public
domain/common source of such work rather than misappropriating the effort and
investment of the appellants by copying the version of such work which was created by
them by independent expenditure of skill, labour and capital. To buttress his submissions,
the learned senior counsel placed reliance on various foreign judgments and judgments of
the Indian High Courts which are considered hereinafter.
16. Ladbroke (Football) Ltd. v. Willim Hill (Football) Ltd., [1964] 1 WLR 273 (HL), is a
case where the concept of originality was considered on the basis of skill, judgment
and/or labour in the context of compilation. Since 1951 the respondents, who were well-
known bookmakers, had sent their customers each week fixed odds football betting
coupons arranged in a certain general form. In 1959 the appellants, who were also
bookmakers, started sending out coupons closely resembling the respondents coupons. A
coupon was a sheet of paper on which were printed several lists of forthcoming matches.
Beside each list were columns of squares on which the punter could indicate his forecast
of the result of each match. Some of the lists included all the matches to be played; others
included only a selection of them. The bets varied in character. A great variety of bets was
offered and the odds offered differed widely from 5-2 to 20,000-1. The respondents"
coupon contained 16 lists, each with an appropriate name. The appellants" coupon, which
contained 15 lists, closely resembled the respondents". The lists offered by the appellants
were almost identical with those offered by the respondents in their corresponding lists.
The respondents brought action claiming copyright in the coupons. The House of Lords
was called upon to determine whether or to what extent copyright attached to these
coupons. The respondents said that a coupon must be regarded as a single work and that
as such it was protected by copyright. The appellants sought to dissect the coupon. It was
contended by the respondents that there had been a breach of copyright by the appellants,
since the respondents" compilation, which must be regarded as a single work, was
original and protected by copyright and the part taken by the appellants from the
respondents" work was substantial. It did not follow that because the fragments of the
compilation, taken separately, would not be copyright, the whole could not be copyright.
It was submitted by the appellants that the derivative work of the respondents not being
original, no copyright can be claimed and the inputs put, if considered separately, are of
insignificant value and thus the respondents could not claim copyright. The word
"original" does not mean that the work must be the expression of original or inventive
thought. Copyright Acts are not concerned with the originality of ideas, but with the
expression of thought, and in the case of literary work, with the expression of thought in
print or writing. The originality which is required relates to the expression of the thought.
But the Act does not require that the expression must be in an original or novel form, but
that the work must not be copied from another work - that it should originate from the
author; and as regards compilation, originality is a matter of degree depending on the
amount of skill, judgment or labour that has been involved in making the compilation.
The words "literary work" cover work which is expressed in print or writing irrespective
of the question whether the quality or style is high. The commonplace matter put together
or arranged without the exercise of more than negligible work, labour and skill in making
the selection will not be entitled to copyright.
The word original does not demand original or inventive thought, but only that the work
should not be copied but should originate from the author. In deciding, therefore, whether
a work in the nature of a compilation is original, it is wrong to consider individual parts
of it apart from the whole. For many compilations have nothing original in their parts, yet
the sum total of the compilation may be original. In such cases the courts have looked to
see whether the compilation of the unoriginal material called for work or skill or expense.
If it did, it is entitled to be considered original and to be protected against those who wish
to steal the fruits of the work or skill or expense by copying it without taking the
trouble to
@page-SC833
compile it themselves. In each case, it is a question of degree whether the labour or skill
or ingenuity or expense involved in the compilation is sufficient to warrant a claim to
originality in a compilation.
17. While considering the question whether the copyright protection is available to the
work created as a whole or the fragment of the work would be considered piecemeal and
individually apart from the whole, the House of Lords said as under:
.......One test may be whether the part which he has taken is novel or striking, or is merely
a commonplace arrangement of ordinary words or well-known data. So it may sometimes
be a convenient short-cut to ask whether the part taken could by itself be the subject of
copyright. But, in my view, that is only a short-cut, and the more correct approach is first
to determine whether the plaintiffs work as a whole is "original" and protected by
copyright, and then to inquire whether the part taken by the defendant is substantial.
A wrong result can easily be reached if one begins by dissecting the plaintiffs" work and
asking, could section A be the subject of copyright if it stood by itself, could section B be
protected if it stood by itself, and so on. To my mind, it does not follow that, because the
fragments taken separately would not be copyright, therefore, the whole cannot be. ........"
18. In the case of Walter and Another v. Lane, [1900] AC 539 (HL), the Earl of Rosebery
on five occasions in 1896 and 1898 delivered to the public audience speeches on subjects
of public interest. The Reporter of "The Times" took down the speeches in shorthand,
wrote out their notes, corrected, revised and punctuated them and the reports were
published in "The Times", the speeches being given verbatim as delivered by Lord
Rosebery. The reporters were employed under the terms that the copyright in all reports
and articles composed by "The Time" magazine should belong to the proprietors. In the
year 1899, the respondent published a book called "Appreciations and Addresses: Lord
Rosebery", which contained the reports of the above speeches of Lord Rosebery and it
was admitted that these reports were taken from the reports in "The Times". Lord
Rosebery made no claim. The appellants brought an action against the respondent
claiming a declaration that a copyright of the articles and reports was vested in the
proprietors of "The Times". The issue involved in the case was whether a person who
makes notes of a speech delivered in public, transcribes them and publishes in the
newspaper a verbatim report of the speech, is the author of the report within the meaning
of the Copyright Act, 1842, and is entitled to the copyright in the report. The House of
Lords held that each reporter is entitled to report and each undoubtedly would have a
copyright in his own published report. It was of course open to any other reporter to
compose his own report of Lord Rosebery's speech, and to any other newspaper and book
to publish that report; but it is a sound principle that a man shall not avail himself of
another's skill, labour and expense by copying the written product thereof; and copyright
has nothing to do with the originality or the literary merits of the author or composer. It
may exist in the information given by a street dictionary. If a person chooses to compose
and write a volume devoid of the faintest spark of literary or any other merit, there is no
legal reason why he should not, if he desires, become the first publisher of it and register
his copyright, worthless and insignificant as it would be.
19. In the case of Designers Guild Ltd. v. Russell Williams (Textiles) Ltd., [2000] 1 WLR
2416 (HL), the plaintiff brought proceedings claiming that the defendant had infringed
the plaintiff’s copyright by copying one of its fabric designs, i.e. for the fabric design
Ixia. The infringement of which the plaintiff complained was that for the purpose of
creating its own design Marguerite by the defendant. The defendant had copied a
substantial part of Ixia. There were mainly two main issues at the trial. First, what, if
anything had the designer of Marguerite copied from Ixia. Secondly, did what had been
copied amount to “the whole or a substantial part" of Ixia? It was said by the House of
Lords that the law of copyright rests on a very clear principle that anyone who by his or
her own skill and labour creates an original work of whatever character shall enjoy an
exclusive right to copy that work. No one else may for a season reap what the copyright
owner had sown.
20. University of London Press Limited v. University Tutorial Press Limited, [1916] 2 Ch
601, is perhaps the most cited judgment regarding originality. Originality was
@page-SC834
held to be not required to be novel form but the work should not be copied from other
work, that is, it should be original. The judgment was based on the following facts:
Certain persons were appointed as examiners for matriculation examination of the
University of London on a condition that any copyright in the examination papers should
belong to the University. The University assigned the copyright to the plaintiff company.
After the examination, the defendant company brought out a publication containing a
number of the examination papers, including three which had been set by two examiners
appointed by the University. The plaintiff company brought a case of copyright
infringement against the defendant company. It was argued that since the setting of the
papers entailed the exercise of brainwork, memory, and trained judgment, and even the
selection of passages from other author's work involved careful consideration, discretion
and choice they constituted original literary work. On the other hand, the defendants
claimed that what they had done was fair dealing for the purposes of private study which
was permissible under the law. The court agreed that the material under consideration
was a literary work. The words "literary work" cover work which is expressed in print or
writing, irrespective of the question whether the quality or style is high. The word
"literary" seems to be used in a sense somewhat similar to the use of the word "literature"
in political or electioneering literature and refers to written or printed matter. With respect
to the originality issue, the Court held that the term "original" under the Act does not
imply original or novel form of ideas or inventive thought, but the work must not be
copied from another work - that it should originate from the author.
21. In Kelly v. Morris, (1866) LR 1 Eq 697, School of thought propounded is that, at least
in respect of compilations, only time and expenses are necessary which is "industrious
collection". The plaintiff was the owner and publisher of the first directory. The defendant
came out with another directory. The plaintiff sought an injunction against the defendant
to restrain the publication of the defendant's directory on the allegations that the
defendant was guilty of appropriating the information contained in the plaintiff's
directory and obtained the benefit of many years of incessant labour and expense. The
defendant, on the other hand, contended that there had been no unfair or improper use of
the plaintiff's work. Information which was given in the plaintiff's directory was entitled
to be used and adopted as long as he did not servilely copy it. The defendant had
bestowed his independent time, labour and expense on the matter and thus had in no way
infringed the copyright of the plaintiff. Granting injunction, the Court held that in the
case of a directory when there are certain common objects of information which must, if
described correctly, be described in the same words, a subsequent compiler is bound to
set about doing for himself that which the first compiler has done. In case of a road-book,
he must count the milestones for himself. In the case of a map of a newly discovered
island he must go through the whole process of triangulation just as if he had never seen
any former map, and, generally he is not entitled to take one word of the information
previously published without independently working out the matter for himself, so as to
arrive at the same result from the same common sources of information, and the only use
that he can legitimately make of a previous publication is to verify his own calculations
and results when obtained. The compiler of a directory or guidebook, containing
information derived from sources common to all, which must of necessity be identical in
all cases if correctly given, is not entitled to spare himself the labour and expense of
original inquiry by adopting and re-publishing the information contained in previous
works on the same subject.
22. In the case of Parry v. Moring and Gollancz, Cop Cas (1901-1904) 49, the plaintiff,
after obtaining permission from the representatives of the owner of certain letters,
updated, chronologically arranged and translated them into modern English for their
inclusion in his book. Later, the defendant published, as one of the series, an edition of
the letters prepared by the plaintiff. The plaintiff, therefore, brought an action against the
defendant alleging infringement of his copyright. The plaintiff maintained his copyright
in his version of the text apart from the copyright in the text. It was held that there is
copyright in the work of editing the text of a non-copyright work. The editor of a non-
copyright work is not entitled to take the text from the edition of a rival
@page-SC835
editor and use it as a copy for the purpose of his own work.
23. In Gopal Das v. Jagannath Prasad and Another, AIR 1938 All 266, the plaintiffs were
the printers and publishers of the books. The book titled "Sachitra Bara Kok Shastra" was
printed for the first time in 1928 and had run into four editions since. The defendants
printed and published another book titled "Asli Sachitra Kok Shastra" in 1930. The
plaintiffs case was that the book published by the defendants was a colourable imitation
of their book and an infringement of plaintiffs copyright. It was held by the Court that
the plaintiffs compiled their book with considerable labour from various sources and
digested and arranged the matter taken by them from other authors. The defendant instead
of taking the pains of searching into all the common sources and obtaining his subject-
matter from them, obtained the subject matter from the plaintiffs book and availed
himself of the labour of the plaintiffs and adopted their arrangement and subject-matter
and, thus, such a use of plaintiffs book could not be regarded as legitimate. It was held
that a person whose work is protected by copyright, if he has collected the material with
considerable labour, compiled from various sources of work in itself not original, but
which he has digested and arranged, the defendant could not be permitted to compile his
work of like description, instead of taking the pains of searching into all the common
sources and obtaining the subject-matter from them and to adopt his arrangement with a
slight degree of colourable variation thereby saving pains and labour which the plaintiff
has employed. The act of the defendant would be illegitimate use. The Court held that no
one is entitled to avail himself of the previous labour of another for the purpose of
conveying to the public the same information, although he may append additional
information to that already published.
24. In V. Govindan v. E. M. Gopala-krishna Kone and Another, AIR 1955 Madras 391,
the respondents had published an English-English Tamil Dictionary in 1932. The
appellants were the publishers of similar Dictionary in 1947. An action was brought
regarding the publication and sale of the dictionary by the appellants which was alleged
to be constituting an infringement of the respondents copyright. The lower court went
through both the books minutely and found, page after page, word after word, slavishly
copied, including the errors, and found the sequence, the meanings, the arrangement and
everything else practically the same, except for some deliberate differences introduced
here and there to cover up the piracy". The High Court referred to Copinger and James on
Law of Copyright wherein the law has been neatly summarized that : In the case of
compilations such as dictionaries, gazetteers, grammars, maps, arithmetics, almanacs,
encyclopaedias and guide books, new publications dealing with similar subject-matter
must of necessity resemble existing publications, and the defence of common source is
frequently made where the new publication is alleged to constitute an infringement of an
earlier one". The Court held that in law books and in books as mentioned above there is
very little amount of originality but the same is protected by law and no man is entitled to
steal or appropriate for himself the result of another's brain, skill or labour even in such
works". The Court further clarified that where there is a "common source", the person
relying on it must prove that he actually went to the common source from where he
borrowed, employing his own skill, labour and brains and that he did not merely copy.
25. In C. Cunniah and Co. v Balraj and Co., AIR 1961 Madras 111, the appellant firm
was carrying on the business in pictures, picture frames, etc. One Sri T.M. Subramaniam
drew a picture of Lord Balasubramanya and gave it the title of Mayurapriya and a
copyright was assigned to the appellant. It came to the knowledge of the appellant firm
that the respondent firm was printing and selling copies of a close and colourable
imitation of the appellant's picture under the style of Bala Murugan. The case of the
defence was that their picture was an independent production and that the appellant had
not acquired copyright in the picture and the subject dealt with in that picture was a
common subject, in which no copyright could be acquired by anyone. The Court held that
in order to obtain copyright production for literary, domestic, musical and artistic works,
the subject dealt with need not to be original, nor the ideas expressed be something novel.
What is required is the expenditure of original skill or labour in execution and not
originality of thought.
26. In Agarwala Publishing House v.
@page-SC836
Board of High School and Intermediate Education and Another, AIR 1967 All 91, a writ
petition was filed by a publisher firm challenging an amendment of the Regulations of the
Board declaring that copyright of the question papers set at all examinations conducted
by the Board shall vest in the Board and forbidding the publication of such question
papers without the Board's permission. The question involved in the case was whether the
question papers are "original literary work" and come within the purview of Section 13 of
the Copyright Act, 1957. It was urged that no copyright can exist in examination papers
because they are not "original literary work". It was held that the "original literary
works" referred to in Section 13 of the Copyright Act, 1957, are not confined to the
works of literature as commonly understood. It would include all works expressed in
writing, whether they have any literary merits or not. This is clear from the definition
given in Section 2(o) of the Act which states that literary work includes tables and
compilations. The Court further held that the word "original" used in Section 13 does not
imply any originality of ideas but merely means that the work in question should not be
copied from some other work but should originate in the author, being the product of his
labour and skill.
27. In the case of Gangavishnu Shrikisondas v. Moreshvar Bapuji Hegishte and Others,
ILR 13 Bom 358, the plaintiff, a book seller, in 1984 brought out a new and annotated
edition of a certain well-known Sanskrit work on religious observances entitled "Vrtraj",
having for that purpose obtained the assistance of the pandits, who re-cast and re-
arranged the work, introduced various passages from other old Sanskrit books on the
same subject and added footnotes. Later on, the defendant printed and published an
edition of the same work, the text of which is identical with that of the plaintiff's work,
which moreover contained the same additional pages and the same footnotes, at the same
places, with many slight differences. The foundation of both plaintiff's and defendant's
books is an old Sanskrit work on Hindu ceremonial, which could have been published by
anyone. The copyright claimed by the plaintiff was on the additions and alterations to the
original text, which the parties admit to be material and valuable, and in which the
copyright is claimed of its prior publication. The defendants argued that there was
nothing really original in the plaintiff’s book and, therefore, he was not entitled to
copyright in the book. It was held by the Court that a new arrangement of old matters will
give a right to the protection afforded by the law of copyright. If anyone by pains and
labour collects and reduces it as a systematic compilation in the form of a book it is
original in the sense that that entitles the plaintiff to the copyright. The plaintiff worked
for such a new arrangement of old matters as to be an original work and was entitled to
the protection; and that as the defendants had not gone to independent sources of the
material but had pirated the plaintiff's work, they were restrained by injunction.
28. In Rai Toys Industries and Others v. Munir Printing Press, 1982 PTC 85, the plaintiff
had published a Tambola ticket book containing 1500 different tickets in 1929. The
plaintiffs alleged that the defendants had brought out another ticket book which the
plaintiffs claimed to have written in 1929 and registered as copyright. The ticket book
brought out by the defendants was alleged to contain 600 different tickets and the same
had been copied identically from the books of the plaintiff. On this basis, a suit for
injunction and rendition of account was filed by the plaintiff. The question before the
court was whether the ticket-books in the form of tables constitute literary work; and
whether copyright has been violated or not? It was held by the High Court that
preparation of tickets and placing them in tables required a good deal of skill and labour
and would thus satisfy the test of being original literary work. It was recognized that the
arrangement of numbers is individual work of a person who prepares it; it bears his
individuality and long hours of labour. It is not information which could be picked up by
all and sundry. The preparation of tickets is an individualized contribution and the
compilation eminently satisfies the test of being an original literary work. Hence it was
held to be a clear case of copyright violation when the defendant decided to pick and
choose 600 tables on the sly and publish them as his individual work.
29. In Macmillan and Another v. Suresh Chandra Deb, ILR 17 Cal 952, the plaintiffs
were proprietors of the copyright of a selection of songs and poems composed by various
authors, which was published in 1861. In 1889, the defendants published a book
@page-SC837
containing same selection of poems and songs as was contained in plaintiffs book, the
arrangement, however, being different. The plaintiffs claimed copyright in the selection
made by them. The defendants, on the other hand, contended that there could be no
copyright in such selection. The Court held that in the case of works not original in the
proper sense of the term, but composed of, or compiled or prepared from material which
are open to all, the fact that one man has produced such a work does not take away from
any one else the right to produce another work of the same kind, and in doing so to use all
the materials open to him. But, as the law is concisely stated by Hall, V.C., in Hogg v
Scott, L.R. 18 Eq. 444, "the true principle in all these cases is, that the defendant is not at
liberty to use or avail himself of the labour which the plaintiff has been at for the purpose
of producing his work, that is, in fact, merely to take away the result of another man's
labour, or, in other words, his property". It is enough to say that this principle has been
applied to maps, to road books, to guide books, to compilations on scientific and other
subjects. This principle seems to be clearly applicable to the case of a selection of a
poem. It was held that for such a selection as the plaintiff had made obviously required
extensive reading, careful studying and comparison and the exercise of taste and
judgment to make a selection for himself. But, if one spares himself this trouble and
adopts some other person's selection, he offends against the principle. The Court was of
the opinion that the selection of poems made by the plaintiff and embodied in the Golden
Treasury was the subject of copyright and that the defendant's book had infringed that
right.
30. These decisions are the authority on the proposition that the work that has been
originated from an author and is more than a mere copy of the original work, would be
sufficient to generate copyright. This approach is consistent with the "sweat of the brow"
standards of originality. The creation of the work which has resulted from little bit of
skill, labour and capital are sufficient for a copyright in derivative work of an author.
Decisions propounded a theory that an author deserves to have his or her efforts in
producing a work, rewarded. The work of an author need not be in an original form or
novel form, but it should not be copied from another's work, that is, it should originate
from the author. The originality requirement in derivative work is that it should originate
from the author by application of substantial degree of skill, industry or experience.
Precondition to copyright is that work must be produced independently and not copied
from another person. Where a compilation is produced from the original work, the
compilation is more than simply a re-arranged copyright of original, which is often
referred to as skill, judgment and or labour or capital. The copyright has nothing to do
with originality or literary merit. Copyrighted material is that what is created by the
author by his skill, labour and investment of capital, may be it is derivative work. The
courts have only to evaluate whether derivative work is not the end-product of skill,
labour and capital which is trivial or negligible but substantial. The courts need not go
into evaluation of literary merit of derivative work or creativity aspect of the same.
31. Mr. P. N. Lekhi, learned senior counsel appearing for the respondents in C.A. No.
6472/2004 has submitted that the judgment of the court is a government work as defined
under Section 2(k)(iii) and on account of Section 17 (d), the Government in the absence
of any agreement to the contrary be the first owner of the copyright therein. Section
52(1)(q)(iv) provides that the publication of any judgment or order of a court, tribunal or
other judicial authority, unless the reproduction of publication of such judgment or order
is prohibited, would not constitute an infringement of the copyright. Therefore,
publication of the judgments of the apex court by the respondents would not tantamount
to infringement of the copyright of the appellants. It is further urged that the judgments
published in the Supreme Court Cases is nothing but merely a derivative work based
upon the judgments of the court, which lacks originality as it does not depict independent
creation even a modicum of creativity. The inputs put by the appellants is nothing but
expressing an idea which can be expressed in a limited way and as such there cannot be a
copyright. Filling the blanks or gaps by providing names of the parties or citations of the
judgments, both of which are well known and unchangeable parts of that idea, are not
original work. These are not creative at all to warrant copyright protection, either singly
or in combination. The additions made in the reported
@page-SC838
judgment by the editors of the Supreme Court Cases are only the well known extensions
of the reported decision. These extensions lack even the minimal degree of author's
creativity or originality or intellectual labour. These additions do not create additional
knowledge, the protection of which is the very basis of the copyright protection.
32. It is submitted by Ms. Pratibha M. Singh, learned counsel for the respondents in C.A.
No. 6905/2004, that in the present case, the journals of the appellants, including SCC, are
printed and published on the basis of pre-existing judgments. Journals are, therefore, a
derivative work. There is a distinction between a "law report" as understood in England
and a "law journal" as printed in India. The appellants" journal" SCC is not a law report
in the strict sense, inasmuch as the appellants journal reproduces the judgments of the
court verbatim along with inputs. However, a law report known in the traditional English
sense is when a law reporter present in the court would record in his own words and
language the arguments of the counsel on both sides, give a summary of the facts and
incorporate into the said report his transcript of the speech of the Judge. Thus, the
appellants work could only be a law journal and not a law report. The judgments were
specifically made a part of the exception to copyright infringement and thus find place in
Section 52(1)(q) of the Act. The underlying purpose is that it is in public interest to place
judgments in public domain. The work for which the copyright protection is claimed is a
derivative work. For claiming protection of copyright in a derivative work, under the
Indian law originality is a pre-condition and originality means only that the work was
independently created by the author as opposed to copied from other works, and that it
possesses at least some minimal degree of creativity. There is a distinction between
creation and discovery. The first person to find a particular fact has not created the fact,
he or she has merely discovered its existence. Reporting of the judgments of the Supreme
Court with certain inputs could only be said to be a discovery of facts already in
existence. Though for the purposes of creativity neither novelty nor invention is requisite
for copyright protection, but at least some minimal creativity is a must. To create a
copyright by alterations of the text, these must be extensive and substantial practically
making a new version. The English decisions relied upon by the appellants would not
apply to the facts of the present case as all the said authorities are under the old 1842 Act
in U.K. wherein the word original was conspicuously missing in the statute. It is further
urged that the copy-editing inputs of the appellants are only discoveries/facts and there
are limited ways/unique of expressing the various copy-editing inputs and thus no
copyright can subsist in such limited/unique expressions. The facts which are discovered
could be expressed in limited ways and as such ways adopted cannot give copyright
protection to the inputs or the judgments as a whole. It is urged that recognizing the
copyright in the copy-edited version of the law reports would amount to giving the
appellants a monopoly in the judgments of the courts which is against the intendment of
Section 52(1)(q)(iv) and would defeat the purpose of putting judgments in the public
domain. It is submitted by the learned counsel for the respondents that for a derivative
work, the originality test as applied in United States Supreme Court should be made
applicable whereby the author of a derivative work would satisfy that the work has been
produced from his exercise of skill and judgment. The exercise of skill and judgment
required to produce the work must not be so trivial that it could be characterized a purely
mechanical exercise. The work should be independently created by the author as opposed
to copied from the other works and that it possesses at least some minimal degree of
creativity. The case law relied upon by the learned counsel for the respondents is
considered hereinafter.
33. In Feist Publications Inc. v. Rural Telephone Service Co. Inc., 18 USPQ 2d 1275,
Rural Telephone Service Co. publishes a typical telephone directory consisting of white
pages and yellow pages. The white pages list in alphabetical order the names of rural
subscribers together with their towns and telephone numbers. The yellow pages list
Rurals business subscribers alphabetically by category and feature classified
advertisements of various sizes. To obtain white pages listings for its area-wide directory,
Feist Publications Inc. approached different telephone companies operating in North West
Kansas and offered to pay for the right to use their white pages listings.
@page-SC839
of them, only Rural refused. Unable to license Rurals white pages listings, Feist used
them without Rural's consent. Rural sued for copyright infringement in the District Court
taking the position that Feist, in compiling its own directory, could not use the
information contained in Rural's white pages. Rural asserted that Feist's employees were
obliged to travel door to door or conduct a telephone survey to discover the same
information for themselves. Feist responded that such efforts were economically
impractical and, in any event, unnecessary because the information copied was beyond
the scope of copyright protection. The United States Supreme Court held that the sine
qua non of copyright is originality. To qualify for copyright protection, a work must be
original to the author. Original, as the term is used in copyright, means only that the work
was independently created by the author (as opposed to copied from other works), and
that it possesses at least some minimal degree of creativity. The requisite level of
creativity is extremely low; even a slight amount will suffice. The vast majority of works
make the grade quite easily, as they possess some creative spark, no matter how crude,
humble or obvious it might be. Originality does not signify novelty; a work may be
original even though it closely resembles other works so long as the similarity is
fortuitous, not the result of copying. The Court further held that no one claim originality
as to the facts. This is because facts do not owe their origin to an act of authorship. The
distinction is one between creation and discovery: the first person to find and report a
particular fact has not created the fact; he or she has merely discovered its existence.
Factual compilations, on the other hand, may possess the requisite originality. The
compilation author typically chooses which facts to include, in what order to place them,
and how to arrange the collected data so that they may be used effectively by readers.
These choices as to selection and arrangement, so long as they are made independently
by the compiler and entail a minimal degree of creativity, are sufficiently original. Thus,
if the compilation author clothes facts with an original collocation of words, he or she
may be able to claim a copyright in this written expression. The Court goes on to hold
that the primary objective of copyright is not to reward the labour of authors, but to
promote the progress of science and useful arts. To this end, copyright assures authors the
right to their original expression but encourages others to build freely upon the ideas and
information conveyed by a work. Only the compiler's selection and arrangement may be
protected; however, the raw facts may be copied at will. The Court rejected the doctrine
of the "sweat of the brow" as this doctrine had numerous flaws, the most glaring being
that it extended copyright protection in a compilation beyond selection and arrangement -
the compiler's original contributions - to the facts themselves. A subsequent compiler was
not entitled to take one word of information previously published, but rather had to
independently work out the matter for himself, so as to arrive at the same result from the
same common sources of information. "Sweat of the brow" courts thereby eschewed the
most fundamental axiom of copyright law that no one may copyright facts or ideas. The
"sweat of the brow" doctrine flouted basic copyright principles and it creates a monopoly
in public domain materials without the necessary justification of protecting and
encouraging the creation of writings by authors.
34. The judgment in Matthew Bender and Co., Inc. v. West Publishing Co., 158 F.3d 674
(2nd Cir. 1998), is of United States Court of Appeals, Second Circuit, which directly
covers the reports of the judgments of the courts. The facts involved in the case are that
the West Publishing Co. and West Publishing Corp. ("West") obtain the text of judicial
opinions directly from courts. It alters these texts into (i) independently composed
features, such as syllabus, headnotes which summarize the specific points of law recited
in each opinion and key numbers which categorize points of law into different legal
topics and sub-topics and (ii) additions of certain factual information to the text of the
opinions, including parallel or alternative citations to cases, attorney information, and
data on subsequent procedural history. West publishes the case reports in different series
of case reporters collectively known as "National Reporter System". Two series of case
reporters at issue in that case were the Supreme Court Reporter and the Federal Reporter.
HyperLaw publishes and markets CD-ROMs which are compilations of the Supreme
Court and the United States Court of Appeals that cover approximately the same ground.
HyperLaw intends to expand
@page-SC840
its CD-ROM product taking the material from the West publications. HyperLaw
intervened and sought a judgment declaring that the individual West case reports that are
left after redaction of the first category of alterations do not contain copyrightable
material. It was held by the Court that for copyright protection, the material does not
require novelty or invention, but minimal creativity is required. All of West's alterations
to judicial opinions involve the addition and arrangement of facts, or the rearrangement
of data already included in the opinions, and, therefore, any creativity in these elements
of West's case reports lies in West's selection and arrangement of this information. West's
choices on selection and arrangement can reasonably be viewed as obvious, typical and
lacking even minimal creativity. Copyright protection is unavailable for both derivative
works and compilations alike unless, when analysed as a whole, they display sufficient
originality so as to amount to an original work of authorship. Originality requires only
that the author makes the selection or arrangement independently and that it displays
some material with minimal level of creativity. While a copy of something in the public
domain will not, if it be merely a copy, support a copyright, a distinguishable variation
will. To support a copyright there must be at least some substantial variation, not merely a
trivial variation such as might occur in the translation to a different medium. Creativity in
selection and arrangement, therefore, is a function of (i) the total number of options
available, (ii) external factors that limit the viability of certain options and render others
non-creative, and (iii) prior uses that render certain selections "garden variety".
35. In the case of Key Publications, Inc. v. Chinatown Today Publishing Enterprises, Inc.,
945 F 2d 509, Key Publication published an Annual Classified Business Directory for
New York City's Chinese-American community. In 1990, Galore Publication published
the Galore Directory, a classified directory for the New York Chinese American
community. Key brought a suit against Galore Directory charging that Galore Directory
infringed Key's copyright in the 1989-90 Key Directory. The United States Court of
Appeal held that individual components of compilation are generally within the public
domain and thus available for public. There are three requirements for a compilation to
qualify for copyright protection : (1) the collection and assembly of pre-existing data; (2)
selection, co-ordination or arrangement of the data; and (3) the resulting work that comes
into being is original, by virtue of the selection, co-ordination or arrangement of the data
contained in the work. For originality, the work is not required to contain novelty. The
doctrine of "sweat of the brow", rewarded compilers for their efforts in collecting facts
with a de facto copyright to those facts and this doctrine would prevent, preclude the
author absolutely from saving time and effort by referring to and relying upon prior
published material. It extended copyright protection in compilation beyond selection and
arrangement - the compiler's original contribution - to the facts themselves drawn on
"sweat of the brow" is a copyright protection to the facts discovered by the compiler. The
court discarded "sweat of the brow" notion of copyright law.
36. In Macmillan and Company v. K. and J. Cooper, 1924 Privy Council 75, action was
brought by McMillan and Company to restrain the respondent-firm who was carrying on
the trade and business of publishers of educational books, from printing, distributing or
otherwise disposing of copies of the book published by the appellants. The ground on
which the relief was claimed was that the appellants had a copyright in the book entitled
Plutarch's Life of Alexander, Sir Thomas North's Translation and that the respondent
published subsequently a book entitled Plutarch's Life of Alexander the Great, North's
Translation", as it had infringed the copyright to which the appellants were entitled in the
earlier compilation. The Court noted the contents of the book of the appellants as also
that of the respondent. As per the Court, the text of the appellants book consisted of a
number of detached passages, selected from Sir Thomas North's translation, words being
in some instances introduced to knit the passages together so that the text should as far as
possible, present the form of an unbroken narrative. The passages so selected were, in the
original translation, by no means contiguous. Considerable printed matter in many
instances separated the one from the other. The opinion of the Privy Council was that for
the work done by the appellants, great knowledge, sound judgment, literary skill or taste
in the inputs brought to bear upon the
@page-SC841
translation was not required, as the passages of the translation which had been selected
are reprinted in their original form, not condensed, expanded, modified or reshaped to
any extent whatever. The Court observed that the North's translation of Plutarch's Life of
Alexander does not and never did, as the law stands, never can enjoy the protection of
copyright; and the questions which arise for decision must be dealt with upon that
assumption. The Court said that in all cases where the reprint with the text of it consisted
merely of a reprint of passages selected from the work of any author, would never have a
copyright. There may be cases where selecting and reprinting the passages would require
the appreciation upon what has been laid down or established in the book and labour,
accurate scientific knowledge, sound judgment, touching the purpose for which the
selection is made, and literary skill would all be needed to effect the object in view. In
such a case, the copyright might well be acquired for the print of the selected passages.
The Court said that it is the product of the labour, skill and capital of one man which must
not be appropriated by another, not the elements, the raw material, upon which the labour
and skill and capital of the first have been expended. To secure copyright for this product,
it is necessary that the labour, skill and capital expended should be sufficient to impart to
the product some quality or character which the raw material did not possess and which
differentiates the product from the raw material. The Court approved the principles
enunciated in the case of University of London Press, Ltd. v. University Tutorial Press,
Ltd., [1916] 2 Ch. 601, dealing with the meaning of the words "original literary work"
that the original does not mean expression of original or inventive thought. The
Copyright Act is not concerned with the original ideas, but with the expression of
thought. The originality which is required relates to expression of thought and the Act
does not require that the expression must be in original or novel form. The work must not
be copied from another - work that it should originate from the author.
37. The Supreme Court of Canada in the matter of CCH Canadian Ltd. v. Law Society of
Upper Canada, 2004 (1) SCR 339 (Canada) has noticed the competing views on the
meaning of "original" in copyright law wherein some courts have held that a work which
has originated from an author and is more than a mere copy of a work, is sufficient to
give copyright. This approach is held to be consistent with the "sweat of the brow" or
"industriousness" standard of originality on the premise that an author deserves to have
his or her efforts in producing a work rewarded. Whereas the other courts have held that a
work must be creative to be original and thus protected by the copyright Act, which
approach is consistent with a natural rights theory of property law; however, it is less
absolute in that only those works that are the product of creativity will be rewarded with
copyright protection and it was suggested in those decisions that the creativity approach
to originality helps ensure that copyright protection is extended to the expression of ideas
as opposed to the underlying ideas or facts. The Court has also noticed that those cases
which had adopted the sweat of the brow approach to originality should not be interpreted
as concluding that labour, in and of itself, would be a ground for finding of originality.
The question for consideration of the copyright has arisen on the following fact
foundation. The appellant, Law Society of Upper Canada, has maintained and operated
the Great Library at Osgoode Hall in Toranto, a reference and research library. The Great
Library provides a request-based photocopy service for Law Society members, the
judiciary and other authorized researchers. Under the custom photocopy service, legal
materials are reproduced and delivered to the requesters. The Law Society also maintains
self-service photocopiers in the Great Library for use by its patrons. The respondents,
CCH Canadian Ltd., Thomson Canada Ltd. and Canada Law Book Inc. publish law
reports and other legal materials. The law book publishers commenced copyright
infringement action against the Law Society claiming ownership of copyright in 11
specific works on the ground that the Law Society had infringed copyright when the
Great Library reproduced a copy of each of the works. The publishers further sought
permanent injunction prohibiting the Law Society from reproducing these 11 works as
well as any other works that they published. The Law Society denied liability and
submitted that the copyright is not infringed when a single copy of a reported decision,
case summary, statute, regulation or a limited selection of text from a treatise is made by
the Great Library staff
@page-SC842
or one of its patrons on a self-service photocopier for the purpose of research. The Court
was called upon to decide the question as to what shall be the originality in the work of
compilation. On consideration of various cases, it was held that to be original under the
Copyright Act the work must originate from an author, not be copied from another work,
and must be the product of an authors exercise of skill and judgment. The exercise of skill
and judgment required to produce the work must not be so trivial that it could be
characterized as a purely mechanical exercise. Creative works by definition are original
and are protected by copyright, but creativity is not required in order to render a work
original. The original work should be the product of an exercise of skill and judgment and
it is a workable yet fair standard. The sweat of the brow approach to originality is too low
a standard which shifts the balance of copyright protection too far in favour of the
owner’s right, and fails to allow copyright to protect the publics interest in maximizing
the production and dissemination of intellectual works. On the other hand, the creativity
standard of originality is too high. A creative standard implies that something must be
novel or non-obvious - concepts more properly associated with patent law than copyright
law. By way of contrast, a standard requiring the exercise of skill and judgment in the
production of a work avoids these difficulties and provides a workable and appropriate
standard for copyright protection that is consistent with the policy of the objectives of the
Copyright Act. Thus, the Canadian Supreme Court is of the view that to claim copyright
in a compilation, the author must produce a material with exercise of his skill and
judgment which may not be creativity in the sense that it is not novel or non-obvious, but
at the same time it is not the product of merely labour and capital.
38. It is the admitted position that the reports in the Supreme Court Cases (SCC) of the
judgments of the Supreme Court is a derivative work in public domain. By virtue of
Section 52(1) of the Act, it is expressly provided that certain acts enumerated therein
shall not constitute an infringement of copyright. Sub-clause (iv) of clause (q) of Section
52(1) excludes the reproduction or publication of any judgment or order of a Court,
Tribunal or other judicial authority, unless the reproduction or publication of such
judgment or order is prohibited by the Court, the Tribunal or other judicial authority from
copyright. The judicial pronouncements of the Apex Court would be in the public domain
and its reproduction or publication would not infringe the copyright. That being the
position, the copy-edited judgments would not satisfy the copyright merely by
establishing amount of skill, labour and capital put in the inputs of the copy-edited
judgments and the original or innovative thoughts for the creativity are completely
excluded. Accordingly, original or innovative thoughts are necessary to establish
copyright in the author’s work. The principle where there is common source the person
relying on it must prove that he actually went to the common source from where he
borrowed the material, employing his own skill, labour and brain and he did not copy,
would not apply to the judgments of the courts because there is no copyright in the
judgments of the court, unless so made by the court itself. To secure a copyright for the
judgments delivered by the court, it is necessary that the labour, skill and capital invested
should be sufficient to communicate or impart to the judgment printed in SCC some
quality or character which the original judgment does not possess and which
differentiates the original judgment from the printed one. The Copyright Act is not
concerned with the original idea but with the expression of thought. Copyright has
nothing to do with originality or literary merit. Copyrighted material is that what is
created by the author by his own skill, labour and investment of capital, may be it is a
derivative work which gives a flavour of creativity. The copyright work which comes into
being should be original in the sense that by virtue of selection, co-ordination or
arrangement of pre-existing data contained in the work, a work somewhat different in
character is produced by the author. On the face of the provisions of the Indian Copyright
Act, 1957, we think that the principle laid down by the Canadian Court would be
applicable in copyright of the judgments of the Apex Court. We make it clear that the
decision of ours would be confined to the judgments of the courts which are in the public
domain as by virtue of Section 52 of the Act there is no copyright in the original text of
the judgments. To claim copyright in a compilation, the author must produce the material
with
@page-SC843
exercise of his skill and judgment which may not be creativity in the sense that it is novel
or non-obvious, but at the same time it is not a product of merely labour and capital. The
derivative work produced by the author must have some distinguishable features and
flavour to raw text of the judgments delivered by the Court. The trivial variation or inputs
put in the judgment would not satisfy the test of copyright of an author.
39. On this touchstone, we shall take into consideration the inputs put by the appellants in
their journal "SCC". The appellants have added in the copy-edited version the cross-
citations to the citation(s) already given in the original text; added names of cases and
cross-citations where only the citation of the case is given; added citation and cross-
citations where only name of the case is given; inserted citation in case history where
only the title and year of the impugned/earlier order is given; presented in their own style
the cases when they are cited repeated in the judgment; provided precise references to the
quoted matter in the judgment by giving exact page and paragraph number as in the
original case source/treatise/reference material; added margin headings to quoted extracts
from statutes/rules, etc., when they are missing from the original text of the judgment;
added the number of the Section/Rule/Article/paragraph to the extract quoted in the
original text; added the names of Judges on whose behalf opinion given by giving
expressions such as "for himself and Pathak, C.J." etc.; done verification of first word of
the quoted extract and supplied emphasis on such verification; added ellipsis "...." to
indicate breaks in quoted extract; provided and supplied the matter inadvertently missed
in quoted extracts in the original text of the judgment; completed/corrected the
incomplete/incorrect case names or citations; renumbered correctly the clauses/sub-
clauses in terms of the questions framed which were numbered in terms of answers to
questions framed by learned Judge; changed the text as per corrigenda issued, which has
been issued upon SCC Editors request and suggestions; done compressing/simplification
of information relating to the case history; followed certain norms at SCC for giving case
names; omitted the words like "Section", "Sec.", "Rule", etc. and given only the number
of the Section/rule at the beginning of the quoted extract; made margin heading and the
first clause/sub-section or initial matter of section/rule etc. to run-on instead of being let
to start from a fresh line; done compressing of unquoted referends and use of *** for
parts; replaced the series of dots in the raw text with ellipsis; removed abbreviations such
as sec., R., cl. and substituted them with full word, i.e. Section, Rule, clause; added
hyphenation after the section/rule numbers which have alphabets suffixed to them;
applied indentation of quoted extracts; removed full stops or word "No." ; and given full
forms of abbreviations to enhance readability and clarity. In addition to the above,
capitalization and italicization is also made wherever necessary in the raw text; and
punctuation, articles, spellings and compound words are also checked and corrected, if
required, in the original text.
40. The aforesaid inputs put by the appellants in the judgments would have had a
copyright had we accepted the principle that any one who by his or her own skill and
labour creates an original work of whatever character, shall enjoy an exclusive right to
copy that work and no one else would be permitted to reap the crop what the copyright
owner had sown. No doubt the appellants have collected the material and improved the
readability of the judgment by putting inputs in the original text of the judgment by
considerable labour and arranged it in their own style, but that does not give the flavour
of minimum requirement of creativity. The exercise of the skill and judgment required to
produce the work is trivial and is on account of the labour and the capital invested and
could be characterized as purely a work which has been brought about by putting some
amount of labour by the appellants. Although for establishing a copyright, the creativity
standard applies is not that something must be novel or non-obvious, but some amount of
creativity in the work to claim a copyright is required. It does require a minimal degree
of creativity. Arrangement of the facts or data or the case law is already included in the
judgment of the court. Therefore, creativity of SCC would only be addition of certain
facts or material already published, case law published in another law report and its own
arrangement and presentation of the judgment of the court in its own style to make it
more user- friendly. The selection and arrangement can be viewed as typical and at best
result of the labour, skill and investment of capital
@page-SC844
lacking even minimal creativity. It does not as a whole display sufficient originality so as
to amount to an original work of the author. To support copyright, there must be some
substantive variation and not merely a trivial variation, not the variation of the type where
limited ways/unique of expression available and an author selects one of them which can
be said to be a garden variety. Novelty or invention or innovative idea is not the
requirement for protection of copyright but it does require minimal degree of creativity.
In our view, the aforesaid inputs put by the appellants in the copy-edited judgments do
not touch the standard of creativity required for the copyright.
41. However, the inputs put in the original text by the appellants in (i) segregating the
existing paragraphs in the original text by breaking them into separate paragraphs; (ii)
adding internal paragraph numbering within a judgment after providing uniform
paragraph numbering to the multiple judgments; and (iii) indicating in the judgment the
Judges who have dissented or concurred by introducing the phrases like "concurring",
"partly concurring", "partly dissenting", "dissenting", "supplementing", "majority
expressing no opinion", etc., have to be viewed in a different light. The task of paragraph
numbering and internal referencing requires skill and judgment in great measure. The
editor who inserts para numbering must know how legal argumentation and legal
discourse is conducted and how a judgment of a court of law must read. Often legal
arguments or conclusions are either clubbed into one paragraph in the original judgment
or parts of the same argument are given in separate paragraphs. It requires judgment and
the capacity for discernment for determining whether to carve out a separate paragraph
from an existing paragraph in the original judgment or to club together separate
paragraphs in the original judgment of the court. Setting of paragraphs by the appellants
of their own in the judgment entailed the exercise of the brain work, reading and
understanding of subject of disputes, different issues involved, statutory provisions
applicable and interpretation of the same and then dividing them in different paragraphs
so that chain of thoughts and process of statement of facts and the application of law
relevant to the topic discussed is not disturbed, would require full understanding of the
entire subject of the judgment. Making paragraphs in a judgment could not be called a
mechanical process. It requires careful consideration, discernment and choice and thus it
can be called as a work of an author. Creation of paragraphs would obviously require
extensive reading, careful study of subject and the exercise of judgment to make
paragraph which has dealt with particular aspect of the case, and separating intermixing
of a different subject. Creation of paragraphs by separating them from the passage would
require knowledge, sound judgment and legal skill. In our opinion, this exercise and
creation thereof has a flavour of minimum amount of creativity. The said principle would
also apply when the editor has put an input whereby different Judges opinion has been
shown to have been dissenting or partly dissenting or concurring, etc. It also requires
reading of the whole judgment and understanding the questions involved and thereafter
finding out whether the Judges have disagreed or have the dissenting opinion or they are
partially disagreeing and partially agreeing to the view on a particular law point or even
on facts. In these inputs put in by the appellants in the judgments reported in SCC, the
appellants have a copyright and nobody is permitted to utilize the same.
42. For the reasons stated in the aforesaid discussion, the appeals are partly allowed. The
High Court has already granted interim relief to the plaintiff-appellants by directing that
though the respondent-defendants shall be entitled to sell their CD-ROMS with the text
of the judgments of the Supreme Court along with their own head notes, editorial notes, if
any, they should not in any way copy the head notes of the plaintiff-appellants; and that
the defendant-respondents shall also not copy the footnotes and editorial notes appearing
in the journal of the plaintiff-appellants. It is further directed by us that the defendant-
respondents shall not use the paragraphs made by the appellants in their copy-edited
version for internal references and their editors judgment regarding the opinions
expressed by the Judges by using phrases like "concurring", "partly dissenting", etc. on
the basis of reported judgments in SCC. The judgment of the High Court is modified to
the extent that in addition to the interim relief already granted by the High Court, we have
granted the above-mentioned additional relief to the appellants.
@page-SC845
43. In view of the decision rendered by us in the civil appeals, we do not think it
necessary to pass any order on the contempt petition. The contempt petition stands
disposed of accordingly.
44.There shall be no order as to costs.
Order accordingly.
AIR 2008 SUPREME COURT 845 "National Insurance Co. Ltd. v. Indira Srivastava"
(From : 2007 (6) ALJ (DOC) 300 : 2007 (3) All WC 2605)
Coram : 2 S. B. SINHA AND H. S. BEDI, JJ.
Civil Appeal No. 5830 of 2007 (arising out of SLP (C) No. 14452 of 2007) D/- 12 -12
-2007.
National Insurance Co. Ltd. v. Indira Srivastava and Ors.
Motor Vehicles Act (59 of 1988), S.168 - MOTOR VEHICLES - Accident compensation
- "Income" of victim - Not only pay packet but perks which are beneficial to his family -
Must be considered.
2004 ACC 533 (SC), Held Per incuriam.
Section 168 uses the word ‘just compensation" which should be assigned a broad
meaning. The term "income" has different connotations for different purposes. A Court of
law, having regard to the change in societal conditions must consider the question not
only having regard to pay packet the employee carries home at the end of the month but
also other perks which are beneficial to the members of the entire family. In determining
what constitutes income it cannot be lost sight that the private sector companies in place
of introducing a pension scheme takes recourse to payment of contributory Provident
Fund, Gratuity and other perks to attract the people who are efficient and hard working.
Different offers made to an officer by the employer, same may be either for the benefit of
the employee himself or for the benefit of the entire family. If some facilities are being
provided whereby the entire family stands to benefit, the same, must be held to be
relevant for the purpose of computation of total income on the basis whereof the amount
of compensation payable for the death of the kith and kin of the applicants is required to
be determined. 2004 ACC 533 held per incuriam. (Paras 8, 9)
The amounts, which were required to be paid to the deceased by his employer by way of
perks, should be included for computation of his monthly income as that would have
been added to his monthly income by way of contribution to the family as
contradistinguished to the ones which were for his benefit. (Para 17)
Cases Referred : Chronological Paras
2007 AIR SCW 1316 : AIR 2007 SC 1243 : 2007 (2) ALJ 766 (Ref.) 22
2007 AIHC 1921 (Mad) (Approved) 13
2005 AIR SCW 1801 : AIR 2005 SC 2157 22
2005 AIR SCW 2542 : AIR 2005 SC 2985 (Ref.) 21
2004 (1) ACC 533 (SC) (Held per Incuriam) 15, 16
AIR 2002 (NOC) 211 : 2002 AIHC 2633 (AP) (Approved) 14
2001 AIR SCW 1074 : AIR 2001 SC 1333 16
(1999)1 TAC 8497 (MP) 3
1998 AIR SCW 3105 : AIR 1998 SC 3191 14
AIR 1979 SC 1666 14
AIR 1985 SC 106 (Rel. on) 20
AIR 1930 Mad 626 18
AIR 1921 Mad 427 (SB) 18
51 LJ Ch 938 18
42 LJ Ch 336 18
Joy Basu, B. K. Satija, for Appellant; L. N. Rao, Sr. Advocate (A. C.) for Respondent,
Ms. Indira Srivastava, Caveator in person.
Judgement
S. B. SINHA, J. :- Leave granted.
2. Connotation of the term "income" for the purpose of determination of just
compensation envisaged under Section 168 of the Motor Vehicles Act, 1988 (the Act)
calls for question in this appeal which arises out of a judgment and order dated 6.4.2007
passed by the High Court of Judicature at Allahabad, Lucknow Bench at Lucknow in
FAFO No.171 of 2001. Respondent's husband R.K. Srivastava was employed in a
company named Gabriel India Ltd. While he was travelling in an auto rickshaw from
Charbagh Railway Station, Lucknow to his residence situated at Ashok Marg, the same
met with an accident with a Mahindra Commander Jeep driven rashly and negligently. He
sustained injuries and ultimately succumbed thereto. Respondents herein filed a claim
petition before the learned Tribunal. A salary certificate was produced in the said
proceedings which is in the following terms:
@page-SC846

Earnings Amount Deductions Amount


Basic 3420.00 CPF(S) 488.00
Special Pay 70.00 CPF (Add)
FDA 350.00 GIS 3.75
VDA 1040.00 LIC/GIS 509.10
CCA 100.00 HRR
HRA 1047.00 MSPI 60.00
Washing All. 75.00 Society 576.00
Conv. 225.00 Union 3.00
Cant.sub. 265.00 HBA 340.00
C.E.A. 2040.00 B.Fund10.00
Total 8632.00 Total 1989.85

3. The learned Tribunal opined that in computing his income, the element of conveyance
allowance only would fall outside the purview of income. On the aforementioned basis,
the monthly income of the deceased was assessed at Rs.20364/-. Applying the multiplier
of 13, as the age of the deceased was 45 years, it was held :
As such, on using multiple of 13 to the annual income of deceased at Rs.2,32,372/?, the
amount works out to Rs.30,20,836/-. The deceased would have spent 1/3rd of this amount
on himself, hence on deducting 1/3rd from this amount, 2/3rd compensation amount
comes to Rs.20,13,890/-."
It was concluded :
"Considering all these facts, I reach to this finding that the petitioners are entitled to get
2/3rd of the total income of deceased worked out by using multiple of 13 i.e. about
Rs.20,00,000/-. Issue No.5 is decided accordingly. It is the liability of opposite party No.3
Insurance Company. On behalf of opposite party No.3, the ruling of Hon'ble High Court
Smt. Lalta Devi Vs. Suresh and Ors., T.A.C. 8, 1999 (1) page 847 has been filed before
me, but this ruling does not extend any specific benefit to opposite party No.3. Hence,
while deciding this issue No.5, I come to this conclusion that the petitioners are entitled
to get Rs.20,00,000/- (Rs. Twenty Lakhs) as compensation."
4. The High Court, on an appeal having been preferred both by the appellant as also the
respondents, partly allowed the same by a common judgment holding that claimants were
entitled to compensation calculated in case of the deceased at Rs.19,53,224/- along with
interest @ 9% from the date of presentation of the claim petition till its realization,
holding that travelling reimbursement could not be taken into consideration for
computation of net income of the deceased.
5. Appellant is, thus, before us.
Keeping in view the importance of the question involved and furthermore in view of the
fact that the first respondent was appearing-in- person, we had requested Mr. L.N. Rao,
learned senior counsel, to assist us in the matter.
6. Submission of Mr. Satija, learned counsel appearing on behalf of the appellant, is that
for the purpose of computation of the amount of compensation what was material is the
basic pay and not other allowances and, in that view of the matter, the High Court has
committed a serious error in opining otherwise. The learned counsel contended that
emphasis by this Court are being laid on computation of damages based on net income
and not gross income. It was also contended that in any event the amount of
compensation awarded by the High Court is on higher side.
7. Mr. Rao, however, submitted that apart from the basic salary, contributions made by
the employee should also be taken into consideration for calculation of the amount of
compensation, inter alia, on the premise that the same would have become payable to him
at a future date as, for example, voluntary retirement, superannuation etc. which would be
beneficial to the entire family. It was pointed out that the contributions towards Provident
Fund, Life Insurance Corporation, gratuity etc. are includible in the definition of income.
8. The term "income" has different connotations for different purposes. A court of law,
having regard to the change in societal conditions must consider the question not only
having regard to pay packet the employee carries home at the end of the month but also
other perks which are beneficial to the members of the entire family. Loss caused to the
family on a death of a near and dear one can hardly be compensated on monetary terms.
9. Section 168 of the Act uses the word "just compensation" which, in our opinion,
should be assigned a broad meaning. We cannot, in determining the issue involved in the
matter, lose sight of the fact that the private sector companies in place of introducing a
pension scheme takes recourse to payment of contributory Provident Fund, Gratuity and
other perks to attract the
@page-SC847
people who are efficient and hard working. Different offers made to an officer by the
employer, same may be either for the benefit of the employee himself or for the benefit of
the entire family. If some facilities are being provided whereby the entire family stands to
benefit, the same, in our opinion, must be held to be relevant for the purpose of
computation of total income on the basis whereof the amount of compensation payable
for the death of the kith and kin of the applicants is required to be determined. For the
aforementioned purpose, we may notice the elements of pay, paid to the deceased :

?BASIC : 63,400.00
CONVEYANCE ALLOWANCE : 12,000.00
RENT CO LEASE : 49,200.00
BONUS (35% OF BASIC) : 21,840.00
___________
TOTAL : 1,45, 440.00
___________

In addition to above, his other entitlements were :

Con. to PF 10% of Basic Rs. 6,240/- (p.a.)


LTA reimbursement Rs. 7,000/- (p.a.)
Medical reimbursement Rs. 6,000/- (p.a.)
Superannuation 15% of BasicRs. 9,360/- (p.a.)
Gratuity Cont.5.34% of BasicRs. 3,332/- (p.a.)
Medical Policy-self and Family @ Rs.55,000/- (p.a.)
Education Scholarship @ Rs.500 Rs.12,000/- (p.a.)
Payable to his two children directly?
10. There are three basic features in the aforementioned statement which require our
consideration :
1. Reimbursement of rent would be equivalent to HRA;
2. Bonus is payable as a part of salary; and
3. Contribution to the Provident Fund.
11. We may furthermore notice that apart therefrom, superannuation benefits,
contributions towards gratuity, insurance of medical policy for self and family and
education scholarship were beneficial to the members of the family.
12. We have, however, no doubt in mind that medical reimbursement which provides for
a slab and which keeping in view the terminology used, would mean reimbursement for
medical expenses on production of medical bills and, thus, the same would not come
within the purview of the aforementioned category.
13

. The question came for consideration before a learned Single Judge of the Madras High
Court in The Manager, National Insurance Co. Ltd. v. Padmavathy and Ors. [CMA
No.114 of 2006 decided on 29.1.2007], wherein it was held : "Income tax, Professional
tax which are deducted from the salaried person goes to the coffers of the government
under specific head and there is no return. Whereas, the General Provident Fund, Special
Provident Fund, L.I.C., Contribution are amounts paid specific heads and the contribution
is always repayable to an employee at the time of voluntary retirement, death or for any
other reason. Such contribution made by the salaried person are deferred payments and
they are savings. The Supreme Court as well as various High Courts have held that the
compensation payable under the Motor Vehicles Act is statutory and that the deferred
payments made to the employee are contractual. Courts have held that there cannot be
any deductions in the statutory compensation, if the Legal Representatives are entitled to
lump sum payment under the contractual liability. If the contributions made by the
employee which are otherwise savings from the salary are deducted from the gross
income and only the net income is taken for computing the dependency compensation,
then the Legal Representatives of the victim reported in 2007 AIHC 1921

@page-SC848
would lose considerable portion of the income. In view of the settled proposition of law, I
am of the view, the Tribunal can make only statutory deductions such as Income tax and
professional tax and any other contribution, which is not repayable by the employer, from
the salary of the deceased person while determining the monthly income for computing
the dependency compensation. Any contribution made by the employee during his
lifetime, form part of the salary and they should be included in the monthly income,
while computing the dependency compensation."
14

. Similar view was expressed by a learned Single Judge of Andhra Pradesh High Court in
S. Narayanamma and Ors. v. Secretary to Government of India, Ministry of
Telecommunications and Ors. [2002 ACC 582], holding : "In this background, now we
will examine the present deductions made by the tribunal from the salary of the deceased
in fixing the monthly contribution of the deceased to his family. The tribunal has not even
taken proper care while deducting the amounts from the salary of the deceased, at least
the very nature of deductions from the salary of the deceased. My view is that the
deductions made by the tribunal from the salary such as recovery of housing loan, vehicle
loan, festival advance and other deductions, if any, to the benefit of the estate of the
deceased cannot be deducted while computing the net monthly earnings of the deceased.
These advances or loans are part of his salary. So far as House Rent Allowance is
concerned, it is beneficial to the entire family of the deceased during his tenure, but for
his untimely death the claimants are deprived of such benefit which they would have
enjoyed if the deceased is alive. On the other hand, allowances, like Travelling
Allowance, allowance for newspapers/periodicals, telephone, servant, club-fee, car
maintenance etc., by virtue of his vocation need not be included in the salary while
computing the net earnings of the deceased. The finding of the tribunal that the deceased
was getting Rs.1,401/- as net income every month is unsustainable as the deductions
made towards vehicle loan and other deductions were also taken into consideration while
fixing the monthly income of the deceased. The above finding of the tribunal is contrary
to the principle of just compensation enunciated by the Supreme Court in the judgment in
Helen's case (1 supra). The Supreme Court in Concord of India Insurance Co. v.
Nirmaladevi and Ors., 1980 ACJ 55 (SC) held that determination of quantum must be
liberal and not niggardly since law values life and limb in a free country ‘in generous
scales." AIR 2002 (NOC) 211
1998 AIR SCW 3103
AIR 1979 SC 1666

15. We may, however, notice that a Division Bench of this Court in Asha and Ors. v.
United Indian Insurance Co. Ltd. and Anr. [2004 ACC 533], whereupon reliance has been
placed by Mr. Satija, was considering a case where, like the present one, several perks
were included in salary. We may reproduce the salary certificate hereto below :
This is to certify that Shri A.M. Raikar was working as AG 111 in this organisation has
been paid the following Pay and Allowances for the month of May, 1995 :

Earnings Amount Deductions Amount


Basic 3420.00 CPF(S) 488.00
Special Pay 70.00 CPF (Add)
FDA 350.00 GIS 3.75
VDA 1040.00 LIC/GIS 509.10
CCA 100.00 HRA
HRA 1047.00 MSPI 160.00
Washing All. 75.00 Society 576.00
Conv. 225.00 Union 3.00
Cant.sub. 265.00 HBA 340.00
C.E.A. 2040.00 B.Fund10.00
Total 8632.00 Total 1989.85

Net Payable Rs. 6642.00 (Rupees six thousand six hundred forty two only)."
In that case, this Court held :
Lastly it was submitted that the salary certificate shows that the salary of the deceased
was Rs.8,632/-. It was submitted that the High Court was wrong in taking the salary to be
Rs.6,642/-. It was submitted that the High Court was wrong in deducting the allowances
and amounts paid towards LIC, Society charges and HBA etc. We are unable to accept
this submission also. The claimants are entitled to be compensated for the loss suffered
by them. The loss suffered by them is the amount which they would have been receiving
at the time when the deceased was alive. There can be no doubt that the dependents
would only be receiving the net amount less l/3rd for his personal expenses. The High
Court was therefore right in so holding."
@page-SC849
This Court in Asha (supra) did not address itself the questions raised before us. It does
not appear that any precedent was noticed nor the term just compensation was considered
in the light of the changing societal condition as also the perks which are paid to the
employee which may or may not attract income tax or any other tax.
What would be "just compensation" must be determined having regard to the facts and
circumstances of each case. The basis for considering the entire pay packet is what the
dependents have lost due to death of the deceased. It is in the nature of compensation for
future loss towards the family income.
16

. In Rathi Menon v. Union of India [(2001) 3 SCC 714], this Court, upon considering the
dictionary meaning of compensation held : 2001 AIR SCW 1074

“In this context a reference to Section 129 of the Act appears useful. The Central
Government is empowered by the said provision to make rules by notification "to carry
out the purposes of this Chapter". It is evident that one of the purposes of this chapter is
that the injured victims in railway accidents and untoward incidents must get
compensation. Though the word "compensation" is not defined in the Act or in the Rules
it is the giving of an equivalent or substitute of equivalent value. In Black’s Law
Dictionary , "compensation" is shown as
"equivalent in money for a loss sustained; or giving back an equivalent in either money
which is but the measure of value, or in actual value otherwise conferred; or recompense
in value for some loss, injury or service especially when it is given by statute."
It means when you pay the compensation in terms of money it must represent, on the date
of ordering such payment, the equivalent value.
25. In this context we may look at Section 128(1) also. It says that the right of any person
to claim compensation before the Claims Tribunal as indicated in Section 124 or 124-A
shall not affect the right of any such person to recover compensation payable under any
other law for the time being in force. But there is an interdict that no person shall be
entitled to claim compensation for more than once in respect of the same accident. This
means that the party has two alternatives, one is to avail himself of his civil remedy to
claim compensation based on common law or any other statutory provision, and the other
is to apply before the Claims Tribunal under Section 124 or 124-A of the Act. As he
cannot avail himself of both the remedies he has to choose one between the two. The
provisions in Chapter XIII of the Act are intended to provide a speedier remedy to the
victims of accidents and untoward incidents. If he were to choose the latter that does not
mean that he should be prepared to get a lesser amount. He is given the assurance by the
legislature that the Central Government is saddled with the task of prescribing fair and
just compensation in the Rules from time to time. The provisions are not intended to give
a gain to the Railway Administration but they are meant to afford just and reasonable
compensation to the victims as a speedier measure. If a person files a suit the amount of
compensation will depend upon what the court considers just and reasonable on the date
of determination. Hence when he goes before the Claims Tribunal claiming compensation
the determination of the amount should be as on the date of such determination."
17. The amounts, therefore, which were required to be paid to the deceased by his
employer by way of perks, should be included for computation of his monthly income as
that would have been added to his monthly income by way of contribution to the family
as contradistinguished to the ones which were for his benefit. We may, however, hasten to
add that from the said amount of income, the statutory amount of tax payable thereupon
must be deducted.
18. The term "income" in P. Ramanatha Aiyar's Advanced Law Lexicon (3rd Ed.) has
been defined as under :
"The value of any benefit or perquisite whether convertible into money or not, obtained
from a company either by a director or a person who has substantial interest in the
company, and any sum paid by such company in respect of any obligation, which but for
such payment would have been payable by the director or other person aforesaid,
occurring or arising to a person within the State from any profession, trade or calling
other than agriculture."
It has also been stated :
"INCOME" signifies "what comes in" (per Selborne, C., Jones v. Ogle, 42 LJ Ch.336). "It
is as large a word as can be used" to denote a person's receipts (per Jessel, M.R. Re
@page-SC850
Huggins, 51 LJ Ch.938.) income is not confined to receipts from business only and
means periodical receipts from one's work, lands, investments, etc. AIR 1921 Mad 427
(SB). Ref. 124 IC 511 : 1930 MWN 29 : 31 MLW 438 AIR 1930 Mad 626 : 58 MLJ
337."
19. If the dictionary meaning of the word "income" is taken to its logical conclusion, it
should include those benefits, either in terms of money or otherwise, which are taken into
consideration for the purpose of payment of income-tax or profession tax although some
elements thereof may or may not be taxable or would have been otherwise taxable but for
the exemption conferred thereupon under the statute.
20. In N. Sivammal and Ors. v. Managing Director, Pandian Roadways Corporation and
Ors. [(1985) 1 SCC 18], this Court took into consideration the pay packet of the
deceased.
21

. We may notice that in T. N. State Transport Corporation Ltd. v. S. Rajapriya and Ors.
[(2005) 6 SCC 236], this Court held :2005 AIR SCW 2542

"8. The assessment of damages to compensate the dependants is beset with difficulties
because from the nature of things, it has to take into account many imponderables e.g. the
life expectancy of the deceased and the dependants, the amount that the deceased would
have earned during the remainder of his life, the amount that he would have contributed
to the dependants during that period, the chances that the deceased may not have lived or
the dependants may not live up to the estimated remaining period of their life expectancy,
the chances that the deceased might have got better employment or income or might have
lost his employment or income together.
9. The manner of arriving at the damages is to ascertain the net income of the deceased
available for the support of himself and his dependants, and to deduct therefrom such part
of his income as the deceased was accustomed to spend upon himself, as regards both
self-maintenance and pleasure, and to ascertain what part of his net income the deceased
was accustomed to spend for the benefit of the dependants. Then that should be
capitalised by multiplying it by a figure representing the proper number of years"
purchase.
10. Much of the calculation necessarily remains in the realm of hypothesis and in that
region arithmetic is a good servant but a bad master" since there are so often many
imponderables. In every case it is the overall picture that matters", and the court must try
to assess as best as it can the loss suffered."
22

. Yet again in New India Assurance Co. Ltd. v. Charlie and Anr. [(2005) 10 SCC 720], the
same view was reiterated. However, therein although the words "net income" has been
used but the same itself would ordinarily mean gross income minus the statutory
deductions. We must also notice that the said decision has been followed in New India
Assurance Co. Ltd. v. Kalpana (Smt.) and Ors. [(2007) 3 SCC 538]. 2005 AIR
SCW 1801
2007 AIR SCW 1316

23. The expression just must also be given its logical meaning. Whereas it cannot be a
bonanza or a source of profit but in considering as to what would be just and equitable,
all facts and circumstances must be taken into consideration.
24. In view of our finding abovemention-ed, the appeal is to be allowed in part in so far
as the High Court had directed deduction of medical reimbursement and tax elements on
the entire sum which according to the statute constitute income. But we decline to do so
for two reasons. Firstly, the accident had taken place as far back as on 1st September,
1997 and secondly the Tribunal as also the High Court failed to take into consideration
rise in income of the deceased by way of promotion or otherwise.
25. For the aforementioned reasons, we are not inclined to interfere with the impugned
judgment. This appeal is, therefore, dismissed. In the facts and circumstances of the case,
there shall be no order as to costs.

AIR 2008 SUPREME COURT 850 "Dharmarajan v. Valliammal"


(From : Madras)*
Coram : 2 H. K. SEMA AND V. S. SIRPURKAR, JJ.
Civil Appeal Nos. 4535-4536 of 2001, D/- 11 -12 -2007.
Dharmarajan and Ors. v. Valliammal and Ors.
(A) Civil P.C. (5 of 1908), S.100 - APPEAL - HIGH COURT - Second appeal - Questions
formulated were neither questions of law nor
@page-SC851
substantial questions of law - High Court finding out new case which was not even
pleaded - High Court entering into appreciation of evidence on basis of non-existent
substantial questions of law - Order of High Court liable to be set aside.
S. A. Nos. 2235 and 2236 of 1986, D/- 25-6-1999 (Mad.), Reversed. (Paras 8, 11,
12, 14, 17)
(B) Limitation Act (36 of 1963), Art.64, Art.65 - LIMITATION - ADVERSE
POSSESSION - DOCUMENTS - INJUNCTION - Adverse possession - Possession of
claimant found to be permissive - Legal heirship of person claiming through original
claimant not decisively proved - No plea as to against whom adverse possession was
claimed - Documentary evidence to show that house registered in name of plaintiff and
he was paying house tax - In absence of revenue records in favour of defendant there was
no question of their title over property - Held that, defendant was not entitled to
injunction prayed for on basis of plea of adverse possession. (Paras 8, 11, 16)
Cases Referred : Chronological Paras
2006 AIR SCW 2404 : 2006 AIR SCW 1975 : 2006 (4) AIR Kar R 78 (Foll.) 14
2004 AIHC 4001 (Mad) (Disting.) 12
F. R. Kumar (for M/s. Parekh and Co.), for Appellants; P. Krishnamoorthy, Sr. Advocate,
R. Nedumaran and Sunil Kumar Singh, for Respondents.
* S.A. Nos. 2235 and 2236 of 1986, D/- 25-6-1999 (Mad).
Judgement
1. V. S. SIRPURKAR, J. :-A common judgment passed by Madras High Court allowing
two Second Appeals is in challenge before us. The Single Judge of the Madras High
Court set aside the appellate judgment, again a common one allowing appeals against the
common judgment passed by District Munsiff, Bhawani whereby the District Munsiff
had decreed the suit filed by one Muthuswami Gounder and dismissed the other suit filed
by Dharmarajan, the appellant herein. A short history of the case would be essential.
2. K. Muthuswami Gounder filed a suit registered as O.S. No.555 of 1991 for declaration
and injunction alleging that he had purchased suit property Survey No.324/D1 under a
Sale Deed dated 10-10-1980 from one Doraiswamy who was in possession and
enjoyment of the property. The said Doraiswamy was claimed to be a foster son of one
Karupayee who had expired in the year 1961 and who was claimed to be in possession
and enjoyment of the suit property wherein she had put up a thatched shed and was
residing for more than 30 years. It is claimed that after Karupayee her foster son who was
none else but his sister's son obtained the possession and enjoyed the said suit property.
Before this sale deed dated 10.10.1980, he had executed a Mortgage Deed in respect of
the suit property in favour of the plaintiff Muthuswami Gounder dated 15.6.1980. It was
further claimed that Doraiswamy was permitted to occupy the suit property as tenant on
monthly rent of Rs.50/-. It was further asserted that defendants 1 to 7, i.e., the present
appellants had also wanted to purchase the property from Doraiswamy but having failed,
they were falsely claiming certain rights in the suit property by creating some false
documents and that they had no right, title or possession. It was claimed that the plaintiff
and his predecessor, namely, Doraiswamy had acquired the title by adverse possession for
more than 60 years. It is on this basis that Muthuswamy Gounder claimed a decree for
declaration of his ownership as also for the injunction against the present appellants.
3. As against this, the present appellants claimed that this property in fact belonged to
first defendant therein, (the appellant No.1 herein) in so far as the Eastern half of the
property was concerned since it was purchased by the first defendant from one
Venkataramana Iyer. It was claimed that the suit property originally belonged to one K.V.
Krishnasamy and others and they were throughout in possession and enjoyment of the
suit property and were paying house tax also. The other appellants claimed the other half
of the property on the plea that they had purchased the same from the other co-sharer
Venugopal Iyer who had inherited the property from K.V. Krishnasamy and others. It was
claimed that Karuppayee was working as a maid servant under one Venugopala Iyer and
it was he who had permitted her to put up the thatched shed in the suit property and after
the death of Karuppayee, Doraiswamy started working as a servant of Venugopala Iyer
and as such he was in occupation of the thatched salai (house) with the permission of
Venugopala Iyer. It was claimed that the property stood in the name of Venugopala Iyer in
Kavundapady Panchayat. In short it was contended that the present appellants were
@page-SC852
owners of the property which they had purchased on 15.7.1980 and 27.8.1980 vide
different sale deeds. It was further claimed that after the purchase of the suit property, the
present appellants who were the defendants in Suit No.555 of 1981 were paying the taxes
and Doraiswamy was staying in the property with their permission. The original
defendants, the appellants herein stoutly denied the right of ownership on the part of
Doraiswamy to transfer the property in favour of the plaintiff. They also denied that
Karuppayee and after her Doraiswamy were in independent possession of the property.
They also denied that Karuppayee or, as the case may be Doraiswamy, had perfected their
title by adverse possession. Thus, the Appellant No.1 Dharmarajan claimed half of the
property whereas the rest of the appellants claimed the other half of the property being
purchasers from the members of Iyer family.
4. The Appellant No.1 Dharmarajan also filed a suit being OS No.280 of 1982 in respect
of the Eastern one half portion of the suit property of which he claimed the ownership
through the sale deed in the earlier suit. This suit was filed against Valliammal and
Palaniammal, who were the legal heirs of Doraiswamy. It must be stated here that
Doraiswamy had by then expired. This was also a suit for declaration of title of
Dharmarajan. In this suit it was claimed that the property originally belonged to the father
of Krishnasamy Iyer, Kandsamy Iyer and the father of one Vengugopala Iyer. In the
family arrangement the suit property was allotted to the father of Kandasamy Iyer and
Venkatasubramania Iyer, the son of Krishnasamy Iyer and the first appellant Dharmarajan
had purchased the suit property from Venkataramana Iyer on 15.7.1990 who was none
else but the son of Krishnaswami Iyer, both of whom were the heirs of Kandasamy Iyer.
An injunction was also claimed against the defendants. Valliammal and Palaniammal
firstly claimed that one suit was already filed against Doraiswamy being OS No.531 of
1981 and the said suit was dismissed. Doraiswamy had expired on 18.5.1981 and since
Valliammal and Palaniammal were the legal heirs of Doraiswamy, the suit was not
maintainable against them. Both these ladies claimed that they were in possession of the
suit property as the tenants under Muthuswami Gounder, the plaintiff in OS No.555 of
1981. They denied the ownership of the Iyer family on the suit property and claimed that
it was false to allege that the suit property was ever allotted to the father of Kandasamy
Iyer and Venkatasubramania Iyer. It was, therefore, pleaded that the vendors of the
plaintiff- Appellant No.1 herein were not entitled to the suit property and they were never
in possession of the same. It was claimed that the suit property was a poramboke land and
it was throughout in possession of Karuppayee Ammal who had perfected title to the
same by adverse possession. It was only Karuppayee who had put up thatched salai in the
suit property and her successor Doraiswamy was the husband of the first defendant
Valliammal and father of Palaniammal and after the death of Karuppayee Ammal he
continued to be in possession of the suit property as the heir of Karuppayee Ammal.
Karuppayee Ammal had died 20 years ago and after the death Doraiswamy had
mortgaged the suit property to Muthuswamy Gounder on 10.10.1980 and thereafter the
Doraiswamy and defendants 1 and 2 continued to be in possession of the suit property as
tenants of Muthuswamy and on that count the suit was liable to be dismissed.
5. The Trial Court decreed the Suit No.555 of 1981 and dismissed Suit No.280 of 1982
filed by the appellant No.1 in respect of the half of the suit property. Two appeals came to
be filed which were allowed whereby the Appellate Court dismissed Suit No.555 of 1981
and decreed Suit No.280 of 1982 only to the extent of the decree of declaration of title.
However, since the plaintiff therein (the appellant herein) had not terminated the licence
of Valliammal and Palaniammal in respect of the suit property that relief was denied to
the appellant No.1 herein and the suit succeeded only partly. As stated earlier, the plaintiff
Muthuswamy Gounder filed Second Appeal No.2236 of 1986 while Valliammal filed
Second Appeal No.2235 of 1986 which appeals have been allowed by the learned Single
Judge of the High Court and that is how the parties are before us in the present two
appeals.
6. Learned counsel appearing for the appellant seriously criticized the High Court
judgment firstly that the High Court had entered into a prohibited arena of re-appreciation
of evidence. It was contended that the appellate court was the final court of facts and yet
even without discussing the
@page-SC853
appellate court judgment, considering the approach thereof, the High Court had re-
appreciated the evidence and had upset the well considered judgment of the appellate
court. Secondly, the learned counsel urged that an entirely new case which was not even
pleaded by the plaintiff in Suit No.555 of 1981 was found out by the High Court and on
that basis chose to decree the said suit which was dismissed by the appellate court. It was
further pointed out that the sole plea raised in the plaint was that the plaintiff had derived
his title vide a Sale Deed from Doraiswamy who himself had continued to be in adverse
possession after Karupayee Ammal. In short the basis of the plea of plaintiff was his valid
title. Learned counsel was at pains to point out that the case regarding adverse possession
was very rightly held not proved by the appellate court and indeed there could not be any
adverse possession since the adverse nature of possession was not proved at all. Learned
counsel pointed out that the plea regarding adverse possession was a confused plea
inasmuch as it was not even pleaded as to against whom was the possession of Karupayee
Ammal and Doraiswamy adverse. Learned counsel, therefore, pleaded that once that plea
was rejected, there was no question of decreeing the suit and the suit should have been
straightway dismissed as was done by the appellate court. Instead the High Court had
found entirely different theory by trying to re-appreciate the evidence even regarding the
boundaries of the plot and the identification thereof which was nobody's case.
7. As against this the learned counsel for the respondent supported the judgment and
suggested that though the plea of adverse possession was not proved, still what was
transferred by Doraiswamy was a possessory title. Learned counsel tried to urge that
Karupayee Ammal continued on the land and she became the owner of the land in
question because of her long possession over natham poramboke and hence Doraiswamy
who continued after her demise would inherit the same rights, he being her legal
representative. It is these rights which he had transferred in favour of Muthuswami
Gounder and, therefore Muthuswamy Gounder had a better title as against the present
appellant Dharmarajan who merely claimed a Sale Deed from non-existent owner.
8. A glance at the High Court judgment suggests that the High Court has gone into a
dangerous area of appreciation of evidence, that too on the basis of non existent
substantial questions of law. The five questions of law framed by the High Court were as
follows :
"(1) Whether the admitted long possession of the original owner Karupayee and that of
Doraiswamy who claims title through her cannot be tacked together in law for the
purpose of adverse possession?
(2) Whether the burden is not on the plaintiff who is out of possession to prove that he
has got valid title in the suit properties as laid down by this Court?
(3) Whether non-examination of the vendors of the plaintiff is not fatal to the case of the
plaintiff?
(4) Whether Ex.A-8 is not admissible in evidence? And
(5) Whether lower appellate court is justified in decreeing the suit for declaration, having
found that the defendants are in possession and having refused to grant injunction in
favour of the plaintiff?"
In our opinion none of these questions could be said to be either question of law or a
substantial question of law arising out of the pleadings of the parties. The first referred
question of law could not and did not arise for the simple reason that the plea of adverse
possession has been rightly found against the plaintiff. Karupayee Ammal's possession,
even if presumed to be in a valid possession in law, could not be said to be adverse
possession as throughout it was the case of the appellant Dharmarajan that it was a
permissive possession and that she was permitted to stay on the land belonging to the
members of the Iyer family. Secondly it has nowhere come as to against whom was her
possession adverse. Was it adverse against the Government or against the Iyer family? In
order to substantiate the plea of adverse possession, the possession has to be open and
adverse to the owner of the property in question. The evidence did not show this
openness and adverse nature because it is not even certain as to against whom the adverse
possession was pleaded on the part of Karupayee Ammal. Further even the legal
relationship of Doraiswamy and Karupayee Ammal is not pleaded or proved. All that is
pleaded is that after Karupayee Ammal's demise Doraiswamy as
@page-SC854
her foster son continued in the thatched shed allegedly constructed by Karupayee Ammal.
There was no question of the tacking of possession as there is ample evidence on record
to suggest that Doraiswamy also was in the service of Iyer family and that he was
permitted to stay after Karupayee Ammal. Further his legal heirship was also not
decisively proved. We do not, therefore, see as to how the first substantial question of law
came to be framed. This is apart from the fact that ultimately High Court has not granted
the relief to the respondents on the basis of the finding of this question. On the other hand
the High Court has gone into entirely different consideration based on reappreciation of
evidence. The second and third questions are not the questions of law at all. They are
regarding appreciation of evidence. The fourth question is regarding the admissibility of
Exhibit A-8. In our opinion there is no question of admissibility as the High Court has
found that Exhibit A-8 was not admissible in evidence since the Tehsildar who had issued
that certificate was not examined. Therefore, there will be no question of admissibility
since the document itself was not proved. Again the finding of the High Court goes
against the respondent herein. Even the fifth question was a clear cut question of fact and
was, therefore, impermissible in the Second Appeal.
9. It must be remembered that plaintiff Muthuswamy Gounder had claimed the title and
possession in respect of the suit property by virtue of Exhibit A-1, Sale Deed dated
10.10.1980 and before which he had also obtained the mortgage in respect of this
property from Doraiswamy. It was, therefore, imperative on the part of the plaintiff to
prove a valid title on the part of the Doraiswamy. The High Court has rightly not accepted
the case of adverse possession though it has given a confused finding about it. However,
one look at pleadings suggests that the only plea regarding the ownership of Doraiswamy
was based solely on the plea of his adverse possession. Once that position is clear, the
High Court could not have gone into any other aspect which was not even pleaded in the
plaint. Instead of discussing the evidence of the plaintiff since the burden was entirely on
the plaintiff Muthuswamy Gounder, his being a prior suit, the High Court went on to
discuss the evidence on the part of defendant Dharmarajan who was the purchaser of the
Eastern half of the suit property under Exhibits B-12 and B-13 and the other defendants 2
to 5 who had purchased the Western half of the suit property under Exhibits B-1 and B-2.
Thereafter the High Court has given a finding that Karupayee Ammal was in possession
of the land for 50 years or so and thereafter her foster son Doraiswamy continued and,
therefore, the possession of Karupayee Ammal and Doraiswamy could be tacked together
and that the appellate court was wrong in treating the possession of Karupayee Ammal
and subsequently by Doraiswamy as distinct and separate. All these findings are of no use
whatsoever for the simple reason that the theory of adverse possession had already failed.
Even the High Court has observed that it is not as if the plaintiff is claiming the right only
by adverse possession. Further the High Court found out that the property was a village
Natham and, therefore, the person who first occupied the same and was residing therein is
entitled to title. The High Court has, from nowhere, found out that it was an unoccupied
Natham and Karupayee Ammal has entered the possession and was residing there by
putting up a house and fencing the property and that she would be entitled to declaration
of her occupancy rights or title because the Government is not claiming it as a poramboke
or its vesting with the Government. We fail to follow any basis for this finding of the
High Court. There is no pleading about this. There is not even an iota of evidence in the
village records in favour of either Karupayee Ammal or Doraiswamy and their so-called
rights. There is a Gram Panchayat in the village and we are certain that there would have
been some evidence in the shape of revenue records in favour of either of these two, had
the case of uninterrupted possession of Karupayee Ammal on village Natham for 50
years, was true. The High Court has found out an entirely different case. The High Court
has lastly held that a continuous possession independently by the person in possession
will definitely entitle him to the property in view of the fact that the property is only a
Natham and not a poramboke. We are afraid this was not a case pleaded in the plaint at
all. In fact excepting the plea of adverse possession, no other plea has been raised.
Therefore, the High Court has clearly erred in this aspect. Similarly the High Court in
para 13 went into the question of identity of suit property
@page-SC855
without there being any pleading and a long and unnecessary discussion.
10. There was a previous litigation in OS No.49 of 1963 before the Subordinate Judge,
Erode which was the suit for partition and separate possession filed by one Venugopal
Iyer against Venkataramana Iyer and his sons. The appellant Dharmarajan had produced
Exhibit B-6, the Judgment which showed that the Brahmin family under whom the
present defendants claimed title was represented by four brothers representing four
branches and they were Ramaswamy Iyer, Venkatasubba Iyer, Krishnaswamy Iyer and
Subramaniya Iyer. There was a partition between these four brothers and as per the
Agreement Krishnaswamy Iyer and Subramania Iyer were allotted the property jointly as
against their shares, Ramaswamy Iyer and Venkatasubba Iyer were dealing separately
their respective shares. It was Subramania Iyer's son Venugopal Iyer who was the plaintiff
in the said suit while Venkataramana Iyer and his sons who were the descendants of
Krishnaswamy Iyer were the main defendants therein. The High Court has gone through
this judgment and has recorded that ultimately the partition was granted only in respect of
Survey No.361/D of Kavundapadi village which was Plaint "A" Schedule and the
backyard of the house which was Plaint "B" Schedule and in other aspects the suit was
dismissed. What the High Court has failed to see is that there is a mention of the property
in the suit in this litigation. The High Court ultimately gave a finding that the suit
property was not the subject-matter as it was lying South of Kattabomman Street and this
property was not, therefore, partitioned in the said suit. In its enthusiasm the High Court
has given a finding that the house was extended further south to the East West
Kattabomman Street and, therefore, the vendors of the defendants (Dharamrajan and
others) had not chosen to deal with suit property even as early as 1957. Ultimately the
High Court has given a finding that the suit property was not the subject- matter of
Exhibit A-11 partitioned in the year 1957.
11. It was pointed out by the learned counsel appearing on behalf of the appellant that
there is a definite mention in Suit OS No.49 of 1963 of the suit property. As if this was
not sufficient, the learned counsel has also pointed out that the suit property was
registered in the name of Venugopala Iyer in Kavundapadi Panchayat and he has also
paid house tax to the Panchayat for the suit property. There is a receipt (Exhibit B-3) on
record of the house tax paid by the Iyer family which is long prior to the suit. There is
also a certificate (Exhibit B-5) to show that house was registered in the name of
Venugopala Iyer for a period even prior to 1977. The appellate court had accepted this
documentary evidence. In our opinion that would be the end of the matter and in the
absence of any revenue records in favour of either Karupayee Ammal or Doraiswamy,
there was no question of their title over the land. The High Court has, in para 24,
recorded:
"The question of adverse possession does not actually arise because the Brahmin family
never asserted title over the suit property and the defendants only with a view to harass
the plaintiff has gone and taken sale deeds from the members of the Brahmin family in
the year 1980 knowing fully well that all along the family never asserted title and had
never been in possession of the suit property."
In our opinion these findings are entirely erroneous and the High Court has gravely erred
in interfering with a well considered judgment and findings of fact of the appellate court
who has accepted the case of the defendant and has chosen to hold that the plaintiff in OS
No.555 of 1981 was not able to discharge the burden at all.
12

. In the absence of pleadings, the High Court gravely erred in finding out an entirely new
case on the basis of unpleaded facts and non existent rights. Learned counsel for the
respondent tried to suggest that this was a Natham and the parties had proceeded on that
basis and, therefore, the long standing possession of Karupayee Ammal and thereafter of
Doraiswamy would clothe them with the ownership. In the first place that it was a
Natham was not pleaded. Secondly, there is nothing to suggest that this long standing
possession could clothe the Karupayee Ammal and Doraiswamy with the ownership
rights. That was neither a case pleaded nor proved. Again there was nothing in the shape
of revenue records in favour of Karupayee Ammal and Doraiswamy. Learned counsel
relied upon the judgment of the Madras High Court in The Executive Officer, Kadathur
Town Panchayat, Harur Taluk, 2004 AIHC 4001

@page-SC856
Dharama-puri District v. I.V. Swaminatha and Ors. [(2004) 3 L.W. 278] delivered by the
Division Bench thereof in support of his contention that a long possession over Gram
Natham ripens into the ownership rights. We are afraid the judgment is being read too
broadly. No such proposition of law emerges from that judgment.
13. On the other hand the appellate court has rightly relied on the tax receipts and the
entry in the name of Venugopala Iyer in respect of the suit land in Survey No.324 Ward
No.4. The appellte court had also correctly held that the suit property was mentioned in
Exhibits B-6 and B-7 as also in the decree in OS 49 of 1963 in Item No.1 of Schedule C
property which was not divided. All the lengthy discussion by the High Court over that
issue was not only uncalled for but the High Court has gravely erred in setting aside the
finding of the appellate court that the suit property was the property of Item No.I of
Schedule C in Exhibit B-7.
14

. This Court has, time and again, explained the scope of Section 100 CPC, more
particularly in Gurudev Kaur and Others vs. Kaki and Others [(2007) 1 SCC 546] where
it was held that even before the 1976 amendment the scope of such interference under
Section 100 drastically curtailed and narrowed down. It is specifically held that the High
Court would have jurisdiction of interfering only in a case where substantial questions of
law are involved and those questions are clearly formulated in the Memorandum of
Appeal. We have already shown that the questions formulated were neither the questions
of law nor substantial questions of law. This is apart from the fact that in the present case
the High Court has completely gone astray inasmuch as it is not even realized that it was
a case which was not even pleaded. In Gurudev Kaur's case the above mentioned position
stated by us in respect of substantial question of law has been reiterated. Thus, the
judgment suffers from error of law. 2006 AIR SCW 2404

15. Learned counsel for the respondent lastly suggested that Doraiswamy had transferred
the possessory title and, therefore, the plaintiff in OS No.555 of 1981 was justified in
filing the suit against the present defendants. This was not even the case pleaded. On the
other hand what was pleaded was adverse possession alone. This is apart from the fact
that all through the plaintiff claimed a title and ownership from Doraiswamy, who
according to the plaintiff, had both ownership and the title to the suit property. The term
possessory title was not even whispered anywhere. We are, therefore, unable to accept the
contention of the learned counsel on behalf of the respondent.
16. Once the suit of Muthuswamy Gounder fails, then the other suit filed by Dharamrajan
being OS 280 of 1982 in respect of the Eastern half portion of the suit property must
succeed. The appellate court has rightly granted the declaration in that suit and has also
restricted the relief only to the declaration since Dharmrajan and the other defendants had
not terminated or revoked the licence of Doraiswamy or his wife Valliammal or daughter
Palaniammal. The First Appellate Court had also correctly held that appellant in AS
No.10/1995 in OS No.280/1982 had established title of his vendors and further that his
vendors has passed a valid title to him with respect to the suit property under Exhibits B-
12 and B-13. We also accept the judgment of the appellate court that Dharamrajan and
other defendants were not entitled to the injunction prayed for.
17. In the result the appeals succeed with costs. The judgment of the High Court is set
aside and that of the First Appellate Court is restored.
Order accordingly.
AIR 2008 SUPREME COURT 856 "Union of India v. S. R. Dhingra"
Coram : 2 A. K. MATHUR AND MARKANDEY KATJU, JJ.
Transferred Case (Civil) No. 106 of 2006, D/- 14 -12 -2007.
Union of India v. S. R. Dhingra and Ors.
Constitution of India, Art.16 - Railway Establishment Code, R.2544(g) - EQUALITY IN
PUBLIC EMPLOYMENT - RAILWAY - PENSION - PAY COMMISSION - Pension -
'Running staff' of railways - Inclusion of "running allowance" to arrive at average
emoluments - Pre-1986 retirees - Recomputation of pension to give benefits of 5th Pay
Commission - Fixation of notional pay of pre-1986 retirees as on 1-1-86 - Running
allowance that has been considered at time of retirement - Not to be considered again
while fixing notional pay. (Paras 23, 24, 26)
@page-SC857
Cases Referred : Chronological Paras
2006 AIR SCW 5252 : 2006 Lab IC 4210 : 2007 (1) AIR Kar R 33 25
2000 AIR SCW 3422 : AIR 2000 SC 3113 : 2000 Lab IC 3298 25
1998 AIR SCW 3431 : AIR 1999 SC 81 : 1998 Lab IC 3579 25
1997 AIR SCW 3747 : AIR 1997 SC 3828 : 1998 Lab IC 100 7, 18
1991 AIR SCW 327 : AIR 1991 SC 1182 : 1991 Lab IC 573 25
AIR 1990 SC 1782 : 1990 Lab IC 1490 25
AIR 1983 SC 130 : 1983 Lab IC 1 25
A. Sharan, ASG, Kumar Rajesh Singh, Satyakam, Amit Anand, B. Krishna Prasad, for
Petitioner; R. Venkataramani, Sr. Advocate, J. M. Khanna, Y. Raja Gopala Rao, Y.
Ramesh, Ms. Y. Vismai Rao, for Respondents.
Judgement
1. MARKANDEY KATJU, J.:-Writ Petition No. 4648/2002 titled Union of India and
another vs. S.R. Dhingra and others was filed in the Delhi High Court and was thereafter
transferred to this Court by order dated 9.5.2006 in Transfer Petition (Civil) No. 278 of
2005.
2. It appears that similar matters were pending before the Delhi High Court, Punjab and
Haryana High Court and Central Administrative Tribunal and further proceedings in
those cases were ordered to be stayed awaiting the judgment in the matter which was
transferred to this Court by the order dated 9.5.2006.
3. The facts of the case are that in the Railways there are certain employees such as
drivers, guards, shunters etc, who go along with the railway train and are categorized as
Running Staff. Such staffs are entitled to an allowance called Running Allowance which
is apart from their salary. Computation of pension after retirement in the railways is made
on the basis of average emoluments plus a part of the running allowance which is
included in average emoluments in terms of Rule 2544(g) of the Indian Railway
Establishment Code. The present dispute relates to the pension for the running staff who
retired prior to 1986.
4. Prior to its amendment Rule 2544(g) running allowance up to a maximum of 75% of
pay and other allowances was added to the pay for computing pension. Subsequently, by
notification dated 5.12.1988 Rule 2544 was amended, and the maximum limit of the
running allowance was fixed at 45% of the pay in the revised scale of pay. Thereafter by
another amendment this was raised to 55% of the average pay.
5. The validity of this amendment was challenged before the Central Administrative
Tribunal by means of a petition which was allowed by the Ernakulam Bench by the order
dated 20.4.1990 and the impugned notification was quashed to the extent that the
amendment of Rule 2544(g) was given retrospective effect.
6. A contrary view was taken by another Bench of the Tribunal and hence the matter was
referred to a Full Bench, and the Full Bench agreed with the view taken by the
Ernakulam Bench.
7

. The matter was then carried in appeal to this Court in Chairman, Railways Board and
others vs. C.R. Ranga-dhamaiah and others, 1997(6) SCC 623 which gave its judgment
on 25.7.1997, upholding the decision of the Full Bench of the Tribunal. 1997 AIR
SCW 3747

8. It may be mentioned that the Tribunal had struck down the retrospective operation of
the notification dated 5.12.1988 issued in exercise of the power of the President under the
proviso to Article 309 of the Constitution whereby Rule 2544 of the Indian Railway
Establishment Code was amended with retrospective effect. In the aforesaid decision this
Court dismissed the appeals filed by the Union of India and the Railway Administration
and upheld the view taken by the Full Bench of the Tribunal.
9. To implement the aforesaid decision of this Court the Railway Board issued
instructions on 14.10.1997 that the pension and other retiral benefits of the railway
running staff who retired between 1.1.1973 to 4.12.1988 should be re-computed, and
arrears on account of re-computation of pension be paid to them accordingly.
10. In the meanwhile the Fifth Pay Commission report was rendered and in
implementation of some of the recommendations in respect of the pre-1986 retirees the
Department of Personnel and PW issued OM dated 27.10.1997 that pension/family
pension will be consolidated w.e.f. 1.1.1996 by adding together their existing
pension/family pension, dearness relief, interim relief I and interim relief II and fitment
weightage of 40%. The said revision was to be done on the basis of existing pension. It is
not in dispute that the said relief was extended to the
@page-SC858
respondents herein and all those similarly placed with the respondents.
11. In order to implement another recommendation of the 5th Pay Commission to give
parity to the pre-1986 retirees with the post 1986 retirees, it was decided to fix the pay of
the pre-1986 retirees as on 1.1.1986. However, it was specifically provided that the pay
so fixed would be treated as average emoluments. Accordingly, on 10.2.1998, a further
Office Memorandum was issued providing for fixation of notional pay of all those
Government servants who retired prior to 1.1.1986 as on 1.1.1986. The pay was to be
fixed on notional basis in the scale revised with effect from 1.1.1986. It was provided that
the notional pay so arrived as on 1.1.1986 shall be treated as average emoluments for the
purpose of calculation of pension and accordingly the pension shall be calculated as on
1.1.1986 as per the pension formula then prescribed. (The then formula was 50% of
average emoluments. Thus 50% of the pay notionally fixed was to be treated as pension
of the respective pre-1986 retirees).
12. Learned Addl. Solicitor General, Shri Amarendra Sharan, appearing for the appellants
submitted that the OM dated 10.2.1998 is very clear that what is to be fixed is the pay on
notional basis, and pay, according to the rules, does not include running allowance.
However, running allowance is a relevant factor for calculating average emoluments. He
submitted that the said OM clearly provides that the notional pay so arrived at will be
treated as average emoluments, meaning thereby that all other elements provided in rule
2544 which otherwise could have been added to pay for arriving at average emoluments
were expressly excluded. He further submitted that the pay notionally fixed includes
existing (pre-1986) basic pay, dearness pay, additional dearness allowance, ad hoc
dearness allowance, first and second installments of interim relief calculated on basic pay
plus 30% basic pay in the existing scale as pay element of running allowance, and fitment
weightage at the rate of 20% of existing basic pay.
13. On 24.7.1998, it was clarified by the department that in cases where the amount of
revised pension arrived at on the basis of notional fixation of pay as on 1.1.1986 happens
to be less than the amount of consolidated pension already drawn by pre-1986 retirees,
there is no need to revise the existing pension and issue a revised PPO.
14. Due to certain doubts and confusion, a clarification was sought as to whether running
allowance was to be added again while fixing the pay notionally as per the OM dated
10.2.1998. On 29.12.1999, the Railway Board issued clarification that running allowance
was not to be taken into consideration at the time of re-fixation of pay on notional basis
on 1.1.1986.
15. The said clarification was challenged by way of an Original Application before the
Central Administrative Tribunal by the respondents. On 22.10.2000 the appellants
(respondents in OA) contested the original application of the respondents (appellants in
OA) before the Central Administrative Tribunal, Principal Bench, New Delhi by filing
their detailed counter reply dated 28.5.2001 in which preliminary objections were also
raised regarding maintainability of the original applications. It was pointed out that only
the pay was to be notionally fixed as per the OM dated 10.2.1998, and therefore, there
was no need to add running allowance again since running allowance would be added
only if on the basis of pay so fixed the average emoluments was to be recalculated. The
said course of recalculation of average emoluments was specifically excluded by the said
OM dated 10.2.1998.
16. On 22.1.2002, the Central Administrative Tribunal, Principal Bench, New Delhi
passed an order allowing the O.A. of the respondents herein and quashed the Railway
Boards clarification dated 29.12.1999. Thereafter various writ petitions were filed in the
Delhi High Court and ultimately writ petition No. 4648/2002 was transferred to this
Court by order of this Court dated 9.5.2006 (as already stated above).
17. We have carefully perused the record and heard learned counsel for the parties.
18

. Leaned counsel for the appellant submitted that the pension of the respondents which
was recomputed on the basis of the judgment of this Court in Chairman, Railways Board
and others vs. C.R. Rangadhamaiah and others (supra) is fully protected and the
respondents will continue to take the benefits in future also. However, he submitted that
the pensionary benefits of the respondents who retired prior to 1.1.1986 was to be
determined by fixing notional pay w.e.f. 1997 AIR SCW 3747

@page-SC859
1.1.1986 and further pension was to be re-fixed w.e.f. 1.1.1986 without taking into
consideration the running allowance as per the policy of the Railway Board for
implementation of the recommendation of the 5th Central Pay Commission while
revising the retiral benefits w.e.f. 1.1.1986. He submitted that there was no infirmity in
the policy issued by the Department of Personnel and PW and adopted by the Railway
Board, and subsequent circulars issued by the Railway Board regarding release of retiral
benefits to the employees who retired prior to 1.1.1986.
19. Learned Additional Solicitor General submitted that while fixing the pension
notionally, due to a clerical mistake it had been fixed at much higher amount, and this
error, when discovered, was later rectified and the pension of the respondents was
correctly revised w.e.f. 1.1.1986.
20. We have carefully considered the submission of the learned counsels for the parties
and we are in agreement with the submission of the learned Additional Solicitor General.
21. In this connection it may be mentioned that the Railway Board vide its letter dated
29.12.1999 issued a clarification to the earlier instructions dated 10.2.1998 issued by the
Department of Personnel and PW, which is the nodal department of the government of
India for framing policy instructions on pensionary matters.
22. It has also been clarified by the Department of Personnel and PW vide their O.M.
No. 45/86/97-P and W (A) Pt. III dated 24.7.1998 circulated vide Railway Boards letter
No. F(E)-III/98/PN-1/2 dated 2.9.1998 that if the pension revised on notional fixation of
pay as on 1.1.1986 happens to be less than the pension already drawn by the pensioner
the same should not be reduced to their disadvantage. In the case of Medical Officers, the
Department of Personnel and Pensioners Welfare have already clarified vide their OM
No. 45/3/99-P and PW (A) dated 12.10.1999 circulated vide Railway Boards letter No.
F(E)-III/98/PN-1/29 dated 12.11.1999 that non-practising allowance which was already
taken into account for calculating the pension and other benefits at the time of retirement
is not to be added to the pay of the pre-1986 retirees revised on notional basis as on
1.1.1986, as the same is not permissible in terms of para No. 2 of their instructions dated
10.2.1998. The nature of running allowance is similar to that of non-practising allowance,
and the Railway Board issued instructions dated 29.12.1999 clarifying that running
allowance is not to be added to the pay of pre-1986 retirees revised on notional basis as
on 1.1.1986.
23. We are of the opinion that the clarification of the Railway Board issued dated
29.12.1999 clarifying that the running allowance which was already taken into account
for pension and other benefits at the time of retirement is not to be added to the pay of
pre-1986 retirees revised on notional basis as on 1.1.1986 is valid. It appears that due to a
clerical error the notional benefits of the respondents w.e.f. 1.1.1986 was wrongly fixed
and such retired employees are getting excess pension. It is well-settled that a mistake
does not confer any right to any party, and can be corrected.
24. We are of the opinion that the benefit of running allowance has to be taken into
consideration for computing pension only once. It had been taken into consideration
while fixing the pension of the respondents at the time of their retirement. In our opinion
it is not required to be taken into account again for any future calculation.
25

. It is well settled that when two sets of employees of the same rank retire at different
points of time, one set cannot claim the benefit extended to the other set on the ground
that they are similarly situated. Though they retired with the same rank, they are not of
the same class or homogeneous group. Hence Article 14 has no application. The
employer can validly fix a cut-off date for introducing any new pension/retirement
scheme or for discontinuance of any existing scheme. What is discriminatory is
introduction of a benefit retrospectively (or prospectively) fixing a cut-off date arbitrarily
thereby dividing a single homogenous class of pensioners into two groups and subjecting
them to different treatment (vide Col. B.J. Akkara (Retd.) vs. Govt. of India (2006) 11
SCC 709, D.S. Nakara vs. Union of India (1983) 1 SCC 305, Krishna Kumar vs. Union
of India (1990) 4 SCC 207, Indian Ex-Services League vs. Union of India (1991) 2 SCC
104, V. Kasturi vs. Managing Director, State Bank of India 2006 AIR SCW 5252

AIR 1983 SC 130


AIR 1990 SC 1782
1991 AIR SCW 327
1998 AIR SCW 3431
2000 AIR SCW 3422

@page-SC860
(1998) 8 SCC 30 and Union of India vs. Dr. Vijayapurapu Subbayamma (2000) 7 SCC
662).
26. In view of the above, we are of the opinion that the benefit of running allowance
which has been given to the respondent at the time of retirement is not to be given again
vide Office Memorandum dated 10.2.1998.
27. Accordingly Writ Petition No. 4648/2002 which has been transferred to this Court is
allowed and the order of the Central Administrative Tribunal dated 22.1.2002 is set aside.
This decision will govern all similar matters pending in the High Courts or Tribunal.
28. However, any amount already paid to the respondents and other similarly situated
persons shall not be recovered from them.
29. The Transfer Case stands allowed.
Order accordingly.
AIR 2008 SUPREME COURT 860 "Union of India v. Smt. Sadhana Khanna"
(From : Delhi)
Coram : 2 A. K. MATHUR AND MARKANDEY KATJU, JJ.
Civil Appeal No. 8208 of 2001, D/- 14 -12 -2007.
Union of India v. Smt. Sadhana Khanna.
Constitution of India, Art.16 - EQUALITY IN PUBLIC EMPLOYMENT -
PROMOTION - SENIORITY - Promotion - Select list (seniority quota) - Inclusion in -
Minimum eligibility service requirement - Letter of appointment was offered to senior,
respondent on 5-7-1983 i.e. after 1-7-1983, a date from which eligibility was to be
counted - However juniors to respondent were issued letters offering appointment prior to
1-7-1983 - It was Department which is to be blamed for this - Office memorandum
stating that where junior had completed eligibility requirement of promotion then their
seniors will also be considered even if they have not completed eligibility period - Thus
inclusion of juniors and not senior in select list - Not proper.
AIR 1988 SC 902, Foll. (Para 11)
Cases Referred : Chronological Paras
AIR 1988 SC 902 : 1988 Lab IC 999 (Foll.) 9
T. S. Doabia, Sr. Advocate, Mrs. Kiran Bhardwaj, Mrs. Sushma Suri, Manpreet Singh
Doabia, for Appellant; Ms. Rachna Gupta, Dr. Indra Pratap Singh, Amrit Singh, for
Respondent.
Judgement
1. MARKANDEY KATJU, J. :-This appeal has been filed against the impugned
judgment of the Delhi High Court dated 21.3.2000 in Civil Writ Petition No.1311 of
2000.
2. Heard learned counsel for the parties and perused the record.
3. The facts of the case are that the respondent joined the Central Secretariat Service on
13.7.1983 after passing Assistants Grade Examination held in October, 1981. The
respondent was allocated to the finance cadre comprising the Ministry of Finance.
4. After completion of eight years of regular service in the grade of Assistant, the
respondent was granted a short-term promotion to the grade of Section Officer on
24.7.1991. The respondent continues to work in this post till date.
5. Select List (Seniority quota) for promotion to the Section Officers Grade for the year
1991 was issued on 28th May, 1993. The respondents name has not been included in this
select list although the respondent was qualified to be included in the select list. Officers
junior to the respondent who have secured lower rank in the Assistant Grade Examination
1981 have been included in the select list for Section Officers 1991.
6. The seniority list of Assistants Grade had been issued on 1st October, 1990. In this list
the respondent ranks at S.N.29 whereas other officers of the same grade at S.Nos.30, 32,
34, 25 and so on who are junior to the respondent in this seniority list have superseded
the respondent and promoted as Section Officer on the basis of the Select List of 1991.
Promotion to the post of section officers from the grade of Assistant to the grade of
Section Officers (seniority quota) is on non-selection basis, based only on seniority.
7. The grievance of the respondent was that her juniors were included in the select list but
her name was not so included. Hence, she filed an O.A. before the Central Administrative
Tribunal.
@page-SC861
8. In the counter affidavit filed before the Tribunal the appellant herein (respondent
before the Tribunal) alleged that the respondent was not eligible for inclusion in the
Select List of 1991, since on 1-7-1991 she was short of the minimum eligibility service
requirement of eight years by twelve days. The respondent joined as Assistant on 13-7-
1983 and as such she could not be placed on the select list.
9

. The Tribunal allowed the O.A. by its order dated 24-9-1999. In the said O.A. it was held
that the Department of Personnel and Training had issued an Office Memorandum dated
19.7.1989 soon after the decision of this Court in R. Prabha Devi and others vs.
Government of India Through Secretary, Ministry of Personnel and Training,
Administrative Reforms and others 1988 (2) SCC 233 stating that where the junior had
completed the eligibility requirement of promotion then their seniors will also be
considered even if they have not completed the eligibility period. AIR 1988 SC 902

10. The appellant filed a writ petition before the Delhi High Court which was dismissed
and hence this appeal.
11. It may be noted that the respondent was offered appointment vide letter dated 5-7-
1983 which is after 1-7-1983 from which the eligibility was to be counted. Hence, it is
the Department which is to blame for sending the letter offering appointment after 1-7-
1983. In fact, some of the candidates who were juniors to the respondent were issued
letters offering appointment prior to 1-7-1983. Hence it was the Department which is to
blame for this. Moreover, in view of the Office Memorandum of the Department of
Personnel and Training dated 18-3-1988 and 19-7-1989 the respondent was also to be
considered, otherwise a very incongruous situation would arise namely that the junior
will be considered for promotion but the senior will not.
12. In view of the above there is no merit in this appeal and it is dismissed.
Appeal dismissed.
AIR 2008 SUPREME COURT 861 "State of Gujarat v. Gajanand M. Dalwadi"
(From : Gujarat)*
Coram : 2 S. B. SINHA AND H. S. BEDI, JJ.
Civil Appeal No. 2322 of 2006, D/- 14 -12 -2007.
State of Gujarat v. Gajanand M. Dalwadi (D) by L.Rs.
(A) Constitution of India, Art.311 - TERMINATION OF SERVICE - MISCONDUCT -
FORGERY - SERVICE MATTERS - Dismissal from service - Misconduct - Delinquent
officer working in accounts Department and not in licence Department of Regional
Transport Office - Issuance of forged driving licence by him though it was not his
function - By itself a misconduct - Order dismissing him from service - Proper -
Misconduct, of such a magnitude, when proved, cannot be ignored on surmises and
conjectures - Equity, in a case of this nature, would have no role to play - Forgery of
licence cannot be condoned only because it has been done at instance of colleague, even
if it be so assumed.
L. P. A. No. 593 of 2004 in Spl. Civil Appln. No. 6283 of 2000, D/- 29-7-2004 (Guj.),
Reversed. (Paras 8, 9, 12)
(B) Constitution of India, Art.311 - TERMINATION OF SERVICE - LICENSE -
SERVICE MATTERS - Imposition of punishment - Proportionality - Delinquent officer
found guilty of issuance of forged driving licence - Order of dismissal /removal is
appropriate punishment.
2007 AIR SCW 7237, 2007 AIR SCW 4136, 2007 AIR SCW 3656, Foll. (Para 10)
Cases Referred : Chronological Paras
2007 AIR SCW 3656 : AIR 2007 SC 2129 (Foll.) 10
2007 AIR SCW 4136 : 2007 Lab IC 3266 : 2007 (5) ALJ 6 (Foll.) 10
2007 AIR SCW 7237 (Foll.) 10
Yashank Adhyaru, Sr. Advocate, Ms. Hemantika Wahi, with him for Appellant; H. K.
Puri, for Respondents.
* L. P. A. No. 593 of 2004 in Spl. Civ. Appln. No. 6283 of 2000, D/- 29-7-2004 (Guj.)
Judgement
S. B. SINHA, J. :-Gajanand M. Dalwadi, since deceased (delinquent officer) was working
in the Regional Transport Office under the Commissioner of Transport in the State of
Gujarat. He had been working in the Department for Grant of Licence. At the relevant
time, however, he was
@page-SC862
serving in the Accounts Department as a Summary Clerk.
2. An inspection was conducted in the Licence Branch of the Regional Transport Office
during the period 21.8.1995 to 13.9.1995.
3. Several misconducts committed by the delinquent officer came to the notice of the
authorities. It was found that a forged license was granted to one Narendra Kumar who
had met with an accident although at the relevant point of time, he was possessing a valid
driving licence. A chargesheet was issued against him. Upon holding a disciplinary
proceeding, the enquiry officer submitted a Report on 6.12.1997 stating that the charges
against him have been proved. The disciplinary authority directed his removal from
service by an Order dated 26.10.1998. Aggrieved by the said Order imposing punishment
upon him, he filed an application before the Gujarat Civil Services Tribunal. The said
application was allowed holding that misconduct on his part, if any, was committed by
him at the request of another clerk; viz. one Dudhrechia. It was further held;
"15. From the Department, it is submitted that Dudhrechia has denied entrusting the work
to appellant but as stated above Dudhrechia would never admit and the submission of
appellant gets credence that this is not an afterthought in the appeal but it was put to the
concerned clerk at the enquiry, at first in point of time.
16. Also the order is too harsh. The Disciplinary Authority must given reasons why it is
proper to pass such orders. In the Discipline Appeals and Rules providing for major
penalties step by step, the punishments are given with a view that penalty must be in
consonance with the act complained or charges proved or the misconduct of the staff. The
appellant is not a chip of dead wood that he must be removed. Also punishment such as
harsh as this would also required (sic) to consider rising an employment in the State. Not
that we want to protect dishonest or bad people but reasons must be given and
satisfaction must be reached that this punishment is proper."
4. A learned Single Judge of the said Court allowed the Writ Petition filed by the
appellant holding that the delinquent had all the opportunities to reply to the charge-sheet
and take part in the disciplinary proceeding. The learned Single Judge held that the
decision of the Tribunal resulted in miscarriage of justice warranting the Courts
intervention under the supervisory jurisdiction conferred upon the High Court under
Article 227 of the Constitution of India stating;
"It is evident that on the date when Driver Narendra Kumar met with the accident, he did
not possess a valid driving licence. In the circumstances the owner of the vehicle
Sugarmal Bherumal, could not have claimed insurance money for the damage caused to
the vehicle. With a view to facilitating the insurance claim, the said Sagarmal Bherumal
arranged for issuance of a duplicate licence in the name of driver-Narendra Kumar for the
period covering the date of the accident. Indisputably, the duplicate licence was issued by
the delinquent. Obviously, the duplicate licence was obtained by the owner Sagarmal
Bherumal with an intention to defraud the insurance Company. The delinquent played an
important role in this fraudulent scheme by issuing duplicate licence. Indisputably, it was
neither the function of the delinquent to issue such licence nor was it his defence that the
said licence was issued by him at the request of the concerned Clerk Shri Dudhrejia or
any other officer. Such defence was taken by the delinquent at a much later stage in the
disciplinary inquiry, though unsuccessfully.
It is quite possible that apart from the delinquent, there were other persons involved in the
aforesaid fraudulent scheme and a further inquiry could have revealed the names of the
other persons involved. However, merely because further inquiry was not made, the
delinquent cannot be exonerated even though by evidence on record the charge against
him has been proved.
As to the second charge, there is no denial by the delinquent that he had left certain
licence numbers blank while issuing the licence numbers. He has not even explained why
such blanks were maintained nor he has denied that the said blanks were maintained with
an ulterior intention to issue bogus licence at a later date. In absence of even a bare
denial, the charge has rightly been held to be proved by the disciplinary authority. The
fact that no licence was issued in the said numbers at any point of time thereafter is of no
consequence.
Even the third charge has been proved
@page-SC863
by the statement of the concerned persons i.e. Shri B.K. Chauhan and Shri N.P. Ptni. It
should also be noted that even in answer to the report of the inquiry officer, the
delinquent has not made out any case based on the evidence on record. Even the said
reply is evasive."
5. The Division Bench of the High Court, however, on an appeal preferred by the
delinquent officer, allowed the said appeal holding;
"Yes, the deceased Gajanand Dalwadi should have been more careful while preparing the
duplicate licence, he may have acted designedly. After all, he may not have understood
the nature of work and manner of transacting it since it was not his function since he was
working in the accounts. Therefore, the conclusions drawn by the Tribunal were justified
and there could be no reason to upturn them."
(Emphasis supplied)
6. Mr. Yashank Adhyaru, the learned senior counsel appearing on behalf of the appellant
submitted that the approach of the Division Bench of the High Court is wholly erroneous
and thus is liable to be set aside.
7. Mr. H.K. Puri, learned counsel appearing on behalf of the respondent, on the other
hand, would support the judgment.
8. Forgery of a licence is a serious charge. It cannot be condoned only because it has been
done at the instance of a colleague, even if it be so assumed. As noticed hereinbefore,
even the employee concerned has denied that the licence was issued at his instance.
9. The learned Tribunal as also the Division Bench of the High Court, with respect,
misdirected themselves in law, as they posed unto themselves wrong questions.
Misconduct, of such a magnitude, when proved, cannot be ignored on surmises and
conjectures. Equity, in a case of this nature, would have no role to play.
10

. When a forgery is committed with a view to assist a person to make unlawful gain for
himself or to cause unlawful loss to another, the matter should be viewed seriously. The
Tribunal is not an appellate authority, its jurisdiction was also limited. It could not have
ordinarily interfered with the quantum of punishment unless it was held to be wholly
disproportionate to the imputation of charges. If ordinarily in regard to the commission of
the offence of forgery, an Order of dismissal/removal is an appropriate punishment; as
has been held in a large number of cases, the same could not have been sidetracked. See
U.P.S.R.T.C. v. Ram Kishan Arora, [2007 (6) SCALE 721], Ramesh Chandra Sharma v.
Punjab National Bank and Anr. [2007(8) SCALE 240] and UCO Bank and Anr. v.
Rajinder Lal Capoor [(2007) 6 SCC 694]. 2007 AIR SCW 7237
2007 AIR SCW 4136
2007 AIR SCW 3656

11. The approach of the learned Single Judge, in our opinion was the correct one.
12. Once, it was held that the delinquent had acted designedly, it could not have also been
held that he might not have understood the nature of work or manner of transacting it,
since it was not his function as he had been working in the accounts. Finding of fact
arrived at by the Enquiry Officer which was accepted by the learned Single Judge, was
that the issuance of licence, which it was not his job, was itself a misconduct. The
Division Bench of the High Court clearly overlooked the fact that it is the positive case of
the State that the delinquent officer was working in the Licence Department prior to his
transfer to the Accounts Department and, therefore, he knew about the modalities of grant
of licence. An application for grant of licence must be processed having regard to the
provisions of the Central Motor Vehicles Rules. An application in Form 4 is required to
be filed as envisaged under Rule 14. Only, upon proper scrutiny thereof, a licence could
be granted in Form 6 as envisaged under Rule 16 of the Rules. Issuance of a forged
licence, having regard to the said provisions, is a serious matter, which could not have
been ignored on the ipse dixit of the Tribunal.
13. For the reasons aforementioned, the impugned Judgment cannot be sustained which is
set aside accordingly. Appeal is allowed. No costs.
Appeal dismissed.
AIR 2008 SUPREME COURT 863 "Govt. of Karnataka v. Gowramma"
(From : Karnataka)*
Coram : 2 Dr. A. PASAYAT AND P. SATHASIVAM, JJ.
Civil Appeal No. 2874 of 2001, D/- 14 -12 -2007.
Govt. of Karnataka and Ors. v. Gowramma and Ors. @page-SC864
(A) Constitution of India, Art.141 - PRECEDENT - Binding precedent - A decision is a
precedent on its own facts - Each case presents its own features - It is not everything said
by a Judge while giving a judgment that constitutes a precedent - Only thing in a Judge's
decision binding on a party is principle upon which case is decided.
According to the well-settled theory of procedents, every decision contains basic
postulates - (i)) findings of material facts, direct and inferential. An inferential finding of
facts is the inference which the Judge draws from the direct, or perceptible facts; (ii)
statements of the principles of law applicable to the legal problems disclosed by the facts;
and (iii) judgment based on the combined effect of the above. A decision is an authority
for what it actually decides. What is of the essence in a decision is its ratio and not every
observation found therein nor what logically flows from the various observations made in
the judgment. The enunciation of the reason or principle on which a question before a
Court has been decided is alone binding as a precedent.
AIR 1968 SC 647; 1996 AIR SCW 4020; (1901) AC 495 (HL)), Relied on. (Para
9)
(B) Constitution of India, Art.141 - PRECEDENT - Binding precedent - Circumstantial
flexibility, one additional or different fact may make a world of difference between
conclusions in two cases - Disposal of cases by blindly placing reliance on a decision is
not proper. (Para 12)
(C) INTERPRETATION OF STATUTES - Interpretation of Statutes - Judges interpret
statutes - They do not interpret judgments.
1970 (2) All ER 294; 1951 AC 737; (1971) 1 WLR 1062; 1972 (2) WLR 537, Rel. on.
(Para 10)
(D) Karnataka Preservation of Trees Act (76 of 1976), S.8 - TREES - Felling of trees -
Claim for value of trees - Plaintiffs, owners of trees, seeking permission for cutting and
felling of silver and jungle wood - Permission granted with condition to transport trees
after felling to Govt. warehouse - There was no challenge to stipulated conditions -
Admittedly trees in question were reserved trees - Order allowing claim of owners
without examining that trees were reserved trees - Not proper.
R. F. A. No. 816 of 1995, D/- 13-4-2000 (Kar), Reversed. (Para 14)
Cases Referred : Chronological Paras
1996 AIR SCW 4020 (Rel. on Pt.A) 9
(1972) 2 WLR 537 : (1972)1 All ER 749 (Rel. on) 11
(1971) 1 WLR 1062 (Rel. on)11
(1970) 2 All ER 294 : (1970)2 WLR 1140 (Rel. on) 11
AIR 1968 SC 647 (Rel. on Pt. A) 9
1951 AC 737 (Rel. on) 10
1901 AC 495 (HL) (Rel. on)) 9
Sanjay R. Hegde and Amit Kr. Chawla, for Appellants; S. N. Bhat, N. P. S. Panwar and
D. P. Chaturvedi, for Respondents.
* R. F. A. No. 816 of 1995, D/- 13-4-2000 (Kant).
Judgement
Dr. ARIJIT PASAYAT, J. :-Heard learned counsel for the parties.
2. Challenge in this appeal is to the judgment of a learned Single Judge of the Karnataka
High Court allowing the appeal filed by the respondents.
3. Plaintiffs, who are the respondents in the present appeal filed a Suit for recovery of a
sum of Rs.1,47,965.20 on the ground that being owners of the Trees which were
transported to the Government godown on the basis of the permission granted by the
present appellants, the value of the Trees has to be paid by the government.
4. The case of the plaintiff, as culled out from the averments in the plaint is that they are
the owners of the suit schedule property. The plaintiffs and their predecessor had grown
silver wood, jungle wood and other varieties of trees in the schedule land by spending lot
of money and had cultivated the said land with coffee crop. In order to regulate the shade
in the schedule property and also for cutting and felling of silver wood, jungle wood and
other trees, the plaintiffs had applied for permission for cutting and felling of the silver
wood, jungle wood and other trees. Before granting the felling permission of the said
trees, a joint survey was carried out by the forest authorities as well as the revenue
surveyors. Thereafter, the second defendant granted permission for felling of the trees
situated in the schedule properties. In terms of the permission, the plaintiffs cut and felled
the trees. While issuing the transport permit to the plaintiffs,
@page-SC865
the second defendant had directed issuance of transport permit for a portion of the trees
and ordered to transfer 1050 CFT of timber valued at Rs.1,31,250/- to an earmarked
forest depot. The firewood of 22-1/2 meters valued at Rs.10,000/- was also transported to
the same depot. Therefore, the claim was made that the plaintiffs are entitled to the value
of the Timber @ Rs.125/- per CFT and at Rs.150/- per CFT at the prevailing rates.
Defendants took the stand that the permission was conditional and there was never any
challenge to the conditional permission granted. After having accepted the permission
with the conditions stipulated, it was not open to the plaintiffs to lay a claim for the value
of the trees. The Trial Judge dismissed the Suit, inter alia, holding that in the absence of a
challenge to the conditional permission, there was no question of the plaintiff's making a
claim for value of the timber transported.
5. An appeal was filed before the High Court, which, by the impugned judgment,
accepted the stand of the plaintiffs. For granting relief to the plaintiffs, i.e. the present
respondents, reliance was placed on certain judgments of the High Court where it was
held that in respect of reserved trees, the ownership was not with the Government but was
with the owner of the land. Accordingly, as noted above, the appeal was allowed.
6. In support of the appeal, learned counsel for the appellant-State submitted that the
grant of permission was governed by the Karnataka Preservation of Trees Act, 1976 (in
short the Act'). Permission is required for felling of all trees irrespective of whether they
are situated in private or in government land. The permission undisputedly is subject to
the stipulated conditions. There is a provision for preferring an appeal in case of refusal
to grant permission. The permission was granted on 30.3.1999 and there was a specific
condition which stipulated that 27 trees of a particular variety which are reserved trees
are to be transported to the Government Nata Warehouse after felling. There was no
challenge to the order in this regard. Since the conditions were not challenged, the High
Court should not have granted relief to the respondents-plaintiffs relying on certain
decisions which were rendered in different context and had no application to the facts of
the present case.
7. Learned counsel for the respondents, on the other hand, submitted that merely because
the trees which were permitted to be cut were reserved trees, that did not mean that
government was the owner of the trees. Reference is made to certain provisions of the
Karnataka Forest Act, 1963 to contend that the ownership of the Government in respect
of the trees is restricted only to sandalwood trees.
8. It is an admitted position that the permission was granted with conditions. It is also not
disputed that PW-l, who was examined in support of the plaintiffs's case, accepted that
the trees in question were reserved trees. The Trial Court took note of this fact and noted
that in the cross-examination of PW-1, he has specifically admitted that the Nandi trees
are reserved trees. Further, the High Court lightly brushed aside the stand of the State and
its functionaries that in the absence of any challenge to the conditions stipulated in the
permission granted, it was not open to the plaintiffs to claim value of the Timber. The
High Court, in the impugned judgment, referred to some judgments rendered in writ
petitions.
9

. Reliance on the decision without looking into the factual background of the case before
it is clearly impermissible. A decision is a precedent on its own facts. Each case presents
its own features. It is not everything said by a Judge while giving a judgment that
constitutes a precedent. The only thing in a Judges decision binding a party is the
principle upon which the case is decided and for this reason it is important to analyse a
decision and isolate from it the ratio decidendi. According to the well-settled theory of
precedents, every decision contains three basic postulates (i) findings of material facts,
direct and inferential. An inferential finding of facts is the inference which the Judge
draws from the direct, or perceptible facts; (ii) statements of the principles of law
applicable to the legal problems disclosed by the facts; and (iii) judgment based on the
combined effect of the above. A decision is an authority for what it actually decides.
What is of the essence in a decision is its ratio and not every observation found therein
nor what logically flows from the various observations made in the judgment. The
enunciation of the reason or principle on which a question before a Court has been
decided is alone binding as a precedent. (See: State of Orissa v. Sudhansu Sekhar Misra
1996 AIR SCW 4020

@page-SC866
and Ors. (AIR 1968 SC 647) and Union of India and Ors. v. Dhanwanti Devi and Ors.
(1996 (6) SCC 44). A case is a precedent and binding for what it explicitly decides and no
more. The words used by Judges in their judgments are not to be read as if they are words
in Act of Parliament. In Quinn v. Leathem (1901) AC 495 (H.L.). Earl of Halsbury LC
observed that every judgment must be read as applicable to the particular facts proved or
assumed to be proved, since the generality of the expressions which are found there are
not intended to be exposition of the whole law but governed and qualified by the
particular facts of the case in which such expressions are found and a case is only an
authority for what it actually decides.
10. Courts should not place reliance on decisions without discussing as to how the factual
situation fits in with the fact situation of the decision on which reliance is placed.
Observations of Courts are neither to be read as Euclids theorems nor as provisions of the
statute and that too taken out of their context. These observations must be read in the
context in which they appear to have been stated. Judgments of Courts are not to be
construed as statutes. To interpret words, phrases and provisions of a statute, it may
become necessary for Judges to embark into lengthy discussions but the discussion is
meant to explain and not to define. Judges interpret statutes, they do not interpret
judgments. They interpret words of statutes; their words are not to be interpreted as
statutes. In London Graving Dock Co. Ltd. V. Horton (1951 AC 737 at p.761), Lord Mac
Dermot observed :
The matter cannot, of course, be settled merely by treating the ipsissima vertra of Willes,
J as though they were part of an Act of Parliament and applying the rules of interpretation
appropriate thereto. This is not to detract from the great weight to be given to the
language actually used by that most distinguished Judge.
11. In Home Office v. Dorset Yacht Co. (1970 (2) All ER 294) Lord Reid said, "Lord
Atkins speech.....is not to be treated as if it was a statute definition. It will require
qualification in new circumstances. Megarry, J in (1971) 1 WLR 1062 observed: One
must not, of course, construe even a reserved judgment of Russell L.J. as if it were an Act
of Parliament". And, in Herrington v. British Railways Board (1972 (2) WLR 537) Lord
Morris said:
There is always peril in treating the words of a speech or judgment as though they are
words in a legislative enactment, and it is to be remembered that judicial utterances made
in the setting of the facts of a particular case.
12. Circumstantial flexibility, one additional or different fact may make a world of
difference between conclusions in two cases. Disposal of cases by blindly placing
reliance on a decision is not proper.
13. The following words of Lord Denning in the matter of applying precedents have
become locus classicus:
"Each case depends on its own facts and a close similarity between one case and another
is not enough because even a single significant detail may alter the entire aspect, in
deciding such cases, one should avoid the temptation to decide cases (as said by
Cordozo) by matching the colour of one case against the colour of another. To decide
therefore, on which side of the line a case falls, the broad resemblance to another case is
not at all decisive."
*** *** ***
"Precedent should be followed only so far as it marks the path of justice, but you must cut
the dead wood and trim off the side branches else you will find yourself lost in thickets
and branches. My plea is to keep the path to justice clear of obstructions which could
impede it."
14.As noted above, there was no challenge to the conditions stipulated and it was
accepted that the trees were reserved trees. What is the effect of this admission, was not
examined by the High Court. Therefore, looked at from any angle, the judgment of the
High Court is clearly unsustainable and is set aside. The appeal is allowed but without
any order as to costs.
Appeal allowed.
@page-SC867
AIR 2008 SUPREME COURT 867 "Hari Yadav v. State of Bihar"
(From : Patna)
Coram : 2 S. B. SINHA AND H. S. BEDI, JJ.
Criminal Appeal No. 1721 of 2007 (arising out of SLP (Cri.) No. 5256 of 2007), D/- 14
-12 -2007.
Hari Yadav v. State of Bihar.
(A) Penal Code (45 of 1860), S.300 - MURDER - ASSAULT - LIFE IMPRISONMENT -
Murderous assault - Proof - Accused persons allegedly assaulted deceased - Witnesses
stated in details about role played by appellant in inflicting farsa blow and mode and
manner of incident - Presence of appellant at scene of incidence was beyond any dispute -
Autopsy surgeon opined that injury was sufficient in ordinary course of nature to cause
death - Merely because there is possibility of injury being caused by hard and blunt
substance, same by itself cannot be a ground for holding that ocular evidence should be
disbelieved - It was not a case where there was a sudden fight - Accused came prepared at
place of occurrence - Conviction of appellant under S. 302 and sentence of life
imprisonment, proper.
2006 (9) Scale 681, Relied on. (Paras 12, 18, 20, 21, 23)
(B) Criminal P.C. (2 of 1974), S.172 - INVESTIGATION - Case diary - Slip attached to
injury report which formed part of case diary - Reliance on note made in slip - Without
proof of such document and without drawing attention of investigating officer towards it
- Improper. (Para 18)
Cases Referred : Chronological Paras
2006 (9) Scale 681 (Rel. on, Pnt. A) 20, 22
AIR 1958 SC 465 22
Sushil Kumar, Sr. Advocate, Vijay Kumar and Vishwajit Singh, for Appellant; Gopal
Singh, for Respondent.
Judgement
S. B. SINHA, J. :-Leave granted.
1. Appellant is before us aggrieved by and dissatisfied with a judgment dated 21st
January, 2004 passed by a Division Bench of the Patna High Court in Criminal Appeal
No. 341 of 1991 arising out of the judgment of conviction and sentence dated 2nd
September, 1991 passed by Additional District and Sessions Judge XIth, Gaya, in
Sessions Trial No. 12 of 1991.
2. A land dispute arose between the accused and the prosecution witnesses in relation to a
piece of land bearing plot No. 1/555 of village Gamahariya. While measurement of the
said land was going on, both the parties, viz., Kameshwar Yadav and Chander Dusadh,
claimed their right thereupon, on the basis of purchases made by them under their
respective registered deeds of sale.
3. A quarrel ensued between both the parties. Kameshwar Yadav exhorted others to
eliminate Chander Dusadh. It was followed by an assault on Kali Dusadh by a hard and
blunt object by him. Jaldhari Yadav assaulted Kali Dusadh on his right arm. Parmeshwar
Yadav assaulted him on his back. Appellant herein inflicted a farsa blow on the head of
Chander Dusadh, the deceased. Kameshwar Yadav gave another blow on his hand by a
hard and blunt object. As alarm was raised by Kali Dusadh, whereupon assailants fled
from the field. They allegedly resorted to firing also.
4. Thereafter, injured Chander Dusadh was taken to the Police Station in an unconscious
state.
5. A First Information Report in regard to the said incident was lodged at 10 pm on 11th
August, 1981. Appellant amongst others was named therein. The prosecution during trial
examined a number of witnesses in support of its case. PW 1, Bisu Bhuiya categorically
stated about infliction of garassa blow by Hari Yadav and lathi blow by Kameshwar
Yadav on the deceased. Role played by Kameshwar Yadav, Kishun Yadav, Hari Yadav,
Parmeshwar Yadav, Bhuja Yadav, Rohan Yadav and Gopal Yadav in inflicting injuries on
Kali Dusadh were also categorically stated by him in his deposition.
6. Similarly, Barat Dusadh (PW 3) deposed about the role played by the Appellant in
inflicting a farsa blow on the head of Chander Dusadh, having been exhorted to do so by
his father. He not only stated in details in regard to the mode and manner in which the
incident took place but also the cause thereof. Similar is the evidence of Aminullah Khan
(PW 5) and Gazi Khijer Heyat (PW 6).
7. P.W. 8 Dr. M.K. Sinha, who conducted the post mortem examination on the dead body
of the deceased, in his report noticed three ante mortem injuries which are as under:
"(i) Recently healed linear wound of
@page-SC868
length 4" over top of head. On dissection underlined tissues were found infiltrated with
blackish blood clot. There was fracture of both parietal bones. On removal of skull cap,
there was presence of extradural and subdural haematoma over superior surface of both
cerebral hemisphere. Brain and meninges were found grossly congested.
(ii) Healed abrasion over front of lower on part of the right knee with presence of dry,
black scale over the area, size 1-1/4' x ½'.
(iii) Swelling over antero lateral aspect of right arm upper part size 2-1/2" circumference.
"
8. On analyzing the materials brought on records, the Learned Sessions Judge found the
appellant guilty of commission of offence under Section 302 of the Indian Penal Code.
The Learned Judge, however, while considering the facts and circumstances of each of
the accused at the time of occurrence imposed a sentence of ten years' rigorous
imprisonment under Section 302 of the Indian Penal Code to accused Kameshwar Yadav
and Hari Yadav and one year's rigorous imprisonment to Jaldhari Yadav and Parmeshwar
Yadav and Kameshwar Yadav under Section 323 of the Indian Penal Code.
9. Three Criminal Appeals were preferred by the accused persons. The High Court
dismissed the said appeals, but keeping in view the fact that the Learned Sessions Judge
committed a serious error in imposing the punishment of 10 year's rigorous imprisonment
for commission of an offence under Section 302 of the Indian Penal Code, imposed the
sentence of rigorous imprisonment for life. Appellant is, thus, before us.
10. Mr. Sushil Kumar, learned Senior Counsel appearing on behalf of the appellant, inter
alia, would submit that the occurrence having taken place on 11.8.1981 and the death of
deceased having taken place on 1.9.1981, the appellant cannot be stated to have
committed any offence under Section 302. It was pointed that according to the Doctor,
the deceased left the hospital on 27.8.1981 without his permission and thus he might have
died of some other disease in between the period 27.8.1981 and 1.9.1981. The Learned
Counsel submitted that keeping in view the injuries suffered by the deceased, the same
were possibly caused by a lathi.
11. Mr. Gopal Singh, learned counsel appearing on behalf of the State on the other hand,
supported the impugned judgment.
12. Presence of the appellant at the scene of the incidence is beyond any dispute. The
autopsy surgeon in his evidence while proving his report, identified three injuries
appearing on the person of the deceased. Injury No. 1 was found to be grievous in nature
and dangerous to life which was likely to be caused by sharp weapon such as farsa
whereas other injuries which were simple in nature could have been caused by hard and
blunt object (may be lathi). He was of the categorical opinion that the injury No.1, in
ordinary course of nature, was sufficient to cause death.
13. In cross examination his attention was drawn to the book "Modi's Medical
Jurisprudence and Toxicology (22nd Edition)". In answer to a question, he stated: Linear
abrasion can be possible by lathi as well. Volunteers it can also be caused by sharp
weapon. If the margin of the wound is sharp, it is inferred that it was caused by sharp
weapon.
14. We may notice that it was categorically stated by the said witness that there was no
provision for treatment of such injury and such cases are ordinarily referred to neuro
surgeon at Ranchi.
15. It appears that on 27.8.1981, a report was sent that deceased died in Gaya hospital on
1.9.1981.
16. Our attention has been drawn to one slip attached to the said report wherein a prayer
was made for insertion of Section 302 of Indian Penal Code in the said case, which reads
as under:
In continuation of Injury Report of Chandra Gorait of Singh Pokhar, Sherghati I have to
inform you that he has developed unomiplagia and he left the hospital on 27.8.1981
without the knowledge of undersigned. He has not submitted x-ray of right hand till now.
This is for information and necessary action."
17. Submission of the learned senior counsel Sh. Sushil Kumar is that the deceased left
hospital on 27.8.1981 without any information to the doctor therefore, cannot be
accepted. The fact that the deceased died in the District Hospital is not in dispute. We
have noticed hereinbefore that the Doctor himself suggested that there was no provision
for treatment of such patients at Sherghatti. Evidently, therefore,
@page-SC869
the relatives of the deceased took him to the District Hospital for better treatment. For the
said purpose, the consent of the doctor might not have been taken or brought to the
personal knowledge of the doctor concerned.
18. It is, however, significant that the aforementioned quotation was made from the
purported note made by somebody which formed part of the case diary. The said
document was not proved. Attention of the investigating officer was not drawn thereto.
No such question appears to have been raised before the High court. We are really at a
loss to understand as to how reliance has been placed thereupon on the basis of a piece of
paper which appeared in the case diary. We deprecate such a practice.
It may be of some interest to notice that Dr. S.P. Gupta has used the word unomiplagia.
We have not been able to find what it means in the medical dictionary.
19. Deposition of Dr. S.P. Gupta who was examined before the Learned Sessions Judge as
PW 10 has not been brought on record by the appellants. The reason therefor appears to
us as obvious. Several unsustainable pleas have been raised before the Trial Court on
behalf of the appellant. It appears that at one point of time a plea of insanity has as also
his having no relationship with the other accused, had also been taken. It appears from
the records that he had also absconded for some time.
20. Submission of Mr. Sushil Kumar that such an injury can be caused by hard and blunt
substance may be correct in view of the statements made by the autopsy surgeon but
merely because there is a possibility in regard thereto, the same by itself cannot be a
ground for holding that ocular evidence should be disbelieved. There are a large number
of authorities of this Court which clearly show that in certain situations, the wound
produced by a blunt instrument may similarly seem to be an incised one. [See Kailash v.
State of MP, 2006 (9) SCALE 681].
21. It was not a case where there was a sudden fight. The accused came prepared at the
place of occurrence. An altercation might have taken place but not only repeated assaults
were made, other witnesses also suffered injuries.
22. Each case must be decided on its own facts as has been held in Kailash (supra).

The law in this regard was laid down in Kailash (supra) in the following terms: In Virsa
Singh v. The State of Punjab [1958 SCR 1495] wherein Vivian Bose, J. opined that
infliction of one injury by accused may be sufficient to hold him guilty for commission of
an offence under Section 302 of the Indian Penal Code stating : AIR 1958 SC 465

In considering whether the intention was to inflict the injury found to have been inflicted,
the enquiry necessarily proceeds on broad lines as, for example, whether there was an
intention to strike at a vital or a dangerous spot, and whether with sufficient force to
cause the kind of injury found to have been inflicted. It is, of course, not necessary to
enquire into every last detail as, for instance, whether the prisoner intended to have the
bowels fall out, or whether he intended to penetrate the liver or the kidneys or the heart.
Otherwise, a man who has no knowledge of anatomy could never be convict, for, if he
does not know that there is a heart or a kidney or bowels, he cannot be said to have
intended to injure them. Of course, that is not the kind of enquiry. It is broad-based and
simple and based on commonsense; the kind of enquiry that twelve good men and true
could readily appreciate and understand.
To put it shortly, the prosecution must prove the following facts before it can bring a case
under Section 300. First, it must establish, quite objectively, that a bodily injury is
present;
Secondly, the nature of the injury must be proved; These are purely objective
investigations.
Thirdly, it must be proved that there was an intention to inflict that particular bodily
injury, that is to say, that it was not accidental or unintentional, or that some other kind of
injury was intended.
Once these three elements are proved to present, the enquiry proceeds further and,
Fourthly, it must be proved that the injury of the type just described made up of the three
elements set out above is
@page-SC870
sufficient to cause death in the ordinary course of nature. This part of the enquiry is
purely objective and inferential and has nothing to do with the intention of the offender.
23. For the reasons aforementioned, there is no merit in this appeal, which accordingly is
dismissed.
Appeal dismissed.
AIR 2008 SUPREME COURT 870 "Poonam Verma v. Delhi Development Authority"
(From : Delhi)*
Coram : 2 S. B. SINHA AND H. S. BEDI, JJ.
Civil Appeal No. 5874 of 2007 (arising out of SLP (Civ.) No. 1322 of 2007) D/- 13 -12
-2007.
Poonam Verma and Ors. v. Delhi Development Authority.
(A) Delhi Development Act (61 of 1957), S.41 - ALLOTMENT OF PREMISES -
PLANNING AND DEVELOPMENT - GENERAL CLAUSES - EQUALITY - Powers of
Central Govt. -Scope - Allotment of flats in scheme floated by Development Authority -
Central Govt. have no power to direct out of turn allotment - Ss. 41, 56(2)(r) not
applicable - S. 21 of General Clauses Act would also not be attracted - Moreso when
scheme was closed - Plea of legitimate expectation on basis of order of Central Govt. -
Not tenable.
General Clauses Act (10 of 1897), S.21.
Constitution of India, Art.14.
The Central Govt. not shown to have any power to interfere with the day to day affairs of
the respondent Development Authority. S. 41 only envisages that the respondent would
carry out such directions that may be issued by the Central Govt. from time to time for
the efficient administration of the Act. The same does not take within its fold an order,
which can be passed by the Central Govt. in the matter of allotment of flats by the
Authority. S. 41 speaks about policy decision. Any direction issued must have a nexus
with the efficient administration of the Act. It has nothing to do with carrying out of the
plans of the authority in respect of a particular scheme. The Central Govt. does not have
any quota under the Act. It did not have any quota under the Scheme. The Central Govt.
had no say in the matter either on its own or under the Act. In terms of the Brochure, S.
41 does not clothe any jurisdiction upon the Central Govt. to issue such a direction. The
Central Govt. could issue the said direction in exercise of its rule making power under S.
56 of the Act is wholly misplaced. In issuing the said letter, the Central Govt. did not
exercise its legislative power nor could it do so. The Central Govt. in terms of the Act
apart from S. 41 did not have any power and, thus, could not have issued any direction in
terms thereof. If S. 41 of the Act or for that matter S. 56(2) (r) thereof were not
applicable, the question of issuing any direction purported to be in terms of S. 21 of the
General Clauses Act, did not arise. (Paras 15, 16)
Having professed to abide by the Brochure which contained the policy of reservation for
allotment of flats under Self Financing Scheme floated by Authority, the Central Govt.
could not in absence of any statutory provision directed creation of any quota and that too
after closure of the Scheme. The Scheme after its closure could not even have been
revived. (Paras 22, 23)
Legitimate expectation would apply only when a practice is found to be prevailing. It has
a positive concept. But, in a case of this nature where purported expectation is based on
an illegal and unconstitutional order, the same is wholly inapplicable, as the same cannot
be founded on an order, which is per se illegal, and without foundation. (Para 27)
(B) Constitution of India, Art.226 - WRITS - ADMINISTRATIVE LAW -
DEVELOPMENT AUTHORITY - Administrative law - Recourse to administrative
remedies - Registrants under flat scheme - Having failed to establish any legal right in
themselves as also purported deficiency in services on part of Development Authority
before competent legal forums - Not entitled to take recourse to remedies on
administrative side which stricto sensu were not available. (Para 12)
(C) Constitution of India, Art.245 - LEGISLATION - Statute - Guidelines per se do not
partake to character of statute - Such guidelines in absence of the statutory backdrop are
advisory in nature - Do not confer any legal right. (Para 24)
Cases Referred : Chronological Paras
2007 AIR SCW 6395 (Ref.) 22
2006 AIR SCW 3120 : AIR 2006 SC 2360 (Ref.) 17
@page-SC871

2006 AIR SCW 5312 : 2006 Lab IC 4241 : 2007 (1)) AIR Jhar R 373 (Expln.) 27
2003 AIR SCW 1848 : AIR 2003 SC 1405 (Ref.) 28
(2002)100 DLT 497 (Ref.) 24
1992 AIR SCW 1247 : AIR 1992 SC 1277 : 1992 Lab IC 1371 (Ref.) 26
AIR 1989 SC 100 (Disting.) 25
AIR 1989 SC 2138 (Ref.) 24
AIR 1988 SC 1681 (Ref.) 24
AIR 1981 SC 234 (Ref.) 24
AIR 1979 SC 1628 (Ref.) 22
AIR 1973 SC 303 : 1973 Lab IC 191 (Ref.) 30
(1959) 359 US 535 22
Poonam Verma Appellant; in person, Ashwani Kumar, for Respondent.
* C. W. Nos. 19633-35 of 2005, Judgment Dated 18-8-2006 with orders dated 25-4-2006
and 20-3-2006 (Del).
Judgement
S. B. SINHA, J. :- Leave granted.
2. Respondent is an authority created under the Delhi Development Act, 1957 (for short
the Act). The Act was enacted to provide for the development of Delhi according to plan
and for matters connected therewith or ancillary thereto.
Respondent floated a scheme known as Fifth Self Financing Housing Registration
Scheme, 1982 (for short the Scheme). Appellants herein pursuant to an advertisement
issued in this behalf registered themselves; their registration numbers being 13463, 16602
and 13464. For the purpose of allotment of flats, lots were drawn on various occasions,
viz., in June, 1987, November, 1987, March, 1989, July, 1990, January, 1991, January,
1993. Appellants were not successful therein and, thus, were unable to get flats in locality
of their choice. The Scheme was closed. However, with a view to give a chance to those
who were not successful in the lots on the earlier occasions, a public notice was issued in
some newspapers on 8.12.1993 for release of about 3000 flats which included some built
and ready-built ones situated in Kondli-Gharoli. Registrants under the Scheme were
entitled to apply therefor. In the public notice, it was categorically stated that the
registrants of the said scheme who had not applied for an allotment in that release would
not be eligible to apply again for allotment. It was further stated that in the case
registrants of 5th SFS did not avail of this opportunity or if they surrendered allotment/
allocation after being successful, they shall be deemed to have opted out of the scheme
and action shall be taken to refund their registration money.
3. Appellants did not respond to the said notice. Despite the same allegedly they had been
allotted Category- III flats. They were called upon to pay the price specified therefor and
to take delivery thereof. They declined to do so. They asked their names to be included in
the VI and VI-A Self Financing Schemes which were issued later on. Respondent did not
agree thereto.
4. A complaint was filed by the appellants before the Consumer Disputes Redressal
District Forum II on or about 16.01.1995 inter alia for a direction upon the respondent
herein that their registration should not be cancelled and they should be considered in
future draw of lots till they could be allotted flats in the locality of their choice. By a
judgment and order dated 24.07.1995, the said application was allowed holding that the
action of the respondent in not considering the cases of the appellants for allotment
through the process of draw of lots amounted to unfair trade practice, apart from being
unilateral and unjustified. Aggrieved by and dissatisfied therewith, the respondent
preferred an appeal before the State Consumer Disputes Redressal Commission, New
Delhi and by an order dated 30.11.1998 allowed the said appeal and set aside the order of
the District Forum. Appellants herein thereafter filed a revision application before the
National Consumer Disputes Redressal Commission. During pendency of the said
application, they approached the Finance Member and Chairman of the respondent to
place their case before the out of court settlement committee. By an order dated
25.11.1999, the National Commission dismissed the revision petition filed by the
appellants herein relying inter alia on Clause 16 of the Brochure wherein it had
categorically been stated that DDA reserves the right to withdraw the Scheme at any
time. A Special Leave Petition preferred thereagainst was dismissed.
5. Although the appellants were not successful in their attempt to obtain any remedy on
the judicial side, they purported to approach the Ministry of Urban Affairs in 1997. They
made certain representations. Allegedly, the Joint Secretary (D and L) by reason of a
letter dated 24.08.2000 addressed to the Vice-Chairman of Delhi Development Authority
directed as under :
"I am directed to refer to D.O. letter No. F.1 (Misc.) 5th SFS and onwards/2000/SFS,
dated the 15th May, 2000 from Shri Arvind
@page-SC872
Kumar, the then Commissioner (Housing), on the subject noted above, and to state that
the matter pertaining to giving one more opportunity to the left out registrants of 5th and
subsequent Self Financing Schemes was discussed in the Chamber of UDM with VC,
DDA some time back. After discussion, it was agreed that instead of a general scheme,
VC, DDA would cover the pending petitioners, especially, the hard cases under the OTA
quota. It was also mentioned by the VC, DDA that there are only three such cases. It is,
therefore, requested that further action to allot the flats to these three petitioners may
please be taken and action taken in the matter may be intimated to this Ministry in due
course."
6. The State Consumer Disputes Redressal Commission was again approached. The
application of the appellants was dismissed. Another application was filed by them before
the Permanent Lok Adalat for non-compliance of orders of Ministry of Urban
Development despite availability of flats. By an order dated 6.09.2005, the Lok Adalat
observed :
"On 12.4.2005, Lok Adalat had recommended that the case of the petitioner is a hard case
and instead of General Scheme the case of the petitioner should be considered under the
Out of Turn Allotment quota particularly when there are only three cases left. In this
connection a letter of Minister of Urban Development and Poverty Alleviation dated
24.8.2005 refers to. This letter clearly provides that according to the Vice-Chairman,
DDA there are only three such cases left and in such a situation the case of the petitioner
should be covered under OTA Quota being a hard case. This recommendation has not
been accepted by the DDA presumably for the reason that the scheme of SFS under
which the petitioner had applied had become defunct. The scheme of OTA under the
quota is also no longer in existence and as such the case of the petitioner cannot be
considered under this category. The petitioner cannot be allotted a flat as the flats which
are lying vacant for which the petitioner has applied for the DDA has merged the flats
with the Higher Income Group. In other words, the DDA in the aforesaid circumstances
has opposed such allotment to the petitioner. There is no meeting ground between the
parties, the matter is closed as unsettled. The petitioner is at liberty to approach
appropriate Forum/ Court of Law for redressal of his grievances if she is so advised."
7. Thereafter, a writ petition was filed before the Delhi High Court which was marked as
Civil Writ No. 19633-35 of 2005. By an order dated 20.03.2006, the said petition was
dismissed. Letters Patent Appeal preferred thereagainst by the appellants being LPA No.
652-654 of 2006 has also been dismissed by reason of the impugned judgment dated
25.04.2006. A review application filed thereagainst also stand dismissed.
8. Mr. Ram Prakash, representing the appellants, in support of this appeal inter alia would
submit :
(i) As in the Brochure, a policy of reservation was provided, the High Court committed a
serious illegality in opining that no legal right accrued in their favour in terms of the said
letter dated 24.08.2000.
(ii) The Central Government, having regard to Section 41 read with Section 56(2)(r) of
the Act, could direct allotment of flats from out of turn quota keeping in view the cases of
the appellants who were three in number, as falling in the category of hard cases.
(iii) The Central Government in a situation of this nature was entitled to formulate a
Scheme for the left out registrants. The authorities of the respondent having participated
in the meeting with the Minister of Urban Development, pursuant whereto the said letter
dated 24.08.2000 was issued, the respondent was bound to implement the same in view
of the principles of Legitimate Expectation and Promissory Estoppel.
(iv) As a large number of flats had been vacant, as would appear from the statement made
by the Vice-Chairman of the respondent on 8.11.2002 by reason of allotment of the flats,
nobody else would be prejudiced.
9. Mr. Ashwani Kumar, learned counsel appearing on behalf of the respondent, on the
other hand, submitted :
(i) Appellant do not have any legal right in obtaining allotment of flats.
(ii) They having failed to deposit the amount as far back in 1994 cannot now be permitted
to claim an equitable right despite their unsuccessful attempt before the Forums created
under the Consumer Protection Act, 1985.
10. Indisputably, the Scheme was an independent one. It was a Self Financing Housing
Registration Scheme. Other similar
@page-SC873
schemes following the same were also wholly independent of each other. The Brochure
issued for enforcing the said Scheme is a self-contained document. It provides for the
mode and manner in which flats are to be allotted, the categories of the allotment of flats
thereof, mode of payment as also cancellation thereof. Indisputably, despite the fact that
the appellants were not successful in obtaining the flats by reason of draw of lots and
despite the fact that they did not respond to the notice issued by the respondent, those
cases had not been considered in the year 1994. On what ground, we do not know, flats
were allotted in their favour. They were asked to make deposits. They did not do so.
They, on the other hand, made a totally untenable claim of continuing their registration
again in VI and VI-A Schemes.
11. We have noticed hereinbefore that their claim based on deficiency of service and/ or
unfair trade practice was rejected by the Higher Forum on the part of the respondent.
They lost their battle up to this Court in the first round of litigation.
12. Having failed to establish any legal right in themselves as also purported deficiency
in services on the part of the respondent before competent legal forums, they took
recourse to remedies on administrative side which stricto sensu were not available. It has
not been shown as to on what premise the Central Government can interfere with the day
to day affairs of the respondent. Section 41 of the Act, only envisages that the respondent
would carry out such directions that may be issued by the Central Government from time
to time for the efficient administration of the Act. The same does not take within its fold
an order which can be passed by the Central Government in the matter of allotment of
flats by the Authority. Section 41 speaks about policy decision. Any direction issued must
have a nexus with the efficient administration of the Act. It has nothing to do with
carrying out of the plans of the authority in respect of a particular scheme.
13. The Central Government does not have any quota under the Act. It did not have any
quota under the Scheme. The reservations envisaged in terms of the Scheme were as
under :
a) 25% of the flats for the persons belonging to SC/ST.
b) 3% of the flats for MPs.
c) 2% of the flats for persons who have won national recognition in the field of sports, art
and music.
d) 1% of the flats for physically handicapped.
14. Evidently, the Central Government had no say in the matter either on its own or under
the Act. In terms of the Brochure, Section 41 of the Act does not clothe any jurisdiction
upon the Central Government to issue such a direction.
15. Submission of Mr. Ram Prakash that the Central Government could issue the said
direction in exercise of its rule making power under Section 56 of the Act is wholly
misplaced. In issuing the said letter, the Central Government did not exercise its
legislative power nor could it do so. The Central Government in terms of the Act apart
from Section 41 did not have any power and, thus, could not have issue any direction in
terms thereof.
16. If Section 41 of the Act or for that matter Section 56(2)(r) thereof were not applicable,
the question of issuing any direction purported to be in terms of Section 21 of the General
Clauses Act, as has been submitted by Mr. Ram Prakash, did not arise.

17

. M.P. Gangadharan and Others v. State of Kerala and Others [(2006) 6 SCC 162],
whereupon reliance has been placed by Mr. Ram Prakash, has no application in the
instant case. 2006 AIR SCW 3120

18. The Scheme in question was closed as far back as in the year 1994. The Central
Government in terms of the provisions of the Act or otherwise had no jurisdiction to
revive the same.
19. All the authorities under the Act including the Central Government being the creature
of statute were bound to act within the four corners thereof. A specific grievance was
raised by the appellants herein that the action on the part of the authority amounted to
unfair trade practice. Deficiency of service was also pleaded. The same had been
negatived. The courts having appropriate jurisdiction having found neither unfair trade
practice nor there being deficiency in service and in that view of the matter, the Central
Government ordinarily ought not to have interfered in the matter.
20. The purported letter dated 24.08.2000 does not specify as to how the Central
Government assumed any
@page-SC874
jurisdiction in the matter.
21. Some officers of the respondent by themselves could not have evolved a Scheme
which was beyond the purview and scope of the Act. Respondent being a State within the
meaning of Article 12 of the Constitution of India is bound to fulfill the constitutional
scheme contained in Article 14 thereof. It could not, going behind the professed scheme
as contained in the Brochure, create a quota. Such a purported decision being wholly
without jurisdiction, is a nullity. The Central Government itself directed the authority to
confine the 'out of turn allotment' quota by reason of a direction issued in June, 2000 only
for widows of:
(a) Government servants who dies in harness.
(b) Those who were killed by terrorists.
It would be preposterous to suggest that the Central Government could act beyond its
professed policy decision.
22. The Central Government, thus, acted illegally and without jurisdiction in purporting
to take a decision that the hard cases may be brought within the purview of the 'Out of
Turn Allotment' Quota, as therefor there was no legal sanction.
Justice Frankfurter in Vitarelli v. Seaton [359 US 535] stated :
"An executive agency must be rigorously held to the standards by which it professes its
action to be judged................. Accordingly, if dismissal from employment is based on a
defined procedure, even though generous beyond the requirements that bind such agency,
that procedure must be scrupulously observed. .........This judicially evolved rule of
administrative law is now firmly established and, if I may add, rightly so. He that takes
the procedural sword shall perish with the sword."

[See also Ramana Dayaram Shetty v. The International Airport Authority of India and
Others, AIR 1979 SC 1628 : (1979) 3 SCC 489; Harjit Singh and Anr. v. The State of
Punjab and Anr., 2007 (3) SCALE 553) ] 2007 AIR SCW 6395

23. Having professed to abide by the Brochure which contained the policy of reservation,
as noticed hereinbefore, the Central Government could not in absence of any statutory
provision directed creation of any quota and that too after closure of the Scheme. The
Scheme after its closure could not even have been revived.
24. Guidelines per se do not partake to the character of statute. Such guidelines in
absence of the statutory backdrop are advisory in nature. Mr. Ram Prakash himself has
relied upon a decision of this Court in Narendra Kumar Maheshwari v. Union of India
and Others [AIR 1989 SC 2138] wherein it has been laid down :
"100 ........This is because guidelines, by their very nature, do not fall into the category of
legislation, direct, subordinate or ancillary. They have only an advisory role to play and
non-adherence to or deviation from them is necessarily and implicitly permissible if the
circumstances of any particular fact or law situation warrants the same. Judicial control
takes over only where the deviation either involves arbitrariness or discrimination or is so
fundamental as to undermine a basic public purpose which the guidelines and the statute
under which they are issued are intended to achieve."

[See also Narendra Kumar Maheshwari v. Union of India and others, 1990 (Supp) SCC
440 at 508; Maharao Sahib Shir Bhim Singhji v. Union of India and others, (1981) 1 SCC
166 at 232; J.R. Raghupathy and others v. State of A.P. and others, (1988) 4 SCC 464
(paragraph 31); Uttam Parkash Bansal and others v. L.I.C. of India, (2002) 100 DLT 487]
AIR 1989 SC 2138
AIR 1981 SC 234
AIR 1988 SC 1681

Guidelines being advisory in character per se do not confer any legal right.
25. Reliance has also been placed upon P. M. Ashwathanarayana Setty and others v. State
of Karnataka and others [AIR 1989 SC 100] for the proposition that the State cannot rely
on an evasive reason. We fail to understand how a case relating to Court Fees and Suits
Valuation Act, would assist us in invoking the principles in regard to the discriminatory
impact of the matter in a case of this nature.
26
. Mr. Ram Prakash has also placed reliance upon State of Himachal Pradesh and Another
v. Kailash Chand Mahajan and Others [1992 Supp (2) SCC 351] wherein this Court was
considering the statutory conditions of services framed under a regulation made in terms
of Electricity (Supply) Act. In that context, this Court considered the question as to
whether the term of appointment can be 1992 AIR SCW 1247

@page-SC875
confined to a single person. Reliance placed on the said decision is wholly misplaced. A
reasonable classification is permissible although a class legislation is not, but the same
will have no application in a case where an executive order was passed wholly without
jurisdiction and contrary to the constitutional scheme relating to fixation of quota for
certain categories of persons.
27. An endeavour has been made to invoke the principles of Legitimate Expectation and
Promissory Estoppel. The doctrine of Legitimate Expectation would apply only when a
practice is found to be prevailing. It has a positive concept. But, in a case of this nature
where purported expectation is based on an illegal and unconstitutional order, the same is
wholly inapplicable, as the same cannot be founded on an order which is per se illegal
and without foundation.

Strong reliance has also been placed on a decision of this Court in Ram Pravesh Singh
and Others v. State of Bihar and Others [(2006) 8 SCC 381] wherein a Bench of this
Court opined : 2006 AIR SCW 5312

"15. What is legitimate expectation? Obviously, it is not a legal right. It is an expectation


of a benefit, relief or remedy, that may ordinarily flow from a promise or established
practice. The term established practice refers to a regular, consistent, predictable and
certain conduct, process or activity of the decision-making authority. The expectation
should be legitimate, that is, reasonable, logical and valid. Any expectation which is
based on sporadic or casual or random acts, or which is unreasonable, illogical or invalid
cannot be a legitimate expectation. Not being a right, it is not enforceable as such. It is a
concept fashioned by the courts, for judicial review of administrative action. It is
procedural in character based on the requirement of a higher degree of fairness in
administrative action, as a consequence of the promise made, or practice established. In
short, a person can be said to have a legitimate expectation of a particular treatment, if
any representation or promise is made by an authority, either expressly or impliedly, or if
the regular and consistent past practice of the authority gives room for such expectation
in the normal course. As a ground for relief, the efficacy of the doctrine is rather weak as
its slot is just above fairness in action but far below promissory estoppel. It may only
entitle an expectant: ( a ) to an opportunity to show cause before the expectation is
dashed; or ( b ) to an explanation as to the cause for denial. In appropriate cases, the
courts may grant a direction requiring the authority to follow the promised procedure or
established practice. A legitimate expectation, even when made out, does not always
entitle the expectant to a relief. Public interest, change in policy, conduct of the expectant
or any other valid or bona fide reason given by the decision-maker, may be sufficient to
negative the legitimate expectation. The doctrine of legitimate expectation based on
established practice (as contrasted from legitimate expectation based on a promise), can
be invoked only by someone who has dealings or transactions or negotiations with an
authority, on which such established practice has a bearing, or by someone who has a
recognised legal relationship with the authority.........."
The said decision, thus, instead of assisting the appellants runs counter to their
contention.
28

. Mr. Ram Prakash has also placed strong reliance on J. P. Bansal v. State of Rajasthan
[(2003) 5 SCC 134 : 2003 (3) Scale 154]. Therein itself, it is laid down : 2003 AIR
SCW 1848, (Para 8)

"......The Constitution requires that action must be taken by the authority concerned in the
name of the Governor. It is not till this formality is observed that the action can be
regarded as that of the State. Constitutionally speaking, the Council of Ministers are
advisers and as the Head of the State, the Governor is to act with the aid or advice of the
Council of Ministers. Therefore, till the advice is accepted by the Governor, views of the
Council of Ministers do not get crystallised into action of the State.........."
29. This decision is, therefore, an authority for the proposition that the government order,
so as to confer a legal right, must conform to the provisions contained in Article 166 of
the Constitution of India.
30. Questioning the correctness of the observation of the Division Bench that the
communication contained in the letter dated 24.08.2000 did not confer any legal right,
Mr. Ram Prakash, would submit that an administrative order may also confer a legal
right. No doubt, it was so stated in Union of India v. K. P. Joseph and others [(1973) 1
SCC 194 : AIR 1973 SC 303] but then it was
@page-SC876
a case where an executive order was passed which was within the jurisdiction of the State
in terms of the proviso appended to Article 309 of the Constitution of India. The Bench, it
is interesting to note, hastened to add :
"11. We should not be understood as laying down any general proposition on this
question. But we think that the Order in question conferred upon the first respondent the
right to have his pay fixed in the manner specified in the Order and that was part of the
conditions of his service. We see no reason why the Court should not enforce that right."
31. We, therefore, find no merit in this appeal which is dismissed accordingly. In the facts
and circumstances of this case, however, there shall be no order as to costs.
Appeal dismissed.
AIR 2008 SUPREME COURT 876 "New India Assurance Co. Ltd. v. Nusli Neville
Wadia"
(From : Bombay)*
Coram : 2 S. B. SINHA AND H. S. BEDI, JJ.
Civil Appeal Nos. 5879 with 5871 of 2007 (arising out of SLP (C) Nos. 8232 with 10348
of 2006), D/- 13 -12 -2007.
New India Assurance Co. Ltd. v. Nusli Neville Wadia and Anr.

With
New India Assurance Co. Ltd. v. KLM Engineering Co. Pvt. Ltd. and Ors.
(A) Public Premises (Eviction of Unauthorised Occupants) Act (40 of 1971), S.5 - Public
Premises (Eviction of Unauthorised Occupants) Rules (1971), R.5 - PUBLIC PREMISES
- EVICTION - UNAUTHORISED OCCUPATION - Eviction - Notice issued to occupant
- He not only entitled to show cause but also entitled to produce evidence in support of
cause shown.
The Estate Officer with a view to determine the lis between the parties must record
summary of the evidence. Summary of the evidence and the documents shall also form
part of the record of the proceedings.
Procedure laid down for recording evidence is stated in the Rules. The Estate Officer
being a creature of the statute must comply with the same. When a notice for eviction is
issued, the occupant of the public premises would not only be entitled to show cause but
would also be entitled to produce evidence in support of the cause shown. (Para 34)
(B) Public Premises (Eviction of Unauthorised Occupants) Act (40 of 1971), S.5 -
PUBLIC PREMISES - EVICTION - UNAUTHORISED OCCUPATION - Eviction -
Proceedings for - Procedural aspect as to who should lead evidence first - Have to be
determined on basis of issues arising in matter.
The procedural aspect as to who should lead evidence first in the proceedings for eviction
of unauthorised occupant, may have to be determined on the basis of the issues arising in
the matter. It does not mean that the procedure involved being a summary one, the issues
are required to be specifically framed but that which is the principal issue (s) between the
parties must be known to the Estate Officer.
Thus under the Act, the occasion would arise for multi-level inquiry : Primary inquiry
will be to arrive at a conclusion on "Unauthorized occupant"; and intermediate inquiry
would be as to the eviction of "Unauthorized occupant". (Para 35)
The statute, although, does not require a lengthy hearing or a lengthy cross-examination
but the noticee should be given an opportunity to file an effective show cause. An
effective show cause can be filed when eviction is sought for a specified ground and the
occupants must know the particulars in relation thereto. (Para 38)
If some facts are to be proved by the landlord, indisputably the occupant should get an
opportunity to cross-examine. The witness who intends to prove the said fact has the right
to cross-examine the witness. This may not be provided by under the statute, but is being
a part of the principle of natural justice should be held to be indefeasible right. (Para
44)
(C) Public Premises (Eviction of Unauthorised Occupants) Act (40 of 1971), S.5 -
EVIDENCE - EQUALITY - INTERPRETATION OF STATUTES - DOCTRINES -
Eviction of unauthorised occupant - Proceeding for - Adduction of evidence - Literal
construction of provisions of Act and Rules place entire onus on assessee - Leads to
anomalous situation - Thus they are to be construed in light of action of State as
envisaged under Art. 14 of Constitution - For that doctrine of purposive construction may
be resorted to.
@page-SC877

Constitution of India, Art.14.


Interpretation of Statutes - Purposive Construction.
Section 5, on a plain reading, would place the entire onus upon a noticee unauthorised
occupant. It, in no uncertain terms, states that once a notice under Section 4 is issued by
the Estate Officer on formation of his opinion as envisaged therein it is for the noticee not
only to show cause in respect thereof but also adduce evidence and make oral
submissions in support of his case. Literal meaning in a situation of this nature would
lead to a conclusion that the landlord is not required to adduce any evidence at all nor it is
required even to make any oral submissions. Such a literal construction would lead to an
anomalous situation because the landlord may not be heard at all. It may not even be
permitted to adduce any evidence in rebuttal to the one adduced by the noticee nor it
would be permitted to advance any argument. This is not contemplated in law. When a
landlord files an application, it in a given situation must be able to lead evidence either at
the first instance or after the evidence is led by the noticee to establish its case and/or in
rebuttal to the evidence led by the noticee.
The literal interpretation of the staute, if resorted to, would also lead to the situation that
it would not be necessary for the landlords in any situation to plead in regard to its need
for the public premises. It could just terminate the tenancy without specifying any cause
for eviction.
Except in the first category of cases in which occupants continuing to occupy premises
without any authority, Sections 4 and 5 may have to be construed differently. If the
landlord being a State within the meaning of Art. 12 of the Constitution of India is
required to prove fairness and reasonableness on its part in initiating a proceeding, it is
for it to show how its prayer meets the constitutional requirements of Article 14 of the
Constitution of India. For proper interpretation not only the basic principles of natural
justice have to be borne in mind, but also principles of constitutionalism involved therein.
With a view to read the provisions of the Act in a proper and effective manner, literal
interpretation, if given, may give rise to an anomaly or absurdity which must be avoided.
So as to enable a superior Court to interpret a statute in a reasonable manner, the Court
must place itself in the chair of a reasonable legislator/author. So done, the rules of
Purposive Construction have to be resorted to which would require the construction of
the Act in such a manner so as to see that the object of the Act fulfilled; which in turn
would lead the beneficiary under the statutory scheme to fulfill its constitutional
obligations. (Paras 48, 49, 50)
The provisions of the Act and the Rules in this case, are thus required to be construed in
the light of the action of the State as envisaged under Art. 14 of the Constitution of India.
With a view to give effect thereto, the doctrine of purposive construction may have to be
taken recourse to. (Para 53)
Act Provided for a speedy remedy. The Estate Officer is expected to arrive at a decision
as expeditiously as possible. The provisions of the Code of Civil Procedure and Evidence
Act being not applicable, what is necessary to be complied with is the principles of
natural justice. (Para 56)
(D) Public Premises (Eviction of Unauthorised Occupants) Act (40 of 1971), S.5 -
PUBLIC PREMISES - EVICTION - ADJOURNMENTS - Eviction - Proceedings for -
Authority filed affidavit - Pleaded requirements of premises on urgent basis - However, it
kept on taking adjournments for more than 2 years - Reason for not producing witnesses
before Estate Officer for cross-examination for such a long time not known - After a long
period, application was filed asking occupants, respondents to show cause - Cause
already shown by respondents, pleading that no case made out for their eviction - Order
passed by Estate Officer thereafter directing respondents to lead evidence - Held, not
proper. (Para 57)
Cases Referred : Chronological Paras
2007 AIR SCW 326 (Ref.) 47
2007 AIR SCW 1357 : 2007 Lab IC 1560 (Ref.) 55
2007 AIR SCW 3734 : AIR 2007 SC 1971 (Rel. on) 53
(2007) 6 SCC 81 (Ref.) 52
2003 AIR SCW 2938 (Ref.) 44
(2002) 100 Delhi LT 497 (Ref.) 22
1999 AIR SCW 4150 : AIR 2000 SC 33 (Ref.) 55
AIR 1992 Bom 375 37
1992 (2) CLR 457 (Ref.) 22
@page-SC878

AIR 1991 SC 855 (Rel. on) 39, 40, 43, 50


AIR 1989 SC 1642 (Ref.) 39, 40
AIR 1989 SC 2138 (Ref.) 22
AIR 1988 SC 1681 (Ref.) 22
AIR 1988 SC 1708 (Ref.) 42
AIR 1984 SC 273 : 1983 Lab IC 1680 (Ref.) 44
AIR 1984 SC 289 (Ref.) 55
AIR 1981 SC 234 (Ref.) 22
AIR 1974 SC 2009 (Ref.) 41
AIR 1972 SC 330 : 1972 Lab IC 188 (Ref.) 45
AIR 1972 SC 2205 (Ref.) 42
AIR 1968 Cal 1 (FB) 39
AIR 1967 SC 1581 (Ref.) 41
AIR 1958 Punjab 1 41
AIR 1956 All 507 41
(1954) 58 Cal WN 1066 41
(1941) 2 All ER 165 54
P. N. Lekhi, Sr. Advocate, Samar Bansal, Rajan Chaurasia, Lokesh Kumar and M. K.
Garg with him, for Appellant; E. S. Nariman, Anil Divan and Shyam Divan, Sr.
Advocates, Parag Gandhi, R. N. Karanjawala, Ms. Nandini Gole, Ms. Debmalya
Banerjee, Ms. Simran Brar, Mrs. Manik Karanjawala, Mahesh Agarwala, Rishi Agrawala
and E. C. Agrawala with them for Respondents.
Judgement
1. S. B. SINHA, J. :-Leave granted.
Introduction
2. Who should begin to lead evidence in a proceeding under the Public Premises
(Eviction of Unauthorised Occupants) Act, 1971 (in short 'the Act') is the question
involved in these appeals.
Background Facts
3. Appellant is a company incorporated under the Companies Act, 1956. It is a State
within the meaning of Article 12 of the Constitution of India. It owns a building in the
town of Mumbai commonly known as Mayfair Gardens. Respondents herein are the
tenants occupying two apartments in the said building. We would notice the fact of each
matter in brief separately.
Facts in appeal-Nusli Neville Wadia and Anr.
4. In this appeal a notice terminating the tenancy of first respondent was issued on 9th
February, 2001. However, on the premise that no ground of eviction had been mentioned
therein, another notice was issued on 18th February, 2002 enumerating the grounds of
eviction. Replies were given thereto by the respondent. Appellant filed an application
before the Estate Officer praying for eviction of the respondent and for damages for
unauthorized occupation of the premises with effect from 1st April, 2002 @
Rs.4,91,700/- per month with interest @ 9 % per annum thereupon.
5. The Estate Officer purported to be satisfied that the occupation of the first respondent
is unauthorized and that an order of eviction may have been passed against it issued a
show cause notice to respondent No.1 on 28th July, 2003 under Section 4 of the Act. The
grounds for first respondent's eviction as set out in the application were:-
"1. The New India Assurance Co. Ltd. needs and requires the premises for its own use
and occupation for accommodating its own senior executives;
2. The New India Assurance Co. Ltd. is being evicted from tenanted premises and being
called upon to pay exhorbitant rents for tenanted premises;
3. Increase in business, globalization of economy and liberalization of policies have
necessitated the New India Assurance Co. Ltd. to use and occupy the New India
Assurance Co. Ltd.s own properties ;
4. The tenancy of Respondent No. 1 has been duly terminated by the New India
Assurance Co. Ltd.'s notice dated 18.02.2002;
5. Respondent No.1 is a rich and wealthy industrialist who has several flats for his
residence."
6. Notice was also issued on the same date i.e. 28th July, 2003 under the provisions of
Section 7(3) of the Act calling upon the tenant-respondent to show cause why he should
not be required to pay damages. Reply to the formal show cause notices were filed by the
first respondent on 23rd February, 2004. A detailed written statement was filed by him on
3rd September, 2004.
7. Upon an application filed by the appellant, the Estate Officer by an order dated 26th
August, 2005 directed the first respondent to lead evidence wherein his advocate was
directed to file an affidavit of evidence. The matter was adjourned for cross-examination
of the first respondent's witnesses by the appellant's advocate. First Respondent, however,
filed an application before the Estate Officer praying inter alia that he should vary his
order dated 26th August, 2005 and direct the appellant to lead its evidence first and offer
its witnesses for cross-examination by the first respondent whereafter he would file his
affidavit of evidence. The said
@page-SC879
application was rejected by the Estate Officer by an order dated 12th January, 2006.
Aggrieved by and dissatisfied therewith the first respondent filed a writ petition before
the Bombay High Court which, by reason of the impugned judgment, has been allowed.
Facts in appeal - KLM Engineering Co. Pvt. Ltd. and Ors.
8. Respondent No.1 was inducted as a licensee in a furnished flat in an apartment in the
said building. The period of lease was for 5 years beginning from 1st October, 1994.
Allegedly on the premise that the respondent No.1 did not renew the 'licence' by giving
two months' prior notice as required, a notice to quit and hand over possession was issued
on 13th December, 1999. In response to the said notice the respondents contended that
they had exercised their option to renew the licence vide their letter dated 9th December,
1999. Thereafter by a notice dated 9th February, 2001 the tenancy of respondent No.1
was terminated by the appellant. As the said notice did not contain any ground for
termination of tenancy, another notice was issued on 18th February, 2002. Composite
application was filed by the appellant under Sections 4 and 7 of the Act before the Estate
Officer on 16th January, 2003 whereupon two show cause notices were issued by the
Estate Officer to the first respondents in terms of Sections 4 and 7(3) of the Act on 21st
February, 2003.
9. Before the Estate Officer, the appellant filed its affidavits of evidence. Dates after dates
were fixed for cross-examination of the witnesses of the appellant. Appellant had been
taking adjournments in the matter. On 22nd August, 2005 the first respondent moved an
application for direction before the Estate Officer. On or about 20th September, 2005 the
appellant had filed an application inter alia stating that as it had already placed all the
evidence on record, it was for the first respondent to file its evidence and produce
witnesses first for cross-examination by it and prayed inter alia for the following relief :-
"the Opposite Party be directed to show cause to the Statutory Notice issued by the
erstwhile Learned Estate Officer under sections 4 and 7 of the PP Act and the case
submitted by the Applicants."
By order dated 30th January, 2006 the Estate Officer directed the first respondent to lead
evidence by the following order :-
"Both applications disposed of. Application dated 22nd August, 2005 of the Opposite
Party is not allowed. As regards application dated 20th September, 2005 taken out by the
Applicants, I direct the Opposite Party to file their documents and witness affidavits in
lieu of evidence-in-chief on or before 21.02.2006 complete inspection of documents, if
any, by 28.02.2006 and adjourn the matter to 03.03.2006 at 3.30 p.m. for further
directions."
10. Aggrieved by and dissatisfied therewith, the Respondent Nos. 1 and 2 filed a writ
petition before the High Court being W.P. No. 557 of 2006 which has been allowed by
reason of the impugned order.
Contentions
11. Mr. P. N. Lekhi, learned senior counsel appearing on behalf of the appellant inter alia
submitted :
i) Having regard to the scheme of the 1971 Act and the Rules framed thereunder the
respondent-tenant should have been directed to lead evidence.
ii) The Estate Officer having satisfied itself that the respondent-tenant was in
unauthorized occupation of the public premises, it was for the tenant to establish that his
occupation is authorized.
iii) The premises in question being a public premises the Noticee does not enjoy any
protection as envisaged under the provisions of the Maharashtra Rent Control Act, 1999
and in that view of the matter termination of tenancy itself is sufficient for directing
eviction of the tenant and thus, the onus would lie upon the respondent to show that it is
not in unauthorized occupation.
iv) The procedure prescribed under the Act for eviction of the unauthorized occupants
being governed by the provisions of the Act and the Rules thereunder, the provisions of
the Code of Civil Procedure or the Evidence Act would not be attracted.
v) The High Court committed a serious error in passing the impugned judgment relying
on or on the basis of the guidelines issued by the Central Government which have no
statutory force; being advisory in character, and as such the same could not have been
relied upon ignoring the statutory enactment.
vi) The Division Bench of the High Court acted illegally in so far as it failed to take
@page-SC880
into consideration several binding precedents operating in the field.
12. Mr. F. S. Nariman, learned senior counsel, appearing on behalf of the first respondent
in Civil Appeal arising out of SLP (C) No. 8232 of 2006 would submit that :
(a) Section 4 of the Act deals with two types of cases; first-those who are in unauthorised
occupation in the sense of being in occupation without any authority therefor which is
governed by the first part of Section 4 of the Act; and second, those who have continued
in occupation of public premises as Rent Control Acts permitted them to continue to
occupy but in respect whereof the tenancy has been terminated "for any reason
whatsoever" as envisaged in Section 2(g) of the Act.
(b) Whereas in the first group of cases the onus of proof would be on the tenant to
establish that no order of eviction should be passed, however, in the second group of
cases it would be for the landlord to establish their bona fide need, although the
provisions of the Code of Civil Procedure and the Evidence Act stricto sensu are not
applicable, having regard to the statutory scheme as also the principles of natural justice
which are required to be complied with.
(c) As the Act and the Rules envisage a lis between the parties and the decision of the
Estate Officer is an appealable one, not only evidence is required to be recorded, an
opportunity to cross-examine the witnesses must also to be given.
(d) A party to the lis cannot ordinarily be directed to prove a negative and in that view of
the matter, it would be for the insurance company to lead evidence at the first instance.
13. Mr. Anil B. Diwan, learned senior counsel appearing on behalf of respondent Nos. 1
and 2 in Civil Appeal arising out of SLP (C) No. 10348 of 2006 submitted that :
(1) As a composite application for eviction as also the damages has been filed and a
composite notice having been issued, the onus of proof lay upon the appellant.
(2) Appellant having filed an affidavit of its witnesses accepted that it was for it to lead
evidence and having taken a large number of adjournments on the dates fixed for cross-
examination of the said witnesses, could not have gone back thereupon and contended
that it was for the respondents to lead evidence.
(3) As the principle of natural justice contemplate cross-examination of the witnesses as
also inspection of documents, the High Court cannot be said to have committed an error
in passing the impugned judgment.
THE ACT
14. Indisputably the respondents herein are not protected tenants under the provisions of
the Maharashtra Rent Control Act, 1999.
15. Section 15 of the Act bars the jurisdiction of a civil court or any other court to
entertain any proceedings in respect of eviction of any person who is in unauthorized
occupation of any public premises. Public premises has been defined in Section 2(e) to
mean any premises belonging to, or taken on lease by any corporation established by or
under a Central Act and owned or controlled by the Central Government. It is not in
dispute that the premise in question is a public premise.
16. Section 3 of the Act provides for appointment of an Estate Officers. Section 4
provides for issuance of a show cause notice in the following terms :-
"Section 4 - Issue of notice to show cause against order of eviction - (1) If the estate
officer is of the opinion that any persons are in unauthorised occupation of any public
premises and that they should be evicted, the estate officer shall issue in the manner
hereinafter provided a notice in writing calling upon all persons concerned to show cause
why an order of eviction should not be made.
(2) The notice shall-
(a) specify the grounds on which the order of eviction is proposed to be made; and
(b) require all persons concerned, that is to say, all persons who are, or may be, in
occupation of, or claim interest in, the public premises,-
(i) to show cause, if any, against the proposed order on or before such date as is specified
in the notice, being a date not earlier than seven days from the date of issue thereof, and
(ii) to appear before the estate officer on the date specified in the notice along with the
evidence which they intend to produce in support of the cause shown, and also for
personal hearing, if such hearing is desired.
(3) The estate officer shall cause the notice to be served by having it affixed on the
@page-SC881
outer door or some other conspicuous part of the public premises, and in such other
manner as may be prescribed whereupon the notice shall be deemed to have been duly
given to all persons concerned."
17. Section 5 deals with the procedure for eviction of unauthorized occupants. It reads :-
"Section 5 - Eviction of unauthorised occupants.- (1) If, after considering the cause, if
any, shown by any person in pursuance of a notice under section 4 and any evidence
produced by him in support of the same and after personal hearing, if any, given under
clause (b) of sub-section (2) of section 4, the estate officer is satisfied that the public
premises are in unauthorised occupation, the estate officer may make an order of eviction,
for reasons to be recorded therein, directing that the public premises shall be vacated, on
such date as may be specified in the order, by all persons who may be in occupation
thereof or any part thereof, and cause a copy of the order to be affixed on the outer door
or some other conspicuous part of the public premises.
(2) If any person refused or fails to comply with the order of eviction on or before, the
date specified in the said order or within fifteen days of its publication under sub-section
(1) whichever is later, the estate officer or any other officer duly authorized by the estate
officer in this behalf may evict that person from, and take possession of, the public
premises and may, for that purpose, use such force as may be necessary."
18. Section 7 empowers the Estate Officer to require payment of rent or damages in
respect of public premises by the person who is in unauthorized occupation thereof.
19. Section 9 provides for appeal from an order of the Estate Officer.
THE RULES;
20. Rules have been framed by the Central Government in exercise of its powers under
Section 18 of the Act known as the Public Premises (Eviction of Unauthorised
Occupants) Rules, 1971. Rule 5, which is material for these appeals reads as under:-
"5. Holding of inquiries.- (1) Where any person on whom a notice or order under this Act
has been served desires to be heard through his representative he should authorize such
representative in writing.
(2) The estate officer shall record the summary of such evidence and any relevant
documents filed before him shall form part of the records of the proceedings."
GUIDELINES
21. A tenant of a public premise although ordinarily does not get any protection from
eviction from the tenanted premises under the provisions of the Maharashtra Rent Control
Act, 1999, it is accepted that the action on the part of the landlord, which is a State within
the meaning of Article 12 of the Constitution of India must in this behalf be fair and
reasonable. In other words the action of the State in terms of the provisions of the Act
should not be arbitrary, unreasonable or mala fide. With that end in view only, and for
determining the legal effect arriving thereunder, the Central Government had, from time
to time, issued several guidelines. The guidelines so issued are dated 14th January, 1992 ;
5th August, 1992 ; 7th July, 1993 ; 14th July, 1993; 23rd July, 1993; 9th June, 1998, 2nd
September, 2002 and 23rd July, 2003. In terms of the said guidelines, however, a
distinction is sought to be made between a tenant who is rich or industrialist etc. vis-a-vis
a person who is poor and uses the tenanted premises only for his residence as would
appear from the guidelines dated 23rd July, 2003, the relevant portion whereof reads as
under :-
"3. The Government Resolution dated 30.05.2002 embodies the guidelines dated
14.01.1992 for observance by the Public Sector Undertakings. However, clarification was
issued vide OM No. 21011/790 Pol.1 IV.H.11 dated 07.07.1993 that the guidelines are
meant for genuine non-affluent tenants and these are not applicable to the large business
houses and commercial entrepreneurs.
22

. Issuance of such guidelines, however, is not being controlled by statutory provisions.


The effect thereof is advisory in character and thereby no legal right is conferred upon the
tenant. (See 1990 (Supp) SCC 440 at 508, Narendra Kumar Maheshwari vs. Union of
India and others; (1981) 1 SCC 166 at 232, Maharao Sahib Shir Bhim Singhji vs. Union
of India and others; (1988) 4 SCC 364 (paragraph 31), J. R. Raghupathy and others vs.
State of A.P. and others ; (2002) 100 DLT 497, Uttam Parkash Bansal and others vs.
L.I.C. of India and 1992 (2) CLR 457, AIR 1989 SC 2138, at Pp. 2181
AIR 1981 SC 234
AIR 1988 SC 1681, Para 30

@page-SC882
Punjab National Bank vs. M/s. The Lord Krishna Paper Industries and others.
23. We may, however, hasten to add that having regard to the fact that the appellants
themselves referred to guidelines issued by the Central Government from time to time, its
ultimate effect on the application need not be finally determined by us.
APPLICATION OF THE ACT AND THE RULES
24. Where an application is filed for eviction of an unauthorized occupant it obligates the
Estate Officer to apply his mind so as to enable him to form an opinion that the
respondent is a person who has been in unauthorized occupation of the public premises
and that he should be evicted. When a notice is issued in terms of Section 4 of the Act,
the noticee may show cause. Section 5 of the Act postulates that an order of eviction must
be passed only upon consideration of the show cause and any evidence produced by him
in support of its case also upon giving him a personal hearing, if any, as provided under
clause (ii) of sub-section (2) of Section 4 of the Act.
25. Although Section 5 ex facie does not make any classification in regard to the two
classes of tenancies but the same is evident from the decisions rendered by this Court as
also by the different High Courts.
26. The occupants of public premises may be trespassers, or might have breached the
conditions of tenancy, or have been occupying the premises as a condition of service, but
were continuing to occupy the premises despite cessation of contract of service.

27. However, there may be another class of tenants who are required to be evicted not on
any of the grounds mentioned hereinbefore but inter alia on the ground, which requires
proof of the fairness and reasonableness on the part of the landlord which may include
requirement for its own use and occupation.
28. Furthermore a proceeding may be initiated under Section 4 simpliciter. A composite
proceedings may also be initiated both under Sections 4 and 7 of the Act. In the latter
category of cases the landlord would be required to establish not only the bona fide need
on its part but also quantum of damages to which it may hold to be entitled to, in the
event that an order is passed in favour of the establishment.
29. Admittedly in these cases two notices for eviction were issued. If the contention of
Mr. Lekhi is correct, the first notice was not required to be withdrawn and the second
notice was not required to be issued, specifying the grounds on which the eviction of the
respondents were sought for.
30. When an application for eviction is based on such grounds, which require production
of positive evidence on part of the landlord, in our opinion, it would be for it to adduce
evidence first; moreso in a composite application where the evidence is also required to
be led on the quantum of damages to be determined by the Estate Officer.
31. There may be a case where the tenant may take a defence which discloses no prima
facie case in which event the Estate Officer may ask him to lead evidence. But there may
be cases where the ground of eviction, having regard to the defence taken by the
occupants, may be required to be gone into.
32. Appellant's stand in this case is clear and unambiguous. It intends to evict the
respondents on the grounds specified in the notices issued by the Estate officer.
33. The Estate Officer with a view to determine the lis between the parties must record
summary of the evidence. Summary of the evidence and the documents shall also form
part of the record of the proceedings.
34. Procedure laid down for recording evidence is stated in the Rules. The Estate Officer
being a creature of the statute must comply the same. When a notice is issued, the
occupant of the public premises would not only be entitled to show cause but would also
be entitled to produce evidence in support of the cause shown.
CONCEPT OF FAIRNESS
35. The procedural aspect as to who should lead evidence first, thus may have to be
determined on the basis of the issues arising in the matter. When we say so, we do not
mean that the procedure involved being a summary one, the issues are required to be
specifically framed but that which is the principal issue(s) between the parties must be
known to the Estate Officer.
36. Thus under the Public Premises (Eviction of Unauthorized Occupants) Act, 1971 the
occasion would arise for multi-level inquiry: Primary inquiry will be to arrive at a
conclusion on "unauthorized occupant"; and intermediate inquiry would be as to the
@page-SC883
eviction of "unauthorized occupant".
37. The question has been succinctly dealt with by a Division Bench of the Bombay High
Court in Minoo Framroze Balsara vs. Union of India and others (AIR 1992 Bom 375)
wherein Bharucha, J. (as the learned Chief Justice then was) opined: "the Government
company or corporation must so act not only when terminating the authority of an
occupant of public premises of its ownership to occupy the same but also when,
thereafter, it seeks his eviction therefrom".
38. The statute, although, does not require a lengthy hearing or a lengthy cross-
examination but the noticee should be given an opportunity to file an effective show
cause. An effective show cause can be filed when eviction is sought for a specified
ground and the occupants must know the particulars in relation thereto.
39

. For the said purpose, Sections 4 and 5 of the Act must be read together. Even the Rules
which are validly framed must be read along with the statutory provisions. Ordinarily
although a tenant occupying the property belonging to a Government may be somewhat
in a worse position than a tenant having protection under the Rent Control Act as has
been held by a Full Bench of the Calcutta High Court in AIR 1968 Calcutta 1 : Standard
Literature Co. Private Ltd. and Ors. vs. Union of India, but with a view to interpret the
provisions of the Act, we must take into consideration the decisions of this Court laying
down the concept of 'bona fide act' and the 'fair action' on the part of the owner as laid
down in (1989) 3 SCC 293, Dwarkadas Marfatia and Sons vs. Board of Trustees of the
Port of Bombay and (1990) 4 SCC 406, Ashoka Marketing Ltd. vs. Punjab National
Bank. AIR 1989 SC 1642
AIR 1991 SC 855

40

. In Dwarkadas Marfatia (supra) this Court clearly held that "the public authorities which
enjoy this benefit without being hidebound by the requirements of the Rent Act must act
for public benefit. Hence, to that extent, that is liable to be gone into and can be the
subject-matter of adjudication". Dwarkadas Marfatia was applied in Ashoka Marketing
(supra) stating :- AIR 1989 SC 1642
AIR 1991 SC 855

"69. It has been urged by the learned counsel for the petitioners that many of the
corporations referred to in Section 2(e)(2)(ii) of the Public Premises Act, like the
nationalised banks and the Life Insurance Corporation, are trading corporations and under
the provisions of the enactments whereby they are constituted these corporations are
required to carry on their business with a view to earn profit, and that there is nothing to
preclude these corporations to buy property in possession of tenants at a low price and
after buying such property evict the tenants after terminating the tenancy and thereafter
sell the said property at a much higher value because the value of property in possession
of tenants is much less as compared to vacant property. We are unable to cut down the
scope of the provisions of the Public Premises Act on the basis of such an apprehension
because as pointed out by this Court in Dwarkadas Marfatia and Sons v. Board of
Trustees of the Port of Bombay (SCC p. 306, para 27) AIR 1989 SC 1642, at P.
1649

"...every activity of a public authority especially in the background of the assumption on


which such authority enjoys immunity from the rigours of the Rent Act, must be
informed by reason and guided by the public interest. All exercise of discretion or power
by public authorities as the respondent, in respect of dealing with tenants in respect of
which they have been treated separately and distinctly from other landlords on the
assumption that they would not act as private landlords, must be judged by that standard."
These observations were made in the context of the provisions of the Bombay Rents,
Hotel and Lodging House Rates Control Act, 1947 whereby exemption from the
provisions of the Act has been granted to premises belonging to the Bombay Port Trust.
The consequence of giving overriding effect to the provisions of the Public Premises Act
is that premises belonging to companies and statutory bodies referred to in clauses (2)
and (3) of Section 2( e ) of the Public Premises Act would be exempted from the
provisions of the Rent Control Act. The actions of the companies and statutory bodies
mentioned in clauses (2) and (3) of Section 2(e) of the Public Premises Act while dealing
with their properties under the Public Premises Act will, therefore, have to be judged by
the same standard."
Constitutional Backdrop
41

. Constitutional validity of the Act as also its predecessors' Act being Public Premises
(Eviction of Unauthorised Occupants) AIR 1958 Punjab 1
AIR 1967 SC 1581
AIR 1974 SC 2009

@page-SC884
Act, 1958 and the Government Public Premises Eviction Act, 1950 was challenged in
several proceedings. The Public Premises Act, 1950 was struck down in AIR 1956 All
507 (DB), Brigadier Commandant, Meerut vs. Gangaprasad; 58 CWN 1066, Jaggu Singh
vs. Shaukat Ali and 1957 (59) PLR 621, Satish Chander vs. Delhi Improvement Trust.
1950 Act was repealed by the Public Premises (Eviction of Unauthorised Occupants) Act,
1958 wherein, however the jurisdiction of the civil court was not barred. A Constitution
Bench of this Court in 1967 (3) SCC 399, Northern India Caterers Pvt. Ltd. vs. State of
Punjab, held Section 5 thereof to be void as an additional remedy over and above the
usual remedy by way of a suit was conferred thereby providing for two alternative
remedies or leaving it to the unguided discretion of the Statutory Authorities to resort to
one or the other procedure. Northern India Caterers Pvt. Ltd. (supra), however, was
overruled by a Bench of 7-Judges of this Court in (1974) 2 SCC 402, Maganlal Chaganlal
vs. Municipal Corporation. We must also notice that 1958 Act was struck down by Delhi
High Court in P. L. Mehra vs. D.R. Khanna (Civil Writ No. 431 of 197).
42

. On the aforementioned premises the 1971 Act was enacted after removing the vice
which led to it having been declared as void with effect from 16th September, 1958. It
suffered another challenge in (1972) 2 SCC 259, Hari Singh vs. Military Estate Officer.
However, the challenge to its validity was negatived holding that the 1971 Act did not
provide for two procedures but only one procedure. Yet again in (1988) 4 SCC 324,
Accountant and Secretarial Services vs. Union of India, challenge to the validity of the
Act on the premise that one of the officers of the Statutory Authority may be appointed as
an Estate Officer and thus violative of Article 14. However, the Court negatived the
challenge and observed : AIR 1972 SC 2205
AIR 1988 SC 1708, Para 26

"32. Dr. Chitale, while initially formulating his contentions, outlined an argument that the
provision in the 1971 Act appointing one of the officers of the respondent-Bank as the
Estate Officer is violative of Article 14. We do not see any substance in this contention. In
the very nature of things, only an officer or appointee of the Government, statutory
authority or corporation can be thought of for implementing the provisions of the Act.
That apart, personal bias cannot necessarily be attributed to such officer either in favour
of the bank or against any occupant who is being proceeded against, merely because he
happens to be such officer. Moreover, as pointed out earlier, the Act provides for an
appeal to an independent judicial officer against orders passed by the Estate Officer.
These provisions do not, therefore, suffer from any infirmity. In fact, Dr. Chitale did not
pursue this objection seriously."
43

. It was on the aforementioned premise that the dicta laid down in Ashoka Marketing Ltd.
(supra) must be considered wherein this Court held that the Act overrides Delhi Rent
Control Act, 1958, although both were Acts of Parliament. AIR 1991 SC 855

Natural Justice Issue :


44

. If some facts are to be proved by the landlord, indisputably the occupant should get an
opportunity to cross-examine. The witness who intends to prove the said fact has the right
to cross-examine the witness. This may not be provided by under the statute, but it being
a part of the principle of natural justice should be held to be indefeasible right. [See 1984
(1) SCC 43, K.L. Tripathi vs. State Bank of India and others and 2005 (10) SCC 634,
Lakshman Exports Limited vs. Collector of Central Excise] AIR 1984 SC 273
2003 AIR SCW 2938

45
. We may also take note of the fact that this Court in 1972 (1) SCR 241, Bareilly
Electricity Supply Co. Ltd. vs. The Workmen, this Court held as under :- AIR 1972 SC
330, Para 14

"The application of the principle of natural justice does not imply that what is not
evidence can be acted upon. On the other hand what it means is that no material can be
relied upon to establish a contested fact which are not spoken to by persons who are
competent to speak about them and are subjected to cross-examination by the party
against whom they are sought to be used."
46. It is axiomatic that when in support of its case the landlord intends to rely upon a
document which is to be taken on record, it would be obligatory on the part of the Estate
Officer to allow inspection thereof to the
@page-SC885
noticee. Denial of such inspection of documents shall be violative of the principle of
natural justice. It would run counter to the doctrine of fairness in the matter of
determination of a lis between the parties.
47

. We may also notice that in (2007) 1 SCC 174, Sarbananda Sonowal (II) vs. Union of
India, this Court having regard to the fact that burden of proof was on the notice held :
2007 AIR SCW 326, Paras 39 and 40

"56. Status of a person, however, is determined according to statute. The Evidence Act of
our country has made provisions as regards "burden of proof". Different statutes also lay
down as to how and in what manner burden is to be discharged. Even some penal statutes
contain provisions that burden of proof shall be on the accused. Only because burden of
proof under certain situations is placed on the accused, the same would not mean that he
is deprived of the procedural safeguard."
It was observed :
"60. Having regard to the fact that the Tribunal in the notice to be sent to the proceedee is
required to set out the main grounds; evidently the primary onus in relation thereto would
be on the State. However, once the Tribunal satisfied itself about the existence of
grounds, the burden of proof would be upon the proceedee."
Interpretative Approval
48. Section 5 of the Act, on a plain reading, would place the entire onus upon a noticee.
It, in no uncertain terms, states that once a notice under Section 4 is issued by the Estate
Officer on formation of his opinion as envisaged therein it is for the noticee not only to
show cause in respect thereof but also adduce evidence and make oral submissions in
support of his case. Literal meaning in a situation of this nature would lead to a
conclusion that the landlord is not required to adduce any evidence at all nor it is required
even to make any oral submissions. Such a literal construction would lead to an
anomalous situation because the landlord may not be heard at all. It may not even be
permitted to adduce any evidence in rebuttal to the one adduced by the noticee nor it
would be permitted to advance any argument. Is this contemplated in law? The answer
must be rendered in the negative. When a landlord files an application, it in a given
situation must be able to lead evidence either at the first instance or after the evidence is
led by the noticee to establish its case and/or in rebuttal to the evidence led by the
noticee.
49. The literal interpretation of the statute, if resorted to, would also lead to the situation
that it would not be necessary for the landlords in any situation to plead in regard to its
need for the public premises. It could just terminate the tenancy without specifying any
cause for eviction.
50

. Except in the first category of cases, as has been noticed by us hereinbefore, Sections 4
and 5 of the Act, in our opinion, may have to be construed differently in view of the
decisions rendered by this Court. If the landlord being a State within the meaning of
Article 12 of the Constitution of India is required to prove fairness and reasonableness on
its part in initiating a proceeding, it is for it to show how its prayer meets the
constitutional requirements of Article 14 of the Constitution of India. For proper
interpretation not only the basic principles of natural justice have to be borne in mind, but
also principles of constitutionalism involved therein. With a view to read the provisions
of the Act in a proper and effective manner, we are of the opinion that literal
interpretation, if given, may give rise to an anomaly or absurdity which must be avoided.
So as to enable a superior court to interpret a statute in a reasonable manner, the court
must place itself in the chair of a reasonable legislator/ author. So done, the rules of
purposive construction have to be resorted to which would require the construction of the
Act in such a manner so as to see that the object of the Act fulfilled; which in turn would
lead the beneficiary under the statutory scheme to fulfill its constitutional obligations as
held by the court inter alia in Ashoka Marketing Ltd. (supra). AIR 1991 SC 855

51. Barak in his exhaustive work on 'Purposive Construction' explains various meanings
attributed to the term "purpose". It would be in the fitness of discussion to refer to
Purposive Construction in Barak's words :
"Hart and Sachs also appear to treat "purpose" as a subjective concept. I say "appear"
because, although Hart and Sachs claim that the interpreter should imagine himself or
herself in the legislator's shoes, they introduce two elements of objectivity: First, the
interpreter should assume that the legislature
@page-SC886
is composed of reasonable people seeking to achieve reasonable goals in a reasonable
manner; and second, the interpreter should accept the non-rebuttable presumption that
members of the legislative body sought to fulfill their constitutional duties in good faith.
This formulation allows the interpreter to inquire not into the subjective intent of the
author, but rather the intent the author would have had, had he or she acted reasonably."
(Aharon Barak, Purposive Interpretation in Law, (2007) at pg. 87)
52. In Bharat Petroleum Corpn. Ltd. v. Maddula Ratnavalli and Ors., (2007) 6 SCC 81,
this Court held :
"The Parliament moreover is presumed to have enacted a reasonable statute (see Breyer,
Stephen (2005): Active Liberty: Interpreting Our Democratic Constitution, Knopf
(Chapter on Statutory Interpretation - pg. 99 for "Reasonable Legislator Presumption")."
53
. The provisions of the Act and the Rules in this case, are, thus required to be construed in
the light of the action of the State as envisaged under Article 14 of the Constitution of
India. With a view to give effect thereto, the doctrine of Purposive Construction may
have to be taken recourse to. [See 2007 (7) Scale 753, Oriental Insurance Co. Ltd. vs. Brij
Mohan and others.] 2007 AIR SCW 3734

Conclusion :
54. Although the provisions of the Evidence Act are not applicable, the underlying
principles of Section 101 thereof would apply. In Sarkar on Law of Evidence 16th
Edition Volume 2 at pg. 1584 it is stated as under :-
"Principle and Scope.- This section is based on the rule, ie incumbit probation qui dicit,
non qui negat the burden of proving a fact rests on the party who substantially asserts the
affirmative of the issue and not upon the party who denies it; for a negative is usually
incapable of proof. "It is an ancient rule founded on consideration of good sense and
should not be departed from without strong reasons". [per LORD MAUGHAM in
Constantine Line vs. I S Corpn. (1941) 2 All ER 165, 179]. This rule is derived from the
Roman law, and is supportable not only upon the ground of fairness, but also upon that of
the greater practical difficulty which is involved inproving a negative than in proving an
affirmative [Hals 3rd Ed .Vol. 15 para 488].
(Emphasis supplied)
55

. The said principle has been approved by this Court in (1983) 4 SCC 491, Shambhu Nath
Goyal vs. Bank of Baroda and others; (1999) 8 SCC 744; Garden Silk Mills Ltd. and
another vs. Union of India and others and (2007) 2 SCC 433 (para 18), J.K. Synthetics
Ltd. vs. K.P. Agrawal and another. AIR 1984 SC 289
1999 AIR SCW 4150
2007 AIR SCW 1357

56. We, however, must not shut our eyes to the objects for which the Act was enacted. It
provided for a speedy remedy. The Estate Officer is expected to arrive at a decision as
expeditiously as possible. The provisions of the Code of Civil Procedure and Evidence
Act being not applicable, what is necessary to be complied with is the principles of
natural justice.
57. Even if we assume that in terms of the statutory provisions the respondents must lead
evidence first the same can be waived, Appellant not only had filed affidavits in one of
the cases but time and again sought adjournments when the deponent of the affidavit was
to be cross-examined. Although the appellant had pleaded requirements of the premises
on an urgent basis, it kept on taking adjournments for more than 2 years. Why the
witnesses were not produced before the Estate Officer for cross-examination for such a
long time is not known. Only after a long period, an application was filed asking the
respondents to show cause. Cause had already been shown by the respondents. They
pleaded that no case has been made out for their eviction. We, therefore, fail to
understand on what basis the Estate Officer passed the order impugned before the High
Court.
58. We, therefore, direct that both the parties must file their documents within a week
from today and the Estate Officer must give both the parties inspection of the said
documents within a week thereafter. In the appeal arising out of SLP (C) No. 10348 of
2006 the appellant must file the affidavits of its witnesses within two weeks and thus
shall be produced for cross-examination within one week thereafter. In appeal arising out
of SLP (C) No. 8232 of 2006 the witnesses must be produced for cross-examination as
expeditiously as possible, but not
@page-SC887
beyond a period of two weeks.
59. The proceedings before the Estate Officer, being summary in nature, the cases must
go on a day-to-day basis. The Estate Officer is directed to pass a final order, as
expeditiously as possible but not beyond a period of 10 weeks from the date of receipt of
a copy of this order.
60. These appeals are dismissed with the aforesaid direction with costs. Counsel fee in
each case is assessed at Rs. 25,000/- (Rupees twenty five thousand only).
Appeal dismissed.
AIR 2008 SUPREME COURT 887 "Soni Dineshbhai Manilal v. Jagjivan Mulchand
Chokshi"
(From : AIR 2005 Guj 293)
Coram : 2 S. B. SINHA AND H. S. BEDI, JJ.
Civil Appeal No. 5945 with 5946 of 2007 (arising out of SLP (C) No. 19295 of 2005 with
7818 of 2006), D/- 14 -12 -2007.
Soni Dineshbhai Manilal and Ors. v. Jagjivan Mulchand Chokshi

With
Jagjivan Mulchand Chokshi v. Soni Dineshbhai Manilal and Ors.
(A) Civil P.C. (5 of 1908), O.43, R.1, O.26, R.166, S.105 - APPEAL - COMMISSIONS -
Commissioner to examine accounts - Objection to report - Appeal against final order
rejecting objection - Interlocutory orders passed earlier can also be challenged - But
specific challenge has to be raised thereagainst in memorandum of appeal - Neither
revision was filed against interlocutory orders nor they had been specifically challenged
in Memorandum of Appeal - Each of those orders attained finality. (Paras 21)
(B) Civil P.C. (5 of 1908), S.100, O.26, R.16, O.43, R.1 - APPEAL - COMMISSIONS -
Second appeal - Tenability - Order allowing objections filed against Commissioner's
report - Second appeal against - Technically not maintainable in view of O.43, R. 1 -
However scope of second appeal being narrower than appeal under O. 43 R.1 - And
appeal having been entertained after hearing parties - S. C. refused to interfere.
Constitution of India, Art.133.
It may be true that in view of R. 1 of O. 43 a second appeal against order rejecting
objections raised to commissioner's report was not maintainable. But the scope of an
appeal under S. 100 is narrower. If the appeal had been entertained upon hearing both the
parties, Supreme Court may not exercise its extra ordinary jurisdiction to set aside that
order, as what matters most is to see whether substantial justice has been done to the
parties and not the technicalities involved therein. In a given case the appellate Court in
exercise of its inherent jurisdiction can convert one type of appeal to the other. Forum for
preferring a second appeal as also an appeal under O.43, R. 1 is the same, namely the
High Court. As the scope of an appeal under O.43, R. 1 is wider than a second appeal, the
appellants on their own showing are not prejudiced in any manner, if the High Court
proceeded to consider the question involved in the appeal in its impugned judgment.
(Paras 23, 24)
Cases Referred : Chronological Paras
2007 AIR SCW 3595 25
AIR 1970 SC 1 21
Pravin Satale, Naresh Kumar, Ms. Meenakashi, Ms. D. Singh, for the Appearing Parties.
Judgement
1. S. B. SINHA, J. :-Leave granted.
2

. Appellants in both the appeals are before us, aggrieved by and dis-sastisfied with the
judgment and order dated 6th April, 2005 passed by a learned Single Judge of the High
Court of Gujarat in Second Appeal No.37 of 1998. Reported in AIR 2005 Guj 293

3. For the purpose of determining the question involved in these appeals, Soni
Dineshbhai Manilal and others are being referred to as the appellants, while Jagjivan
Mulchand Chokshi is being referred to as the respondent.
4. Appellants' father and the respondent were partners of a partnership firm known as
'Bhagyoday Engineering Company'. A decision was taken to dissolve the said firm. A
deed of dissolution was entered into on the 9th day of September, 1965. A suit for
dissolution of the partnership firm and accounts was filed by the respondent herein, inter
alia on the premise of the existing dispute in regard to shares of the parties in the said
partnership firm.
5. The suit was dismissed on 29th July, 1975. However, the appeal preferred thereagainst
was allowed holding that the respondent-plaintiff was entitled to 56 % shares, whereas
the father of the appellants was entitled to 44 % share. A decree was also
@page-SC888
passed for accounts for the period 19th January, 1960 and 9th September, 1965. Father of
the appellants filed a second appeal before the High Court which was partly allowed,
whereby the shares of the parties were determined at 50 % each. The said decree was
affirmed by this Court by its order dated 25th February, 1994.
6. An application was thereafter filed for initiation of a final decree proceedings. A Court
Commissioner was appointed for taking accounts. The Commissioner submitted his
report on 13th August, 1986. Objection thereto was filed by the father of the appellants.
An application was also filed for permission to cross-examine the Court Commissioner.
The same was rejected. A civil revision application was filed by the father of the
appellants which was dismissed by an order dated 22nd April, 1996, stating :-
"Mr. D.K. Acharya, learned Advocate for the petitioner seeks leave to withdraw the Civil
Revision Application. Leave granted. Rejected as withdrawn.
It is, however, clarified that the petitioner-defendant would be entitled to prove or
disprove the accounts that may be submitted by the parties with regard to the partnership
firm."
7. An application was also filed by the father of the appellants before the trial court to
allow a Chartered Accountant to verify the records and, accounts books. The said prayer
was also rejected.
8. By an order dated 2nd May, 1997 the trial court rejected the objections of the
appellants opining :-
"Taking into consideration, the submissions, replies and evidences of both the sides, Civil
Court had rejected the said suit and against the said order, Appeal No. 79 of 1975 was
being admitted in the District Court. The appellate Court had quashed the order of the
civil Court and share of the Plaintiff was decided and it was ordered in respect of
accounts to appoint the Court Commissioner for taking accounts. Thereafter in the
Honble High Court and in the Honble Supreme Court, the said matter was filed and
thereafter to draw final decree the same matter was adjourned.
In the said case, being kept for hearing on the debated point in respect of legal question,
but both the parties were given proper time and reasonable opportunity, even though,
their rights were closed as there was no submissions.
In the said case, Commissioner's Report at Mark 44/1, which was being admitted in
evidence according to provisions of Order 2-G, Rules 11, 12, which was taken on record
by Exh. 124 for taking into consideration for evidence. In the said case, the record and
Commissioner's report which were produced before me, being taken into consideration
and if determined as per law, the Commissioner's report and the finding which were given
by taking into account the fact, are found reasonable and when said report was given by
the defendant by violating the provisions of existing law, have been failed to prove the
same, in that circumstances, it is found that it is reasonable and just to give sanction to
the details of the report of Court Commissioner. So taking into consideration the facts and
documentary evidence produced, I pass the following order in the interest of justice."
9. An appeal preferred thereagainst, however, was allowed by an order dated 11th
December, 1997. Cross-objection was filed by the respondent therein and while rejecting
the said cross-objection, the first appellate court observed :-
"The cross objections Ex.11 filed by the respondent plaintiff are hereby rejected.
However, the learned trial Judge is directed to allow the said party to agitate the question
regarding interest and the same be decided as per law."
10. Respondent filed a second appeal thereagainst which has been allowed by reason of
the impugned judgment.
11. Mr. Pravin Satale, learned counsel appearing on behalf of the appellant submitted :
i) having regard to the provisions of Order XLIII, Rule 1, sub-rule (u) of the Code of
Civil Procedure the second appeal was not maintainable;
ii) High Court committed a serious error in relying upon the orders passed by the trial
court from time to time without taking into consideration the fact that in view of Section
105 of the Code of Civil Procedure, such orders are open to challenge in an appeal
preferred against a final order;
iii) The Commissioner appointed to take accounts should be allowed to be cross-
examined by a party taking objection to his report and in any event, he is entitled to
adduce his own evidence in support of his objection.
@page-SC889
iv) The Commissioner having ignored vital facts including non-production of books of
accounts and ledger, his report could not have been accepted.
12. Ms. Meenakshi Arora, learned counsel appearing on behalf of the respondent, on the
other hand, contended :
i) The appeal preferred by the respondent being a composite one both against the order
dismissing the cross-objection as also the appeal preferred by the appellants, a second
appeal was maintainable.
ii) A distinction must be made between a Commissioner appointed to examine accounts
and other Commissioners inasmuch as the report in the former case is to be treated as
evidence in the suit. In any view of the matter, keeping in view the facts and
circumstances of the case, in particular the fact that the preliminary decree was passed as
far back as 13th October, 1978, the impugned judgment should not be interfered with.
13. The learned trial Judge inter alia opined that opportunities have been granted to the
appellants to adduce evidence which they did not avail.
14. The learned Court of Appeal, on the other hand, held that the appellants were
prejudiced as the objections filed by them had not been considered.
15. In the final decree proceeding, one Shri Vardhilal A. Shah was appointed as a
Commissioner. He was asked to examine the accounts of the dissolved partnership firm.
He submitted a detailed report. It is not the case of the appellants that while preparing the
said report he was not allowed to place any document before him or call for any
document which was in custody or possession of the respondent. For the purpose of
determining the issue referred to him by the Court in terms of OrderXXVI, Rule 11 of the
Code of Civil Procedure, principally the books of accounts which were maintained by the
firm were required to be taken into consideration. If any additional books of accounts or
any other document was required to be taken into consideration therefor, it was for the
appellants to point out the same. It appears that the first objection which was taken by the
appellant was non production of "ublek" books and stock books. An objection was filed
to that effect on 1st July, 1985 which was rejected by an order dated 26th July, 1985. It
does not appear that any civil revision application was filed thereagainst. The said order,
therefore, attained finality.
16. Another application was filed in 1994 to cross-examine the Commissioner. The same
was rejected on 29th April, 1995. However, an observation was made by the trial court
that the "averments are fabricated with bad intention by the defendant and that the delay
may be caused for recovery of decretal amount". Another objection filed by the
respondent was rejected by a very detailed order dated 31st January, 1996 not only taking
into consideration the provisions of law but also the precedents operating in the field.
Conduct of the parties had also been taken into consideration therein. The learned Judge
also considered the nature of the objections raised, one of which, we may notice, is that
the Commissioner was not an expert in accounts. It was pointed that no such objection
was raised at the time of the appointment of the Commissioner.
17. Appellants' principal grievance centers round the non-production of "ublak" books
which, as noticed hereinabove, had been dealt with in the earlier orders of the court. It
was pointed out that the Commissioner had prepared a balance sheet inter alia on the
basis of the purchase bills and the sales bills.
18. A civil revision application, as noticed hereinbefore, was filed against one of the
orders, which was later withdrawn. It is accepted at the Bar that the other civil revision
application was also withdrawn.
19. The High Court in its impugned judgment had taken the said facts into consideration.
Order XXVI, Rule 11 of the Code of Civil Procedure provides for appointment of a
Commissioner to examine or adjust accounts, if necessary. He is competent to decide all
questions raised before him, taking into consideration all aspects of the matter. He is to
assist the Court. A Commissioner's report can be set aside only upon assignment of
proper and sufficient reasons. In the event any defect in the conduct of enquiry by him is
found out, the court may issue any further directions. A further enquiry can also be
ordered. A report of the Commissioner is a part of the record. It is to be treated as
evidence in the suit.
20. Rule 16 of Order XXVI of the Code of
@page-SC890
Civil Procedure provides for powers of the Commissioners which is in the following
terms :-
"16. Powers of Commissioners - Any Commissioner appointed under this Order may,
unless otherwise directed by the order of appointment,
(a) examine the parties themselves and any witness whom they or any of them may
produce, and any other person whom the Commissioner thinks proper to call upon to give
evidence in the matter referred to him;
(b) call for and examine documents and other things relevant to the subject of inquiry;
(c) at any reasonable time enter upon or into any land or building mentioned in the order."
21. Appellants' father was, therefore, entitled to raise all the contentions in regard to non-
production of books of accounts and other matters. It was also permissible for him to
examine witnesses in support of his case before the Commissioner. It may be true that
any order passed can be questioned in the grounds taken in the appeal against the final
orders, but such interlocutory orders are required to be challenged. Nothing has been
shown before us that such interlocutory orders and particularly those which are referred
to hereinbefore had specifically been challenged in the Memorandum of Appeal but the
said interlocutory orders were not subjected to revision. What is essential is that they
should not have been appealed against. If a revision has been filed which is a part of the
appellate jurisdiction, although stricto sensu, doctrine of merger may not apply but
Section 105 of the Code of Civil Procedure also would not apply in such cases. Each of
those orders attained finality.
It has been held in Shankar Ramchandra Abhyankar v. Krishnaji Dattatreya Bapat, AIR
1970 SC 1, that civil revision is a part of appellate jurisdiction.
22. As noticed hereinbefore, before the Court, objections to the report of the
Commissioner had been taken. Several orders were passed. There is nothing on record to
show that the appellant intended to adduce any evidence in support of his case. In fact he
was permitted to do so.
23. It may be true that in view of Rule 1(u) of Order XVIII a second appeal was not
maintainable but the scope of an appeal under Section 100 of the Code of Civil Procedure
is narrower. If the appeal had been entertained upon hearing both the parties, this Court
may not exercise its extra ordinary jurisdiction to set aside that order, as what matters
most is to see whether substantial justice has been done to the parties and not the
technicalities involved therein.
24. In a given case the appellate court in exercise of its inherent jurisdiction can convert
one type of appeal to the other. Forum for preferring a second appeal as also an appeal
under Order XVIII, Rule 1(u) is the same, namely the High Court. As the scope of an
appeal under Order XVIII, Rule 1(u) is wider than a second appeal, the appellants on
their own showing are not prejudiced in any manner, if the High Court proceeded to
consider the question involved in the appeal in its impugned judgment.
25

. Even substantial questions of law were framed and the same have been answered. We,
however, although agree that technically a second appeal was not maintainable from one
part of the judgment, keeping in view of the fact that the matter is pending for more than
40 years and in view of the nature of the dispute as also the quantum of amount involved,
we are of the opinion that it is not a fit case where we should exercise our discretionary
jurisdiction under Article 136 of the Constitution of India. It is now well settled that this
Court may decline to exercise its jurisdiction, although it would be lawful to do so. [See
Management, Pandiyan Roadways Corporation Ltd. vs. N. Balakrishnan, 2007 (7) Scale
758 ]. 2007 AIR SCW 3595

26. In the above circumstances both the appeals fail and are dismissed. However, in the
facts and circumstances of the case there shall be no order as to costs.
Appeals dismissed.
AIR 2008 SUPREME COURT 890 "Rameshwar Dass v. State of Punjab"
(From : 2002 (2) Rec Cri R 83 (Punj and Hary))
Coram : 2 S. B. SINHA AND H. S. BEDI, JJ.
Criminal Appeal No. 1021 of 2002, D/- 13 -12 -2007
Rameshwar Dass v. State of Punjab and Anr.
(A) Criminal P.C. (2 of 1974), S.102 - POLICE OFFICERS - SEARCH AND SEIZURE -
DOWRY DEATH - Trial for offence u/S. 304-B, I. P. C. - Seizure list - Non-mention of
letters
@page-SC891
purported to be written by deceased to accused in list - Such letters sought to be proved
by defence witness in support of defence that relationship between deceased and accused
was cordial - Testimony of such witness rightly held as wholly unnatural - Investigating
Officer would not seize letters without mentioning it in seizure list that was prepared.
Penal Code (45 of 1860), S.304B. (Para 18)
(B) Penal Code (45 of 1860), S.304B, S.405 and S.406 - DOWRY DEATH - BREACH
OF TRUST - Dowry death -Acquittal of accused in prosecution u/Ss. 405, 406 - Effect on
prosecution for offence u/S. 304-B - Acquittal being in respect of different transaction,
held, had no relevance for offence u/S. 304-B. (Para 20)
(C) Evidence Act (1 of 1872), S.3 and S.114 - EVIDENCE - DOWRY DEATH - Dowry
death - Death of deceased in hospital - First informant, the brother of deceased, reaching
hospital immediately on receiving news of his sister's death - Accused-husband not found
in hospital - Informant, held, would not make imaginary statement so as to implicate
accused falsely - Demand of money at time of marriage accepted by accused - No
evidence that accused admitted deceased in hospital - Accused absconding for 6 days
after incident - Onus on prosecution stands discharged - Accused liable to be convicted
on failure to prove his defence.
Penal Code (45 of 1860), S.304B. (Paras 21, 25)
(D) Evidence Act (1 of 1872), S.114 - EVIDENCE - DOWRY DEATH - Dowry death -
Deceased pregnant - Defence of suicide - Pregnant woman ordinarily would not commit
suicide unless relationship with her husband comes to such a pass that she would be
compelled to do so.
Penal Code (45 of 1860), S.304B. (Para 25)
Cases Referred : Chronological Paras
2003 AIR SCW 241 : AIR 2003 SC 809 : 2003 Cri LJ 900 (Rel. on) 25
Rajiv K. Garg and Ashish Garg (for Annam D. N. Rao), for Appellant; Gagan Deep
Sharma, Ajay Pal, Kuldip Singh, V. Kadam and Ms. Suruchii Aggarwal, for Respondents.
Judgement
1. S. B. SINHA, J. :-Appellant stood trial for commission of an offence under Section
304B of the Indian Penal Code in connection with unnatural death of his wife Sushma
Rani.
2. They were married on 11.03.1986. An engagement ceremony took place twenty days
prior thereto. A demand for dowry was made. It was met in part. A sum of Rs. 25,000/- by
way of a demand draft was handed over to the husband's family at Mansa when shagun
ceremony was performed. Allegedly another sum of Rs. 11,000/- was paid in cash, as
further demand was made by the family of the appellant. However, allegedly at the time
of marriage the parents of the appellant demanded a further sum of Rs. 40,000/- in cash
which could not be fulfilled. For non-fulfilment of the said demand, the deceased was
allegedly tortured.
3. Appellant had sent a telegram on 16.04.1988 to Des Raj stating :
"MUTUAL MISUNDERSTANDING REACH IMMEDIATELY RAMESHWAR"
On or about 4.06.1986, Sat Paul, brother of the deceased (PW-1), visited the house of his
brother-in-law Raj Kumar (PW-2), which is situate in the village, where the incident took
place and stayed there for the night. On the morning of the next day, both of them visited
the house of the appellant. Appellant and the deceased were found quarrelling with each
other on account of demand of dowry of Rs. 40,000/-. He tried to pacify them. Appellant
left his house at about 9 a.m. Sat Paul and Raj Kumar also went to the town. However,
when they returned after three or four hours, the house was found to be locked. On an
enquiry having been made from the neighbours, they were informed that the deceased
had been taken to hospital as she had consumed something. At about 1.30 p.m., they
reached the hospital at Bhatinda and came to know that Sushma Rani had expired. She
admittedly was pregnant at that time.
4. On receipt of a report in this behalf, ASI Mal Singh visited the hospital at about 1.20
p.m. It was at a distance of about 1 km. from the police post. Statement was made before
him by Sat Paul at about 2.45 p.m. The said statement was sent to the police station and
the same was recorded at 4 p.m. on the same day on the basis whereof the F.I.R. was
recorded. The statement of PW-2 Raj Kumar was also recorded on the
@page-SC892
same day. Appellant was arrested on 12.06.1988.
5. The learned Sessions Judge, Bhatinda convicted the appellant under Section 304B of
the Indian Penal Code and sentenced him to undergo rigorous imprisonment for seven
years. An appeal filed thereagainst by the appellant was dismissed by the High Court by
reason of the impugned judgment.
6. Mr. Rajiv K. Garg, learned counsel appearing on behalf of the appellant in support of
this appeal inter alia would submit:
(i) The relation between the parties being cordial, the prosecution cannot be said to have
proved its case of causing harassment by the appellant to his wife.
(ii) The prosecution has not been able to prove demand of dowry of Rs. 40,000/?.
(iii) Des Raj in his previous statement did not state about the demand of dowry of Rs.
40,000/-. A complaint petition was later on filed under Sections 405 and 406 of the Indian
Penal Code and Sections 3 and 4 of the Dowry Prohibition Act which ended in acquittal
by a judgment dated 5.11.1992 and a revision petition filed thereagainst by Des Raj was
withdrawn on 2.03.1995 which clearly establishes innocence on the part of the Appellant.
(iv) The family of the deceased having suffered several deaths, the deceased started
suffering from severe depression wherefor she was given a 'taveej' which also goes to
show that she had committed suicide.
(v) Even after the conviction, a settlement had been arrived at by and between the parties
and in fact the cousin of the deceased was married to the appellant whereafter an
application for compromise had also been filed.
7. The learned counsel for the State, on the other hand, supported the impugned
judgment.
8. The fact that the deceased committed suicide is not in dispute. The only question is
whether the appellant is guilty of commission of an offence under Section304B of the
Indian Penal Code. Before the learned Sessions Judge, the prosecution inter alia
examined PW-1 Sat Paul, PW-2 Raj Kumar, PW-3 Dr. Balbir Singh, PW-4 Dr. S.K.
Gupta, PW-5 Gurjant Singh Record Keeper), PW-6 Anup Krishan (Branch Manager,
State Bank of Patiala, Chandigarh), PW-7 G.S. Mann, (Manager, State Bank of Patiala,
Bhatinda), PW-8 Gulab Chand (Supervisor Telegraph Office), Bhatinda, PW-11 Des Raj
and PW-12 ASI Mal Singh.
9. The defence of the appellant was that she had been suffering from several diseases like
depression, leucorrhea, abdominal trouble and was on medicine. He also denied that Sat
Paul came to Bhatinda on 4.06.1988. According to him, Shri Joginder Singh Bedi was
requested to inform his in-laws on telephone about the death of Sushma, whereupon his
father-in-law and other relations came in the night and demanded valuables including the
ornaments and on his refusal to give the same, they had filed a false case. He also alleged
that he was detained by the police in the hospital.
10. To prove mal treatment of the deceased on the part of the appellant, three witnesses
were examined, viz., PW-1 Sat Paul, PW-2 Raj Kumar and PW-11 Des Raj. PW-3 Dr.
Balbir Singh examined the deceased when she was brought to the Civil Hospital. PW-4
Dr. S.K. Gupta conducted the post mortem examination at 9 a.m. on 6.06.1988. The
report reads as under :
"Post-mortem staining was present on the dependant part of the body. Rigor mortis was
present. Eyes were congested enclosed. Fluid was coming out from nostrils and mouth.
Level of the uterus was 3" above the umbilicus. Ants were creeping on the body. Multiple
abrasions were seen on the waist line, neck, arm with clotted blood. These abrasions and
clotted blood was post-mortem and is caused by ants. Lungs were congested. Stomach
contained about 1/4th ounce of semi-digested food small intestines contained a small
amount of chime. Large intestines contained some faecal matter. Bladder contained 2
ounces of urine. On dissection of the uterus, it was engorged with dilated veins and going
into the abdomen about 2½" above the umbilicus. A male dead foetus was lyng in the
utering cavity. The length of the foetus was 37 cms and weight was 1-2/4 kgs. Some
cutaneus fat was present. Meconium was present in the large intestines. There was a
centre of ossification of the talus bone. Skin was red. No centre was seen in the upper end
of femur. The probable time that elapsed between injuries and death will be known after
receiving the report of the Chemical Examiner, Patiala. The time between death and post-
mortem was within 24 hours.
@page-SC893
11. The Branch Manager of the State Bank of India Shri Anup Krishan PW-6 proved
preparation of the demand draft of Rs. 25,000/- from Sector 22-D Branch of Chandigarh
on 1.02.1986 as also a draft dated 19.03.1986 for a sum of Rs. 5,000/- which was issued
from Chandigarh in favour of Rameshwar Dass (appellant) payable at Mansa. PW-7 G.S.
Mann, Manager, State Bank of Patiala, Bhatinda proved payment of the said drafts to the
appellant.
12. Apart from the said draft of Rs. 25,000/- dated 1.02.1986 which was encashed on
3.02.1986 at Mansa, the prosecution, thus, had also been able to prove payment of a sum
of Rs. 5,000/- to the appellant by demand draft dated 19.03.1986.
13. The learned Sessions Judge relied upon the testimonies of Sat Paul and Des Raj. The
court found corroboration in their testimonies from the fact of payment of money by way
of demand drafts. The learned Sessions Judge also took into consideration the fact that
some quarrel must have taken place between the spouses wherefor the telegram dated
16.04.1988 was sent to Des Raj by the appellant himself.
14. The fact that the death took place within seven years from the date of marriage is not
in dispute. Commission of suicide by Sushma Rani is also not in dispute.
15. The only question which is required to be taken into consideration is as to whether the
deceased soon before her death was subjected to cruelty or harassment by her husband for
or in connection with the demand of dowry.
16. The defence of the appellant, as noticed hereinbefore, was that the relationship
between the parties was cordial. An attempt was made to prove a few letters allegedly
written by the deceased to the accused. The said letters were sought to be proved by one
Joginder Singh Bedi. He was cited as a witness on behalf of the prosecution. He had
made his statement before the police under Section 161 of the Indian Penal Code.
Evidently, he was given up. The learned Sessions Judge disbelieved his testimony
opining that it is wholly unnatural that he would see the deceased writing a letter. He
made a statement that the deceased was writing letters to him as also his wife. The said
letters were not produced. The learned Sessions Judge also took into consideration the
fact that there was nothing to show as was contended by the appellant that the
investigating officer took possession of the said letters wherefor no seizure memo was
prepared and handed over the same to the accused. Such a statement made by Joginder
was held to be wholly unnatural. The High Court in its impugned judgment endorsed the
said view.
17. A document in terms of Section 65 of the Evidence Act is to be proved by a person
who is acquainted with the handwriting of the author thereof. DW-1 Joginder Singh Bedi
claimed his acquaintance with the handwriting of the deceased on the basis of the letters
written to him and his wife. He claimed that the deceased had written a large number of
letters. It was, therefore, expected that some of them would be preserved. He did not
produce any letter.
18. DW-1, as noticed hereinbefore, was a prosecution witness. He evidently was won
over and, thus, prosecution did not examine him as its witness. Submission of Mr. Garg
that he was the only independent witness who was available and was not examined must
be considered from that angle. In response to the questions posed by the learned Sessions
Judge, he even denied making any statement before the police. We wish that the learned
Magistrate had recalled the investigating officer to bring the earlier statements of DW-1
to his notice. Unfortunately, no such step was taken. Furthermore, we fail to comprehend
as to why the father and brother of the deceased in the course of their depositions were
not confronted with the said letters. In ordinary course, it should have been done. Why
such a course of action was not taken recourse to is anybody's guess. The purported
explanation of the appellant that the said letters were taken into custody of the
investigating officer and later on returned to him is wholly unbelievable. An investigating
officer would not seize letters without preparing any seizure list. When the investigating
officer visited the place of occurrence, the appellant was not present. Even a copy of the
telegram which was found in the house had been seized. Other articles had also been
seized. If seizure list had been prepared, there does not appear to be any reason
whatsoever as to why the letters purported to have been written by the deceased would
not be mentioned in the seizure list. We, therefore, do not find any reason to differ with
the view taken by the learned Sessions
@page-SC894
Judge as also the High Court in this behalf.
19. So far as the submission of Mr. Garg that the subsequent complaint made by father of
the deceased as against the appellant for commission of an offence under Sections 405
and 406 of the Indian Penal Code read with Sections 3 and 4 of the Dowry Prohibition
Act ended in acquittal, is concerned, in our opinion, the same is not very relevant. The
said complaint was in relation to a demand made before the marriage took place.
Allegations made therein were confined to non-return of the articles which was allegedly
given to the bride for her own use. Misappropriation of the said articles was alleged on
the part of the appellant therein. The learned Magistrate while passing the said judgment
of acquittal inter alia opined that whereas the letter containing the draft of Rs. 5,000/- was
addressed to Rameshwar Dass, it was encashed by Rameshwar Garg.
Even in the purported letters written by the deceased to the appellant, the name of the
appellant was stated to be Rameshwar Dass Garg. The learned Magistrate, therefore, in
our opinion, misdirected himself in making such observations but it is not necessary for
us to express any opinion thereupon.
20. The said judgment of acquittal was passed on 5.11.1992, whereas the learned
Sessions Judge passed his judgment on 18.10.1989 in the instant case. The charges in
both the cases were not same. They were based on different transactions. Surprisingly, for
reasons best known to the appellant, no attempt was made even to bring the said
judgment dated 5.11.1992 to the notice before the High Court. Had an application for
adducing additional evidence been filed, the High Court not only could have considered
the same, it could have also called for the record of the said case to examine the matter at
some length.
We, however, gave an opportunity to the appellant, in our anxiety to do justice to him, to
file an application for bringing the same on records. Such an application has been filed
and we have taken into consideration the effect of the said subsequent judgment. As two
cases, it will bear repetition to state, are on in respect of two different transactions, we are
of the opinion that the same does not have any relevance in the present case. For the self-
same reasons, the purported withdrawal of the criminal revision application filed by Des
Raj thereagainst is also not of much relevance. Both PWs 1 and 2 in no uncertain terms
stated about the demand of dowry of Rs. 40,000/-.
21. Apart therefrom, we have noticed hereinbefore that the fard-bayan was recorded in
the hospital. When the investigating officer on receipt of the report went to the hospital,
there was none. In normal course, the appellant should have been in the hospital. The first
informant arrived at the hospital within a few minutes. He upon receipt of the news of his
sisters death made a statement at the hospital at about 2.30 p.m. In a situation of this
nature, he would not make a statement which is imaginary so as to implicate the appellant
falsely. Apart from the oral testimony of the prosecution witnesses, the very fact that the
appellant himself had sent the telegram asking Des Raj to come immediately is also a
pointer to the fact that the spouses were not on good terms.
22. There appears to be a ring of truth in the statement of the first informant that a
telegram was sent. He came to see his sister. His brother-in-law was the resident of the
same village. After he spent his night at his brother-in law's place, he had visited the
appellant's house together with him. He tried to pacify them. Only when the appellant
went out, they also went out and came back after a few hours to find the door closed.
Appellant evidently tried to show that he had gone to office. It was a Sunday and as such
the question of his going to office did not arise.
23. Another factor which is of some significance cannot also be lost sight of. The
deceased was in the family way. She was carrying for 5-6 months. Apart from the
statement of her father and brother to that effect, even the post mortem report clearly
proved the said fact. Des Raj in his testimony denied and disputed that a "taveej" was
given to her because she was suffering from depression. According to him, it was given
for the birth of a child. A pregnant woman ordinarily would not commit suicide unless
relationship with her husband comes to such a pass that she would be compelled to do so.
24. An attempt had also been made to show that there was death of two children in the
family as also a brother of the deceased. It was stated that one of sisters committed
suicide and the 'taveej' was given for her warding off the evil spirits of her deceased
sister. No such evidence has been
@page-SC895
brought on record.
25. We must also notice the conduct of the appellant.
In his statement under Section 313 of the Code of Criminal Procedure, the appellant
accepted that at the time of his marriage, a demand for Rs. 40,000/- was made from the
parents of the deceased and they showed their inability to pay the same.

ASI Mal Singh, investigating officer stated that when he had visited the Hospital, nobody
was present. If the statement of the appellant is to be believed that he immediately rushed
to the hospital, there was no reason of his not being there. According to him, he was
detained in the hospital but the same has not been proved. It was only on 12.06.1988 that
Sub-Inspector Sukhdev Ram had arrested him. Why he was not available for six days has
not been explained by the appellant. Even there was nothing on record to show that it was
either DW-1 or the appellant, who got the deceased admitted in the hospital. The
prosecution has discharged its primary onus, as envisaged under Section 304B of the
Indian Penal Code. In terms of Section 113B of the Indian Evidence Act, onus of proof
was upon the appellant. [See State of Karnataka v. M.V. Manjunathegowda and Anr.,
(2003) 2 SCC 188]. As the defence taken by the appellant has not been established, he
cannot be held to have discharged the said onus. 2003 AIR SCW 241

26. We, therefore, are of the opinion that there is no merit in this appeal which is
dismissed accordingly.
27. Before parting, however, we must notice the submissions made by Mr. Garg that 21
years have passed, the appellant has married a cousin of the deceased and an application
has been filed by Des Raj for condoning the offence.
28. An offence under Section 304B of the Indian Penal Code is not compoundable. Why
and under what circumstances the cousin of the deceased, if any, has been married to the
appellant is not known. Only because such a marriage has allegedly taken place, the same
by itself cannot be said to be a ground for rejecting the prosecution story. These
contentions cannot also persuade us to impose a lesser punishment upon the appellant as
in terms of Section 304B of the Indian Penal Code the minimum sentence is seven years
rigorous imprisonment. Appellant has been awarded only the minimum sentence. We,
therefore, cannot even interfere with the quantum of the sentence.
29. The appeal, for the reasons stated above, is dismissed.
Appeal dismissed.
AIR 2008 SUPREME COURT 895 "Rugmini Ammal v. V. Narayana Reddiar"
(From : 2001 Cri LJ 81 (Kerala))
Coram : 2 Dr. A. PASAYAT AND P. SATHASIVAM, JJ.
Civil Appeal No. 564 of 2002, D/- 13 -12 -2007.
Rugmini Ammal (Dead by L. Rs.) v. V. Narayana Reddiar and Ors.
Criminal P.C. (2 of 1974), S.340 - CONTEMPT OF LAWFUL AUTHORITY -
DOCUMENTS - FORGERY - Forged document - Production of - Prayer for conduct of
enquiry into forged document and filing of complaint by Court - Document in question
was produced before Government and not before Court - Initiation of proceeding under S.
340 by Court was without jurisdiction.
AIR 1998 SC 1121; 2005 AIR SCW 1929, Rel. on.
1996 AIR SCW 1850, Ref. (Paras 6, 8, 11, 13)
Cases Referred : Chronological Paras
2005 AIR SCW 1929 : AIR 2005 SC 2119 : 2005 Cri LJ 2161 (Rel. on) 11, 12
1998 AIR SCW 932 : AIR 1998 SC 1121 : 1998 Cri LJ 1565 (Rel. on) 8, 9, 11, 13
1996 AIR SCW 1850 : AIR 1996 SC 1592 : 1996 Cri LJ 2304 (Ref.) 9, 11
1992 (2) Ker LT 358 7
P. Krishnamoorthy Sr. Advocate, Romy Chacko for Appellant; E. M. S. Anam, G.
Prakash, M. P. Vinod, Ajay K. Jain and Vimelsh Kumar for Respondents.
Judgement
Dr. ARIJIT PASAYAT, J. :- Challenge in this appeal is to the judgment of a Division
Bench of the Kerala High Court by which the judgment of the learned Single Judge was
set aside and the writ appeal was allowed.
2. Background facts in a nutshell are as follows:
Respondent No.4 in CMP No.35930 of 1998 in O.P. No.12701 of 1998 was the 4th
respondent in writ petition also. According to the respondent No.1 he was the tenant of a
building called 'Jaya building' Main Road,
@page-SC896
Kollam. The tenancy was given by one Durairaja Reddiar by executing an agreement of
lease dated 6.1.1994. This lease deed enabled him to make alterations in the building.
Accordingly he effected some alterations in the building. When it was found that the
alterations were effected he received from the Kollam Municipality an order directing
him to demolish the structure which according to the Municipality was unauthorized.
Against the order of the Municipality respondent No.1 approached the Government. The
Government issued an order dated 22.6.1998 which was annexed as Ext.P5 to the writ
petition, directing the respondent No.1 to submit an application to the local authority
seeking regularization of the additional structure made by him. Rugmini Ammal, the first
respondent in the Writ Appeal filed a writ petition. The contention raised by her was that
there was no lease agreement and that the construction was unauthorized. It was stated
that the construction cannot be legalized on other grounds.
3. Respondent No.1 filed a counter-affidavit. Along with counter-affidavit photocopy of
the agreement of lease dated 6.1.1994 was annexed. Thereafter Rugmini Ammal filed
CMP No.35930 of 1998. The contentions taken in the CMP was that the purported
agreement of lease is a forged document. It was further stated that she sought the opinion
of Professor B.B. Kashyap, a renowned handwriting and finger print expert. The
signatures in the purported lease agreement, Exh. R4(a) was compared with the admitted
signatures of Durairaj Reddiar in Ext. P7. The expert gave his opinion, the copy of which
was produced as Exh.P18. According to it the signatures in Exh.R4(a) did not tally with
the admitted signatures. Hence the handwriting expert was of the opinion that the five
disputed signatures were not written by the writer of the admitted signatures.
4. A counter-affidavit was filed in CMP No.35930 of 1998. In the counter-affidavit, it was
stated that Exh. R4(a) was produced before the Government and Exh. P5 order itself goes
to show that this was produced before the Government. The fabrication of Exh.R4(a) was
denied. A reply affidavit was filed in which Rugmini Ammal denied the execution of
certain documents signed by Reddiar and produced by the appellant in the writ appeal.
The prayer in CMP No.35930 of 1998 was to conduct enquiry into the production of
Exh.R4(a) forged document and made a complaint thereof and forward it to the
Magistrate of the First Class having jurisdiction.
5. Learned Single Judge relied on the opinion given by the handwriting expert and prima
facie came to the conclusion that Exts.R4(a), R4(e), R4(i) and R4(j) were fabricated and
hence there is a reasonable likelihood to establish the offences punishable under Sections
463, 471, 475 and 476 of the Indian Penal Code, 1860 (in short the 'IPC'). The learned
Single Judge directed the Registrar of the Court to make a complaint for the purpose in
writing and send it to the Magistrate of the First Class having jurisdiction. Against that
order the writ appeal was filed.
6. Stand of the appellant before the High Court was that proceedings under Section 340
of the Code of Criminal Procedure, 1973 (in short the 'Cr.P.C'.) cannot be initiated
because there was no allegation that the fabrication was made after the document was
produced. Certain other stands were also taken with which we are not very much
concerned.
7. Stand of the respondents in the writ appeal was that the writ appeal was not
maintainable and Section 341 of Cr.P.C. does not provide for an order passed by the High
Court. It was submitted that Section 340 Cr.P.C. has been rightly initiated. The High
Court was of the view that though Section 341 Cr.P.C. does not provide for an appeal
from an order passed under Section 340 Cr.P.C. it does not mean that there was no other
provision by which appeal cannot be filed. In fact it was held that Section 5 of the Kerala
High Court Act provides for such an appeal. Reference in this context was made to a Five
Judge Bench decision of the High Court in K.S. Dass v. State of Kerala [1992 (2) KLT
358]. Reference was also made to some other full Bench judgments.
8

. Coming to the question about the applicability of Section 340 to the facts of the case it
was held that stage for initiation of the proceeding, if any, under Section 340 Cr.P.C. had
not come. Reference was made to a decision of this Court in Sachida Nand Singh and
Anr. v. State of Bihar and Anr. [AIR 1998 SC 1121]. Therefore, it was held that initiation
of the proceeding under Section 340 Cr.P.C. was without jurisdiction. The writ appeal
was accordingly allowed. 1998 AIR SCW 932

@page-SC897
9

. Learned counsel for the appellant submitted that there is a conflict in view between the
decision in Sachida Nands case (supra) and Surjit Singh and Ors. v. Balbir Singh (1996
(3) SCC 533). 1996 AIR SCW 1850

10. Learned counsel for the respondents on the other hand supported the order of the
High Court.
11

. At this juncture it is to be noted that in view of the conflict of language between two
decisions of this Court each rendered by a Bench of three learned Judges in Sachida
Nand's case (supra) and Surjit Singh's case (supra) regarding interpretation of Section
195(1)(b)(ii) Cr.P.C. the matter was placed before a five-judge Bench in Iqbal Singh
Marwah v. Meenakshi Marwah [2005 (4) SCC 370]. After referring to the provisions
contained in Sections 190, 195(1)(b)(ii) and 340 Cr.P.C. it was held that the decision in
Sachida Nand's case (supra) correctly decided and the view taken is the correct view. It
was, inter alia, observed as follows: 1998 AIR SCW 932
1996 AIR SCW 1850
2005 AIR SCW 1929, Paras 16, 21 and 25
"19. As mentioned earlier, the words "by a party to any proceeding in any court"
occurring in Section 195(1)( c ) of the old Code have been omitted in Section 195(1) (b)
(ii) Cr.P.C. Why these words were deleted in the corresponding provision of the Code of
Criminal Procedure, 1973 will be apparent from the 41st Report of the Law Commission
which said as under in para 15.39 :
"15.39. The purpose of the section is to bar private prosecutions where the course of
justice is sought to be perverted leaving to the court itself to uphold its dignity and
prestige. On principle there is no reason why the safeguard in clause (c) should not apply
to offences committed by witnesses also. Witnesses need as much protection against
vexatious prosecutions as parties and the court should have as much control over the acts
of witnesses that enter as a component of a judicial proceeding, as over the acts of parties.
If, therefore, the provisions of clause (c) are extended to witnesses, the extension would
be in conformity with the broad principle which forms the basis of Section 195."
20. Since the object of deletion of the words "by a party to any proceeding in any court"
occurring in Section 195(1)(c) of the old Code is to afford protection to witnesses also,
the interpretation placed on the said provision in the earlier decisions would still hold
good.
21. Section 190, Cr.P.C. provides that a Magistrate may take cognizance of any offence
(a) upon receiving a complaint of facts which constitute such offence, (b) upon a police
report of such facts, and (c) upon information received from any person other than a
police officer, or upon his own knowledge, that such offence has been committed. Section
195, Cr.P.C. is a sort of exception to this general provision and creates an embargo upon
the power of the court to take cognizance of certain types of offences enumerated therein.
The procedure for filing a complaint by the court as contemplated by Section 195(1),
Cr.P.C. is given in Section 340, Cr.P.C. and sub-sections (1) and (2) thereof are being
reproduced below :
"340. Procedure in cases mentioned in Section 195.- (1) When, upon an application made
to it in this behalf or otherwise, any court is of opinion that it is expedient in the interests
of justice that an inquiry should be made into any offence referred to in clause (b) of sub-
section (1) of Section 195, which appears to have been committed in or in relation to a
proceeding in that court or, as the case may be, in respect of a document produced or
given in evidence in a proceeding in that court, such court may, after such preliminary
inquiry, if any, as it thinks necessary,-
(a) record a finding to that effect;
(b) make a complaint thereof in writing;
(c) send it to a Magistrate of the First Class having jurisdiction;
(d) take sufficient security for the appearance of the accused before such Magistrate, or if
the alleged offence is non-bailable and the court thinks it necessary so to do, send the
accused in custody to such Magistrate; and
(e) bind over any person to appear and give evidence before such Magistrate.
(2) The power conferred on a court by sub-section (1) in respect of an offence may, in any
case where that court has neither made a complaint under sub-section (1) in respect of
that offence nor rejected an application for the making of such complaint, be exercised by
the court to which such former court is subordinate within the meaning of sub-section (4)
of Section 195.
@page-SC898
Section 341, Cr.P.C. provides for an appeal to the court to which such former court is
subordinate within the meaning of sub-section (4) of Section 195, against the order
refusing to make a complaint or against an order directing filing of a complaint and in
such appeal the superior court may direct withdrawal of the complaint or making of the
complaint. Sub-section (2) of Section 343 lays down that when it is brought to the notice
of a Magistrate to whom a complaint has been made under Section 340 or 341 that an
appeal is pending against the decision arrived at in the judicial proceeding out of which
the matter has arisen, he may, if he thinks fit, at any stage, adjourn the hearing of the case
until such appeal is decided.
23. In view of the language used in Section 340, Cr.P.C. the court is not bound to make a
complaint regarding commission of an offence referred to in Section 195(1)(b), as the
section is conditioned by the words "court is of opinion that it is expedient in the interests
of justice." This shows that such a course will be adopted only if the interest of justice
requires and not in every case. Before filing of the complaint, the court may hold a
preliminary enquiry and record a finding to the effect that it is expedient in the interests
of justice that enquiry should be made into any of the offences referred to in Section
195(1)(b). This expediency will normally be judged by the court by weighing not the
magnitude of injury suffered by the person affected by such forgery or forged document,
but having regard to the effect or impact, such commission of offence has upon
administration of justice. It is possible that such forged document or forgery may cause a
very serious or substantial injury to a person in the sense that it may deprive him of a
very valuable property or status or the like, but such document may be just a piece of
evidence produced or given in evidence in court, where voluminous evidence may have
been adduced and the effect of such piece of evidence on the broad concept of
administration of justice may be minimal. In such circumstances, the court may not
consider it expedient in the interest of justice to make a complaint. The broad view of
clause (b)(i), as canvassed by learned counsel for the appellants, would render the victim
of such forgery or forged document remediless. Any interpretation which leads to a
situation where a victim of a crime is rendered remediless, has to be discarded.
There is another consideration which has to be kept in mind. Sub-section (1) of Section
340, Cr.P.C. contemplates holding of a preliminary enquiry. Normally, a direction for
filing of a complaint is not made during the pendency of the proceeding before the court
and this is done at the stage when the proceeding is concluded and the final judgment is
rendered. Section 341 provides for an appeal against an order directing filing of the
complaint. The hearing and ultimate decision of the appeal is bound to take time. Section
343(2) confers a discretion upon a court trying the complaint to adjourn the hearing of the
case if it is brought to its notice that an appeal is pending against the decision arrived at in
the judicial proceeding out of which the matter has arisen. In view of these provisions,
the complaint case may not proceed at all for decades specially in matters arising out of
civil suits where decisions are challenged in successive appellate fora which are time-
consuming. It is also to be noticed that there is no provision of appeal against an order
passed under Section343(2), whereby hearing of the case is adjourned until the decision
of the appeal. These provisions show that, in reality, the procedure prescribed for filing a
complaint by the court is such that it may not fructify in the actual trial of the offender for
an unusually long period. Delay in prosecution of a guilty person comes to his advantage
as witnesses become reluctant to give evidence and the evidence gets lost. This important
consideration dissuades us from accepting the broad interpretation sought to be placed
upon clause(b)(ii).

25. An enlarged interpretation to Section 195(1)(b)(ii), whereby the bar created by the
said provision would also operate where after commission of an act of forgery the
document is subsequently produced in court, is capable of great misuse. As pointed out in
Sachida Nand Singh 2 after preparing a forged document or committing an act of forgery,
a person may manage to get a proceeding instituted in any civil, criminal or revenue
court, either by himself or through someone set up by him and simply file the document
in the said proceeding. He would thus be protected from prosecution, either at the
instance of a private party or the police until the court, where the document has been
filed, itself chooses to file a complaint. The litigation may be a prolonged one due to
1998 AIR SCW 932

@page-SC899
which the actual trial of such a person may be delayed indefinitely. Such an interpretation
would be highly detrimental to the interest of the society at large.
26. Judicial notice can be taken of the fact that the courts are normally reluctant to direct
filing of a criminal complaint and such a course is rarely adopted. It will not be fair and
proper to give an interpretation which leads to a situation where a person alleged to have
committed an offence of the type enumerated in clause (b)(ii) is either not placed for trial
on account of non-filing of a complaint or if a complaint is filed, the same does not come
to its logical end. Judging from such an angle will be in consonance with the principle
that an unworkable or impracticable result should be avoided. In Statutory Interpretation
by Francis Bennion (3rd Edn.), para 313, the principle has been stated in the following
manner :
"The court seeks to avoid a construction of an enactment that produces an unworkable or
impracticable result, since this is unlikely to have been intended by Parliament.
Sometimes, however, there are overriding reasons for applying such a construction, for
example, where it appears that Parliament really intended it or the literal meaning is too
strong."
In view of the discussion made above, we are of the opinion that Sachida Nand Singh 2
has been correctly decided and the view taken therein is the correct view. Section 195(1)
(b)(ii) Cr.P.C. would be attracted only when the offences enumerated in the said provision
have been committed with respect to a document after it has been produced or given in
evidence in a proceeding in any court i.e. during the time when the document was in
custodia legis.
12

. The above position was highlighted in Iqbal Singh Marwah v. Meenakshi Marwah
(supra). 2005 AIR SCW 1929

13

. The High Court was, therefore, right in placing reliance on Sachida Nand's case (supra).
1998 AIR SCW 932
14. The appeal is, therefore, without merit and is, therefore, dismissed. There will be no
order as to costs.
Appeal dismissed.
AIR 2008 SUPREME COURT 899 "Anil Kumar Goel v. Kishan Chand Kaura"
(From : Punjab and Haryana)*
Coram : 2 Dr. A. PASAYAT AND AFTAB ALAM, JJ.
Criminal Appeal No. 1704 of 2007 (arising out of SLP (Cri.) No. 2429 of 2006), D/- 12
-12 -2007.
Anil Kumar Goel v. Kishan Chand Kaura.
Negotiable Instruments Act (26 of 1881), S.138, S.142(b), Proviso (as inserted by Act 55
of 2002) - DISHONOUR OF CHEQUE - COMPLAINT - COGNIZANCE OF
OFFENCE - GENERAL CLAUSES - Dishonour of cheque - Filing of complaint - Taking
of cognizance of offence - Extended period of limitation provided under proviso to S.
142(b) - Said provisol, hold not intended to operate retrospectively.
Crl. M. No. 10233-M of 2006, D/- 16-3-2006 (P and H), Reversed.
General Clauses Act (10 of 1897), S.5.
2004 AIR SCW 5174, Ref.
AIR 1984 SC 87, Relied on. (Paras 8, 9, 10)
Cases Referred : Chronological Paras
2004 AIR SCW 5174 : AIR 2004 SC 4674 : 2004 Cri LJ 4874 (Ref.) 2
AIR 1984 SC 87 (Rel. on) 8
Gagan Gupta, Ms. Gurpreet Bawa and Parmanand Gaur, for Appellant.
* Cri. M. No. 10233-M of 2006, D/- 16-3-2006 (PandH)
Judgement
Dr. ARIJIT PASAYAT, J .:- Leave granted.
2

. Challenge in this appeal is to the order passed by a learned Single Judge of the Punjab
and Haryana High Court dismissing the application filed in terms of Section 482 of the
Code of Criminal Procedure, 1973 (in short the 'Cr.P.C.'). Appellant had filed a petition
for quashing the complaint filed by the respondent in terms of Section 138 of the
Negotiable Instruments Act, 1881 (in short 'the Act') In the complaint it was averred that
a cheque was issued by the appellant on 31.3.1998 which was dishonoured by the bank
when presented on 11.4.1998. Notice dated 27.4.1998 was duly served on the appellant.
Since the accused appellant assured that the cheque will be honoured if it is presented
again, the cheque was presented but was again dishonoured on 30.9.1998 for which
notice dated 13.10.1998 was again served on the appellant. But no payment was made.
Appellant filed an application in terms of 2004 AIR SCW 5174

@page-SC900
Section 245 of the Code of Criminal Procedure, 1973 (in short the 'Cr.P.C'.) before the
trial Court for discharge. It was averred that the application was clearly barred by time
and therefore the said application ought to be dismissed at the outset. The motion was
opposed by the respondent. The learned Judicial Magistrate dismissed the application
stating that in view of the judgment in Adalat Prasad v. Rooplal Jindal and Others [2004
(7) SCC 338], the trial Court cannot review or reconsider the order issuing process; once
process has been issued pursuant to an order passed in a complaint case. Appellant filed a
petition in terms of Section 482 Cr.P.C. which as noticed above was dismissed. It is to be
noted that the only stand of the appellant before the High Court was that even if the
position as stated by the respondent is accepted to be correct, in view of Section 142 B of
the Act, a complaint was not to be entertained. High Court dismissed the application on
the ground that proviso of Section 142 (b) of the Act was inserted vide Act 55 of 2002
which empowered the court to extend the period of limitation on sufficient cause being
shown. Therefore, the petition was to be dismissed.
3. In support of the appeal, learned counsel for the appellant submitted that the
amendment inserted by Act 55 of 2002 had no application to the facts of the case as the
various events took place much prior to 2002 and in fact the complaint was filed on
28.11.1998. It was further pointed out that the case of respondent was not that case in
hand was covered by the amendment. There is no such plea taken. The High Court could
not have made out a new case.
4. There is no appearance on behalf of the respondent.
5. For resolution of the controversy Sections 138 and 142 of the Act are relevant. They
read as follows :
"Section 138 :
Dishonour of cheque for insufficiency, etc. of funds in the account - Where any cheque
drawn by a person on an account maintained by him with a banker for payment of any
amount of money to another person from out of that account for the discharge, in whole
or in part, of any debt or other liability, is returned by the bank unpaid, either because of
the amount of money standing to the credit of that account is insufficient to honour the
cheque or that it exceeds the amount arranged to be paid from that account by an
agreement made with that bank, such person shall be deemed to have committed an
offence and shall, without prejudice to any other provision of this Act, be punished with
imprisonment for a term which may extend to one year, or with fine which may extend to
twice the amount of the cheque, or with both :
Provided that nothing contained in this section shall apply unless-
(a) the cheque has been presented to the bank within a period of six months from the date
on which it is drawn or within the period of its validity, whichever is earlier;
(b) the payee or the holder in due course of the cheque, as the case may be, makes a
demand for the payment of the said amount of money by giving a notice in writing, to the
drawer of the cheque, within fifteen days of the receipt of information by him from the
bank regarding the return of the cheque as unpaid; and
(c) the drawer of such cheque fails to make the payment of the said amount of money to
the payee or as the case may be, to the holder in due course of the cheque within fifteen
days of the receipt of the said notice.
Explanation - For the purposes of this section, "debt or other liability" means a legally
enforceable debt or other liability.
Section 142 :
Cognizance of offences - Notwithstanding anything contained in the Code of Criminal
Procedure, 1973 (2 of 1974),-
(a) no court shall take cognizance of any offence punishable under Section 138 except
upon a complaint, in writing, made by the payee or, as the case may be, the holder in due
course of the cheque;
(b) such complaint is made within one month of the date on which the cause of action
arises under clause (c) of the proviso to Section 138;
(Provided that the cognizance of a complaint may be taken by the Court after the
prescribed period, if the complainant satisfies the Court that he had sufficient cause for
not making a complaint within such period.)
(c) no court inferior to that of a Metropolitan Magistrate or a Judicial Magistrate of the
first class shall try any offence punishable under Section 138."
@page-SC901
6. Before the amendment, the proviso, as quoted above, was not there. Clause (a) of the
proviso to Section 138 does not put any embargo upon the payee to successively present a
dishonoured cheque during the period of its validity. This apart, in the course of business
transactions it is not uncommon for a cheque being returned due to insufficient funds or
similar such reasons and being presented again by the payee after sometime, on his own
volition or at the request of the drawer, in expectation that it would be encashed. The
primary interest of the payee is to get his money and not prosecution of the drawer,
recourse to which, normally, is taken out of compulsion and not choice. On each
presentation of the cheque and its dishonour, a fresh right-and not a cause of action -
accrues in his favour. He may, therefore, without taking pre-emptory action in exercise of
his such right under clause (b) of Section 138, go on presenting the cheque so as to enable
him to exercise such right at any point of time during the validity of the cheque.
7. Section 5 of the General Clauses Act, 1897 (in short the 'General Clauses Act') also
throws considerable light on the controversy. Section 5 reads as follows:
"5.Coming into operation of enactments- (1) Where any Central Act is not expressed to
come into operation on particular day, then it shall come into operation on the day on
which it receives the assent,-
(a) In the case of a Central Act made before the commencement of the Constitution of the
Governor-General and
(b) In the case of an Act of Parliament of the President.
(c) Unless the contrary is expressed a Central Act or Regulation shall be construed as
coming into operation immediately on the expiration of the day preceding its
commencement."
8. All laws that affect substantive rights generally operate prospectively and there is a
presumption against their retrospectivity if they affect vested rights and obligations,
unless the legislative intent is clear and compulsive. Such retrospective effect may be
given where there are express words giving retrospective effect or where the language
used necessarily implies that such retrospective operation is intended. Hence the question
whether a statutory provision has retrospective effect or not depends primarily on the
language in which it is couched. If the language is clear and unambiguous, effect will
have to be given to the provision is question in accordance with its tenor. If the language
is not clear then the Court has to decide whether, in the light of the surrounding
circumstances, retrospective effect should be given to it or not. (See : M/s Punjab Tin
Supply Co., Chandigarh etc. etc. v. Central Government and Ors. AIR 1984 SC 87).
9. There is nothing in the amendment made to Section 142(b) by the Act 55 of 2002 that
the same was intended to operate retrospectively. In fact that was not even the stand of
the respondent. Obviously, when the complaint was filed on 28.11.1998, the respondent
could not have foreseen that in future any amendment providing for extending the period
of limitation on sufficient cause being shown would be enacted.
10. That being so the High Court's view is clearly unacceptable. The impugned order of
the High Court is set aside. The proceeding pursuant to respondents complaint i.e.
Complaint No.120 of 1998 in the Court of JMIC, Chandigarh, is quashed.
11. The appeal is allowed.
Appeal allowed.
AIR 2008 SUPREME COURT 901 "Gurunath Manohar Pavaskar v. Nagesh Siddappa
Navalgund"
(From : Karnataka)*
Coram : 2 S. B. SINHA AND H. S. BEDI, JJ.
Civil Appeal No. 5794 of 2007 (arising out of SLP (C) No. 20584 of 2005), D/- 11 -12
-2007.
Gurunath Manohar Pavaskar and Ors. v. Nagesh Siddappa Navalgund and Ors.
(A) Specific Relief Act (47 of 1963), S.38 - INJUNCTION - DECREE -
ENCROACHMENT - Suit for permanent injunction - Directing defendants to demolish
construction made on encroached land and handover vacant possession - Suit allowed on
basis of entries in Revenue records - Decree for permanent injunction in mandatory form
passed without deciding title of plaintiff - Is improper.
R. S. A. No. 135 of 2003, D/- 4-7-2005, (Kar.), Reversed.
It is one thing to say that the Courts could pass an interlocutory order in the nature of
mandatory injunction in exercise of its
@page-SC902
jurisdiction under S. 151 of Civil P. C. on the premise that a party against whom an order
of injunction was passed, acted in breach thereof; so as to relegate the parties to the same
position as if the order of injunction has not been violated, but, it is another thing to say
that the Courts shall exercise the same power while granting a decree of permanent
injunction in mandatory form without deciding the question of title and /or leaving the
same open. The decree for permanent mandatory injunction passed by relying upon
revenue records and without deciding title of plaintiff was liable to be set aside.
(Para 13)
(B) Evidence Act (1 of 1872), S.35, S.114 - EVIDENCE - POSSESSION - Revenue
record - Not document of title - It merely raises presumption of possession. (Para
12)
Cases Referred : Chronological Paras
2007 AIR SCW 4165 : AIR 2007 SC 2349 11
S. N. Bhat, for Appellant; Ms. Kiran Suri, Rajesh Mahale, for Respondents.
* R. S. A. No. 135 of 2003, D/- 4-7-2005 (Kar.)
Judgement
S. B. SINHA, J. :- Leave granted.
2. Defendants before the Trial Court are the appellants herein.
3. Plaintiffs Respondents filed a suit against the appellants praying inter alia for the
following reliefs :
"(a) That the encroached portion of the suit property by erection of structure measuring
369 1/9 sq. yards be directed to be demolished at the cost and risk of Defendant No. 1 to
5 consequently defendants be further directed to maintain the rules of set-back in respect
of his remaining construction enabling plaintiff to use and enjoy the free light and air to
his property and similarly defendants No. 6 be directed to remove the sign board and the
firm from the encroached area of the suit property. Further defendants be directed to give
the respective vacant possession of the suit land to the plaintiffs.
(aa) A decree of permanent injunction against defendants, their agents, their relative or
any body on their behalf to interfere with the plaintiffs peaceful possession and
enjoyment of suit property"
4. Respondents contended that they are owners of a portion of Survey No. 1008/1 bearing
CTS Nos. 4823/A-17 and 4823/A-18 measuring 662 2/9 and 533 3/9 square yards
respectively and the appellants who are the owners of the abutting land bearing CTS No.
4823/A-1 had encroached upon a portion of CTS Nos. 4823/A-17 and 4823/A-18
measuring 249 1/9 and 120 square yards respectively. Plaintiffs purchased the said plots
by a deed of sale dated 7.11.1984, whereas the date of purchase made by the defendants
dated 17.8.1992
5. The learned Trial Judge having regard to the pleadings of the parties framed issues;
issue No. 3 whereof reads as under :
"3. Whether the defendant Nos. 1 to 5 proves that the vendor of the plaintiff by way of
fabrication of false documents had sold the suit schedule property to these plaintiffs, thus,
the plaintiffs are not the owners of the suit schedule property?"
It was answered stating :
"My answers to the above issues are as follows :
*** *** ***
Issue No. 3 - Does not arise."
6. During the pendency of the said suit, an application for injunction was filed. Allegedly,
the appellants raised constructions upon the suit land in violation of the said order of
injunction. The learned Trial Judge in regard to the title of the plaintiffs over the suit land
held :
"..........According to the learned counsel for the plaintiff since CTS No. 4823/A1 is
completely acquired by the Municipal Corporation Belgaum for Malmaruti Extension
scheme then the property of the defendant No. 1 to 6 is not in existence in the name of
defendants. But according to me since the defendant No. 1 to 5 also have purchased the
property through a registered sale deed and also their vendors have also purchased the
said property through a registered sale deed and as such it cannot be said that the property
of defendants are not in existence. But at the same time the say of the defendant cannot
be taken into believed (sic) that the CTS No. 4823/A17 and 4823/A18 are not in
existence. When in the survey map as well as in other documents these properties are
clearly demarcated and identified then according to me, these properties have been
clearly demarcated in relevant records..........."
7. The High Court affirmed the said findings stating :
@page-SC903
"It is also clear from the perusal of the judgment and decree passed by the courts below
that both the courts below have rightly decided on the basis that it is unnecessary to give
any decision on the title of the property as the suit is for permanent and mandatory
injunction and the trial Court has rightly observed that it is always open to the defendants
to work out their remedy in accordance with law, regarding their title to the property CTS
No. 4823/A1 and no finding could be given on title in the present case and when there is
no finding on the title of the property in the present case, it is clear that it is always open
to the defendants to work out their remedy, in accordance with law. It is clear from the
perusal of the material on record that defendant No. 6 who also suffered decree of
injunction and permanent injunction though had filed first appeal before the lower
appellate court has not chosen to challenge the judgment and decree passed by first
appellate court in RA 252/2001.............."
8. Indisputably, an Advocate-Commissioner was appointed. He filed a report. An
objection thereto was also filed. He, however, could not be cross-examined. His report,
therefore, could not have been taken into consideration although the same formed part of
the record.
9. The High Court although took into consideration the fact that the plaintiffs did not seek
for any declaration of title, as noticed hereinbefore, opined that the question of title can
be gone into in an appropriate suit. All the Courts relied on Ex. P-35 which was allegedly
produced by the appellants but were made use of by the respondents, wherein it had been
shown that the chalta No. 63 was allotted in respect of CTS No. 4823/A-1, chalta No. 62-
A was allotted in respect of CTS No. 4823/A-17 and chalta No. 62-B was allotted in
respect of CTS No. 4823/A-18.
10. It is one thing to say that there does not exist any ambiguity as regards description of
the suit land in the plaint with reference to the boundaries as mentioned therein, but it is
another thing to say that the land in suit belongs to the respondents.
It was for the plaintiffs to prove that the land in suit formed part of CTS Nos. 4823/A-17
and 4823/A-18. It was not for the defendants to do so. It was, therefore, not necessary for
them to file an application for appointment of a Commissioner nor was it necessary for
them to adduce any independent evidence to establish that the report of the Advocate-
Commissioner was not correct. The suit could not have been, therefore, decreed inter alia
on the basis of Ex. P-35 alone. In a case of this nature, even Section 83 of the Indian
Evidence Act would not have any application.
11

. Furthermore, the High Court committed an error in also throwing the burden of proof
upon the defendants appellants without taking into consideration the provisions of
Section 101 of the Indian Evidence Act. In Narain Prasad Aggarwal (D) by LRs. v. State
of M.P. [2007 (8) Scale 250], this Court opined: 2007 AIR SCW 4165

"22. Record of right is not a document of title. Entries made therein in terms of Section
35 of the Indian Evidence Act although are admissible as a relevant piece of evidence and
although the same may also carry a presumption of correctness, but it is beyond any
doubt or dispute that such a presumption is rebuttable."
12. A revenue record is not a document of title. It merely raises a presumption in regar
d to possession. Presumption of possession and/ or continuity thereof both forward and
backward can also be raised under Section 110 of the Indian Evidence Act. The Courts
below, were, therefore, required to appreciate the evidence keeping in view the correct
legal principles in mind.
13. The Courts below appeared to have taken note of the entries made in the revenue
records wherein the name of the Municipal Corporation, Belgaum appeared in respect of
CTS No. 4823/A-1. We have, however, noticed that the learned Trial Judge proceeded on
the basis that the said property may be belonging to the defendants appellants. The Courts
below not only passed a decree for prohibitory injunction but also passed a decree for
mandatory injunction. The High Court opined that the Trial Court could exercise
discretion in this behalf. It is again one thing to say that the Courts could pass an
interlocutory order in the nature of mandatory injunction in exercise of its jurisdiction
under Section 151 of the Code of Civil Procedure on the premise that a party against
whom an order of injunction was passed, acted in breach thereof; so as to relegate the
parties to the same position as if the order of injunction has not been violated, but, it is
another thing to say that the Courts shall exercise the
@page-SC904
same power while granting a decree permanent injunction in mandatory form without
deciding the question of title and/or leaving the same open. How, in the event the
structures are demolished, it would be possible for the appellants to work out their
remedies in accordance with law in regard to the title of the property has not been spelt
out by the High Court.
14. We, therefore, are of the opinion that the interest of justice would be subserved if the
impugned judgments are set aside and the matter is remitted to the learned Trial Judge for
consideration of the matter afresh. The plaintiffs may, if they so desire, file an application
for amendment of plaint praying inter alia for declaration of his title as also for damages
as against the respondents for illegal occupation of the land. It would also be open to the
parties to adduce additional evidence(s). The learned Trial Judge may also appoint a
Commissioner for the purpose of measurement of the suit land whether an Advocate -
Commissioner or an officer of the Revenue Department.
15. Before us, additional documents have been filed by the appellants showing some
subsequent events. It would be open to the defendants to file an application for adduction
of additional evidence before the Trial Judge which may be considered on its own merits.
16. The appeal is allowed with the aforementioned observations. We would request the
Trial Court to consider the desirability of disposing of the matter as expeditiously as
possible and preferably within a period of six months from the date of communication of
this order. Costs of this appeal shall be the cost in the suit.
Appeal allowed.
AIR 2008 SUPREME COURT 904 "Sumersinbh Umedsinh Rajput v. State of Gujarat"
(From : Gujarat)*
Coram : 2 S. B. SINHA AND H. S. BEDI, JJ.
Criminal Appeal No. 1696 of 2007 (arising out of SLP (Cri.) No. 2557 of 2007), D/- 11
-12 -2007.
Sumersinbh Umedsinh Rajput alias Sumersinh v. State of Gujarat.
Penal Code (45 of 1860), S.307 - ATTEMPT TO MURDER - Attempt to murder - Car
driven by accused intercepted by complainant Police Officer - Other inmates fleeing
away - Scuffle ensuing when complainant tried to apprehend accused - Accused alleged
to have snatched service revolver of complainant and fired single shot - Pant and vest of
complainant both having one bullet hole - Bullet holes are incompatible with case of
single shot - Nature of injury suffered by complainant also incompatible with gun shot
injury - Seizure witnesses turning hostile - Prosecution case suffers from lot of
discrepancies - Conviction of accused liable to be set aside.
Cri. A. No. 1832 of 2006, D/- 7-12-2006 (Guj), Reversed. (Paras 9, 13, 15, 16)
Cases Referred : Chronological Paras
2004 AIR SCW 5779 : AIR 2004 SC 5068 : 2004 Cri LJ 4978 (Ref.) 14
2000 AIR SCW 2225 : AIR 2000 SC 2161 : 2000 Cri LJ 3182 (Ref.) 14
1996 AIR SCW 2094 : AIR 1996 SC 3236 : 1996 Cri LJ 2465 (Ref.) 14
S. B. Upadhyay, Sr. Advocate, Santosh Mishra, Shivmangal Sharma, Rajesh R. Dubey,
Tejmal Raka and Ms. Sharmila Upadhyay with him, for Appellant; V. Madhukar, Ms.
Pinky, Ms. Jesal Wahi and Ms. Sangeeta Singh (for Ms. Hemantika Wahi), for
Respondent.
* Cri. A. No. 1832 of 2006, D/- 7-12-2006 (Guj).
Judgement
S. B. SINHA, J. :- Leave granted.
2. Appellant was charged with and convicted for commission of offences under Sections
307 and 353 of the Indian Penal Code as also Section 25(1)(a) of the Arms Act; and
sentenced to undergo rigorous imprisonment of five years and fine of Rs. 5,000/-, two
years and fine of Rs. 1000/- and three years and fine of Rs. 1000/- respectively.
3. Prosecution case shortly stated is as under:
Appellant was a driver of a Tata Spacio Car. Three other persons were accompanying
him. They were sitting on the back seat. The said car was intercepted by the complainant
PSI Babaji Javanji Vaghela (PW-8) and other police officers. The said persons ran away.
The complainant Vaghela tried to pull the appellant out of the car. Allegedly, he resisted.
Force was applied to take him out of the car. A scuffle ensued, during which allegedly he
snatched the service revolver of the complainant and fired at him. Injury suf-fered by the
complainant Vaghela (PW-8) as appearing from the medical report, is as under :
@page-SC905
"H/O Firing has (sic) done by accused from the service revolver.
(illegible) on right side of loin (illegible)
1 x ½ cm abrasion (illegible) superficial
Black gas seen on cloth and puncture and baniyan occurs"
4. The clothes of the complainant as also the revolver with the cartridges were sent for
testing to the Forensic Science Laboratory. It was found :
"Sample-A : It is a pant. On being performing (sic) chemical analysis and microscopic
examination of the hole on the pocket of the said pant, it suggests that the hole on sample
A has occurred due to fire arms discharge. The hole on the said pant can take place with
the help of bullet of sample F.
Sample-B : It is a shirt. On being performing chemical analysis of the black spot that is
seen on the right hand side of the waist of the said shirt it is found that the black spot on
the right hand side of the waist of the said shirt has occurred due to fire arms discharge.
Sample-D: It is 0.38" revolver of Lama Company made in Spain.
On being analyzing barrel wash (before performing test firing in this laboratory) of the
said revolver the presence of residuals of nitrate and lead of the fire arms were seen. It
suggests that firing was done from the revolver of the said Sample D before it has been
brought to this laboratory.
On being firing from the chamber of the revolver of Sample D by taking two cartridges of
0.38" revolver from the stock of this laboratory, the same has been fired successfully. It
suggests that the revolver of the said Sample D is in working condition.
Sample-E: It is empty case of cartridge of K.F. 0.38" revolver. There was indentation
mark on the percussion cap of the said empty case of the cartridge. While performing
examination and comparison in the microscope about the characteristics of the
indentation mark on the percussion cap of the said cartridge and firing pin mark on the
percussion cap of the cartridge that was test fired from the revolver of Sample D, they
were found similar. It suggests that sample of cartridge of Sample E is fired from the
revolver of Sample D.
Sample-F: It is one copper jacketed bullet of 0.38" revolver cartridge. While performing
examination and comparison in the microscope about the characteristics of rifling mark
on the said bullet and rifling mark on the bullet that was test fired from the revolver of
Sample D, they were found similar. It suggests that bullet of Sample F is fired from the
revolver of Sample D.
Note : Two cases of cartridge test fired from Sample D and Bullet is enclosed with
parcel D.
The test report of blood present on the banyan of Parcel B (Sample B) will be sent
separately on being received from the biology department."
5. The complainant examined himself as PW-8. One Amratlal (PW-2) who is the PSI of
CID and had allegedly accompanied the complainant sought to support the prosecution
case. However, he did not have any personal knowledge about the incident. He heard
thereabout only from the complainant. In regard to seizure of the article, PW-7
Khengarbhai stated :
"How many panchanamas were prepared by police, that I do not know. I put my signature
in 4 to 5. The panchnama with regard to clothes was prepared first, thereafter panchnama
with regard to revolver was prepared. As soon as first panchnama was concluded, second
panchanama was prepared. When I went to police station that time clothes and revolver
were lying on table in police station. The police, who has prepared panchnama informed
me that those clothes belonged to PSI Vaghela. Tharad Police has shown revolver.
Vaghela was sitting there. The said revolver was empty however it did not open. How
many cartridges were present inside, I have not seen them. I have seen hole in vest and
trouser. The hole was present in left side of trouser. It was small and round, whatever has
been shown to me in round hole ;in our language. I do not remember now. Today, the
trouser which is shown to me has hole on right side."
6. All the witnesses who were said to be independent witnesses, viz., PWs-3, 6, 7 and 10
turned hostile. According to them, they were made to become witnesses of seizure of the
clothes, etc., which had been kept in the police station.
7. Dr. Deepak Kumar examined himself as PW-5. He in his evidence proved the medical
report. In his deposition for all intent and purport, he conceded the deficiencies in the
prosecution case vis-a-vis the report
@page-SC906
prepared by him, stating;
"It is true that I have written history in certificate, that history was recorded in Yaadi. If
vest has hole then shirt worn on that should have hole on it or if Bushirt is torn then shirt
also should have hole on it or Bushirt worn is found torn.
It is true that looking at trouser, I say that one circle is made on it with pencil. That is not
torn with bullet. It is true that looking at the trouser I say that, it is not entry cut. It is true
that if vest has hole then two holes should have found, one is entry and other exit hole.
Otherwise, in case of scratch, vest is found in similar torn manner.
It is true that I have not mentioned fire arm's marks. It is true that if any injury is caused
with fire arm or bullet then the edge has burn mark. In present case no burn injury is
found. It is true that if shooter fires from point blank range then black colour is found
near wound. When I saw injury of patient, it did not have such black mark on that. Shirt
had black mark. It is true that scratch mark can occur due to rubbing on rough substance."
8. From the statements made by PW-5, it is evident that even in relation to the purported
marks of entry of the bullet through the garments worn by the complainant, there existed
a lot of discrepancies. Some sort of make-shift report was placed before him by way of
"Yaadi", which prepared by the complainant and whereupon he completely relied. If no
burn injury was found in the clothes, it is difficult to believe that some burn injury was
noticed in the wound. Prosecution did not obtain any clarification from him as to whether
the nature of the injury which the complainant suffered could not take place due to
rubbing of the skin on a rough substance.
9. We must also notice that the injury received by the complainant was allegedly caused
to his loin. How such a simple injury could be caused from a shot fired from a fire-arm is
open to question. So far as the report of the Forensic Science Laboratory is concerned,
the clothes had not been identified as belonging to the injured. It may also be recorded
that two bullets were sent to the Forensic Science Laboratory, although the specific case
of the prosecution is that only one shot was fired. Two bullet holes were, therefore, not
possible to be caused, one in the trouser and other in the waist, by one shot of fire. It has
not been disclosed as to wherefrom the bullet was recovered. The mazhar witnesses did
not say that any bullet was recovered from the place of occurrence in their presence.
10. According to PW-8, he came to know about the firing on hearing of sound of fire. He
had immediately put his finger in the trigger of the revolver and caught the appellant
from his wrist. If the finger of the complainant himself was on the trigger of the revolver,
it is difficult to believe that the appellant was responsible for the act complained of.
According to him, seizure took place at the place of occurrence but panch witnesses
contradicted him as according to them, they were made to sign the seizure list only at the
police station. In his statement before the investigating officer under Section 161 of the
Code of Criminal Procedure, PW-8 stated :
".........three persons were there in Spacio but they all started running in the farm by
opening the doors of the vehicle and as the driver of the vehicle was sitting on the stirring
(sic for steering) wheel, we along with police personnel get down from our mobile van
and approach to catch the driver of Spacio........."
11. He resiled from the said statement and built up another story in his examination-in-
chief that other police personnel chased them and that they had fled away.
12. PW-9 Maan Singh in his deposition stated that Vaghela had held the hand of the
appellant and had been asking him to get down from his vehicle only when the scuffle
took place. The said witness stated that blood had oozed out but the vest of the
complainant did not contain any blood-stain. Significantly, PW-9 stated that the doors of
the vehicle near the driving seat were locked.
In his deposition, he stated :
"That time I have not seen him pulling out revolver. However, I saw revolver in his hand.
After firing sir hold his wrist. The hand of accused were tied from wrist. That time his
hand were in up side. That time firing did not occur. That is not true. Accused has not
done firing and sir did not get injury that is not true. Sir got scratch mark during scuffle.
Three accused who escaped and ran away, they were not caught."
13. There, thus, exists a lot of discrepancies in regard to the manner in which the incident
had taken place. The complainant
@page-SC907
himself in his evidence did not say that all the three persons, who had got down from the
rear seat and ran away, were chased by anybody.
14. Even assuming that PW-8 received a fire-arm injury which in the facts and
circumstances of the case does not appear to be plausible, having regard to the positive
evidence of the prosecution as has been stated by PW-4 Neelabhai it seems certain that a
scuffle had ensued. A case of Section 307 of the Indian Penal Code, therefore, has not
been made out.
The ingredients of Section 307 are :
(i) an intention of or knowledge relating to commission of murder; and
(ii) the doing of an act towards it.

[See Parsuram Pandey and others v. State of Bihar (2004) 13 SCC 189; Sagayam v. State
of Karnataka (2000) 4 SCC 454 and Merambhai Punjabhai Khachar and others v. State of
Gujarat, AIR 1996 SC 3236]. 2004 AIR SCW 5779
2000 AIR SCW 2225
1996 AIR SCW 2094

15. If the prosecution case of attempt to murder of PW-8 by gun-shot injury fails,
resultantly, the prosecution under Section 25 of the Arms Act would also fail.
16. Having regard to the facts and circumstances of this case, we are of the opinion that
no case has been made out even under Section 353 of the Indian Penal Code. The appeal
is allowed. Appellant is directed to be set at liberty unless wanted in connection with any
other case.
Appeal allowed.
AIR 2008 SUPREME COURT 907 "Sakiri Vasu v. State of U. P."
(From : Allahabad)
Coram : 2 A. K. MATHUR AND MARKANDEY KATJU, JJ.
Criminal Appeal No. 1685 of 2007 (arising out of SLP (Cri.) No. 6404 of 2007), D/- 7
-12 -2007.
Sakiri Vasu v. State of U.P. and Ors.
(A) Constitution of India, Art.226 - Delhi Special Police Establishment Act (25 of 1946),
S.3 - POLICE OFFICERS - CBI - INHERENT POWERS - INVESTIGATION - CBI
Investigation - Directions for - Should be issued only in some rare and exceptional case -
Dead body of Army personnel found on Railway Station - Finding by two Courts of
Inquiry held by Army Authorities and GRP that it was a case of suicide - Mere allegation
by father that his son was murdered because he had discovered some corruption cannot
justify a CBI inquiry - Refusal by writ Court to direct CBI enquiry, proper.
Criminal P.C. (2 of 1974), S.482. (Paras 32, 34)
(B) Constitution of India, Art.226 - WRITS - FIR - INVESTIGATION - INHERENT
POWERS - Writ petition - Alternative remedy - Petition against - Non-registration of F. I.
R. by Police - Not to be entertained - Aggrieved person can approach the Superintendent
of Police under Section 154(3), Cr. P. C. by an application in writing or can file
application before Magistrate under S. 156(3).
Criminal P.C. (2 of 1974), S.482, S.154, S.156. (Para 11)
Cases Referred : Chronological Paras
2007 AIR SCW 5899 : AIR 2007 SC 3234 : 2007 Cri LJ 4709 (Ref.) 13
2006 AIR SCW 95 : AIR 2006 SC 705 : 2006 Cri LJ 788 : 2006 (2) ALJ 8 : 2006 (1) AIR
Jhar R 641 (Ref.) 12
2003 AIR SCW 558 : AIR 2003 SC 1043 : 2003 AIR - Kant HCR 532 22
2003 AIR SCW 3258 : AIR 2003 SC 2612 : 2003 Cri LJ 3117 (Ref.) 29
2002 AIR SCW 2333 : AIR 2002 SC 2225 : 2002 Cri LJ 2942 : 2002 All LJ 1470 (Ref.)
33
2001 AIR SCW 305 : AIR 2001 SC 668 : 2001 Cri LJ 968 30, 31
1996 AIR SCW 384 : AIR 1996 SC 646 (Ref.) 22
1996 AIR SCW 3682 : AIR 1996 SC 3527 22
1996 AIR SCW 4249 : 1997 Cri LJ 63 : AIR 1997 SC 93 (Ref.) 10
(1996) 11 SCC 23 22
AIR 1991 SC 696 22
AIR 1986 SC 984 : 1986 Cri LJ 41 (Ref.) 23
AIR 1980 SC 326 : 1980 Cri LJ 98 (Ref.) 16, 30
AIR 1969 SC 430 21
AIR 1961 SC 1117 : 1961 (2) Cri LJ 161 (Ref.) 30
Dinesh Kumar Garg, for Appellant.
Judgement
1. MARKANDEY KATJU, J. :- Leave granted.
2. This appeal is directed against the impugned judgment and order dated 13.7.2007
passed by the Allahabad High
@page-SC908
Court in Criminal Misc. Writ Petition No. 9308 of 2007.
3. Heard learned counsel for the parties and perused the record.
4. The son of the appellant was a Major in the Indian Army. His dead body was found on
23.8.2003 at Mathura Railway Station. The G.R.P, Mathura investigated the matter and
gave a detailed report on 29.8.2003 stating that the death was due to an accident or
suicide.
5. The Army officials at Mathura also held two Courts of Inquiry and both times
submitted the report that the deceased Major S. Ravishankar had committed suicide at the
Railway track at Mathura junction. The Court of Inquiry relied on the statement of the
Sahayak (domestic servant) Pradeep Kumar who made a statement that deceased Major
Ravishankar never looked cheerful; he used to sit on a chair in the verandah gazing at the
roof with blank eyes and deeply involved in some thoughts and used to remain oblivious
of the surroundings. The Court of Inquiry also relied on the deposition of the main eye-
witness, gangman Roop Singh, who stated that Major Ravishankar was hit by a goods
train that came from Delhi.
6. The appellant who is the father of Major Ravishankar alleged that in fact it was a case
of murder and not suicide. He alleged that in the Mathura unit of the Army there was
rampant corruption about which Major Ravishankar came to know and he made oral
complaints about it to his superiors and also to his father. According to the appellant, it
was for this reason that his son was murdered.
7. The first Court of Inquiry was held by the Army which gave its report in September,
2003 stating that it was a case of suicide. The appellant was not satisfied with the findings
of this Court of Inquiry and hence on 22.4.2004 he made a representation to the then
Chief of the Army Staff, General N.C. Vij, as a result of which another Court of Inquiry
was held. However, the second Court of Inquiry came to the same conclusion as that of
the first inquiry namely, that it was a case of suicide.
8. Aggrieved, a writ petition was filed in the High Court which was dismissed by the
impugned judgment. Hence this appeal.
9. The petitioner (appellant herein) prayed in the writ petition that the matter be ordered
to be investigated by the Central Bureau of Investigation (in short 'CBI'). Since his prayer
was rejected by the High Court, hence this appeal by way of special leave.
10

. It has been held by this Court in CBI and another vs. Rajesh Gandhi and another, 1997
Cri.LJ 63 (vide para 8) that no one can insist that an offence be investigated by a
particular agency. We fully agree with the view in the aforesaid decision. An aggrieved
person can only claim that the offence he alleges be investigated properly, but he has no
right to claim that it be investigated by any particular agency of his choice. 1996
AIR SCW 4249

11. In this connection we would like to state that if a person has a grievance that the
police station is not registering his FIR under Section 154, Cr.P.C., then he can approach
the Superintendent of Police under Section 154(3), Cr.P.C. by an application in writing.
Even if that does not yield any satisfactory result in the sense that either the FIR is still
not registered, or that even after registering it no proper investigation is held, it is open to
the aggrieved person to file an application under Section 156 (3), Cr.P.C. before the
learned Magistrate concerned. If such an application under Section 156 (3) is filed before
the Magistrate, the Magistrate can direct the FIR to be registered and also can direct a
proper investigation to be made, in a case where, according to the aggrieved person, no
proper investigation was made. The Magistrate can also under the same provision
monitor the investigation to ensure a proper investigation.
12

. Thus in Mohd. Yousuf vs. Smt. Afaq Jahan and Anr., JT 2006 (1) SC 10, this Court
observed : 2006 AIR SCW 95, (Para 11)

"The clear position therefore is that any Judicial Magistrate, before taking cognizance of
the offence, can order investigation under Section 156(3) of the Code. If he does so, he is
not to examine the complainant on oath because he was not taking cognizance of any
offence therein. For the purpose of enabling the police to start investigation it is open to
the Magistrate to direct the police to register an FIR. There is nothing illegal in doing so.
After all registration of an FIR involves only the process of entering the substance of the
information relating to the commission of the cognizable offence in a book kept by the
officer-in-charge of the police station as indicated in Section 154 of the
@page-SC909
Code. Even if a Magistrate does not say in so many words while directing investigating
under Section 156(3) of the Code that an FIR should be registered, it is the duty of the
officer-in-charge of the police station to register the FIR regarding the cognizable offence
disclosed by the complaint because that police officer could take further steps
contemplated in Chapter XII of the Code only thereafter."
13

. The same view was taken by this Court in Dilawar Singh vs. State of Delhi, JT 2007
(10) SC 585 (vide para 17). We would further clarify that even if an FIR has been
registered and even if the police has made the investigation, or is actually making the
investigation, which the aggrieved person feels is not proper, such a person can approach
the Magistrate under Section 156(3), Cr.P.C., and if the Magistrate is satisfied he can
order a proper investigation and take other suitable steps and pass such order as he thinks
necessary for ensuring a proper investigation. All these powers a Magistrate enjoys under
Section 156(3), Cr.P.C. 2007 AIR SCW 5899

14. Section 156 (3) states:


"Any Magistrate empowered under Section 190 may order such an investigation as
abovementioned."
The words 'as abovementioned' obviously refer to Section 156 (1), which contemplates
investigation by the officer in charge of the Police Station.
15. Section 156(3) provides for a check by the Magistrate on the police performing its
duties under Chapter XII, Cr.P.C. In cases where the Magistrate finds that the police has
not done its duty of investigating the case at all, or has not done it satisfactorily, he can
issue a direction to the police to do the investigation properly, and can monitor the same.
16. The power in the Magistrate to order further investigation under Section 156(3) is an
independent power, and does not affect the power of the investigating officer to further
investigate the case even after submission of his report vide Section 173(8). Hence the
Magistrate can order re-opening of the investigation even after the police submits the
final report, vide State of Bihar vs. A.C. Saldanna, AIR 1980 SC 326 (para 19).
17. In our opinion Section 156(3), Cr.P.C. is wide enough to include all such powers in a
Magistrate which are necessary for ensuring a proper investigation, and it includes the
power to order registration of an F.I.R. and of ordering a proper investigation if the
Magistrate is satisfied that a proper investigation has not been done, or is not being done
by the police. Section 156(3), Cr.P.C., though briefly worded, in our opinion, is very wide
and it will include all such incidental powers as are necessary for ensuring a proper
investigation.
18. It is well-settled that when a power is given to an authority to do something it
includes such incidental or implied powers which would ensure the proper doing of that
thing. In other words, when any power is expressly granted by the statute, there is
impliedly included in the grant, even without special mention, every power and every
control the denial of which would render the grant itself ineffective. Thus where an Act
confers jurisdiction it impliedly also grants the power of doing all such acts or employ
such means as are essentially necessary to its execution.
19. The reason for the rule (doctrine of implied power) is quite apparent. Many matters of
minor details are omitted from legislation. As Crawford observes in his Statutory
Construction (3rd Edn. page 267) :-
"If these details could not be inserted by implication, the drafting of legislation would be
an indeterminable process and the legislative intent would likely be defeated by a most
insignificant omission."
20. In ascertaining a necessary implication, the Court simply determines the legislative
will and makes it effective. What is necessarily implied is as much part of the statute as if
it were specifically written therein.
21. An express grant of statutory powers carries with it by necessary implication the
authority to use all reasonable means to make such grant effective. Thus in ITO,
Cannanore vs. M.K. Mohammad Kunhi, AIR 1969 SC 430, this Court held that the
Income-tax Appellate Tribunal has implied powers to grant stay, although no such power
has been expressly granted to it by the Income-tax Act.
22

. Similar examples where this Court has affirmed the doctrine of implied powers are
Union of India vs. Paras Laminates, AIR 1991 SC 696; Reserve Bank of India vs.
Peerless General Finance and Investment Company Ltd., AIR 1996 SC 646 (at p. 656);
Chief 1996 AIR SCW 384
1996 AIR SCW 3682

@page-SC910
Executive Officer and Vice-Chairman, Gujarat Maritime Board vs. Haji Daud Haji
Harun Abu, 1996 (11) SCC 23; J. K. Synthetics Ltd. vs. Collector of Central Excise, AIR
1996 SC 3527; State of Karnataka vs. Vishwabharati House Building Co-op Society,
2003 (2) SCC 412 (at p. 432) etc.
23. In Savitri vs. Govind Singh Rawat, AIR 1986 SC 984, this Court held that the power
conferred on the Magistrate under Section 125, Cr.P.C. to grant maintenance to the wife
implies the power to grant interim maintenance during the pendency of the proceeding,
otherwise she may starve during this period.
24. In view of the abovementioned legal position, we are of the view that although
Section 156(3) is very briefly worded, there is an implied power in the Magistrate under
Section 156(3), Cr.P.C. to order registration of a criminal offence and/or to direct the
officer-in-charge of the concerned police station to hold a proper investigation and take
all such necessary steps that may be necessary for ensuring a proper investigation
including monitoring the same. Even though these powers have not been expressly
mentioned in Section 156(3), Cr.P.C., we are of the opinion that they are implied in the
above provision.
25. We have elaborated on the above matter because we often find that when someone
has a grievance that his FIR has not been registered at the police station and/or a proper
investigation is not being done by the police, he rushes to the High Court to file a writ
petition or a petition under Section 482, Cr.P.C. We are of the opinion that the High Court
should not encourage this practice and should ordinarily refuse to interfere in such
matters, and relegate the petitioner to his alternating remedy, firstly under Section 154(3)
and Section 36, Cr.P.C. before the concerned police officers, and if that is of no avail, by
approaching the concerned Magistrate under Section 156(3).
26. If a person has a grievance that his FIR has not been registered by the police station
his first remedy is to approach the Superintendent of Police under Section 154(3), Cr.P.C.
or other police officer referred to in Section 36, Cr.P.C. If despite approaching the
Superintendent of Police or the officer referred to in Section 36 his grievance still
persists, then he can approach a Magistrate under Section 156(3), Cr.P.C. instead of
rushing to the High Court by way of a writ petition or a petition under Section 482,
Cr.P.C. Moreover he has a further remedy of filing a criminal complaint under Section
200, Cr.P.C. Why then should writ petitions or Section 482 petitions be entertained when
there are so many alternative remedies?
27. As we have already observed above, the Magistrate has very wide powers to direct
registration of an FIR and to ensure a proper investigation, and for this purpose he can
monitor the investigation to ensure that the investigation is done properly (though he
cannot investigate himself). The High Court should discourage the practice of filing a
writ petition or petition under Section 482, Cr.P.C. simply because a person has a
grievance that his FIR has not been registered by the police, or after being registered,
proper investigation has not been done by the police. For this grievance, the remedy lies
under Sections 36 and 154(3) before the concerned police officers, and if that is of no
avail, under Section 156(3), Cr.P.C. before the Magistrate or by filing a criminal
complaint under Section 200, Cr.P.C. and not by filing a writ petition or a petition under
Section 482, Cr.P.C.
28. It is true that alternative remedy is not an absolute bar to a writ petition, but it is
equally well settled that if there is an alternative remedy the High Court should not
ordinarily interfere.
29

. In Union of India vs. Prakash P. Hinduja and another, 2003 (6) SCC 195 (vide para 13),
it has been observed by this Court that a Magistrate cannot interfere with the
investigation by the police. However, in our opinion, the ratio of this decision would only
apply when a proper investigation is being done by the police. If the Magistrate on an
application under Section 156(3), Cr.P.C. is satisfied that proper investigation has not
been done, or is not being done by the officer-in-charge of the concerned police station,
he can certainly direct the officer- in-charge of the police station to make a proper
investigation and can further monitor the same (though he should not himself
investigate). 2003 AIR SCW 3258

30

. It may be further mentioned that in view of Section 36, Cr.P.C. if a person is aggrieved
that a proper investigation has not been made by the officer-in-charge of the concerned
police station, such aggrieved person can approach the Superintendent of 2001 AIR
SCW 305
AIR 1980 SC 326
@page-SC911
Police or other police officer superior in rank to the officer-in-charge of the police station
and such superior officer can, if he so wishes, do the investigation vide CBI vs. State of
Rajasthan and another, 2001 (3) SCC 333 (vide para 11); R. P. Kapur vs. S.P. Singh, AIR
1961 SC 1117 etc. Also, the State Government is competent to direct the Inspector
General, Vigilance to take over the investigation of a cognizable offence registered at a
police station vide State of Bihar vs. A.C. Saldanna (supra).
31. No doubt the Magistrate cannot order investigation by the CBI vide CBI vs. State of
Rajasthan and another (supra), but this Court or the High Court has power under Article
136 or Article 226 to order investigation by the CBI. That, however should be done only
in some rare and exceptional case, otherwise, the CBI would be flooded with a large
number of cases and would find it impossible to properly investigate all of them.
32. In the present case, there was an investigation by the G.R.P., Mathura and also two
Courts of Inquiry held by the Army Authorities and they found that it was a case of
suicide. Hence, in our opinion, the High Court was justified in rejecting the prayer for a
CBI inquiry.
33

. In Secretary, Minor Irrigation and Rural Engineering Services, U.P. and others vs.
Sahngoo Ram Arya and another, 2002 (5) SCC 521 (vide para 6), this Court observed that
although the High Court has power to order a CBI inquiry, that power should only be
exercised if the High Court after considering the material on record comes to a
conclusion that such material discloses prima facie a case calling for investigation by the
CBI or by any other similar agency. A CBI inquiry cannot be ordered as a matter of
routine or merely because the party makes some allegation. 2002 AIR SCW 2333

34. In the present case, we are of the opinion that the material on record does not disclose
a prima facie case calling for an investigation by the CBI. The mere allegation of the
appellant that his son was murdered because he had discovered some corruption cannot,
in our opinion, justify a CBI inquiry, particularly when inquiries were held by the Army
authorities as well as by the G.R.P. at Mathura, which revealed that it was a case of
suicide.
35. It has been stated in the impugned order of the High Court that the G.R.P. at Mathura
had investigated the matter and gave a detailed report on 29.8.2003. It is not clear
whether this report was accepted by the Magistrate or not. If the report has been accepted
by the Magistrate and no appeal/revision was filed against the order of the learned
Magistrate accepting the police report, then that is the end of the matter. However, if the
Magistrate has not yet passed any order on the police report, he may do so in accordance
with law and in the light of the observations made above.
36. With the above observations, this appeal stands dismissed.
37. Let a copy of this judgment be sent by the Secretary General of this Court to the
Registrar Generals/Registrars of all the High Courts, who shall circulate a copy of this
Judgment to all the Honble Judges of the High Courts.
Appeal dismissed.
AIR 2008 SUPREME COURT 911 "Bogidhola Tea and Trading Co. Ltd. v. Hira Lal
Somani"
(From : Gauhati)*
Coram : 2 S. B. SINHA AND G. S. SINGHVI, JJ.
Civil Appeal No. 5771 of 2007 (arising out of SLP (C) No. 6404 of 2007), D/- 7 -12
-2007.
Bogidhola Tea and Trading Co. Ltd. and Anr. v. Hira Lal Somani.
Civil P.C. (5 of 1908), O.8, R.10, O.7, R.11 - PLAINT - DECREE - Written statement not
filed - Ex parte decree - Not to be passed unless averments made in plaint are established
- Decree also passed without considering point of limitation - Liable to be set aside.
(Para 12)

Vijay Hansaria, Sr. Advocate, Ms. Sneha Kalita and Shankar Divate, with him for
Appellants; Manish Goswami, Ashok Panigrahi (for M/s. Map and Co.), for
Respondent.
* M. C. No. 3398 of 2004 in R. F. A. No. 122 of 2004, D/- 3-1-2007 (Gau)).
Judgement
S. B. SINHA, J. :-Leave granted.
2.This appeal is directed against the judgment and order dated 3.1.2007 passed by the
Gauhati High Court in MC No.3398/2004 whereby and whereunder the appeal preferred
by the appellants herein from a judgment and decree dated 19.4.1990
@page-SC912
passed in Suit No.2/89, was dismissed on the premise that the appellants had not shown
sufficient cause for condonation of 10 months' delay in filing the said appeal.
3. The parties herein were on business terms. Appellants were to supply 22,000 Kgs. of
'made tea' for 1984 season and 50,000 Kgs. of 'made tea' for 1985 season to the
respondent. However, the appellants supplied only 5,547 Kgs. of 'made tea' for 1984
season and 18.245 Kgs. of 'made tea' for 1985 season. Respondent filed a suit for a decree
for a sum of Rs.5,22,69.66 paise together with interest thereon at the rate of 18% per
annum. A suit was filed towards the price of the remaining amount for terminal tea
supply. In paragraph 5 of the plaint, the respondent inter alia averred:
"The price for other remaining quantity of made tea of 1985 season made available by the
defendants to the plaintiff, could not be finalized as the same were found to have no
proper market and hence were not saleable at Jorhat. At such as per instruction/discussion
of the defendants, remaining qualities of 14,796 of made tea of 1985 season were sent to
the tea auction market at Guwahati and in Calcutta. The sale proceeds of the said tea on
sale of auction markets were to be adjusted with the advances already made by the
plaintiff to the deponents. After 18.04.85 the plaintiff had paid a total sum of Rs. 6,22,116
inclusive of Rs. 1,30,000 as shown in schedule 'A' below.
4. The first Bill referred to in this appeal reads thus :

"Bill dated 5.6.85 for..... Rs. 46,594.80


Bill dated 5.6.85 for...... Rs. 86,225.00
For sale proceeds on 16.8.85 Rs. 79,824.91
For sale proceeds on 26.8.85 Rs. 4,608.60
For sale proceeds on 9.9.85 Rs. 9,101.83
For sale proceeds on 19.9.85 Rs. 3766.70
For sale proceeds on 12.11.85Rs. 2502.54
For sale proceeds on 9.12.85 Rs. 30,615.48
For sale proceeds on 23.12.85Rs. 30,9119.62
For sale proceeds on 3.1.86 Rs. 5,945.78
For sale proceeds on 20.1.86 Rs. 9,784.28
Rs. 3,18,089.54

5. Allegedly, despite service of summons the appellants did not appear. The plaintiff-
respondent made a prayer before the Trial Court that a decree be passed under order 8
Rule 10 of the CPC. He declined to examine any witness. The learned Trial Court by a
judgment and order dated 19.4.1990 decreed the suit stating :
"Learned lawyer of the plaintiff side is present filing his hazira. Defendants side is absent
without any step. Seen the previous orders of this Court. The suit is taken up for ex-parte
hearing. Heard learned counsel for the plaintiff. He submits Court to take action under
Order VIII Rule 10 of the C.P.C. and declined to examine any P.W. Hence, perused the
plaint and the relevant documents submitted by the plaintiff in support of his plaint.
Prima facie case is held proved in favour of the plaintiff as per plaint.
The suit is decreed on ex-parte for realization of Rs.5,22,669.66 p with costs of the suit
and future interest per plaint as prayed for."
6. Appellants contended that they were not aware of passing of the said decree. In the
year 1997, an execution case was filed. Summons in the said execution case were served
upon the appellants. One Shri Tapan Gogoi was appointed as an Advocate in the said
execution case. However, no further steps were taken. The execution proceeding was
stayed on 15.7.2000.
7. In the month of September, 2000, the appellants herein filed an application under
Order 9 Rule 13 of CPC for setting aside the said ex-parte decree. An application for
condonation of delay in filing the suit was also filed. The said application was dismissed
by an order dated 22.9.2003 as the appellants could not allegedly satisfactorily explain
the cause for delay in filing the said application as also in view of Article 123 of the
Schedule appended to the Limitation Act, 1923.
8. A civil revision application was preferred thereagainst which was also dismissed by the
High Court by its order dated 2.1.2004. Liberty, however, was granted to the appellants to
prefer an appeal against the original decree. An appeal thereafter was filed by the
appellants along with an application for condonation of delay. The High Court by reason
of the impugned judgment refused to condone the delay and consequently dismissed the
appeal.
9. Mr. Hansaria, learned senior counsel appearing on behalf of the appellants, inter alia,
would submit that the High Court committed a serious error in passing the impugned
judgment in so far as it failed to take into consideration that assuming that the defendant-
appellants had not filed written statement, it was obligatory on the part of
@page-SC913
the Trial Judge to satisfy itself about the bona fide of the claim of the plaintiff-
respondent. Learned counsel urged that having regard to the fact that the last advance was
purported to have been made on 19.6.1985, the suit which was filed on 2.1.1989 was
barred by limitation.
10. Learned counsel appearing on behalf of the respondent, on the other hand, submitted
that assuming that the learned Trial Judge should not have granted a decree in terms of
Order 8 Rule 10 of the CPC, the appellants were obligated to explain the delay in
preferring an appeal. The appeal being continuation of the suit, the learned counsel would
submit that if the same could not have been entertained on the ground of being barred by
limitation, the question of setting aside the decree by the High Court in exercise of its
appellate jurisdiction did not and could arise.
11. While issuing notice in terms of order dated 16.04.2007, this Court directed the
appellants to deposit a sum of Rs. 2 lakhs before the Executing Court within four weeks
from the said date. It is stated before us by learned senior counsel Mr. Hansaria, that the
aforementioned sum has been deposited on or about 25.6.2007.
12. Ordinarily, we would not have interfered in such matter. However, it appears to be a
gross case. Appellants before us have been able to show that the ex-parte decree dated
19.4.1990 passed by the learned Additional District and Sessions Judge, Jorhat, ex-facie
suffers from non-application of mind. Had the learned Judge applied its mind even to the
averments made in the plaint, he should have asked himself the question as to whether in
absence of any acknowledgment in writing, as a result whereof the period of limitation
would start running afresh, the suit could have been decreed. Section 3 of the Limitation
Act, 1963 mandates that a Court would not exercise its jurisdiction for any relief in
favour of a party if the same is found to be barred by limitation. Although such a defence
has not been raised, the statute obligated upon the Court of law to consider as to whether
a suit is barred by limitation or not. In the event it was found that the suit was barred by
limitation, the Court had no jurisdiction to pass a decree. It was, therefore, essential for
the learned Trial Judge to pose unto itself the right question, particularly when without
adduction of oral evidence the pleading raised in the plaint could not be said to have been
established. It was, therefore, not a case where the Court could have invoked the
provisions of Order 10 Rule 8 of the CPC. Even otherwise, the suit was set down for ex-
parte hearing. The learned Trial Judge stated that only a prima-facie case was found out
from the plaint and other documents which were not sufficient for passing a decree as
therefor the plaintiff was bound to prove his case.
13. For the reasons aforementioned, having regard to the peculiar facts and circumstances
of this case, we think that it is a fit case where the High Court should have condoned the
delay. We, therefore, set aside the judgment of the High Court. Ordinarily, we would have
remitted the matter back to the High Court for consideration thereof on merit of the
appeal, but as we have ourselves looked to the records of the case, we are of the opinion
that interest of justice would be subserved if we set aside the ex-parte decree dated
19.4.1990. We direct accordingly.
14. This order shall, however, be subject to the condition that the appellants shall deposit
a further sum of Rs.1 lakh before the Executing Court which shall be subject to the
outcome of the suit. Appellants shall further pay a sum of Rs. 25,000/- to the respondent
towards costs. The respondent shall be entitled to withdraw the sum deposited by the
appellants, upon furnishing security.
15. Appellants may file written statements before the Trial Court within six weeks and the
learned Trial Judge may consider the desirability of disposing of the suit within three
months from the date of receipt of this order.
16. The appeal is disposed of with the aforementioned observations and direction.
Order accordingly.
AIR 2008 SUPREME COURT 913 "Holicow Pictures Pvt. Ltd., M/s. v. Prem Chandra
Mishra"
(From : 2007 (1) Pat LJR 372)
Coram : 2 Dr. A. PASAYAT AND P. SATHASIVAM, JJ.
Civil Appeal No. 5671 of 2007 (arising out of SLP (C) No. 187 of 2007), D/- 6 -12 -2007.
M/s. Holicow Pictures Pvt. Ltd. v. Prem Chandra Mishra and Ors.
(A) Constitution of India, Art.32, Art.226 - WRITS - PUBLIC INTEREST LITIGATION
- Public interest litigation - Courts are
@page-SC914
flooded with large number of PIL - It is desirable for Courts to filter out frivolous
petitions and dismiss them with costs.
Public interest litigation is a weapon which has to be used with great care and
circumspection and the judiciary has to be extremely careful to see that behind the
beautiful veil of public interest an ugly private malice, vested interest and /or publicity
seeking is not lurking. It is to be used as an effective weapon in the armory of law for
delivering social justice to the citizens. The attractive brand name of public interest
litigation should not be used for suspicious products of mischief. It should be aimed at
redressal of genuine public wrong or public injury and not publicity oriented or founded
on personal vendetta. The Court must be careful to see that a body of persons or member
of public, who approaches the Court is acting bona fide and not for personal gain or
private motive or political motivation or other oblique considerations. The Court must not
allow its process to be abused for oblique considerations by masked phantoms who
monitor at times from behind. Some persons with vested interest indulge in the pastime
of meddling with judicial process either by force of habit or from improper motives, and
try to bargain for a good deal as well to enrich themselves. Often they are actuated by a
desire to win notoriety or cheap popularity. The petitions of such busy bodies deserve to
be thrown out by rejection at the threshold, and in appropriate cases with exemplary
costs. (Para 18)
A time has come to weed out the petitions, which though titled as public interest
litigations are in essence something else. The Courts are flooded with large number of so
called public interest litigations where even a minuscule percentage can legitimately be
called as public interest litigations. Though the parameters of public interest litigation
have been indicated by Supreme Court in large number of cases, yet unmindful of the real
intentions and objectives, Courts are entertaining such petitions and wasting valuable
judicial time which, could be otherwise utilized for disposal of genuine cases. It is also
noticed that petitions are based on newspaper reports without any attempt to verify their
authenticity. As observed by the Supreme Court in several cases newspaper reports do not
constitute evidence. A petition based on unconfirmed news reports, without verifying
their authenticity should not normally be entertained. Such petitions do not provide any
basis for verifying the correctness of statements made and information given in the
petition. It would be desirable for the Courts to filter out the frivolous petitions and
dismiss them with costs so that the message goes in the right direction that petitions filed
with oblique motive do not have the approval of the Courts.(Para 22)
Public Interest Litigation which has now come to occupy an important field in the
administration of law should not be "publicity interest litigation" or "private interest
litigation" or "politics interest litigation" or the latest trend "paise income litigation."
(Para 10)
(B) Constitution of India, Art.226 - WRITS - PUBLIC INTEREST LITIGATION -
ALLOTMENT OF PREMISES - HIGH COURT - Public interest litigation - Against
allotment of land by State Govt. at throw away prices for political considerations -
Allotments set aside by High Court - High Court's conclusions were drawn after going
through files - However, High Court did not ask the parties to clarify any doubt it
entertained as regards certain crucial aspects - Said aspects assume considerable
importance as they formed foundation of High Court's conclusions about
irregularity/illegality in allotment - Matter remitted for fresh consideration. (Paras
27, 28)
Cases Referred : Chronological Paras
2005 AIR SCW 46 : AIR 2005 SC 540 : 2005 AIR Jhar HCR 532 (Ref) 25
2004 AIR SCW 1494 : AIR 2004 SC 1923 (Ref) 25
2003 AIR SCW 6105 : AIR 2004 SC 280 (Ref) 25
1996 AIR SCW 3356 : AIR 1996 SC 2687 : 1996 Cri LJ 3983 (Ref) 21
1994 AIR SCW 1953 : AIR 1994 SC 2151 (Ref) 21
1994 AIR SCW 2190 (Ref) 10
(1994) 2 SCC 481 (Ref) 21
(1994) 6 SCC 620 (Ref) 10
1993 AIR SCW 248 : AIR 1993 SC 892 : 1993 Cri LJ 600 (Ref) 10, 13
1992 AIR SCW 3460 : AIR 1993 SC 852 (Ref) 10
AIR 1985 SC 910 (Ref) 24
AIR 1982 SC 149 (Ref) 23
P. S. Patwalia, Sr. Advocate, Mohd. Guzail Khan, Anil Kumar Jha, Braj Kishore Mishra,
@page-SC915
Ms. Aparna Jha, Tanushree Sinha, Abhishek Yadav, Vikrant, U. K. Jha and Gopal Singh,
for Respondents.
Judgement
1. Dr. ARIJIT PASAYAT, J. :-Leave granted.
2. Challenge in this appeal is to the order passed by a Division Bench of the Patna High
Court disposing the writ petition filed by the respondent No.1 styled as a "Public Interest
Litigation". The order gave certain directions and nullifying certain allotments of land
made in favour of respondent No. 5. In the writ petition action of the State Government
of Bihar in granting appellant through its Director Prakash Jha land pieces in the
Industrial areas in Patna, Hajipur, Muzaffarpur, Sitamarhi and Buxar. The writ petitioner
alleged that the said Prakash Jha, Director of present appellant who was respondent No. 5
in the writ petition was given land in return of services rendered by him to help the
present Chief Minister to win last assembly elections. It was alleged that the action of the
Government amounted to doling out valuable State property as largess at throw away
prices for political considerations.
3. The writ petition was filed, and was claimed to be, in public interest. The respondent
described himself as the Chief Spokesperson of the Indian National Congress, a
recognizd political party. His party contested the previous election in alliance with the
party that was in power at that time. The alliance got worsted in the election and
Congress party was returned as a poor fourth. Respondent No. 5 is a private limited
company; it is represented through a person who is well known as a film maker but who
also takes part in electoral politics. In the supplementary affidavit filed by the petitioner,
it was stated that Prakash Jha had fought the 2004 Lok Sabha Election from the Bettiah
Lok Sabha Constituency. It is further stated that in the last assembly election held in
October-November, 2005 though not a candidate himself, he addressed public meetings
jointly with the present Chief Minister in various parts of the State. As both the petitioner
and said Prakash Jha are political persons, it is not surprising that the pleadings are heavy
with political invectives.
4. In the writ petition, it was stated that the Bihar Industrial Area Development Authority
(hereinafter referred to as "the Authority") has given large areas of land at various places
to respondent No. 5 at throwaway prices. It was further stated that allotments of land
were made to respondent No. 5 without issuing any notice, inviting competitive bids, and
the land areas were granted to respondent No. 5 at very cheap rates without following the
established norms and procedure. According to him the market value of the said land
plots was much higher. For instance, in Patliputra Industrial Area, Patna one acre land
was given to respondent No. 5 for Rs. 14,65,000.00. This piece of land should have
fetched the State about rupees five crores if allotments were to be made on the basis of
competitive bidding. It was also stated that in a blatant show of favour, respondent No. 5
was also given the Authority's Office building along with the land in Patliputra Industrial
Area. The favour was crowned by putting a very low value for the building. It was also
alleged that the action of the State Government/Authority in granting to respondent No. 5
land at different places was an act of mala fide and called for institution of criminal cases
against the concerned people and an investigation by the Central Bureau of Investigation.
5. The writ petition, in brief, made three allegations. (i) land plots were given to
respondent No. 5 at different places in the State without issuing any notice, inviting bids
and at value far below the market price of those land plots, (ii) the allotment was made
without following any norms or procedure and (iii) the action of the State Government
caused heavy loss to the State; the action was based on political considerations and was,
therefore, mala fide.
6. The respondents in the writ petition questioned bona fides of the writ petitioner. They
took the stand that there was nothing illegal and the entire action was bona fide in the
greater interests of the State. It was pointed out that the mala fides of the writ petitioner
are clear from the fact that the writ petitioner did not question legality of the action of the
then State Government who had in the year 1996 allotted the land in the Patliputra
Industrial areas to one M/s. Dynax Digital Studio (Ind) Pvt. Ltd. for consideration of Rs.
5.5 Lakhs. The allotment made was subsequently cancelled and the consideration for
allotment was fixed by raising the land value at the rate of 10% only as provided in Govt.
letter dated 17-7-1982. It was also submitted that the petition was an abuse of Public
Interest Litigation and
@page-SC916
deserves to be rejected outright.
7. After referring to the various stands, ultimately, the High Court came to observe that
there was more than what met the eye and the allotment of land plots to respondent No. 5
was done in a thoroughly irregular manner and the allotments are completely untenable.
So far as the charge of mala fides is concerned the State's submission was accepted by
giving "benefit of doubt". It was, however, observed that the authorities were in a hurry to
go for private investment and that might have led to non-observance to some of the
official norms was held to be completely unacceptable. The writ petition was disposed of
with following observations and directions.
"In the facts and circumstances discussed above, I reject the allegation of mala fide. But
at the same time, I am unable to accept the submission that the Court should not interfere
in the matter because the writ petition may not qualify as Public Interest Litigation. In
view of the facts coming to its notice, the only proper course for the Court is to intervene
and to set things right. I therefore, feel constrained to interfere with the allotments made
by the Authority in favour of respondent No. 5. All the allotments of lands made in favour
of respondent No. 5 are accordingly quashed. The Authority is directed to resume
possession of the lands. It will be open to respondent No. 5 to make fresh applications
with proper Project Reports and supporting documents for allotment of lands to it at
different centres. In case such applications made, the authority shall consider them in
accordance with law and take a decision on those applications within three months of the
date of their receipt in its office. It will be open to respondent No. 5 either to get back all
its money deposited with the Authority or in case it makes fresh applications to wait till
the final decision on those applications is taken by the Authority.
This writ petition is disposed of with the aforesaid observations and directions. There will
be no order as to costs."
8. In support of the appeal, learned counsel for the appellant submitted that there was
complete violation of the principles of natural justice in the instant case. The High Court's
observations were contrary to the materials on record. The High Court appears to have
based its conclusions on a reading of the files without any opportunity to either the
authorities or the State Government and in any event not to the present appellant to
explain the fact situation. If the High Court had any doubt about any aspect which
according to it was relevant, opportunity in that regard should have been given.
Unfortunately that has not been done. It is pointed out that the High Court has rightly
rejected the stand of the writ petitioner about mala fides. That was sufficient to throw out
the writ petition at the threshold. Instead of that the High Court referred to the records
and came to conclusions finding alleged discrepancies without grant of opportunity. The
conclusions are contrary to the materials available and in any event the High Court ought
not to have relied solely on the counter affidavits filed which were in fact replies to the
averments made in the writ petition. Most of the conclusions of the High Court related to
aspects which were not even pleaded in the writ petition. That being so, there was no
scope for the respondents in the writ petition to throw any light on aspects which
ultimately were taken note of by the High Court. It is pointed out that the writ petitioner
himself accepted that he was a functionary of a political party. The petition is nothing but
political vendata unleashed.
9. Learned counsel for the State of Bihar and the Authority supported the stand taken by
the appellant. On the contrary the respondent No. 1 writ petitioner submitted that merely
because the High Court has given the "benefit of doubt" to the State Government, it could
not have closed eyes to the apparent illegalities which a bare perusal of the records
revealed. The State and the Authorities, it is submitted, filed affidavits taking contrary
stands. Stand which was stated in one affidavit was subsequently departed from.
10
. When there is material to show that a petition styled as a public interest litigation is
nothing but a camouflage to foster personal disputes, the said petition is to be thrown out.
Before we grapple with the issue involved in the present case, we feel it necessary to
consider the issue regarding public interest aspect. Public Interest Litigation which has
now come to occupy an important field in the administration of law should not be
"publicity interest litigation" or "private interest litigation" or "politics interest litigation"
or the latest trend "paise income litigation". If not properly regulated 1993 AIR
SCW 248
1994 AIR SCW 2190
1992 AIR SCW 3460

@page-SC917
and abuse averted, it becomes also a tool in unscrupulous hands to release vendetta and
wreck vengeance, as well. There must be real and genuine public interest involved in the
litigation and not merely an adventure of knight errant borne out of wishful thinking. It
cannot also be invoked by a person or a body of persons to further his or their personal
causes or satisfy his or their personal grudge and enmity. Courts of justice should not be
allowed to be polluted by unscrupulous litigants by resorting to the extraordinary
jurisdiction. A person acting bona fide and having sufficient interest in the proceeding of
public interest litigation will alone have a locus standi and can approach the Court to
wipe out violation of fundamental rights and genuine infraction of statutory provisions,
but not for personal gain or private profit or political motive or any oblique consideration.
These aspects were highlighted by this Court in The Janta Dal v. H. S. Chowdhary (1992
(4) SCC 305) and Kazi Lhendup Dorji v. Central Bureau of Investigation, (1994 Supp (2)
SCC 116). A writ petitioner who comes to the Court for relief in public interest must
come not only with clean hands like any other writ petitioner but also with a clean heart,
clean mind and clean objective. (See Ramjas Foundation v. Union of India, (AIR 1993
SC 852) and K. R. Srinivas v. R. M. Premchand, (1994 (6) SCC 620).
11. It is necessary to take note of the meaning of expression 'public interest litigation'. In
Stroud's Judicial Dictionary, Volume 4 (IV Edition), 'Public Interest' is defined thus :
"Public Interest (1) a matter of public or general interest does not mean that which is
interesting as gratifying curiosity or a love of information or amusement but that in which
a class of the community have a pecuniary interest, or some interest by which their legal
rights or liabilities are affected."
12. In Black's Law Dictionary (Sixth Edition), "public interest" is defined as follows :
"Public Interest something in which the public, or some interest by which their legal
rights or liabilities are affected. It does not mean anything the particular localities, which
may be affected by the matters in question. Interest shared by national government.........."
13

. In Janata Dal case (supra) this Court considered the scope of public interest litigation. In
para 52 of the said judgment, after considering what is public interest, has laid down as
follows : 1993 AIR SCW 248

"The expression 'litigation' means a legal action including all proceedings therein
initiated in a Court of law for the enforcement of right or seeking a remedy. Therefore,
lexically the expression "PIL" means the legal action initiated in a Court of law for the
enforcement of public interest or general interest in which the public or a class of the
community have pecuniary interest or some interest by which their legal rights or
liabilities are affected."
14. In paras 60, 61 and 62 of the said judgment, it was pointed out as follows :
"Be that as it may, it is needless to emphasis that the requirement of locus standi of a
party to a litigation is mandatory, because the legal capacity of the party to any litigation
whether in private or public action in relation to any specific remedy sought for has to
primarily ascertained at the threshold".
15. In para 96 of the said judgment, it has further been pointed out as follows :
"While this Court has laid down a chain of notable decisions with all emphasis at their
command about the importance and significance of this newly developed doctrine of PIL,
it has also hastened to sound a red alert and a note of severe warning that Courts should
not allow its process to be abused by a mere busy body or a meddlesome interloper or
wayfarer or officious intervener without any interest or concern except for personal gain
or private profit or other oblique consideration."
16. In subsequent paras of the said judgment, it was observed as follows :
"It is thus clear that only a person acting bona fide and having sufficient interest in the
proceeding of PIL will alone have as locus standi and can approach the Court to wipe out
the tears of the poor and needy, suffering from violation of their fundamental rights, but
not a person for personal gain or private profit or political motive or any oblique
consideration. Similarly a vexatious petition under the colour of PIL, brought before the
Court for vindicating any personal grievance, deserves rejection at the threshold".
@page-SC918
17. It is depressing to note that on account of such trumpery proceedings initiated before
the Courts, innumerable days are wasted, the time which otherwise could have been spent
for disposal of cases of the genuine litigants. Though we spare no efforts in fostering and
developing the laudable concept of PIL and extending our long arm of sympathy to the
poor, the ignorant, the oppressed and the needy, whose fundamental rights are infringed
and violated and whose grievances go unnoticed, un-represented and unheard; yet we
cannot avoid but express our opinion that while genuine litigants with legitimate
grievances relating to civil matters involving properties worth hundreds of millions of
rupees and criminal cases in which persons sentenced to death facing gallows under
untold agony and persons sentenced to life imprisonment and kept in incarceration for
long years, persons suffering from undue delay in service matters-goverment or private,
persons awaiting the disposal of cases wherein huge amounts of public revenue or
unauthorized collection of tax amounts are locked up, detenu expecting their release from
the detention orders etc. etc. are all standing in a long serpentine queue for years with the
fond hope of getting into the Courts and having their grievances redressed, the
busybodies, meddlesome interlopers, wayfarers or officious interveners having absolutely
no public interest except for personal gain or private profit either of themselves or as a
proxy of others or for any other extraneous motivation or for glare of publicity break the
queue muffing their faces by wearing the mask of public interest litigation and get into
the Courts by filing vexatious and frivolous petitions and thus criminally waste the
valuable time of the Courts and as a result of which the queue standing outside the doors
of the Courts never moves, which piquant situation creates frustration in the minds of the
genuine litigants and resultantly they loose faith in the administration of our judicial
system.
18. Public interest litigation is a weapon which has to be used with great care and
circumspection and the judiciary has to be extremely careful to see that behind the
beautiful veil of public interest an ugly private malice, vested interest and /or publicity
seeking is not lurking. It is to be used as an effective weapon in the armory of law for
delivering social justice to the citizens. The attractive brand name of public interest
litigation should not be used for suspicious products of mischief. It should be aimed at
redressal of genuine public wrong or public injury and not publicity oriented or founded
on personal vendetta. As indicated above, Court must be careful to see that a body of
persons or member of public, who approaches the Court is acting bona fide and not for
personal gain or private motive or political motivation or other oblique considerations.
the Court must not allow its process to be abused for oblique considerations by masked
phantoms who monitor at times from behind. Some persons with vested interest indulge
in the pastime of meddling with judicial process either by force of habit or from improper
motives, and try to bargain for a good deal as well to enrich themselves. Often they are
actuated by a desire to win notoriety or cheap popularity. The petitions of such busy
bodies deserve to be thrown out by rejection at the threshold, and in appropriate cases
with exemplary costs.
19. The Council for Public Interest Law set up by the Ford Foundation in USA defined
the "public interest litigation" in its report of Public Interest Law, USA, 1976 as follows :
"Public Interest Law is the name that has recently been given to efforts provide legal
representation to previously unrepresented groups and interests. Such efforts have been
undertaken in the recognition that ordinary market place for legal services fails to provide
such services to significant segments of the population and to significant interests. Such
groups and interests include the proper environmentalists, consumers, recial and ethnic
minorities and others."
20. The Court has to be satisfied about (a) the credentials of the applicant; (b) the prima
facie correctness or nature of information given by him; (c) the information being not
vague and indefinite. The information should show gravity and seriousness involved.
Court has to strike balance between two conflicting interests; (i) nobody should be
allowed to indulge in wild and reckless allegations besmirching the character of others;
and (ii) avoidance of public mischief and to avoid mischievous petitions seeking to assail,
for oblique motives, justifiable executive actions. In such case, however, the Court cannot
afford to be liberal. It has to be extremely careful to see that under the guise of redressing
a public
@page-SC919
grievance, it does not encroach upon the sphere reserved by the Constitution to the
Executive and the Legislature. The Court has to act ruthlessly while dealing with
imposters and busybodies or meddlesome interlopers impersonating as public-spirited
holy men. They masquerade as crusaders of justice. They pretend to act in the name of
Pro Bono Publico, though they have no interest of the public or even of their own to
protect.
21
. Courts must do justice by promotion of good faith, and prevent law from crafty
invasions. Courts must maintain the social balance by interfering where necessary for the
sake of justice and refuse to interfere where it is against the social interest and public
good. (See State of Maharashtra v. Prabhu, (1994 (2) SCC 481), and Andhra Pradesh
State Financial Corporation v. M/s. GAR Re-Rolling Mills and Anr. (AIR 1994 SC 2151).
No litigant has a right to unlimited draught on the Court time and public money in order
to get his affairs settled in the manner as he wishes. Easy access to justice should not be
misused as a licence to file misconceived and frivolous petitioners. (See Dr. B. K.
Subbarao v. Mr. K. Parasaran, (1996 (7) JT 265). Today people rush to Courts to file
cases in profusion under this attractive name of public interest. They must inspire
confidence in Courts and among the public. 1994 AIR SCW 1953
1996 AIR SCW 3356

22. As noted supra, a time has come to weed out the petitions, which though titled as
public interest litigations are in essence something else. It is shocking to note that Courts
are flooded with large number of so called public interest litigations where even a
minuscule percentage can legitimately be called as public interest litigations. Though the
parameters of public interest litigation have been indicated by this Court in large number
of cases, yet unmindful of the real intentions and objectives, Courts are entertaining such
petitions and wasting valuable judicial time which, as noted above, could be otherwise
utilized for disposal of genuine cases. It is also noticed that petitions are based on
newspaper reports without any attempt to verify their authenticity. As observed by this
Court in several cases newspaper reports do not constitute evidence. A petition based on
unconfirmed news reports, without verifying their authenticity should not normally be
entertained. As noted above, such petitions do not provide any basis for verifying the
correctness of statements made and information given in the petition. It would be
desirable for the Courts to filter out the frivolous petitions and dismiss them with costs as
afore-stated so that the message goes in the right direction that petitions filed with
oblique motive do not have the approval of the Courts.
23

. In S. P. Gupta v. Union of India (1981 Supp. SCC 87), it was emphatically pointed out
that the relaxation of the rule of locus standi in the field of PIL does not give any right to
a busybody or meddlesome interloper to approach the Court under the guise of a public
interest litigant. He has also left the following note of caution : (SCC p. 219, para 24) :
AIR 1982 SC 149, (Para 23)

"But we must be careful to see that the member of the public, who approaches the Court
in cases of this kind, is acting bona fide and not for personal gain or private profit or
political motivation or other oblique consideration. The Court must not allow its process
to be abused by politicians and others to delay legitimate administrative action or to gain
a political objective."
24
. In State of H. P. v. A Parent of a Student of Medical College, Simla and Ors. (1985 (3)
SCC 169), it has been said that public interest litigation is a weapon which has to be used
with great care and circumspection. AIR 1985 SC 910

25

. These aspects have been highlighted in Ashok Kumar Pandey v. State of West Bengal
(2004 (3) SCC 349) and Dr. B. Singh v. Union of India and Ors. (2004 (3) SCC 363)
and Dattaraj Nathuji Thaware v. State of Maharashtra and Ors. (2005 (1) SCC 590).
2003 AIR SCW 6105
2004 AIR SCW 1494
2005 AIR SCW 46

26. It is true that in certain cases even though the Court comes to the conclusion that the
writ petition was not in a public interest, yet if it finds that there is scope for dealing with
the matter further in greater public interest, it can be done. This can be done by keeping
the writ petitioner out of picture and appointing an amicus curiae. This can only be done
in exceptional cases and not in a routine manner.
27. It is true as contented by learned counsel for the appellant that the High Court's
conclusions were drawn after going through the files. It is apparent from records that the
High Court did not ask the parties to clarify any doubt it entertained as regards
@page-SC920
certain crucial aspects. These aspects assume considerable importance because they have
formed the foundation of the High Court's conclusions about irregularity/illegality in
allotment.
28. In the circumstances we set aside the order of the High Court and remit the matter to
it for fresh consideration. Needless to say the parties shall be permitted to place material
in support of their respective stand, in addition to those which are already on record and
the High Court shall thereafter take a decision in the matter within four months. Further
affidavits shall be filed with all relevant details /documents by the parties. We make it
clear that we have not expressed any opinion on the merits of the case.
29. The appeal is allowed to the aforesaid extent.
Appeal partly allowed.
AIR 2008 SUPREME COURT 920 "Dharam Pal v. State of U.P."
(From : Allahabad)*
Coram : 2 C. K. THAKKER AND TARUN CHATTERJEE, JJ.
Criminal Appeal No. 884 of 2001, D/- 4 -1 -2008.
Dharam Pal and Ors. v. State of U.P.
(A) Criminal P.C. (2 of 1974), S.385, S.386 - APPEAL - APPELLATE COURT - Appeal
- Can be decided on merits by High Court in absence of appellants - Moreover, counsel
appeared for appellants expressed his inability to argue case as there was no response
from appellants despite repeated reminders to them - Thus, it cannot be said that notice of
appeal was not served on them.
It cannot be said that the High Court ought not to have decided the appeal on merits in the
absence of the appellants as the High Court had no power or jurisdiction under S. 385 or
386 to do so. Further, it cannot be said that the notice of appeal was not served upon the
appellants and, therefore, without a proper service of notice of appeal on the appellants
and without giving them any opportunity of hearing to proceed with the appeal, the High
Court erred in proceeding with the appeal and deciding the same on merits. Even if it was
assumed that the notice of appeal was not served on the appellants, then also, it was an
admitted position that the counsel for the appellants appeared for them to prosecute the
appeal and, therefore, after appearance of the counsel for the appellants, it must be held
that the notice of appeal was duly served. Moreover, the counsel for the appellants
submitted before the High Court that despite repeated reminders to the appellants, the
appellants were not responding and, therefore, the counsel for the appellants expressed
his inability to argue the case before the High Court.(Para 7)
When the accused does not appear, it is the bounden duty of the High Court to look into
the records and the other materials on record, including the judgment of the trial Court
and thereafter, decide the appeal on merits which would be due compliance with Ss. 385
and 386 in disposing of criminal appeals. The Appellate Court must dispose of the appeal
after perusal of the record and judgment of the trial Court even if the appellant or his
counsel was not present at the time of hearing of the appeal. The only exception is that if
the appellant is in jail and his counsel is not present, the Court should adjourn the case to
facilitate the appearance of the appellant. There is yet another exception to this rule,
namely, that in an appropriate case, the Court can appoint a lawyer at the State expense to
assist the Court. Therefore, the High Court was justified in taking the assistance of the
Assistant Government Advocate and after taking such assistance and considering the
entire evidence on record, the High Court passed the judgment under appeal.
1996 AIR SCW 2986, Followed. (Para 8)
It cannot be said that the High Court had ignored the basic principles of criminal justice
while disposing of the appeal ex parte. Therefore, the matter cannot be remitted back to
the High Court for decision afresh after giving opportunity of hearing to the appellants.
(Para 9)
(B) Criminal P.C. (2 of 1974), S.154 - FIR - DYING DECLARATION - F.I.R. - Deceased
himself dictated F.I.R. to Police - Same was read over to him and he put his thumb
impression on same - Said report is admissible as dying declaration under S. 32 of
Evidence Act.
Evidence Act (1 of 1872), S.32.
@page-SC921
The report of occurrence was dictated by the deceased himself and the same was read
over to him after which he had put his thumb impression on the same. This report is
admissible under S. 32 of the Evidence Act as a dying declaration. It is true that the
original document signed by the deceased was not brought on record but the FIR has
rightly been admitted as a dying declaration. There appears no reason for the police to
falsely implicate any one of the accused inasmuch as, initially, the report dictated by the
deceased was taken down as a non-cognizable report under S.323 of the IPC. If the police
were to implicate the accused, they would have not taken down the report as a non-
cognizable report in the very first place itself. (Paras 10, 11)
Further, the names of the accused and the important features of the case have been clearly
mentioned in the report. It contains a narrative by the deceased as to the cause of his
death, which finds complete corroboration from the testimony of eye-witnesses and the
medical evidence on record. There is nothing on record to show that the deceased was not
in a position to speak at the time when he dictated the report of occurrence. On the other
hand, the materials and the other evidence on record show that the deceased was in a
position to speak when he dictated the report of occurrence. Therefore, the High Court
was fully justified in holding that the deceased was in a fit state of mind at the time of
making the statement. Furthermore, the dying declaration was fully corroborated by the
other evidence on record. (Para 11)
(C) Criminal P.C. (2 of 1974), S.154 - FIR - EVIDENCE - FIR - Non-production of -
Effect - Original FIR could not be produced as registers relating to non-cognizable
offences were destroyed after a lapse of two years - Loss of original FIR was duly proved
by prosecution witness -Accordingly, secondary evidence adduced by prosecution was
accepted - Therefore, no adverse inference could be drawn against prosecution for non-
production of original FIR.
Evidence Act (1 of 1872), S.3. (Para 11)
(D) Evidence Act (1 of 1872), S.3 - EVIDENCE - WITNESS - Interested witnesses -
Evidence of witnesses who were relatives of deceased - Cannot be discarded in the
absence of any infirmity in said evidence. (Para 12)
(E) Penal Code (45 of 1860), S.304, Part II - CULPABLE HOMICIDE - MURDER -
Culpable homicide - Fight between parties resulted on drinking of water from hand pipe
after exchange of abuses - There was no premeditation - There was no intention on part
of accused to cause death of deceased - Nor were injuries caused with intention of
causing such bodily injuries as the accused knew were likely to cause death - Thus
offence committed by them would be culpable homicide not amounting to murder -
Accused liable to be convicted under S. 304, Part II and not under S. 302. (Para 13)
Cases Referred : Chronological Paras
2004 AIR SCW 5754 : AIR 2004 SC 5056 : 2004 Cri LJ 4973 12
1996 AIR SCW 2986 : AIR 1996 SC 2439 : 1996 Cri LJ 3491 (Foll.) 6, 7, 8, 9
AIR 1971 SC 1606 : 1971 Cri LJ 1177 6
K. C. Lamba (for M.P. Shorawala), for Appellants; Pramod Swarup, Ms. Subvira Lal,
Anuvrat Sharma and Sanjay Kumar Singh, for Respondent.
* Cri. A. No. 1303 of 1982, D/- 4-11-1999 (All.)
Judgement
1. TARUN CHATTERJEE, J. :-This appeal arises from the judgment and order dated 4th
of November, 1999 of the High Court of Judicature at Allahabad whereby the High Court
had partly allowed the appeal of the accused/appellants herein thereby setting aside their
conviction and sentence of imprisonment for life under Section 302/34 of the Indian
Penal Code (for short "the IPC") imposed by the VIIIth Additional Sessions Judge,
Bareilly, U.P. and instead convicting and sentencing them to 7 years' rigorous
imprisonment under Section 304, Part II read with Section 34 of the IPC. The
accused/appellants (for short "the appellants") before us are Mahabir, Najjoo, Dharam Pal
and Sheru whose fluctuating fortunes shall be set at rest by us in this appeal.
2. In order to appreciate the controversy involved, we propose to give a brief narrative of
the prosecution case relevant for our consideration.
3. The incident took place on 5th of June, 1978 in Village Khalanpur where the deceased
Rajpal had come to see a fair. At about 2 p.m., he went to drink water at a hand pipe
towards the north of Ram Das
@page-SC922
Teli's House. An altercation took place between Mahabir and Rajpal deceased on drinking
of water. There was an exchange of abuses between Dharam Pal and Rajpal. Thereafter,
Rajpal left the place and proceeded towards the southern side. Meanwhile, all the four
accused came there and assaulted Rajpal with lathis who sustained head injuries and fell
down. The accused thereafter fled from the spot. Raghu, father of Rajpal arrived there
shortly and took him to Faridpur Police Station on a bullock cart where Rajpal himself
dictated a report of occurrence. The report was registered under Section 323 of the IPC
against the four accused as a non-cognizable report at 21.10 hours on 5th of June, 1978.
Rajpal was medically examined at the Primary Health Center, Faridpur at 10.00 p.m. on
the same night. He, however, succumbed to his injuries at about 1.00 p.m. on 7th of June,
1978.
4. After Rajpal died, information was sent to the police station and the case was
converted into one under section 304 of the IPC. Thereafter, the case was investigated by
Sub-Inspector P.C. Sharma, who submitted the charge-sheet against the appellants on
28th of October, 1978. The learned Magistrate took cognizance of the offence and
committed the case to the Court of Session. The Sessions Judge framed charge under
Section 302/34 of the IPC against all the appellants who pleaded not guilty and claimed
to be tried. Nine witnesses including three eye-witnesses were examined from the side of
the prosecution. Two witnesses were examined by the appellants in their defence. In their
statement under Section 313 of the Code of Criminal Procedure (for short "the Code"),
the appellants denied the prosecution case and alleged false implication on account of
enmity. The Sessions Judge, as noted hereinabove, believed the case of the prosecution
and convicted the appellants and sentenced them to imprisonment for life under Section
302/34 of the IPC. Against this decision of the Sessions Judge, an appeal was preferred
before the Allahabad High Court by the appellants. It may be kept on record that when
the appeal was taken up for hearing before the High Court, the learned counsel for the
appellants made a statement that despite repeated letters, the appellants were not
responding and therefore he was not in a position to argue the appeal. The High Court,
thereafter, scrutinized the entire record with the assistance of Learned Assistant
Government Advocate. As noted hereinabove, the appeal was partly allowed and the
appellants were convicted and sentenced to rigorous imprisonment of 7 years under
Section 304, Part II read with Section 34 of the IPC. It is this judgment of the High Court
which is impugned in this appeal.
5. We have heard the learned counsel for the parties and examined the entire materials on
record. We shall now deal with each of the questions raised before us by the learned
counsel for the parties.
6

. The learned counsel for the appellants, at the first instance, submitted that since the
appellants were not served with a notice of appeal in the High Court, the appeal was
disposed of by the High Court ex parte without affording any opportunity of hearing to
the appellants. Our attention was drawn to the decision of this court in Bani Singh v. State
of U.P. [(1996) 4 SCC 720] to drive home the point that the High Court was duty-bound
to ensure proper compliance with Sections 385 and 386 of the Code in disposing of
criminal appeals when the accused did not appear and that the Appellate Court must
dispose of the appeal on merits after perusal and scrutiny of the record. Relying on the
decision of this court in the case of Bani Singh [supra], the learned counsel for the
appellants sought to argue that the High Court was not justified in deciding the appeal on
merits without giving any opportunity of hearing to the appellants. He submitted that a
further date for hearing the appeal ought to have been fixed by the High Court and not
having done so, it had acted illegally and with material irregularity in deciding the appeal
on merits. This submission of the learned counsel for the appellants was, however,
contested by the learned counsel appearing on behalf of the respondent. The learned
counsel for 1996 AIR SCW 2986
1996 AIR SCW 2986

@page-SC923
the respondent submitted that the High Court was fully justified in deciding the appeal on
merits even in the absence of the learned counsel for the appellants as from the record, it
would be clear that the notice of appeal was duly served on the appellants and in spite of
such service of notice and also in view of the fact that a learned advocate had appeared
for the appellants, it would not be justified to say that a further date ought to have been
fixed by the High Court for hearing of the appeal. The learned counsel for the respondent
further contended that the High Court had followed the principles laid down by this court
in Bani Singh's case [supra] and disposed of the appeal on merits in the absence of the
appellants or their learned counsel. In Bani Singh's case [supra], this court observed in
paragraph 10 as under:-
"10. In Shyam Deo case1, this Court ruled that the Appellate Court must peruse the
record before disposing of the appeal; the appeal has to be disposed of on merits even if it
is being disposed of in the absence of the appellant or his pleader. Interpreting Section
423 of the Old Code (the corresponding provisions are Sections 385-386 of the present
Code), this Court in paragraph 19 of the judgment held as under (SCC p. 861, Para 19):
1 (1971) 1 SCC 855 : 1971 SCC (Cri) 353) : AIR 1971 SC 1606.
"The consideration of the appeal on merits at the stage of final hearing and to arrive at a
decision on merits and to pass final orders will not be possible unless the reasoning and
findings recorded in the judgment under appeal are tested in the light of the record of the
case. After the records are before the court and the appeal is set down for hearing, it is
essential that the Appellate Court should (a) peruse such record, (b) hear the appellant or
his pleader, if he appears, and (c) hear the public prosecutor, if he appears. After
complying with these requirements, the Appellate Court has full power to pass any of the
orders mentioned in the section. It is to be noted that if the appellant or his pleader is not
present or if the public prosecutor is not present, it is not obligatory on the Appellate
Court to postpone the hearing of the appeal. If the appellant or his counsel or the public
prosecutor, or both, are not present, the Appellate Court has jurisdiction to proceed with
the disposal of the appeal; but that disposal must be after the Appellate Court has
considered the appeal on merits. It is clear that the appeal must be considered and
disposed of on merits irrespective of the fact that whether the appellant or his counsel or
the public prosecutor is present or not. Even if the appeal is disposed of in their absence,
the decision must be after consideration on merits.
(Emphasis added)
11. In our view, the above-stated position is in consonance with the spirit and language of
Section 386 and, being a correct interpretation of the law, must be followed."
7. Before we proceed further, we keep it on record that in the present case, the appellants
were granted bail and in fact, at the time of hearing of the appeal, they were already
enlarged on bail. Only after the judgment was delivered by the High Court, the bail was
cancelled and they were directed to surrender before the appropriate authority. At this
stage, we may note the relevant provisions under the Code of Criminal Procedure (for
short the Code). Chapter XXIX of the Code deals with appeals under the Code. Sections
385 and 386 of the Code, which are the most important provisions for dealing with the
case in hand, are reproduced as under :-
"385. Procedure for hearing appeals not dismissed summarily- (1) If the Appellate Court
does not dismiss the appeal summarily, it shall cause notice of the time and place at
which such appeal will be heard to be given
(i) to the appellant or his pleader :
(ii) ...
(iii) ...
(iv) ...
(2) The Appellate Court shall then send for the record of the case, if such record is not
already available in that Court, and hear the parties :
Provided that if the appeal is only as to the extent or the legality of the sentence, the
Court may dispose of the appeal without sending for the record.
(3) ...
386. Powers of the Appellate Court - After perusing such record and hearing the appellant
or his pleader, if he appears, and the Public Prosecutor, if he appears, and in case of an
appeal under Section 377 or Section 378, the accused, if he appears, the Appellate Court
may, if it considers that there is no sufficient ground for interfering, dismiss the appeal, or
may-
xxx xxx xxx xxx"

Having examined the provisions under Sections 385 and 386 of the Code, as noted
hereinabove, and applying the principles laid down by this court in the case of Bani Singh
[supra], we are not in agreement with the argument advanced by the learned counsel for
the appellants that the High Court ought not to 1996 AIR SCW 2986

@page-SC924
have decided the appeal on merits in the absence of the appellants as the High Court had
no power or jurisdiction under Section 385 or 386 of the Code to do so. So far as the
service of notice of the appeal on the appellants by the High Court is concerned, we are
unable to agree with the learned counsel for the appellants that the notice of appeal was
not served upon them and, therefore, without a proper service of notice of appeal on the
appellants and without giving them any opportunity of hearing to proceed with the
appeal, the High Court erred in proceeding with the appeal and deciding the same on
merits. Even if we assume that the notice of appeal was not served on the appellants, then
also, it was an admitted position that the learned counsel for the appellants appeared for
them to prosecute the appeal and therefore, after appearance of the learned counsel for
the appellants, it must be held that the notice of appeal was duly served. At the risk of
repetition, we may note that the learned counsel for the appellants submitted before the
High Court that despite repeated reminders to the appellants, the appellants were not
responding and, therefore, the learned counsel for the appellants expressed his inability to
argue the case before the High Court.
8

. That apart, the decision of this court in Bani Singh's case [supra] would clearly show
that when the accused does not appear, it is the bounden duty of the High Court to look
into the records and the other materials on record, including the judgment of the trial
court and thereafter, decide the appeal on merits which would be due compliance with
Sections 385 and 386 of the Code in disposing of criminal appeals. While dealing with
the procedure for disposing of a criminal appeal, this court in Bani Singh's case [supra]
has clearly laid down that the dismissal of an appeal for default or non-prosecution
without going into the merits of the case is clearly illegal and that the Appellate Court
must dispose of the appeal on merits after perusal and scrutiny of record and after giving
a hearing to the parties, if present, before disposal of the appeal on merits. This court, in
that decision, further held that the Appellate Court must dispose of the appeal after
perusal of the record and judgment of the trial court even if the appellant or his counsel
was not present at the time of hearing of the appeal. The only exception, as we find from
the aforesaid decision of this court, is that if the appellant is in jail and his counsel is not
present, the court should adjourn the case to facilitate the appearance of the appellant.
There is yet another exception to this rule, namely, that in an appropriate case, the court
can appoint a lawyer at the State expense to assist the court. Therefore, the High Court, in
our view, was justified in taking the assistance of the Assistant Government Advocate and
after taking such assistance and considering the entire evidence on record, the High Court
passed the judgment under appeal before us holding that the appellants were guilty of the
offence, not under Section 302/34 of the IPC but under Section 304, Part II of the IPC
and directed them to undergo 7 years' rigorous imprisonment. In doing so, the High Court
affirmed the findings of the trial court but differed on the point of the offence committed
by the appellants and the corresponding punishment to be awarded to them. After a
thorough appreciation of the evidence on record, the High Court recorded the following
findings :- 1996 AIR SCW 2986

"1. Both the eye-witnesses PW 2 Dannu and PW 3 Om Prakash had stated that they were
present in the fair and had seen the occurrence. In spite of lengthy cross-examination of
these witnesses, their testimony that they had seen the occurrence could not been
shattered in any manner.
2. PW2 Dannu and PW3 Om Prakash had stated in their testimony that all the four
accused assaulted Rajpal with dandas near the pakar tree who fell down after receiving
injuries on his head.
3. The medical evidence corroborated the testimony of the eye-witnesses that the assault
was made upon Rajpal by danda, which is a blunt weapon.
4. The names of PW2 Dannu and PW4 Satyapal were mentioned in the N.C.R. lodged by
Rajpal. There is no reason to doubt the presence of PW2 Dannu and PW4 Satyapal on the
spot, who saw the occurrence. PW2 Dannu and PW4 Satyapal were truthful and reliable
witnesses and implicit reliance could be placed on their testimonies.
5. The FIR of the occurrence was lodged by the deceased Rajpal himself. The report
dictated by Rajpal was initially taken down as a non-cognizable report under Section 323
of the IPC. Therefore, there was no
@page-SC925
occasion for either falsely implicating any one as accused or exaggerating the role played
by any accused.
6. The testimony of PW6 Ram Swaroop Mishra, Head Constable showed that after the
report had been dictated by Rajpal, the same was read over to him and thereafter he had
put his thumb impression over the same. This act found mention in the report itself.
7. The report was admissible under Section 32 of the Evidence Act as a dying declaration
of the deceased Rajpal. The names of the accused and the important features of the case
had been mentioned therein. The report contained a truthful version of the incident as
narrated by Rajpal as to the cause of his death.
8. The version given in the FIR found complete corroboration from the testimony of eye-
witnesses and the medical evidence on record.
9. The evidence did not show that the deceased was not in a position to speak at the time
when he dictated the report of the occurrence.
10. The testimony of defence witnesses did not inspire confidence and was not worthy of
belief.
11. It cannot be said that the accused had any intention of causing the death of Rajpal nor
were the injuries caused with the intention of causing such bodily injuries as the accused
knew were likely to cause death.
12. The knowledge that death is likely to be caused could be inferred as they gave the
blow on the head. The accused had therefore committed offence under Section 304, Part
II of the IPC."
9

. From the above findings of the High Court, it is abundantly clear that the High Court
had arrived at a well-merited judgment after a careful consideration of the materials on
record. The position, of course, would have been different if the High Court had simply
dismissed the appeal without going into the merits. However, nothing of this sort has
been done in the present case. The judgment of the High Court clearly shows that
evidence before the trial court has been carefully deliberated upon and weighed and it is
only then that the conclusions have been arrived at. Therefore, relying on the aforesaid
principles and in view of the discussions made hereinabove, we are afraid that the
decision of this Court in Bani Singh's case [supra] is of no help to the appellants but on
the other hand, the High Court, while dealing with the appeal ex parte had followed the
guidelines laid down in that case. That being the position, it cannot be said that the High
Court had ignored the basic principles of criminal justice while disposing of the appeal ex
parte. In our view, there has been substantial compliance with the guidelines made in
Bani Singh's case [supra]. Accordingly, we are unable to agree with the learned counsel
for the appellants that the matter should be remitted back to the High Court for decision
afresh after giving opportunity of hearing to the appellants. 1996 AIR SCW 2986

10. The learned Counsel for the appellants further argued before us that the alleged dying
declaration which was given the shape of an FIR could not be made the basis of
conviction when the original document signed by the deceased was not brought on
record. The learned counsel for the appellants tried to prove before us that the deceased
was not in a position to speak and which becomes apparent from the testimony of his
father. However, it would not be correct to say so. The evidence of PW 7 Dr. R.P. Goel
shows that the condition of the deceased was good and that he was in a position to speak.
It would not be appropriate for us to read between the lines by giving unnecessary
meanings to the testimony of Raghu. It cannot be left out of sight that Raghu also said
that the deceased dictated the FIR to the police. In any view of the matter, the report of
occurrence was dictated by the deceased himself and the same was read over to him after
which he had put his thumb impression on the same. This report is admissible under
Section 32 of the Evidence Act as a dying declaration. It is true that the original
document signed by the deceased was not brought on record, but in our view, the FIR has
rightly been admitted as a dying declaration. There appears no reason for the police to
falsely implicate any one of the accused inasmuch as, initially, the report dictated by the
deceased was taken down as a non-cognizable report under section 323 of the IPC. If the
police were to implicate the accused, they would have not taken down the report as a
non-cognizable report in the very first place itself.
11. That apart, the report dictated by the deceased fully satisfied all the
@page-SC926
ingredients for being made admissible as a dying declaration. To ascertain this aspect, we
may refer to some of the general propositions relating to a dying declaration. Section
32(1) of the Indian Evidence Act deals with dying declaration and lays down that when a
statement is made by a person as to the cause of his death, or as to any of the
circumstances of the transaction which resulted in his death, such a statement is relevant
in every case or proceeding in which the cause of the person's death comes into question.
Further, such statements are relevant whether the person who made them was or was not
at the time when they were made under expectation of death and whatever may be the
nature of the proceedings in which the cause of his death comes into question. The
principle on which a dying declaration is admissible in evidence is indicated in the
Maxim Nemo Moriturus Praesumitur Mentire, which means that a man will not meet his
maker with a lie in his mouth. Thus it is clear that a dying declaration may be relating
to :-
a) as to the cause of death of the deceased,
b) as to "any of the circumstances of the transaction" which resulted in the death of the
deceased.
It is also clear that it is not necessary that the declarant should be under expectation of
death at the time of making the statement. If we look at the report dictated by the
deceased in the light of the aforesaid propositions, it emerges that the names of the
accused and the important features of the case have been clearly mentioned in the report.
It contains a narrative by the deceased as to the cause of his death, which finds complete
corroboration from the testimony of eye-witnesses and the medical evidence on record.
There is nothing on record to show that the deceased was not in a position to speak at the
time when he dictated the report of occurrence. On the other hand, the materials and the
other evidence on record would conclusively show, as rightly held by the High Court, that
the deceased was in a position to speak when he dictated the report of occurrence.
Therefore, in our view, the High Court was fully justified in holding that the deceased
was in a fit state of mind at the time of making the statement. In the present case, as noted
hereinabove, the dying declaration was fully corroborated by the other evidence on
record. That apart, in our view, the submission of the learned counsel for the appellants
that the dying declaration which was given the shape of an FIR could not be made the
basis of conviction when the original document signed by the deceased was not brought
on record is not acceptable. It is an admitted position that despite best efforts, the original
FIR could not be produced as the registers relating to non-cognizable offences were
destroyed after a lapse of two years. For this reason, the Sessions Court had duly
considered this aspect of the matter and found that the loss of the original FIR was duly
proved by PW 6 and accordingly, the secondary evidence adduced by the prosecution was
accepted. We do not find any infirmity in the said finding when, admittedly, the original
register was destroyed after a lapse of two years. Therefore, no adverse inference could
be drawn against the prosecution for non-production of the original FIR. That being the
position and in view of our discussions, we are not inclined to accept the argument of the
learned counsel for the appellant that the deceased was not in a position to speak when he
dictated the report or that the alleged dying declaration could not be admissible in
evidence because of the other infirmities, as noted hereinabove.
12

. This takes us to the next question viz. whether the other lacunae pointed out by the
learned counsel for the appellants are fatal to the prosecution case. We agree that the High
Court erred in relying on the evidence of PW4, who admittedly was declared a hostile
witness. Nevertheless, we feel that in the face of the other evidence of PW2 Dannu, PW3
Om Prakash who were corroborated in all material respects by PW7 Dr. R.P.Goyal and by
PW9, Dr. U. Kanchan, the evidence of PW4, even if discarded, is inconsequential. The
evidentiary value of a dying declaration and the principles underlying the importance of a
dying declaration have already been discussed herein earlier. Simply because PW2 and
PW3, in their cross-examination, have been shown to be related to the deceased does not
mean that their testimony has to be rejected. It is well settled that evidence of a witness is
not to be rejected merely because he happens to be a relative of the deceased. In State of
Himachal Pradesh v. Mast Ram [(2004) 8 SCC 660], this Court observed as under:-
2004 AIR SCW 5754

".....The law on the point is well settled


@page-SC927
that the testimony of the relative witnesses cannot be disbelieved on the ground of
relationship. The only main requirement is to examine their testimony with caution. Their
testimony was thrown out at the threshold on the ground of animosity and relationship.
This is not a requirement of law.. ......."
In this view of the matter and this being the well-settled law, it is difficult for us to
discard the evidence of the witnesses, as discussed hereinabove, only on the ground that
they were related to the deceased, in the absence of any infirmity in the said evidence.
13. In the light of the aforesaid discussions, let us now see whether the High Court was
justified, in the facts and circumstances of the present case, to convert the offence from
Section 302/34 of the IPC to Section 304, Part II of the IPC. In this regard, we may again
note the findings recorded by the High Court, as noted herein earlier, in clauses 11and 12.
The High Court observed that the accused did not have any intention of causing the death
of Rajpal nor were the injuries caused with the intention of causing such bodily injuries
as the accused knew were likely to cause death. The High Court further observed that the
knowledge that death was likely to be caused could be inferred as the accused gave the
blow on the head. Let us now see whether the aforesaid act would warrant a punishment
under Section 302 or Section 304 of the IPC. In our view, the facts disclose that there was
no premeditation and the fight resulted on drinking of water from the hand pipe after an
exchange of abuses. There appeared no intention on the part of the appellants to cause the
death of the deceased Rajpal. Therefore, the offence committed by the appellants, in our
view, is culpable homicide not amounting to murder because, in our view, it falls within
Exception 4 to Section 300 which reads as under :-
"Exception 4- Culpable homicide is not murder if it is committed without premeditation
in a sudden fight in the heat of passion upon a sudden quarrel and without the offender
having taken undue advantage or acted in a cruel or unusual manner.
Explanation- It is immaterial in such cases which party offers the provocation or commits
the first assault."
Section 304 of the IPC lays down the punishment for culpable homicide not amounting to
murder and reads as under : -
"Whoever commits culpable homicide not amounting to murder shall be punished with
[imprisonment for life], or imprisonment of either description for a term which may
extend to ten years, and shall also be liable to fine, if the act by which the death is caused
is done with the intention of causing death, or of causing bodily injury as is likely to
cause death, or with imprisonment of either description for a term which may extend to
ten years, or with fine, or with both, if the act is done with the knowledge that it is likely
to cause death, but without any intention to cause death, or to cause such bodily injury as
is likely to cause death."
We have already gone through the evidence and the other materials on record. From the
evidence on record, we cannot find any ground to discard the finding of the High Court
that it cannot be said that the accused had any intention of causing the death of Rajpal,
the deceased, nor were the injuries caused with the intention of causing such bodily
injuries as the accused knew were likely to cause death. Therefore, in the absence of any
intention of causing the death of the deceased Rajpal, we are in agreement with the High
Court that the accused must be convicted of the offence under Section 304, Part II of the
IPC and not under Section 302 of the IPC.
14. For the reasons aforesaid, we do not find any cogent reason to interfere with the
judgment of the High Court converting the offence to Section 304, Part II of the IPC from
Section 302 of the IPC. Accordingly, the appeal fails and is dismissed with no order as to
costs.
Appeal dismissed.
AIR 2008 SUPREME COURT 927 "Ramesh Krishna Madhusudan Nayar v. State of
Maharashtra"
(From : Bombay)*
Coram : 2 Dr. A. PASAYAT AND AFTAB ALAM, JJ.
Criminal Appeal No. 12 of 2008 (arising out of SLP (Cri.) No. 4630 of 2006), D/- 7 -1
-2008.
Ramesh Krishna Madhusudan Nayar v. State of Maharashtra.
(A) Evidence Act (1 of 1872), S.134 - WITNESS - MURDER - Solitary witness -
Testimony wholly reliable - Conviction can be based.
Penal Code (45 of 1860), S.300.
@page-SC928
On the basis of a solitary evidence conviction can be maintained. Section 134 clearly
states that no particular number of witnesses is required to estabish the case. Conviction
can be based on the testimony of a single witness if he is wholly reliable. Corroboration
may be necessary when he is only partially reliable. If the evidence is unblemished and
beyond all possible criticism and the Court is satisfied that the witness was speaking the
truth then on his evidence alone conviction can be maintained. (Para 7)
(B) Penal Code (45 of 1860), S.300, Exception 4, Exception 1 - MURDER - CULPABLE
HOMICIDE - Murder - Sudden fight - Quarrel taking place between accused and
deceased - Accused thereupon giving two blows by wooden log on head of deceased -
Entitled to benefit of Exception 4 to S. 300 - Liable to be convicted under 304, Part 2 -
Not for murder - Distinction between Exceptions 1 and 4 of S.300 explained. (Paras
9, 11)
Cases Referred : Chronological Paras
2006 AIR SCW 1678 : 2006 Cri LJ 2111 : 2006 (2) AIR Jhar R 666 : 2006 (3) AIR Bom
R 321 10
2006 AIR SCW 3860 : AIR 2006 SC 2763 : 2006 Cri LJ 4041 10
2004 AIR SCW 4480 : AIR 2004 SC 4100 : 2004 Cri LJ 3875 10
2004 AIR SCW 4650 : AIR 2004 SC 4496 10
2004 AIR SCW 5672 : AIR 2004 SC 5039 : 2004 Cri LJ 4660 : 2004 All LJ 3822 10
Bimal Roy Jad, Ms. Sunita Pandit, for Appellant; R. K. Adsure, for Respondent.
* Cri. A. No. 10 of 2001, D/- 8-9-2004 (Bom.)
Judgement
1. Dr. ARIJIT PASAYAT, J. :- Leave granted.
2. Challenge in this appeal is to the judgment of Bombay High Court, Aurangabad Bench,
dismissing the appeal of the appellant who faced trial for alleged commission of offence
punishable under Section 302 of the Indian Penal Code, 1860 (in short IPC) and was
sentenced to imprisonment for life by learned Additional Sessions Judge, Ahmednagar.
3. Background facts in a nutshell are as follows:
The complainant Sajay Vithal was serving as a waiter in Sanjog Hotel for 2-1/2 months
prior to the incident. Pradip Panjabi is the owner of the said hotel. Business in the hotel is
conducted from 5 p.m. to 11 p.m. After closure of the hotel, complainant Sanjay
alongwith 5 workers of the hotel used to reside in a staff room. Hotel was closed on
3.11.1999 at 11.30 p.m. Pradip Panjabi and other staff members went out at about 1 a.m.
Thereafter on 4.11.1999 around 1.30 a.m. in the night, altercations took place between
Ramesh Nayar and Anna Devraj (hereinafter referred to as the deceased) on the point of
switching off the lights. Both used to reside in the staff room. At that time, complainant,
Kundlik Chavhan and Chhotu intervened. Thereafter complainant and Anna Devraj slept
in the staff room. At about 8.30 a.m. complainant heard loud noise relating to a quarrel
and got up. He saw the accused and the deceased quarrelling and accused inflicting two
blows by a wooden log on the head of Anna Devraj. Ramesh Nayar threatened the
complainant that if he disclosed anything to anybody, he will teach him a lesson. Hence
complainant went out of the room. He disclosed the incident to the persons in the hotel
working as gardeners in the morning. At that time, Anna Devraj was not speaking
anything. He was lying unconscious and moaning. Thereafter owner of the hotel was
informed on phone. He came and the deceased was shifted to Civil Hospital for
treatment. His right ear was bleeding. Thereafter, the complainant and hotel owner went
to Tophkhana Police Station and reported the matter to police as per Exh.26. A.S.I. Puri
registered the offence as Crime No.227/99 under Sections 307, 506 of IPC and handed
over investigation to PW.7, P.S.I. Jyoti Madhav Karandikar. After completion of
investigation, charge sheet was placed and accused-appellant faced trial as he denied the
occurrence and pleaded false implication. The trial Court placed reliance on the evidence
of Sanjay Diwate (PW-5). It is to be noted that certain other persons i.e. Dhirendera
Suryavanshi (PW-2), Ashok Palve (PW-3) and Datta Pingale (PW-6) were claimed to be
eye-witnesses, but they made departure from the statements given during investigation.
The trial Court found the evidence of PW-5 to be credible and cogent and recorded his
conviction and imposed the sentence of imprisonment for life.
4. The conviction and sentence were challenged before the High Court, which as noted
above, dismissed the appeal.
5. In support of the appeal, leaned
@page-SC929
coun-sel for the appellant submitted that the conviction could not have been recorded
solely on the testimony of one alleged eye-witness PW-5. Alternatively, it is submitted
that Section 302, IPC has no application to the facts of the case in view of the factual
scenario highlighted. According to him in course of a sudden quarrel the incident
happened. In other words, according to him Exception 4 to Section 300, IPC applies.
6. Learned counsel for the respondent-State on the other hand supported the judgment of
conviction and sentence.
7. Coming to the question whether on the basis of a solitary evidence conviction can be
maintained, a bare reference to Section 134 of the Evidence Act, 1872 (in short the
Evidence Act) would suffice. The provision clearly states that no particular number of
witnesses is required to establish the case. Conviction can be based on the testimony of a
single witness if he is wholly reliable. Corroboration may be necessary when he is only
partially reliable. If the evidence is unblemished and beyond all possible criticism and the
Court is satisfied that the witness was speaking the truth then on his evidence alone
conviction can be maintained.
8. For bringing in operation of Exception 4 to Section 300, IPC, it has to be established
that the act was committed without premeditation, in a sudden fight in the heat of passion
upon a sudden quarrel without the offender having taken undue advantage and not having
acted in a cruel or unusual manner.
9. The Fourth Exception of Section 300, IPC covers acts done in a sudden fight. The said
exception deals with a case of prosecution not covered by the first exception, after which
its place would have been more appropriate. The exception is founded upon the same
principle, for in both there is absence of premeditation. But, while in the case of
Exception 1 there is total deprivation of self-control, in case of Exception 4, there is only
that heat of passion which clouds men's sober reasons and urges them to deeds which
they would not otherwise do. There is provocation in Exception 4 as in Exception 1; but
the injury done is not the direct consequence of that provocation. In fact Exception 4
deals with cases in which notwithstanding that a blow may have been struck, or some
provocation given in the origin of the dispute or in whatever way the quarrel may have
originated, yet the subsequent conduct of both parties puts them in respect of guilt upon
equal footing. A sudden fight implies mutual provocation and blows on each side. The
homicide committed is then clearly not traceable to unilateral provocation, nor in such
cases could the whole blame be placed on one side. For if it were so, the Exception more
appropriately applicable would be Exception 1. There is no previous deliberation or
determination to fight. A fight suddenly takes place, for which both parties are more or
less to be blamed. It may be that one of them starts it, but if the other had not aggravated
it by his own conduct it would not have taken the serious turn it did. There is then mutual
provocation and aggravation, and it is difficult to apportion the share of blame which
attaches to each fighter. The help of Exception 4 can be invoked if death is caused (a)
without premeditation, (b) in a sudden fight; (c) without the offenders having taken undue
advantage or acted in a cruel or unusual manner; and (d) the fight must have been with
the person killed. To bring a case within Exception 4 all the ingredients mentioned in it
must be found. It is to be noted that the fight occurring in Exception 4 to Section 300,
IPC is not defined in the IPC. It takes two to make a fight. Heat of passion requires that
there must be no time for the passions to cool down and in this case, the parties have
worked themselves into a fury on account of the verbal altercation in the beginning. A
fight is a combat between two and more persons whether with or without weapons. It is
not possible to enunciate any general rule as to what shall be deemed to be a sudden
quarrel. It is a question of fact and whether a quarrel is sudden or not must necessarily
depend upon the proved facts of each case. For the application of Exception 4, it is not
sufficient to show that there was a sudden quarrel and there was no premeditation. It must
further be shown that the offender has not taken undue advantage or acted in cruel or
unusual manner. The expression undue advantage as used in the provision means unfair
advantage.
10

. The aforesaid aspects have been highlighted in Sridhar Bhuyan v. State of Orissa (2004
(6) JT SC 299), Prakash Chand v. State of H.P. (2004 (6) JT SC 302), Sachchey Lal
Tiwari v. State of Uttar Pradesh (2004 (8) JT SC 534), Sandhya Jadhav v. State of
Maharashtra [2006(4) SCC 653] 2004 AIR SCW 4480
2004 AIR SCW 4650
2006 AIR SCW 1678
2006 AIR SCW 3860

@page-SC930
and Lachman Singh v. State of Haryana [2006 (10) SCC 524].
11. Considering the factual background the inevitable conclusion is that the appropriate
conviction would be under Section 304, Part I, IPC and not Section 302 IPC. Custodial
sentence of 10 years would meet the ends of justice.
12. The appeal is allowed to the aforesaid extent.
Order accordingly.
AIR 2008 SUPREME COURT 930 "State of Himachal Pradesh v. Paras Ram"
(From : Himachal Pradesh)*
Coram : 2 Dr. A. PASAYAT AND AFTAB ALAM, JJ.
Criminal Appeal No. 1 of 2008 (arising out of SLP (Cri.) No. 1040 of 2007), D/- 3 -1
-2008.
State of H.P. v. Paras Ram and Ors.
(A) Criminal P.C. (2 of 1974), S.378(3) -APPEAL - EQUALITY - Appeal against
acquittal - Grant/refusal of leave - Resoned order has to be passed.
Cri. M.P. (M) No. 623 of 2006, D/- 12-10-2006 (H.P.), Reversed.
Constitution of India, Art.14.
An application for grant of leave to file appeal against acquittal has to be disposed of by a
reasoned order. Reasons introduce clarity in an Order. On plainest consideration of
justice, the High Court ought to set forth its reasons, howsoever brief, in its order
indicative of an application of its mind, all the more when its order is amenable to further
avenue of challenge. The absence of reasons renders the High Court order not
sustainable. Right to reason is an indispensable part of a sound judicial system, reasons at
least sufficient to indicate an application of mind to the matter before Court. Another
rationale is that the affected party can know why the decision has gone against him. One
of the salutary requirements of natural justice is spelling out reasons for the order made,
in other words, a speaking out. The "inscrutable face of a sphinx" is ordinarily
incongruous with a judicial or quasi-judicial performance.
Cri. M.P. (M) No. 623 of 2006, D/- 12-10-2006 (H.P.), Reversed. (Paras 8, 11)
(B) Constitution of India, Art.141 - PRECEDENT - SUPREME COURT - JUDICIAL
DISCIPLINE - Supreme Court decision - Binding force - Judicial dicipline to abide by
S.C. decision cannot be forsaken, under any pretext by any authority or Court, be it even
High Court. (Para 8)
Cases Referred : Chronological Paras
2004 AIR SCW 102 : AIR 2004 SC 1203 : 2004 Cri LJ 916 (Ref.) 10
(2001) 10 SCC 607 (Ref.) 8
AIR 1987 SC 724 : 1987 Cri LJ 698 (Rel. on) 8
AIR 1982 SC 1215 : 1982 Cri LJ 1743 (Rel. on) 8
1974 LCR 1209
(1971) 1 All ER 1148 : (1971) 2 WLR 742 9
J. S. Attri, for Appellant; Dr. I. B. Gaur, for Respondents.
* Cri. M. P. (M) No. 623 of 2006, D/- 12-10-2006 (HP).
Judgement
1. Dr. ARIJIT PASAYAT, J. :-Leave granted.
2. Refusal to grant leave to question judgment of acquittal in terms of Section 378(3) of
the Code of Criminal Procedure, 1973 (in short 'Cr.P.C.') is the subject-matter of
challenge in this appeal. According to the appellant-State of Himachal Pradesh, the one
line order "Dismissed" of the Himachal Pradesh High Court without assigning reasons
therefor does not meet the requirement of law.
3. Respondents (hereinafter referred to as the 'accused' faced trial for alleged commission
of offences punishable under Sections 436, 447, 427, 147 and 506 of the Indian Penal
Code, 1860 (in short 'IPC').
4. The prosecution case, in brief, is that the complainant Smt. Manjit Kaur is a resident of
Village Abada Barana, Distt. Una, H. P. She is a housewife. On 30-6-2003 at about 5.45
p.m. she and her sister-in-law Nirmala Devi were watching television in their house and
their children were playing outside, while her husband Gurdial Singh had gone to Kuthar
to bring medicines. In the meantime daughter of her sister-in-law aged about 13 years
named Poonam came there and told that some persons were uprooting creepers from their
field. On this they both came out and saw that the above named accused were doing the
same and on seeing the complainant and her sister-in-law the accused came towards the
house of the complainant and challenged them that they would set their house on fire.
The accused Balbir Singh, Baldev Singh, Jai Gopal
@page-SC931
and Radhey Shyam were carrying 'Mashals' in their hands and with the help of those
Mashals they lit their thatched house on fire from three sides. When the complainant and
her sister-in-law tried to take out their articles from the house, the accused threatened
them to throw in the fire. On this, the complainant got frightened and she along with her
sister-in-law and children rushed towards Kuthar Kalan while raising cries. On listening
their cries Santosh Kumari, w/o Jog Raj and Yash Pal, s/o Bihari Lal, both resident of
Kuthar Kalan, reached at the spot. Thereafter the complainant went to Kuthar Kalan and
told about this incident to her husband who informed the fire brigade and police. The
police reached at the spot and recorded statement of complainant Ex.PW-1/A under
Section 154 of the Code of Criminal Procedure, 1973 (in short 'Cr.P.C.') upon which FIR
Ex.PW-12/A was recorded against the accused. During investigation the police prepared
the photographs of the spot and obtained demarcation of the land over which the house in
question was situated. After completion of investigation charge-sheet was filed and the
accused persons faced trial. Thirteen persons were examined as witnesses. PWs. 1 and 4
apart from other were stated to be eye-witnesses. The High Court found that there was
some delay in lodging the FIR and though large number of people were claimed to have
gathered at the spot, the witnesses could not have seen the accused persons. The High
Court also found that there was some dispute between the parties and, therefore, the
prosecution version was suspect. Accordingly, the accused persons were acquitted.
5. The appellant-State filed an application for grant of leave. High Court disposed of the
application in the following manner :
"Dismissed"
6. According to learned counsel for the appellant-State it was imperative on the High
Court to indicate reason as to why the prayer for grant of leave was found untenable. In
the absence of any such reasons the order of the High Court is indefensible. Learned
counsel for the respondents supported the order.
7. Section 378(3) of the Code deals with the power of the High Court to grant leave in
case of acquittal. Section 378(1) and (3) of the Code reads as follows :
"378(1) Save as otherwise provided in sub-section (2) and subject to the provisions of
sub-sections (3) and (5), the State Government may, in any case, direct the Public
Prosecutor to present an appeal to the High Court from an original or appellate order of
acquittal passed by any Court other than a High Court or an order of acquittal passed by
the Court of Session in revision.
(3) No appeal under sub-section (1) or sub-section (2) shall be entertained except with the
leave of the High Court".
8

. The trial Court was required to carefully appraise the entire evidence and then come to a
conclusion. If the trial Court was at lapse in this regard the High Court was obliged to
undertake such an exercise by entertaining the appeal. The trial Court on the facts of this
case did not perform its duties, as was enjoined on it by law. The High Court ought to
have in such circumstances granted leave and thereafter as a first Court of appeal, re-
appreciated the entire evidence on the record independently and returned its findings
objectively as regards guilt or otherwise of the accused. It has failed to do so. The
question involved were not trivial. The High Court has not given any reasons for refusing
to grant leave to file appeal against acquittal, and seems to have been completely
oblivious to the fact that by such refusal, a close scrutiny of the order of acquittal, by the
appellate forum, has been lost once and for all. The manner in which appeal against
acquittal has been dealt with by the High Court leaves much to be desired. Reasons
introduce clarity in an order. On plainest consideration of justice, the High Court ought to
have set forth its reasons, howsoever brief, in its order indicative of an application of its
mind, all the more when its order is amenable to further avenue of challenge. The absence
of reasons has rendered the High Court order not sustainable. Similar view was expressed
in State of U.P. v. Battan and Ors. (2001 (10) SCC 607). About two decades back in State
of Maharashtra v. Vithal Rao Pritirao Chawan (AIR 1982 SC 1215) the desirability of a
speaking order while dealing with an application for grant of leave was highlighted. The
requirement of indicating reasons in such cases has been judicially recognized as
imperative. The view was reiterated in Jawahar Lal Singh v. Naresh Singh and Ors.
(1987 (2) SCC 222). Judicial discipline to abide by declaration of law by this Court,
cannot be forsaken, AIR 1987 SC 724

@page-SC932
under any pretext by any authority or Court, be it even the Highest Court in a State,
oblivious to Article 141 of the Constitution of India, 1950 (in short the 'Constitution').
9. Even in respect of administrative orders Lord Denning M.R. in Breen v. Amalgamated
Engineering Union (1971 (1) All ER 1148) observed "The giving of reasons is one of the
fundamentals of good administration". In Alexander Machinery (Dudley) Ltd. v. Crabtree
(1974 LCR 120) it was observed :
"Failure to give reasons amounts to denial of justice". Reasons are live links between the
mind of the decision taker to the controversy in question and the decision or conclusion
arrived at". Reasons substitute subjectivity by objectivity. The emphasis on recording
reasons is that if the decision reveals the "inscrutable face of the sphinx", it can, by its
silence, render it virtually impossible for the Courts to perform their appellate function or
exercise the power of judicial review in adjudging the validity of the decision. Right to
reason is an indispensable part of a sound judicial system, reasons at least sufficient to
indicate an application of mind to the matter before Court. Another rationale is that the
affected party can know why the decision has gone against him. One of the salutary
requirements of natural justice is spelling out reasons for the order made, in other words,
a speaking out. The "inscrutable face of a sphinx" is ordinarily incongruous with a
judicial or quasi-judicial performance.

10

. The aforesaid aspects were highlighted by this Court in State of Punjab v. Bhag Singh
(2004 (1) SCC 547). 2004 AIR SCW 102
11. In view of the aforesaid legal position, the impugned judgment of the High Court is
unsustainable and is set aside. We grant leave to the State to file the appeal. The High
Court shall entertain the appeal and after formal notice to the respondents hear the appeal
and dispose of it in accordance with law, uninfluenced by any observation made in the
present appeal. The appeal is allowed to the extent indicated.
Appeal allowed.
AIR 2008 SUPREME COURT 932 "State of U.P. v. Punni"
(From : 1999 Cri LJ 4641 (Allahabad))
Coram : 2 C. K. THAKKER AND TARUN CHATTERJEE, JJ.
Criminal Appeal No. 463 of 2001, D/- 4 -1 -2008.
State of U.P. v. Punni and Ors.
(A) Evidence Act (1 of 1872), S.3, S.114 - Penal Code (45 of 1860), S.399, S.402 -
EVIDENCE - DACOITY - WITNESS - Non-examination of witness - Adverse inference
- Case of attempt to commit dacoity - Station Officer who had prepared plan to prevent
dacoity and nabbed dacoits - Had also dictated F.I.R. - Not examined by prosecution -
I.O. also not examined - Non-examination of S.O. and I.O. held was fatal to prosecution.
(Para 9)
(B) Penal Code (45 of 1860), S.399, S.402 - DACOITY - FIR - Preparation to commit
dacoity - Prosecution case that police nabbed dacoits when they had assembled in a grove
- No adjoining grove holder, though present at time of incident, examined - Adverse
inference can be drawn against prosecution - Arrest of dacoits was without any resistance
or struggle on their part - Statement in F.I.R. that police got information from an informer
- Not supported by Sub-Inspector - One of accused having enmity with co-accused
making his participation improbable - Accused liable to be acquitted. (Para 9)
(C) Constitution of India, Art.136 - SPECIAL LEAVE APPEAL - EVIDENCE - Appeal -
Against order of acquittal - Interference only if finding is vitiated by some glaring
infirmity in the appraisement of evidence or such finding was perverse or arbitrary.
(Para 11)
Cases Referred : Chronological Paras
(2004) 9 SCC 193 (Disting.) 10
2001 AIR SCW 4350 : AIR 2001 SC 2902 (Disting.) 10
1995 AIR SCW 950 : AIR 1995 SC 975 : 1995 Cri LJ 1456 (Rel. on, Pnt. C) 11
(1974) 6 SCC 29 (Rel. on, Pnt. C) 11
AIR 1974 SC 1890 : 1974 Cri LJ 1297 (Rel. on, Pnt. C) 11
AIR 1973 SC 2673 : 1973 Cri LJ 1807 (Rel. on, Pnt. A) 9
AIR 1954 SC 51 : 1954 Cri LJ 338 (Rel. on) 8
@page-SC933

AIR 1915 Cal 545 : 1915 (16) Cri LJ 170 8


Pramod Swarup, Praveen Swarup, Ms. Reena Singh and Anuvrat Sharma, for Appellant;
Rana Ranjit Singh, Manish Vashishtha, Prakash Kumar and S. K. Sinha, for Respondents.
Judgement
1
. TARUN CHATTERJEE, J. :- This is an appeal against the judgment and order dated
21st of May, 1999 passed by the learned Judge of the High Court of Judicature at
Allahabad in Criminal Appeal No.2921 of 1980 whereby the High Court had allowed the
appeal of the accused/respondents and set aside the judgment and order dated 29th of
November, 1980 passed by the Additional Sessions Judge, VIth Court at Etah in Sessions
Trial No.406 of 1978 (State of U.P. v. Punni and 5 others) convicting the
accused/respondents of the offences under Sections 399 and 402 of the Indian Penal Code
(for short the IPC) and sentencing each one of them to undergo rigorous imprisonment
for a period of 4 years and 2 years respectively and further convicting each one of them
under Section 27 of the Arms Act and sentencing them to undergo rigorous imprisonment
for a period of 6 months. However, all the aforesaid sentences were ordered to run
concurrently. Reported in 1999 Cri LJ 4641 : 1999 All LJ 2028

2. The relevant facts leading to the filing of this appeal may be narrated, in a nutshell,
which are as follows :-
3. The case of the prosecution, inter alia, was that on 15th of October, 1977, one Ram
Charan Singh, Station Officer (S.O.), Police Station Sikanderpur Vaish, Assistant Sub-
Inspector (A.S.I.) Gaya Prasad along with constables Bhanwar Singh, Lakhan Singh,
Mulaim Singh, Dina Nath, Jamuna Prasad, Rajendera Singh and Head Constable Hajari
Singh were returning to the Police Station from the patrol duty. When they reached near
village Nagla Abdal at about 8 p.m., a reliable informer notified the S.O. Ram Charan
Singh that the gang of Punni Habda shall assemble in the grove of Pandit Lakhan Singh
of Rani Damer at about 1:00 am to commit dacoity and would loot Nagla Karan. On
receiving this information, the S.O. and the A.S.I. had called for two witnesses, namely,
Nakdey and Sri Pal from village Nagla Abdal and also procured their licenced firearms
through Constable Bhanwar Singh. Thereafter, they came to village Rani Damer
wherefrom witnesses, namely, Ranvir, Ram Prakash, Deo Singh and Soran were also
taken with them. Out of these witnesses, Ram Prakash (PW 2) was carrying his licensed
gun. Thereafter, all of them came to the Madha of Pradhan in village Rani Damer where
the witnesses were told the purpose of calling them and the necessary instructions
pertaining to dacoity prevention scheme were given. A search was also made on all the
persons present to ensure that none possessed any illegal weapons. Thereafter, two parties
were formed by the S.O., one in his leadership, which comprised Ram Prakash (PW 2)
and the other in the leadership of Gaya Prasad, A.S.I. (PW 1). At 11.00 p.m., the two
parties had reached near the grove of Pandit Lakhan Singh. The party headed by the S.O.
positioned themselves on the north of the grove while the other party positioned on the
east of the grove. While they were waiting there, some persons entered the grove and
started conversing and smoking biri and cigarette. One of them was over heard saying
"Nagla chhota hi hai. Ustad nahin aaye hain. Darney ki koyee bat nahin hai. Chalo
chalkar loot lengey." On hearing this conversation, the S.O. and the witnesses were
convinced that the assembled persons were a gang of dacoits and that they had assembled
there to commit dacoity. The S.O. then challenged the dacoits telling them that they were
under siege and commanded them to surrender their weapons, lest they would be done to
death. A V.L.P. shot was fired by the Head Constable, whereupon these persons started to
run away from that site. The police party arrested 6 persons while 3 managed to escape.
The arrested persons were questioned and they revealed their names to the police. On
search being taken, one gun and 8 live cartridges were recovered from Punni, one
Tamancha and 4 live cartridges were recovered from Munshi, one Tamancha and 5 live
cartridges and one torch were recovered from Saligram, one Tamancha and 3 live
cartridges were recovered from Sultan, one Bhala and a torch were recovered from Ram
Murti, all without licence, and a Bamboo lathi was recovered from Ram Bharose. The
empty V.L.P. shot, half burnt pieces of biris and match sticks were also collected. The
recovery memos were prepared on the spot and the collected articles were sealed in
separate bundles. Thereafter, the police party returned to the Police Station alongwith the
accused and the recovered articles. On 16th of October, 1977, a chik report was prepared
in accordance with the dictation given by
@page-SC934
the S.O. and a case was registered under Sections 399 and 402 of the IPC and separate
cases were registered under Sections 25 and 4/25 of the Arms Act. The investigation was
completed and the charge sheets were prepared and submitted to the concerned
Magistrate who had committed the case to the Court of Session.
4. The accused/respondents after appearance pleaded not guilty and claimed to be tried. It
was contended by the accused/respondents that they were falsely implicated in the case.
The witnesses and Thakurs took Begar from them and when they declined to comply with
their demand, they connived with the police to implicate them. The witnesses were the
Dalals of the Police and the Police had enmity against them. The main ground of attack of
the accused/respondents was the non-examination of the S.O., which, according to them,
was fatal. No injuries were caused to the police party and therefore, the case was not
probable. The respondent, Ram Bharosey, was examined as a witness and he sought to
show that he was not on good terms with the co-accused Munshi and one Ram Chadra
who was the brother of another co-accused Ram Murti. In order to show enmity, certified
copies of some police FIRs were submitted by Ram Bharosey. Thus, it was highly
improbable for him to form a gang with them to commit dacoity. Therefore, according to
the respondents, they were falsely implicated in the case and they should have been
acquitted.
5. The Additional Sessions Judge, VIth Court at Etah relying on the evidence of the two
witnesses, namely, PW1 and PW2 and after rejecting the testimony of the accused Ram
Bharosey convicted the six accused for the offences under Sections 399 and 402 of IPC
and Section 27 of the Arms Act and sentenced them in the manner indicated herein
earlier. Feeling aggrieved by the said decision of the Additional Sessions Judge, VIth
Court at Etah, an appeal was preferred by the accused/respondents which, by the
impugned order was allowed and the said judgment of the Additional Sessions Judge,
VIth Court at Etah was set aside. It is this order of the High Court, which is impugned in
this appeal.
6. Since this is a case where the High Court acquitted the accused/respondents thereby
setting aside the order of conviction of the Additional Sessions Judge, VIth Court at Etah,
it would be appropriate to consider the findings arrived at by the Additional Sessions
Judge, VIth Court at Etah as well as by the High Court. The Additional Sessions Judge,
VIth Court at Etah convicted the accused/respondents, inter alia, on the following
findings :-
(i) On each broad and important aspect of the case, the two witnesses PW 1 and PW 2
had given cogent evidence proving the case and that their testimonies fully answered the
test of credibility.
(ii) PW 2 Ram Prakash had no reason to give false evidence against the accused.
(iii) The accused were arrested on the spot and there was nothing to show that they were
arrested from their houses.
(iv) The properties were recovered from their possession and the V.L.P. shot was fired.
(v) PW 1 and the S.O Ram Charan Singh were present throughout and the examination of
the S.O. would not have brought any improvement.
(vi) The accused had only country lethal weapons and if their weapons could not be put
to use, the case of the prosecution could not be thrown out.
(vii) The accused Ram Bharosey did not deny the charges after entering the witness box.
(viii) The defence could not point out any infirmity in the prosecution case and the case
stood fully proved beyond doubt.
On the aforesaid findings made by the Additional Sessions Judge, VIth Court at Etah, the
accused/respondents were convicted under Sections 399 and 402 of the IPC and also
under Section 27 of the Arms Act.
The High Court, as mentioned herein earlier, had set aside the conviction on appeal.
While setting aside the conviction, the High Court, inter alia, recorded the following
findings :-
(i) The S.O. Ram Charan Singh was not examined, although the FIR of the case was
dictated by him.
(ii) The I.O. of the case was also not examined at the trial.
(iii) None of the adjoining grove holders or land holders were said to have been present in
the grove at the time of occurrence although the site plan prepared by the investigating
officer shows that the grove of
@page-SC935
Lakhan Singh was surrounded on three sides by groves.
(iv) The accused/respondents were said to have been caught on the spot without any
resistance or struggle on their part.
(v) There were discrepancies in the examination-in-chief and cross examination of PW 1
as to the time of his departure from the police station and also as to the fact of his leaving
the station with SO Ram Charan Singh.
(vi) There were contradictions in the testimony of PW 1 and the FIR as to the fact of their
leaving the police station alone or with others.
(vii) The S.O. had claimed in the FIR that an informer had given the information that
dacoity would be committed in Nagla Karan at 3 a.m. but PW 1 did not testify to the
giving of any such information by the informer.
(viii) There were contradictions in the cross examination of PW 1 and the medical
evidence as to the fact of receiving of danda blows on the body of the accused.
(ix) There appeared substance in the defence plea that the accused Ram Bharosey could
not have joined hands with Munshi and Ram Murti for committing dacoity on account of
their enmity.
The High Court thus concluded that all these circumstances tended to show that the
accused had been bundled together by the police and implicated in the case of assembly
and preparation to commit dacoity.
7. Before we consider whether the High Court was justified in reversing the order of
conviction of the Additional Sessions Judge, VIth Court at Etah, and passing an order of
acquittal in appeal, we may briefly highlight the issues raised before us. The Learned
Counsel for the appellant argued that the High Court had erred in taking the adverse view
on account of non-examination of the I.O. when the A.S.I. Gaya Prasad had adduced the
entire sequence of events in a natural and convincing manner. He also sought to argue
that the reasons of acquittal recorded by the High Court were erroneous and against the
weight of the evidence proved on record. On the other hand, the learned counsel for the
accused/respondents sought to argue that the High Court, while acquitting the accused
and reversing the judgment of the Additional Sessions Judge, VIth Court at Etah, who
convicted them, had taken into consideration all the aspects of the matter and the
evidence on record and came to a conclusion that the judgment of the Additional Sessions
Judge could not be accepted, after giving proper and cogent reasons for the same.
Accordingly, the learned counsel for the accused/respondents sought for dismissal of the
appeal by this Court.
8. Having heard the learned counsel for the parties and after examining the submissions
made by them and also the judgment of the High Court as well as of the Additional
Sessions Judge, VIth Court at Etah and the other materials on record, including the
findings and the reasoning given by the Additional Sessions Judge as well as the High
Court, we do not find any ground to hold that the High Court was not justified in setting
aside the order of conviction and passing an order of acquittal in appeal. While doing so,
the High Court had given due reasons after considering the entire materials and the
evidence on record and had also given the reasons as to why the non-examination of the
S.O. and the I.O. was fatal in the facts and circumstances of the case. In our view, the
High Court was justified in holding that it was necessary for the prosecution to prove the
case made out under Sections 399 and 402 of the IPC beyond reasonable doubt and to
examine the S.O. and the I.O. for unfolding the prosecution story. The High Court had
also given its reasons, in our view, correctly, that the evidence of PW 1 and PW 2 on
which, strong reliance was placed by the Additional Sessions Judge in order to pass an
order of conviction could not be relied upon. On the question of non-examination of the
S.O. and the I.O., which led to an adverse inference being drawn by the High Court
against the prosecution, the fact that the same was fatal would also be clear from a
decision of this Court in the case of Habeeb Mohammad v. State of Hyderabad [AIR
1954 SC 51] in which this Court at paragraph 11 observed as follows:-

"It is said that the state of things above described arose because of a supposed obligation
on the prosecution to call every available witness on the principle laid down in such a
case as Ram Ranjan Roy v. Emperor (ILR 42 Cal 422), to the effect that all available eye-
witnesses should be called by the prosecution even though, as in the case cited, their
AIR 1915 Cal 545

@page-SC936
names were on the list of defense witnesses. Their Lordships do not desire to lay down
any rules to fetter discretion on a matter such as this which is so dependent on the
particular circumstances of each case. Still less do they desire to discourage the utmost
candour and fairness on the part of those conducting prosecutions; but at the same time
they cannot, speaking generally, approve of an idea that a prosecution must call witnesses
irrespective of considerations of number and of reliability, or that a prosecution ought to
discharge the functions both of prosecution and defense. If it does so confusion is very
apt to result, and never is it more likely to result than if the prosecution calls witnesses
and then proceeds almost automatically to discredit them by cross-examination.
Witnesses essential to the unfolding of the narrative on which the prosecution is based,
must, of course, be called by the prosecution, whether in the result the effect of their
testimony is for or against the case for the prosecution."
9

. Relying on the aforesaid observations of this Court in the above-mentioned case, we,
therefore, agree with the findings and the reasoning of the High Court, while setting aside
the order of conviction, on the question of non-examination of the S.O., who was the
architect of the facts of the case. In Ram Prasad and Ors. v. State of U.P. [1974 (3) SCC
388], this Court has held that in case the Court finds that the prosecution has not
examined the witnesses for reasons not tenable or not proper, the Court would be justified
in drawing an adverse inference against the prosecution. In view of the non-examination
of the S.O. and the I.O. and also in view of the glaring discrepancies pointed out by the
High Court in its judgment, as noted herein earlier, we are, therefore, in agreement with
the High Court that in the facts and circumstances of the present case and on the evidence
on record, the order of acquittal was reasonably possible to arrive at and that being the
position, we do not find any reason to interfere with the judgment of acquittal in the
exercise of our jurisdiction under Article 136 of the Constitution. At the risk of repetition,
we may also reiterate that the High Court, after consideration of all the evidence and
materials on record had come to a conclusion of fact that the prosecution story as made
out to convict the accused/respondents under Sections 399 and 402 of the IPC could not
at all be believed and therefore, the order of conviction of the Additional Sessions Judge,
VIth Court at Etah was needed to be interfered with. There is one further aspect of this
matter. In our view, the High Court was justified in drawing an adverse inference against
the prosecution as it had failed to examine the adjoining grove holders or land holders
who were said to have been present in the grove at the time of occurrence. That apart, it
was rightly pointed out by the High Court that adverse inference ought to have been
drawn against the prosecution as admittedly, the persons who were caught on the spot
were caught without any resistance or struggle from their side. From the judgment of the
High Court, it is also evident that the High Court had found discrepancies in the
examination-in-chief and the cross-examination of PW 1 as to the time of his departure
from the police station and also as to the fact of his leaving the station with the S.O., Ram
Charan Singh. At this stage, we may further reiterate that the Additional Sessions Judge,
VIth Court at Etah, while convicting the accused/respondents had practically relied on the
evidence of PW 1, whose evidence, in fact, was rightly not accepted by the High Court in
view of the discrepancies found in his evidence. Finally, in our view, the High Court,
while reversing the order of conviction, had also noted other contradictions viz., vii, viii
and ix, as noted herein earlier, which, in our view, are material contradictions which
would lead to acquitting the accused/respondents. AIR 1973 SC 2673, Para 13

10
. Before we part with our discussion on the findings of the High Court while setting aside
the order of conviction of the Additional Sessions Judge, VIth Court at Etah, we may note
that reliance was placed at the bar on the case of Kashiram and others v. State of M.P.
[(2002) 1 SCC 71]. In that decision, this Court while considering the power of the High
Court to interfere with an order of acquittal of the trial Court held that when two views
are possible, the High Court should not interfere only because it feels that sitting as a trial
Court, it would have preferred conviction and that the High Court should consider every
reason given by the trial Court in favour of an acquittal and then dislodge them. It was
also held in that decision that while deciding an appeal against an order of acquittal, the
High Court can reappraise 2001 AIR SCW 4350

@page-SC937
the evidence, arrive at findings at variance with those recorded by the trial Court in its
order of acquittal and arrive at its own findings, yet, the salutary principle, which would
guide the High Court is if two views are reasonably possible, one supporting the acquittal
and the other recording a conviction, the High Court would not interfere merely because
it feels that sitting as a trial Court, its view would have been one of recording a
conviction. It was further held in that decision that as a necessary corollary, it was
obligatory on the High Court, while reversing an order of acquittal, to consider and
discuss each of the reasons given by the trial Court to acquit the accused and then to
dislodge those reasons and if the High Court failed to discharge this obligation, it would
constitute a serious infirmity in the judgment of the High Court. Reliance was also placed
on the decision of this Court in Kunju Muhammed alias Khumani and another v. State of
Kerala [(2004) 9 SCC 193] wherein this Court has held that the judgment of the trial
Court acquitting the accused cannot be reversed by the High Court when the findings of
the trial Court were neither perverse nor they could not be reached by a reasonable person
and the view taken by the trial Court was the only possible view. However, in the present
case, we are not concerned with the situation, which had arisen in the aforesaid two
decisions. In this case, the Additional Sessions Judge, VIth Court at Etah, convicted the
accused/respondents and such order of conviction was set aside in appeal by the High
Court. Therefore, in our view, the principles laid down in the aforesaid decisions are not
applicable to the facts of the present case although, from the aforesaid two decisions, it is
at least clear that while dealing with an appeal under Sections 378 and 386 of the Code of
Criminal Procedure, the salutary principle which would guide the High Court is if two
views are reasonably possible, one supporting the acquittal and the other recording a
conviction, the High Court would not interfere merely because it feels that sitting as a
trial Court, its view would have been one of recording a conviction. It was, however,
made clear in the aforesaid decisions by this Court that the High Court while hearing an
appeal against an acquittal has powers as wide and comprehensive as in an appeal against
a conviction and while exercising its appellate jurisdiction, the High Court can reappraise
the evidence, arrive at findings at variance with those recorded by the trial Court in its
order of acquittal and arrive at its own findings.
11
. In any view of the matter, we are of the view that this Court, while dealing with the
order of acquittal of the High Court, would not ordinarily interfere with the findings of
the High Court unless it is satisfied that such finding is vitiated by some glaring infirmity
in the appraisement of evidence or such finding was perverse or arbitrary. (See State of
U.P. v. Harihar Bux Singh [AIR 1974 SC 1890]. In State of Punjab v. Ajaib Singh [(1995)
2 SCC 486], this Court, on the same lines, held that if the order of acquittal was not
perverse or palpably erroneous, this Court would not interfere with such finding of the
High Court acquitting the accused/respondents from the offences charged against them.
While considering the scope of Article 136 of the Constitution as to when this Court is
entitled to interfere with an order of acquittal, this Court observed in State of U.P. v.
Babul Nath [(1994) 6 SCC 29] as follows : 1995 AIR SCW 950

"At the very outset we may mention that in an appeal under Article 136 of the
Constitution this Court does not normally reappraise the evidence by itself and go into the
question of credibility of the witnesses and the assessment of the evidence by the High
Court is accepted by the Supreme Court as final unless, of course, the appreciation of
evidence and finding is vitiated by any error of law of procedure or found contrary to the
principles of natural justice, errors of record and misreading of the evidence, or where the
conclusions of the High Court are manifestly perverse and unsupportable from the
evidence on record."
In view of our discussions made herein above, we do not find any ground to interfere
with the decision of the High Court, which on consideration of all the materials on record
and the evidence adduced by the parties had acquitted the accused/respondents and
therefore, no interference is warranted in the exercise of our power under Article 136 of
the Constitution.
12. For the reasons aforesaid, we do not find any reason to interfere with the judgment of
the High Court acquitting the accused/respondents. The appeal is thus dismissed. There
will be no order as to costs.
Appeal dismissed.
@page-SC938
AIR 2008 SUPREME COURT 938 "Union of India v. Ranchod"
(From : Madhya Pradesh)*
Coram : 2 G. P. MATHUR AND G. S. SINGHVI, JJ.
Civil Appeal Nos. 2108-2194 of 2003, with C. A. Nos. 2266, 2269-2282, 2099-2107,
2283-2294, 2195-2254, 2267, 2268, 2255-2265 and 9511-9522 of 2003 and C. A. Nos.
5609-5610 of 2007 (SLP (C) Nos. 711-714, 731-732, 735-736 and 740-741 of 2004), D/-
4 -12 -2007
Union of India and Anr. v. Ranchod and Ors.
Land Acquisition Act (1 of 1894), S.54 - ACQUISITION OF LAND - APPEAL -
REMAND OF MATTER - HIGH COURT - Compensation - Appeal - Cryptic order -
Appeal against compensation by landholders as well as Union of India - Disposal of by
High Court with observation that it would be appropriate to end litigation in "let be gones
be gones" spirit without considering evidence produced by parties - Is not in accordance
with O 41 R 31 - Matter remanded back for disposal according to law.
F. A. Nos. 297, 338, 339, 351 etc. of 1997, D/- 1-9-1999 (M.P.) Reversed.
Civil P.C. (5 of 1908), O.41, R.31.
In the matter of determination of compensation large number of factors have to be seen,
namely, nature and quality of land, whether irrigated or unirrigated, facilities for
irrigation like existence of well etc. presence of fruit bearing trees, the location of the
land, closeness to any road or highway, the evenness of the land, namely, whether its
level is even or there are pits etc., its position in rainy season, namely, whether water gets
accumulated in rains, existence of any building or structure and a host of factors having
bearing on valuation of the land.
In the instant case the High Court has not referred to even an iota of evidence which was
adduced by the parties. There were large number of landholders whose land was acquired
and they had filed separate objections under Section 9 of the Act, and had separately
sought reference under Section 18 of the Act. They had separately led evidence in support
of their cases before the reference Court. It is not a case where a single case may have
number of parties and the evidence adduced is common for all of them. High Court has
not at all adverted to even a single piece of evidence and there is absolutely no indication
in the judgment as to how it has come to a conclusion that the findings recorded by the
reference Court require to be affirmed.
There being total non-compliance of the mandatory provisions of Order XLI Rule 31
CPC the judgment of the High Court be liable to be set aside. Matter remanded to the
High Court for fresh consideration of the appeals.
F. A. Nos. 297, 338, 339, 351 etc. of 1997, D/- 1-9-1999 (M.P.), Reversed. (Paras 8, 9,
10)
Cases Referred : Chronological Paras
(2006)3 SCC 224 6
2001 AIR SCW 723 : AIR 2001 SC 965 7
AIR 1967 SC 1124 6, 7
Ms. Shipra Ghose, Ms. Sadhana Sandhu, Sunil Roy, R. C. Kathuria, Shail Kumar
Dwivedi, B. K. Prasad, Mrs. Anita Sahani, Mrs. Anil Katiya, Niraj Sharma, B. S. Banthia,
B. V. Balaram Das, Siddhartha Dave, Ms. Vibha Datta Makhija, M. M. Kashyap, Ms.
Meera Mathur, Ms. Aishwarya Bhati and C. D. Singh, for the Appearing Parties.
* F. A. Nos. 297, 338, 339, 351, 402 of 1997, 217, 372, 454 and etc. of 1998, 44, 164, 30
and etc. of 1999, D/- 1-9-1999 (MP).
Judgement
G. P. MATHUR, J. :- Delay condoned. Leave granted in special leave petitions.
2. These appeals, by special leave, have been preferred against the judgment and decree
dated 1-9-1999 of Madhya Pradesh High Court by which the appeals filed by the
landholders and also by the Union of India were dismissed.
3. The Government of India issued notifications under Sections 4(1) and 6(1) of the Land
Acquisition Act, 1894 (hereinafter referred to as the 'Act') for acquisition of large area of
land (4827.63 hectares) situate in various villages in Tehsil Mhow, District Indore for
establishing two firing ranges, namely, Bercha and Hema for the artillery wing of the
army. An area of 2917.160 hectares was acquired for Bercha Firing Range and 1910.464
hectares for Hema Firing Range. After receipt of notice under Section 9 of the Act, the
landholders submitted objections. the Collector, Indore, after considering the objections
of the landholders and making relevant inquiry, gave an Award regarding the
compensation which was to be paid to the landholders. The landholders being dissatisfied
with the Award of the Collector asked for a reference to be made to the Court in
accordance with Section 18 of the Act. The reference Court after taking into
@page-SC939
consideration the evidence adduced by the parties gave an Award. It awarded
compensation @ Rs. 58,000 per hectare for unirrigated and uncultivable land and Rs.
88,000/- per hectare for irrigated land in Bercha Firing Range. With regard to Hema
Firing Range compensation was awarded @ Rs. 40,000/- per hectare for uncultivable
land, Rs. 58,000/- per hectare for unirrigated land and Rs. 88,000/- per hectare for
irrigated land. The landholders and also the Union of India preferred appeals against the
Award of the reference Court before the High Court. The High Court decided all the
appeals by a common order, which is the subject matter of challenge in the present
appeals. The High Court passed a short order and the relevant part of the judgment
dealing with the controversy is reproduced below :-
"5. We would have very much liked to examine the merit of rival contentions, but it
would serve the interests of none. It could only prolong the agony of petty landholders
without resulting in the gain to union coffers. Assuming appeals filed by the Union were
to be allowed, it could prove futile because compensation amount awarded by reference
Court stood paid or was in the process of being paid to landholders under the orders of
this Court with little or no prospects of its recovery. Similarly if landholders' plea was to
be entertained, it could entail remand to the reference Court and protract the proceedings
for years on to their disadvantage and detriment. Therefore taking all this into
consideration and given regard to the interest of both parties we deem it appropriate to
end this litigation in "let be gones be gones" spirit, because adverting to the issues raised
by the parties would have opened Pandora's Box resulting in unending litigation causing
avoidable hardship and inconvenience more particularly to poor landholders who have
reportedly gone through considerable sufferings during the last 11 years for the sake of
National Defence. This is not to shy away from taking the adjudication to logical end but
to terminate the litigation to the mutual advantage and benefit of both sides."
4. Learned counsel for both the sides have submitted that the High Court has not at all
considered the evidence on record and has decided the appeals by a short and cryptic
order which shows a total non-application of mind. It has been submitted by learned
counsel for the parties that the matter requires fresh consideration in the light of evidence
which has been adduced by the parties.
5. Section 54 of the Act, insofar as relevant for the purposes of the present appeals, says
that subject to the provisions of the Code of Civil Procedure, 1908, applicable to appeals
from original decrees, and notwithstanding anything to the contrary in any enactment for
the time being in force, an appeal shall only lie in any proceedings under this Act to the
High Court from the award, or from any part of the award of the Court.
6. Order XLI, CPC deals with appeals from original decrees. Order XLI, Rule 31 lays
down that the judgment of the appellate Court shall be in writing and shall state (a) the
points for determination, (b) the decision thereon, (c) the reasons for the decision, and (d)
where the decree appealed from is reversed or varied, the relief to which the appellant is
entitled. This provision has come up for consideration in innumerable occasions and its
meaning and scope has been explained. It is not necessary for us to refer to various
decisions but we will refer to one of the recent judgments given in G. Amalorpavam and
others v. R. C. Diocese of Madurai (2006) 3 SCC 224, wherein this Court observed as
under :-
"The question whether in a particular case there has been substantial compliance with the
provisions of Order 41, Rule 31, CPC has to be determined on the nature of the judgment
delivered in each case. Non-compliance with the provisions may not vitiate the judgment
and make it wholly void, and may be ignored if there has been substantial compliance
with it and the higher appellate Court is in a position to ascertain the findings of the lower
appellate Court. It is no doubt desirable that the appellate Court should comply with all
the requirements of Order 41, Rule 31, CPC. But if it is possible to make out from the
judgment that there is substantial compliance with the said requirements and that justice
has not thereby suffered, that would be sufficient. Where the appellate Court having
considered the entire evidence on record and discussed the same in detail, come to any
conclusion and its findings are supported by reasons even though the point has not been
framed by the appellate Court there is substantial compliance with the provisions of
@page-SC940
Order 41, Rule 31, CPC and the judgment is not in any manner vitiated by the absence of
a point of determination. Where there is an honest endeavour on the part of the lower
appellate Court to consider the controversy between the parties and there is proper
appraisement of the respective cases and weighing and balancing of the evidence, facts
and the other considerations appearing on both sides is clearly manifest. by the perusal of
the judgment of the lower appellate Court, it would be a valid judgment even though it
does not contain the points for determination. The object of the rule in making it
incumbent upon the appellate Court to frame points for determination and to cite reasons
for the decision is to focus attention of the Court on the rival contentions which arise for
determination and also to provide litigant parties opportunity in understanding the ground
upon which the decision is founded with a view to enable them to know the basis of the
decision and, if so considered appropriate and so advised, to avail the remedy of second
appeal conferred by Section 100, CPC."
In Girja Nandini Devi v. Bijendra Narain Choudhury, AIR 1967 SC 1124, an observation
was made that it is not the duty of the appellate Court when it agrees with the view of the
trial Court on the evidence either to restate the effect of the evidence or to reiterate the
reasons given by the trial Court. Expression of general agreement with reasons given by
the Court decision of which is under appeal would ordinarily suffice.
7

. The aforesaid observation in Girja Nandini Devi (supra) is often misunderstood and
sometimes the Courts while delivering a judgment of affirmance have adopted a shortcut
method of not considering the evidence but merely expressing a general agreement with
the reasons given by the trial Court. This case was considered in Santosh Hazari v.
Purushottam Tiwari (2001)3 SCC 179, wherein it was observed as below :- 2001
AIR SCW 723

"The appellate Court has jurisdiction to reverse or affirm the findings of the trial Court.
First appeal is a valuable right of the parties and unless restricted by law, the whole case
is therein open for rehearing both on questions of fact and law. The judgment of the
appellate Court must, therefore, reflect its conscious application of mind and record
findings supported by reasons, on all the issues arising along with the contentions put
forth, and pressed by the parties for decision of the appellate Court. The task of an
appellate Court affirming the findings of the trial Court is an easier one. The appellate
Court agreeing with the view of the trial Court need not restate the effect of the evidence
or reiterate the reasons given by the trial Court; expression of general agreement with
reasons given by the Court, decision of which is under appeal, would ordinarily suffice
(See Girja Nandini Devi v. Bijendra Narain Choudhury). We would, however, like to
sound a note of caution. Expression of general agreement with the findings recorded in
the judgment under appeal should not be a device or camouflage adopted by the appellate
Court for shirking the duty cast on it. While writing a judgment of reversal the appellate
Court must remain conscious of two principles. Firstly, the findings of fact based on
conflicting evidence arrived at by the trial Court must weigh with the appellate Court,
moreso when the findings are based on oral evidence recorded by the same Presiding
Judge who authors the judgment. This certainly does not mean that when an appeal lies
on facts, the appellate Court is not competent to reverse a finding of fact arrived at by the
trial Judge. As a matter of law if the appraisal of the evidence by the trial Court suffers
from a material irregularity or is based on inadmissible evidence or on conjectures and
surmises, the appellate Court is entitled to interfere with the finding of fact."
8. In the case in hand the High Court has not referred to even an iota of evidence which
was adduced by the parties. There were large number of landholders whose land was
acquired and they had filed separate objections under Section 9 of the Act and had
separately sought references under Section 18 of the Act. They had separately lead
evidence in support of their cases before the reference Court. It is not a case where a
single case may have large number of parties and the evidence adduced is common for all
of them. In the matter of determination of compensation large number of factors have to
be seen, namely, nature and quality of land, whether irrigated or unirrigated, facilities for
irrigation like existence of well etc., presence of fruit bearing trees, the location of the
land, closeness to any road or highway, the evenness of the land, namely, whether its
level is even or there are pits etc., its position in rainy season,
@page-SC941
namely, whether water gets accumulated in rains, existence of any building or structure
and a host of factors having bearing on valuation of the land.
9. The High Court has not at all adverted to even a single piece of evidence and there is
absolutely no indication in the judgment as to how it has come to a conclusion that the
findings recorded by the reference Court require to be affirmed.
10. There being total non-compliance of the mandatory provisions of Order XLI, Rule 31,
CPC we have no option but to set aside the judgment of the High Court and remand the
matter to the High Court for fresh consideration of the appeals.
11. In the result, the appeals are allowed and the judgment and decree dated 1-9-1999 of
the High Court is set aside. The appeals (both by the landholders and also by the Union of
India) are remitted to the High Court for fresh decision in accordance with law. Parties to
bear their own costs.
CIVIL APPEAL Nos. of 2007
@ S. L. P. (C Nos. 740-41 of 2004
12. These appeals, by special leave, have been preferred against the judgment and decree
dated 27-6-2000 of Madhya Pradesh High Court.
13. The State Government issued notifications under Sections 4(1) and 6(1) of the Land
Acquisition Act, 1894 (hereinafter referred to as the 'Act') for acquiring large area of land
for a public purpose. After receipt of the notice under Section 9 of the Act the landholders
filed objections. The Collector, after consideration of the objections, made an Award
awarding compensation @ Rs. 12,000/- per hectare for dry land and Rs. 18,000 per
hectare for irrigated land. Feeling aggrieved by the Award the landholders asked for a
reference under Section 18 of the Act. The reference Court enhanced the compensation
and awarded Rs. 22,000/- per hectare for dry land and Rs. 33,000 per hectare for irrigated
land. Against the Award of reference Court the landholders preferred appeals before the
High Court under Section 54 of the Act. The High Court decided the appeals by
extremely short and cryptic order and the entire judgment passed by the Division Bench
of the High Court, comprising Justice R. D. Vyas and Justice Shambhoo Singh, is
reproduced below :-
"These appeals arise out of the similar acquisition proceedings as in the case of First
Appeal No. 254/97 and the group decided by this Court in the Division Bench consisting
of Justice B. A. Khan and Justice Shambhoo Singh on 1-9-1999 are taken together. First
appeal No. 134/95 and First Appeal No. 223/96 are taken up for hearing today itself at the
request of the parties and disposed of along with the group.
The compensation will be governed by the principle and amount decided in the aforesaid
judgment of this Court meaning thereby that the unirrigated lands will be paid Rs.
58,000/- per hectare and the irrigated land Rs. 88,000/- per hectare as confirmed in the
said judgment. The appeals are accordingly disposed of. The Court-fees deposited in the
appeal filed by the claimants/land holders shall be returned to them as per the directions
in the aforesaid judgment. A copy of this judgment be placed in the connected appeal."
14. The appellant State of Madhya Pradesh preferred a review petition against the
aforesaid judgment and decree dated 27-6-2000. In the review petition the High Court
took the view that the case did not fall within the purview of Order 47, Rule 1 of the
Code of Civil Procedure and accordingly dismissed the same by a short order of eight
lines.
15. Since in the present case the High Court has followed the judgment and decree dated
1-9-1999, which has been quoted above and since we have set aside the said judgment
and have remanded the matter to the High Court for fresh consideration, the judgments
and decrees under challenge in the present appeals have also to be set aside.
16. The appeals are accordingly allowed and the judgment and decree dated 27-6-2000
and also the order passed in the review petition dated 22-3-2002 are set aside and the
appeals are remitted to the High Court for a fresh consideration in accordance with law.
Parties to bear their own costs.
Appeals allowed.
@page-SC942
AIR 2008 SUPREME COURT 942 "Rajesh Ranjan Yadav v. CBI through its Director"
Coram : 2 S. B. SINHA AND H. S. BEDI, JJ.
Criminal M.P. No. 9066 and 11845 of 2007 in Cri. Appeal No. 1172 of 2006, D/- 30 -11
-2007.
Rajesh Ranjan Yadav @ Pappu Yadav v. CBI through its Director.
(A) Criminal P.C. (2 of 1974), S.439 - BAIL - HIGH COURT - Bail - Accused charged of
serious offences - Bail sought on ground that because of death of his father nobody is left
to pursue trial - Not tenable particularly in view of nature of charges against him. (Para
2)
(B) Criminal P.C. (2 of 1974), S.439 - BAIL - TRIAL - Bail - Delay in completing trial -
Material adduced however showing that accused and not prosecution was more
responsible for delay - Bail not liable to be granted - Accused held cannot also be
released on health ground. (Paras 6, 7)
Cases Referred : Chronological Paras
2006 AIR SCW 5853 : AIR 2007 SC 451 : 2007 Cri LJ 304 : 2007 (1) AIR Jhar R 761
3
Rakesh Kumar Singh, Jitendra Kumar, Prem Malhotra, for Appellant; A. Sharan, ASG,
Amit Anand Tiwari, P. Parmeswaran, for Respondent.
Judgement
1. HARJIT SINGH BEDI, J. :-This application for bail has been filed directly in this
Court on the following grounds :
1) that the appellant has been in custody for more than seven years and that his conduct in
jail has been exemplary;
2) that on account of the death of his father, there is nobody available to him to pursue the
present case,
3) that no inculpatory evidence has come on record justifying his continued incarceration,
4) despite the orders of this Court from time to time, the trial was no where near
completion and, finally,
5) that his medical condition required sophisticated life saving treatment which was only
possible outside jail.
2. We are of the opinion that in the light of the facts that several bail applications filed by
the appellant raising almost similar issues have been rejected no case for release on bail is
made out. We are also of the opinion that the demise of the appellant's father also does
not ipso facto mean that he should be released on bail more particularly on account of the
serious charges against him. We are therefore left with the last two points for
consideration.
3

. Mr. Rakesh Kumar Singh, the learned counsel for the appellant has very strenuously
urged that despite the directions of this Court in Rajesh Ranjan Yadav @ Pappu Yadav vs.
CBI through its Director, (2007) 1 SCC 70 while dismissing one of the bail applications
filed by the appellant that the trial court was to ensure that the defence witnesses were
examined on a day-to-day basis in accordance with a fixed time schedule so that the trial
was completed as expeditiously as possible and the judgment delivered, the defence
evidence had so far not been completed on account of the delaying tactics on the part of
the CBI and it was therefore appropriate that the appellant be released on bail. It has also
been pointed out that a direction had also been issued that as the appellant was lodged in
Tihar Jail in Delhi and the trial was being conducted in Patna, video conference facilities
be provided to the appellant in order to enable him oversee the proceedings in the trial but
the said facilities were not being made available to him as the equipment had been
damaged. It has also been argued that as the appellant was grossly overweight, he was
required to undergo some invasive surgical process which required special care and
nursing which could not be made available while the appellant remained in custody.
Several documents in support of the appellant's medical condition have been handed over
to us in Court. 2006 AIR SCW 5853

4. In reply a counter affidavit on behalf of the CBI has been filed and Mr. A. Sharan,
learned ASG has drawn our attention to the enclosures appended therewith to submit that
the delay, if any, in the completion of the trial was on account of repeated applications
filed by the appellant in the trial court asking for one or other information or the recall of
witnesses and as such it did not lie in him to state that the trial was being inordinately
delayed. He has also pointed out that the CBI had completed its evidence on 7.6.2006 and
that a list of 43 defence witnesses had been given by the appellant of whom only a few
had been examined and the case had been adjourned time and again at the instance of the
accused or to secure
@page-SC943
the presence of the remaining defence witnesses. He has also submitted that in the light of
Sections 273 and 317 of Cr.P.C the trial could go on even if an accused was not
personally present and as such directions should be given by this Court that
notwithstanding the fact that the video conference facility was out of order the court
should go ahead and complete the trial. He has also pleaded that the appellant had been
referred to arguably the best medical facility in Delhi i.e. All India Institute of Medical
Sciences (AIIMS) and that all medical aid would be provided to him as per his needs.
5. We have heard learned counsel for the parties and gone through the record very
carefully. In the cited case it has been observed that the appellant had filed bail
applications ad nauseam in the High Court and in this Court and this amounted to a
misuse of the legal process and it had accordingly been ordered that no further bail
application on his behalf be entertained by any Court. An application for review was
thereafter filed in the aforesaid matter and was allowed on 27.4.2007 only to the extent
that "in the event any occasion arises, the petitioner may move this Court for grant of
bail". The present application filed within a month of that date, is yet another in
continuation of the series of applications raising almost identical issues which have
already been rejected by this Court. However, as some additional points have been raised,
we must deal with them as well. It is clear from the orders that have been put on record
and the additional counter affidavit on behalf of the CBI sworn by Sh. Pyare Lal Meena,
Additional Superintendent of Police CBI, that the defence evidence had not been
completed because the defence had often sought adjournments or the defence witnesses
had not been present. We find from a perusal of the -imni orders of the trial court from
2.5.2007 to 20.9.2007 that the defence has been procrastinating in the matter and not
permitting the defence evidence to proceed to its conclusion. It is true that on a few
occasions the trial had been adjourned on account of the non-availability of the video
conference facility whereas the record reveals that the adjournments had largely been
sought either by the co-accused Anil Kumar Yadav or the appellant, on one pretext or the
other. It is also clear that several miscellaneous applications have been filed by the
appellant praying for a recall of witnesses and as they have been rejected the matters are
in the High Court by way of appeal/revision.
6. Mr. Rakesh Kumar Singh, the learned counsel for the appellant has however submitted
that the appellant was only exercising his legal rights in accordance with law and could
therefore not be faulted on that account. We agree with the learned counsel to the extent
that the appellant was fully justified in exercising his legal rights but it does not then
behove him to say that the trial was being unduly delayed. On the other hand, as has
already been noted above, adjournments have been taken time and again for the
completion of the defence evidence whereas Mr. Sharan has, on the contrary, made a
statement that the CBI would complete its arguments within a week of the
commencement thereof.
7. We have also carefully gone through the appellant's medical papers that have been
produced before us in court. We are of the opinion that they do not as of now justify his
release on bail even on medical grounds the moreso as all facilities are being made
available to him by the jail authorities. We accordingly dismiss the application but while
doing so issue the following directions :
1) Every effort will be made to provide Video Conference Facilities to the appellant but
in the light of Sections 273 and 317 of the Cr.P.C, the trial will go on to its conclusion
even if they are not available;
2) that in the event that the video conference facilities are available, the appellant would
be allowed access to his lawyers through the aforesaid facility in addition for one hour on
each day that the final arguments in the trial proceed;
3) that the Tihar jail authorities will ensure that all the directions issued by the attending
doctors with respect to the appellant will be observed scrupulously ; and
4) should the appellant's medical condition require further orders from the Courts at a
later stage, he would be at liberty to approach this Court yet again.
Order accordingly.
@page-SC944
AIR 2008 SUPREME COURT 944 "Special Deputy Collector (L. A.) v. N. Vasudeva
Rao"
(From : 2004 (1) Andh WR 418)
Coram : 2 Dr. A. PASAYAT AND LOKESHWAR SINGH PANTA, JJ.
Civil Appeal Nos. 4649-4650 of 2004, D/- 28 -11 -2007.
Special Deputy Collector (L. A.) v. N. Vasudeva Rao and Ors.
(A) Letters Patent (A. P.), Cl.15 - LETTERS PATENT APPEAL - Contempt petition
arising out of order passed by single Judge in writ jurisdiction - Letters Patent Appeal
against order passed in contempt proceedings - Maintainable.
2004 (1) Andh WR 418, Reversed.
2006 AIR SCW 2766, Foll. (Para 13)
(B) Constitution of India, Art.141 - PRECEDENT - HIGH COURT - JUDICIAL
DISCIPLINE - Judicial discipline - High Court distinguishing judgment of Apex Court
on ground that there was no elaborate discussion and therefore, no reason is discernible -
It is clearly violative of judicial discipline - Alleged distinguishing feature cannot be said
to be graceful. (Para 15)
(C) Contempt of Courts Act (70 of 1971), S.14 - Constitution of India, Art.215 -
CONTEMPT OF COURT - HIGH COURT - Resumption of land by Govt. - Contempt
petition arising out of order passed in writ jurisdiction to pay ex-gratia amount - Dispute
about area of land for which ex-gratia was payable - Held, no direction could be given in
contempt proceedings regarding ex-gratia payment for entire extent of land.
2004 AIR SCW 4742, 2006 AIR SCW 342 and 2000 (10) SCC 285, Foll. (Para 13)
Cases Referred : Chronological Paras
2006 AIR SCW 342 : AIR 2006 SC 909 : 2006 Cri LJ 971 : 2006 (2) AIR Jhar R 78
(Foll.) 11
2006 AIR SCW 2766 : AIR 2006 SC 2190 : 2006 Cri LJ 2903 : 2006 (4) AIR Bom R 322
(Rel. on) (Pt. A) 13
2004 AIR SCW 4742 : AIR 2004 SC 4277 : 2004 Cri LJ 4848 : 2004 AIR-Jhar R 2593
(Foll.) 12
2000 AIR SCW 4675 (Ref.) 11
(2000) 10 SCC 285 (Foll.) 14, 15
1994 AIR SCW 4481 : AIR 1995 SC 308 (Ref.) 11
R. Sundervardhan, Sr. Advocate, Manoj Saxena, Rajnish Kr. Singh, Rahul Shukla and T.
V. George, for Appellant; A. Subba Rao, for Respondents.
Judgement
Dr. ARIJIT PASAYAT, J. :- Challenge in these appeals is to the order passed by a
Division Bench of the Andhra Pradesh High Court disposing of four Letters Patent
Appeals i.e. LPA Nos. 184 and 185 of 2002 and 33 and 34 of 2003, filed by the
appellants.
2. Background facts need to be noted in brief before dealing with the rival contentions.
3. The land in question to an extent of 24 acres 82 cents is Government land which was
said to have been assigned to the respondents herein on the basis of D Form pattas. As per
the terms and conditions of the pattas, whenever the land is required for any public
purpose, the same can be resumed by the Government on payment of certain ex-gratia
amount.
4. The concerned Executive Engineer SRBC Division, Koilakuntla is said to have sent
proposals for acquiring about 24 acres 82 cents that is the land in question situated in
Cherlopalli village of Owk Mandal. Accordingly, the Revenue authorities surveyed the
land and arrived at the actual extent of land involved to be only 20 Acres 75 cents and the
said land is Government land and therefore proposal for resumption of the land was said
to have been initiated.
5. Respondents herein filed W.P. Nos. 6511 of 1999 and W.P. No. 6513 of 1999, inter alia,
contending that the appellants have resumed their land without paying ex- gratia amounts
in terms of GOMs. No. 1307 dated 23.12.1993.
6. Learned Single Judge by common judgment and order dated 11.8.1999 in Writ Petition
Nos. 6511 and 6513 of 1999 disposed of the writ petitions directing the respondents
herein to make a detailed representation to the authorities within four weeks and the
authorities were directed to consider the same and pass appropriate order within a period
of six weeks. Subsequently, respondents herein filed Contempt Case No. 493 of 2001 and
Contempt Case No. 1211 of 2001 before the High Court inter alia alleging that despite
court's order the ex-gratia payment was not made. The appellants filed detailed counter
affidavits in the said contempt petitions inter alia indicating that the land in question was
not resumed and out of the alleged land in question i.e. out of Acre 24.82 cents, soil was
stated to have been
@page-SC945
excavated only in Acre 2.40 cents and as such they are not entitled to any ex-gratia
payment in respect of the entire extent.
7. A learned Single judge of the High Court on consideration of the matter by a common
order dated 11.9.2002 in the two contempt cases held that the department has resumed the
land in question, exonerated the concerned officer and directed payment of ex-gratia
payment in terms of GOMs. No. 1307 dated 23.12.1993 for the entire extent of the land.
Appellant filed LPA Nos. 184 and 185 of 2002 before the High Court against the order
passed by learned Single Judge. The other LPA No. 33 of 2003 was filed by the
Commissioner of Municipality, Tuni. The third parties filed LPA No. 34 of 2003 along
with a Misc. Petition seeking permission of the Court to condone delay in filing of the
LPA against the order. They also filed another Misc. petition to direct the Municipality to
deliver possession of the shops as per the terms and conditions of the auction. These two
appeals have been filed by the functionaries of the State Government against the
combined order of the High Court in the Letters Patent Appeal.
8. Stand of the appellant in these appeals is as follows :
9. Primarily, it is contented that the learned Single Judge has no jurisdiction to give any
direction in the manner done while dealing with the contempt petition. In any event, a
learned Single Judge has no jurisdiction as his order merging to the order of Division
Bench. Finally it is submitted that the LPA was not maintainable.
10. Learned counsel for the respondents on the other hand submitted that there was clear
violation of the order passed in the writ petitions and there was blatant attempt to deny
the legitimate claim of the respondents herein. The land was resumed on 18.11.1998 and
till now nothing has been paid to the respondent as compensation. Reference has been
made to several correspondences between Municipal Revenue Officer Owk Mandalam
and the Special Deputy Collector, Nandyal to show that the lands of the respondents were
resumed.
11

. The law as to nature of order that can be passed in contempt proceedings had been
elaborately dealt with by this Court in several cases. In Union of India and Ors. v.
Subedar Devassy PV [2006(1) SCC 613] it was held as follows : 2006 AIR SCW 342

"2. While dealing with an application for contempt, the court is really concerned with the
question whether the earlier decision which has received its finality had been complied
with or not. It would not be permissible for a court to examine the correctness of the
earlier decision which had not been assailed and to take a view different from what was
taken in the earlier decision. A similar view was taken in K. G. Derasari v. Union of India
[(2001) 10 SCC 496]. The court exercising contempt jurisdiction is primarily concerned
with the question of contumacious conduct of the party who is alleged to have committed
default in complying with the directions in the judgment or order. If there was no
ambiguity or indefiniteness in the order, it is for the party concerned to approach the
higher court if according to him the same is not legally tenable. Such a question has
necessarily to be agitated before the higher court. The court exercising contempt
jurisdiction cannot take upon itself the power to decide the original proceedings in a
manner not dealt with by the court passing the judgment or order. Though strong reliance
was placed by learned counsel for the appellants on a three-Judge Bench decision in Niaz
Mohd. v. State of Haryana [(1994) 6 SCC 332], we find that the same has no application
to the facts of the present case. In that case the question arose about the impossibility to
obey the order. If that was the stand of the appellants, the least it could have done was to
assail correctness of the judgment before the higher Court." 2000 AIR SCW 4675
1994 AIR SCW 4481

12

. The above position was earlier highlighted in Prithawi Nath Ram v. State of Jharkhand
[(2004) 7 SCC 261]. 2004 AIR SCW 4742

13

. It appears that there is also dispute about the area, so in the contempt petition no
direction could have been given in the manner done. The Division Bench has held that
the LPA is not maintainable. In view of what has been stated in Midnapore Peoples'
Coop. Bank Ltd. and Ors. v. Chunilal Nanda and Others [2006 (5) SCC 399], the LPA
was clearly maintainable. 2006 AIR SCW 2766

14. In Lalith Mathur v. L. Maheswara Rao [2000(10) SCC 285] it was inter alia held as
follows :
"3. The above will show that the High Court has directed the State Government to
@page-SC946
absorb the respondent against a suitable post either in a government department or in any
public sector undertaking. This order, in our opinion, is wholly without jurisdiction and
could not have been made in proceedings under the Contempt of Courts Act or under
Article 215 of the Constitution."
15. Reliance was placed on two Division Bench Judgments holding that contempt
petition was not maintainable before Learned Single Judge as his order had merged with
the Division Bench order. As regards Lalith Mathur's case (supra), the High Court
distinguished the judgment on the ground that there was no elaborate discussion in the
judgment and therefore no reason is discernible. To say the least, the alleged
distinguishing feature as pointed out by the High Court not to follow the judgment cannot
be said to be graceful. It is clearly violative of the judicial discipline. It has been stated
that payments have been made to some persons and no departure could be made in the
present case. Actually there is no definite material as to whether the land was resumed or
it was an excavated land.
16. It appears from record that three counter affidavits have been filed and one of the
basic issues was whether the land was resumed or excavated land. There is no definite
material in this regard brought by the respondents on record. Three counter affidavits
filed by the respondents clearly indicate their definite stand. Neither learned Single Judge
nor the Division Bench addressed the basic issues and on the other hand came to abrupt
conclusions. Therefore, the orders passed by learned Single Judge and the Division
Bench deserve to be set aside, which we direct. The authorities shall however consider
the matter in detail and record findings keeping in view the GO, the factual position and
evidence led before it. The appeals are accordingly disposed of without any order as to
costs.
Order accordingly.
AIR 2008 SUPREME COURT 946 "Sarva Shramik Sanghatana (K. V.), Mumbai v. State
of Maharashtra"
(From : 2007 (5) Bom CR 498)
Coram : 2 C. K. THAKKER AND MARKANDEY KATJU, JJ.
Civil Appeal No. 5458 of 2007 (arising out of SLP (C) No. 15199 of 2007), D/- 28 -11
-2007.
Sarva Shramik Sanghatana (K.V.), Mumbai v. State of Maharashtra and Ors.
(A) Industrial Disputes Act (14 of 1947), S.25 - INDUSTRIAL DISPUTE - CLOSURE
OF UNDERTAKING - Closure of undertaking - Application for - Withdrawal of, on
receiving letter from Deputy Labour Commissioner calling for meeting of parties so that
effort could be made for amicable settlement - Withdrawal is bona fide - Not for Bench
hunting.
It often happens that during the hearing of a petition the Court makes oral observations
indicating that it is inclined to dismiss the petition. At this stage the counsel may seek
withdrawal of his petition without getting a verdict on the merits, with the intention of
filing a fresh petition before a more convenient bench. (Para 14)
Where the application for withdrawal of the first petition under S. 25-O(1) was made
because the company had received a letter from the Deputy Labour Commissioner calling
for a meeting of the parties so that an effort could be made for an amicable settlement and
Company did not wait for the expiry of 60 days from the date of filing of its application
under S. 25-O(1), on the expiry of which the application would have deemed to have
been allowed under S. 25O(3), it was held that application for withdrawal was bona fide,
and that it was not a case of bench hunting - It was not a case where withdrawal
application was made when it was found that an adverse order was likely to be passed
against it. (Para 20)
(B) Industrial Disputes Act (14 of 1947), S.25-O - INDUSTRIAL DISPUTE -
WITHDRAWAL OF SUIT - CLOSURE OF UNDERTAKING - RES JUDICATA -
Application for permission to close down undertaking - Withdrawal - Provisions of O. 23,
R. 1 (4) CPC do not apply.
Civil P.C. (5 of 1908), S.11, O.23, R.1(4).
Although Civil P C does not strictly apply to proceedings under S. 25-O (1) of the
Industrial Disputes Act, or other judicial or quasi-judicial proceedings under in any other
Act, some of the general priciples in the CPC may be applicable. For instance, even if S.
11 of the CPC does not in terms strictly apply because both the proceedings may not be
suits, the general principle of res judicata apply. However, this does not mean that all
provisions in the CPC will strictly apply to proceedings, which are not suits. Thus, an
application under S. 25-O(1)
@page-SC947
is not a suit, and hence, the said provision will not apply to such an application under S.
25. (Paras 21, 24)
(C) Industrial Disputes Act (14 of 1947), S.25-O (5) - INDUSTRIAL DISPUTE -
CLOSURE OF UNDERTAKING - Application for closure of establishment -
Withdrawal, of - Order permitting withdrawal - No order on merits was passed - Second
application maintainable - S. 25 (O) (5) providing that order of grant or refusal to grant
permission shall be final, has no application. (Para 26)
Cases Referred : Chronological Paras
2005 AIR SCW 6314 : AIR 2006 SC 586 : 2006 (1) ALJ 577 (Disting.) (Pt. B) 22
2004 AIR SCW 5457 : AIR 2004 SC 4778 (Ref.) 18
2003 AIR SCW 5725 : AIR 2003 SC 4701 : 2003 Lab IC 3709 (Foll.) (Pt. B) 21
2002 AIR SCW 4939 : AIR 2003 SC 511 (Ref.) 17
AIR 1987 SC 88 (Disting.) 10, 11, 13, 14, 19, 20
AIR 1987 SC 1073 (Ref.) 16
(1972) 2 WLR 537 18
(1971) 1 WLR 1062 18
(1970) 2 All ER 294 18
(1951) AC 737 18
(1901) AC 495 (Foll.) 15
(1898) AC 1 15
K. K. Singhvi, Sr. Advocate, Susheel Madhadeshwar, Uday B. Dube and Kuldip Singh,
for Appellant; K. K. Venugopal, Sr. Advocate, Gopal Jain, Mrs. Manik Karanjawala, Ms.
Nandini Gore, Debmalya Banerjee, Ms. Prachi Goel, R. N. Karanjawala, Ms. Haripriya
Padmanabhan, for Respondents.
Judgement
1. MARKANDEY KATJU, J. :-Leave granted.
2. This appeal has been filed against the impugned judgment dated 16.8.2007 passed by
the High Court of Bombay in Writ Petition No. 1240 of 2007.
3. Heard learned counsel for the parties and perused the record.
4. Respondent No. 3, Century Industries Textiles Limited, is a company registered under
the Indian Companies Act, 1956. It had about 7500 employees in its textile mill at
Mumbai which suffered heavy loss due to high increase in the cost of production and
competition both in the domestic as well as international market. With the object to
reduce its operational cost, agreements dated 6.7.2004 and 5.9.2005 were entered into by
the company with its recognized union for reducing the workforce through an offer of
Voluntary Retirement Scheme (hereinafter in short 'VRS'). However, there was hardly
any success in this exercise, and only about 800 employees opted for the VRS which left
with 6700 employees still on its roll. Finally, a highly upgraded VRS was offered to the
employees unilaterally by the respondent-company on 13.11.2006 which offer was valid
till 12.12.2006. There was an overwhelming response to the said VRS and more than
6300 employees opted for the new VRS, and were accordingly relieved from service on
payment of VRS benefits and all other legal dues. Only about 275 employees did not
accept the abovementioned VRS and 230 of these were the petitioners before the High
Court.
5. The respondent-company further alleged that its manufacturing activities in its textile
mill came to an end on 13.12.2006 since it was left with only 275 workers. All
supervisors and departmental heads had left after taking the VRS. In these circumstances,
the respondent-company was constrained to file an application seeking permission for
closure under Section 25-O of the Industrial Disputes Act (hereinafter in short 'the Act')
vide application dated 13.2.2007.
6. Before the aforesaid application under Section 25-O could be decided, the respondent-
company received a letter dated 5.4.2007 from the Deputy Commissioner of Labour,
Mumbai, a copy of which is Annexure P-1 to this appeal. This letter states that as per the
directions of the Hon'ble Minister for Labour, Maharashtra Government, a meeting has
been convened for discussing the matter in dispute at 11.00 a.m. on 9.4.2007 in the
Chambers of the Hon'ble Minister in Vidhan Bhavan.
7. In response, the respondent-company wrote a letter to the Hon'ble Minister for Labour
dated 11.4.2007 stating that it was willing to discuss the matter in dispute and would
attend the meeting. However, in the same letter dated 11.4.2007 the respondent-company
also mentioned that under Section 25-O(3) of the Industrial Disputes Act, an application
under Section 25-O(1) has to be decided within 60 days, otherwise it would be deemed to
have been allowed. Since the application was made on 13.2.2007, the 60
@page-SC948
days' limitation was shortly about to expire and then the application would be deemed to
have been allowed. However, in order to create a conducive atmosphere for discussing
the problems of the remaining employees who had not taken VRS, the respondent-
company was withdrawing its application under Section 25-O(1), but reserving its right
to move fresh application under Section 25-O as and when necessary. Accordingly, the
Commissioner of Labour, Mumbai by his order dated 12.4.2007 allowed the respondent-
company to withdraw its application under Section 25-O (1). The respondent-company
alleged that it could have very easily pretended to discuss the matter with the workers'
Union and bided its time till 13.4.2007 and then claimed the benefit of deemed grant of
permission for closure. But, instead of doing so, the respondent-company decided to bona
fide explore the possibility of an overall settlement with the remaining employees. Since
that could not have been done within the remaining 4 days, the respondent-company
withdrew its application under Section 25-O(1) so that an attempt for settlement could be
made. Thus, the respondent-company alleged that its conduct was bona fide in seeking
withdrawal of its closure application.
8. It appears, however, that the effort for an amicable settlement failed. Hence the
respondent-company filed fresh application under Section 25-O(1) on 11.5.2007 before
the Commissioner of Labour, Mumbai.
9. The appellant, which represents the workmen concerned, opposed the very
entertainment of the second closure application under Section 25-O on the ground that
the first application was withdrawn but without liberty from the concerned authority to
file a fresh application. The appellant filed a writ petition under Article 226 of the
Constitution before the Bombay High Court praying that the Deputy Commissioner of
Labour should be directed not to take any further proceedings in relation to the closure
application dated 11.5.2007 under Section 25-O. Since that writ petition was dismissed,
hence this appeal by way of Special Leave Petition.
10. Learned counsel for the appellant has strongly relied on the decision of this Court in
Sarguja Transport Service vs. State Transport Appellate Tribunal, Gwalior and others,
AIR 1987 SC 88. He has submitted that in that decision this Court has laid down that if a
writ petition filed in a High Court is withdrawn without permission to file a fresh writ
petition, a second writ petition for the same relief is barred. Learned counsel for the
appellant submitted that in the order of the Labour Commissioner dated 12.4.2007, a
copy of which is Annexure P-4 to this appeal, it is only mentioned that the applicant
company is allowed to withdraw its application under Section 25-O(1) seeking
permission for closure of its textile mill, but there is no mention in the said order that the
Company is given liberty or permission to file a fresh application under Section 25-O(1).
Accordingly, he submitted that the decision of Sarguja Transport case (supra) squarely
applies to the present case. He submitted that although the decision in Sarguja Transport
case (supra) related to a writ petition, the ratio of that decision was based on public
policy, and hence it was also application to proceedings under Section 25-O of the
Industrial Disputes Act.
11. We have carefully examined the decision of the Sarguja Transport Service case
(supra). In the said decision it is mentioned in paragraph 8 as follows :
"It is common knowledge that very often after a writ petition is heard for some time when
the petitioner or his counsel finds that the Court is not likely to pass an order admitting
the petition, request is made by the petitioner or by his counsel, to permit the petitioner to
withdraw the writ petition without seeking permission to institute a fresh writ petition. A
Court which is unwilling to admit the petition would not ordinarily grant liberty to file a
fresh petition while it may just agree to permit the withdrawal of the petition."
12. In paragraph 9 of the said decision, it is also mentioned as follows:
"But we are of the view that the principle underlying R.1 of O. XXIII of the Code should
be extended in the interest of administration of justice to cases of withdrawal of writ
petition also, not on the ground of res judicata but on the ground of public policy as
explained above. It would also discourage the litigant from indulging in bench-hunting
tactics.
13. We are of the opinion that the decision in Sarguja Transport case (supra) has to be
understood in the light of the observations in paragraphs 8 and 9 therein, which have
been quoted above. The said decision
@page-SC949
was given on the basis of public policy that, if while hearing the first writ petition the
Bench is inclined to dismiss it, and the learned counsel withdraws the petition so that he
could file a second writ petition before what he regards as a more suitable or convenient
bench, then if he withdraws it he should not be allowed to file a second writ petition
unless liberty is given to do so. In other words, bench-hunting should not be permitted.
14. It often happens that during the hearing of a petition the Court makes oral
observations indicating that it is inclined to dismiss the petition. At this stage the counsel
may seek withdrawal of his petition without getting a verdict on the merits, with the
intention of filing a fresh petition before a more convenient bench. It was this malpractice
which was sought to be discouraged by the decision in Sarguja Transport case (supra).
15. On the subject of precedents Lord Halsbury, L.C., said in Quinn v. Leathem, 1901 AC
495 :
"Now before discussing the case of Allen v. Flood (1898) AC 1 and what was decided
therein, there are two observations of a general character which I wish to make, and one
is to repeat what I have very often said before, that every judgment must be read as
applicable to the particular facts proved, or assumed to be proved, since the generality of
the expressions which may be found there are not intended to be expositions of the whole
law, but are governed and qualified by the particular facts of the case in which such
expressions are to be found. The other is that a case is only an authority for what it
actually decides. I entirely deny that it can be quoted for a proposition that may seem to
follow logically from it. Such a mode of reasoning assumes that the law is necessarily a
logical Code, whereas every lawyer must acknowledge that the law is not always logical
at all."
We entirely agree with the above observations.
16

. In Ambica Quarry Works vs. State of Gujarat and others (1987) 1 SCC 213 (vide
paragraph 18) this Court observed :- AIR 1987 SC 1073, (Para 18)

"The ratio of any decision must be understood in the background of the facts of that case.
It has been said a long time ago that a case is only an authority for what it actually
decides, and not what logically follows from it."
17

. In Bhavnagar University vs. Palitana Sugar Mills Pvt. Ltd. (2003) 2 SCC 111 (vide
paragraph 59), this Court observed :- 2002 AIR SCW 4939

"It is well settled that a little difference in facts or additional facts may make a lot of
difference in the precedential value of a decision."
18

. As held in Bharat Petroleum Corporation Ltd. and another vs. N.R.Vairamani and
another (AIR 2004 SC 4778), a decision cannot be relied on without disclosing the
factual situation. In the same Judgment this Court also observed :- 2004 AIR SCW 5457,
(Para 9)

"Courts should not place reliance on decisions without discussing as to how the factual
situation fits in with the fact situation of the decision on which reliance is placed.
Observations of Courts are neither to be read as Euclid`s theorems nor as provisions of
the statute and that too taken out of the context. These observations must be read in the
context in which they appear to have been stated. Judgments of Courts are not to be
construed as statutes. To interpret words, phrases and provisions of a statute, it may
become necessary for judges to embark into lengthy discussions but the discussion is
meant to explain and not to define. Judges interpret statutes, they do not interpret
judgments. They interpret words of statutes; their words are not to be interpreted as
statutes.
In London Graving Dock Co. Ltd. vs. Horton (1951 AC 737 at page 761), Lord Mac
Dermot observed :
"The matter cannot, of course, be settled merely by treating the ipsissima vertra of Willes,
J. as though they were part of an Act of Parliament and applying the rules of
interpretation appropriate thereto. This is not to detract from the great weight to be given
to the language actually used by that most distinguished judge."
In Home Office vs. Dorset Yacht Co. (1970 (2) All ER 294) Lord Reid said, "Lord
Atkin`s speech ............ is not to be treated as if it was a statute definition; it will require
qualification in new circumstances." Megarry, J. in (1971)1 WLR 1062 observed :
"One must not, of course, construe even a reserved judgment of Russell L. J. as if it were
an Act of Parliament."
@page-SC950
And, in Herrington v. British Railways Board (1972 (2) WLR 537) Lord Morris said :
"There is always peril in treating the words of a speech or judgment as though they are
words in a legislative enactment, and it is to be remembered that judicial utterances are
made in the setting of the facts of a particular case."
Circumstantial flexibility, one additional or different fact may make a world of difference
between conclusions in two cases. Disposal of cases by blindly placing reliance on a
decision is not proper. The following words of Lord Denning in the matter of applying
precedents have become locus classicus :
"Each case depends on its own facts and a close similarity between one case and another
is not enough because even a single significant detail may alter the entire aspect, in
deciding such cases, one should avoid the temptation to decide cases (as said by Cardozo,
J. ) by matching the colour of one case against the colour of another. To decide therefore,
on which side of the line a case falls, the broad resemblance to another case is not at all
decisive."
*** *** ***
"Precedent should be followed only so far as it marks the path of justice, but you must cut
the dead wood and trim off the side branches else you will find yourself lost in thickets
and branches. My plea is to keep the path of justice clear of obstructions which could
impede it."
19. We have referred to the aforesaid decisions and the principles laid down therein,
because often decisions are cited for a proposition without reading the entire decision and
the reasoning contained therein. In our opinion, the decision of this Court in Sarguja
Transport case (supra) cannot be treated as a Euclid's formula.
20. In the present case, we are satisfied that the application for withdrawal of the first
petition under Section 25-O(1) was made bona fide because the respondent-company had
received a letter from the Deputy Labour Commissioner on 5.4.2007 calling for a
meeting of the parties so that an effort could be made for an amicable settlement. In fact,
the respondent-company could have waited for the expiry of 60 days from the date of
filing of its application under Section 25-O(1), on the expiry of which the application
would have deemed to have been allowed under Sec-tion25?O(3). The fact that it did not
do so, and instead applied for withdrawal of its application under Section 25-O(1), shows
its bona fide. The respondent-company was trying for an amicable settlement, and this
was clearly bona fide, and it was not a case of bench hunting when it found that an
adverse order was likely to be passed against it. Hence, Sarguja Transport case (supra) is
clearly distinguishable, and will only apply where the first petition was withdrawn in
order to do bench hunting or for some other mala fide purpose.
21

. We agree with the learned counsel for the appellant that although the Code of Civil
Procedure does not strictly apply to proceedings under Section 25-O(1) of the Industrial
Disputes Act, or other judicial or quasi-judicial proceedings under in any other Act, some
of the general principles in the CPC may be applicable. For instance, even if Section 11
of the CPC does not in terms strictly apply because both the proceedings may not be
suits, the general principle of res judicata may apply vide Pondicherry Khadi and
Village Industries Board vs. P. Kulo-thangan and another 2004 (1) SCC 68. However, this
does not mean that all provisions in the CPC will strictly apply to proceedings which are
not suits. 2003 AIR SCW 5725
22

. Learned counsel for the appellant has relied on an observation in the decision of this
Court in U.P. State Brassware Corporation Ltd. vs. Uday Narain Pandey 2006(1) SCC
479, in paragraph 38 of which it is stated: 2005 AIR SCW 6314, (Para 41)

"Order 7 Rule 7 of the Code of Civil Procedure confers powers upon the court to mould
relief in a given situation. The provisions of the Code of Civil Procedure are applicable to
the proceedings under the Industrial Disputes Act.
23. It may be noted that the observation in the aforesaid decision that the provisions of
the CPC are applicable to proceedings under the Industrial Disputes Act was made in the
context of Order 7 Rule 7 of the Code of Civil Procedure which confers powers upon the
Court to mould relief in a given situation. Hence, the aforesaid observation must be read
in its proper context, and it cannot be interpreted to mean that all the provisions of the
CPC will strictly apply to proceedings under the Industrial Disputes Act.
@page-SC951
24. No doubt, Order XXIII, Rule 1(4), CPC states that where the plaintiff withdraws a
suit without permission of the court, he is precluded from instituting any fresh suit in
respect of the same subject-matter. However, in our opinion, this provision will apply
only to suits. An application under Section 25-O(1) is not a suit, and hence, the said
provision will not apply to such an application.
25. Learned counsel for the appellant has relied upon Section 25-O (5) of the Act which
states :
"An order of the State Government granting or refusing to grant permission shall, subject
to the provisions of sub-section (6), be final and binding on all the parties and shall
remain in force for one year from the date of such order."
26. Leaned counsel submitted that the order of the Labour Commissioner dated 12.4.2007
allowing the respondent-company to withdraw its closure application dated 1.2.2007
should be deemed to be an order refusing to grant permission, and hence a fresh
application under Section 25-O(1) could not be filed before the expiry of one year from
the date of the said order. We do not agree. In our opinion, Section 25-O(5) only applies
when an order is passed on merits either granting or refusing to grant permission for
closure. Since in the present case no order on merits was passed, but only an order
permitting withdrawal of the closure application was passed, Section 25-O(5) has no
application.
27. For the reasons given above this appeal fails and is hereby dismissed. There shall be
no order as to costs.
28. Since it has been alleged by the respondent-company that it is suffering a liability of
Rs. 2.84 lakhs per day although the Mill is lying closed and the concerned workers are
getting wages for doing nothing for a long time, we direct that the petitioner's application
dated 11.5.2007 be decided very expeditiously by the concerned authority in accordance
with law, preferably within a period of two months of production of copy of this order to
it.
Appeal dismissed.
AIR 2008 SUPREME COURT 951 "Kannan v. V. S. Pandurangam"
(From : Madras)
Coram : 2 A. K. MATHUR AND MARKANDEY KATJU, JJ.
Civil Appeal Nos. 5472-5475 of 2001, D/- 27 -11 -2007.
Kannan (Dead) by L.Rs. and Anr. v. V. S. Pandurangam (Dead) by L.Rs. and Ors.
Civil P.C. (5 of 1908), S.100(4), O.14, R.1 -APPEAL - CIVIL PROCEDURE -
Substantial question of law - Non-formulation - Order passed not always liable to be set
aside - Prejudice to party necessary for order to be set aside. (Paras 9, 13, 14)
Cases Referred : Chronological Paras
2003 AIR SCW 3680 : AIR 2003 SC 2985 (Rel. on) 12
AIR 1990 SC 553 (Ref.) 6
AIR 1989 SC 1530 (Rel. on) 12
AIR 1986 SC 1889 : 1986 All LJ 1061 (Ref.) 6
AIR 1970 SC 61 (Rel. on) 12
AIR 1963 SC 884 (Rel. on) 11
AI 1957 SC 314 (Ref.) 6
S. Mahendran, for Appellants; Mahabir Singh, Sr. Advocate, P. B. Suresh, Vipin Nair (for
M/s. Temple Law Firm) for Respondents.
Judgement
1. ORDER :- These appeals are directed against the impugned judgment of the Madras
High Court dated 17.8.2000 in Second Appeal Nos. 1601-04/1986.
2. Heard learned counsel for the parties and perused the record.
3. The respondent in these appeals, Pandurangan filed a suit being Original Suit No. 807
of 1982 (OS No. 135 of 1982 at Cuddalore) which was decreed on 20.8.1984 by the trial
court. In that suit the plaintiff alleged that he is the owner of the property in question, and
he prayed for declaration of his title and for a decree of possession against the defendant.
4. Against the judgment and decree of the trial court the appellant herein filed an appeal
which was allowed by the Additional Sub-ordinate Judge, Cuddalore on 30.12.1985. The
First Appellate Court set aside the judgment of the trial court and allowed the appeal and
dismissed the plaintiff's suit, holding that the defendant had acquired title by adverse
possession over the property in dispute.
5. Against the aforesaid decision the plaintiff (respondent herein), filed a second
@page-SC952
appeal which was allowed by the High Court by the impugned judgment dated 17.8.2000.
6. The High Court relying on several decisions held that the ingredients of adverse
possession (nec vi, nec clam, nec precario vide P. Lakshmi Reddy vs. L. Lakshmi Reddy,
AIR 1957 SC 314; Suraj Mal and Another vs. Ram Singh and Others, AIR 1986 SC 1889;
Achal Reddi vs. Ramakrishna Reddiar and Others, AIR 1990 SC 553, etc.) have not been
satisfied by the defendant and hence the plaintiff's suit deserves to be decreed, since
admittedly the plaintiff was the owner of the property in dispute.
7. Learned counsel for the appellant has submitted that no substantial question of law was
framed by the High Court as required by Section 100 (4), C.P.C. Hence he submitted that
the impugned judgment of the High Court deserves to be set aside.
8. It is true that in this case no substantial question of law has been formulated by the
High Court. However, in our opinion, merely because no substantial question of law has
been formulated by the High Court that does not mean that the judgment of the High
Court automatically becomes a nullity or that it must necessarily be set aside by this
Court on that ground alone. The appellant before us must also show prejudice to him on
this account.
9. Learned counsel for the appellant has shown us several decisions of this Court where
the judgments of the High Court in Second Appeal were set aside on the ground that no
substantial question of law had been framed by the High Court as required by Section
100 (4), C.P.C. In our opinion these decisions cannot be said to have laid down any
absolute proposition of law that whenever a second appeal is decided by the High Court
without formulating a substantial question of law that judgment must necessarily be set
aside. In our opinion, the judgment of the High Court should not be set aside on this
ground alone if no prejudice had been caused to the appellant before us on this account.
10. In the present case both the parties knew that the question involved was whether the
defendant (appellant) in this case had been able to prove his title by adverse possession.
Hence the non-framing of a substantial question of law in this case did not prejudice the
appellant at all before the High Court.
11. By a series of decisions of this Court it has been settled that omission to frame an
issue as required under Order XIV, Rule 1, C.P.C. would not vitiate the trial in a suit
where the parties went to trial fully knowing the rival case and led evidence in support of
their respective contentions and to refute the contentions of the other side vide Nedunuri
Kameswaramma vs. Sampati Subba Rao, AIR 1963 SC 884.
12

. In Sayeda Akhtar v. Abdul Ahad, AIR 2003 SC 2985 it was held by this Court that even
if no specific issue has been framed but if the parties were aware of that issue and have
led evidence on it, the Appellate Court should not interfere with the findings of the trial
court. A similar view was taken in Kali Prasad Agarwalla and others v. M/s Bharat
Coking Coal Limited and others, 1989 Supp (1) SCC 628 (vide paragraph 19) and in
Shaikh Mahamad Umarsaheb v. Kadalaskar Hasham Karimsab and others AIR 1970 SC
61( vide paragraph 9) as well as in several other decisions. 2003 AIR SCW 3680
AIR 1989 SC 1530

13. In the present case, the parties knew well that the question of adverse possession has
been pleaded by the defendant appellant and evidence was led on this issue. Hence no
prejudice has been caused to the appellant by non-framing of a substantial question of
law by the High Court. In our opinion, the ratio of the decisions on Order XIV, Rule 1,
C.P.C. will also apply when a judgment of the High Court is challenged on the ground
that a substantial question of law was not formulated by the High Court as required by
Section 100 (4), C.P.C. In our opinion, this Court should not take an over technical view
of the matter to declare that every judgment of the High Court in Second Appeal would
be illegal and void, merely because no substantial question of law was formulated by the
High Court. Such an over technical view would only result in remitting the matter to the
High Court for a fresh decision, and thereafter the matter may again come up before us in
appeal. The judiciary is already overburdened with heavy arrears, and we should not take
a view which would add to the arrears.
14. In our opinion, the judgment of the High Court should only be set aside on the ground
of non compliance with Section 100(4) if some prejudice has been caused to the appellant
before us by not formulating
@page-SC953
such a substantial question of law.
15. In the present case, we agree with the view taken by the High Court that the
defendant appellant has not been able to establish that the ingredients of plea of adverse
possession (nec vi, nec clam, nec precario) had been established by the defendant-
appellant. Hence there is no force in these appeals which are accordingly dismissed. No
costs.
16. Normally, we grant six months' time to the tenant to vacate the residential premises
but looking to the fact that the appellants have been in possession of the suit premises for
a long time, therefore, as a special case we grant to the appellants time till 31.12.2008 to
vacate and hand over vacant physical possession of the suit premises subject to the
appellants' filing the usual undertaking before this Court within a period of eight weeks
from today.
Order accordingly.
AIR 2008 SUPREME COURT 953 "Rameshwar Das Agrawal v. Kiran Agrawal"
(From : Allahabad)
Coram : 2 G. P. MATHUR AND P. SATHASIVAM, JJ.
Civil Appeal No. 5366 of 2007 (arising out of SLP (C) No. 1822 of 2006), D/- 23 -11
-2007.
Rameshwar Das Agrawal and Anr. v. Kiran Agrawal and Ors.
(A) Arbitration and Conciliation Act (26 of 1996), S.11(6) - Constitution of India, Art.136
- ARBITRATION AND CONCILIATION - SPECIAL LEAVE APPEAL -
APPOINTMENT - CHIEF JUSTICE - Appointment of arbitrator - Power exercised by
Chief Justice or designated Judge under Section 11 is not an administrative power but is a
judicial power - An appeal would lie against that order only under Article136 of
Constitution. (Para 8)
(B) Arbitration and Conciliation Act (26 of 1996), S.11(6) - ARBITRATION AND
CONCILIATION - APPOINTMENT - CHIEF JUSTICE - Appointment of arbitrator -
Order appointing arbitrator was passed without adverting to claim and objection of both
parties - No reason given for appointing arbitrator - Neither any notice was served on
other respondents nor hearing opportunity was afforded - Impugned order passed by
Chief Justice is liable to be set aside.
2005 AIR SCW 5932, Relied on.
2002 AIR SCW 426, Ref.
A. A. No. 54 of 2003, D/- 9-12-2005 (All), Reversed. (Paras 8, 9)
Cases Referred : Chronological Paras
2005 AIR SCW 5932 : AIR 2006 SC 450 : 2005 CLC 1546 (Rel. on) 5, 8, 10
2002 AIR SCW 426 : AIR 2002 SC 778 : 2002 CLC 478 (Ref.) 8
Manoj Swarup and Ms. Lalita Kohli (for M/s. Manoj Swarup and Co. ), for Appellants;
Jay Savla, Ms. Reena Bagga, Gaurav Agrawal and Ashutosh Lohia, for Respondents.
Judgement
1. P. SATHASIVAM, J. :- Leave granted.
2. This appeal is directed against the order dated 09.12.2005 passed by the Hon'ble Chief
Justice of the High Court of Judicature at Allahabad in Arbitration Application No. 54 of
2003 appointing Hon'ble Mr. Justice Giridhar Malviya, a retired Judge of the Allahabad
High Court as Arbitrator in respect of the dispute between the parties.
3. Respondent Nos. 4 and 5 before the High Court are the appellants in this appeal.
According to them, late Hari Prakash Agrawal (father of respondent No.7) and
Rameshwar Das Agrawal (appellant No.1 herein) were very close relatives and they
decided to carry on business of electronics and electrical goods and other items. They
executed a partnership deed on 15.05.1992 which contains an arbitration clause.
Subsequently, a dispute arose between the members of their two families and by
agreement dated 13.09.2002 signed by the partners, Shri Gopal Goel of Ravindrapuri,
Varanasi was appointed as sole Arbitrator to decide all the disputes concerning the
business. Since the entire disputes between the families were reconciled, fresh Deed of
Partnership reconstituting the three partnership firms were executed on 13.09.2002 and
signed by all the partners and witnessed by the sole Arbitrator - Shri Gopal Goel and one
Shri Vinod Kumar Jindal, one of the advisors to the Arbitrator. This was intimated to the
bank and sales tax authorities. After retirement of Smt. Kiran Agrawal and her husband
Shiv Kumar Agrawal on 13.09.2002, a fresh Retirement Deed was executed on
05.07.2003 which was also duly signed by the parties concerned. Thereafter, first
respondent herein filed an application dated 07.07.2003 under Section 11 of the
Arbitration and Conciliation Act, 1996 (in short the 'Act') for appointment of an
Arbitrator based on clause 21 of the agreement dated 15.05.1992. On 17.10.2003, the
High Court issued notice to all the 8 respondents
@page-SC954
therein. Thereafter, the matter was listed on 09.12.2005 and as per the office report, most
of the respondents had not been served. Shri Rameshwar Das Agrawal, appellant No.1-
herein was represented in the High Court through his counsel and prayed time to file
counter-affidavit. It was also stated that no dispute remained for adjudication. The High
Court, after rejecting the request of the first appellant-herein, by order dated 09.12.2005,
appointed Hon'ble Mr. Justice Giridhar Malviya as an Arbitrator. Aggrieved by the said
order, the appellants preferred this appeal.
4. We heard Mr. Manoj Swarup, learned counsel for the appellants and Mr. Jay Savla and
Mr. Gaurav Agrawal, learned counsel for respondent Nos. 1 and 7 respectively.
Respondent Nos. 2 to 6 though duly served notice not chosen to contest the appeal.
5

. Mr. Manoj Swarup, learned counsel for the appellants placing reliance on a Seven-Judge
Bench decision of this Court in SBP and Co. vs. Patel Engineering Ltd. and another,
(2005) 8 SCC 618, which was pronounced on 26.10.2005, submitted that the decision on
the application under Section 11 of the Act is a judicial pronouncement, the impugned
order of the Hon'ble Chief Justice which does not contain any reason cannot be sustained
and the same deserves to be set aside. He also contended that the Hon'ble Chief Justice,
who passed the impugned order, has not taken care to verify whether notice had been
duly served on all the respondents. He further contended that in any event, the High
Court ought to have granted reasonable time to file their objections. On the other hand,
Mr. Jay Savla and Mr. Gaurav Agrawal, learned counsel for the contesting respondents
submitted that inasmuch as the appellants herein (respondents before the High Court) did
not utilize the ample time provided by the High Court for filing their objection, the
ultimate order of the Hon'ble Chief Justice cannot be faulted with. He also submitted that
there is no violation of the law as declared by this Court. 2005 AIR SCW 5932
6-7. We have carefully perused the relevant materials and considered the rival
submissions.
8

. Before analyzing the claim of both the parties, it is relevant to note that the Hon'ble
Chief Justice of the Allahabad High Court passed the impugned order appointing a retired
Judge of the High Court as an Arbitrator on 09.12.2005. On 26.10.2005, a seven-Judge
Bench of this Court in SBP and Co. v. Patel Engineering Ltd. and another (supra)
reviewed the entire legal position and issued various directions in the matter of
appointment of Arbitrator. The larger Bench has also overruled the earlier decision in
Konkan Railway Corporation Ltd. vs. Rani Construction Private Limited, (2002) 2 SCC
388. It is useful to refer to the conclusions arrived at by the larger Bench which read
thus : 2005 AIR SCW 5932, (Para 46)
2002 AIR SCW 426

"47. We, therefore, sum up our conclusions as follows :


(i) The power exercised by the Chief Justice of the High Court or the Chief Justice of
India under Section 11(6) of the Act is not an administrative power. It is a judicial power.
(ii) The power under Section 11(6) of the Act, in its entirety, could be delegated, by the
Chief Justice of the High Court only to another Judge of that Court and by the Chief
Justice of India to another Judge of the Supreme Court.
(iii) In case of designation of a Judge of the High Court or of the Supreme Court, the
power that is exercised by the designated Judge would be that of the Chief Justice as
conferred by the statute.
(iv) The Chief Justice or the designated Judge will have the right to decide the
preliminary aspects as indicated in the earlier part of this judgment. These will be his own
jurisdiction to entertain the request, the existence of a valid arbitration agreement, the
existence or otherwise of a live claim, the existence of the condition for the exercise of
his power and on the qualifications of the arbitrator or arbitrators. The Chief Justice or
the designated Judge would be entitled to seek the opinion of an institution in the matter
of nominating an arbitrator qualified in terms of Section11(8) of the Act if the need arises
but the order appointing the arbitrator could only be that of the Chief Justice or the
designated Judge.
(v) Designation of a District Judge as the authority under Section 11(6) of the Act by the
Chief Justice of the High Court is not warranted on the scheme of the Act.
(vi) Once the matter reaches the Arbitral Tribunal or the sole arbitrator, the High Court
would not interfere with the orders
@page-SC955
passed by the arbitrator or the Arbitral Tribunal during the course of the arbitration
proceedings and the parties could approach the Court only in terms of Section 37 of the
Act or in terms of Section 34 of the Act.
(vii) Since an order passed by the Chief Justice of the High Court or by the designated
Judge of that Court is a judicial order, an appeal will lie against that order only under
Article 136 of the Constitution to the Supreme Court.
(viii) There can be no appeal against an order of the Chief Justice of India or a Judge of
the Supreme Court designated by him while entertaining an application under Section
11(6) of the Act.
(ix) In a case where an Arbitral Tribunal has been constituted by the parties without
having recourse to Section 11(6) of the Act, the Arbitral Tribunal will have the
jurisdiction to decide all matters as contemplated by Section 16 of the Act.

(x) Since all were guided by the decision of this Court in Konkan Rly. Corpn. Ltd. v. Rani
Construction (P) Ltd. and orders under Section 11(6) of the Act have been made based on
the position adopted in that decision, we clarify that appointments of arbitrators or
Arbitral Tribunals thus far made, are to be treated as valid, all objections being left to be
decided under Section 16 of the Act. As and from this date, the position as adopted in this
judgment will govern even pending applications under Section 11(6) of the Act. 2002
AIR SCW 426

(xi ) Where District Judges had been designated by the Chief Justice of the High Court
under Section 11(6) of the Act, the appointment orders thus far made by them will be
treated as valid; but applications, if any, pending before them as on this date will stand
transferred, to be dealt with by the Chief Justice of the High Court concerned or a Judge
of that Court designated by the Chief Justice.
(xii) The decision in Konkan Rly. Corpn. Ltd. v. Rani Construction (P) Ltd. is overruled."

From the above, it is clear that the power being exercised by the Chief Justice or the
designated Judge under Section 11 is not an administrative power but it is a judicial
power. It is also clear that an appeal would lie against that order only under Article136 of
the Constitution of India to this Court. Though the decision in Konkan Railway Corpn.
Ltd. (supra) has been overruled, the Bench has clarified that appointment of arbitrators or
Arbitral Tribunals, therefore, made are to be treated as valid, all objections being left to
be decided under Section 16 of the Act. Unfortunately, the above decision in SBP and
Co. vs. Patel Engineering Ltd. and another (supra) though decided earlier i.e. on
26.10.2005 has not been brought to the notice of the Chief Justice, who passed an order,
subsequent to the same i.e. on 09.12.2005. In view of the fact that an order passed under
Section 11(6) is a judicial order and in the light of the stand of the contesting respondents
before the High Court, the appellants in this Court, the impugned order appointing an
Arbitrator without adverting to the claim and objection of both parties cannot be
sustained. The order of the High Court reads as under: 2002 AIR SCW 426
2005 AIR SCW 5932

"Shri Vijay Kumar Singh has appeared for the respondent. The prayer for filing affidavit
is turned down.
For the purpose of acting as Arbitrator in this matter Hon'ble Giridhar Malviya of 26,
Hamilton Road, a retired Judge of this High Court is hereby nominated.
Sd/-
Ajoy Nath Ray
C.J."
As rightly pointed out by learned counsel for the appellants, the order does not show any
reason for appointing an Arbitrator. As said earlier, after the decision of this Court in SBP
and Co. vs. Patel Engineering Ltd. and another (supra) it is incumbent on the part of the
Chief Justice or a designated Judge to consider the claim of both parties and pass a
reasoned order.
Apart from the above infirmity, learned counsel for the appellants has also brought to our
notice that in spite of a request made for filing an affidavit opposing the application for
appointment of an Arbitrator, the Chief Justice has not afforded further time. It is also
pointed out that except respondent No.4-therein, notice had not been served on the other
respondents and without hearing them an order has been passed appointing an Arbitrator.
We verified the order sheet of the High Court (Annexure-P4) which is available at page
50 of the paper-book. The relevant details are reproduced hereunder :
@page-SC956
"ORDER SHEET
Arbitration Case No. 54 of 2002
xxxx xxxx xxxx
14.07.05 Case
Shri Y. P. Singh Advocate and Ajay Kumar Singh have filed Vakalatnama on behalf of the
respondent No. 4
Notices issued to respondents fixing 17.12.2002 have been returned after service as under
:

Respondent No.7 Returned undelivered cover with report "Not Known".


Respondent No.6 Returned undelivered cover with report "Not Known".
Respondent No.8 Returned undelivered cover with report "Not Known".
Respondent No.1 Returned undelivered cover with report "Not Known".
Respondent No.3 Returned undelivered cover with report "Not Known".
Respondent No.2 Notice has not returned after service Put up for Orders

Sd/-
Section Officer
Copying (D) Department
High Court, Allahabad."
As rightly pointed out that whether notice duly served on all the respondents was not
verified before passing the order on 09.12.2005. In our opinion, the following conclusion
would emerge :
i) All the respondents therein except respondent No. 4, notice was not served in the
application for appointment of arbitrator.
ii) Even the served respondent was not afforded adequate opportunity to file his
objection.
iii) The order does not satisfy the requirement of law laid down by this Court in SBP and
Co. vs. Patel Engineering Ltd. and another (supra).
9. In view of the above, we have no other option except to set aside the impugned order
and remit the same for passing fresh order. Since respondents 1 and 7 herein are
represented by their counsel and notice had duly been served on the other respondents in
this Court and none appeared for them, they are permitted to file their objections, if they
so desire within a period of 4 weeks from the date of receipt of copy of this judgment.
Considering the fact referred to above, we make it clear that no further notice need be
issued by the High Court. We constrain to arrive at such conclusion since all of them
(except Shivkumar Agrawal) are members of one family residing at No. 20, Gurdas
Colony, Varanasi and all of them were duly served notice in this Court.
10

. We, therefore, set aside the impugned order dated 09.12.2005 passed by the Chief
Justice of Allahabad High Court and remit the same to the High Court to pass fresh order
as early as possible as observed above and in the light of the principles laid down in SBP
and Co. vs. Patel Engineering Ltd. and another (supra). 2005 AIR SCW 5932

11. The Civil Appeal is allowed to the extent mentioned above. No costs.
Appeal allowed.
AIR 2008 SUPREME COURT 956 "Abdul Raheem v. Karnataka Electricity Board"
(From : Karnataka)
Coram : 2 S. B. SINHA AND H. S. BEDI, JJ.
Civil Appeal No. 5320 of 2007 (arising out of SLP (C) No. 24595 of 2005), D/- 20 -11
-2007.
Abdul Raheem v. Karnataka Electricity Board and Ors.
(A) Civil P.C. (5 of 1908), S.100 - APPEAL - AGREEMENT TO SELL - CONTRACT -
Substantial question of law - Appeal arising out of suit for specific performance of
agreement for sale - Question whether plaintiff was ready and willing to perform his part
of contract - Not a question of law - Substantial question of law should be formulated
relying on or on basis of findings of fact arrived at by Trial Court and First Appellate
Court.
Specific Relief Act (47 of 1963), S.16. (Para 11)
(B) Civil P.C. (5 of 1908), S.100 - Specific Relief Act (47 of 1963), S.16 - AGREEMENT
TO SELL - APPEAL - SUPREME COURT - Second appeal - Formulation of wrong
question of law - Suit for specific performance of agreement to sell - Plaintiff-purchaser
had parted with substantial portion of consideration - Supreme Court, in interest of
justice, remitted matter back to High Court for proper
@page-SC957
framing of question of law. (Para 13)
(C) Civil P.C. (5 of 1908), S.100 - APPEAL - Substantial question of law - Finding
arrived at by considering irrelevant fact - Or by non-consideration of relevant fact - Gives
rise to substantial question of law. (Para 12)
Cases Referred : Chronological Paras
2007 AIR SCW 1694 (Ref., Pnt. C) 14
2007 AIR SCW 4002 : AIR 2007 SC 2306 (Ref., Pnt. C) 14
2005 AIR SCW 993 : AIR 2005 SC 1292 14
2004 AIR SCW 2304 : AIR 2004 SC 3142 (Ref.) 14
2002 AIR SCW 4212 : AIR 2002 SC 3588 14
1999 AIR SCW 1039 : AIR 1999 SC 1341 (Ref., Pnt. C) 12
1999 AIR SCW 1129 : AIR 1999 SC 1441 (Rel. on, Pnt. C) 12
AIR 1990 SC 656 (Ref.) 14
AIR 1988 SC 2176 (Ref.) 14
AIR 1980 SC 1754 (Ref.) 14
Nagendra Rai, Sr. Advocate, Praveen Swarup, Anis Ahmed Khan, R. K. Singh, Shoaib
Ahmad and Rahmat Ullah Kotwal, for Appellant; Basava Prabhu S. Patil, V. N.
Raghupathy, B. Subrahmanya Prasad, Narayan P. Kengasur and Chander Shekhar Ashri,
for Respondents.

Judgement
1. S. B. SINHA, J. :-Leave granted.
2. Defendant in a suit for specific performance of contract is before us aggrieved by and
dissatisfied with a judgment and order dated 15.09.2005 passed by the High Court of
Karnataka in R.S. A. No. 238 of 2000 whereby and whereunder the appeal preferred by
Respondent No. 1 herein arising out of a judgment and decree dated 25.01.2000 passed in
R.A. No. 5 of 1992 was dismissed.
3. An agreement for sale was entered into by and between the defendant-appellant and the
plaintiff-respondent No. 1 for sale of 4 acres, 4 guntas of land in Survey No. 112/A,
Chhidri village. A suit for specific performance of contract was filed by the plaintiff-
respondent No. 1 on the premise that the appellant did not perform his part of contract.
The said suit was dismissed. It was, however, inter alia held by the Trial Court that
Respondent No. 1 was all along ready and willing to perform its part of contract. An
appeal preferred thereagainst by the respondent No.1 was dismissed.
4. The First Appellate Court in arriving at its decision inter alia held :
(i) Respondent No. 1 without any reason withheld payment of balance consideration of
Rs. 13,100/- and, thus, failed to perform its part of contract.
(ii) Respondent No. 1 was not always ready and willing to perform its part of contract
and somehow wanted to transfer liability on the defendant as regards conversion fine and
measurement charges.
(iii) The findings of the Trial Court that the plaintiff-respondent No. 1 was always ready
and willing to perform its part of contract was not correct.
(iv) The plaintiff-respondent No. 1 did not approach the court with clean hands and, thus,
was not entitled to the discretionary relief of specific performance of contract.
5. On a second appeal having been preferred by Respondent No. 1 before the High Court,
the following substantial questions of law were framed :
"i) Whether both the courts have erred in refusing the specific performance although the
respondents received full consideration amount and a sum of Rs. 8,000/- towards
development charges?
ii) Whether the appellate court is justified in holding that the plaintiff was not ever ready
and willing to perform his part of the contract?
iii) Whether the courts below have not committed any error in directing to refund the
earnest money?"
6. The High Court inter alia considering the stipulations made in agreement for sale and
other evidences brought on records, opined :
(i) It was for the defendant to bear the conversion expenses. Admittedly when the amount
of Rs. 94,000/- and odd was paid to him, he was bound to perform his part of contract by
executing a deed of sale in favour of the plaintiff Respondent No. 1.
(ii) The First Appellate Court failed to interpret the clauses of the agreement in their
proper perspective.
(iii) It applied the provisions contained in Section 22 of the Specific Relief Act directing
refund of the earnest amount only in a mechanical manner.
7. The plea of Respondent No. 1 that it
@page-SC958
was ready and willing to forgo four guntas of land was held by the High Court sufficient
to meet the demand of the defendant appellant stating :
"15. For the foregoing reasons, it is to be held that both the Courts below have erred in
interpreting Ex. P.1 in the proper perspective and rejecting for specific performance.
Hence, the substantial question No. 1 rests in favour of the appellant and further the
finding of the lower appellate Court for the plaintiff was not ready and willing to perform
his part of contract is with a basis and as such the same is to be reversed and as also the
2nd substantial question of law to be held in favour of the appellant. In so far as the 3rd
substantial question of law is concerned in the event if the courts below have considered
the fact of hardship and in the event if the courts below would have ordered for specific
performance, then order for refund of money instead of ordering for specific performance
would be perverse. Accordingly, it is held necessarily in favour of the appellant.
8. Mr. Nagendra Rai, learned senior counsel appearing on behalf of the appellant
submitted that the purported substantial questions of law formulated by the High Court
do not meet the requirements of Section 100 of the Code of Civil Procedure (Code). It
was further urged that having regard to the factual findings arrived at by the First
Appellate Court, the High Court should not have interfered therewith in exercise of its
power under Section 100 of the Code.
9. Mr. Basava Prabhu S. Patil, learned counsel appearing on behalf of the respondents, on
the other hand, drew our attention to the factual matrix involved in the matter. It was
contended that Respondent No. 1 was put in possession pursuant to the agreement for
sale. A question, however, arose as to who on conversion of the user of the land would
pay the conversion fine. It was submitted that Respondent No. 1 paid the said amount
also. In the aforementioned situation, interpretation of Clause 3 of the agreement arose
for consideration in the factual matrix obtaining in the matter, viz., Respondent No. 1 not
only paid a sum of Rs. 73,000/- out of the total amount of consideration of Rs. 86,100/-
but also paid a sum of Rs. 21,431.55 and Rs. 35.00 towards the conversion fine and
measurement fees respectively.
10. A substantial question of law ordinarily would arise from the finding of facts arrived
at by the Trial Court and the First Appellate Court. The High Courts jurisdiction in terms
of Section 100 of the Code is undoubtedly limited.
11. The question as to whether the plaintiff was ready and willing to perform its part of
contract by itself may not give rise to a substantial question of law. Substantial question
of law should admittedly be formulated relying on or on the basis of findings of fact
arrived at by the Trial Court and the First Appellate Court.
12. However, there cannot be any doubt whatsoever that consideration of irrelevant fact
and non-consideration of relevant fact would give rise to a substantial question of law.
Reversal of a finding of fact arrived at by the First Appellate Court ignoring vital
documents may also lead to a substantial question of law.
In Vidhyadhar v. Manikrao and Another [(1999) 3 SCC 573], this Court held : 1999
AIR SCW 1129

"23. The findings of fact concurrently recorded by the trial court as also by the lower
appellate court could not have been legally upset by the High Court in a second appeal
under Section 100, CPC unless it was shown that the findings were perverse, being based
on no evidence or that on the evidence on record, no reasonable person could have come
to that conclusion."

[See also Iswar Bhai C. Patel alias Bachu Bhai Patel v. Harihar Behera and Another
(1999) 3 SCC 457] 1999 AIR SCW 1039

13. Ordinarily, we would have allowed the appeal on the failure of the High Court to
formulate substantial questions of law within the meaning of Section 100 of the Code,
but, we feel that as the plaintiff Respondent No. 1 had already parted with a substantial
portion of the consideration amount as also upon having paid a large sum towards
conversion charges, in the interest of justice another opportunity should be given to the
High Court to frame proper substantial questions of law arising in the matter.
14

. We may, however, notice a few decisions in regard to the jurisdiction of the High Court
under Section 100 of the Code. In Commissioner of Customs (Preventive) v. Vijay
Dasha-rath Patel [(2007) 4 SCC 118], 2007 AIR SCW 1694, (Paras 20 to 24)

@page-SC959
this Court held :
"22. We are not oblivious of the fact that the High Courts jurisdiction in this behalf is
limited. What would be substantial question of law, however, would vary from case to
case.

23. Moreover, although, a finding of fact can be interfered with when it is perverse, but, it
is also trite that where the courts below have ignored the weight of preponderating
circumstances and allowed the judgment to be influenced by inconsequential matters, the
High Court would be justified in considering the matter and in coming to its own
independent conclusion. (See Madan Lal v. Gopi.) AIR 1980 SC 1754

24. The High Court shall also be entitled to opine that a substantial question of law arises
for its consideration when material and relevant facts have been ignored and legal
principles have not been applied in appreciating the evidence. Arriving at a decision,
upon taking into consideration irrelevant factors, would also give rise to a substantial
question of law. It may, however, be different that only on the same set of facts the higher
court takes a different view. [See Collector of Customs v. Swastic Woollens (P) Ltd. and
Metroark Ltd. v. CCE.] AIR 1988 SC 2176
2004 AIR SCW 2304
25. Even in a case where evidence is misread, the High Court would have power to
interfere. (See W.B. Electricity Regulatory Commission v. CESC Ltd. and also Commr.
of Customs v. Bureau Veritas.) 2002 AIR SCW 4212

26. In Dutta Cycle Stores v. Gita Devi Sultania this Court held : (SCC p. 587, para 4)
AIR 1990 SC 656, Para 4

"4. Whether or not rent for the two months in question had been duly paid by the
defendants is a question of fact, and with a finding of such fact, this Court does not
ordinarily interfere in proceedings under Article 136 of the Constitution, particularly
when all the courts below reached the same conclusion. But where the finding of fact is
based on no evidence or opposed to the totality of evidence and contrary to the rational
conclusion to which the state of evidence must reasonably lead, then this Court will in the
exercise of its discretion intervene to prevent miscarriage of justice."

[See also P. Chandrasekharan and Others v. S. Kanakarajan and Others, (2007) 5 SCC
669]. 2007 AIR SCW 4002

15. We, therefore, set aside the impugned judgment and remit the matter back to the High
Court for consideration of the matter afresh upon formulation of a substantial question of
law. The appeal is allowed. No costs.
Order accordingly.
AIR 2008 SUPREME COURT 959 "Management of Indian Bank v. G. Ramachandran"
(From : Madras)*
Coram : 2 S. B. SINHA AND PRAKASH PRABHAKAR NAOLEKAR, JJ.
Civil Appeal Nos. 5238-5242 of 2007 (arising out of SLP (C) Nos. 24710-24714 of
2005), D/- 2 -11 -2007.
Management of Indian Bank and Anr. v. G. Ramachandran and Ors.
(A) Banking Companies (Acquisition and Transfer of Undertakings) Act (40 of 1980),
S.19 - Indian Bank (Employees') Pension Regulations (1995), Regn.17 - Indian Bank
Officers' Service Regulations (1979), Regn.37 - BANKING - PENSION - SERVICE
MATTERS - Pension - Qualifying service - Computation - Inclusion of leave with loss of
pay - Permissible only when sanctioning authority so directs - Holding that such direction
must be given simultaneously while sanctioning leave - Not proper.
W. P. Nos. 44153, 44157, etc. of 2002 and 3700 of 2003, D/- 13-9-2005 (Mad), Reversed.
Regn. 17 of Pension Regulations provides that all leave during service in the Bank for
which leave salary is payable shall count as qualifying service. The proviso appended to
Regn. 17, however, categorically states that extraordinary leave on loss of pay shall not
count as qualifying service except when the sanctioning authority has directed that such
leave, not exceeding twelve months during the entire service, may count as service for all
purposes including pension. Leave is sanctioned under Service Regulations. Service
Regulations and Pension Regulations, operate in different fields. To hold that while
sanctioning leave the authority must simultaneously give direction as regards its inclusion
in qualifying service would be improper. The question of application of mind on the part
of sanctioning authority in terms of proviso appended to Regn. 17 of the
@page-SC960
Pension Regulations would arise only at the end of the service of the employee concerned
and not at the time when the leave is granted.
W. P. Nos. 44153, 44157, etc. of 2002 and 3700 of 2003, D/- 13-9-2005 (Mad), Reversed.
(Paras 11, 13, 14)
(B) INTERPRETATION OF STATUTES - Interpretation of Statutes - External aids - For
construing one statute reference to another statute - Not permissible. (Para 12)

Raju Ramachandran, Sr. Advocate, Y. K. Rao, Saket Sikri, Ms. Madhu Sikri, for
Appellants; Jugal Kishore Tiwari, P. V. Yogeswaran, for Respondents.
* W.P. Nos. 44153, 44157, 44158, 44178 of 2002 and 3700 of 2003, D/- 13-9-2005
(Mad.)
Judgement
S. B. SINHA, J. :- Leave granted.
2. This appeal is directed against the judgment and order dated 13.9.2005 passed by a
Division Bench of the Madras High Court in civil writ petitions filed by the respondents
herein.
3. The short question involved in these appeals related to interpretation of Regulation 17
of the Indian Bank (Employees') Pension Regulations, 1995 (hereinafter referred to as
"Pension Regulations"), vis-a-vis Regulation 37 of the Indian Bank Officers' Service
Regulations, 1979, (hereinafter referred to as "Service Regulations").
4. Respondents have joined the service of the appellant-Bank on diverse dates. They,
indisputably, have served the Bank for more than 10 years. They opted for the Pension
Regulation as and when the same was framed and sought for voluntary retirement from
services in 2001. The said offer of retirement has been accepted.
5. The representation of the respondents that their entire period of service, including the
period of leave availed by them on loss of pay, should be taken into consideration for the
purpose of computing the pensionary benefits, having been rejected by the Bank by its
order dated 16.8.2001 and 17.8.2001, several writ applications came to be filed by the
respondents. The High Court by an order dated 23.9.2002 directed the appellant-Bank to
give a personal hearing to them. The said order was complied with. However, by an order
dated 11.10.2002 passed by the Bank, the representations of the respondents were
rejected.
6. Respondent Nos.1, 3, 4 and 5 herein again filed writ applications in November 2002
challenging the aforesaid order dated 11.10.2002; whereas respondent No.2 filed a writ
petition before the High Court in March 2003, questioning the validity or otherwise of the
order dated 3.8.2001. By reason of the impugned judgment a Division Bench of the High
Court allowed the said writ applications filed by the respondents herein. Appellants are,
thus, before us.
7. Before embarking on the rival contentions advanced by the learned counsel for the
parties, we may notice that Chapter VII of the Service Regulations provide for different
kinds of leave, namely, casual leave, privilege leave, sick leave, special sick leave,
maternity leave, extraordinary leave on loss of pay and special casual leave and special
leave.
8. The fact that the concerned respondents had obtained leave on medical ground, without
pay is not in dispute.
9. Extraordinary leave is granted in terms of Regulation 37 of the Service Regulations
which reads thus :
"An officer shall be eligible for extraordinary leave on loss of pay for not more than 360
days during the entire period of service. Such leave may not be availed of except for
sufficient reasons on more than 90 days at a time provided that in very special
circumstances the Board may grant extraordinary leave on loss of pay to an officer upto a
total period of 720 days."
10. Whereas grant of leave is governed by the Service Regulations, grant of pension
and/or determination of the quantum thereof is governed by the Pension Regulations.
Regulation 14 of the Pension Regulations provides for qualifying service in the following
terms :
"Subject to the other conditions contained in these regulations, an employee who has
rendered a minimum of ten years of service in the Bank on the date of his retirement or
the date on which he is deemed to have retired shall qualify for pension."
11. Regulation 17 provides that all leave during service in the Bank for which leave
salary is payable shall count as qualifying service. The proviso appended to Regulation
17, however, categorically states that extraordinary leave on loss of pay shall not count as
qualifying service except when the sanctioning authority has directed that such leave, not
exceeding twelve months during the entire service, may count as service for
@page-SC961
all purposes including pension. The High Court in its impugned judgment opined that
while granting extraordinary leave, the sanctioning authority must be held to have
sanctioned leave in terms of Regulation 17 of the Pension Regulations as well. For the
said proposition, reliance has been placed on Rule 21 of the Central Civil Services
Pension Rules. With a view to complete the narration of facts, we may take note of
Regulation 21 also.
"Counting of period spent on leave. All leave during service for which leave salary is
payable and all extraordinary leave granted on medical certificate shall count as
qualifying service.
Provided that in the case of extraordinary leave other than extraordinary leave granted on
medical certificate, the appointing authority may, at the time of granting such leave, allow
the period of that leave to count as qualifying service if such leave is granted to a
Government servant-
i) Omitted.
ii) Due to his inability to join or rejoin duty on account of civil commotion or;
iii) For prosecuting higher scientific and technical studies."
12. We, at the outset, must express our reservation in regard to the approach of the High
Court in so far as reference has been made to a regulation which is not applicable to the
case of the respondents herein. Regulation 21 itself clearly suggests that only in a case
where leave has been granted on medical ground, the appointing authority at the time of
granting such leave may allow the period thereof to be counted as qualifying service if
such leave is granted to a Government servant. The limited area in which Regulation 21
operates is evidently centres round grant of medical leave which is not the case here.
Pension Regulations framed by the appellants do not postulate such a contingency. It is
now a trite law that for the purpose of construing a statute, reference to another statute is
not permissible and, thus, Regulation 21 of the Civil Services Pension Rules
contemplates a different situation, the same will have no application in the instant case.
The High Court, therefore, committed an error in relying on the said provision.
13. The High Court has held that the "entries regarding service being qualifying or
otherwise are required to be made simultaneously with the event, but in this case it is not
done." We are afraid that such an inference could not have been drawn in the instant case.
Service Regulations operate in the matter of grant of leave. It may be possible that the
highest authority had granted leave in favour of the respondents but the same would not
mean that in all such events, the authority to grant leave in terms of Service Regulations
i.e. the sanctioning authority under Regulation 17 would be one and the same.
Construction of a statute should not be premised on surmises and conjectures.
14. The question of application of mind on the part of sanctioning authority in terms of
proviso appended to Regulation 17 of the Pension Regulations would arise only at the
end of the service of the employee concerned and not at the time when the leave is
granted. Service Regulations and Pension Regulations, thus, operate in different fields.An
employee of the Bank would be entitled to the benefit of the proviso appended to
Regulation 17 of the Pension Regulations in the event a direction is issued by the
sanctioning authority. If no such direction is issued, the question of granting leave by the
competent authority for the purpose of pensionary benefits would not arise.
15. We, therefore, are of the opinion that the impugned judgment of the High Court
cannot be sustained and it is set aside accordingly. The appeal is allowed. However, in the
facts and circumstances of this case, there shall be no orders as to costs.
Appeal allowed.
AIR 2008 SUPREME COURT 961 "Vijaykumar Baldev Mishra v. State of Maharashtra"
Coram : 2 S. B. SINHA AND MARKANDEY KATJU, JJ.
Criminal Appeal No. 602 of 2004*, D/- 18 -5 -2007.
Vijaykumar Baldev Mishra @ Sharma v. State of Maharashtra.
(A) Criminal P.C. (2 of 1974), S.321 - WITHDRAWAL FROM PROSECUTION -
TERRORIST AND DISRUPTIVE ACTIVITIES - TADA case - Application for
withdrawal of prosecution - Refusal of consent - Validity - Expressing opinion in merit of
matter and effect of confessions - Not warranted.
@page-SC962

TADA Sessions Case No. 8 of 1993 (Pune), D/- 15-3-2003, Reversed.


While refusing to grant permission, the Designated Court, is not correct in expressing its
opinion in the merit of the matter and the effect of confessions made in terms of the
provisions of TADA. It is, however, also not necessary to consider as to whether, the
action of the Public Prosecutor as also the State is bona fide or not. Moreover, bona fide
on the part of the Public Prosecutor itself cannot automatically lead to grant of consent.
There are other circumstances also which are required to be taken into consideration.
(Para 17)
Application of mind on the part of the Court, is necessary in regard to the grounds for
withdrawal from the prosecution in respect of any one or more of the offences for which
the accused is tried. The Public Prosecutor in terms of the statutory scheme laid down
under the Code plays an important role. He is supposed to be an independent person.
While filing such an application, the Public Prosecutor also is required to apply his own
mind and the effect thereof on the society in the event such permission is granted. (Paras
12, 13)
(B) Terrorist and Disruptive Activities (Prevention) Act (28 of 1987), S.1(4) (since
repealed) - TERRORIST AND DISRUPTIVE ACTIVITIES - CONSTITUTIONALITY
OF AN ACT - EQUALITY - Constitutional validity - Section violates Art. 14 of
Constitution.
Constitution of India, Art.14.
Per Katju, J. - Section 1(4) of the Act says that the offence created by S. 3 of the Act will
be punishable as a crime if the act was committed on or before 24-5-1995, but if the same
act was committed after 24-5-1995, it will not be a crime. This is ex facie violation of
Art. 14 of the Constitution and hence S. 1(4) of Act to the extent it says that acts
mentioned in S. 3 committed on or before 24-5-1995 can still be treated as a crime and
punished under the TADA, though the same act committed after 24-5-1995 cannot, is
clearly ultra vires Art. 14 and hence is liable to be struck down as unconstitutional. As
regards those who have already undergone the entire sentence for which they were
convicted under TADA obviously nothing can be done, but regarding those who have
undergone only part of the sentence or regarding those who are facing prosecution or
investigation under TADA such prosecution or investigation are liable to be quashed.
(Paras 28, 30)
Cases Referred : Chronological Paras
2005 AIR SCW 576 : AIR 2005 SC 910 : 2005 Cri LJ 963 15
2005 AIR SCW 6054 : AIR 2006 SC 413 : 2006 Cri LJ 148 15
2003 AIR SCW 6468 : AIR 2004 SC 524 : 2004 Cri LJ 583 15
2001 AIR SCW 1156 : AIR 2001 SC 1375 : 2001 Cri LJ 1697 5
1999 AIR SCW 1889 : AIR 1999 SC 2640 : 1999 Cri LJ 3124 6, 10
1996 AIR SCW 2454 : AIR 1996 SC 2047 : 1996 Cri LJ 2872 3
1994 Cri LJ 3139 (SC) 2, 3, 10
AIR 1990 SC 1962 5
AIR 1987 SC 877 : 1987 Cri LJ 793 14
AIR 1957 SC 389 : 1957 Cri LJ 567 16
R. F. Nariman, Sr. Advocate, K. V. Vishwanathan, Anup Kumar and Atishi Dipankar, for
Appellant; Prashant Chaudhary and Ravindra Keshavrao Adsure, for Respondent.
* From Judgment and Order of Designated Court (TADA), Pune in EXH. No. 282 in
TADA Sessions Case No. 8 of 1993, D/- 15-3-2003.
Judgement
1. S. B. SINHA, J. :- One Anna Shetty was facing trial for murder of one Duni Chand
Kalani. Duni Chand Kalani is said to be the uncle of one Pappu Kalani. He was an
accused in a murder case. The said Anna Shetty was murdered in jail. Anna Shetty was
released from jail on 15.10.1990. He was murdered on the same day. The appellant was
an accused therein with many others. The said murder took place in view of the long
standing enmity between two gangs belonging to Gopal Rajwani and Pappu Kalani. A
First Information Report was lodged under Section 302/307, IPC as also under the Arms
Act. Appellant, however, along with others, were also charged under Terrorist and
Disruptive Activities (Prevention) Act (hereinafter referred to as 'TADA').
2
. Indisputably, a Constitution Bench of this Court in Kartar Singh vs. State of Punjab,
[1994 (3) SCC 569], while upholding the validity of TADA directed constitution of a
Committee to review the cases pending thereunder for the purpose of making
recommendations to the Government, so as to enable it to consider the matters where in
its opinion, the charges under TADA were required to be dropped and the matters for the
prosecution thereunder should continue. 1994 Cri LJ 3139

@page-SC963
3

. Kartar Singh (supra) was explained by this Court in R.M. Tewari v. State (NCT of
Delhi) and others [1996 (2) SCC 610] in the following terms : 1996 AIR SCW 2454,
(Paras 9, 10)

"10. The observations in Kartar Singh have to be understood in the context in which they
were made. It was observed that a review of the cases should be made by a High Power
Committee to ensure that there was no misuse of the stringent provisions of the TADA
Act and any case in which resort to the TADA Act was found to be unwarranted, the
necessary remedial measures should be taken. The Review Committee is expected to
perform its functions in this manner. If the recommendation of the Review Committee,
based on the material present, is, that resort to provisions of the TADA Act is
unwarranted for any reason which permits withdrawal from prosecution for those
offences, a suitable application made under Section 321, Cr. P.C. on that ground has to be
considered and decided by the Designated Court giving due weight to the opinion formed
by the Public Prosecutor on the basis of the recommendation of the High Power
Committee.
11. It has also to be borne in mind that the initial invocation of the stringent provisions of
the TADA Act is itself subject to sanction of the Government and, therefore, the revised
opinion of the Government formed on the basis of the recommendation of the High
Power Committee after scrutiny of each case should not be lightly disregarded by the
Court except for weighty reasons such as mala fides or manifest arbitrariness. The worth
of the material to support the charge under the TADA Act and the evidence which can be
produced, is likely to be known to the prosecuting agency and, therefore, mere existence
of prima facie material to support the framing of the charge should not by itself be treated
as sufficient to refuse the consent for withdrawal from prosecution. It is in this manner an
application made to withdraw the charges of offences under the TADA Act pursuant to
review of a case by the Review Committee has to be considered and decided by the
Designated Courts."
4. Bombay High Court also took the same view and issued similar directions.
5. Pursuant to or in furtherance of the recommendations of this Court as also the Bombay
High Court, a Review Committee was constituted, headed by a retired Judge of the High
Court of Bombay. The Review Committee took up for consideration 27 cases including
the one pending against the appellant herein. In its report, the Review Committee opined :
"The murder of Anna Shetty appears to be out of personal enmity and not intended to
cause fear or terror in the minds of the people. It is so stated in the chargesheet itself.
"To take revenge of the murder of his uncle and to establish supremacy over the rival
gang, he made conspiracy to kill Anna Shetty by providing money, weapons, manpowers,
vehicles and other assistance. He was the mastermind behind the killing of Anna Shetty."

It also reveals that the genesis of two murder cases i.e. TCS No. 25/92 and TCS 8/93 are
inter-connected as observed by the Supreme Court in its order dated 2nd March, 2001 in
Criminal Appeal No. 12981, 1299 of 1998 as under : reported in 2001 AIR SCW
1156, (Para 7)

"According to the prosecution there are two groups in Ulhasnagar, one is headed by
Gopal Rajwani and other by Pappu Kalani. The deceased Maruti Jadhav and one Krishna
Pillay were eye-witness to the murder of Lalu in the year 1989 at Hotel Sun and Sand,
Bombay. The said Krishna was murdered and thereafter Maruti Jadhav remained the only
eye-witness."

It is heard in the case of Niranjan Singh Karan Singh Punjabi vs. Jitendera Bhimraj Bijj,
AIR 1990 SC 1962 (1969). (Para 11)

"A mere statement to the effect that the show of such violence would create terror or fear
in the minds of the people and none would care to oppose them cannot constitute an
offence under Section 3 (1) of TADA Act."
It was observed :
"Although the murder of Anna Shetty took place in 1990 and the case was charge-sheeted
in 1993, the trial has not commenced. There are three accused persons who are still in jail
for more than seven years and are yet to be tried. In view of the observation of the
Hon'ble Supreme Court in the various judgments cited above, the long time that has
elapsed, periods spent in jail and the fact that the crime is committed due to personal
enmity, the Review Committee is
@page-SC964
of the view to drop the proceedings under TADA against all the accused and they may be
prosecuted for the charges under Section 302, IPC etc. in the regular Sessions Court.
6. It was opined that no case for continuation of a case under TADA had been made out
against the appellant. Pursuant to or in furtherance of the said recommendations, the
Special Public Prosecutor filed an application for withdrawal of the charges under TADA
inter alia as against the appellant stating :
"The reasons for withdrawal are as under :

(A) The stringent provision of TADA need not to be attracted in the instant case and the
Government after proper discussion on the facts of the case and the evidence, reports and
letters available on the record has decided in the enclosed list and the Government has
perused all records and considered the opinion of the Review Committee formulated by
the State Government under orders of the Supreme Court. It would be just and proper that
this case need not be proceeded further. The request is being made to withdraw from
prosecution as against the offences punishable under the TADA Act, and the stringent and
hard provisions of TADA were not necessary to deal with such situations. In fact, from
the record it has also been seen that the provision of the said TADA Act also could not be
said to be attracted as the said provision cannot be invoked nor could the activities be said
to be disruptive activities as the said incident seems to have taken place because of the
personal rivalry and as held by the Hon'ble Supreme Court in the case of State vs. Nalini
and others, reported in 1999 (5) SCC 253, it could not be said that the provisions of
Section of TADA are applicable. It is worthwhile to note that after having seen the record
that concession cannot form the basis for framing charges. 1999 AIR SCW 1889

(B) The State Government having considered all this and the Special Public Prosecutor
having applied his mind to this aspect, seeks to withdraw from the prosecution in respect
of offences under TADA Act as against the accused in the instant case."
7. The Designated Court TADA dismissed the said application by an order dated
15.5.1993 inter alia opining :
"The case No. 8/93 pertains to the murder of Anna Shetty and constable Surve. He has
also referred to the murder of one Ghanasham Bhatija and Inder Bhatija on 27.7.1990 and
28.4.90 respectively, who were also accused in case of murder of Dunichand Kalani. In
the order dated 10.11.98 in connected TADA Sessions Case Nos. 25/92 and 9/93 below
Exhs. 27, 31, 32 etc. my learned predecessor made reference to the confessional
statements of accused recorded under the provisions of TADA Act. Leaving aside the
infirmities and/or evidentiary value of these statements which will be assessed during
trial the fact remains that all this evidence collected after strenuous investigation will
have to be ignored. In both these cases allegations are that conspiracy was hatched to
eliminate Anna Shetty. Maruti Jadhav and in pursuance of this plan was executed on
different dates. Thus confessional statements recorded under Section 15 of the TADA Act
are very relevant and important piece of evidence to unfold conspiracy and to unfold act
of main perpetrator of crime. Keeping in view this aspect of the matter, if applications for
dropping of the charges under the provisions of TADA Act are considered, it would be
difficult to conclude that the withdrawal from the prosecution or dropping of the charges
under the provisions of TADA Act would hardly serve any public interest or would
advance course of justice. This is apart from the fact that in both these matters there are
orders of Hon'ble Supreme Court wherein applicability of TADA has been upheld and
trials have been expedited. It is, therefore, expected of the State to render assistance to the
Court to dispose of the matters expeditiously."
8. The State of Maharashtra being aggrieved by the said order filed a writ petition before
the Bombay High Court which was marked as Criminal Writ Petition 562 of 2003. A
Division Bench of the said Court by a judgment and order dated 4.7.2003 while declining
to entertain the same opined that as an appeal against the order of the learned Designated
Court TADA is maintainable under Section 19 of the Act, the petitioner should take
recourse thereto stating:
"In our opinion therefore the objection raised by Mr. Nitin Pradhan that the writ petition
is not maintainable in view of the fact that alternate and efficacious remedy by way of
appeal is available to the
@page-SC965
prosecution is well founded.
In all humility, in our opinion, a remedy of appeal under Section 19 of the TADA Act is
far more efficacious and better for all concerned than seeking exercise of writ jurisdiction
of this Court under Article 226 of the Constitution.
Yet another important aspect to be noted is that this very interveners in this case i.e.
representatives of the victim are also the interveners of the appeal in the Supreme Court
which was decided by the order dated 2nd March, 2001. It is pertinent to note that they
do not in our opinion, rightly raise any objection about maintainability of the appeal in
Supreme Court on the ground that the order challenged is an interlocutory order."
9. Appellant has, thus, preferred this appeal under Section 19 of the TADA (P) Act, 1987
from the said judgment of the Designated Court TADA dated 10.3.2003.
10

. Mr. R. F. Nariman, the learned senior counsel appearing on behalf of the appellant
submits that keeping in view the recommendations of the Review Committee, the learned
Designated Court TADA committed a manifest error in refusing to allow the public
prosecutor to withdraw the case solely on the ground that certain confessions having been
made thereunder, the same would not be available if the appellant is not proceeded
against under TADA Act. The Review Committee, as noticed hereinbefore, was
constituted in view of the directions issued by this Court in Kartar Singh (supra) and a
decision of the Bombay High Court in Criminal Writ Petition No. 289 of 2002. The
Review Committee reviewed only those cases where the trial had not started. They
formulated their own guidelines for scrutiny of the matter. It noticed the decision of this
Court in State vs. Nalini reported in (1999) 5 SCC 253 wherein it was held : 1994
Cri LJ 3139
1999 AIR SCW 1889, (Para 44)

"A reading of the first sub-section shows that the person who does any act by using any
of the substances enumerated in the sub-section in any such manner as are specified in
the sub-section, cannot be said to commit a terrorist act unless the act is done "with
intent" to do any of the four things : (1) to overawe the Government as by law
established; or (2) to strike terror in people or any section of the people; or (3) to alienate
any section of the people; or (4) to adversely affect the harmony amongst different
sections of the people"
11. Mr. R.K. Adsure, learned counsel appearing on behalf of the State supported the
contention of Mr. Nariman.
12. Section 321 of the Criminal Procedure Code, 1973 provides for withdrawal from
prosecution at the instance of the public prosecutor or Assistant public prosecutor.
Indisputably therefor the consent of the Court is necessary. Application of mind on the
part of the Court, therefore, is necessary in regard to the grounds for withdrawal from the
prosecution in respect of any one or more of the offences for which the appellant is tried.
The provisions of TADA could be attracted only in the event of one or the other of the
four 'things' specified in Nalini (supra) is found applicable and not otherwise. The Review
Committee made recommendations upon consideration of all relevant facts. It came to its
opinion upon considering the materials on record. Its recommendations were based also
upon the legality of the charges under TADA in the fact situation obtaining in each case.
It came to the conclusion that in committing the purported offence, the appellant inter alia
had no intention to strike terror in people or any section of the people and in fact the
murder has been committed only in view of group rivalry and because the parties
intended to take revenge, the provisions of the TADA should not have been invoked.
13. The Public Prosecutor in terms of the statutory scheme laid down under the Code of
Criminal Procedure plays an important role. He is supposed to be an independent person.
While filing such an application, the public prosecutor also is required to apply his own
mind and the effect thereof on the society in the event such permission is granted.
14

. In Sheonandan Paswal v. State of Bihar [(1987) 1 SCC 288], Khalid, J, opined: AIR
1987 SC 877, (Para 67)

"73. Section 321 gives the Public Prosecutor the power for withdrawal of any case at any
stage before judgment is pronounced. This presupposes the fact that the entire evidence
may have been adduced in the case, before the application is made. When an application
under Section 321, CrPC is made, it is not necessary for the court to assess the evidence
to discover whether the case would end in conviction or acquittal. To
@page-SC966
contend that the court when it exercises its limited power of giving consent under Section
321 has to assess the evidence and find out whether the case would end in acquittal or
conviction, would be to rewrite Section 321, CrPC and would be to concede to the court a
power which the scheme of Section 321 does not contemplate. The acquittal or discharge
order under Section 321 are not the same as the normal final orders in criminal cases. The
conclusion will not be backed by a detailed discussion of the evidence in the case of
acquittal or absence of prima facie case or groundlessness in the case of discharge. All
that the court has to see is whether the application is made in good faith, in the interest of
public policy and justice and not to thwart or stifle the process of law. The court after
considering these facets of the case, will have to see whether the application suffers from
such improprieties or illegalities as to cause manifest injustice if consent is given. In this
case, on a reading of the application for withdrawal, the order of consent and the other
attendant circumstances, I have no hesitation to hold that the application for withdrawal
and the order giving consent were proper and strictly within the confines of Section 321,
Cr PC."
15. In regard to the Courts' function in the matter of grant of consent, while opining that
the grant should not be a matter of course, this Court held that even a detailed reasoned
order is not necessary to be passed therefor. The Court took into consideration the
jurisdiction of the Court under Section 321 of the Criminal Procedure Code vis-a-vis
other provisions laid down therein opining :
"85. The scope of Section 321 can be tested from another angle and that is with reference
to Section 320 which deals with "compounding of offences". Both these sections occur in
Chapter 24 under the heading "General Provisions as to Enquiries and Trials". Section
320(1) pertains to compounding of offences, in the table, which are not of a serious
nature while Section 320(2) pertains to offences of a slightly serious in nature but not
constituting grave crimes. The offences in the table under Section 320(1) may be
compounded by the persons mentioned in the third column of the table without the
permission of the court and those given in the Table II, under Section 320(2) can be
compounded only with the permission of the court. Under sub-section 4(a), when a
person who would otherwise be competent to compound an offence under Section 320, is
under the age of 18 years or is an idiot or a lunatic, any person competent to contract on
his behalf may, with the permission of the court, compound such offence. Sub-section
4(b) provides that when a person who would otherwise be competent to compound an
offence under this section is dead, the legal representative, as defined in the Code of Civil
Procedure, of such person may, with the consent of the court, compound such offence.
86. These two sub-sections use the expression "with the permission of the court" and
"with the consent of the court" which are more or less ejusdem generis. On a fair reading
of the abovementioned sub-sections it can be safely presumed that the sections confer
only a supervisory power on the court in the matter of compounding of offences in the
manner indicated therein, with this safeguard that the accused does not by unfair or
deceitful means, secure a composition of the offence. Viewed thus I do not think that a
plea can be successfully put forward that granting permission or giving consent under
sub-section (4)(a) or (4)(b) for compounding of an offence, the court is enjoined to make
a serious detailed evaluation of the evidence or assessment of the case to be satisfied that
the case would result in acquittal or conviction. It is necessary to bear in mind that an
application for compounding of an offence can be made at any stage. Since Section 321
finds a place in this chapter immediately after Section 320, one will be justified in saying
that it should take its colour from the immediately preceding section and in holding that
this section, which is a kindred to Section 320, contemplates consent by the court only in
a supervisory manner and not essentially in an adjudicatory manner, the grant of consent
not depending upon a detailed assessment of the weight or volume of evidence to see the
degree of success at the end of the trial. All that is necessary for the court to see is to
ensure that the application for withdrawal has been properly made, after independent
consideration, by the Public Prosecutor and in furtherance of public interest.
It was furthermore held :
"90. Section 321 CrPC is virtually a step by way of composition of the offence by the
State. The State is the master of the
@page-SC967
litigation in criminal cases. It is useful to remember that by the exercise of functions
under Section 321, the accountability of the concerned person or persons does not
disappear. A private complaint can still be filed if a party is aggrieved by the withdrawal
of the prosecution but running the possible risk of a suit of malicious prosecution if the
complaint is bereft of any basis."

(See also S. K.Shukla and Others vs. State of U.P. and Others [2006(1) SCC 314], Rahul
Agarwal vs. Rakesh Jain and Anr. [2005 (2) SCC 377] and K. Anbazhagan vs.
Superintendent of Police and Others [2004(3) SCC 767]) 2005 AIR SCW 6054
2005 AIR SCW 576
2004 AIR SCW 6468

16

. Noticing that no guidelines have been provided for in the matter of grant or withdrawal
of the consent by the Court, the Constitution Bench referred to the ratio of its earlier
decision in State of Bihar vs. Ram Naresh Pandey [1957 (1) SCR 279] wherein it was
held : AIR 1957 SC 389
"His discretion in such matters has necessarily to be exercised with reference to such
material as is by then available and it is not a prima facie judicial determination of any
specific issue. The Magistrate's functions in these matters are not only supplementary, at a
higher level, to those of the executive but are intended to prevent abuse. Section 494
requiring the consent of the court for withdrawal by the Public Prosecutor is more in line
with this scheme, than with the provisions of the Code relating to inquiries and trials by
court. It cannot be taken to place on the court the responsibility for a prima facie
determination of a triable issue. For instance the discharge that results therefrom need not
always conform to the standard of 'no prima facie case' under Sections 209(1) and 253(1)
or of 'groundlessness' under Sections 209(2) and 253(2). This is not to say that a consent
is to be lightly given on the application of the Public Prosecutor, without a careful and
proper scrutiny of the grounds on which the application for consent is made."
17. While refusing to grant permission, the Designated Court, in our opinion, was not
correct in expressing its opinion in the merit of the matter and the effect of confessions
made in terms of the provisions of TADA. It was, however, also not necessary to consider
as to whether, the action of the public prosecutor as also the State was bona fide or not.
Moreover, bona fide on the part of the public prosecutor itself cannot automatically lead
to grant of consent. There are other circumstances also which are required to be taken
into consideration.
18. For the reasons aforementioned, the appeal is allowed. The application filed by the
State for withdrawal of the charges under TADA against the appellant shall stand
allowed. The learned Designated Judge may now proceed with the matter in accordance
with law.
19. MARKANDEY KATJU, J.I have perused the judgment of my learned brother
Hon'ble S. B. Sinha, J. in this case. The facts of the case have been narrated in the
judgment of my learned brother and hence I am not repeating the same. I entirely agree
with the reasoning and conclusion of my learned brother.
20. However, there is an important constitutional point which though not taken in the
Criminal Appeal before us, is of such great importance that I wish to express my opinion
on the same.
21. The Terrorist and Disruptive Activities (Prevention) Act, 1987 (hereinafter referred to
as "The Act") stated initially in Section 1(4) thereof that the said Act will remain in
operation for a period of two years from 24.5.1987, but thereafter by amendments from
time to time the period of two years was extended to four years, then six years and lastly
for eight years.
22. Thus Section 1(4) of the Act as it stood ultimately read as follows :
"It shall remain in force for a period of [eight years] from the 24th day of May, 1987, but
its expiry under the operation of this sub-section shall not affect-
(a) the previous operation of, or anything duly done or suffered under this Act or any rule
made thereunder or any order made under any such rule, or
(b) any right, privilege, obligation or liability acquired, accrued or incurred under this Act
or any rule made thereunder or any order made under any such rule, or
(c) any penalty, forfeiture or punishment incurred in respect of any offence under this Act
or any contravention of any rule made under this Act or of any order made under any
such rule, or
(d) any investigation, legal proceeding or remedy in respect of any such right,
@page-SC968
privilege, obligation, liability, penalty, forfeiture or punishment as aforesaid
and any such investigation, legal proceeding or remedy may be instituted, continued or
enforced and any such penalty, forfeiture or punishment may be imposed as if this Act
had not expired."
23. There was no further extension of the period for which the Act remained in force, and
hence the Act automatically came to an end on 24.5.1995.
24. However, Section 1(4) of the Act also stated that the expiry of the aforesaid period of
the Act would not affect any right or liability incurred when the Act was in force, and
legal proceedings can be instituted or continued as if the Act has not expired, provided
the act in question was committed when the Act was in force. It is under this provision
that prosecutions under TADA have been continued although the life of the Act has
expired.
25. In my opinion the provision that legal proceedings can be instituted or continued even
after the Act has expired, is clearly violative of Article 14 of the Constitution of India. We
may test the constitutional position by taking a hypothetical case. Supposing a law is
made which makes an act committed by a person a crime on one day, but the same law
says that the same act will not be a crime if committed on the next day. Surely such a law
will be violative of Article 14 unless there is very strong and rational basis for such
classification and differentiation.
26. From the common sense point of view too, it does not stand to reason that a certain
act will be treated as a crime if committed within one time period, but it will not be a
crime if it is committed thereafter.
27. Of course, if there is good rational ground for making such a differentiation, Article
14 may not be violated but then it will be for the State Authorities to justify such a
classification on some reasonable and rational basis, failing which it will clearly violate
Article 14 of the Constitution.
28. In the present case, Section 1(4) of the Act says that the offence created by Section 3
of the Act will be punishable as a crime if the act was committed on or before 24.5.1995,
but if the same act was committed after 24.5.1995, it will not be a crime. To my mind this
is ex facie violation of Article 14 of the Constitution and hence Section 1(4) of Act to the
extent it says that acts mentioned in Section 3 committed on or before 24.5.1995 can still
be treated as a crime and punished under the TADA, though the same act committed after
24.5.1995 cannot, is in my opinion clearly ultra vires Article 14 and hence is liable to be
struck down as unconstitutional.
29. Since this point has not been raised in the appeal I am not giving any final opinion in
the matter, but the point, to my mind, is of such a vital and wide constitutional
importance that I thought it fit to express my opinion on the same, so that this opinion can
be considered in other cases where prosecutions under TADA are going on or where
convictions have been made in relation to the offences under Section 3 alleged to have
been committed before 24.5.1995.
30. As regards those who have already undergone the entire sentence for which they were
convicted under TADA obviously nothing can be done, but regarding those who have
undergone only part of the sentence or regarding those who are facing prosecution or
investigation under TADA such prosecution or investigation are liable to be quashed in
view of the opinion expressed above.
31. It is of course made clear that the above opinion has nothing to do with Article 20(1)
of the Constitution, which deals with ex post facto laws.
32. With the above observation the appeal is allowed and the impugned order is set aside.
The appellant is directed to be released forthwith unless required in some other case.
Appeal allowed.
AIR 2008 SUPREME COURT 968 "Hamdard Laboratories v. Deputy Labour
Commissioner"
(From : Allahabad)*
Coram : 2 S. B. SINHA AND MARKANDEY KATJU, JJ.
Civil Appeal No. 2204 of 2007 (arising out of SLP (C) No. 17526 of 2006), D/- 27 -4
-2007.
M/s. Hamdard (Wakf) Laboratories v. Deputy Labour Commissioner and Ors.
(A) Payment of Bonus Act (21 of 1965), S.22 - U.P. Industrial Disputes Act (28 of 1947),
S.6H(1) - BONUS - INDUSTRIAL DISPUTE - Claim for bonus - Has to be raised as an
industrial dispute -
@page-SC969
Application therefor under S. 6H(1) of 1947 Act - Would not be maintainable.
AIR 1986 Bom 340, Overruled. (Paras 36, 37, 38, 39)
(B) INTERPRETATION OF STATUTES - WORDS AND PHRASES - Interpretation of
Statutes - Definition clause - Word 'includes' - Is extensive - Words 'means and includes' -
However makes provision exhaustive. (Para 34)
(C) U.P. Industrial Disputes Act (28 of 1947), S.6C - INDUSTRIAL DISPUTE - Award -
Construction - Words used in award - Must be given same meaning as given in statute.
An award made in favour of one party and against the other must be clear and certain. A
person keeping in view the limited relief granted in favour of one party to the dispute
may not question the correctness or otherwise thereof. With a view to ascertain the
certainty in regard to the meaning of the words used by a competent Court of Law and
that too by an experienced judicial officer, they must be given the same meaning which
are given in a statute. A judgment, it is trite, must be reasonable. It must be construed in
such a manner so as not to offend the provisions of any statute. It must not be held to be
contrary to any statutory provisions. (Paras 33, 34)
(D) U.P. Industrial Disputes Act (28 of 1947), S.11B, S.2(y), S.6H - INDUSTRIAL
DISPUTE - BACK WAGES - BONUS - Back wages - Would not include bonus - Bonus
is not deferred wages - Claim for bonus for period back wages were allowed under award
- Not tenable under S. 6-H.
Spl. Appeal No. 443 of 2003, D/- 25-7-2006 (All.), Reversed. (Paras 20, 21, 22, 40)
Cases Referred : Chronological Paras
2007 AIR SCW 2972 : AIR 2007 SC 1782 (Rel. on, Pnt. B) 35
2001 AIR SCW 2212 : AIR 2001 SC 2370 (Rel. on, Pnt. C) 25
AIR 1986 Bom 340 : 1986 Lab IC 1055 (FB), (overruled) 11, 37
AIR 1969 SC 530 : 1969 Lab IC 854 (Rel. on, Pnt. A) 11, 31, 37
AIR 1958 SC 518 (Disting.) 30
AIR 1955 SC 170 (Rel. on, Pnt. D) 28, 40
Dinesh Dwivedi, S. B. Upadhyay, Sr. Advocates, Shiv Mangal Sharma, Ms. Sharmila
Upadhyay, for Appellant; Bharat Sangal, for Respondents.
* Special Appeal No. 443 of 2003, D/- 25-7-2006 (All.)
Judgement
S. B. SINHA, J. :- Leave granted.
2. Relationship between the parties hereto is employer and workmen. As far back in the
year 1983, the appellant terminated the services of 37 workmen allegedly on the ground
that they had gone on an illegal strike. It gave rise to an industrial dispute. The
management and its 19 workmen entered into compromise. One workman died during
pendency of the said dispute. Claim of 17 workmen, therefore, survived for adjudication
in the aforementioned industrial dispute. By an award dated 26.05.1993, the industrial
court, to which reference of the dispute was made by the appropriate government,
directed:
".........Accordingly, the Employers are directed to reinstate these 17 workers on duty on
the original post and payscale within one month after the date of publication of this
Award. So far as the question of back-wages is concerned, these workmen are to be paid
50% of their wages/ allowances which they were getting on 2-6-83, for the period 1-8-87
till the date of their joining the duty, within 2 months of publication of this Award. As
regards the deceased Komal Singh, his Provident Fund, Insurance money and wages/
allowances upto 30-9-91 to be calculated in the same manner as was paid on 2-6-83 and
50% of the same is to be paid by the Employer to his wife Smt. Shakuntala. This is my
Award in this dispute."
3. The said award ultimately attained finality as the writ petition preferred thereagainst by
the appellant was dismissed by an order dated 3.11.1995. A Special Leave Petition filed
thereagainst has also been dismissed.
4. On or about 2.08.1994, an application purported to be under Section 6-H(1) of the U.P.
Industrial Disputes Act, 1947 (for short "the Act") claiming backwages and bonus was
filed wherein the total amount of claim was for a sum of Rs. 20,70,020.44.
5. The Additional Labour Commissioner, however, on an objection raised by the
appellant to the effect that the amount of bonus could not be included in the claim
application issued a recovery certificate for a sum of Rs. 17,61,755.18. A review
application, however, was filed inter alia on the premise that the workmen were not
entitled to claim any bonus. By an order dated 9.11.1994, the said plea on the part of the
appellant was accepted as a result whereof the claim was reduced to Rs. 5,31,030.90. The
said direction admittedly has been complied with.
@page-SC970
6. The workmen, however, filed another application before the Labour Commissioner,
Ghaziabad claiming bonus for the period 1987 to 1996. In its objection filed thereto, the
appellant contended :
"It is respectfully submitted that the present claim of Bonus for the period 1987 to 1996
have been filed on the last date of hearing on 04.7.96. The workmen have earlier also
filed a claim u/S. 6-H(1) vide their application dtd. 02.8.94 and also submitted list
claiming Bonus, yearly increments, leave with wages, etc. The predecessor of the office
Sh. Arjun Ram the then Addl. Labour Commissioner heard the parties at length and
passed an order dtd. 26.9.94 amounting to Rs. 17,61,755.18. The employers/ management
filed an application to review the said order on 15.10.94. The review application was
heard in presence of the parties and the earlier order dtd. 26.9.94, was reviewed, order
modified to the extent of Rs. 5,31,030.00. The then Addl. Labour Commissioner rejected
the claim of Bonus, yearly increments etc. since the claim of Bonus yearly increments
etc. have already been rejected by a competent authority the same can't be heard again.
That the claim of Bonus does not fall in the definition of 'wages' as defined in Section
2(y) of the U.P. I.D. Act, 1947 hence the said claim cannot be maintainable u/S. 6-H(1) of
the U.P. I.D. Act, 1947 and deserves to be dismissed outrightly.
That the Hon'ble Labour Court (I), Ghaziabad who passed the Award in Adj. Case No.
275/87 have not given any consequential relief. Hence the workmen are not entitled to
any relief/ benefit such as Bonus, leave etc. for the period Sep. 87 to June 95.
That on perusal of the Award, dtd. 26.5.98 made by the Hon'ble Labour Court (I)
Ghaziabad, it is specifically mentioned in the conclusion at page No. 12 that the workmen
are only entitled to 50% back wages at the rate of wages which they were drawing on
2.6.83."
7. Rejecting the said contention, however, the Labour Commissioner, Ghaziabad, by an
order dated 8.08.1996 held :
"After hearing the parties, I have come to the conclusion that after the publication of the
Award, the employer has made the payment of wages to the workmen but did not
attribute them the work. Therefore, these all workmen are completely entitled for the
bonus, because bonus is deferred wage. All workmen are entitled for the bonus at the rate
on which other workmen have been paid bonus in the organization. Therefore, the
Management shall calculate the same for the period from 6.11.93 till the year 94-95. The
another issue is related to the grant of bonus for the period prior to the publication of
Award. In the Award in question, the Hon'ble Labour Court has passed the order only for
payment of the 50% of the wages to the Workmen on the issue of back wages. In this
regard, the recovery order passed by the Previous Ld. Addl. Commissioner does not
include the amount of bonus. The Hon'ble Court has not used the word "other benefits"
along with the Pay and allowances. But, in my opinion, the bonus is deferred wages and
the same is included in the Pay and salary. Therefore, I do not agree with this pleading of
the employer that the matter shall be referred to the Labour Court for interpretation of the
Payment/ Wage under Section 11(B) of the U.P. Industrial Disputes Act, 1947. Since in
the Award the order for payment of 50 per cent amount of back wages has been passed,
thus, accordingly the 50% of the bonus amount at the rate payable to other workmen of
the organization shall be payable..........."
8. A review application filed thereagainst was dismissed. A writ petition was filed by the
appellant before the Allahabad High Court aggrieved by and dissatisfied therewith. A
learned Judge of the said Court by an order dated 9.04.2003 held :
"Coming to the facts of the Writ Petition No. 35708 of 1996, the facts being the same,
claims being only for the payment of bonus for the disputed period. Once the employer
themselves have paid the wages upto the month of June, 1996, and since this Court has
also rejected the writ petition with regard to the payment of wages for the month of July,
1996, needless to say for the reasons and the ground stated in this judgment with regard
to writ petition No. 41691 of 1996, this writ petition also deserves to be dismissed and is
hereby dismissed."
9. An intra-Court appeal preferred thereagainst was dismissed by a Division Bench by
reason of the impugned judgment holding :
"........The accepted translation of these two Hindi words as amongst learned counsel
appearing is "wages and allowances".
@page-SC971
A submission is made that the definition of the word "wages" in the U.P. Industrial
Disputes Act, 1947 specifically excludes bonus. Therefore, it is argued, the mention of
wages in the award cannot include bonus and the passing of the Labour Commissioner's
order under Section 6-H(1) including bonus is without authority as the original award
cannot be said to have included it.
In our opinion, this argument suffers from a fallacy. The definition of the word "wages" is
meant for construing the U.P. Industrial Disputes Act. Such definition in the Act is not
meant to govern or limit the use of the word "wages" made by any and every authority
exercising jurisdiction under the Act or passing orders under the Act. The Labour Court's
award mentioning the phrase "wages and allowance" has to be read in its proper and
normal context. The Labour Commissioner did not in any manner misconstrue the said
two words in including bonus within the term wages and allowances. Simply put,
whatever the other similarly situated workers got during the period the seventeen
workmen were kept out of employment, and whatever the seventeen workmen would
have got themselves had they not been put out of employment improperly, they were to
get 50% of all that. That is the plain and simple reading of the Labour Court's award. The
order of the Labour Commissioner has proceeded on this basis. As such the challenge by
way of the second writ petition to payment of 50% bonus also fails."
10. Mr. Dinesh Dwivedi, learned senior counsel appearing on behalf of the appellant,
would submit that in view of the definition of 'wages' contained in Section2(y) of the Act
and Section 2(21) of the Payment of Bonus Act, in terms whereof bonus is neither wages
nor allowance; the Labour Commissioner committed a manifest error in directing
payment thereof on the spacious plea that it is deferred wages. It was urged that in order
to interpret a judgment, the terms used therein, in the event of any ambiguity, must be
interpreted in the light of the statute operating in the field.
11

. Mr. Bharat Sangal, learned counsel appearing on behalf of the respondents, on the other
hand, would submit that bonus being a part of 'remuneration', a claim in relation thereto
can also be made under the Payment of wages Act. It was submitted that the claim
petition was not filed for enforcement of the award but as an independent claim in terms
of the provisions of the Payment of Bonus Act in regard whereto an application under
Section 6-H(1) of the Act before the Labour Commissioner was maintainable. Strong
reliance in this behalf has been placed on Sanghi Jeevaraj Ghewar Chand and Ors. v.
Secretary, Madras Chillies, Grains, Kirana Merchants Workers' Union and Anr. [(1969)
1 SCR 366] and Kohinoor Tobacco Products Pvt. Ltd., Adyal v. Presiding Officer, Second
Labour Court, Nagpur and Others [AIR 1986 Bom 340]. AIR 1969 SC 530

12. The term 'Wages' has been defined in Section 2(y) of the Act in the following terms :
"2(y) 'wages' means all remuneration capable of being expressed in terms of money,
which would, if the terms of employment, expressed or implied, were fulfilled, be
payable to a workman in respect of his employment, or of work done in such
employment, and includes -
(i) such allowances (including dearness allowance) as the workman is for the time being
entitled to;
(ii) the value of any house accommodation, or of supply of light, water, medical
attendance or other amenity or of any service or of any concessional supply of foodgrains
or other articles;
(iii) any travelling concession;
but does not include-
(a) any bonus;
(b) any contribution paid or payable by the employer to any pension fund or provident
fund or for the benefit of the workman under any law for the time being in force;
(c) any gratuity payable on the termination of his service;"
[Emphasis supplied]
13. Section 2(rr) of the Industrial Disputes Act, 1947 defining the term 'Wages' is in pari
materia with Section 2(y) of the Act, 1947.
14. The term "salary or wage" has been defined under Section 2(21) of the Payment of
Bonus Act as under:
"(21) "salary or wage" means all remuneration (other than remuneration in respect of
over-time work) capable of being expressed in terms of money, which would, if the terms
of employment, express or implied, were fulfilled, be payable to an employee in respect
of his employment or of work done
@page-SC972
in such employment and includes dearness allowance (that is to say, all cash payments,
by whatever name called, paid to an employee on account of a rise in the cost of living),
but does not include-
(i) any other allowance which the employee is for the time being entitled to;
(ii) the value of any house accommodation or of supply of light, water, medical
attendance or other amenity or of any service or of any concessional supply of foodgrains
or other articles;
(iii) any travelling concession;
(iv) any bonus (including incentive, production and attendance bonus);
(v) any contribution paid or payable by the employer to any pension fund or provident
fund or for the benefit of the employee under any law for the time being in force;
(vi) any retrenchment compensation or any gratuity or other retirement benefit payable to
the employee or any ex gratia payment made to him;
(vii) any commission payable to the employee.
Explanation.- Where an employee is given in lieu of the whole or part of the salary or
wage payable to him, free food allowance or free food by his employer, such food
allowance or the value of such food shall, for the purpose of this clause, be deemed to
form part of the salary or wage of such employee;"
15. Section 2(vi) of the Payment of Wages Act, 1936 defines "wages" in the following
terms :
"(vi) "wages" means all remuneration (whether by way of salary, allowances, or
otherwise) expressed in terms of money or capable of being so expressed which would, if
the terms of employment, express or implied, were fulfilled, be payable to a person
employed in respect of his employment or of work done in such employment, and
includes-
(a) any remuneration payable under any award or settlement between the parties or order
of a Court;
(b) any remuneration to which the person employed is entitled in respect of overtime
work or holidays or any leave period;
(c) any additional remuneration payable under the terms of employment (whether called a
bonus or by any other name);
(d) any sum which by reason of the termination of employment of the person employed is
payable under any law, contract or instrument which provides for the payment of such
sum, whether with or without deductions, but does not provide for the time within which
the payment is to be made;
(e) any sum to which the person employed is entitled under any scheme framed under any
law for the time being in force,
but does not include-
(1) any bonus (whether under a scheme of profit sharing or otherwise) which does not
form part of the remuneration payable under the terms of employment or which is not
payable under any award or settlement between the parties or order of a Court;
(2) the value of any house-accommodation, or of the supply of light, water, medical
attendance or other amenity or of any service excluded from the computation of wages by
a general or special order of the State Government;
(3) any contribution paid by the employer to any pension or provident fund, and the
interest which may have accrued thereon;
(4) any travelling allowance or the value of any travelling concession;
(5) any sum paid to the employed person to defray special expenses entailed on him by
the nature of his employment; or
(6) any gratuity payable on the termination of employment in cases other than those
specified in sub-clause (d)."
16. Different statutes, enacted by the Parliament from time to time, although beneficial in
character to the workmen, seek to achieve different purposes. Different authorities have
been prescribed for enforcing the provisions of the respective statutes. The authority
under the Payment of Wages Act is one of them.
17. In view of the fact that diverse authorities exercise jurisdiction which may be
overlapping to some extent, the courts while interpreting the provisions of the statutes
must interpret them in such a manner so as to give effect thereto.
18. Section 6-H(1) of the Act provides for a proceeding which is in the nature of an
execution proceeding. The said provision can be invoked inter alia in the event any
money is due to a workman under an award. They cannot be invoked in a case where
ordinarily an industrial dispute can be raised and can be referred to for adjudication by
@page-SC973
the appropriate Government to an industrial court. The authorities to determine a matter
arising under Section 6-H(1) of the Act and an industrial dispute raised by the workmen
are different. Section 6-H(1) of the Act, it will bear repetition to state, is in the nature of
an execution provision. The authority vested with the power thereunder cannot determine
any complicated question of law. It cannot determine a dispute in regard to existence of a
legal right. It cannot usurp the jurisdiction of the State Government under Section 11-B of
the Act.
19. A Labour Commissioner is not a judicial authority. In view of Section 11-B of the
Act, it is for the State Government to construe an award, in the event any dispute arises in
giving effect thereto.
20. The Labour Court in its award directed reinstatement of 17 workmen on the original
post and payscale. No increment was granted; no continuity of service was directed. What
was directed was payment of 50% of the backwages/ allowance while considering the
question of backwages.
21. Definition of 'wages' within the meaning of the Act does not include "bonus". It,
however, includes allowance. Payment of Bonus Act also excludes bonus for the purpose
of calculating the amount of bonus to be determined in terms of Section 10 thereof.

22. Presiding Officer of the Labour Court is a judicial authority. He is supposed to know
the definition of 'wages' as contained in the Act. The rights and obligations of the parties
were being determined only under the Act and not in terms of any other law.
23. An award made in favour of one party and against the other must be clear and certain.
A person keeping in view the limited relief granted in favour of one party to the dispute
may not question the correctness or otherwise thereof. With a view to ascertain the
certainty in regard to the meaning of the words used by a competent court of law and that
too by an experienced judicial officer, they must be given the same meaning which are
given in a statute.
24. A judgment, it is trite, must be reasonable. It must be construed in such a manner so
as not to offend the provisions of any statute. It must not be held to be contrary to any
statutory provisions.
25

. In Gajraj Singh and Others v. State of U.P. and Others [(2001) 5 SCC 762], a 3-Judge
Bench of this Court held : 2001 AIR SCW 2212, Para 8

"...........A doubt arising from reading a judgment of the Court can be resolved by
assuming that the judgment was delivered consistently with the provisions of law and
therefore a course or procedure in departure from or not in conformity with statutory
provisions cannot be said to have been intended or laid down by the Court unless it has
been so stated specifically."
26. Bonus either in its ordinary meaning or statutory ones would not include wages.
27. What is a 'bonus' within the meaning of a provision before the coming into force of
Payment of Bonus Act, 1965 came up for consideration before this Court on various
occasions. Although reference thereto may not be strictly necessary, as the learned
counsel appearing for the parties have referred to the same, we may take notice thereof.
28

. In Muir Mills Co. Ltd. v. Suti Mills Mazdoor Union, Kanpur [(1955) 1 SCR 991], this
Court held : AIR 1955 SC 170, Para 14

"It is therefore clear that the claim for bonus can be made by the employees only if as a
result of the joint contribution of capital and labour the industrial concern has earned
profits. If in any particular year the working of the industrial concern has resulted in loss
there is no basis nor justification for a demand for bonus. Bonus is not a deferred wage.
Because if it were so it would necessarily rank for precedence before dividends. The
dividends can only be paid out of profits and unless and until profits are made no
occasion or question can also arise for distribution of any sum as bonus amongst the
employees. If the industrial concern has resulted in a trading loss, there would be no
profits of the particular year available for distribution of dividends, much less could the
employees claim the distribution of bonus during that year............"
29. Bonus may be a deferred wage but the same must be construed in a different context.
When used in the context of 'backwages' and that too 50% of it, the same would not
include backwages. It is expected that had the Labour Court intended to include the same,
he would have explicitly said so. Even now, under the Payment of Wages Act, bonus does
not come within the purview of wages. The decision was rendered
@page-SC974
when Payment of Bonus Act had not been enacted.
30

. The question came up for consideration, yet again, in Bala Subrahmanya Rajaram v.
B.C. Patil and Others [(1958) SCR 1504] wherein bonus was equated with remuneration
but therein the question which arose for consideration was the quantum of bonus and in
that context the court went into the question as to whether the same can be claimed under
the provisions of the Payment of Wages Act. When the bonus was considered to be a part
of remuneration, what was in the mind of this Court, was the definition of 'wages' under
the Payment of Wages Act, as it existed at the relevant time. In the factual matrix
obtaining therein, this Court held that 'bonus' would come within the purview of the term
'remuneration'. Evidently, 'bonus' would not come within the meaning of the said term as
it stands now and in view of the controversy involved herein, particularly, in view of the
fact that 'bonus' now stands explicitly excluded by reason of the Payment of Wages
(Amendment) Act, 1957 which came into effect from 1.04.1958. This Court therein had
no occasion to consider the question with which we are beset with. AIR 1958 SC 518

31

. In Sanghi Jeevaraj Ghewar Chand (supra), this Court took into consideration the history
of the term "bonus" stating that a claim in regard to bonus can be raised under the
provisions of the Industrial Disputes Act. Having regard to Sections 22 and 39 of the
Payment of Bonus Act, it was stated :AIR 1969 SCW 530, Para 6

"..........If a dispute, for instance, were to arise as regards the quantum of available
surplus, such a dispute not being one falling under Section 22, Parliament had to make a
provision for investigation and settlement thereof. Though such a dispute would not be an
industrial dispute as defined by the Industrial Disputes Act or other corresponding Act in
force in a State, Section 39 by providing that the provisions of this Act shall be in
addition to and not in derogation of the Industrial Disputes Act or such corresponding law
makes available the machinery in that Act or the corresponding Act available for
investigation and settlement of industrial disputes thereunder for deciding the disputes
arising under this Act. As already seen Section 22 artificially makes two kinds of disputes
therein referred to industrial disputes and having done so applies the provisions of the
Industrial Disputes Act and other corresponding law in force for their investigation and
settlement. But what about the remaining disputes? As the Act does not provide any
machinery for their investigation and settlement, Parliament by enacting Section 39 has
sought to apply the provisions of those Acts for investigation and settlement of the
remaining disputes, though such disputes are not industrial disputes as defined in those
Acts. Though, the words "in force in a State" after the words "or any corresponding law
relating to investigation and settlement of industrial disputes" appear to qualify the words
"any corresponding law" and not the Industrial Disputes Act, the Industrial Disputes Act
is primarily a law relating to investigation and settlement of industrial disputes and
provides machinery therefor. Therefore the distinction there made between that Act and
the other laws does not seem to be of much point. It is thus clear that by providing in
Section 39 that the provisions of this Act shall be in addition to and not in derogation of
those Acts, Parliament wanted to avail of those Acts for investigation and settlement of
disputes which may arise under this Act. The distinction between Section 22 and Section
39, therefore, is that whereas Section 22 by fiction makes the disputes referred to therein
industrial disputes and applies the provisions of the Industrial Disputes Act and other
corresponding laws for the investigation and settlement thereof, Section 39 makes
available for the rest of the disputes the machinery provided in that Act and other
corresponding laws for adjudication of disputes arising under this Act. Therefore, there is
no question of a right to bonus under the Industrial Disputes Act or other corresponding
Acts having been retained or saved by Section 39. Neither the Industrial Disputes Act nor
any of the other corresponding laws provides for a right to bonus. Item 5 in Schedule 3 to
the Industrial Disputes Act deals with jurisdiction of tribunals set up under Sections 7, 7-
A and 7-B of that Act, but does not provide for any right to bonus. Such a right is
statutorily provided for the first time by this Act."
32. The Labour Court was not determining any right under the Payment of Bonus Act. It
was while making its award determining the rights and liabilities under the Act.
@page-SC975
33. It, therefore, must have in mind the provisions of the Act alone. The aforementioned
decisions, therefore, have no application to the facts and circumstances of the present
case.
34. When an interpretation clause uses the word "includes", it is prima facie extensive.
When it uses the word "mean and include", it will afford an exhaustive explanation to the
meaning which for the purposes of the Act must invariably be attached to the word or
expression [See G.P. Singh's Principles of Statutory Interpretation, 10th Edition, Pages
173 and 175].
35

. Recently, in N.D.P. Namboodripad (Dead) by L.Rs. v. Union of India (UOI) and Ors.
[2007 (4) Scale 361], this Court held : 2007 AIR SCW 2972

"17. If the words 'and includes' were intended to rope in certain items which would not be
part of the meaning, but for the definition, then Rule 62 would have specified only
'dearness pay' as the item to be included but not 'pay'. If pay, dearness allowance and
other allowances were already included in 'emolument' with reference to its general or
normal meaning, as contended by appellant, there was no reason to specifically again
include 'pay' in Rule 62. Inclusion of 'pay' and 'dearness pay' and non-inclusion of
'dearness allowance or other allowances' in the definition of 'emolument' is significant.
The definition in Rule 62 is intended to clarify that only pay and dearness pay would be
considered as 'emolument' for purposes of calculating pension. The words 'and includes'
have been used in Rule 62, as meaning 'comprises' or 'consists of."
36. There is yet another aspect of the matter which cannot be lost sight of. A claim for
bonus in the context of Section 22 of the Payment of Bonus Act can be raised only by
raising an industrial dispute. It cannot be raised by way of an execution application. If a
claim had been made under an award, the same attained finality when the amount payable
thereunder had been calculated. Bonus was a subject matter of claim in the first
application filed under Section 6-H(1) of the Act. The amount payable thereunder had
been determined. Another application under Section 6-H(1) of the Act for the purpose of
enforcement of award, therefore, was, in our opinion, not maintainable.
37

. When the second application was filed, the same was dehors the award. It was an
independent claim. Such an independent claim, thus, on a plain reading of Section 22 of
the Payment of Bonus Act could have been raised as an industrial dispute in the light of
the decision of this Court in Sanghi Jeevaraj Ghewar Chand (supra). The decision of the
Full Bench of the Bombay High Court in Kohinoor Tobacco Products Pvt. Ltd (supra), in
our opinion, to that extent is not correct. When the statute provides for a remedy in a
particular manner, the same cannot be achieved by filing an application which subserves
a different purport and object.AIR 1969 SC 530
AIR 1986 Bom 340

38. Such an application was, thus, not maintainable under Section 6-H(1) of the Act
which corresponds to Section 33C(1) of the Industrial Disputes Act. Even the jurisdiction
of a Labour Court in terms of Section 33C(2) of the Industrial Disputes Act would be
limited.
39. An application under Section 33C(1) of the Industrial Disputes Act, 1947 must be for
enforcement of a right. If existence of right, thus, is disputed, the provisions may not be
held to have any application.
40

. The Labour Commissioner in view of the decision of this Court in Muir Mills Co. Ltd.
(supra) has evidently committed a manifest error in opining that bonus is deferred wages.
Once it is excluded from the purview of the term 'wages' under the Act, such a view was
impermissible in law, particularly, when the appellant denied and disputed the right of the
workmen to claims. Both the learned Single Judge and the Division Bench of the High
Court also fell to the same error. The learned Judges even did not address themselves the
right questions. They, thus, misdirected themselves in law. AIR 1955 SC 170

41. We, therefore, are of the opinion that the impugned judgment cannot be sustained
which is set aside accordingly. The appeal is allowed. No costs.
Appeal allowed.
@page-SC976
AIR 2008 SUPREME COURT 976 "Maharashtra Electricity Regulatory Commission v.
Reliance Energy Ltd"
Coram : 2 A. K. MATHUR AND MARKANDEY KATJU, JJ.
Civil Appeal No. 2846 of 2006 with C. A. No. 3551 of 2006, D/- 14 -8 -2007.
Maharashtra Electricity Regulatory Commission v. Reliance Energy Ltd. and Ors.
(A) Electricity Act (36 of 2003), S.86(1)(f ) -ELECTRICITY - STATE COMMISSION -
OBJECT OF AN ACT - Function of State Commission - Adjudicatory function - Scope -
Limited to matter prescribed in S. 86(1)(f) - Cannot adjudicate disputes relating to
grievances of individual consumers. (Para 12)
(B) Electricity Act (36 of 2003), S.128(6) -ELECTRICITY - COMMISSIONS -
Conditions of licence - Enforcement of - Commission is empowered under S. 128(6) -
Issuing supplementary/amended bills and charging excess amounts for supply of
electricity by licencees/distribution companies - Complaints by consumers - Blanket
direction/order by Commission directing refund of amounts collected by
licensees/distribution companies without making any investigation - Not permissible -
Licensees/distribution companies directed to decide individual grievances received by
them, on giving public notice, and after giving a fair opportunity of hearing to the
consumers. (Paras 14, 15, 16, 17, 18, 19)

Altaf Ahmad, Sr. Advocate, M. G. Ramachandran, K. V. Balakrishnan, Anand K.


Ganesan, K. V. Mohan, Varun Thakur, A. S. Bhasme, for Appellant; J. J. Bhat, Shyam
Divan and Rajiv Shakdher, Sr. Advocate, Ms. Sunita Chandurkar, Syed Naqvi, Ms.
Smieetaa Inna, Rajesh Kumar, Ms. Swati Sinha, Ms. Jayasree Singh, Ms. Shefali Jain (for
M/s. Fox Mandal and Co.), Prashant Bhushan, Ramji Srinivasan, M. V. Kini, Ravi Kini,
V. B. Joshi (for M/s. P. H. Parekh and Co.), Ms. Rukhsana Choudhury, U. A. Rana,
Abhishek Rao and Nitesh Jain (for M/s. Gagrat and Co.), for Respondents.
Judgement
JUDGMENT :-This appeal under Section 125 of the Electricity Act, 2003 (hereinafter for
short "the Act") is directed against the judgment and order dated 29th March, 2006 passed
by the Appellate Tribunal for Electricity whereby the Appellate Tribunal has allowed the
appeals filed by the distribution companies and set aside the orders passed by the
Maharashtra Electricity Regulatory Commission (hereinafter for short "The
Commission") dated 23.2.2005. The Commission on 3.8.2004 addressed a notice to all its
licensees/distribution companies in Maharashtra and made an inquiry from them with
regard to raising of the bills by the said licensees/distribution companies on the basis
other than the actual meter reading for the relevant period, when large variations in
consumption were noticed, or for other reasons. The notice dated 3.8.2004 sent by the
Commission to all its licensees/distribution companies reads as under :-
"Several instances have come to the Commission's notice of so-called "amendment",
"supplementary" or other such bills being raised by some licensees to consumers, often
several years later, on a basis other than the actual meter reading for the relevant period,
when large variations in consumption are noticed, or for other reasons. Computerised
systems have sometimes been put in place which generate such bills automatically.
Wide variations observed in recorded consumption and other such apparent anomalies
may be useful for monitoring, checking/testing of meters and for taking corrective action.
However, billing on a basis other than recorded consumption, and raising amended bills
accordingly (often after several years later, and without giving reasons), is not mandated
by law.
The electricity statutes (in the past, and at present) provide inter alia that, in case of
metered consumers, energy consumption charges have to be billed on the basis of meter
readings. Moreover, the licensee, and not the consumer, is responsible for maintaining,
rectifying, or having such meters replaced where necessary. Thus, no "amendment" bills
of the kind referred to above can be raised, and any additional billing has to follow due
process and the provision of law.
In the context of such "amendment" bills, I am directed to ask that the billing practices
followed be immediately reviewed and brought into conformity with the statutory
provisions. An affidavit stating the corrective action taken (including withdrawal of all
such pending bills, and refund, though adjustment in energy bills or otherwise, of
amounts received from consumers on or after 10.6.2003 may be furnished by 3rd
September, 2004."
2. In response to the said notice all the
@page-SC977
licensees/distribution companies in Maharashtra made their respective submissions
before the Commission explaining under what circumstances the supplementary/amended
bills were sent to the consumers. They tried to justify raising of such bills and stated that
these bills were rightly sent as they found that some time the meters were not registering
proper consumption and on that basis they tried to justify their action.
3. The Commission examined the matter in detail and vide its order dated 23.2.2005 in
para 46 directed as under :-
"46. After considering all these factors and the submissions made, the Commission
directs that the supplementary/amendment bills issued in the circumstances set out at
paras 42 and 43 above from 10th June, 2003 (the date of coming into force of EA, 2003)
and upto notification of the Supply Code.
a. should be withdrawn, if due meter testing has not been done with the results intimated
to the consumer.
b. any amounts collected should be refunded to the concerned consumers (without
interest considering the earlier lack of clarity on this meter on the part of the licensees);
c. where meters have been found to be defective upon subsequent due testing (and the
results intimated to the consumer), the bills may be adjusted for upto 3 months prior to
the date of testing or meter replacement, whichever is earlier, and any amounts recovered
in excess refunded without interest (in the case of 'stopped' meters, the analogy of the
Supply Code provisions should be applied for assessment);
d. the above action should be completed by 30th May, 2005, so as to give the licensees
more than 3 months' time in view of the work likely to be involved;
e. compliance should be submitted on affidavit by 15th June, 2005, with a list of
consumers involved, and certifying that no further action remains to be done in terms of
this Order."
By another order dated 23.8.2005 in the case of M/s. Prayas (Energy Group) Pune, the
Commission in para 45 directed as under :-
"45. Considering the foregoing, the Commission disposes of Prayas' petition with the
following directions, which would apply for the period from 1st June, 2004 (i.e. around 3
months after the detailed Tariff Order dated 10th March, 2004, uptil 19th January, 2005
(following which the Supply Code Regulations were notified) :
(a) no billing using past consumption or some related 'average' basis should be resorted to
for more than a period of 3 months. (Where average billing has been continuing for more
than that period just prior to 1st June, 2004, then it cannot be continued from that date. In
case average billing has been resorted to for, say, 2 months prior to that, it can be
continued only for upto one month more). During that period of 3 months, the meter
should have been tested/replaced, with the results intimated to the consumer, and
appropriate bill adjustments carried out thereafter (where such average billing is being
done on the basis of presumed faulty meter, and where defectiveness of the meter has
accordingly been established). If due and timely diligence has not been exercised by the
licensee, he cannot claim the right to continue billing on a presumptive, average basis.
The same principle will apply to all other situations in which such 'average' billing has
been resorted to, except in cases where the meter is not accessible. (However, the
Commission notes that, in the case of locked/inaccesible meters, the licensees have
recourse to the remedies provided under Section 163 of EA, 2003, and it would be
expected that MSEB would exercise it sooner rather than later).

(b) In all cases where bills have been raised and/or recoveries made which are not in
accordance with (a) above, the bills should be withdrawn and/or amounts refunded to the
consumers, through energy bills or other means, as may be relevant, by 30th November,
2005, with interest at the same rate as payable by consumers to MSEB for delayed
payments."
4. Aggrieved against both these orders, the matter was taken up in appeal before the
Appellate Authority. The Appellate Authority by the impugned order dated 29th March,
2006 set aside the orders passed by the Commission and directed that each consumer
should approach the forum created under Section 42(5) of the Act for the individual
grievances.
5. Aggrieved against the order dated 29.3.2006 passed by the Appellate Authority, the
present appeal has been filed under Section 125 of the Act.
@page-SC978
6. We have heard learned counsel for the parties and perused the record.
7. Learned counsel for the appellant-Commission has submitted before us that the
Commission has the power to give a general direction to its licensees/distribution
companies and he also submitted that in exercise of the power under the Act, the
Commission was competent to issue the aforesaid direction. As against this, the learned
counsel appearing for the respondent-licensees/distribution companies submitted that the
Commission has no power to issue a direction like the one issued in the present case and
entertain individual complaints and direct refund of the whole amount by a blanket order.
8. The question before us is : what is the power of the Commission and to what extent the
Commission can issue directions. Suffice it to say that the Regulatory Commission was
constituted under the Electricity Act, 2003. The Act was a new enactment which was
promulgated by superseding the Indian Electricity Act, 1910 and the Electricity (Supply)
Act, 1948. The Statement of Objects and Reasons of the Act which have been
summarized in the Preamble, reads as under :-
"An Act to consolidate the laws relating to generation, transmission, distribution, trading
and use of electricity and generally for taking measures conducive to development of
electricity industry, promoting competition therein, protecting interest of consumers and
supply of electricity to all areas, rationalisation of electricity tariff, ensuring transparent
policies regarding subsidies, promotion of efficient and environmentally benign policies,
constitution of Central Electricity Authority, Regulatory Commissions and establishment
of Appellate Tribunal and for matters connected therewith or incidental thereto."
"Appropriate Commission" as defined in Section 2(4) of the Act means the "Central
Regulatory Commission referred to in sub-section (1) of Section 76 or the State
Regulatory Commission referred to in Section 82 or the Joint Commission referred to in
Section 83, as the case may be". In exercise of its power under Section 82 of the Act, the
State of Maharashtra constituted the Maharashtra Electricity Regulatory Commission.
The Commission exercises all the powers which are enumerated in the Act. Though
various provisions were pointed out to us by learned counsel for the parties, but Section
82 which is relevant for our purposes reads as under :-
"82. Constitution of State Commission- (1) Every State Government shall, within six
months from the appointed date, by notification, constitute for the purposes of this Act, a
Commission for the State to be known as the (name of the State) Electricity Regulatory
Commission :
Provided that the State Electricity Regulatory Commission, established by a State
Government under Section 17 of the Electricity Regulatory Commissions Act, 1998 (14
of 1998) and the enactments specified in the Schedule, and functioning as such
immediately before the appointed date shall be the State Commission for the purposes of
this Act and the Chairperson, Members, Secretary, and officers and other employees
thereof shall continue to hold office on the same terms and conditions on which they were
appointed under those Acts :
Provided further that the Chairperson and other Members of the State Commission,
appointed, before the commencement of this Act, under the Electricity Regulatory
Commissions Act, 1998 (14 of 1998) or under the enactments specified in the Schedule,
may, on the recommendations of the Selection Committee constituted under sub-section
(1) of Section 85, be allowed to opt for the terms and conditions under this Act by the
concerned State Government.
(2) The State Commission shall be a body corporate by the name aforesaid, having
perpetual succession and a common seal, with power to acquire, hold and dispose of
property, both movable and immovable, and to contract and shall, by the said name, sue
or be sued.
(3) The head office of the State Commission shall be at such place as the State
Government may, by notification, specify.
(4) The State Commission shall consist of not more than three Members, including the
Chairperson.
(5) The Chairperson and Members of the State Commission shall be appointed by the
State Government on the recommendation of a Selection Committee referred to in
Section 85."
9. We are not concerned with the provisions of appointment of Members of the
Commission as they are dealt with by Sections 84 and 85 of the Act. Section 86 deals
@page-SC979
with the functions of the Commission and is relevant for our purposes. For ready
reference, the same is reproduced hereunder.
"86. Functions of State Commission :- (1) The State Commission shall discharge the
following functions, namely :-
(a) determine the tariff for generation, supply, transmission and wheeling of electricity,
wholesale, bulk or retail, as the case may be, within the State :
Provided that where open access has been permitted to a category of consumers under
section 42, the State Commission shall determine only the wheeling charges and
surcharge thereon, if any, for the said category of consumers;
(b) regulate electricity purchase and procurement process of distribution of licensees
including the price at which electricity shall be procured from the generating companies
or licensees or from other sources through agreements for purchase of power for
distribution and supply within the State;
(c) facilitate intra-State transmission and wheeling of electricity;
(d) issue licences to persons seeking to act as transmission licensees, distribution
licensees and electricity traders with respect to their operations within the State;
(e) promote congeneration and generation of electricity from renewable sources of energy
by providing suitable measures for connectivity with the grid and sale of electricity to any
person, and also specify, for purchase of electricity from such sources, a percentage of the
total consumption of electricity in the area of a distribution licensee;
(f) adjudicate upon the disputes between the licensees and generating companies and to
refer any dispute for arbitration;
(g) levy fee for the purposes of the Act;
(h) specify State Grid Code consistent with the Grid Code specified under clause(h) of
sub-section (1) of Section 79;
(i) specify or enforce standards with respect to quality, continuity and reliability of
service by licensees;
(j) fix the trading margin in the intra-State trading of electricity, if considered, necessary;
(k) discharge such other functions as may be assigned to it under this Act.
(2) The State Commission shall advise the State Government on all or any of the
following matters, namely :-
(i) promotion of competition, efficiency and economy in activities of the electricity
industry;
(ii) promotion of investment in electricity industry;
(iii) reorganisation and restructuring of electricity industry in the State;
(iv) matters concerning generation, transmission, distribution and trading of electricity or
any other matter referred to the State Commission by that Government;
(3) The State Commission shall ensure transparency while exercising its powers and
discharging its functions.
(4) In discharge of its functions, the State Commission shall be guided by the National
Electricity Policy, National Electricity Plan and Tariff Policy published under Section 3.
10. Thus, from the above provision it is clear that the primary purpose of the Commission
is to determine tariff for generation, supply, transmission of electricity etc. and to regulate
the electricity purchase and procurement process of distribution licensees, to facilitate
intra-State transmission, to promote congeneration and generation of electricity from
renewable sources of energy, to adjudicate upon the disputes between the licensees and
generation companies and to refer any dispute for arbitration, to levy fee for the purposes
of this Act, specify State Grade Code consistent with the Grid Code specified under
clause (h) of sub-section (1) of Section 79. Sub-section (2) of Section 86 also empowers
the State Commission to advise the State Government on any of the matters including
promotion of competition, efficiency, matters concerning generation, transmission,
distribution and trading of electricity etc. Sub-section (3) provides that the Commission
shall ensure transparency while exercising its powers and discharging its functions. Sub-
section (4) provides that in discharge of its functions the Commission shall be guided by
the National Electricity Policy, National Electricity Plan and Tariff Policy published
under Section 3.
11. We are not going into other questions as to how licenses were granted to all these
utilities, i.e., various distribution companies. It is not necessary for us to go into these
questions as in the present case, we are primarily concerned to decide the powers of the
Commission and to what extent it
@page-SC980
can issue directions and whether the direction given by the Commission in the present
case is sustainable or not.
12. It may be noted from a perusal of Section 86(1)(f) of the Act that the State
Government has only power to adjudicate upon disputes between licensees and
generating companies. It follows that the Commission cannot adjudicate disputes relating
to grievances of individual consumers. The adjudicatory function of the Commission is
thus limited to the matter prescribed in Section 86(1)(f).
13. Section 14 of the Act provides for grant of licence; Section 16 provides for conditions
of licence; Section 61 lays down the tariff regulations and Section 62 provides for
determination of tariff. The Commission under Section 94 has civil powers also and
under Section 96 it has power of entry and seizure. Under Section 126 the Commission
has the power to investigate and make assessment. Section 127 provides for an appeal to
the appellate authority. Under Section 128 the Commission can make investigation of
certain matters where it is satisfied that the licensee has failed to comply with any of the
conditions of licence or failed to comply with any of the provisions of the Act or the
Rules and Regulations made thereunder. Sub-section (6) of Section128 empowers the
Commission to take any action against the licensee/generating company. Sub-section (6)
reads as under :-
(6) On receipt of any report under sub-section (1) or sub-section (5), the Appropriate
Commission may, after giving such opportunity to the licensee or generating company, as
the case may be, to make a representation in connection with the report as in the opinion
of the Appropriate Commission seems reasonable, by order in writing :-
(a) require the licensee or the generating company to take such action in respect of any
matter arising out of the report as the Appropriate Commission may think fit; or
(b) cancel the licence; or
(c) direct the generating company to cease to carry on the business of generation of
electricity.
Section 142 of the Act provides for punishment for non-compliance of directions issued
by the Commission and Section 143 empowers the Commission to adjudicate after
holding an inquiry in such manner as may be prescribed by the Government. Section 181
empowers the Commission to make regulations.
14. A comprehensive reading of all these provisions leaves no manner of doubt that the
Commission is empowered with all powers right from granting licence and laying down
the conditions of licence and to frame Regulations and to see that the same are properly
enforced and also power to enforce the conditions of licence under sub-section (6) of
Section 128.
15. Thus, insofar as the first contention of the learned counsel for the respondents that the
Commission has no power is concerned, we are of the view that the same is wrong. In
this behalf the provisions of The Electricity Act, 2003 are quite clear and categoric and
Section 128(6) empowers the Commission to get the conditions of licence enforced. But
the question is whether the said power under Section128(6) has been rightly exercised by
the Commission or not. After clearing the first hurdle, that the Commission has power to
issue directions, we shall now examine whether the direction given by the Commission in
the present case is correct or not.
16. When the Commission received a spate of complaints from consumers against its
licensees/distribution companies that they are arbitrarily issuing supplementary/amended
bills and charging excess amounts for supply of electricity, it felt persuaded to invoke its
general power to supervise the licensees/distribution companies and in that connection
issued notice dated 3.8.2004. There can be no manner of doubt that the Commission has
full power to pull up any of its licensee or distribution company to see that the Rules and
Regulations laid down by the Commission are properly complied with. After all, it is the
duty of the Commission under Sections 45(5), 55(2), 57, 62, 86, 128, 129, 181 and other
provisions of the Act to ensure that the public is not harassed.
17. In exercise of this general power notice dated 3.8.2004 was issued when mass scale
supplementary/amended bills were issued to the consumers. When these consumers
approached the Commission, the Commission directed its licensees to immediately
review their billing policies and bring the same in conformity with the statutory
provisions of the Act. The Commission did not get an investigation made under Section
128(1) which it could have done, and
@page-SC981
without that, and without getting a report under Section 128(5) it passed an order
directing refund of the amounts collected by the licensees/distribution companies, which
in our opinion was not permissible, since such a direction could, if at all, be given after
getting a report of the investigation agency. The Commission could have made an
investigation and got a report from the investigation agency and on that basis directions
could have been given. However, that was not done. In these circumstances, in our
opinion, the view taken by the Appellate Authority in the impugned order to that extent is
correct that the individual consumers should have approached the appropriate forum
under Section 42(5) of the Act.
18. Thus while we hold that the Commission has power to issue a general direction to
licencees that they should abide by conditions of the licence issued by them and charge
only as per the tariff fixed under the Act so that the public at large should not be harassed,
we are of the opinion that so far as the blanket direction given by the Commission for
refunding the entire amount without making a proper investigation whether the issue of
supplementary/amended bills was really warranted in every case or not is unsustainable.
Here the Commission has gone beyond its jurisdiction. After all the distribution/
generating companies have to incur expenses for generation/distribution of power, and
we cannot at the same time give license to the consumers to commit theft of electricity or
to be benefited by improper functioning of the meter to the disadvantage of the
distribution/generating company. Thus, keeping in view the equity of both the parties, we
think it will be proper for us to direct that all the licensees/distribution companies in the
State of Maharashtra issue a general public notice in two daily newspapers having wide
circulation in the State, one English newspaper and one in vernacular language. The
notice shall state that whoever feels aggrieved by the supplementary/amended bill, he/she
can approach the licensee/distribution company for redressal of their grievance within a
period of three months from the date of publication of the notice. In our view, that would
meet the ends of justice instead of passing a blanket order as given by the Commission
for refunding the money charged by the licensees/distribution companies by issuing
supplementary/amended bills. The individual consumers may make a grievance before
the licensee/distribution company that they have not consumed the electricity for which
they are charged or that the meter reading was not proper or that they have been
excessively charged for the power which they have not actually consumed. Therefore, we
direct that all the licensees/distribution companies shall issue a public notice in two daily
newspapers having wide circulation in the State of Maharashtra, one in English language
and the other in vernacular language requiring their respective consumers to make their
representations for redressal of their grievances in respect of the supplementary/amended
bills. The licensees/distribution companies shall decide the individual cases received by
them after giving a fair opportunity of hearing to the consumers. The consumers who still
feel not satisfied with the order passed by the licensees/distribution companies can
approach the appropriate forum constituted under Section 42(5) of the Act and, if still not
satisfied, with the order passed by the appropriate forum to approach the Ombudsman
under Section 42(6) of the Act. Accordingly, we hold that while the Commission had a
power to issue general directions to prevent harassment to the public at large by its
licensees/distribution companies, but a blanket direction to refund the amounts collected
by the licensees/distribution companies which has been given by the Commission was not
warranted.
19. Although, the Appellate Authority has set aside the order passed by the Commission
and issued a direction that the individual consumers may approach the appropriate orders
under Sections 42(5) and (6) we are not interfering with that direction, but we direct that
before that the licensees/distribution companies shall hear the parties as directed
hereinabove and decide whether the supplementary/amended bills issued by them are
proper or not.
20. In view of the above discussion, this appeal stands disposed of with no order as to
costs.
Order accordingly.
@page-SC982
AIR 2008 SUPREME COURT 982 "State of Rajasthan v. Jaggu Ram"
(From : Rajasthan)*
Coram : 2 G. P. MATHUR AND G. S. SINGHVI, JJ.
Criminal Appeal No. 1133 of 2000, D/- 4 -1 -2008.
State of Rajasthan v. Jaggu Ram.
(A) Penal Code (45 of 1860), S.304B - DOWRY DEATH - Dowry death - Husband of
deceased charged along with his father and mother - Husband not present in village at
time of occurrence - Acquittal of husband by giving benefit of doubt - Would not justify
conclusion that prosecution was unable to prove charge against father-in-law and mother-
in-law of deceased. (Para 17)
(B) Penal Code (45 of 1860), S.304B - Evidence Act (1 of 1872), S.106 - DOWRY
DEATH - EVIDENCE - Dowry death - Subsequent conduct of accused - Relevancy -
Death of deceased due to head injuries within one and half years of marriage -
Harassment for dowry proved - No satisfactory explanation given by accused for head
injuries of deceased - Accused and his family members cremating deceased without
informing her parents or police - Conduct of accused clearly establish that accused had
done so with sole object of concealing the real cause of death - Accused liable to be
convicted.
S. B. Cri. Misc. Appeal No. 387 of 1995, D/- 27-10-1999 (Raj.), Reversed. (Paras 18, 19,
20, 23)
Cases Referred : Chronological Paras
2007 AIR SCW 295 : 2007 Cri LJ 1174 (Ref. Pnt. B) 22
2006 AIR SCW 4068 : AIR 2006 SC 2855 : 2006 Cri LJ 4070 11
2006 AIR SCW 5300 : 2007 Cri LJ 20 (Rel. on) 21
2006 AIR SCW 5768 : AIR 2007 SC 144 (Ref. Pnt. B) 22
2005 AIR SCW 770 : AIR 2005 SC 785 : 2005 Cri LJ 1418 11
2004 AIR SCW 1566 : AIR 2004 SC 1933 : 2004 Cri LJ 1791 (Ref.) 11
(2004) 8 SCC 251 (Ref.) 11
2003 AIR SCW 4065 : AIR 2003 SC 3609 : 2003 Cri LJ 3892 (Ref. Pnt. B) 21
2003 AIR SCW 4387 : AIR 2003 SC 3828 : 2003 Cri LJ 4321 11
2001 AIR SCW 3802 : AIR 2001 SC 2842 : 2001 Cri LJ 4632 (Rel.on) 20
1944 AC 315 21
Naveen Kumar Singh (for Aruneshwar Gupta), for Appellant; Badri Prasad Singh, for
Respondent.
* S. B. Cri. Misc. Appeal No. 387 of 1995, D/- 27-10-1999 (Raj.)
Judgement
G. S. SINGHVI, J. :-This appeal is directed against the judgment dated 27.10.1999 of the
learned Single Judge of Rajasthan High Court whereby he allowed the appeal preferred
by respondent Jaggu Ram and acquitted him of the charge under Sections 304-B and 201,
Indian Penal Code.
2. The facts necessary for deciding the appeal are as under :
On 30th March, 1993 one Suresh Khateek informed Atma Ram (PW-1) that his daughter
Shanti @ Gokul had died at her in-laws place. Upon this Atma Ram lodged First
Information Report at Police Station, Srimadhopur stating therein that his daughter Shanti
@ Gokul was married to Jeevan Ram, son of Jagdish Balai (Jaggu Ram), resident of
Nathusar about eighteen months ago; that he gave dowry according to his capacity; that
immediately after the marriage, Jaggu Ram, his son Jeevan Ram and wife Nathi started
harassing Shanti in connection with dowry; that after three days of marriage they left
Radio, Press etc. and demanded watch and jewellery; that they with a view to save his
daughter from harassment, he gave silver ornaments viz. Paizeb (anklet), Tagadi, Locket
and Ear-rings apart from cash of Rs. 10,000/- to her in-laws, but this did not satisfy them
and after three months, they again started harassing her and left her at village Abhawas,
where she stayed at his house for 8 months; that a meeting was held at village Abhawas
which was attended by 22-25 people including Jaggu Ram (the respondent herein),
Rameshwar Mali, Chhitar Khateek and Bhagega Balai of village Nathusar. In that
meeting Jaggu Ram assured that he will keep Gokul without creating any problem; that
thereafter he went to Nathusar six to seven times to bring her daughter to Abhawas but
her husband and in-laws did not send her and demanded colour television. They also
threatened that if additional dowry is not brought, then his daughter will be finished. He
told the villagers about the demand made by Jaggu Ram and his family and the threat
given by them. Upon this, the villagers sent a message to Jaggu Ram that it was not
proper. Jaggu Ram and his family members got annoyed by this
@page-SC983
development and they killed his daughter by burning with kerosene and cremated her
body at 5.00 a.m. on 30.3.1993. Thereupon, the police registered Criminal Case No.
48/93 under Sections 498-A, 304-B and 201 of the IPC. During the investigation, the
police recorded the statements of Atma Ram and other persons under Section 161,
Cr.P.C., collected the hospital record and arrested the appellant, his wife Nathi and son
Jeevan Ram. A lathi was recovered at the instance of Jeevan Ram. After completing the
investigating, the police filed challan against all the accused in the court of Addl. Chief
Judicial Magistrate, Shrimadhopur, who committed them to the Sessions Court for trial.
3. The prosecution examined PW-1 Atmaram, PW-2 Kaluram, PW-3 Babulal, PW-4
Doctor Vijay Kumar, PW-5 Kamal Surana, PW-6 Shankar Lal, PW-7 Kanaram, PW-8
Gopiram, PW-9 Mewaram, PW-10 Rameshwar, PW-11 Banwari Lal, PW-12 Ram
Kishore, PW-13 Chhitar, PW-14 Maliram, PW-15 Saidduram, PW-16 Suresh, PW-17
Shrawan Singh, PW-18 Jamal, PW-19 Ramdhan, PW-20 Doctor Shyam Lal Khuteta and
PW-21 Om Prakash Godara and also produced documents marked Exhibit P1-Indoor
Ticket of deceased Gokul, Exhibit P3A to P5A Recovered bloodstained clothes, Exhibit
P6-Recovered bones and ash and Exhibit P7- Original report, PW 8- First Information
Report, Exhibit P9-Statement of Rameshwar given to the Police, Exhibit P10-Site map of
the spot, Exhibit P11-Statement of Chhitar given to the Police, Exhibit P12- Recovered
Lathi, Exhibit P13-Recovery site map, Exhibit P14-Statement of Suresh given to the
Police, Exhibit P15-Receipt of FSL, Exhibit P16 and Exhibit P17-Statement of Jamal to
the Police, Exhibit P18-Photo copy of Register of Store, Exhibit P19-Injury Report of
Gokul Devi, Exhibit P20-Arrest Jeevanram, Exhibit P21-Arrest Jagguram, Exhibit P22-
Arrest Nathi Devi and Exhibit P24-Information.
4. The accused were examined under Section 313, Cr.P.C. In his statement, Jeevan Ram
gave out that he had gone out of village on 29.3.1993 to appear in an examination and
that he had been falsely implicated. He denied the allegation of demand of dowry. He
gave out that the deceased was suffering from fits and she died due to injuries caused
when she collided with the door (chaukhat) of the house. The other accused denied the
charges levelled against them. The defence produced the documents marked Exhibit D1-
Statement of Shankar Lal given to the Police, Exhibit D2-Statement of Gopiram given to
the Police, Exhibit D3-Statement of Mevaram given to the Police, Exhibit D4-Statement
of Atmaram given to the Police, Exhibit D5-Statement of Ram Kishore to the Police,
Exhibit D6-Programme of Examination of Secondary School, Exhibit D7-Admission
Certificate for the Examination, Exhibit D8 and Exhibit D9-Entrance Card of
Examination.
5. The learned Additional Sessions Judge, Neem Ka Thana (hereinafter referred to as the
Trial Judge), framed the following points for determination:
1. Whether Shrimati Gokul died due to the injuries on her body?
2. Whether Shrimati Gokul died within 7 years of her marriage?
3. Whether Shrimati Gokul was subjected to cruelty and harassment by her husband,
mother-in-law and father-in-law immediately after the marriage and till her death in
connection with the demand of dowry?
4. Whether the accused destroyed the evidence relating to the death of Shrimati Gokul by
cremating her without informing her family members and the police and without getting
the post-mortem conducted?
6. The learned Trial Judge analysed the facts, evaluated the prosecution and defence
evidence and concluded that the prosecution has succeeded in proving the charge of
demand of dowry by the accused and that they were guilty of torturing and treating her
with cruelty immediately after marriage till her death. The learned Trial Judge further
held that Shanti @ Gokul died due to head injuries within seven years of her marriage.
He then held that the defence has failed to explain the cause of death of Shanti @ Gokul.
He rejected the defence theory that the deceased was suffering from epilepsy and she died
due to injuries suffered because during the bout of fits, her head collided against the door
of the house. Accordingly, he convicted the respondent and his wife Shrimati Nathi under
Section 304-B and sentenced them to seven years' rigorous imprisonment. He also
convicted them under Section 498-A and sentenced to one year's rigorous imprisonment
and imposed a fine of Rs. 500/- with a direction that if they fail to deposit the amount of
fine, the accused shall undergo further imprisonment
@page-SC984
of three months. The respondent and his wife were also convicted under Section 201, IPC
and sentenced to one year rigorous imprisonment with a fine of Rs. 500/- and in default
to undergo simple imprisonment of three months. The learned Trial Judge directed that all
the sentences shall run concurrently. He, however, accepted the defence version that
Jeevan Ram was not in the village at the time of death of Shanti and acquitted him.
7. On appeal, the learned Single Judge of the High Court confirmed the finding that the
respondent and his wife Nathi were guilty of demanding dowry but acquitted them of the
charge under Section 304-B, IPC on the premise that Jeevan Ram had been acquitted and
the State had not preferred appeal against his acquittal. However, he upheld their
conviction under Section 498-A and confirmed the sentence of one year's rigorous
imprisonment with fine of Rs. 500/- and to undergo further imprisonment of three months
in the case of default.
8. We have heard Shri Naveen Kumar Singh, learned advocate appearing for the
appellant-State of Rajasthan and scrutinized the entire record.
9. At the outset we consider it proper to mention that with a view to curb the growing
menace of dowry deaths, the Parliament amended the Indian Penal Code and the
Evidence Act and inserted Section 304-B and 113-B respectively in the two statutes. This
was done keeping in view the recommendations made by the Law Commission of India
in its 21st Report. Section 304-B (1), IPC lays down that where the death of a woman is
caused by burns or bodily injury or occurs otherwise than under normal circumstances
within seven years of her marriage and it is shown that soon before her death she was
subjected to cruelty or harassment by her husband or any relative of her husband for, or in
connection with, any demand for dowry, such death shall be called dowry death, and such
husband or relative shall be deemed to have caused her death. Explanation appearing
below sub-section (1) of Section 304-B declares that for the purpose of this sub-section,
dowry shall have the same meaning as in Section 2 of the Dowry Prohibition Act, 1961.
Sub-section (2) of Section 304-B prescribes the minimum punishment for dowry death as
seven years which can be extended up to imprisonment for life. The ingredients necessary
for the application of Section 304-B, IPC are :
1. that the death of a woman has been caused by burns or bodily injury or occurs
otherwise than under normal circumstances,
2. that such death has been caused or has occurred within seven years of her marriage,
and,
3. that soon before her death the woman was subjected to cruelty or harassment by her
husband or any relative of her husband in connection with any demand for dowry.
10. Section 113-B of the Evidence Act lays down that if soon before her death a woman
is subjected to cruelty or harassment for, or in connection with any demand for dowry by
the person who is accused of causing her death then the court shall presume that such
person has caused the dowry death. The presumption under Section 113-B is a
presumption of law and once the prosecution establishes the essentials ingredients
mentioned therein it becomes the duty of the court to raise a presumption that the accused
caused the dowry death.
11

. A conjoint reading of Section 304-B, IPC and Section 113-B, Evidence Act shows that
in order to prove the charge of dowry death, prosecution has to establish that the victim
died within 7 years of marriage and she was subjected to cruelty or harassment soon
before her death and such cruelty or harassment was for dowry. The expression soon
before her death has not been defined in either of the statutes. Therefore, in each case the
court has to analyse the facts and circumstances leading to the death of the victim and
decide whether there is any proximate connection between the demand of dowry, the act
of cruelty or harassment and the death - State of A.P. v. Raj Gopal Asawa and Anr. [2004
(4) SCC 470]; Arun Garg v. State of Punjab and Anr. [2004 (8) SCC 251];
Kaliyaperumal and Anr. v. State of Tamil Nadu [2004 (9) SCC 157]; Kamesh Panjiyar
@ Kamlesh Panjiyar v. State of Bihar [2005 (2) SCC 388]; Ram Badan Sharma v. State
of Bihar [2006 (10) SCC 115]. 2004 AIR SCW 1566
2003 AIR SCW 4387
2005 AIR SCW 770
2006 AIR SCW 4068

12. In the light of the above, we shall now consider whether the prosecution succeeded in
establishing the existence of the ingredients of Section 304-B, IPC and the High Court
committed an error by acquitting the
@page-SC985
respondent only on the ground that Jeevan Ram had been acquitted by the trial court and
the State did not appeal against his acquittal.
13. In order to prove that Shanti @ Gokul died as a result of injuries inflicted on her
body, the prosecution examined PW-1 Atma Ram who largely reiterated the story set out
in the first information report. He also explained the apparent discrepancy in the First
Information Report and medical report regarding the cause of the death by stating that he
mentioned about the burning of his daughter because she had earlier told about such
threat held out by her in-laws, but on reaching the spot, he came to know that she died
due to injuries on her head. In cross-examination he gave details of dowry items. He
categorically denied that his daughter was mentally ill and that he and the accused had
taken her out to Dr. Shiv Gautam, a Psychiatrist at Jaipur. He also denied that he had kept
the daughter with him for her treatment or that the story of Panchayat was fabricated. The
prosecution also examined PW-6 Shankar Lal, PW-7 Kana Ram and PW -12 Ram
Kishore, who supported the statement of Atma Ram. In their corss-examination each of
these witnesses denied the suggestion that the deceased was suffering from epilepsy and
she used to get fits. PW-4 Dr. Vijay Kumar gave out that Gokul Devi was admitted in the
hospital with head injuries. He was told by the relatives of the injured that the injuries
were caused due to fall. They also told that she was an old patient of epilepsy. According
to Dr. Vijay Kumar, Shanti remained in the hospital till 4'O clock and in the evening her
relatives took her by saying that they will be going to Jaipur. PW-20 Dr. Shyam Lal
Khuteta supported the version of Dr. Vijay Kumar that Shrimati Shanti had two injuries
on her head. He gave out that first injury - 3½" x ½" was a bone deep crushed wound on
the right side of the temple and the second was punctured wound of ½ cm. deep in the
bone from which the fresh blood was oozing. According to Dr. Shyam Lal Khuteta injury
No. 1 was inflicted by heavy weapon and injury No. 2 was inflicted by blunt weapon. The
learned Trial Judge relied on the statements of these witnesses in conjunction with the
medical reports and concluded that Shanti @ Gokul died as a result of the injuries
sustained by her on her head. He then considered the defence plea that Gokul was
suffering from epilepsy and she sustained head injuries by colliding with the frame of the
door during the bout of fits. This plea of the defence was based on the statement of
Jeevan Ram that he used to take Shanti to the doctors for treatment and on one occasion
he had taken her to Dr. Shiv Gautam, a mental doctor at Jaipur as also the statements of
PW-10 Rameshwar Mali, PW-13 Chittar, PW-14 Maliram, PW-16 Suresh, who were
declared hostile, that Gokul was suffering from Epilepsy. In their cross-examination,
these witnesses generally stated that Shanti @ Gokul suffered from fits and Jeevan Ram
used to take her to Jaipur for treatment. In his cross-examination, PW20- Dr. Shyam Lal
Khuteta also stated that long time ago, the deceased had come to him for her treatment of
Epilepsy. The learned Trial Judge observed that the story of Gokul suffering from
Epilepsy and her having suffered injuries on the head due to fall and consequential
striking against the door frame was concocted and was not acceptable because no
evidence was produced regarding her treatment for Epilepsy and held that in the absence
of any cogent explanation, it was reasonable to infer that the injuries on the head of the
deceased were caused by her in-laws. In this regard the learned trial judge also referred to
the factum of recovery of lathi at the instance of Jeevan Ram.
14. The High Court overturned this finding by observing that the prosecution has failed to
prove the allegation that the deceased was burnt to death. The learned Single Judge took
note of the so-called discrepancy in the First Information Report and the statement of
PW1-Atma Ram and held that injuries on the head of Gokul were caused due to her
having collided with door frame during bout of fits. For this purpose, he relied on the
statements of PW10-Rameshwar , PW13-Chhitar, PW16-Suresh and PW20-Dr. Shyam
Lal Khuteta. The learned Single Judge also opined that in view of the acquittal of Jeevan
Ram, the other accused cannot be convicted for offence under Section 304-B, IPC.
15. On the third point framed by him, the learned Trial Judge relied on the testimony of
PW-1 Atma Ram (father of the deceased), PW-6 Shankar Lal and PW-12 Ram Kishore
(brothers of the deceased), PW-5 Kamal Surana, who was in-charge of Mahila Jagran,
Shrimadhopur, PW-7 Kanaram, PW-8 Gopiram and PW-9 Mewaram who had
@page-SC986
participated in the Panchayat and held that the detailed narration given by the father and
the brothers of the deceased about cruel treatment and harassment meted out to the Gokul
was amply supported by the contents of First Information Report, the Panchayat held at
village Abhawas to discuss the issue relating to dowry and the statement of PW-5 Kamal
Surana in whose presence the dowry case of Gokul @ Shanti, daughter of Atma Ram,
was discussed. The learned Trial Judge held that the evidence produced by the
prosecution was sufficient to show that Gokul was subjected to harassment and torture
and was being treated with cruelty immediately after the marriage till her death because
she did not bring sufficient dowry. The High Court reversed this finding only on the
premise that the Panchayat was convened two and half months before the death of Gokul
and nothing had been brought on record to prove that during that period she was
subjected to cruel treatment. The learned Single Judge heavily relied on some
discrepancies and omissions in the FIR and held that the prosecution has not been able to
prove that Shanti @ Gokul was subjected to cruelty or harassment soon before her death.
16. On point No. 4, the learned Trial Judge relied on the statements of PW-1 Atma Ram,
PW-6 Shankar Lal, PW-7 Kanaram, PW-8 Gopiram, PW-9 Mewaram, PW-12 Ram
Kishore, all of whom stated that Shrimati Gokul was cremated before they reached
Nathusar. The learned Trial Judge noted that the defence has not produced any evidence
to show that the in-laws of the deceased had informed the police about the death or that
the post-mortem was got conducted. They also did not inform the parents of the deceased.
In the opinion of the Trial Judge, all this was sufficient to prove the charge of destroying
evidence. Learned Single Judge of the High Court did not at all discuss this issue.
17. In our considered view, the High Court committed serious illegality by acquitting the
respondent of the charge under Sections 304-B and 201, IPC on the premise that Jeevan
Ram had been acquitted. It is true that the learned trial Judge accepted the defence
version that Jeevan Ram was not present in the village Nathusar at the time when Shanti
@ Gokul suffered injuries on her head and acquitted by giving benefit of doubt and the
State did not challenge the same by filing appeal but that by itself did not justify a
conclusion that the prosecution had failed to prove the charge under Sections 304-B and
201 IPC against the remaining accused.
18. A critical analysis of the facts and evidence brought on record shows that the
prosecution had succeeded in proving that Shanti @ Gokul died within one and a half
years of her marriage. In their statements, PW1-Atma Ram, PW6-Shankar Lal and
PW12-Ram Kishore (father and brothers of the deceased) categorically stated that the
deceased was subjected to harassment and cruelty by her husband-Jeevan Ram, father-in-
law-Jaggu Ram, mother-in-law, Nathi Devi, immediately after marriage on the ground
that she did not bring sufficient dowry. When the deceased visited her parent's house, she
made a complaint about the harassment. Thereupon, Atma Ram gave Silver jewellery and
Rupees ten thousand cash. Even this also did not satisfy the accused who continued to
harass her. After sometime, the deceased was left at her father's place. She stayed there
for eight months. About two and a half months before the death, a meeting was convened
in the village, which was attended by 20-25 persons, including PW1-Atma Ram, PW6-
Shankar Lal, PW12-Ram Kishore, PW7-Kana Ram, PW8-Gopi Ram, PW9-
Mewaram(ex-Sarpanch of Gram Panchayat), Jaggu Ram and four others of village
Nathusar also attended the Panchayat. PW7-Kana Ram, PW8-Gopi Ram, PW9-Mewa
Ram were independent witnesses. They confirmed that a meeting was held in village
Abhawas to discuss the issue of dowry and reiterated what PW1-Atma Ram had told
them about the demand of dowry. They were cross-examined at length but the defence
could not shake their testimony. Rather, they reiterated the factum of holding the meeting
of Panchayat at Amawas wherein the issue of dowry was generally discussed. PW5-
Kamal Surana, In-charge, Women Development Agency, Data Ramgarh, is also an
independent witness. She gave details of the discussion made in the meeting held on
11.8.1992 where the case of dowry of Shanti @ Gokul, daughter of Atma Ram was
considered. According to Kamal Surana, Shanti was very scared and was not able to say
anything but her father gave the details of harassment. PW5 also stated that she had gone
to the in-laws whereupon, the
@page-SC987
respondent is said to have objected to her intervention. She also produced a diary
maintained by her in which the factum of her meeting at Amawas and Nathusar were
recorded.
19. If the prosecution evidence is considered in the backdrop of the fact that the defence
failed to produce any evidence to controvert the facts relating to the demand of dowry, it
must be held that the deceased was subjected to cruelty and harassment in connection
with dowry immediately after her marriage and such harassment continued till her death
and the learned trial Judge rightly held the charge under Section 304-B, IPC as proved,
against the accused. The learned Single Judge of the High Court gave undue weightage to
the minor discrepancies in the first information report and the statement of PW 1 Atma
Ram and some alleged omission in the first information report and acquitted the accused
ignoring the most important factor that the deceased suffered injuries in a dwelling unit
belonging to her in-laws and in their presence, that she died due to those injuries and that
the defence failed to offer any satisfactory explanation for the injuries on the head of the
deceased. The defence did introduce the story of the deceased suffering with epilepsy and
her being treated for the same, but no documentary evidence was produced to show that
she was ever treated for epilepsy. In their cross-examination, the father and brothers of
the deceased and the other prosecution witnesses categorically denied that the deceased
was suffering from epilepsy and she used to have bouts of fits. Atma Ram also denied the
suggestion that she and the accused had taken Shanti @ Gokul for treatment to a
Psychiatrist at Jaipur. Some of the Prosecution witnesses who were declared hostile, did
try to support the theory that the deceased used to have fits, but their statements can be of
no help to the accused because no documentary evidence in the form of prescriptions of
doctors or the bills of the treatment and purchase of medicines were produced to prove
that the deceased was suffering from epilepsy and used to have fits. The statement of Dr.
Shyam Lal Khuteta is also of no help to the accused because he too did not produce
record relating to the treatment allegedly given to the deceased for epilepsy long time
ago. The conduct of the accused and his family members in not informing the parents of
the deceased about the injuries caused on her head and consequential death and the fact
that the cremation of the dead body was conducted in the wee hours of 30.3.1993 without
informing the parents or giving an intimation to the Police so as to enable it to get the
post-mortem of the dead body conducted go a long way to show that the accused had
deliberately concocted the story that Shanti @ Gokul was suffering from epilepsy and she
suffered injuries on her head by colliding against the door bar during the bout of fits. The
disposal of dead body in a hush-hush manner clearly establish that the accused had done
so with the sole object of concealing the real cause of the death of Shanti @ Gokul.
20

. In our considered view, this was a fit case for invoking Section 106 of the Evidence Act,
which lays down that when any fact is especially within the knowledge of the any person,
the burden of proving that fact is upon him. In Ram Gulam Chaudhary vs. State of Bihar
[2001 (8) SCC 311] this Court considered the applicability of Section 106 of the
Evidence Act in a case somewhat similar to the present one. This Court noted that the
accused after brutally assaulting a boy carried him away and thereafter the boy was not
seen alive nor his body was found. The accused, however, offered no explanation as to
what they did after they took away the boy. It was held that for the absence of any
explanation from the side of the accused about the boy, there was every justification for
drawing an inference that they had murdered the boy. It was further observed that even
though Section 106 of the Evidence Act may not be intended to relieve the prosecution of
its burden to prove the guilt of the accused beyond reasonable doubt, but the section
would apply to cases like the present, where the prosecution has succeeded in proving
facts from which a reasonable inference can be drawn regarding death. The accused by
virtue of their special knowledge must offer an explanation which might lead the court to
draw a different inference. 2001 AIR SCW 3802

21

. In Trimukh Maroti Kirkan vs. State of Maharashtra [2006 (1) SCC 681], a two judge-
Bench of which one of us (G. P. Mathur, J.) was a member, considered the applicability of
Section 106 of the Evidence Act and observed: 2006 AIR SCW 5300, Paras 11 and
12

"The demand for dowry or money from


@page-SC988
the parents of the bride has shown a phenomenal increase in the last few years. Cases are
frequently coming before the courts, where the husband or in-laws have gone to the
extent of killing the bride if the demand is not met. These crimes are generally committed
in complete secrecy inside the house and it becomes very difficult for the prosecution to
lead evidence. No member of the family, even if he is a witness of the crime, would come
forward to depose against another family member. The neighbours, whose evidence may
be of some assistance, are generally reluctant to depose in court as they want to keep
aloof and do not want to antagonise a neighbourhood family. The parents or other family
members of the bride being away from the scene of commission of crime are not in a
position to give direct evidence which may inculpate the real accused except regarding
the demand of money or dowry and harassment caused to the bride. But, it does not mean
that a crime committed in secrecy or inside the houses should go unpunished.
If an offence takes place inside the privacy of a house and in such circumstances where
the assailants have all the opportunity to plan and commit the offence at the time and in
circumstances of their choice, it will be extremely difficult for the prosecution to lead
evidence to establish the guilt of the accused if the strict principle of circumstantial
evidence, as noticed above, is insisted upon by the courts. A Judge does not preside over
a criminal trial merely to see that no innocent man is punished. A Judge also presides to
see that a guilty man does not escape. Both are public duties. (See Stirland v. Director of
Public Prosecutions [1944 AC 315] quoted with approval by Arijit Pasayat, J. in State of
Punjab v. Karnail Singh [2003 (11) SCC 271]. The law does not enjoin a duty on the
prosecution to lead evidence of such character which is almost impossible to be led or at
any rate extremely difficult to be led. The duty on the prosecution is to lead such
evidence which it is capable of leading, having regard to the facts and circumstances of
the case. Here it is necessary to keep in mind Section 106 of the Evidence Act which says
that when any fact is especially within the knowledge of any person, the burden of
proving that fact is upon him. Illustration (b) appended to this section throws some light
on the content and scope of this provision and it reads: 2003 AIR SCW 4065

"(b) A is charged with travelling on a railway without ticket. The burden of proving that
he had a ticket is on him.
Where an offence like murder is committed in secrecy inside a house, the initial burden to
establish the case would undoubtedly be upon the prosecution, but the nature and amount
of evidence to be led by it to establish the charge cannot be of the same degree as is
required in other cases of circumstantial evidence. The burden would be of a
comparatively lighter character. In view of Section 106 of the Evidence Act there will be
a corresponding burden on the inmates of the house to give a cogent explanation as to
how the crime was committed. The inmates of the house cannot get away by simply
keeping quiet and offering no explanation on the supposed premise that the burden to
establish its case lies entirely upon the prosecution and there is no duty at all on an
accused to offer any explanation."
22

. Similar view has been expressed in State of Punjab vs. Karnail Singh [2003 (11) SCC
271]; State of Rajasthan vs. Kashi Ram [2006 (12) SCC 254]; Raj Kumar Prasad
Tamakar vs. State of Bihar [2007 (1) SCR 13]. 2003 AIR SCW 4065
2007 AIR SCW 295

23. We are sure, if the learned Single Judge of the High Court had adverted to Section
106 of the Evidence Act and correctly applied the principles of law, he would not have
committed the grave error of acquitting the respondent.
24. In the result, the appeal is allowed. The impugned judgment is set aside and the
conviction of the respondent under Section 304-B read with 201, IPC is restored. He is
sentenced to seven years' rigorous imprisonment. He shall also pay fine of Rs.500/- and
suffer further imprisonment of three months in case of default. If the respondent has
already undergone sentence of one year under Section 498-A, IPC in furtherance of the
judgment of the High Court, then he shall serve out six years' imprisonment, apart
@page-SC989
from paying fine. The respondent shall be immediately taken into custody to serve out his
sentence.
Appeal allowed.
AIR 2008 SUPREME COURT 989 "Ghulam Mohammad Dar v. State of J. and K."
(From : Jammu and Kashmir)
Coram : 2 G. P. MATHUR AND P. SATHASIVAM, JJ.
Civil Appeal No. 12 of 2008 (arising out of SLP (C) No. 16417 of 2006), D/- 4 -1 -2008.
Ghulam Mohammad Dar v. State of J. and K. and Ors.
Arbitration Act (10 of 1940), S.29, S.14(2) - ARBITRATION - INTEREST - DECREE -
Interest - Arbitrator awarding interest from date of award till final realisation - Decree
passed to pay interest from date of decree - Considering fact that judgment-debtor was
State Govt. - Decree held proper. (Para 8)

M. L. Bhat, Sr. Advocate, Ms. Purnima Bhat, for Appellant; Anis Suhrawardy, S. Mehdi
Imam, for Respondents.
Judgement
1. P. SATHASIVAM, J. :-Leave granted.
2. This appeal is directed against the order dated 01.09.2006 passed by the High Court of
Jammu and Kashmir at Sri Nagar in Civil Revision No. 47 of 2006 whereby the High
Court dismissed the civil revision filed by the appellant herein.
3. In view of the limited issue i.e., interest payable by the respondents, there is no need to
traverse the entire factual matrix except relating to the issue in question.
4. According to the appellant, the executing Court has wrongly interpreted the judgment
passed by the High Court as well as the provisions of the Arbitration Act and erroneously
refused to release the interest on the Award/decretal amount from the date of Award till
passing of the decree.
5. It is not in dispute that an Award came to be passed on 05.09.1995 which was made a
Rule of the Court and accordingly decree came to be passed on 30.04.1998.
6. It is relevant to reproduce the Award of the Arbitrator in respect of the interest which
reads as under:
"The claimant shall be entitled to 10% S.I.P.A. beyond 10.11.1995 till payment is made in
full by the respondents of the full awarded amount. Respondent No.3 shall be liable to
discharge and pay the final bill pending since 27.12.1993 with him failing which 18%
P.A. simple interest shall be paid over and above from 1.2.1994 till date of actual
payment."
Though there is little confusion in the direction of the Arbitrator, it is presumed that the
Arbitrator has granted interest @ 10% (simple interest p.a.) from 10.11.1995 till payment
is made in full by the respondents. The latter part of the said direction shows that in case
of default, the Award amount carries interest @ 18% simple interest per annum from
01.02.1994 till date of actual payment.
7. On 30.04.1998, learned single Judge of the High Court disposed of Arbitration Petition
No. 171 of 1991 by passing the following order:
"In the totality of the circumstances, I order that let award be made rule of the Court and
the amounts found due along with interest awarded by the Arbitrator be paid from the
date of decree with interest at 18% till final realization of the decretal amount of the
petition. Let decree be prepared accordingly."
8. Learned senior counsel appearing for the appellant, by drawing our attention to the
direction of the Arbitrator as well as the ultimate order passed by the High Court,
submitted that in view of default in payment of the amount within the stipulated time, the
appellant is entitled to interest @ 18% p.a. from the date of the Award and not from the
date of the decree. In the light of the controversy, we verified the direction of the
Arbitrator and the order passed by the High Court both in the Arbitration and Revision
Petition. On perusal of the same and of the fact that the respondents are none other than
the State Government, we agree with the order of the High Court dated 30.04.1998
passed in Arbitration Petition No. 171 of 1991 and hold that the claimant is entitled to
interest @ 18% p.a. for the award amount from the date of the decree till realization. To
this extent, we clarify the position. The Civil Appeal is disposed of on the above terms.
No costs.
Order accordingly.
@page-SC990
AIR 2008 SUPREME COURT 990 "Bhagwan Dass v. Punjab State Electricity Board"
(From : Punjab and Haryana)*
Coram : 2 G. P. MATHUR AND AFTAB ALAM, JJ.
Civil Appeal No. 8 of 2008 (arising out of SLP (C) No. 26357 of 2005), D/- 4 -1 -2008.
Bhagwan Dass and Anr. v. Punjab State Electricity Board.
(A) Persons with Disabilities (Equal Opportunities, Protection of Rights and Full
Participation) Act (1 of 1996), S.47 - DISABLED PERSONS - APPLICABILITY OF AN
ACT - REPEAL AND SAVINGS - Applicability - Appellant- employee declared
completely blind on 17-1-1994 - When Act came into force on 7-2-1996 he was
undeniably in service and his contract of employment with Electricity Board was
subsisting - His case was, therefore, squarely covered by provisions of the Act. (Para
6)
(B) Persons with Disabilities (Equal Opportunities, Protection of Rights and Full
Participation) Act (1 of 1996), S.47 - DISABLED PERSONS - TERMINATION OF
SERVICE - ELECTRICITY - Termination of service - Appellant-employee became
completely blind - Wrote a letter to employer, Electricity Board to explain his absence
from duty, when charge-sheet was issued to him - In said letter he requested to be retired
but at the same time asked that his wife should be given a suitable job in his place - Said
letter cannot be read as a voluntary offer for retirement - Superior Officers of Board
failed to explain him correct legal position and telling him about his legal rights - Threw
him out of service by picking up a sentence from his letter, completely out of context -
Conduct of Superior Officers, deprecated - Termination of service of said employee, held,
illegal. (Paras 12, 13, 15)
(C) Persons with Disabilities (Equal Opportunities, Protection of Rights and Full
Participation) Act (1 of 1996), S.47 - DISABLED PERSONS - EQUALITY - Disabled
employee - His Superiors were duty-bound to follow law and it was not open to them to
allow their bias to defeat his lawful rights - Denial of rights to disabled would not only be
unjust and unfair to them and their families but would create larger and graver problems
for society at large - What the law permits to them is no charity or largess but their right
as equal citizens of country.
Constitution of India, Art.14. (Para 14)
Cases Referred : Chronological Paras
2003 AIR SCW 1013 : AIR 2003 SC 1623 : 2003 Lab IC 1133 (Ref.) 4
1994 AIR SCW 235 (Ref.) 11
Vipin Gogia (for Ms. Jaspreet Gogia), for Appellants; Mrs. Jayashree Anand (for
Ashwani Bhardwaj), for Respondent.
* CWP No. 12534 of 2005, D/- 11-8-2005 (PandH)
Judgement
AFTAB ALAM, J. :-Leave granted.
2. This case highlights the highly insensitive and apathetic attitude harboured by some of
us, living a normal healthy life, towards those unfortunate fellowmen who fell victim to
some incapacitating disability. The facts of the case reveal that officers of the Punjab
State Electricity Board were quite aware of the statutory rights of appellant No.1 and
their corresponding obligation yet they denied him his lawful dues by means that can
only be called disingenuous.
3. The facts of the case are brief and are all taken from the (Reply) Affidavit filed on
behalf of the Punjab State Electricity Board and its officers (the respondents in the
appeal). Appellant No.1 joined the respondent Board on July 19, 1977, on ad hoc/work-
charged basis. His services were regularized as an Assistant Lineman on June 16, 1981.
While in service he became totally blind on January 17, 1994 and a certificate to that
effect was issued by the Civil Surgeon, Faridkot.
4. Here, it may be noted that the rights of an employee who acquires a disability during
his service are protected and safeguarded by Section 47 of the Persons with Disabilities
(Equal Opportunities, Protection of Rights and Full Participation) Act, 1995. Section 47
reads as follows :
"47. Non-discrimination in Government employments- (1) No establishment shall
dispense with, or reduce in rank, an employee who acquires a disability during his
service:
Provided that, if an employee, after acquiring disability is not suitable for the post he was
holding, could be shifted to some other post with the same pay scale and service
benefits :
Provided further that if it is not possible to adjust the employee against any post, he may
be kept on a supernumerary post until a suitable post is available or he attains the
@page-SC991
age of superannuation, whichever is earlier.
(2) No promotion shall be denied to a person merely on the ground of his disability:-
Provided that the appropriate Government may, having regard to the type of work carried
on in any establishment, by notification and subject to such conditions, if any, as may be
specified in such notification, exempt any establishment from the provisions of this
section.

It may further be noted that the import of Section 47 of the Act was considered by this
court in Kunal Singh vs. Union of India and Anr. [2003 (4) SCC 524] and in paragraph 9
of the decision it was observed and held as follows : 2003 AIR SCW 1013, Para 9
"Chapter VI of the Act deals with employment relating to persons with disabilities, who
are yet to secure employment. Section 47, which falls in Chapter VIII, deals with an
employee, who is already in service and acquires a disability during his service. It must
be borne in mind that Section 2 of the Act has given distinct and different definitions of
"disability" and "person with disability". It is well settled that in the same enactment if
two distinct definitions are given defining a word/expression, they must be understood
accordingly in terms of the definition. It must be remembered that a person does not
acquire or suffer disability by choice. An employee, who acquires disability during his
service, is sought to be protected under Section 47 of the Act specifically. Such
employee, acquiring disability, if not protected, would not only suffer himself, but
possibly all those who depend on him would also suffer. The very frame and contents of
Section 47 clearly indicate its mandatory nature. The very opening part of the section
reads "no establishment shall dispense with, or reduce in rank, an employee who acquires
a disability during his service." The section further provides that if an employee after
acquiring disability is not suitable for the post he was holding, could be shifted to some
other post with the same pay scale and service benefits; if it is not possible to adjust the
employee against any post he will be kept on a supernumerary post until a suitable post is
available or he attains the age of superannuation, whichever is earlier. Added to this no
promotion shall be denied to a person merely on the ground of his disability as is evident
from sub-section (2) of Section 47. Section 47 contains a clear directive that the
employee shall not dispense with or reduce in rank an employee who acquires a disability
during the service. In construing a provision of a social beneficial enactment that too
dealing with disabled persons intended to give them equal opportunities, protection of
rights and full participation, the view that advances the object of the Act and serves its
purpose must be preferred to the one which obstructs the object and paralyses the purpose
of the Act. Language of Section 47 is plain and certain casting statutory obligation on the
employer to protect an employee acquiring disability during service.
(Emphasis added)
5. After the Act came into force with effect from December 7, 1996 (vide S.O.107(E),
dated 7th February, 1996), the Government of Punjab, Department of Personnel and
Administrative Reforms, issued a letter dated September 24, 1996 directing all the Heads
of Departments to comply with Section 47 of the Act. The Punjab State Electricity Board
too adopted the Government letter under its Circular No.6/97, dated February 17, 1997.
6. In view of Section 47 of the Act and the Circulars issued by the State Government and
the Board it is clear that notwithstanding the disability acquired by the appellant the
Board was legally bound to continue him in service. But on behalf of the respondent it is
stated that the disabled employee himself wanted to retire from service and, therefore, the
provisions of Section 47 had no application to his case. Here it needs to be made clear
that at no stage any plea was raised that since the appellant was declared completely blind
on January 17, 1994 he was not covered by the provisions of the Act that come into force
on February 7, 1996. Such plea cannot be raised because on February 7, 1996 when the
Act came into force the appellant was undeniably in service and his contract of
employment with the Board was subsisting. His case was, therefore, squarely covered by
the provisions of the Act.
7. Coming now to the reason assigned by the Board to deny him the protection of Section
47 of the Act, it is stated on behalf of the respondents that he remained absent from duty
without any sanctioned leave from January 18, 1994 to March 21, 1997. He was directed
by the Executive Engineer to resume duties vide Memo No.412, dated
@page-SC992
March 16, 1994 and Memo No.6411, dated August 4, 1994. He, however, failed to report
for duty and on September 13, 1994, a charge-sheet was issued initiating disciplinary
proceedings against him for gross misconduct under Regulation 8 of the Punjab State
Electricity Board Employees Punishment and Appeal Regulation, 1971.
8. The matter appears to have lain dormant for sometime and then it is stated that the
appellant by his letter dated July 17, 1996 requested the Board to retire him from service.
As a matter of fact by this letter the appellant sought to explain his absence from duty and
requested that his wife might be employed in his place. But it was made the basis for
denying the appellant his lawful dues. Since the whole case of the respondents is based
on this letter it would be appropriate to reproduce it in full :
"Sir,
I explain as under the subject cited unnatural happening which I met,
When I was returning home after performing my duty on 17-1-94 then vision of my eyes
lessened suddenly. I got treatment from far and near for eye-sight/lessening of vision of
my eyes. But I became completely blind. Now I cannot perform my hard work duty. I
want to retire from service. I may be retired and my wife may be provided with suitable
job against me. Yourself will be genesis to me.
(Emphasis added)
At this stage some internal correspondences took place between the officers of the Board
over the question how to deal with the appellant. On July 10, 1997, the Senior Executive
Engineer (OP) Division, Malout wrote to the Deputy Chief Engineer, Operation Circle,
Muktsar, asking for instructions in the matter. Paragraphs 2, 3 and 4 of the letter are
relevant and are reproduced below :
"2) As per report of Medical Board the official is unfit for duty, he cannot perform any
duty.
3) But as per instructions contained in Punjab Government Memo No.17/16/94-5 PP-
1/6546 adopted by PSEB vide its Circular No.6/97 the official/officer it (sic is) not to be
retired from service who become disable during service.
4) The official has represented that he may be retired from duty and his wife be provided
with suitable job."
The Senior Executive Engineer received the reply from the Secretary of the Board vide
letter dated February 17, 1998 in which he was advised as follows :
"It is advisable to retire the official as per Rules and Regulations of the Board if the
employee is not otherwise interested in taking the benefit of Board's Circular No.6/97.
For the purpose of clarification as to whether employee is entitled to the benefits,
otherwise admissible under Rules/Regulations of the Board in preference to benefits
admissible under Circular No.6/97, if he so desires, can be obtained from the Office
concerned which issued said circular.
Later on, the charge-sheet issued against the appellant was withdrawn by the Senior
Executive Engineer vide Office Order No.14, dated January 13, 1999 and the appellant
was asked to submit leave application for the period of absence.
9. Next in series is a letter, dated November 15, 1999, from the Director/IR, PSEB,
Patiala to the Senior Executive Engineer, (OP) Division, Malout. In this letter it was
stated as follows :
"As per cited subject it is made clear that employee who is blind shall not be retired as
per instructions of the Board. But is (sic. if) such employee himself make request for
retirement then he can be given retirement on medical ground.
Finally, the Senior Executive Engineer, issued Office Order No.559, dated December 14,
1999, by which the appellant was relieved from service with effect from March 21, 1997
(the date of issuance of Medical Certificate) as per Rule 5.11 of Civil Services Rules-
Vol.II.
10. It appears that the appellant protested against the action of the Board in relieving him
from service and made representations. The representations, it seems, were forwarded to
the Superior Authorities and the Board's decision was communicated to the Senior
Executive Engineer vide letter dated February 18, 2000 from the Director/IR, PSEB,
Patiala. The contents of the letter are as follows :
"With regard to cited subject it is made clear that there are instructions of the Board on
which blind employee is not liable to be retired. But in the case of Shri Bhagwan Dass
ALM advice of retirement was given as he himself made request for his retirement
@page-SC993
on Medical Ground. So the case of this employee is not likely considered for his rejoining
of duty."
The appellant then filed an affidavit before the concerned officers. A copy of the affidavit
is at Annexure R-12 to the respondents' affidavit. In the affidavit he pathetically pleaded
that he had no knowledge about the Rules of the Electricity Board and represented for
retirement unknowingly. He further stated that when he came to know that there was no
need for retirement for those who were disabled during service he again represented that
he might not be retired and might be retained in service as per the instructions of the
department. The affidavit did not evoke any response but the severance was completed by
making payment of his terminal dues.
11

. The disabled employee then approached the Punjab and Haryana High Court in Civil
Writ Petition No.12534 of 2004 seeking relief in terms of section 47 of the Act and the
Circulars issued by the State Government and the Board in its furtherance. In the writ
petition he was joined by his son, appellant No.2, and an alternative relief was sought for
employment of his son in his place. Unfortunately, before the High Court it was the
second relief that came into focus and the High Court dismissed the writ petition by a
brief order referring to the decision of this Court in Umesh Nagpal vs. State of Haryana
[1994 (3) SCT 174]. In the High Court order there is no mention of Section 47 of the Act
and the disabled employees' claim/right on that basis. Against that order this appeal is
preferred in which the disabled employee agitates his rights on the basis of Section 47 of
the Act. 1994 AIR SCW 2305

12. From the materials brought before the court by none other than the respondent-Board
it is manifest that notwithstanding the clear and definite legislative mandate some
Officers of the Board took the view that it was not right to continue a blind, useless man
on the Board's rolls and to pay him monthly salary in return of no service. They
accordingly persuaded each other that the appellant had himself asked for retirement from
service and, therefore, he was not entitled to the protection of the Act. The only material
on the basis of which the Officers of the Board took the stand that the appellant had
himself made a request for retirement on medical grounds was his letter dated July 17,
1996. The letter was written when a charge sheet was issued to him and in the letter he
was trying to explain his absence from duty. In this letter he requested to be retired but at
the same time asked that his wife should be given a suitable job in his place. In our view
it is impossible to read that letter as a voluntary offer for retirement.
13. Appellant No.1 was a Class IV employee, a Lineman. He completely lost his vision.
He was not aware of any protection that the law afforded him and apparently believed
that the blindness would cause him to lose his job, the source of livelihood of his family.
The enormous mental pressure under which he would have been at that time is not
difficult to imagine. In those circumstances it was the duty of the Superior Officers to
explain to him the correct legal position and to tell him about his legal rights. Instead of
doing that they threw him out of service by picking up a sentence from his letter,
completely out of context. The action of the concerned Officers of the Board, to our
mind, was deprecatable.
14. We understand that the concerned Officers were acting in what they believed to be the
best interests of the Board. Still under the old mind-set it would appear to them just not
right that the Board should spend good money on someone who was no longer of any
use. But they were quite wrong, seen from any angle. From the narrow point of view the
officers were duty-bound to follow the law and it was not open to them to allow their bias
to defeat the lawful rights of the disabled employee. From the larger point of view the
officers failed to realise that the disabled too are equal citizens of the country and have as
much share in its resources as any other citizen. The denial of their rights would not only
be unjust and unfair to them and their families but would create larger and graver
problems for the society at large. What the law permits to them is no charity or largess
but their right as equal citizens of the country.
15. In light of the discussions made above, the action of the Board in terminating the
service of the disabled employee (appellant No.1) with effect from March 21, 1997 must
be held to be bad and illegal. In view of the provisions of Section 47 of the Act, the
appellant must be deemed to be in service and he would be entitled to all service benefits
including annual increments
@page-SC994
and promotions etc. till the date of his retirement. The amount of terminal benefits paid to
him should be adjusted against the amount of his salary from March 22, 1997 till date. If
any balance remains, that should be adjusted in easy monthly installments from his future
salary. The appellant shall continue in service till his date of superannuation according to
the service records. He should be reinstated and all due payments, after adjustments as
directed, should be made to him within six weeks from the date of presentation of a copy
of the judgment before the Secretary of the Board.
16. In the result the appeal is allowed with costs quantified at Rs. 5,000/-.
Appeal allowed.
AIR 2008 SUPREME COURT 994 "Reserve Bank of India v. M. Hanumaiah"
(From : Karnataka)*
Coram : 2 G. P. MATHUR AND AFTAB ALAM, JJ.
Civil Appeal No. 9 of 2008 (arising out of SLP (C) No. 13664 of 2005), D/- 4 -1 -2008.
Reserve Bank of India v. M. Hanumaiah and Ors.
Karnataka Co-operative Societies Act (11 of 1959), S.30(5) - Constitution of India, Art.14
- CO-OPERATIVE SOCIETIES - EQUALITY - RESERVE BANK OF INDIA - Co-
operative Bank - Supersession of Managing Committee - Hearing to affected
Bank/Committee - Not necessary when supersession is on request from Reserve Bank.
W. A. No. 6120 of 2002 (CS-RES), D/ 3-3-2005 (Kant), Reversed.
On receipt of a requisition in writing from the Reserve Bank of India the Registrar, Co-
operative Societies is statutorily bound to issue the order of supersession of the
Committee of Management of the Co-operative Bank. At that stage the affected Bank/its
Managing Committee has no right of hearing or to raise any objections. Sub-sections (1
to (4) of S. 30 relate to removal of the Committee of the Co-operative Society and sub-
section (5) relates to supersession of the Managing Committee of a Co-operative Bank. In
case of removal of the Committee of a Co-operative Society compliance with the
principles of natural justice is expressly required inasmuch in sub-section (1) of S. 30 it is
stipulated that the Registrar would pass the order of removal only 'after giving the
committee an opportunity to state its objections'. On the other hand the requirement of
any hearing is absent in sub-section (5) which starts with a non obstante clause that also
covers the provisions of the earlier sub-sections of Section 30.
W. A. No. 6120 of 2002 (CS-RES), D/- 3-3-2005 (Kant), Reversed.
1992 Mah LJ 1442, 2002 (2) Mah LJ 844 Approved.
AIR 1962 SC 1371, Rel. on. (Paras 10, 19)
Cases Referred : Chronological Paras
2002 (2) Mah LJ 844 (Approved) 15
1992 Mah LJ 1442 (Approved) 15
1992 CTJ 729 (Bom) 15
AIR 1962 SC 1371 (Rel. on) 11, 14
AIR 1957 SC 896 (Ref.) 14
R. N. Trivedi, Sr. Advocate, Kuldeep Parihar, Ms. Shweta Garg, H. S. Parihar, for
Appellant; A. Deb Kumar, K. Rajeev, for Respondents.
* W.A. No. 6120 of 2002 (CS-RES), D/- 3-3-2005 (Kant).
Judgement
AFTAB ALAM, J. :- Leave granted.
2. Whether the principles of natural justice have any application at the stage when the
Registrar Co-operative Societies, on being so required in writing by the Reserve Bank of
India passes an order removing the Committee of Management of a Co-operative Bank
and appointing an Administrator to manage its affairs for such period, as may be specified
by the Reserve Bank of India? This is the question that falls for consideration in this case.
3. The facts and circumstances in which the question arises are brief and simple and may
be stated thus :
On inspection of Kalidasa Co-operative Bank Ltd. (respondent No.16) (hereinafter
referred to as the 'Co-operative Bank' or 'the Bank') made on June 30, 1994 under Section
35 read with Section 56 of the Banking Regulation Act, the Reserve Bank of India (the
appellant before us) found a number of serious irregularities in its affairs. It sent a copy of
the inspection report to the Co-operative Bank and called the members of its Board of
Directors for discussion on the findings in the report. It also forwarded a copy of the
inspection report to the Joint Registrar, Co-operative Societies. The Joint Registrar
advised the Reserve Bank to make requisition for supersession of the Committee of
Management of the Bank. The Reserve Bank, however, withheld any action in that
@page-SC995
regard but called the members of the Board of Directors of the Bank for several rounds of
discussions at different levels. The Board of Directors was repeatedly urged to take
stringent actions to improve the financial health of the Bank. Apparently, no remedial
measures were taken and the affairs of the Co-operative Bank continued in a state of
financial distress. Finally, the Reserve Bank issued a requisition to the Registrar, Co-
operative Societies, Karnataka on January 22, 2002 requiring him to supersede the Board
of Directors of the Co-operative Bank and to appoint an Administrator for a period of one
year as provided under Section 30(5) of the Karnataka Co-operative Societies Act. The
requisition was made in public interest and for preventing the affairs of the Bank being
conducted in a manner detrimental to the interest of the depositors and for securing
proper management of the Bank.
4. In compliance with the requisition made by the Reserve Bank the Registrar, Co-
operative Societies issued an order on January 31, 2002 superseding the Board of
Directors of the Bank and appointing an Administrator in its place.
5. The order of supersession issued by the Registrar was challenged before the Karnataka
High Court by respondents 2 to 13 (members of the Committee of Management of the
Co-operative Bank that was in existence at that time) in W.P.No.6706 of 2003 (CS-RES).
The writ petition was allowed by a learned Single Judge of the Court by order dated
September 21, 2002. It is a brief order in which after noticing the relevant provision as
contained in Section 30(5) of the Karnataka Co-operative Societies Act, the learned Judge
simply observed as follows :
"From the order, I find that the supersession is at the instance of the Reserve Bank of
India since it is referred to in the impugned order. Further, the reason given by the
Reserve Bank of India in order to supersede the Committee of Management in the public
interest has not been disclosed in the impugned order. Further, no opportunity of hearing
also has been afforded before passing an order by the Co-operative Bank. In the result, I
pass the following order :
(a) Writ Petition is allowed.
(b) The impugned order is quashed."
(Emphasis added)
6. Against the order passed by the learned Single Judge, the Reserve Bank of India
preferred Writ Appeal No.6120 of 2002 (CS-RES). When the appeal was taken up on
March 31, 2003, the Court was told that fresh elections for the Committee of
Management were to take place on March 20. The Division Bench took the view that this
development had rendered the writ appeal infructuous and disposed it of as such, leaving
it open 'to the Reserve Bank to proceed against the Bank, if necessary, in accordance with
law'.

7. Mr. R. N.Trivedi, learned senior counsel, appearing on behalf of the appellant,


submitted that both the learned Single Judge and the Division Bench of the High Court
seriously erred in the matter, the learned Single Judge by introducing the elements of
natural justice where none existed and the Division Bench by treating the appeal as
infructuous.
8. The learned counsel submitted that the Division Bench overlooked the main issue and
failed to appreciate that as long as the Registrar was held obliged to give an opportunity
of hearing to the Co-operative Bank it was pointless to say that it 'would be open to
Reserve Bank of India to proceed against the bank, if necessary, in accordance with law'.
Counsel further submitted that the learned Single Judge had set aside the supersession
order on two grounds. The first ground was wrong on facts and the second was flawed
legally. It was incorrect to say that the order of the Registrar did not disclose the reasons
for supersession. The reasons were stated in the preamble of the order. Moreover, the
reasons for supersession were stated in detail in the requisition made by the Reserve
Bank. But it was the second ground in regard to the opportunity of hearing to the Co-
operative Bank that was fundamentally bad as it tended to defeat the very object and
purpose of supersession of the Managing Committee of the Bank. Learned counsel
submitted that the order of the learned Single Judge would in effect give rise to a process
of adjudication at the level of the Registrar. In other words, the Reserve Bank which is
the apex expert body in the country in regard to banking affairs would be required to go
to the Registrar and satisfy him about the need for supersession of the Management of the
Bank. What is worse is that this process of adjudication might take a few weeks' time and
thus
@page-SC996
completely frustrate the need for an urgent intervention by the Reserve Bank in order to
protect the interests of small depositors.
9. We are satisfied that Mr.Trivedi is right in his submission and though the Managing
Committee of the Co-operative Bank for the supersession of which action was taken by
the Reserve Bank may no longer be in existence the issue involved in the case needs to be
decided as it is likely to crop up in future in regard to the respondent-Bank or other Co-
operative Banks.
10. In order to examine the question it would be best to begin with the legal provision.
Section 30 of the Karnataka Co-operative Societies Act, 1959 is as follows :
"30. Supersession of committee- (1) If, in the opinion of the Registrar-
(a) the committee of a co-operative society persistently makes default or is negligent in
the performance of the duties imposed on it by this Act or the rules or the bye-laws or
commits any act which is prejudicial to the interests of the society or its members, or is
otherwise not functioning properly; or
(b) a co-operative society is not functioning in accordance with the provisions of this Act,
the rules or bye-laws or any order or direction issued by the State Government or the
Registrar, "including the direction issued under Section 30-B."
the Registrar may, after giving the committee an opportunity to state its objections, if any,
by order in writing remove the said committee, and appoint an administrator to manage
the affairs of the society for such period, not exceeding [six months], as may be specified
by the Registrar. The Registrar may for the reasons to be recorded in writing extend the
period of such appointment for a further period of six months at a time and in any case
such extension shall not exceed one year in aggregate.
(2) The administrator so appointed shall, subject to the control of the Registrar and such
instructions as he may give from time to time, exercise all or any of the functions of the
committee or of any [office bearer] of the co-operative society and take such action as he
may consider necessary in the interest of the society.
(3) The administrator shall, before the expiry of his term of office arrange for the
constitution of a new committee after holding the election in accordance with this Act,
the rules and the bye-laws of the co-operative society :
Provided that in such an election, no member of the committee removed under sub-
section (1) shall, notwithstanding anything contained in this Act, the rule or the bye-laws,
be eligible for being elected as a member of the Committee, for a period of four years
from the date of supersession of the committee under the said sub-section.
(4) Before taking any action under sub-section (1) in respect of a co-operative society, the
Registrar shall consult the financial banks to which it is indebted.
(5) Notwithstanding anything contained in this Act, the Registrar shall, in the case of a
co-operative bank, if so required in writing by the Reserve Bank of India in public
interest or for preventing the affairs of the co-operative bank being conducted in a
manner detrimental to the interest of the depositors or for securing the proper
management of the co-operative bank, by order in writing, remove the committee of that
co-operative bank and appoint an administrator to manage the affairs of the co-operative
bank for such period as may, from time to time, be specified by the Reserve Bank of
India."
(Emphasis added)
Sub-sections (1) to (4) relate to removal of the Committee of the Co-operative Society
and sub-section (5) relates to supersession of the Managing Committee of a Co-operative
Bank. It is to be seen that in case of removal of the Committee of a Co-operative Society
compliance with the principles of natural justice is expressly required inasmuch in sub-
section (1) it is stipulated that the Registrar would pass the order of removal only 'after
giving the Committee an opportunity to state its objections'. On the other hand the
requirement of any hearing is absent in sub-section (5) which starts with a non obstante
clause that also covers the provisions of the earlier sub-sections of Section 30. Mr.
Trivedi submitted that in case of supersession of the Management of a Co-operative bank
there was no application of the principles of natural justice for two reasons; one was that
the Reserve Bank of India was the apex expert body in the country in banking matters
and once the Reserve Bank of India was satisfied in regard to the need of supersession of
the Banks Management, the Registrar Co-operative Societies who had no experience in
the affairs of
@page-SC997
Banks was simply obliged to carry out the instructions of the Reserve Bank; secondly,
once the decision of supersession was taken it was necessary to have it effected speedily
because any delay would cause irreparable loss and harm to the interests of small
depositors of the Bank. It was, therefore, by design that no opportunity of hearing was
mentioned in sub-section (5) even though it was stipulated earlier (in sub-section (1)) in
the same section.
11. Mr. Trivedi submitted that a similar question arose before this Court when the validity
of section 38 of the Banking Companies Act, 1956 came in question in the case of Joseph
Kurnvilla Velukunnel vs. Reserve Bank of India and Ors. [AIR 1962 SC 1371] relating
to the winding up of the Palai Central Bank Ltd., Kerala. The Reserve Bank of India
made an application in the High Court of Kerala under Section 38 of the Banking
Companies Act read with some allied provisions of the Indian Companies Act for the
winding up of the Palai Central Bank Limited and for appointment of the Official
Liquidator etc. The High Court allowed the application and the decision of the High
Court came to be challenged before this Court in appeal in which the main question
related to the constitutional validity of Section 38 of the Banking Companies Act. A
Constitution Bench upheld the validity of the provision by a majority of 3 to 2.
12. Section 38 of the Banking Companies Act laid down as follows :
"38(1). Notwithstanding anything contained in Section 391, Section 392, Section 433 and
Section 583 of the Companies Act, 1956, but without prejudice to its powers under sub-
section (1) of Section 37 of this Act, the High Court shall order the winding up of a
banking company-
(a) if the banking company is unable to pay its debts; or
(b) if an application for its winding up has been made by the Reserve Bank under Section
37 of this Section.
(2) The Reserve Bank shall make an application under this section for the winding up of a
banking company if it is directed so to do by an order under clause (b) of sub-section (4)
of Section 35.
(3) The Reserve Bank may make an application under this section for the winding up of a
banking company -
xxx xxx xxx xxx
(b) if in the opinion of the Reserve Bank-
xxx xxx xxx xxx
(iii) the continuance of the banking company is prejudicial to the interests of its
depositors.
13. Mr. Trivedi argued that in case of Palai Bank the issue was far more fundamental and
grave than the issue in the case in hand. In Palai Bank the provision of Section 38 ousted
the authority and power of the High Court and not merely that of a Registrar, Cooperative
Societies; furthermore, the provision allowed for the winding up of a banking company
and thus interfered with the fundamental right to carry on business. In the case in hand
the business of the cooperative bank would go unhindered and interference was limited
only to the management of the bank.
14

. One of the grounds on which the validity of Section 38 was challenged was that it
offended the principles of natural justice. In paragraphs 30 to 31 of the judgment this
Court noticed the grounds on which the provisions were assailed and observed as
follows : AIR 1962 SC 1371, at p. 1383

"(30) The main ground of attack is the way Ss.38(1) and (3)(b)(iii) make it mandatory for
the High Court to pass an order winding up a banking company whenever the Reserve
Bank under its powers or under an order of the Central Government makes an application
for the winding up a banking company. It is argued that such a power to the Reserve
Bank is an uncontrolled and despotic power and to crown all, access to Courts is not
possible because the Court itself must pass an order without deciding whether the affairs
of the banking company are being conducted in a manner detrimental to the interests of
the depositors - a fact capable of being proved like any other fact. It is argued as a matter
of principle that any law which bars a decision by the Court is itself unreasonable without
more. Mr.Pathak, in supplementing the above contentions of Mr.Nambiar, also contends
that by the law in question a judicial process has been converted into an executive action,
and subjective determination has taken the place of judicial determination. He also
contends that the Reserve Bank accuses a banking company, and then tries the issue to
the complete exclusion of Courts.
(31) It must not be overlooked that the winding up of a banking company takes
@page-SC998
place before the High Court and under the process of law. The judicial process is
excluded only to respect of the momentous decision whether a winding up order should
be made or not. This opinion is left to the Reserve Bank, and the Court merely passes an
order according to the Reserve Bank's opinion, and then proceeds to wind up the banking
company according to law. The narrow question is whether in leaving this decision to the
Reserve Bank the law offends the principles of natural justice and becomes so
unreasonable, viewed in the light of Art.19, as to become void. This is the point on which
the respective parties joined issue and had much to say, and this is the crucial point in this
case.
(Emphasis added)

Rejecting the submissions the majority decision referred to an earlier decision of this
Court in Virendra vs. The State of Punjab [1958 SCR 308] relied upon by the Attorney
General and in paragraphs 44 and 45 observed as follows : AIR 1957 SC 896

"(44) These observations lay down clearly that there may be occasions and situations in
which the legislature, may with reason, think that the determination of an issue may be
left to an expert executive like the Reserve Bank rather than to Courts without incurring
the penalty of having the law declared void. The law thus made is justified on the ground
of expediency arising from the respective opportunities for action. Of course, the
exclusion of Courts is not lightly to be inferred nor lightly to be conceded. The
reasonableness of such a law in the total circumstances will, if challenged, have to be
made out to the ultimate satisfaction of this Court and it is only when this Court considers
that it is reasonable in the individual circumstance that the law will be upheld.
(45) In the present case, in view of the history of the establishment of the Reserve Bank
as a central bank for India, its position as a Bankers Bank, its control over banking
companies and banking in India, its position as the issuing bank, its power to license
banking companies and cancel their licenses and the numerous other powers, it is
unanswerable that between the court and the Reserve Bank, the momentous decision to
wind up a tottering or unsafe banking company in the interest of the depositors, may
reasonably be left to the Reserve Bank. No doubt, the Court can also, given the time
perform this task. But the decision has to be taken without delay, and the Reserve Bank
already knows intimately the affairs of the banking companies and has had access to their
books and accounts. If the Court were called upon to take immediate action, it would
almost always be guided by the opinion of the Reserve Bank. It would be impossible for
the Court to reach a conclusion unguided by the Reserve Bank if immediate action was
demanded. But the law which gives the same position to the opinion of the Reserve Bank
is challenged as unreasonable. In our opinion such a challenge has no force. The situation
that arose in this case is typical of the occasions on which this extraordinary power would
normally be exercised, and, as we have said already, if the power is abused by the
Reserve Bank, what will be struck down would be the action of the Reserve Bank but not
the law. An appeal against the Reserve Banks action or a provision for an ex post facts
finding by the Court is hardly necessary. An appeal to the Central Government will be
only an appeal from Caesar to Caesar, because the Reserve Bank would hardly act
without the concurrence of the Central Government and the finding by the Court would
mean, to borrow the macabre phrase of Raman Nayar, J. a post-mortem examination of
the corpse of the banking company.
(Emphasis added)
The decision in the case of Palai Bank undoubtedly goes a long way to support the
contention of the appellant in the case in hand.
15. Mr.Trivedi also submitted that the Maharashtra Cooperative Societies Act, 1960 had a
similar provision in Section 110A like the one contained in Section 30(5) of the
Karnataka Act. Sub-section (ii) of Section 110A provided that an order for the winding up
of the bank would be made by the Registrar, if so required by the Reserve Bank of India
in the circumstances referred to in section 13-D of the Deposit Insurance Corporation
Act, 1961. Dealing with the provisions the Bombay High Court had held that the power
conferred under Section 110A of the Maharashtra Cooperative Societies Act should not
be hindered by reading into it the requirement of show cause notice. Learned counsel
cited before us two decisions of the Bombay High Court. One in Mahendra Husanji
Gadkari vs. State of Maharashtra and Ors. [1992 Mah.L.J.1442]
@page-SC999
and the other in Ishwardas Premkumar Choradiya and Anr. vs. State of Mahrashtra and
Ors. [2002 (2) Mah.L.J.844]. In the latter decision, a learned Single Judge of the Bombay
High Court held as follows :
"The question is : whether under Section 110A of the Maharashtra Cooperative Societies
Act, 1960, respondent No.5 was duly bound to give a show cause notice to the petitioners
herein. In the first instance, the section does not provide for a show cause notice. Once
that be so, the question is : whether it can be implied in the absence of provision of show
cause notice whether by implication it is required that a show cause notice must be issued
as it involves civil consequences. Sub-section (3) of Section 110A of the Maharashtra
Cooperative Societies Act, 1960, came up for consideration before a Division Bench of
this Court in the case of Mahendra Husanji vs. State of Maharashtra, 1992 Mah.L.J.1442.
The Division Bench of this Court, after considering the provisions of sub-section (3) of
Section 110A of the Maharashtra Cooperative Societies Act, has held that the Reserve
Bank of India can issue directions only when the situation contemplated by Section 110A
of the Act exists. The directions issued are binding on the Registrar. In other words, once
a direction is issued by the Reserve Bank of India, the Registrar has no discretion in the
matter, but to supersede and appoint an Administrator. Once that be so, and as there is no
discretion left in respondent No.5, it must mean that the right of hearing is excluded.
Once that be so, there was no question of issuing a show cause notice to the petitioner
herein before passing the impugned order. In fact, though not directly in issue in the case
of L. V. Sasmile vs. State of Maharashtra ,1992 CTJ 729, another Division Bench,
considering the material on record, had directed the appointment of an Administrator
under Section 110A of the Maharashtra Cooperative Societies Act. That also would
indicate that there is no requirement under Section 110 for hearing.
16. In our opinion the Bombay High Court has taken the correct view of the matter.
17. On hearing Mr.Trivedi, counsel for the appellant, and on a careful consideration of
the relevant provisions of law and the decisions cited before us we have no hesitation in
accepting the submissions made on behalf of the appellant. We accordingly answer the
question (framed in the beginning of the judgment) in the negative and hold and find that
on receipt of a requisition in writing from the Reserve Bank of India the Registrar
Cooperative Societies is statutorily bound to issue the order of supersession of the
committee of management of the cooperative bank. At that stage the affected bank/its
managing committee has no right of hearing or to raise any objections.
18. The question may here arise whether the principles of natural justice are completely
excluded from the process or it may be that against the requisition, the affected bank may
move the Reserve Bank itself and try to show that it had wrongly arrived at the decision
for its supersession. The other course may be that after the supersession order was issued
by the Registrar that may be challenged before a court of law and in that proceeding one
of ground for assailing the order might be that the decision of the Reserve Bank was
arrived at without giving the affected cooperative bank a proper opportunity of hearing.
We, however, refrain from going into that question as it does not arise in the facts of the
present case.
19. In light of the discussions made above, both the orders passed by the learned Single
Judge and the Division Bench appear quite untenable. Both the orders are accordingly set
aside. However, since the matter has become quite old it needs to be clarified that the
order of supersession passed by the Registrar on January 31, 2002 shall not be
automatically revived but in case the Reserve Bank of India is of the opinion that the
situation so warrants it may issue a fresh requisition to the Registrar Cooperative
Societies, Karnataka, who would on that basis pass the order of supersession as held in
the judgment.
20. The appeal is, accordingly, allowed but with no order as to costs.
Appeal allowed.
AIR 2008 SUPREME COURT 999 "Sat Narain v. State of Haryana"
(From : Punjab and Haryana)
Coram : 2 C. K. THAKKER AND DALVEER BHANDARI, JJ.
Criminal Appeal No. 38 of 2008 (arising out of SLP (Cri. No. 1385 of 2007), D/- 8 -1
-2008
Sat Narain v. State of Haryana through Ministry of Home.
(A) Penal Code (45 of 1860), S.302, S.307 - ATTEMPT TO MURDER - MURDER -
HIGH COURT -
@page-SC1000
Murder - Intention - Quarrel between son of accused and 'D' brother of deceased -
Accused wanted to teach lesson to complainant party for siding with 'D' - Complainant
party was unarmed and not agressors - Accused fired 3 gun shots causing death of
deceased, injury to wife of deceased and brother of deceased avoided shot - It shows
intention of accused to finish complainant side - Evidence of complainant and injured
witnesses found to be reliable - In facts plea of right of self defence, not available to
accused - Conviction of accused under Ss. 302, 307 by Trial Court - Confirmed by High
Court - Not liable to be interfered with. (Paras 14, 17, 22, 24)
(B) Penal Code (45 of 1860), S.300, S.307 - MURDER - ATTEMPT TO MURDER -
Murder - Accused fired gun shots one after other - Killed deceased and also attempted to
kill two persons from complainant party - Fact that two co-accused convicted by Trial
Court but acquitted by High Court by giving benefit of doubt - Would not mean that
accused is entitled to acquittal on that ground. (Para 21)
Cases Referred : Chronological Paras
AIR 1979 SC 1114 : 1979 Cri LJ 916 (Disting.) 23
AIR 1976 SC 966 : 1976 Cri LJ 697 (Disting.) 25
AIR 1974 SC 1570 : 1974 Cri LJ 1035 (Ref.) 26
Puneet Bali, Satinder S. Gulati, Hiten Nehra and Ms. Kamaldeep Narang, for Appellant;
Rajeev Gaur 'Naseem'; Rajesh Ranjan and T. V. George, for Respondent.
Judgement
C. K. THAKKER, J. :- Leave granted.
2. The present appeal is filed against the judgment and order passed by the Addl. Sessions
Judge, Sirsa on May 5, 2004 in Sessions Case No.140 of 1999/2003 and partly confirmed
by the High Court of Punjab and Haryana on August 17, 2006 in Criminal Appeal No.
533-DB of 2004.
3. The case of the prosecution was that the occurrence took place on June 17, 1999 at
about 3.30 a. m. In the said incident, one Chhotu Ram @ Lal Bahadur was killed and
PW14- Ravinder Kumar real brother of deceased Chhotu Ram, as also PW-15 Sakeela-
wife of deceased Chhotu Ram sustained injuries. According to the prosecution, on that
day, a telephonic message was received from civil Hospital, Sangaria Police Station by
ASI Hari Ram relating to the incident. He, therefore, reached Civil Hospital, Sangaria to
find out whether injured Ravinder Kumar, Chhotu Ram and Sakeela were in a position to
give statements. ASI Hari Ram also sent wireless message to the Police Station, Sadar
Dabwali. After some time, he received an information that Chhotu Ram, injured had
succumbed to the injuries. Meanwhile, PW-18-ASI Umed Singh of Sadar Dabwali Police
Station reached the hospital at Sangaria and recorded the statement of injured Ravinder
Kumar, resident of Village Chamar Khera, Police Station Sadul Shahar, District Ganga
Nagar, Rajasthan wherein he stated that he was the resident of Village Chamar Khera. His
sister, Sunita Devi, got married to one Darshan Singh, resident of Dhani Choutala. On the
previous day, i.e. on June 16, 1999, at about 8 p. m., the complainant had gone to see his
sister Sunita Devi at Dhani Choutala where there was a quarrel between his brotherin-law
Darshan Singh and Bablu, son of Sat Narain (appellant herein). Darshan Singh then went
to call Chhotu Ram (Deceased) brother of the complainant, from village Chamar Khera in
a jeep. In the night at about 1.30 a.m., Darshan Singh returned with deceased Chhotu
Ram and his wife Sakeela. At about 3.30 a.m. on June 17, 1999, Sat Narain (appellant
herein) armed with a gun, his son Bablu armed with a gandasi, his wife Savitri armed
with a kasoli and Avtar Singh armed with lathi came in a Maruti car at Dhani of Darshan
Singh and stopped the car on the way. They started abusing Darshan Singh, his wife and
complainant Ravinder Kumar. Chhotu Ram (deceased) asked Sat Narain to go back,
saying that they would talk in the morning. The complainant Ravinder Kumar with his
brother Chhotu Ram (deceased) and Sakeela PW15 then boarded a jeep and all of them
started for Village Sangaria. The deceased Chhotu Ram was on the steering wheel of the
jeep, complainant Ravinder Kumar was sitting by his side and Sakeela was sitting on the
back seat. When their jeep reached near the car of the appellant, all accused raised a
lalkara to teach a lesson to the complainant party for siding with Darshan Singh. The
appellant Sat Narain fired a shot at Chhotu Ram from his licensed gun with intention to
kill him and the pellets hit the right shoulder of the deceased. Sat Narain fired another
shot towards the complainant
@page-SC1001
Ravinder Kumar but he saved himself by taking shelter behind the jeep. The appellant
fired third shot towards Sakeela and the pellets hit on her right hand. Accused Bablu,
Savitri and Avtar Singh also caused injuries with their respective weapons to the
complainant party. The pellets also hit on front portion of the jeep of the complainant
party. They raised alarm and on hearing the noise, Darshan Singh, Vikram Singh and
Sunita came running and raised a lalkara that complainant party need not be frightened
and they had come. On seeing those persons coming near the scene of offence, the
accused fled away with their respective weapons. The injured were then shifted to Civil
Hospital, Sangaria. Chhotu Ram, however, succumbed to the injuries and died. The
motive behind the attack was the dispute between Darshan Singh on the one hand and his
brother Sat Narain on the other hand and complainant party was supporting the case of
Darshan Singh.
4. After the usual investigation, charges were framed against three accused persons Bablu
(A1), son of Sat Narain (appellant), Sat Narain (appellant) (A2) and Avtar Singh (A3).
Savitri was declared as proclaimed offender and the case could not be proceeded against
her.
5. At the trial, the prosecution examined the complainant PW-14 Ravinder Kumar,
brother of deceased Chhotu Ram, first informant and an injured eye-witness, Sakeela
PW15 wife of deceased Chhotu Ram, another injured eye witness. It also examined
PW12 Darshan Lal and PW13 Vikram to prove the motive on the part of the accused
persons in committing the crime.
6. In the statement under Section 313 of the Code of Criminal Procedure, 1973, accused
Nos. 1 and 3 pleaded not guilty. Their case was of total denial. According to them, they
were falsely implicated in the incident. So far as appellant Sat Narain accused No.2 is
concerned, he admitted his presence at the scene of offence. His defence, however, was
that after taking the turn of water in his field he was returning with his licensed gun in the
car. There the complainant party surrounded him and threatened to finish him. The
appellant got frightened and ran to save himself but the complainant party chased him
and stopped him by striking the jeep against his car. The complainant party was armed
with deadly weapons. The appellant accused, in the circumstances, fired a shot in his
defence which hit the deceased Chhotu Ram and on seeing the injuries on the person of
Chhotu Ram, the complainant party ran away from the spot. It was, therefore, his case
that he had committed no offence and should be acquitted.
7. The trial Court, on the basis of the prosecution evidence and considering the defence
version particularly of accused No. 2 (appellant herein), held that the prosecution was
successful in proving the incident in question. It was also established from the evidence
that the complainant party was not the aggressor. No right of self defence was, therefore,
available to appellant Sat Narain. From the injuries sustained by the deceased Chhotu
Ram, PW14 Ravinder Kumar (complainant) and PW15 Sakeela, it was proved beyond
reasonable doubt that the appellant accused had used gun which caused death of deceased
Chhotu Ram and injuries to Sakeela. Accused Bablu and Avtar Singh had also caused
injuries to deceased Chhotu Ram, Ravinder Kumar and Sakeela. Accordingly, the trial
Court convicted the appellant herein for an offence punishable under Section 302 of
Indian Penal Code ('IPC for short) for causing death of deceased Chhotu Ram and
sentenced him to undergo rigorous imprisonment for life and to pay a fine of Rs.
10,000/-, in default of payment of fine, to further undergo imprisonment for two years. It
also convicted the appellant for an offence punishable under Section 307, IPC for attempt
to commit murder of Sakeela, wife of deceased Chhotu Ram, and ordered him to undergo
imprisonment for life and to pay a fine of Rs. 5,000/- in default of payment of fine to
further undergo imprisonment for one year. The Court also convicted the appellant for an
offence punishable under Sections 323 and 324 read with Section 34, IPC for causing
injuries to Ravinder Kumar and Sakeela. Accused Avtar Singh was thus convicted for an
offence punishable under Sections 302 and 307, IPC substantively. He was also convicted
and sentenced for committing offences punishable under Sections 323 and 324 read with
Section 34, IPC. Accused. Nos. 1 Bablu and 3 Avtar Singh were also convicted by the
trial Court.
8. Being aggrieved by the order of conviction and sentence, all the three accused
preferred Criminal Appeals before the High Court. The State of Haryana and complainant
@page-SC1002
Ravinder Kumar filed Revision petitions. All the matters were taken up by the High
Court together. The High Court, by the impugned order, dismissed both the Revisions
filed by the State and the complainant. It allowed Criminal Appeals of accused No. 1
Bablu and accused No. 3 Avtar Singh and held that it was not proved beyond reasonable
doubt that they had committed the offences with which they were charged. Hence, by
giving benefit of doubt, the High Court acquitted them.
9. So far as the appellant is concerned, the High Court, in the light of acquittal recorded
in favour of accused Nos. 1 and 3, acquitted him for the offences punishable under
Sections 323 and 324 read with Section 34, IPC. But the High Court held that the
appellant was rightly convicted by the trial Court for offences punishable under Section
302, IPC substantively as also under Section 307, IPC substantively. Accordingly, the
order of conviction and sentences recorded by the trial Court against the appellant in
respect of those offences was confirmed.
10. Against the order passed by the High Court acquitting Bablu and Avtar Singh,
complainant Ravinder Kumar had approached this Court by filing Special Leave to
Appeal (Criminal) No. 1274 of 2006 which was dismissed on January 15, 2007. In the
present matter by accused No. 2 Sat Narain, notice was issued on March 7, 2007. Records
and proceedings were thereafter called for and the matter was ordered to be placed for
hearing on a non miscellaneous day.
11. We have heard learned counsel for the parties.
12. The learned counsel for the appellant contended that both the Courts were wrong in
convicting the appellant. It was submitted that when the other two accused were acquitted
by the High Court, the appellant could not have been convicted on the basis of the same
evidence. Acquittal of other accused goes to show that the case of the prosecution was not
reliable and it was not proved that the incident took place in the manner as described by
the prosecution. The appellant, in the circumstances, ought to have been granted benefit
of doubt. It was urged that the case of the appellant was that the complainant party was
the aggressor and in exercise of right of private defence, the appellant was compelled to
use licensed fire arm. Only one shot was fired which resulted into unfortunate death of
Chhotu Ram and injuries to Sakeela. The fire arm, however, was used because there was
reasonable apprehension in the mind of the appellant that if he would not use it, he would
either be killed or seriously injured. Both the Courts were, therefore, wrong in not giving
benefit of right of self defence. Finally, it was urged that on the facts and in the
circumstances of the case, at the most, it was a case of exceeding of right of self defence.
The Courts ought to have considered the facts of the case and ought not to have convicted
the appellant for an offence punishable under Section 302, IPC for causing death of
deceased Chhotu Ram and ought not to have awarded imprisonment for life. The Courts
were also in error in convicting the appellant for an offence punishable under Section
307, IPC for attempt to commit murder of PW 15 Sakeela, wife of deceased Chhotu Ram
and in awarding imprisonment for life which is the maximum penalty. It was, therefore,
submitted that in any case, the conviction of the appellant should be converted for an
offence punishable under Section 304, Part II or under Section 304, Part I and to that
extent, the appeal deserves to be allowed.
13. The learned counsel for the respondent-State submitted that murder of deceased
Chhotu Ram was committed by the appellant accused No. 2 when the members of the
complainant party, who were unarmed, were going to their village in a jeep. According to
the counsel, they were prevented from proceeding towards the village by putting Maruti
car in the way which caused obstruction and the complainant party was attacked by the
accused party. The counsel submitted that the appellant herein had fired three shots, out
of them one injured deceased Chhotu Ram who succumbed to the injuries, the second
shot was aimed at complainant Ravinder Kumar but he escaped it by taking shelter
behind the jeep and the third shot injured Sakeela, wife of the deceased. The trial Court,
after appreciating the evidence, believed the story of the prosecution witnesses so far as
the role of the appellant is concerned and convicted him. The High Court confirmed the
view taken by the trial Court. In the circumstances, acquittal of other two accused by the
High Court would not make conviction and finding recorded against the appellant
vulnerable. It was also submitted that in the
@page-SC1003
totality of facts, both the Courts were right in convicting the appellant for an offence
punishable under Section 302 as also under Section 307, IPC and in imposing
imprisonment for life and the order does not suffer from any infirmity.
14. Having heard learned counsel for the parties, in our opinion, no case has been made
out for interference by this Court against the order passed by the trial Court and
confirmed by the High Court so far as the present appellant is concerned. As already
noted by us, both the Courts believed the story of the prosecution, particularly evidence
of PW14 complainant Ravinder Kumar, real brother of the deceased and injured as also
PW 15 Sakeela, wife of deceased Chhotu Ram, another injured eye witness. The Courts
rightly held that the appellant fired three times. The resultant effect was loss of life by
Chhotu Ram and injuries to Sakeela, wife of Chhotu Ram. The appellant also tried to hit
the complainant Ravinder Kumar but he could escape himself by taking shelter of his
vehicle (jeep) and thus could avoid injury by a fire arm. From the evidence, it is also
clearly established and believed by the Courts that the appellant had reloaded his gun
which proves his intention to kill Chhotu Ram and attempt to kill other members of the
complainant party.
15. So far as right of self defence is concerned, in our opinion, both the Courts were right
in holding that the complainant party was not the aggressor. It is also clear from intrinsic
evidence and circumstances, viz. (i) presence of Sakeela at the time of occurrence at 3.30
a.m.; (ii) complainant side was unarmed and empty handed; (iii) all the injured persons
were from complainant's family, etc. It has also come in evidence that it was the accused
side who abused the complainant party. Deceased Chhotu Ram tried to pacify the accused
stating that they would discuss the matter in the morning. Accused party, however, did
not oblige the deceased and the complainant party and did not allow them to leave. When
the complainant party left the place for going to their village in a jeep, they found Maruti
car of the accused in the way which prevented them from proceeding towards their
village and at that stage, the accused attacked them. In the circumstances, in our opinion,
it cannot be said that either the trial Court or the High Court had committed any error of
fact or of law in convicting the appellant.
16. The trial Court considered the defence of the appellant and his right of self defence.
Dealing with the evidence on record and negating the defence theory, it observed;
"There is no reason to disbelieve the oral assertions of the prosecution witnesses which
has been well corroborated from the medical evidence. Even the occurrence has been
admitted by the accused. Even the accused Sat Narain has taken the plea of self defence.
The accused have also admitted the presence of the witnesses Ravinder Kumar, Sakila,
Vikram Singh, Darshan Lal and deceased Chhotu Ram at the place of occurrence. The
defence version of the accused is that the complainant party was the aggressor. They
attacked accused Sat Narain and the accused Sat Narain in his self defence opened fire
but except the suggestion put to the prosecution witnesses there is no evidence on behalf
of the accused to prove that the complainant party ever chased accused Sat Narain armed
with deadly weapons with intention to harm his person and property. So, the plea of self
defence set up by the accused fails. There is specific evidence against the accused that
accused Sat Narain was armed with the double barrel licensed gun, accused Bablu was
armed with a gandasi, accused Avtar Singh was armed with a lathi and accused Savitri
(since declared proclaimed offender) was armed with a kasuli. They all came to the Dhani
of Darshan Lal at 3.30 a.m. at night. So, their common intention can be inferred. Had
they no common intention to cause the death of Chhotu Ram, they might not have come
to the Dhani of Darshan Lal (PW12) at odd hours armed with deadly weapons. Specific
injury and role has been attributed to each of the accused. The testimony of the witness
namely Darshan Lal (PW12), Vikram Singh (PW13), Ravinder Kumar,
complainant/injured (PW14) and Sakeela, injured (PW15) is well corroborated by Dr. R.
C. Ola (PW7) and medico legal reports Ex. PE, Ex. PG and Ex. PH and post mortem
report Ex. PM. Hence, it is proved that the accused on 17-6-1999 in the area of village
Choutala accused Sat Narain in furtherance of common intention of his co-accused
Bablu, Avtar Singh and Savitri (proclaimed offender) caused the death of Chhotu Ram
alias Lal Bahadur by firing shot at him and he also fired at Sakeela in furtherance of
common intention of his remaining accused with the
@page-SC1004
intentio to cause her death and accused Avtar Singh and Bablu in furtherance of common
intention of his co-accused Sat Narain, and Savitri (since declared proclaimed offender)
caused simple hurt with sharp and blunt weapon to Ravinder Kumar. Therefore, the
accused committed offences punishable under Sections 302, 307, 323 and 324 read with
Section 34 of the Indian Penal Code. So, accused Sat Narain is held guilty for the
commission of offences punishable under Sections 302 and 307 of the Indian Penal Code
whereas accused Bablu and Avtar Singh are held guilty for the commission of offences
punishable under Sections 3022 and 307 read with Section 34 of the Indian Penal Code
and accused Bablu and Avtar Singh are further held guilty for the commission of offences
punishable under Sections 323 and 324 of the Indian Penal Code and accused Sat Narain
is held guilty for the commission of offences punishable under Sections 323 and 324 read
with Section 34 of the Indian Penal Code and are convicted thereunder accordingly."
17. It is no doubt true that the High Court allowed the appeals of the other two accused,
i.e. accused Nos. 1 and 3 extending benefit of doubt to them observing that they might
have been roped belatedly. But to us, the High Court was wholly right in holding that
when accused No.2 (appellant) had admitted his presence at the scene of offence as also
use of fire-arm and pleaded the right of self defence, there was no question of his alibi. It
was, therefore, for him to place before the Court the circumstances in which he exercised
the said right. We are conscious and mindful that a right of private defence cannot be
weighed in a 'golden scale' and even in absence of physical injury, in a given case, such
right may be upheld by the Court provided there is reasonable apprehension to life or
grievous hurt to such person. We are equally aware of settled legal position that the onus
of proof on the accused as to exercise of right of private defence is not as heavy as on the
prosecution to prove guilt of the accused and it is sufficient for him to prove the defence
on the touchstone of preponderance of probability. But in the instant case, from the facts
and evidence on record, the right of self defence was not at all available to the appellant.
The complainant party was unarmed and also not an aggressor. It was the appellant, who
came with a loaded gun and fired shots one after the other which resulted in death of
Chhotu Ram, ijury to Sakeela, though Ravinder Kumar could avoid the shot and
remained unhurt. The gun was re-loaded by the appellant which goes a long way to
exhibit his intention to finish the complainant side.
18. The trial Court, in the light of the evidence, did not grant benefit of private defence to
the appellant. The High Court again considered the plea of the accused and negatived it
by observing as under;
"If we take the plea as set up by Sat Narain to be true then in that eventuality some one
else along with Chhotu Ram, Sakeela and Ravinder would have certainly received some
injuries. It is not so. The hired persons do not come without any weapon. Sat Narain does
not say a word about any weapon being carried by the complainant side. He simply states
that he was attacked by them. This goes to show that at the time of occurrence there was
no one else except Chhotu Ram, his wife and Ravinder who were going to village
Babrian from Dhani of Darshan Lal. If the entire scene is visualized in the present set of
circumstances, it can be said that the plea raised by Sat Narain to the effect that he was
called in the Dhani and threatened by Chhotu Ram and others goes against him for the
reason that in any case he had the grudge in his bosom against Chhotu Ram (since
deceased) an outsider for extending help to Darshan Lal. Chhotu Ram in any case would
not be the aggressor."
19. The High Court rightly concluded;
"Therefore, in our considered view, it is a clear case of murder for which he deserves to
be punished under Section 302, IPC".
20. Regarding an offence punishable under Section 307, IPC, the High Court stated;
"We have also appreciated the case of the prosecution with regard to charge of Section
307 IPC qua Sat Narain appellant. We do find some discrepancies in the eye-version
account vis-a-vis the investigation conducted but the same is of no help to him. The case
set up by the prosecution is that Sat Narain had fired three shots; one at Chhotu Ram
hitting on his right side; second at Ravinder, who incidentally escaped unhurt and the
third at Sakeela causing her injuries, who was sitting behind Chhotu Ram on the back
seat of the jeep. The substantive evidence before us is that the gun was reloaded by Sat
Narain. The argument
@page-SC1005
advanced by Mr. Ghai is that only one shot was fired by Sat Narain and a crude padding
has been done by the investigating agency in connivance with the complainant side in
order to project it as a case of repeated firing of shots is of no consequence as we have
already held him guilty for the charge under Section 302 IPC substantively for causing
murder of Chhotu Ram. Still after rescanning the entire case in its right perspective, we
are of the firm view that it is a case of more than one shots, causing pellet injuries to
Sakeela PW also. Therefore, the conviction of Sat Narain appellant under Section 307
IPC also deserves to be re-affirmed. Resultantly his conviction under Sections 302 IPC
and 307 IPC is upheld."
21. It may be mentioned here that the trial Court had convicted accused Nos. 1 and 3 also
but the High Court felt that they were entitled to benefit of doubt and hence they were
ordered to be acquitted. But that does not mean that the appellant who had used his gun,
killed deceased Chhotu Ram and also attempted to kill Sakeela and Ravinder Kumar is
entitled to acquittal or that his conviction and sentence was not in consonance with law.
22. We are also not impressed by the argument of the learned counsel for the appellant
that no case for an offence punishable under Section 302 or 307, IPC was made out
against the appellant and at the most he could be convicted for an offence punishable
under Section 304 Part II or Part I, IPC.
23

. The counsel drew our attention to a decision in Chanan Singh v. State of Punjab,
(1979)4 SCC 399. In that case, reversing the order passed by the High Court and
extending the benefit of right of self defence, this Court set aside the conviction of the
appellant accused. In our opinion, however, the facts of Chanan Singh were totally
different. As observed in the decision, even the High Court had observed in the order that
it was difficult to hold that the eye-witnesses in the case had given true version of the
fight. The presence of injuries on the persons of the accused went to show that there was
a quarrel between them on one side and the complainant party on the other and the
prosecution did not put forward the genesis and the origin of fight correctly. Obviously,
therefore, this Court held that the High Court was wrong in convicting the accused.
AIR 1979 SC 1114

24. In the instant case, both the Courts believed the prosecution version and we are
satisfied that the Courts were right in relying on the evidence of prosecution and in
holding that they were not aggressors and the appellant could not have claimed right of
private defence.
25

. Pratap v. State of Uttar Pradesh, (1976) 2 SCC 798 also does not take the case of the
appellant anywhere. There it was held that burden to prove of self defence on the accused
is not as onerous as that which lies on the prosecution. We are in respectful agreement
with the said proposition of law. It is settled law that the prosecution has to prove its case
beyond reasonable doubt. But it is sufficient if the accused, when he is called upon to
take a defence, proves it on the basis of reasonable probability. He need not prove it
beyond reasonable doubt. But in the instant case, complainant party was not the aggressor
and the right of self defence was not available to the appellant and hence he could not
have committed death of Chhotu Ram nor could have caused injuries to Sakeela. AIR
1976 SC 966

26

. State of U. P. v. Ram Swarup and Anr., (1974) 4 SCC 764 is again on a question of law
and as stated by us, law is well settled on the point and even the State counsel has not
disputed the availability of right of private defence to accused under the IPC. AIR
1974 SC 1570

27. Since both the Courts were right in believing prosecution witnesses who were injured
at the time of incident and since the appellant had used fire arm which caused death of
deceased Chhotu Ram and injuries to PW-15 Sakeela without there being aggression by
them, the order of conviction and sentence recorded by the trial Court and confirmed by
the High Court against the appellant cannot be said illegal or contrary to law. The appeal,
therefore, deserves to be dismissed.
28. For the foregoing reasons, the order of conviction and sentence recorded by the trial
Court and confirmed by the High Court against the appellant is legal, valid and in
consonance with law and calls for no interference. The appeal is, therefore, dismissed.
Appeal dismissed.
@page-SC1006
AIR 2008 SUPREME COURT 1006 "Sunil Poddar v. Union Bank of India"
(From : Allahabad)
Coram : 2 C. K. THAKKER AND ALTAMAS KABIR, JJ.
Civil Appeal No. 86 of 2008 (arising out of SLP (C) No. 3935 of 2006), D/- 8 -1 -2008.
Sunil Poddar and Ors. v. Union Bank of India.
(A) Recovery of Debts Due to Banks and Financial Institutions Act (51 of 1993), S.22(2)
(g) - Civil P.C. (5 of 1908), O.9, R.13 - RECOVERY OF DEBT - DEBT RECOVERY
TRIBUNAL - DECREE - SUMMONS - Ex parte decree - Setting aside - Non-service of
summons - Decree passed by DRT transferred suit - Defendants had put in appearance in
suit and raised preliminary objections - Plea that fresh summonses were not issued on
transfer of suit to DRT - Not tenable as defendants had knowledge of proceedings -
Moreover fresh summonses were issued on same address on which they were earlier
served and even paper publication was done.(Paras 11, 19)
(B) Civil P.C. (5 of 1908), O.5, R.20 - SUMMONS - PLEA - BOOKS, NEWS PAPERS
AND PRESS - Summons - Service - By newspaper publication - Plea that person sought
to be served does not read such newspaper - Not open. (Para 11)
(C) Recovery of Debts Due to Banks and Financial Institutions Act (51 of 1993), S.22(2)
(g) - Civil P.C. (5 of 1908), O.9, R.13 - RECOVERY OF DEBT - DECREE - DEBT
RECOVERY TRIBUNAL - Ex parte decree - Setting aside - Wilful suppression of
relevant facts - Ex parte decree passed by DRT in transferred suit - Applicants defendants
concealing fact that they had appeared before civil court and had raised preliminary
objections - Impression sought to be created that defendants for first time came to know
that an ex parte order had been passed against them by DRT - Order dismissing
application to set aside ex parte decree - Proper. (Paras 20, 22)

Subrat Birla, Subhash Chandra Birla, for Appellants; Jyoti Saxena, Vipin Saxena, M. P.
Shorawala, Hemant Choudhary, S. W. Haider, Ram Niwas, Neeraj Kumar, Sanjeev
Malhotra, for Respondent.
Judgement
1. C. K. THAKKER, J. :-Leave granted.
2. The present appeal is directed against the order dated November 23, 2005 passed by
the High Court of Judicature at Allahabad in Civil Miscellaneous Writ Petition No. 67297
of 2005. By the said order, the High Court dismissed the writ petition filed by the
appellant-writ petitioners and confirmed the order of Debt Recovery Appellate Tribunal,
Allahabad dated September 13, 2005 which in turn affirmed the order passed by the Debt
Recovery Tribunal, Jabalpur on December 20, 2001.
3. To appreciate the controversy raised in the present appeal, few relevant facts may be
stated. It is the case of the appellant that Adhunik Detergent Ltd. (Defendant No. 1 in Suit
No. 44A of 1993 instituted by respondent-Union Bank of India) was incorporated as
Company under the Indian Companies Act, 1956. There was another Company also
known as Adhunik Synthetics Ltd. which was floated by the Directors of Adhunik
Detergent Ltd. According to the appellants, initially, Adhunik Detergent Ltd. had seven
Directors, namely, (1) Satyanarayan Jalan, (2) Krishna Jalan, (3) Chakrapani Jalan, (4)
K.K. Jalan, (5) Sunil Poddar, (6) Sushil Kumar Kanodia and (7) Radhey Shyam Poddar.
Adhunik Detergent Ltd. had taken loan from the respondent-Bank. The appellants herein
as Directors of Adhunik Detergent Ltd. at the relevant time became guarantors for
repayment of loan and executed certain documents in favour of the respondent-Bank. It is
the say of the appellants that there was division of business among the Directors of
Adhunik Detergent Ltd. and Adhunik Synthetics Ltd. Consequent upon the division, the
appellants herein, who were Directors 5, 6 and 7 had resigned as Directors from Adhunik
Detergent Ltd. on August 18, 1989 and they got exclusive control over Adhunik
Synthetics Ltd. From that date onwards, the appellants no more remained as Directors of
Adhunik Detergent Ltd.
4. It was alleged by the respondent-Bank that since Adhunik Detergent Ltd. did not repay
the loan amount, a civil suit came to be filed by the Bank in the Court of District Judge,
Raipur, Madhya Pradesh for recovery of Rs. 1,07,17,177.60 p. In the said suit, over and
above the Company, all the Directors were also joined as defendants. A prayer was made
in the plaint to hold all the defendants jointly and severally liable to pay the amount
claimed by the plaintiff-Bank along with interest, costs and other expenses. Summonses
were issued by the Court and the defendants appeared. So far as the
@page-SC1007
present appellants are concerned, they were not served with the summonses but when
they came to know about the filing of the suit, they appeared and filed written statement
on March 9, 1995 contending inter alia that they had resigned from the Directorship of
the Company (Adhunik Detergent Ltd.) with effect from August 18, 1989 and the Bank
was intimated about such resignation. It was, therefore, contended that they were not
responsible for repayment of loan amount and suit against them was not maintainable.
The appellants, therefore, prayed that they may be deleted from the array of parties. On
March 14, 1995, the appellants also filed an application by raising preliminary objection
as to maintainability of civil suit against them. It was stated in the said application that
preliminary objections were raised in the written statement by the appellants (defendant
Nos. 7, 8 and 9) that no suit against them would lie. It was stated that the preliminary
objection raised by them was fundamental in nature and went to very root of the
jurisdiction of the Court. It was, therefore, prayed that an issue as to maintainability of
suit against defendant Nos. 7 to 9 be framed and decided as preliminary issue before
trying the suit on merits. Another application was also made in November, 1995 raising a
similar objection contending that the suit was not instituted in accordance with law. The
plaint which was filed was not signed by a person authorized to do so and on that count
also, the suit was not tenable. It was further stated that suit against defendant Nos. 7 to 9
was not maintainable. A prayer was made to frame two issues under Order XIV, Rule 1 of
the Code of Civil Procedure, 1908 ("Code" for short) as preliminary issues and to decide
them as such.
5. It may, however, be stated that during the pendency of the suit before the Civil Court,
the Recovery of Debts Due to Banks and Financial Institutions Act, 1993 (hereinafter
referred to as "the Act") came into force and in 1998 the suit filed by the respondent-
Bank came to be transferred to the Debt Recovery Tribunal, Jabalpur ('DRT' for short).
The appellants had no knowledge about the transfer of the suit to DRT nor summonses
were issued by DRT to the appellants at the new address. In the circumstances, nobody
appeared before the DRT and the DRT vide its ex parte judgment and order dated
December 15, 2000 decreed the suit filed by the plaintiff-Bank holding that the Bank was
entitled to recover 1,07,17,177/- with interest and cost from the defendant Nos. 1-9
jointly and severally. The defendants were also restrained from transferring, alienating or
otherwise dealing with or disposing of the hypothecated/mortgaged properties without the
prior permission of DRT.
6. It is asserted by the appellants that they were not aware of the proceedings before the
DRT and no summonses were served upon them. In the circumstances, they could not
remain present before the DRT. It was on December 16, 2000 when Mr. G. Karmakar,
who was working for the appellants, happened to visit the office of M.P. Audyogik Vikas
Nigam Ltd. at Bhopal for some official work that the officials of the Nigam informed him
that a suit pending in the Civil Court, Raipur was transferred to DRT, Jabalpur and an ex
parte decree had been passed against the appellants. Immediately on December 18, 2000,
Mr. Karmakar went to DRT, Jabalpur for getting requisite information and came to know
that notice was sent to the appellants at the old address though new address was available.
An advertisement was also published in a Hindi daily. He also came to know that since
nobody appeared on behalf of the appellants, ex-parte decree had been passed. In the
circumstances, the appellants herein made an application under Section 22(2)(g) of the
Act on January 10, 2001 for setting aside an ex-parte order passed by the DRT. The DRT,
however, on December 20, 2001 dismissed the application. The appellants appealed
against the order passed by the DRT, but the Debt Recovery Appellate Tribunal,
Allahabad ('DRAT' for short) also dismissed the appeal. A writ petition filed against the
order of DRAT also met with the same fate. The High Court dismissed the writ petition.
All these orders have been challenged by the appellants in the present appeal.
7. Notice was issued by this Court on March 6, 2006. After hearing the parties, execution
proceedings were stayed and the matter was ordered to be posted for final hearing. That is
how the matter has been placed before us.
8. We have heard the learned counsel for the parties.
9. The learned counsel for the appellants contended that DRT committed grave error
@page-SC1008
of law and jurisdiction in proceeding with the application and deciding it on merits ex
parte in absence of the appellants. It was submitted that no summonses were served upon
the appellants and thus no opportunity of hearing was afforded to them before passing the
impugned order which is liable to be set aside. The DRT in the circumstances, ought to
have allowed the application for setting aside ex parte order. By not doing so, the DRT
had committed grave error and the said order deserves to be quashed. It was also
submitted that appellants were not informed about the transfer of case from Civil Court to
DRT and no summonses were served upon them. According to the appellants, they had
changed their address and new address was available with the Bank. In spite of that, with
mala fide intention and oblique motive, summonses were sought to be served upon
appellants at an old address but the appellants were not served because of change of
address. Summonses were then published in a Hindi newspaper which had no 'wide
circulation'. That action was also taken with a view to deprive the appellants from
knowing about the proceedings before the DRT so that they may not be able to appear
and defend themselves and the Bank would be able to obtain ex parte order. The
appellants had led the evidence in support of their say that they were not in Mumbai at
the relevant time and they were not subscribers of Hindi newspaper "Navbharat Times".
They had produced necessary particulars and yet the DRT failed to consider the said
evidence in its proper perspective and dismissed the application observing that the
appellants must be deemed to be aware of the proceedings. According to the DRT, the
appellants appeared in Civil Court, filed written statement but all those facts were
suppressed by them while filing the application before the DRT for setting aside ex parte
order. The same mistake has been repeated by the Appellate Tribunal as also by the High
Court. It was submitted that all those facts were not relevant in the present proceedings.
On all these grounds, the orders are liable to set aside by directing the Debt Recovery
Tribunal, Jabalpur to consider the matter afresh and to decide it in accordance with law.
10. The learned counsel for the respondent-Bank, on the other hand, supported the order
passed by the DRT, confirmed by the DRAT as well as by the High Court. An affidavit-
in-reply is filed by Senior Manager (Law) of the respondent-Bank, wherein it was
submitted that the appellants were aware of the proceedings initiated by the Bank against
them. In civil suit, the appellants were joined as defendant Nos. 7-9. They appeared
before the Court through an advocate and filed written statement in March, 1995. They
also raised preliminary objections by filing applications, requesting the Court to treat the
issues as to maintainability of suit and liability of the appellants as preliminary issues. It
was, therefore, clear that they were served with the summonses and were in know of the
proceedings. It was thereafter their duty to take care of their interest, when the suit was
transferred to DRT, Jabalpur. It was further stated that summonses were issued to the
appellants at the addresses at which they were earlier served. In fact, according to the
respondent-Bank, it was the same address which was given by the appellants themselves
before both the Tribunals and before the High Court. But with a view to deprive the Bank
of the legitimate dues and to delay the proceedings initiated against them, they did not
appear before the DRT. Though it was not necessary for the Bank to serve the appellants
once again, they made a prayer to the Bank to get the summonses published in a
newspaper which was done and in "Navbharat Times", Bombay as well as Navbharat
Times, Raipur summonses were published. "Navbharat Times" is having very wide
circulation at both the places, i.e. Bombay as well as at Raipur. It was, therefore, not open
to the appellants to contend that they were not subscribing and/or reading a Hindi
newspaper by producing a bill from a newspaper agent. Such a bill can be obtained from
any vendor. No reliance can be placed on such evidence. Moreover, an extremely
important fact which weighed with both the Tribunals as well as with the High Court was
that in an application under Section 22(2)(g) of the Act for setting aside ex parte order
passed by DRT, the appellants have suppressed material and extremely important fact that
they had appeared before the Civil Court and had filed written statement. The application
proceeded on the footing as if the appellants were never aware of any proceedings
initiated against them by the plaintiff-Bank. The DRT was, therefore, wholly right in
dismissing the application and the said order was correctly confirmed by the DRAT and
by the High Court. No case
@page-SC1009
can be said to have been made out by the appellants to interfere with those orders and the
appeal deserves to be dismissed.
11. Having heard the learned counsel for the parties, in our opinion, the appellants have
not made out any ground on the basis of which the order passed by the DRT, confirmed
by the DRAT and by the High Court can be set aside. From the record, it is clearly
established that the suit was instituted by the plaintiff-Bank as early as in August, 1993.
The appellants who were defendant Nos. 7 to 9 were aware of the proceedings before the
Civil Court. They appeared before the Court, engaged an advocate and filed a written
statement. They raised preliminary objections as also objections on merits. They filed
applications requesting the Court to raise certain issues and try them as preliminary
issues. It was, therefore, obligatory on their part to appear before the DRT, Jabalpur when
the matter was transferred under the Act. The appellants, however, failed to do so. We are
not impressed by the argument of the learned counsel for the appellants that they were not
aware of the proceedings before the DRT and summonses could not be said to have been
duly served. As is clear, summonses were issued earlier and on the same address,
summonses were sought to be served again after the case was transferred to DRT. There
is substance in the submission of the learned counsel for the respondent-Bank that the
appellants had avoided service of summons as they wanted to delay the proceedings. We
are also inclined to uphold the argument of learned counsel for the Bank that in view of
the fact that the appellants were appearing before the Civil Court, it was not necessary for
the Bank to get summonses published in a newspaper after the matter was transferred in
accordance with law to the DRT, Jabalpur. But even that step was taken by the
respondent-Bank. In "Navbharat Times", a Hindi newspaper having wide circulation in
Bombay and Raipur, summonses were published. It cannot be argued successfully that
the appellants were not the subscribers of the said newspaper and were not reading
'Navbharat Times' Hindi Edition. But even otherwise, such contention is wholly
irrelevant. As to bills said to have been produced from the newspaper agent, to us, both
the Tribunals were right in observing that such a bill can be obtained at any time and no
implicit reliance can be placed on that evidence. It is immaterial whether appellants were
subscribers of the said newspaper and whether they were reading it. Once a summons is
published in a newspaper having wide circulation in the locality, it does not lie in the
mouth of the person sought to be served that he was not aware of such publication as he
was not reading the said newspaper. That ground also, therefore, does not impress us and
was rightly rejected by the Tribunals.
12. While dealing with the contention raised by the appellants, the DRT observed;
"When summons are published in newspaper, the Court has to be cautious that it is
published in a newspaper, circulated and widely read in an area where the defendant
stays. Navbharat Times is a national newspaper read not only in Mumbai but also
elsewhere in this country. The summons were published also in a newspaper circulated in
Raipur from where the loan was disbursed. As stated in the main order, the Court is
satisfied that summons were properly published and summons has been properly served."
13. But the fundamental objection which had been raised by the respondent-Bank and
upheld by the Tribunals is legally well-founded. In the application filed by the appellants
before the DRT, Jabalpur under Section 22(2)(g) of the Act, there is no murmur that the
applicants were defendants in the suit instituted in Civil Court; they were served and they
appeared through an advocate and also filed a written statement and other applications
requesting the Court to try certain issues as preliminary issues. It was expected of the
appellants to disclose all those facts. Apart from suppression of fact as to service of
summons and appearance of defendants before the Court, even on legal ground, it was
not obligatory that the appellants should have been served once again.
14. In this connection, we may refer to the provisions of Section 22 of the Act which lays
down procedure to be followed by the Tribunals. The relevant part of the said section
reads thus;
22. Procedure and powers of the Tribunal and the Appellate Tribunal.- (1) The Tribunal
and the Appellate Tribunal shall not be bound by the procedure laid down by the Code of
Civil Procedure, 1908 (5 of 1908), but shall be guided by the principles of
@page-SC1010
natural justice and, subject to the other provisions of this Act and of any rules, the
Tribunal and the Appellate Tribunal shall have powers to regulate their own procedure
including the places at which they shall have their sittings.
(2) The Tribunal and the Appellate Tribunal shall have, for the purposes of discharging
their functions under this Act, the same powers as are vested in a civil Court under the
Code of Civil Procedure, 1908 (5 of 1908), while trying a suit, in respect of the following
matters, namely:-
(a) summoning and enforcing the attendance of any person and examining him on oath;
(b) requiring the discovery and production of documents;
(c) receiving evidence on affidavits;
(d) issuing commissions for the examination of witnesses or documents;
(e) reviewing its decisions;
(f) dismissing an application for default or deciding it ex parte;
(g) setting aside any order of dismissal of any application for default or any order passed
by it ex parte;
(h) . . . . . . . . . . . . . . . . . . . . . . .
15. Bare reading of the above provision makes it clear that the DRT and the DRAT have,
for the purpose of discharging their functions under the Act, the same powers as are
vested in Civil Court under the Code of Civil Procedure, 1908. Clause (g) of sub-section
(2) of Section 22, therefore, has to be read with Rule 13 of Order IX of the Code which
provides for setting aside ex parte decree passed against a defendant. Rule 13 of Order IX
as originally enacted in the Code of 1908 read thus;
13. Setting aside decree ex parte against defendant.In any case in which a decree is
passed ex parte against a defendant, he may apply to the Court by which the decree was
passed for an order to set it aside; and if he satisfies the Court that the summons was not
duly served, or that he was prevented by any sufficient cause from appearing when the
suit was called on for hearing, the Court shall make an order setting aside the decree as
against him upon such terms as to costs, payment into Court or otherwise as it thinks fit,
and shall appoint a day for proceeding with the suit:
Provided that where the decree is of such a nature that it cannot be set aside as against
such defendant only it may be set aside as against all or any of the other defendants also.
16. Original Rule 13 of Order IX of the Code thus provided that when a decree had been
passed ex parte against the defendant who satisfied the Court that summons was not duly
served upon him, the Court was bound to set aside the decree. It was immaterial whether
the defendant had knowledge about the pendency of suit or whether he was aware as to
the date of hearing and yet did not appear before the Court. The Law Commission
considered that aspect and the expression duly served. In its Twenty-seventh Report, the
Commission stated;
1. Under Order IX, Rule 13, if the court is satisfied either that the summons has not been
served, or that the defendant was prevented by sufficient cause from appearing, etc., the
ex parte decree should be set aside. The two branches of the rule are distinctive and the
defendant, whatever his position may be in respect of one branch, is the Court that he has
made good his contention in respect of the other branch.
2. Now, cases may arise where there has been a technical breach of the requirements of
due service, though the defendant was aware of the institution of the suit. It may well be,
that the defendant had knowledge of the suit in due time before the date fixed for hearing,
and yet, apparently he would succeed if there is a technical flaw. This situation can arise
e.g., where the acknowledgment on the duplicate of the summons has not been signed.
There may be small defects in relation to affixation, etc., under Order V, Rule 15. At
present, the requirements of the rules regarding service must be strictly complied with,
and actual knowledge (of the defendant) is immaterial. (There are not many decisions
which hold that even where there has not been due service, yet the decree can be
maintained, if the defendant knew the date of hearing.)
3. Where a literal conformity with the C.P.C. is wanting, the second part of column third
of Article 164, Indian Limitation Act, 1908 (now Article 123, Limitation Act, 1963)
applies. As to substituted service, see discussion in under-mentioned decision.
4. The matter was considered exhaustively by the Civil Justice Committee, which
recommended a provision that a decree
@page-SC1011
should not be set aside for mere irregularity. Local Amendments made by several High
Courts (including Allahabad, Kerala, Madhya Pradesh, Madras and Orissa) have made a
provision on the subject, though there are slight variations in the language adopted by
each. Such a provision appears to be useful one, and has been adopted on the lines of the
Madras Amendment.
17. The Commission again considered the question and in its Fifty-fourth Report,
reiterated;
9.12. Under Order 9, Rule 13, if the Court is satisfied either that the summons has not
been served, or that the defendant was prevented by sufficient cause from appearing, etc.,
the ex parte decree should be set aside. The two branches of the rule are distinctive, and
the defendant, whatever his position may be in respect of one branch, is entitled to benefit
of the other branch, if he satisfies the Court that he has made good his contention in
respect of the other branch.
9.13. In the earlier Report, several points were considered with reference to this rule, and
amendments suggested on one point,-the broad object being to ensure that a decree shall
not be set aside merely on the ground of irregularity in service, if the defendant had
knowledge of the decree. After consideration of the points discussed in the earlier Report,
we have reached the same conclusion.
18. Accepting the recommendations of the Law Commission, the rule was amended by
the Code of Civil Procedure (Amendment) Act, 1976. Rule 13 of Order IX with effect
from February 1, 1977 now reads thus;
13. Setting aside decree ex parte against defendant- In any case in which a decree is
passed ex parte against a defendant, he may apply to the Court by which the decree was
passed for an order to set it aside; and if he satisfies the Court that the summons was not
duly served, or that he was prevented by any sufficient cause from appearing when the
suit was called on for hearing, the Court shall make an order setting aside the decree as
against him upon such terms as to costs, payment into Court or otherwise as it thinks fit,
and shall appoint a day for proceeding with the suit;
Provided that where the decree is of such a nature that it cannot be set aside as against
such defendant only it may be set aside as against all or any of the other defendants also:
Provided further that no Court shall set aside a decree passed ex parte merely on the
ground that there has been an irregularity in the service of summons, if it is satisfied that
the defendant had notice of the date of hearing and had sufficient time to appear and
answer the plaintiffs claim.
Explanation.- Where there has been an appeal against a decree passed ex parte under this
rule, and the appeal has been disposed of on any ground other than the ground that the
appellant has withdrawn the appeal, no application shall lie under this rule for setting
aside that ex parte decree.
(Emphasis supplied)
19. It is, therefore, clear that the legal position under the amended Code is not whether
the defendant was actually served with the summons in accordance with the procedure
laid down and in the manner prescribed in Order V of the Code, but whether (i) he had
notice of the date of hearing of the suit; and (ii) whether he had sufficient time to appear
and answer the claim of the plaintiff. Once these two conditions are satisfied, an ex parte
decree cannot be set aside even if it is established that there was irregularity in service of
summons. If the Court is convinced that the defendant had otherwise knowledge of the
proceedings and he could have appeared and answered the plaintiffs claim, he cannot put
forward a ground of non-service of summons for setting aside ex parte decree passed
against him by invoking Rule 13 of Order IX of the Code. Since the said provision
applies to Debt Recovery Tribunals and Appellate Tribunals under the Act in view of
Section 22(2)(g) of the Act, both the Tribunals were right in observing that the ground
raised by the appellants could not be upheld. It is not even contended by the appellants
that though they had knowledge of the proceedings before the DRT, they had no
sufficient time to appear and answer the claim of the plaintiff-bank and on that ground, ex
parte order deserves to be set aside.
20. In our opinion, the Tribunals were also right in commenting on the conduct of the
appellants/defendants that they were appearing before Civil Court through an advocate,
had filed written statement as also applications requesting the Court to treat and try
certain issues as preliminary issues. All those facts were material facts. It was,
@page-SC1012
therefore, incumbent upon the appellants to disclose such facts in an application under
Section 22(2)(g) of the Act when they requested the DRT to set aside ex parte order
passed against them. The appellants deliberately and intentionally concealed those facts.
There was no whisper in the said application indicating that before the Civil Court they
were present and were also represented by an advocate. An impression was sought to be
created by the defendants/appellants as if for the first time they came to know in
December, 2000 that an ex parte order had been passed against them and immediately
thereafter they had approached the DRT. The Debt Recovery Tribunal, Jabalpur,
therefore, in our opinion was right in dismissing the said application. In an appeal against
the said order, the DRAT observed that the appellants had willfully suppressed the fact
that they were not in the know of the proceedings when the same was proceeding in the
Civil Court. The DRAT correctly stated that even if it is taken to be true that the
appellants did not receive notice from the DRT, it was their duty to make necessary
inquiry in the proceedings when the case had been transferred to the DRT. The Appellate
Tribunal rightly concluded;
"In the present case, the appellants very artistically have suppressed the fact of their filing
of written statement in the case while it was proceeding in the Civil Court and were being
represented by their lawyer till the date of its transfer to the Tribunal at Jabalpur."
21. The High Court, in our judgment, was equally right in dismissing the petition
confirming the finding of the DRAT that the appellant had artistically suppressed material
fact and no interference was called for.
22. Finally, we are exercising discretionary and equitable jurisdiction under Article 136 of
the Constitution. From the facts and circumstances of the case in their entirety, we do not
feel that there is miscarriage of justice. On the contrary, we are convinced that the
appellants had not come forward with clean hands. They wanted to delay the proceedings.
Though they were aware of the proceedings pending against them, had appeared before
the Civil Court, but then did not care to inquire into the matter. Even after ex parte order
was passed, in an application for setting aside the order, they had not candidly disclosed
all the facts that they were aware of such proceedings and were represented by a counsel.
In the light of all these facts and keeping in view the provisions of Section 22 (2)(g) of
the Act read with Rule 13 of Order IX of the Code, if the Debt Recovery Tribunal
dismissed the application and the said order was confirmed by the Debt Recovery
Appellate Tribunal and by the High Court, it cannot be held that those orders were wrong
and ex parte order should be quashed. The prayer of the appellants, therefore, has no
substance and cannot be accepted.
23. For the foregoing reasons, the appeal deserves to be dismissed and is accordingly
dismissed with costs.
Appeal dismissed.
AIR 2008 SUPREME COURT 1012 "Gaudiya Mission v. Shobha Bose"
(From : Allahabad)
Coram : 2 C. K. THAKKER AND ALTAMAS KABIR, JJ.
Civil Appeal No. 398 of 2008 (arising out of SLP (C) No. 12465 of 2006), D/- 15 -1
-2008.
Gaudiya Mission v. Shobha Bose and Anr.
(A) Succession Act (39 of 1925), S.384 - SUCCESSION - APPEAL - OBJECT OF AN
ACT - DISMISSAL - Appeal - Scope - Testamentary suit filed on original side of High
Court - Single Judge deciding suit by entering into merits of case - Appeal against, filed
before Division Bench - Is statutory appeal - All questions both of facts and law could be
urged in appeal - Dismissal of appeal in limine - Not proper.
Spl. Appeal No. 527 of 2006, D/- 24-5-2006 (All.), Reversed. (Para 14)
(B) Evidence Act (1 of 1872), S.73 - DOCUMENTS - WILL - SALE DEED -
Comparison of handwriting - Power of Court - Signature on registered Will disputed on
basis of sale deed executed by testator long back - Court rejecting prayer for sending
documents to expert - Comparing signatures itself and giving finding - Modality adopted
by Court is improper. (Para 14)
(C) Allahabad High Court Rules (1952), Chap.VIII, R.5 - HIGH COURT - APPEAL -
OBJECT OF AN ACT - Intra Court appeal - Scope - Not limited to question of law - All
questions, whether of facts or of law, could be raised. (Paras 15, 18)
Cases Referred : Chronological Paras
AIR 1974 SC 2048 (Rel. on) (Pt. C) 16
@page-SC1013

Dr. R. G. Padia, Sr. Advocate, Rajiv Mehta, A. Henry, B. Aggrawal, for Appellant; E. C.
Agrawala, Mahesh Agarwal, Rishi Agrawala, Gaurav Goel, Amit Sharma, Ms. Neha
Aggrawal, for Respondents.
Judgement
1. C. K. THAKKER, J.:-Leave granted.
2

. The present appeal is directed against summary dismissal of Special Appeal No. 527 of
2006 by a Division Bench of the High Court of Judicature at Allahabad on May 24, 2006.
By the said order, the Division Bench of the High Court confirmed the judgment and
order dated March 28, 2006 passed by a Single Judge of that Court in Testamentary Case
No. 8 of 2000 on the Original Side of Testamentary and Intestate Jurisdiction. 2006
(4) ALJ (NOC) 658

3. Brief facts of the case are that one Narendra Nath Bose, resident of Allahabad and
working as Lecturer, Government Girls Inter College, Gonda (U.P.) was having his
family consisting of his wife Radha Rani Bose and three daughters, (i) Asha Bose, (ii)
Uma Bose and (iii) Shobha Bose (respondent herein). All the three sisters (daughters of
deceased Narendra Nath) decided not to marry. Kum. Asha Bose died on June 9, 1990.
4. Kum. Uma Bose was serving as a Lecturer in Government Girls College, Gonda, U.P.
and was a disciple of Gaudia Mission (hereinafter referred to as the Mission) a Society,
registered under the Societies Registration Act, 1860 (appellant herein). She was closely
associated with the activities of the Mission and also got printed various religious books
and literature for the Mission by spending considerable amount. It is the case of the
appellant that after her retirement from the College, she was living in Vrindaban and not
with her sister Kum. Shobha Bose-respondent at Allahabad. She continued to take interest
in the activities of the Mission. Because of her attachment and dedication towards work
of the Mission, she executed a Will on December 28, 1994 bequeathing her properties in
favour of the Mission. The Will was duly registered. On September 9, 1996, said Kum.
Uma Bose executed a Codicil in favour of the appellant Mission in relation to certain
further sums and movable properties which had come to her share and also her share in
immovable properties at Vrindaban. The Codicil was also registered on September 19,
1996. On November 18, 1996, Kum. Uma Bose died in Vrindaban.
5. According to the appellant Mission, Kum. Shobha Bose-respondent No.1 herein-real
sister of late Uma Bose, never kept any relation with her and never looked after her and
continued to stay at Allahabad only. It is the case of the appellant that in 1997, respondent
No. 1 Kum. Shobha Bose applied for a Letter of Administration to the estate of deceased
Kum. Uma Bose concealing real facts of Will and Codicil in favour of appellant-Mission.
The Letter of Administration was granted to her on September 26, 1997 but on
application at the instance of the appellant Mission, the certificate was cancelled. The
appellant-Mission applied for Probate for the Will executed by deceased Kum. Uma Bose
by filing Probate Case No. 174 of 1997 before the Division Bench of Allahabad High
Court. The application was, however, withdrawn by the appellant with liberty to file fresh
proceedings. Respondent No. 1-Kum. Shobha Bose filed Testamentary Suit for the estate
of deceased Kum. Uma Bose. The appellant-Mission filed its objections to the said suit
and claimed that it was the appellant who was entitled to the property of deceased Uma
Bose and Kum. Shobha Bose had no right, title or interest in the estate of late Kum. Uma
Bose. Issues were framed by the Court and witnesses were examined. Respondent No.1-
Kum. Shobha Bose produced a sale deed said to have been executed by Kum. Uma Bose
long back and contended that signature on the sale deed and that in the Will did not tally.
The appellant-Mission applied to the High Court that the signatures be examined by
hand-writing expert. But the prayer was not granted by the Court.
6. The matter was then heard by a Single Judge of the High Court and by judgment and
order dated March 28, 2006, the learned Judge himself compared the hand-writing of
deceased Kum. Uma Bose in sale deed and in the Will and held that the Will was
surrounded by suspicious circumstances. He also observed that the deceased was not
living in Vrindaban and was not being looked after by the Mission as claimed by the
Mission. He held that Kum. Shobha Bose was the real sister of deceased Kum. Uma
Bose. In that capacity, she was entitled to a Letter of Administration of the estate and
credits of late Kum. Uma Bose. The Court consequently ordered payment of amount
received from the Bank by the deceased to Kum.
@page-SC1014
Shobha Bose holding that she was entitled to the said amount. She was also held to be the
owner of the house. The learned Judge also imposed cost of Rs.25,000/- on the Mission-
appellant herein.
7. Being aggrieved by the order passed by the trial Court (Single Judge), Special Appeal
was filed by the appellant herein before the Division Bench of the High Court which, as
stated above, was dismissed in limine by the Division Bench holding that the view taken
by the Single Judge was correct and appeal did not require admission. Hence, the present
appeal has been preferred by the appellant-Mission.
8. Notice was issued by this Court on August 11, 2006. Interim stay of recovery was also
granted. Affidavit in reply and affidavit in rejoinder were thereafter filed. The matter was
ordered to be posted for final hearing and that is how the matter has been placed before
us.
9. We have heard learned counsel for the parties.
10. The learned counsel for the appellant Mission has raised several contentions. He
submitted that an appeal filed before Division Bench of the High Court was a regular
statutory appeal. It was in the nature of First Appeal and all questions of fact as well as of
law could be agitated. It was, therefore, incumbent on the Division Bench of the High
Court to admit the appeal and to decide it after appreciating the evidence on record by a
detailed and reasoned judgment. Dismissal of appeal in limine by the Division Bench was
improper and on that ground alone, the impugned order deserves to be set aside. It was
also urged that even if the appeal is considered to be an intra-court appeal, all questions
of fact and of law could be argued and the Division Bench cannot refuse to admit the
appeal by dismissing it at the threshold observing that it agrees with the finding recorded
by the trial Court. It was also urged that as many as thirteen issues were framed by the
trial Court and all those issues were required to be considered by the Division Bench.
They raised disputed questions of fact which necessitated appreciation of evidence,
application of mind by the Division Bench and a reasoned judgment. The counsel argued
that from the facts, it was clearly established that there was a Will executed by the
deceased Kum. Uma Bose in 1994 which was duly registered. Similarly, there was a
Codicil of 1996 which was also registered. They ought to have been considered but they
were not considered by the Division Bench in their proper perspective. According to the
counsel, if respondent No.1 challenged legality and validity of the Will, the proceedings
initiated on the Original Side of the High Court were not maintainable and the only Court
which had jurisdiction to decide such question was a competent Civil Court which had
exclusive jurisdiction in such matters. Testamentary Suit instituted by the respondent No.
1 before the learned Single Judge on the Original Side of the High Court was, therefore,
not maintainable. The counsel made serious grievance against the order passed by the
learned Single Judge and confirmed by the Division Bench on the ground that they had
committed grave error in comparing signatures and hand-writings of deceased Kum. Uma
Bose on sale deed and in the Will and in coming to the conclusion that the signatures and
the hand-writings differed and they were not of one and the same person. The counsel
urged that this Court has held in several cases that no comparison of hand-writing should
be made by a Court as it is the function of an expert. It is dangerous, hazardous and risky
to record a finding on comparison of hand-writings on different documents and it should
be avoided. In the instant case, though the said objection was taken before the trial Court
by the appellant and an application was also made to send the hand-writings to experts,
the prayer was rejected and the Court proceeded to undertake the exercise which was not
warranted. But, even otherwise, the appellant contended that the Court was in error in
comparing handwritings of Kum. Uma Bose in the Will on the one hand and in the sale
deed on the other hand. So far as sale deed is concerned, it was said to have been
executed in 1987 whereas Will was executed in 1994. There was thus substantial time lag
between the sale deed and the Will and the said important and vital fact had not been
considered properly by the learned Single Judge. On all these grounds, it was submitted
that the appeal deserves to be allowed by setting aside the orders passed by both the
Courts or in any case by the Division Bench of the High Court by remitting the matter to
the appellate Court and directing it to admit the appeal and to decide it by a reasoned
judgment.
11. The learned counsel for the
@page-SC1015
respondent, on the other hand, supported the order passed by the Division Bench of the
High Court. According to him, the Division Bench did not think it fit to admit the appeal
since it agreed with the reasons recorded and conclusions reached by the learned Single
Judge who had considered all points and no fault can be found against such order. It was
argued that on the basis of the evidence on record, the learned Single Judge held that as a
sole surviving member of the family of Narendra Nath Bose, respondent No.1-Kum.
Shoba Bose was entitled to the property of her elder sister late Kum. Uma Bose and no
disputed questions of fact were involved in the suit. On the basis of evidence adduced by
the parties, the learned Single Judge held that there was nothing to show that the
appellant-Mission was entitled to any relief and the respondent No. 1 had right to apply
for Letter of Administration who could get the said relief and the Division Bench agreed
with the said conclusion. Regarding comparison of signature and hand-writings, it was
submitted by the counsel that it is open to a Court of Law to compare hand-writings and
reliance in this connection was placed on Section 73 of the Evidence Act, 1872. It was
submitted that the law enables a Court to make comparison of hand-writings and if the
statutory power was exercised by the learned Single Judge, it cannot be contended that
the Court was wrong. The appeal, hence, deserves to be dismissed.
12. Having heard learned counsel for the parties, in our opinion, on a short ground, the
appeal deserves to be allowed. From the facts, it is clear that great many questions were
involved in the Testamentary Suit instituted by the respondent No. 1. Several issues were
framed by the trial Court and the suit was decided by a detailed judgment entering into
merits of the matter. In our opinion, therefore, the learned counsel for the appellants is
right in submitting that the Division Bench of the High Court ought to have admitted the
appeal. It was not right in dismissing statutory appeal in limine. No doubt, an order
dismissing the appeal is a speaking order containing few pages. But, in our opinion, the
appeal instituted by the appellant before the Division Bench was a statutory appeal under
Section 384 of the Indian Succession Act, 1925.
13. Section 384 reads thus:
384. Appeal.- (1) Subject to the other provisions of this Part, an appeal shall lie to the
High Court from an order of a District Judge granting, refusing or revoking a certificate
under this Part, and the High Court may, if it thinks fit, by its order on the appeal, declare
the person to whom the certificate should be granted and direct the District Judge, on
application being made therefor, to grant it accordingly, in supersession of the certificate,
if any, already granted.
(2) An appeal under sub-section (1) must be preferred within the time allowed for an
appeal under the Code of Civil Procedure, 1908 (5 of 1908).
(3) Subject to the provisions of sub-section (1) and to the provisions as to reference to
and revision by the High Court and as to review of judgment of the Code of Civil
Procedure, 1908, (5 of 1908) as applied by Section 141 of that Code, an order of a
District Judge under this Part shall be final.
14. Bare reading of the aforesaid provision leaves no room for doubt that it is a regular
appeal and all questions i.e. questions of fact and of law are open to urge before the
appellate Court. In the circumstances, it was expected of the Division Bench to consider
all submissions and contentions of the parties. We are also of the view that the argument
of the learned counsel for the appellants that the Will as well as Codicil were executed in
1994 and 1996 and both were duly registered, was one of the relevant factors which
ought to have been kept in mind by the Division Bench. Again, even if it is held that a
Court of law has power, authority and jurisdiction to compare hand-writings under
Section 73 of the Evidence Act, the point raised as to whether on the facts and in the
circumstances of the case and in the light of an application made by the appellant-
Mission that they may be sent to hand-writing expert, the Court should have undertaken
the exercise of comparison of hand-writings was a relevant issue. This is coupled with the
fact that the sale deed said to have been executed by deceased Kum. Uma Bose was of
1987 and Will and Codicil were of 1994 and 1996 respectively.
15. We are not prepared to agree with the learned counsel for the respondent that the
scope of appeal before the Division Bench was very much limited. Even in an appeal
from a decision of a Single Judge of the High Court in First Appeal, a Division Bench of
the High Court has power to
@page-SC1016
consider all questions, whether of facts or of law, which could be raised before a Single
Judge. In other words, the party aggrieved before the Division Bench in Intra-
Court/Letters Patent Appeal can raise all those questions which could be raised before a
Single Judge of the High Court in First Appeal.
16. In Asha Devi v. Dukhi Sao, (1975) 1 SCR 611: AIR 1974 SC 2048, a similar question
came up for consideration before this Court. There, a First Appeal came up for hearing
before a Single Judge of the High Court and was disposed of. Against the said order, a
Letters Patent Appeal was filed. A preliminary objection was raised on behalf of the
respondents that since it was an appeal from an order passed by a Single Judge of the
High Court in First Appeal, the appeal before the Division Bench was really in the nature
of Second Appeal and questions of law only could be agitated in such Letters Patent
Appeal.
17. Negativing the contention and holding that the scope of appeal before the Division
Bench was similar to one before a Single Judge, this Court stated:
"There is no dispute that an appeal lies to a Division Bench of the High Court from the
judgment of a Single Judge of that Court in appeal from a judgment and decree of a court
subject to the superintendence of the High Court. The only question is whether the power
of a Division Bench hearing a Letters Patent appeal under Clause 10 of the Letters Patent
of Patna High Court or under the analogous provisions in the Letters Patent of other High
Courts is limited only to a question of law under Section 100 of the CPC or has it the
same power which the Single Judge has as a first Appellate Court in respect of both
questions of fact and of law. The limitations on the power of the Court imposed by
Sections 100 and 101 of the CPC cannot be made applicable to an Appellate Court
hearing a Letters Patent appeal from the judgment of a Single Judge of that High Court in
a first appeal from the judgment and decree of the Court subordinate to the High Court,
for the simple reason that a Single Judge to the High Court is not a Court subordinate, to
the High Court".
18. From what has been observed by this Court in Asha Devi and considering the fact that
an appeal under Section 384 of the Indian Succession Act is a regular appeal, we are of
the view that arguable points had been raised by the appellant-Mission in the appeal
which ought to have been admitted by the Division Bench.
19. On overall considerations, in our judgment, the appeal deserves to be allowed by
setting aside the order passed by the Division Bench and by ordering admission of appeal
remitting it to the Division Bench of the High Court to be decided in accordance with law
after recording reasons.
20. For the foregoing reasons, the appeal is allowed. The order passed by the Division
Bench in Gaudiya Mission v. Km. Shobha Bose and Anr. is set aside. The appeal will
stand admitted. The Division Bench will now hear the parties on merits and decide the
case in accordance with law by a reasoned judgment. On the facts and in the
circumstances of the case, however, there shall be no order as to costs.
21. Before parting with the matter, we may clarify that we have not expressed any
opinion on merits of the matter one way or the other. All the observations made by us
hereinabove have been made only for the purpose of deciding the present appeal and as
and when the matter will be placed for hearing before the Division Bench, the same will
be decided strictly on its own merits without being influenced by the above observations.
22. The appeal is accordingly allowed.
Appeal allowed.
AIR 2008 SUPREME COURT 1016 "Atul Singh v. Sunil Kumar Singh"
(From : (2005) 4 Pat LJR 110)
Coram : 2 G. P. MATHUR AND AFTAB ALAM, JJ.
Civil Appeal No. 10 of 2008 (arising out of SLP (C) No. 24729 of 2005), D/- 4 -1 -2008.
Atul Singh and Ors. v. Sunil Kumar Singh and Ors.
(A) Arbitration and Conciliation Act (26 of 1996), S.8(1) - ARBITRATION AND
CONCILIATION - PARTNERSHIP DEED - DOCUMENTS - Reference to arbitration -
Dispute relating to reconstituted partnership firm - Neither 'R' nor his heirs, plaintiffs
were parties to reconstituted partnership deed dated 17-2-1992 - No document produced
as defined in S. 7 of 1996 Act which may contain signature of either 'R' or plaintiffs his
heirs - No document to show that either 'R' or plaintiffs were parties to clause relating to
arbitration contained in partnership
@page-SC1017
deed dated 17-2-1992 - Thus S. 8 would not be applicable to any dispute concerning said
partnership deed dated 17-2-1992 - Therefore matter cannot be referred to arbitration.
(Para 8)
(B) Arbitration and Conciliation Act (26 of 1996), S.8(1) - ARBITRATION AND
CONCILIATION - PARTNERSHIP DEED - DEED - Reference to arbitration - Suit for
declaration that reconstituted partnership deed was void and illegal and for rendition of
accounts and share of profits - Neither plaintiffs nor their father 'R' from whom they
derive title were parties to said deed - Therefore, in order to get reliefs it was absolutely
essential for plaintiffs to have partnership deed declared as illegal, void and inoperative -
Relief for such declaration could only be granted by civil Court and not by arbitrator -
Matter could not therefore be referred to arbitration.
2005 (4) Pat LJR 110, Reversed. (Para 9)
(C) Arbitration and Conciliation Act (26 of 1996), S.8(2) - ARBITRATION AND
CONCILIATION - AGREEMENT - Reference to arbitration - Failure of applicant to file
original arbitration agreement or duly certified copy thereof - Amounts to non-
compliance of mandatory provision of sub-section (2) of S. 8 - Dispute could not
therefore be referred to arbitration. (Para 10)
Cases Referred : Chronological Paras
AIR 1962 SC 1810 (Ref.) 7
AIR 1959 SC 1357 7
AIR 1959 SC 1362 (Ref.) 7
AIR 1954 Mad 528 (Ref.) 7
AIR 1952 SC 119 (Ref.) 7
ILR (1948))2 Cal 171 (Ref.) 7
1942 AC 356 (Ref.) 7
Ranjit Kumar, Sr. Advocate, Ms. Binu Tamta, Kamal Gupta, for Appellants; S. B. Sanyal,
Sr. Advocate, Devashish Bharuka, Abhishek Sinha, Dr. Sushil Balwada, for Respondents.
Judgement
G. P. MATHUR, J.:-Leave granted.
2. This appeal, by special leave, has been preferred against the judgment and order dated
4.8.2005 of Patna High Court, by which the Civil Revision Petition preferred by Sunil
Kumar Singh (defendant No.3 in the suit) was allowed and the order passed by the trial
Court on 17.3.2005 rejecting his prayer for referring the dispute for arbitration under
Section 8 of the Arbitration and Conciliation Act, 1996 (hereinafter referred to as the
1996 Act) was set aside.
3. In order to understand the controversy raised, it is necessary to mention the basic facts
of the case. The appellants herein filed Title Suit No.296 of 1998 in the Court of Sub-
Judge-I, Patna, against Sunil Kumar Singh (defendant No.3) and 5 others for a
declaration that the reconstituted partnership deed dated 17.2.1992 (effective from
1.4.1992) is illegal, void and without jurisdiction and was also without any intention or
desire of Shri Rajendra Prasad Singh (who died after 17.2.1992) to retire from the
partnership. A declaration was also sought that the plaintiffs being heirs of late Shri
Rajendra Prasad Singh will be deemed to be continuing as partners to the extent of his
share. It was further prayed that a decree for rendition of accounts of the firm from
1.4.1992 up to date may be passed and the defendants may be directed to pay to the
plaintiffs their share of the profits of the partnership as well as interest and principal
amount of unsecured loan advanced by the firm. A further relief for grant of an ad interim
injunction restraining the respondents from mismanaging and misappropriating the funds
of the firm was also sought, besides appointment of a Receiver during the pendency of
the suit to manage the firm.
4. The case of the plaintiffs, as set out in plaint, in brief, is as under. A partnership firm in
the name and style of M/s. Veena Theatres Pvt. Ltd. was formed by a deed of partnership
on 25.12.1959 and the business of the firm was to book pictures with film distributors at
various places and to get them screened or exhibited in the picture hall owned by M/s
Veena Theatres Pvt. Ltd. The capital in the firm was invested by the members of the
family of Shri Shatrughan Prasad Singh. Shri Rajendra Prasad Singh also subsequently
became a partner of the firm by making investments and a deed of partnership was
executed on 20.12.1972. The partnership was reconstituted on 21.5.1976, in which the
share of Shri Rajendra Prasad Singh was 21% and on the death of Brij Mohan Prasad
Singh, his widow Smt. Sona Devi was inducted as a partner and a fresh deed was
executed on 13.1.1989 in which Shri Rajendra Prasad Singh continued to be a partner
having 21% share. Shri Rajendra Prasad Singh died on 5.9.1992 leaving behind plaintiff
Nos.2, 3, 5 and 7, who are his grandsons, as his heirs. The wife and two sons of Shri
Rajendra
@page-SC1018
Prasad Singh had predeceased him. The case of the plaintiffs further is that the defendants
fraudulently executed another partnership deed on 17.2.1992, in which Shri Rajendra
Prasad Singh was not shown as one of the partners, though he had neither given any
consent nor had expressed his desire for retiring from the partnership. The plaintiffs made
a request to the defendants to give the accounts of the partnership firm and give them
their share of profits, but the defendants refused to do so on the ground that they or their
predecessor-in-interest viz. Shri Rajendra Prasad Singh were not partners in the
partnership deed which was executed on 17.2.1992. The suit was accordingly filed on
1.8.1998 for the reliefs mentioned above.
5. The suit proceeded ex parte against all the defendants except Birendra Kumar Singh
(defendant No.2), who appeared before the trial Court and moved an application for
giving time to file written statement. He also moved an application for rejecting the plaint
under Order VII, Rule 11, CPC on 18.9.1998, which was rejected on 16.1.2002. A review
petition seeking review of the aforesaid order was filed but the same was dismissed on
29.4.2004. He, thereafter, moved an application on 3.8.2004 for referring the dispute for
arbitration, but subsequently his counsel conceded that the said application was not
maintainable.
6. Sunil Kumar Singh (defendant No.3), who is son of Birendra Kumar Singh (defendant
No.2) did not put in appearance despite service of summons and the trial Court vide order
dated 28.6.1999 directed to proceed ex parte against him. After more than 5 years
defendant No.3 moved two applications on 14.10.2004 for setting aside the order dated
28.6.1999 by which the Court had directed to proceed ex parte against him and also
sought time to file written statement. On the concession made by the plaintiffs, the order
to proceed ex parte against defendant No.3 was set aside on 3.11.2004. The defendant
No.3 thereafter moved an application on 25.11.2004 under Section 34 of the Arbitration
Act, 1940 praying that in view of the arbitration clause in the agreement dated 13.1.1989,
the proceedings in the suit may be stayed and the matter may be referred to arbitration.
The plaintiffs filed an objection to the application on 1.12.2004. On 16.12.2004,
defendant No.3 filed a supplementary petition in support of his earlier petition dated
25.11.2004 reiterating the prayer for referring the dispute to arbitration. Subsequently, on
28.2.2005, defendant No.3 moved a petition purporting to be supplementary petition to
the petitions dated 25.11.2004 and 16.12.2004, wherein it was averred that as the suit is
of the year 1998, to avoid any chances of confusion, his earlier petitions may be treated to
have been filed under Section 8 of the Arbitration and Conciliation Act, 1996. This
petition was opposed by the plaintiff appellants. The trial Court dismissed the petition by
the order dated 17.3.2005 mainly on the ground that as Shri Rajendra Prasad Singh
(predecessor-in-interest of the plaintiffs) was not a party to the partnership deed which
was executed on 17.2.1992, and as the main relief sought in the suit was that the said
partnership deed dated 17.2.1992 was illegal and void, which question could only be
decided by the civil Court, the dispute could not be referred to arbitration. The defendant
No.3 challenged the aforesaid order by filing a Civil Revision Petition which was allowed
by the High Court by the impugned order dated 4.8.2005. The operative portion of the
order passed by the High Court only says that the Court below has committed error in
passing the impugned order. Accordingly, the impugned order is set aside and this civil
revision is allowed. No specific order making reference to arbitration was passed.
7. Shri Ranjit Kumar, learned senior counsel for the appellants, has submitted that the
main relief claimed in the suit is that a declaration be made that the reconstituted
partnership deed dated 17.2.1992 was illegal, void and without jurisdiction as there was
no intention or desire on the part of Shri Rajendra Prasad Singh to retire from the
partnership and that the plaintiffs being heirs of Shri Rajendra Prasad Singh, shall be
deemed to be continuing as partners to the extent of his share. The other relief regarding
rendering of accounts of all transactions from 1.4.1992 onwards was dependent upon the
first relief inasmuch as Shri Rajendra Prasad Singh or the plaintiffs were admittedly not
shown as partners of the firm in the deed dated 17.2.1992 and unless the said document
was declared as void, they could not claim any rights on the basis of earlier deed dated
13.1.1989. Learned counsel has submitted that Shri Rajendra Prasad Singh or the
plaintiffs
@page-SC1019
being not parties to the deed dated 17.2.1992, Section 8 of the 1996 Act can have no
application to the facts and circumstances of the case and the High Court committed
manifest error of law in setting aside the order of the trial Court and allowing the revision
petition filed by defendant No.3. Learned counsel has also submitted that having regard
to the facts of the case, the relief of declaration that the partnership deed is illegal or void
or the relief of cancellation thereof can only be granted by the Civil Court and not by an
arbitrator. In support of his submission Shri Ranjit Kumar has placed reliance on the
following observations made in Khardah Company Ltd. v. Raymon and Company
(India) Pvt. Ltd., AIR 1962 SC 1810 :
It cannot be disputed that the expression arising out of or concerning or in connection
with or in consequence of or relating to this contract occurring in an arbitration clause in
an agreement to purchase goods are of sufficient amplitude to take in a dispute as to the
validity of the agreement. But the arbitration clause cannot be enforced when the
agreement of which it forms an integral part is held to be illegal. On principle it must be
held that when an agreement is invalid every part of it including the clause as to
arbitration contained therein must also be invalid. (1942) AC 356 and AIR 1959 SC 1362
and ILR (1948) 2 Cal 171 and AIR 1954 Mad 528(531), Rel. on; AIR 1952 SC 119, Ref.
(Para 4).
Accordingly, a dispute that the contract of which the arbitration clause forms an integral
part is illegal and void is not one which the arbitrators are competent to decide under the
arbitration clause although it is of sufficient amplitude to take in a dispute as to the
validity of the agreement and in consequence a party to the contract is entitled to
maintain an application under S. 33 for a declaration that the contract is illegal and that in
consequence the proceedings taken thereunder before the arbitrators and the award in
which they resulted were all void : AIR 1959 SC 1357, Rel. on. (para 13)
Learned counsel has also submitted that the suit was filed on 1.8.1998 and defendant
No.2 having failed in his attempt to get the matter referred to arbitration, his son Sunil
Kumar Singh (defendant No.3) who was set ex parte on 28.6.1999 moved the application
for staying the suit under Section 34 of Arbitration Act, 1940 on 5.11.2004 and then
moved the application giving rise to the order under challenge on 28.2.2005 and such an
application having been moved after an inordinate delay, it was wholly improper on the
part of the High Court to have accepted his prayer. It has been further urged that there
was non-compliance of sub-section (2) of Section 8 of the 1996 Act as the application
moved by defendant No.3 was not accompanied by the original arbitration agreement or a
duly certified copy thereof and, therefore, the same ought to have been rejected. Shri S.B.
Sanyal, learned senior counsel for the respondent, has submitted that the plaintiffs in fact
are claiming rendition of accounts and their share in the partnership business for which
they are basing their claim on the partnership deed dated 13.1.1989 to which Shri
Rajendra Prasad Singh was a party and the said deed contains an arbitration clause. In
such circumstances, the High Court rightly referred the dispute for arbitration and the
contention raised by learned counsel for the plaintiffs has no substance.
8. In order to appreciate the contention raised by learned counsel for the parties, it will be
convenient to set out Sections 7 and 8 of the 1996 Act:
7. Arbitration agreement.- (1) In this Part, 'arbitration agreement' means an agreement by
the parties to submit to arbitration all or certain disputes which have arisen or which may
arise between them in respect of a defined legal relationship, whether contractual or not.
(2) An arbitration agreement may be in the form of an arbitration clause in a contract or
in the form of a separate agreement.
(3) An arbitration agreement shall be in writing.
(4) An arbitration agreement is in writing if it is contained in-
(a) a document signed by the parties;
(b) an exchange of letters, telex, telegrams or other means of telecommunication which
provide a record of the agreement; or
(c) an exchange of statements of claim and defence in which the existence of the
agreement is alleged by one party and not denied by the other.
(5) The reference in a contract to a document containing an arbitration clause constitutes
an arbitration agreement if the contract is in writing and the reference is such
@page-SC1020
as to make that arbitration clause part of the contract.
8. Power to refer parties to arbitration where there is an arbitration agreement.- (1) A
judicial authority before which an action is brought in a matter which is the subject of an
arbitration agreement shall, if a party so applies not later than when submitting his first
statement on the substance of the dispute, refer the parties to arbitration.
(2) The application referred to in sub-section (1) shall not be entertained unless it is
accompanied by the original arbitration agreement or a duly certified copy thereof.
(3) Notwithstanding that an application has been made under sub- section (1) and that the
issue is pending before the judicial authority, an arbitration may be commenced or
continued and an arbitral award made.
Sub-section (1) of Section 8 of the 1996 Act says that a judicial authority before which an
action is brought in a matter which is the subject of an arbitration agreement shall, if a
party so applies not later than when submitting his first statement on the substance of the
dispute, refer the parties to arbitration. Therefore, for application of Section 8, it is
absolutely essential that there should be an arbitration agreement between the parties. It is
an admitted fact that neither Shri Rajendra Prasad Singh nor the plaintiffs are parties to
the partnership deed dated 17.2.1992. There is no document as defined in Section 7 of
1996 Act which may contain the signature of either Shri Rajendra Prasad Singh or the
plaintiffs. Similarly, there is no document as contemplated by clause (b) or (c) of sub-
section (4) of Section 7 of 1996 Act from which it may be spelled out that either Rajendra
Prasad Singh or the plaintiffs were parties to clause relating to arbitration contained in the
partnership deed dated 17.2.1992. It is also an admitted fact that Shri Rajendra Prasad
Singh was alive when the said partnership deed dated 17.2.1992 was executed. Therefore,
on the face of it Section 8 of 1996 Act would not apply to any dispute concerning the said
partnership deed dated 17.2.1992 and the matter cannot be referred to arbitration.
9. The first relief claimed by the plaintiffs in the suit is a decree for declaration that the
reconstituted partnership deed dated 17.2.1992 was illegal and void and there was no
intention or desire of Shri Rajendra Prasad Singh to retire from the partnership and
further that the plaintiffs being heirs of Shri Rajendra Prasad Singh will be deemed to be
continuing as partners to the extent of his share. It is true that the plaintiffs have also
sought rendition of accounts and their share of profits from the partnership as well as
interest over the unsecured loan and the principal amount of unsecured loan on rendition
of accounts. For getting this relief, the plaintiffs undoubtedly rely upon the partnership
deed dated 13.1.1989. However, this deed of 1989 could be relied upon and form the
basis of the claim of the plaintiffs only if the partnership deed dated 17.2.1992 was
declared as void. If the deed dated 17.2.1992 was not declared as void and remained valid
and operative, the plaintiffs could not fall back upon the earlier partnership deed dated
13.1.1989 to claim rendition of accounts and their share of profits. Therefore, in order to
get their share of profits from the partnership business, it was absolutely essential for the
plaintiff appellants to have the partnership deed dated 17.2.1992 declared as illegal, void
and inoperative. The relief for such a declaration could only be granted by the civil Court
and not by an arbitrator as they or Shri Rajendra Prasad Singh through whom the
plaintiffs derive title, are not party to the said deed. The trial Court had, therefore, rightly
held that the matter could not be referred to arbitration and the view to the contrary taken
by the High Court is clearly illegal.
10. Sub-section (2) of Section 8 of the 1996 Act says that the application referred to in
sub-section (1) shall not be entertained unless it is accompanied by the original
arbitration agreement or a duly certified copy thereof. As already stated in the earlier part
of the judgment, defendant No.3 had moved an application on 25.11.2004 under Section
34 of the Arbitration Act, 1940 for staying the proceedings of the title suit and for
referring the matter to arbitration. He filed a supplementary petition to the aforesaid
application on 16.12.2004. Herein also reference was made to Section 34 of Arbitration
Act, 1940. Thereafter, he filed an application on 28.2.2005 praying that as the Arbitration
Act, 1940 had been repealed and the suit is of 1998, to avoid any confusion, his earlier
petitions may be treated to have been filed under Section 8 of the Arbitration Act, 1996.
None of these petitions were
@page-SC1021
accompanied by the original arbitration agreement dated 17.2.1992 or a duly certified
copy thereof. In fact, there is no requirement of filing the original arbitration agreement
or a duly certified copy thereof under Section 34 of Arbitration Act, 1940 and as such
there was no occasion for defendant No.3 to file the aforesaid document. The third
petition filed on 28.2.2005 contained the following prayer:
"It is, therefore, prayed that your honour may graciously be pleased to treat the petitions
dated 25.11.04, 16.12.04 and the present petition as supplement and part of each other for
deciding the prayer with regard to stay of the proceedings of the aforesaid suit and/or to
refer to arbitration in view of the arbitration agreement covering the subject matter of this
suit."
There is no whisper in the petition dated 28.2.2005 that the original arbitration agreement
or a duly certified copy thereof is being filed along with the application. Therefore, there
was a clear non-compliance of sub-section (2) of Section 8 of 1996 Act which is a
mandatory provision and the dispute could not have been referred to arbitration. Learned
counsel for the respondent has submitted that a copy of the partnership deed was on the
record of the case. However, in order to satisfy the requirement of sub-section (2) of
Section 8 of the Act, defendant No.3 should have filed the original arbitration agreement
or a duly certified copy thereof along with the petition filed by him on 28.2.2005, which
he did not do. Therefore, no order for referring the dispute to arbitration could have been
passed in the suit.
11. In view of the discussions made above, the appeal is allowed with costs and the
impugned order dated 4.8.2005 passed by the High Court in Civil Revision No.1010 of
2005 is set aside.
Appeal allowed.
AIR 2008 SUPREME COURT 1021 "Mani v. State of Tamil Nadu"
(From : 2003 Mad LJ (Cri) 906)
Coram : 2 PRAKASH PRABHAKAR NAOLEKAR AND V. S. SIRPURKAR, JJ.
Criminal Appeal No. 443 of 2006, D/- 8 -1 -2008.
Mani v. State of T.N.
(A) Penal Code (45 of 1860), S.300 - MURDER - EVIDENCE - Murder - Circumstantial
evidence - No evidence to prove that, house where deceased was murdered belonged to
accused - Discovery of articles found to be farce - All discovered articles were lying open
barely 300 feet away from body of deceased - Such articles discovered after more than 10
days of incident - No attempt made by prosecution to prove that discovered clothes
belonged to accused - No motive for murder - Evidence of prosecution witness who had
found corpse at night did not report the matter till 10 o'clock of next day morning - Held
that, this was clear case of benefit of doubt.
2003 Mad LJ (Cri) 906, Reversed. (Paras 16, 20, 21, 25)
(B) Penal Code (45 of 1860), S.300 - MURDER - EVIDENCE - Murder - Recovery of
articles on disclosure by accused - It is weak kind of evidence - Conviction in serious
matter cannot be based on discovery - Discovery of relevant articles made from open
ground after more than 10 days of incident and about 300 feet away from dead body -
Fact that Inspector did not search nearby spots and that all articles remained in open,
unguarded for more than 10 days - Unbelievable - Discovery, held, was farcicle and could
not be accepted.
Evidence Act (1 of 1872), S.27.
2003 Mad LJ (Cri) 906, Reversed. (Paras 20, 21)

Prasanthi Prasad, for Appellant; V. Krish-namoorthy, Sr. Advocate, V. G. Pragasam and S.


Prabu Ramasubramanian, for Respondent.
Judgement
1. V. S. SIRPURKAR, J.:- This appeal challenges the judgment of the High Court
whereby the conviction of the appellant-Mani for an offence under Section302, Indian
Penal Code, 1860 read with Section 34 as also the conviction under Section 201, I.P.C.
have been confirmed by the High Court.
2. Originally, there were two accused persons, however, accused No.2 Moyyasamy has
been acquitted by the High Court. State has not challenged the acquittal of accused
Moyyasamy. We have to therefore consider only whether the High Court was justified in
confirming the conviction of the appellant for the offences stated above.
3. The prosecution case was that the two accused persons had murdered one
@page-SC1022
deceased Sivakumar who had strained relationship with Accused No. 2 Moyyasamy. The
relationship was strained on account of the cattle belonging to the father of the deceased
namely P.W.1 Arunachalam having grazed in the field belonging to accused Moyyasamy.
In fact, both P.W.1 Aruna-chalam and accused Moyyasamy are the real brothers while
deceased Sivakumar and P.W.2 Amulnathan were the sons of Arunachalam. On account
of the cattle trespass into the field of A.2 Moyyasamy, there was a quarrel. This trespass
incident, seems to have taken place on 19.9.1996. Since there was an altercation between
accused Moyyasamy and his brother in which accused Moyyasamy had stabbed on the
neck of Arunachalam, he (Arunachalam) had reported the matter to the police.
4. The occurrence in question, according to the prosecution, took place about two months
thereafter on 24.11.1996. It seems that according to the prosecution, P.W.1 Arunachalam
and deceased Sivakumar were chatting with the family members when accused No. 1-
Mani came there and had a cup of coffee with P.W.1 Arunachalam and his family. Mani is
said to have taken Sivakumar to his house. This was at about 6.00 p.m. Since Mani did
not turn up till 10.00 p.m., P.W.1 went to the house of the appellant Mani and found that
blood was oozing from the house through the water passage underneath the door frame.
The house was found locked. P.W.1 Arunachalam therefore came back and he was
informed by P.W.2 Amulnathan that Sivakumar was found in the company of appellant
Mani and acquitted accused Moyyasamy at about 7.00 p.m. Though P.W.1 Arunachalam
was disturbed on account of this, he did nothing and only on the next morning at 6 O'
clock he went to the house of the accused and found trail of blood near the house of Mani
and ultimately from that he traced out the body of the deceased which was lying down in
the nearby field belonging to one P.W.5 Chinnammal. It is then that P.W.1 Moyyasamy
rushed to Yercaud police station and gave a complaint. The case was registered under
Section 302, I.P.C.
5. During investigation which was done on 25.11.1996 the house of appellant which was
locked was broken open by the police and the house was searched. Some blood-stained
materials like cement flooring, human hair and blood-stained brass lock were seized by
the investigating officer. It was found in the post-mortem examination that the deceased
had suffered six injuries on various parts of the body including the neck and that he had
died due to the injuries to the vital organs and also due to the hemorrhage. The seized
material was sent for the chemical analysis.
6. The investigating officer came to know that accused appellant had surrendered before
the Court. He, therefore, filed an application for the police custody on 28.11.1996 and
ultimately obtained the same on 06.12.1996. The prosecution further contended that on
the same day on the basis of the discovery statement made by the accused appellant,
blood-stained clothes of the accused namely M.Os. 15-17 and blood-stained Koduval
namely M.O. 18 were recovered from the place pointed out by the accused. The second
accused (Moyyasamy) was arrested on 08.12.1996 and his blood-stained clothes were
also recovered. On the basis of the investigation, charge-sheet came to be filed.
7. As many as 15 witnesses were examined. The accused had denied their guilt. The Trial
Court concluded that the prosecution established its case beyond reasonable doubt and
convicted both the accused of the offences as aforesaid.
8. During the appeal, the High Court came to the conclusion that there was no evidence
worth the name against accused No. 2 Moyyasamy. The High Court further held that the
motive attributed by the prosecution to Moyyasamy was not established. Thus, the High
Court held on the basis of the fact that the prosecution had not placed any material to
show that subsequent to 19.9.1996, any incident took place so as to provide motive for
murder of the deceased. The High Court also held that there was no evidence led in to
show that appellant Mani and accused No. 2 Moyyasamy were the close associates and
that accused Moyyasamy had set up the present appellant to commit the murder of the
deceased. Thus, the High Court held that there was nothing to link the accused
Moyyasamy with the crime particularly because there could be no motive suggested to it.
The High Court also disbelieved the discovery made by accused Moyyasamy of the
blood-stained clothes on the ground that the discovery was belated. For these reasons,
accused Moyyasamy was acquitted. However, the
@page-SC1023
High Court dismissed the appeal in so far as the present appellant is concerned.
9. Challenging the judgment of the High Court, learned counsel points out that firstly
there could be no motive whatsoever on the part of the appellant. Our attention was
invited to the finding that there was no thick relationship between the two accused
persons so that the present appellant could take up the cause of the acquitted accused
Moyyasamy and go to the extent of murdering the deceased. It is further pointed out that
if at all, there was any motive, it could be on the part of the acquitted accused since it was
he who had fought on 19.9.1996 with the father of the deceased and it was he who was
reported against by P.W.1 Arunachalam. Learned Counsel was at pains to point out that
the appellant had nothing to do either in the first incident of the cattle straying into the
complainant's field or the subsequent altercation between Moyyasamy and P.W.1
Arunachalam wherein P.W.1 Arunachalam was allegedly injured by accused Moyyasamy.
Learned counsel further pointed out that the theory of last seen together if at all was to be
believed, then it applied with equal force to the second accused also. At any rate, the
theory was unworkable as the exact time of death was not established by the medical
evidence. Learned counsel argued that at any rate, this could not be a clinching
circumstance against the appellant particularly when the second accused was acquitted by
the High Court.
10. The counsel further urged that the High Court erred in relying upon the discovery
evidence as admittedly the so-called discovered articles were found from the open ground
barely 300 feet away from the body and that it was nothing but a farce. Learned counsel
pointed out that the investigating officer was bound to know about the so-called blood-
stained clothes (Koduval) as those articles were lying barely within three hundred feet
from the body. Lastly, learned counsel urged that the evidence of discovery could not be
held to be sufficient and could not be relied upon to convict the appellant. Learned
counsel also urged that the prosecution had miserably failed to establish that the alleged
house which was visited by P.W.1 Arunachalam at about 10 O'clock at night actually
belonged to and was possessed by the appellant. The learned counsel also pointed out that
though P.W.1 Arunachalam had found the body at night in the field of P.W.5, Chinnamal
he did not even bother to report the matter to the police and in fact the First Information
Report was hopelessly delayed. He pointed out that this delay was also not considered by
the High Court.
11. As against this, learned senior counsel appearing for the State supported the
prosecution and tried to point out that though this was a case of circumstantial evidence,
the prosecution has not only established each circumstance but had also discharged its
burden to show that the circumstances pointed out only to the guilt of the appellant and
also suggested that he alone and nobody else was responsible for the murder.
12. We have closely examined the evidence as also the original records of the matter and
we are convinced that the prosecution has not been able to establish the guilt of the
appellant beyond reasonable doubt and that the High Court as well as the Trial Court
have erred in convicting the accused. Our reasons are as follows :
13. There can be no dispute that deceased Sivakumar had died homicidal death. The
question is whether the prosecution has been able to connect the present appellant with
the crime? The case undoubtedly depends upon circumstantial evidence. It will be
therefore our task to examine all the circumstances relied upon by the courts below.
14. Though the Trial Court has discussed nine circumstances, the High Court has not
discussed all the circumstances in seriatum.
15. The first circumstance held by the High Court against the appellant is that the
Koduval (M.O.18) was found with sticking hair and it is proved that the scalp hair of the
deceased was found similar to the hair sticking to the Koduval and that M.Os. 15, 16 and
17 were the clothes found with stained blood. In our opinion, this circumstance by itself
is of no consequence unless those material objects are connected with appellant.
16. An interesting statement was made by the High Court suggesting that if the appellant
took the deceased at 6.00 p.m. on 24.11.1996 to his house where the deceased was done
away with, the burden shifted on the first accused to show how the deceased died in his
house. In our opinion, this is not
@page-SC1024
the correct position of law. In order to hold this circumstance, the High Court has
recorded the finding that the house belonged to the present appellant. The appellant had
very clearly stated in his examination under Section 313, Cr.P.C. that the house did not
belong to his father and that it was lying vacant and nobody had occupied it. In our
opinion, at least from the evidence on record, it cannot be concluded that the house
belonged to the appellant. There is no evidence worth the name led by the prosecution to
suggest the exclusive ownership or the possession of the house belonged to the appellant.
Both the courts have proceeded on the presumption that the house was owned or
possessed exclusively by the appellant. Much could have been done to establish its
ownership by filing the revenue record of that house. No such documentary evidence was
collected by the prosecution. The High Court has not discussed this aspect of exclusive
ownership and possession at all and has proceeded on the presumption that the house
belonged to and was possessed by the appellant herein.
17. The Panch witness P.W.6 Ganesan, though had referred to the said house as the house
of the appellant, has clearly admitted in his cross-examination that he did not know as to
in whose name stood the said house. It is very significant to note that he has lastly given
the admission to the effect "to say that (blood-stained) that house is not Mani's house and
it was built by Mani's father, cannot be objected". This witness was a Village
Administrative Officer through whom the investigating officer could have easily obtained
the records of this house. Unfortunately, that was not done.
18. The only other evidence in this behalf is that of PW-14 Karunakaran who was one of
the Investigating Officer. He has never asserted that the concerned house was appellant's
house though he, in his examination-in-chief referred to that house as Mani's house. He
had to admit in his cross-examination that he did not interrogate any other residents
residing near Andiammal's house. He also had admitted that he had never questioned the
Village Administrative Officer as to in whose name was the said house. Though this
witness commonly referred to that house as Mani's house, it is difficult to hold that the
prosecution had established the exclusive ownership and possession of that house as
against the appellant.
19. The High Court has undoubtedly referred to the written statement filed by the accused
under Section 313, Cr.P.C. but has chosen not to discuss the matter further. It is therefore
difficult to hold that the said house where allegedly the blood-stains were found belonged
to the appellant.
20. Same and similar is the story regarding the alleged discovery. Both the witnesses
namely P.W. 8 Dilip Kumar and P.W.9 Loganathan had turned hostile. They completely
disowned the prosecution case that the appellant was examined by the Inspector and
during that examination, Mani was enquired and he gave confessional statement and that
he would show the hidden clothes which he worn at the time of the incident and also the
koduval which was used at the time of the incident. They only accepted that they had
signed the said statement. Now, when we consider the evidence regarding the discovery, a
very important circumstance was missed by both the Trial Court as well as by the High
Court about the place where the articles were found. P.W. 15 Govindan who was the
Investigating Officer in respect of this discovery deposed that the accused Mani had made
voluntary confession statement which he had recorded in presence of the two witnesses
namely P.W. 8-Dilip Kumar and Loganathan. We are already seen that both the witnesses
had completely disowned the stouts There is no cross-examination worth the name to
these witnesses by the Public Prosecutor. Both these witnesses were chance witnesses,
they being the regular visitors to the police station, and it appears that they were merely
used by the investigating agency. In fact, there is no explanation on how these two
witnesses reached the police station at all. P.W. 15 Govindan did assert that he recovered
one blood-stained rose colour full sleeve shirt, blood-stained green coloured sweater,
blood-stained lungi having green, red and black stripes and one blood-stained Koduval
with human hair produced by the accused at 11.30 a.m. near the sloppy rock on the way
to Chinnammal Coffee Estate in Salaipara village. Accordingly, the witness asserts that
he recovered M.Os. 15, 16, 17 and 18 under Exhibit P.26 Mahazar. Now, unfortunately,
the High Court has missed a very important of evidence in his cross- examination where
the witness very specifically admitted that the material objects said to
@page-SC1025
have been produced by Accused No. 1 and Accused No. 2 were recovered about 300 feet
away from the dead body of Sivakumar. Now, it is nobody's case that at the time the
discovery was made by accused No. 1, accused No. 2 also made certain discoveries.
Therefore, the witness was not certain as to who made the discovery. This is apart from
the fact that discovery admittedly was made from 300 feet away from the dead body of
Sivakumar and after Sivakumar's body was inspected by P.W.14 as early as on
25.11.1996. It would be impossible to believe that the Inspector did not search the nearby
Spots and that all the articles would remain in open unguarded till 6th December, 1996
when the discovery had allegedly been made. This was nothing but a farce of discovery
and could never have been accepted particularly because all the discovered articles were
lying bare open barely 300 feet away from the body of the deceased Sivakumar. Even this
witness had to admit that he never enquired as to in whose name the house of Mani stand.
He claims that P.W.14 had done the same whereas P.W.14 is completely silent about such
investigation. It is, therefore, obvious this discovery could have never been accepted by
both the courts below and both the courts have completely ignored this vital admission. It
need not be stated that where the discovery of the relevant articles have been made from
the open ground though under the bush, that too after more than 10 days of the incident,
such discovery would be without any credence. It does not stand to any reasons that the
concerned investigating officer did not even bother to look hither and thither when the
dead body was found. We are, therefore, not prepared to accept such kind of farcical
discovery which has been relied by the courts below without even taking into
consideration the vital facts which we have shown above.
21. The discovery is a weak kind of evidence and cannot be wholly relied upon and
conviction in such a serious matter cannot be based upon the discovery. Once the
discovery fails, there would be literally nothing which would support the prosecution
case. We have already held that the prosecution has failed to prove that the house where
alleged blood-stains were found belonged exclusively or was possessed exclusively by
the appellant, we have further pointed out that the discovery was absolutely farcical.
There is one other very relevant factor ignored by both the courts that the prosecution
never made any attempts to prove that the clothes belonged to the appellants. There is
literally no evidence to suggest anything to that effect. Therefore, even if we accept the
discovery, it does not take us anywhere near the crime. Both the Courts below have
ignored this very important aspect. Once these two important circumstances are
disbelieved, there is nothing which would remain to support the prosecution theory. We
also fail to understand the finding of the High Court in respect of the motive. In our
opinion, there was no motive whatsoever much less entertainable by the present
appellant. He had nothing to do with the straying cattle nor was he a party to subsequent
altercation between P.W.1 Arunachalam and the accused No. 2 Moyyasamy. Lastly, there
is nothing on record to show that he was a henchman set up to take revenge by accused
No. 2 Moyyasamy and he was set up by the accused No. 1 to revenge. We also did not
understand that if there was no motive for Moyyasamy, how could there be any motive
entertainable by the appellant. Therefore, even for that circumstance has to go.
22. Even if we accept that the Koduval and the alleged clothes, i.e. the material object
Nos. 15-18 had the blood stains that does not connect the appellant with the crime.
23. The only other circumstance left with is that Mani called Sivakumar at 6 O'clock and
took him away in the absence of any other clinching circumstances, this circumstance by
itself cannot lead to the only conclusion that Mani murdered Sivakumar. Therefore, we
have to ignore that circumstance.
24. Lastly, the Trial Court has obviously committed an error in suggesting that the
appellant was absconding and that he was surrendered before the Court on 06.12.1996. It
is mentioned in sub-para 6 of the judgment of the Trial Court of para 28. We are afraid
this is a factual mistake because it has come in the evidence of the prosecution witnesses
that the accused had surrendered before the Magistrate much earlier to that , i.e. on the
very next day or so and that it took the investigating officer almost 10 days to obtain his
custody. That has clearly come in the evidence of P.W.14.
25. We are also not impressed by the evidence of P.W.1 Arunachalam who had
@page-SC1026
though found the corpse at night or at least had realized that something unusual had
happened, did not bother to go till next day at 10 O'clock for reporting the matter. We
cannot ignore the evidence of P.W.5 Chinamal in whose field the body was found. She
had specifically claimed that this fact was known to P.W. 1 Arunachalam. Having seen all
the evidence and having considered both the judgments very carefully, we are of the clear
opinion that this cannot be a case where the prosecution has proved the guilt of the
accused beyond reasonable doubts. We would therefore accept the plea of the accused
raised by the defence counsel that this is clearly a case for benefit of doubt. We would
therefore chose to grant the benefit of doubt to the accused holding that the prosecution
has not been able to prove its case beyond the reasonable doubt. We therefore, allow the
appeal. Accordingly the appeal is allowed. Impugned Judgments and conviction are set
aside.
The accused be set to liberty forthwith unless required in any other case.
Appeal allowed.
AIR 2008 SUPREME COURT 1026 "Union of India v. Dineshan K. K."
(From : Gauhati)
Coram : 2 C. K. THAKKER AND D. K. JAIN, JJ.
Civil Appeal No. 25 of 2008 (arising out of SLP (C) No. 21222 of 2005), D/- 4 -1 -2008.
Union of India and Ors. v. Dineshan K. K.
(A) Constitution of India, Art.16, Art.39(d), Art.226 - EQUALITY IN PUBLIC
EMPLOYMENT - DIRECTIVE PRINCIPLES - WRITS - PAY PARITY - Pay parity -
Claim for - Generally to be left to executive and expert bodies - Jurisdiction of Court is
however not altogether outsted.
It is true that the principle of equal pay for equal work had no mathematical application in
every case of similar work. Equation of posts and equation of pay structure being
complex matters are to be generally left to the Executive and expert bodies like the Pay
Commission etc. Nevertheless, it will not be correct to lay down as an absolute rule that
merely because determination and granting of pay scales is the prerogative of the
Executive, the Court has no jurisdiction to examine any pay structure and an aggrieved
employee has no remedy if he is unjustly treated by arbitrary State action or inaction,
except to go on knocking at the doors of the Executive or the Legislature. Undoubtedly,
when there is no dispute with regard to the qualifications, duties and responsibilities of
the persons holding identical posts or ranks but they are treated differently merely
because they belong to different departments or the basis for classification of posts is ex
facie irrational, arbitrary or unjust, it is open to the Court to intervene. (Para 10)
(B) Constitution of India, Art.16, Art.39(d) - EQUALITY IN PUBLIC EMPLOYMENT -
DIRECTIVE PRINCIPLES - PAY PARITY - PAY COMMISSION - Pay parity - Radio
mechanics in Assam Rifles claiming parity with their counterparts in CRPF and BSF - No
dissimilarity between two as to academic qualification, duties or responsibility - Disparity
caused because of anomaly in pre-revised and revised scales of personnel of Assam
Rifles prior to the recommendations of the Fourth Pay Commission - Decision of Govt. to
perpetuate disparity is clearly irrational and arbitrary. (Paras 18, 20)
Cases Referred : Chronological Paras
2007 AIR SCW 5480 : AIR 2007 SC 3021 : 2007 Lab IC 4043 7
2007 AIR SCW 7025 7
2005 AIR SCW 5632 : AIR 2006 ScC 161 : 2005 Lab IC 4322 13
(2004) 1 SCC 347 (Rel. on, Pnt. A) 13
2003 AIR SCW 2513 : 2003 Lab IC 2038 (Rel. on, Pnt A) 13
2003 AIR SCW 3382 : AIR 2003 SC 2658 (Ref.) 12, 13
2002 AIR SCW 1881 : AIR 2002 SC 1955 : 2002 Lab IC 1682 (Ref.) 11
2002 AIR SCW 2896 : AIR 2002 SC 2589 : 2002 Lab IC 2630 (Ref.) 10
1997 AIR SCW 157 : AIR 1997 SC 1788 13
1992 AIR SCW 1071 : AIR 1992 SC 1203 : 1992 Lab IC 1047 (Ref.) 10
AIR 1983 SC 130 : 1983 Lab IC 1 (Ref.) 9
AIR 1982 SC 879 : 1982 Lab IC 806 (Ref.) 9
B. Dutta, ASG, Ms. Sunita Sharma and M. Sushma Suri, for Appellants; Ranjit Kumar,
Sr. Advocate, K. V. Vishwanathan, Hiren Dasan, Kalyan Bhaumik, Dhirendra Kumar
Misra and Mrs. Sarla Chandra, for Respondent.
Judgement
D. K. JAIN, J. :- Delay condoned.
2. Leave granted.
3. This appeal by the Union of India and the Director General of Assam Rifles arises out
of the judgment and order dated 11th
@page-SC1027
February, 2005 rendered by the Gauhati High Court in WP (C) No. 497 of 2001. By the
impugned order, while allowing the writ petition, directions have been issued that the
permission granted by the Union of India vide its letter dated 3rd March, 1998, to re-
designate the rank of Havildar (Radio Mechanic) as Warrant Officer as recommended by
the Ministry of Home Affairs shall be carried out and the pay scale as admissible to their
counterparts in the Central Reserve Police Force (CRPF) and the Border Security Force
(BSF) shall be granted from the same date.
4. The nub of the grievance of the writ petitioner, working in the rank of a Radio
Mechanic in the Assam Rifles was that the Ministry of Home Affairs and the Director
General of Assam Rifles having accepted in principle that the members of the Assam
Rifles, should be given the same rank and pay structure as was given to other central
paramilitary forces, yet the same had been denied to them. It was pleaded that as the
Ministry of Home Affairs had conveyed its decision to rationalize the rank structure of
non-gazetted personnel of central paramilitary forces vide order dated 26th January,
1998, equal pay structure in other ranks, including the Radio Mechanics in the Assam
Rifles could not be denied. His further grievance was that after the implementation of the
Fourth Pay Commission, the pay of the Havildar/GD and Head Constable/Radio
Mechanic was fixed in the pay scale of Rs. 975-1660, without any discrimination
between the general duty and technical categories but the discrimination surfaced when
higher pay scale of Rs.1200-2040 was given to the Radio Mechanics working in the BSF,
denying the same pay scale to the Radio Mechanics in the Assam Rifles. It was also
pointed out that the Radio Mechanics working in the Delhi Police organization had been
given a much higher pay scale on 10th October, 1997 which was being denied to the
similar rank holders in the Assam Rifles.
5. The writ petition was contested by the Union of India. In the counter-affidavit filed on
its behalf, it was stated that on the recommendation of the Fourth Pay Commission, with
effect from 1st January, 1986, the Assam Rifles personnel had been granted revised pay
scales and allowances entirely on the lines of other central paramilitary forces. However,
as the changes in the rank structure were not carried out in the Assam Rifles like in other
central paramilitary forces, an "apparent disparity" in the service conditions of certain
category of personnel including the rank of Radio Mechanic had arisen. It was also
pointed out that the Assam Rifles Directorate had brought this disparity to the notice of
the Ministry of Home Affairs in February, 1998, and had recommended the re-
designation of Radio Mechanic and Head Constable in Assam Rifles as Warrant Officer
and for replacement of pay scale of Rs.4000-6000 to bring them at par with their
counterparts in other central police organizations. It was stated that in response to the said
recommendation, the Ministry of Home Affairs vide letter dated 3rd March, 1998, had
informed the Assam Rifles that they could re-designate the Head Constable (Radio
Mechanic) as Warrant Officer provided their pre-revised and revised pay scales were
identical to the pay scales of their counterparts in CRPF and BSF. However, the re-
designation of the ranks could not be carried out in the light of the said communication as
there was disparity between the pay scales of a Radio Mechanic in Assam Rifles and their
counterparts in CRPF and BSF. It was conceded that though the academic qualification
for recruitment to the post of Radio Mechanic in Assam Rifles as well as in CRPF and
BSF was the same yet there was disparity in the revised pay scales between the Assam
Rifles and the said two other paramilitary forces. The claim of the petitioner for higher
pay scales on the lines of the pay scales of Delhi Police organization was seriously
contested on the ground that the Assam Rifles being a central police organisation, it could
not claim parity with Delhi Police organization, which was not a central paramilitary
force.
6. Taking note of the admission on the part of the Union of India that there was disparity
between the pay scales of the members of the Assam Rifles and similarly ranked
personnel of other paramilitary forces, the High Court felt that it would be unreasonable
and discriminatory if the pay scales given to Radio Mechanics in CRPF and BSF were
denied to the Radio Mechanics in Assam Rifles, when the qualifications and service
requirements in all the three organizations were identical. Consequently, the High Court
issued the aforenoted directions, which are questioned in this appeal.
7

. Mr. B. Dutta, learned Additional Solicitor General, appearing for the Union of 2007
AIR SCW 5480
2007 AIR SCW 7025

@page-SC1028
India contended that the direction given by the High Court is manifestly contrary to the
settled legal position, enunciated by this Court in several decisions that pay fixation is
essentially an executive function, ordinarily undertaken by an expert body like the Pay
Commission, whose recommendations are entitled to a great weight though not binding
on the Government. It was argued that the recommendations of an expert body are not
justiciable since the Court is not equipped to take upon itself the task of job evaluation,
which is a complex exercise. In support of the proposition, reliance is placed on two
decisions of this Court in S. C. Chandra and Ors.1v. State of Jharkhand and Ors. and
Union of India and Ors. v. Hiranmoy Sen and Ors2.
1 (2007) 9 SCR 130
2 2007 (12) Scale 170
8. Mr. Ranjit Kumar, learned senior counsel, appearing on behalf of the respondent, on
the other hand, submitted that the petitioners having themselves admitted that there was
an anomaly in the pay scales of the personnel of Assam Rifles, particularly, the Radio
Mechanics, as compared to their counterparts in other paramilitary forces, the High Court
was fully justified in giving the impugned directions. It was pointed out that, in fact, the
Director General, Assam Rifles, who is one of the petitioners in the present appeal, had
himself recommended to the Ministry of Home Affairs that the anomaly in the pay scales
of the Radio Mechanics should be rectified. Learned counsel submits that it is unfair on
the part of the Director General to take a somersault and oppose the direction given by
the High Court which is in consonance with his recommendation. Learned counsel,
however, stated that the respondent was not pressing for parity with the personnel of the
Delhi Police.
9

. The principle of 'equal pay for equal work' has been considered, explained and applied
in a catena of decisions of this Court. The doctrine of 'equal pay for equal work' was
originally propounded as part of the Directive Principles of the State Policy in Article
39(d) of the Constitution. In Randhir Singh v. Union of India and Ors.3a bench of three
learned Judges of this Court had observed that principle of equal pay for equal work is
not a mere demagogic slogan but a constitutional goal, capable of being attained through
constitutional remedies and held that this principle had to be read under Articles 14 and
16 of the Constitution. This decision was affirmed by a Constitution Bench of this Court
in D. S. Nakara and Ors. v. Union of India4. Thus, having regard to the constitutional
mandate of equality and inhibition against discrimination in Articles 14 and 16, in service
jurisprudence, the doctrine of equal pay for equal work has assumed status of a
fundamental right. AIR 1982 SC 879
AIR 1983 SC 130

3 (1982) 1 SCC 618


4 (1983) 1 SCC 305
10

. Initially, particularly in the early eighties, the said principle was being applied as an
absolute rule but realizing its cascading effect on other cadres, in subsequent decisions of
this Court, a note of caution was sounded that the principle of equal pay for equal work
had no mathematical application in every case of similar work. It has been observed that
equation of posts and equation of pay structure being complex matters are generally left
to the Executive and expert bodies like the Pay Commission etc. It has been emphasized
that a carefully evolved pay structure ought not to be ordinarily disturbed by the Court as
it may upset the balance and cause avoidable ripples in other cadres as well. (Vide :
Secretary, Finance Department and Ors. vs. West Bengal Registration Service
Association and Ors.5and State of Haryana and Anr. vs. Haryana Civil Secretariat
Personal Staff Association6. Nevertheless, it will not be correct to lay down as an
absolute rule that merely because determination and granting of pay scales is the
prerogative of the Executive, the Court has no jurisdiction to examine any pay structure
and an aggrieved employee has no remedy if he is unjustly treated by arbitrary State
action or inaction, except to go on knocking at the doors of the Executive or the
Legislature, as is sought to be canvassed on behalf of the appellants. Undoubtedly, when
there is no dispute with regard to the qualifications, duties and responsibilities of the
persons holding identical posts or ranks but they are treated differently merely because
they belong to different departments or the basis 1992 AIR SCW 1071
2002 AIR SCW 2896

@page-SC1029
for classification of posts is ex-facie irrational, arbitrary or unjust, it is open to the Court
to intervene.
5 (1993) Supp (1) SCC 153
6 (2002) 6 SCC 72
11

. In State Bank of India and Anr. vs. M.R. Ganesh Babu and Ors.7, a three-Judge
Bench of this Court, dealing with the same principle, opined that principle of equal pay is
dependent upon the nature of work done. It cannot be judged by the mere volume of
work; there may be qualitative difference as regards reliability and responsibility. The
functions may be the same but the responsibilities do make a difference. It was held that
the judgment of administrative authorities, concerning the responsibilities which attach to
the post, and the degree of reliability expected of an incumbent, would be a value
judgment of the authorities concerned which, if arrived at bona fide, reasonably and
rationally, was not open to interference by the Court. 2002 AIR SCW 1881

7 (2002) 4 SCC 556


12

. In State of Haryana and Anr. vs. Tilak Raj and Ors.8, it has been observed that the
principle of 'equal pay for equal work' is not always easy to apply as there are inherent
difficulties in comparing and evaluating the work of different persons in different
organizations or even in the same organisation. It has been reiterated that this is a concept
which requires for its applicability, complete and wholesale identity between a group of
employees claiming identical pay scales and the other group of employees who have
already earned such pay scales. It has been emphasized that the problem about equal pay
cannot be translated into a mathematical formula. 2003 AIR SCW 3382

8 (2003) 6 SCC 123


13

. Yet again in a recent decision in State of Haryana and Ors. vs. Charanjit Singh and
Ors.9, a Bench of three learned Judges, while affirming the view taken by this Court in
the cases of State of Haryana and Ors. v. Jasmer Singh and Ors.10, Tilak Raj (supra),
Orissa University of Agriculture and Technology and Anr. vs. Manoj K. Mohanty11and
Government of W.B. vs. Tarun Roy and Ors.12has reiterated that the doctrine of equal
pay for equal work is not an abstract doctrine and is capable of being enforced in a court
of law. Inter alia, observing that equal pay must be for equal work of equal value and that
the principle of equal pay for equal work has no mathematical application in every case,
it has been held that Article 14 permits reasonable classification based on qualities or
characteristics of persons recruited and grouped together, as against those who are left
out. Of course, the qualities or characteristics must have a reasonable relation to the
object sought to be achieved. Enumerating a number of factors which may not warrant
application of the principle of equal pay for equal work, it has been held that since the
said principle requires consideration of various dimensions of a given job, normally the
applicability of this principle must be left to be evaluated and determined by an expert
body and the Court should not interfere till it is satisfied that the necessary material on
the basis whereof the claim is made is available on record with necessary proof and that
there is equal work and equal quality and all other relevant factors are fulfilled. 2005
AIR SCW 5632
1997 AIR SCW 1057
2003 AIR SCW 3382
2003 AIR SCW 2513

9 (2006) 9 SCC 321


10 (1996) 11 SCC 77
11 (2003) 5 SCC 188
12 (2004) 1 SCC 347
14. Tested on the touchstone of the aforenoted broad guidelines and not cast-iron
imperatives, we are of the opinion that in the present case, on the pleadings and the
material placed on record by the parties in support of their respective stands, the High
Court was justified in issuing the impugned directions.
15. Vide order dated 10th October, 1997 passed by the Ministry of Home Affairs in
pursuance of para 7 of the Ministry of Finance, Department of Expenditure Resolution
dated 30th September, 1997, it was notified that the President was pleased to rationalize
the rank structure and pay scales of non gazetted cadre of central police organizations and
as a result of this exercise certain ranks were to be merged and the rank structure was
communicated in the order along with the revised pay scales and replacement pay scales.
Copy of this order was sent to all the paramilitary forces, including the Assam Rifles. On
22nd January, 1998, an office memorandum was issued by the Government of India,
Ministry of Home Affairs, by way of a clarification. In the said letter, it was clarified that
order dated 10th October, 1997 was equally applicable to all advertised categories. In the
@page-SC1030
said letter, direction with regard to the re-designation of the three posts including Head
Constable (RM) as ASI in central paramilitary forces along with their replacement pay
scales were also ordered. It appears that the disparity in rank and pay in various central
paramilitary forces could not be resolved and on 24th April, 2001, the Director General
Assam Rifles submitted a report to the Government with regard to the progress on pay
anomaly cases. Para 4 of the said letter is of some relevance to the issue at hand and it
reads as follows :
"Rank and pay of Technical Cadre Person RM. Ptmn, Pharma, and Compounder of AR
with the same intake QR for remounts are given the rank of HAV wherein they are
counterparts in CPOs are given ASI. The MHA had ordered to submit proposal in
directing cadre to cadre comparison with BSF where the rank of ASI is available in other
tech and also along with fin implication. The proposal along with fin implication has been
submitted to MHA and the case is lying with MOF for approval."
16. Having failed to receive any positive response from the Government, one of the
Radio Mechanics issued a Notice of Demand to the Ministry of Home Affairs and
Director General of Assam Rifles, inter alia, praying for giving effect to office order
dated 10th October, 1997 and office memorandum dated 22nd January, 1998. Vide order
dated 26th December, 2001, the Ministry of Home Affairs informed the Director General
of Assam Rifles that his proposal had been examined in consultation with Ministry of
Finance and it was found that there was no point for comparison of grades and scales of
pay for such posts across various central paramilitary forces. It was stated that the
proposed upgradation may disturb relativities of various trades and grades within the
Assam Rifles and there was no functional justification for upgrading these posts. It is
evident that on rejection of the recommendation made by the Director General of the
Force, the respondent herein was left with no option but to approach the High Court for
redressal of his grievance.
17. As noted above, the writ petition was opposed by the petitioners herein by filing
counter affidavit. For the sake of ready reference, the relevant portions in some of the
paragraphs of the counter affidavit are extracted below :
"That, with regards to the averments of the petitioner made in the writ petition in
paragraph 5, I submit that Assam Rifles personnel were in receipt of pay and allowances
on Army analogy with various groups in terms of Group A, B, C, D, and E to conform to
their functional qualitative requirements of these groups which had varying pay scales. I
submit that on the recommendation of the fourth pay commission w.e.f. 01.01.1986 for
Force had been granted and pay and allowance entirely on the lines of Central Para
Military Forces while no change in the rank structure was carried out and this difference
in rank structure has resulted in an apparent disparity in their service conditions and
certain category of personnel who were placed in the erstwhile higher groups including
radio mechanics category have also been deprived of pay scales either at par with their
counterparts in the Army or in the Central Police Organisation."
..... ..... ..... ..... ..... ..... ..... .....
"That, with regard to the averments of the petitioner made in the writ petition in
paragraph 8, I beg to reiterate that Assam Rifles personnel were in receipt of pay and
allowances on Army analogy with various groups in terms of group B, C, D, and E, to
conform to their functional and qualitative requirements of these groups which had
varying pay scales in diminishing order. On the recommendation of the Fourth Pay
Commission w.e.f. 1st January, 1986 the Force had been granted pay and allowances
entirely on the lines of Central Para Military forces shortly called as CPMFs while no
change in the rank structure was carried out, and this difference in the rank structure has
resulted in an apparent disparity in their service condition.
..... ..... ..... ..... ..... ..... ..... .....
"That, with regards to the averments of the petitioner made in the writ petition in
paragraphs 10 to 13, I beg to submit that on receipt of MHA letter No.27011/1103/97-
PF.1/56 dated 22nd January, 1998, Assam Rifles Directorate by letter No.A/Pers/5th
CPC/Vol.III/98 dated 18th February, 1998 had taken up a case with HA to redesignate
Hav/RM-Gde I and II of Assam Rifles as Warrant Officer and for replacement of pay
scale of Rs.4000-1000-6000/- to bring them at par with their counterparts in other Central
Police Organisation. I submit that attention of MHA was also drawn regarding placement
of Hav/RM Gde-I and II in the lower scale of pay consequent to
@page-SC1031
implementation of IV Pay Commission. In reply to the Assam Rifles Directorate letter the
MHA had ruled out vide their letter No.27011/103/97-P.F.1 dated 3rd March, 1998 that
Assam Rifles can redesignate HC (RM) as Warrant Officer if pre-revised and revised pay
scale of Hav(RM) in Assam Rifles are identical to the pay scale of HC(RM) in BSF and
CRPF. I submit that the main hurdle in implementing the said order in Assam Rifles is
that there is disparity in pay scales of RM in Assam Rifles to that of BSF and CRPF. The
HAV(RM) of Assam Rifles were drawing pay scales of Rs.975-1660/- w.e.f. 1st January,
1986 and replacement scale as given in the 5th Central Pay Commission is Rs.3200-
4900/- per month whereas in CRPF and BSF the HAV (RM) was drawing pay scale of
Rs.1200-30-1560-40-2040/- per month whose replacement scale in the 5th Central Pay
Commission is Rs.4000-100-6000/-. It is also pertinent to clarify here that the
qualification of HC(RM) in other Central Police Organisations that of Assam Rifles Hav
(RM) is almost at par"................
..... ..... ..... ..... ..... ..... ..... .....
"As per averment made in Para 13 of the writ petition, the petitioner is seeking higher pay
scale viz. 5000-150-8000/- admissible to Delhi Police personnel. I submit that since the
Assam Rifles is at par with other central police organization, the demand of the petitioner,
for parity with an entirely another department is not possible. In view of the facts narrated
above and to bring parity with other central police organization, it is proposed to grant
warrant officers rank (Equivalent to Assistant Sub Inspector) to technical categories
including radio mechanics vide Assam Rifles Directorate Letter No.A/Pers/45th CPC/Vol
III/98/77 dated 6th April, 1998 and subsequent queries sought by the MHA has been
replied. I submit that MHA has also informed to the LOAR (Liaison Office, Assam
Rifles) that the case for introduction of Warrant Officers rank to technical categories is
presently lying with Ministry of Finance (E-III) since 29th August, 2000."
(Emphasis supplied)
18. From the afore-extracted paragraphs of the counter affidavit and the resume of
correspondence referred to above, it clearly stands admitted by the petitioners herein that:
(i) all the paramilitary forces, including Assam Rifles are at par with each other and (ii)
there was apparent disparity in the pay scales of the personnel of Assam Rifles with their
counterparts in other central paramilitary forces. In order to rectify this disparity, Director
General Assam Rifles, petitioner No.2 herein, vide his letter dated 18th February, 1998
had, in fact, taken up the grievance of the respondent with the Ministry of Home Affairs,
inter alia recommending re-designation of Havildar (RM) Gd.-I and II of Assam Rifles as
Warrant Officer and for replacement of pay scale of Rs.4000-100-6000 to bring them at
par with their counterparts in other central police organization. However, the Ministry of
Home Affairs vide letter dated 3rd March, 1998 while accepting the said proposal had
recommended re-designation of HAV/RM as Warrant Officer but subject to the condition
that the pre-revised and revised pay scales of HAV/RM in other paramilitary forces were
identical to the pay scales of Head Constable (RM) in CRFP and BSF. Manifestly, in the
instant case, the differentiation in the pay scales of the two paramilitary forces is sought
to be achieved not on the ground of dissimilarity of academic qualification or the nature
of duties and responsibilities but only on the ground that there was initial anomaly in the
Fourth Central Pay Commission Report. The counter affidavit does not even attempt to
explain how the case of the HAV/RM in Assam Rifles is different from that of Radio
Mechanics in other central paramilitary forces.
19. In the present case, therefore, in the light of the admitted factual position, the question
of examination of external comparisons, internal relativities and other factors, to be kept
in view for job evaluation, considered to be a complex issue to be studied only by expert
bodies, does not arise. As a necessary corollary, the issue as to whether there is a
complete or wholesale identity between the said paramilitary forces, does not survive for
consideration.
20. Thus, the short question requiring our consideration is whether having admitted in
their affidavit referred to hereinabove, the apparent disparity and anomaly in the pay
scales of Radio Mechanics, the administrative authorities, the petitioners herein, could be
permitted to perpetuate apparent discriminatory differentiation in the pay scales because
of the disparity in pre-revised and revised scales of the personnel of Assam Rifles prior to
the recommendations of the
@page-SC1032
Fourth Pay Commission, irrespective of the identity of their powers, duties and
responsibilities with other paramilitary forces. In our considered opinion, in view of the
total absence of any plea on the part of the Union of India that Radio Mechanics in other
paramilitary forces were performing different or more onerous duties as compared to the
Radio Mechanics in Assam Rifles, the impugned decision of the Government was clearly
irrational and arbitrary and thus, violative of Article 14 of the Constitution.
21. On a conspectus of the factual scenario noted above, we do not find any infirmity in
the impugned directions given by the High Court, warranting interference. There is no
merit in this appeal and it is dismissed accordingly with costs.
Appeal dismissed.
AIR 2008 SUPREME COURT 1032 "L.M.T. Limited v. State of U. P."
(From : AIR 2001 All 321 : 2001 All LJ 2520)
Coram : 2 S. B. SINHA AND H. S. BEDI, JJ.
Civil Appeal No. 5789 of 2002 with C. A. Nos. 1106, 1622-1628, 1716 of 2007 and SLP
(C) No. 6721 of 2007, D/- 13 -12 -2007.
L.M.T. Limited v. State of U. P. and Ors.
WITH
Executive Engineer, Uttaranchal Power Corporation v. M/s. Kashi Vishwanath Steel Ltd.
and Ors.
WITH
Diamond Cement v. U. P. Power Corporation Ltd. and Ors.
WITH
M/s. Saini Alloys Pvt. Ltd. and etc. v. U.P. Power Corporation and Ors. etc.
WITH
M/s. Sandila Metal Wires (P) Ltd. v. U. P. Power Corporation Ltd. and Ors.
AND
M/s. Jagannath Steels Pvt. Ltd. v. U. P. Power Corporation Ltd. and Ors.
(A) U.P. Electricity Reforms Regulation Act (24 of 1999), S.24 - ELECTRICITY -
ELECTRICITY REGULATORY COMMISSION - DOCTRINES - State Electricity
Regulatory Commission - Powers of - Tariff - Licensee wanted some alteration -
Commission expected to take decision forthwith - It should not have whiled away time
and allowed Power Corporation to proceed with its proposal - Such a conduct on part of
Commissioner may invite doctrine of acceptance sub silentio.
Doctrines - Doctrine of sub silentio. (Para 41)
(B) U.P. Electricity Reforms Regulation Act (24 of 1999), S.24 - ELECTRICITY -
ELECTRICITY REGULATORY COMMISSION - WITNESS - Tariff - Fixation of, by
Electricity Regulatory Commission - Licensee thereafter calling for options for
uninterrupted simply from consumers of electrical energy at extra surcharge - Consumers
exercising options for continuous supply - Said Circular cancelled /rescinded being found
illegal by High Court - Raising bills retrospectively by licensee on ground of promise
made by consumers - Not proper - For mistake on part of licensee, consumers should not
suffer.
Evidence Act (1 of 1872), S.155. (Paras 42, 48, 49, 50)
Cases Referred : Chronological Paras
2007 AIR SCW 3752 : AIR 2007 SC 1984 (Ref.) 39
2004 AIR SCW 3703 : AIR 2004 SC 4559 (Ref.) 39
2003 AIR SCW 6931 : AIR 2004 SC 760 (Disting.) 46
2002 AIR SCW 1191 : AIR 2002 SC 1361 (Disting.) 45
2002 AIR SCW 4212 : AIR 2002 SC 3588 (Disting.) 45
2001 AIR SCW 5073 : AIR 2002 SC 453 (Ref.) 44
1999 AIR SCW 3337 : AIR 1999 SC 3341 (Ref.) 43
AIR 1986 SC 872 (Ref.) 40
(1951) AC 837 40
Ms. Rachana Srivastava, Addl. Advocate General, M. L. Lahoti, Ravindra Shrivastava, C.
S. Vaidyanathan, T. S. Doabia and Rakesh Dwivedi, Sr. Advocates, E. C. Agrawala,
Mahesh Agarwal, Rishi Agrawal, Gaurav Goel, Amit Sharma, Ms. Neha Aggarwal,
Ananaya Kumar, Kunal Verma, Rajul Shrivastava, Ms. Vibha Datta Makhija, Arbind
Kumar Shukla, Sunil Kr. Shukla, Vishal Dixit, Alok Shukla, Irshad Ahmad, Niraj
Sharma, Manpreet Singh Doabia, Vikrant Singh Bais, Aseem Chandra, Anurag Singh, C.
Murlikrishna, C. Balakrishna, Ramesh Singh, Mrs. Sheela Goel, R. Santhanam, R. C.
Gupta, Ms. Manjula Gupta, Hari Shankar K. , E. C. Vidya Sagar, Pradeep Misra, Manoj
Swarup (For M/s. Manoj Swarup and Co.), Amit Bhandari, Vikas Mehta and Rajiv
Mehta, for the appearing parties.
@page-SC1033

Judgement

S. B. SINHA, J :- Effect of two Circular letters issued by the U.P. Power Corporation
Limited is involved in these appeals, which arise out of a judgment and order dated 25th
April, 2001 of the High Court of Judicature at Allahabad in CMWP No.40692 of 2000 ;
judgment and order dated 17th January, 2007 passed by the High Court of Uttaranchal at
Nainital in WP No. 936 of 2001 and judgment and order dated 19th October, 2006 of the
High Court of Judicature at Allahabad, Lucknow Bench, Lucknow, in Appeal No.82 of
2002 etc. etc. Reported in AIR 2001 All 321 : 2001 All LJ 2520

2. State of Uttar Pradesh constituted Uttar Pradesh Electricity Board in terms of the
provisions of the Electricity (Supply) Act, 1948. In the year 1999 Uttar Pradesh
Electricity Reforms Act, 1999 (for short, the 1999 Act) was enacted, in terms whereof the
U.P. Electricity Regulatory Commission (for short, 'the Commission') was constituted.
Indisputably, three licensees, namely, (i) U.P. Power Corporation Ltd. (for short,
'UPPCL'), (ii) Kanpur Electricity Supply Company (for short, 'KESCO'); and (iii)
NOIDA Power Company Ltd. (for short 'NPCL') filed applications before the
Commission for determination of 'tariff'.
3. By reason of a notification dated 07.08.2000, tariff was framed which was to come into
force from 09.08.2000, inter alia, providing for :
"RATE SCHEDULE HV-2 LARGE AND HEAVY POWER
1. Applicability :
This rate schedule shall apply to all consumers who have contracted load of more than 75
KW (100 BHP) for industrial and/or processing purposes as well as to Acr/Induction,
Furnaces Rolling/Re-Rolling Mills, Mini Steel Plants and to any other power consumers
not covered under any other rate schedule.
This rate schedule shall also apply to commercial light, fan and power consumers
(LMV-2) and power consumers of Rate Schedule LMV-6, subject to the condition that
they opt for this Rate Schedule.
The contracted demand shall be expressed in whole number only.
2. .... ... ...
3. ... ... ...
4. Rate of charge

Description Demand Charge Energy Charge

A. Basic Rate (Applicable to Urban Consumers) Rs. 130/- per KVA/Month PLUS
390 paise/KWH

Notes : (a) In respect of consumers who opt for power supply during restricted/peak
hours an additional surcharge of 15% on the amount billed at the Rate of Charge under
item 4-A above, i.e. Demand Charge and Energy Charge shall be levied.
However, in respect of consumers getting power supply on independent feeders
emanating from 400/220/132 KV sub-stations an additional surcharge of 15% on demand
and energy charges shall be charged further subject to the condition that these consumers
will get an assured supply of minimum 500 hours in a month. In case of short fall in
above guaranteed hours of supply a rebate @ 1% for each 10 hours short fall will be
admissible on the bill amount computed under Rate of Charge."
(b) ... ...
...
(c) ... ...
...
(d) In respect of supply during peak hours/restricted hours, the consumers shall have to
take the permission from UPPCL."
4. Appellant LML Ltd. prior to framing of the said tariff and bifurcation of U.P. State
Electricity Board had been taking supply of electrical energy in the form of a three phase
alternatives current at declared pressure of 132 K.V. and a power not exceeding 8000
K.V. in their respective factories. Whereas in the case of L.M.L. Limited, their factory
being situated at Kanpur, electrical energy was supplied by KESCO, but so far as other
consumers are concerned, electrical energy was supplied to them by UPPCL
5. Appellants-consumers herein claimed that although they had been running a non-
continuous process industry but was not to observe peak hours restriction and in terms
thereof they did not consume power from 6.00 p.m. to 11.00 p.m. (being the peak hours).
6. A confusion arose in regard to interpretation of the said purported levy of 15%
surcharge on demand and energy charge on independent feeders from 400/220/132 KV
@page-SC1034
sub-stations having assured supply of minimum 500 hours in a month. In the event, the
consumers were to get power supply from independent feeders, were to get supply of
minimum 500 hours in a month, indisputably, they were to pay 15% surcharge on
demand.
7. UPPCL, however, on construction of the said provisions of the statute issued a circular
letter dated 08.09.2000 calling for options from the consumers of electrical energy, who
did not intend to have a continuous power supply of 500 hours in a month. A copy of the
said circular letter admittedly was sent to the Secretary of the Commission, the relevant
paragraphs whereof read thus :
"Some other important guidelines/directions are being issued with the request that please
make aware to all your concerned subordinate officers and ensure its strict compliance.
1. 15% surcharge will be payable for Electricity use in prohibited period in new rate list
of L.M.V.-6 and HV-2. Consumers who were notified by U.P. Government under
continuous category before new tariff should be necessarily imposed 15% surcharge in
their bills. The facility of Electricity supply in prohibited period should be continued as
before to consumers falling under this category and option letter should not be asked
from them.
In addition, consumers of non-continuous category will not be provided the facility to use
Electricity in prohibited period. But, if the consumer of this category wants to use
electricity in prohibited period, he will intimate to concerned Executive Engineer through
registered letter.
Executive Engineer within three days of receiving this letter will issue office circular
which will indicate the date from which this facility can be provided . 15% surcharge will
be payable by the consumer from the said date mentioned in above letter. This option
once given will not be revoked.
2(a) Consumers connected to independent feeders will be charged 15% surcharge against
guarantee of 500 hours electricity supply from sub-stations of 400 KV, 220 KV and 132
KV in HV-2 rate list. 500 hours electricity supply will be ensured to the consumers of this
category. 1% rebate will be given on Electricity Bill of 10 hours or its part, if they receive
electricity supply less than 500 hours. If the consumers connected to these independent
feeders who do not want guaranteed supply of 500 hours electricity supply then 15%
surcharge will not be charged on their electricity bills. These type of consumers will
intimate to Executive Engineer (Distribution) if they do not want 500 hours guarantee of
electricity supply. Executive Engineer will issue office memo in this regard. If any
consumer of this category does not exercise this option, then he will be guaranteed 500
hours electricity supply and will be charged 15% surcharge. It will be the responsibility
of SSO/Assistant Engineer to ensure that consumers of this category should not use
electricity in the restricted period. In case consumers of this category use electricity in the
restricted period then they will be charged (15+15) 30 % surcharge.
2(b) Normally the availability of electricity supply to the consumers from these feeders
will depend upon data of electronic meters installed in their establishments and no officer
will be authorized for issuing any certificate and nor such certificate will be acceptable.
2(c) In case electric meter is not available at consumers' establishment or is defective,
then during this period only no employee below the level of Asstt. Engineer will issue
any certificate under his signatures under any circumstances regarding period of
electricity supply/hours etc. and in case this is issued the same will not be accepted, and
the concerned officer/employee will be deemed guilty of indiscipline and appropriate
action will be taken against them. As per requirement, this type of certificate can be
issued by Asstt. Engineer or above level officer and they may get the signature of
subordinate officer/employee if they wish. This certificate will be made available to
concerned Executive Engineer (Distribution) for each month.
2(d) Every month's intimation/certificates of electricity supply hours along with the
reason of less supply hours will be provided by Sub-division Officer of sub station of
400, 220 and 132 KV to Executive Engineer (Distribution) for the purpose of issuing bill
to consumer.
The supply hours should tally with the hours written in the log book of sub station. Along
with this, the sub-division officer will provide the certificate confirming whether
electricity was supplied in peak hours or not? In case the power supply to large and
@page-SC1035
heavy power consumers is less than prescribed hours for two consecutive months, then,
concerned Dy. General Manager of the sub-station will review the situation at his level
and resolve the same.
Review of power supply to small and medium consumers shall be done by Executive
Engineer of concerned sub-station."
8. On or about the 14.09.2000, the Executive Engineer of UPPCL issued notices to the
parties, inter alia, stating :
"As per the Extra-ordinary Gazette dated 27.07.2000 of Government of U.P., the U.P.P.C.
Ltd. has revised the tariff of consumers of all the categories from 9.8.2000. Accordingly,
both the categories i.e. Continuous and Non-continuous have been amalgamated. The
restriction is that they will have to submit their separate option for use of electricity
consumption in peak hours and restricted use of electricity that if they want to consume
the electricity in peak hours and restricted period, they will be required to pay 15% extra
surcharge on the amount worked out as per category 4 of the tariff rate. Without
permission of U.P.P.C. Ltd., the consumption of electricity in this period is prohibited
otherwise action as per rules will be taken.
You are, therefore, hereby requested that you intimate in writing to this office within 15
days of receipt of this letter that whether you want to consume the electricity during the
peak hours and restricted use of electricity period or not so that you tariff rate could be
fixed accordingly in H.V. 2 category. The consumption of electricity during the said
period will be prohibited without permission of U.P.P.C. Ltd. In case of violation, you
will be liable to financial and other losses. Option given by you shall be effective from
9.8.2000."
9. Pursuant thereto and in furtherance thereof, by reason of a letter dated 16.09.2000, the
consumer opted for not having a continuous power supply of 500 hours, a sample copy
whereof is as under :
"As you already know that we are electricity consumer in the category of Non-continuous
process of 132 KVA. We have to inform you that we shall not be consuming the same
during the peak hour restrictions. Further we are not opting for such guaranteed supply of
electricity for 500 hours per month and in default thereof a rebate of 1% for every 10
hours of electricity non-supply. This does not, however, mean that you shall subject us to
any unscheduled and arbitrary cuts in the supply in future
We are sure that you shall continue to supply electricity as in the past from the same
feeder line. This letter is in compliance of the requirement of the above notification dated
8.9.2000, and hence the additional surcharge of Rs.6,33,898.45 shall be withdrawn from
our bill dated 5.9.2000. The payment of Rs.53,01,727/- having been made by cheque
No.207076 dated 11.9.2000 (handed over in the Court of Chief Justice, Allahabad on
13.9.2000). Thus the aforesaid bill stands finally paid.
We are sure that in future our bills shall not be loaded with additional surcharge of 15%."
10. It appears that meetings were also held by and between the consumers and Secretary
and Chairman of UPPCL at PHD Chambers of Commerce at Delhi, wherein it was
decided that only thermal industries would not be charged 15% additional surcharge who
did not want to go for assured supply of 500 hours.
11. It further appears that UPPCL issued another circular letter dated 15.12.2000, the
relevant portion whereof reads thus :
"U.P Electricity Regulatory Commission in its revised tariff for the year 2000-01
applicable to HV-2 rate schedule consumers who are getting supply from independent
feeders for levy of 15% surcharge on the guarantee of 500 hours of power supply per
month.
In this regard, detailed guidelines have been issued by this office vide letter No. 1423
dated 9.8.2000.
In this regard, it is directed that those consumers who will exercise option, of not availing
500 hours guaranteed supply, through a registered letter to Executive Engineer
(Distribution) by 31.12.2000, they will not be charged 15% surcharge from the very date
of its applicability i.e. 7.8.2000. For consumers, who will submit their option after
31.12.2000, this facility will be applicable from the date of receipt of the application."
12. Although no such circular letter was issued by KESCO, relying on or on the basis of
circular letter issued by UPPCL, L.M.L. Limited filed a writ petition in the Allahabad
High Court.
@page-SC1036
13. Upon taking into consideration the jurisdiction of the UPPCL to implement the tariff
fixed by the Commission vis-a-vis the procedure required to be adopted therefor, the
High Court by reason of the impugned judgment and order dated 25.04.2001 opined that
it had absolutely no jurisdiction to make any modification in the tariff and in that view of
the matter the purported circular letter issued on 08.09.2000 was invalid in law, inter alia,
stating :
"The contention raised on the basis of circular dated 8.9.2000 issued from the office of
Chief General Manager (Commercial), UPPCL, is equally untenable. The provision in
later part of paragraph 2 Ka thereof which lays down that 15 per cent surcharge would
not be levied in case a consumer getting supply from an independent feeder emanating
from 400/220/132 KV sub-station gave an option that he did not want a guarantee of 500
hours of supply in a month, is contrary to the tariff approved by the Commission. The
Commission in its order approving the tariff had merely provided that in case of shortfall
in 500 hours of assured supply in a month, a rebate of 1 per cent for each 10 hours
shortfall will be admissible on the total amount computed under Rate of Charge. The
Circular while retaining this provision has made an additional provision to the effect that
if such type of consumer gave an option that he did not want an assured supply of
minimum 500 hours in a month, the 15 per cent surcharge shall be not levied. This is a
clear alteration of the approved tariff which is not permissible in law.
14. Relying on or on the basis of the said judgment of the Division Bench of the
Allahabad High Court delivered in the case of L.M.L. Limited, UPPCL issued another
circular dated 31.08.2001 cancelling the earlier circulars, stating :
"Since some confusion has arisen amongst the Field/Regional Officers on this provision,
Commercial Division vide its letter Nos. 1423-HC/UPPCL/Five-1974-1204 dated
8.09.2000 and No. 3046/HC /Tariff/SAMA/Nirdesh dated 15.12.2000 had issued
clarifications after discussions with U.P. Electricity Regulatory Commission.
LML Kanpur had filed a Writ Petition No.40692/2000 before the Honble Allahabad High
Court on this subject. Honble Allahabad High Court in its order has directed that tariff as
approved by Electricity Regulatory Commission only will be applicable and the licensee
cannot amend the tariff. Therefore, in the light of Hon'ble Allahabad High Court's
judgment Circular No. 1423 dated 8.9.2001 (Point No. 2) and letter No. 3046 dated
15.12.2000 stand rescinded from the date of their issue."
Bills were issued in October 2001 with retrospective effect from November 2000.
15. Appellants other than L.M.L. Limited filed several writ petitions questioning the
legality and/or validity of the said circular dated 31.08.2001.
Several contentions were raised in the writ petitions including the jurisdiction of UPPCL
to issue bills with retrospective effect.
It was furthermore contended that the appellants having altered their position pursuant to
or in furtherance of the promise made by UPPCL in terms of its circular letter dated
08.09.2000, they were estopped and precluded from raising any bill, with retrospective
effect or otherwise.
Attention of the High Court in the subsequent writ petitions were also drawn to the fact
that UPPCL had carried out extensive consultation with the Commission on several dates.
16. It was pointed out that UPPCL itself in its counter affidavit filed in the case of Modi
Pon Fibre Company, Ghaziabad before the High Court had stated as under :
"4. That there was some confusion in the category of consumers who were covered by
both category (i) and (ii) above, and who on plain reading of the tariff were liable to pay
surcharge of 15% plus 15%. To clarify the above UP Power Corporation Limited,
hereinafter referred to in brief as UPPCL, held discussions both with the Commission and
the Government of Uttar Pradesh through Principal Secretary, Power. The above
discussions culminated in the passing of Circular No.1423-HC/UPPCL/5-1974-1204-
C/2000 dated 8.9.2000 by UPPCL. The above circular as per its Para 2(Ka) gives an
option to the consumers under category (ii) that in case they do not want to receive
supply of assured 500 hours in a month no surcharge of 15% shall be charged from them.
It was provided in the above circular that the concerned consumer may give their option
of waiver of assured supply by registered post to the concerned Executive Engineer
(Distribution). It was further provided
@page-SC1037
in the above circular that in case the consumer fails to exercise the above option, he will
be assured supply of 500 hours and he shall be liable to pay surcharge of 15%. A copy of
the above circular is endorsed to the Secretary of the Commission for information and
necessary action. A copy of the above circular dated 8.9.2000 is appended to this Short
Counter Affidavit as its Annexure CA-1.
5. By another Circular No.3046-HC/Tariff/general instruments, dated 15.12.2000, it was
provided that the consumers of category (ii) above may exercise their option of not
availing 500 hours guaranteed supply through a registered letter to Executive Engineer
(Distribution) by 31.12.2000. A copy of the above circular was endorsed to the Secretary
of the Commission for information and necessary action. A copy of the above circular
dated 8.9.2000 is appended to this Short Counter Affidavit as its Annexure CA-2.
It was further stated therein :
7. Since the circular dated 8.9.2000 now stands rescinded pursuant to the orders of the
Hon'ble Allahabad High Court and since UPPCL has failed to elicit any response from
the Commission to its letters (Annexure Nos. CA-3, 3A and 3B), UPPCL has initiated
action for charging 15% surcharge from consumers of category (ii) above which as per
the tariff order dated 27.7.2000 passed by the Commission. A circular No. 925
HC/LML/LS-15 dated 31.8.2001 has been issued by the respondent to the above effect. A
copy of the above circular dated 31.8.2001 is appended to this Short Counter Affidavit as
its Annexure No. CA-4 "
17. Before the High Court, several other documents were brought on record, including a
letter dated 11.06.2001 which had been filed before the Commission, which was in the
following terms :
"In accordance with the rates specified by U.P.E.R.C. in its Tariff Orders dated 27.7.2000,
it was provided in the Notification for rate Schedule for HV-2 category issued by
U.P.P.C.L. that 15% surcharge will be levied on consumers who opt for power supply
during restricted/peak hours. It was also provided that additional surcharge of 15% on
demand and energy charges will be payable by the consumers getting supply on
independent feeders subject to the condition that they will get assured supply of 500
hours in a month. Subsequently, as per discussions in the Honble Commission it was
clarified by UPPCL vide letter No. 1423-HC/UPPCL/V-1974-1204-C/2000 dated
8.9.2000 that the levy of 15% surcharge on consumers on independent feeder will be
optional subject to their giving the option.
A writ was filed by M/s LML, Kanpur who is a consumer of KESCO claiming that 15%
additional surcharge for independent feeder should not be levied on them as provided in
circular no. 1423-HC/UPPCL dated 8.9.2000 referred to above. The Hon. High Court,
Allahabad have held that the provision of para-2(Ka) of above referred circular dated
8.9.2000 giving option to the consumers on independent feeders is a clear alteration of
the approved tariff. They have further held that the circular of UPPCL insofar as it is
inconsistent with the tariff approved by the Commission is void and wholly inoperative in
law. The petitioner, therefore, cannot get any advantage by exercising an option in terms
of circular by way of informing through the registered post that he did not want an
assured supply of 500 hours in a month.
It may kindly be recalled that the clarification issued vide above referred letter No. 1423
dated 8.9.2000 was subsequent to the detailed discussions held in the Commission as well
the then Pramukh Sachiv Oorja.
It is, therefore, requested that the above facts may kindly be brought to the notice of the
Hon'ble Commission and further directions may kindly be issued so that the same may be
implemented as ordered by the Hon. High Court, Allahabad."
18

. The Chief General Manager, UPPCL by reason of a letter dated 23.06.2001 drawing the
attention of the Commission to the said letter dated 11.06.2001 had requested it to issue
necessary guidelines in the light of the order dated 25.04.2001 passed by the Allahabad
High Court in W.P. No.40692 of 2000. Reported in AIR 2001 All 321 : 2001 All LJ
2520

19. Yet again, on or about 24.08.2001, the Executive Director, UPPCL, referring to its
earlier letter dated 11.06.2001 as also a reminder letter dated 23.06.2001 requested the
Secretary of the Commission to issue necessary guidelines in regard to the levy of 15%
surcharge , inter alia, stating :
"........It may also be brought to the kind notice of the Commission that at present
@page-SC1038
field unit of UPPCL are not charging 15% surcharge from such consumers on
independent feeders who have given option for not availing 500 Hrs. of guaranteed
supply during a month."
20. Other Division Benches of the Allahabad High Court, however, chose to follow its
earlier decision in L.M.L. Limited (supra).
21. We may notice that some of the appellants herein had filed reference applications
before the Commission, which were found to be not maintainable. A Review Application
was also filed whereafter, the First Appeals were filed before the High Court. It may,
however, be placed on record that in regard to the meaning of independent feeders some
matters are still pending before the Commission.
22. We may also note that on similar questions, the Uttaranchal High Court has allowed
the writ applications filed before it.
23. The learned counsel appearing on behalf of the appellants, inter alia, would submit
that the High Court committed a manifest error in passing the impugned judgment insofar
as it failed to take into consideration that in terms of sub-section (6) of Section 24 of the
1999 Act, it was for the licensee to modify the tariff and in view of the fact that before
doing so, they had held extensive consultation with the Commission; the impugned
judgments are wholly unsustainable.
It was also submitted that in any event, the doctrine of promissory estoppel could
squarely be applicable in the instant case as the appellants herein had altered their
position relying on or on the basis of the representation so made.
24. Mr. Rakesh Dwivedi, learned Senior Counsel appearing on behalf of the respondents,
on the other hand, submitted :
(i) No promise having been made by KESCO, the principle of promissory estoppel will
have no application.
(ii) In any event there cannot be any estoppel against the statute.
(iii) So far as UPPCL is concerned, having regard to the provisions of the 1999 Act in
terms whereof the Commission alone possessed the power to modify the tariff, the
impugned judgments are unassailable.
25. The 1999 Act was enacted to provide for the restructuring of the electricity industry,
the rationalization of generation, transmission, distribution and supply of electricity,
regulation by an independent electricity regulatory Commission of the electricity industry
including the purchase, distribution, supply and utilization of electricity, the quality of
service, tariff and other charges keeping in view the interest of the consumers and
utilities, creation of an environment which will attract participation of private sector
entrepreneurs in the electricity industry in the State and generally for taking measures
conducive to the development and management of the electricity industry in the State in
an efficient, economical and competitive manner and for matters connected therewith or
incidental thereto.
26. Commission is defined in Section 2(f) of the 1999 Act to mean the Uttar Pradesh
State Electricity Regulatory Commission referred to in Section 3 thereof. Section 10 of
the 1999 Act provides for the functions of the Commission including the one to determine
the tariff for electricity - wholesale, bulk, grid or retail, as the case may be.
27. Section 13 provides for formation and functions of the Uttar Pradesh Power
Corporation.
28. Section 24 occurring in Chapter VII of the 1999 Act provides for licensee's revenue
and tariffs. Sub-section (1) of Section 24 states that the licensee shall follow the
procedure prescribed in the regulations in calculating the expected revenue from charges
which he is permitted to recover and in determining tariffs. Sub-section (2) of Section 24
provides for the factors which are relevant for the purpose of determining the tariffs in the
following terms :
"24. Licensees revenues and tariffs.-
(1) ... ... ...
(2) Save as provided in sub-section (3), the Commission may specify in regulations the
terms and conditions for the determination of the revenue and tariffs and, in doing so, the
Commission shall be guided by the following, namely :-
(a) the financial principles and their application provided in Sections 46, 57 and 57-A of
the Electricity (Supply) Act, 1948 and in the Sixth Schedule thereto;
(b) the factors which would encourage efficiency; economical use of the resources, good
performance, optimum investments, observance of the conditions of the licence and other
matters which the Commission
@page-SC1039
may consider appropriate for the purposes of this Act; and
(c) the interest of the consumers."
29. Sub-section (3) of Section 24 of the Act provides that in the event the Commission
departs from the factors specified in clauses (a) to (c) of sub-section (2), reasons therefor
shall be assigned. Sub-section (6) of Section 24 read as under :
"(6) The Commission may, after notifying its decision on the licensee's calculations as
provided in sub-section (5), determine whether the tariff charged by the licensee is
required to be modified, and if so, require the licensee to modify the tariff or any part
thereof with immediate effect.
30. Section 27 provides for enforcement of the orders and directions of the Commission.
Section 28 provides for penal provisions. Section 36 provides for appeals from the orders
of the Commission to the High Court.
31. The Commission in this case proceeded to determine the tariff keeping in view the
fact that the electricity rates for industries in the State of Uttar Pradesh were quite high
and any sharp increase in the rates would be counter productive. It, however, though to
impose 15% surcharge in relation to two types of supply, inter alia, keeping in view : (i)
supply during peak hours; and (ii) supply of independent feeders in terms whereof
continuous supply of minimum 500 hours in a month shall be assured.
32. Surcharge, therefore, was levied when the supply was to be made by the licensee on
fulfilment of conditions laid down therein.
33. We may notice that the Commission itself directed discontinuance of the said
surcharge with effect from 01.09.2001 by issuing a tariff order in the following terms:
"The U.P.E.R.C. in terms has recorded that discontinuation of 15% surcharge is due to (i)
inability/incapability on the part of UPPCL for technical and operational reasons to
ensure the guaranteed supply of 500 hours, (ii) it was difficult for UPPCL even to
distinguish between the two consumers on independent feeder who asked for assured
supply and who do not, (iii) most of the consumers having opted against this agreement
and (iv) the financial implication was also negligible if the scheme was discontinued."
34. Appellants-Consumers at all material times had been complaining in regard to
irregular supply of electrical energy by the licensee.
35. A supplier of electrical energy is presumed to know as to whether it would be in a
position to abide by the terms of supply imposed by the Commission. It was required to
gauze its capacity to make uninterrupted supply of electrical energy to a class of
consumers. Manufacturers of electrical energy belong to different classes. Manufacturers
of certain categories of goods having regard to the nature of their products would require
continuous supply of electrical energy; be it peak hours or otherwise. The licensees in
such cases are required to make special arrangements for continuous supply of electrical
energy to such class of consumers.
36. We have noticed hereinbefore that the consumers of electrical energy, who are before
us, did not intend to have supply of electrical energy during peak hours. Their need in
relation thereto, therefore, was not such which would have required continuous supply of
electrical energy. If keeping in view such a contingency, the suppliers intended to have an
assessment of their own capacity to supply uninterrupted electrical energy by asking for
option of the consumers concerned, we do not see as to how thereby they can be said to
have deviated from the tariff determined by the Commission. If one of the objects of the
Commission was to ensure uninterrupted supply of electrical energy, it was for the
supplier itself to assess its own capacity therefor. Surcharge may or may not be a part of
tariff. Even if it is a part of tariff in respect thereof, the levy was conditional. If the
supplier was not itself in a position to fulfil the condition, the question of insisting on
implementation of the said provision would not arise.
37. While we say so, we are not unmindful of the fact that imposition of 15% surcharge
was not dependent upon the exercise of option in terms of the tariff provision, which was
confined to the supply of electrical energy during peak hours.
38. Those suppliers, who keeping in view of their capacity to supply uninterrupted
electrical energy had made a representation and pursuant thereto the consumers had
altered their position, cannot be permitted to take a different stand as the doctrine of
promissory estoppel would apply against them. The said doctrine is premised on the
@page-SC1040
conduct of party making a representation to the other so as to enable him to arrange its
affairs in such a manner as if the said representation would be acted upon. It provides for
a cause of action. It need not necessarily be a defence.
39

. Application of said doctrine has been analysed by this Court in several judgments. We
would only refer to some of them. In Southern Petrochemical Industries Co. Ltd. vs.
Electricity Inspector and Etio and others, (2007) 5 SCC 447 this Court upon noticing a
large number of precedents including State of Punjab vs. Nestle India Ltd. and another,
(2004) 6 SCC 465 opined as under :- 2007 AIR SCW 3752
2004 AIR SCW 3703

"The doctrine of promissory estoppel would undoubtedly be applicable where an


entrepreneur alters his position pursuant to or in furtherance of the promise made by a
State to grant inter alia exemption from payment of taxes or charges on the basis of the
current tariff. Such a policy decision on the part of the State shall not only be expressed
by reason of notifications issued under the statutory provisions but also under the
executive instructions. Appellants had undoubtedly been enjoying the benefit of payment
of tax in respect of sale/ consumption of electrical energy in relation to the cogenerating
power plants."
40

. In Express Newspapers Pvt. Ltd. and others vs. Union of India and others, (1986) 1 SCC
133 this Court held :- AIR 1986 SC 872 (Para 178)

"179. It would appear that Denning, J. evoked two doctrines : (1) that assurances intended
to be acted upon and in fact acted upon were binding; and (2) that where a Government
department wrongfully assumes authority to perform some legal act, the citizen is entitled
to assume that it has that authority, and he dismissed the contention that estoppels do not
bind the Crown by saying that 'that doctrine has long been exploded' and that the Crown
cannot fetter its future executive action. Professor Wade points out that the proposition
about wrongful assumption of authority evoked by Denning, J. was immediately
repudiated by the House of Lords in a later case in which Denning, LJ. had again put it
forward in Howell v. Falmouth Boat Construction Company Ltd., L.R. [1951] AC 837, it
is beyond the scope of this judgment to enter into a discussion as to how far Denning, J.'s
dictum can still be regarded as part of the common law in England. But there appears to
be a school of thought in India laying down that the doctrine of promissory estoppel
applies to the Government except under certain circumstances.
41. We may also notice that the Commission did not take any decision despite repeated
communications by the Power Corporation. If in a situation of this nature where the
licensee wanted some alteration in the tariff, it was expected of it to take a decision
forthwith. It should not have whiled away the time and allowed the Power Corporation to
proceed with its proposal. Such a conduct on the part of the Commission may invite the
doctrine of acceptance sub silentio. The statute provides for a consultation and not a
concurrence. It does not provide for the consequence of any alteration of tariff applicable
to a particular category of consumer. It merely, as indicated hereinbefore, brings about the
situation where a licensee found itself unable to supply electrical energy uninterruptedly
to the consumer.
42. There can, however, be no doubt that ordinarily the doctrine of promissory estoppel
would not be applied against statute. Sub-section 6 of Section 24 of 1999 Act inter alia
empower the holder of a licence, to modify the tariff. If the implementation of tariff was
dependent upon fulfilment of certain conditions precedent which in turn would be
dependent upon the capacity of the producer of electrical energy to fulfil the same, in our
opinion, no impropriety was caused by the Power Corporation to ask for the said option.
The fact, that such an option had indeed been called for and pursuant thereto the
consumers had altered their position is not in dispute. While dealing with a question as to
whether an action on the part of the State to make a representation is contrary to a statute
or not, in our opinion, a distinction should be borne in mind between an act which goes
clearly contrary to the mandatory provisions thereof and a case where irregularities have
been committed.
43

. We may notice that in The Paper Products Ltd. vs. Commissioner of Central Excise,
1999 (7) SCC 84 this Court held :- 1999 AIR SCW 3337, (Para 7)

"As stated above, it is an admitted fact that by virtue of Circular No. 4/85 dated
@page-SC1041
23-7-1986 as clarified by Circular dated 7-8-1987, all the three products of the appellant
are to be treated as the products of the printing industry and not that of the packaging
industry. A change in the said view of the Board occurred for the first time by virtue of
the Circular No. 6/89 dated 16-1-1989. Further, the Board itself by its subsequent
Circular No. 29/89 dated 5-5-1989 has made it abundantly clear that the change notified
in Circular No. 6/89 will be prospective from the date of issuance of Circular No. 6/89,
that is, from 16-1-1989. Therefore, it is clear that till the issuance of Circular No. 6/89
which is dated 16-1-1989 the products of the appellant, by virtue of the two Circulars
dated 23-7-1986 and 7-8-1987, have to be classified under Chapter 49 of the Act as being
products of the printing industry eligible for exemption of duty under Notification Nos.
122/75 and 234/82 as applicable at the relevant time. The impugned show cause notices
and consequent demand being ab initio bad inasmuch as the same was contrary to the
existing Circulars of the Board, the same cannot be sustained."
44

. In Collector of Central Excise, Vadodra vs. Dhiren Chemical Industries, (2002) 2 SCC
127 this Court held :- 2001 AIR SCW 5773, (Para 11)

" We need to make it clear that, regardless of the interpretation that we have placed on
the said phrase, if there are circulars which have been issued by the Central Board of
Excise and Customs which place a different interpretation upon the said phrase, that
interpretation will be binding upon the Revenue."
45

. The latter decision is also an authority for the proposition that a circular would be
binding on the State in appropriate cases. We are not oblivious of the decisions of this
Court where the Commission has been held to be the sole tariff making authority. [See
Association of Industrial Electricity Users vs. State of Andhra Pradesh and others, (2002)
3 SCC 711] and West Bengal Electricity Regulatory Commission vs. C.E.S.C. Ltd. etc.
etc., (2002) 8 SCC 715. In CESC (supra) this Court observed :- 2002 AIR SCW 1191
2002 AIR SCW 4212, (Para 57)
"58. Having carefully considered the provisions of the Act as also the arguments
advanced in this regard, we are of the opinion that under the 1998 Act, it is the
Commission concerned and in the instant case the State Commission of West Bengal,
which is the sole authority to determine the tariff, of course as per the procedure in the
said Act."
46

. We may also notice that in BSES Ltd. vs. Tata Power Co., Ltd. and others, (2004) 1
SCC 195 this Court held :- 2003 AIR SCW 6931

"16. The word "tariff" has not been defined in the Act. "Tariff' is a cartel of commerce
and normally it is a book of rates. It will mean a schedule of standard prices or charges
provided to the category or categories of customers specified in the tariff. Sub-section (1)
of Section 22 clearly lays down that the State Commission shall determine the tariff for
electricity (wholesale, bulk, grid or retail) and also for use of transmission facilities. It
has also the power to regulate power purchase of the distribution utilities including the
price at which the power shall be procured from the generating companies for
transmission, sale, distribution and supply in the State. 'Utility' has been defined in
Section 2(1) of the Act and it means any person or entity engaged in the generation,
transmission, sale, distribution or supply, as the case may be, of energy. Section 29 lays
down that the tariff for intra-State transmission of electricity and tariff for supply of
electricity, wholesale, bulk or retail in a State shall be subject to the provisions of the Act
and the tariff shall be determined by the State Commission. Sub-section (2) of Section 29
shows that terms and conditions for fixation of tariff shall be determined by Regulations
and while doing so, the Commission shall be guided by the factors enumerated in Clauses
(a) to (g) thereof. The Regulations referred to earlier show that generating companies and
utilities have to first approach the Commission for approval of their tariff whether for
generation, transmission, distribution or supply and also for terms and conditions of
supply. They can charge from their customers only such tariff which has been approved
by the Commission. Charging of a tariff which has not been approved by the Commission
is an offence which is punishable under Section 45 of the Act. The provisions of the Act
and Regulations show that the Commission has the exclusive power to determine the
tariff. The tariff approved by the Commission is final and binding and it is not
permissible for the licensee, utility or any one else to charge a different
@page-SC1042
tariff."
47. The abovesaid three decisions are distinguishable on facts. They were not dealing
with a situation of the present kind. It was not a case where the supplier had difficulty of
supplying uninterrupted electrical energy.
48. The proximity of issuance of the circular vis-a-vis Notification must also be noticed.
The tariff was framed on 7th August, 2000 which came into force from 9th August, 2000
whereas the circular was issued on 8th September, 2000. The consumers exercised their
option on 31st October, 2000. The judgment in the case of LML (supra) was delivered on
25th April, 2001. The circular dated 31st August, 2001 undoubtedly was issued in view of
the said judgment. The said judgment did not deal with the questions raised before us. In
any event if the licensee violates the tariff approved by the Commission appropriate legal
action can be taken against it. But it would be too much to contend that for a mistake on
the part of the Corporation, the consumers would suffer. In this view of the matter, we are
of the considered view that the doctrine of estoppel shall apply in the cases where the
promise was made. However, the principle of said doctrine would, however, not be
applicable where no such promise was made.
49. Respondent-Kanpur Electricity Supply Company would not be bound thereby. Tariff
is fixed for providing a service. Supply of electrical energy is a public utility service.
While carrying out a function of this nature, the court of law must keep in mind the
equitable principles also. Equity does not postulates that although the supplier did not
fulfil its obligation, still it would be entitled to the benefits envisaged under the law.
50. Similarly Uttaranchal Power Corporation also does not appear to have made such a
promise. The doctrine of promissory estoppel in those cases also will have no application.
51. In view of the fact that several matters are pending before the Commission on
question of independent feeder we need not express any opinion thereupon. If any appeal
is pending before the Commission on the said question it would decide the same
independent of the same irrespective of the result of this decision. We, therefore, without
expressing any opinion on the said question, permit the appellants to agitate the same
point before the Commission.
52. We, therefore, allow these appeals only to the extent mentioned hereinbefore in terms
of the promise made by the U.P. Power Corporation and allow the appeals on question of
independent feeder to be withdrawn subject to the observations made by us hereinabove.
53. Civil Appeal No.5789 of 2002 which relates to Kanpur Electricity Supply Company
is dismissed.
54. Civil Appeal No.1106 of 2007 filed on behalf of the Uttaranchal Power Corporation is
allowed.
55. There shall, however, be no order as to costs.
SLP (C) NO. 6721/2007
The only issue involved in this petition is the question of independent feeder and the
appeal being pending before the Commission, this special leave petition is permitted to be
withdrawn.
Order accordingly.
AIR 2008 SUPREME COURT 1042 "M.S.E.D. Co. Ltd. v. Lloyds Steel Industries Ltd."
Coram : 2 A. K. MATHUR AND MARKANDEY KATJU, JJ.
Civil Appeal No. 3551 of 2006, D/- 14 -8 -2007.
Maharashtra State Electricity Distribution Co. Ltd. v. Lloyds Steel Industries Ltd.
Electricity Act (36 of 2003), S.86, S.42 - ELECTRICITY - ELECTRICITY
REGULATORY COMMISSION - COMPLAINT - State Electricity Regulatory
Commission - Jurisdiction - Complaints of individual consumers - Outside jurisdiction of
State Commission - Separate Forum for redressal of individual customer's grievances has
been created under S. 42 of Act - All individual customers' grievances should therefore,
be raised before such Forum.
(2006) 132 DLT 339, Approved. (Paras 7, 8)
Cases Referred : Chronological Paras
2006 (132) Delhi LT 339 (Approved)7
Judgement
1. JUDGMENT :-This appeal under Section 125 of the Electricity Act, 2003 (hereinafter
for short "the Act") is directed against the judgment passed by the Appellate Tribunal for
Electricity (hereinafter for short "the Appellate Tribunal") dated 5th April, 2006 in
Appellant's appeal No. 191/2005 and the order dated 2nd June, 2006
@page-SC1043
passed by the Appellate Tribunal in Review Petition No. 3/2006 and I.A. No.60/2006.
2. It is not necessary for us to go into the detailed facts. Suffice it to say that the
respondent company approached the Maharashtra Electricity Regulatory Commission
(hereinafter for short referred to as "the Commission") with the grievance that a demand
notice dated 26.8.2002 issued by the Appellant's Wardha office be declared as illegal and
may be set aside and quashed and the respondent company be permitted to avail power
supply to the limit of 90 MVA without recovery of any additional charge either on
account of service connection charges or the service line charges and to further direct the
appellant herein to refund the amount of Rs. 227.9 lakhs so collected for re-instatement of
the contract demand to the original level of 90 MVA along with interest @ 12% from the
date of payment till the date of refund. The respondent company was a consumer of the
Maharashtra State Electricity Board and originally they had a connection of 90 MVA
which was subsequently reduced to 80 MVA and finally to 56 MVA on a request made by
the company. Thereafter again they applied in June, 2002 for enhancement of their
contract demand upto 90 MVA. Their request for enhancement of contract demand upto
90 MVA was granted though it was clearly mentioned in the order dated 12.8.2001 while
reducing the contract demand to 56 MVA that in case enhancement of contract demand
was subsequently required by the respondent company, it would attract payment of
service line and other charges as per then prevailing conditions of supply. However, the
regular supply of 90 MVA was restored on the request of the respondent company. The
supply of 90 MVA was restored in June, 2002 and thereafter a demand was raised in
terms of letter dated 02.08.2001 for service line charges, which was agreed to be paid by
the respondent company, but in installments. Aggrieved against the said order the
respondent-company filed a petition before the Commission on the ground that the
demand of Rs.227.9 lakhs so raised for reinstatement of contract demand of 90 MVA is
not proper. An objection was raised before the Commission that the Commission has no
jurisdiction in the matter in view of Section 42 of the Act and that the consumer should
approach the Consumer Grievance Redressal Forum and thereafter, if still aggrieved, the
Ombudsman created under the Act for redressal of their grievances. The Commission
overruled this objection by making a reference to some decision of the Bombay High
Court and proceeded to assume jurisdiction and directed the refund of the aforesaid
amount to the respondent company.
3. Aggrieved against the said order dated 18th October, 2005, the Maharashtra State
Electricity Distribution Company Ltd. (hereinafter for short "MSEDCL") approached the
Appellate Tribunal for Electricity created under the Act. The Appellate Tribunal vide its
order dated 5th April, 2006 affirmed the order passed by the Commission. Thereafter a
review petition was filed by the MSEDCL before the Appellate Tribunal, which was also
rejected vide order dated 2nd June, 2006. Aggrieved against both these orders, the
MSEDCL has approached this Court by the present appeal under Section 125 of the Act.
4. We have heard learned counsel for the parties and perused the record.
5. The basic question which arises for our consideration in this appeal is whether the
individual consumer can approach the Commission under the Act or not.
6. For deciding this question, the relevant provision is Section 42(5) of the Act, which
reads as under:-
42. Duties of distribution licensees and open access.-
(1) x x x x x x
(2) x x x x x x
(3) x x x x x x
(4) x x x x x x
(5) Every distribution licensee shall, within six months from the appointed date or date of
grant of licence, whichever is earlier, establish a forum for redressal of grievances of the
consumers in accordance with the guidelines as may be specified by the State
Commission."
7. As per the aforesaid provision, if any grievance is made by a consumer, then they have
a remedy under Section 42(5) of the Act and according to sub-section (5) every
distribution licensee has to appoint a forum for redressal of grievances of the consumers.
In exercise of this power the State has already framed The Maharashtra Electricity
Regulatory Commission (Consumer Grievance Redressal Forum and
@page-SC1044
Ombudsman) Regulations, 2003 (hereinafter referred to as "2003 Regulations") and
created Consumer Grievance Redressal Forum and Ombudsman. Under these 2003
Regulations a proper forum for redressal of the grievances of individual consumers has
been created by the Commission. Therefore, now by virtue of sub-section (5) of Section
42 of the Act, all the individual grievances of consumers have to be raised before this
forum only. In the face of this statutory provision we fail to understand how could the
Commission acquire jurisdiction to decide the matter when a forum has been created
under the Act for this purpose. The matter should have been left to the said forum. This
question has already been considered and decided by a Division Bench of the Delhi High
Court in the cases of Suresh Jindal vs. BSES Rajdhani Power Ltd. and Ors. reported in
132 (2006) DLT 339 (DB) and Dheeraj Singh vs. BSES Yamuna Power Ltd. and we
approve of these decisions. It has been held in these decisions that the Forum and
Ombudsman have power to grant interim orders. Thus a complete machinery has been
provided in Section 42(5) and 42(6) for redressal of grievances of individual consumers.
Hence wherever a Forum/Ombudsman have been created the consumers can only resort
to these bodies for redressal of their grievances. Therefore, not much is required to be
discussed on this issue. As the aforesaid two decisions correctly lay down the law when
an individual consumer has a grievance he can approach the forum created under sub-
section (5) of Section42 of the Act.
8. In this connection, we may also refer to Section 86 of the Act which lays down the
functions of the State Commission. Sub-Section (1) (f) of the said Section lays down the
adjudicatory function of the State Commission which does not encompass within its
domain complaints of individual consumers. It only provides that the Commission can
adjudicate upon the disputes between the licensees and generating companies and to refer
any such dispute for arbitration. This does not include in it an individual consumer. The
proper forum for that is Section 42(5) and thereafter Section 42(6) read with Regulations
of 2003 as referred to hereinabove.
9. Therefore, in the facts and circumstances of the present case, we are of the opinion that
the view taken by the Commission as well as the Appellate Authority are unsustainable
and they have erred in coming to the conclusion that the Commission has jurisdiction.
Consequently, we set aside the order dated 18th October, 2005 passed by the Commission
and the orders dated 5th April, 2006 and 2nd June, 2006 passed by the Appellate
Authority and remit the matter to the proper Forum created under Section 42(5) of the
Act to decide the grievance of the respondent herein in accordance with law. We make it
clear that we have not made any observation with regard to the merits of the demand
raised by the appellant upon the respondent company and it will be open for the proper
forum to adjudicate the same. The payment, if any, made by the company will not operate
as an estoppel against the respondent company. We hope that the forum will decide the
matter expeditiously.
10. With the above observations, the appeal is allowed with no order as to costs.
Appeal allowed.
AIR 2008 SUPREME COURT 1044 "Kanhaiyalal v. Union of India"
(From : Madhya Pradesh)*
Coram : 2 ALTAMAS KABIR AND B. SUDERSHAN REDDY, JJ.
Criminal Appeal No. 788 of 2005, D/- 9 -1 -2008
Kanhaiyalal v. Union of India.
Narcotic Drugs and Psychotropic Substances Act (61 of 1985), S.53, S.67, S.42 -
NARCOTIC DRUGS - CONFESSION - INVESTIGATION - Statement by accused
under S. 67 - Is not same as statement made under S. 161 Cr. P. C. - Can be used as
confession against him - Is excluded from operation of Ss. 24 to 27 of Evidence Act -
Conviction of accused can be based on it - Moreso, when it is corroborated by other
evidence.
Evidence Act (1 of 1872), S.25.
An officer vested with the powers of an officer-in-charge of a Police Station under S. 53
of N. D. P. S. Act is not a "Police Officer" within the meaning of S. 25 of the Evidence
Act. Thus, it is clear that a statement made under S. 67 of N. D. P. S. Act is not the same
as a statement made under Section 161 of Cr. P. C., unless made under threat or coercion.
It is this vital difference, which allows a statement made under S. 67 of the N. D. P. S. Act
to be used as a
@page-SC1045
confession against the person making it and excludes it from the operation of Ss. 24 to 27
of the Evidence Act. In the instant case there is nothing on record to suggest that the
appellant accused was compelled under threat to make the statement after he had been
placed under arrest which renders such statement inadmissible and not capable of being
relied upon in order to convict him. On the other hand, there is the evidence of
prosecution officer upon which the High Court has relied in convicting the appellant. No
question in cross-examination had been put to said prosecution witness in this regard and
the version of the said witness must be accepted as corroborative of the statement made
by the accused.
AIR 1991 SC 45, Followed. (Paras 38, 39)
Though an application was made for retracting the confession made by the appellant,
neither was any order passed on the said application nor was the same proved during the
trial so as to water down the evidentiary value of the said statement. On the other hand, in
the absence of such evidence on record, the High Court had no option but to proceed on
the basis of the confession as made by the appellant under S. 67 of the N.D.P.S. Act.
Since an officer for the purposes of S. 67 of the N.D P.S. Act read with S. 42 thereof, is
not a police officer, the bar under Sections 24 and 27 of the Evidence Act cannot be
attracted and the statement made by a person directed to appear before the officer
concerned may be relied upon as a confessional statement against such person. Since a
conviction can be maintained solely on the basis of a confession made under S. 67 of the
N.D.P.S. Act, the conviction of the accused on that basis was proper. (Para 40)
Cases Referred : Chronological Paras
2005 AIR SCW 4148 : AIR 2005 SC 3820 : 2005 Cri LJ 3950 (Ref.) 26
2004 AIR SCW 4930 : AIR 2004 SC 4197 : 2004 Cri LJ 4197 (Ref.) 25
2003 AIR SCW 4975 : AIR 2003 SC 4311 : 2003 Cri LJ 4996 (Ref.) 31
(2001)10 SCC 203 (Ref.) 28, 31
1999 AIR SCW 2457 : AIR 1999 SC 2355 : 1999 Cri LJ 3663 (Ref.) 29, 31, 36
AIR 1991 SC 45 : 1991 Cri LJ 97 37, 38
(1971) 3 SCC 950 (Ref.) 26
AIR 1968 SC 832 : 1968 Cri LJ 1017 (Ref.) 26
AIR 1964 SC 1184 : 1964 Cri LJ 344 (Ref.) 30
AIR 1963 SC 1094 : 1963 Cri LJ 178 (Ref.) 26, 27, 37
AIR 1961 SC 1808 : 1961 (2) Cri LJ 856 (Ref.) 34
AIR 1954 SC 4 : 1954 Cri LJ 236 (Ref.) 23
AIR 1953 SC 459 : 1953 Cri LJ 1925 (Ref.) 24
S. K. Gambhir, Sr. Advocate, T. N. Singh, Anil Sharma, A. K. Sinha, V. K. Singh and B.
K. Sharma, with him for Appellant; B. B. Singh, Ms. Binu Tamta, D. S. Mehra and Ms.
Sushma Suri, for Respondent.
* Cri. A. No. 108 of 2003, D/- 18-3-2005 (MP) (Indore Bench)
Judgement
ALTAMAS KABIR, J. :- The appellant herein, along with one Phool Chand and Ram
Prasad, was accused of offences under the Narcotic Drugs and Psychotropic Substances
Act, 1985 (hereinafter referred to as the NDPS Act) and ultimately charges were framed
against them by the Special Judge under Section 8/18 and in the alternative under Section
8/18/29 of the aforesaid Act. On denying the charges framed against them the accused
persons were sent to trial.
2. The learned Special Judge framed several issues and ultimately held that the charges
had been fully proved against the accused Phool Chand, from whose possession 19 kg
200 gms. of opium was seized. Phool Chand was found guilty and convicted under
Section 8/18 of the NDPS Act and sentenced to suffer 10 years R.I. together with fine of
Rs.1 lakh, in default, to undergo 2 years R.I. separately.
3. As far as the appellant herein and Ram Prasad are concerned, the learned Trial Judge
found that the charges against them had not been proved and acquitted them of the
charges under Section 8/18 read with Section 29 of the NDPS Act.
4. Aggrieved by his conviction and sentence, Phool Chand preferred Criminal Appeal
No.47 of 2002 before the Indore Bench of the Madhya Pradesh High Court. On the other
hand, the Union of India also preferred Criminal Appeal No.108 of 2003 against the
acquittal of Ram Prasad and the appellant herein.
5. The High Court heard both the appeals together and ultimately dismissed the appeal
preferred by Phool Chand and relying on the statement made by Ram Prasad and the
appellant herein under Section 67 of the NDPS Act, found them also guilty of the charges
framed against them and
@page-SC1046
allowed the appeal filed by the Union of India. Ram Prasad and Kanhaiyalal (the
appellant herein) were also sentenced to 10 years R.I. each and fine of Rs. 1 lakh each,
and in default, to further undergo a jail sentence of 6 months R.I.
6. It is against the said judgment of the High Court that the instant appeal had been
preferred by Kanhaiyalal.
7. Since the appellant, Kanhaiyalal was convicted on the basis of the statement made by
him under Section 67 of the NDPS Act, a question has been raised whether such
statement made to an officer within the meaning of Section 42 of the said Act could be
treated as a confessional statement and whether the accused could be convicted on the
basis thereof in the absence of any other corroborative evidence.
8. As will appear from the records, the case of the prosecution was that on 22.2.1997 an
information had been received by Shri Rajesh Nagpal, Assistant Narcotics Commissioner
of the Central Bureau of Narcotics, Neemuch, that accused Phoolchand and Ram Prasad
were involved in illegal dealing in opium and they had entered into an agreement to buy
25 kgs of opium from the appellant and the delivery of the said contraband was to be
made at the site of the well of accused Ram Prasad situated in Haspur. On receipt of the
said information, Shri Nagpal went to his office and reported the same. Subsequently, the
same was produced before the Deputy Commissioner, Shri Prem Raj. On the basis of the
said information a preventive party was constituted which proceeded to the identified
site. The raiding party purportedly reached village Hadipiplya Police Station, Manasa, by
a Government vehicle at about 3.00 p.m. on the same day and after calling two
independent witnesses, Madan Lal and Ram Rattan, who belonged to Hadipatiya, the
raiding party proceeded towards the well in question belonging to Ram Prasad's father
Mangi Lal. The raiding party led by Lala Ram Dinkar, Inspector, along with the said two
witnesses reached the said well at about 4.00 p.m. and found two persons sitting there
with 3 bags. On seeing the raiding/preventive party, one of the two persons sitting there
ran away and although he was chased by some members of the raiding party, they did not
manage to catch him. Inspector Lal Ram Dinkar went up to the other person sitting near
the well who disclosed his name as Phool Chand son of Sita Ram and resident of
Bardiya. According to the prosecution case, on being asked as to who was the person who
had fled, Phool Chand indicated the name of the appellant herein. On being further asked
as to the ownership of the bags lying at the spot, accused Phool Chand admitted that the
bags belonged to him and to the appellant herein.
9. Thereafter, Phool Chand was told about the confidential information that had been
received and that the raiding party had reasons to believe that the bags in question
contained intoxicating materials. He was also informed that for such reasons he would
have to be searched along with the bags. In keeping with the provisions of Section 50 of
the NDPS Act Phool Chand was given the option of the search being conducted by any
nearby Magistrate or before any Gazetted Officer. According to the prosecution, Phool
Chand was willing to have the bag searched before the Gazetted Officer. Accordingly,
Shri R. K. Sinha, Superintendent of the Narcotics Department, who was present with the
raiding/preventive party introduced himself as a competent Gazetted Officer to Phool
Chand who gave his consent in writing about such search in the Panchnama which was
drawn before Shri R.K. Sinha. Pursuant to the above, search of the bags was undertaken
from which the contraband opium was found and on measurement the bags were found to
contain the following amounts of opium :-

i) Bag A 11 kgs. 500 gms.


ii) Bag B 4 kgs. 700 gms, and
iii) Bag C 300 gms

From each of the bags two samples of 25 grams each were taken out for chemical
examination while the remaining opium was sealed under the Panchnama (Exh.P.2)
10. On the basis of the proceeding in terms of Section 57 of the NDPS Act and the First
Information Report filed before the Narcotic Superintendent, one Suresh Badlani was
appointed as Investigating Officer and the seized opium along with samples were
deposited in the Malkhana. At this point of time the statement of accused Phool Chand
was taken where he confessed to the offence complained of.
11. During investigation, accused Ram Prasad and Kanhaiyalal (appellant herein) were
served with summons under Section 67 of the NDPS Act. While Ram Prasad was
@page-SC1047
present in the Neemuch office on 24.2.1997 in pursuance of the summons, the appellant
herein did not appear before the concerned officer. Accordingly, charge-sheet was filed
before the Special Judge under the NDPS Act against Phool Chand and Ram Prasad on
20.5.1997. Subsequently, after fresh summons were issued to the appellant herein, he also
appeared before the concerned officer and his statement was taken under Section 67 of
the NDPS Act. In his statement the appellant confessed that he too was involved in the
smuggling of opium with the co-accused. According to the prosecution, on the basis of
his confessional statement, Kanhaiyalal was also arrested in connection with the offence
and supplementary challan was filed against him before the Special Judge on 7.8.1997.
12. The defence of the accused was that they have been falsely implicated and accused
Phool Chand took a specific plea that on 21.2.1997 he was playing Kabbadi at the sports
ground.
13. The Trial Judge framed 4 questions for the purpose of deciding the case, namely, :-
i) Whether seized material in the case is opium?
ii) Whether aforesaid opium seized illegally kept in possession of accused Phool Chand
in village Hadipipliya at about 4 oclock on 22.2.1997 i.e. the day of incident?
iii) Whether aforesaid opium was collected for sale by all the three accused Ram Prasad
and Kanhaiya Lal with co-accused Phool Chand in co-partnership ? and
iv) Whether in this case compliance of necessary legal provisions of NDPS Act is done ?
14. The first question was answered by the learned Trial Judge in the affirmative upon
holding that the seized material was proved to be opium.
15. On the second question, the Trial Judge came to the definite finding that it had been
fully proved that the opium had been seized from the possession of accused Phool Chand
for which he did not have any valid licence.
16. The answer to the third question, which is relevant to this appeal, was answered in
favour of the accused Ram Prasad and the appellant herein and the Special Judge
concluded that except for the contradictory confession of these two accused there was no
other corroborative evidence and the prosecution had failed to prove that they were
included in the smuggling operation.
17. As far as the fourth question is concerned the Special Judge was satisfied that all the
provisions of the NDPS Act had been duly complied with.
18. On the basis of his findings on the first, second and fourth questions, the Special
Judge under the NDPS Act convicted Phool Chand of the offences under Section 8/18 of
the NDPS Act, but acquitted both Ram Prasad and the appellant herein of the said charge
on his findings with regard to question No.3.
19. As mentioned hereinabove, the High Court while dismissing the appeal filed by Phool
Chand allowed the appeal filed by the Union of India against the acquittal of Ram Prasad
and Kanhaiyalal upon holding that the statement made by Ram Prasad and the appellant
herein under Section 67 of the NDPS Act did not require any corroboration and were
sufficient in themselves to convict the said two accused.
20. Mr. S.K. Gambhir, learned Senior Advocate, contended on behalf of the appellant,
Kanhaiyalal, that the High Court had incorrectly stated the law regarding statements
made under Section 67 of the NDPS Act before officers empowered under Section 42
thereunder. It was his specific case that once the appellant had been summoned in an
inquiry under Section 67 of the aforesaid Act and was placed under arrest, any statement
made by him thereafter would be hit by the provisions of Sections 24 to 27 of the Indian
Evidence Act, 1872. Apart from the above, Mr. Gambhir also submitted that after making
the statement in terms of Section 67 of the NDPS Act the appellant had retracted such
statement and in the absence of corroborative evidence, the said retracted
statement/confession could not be relied upon in order to convict the appellant.
Furthermore, there was no independent evidence to corroborate the retracted confession,
which fact had weighed with the trial court in acquitting the appellant.
21. Mr. Gambhir submitted that although from the arrest Memo it would be clear that
Kanhaiyalal was arrested on 8.6.1997 at 5.30 p.m., he was produced before the
Magistrate on 9th June, 1997, and on the same day he made an
@page-SC1048
application in writing to the Court that his signature had been forcibly obtained on blank
papers under threat that if he did not sign he would be involved in other serious cases and
the same were subsequently used for preparing statements under Section 67 of the
aforesaid Act as if the same had been voluntarily made by him. Mr. Gambhir submitted
that the appellant had already been arrested and detained in custody when the statement
under Section 67 of the NDPS Act was recorded and, accordingly the same came within
the mischief of Sections 24 to 27 of the Evidence Act. Mr. Gambhir pointed out that since
the Trial Court had taken cognizance of the said application and recorded a finding in the
judgment itself that Kanhaiyalal had retracted his confession, it would be incorrect to say
that the said application made by Kanhaiyalal had not been considered by the Court. It
was also pointed out by Mr. Gambhir that although on behalf of the prosecution it had
been submitted that Kanhaiyalal in his statement under Section 313 of the Criminal
Procedure Code had stated that he had been compelled to sign on blank papers under
threat, the truth was otherwise since in his statement under Section 313 Kanhaiyalal
answered galat hai which had to be taken as a denial that he had given such statement.
Mr. Gambhir submitted that the appellants positive response had to be read along with his
application dated 9.6.1997 retracting his confessional statement.
22. Mr. Gambhir submitted that the High Court had erred in relying upon the appellants
statement made under Section 67 of the NDPS Act, although, not only had the same been
retracted immediately thereafter before the learned Magistrate, but the same was not
admissible under the above-mentioned provisions of the Evidence Act. It was submitted
that since apart from the above statement there was no other evidence, which linked the
appellant with the alleged offence, the High Court should have maintained the judgment
of the acquittal passed by the learned trial Court.
23. In support of his aforesaid submission that in the absence of other corroborating
evidence the retracted confession had been wrongly relied upon by the High Court to
convict the appellant, Mr. Gambhir referred to the three Judge Bench decision of this
Court in Muthuswami vs. State of Madras (AIR 1954 SC 4) in which, it was indicated
that no hard and fast rule could be laid down regarding the necessity of corroboration in
the case of a retracted confession in order to base conviction thereupon. But apart from
the general rule of prudence, if the circumstances of a particular case raised doubts as to
the genuineness of a confession, it would be sufficient to require corroboration of a
retracted confession.
24

. In Puran vs. State of Punjab (AIR 1953 SC 459) the same view has been expressed as
follows :- Para 9 of AIR

"It is a settled rule of evidence that unless a retracted confession is corroborated in


material particulars, it is not prudent to base a conviction in a criminal case on its strength
alone."
25

. The same view was also expressed by this Court in Parmananda Pegu vs. State of Assam
(2004 (7) SCC 779), which involved a conviction made on the basis of a confession made
before a Judicial Magistrate in accordance with Section 164 of the Code of Criminal
Procedure. 2004 AIR SCW 4930

26

. In the same context Mr. Gambhir strongly relied on the decision of this Court in State
(NCT of Delhi) vs. Navjot Sandhu 2005 (11) SCC 600) (commonly known as the
Parliament Attack case)wherein while hearing several appeals, this Court had occasion to
go into the question of confessions and retracted confessions in some detail. Referring to
confessions in general, this Court made a distinction between "confession" and
"admission". It observed that "an admission is a statement oral or documentary which
enables the Court to draw an inference as to any fact in issue or relevant fact. It is trite to
say that every confession must necessarily be an admission, but, every admission does
not necessarily amount to a confession". This Court also cautioned that before acting
upon a confession, the Court has to satisfy itself that it was freely and voluntarily made,
having regard to the language of Section 24 of the Evidence Act. However, while
examining the evidentiary value of a retracted confession against the confessor, the
learned Judges had occasion to consider three previous decisions of this Court in Bharat
vs. State of U.P. (1971 3 SCC 950) and Haroon Hazi Abdulla vs. State of Maharashtra
(1968 2 SCR 641) and Pyare Lal Bhargava vs. State 2005 AIR SCW 4148
AIR 1968 SC 832
AIR 1963 SC 1094

@page-SC1049
of Rajasthan (1963 Suppl.1 SCR 689). The learned Judges extracted paragraph 7 of the
judgment in Bharat vs. State of U.P. (supra) wherein a comparison has been made
between "confession" and "retracted confession". While in the former case, it was
observed that confessions could be acted upon, if the Court was satisfied that they were
voluntarily made and they were true, retracted confessions, stood on a slightly different
footing. In that context, it was observed that a Court may take into account the retracted
confession, but it must look for the reasons for the making of the confession as well as for
its retraction, and must weigh the two, to determine whether the retraction affects the
voluntary nature of the confession or not". The learned Judges of the three Judge Bench
went on to observe that upon being satisfied, it was for the court to decide whether to use
the retracted confession or not, but all the same, the courts did not normally act upon a
retracted confession without finding some other evidence as to the guilt of the accused.
The learned Judges concluded that a true confession voluntarily made could be acted
upon with slight evidence to corroborate the same, but a retracted confession requires the
Court to be assured that the retraction was an after-thought and that the earlier statement
was true.
27

. Similar views were expressed in the other two cases referred to hereinabove, but it
would be profitable to reproduce the views of the four Judge Bench in Pyare Lal
Bharagava's case (supra) which has been reproduced in Navjot Sandhu's case, in the
following terms :- AIR 1963 SC 1094
2005 AIR SCW 4148, Para 8

"A retracted confession may form the legal basis of a conviction if the court is satisfied
that it was true and was voluntarily made. But it has been held that a Court shall not base
a conviction on such a conviction without corroboration. It is not a rule of law, but is only
rule of Prudence. It cannot even be laid down as an inflexible rule of practice of prudence
that under no circumstances can such a conviction be made without corroboration, for a
court may, in a particular case, be convinced of the absolute truth of a confession and
prepared to act upon it without corroboration; but it may be laid down as a general rule of
practice that it is unsafe to rely upon a confession much less on a retracted confession,
unless the court is satisfied that the retracted confession is true and voluntarily made and
has been corroborated in material particulars."
28. While the above mentioned decisions dealt with other criminal enactments, the next
case referred to by Mr. Gambhir, namely, A. K. Mehaboob vs. Intelligence Officer,
Narcotics Control Bureau, (2001 (10) SCC 203) is a decision under the NDPS Act with
due regard to the provisions of Sections 42 and 67 thereof. The criminal appeal of Shri A.
K. Mehaboob was heard alongwith the appeal filed by Shri P.K. Naushad, who were the
second and third accused facing charges for having committed an offence under Sections
21 and 29 of the NDPS Act. While the first accused, Divakaran, was found guilty and
convicted on the strength of recovery of brown sugar from him, the two appellants before
this Court were acquitted by the trial court but were convicted by the High Court in
appeal preferred by the Narcotics Control Bureau. From the facts of the said case, it
appears that apart from the statement made by Naushad under Section 67 of the NDPS
Act there was other evidence to indicate that business in Narcotic drugs was being
transacted from his house. His appeal was therefore rejected. As far as Mehaboob was
concerned, his statement did not contain any statement, which could involve him in the
offence. The High Court, therefore, allowed the appeal filed by Mehaboob and set aside
his conviction.
29

. On this line of decisions, Mr. Gambhir lastly referred to a decision of this Court in Pon
Adhithan vs. Deputy Director, Narcotics Control Bureau, Madras (1999 (6) SCC 1)
wherein, in fact, the confessional statement made by the accused-appellant while in
custody of the Intelligence Officer, Narcotics Intelligence Bureau, was relied upon to
convict the said accused, on it being held that the said statement had been voluntarily
made as no complaint of threat or pressure had been made by the accused when he was
produced before the Magistrate. Mr. Gambhir sought to distinguish the said decision on
the ground that while in Pon Adhithan's case, no complaint had been made by the accused
of any threat or compulsion for making such statement, in the present case the confession
made by the appellant herein was immediately retracted on the very next date when the
appellant was produced before the 1999 AIR SCW 2457

@page-SC1050
Magistrate and that, too, in writing by way of an application. Mr. Gambhir reiterated that
the said application was referred to by the learned trial court in its judgment. Mr.
Gambhir also submitted that since the trial court had chosen not to rely on the statement
made by the appellant under Section 67 of the NDPS Act, in the absence of any
corroborating evidence, the evidentiary value of the retracted confession had not been
gone into by the learned trial Court.
30

. Reliance was also placed by Mr. Gambhir on the Constitution Bench decision in
Haricharan Kurmi vs. State of Bihar (1964 (6) SCR 623) in support of his submission that
apart from the statement of Phoolchand, the main accused, there was no other evidence to
connect the appellant with the alleged offence under the NDPS Act. In the said case, the
Constitution Bench, held that though a confession within the meaning of Section 30 of
the Indian Evidence Act, cannot strictly speaking be treated as evidence as defined in
Section 3, it is an element which may be taken into consideration by the criminal courts
but the court cannot start with the confession of a co-accused person, but must begin with
other evidence adduced by the prosecution before relying on the confession of the co-
accused. AIR 1964 SC 1184
31

. On behalf of the Union of India, Mr. B.B. Singh, learned Advocate, submitted that
having regard to the decisions of this Court in Pon Adithan vs. Dy. Director Narcotics
Bureau, (supra), A.K. Mehaboob vs. Intelligence Officer, Narcotics, (supra) and
M.Prabhulal vs. The Assistant Director, Directorate of Revenue Intelligence (2003) 8
SCC 449, the appellant had been rightly convicted on his confessional statement made
under Section 67 of the NDPS Act alongwith the evidence of PW 9 Suresh Badlani,
Inspector in the office of the Deputy Narcotics Commissioner, Lucknow, U.P., before
whom such statement had been made. Mr. Singh also submitted that even if the person
was in custody at the time when he made such confessional statement, the same would
not attract the bar under Article 20(3) of the Constitution, if it was made voluntarily. Mr.
Singh also emphasised that in all the three cases referred to hereinabove the accused were
in custody when their confessional statements were recorded under Section 67 of the
NDPS Act. 1999 AIR SCW 2457
2003 AIR SCW 4975

32. Mr. Singh further submitted that although PW 9 had deposed that the confessional
statement had been made by the appellant on 8.6.97 before him in the office of the
Deputy Commissioner, there was no cross-examination on this point on behalf of the
appellant and consequently the confessional statement of the appellant had to be
accepted.
33. On the issue involving retraction by the appellant of his statement made before the
Inspector under Section 67 of the NDPS Act, Mr. Singh submitted that the application
which the appellant is said to have filed before the Magistrate was never proved or
exhibited in the Trial Court and did not form part of the evidence on record. He submitted
that the same could not, therefore, be taken note of or be relied upon in support of the
appellants case of retracted confession.
34

. In support of his aforesaid submission Mr. Singh referred to the decision of the eleven
Judge Bench of this Court in State of Bombay vs. Kathi Kalu Oghad and others (1962) 3
SCR 10 wherein it was, inter alia, concluded that the accused person cannot be said to
have been compelled to be a witness against himself simply because he made a statement
while in police custody, without anything more. In other words, just being in Police
custody when the statement was made would not, by itself, give rise to an inference that
the accused had been compelled to make such statement. It was also held that to bring the
statement within the prescription of Article 20(3), the person accused must have stood in
the character of an accused person at the time he made the statement. It is not enough that
he should become an accused any time after the statement had been made. AIR 1961 SC
1808

35. From the facts of the case and the submissions made on behalf of the respective
parties the point which emerges for determination is upto what extent can a statement
under Section 67 of the NDPS Act be relied upon for convicting a person accused of
having committed an offence under the provisions of the said Act. In order to arrive at a
decision in regard to the above, it will also have to be considered whether such a
statement would attract the bar both of Sections 24 to 27 of the Indian Evidence Act as
also Article 20(3) of the Constitution of India. For the aforesaid purpose, the
@page-SC1051
provisions of Section 67 of the N.D.P.S. Act are reproduced hereinbelow :-
"67. Power to call for information, etc. Any officer referred to in Section 42 who is
authorised in this behalf by the Central Government or a State Government may, during
the course of any enquiry in connection with the contravention of any provisions of this
Act,-
(a) call for information from any person for the purpose of satisfying himself whether
there has been any contravention of the provisions of this Act or any rule or order made
thereunder;
(b) require any person to produce or deliver any document or thing useful or relevant to
the enquiry;
(c) examine any person acquainted with the facts and circumstances of the case.
36

. A parallel may be drawn between the provisions of Section 67 of the NDPS Act and
Sections 107 and 108 of the Customs Act and to a large extent Section 32 of the
Prevention of Terrorism Act, 2002 and Section 15 of the Terrorist and Disruptive
Activities (Prevention) Act, 1987. These are all special Acts meant to deal with special
situations and circumstances. While the provisions of the Prevention of Terrorism Act,
2002, and TADA Act, 1987, are much more stringent and excludes from its purview the
provisions of Sections 24 to 27 of the Evidence Act with regard to confession made
before a police officer, the provisions relating to statements made during inquiry under
the Customs Act and under the NDPS Act are less stringent and continues to attract the
provisions of the Evidence Act. In the case of both the latter enactments, initially an
inquiry is contemplated during which a person may be called upon to provide any
information relevant to the inquiry as to whether there has been any contravention of the
provisions of the Act or any Rule or Order made thereunder. At that stage the person
concerned is not an accused although he may be said to be in custody. But on the basis of
the statements made by him he could be made an accused subsequently. What is
important is whether the statement made by the person concerned is made during inquiry
prior to his arrest or after he had been formally charged with the offence and made an
accused in respect thereof. As long as such statement was made by the accused at a time
when he was not under arrest, the bar under Sections 24 to 27 of the Evidence Act would
not operate nor would the provisions of Article 20(3) of the Constitution be attracted. It is
only after a person is placed in the position of an accused that the bar imposed under the
aforesaid provision will come into play. Of course, this Court has also held in Pon
Adithans case (supra) that even if a person is placed under arrest and thereafter makes a
statement which seeks to incriminate him, the bar under Article 20(3) of the Constitution
would not operate against him if such statement was given voluntarily and without any
threat or compulsion and if supported by corroborating evidence. 1999 AIR SCW 2457

37
. The law involved in deciding this appeal has been considered by this Court from as far
back as in 1963 in Pyare Lal Bhargava's case (supra). The consistent view which has been
taken with regard to confessions made under provisions of Section 67 of the NDPS Act
and other criminal enactments, such as the Customs Act, 1962, has been that such
statements may be treated as confessions for the purpose of Section 27 of the Evidence
Act, but with the caution that the Court should satisfy itself that such statements had been
made voluntarily and at a time when the person making such statement had not been
made an accused in connection with the alleged offence. In addition to the above, in the
case of Raj Kumar Karwal v. Union of India and others (1990) 2 SCC 409, this Court
held that officers of the Department of Revenue Intelligence who have been vested with
powers of an Officer-in-Charge of a police station under Section 53 of the NDPS Act,
1985, are not "police officers" within the meaning of Section 25 of the Evidence Act.
Therefore, a confessional statement recorded by such officer in the course of
investigation of a person accused of an offence under the Act is admissible in evidence
against him. It was also held that power conferred on officers under the NDPS Act in
relation to arrest, search and seizure were similar to powers vested on officers under the
Customs Act. Nothing new has been submitted which can persuade us to take a different
view. AIR 1963 SC 1094
AIR 1991 SC 45

38

. Considering the provisions of Section 67 of the N.D.P.S. Act and the views expressed by
this Court in Raj Kumar Karwals case (supra), AIR 1991 SC 45

@page-SC1052
with which we agree, that an officer vested with the powers of an Officer-in-Charge of a
Police Station under Section 53 of the above Act is not a "Police Officer" within the
meaning of Section 25 of the Evidence Act, it is clear that a statement made under
Section 67 of the N.D.P.S. Act is not the same as a statement made under Section 161 of
the Code, unless made under threat or coercion. It is this vital difference, which allows a
statement made under Section 67 of the N.D.P.S. Act to be used as a confession against
the person making it and excludes it from the operation of Sections 24 to 27 of the
Evidence Act.
39. There is nothing on record to suggest that the appellant was compelled under threat to
make the statement after he had been placed under arrest which renders such statement
inadmissible and not capable of being relied upon in order to convict him. On the other
hand, there is the evidence of PW9 upon which the High Court has relied in convicting
the appellant. It may once again be mentioned that no question in cross-examination had
been put to PW9 in this regard and the version of the said witness must be accepted as
corroborative of the statement made by the accused.
40. It may also be recalled that though an application was made for retracting the
confession made by the appellant, neither was any order passed on the said application
nor was the same proved during the trial so as to water down the evidentiary value of the
said statement. On the other hand, in the absence of such evidence on record, the High
Court had no option but to proceed on the basis of the confession as made by the
appellant under Section 67 of the NDPS Act. Since it has been held by this Court that an
officer for the purposes of Section 67 of the NDPS Act read with Section 42 thereof, is
not a police officer, the bar under Sections 24 and 27 of the Evidence Act cannot be
attracted and the statement made by a person directed to appear before the officer
concerned may be relied upon as a confessional statement against such person. Since a
conviction can be maintained solely on the basis of a confession made under Section 67
of the NDPS Act, we see no reason to interfere with the conclusion of the High Court
convicting the appellant.
41. We, accordingly, dismiss the appeal and maintain the order of conviction and sentence
passed against the appellant by the High Court.
Appeal dismissed.
AIR 2008 SUPREME COURT 1052 "State of A.P. v. A. S. Peter"
(From : Andhra Pradesh)
Coram : 2 S. B. SINHA AND LOKESHWAR SINGH PANTA, JJ.
Criminal Appeal No. 1119 of 2004, D/- 13 -12 -2007
State of A.P. v. A. S. Peter.
Criminal P.C. (2 of 1974), S.173, S.36 - INVESTIGATION - CHARGE SHEET -
POLICE OFFICERS - Further investigation - Law does not mandate taking of prior
permission - Carrying out of further investigation even after filing of charge sheet is a
statutory right of police.
Cri. Petn. No. 3955 of 2000, D/- 3-10-2002 (AP), Reversed.
A charge-sheet was filed upon completion of the investigation in the Court of Magistrate,
for alleged commission of offence under Sections 199, 200 and 204 of the Indian Penal
Code. As allegations had been made against the officer of a local police station in regard
to the mode and manner in which investigation was carried out, a further investigation
was directed. The Court was informed thereabout. Although no express permission was
granted, but evidently, such a permission was granted by necessary implication as further
proceeding was stayed by the Magistrate. It was not a case where two charge-sheets were
filed before two different Courts. The Court designated to deal with the matters wherein
investigation had been carried out by the CID, was located at Chittoor. Indisputably, the
law does not mandate taking of prior permission from the Magistrate for further
investigation. Carrying out of a further investigation even after filing of the charge-sheet
is a statutory right of the police. A distinction also exists between further investigation
and re-investigation. Whereas re-investigation without prior permission is necessarily
forbidden, further investigation is not. It is not correct to contend that the investigation
was taken up by a different agency. The CID is a part of the investigation authorities of
the State. Section 36 of the Criminal P. C., empowers a police officer, superior in rank to
an officer in charge of a police station, to exercise the same powers throughout the local
area to which they are appointed, as
@page-SC1053
may be exercised by such officer within the limits of his station.
AIR 1961 SC 1117; AIR 1980 SC 326; 2004 AIR SCW 5017, Relied on.
1998 AIR SCW 1852, Disting.
Cri. Petn. No. 3955 of 2000, D/- 3-10-2002 (AP), Reversed. (Paras 5, 7, 12, 14)
Cases Referred : Chronological Paras
2004 AIR SCW 5017 : AIR 2004 SC 4320 : 2004 Cril LJ 4219 (Rel. on) 9
1998 AIR SCW1852 : AIR 1998 SC 2001 : 1998 Cri LJ 2897 (Disting.) 3, 4, 13
AIR 1980 SC 326 : 1980 Cri LJ 98 (Rel. on) 9
AIR 1979 SC 1791 : 1979 Cri LJ 1346 3, 4, 10
AIR 1961 SC 1117 : 1961 (2) Cri LJ 161 (Rel. on) 6
Mrs. D. Bharathi Reddy, for Appellant; Nagendra Rai, Sr. Advocate, A. V. Rao, Parnam
Prabhakar and Venkateswara Rao Anumolu, with him for Respondent.
Judgement
S. B. SINHA, J :- The State of Andhra Pradesh is before us aggrieved by and dissatisfied
with a judgment and order dated 3.10.2002 passed by the High Court of Judicature of
Andhra Pradesh in Criminal Petition No. 3955 of 2000 allowing a criminal revision
application filed by the respondent herein.
2. Respondent (Accused No. 1) carries on business in Red Sanders hard wood and was
having a godown at Renigunta in the District of Chittoor. A fire broke out in the said
godown on 28/29.06.1996 resulting in destruction of red sanders hard wood, undressed
wood as also nine cutting machines.
The said godown was insured. The concerned Forest Officer gave an information to the
police station that the respondent had made a false declaration regarding the stock shown
in the godown and inflated the same in order to make unlawful gain, whereupon a First
Information Report was lodged. Investigation was carried out upon obtaining permission
of the concerned Magistrate. A charge-sheet was filed upon completion of the
investigation in the Court of III Additional Munsif Magistrate, Tirupati for alleged
commission of offence under Sections 199, 200 and 200 of the Indian Penal Code.
Subsequently, however, as some allegations had been made against the manner in which
the local police conducted the investigation, the Additional Director General of Police,
CID entrusted the case for further investigation to the Inspector of Police, CID, Prakasam
District on 5.11.1997.
Before carrying out the said investigation, the Inspector of Police, CID filed a memo in
the said Court with the prayer that the matter be adjourned. Although it does not appear
that any express permission was granted for carrying out further investigation, the prayer
of adjournment was allowed in terms of the said Memo. Further investigation was carried
out whereafter an additional charge-sheet was filed against Accused Nos. 1 to 3 in the
Court of IV Additional Munsif Magistrate, Chittoor for offences under Sections 199, 200,
204 and 120 of the Indian Penal Code. More accused persons were also added in the
charge-sheet in the category of accused. Indisputably, the case was transferred from the
Tirupati Court to a Designated Court at Chittoor.
Appellant filed an application before the High Court of Andhra Pradesh for quashing of
the criminal proceedings inter alia on the ground that prior permission of the Magistrate
was not obtained for further investigation as also on the ground that the same was
conducted entirely by a different investigating agency.
A learned Single Judge of the High Court by reason of the impugned order has allowed
the said application.
3

. Mrs. D. Bharathi Reddy, learned counsel appearing on behalf of the appellant in support
of this appeal submitted that the High Court committed a manifest error in taking the
view that the investigation in question was a fresh investigation or it was imperative on
the part of the investigating officer to obtain express permission from the Magistrate
concerned. Decisions of this Court in Ram Lal Narang v. State (Delhi Administration)
[(1979) 2 SCC 322] and K. Chandrasekhar v. State of Kerala and Others [(1998) 5 SCC
223] whereupon the High Court relied upon, Mrs. Reddy, would contend, have no
application to the facts of the present case. AIR 1979 SC 1791
1998 AIR SCW 1852

4. Mr. Nagendra Rai, learned Senior counsel appearing on behalf of the respondent, on
the other hand, submitted that in view of the fact that not only a re-investigation was
conducted by a different investigating agency, even another case was
@page-SC1054
instituted at a different place without obtaining the prior permission of the Magistrate
concerned and, thus, the impugned judgment is unassailable in view of the decisions of
this Court in Ram Lal Narang (supra) and K. Chandrasekhar (supra).
5. Indisputably, the law does not mandate taking of prior permission from the Magistrate
for further investigation. Carrying out of a further investigation even after filing of the
chargesheet is a statutory right of the police. A distinction also exists between further
investigation and re-investigation. Whereas re-investigation without prior permission is
necessarily forbidden, further investigation is not.
6

. In R. P. Kapur and Others v. Sardar Pratap Singh Kairon and Others [1961 (2) SCR
143], this Court laid down the law in the following terms : AIR 1961 SC 1117, Para 10

"..........The Additional Inspector General of Police to whom Sethi's complaint was sent
was, without doubt, a police officer superior in rank to an officer in charge of a police
station. Sardar Hardayal Singh, Deputy Superintendent of Police, CID, Amritsar, was also
an officer superior in rank to an officer in charge of a police station. Both these officers
could, therefore, exercise the powers, throughout the local area to which they were
appointed, as might be exercised by an officer in charge of a police station within the
limits of his police station............"
It was further held :
"..........If the police officer concerned thought that the case should be investigated by the
C.I.D. - even though for a reason which does not appeal to us - it cannot be said that the
procedure adopted was illegal................"
7. It is not correct to contend that the investigation was taken up by a different agency.
The CID is a part of the investigating authorities of the State. A further investigation was
directed by the Additional Director General of Police. Section 36 of the Code of Criminal
Procedure, 1973 empowers a police officer, superior in rank to an officer in charge of a
police station, to exercise the same powers throughout the local area to which they are
appointed, as may be exercised by such officer within the limits of his station.
8. It was, therefore, permissible for the higher authority to carry out or direct further
investigation in the matter.
9
. This aspect of the matter is covered by a decision of this Court in State of Bihar and
Another v. J.A.C. Saldanha and Others [(1980) 1 SCC 554], wherein it was held: AIR
1980 SC 326

"19. ..........This provision does not in any way affect the power of the investigating
officer to further investigate the case even after submission of the report as provided in
Section 173(8). Therefore, the High Court was in error in holding that the State
Government in exercise of the power of superintendence under Section 3 of the Act
lacked the power to direct further investigation into the case. In reaching this conclusion
we have kept out of consideration the provision contained in Section 156(2) that an
investigation by an officer in charge of a police station, which expression includes police
officer superior in rank to such officer, cannot be questioned on the ground that such
investigating officer had no jurisdiction to carry on the investigation; otherwise that
provision would have been a short answer to the contention raised on behalf of
Respondent 1."

[See also Upkar Singh v. Ved Prakash, (2004) 13 SCC 292] 2004 AIR SCW 5017

10

. In Ram Lal Narang (supra), this Court was concerned with a case where two
conspiracies were alleged; one being part of a larger conspiracy. Two investigations were
carried out. This Court, while opining that further investigation is permissible in law, held
that the Magistrate has a discretion in the matter to direct further investigation, even if he
had taken cognizance of the offence, stating : AIR 1979 SC 1791, Para 21

"........The criticism that a further investigation by the police would trench upon the
proceeding before the court is really not of very great substance, since whatever the
police may do, the final discretion in regard to further action is with the Magistrate. That
the final word is with the Magistrate is sufficient safeguard against any excessive use or
abuse of the power of the police to make further investigation. We should not, however,
be understood to say that the police should ignore the pendency of a proceeding before a
court and investigate every fresh fact that comes to light as if no cognizance had been
taken by the Court of any offence.
@page-SC1055
We think that in the interests of the independence of the magistracy and the judiciary, in
the interests of the purity of the administration of criminal justice and in the interests of
the comity of the various agencies and institutions entrusted with different stages of such
administration, it would ordinarily be desirable that the police should inform the court
and seek formal permission to make further investigation when fresh facts come to light."
While acknowledging the power of the police authorities to carry out further investigation
in terms of Section 173 of the Code of Criminal Procedure, an observation was made
therein to the following effect :
"....In our view, notwithstanding that a Magistrate had taken cognizance of the offence
upon a police report submitted under Section 173 of the 1898 Code, the right of the
police to further investigate was not exhausted and the police could exercise such right as
often as necessary when fresh information came to light. Where the police desired to
make a further investigation, the police could express their regard and respect for the
Court by seeking its formal permission to make further investigation."
11. Even in regard to an independent investigation undertaken by the police authorities, it
was observed:
"22. In our view, notwithstanding that a Magistrate had taken cognizance of the offence
upon a police report submitted under Section 173 of the 1898 Code, the right of the
police to further investigate was not exhausted and the police could exercise such right as
often as necessary when fresh information came to light. Where the police desired to
make a further investigation, the police could express their regard and respect for the
court by seeking its formal permission to make further investigation......."
12. It is not a case where investigation was carried out in relation to a separate
conspiracy. As allegations had been made against the officer of a local police station in
regard to the mode and manner in which investigation was carried out, a further
investigation was directed. The court was informed thereabout. Although, no express
permission was granted, but evidently, such a permission was granted by necessary
implication as further proceeding was stayed by the learned Magistrate. It is also not a
case where two chargesheets were filed before two different courts. The Court designated
to deal with the matters wherein investigation had been carried out by the CID, is located
at Chitoor. It is in the aforementioned situation, the Sessions Judge transferred the case
pending in the Tirupati Court to the Designated Court at Chittoor. Cognizance of further
offence had also been taken by the Chittoor Court.
13

. Reliance placed by the High Court as also by Mr. Rai in K. Chandrasekhar (supra) is
misplaced. Therein investigation had been carried out by the Central Bureau of
Investigation with the consent of the State. However, the State withdrew the same. The
question which arose for consideration therein was as to whether it was permissible for
the State to do so. The said issue was answered in the negative stating that the
investigating officer must be directed to complete the investigation. It was in the
aforementioned situation opined : 1998 AIR SCW 1852

"24. From a plain reading of the above section it is evident that even after submission of
police report under sub-section (2) on completion of investigation, the police has a right
of "further" investigation under sub-section (8) but not "fresh investigation" or
"reinvestigation". That the Government of Kerala was also conscious of this position is
evident from the fact that though initially it stated in the Explanatory Note of their
notification dated 27-6-1996 (quoted earlier) that the consent was being withdrawn in
public interest to order a "reinvestigation" of the case by a special team of State police
officers, in the amendatory notification (quoted earlier) it made it clear that they wanted a
"further investigation of the case" instead of "reinvestigation of the case." The dictionary
meaning of "further" (when used as an adjective) is "additional; more; supplemental".
"Further" investigation therefore is the continuation of the earlier investigation and not a
fresh investigation or reinvestigation to be started ab initio wiping out the earlier
investigation altogether. In drawing this conclusion we have also drawn inspiration from
the fact that sub-section (8) clearly envisages that on completion of further investigation
the investigating agency
@page-SC1056
has to forward to the Magistrate a "further" report or reports and not fresh report or
reports - regarding the "further" evidence obtained during such investigation. Once it is
accepted - and it has got to be accepted in view of the judgment in Kazi Lhendup Dorji -
that an investigation undertaken by CBI pursuant to a consent granted under Section 6 of
the Act is to be completed, notwithstanding withdrawal of the consent, and that "further
investigation" is a continuation of such investigation which culminates in a further police
report under sub-section (8) of Section 173, it necessarily means that withdrawal of
consent in the instant case would not entitle the State Police, to further investigate into
the case. To put it differently, if any further investigation is to be made it is the CBI alone
which can do so, for it was entrusted to investigate into the case by the State Government.
Resultantly, the notification issued withdrawing the consent to enable the State Police to
further investigate into the case is patently invalid and unsustainable in law. In view of
this finding of ours we need not go into the questions, whether Section 21 of the General
Clauses Act applies to the consent given under Section 6 of the Act and whether consent
given for investigating into Crime No. 246 of 1994 was redundant in view of the general
consent earlier given by the State of Kerala."
We do not see any application of the said ratio herein.
14. We, therefore, are of the opinion that the judgment of the High Court cannot be
sustained.
15. Mr. Rai submitted that the High Court did not go into the other contentions raised by
the respondent in quashing the application. We have examined the application filed by the
respondent under Section 482 of the Code of Criminal Procedure and are satisfied that the
respondent herein only raised the contention of validity of the chargesheet filed upon
completion of the second investigation.
16. For the reasons aforementioned, the appeal is allowed.
Appeal allowed.
AIR 2008 SUPREME COURT 1056 "Brajendra Singh v. State of M. P."
(From : Madhya Pradesh)*
Coram : 2 Dr. A. PASAYAT AND P. SATHASIVAM, JJ.
Civil Appeal No. 7764 of 2001, D/- 11 -1 -2008.
Brajendra Singh v. State of M.P. and Anr.
(A) Hindu Adoptions and Maintenance Act (78 of 1956), S.8(c) - ADOPTION - Adoption
- Hindu wife - Capacity to adopt - Husband's consent does not clothe wife with such
capacity.
Section 8 brings about a very important and far reaching change in the law of adoption as
used to apply earlier in case of Hindus. It is now permissible for a female Hindu who is
of sound mind and has completed the age of 18 years to take a son or a daughter in
adoption to herself in her own right provided that - (a) she is not married; (b) or is a
widow; (c) or is a divorcee or after marriage her husband has finally renounced the world
or is ceased to be a Hindu or has been declared to be of unsound mind by a Court having
jurisdiction to pass a declaratory decree to that effect. It follows from Clause (c) of
Section 8 that Hindu wife cannot adopt a son or daughter to herself even with the consent
of her husband because the section expressly provides for cases in which she can adopt a
son or daughter to herself during the lifetime of the husband. (Para 10)
(B) Hindu Adoptions and Maintenance Act (78 of 1956), S.8(c) - ADOPTION - Adoption
- Capacity of female Hindu - Married female Hindu - Not a divorcee but leading life like
a divorcee - Cannot legally adopt.
Section 8(c) lays down the situations in which a married female Hindu wife can adopt. It
lays down that only a female Hindu who is married and whose marriage has been
dissolved i.e. who is a divorcee has the capacity to adopt. A Hindu wife who is not
divorced but who is leading a life like a divorced woman cannot, therefore, adopt a son or
daughter. There is conceptual and contextual difference between a divorced woman and
one who is leading life like a divorced woman. Both cannot be equated. The appellant
because of her physical deformity lived separately from her husband and that too for a
very long period right from the date
@page-SC1057
of marriage. But in the eye of law they continued to be husband and wife because there
was no dissolution of marriage or a divorce in the eye of law. Son adopted by her so that
he can look after her, despite social issue involved, cannot be declared to be validly
adopted son. (Para 10)
(C) HINDU LAW - ADOPTION - Hindu Law - Adoption - Origin of custom and
objectives behind it, discussed. (Para 13)
Cases Referred : Chronological Paras
1994 AIR SCW 2836 (Ref.) 5, 19
AIR 1963 SC 185 (Ref.) 14
AIR 1959 SC 504 (Ref.) 17
AIR 1954 SC 581 (Ref.) 15
AIR 1933 PC 155 (Ref.) 15, 17
Raju Ramachandran, Sr. Advocate, Pra-kash Shrivastava, Ms. Pradipti Shrivastava and P.
Mahapatra, for Appellant; Dr. N. M. Ghatate, Sr. Advocate, C. D. Singh, Meru-sagar
Samantaray, Vairagya Vardhan and Sunny Chowdhary, for Respondents.
* S.A. No. 482 of 1996, D/- 25-9-2000 (MP).
Judgement
Dr. ARIJIT PASAYAT, J. :-The present appeal involves a very simple issue but when the
background facts are considered it projects some highly emotional and sensitive aspects
of human life.
2. Challenge in this appeal is to the judgment of the Madhya Pradesh High Court at
Jabalpur in a Second Appeal under Section 100 of the Code of Civil Procedure, 1908 (in
short the C.P.C.).
3. Background facts sans unnecessary details are as follows:
Sometime in 1948, one Mishri Bai, a crippled lady having practically no legs was given
in marriage to one Padam Singh. The aforesaid marriage appears to have been
solemnized because under the village custom, it was imperative for a virgin girl to get
married. Evidence on record shows that Padam Singh had left Mishri Bai soon after the
marriage and since then she was living with her parents at Village Kolinja. Seeing her
plight, her parents had given her a piece of land measuring 32 acres out of their
agricultural holdings for her maintenance. In 1970, Mishri Bai claims to have adopted
appellant Brajendra Singh. Padam Singh died in the year 1974. The Sub-Divisional
Officer, Vidisha served a notice on Mishri Bai under Section 10 of the M.P. Ceiling on
Agricultural Holdings Act, 1960 (in short the Ceiling Act) indicating that her holding of
agricultural land was more than the prescribed limit. Mishri Bai filed a reply contended
that Brajendra Singh is her adopted son and both of them constituted a Joint family and
therefore are entitled to retain 54 acres of land. On 28.12.1981, the Sub Divisional officer
by order dated 27.12.1981 disbelieved the claim of adoption on the ground inter alia that
in the entries in educational institutions adoptive fathers name was not recorded. On
10.1.1982, Mishri Bai filed Civil Suit No. SA/82 seeking a declaration that Brajendra
Singh is her adopted son. On 19.7.1989, she executed a registered Will bequeathing all
her properties in favour of Brajendra Singh. Shortly thereafter, she breathed her last on
8.11.1989. The trial court by judgment and order dated 3.9.1993 decreed the suit of
Mishri Bai. The same was challenged by the State. The first appellate court dismissed the
appeal and affirmed the judgment and decree of the trial court. It was held concurring
with the view of the trial court that Mishri Bai had taken Brajendra Singh in adoption and
in the Will executed by Mishri Bai the factum of adoption has been mentioned.
Respondents filed Second Appeal No. 482 of 1996 before the High Court. A point was
raised that the adoption was not valid in the absence of the consent of Mishri Bai's
husband. The High Court allowed the appeal holding that in view of Section 8(c) of
Hindu Adoptions and Maintenance Act, 1956 (in short the Act) stipulated that so far as a
female Hindu is concerned, only those falling within the enumerated categories can adopt
a son.
4. The High Court noted that there was a great deal of difference between a female Hindu
who is divorced and who is leading life like a divorced woman. Accordingly the High
Court held that the claimed adoption is not an adoption and had no sanctity in law. The
suit filed by Mishri Bai was to be dismissed.
5

. In support of the appeal learned counsel for the appellant submitted that as the factual
position which is almost undisputed goes to show, there was in fact no consummation of
marriage as the parties were living separately for a very long period practically from the
date of marriage. That being so, an inference that Mishri Bai ceased to be a married
woman, has been rightly recorded by the trial court and the first appellate court. It was
also pointed out that the question of law framed proceeded on a wrong 1994 AIR
SCW 2836

@page-SC1058
footing as if the consent of husband was necessary. There was no such stipulation in law.
It is contented that the question as was considered by the High Court was not specifically
dealt with by the trial court or the first appellate court. Strong reliance has been placed on
a decision of this Court in Jolly Das (Smt.) Alias Moulick v. Tapan Ranjan Das [1994 (4)
SCC 363] to highlight the concept of "Sham Marriage".
6. It was also submitted that the case of invalid adoption was specifically urged and taken
note of by the trial court. Nevertheless the trial court analysed the material and evidence
on record and came to the conclusion that Mishri Bai was living like a divorced woman.
7. Learned counsel for the respondents on the other hand submitted that admittedly
Mishri Bai did not fall into any of the enumerated categories contained in Section 8 of the
Act and therefore, she could not have validly taken Brajendra Singh in adoption.
8. It is to be noted that in the suit there was no declaration sought for by Mishri Bai either
to the effect that she was not married or that the marriage was sham or that there was any
divorce. The stand was that Mishri Bai and her husband were living separately for very
long period.
9. Section 8 of the Act reads as follows:
"8. Capacity of a female Hindu to take in adoption - Any female Hindu-
(a) who is of sound mind,
(b) who is not minor, and
(c) who is not married, or if married, whose marriage has been dissolved or whose
husband is dead or has completely and finally renounced the world or has ceased to be a
Hindu or has been declared by a court of competent jurisdiction to be of unsound mind,
has capacity to take a son or daughter in adoption."
10. We are concerned in the present case with clause (c) of Section 8. The Section brings
about a very important and far reaching change in the law of adoption as used to apply
earlier in case of Hindus. It is now permissible for a female Hindu who is of sound mind
and has completed the age of 18 years to take a son or daughter in adoption to herself in
her own right provided that (a) she is not married; (b) or is a widow; (c) or is a divorcee
or after marriage her husband has finally renounced the world or is ceased to be a Hindu
or has been declared to be of unsound mind by a court having jurisdiction to pass a
declaratory decree to that effect. It follows from Clause (c) of Section 8 that Hindu wife
cannot adopt a son or daughter to herself even with the consent of her husband because
the Section expressly provides for cases in which she can adopt a son or daughter to
herself during the lifetime of the husband. She can only make an adoption in the cases
indicated in clause (c). It is important to note that Section 6(1) of the Act requires that the
person who wants to adopt a son or a daughter must have the capacity and also the right
to take in adoption. Section 8 speaks of what is described as 'capacity.' Section 11 which
lays down the condition for a valid adoption requires that in case of adoption of a son, the
mother by whom the adoption is made must not have a Hindu son or son's son or grand
son by legitimate blood relationship or by adoption living at the time of adoption. It
follows from the language of Section 8 read with Clauses (i) and (ii) of Section 11 that
the female Hindu has the capacity and right to have both adopted son and adopted
daughter provided there is compliance of the requirements and conditions of such
adoption laid down in the Act. Any adoption made by a female Hindu who does not have
requisite capacity to take in adoption or the right to take in adoption is null and void. It is
clear that only a female Hindu who is married and whose marriage has been dissolved i.e.
who is a divorcee has the capacity to adopt. Admittedly in the instant case there is no
dissolution of the marriage. All that the evidence led points out is that the husband and
wife were staying separately for a very long period and Mishri Bai was living a life like a
divorced woman. There is conceptual and contextual difference between a divorced
woman and one who is leading life like a divorced woman. Both cannot be equated.
Therefore in law Mishri Bai was not entitled to the declaration sought for. Here comes the
social issue. A lady because of her physical deformity lived separately from her husband
and that too for a very long period right from the date of marriage. But in the eye of law
they continued to be husband and wife because there was no dissolution of marriage or a
divorce in the eye of law. Brajendra Singh was adopted by Mishri Bai so that he can look
after her. There is
@page-SC1059
no dispute that Brajendra Singh was in fact doing so. There is no dispute that the property
given to him by the Will executed by Mishri Bai is to be retained by him. It is only the
other portion of the land originally held by Mishri Bai which is the bone of contention.
11. Section 5 provides that adoptions are to be regulated in terms of the provisions
contained in Chapter II. Section 6 deals with the requisites of a valid adoption. Section 11
prohibits adoption; in case it is of a son, where the adoptive father or mother by whom
the adoption is made has a Hindu son, son's son, or son's son's son, whether by legitimate
blood relationship or by adoption, living at the time of adoption. Prior to the Act under
the old Hindu law, Article 3 provided as follows:
"3. (1) A male Hindu, who has attained the age of discretion and is of sound mind, may
adopt a son to himself provided he has no male issue in existence at the date of the
adoption.
(2) A Hindu who is competent to adopt may authorize either his (i) wife, or (ii) widow
(except in Mithila) to adopt a son to himself."
12. Therefore, prior to the enactment of the Act also adoption of a son during the lifetime
of a male issue was prohibited and the position continues to be so after the enactment of
the Act. Where a son became an outcast or renounced the Hindu religion, his father
became entitled to adopt another. The position has not changed after the enactment of the
Caste Disabilities Removal Act (21 of 1850), as the outcast son does not retain the
religious capacity to perform the obsequial rites. In case parties are governed by
Mitakshara law, additionally adoption can be made if the natural son is a congenital
lunatic or an idiot.
13. The origin of custom of adoption is lost in antiquity. The ancient Hindu law
recognized twelve kinds of sons of whom five were adopted. The five kinds of adopted
sons in early times must have been of very secondary importance, for, on the whole, they
were relegated to an inferior rank in the order of sons. Out of the five kinds of adopted
sons, only two survive today, namely, the dattaka form prevalent throughout India and the
kritrima form confined to Mithila and the adjoining districts. The primary object of
adoption was to gratify the means of the ancestors by annual offerings and, therefore, it
was considered necessary that the offerer should be as much as possible a reflection of a
real descendant and had to look as much like a real son as possible and certainly not be
one who would never have been a son. Therefore, the body of rules was evolved out of a
phrase of Saunaka that he must be "the reflection of a son". The restrictions flowing from
this maxim had the effect of eliminating most of the forms of adoption. (See Hindu Law
by S.V. Gupte, 3rd Edn., at pp. 899-900.) The whole law of dattaka adoption is evolved
from two important texts and a metaphor. The texts are of Manu and Vasistha, and the
metaphor that of Saunaka. Manu provided for the identity of an adopted son with the
family into which he was adopted. (See Manu, Chapter IX, pp. 141-42, as translated by
Sir W. Jones.) The object of an adoption is mixed, being religious and secular. According
to Mayne, the recognition of the institution of adoption in the early times had been more
due to secular reasons than to any religious necessity, and the religious motive was only
secondary; but although the secular motive was dominant, the religious motive was
undeniable. The religious motive for adoption never altogether excluded the secular
motive. (See Mayne's Hindu Law and Usage, 12th Edn., p. 329.)
14. As held by this Court in V.T.S. Chandrasekhara Mudaliar v. Kulandaivelu Mudaliar
(AIR 1963 SC 185) substitution of a son for spiritual reasons is the essence of adoption,
and consequent devolution of property is mere accessory to it; the validity of an adoption
has to be judged by spiritual rather than temporal considerations and devolution of
property is only of secondary importance.
15. In Hem Singh v. Harnam Singh (AIR 1954 SC 581) it was observed by this Court that
under the Hindu law adoption is primarily a religious act intended to confer spiritual
benefit on the adopter and some of the rituals have, therefore, been held to be mandatory,
and compliance with them regarded as a condition of the validity of the adoption. The
first important case on the question of adoption was decided by the Privy Council in the
case of Amarendra Man Singh Bhramarbar v. Sanatan Singh (AIR 1933 PC 155). The
Privy Council said:
Among the Hindus, a peculiar religious significance has attached to the son,
@page-SC1060
throughBrahminical influence, although in its origin the custom of adoption was perhaps
purely secular. The texts of the Hindus are themselves instinct with this doctrine of
religious significance. The foundation of the Brahminical doctrine of adoption is the duty
which every Hindu owes to his ancestors to provide for the continuance of the line and
the solemnization of the necessary rites.
16. With these observations it decided the question before it viz. that of setting the limits
to the exercise of the power of a widow to adopt, having regard to the well-established
doctrine as to the religious efficacy of sonship. In fact, the Privy Council in that case
regarded the religious motive as dominant and the secular motive as only secondary.
17. The object is further amplified by certain observations of this Court. It has been held
that an adoption results in changing the course of succession, depriving wife and
daughters of their rights, and transferring the properties to comparative strangers or more
remote relations. [See: Kishori Lal v. Chaltibai (AIR 1959 SC 504)]. Though undeniably
in most of the cases, motive is religious, the secular motive is also dominantly present.
We are not concerned much with this controversy, and as observed by Mayne, it is unsafe
to embark upon an enquiry in each case as to whether the motives for a particular
adoption were religious or secular and an intermediate view is possible that while an
adoption may be a proper act, inspired in many cases by religious motives, courts are
concerned with an adoption, only as the exercise of a legal right by certain persons. The
Privy Councils decision in Amarendra Man Singhs case (supra) has reiterated the well-
established doctrine as to the religious efficacy of sonship as the foundation of adoption.
The emphasis has been on the absence of a male issue. An adoption may either be made
by a man himself or by his widow on his behalf with his authority conveyed therefor. The
adoption is to the male and it is obvious that an unmarried woman cannot adopt, for the
purpose of adoption is to ensure spiritual benefit for a man after his death and to his
ancestors by offering of oblations of rice and libations of water to them periodically. A
woman having no spiritual needs to be satisfied, was not allowed to adopt for herself. But
in either case it is a condition precedent for a valid adoption that he should be without
any male issue living at the time of adoption.
18. A married woman cannot adopt at all during the subsistence of the marriage except
when the husband has completely and finally renounced the world or has ceased to be a
Hindu or has been declared by a court of competent jurisdiction to be of unsound mind. If
the husband is not under such disqualification, the wife cannot adopt even with the
consent of the husband whereas the husband can adopt with the consent of the wife. This
is clear from Section 7 of the Act. Proviso thereof makes it clear that a male Hindu cannot
adopt except with the consent of the wife, unless the wife has completely and finally
renounced the world or has ceased to be a Hindu or has been declared by a Court of
competent jurisdiction to be of unsound mind. It is relevant to note that in the case of a
male Hindu the consent of the wife is necessary unless the other contingency exists.
Though Section 8 is almost identical, the consent of the husband is not provided for. The
proviso to Section 7 imposes a restriction on the right of male Hindu to take in adoption.
In this respect the Act radically departs from the old law where no such bar was laid
down to the exercise of the right of a male Hindu to adopt oneself, unless he dispossess
the requisite capacity. As per the proviso to Section 7 the wife's consent must be obtained
prior to adoption and cannot be subsequent to the act of adoption. The proviso lays down
consent as a condition precedent to an adoption which is mandatory and adoption without
wife's consent would be void. Both proviso to Sections 7 and 8(c) refer to certain
circumstances which have effect on the capacity to make an adoption.
19

. At this juncture it would be relevant to take note of Jolly Das's case (supra). The
decision in that case related to an entirely different factual scenario. There was no
principle of law enunciated. That decision was rendered on the peculiar factual
background. That decision has therefore no relevance to the present case. 1964 AIR
SCW 2936

20. Learned counsel for the appellant submitted that in any event, the land which is
declared to be in excess of the prescribed limit vests in the Government to be allotted to
persons selected by the Government. It was submitted that in view of the peculiar
background, the Government may be directed to consider the appellant's case for
@page-SC1061
allotment of the land from the surplus land so that the purpose for which adoption was
made and the fact that the appellant nourished a crippled lady treating her to be his own
mother would set a healthy tradition and example. We express no opinion in that regard.
It is for the State Government to take a decision in the matter in accordance with law. But
while dismissing the appeal, we permit the appellant to be in possession of land for a
period of six months by which time the Government may be moved for an appropriate
decision in the matter. We make it clear that by giving this protection we have not
expressed any opinion on the acceptability or otherwise of the appellant's request to the
State Government to allot the land to him.
21. The appeal is dismissed subject to the aforesaid observations.
Appeal dismissed.
AIR 2008 SUPREME COURT 1061 "Venture Global Engineering v. Satyam Computer
Services Ltd."
(From : 2007 CLC 1086 (AP))
Coram : 2 TARUN CHATTERJEE AND P. SATHASIVAM, JJ.
Civil Appeal No. 309 of 2008 (arising out of SLP (C) No. 8491 of 2007), D/- 10 -1 -2008.
Venture Global Engineering v. Satyam Computer Services Ltd. and Anr.
(A) Arbitration and Conciliation Act (26 of 1996), Part I, S.34, S.48 - ARBITRATION
AND CONCILIATION - APPLICABILITY OF AN ACT - Applicability - Provisions of
Part I of Act would apply to all arbitrations including international commercial
arbitrations and to all proceedings relating thereto - Matter held concluded by decision in
2002 AIR SCW 1285.
The provisions of Part I of the Act would apply to all arbitrations including international
commercial arbitrations and to all proceedings relating thereto. Where such arbitration is
held in India, the provisions of Part-I would compulsorily apply and parties are free to
deviate to the extent permitted by the provisions of Part-I. Even in the case of
international commercial arbitrations held out of India provisions of Part-I would apply
unless the parties by agreement, express or implied, exclude all or any of its provisions.
Such an interpretation does not lead to any conflict between any of the provisions of the
Act and there is no lacuna as such. The matter, therefore, is concluded by the decision in
2002 AIR SCW 1285, Foll. (Paras 17, 29)
(B) Arbitration and Conciliation Act (26 of 1996), S.34, S.48 - ARBITRATION AND
CONCILIATION - INJUNCTION - Foreign award - Enforcement of - Award directing
appellant, foreign company to transfer shares in Indian company, Joint Venture Company,
to respondent - Seeking enforcement of said transfer of shares by respondent in Foreign
Court - Held, was violative of Shareholders Agreement - Appellant in circumstances can
challenge award in Indian Courts - Further in view of injunction restraining respondent
from seeking or effecting transfer of shares, respondent ought not to have proceeded in
issue in foreign Court without getting earlier interim orders vacated.
2007 CLC 1086 (AP), Reversed.
Foreign award which was passed outside India cannot be said to be not enforceable in
India by invoking the provisions of the Act or the CPC. The very fact that the decision in
2002 AIR SCW 1285 holds that it would be open to the parties to exclude the application
of the provisions of Part I by express or implied agreement, would mean that otherwise
the whole of Part I would apply. In any event, to apply S. 34 to foreign international
awards would not be inconsistent with S. 48 of the Act, or any other provision of Part II
as a situation may arise, where, even in respect of properties situate in India and where an
award would be invalid if opposed to the public policy of India, merely because the
judgment-debtor resides abroad, the award can be enforced against properties in India
through personal compliance of the judgment-debtor and by holding out the threat of
contempt as is being sought to be done in the present case. In such an event, the
judgment-debtor cannot be deprived of his right under S. 34 to invoke the public policy
of India, to set aside the award. The public policy of India includes - (a) the fundamental
policy of India; or (b) the interests of India; or (c) justice or morality; or (d) in addition, if
it is patently illegal. This extended definition of public policy can be by-passed by taking
the award to a foreign country for enforcement - In company law, the word 'transfer' has a
definite connotation which would require the ownership of the shares to be transferred to
the transferee, which would involve the steps being taken under the Companies Act and
the rules and regulations thereunder, as well
@page-SC1062
as the Foreign Exchange Management Act, 1999 (FEMA). Respondent in enforcing the
Award passed by arbitration foreign country directing foreign company a shareholder in
joint venture company, Indian company to transfer shares in favour of respondent
company in the US District Court instead of Indian Courts can be said to have been
motivated by the intention of evading the legal and regulatory scrutiny to which this
transaction would have been subject to had it been enforced in India. Therefore,
respondent was not prepared to enforce the Award in spite of intimate and close nexus to
India and its laws, the appellant would certainly not be deprived of the right to challenge
the award in Indian Courts. (Paras 20, 21)
The fact that the appellant having participated and consented in proceedings filed in
foreign Court by the respondent for execution of the award, would not be precluded from
re-opening the vary same issue by filing a suit in a Court. Moreover, when the High
Court in India restrained the respondent from effecting transfer of the shares pending
further orders by the City Civil Court, the respondent could not have proceeded the issue
before the Foreign District Court, without getting the said interim orders/directions
vacated. (Paras 22, 25)
Furthermore Section 10.05(b) and (c) of the shareholders Agreement between the parties
precluding the - Respondent from approaching the Foreign Courts in regard to the
enforcement of the Award, cannot be ignored lightly. The non-obstante clause in sub-sec.
(c) of said section would override the entirety of the agreement including sub-section (b)
which deals with settlement of the dispute by arbitration. Sub-section (c), therefore,
would apply to the enforcement of the Award which declares that, notwithstanding that
the proper law or the governing law of the contract is the law of the concerned foreign
State, their shareholders shall at all times act in accordance with the Companies Act and
other applicable Acts/Rules being in force in India at any time. Necessarily, enforcement
has to be in India, as declared by this very section which overrides every other section in
the Shareholders Agreement. Respondent, therefore, totally violated the agreement
between the parties by seeking enforcement of the transfer of the shares in the Indian
Company by approaching the Foreign Courts. The claim of the respondent that the
S.11.05(c) of the SHA cannot be construed to mean that Indian Law is a substantive law
of the contract or that Indian law would governed the dispute resolution clause in S.
11.05(b) are not acceptable. as the non obstante clause would override the entirety of the
agreement including sub-section (b) which deals with the settlement of the dispute by
arbitration and, therefore, Section 3 would apply to the enforcement of the award. In such
event, necessarily enforcement has to be in India as declared by the very section which
overrides every other section.
2007 CLC 1086 (AP), Reversed. (Paras 26, 27, 28, 29)
Cases Referred : Chronological Paras
2007 AIR SCW 652 : AIR 2007 SC 909 (Ref.) 24
(2007) EWHC 1541 15
(2006) 1 SCC 419 24
2006 (1) Arbi LR 38 (Del) 16
2005 (2) Bom CR 36416
AIR 2005 Chhat 21 16
AIR 2004 Guj 274 : 2004 CLC 1108 16
2003 AIR SCW 3041 : AIR 2003 SC 2629 (Ref.) 13, 21
AIR 2003 Guj 145 16
2002 AIR SCW 1285 : AIR 2002 SC 1432 : 2002 CLC 647 (Foll.) 5, 8, 10, 11, 12, 14,
17, 22, 28, 29
1998 (1) LJ 977 (Bom) 16
AIR 1960 Cal 702 13
87 F 3d 844 15
126 F 3d 15 15
364 F 3d 274 15
745 F Supp 172 15
K. K. Venugopal, Sr. Advocate, Vinay Kumar Misra, Rajat Tamni and B. Parthasarathi,
for Appellant; R. F. Nariman, Sr. Advocate, Subhash Chandra Birla and Subrat Birla, for
Respondents.
Judgement
P. SATHASIVAM, JLeave granted.
2

. Appellant - Venture Global Engineering (in short 'VGE'), a company incorporated in the
United States of America with its principal office at 33662, James J Pampo Drive, Fraser,
Michigan, USA 48026 through its Constituted Attorney, Mr. Pradeep Yadav filed this
appeal challenging the final order and judgment dated 27.2.2007 passed by the High
Court of Judicature, Andhra Pradesh at Hyderabad in City Civil Court Appeal No. 26 of
2007 Reported in 2007 CLC 1086

@page-SC1063
whereby the Division Bench of the High Court dismissed their appeal.
3. The facts, which are necessary for the disposal of this appeal, are as under:
On 20.10.1999, Appellant-Company and respondent No.1- Satyam Computer Services
Limited (in short "SCSL"), a registered company having its office at Mayfair Centre, S.P.
Road, Secunderabad entered into a Joint Venture Agreement to constitute a company
named Satyam Venture Engineering Services Ltd. respondent No.2 herein (in short
"SVES") in which both the appellant and respondent No.1 have 50 per cent equity
shareholding. Another agreement was also executed between the parties on the same day
being the Shareholders Agreement (in short "SHA") which provides that disputes have to
be resolved amicably between the parties and failing such resolution, the disputes are to
be referred to arbitration. Section 11.05 of the SHA provides for certain terms and
conditions as regards the resolution of the disputes. In February, 2005, disputes arose
between the parties. Respondent No.1 alleged that the appellant had committed an event
of default under the SHA owing to several venture companies becoming insolvent and
they had exercised its option to purchase the appellant-company's shares in SVES at its
book value. On 25.07.2005, respondent No.1 filed a request for arbitration with the
London Court of International Arbitration which appointed Mr. Paul B Hannon as sole
arbitrator on 10.9.2005. The sole Arbitrator on 3.4.2006 passed an award directing the
appellant VGE to transfer the shares to respondent No.1. On 14.4.2006, respondent No.1
filed a petition to recognize and enforce the award before the United States District
Court, Eastern District Court of Michigan (US Court). The appellant entered appearance
to defend this proceeding before the US Court by filing a cross petition. In the said
petition, it objected to the enforcement of the Award which ordered transfer of shares
which was in violation of Indian Laws and Regulations specifically the Foreign Exchange
Management Act, 1999 (in short "FEMA") and its notifications. The appellant filed a suit
being O.S. No. 80 of 2006 before the Ist Additional Chief Judge, City Civil Court,
Secunderabad on 28.4.2006 seeking declaration to set aside the award and permanent
injunction on the transfer of shares under the Award. On 15.6.2006, the District Court
passed an ad-interim ex parte order of injunction, inter alia, restraining respondent No.1
from seeking or effecting the transfer of shares either under the terms of the Award or
otherwise. Challenging the said order, respondent No.1 filed an appeal before the High
Court of Andhra Pradesh. The High Court admitted respondent's appeal and directed
interim suspension of the order of the District Court but made it clear that respondent
No.1 would not effect the transfer of shares until further orders. On 13.07.2006, in
response to the summons served upon the respondents, respondent No.1 appeared in the
Court and filed a petition under Order VII Rule 11 C.P.C. for rejection of the plaint. The
appellant filed objection to the application. The trial Court, by its order dated 28.12.2006,
allowed the said application and rejected the plaint of the appellant. Challenging the said
order, the appellant filed an appeal before the High Court. On 27.2.2007, the High Court
dismissed the appeal holding that the award cannot be challenged even if it is against the
public policy and in contravention of statutory provisions. Against the said order, the
appellant preferred the above appeal by way of special leave petition.
4. Heard Mr. K.K. Venugopal, learned senior counsel, appearing for the appellant and Mr.
R.F. Nariman, learned senior counsel, appearing for respondent No.1.
5. After taking us through agreements entered into by both the parties, subsequent
developments such as alleged violations, Award by an Arbitrator at U.K., proceedings
before the District Court, Michigan, USA and the impugned proceedings of the Ist
Additional Chief Judge-City Civil Court, Secunderabad as well as the order of the High
Court, Mr. K.K. Venugopal learned senior counsel appearing for the appellant has raised
the following contentions:

(i) The claim that Part I of the Arbitration and Conciliation Act, 1996 (in short "the Act")
applies to foreign awards is covered by the judgment of this Court in Bhatia International
vs. Bulk Trading S.A. and Anr., (2002) 4 SCC 105.2002 AIR SCW 1285

(ii)The first respondent - Satyam Computer Services Ltd. could not have pursued the
enforcement proceedings in the District Court in Michigan, USA in the teeth of the
injunction granted by the Courts in India
@page-SC1064
which also, on the basis of the Comity of Courts should have been respected by the
District Court in Michigan.
(iii) The overriding Section 11.5 (c) of the SHA would exclude respondent No.1- Satyam
Computer Services Ltd. approaching the US Court in regard to the enforcement of the
Award.
6. On the other hand, Mr. R.F. Nariman, learned senior counsel, appearing for the first
respondent, submitted that,
(i) In view of Section 44 of the Act and the terms of the agreement, no suit would lie in
India to set aside the Award, which is a foreign Award.
(ii) No application under Section 34 of the Act would lie to set aside the Award.
(iii) In view of the provisions of the Act and the terms of the agreement, the first
respondent rightly sought enforcement of the Award in Michigan, USA, hence the civil
suit filed at Secunderabad is not maintainable.
(iv) Section 11.5(c) of the SHA only deals with the rights and obligations of the appellant
and the first respondent while acting as shareholders of the 2nd respondent it has nothing
to do with the enforcement of foreign Award.
(v) In terms of the agreement, having participated in the arbitration proceedings in UK,
filed cross-suit/objection in the District Court, Michigan opposing the Award, the
appellant cannot agitate the very same issue in the Indian Courts namely, District Court,
Secunderabad. In other words, the appellant, VGE, cannot ride two horses at the same
time.
7. We perused all the relevant materials, Annexures and considered the rival contentions.
8. Since both Mr. K.K. Venugopal, learned senior counsel for the appellant and Mr. R. F.
Nariman, learned senior counsel, for respondent No.1 heavily relied on a judgment of this
Court in Bhatia International (supra), in support of their respective stand, let us consider
the facts in that case and ultimate conclusion arrived at therein.
9. Bhatia International filed an Appeal before this Court against the judgment of the M.P.
High Court in W.P. No. 453 of 2000. The appellant-Bhatia International entered into a
contract with the first respondent Bulk Trading on 9.5.1997. This contract contained an
arbitration clause which provided that arbitration was to be as per the Rules of the
International Chamber of Commerce (for short "ICC"). On 23.10.1997, the Ist respondent
made a request for arbitration with ICC. Parties had agreed that the arbitration be held in
Paris, France. ICC has appointed a sole arbitrator. The first respondent filed an
application under Section 9 of the Act before the 3rd Additional District Judge, Indore,
M.P. against the appellant and the 2nd respondent. One of the interim reliefs sought for
was an order of injunction restraining these parties from alienating, transferring and/or
creating third-party rights, disposing of, dealing with and/or selling their business assets
and properties. The appellant raised the plea of maintainability of such an application.
The appellant contended that Part I of the Act would not apply to arbitrations where the
place of arbitration was not in India. The application was rejected by the 3rd Additional
District Judge on 1-2-2000. It was held that the court at Indore (M.P.) had jurisdiction and
the application was maintainable. The appellant filed a writ petition before the High
Court of Madhya Pradesh, Indore Bench and the same was dismissed by the impugned
judgment dated 10-10-2000. Several contentions have been raised on behalf of the
appellant, namely, Part I of the Act only applies to arbitrations where the place of
arbitration is in India and if the place of arbitration is not in India then Part II of the said
Act would apply. Sub-section (2) of Section 2 of the Act makes it clear that the provisions
of Part I do not apply where the place of arbitration is not in India. The Court at Indore
could not have entertained the application under Section 9 of the Act as Part I did not
apply to arbitrations which had taken place outside India. On the other hand, on behalf of
respondent No.1, it was submitted that a conjoint reading of the provisions shows that
Part I is to be applied to all arbitrations. It was further submitted that unless the parties by
their agreement exclude its provisions, Part I would also apply to all International
Commercial arbitrations including those that take place out of India.
10
. The above contentions were considered in detail. In view of the assertion of both the
senior counsel, the decision in Bhatia International (supra) has very much bearing on the
issue raised in this case. The relevant paragraphs are reproduced hereunder: 2002
AIR SCW 1285

@page-SC1065
"14. At first blush the arguments of Mr. Sen appear very attractive. Undoubtedly sub-
section (2) of Section 2 states that Part I is to apply where the place of arbitration is in
India. Undoubtedly, Part II applies to foreign awards. Whilst the submissions of Mr. Sen
are attractive, one has to keep in mind the consequence which would follow if they are
accepted. The result would:
(a) Amount to holding that the legislature has left a lacuna in the said Act. There would
be a lacuna as neither Part I or II would apply to arbitrations held in a country which is
not a signatory to the New York Convention or the Geneva Convention (hereinafter called
"a non-convention country"). It would mean that there is no law, in India, governing such
arbitrations.
(b) Lead to an anomalous situation, inasmuch as Part I would apply to Jammu and
Kashmir in all international commercial arbitrations but Part I would not apply to the rest
of India if the arbitration takes place out of India.
(c) Lead to a conflict between sub-section (2) of Section 2 on one hand and sub-sections
(4) and (5) of Section 2 on the other. Further, sub-section (2) of Section 2 would also be
in conflict with Section 1 which provides that the Act extends to the whole of India.
(d) Leave a party remediless inasmuch as in international commercial arbitrations which
take place out of India the party would not be able to apply for interim relief in India even
though the properties and assets are in India. Thus a party may not be able to get any
interim relief at all."
"16. A reading of the provisions shows that the said Act applies to arbitrations which are
held in India between Indian nationals and to international commercial arbitrations
whether held in India or out of India. Section 2(1)( f ) defines an international
commercial arbitration. The definition makes no distinction between international
commercial arbitrations held in India or outside India. An international commercial
arbitration may be held in a country which is a signatory to either the New York
Convention or the Geneva Convention (hereinafter called "the convention country)." An
international commercial arbitration may be held in a non-convention country. The said
Act nowhere provides that its provisions are not to apply to international commercial
arbitrations which take place in a non-convention country. Admittedly, Part II only
applies to arbitrations which take place in a convention country. Mr. Sen fairly admitted
that Part II would not apply to an international commercial arbitration which takes place
in a non-convention country. He also fairly admitted that there would be countries which
are not signatories either to the New York Convention or to the Geneva Convention. It is
not possible to accept the submission that the said Act makes no provision for
international commercial arbitrations which take place in a non-convention country."
"17. Section 1 of the said Act reads as follows:
"1. Short title, extent and commencement.- (1) This Act may be called the Arbitration and
Conciliation Act, 1996.
(2) It extends to the whole of India:
Provided that Parts I, III and IV shall extend to the State of Jammu and Kashmir only
insofar as they relate to international commercial arbitration or, as the case may be,
international commercial conciliation.
The words "this Act" mean the entire Act. This shows that the entire Act, including Part I,
applies to the whole of India. The fact that all Parts apply to the whole of India is clear
from the proviso which provides that Parts I, III and IV will apply to the State of Jammu
and Kashmir only so far as international commercial arbitrations/conciliations are
concerned. Significantly, the proviso does not state that Part I would apply to Jammu and
Kashmir only if the place of the international commercial arbitration is in Jammu and
Kashmir. Thus if sub-section (2) of Section 2 is read in the manner suggested by Mr. Sen
there would be a conflict between Section 1 and Section 2(2). There would also be an
anomaly inasmuch as even if an international commercial arbitration takes place outside
India, Part I would continue to apply in Jammu and Kashmir, but it would not apply to
the rest of India. The legislature could not have so intended."
"21. Now let us look at sub-sections (2), (3), (4) and (5) of Section 2. Sub-section (2) of
Section 2 provides that Part I would apply where the place of arbitration is in India. To be
immediately noted, that it is not providing that Part I shall not apply where the place of
arbitration is not in India. It is also not providing that Part I will "only" apply where the
place of arbitration is in India
@page-SC1066
(Emphasis supplied). Thus the legislature has not provided that Part I is not to apply to
arbitrations which take place outside India. The use of the language is significant and
important. The legislature is emphasizing that the provisions of Part I would apply to
arbitrations which take place in India, but not providing that the provisions of Part I will
not apply to arbitrations which take place out of India. The wording of sub-section (2) of
Section 2 suggests that the intention of the legislature was to make provisions of Part I
compulsorily applicable to an arbitration, including an international commercial
arbitration, which takes place in India. Parties cannot, by agreement, override or exclude
the non-derogable provisions of Part I in such arbitrations. By omitting to provide that
Part I will not apply to international commercial arbitrations which take place outside
India the effect would be that Part I would also apply to international commercial
arbitrations held out of India. But by not specifically providing that the provisions of Part
I apply to international commercial arbitrations held out of India, the intention of the
legislature appears to be to ally (sic allow) parties to provide by agreement that Part I or
any provision therein will not apply. Thus in respect of arbitrations which take place
outside India even the non-derogable provisions of Part I can be excluded. Such an
agreement may be express or implied.
"26. Mr. Sen had also submitted that Part II, which deals with enforcement of foreign
awards does not contain any provision similar to Section 9 or Section 17. As indicated
earlier, Mr. Sen had submitted that this indicated the intention of the legislature not to
apply Sections 9 and 17 to arbitrations, like the present, which are taking place in a
foreign country. The said Act is one consolidated and integrated Act. General provisions
applicable to all arbitrations will not be repeated in all Chapters or Parts. The general
provisions will apply to all Chapters or Parts unless the statute expressly states that they
are not to apply or where, in respect of a matter, there is a separate provision in a separate
Chapter or Part. Part II deals with enforcement of foreign awards. Thus Section 44 (in
Chapter I) and Section 53 (in Chapter II) define foreign awards, as being awards covered
by arbitrations under the New York Convention and the Geneva Convention respectively.
Part II then contains provisions for enforcement of "foreign awards" which necessarily
would be different. For that reason special provisions for enforcement of foreign awards
are made in Part II. To the extent that Part II provides a separate definition of an arbitral
award and separate provisions for enforcement of foreign awards, the provisions in Part I
dealing with these aspects will not apply to such foreign awards. It must immediately be
clarified that the arbitration not having taken place in India, all or some of the provisions
of Part I may also get excluded by an express or implied agreement of parties. But if not
so excluded the provisions of Part I will also apply to foreign awards. The opening words
of Sections 45 and 54, which are in Part II, read "notwithstanding anything contained in
Part I". Such a non obstante clause had to be put in because the provisions of Part I apply
to Part II."
"32. To conclude, we hold that the provisions of Part I would apply to all arbitrations and
to all proceedings relating thereto. Where such arbitration is held in India the provisions
of Part I would compulsorily apply and parties are free to deviate only to the extent
permitted by the derogable provisions of Part I. In cases of international commercial
arbitrations held out of India provisions of Part I would apply unless the parties by
agreement, express or implied, exclude all or any of its provisions. In that case the laws
or rules chosen by the parties would prevail. Any provision, in Part I, which is contrary to
or excluded by that law or rules will not apply."
"35. Lastly, it must be stated that the said Act does not appear to be a well-drafted
legislation. Therefore the High Courts of Orissa, Bombay, Madras, Delhi and Calcutta
cannot be faulted for interpreting it in the manner indicated above. However, in our view
a proper and conjoint reading of all the provisions indicates that Part I is to apply also to
international commercial arbitrations which take place out of India, unless the parties by
agreement, express or implied, exclude it or any of its provisions. Such an interpretation
does not lead to any conflict between any of the provisions of the said Act. On this
interpretation there are no lacunae in the said Act. This interpretation also does not leave
a party remediless. Thus such an interpretation has to be preferred to the one adopted by
the High Courts of Orissa, Bombay, Madras, Delhi and Calcutta. It will,
@page-SC1067
therefore, have to be held that the contrary view taken by these High Courts is not good
law."
11

. Mr. K.K. Venugopal, learned senior counsel, has pointed out that paragraph 14 of the
judgment of Bhatia International (supra) sets out four independent reasons for arriving at
the conclusion that Part I would apply to foreign Awards that are as follows : 2002
AIR SCW 1285

i) to hold to the contrary would result in a lacunae as Non-Convention country awards


cannot be enforced in India.
ii) Section 1(2) expressly extends Part I to the State of Jammu and Kashmir so far as it
relates to international commercial arbitration giving rise to an anomaly so far as the rest
of India is concerned unless Part I applies to international commercial arbitrations in the
other States as well.
iii) If the word "only" is read into Section 2(2), it would then render the sub-section
inconsistent with sub-sections (4) and (5) of Section 2 which apply Part I to all
arbitrations, meaning thereby, including foreign international arbitrations.
iv) As otherwise, no relief can be sought in India even though the properties and assets
are situated in India, merely because the arbitration is an international commercial
arbitration.
Further, by drawing our attention to the specific conclusion arrived in paragraphs 32 and
35, he reiterated that the issue has been very well concluded and the argument based on
paragraph 26 is not acceptable.
12. Mr. Nariman heavily relied on paragraph 26 of the judgment in Bhatia International
which we have extracted supra. According to him, the said paragraph contains not only
the submissions of Mr. Sen, who appeared for Bhatia International therein but also the
ultimate conclusion of the Bench. He reiterated that the Court concluded "Thus Section
44 (in Chapter I) and Section 53 (in Chapter II) define foreign Awards, as being awards
covered by arbitrations under the New York Convention and the Geneva Convention
respectively. Part II then contains provisions for enforcement of "foreign awards" which
necessarily would be different. For that reason, special provisions for enforcement of
foreign awards are made in Part II. To the extent that Part II provides a separate definition
of an arbitral award and separate provisions for enforcement of foreign awards, the
provisions in Part I dealing with these aspects will not apply to such foreign awards. It
must immediately be clarified that the arbitration not having taken place in India, all or
some of the provisions of Part I may also get excluded by an express or implied
agreement of parties. But if not so excluded, the provisions of Part I will also apply to
"foreign awards". The opening words of Sections 45 and 54, which are in Part II, read
"notwithstanding anything contained in Part I." Such a non obstante clause had to be put
in because the provisions of Part I apply to Part II.
13

. According to Mr. K.K. Venugopal, paragraphs 26 and 27 start by dealing with the
arguments of Mr. Sen who argued that Part I is not applicable to foreign awards. He
further pointed out that it is only in the sentence starting at the bottom of para 26 that the
phrase "it must immediately be clarified that the finding of the Court is rendered. That
finding is to the effect that an express or implied agreement of parties can exclude the
applicability of Part I. He further pointed out that the finding specifically states that, "But
if not so excluded, the provisions of Part I will also apply to all "foreign awards". This
exception which is carved out, based on agreement of the parties. By omitting to provide
that Part I will not apply to international commercial arbitrations which take place outside
India the effect would be that Part I would also apply to international commercial
arbitrations held out of India. But by not specifically providing that the provisions of Part
I apply to international commercial arbitrations held out of India, the intention of the
legislature appears to be to allow parties to provide by agreement that Part I or any
provision therein will not apply. Thus in respect of arbitrations which take place outside
India even the non-derogable provisions of Part I can be excluded. Such an agreement
may be express or implied. He further pointed out the very fact that the judgment holds
that it would be open to the parties to exclude the application of the provisions of Part I
by express or implied agreement, would mean that otherwise the whole of Part I would
apply. In any event, according to him, to apply Section 34 to foreign international awards
would not be inconsistent with Section 48 of the Act, or any other provision of Part II as a
situation may arise, where, even in respect of properties situate in India and where an
award would be invalid if opposed to the 2003 AIR SCW 3041

@page-SC1068
public policy of India, merely because the judgment-debtor resides abroad, the award can
be enforced against properties in India through personal compliance of the judgment-
debtor and by holding out the threat of contempt as is being sought to be done in the
present case. In such an event, the judgment-debtor cannot be deprived of his right under
Section 34 to invoke the public policy of India, to set aside the award. He very much
relied on the judgment of this Court in Oil and Natural Gas Corporation Ltd. vs. Saw
Pipes Ltd. (2003) 5 SCC 705, wherein particularly, in paragraphs 30 and 31, the public
policy of India has been defined to include-
(a) the fundamental policy of India; or
(b) the interests of India; or
(c) justice or morality; or
(d) in addition, if it is patently illegal.
He pointed out that this extended definition of public policy can be by-passed by taking
the award to a foreign country for enforcement. In such circumstances, according to him,
there is nothing inconsistent between Section 48 which deals with enforcement and
Section 34 which deals with a challenge to the Award. He also relied on a decision of the
Division Bench of the Calcutta High Court in Pratabmull Rameshwar vs. K.C. Sethia
Ltd., AIR 1960 Calcutta 702. In paragraphs 45 and 63, the Calcutta High Court while
dealing with Arbitration Act of 1940 sets out the reasoning in support of a challenge
being permissible in India to a foreign award.
14

. In order to find out an answer to the first and prime issue and whether the decision in
Bhatia International (supra) is an answer to the same, let us go into the details regarding
the suit filed by the appellant as well as the relevant provisions of the Act. The appellant -
VGE filed O.S. No. 80 of 2006 on the file of the Ist Additional District Court,
Secunderabad, for a declaration that the Award dated 3.4.2006 is invalid, unenforceable
and to set aside the same. Section 5 of the Act makes it clear that in matters governed by
Part I, no judicial authority shall intervene except where so provided. Section 5 which
falls in Part I, specifies that no judicial authority shall intervene except where so
provided. The Scheme of the Act is such that the general provisions of Part I, including
Section 5, will apply to all Chapters or Parts of the Act. Section 2(5) which falls in Part I,
specifies that "this part shall apply to all arbitrations and to all proceedings relating
thereto." It is useful to refer Section 45 which is in Part II of the Act which starts with
non obstante clause namely, "Notwithstanding anything contained in Part I or in Code of
Civil Procedure. . . . . . . . ." Section 52 in Chapter I of Part II of the Act provides that
"Chapter II of this Part shall not apply in relation to foreign awards to which this Chapter
applies." As rightly pointed out, the said section does not exclude the applicability of Part
I of the Act to such awards. 2002 AIR SCW 1285

15. Part II of the Act speaks about the enforcement of certain foreign awards. Section 48
speaks about conditions for enforcement of foreign awards. Section 48(1) (e) read with
Section 48(3) of the Act specify that an action to set aside the Award would lie to the
competent authority. Mr. Nariman, after taking us through the relevant provisions of
Chapter I, Part II submitted that Section 48(1)(e) read with Section 48(3) of the Act
specifies that an action to set aside a foreign award within the meaning of Section 44 of
the Act would lie to the "competent authority of the country in which, or under the law of
which, that award was made". According to him, the phrase "the country . . . . . . . . under
the law of which, that award was made" refers to the country of the curial law of
arbitration, in the extremely rare situation where the parties choose a curial law other than
the law of the country of the seat of arbitration. He further pointed out that therefore such
a challenge would lie only to the competent Court of the country in which the foreign
award was made. He also submitted that the said principle is recognized internationally
by Courts in US and UK as well as by several High Courts in India. The US decisions
which support/recognize the above principle are :
(1) International Standard Electric Corp. vs. Bridas Sociedad Anonima Petrolera,
Industrial Y Comercial, 745 F.supp.172
(2) M and C Corporation vs. ERWIN BEHR GmbH and Co., KG, a foreign
Corporation, 87 F.3d 844
(3) Yusuf Ahmed Alghanim and Sons vs. Toys "R" US. INC. Thr. (HK) Ltd., 126 F3d 15
(4) Karaha Bodas Co. L.L.C. vs. Perusahaan Pertambangan Minyakdan Gas
@page-SC1069
Bumi Negara, 364 F3d 274
(5) C v. D (2007) EWHC 1541
16. Apart from the above US decisions, Mr. R.F. Nariman, pointed out that all the Indian
High Courts except the Gujarat High Court in Nirma Ltd. vs. Lurgi Energie Und
Entsorgung GMBH, Germany, AIR 2003 Gujarat 145, have taken this consistent view in
the following judgments:
(a) Bombay Gas Company Limited vs. Mark Victor Mascarenhas and Ors., 1998 1 LJ
977
(b) Inventa Fischer Gmbh and Co., K.G. vs. Polygenta Technologies Ltd., 2005 (2) Bom
CR 364
(c) Trusuns Chemical Industry Ltd. vs. Tata International Ltd., AIR 2004 Gujarat 274
(d) Bharat Aluminium Co. Ltd. vs. Kaiser Aluminium Technical Services, AIR 2005
Chhattisgarh 21
(e) Bulk Trading SA vs. Dalmia Cement (Bharat) Limited, (2006) 1 ArbLR 38 (Delhi)
17

. On close scrutiny of the materials and the dictum laid down in three-Judge Bench
decision in Bhatia International (supra), we agree with the contention of Mr.
K.K.Venugopal and hold that paragraphs 32 and 35 of the Bhatia International (supra)
make it clear that the provisions of Part I of the Act would apply to all arbitrations
including international commercial arbitrations and to all proceedings relating thereto.
We further hold that where such arbitration is held in India, the provisions of Part-I would
compulsorily apply and parties are free to deviate to the extent permitted by the
provisions of Part-I. It is also clear that even in the case of international commercial
arbitrations held out of India provisions of Part-I would apply unless the parties by
agreement, express or implied, exclude all or any of its provisions. We are also of the
view that such an interpretation does not lead to any conflict between any of the
provisions of the Act and there is no lacuna as such. The matter, therefore, is concluded
by the three-Judge Bench decision in Bhatia International (supra). 2002 AIR SCW 1285
2002 AIR SCW 1285

18. Learned senior counsel for the respondent based on para 26 submitted that in the case
of foreign award which was passed outside India is not enforceable in India by invoking
the provisions of the Act or the CPC. However, after critical analysis of para 26, we are
unable to accept the argument of learned senior counsel for the respondent. Paras 26 and
27 start by dealing with the arguments of Mr. Sen who argued that Part I is not applicable
to foreign awards. It is only in the sentence starting at the bottom of para 26 that the
phrase "it must immediately be clarified" that the finding of the Court is rendered. That
finding is to the effect that an express or implied agreement of parties can exclude the
applicability of Part I. The finding specifically states: "But if not so excluded, the
provisions of Part I will also apply to all "foreign awards". This exception which is
carved out, based on agreement of the parties, in para 21 (placitum (e) to (f) is extracted
below :
"By omitting to provide that Part I will not apply to international commercial arbitrations
which take place outside India the effect would be that Part I would also apply to
international commercial arbitrations held out of India. But by not specifically providing
that the provisions of Part I apply to international commercial arbitrations held out of
India, the intention of the legislature appears to be to allow parties to provide by
agreement that Part I or any provision therein will not apply. Thus in respect of
arbitrations which take place outside India even the non-derogable provisions of Part I
can be excluded. Such an agreement may be express or implied."
19. The very fact that the judgment holds that it would be open to the parties to exclude
the application of the provisions of Part I by express or implied agreement, would mean
that otherwise the whole of Part I would apply. In any event, to apply Section 34 to
foreign international awards would not be inconsistent with Section 48 of the Act, or any
other provision of Part II as a situation may arise, where, even in respect of properties
situate in India and where an award would be invalid if opposed to the public policy of
India, merely because the judgment-debtor resides abroad, the award can be enforced
against properties in India through personal compliance of the judgment-debtor and by
holding out the threat of contempt as is being sought to be done in the present case. In
such an event, the judgment-debtor cannot be deprived of his right under Section 34 to
invoke the public policy of India, to set aside the award. As observed earlier, the public
policy of India includes -
@page-SC1070
(a) the fundamental policy of India; or (b) the interests of India; or (c) justice or morality;
or (d) in addition, if it is patently illegal. This extended definition of public policy can be
by-passed by taking the award to a foreign country for enforcement.
20. Mr. K.K.Venugopal also highlighted that in Company Law, the word 'transfer' has a
definite connotation which would require the ownership of the shares to be transferred to
the transferee, which would involve the following steps being taken under the Companies
Act and the Rules and Regulations thereunder, as well as the Foreign Exchange
Management Act, 1999 (FEMA):
i) Obtaining a Share Transfer Form 7-B and having it endorsed by the prescribed
authority under the Companies Act, 1956 in compliance with Section 108.
ii) Execution of Share Transfer Form 7-B by the appellant and respondent.
iii) Payment of stamp duty on the transfer of shares.
iv) Sending duly executed Share Transfer Form 7-B and the Share Certificates to SVES,
the respondent No.2 herein under Section 110 of Companies Act.
v) Respondent No.2 approving the transfer of shares and causing alteration in its Register
of Members under Section 111A.
vi) Compliance with Rules and Regulations, completing prescribed forms, giving relevant
undertakings in accordance with Indian Foreign Exchange Laws and Regulations such as
the Foreign Exchange Management Act, 1999 and its notifications, given that the
transaction involved transfer of shares from a non-resident to a resident.
By pointing out, he submitted that respondent No.1, in enforcing the Award in the US
District Court instead of Indian Courts was motivated by the intention of evading the
legal and regulatory scrutiny to which this transaction would have been subject to had it
been enforced in India. In the light of the statutory provisions as provided in the
Companies Act and FEMA, we agree with the submission of Mr. K.K.Venugopal.
21

. As rightly pointed out the effort of respondent No.1 was to avoid enforcement of the
Award under Section 48 of the 1996 Act which would have given the appellant herein the
benefit of the Indian Public Policy rule based on the judgment in the Saw Pipes case
(supra) and for avoiding the jurisdiction of the Courts in India though the award had an
intimate and close nexus to India in view of the fact that, (a) the company was situated in
India; (b) the transfer of the 'ownership interests' shall be made in India under the laws of
India as set out above; (c) all the steps necessary have to be taken in India before the
ownership interests stood transferred. If, therefore, respondent No.1 was not prepared to
enforce the Award in spite of this intimate and close nexus to India and its laws, the
appellant herein would certainly not be deprived of the right to challenge the award in
Indian Courts. 2003 AIR SCW 3041

22

. Mr. R.F. Nariman by placing the factual details, namely, filing of petition before the
Michigan Court for execution of the Award the objection petition filed by the first
respondent herein as well as the orders passed by the Court of Michigan, US submitted
that the appellant having participated and consented in those proceedings is precluded
from re-opening the very same issue by filing a suit in a court at Secunderabad which is
not permissible either under law or in terms of their conduct. In view of the legal position
derived from Bhatia International (supra), we are unable to accept Mr. Nariman's
argument. It is relevant to point out that in this proceeding, we are not deciding the merits
of the claim of both parties, particularly, the stand taken in the suit filed by the appellant-
herein for setting aside the award. It is for the concerned court to decide the issue on
merits and we are not expressing anything on the same. The present conclusion is only
with regard to the main issue whether the aggrieved party is entitled to challenge the
foreign award which was passed outside India in terms of Section 9/34 of the Act.
Inasmuch as the three-Judge Bench decision is an answer to the main issue raised, we are
unable to accept the contra view taken in various decisions relied on by Mr. Nariman.
Though in Bhatia International (supra) the issue relates to filing a petition under Section 9
of the Act for interim orders the ultimate conclusion that Part I would apply even for
foreign awards is an answer to the main issue raised in this 2002 AIR SCW 1285

@page-SC1071
case.
23. Mr. K.K. Venugopal, learned senior counsel, next contended that the overriding
section 11.05 (c) of the Shareholders Agreement would exclude respondent No.1
approaching the US Courts in regard to enforcement of the Award. Section 11.05 (b) and
(c) of the Shareholders Agreement between the parties read as follows:
"(b) This Agreement shall be construed in accordance with and governed by the laws of
the State of Michigan, United States, without regard to the conflicts of law rules of such
jurisdiction. Disputes between the parties that cannot be resolved via negotiations shall be
submitted for final, binding arbitration to the London Court of Arbitration.
(c) Notwithstanding anything to the contrary in this agreement, the Shareholders shall at
all times act in accordance with the Companies Act and other applicable Acts/Rules being
in force, in India at any time."
It was pointed out that the non obstante clause would override the entirety of the
agreement including sub-section (b) which deals with settlement of the dispute by
arbitration. It was further pointed out that sub-section (c), therefore, would apply to the
enforcement of the Award which declares that, notwithstanding that the proper law or the
governing law of the contract is the law of the State of Michigan, their shareholders shall
at all times act in accordance with the Companies Act and other applicable Acts/Rules
being in force in India at any time. In such circumstances, it is the claim of the appellant
that necessarily enforcement has to be in India, as mentioned in clause (c) which
overrides every other clause in the Shareholders Agreement. Mr. K.K. Venugopal further
pointed out that respondent No.1 totally violated the agreement between the parties by
seeking enforcement of the transfer of the shares in the Indian company by approaching
the District Court in the United States. On the other hand, Mr. Nariman pointed out that
Section 11.05(b) of the Shareholders Agreement alone governs the rights and obligations
between the appellant and the first respondent inter se and dispute resolution thereof. In
view of our discussion supra, we agree with the stand of the learned senior counsel for
the appellant.
24

. Coming to the other contentions particularly the fact that the suit has been filed before
the trial Court which is a court of competent jurisdiction under Section 2(e) of the Act
and not an application under Section 34 of the Act, Mr. K.K. Venugopal pointed out that
it would not affect the issue of jurisdiction as this Court has upheld the conversion of a
suit into a Section 9 petition under the Act. (vide Sameer Barar and Ors. v. Ratan
Bhushan Jain and Ors. (2006) 1 SCC 419) and in another instance, converted a writ
petition into a first appeal under the Civil Procedure Code. (vide Ajay Bansal vs. Anup
Mehta and Ors. (2007) 2 SCC 275). Even otherwise, if the Court in question is not
having jurisdiction in the interest of justice the suit/proceeding has to be transferred to the
court having competent jurisdiction. 2007 AIR SCW 652

25. Learned senior counsel for the appellant submitted that the first respondent - Satyam
Computer Services Ltd. could not have pursued the enforcement proceedings in the
District Court in Michigan, USA in the teeth of the injunction granted by the Courts in
India which also, on the basis of the Comity of Courts, should have been respected by the
District Courts in Michigan, USA. Elaborating the same, he further submitted that the
injunction of the trial court restraining the respondents from seeking or effecting the
transfer of shares either under the terms of the Award or otherwise was in force between
15.06.2006 and 27.06.2006. The injunction of the High Court in the following terms
"appellant (i.e. respondent No.1) shall not effect the transfer of shares of the respondents
pending further orders" was in effect from 27.06.2006 till 28.12.2006. The judgment of
the US District Court was on 13.07.2006 and 31.07.2006 when the Award was directed to
be enforced as sought by respondent No.1, notwithstanding the injunction to the effect
that the appellant (respondent No.1 herein) "shall not effect the transfer of shares of the
respondents pending further orders." The first respondent pursued his enforcement suit in
Michigan District Courts to have a decree passed directing - ".........VGE shall deliver to
Satyam or its designee, share certificates in a form suitable for immediate
@page-SC1072
transfer to Satyam evidencing all of the appellant's ownership interest in Satyam Ventures
Engineering Services (SVES), the party's joint venture company." Further, the "VGE
(appellant herein) shall do all that may otherwise be necessary to effect the transfer of its
ownership interest in SVES to Satyam (or its designee)". It is pointed out that obtaining
this order by pursuing the case in the US District Courts, in the teeth of the prohibition
contained in the order of the High Court, would not only be a contempt of the High Court
but would render all proceedings before the US courts a brutum fulmen, and liable to be
ignored. Though Mr. R.F.Nariman has pointed out that the High Court only restrained the
respondent from effecting transfer of the shares pending further orders by the City Civil
Court, Secunderabad, after the orders of the trial Court as well as limited order of the
High Court, the first respondent ought not to have proceeded the issue before the District
Court, Michigan without getting the interim orders/directions vacated.
26. Finally, the overriding section 11.5 (c) of the SHA cannot be ignored lightly. As
pointed out, the said section would exclude respondent No.1- Satyam Computer Services
Ltd. approaching the US Courts in regard to the enforcement of the Award. Section 11.05
(b) and (c) of the Shareholders Agreement between the parties which is relevant has
already been extracted in para 23.
The non obstante clause would override the entirety of the agreement including sub-
section (b) which deals with settlement of the dispute by arbitration. Sub-section (c),
therefore, would apply to the enforcement of the Award which declares that,
notwithstanding that the proper law or the governing law of the contract is the law of the
State of Michigan, their shareholders shall at all times act in accordance with the
Companies Act and other applicable Acts/Rules being in force in India at any time.
Necessarily, enforcement has to be in India, as declared by this very section which
overrides every other section in the Shareholders Agreement. Respondent No.1, therefore,
totally violated the agreement between the parties by seeking enforcement of the transfer
of the shares in the Indian company by approaching the District Courts in the United
States.
27. The claim of the first respondent that the section, namely, 11.05(c) of the SHA cannot
be construed to mean that Indian law is a substantive law of the contract or that Indian
law would govern the dispute resolution clause in Section 11.05(b) are not acceptable. As
rightly pointed out and observed earlier, the non obstante clause would override the
entirety of the agreement including sub-section (b) which deals with the settlement of the
dispute by arbitration and, therefore, section 3 would apply to the enforcement of the
award. In such event, necessarily enforcement has to be in India as declared by the very
section which overrides every other section.
28. The above-mentioned relevant aspects, the legal position as set out in three-Judge
Bench decision in Bhatia International (supra), specific clause in the Shareholders
Agreement (SHA), conduct of the parties have not been properly adverted to and
considered by the trial Court as well as the High Court. Accordingly, both the orders
passed by the City Civil Court and of the High Court are set aside.
29

. In terms of the decision in Bhatia International (supra), we hold that Part I of the Act is
applicable to the Award in question even though it is a foreign Award. We have not
expressed anything on the merits of claim of both the parties. It is further made clear that
if it is found that the Court in which the appellant has filed a petition challenging the
Award is not competent and having jurisdiction, the same shall be transferred to the
appropriate Court. Since from the inception of ordering notice in the special leave
petition both parties were directed to maintain status quo with regard to transfer of shares
in issue, the same shall be maintained till the disposal of the suit. Considering the nature
of dispute which relates to an arbitration Award, we request the concerned Court to
dispose of the suit on merits one way or the other within a period of six months from the
date of receipt of copy of this judgment. Civil appeal is allowed to this extent. No costs.
2002 AIR SCW 1285

Order accordingly.
@page-SC1073
AIR 2008 SUPREME COURT 1073 "Premkumari v. Prahlad Dev"
(From : Madhya Pradesh)*
Coram : 2 Dr. A. PASAYAT AND P. SATHASIVAM, JJ.
Civil Appeal No. 490 of 2008 (arising out of S.L.P. (C) No. 7373 of 2005), D/- 18 -1
-2008.
Premkumari and Ors. v. Prahlad Dev and Ors.
(A) Motor Vehicles Act (59 of 1988), S.149 - MOTOR VEHICLES - INSURANCE -
LICENSE - Liability of insurer - Driver having fake licence - Not by itself sufficient to
absolve insurer of liability - It has to further prove that owner was aware or noticed that
licence was fake and still permitted driver to drive. (Para 6)
(B) Motor Vehicles Act (59 of 1988), S.149 - Constitution of India, Art.136 - MOTOR
VEHICLES - SPECIAL LEAVE APPEAL - INSURANCE - Liability of insurer - Driver,
brother of owner of motor vehicle - Holding fake licence at time of accident - Insurer is
not liable to pay compensation - However, insurer directed not to recover amount already
paid by it from claimants - Such amount instead to be recovered from owner of vehicle -
Rest of amount to be recovered by claimants from owner and driver of motor vehicle.
(Para 10)
Cases Referred : Chronological Paras
2007 AIR SCW 2279 : AIR 2007 SC 1563 (Ref.) 8, 9
2007 AIR SCW 2362 : AIR 2007 SC 1609 : 2007 (3) ALJ 596 (Ref.) 9
2006 AIR SCW 1649 : AIR 2006 SC 3440 (Ref.) 10
2004 AIR SCW 663 : AIR 2004 SC 1531 (Ref.) 7, 8, 9, 10
2004 AIR SCW 952 : AIR 2004 SC 1630 (Rel. on) 10, 11
2003 AIR SCW 1695 : AIR 2003 SC 1292 (Ref.) 6
2001 AIR SCW 1340 : AIR 2001 SC 1419 6
1996 AIR SCW 3271 : AIR 1996 SC 2627 6
AIR 1987 SC 1184 6
Vikrant Singh Bais and Niraj Sharma, for Appellants; Ms. Manjeet Chawla and P. K.
Seth, for Respondents.
* Review Appln. M.C.C. No. 41 of 2004, D/- 22-4-2004 (M.P.) (Indore Bench).
Judgement
1. P. SATHASIVAM, J.Leave granted.
2. Whether the Tribunal was right in holding that the insurer was not liable as the driver
had a fake licence is the question to be decided in this appeal?
3. BACKGROUND FACTS:
One Ramdhan, who was husband of appellant No.1 and father of appellant Nos. 2 and 3
who were minor children, died in a motor vehicle accident while he was going on his
bicycle and hit by a truck bearing Registration No. CPW 7344 which was being driven in
a rash and negligent manner by respondent No.2 herein, owned by respondent No.1
herein and was insured by respondent No.3 herein - National Insurance Company.
According to the appellants/claimants at the time of accident, the deceased was aged
about 36 years and working as a carpenter and he was getting an income of Rs.125/- to
Rs.150/- per day. The claimants filed Claim Case No. 154 of 1997 before the Motor
Accident Claims Tribunal, Indore claiming a total compensation of Rs. 7 lacs under
Sections 166-A and 140 of the Motor Vehicles Act, 1988. Respondent No.3 filed a written
statement denying the claim and also pleaded that the driver of the offending vehicle did
not have a valid and effective driving licence on the date of the accident. The Tribunal
based on the materials placed and the evidence on record found that death was caused
due to rash and negligent driving of respondent No.2. On 08.02.2000, the Tribunal
awarded a compensation of Rs.2,56,000/- to the appellants along with interest @ 9% p.a.
from the date of filing of the claim application. The respondent No.3-Insurance Company
was exonerated from its liability to pay compensation on the ground that the driver of the
offending vehicle did not have a valid and effective driving licence on the date of
accident.
4. Aggrieved by the award of the Tribunal, the claimants filed Misc. Appeal No.
1665/2002 in the High Court of Madhya Pradesh, Bench at Indore challenging the
quantum of the award as well as exoneration of respondent No.3-Insurance Company
from its liability of making payment of compensation to them. The High Court,
considering the merits of the case and finding that duplicate licence was issued to
respondent No.2 who is not having a valid and effective licence on the date of the
accident, held that Insurance Company was not liable for the compensation amount as
determined. However, considering the merits of the case, age and income of the deceased
and dependents being wife and minor children enhanced the compensation amount to
Rs.3,50,000/- and directed respondent Nos. 1 and 2 i.e. owner and driver of the vehicle to
pay the same.
@page-SC1074
The review petition filed by the appellants in Misc. Civil Case No. 41 of 2004
exonerating respondent No.3 from its liability has been dismissed by the High Court by
order dated 22.04.2004. Questioning those orders, the claimants filed the present appeal
after obtaining leave.
5. Heard Mr. Vikrant Singh Bais, learned counsel for the appellants and Ms. Manjeet
Chawla, learned counsel for the 3rd respondent and none appeared for respondent Nos. 1
and 2 perused the materials placed before us and the annexures filed.
6

. In this appeal, the appellants mainly concerned about the orders of the Tribunal and the
High Court exonerating the Insurance Company from its liability. Before considering the
relevant decisions of this Court and the issue in question, let us note certain factual
details. The first respondent is the owner of the offending vehicle and respondent No.2 is
the driver of the said vehicle, who is none other than the brother of the first respondent.
Before the Tribunal, the Insurance Company contended that the driver was not having a
valid and effective driving licence. Considering the materials in the form of oral and
documentary evidence placed by the Insurance Company the Tribunal found that opposite
party No.2, namely, driver of the offending vehicle did not have a valid and effective
licence on the date of the accident. Based on the said conclusion, it exonerated the
Insurance Company from its liability. When this specific finding was challenged by way
of review application before the High Court, the judgment of this Court in United India
Insurance Co. Ltd. vs. Lehru and others, (2003) 3 SCC 338, was pressed into service. In
the said judgment, after considering Section 96(2)(b)(ii) of the old Motor Vehicles Act
and similar provision i.e. 149(2)(a)(ii) in the Motor Vehicles Act, 1988, this Court held as
under :- 2003 AIR SCW 1695

"17. xxx xxx xxx


Thus under sub-section (1) the insurance company must pay to the person entitled to the
benefit of the decree, notwithstanding that it has become "entitled to avoid or cancel or
may have avoided or cancelled the policy". The words "subject to the provisions of this
section" mean that the insurance company can get out of the liability only on grounds set
out in Section 149. Sub-section (7), which has been relied on, does not state anything
more or give any higher right to the insurance company. On the contrary, the wording of
sub-section (7) viz. "no insurer to whom the notice referred to in sub-section (2) or sub-
section (3) has been given shall be entitled to avoid his liability" indicates that the
legislature wanted to clearly indicate that insurance companies must pay unless they are
absolved of liability on a ground specified in sub-section (2). This is further clear from
sub-section (4) which mandates that conditions, in the insurance policy, which purport to
restrict insurance would be of no effect if they are not of the nature specified in sub-
section (2). The proviso to sub-section (4) is very illustrative. It shows that the insurance
company has to pay to third parties but it may recover from the person who was primarily
liable to pay. The liability of the insurance company to pay is further emphasised by sub-
section (5). This also shows that the insurance company must first pay, then it can
recover. If Section 149 is read as a whole it is clear that sub-section (7) is not giving any
additional right to the insurance company. On the contrary it is emphasising that the
insurance company cannot avoid liability except on the limited grounds set out in sub-
section (2).

18. Now let us consider Section 149(2). Reliance has been placed on Section 149(2)(a)
(ii). As seen, in order to avoid liability under this provision it must be shown that there is
a "breach". As held in Skandia (1987) 2 SCC 654 and Sohan Lal Passi (1996) 5 SCC 21
cases, the breach must be on the part of the insured. We are in full agreement with that.
To hold otherwise would lead to absurd results. Just to take an example, suppose a
vehicle is stolen. Whilst it is being driven by the thief there is an accident. The thief is
caught and it is ascertained that he had no licence. Can the insurance company disown
liability? The answer has to be an emphatic "No". To hold otherwise would be to negate
the very purpose of compulsory insurance. The injured or relatives of the person killed in
the accident may find that the decree obtained by them is only a paper decree as the
owner is a man of straw. The owner himself would be an innocent sufferer. It is for this
reason that the legislature, in its wisdom, has made insurance, at least third-party
insurance, compulsory. The aim and purpose being that an insurance company would be
available to pay. The business of the company is insurance. In all businesses there is an
element of risk. All persons carrying AIR 1987 SC 1184
1996 AIR SCW 3271

@page-SC1075
on business must take risks associated with that business. Thus it is equitable that the
business which is run for making profits also bears the risk associated with it. At the same
time innocent parties must not be made to suffer or loss. These provisions meet these
requirements. We are thus in agreement with what is laid down in the aforementioned
cases viz. that in order to avoid liability it is not sufficient to show that the person driving
at the time of accident was not duly licensed. The insurance company must establish that
the breach was on the part of the insured."

"20. When an owner is hiring a driver he will therefore have to check whether the driver
has a driving licence. If the driver produces a driving licence which on the face of it looks
genuine, the owner is not expected to find out whether the licence has in fact been issued
by a competent authority or not. The owner would then take the test of the driver. If he
finds that the driver is competent to drive the vehicle, he will hire the driver. We find it
rather strange that insurance companies expect owners to make enquiries with RTOs,
which are spread all over the country, whether the driving licence shown to them is valid
or not. Thus where the owner has satisfied himself that the driver has a licence and is
driving competently there would be no breach of Section 149(2)(a)(ii). The insurance
company would not then be absolved of liability. If it ultimately turns out that the licence
was fake, the insurance company would continue to remain liable unless they prove that
the owner/insured was aware or had noticed that the licence was fake and still permitted
that person to drive. More importantly, even in such a case the insurance company would
remain liable to the innocent third party, but it may be able to recover from the insured.
This is the law which has been laid down in Skandia (1987) 2 SCC 654, Sohan Lal Passi
(1996) 5 SCC 21 and Kamla (2001) 4 SCC 342 cases. We are in full agreement with the
views expressed therein and see no reason to take a different view."AIR 1987 SC 1184
2001 AIR SCW 1340

It is clear from the above decision when the owner after verification satisfied himself that
the driver has a valid licence and driving the vehicle in question competently at the time
of the accident there would be no breach of Section 149(2)(a)(ii), in that event, the
Insurance Company would not then be absolved of liability. It is also clear that even in
the case that the licence was fake, the Insurance Company would continue to remain
liable unless they prove that the owner was aware or noticed that the licence was fake and
still permitted him to drive.
7

. Learned counsel for the appellants placing reliance on a three-Judge Bench decision of
this Court in National Insurance Co. Ltd. vs. Swaran Singh and others, (2004) 3 SCC
297, contended that in view of marshalling of the case laws and principles arrived therein,
the Insurance Company cannot escape its liability to indemnify the owner even in the
case of breach of licence conditions. After analyzing the relevant provisions in the old
Motor Vehicles Act as well as the 1988 Act and the entire case laws, this Court
summarized its findings as under: 2004 AIR SCW 663

"110. The summary of our findings to the various issues as raised in these petitions is as
follows:
(i) Chapter XI of the Motor Vehicles Act, 1988 providing compulsory insurance of
vehicles against third-party risks is a social welfare legislation to extend relief by
compensation to victims of accidents caused by use of motor vehicles. The provisions of
compulsory insurance coverage of all vehicles are with this paramount object and the
provisions of the Act have to be so interpreted as to effectuate the said object.
(ii) An insurer is entitled to raise a defence in a claim petition filed under Section 163-A
or Section 166 of the Motor Vehicles Act, 1988, inter alia, in terms of Section 149(2)(a)
(ii) of the said Act.
(iii) The breach of policy condition e.g. disqualification of the driver or invalid driving
licence of the driver, as contained in sub-section (2)(a)(ii) of Section 149, has to be
proved to have been committed by the insured for avoiding liability by the insurer. Mere
absence, fake or invalid driving licence or disqualification of the driver for driving at the
relevant time, are not in themselves defences available to the insurer against either the
insured or the third parties. To avoid its liability towards the insured, the insurer has to
prove that the insured was guilty of negligence and failed to exercise reasonable care in
the matter of fulfilling the condition of the policy regarding use of vehicles by a duly
licensed driver or one who was not disqualified to drive at the relevant time.
(iv) Insurance companies, however, with a view to avoid their liability must not only
establish the available defence(s) raised in
@page-SC1076
the said proceedings but must also establish "breach" on the part of the owner of the
vehicle; the burden of proof wherefor would be on them.
(v) The court cannot lay down any criteria as to how the said burden would be
discharged, inasmuch as the same would depend upon the facts and circumstances of
each case.
(vi) Even where the insurer is able to prove breach on the part of the insured concerning
the policy condition regarding holding of a valid licence by the driver or his qualification
to drive during the relevant period, the insurer would not be allowed to avoid its liability
towards the insured unless the said breach or breaches on the condition of driving licence
is/are so fundamental as are found to have contributed to the cause of the accident. The
Tribunals in interpreting the policy conditions would apply "the rule of main purpose"
and the concept of "fundamental breach" to allow defences available to the insurer under
Section 149(2) of the Act.
(vii) The question, as to whether the owner has taken reasonable care to find out as to
whether the driving licence produced by the driver (a fake one or otherwise), does not
fulfil the requirements of law or not will have to be determined in each case.
(viii) If a vehicle at the time of accident was driven by a person having a learner's licence,
the insurance companies would be liable to satisfy the decree.
(ix) The Claims Tribunal constituted under Section 165 read with Section 168 is
empowered to adjudicate all claims in respect of the accidents involving death or bodily
injury or damage to property of third party arising in use of motor vehicle. The said
power of the Tribunal is not restricted to decide the claims inter se between claimant or
claimants on one side and insured, insurer and driver on the other. In the course of
adjudicating the claim for compensation and to decide the availability of defence or
defences to the insurer, the Tribunal has necessarily the power and jurisdiction to decide
disputes inter se between the insurer and the insured. The decision rendered on the claims
and disputes inter se between the insurer and insured in the course of adjudication of
claim for compensation by the claimants and the award made thereon is enforceable and
executable in the same manner as provided in Section 174 of the Act for enforcement and
execution of the award in favour of the claimants.
(x) Where on adjudication of the claim under the Act the Tribunal arrives at a conclusion
that the insurer has satisfactorily proved its defence in accordance with the provisions of
Section 149(2) read with sub-section (7), as interpreted by this Court above, the Tribunal
can direct that the insurer is liable to be reimbursed by the insured for the compensation
and other amounts which it has been compelled to pay to the third party under the award
of the Tribunal. Such determination of claim by the Tribunal will be enforceable and the
money found due to the insurer from the insured will be recoverable on a certificate
issued by the Tribunal to the Collector in the same manner under Section 174 of the Act
as arrears of land revenue. The certificate will be issued for the recovery as arrears of
land revenue only if, as required by sub-section (3) of Section 168 of the Act the insured
fails to deposit the amount awarded in favour of the insurer within thirty days from the
date of announcement of the award by the Tribunal.
(xi) The provisions contained in sub-section (4) with the proviso thereunder and sub-
section (5) which are intended to cover specified contingencies mentioned therein to
enable the insurer to recover the amount paid under the contract of insurance on behalf of
the insured can be taken recourse to by the Tribunal and be extended to claims and
defences of the insurer against the insured by relegating them to the remedy before
regular court in cases where on given facts and circumstances adjudication of their claims
inter se might delay the adjudication of the claims of the victims."
Among the above findings, for our purpose clause (iii) and (iv) are relevant.
8

. The effect and implication of the principles laid down in Swaran Singh's case (supra)
has been considered and explained by one of us (Dr. Justice Arijit Pasayat) in National
Insurance Co. Ltd. vs. Laxmi Narain Dhut, (2007) 3 SCC 700. The following conclusion
in para 38 are relevant: 2004 AIR SCW 663
2007 AIR SCW 2279

"38. In view of the above analysis the following situations emerge:


1. The decision in Swaran Singh case has no application to cases other than third-party
risks.
2. Where originally the licence was a fake
@page-SC1077
one, renewal cannot cure the inherent fatality.
3. In case of third-party risks the insurer has to indemnify the amount, and if so advised,
to recover the same from the insured.
4. The concept of purposive interpretation has no application to cases relatable to Section
149 of the Act."
9

. In the subsequent decision Oriental Insurance Co. Ltd. vs. Meena Variyal and others,
(2007) 5 SCC 428, which is also a two-Judge Bench while considering the ratio laid
down in Swaran Singh's case (supra) concluded that in a case where a person is not a
third party within the meaning of the Act, the Insurance Company cannot be made
automatically liable merely by resorting to Swaran Singh's case (supra). While arriving at
such a conclusion the Court extracted the analysis as mentioned in para 38 of Laxmi
Narain Dhut (supra) and agreed with the same. In view of consistency, we reiterate the
very same principle enunciated in Laxmi Narain Dhut (supra) with regard to
interpretation and applicability of Swaran Singh's case (supra). 2007 AIR SCW 2362
2004 AIR SCW 663
2007 AIR SCW 2279

10

. In the case of National Insurance Co. Ltd. vs. Kusum Rai and others, (2006) 4 SCC 250,
the vehicle was being used as a taxi. It was, therefore, a commercial vehicle. The driver
of the said vehicle was required to hold an appropriate licence therefor. Ram Lal, who
allegedly was driving the said vehicle at the relevant time, was holder of a licence to
drive light motor vehicle only. He did not possess any licence to drive a commercial
vehicle. Therefore, there was a breach of condition of the contract of insurance. In such
circumstances, the Court observed that the appellant-National Insurance Co. Ltd.,
therefore, could raise the said defence while considering the stand of the Insurance
Company. This Court, pointing out the law laid down in Swaran Singh (supra) concluded
that the owner of the vehicle cannot contend that he has no liability to verify the fact as to
whether the driver of the vehicle possessed a valid licence or not. However, taking note
of the fact that the owner has not appeared, the victim was aged only 12 years, the
claimants are from a poor background and to avoid another round of litigation applying
the decision in Oriental Insurance Co. Ltd. vs. Nanjappan, (2004) 13 SCC 224, and
finding that though the appellant-Insurance Company was not liable to pay the claimed
amount as the driver was not possessing a valid licence and the High Court committed an
error in holding otherwise, in the peculiar facts and circumstances of the case and in
exercise of jurisdiction under Article 136 of the Constitution declined to interfere with the
impugned judgment therein and permitted the appellant-Insurance Company to recover
the amount from the owner of the vehicle. 2006 AIR SCW 1649
2004 AIR SCW 663
2004 AIR SCW 952

11. In the light of the various principles, the factual finding of the Tribunal, namely, the
second respondent, driver was not holding a valid licence on the date of the accident and
also of the fact that the appellants are none else than widow and minor children of the
deceased, we pass the following order:-

(i) In view of the order of this Court dated 08.12.2006 granting stay of further
proceedings of the recovery initiated by the Insurance Company for refund of the amount
of Rs.50,000/- with interest claimed to have been paid to the appellants, we make it clear
that the appellants need not repay the said amount in spite of our conclusion which is in
favour of the Insurance Company. However, we permit the third respondent-Insurance
Company to recover the said amount from the owner of the vehicle in the same manner as
was directed in Nanjappan (supra); 2004 AIR SCW 952

(ii) The appellants are permitted to proceed and recover the rest of the amount from the
owner and driver of the vehicle - respondent Nos. 1 and 2 herein in accordance with law.
12. The appeal is disposed of with the above directions. No costs.
Order accordingly.
AIR 2008 SUPREME COURT 1077 "S. K. Dua v. State of Haryana"
(From : Punjab and Haryana)
Coram : 2 C. K. THAKKER AND D. K. JAIN, JJ.
Civil Appeal No. 184 of 2008 (arising out of SLP (C) No. 21311 of 2005), D/- 9 -1 -2008.
S. K. Dua v. State of Haryana and Anr.
(A) Constitution of India, Art.16, Art.14, Art.19, Art.21 - EQUALITY IN PUBLIC
EMPLOYMENT - RIGHT TO LIFE - RETIREMENT - FREEDOM OF TRADE -
Retiral benefits - Interest - Retiral benefits given to retiree four years after his retirement -
He would be entitled to interest on such benefits - It
@page-SC1078
can be claimed by relying on Rules occupying field - Even in absence of Rules employee
can claim interest by relying on Articles 14, 19 and 21.
C.W.P. No. 10025 of 2005, D/- 7-7-2005 (P and H), Reversed. (Paras 11, 12, 13)
(B) Constitution of India, Art.226 - WRITS - INTEREST - SUPREME COURT -
DISMISSAL - Writ petition by employee seeking interest on retiral benefits - Dismissal
in limine by High Court even without issuing notice to respondents - Not proper - In view
of alleged pendency of vigilance enquiry against retiree - Supreme Court instead of
directing Court for payment of interest, remitted matter to High Court for consideration
on merits. (Paras 14, 15)

M. N. Krishnamani, Sr. Advocate, S. Pani and Ansar Ahmed Chaudhary, for Appellant;
Manjit Singh, A. A. G. (Haryana) and T. V. George, for Respondents.
Judgement
1. C. K. THAKKER, J. :-Leave granted.
2. This appeal is directed against an order passed by the High Court of Punjab and
Haryana at Chandigarh on July 7, 2005 in Writ Petition (C) No. 10025 of 2005. By the
impugned order, the High Court dismissed the petition in limine relegating the appellant
writ petitioner to avail a remedy by approaching a Civil Court.
3. Facts in brief are that the appellant was working as an Engineer-in-Chief in the
Department of Irrigation, Haryana. According to him, he joined the service in Irrigation
Department of the erstwhile State of Punjab in August, 1961 and was allocated to the
Department of Irrigation and Power in the State of Haryana. He was promoted as
Engineer-in-Chief on May 31, 1996 and worked in that capacity till he attained the age of
superannuation in June, 1998. The appellant had an unblemished record of service for 37
years. During the course of his duties as Head of the Department, he submitted reports in
or about April-May, 1998 to the Government highlighting certain irregularities and mal-
practices said to have been committed by Mr. S.Y. Quraishi, the then Secretary, Irrigation
and Power and requested the Government to make enquiry through Central Bureau of
Investigation (CBI). According to the appellant, in pursuance of the complaint made by
him, the Government removed Mr. Quraishi as Secretary, Irrigation allowing him to work
only as Secretary, Department of Power.
4. The appellant has alleged that, as a measure of vendetta, Mr. Quraishi organized to
send the appellant on deputation on May 15, 1998 to a lower and unimportant specially
created post of Engineer-in-Chief, Command Area Development Agency by upgrading it
just few weeks before his retirement. In addition to the said action, the appellant was
served with three charge-sheets/ show cause notices in June, 1998, few days before his
retirement. The appellant, however, retired on June 30, 1998 on reaching the age of
superannuation. The appellant was paid provisional pension, but other retiral benefits
were not given to him which included Commuted Value of Pension, Leave Encashment,
Gratuity, etc. totalling to about Rs. 12 lakhs. They were withheld till finalization of
disciplinary proceedings. The appellant submitted replies to the charge-sheets/show cause
notices, inter alia, denying allegations and asserting that they were uncalled for and were
issued with mala fide intention and oblique motive. He further submitted that he had
acted in public interest in salvaging damage likely to be caused to public exchequer. The
replies submitted by the appellant were accepted by the authorities and the appellant was
exonerated of all the charges. All retiral benefits were thereafter given to him between
June 11 and July 18, 2002. Thus, according to the appellant though he retired in June,
1998, retiral benefits to which he was otherwise entitled, were given to him after four
years of his superannuation.
5. The appellant has stated that, in the aforesaid circumstances, he was entitled to interest
on the amount which had been withheld by the respondents and paid to him after
considerable delay. He, therefore, made several representations. He also issued legal
notice on June 3, 2005 claiming interest at the rate of 18% per annum for delayed
payment. He had invited the attention of the Government to Administrative Instructions
issued by the Government under which an employee is entitled to claim interest. Even
otherwise, the action of non-payment of interest was arbitrary, unreasonable and violative
of Articles 14 and 21 of the Constitution. There was, however, no reply whatsoever from
the Government. The appellant as a senior citizen of 65 years of age then approached the
High Court of Punjab and Haryana by filing a writ petition under Article 226 of the
Constitution. But the High Court summarily dismissed the writ petition without even
issuing notice to the
@page-SC1079
respondents. The appellant has challenged the said order in the present appeal.
6. On October 28, 2005, notice was issued by this Court. Affidavits and further affidavits
were filed thereafter and the Registry was directed to place the matter for final hearing.
Accordingly, the matter has been placed before us for final disposal.
7. We have heard learned counsel for the parties.
8. The learned counsel for the appellant contended that the High Court was totally
unjustified in dismissing the writ petition in limine and the said order is liable to be set
aside. He submitted that no questions of fact, much less, disputed questions of fact were
involved in the petition and the High Court was wrong in summarily dismissing it. It is
well settled law, submitted the counsel, that retiral benefits are not in the nature of bounty
and an employee is entitled as of right to get those benefits immediately after
superannuation unless they are withdrawn or withheld as a matter of punishment.
According to the appellant, he had always acted in the interest of the Government and
saved public exchequer by inviting the attention to mal-practices committed by high
ranking officers. As a measure of revenge against the appellant, charge-sheets were
issued, but after considering the explanation submitted by the appellant, all proceedings
against him were dropped. In view of exoneration of the appellant, the Government ought
to have paid interest on retiral benefits which were given to him after long time. As per
the Guidelines and Administrative Instructions issued by the Government, the appellant
was entitled to such benefit with interest. The High Court ought to have allowed the writ
petition of the appellant and ought to have awarded those benefits. It was, therefore,
submitted that the appeal deserves to be allowed by directing the respondents to pay
interest on the retiral dues payable to the appellant which were actually paid to him after
considerable delay.
9. An affidavit-in-reply is filed by Special Secretary, Government of Haryana, Irrigation
Department. In the counter-affidavit which was filed in January, 2005, the deponent has
stated that the appellant was paid all his retiral dues as soon as he was exonerated of the
charges levelled against him. The deponent referred to the Haryana Civil Service
(Punishment and Appeal) Rules, 1987 relating to benefits to which an employee is
entitled and contended that after the charge-sheets were finally dropped, the appellant
was paid all retiral benefits within three months from the date of dropping of the charge-
sheets. But it was further stated that certain vigilance enquiries are 'still pending' against
the appellant. In the circumstances, according to the deponent, the appellant was not
entitled to interest and the action taken by the Government could not be said to be illegal
or otherwise unreasonable. A prayer was, therefore, made to dismiss the appeal.
10. In rejoinder-affidavit, the appellant reiterated what he had pleaded in the petition for
leave to appeal and submitted that the stand taken by the Government in counter-affidavit
is misconceived and he is entitled to the relief prayed in the petition before the High
Court and in the present appeal.
11. Having heard the learned counsel for the parties, in our opinion, the appeal deserves
to be partly allowed. It is not in dispute by and between the parties that the appellant
retired from service on June 30, 1998. It is also undisputed that at the time of retirement
from service, the appellant had completed more than three decades in Government
Service. Obviously, therefore, he was entitled to retiral benefits in accordance with law.
True it is that certain charge-sheets/show cause notices were issued against him and the
appellant was called upon to show cause why disciplinary proceedings should not be
initiated against him. It is, however, the case of the appellant that all those actions had
been taken at the instance of Mr. Quraishi against whom serious allegations of mal-
practices and misconduct had been levelled by the appellant which resulted in removal of
Mr. Quraishi from the post of Secretary, Irrigation. The said Mr. Quraishi then became
Principal Secretary to the Chief Minister. Immediately thereafter charge-sheets were
issued to the appellant and proceedings were initiated against him. The fact remains that
proceedings were finally dropped and all retiral benefits were extended to the appellant.
But it also cannot be denied that those benefits were given to the appellant after four
years. In the circumstances, prima facie, we are of the view that the grievance voiced by
the appellant appears to be well-founded that he would be entitled to interest on such
benefits. If there are Statutory Rules occupying the field, the appellant could claim
payment of interest relying on such Rules. If there
@page-SC1080
are Administrative Instructions, Guidelines or Norms prescribed for the purpose, the
appellant may claim benefit of interest on that basis. But even in absence Statutory Rules,
Administrative Instructions or Guidelines, an employee can claim interest under Part III
of the Constitution relying on Articles 14, 19 and 21 of the Constitution. The submission
of the learned counsel for the appellant, that retiral benefits are not in the nature of
'bounty' is, in our opinion, well-founded and needs no authority in support thereof. In that
view of the matter, in our considered opinion, the High Court was not right in dismissing
the petition in limine even without issuing notice to the respondents.
12. To us, the plea of the learned counsel for the appellant that the High Court ought to
have entered into the merits of the matter which is based on documentary evidence is
well-taken. In our considered view, the writ petition ought to have been admitted by
issuing Rule nisi and ought to have been decided on merits. The High Court, however,
dismissed the petition by a cryptic order which reads thus:
"The petitioner seeks only payment of interest on the delayed payment of retiral benefits.
We, however, relegate the petitioner to avail of his remedies before the Civil Court, if so
advised.
Dismissed with the above observations."
13. The order passed by the High Court, therefore, must be quashed and set aside.
14. The learned counsel for the appellant submitted that an appropriate direction may be
issued to the Government to pay interest to the appellant who had retired on June 30,
1998 and about a decade has passed even thereafter. He, therefore, submitted that the
matter may be finally concluded by this Court by passing appropriate orders. We would
have certainly considered this aspect and prayer made by the appellant but for the fact
that the High Court had not entertained the petition and it was summarily dismissed. The
High Court thus was not having the affidavit on behalf of the respondent-Authorities. In
the affidavit filed by the State-Authorities in this Court, the stand taken by Government is
that 'vigilance enquiries' are 'still pending' against the appellant. The said affidavit is of
January, 2005. In the affidavit-in-rejoinder, the writ-petitioner has stated that "the alleged
pendency of the 'vigilance enquiry' if any is insignificant". We are also not aware as to
what has happened thereafter though considerable period has elapsed. In view of all these
facts, in our opinion, it would be in the interest of both the parties that we may remit the
matter to the High Court so as to enable the High Court to consider the matter on merits
and pass an appropriate order in accordance with law. We are mindful that the appellant is
a senior citizen and the prayer relates to interest on retiral dues paid to him after four
years. Keeping in view the totality of facts and circumstances, we request the High Court
to give priority to the case and decide it finally as expeditiously as possible, preferably
before June 30, 2008.
15. For the foregoing reasons, the appeal is partly allowed. The order passed by the High
Court is set aside and the matter is remitted to the High Court for fresh disposal in
accordance with law. In the facts and circumstances of the case, however, there shall be
no order as to costs.
16. Before parting with the matter, we may clarify that we may not be understood to have
expressed any opinion on the merits of the matter, one way or the other. As and when the
writ petition will be placed before the High Court, it will be decided on its own merits
without being influenced by any observations made by us hereinabove.
Order accordingly.
Appeal partly allowed.
AIR 2008 SUPREME COURT 1080 "Bhadrappa v. Tolacha Naik"
(From : Karnataka)*
Coram : 2 Dr. A. PASAYAT AND P. SATHASIVAM, JJ.
Civil Appeal Nos. 7782 and 7799 of 2001, D/- 8 -1 -2008.
Bhadrappa (D) by L.Rs. v. Tolacha Naik.
Karnataka Scheduled Castes and Scheduled Tribes (Prohibition of Transfer of Certain
Lands) Act (2 of 1979), S.4 and S.5 - SCHEDULED CASTES AND SCHEDULED
TRIBES - LAND - Transfer of granted land during period of prohibition - Burden of
proof is on purchaser to prove that his possession is in accordance with law - Burden not
discharged - Order of resumption is proper.
2005 AIR SCW 3657, Foll. (Para 8)
Cases Referred : Chronological Paras
2005 AIR SCW 3657 : AIR 2005 SC 4013 : 2005 AIR-Kant HCR 2075 (Foll.) 8
@page-SC1081

AIR 1984 SC 1151 8


Naresh Kaushik, Mrs. Lalita Kaushik, Satish Daya Nandan and G. S. Pandey, for
Appellants; S. K. Kulkarni, G. Gireesh Kumar and Khwairakpam Nobin Singh, for
Respondent.
* W.A. No. 1886 of 2000, D/- 9-11-2000 (Kar).
Judgement
Dr. ARIJIT PASAYAT, J. :-Civil Appeal No. 7782 of 2001
1. Heard learned counsel for the parties.
2. Challenge in this appeal is to the order passed by a Division Bench of the Karnataka
High Court dismissing the writ appeal filed under Section 4 of the Karnataka High Court
Act (in short the 'High Court Act'). Challenge in the appeal was to the order passed by a
learned Single Judge who had dismissed the writ petition filed by the appellant-
Bhadrappa. After the death of Bhadrappa, his legal heirs were brought on record and they
are the appellants before this Court.
3. Background facts in a nutshell are as follows:
The land in question was granted some time in the year 1955 in favour of one Gopya
Naik who is referred hereinafter as grantee. Saguvali Chit was issued on 11.10.1956.
Seetamma, widow of the grantee who was also the mother of respondent No.3 sold the
land in the year 1959 in favour of one Gangappa who in turn sold the said land to Ahmad
Pasha and there was subsequent sale by Ahmad Pasha to Bhadrappa. The land in question
bears Survey No.106 measuring 3 acres and 5 guntas.
4. Proceedings were initiated on the basis of an application that the alienation was hit by
Section 4 of Karnataka Scheduled Castes and Schedules Tribes (Prohibition of Transfer
of Certain Lands) Act, 1978 (in short the 'Act').
5. Sections 4 and 5 of the Act read as follows:
"4. Prohibition of transfer of granted lands.- (1) Notwithstanding anything in any law,
agreement, contract or instrument, any transfer of granted land made either before or after
the commencement of this Act, in contravention of the terms of the grant of such land or
the law providing for such grant, or sub-section (2) shall be null and void and no right,
title or interest in such land shall be conveyed or be deemed ever to have conveyed by
such transfer.
(2) No person shall, after the commencement of this Act, transfer or acquire by transfer
any granted land without the previous permission of the Government.
(3) The provisions of sub-sections (1) and (2) shall apply also to the sale of any land in
execution of a decree or order of a civil court or of any award or order of any other
authority.
5. Resumption and restitution of granted lands.- (1) Where, on application by any
interested person or on information given in writing by any person or suo motu, and after
such enquiry as he deems necessary, the Assistant Commissioner is satisfied that the
transfer of any granted land is null and void under sub-section (1) of section 4, he may,-
(a) by order take possession of such land after evicting all persons in possession thereof
in such manner as may be prescribed:
Provided that no such order shall be made except after giving the person affected a
reasonable opportunity of being heard;
(b) restore such land to the original grantee or his legal heir. Where it is not reasonably
practicable to restore the land to such grantee or legal heir; such land shall be deemed to
have vested in the Government free from all encumbrances. The Government may grant
such land to a person belonging to any of the Scheduled Castes or Scheduled Tribes in
accordance with the rules relating to grant of land.
(1A) After an enquiry referred to in sub-section (1) the Assistant Commissioner may, if
he is satisfied that transfer of any granted land is not null and void pass an order
accordingly.
(2) Subject to the orders of the Deputy Commissioner under section 5A, any order passed
under sub-sections(1) and (1A) shall be final and shall not be questioned in any court of
law and no injunction shall be granted by any court in respect of any proceeding taken or
about to be taken by the Assistant Commissioner in pursuance of any power conferred by
or under this Act.
(3) For the purposes of this section, where any granted land is in the possession of a
person, other than the original grantee or his legal heir, it shall be presumed, until the
contrary is proved, that such person has acquired the land by a transfer which is null and
void under the provisions of sub-section (1) of section 4."
6. An order was passed in the proceeding under Section 5 of the Act to the effect that the
alienation had been effected within
@page-SC1082
the period of prohibition. The appellant took the stand that the land was not a free grant
land. It was a grant for upset price. The authorities concluded that it was a free grant. The
writ petition was dismissed.
7. The stand before the learned Single Judge and the Division Bench were reiterated.
8

. Section 5(3) of the Act clearly provides that any person other than the grantee or his
legal heirs in possession of the granted land, shall be deemed to be in possession under a
transfer which is null and void under Sections 4(1) and 4(2) until and unless anything
contrary is established. Burden, therefore, is on the person in possession to prove that his
possession was valid in accordance with law. It was found factually that the writ
petitioner had failed to establish the same. The transfer in favour of Gangappa was in
violation of the prohibition of the Act. That being so, the High Court was right in
dismissing the writ petition and the writ appeal. In Guntaiah and Ors. v. Hamb-amma and
Ors. (2005 (6) SCC 228 at para 14) it was noted as follows : 2005 AIR SCW 3657
(Para 13)

"It is also pertinent to note that the prohibition regarding alienation is a restrictive
covenant binding on the grantee. The grantee is not challenging that condition. In all
these proceedings, challenge is made by the third party who purchased the land from the
grantee. The third party is not entitled to say that the conditions imposed by the grantor to
the grantee were void. As far as the contract of sale is concerned, it was entered into
between the Government and the grantee and at that time the third-party purchaser had no
interest in such transaction. Of course, he would be entitled to challenge the violation of
any statutory provisions but if the grant by itself specifically says that there shall not be
any alienation by the grantee for a period of 15 years, that is binding on the grantee so
long as he does not challenge that clause, moreso when he purchased the land, in spite of
being aware of the condition. The Full Bench seriously erred in holding that the land was
granted under Rule 43-J and that the Authorities were not empowered to impose any
conditions regarding alienation without adverting to Section 4 of Act 2 of 1979. These
lands were given to landless persons almost free of cost and it was done as a social
welfare measure to improve the conditions of poor landless persons. When these lands
were purchased by third parties taking advantage of illiteracy and poverty of the grantees,
Act 2 of 1979 was passed with a view to retrieve these lands from the third-party
purchasers. When Act 2 of 1979 was challenged, this Court observed in Manchegowda v.
State of Karnataka (SCC pp. 310-11, para 17) AIR 1984 SC 1151

"17. Granted lands were intended for the benefit and enjoyment of the original grantees
who happen to belong to the Scheduled Castes and Scheduled Tribes. At the time of the
grant, a condition had been imposed for protecting the interests of the original grantees in
the granted lands by restricting the transfer of the same. The condition regarding the
prohibition on transfer of such granted lands for a specified period, was imposed by
virtue of the specific term in the grant itself or by reason of any law, rule or regulation
governing such grant. It was undoubtedly open to the grantor at the time of granting lands
to the original grantees to stipulate such a condition, the condition being a term of the
grant itself, and the condition was imposed in the interests of the grantee. Except on the
basis of such a condition the grantor might not have made any such grant at all. The
condition imposed against the transfer for a particular period of such granted lands which
were granted essentially for the benefit of the grantees cannot be said to constitute any
unreasonable restriction. The granted lands were not in the nature of properties acquired
and held by the grantees in the sense of acquisition, or holding of property within the
meaning of Article 19(1)( f ) of the Constitution. It was a case of a grant by the owner of
the land to the grantee for the possession and enjoyment of the granted lands by the
grantees and the prohibition on transfer of such granted lands for the specified period was
an essential term or condition on the basis of which the grant was made. It has to be
pointed out that the prohibition on transfer was not for an indefinite period or perpetual. It
was only for a particular period, the object being that the grantees should enjoy the
granted lands themselves at least for the period during which the prohibition was to
remain operative. Experience had shown that persons belonging to Scheduled Castes and
Scheduled Tribes to whom the lands were granted were, because of their poverty, lack of
education and general backwardness, exploited by various persons who could and would
take advantage of the sad plight of
@page-SC1083
these poor persons for depriving them of their lands. The imposition of the condition of
prohibition on transfer for a particular period could not, therefore, be considered to
constitute any unreasonable restriction on the right of the grantees to dispose of the
granted lands. The imposition of such a condition on prohibition in the very nature of the
grant was perfectly valid and legal."
Civil Appeal No.7799 of 2001:
9. In view of the position of law indicated in the connected Civil Appeal No.7782 of 2001
this appeal is sans merit.
10. Above being the position, there is no merit in these appeals which are accordingly
dismissed with no order as to costs.
Appeals dismissed.
AIR 2008 SUPREME COURT 1083 "State of Haryana v. Dinesh Kumar"
(From : 2006 (3) Pun LR 323)
Coram : 2 C. K. THAKKER AND ALTAMAS KABIR, JJ.
Civil Appeal No. 84 of 2008 and 85 of 2007 (arising out of SLP (C) Nos. 1840 and 14939
of 2007), D/- 8 -1 -2008.
State of Haryana and Ors. v. Dinesh Kumar.
(A) Criminal P.C. (2 of 1974), S.46, S.439 - ARREST - BAIL - Arrest and custody - What
amounts to - Person whose control is taken over by law - Whether by an officer with
coercive power or on voluntary surrender before Court - Is in custody as regards criminal
proceedings.
1984 Cri LJ 134 : AIR 1984 (NOC) 103 (Mad), Overruled.
C. W. P. No. 18 of 2006, D/- 1-8-2006 (P and H), Reversed.
AIR 1980 SC 785, Foll.
AIR 1960 SC 1125, Rel. on. (Para 25)
(B) Constitution of India, Art.16 - EQUALITY IN PUBLIC EMPLOYMENT -
APPOINTMENT - Appointment - Application form - Deliberate and willful
misrepresentation - Applicant involved in criminal case - Appearing before Court and
obtaining bail before formal arrest - Answering query in application form about arrest in
negative - Cannot be said to have deliberately concealed facts - Can be given benefit of
mistaken impression.
C. W. P. No. 18 of 2006, D/- 1-8-2006 (P and H), Affirmed. (Para 29)
Cases Referred : Chronological Paras
AIR 1984 (NOC) 103 : 1984 Cri LJ 134 (Mad) (FB) (Overruled) 19, 20, 25
AIR 1980 SC 785 : 1980 Cri LJ 426 (Foll.) 22, 23, 24
AIR 1960 SC 1125 : 1960 Cri LJ 1504 (Rel. on) 26
Anoop G. Chaudhary, Sr. Advocate, Manjeet Singh, Roopansh Purohit, Devendra Kumar
Singh, T.V. George, Ved Prakash Singh, Ramesh Kumar Shokeen and Kamal Mohan
Gupta, for Appellants; P.S. Patwalia, Sr. Advocate, Anil Hooda, Dr. Sushil Balwada, Ms.
Devesh Tripathi, Amanpreet Singh, Vinod Kumar Ahlawat, Jagjit Singh Suhag, Suresh
Kumar Nandal and Satbir Pillania, for Respondent.
Judgement
1. ALTAMAS KABIR, J. :- Leave granted.
2. These two appeals have been taken up for hearing and disposal together, inasmuch as,
the issues to be decided in these appeals are common to both, but have been decided
differently by two co-ordinate Benches of the same High Court giving rise to a question
of law which is of great public importance. In these appeals we are called upon to decide
what constitutes "arrest" and "custody" in relation to a criminal proceeding and the
decision in respect thereof may have a bearing on the fate of the respondent in this appeal
and that of the appellants in the other appeal in relation to their recruitment as Constable-
Drivers in the Haryana Police.
3. The respondent in the first of these two appeals and the appellants in the other appeal
applied for appointment as Constable-Drivers under the Haryana Police and submitted
their respective application forms, which contained two columns, namely, 13(A) and 14,
which read as follows:-
13(A) : Have you ever been arrested?
14: Have you ever been convicted by the Court of any offence?
4. As far as the respondent in SLP(C) No. 1840 of 2007, Dinesh Kumar, is concerned, he
answered the said two queries in the negative. Subsequently, during verification of the
character and antecedents of the said respondent, it was reported that he had been arrested
in connection with a case arising out of FIR No. 168 of 13th October, 1994, registered at
Kalanaur Police Station under Sections 323/324/34, Indian Penal Code. He and his family
members were ultimately acquitted of the charges framed against them on 6th January,
1998, by the Judicial Magistrate, Ist Class, Rohtak. The appellant, however, alleged that
the respondent had concealed these facts from the Selection
@page-SC1084
Committee and had not correctly furnished the information in columns 13(A) and 14 of
the application form submitted by him for recruitment to the post in question.
5. Since, according to the appellants, the respondent had fai1ed to disclose the aforesaid
criminal case, which had been registered against all his family members, he was not
offered any appointment. The appeal filed by the respondent was rejected by the Director
General of Police, Haryana, by his order dated 18th November, 2005.
6. Before the High Court, it was contended by the respondent that in connection with the
aforesaid FIR No. 168, dated 13th October, 1994, he had been granted bail on 17th
October, 1994 without having been arrested. It was, therefore, contended on his behalf
that since he had not been actually arrested and the case against him having ended in
acquittal, it must be deemed that no case had ever been filed against him and hence he
had not suppressed any information by replying in the negative to the questions contained
in columns 13(A) and 14.
7. The rejection of the respondent's claim for appointment as Constable-Driver on the
above-mentioned ground was challenged by him before the Punjab and Haryana High
Court in Civil Writ Petition No. 18 of 2006. Taking the view that the appellant had not
suppressed any material while filling up the said columns 13(A) and 14, the High Court
quashed the order of rejection by the Director General of Police, Haryana on 18th
November, 2005 and directed the appellants herein to take steps to issue an appointment
letter to the respondent subject to fulfillment of other conditions by him.
8. In order to arrive at the aforesaid conclusion, the High Court held that since the
petitioner had been acquitted from the criminal case in question, he had quite truthfully
answered the query in column 14 by stating that he had never been convicted by any
Court for any offence. The High Court also held that even column 13(A) had been
correctly answered because the High Court was of the view that the appellant had never
been arrested, though he had obtained bail in connection with the said case.
9. In the other writ petition filed by Lalit Kumar and Bhupinder, a co-ordinate Bench of
the same High Court took a different view. In the said matter the appellants had been
involved in a criminal case, being FIR No.212, dated 3rd November, 2000, registered at
Police Station Sadar, Narwana, for offences punishable under Sections
148/149/307/325/323 of the Indian Penal Code, but they had been subsequently acquitted
of the said charges on 10th September, 2001. On behalf of the State, the same stand was
taken that the aforesaid piece of information had been withheld by the writ petitioners
while filling column 14 of the application form. The High Court was of the view that
since the writ petitioners had withheld important information it clearly disentitled them to
appointment, as it revealed that they could not be trusted to perform their duties honestly.
The High Court, accordingly, dismissed the writ petitions as being without merit.
10. In the first of the two appeals, the respondent had not surrendered to the police but
had appeared before the Magistrate with his lawyer of his own volition and was
immediately granted bail. Admittedly, therefore, the respondent had not surrendered to
the police but had voluntarily appeared before the Magistrate and had prayed for bail and
was released on bail, so that as per the respondent's understanding at no point of time was
he taken into custody or arrested.
11. As to the second of the two appeals, the appellants in response to the query in column
14, had quite truthfully answered that they had not been convicted by any Court of any
offence, since they had been acquitted of the charges brought against them. With regard
to column 13(A), the appellants who had been implicated in FIR 108, dated 26th May,
2002 under Sections 323/324/34, Indian Penal Code of Police Station Nangal Chaudhary,
Mahendergarh, appeared before the Ilaka Magistrate on 7th June, 2002, and were released
on their personal bonds without being placed under arrest or being taken into custody.
The information disclosed by them was held to be suppression of the fact that they had
been involved in a criminal case though the tenor of the query was not to that effect and
was confined to the question as to whether they had been arrested.
12. One of the common questions which, therefore, need to be answered in both these
appeals is whether the manner in which they had appeared before the Magistrate and had
been released without being taken into
@page-SC1085
formal custody, could amount to "arrest" for the purpose of the query in Column 13(A).
As mentioned hereinbefore, the same High Court took two different views of the matter.
While, on the one hand, one Bench of the High Court held that since the accused had
neither surrendered nor had been taken into custody, it could not be said that he had
actually been arrested, on the other hand, another Bench of the same High Court
dismissed similar writ petitions filed by Lalit Kumar and Bhupinder, without examining
the question as to whether they had actually been arrested or not. The said Bench decided
the writ petitions against the writ petitioners upon holding that they had withheld
important information regarding their prosecutions in a criminal case though ultimately
they were acquitted.
13. In order to resolve the controversy that has arisen because of the two divergent views,
it will be necessary to examine the concept of "arrest" and "custody" in connection with a
criminal case. The expression "arrest" has neither been defined in the Code of Criminal
Procedure (hereinafter referred to as the "Code") nor in the Indian Penal Code or any
other enactment dealing with criminal offences. The only indication as to what would
constitute "arrest" may perhaps be found in Section 46 of the Code which reads as
follows:-
"Arrest how made- (1) In making an arrest the police officer or other person making the
same shall actually touch or confine the body of the person to be arrested, unless there be
a submission to the custody by word or action.
(2) If such person forcibly resists the endeavour to arrest him, or attempts to evade the
arrest, such police officer or other person may use all means necessary to effect the arrest.
(3) Nothing in this section gives a right to cause the death of a person who is not accused
of an offence punishable with death or with imprisonment for life.
{(4) Save in exceptional circumstances, no woman shall be arrested after sunset and
before sunrise, and where such exceptional circumstances exist, the woman police officer
shall, by making a written report, obtain the prior permission of the Judicial Magistrate of
the first class within whose local jurisdiction the offence is committed or the arrest is to
be made.}"
14. We are concerned with sub-sections (1) and (2) of Section 46 of the Code from which
this much is clear that in order to make an arrest the police officer or other person making
the same shall actually touch or confine the body of the person to be arrested, unless there
be submission to the custody by word or action.
15. Similarly, the expression "custody" has also not been defined in the Code.
16. The question as to what would constitute "arrest" and "custody" has been the subject-
matter of decisions of different High Courts, which have been referred to and relied upon
by Mr. Patwalia appearing for Dinesh Kumar, respondent in the first of the two appeals.
This Court has also had occasion to consider the said question in a few cases, which we
will refer to shortly. Reliance was also placed on the dictionary meaning of the two
expressions which will also be relevant to our decision.
17. Mr. Anoop Chaudhary, learned senior advocate, who appeared for the State of
Haryana, in both the appeals, submitted that when the respondent in the first appeal and
the appellants in the second appeal had appeared before the Magistrates and prayed for
bail, it must be understood that they had surrendered to the custody of the court, as
otherwise, the provisions of Section 439 of the Code would not have had application. Mr.
Chaudhary also submitted that it did not matter as to whether the accused persons had
been arrested and detained in custody by the police or not, the very fact that they
voluntarily appeared before the Magistrate and prayed for bail amounted to arrest of their
movements, since thereafter they were confined to the Court room and were no longer
free to leave the court premises of their own choice.
18. Mr. Chaudhary submitted that the ordinary dictionary meaning of "arrest" is to legally
restrain a person's movements for the purpose of detaining a person in custody by
authority of law. He submitted that in Dinesh Kumar's writ petition the High Court had
erred in coming to a finding that he had never been arrested since he had voluntarily
appeared before the Magistrate and had been granted bail immediately.
19

. Opposing Mr. Chaudhary's submission, Mr. Patwalia, relying on various decisions of


different High Courts and in particular a Full Bench decision of the Madras AIR
1984 (NOC) 103

@page-SC1086
High Court in the case of Roshan Beevi and Anr. vs. Joint Secretary to the Govt. of Tamil
Nadu and Ors. (1984 Criminal Law Journal 134) submitted that although technically the
appearance of the accused before the Magistrate might amount to surrender to judicial
custody, in actuality no attempt had been made by anyone to restrict the movements of
the accused which may have led him to believe that he had never been arrested. It is on a
layman's understanding of the principle of "arrest" and "custody" that prompted the
respondent in the first of the two appeals and the appellants in the second appeal to
mention in column 13(A) that they had never been arrested in connection with any
criminal offence.
20. Mr. Patwalia referred to certain decisions of the Allahabad High Court, the Punjab
High Court and the Madras High Court which apparently supports his submissions. Of
the said decisions, the one in which the meaning of the two expressions "arrest" and
"custody" have been considered in detail is that of the Full Bench of the Madras High
Court in Roshan Beevis case (supra). The said decision was, however, rendered in the
context of Sections 107 and 108 of the Customs Act, 1962. Sections 107 and 108 of the
Customs Act authorises a Customs Officer empowered in that behalf to require a person
to attend before him and produce or deliver documents relevant to the enquiry or to
summon such person whose attendance is considered necessary for giving evidence or
production of a document in connection with any enquiry being undertaken by such
officer under the Act. In such context the Full Bench of the Madras High Court returned a
finding that "custody" and "arrest" are not synonymous terms and observed that it is true
that in every arrest there is a custody but not vice-versa. A custody may amount to
"arrest" in certain cases, but not in all cases. It is in the aforesaid circumstances that the
Full Bench came to the conclusion that a person who is taken by the Customs Officer
either for the purpose of enquiry or interrogation or investigation cannot be held to have
come into the custody and detention of the Customs Officer and he cannot be deemed to
have been arrested from the moment he was taken into custody.
21. In coming to the aforesaid conclusion, the Full Bench had occasion to consider in
detail the meaning of the expression "arrest". Reference was made to the definition of
arrest in various legal dictionaries and Halsbury's Laws of England as also the Corpus
Juris Secondum. In paragraph 16 of the judgment it was observed as follows:
"16. From the various definitions which we have extracted above, it is clear that the word
'arrest' when used in its ordinary and natural sense, means the apprehension or restraint or
the deprivation of one's personal liberty. The question whether the person is under arrest
or not, depends not on the legality of the arrest, but on whether he has been deprived of
his personal liberty to go where he pleases. When used in the legal sense in the procedure
connected with criminal offences, an arrest consists in the taking into custody of another
person under authority empowered by law, for the purpose of holding or detaining him to
answer a criminal charge or of preventing the commission of a criminal offence. The
essential elements to constitute an arrest in the above sense are that there must be an
intent to arrest under the authority, accompanied by a seizure or detention of the person in
the manner known to law, which is so understood by the person arrested. In this
connection, a debatable question that arises for our consideration is whether the mere
taking into custody of a person by an authority empowered to arrest would amount to
'arrest' of that person and whether the terms 'arrest' and 'custody' are synonymous."
22. Faced with the decision of this Court in Niranjan Singh vs. Prabhakar (AIR 1980 SC
785) the Full Bench distinguished the same on an observation made by this Court that
equivocatory quibbling that the police have taken a man into informal custody but have
not arrested him, have detained him in interrogation but have not taken him into formal
custody, were unfair evasion of the straightforwardness of the law. This Court went on to
observe further that there was no necessity of dilating on the shady facet as the Court was
satisfied that the accused had physically submitted before the Sessions Judge giving rise
to the jurisdiction to grant bail. Taking refuge in the said observation, the Full Bench
observed that the decision rendered by this Court could not be availed of by the learned
counsel in support of his contentions that the mere taking
@page-SC1087
of a person into custody would amount to arrest. The Full Bench observed that mere
summoning of a person during an enquiry under the Customs Act did not amount to arrest
so as to attract the provisions of Article 22(2) of the Constitution of India and the stand
taken that the persons arrested under the Customs Act should be produced before a
Magistrate without unnecessary delay from the moment the arrest is effected, had to fail.
23

. We are unable to appreciate the views of the Full Bench of the Madras High Court and
reiterate the decision of this Court in Niranjan Singhs case (supra). In our view, the law
relating to the concept of "arrest" or "custody" has been correctly stated in Niranjan
Singh's case (supra). Paragraphs 7, 8 and the relevant portion of paragraph 9 of the
decision in the said case states as follows:- AIR 1980 SC 785

"7. When is a person in custody, within the meaning of S. 439, Cr. P.C.? When he is, in
duress either because he is held by the investigating agency or other police or allied
authority or is under the control of the court having been remanded by judicial order, or
having offered himself to the court's jurisdiction and submitted to its orders by physical
presence. No lexical dexterity nor precedential profusion is needed to come to the
realistic conclusion that he who is under the control of the court or is in the physical hold
of an officer with coercive power is in custody for the purpose of S.439. This word is of
elastic semantics but its core meaning is that the law has taken control of the person. The
equivocatory quibblings and hide-and-seek niceties sometimes heard in court that the
police have taken a man into informal custody but not arrested him, have detained him
for interrogation but not taken him into formal custody and other like terminological
dubiotics are unfair evasion of the straightforwardness of the law. We need not dilate on
this shady facet here because we are satisfied that the accused did physically submit
before the Sessions Judge and the jurisdiction to grant bail thus arose.
8. Custody, in the context of S.439, (we are not, be noted, dealing with anticipatory bail
under S.438) is physical control or at least physical presence of the accused in court
coupled with submission to the jurisdiction and order of the court.
9. He can be in custody not merely when the police arrest him, produces him before a
Magistrate and gets a remand to judicial or other custody. He can be stated to be in
judicial custody when he surrenders before the court and submits to its
directions..................."
Sections 107 and 108 of the Customs Act do not contemplate immediate arrest of a
person being summoned in connection with an enquiry, but only contemplates
surrendering to the custody of the Customs Officer which could subsequently lead to
arrest and detention.
24. We also agree with Mr. Anoop Chaudhary's submission that unless a person accused
of an offence is in custody, he cannot move the Court for bail under Section 439 of the
Code, which provides for release on bail of any person accused of an offence and in
custody (Emphasis supplied). The pre-condition, therefore, to applying the provisions of
Section 439 of the Code is that a person who is an accused must be in custody and his
movements must have been restricted before he can move for bail. This aspect of the
matter was considered in Niranjan Singh's case where it was held that a person can be
stated to be in judicial custody when he surrenders before the Court and submits to its
directions.
25. It is no doubt true that in the instant case the accused persons had appeared before the
concerned Magistrates with their learned advocates and on applying for bail were granted
bail without being taken into formal custody, which appears to have swayed one of the
Benches of the Punjab and Haryana High Court to take a liberal view and to hold that no
arrest had actually been effected. The said view, in our opinion, is incorrect as it goes
against the very grain of Sections 46 and 439 of the Code. The interpretation of "arrest"
and "custody" rendered by the Full Bench in Roshan Beevi's case (supra) may be relevant
in the context of Sections 107 and 108 of the Customs Act where summons in respect of
an enquiry may amount to "custody" but not to "arrest", but such custody could
subsequently materialize into arrest. The position is different as far as proceedings in the
court are concerned in relation to enquiry into offences under the Indian Penal Code and
other criminal enactments. In the latter set of cases, in order to obtain the benefit of bail
an accused has to surrender to the
@page-SC1088
custody of the Court or the police authorities before he can be granted the benefit
thereunder. In Vol.11 of the 4th Edition of Halsbury's "Laws of England" the term "arrest"
has been defined in paragraph 99 in the following terms:-
"99. Meaning of arrest.- Arrest consists in the seizure or touching of a person's body with
a view to his restraint; words may, however, amount to an arrest if, in the circumstances
of the case, they are calculated to bring, and do bring, to a person's notice that he is under
compulsion and he thereafter submits to the compulsion."
26

. The aforesaid definition is similar in spirit to what is incorporated in Section 46 of the


Code of Criminal Procedure. The concept was expanded by this Court in State of Uttar
Pradesh vs. Deomen (AIR 1960 SC 1125) wherein it was inter alia observed as follows:-
(Para 12)

"Section 46, Cr.P.C. does not contemplate any formality before a person can be said to be
taken in custody. Submission to the custody by words of mouth or action by a person is
sufficient. A person directly giving a police officer by word of mouth information which
may be used as evidence against him may be deemed to have submitted himself to the
custody of the Police Officer."
27. The sequester of the above is that when a person, who is not in custody, approaches
the police officer and provides information, which leads to the discovery of a fact, which
could be used against him, it would be deemed that he had surrendered to the authority of
the investigating agency.
28. It must, therefore, be held that the views expressed by the High Court in Dinesh
Kumar's writ petition regarding arrest were incorrect, while the views expressed in the
writ petitions filed by Lalit Kumar and Bhupinder correctly interpreted the meaning of
the expressions "arrest" and "custody". However, how far the same would apply in the
ultimate analysis relating to the filling up of column 13(A) is another matter altogether.
29. In our view, the reasoning given in Dinesh Kumar's case in that context is a possible
view and does not call for interference under Article 136 of the Constitution. Conversely,
the decision rendered in the writ petitions filed by Lalit Kumar and Bhupinder has to be
reversed to be in line with the decision in Dinesh Kumar's case. When the question as to
what constitutes "arrest" has for long engaged the attention of different High Courts as
also this Court, it may not be altogether unreasonable to expect a layman to construe that
he had never been arrested on his appearing before the Court and being granted bail
immediately. The position would have been different, had the person concerned not been
released on bail. We would, in the facts of these cases, give the benefit of a mistaken
impression, rather than that of deliberate and wilful misrepresentation and concealment of
facts, to the appellants in the second of the two appeals as well, while affirming the view
taken by the High Court in Dinesh Kumar's case.
30. Accordingly, although, we are of the view that the legal position as to what constitutes
arrest was correctly stated in the writ petitions filed by Lalit Kumar and Bhupinder, we
confirm the order passed in Dinesh Kumar's case and extend the same benefit to Lalit
Kumar and Bhupinder also.
31. In the result, the Civil Appeal arising out of SLP(C) No. 1840 of 2007 is dismissed,
while the Civil Appeal arising out of SLP(C) No.14939 of 2007 is allowed. The Judgment
of the High Court dated 22nd September, 2005, impugned in the said appeal, is set aside
and the concerned respondents are directed to take steps to issue appointment letters to
the appellants in the said appeals subject to fulfillment of other conditions by them. It is
also made clear that the appellants will be deemed to have been appointed as Constable-
Drivers with effect from the date, persons lower in merit to them were appointed.
However, while they will be entitled to the notional benefits of such continuous
appointment, they will be entitled to salary only from the date of this judgment on the
basis of such notional benefits.
32. The appeals are disposed of accordingly.
33. In the peculiar facts of the case, the parties will bear their own costs.
Order accordingly.
@page-SC1089
AIR 2008 SUPREME COURT 1089 "Commr., Municipal Corporation, Hyderabad v. P.
Mary Manoranjani"
(From : Andhra Pradesh)*
Coram : 2 S. B. SINHA AND H. S. BEDI, JJ.
Civil Appeal No. 341 of 2008 (arising out of S.L.P. (C) No. 5885 of 2005), D/- 11 -1
-2008.
Commissioner, Municipal Corporation, Hyderabad and Ors. v. P. Mary Manoranjani and
Anr.
Constitution of India, Art.14, Art.16, Art.309 - EQUALITY IN PUBLIC
EMPLOYMENT - SERVICE MATTERS - APPOINTMENT - TERMINATION OF
SERVICE - Appointment - Voluntary workers - Appointment to posts in Municipal
Corporation - Exemption from requirements of sponsorship of candidate by Employment
Exchange - Does not exempt employer-Municipal Corporation from obligations to
comply with requirements of Art.14 and Art.16 or other statutory Rules - Candidate not
only failing to appear at written test but also failing to attend duties for number of years -
Such candidate, held, did not have right to continue in service - Termination of services
challenged in writ jurisdiction - Direction by High Court to reconsider case in light of
exemption that was granted, held not sustainable.
W.A. No. 1714 of 2004, D/-04-11-2004 (AP), Reversed. (Paras 4, 5, 13, 14, 17)
Cases Referred : Chronological Paras
2006 AIR SCW 1991 : AIR 2006 SC 1806 : 2006 (3) AIR Kar R 320 (Rel. on) 14
L.N. Rao, Sr. Advocate, G. Ramakrishna Prasad, Suyodhan Byrapaneni, Siddharth
Patnaik, G. Arun, for Appellants; Anil Kumar Tandale, for Respondents.
* W.A. No. 1714 of 2004, D/- 4-11-2004 (AP).
Judgement
1. S. B. SINHA, J. :- Leave granted.
2. Respondent No. 1 joined the services of Appellant-Corporation as Balwadi Teacher on
an honorarium of Rs. 100/- per month.
3. The Corporation requested the State of Andhra Pradesh to grant exemption in regard to
requirements of sponsorship of the candidates by the Employment Exchange for
appointment in the regular posts, pursuant whereto G.O. Ms. No.27 M.A. (Q), dated 16th
January, 1991 was issued, stating :-
The Commissioner, Municipal Corporation of Hyderabad in his letter 2nd read above has
stated that the Municipal Corporation of Hyderabad has been implementing the Urban
Community Development Programme since 1967 that under this programme a number of
Balwadi and Sewing Centres were opened in the slum areas for the benefit of the slum
Woman and Children and a grant of Rs.250/- p.m. was paid by the Municipal Corporation
of Hyderabad to the Balwadi Teachers and that there is a long standing demand from
these persons for absorption into posts with a regular scale of pay as most of them are
working as Teachers from 10 to 15 years. Therefore, the Commissioner, Municipal
Corporation of Hyderabad has requested the Government to exempt the voluntary
workers from the Employment Exchange Procedure so that they may be considered for
appointment in the existing and future vacancies of Municipal Corporation of Hyderabad
as Lower Division Clerks, Lower Division Typists, Bill Collectors, Record Assistants or
any other posts for which they are eligible.
2. The Government having carefully examined the proposal of the Commissioner,
Municipal Corporation of Hyderabad hereby accord exempting 214 voluntary workers
listed in the annexure to this order from Employment Exchange procedure so that they
may be considered for appointment as LDCs, L.D. Typists, Bill Collectors, Record
Assistants or any other posts for which they are eligible in the existing and future
vacancies.
3. The Commissioner, Municipal Corporation of Hyderabad is requested to take
necessary action accordingly.
4. The said order clearly postulates that what was exempted was requirement of
sponsorship of the candidates by the Employment Exchange and not the selection process
itself.
5. Appellant is a State within the meaning of Article 12 of the Constitution of India. It
was, therefore, obligated to undertake the selection process in terms of the constitutional
scheme envisaged under Articles 14 and 16 of the Constitution of India.
6. Respondent, for the purpose of her selection in a regular post created by the appellant-
Corporation, was thus, required to appear for a vive voce before a Selection Committee
as also a written test. She appeared in the interview on 24th December, 1991. She,
however, did not appear in the written test. From April, 1989 she absented herself from
her duties. As she had unauthorisedly remained absent continuously for long time, a letter
dated 2nd June,
@page-SC1090
1992 was received by the Corporation from Jai Prakash Nagar Welfare Association
requesting the Corporation to terminate her services.
7. Having come to learn thereabout the respondent made a request before the appellant on
12th February, 1993 that she be provided with a job of typist stating :-
"I am to state here that I passed Higher Typewriting in English and S.S.C. passed I have
attended the interview on 26.12.91, by (but) could not attend written test and Typewriting
exam as I am late and there was a communication gap.
I request you sir kindly to appoint me a typist as I am fully qualified to hold the post. My
colleagues have already been appointed. If I am provided with the job, I shall ever remain
grateful."
8. It, therefore, stands admitted that she had not appeared at the written test as also
typewriting examination. She also having realized that she had been absenting from her
duties continuously, purported to file some medical certificates for treating her to be on
leave from 1st May, 1989 to 19th February, 1993. Appellant by its letter dated 10th
March, 1993 asked her to produce copies of the representations made by her during the
period of her absence as also the acknowledgement receipts thereto to examine her case.
The period of absence having not satisfactorily been explained by the respondent, her
services were terminated by an order dated 24th July, 1998 stating :-
13. The request of Smt. Mary Manoranjani has been examined with reference to the
records available and it is clearly established that she had absented from duty w.e.f. April,
1989 onwards for reasons best known to her. There is nothing on record to show that she
has either submitted any leave application or any medical certificate during the period of
her absence, until she again submitted application on 20.02.1993 requesting for
permission to join today. The Balwadi/Sewing teachers have been appointed by the
respective welfare associations located in slums and they were not appointed by MCH.
Smt. Mary Manoranjani has already been communicated that her request for rejoining
into duty is rejected vide this office letter 7th cited. Smt. Mary Manoranjani has not put
forth any new grounds to reconsider her case."
9. Aggrieved thereby she filed a writ petition before the High Court of Judicature at
Andhra Pradesh in August, 1998. By a judgment and order dated 9th March, 2004 a
learned Single Judge of the High Court, without entering into the merit of the matter,
allowed the said petition stating :-
"Admittedly, the Government has issued G.O.Ms. No.27 on 16.1.1991 and as per the
contents of the said G.O. voluntarily workers working in the Municipal Corporation of
Hyderabad should be absorbed in regular vacancies by relaxing the rules relating to
employment exchange etc., and from the papers produced by the learned counsel for the
petitioner, it is clear that a list of casual workers was prepared and the petitioner is placed
at serial No. 100. When once the petitioner is in the list of candidates and the purpose of
G.O.Ms. No.27 is to give relaxation of the existing rules and for absorption of those who
are working as casual workers as on the date of the G.O., it cannot be said that the
petitioners case cannot be considered simply on the ground that as on the date of
interview she was not in actual service. Hence, I deem fit to direct the respondents to
consider the case of the petitioner for appointment as Lower Division Typist or in any
other equivalent post in terms of G.O.Ms. No.27 dated 16.1.1991."
10. An intra Court appeal preferred by the appellant thereagainst has been dismissed by a
Division Bench of the said High Court by reason of the impugned judgment.
11. Mr. L.N. Rao, learned senior counsel appearing on behalf of the appellants, submitted
that the impugned orders of the High Court are ex facie illegal as in a case of this nature
the aforesaid G.O.Ms, cannot be said to have any application whatsoever.
12. Mr. Anil Kumar Tandale, learned counsel appearing on behalf of the respondent, on
the other hand, submitted that keeping in view the tenor of the order passed by the
learned Single Judge which has been affirmed by the Division Bench, as a mere direction
for consideration of the case of the respondent in terms of the said G.O.M. has been
made, no interference therewith by this Court is warranted.
13. Constitutional scheme in regard to public employment as enumerated in Articles 14
and 16 of the Constitution of India is explicit. Any appointment made by a State within
the meaning of Article 12 of the Constitution of India must be subject to the
@page-SC1091
constitutional scheme. In making appointments the State is obligated to comply with the
same as also statutory requirements, if any. Neither the appellant nor the State could grant
any exemption in regard to compliance of the statutory requirements.
14

. The G.O.M. granted only exemption from sponsorship of the names by the Employment
Exchange. Appellant did not and in fact could not ask for grant of any exemption from its
obligation to comply with the requirements of Articles 14 and 16 of the Constitution of
India or other Statutory Rules operating in the field in this behalf. It is now a well settled
principle of law that any appointment made in violation of the statute or the constitutional
provision would be illegal. (See Secretary State of Karnataka and others v. Uma Devi (3)
and others : (2006) 4 SCC 1. 2006 AIR SCW 1991

15. G.O. Ms. No. 27 M.A. (Q) dated 16th January, 1991, therefore, is not at all applicable
in a case of this nature, where a candidate not only did not appear at the written test for
the purpose of recruitment to the regular post but also failed to attend to her duties for a
number of years. Respondent was engaged for a particular purpose, namely to impart
education to the poor children. She failed to carry out her contractual obligations.
16. Only when she came to learn of the fact that a complaint had been made against her,
she requested either for her recruitment as a typist or grant her leave on medical ground,
which ex-facie appears to be mala fide.
17. We, therefore, are of the opinion that she did not have any legal right to continue in
the said post. The direction of the High Court to consider her case in the light of G.O.Ms.
No. 27 M.A. (Q) dated 16th January, 1991 is eminently unsustainable as the said
Government order would have no application to the facts and circumstances of the case.
18. For the reasons abovementioned the impugned judgment cannot be sustained and is
set aside accordingly. The appeal is allowed. However, there shall be no order as to costs.
Appeal allowed.
AIR 2008 SUPREME COURT 1091 "Shaikh Majid v. State of Maharashtra"
(From : Bombay)*
Coram : 2 Dr. A. PASAYAT AND AFTAB ALAM, JJ.
Criminal Appeal No. 132 of 2008 (arising out of S.L.P. (Cri.) No. 2809 of 2007), D/- 21
-1 -2008.
Shaikh Majid and Anr. v. State of Maharashtra and Ors.
(A) Penal Code (45 of 1860), S.300 - MURDER - Murder - Non-explanation of injuries
on accused - May affect prosecution case - Non-explanation however not always fatal.
Non-explanation of the injuries on the accused by the prosecution may affect the
prosecution case. But such a non-explanation may assume greater importance where the
defence gives a version which competes in probability with that of the prosecution. But
where the evidence is clear, cogent and creditworthy and where the Court can distinguish
the truth from falsehood, the mere fact that the injuries are not explained by the
prosecution cannot by itself be a sole basis to reject such evidence, and consequently, the
whole case. Much depends on the facts and circumstances of each case. It is not an
invariable rule that the prosecution has to explain the injuries sustained by the accused in
the same occurrence. When the prosecution comes with a definite case that the offence
has been committed by the accused and proves its case beyond any reasonable doubt, it
becomes hardly necessary for the prosecution to again explain how and under what
circumstances injuries have been inflicted on the person of the accused. It is more so
when the injuries are simple or superficial in nature. (Paras 9, 10)
(B) Penal Code (45 of 1860), S.300, Exception 4 - MURDER - APPLICABILITY OF AN
ACT - CULPABLE HOMICIDE - Applicability - Ingredients to be established - Incident
taking place in sudden fight between two religions groups over offering of prayers -
Single knife blow given to deceased - Knife used was also small - Accused not guilty of
murder - Liable to be convicted u/S.304, Part I. (Paras 13, 17)
(C) Penal Code (45 of 1860), S.300 - MURDER - APPLICABILITY OF AN ACT -
@page-SC1092
Murder - Single blow - Does not always rule out applicability of S.300. (Para 16)
Cases Referred : Chronological Paras
2006 AIR SCW 1678 : 2006 Cri LJ 2111 (Rel. on Pnt. B) 15
AIR 1990 SC 1459 : 1990 Cri LJ 1510 : 1990 All LJ 415 (Rel. on) 9
AIR 1976 SC 2263 : 1976 Cri LJ 1736 (Ref.) 7, 8
AIR 1972 SC 2593 : 1973 Cri LJ 44 (Rel. on) 10
AIR 1968 SC 1281 : 1968 Cri LJ 1479 (Ref.) 6, 9
Shakil Ahmed Syed, Faiyz Ahmad Syed, Mohd. Yasir Abbasi, Firasat Ali Siddiqui, for
Appellants; Ravindra Keshavrao Adsure, for Respondents.
* Cri. Appeal No. 314 of 1991, D/- 24-1-2007 (Bom.) (Aurangabad Bench).
Judgement
Dr. ARIJIT PASAYAT, J. :- Leave granted.
2. Challenge in this appeal is to the judgment of the Division Bench of the Bombay High
Court at Aurangabad, dismissing the appeal filed by the appellants. By a common
judgment two appeals were disposed of. Both the appeals were directed against the
judgment and order passed by learned Third Additional Sessions Judge, Aurangabad, in
Sessions Case No. 39 of 1990. Originally there were 37 accused persons in the Sessions
Case who faced trial for offences punishable under Ss. 302, 307, 395, 436, 324, 323, 147,
148 and 149 of the Indian Penal Code, 1860 (in short the 'IPC'), S. 25 of the Indian
Telegraph Act, (in short the 'Telegraph Act') and S. 3 of Prevention of Damage to Public
Properties Act, 1984 (in short the 'Public Property Act') and Ss. 3 and 4 of the Religious
Institutions (Prevention of Misuse) Act, 1988 (in short the 'Religious Institutions Act').
Learned trial Judge convicted the present appellants who were accused Nos. 1 and 26
respectively for the offences punishable under S. 302 read with S. 34 and S.324 read with
S. 34, I.P.C. Imprisonment for life and fine of Rs. 1,000/ with default stipulations and
three years imprisonment and fine of Rs. 1,000/- with default stipulation were imposed.
Appellant No. 2-Shaikh Abbas was also convicted for offences punishable under S. 452
read with S. 149, I.P.C. and was sentenced to five years' rigorous imprisonment and fine
of Rs. 1000/- with default stipulation. He was also convicted for offences punishable
under S. 435 read with S. 149, I.P.C., and for offences punishable under S. 323 read with
S. 149, I.P.C. The sentences were directed to run concurrently and it was further directed
that in case the fine amount was realised, part of it, i.e. Rs. 10,000/-, was to be paid to the
widow of Jayanarayan (hereinafter referred to as the 'deceased'). Three prosecution
witnesses were also directed to be paid compensation of Rs. 1,000/-. Law was set into
motion on the basis of the statement given by the one Krishna (P.W. 4), Sandu, Police
Inspector (P.W. 11), recorded the statement which was treated as the First Information
Report (in short the 'FIR'). The information given by him was to the following effect :
"Krishna P.W. 4 has alleged in the First Information Report Exhibit 63 that the occurrence
in the question took place on 10-10-1989. On that day at about 4 p.m., he along with
about 200 villagers had been to a temple for darshana and also for performing pooja on
the eve of Dashera festival. All of them went to temple and Shami tree near the cremation
ground on the outskirts of village Balanagar, Tq. Paithan. As usual, the villagers had been
to the place of pooja in a procession with drums etc. After pooja they entered the gate of
village (Ves) and had been to the Ram and Maroti temples for offering their prayers. At
this time, about 250-300 people from mosque, adjacent to Ram temple, had started
pelting stones at them. While pelting stones they were exhorting that it was a time for
offering prayer (Namaz) and, therefore, these people should leave the place immediately.
Krishna P.W. 4, deceased-Jayanarayan, Pandharinath, Babasaheb and other villagers have
noted the presence of about 3-32 accused persons. According to them, these 32 accused
persons along with 200-250 persons assaulted them with sticks, stones, knives etc. the
assault was on the people belonging to Hindu religion. P.W. 4 Krishna had been assaulted
by accused No. 1-Shaikh Majid and accused No. 26-Shaikh Abbas with knives. Such
assault was on his right hand and below shoulder, on right shoulder as well as in the
stomach. He suffered wounds. Such stabbing is suffered by Ranganath, Laxman,
Harichand, Gorakh, Hanuman, Uttarm, Bhausaheb, Badri Narayan and Dr. Gopi Kisan as
well as Laxman Shamrao. Other persons also sustained serious injuries.
3. As noted above, on the basis of information given by Krishna (P.W. 4) case was
@page-SC1093
registered under Ss. 307, 147, 148, 149 and 323, I.P.C. Investigation was undertaken and
on completion thereof charge-sheet was filed. At this juncture, it is to be noted that
another FIR was lodged at about 6 p.m. by appellant-accused No. 1 against Krishna (P.W.
4) and others. Charge-sheet thereafter was filed. Undisputedly, there was order of
acquittal in the said case. The trial Court placing reliance on the evidence adduced, held
present appellants guilty of offence committed punishable under S.302 read with S. 34,
I.P.C. They were also found guilty of having committed offence punishable under S. 324
in relation to the injury caused to Krishna P.W. 4. As noted above, appellant-Shaikh
Majid was also convicted under various other provisions. The stand of the appellants
before the High Court was that the evidence of the so-called eye-witnesses P.Ws. 4, 5, 6
and 7 was not consistent. Only one blow was given and that in course of a sudden
quarrel, injuries were sustained by the accused persons. The prosecution has, therefore,
suppressed the truth. The plea was not accepted and as noted above their appeal was
dismissed.
4. The stand taken by the appellant before the High Court was reiterated.
5. Learned counsel for the State supported the judgment of the trial Court as affirmed by
the High Court.
6. One of the pleas is that the prosecution has not explained the injuries on the accused.
The issue is, if there is no such explanation what would be its effect? We are not prepared
to agree with the learned counsel for the defence that in each and every case where the
prosecution fails to explain the injuries found on some of the accused, the prosecution
case should automatically be rejected, without any further probe. In Mohar Rai v. State of
Bihar (AIR 1968 S 1281) it was observed (AIR p. 1284, para 6) :
"In our judgment the failure of the prosecution to offer any explanation in that regard
shows that evidence of the prosecution witnesses relating to the incident is not true or at
any rate not wholly true. Further those injuries probabilise the plea taken by the
appellants."
7

. In another important case Lakshmi Singh v. State of Bihar (1976 (4) SCC 394) after
referring to the ratio laid down in Mohar Rai case (supra) this Court observed (SCC p.
401, para 12) : AIR 1976 SC 2263 at p. 2269, Para 11

"Where the prosecution fails to explain the injuries on the accused, two results follow :
(1) that the evidence of the prosecution witnesses is untrue; and (2) that the injuries
probabilise the plea taken by the appellants."
8

. It was further observed that (SCC p. 401, para 12) : AIR 1976 SC 2263 at p.
2269, Para 11

"In a murder case, the non-explanation of the injuries sustained by the accused at about
the time of the occurrence or in the course of altercation is a very important circumstance
from which the Court can draw the following inferences :
(1) that the prosecution has suppressed the genesis and the origin of the occurrence and
has thus not presented the true version;
(2) that the witnesses who have denied the presence of the injuries on the person of the
accused are lying on a most material point and, therefore, their evidence is unreliable;
(3) that in case there is a defence version which explains the injuries on the person of the
accused it is rendered probable so as to throw doubt on the prosecution case.
The omission on the part of the prosecution to explain the injuries on the person of the
accused assumes much greater importance where the evidence consists of interested or
inimical witnesses or where the defence gives a version which competes in probability
with that of the prosecution one."
9

. In Mohar Rai case it is made clear that failure of the prosecution to offer any
explanation regarding the injuries found on the accused may show that the evidence
related to the incident is not true or at any rate, not wholly true. Likewise in Lakshmi
Singh case, it is observed that any non-explanation of the injuries on the accused by the
prosecution may affect the prosecution case. But such a non-explanation may assume
greater importance where the defence gives a version which competes in probability with
that of the prosecution. But where the evidence is clear, cogent and creditworthy and
where the Court can distinguish the truth from falsehood, the mere fact that the injuries
are not explained by the prosecution cannot by itself be a sole basis to reject such
evidence, AIR 1968 SC 1281
AIR 1990 SC 1459

@page-SC1094
and consequently, the whole case. Much depends on the facts and circumstances of each
case. These aspects were highlighted by this Court in Vijayee Singh v. State of U.P. (1990
(3) SCC 190).
10

. Non-prosecution of injuries by the prosecution will not affect the prosecution case
where injuries sustained by the accused are minor and superficial or where the evidence
is so clear and cogent, so independent and disinterested, so probable, consistent and
creditworthy, that it outweighs the effect of the omission on the part of the prosecution to
explain the injuries. As observed by this Court in Ramlagan Singh v. State of Bihar (1973
(3) SCC 881) the prosecution is not called upon in all cases to explain the injuries
received by the accused persons. It is for the defence to put questions to the prosecution
witnesses regarding the injuries on the accused persons. When that is not done, there is no
occasion for the prosecution witnesses to explain any injury on the person of an accused.
In Hare Krishna Singh v. State of Bihar (1988 (2) SCC 98), it was observed that the
obligation of the prosecution to explain the injuries sustained by the accused in the same
occurrence may not arise in each and every case. In other words, it is not an invariable
rule that the prosecution has to explain the injuries sustained by the accused in the same
occurrence. If the witnesses examined on behalf of the prosecution are believed by the
Court in proof of guilt of the accused beyond reasonable doubt, question of obligation of
the prosecution to explain injuries sustained by the accused will not arise. When the
prosecution comes with a definite case that the offence has been committed by the
accused and proves its case beyond any reasonable doubt, it becomes hardly necessary
for the prosecution to again explain how and under what circumstances injuries have
been inflicted on the person of the accused. It is more so when the injuries are simple or
superficial in nature. AIR 1972 SC 2593
11. The residual question is the applicability of S. 302, I.P.C. As noted above it was
submitted that only one blow was given and that too in the course of a sudden quarrel.
12. In essence, the stand of learned counsel for the appellant is that Exception IV to S.
300, I.P.C. would apply to the facts of the case.
13. For bringing in operation of Exception 4 to S. 300, I.P.C., it has to be established that
the act was committed without premediation, in a sudden fight in the heat of passion upon
a sudden quarrel without the offender having taken undue advantage and not having acted
in a cruel or unusual manner.
14. The Fourth Exception to S. 300, I.P.C. covers acts done in a sudden fight. The said
exception deals with a case of prosecution not covered by the First Exception, after which
its place would have been more appropriate. The Exception is founded upon the same
principle, for in both there is absence of premediation. But, while in the case of Exception
1 there is total deprivation of self-control, in case of Exception 4, there is only that heat
of passion which clouds men's sober reason and urges them to deeds which they would
not otherwise do. There is provocation in Exception 4 as in Exception 1; but the injury
done is not the direct consequence of that provocation. In fact Exception 4 deals with
cases in which notwithstanding that a blow may have been struck, or some provocation
given in the origin of the dispute or in whatever way the quarrel may have originated, yet
the subsequent conduct of both parties puts them in respect of guilt upon equal footing. A
"sudden fight" implies mutual provocation and blows on each side. The homicide
committed is then clearly not traceable to unilateral provocation, nor in such cases could
the whole blame be placed on one side. For if it were so, the Exception more
appropriately applicable would be Exception 1. There is no previous deliberation or
determination to fight. As fight suddenly takes place, for which both parties are more or
less to be blamed. It may be that one of them starts it, but if the other had not aggravated
it by his own conduct it would not have taken the serious turn it did. There is then mutual
provocation and aggravation, and it is difficult to apportion the share of blame which
attaches to each fighter. The help of Exception 4 can be invoked if death is caused (a)
without premediation; (b) in a sudden fight; (c) without the offender having taken undue
advantage or acted in a cruel or unusual manner; and (d) the fight must have been with
the person killed. To bring a case within Exception 4 all the ingredients mentioned in it
must be found. It is to be noted that the "fight" occurring in Exception 4 to S. 300, I.P.C.
is not defined in I.P.C. It takes two to make a fight. Heat of
@page-SC1095
passion requires that there must be no time for the passions to cool down and in this case,
the parties have worked themselves into a fury on account of the verbal altercation in the
beginning. A fight is a combat between two or more persons whether with or without
weapons. It is not possible to enunciate any general rule as to what shall be deemed to be
a sudden quarrel. It is a question of fact and whether a quarrel is sudden or not must
necessarily depend upon the proved facts of each case. For the application of Exception
4, it is not sufficient to show that there was a sudden quarrel and that there was no
premediation. It must further be shown that the offender has not taken undue advantage
or acted in cruel or unusual manner. The expression "undue advantage" as used in the
provision means "unfair advantage."
15
. The above position is highlighted in Sandhya Jadhav v. State of Maharashtra (2006) 4
SCC 653). 2006 AIR SCW 1678

16. It cannot be said that whenever a single blow is given, that would not attract S. 302,
I.P.C.
17. In the instant case considering the background facts and the nature of the weapon
used i.e. small knife the appropriate conviction would be under S. 304, Part I, I.P.C. and
not S. 302, I.P.C. Accordingly the conviction is altered, custodial sentence of ten years
would suffice.
18. The appeal is allowed to the aforesaid extent.
Order accordingly
AIR 2008 SUPREME COURT 1095 "K. T. Palanisamy v. State of Tamil Nadu"
(From : Madras)*
Coram : 2 S. B. SINHA AND DALVEER BHANDARI, JJ.
Criminal Appeal No. 1107 of 2005, D/- 11 -1 -2008.
K.T. Palanisamy v. State of T.N.
(A) Penal Code (45 of 1860), S.300 - Evidence Act (1 of 1872), S.3 - MURDER -
EVIDENCE - Murder - Circumstantial evidence - Deceased who allegedly performed
pooja near river on advise of appellant did not return home - Circumstances of last seen
with appellant and recovery of gold chain of deceased relied on - However, corpus delicti
had not been proved - Dead body of deceased was not recovered - All witnesses being
related witnesses, chances of their deposing falsely not ruled out - No missing report was
lodged - Recovery of gold chain by itself would not prove guilt - Conviction of accused is
not proper.
1991 AIR SCW 1368, Disting.
Crl. A. No. 996 of 1998, D/-18-4-2002 (Mad), Reversed. (Paras 11, 12, 13, 14, 16, 17)
(B) Evidence Act (1 of 1872), S.3 - EVIDENCE - MURDER - Murder case -
Circumstantial evidence - All the links in chain must be found to be completed, to form
basis for conviction.
AIR 1984 SC 1622, Foll. (Para 12)
Cases Referred : Chronological Paras
1991 AIR SCW 1368 : AIR 1991 SC 1463 : 1991 Cri LJ 1845 (Disting.) 17
AIR 1984 SC 1622 : 1984 Cri LJ 1738 (Foll.) 12
AIR 1973 SC 2622 : 1973 Cri LJ 1783 12
C. Rose, Murali C. Krishna, C. Balakrishna, for Appellant; V. Kanakaraj, S. Joseph
Aristotle, S. Prabu Ramasubramanian, V.G. Pragasam, for Respondent.
* Cri. A. No. 996 of 1998, D/- 18-4-2002 (Mad).
Judgement
S. B. SINHA, J. :- Appellant with two others, namely, A2, Vellingiri, and A3, Officer @
Paramasivam, were prosecuted for commission of the offence of murder of one
Somasundaram.
Appellant was an astrologer. The deceased was passing through tough times. His son had
also remained ill for long time. He allegedly was advised by the appellant to perform
some poojas on the bed of the river Bhavani situated at Nanjaipuliyampatti on or about
29-4-1996. He went to the place for the said purpose along with the accused persons. The
deceased at that time was said to be wearing a gold chain (M. O.1) and two gold rings
(M. Os. 2 and 3 engraved with the letters "P. Mani". The deceased was allegedly last seen
by PW3 and PW4 namely, Nallasamy and K. Devaraj respectively and his brother-in-law
PW, Muthusamy. He, however, did not come back. On inquiries having been made from
the appellant, the family members of the deceased were informed that he had gone away
after performing the said pooja for attending some function.
@page-SC1096
He was even thereafter not heard of for a long time.
An advertisement was issued in a local newspaper on 12-5-1996. Poongodi (PW1), the
wife of the deceased, in order to search out her husband, in turn, sought for the assistance
of the appellant, who advised them to perform a pooja at Bannavi Amman Temple.
However, when the parents of the deceased and his grand-mother left for Bannavi
Amman Temple for performing the pooja in the company of the appellant, they also did
not return home. From a report which appeared in a local newspaper on 3-6-1996 PW1
came to learn that the said three persons have been found murdered at Erode. He went to
Erode and lodged first information report before the Erode, South Police Station, which
was recorded by Manoharan (PW16). A first information report was registered being
crime No. 415/96 under the caption "man missing". On 4-7-1996 the statement of PW-1
was recorded by the Investigating Officer PW-18. The Banglaputhur Police station which
had the jurisdiction to investigate into the said matter received the said first information
report on 2-9-1996, on the basis whereof PW17 the Head Constable of the said police
station registered a case as Crime No. 406/96 against the accused under Sections 302 and
379 of the Indian Penal Code (for short the 'IPC'). Accused Nos. 1 and 2 were arrested on
6-7-1996 at a bus stop at Erode in the presence of Abdulhasan Ansari (PW-11). Allegedly,
on the basis of a purported confessional statement made by the appellant, the
Investigating Officer and others went to Coimbatore for recovery of a gold chain which is
said to have been sold to one Dhanasekaran. The said gold chain and some bangles,
however, were seized in connection with the murder of the deceased's parents and grand-
mother. He also took the police party to a jewellery shop belonging to one Dhanasekaran
who produced a long golden chain. The same was seized and marked as M. O. 1.
2. Accused No. 2 allegedly was having a gold ring in his finger which was seized and
marked before the learned Trial Judge as M. O. 3.
3. Accused No.3 was arrested near a municipal guest house at Erode on 7th July, 1996.
Two criminal cases were instituted against the appellants; -
one for commission of murder of Somasundaram; and the other for murder of his parents
and grand-mother.
Whereas the first case was registered as S. C. No. 70/97, the second one was registered as
S. C. No. 100/97.
4. The learned Sessions Judge convicted the appellant and other two accused persons for
commission of offence under Sections 120B, 302 read with Sections 34 and 109 of the
IPC as also Sections 379 and 201 thereof. They were sentenced to undergo rigorous
imprisonment for life under Sections 120B, 302 read with Section 34 and Section 109 of
the IPC and one year's rigorous imprisonment under Section 379 and three years rigorous
imprisonment under Section 201 of the IPC.
5. Before the learned Trial Judge, a large number of witnesses were examined on behalf
of the prosecution. PW1 is the widow of the deceased. Her evidence centered around the
fact that the deceased used to consult him as he was an astrologer. She further stated that
the deceased had gone with the accused for performing the said pooja and never came
back thereafter. She identified Material Objects 1, 2, 3. According to her, a letter was
received from one Bala in the name of the first accused stating that her husband was
keeping well but was in a depressed mind and that within a month he would go back
home.
6. Allegedly, on the hope generated from the said letter, no first information report was
lodged. As noticed hereinbefore, she went back to the appellant and requested him to find
out her husband, who in turn advised her to perform a pooja at Pannari Mariamman
temple. Her in-laws and mother of her mother-in-law went there on 28-6-1996 and the
news about their murder appeared in the newspaper on 30th June, 1996. According to her,
she became suspicious on learning that her in-laws have been murdered and as her
husband was also missing, she filed a first information report on 3-7-1996 at Erode.
In the cross examination of the said witness, several discrepancies have been brought on
record particularly the omissions in her statement under Section 161 of the Code of
Criminal Procedure, that her husband had been putting on M. Os. 1 to 3. She accepted
that there used to be quarrel between her and her husband. The distance
@page-SC1097
between her residence and that of her in-laws was 5 to 6 kms. They were living
separately. She admitted that her husband had not been in sound mental health. She had
been informed that her husband was to go to a function after performance of the said
pooja and the same statement was made by her before the police. Loganathan (PW2) was
to be the brother-in-law of the deceased appellant. According to him, he used to run an
electrical shop with the deceased Somasundaram and one Nallasamy. The deceased,
according to him, used to come to the appellant to discuss astrological aspects as regards
his child who had not been keeping good health. He deposed that on 29-4-1996
Somasudaram came to his shop along with the appellant and informed him that they were
going to perform a pooja in the Vannan temple whereafter he would go to some function.
Although according to him the deceased was wearing gold ornaments (viz. the rings and
the chain) but he did not know to whom they belonged to.
7. PW - 3 was also a partner in the said electrical shop in which the deceased and PW2
were running. According to him on 29-4-1996 he met the deceased at Gobi
Chettipalayam when he informed him that he had been coming from Nanjai Pulliampatti
after performing the pooja. He furthermore informed that they were going to a temple
whereupon he came back by bus. He could not remember the date when Somasundaram
brought the three accused to his shop.
8. PW4 is K. Devaraj. He was a valuer working at the Veerappan Chattiram Cooperative
Bank. He also saw the deceased going to perform pooja with the accused.
PW-5 is Muthusamy. He is an electrician. He is the brother-in-law of the deceased.
Allegedly, he saw the deceased and the other accused sitting in the bus and on a querry
made by him, they informed that they were going to the temple at Nanjaipulimpatti and at
about 8.30 p. m. on that day when he was returning from Bangalaputhur, all the three
accused also boarded the same bus. On a querry again made by him in regard to
whereabouts of the deceased, he was told that he had gone to attend some function.
Although he is a resident of the area he did not know as whether there was a river bridge
at Puliampatty or not.
9. PW-6 is Thangavel. He is the brother-in-law of the deceased. He is said to have given
advance of a sum of Rs. 10,000/- on execution of a pronote to accused No. 3, at the
instance of the deceased. The letter which was marked as Ex. A6, according to him,
contained some zig-zag lines at the time of his deposition which were not there when it
was received.
10. PW-7 is Marisamy. According to him, Dhanasekaran had paid a sum of Rs. 15,000/-
to the accused persons. He accepted that he was a police informer and had been appearing
for them as a witness.
PW8 has, however, denied that he was a jeweller or that he had a shop.
11. On analysis of the entire materials brought on records by the prosecution, the only
relevant evidence is the purported recovery of chain (M. O. 1) at the instance of the
appellant
The dead body of the deceased was not recovered. There is no evidence in regard to
death. Nothing has been brought on record to show that there was enough water in the
river or the current in the water was such so as to take a dead body away.
All the prosecution witnesses are related to the deceased. It is difficult for us to believe
that all the witnesses saw the deceased accompanying the accused persons one after the
other at different places. Therefore, chances of their deposing falsely cannot be ruled out.
Be that as it may, when the offence is said to have been committed and the circumstantial
evidence is made the basis for establishing the charge against the appellant, indisputably
all the links must be completed to form the basis for his conviction.
12

. It is now well settled that in a case where an offence is said to have been established on
circumstantial evidence alone, indisputably all the links in the chain must be found to be
complete as has been held in Sharad Birdhichad Sarda v. State of Maharashtra (AIR 1984
SC 1662) in the following terms : (Para 152)

"A close analysis of this decision would show that the following conditions must be
fulfilled before a case against an accused can be said to be fully established :
(1) the circumstances from which the conclusion of guilt is to be drawn should be fully
established.

It may be noted here that this Court indicated AIR 1973 SC 2622

@page-SC1098
that the circumstances concerned 'must or should' and not 'may be' established. There is
not only a grammatical but a legal distinction between 'may be proved' and 'must be or
should be proved as was held by this Court in Shivaji Sahebrao Bobade v. State of
Maharashtra where the following observations were made :
certainly, it is a primary principle that the accused must be and not merely may be guilty
before a Court can convict, and the mental distance between 'may be' and 'must be' is
long and divides vague conjectures from sure conclusions.
(2) the facts so established should be consistent only with the hypothesis of the guilt of
the accused, that is to say, they should not be explainable on any other hypothesis except
that the accused is guilty.
(3) the circumstances should be of a conclusive nature and tendency.
(4) they should exclude every possible hypothesis except the one to be proved, and
(5) there must be a chain of evidence so complete as not to leave any reasonable ground
for the conclusion consistent with the innocence of the accused and must show that in all
human probability the act must have been done by the accused.
153. These five golden principles, if we may say so, constitute the panchsheel of the
proof of a case based on circumstantial evidence."
13. In this case, corpus delicti has not been proved. The same need not be but the death as
a fact must be proved. Even death has not been proved in this case. No piece of mortal
remains of the deceased was found. If the prosecution witnesses are to be believed they
had no reason to suspect the appellant herein at the relevant point of time. They knew that
the deceased was to attend another function. We fail to understand as to why the deceased
would take all the accused to the shop of PW2 or allowed to be found in their company
by all of his relations and partners. None of the witnesses testified that they were seen
near the place of worship. None said that they were found to be performing any pooja. No
evidence was adduced to show that any pooja was performed in a temple.
14. In a situation of this nature, it is difficult to hold that a judgment of conviction can be
founded on the sole circumstance of the deceased's having been last seen with the
appellant by the prosecution witnesses who are all interested and partisan witnesses.
More significant is the conduct of the prosecution witnesses. On the day of the alleged
crime, they did not suspect the appellant in any manner whatsoever. They did not even go
to the place of the occurrence. Despite the fact that he was missing, the purported
explanation of the appellant was taken for granted. Even no missing report was lodged. It
was expected that such missing report should have been lodged immediately and that
details of his wearing apparels as also the fact that he had two rings on his finger and one
gold chain would have been mentioned.
The fact that the deceased was last seen with the appellant should have been specifically
disclosed in the first information report. Suspicion was raised about the involvement of
the appellant only because three other dead bodies were recovered. We do not know the
nature of evidence that has been adduced in that case. We need not enter into any surmise
in this behalf.
15. In any event, the circumstantial evidence which formed part of the records of SC 100
of 1997 could not be relied upon for arriving at the conclusion that the appellant herein is
guilty of commission of the said offence.
16. The only other circumstance is recovery of the golden chain. It was allegedly sold to
PW8. He, however, has denied his involvement. Even assuming that golden chain was
recovered at the instance of the appellant herein, the same by itself, in our considered
view, would be sufficient for upholding the judgment and conviction under Section 302
of the IPC.
17

. Mr. V. Kanakaraj, learned senior counsel appearing on behalf of the respondent, has
placed strong reliance on a decision of this Court in Sevaka Perumal and Anr. v. State of
Tamil Nadu reported in (1991 (3) SCC 471). Therein also it was held that the fact of the
death of the deceased must be established like any other fact. In that case it was not done.
This Court in that case gave an instance where a corpus delicti is not possible to be traced
or recovered. The same being that the murder was committed and the dead body was
thrown into the river, 1991 AIR SCW 1368

@page-SC1099
stream or burnt out. Even such is not the case here.
As indicated herein before, the fact that the river was a tidal one had not been proved.
There is, thus, no reliable or acceptable evidence that the offence has been committed by
the appellant. Neither any direct nor circumstantial evidence had been brought on record
to establish the guilt on the part of the appellant herein.
18. We, therefore, are of the opinion that the impugned judgment cannot be sustained,
which is set aside accordingly. The appeal is allowed. The appellant is in jail. He is
directed to be set at liberty unless wanted in connection with any other case.
Appeal allowed.
AIR 2008 SUPREME COURT 1099 "Deputy Inspector General of Police v. K. Ravinder
Rao"
(From : Andhra Pradesh)*
Coram : 2 A. K. MATHUR AND H. S. BEDI, JJ.
Civil Appeal No. 499 of 2008 (arising out of SLP (C) No. 18176 of 2006), D/- 18 -1
-2008.
Deputy Inspector General of Police and Anr. v. K. Ravinder Rao.
Constitution of India, Art.226, Art.311 - WRITS - TERMINATION OF SERVICE -
CONCURRENT FINDINGS - ADMINISTRATIVE TRIBUNAL - REINSTATEMENT -
SERVICE MATTERS - Writ jurisdiction - Concurrent finding of fact - Interference -
Termination of service - Police Constable charged of misbehaving with lady by going to
her house in drunken state for satisfying his sensual lust - Concurrent finding by Inquiry
Officer and Administrative Tribunal that delinquent was guilty of misconduct -
Reapprecia-tion of evidence by High Court to reach at different conclusion is unjustified -
Order of High Court directing reinstatement of Police constable liable to be set aside.
W.P. No. 32232 of 1997, D/-29-08-2005 (A.P.), Reversed. (Paras 5, 6)

Mrs. D. Bharathi Reddy, for Appellants; Jayanth Muth Raj and C.K. Sasi, for
Respondents.
* W.P. No. 32232 of 1997, D/- 29-8-2005 (A.P.)
Judgement
1. A. K. MATHUR, J. :- Delay condoned.
2. Leave granted.
3. This appeal is directed against the order passed by the Division Bench of the Andhra
Pradesh High Court whereby the Division Bench of the High Court has set aside the
order of the A. P. Administrative Tribunal and directed reinstatement of the respondent
with 50% backwages. Aggrieved against this order the present appeal has been filed by
the Deputy Inspector General of Police, Hyderabad Range and another.
4. Brief facts which are necessary for disposal of this appeal are the respondent herein
was appointed as a Police Constable in 1979 and while he was working as such at Uppal
Police-Station he was placed under suspension on 9.5.1985 on the ground that he visited
the house of one Smt. Kamasani Susheela in a drunken state and demanded her to provide
girls for satisfying his sexual lust and when she refused, the respondent scuffled with her.
When she raised alarm another Police Constable, Jagan Mohan Reddy, who was on
picket duty rushed there and pulled the respondent from the house. On this misconduct by
the respondent an inquiry was conducted by serving a proper charge-sheet. The Inquiring
Officer after hearing both the parties, found the respondent guilty and thereafter, his
explanation was called for as to why he should not be removed from service.
Subsequently he was removed from service by order dated 1.10.1993. Then he filed an
appeal before the appellate authority. That was rejected by order dated 30.4.1994. After
that the respondent challenged that order by filing an original application being
O.A.No.1489 of 1994 before the Administrative Tribunal. The Administrative Tribunal
also affirmed the impugned order of removal from service. Aggrieved against that the
respondent filed a writ petition before the High Court and the grievance of the respondent
was that he was not afforded sufficient opportunity, the documents were not given to him
and the finding of the Inquiring Officer was perverse and unsustainable. The writ petition
was opposed by the appellants before the High Court. The High Court after reviewing the
evidence and after going through the statement of P.W.2, Smt. Kamasani Susheela and the
statement of P.W.3, her sister and other two constables
@page-SC1100
on duty, found the charges established but the High Court observed that the Inquiring
Officer has given too much importance on the evidence of P.Ws. 1 and 12 who were his
superior officers. But the High Court found that there was no evidence that the
respondent went to the house of Smt. Kamasani Susheela in a drunken state as alleged.
But at the same time the High Court observed that there was some evidence that the
respondent misbehaved with P.W.2. The Division Bench of the High Court held that
imposition of punishment of removal from service was certainly disproportionate. The
Division Bench of the High Court further observed that the finding of the disciplinary
authority appeared to be perverse and the evidence has not been properly evaluated in its
proper perspective. Consequently, the High Court set aside the order of the Tribunal as
well as the order of the Inquiring Officer and directed reinstatement of the respondent in
service as he has been out from service for 12 years i.e. since 1993 and directed payment
of 50% of backwages. The High Court observed that this may not be treated as a
precedent in future. Aggrieved against this impugned order of the High Court the
appellants have preferred the present appeal.
5. We have heard learned counsel for the parties and perused the records. We have gone
through the order passed by the Tribunal as well as the Inquiring Officer. We regret that
the view taken by the High Court does not appear to be well founded. It is unfortunate
that a Police Constable who is supposed to safeguard the public makes such a
unreasonable demand on going to someone's house for satisfying his sensual lust. It is
disgrace in uniform. The Tribunal has examined the matter in detail and after considering
the matter affirmed the order of removal of the respondent. The Tribunal has also found
that the findings given by the Inquiring Officer are sound and proper. The Tribunal
examined the evidence and found that the testimony of P.W.2 has been corroborated by
the evidence of P.W.3, Smt. Kamasani Laxmi and P.W. 1, Circle Inspector of Police, who
submitted a report finding the allegation true. P.Ws.7 and 9, both Constables supported
the version of P.W.2. The Circle Inspector investigated the matter further and confirmed
the incident that the respondent misbehaved with Smt. Kamasani Susheela and he was in
a drunken condition so much so that when the respondent went to the Doctor for some
medical treatment at the relevant time the Doctor declined to administer injection as the
respondent was drunk. Therefore, all the evidence has been again examined by the
Tribunal in objective manner and rightly affirmed the report of the Inquiring Officer.
6. It is strange that the High Court sitting under Article 226 of the Constitution of India
re-appreciated the evidence and came to a different conclusion which is not within the
scope of the High Court. The finding given by the Inquiring Officer has been affirmed in
appeal and the same having been examined by the Tribunal in threadbare there was no
justification for the High Court to come to its own conclusion when there was concurrent
finding given by the Inquiring Officer and the Tribunal. But the High Court appreciated
the whole evidence which was unwarranted. The respondent was drunk as is apparent
from the testimony of the Doctor to whom the appellant had approached for some
treatment and wanted to administer injection but having seen him in a drunken state the
Doctor declined. This is sufficient to prove that the appellant was drunk. Secondly, when
the testimony of Smt. Kamasani Laxmi, supported by two beat Constables as well as by
the Circle Inspector that the respondent went to the house of Smt. Kamasani Susheela and
approached her for providing some girls to satisfy his sensual lust, the High Court went
wrong in recording its finding. Time and again this Court has emphasized that under
Article 226 of the Constitution of India, appreciation of evidence should not be done in
matters of this nature unless the finding appears to be perverse. In the present case the
finding having been examined in detail by the Tribunal and the Tribunal also having
found no perversity in the finding of the Inquiring Officer, we fail to appreciate the
approach of the High Court. Hence, we allow this appeal and set aside the order of the
High Court and confirm the order of the Tribunal. There would be no order as to costs.
Appeal allowed.
@page-SC1101
AIR 2008 SUPREME COURT 1101 "Food Corporation of India v. M/s. Seil Ltd."
(From : Delhi)*
Coram : 2 S. B. SINHA AND J. M. PANCHAL, JJ.
Civil Appeal No. 370 with 372 and 371 of 2008 (arising out of SLP (C) No. 5599 with
7316 and 7666 of 2006), D/- 11 -1 -2008.
Food Corporation of India and Anr. v. M/s. Seil Ltd. and Ors.
(A) Essential Commodities Act (10 of 1955), S.3(2)(f) - ESSENTIAL COMMODITIES -
Levy sugar - Supply of, by respondents to appellants-FCI - Withholding of payment by
appellants on ground of alleged shortage in supply during earlier years - Supply of sugar
was made in terms of a statutory order as also on directions issued by Central
Government - Central Govt. scrutinised bills and verified claims of respondents before
issuing direction to make payment - There was no protest or demur of shortages by third
party or recipient - Appellants could not have withheld payment on purported shortages
made by respondent many years back - Direction therefore issued to appellants to pay
amount due with interest. (Paras 10, 11, 18, 19)
(B) Constitution of India, Art.226 - WRITS - Writ jurisdiction - Contractual disputes
involving public law element - Are amenable to writ jurisdiction. (Para 10)
(C) Constitution of India, Art.14 - EQUALITY - OBJECT OF AN ACT - Scope - Art.14
has received liberal interpretation over the years - Its scope has also been expanded by
creative interpretation of Court. (Para 17)
(D) Constitution of India, Art.226 - WRITS - REVIEW - PRINCIPLES - Writ Court -
Power of review - While exercising said jurisdiction, it not only acts as a Court of law but
also as a Court of equity - A clear error or omission on the part of Court to consider a
justifiable claim as to payment of interest on principal sum - Would be thus subject to
review; amongst others on principle of actus curiae neminem gravabit (An act of the
Courts shall prejudice none). (Para 17)
Cases Referred : Chronological Paras
(2004) 3 SCC 553 15
AIR 1962 SC 1320 16
Amarendra Sharan, A.S.G., Ajit Pudussery, K. Vijay Kumar, Ms. Rekha Bakshi and
Vivek Sigh, with him for Appellants; Sudhir Chandra Agarwal, Sr. Advocate, Praveen
Kumar, Manish Bishnoi and T.V. George, with him for Respondents.
* L.P.A. Nos. 331-352, 451, 452, 1131 of 2005, D/-07-12-2005, reported in AIR 2006
(NOC) 816 : 2006 (127) Delhi LT 611 and C. M. No. 1252 of 2006 in L. P. A. No. 331 of
2005, D/- 24-1-2006 (Del.)
Judgement
S. B. SINHA, J. :- Leave grated.
1. Parliament of India enacted Essential Commodities Act, 1995 (The Act). In terms of
Section 3(2)(f) thereof, the Central Government is empowered to direct any manufacturer
of sugar to sell the said commodity to the Central Government or a State Government or
to a body owned or controlled by them for the purpose of making it available to the
public at a fair price. It is commonly known as 'levy sugar.' Price of such levy sugar is
fixed by the Central Government in exercise of its power under Section 3(3C) of the Act
on yearly basis. 'Sugar year' commences from the month October of the year. Price of
levy sugar although is required to be notified at that time, admittedly, there exists a
practice to notify the previous year's price as a levy sugar on an ad hoc basis price in
October and final price therefor is notified later on.
2. Pursuant to or in furtherance of a notification issued by the Central Government under
the Act and the directions issued by the competent authority from time to time, levy sugar
was supplied by the respondents to the agencies of Central Government as also the
appellant.
3. Respondents herein received allotment letters for supply of sugar both to FCI as also
UPPCF. Claims were lodged for the price of levy sugar both with the FCI as also the
Directorate of Sugar, Ministry of Food. The Central Government sanctioned the claim of
the respondent in respect of the sugar supplied to UPPCF. It made similar claim in respect
of the sugar supplied to the appellant. Appellant, however, demanded for a no dues
certificate. It raised other objections including weight and quality of the sugar in relation
to the supplies made to the Central Government. Respondents contended that no
complaint having been made by the Central Government in this connection the action of
the appellant was totally unjustified.
We may, however, notice that withholding of payment was, inter alia, made by the
@page-SC1102
appellant for the alleged shortages in supply of sugar during the period 1983 to 1995.
4. Respondents filed writ applications before the High Court of Delhi. A learned single
Judge of the said Court classified the cases into two categories; (1) supplies made to the
State Government, the Central Government; and their other agencies in respect whereof
the appellant only had the authority to make payment, and (2) supplies made to the
appellant.
5. So far as the supplies made to the Central Government and other agencies are
concerned, it was held that a direction for making the payment should be made but in
respect of the supplies made to the appellant; any resolution setting the controversy was
held to be impermissible in a writ proceeding therefor and the respondents were relegated
to the remedy of a civil suit for recovery of respect of the amount claimed by them.
6. A review petition was filed by the respondents pointing out that no direction has been
made in regard to payment of interest and by a judgment and order dated 29th April, 2005
interest was directed to be paid.
7. Appeals preferred there against by the appellant herein have been dismissed by a
Division Bench of the High Court by reason of the impugned judgment.
8. Mr. Amarendra Sharan, learned Additional Solicitor General of India appearing on
behalf of the appellant submitted :
1. Transactions between the parties being contractual in nature, no writ petition was
maintainable.
2. Respondents, having alleged breach of contract on the part of the appellant, the writ
petition should not have been entertained.
3. Several disputed questions of fact including the quality and quantity of sugar having
been raised, the High Court committed a serious error in determining the said question in
a writ proceeding.
4. In any event, direction to pay interest in the review proceeding was wholly
impermissible in law.
Mr. Sudhir Chandra Agarwal, learned senior counsel appearing for the respondents, on
the other hand, contended :
1. Supply of sugar having been made in terms of a statutory order, the writ petition was
maintainable.
2. Food Corporation of India could not have withheld payment in respect whereof there
was no dispute.
3. Lawful payment cannot be withheld on the purported plea of non-supply of entire
quantity of sugar in the earlier years.
8A. Admittedly, supplies were made to FCI and UPPCF in terms of the allotment orders
received by the respondents. The Central Government verified the bills in terms of the
circular letters issued by it from time to time. The claim in terms of the said circulars was
to be submitted to the Directorate of Sugar directly. Appellant was merely to pay the
difference in the prices of sugar for the years in question keeping in view the price
notification dated 22-10-1993 and 17-1-1994. Bills were forwarded to the Food
Corporation of India by the concerned authority for making payment.
Appellant, in its counter affidavit before the High Court, inter alia, averred that as in
respect of supply of sugar in earlier years, certain claims had been made by it, payment
was rightly withheld, stating:
"Since the shortages mentioned in the preceding paras were detailed in the seal intact
wagons, therefore, the petitioners were fully responsible /liable for compensating the
losses caused to the Respondents on this account. However, there happened some delay
in working out compiling the accurate shortages at our level and as such factual position
could not be intimated to the petitioners in time. The shortages relates for the period from
1983 to 1995 i.e. 12 years."
9. We have noticed that the mode in which supplies were to be made have been laid down
in the circular letters issued by the Central Government. The responsibility of the mill
owner was to supply at the rail head. The fact that transportations of the commodity were
made only by rail is not in dispute. If any shortage was found during transit, in terms of
the policy decision of the Central Government, claims were to be raised by the appellant
with the Railway Authorities.
10. When supply of sugar was made in terms of a statutory order as also on the directions
issued by the Central Government and in the cases there did not exist any factual dispute,
we do not see any reason as to why the writ petitions would not be maintainable.
It is now no longer res integra that contractual
@page-SC1103
disputes involving public law element are amenable to writ jurisdiction. In these cases,
the Central Government not only scrutinized the bills but also verified the claims of the
respondents. A direction was issued to make payment. Appellant, which is a 'State' within
the meaning of Article 12 of the Constitution of India, withheld payment without any
legal justification.
11. The High Court referred to several letters issued by the Central Government to arrive
at the conclusion that where sugar had been lifted by a third party without any complaint,
protest or demur of shortages, there was no reason as to why payment therefore could not
be made.
12. Appellant could not have withheld payment on the basis of the purported shortages in
supply of sugar under the contracts made by the respondents many many years back, save
and except under the terms of binding contract.
13. We have noticed hereinbefore that the High Court had divided the cases in two
categories. In regard to the supplies made by the respondents to the Central Government
and/or its agencies wherewith appellants had no concern, it could not have denied
payment on the pretext of shortage or quality of the sugar supplied, particularly, when the
recipient did not raise such a question.
14. The Central Government, issued a letter dated 17th November, 1972 on which
reliance has been placed by the appellant itself before the High Court; clause (vii)
whereof reads as under :
"On receipt of dispatch instructions, the District Manager at dispatching and will arrange
full payment including excise duty to the mills for road movement. As regards,
movement by rail full payment may be made in two instalments; first being @ Rs. 15/-
per quintal. After making initial payment inspection of the stocks should be arranged and
mills should be asked and perused to place indents for wagons immediately. Balance
amount will be paid to the Mills as soon as wagons are placed. To save time lag,
cheques/demand drafts should be kept ready and handed over the mills as soon as wagons
are made available, as the mills may hesitate loading wagons unless full payment is made
particularly when the consignees will be FCI and ownership of the Cargo will be changed
as soon as stocks are loaded.
Excise duty will also be paid along with the final payment for stocks RRs will be freight
to pay and in favour of FCI as consignee. Payment shall be made through cheques and in
case of any objection from the mill regarding acceptance of the cheques, payments may
be made either by demand draft or cheques certified as good for payment. Funds shall be
arranged by the District Managers directly from the Head Office as is being done in the
case of food grain purchase.
Posting of additional staff at the mill point is under consideration and after decision is
taken follow up action should be taken by the Regional Managers. The staff at the mill
would be responsible to undertake inspection of quality, check weighment, indent of
wagons and look to other general arrangements about transport and dispatch......These
transport charges will be incorporated by the mills in the bills and will be paid by FCI.
Wagons will be booked against clear RRs in the name of receiving District Managers and
would be sent to the letter promptly.
Stocks by rail shall move against clear RRs and it shall therefore be the responsibility of
the receiving District Managers to account for the weight of sugar properly. In case of
any shortage/damages of sugar in transit, the claims for the same should be lodged
promptly with the railways, in accordance with the standing instructions on the subject."
15. Jurisdiction of the High Court to entertain a writ application involving contractual
matter was considered by a Bench of this Court in ABL International Ltd. and Anr. v.
Export Credit Guarantee Corporation of India Ltd. and Ors. ((2004) 3 SCC 553) wherein
upon referring to a large number of decisions, it was held :
"23. It is clear from the above observations of this Court, once the State or an
instrumentality of the State is a party of the Contract, it has an obligation in law to act
fairly, justly and reasonably which is the requirement of Article 14 of the Constitution of
India. Therefore, if by the impugned repudiation of the claim of the appellants the first
respondent as an instrumentality of the State has acted in contravention of the abovesaid
requirement of Article 14, then we have no hesitation in holding that a writ Court can
issue suitable directions to set
@page-SC1104
right the arbitrary actions of the first respondent."
16. Reliance placed by Mr. Sharan on M/s. Burmah Construction Company v. The State
of Orissa and Ors. (AIR 1962 SC 1320) is not apposite. Claim made therein was a pure
money claim. It was in that situation observed that the High Court normally does not
entertain a petition under Article 226 of the Constitution to enforce a civil liability arising
out of a breach of contract to pay an amount of money due to the claimant.
17. Article 14 of the Constitution of India has received a liberal interpretation over the
years. Its scope has also been expanded by creative interpretation of the Court. The law
has developed in this field to a great extent. In this case, no disputed question of fact is
involved.
The High Court, in an appropriate case, may grant such relief to which the writ petitioner
would be entitled to in law as well as in equity.
We do not, thus, find any substance in the contention of Mr. Sharan that while exercising
its review jurisdiction, no interest on the principal sum could have been directed to be
granted by the High Court. A writ court exercises its power of Review under Article 226
of the Constitution of India itself. While exercising the said jurisdiction, it not only acts
as a Court of law but also as a court of equity. A clear error or omission on the part of the
Court to consider a Justifiable claim on its part would be subject to review; amongst
others on the principle of actus curiae neminem gravabit (An act of the Courts shall
prejudice none). We appreciate the manner in which the learned Judge accepted his
mistake and granted relief to the respondents.
18. We, however, although agree with the opinion of the Division Bench of the High
Court on the legal principle in regard to payment of interest, as has been enunciated by it,
having regard to the fact that the respondents did not prefer any appeal, are of the opinion
that increase in the rate of interest, as has been directed by the Division Bench, cannot be
upheld.
19. We, therefore, in modification of the order passed by the Division Bench, direct that
the appellant would pay the amount in question with interest as awarded by the learned
single Judge of the High Court.
20. Subject to above, the appeals are dismissed. Respondents are also entitled to costs
quantified at Rs. 1,00,000/- (Rupees one lakh only) in each case.
Appeals dismissed.
AIR 2008 SUPREME COURT 1104 "S. Raju v. C. Sathammai"
(From : Madras)*
Coram : 2 G. P. MATHUR AND AFTAB ALAM, JJ.
Civil Appeal No. 480 of 2008 (arising out of SLP (C) No. 128 of 2007), D/- 17 -1 -2008.
S. Raju v. C. Sathammai.
Civil P.C. (5 of 1908), O.37, R.3 - SUMMARY SUIT - PROMISSORY NOTE -
Summary suit - Leave to defend - Suit instituted on basis of promissory note - Defendant
uneducated person seeking leave to defend - Alleged that plaintiff took his signatures on
blank stamp papers on pretext that those were for receipts of payments made to him for
work done - Plaintiff used one of those signatures to forge promissory note for filing suit
- Rejection of application of defendant on ground of inconsistency in his stand - Liable to
be set aside being based on technical view of matter - Leave to defend granted subject to
condition of depositing part of plaintiffs claim.
CPR (NPD) No. 1226 of 2000, D/-27-09-2006 (Mad), Reversed. (Para 9)

S.G.K. Kumar and Ms. Srikala Gurukrishan Kumar, for Appellant.


* CRP (NPD) No. 1226 of 2000, D/- 27-9-2006 (Mad.)
Judgement
JUDGMENT :-Heard counsel for the appellant. No one appears for the respondent
despite notice.
2. Leave granted.
3. This appeal is directed against the orders passed by the City Civil Court and the High
Court denying the appellant-defendant the leave to defend the suit filed by the respondent
under Order 37 of the Code of Civil Procedure (CPC for short).
4. The respondent-plaintiff instituted the suit on the basis of a promissory note, dated
November 11, 2004, for Rs. 1,50,000/- along with interest at the rate of 25% per annum
allegedly signed by the appellant in the presence of two witnesses.
@page-SC1105
5. On notice by the court, the appellant filed a petition under Rule 3, Order 37 of CPC
seeking leave to defend the suit without any condition. On behalf of the appellant, it was
stated that the promissory note, forming the basis of the defendants claim was completely
sham and fabricated. It was further stated that he was an uneducated and illiterate person,
engaged in the work of civil construction, as a contractor. He lived in the same locality
and had agreed to build the house of the appellant's son. He completed the construction of
the house at a relatively much cheaper rate of Rs. 430/- per square ft. The defendant-
respondent/her son used to take his signatures on blank stamp papers telling him that
those were for receipts of the payments made to him and were required for income tax
purposes. Being a simple, uneducated person he put his signatures on blank papers
without any question and in good faith. It was alleged that one of the signatures made by
him was later used to forge the promissory note for filing the suit.
6. It was also stated on his behalf that the alleged signatures on the promissory note were
not his signatures as would be apparent from the fact that there were two signatures on
the promissory note, one in English and the other in Telugu.
7. The trial court noted that the contentions raised by the appellant for defending the suit
were quite inconsistent. On the one hand, he denied the signatures on the promissory note
as his signatures and, on the other hand, it was stated that his signatures were obtained on
blank stamp papers on the pretext that those were to be made into receipts for payments
made to him and one of those signatures was used for creating the promissory note. The
trial court accordingly rejected the petition filed by the appellant under Order 37, Rule 3,
CPC.
8. Against the order passed by the trial court, the appellant moved the High Court in
revision but the High Court dismissed the revision and affirmed the order passed by the
trial court primarily on the ground that there was an inherent inconsistency in the case of
the appellant.
9. On hearing the counsel for the appellant and on going through the materials on record,
we feel that the trial court and High Court have taken a rather technical view of the
matter. On a careful consideration of the matter, we are satisfied, that in the overall facts
and circumstances of the case, the petitioner ought to have at least been allowed to defend
the suit, subject to the condition of depositing a part of the plaintiffs claim. We
accordingly allow the appeal, set aside the orders of the trial court and the High Court
and direct that the petitioner may be granted leave to defend the suit subject to deposit of
Rs. 50,000/- in the trial court. The leave shall be granted to the appellant provided the
amount, as directed above, is deposited within two months from today.
Appeal allowed.
AIR 2008 SUPREME COURT 1105 "K. Sharada Bai v. Shamshunnisa"
(From : 2003 (1) Andh LT 467)
Coram : 2 Dr. A. PASAYAT AND P. SATHASIVAM, JJ.
Civil Appeal Nos. 1526-1527 of 2005, D/- 24 -1 -2008.
K. Sharada Bai and Anr. v. Smt. Shamshunnisa and Ors.
Constitution of India, Art.133 - A.P. Land Grabbing (Prohibition) Act (12 of 1982),
S.2(d), S.8(1) - APPEAL - LAND - INJUNCTION - SPECIAL COURT - Finding based
on evidence - Interference with - Perpetual injunction - To restrain appellants from
interfering with exclusive possession of land of respondent - Respondent alleged that
appellants had grabbed portion of her land - She had duly established her case by placing
oral and documentary evidence - Special Court concluded that respondent is owner of
land in question and rival title set up by appellants is not true and valid - Finding that
appellants are land grabbers was based on appreciation of evidence, revenue records and
report of Commissioner - No interference. (Paras 7, 8)
Roy Abraham, Mrs. Seema Jain, Vimlesh Kumar and Himinder Lal, for Appellants; Mrs.
K. Amreswari. Sr. Advocate, V.S. Raju and Debasis Misra, for Respondents.
Judgement
P. SATHASIVAM, J. :- These appeals are directed against the final judgment and order
dated 25.06.2002 passed by the High Court of Judicature, Andhra Pradesh at Hyderabad
in Writ Petition Nos. 29675 and 29712 of 1997 in and by which the High Court dismissed
these writ petitions filed by the appellants herein.
2. BRIEF FACTS :
@page-SC1106
The schedule land forms part of Sy. No. 30 of Taranagar Village, Serilingampally
Mandal, Ranga Reddy District and the total extent of which is Ac. 3.19 guntas. Out of the
said extent, one Chakali Ramaiah owned an extent of Ac. 1.29 guntas of land and one
Katika Baloji owned an extent of Ac. 1.30 guntas. Out of total extent of Ac. 1.29 guntas,
Chakali Ramaiah sold an extent of Ac. 1.00 to Smt. Shamshunnisa Begum, contesting
respondent No.1 herein and 20 guntas to one Jahangir and retained the balance of 9
guntas. Katika Baloji sold an extent of 30 guntas to Smt. K. Sharada Bai, appellant No. 1
herein and 1 acre to H. Padmini Bhai, appellant No.2 herein. The contesting respondent
filed O.S. No. 87 of 1988 on the file of the Munsif Magistrate, West and South, R.R. Dist.
for a perpetual injunction restraining appellants herein and others acting on their behalf
from interfering with the exclusive possession and enjoyment of her 1 acre land. By order
dated 14.07.1995, the said suit was transferred to the Special Court constituted under the
A.P. Land Grabbing (Prohibition) Act, 1982, (hereinafter referred to as the Act) and
numbered as L.G.C. No. 133 of 1995. On its transfer, it was tried along with L.G.C. No.
162 of 1994 which was filed by respondent No. 1 herein alleging that the appellants
grabbed 12.5 guntas of land out of her 1 acre land. During the pendency of the
application, the Special Court appointed an Advocate-Commissioner to inspect and
measure the disputed land and the Commissioner filed a report before the Court which is
filed as Annexure P-1 along with the S.L.P. On 15-10-1997, the Special Court by a
common judgment allowed both the L.G.Cs holding the appellants herein as land
grabbers and directed to deliver the vacant possession of 12.5 guntas of land to the 1st
respondent. Aggrieved by the said order, the appellants filed Writ Petition Nos. 29675
and 29712 of 1997 before the High Court. The High Court dismissed the writ petitions
holding that the Special Court has not committed any error in allowing the L.G.Cs.
Questioning the same, the appellants filed the above appeals by way of special leave.
3. We heard Mr. Roy Abraham, learned counsel appearing for the appellants and Mrs. K.
Amareswari, learned senior counsel appearing for respondent No. 1.
4. The only question to be considered in these appeals is whether the order passed by the
Special Court and the impugned order of the High Court upholding the decision of the
Special Court is sustainable or not?
5. Though in the grounds of appeal an objection was raised about the jurisdiction of the
Special Court constituted under the Act, no argument was advanced with regard to the
same. On the other hand, the appellants challenged the merits of the impugned orders and
the ultimate conclusion arrived. The contesting respondent filed an application under
Section 8(1) of the Act to declare the appellants herein as land grabbers and evict them
from an extent of 15 guntas of land forming part of Sy. No. 30 of Taranagar Village. She
filed counter contending that she is bona fide purchaser and she is in possession and
enjoyment of her property since the date of purchase and perfected title to the schedule
property by adverse possession. Before the Special Court, common evidence was
recorded. On behalf of the petitioners, PWs 1 and 2 were examined and Ex. A-l to A-21
were marked. On behalf of the respondents, RW 1 was examined and EX. B1 to B-14
were marked. The Special Court examined CW-1 and Ex. Cl to C-8 were marked. The
Special Court, on appreciation of oral and documentary evidence, found that the applicant
before it is the owner of 12½ guntas of land forming part of Sy. No. 30 as specifically
shown in the sketch of the Commissioner and declared the respondents as land grabbers
and directed to deliver possession as far as L.G.C. No. 162 of 1994 is concerned and
granted permanent injunction to an extent of 27½ guntas of land in S.No. 30 against the
respondents in L.G.C. No. 133 of 1995. The said order was confirmed by the High Court.
6. It is useful to refer the definition of Land Grabbing and Land Grabbers as defined in
Section 2(e) and Section 2(d) of the Act respectively :
"Section 2(e) - "land grabbing" means every activity of grabbing of any land (whether
belonging to the Government, a local authority, a religious or charitable institution or
endowment, including a wakf, or any other private person) by a person or group of
persons, without any lawful entitlement and with a view to illegally taking possession of
such lands, or enter into or create illegal tenancies or lease and licences agreements or
any other illegal agreements in respect of such lands, or to construct
@page-SC1107
unauthorized structures thereon for sale or hire, or give such lands to any person on rental
or lease and licence basis for construction, or use and occupation, or unauthorized
structures; and the term to grab land shall be construed accordingly."
"Section 2(d) "land grabber" means a person or a group of persons who commits land
grabbing and includes any person who gives financial aid to any person for taking illegal
possession of lands or for construction of unauthorized structures thereon, or who collects
or attempts to collect from any occupiers of such lands rent, compensation and other
charges by criminal intimidation, or who abets the doing of any of the above mentioned
acts; and also includes the successors-in-interest."
In view of the above statutory provisions and of the claim of the applicant that she is the
original owner of the schedule property and her land was grabbed by the respondents, the
initial burden is on her to prove her right and title to the property and if the same is
discharged, the burden shifts on the respondents. It is not in dispute that the land was
purchased by the applicant and respondent Nos. 1 and 2 forming part of Sy. No. 30 of
Taranagar Village. It is also not disputed that Sy. No. 30 was not sub divided. It is the
claim of the applicant that she purchased the property from Ramaiah and S.
Krishnamurthy under a registered sale deed dated 15-02-1979 which is marked as Ex. A-
1 which is part of Sy. No.30. Before the Special Court, the applicant very much relied on
the report of the Mandal Revenue Officer. On the direction of the Court, a Commissioner
was appointed, who after inspection submitted a report. Based on the oral and
documentary evidence coupled with the report of the Mandal Revenue Officer as well as
the Commissioner, the Special Court found that the applicant is in possession of 27½
guntas of land and the adjoining 12½ guntas of land forming part of Sy. No. 30 which is
claimed by the applicant is adjoining to the said land. The Special Court disbelieved the
claim of the respondents that there is a boundary wall in between those lands i.e., 27½
guntas of land and 12½ guntas of land and rightly rejected their stand.
7. Mrs. K. Amareshwari, learned senior counsel appearing for the contesting respondent
before us by taking us through the relevant portion of the order of the Special Court
submitted that the applicant has duly established her case by placing oral and
documentary evidence and the Special Court after accepting the same and basing reliance
on the records as well as the report of the Commissioner rightly passed an order which
was confirmed by the High Court. In the light of the submission, we verified the order of
the Special Court and the materials placed before it. It shows that after tabulating all the
details furnished by the applicant and the respondents, it concluded as follows :
"8-x ........................ Thus the respondents 1 and 2 or their successors-in-interest are in
occupation of land which does not belong to them. The report of the Commissioner
shows that about 12½ guntas as shown in the sketch appended to the report of the
Advocate-Commissioner is in the occupation of R1 and R2 or their vendees. In the
absence of any evidence to show that 12½ guntas of land belongs to R1 and R2 and that it
lies in Survey Number 30 A, it shall be presumed that the said land which is in Sy. No.30
and which abuts the extent of 27½ guntas of land of the applicant, belongs to the
applicant, particularly when it is shown in Ex. B6 to Ex. B13 that one of the survey
numbers in which plots 50 to 55 lie, is 30 AA. In fact the area covered by plots 49 to 55 is
the disputed land."
"For the foregoing discussion, we hold that the appellant is the owner of 12½ guntas of
land forming part of the land in Sy. No. 30 as shown in the sketch appended to the report
of the Commissioner and that the rival title set up by R1 and R2 over the said land is not
true and valid."
Inasmuch as the above conclusion is based on the appreciation of oral and documentary
evidence led by the applicant and the respondents as well revenue records and the report
of the Commissioner, the said conclusion cannot be faulted with. The High Court, after
analyzing all the materials and finding that the petitioners before them who are appellants
before us are land grabbers and grabbed 12½ guntas of land, concurred with the decision
arrived at by the Special Court and dismissed their writ petitions.
8. In the light of the abundant acceptable materials in the form of oral and documentary
evidence coupled with the report of the Mandal Revenue Officer and of the
Commissioner, we agree with the conclusion arrived at by the Special Court and the High
Court and reject the claim of the appellants.
@page-SC1108
Consequently, both the appeals are liable to be dismissed, accordingly, we do so. No
costs.
Appeals dismissed.
AIR 2008 SUPREME COURT 1108 "Basayya I. Mathad v. Rudrayya S. Mathad"
(From : Karnataka)
Coram : 2 Dr. A. PASAYAT AND P. SATHASIVAM, JJ.
Civil Appeal No. 1349 of 2001, D/- 24 -1 -2008.
Basayya I. Mathad v. Rudrayya S. Mathad and Ors.
(A) Civil P.C. (5 of 1908), S.100 - APPEAL - HIGH COURT - Second appeal - Failure of
High Court to frame substantial question of law - Order of High Court liable to be set
aside.
RSA No. 131 of 1999, D/-16-03-1999 (Kant.), Reversed. (Para 8)
(B) Civil P.C. (5 of 1908), S.100 - APPEAL - HIGH COURT - Finding of fact -
Interference by High Court - Not permissible. (Para 8)
(C) Civil P.C. (5 of 1908), S.100 - APPEAL - HIGH COURT - Powers of High Court -
Impermissible for High Court to arrive at decision that suit property forms part of family
property partible among members of family without adverting to acceptable materials
placed before it in terms of procedure and in accordance with law.
R.S.A. No. 131 of 1999, D/-16-3-1999 (Kant.), Reversed. (Para 8)
(D) Civil P.C. (5 of 1908), O.41, R.27 - APPEAL - EVIDENCE - HIGH COURT -
Additional evidence - Finding of Court based on document produced at time of argument
dehors to R.27 - High Court neither followed conditions for production of additional
evidence nor recorded reason for basing reliance on same - Order of High Court liable to
be set aside.
R.S.A. No. 131 of 1999, D/-16-03-1999 (Kant.), Reversed. (Para 8)
Cases Referred : Chronological Paras
2007 AIR SCW 4002 : AIR 2007 SC 2306 (Foll. Pnt. B) 8
(2007) Civil Appeal No. 2836 of 2001, D/-8-1-2007 (SC (Foll. Pnt. A) 8
2005 AIR SCW 5321 : AIR 2005 SC 4395 (Foll. Pnt. A) 8
(2005)7 SCC 667 (Foll. Pnt. A) 8
(2006)6 SCC 271 (Foll. Pnt. A) 8
2004 AIR SCW 4451 (Foll. A) 8
2002 AIR SCW 5368 : AIR 2003 SC 689 (Foll. Pnt. A) 8
(1996) RFA No. 189 of 1996 D/- 9-8-1996 (Kant) 8
Shankar Divate, for Appellant; S.N. Bhat, N.P.S. Panwar, D.P. Chaturvedi, for
Respondents.
Judgement
1. P. SATHASIVAM, J. :-This appeal is directed against the judgment and order dated
16.03.1999 of the High Court of Karnataka at Bangalore in Regular Second Appeal No.
131 of 1999 in and by which the learned single Judge dismissed the second appeal at the
stage of admission.
2. BRIEF FACTS :
The appellant and Shri Shivayya (since deceased) and two others were brothers. Their
father owned many properties apart from being tenant of suit lands. Their father died in
the year 1952. According to the appellant, he alone was cultivating the suit lands as
tenant excluding all the brothers. The properties were divided among the brothers. The
suit property continued to be in the exclusive possession of the appellant as the same was
a tenanted land. Under Section 44 of the Karnataka Land Reforms Act, 1974 (hereinafter
referred to as the Act) all the lands held by or in possession of tenants stood transferred to
and vested in the Government. Under Section 45 of the Act, tenants were given an option
to be registered as occupants of the vested lands. It is the claim of the appellant that in
view of the provisions of the Act, the suit lands, which were in his possession as on 01-
03-1974, stood vested in the Government. He applied for registration of the occupancy
rights in respect of the suit lands. The respondents herein claiming to be the tenant for a
part of the land sought registration of occupancy rights. The Land Tribunal, after holding
enquiry as required under the provisions of the Act, allowed the application of the
appellant and rejected the application of the respondents. Aggrieved by the same, the
respondents filed an appeal before the Land Reforms Appellate Authority which was also
rejected. The Appellate Authority found that the appellant herein cultivated the suit lands
and other brothers have never cultivated the same. Thereafter, the plaintiff, Shivayya
(since deceased), filed a suit for partition and separate possession of his share from the
@page-SC1109
suit lands. On the basis of the evidence on record, the trial Court dismissed the suit filed
by the plaintiff. The plaintiff preferred an appeal before the appellate Court which also
confirmed the decree and dismissed the appeal on 29-11-1996. The plaintiff preferred a
second appeal before the High Court being R.S.A. No. 105 of 1997. The High Court, by
order dated 10-12-1997, allowed the second appeal and remanded the matter to the first
appellate Court to decide the same in the light of the finding and conclusion arrived at by
it. The first appellate Court, relying upon the opinion of the High Court, held that the suit
land was granted for the benefit of the entire family and the plaintiff is entitled to claim
his share and allowed the appeal on 14.12.1998. Questioning the judgment and decree of
the first appellate Court, the appellant preferred R.S.A. No. 131 of 1999 before the High
Court. The learned single Judge, basing reliance on his opinion in the earlier second
appeal i.e. R.S.A. No. 105 of 1997, which is binding and final, dismissed the second
appeal in limine. Aggrieved by the judgment and decree of the High Court, the appellant
filed the present appeal before this Court.
3. Heard Mr. Shankar Divate, learned counsel appearing for the appellant and Mr. S.N.
Bhat, learned counsel appearing for the respondents, perused the entire annexures and
other relevant materials filed before this Court.
4. Before considering the impugned judgment of the High Court, it is useful to refer the
notice issued by this Court on 15-12-1999 when S.L.P.(c) No. 13747/1999 came up for
hearing. The order passed, while issuing notice to the respondents, reads as under :
"Issue notice to show cause why the earlier judgment of the High Court dated 10-12-
1997, giving the finding that the property shall be treated as a family property, should not
be set aside on the ground that interference on question of fact was not permissible under
Section 100 CPC. Status quo as of possession on the spot shall be maintained."
From the above order, it is clear that the respondents were put on notice to the effect that
while hearing this special leave petition the correctness of the earlier finding in the
judgment of the High Court dated 10-12-1997 in R.S.A. No. 105 of 1997 would be gone
into by this Court. In view of the same, though the decision in R.S.A. No. 105 of 1997
has not been challenged in this Court, in view of the reasons which we refer hereunder
this Court is justified in considering the same.
5. As observed in the notice issued by this Court on 15-12-1999, let us first consider
whether the learned Judge of the High Court is justified in interfering with the factual
aspect and concurrent findings of both the Courts below and ultimate order of remand is
warranted. We carefully analysed the order of the High Court dated 10-12-1997 passed in
R.S.A. No. 105 of 1997. The said second appeal came to be filed by the plaintiff-
appellant against the dismissal of a suit and the learned Judge after referring the
unreported decision of the Division Bench of the same Court in R.F.A. No. 189 of 1996
dated 9-8-1996 directed the contesting respondent for production of an order of the grant
passed by the Authority and on going through the same arrived at a conclusion that the
findings of the Courts below were contrary to the grant and set aside the same. In
paragraph 3 of his order, the learned Judge concluded, the Court shall treat this property
also as a family property partible among the members of the family. By observing so,
remanded the matter to the first appellate Court with a direction to dispose of the same in
accordance with law in the light of the decision referred to in paragraph 2 as well as his
finding in paragraph 3.
6. Learned counsel appearing for the appellant vehemently contended that the order of the
learned Judge dated 10-12-1997 allowing the second appeal without framing the
substantial question of law in terms of Section 100, C.P.C. cannot be sustained. He also
submitted that forgetting that the learned Judge was hearing a second appeal filed against
the concurrent findings of both the Courts below, received a copy of the order of grant
passed by the authority, entertained the same and basing reliance on it, set aside the
judgment and decree of both the Courts below and remanded the matter to the first
appellate Court for fresh consideration. Learned counsel commented that the course
adopted by the learned Judge is unknown to law and is not justified in reversing the
concurrent finding of the Courts below by merely perusing a document which was
entertained without following the recourse provided under Order XLI, Rule 27, C.P.C.,
the High Court committed an error
@page-SC1110
in accepting the document and upsetting factual findings arrived at by the Courts below.
It is the argument of the learned counsel for the appellant that because of the erroneous
conclusion by the High Court, the lower appellate Court has no other option except to
follow the same and allowed Regular Appeal No. 9 of 1994 and decreed the suit of the
plaintiff in O.S. No. 517 of 1989.
7. Mr. S. N. Bhat, learned counsel appearing for the respondents, submitted that inasmuch
as no appeal had been filed against the decision of the High Court in R.S.A. No. 105 of
1997, the correctness or otherwise of the said order cannot be canvassed in this appeal. It
is true that against the order of remand in R.S.A. No. 105 of 1997, the appellant has not
filed appeal before this Court. However, after remand, the first appellate Court, based on
the direction of the High Court, allowed the appeal and decreed the suit which was
challenged by way of second appeal being R.S.A. No. 131 of 1999 before the High Court.
As stated earlier, the High Court, by judgment and order dated 16.3.1999, dismissed the
second appeal in limine which is the subject-matter of the present appeal.
8. On going through the entire materials, we are of the view that this Court is justified in
considering the earlier order of the High Court dated 10-12-1997 in R.S.A. No. 105 of
1997 for the following reasons :
i) It is not in dispute that the parties in the earlier proceeding, namely, R.S.A. No. 105 of
1997 and in the impugned proceeding - R.S.A. No. 131 of 1999 are one and the same.
Interestingly, the very same learned Judge had passed both the orders. The appellant had
placed judgment of the High Court rendered in R.S.A. No. 105 of 1997 as Annexure-P2
which is available on page 35 of the paper-book. A perusal of the same shows that after
reproducing unreported decision, namely, R.F.A. No. 189 of 1996 dated 09-08-1996
(Jarappa Poojari and others vs. Smt. Ramakku and others), the learned Judge called the
respondent for production of the order of grant passed by the authority and after perusing
the same arrived at a finding that the suit property also be treated as a family property and
partible among the members of the family. By arriving at such a conclusion, he set aside
the orders of both the Courts below. It is not in dispute that the learned Judge heard and
disposed of the second appeal filed under Section 100 CPC which reads as under :-
"100. Second appeal.- (1) Save as otherwise expressly provided in the body of this Code
or by any other law for the time being in force, an appeal shall lie to the High Court from
every decree passed in appeal by any Court subordinate to the High Court, if the High
Court is satisfied that the case involves a substantial question of law.
(2) An appeal may lie under this section from an appellate decree passed ex parte.
(3) In an appeal under this section, the memorandum of appeal shall precisely state the
substantial question of law involved in the appeal.
(4) Where the High Court is satisfied that a substantial question of law is involved in any
case, it shall formulate that question.
(5) The appeal shall be heard on the question so formulated and the respondent shall, at
the hearing of the appeal, be allowed to argue that the case does not involve such question
:
Provided that nothing in this sub-section shall be deemed to take away or abridge the
power of the Court to hear, for reasons to be recorded, the appeal on any other substantial
question of law, not formulated by it, if it is satisfied that the case involves such
question."
The above provision was amended and incorporated by amending Act 104 of 1976 which
came into effect from 1-2-1977. The learned Judge disposed of the second appeal being
R.S.A. No. 105 of 1997 on 10-12-1997 well after the amendment to Section 100. It is
clear from the above provision that only if the High Court is satisfied that the case
involves a substantial question of law, after formulating questions and hearing those
questions so formulated dispose of the same based on the materials placed before it. This
Court, in a series of decisions, has held that allowing a second appeal without framing
substantial question of law is clearly contrary to the mandate of Section 100, CPC vide :
(a) Gian Dass vs. Gram Panchayat, Village Sunner Kalan and others, (2006) 6 SCC 271 :
(b) Joseph Severance and Others vs. Benny Mathew and others, (2005) 7 SCC 667;

(c) Sasikumar and others vs. Kunnath Chellappan Nair and 2005 AIR SCW 5321

@page-SC1111
others, (2005) 12 SCC 588;

(d) Chadat Singh vs. Bahadur Ram and others, (2004) 6 SCC 359; 2004 AIR SCW 4451

(e) Kanhaiyalal and others vs. Anupkumar and others, (2003) 1 SCC 430; 2002 AIR
SCW 5368

(f) Civil Appeal No. 2836 of 2001 - Town Planning Municipal Council vs. Rajappa and
Anr. dated 08.01.2007 (Dr. Justice Arijit Pasayat and Justice P. Sathasivam)
In view of the settled legal position and of the fact that the High Court has not adhered to
the same, failed to formulate substantial question of law thereby committed an error in
allowing the second appeal. On this ground, the judgment and order of the learned Judge
in R.S.A. No. 105 of 1997 is liable to be set aside.
ii) Apart from the above infirmity, the High Court has committed an error in interfering
on a question of fact which was not permissible under Section 100, CPC vide P.
Chandrasekharan and others vs. S. Kanakarajan and others, (2007) 5 SCC 669. 2007
AIR SCW 4002

iii) It is relevant to point out that it is impermissible for a High Court to arrive at a
decision that the suit property forms part of family property partible among the members
of the family without adverting to acceptable materials placed before it in terms of the
procedure and in accordance with law. On this ground also, the decision of the High
Court is liable to be interfered with.
iv) The third infirmity is that though the parties to the proceeding can produce a
document as additional evidence even in Appellate Court, undoubtedly, they have to
adhere and satisfy the mandates provided under Order XLI Rule 27. For clarity, we
hereby reproduce the same.
"27. Production of additional evidence in Appellate Court.- (1) The parties to an appeal
shall not be entitled to produce additional evidence, whether oral or documentary, in the
Appellate Court. But if -
(a) the Court from whose decree the appeal is preferred has refused to admit evidence
which ought to have been admitted, or
(aa) the party seeking to produce additional evidence, establishes that notwithstanding the
exercise of due diligence, such evidence was not within his knowledge or could not, after
the exercise of due diligence, be produced by him at the time when the decree appealed
against was passed, or
(b) the Appellate Court requires any document to be produced or any witness to be
examined to enable it to pronounce judgment, or for any other substantial cause, the
Appellate Court may allow such evidence or document to be produced, or witness to be
examined.
(2) Wherever additional evidence is allowed to be produced by an Appellate Court, the
Court shall record the reason for its admission.
It is clear that parties to the lis are not entitled to produce additional evidence as of course
or routine but must satisfy the conditions stated in sub-clauses (a)and(aa). Admittedly,
such recourse has not been resorted to neither by the party concerned nor adhered those
principles by the High Court. Paragraph 3 of his order shows that the learned Judge
verified the document produced on his direction without complying the mandate as
provided under Rule 27 of Order XLI. Hence, we are of the view that the finding of the
learned Judge based on a document produced at the time of argument dehors to Rule 27
referred above cannot be sustained in the eye of law. In such circumstances, his ultimate
conclusion treating the suit property as a family property partible among the members of
the family is also liable to be set aside. In fact, sub-clause (2) of Rule 27 mandates that
wherever additional evidence is allowed to be produced by an Appellate Court, it shall
record the reason for its admission. It is needless to mention that the High Court neither
followed those conditions for production of additional evidence nor recorded the reason
for basing reliance on the same.
9. It is relevant to point out that in the ultimate paragraph (para 4), the learned Judge,
after remitting the matter to the first Appellate Court directed "to dispose of the matter in
accordance with law in the light of the decision mentioned (supra) and my finding
rendered above." Based on the said positive direction, the first Appellate Court has no
other option and, by judgment dated 14-12-1998, allowed Regular Appeal No. 9 of 1994
and granted preliminary decree for partition.
10. Though Mr. S.N. Bhat, learned counsel for the respondents reiterated his earlier
@page-SC1112
stand that the decision in R.S.A. 105 of 1997 cannot be gone into in the absence of appeal
against the same, in the light of our above-mentioned discussion, reasons thereon coupled
with the infirmities pointed above and the earlier decision is not in terms of Section 100
as well as Order XLI, Rule 27, CPC, we are unable to accept the said objection and pass
the following order :-
1. The finding of the High Court in RSA No. 105 of 1997 dated 10-12-1997 treating the
suit property also as family property partible among the members of the family is set
aside.
2. Since the lower Appellate Court i.e. Civil Judge, Senior Division, Gokak allowed the
Regular Appeal No. 9 of 1994 on 14-12-1998 based on the finding and positive direction
of the High Court dated 10.12.1997 in RSA No. 105 of 1997, his ultimate decision
allowing the appeal and granting preliminary decree is also set aside.
3. In view of our conclusion in sub-paras 1 and 2, the impugned order of the High Court
dated 16-03-1999 in R.S.A. No. 131 of 1999 is set aside.
4. The Civil Judge, Senior Division, Gokak is directed to restore Regular Appeal No. 9 of
1994 on his file and dispose of the same afresh uninfluenced by any of the observation
made by us.
5. Both parties are at liberty to file appropriate petition, if they so desire, for production
of any material as additional evidence subject to satisfying the conditions prescribed in
Rule 27 of Order XLI, CPC.
6. It is made clear that we have not expressed anything on the merits of the claim of
either parties. Our above conclusion mainly relates to the illegality or irregularity in the
order of the High Court in allowing the second appeal (RSA No. 105 of 1997)
7. Taking note of the fact that suit for partition was instituted even in the year 1989 and
yet to reach its finality, we request the Civil Judge. Gokak to dispose of the appeal, as
directed above and in accordance with law within a period of six months from the date of
receipt of copy of this judgment.
11. The civil appeal is allowed on the above terms. No costs.
Appeal allowed.
AIR 2008 SUPREME COURT 1112 "Om Prakash v. State of U. P."
(From : Allahabad)*
Coram : 2 Dr. A. PASAYAT AND P. SATHASIVAM, JJ.
Criminal Appeal No. 145 of 2008 (arising out of SLP (Cri.) No. 6188 of 2007), D/- 22 -1
-2008.
Om Prakash v. State of U.P.
Railway Property (Unlawful Possession) Act (29 of 1966), S.3 - RAILWAY - ILLEGAL
POSSESSION - THEFT - Theft of railway property - Unlawful possession - Truck found
to be loaded with cast iron Grade I - Accused described as contractor was present near
truck - On being asked to unload stolen iron he got it unloaded through his labourers -
Plea of accused that he was only a labourer - Not tenable - Accused can be said to be in
unlawful possession of cast iron Grade I - Conviction u/S.3, proper. (Paras 7, 8,
11)
Cases Referred : Chronological Paras
AIR 1979 SC 1825 : AIR 1979 SC 1193 (Ref) 10
B.S. Jain, Ajay Veer Singh, Manish Raghav, Anand Mishra and Dr. (Mrs.) Vipin Gupta,
for Appellant; Anil K. Jha, (N.P.) for Respondent.
* Cri. R. No. 156 of 2000, D/- 4-5-2007 (All.)
Judgement
1. Dr. ARIJIT PASATAT, J :- Leave granted.
2. Challenge in this appeal is to the order passed by a learned Single Judge of the
Allahabad High Court dismissing the revision petition filed by the appellant under
Section 397, read with Section 401 of the Code of Civil Procedure, 1973 (in short 'the
Code').
3. Challenge in the revision was to the order passed by a learned Sessions Judge in
Criminal Appeal No.2060 of 1990 by which the order of conviction and sentence, as
recorded by the learned Additional CJM, was confirmed.
4. Background facts in a nutshell are as follows :
On getting information on 22-1-1990 that one truck No.UP-93, 1665 of Minakshi Traders
was being loaded at scrap yard of Jhansi with Cast Iron Grade-I illegally with the Cast
Iron Grade-II, the inspector R.K. Rajput, along with Dy. Superintendent M.U. Farooki
went to the spot and found a truck No.UP-93-1665 near the Auction Hall which was
@page-SC1113
loaded with Cast Iron Grade II. Superintendent Incharge called the Head Constable 878
Taradat Sati and DSK Lala Ram. They were asked to climb up the truck and take a look,
and after examining, they reported that some Cast Iron Grade-I was loaded in the truck.
At that point of time, the contractor, the present appellant was also present near the truck
along with some persons. The contractor called his labourers and unloaded the Cast Iron
Grade-I from the truck. It was found that 22 carat Cast Iron Grade-I were without Tie
Bars. Inside the truck approximately, 7 Tons of Cast Iron Grade-II were loaded. It was
accepted that no Cast Iron Grade-I could have been loaded. Necessary examination was
done and it was found that the railway property, i.e. Cast Iron Grade-I was unauthorizedly
being transported. A complaint was lodged and after analysing the evidence on record,
learned Additional Chief Judicial Magistrate, Jhansi found the appellant guilty under
Section 3 of the Railway Property (Unlawful Possession) Act, 1966 (in short 'the Railway
Act') and sentenced him to undergo imprisonment for one year with fine of Rs.1,000/-
with default stipulations.
5. As noted above, an appeal before the learned Sessions Judge, Jhansi did not bring any
relief to the appellant and so also was the revision before the High Court.
6. Learned counsel for the appellant took the stand that the appellant was not the owner
of the articles and he was only a labourer. It was also submitted that the appellant has
already undergone sentence of more than eight months and since the sentence is only one
year, the same may be reduced to the period already undergone by the appellant.
7. From the evidence on record, it was found that the presence of Cast Iron Grade-I has
not been disputed. The stand presently taken is that somebody else was the auction
purchaser and the appellant had no role to play. But at all stages, it appears that the
appellant was present near the truck, he was described as the contractor and in his
presence the analysis was done, and from the material available on record, it is also clear
that he, as the contractor, was asked to unload the articles and he had called his labourers
to unload the articles.
8. Therefore, the finding that he was in unlawful possession of Cast Iron Grade-I is a
finding which does not warrant interference. Railway property, as defined in Section 2,
clause (d) of the Act reads as follows :
"Section 2(d) "railway property" includes any goods, money or valuable security or
animal, belonging to, or in the charge or possession of, a railway administration."
9. Section 3 deals with penalty for unlawful possession of railway property. The same
reads as follows :
"3. Whoever is found, or is proved to have been, in possession of any railway property
reasonably suspected of having been stolen or unlawfully obtained shall, unless he proves
that the railway property came into his possession lawfully, be punishable -
(a) for the first offence, with imprisonment for a term which may extend to five years, or
with fine, or with both and in the absence of special and adequate reasons to be
mentioned in the judgment of the court, such imprisonment shall not be less than one year
and such fine shall not be less than one thousand rupees;
(b) for the second or a subsequent offence, with imprisonment for a term which may
extend to five years and also with fine and in the absence of special and adequate reasons
to be mentioned in the judgment of the court, such imprisonment shall not be less than
two years and such fine shall not be less than two thousand rupees."
10

. In State of Maharashtra vs. Vishwanath Tukaram (1979 (4) SCC 23), it was observed
that the following ingredients are necessary to bring in application of Section 3 : AIR
1979 SC 1825

(i) The property in question should be railway property;


(ii) It should be reasonably suspected of having been stolen or unlawfully obtained; and
(iii) it should be found or proved that the accused was or had been in possession of that
property.
11. In the instant case, all the ingredients have been established. So far as the sentence is
concerned, for offence committed for the first time, a minimum punishment of one year
has been prescribed. That
@page-SC1114
being so, the courts below have rightly imposed sentence of one year.
12. Above being the position, there is no merit in this appeal which is, accordingly,
dismissed.
Appeal dismissed.
AIR 2008 SUPREME COURT 1114 "Uttar Haryana Bijli Vitran Nigam Ltd. v. Surji
Devi"
(From : Punjab and Haryana)
Coram : 2 S. B. SINHA AND V. S. SIRPURKAR, JJ.
Civil Appeal No. 576 of 2008 with C. A. Nos. 577 and 587-588 of 2008 (arising out of
SLP (C) Nos. 4392 and 9311 and 19530-19531 of 2004), D/- 22 -1 -2008.
Uttar Haryana Bijli Vitran Nigam Ltd. and Ors. v. Surji Devi.
Constitution of India, Art.166, Art.309 - Punjab Civil Services Rules (1959), Vol.2,
R.3.12, R.3.17 - GOVERNMENT BUSINESS - CIVIL SERVICE - FAMILY PENSION
SCHEME - PROVIDENT FUND - Family pension - Work charge employee - Not
entitled to family pension under service Rules - Deceased was member of contributory
Provident Fund - Has opted for non-pensionable scheme - Question of his being entitled
to pension or for that matter his family members becoming entitled to family pension
could not arise.
C. W. P. No. 1110 of 2003, D/-18-9-2003 (PandH). Reversed.
Employees' Provident Funds and Miscellaneous Provisions Act (19 of 1952), S.6A.
(Para 14)
Cases Referred : Chronological Paras
2007 AIR SCW 6967 14
2006 AIR SCW 5051 : 2007(1) AIR Jhar R 346 (Ref) 14
2006 AIR SCW 5957 : 2007 Lab IC 2270 : 2007 (1) ALJ 302 : 2007 (1) AIR Jhar R 836
(Ref) 14
2006 (13) Scale 319 14
2005 AIR SCW 654 : AIR 2005 SC 851 : 2005 Lab IC 864 (Ref) 14
(1999 Civ WP No. 7506 of 1998, D/- 16-12-1999 (reported in 2000 (2) SCT 32) (PandH)
5, 7, 12, 14, 15
Neeraj Kumar Jain, Bharat Singh, Sanjay Singh, Sandeep Chaturvedi, Umang Shankar,
Ugra Shankar Prasad, D.P. Singh, Sanjay Jain, Ms. Priyanka Singh, Rajat Vohra, Arvind
Nayyar, Ms. Kavita Wadia, for Appellants; Jasbir Singh Malik, S.K. Sabharwal,
Kamakshi S. Mehiwal, Vikash Chatrath, M.K. Verma, (for Anis Ahmed Khan) for
Respondent.
Judgement
S. B. SINHA, J. :-Leave granted.
2. The short question involved in these appeals, arising out of the judgments and orders
dated 18-9-2003 and 5-3-2004 passed by the High Court of Punjab and Haryana in
C.W.P. Nos. 631, 1110 of 2003 and Review Application No. 71 of 2004 respectively, is as
to whether family members of a deceased employee who was appointed on a work-
charged basis would be entitled to family pension?
3. For the purpose of disposal of these appeals, we would note the factual matrix only
from the Civil Appeal arising out of SLP (C) No. 4392 of 2004 titled Uttar Haryana Bijli
Vitran Nigam Ltd. and Ors. v. Surji Devi.
4. Appellant No. 1 was the successor of Haryana State Electricity Board which was
constituted under Section 5 and incorporated under Section 12 of the Electricity (Supply)
Act, 1948. Respondent (Surji Devi) is the widow of Late Shri Krishan. He was appointed
on a work-charge basis on or about 12-8-1974. Indisputably he continued to serve the
appellant No. 1 in the same capacity. While in service, he expired on 11-8-1985.
Respondent was appointed on compassionate ground in the services of the appellant
Corporation in terms of an ex gratia scheme.
Concededly, the deceased was a member of a Contributory Provident Fund constituted
under a Scheme.
Despite the same, the respondent filed an application for grant of family pension, which
pertains to altogether a different scheme.
5. Concededly, Late Shri Krishan's services were never regularized. The scheme for
regularization also came into force in 1986.
As the claim of the respondent No. 1 for grant of family pension was declined, she filed a
writ petition before the High Court of Punjab and Haryana. The High Court by reason of
the impugned judgment dated 18-9-2003, relying on or on the basis of its earlier decision
rendered in Civil Writ Petition No. 7506 of 1998 titled Kanta Devi v. State of Haryana
and others decided on 16-12-1999, allowed the same directing :
"...........It is the conceded position that the petitioner had received the benefit of pension
under the EPF scheme, but it is also
@page-SC1115
the admitted position that the amount which the petitioner would now receive on account
of family pension will be higher than the amount received by her under the EPF scheme.
Mr. Malik accordingly undertakes that the petitioner will refund/ adjust the amount,
which she had already received towards the amount, which she will now receive by way
of family pension."
6. Mr. Neeraj Kumar Jain, learned counsel appearing on behalf of the appellants, in
support of the appeal would submit :
(i) Having regard to the Punjab Civil Services Rules, Volume 2 as applicable to the State
of Haryana, the impugned judgment is wholly unsustainable.
(ii) Respondents husband having been a member of the Contributory Provident Fund, the
Family Pension Scheme was not applicable in her case.
7. Mr. Jasbir Singh Malik, learned counsel appearing on behalf of the respondent, on the
other hand, urged :
(i) Appellants having not questioned the correctness of Kanta Devi (supra), now cannot
turn round and contend that the Family Pension Scheme is not applicable.
(ii) The High Court in Kanta Devi (supra) having interpreted para 4 of the Family
Pension Scheme, the appellants are bound thereby.
8. The State of Punjab made the Punjab Civil Services Rules. The said Rules, subject to
modifications, became applicable to the State of Haryana. Volume 2 of the said Rules
inter alia provide for service qualifying for pension. Rule 3.12 thereof reads as under :
"3.12 The service of a Government employee does not qualify for pension unless it
conforms to the following three conditions
First - The service must be under Government.
Second - The employment must be substantive and permanent.
Third - The service must be paid by Government."
9. Rule 3.17 of the Rules provides that in the case of an officer retiring on or after 5th
January, 1961, if he was holding sub-stantively a permanent post on the date of his
retirement, his temporary or officiating service under the State Government, followed
without interruption by confirmation in the same or another post, shall count in full as
qualifying service except in respect of the pension period of temporary or officiating
service in non-pensionable establishment.
10. Rule 3.17-A(g) of the Rules inter alia provides that the entire service rendered by an
employee as work-charged shall be reckoned towards retirement benefits provided :
(i) such service is followed by regular employment :
(ii) there is no interruption in the two or more spells of service or the interruptions fall
within condonable limits; and
(iii) such service is a whole time employment and not part-time or portion of day.
[Emphasis supplied]
11. Indisputably, there exist two schemes; one in relation to Contributory Provident Fund
and another in relation to Pension. The Scheme of grant of Family Pension is contained in
Appendix 1 of the said Rules. Relevant portion of Para 4 of the said Scheme reads as
under :
"4. This scheme is administered as below :-
(i) The family pension is admissible in case of death while in service or after retirement
on or after the 1st July, 1964, if at the time of death, the retired officer was in receipt of a
compensation, invalid, retiring or superannuation pension. The family pension will not be
admissible in case of death after retirement if the retired employee at the time of death
was in receipt of gratuity only. In case of death while in service a Government employee
should have completed a minimum period of one year of continuous service without
break.
Note 1. - The term one year continuous service used in para- 4(i) above is inclusive of
permanent/temporary service in a pensionable establishment but does not include periods
of extraordinary leaves, boy service and suspension period unless that is regularized by
the competent authority or before completion of one year continuous service provided the
deceased Government employee concerned immediately prior to his recruitment to the
service or post was examined by the appropriate Medical Authority
@page-SC1116
and declared fit by that authority for Government service.
*** *** ***
(iii) The pension is admissible :-
(a) in the case of widow/widower upto the date of death or remarriage, whichever is
earlier; and
(b) in the case of son/unmarried daughter until he/she attains the age of 25 years."
12. Para 11 of the said Scheme excludes the applicability of the scheme inter alia in
relation to the work-charge staff. We may notice that in Kanta Devi (supra) the husband
of the appellant therein was in temporary service. Construing Para 4(i) as also Note 1
appended thereto, the High Court held that as the husband of Kanta Devi completed more
than one year in temporary service, she was entitled to family pension.
13. We have noticed hereinbefore that Shri Krishan was a member of the Contributory
Provident Fund. It has furthermore been noticed by us that even before the High Court
the said position stood conceded but she opted for the Pension Scheme only because
thereby she considered herself to be entitled to a higher amount.
14

. The scheme relating to grant of Family Pension was made under a statute. A person
would be entitled to the benefit thereof subject to the statutory interdicts. From a bare
perusal of the provisions contained in the Punjab Civil Services Rules, Volume 2 vis-a-vis
the Family Pension Scheme, it would be evident that the respondent was not entitled to
the grant of any family pension. Husband of the respondent was a work-charge employee.
His services had never been regularized. It may be unfortunate that he had worked for 11
years. He expired before he could get the benefit of the regularization scheme but
sentiments and sympathy alone cannot be a ground for taking a view different from what
is permissible in law. [See Maruti Udyog Ltd. v. Ram Lal and others, (2005) 2 SCC 638;
State of Bihar and Ors. v. Amrendra Kumar Mishra, 2006 (9) Scale 549; Regional
Manager, SBI v. Mahatma Mishra, 2006 (11) Scale 258; State of Karnataka v. Ameerbi
and Ors. 2006 (13) Scale 319 and State of M.P. and Ors. v. Sanjay Kumar Pathak and
Ors. [2007 (12) Scale 72] 2005 AIR SCW 654
2006 AIR SCW 5051
2006 AIR SCW 5957
2007 AIR SCW 6967

The statutory provisions, as noticed hereinbefore, debar grant of family pension in favour
of the family members as the deceased employee if was a work-charge employee and not
a permanent employee or temporary employee. The period during which an employee
worked as a work-charge employee could be taken into consideration only when his
services are regularized and he becomes permanent and not otherwise.
Furthermore, there exists a distinction between a pensionable and non-pensionable
establishment. Shri Krishan being a member of a non-pensionable establishment, Family
Pension was not admissible. It is not a case where an employee had been given an option
to opt for one or the other schemes. Once a person had opted for non-pensionable
scheme, the question of his being entitled to pension or for that matter his family
members becoming entitled to family pension did not and could not arise. The High
Court only followed Kanta Devi (supra) without noticing the distinctive features thereof.
As it is not necessary, we have not gone into the question as to whether Kanta Devi
(supra) was correctly decided. Apart from the fact that the fact therein was different,
evidently the questions which have been raised before us were not raised therein. The
High Court, therefore, committed a serious error in applying Kanta Devi (supra) to the
fact of the present case.
15. Mr. Malik contended that it has wrongly been stated in the list of dates that the appeal
against Kanta Devi (supra) has remained pending before this Court and, thus, it being a
mis-statement, the leave granted should be revoked. It may be so but in a case of this
nature this Court is required to lay down the law. We do not, thus, intend to revoke the
leave. However, we direct that any benefit paid to the respondent should not be
recovered.
16. For the reasons aforementioned, the appeal is allowed with the aforementioned
directions. However, in the facts and circumstances of this case, there shall be no order as
to costs.
Appeal allowed.
@page-SC1117
AIR 2008 SUPREME COURT 1117 "Neebha Kapoor v. Jayantilal Khandwala"
(From Bombay)*
Coram : 2 S. B. SINHA AND V. S. SIRPURKAR, JJ.
Civil Appeal No. 573 of 2008 (arising out of SLP (C) No. 5629 of 2007), D/- 22 -1 -2008.
Neebha Kapoor v. Jayantilal Khandwala and Ors.
Civil P.C. (5 of 1908), O.37, R.3 - SUMMARY SUIT - PROMISSORY NOTE -
DOCUMENTS - DECREE - Summary suit - Leave to defend - Suit filed to recover
money advanced on promissory note - Plaintiff unable to file originals of promissory note
and cheques dishonoured - Grant of unconditional leave to defend - Not improper - For
obtaining summary judgment original documents must be produced - Merely because
summary suit has been entertained, decree therein is not automatic - Court can always
refuse to exercise jurisdiction if original documents are not filed. (Paras 10, 12)
Cases Referred : Chronological Paras
AIR 2004 MP 158 (Rel. on) 10
AIR 1977 SC 577 (Ref) 11
AIR 1949 Cal 479 11
Shekhar Naphede, Sr. Advocate, Balraj Dewan, Satish K. Srivastava and Shekhar Gupta,
for Appellant; Jatin Zaveri, for Respondents.
* S. J. No. 9 of 2007 in S. S. No. 501 of 2006, D/- 13-2-2007 (Bom.)
Judgement
S. B. SINHA, J. :- Leave granted.
2. Appellant herein question a judgment and order dated 13-2-2007 passed by a learned
single Judge of the Bombay High Court granting unconditional leave to defend in a
summary suit wherein summons for judgment had been taken out.
3. Appellant filed the aforementioned suit for recovery of a sum of Rs. 25,00,000/- with
interest, which amount he is said to have advanced to the respondents by a cheque.
Respondents allegedly executed a promissory note for the said amount. An amount of Rs.
5,27,293/- was said to have been repaid by way of interest. A certificate of deducation of
tax at source under Section 203 of the Income Tax Act, 1961 for the amount of tax
deducted is said to have been issued to the appellant. A post dated cheque for Rs.
25,00,000/- was also given. Respondents also allegedly issued the following cheques
towards payment of interest accrued, the details whereof are as under :

Cheque No. Date Amount


948921 2-1-2003 67,903/-
948928 31-1-2003 12,500/-
948929 3-2-2003 12,500/-

4. All the four cheques having not been honoured, complaint petitions were filed.
Allegedly all original documents, viz., promissory note and four cheques, which were
filed in the criminal Court were misplaced.
The writ of summons in the summary suit was served upon the respondents. They
appeared on 14-8-2006. An application for a judgment in the said suit was applied for by
way of Summons for Judgment on 7-12-2006, which by reason of the impugned
judgment has been disposed of.
5. Mr. Shekhar Naphede, learned senior counsel appearing on behalf of the appellant,
would submit that a suit having been filed on the basis of bill of exchange within the
meaning of Order 37, Rule 1 of the Code of Civil Procedure (for short "the Code") read
with Section 6 of the Negotiable Instruments Act, 1881 (for short "the Act"), The High
Court committed a manifest error in passing the impugned order.
In any event, the learned counsel would contend that the Court ought to have, keeping in
view the facts and circumstances of the case, imposed conditions.
6. Mr. Jatin Zaveri, learned counsel appearing on behalf of the respondents, on the other
hand, would submit that the promissory note, having not been properly stamped, was not
admissible in evidence and as such even a summary suit is not maintainable.
7-8. A summary suit, as provided for in Order 37, Rule 1 of the Code is maintainable if it
is filed on bills of exchange, hundis and promissory notes. A cheque is a bill of exchange
within the meaning of Section 6 of the Act. Order 37, Rule 2 of the Code provides as to
what should a 'Plaint' contain. Rule 3 thereof provides for the procedure to be adopted in
such a suit. Sub-Rule (1) of rule 3 provides for entrance of appearance by the defendant
within ten days from the date of service of summons. Sub-rule (4) of Rule 3 provides for
service of a summons for judgment in Form No. 4A upon the defendant. Defendant
within ten days from the service of such summons by affidavit or otherwise may disclose
facts which would be deemed sufficient to entitle him to defend,
@page-SC1118
apply for leave to defend such suit. Leave to defend, however, may be granted
unconditionally upon such terms as may appear to the court to be just. Sub-rule (6) of
Rule 3 of Order 37 of the Code provides for hearing of such summons for judgment
stating :

"6. Recovery of cost of noting non-acceptance of dishonoured bill or note.- The holder of
every dishonoured bill of exchange or promissory note shall have the same remedies for
the recovery of the expenses incurred in noting the same for non acceptance or non-
payment, or otherwise, by reason of such dishonour, as he has under this Order for the
recovery of the amount of such bill or note."
9. A decree in a summary suit is to be granted provided it fulfills all the criteria laid down
therein. What is mandatory is the entering of appearance by the defendant in the suit.
Appellant took out summons for judgment under Order 37, Rule 3 on 5-1-2007. It was
served on the respondents on 8-1-2007. It was listed for hearing on 13-2-2007. Time was
sought for by the respondents to file their affidavit in reply. However, an unconditional
leave to defend was granted by the learned Judge having regard to the admitted position
that the appellant was not in a position to produce the original documents.
10. For the purpose of obtaining a summary judgment in terms of Order 37 of the Code,
ordinary the original documents must be produced. Original documents are not available.
Appellant, therefore, is obligated to prove the loss of documents. Only because a suit has
been entertained as a summary suit, the same by itself may not be a ground for passing of
a judgment on mere asking. We have noticed the fact situation obtaining herein. The High
Court was of the opinion that it is a case where unconditional leave should be granted.
The question as to whether the defence of the respondents is 'moonshine' or not a matter
which required consideration of the High Court at that stage. A decree could not have
been granted on the basis of even photostat copies of the documents. (See Food
Corporation of India v. Dena Bank, Indore and another AIR 2004 MP 158). Presumption
in regard to a negotiable or a bill of exchange in terms of Section 118 of the Act is also an
evidence. It is true that a presumption can be raised that a bill of exchange was correctly
stamped as provided for under Clause (f) of sub-section (2) of Section 128 of the Code
but a decree is to be passed by a Court of law upon application of mind.
Order 37 of the Code has been prescribed in terms of the provisions contained in Clause
(f) of sub-section (2) of Section 128 of the Code so as to expedite trial of suits specified
therein. We have no doubt in our mind that the underlying public policy behind Order 37
is expeditious disposal of suits of commercial nature. It provides for such disposal as
expeditiously as possible by prescribing time frame therefor. Where, however,
applicability of Order 37 of the Code itself is in question which appears to be the
principal reason behind the impugned judgment, in our opinion, grant of leave may be
permissible. The Court before passing a decree was entitled to take into consideration the
consequences therefor.
11

. Reliance has been placed by Mr. Naphede on a decision of this Court in M/s. Mechelec
Engineers and Manufacturers v. M/s. Basic Equipment Corporation ((1976)4 SCC 687)
wherein this Court quoted with approval a decision of the Calcutta High Court in Smt.
Kiranmoyee Dassi v. Dr. J. Chatterjee (49 CWN 246 : AIR 19949 Cal 479 in the
following terms : AIR 1997 SC 577

"(a) If the defendant satisfies the Court that he has a good defence to the claim on its
merits the plaintiff is not entitled to leave to sign judgment and the defendant is entitled
to unconditional leave to defend.
(b) If the defendant raises a triable issue indicating that he has a fair or bonafide or
reasonable defence although not a positively good defence the plaintiff is not entitled to
sign judgment and the defendant is entitled to unconditional leave to defend.
(c) If the defendant discloses such facts as may be deemed sufficient to entitle him to
defend, that is to say, although the affidavit does not positively and immediately make it
clear that he has a defence, yet, shows such a state of facts as leads to the inference that at
the trial of the action he may be able to establish a defence to the plaintiffs claim the
plaintiff is not entitled to judgment and the defendant is entitled to leave to defend but in
such a case the Court may in its discretion impose conditions as to the time or mode of
trial but not as to payment into Court or furnishing security.
@page-SC1119
(d) If the defendant has no defence or the defence set up is illusory or sham or practically
moonshine then ordinarily the plaintiff is entitled to leave to sign judgment and the
defendant is not entitled to leave to defend.
(e) If the defendant has no defence or the defence is illusory or sham or practically
moonshine then although ordinarily the plaintiff is entitled to leave to sign judgment, the
Court may protect the plaintiff by only allowing the defence to proceed if the amount
claimed is paid into Court or otherwise secured and give leave to the defendant on such
condition, and thereby show mercy to the defendant by enabling him to try to prove a
defence."
12. Admissibility of secondary evidence would be subject matter to evidence. Only if a
Court is to go into the evidence, presumptive evidence could also be taken into
consideration. Although the burden may be on the defendant, he may discharge the same
only when it is raised. The Code does not put any embargo on the Courts exercising a suo
motu power of granting leave in a case of this nature. If a Court does so even when an
application was not filed, keeping in view the admitted position of the case, we do not see
any illegality therein. As a decree in summary suit may not be automatic and the Court
can always refuse to exercise its discretion as the original documents were not produced
and, thus, the plaintiff is called upon to prove that the documents are lost in the criminal
proceedings.
13. In view of the fact that no application for leave was filed, it is not possible for us to
consider submission of Mr. Naphede in regard to the presumptions arising under Clause
(f) of sub-section (2) of Section 128 of the Code or purported acknowledgement
contained in the balance sheet of the respondents.
We, however, are of the opinion that the question as to whether the respondents should be
put to any terms or not should be determined afresh by the High Court as the High Court
did not address itself on the aforementioned questions. We, however, express no opinion
thereupon.
14. For the reasons aforementioned, we are of the opinion that the impugned judgment
warrants no interference at this stage.
The appeal is dismissed accordingly, subject, however, to the aforementioned
observations. No costs.
Appeal dismissed.
AIR 2008 SUPREME COURT 1119 "Regional Manager, Uttaranchal Rd. Tpt. Corpn. v.
Than Singh"
(From : Uttaranchal)*
Coram : 2 Dr. A. PASAYAT AND P. SATHASIVAM, JJ.
Civil Appeal No. 471 of 2008 (arising out of SLP (C) No. 3762 of 2006), D/- 17 -1 -2008.
Regional Manager, Uttaranchal Rd. Tpt. Corpn. v. Than Singh and Anr.
Road Transport Corporation Act (64 of 1950), S.45 - ROAD TRANSPORT
CORPORATION - TERMINATION OF SERVICE - SERVICE MATTERS - Termination
of service - Order passed against Conductor for not issuing tickets to 20 passengers
though fare was taken from them - No entry was made in waybill for 23 passengers out of
48 passengers - Issuance of tickets on basis of instructions of checking staff cannot
legalize illegality committed by him - Finding by High Court while setting aside order of
termination that when bus was checked, the tickets were issued to passengers but only
entries were not made in waybill - Contrary to materials on record - Matter remitted for
fresh consideration. (Para 7)

Ms. Sangeeta Kumar, for Appellant; Brij Bhusan, for Respondents.


* W. P. No. 690 (M/S) of 2005, D/- 5-10-2005 (Utr).
Judgement
1. ARIJIT PASAYAT, J. :- Leave granted.
2. Challenge in this appeal is to the judgment rendered by a learned Single Judge of the
Uttaranchal High Court allowing the writ petition filed by the respondent No. 1
(hereinafter referred to as the "employee"). Challenge before the High Court was to the
award dated 27-7-2004 passed by the Presiding Officer, Industrial Tribunal and Labour
Court, Haldwani (in short the "Tribunal") in Adjudication Case No. 21 of 1995.
3. Background facts in a nutshell are as follows :
Respondent No. 1 was appointed as a conductor on 21-11-1989 under the Appellant-
Corporation, hereinafter referred to as the "Corporation"). On 8/9-9-1990 the appellant
was the conductor in Bus No. UP 78-9254.
@page-SC1120
The Transport Inspector as a part of the checking operation stopped the bus. There were
48 passengers travelling in the bus and out of them 20 did not have any ticket and there
was no entry made in the Way Bill for 23 passengers. The employee made a statement
that he could not issue tickets though he had collected the fares from 20 persons. The
conductor was made to issue tickets to passengers to whom tickets had not been issued.
The inspector made an entry for closing of ticket issuance and he also directed the
employee to make an entry in respect of 23 passengers in the waybill. Proceedings were
initiated against the erring employee and his services were terminated on the basis of the
materials collected during departmental enquiry. A reference was made under the Uttar
Pradesh Industrial Disputes Act, 1947 (in short the "Act") at the prayer of the respondent.
The Tribunal came to hold that the order of termination was legal and justified and the
concerned workman was not entitled to any relief. It is to be noted that the enquiry officer
had in the enquiry report noted that the conductor had issued the tickets later though there
were no entries in the way bill.
4. The High Court in the writ petition filed by the respondent came to hold that the
concerned employee had taken fare from 20 passengers in presence of checking staff. It
was also noted that when the bus was checked, the tickets were issued to the passengers
but only entries were not made in the waybill. Accordingly the impugned award before it
was set aside by the High Court and the respondent No. 1-Employee was directed to be
reinstated in service with continuity of all retrial benefit but without back wages.
5. In support of the appeal, learned counsel for the appellant-Corporation submitted that
the findings recorded by the High Court are clearly contrary to record. It was not a fact
that the tickets had been issued as observed by the High Court. As a matter of fact, after
detection by the checking staff direction was given by the Traffic Inspector to issue
tickets and to make entries in the way bill for regularizing the travel of the passengers.
6. Learned counsel for the respondent with reference to the report of the enquiry officer
submitted that the High Court has correctly recorded the facts.
7. It appears from the statement of the respondent No. 1-employee that he himself
accepted that though he had collected the fare, he had not issued tickets to 20 passengers
and had only issued tickets to three passengers. The confusion appears to have arisen
because the High Court apparently proceeded on the basis that after the tickets were
issued only the entries in the waybill were to be recorded. This is really not so, because
the respondent No. 1 himself had accepted that tickets had not been issued to 20
passengers. The material on record also shows that the checking staff with a view to
regularize the entries and regularizing the travel of the passengers had directed issuance
of tickets to those 20 passengers to whom respondent No. 1 had not issued tickets. This is
evident from the fact that the Tribunal had categorically noted that 20 passengers were
issued tickets by the checking staff and the respondent No. 1 was directed to make entries
in the way bill. Issuance of tickets on the basis of the instructions of the checking staff
cannot legalize the illegality committed by the respondent No. 1 -employee. That being
so, the approach of the High Court was clearly wrong and the conclusions drawn are
contrary to the materials on record. Since the High Court has not considered the materials
in the proper perspective, the impugned order is set aside and the matter is remitted to the
High Court for fresh consideration in accordance with law.
8. The appeal is allowed to the aforesaid extent without any orders as to costs.
Order accordingly.
AIR 2008 SUPREME COURT 1120 "Sudesh Kumar v. State of Uttarakhand"
(From : Uttarakhand)*
Coram : 2 C. K. THAKKER AND PRAKASH PRABHAKAR NAOLEKAR, JJ.**
Criminal Appeal No. 204 of 2008 (arising out of SLP (Cri.) No. 5639 of 2007), D/- 29 -1
-2008
Sudesh Kumar v. State of Uttarakhand
(A) Probation of Offenders Act (20 of 1958), S.6 - Constitution of India, Art.136 -
PROBATION OF OFFENDERS - SPECIAL LEAVE PETITION - PLEA -
@page-SC1121
Benefit of probation - Plea raised for first time in S. L. P. - Not also supported by credible
and trustworthy evidence - Cannot be allowed (Per Court).
AIR 1977 SC 698, Relied on. (Paras 4, 24)
(B) Probation of Offenders Act (20 of 1958), S.6 - PROBATION OF OFFENDERS -
JUVENILE JUSTICE - Benefit of probation - Availability - Relevant date to determine
age of accused - Is date of imposition of punishment by Trial Court - Not date of offence.
AIR 1983 SC 654, Held per incuriam.
Juvenile Justice Act (53 of 1986), S.22.
Juvenile Justice (Care and Protection of Children) Act (56 of 2000), S.2(1).
Per P.P. Naolekar, J. :-Section 6 of the Act would apply to the accused who is under 21
years of age on the date of imposition of punishment by the trial Court and not on the
date of commission of the offence. If on the date of the order of conviction and sentence
by the trial Court the accused is below 21 years of age the provisions of Section 6 of the
Act applies in full force.
AIR 1983 SC 654, Held per incuriam. (Para 22)
The object and purpose of the Probation of Offenders Act, 1958 for applying the relevant
provisions to the accused are different and cannot be said in pari materia with the
Juvenile Justice Act, 1986 and the Juvenile Justice (Care and Protection of Children) Act,
2000. The Court would not construe a Section of a statute with reference to that of
another statute unless the latter is in pari materia with the former. Therefore, a decision
made on a provision of a different statute will be of no relevance unless underlying
objects of the two statutes are in pari materia. The decision interpreting various
provisions of one statute will not have the binding force while interpreting the provisions
of another statute. (Para 22)
(C) INTERPRETATION OF STATUTES - Interpretation of Statutes - External aids -
Provisions of other statute - Useful only if provisions of both statutes are pari materia.
(Para 22)
Cases Referred : Chronological Paras
2005 AIR SCW 3088 : AIR 2005 SC 2731 : 2005 Cri LJ 3091 : 2005 AIR Jhar HCR 1841
(Disting) 20, 22
AIR 1984 SC 237 : 1984 Cri LJ 168 (Ref.) 4
AIR 1984 SC 1232 : 1984 Cri LJ 990 (Ref.) 3
AIR 1983 SC 654 : 1983 Cri LJ 1043 (Held per Incuriam) 17, 18
AIR 1977 SC 698 : 1977 Cri LJ 340 (Rel. on (Pt. A)) 2, 24
AIR 1972 SC 1554 : 1972 Cri LJ 1042 (Ref.) 14
AIR 1972 SC 2434 : 1972 Cri LJ 1517 (Ref.) 13, 14
AIR 1963 SC 1088 : 1963 (2) Cri LJ 173 (Rel. on) (Pt. B) 18, 20, 22
KTS Tulsi, Sr. Advocate, Abhishek Garg, Ms. Evneet Uppan, Ravinder Singh, Kuba
Bodah, Dr. Bheem Pratap Singh, Pawan Kumar and Dinesh Kumar Garg, for Appellant;
Jatinder Kumar Bhatia, B. N. Jha and Sahdev Singh, for Respondent.
* Cri. Appeal No. 311 of 2001, D/- 9-7-2007 (UTR).
** (The judgments are printed in the order in which they are given in the certified Copy -
Ed.)
Judgement
C. K. THAKKER, J. :- I have had the benefit of going through the judgment prepared by
my learned Brother. I am in agreement with him that the appeal deserves to be dismissed.
I, however, decide the appeal on the second ground that on the facts and in the
circumstances of the case, the appellant has failed to make out a ground that he was less
than 21 years of age at the time of commission of offence.
2

. As observed by my learned Brother, the accused had not claimed benefit of Section 6 of
the Probation of Offenders Act, 1958 either before the trial Court or before the High
Court. My learned Brother has also referred to Yaduraj Singh and Ors. v. State of U.P.,
(1976) 4 SCC 310 wherein this Court did not allow AIR 1977 SC 698

. In Sushil Kumar Mehrotra v. State of Uttar Pradesh, (1984) 3 SCC 123, a similar plea
was raised for the first time by the appellant-accused in this Court against his conviction
for an offence punishable under Section 302 read with Section 34 and Section 394 of the
Indian Penal Code (IPC). It was held that the contention of the accused that he was 15½
years of age at the time of occurrence was 'a complete afterthought' and refused to grant
the benefit on that basis. AIR 1984 SC 1232

. It is, no doubt, true that the provision is beneficial and benevolent in nature and no
'technical' objection should be raised that such plea was not taken before the Courts
below. AIR 1984 SC 237

@page-SC1122
[Gopinath Ghosh v. State of West Bengal, (1984) Supp. SCC 228]. But in my opinion,
there must be credible and trustworthy evidence in support of such plea. In the present
case, a certificate in the form of 'Scholar Record and Transfer Certificate' is annexed
wherein the date of birth of the appellant was shown as June 28, 1962. The certificate was
not on record either before the trial Court or before the High Court. From the True Copy',
it is clear that it is purported to have been issued by the Principal only on February 10,
2007. Thus, it cannot be said that there is 'credible evidence' or 'trustworthy material' that
the appellant was less than 21 years of age at the time of commission of offence. In my
considered opinion, such question cannot be permitted to be raised for the first time in
this Court and I am in agreement with my learned Brother on that point.
5. Since the appeal can be decided on this ground, I refrain from expressing any opinion
on the question dealt with and decided by my learned Brother on interpretation of Section
6 of the Act.
6. The appeal is accordingly dismissed.
7. P. P. NAOLEKAR, J.:-Leave granted.
8. The appellant was convicted by the judgment and order dated 26-7-1985 passed by the
Additional District and Sessions Judge, Dehradun, along with another accused person,
under Section 392 read with Section 34 of the Indian Penal Code (IPC) and sentenced to
undergo five years' rigorous imprisonment and further to pay a fine of Rs.5,000/- and in
default of payment of fine to undergo further rigorous imprisonment for six months. The
appellant was further convicted under Section 25 of the Arms Act and sentenced to
undergo rigorous imprisonment for one year. In appeal preferred by the appellant, the
High Court has confirmed the order of conviction and sentence by its order dated 9-7-
2007.
9. The case of the prosecution in brief is that one Jagdish Prasad was wholesale beedi
merchant and carried on his business in the name and style of M/s. Madrasee Basant
Beedi in Vikasnagar, District Dehradun. Jagdish Prasad used to go to collect his dues
from the retailers on every 15th day. On 7-3-1981, he went to Purola, Badkot for
realization of his dues. Along with other persons, he was travelling in the car which was
being driven by the driver Gyanendra Singh. While returning to Vikasnagar from Purola,
they had stopped at the curve of Katta Pather and alighted from the car. Four miscreants
came on scooter and parked the said scooter in front of the motor car. Two miscreants
were armed with revolvers and the remaining two had khukhries with them. All of them
surrounded Jagdish Prasad and ordered him to hand over money bag to them. They also
threatened him to shoot and kill him if he made any protest. Jagdish Prasad quietly
handed over the money bag containing about Rs. 25,000/-. He also gave his wrist watch
and a golden ring. Another occupant of the car was compelled to give cash of Rs.230/-
and the driver gave cash of Rs.600/- to them. One person sitting in the car was forced to
give his three wrist watches. The miscreants snatched away the keys of the car from its
driver. One of the miscreants ran away on the scooter along with the money bag, while
the remaining three boarded the car and fled away. On appreciation of the evidence
brought on record, the Additional District and Sessions Judge found the accused persons
guilty and imposed the punishment which was confirmed by the High Court as mentioned
hereinabove. The accused appellant-Sudesh Kumar has preferred this appeal against the
order of conviction and sentence.
10. Shri K.T.S. Tulsi, learned senior counsel appearing for the appellant has submitted
only one point that the accused at the time of the commission of the crime was below 21
years of age which is apparent from the statement recorded under Section 313 Cr.P.C. of
the accused wherein age of the accused was given by the accused as 20 years and from
the transfer certificate, filed along with special leave petition, issued by the Principal,
Sanatan Dharma Junior High School, Dehradun, which shows that the appellant was born
on 28-6-1962. It is, therefore, submitted that it is clearly established that the accused
appellant on the date of the offence, i.e. 7-3-1981, was below 21 years of age and as such
was entitled to consideration and benefit under Section 6 of the Probation of Offenders
Act, 1958 (hereinafter referred to as the Act for convenience).
11. On the other hand, it is urged by Shri Jatinder Kumar Bhatia, learned counsel for the
State that the accused having not raised the question of his age either before the trial court
or before the High Court, and in the absence of any reliable material, could not
@page-SC1123
ask for consideration of his case and benefit under Section 6 of the Act. It is further
submitted that it is the date on which the sentence is passed which shall be the relevant
date for applicability of Section 6 of the Act.
12. The question involved in this case is of interpretation of Section 6 of the Act. It
would, therefore, be appropriate to reproduce Section 6 which reads as under :
"6. Restriction on imprisonment of offenders under twenty-one years of age. - (1) When
any person under twenty-one years of age is found guilty of having committed an offence
punishable with imprisonment (but not with imprisonment for life), the court by which
the person is found guilty shall not sentence him to imprisonment unless it is satisfied,
that having regard to the circumstances of the case including the nature of the offence and
the character of the offender, it would not be desirable to deal with him under section 3 or
section 4, and if the court passes any sentence of imprisonment on the offender, it shall
record its reasons for doing so.
(2) For the purpose of satisfying itself whether it would not be desirable to deal under
section 3 or section 4 with an offender referred to in sub-section (1), the court shall call
for a report from the Probation Officer and consider the report, if any; and any other
information available to it relating to the character and physical and mental condition of
the offender."
13

. While interpreting Section 6 of the Act, a 3-Judge Bench of this Court in the case of
Daulat Ram v. The State of Haryana, (1972) 2 SCC 626, has said that the object of
Section 6 of the Act, broadly speaking, is to see that young offenders are not sent to jail
for the commission of less serious offences mentioned therein because of grave risk to
their attitude to life to which they are likely to be exposed as a result of their close
association with the hardened and habitual criminals who may happen to be the inmates
of the jail. The Court laid down that Section 6 places restrictions on the courts power to
sentence a person under 21 years of age for the commission of crimes mentioned therein
unless the court is satisfied that it is not desirable to deal with the offender under Sections
3 and 4 of the Act. The court is also required to record reasons for passing sentence of
imprisonment on such offender. AIR 1972 SC 2434

14

. In another case in the matter of Satyabhan Kishore and Another v. The State of Bihar,
(1972) 3 SCC 350, this Court (a 3-Judge Bench) reiterated the principle laid down by the
Court in Daulat Ram case (supra) and Shelat, J. speaking for the Court held that Section 6
lays down an injunction as distinguished from discretion under Sections 3 and 4 not to
impose a sentence of imprisonment on an offender, unless reasons are recorded. AIR
1972 SC 1554
AIR 1972 SC 2434

15. From the aforesaid judgments, it is apparent that while imposing a sentence on an
accused who is below 21 years of age and who is found guilty of having committed an
offence punishable with imprisonment which is not the imprisonment for life, the court
shall not sentence him to imprisonment unless it is satisfied that having regard to the
circumstances of the case including the nature of the offence and the character of the
offender it is not desirable to deal with him under Section 3 or Section 4 of the Act. It
further mandates that if the court wants to impose a sentence of imprisonment on the
offender who is below 21 years of age it shall record its reasons for doing so. Thus, the
court imposing a sentence of imprisonment on an accused who is below 21 years of age
would record reasons as to why it does not find it desirable to deal with him under
Section 3 or Section 4 of the Act.
16. It can be noticed that the question of the offender being of 21 years or below on the
date of the commission of offence or on the date of imposition o. sentence of
imprisonment was not dealt with in the above-mentioned cases.
17

. The learned counsel for the appellant has relied upon a 2-Judge Bench judgment of this
Court in the case of Masarullah v. State of Tamil Nadu, (1982) 3 SCC 458, wherein this
Court held as under : AIR 1983 SC 653

"6. In case of an offender under the age of 21 years on the date of commission of the
offence, the court is expected ordinarily to give benefit of the provisions of the Act and
there is an embargo on the power of the court to award sentence unless the court
considers otherwise, 'having regard to the circumstances of the case including nature of
the offence and the character of the offender', and reasons for awarding sentence have to
be recorded. Considerations relevant to the
@page-SC1124
adjudication of this aspect are, circumstances of the case, nature of the offence and
character of the offender. It is, therefore, necessary to keep in view the aforementioned
three aspects while deciding whether the appellant should be granted the benefit of the
provisions of the Act."
18. It appears that in Masarullah case (supra), the Court did not notice a 4-Judge Bench
judgment delivered by Ayyangar, J. in Ramji Missar and Another v. State of Bihar, AIR
1963 SC 1088 : ((1963) Supp. 2 SCR 745), wherein this Court has noticed argument
before the High Court that the Sessions Judge erred in not applying the provisions of
Section 6 of the Act to the accused. The High Court repelled the contention holding that
although the accused might have been under 21 years of age on the date of the offence, he
was not a person under 21 years of age on the date when the Sessions Judge found him
guilty and sentenced him to a term of imprisonment, and held that the crucial date on
which the age had to be determined being not the date of offence but the date on which as
a result of a finding of guilty sentence had to be passed against the accused. In the factual
matrix of that case, this Court held as under :
"6. Taking first the case of Ramji, the elder brother, we entirely agree with the High Court
in their construction of S.6. The question of the age of the person is relevant not for the
purpose of determining his guilt but only for the purpose of the punishment which he
should suffer for the offence of which he has been found, on the evidence, guilty. The
object of the Act is to prevent the turning of youthful offenders into criminals by their
association with hardened criminals of mature age within the walls of a prison. The
method adopted is to attempt their possible reformation instead of inflicting on them the
normal punishment for their crime. If this were borne in mind it would be clear that the
age referred to by the opening words of S.6(l) should be that when the court is dealing
with the offender, that being the point of time when the court has to choose between the
two alternatives which the Act in supersession of the normal penal law vests in it, viz.,
sentence the offender to imprisonment or to apply to him the provisions of S.6(l) of the
Act....."
19. The Court further said :
" 19. We shall now proceed to consider one question which was mooted before us in
regard to the crucial date for reckoning the age where an appellate court modifies the
judgment of the trial Judge, when S.6 becomes applicable to a person only on the
decision of an appellate or a revisional court. Is the age of the offender to be reckoned as
at the date of the judgment of the trial Judge or is it the date when the accused is, for the
first time, in a position to claim the benefit of S.6. We consider that on the terms of the
section, on grounds of logic as well as on the theory that the order passed by an appellate
court is the correct order which the trial court should have passed, the crucial date must
be that upon which the trial court had to deal with the offender........."
From the judgment of the Court, it is apparent that the date of the judgment of the trial
court would be the crucial date for consideration of the age of the accused while applying
Section 6 of the Act.
20

. Faced with the 4-Judge judgment of this Court in Ramji Missar (supra), the learned
senior counsel for the appellant contended that while considering the pan materia
provisions under the Juvenile Justice Act, 1986, a Constitution Bench of this Court in
Pratap Singh v. State of Jharkhand and Another, (2005) 3 SCC 551, has held that
reckoning date for determining the age of a juvenile is the date of the commission of the
offence and not the date when he is produced before the competent authority or in the
court and, therefore, the provisions of Section 6 of the Act should be construed in the
same light, and the age of the accused for applying Section 6 of the Act has to be the date
on which the offence was committed. While interpreting the provisions of the Juvenile
Justice Act, 1986 (for short "the 1986 Act") and the Juvenile Justice (Care and Protection
of Children) Act, 2000 (for short "the 2000 Act"), this Court has observed that these Acts
provide for the care, protection, treatment, development and rehabilitation of juveniles.
The Acts being benevolent legislations, such interpretation must be given which would
advance the cause of the legislations, i.e. to give benefit to juveniles. Section 2(1) of the
2000 Act defines 'juvenile in conflict with law' as meaning a juvenile who is alleged to
have committed an offence. The definition of 'delinquent juvenile' in the 1986 Act is
referable to an offence said to have been committed by him. It is the date of offence
AIR 1963 SC 1088
2005 AIR SCW 3088

@page-SC1125
that he was conflict with law. When a juvenile is produced before the competent authority
and/or court, he has not committed an offence on that date, but he was brought before the
authority for the alleged offence which he has been found guilty to have committed.
Therefore, what was implicit in the 1986 Act has been made explicit in the 2000 Act.
Sinha, J. in his concurring judgment said that having regard to the constitutional and
statutory scheme it was not necessary for Parliament to specifically state that the age of
juvenile must be determined as on the date of commission of the offence and the same is
inbuilt in the statutory scheme.
21. From the aforesaid, it is apparent that while determining the age of a juvenile the
Court has interpreted the provision for giving benefit to a juvenile who has committed an
offence and was in conflict with law. The offence having been committed, he came in
conflict with law on the date of commission of the offence which is relevant for
determining the age for giving protection under the 1986 Act and the 2000 Act.
22

. It can be noticed from Ramji Missar case (supra) and Pratap Singh case (supra) that the
object and purpose of the Probation of Offenders Act, 1958 for applying the relevant
provisions to the accused are different and cannot be said in pari materia with the
Juvenile Justice Act, 1986 and the Juvenile Justice (Care and Protection of Children) Act,
2000. The Court would not construe a Section of a statute with reference to that of
another statute unless the latter is in pari materia with the former. Therefore, a decision
made on a provision of a different statute will be of no relevance unless underlying
objects of the two statutes are in pari materia. The decision interpreting various
provisions of one statute will not have the binding force while interpreting the provisions
of another statute. Section 6 of the Act has been construed by a 4-Judge Bench of this
Court in Ramji Missar case (supra) and that will have the binding force while interpreting
the same Section in same statute and the decision of the Constitution Bench interpreting
provisions of the 1986 Act and the 2000 Act would not be held to be a decision on
interpretation of Section 6 of the Act. Section 6 of the Act would apply to the accused
who is under 21 years of age on the date of imposition of punishment by the trial court
and not on the date of commission of the offence. If on the date of the order of conviction
and sentence by the trial court the accused is below 21 years of age the provisions of
Section 6 of the Act applies in full force. AIR 1963 SC 1088
2005 AIR SCW 3088

23. That being the case, even if the date of birth of the accused is held to be 28-6-1962 as
alleged by him in the petition, on the date of delivery of judgment of conviction and
sentence on 26-7-1985 by the Additional District and Sessions Judge he was more than
21 years of age and thus was not entitled to the benefit under Section 6 of the Act.
24

. That apart, the question of applicability of the Act has been raised for the first time
while filing the special leave petition. The accused has not claimed benefit under Section
6 of the Act during the trial before the Additional District and Sessions Judge or before
the High Court. Only material which was placed before the Sessions Judge or the High
Court is the statement recorded of the accused appellant under Section 313 Cr.P.C.
wherein the age of the accused was given as 20 years. In the similar circumstances, in
Yuduraj Singh and Others v. State of U.P., (1976) 4 SCC 310, this Court held as under :
AIR 1977 SC 698
"2. The learned counsel appearing for the appellants argues that on August 30, 1969 when
the incident took place, appellants 3 and 4 were less than 21 years of age and, therefore,
they ought to have been given the benefit of the Probation of Offenders Act. This
contention was neither taken in the sessions court nor in the High Court. True, that this
Court has taken the view that in appropriate cases such a contention may be entertained
by this Court for the first time. But the difficulty in accepting the submission of the
learned counsel is that there is no credible evidence on the record showing that appellants
3 and 4 were less than 21 years of age when the offence was committed. Counsel says
that those two accused had given their ages in their statements under Section 342, Code
of Criminal Procedure, and if the trial Judge doubted the correctness thereof, he could
have had the two accused medically examined in order to ascertain their age. This seems
to us a difficult burden for any trial Judge to undertake. The age given by the two accused
in their statements had no special significant in the absence of a proper plea under the
@page-SC1126
Probation of Offenders Act.........."
25. For the aforesaid reasons, the appeal being devoid of any merit, is dismissed.
Appeal dismissed.
AIR 2008 SUPREME COURT 1126 "Som Mittal v. Govt. of Karnataka"
(From : Karnataka)
Coram : 2 H. K. SEMA AND MARKANDEY KATJU, JJ.*
Criminal Appeal No. 206 of 2008 (arising out of SLP (Cri.) No. 1719 of 2006), D/- 29 -1
-2008.
Som Mittal v. Govt. of Karnataka.
(A) Criminal P.C. (2 of 1974), S.482 - INHERENT POWERS - Inherent power - Is to be
exercised sparingly with circumspection and in rarest of rare cases.
H. K. Sema, J. :- Inherent power of the High Court should not be exercised according to
whims and caprice and it has to be exercised sparingly, with circumspection and in the
rarest of rare cases. It is seen often that High Court exercising the inherent power under
S. 482 Cr. P. C. in a routine manner at its whims and caprice setting at naught the
cognizance taken and the FIR lodged at the threshold committing grave miscarriage of
justice. While it is true that so long as the inherent power of S. 482 is in the Statute Book,
exercise of such power is not impermissible but it must be noted that such power has to
be exercised sparingly with circumspection and in the rarest of rare cases, the sole aim of
which is to secure the ends of justice. The power under S. 482 is not intended to scuttle
justice at the threshold. (Paras 10, 19)
Markandey Katju, J. (Contra) :- The power under S. 482 Cr. P. C. is to be exercised
sparingly. But it cannot be said that it should be exercised in the 'rarest of the rare cases'.
(Paras 38, 39)
(B) Karnataka Shops and Commercial Establishments Act (8 of 1962), S.25, S.30(1) -
SHOPS AND ESTABLISHMENT - COGNIZANCE OF OFFENCE - SPECIAL LEAVE
APPEAL - MAGISTRATE - Accused charged for offence under - Order taking
cognizance of offence by Magistrate - No infirmity found by High Court - Order not
liable to be interfered with.
Constitution of India, Art.136. (Paras 21, 25)
(C) Karnataka Shops and Commercial Establishments Act (8 of 1962), S.3(1)(h), S.30 -
SHOPS AND ESTABLISHMENT - APPLICABILITY OF AN ACT - AMENDMENT -
Person in management - Exclusion from applicability of Act - Apparent conflict between
S.30 and S.3(1) (h) - S.3(1)(h) require amendment - Court cannot fill up a casus omissus -
Matter left open. Per Markandey Katju, J. (Paras 35, 36, 37)
(D) Criminal P.C. (2 of 1974), S.438 - ANTICIPATORY BAIL - INHERENT POWERS -
Anticipatory bail - Deletion of provision u/S.438 Criminal P.C. by S.9 of U. P. Act 16 of
1976 - Entailed filing of thousands of writ petitions and S.482 Criminal P. C. applications
in Allahabad High Court for stay of petitioner's arrest and/or quashing FIR - Deletion
causing hardship to public, and overcrowding in jails - Problem will be obviated by
restoring provision for anticipatory bail in State of U. P. - Court made strong
recommendation to U. P. Government to immediately issue Ordinance to restore
provision for anticipatory bail - Further, decision in 2005 Cri LJ 755 (All) (FB) directed
to be implemented in letter and spirit. (Obiter) (Per Markandey Katjn, J.) (Paras 47, 51,
52, 59, 61)
(E) Constitution of India, Art.136, Art.245 - SPECIAL LEAVE APPEAL - STATE
LEGISLATURE - Powers of Court - Recommendation of Court for restoration of deleted
statutory provision - Is not binding on State Govt./ State Legislature - But still it should
be seriously considered, and not simply ignored - Court usually makes recommendation
when it feels that public is facing some hardship - Such recommendation should,
therefore, be given respect and serious consideration. (Obiter) (Per Markandey Katjn, J.)
(Para 58)
Cases Referred : Chronological Paras
2005 AIR SCW 4740 : AIR 2005 SC 3212 : 2005 Cri LJ 4140 : 2005 CLC 1382
(Disting.) 20
2005 Cri LJ 755 (All) (FB) (Ref.) 60, 61
2002 Cri LJ 4561 : 2002 All LJ 2233 (Ref.) 58
1994 AIR SCW 1886 : AIR 1994 SC 1349 : 1994 Cri LJ 1981 (Ref.) 42, 45, 54, 56,
62
1992 AIR SCW 2237 : AIR 1992 SC 604 : 1992 Cri LJ 527 (Ref.) 18
@page-SC1127

AIR 19989 SC 1 : 1989 Cri LJ 1055 (Ref.) 17


AIR 1980 SC 326 : 1980 Cri LJ 98 (Ref.) 13
AIR 1980 SC 898 : 1980 Cri LJ 636 (Ref.) 39
AIR 1980 SC 1632 : 1980 Cri LJ 1125 (Ref.) 55
AIR 1978 SC 597 (Ref.) 49
AIR 1977 SC 366 : 1977 Cri LJ 225 (Ref.) 53
AIR 1977 SC 2229 : 1977 Cri LJ 1900 (Ref.) 16
AIR 1974 SC 1146 : 1974 Cri LJ 802 (Ref.) 15
AIR 1972 SC 484 : 1972 Cri LJ 298 (Ref.) 14
(1970) 1 QB 693 : (1969) 3 All ER 1700 49
AIR 1969 SC 110 (Disting.) 20
AIR 1965 SC 111 (Disting.) 20
AIR 1962 SC 1893 (Disting.) 20
K.K. Venugopal and K.G. Raghavan, Sr. Advocates, Amit Dhingra and Aman Leekha (for
M/s. Dua Associates), with them for Appellant; Ms. Anitha Shenoy and Sanjay R. Hegde,
for Respondent.
* [The Judgements are printed in the order in which they are given in the certified copy -
Ed.]
Judgement
1. H. K. SEMA, J. :-Leave granted.
2. Heard learned counsel for the parties.
3. This appeal is directed against the judgment and order dated 28th March, 2006 passed
by the High Court of Karnataka at Bangalore in Criminal Petition No. 1535 of 2006 filed
under Section 482 of the Code of Criminal Procedure with a prayer to quash cognizance
of offence under Sections 25 and 30(3) of the Karnataka Shops and Commercial
Establishments Act, 1961 (in short the Act) by Metropolitan Magistrate Traffic Court III.
4. In view of the order that we propose to pass, it may not be necessary to recite the entire
facts leading to the filing of the present appeal. Suffice it to say that an unfortunate
incident had occurred on 13th December, 2005 in which late Smt. Pratibha Srikant
Murthy was stated to have been murdered on her way to work from her residence.
Pursuant to the aforesaid incident a complaint was filed on 27th December, 2005 against
the appellant alleging violation of Sections 25 and 30(3) of the Act before the
Metropolitan Magistrate. On 30th December, 2005, the Metropolitan Magistrate took
cognizance of the offences under aforesaid sections of the Act. On 23rd March, 2006, a
petition under Section 482 of the Code of Criminal Procedure for quashing of the
complaint and cognizance was filed before the High Court. The High Court, by its
impugned order dated 28th March, 2006, dismissed the petition. Hence, the present
appeal by special leave.
5. The High Court, by its impugned order, has altered the cognizance taken by the
Magistrate under Section 25 read with Section 30(3) to that one under Section 25 read
with Section 30(1) of the Act. The High Court was of the view that taking cognizance
against the appellant cannot be found fault with and dismissed the petition.
6. It is noticed, therefore, that petition under Section 482 was filed at the threshold for
quashing of the cognizance taken by the Magistrate.
7. Mr. K. K. Venugopal, learned Senior counsel for the appellant has addressed us on
merits of the case. He would contend that the appellant is a Managing Director and
occupying the position of management and, therefore, he would be entitled for exemption
under Section 3(h) of the Act. He would further contend that the appellant, being
Managing Director of the company, would not be liable for prosecution under Section 25
read with Section 30(1) of the Act.
8. Per contra, Ms. Anitha Shenoy, learned counsel appearing on behalf of the respondent,
contended that Chapter VIII of the Act deals with a penal provision. She would contend
that the language, Whoever contravenes employed in Section 30 of the Act would include
the Managing Director.
9. At this stage we are not prepared to enter into the merits of the case on the basis of
contentions urged by the respective counsel. Here are our reasons :
10. In a catena of decisions this Court has deprecated the interference by the High Court
in exercise of its inherent powers under Section 482 of the Code in a routine manner. It
has been consistently held that the power under Section 482 must be exercised sparingly,
with circumspection and in rarest of rare cases. Exercise of inherent power under Section
482 of the Code of Criminal Procedure is not the rule but it is an exception. The
exception is applied only when it is brought to the notice of the Court that grave
miscarriage of justice would be committed if the trial is allowed to proceed where the
accused would be harassed
@page-SC1128
unnecessarily if the trial is allowed to linger when prima facie it appears to Court that the
trial would likely to be ended in acquittal. In other words, the inherent power of the Court
under Section 482 of the Code of Criminal Procedure can be invoked by the High Court
either to prevent abuse of process of any Court or otherwise to secure the ends of justice.
11. This Court, in a catena of decisions, consistently gave a note of caution that inherent
power of quashing a criminal proceeding should be exercised very sparingly and with
circumspection and that too in the rarest of rare cases. This Court also held that the High
Court will not be justified in embarking upon an inquiry as to the reliability or
genuineness or otherwise of the allegations made in the F.I.R. or the complaint and that
the extraordinary or inherent powers do not confer an arbitrary jurisdiction on the court to
act according to its whims and caprice.
12. We now refer to a few decisions of this Court deprecating the exercise of
extraordinary or inherent powers by the High Court according to its whims and caprice.
13

. In State of Bihar v. J.A.C. Saldanha (1980) 1 SCC 554 this Court pointed out at SCC p.
574 : AIR 1980 SC 326, Para 28

"The High Court in exercise of the extraordinary jurisdiction committed a grave error by
making observations on seriously disputed questions of facts taking its cue from
affidavits which in such a situation would hardly provide any reliable material. In our
opinion the High Court was clearly in error in giving the direction virtually amounting to
a mandamus to close the case before the investigation is complete. We say no more."
14

. In Hazari Lal Gupta v. Rameshwar Prasad (1972) 1 SCC 452 this Court at SCC p. 455
pointed out : AIR 1972 SC 484, Para 12

"In exercising jurisdiction under Section 561-A of the Criminal Procedure Code, the High
Court can quash proceedings if there is no legal evidence or if there is any impediment to
the institution or continuance of proceedings but the High Court does not ordinarily
inquire as to whether the evidence is reliable or not. Where again, investigation into the
circumstances of an alleged cognizable offence is carried on under the provisions of the
Criminal Procedure Code, the High Court does not interfere with such investigation
because it would then be the impeding investigation and jurisdiction of statutory
authorities to exercise power in accordance with the provisions of the Criminal Procedure
Code."
15

. In Jehan Singh v. Delhi Administration (1974) 4 SCC 522 the application filed by the
accused under Section 561-A of the old Code for quashing the investigation was
dismissed as being premature and incompetent on the finding that prima facie, the
allegations in the FIR, if assumed to be correct, constitute a cognizable offence. AIR
1974 SC 1146

16

. In Kurukshetra University v. State of Haryana (1977) 4 SCC 451, this Court pointed
out : It surprises us in the extreme that the High Court thought that in the exercise of its
inherent powers under Section 482 of the Code of Criminal Procedure, it could quash a
first information report. The police had not even commenced investigation into the
complaint filed by the Warden of the University and no proceeding at all was pending in
any court in pursuance of the FIR. It ought to be realized that inherent powers do not
confer an arbitrary jurisdiction on the High Court to act according to whim or caprice.
That statutory power has to be exercised sparingly, with circumspection and In the rarest
of rare cases. AIR 1977 SC 2229, Para 2

(Emphasis supplied)
17

. In State of Bihar v. Murad Ali Khan (1988) 4 SCC 655 this Court held that the
jurisdiction under Section 482 of the Code has to be exercised sparingly and with
circumspection and has given the working that in exercising that jurisdiction, the High
Court should not embark upon an enquiry whether the allegations in the complaint are
likely to be established by evidence or not. AIR 1989 SC 1

18

. In State of Haryana and ors. (appellant) v. Bhajan Lal and ors. (respondents) 1992 Supp.
(1) SCC 335, this Court after referring to various decisions of this Court, enumerated
various categories of cases by way of illustration wherein the inherent power under
Section 482 of the Code should be exercised by the High Court. They are : 1992 AIR
SCW 237

"(1) Where the allegations made in the first information report or the complaint, even if
they are taken at their face value and accepted in their entirety do not prima facie
@page-SC1129
constitute any offence or make out a case against the accused.
(2) Where the allegations in the first information report and other materials, if any,
accompanying the FIR do not disclose a cognizable offence, justifying an investigation
by police officers under Section 156(1) of the Code except under an order of a Magistrate
within the purview of Section 155(2) of the Code.
(3) Where the uncontroverted allegations made in the FIR or complaint and the evidence
collected in support of the same do not disclose the commission of any offence and make
out a case against the accused.
(4) Where, the allegations in the FIR do not constitute a cognizable offence but constitute
only a non-cognizable offence, no investigation is permitted by a police officer without
an order of a Magistrate as contemplated under Section 155(2) of the Code.
(5) Where the allegations made in the FIR or complaint are so absurd and inherently
improbable on the basis of which no prudent person can ever reach a just conclusion that
there is sufficient ground for proceeding against the accused.
(6) Where there is an express legal bar engrafted in any of the provisions of the Code or
the concerned Act (under which a criminal proceeding is instituted) to the institution and
continuance of the proceedings and/or where there is a specific provision in the Code or
the concerned Act, providing efficacious redress for the grievance of the aggrieved party.
(7) Where a criminal proceeding is manifestly attended with mala fide and/or where the
proceeding is maliciously instituted with an ulterior motive for wreaking vengeance on
the accused and with a view to spite him due to private and personal grudge."
19. We may observe here that despite this Court consistently held in catena of decisions
that inherent power of the High Court should not be exercised according to whims and
caprice and it has to be exercised sparingly, with circumspection and in the rarest of rare
cases, we often come across the High Court exercising the inherent power under Section
482 of the Code of Criminal Procedure in a routine manner at its whims and caprice
setting at naught the cognizance taken and the FIR lodged at the threshold committing
grave miscarriage of justice. While it is true that so long as the inherent power of Section
482 is in the Statute Book, exercise of such power is not impermissible but it must be
noted that such power has to be exercised sparingly with circumspection and in the rarest
of rare cases, the sole aim of which is to secure the ends of justice. The power under
Section 482 is not intended to scuttle justice at the threshold.
20

. The rulings cited by Mr. K. K. Venugopal East India Commercial Co. Ltd., Calcutta and
Anr. v. The Collector of Customs, Calcutta 1963 (3) SCR 338; T. Prem Sagar v. The
Standard Vacuum Oil Company Madras and Ors. 1964 (5) SCR 1030; Boothalinga
Agencies v. V.T.C. Poriaswami Nadar 1969 (1) SCR 65; and S.M.S. Pharmaceuticals Ltd.
v. Neeta Bhalla and Anr. (2005) 8 SCC 89 are not applicable in the facts of this case at
this stage in view of our view above. AIR 1962 SC 1893
AIR 1965 SC 111
AIR 1969 SC 110
2005 AIR SCW 4740

21. In the result, there is no infirmity in the order passed by the High Court warranting
our interference in exercise of our power under Article 136 of the Constitution. This
appeal is, accordingly, dismissed.
22. We clarify that we do not express any opinion on the merits of the case. The trial
court shall decide the matter expeditiously uninfluenced by any observations made by
this Court or the High Court. The trial court shall decide the maintainability of the
complaint at the time of consideration of the charge. We further make it clear that it is
open to the parties to urge all the contentions as available under the law, including the
maintainability of the complaint before the trial judge at the time of consideration of this
charge.
23. With these observations and directions, the appeal is dismissed.
24. MARKANDEY KATJU, J. :- I have perused the judgment of my learned brother
Hon'ble H.K. Sema, J. in this appeal.
25. I respectfully agree with his conclusion that the appeal be dismissed but only because
of the observations in his judgment that we are not expressing any opinion on the merits
of the case. However, I think it is necessary to give my separate concurrent judgment in
this case.
26. The appellant before us, Mr. Som Mittal, is the Managing Director of Hewlett
Packard Global Soft Ltd. He filed a petition
@page-SC1130
under Section 482, Cr.P.C. before the Karnataka High Court challenging the order dated
30-12-2003 passed by the Metropolitan Magistrate Traffic Court III, Bangalore, taking
cognizance of an offence under Section 25 of the Karnataka Shops and Commercial
Establishments Act, 1961 (in short the Act) read with Section 30(3) of the same and also
the conditions imposed by the Karnataka Government in its order dated 9-2-2005. It may
be mentioned that cognizance was taken on a complaint filed by the respondent through
its Senior Labour Inspector, 18th Circle, Bangalore.
27. Section 25 as amended by Act No. 14 of 2002 reads as follows :
"25. Prohibition of employment of women and young persons during night : No woman,
or a young person, shall be required or allowed to work whether as an employee or
otherwise in any establishment during nights.
Provided that the State Government may, by notification exempt any establishment of
Information Technology or Information Technology enabled service from the provisions
of this section relating to, employment of women during night subject to the condition
that the establishment provides facilities of transportation and security to such women
employees and subject to any other condition as may be specified in the notification."
28. It may be noted from the above provision that while the main part of Section 25 is
prohibition of employment of women and young persons in a shop or commercial
establishment during night, the proviso enables the State Government to exempt any
establishment of Information Technology from the provisions of the section subject to the
condition that the establishment provides facility for transportation and security to the
woman employees.
29. The Deputy Labour Commissioner, Region 2, Bangalore, in exercise of the power
under the proviso to Section 25 issued an office order in terms of Section 25 read with
Rule 24(b) of the Karnataka Shops and Commercial Establishments Rules 1963 granting
exemption. Condition No. 2 of the said Order stated :
"Transport facilities from the residence to workplace and back shall be provided free of
cost and with adequate security." .
30. It appears that on 13-12-2005 at about 2 a.m. a woman employee of the Company of
which the appellant was Managing Director was travelling from her house to the
workplace situated in Electronic City, Bangalore. While on the way the vehicle driver
took the vehicle to a secluded place and raped and killed the said woman employee. This
fact finds reference in the letter of the Bangalore City Police Commissioner dated 26-12-
2005 addressed to the Labour Commissioner, and in the said letter it is stated that
adequate security had not been provided to the said woman employee during her travel
from her home to the workplace. It is on the basis of this letter that the complaint was
filed on the basis of which cognizance was taken by the learned Magistrate.
31. Shri K.K. Venugopal, learned counsel for the appellant, has invited our attention to
Section 3(l)(h) of the Act which states
"3(1) Nothing in this Act shall apply to - (h) person occupying positions of management
in any establishment."
32. We agree with Shri Venugopal that the Managing Director is surely a person
occupying a position of management in the establishment and hence Section 3(l)(h) is
clearly attracted to the facts of this case.
33. However, learned counsel for the State Government has relied on Section 2(h) of the
Act which states :
"2(h) "Employer" means a person having charge of or owning or having ultimate control
over the affairs of an establishment and includes members of the family of an employer, a
manager, agent or other person acting in the general management or control of an
establishment;"
34. Learned counsel for the respondent submitted that Section 30(1) of the Act states that
"Whoever contravenes any of the provisions of Sections 4, 5 -------, 25 and 39, shall, on
conviction, be punished with fine....." She submitted that the word "whoever" in section
30 is broad enough to include the Managing Director also.
35. To my mind, there seems to be some apparent conflict between section 30 and section
3(1)(h) of the Act since while the latter provision states that a person in a position of
management is outside the purview of the Act, it is contended by counsel for the
respondent that the former provision includes a person in management also since
@page-SC1131
the word whoever is very wide.
36. Since section 30 is also part of the Act, hence prima facie it seems that a Managing
Director does not come within the purview of the Act in view of section 3(l)(h). It prima
facie seems that only persons not in a position of management will come within the
purview of the Act, and hence they alone can be penalized under Section 30. If persons in
a position of management are also intended to be penalized then that will require an
amendment to the Act, in particular Section 3(l)(h) thereof. The Court cannot amend an
Act of the legislature, and cannot fill up a casus omissus.
37. However, I am not expressing any final opinion on the merits of the matter, and it is
left open for the court concerned to interpret the various provisions of the Act.
38. While I agree with my learned brother, Hon'ble Sema, J. that the power under section
482 Cr.P.C. is to be exercised sparingly, I cannot agree with my learned brother that it
should be exercised in the rarest of the rare cases.
39. The expression rarest of the rare cases was used in connection with Section 302, IPC
to hold that death penalty should only be imposed in rarest of rare cases vide Constitution
Bench decision of this Court in Bachan Singh v. State of Punjab (vide para 207), AIR
1980 SC 898. In my opinion, this expression cannot be extended to a petition under
Section 482, Cr.P.C.. Though I agree with my learned brother Hon'ble Sema, J. that the
power under Section 482, Cr.P.C. should be used sparingly, yet there may be occasions
where in the interest of justice the power should be exercised.
40. In this connection, I would also like to refer to the situation prevailing in the State of
Uttar Pradesh where due to deletion of the provision for anticipatory bail under Section
438, Cr.P.C. by Section 9 of the U.P. Act 16 of 1976, huge difficulties have been created
both for the public as well as for the Allahabad High Court.
41. It may be noted that in U.P. such provision for anticipatory bail has been deleted
while it continues to exist in all other States in India, even in terrorist affected States. The
result is that thousands of petitions under Section 482 are filed every year in Allahabad
High Court praying for stay of arrest or for quashing the FIR, because in the absence of
the provision of anticipatory bail many persons who are innocent cannot get anticipatory
bail even though the FIR filed against them may be frivolous and/or false. Even if such
persons get regular bail under Section 439, before that they will have to go to jail, and
thus their reputation in society may be irreparably tarnished.
42

. It has been held by this Court in Joginder Kumar v. State of U.P. and others AIR 1994
SC 1349 (vide para 24) that No arrest can be made because it is lawful for the Police
Officer to do so. The existence of the power to arrest is one thing and the justification for
the exercise of it is quite another. The Police Officer must be able to justify the arrest
apart from his power to do so. Arrest and detention in police lock up of a person can
cause incalculable harm to the reputation and self esteem of a person. No arrest can be
made in a routine manner on a mere allegation of commission of an offence made against
a person. It would be prudent for a Police Officer in the interest of protection of the
constitutional right of a citizen and perhaps in his own interest that no arrest should be
made without a reasonable satisfaction reached after some investigation as to the
genuineness and bona fides of a complaint and a reasonable belief both as to the person's
complicity and even so as to the need to effect arrest. Denying a person of his liberty is a
serious matter. The recommendation of the Police Commissioner merely reflects the
constitutional concomitants of the fundamental right to personal liberty and freedom. A
person is not liable to arrest merely on the suspicion of complicity in an offence. There
must be a reasonable justification in the opinion of the officer effecting the arrest that
such arrest is necessary and justified. Except in heinous offences, an arrest must be
avoided if a police officer issues notice to a person to attend the Station House and not to
leave Station without permission would do." 1994 AIR SCW 1886

43. In para 13 of the same judgment this Court has also referred to the Third Report of the
National Police Commission which stated that by and large nearly 60% of the arrests in
the country were unnecessary or unjustified. Also, 43.2 % of the expenditure in jails was
over such prisoners only who need not have been arrested at all.
44. Despite this categorical judgment of the Supreme Court it appears that the police
@page-SC1132
is not at all implementing it. What invariably happens is that whenever an FIR of a
cognizable offence is lodged the police immediately goes to arrest the accused person.
This is clear violation of the aforesaid judgment of the Supreme Court.
45

. It may be noted that Section 2(c), Cr.P.C. defines a cognizable offence as an offence in
which a police officer may arrest without warrant. Similarly Section 41, Cr.P.C. states a
police officer may arrest a person involved in a cognizable offence. The use of the word
'may' shows that a police officer is not bound to arrest even in a case of a cognizable
offence. When he should arrest and when not is clarified in Joginder Kumar's case
(supra). 1994 AIR SCW 1886

46. Again in Section 157(1), Cr.P.C. it is mentioned that a police officer shall investigate
a case relating to a cognizable offence, and if necessary take measures for the arrest of the
offender. This again makes it clear that arrest is not a must in every case of a cognizable
offence.
47. Because of absence of the provision for anticipatory bail in U.P., thousands of writ
petitions and Section 482, Cr.P.C. applications are being filed in the Allahabad High
Court praying for stay of the petitioners arrest and/or quashing the FIR. This is
unnecessarily increasing the work load of the High Court and adding to the arrears, apart
from the hardship to the public, and overcrowding in jails.
48. The right to liberty under Article 21 of the Constitution is a valuable right, and hence
should not be lightly interfered with. It was won by the people of Europe and America
after tremendous historical struggles and sacrifices. One is reminded of Charles Dickens
novel 'A Tale of Two Cities' in which Dr. Manette was incarcerated in the Bastille for 18
years on a mere 'lettre de cachet' of a French aristocrat, although he was innocent.
49. In Ghani vs. Jones (1970) 1 Q.B. 693 (709) Lord Denning observed :
"A man's liberty of movement is regarded so highly by the Law of England that it is not
to be hindered or prevented except on the surest grounds."
The above observation has been quoted with approval by a Constitution Bench decision
of this Court in Maneka Gandhi v. Union of India, AIR 1978 SC 597 (vide para 99).
50. Despite this clear enunciation of the law many people are arrested and sent to the jail
on the basis of false and/or frivolous FIRs.
51. In my opinion the problem will be obviated by restoring the provision for anticipatory
bail which was contained in Section 438, Cr.P.C. but was deleted in U.P. by Section 9 of
U.P. Act 16 of 1976.
52. It is surprising that the provision for anticipatory bail has been deleted in U.P
although it exists in all other States in India, even in terrorist affected States. I do not
understand why this provision should not exist in U.P. also.
53. As pointed out in Balchand Jain v. State of Madhya Pradesh, AIR 1977 SC 366, the
provision for anticipatory bail was included in the Cr.P.C. of 1973 in pursuance of the
Forty First Report of the Law Commission which observed :-
"The necessity for granting anticipatory bail arises mainly because sometimes influential
persons try to implicate their rivals in false cases for the purpose of disgracing them or
for other purposes by getting them detained in jail for some days. In recent times, with
the accentuation of political rivalry, this tendency is showing signs of steady increase.
Apart from false cases, where there are reasonable grounds for holding that a person
accused of an offence is not likely to abscond, or otherwise misuse his liberty while on
bail, there seems no justification to require him first to submit to custody and remain in
prison for some days and then apply for bail."
54

. Thus the provision for anticipatory bail was introduced in the Cr.P.C. because it was
realized by Parliament in its wisdom that false and frivolous cases are often filed against
some persons and such persons have to go to jail because even if the First Information
Report is false and frivolous a person has to obtain bail, and for that he has to first
surrender before the learned Magistrate, and his bail application is heard only after
several days (usually a week or two) after giving notice to the State. During this period
the applicant has to go to jail. Hence even if such person subsequently obtains bail his
reputation may be irreparably tarnished, as held by the Supreme Court in Joginder
Kumar's case (supra). The reputation of a person is a valuable asset for him just as in law
the good will of a firm is an intangible 1994 AIR SCW 1886

@page-SC1133
asset. In Gita, Lord Krishna said to Arjun :
lEHkkforL; pkdhfrZej, kknfrfjP; rss
"For a self-respecting man, death is
preferable to dishonour"
(Gita Chapter 2, Shloka 34)
55. No doubt anticipatory bail is not to be granted as a matter of course by the Court but
only in accordance with the principles laid down by the Supreme Court in Gurbaksh
Singh v. State of Punjab, AIR 1980 SC 1632. However, we are of the view that there must
be a provision for anticipatory bail in U.P. for the reason already mentioned above.
56

. Experience has shown that the absence of the provision for anticipatory bail has been
causing great injustice and hardship to the citizens of U.P. For instance, often false FIRs
are filed e.g. under Section 498AIPC, Section 3/4 Dowry Prohibition Act etc. Often aged
grandmothers, uncles, aunts, unmarried sisters etc. are implicated in such cases, even
though they may have nothing to do with the offence. Sometimes unmarried girls have to
go to jail, and this may affect their chances of marriage. As already observed by me
above, this is in violation of the decision of this Court in Joginder Kumar's case (supra),
and the difficulty can be overcome by restoring the provision for anticipatory bail. 1994
AIR SCW 1886

57. Moreover, the Allahabad High Court is already over burdened with heavy arrears and
overloaded with work. This load is increasing daily due to the absence of the provision
for anticipatory bail. In the absence of such provision whenever an FIR is filed the
accused person files a writ petition or application under Section 482, Cr.P.C. and this has
resulted in an unmanageable burden on this Court. Also jails in U.P. are overcrowded.
58. The Allahabad High Court had on several occasions requested the State Government
to issue an Ordinance immediately to restore the provision for anticipatory bail, (e.g. in
Vijay Kumar Verma v. State of U.P., 2002 Cr.L.J. 4561) but all its requests seem to have
fallen on deaf ears. It seems that there is an impression in some quarters that if the
provision for anticipatory bail is restored crimes will increase. In my opinion this is a
specious argument, since it has not made much difference to the crime position in the
States where the provision for anticipatory bail exists, even in terrorist affected States. No
doubt the recommendation of a Court is not binding on the State Government/State
Legislature but still it should be seriously considered, and not simply ignored. The Court
usually makes a recommendation when it feels that the public is facing some hardship.
Such recommendation should, therefore, be given respect and serious consideration.
59. I, therefore, make a strong recommendation to the U.P. Government to immediately
issue an Ordinance to restore the provision for anticipatory bail by repealing Section 9 of
U.P. Act No. 16 of 1976, and empowering the Allahabad High Court as well as the
Sessions Courts in U.P. to grant anticipatory bail.
60. In this connection I may also refer to the decision of the Seven Judge Full Bench of
Allahabad High Court in Smt. Amarawati and another v. State of U.P. (2005 Crl. L.J. 755)
in which the Full Bench has mentioned that the Sessions Judge while considering a bail
application under Section 439, Cr.P.C. can grant interim bail till the final disposal of the
bail application subsequently. This will enable innocent persons to avoid going to jail
pending consideration of their bail application.
61

. I am informed that despite this Seven Judge Full Bench judgment which has clearly
mentioned that a Sessions Judge can grant interim bail, the Session Courts in U.P. are
ignoring the said judgment and are not granting interim bail pending disposal of the final
bail application even in appropriate cases. This is wholly improper. Decisions of this
Court and of the High Court must be respected and carried out by the sub-ordinate courts
punctually and faithfully. It is, therefore, directed that Amarawati's case (supra) must be
implemented in letter and spirit by the Sessions Courts in U.P. and in this connection the
Registrar General of Allahabad High Court will circulate letters to all the District Judges
in U.P. along with a copy of this judgment to ensure faithful compliance of the decision
of the Full Bench decision of the High Court in Amarawati's case (supra). 2005 AIR
SCW 755

62

. The Secretary General of this Court shall send a copy of my judgment to the Chief
Secretary, Home Secretary and Law Secretary of U.P. as well as to the Registrar General
of Allahabad High Court and also to the 1994 AIR SCW 1886

@page-SC1134
President/Secretary of Allahabad Bar Association and the Allahabad High Court
Advocates Association as well as Oudh Bar Association, Lucknow forthwith. A copy
shall also be sent to the Chief Secretary, Home Secretary and Law Secretary of all State
Governments/Union Territories in India who shall direct all officials to strictly comply
with the judgment of this Court in Joginder Kumar's case (supra).
Order accordingly.
AIR 2008 SUPREME COURT 1134 "Gobarbhai Naranbhai Singala v. State of Gujarat"
(From : Gujarat)*
Coram : 2 ASHOK BHAN AND ALTAMAS KABIR, JJ.
Criminal Appeal Nos. 198 with 199 of 2008 (arising out of SLP (Cri.) No. 6646 of 2005
with 4283 of 2006), D/- 29 -1 -2008.
Gobarbhai Naranbhai Singala v. State of Gujarat and Ors.

WITH
Jayeshbhai @ Panchabhai Muljibhai Satadiya v. Jayrajsinh Temubha Jadeja and Anr.
(A) Criminal P.C. (2 of 1974), S.437 - BAIL - MURDER - Bail - Grant of - Accused
charged for murder - Fact that he did not misuse his liberty while on temporary bail twice
- By Itself is no ground to grant bail.
The fact that accused did not misuse his liberty while on temporary bail twice by itself is
no ground to grant bail in a murder case especially when he was allegedly involved in a
subsequent case of murder. Further, apart from the present two cases of murder, the
accused has been named in 10 other criminal cases in the last 25 years or so, out of which
5 cases were under S. 307 IPC for attempt to murder and another under S. 302 IPC for
committing murder. Furthermore, the accused has been acquitted in most of the cases for
want of sufficient evidence. This speaks volumes. (Paras 20, 21)
(B) Criminal P.C. (2 of 1974), S.437 - BAIL - MURDER - Bail - Grant of - Murder case -
Complainants were not instrumental in delaying the trial - Grant of bail to accused on
ground that trial of case has not progressed/ begun - Not proper. (Paras 20, 22)
(C) Criminal P.C. (2 of 1974), S.439 - BAIL - SUPREME COURT - Cancellation of bail -
Basic requirement necessary for grant of bail completely ignored by High Court -
Cancellation of bail by Supreme Court - Permissible. (Paras 24, 27)
(D) Criminal P.C. (2 of 1974), S.437 - BAIL - MURDER - INVESTIGATION - Bail -
Murder case - Three eye-witnesses unequivocally stated in their statements u/S.161 Cr. P.
C. that accused was present at time of occurrence and he had fired with his gun - Prima
facie a case for grant of bail could not be said to have been made out.
Crl. Misc. A. No. 8305 of 2004. D/-26-10-2005 (Guj.), Reversed. (Paras 24, 29)
Cases Referred : Chronological Paras
2005 AIR SCW 1103 : AIR 2005 SC 1299 : 2005 Cri LJ 1721 (Ref.) 26
2005 AIR SCW 4763 : AIR 2005 SC 3490 : 2005 Cri LJ 4149 : 2005 All LJ 3368 (Ref.)
23, 25, 28
Arun Jaithley, N.D. Nanavati, Sr. Advocates, Sushil Kumar Jain, Bharat T. Rao, Puneet
Jain, N. Ganpathy, Huzefa Ahmadi, Ejaz Maqbool, Vikash Singh, Pradhuman Goghil,
Abhijeet Sinha, Ms. Pinky Behera and Ms. Hemantika Wahi, with them for the appearing
parties.
* Cri. Misc. A. No. 835 of 2004, D/- 26-1-2005 (Guj.)
Judgement
BHAN, J. :- Leave granted.
2. This judgment shall dispose of the Criminal Appeal arising out of SLP(Crl) No.
4283/2006 (for short 1st case) and Criminal Appeal arising out of SLP(Crl) No. 6646/
2005 (for short 2nd case).
3. The 1st case has been filed by the complainant Jayeshbhai @Panchabhai M. Satodiya
seeking cancellation of the bail granted to respondent Shri Jayrajsinh Temubha Jadeja
who, at that time, was a sitting Member of Legislative Assembly of Gujarat (the first
accused in Criminal Case No. 1-25/2004 registered under Sections 302, 307, 143, 147,
148, 149, 341, 120B, 201 IPC and Sections 25(1) A, (1-a), 27 of the Arms Act and
Section 135 of the Bombay Police Act at Police Station Gondal City) for the alleged
murder of Nilesh.
4. The 2nd case has been filed by the complainant, namely, Gobarbhai Naranbhai Singala
seeking cancellation of the bail granted to the respondents Shri Shivbhadrasinh @
Gopalsinh Giriraj Jadeja (respondent No.2 herein) and Shri
@page-SC1135
Jayrajsinh Themubha Jadeja (respondent No.3 herein) [2nd and 3rd accused in Criminal
Case No.1-173/2005, wrongly mentioned as 1-102/2004 dated 19-3-2004] registered at
Police Station Malviya Nagar, Rajkot City, Rajkot under Sections 143, 148, 149, 449, 302
IPC and 25(l)(b)(a) of the Arms Act, for the alleged murder of appellant's son Vinodrai
Singala.
5. The facts in brief culminating into filing of these appeals are narrated below.
6. That, on 9th February, 2004 the complainant Jayeshbhai [appellant in 1st case] along
with Nilesh Rayani and Ramjibhai Markana had allegedly gone to hostel. While coming
back, they were followed by a car, which overtook Jayeshbhai's vehicle driven by
Ramjibhai. When they reached near Central Talkies at about 8.15 p.m., three persons,
namely Jairajsinh Jadeja, Amarjit Singh and Bhagat came out of the car and allegedly
fired at the Jayeshbhai's car. The said fire hit the glass of the car. Jayeshbhai was hit by
the shattered pieces of glass. Ramjibhai, who was driving the car at that point of time, to
save himself, came out of the vehicle and tried to run away. But finding that there was
another car which was following them, he again entered the vehicle. Meanwhile, the
deceased Nilesh started driving the vehicle. The respondent-Jayrajsinh Jadeja again fired
which hit Nilesh Rayani. The car went out of control and hit another car and thereafter
side railing on the road. The appellant and Ramjibhai ran away from the scene and, while
running, they saw that the Nilesh Rayani was being beaten by other persons. Nilesh died
on the spot. The appellant came to his house where Vinubhai Singala and others were
present. Appellant thereafter narrated the story to Vinubhai Singala and then they went to
meet Ramjibhai, who was hiding in the town hall out of fear. They lodged the report, i.e.,
Case No. 1-25/2004 under Sections 302, 143, 147, 148, 149, 341, 307, 120B, 201, IPC,
Sections 25(l)(a) and 27 of the Arms Act and Section 135 of the Bombay Police Act.
7. On 17th February, 2004 the respondent was released on anticipatory bail by the
Additional Sessions Judge, Rajkot in Criminal Misc. Application No. 28/2004. The State
of Gujarat filed CPA No. 102/2004 in the High Court, seeking cancellation of the
anticipatory bail granted to the respondent. Appellant also filed CRA No.92/2004 for the
same relief. Anticipatory bail granted to the respondent was cancelled by the High Court
on 5th March, 2004.
8. On 19th March, 2004 Vinubhai Singala (Vinod Rai Singala) was murdered. The
respondent was named in the FIR as one of the assailants. The case was registered as
Criminal Case No. 1-173/05 at P.S. Malviya Nagar, Rajkot. The said FIR was lodged by
Gobarbhai Naranbhai Singala, father of the deceased Vinubhai Singala, appellant in the
1st Appeal under Sections 143, 148, 149, 447, 302 IPC and 23(l)(b) of Arms Act. In the
said FIR it was inter alia alleged :
"Today at about 8/00 to 8/30 a.m. my son Vinodrai was reading newspaper in garden of
bungalow compound and was seating in the coat, nearby Vanupuriyabhai was seating.
And after giving grass to my cow myself came in garden and heard a sound of firing and
when I seen that were two persons who were fining out of which one Viranndev and other
Gopalsinh after firing both of them ran away to the east side of building wall which I
have seen I know them they are residing in Gondal. I do not know full name of the
person, they were jumped the wall at that time on wall M.L.A. Jayrajsinh Jadeja was
standing and the person who run away has told him that they have completed Vinodrai
Singala and revenge has been taken and telling this they got down the wall and run away
with them." (sic)
9. Pursuant to the cancellation of bail by the High Court in Criminal Case No.1-25/2004,
the respondent surrendered on 20th of March, 2004. After his arrest, the respondent filed
an application for bail in Criminal Case-No.1-25/2004.
10. The High Court by its order dated 14th September, 2004 refused to grant bail to the
respondent, inter alia, observing :
"From the aforesaid facts and looking to the statements of the witnesses, panchnama,
reports, including ballistic report, prima facie reflects that there is direct involvement of
the applicants in commissioning of the offences. The applicants were not available after
the offence is committed as narrated in para 6 and para 8 of the judgment by the trial
court. Looking to gravamen of charge against the present applicants, their involvement
(prima facie clearer from the record of the case), quantum of
@page-SC1136
punishment, their tenancy (sic) to remain away from police (as per para 6 and para 8 of
judgment of the trial court), leads me to believe that they may not be available at the time
of trial and due to cumulative effect of all the aforesaid factors, I am not inclined to use
discretionary power, to enlarge the present applicants on bail, there is no substance in the
present application and hence, the present application is rejected. Notice is discharged."
11. Aggrieved against the rejection of bail by the High Court in Crl. Misc. Application
No. 7579/2004, the respondent filed a petition being SLP (Crl) No. 1128/2005 before this
Court.
12. This Court while rejecting the petition on 18th February, 2005, observed as under :
"Delay condoned.
It is stated that the petitioner has been in custody since 28th March 2004 and in the case
of the co-accused whose bail application was rejected, this Court made an observation on
1-11-2004, that the bail application could be renewed after six months.
Though we are not inclined to interfere at this stage, we would like to give liberty to the
petitioner to renew bail application before the High Court after four months. Such bail
application, if filed, shall be considered on its own merits." (sic)
13. Respondents filed M.A. No.8305/ 2004 seeking bail. On 26th October, 2005 the High
Court granted bail to the respondents herein in CR No.1-173/2005, lodged by the
appellant in the 2nd case - Gobarbhai.
14. Against the aforesaid order, the complainant Gobarbhai has filed the appeal arising
out of SLP(Crl) No. 6646/2005, seeking cancellation of the bail granted to the
respondents, in which this Court issued notice on 16th of December, 2005.
15. In Criminal Case No. 1-25/2004 (incident of 9th February 2004) respondent filed an
application for temporary bail, which was granted by the High Court on 23rd of
December, 2005 for one month. Respondent was released on bail on 27th December,
2005. Appellant filed an application for cancellation of temporary bail granted to the
respondent. In the meantime, on expiry of the period of temporary bail granted to the
respondent, the respondent had surrendered on 27-1-2006. Thus, application filed by the
appellant, seeking cancellation of the temporary bail was dismissed as infructuous on
10th February, 2006, as the respondent had already surrendered.
16. On 3rd of March, 2006, the respondent filed another application for temporary bail in
Criminal Case No. 1-25/2004, which was granted by the High Court for a period of one
month from 6th March, 2006 to 5th of April, 2006. After expiry of the period of
temporary bail, the respondent surrendered and thereafter filed a Criminal Misc.
Application, which came up for hearing before the same Hon'ble Judge who had heard
Crl.Misc. Application No. 7579/2004 filed earlier by the respondent in which prayer for
grant of bail was turned down by his order dated 14th September, 2004. This time, the
Hon'ble Judge granted regular bail to the respondent by observing thus :
"5. Having heard the learned advocates for the rival sides and looking to the facts and
circumstances of the case, it appears that the applicant is in judicial custody since March,
2004. Sessions case has not yet commenced and no prosecution witness has been
examined. Moreover, this Court has twice granted temporary bail to the present applicant,
initially for the period from 27th December, 2005 to 27th January, 2006 with stringent
conditions and the present applicant had surrendered to the judicial custody in time
without any breach of conditions. Similarly, for the second time also, this Court had
granted temporary bail to the present applicant for the period from 6th March, 2006 to 5th
March, 2006 (sic) with stringent conditions and at that time also, the applicant had
surrendered to judicial custody in time without any breach of conditions. The offence
being Cr. No.1-102 of 2004 was registered in the intervening period wherein charge sheet
has already been filed which is not against the present applicant. Thus, twice this Court
has tested the present applicant for one month. On each of the occasions, the conditions
imposed by this Court have been fulfilled and obeyed by the applicant coupled with the
fact that no prosecution witness has yet been examined though period of more than two
years have elapsed.
6. In view of the above facts and circumstances of the case, this application is required to
be allowed and the applicant is required to be enlarged on bail. Accordingly, the applicant
is hereby ordered to be enlarged on bail in pursuance of the offence
@page-SC1137
registered bearing C.R. No.I-25/2004 at Gondal Police Station on his furnishing a bond of
Rs. 50,000/- (Rs. Fifty Thousand) and solvent surety of the like amount, on the following
terms and conditions that he shall :
(a) not take undue advantage of his liberty or abuse his liberty;
(b) not act in a manner injurious to the interest of the prosecution;
(c) maintain law and order;
(d) mark his presence on every Tuesday and Friday in a week at Sector 21, Police Station,
Gandhinagar between 9.00 a.m. to 2.00 p.m.;
(e) not leave the State of Gujarat without prior permission of the Sessions Court
concerned;
(f) furnish the address of his residence at the time of execution of the bond and shall not
change the residence without prior permission of this Court;
(g) not enter into the local limits of district Rajkot without prior permission of this Court,
but for attending the Court in connection with this case he will be free to enter the limits
for a period to the extent necessary and will leave the limits thereafter soon after the case
is adjourned;
(h) surrender his passport, if any, to the lower court within a week.
17. Aggrieved against the aforesaid order granting bail to the respondent in Crl. Case No.
I-25/2004, Jayeshbhai has filed the Appeal arising out of SLP(Crl) No. 4283/ 2006,
seeking cancellation of bail.
18. Heard learned counsel appearing for the parties.
19. The High Court by the impugned order has granted bail to the respondent-Jayrajsinh
Jadeja (in 2nd case) on three grounds (i) that the respondent was in judicial custody since
March, 2004; (ii) that trial had yet not commenced and no prosecution witness had been
examined; and (iii) that the Court had tested the respondent twice by granting temporary
bail to him with stringent conditions for a duration of one month each, i.e., from 27th
December 2004 to 27th January, 2005 and 6th March, 2006 to 5th April, 2006 and, on
both the occasions, the respondent had surrendered within time, without breach of any of
the conditions.
20. From a reading of the impugned order it is found that the learned Judge, who
incidentally happens to be the same Judge who had declined to release the respondent on
bail earlier, did not advert to any of the reasons given by him declining to release the
respondent on bail. There was no change of circumstances. The reasons given by the
learned Judge in the impugned order for grant of bail are untenable.
21. That the respondent did not misuse his liberty while on temporary bail twice by itself
is no ground to grant bail in a murder case especially when he was allegedly involved in a
subsequent case of murder. It may be mentioned here that apart from the present two
cases of murder, respondent has been named in 10 other criminal cases in the last 25
years or so, out of which 5 cases were under Section 307, IPC for attempt to murder and
another under Section 302, IPC for committing murder. We are informed at the Bar that
the respondent has been acquitted in most of the cases for want of sufficient evidence.
This speaks volumes. We refrain from saying anything further, lest it may prejudice the
trial in these two cases.
22. The other reason given in the impugned order is that the trial of the case has not
progressed/begun. We find from the record that between 2nd June, 2004 and 19th
December, 2005 the case was listed before the trial Court 31 times and on each date, it
had to be adjourned on the ground that one or the other accused was not present. There
are 16 accused in the case. It is not clear from the record whether the accused were not
brought by the police from the jail or that they were on bail and had not appeared of their
own, but the fact remains that the complainants were not in any way instrumental in
delaying the trial between 2nd June, 2004 and 19th December, 2005. It was brought to
our notice that the only witness who has been examined so far has turned hostile. Trial
was stayed by the High Court on 15th February, 2007 at the instance of the appellant as
Shri R.R. Trivedi, A.P.P., to whom the case had been assigned for conducting the trial and
was allegedly the counsel for the respondent in some other case earlier, continued to
appear in the case in spite of the fact that he was replaced by another A.P.P. It just shows
that the trial was not progressing smoothly. In any case, complainant party was in no way
responsible for any delay in trial.
23. The third reason given by the High Court for grant of bail, that the respondent
@page-SC1138
had been in jail for the last more than 2 years, is equally untenable in view of the
observations made by this Court in State of U.P. v. Amarmani Tripathi 1(2005) 8 SCC 21]
:
"the condition laid down under Section 437(1)(i) is sine qua non for granting bail even
under Section 439 of the Code. In the impugned order it is noticed that the High Court
has given the period of incarceration already undergone by the accused and the
unlikelihood of trial concluding in the near future as grounds sufficient to enlarge the
accused on bail, in spite of the fact that the accused stands charged of offences punishable
with life imprisonment or even death penalty. In such cases, in our opinion, the mere fact
that the accused has undergone certain period of incarceration (three years in this case) by
itself would not entitle the accused to being enlarged on bail, nor the fact that the trial is
not likely to be concluded in the near future either by itself or coupled with the period of
incarceration would be sufficient for enlarging the appellant on bail when the gravity of
the offence alleged is severe and there are allegations of tampering with the witnesses by
the accused during the period he was on bail."
[Underlining is ours]
24. Shri Arun Jaitley, learned senior counsel appearing for the respondents, submitted that
this Court should not ordinarily interfere in the matters relating to bail. It was pointed out
that in the last two years, the respondent has not misused the liberty granted to him. There
is no doubt that this Court does not ordinarily interfere in the matters granting bail but the
same is subject to certain exceptions. When the basic requirements necessary for grant of
bail are completely ignored by the High Court, this Court would be justified in cancelling
the bail. In the present case, three witnesses, who had allegedly seen the occurrence, have
unequivocally in their statements under Section 161, Cr.P.C. have stated that the
respondent, was present at the time of occurrence and he had fired with his gun. Prima
facie a case for grant of bail was not made out.
25

. This Court in Amarmani Tripathi's case (supra) had held that while considering the
application for bail, what is required to be looked is, (i) whether there is any prima facie
or reasonable ground to believe that the accused had committed the offence; (ii) nature
and gravity of the charge; (iii) severity of the punishment in the event of conviction; (iv)
danger of accused absconding or fleeing if released on bail; (v) character, behaviour,
means, position and standing of the accused; (vi) likelihood of the offence being
repeated; (vii) reasonable apprehension of the witnesses being tampered with; and (viii)
danger, of course, of justice being thwarted by grant of bail.2005 AIR SCW 4763

26

. In Panchanan Mishra v. Digambar Mishra, [(2005) 3 SCC 143], this Court while
considering the question of cancellation of bail, observed : 2005 AIR SCW 1103, Para
13

"The object underlying the cancellation of bail is to protect the fair trial and secure justice
being done to the society by preventing the accused who is set at liberty by the bail order
from tampering with the evidence in the heinous crime.... It hardly requires to be stated
that once a person is released on bail in serious criminal cases where the punishment is
quite stringent and deterrent, the accused in order to get away from the clutches of the
same indulge in various activities like tampering with the prosecution witnesses,
threatening the family members of the deceased victim and also create problems of law
and order situation."
27. We are of the view that the High Court has completely ignored the general principles,
for grant of bail in a heinous crime of commission of murder in which the sentence, if
convicted, is death or life imprisonment.
28

. In the second case, another learned Judge has granted the bail by the impugned order
which runs into 22 pages. The findings recorded therein touch upon the merits of the
case. The learned Judge has proceeded as if an order of acquittal is being passed. This
Court in Amarmani Tripathi's case (supra) has held that a detailed examination of the
evidence is to be avoided while considering the question of bail, to ensure that there is no
pre-judging and no prejudice is caused. Only a brief examination is to be done to satisfy
about the facts and circumstances or otherwise of a prima facie case. 2005 AIR
SCW 4763

29. Taking the overall view of the entire matter and in particular to the antecedents of the
respondent-Jayrajsinh Temubha Jadeja, the alleged statements made by the witnesses,
who were present at the spot, to
@page-SC1139
the police and the admitted enmity between the parties (which is a double edged weapon
to commit the crime as well as to falsely implicate), we are of the view that it was not a
fit case to grant bail to the respondents in this case as well. Without elaborating further,
we set aside the impugned orders granting bail to the respondents. Respondents are
directed to surrender to the judicial custody forthwith. In case, the respondents do not
surrender within seven days, steps be taken, in accordance with law, to apprehend them.
30. The counsel appearing for the State of Gujarat has informed us that Shri R.R. Trivedi
has been replaced by Ms. Amita Ben Sippy as the new A.P.P. to conduct the trial. She has
assured us that Shri R.R. Trivedi will not appear and conduct the trial in either of these
two cases. Thus, the grievance of the appellants on the basis of which the trial was stayed,
stands redressed. Stay of trial granted by the High Court on 15th February, 2007 is
vacated. Trial of the case to begin forthwith. Trial Court is directed to take up the trial on
day to day basis and, if possible, conclude the same within the next six months from the
date of production of a certified copy of this judgment. The prosecution as well as
defence counsel is directed to co-operate in conducting the trial on day to day basis. In
case, any of the accused who is on bail and does not appear, then his bail be cancelled and
he be taken into custody.
31. Nothing stated herein above be taken as an expression of opinion on merits of the
matter. The trial court shall proceed with the trial in accordance with law, without in any
manner being influenced by the observations made herein above or in the Orders passed
by the High Court granting bail to the respondents.
32. With these observations, the appeals are allowed.
Appeals allowed.
AIR 2008 SUPREME COURT 1139 "Fulchand Munda v. State of Bihar"
(From : Patna)*
Coram : 2 PRAKASH PRABHAKAR NAOLEKAR AND DALVEER BHANDARI,
JJ.
Civil Appeal No. 3267 of 2001, D/- 24 -1 -2008.
Fulchand Munda v. State of Bihar and Ors.
(A) Chhotanagpur Tenancy Act (6 of 1908), S.46, S.71A - Civil P.C. (5 of 1908), S.11 -
TENANCY - RES JUDICATA - DECLARATION OF TITLE - MORTGAGE -
Restriction on transfer by raiyat - Suit for declaration of title and restoration of possession
by Landlords - Dismissed by High Court on ground that oral usufructuary mortgage was
invalid under Transfer of Property Act - Defendants are, therefore, not holding under
mortgage - Plaintiff not raising plea that mortgage was barred by S.46 of 1908 Act -
Cannot be raised in subsequent application for restoration of possession u/S.71A. (Para
5)
(B) Chhotanagpur Tenancy Act (6 of 1908), S.71A, S.46 - TENANCY - RESTORATION
- POSSESSION - LIMITATION - Restoration of possession of land unlawfuly
transferred - Limitation - Not prescribed - But party has to approach authority within
reasonable time - Restoration cannot be ordered in case party approaches after more than
50 years. (Para 5)

S.B. Upadhyay, Sr. Advocate, Kumud L. Das, Shiv Mangal Sharma, Rajesh R. Dubey,
Ms. Santosh Mishra, Pawan Upadhyay and Ms. Sharmila Upadhyay, for Appellant; Sunil
Kumar, Sr. Advocate, Manish Mohan, Mrs. Anit Kanuga, Ms. Mridula Ray Bharadwaj,
Ashok Mathur, Ajit Kumar Sinha and Nitish Masey, for Respondents.
* L.P.A. No. 145 of 1999 (R)., D/- 19-1-2000 (Pat)
Judgement
1. P. P. NAOLEKAR, J. :-The brief facts of the case necessary for deciding the questions
involved are that the land of Plot Nos. 1695, 517 and 802 under Khata No. 288 within
Khewat No. 6/1 of Village Hocher, P.S. Kanke, District Ranchi was recorded in the
record of rights as Bakast Bhuinhari land in the name of Chamtu Pahan and others as
landlords. In the record of rights in the remarks column, these lands were shown in
possession of Kolha Kumhar and others, the predecessors-in-interest of the private
respondents herein as Beyayani Bakbaje. The recorded bhumidar Chamtu Pahan and
others filed a title suit against Kolha Kumhar and others for relief of declaration of title
and recovery of possession. The said suit was decreed by the trial court and the appeal
preferred by the predecessors-in-interest of the respondents herein was dismissed. A
second appeal being Appeal from Appellate Decree No. 1909 of 1948 filed by the
defendants in the original suit was allowed by the High Court on 20-9-1951 and the
judgment and decree passed by the trial court and that of the first appellate court
@page-SC1140
was set aside. The Court came to the finding that the appellants-predecessors neither
redeemed mortgage nor came in possession of the land and that the suit for recovery of
possession was not maintainable. The Court recorded the finding that there was an oral
usufructuary mortgage as not yet been repaid and that mortgage, under Section 59 of the
Transfer of Property Act, is bad in law and as such the defendants' possession as
mortgagees must be ignored. After commencement of the Bihar Scheduled Areas
Regulation, 1969 (Regulation 1 of 1969), successive applications were filed under
Section 71A of the Chhotanagpur Tenancy Act, 1908 (for short "the CNT Act") by the
predecessors-in-interest of Chamtu Pahan bearing SAR Nos. 65/76, 82/77 and 543/ 83.
All these applications were ultimately rejected by the Special Officer, Scheduled Areas
Regulation, in terms of the orders dated 16-9-1976, 7-7-1977 and 31-12-1983
respectively holding that the predecessors-in-interest of the respondents had perfected
their title and the applications for restoration were barred by limitation. Despite rejection
of the suit and the applications moved under Section 71A of the CNT Act, a fresh
application was moved by the appellant claiming himself to be the heir of Chamtu Pahan
alleging therein that he by caste is Munda and is a member of the Scheduled Tribes and is
the priest (Pahan) of his village and the land in question measuring a total area of 6.38
acres is Bakast Bhuinhari Pahani land recorded in the name of his grandfather Chamtu
Munda/Pahan and others in the record-of-rights. It was alleged that the land in question is
community land, the usufruct of which is used for the community feast at the time of
Sarna Puja or Bhut Puja held by the community members on several occasions of the
agricultural year and the said land cannot be transferred to a person other than the
members of a Bhuinhari family as provided under Section 48 of the CNT Act. It was
further alleged that although such land is non-alienable, the ancestors of the respondents
by playing fraud on the grandfather of the appellant, namely, Chamtu Munda, took the
same on oral zerpesgi (mortgage) for Rupees 154/- for a period of 20 years as mentioned
in the record-of-rights in the year 1922 and, thus, the transfer being in contravention of
Section 46 of the CNT Act, possession of the land be restored. The application moved by
the appellant was allowed vide order dated 21-12-1987 by the Special Officer, Scheduled
Areas Regulation, who directed restoration of possession of the land in favour of the
appellant. The private respondents herein thereupon preferred an appeal before the
Additional Collector, Ranchi which was allowed by him. Considering the judgment and
order passed in the second appeal by the High Court as also the orders passed on
successive applications under Section 71A of the CNT Act, he came to the conclusion
that fresh application under Section 71A was not maintainable. Consequently, the order of
restoration of possession was set aside. The appellant preferred a revision before the
Divisional Commissioner under Section 217 of the CNT Act, which was allowed and
restoration of possession order was restored. That was challenged by the respondents by
filing a writ petition in the High Court. Learned Single Judge of the High Court while
allowing the writ petition held that the revisional authority committed an error in ignoring
the findings arrived at by the High Court in the second appeal and also the successive
orders passed by the Special Officer earlier rejecting the applications for restoration filed
by the predecessors-in-interest of the appellant. The Court also held that the
Commissioner totally ignored the effect of Section 27 of the Limitation Act and failed to
see that the application for restoration was barred by limitation as also by the principle of
res judicata. The order of the learned Single Judge was upheld by the Division Bench in
Letters Patent Appeal. That is how the matter has come before us.
2. It is contended by Mr. S.B. Upadhyay, learned senior counsel for the appellant that the
orders of the High Court are contrary to the provisions, intendment, letter and spirit of the
Bihar Scheduled Areas Regulation, 1969 (Regulation 1 of 1969) which is a welfare
legislation concerning the members of the Scheduled Tribes, which is mainly intended,
by insertion of Section 71A in the CNT Act, for restoration of their lands transferred in
favour of non-tribals fraudulently or in contravention of Sections 46 and 48 and other
provisions of the CNT Act. It is further urged by the learned senior counsel that there is
no limitation prescribed for resorting to the provision of Section 71A of the CNT Act; and
that the earlier decision of the High Court will not operate as res judicata. Whereas, it is
contended by Mr. Sunil
@page-SC1141
Kumar, learned senior counsel for the private respondents that when successive
applications under Section 71A of the CNT Act moved by the predecessors-in-interest of
the appellant have been rejected, the Special Officer committed an error in entertaining
the fresh application moved by the appellant. It is further urged that the earlier decision of
the High Court operates as res judicata and in any case the principle of constructive res
judicata would be applicable as all the questions available with the appellant to be
agitated before the court shall be deemed to have been adjudicated against him.
3. To better appreciate the arguments advanced by the counsel on both sides, it would be
pertinent to note the relevant provisions of the Chhotanagpur Tenancy Act, 1908 (CNT
Act). The relevant provisions of Section 46(1) of the CNT Act as it stood in 1908 Act and
substituted by Amendment Act of 1947 which came into force with effect from 5-1-1948
read as under :
Restrictions on transfer of their rights by raiyats :-(1) No transfer by a raiyat of his right
in his holding or any portion thereof -
(a) by mortgage or lease for any period expressed or implied which exceeds or might in
any possible event exceed five years, or
(b) by sale, gift or any other contract or agreement,
shall be valid to any extent :
XXX XXX XXX
4. In the CNT Act, Section 71A was inserted by the Bihar Scheduled Areas Regulation,
1969 (Regulation 1 of 1969). Later on, by the Bihar Scheduled Areas (Amendment)
Regulation, 1985 (Regulation 1 of 1985), after the word 'raiyat', the words 'or a Mundari
Khunt Kattidar or a Bhuinhar' were inserted. Section 71A, as amended by Bihar
Scheduled Areas (Amendment) Regulation, 1985, reads as under :
"Power to restore possession to member of the Scheduled Tribes over land unlawfully
transferred. - If at any time it comes to the notice of the Deputy Commissioner that
transfer of land belonging to a raiyat or a Mundari Khunt Kattidar or a Bhuinhar who is a
member of the Scheduled Tribes has taken place in contravention of Section 46 or any
other provision of this Act or by any fraudulent method, including decrees obtained in
suit by fraud and collusion he may, after giving reasonable opportunity to the transferee
who is proposed to be evicted, to show cause and after making necessary enquiry in the
matter, evict the transferee from such land without payment of compensation and restore
it to the transferor or his heir, or in case the transferor or his heir is not available or is not
willing to agree to such restoration, resettle it with another raiyat belonging to the
Scheduled Tribes according to the village custom for the disposal of an abandoned
holding :
xxx xxx xxx
5. As per Section 46 of the CNT Act, 1908. as it stood in 1922, no transfer by a raiyat of
his right in his holding or any portion thereof by mortgage or lease for any period
expressed or implied would be effected which exceeds or might in any possible event
exceed five years. It further restricted transfer by way of sale, gift or any other contract or
agreement and such transfer shall not be valid to any extent. The suit of the appellant's
predecessors for possession on the basis of oral mortgage was culminated into a decision
by the High Court in second appeal (AFAD No. 1909/1948) where a clear-cut finding
was recorded that there could not have been an oral usufructuary mortgage of immovable
property for value of more than Rs.100/- under Section 59 of the Transfer of Property
Act, the same being bad in law. Thus, the predecessors of the respondents could not be
treated to be in possession under the mortgage. Under the CNT Act as it stood in the year
1922, the transfer could have been challenged as it contravenes Section 46 of the CNT
Act, being a contract or agreement of transfer. That plea having not been taken by the
appellant's predecessors, the appellant and his predecessors were not entitled to raise the
question of transfer being invalid under Section 46 of the CNT Act as it stood in 1922 on
the principle of constructive res judicata. Section 46 of the CNT Act, by virtue of its
amendment with effect from 5-1-1948, restricts and prohibits transfer by a raiyat of his
right in his holding or any portion thereof by mortgage or lease for any period expressed
or implied, which exceeds or might in any possible event exceed five years. It
@page-SC1142
further restricts transfer by a raiyat of his right in his holding or any portion thereof, apart
from mortgage etc., by way of sale, gift or any other contract or agreement and if such
transfer is effected it shall be invalid. Section 71A of the CNT Act authorizes the Deputy
Commissioner to evict the transferee from such land and to restore possession to the
raiyat if the transfer is being effected in contravention of Section 46 or any other
provision of the CNT Act. Thus, if there is contravention of Section 46, the Deputy
Commissioner is authorized to evict the transferee from such land and restore it to the
transferor under Section 71A of the CNT Act. The predecessors of the respondents could
not be treated to be in possession in contravention of Section 46 as possession of land by
them has been upheld by the High Court in its decision. The decision of the High Court
cannot be reopened by taking advantage of amendment in Section 46 which came into
force with effect from 5-1-1948. Section 71A of the CNT Act would be attracted only in
case the Deputy Commissioner finds that the impugned transfer was made in
contravention of Section 46 or any other provision of the CNT Act. The decision of the
High Court comes in the way of the Deputy Commissioner in arriving at any such
findings. The possession having been denied to the appellant's predecessors holding that
there was no contravention of Section 46 as it stood in 1922, the appellant cannot be
permitted to take advantage under Section 46 on same having been amended by an Act of
1947. That apart, although there is no period of limitation prescribed for exercising the
power under Section 71A by the Deputy Commissioner, the party affected is called upon
to approach the appropriate authority or the power has to be exercised by the Deputy
Commissioner within a reasonable period of time. The gap of more than 50 years for
challenging the transaction of 1922 cannot be said to be a reasonable time for exercising
the power even if it is not hedged in by a period of limitation.
6. For the aforesaid reasons, the appeal is without substance and is dismissed.
Appeal dismissed.
AIR 2008 SUPREME COURT 1142 "Vinod v. State of Haryana"
(From : 2006 (2) All. Cri. L.R. 201 (Pun and Har))
Coram : 2 Dr. A. PASAYAT AND P. SATHASIVAM, JJ.
Criminal Appeal No. 165 of 2008 (arising out of SLP (Cri.) No. 2242 of 2006), D/- 24 -1
-2008.
Vinod v. State of Haryana.
Penal Code (45 of 1860), S.364A - KIDNAPPING - LIFE IMPRISONMENT -
Kidnapping for ransom - Kidnapping of minor boy - Demand at ransom and its
communication established - Evidence showing that seized currency notes from accused
were produced during trial - Offence u/S.364-A thus, proved - Sentence of life
imprisonment - Not interfered with - Considering alarming rise in kidnapping young
children for ransom, legislature has in its wisdom provided for stringent sentence. (Para
21)

Prem Malhotra, for Appellant; Rajeev Gaur 'Naseem', and T.V. George, for Respondent.
Judgement
1. Dr. ARIJIT PASAYAT, J. :-Leave granted.
2. Challenge in this appeal is to the judgment of a Division Bench of the Punjab and
Haryana High Court upholding conviction of the appellant for offence punishable under
Section 364A of the Indian Penal Code, 1860 (in short the 'IPC') as recorded by the
learned Additional Sessions Judge, Panipat.
3. By the impugned common judgment two Criminal Appeal Nos. 255 and 307 of 2001
were disposed of. Ten persons including one Virender who was declared a proclaimed
offender were sent for trial. After trial all the accused except Virender, whose presence
the investigating agency was not able to secure during trial were convicted under Section
364-A of IPC. They were convicted for offence punishable under Section 364-A IPC and
sentenced to imprisonment for life and a fine of Rs. 1,000/- with default stipulation.
4. Prosecution version as unfolded during trial is as follows :
Amit Kumar son of Madan Mohan (PW 2) was aged 9 years and on the date of the
incident i.e. 29-5-1996 was studying in class III. He used to live with his father in House
No. 212 Old Housing Board Colony, Panipat at a distance of about 100 yards from
@page-SC1143
Salarganj Gate, Panipat. At about noon time on 29-5-1996 the boy had gone to Salarganj
to play with his friends and when he did not return home, the family made all efforts to
trace him which proved to be futile. Apprehending that he had been kidnapped, Madan
Mohan (PW-1) proceeded to the Police Station City, Panipat to lodge a report. On the
way he met a police party at Sukhdev Nagar where he made his statement Ex. PA and on
its basis formal FIR (Ex. PA/2) was recorded.
On 2-6-1996, Sanjiv Jain and Faqir Chand (PW-4) came and apprised Madan Mohan that
Yashpal, a resident of Panipat, had come to them and told about their having received a
telephonic message from Saharanpur that Amit Kumar was well but his abductors were
demanding a ransom of Rs. 10,00,000/- failing which they were threatening to kill Amit
Kumar. The abductors had further informed Yashpal that in case the matter was reported
to the Police even then Amit Kumar would be killed. Yashpal had also been apprised of
the manner in which the money was required to be paid, which mode required Yashpal to
travel in Car No. HR-06B-244 belonging to Sanjiv Tayal, the younger brother of Madan
Mohan, display a white cloth for identification before reaching the Railway Crossing
before Rampur at 10.00 p.m. On reaching there, the car was to give a signal with the
dipper. On getting this information, Madan Mohan, Sanjiv Jain and Faqir Chand had
decided not to report the matter to the police and had arranged for the requisite amount
taking Rs. 3,50,000/- from M/s. Surya International (a factory owned by Madan Mohan),
Rs. 50,000/- from M/s. Design Rug owned by the younger brothers of Madan Mohan, Rs.
2 00,000/- from M/s. Surya International in the name of Faqir Chand, an amount of Rs.
2,75,000/- from Sintex Handicraft, Panipat in the name of wife of Madan Mohan as she
was partner of the firm.
On 3-6-1996 Sanjiv Jain and Faqir Chand had again informed Madan Mohan that Yashpal
had come and told them that he had received another message that in case the amount of
Rs. 10,00,000/- was not paid that day itself, Amit Kumar would be killed. Yashpal had
further assured them not to worry and had taken the entire responsibility for the safety of
the child. Thereupon the currency notes already collected had been arranged in the
denomination of Rs. 500/-, Rs. 100/- and Rs. 50/- respectively and the first and the last
notes of the bundles were initialled as "MM" by Madan Mohan. Sanjiv Jain had then
called Yashpal at the residence of Madan Mohan and handed over the bag containing
currency notes of Rs. 10,000,000/- to him. Yashpal had taken away the bag in the car
bearing registration No. HR-06B-244, which he had driven away himself.
On 4-6-1996, Yashpal brought back Amit Kumar and handed him over to Madan Mohan.
Amit Kumar told his father that on 30-5-1996 he was accosted by Virender the
absconding accused, who apprised him that his father was calling him. On hearing this,
Amit Kumar accompanied Virender for some distance where two young men, namely,
Vinod and Sohan were positioned near a Yamaha Motor Cycle. Vinod was standing near
the Motor Cycle while Sohan was sitting on the pillion. Sohan had caught hold of Amit
Kumar and closed his mouth and made him sit on the motor cycle whereafter the motor
cycle, was driven away by Vinod to the G.T. Road via bus stand from where it was taken
to Gharaunda. When the motor cycle reached the Yamuna bridge, Amit Kumar was given
water to drink and the accused threw a coin in the river. After this his abductors took him
to Railway Station Sona Arjunpur where it started raining. Thereupon they made him to
sit on the ticket window. When the rain stopped, Vinod and Sohan had removed him on
the motor cycle to a garden where 4/5 persons were taking liquor. One of them asked
Vinod whether the work had been done and another one of them had enquired as to where
Virender was, to which enquiry Vinod replied that Virender had been left at Panipat with
Yashpal. The group was addressing each other by their names as Sohan, Pawan, Pappu,
Jagbir, Sunder Pal and Vikas. They had served meals to Amit Kumar in the garden.
On the next day, Virender also came there and thereafter Virender and Vinod had taken
Amit Kumar on the Yamaha motor cycle to the house of Vidya Sagar Chawla at
Saharanpur. Vinod stayed with Amit Kumar whereas Virender used to go out at times.
Vidya Sagar Chawla also remained present in the house and the entire incident was
narrated by Amit Kumar to him that night. On the following day, after Virender had
returned, he and Vinod took Amit Kumar to a sugarcane field on a Yamaha motor cycle.
On the pavement of nearby canal, an Ambassador
@page-SC1144
car was standing and then Sohan, Pawan, Pappu, Jagbir, Sunder Pal and Vinod had taken
Amit Kumar for making a telephonic call to his father asking him to reach soon. On the
way, Vinod had told them that the uncle of Amit Kumar and Kakku had reached and,
therefore, they should escape. On hearing this, the appellants took Amit Kumar back to
the sugar cane field. During the night car belonging to the uncle of Amit Kumar came
back near the sugarcane field and Yashpal got down from the same. He called for the
aforementioned persons whereupon Amit Kumar was taken near the car. Vinod enquired
from Yashpal whether everything was alright at the house of Amit Kumar and Yashpal
replied in the affirmative. Vinod handed over Amit Kumar to Yashpal, who in turn handed
over a bag to Vinod. Yashpal further told that they shall count for the money after some
time. Yashpal thereafter took Amit Kumar to Saharanpur where meals were taken and
Yashpal had left him at the residence of Madan Mohan.
After his release, Amit Kumar had told his father that he could point out the places where
he had been taken. He had also made a similar statement before the police. On 8-6-1996,
Madan Mohan and his son Amit Kumar accompanied the police party. The boy had first
taken them to Salarganj gate from where he had been kidnapped and thereafter to Sona
Arjunpur Railway Station in Uttar Pradesh. From there, he had taken them to a garden
where he had been kept and from there to a sugarcane field which was at some distance
from the Railway Station. Thereafter, Amit Kumar had taken them to the house of Vidya
Sagar Chawla at Saharanpur. The Police had been carrying out raids to apprehend the
accused and during one such raid on 19-6-1996, in which Pawan Kumar (PW-3) had
joined, the police had gone to Sona Arjunpur where a person, whose name did not come
forth in the investigation, had disclosed that Sunder Pal and Pawan were sleeping under a
tree in the field. The police had then raided the field and apprehended both of them. On
interrogation, Sunder Pal made a disclosure statement (Ex. PF) that out of the ransom of
Rs. 50,000/-, he had spent Rs. 1,000/ and had kept concealed the remaining amount of
Rs. 49,000/- wrapped in a polythene paper underneath the ground in the fields. His
disclosure statement which runs into five pages interestingly incorporated the entire
details of the kidnapping including the portions wherein even he had not been associated
and bears his thumb impression and is attested by Pawan Kumar and Jai Narain. Pursuant
to this disclosure statement Sunder Pal had got recovered Rs. 49,000/- currency notes of
the denomination of Rs. 100/ which were taken into possession through recovery memo
Ex. PF/1. Pawan too made an equally detailed disclosure statement Ex. PG which was
reduced into writing and signed by him and attested by Pawan Kumar and Jai Narain in
the presence of S1 Krishan Pal and subsequent thereto, he too got recovered Rs. 45,000/-
of the denomination of Rs. 100/- each from the field indicated by him in the disclosure
statement. It was taken into possession through recovery memo Ex. PG/1.
On 20-6-1996, when Madan Mohan was standing at Mayur Chowk, a Sub Inspector, an
Assistant Sub Inspector and three constables met him and they together proceeded
towards the Railway Station, Panipat. When they were standing outside the cycle stand,
Railway Station, Madan Mohan noticed accused Yashpal coming towards the Railway
Station. Yashpal was apprehended by the police and on his personal search a country
made pistol and three live cartridges from the left pocket of his trouser were recovered,
which were taken into possession through recovery memo. On interrogation, in the
presence of Madan Mohan, Yashpal had made a disclosure statement Ex. PB in which
after giving the details of the persons involved in the kidnapping and the amount of
ransom taken, he disclosed that his share in the ransom came to Rs. 6,00,000/- out of
which Rs. 5,75,000/- have been kept in the bag in a Almirah at his residential house and
an amount of Rs. 25,000/- had been kept concealed in the house of his sister in Ludhiana
and a pistol had been kept concealed in a house of his sister in Saharanpur. Pursuant to
this disclosure statement, Yashpal got recovered Rs. 5,75,000/- and the black coloured
bag bearing the words "M.M. Tayal" embroidered thereon from his house, which were
taken into possession through recovery memo Ex. PC, which was attested by Madan
Mohan and Ramesh Chand.
On the same day, Inspector Krishan Pal (PW-8) joined Sanjay Tayal in the investigation
and after receipt of the secret information about Vinod, Vikas and Vidya Sagar, he went to
the Truck Union, Panipat and
@page-SC1145
apprehended all of them from a hut near a tube well. The inspector had recovered a sum
of Rs. 42,000/- from a bag which was being carried by Vikas. The notes were in the four
packets of Rs. 100/- denomination and four notes were of the denomination of Rs. 500/-
The first and the last notes bore the initials of "MM". These were taken into possession
through recovery memo Ex. PR, which was attested by the witnesses. A similar search of
Vinod led to the recovery of Rs. 41,000/-, which were carried by him in a bag. The notes
were in four packets of Rs. 100/- denomination and two packets of Rs. 500 denomination.
The first and the last currency notes of the packet of Rs. 500/-denomination bore the
initial MM, the signatures of Madan Mohan (PW-1). The notes were taken into
possession through recovery memo Ex. PS. A similar search conducted on the person of
Vidya Sagar led to the recovery of Rs. 24,000/-. All the notes were of the denomination
of Rs. 500/- and 12 notes were recovered from the right pocket of Vidya Sagar while 36
notes were recovered from the back pocket of the pant of Vidya Sagar. All the notes bore
the initials "MM", which were identified by Sanjay Tayal and taken into possession
through recovery memo Ex. PT. During the personal search of Vinod and Vikas one pistol
of 12 bore along with two live cartridges were recovered from Vinod whereas from Vikas
one pistol 315 bore along with three live cartridges were recovered.
On the same day, Inspector Ravinder Kumar (PW-9) along with SI Yad Ram and other
police officials had gone to Sona Arjunpur in search of the accused and there one Jaswant
Rai was joined in the investigation. There the police party got secret information that
Virender, Sohan and Vishav Pal were coming from Shamli to Panipat on a Yamaha motor
cycle whereupon he set up a naka. On their arrival, Sohan, Virender and Vishav Pal were
apprehended and a sum of Rs. 40,000/- were recovered from Vishav Pal, which were
carried by him in a bag of black colour, which he was holding in his hand. A similar sum
of Rs. 40,000/- was recovered from a bag which was being carried by Sohan. All the
notes were of the denomination of Rs. 100/- each and bore the initials "MM" of Madan
Mohan complainant, who has been described as Madan Gopalcomplainant. On the
personal search of Virender, 86 currency notes of the denomination of Rs. 500/- each i.e.
Rs. 43,000/-were recovered. All the aforesaid currency notes were recovered through
recovery memos Ex. PJ, PK and PL respectively. The motor cycle was also taken into
possession through recovery memo Ex. PM.
On 4-8-1996, Inspector Ravinder Kumar (PW-9) along with SI Randhir Singh and other
police officials and the complainant went to Village Sona Arjunpur in search of Jagbir
appellant where he came to know that he had gone to Panipat in order to surrender in the
Court. When the police party was present near the bridge of Yamuna, the Inspector
received secret information that Jagbir had gone to Panipat. When the police party
reached Sanjay Chowk, Panipat, the complainant pointed out towards Jagbir who was
standing near a three wheeler. He was apprehended. On 7-8-1996, on interrogation Jagbir
made a disclosure statement Ex. PD to the effect that he had kept concealed an amount of
Rs. 5,000/- in a wax paper in the Baithak of his house situated in Village Sona Arjunpur.
Thereafter, in pursuance of his disclosure statement, he got recovered a sum of Rs.
5,000/- of the denomination of Rs. 100/- each. The same were taken into possession
through recovery memo Ex. PE. One of the notes was bearing the initial of "MM".
5. On completion of the investigation, a challan was put in the Court of the Illaqa
Magistrate, who committed the case to the Court of Session as the offences disclosed
therein were exclusively triable by that Court.
6. On going through the challan papers, Learned Additional Sessions Judge framed
charge under Section 364-AIPC against all the appellants to which they pleaded not
guilty.
7. In order to bring home charge against the appellants, the prosecution examined Madan
Mohan (PW-1), Amit Kumar (PW2), Pawan Kumar (PW3), Faqir Chand (PW4), SI Yad
Ram (PW5), ASI Dalel Singh (PW6), Inspector Rajinder Singh (PW7), SI Krishan Pal
(PW8) and Inspector Ravinder Kumar (PW9).
8. When examination under Section 313 of the Code of Criminal Procedure, 1973 (in
short Cr.P.C.) in order to explain the incriminating circumstances appearing in evidence
against them, Vikas, Vishav Pal, Pawan
@page-SC1146
Kumar, Vidya Sagar, Sunder Pal, Vinod, Sohan and Jagbir pleaded innocence and false
implication.
9. Placing reliance on the evidence of victim Amit Kumar (PW-2), Madan Mohan (PW-1)
and Pawan Kumar (PW-3) as noted above the accused persons were found guilty and
sentenced.
10. Before the High Court stand of the appellant was that offence under Section 364-A
IPC has not been made out and in any event the seized money have not been produced
during trial which prompted the High Court to take serious note of the lapse. But the
High Court upheld the conviction and the sentence of the appellant.
11. It was pleaded that the prosecution version should not have been accepted. In any
event, according to learned counsel for the appellant Section 364-A has no application.
12. In response, learned counsel for the respondent-State submitted that the High Court
erroneously observed that the seized notes were not produced during trial. In any event
the High Court was right in dismissing the appeal. It would be appropriate to deal with
the plea that seized currency notes were not produced. Following observations of the trial
Court are relevant :
".....Similarly accused Pawan Kumar suffered a disclosure statement Ex. PG and got
recovered an amount of Rs. 45,000/- which was taken into possession vide memo Ex.
PG/1. Ex. P4 is the currency notes. He also prepared the rough site-plans Ex. PP and Ex.
PQ regarding the aforesaid recoveries. He further stated that on 20-6-1996 he joined
Sanjay Tayal in the investigation of this case and after receipt of a secret information, he
rushed to Truck Union Panipat and there he apprehended accused Vinod, Vikas and Vidya
Sagar. He conducted the personal search of the aforesaid accused and recovered an
amount of Rs. 42,000/- from accused Vikas. The currency notes were in four packets of
Rs. 100/- denomination and four notes were of the denomination of Rs. 500/-. The first
and the last note of each packet were bearing the initial of "M.M." which also identified
Sanjay Tayal of his brother Madan Mohan. He took the same into possession and the bag
is Ex. P7 and currency notes are Ex. P4. The recovery memo is Ex. PR. He also
conducted the personal search of Vinod and recovered an amount of Rs. 41,000/- which
were in a bag which is Ex. P8 which accused was carrying. The aforesaid currency notes
were in four packets having a denomination of Rs. 100/- and two notes were of the
denomination of Rs. 500/ The first and the last note of every bundle was having the initial
of, M.M. and PW Sanjay Tayal identified the same. The currency notes were Ex. P4. He
took into possession the bag Ex. PS which bears his signature as well as signature of
Sanjay Tayal. He also conducted the personal search of Vidya Sagar accused and
recovered a sum of Rs. 24,000/-. All the currency notes were of the denomination of Rs.
500/-. The 12 notes were recovered from the right pocket of the accused Vidya Sagar
whereas 36 currency notes were recovered from back side pocket of the pant of the
accused. All the currency notes were bearing the initial of Madan Mohan which were
identified by Sanjay Tayal."
13. It is to be noted that before the High Court challenge was not raised to shake the
credibility of the testimony of Madan Mohan (PW-1) and Amit Kumar (PW-2) during
arguments.
14. Section 364-A deals with 'Kidnapping for ransom etc.' This Section reads as follows :
"Whoever kidnaps or abducts any person or keeps a person in detention after such
kidnapping or abduction and threatens to cause death or hurt to such person, or by his
conduct gives rise to a reasonable apprehension that such person may be put to death or
hurt, or causes hurt or death to such person in order to compel the Government or (any
foreign State or international inter governmental organization or any other person) to do
or abstain from doing any act or to pay a ransom, shall be punishable with death, or
imprisonment for life, and shall also be liable to fine."
15. The Section refers to both "Kidnapping" and "Abduction". Section 359 defines
Kidnapping. As per the said provision there are two types of kidnapping i.e. (1)
kidnapping from India; and (2) kidnapping from lawful guardianship.
16. Abduction is defined in Section 362. The provision envisages two types of abduction
i.e. (1) by force or by compulsion; and/ or (2) inducement by deceitful means. The
@page-SC1147
object of such compulsion or inducement must be the going of the victim from any place.
The case at hand falls in the second category.
17. To "Induce" means "to lead into". Deceit according to its plain dictionary meaning
signifies anything intended to mislead another. It is a matter of intention and even if
promise held out by the accused was fulfilled by him, the question is : whether he was
acting in a bona fide manner?
18. The offence of abduction is a continuing offence. This Section was amended in 1992
by Act XLII of 1993 with effect from 22-5-1993 and it was subsequently amended in
1995 by Act XXIV of 1995 with effect from 26-5-1995. The Section provides punishment
for kidnapping, abduction or detaining for ransom.
19. To attract the provisions of Section 364-A what is required to be proved is (1) that the
accused kidnapped or abducted the person; and (2) kept him under detention after such
kidnapping and abduction; and (3) that the kidnapping or abduction was for ransom.
20. To pay a ransom as per Black's Law Dictionary means "to pay price or demand for
ransom". The word "demand" means "to claim as ones due"; "to require"; "to ask relief;
"to summon"; "to call in Court"; "An imperative request preferred by one person to
another requiring the latter to do or yield something or to abstain from some act;" "An
asking with authority, claiming." The definition as pointed out above would show that the
demand has to be communicated. It is an imperative request or a claim made.
21. When the evidence on record is analysed in the background of Section 364-A IPC, the
inevitable conclusion is that the prosecution has clearly established commission of the
said offence. Considering the alarming rise in kidnapping young children for ransom, the
legislature has in its wisdom provided for stringent sentence. Therefore, the High Court
rightly refused to interfere in the matter. In our view, the impugned judgment of the High
Court does not suffer from any infirmity to warrant interference. The appeal fails and is
dismissed.
Appeal dismissed.
AIR 2008 SUPREME COURT 1147 "Usha Devi v. Rijwan Ahmad"
(From : Jharkhand)*
Coram : 2 G. P. MATHUR AND AFTAB ALAM, JJ.
Civil Appeal No. 481 of 2008 (arising out of SLP (C) No. 20203 of 2006), D/- 17 -1
-2008.
Usha Devi v. Rijwan Ahmad and Ors.
(A) Civil P.C. (5 of 1908), O.6, R.17 - AMENDMENT - PLAINT - SUPREME COURT -
Amendment of plaint - Correction of description of suit property - Plaintiff not diligent -
Did not seek amendment at early stage though wrong description was pointedly brought
up by defendants not only in written statement but also in course of proceedings -
Proposed amendment necessary for bringing to fore real controversy between parties -
Supreme Court allowed prayer for amendment in view of decision in 2005 (13) SCC 89 -
Proposed amendment allowed subject to cost of Rs. 10,000/- as defendants made to suffer
injunction for a long time with regard to their own property because of said wrong
description.
W.P. (C) No. 2325 of 2006, D/-13-07-2006 (Jhar), Reversed. (Paras 10, 12)
(B) Civil P.C. (5 of 1908), O.6, R.17 - AMENDMENT - PLAINT - Amendment of plaint
- Grant of permission - Merit of amendment is hardly a relevant consideration - It is open
to defendant to raise their objection in regard to amended plaint by making any
corresponding amendments in their written statement. (Para 11)
Cases Referred : Chronological Paras
2007 AIR SCW 513 : AIR 2007 SC 806 : 2007 (2) AIR Kar R 17 (Ref.) 8
2006 AIR SCW 3956 : AIR 2006 SC 2832 : 2006 (5) AIR Kar R 457 (Ref.) 9
2005 AIR SCW 2346 : AIR 2005 SC 2441 8 (2005) 13 SCC 89 (Foll.) 9, 10
Devashish Bharuka, Mrs. Hansa Bharuka, Abhiseke Mohan Sinha, Dr. Sushil Balwada,
for Appellant; S.R. Sharma, S. Balaji, for Respondents.
* W. P. (C) No. 2325 of 2006, D/- 13-7-2006 (Jhar)
Judgement
AFTAB ALAM, J. :-Leave granted.
2. This appeal is directed against the order dated July 13, 2006, passed by the High Court
in W.P.(C) No.2325 of 2006. It is a brief and non-speaking order by which the
@page-SC1148
High Court dismissed the writ petition and affirmed the order passed by the trial court
which, in turn, had rejected the appellant's petition under Order 6, Rule 17 of the Code of
Civil Procedure ('CPC' for short) for amendment of the suit property as described in the
Schedule to the plaint.
3. The material facts are brief and simple. In the year 2002, the appellant filed a suit, inter
alia, seeking permanent injunction restraining the respondents-defendants from
interfering with her rights over the suit property and further directing them not to build or
demolish the building already existing on the suit land. In the Schedule to the plaint, the
description of the suit premises was given as follows :
"Southern half portion of measuring an area of 1937.97 sq.feet = 0.04.448 acres or
0.04.9/20 acres bearing at present holding Nos. 304, before that 275 and presently 201,
Ward No.IV(Old) New 13, of Giridih Municipality having double storied house together
the land over which it stands bounded as follows :-

xxx xxx xxx xxx


xxx xxx xxx xxx
xxx xxx xxx xxx

The defendant-respondents filed their written statement in which objection was especially
taken to the description of the suit property as given in the plaint. On behalf of the
respondents it was stated that the area of land that might possibly be the subject-matter of
any dispute was much smaller and the plaintiff had described properties lawfully
belonging to them as the suit property. No rejoinder to the written statement was filed on
behalf of the plaintiff and on the basis of the pleadings issues were framed on August 13,
2002. Thereafter, the proceedings in the suit remained in abeyance but on August 5, 2002,
the appellant-plaintiff filed a Misc. Petition under Order 39, Rule 2(A) read with Section
151 C.P.C. (registered as Misc.Case No.28/2002) for alleged breach of an interim
injunction earlier granted in her favour. In that proceeding, the husband of the plaintiff
was examined as one of the witnesses. In course of his cross-examination, it was
repeatedly put to him that he did not have any idea of the suit land and that he would not
claim all the area mentioned in the Schedule to the plaint but the plaintiff's claim would
be only over one decimal of land. It was also suggested to him that the rest of the land
admittedly belonged to the defendants and further that any alleged dispute between the
parties could only be over a very limited area and not the entire property as stated in the
Schedule to the plaint. The witness (the appellant's husband), however, denied the
suggestions made on behalf of the defendants and stuck to the stand that the disputed
property was correctly described in the plaint and that was the subject-matter of the suit.
Later, on September 29, 2004, the amendment petition was filed that gives rise to the
present appeal. In the amendment petition it was stated that due to inadvertence the suit
land was wrongly described in the Schedule to the plaint and the mistake required to be
corrected. It was further stated that, as a matter of fact, one decimal equivalent to 9
chhatak by standard measurement, i.e., 414 square feet of land (along with some
structure) was the subject matter of the suit. Accordingly, it was prayed that from the
description of the suit property in the plaint the opening words "southern half portion of
measuring an area of 1937.97 square feet = 0.04.448 acre or 0.04.9/20 acres" be deleted
and substituted by the following :
"1 decimal (one decimal) equivalent to about 9 chhatak (Nine chhatak) by standard
measurement that is 414 square feet land along with old double storied house consisting
of four rooms, two rooms in ground floor and two rooms in first floor and one verandah
towards west that is in road side covered with cogurated sheet, a stair case for going to
upper floor rooms." bearing at present holding number No. 304.........
4. The trial court rejected the petition by order dated February 2, 2006, observing as
follows :
"As such it cannot be said that plaintiff in spite of due diligence could not have raised this
discrepancy in the plaint prior to 29-09-04 i.e. after nearly 2 years of the settlement of the
issues and after witnesses have been examined on oath in the Misc. Case 28/2002 arising
out of T.S.58/ 2002.
"Hence it is clear that the plaintiff in spite of ample opportunity to have corrected the
discrepancy in the Schedule of the plaint did not care to remove the same instead kept of
(sic) insisting and asserting the correctness of the land and boundary mentioned in the
Schedule."
@page-SC1149
The order of the trial court was challenged before the High Court in a writ petition which
was dismissed with the observation that there was no illegality in the impugned order.
5. Amendment of pleadings used to be one of the easiest things in the course of judicial
proceedings before the amendments came to be made in the C.P.C. in the year 1999. It
was felt that the provision for amendment of pleadings (Order 6, Rule 17) was greatly
abused and it was one of the significant sources of delay in the judicial process.
Accordingly, as per the recommendation of the Law Commission, the provision for
amendment of pleadings was altogether deleted by Act 46/1999. The deletion of the
provision led to widespread protests by lawyers and different legal bodies and as a result
the provision was once again introduced, albeit with a rider, by Act 22/ 2002, with effect
from July 1, 2002. In its amended form, Rule 17, Order VI carries a proviso that bars any
amendment after the commencement of trial unless the court came to the conclusion that
in spite of due diligence the party could not have raised the matter before the
commencement of trial.
6. As noted above, the trial court found and held that there was singular lack of due
diligence on the part of the appellant-plaintiff inasmuch as the wrong description of the
suit property was pointedly brought up by the defendants not only in the written
statement but also in course of the proceedings of the Misc.Case.
7. Mr. Devashish Bharuka, learned counsel appearing on behalf of the appellant,
submitted that the proviso to rule would come into play only after the commencement of
trial and in this case the trial court was in error in rejecting the appellant's prayer invoking
the due diligence clause in the proviso. Learned counsel further submitted that neither the
framing of issues nor the proceedings of Misc. case could be taken as commencement of
trial. The prayer for amendment was made at the pre trial stage and hence, the prayer
should have been allowed without difficulty as was the position under the unamended
Rule 17.
8

. Mr. S.R. Sharma, learned counsel appearing for the respondents-defendants, on the other
hand, submitted that the plaintiff-appellant had obtained interim injunction against the
defendants in regard to the property as described in the plaint and now the proposed
amendment made it manifest that the defendants were made to suffer injunction for a
long time with regard to their own property. The prayer for amendment, according to him,
was fit to be rejected on that ground alone and allowing the prayer would be quite
unreasonable, unjust and unfair. He further submitted that on the plaintiffs own showing
the suit in its present form was bound to fail and the permission to amend the plaint
would, therefore, amount to giving an undue advantage to the plaintiff. He further
submitted that the proposed amendment would not only change the suit property but
would also change the cause of action and would thus render the suit not maintainable in
any event. He lastly submitted that the prayer for amendment was made after the
commencement of the trial and the trial court had, therefore, rightly rejected the prayer.
He maintained that the trial of the suit would commence with the settlement of the issues.
In support of the submission that the framing of the issues marked the commencement of
trial of the suit, Mr. Sharma, relied upon the decision of this Court in Ajendraprasadji N.
Pandey and Ann v. Swami Keshavprakeshdasji N. and Ors. [2006 (12) SCC 1]. In
paragraph 57 of the decision, it was observed as follows : 2007 AIR SCW 513, Para 54

"It is submitted that the date of settlement of issues is the date of commencement of trial.
(Kailash v. Nanhku [(2005) 4 SCC 480] Either treating the date of settlement of issues as
date of commencement of trial or treating the filing of affidavit which is treated as
examination-in-chief as date of commencement of trial, the matter will fall under proviso
to Order 6 Rule 17 CPC. The defendant has, therefore, to prove that in spite of due
diligence, he could not have raised the matter before the commencement of trial. We have
already referred to the dates and events very elaborately mentioned in the counter-
affidavit which proves lack of due diligence on the part of the defendants 1 and 2 (the
appellants)." 2005 AIR SCW 2346

From the above-quoted passage, it appears that the decision did not hold that settlement
of issues marks the commencement of trial. Earlier in the decision, the court exhaustively
examined the proceedings from date to date and on that basis came to hold and find that
the prayer for amendment was made after the commencement of trial.
@page-SC1150
9

. Mr. Bharukha, on the other hand, invited our attention to another decision of this Court
in Baldev Singh and Ors. v. Manohar Singh and Anr. [2006 (6) SCC 498J. In paragraph
17 of the decision, it was held and observed as follows : 2006 AIR SCW 3956, Para
17

"Before we part with this order, we may also notice that proviso to Order 6 Rule 17 CPC
provides that amendment of pleadings shall not be allowed when the trial of the suit has
already commenced. For this reason, we have examined the records and find that, in fact,
the trial has not yet commenced. It appears from the records that the parties have yet to
file their documentary evidence in the suit. From the record, it also appears that the suit
was not on the verge of conclusion as found by the High Court and the trial court. That
apart, commencement of trial as used in proviso to Order 6 Rule 17 in the Code of Civil
Procedure must be understood in the limited sense as meaning the final hearing of the
suit, examination of witnesses, filing of documents and addressing of arguments. As
noted hereinbefore, parties are yet to file their documents, we do not find any reason to
reject the application for amendment of the written statement in view of proviso to Order
6 Rule 17 CPC which confers wide power and unfettered discretion to the court to allow
an amendment of the written statement at any stage of the proceedings."
Mr. Bharukha also invited our attention to a three-Judge Bench decision of this Court in
Sajjan Kumar v. Ram Kishan [2005 (13) SCC 89). In this decision too the proposed
amendment related to correction of the description of the suit premises in the plaint. The
amendment was sought on the plea that the description of the property given in the rent
note itself was incorrect and the same description was repeated in the plaint and there
would be complications at the stage of execution to avoid which the description of the
suit premises as given in the plaint needed to be corrected. Another similarity with the
case in hand was that the prayer for amendment was opposed by the defendant-
respondent on the principal ground that although the defendant had taken the plea in the
written statement itself that the suit premises were not correctly described, yet the
plaintiff-appellant proceeded with the trial of the suit and did not take care to seek the
amendment at an early stage. The trial court rejected the prayer for amendment and the
High Court dismissed the civil revision against the order of the trial court. Allowing the
prayer for amendment this Court in paragraph 5 of the decision observed as follows :
"Having heard the learned counsel for the parties, we are satisfied that the appeal
deserves to be allowed as the trial court, while rejecting the prayer for amendment has
failed to exercise the jurisdiction vested in it by law and by the failure to so exercise it,
has occasioned a possible failure of justice. Such an error committed by the trial court
was liable to be corrected by the High Court in exercise of its supervisory jurisdiction,
even if Section 115 CPC would not have been strictly applicable. It is true that the
plaintiff-appellant ought to have been diligent in promptly seeking the amendment in the
plaint at an early stage of the suit, more so when the error on the part of the plaintiff was
pointed out by the defendant in the written statement itself. Still, we are of the opinion
that the proposed amendment was necessary for the purpose of bringing to the fore the
real question in controversy between the parties and the refusal to permit the amendment
would create needless complications at the stage of the execution in the event of the
plaintiff-appellant succeeding in the suit."
10. In view of the decision in Sajjan Kumar, we are of the view that this appeal too
deserves to be allowed. We may clarify here that in this order we do not venture to make
any pronouncement on the larger issue as to the stage that would mark the
commencement of trial of a suit but we simply find that the appeal in hand is closer on
facts to the decision in Sajjan Kumar and following that decision the prayer for
amendment in the present appeal should also be allowed.
11. As to the submission made on behalf of the respondents that the amendment will
render the suit non-maintainable because it would not only materially change the suit
property but also change the cause of action it has only to be pointed out that in order to
allow the prayer for amendment the merit of the amendment is hardly a relevant
consideration and it will be open to the defendants-respondents to raise their objection in
regard to the amended plaint by making any corresponding amendments in their written
statement.
12. The counsel for the respondents also submitted that as a result of the description
@page-SC1151
the suit property in the plaint the defendants-respondents had to suffer injunction against
their own property. We feel that the ends of justice would meet by allowing the proposed
amendment subject to a cost of Rs. 10,000/ .
13. This appeal is accordingly allowed. The orders of the trial court and High Court are
set aside and it is directed that the appellant may be allowed to make the proposed
amendment in the plaint subject to payment of Rs. 10,000/- as cost to the respondents-
defendants. The amendment will be allowed in case the amount of cost is paid within two
months from today.
Appeal allowed.
AIR 2008 SUPREME COURT 1151 "Tamil Nadu Housing Board v. Sea Shore
Apartments Owners' Welfare Association"
Coram : 2 C. K. THAKKER AND P. SATHASIVAM, JJ.
Civil Appeal Nos. 7907-7913 of 2003, D/- 9 -1 -2008.
Tamil Nadu Housing Board and Ors. v. Sea Shore Apartments Owners' Welfare
Association.
(A) Consumer Protection Act (68 of 1986), S.2(1)(g), S.21 - CONSUMER
PROTECTION - HOUSING BOARD - Deficiency in service - Housing construction -
Demand of additional price by Housing Board - Price quoted in initial advertisement
issued for registration of intending purchasers was tentative price - On alteration of
Scheme, plinth area and ground area was increased - Allottees were again informed that
price fixed were tentative and they would have to pay price finally determined by Board -
Demand of additional price considering alterations in Scheme and increased land
acquisition compensation - Not arbitrary - Not deficiency in service. (Paras 27, 30)
(B) Consumer Protection Act (68 of 1986), S.11, S.17, S.21 - CONSUMER
PROTECTION - Consumer Forum - Jurisdiction - Price fixation - Issue depends upon
various factors - Normally Consumer Forum should not decide such question. (Para
20)
(C) Evidence Act (1 of 1872), S.115 - ESTOPPEL - ALLOTMENT OF PREMISES -
HOUSING BOARD - Estoppel - Allottees of Flats in Housing Scheme - Demand of
additional price by Housing Board - Allottees paying additional price and taking
possession of Flats - Cannot go behind concluded contract and challenge demand of
additional price. (Para 27)
Cases Referred : Chronological Paras
2006 AIR SCW 1943 : AIR 2006 SC 1758 : 2006 (3) ALJ 494 (Ref.) 26
(1996) 1 CPJ 103 (NCC) 18
1994 AIR SCW 97 : AIR 1994 SC 787 (Ref.) 19
(1993) 3 CPJ 351 (NCC) 18
AIR 1989 SC 1076 (Rel. on, Pnt B) 24, 25
AIR 1980 SC 738 (Rel. on, Pnt B) 21
AIR 1979 SC 1628 (Ref.) 24, 25
V. Krishnamurthy, Sr. Advocate, H. Harish Kumar, Dr. R. Prakash and P.N. Ramalingam
with him. for Appellants; M.N. Rao, Sr. Advocate, K.K. Mani, C.K.R. Linin Shekar and
Mayur R. Shah with him, for Respondent.
Judgement
1. C. K. THAKKER, J. :-The present appeals are filed against an order passed by the
State Consumer Disputes Redressal Commission, Madras ('State Commission' for short)
on July 24, 1995 in Original Petition Nos. 143-149 of 1995 and confirmed by the
National Consumer Disputes Redressal Commission, New Delhi ('National Commission'
for short) on February 25, 2002 in First Appeal Nos. 500-506 of 1995.
2. Shortly stated the facts are that the Tamil Nadu Housing Board (hereinafter referred to
as 'the Board') was constituted under the Tamil Nadu Housing Board Act, 1961 (Act 17 of
1961). The primary object of creation of the Housing Board was to acquire land in the
neighbourhood areas of developed cities at a reasonable price and to construct tenements,
houses and flats thereon for providing residential accommodation to needy people of
different income groups and categories. In the year 1982, vast piece of land admeasuring
abour 28 acres of Thiruvanmiyur, Chennai was acquired by the State of Tamil Nadu
under the Land Acquisition Act, 1894 for a public purpose, viz. for the development of
the area known as South Madras Neighbourhood Scheme. On February 27, 1991 the
Board approved a proposal to construct seven different types of flats. It proposed to
construct 102 flats under its High Income Group Scheme ('HIG Scheme' for short). In
order to assess demand from public, an advertisement was issued by the Board on March
21, 1991 inviting applications for registration under the title "Avail a chance of owning
your own flat" in Thiruvanmiyur Extension, Madras. Seven
@page-SC1152
types of flats were mentioned in the said advertisement along with plinth area, tentative
price, initial deposit, monthly instalment, repayment period, amount of deposit for
registration, etc. It was stated that pursuant to the said advertisement applications were
made by interested persons. There was overwhelming demand and several persons
applied. The record reflects that finally instead of seven types of flats, fifteen types of
flats were constructed under HIG Scheme. The Board issued letters on August 13, 1993
to the applicants asking them whether they were willing to purchase flats. Necessary
details of the type, design, plinth area, tentative selling price and other particulars were
supplied. Draw was conducted on October 15, 1993 and provisional allotment letters
were issued on October 19, 1993. Tentative cost was specified in the letter which was to
be paid within a period of 21 days. Final allotment order was made on August 9, 1994
wherein final cost of the flat was mentioned. An agreement was entered into between the
Housing Board and allottees on August 22, 1994. In the said agreement, it was mentioned
that it was agreed between the parties that the ultimate cost of the total construction of the
flat was subject to the outcome in the award of compensation in land acquisition
proceedings pending adjudication and the final amount will be fixed on that basis which
will be paid by the members. Thereafter possession of flats was given to all allottees. The
members were then asked to pay additional amount. The respondent-Sea Shore
Apartments Owners Welfare Association ('Association' for short) felt that the demand
made and amount recovered by the Housing Board was neither legal nor proper. It could
not have demanded more amount. The amount which was fixed earlier was already paid
and the members of the Association were not treated fairly. It, therefore, made
representation on December 26, 1994 against the additional amount. In the said
representation, the Association asked the Board to give reasons for enhancement of price
of flats as also for reduction of period of payment of instalments from 15 years to 13
years. The Board, however, did not reply to the said letter. Even subsequent letter was not
responded. Seven complaints were, therefore, filed by the allottees before the State
Commission on May 26, 1995 under Section 12 of the Consumer Protection Act, 1986
(hereinafter referred to as 'the Act'). Prayers were made in the complaints to direct the
Board and its officers to return the escalation amount paid by the members of the
Association with interest thereon; to restrain the Board and its officers from insisting on
payment of excess amount as demanded; to direct the Board to collect the instalments in
15 years as per the order of allotment issued earlier; to pay compensation of rupees one
lakh for the loss sustained and mental agony suffered by the members of the Association
and to pay costs of the complaints. It was also stated that the complainants had claimed
relief for those members also whose names had been given in the Annexure to the
complaints.
3. A reply was filed by the Board controverting averments made and allegations levelled
in the complaints. It was stated that under the Demand Assessment Scheme, the price
mentioned in the advertisement was only 'tentative'. Originally, the proposal was for
construction of seven types of flats but because of great demand, it was finalized into
fifteen types of flats. It was also stated that the construction cost was increased because of
increase in ground area, plinth area and also because of payment of excess compensation
to the land owners whose lands had been acquired for the purpose of construction of flats.
It was contended that if the allottees were really aggrieved over the increase in cost, they
could have well surrendered the flats. But they did not do so. They accepted the increase
in price and took over possession of property. It was also contended that the Consumer
Forum had no jurisdiction to deal with and decide the matters relating to fixation of price
of flats and on that ground also, the complaints were not maintainable. It was submitted
that the demand of price could not be said to be illegal, fanciful or otherwise
unreasonable and the complaints were liable to be dismissed.
4. The State Commission considered the rival contentions of the parties and held that
there had been 'deficiency in service' on the part of Board inasmuch as there was illegal
demand by the Board of additional amount which was neither legal nor proper. The
Commission observed that when the possession was sought to be given to the allottees,
they had no option, but to take possession of the flats and that is how possession was
taken over by the members and the said circumstance could not go against them.
According to the State Commission, the complaint of the complainant-Association
@page-SC1153
that escalation was unjust, unwarranted and illegal was well founded and ought to be
upheld. According to the State Commission, "threefold defence" put forward by the
Board had no basis whatsoever. In the opinion of the State Commission, the defences as
to (i) increase in the plinth area, (ii) increase in the area of land, and (iii) payment of
excess amount of compensation to the land owners were vague and no particulars were
furnished. No details were supplied as to excess payment of compensation. It was also
not clear whether the entire excess amount of compensation paid to the land owners was
in respect of land on which flats were constructed by the Board and allotted to the
members of the Association. It was not open to the Board, commented the State
Commission, to demand from members of the Association, the entire amount which it
had paid to the land owners towards enhanced compensation. The State Commission also
held that the Board had no right to reduce the period of recovery of amount by
instalments from 15 years to 13 years and the said action was illegal. Accordingly, all the
complaints were allowed and the demand made by the Board was quashed and set aside.
Refund of amount was also ordered.
5. Being aggrieved by the order passed by the State Commission, the Board approached
the National Commission. The National Commission by a short order dated February 25,
2002 dismissed all the appeals observing inter alia that the State Commission recorded
that "not a scrap of paper has been filed by the opposite party to show that there was any
land acquisition proceedings before any Court in respect of the lands in question".
According to the National Commission, the action of the Board in increasing price was
on non-existing grounds and hence the demand was not legal. The appeals were
accordingly dismissed.
6. The Board has challenged these decisions by filing present appeals. On November 25,
2002, notice was issued. On September 15, 2003, leave was granted after hearing the
parties. Operation of the impugned order was also stayed subject to the appellants
depositing the disputed amount in the Court within a period of four weeks from the date
of the order. The Registry was directed to invest the said amount. The matters were
thereafter ordered to be posted for hearing. That is how the matters are before us.
7. We have heard the learned counsel for the parties.
8. The learned counsel for the Board strenuously urged that the Commissions were
clearly in error in invoking the provisions of the Act and in observing that there was
'deficiency in service'. According to the learned counsel, dispute in the instant case
related to fixation and determination of price of flats. Such dispute cannot be resolved
under the Act. Consumer Commission has no power, authority or jurisdiction to inquire
into, deal with and decide such questions. Even otherwise, in view of allegations and
counter-allegations and assertions and retractions, only civil Court can enter into disputed
questions of fact on the basis of evidence adduced by the parties and Commissions
exercising summary power were in error in encroaching the jurisdiction of civil Court
which could not have been done.
9. It was also submitted that from the facts it was clearly established that in 1991 what
was done by the Board was to formulate a scheme and tentative price was fixed. In view
of overwhelmed response, the scheme was changed from seven types to fifteen types
flats. There was increase in plinth area, in ground area as also payment of excess
compensation to land owners. It was, therefore, clearly stated in 1993 to all the applicants
whose names had been registered in 1991 about the revised price, the period within
which the amount was to be paid and the reasons for fixation of higher price. It was also
stated that at the time of registration in 1991, it was clearly indicated that for those who
opted to make payment in instalments, the period of repayment was 13 years. In 1993,
however, when applications for allotment were called for, the period was indicated as 15
years. The said mistake was rectified at the time of final allotment. With an open eye, it
was accepted by the allottees and agreements were signed by them giving undertakings. It
was thereafter not open to the allottees to challenge fixation of price of flats by the Board.
They were estopped from doing so under the doctrine of promissory estoppel. It was also
submitted that when complaints were filed before the State Forum, a counter-affidavit
was filed on behalf of the Board wherein it was asserted that there were three-fold
reasons for increase of price; viz., (i) increase in plinth area, (ii) increase in ground area,
@page-SC1154
and (iii) payment of enhanced compensation to land owners. In view of the above pleas
and defences, the State Commission ought to have dismissed the complaints. The State
Commission, however, failed to do so. But even otherwise, the State Commission did not
consider all the defences in their proper perspective and held that the Board was not
entitled to claim additional amount and issued certain directions including refund of
amount with interest. Obviously, the Board was aggrieved and it approached the National
Commission. But the National Commission also, without considering the points raised by
the Board confirmed the order passed by the State Commission and dismissed the
appeals. Both the orders, therefore, are not in consonance with law and are liable to be set
aside.
10. The learned counsel for the complainants supported the order passed by the State
Commission and confirmed by the National Commission. He submitted that the State
Commission has considered all the contentions raised by the Board and after perusing the
materials placed before it, recorded a finding that none of the three defences raised by the
Board was well founded and hence could not be upheld. It was a pure finding of fact
based on evidence. The National Commission affirmed the order passed by the State
Commission observing that the findings recorded by the State Commission were findings
of fact and they did not call for interference. Such order cannot be said to be illegal or
otherwise unreasonable which can be interfered with in exercise of discretionary
jurisdiction of this Court under Article 136 of the Constitution and the appeals may be
dismissed.
11. Having heard the learned counsel for the parties, in our opinion, all the appeals should
be allowed. From the record, it is clear that in 1982, a huge land admeasuring about 28
acres at Thiruvanmiyur Extension, Chennai was acquired by the State under the Land
Acquisition Act for public purpose, namely, for the purpose of development of area
known as South Madras Neighbourhood Scheme. Amount of compensation was paid to
the land owners as per the award but it was enhanced in reference proceedings. The
Board came up to this Court, but the enhanced compensation was confirmed. It is also
clear from the Scheme initially prepared, i.e. seven types scheme and fifteen types
scheme which was subsequently finalized, there was difference in plinth area as also
ground area. So far as price is concerned, in 1991, when the names of applicants were
registered, it was clarified that the price indicated was 'tentative price' and it was subject
to 'final price' being fixed by the Board. In any case when the scheme was altered from
seven types to fifteen types flats, it was stated that the amount shown was merely
"tentative selling price." The intending purchasers, therefore, were aware of the fact that
the final price was to be fixed by the Board. In fact an agreement to that effect was
executed by all prospective allottees wherein they agreed that they would pay the amount
which would be finally fixed by the Board.
12. Clause 18 of the Agreement entered into between the parties and signed by all
allottees is extremely important and reads thus;
"It is expressly agreed between both the parties that after the finalization of the total cost
of construction of flat and the value of the land in accordance with the award of
compensation declared by the Tribunals and Courts the Purchaser shall pay to the Vendor
on demand before the registration of the Sale Deed the difference between the amount
already paid by the purchaser as per clause 2 above and the price amount finally fixed by
the Chairman the Vendor."
13. In the circumstances, it cannot be said that the allottees were not aware of the above
condition and they were compelled to make payment and thus were treated unfairly or
unreasonably by the Board.
14. The State Commission in the impugned order observed that it was the case of the
Board that excess amount of compensation was awarded to the land owners. It proceeded
to state that the excess compensation had been awarded in respect of lands covered by
other schemes in the neighbourhood and the Board attempted to shift the burden of the
excess amount on the allottees of Thiruvanmiyur Extension Scheme. It also stated that no
evidence was produced by the Board to show that there was any land acquisition
proceeding before any Court in respect of land covered by HIG Scheme No. 102 (though
Clause 18 of the Agreement extracted hereinabove expressly refers to such proceedings).
It also observed that an affidavit was filed by the Secretary of the Complainant-
Association that HIG Scheme No. 102 was not involved in any land
@page-SC1155
acquisition proceedings before any Court and the said averment has not been rebutted by
the Board. (It may, however, be stated that in the reply filed by the Board before the State
Commission, it was asserted that one of the reasons for increase in cost was due to excess
amount of compensation allowed to the land-owners). The State Commission observed
that all the three defences raised by the Board were 'delectably vague', without any
particulars as to how much escalation was due to plinth area, how much was due to
increase in the land area and how much was due to payment of enhanced compensation to
land owners. It went on to state that the cost of enhanced compensation and increased
area "must also have been taken into consideration in fixing the tentative selling price".
The action of the Board, in the opinion of the State Commission was, therefore, unjust
and arbitrary.
15. It was also held that reduction of period of payment of balance amount from 15 years
to 13 years by monthly instalments amounted to 'deficiency in service' and that part was,
therefore, illegal. Accordingly, the following directions were issued by the State
Commission;
"1. It is declared that the opposite parties are entitled to claim from the members of the
complainant-Association for the flats allotted to them under No. 102 HIG Scheme at
Thiruvanmiyur Extension only the selling price mentioned in Ex. A2 (a) containing the
particulars of this Scheme.
2. The opposite parties are directed to refund to the members of the complainant-
Association who have made full payment, the excess amount collected with interest
thereon at 12% from the date of collection till payment.
3. In respect of the Members of the Complainant-Association who have opted for
payment in instalments, the opposite parties are directed to re-schedule the balance of
payment as per Ex. A2(a) in monthly instalments for 15 years instead of 13 years and
adjust the excess payment made, if any, towards future instalments.
4. The opposite parties are also directed to pay a consolidated sum of Rs. 7,000/- as costs
to the Complainant-Association at the rate of Rs. 1,000/- per complaint."
16. The National Commission, without discussing the evidence on record as also
contentions raised by the Board, conclusions arrived at and reasons weighed with the
State Commission, confirmed the findings by a brief order.
17. As observed earlier, it was contended by the Board before the State Commission and
National Commission that fixation of price of flats cannot fall within the purview of the
Commission. It is, no doubt, true that 'housing construction' had been included in the
definition of 'service' in clause (o) of Section 2(1) of the Act by the Consumer Protection
(Amendment) Act, 1993 (Act 50 of 1993). But it was submitted that the 'fixation of price'
cannot be made subject-matter of dispute and Consumer Commission could not deal with
the question as to adequacy of price. A specific contention was raised by the Board before
the State Commission and National Commission, but it was decided against the Board
though according to the Board, the point was covered by earlier decisions of the National
Commission itself.
18. The learned counsel for the Board referred to a decision of the National Commission
in Gujarat Housing Board v. Akhil Bhartiya Grahak Panchayat and Ors., (1996) 1 CPJ
103. Considering the provisions of the Act, the National Commission held that the
Consumer Commission had no jurisdiction to go into the question of pricing of houses
and plots, sold or allotted on hire-purchase system by the Development Authority or
Housing Board. The Commission relied upon its earlier decision in Gujarat Housing
Board v. Datania Amritlal Fulchand and Ors., (1993) 3 CPJ 351.
19

. True it is that in Lucknow Development Authority v. M. K. Gupta, (1994) 1 SCC 243,


this Court stated; 1994 AIR SCW 97, Para 5

"When private undertakings are taken over by the Government or Corporations are
created to discharge what is otherwise State's function, one of the inherent objectives of
such social welfare measures is to provide better, efficient and the cheaper services to the
people. Any attempt, therefore, to exclude services offered by statutory or official bodies
to the common man would be against the provisions of the Act and spirit behind it. It is
indeed unfortunate that since enforcement of the Act there is a demand and even political
pressure is built up to exclude one or the other class from operation of the Act. How
ironical it is that official or semi-official bodies which insist on numerous benefits, which
are otherwise available
@page-SC1156
in private sector, succeed in bargaining for it on threat of strike mainly because of larger
income accruing due to rise in number of consumers and not due to better and efficient
functioning claim exclusion when it comes to accountability from operation of the Act.
The spirit of consumerism is so feeble and dormant that no association, public or private
spirited, raises any finger on regular hike in prices not because it is necessary but either
because it has not been done for sometime or because the operational cost has gone up
irrespective of the efficiency without any regard to its impact on the common man. In our
opinion, the entire argument found on being statutory does not appear to have any
substance. A Government or semi-Government body or a local authority is as much
amenable to the Act as any other private body rendering similar service. Truly speaking it
would be a service to the society if such bodies instead of claiming exclusion subject
themselves to the Act and let their acts and omissions scrutinized as public accountability
is necessary for healthy growth of society."
20. The above observations make it clear that when private undertakings are taken over
by the State or its Instrumentalities, any attempt to exclude the services offered by such
statutory bodies to the common-man from the application of the Act must be discouraged.
It would be against the spirit behind the benevolent legislation. At the same time,
however, it cannot be overlooked that 'price fixation' depends on several factors.
Normally, therefore, it would not be appropriate to enter into adequacy of price.
21

. It may be profitable at this stage to refer to a decision of this Court in Premji Bhai
Parmar and Ors. v. Delhi Development Authority and Anr. (1980) 2 SCC 129. The
petitioner in that case purchased a plot offered by the respondent-Authority and after
payment of price took possession thereof. Subsequently, however, he filed a petition
under Article 32 in this Court contending that the surcharge collected by the authority
was illegal and violative of Article 14. Dismissing the petition, this Court held that the
remedy sought by the petitioner to reopen the concluded contract with a view to getting
back a part of the purchase price paid and benefit taken was not proper. AIR 1980 SC
738

22

. The Court stated : (Para 8 of AIR)

"Conceding for this submission that the Authority has the trappings of a State or would be
comprehended in 'other authority' for the purpose of Article 12, while determining price
of flats constructed by it, it acts purely in its executive capacity and "is bound by the
obligations which dealings of the State with the individual citizens import into every
transaction entered into the exercise of its constitutional powers. But after the State or its
agents have entered into the field of ordinary contract, the relations are no longer
governed by the Constitutional provisions by the legally valid contract which determines
rights and obligations of the parties inter se. No question arises of violation of Article 14
or of any other constitutional provision when the State or its agents, purporting to act
within this field, perform any act. In this sphere, they can only claim rights conferred
upon them by contract and are bound by the terms of the contract only unless some
statute steps in and confers some special statutory power or obligation on the State in the
contractual field which is apart from contract."
23

. The Court went on to state : (Para 9 of AIR)

"The principal contention canvassed on behalf of the petitioners is that the treatment
meted to them by the Authority is discriminatory inasmuch as no surcharge was levied on
flats in MIG Scheme constructed and allotted prior to November 1976 and after January
1977. MIG flats involved in these petitions were constructed and were available for
allotment in November 1976 and the lots were drawn in January 1977. There is one more
MIG Scheme at Munirka where the allotment took place at or about the same time but in
which case no surcharge was levied. The contention is that once for the purpose of
eligibility to acquire a flat, the criterion is grounded in income brackets, MIG, LIG, et.
al., those in the same income bracket form one class even for the purpose of determining
disposal price of flat allottable to them irrespective of situation, location or other relevant
determinants which enter into price calculation and therefore, in the same income group
there cannot be differentiation by levying of surcharge in some cases and charging only
the cost price in other cases and that the discrimination is thus writ large on the face of
the record because by levying surcharge in case of petitioners they have been treated
unequally and with an evil eye. It is difficult to
@page-SC1157
appreciate how Article 14 can be attracted in the circumstances hereinabove mentioned.
Cost price of a property offered for sale is determined according to the volition of the
owner who has constructed the property unless it is shown that he is under any statutory
obligation to determine cost price according to certain statutory formula. Except the
submission that the Authority has a proclaimed policy of constructing and offering flats
on 'no profit no loss' basis which according to Mr. Nariman has a statutory flavour in the
regulations enacted under the Act, the Authority is under no statutory obligation about its
pricing policy of the flats constructed by it. When the flats were offered to the petitioners
the price in round figure in respect of each flat was mentioned and surcharge was not
separately set out and this price has been accepted by the petitioners. The obligation that
regulations are binding on the Authority and have provided for a statutory price fixation
formula on 'no profit no loss' basis will be presently examined but save this the Authority
is under no obligation to fix price of different flats in different schemes albeit in the same
income group at the same level or by any particular statutory or binding formula. The
Authority having the trappings of a State might be covered by the expression 'other
authority' in Article 12 and would certainly be precluded from according discriminatory
treatment to persons offering to purchase flats in the same scheme. Those who opt to take
flats in a particular income-wise area-wise scheme in which all flats came up together as
one project, may form a class and any discriminatory treatment in the same class may
attract Article 14. But to say that throughout its course of existence the Authority would
be bound to offer flats income-group-wise according to the same price formula is to
expect the Authority to ignore time, situation, location and other relevant factors, which
all enter the price structure. In price fixation executive has a wide discretion and is only
answerable provided there is any statutory control over its policy of price fixation and it
is not the function of the Court to sit in judgment over such matters of economic policy as
must be necessarily left to the Government of the day to decide. The experts alone can
work out the mechanics of the price determination; Court can certainly not be expected to
decide without the assistance of the experts".
24

. Again, in Bareilly Development Authority v. Ajai Pal Singh, (1989) 2 SCC 116, the
Authority (BDA) constructed plots for persons belonging to different income groups. The
terms and conditions contained in the brochure empowered the BDA to revise the cost of
price and to enhance the rate of flats. The petitioners got themselves registered for
allotment of flats. Notices were issued by the BDA intimating the petitioners regarding
the costs of flats and the rate of instalments. The said action was challenged under Article
226 of the Constitution. The High Court of Allahabad, placing reliance on R. D. Shetty v.
International Airports Authority, (1979) 3 SCC 489 held that the BDA acted arbitrarily
and unreasonably in unilaterally enhancing the cost of flats and the rate of instalments
and directed the BDA to redetermine the issue. The BDA approached this Court. AIR
1989 SC 1076
AIR 1979 SC 1628

25

. Allowing the appeal, setting aside the judgment of the High Court and distinguishing
International Airports Authority, this Court observed; AIR 1979 SC 1628

"Even conceding that the BDA has the trappings of a State or would be comprehended in
'other authority' for the purpose of Article 12 of the Constitution, while determining price
of the houses/flats constructed by it and the rate of monthly instalments to be paid, the
'authority' or its agent after entering into the field of ordinary contract acts purely in its
executive capacity. Thereafter the relations are no longer governed by the constitutional
provisions but by the legally valid contract which determines the rights and obligations of
the parties inter se. In this sphere, they can only claim rights conferred upon them by the
contract in the absence of any statutory obligations on the part of the authority (i.e. BDA
in this case in the said contractual field." AIR 1989 SC 1076, Para 20

(Emphasis supplied)
26

. Recently, in Chief Administrator, PUDA v. Shabnam Virk, (2006) 4 SCC 74, the allottee
had filed an affidavit clearly indicating that she would undertake to abide by all the terms
and conditions of allotment letter and the amount indicated therein for allotment of a
house. There was nothing to show that the increase was possible only when there was
increase in the 2006 AIR SCW 1943, Paras 14 and 15

@page-SC1158
cost of construction. It was held by this Court that the allottee was liable to pay amount as
stipulated in the allotment letter. It was observed;
"It is to be noted that the respondent herself had accepted in the undertaking that she
accepted the allotment of the house and undertook to abide by all the terms and
conditions of the allotment letter. It is not in dispute that in the allotment letter the figure
as demanded has been reflected. That being so the respondent was liable to pay the
amount as stipulated in the allotment letter.
As there is no dispute that the respondent had in fact filed an affidavit clearly indicating
that she undertook to abide by all the terms and conditions of the allotment letter, the
amount indicated in the allotment letter was the amount in respect of the allotment of the
house. We find nothing in the quoted clause to show that the increase was possible only
when there was an increase in the cost of construction. The clause quoted above does not
reflect any such intention of the parties."
27. In our considered opinion, the State Commission as well as National Commission
ought to have considered all these aspects. Even if they were of the view that after the
amendment of the Act in 1993 and in the light of inclusion of 'housing construction'
within the meaning of 'service' in clause (o) of Section 2(1), the Commission had
jurisdiction to deal with and decide disputes relating to deficiency in service under the
Act which included the issues raised, it was obligatory on them to consider whether the
controversy raised in the proceedings with regard to fixation of price would be justiciable
on the facts and in the circumstances of the case, particularly in the light of the
contentions raised by the Board that there was increase in plinth area, ground area and
payment of enhanced compensation to land owners. They were also required to consider
that the Board does not have land of its own and the land was acquired under the Land
Acquisition Act by paying compensation as determined in accordance with the provisions
of that law. The Commissions also could not ignore the fact that when the advertisement
was issued for the purpose of registration of intending purchasers of flats they were
clearly intimated that the price shown was merely a 'tentative price'. Again, when the
scheme was altered the intending purchasers were informed that the price was tentative
and they would have to pay price finally determined by the Board. They consented and
entered into an agreement by giving an undertaking that they would pay the price
determined by the Board. When the question of giving possession of flats came up, the
Board informed them to pay the remaining amount so that possession could be delivered
to them. They made such payment and obtained possession. It was, therefore, contended
by the Board that the allottees were estopped from raising the contention that additional
amount could not have been recovered from them. It was open to the allottees not to pay
the additional amount demanded by the Board and not to take possession. By agreeing to
pay the amount and by paying such amount and taking possession, now they want to go
behind the concluded contract between the parties. In our considered opinion, all these
questions were required to be gone into by the State Commission as also by the National
Commission. The orders passed by both the Fora are, therefore, liable to be set aside.
28. Before we part with the matter, we may refer to one more aspect. After the Board
approached this Court and notice was issued, the respondent-Association filed a counter
affidavit in this Court through Secretary of the Association. In the said affidavit, the
orders passed by the State Commission and affirmed by the National Commission were
sought to be supported. One may appreciate allottees taking such stand supporting the
orders which were passed in their favour. But while doing so, certain averments and
remarks have been made which were not necessary for determining the question. For
instance in paragraph 12 of the affidavit-in-reply, it was stated;
"A public undertaking like the Housing Board has not only to act fairly, but also openly it
cannot suppress vital documents and play the game of hide and seek. We have given to
ourselves a democratic Constitution. Accountability and transparency are the pillars of
democracy. There must be sun shine in the corridors of power. It is lamentable that the
bureaucrates of the Housing
@page-SC1159
Board are still living in the atmosphere of British Raj and accountability and transparency
are anaethima to them."
29. In paragraph 16 of the counter, similar allegations have been levelled. It was stated
that an instrumentality of State is expected to conduct its affairs in transparent manner,
but the Board failed to do so. At another place, it was said that service oriented body like
the Housing Board cannot act like private bodies and take a 'Shylockean attitude'. In our
opinion, all those observations could have been easily avoided. Since we are setting aside
both the orders and remitting the cases to the State Commission for deciding afresh in
accordance with law, it would not be appropriate to say anything more on this. Let the
matter rest there.
30. For the foregoing reasons, all the appeals are allowed. The order passed by the State
Commission and confirmed by the National Commission is set aside. All the complaints
are remitted to the State Commission to decide them in accordance with law after hearing
the parties. On the facts and in the circumstances of the case, there shall be no order as to
costs. Amount if any, deposited by the appellant-Board in this Court may be refunded to
the Board with accrued interest thereon. Since the original complaints were filed in 1995,
the State Commission will give priority to the cases and decide them as expeditiously as
possible preferably before June 30, 2008.
31. At this stage, we may clarify that we should not be understood to have expressed any
opinion one way or the other on the controversy raised by the parties. All the observations
made by us hereinabove are limited for the purpose of holding that the State Commission
as also National Commission ought to have dealt with and decided the contentions raised
by the Housing Board. Therefore, as and when the complaints will be placed for hearing
before the Commissions, they will be decided strictly on their own merits without being
inhibited by those observations.
32. Ordered accordingly.
Appeals allowed.
AIR 2008 SUPREME COURT 1159 "Narayan Ghosh v. State of Orissa"
(From : Orissa)*
Coram : 2 S. B. SINHA AND V. S. SIRPURKAR, JJ.
Criminal Appeal No. 251 with 252 of 2008, (arising out of S.L.P. (Cri.) Nos. 6875 with
5941 of 2007), D/- 4 -2 -2008.
Narayan Ghosh alias Nantu v. State of Orissa

WITH
Sankar Adeya v. State of Orissa.
Criminal P.C. (2 of 1974), S.439 - BAIL - CRIMINAL CONSPIRACY - MURDER - Bail
- Accused charged of criminal conspiracy to commit murder - Had common grudge
against deceased - Trial had almost come to an end only few witnesses remained to be
examined - Some of witnesses to conspiracy had turned hostile - Accused politically
influential and financially strong - Had capacity to influence witnesses - Releasing
accused when trial is at such precarious stage would not be proper - Moreover accused
are residents of border districts and possibility of they fleeing from judicial process
cannot be ruled out - Accused held, not entitled to be released on bail. (Paras 9, 10)
Cases Referred : Chronological Paras
2005 AIR SCW 323 : AIR 2005 SC 716 : 2005 Cri LJ 883 (Ref.) 11
Basudev Panigarhi, Uday Umesh Lalit, Sr. Advocates, Deepak Kumar Jena, Ms.
Minakshi Jena, Humanu Sahu, Bijan Kumar Ghosh, Dipankar Burman, S.K. Poddar,
Anurag Pandey, for Appellant; Janaranjan Das, Swetaketu Mishra, for Respondent.
* BLAPL No. 4487 of 2007, D/- 4-9-2007 (Ori).
Judgement
1. V. S. SIRPURKAR, J. :- Leave granted.
2. Being aggrieved by the refusal of bail by the High Court both the accused persons have
come up before us by filing separate appeals.
3. The accused before us along with six other accused persons are facing the prosecution
for the offences of criminal conspiracy under Section 120B, IPC, murder under Section
302/34, IPC as also the offences under Sections 25 and 27 of the Indian Arms Act. The
case of the prosecution appears to be that the 8 accused persons conspired and in
pursuance of that conspiracy
@page-SC1160
one Tapas Mitra was murdered on the Puri Sea Beach. Immediate report with regard to
the murder was lodged at the Puri Sea Beach Police Station by one Prashanta Bala to the
effect that the informant along with some others, all of whom were the members of
Baragaon Motor Association, about 250 members, had come to Puri and stayed in the
hotels and deceased Tapas Mitra who was a Councilor of Baragaon Municipality was one
of them. It was alleged that he was a member of the Trade Union and was an invited
guest and was staying at Hotel Mayur and on 22.6.2006 at about 9.20 p.m. when the
informant along with deceased Pallav Das, Tapas Ghosh and Swapan Seth were sitting on
the sea beach in front of Hotel R.L., a man suddenly came on the spot and fired at Tapas
Mitra, as a result of which Tapas Mitra sustained bleeding injuries. It was further averred
that the persons present there tried to catch the assailant but he was able to escape. It was
expressed specifically in the FIR that it was due to the political rivalry and previous
enmity that Tapas Mitra was murdered. On the basis of the investigation some persons
were arrested and a charge-sheet came to be filed against eight accused persons.
However, after further investigation as per Section 173(8), Cr.P.C., the SDJM, Puri by his
orders dated 2-1-2007 took cognizance of the offences under Section 302/34, IPC as also
under Sections 25/ 27 of the Arms Act against the accused persons and as such they were
arrested on 30th March, 2007 by Banagaon Police Station on the requisition of Puri Sea
Beach Police Station.
4. To begin with, the appellants were released by Calcutta High Court on interim bail by
order dated 5-4-2007 on certain conditions and they were further directed to surrender
before the appropriate court, i.e., SDJM, Puri within two weeks from the date of their
release. The appellants accordingly surrendered on 20-4-2007 and prayed for the bail.
However, that prayer was rejected. The appellants, therefore, moved the Sessions Judge,
Puri. Even the Sessions Judge dismissed their bail applications. They thereafter moved
the High Court of Orissa. However, even the High Court seems to have dismissed their
bail applications. The appellants have now come up before us.
5. We have heard Shri Uday Umesh Lalit and Shri Panigrahi, learned Senior Counsel
appearing on behalf of both the appellants. It was urged that both the appellants have
been falsely implicated on account of their alleged political differences. It is further stated
that the theory of conspiracy has no legs to stand as there is absolutely no evidence to
support the same. It is also pointed out that the three witnesses who were set up in
support of the conspiracy were already examined before the Sessions Court and all of
them did not support the theory, in the sense that they turned hostile. Learned counsel
further urged that the only possible material against them would be the confessions of the
co-accused and even assuming that such confessions are admissible under Section 10 of
the Indian Evidence Act, they would be of no consequences since such confessions
cannot be used as substantive evidence. It is then pointed out that otherwise there is
absolutely no material implicating the appellants and, therefore, they are liable to be
released on bail.
6. Shri Janaranjan Das, learned counsel for the prosecution vehemently opposed the
appeals. It is pointed out that the trial has already commenced and has substantially
proceeded inasmuch as a large number of witnesses have already been examined. It is
also expressed that at this stage of the trial it would not be proper to release the accused
on bail as such release was likely to affect adversely the evidence of the proposed
witnesses in the sense that there was every likelihood of the witnesses being intimidated
because of the release of the appellants on bail.
7. Shri Lalit also supported his arguments by suggesting that the appellant Sanker Adeya
was suffering from a serious kidney disorder and was in precarious health and pressed the
ground of health in support of plea of bail. Learned counsel also further pointed out that
there are some inherent defects in the prosecution story inasmuch as though Sanker
Adeya was in Bangladesh from 19-5-2006 to 30-5-2006 yet it was suggested by
prosecution witness Swapan Mondal that he had met Sankar Adeya and Naryana Ghosh
at Tarapith temple and had a feast where the conspiracy for the murder was hatched.
Learned counsel also pointed out that the statement of witnesses like Nirmal Biswas and
Ashok Das @ Putke were recorded late and after the arrest of the appellant Sankar
Adeya. All this, according to the learned counsel, went on to suggest that the appellants
were actually
@page-SC1161
innocent and were unnecessarily implicated.
8. Shri Janaranjan Das, learned counsel appearing for the prosecution pointed out that the
prosecution has the material to show that as many as six accused persons, namely,
Satyajit Lohar, Tarun Kumar Bhowmik @ Raja, Samir Durlav, Bapi Roy, Rajen Biswas
and Samir Das had boarded a vehicle Tata Sumo from Chakdaha in the evening of 18-6-
2006 and came to the Siromani Mondal at Gaganpur where they halted and next morning
all these six persons along with three others, namely, Buro @ Akas, Kalo, etc. left for
Puri and arrived there on 19-6-2006 and stayed in Hotel Kingfisher. Next day they were
joined by Kaka and Munna who had arrived by Puri Express. All these persons, helped by
two other persons to identify Tapash Mitra @ Tofan Mitra, firstly attempted to murder
him in the afternoon but failed. However, in the evening on information that Tapas Mitra
had gone to Puri Sea Beach, the accused persons located Tapas Mitra who was sitting
there. Accused Raju, Bapi Roy and Akas @ Buro went and sat behind Tapas Mitra and
one of them fired at Tapas Mitra, resulting in his death. The accused thereafter fled from
the scene of occurrence. It was then contended that Bapi Roy had confessed about the
conspiracy. It has also come out that both the appellants were friends and they had
common grudge against Tapas Mitra.
9. Considering everything, we are of the clear opinion that it will not be possible to
release the accused on bail at this stage. It is an admitted position that the Sessions trial
had almost come to an end, and there are only few more witnesses to be examined. The
prosecution has expressed that the appellants are politically influential and financially
strong and are capable of influencing the witnesses. It has also been expressed that the
appellants are residents of Banagaon District which is on the Bangladesh border and,
therefore, there is every likelihood of their fleeing from the judicial process.
10. It is an admitted position that the appellants Sankar Adeya and Narayan Ghosh are the
residents of Banagaon District which is a border District. Therefore, it cannot be said that
the apprehension expressed by the learned counsel for the Prosecution is totally
unfounded. Learned counsel, however, insisted that we should consider the material and
more particularly the evidence regarding the conspiracy. We do not think that it would be
proper for us to discuss the evidence threadbare as any expression of ours would
undoubtedly affect the trial. It was admitted during the debate that some witnesses who
were the witnesses for conspiracy were examined and had to be declared hostile. If that is
so, that is all the more reason for us not to release the appellants when the trial is at a
precarious stage.
11

. Much debate was devoted about the non admissibility of the confessions of the co-
accused which were likely to be relied upon by the prosecution. Reference was made to
the reported judgments more particularly of Jayendra Saraswathi Swamigal v. State of
Tamil Nadu [(2005) 3 SCC 13]. It was urged, relying on that decision, that there was no
reasonable ground to believe that two or more persons in this case had conspired together
to commit an offence and if there was no prima facie evidence of the existence of
conspiracy, then there was no question of any evidence of the acts and statements made
by any of the accused in furtherance of the common object being admissible at all.
Learned counsel strenuously argued that there was no prima facie evidence to show that
the two appellants were party to the conspiracy and had conspired together between
themselves or with any other accused persons. It was pointed out from the reported
decision that it was only when the conspiracy was being hatched, whatever was said
could become admissible. Our attention was invited to the following observations:2005
AIR SCW 323

"The words of Section 10 are not capable of being widely construed so as to include a
statement made by one conspirator in the absence of the other with reference to past acts
done in the actual course of carrying out the conspiracy, after it has been completed. The
words 'common intention' signify a common intention existing at the time when the thing
was said, one or written by one of them. Things said, done or written while the
conspiracy was on foot are relevant as evidence of the common intention, once
reasonable ground has been shown to believe in its existence. But it would be a very
different matter to hold that any narrative or statement or confession made to a third party
after the common intention or conspiracy
@page-SC1162
was no longer operating and had ceased to exist is admissible against the other party.
There is then no common intention to the conspirators to which the statement can have
reference."
There is no dispute about the principles stated in the ruling, however, we do not think that
it would be proper for us to discuss at this juncture about prima facie finding. In our
opinion it would be for the trial court to consider and appreciate the evidence which
comes before it in support of the plea of conspiracy and to arrive at the correct finding.
We will not, at this stage, comment upon the nature of the evidence one way or the other.
In that view of the matter we do not think that the High Court was in error in refusing the
bail to the appellants.
12. However, in view of the plea regarding the health of Sankar Adeya, we direct that all
the timely medical help shall be made available to him. We also further direct that the
trial shall be completed without any unnecessary delay and as far as possible within four
months from today. With these observations, we dismiss both the appeals.
Appeals dismissed.
AIR 2008 SUPREME COURT 1162 "Management, W. B. Colliery of M/s. TISCO Ltd. v.
Ram Pravesh Singh"
(From : Jharkhand)*
Coram : 2 ASHOK BHAN AND DALVEER BHANDARI, JJ.
Civil Appeal No. 892 of 2008 (arising out of S.L.P. (C) No. 13281 of 2006), D/- 1 -2
-2008.
Employers in relation to the Management of West Bokaro Colliery of M/s. TISCO Ltd. v.
The Concerned Workman, Ram Pravesh Singh.
(A) Industrial Disputes Act (14 of 1947), S.11A - MISCONDUCT - LABOUR COURT -
TRIBUNALS - EVIDENCE - Labour Court/Tribunal - Interference with finding of
Inquiry Officer - Court/Tribunal not to act Appellate Court - Finding of I.O. as to
misconduct based on evidence - Interference on ground that no independent witness was
examined - Improper - Once finding of I. O. is found to based on evidence Court not to
substitute finding of I. O. by its own opinion.
L. P. A. No. 389 of 2004, D/- 15-6-2006 (Jhar), Reversed. (Para 17)
(B) Industrial Disputes Act (14 of 1947), Sch.2, Item 6 - INDUSTRIAL DISPUTE -
DISCIPLINARY PROCEEDINGS - CRIMINAL PROCEEDINGS - Disciplinary
proceedings - Not barred by acquittal of delinquent in criminal proceedings - Standard of
proof in criminal proceedings and Disciplinary proceedings are different. (Para 19)
Cases Referred : Chronological Paras
2007 AIR SCW 7649 16
2006 AIR SCW 2801 : AIR 2006 SC 2208 : 2006 (3) AIR Jhar R 356 (Ref.) 20
2004 AIR SCW 5427 : AIR 2004 SC 4761 : 2004 Lab IC 3744 : 2004 AIR - Kant HCR
3103 15
2002 AIR SCW 2777 : AIR 2002 SC 2545 : 2002 Lab IC 2475 : 2002 AIR-Kant HCR
1920 15
AIR 1977 SC 1512 : 1977 Lab IC 845 15
AIR 1973 SC 1227 : 1973 Lab IC 851 (Ref.) 20
Raju Ramachandran, Sr. Advocate, M.K. Dua, for Appellant; Sunil Kumar, Sr. Advocate,
Amitesh Chandra Mishra, Alok Kumar, for Respondent.
* L.P.A. No. 389 of 2004, D/- 15-6-2006 (Jhar).
Judgement
1. BHAN, J.:-Leave granted.
2. Management is in appeal.
3. The respondent-workman was working as Senior Dumper Operator under the
Management of the appellant. The workman was deputed at Open Caste Mine, West
Bokaro on 2nd of March, 1994 during the first shift from 5.00 a.m. to 1.00 p.m.
Respondent left the place of his duty before the end of his shift duty and went to Rajiv
Nagar area where Shri Harbans Kumar, Senior Officer (Security), along with a number of
security personnel and other workers, was discharging his duties in connection with
prevention of unauthorized constructions on the company's land. The respondent-
workman along with few others approached Shri Harbans Kumar and shouted at him
using abusive language and threatened him with dire consequences in case the
unauthorized construction was demolished. The respondent-workman, on being asked not
to behave in the said manner, assaulted Shri Harbans Kumar with his hands and also
resorted to brick-bating as a result of which Shri Harbans Kumar and Shri S.P. Yadav
sustained injuries on the face and other parts of the body.
4. Appellant-Management issued a
@page-SC1163
charge sheet to the respondent-workman whereby he was asked to show-cause as to why
disciplinary action should not be taken against him under Clause 22(18) and 22(5) of the
Standing Orders of the Company for the following misconduct :-
"(a) leaving work without permission (b) indecent, riotous and disorderly behaviour with
a superior as well as co-worker."
5. The respondent-workman submitted his reply denying all charges brought against him.
The Management decided to conduct an enquiry and accordingly appointed Shri
Madhusudan Das, Deputy Manager (Personnel) as Enquiry Officer. The Enquiry Officer
after giving full opportunity to the respondent-workman came to the conclusion that the
charges levelled against him were established beyond reasonable doubt and submitted his
report.
6. The Punishing Authority after going through the Enquiry Report and related enquiry
papers, satisfied himself that charges levelled against the respondent had been established
and recommended the dismissal of the respondent from the Company with immediate
effect. The workman was accordingly dismissed on 23/25th of April, 1994.
7. The respondent raised an industrial dispute and the Government of India, Ministry of
Labour, in exercise of its powers under Section 10(1)(d) of the Industrial Disputes Act,
1947 (for short the Act), referred the following dispute to the Tribunal for adjudication:
"THE SCHEDULE
"Whether the action of the Management of West Bokaro Collieries of M/s. TISCO Ltd.
PO-Ghatotand, Dist. Hazaribagh in dismissing Shri Ram Pravesh, Ex. Sr. Dumper
Operator from the services of the Company w.e.f 25-4-1994 is justified? If not, to what
relief the workman is entitled?"
8. The respondent on 3rd of October, 2003, made a statement before the Labour Court
that he did not want to challenge the legality, fairness and propriety of the domestic
enquiry. On this statement being made, the Labour Court, after careful consideration of
the facts and circumstances and the submissions advanced by the Counsel for the
respondent, held that the domestic enquiry conducted by the Management was fair,
proper and in accordance with the principles of natural justice. The matter was adjourned
to 14th of December, 2001 for hearing argument on merit.
9. The Industrial Tribunal set aside the order of dismissal passed against the respondent
by holding that the Management had failed to substantiate the charges brought against the
concerned workman beyond reasonable doubt. Accordingly, order of dismissal passed
against the concerned workman was set aside and he was ordered to be reinstated with
50% back wages.
10. Management, thereafter, filed the Writ Petition before the High Court which was
dismissed by the Learned Single Judge, aggrieved against which Management filed
Letters Patent Appeal which has also been dismissed by the impugned order.
11. Learned Senior Counsel, Mr. Raju Ramachandran, appearing for the Management
submitted that the findings recorded by the domestic Tribunal based on the evidence
cannot be set aside or interfered with by the Industrial Tribunal or the Courts by
substituting their substantive opinion in place of the one arrived at by the domestic
Tribunal. It is further contended that the Tribunal applied the standard of proof of beyond
reasonable doubt which is required to be proved in criminal cases whereas in the
domestic enquiry and Civil Courts, the standard of proof is of preponderance of
probabilities. It is further contended that the Tribunal erred in relying upon the order of
acquittal passed in favour of the respondent by the Criminal Court as in the criminal
cases, the standard of proof required to prove a charge is materially different than in civil
matters.
12. As against this, Learned Counsel for the respondent contended that the Industrial
Tribunal was fully justified in coming to the different conclusions in exercise of its
powers under Section 11A of the Act.
13. Counsel for the parties have been heard at length.
14. The Tribunal in its order on re-appreciation of evidence came to the conclusion that in
the absence of any independent evidence other than of fellow workman, the charge of
indecent, riotous and disorderly behaviour with superior and co-worker was not proved.
Insofar as the absence from the duty is concerned, Tribunal came to the conclusion that
according to the workman, he had left the place of work at 12.25 P.M. and as the incident
allegedly had taken place
@page-SC1164
at 12.30 P.M., the respondent could not have reached the place of incident at 12.30 P.M.
after collecting his other associates. In para 14 of its order, the Tribunal concluded that
Management had failed to substantiate the charges brought against the workman beyond
reasonable doubt.
15

. This Court in Divisional Controller, KSRTC (NWKRTC) v. A.T. Mane [(2005) 3 SCC
254], held that: - 2004 AIR SCW 5427, (Para 10)

"From the above it is clear that once a domestic tribunal based on evidence comes to a
particular conclusion, normally it is not open to the appellate tribunals and courts to
substitute their subjective opinion in the place of the one arrived at by the domestic
tribunal. In the present case, there is evidence of the inspector who checked the bus
which establishes the misconduct of the respondent. The domestic tribunal accepted that
evidence and found the respondent guilty. But the courts below misdirected themselves in
insisting on the evidence of the ticketless passengers to reject the said finding which, in
our opinion, as held by this Court in the case of Rattan Singh [(1977) 2 SCC 491] is not a
condition precedent. We may herein note that the judgment of this Court in Rattan Singh
has since been followed by this Court in Devendra Swamy v. Karnataka SRTC [(2002] 9
SCC 644]" AIR 1977 SC 1512
2002 AIR SCW 2777

16
. In U.P. State Road Transport Corporation vs. Vinod Kumar [2007 (13) SCALE 690],
this Court again observed that in the absence of a challenge to the legality or fairness of
the domestic enquiry, the Court should be reluctant to either interfere with the finding
recorded by the Enquiry Officer or the punishment awarded by the Punishing Authority.
2007 AIR SCW 7649

17. After going through the order of the Industrial Tribunal, we are of the opinion that the
Tribunal has interfered with the findings recorded by the domestic Tribunal as if it was
the Appellate Tribunal. There was evidence present on record regarding indecent, riotous
and disorderly behaviour of the respondent towards his superiors. The Management
witnesses who were present at the scene of occurrence have unequivocally deposed about
the misbehaviour of the respondent towards his superiors. Their evidence has been
discarded by the Tribunal by observing that in the absence of independent evidence, the
statements of the workmen who were present at the scene of occurrence could not be
believed. Industrial Tribunal fell in error in discarding the evidence produced by the
Management only because the independent witnesses were not produced. It is nobody's
case that the independent witnesses were available at the scene of occurrence and the
Management had failed to produce them. It is possible that at the time of occurrence, only
the workers of the Management and the persons who were trying to put up the
construction unauthorizedly were the persons present and no independent evidence was
available. Statements of the fellow workmen had established the misconduct of the
respondent. Enquiry Officer accepted the testimony of the witnesses produced by the
Management who had clearly implicated the respondent. It was a legitimate conclusion
which could be arrived at and it would not be open to the Industrial Tribunal to substitute
the said opinion by its own opinion.
18. Findings recorded by the Tribunal that the workman had left the place of duty at
12.25 P.M. and, therefore, could not have reached the place of occurrence at 12.30 P.M.
after collecting his other associates, is not based on any evidence. The case of the
Management is that the respondent had left his place of duty at 12.05 P.M. and reached
the place of occurrence at 12.30 P.M. after collecting his fellow workmen. There was
sufficient time for the workman to reach the place of occurrence within half an hour as
the distance between the place of duty and the place of occurrence was only 1 k.m. The
duty of the respondent-workman was up to 1.00 O'clock. Even if, it is accepted that he
left the place of duty at 12.25 P.M., then also, he left the place of duty during his duty
hours.
19. Tribunal has set aside the report of the Enquiry Officer and the order of dismissal
passed by the Punishing Authority by observing that the charges against the respondent
were not proved beyond reasonable doubt. It has repeatedly been held by this Court that
the acquittal in a criminal case would not operate as a bar for drawing up of a disciplinary
proceeding against a delinquent. It is well settled principle of law that yardstick and
standard of proof in a criminal case is different from the one in disciplinary proceedings.
While the standard
@page-SC1165
of proof in a criminal case is proof beyond all reasonable doubt, the standard of proof in a
departmental proceeding is preponderance of probabilities.
20
. Learned Counsel for the respondent cited two cases The Workmen of M/s. Firestone
Tyre and Rubber Co. of India (Pvt.) Ltd. v. The Management and Ors. [(1973) 1 SCC
813] and South Indian Cashew Factories Workers' Union vs. Kerala State Cashew
Development Corpn. Ltd.and Ors. [(2006) 5 SCC 201], to contend that the Labour Court
in exercise of its jurisdiction under Section 11A could have come to a different
conclusion. There is no quarrel with this proposition of law. The Labour Court could have
awarded lesser punishment in the given facts and circumstances of the case. In a case
where two views are possible on the evidence on record, then the Industrial Tribunal
should be very slow in coming to a conclusion other than the one arrived at by the
domestic Tribunal by substituting its opinion in place of the opinion of the domestic
Tribunal. AIR 1973 SC 1227
2006 AIR SCW 2801

21. Labour Court fell into the factual as well as legal error in setting aside the findings
recorded by the domestic Tribunal. Learned Single Judge as well as the Division Bench
have simply affirmed the findings recorded by the Tribunal.
22. For the reasons stated above, we accept this appeal, set aside the order passed by the
High Court as well as the Labour Court. Accordingly, the Order passed by the domestic
Tribunal and the Punishing Authority is restored. There should be no orders as to costs.
Appeal allowed.
AIR 2008 SUPREME COURT 1165 "Johar v. Mangal Prasad"
(From : 2007 Cri LR (SC and MP) 605
(Madh. Pra.))
Coram : 2 S. B. SINHA AND V. S. SIRPURKAR, JJ.
Criminal Appeal No. 215 of 2008 (arising out of SLP (Cri.) No. 2014 of 2007), D/- 30 -1
-2008.
Johar and Ors. v. Mangal Prasad and Anr.
(A) Criminal P.C. (2 of 1974), S.397, S.401 - REVISION - TRIAL COURT -
EVIDENCE - Revision - Court has limited jurisdiction - Order of Trial Court not found
to be passed without considering relevant evidence - Or passed by considering irrelevant
evidence - Interference by entering into merits and re-appreciating entire evidence -
Improper. (Para 9)
(B) Criminal P.C. (2 of 1974), S.397, S.401 - REVISION - Revision - Against judgment
of acquittal - Though not barred, jurisdiction is severally restricted. (Para 17)
(C) Criminal P.C. (2 of 1974), S.401(3) - REVISION - TRIAL COURT - REMAND OF
MATTER - Revision - Prohibition to convert acquittal to conviction - Applies to finding
and not conclusion - Trial Court not relying on eye-witnesses having regard to evidence
of Autopsy Surgeon - Revisional Court criticising testimony of Autopsy Surgeon for
deliberately suppressing injuries - Directing Trial Court to initiate proceedings against
him and relying upon eye-witnesses to hold that accused had inflicted injuries on
deceased - Remanding matter thereafter to Trial Court clearly amounts to reversal of
finding of Trial Court - Order passed is in excess of jurisdiction.
2007 Cri LR (SC and MP) 6O5 (M.P.), Reversed. (Paras 13, 20, 21, 22)
Cases Referred : Chronological Paras
2004 AIR SCW 3626 : AIR 2004 SC 4583 : 2004 Cri LJ 3115 (Rel. on, Pnt. B) 19
2004 AIR SCW 4767 : AIR 2004 SC 4412 : 2004 Cri LJ 4254 (Rel. on) 19
2004 AIR SCW 6646 : AIR 2005 SC 4161 : 2005 Cri LJ 648 (Rel. on) 19
1993 AIR SCW 248 : AIR 1993 SC 892 : 1993 Cri LJ 608 (Rel. on) 19
AIR 1968 SC 707 : 1968 Cri LJ 865 (Rel. on, Pnt. B) 19
AIR 1962 SC 1788 : 1963 (1) Cri LJ 8 (Rel. on, Pnt. B) 19
AIR 1951 SC 196 : (1951) 52 Cri LJ 510 (Rel. on) 11, 19
AIR 1951 SC 316 : (1951) 52 Cri LJ 1248 (Rel. on) 12, 19
Fakhruddin, Sr. Advocate, Abdul Karim Ansari, Abdul Qadri and Aftab All Khan, for
Appellants; Ms. Vibha Datta Makhija, for Respondents.
Judgement
1. S. B. SINHA, J. :- Leave granted.
2. Appellants were charged for commission of offences under Sections 148 and 302 of the
Indian Penal Code and in the alternative under Section 302/149 and Section 120-B of the
Indian Penal Code. They were, however, convicted for commission of an offence under
Section 323 read with Section
@page-SC1166
34 of the Indian Penal Code only, recording that as accused Nos. 1 to 4 had only caused
simple injuries to the deceased Umashankar, the provisions of Sections 148 and 149 of
the Indian Penal Code were not attracted.
3. The State did not prefer any appeal there against. The complainant/respondent,
however, filed a revision application before the High Court. The High Court went into the
evidence adduced on behalf of the prosecution. In regard to the deposition of the official
witnesses including Autopsy Surgeon it was commented :-
"10. If a public servant is corruptly (sic) makes a report in a judicial proceeding it will be
offences under section 193, IPC and section 196, IPC and preparation of document with
an intention to save person from punishment, it will be an offence falling under section
196, IPC. Thus, willful act of the Doctor in not referring to other injuries in the post-
mortem report discloses his intention to protect the respondents who are guilty of
commission of murder. Witnesses were firm on the point of beating of deceased by lathi
and number of injuries received by the deceased. It is held that post-mortem report is
incomplete report prepared by the doctor to give undue advantage to the accused.
Appropriate steps for prosecution of PW9 Dr.Y.K. Malaiya be initiated for intentionally
preparing false evidence." It was opined that having regard to the nature of deposition of
the Autopsy Surgeon, the trial Court committed a grave error in ignoring the other
relevant materials brought on records to pronounce a judgment of acquittal in favour of
the respondents (appellants herein). It was furthermore held that the doctor had willfully
suppressed the head injury and was thus guilty of dereliction of duty. Re-appreciating the
evidence of the prosecution witnesses, it was held :-
"It is natural that when a person is surrounded by number of accused it is difficult for
eye-witness to describe the author of each and every injury. In para 16 of cross-
examination, this witness has clarified that he has seen the body of injured and he found
that Umashankar was having lathi injuries on his entire body and no place on his body
was left where he had not received injuries by lathi."
4. On the premise that the learned trial Judge has mis-appreciated the evidence, the
revision application was allowed, directing :-
"22. In the result, judgment of acquittal passed by the trial court is set aside and the case
is remanded to the trial court to pass the judgment on the basis of evidence on record for
each offence keeping in mind evidence of eye-witnesses wherein it is stated that deceased
had suffered injuries on the whole body. The fact is also referred in Dehati Naleshi and
Panchnama of dead body Ex.P/3. Evidence of doctor will not prevail over the eye-witness
account in this case. This is a case under section 302, IPC and the intention of all the
respondents was to cause death of deceased. Trial court shall also examine and pass
necessary orders against the concerned doctor for preparing document in order to give
undue benefit to the accused".
5. We may, however, before embarking upon the contentions raised before us by the
learned counsel for the parties place on record that one of the accused persons, namely
Roshan, had preferred an appeal before the High Court of Madhya Pradesh at Jabalpur
and by a judgment and order dated 18th November, 2003, it while upholding his
conviction under Section 323 read with Section 34 of the Indian Penal Code set him free
on probation on his furnishing a personal bond for Rs. 3,000/- (Rupees there thousand
only) with one surety of the like amount. The said judgment and order has attained
finality.
6. Mr. Fakhruddln, the learned Senior counsel appearing for the appellant, submitted that
the High Court committed a manifest illegality in passing the impugned judgment which
is in the teeth of sub-section (3) of Section 401 of the Code of Criminal Procedure, 1973.
7. Ms. Vibha Datta Makhija, the learned counsel appearing on behalf of the respondent-
State, on the other hand, urged that it is not a case where the High Court converted a
judgment of acquittal to a judgment of conviction in exercise of its revisional jurisdiction
but merely remitted the matter to the trial court for consideration afresh, this Court should
not interfere therewith.
8. The State did not prefer any appeal from the judgment of the learned Trial Judge. From
the proceedings of the High Court, it appears that the State was not even made a party in
the criminal revision application.
@page-SC1167
Public Prosecutor, however, represented the State before the High Court. Nobody
interestingly appeared on behalf of the complaint-revisionist.
9. Revisional jurisdiction of the High Court in terms of Section 397 read with Section 401
of the Code of Criminal Procedure is limited. The High Court did not point out any error
of law on the part of the learned Trial Judge. It was not opined that any relevant evidence
has been left out of its consideration by the court below or irrelevant material has been
taken into consideration. The High Court entered into the merit of the matter. It
commented upon the credentiality of the Autopsy Surgeon. It sought to re-appreciate the
whole evidence. One possible view was sought to be substituted by another possible
view.
10. Sub-section (3) of Section 401 reads as under :-
"401(3). Nothing in this section shall be deemed to authorize a High Court to convert a
finding of acquittal into one of conviction."
Technically, although Ms. Makhija may be correct that the High Court has not converted
the judgment of acquittal passed by the learned Trial Court to a judgment of conviction,
but for arriving at a finding as to whether the High Court has exceeded its jurisdiction or
not, the approach of the High Court must be borne in mind. For the said purpose, we may
notice a few precedents.
11

. In D. Stephens v. Nosibolla: [1951] 1 SCR284, this Court opined : AIR 1951 SC


196

"10. The revisional jurisdiction conferred on the High Court under section 439 of the
Code of Criminal Procedure is not to be lightly exercised when it is invoked by a private
complainant against an order of acquittal, against which the Government has a right of
appeal under section 417. It could be exercised only in exceptional cases where the
interests of public justice require interference for the correction of a manifest illegality, or
the prevention of a gross miscarriage of justice. This jurisdiction is not ordinarily invoked
or used merely because the lower court has taken a wrong view of the law or
misappreciated the evidence on record."
12

. The same principle was reiterated in Logendra Nath Jha and others vs. Polailal Biswas
[1951 SCR 676] stating: AIR 1951 SC 316, Para 7

"........Though sub-section (1) of section 439 authorises the High Court to exercise, in its
discretion, any of the powers conferred on a court of appeal by section 423, sub-section
(4) specifically excludes the power to "convert a finding of acquittal into one of
conviction. This does not mean that in dealing with a revision petition by a private party
against an order of acquittal the High Court could in the absence of any error on a point
of law re-appraise the evidence and reverse the findings of facts on which the acquittal
was based, provided only it stopped short of finding the accused guilty and passing
sentence on him. By merely characterizing the judgment of the trial court as "perverse"
and "lacking in perspective", the High Court cannot reverse pure findings of fact based on
the trial Court's appreciation of the evidence In the case. That is what the learned Judge in
the court below has done, but could not, in our opinion, properly do on an application in
revision filed by a private party against acquittal........."
13. In the instant case the High Court not only entered into the merit of the matter but
also analysed the depositions of all the witnesses examined on behalf of the prosecution.
It, in particular, went to the extent of criticizing the testimony of Autopsy Surgeon. It
relied upon the evidence of the so-called eye-witnesses to hold that although appellants
herein had inflicted injuries on the head of the deceased, Dr. Y.K. Malaiya, PW-9,
deliberately suppressed the same. He was, for all intent and purport, found guilty of the
offence under Sections 193 and 196 of the Indian Penal Code. The Autopsy Surgeon was
not cross-examined by the State. He was not declared hostile. The State did not even
prefer any appeal against the judgment.
14. In the absence of any such injury on the vital part of the body, the learned trial Judge,
upon analyzing the evidence brought on record by the prosecution, held that only four
accused had committed the offence under Section 323 read with Section 34 alone. We see
no reason as to how the findings of the trial Judge can be said to be perverse. The learned
trial Judge in arriving at his conclusion noticed:-
(i) Names of some of the appellants were not stated in the first information report.
(ii) Some of the accused persons were not present at the time of commission of offence,
@page-SC1168
as their plea of alibi was acceptable.
(iii) The story of recovery of lathis from some of the accused is doubtful.
(iv) Purported recovery of lathi by the investigating officer without any disclosure
statement having been made by the concerned accused, was not relevant.
(v) Some of the accused did not have any dispute whatsoever with the complainant side,
as such they had no motive to commit the crime.
(vi) Only because some of the accused were present at the time of commission of the
offence, having regard to the fact that the incident took place in a very small village, their
presence at the time of occurrence by itself cannot lead to an inference that they
participated therein, particularly when prosecution witnesses did not name them.
(vii) No independent witness had been examined by the prosecution despite the fact that a
large number of persons witnessed the incident.
15. Upon analyzing the entire evidence on record, the learned trial Judge held :-

"58. In view of the discussion and analyses made hereinabove prosecution has proved
that accused persons Nos. 1 to 4 i.e. Johar, Ruplal, Roshan and Santosh inflicted simple
injuries to deceased Umashankar. Against accused persons offence under Sections 148,
302 r/w 149, IPC have been levelled but in the incident only accused Nos. 1 to 4 have
committed and thus participation of the number of accused is proved to be four only and
under sections 148 and 149, IPC the accused persons minimum remained to be five. As
such against accused Nos. 1 to 4 offence under Sections 148 and 149 are not proved."
16. Evidently the High Court raised a presumption that Autopsy Surgeon deliberately did
not disclose the ante mortem head injury purported to have been suffered by the
deceased.
17. The approach of the High Court to the entire case cannot be appreciated. The High
Court should have kept in mind that while exercising its revisional jurisdiction under
Sections 397 and 401 of the Code of Criminal Procedure, it exercises a limited power. Its
jurisdiction to entertain a revision application, although is not barred, but severally
restricted, particularly when it arises from a judgment of acquittal.
18. Ms. Makhija is correct that sub-section (4) of Section 378 of the Code of Criminal
Procedure was not available to the first informant but the same by itself would not mean
that in absence of any appeal preferred by the State, the limited jurisdiction of the court
should be expanded.
19. We may notice a few of the decisions of this Court which are binding on us.

In K. Chinnaswamy Reddy v. State of Andhra Pradesh [1963] 3 SCR 412, this Court
observed :- AIR 1962 SC 1788, Para 7

"It is true that it is open to a High Court in revision to set aside an order of acquittal even
at the instance of private parties, though the State may not have thought fit to appeal; but
this jurisdiction should in our opinion be exercised by the High Court only in exceptional
cases, when there is some glaring defect in the procedure or there is a manifest error on a
point of law and consequently there has been a flagrant miscarriage of justice. Sub-
section (4) of S. 439 forbids a High Court from converting a finding of acquittal into one
of conviction and that makes it all the more incumbent on the High Court to see that it
does not convert the finding of acquittal into one of conviction by the indirect method of
ordering retrial, when it cannot itself directly convert a finding of acquittal into a finding
of conviction. This places limitations on the power of the High Court to set aside a
finding of acquittal in revision and it is only in exceptional cases that this power should
be exercised."

In Mahendra Pratap Singh v. Sarju Singh and Anr. [1968] 2 SCR 287 this Court stated the
law thus :- AIR 1968 SC 707

"8. The practice on the subject has been stated by this Court on more than one occasion.
In D. Stephens v. Nosibolla [[1951] SCR 284], only two grounds were mentioned by this
Court as entitling the High Court to set aside an acquittal in a revision and to order a
retrial. They are that there must exist a manifest illegality in the judgment of the Court of
Session ordering the acquittal or there must be a gross miscarriage of justice. In
explaining these two propositions, this Court further states that the High Court is not
entitled to interfere even if a wrong AIR 1951 SC 196
AIR 1951 SC 316
AIR 1962 SC 1788

@page-SC1169
view of law is taken by the Court of Session or if even there is misappreciation of
evidence. Again, in Logendranath Jha and others v. Shri Polailal Biswas [[1951] SCR.
676], this Court points out that the High Court is entitled in revision to set aside an
acquittal if there is an error on a point of law or no appraisal of the evidence at all. This
Court observes that it is not sufficient to say that the judgment under revision is
"perverse" or "lacking in true correct perspective". It is pointed out further that by
ordering a retrial, the dice is loaded against the accused, because however much the High
Court may caution the Subordinate Court, it is always difficult to re-weigh the evidence
ignoring the opinion of the High Court. Again in K. Chinnaswamy Reddy v. State of
Andhra Pradesh, it is pointed out that an interference in revision with an order of acquittal
can only take place if there is a glaring defect of procedure such as that the Court had no
jurisdiction to try the case or the Court had shut out some material evidence which was
admissible or attempted to take into account evidence which was not admissible or had
overlooked some evidence. Although the list given by this Court is not exhaustive of all
the circumstances in which the High Court may interfere with an acquittal in revision it is
obvious that the defect in the judgment under revision must be analogous to those
actually indicated by this Court."

In Janata Dal vs. HS Chowdhary (1992) 4 SCC 305, this Court stated that the object of
the revisional jurisdiction was to confer power on superior criminal courts to correct
miscarriage of justice arising from misconception of law, irregularity of procedure,
neglect of proper precaution or apparent harshness of treatment. 1993 AIR SCW 248
In State of Maharashtra vs. Jagmohan Singh Kuldip Singh Anand : (2004) 7 SCC 659,
this Court observed :- 2004 AIR SCW 4767

"21. In embarking upon the minutest re-examination of the whole evidence at the
revisional stage, the learned Judge of the High Court was totally oblivious of the self-
restraint that he was required to exercise in a revision under Section 397, Cr. P.C. On
behalf of the accused, reliance is placed on the decision of this Court to which one of us
(Justice Sabharwal) is a party i.e. Ram Briksh Singh v. Ambika Yadav. That was the case
in which the High Court interfered in revision because material evidence was overlooked
by the courts below." 2004 AIR SCW 3626

The judgment of Ram Briksh mentioned above, has since been reported as Ram Briksh
Singh vs. Ambika Yadav (2004) 7 SCC 665, wherein it has been observed :- 2004
AIR SCW 3626

"12. For the aforesaid reasons, we are unable to accept the contention that the High Court
has reappreciated the evidence. The High Court has only demonstrated as to how the
material evidence has been overlooked leading to manifest illegality resulting in gross
miscarriage of justice."
It was, therefore, relevant in the fact-situation obtaining therein.

Yet again in Satyajit Banerjee vs. State of W.B. (2005) 1 SCC 115, this Court has, while
exercising its jurisdiction under Section 142 of the Constitution of India, expressed a note
of caution stating :- 2004 AIR SCW 6646, Para 21

"22. The cases cited by the learned counsel show the settled legal position that the
revisional jurisdiction, at the instance of the complainant, has to be exercised by the High
Court only in very exceptional cases where the High Court finds defect of procedure or
manifest error of law resulting in flagrant miscarriage of justice."
20. We may notice that prohibition contained in sub-section (3) of Section 401 refers to a
finding and not the conclusion.
A bare perusal of the judgment of the High Court clearly demonstrates that in effect and
substance the finding of the learned trial Judge has been reversed. While hearing the
matter afresh in terms of the direction of the High Court, the learned Trial Judge would
be bound by the observations made therein and thus, would have no option but to convict
the appellants.
21. Not only the evidence of the prosecution witnesses has been relied upon and that of
the Autopsy Surgeon has been disbelieved but the Trial Judge has also been asked to
initiate an appropriate proceeding against him.
22. We have, therefore, no hesitation to hold that the High Court exceeded its jurisdiction
in view of the fact that the judgment of the learned Trial Judge could not be
@page-SC1170
termed to be a perverse one.
23. The Trial Court might be wrong as regards analyzing the prosecution evidence but
then it had not relied upon the evidence of the eye-witnesses only having regard to the
opinion of medical expert. The learned Trial Judge considered the plea of alibi on the part
of some of the accused and accepted the same. The High Court did not bestow any
consideration in this behalf. It also failed to take into consideration that even bystanders
have been implicated in the matter.
24. Unfortunately, the High Court did not meet the reasonings of the learned trial Judge
which was its bounden duty.
25. Even the effect of the order dated 18-11-2003 passed by the High Court in the appeal
preferred by Roshan was not taken into consideration.
The said order attained finality. If Roshan was guilty of commission of an offence under
Section 323 of the Indian Penal Code, we fail to see any reason as to how others could be
held guilty for commission of the offence under Section 302 thereof.
In any event, the judgment passed in favour of Roshan could not have been set aside
indirectly which could not be done directly.
26. For the reasons abovementioned, the impugned judgment cannot be sustained, which
is set aside accordingly. The appeal is allowed.
Appeal allowed.
AIR 2008 SUPREME COURT 1170 "Rajendran v. Shankar Sundaram"
(From : AIR 2004 Madras 339)
Coram : 2 S. B. SINHA AND H. S. BEDI, JJ.
Civil Appeal No. 802 of 2008 (arising out of SLP (C) No. 22880 of 2004), D/- 30 -1
-2008.
Rajendran and Ors. v. Shankar Sundaram and Ors.
Civil P.C. (5 of 1908), O.38, R.5 - ATTACHMENT - JUDGMENT - PARTNERSHIP -
Attachment before judgment - Passing of Order - Formation of prima facie opinion by
Court is sufficient - Suit for recovery against Firm and its partners - Cheque admittedly
issued by plaintiff to Firm - Pronote executed by one of partners - Order directing
partners to furnish security or suffer attachment before judgment without going into
merits of contention - Not improper. (Paras 11, 13)

K. Ramamurthy, Sr. Advocate, Mrs. Kawaljit Kochar, Rishi Dewan and Ms. Kusum
Chaudhary, for Appellants; Amit Sharma and B.V. Anupam Lal Das, for Respondent.
Judgement
1. S. B. SINHA, J.:-Leave granted.
2. Appellants herein were defendant Nos. 4 to 7 in the suit. Plaintiff-respondent No. 1
filed the suit against them and four others. They are admittedly partners of defendant No.
1 - Firm, M/s. AR. AS and P.V.PV , registered under the Partnership Act, 1932. Defendant
No. 3 P. Shankar (Respondent No. 4 herein) was also a partner in the said Firm.
3. Allegedly, Defendant No. 2, P.V. Purushothaman (Respondent No. 3 herein), who has
been described as the Managing Partner of the said Firm, fraudulently obtained an
advance from the plaintiff where for a personal guarantee was furnished by the defendant
No. 2. Indisputably a cheque for a sum of Rs. 50 lakhs was issued in the name of the
defendant No. 1.
4. Plaintiff-Respondent filed the aforementioned suit for realisation of a sum of Rs.
70,30,000/- with interest @ 20% per annum inter alia alleging that all the defendants
were jointly and severally liable therefor. An application under Order XXXVIII Rule 5 of
the Code of Civil Procedure was filed by the plaintiff.
5. Appellants in their written statement inter alia raised a contention that since the amount
of Rs. 50 lakhs purported to have been taken in advance by defendant No. 2 in
connivance with defendant Nos. 3 and 8 had not been used for the benefit of the
partnership firm, no order of attachment could be issued as against the appellants herein.
The said contention of the appellants was accepted by a learned Single Judge of the High
Court by his order dated 10th December, 2002 opining :-
"The copy of the partnership deed date 01-4-1996 has been filed by the contesting
defendants in the typed set. A perusal of the same clearly disclosed that the 2nd
Defendant was not a partner in the 1st defendant Firm. Moreover, the plaintiff had also
not filed any record to show that the 2nd defendant was already in a partner (sic) in the
1st defendant Firm and the borrowal was also made only for the Firm. Unless and until, it
is established by the plaintiff, I am
@page-SC1171
of the view that the plaintiff is not entitled to seek any interim order calling upon the
defendants to execute a security."
6. An intra-court appeal was preferred there against wherein a Division Bench of the
High Court by reason of the impugned judgment opined :-
"The Learned Judge has not appreciated that the 3rd Defendant who is the partner of the
firm as per the partnership deed dated 1-4-1996 executed the promissory notes and clause
10 of the partnership firm gives power to a partner to borrow monies (sic) from the 3rd
parties for the purpose of business. The 2nd Defendant gave a letter which is only for
personal guarantee. So, the reasonings given by the Leaned Judge that since the 2nd
defendant is not a partner, the borrowal of money is not for the benefit of the partnership
cannot be countenanced. When the cheque was given in the name of the firm by the
Plaintiff, prima facie, it has to be taken that it is borrowed on behalf of the partnership
firm. When the payment of the money by the Plaintiff in the firm is not in dispute and in
the absence of any specific allegation that the amount was paid personally to the
defendants, 2, 3 and 8, though the cheque was issued in the name of the firma and the
Plaintiff also colluded with them, the argument of the Learned Counsel regarding the
alleged collusion cannot be accepted. Whether the amount is used for the firm or
personally by the defendants 2, 3 and 8 can be gone into only after adducing evidence.
Prima facie, we find that since the amount was paid in the name of the firm and
promissory notes were executed by the partners of the firm and no other partnership deed
is produced before the Court other than the partnership dated 1-4-1996, the learned Judge
is not correct in rejecting the Application as if the plaintiff has no prima facie case. The
learned Judge has not given any other finding as to the necessity for attachment, but
rejected the application only on the ground that the 2nd defendant is not the partner of the
Firm."
On the said findings the appeal preferred by the plaintiff-respondent was allowed.
Appellants are thus before us.
7. Appellants are, thus, before us.
8. Mr. Ramamurthy, learned senior counsel appearing on behalf of the appellants, would
take us through the plaint as well as the written statement to contend that from a perusal
thereof it would appear that in obtaining the said purported loan from the plaintiff-
respondent, defendant Nos. 2, 3 and 8 played a prime role. As defendant No. 2 was stated
to be the Managing Partner of the Firm, which he was not, and in fact only his son
(defendant No. 3) was a partner, the purported loan was granted by the plaintiff without
even caring to ascertain as to who are the partners of the said Firm.
9. Our attention was furthermore drawn to various provisions of the Partnership Act and
in particular, Section 2(a); Section 18; Section 19; Section 22 and Section 28 thereof for
advancing the proposition that the Firm would be bound only when a transaction is
entered into by a partner of the Firm and that too subject to the limitations contained in
the aforementioned provisions.
10. Mr. Amit Sharma, learned counsel appearing on behalf of the respondents, on the
other hand supported the impugned judgment.
11. Concededly, the amount of loan was advanced by a cheque. The said cheque was
drawn in the name of the partnership Firm. Concededly again, the appellants were the
partners thereof at the relevant time, although an endeavour was made before the learned
Single Judge to show that they ceased to be so. Having regard to the fact that they
purported to have retired from the partnership Firm in the year 2001 and the transaction
herein between the parties are of the year 2000, prima facie the liability of the appellants
could not have been ignored.
12. The application for attachment before judgment was filed by the plaintiff so as to
protect his interest in the event the suit is decreed. The court exercises, in such a situation,
jurisdiction under Order XXXVIII, Rule 5 of the Code of Civil Procedure. The Division
Bench of the High Court merely directed the appellants herein to furnish security within
the time specified thereunder. It was directed that only on their failure to do so, an order
of attachment of the 2nd item on the schedule to the petition shall be issued.
13. Appellants, in our opinion, are not seriously prejudiced thereby. The court while
exercising its jurisdiction under Order XXXVIII, Rule 5 of the Code of Civil Procedure is
required to form a prima facie opinion at that stage. It need not go into the correctness or
otherwise of all the contentions
@page-SC1172
raised by the parties. A cheque had been issued in the name of the firm. The appellants
are partners thereof. A pronote had been executed by a partner of the Firm. Thus even
under the Partnership Act prima facie the plaintiff could enforce his claim not only as
against the Firm but also as against its partners.
14. Sections 2(a) ; 18 ; 19 ; 22 and 28 to which our attention has been drawn, instead of
assisting the appellants, prima facie assist the plaintiff-respondent. Allegations against
defendant Nos. 2, 3 and 8 are required to be gone into at the hearing of the suit. The
Court at this stage is required only to form a prima facie opinion. The plaintiff is entitled
to secure his interest keeping in view the amount involved in the suit. For the said
purpose a detailed discussion in regard to the question as to whether defendant No. 2 was
a partner or not is not of much relevance.
15. In any view of the matter as the appellants are not seriously prejudiced if they furnish
the security, this, in our opinion, is not a fit case where this Court should exercise its
jurisdiction under Article 136 of the Constitution of India.
16. For the reasons abovementioned this appeal fails and is dismissed. No order as to
costs.
Appeal dismissed.
AIR 2008 SUPREME COURT 1172 "Kashi Ram v. State of Rajasthan"
(From : Rajasthan)
Coram : 2 S. B. SINHA AND DALVEER BHANDARI, JJ.
Criminal Appeal No. 732 of 2002, D/- 28 -1 -2008.
Kashi Ram and Ors. v. State of Rajasthan.
Penal Code (45 of 1860), S.304, Part II - CULPABLE HOMICIDE - PRIVATE
DEFENCE - UNLAWFUL ASSEMBLY - Right of private defence - When available -
Accused-appellants in group of 15 to 20 persons armed with variety of lethal weapons -
Going to land where complainants who were unarmed, were cultivating and attacking
complainants causing fatal injuries - Accused-appellants being aggressor cannot claim
right of private defence - Consequently, question of exceeding right of private defence
also does not arise - Conviction of appellants u/S.304, Part II read with S.149 - No
interference. (Paras 41, 56, 59)
Cases Referred : Chronologtical Paras
2005 AIR SCW 5798 : AIR 2006 SC 302 (Foll.) 41, 44
(2003) 12 SCC 594 (Ref.) 42
1996 AIR SCW 2389 : AIR 1996 SC 1998 : 1996 Cri LJ 2860 (Ref.) 39
1993 AIR SCW 4034 : AIR 1994 SC 127 : 1994 Cri LJ 615 : 1993 All LJ 1355 (Ref.)
38
(1981) 4 SCC 432 (Ref.) 37
1973 NI 96 (Ref.) 46
1972 Cri LJ 661 (SC) (Ref.) 52
AIR 1971 SC 1491 (Ref.) 36
AIR 1970 SC 1079 (Ref.) 48
(1866) 6 WR (Cri) 89 (Ref.) 34
(1864) 1 WR (Cri) 34 (Ref.) 35
Dr. Sushil Balwada, for Appellants; Naveen Kumar Singh, Shashwat Gupta and
Aruneshwar Gupta, for Respondent.
Judgement
DALVEER BHANDARI, J. :- This appeal is directed against the judgment dated 04-02-
2002 in Criminal Appeal No. 826 of 2001 passed by the High Court of judicature for
Rajasthan at Jodhpur.
2. Brief facts, which are necessary to dispose of this appeal are recapitulated as under :-
The land measuring 21 bighas is located in village Bhinan, Tahsil Taranagar and the
ownership of the same was recorded in the name of Smt. Chhoti Devi w/o Budh Singh
Rajput and after her demise, the land was transferred in the name of Balu Singh.
3. The accused, Nanuram submitted an application before the Tahsildar, Taranagar and
disclosed that he had bought the said land on the basis of agreement to sell from Smt.
Chhoti Devi at a consideration of Rs. 1200/- and he is in possession of the land and is
cultivating the same. It was alleged that the transfer in the name of Balu Singh had been
wrongly recorded in the revenue records. The Tahsildar, after some enquiry cancelled the
entry of transfer recorded in the name of Balu Singh.
4. On 13th June, 1999 at about 10 a.m., the complainant party consisting of Amar Singh
PW4, his father Balu Singh (since deceased), Bahadur Singh PW8, Nanuram Nai PW1
and Prithvi Singh PW17 went to cultivate Khasra No. 512 situated in village Bhinan
Tahsil, Taranagar District Churu. At that time, the accused persons were not there but on
learning about the presence of the complainant party in Khasra No. 512 around 12 noon
on the same day, the accused
@page-SC1173
party consisting of Nanuram accused-appellant along with the acquitted 6 persons came
from the side of village, armed with gandasa, lathis and axes and attacked the members of
the complainant party and caused serious injuries to Amar Singh PW4, Nanuram Nai
PW1 and Balu Singh. Balu Singh succumbed to those injuries in the hospital on the same
day at 6 p.m.
5. Amar Singh PW4 lodged the first information report. The accused persons were
apprehended and on their voluntary disclosure statements, lathis, gandasa and axes were
recovered and after usual examination, they were charged under section 302 read with
sections 149, 148 and 323, IPC. The accused-appellants in their statements under section
313 of the Code of Criminal Procedure denied all the incriminating evidence and pleaded
that they were in possession of the agricultural land and the complainant party wanted to
dispossess them forcibly. In the process of protecting the possession of their land, a
scuffle between the parties took place. Amar Singh PW4 and Balu Singh from the side of
the complainant party received injuries and Gopiram from the side of accused-appellants
also received injuries.
6. According to the members of the complainant party, they were totally unarmed at the
time of the incident and the accused persons who were armed with lathis, gandasa and
axes had inflicted serious injuries on them. The injuries on the person of Balu Singh were
medically examined. The doctor found the following external injuries :
(1) lacerated wound 6 cm x 1 cm x bone deep on vertex of skull,
(2) lacerated wound 5 cm x bone deep in the right frontal prominence region,
(3) lacerated wound 3 cm x 1 cm x bone deep on occipital region of head, and
(4) four abrasions on right middle leg, left knee and posterior region of left leg.
7. All the aforesaid injuries were found to have been caused with blunt weapon and X-ray
was advised in respect of three lacerated wounds.
8. On the post-mortem of Balu Singh's body, it was revealed that apart from abrasions,
three lacerated wounds, haematoma was present and the fracture of bone was detected.
The brain was squeezed. In the opinion of doctor, cause of death of Balu Singh was shock
due to aforesaid three lacerated injuries on his person.
9. On the head of Amar Singh four lacerated wounds on left parietal region, middle of
forehead, right leg and two other lacerated wounds and middle region of left leg were
found by the doctor. According to the doctor, these injuries were caused by a blunt
weapon.
10. On Nanuram, lacerated wound on occipital region of head, upper left near ear region
respectively and contusion on left shoulder were found. All the above three injuries were
caused by a blunt weapon. Gandasa, lathis and other weapons of offence were recovered
at the instance of the accused-appellants. Blood-stained clothes of the deceased-Balu
Singh were seized by the police and clothes, earth etc. were sent to Forensic Laboratory
for examination. In the serological examination human blood was detected in the blood-
stained earth and on the deceased's shirt, dhoti and baniyan, however, no blood was found
on the weapons recovered by the police. In the formal investigation of the case, no case
was made out against Sri Chand, Dula Ram, Lilu Ram and Pappu and charge-sheet
against the remaining 11 accused persons was filed in the court of the learned Judicial
Magistrate, Taranagar. On committal, the case was sent to the Court of Session.
11. The prosecution, in order to support and strengthen its case has examined 25
witnesses and placed reliance on 78 documents on record. The statements of the accused
persons were recorded under section 313, Cr.P.C. wherein the accused denied the
prosecution version and claimed themselves innocent and asserted that a false case has
been made out against them. It was asserted by the appellants that Nanuram and
Kashiram bought the disputed land from Smt. Chhoti Devi through agreement to sell
dated 23-4-1965 and since then Nanuram has been in possession and was paying land
revenue. It was further submitted that on 13-9-1999, on the basis of the information
received that Balu Singh and his sons along with other 15-20 persons went to their field
(Khasra No. 512) on a tractor with the intention to take forcible possession of the field by
cultivating it. About 100-150 people of village Bhinan went to stop them from doing so.
They were armed with variety of weapons. They inflicted serious injuries on Amar Singh
and Balu Singh.
12. The defence has produced DW1 Dr. Haleef, DW2 Mahender Singh and DW3
@page-SC1174
Nanuram. In the documentary evidence, extracts of statements of witnesses Nanuram,
Mohan Kunwar, Amar Singh, Bhawan Singh, Moti Ram Patwari, Bhanwar Singh and
written report by Dr. Mahesh Panwar to the SHO Police Station Taranagar, letter of SHO
and injury report of Gopiram and copies of traced out site plans have been produced.
13. The prosecution mainly relied on PW2 Lal Chand, PW5 Het Ram, PW6 Lilu Ram,
PW7 Moman Ram, PW12 Gulab Singh, PW13 Moti Ram, PW14 Manohar Lal and PW23
Pala Ram, investigating officer.
14. According to the investigating officer, the accused-appellants were in possession of
the field where the occurrence took place. The complainant party went to this field with
the intention to take its possession. The members of the complainant party were asked not
to ply the tractor on the field. Despite the resistance the field was cultivated by the
complainant party. On learning that the complainant parry was cultivating Khasra No.
512, the accused appellants in a group of 15-20 people fully armed with different
weapons, reached the said Khasra and attacked the complainant party. The case of the
appellants as culled out from evidence is that the accused appellants were compelled to
use force in order to protect the lives and property and their case is fully covered by the
right of private defence. In this view of the matter, presence of the accused appellants
cannot be doubted.
15. The entire evidence on record had been scrutinized in detail by the learned Additional
Sessions Judge. On evaluation of the entire evidence it has been fully established by the
learned Additional Sessions Judge that the fatal injuries were inflicted by Kashiram and
other serious injuries were caused by Dharam Pal, Jagdish and Rupa Ram on the persons
of Balu Singh and Amar Singh in furtherance of their common object of killing the
members of the complainant party.
16. The trial court acquitted six accused and convicted five accused appellants.
17. From the analysis of the evidence by the trial court, it is abundantly clear that the
accused appellants were in possession of Khasra No. 512. The complainant party had
gone to cultivate the said Khasra at 10 a.m. on 13th June, 1999. At that time, the accused
appellants were not there but on learning that the complainant party was cultivating the
field, they reached there armed with varieties of weapons and caused serious injuries on
the members of the complainant party. Admittedly, the members of the complainant party
were totally unarmed. The appellants were responsible for causing fatal injury on Balu
Singh and other serious injuries on Amar Singh and Nanuram. According to the findings
of the Sessions Court, the accused appellants had exceeded the right of private defence.
18. Kashiram was convicted under Section 304 Part-II and was sentenced to 5 years'
rigorous imprisonment. Other 4 accused, namely, Dharam Pal, Jagdish, Rupa Ram and
Om Prakash inflicted injuries on Amar Singh and Nanuram were convicted under Section
304 Part-II read with Section 149, IPC and they were also sentenced to 5 years' rigorous
imprisonment. They were also convicted under Section 323, IPC.
19. The High Court again examined the entire evidence and came to a clear conclusion
that the accused appellants had exceeded in their right of private defence. They caused
serious injuries to Balu Singh which proved fatal. They also caused serious injuries to
Amar Singh and Nanuram. Injuries of such serious nature were totally unwarranted
because the members of the complainant party were totally unarmed.
20. The finding of the High Court regarding accused appellants' private defence reads as
under :-
"Therefore, the learned trial court has rightly held that the accused persons have exceeded
their right of private defence of property."
21. The High Court also came to the conclusion that in the facts and circumstances the
trial court has correctly evaluated the entire evidence on record and has taken a very
lenient view. The High Court did not find any mitigating circumstance to interfere with
the quantum of sentence.
22. The appellants aggrieved by the said judgment of the High Court have preferred this
appeal before this Court.
23. It was submitted by the learned counsel appearing for the appellants that the High
Court failed to appreciate that the disputed land was in possession of the accused persons
and the complainant party came to their field to dispossess them and their acts, if any, are
fully covered by the right of self
@page-SC1175
defence. It is also submitted that the appellants had filed a suit against the complainant
party prior to this incident and an injunction was granted against the complainant party by
the Revenue Court on 10-5-1999 and it was found that the accused appellants were in
possession of the disputed land.
24. The appellants also submitted that it is a case of over implication because of previous
enmity. According to the appellants, since they were in possession of the land in dispute,
therefore, no offence under Section 304 Part-II, IPC can be made out against them.
25. We have heard the learned counsel for the appellants and the State. We have also
perused the judgment of the trial court and the record of the case. The Sessions Court and
the High Court found that the appellants were in possession of Khasra No. 512 and the
complainant party at about 10 a.m. on 13th June, 1999 went to cultivate Khasra No. 512.
The appellants were not there. The appellants learnt that the members of the complainant
party were cultivating the said field, the accused appellants armed with gandasa, lathis
and axes came to the field and assaulted the members of the complainant party when they
were unarmed. Appellant Kashiram inflicted gandasa blow on Balu Singh from the
reverse side and that injury proved fatal. The gandasa has been recovered at the instance
of Kashiram. According to the report of the Chemical Examiner, human blood was
detected from the blood-stained clothes of the deceased. The earth collected from the spot
also contained human blood. Since the appellant Kashiram did not use the front side of
gandasa, therefore, the trial Court instead of convicting him under Section 302, IPC
convicted him under Section 304 Part-II, IPC. In view of our finding that the appellants
were in possession of Khasra No. 512 and the appellants had gone to take back
possession of Khasra No. 512 from the members of the complainant party, had inflicted
fatal blow on Babu Singh and other serious injuries on the members of the complainant
party.
26. The question which arises for our adjudication is that in the facts and circumstances
of this case whether the accused appellants are protected by the right of private defence
as enumerated by section 96 of the Indian Penal Code.
27. Sections 96 to 106 deal with various facets of the right of private defence. Before
determining the controversy in this case, we deem it proper to deal with these provisions
in brief.
Section 96, IPC reads as under :
"96. Things done in private defence. -
Nothing is an offence which is done in the exercise of the right of private defence."
28. Section 97 of IPC gives right to a person to defend his body and the property. But,
this right is subject to restrictions contained in section 99. Section 99 IPC reads as
under :-
"99. Acts against which there is no right of private defence. - There is no right of private
defence against an act which does not reasonably cause the apprehension of death or of
grievous hurt, if done, or attempted to be done, by a public servant acting in good faith
under colour of his office, though that act, may not be strictly justifiable by law.
There is no right of private defence against an act which does not reasonably cause the
apprehension of death or of grievous hurt, if done or attempted to be done, by the
direction of a public servant acting in good faith under colour of his office, though that
direction may not be strictly justifiable by law.
There is no right of private defence in cases in which there is time to have recourse to the
protection of the public authorities.
Extent to which the right may be exercised. - The right of private defence in no case
extends to the inflicting of more harm than it is necessary to inflict for the purpose of
defence."
29. The main question that arises for adjudication in this case is whether the accused
appellants had right of private defence and this is the case of exceeding the right of
private defence meaning thereby, inflicting more harm than it was necessary for the
purpose of defence.
30. Section 100 of the Indian Penal Code deals with a situation when the right of private
defence of the body extends of causing death. The relevant portion of the section reads as
under :-
" 100 - When the right of private defence of the body extends to causing death - The right
of private defence of the body extends, under the restrictions mentioned in the last
preceding section, to the voluntary causing
@page-SC1176
of death or of any other harm to the assailant, if the offence which occasions the exercise
of the right be of any of the descriptions hereinafter enumerated, namely :-
First. - Such an assault as may reasonably cause the apprehension that death will
otherwise be the consequence of such assault;
Secondly. - Such an assault as may reasonably cause the apprehension that grievous hurt
will otherwise be the consequence of such assault;

Thirdly. - xxx xxx xxx


Forthly. - xxx xxx xxx

Fifthly.- xxx xxx xxx

Sixthly.- xxx xxx xxx"

31. Section 103, IPC deals with a situation when the right of private defence of property
extends to causing death. Section 103, IPC reads as under :-
" 103. When the right of private defence of property extends to causing death. - The right
of private defence of property extends, under the restrictions mentioned in section 99, to
the voluntary causing of death or of any other harm to the wrong-doer, if the offence, the
committing of which, or the attempting to commit which, occasions the exercise of the
right, be an offence of any of the descriptions hereinafter enumerated, ........"
32. Admittedly, the members of the complainant party were totally unarmed. Even if the
case of the accused appellants is accepted in toto that in order to take back the possession
of Khasra No. 512 some injuries were inflicted but the act of the appellants in causing
death cannot be covered by the ambit of Section, 96 IPC. According to the findings of
courts below, it was clearly a case of exceeding the right of private defence. The
appellants indeed inflicted more harm than it was necessary for the purpose of defence.
33. The right of private defence is codified in Sections 97 to 106 of the Indian Penal Code
and all these sections will have to be read together to ascertain whether in the facts and
circumstances the accused appellants are entitled to right of private defence or they
exceeded the right of private defence. Only when all these sections are read together, we
get comprehensive view of the scope and limitation of that right. The position of law is
well-settled for over a century both in England and India.
34. Almost 150 years ago in Queen v. Fuzza Meeah alias Fuzza Mahomed (1866) 6 WR
(Cr) 89 because of exceeding the right of private defence, the appellants were convicted,
but the sentence of imprisonment was reduced.
35. In another case decided during the same period in Queen v. Shunker Sing, Kukhoor
Sing (1864) 1 WR (Cr) 34, the court for exceeding the right of private defence convicted
the accused and reduced the sentence.
36

. This court also on several occasions dealt with the cases of exceeding the right of
private defence. In The Munney Khan v. State of Madhya Pradesh, (1970) 2 SCC 480,
this court for exceeding the right of private defence converted the sentence of the accused
appellant from under Section 302, IPC to Section 304, IPC. The relevant portion of the
judgment reads as under :- AIR 1971 SC 1491, Paras 4 and 5
"Such a right of private defence is governed by Section 101, I.P.C. and is subject to two
limitations. One is that, in exercise of this right of private defence, any kind of hurt can
be caused, but not death; and the other is that the use of force does not exceed the
minimum required to save the person in whose defence the force is used. In these
circumstances, in the present case, when Zulfiquar was being given fist blows only, there
could be no justification at all for the appellant to stab Reotisingh with a knife and
particularly to give him a blow which could prove fatal by aiming it on his back. The use
of the knife itself was in excess of the right of private defence and it became much more
excessive when the blow with the knife was given on a vital part of the body which, in
the ordinary course of nature, was likely to cause the death of Reotisingh. From the fact
that the blow was given in the back with a knife an inference follows that the appellant
intended to cause death or at least intended to cause such injury as would, in the ordinary
course of nature, result in his death. In adopting this course, the appellant would have
been clearly guilty of the offence of murder had there been no right of private defence of
Zulfiquar at all. Since such a right did exist, the case would fall under the exception under
which culpable homicide does not amount to murder on the ground that the death was
caused in exercise of right of private
@page-SC1177
defence, but by exceeding that right. An offence of this nature is made punishable under
the first part of Section 304, I.P.C. Consequently, the conviction of the appellant must be
under that provision and not under Section 302, I.P.C.
As a result, the appeal is partly allowed, the conviction under Section 302, I.P.C. is set
aside, and the appellant is convicted instead under the first part of Section 304, I.P.C. In
view of the change in the offence for which the appellant is being punished, we set aside
the sentence of imprisonment for life, and instead, award him a sentence of seven years
rigorous imprisonment".
37. In Balmukund and Another v. State of Madhya Pradesh, (1981) 4 SCC 432 this court
while dealing with the facts of similar nature converted the conviction from section 302,
IPC to Section 304, IPC. Relevant observations of the Court reads as under :-
"In rural landscape even today dispute as to possession of agricultural land is a part of
life. Occupancy of land being the only source of survival, emotional attachment apart, the
struggle for survival leads to fierce fight and resort to arms to protect possession because
in the context of tardy slow moving litigative process actual possession has ceased to be
mere nine point in law but it has assumed alarming proportions. Years upon years spent in
legal conundrums moving vertically through hierarchy of courts coupled with the cost
and time to throw out a trespasser or even a rank trespasser provides occasionally
provocation to resort to physical violence. The use of the firearm used to be spasmodic
but it has started becoming a recurring, malady. But right of private defence cannot be
judged step by step or in golden scales. Once we accept the finding of the High Court that
the appellants had the right of private defence of person and property meaning thereby
that the appellants were the victims and the complainants were aggressors, but in the facts
of the case they exceeded the same by wielding a firearm, a sentence of 10 years rigorous
imprisonment would appear to us in the facts and circumstances of the case to be a little
bit too harsh.
Having given our earnest consideration to the question of sentence alone in this case, we
are of the opinion that Balmukund, Appellant 1, should be sentenced to rigorous
imprisonment for five years, and simultaneously the sentence of seven years under
Section 307, Indian Penal Code awarded to Appellants 1 and 2 both be reduced to three
years each. The substantive sentences should run concurrently."
38

. In another case, while dealing with a case of self defence in Dharam Pal and Others v.
State of U.P., 1994 Supp (3) SCC 668, this court for exceeding the right of private
defence instead of convicting the accused appellant under Section 302 read with Section
149, IPC, converted the sentence under Section 304 Part-I, IPC. 1993 AIR SCW 4034

39

. In Mahabir Choudhary v. State of Bihar, (1996) 5 SCC 107, this Court held that the
High Court erred in holding that the appellants had no right of private defence at any
stage. However, this Court upheld the judgment of the Sessions Court holding that since
the appellants had right of private defence to protect their property, but in the
circumstances of the case, the appellants had exceeded their right of private defence and
were, therefore, rightly convicted by the trial court under section 304 Part-I. The court
observed that the right of private defence cannot be used to kill the wrongdoer unless the
person concerned has a reasonable cause to fear that otherwise death or grievous hurt
might ensue in which case that person would have full measure of right of private
defence including killing. 1996 AIR SCW 2389

40. We have examined the cases of exceeding of the right of private defence. In the
instant case, both the Sessions Court and the High Court came to the conclusion that the
accused appellants were guilty of exceeding the right of private defence and instead of
convicting them under Section 302 convicted them under Section 304 Part-II along with
149, IPC.
41

. Both the Sessions Court and the High Court clearly came to the conclusion that the
accused appellants in a group of 15-20 people armed with variety of weapons had gone to
Khasra No. 512 where the complainant party was cultivating. The accused appellants in
order to dispossess the members of the complainant party attacked them and caused
serious injuries to the members of the complainant party in which Balu Singh died.
Admittedly, the members of the complainant party were totally unarmed. From perusal of
the entire evidence on record, it is abundantly clear that the accused appellants were the
aggressor and they attacked 2005 AIR SCW 5798

@page-SC1178
the complainant party when they were totally unarmed. It is settled legal position that the
right of private defence cannot be claimed when the accused are aggressors particularly
when the members of the complainant party were totally unarmed. This Court in the
recent judgment in Bishna alias Bhiswasdeb Mahato and Others v. State of West Bengal,
(2005) 12 SCC 657 exhaustively dealt with this aspect of the matter. The facts of this case
are akin to the facts of the instant cases. In this case, the Court while relying on the earlier
judgments of this Court, clearly came to the conclusion that the right of private defence
cannot be claimed when the accused is an aggressor.
42. In the said case, this Court relied on Preetam Singh v. State of Rajasthan, (2003) 12
SCC 594. In this case, the Court clearly held that the appellants were the aggressors,
therefore, the question of the appellants having the right of private defence or exceeding
it does not arise. The plea of private defence is not at all available to the appellants.
43. In the instant case, the appellants were the aggressor. They inflicted serious injuries
on the unarmed complainant party by a variety of weapons causing the death of Balu
Singh and also inflicted serious injuries on other members of the complainant party.
44

. Private defence can be used only to ward off unlawful, force, to prevent unlawful force,
to avoid unlawful detention and to escape from such detention as held by this Court in
Bishna's case (supra). In the said judgment the relevant portion of Kenny's Outlines of
Criminal Law and Criminal Law by J.C. Smith and Brian Hogan have been quoted. We
deem it appropriate to reproduce the same. 2005 AIR SCW 5798

"It is natural that a man who is attacked should resist, and his resistance, as such, will not
be unlawful. It is not necessary that he should wait to be actually struck, before striking in
self-defence. If one party raises up a threatening hand, then the other may strike. Nor is
the right of defence limited to the particular person assailed; it includes all who are under
any obligation, even though merely social and not legal, to protect him. The old
authorities exemplify this by the cases of a husband defending his wife, a child his parent,
a master his servant, or a servant his master (and perhaps the courts would now take a
still more general view of this duty of the strong to protect the weak)."
45. The learned author further stated that self-defence, however, is not extended to
unlawful force :
"But the justification covers only blows struck in sheer self-defence and not in revenge.
Accordingly if, when all the danger is over and no more blows are really needed for
defence, the defender nevertheless strikes one, he commits an assault and battery. The
numerous decisions that have been given as to the kind of weapons that may lawfully be
used to repel an assailant, are merely applications of this simple principle. Thus, as we
have already seen, where a person is attacked in such a way that his life is in danger he is
justified in even killing his assailant to prevent the felony. But an ordinary assault must
not be thus met by the use of firearms or other deadly weapons........."
46. In Browne, 1973 NI 96 (NI at p. 107] Lowry, L.C.J. with regard to self-defence stated
:
"The need to act must not have been created by conduct of the accused in the immediate
context of the incident which was likely or intended to give rise to that need."
47. As regards self-defence and prevention of crime in Criminal Law by J.C. Smith and
Brian Hogan, it is stated :
"Since self-defence may afford a defence to murder, obviously it may do so to lesser
offences against the person and subject to similar conditions. The matter is now regulated
by Section 3 of the Criminal Law Act, 1967. An attack which would not justify D in
killing might justify him in the use of some less degree of force, and so afford a defence
to a charge of wounding, or, a fortiori, common assault. But the use of greater force than
is reasonable to repel the attack will result in liability to conviction for common assault,
or whatever offence the degree of harm caused and intended warrants. Reasonable force
may be used in defence of property so that D was not guilty of an assault when he struck
a bailiff who was unlawfully using force to enter D's home. Similar principles apply to
force used in the prevention of crime."
48

. The right of private defence is a very valuable right and it has been recognized inAIR
1970 SC 1079

@page-SC1179
all free, civilized and democratic societies within certain reasonable limits (See Gottipulla
Venkatasiva Subbrayanam and Others v. The State of Andhra Pradesh and Another,
(1970) 1 SCC 235.)
49. Russel in his celebrated book on Crimes (11th Edn.) p.491 has stated :-
"A man is justified in resisting by force any one who manifestly intends and endeavours
by violence or surprise to commit a known felony against his person, habitation or
property. In these cases he is not obliged to retreat and not merely to resist the attack
where he stands but may indeed pursue his adversary until the danger is ended. If and in a
conflict between them he happens to kill his attacker such killing is justifiable."
50. Blackstone [Commentaries Book 4; P. 185] also observed as under :-
The party assaulted must, therefore, flee as far as he conveniently can either by reason of
some wall, ditch, or some other impediment; or as far as the fierceness of the assault will
permit him; for it may be so fierce as not to yield a step, without manifest danger of his
life, or enormous bodily harm; and then in his defence he may kill his assailant
instantaneously. And this is the doctrine of universal justice, as well as of the municipal
law." (Emphasis supplied).
51. Halsbury's Laws of England, Fourth Edition, Vol. 11 pp. 630-631 dealt with self-
defence and defence of property. The relevant portion in paras 1180-1181 reads as
under :-
"1180. Self-defence. A person acting in self-defence is normally acting to prevent the
commission of a crime, as is a person acting in defence of another. The test to be applied
in such cases is now established to be the same as for cases of prevention of crime, that is
the force used in self-defence or in defence of another must be reasonable in the
circumstances.
Provided the force used is reasonable a person is entitled to defence not only himself or a
member of his family, but even a complete stranger if the stranger is subject to unlawful
attack by others.
In deciding whether the force used was reasonable, all the circumstances may be
considered. The matter is one of fact and not one of law, hence it cannot be ruled that a
person who is attacked must retreat before retaliating. A persons opportunity to retreat
with safety is a factor to be taken into account in deciding whether his conduct was
reasonable, as is his willingness to temporize or disengage himself before resorting to
force. A man is not obliged to refrain from going where he may lawfully go because he
has reason to believe that he may be attacked, and is not thereby deprived of his right of
self-defence.
1181. Defence of property. Where a person in defending his property is also acting in the
prevention of crime then he may use such force as is reasonable in the circumstances.
Where no crime is involved, as where there is merely a trespass, the same rule of
reasonable force in the circumstances is applicable. If in using reasonable force the
defendant should accidentally kill another, the killing would not amount to murder or
man-slaughter. It would not, in general, be reasonable to kill in defence of property alone,
although it has been held that a man may lawfully kill, a trespasser who would forcibly
dispossess him of his house."
52

. In Mohammad Khan and Others v. State of Madhya Pradesh (1971) 3 SCC 683 in para
11, this Court has rightly concluded that the right of self-defence only arises if the
apprehension is unexpected and one is taken unawares. If one enters into an inevitable
danger with the fullest intimation beforehand and goes there armed to fight out, the right
cannot be claimed. 1972 Cri LJ 661

53. Careful analysis of the right of private defence as codified in Sections 96 to 106, IPC
and the legal position as crystallized by a number of judgments leads to an irresistible
conclusion that the findings of the Sessions Court as upheld by the High Court in the
instant case regarding the appellants exceeding the right of private defence are wholly
erroneous and untenable.
54. The right of private defence is purely preventive and not punitive. This right is
available only to ward off the danger of being attacked; the danger must be imminent and
very real and it cannot be averted by a counter-attack.
55. In view of the facts of this case, the accused appellants did not have the right of
private defence. Therefore, they cannot legitimately claim any benefit by invoking the
@page-SC1180
principle of right of private defence.
56. The acts of the accused appellants of proceeding to a definite destination with lethal
weapons and thereafter causing serious injuries including fatal injuries on the unarmed
members of the complainant party can never legitimately claim the benefit of the
provisions of the right of private defence. Since the accused appellants did not have the
right of private defence, therefore, the findings of the courts below regarding their
exceeding the right of private defence cannot be sustained and are accordingly set aside.
57. Since there is no appeal by the State against acquittal of the accused appellants under
Sections 302, IPC, therefore it is not necessary for us to deal with the aspect whether their
acquittal under Section 302 was justified or not.
58. The Sessions Court convicted accused Kashiram under Section 304 Part-II and the
other appellants under Section 304 Part-II read with Section 149, IPC. In the impugned
judgment the High Court has upheld their conviction.
59. On consideration of the peculiar facts and circumstances of the case the conviction
and sentence of the accused appellants as recorded by the courts below do not warrant
any interference. The appeal being devoid of any merit is accordingly dismissed.
60. The accused appellants are directed to surrender forthwith to suffer the remaining
sentence.
Appeal dismissed.
AIR 2008 SUPREME COURT 1180 "K. T. Varghese v. State of Kerala"
(From : Kerala)*
Coram : 3 Dr. A. PASAYAT, LOKESHWAR SINGH PANTA AND P. SATHASIVAM,
JJ.
Civil Appeal No. 6456 of 2001, D/- 24 -1 -2008.
K.T. Varghese and Ors. v. State of Kerala and Ors.
Mines and Minerals (Regulation and Development) Act (67 of 1957), S.15 - Kerala
Minor Mineral Concession Rules (1967), R.48C - MINES AND MINERALS - LICENSE
- CO-OPERATIVE SOCIETIES - Dealers' licence - Restrictions incorporated therein -
Licensee required to purchase mineral for stock from authorised quarrying permit holders
only - Permitted to sell minerals only within State of Kerala that too for domestic and
agricultural purposes - However Co-operative Societies are not saddled with any such
restrictions - Imposition of said conditions, held not proper in view of 1995 AIR SCW
657 and therefore, struck down. (Para 13)
Cases Referred : Chronological Paras
1995 AIR SCW 657 : AIR 1995 SC 858 (Foll.) 12, 13
P. Krishnamurthy, Sr. Adv., Romy Chacko, Ahanthem Henry, Arpit Gupta, Rajiv Mehta,
for Appellants; G. Prakash, Ms. Beena Prakash, for Respondents.
* W.A. No. 2765 of 1998, D/- 15-12-1999 (Ker).
Judgement
1. Dr. ARIJIT PASAYAT, J. :- Challenge in this appeal is to the order passed by a
Division Bench of the Kerala High Court. Challenge before the High Court was to the
order passed by a learned Single Judge dismissing the Original Petition filed.
2. Background facts as projected by the appellants in a nutshell are as follows :
The appellants are engaged in the business of limeshell. They have been holding the
necessary dealers' licence issued under the Kerala Minor Mineral Concession Rules, 1967
(hereinafter referred to as 'the State Rules') under Section 15 of the Mines and Minerals
(Regulation and Development) Act, 1957 (hereinafter referred to as 'the Act') and the
Minor Mineral Concession Rules, 1967 (in short the 'Rules'). The appellants were given
the licence under Rule 48-C of the State Rules. As per the licence the appellants got the
licence to sell, stock and exhibit for sale minor minerals under the Rules. Along with the
licence certain conditions have also been laid down which the appellants are under
obligation to comply with. When the appellants were not granted the renewal of licence
for the period 1997-98, they approached the High Court by filing O.P.No. 14269/1997
which was disposed of by judgment dated 16-2-1998. The appellants filed a Writ Appeal
against the said judgment and the Division Bench of this Court in Writ Appeal No.
547/1998 directed the first respondent to dispose of the representation filed by the
appellants. Accordingly, the appellants were granted renewal of their licences for the
period 1998-99.
3. The appellants' complaint is that certain conditions in the form of restrictions
@page-SC1181
have been incorporated while issuing the licences. One of such conditions which the
appellants attacks is that the minerals permitted to be stocked were to be purchased only
from authorised quarrying permit holders on that behalf. Another condition is that they
are permitted to sell the minerals only within the State of Kerala that too for domestic and
agricultural purposes. The appellants' complaint is that as far as Co-operative Societies
are concerned, they are not saddled with any such restrictions imposed in the case of the
appellants. Thus, according to the appellants, there is a clear discrimination between the
Co-operative Societies and the individuals in the matter of restrictions imposed in the
licences granted to them. Apart from that there is no legal sanction for such restrictions.
4. Learned Single Judge of the High Court was of the view that the licence was granted
subject to certain conditions and restrictions. The State Government was empowered to
impose such conditions under the Act and the State Rules and the licences were issued in
terms of provisions of the Act and the State Rules. Since the conditional licence was
issued, the licencees cannot take up the benefit of licences without the conditions
imposed.
5. The Division Bench in writ appeal did not specifically refer to these aspects.
6. Learned counsel for the appellants submitted that the condition that sales would be for
agricultural purposes and inside the State condition cannot be imposed under the Rules.
7. Learned counsel for the State Government and its functionaries on the other hand
supported the order of the High Court.
8. It appears that the impugned conditions stipulated run as follows :
"While selling limeshell or the products made using the minerals you should give to the
purchaser Cash memorandum authenticated by the undersigned/Assistant Geologist of
this office before use. Please note that any consignment of minor minerals without a valid
cash memorandum shall be considered as illicit and the competent authority or such
authorized person may recover the mineral from the person concerned."
9. It is to be noted that there is no serious challenge to the Condition No. 1.
10. Similarly another condition was imposed which read as follows :
"For sale within Kerala State only for domestic and Agricultural purpose."
11. Primarily it has been contended that no reason has been indicated as to the basis for
imposition of such conditions and there is no such prescription for licencees who were
co-operative societies.
12

. It is to be noted that dealer does not extract the minerals. In State of Tamil Nadu v.
M.P.P. Kavery Chetty [1995 (2) SCC 402] considering a similar challenge it was inter alia
observed as follows : 1995 AIR SCW 657

"17. Rules 8-D and 19-B were introduced into the said Rules by Government Order No.
214 dated 10-6-1992. The two rules are identical, except that Rule 8-D is in Section II
which relates to Government lands in which the minerals belong to the Government and
Rule 19-B is in Section III which relates to ryotwari land in which the minerals belong to
Government. This being so, it is enough to quote Rule 19-B. It reads thus :
"19-B. Constitution of black, red, pink, grey, green, white or other coloured or
multicoloured granites or any rock suitable for use as ornamental and decorative stones
quarried by the permit-holder etc. - (1) Notwithstanding anything contained in these
rules, on and from 10-6-1992 the sale of the quarried black, red, pink, grey, green, white
or other coloured or multi-coloured granites or any rock suitable for use as ornamental
and decorative stone by every permit-holder who has been granted permission by the
State Government and every person who has been permitted by a competent court having
jurisdiction, for quarrying black, red, pink, grey, green, white or other coloured or multi-
coloured granites or any rock suitable for use as ornamental and decorative stone, shall be
regulated by the State Government or by an officer of the State Government or by a State
Government company or by a corporation owned or controlled by the State Government,
as the State Government may direct in this behalf.
(2) Where the above sale is regulated by - (i) the State Government or by an officer of the
State Government, the minimum price shall be as fixed by the State Government;
(ii) the State Government company or a corporation owned or controlled by the State
Government, the minimum price shall be as
@page-SC1182
fixed by the said company or corporation, as the case may be :
Provided that in fixing the minimum price under this sub-rule, the fair market price
prevailing at the time of the sale shall be taken into account".
18. On the same day that Rules 8-D and 19-B were introduced, that is, 10-6-1992,
Government Order No. 216 was also issued. It directed, under the provisions of the two
rules, that the Tamil Nadu Minerals Limited, a State Government company, would
regulate the sale of quarried black, red, pink, grey, green, white or other coloured or
multicoloured granite or any rock suitable for use as ornamental and decorative stones.
19. The High Court quashed Rules 8-D and 19-B principally on the ground that Section
15 of the said Act gave no power to the State Government to frame rules to regulate
internal or foreign trade in granite after it had been quarried. Section 15 also did not
empower the State Government to frame rules to enable a State Government company or
corporation to fix a minimum price for granite.
20. Learned counsel for the appellant State submitted that Rules 8-D and 19-B were valid
having regard to the Preamble of the said Act and Section 18 thereof. He submitted that
the rule-making power of the State under Section 15(o) was wide enough to encompass
Rules 8-D and 19-B.
23. It is difficult to see how granite resources can be protected by controlling the sale of
granite after its excavation and fixing the minimum price thereof.
24. There is no power conferred upon the State Government under the said Act to
exercise control over minor minerals after they have been excavated. The power of the
State Government, as the subordinate rule-making authority, is restricted in the manner
set out in Section 15. The power to control the sale and the sale price of a minor mineral
is not covered by the terms of Clause (o) of sub-section (1-A) of Section 15. This clause
can relate only to the regulation of the grant of quarry and mining leases and other
mineral concessions and it does not confer the power to regulate the sale of already
mined minerals".
13

. In view of what has been stated by this Court in M.P.P. Kavery Chetty's case (supra) the
impugned conditions stipulated could not have been imposed and are accordingly struck
down. 1995 AIR SCW 657

14. The appeal is allowed to the aforesaid extent without any order as to costs.
Appeal allowed.
AIR 2008 SUPREME COURT 1182 "Ramakrishna Rao v. Rai Murari"
(From : Karnataka)*
Coram : 2 Dr. A. PASAYAT AND P. SATHASIVAM, JJ.
Civil Appeal Nos. 454-455 of 2002, D/- 21 -1 -2008.
Ramakrishna Rao (Dead) by L.R. v. Rai Murari.
Specific Relief Act (47 of 1963), S.37 - Civil P.C. (5 of 1908), S.100 - INJUNCTION -
APPEAL - APPELLATE COURT - POSSESSION - Suit for permanent injunction
against dispossession - Defendant taking plea that she was put in possession under
agreement for sale - Appellate Court taking defendant's case as that of plaintiff - On that
basis directing plaintiff to refund amount received with interest - Direction liable to be set
aside.
R. S. A. No. 183 of 1994, D/-13-02-1998 and R. P. No. 856 of 2000, D/-07-03-2001
(Kant), Reversed. (Paras 3, 5)

S.N. Bhat, N.P.S. Panwar, D.P. Chaturvedi, for Appellant; M.M. Kashyap, for
Respondent.
* R.S.A. No. 183 of 1994, D/- 13-2-1998 and R. P. No 856 of 2000, D/- 7-3-2001 (Kant).
Judgement
1. Dr. ARIJIT PASAYAT, J. :-Challenge in these appeals is to the judgment of a learned
single Judge of the Karnataka High Court who while dismissing the second appeal filed
by the respondent has given certain directions which according to the appellant could not
have been given in the absence of any finding.
2. A suit for permanent injunction was filed by the appellant, with the prayer to direct the
defendant, respondent herein from interfering with the peaceful possession and
enjoyment over the suit schedule land. The trial Court dismissed the suit but the first
appellate Court allowed the appeal. The High Court in the second appeal as noted above
dismissed the same but directed payment of certain amounts by the appellants to the
respondent. The directions in this regard read as follows :
"Now the defendant's grievance is that the
@page-SC1183
plaintiff could not have the benefit of the property as well as money paid by the
defendant's wife on 19-6-1983. In equity and in law, inasmuch as agreement is found to
be valid by the first appellate Court, it is the duty of the plaintiff to return the money of
Rs. 5,000/- together with the interest at 12% per annum. Such amount shall be charged in
the property. The plaintiff is directed to pay the same within a period of six months and
the decree of injunction shall come into operation only after payment is made. Subject to
the above direction, the second appeal is dismissed."
3. Subsequently, the figure of Rs. 5,000/-was substituted with a figure of Rs. 32,000/-.
The High Court proceeded on the basis as if the suit for injunction was on the ground that
plaintiff had entered into an agreement with defendant's wife on 19-6-1983. It was further
observed that on the basis of the agreement the defendant's wife claimed to have been put
in possession of the land and the right to property cannot be taken away by the plaintiff
by way of injunction or otherwise. To say the least, this shows complete non application
of mind. This is not the case of the plaintiff in the suit but as a matter of fact it was the
defendant's case. To add to the vulnerability, the High Court found that the agreement
was found to be valid by the first appellate Court and, therefore, it was the duty of the
plaintiff to return the amount with interest at the rate of 12% per annum. Even on a deep
scanning of the first appellate Court's order it is noticed that there was no such finding
recorded. On the contrary, in paragraphs 19 to 21 of the first appellate Court's order, it is
clear that the first appellate Court found that the stand of the plaintiff about the alleged
existence of an agreement for sale was not established. It is relevant to take note of what
has been stated in the aforesaid paragraphs :
"19. In this connection it was urged on behalf of the plaintiff that the document is a
cooked up one as the wife of the defendant was not in a position to afford such a huge
amount. It is an admitted fact that there has been a loan transaction between the plaintiff
and the defendant and for recovery of the said amount a suit has been filed which has
been decreed by the Court of Munsif. The suit had been filed in O. S. No. 118/84.
According to the plaintiff a sum of Rs. 11,500/- was paid in August, 1983 and for non
payment of balance amount O. S. No. 118/84 has been filed against him. It was urged that
the defendant was not in a position to pay the said amount and it was quite impossible to
enter into an agreement for purchase of the land at the rate of Rs. 2,000/- per acre. It is
stated by D.W.2 that his wife who was the owner of the land at Hokrani village has been
sold by her, but neither the wife of the defendant has been examined before the Court nor
any other material is produced to show that she was in a position to pay the amount so as
to purchase the suit land along with non-suit land.
20. It can also be seen here that the plaintiff has denied the execution of the document
and has refused the said transaction. To enforce such transaction no action has been taken
by the wife of the defendant for specific performance of agreement of sale against the
plaintiff though it has come to the knowledge that the said agreement has been refuted by
the plaintiff.
21. The perusal of the document at Ex. D-2 itself goes to show that Sy. No. 715 appearing
in the document at two places has been over written. It has also been mentioned therein
that in the month of May of next year by taking the balance amount, the document has to
be taken. In spite of lapse of time as contended so far no action has been taken against the
plaintiff, so the right of equity accrued to the wife of the defendant and the defendant who
claims through her, has been defeated by lapse of time."
4. No one appears for the respondent.
5. A review petition was filed highlighting these aspects by the appellants, but the same
came to be rejected by the High Court. Since the High Court proceeded on totally
untenable premises and recorded findings which are contrary to materials on record and
findings recorded, the direction for refund of money is clearly indefensible and is set
aside.
6. The appeals are allowed to the aforesaid extent with no order as to costs.
Appeal allowed.
@page-SC1184
AIR 2008 SUPREME COURT 1184 "Sattatiya v. State of Maharashtra"
(From : Bombay)*
Coram : 2 G. P. MATHUR AND G. S. SINGHVI, JJ.
Criminal Appeal No. 579 of 2005, D/- 16 -1 -2008.
Sattatiya @ Satish Rajanna Kartalla v. State of Maharashtra.
Penal Code (45 of 1860), S.300 - MURDER - EVIDENCE - Murder - Cricumstantial
evidence - Evidence of last seen together - Witness to "last seen" untrustworthy - His
evidence full of contradictions - Witness wanted to win favour from police - Statement
allegedly made by accused expressing his desire to facilitate recovery of clothes and
weapon (half blade) - Not recorded - Place from where weapon was recovered was open
place where everybody had access - Blood stains found on recovered articles could not be
linked with blood of deceased - Evidence of person who sold handkerchief to accused
found by side of dead body could not be believed - Held that, prosecution failed to
establish chain of circumstances which could link accused with crime - Accused
acquitted.

Crl. A. No. 797 of 1998, D/-04-04-2003 (Bom.), Reversed. (Paras 19, 22, 23, 26, 27)
Cases Referred : Chronological Paras
2007 AIR SCW 2226 (Foll.) 13
2006 AIR SCW 1602 : AIR 2006 SC 1656 (Foll.) 13, 14
2006 AIR SCW 1637 : AIR 2006 SC 1708 (Foll.) 13
2005 AIR SCW 4620 : AIR 2005 SC 3478 : 2005 Cri LJ 4131 (Foll.) 13
(2005) 12 SCC 438 (Foll.) 13
2003 AIR SCW 770 : AIR 2003 SC 1433 : 2003 Cri LJ 1297 (Foll.) 13, 15
2002 AIR SCW 3655 : AIR 2002 SC 3164 : 2002 Cri LJ 4664 (Foll.) 13
1992 AIR SCW 640 : AIR 1992 SC 840 : 1992 Cri LJ 1104 (Foll.) 12
AIR 1990 SC 79 : 1990 Cri LJ 605 (Foll.) 10
AIR 1984 SC 1622 : 1984 Cri LJ 1738 (Foll.) 11
AIR 1952 SC 343 : 1953 Cri LJ 129 (Foll.) 9
Ajit Kumar Pande, (AC), for Appellant; Sushil Karanjkar, Ravindra Keshavrao Adsure,
for Respondent.
* Cri. A. No. 797 of 1998, D/- 4- 4- 2003 (Bom.)
Judgement
G. S. SINGHVI, J. :- This appeal by special leave is directed against the judgment of the
Bombay High Court, which upheld the conviction of the appellant under Section 302
I.P.C. and sentence of life imprisonment awarded to him by Additional Sessions Judge,
Greater Bombay in Sessions Case No. 28/1995.
2. On 1-10-1994, PW1 Dr. Rasiklal Dwarkadas Dani, a resident of Pratap Building 173,
Dadiseth Agyari Lane, Mumbai, telephonically informed Assistant Police Inspector
(API), PW14 R.R. Gaekwad of Police Station Tilak Nagar that a man, who was later on,
identified as Satish, was lying on the right side of the stairs of the building in a pool of
blood. API Gaekwad reached the spot and removed that person to G.T. Hospital, where
he was declared brought dead. PW14 recorded the information given by Dr. Dani as Ex.
P6 and treated the same as FIR. He then handed over the investigation to PW13
Shamsherkhan Wazirkhan Pathan, who was acting as night Police Inspector at L.T. Marg
Police Station. The latter prepared Panchnama of the dead body. From the papers found
in the pocket of the clothes of the deceased, the police contacted his brother, PW3
Rajaiyya Pochyya Bandapalli on 1-10-1994 itself and recorded his statement. After two
days, the appellant and one Devabhuma Badapatti were arrested. On the day of his arrest
i.e. 3-10-1994, the appellant is said to have made a statement and then took the police to
Room No. 45 of the third floor of the building known as Ganesh Bhuvan Dadiseth Agyari
Lane, Mumbai and got recovered his pant and shirt which are said to be having stains of
blood. On 4-10-1994, the appellant was medically examined by PW10 Shiv Narain
Daund, who found that the thumb and index finger of the appellants right hand had been
injured sometime back. On the next day i.e., 5-10-1994, the appellant took the police to
PW7 Mohd. Farid Abdul Gani, who claims to have sold the handkerchief, which was
found near the body of the deceased. On 6-10-1994, the appellant is said to have given
some more information to the police and got recovered half blade (marked as Article 7)
which was lying under the wooden platform in front of Ganesh Bhuvan. The clothes of
the deceased, the pant and shirt belonging to the appellant and blade were sent for
chemical examination. As per the Chemical Examiners Report, the clothes of
@page-SC1185
the deceased were having human blood of O group. The pant and shirt, allegedly
recovered at the instance of the appellant also had blood stains, but it could not be
established whether the same was human blood of O group. The stain on the blade was
also said to be of human blood but its identity could not be established by the chemical
examiner.
3. After completing the investigation, the police submitted challan in the Court of the
Metropolitan Magistrate who committed the case to the Court of the Session, Greater
Bombay.
4. The prosecution examined PW1 Dr. Rasiklal Dwarkadas Dani, PW2 Dinesh Dubey,
with whom Devabhuma Badapatti is said to have worked till September 1994, PW3
Rajjaiyya (brother of the deceased), PW4 Hari Oval and PW8 Ranjit Bishram Jaiswal,
who acted as panches for recovery of the clothes from Room No.45 of Ganesh Bhuvan,
PW5 Salim Sheikh, who acted as panch for recovery of half blade beneath wooden board
in front of Ganesh Bhuvan, PW6 Shankar Shripati Ulalkar, who was engaged in the work
of shaving and cutting hair outside shop No.1 of Ganesh Bhuvan, Dadiseth Agyari Lane,
PW7 Mohd. Farid Abdul Gani, who claims to have sold the handkerchief to the appellant,
PW9 Balu Shivram Nalwada, who is said to have witnessed the sale of handkerchief by
PW7 to the appellant, PW10 Shivraj Narayan Daund, who examined the appellant on 4-
10-1994, PW11 Raju Chandu Poojari, who claims to have seen the accused persons with
the deceased on the night of the incident i.e. 30-9-1994, PW12 Dr. Avinash Janardan
Pujari, who performed the autopsy on the dead body, PW13 PI, Shamsherkhan Vazirkhan
Pathan and PW14 API, R.R. Gaikwad. Thereafter, the statements of the appellant and
Devabhuma Badapatti were recorded under Section 313, Cr.P.C. Both of them denied
having committed the crime.
5. The motive of the crime, as projected by the prosecution, was that the appellant was
having illicit relation with Lakshmi wife of the deceased and Devabhuma Badpatti was
having animosity with the latter because of the alleged murder of his father. The
prosecution relied on the circumstantial evidence of last scene, recovery of blood stained
pant and shirt from Room No.45, Ganesh Bhuvan Building, blood stained half blade and
handkerchief found near the body of the deceased to prove the appellants involvement in
the crime.
6. The learned Additional Sessions Judge did not accept the prosecutions theory regarding
motive but relied on the circumstantial evidence and convicted both the accused under
Section 302 read with Section 34, I.P.C. and sentenced them to life imprisonment. On
appeal, the Division Bench of the High Court upheld the conviction of the appellant and
confirmed the sentence of life imprisonment awarded to him, but acquitted Devabhuma
Badpatti on the premise that there was no evidence to show that he was a party to the
crime.
7. Shri Ajit Kumar Pande assailed the findings recorded by the learned Additional
Sessions Judge, which as mentioned, were confirmed by the High Court by arguing that
the entire story was fabricated by the police to falsely implicate the appellant. Learned
counsel invited our attention to the serious discrepancies in the statement of PW 11, Raju
Poojari, who claims to have seen the appellant with the deceased at 10.45 p.m. on 30th
September 1994 and argued that the deliberate attempt made by the witness to conceal
the fact that he was engaged in the business of illicit liquor and was arrested by the police
in connection with the said business should have been treated by the learned Additional
Sessions Judge and High Court sufficient for discarding his testimony. Shri Pande then
argued that the recovery of the blood stained pant and shirt from Room No. 45 of Ganesh
Bhuvan and half blade from under the wooden board in front of Ganesh Bhuvan, are
highly suspicious and no credence should have been given to such recoveries for holding
the appellant guilty of serious offence like murder because they were not proved to be
stained with human blood of O group. He lastly argued that version of PW7 Mohd. Gani
regarding sale of handkerchief to the appellant is unbelievable because there was nothing
from which he could identify the handkerchief allegedly sold more than one month
before the alleged murder. Shri Sushil Karanjakar, learned counsel for the State supported
the judgment under challenge and argued that the High Court rightly upheld the
conviction of the appellant and the sentence awarded to him.
8. We have thoughtfully considered the entire matter. It is settled law that an offence can
be proved not only by direct evidence
@page-SC1186
but also by circumstantial evidence where there is no direct evidence. The Court can draw
an inference of guilt when all the incriminating facts and circumstances are found to be
totally incompatible with the innocence of the accused. Of course, the circumstances
from which an inference as to the guilt is drawn have to be proved beyond reasonable
doubt and have to be shown to be closely connected with the principal fact sought to be
inferred from those circumstances.
9

. In Hanumant Govind Nargundkar v. State of M.P. [AIR 1952 SC 343], which is one of
the earliest decisions on the subject, this court observed as under : Para 10 of AIR

"It is well to remember that in cases where the evidence is of a circumstantial nature, the
circumstances from which the conclusion of guilt is to be drawn should be in the first
instance be fully established and all the facts so established should be consistent only
with the hypothesis of the guilt of the accused. Again, the circumstances should be of a
conclusive nature and tendency and they should be such as to exclude every hypothesis
but the one proposed to be proved. In other words, there must be a chain of evidence so
far complete as not to leave any reasonable ground for a conclusion consistent with the
innocence of the accused and it must be such as to show that within all human probability
the act must have been done by the accused."
10
. In Padala Veera Reddy v. State of A.P. [(1989) Supp (2) SCC 706], this Court held that
when a case rests upon circumstantial evidence, the following tests must be satisfied :
AIR 1990 SC 79

"(1) the circumstances from which an inference of guilt is sought to be drawn, must be
cogently and firmly established;
(2) those circumstances should be of a definite tendency unerringly pointing towards guilt
of the accused;
(3) the circumstances, taken cumulatively, should form a chain so complete that there is
no escape from the conclusion that within all human probability the crime was committed
by the accused and none else; and
(4) the circumstantial evidence in order to sustain conviction must be complete and
incapable of explanation of any other hypothesis than that of the guilt of the accused and
such evidence should not only be consistent with the guilt of the accused but should be
inconsistent with his innocence.
11

. In Sharad Birdhichand Sarda v. State of Maharashtra [(1984) 4 SCC 116], it was held
that the onus was on the prosecution to prove that the chain is complete and falsity or
untenability of the defence set up by the accused cannot be made basis for ignoring
serious infirmity or lacuna in the prosecution case. The Court then proceeded to indicate
the conditions which must be fully established before conviction can be based on
circumstantial evidence. These are : AIR 1984 SC 1622

(1) the circumstances from which the conclusion of guilt is to be drawn should be fully
established. The circumstances concerned must or should and not may be established;
(2) the facts so established should be consistent only with the hypothesis of the guilt of
the accused, that is to say, they should not be explainable on any other hypothesis except
that the accused is guilty;
(3) the circumstances should be of a conclusive nature and tendency;
(4) they should exclude every possible hypothesis except the one to be proved; and
(5) there must be a chain of evidence so complete as not to leave any reasonable ground
for the conclusion consistent with the innocence of the accused and must show that in all
human probability the act must have been done by the accused."
12

. In State of U.P. v. Ashok Kumar Srivastava [(1992) 2 SCC 86], it was pointed out that
great care must be taken in evaluating circumstantial evidence and if the evidence relied
on is reasonably capable of two inferences, the one in favour of the accused must be
accepted. It was also pointed out that the circumstances relied upon must be found to
have been fully established and the cumulative effect of all the facts so established must
be consistent only with the hypothesis of guilt. 1992 AIR SCW 640

13
. The above noted propositions have been reiterated in Bodhraj alias Bodha and Others
vs. State of Jammu and Kashmir [(2002) 8 SCC 45]; Bharat vs. State of M.P. [(2003) 3
SCC 106]; Jaswant Gir vs. State of Punjab [(2005) 12 SCC 438]; Reddy Sampath Kumar
vs. State of A.P. [(2005) 7 SCC 603]; Deepak Chandrakant Patil vs. State of Maharashtra
[(2006) 10 SCC 151]; Ramreddy Rajesh 2000 AIR SCW 3655
2003 AIR SCW 770
2005 AIR SCW 4620
2006 AIR SCW 1637
2006 AIR SCW 1602
2007 AIR SCW 2226

@page-SC1187
Khanna Reddy and Another vs. State of A.P. [(2006) 10 SCC 1721 and State of Goa vs.
Sanjay Thakran and Another [(2007) 3 SCC 755].
14

. In Ramreddy Rajesh Khanna Reddy and Another vs. State of A.P. [(2006) 10 SCC 172],
this Court while reiterating the settled legal position, observed : 2006 AIR SCW 1602,
Para 26

"It is now well settled that with a view to base a conviction on circumstantial evidence,
the prosecution must establish all the pieces of incriminating circumstances by reliable
and clinching evidence and the circumstances so proved must form such a chain of events
as would permit no conclusion other than one of guilt of the accused. The circumstances
cannot be on any other hypothesis. It is also well settled that suspicion, however grave it
may be, cannot be a substitute for a proof and the courts shall take utmost precaution in
finding an accused guilty only on the basis of the circumstantial evidence."
15

. At this stage, we also deem it proper to observe that in exercise of power under Article
136 of the Constitution, this Court will be extremely loath to upset the judgment of
conviction which is confirmed in appeal. However, if it is found that the appreciation of
evidence in a case, which is entirely based on circumstantial evidence, is vitiated by
serious errors and on that account miscarriage of justice has been occasioned, then the
Court will certainly interfere even with the concurrent findings recorded by the trial
Court and the High Court - Bharat vs. State of M.P. [(2003) 3 SCC 106]. 2003 AIR
SCW 770

16. In the light of the above, we shall now consider whether in the present case the
prosecution succeeded in establishing the chain of circumstances leading to an
inescapable conclusion that the appellant had committed the crime.
17. A careful reading of the judgments of the Additional Sessions Judge and High Court
shows that the conviction of the appellant was based on the following circumstances :
"(i) that both the accused were with the deceased when he was last seen alive in the night
of 30/9/1994.
(ii) the accused had residence in the vicinity of the place where the injured was found
while the injured did not reside in the vicinity.
(iii) accused No.l had an injury which could be caused by user of the blade (Art. 7) and
had knowledge where the piece of blade could be found by the Police.
(iv) there was human blood on the piece of blade and stains of human blood on the
clothes of accused No. 1 were not explained to be the stains of blood of his own.
(v) the handkerchief purchased by accused No. 1 was found near the injured with stains
of blood of the injured indicative of presence of accused No. 1 in the vicinity after the
injured had sustained bleeding injuries.
(vi) Accused No.2 used to be with accused No.l many times and had been sleeping at the
place of accused No. 1 for three nights and accused No.2 had borrowed Rs.300/-in the
night of 30th September, 1994 and
(vii) they were caught when they were together.
18. We shall first scrutinize the evidence of last seen, which is in the form of statement of
PW11 Raju Poojari. In the first instance, the witness denied his acquaintance with Babu
Poojari but then volunteered to admit that he knew the latter. He gave out that he was
residing in a temporary shed at Sonapur, Chandanwadi, which was used as tailoring shop.
Later on, he made an improvement by saying that he was doing work at the tailoring
shop. According to him both the accused had passed in front of the shop on 30th
September, 1994 at 10.45 p.m. He demonstrated his acquaintance with both the accused
by saying that they used to come to the tailoring shop. When two photographs of the
deceased (marked as Article 8) were shown to him, PW11 stated that the said person had
come with the accused for getting their clothes stitched from the shop. He expressed his
ignorance about the time when they came to the shop and then stated that they came at
10.30 p.m. 2-4 days before the police came to make enquiries from him. According to
PW11 his signatures were obtained at the police station but nothing was read out to him.
He then stated that something was read out at the police station ten days back when he
was called there and was shown photographs (marked as Article 8). He admitted the
existence of a liquor shop near the tailoring shop, but gave out that the same was owned
by one John.
@page-SC1188
He denied his involvement in the business of illicit liquor. At that stage the public
prosecutor sought and was granted permission to ask questions in the nature of cross
examination. In reply to the querries put by the public prosecutor, PW11 denied the
suggestion that he was doing business of illicit liquor and expressed his ignorance about
the statement given to the police that he was engaged in such business. He also denied
having stated before the police that the accused had come to the liquor shop with the
person in the photograph and that they were offering liquor to him and also asked Babu
Poojari to pour more liquor in his glass because he was their guest. PW11 then stated that
the person shown in the photograph was totally drunk when he came with two accused
and they were supporting him while walking and this happened 4-5 days before when he
was called to the police station. He expressed his ignorance about the number of false
cases registered against him. In cross examination he denied having indulged in any
activity other than tailoring work. He also gave out that he did not know the names of the
accused when they passed in front of the tailoring shop.
19. A critical analysis of the statement of PW11 shows that the same is full of
contradictions. In the examination-in-chief, he demonstrated his acquaintance with the
accused by saying that they used to come to tailoring shop but in cross examination he
admitted that he did not know their names when they were passing in front of the
tailoring shop. The second important contradiction relates to his recognition of the person
shown in the photograph. In the first instance he gave out that the said person had come
with the accused for getting their clothes stitched from the shop but, later on, stated that
he came with the accused and was heavily drunk and was being helped by the accused.
Yet another contradiction which is apparent from the statement of PW11 relates to his
acquaintance with Babu Poojari. In the beginning he flatly denied that he knew Babu
Poojari and then made a u-turn by voluntarily stating that he knows Babu Poojari. He also
denied having stated before the police that he was doing the business of selling illicit
liquor in association with Babu Poojari and that the accused offered liquor to the
deceased and also asked Babu Poojari to pour more liquor in his glass. These
contradictions are evident from the following extracts of the statement of PW11 :
"I know Satish Bandapalli and Devaanna Bandapalli and they were passing in front of my
shop at about 10.45 p.m. I do not know where they were going. I had been knowing those
2 persons as they used to come to the tailoring shop."
"These 2 accused had not done anything else when they passed in front of my tailoring
shop. I did not know their names at that time."
"I had seen the person whose 2 photographs from Article 8 are now shown to me, but I do
not know his name. When I last saw the person in this photograph, he had been with the 2
accused before this Court. Those 3 together had come to my shop. Those 3 had come for
getting their clothes stitched from the shop where I used to be. I do not remember the
time when they had come to the shop. Now I say that they had come at 10.30 p.m. 2-4
days before the police came to make enquiries from me. The police had taken me in the
police station. At the police station my signature was obtained and I was permitted to go.
The police did not read out anything to me at the time they had taken me to the police
station and obtained my signature but something was read out to me 10 days back when I
had been called here. At the time my signature was taken I was shown the photographs
Article 8."
"It did not happen that Babu Poojari came to me and agreed to work with me and we both
started doing the business of illicit liquor. I had not stated so to any one at any time. I
cannot say why portion marked A to that effect has been so recorded. It did not happen
that these 2 accused had come to my illicit liquor business with the person in the
photograph. I had not stated so to any one at any time. I cannot say why portion marked
B to that effect has been so recorded in my alleged statement dated 5-10-1994. I had not
seen these 2 accused offering liquor to the person in the photograph and also asking Babu
Poojari to pour more liquor in his glass as he was their guest. I had not stated so to any
one at any time. I cannot say why portion marked 'C' to that effect has been so recorded
in my alleged statement dated 5-10-1994. The person in the photograph now shown to me
Article 8 was totally drunk when he had come with these 2 accused to our shop. The
person in the photograph was so drunk that these 2 accused

@page-SC1189
had to support the person in the photograph Article 8 for making him walk away and in
that condition I last saw them walking away from the tailoring shop 4-5 days before
police took me to the police station. At present I have been wrongly apprehended by the
police in a case when there was a raid on the illicit liquor shop in the neighbourhood. I do
not know in how many false cases I have been involved after being wrongly
apprehended."
20. It is significant to note that even though PW11 denied having made statements
marked 'A', 'B' and 'C' before the police but the investigating officer. PW13 categorically
asserted that Raju Poojari did make those statements.
21. The learned Sessions Judge as also the High Court noted contradictions in the
statement of PW11 but ignored the same by describing them as minor. In the opinion of
the learned Sessions Judge the variation in the previous statement of PW11 stands
explained by his desire not to incriminate himself. He also observed that the defence had
not brought anything from the cross examination of PW11 to discredit his testimony. The
High Court adopted the same line of reasoning for placing reliance on the evidence of last
scene.
22. In our view, the testimony of PW11 is wholly untrustworthy. He appears to be a
doctored witness, who came forward to support the prosecution cause with a view to win
favour from the police in the cases registered against him in connection with the raid of
illicit liquor shop. This is the reason why he made vacillating statement regarding the
identity of two accused and the deceased and the purpose of their coming to the so-called
tailoring shop where he was residing and also working. It is difficult, if not possible, to
believe that even though the accused persons used to come to the tailoring shop for
getting their clothes stitched, where PW11 is said to be working, he did not know their
names. His attempt to conceal his acquaintance with Babu Poojari who was his associate
in the business of illicit liquor is inexplicable. The suggestive conjecture made by the
learned Additional Sessions Judge that PW11 retracted from the statement made before
the police because he did not want to incriminate himself in offences relating to business
of illicit liquor cannot be accepted because the fact of the matter is that the witness was
arrested by the police in connection with the said business and there was every reason for
him to come forward to support the police case. The testimony of PW11 is also
discredited by the fact that he made self contradictory statements regarding the presence
of the accused and the deceased at the shop. In one breath he stated that they were
passing in front of the shop and thereafter sought to identify them by stating that they had
come for stitching the clothes.
23. The next thing which is to be seen is whether the evidence relating to the recovery of
clothes of the appellant and the half blade, allegedly used for commission of crime, is
credible and could be relied on for proving the charge of culpable homicide against the
appellant. In this context, it is important to note that the prosecution did not produce any
document containing the recording of statement allegedly made by the appellant
expressing his desire to facilitate recovery of the clothes and half blade. The prosecution
case that the accused volunteered to give information and took the police for recovery of
the clothes, half blade and purchase of handkerchief is highly suspect. It has not been
explained as to why the appellant gave information in piecemeal on three dates i.e. 3-10-
1994, 5-10-1994 and 6-10-1994. Room No.45 of 'Ganesh Bhuvan' from which the clothes
are said to have been recovered was found to be unlocked premises which could be
accessed by any one. The prosecution could not explain as to how the room allegedly
belonging to the appellant could be without any lock. The absence of any habitation in
the room also cast serious doubt on the genuineness and bona fides of recovery of
clothes. The recovery of half blade from the road side beneath the wooden board in front
of Ganesh Bhuvan is also not convincing. Undisputedly, the place from which half blade
is said to have been recovered is an open place and everybody had access to the site from
where the blade is said to have been recovered. It is, therefore, difficult to believe the
prosecution theory regarding recovery of the half blade. The credibility of the evidence
relating to recovery is substantially dented by the fact that even though as per the
Chemical Examiners Report the blood stains found on the shirt, pant and half blade were
those of human blood, the same could not be linked with the blood of the deceased.
Unfortunately, the learned Additional Sessions
@page-SC1190
Judge and High Court overlooked this serious lacuna in the prosecution story and
concluded that the presence of human blood stains on the cloths of the accused and half
blade were sufficient to link him with the murder.
24. The over jealous efforts made by the prosecution to link the handkerchief allegedly
found near the body of the deceased of the appellant lends support to the argument of the
learned counsel for the appellant that the police had fabricated the case to implicate the
appellant. In his statement, PW7 Mohd. Farid Abdul Gani, who is said to have sold the
handkerchief to the appellant, admitted that he was not selling branded handkerchiefs and
that there were no particular marks on the goods sold by him. He, however, recognized
the handkerchief by saying that the accused made a lot of bargaining and he was amused
by the latters statement that he will soon become an actor.
25. Both the learned Additional Judge and High Court accepted the testimony of PW7
along with the statement of PW9 ignoring the admission made by the former that he did
not put any special mark on the handkerchief sold by him; that he purchased the
handkerchiefs in wholesale from the market and removed the label of manufacturer
before selling the same and that there are 4 or 5 other persons carrying on the same
business in the locality. Likewise both the courts ignored the fact that PW9 could not
confirm the exact identity of the handkerchief (marked as Article 3), he could only say
that the handkerchief of the appellant was just like Article 3.
26. In our opinion it is extremely difficult to believe that a person engaged in the business
of hawking would remember what was sold to a customer almost two months after the
transaction and that to without identity of the goods sold having been established.
27. On the basis of above discussion we held that the prosecution failed to establish the
chain of circumstances which could link the appellant with the crime. The learned Trial
Court and the High Court committed a serious error by relying on the circumstantial
evidence of last seen, the recovery of pant and shirt from Room No.45 of Ganesh Bhuvan
building, half blade from under the wooden board and the sale of the handker-chief by
PW7 to the appellant.
28. In the result the appeal is allowed. The judgment under appeal and the one of the Trial
Court are set aside and the appellant is acquitted. He shall be released forthwith if not
required in connection with any other offence.
Appeal allowed.
AIR 2008 SUPREME COURT 1190 "State of U. P. v. Roshan Singh"
(From : Allahabad)*
Coram : 2 Dr. A. PASAYAT AND AFTAB ALAM, JJ.
Civil Appeal Nos. 453-455 of 2008 (arising out of SLP (C) Nos. 16970-72 of 2005), D/-
16 -1 -2008.
State of U.P. and Ors. v. Roshan Singh (D) by L.Rs. and Ors.
Civil P.C. (5 of 1908), S.151 - U.P. Imposition of Ceiling on Land Holdings Act (1 of
1961), S.12 - INHERENT POWERS - LAND CEILING - OBJECT OF AN ACT -
APPEAL - Inherent powers - Exercise of - Scope - Inherent power cannot be exercised
for benefit of litigant who has remedy under relevant statute - Declaration of surplus land
- No appeal preferred against such order - Application filed u/S.151 long after period
provided for filing appeal under Act - Not maintainable.
C.M.W.P. No. 17464 of 1984 in C.M.W.P. Nos. 8825 and 19050 of 1995, D/-13-11-2002
(All), Reversed. (Paras 7, 8)
Cases Referred : Chronological Paras
2002 AIR SCW 1432 : AIR 2002 SC 1543 (Ref.) 9
AIR 1964 SC 993 (Ref.) 10
S.K. Dwivedi, AAG, Ratnakar Das, Sr. Adv., Abhishek Chowdhary, Manoj Kr. Dwivedi,
Ms. Vandana Mishra, Gunnam Venkateswara Rao, for Appellants; A. S. Pundir, B.K. Pal,
for Respondents.
* C.M.W.P. No. 17464 of 1984 in C.M.W.P. Nos. 8825 and 19050 of 1995, D/- 13-11-
2002 (All).
Judgement
Dr. ARIJIT PASAYAT, J. :- Leave granted.
2. Challenge in these appeals is to the judgment of the learned Single Judge of the
Allahabad High Court allowing the Civil Misc. Writ Petitions 17464 of 1984, 8825 of
1995 and 19050 of 1995. Challenge in the first writ petition was to the order passed by
the Prescribed Authority under the U.P. Imposition of Ceiling on Land Holdings Act,
@page-SC1191
1954 (in short the Act) and the appellate order passed by the Appellate Authority.
3. Background facts in a nutshell are as follows :
After issuance of notice under Section 10(2) of the Act an area of 17 Bighas 10 Biswas
and 2 Biswansis of land of the respondent- Roshan Singh was declared as surplus.
Thereafter consolidation operation commenced. Proceedings under Section 107 of the
Act were initiated on 28-3-1974 and the respondent-Roshan Singh was granted
opportunity to file his response to the notice. The objection was filed on 25-5-1974 and
by order dated 14-1-1980 the Prescribed Authority after determining the surplus gave
opportunity to the respondent to indicate the choice of land to be retained. The respondent
did not indicate any choice. Therefore by order dated 8-4-1982, 17 Bighas 10 Biswas and
2 Biswansis of land was declared as surplus. Thereafter, possession of the surplus land
was taken. There is a provision for appeal under Section 12 of the Act. But the
respondent-Roshan Singh did not prefer any appeal. On the other hand on 17-2-1984 an
application titled under Section 151 of the Civil Procedure Code, 1908 (in short CPC)
was filed. Stand taken was that in the consolidation proceedings different area was
indicated and, therefore, holding was reduced. Objections were filed by the functionaries
of the State on 23-3-1984 and 30-3-1984. Considering the objections the Prescribed
Authority by order dated 3-4-1984 rejected the claim of the respondent-Roshan Singh. An
appeal was preferred by him i.e. Revenue Appeal No.24 of 1984 in the Court of III
Additional District Judge, Banda, U.P. The appeal was dismissed on 21-8-1984. Civil
Writ Petition No.17464 of 1984 was filed before the Allahabad High Court.
Subsequently, the surplus land was distributed. These were challenged in Civil Writ
Petition No.8825 of 1995 and 19050 of 1995. The first writ petition was allowed by a
learned Single Judge with the following observations :
"Having heard Sri V.K.S. Chaudhary, learned Senior counsel appearing on behalf of the
petitioner and Smt. Archana Srivastava, learned Standing Counsel for the respondents,
this Court is of the view that as the reduction of area made during the consolidation
operation is made for public purposes, the petitioner is entitled to the benefit of said
reduction. The submission made by the learned counsel for the petitioner has got force
and therefore, the writ petition deserves to be allowed."
4. It is to be noted that the above quoted portion was the only basis on which the writ
petition was allowed. Two orders were also allowed following the decision rendered in
the first case.
5. Learned counsel for the appellants submitted that the approach of the High Court is
clearly erroneous. Firstly, petition under Section 151 was not maintainable when
statutorily an opportunity and/or forum is provided which was not availed. Further the
proceedings under the Act and the Consolidation Act operate in different fields and,
therefore, even if the area was different same was on the basis of the parameters under the
Consolidation Act and a belated attempt to re-open concluded issues by resorting to
Section 151 was clearly impermissible.
6. Learned counsel for the respondent submitted that there cannot be two different areas;
one under the Act and the other the Consolidation Act. Therefore, the High Court was
justified in its view.
7. The principles which regulate the exercise of inherent powers by a court have been
highlighted in many cases. In matters with which the CPC does not deal with, the Court
will exercise its inherent power to do justice between the parties which is warranted
under the circumstances and which the necessities of the case require. If there are specific
provisions of the CPC dealing with the particular topic and they expressly or by
necessary implication exhaust the scope of the powers of the Court or the jurisdiction that
may be exercised in relation to a matter, the inherent powers of the Court cannot be
invoked in order to cut across the powers conferred by the CPC. The inherent powers of
the Court are not to be used for the benefit of a litigant who has remedy under the CPC.
Similar is the position visa-vis other statutes. The object of Section 151, CPC is to
supplement and not to replace the remedies provided for in the CPC. Section 151, CPC
will not be available when there is alternative remedy and same is accepted to be a well-
settled ratio of law. The operative field of power being thus restricted, the same cannot be
risen to inherent power. The inherent powers of the Court are in addition to the powers
specifically conferred to it. If there are express provisions covering
@page-SC1192
a particular topic, such power cannot be exercised in that regard. The section confers on
the Court power of making such orders as may be necessary for the ends of justice of the
Court. Section 151, CPC cannot be invoked when there is express provision even under
which the relief can be claimed by the aggrieved party. The power can only be invoked to
supplement the provisions of the Code and not to override or evade other express
provisions. The position is not different so far as the other statutes are concerned.
Undisputedly, an aggrieved person is not remediless under the Act.
8. The conclusions of the High Court are not only cryptic but also without indication of
any basis. As rightly contended by learned counsel for the appellant long after the period
provided for preferring an appeal under Section 12 of the Act, the application under
Section 151, CPC was filed.
9

. This Court in State of W. B. and Ors. v. Karan Singh Binayak and Ors. (2002 (4) SCC
188), inter alia observed as follows : 2002 AIR SCW 1432, Para 17

"The period of 25 years under the lease expired in the year 1976. The notification under
the Act was issued on 11th November, 1954. In 1957 record of rights was prepared under
Section 44 of the Act according to which the land was held retainable under Section 6(l)
(b) of the Act. The possession was handed over to the original owners in 1981 on
liquidation of the lessee on an order being passed by the High Court directing official
liquidator to disclaim the property which was later transferred to the writ petitioners in
terms of the agreements of sale entered in the year 1988 and sale deeds in 1992-93.
Meanwhile, in the year 1991 on proceedings being taken under the ULC Act, 6145.90
square meter of the land was held to be excess under the said Act. In June 1993, the plans
were sanctioned and construction commenced. It can, thus, be seen that after the
preparation of record-of-rights, not only the appellants did not take any steps and slept
over the matter but various steps as above were taken by the respondents in respect of the
land in question. The argument that the proceedings under the ULC Act or the preparation
of record-of-rights were ultra vires and the acts without jurisdiction and, therefore, those
proceedings would not operate as a bar in appellants invoking inherent jurisdiction under
Section 151 CPC by virtue of conferment of such power under Section 57A of the Act is
wholly misconceived and misplaced. The inherent powers cannot be used to reopen the
settled matters. These powers cannot be resorted to when there are specific provisions of
the Act to deal with the situation. It would be an abuse to allow the reopening of the
settled matter after nearly four decades in the purported exercise of inherent powers. It
has not even been suggested that there was any collusion or fraud on behalf of the writ
petitioners or the erstwhile owners. There is no explanation much less satisfactory
explanation for total inaction on the part of the appellants for all these years."
10

. In Arjun Singh v. Mohindra Kumar and Ors. (AIR 1964 SC 993) it was, inter alia,
observed as follows : Para 20 of AIR

"There is one other aspect from which the same question could be viewed. Order IX,
Rule 7 prescribes the conditions subject to which alone an application competent under
the opening words of that rule ought to be dealt with. Now, the submission of Mr. Pathak
if accepted, would mean to ignore the opening words and say that though specific power
is conferred when a suit is adjourned for hearing, the Court has an inherent power even
when (a) it is not adjourned for that purpose, and (b) and this is of some importance when
the suit is not adjourned at all, having regard to the term of Order XX, Rule 1. The main
part of Order IX, Rule 7 speaks of good cause being shown for non-appearance on a
previous day. Now what are the criteria to be applied by the Court when the supposed
inherent jurisdiction of the Court is invoked? Non-constat it need not be identical with
what is statutorily provided in Rule 7. All this only shows that there is really no scope for
invoking the inherent powers of the Court. Lastly, that power is to be exercised to secure
the ends of justice. If at the stage of Rule 7 power is vested in the Court and after the
decree is passed Order IX, Rule 13 becomes applicable and the party can avail himself of
that remedy, it is very difficult to appreciate the ends of justice which are supposed to be
served by the Courts being held to have the power which the learned counsel says must
inhere in it. In this view it is unnecessary to consider whether to sustain the present
submission the respondent must establish that
@page-SC1193
the court was conscious that it lacked specific statutory power and intended to exercise an
inherent power that it believed it possessed to make such orders as may be necessary for
the ends of justice."
11. Looked at from any angle the orders of the High Court impugned in these appeals
cannot be sustained and are set aside. It is to be noted that subsequent two writ petitions
were allowed primarily on the ground that first writ petition was allowed.
The appeals are allowed but in the circumstances without any order as to costs.
Appeals allowed.
AIR 2008 SUPREME COURT 1193 "Tulsa v. Durghatiya"
(From : AIR 2001 MP 188)
Coram : 2 Dr. A. PASAYAT AND P. SATHASIVAM, JJ.
Civil Appeal No. 648 of 2002, D/- 15 -1 -2008.
Tulsa and Ors. v. Durghatiya and Ors.
Evidence Act (1 of 1872), S.114 - EVIDENCE - SALE - Presumption as to marriage -
Continuous living of 'L' with 'R' after death of her husband, established by evidence on
record - She had legitimate claim over property of 'R' as his wife and can be sold by her.
Transfer of Property Act (4 of 1882), S.54.
S. A. No. 451 of 1988, D/-20-6-2000 (MP), Reversed. (Paras 15, 16)
Cases Referred : Chronological Paras
AIR 1978 SC 1557 : 1978 All LJ 1010 (Ref.) 13
AIR 1952 SC 231 (Ref.) 14
AIR 1927 PC 185 10
AIR 1929 PC 135 11
Prakash Shrivastava, for Appellants; Shiv Prakash Pandey, Raj Kumar Tanwar, for
Respondents.
Judgement
Dr. ARIJIT PASAYAT, J. :- Challenge in this appeal is to the judgment rendered by a
learned Single Judge of the Madhya Pradesh High Court at Jabalpur. The appeal under
Section 100 of the Code of Civil Procedure, 1908 (in short the Code) was directed against
the judgment and decree dated 29-10-1988 passed by learned IInd Additional District
Judge, Satna in Civil appeal No. 138-A of 1987. The appeal before the First appellate
court was directed against the judgment and decree dated 26-4-1985 passed by learned
Second Civil Judge Class I, Satna in Civil Suit No. 52-A of 1982. The suit was filed by
the respondents herein for nullifying and setting aside sale deed dated 10-9-1980 and also
for permanent injunction of land at SI. Nos. 4009, 4010, 4011 and 4014. The sale deed
dated 10-9-1980 was in respect of lands at SI. Nos. 3853, 3993, 4002, 4003, 4004, 4009,
4010, 4014, 4015 and 4021 of Mauza Nayagaon, Tehsil Raghurajnagar, District Satna.
According to them the disputed property is the joint ancestral property of Radhika Singh,
Sunder Singh and the husband of plaintiff No.l, Dadau Singh who was the father of the
other two plaintiffs - Smt. Rani and Smt. Butan. Vansh Gopal had three sons, Radhika
Singh, Sunder Singh and Dadau Singh. Sunder died without any legal heir. No partition
had taken place between Radhika and Sunder and Radhika, Sunder and Dadau all used to
do cultivation jointly. As Radhika and Sunder died without leaving legal heirs, the
plaintiffs became the sole owners of the property. Loli, the original defendant No. 1 is the
wife of Mangal Kachhi and his daughter Tulsa Bai, the present appellant was born to Loli
and Mangal Kachhi. After the birth of her daughter Tulsabai, deceased Radhika Singh,
kept defendant No. 1 as a mistress in his house and left for somewhere else taking her
along and came back after many years. She gave birth to three daughters namely Vidya,
Badaniya and Rajaniya. Defendant No. 1 was a Kachhia by caste and was also the
cognitive of deceased Radhika, so she had no legal rights in the property. After the death
of Radhika, Defendant No. 1 was residing with Badri Prasad Pandey. Badri Prasad got
sale deed executed in favour of defendant No. 1 of disputed property with intention to
usurping the land. Plaintiffs are in possession. They came to know about the transaction
when defendant Nos.2 to 4 submitted an application for transfer of land in their names
and then it came to light that defendant No. 1 had no title over the land and the land was
in possession of plaintiffs 1 to 3. On 17-12-1984 plaintiffs got the information that the
defendant Nos.2 and 3 have got their names mutated in respect of certain lands, therefore
the suit was filed. In the written statement filed the defendants took the stand that the
family tree indicated by the plaintiff was correct. Out of the land
@page-SC1194
12 acres owned by the family of Durghatiya, the plaintiff No. 1 had sold her share of
land. About 30 years back partition has taken place between Dadau and Sunder. Dadau
had separated after taking his share. He got the land in certain villages. Radhika and
Sunder used to live jointly and used to do cultivation over the land which they got in
partition. They died while living jointly in the year 1970. Plaintiff-Durghatia and Radhika
had sold their land in the capacity of owners during their lifetime. Sunder did not marry
and had no issue. Defendant No. 1 is the widow of Radhika. They were blessed with five
daughters and one son, out of which one son and one daughter died. The eldest daughter
Tulsa and the younger daughter were given in marriage by Radhika. Plaintiff No. 1 used
to regard defendant No. 1 as her jethani. Radhika and defendant No. 1 lived together for
thirty years as husband and wife and, therefore, she had legitimate claim over the
property as his wife. It was also disputed that defendant No. 1 was living with defendant
Nos.2 to 5. Defendant No. 1 had sold the lands to defendant Nos.2, 3 and 4 had also
given possession. Defendant No. 1 had taken a debt on the marriage of her son and for
that purpose she sold the land. She claimed that she had right to sell the land and
therefore no question of having any illegal possession. Four issues were framed by the
trial court and the important and vital issue was framed as issue No.2 which read as
follows :
"Whether the defendant No. 1 was the wife of Radhika Singh"?
The question was answered in the affirmative. After referring to the evidence of the
witnesses examined by the plaintiffs as well as the defendants, the trial court held that
there was no merit in the suit and accordingly it was dismissed. The judgment and decree
were questioned in appeal before the first appellate Court.
2. As noted above, the first appellate Court allowed the appeal. The trial court noted that
there was a presumption of valid marriage, as for decades Radhika and plaintiff No. 1
lived together, their daughters were given in marriage by Radhika. Loli the defendant No.
1 was earlier married to Mangala Kochhi and after his death she married Radhika. It is to
be noted that the stand of the plaintiffs was that Loli married Radhika during the lifetime
of Mangal Katchhi. The trial Court rejected this plea. The first appellate court observed
that Loli started living with Radhika during the lifetime of Mangal Katchhi, so the
presumption of valid marriage was not there. The judgment and decree of the first
appellate court was challenged before the High Court. The High Court formulated the
following questions for adjudication :
"Whether in the facts and circumstances of the case, the first appellate Court erred in law
in finding that Mst. Lollibai was not the legally married wife of Radhika Singh"?
3. After discussing the respective stand of the parties, the High Court came to a somewhat
peculiar finding. It held that the findings recorded by the appellate court may be
erroneous, but it does not appear to be perverse.
4. It is to be noted that the first appellate court without any evidence or material came to
an abrupt conclusion that the defendant No. 1 Loli started living with Radhika during the
lifetime of her husband. There is no discussion with reference to any material as to the
basis for such a conclusion.
5. Some of the conclusions of the trial court in this regard are relevant. In paragraph 16 of
the judgment it was noted as follows :
"In the content of the aforesaid judgment, now we have to examine this that whether we
have sufficient basis to make a presumption of legal marriage of Lolli and Radhika Singh.
In this connection, plaintiff witness Visheshar had admitted in para 9 of his statement that
there were four daughters and one son born of Lolli and Radhika Singh. The eldest
daughter of Lolli is Tulsi. Rani was born to Lolli after 2-3 years of her arriving in the
village. Three of the daughters of Lolli was married off by Radhika Singh and she had
also contributed.
6. Again at para 18 it was observed as follows :
"Witness Devdhari has also admitted in his statement that after 2-3 years of the birth of
first born Bhaiyalal Mangal Kachhi had died. Lolli used to work as a labourer. She also
used to be labourer with Radhika Singh. Radhika Singh had retained Lolli as his wife.
The daughters of Lolli were married off by Radhika Singh. Ram Milan Singh had
admitted in his statement that all these four daughters were alive. They were born of
Radhika and Lolli. The daughters which were born of Radhika Singh, their Kanyadan
was
@page-SC1195
also performed by Radhika Singh. He has also admitted this in his statement that Radhika
Singh had married off his daughters as Vaishyas and Thakurs married off their daughters.
He had attended the marriage."
7. In para 24 it was observed as follows :
"This has also been argued by learned counsel of the plaintiff that even if this is accepted
that Lolli and Radhika Singh stayed as husband and wife for many days and they were
blessed with children even then it cannot be presumed that Lolli is legitimate wife of
Radhika Singh. Because Lolli moved in with Radhika Singh then her husband had been
alive. His former husband Mangal Kachhi had been alive, till she got divorce by Mangal
Kachhi till then Lolli could not have entered in second marriage with Radhika Singh. I
am no in agreement with this argument of the learned counsel of the plaintiff because the
evidence, which has been adduced from the side of the plaintiff and defendants, from that
it becomes clear, that after Bhaiyalal was born to Lolli from Mangal, Mangal had thrown
Lolli out of the house. Then Lolli worked as a casual labourer for some time and
meanwhile Mangal had died. Thereafter Radhika Singh adopted her as his wife. This fact
has been admitted by Devdhari in para 4 of his statement that Lolli used to frequent
village Bointa from Bandhi to work as a labourer, thereafter she was adopted."
8. In contrast, the first appellate court held that Bhaiyalal (DW2) who was born to Lolli
and Mangal, had stated that he was very young when his father died and when he was
young his mother had left. From that it was inferred that during the lifetime of Mangal
Katchhi, Lolli left the Mangal and was living with Radhika. This conclusion is clearly
contrary to the evidence on record. A bare reading of the evidence of DW 2 shows that he
had clearly stated that Mangal was not alive when Lolli came and stayed with Radhika.
9. At this juncture reference may be made to the Section 114 of the Indian Evidence Act,
1872 (in short the Evidence Act). The provision refers to common course of natural
events, human conduct and private business. The court may presume the existence of any
fact which it thinks likely to have occurred. Reading the provisions of Sections 50 and
114 of the Evidence Act together, it is clear that the act of marriage can be presumed from
the common course of natural events and the conduct of parties as they are borne out by
the facts of a particular case.
10. A number of judicial pronouncements have been made on this aspect of the matter.
The Privy Council, on two occasions, considered the scope of the presumption that could
be drawn as to the relationship of marriage between two persons living together. In first
of them i.e. A. Dinohamy v. W.L. Blahamy [AIR 1927 PC 185] their Lordships of the
Privy Council laid down the general proposition that :
"Where a man and woman are proved to have lived together as man and wife, the law
will presume, unless, the contrary be clearly proved that they were living together in
consequence of a valid marriage, and not in a state of concubinage".
11. In Mohabhat Ali v. Md. Ibrahim Khan [AIR 1929 PC 135] their Lordships of the
Privy Council once again laid down that :
"The law presumes in favour of marriage and against concubinage when a man and
woman have cohabited continuously for number of years."
12. It was held that such a presumption could be drawn under Section 114 of the
Evidence Act.
13. Where the partners lived together for long spell as husband and wife there would be
presumption in favour of wedlock. The presumption was rebuttable, but a heavy burden
lies on the person who seeks to deprive the relationship of legal origin to prove that no
marriage took place. Law leans in favour of legitimacy and frowns upon bastardy. (See :
Badri Prasad v. Dy. Director of Consolidation and Ors. [AIR 1978 SC 1557]).
14. This Court in Gokal Chand v. Parvin Kumari [AIR 1952 SC 231] observed that
continuous cohabitation of woman as husband and wife and their treatment as such for a
number of years may raise the presumption of marriage, but the presumption which may
be drawn from long cohabitation is rebuttable and if there are circumstances which
weaken and destroy that presumption, the Court cannot ignore them.
15. As noted above, the continuous living together of Lolli and Radhika has been
established. In fact the evidence of the witnesses examined by the plaintiff also
established this fact. The conclusion of the first appellate court that they were living
together
@page-SC1196
when Mangal was alive has not been established. The evidence on record clearly shows
that Lolli and Radhika were living together after the death of Mangal.
16. Above being the position, the appeal deserves to be allowed which we direct. The
judgment and decree of the first appellate court and the High Court are set aside and
those of the trial court stand restored.
17. Appeal is allowed but with no order as to costs.
Appeal allowed.
AIR 2008 SUPREME COURT 1196 "Rajib Kumar Paul v. Gurudas Mitra"
(From : Calcutta)
Coram : 2 H. K. SEMA AND LOKESHWAR SINGH PANTA, JJ.
Civil Appeal Nos. 6831-6833 with 6834-6836 of 2001, D/- 30 -1 -2008.
Rajib Kumar Paul and Anr. v. Gurudas Mitra and Ors.
Calcutta Municipal Corporation Act (33 of 1951), S.416 - MUNICIPALITIES -
UNAUTHORISED CONSTRUCTION - Unauthorised construction - Demolition -
Residential building converted to commercial building - No sanction obtained - Owner
subsequently applying for regularisation - Building authorities passing order for
demolition - Plea that Order was passed without considering application for
regularisation - Not tenable as the order itself shows that the application was considered
and order was passed after considering provisions and hearing parties - Owner also not
entitled to benefit of new scheme as it deals with unauthorised use and not unauthorised
construction. (Para 7, 8)

Jaideep Gupta, Sr. Advocate, Sauvik Mandy, D. Bharat Kumar, Anand, Ishit Saharia,
Abhijit Sengupta C. Mukund, Ashok Kumar Jain, Pankaj Jain, Amit Khasera and Bijoy
Kumar Jain, for Appellants; Tapash Ray, Vijay Hansaria, Pradip Ghosh, Sr. Advocates,
Satish Vig, Soumya Chakravorty and Ranjan Mukherjee, for Respondents.
Judgement
H. K. SEMA, J. :- These appeals are directed against the judgment and order dated 18th
April, 2001 passed by the High Court of Calcutta in M.A.T. Nos. 1746, 1626 and 1627 of
1999.
2. We have heard the parties.
3. The undisputed facts are :
Premises No. 4H, Panchanantala Road, Calcutta, was originally sanctioned as residential.
It is also not disputed that the said building was subsequently converted from residential
to commercial (change of land use) unauthorizedly without obtaining sanction from the
appropriate authority. In the said building, the major portion of the garage space was
being used as commercial and the appellants have started business like M/s. Atithi
(Restaurant), M/s. Avinandan (Sweat meats), M/s. Avisar (Furniture) and M/s. Apsara
(Ladies Beauty Parlour). It is also not disputed that the appellants have also started
another business namely M/s. Aithi Guest House during June, 1990. It is alleged that the
said guest house has also been misused for immoral activities. We are not concerned.
4. The aforesaid admitted facts are born out from the application dated 29-6-1993
addressed to the Municipal Commissioner, the Calcutta Municipal Corporation. In the
said application, the appellants prayed for regularization of the entire irregularity and
requested to allow the change of use of the places from residential to commercial. The
application dated 29-6-1993 speaks for itself. It is extracted :
Rajib Kr. Paul and
Rajdeep Kr. Paul
4.H, Panchanathala Road,
Calcutta 700 029.
Dated, Calcutta 29th June, 1993
The Municipal Commissioner, The Calcutta Municipal Corporation, 5, B.N. Banerjee
Road, Calcutta-13
Dear Sir,
Re. : Premises No. 4H, Panchanantala Road, Calcutta 700 020.
Sub : Regularise the change of use from Residential to Commercial.
With reference to our discussion had with your kindself along with the City Architect at
your office on 26-06-93 at about 11 a.m. you were pleased to advise us to apply before
your kindself for change of use and occupation by us at the above-cited premises
including photocopy and measurement to regularize the existing position.
Original the building was sanctioned as Residential, but with effect from 5th October,
1989 in the major portion of the garage space of the said building were being used
@page-SC1197
as Commercial as we have started some business viz. M/s. ATITHI (Restaurant), M/s.
AVINANDAN (Sweat meats), M/s. AVISAR (Furniture) and M/s. APSARA (Ladies
Beauty Parlour) for which we have already obtained Certificate of Enlistment (Trade
Licence) for the year 1989-90 on 16-10-89, xerox copies of Certificate of Enlistment
(Trade Licence) as above are enclosed hereto for your record and ready reference.
We have started another business viz. M/s.AITHI GUEST HOUSE during June, 1990 and
Certificate of Enlistment (Trade Licence) for the same was obtained for the first time on
19-06-90 for the year 1990-91, a xerox copy of the same is also enclosed hereto for your
record and ready reference.
For regularization of the entire irregularity on our part as stated above we also furnish
herewith the actual measurement of each business place along with coloured photocopy
as advised by your kindself, which also you will find enclosed hereto.
In view of the fact as stated above and under the circumstances your honour is earnestly
requested to allow the change of use of the places as cited above from Residential to
Commercial, so that we may continue the business legally in view of C.M.C. Act is
concerned and thus make us free from hazards and encumbrances.
Thanking you in anticipation and a favourable consideration.
We remain,
Yours faithfully,
(Rajib Kr. Paul)
(Rajdeep Kr. Paul)
Encl. : 1. Plan showing the ground floor garage space and 3rd and 4th floor 2 flats
converted into 4 shop rooms and guest house respectively including measurement.
2. No. coloured photo showing all above converted commercial places.
3. Xerox copies of Trade Licence as above.
5. The Special Officer, Building, the Calcutta Municipal Corporation, after taking into
account the application for regularization and the complaint, directed to demolish the
unauthorized construction by its order dated 10-6-1994. Aggrieved thereby, the appellants
preferred an appeal before the Building Tribunal, which upholds the order of demolition
passed by the Special Officer, Building. Aggrieved thereby, Writ Petition No.21521(W)
of 1999 was preferred. The learned single Judge by its judgment and order dated 23-4-
1999 although upheld the order of the Tribunal, remanded the case to the Tribunal for
fresh consideration. The Division Bench of the High Court set aside the order of the
learned Single Judge and allowed the appeal. The Division Bench of the High Court
further directed the Corporation Authority to take immediate steps in terms of the order of
the Special Officer and confirmed by the Tribunal. The authority was further directed to
stop unauthorized use of the building either by guest house or by the bank or any other
organization and/ or firm and/or association under Section 416 of the Calcutta Municipal
Corporation Act, 1980.
6. The main thrust of arguments by the learned counsel for the appellants in
C.A.Nos.6831-6833 of 2001 is that the appellants' application dated 29-6-1993 for
regularization of the change of use from residential to commercial, was not disposed of
by the authority. However, by an order dated 10-6-1994 the Special Officer, Building,
without considering and disposing of the application dated 29-6-1993 ordered the
demolition of the unauthorized construction and, therefore, the order of demolition is not
tenable in law.
7. We are unable to countenance with this contention of the appellants. We have been
taken through the entire order of the Special Officer, Building, passed on 10-6-1994. On
perusal of the aforesaid order, it emerges that the Special Officer, Building, did take note
of the application filed by the appellants on 29-6-1993 and after having considered the
various provisions and after hearing the parties passed the order of demolition. The
consideration of the request of the appellants in C.A.Nos.6831-33 for regularization of
the unauthorized construction and unauthorized land is implicit in the order itself.
Therefore, it cannot be said that the demolition order dated 10-6-1994 has been passed
without considering the application dated 29-6-1993.
8. The next contention of the learned counsel for the appellants is that now a new scheme
has been formulated by the Corporation on 24-12-1998 in which it has been decided that
the use of the building in trades in deviation from the sanctioned use for residential
purpose may be regularized in case of existing trades being carried out with C.E. from
CMC Act, 1980. We may point out at
@page-SC1198
this stage that the same Circular dated 24-12-1998 placed before the High Court was
considered by the High Court and the claim of regularization was rejected. Apart from
that, the said Circular has expressly excluded the cases of unauthorized construction of
any building or part thereof and the same will be guided by the relevant provisions of
CMC Act, 1980. The present is the case of unauthorized construction of building and,
therefore, the aforesaid Circular dated 24-12-1998 is not applicable in the facts of the
present case at hand.
CIVIL APPEAL NOS. 6834-6836 OF 2001
9. These appeals have been preferred by the Allahabad Bank. Learned counsel for the
appellant strenuously urged that the appellant was not a party before the Single Judge and
the Division Bench, and, therefore, there was a clear violation of principles of natural
justice. This contention is belied by the fact. In the order dated 10-6-1994 passed by the
Special Officer, Building, it is clearly recorded that one Ashok Roy, Advocate, appeared
on behalf of the Allahabad Bank, a tenant in respect of the 1st and 2nd Floor of the
premises in question. It is also admitted by the counsel for the appellant that the
Allahabad Bank is functioning on the 1st and 2nd floor of the premises. Undisputedly, the
appellant-Bank is a tenant of the owner of the house. The bank has to sail or drown with
the owner of the house. When the entire building of four floors is ordered to be
demolished, the 1st and 2nd floors alone cannot survive. Therefore, there is no substance
in the contention of the appellant-Bank.
10. For the reasons aforestated, all these appeals are devoid of merits, and are,
accordingly dismissed. No costs.
Appeals dismissed.
AIR 2008 SUPREME COURT 1198 "Vijay Shankar Shinde v. State of Maharashtra"
(From : Bombay)*
Coram : 2 Dr. A. PASAYAT AND P. SATHASIVAM, JJ.
Criminal Appeal No. 95 of 2008 (airsing out of SLP (Cri.) No. 3394 of 2006), D/- 15 -1
-2008.
Vijay Shankar Shinde and Ors. v. State of Maharashtra.
(A) Penal Code (45 of 1860), S.300 - MURDER - EVIDENCE - WITNESS - Murder -
Evidence of eye-witnesses - Widow of deceased and injured exaggerating incident - But
testimony of other eye-witnesses clear and cogent - Witnesses withstanding lengthy
cross-examination - Conviction based on eye-witness evidence - Not liable to be
interfered with. (Paras 8, 9, 10)
(B) Evidence Act (1 of 1872), S.3 - EVIDENCE - WITNESS - Injured witness -
Testimony of - Holds more credence - Normally he would not shield real culprit. (Para
9)

Deepa Mahajan, Bhaskar Y. Kulkarni, for Appellants; Ravindra Kesharvao Adsure, for
Respondent.
* Cri. A. No. 253 of 1999, D/- 21-9-2004 (Bom).
Judgement
1. Dr. ARIJIT PASAYAT, J. :- Leave granted.
2. Challenge in this appeal is to the judgment passed by a Division Bench of Bombay
High Court, dismissing the appeal filed by the appellants who were convicted by learned
Additional Sessions Judge, Satara, for offences punishable under Sections 302, 307, 452
read with Section 34 of the Indian Penal Code, 1860 (in short 'IPC'). For the first two
offences each was sentenced to undergo imprisonment for life and to pay a fine with
default stipulation. For the offence relatable to Section 452, IPC, each was sentenced to
undergo imprisonment for one year and to pay a fine with default stipulation.
3. Prosecution case which led to the trial of the appellants was as follows :
There was family feud between the family of the accused and the family of the
complainant who were close relations. Suits are filed and suits were pending. On 10-10-
1996, around 12.00 p.m. Dattatraya one of the injured persons was assaulted by the
accused persons and on being stopped by mother-in-law of Dattatraya accused ran away.
Thereafter Tanaji (hereinafter referred to as deceased) came home and took Dattatraya by
rickshaw towards hospital. They were accosted at Gandhi Chowk by accused persons
who broke the glass of rickshaw, turned down the rickshaw, pulled out the victims and
assaulted them. This assault was witnessed by Jayashri, wife of Tanaji. She, therefore,
filed a complaint before the police. Investigation was started and on completion of
investigation, the accused persons were charged of having committed murder of the
deceased Tanaji and grievous hurt to Dattatraya.
4. In order to establish its version
@page-SC1199
prosecution examined 18 witnesses. PWs. 11, 12 and 13 were stated to be eye-witnesses
in addition to PW9 who also claimed to have witnessed a part of the incidence.
5. The Trial Court accepted the prosecution version and held that the evidence of PWs 11,
12 and 13 clearly establish the prosecution version. It was noted that PW 12 was injured
in the incident.
6. Before the High Court it was submitted that there were contradictions and omissions
falsifying the prosecution version. The High Court did not accept the version and upheld
the conviction and maintained their sentence.
7. In support of the appeal, leaned counsel for the appellants submitted that PWs 9
presence at the spot appears to be doubtful and in fact the Trial Court noted that she had
not seen the actual incident but after learning about the occurrence she came to the place
and her husband told her that it was the accused who had beaten him. It is also submitted
that PW11 had reason to falsely implicate the accused persons. Learned counsel for the
respondent-State on the other hand supported the judgment of the Trial Court as well as
the High Court.
8. Though the Trial Court observed that PWs 9 and 11 may have tried to exaggerate
because former was the widow and latter was injured victim, the evidence of PWs 12 and
13 establish the prosecution version.
9. The Trial Court was not justified in holding that because PW11 was an injured witness
he may have reason to falsely implicate the accused. However, as rightly observed by the
Trial Court and the High Court, the evidence of PWs 12 and 13 does not suffer from any
deficiency. PWs 11, 12 and 13 were cross-examined at length but nothing substantial
could be elicited to destroy the credibility of their version. As a matter of fact, the
evidence of injured person who is examined as a witness lends more credence, because
normally he would not falsely implicate a person thereby protecting the actual assailant.
10. The Trial Court as well as the High Court have rightly placed reliance on the evidence
of the eye-witnesses and as noted above their evidence was clear and cogent.
11. That being so, the impugned judgment of the High Court does not suffer from any
infirmity to warrant interference.
12. The appeal fails and is dismissed.
Appeal dismissed.
AIR 2008 SUPREME COURT 1199 "Venu v. State of Karnataka"
(From : Karnataka)*
Coram : 2 Dr. A. PASAYAT AND P. SATHASIVAM, JJ.
Criminal Appeal No. 221 of 2008 (arising out of SLP (Cri.) No. 6056 of 2007), D/- 30 -1
-2008.
Venu alias Venugopal and Ors. v. State of Karnataka.
(A) Penal Code (45 of 1860), S.390 - ROBBERY - Robbery - Robbery is only an
aggravated form of offence of theft or extortion - Aggravation is in use of violence of
death, hurt or restraint - Violence must be in course of theft and not subsequently - It is
not necessary that violence actually should be committed but even attempt to commit it is
enough. (Para 8)
(B) Penal Code (45 of 1860), S.390 - ROBBERY - WORDS AND PHRASES - THEFT -
Robbery - Causing death, hurt or wrongful restraint to commit theft - Words "for that
end" in S.390 clearly mean that hurt caused must be with object of facilitating
committing of theft - Or must be caused while offender is committing theft or is
attempting to carry away property obtained by theft. (Para 10)
(C) Penal Code (45 of 1860), S.392 - ROBBERY - Robbery - Proof - Appellants
allegedly intercepted victim and robbed gold and cash by threatening with knife -
Evidence of victim, her husband, factum of recovery of vehicle used clearly established
commission of offence by appellants - Offence was committed on a public road - It was
not a highway - Offence was committed at night - Conviction of appellants is proper.
2003 AIR SCW 6429, Relied on. (Para 12)
Cases Referred : Chronological Paras
2003 AIR SCW 6429 : AIR 2003 SC 433 (Rel. on) 13
Ms. Rachna Joshi (A.C.), for Appellants; Ms. Anitha Shenoy, for Respondent.
* Cri. A. No. 1146 of 2004, D/- 6-6-2006 (Kant).
Judgement
1. Dr. ARIJIT PASAYAT, J. :-Leave granted.
2. Challenge in this appeal is to the order of a learned Single Judge of the Karnataka High
Court holding the appellants
@page-SC1200
guilty of offence punishable under Section 392 of the Indian Penal Code, 1860 (in short
the 'IPC') and sentencing each to 10 years' imprisonment.
3. Prosecution version in a nutshell is as follows :
On 24-6-2001 at 9.00 p.m. on Mulbagal-Punganoor Road PWs 2 and 3 were going on a
Bajaj Scooter. When they were near 'Kirumani Mitta' of 'Buddadoru village", accused
persons 2 to 5 intercepted PWs 2 and 3, and robbed the gold chain, golden ear drops, thali
and cash of Rs. 400/- by threatening with knife. The accused tied the legs and hands of
PW-2 and PW-3 and threatened them not to escape and get out from the place for about
ten minutes after their departure. The victims went to Punganoor Police Station and later
on lodged First Information Report with Nangali Police (Kolar Dist.) on 25-6-2001. The
Traffic Police while checking found A-2, A-3 and A-4 were going on the scooter (M.O.6)
they had robbed from PW-2, the deadly weapons like knives, pistol, iron rod, etc. were
hidden in the scooter. On interrogation, the accused persons admitted the commission of
offence in question. A-5 and A-8 were arrested on the information given by A-2 to A-4.
At the instance of A-2, the gold jewellery (M.Os. 2 and 3) are recovered from PW-6-
Pawn-broker. The Bajaj Scooter (M.O.6) was seized from A-2, A-3 and A-4. PW-13 with
whom the ear-studs and the chain were pledged by A-2, testified to the said fact. PWs 2
and 3 identified A-2 to A-5 as the persons who robbed them. Prosecution claimed that the
identification of accused persons by PWs 2 and 3 coupled with the recovery of jewellery
at the instance of A-2 and seizure of scooter from A-2, A-3 and A-4 clinchingly
established the guilt of A-2 to A-5.
The investigating agency submitted charge-sheet for alleged commission of offence
punishable under Section 395 of IPC. The case was split up against A-1, A-6 and A-7 as
they were absconding.
Learned Additional Sessions Judge, Kolar referred to the evidence of PWs 1 and 2, the
recovery of the scooter, the recovery of stolen articles and identification thereon to
conclude that accused persons are guilty and accordingly A-2 to A-5 were convicted for
offence punishable under Section 395, IPC. Accused 7 and 8 were acquitted as the
evidence was not sufficient to find them guilty. Considering the gravity of the offence,
custodial sentence of 10 years' imprisonment and a fine of Rs. 5,000/- each was imposed.
In appeal, the High Court found that the offence committed was covered under Section
392, IPC, but considering the gravity of the offence upheld the sentence.
4. In support of the appeal, learned counsel for the appellants submitted that the evidence
of PWs 2 and 3 does not show that any knife was used for robbery. On the contrary,
evidence of victim clearly shows that she raised hue and cry when accused persons tried
to snatch the stolen articles from her. It was also submitted that the appellants have
suffered custody of more than nearly 8 years and the sentence deserves to be reduced to
the period already undergone.
5. Learned counsel for the respondent-State on the other hand submitted that there is no
minimum sentence prescribed and the maximum sentence is 10 years. It is submitted that
the robbery was committed on the highway at about 9.00 p.m. That being so, the sentence
can be upto 14 years. Considering the gravity of the offence and the large scale highway
robberies, no leniency should be shown.
6. Section 392, IPC provides for punishment for robbery. The essential ingredients are as
follows :
1. Accused committed theft;
2. Accused voluntarily caused or attempted to cause.
(i) death, hurt or wrongful restraint.
(ii) Fear of instant death, hurt or wrongful restraint.
3. He did either act for the end. (i) to commit theft.
(ii) While committing theft.
(iii) In carrying away or in the attempt to carry away property obtained by theft.
7. It is to be noted that the Section 392 provides punishment for robbery. It is punishment
for the offence defined in Section 390. Punishment is higher if it is committed on a
highway and between sunset and sunrise. Section 390 which defines "robbery" reads as
follows :
390. Robbery. - In all robbery there is either theft or extortion.
When theft is robbery. - Theft is "robbery" if, in order to the committing of the theft, or in
committing the theft, or in carrying away
@page-SC1201
or attempting to carry away property obtained by the theft, the offender, for that end,
voluntarily causes or attempts to cause to any person death or hurt or wrongful restraint,
or fear of instant death or of instant hurt, or of instant wrongful restraint.
When extortion is robbery - Extortion is "robbery" if the offender, at the time of
committing the extortion, is in the presence of the person put in fear, and commits the
extortion by putting that person in fear of instant death, of instant hurt, or of instant
wrongful restraint to that person or to some other person, and, by so putting in fear,
induces the person so put in fear then and there to deliver up the thing extorted.
Explanation - The offender is said to be present if he is sufficiently near to put the other
person in fear of instant death, of instant hurt, or of instant wrongful restraint."
8. The provision defines robbery which is theft or extortion when caused with violence of
death, hurt or wrongful restraint. When there is no theft committed, then as a natural
corollary there cannot be robbery. Robbery is only an aggravated form of offence of theft
or extortion. Aggravation is in the use of violence of death, hurt or restraint. Violence
must be in course of theft and not subsequently. It is not necessary that violence actually
should be committed but even attempt to commit it is enough.
9. The authors of the Code observed as follows :
"In one single class of cases, theft and extortion are in practice confounded together so
inextricably, that no judge, however, sagacious, could discriminate between them. This
class of cases, therefore, has, in all systems of jurisprudence ... been treated as a perfectly
distinct class ... we have, therefore, made robbery a separate crime.
There can be no case of robbery which does not fall within the definition either of theft or
of extortion; but in a practice it will perpetually be a matter of doubt whether a particular
act of robbery was a theft or an extortion. A large proportion of robberies will be half
theft, half extortion. A seizes Z, threatens to murder him, unless he delivers all his
property, and begins to pull off Z's ornaments. Z in terror begs that A will take all he has,
and spare his life, assists in taking off his ornaments, and delivers them to A, Here, such
ornaments as A took without Z's consent are taken by theft. Those which Z delivered up
from fear of death are acquired by extortion. It is by no means improbable that Z's right
arm bracelet may have been obtained by theft, and left-arm bracelet by extortion; that the
rupees in Z's girdle may have been obtained by theft, and those in his turban by extortion.
Probably in nine-tenths of the robberies which are committed, something like this
actually takes place, and it is probable that a few minutes later neither the robber nor the
person robbed would be able to recollect in what proportions theft and extortion were
mixed in the crime; nor is it at all necessary for the ends of justice that this should be
ascertained. For though, in general, the consent of a sufferer is a circumstance which very
materially modifies the character of the offence, and which ought, therefore, to be made
known to the Courts, yet the consent which a person gives to the taking of this property
by a ruffian who holds a pistol to his breast is a circumstance altogether immaterial".
10. The words "for that end" in Section 390 clearly mean that the hurt caused must be
with the object of facilitating the committing of the theft or must be caused while the
offender is committing theft or is carrying away or is attempting to carry away property
obtained by the theft.
11. As the provision itself provides when the highway robbery is committed, deterrent
punishment is called for.
12. In the instant case, the evidence of the victim, her husband, the factum of recovery of
the vehicle used has clearly established the commission of offence by the appellants. The
offence was committed on a public road. There is no dispute that it was not a highway. It
is also not in dispute that the offence was committed during sunset and sunrise that is, at
about 9.00 p.m.
13

. In State of Karnataka v. Puttaraja (2004 (1) SCC 475), it was inter-alia observed as
follows : 2003 AIR SCW 6429, Para 12

"Imposition of sentence without considering its effect on the social order in many cases
may be in reality a futile exercise. The social impact of the crime e.g. where it relates to
offences against women like the case at hand, dacoity, kidnapping, misappropriation of
public money, treason and other offences involving moral turpitude or moral delinquency
which have great impact and
@page-SC1202
serious repercussions on social order and public interest, cannot be lost sight of and per se
require exemplary treatment. Any liberal attitude by imposing meager sentences or taking
too sympathetic a view merely on account of lapse of time or considerations personal to
the accused only in respect of such offences will be resultwise counterproductive in the
long run and against societal interest which needs to be cared for and strengthened by the
required string of deterrence inbuilt in the sentencing system."
14. Above being the position, there is no merit in this appeal which is accordingly
dismissed.
Appeal dismissed.
AIR 2008 SUPREME COURT 1202 "Hari Prasad Bhuyan v. Durga Prasad Bhuyan"
(From : Gauhati)
Coram : 2 Dr. A. PASAYAT AND D. K. JAIN, JJ.
Civil Appeal No. 768 of 2008 (arising out of SLP (C) No. 8870 of 2006), D/- 29 -1 -2008.
Hari Prasad Bhuyan v. Durga Prasad Bhuyan and Ors.
Civil P.C. (5 of 1908), O.22, R.9 - DECREE - Abatement of suit - Setting aside -
Condonation of delay - Plaintiff getting information about death of defendant at time of
rectification of decree - Applying immediately for substitution - Defendants though had
knowledge of death not disclosing fact in earlier round of litigation - Cannot take
advantage from belated approach of plaintiff - Delay liable to be condoned - Order
declaring decree to be nullity on ground of non- substitution of parties liable to be set
aside.
Misc. Case No. 58 of 2003 in S.A, No. 80 of 1986 (Gau.), Reversed. (Para 6)
Cases Referred : Chronological Paras
2002 AIR SCW 4843 : AIR 2003 SC 351 (Ref.) 3
(1885) 30 Ch D 239 3
Rana Mukherjee, Azim H. Laskar, Anand, D. Bharat Kumar and Abhijit Sengupta,
Advocates, for Appellant.
Judgement
1. Dr. ARIJIT PASAYAT, J. :-Leave granted.
2. Challenge in this appeal is to the order passed by a learned Single Judge of the Gauhati
High Court, dismissing the applications for condonation of delay, setting aside of
abatement and substitution of the heirs of the respondent Nos. 13 and 24 in the Second
Appeal No. 80/1986. It was held that the appeal had abated and the judgment and order
dated 18-5-1995 passed by the High Court in Second Appeal No. 80/ 1986 was a nullity
and, therefore, application under Section 152 of the Code of Civil Procedure, 1908 (in
short the "CPC") was not maintainable.
3. Background facts in a nutshell are as follows :
Predecessors-in-interest of the appellant filed Suit TS No. 26/1978 in the Court of
Assistant District Judge No. 1, Gauhati. The said suit, inter alia, was for recovery of
possession, confirmation of possession and declaration of title over the suit properties and
for cancellation of mutation of names of certain defendants. According to the appellant,
the said suit specifically set out the cause of action against each defendant and the prayers
in the suit were also specifically directed against the defendants in respect of the alleged
holding in the scheduled properties. The Trial Court by judgment dated 11-1-1984
dismissed the suit. An appeal was preferred which was numbered as Appeal No. 5/1984
and the same was dismissed by learned District Judge, Gauhati by order dated 30-1-1986.
Plaintiffs filed a Second Appeal No. 80 of 1986 in the Gauhati High Court. During
pendency of the same, some of the plaintiffs died and their legal heirs were substituted.
The Second Appeal filed by the plaintiffs was allowed by the Gauhati High Court and the
suit was decreed. Plaintiffs filed an Execution Petition before the Trial Court which was
numbered as Title Execution Case No. 4 of 1995. The Trial Court drew up the decree
dated 7-4-1996 as directed by the High Court, but mistakenly set out only costs without
setting out the reliefs in the suit which had been decreed. An S.L.P. (CC No. 2275/96)
filed by the respondents against the judgment and order dated 18-8-1995 passed by the
High Court was dismissed by order dated 8-5-1996 with the following observations :
"The Ld. Counsel for the petitioner submits that the petitioners have been advised to
approach the High Court for recall of the order and he had instructions to withdraw this
Special Leave Petition. We record the statements of the Ld. Counsel and dismiss the
Special Leave Petition as withdrawn." In the Execution Petition filed by the appellants
objection under Section 47, CPC was
@page-SC1203
filed on behalf of the heirs of deceased respondent No. 7 and the Trial Court by an order
disposed of the said application, inter alia, observing as follows :
"In the light of the above, I am of the considered view that the decree cannot be executed
in respect of the E Schedule on the ground of nullity but the decree will be executable in
respect of other properties as mentioned in the plaint except those in Schedule E and
against the other judgment-debtors. With this order, the petition stands disposed of. Steps
be taken for execution of the decree."

On 26-8-1997 the trial Court by two separate orders in the suit in the execution
proceedings observed that decree should have contained all the reliefs claimed and
ordered accordingly. On 17-11-1997 the decree was drawn up as per the order dated 26-8-
1997. Respondent No. 6 i.e. Laxmi Ram Bhuyan filed a Civil Revision (CR No.
423/1997) in the Gauhati High Court questioning orders dated 26-8-1997 and decree
dated 17-11-1997. By order dated 29-9-1999 the High Court dismissed the Civil
Revision. A petition was filed seeking review of the High Court's order dated 29-9-1999
in RP No. 6 of 2000. A Special Leave Petition was filed against the order dated 10-4-
2001, by which the High Court rejected the review Petition. On 20-11-2002 this Court
granted liberty to the appellants to approach the High Court under Section 152, CPC for
making appropriate corrections in the decree. The judgment is reported in Lakshmi Ram
Bhuyan vs. Hari Prasad Bhuyan and Ors. (2003 (1) SCC 197). It was inter alia noted as
follows : 2002 AIR SCW 4843

"11. The obligation is cast not only on the trial court but also on the appellate court. In the
event of the suit having been decreed by the trial court if the appellate court interferes
with the judgment of the trial court, the judgment of the appellate court should precisely
and specifically set out the reliefs granted and the modifications, if any, made in the
original decree explicitly and with particularity and precision. Order XLI, Rule 31, CPC
casts an obligation on the author of the appellate judgment to state the points for
determination, the decision thereon, the reasons for the decision and when the decree
appealed from is reversed or varied, the relief to which the appellant is entitled. If the suit
was dismissed by the trial court and in appeal the decree of dismissal is reversed, the
operative part of the judgment should be so precise and clear as it would have been if the
suit was decreed by the trial court to enable a self-contained decree being drawn up in
conformity therewith. The plaintiff, being dominus litus, enjoys a free hand in couching
the relief clause in the manner he pleases and cases are not wanting where the plaintiff
makes full use of the liberty given to him. It is for the court, decreeing the suit, to
examine the reliefs and then construct the operative part of the judgment in such manner
as to bring the reliefs granted in conformity with the findings arrived at on different
issues and also the admitted facts. The trial court merely observing in the operative part
of the judgment that the suit is decreed or an appellate court disposing of an appeal
against dismissal of suit observing the appeal is allowed, and then staying short at that,
without specifying the reliefs to which the successful party has been found entitled
tantamounts to a failure on the part of the author of the judgment to discharge obligation
cast on the Judge by the provisions of the Code of Civil Procedure.
12. In the case at hand, a perusal of the reliefs prayed for in the plaint shows that the
reliefs are not very happily worded. There are some reliefs which may not be necessary
or may be uncalled for, though prayed for. The reliefs may have been considered capable
of being recast or redefined so as to be precise and specific. May be, that the Court was
inclined to grant some other relief so as to effectually adjudicate upon the controversy
and bring it to an end. Nothing is spelled out from the appellate judgment. The trial court,
on whom the obligation was cast by the second appellate judgment to draw up a decree,
was also, as its order shows, not very clear in its mind and thought it safe to proceed on
an assumption that all the reliefs sought for in the plaint were allowed to the plaintiffs.
The learned Single Judge allowing the second appeal, should have clearly and precisely
stated the extent and manner of reliefs to which the plaintiffs were found to be entitled in
his view of the findings arrived at during the course of the appellate judgment. The
parties, the draftsman of the decree and the executing court cannot be left guessing what
was transpiring in the mind of the Judge decreeing the suit or allowing the appeal without
further placing on record the reliefs to which the plaintiffs are held entitled in the Opinion
of
@page-SC1204
the Judge.
13. There is yet another infirmity. Ordinarily the decree should have been drawn up by
the High Court itself. It has not been brought to the notice of this Court by the learned
counsel for either parties if there are any rules framed by the High Court which
countenance such a practice as directing the trial court to draw up a decree in conformity
with the judgment of the High Court.
14. How to solve this riddle? In our opinion, the successful party has no other option but
to have recourse to Section 152, CPC which provides for clerical or arithmetical mistakes
in judgments, decrees or orders or errors arising therein from any accidental slip or
omission being corrected at any time by the court either on its own motion or on the
application of any of the parties. A reading of the judgment of the High Court shows that
in its opinion the plaintiffs were found entitled to succeed in the suit. There is an
accidental slip or omission in manifesting the intention of the court by couching the
reliefs to which the plaintiffs were entitled in the event of their succeeding in the suit.
Section 152 enables the court to vary its judgment so as to give effect to its meaning and
intention. Power of the court to amend its orders so as to carry out the intention and
express the meaning of the Court at the time when the order was made was upheld by
Bowen, L.J. in Swire, Re, Mellor v. Swire subject to the only limitation that the
amendment can be made without injustice or on terms which preclude injustice. Lindley,
L.J. observed that if the order of the court, though drawn up, did not express the order as
intended to be made then
"there is no such magic in passing and entering an order as to deprive the court of
jurisdiction to make its own records true, and if an order as passed and entered does not
express the real order of the court, it would, as it appears to me, be shocking to say that
the party aggrieved cannot come here to have the record set right, but must go to the
House of Lords by way of appeal."
15. For the foregoing reasons the appeal is allowed. The order of the trial court drawing
up the decree is set aside. The parties are allowed liberty of moving the High Court under
Section 152, CPC seeking appropriate rectification in the judgment of the High Court so
as to clearly specify the extent and manner of reliefs to which in the opinion of the High
Court the successful party was found entitled consistently with the intention expressed in
the judgment. The delay which would be occasioned has to be regretted but is
unavoidable. Once the operative part of the judgment is rectified there would be no
difficulty in drawing up a decree by the High Court itself in conformity with the operative
part of the judgment. If the rules of the High Court so require, the ministerial act of
drawing up of the decree may be left to be performed by the trial Court."
4. Accordingly the application was filed under Section 152, CPC before the High Court.
On 26-6-2003, according to the appellant, he came to know about the death of respondent
Nos. 13 and 24 in February 1999 and 1993 respectively. This according to the appellant
came to the knowledge of the appellant from the report of the Process Server dated 26-6-
2003. On 2-8-2003 the appellant filed application for setting aside the abatement,
substitution and for condonation of delay. By the impugned order, the learned Single
Judge while dealing with application under Section 152 CPC declared the decree to be a
nullity on account of death of respondent Nos. 13 and 24 and the belated approach for
bringing their legal heirs on record.
5. Learned counsel for the appellant submitted that the High Court has missed several
relevant factors. Firstly, in the earlier round of litigation which resulted in the decision
Lakshmi Ram Bhuyan's case (supra) it was not pointed out by the respondents about the
death of respondent No. 13 or respondent No. 24. The present respondents were the
appellants in the appeal before this Court. They also did not point out about the death.
There is no decree which was to be drawn up in line with this Court's judgment.
6. There is no dispute regarding the assertion of the appellant that he came to know about
the death of respondents 13 and 24 from the process servers report. Before this Court
earlier also respondents did not disclose about their death. Since that has not been done,
respondents cannot take any advantage from the belated approach by the appellant. This
according to us is a clear case where the prayer for condonation of delay in seeking
substitution by setting aside abatement and condonation of delay should have been
accepted by the High Court. The
@page-SC1205
High Court's order is set aside. The appeal is allowed. There will be no order as to costs.
Appeal allowed.
AIR 2008 SUPREME COURT 1205 "Balasaheb Dayandeo Naik v. Appasaheb Dattatraya
Pawar"
(From : Bombay)
Coram : 2 Dr. A. PASAYAT AND P. SATHASIVAM, JJ.
Civil Appeal No. 647 of 2008 (arising out of SLP (C) No. 16694 of 2005), D/- 24 -1
-2008.
Balasaheb Dayandeo Naik (Dead) Through L.Rs. and Ors. v. Appasaheb Dattatraya
Pawar,
(A) Specific Relief Act (47 of 1963), S.12 - Contract Act (9 of 1872), S.55 - CONTRACT
- AGREEMENT TO SELL - EXECUTION - SALE DEED - Specific performance - Suit
for - Agreement to sell - Defence that time was essence of contract and plaintiff failed to
pay within time - Agreement though specified time for execution of sale deed -
Stipulation for forfeiture of earnest money in event of failure to execute sale deed also
made - Indicates that time was never intended by parties to be of essence - Defendant
could not substantiate plea that time was essence of contract and that it was plaintiffs who
avoided to perform their part - Plaintiffs are, therefore, entitled to decree.
1997 AIR SCW 956, Disting.
(2004) 8 SCC 689, 1993 AIR SCW 1371, Relied on.
F.A. No. 743 of 1993. D/-11-1-2005 (Bom), Reversed. (Paras 10, 12, 13, 14)
(B) Specific Relief Act (47 of 1963), S.22 - CONTRACT - DECREE - Specific
performance - Suits for - Prayer for refund of earnest money - Plaintiff is entitled to seek
alternative relief of refund of earnest money in event decree for specific performance
cannot be granted for any reason. (Para 13)
Cases Referred : Chronological Paras
(2004) 8 SCC 689 (Rel. on) 11
1997 AIR SCW 956 : AIR 1997 SC 1751 (Disting.) 12
1993 AIR SCW 1371 : AIR 1993 SC 1742 (Rel. on) 9, 13
AIR 1988 SC 1074 9
AIR 1977 SC 1005 9
AIR 1967 SC 868 9
Makarand D. Adkar, Vijay Kumar and Vishwajit Singh, for Appellants; V.N. Ganpule, Sr.
Advocate, S.B. Meitei, Naresh Kumar Gaur and Ashok Kumar Singh with him, for
Respondent.
Judgement
1. P. SATHASIVAM, J. :-Leave granted.
2. This appeal is directed against the judgment and order dated 11-01-2005 passed by the
High Court of Judicature at Bombay in First Appeal No. 743 of 1993 in and by which the
High Court set aside the decree for specific performance granted by the trial Court and
consequently dismissed the suit of the plaintiffs.
3. Brief facts in a nutshell are :
The appellants/plaintiffs in Special Civil Suit No. 320 of 1988 filed the same for specific
performance of agreement dated 31-07-1985. According to the plaintiffs, the respondent
herein/defendant is the owner of land Block No. 208 and Block No. 209 respectively
admeasuring Area H. 0.60 R and H. 0.40 R of Village Nagaon in Hatkanangale Tahsil.
The defendant had entered into an agreement for sale of the said lands to the plaintiffs for
a consideration of Rs. 85,000/ - per acre. The agreement was reduced into writing and
according to the terms of the agreement, the sale deed was to be executed by the
defendant within a period of six months. It was agreed that possession of the lands was to
be delivered at the time of execution of sale deed. The defendant has also undertaken the
responsibility of obtaining necessary permission for sale of the lands, if required. On the
date of execution of the agreement, an amount of Rs. 20,000/- was paid by the plaintiffs
to the defendant as earnest money and balance amount of the consideration was to be
paid at the time of execution of the sale deed. The plaintiffs were always ready and
willing to perform their part of the contract but the defendant avoided to receive the
balance amount of consideration and neglected to execute the sale deed. The plaintiffs
sent a legal notice on 16-07-1988 to the defendant through their advocate calling upon
him to perform his part of the obligation under the contract. In spite of the notice, the
defendant did not comply with the requirements which necessitated the plaintiffs to file
the suit for specific performance or in the alternative refund of earnest money with
interest thereon @ 15% per annum.
4. The defendant filed a written statement wherein he denied the plaintiffs claim.
@page-SC1206
It was further stated that though agreement for sale of the suit lands was entered into
between him and the plaintiffs on 31-07-1985, the sale deed was to be executed within a
period of six months from the date of contract as he was in dire need of money for
construction of his house and, therefore, the time was the essence of the contract. He had
called upon the plaintiffs to pay the balance amount of consideration and get the sale deed
executed. But the plaintiffs were not in a position to arrange the balance amount of
consideration and complete the contract. As the market price of the agricultural lands
have now gone up, the plaintiffs by purchasing the suit lands are intending to dispose of
the same to others at a higher price. In view of the same, the plaintiffs are not entitled to
discretionary relief of specific performance of contract.
5. The learned Civil Judge (Senior Division), on 23-02-1993, after finding that the
defendant has failed to prove that time was the essence of contract and the plaintiffs were
and are ready and willing to perform their part of contract decreed the suit as prayed for.
Aggrieved by the aforesaid judgment of the trial Court, the defendant filed First Appeal
No. 743 of 1993 before the High Court of Judicature at Bombay. The learned Single
Judge of the High Court not in agreement with the conclusion of the trial Court and
finding that plaintiffs failed to substantiate their plea allowed the appeal of the defendant
and dismissed the suit. Questioning the judgment and order of the High Court, the
plaintiffs have filed the present appeal by way of special leave. During the pendency of
the appeal before this Court, Balasaheb Dayandeo Naik/first plaintiff died and his legal
representatives were brought on record as per order dated 19-09-2006 in I.A. No. 3 of
2005.
6. We heard Mr. Makarand D. Adkar, learned counsel appearing for the appellants and
Mr. V.N. Ganpule, learned senior counsel appearing for the respondent, perused the entire
annexures and other relevant materials filed before this Court.
7. Having regard to the terms of agreement of sale dated 31-07-1985, reasonings of the
trial Court as well as the High Court and submissions before this Court, only two points
arise for consideration of this Court, namely, (a) whether time is the essence of the
contract? and (b) whether the plaintiffs were ready and willing to perform the contract ?
8. In order to find an answer to the above questions, it would be useful to refer the
relevant recitals from the agreement of sale. Para 3 of the agreement specifically
mentions the details of the land sought to be sold such as extent and boundaries. It also
refers the easement rights and the period in which the sale has to be completed. The
recital reads as under :-
"From the total consideration I have received Rs.20,000/- as an earnest money of which
no independent receipt is necessary. Rest of the amount is to be paid by you at the time of
sale deed of the said lands. It is agreed between the parties that the sale deed is to be
executed within 6 months from today. Possession of the land is to be handed over at the
time of sale deed."
It is also relevant to mention the default clause which reads as under :-
"For completion of the sale deed the permission is required to be obtained by me. If I fail
to execute the said deed within stipulated period then you have to get it executed on the
basis of this agreement. On the contrary if you fail to get execute the sale deed then this
agreement is supposed to be cancelled and the earnest amount will be forfeited. The land
is free from all sorts of encumbrances. This agreement is binding on myself and my legal
heirs etc. dated 31-7-1985."
The above-mentioned details in the agreement of sale clearly show a) that the subject-
matter of the property is an agricultural land/immoveable properties b) the sale deed is to
be executed within six months from the date of sale agreement i.e. 31-07-1985. c)
possession of the land to be handed over at the time of execution of sale deed d) failure to
get execute the sale deed, the earnest money will be forfeited. With these factual details,
let us consider the legal principles enunciated by this Court.
9

. In Chand Rani (Smt.) (dead) by L.Rs. Vs. Kamal Rani (Smt.) (dead) by L.Rs., (1993) 1
SCC 519, a Constitution Bench of this Court has held that in the sale of immoveable
property, time is not the essence of the contract. It is worthwhile to refer the following
conclusion : 1993 AIR SCW 1371, Paras 18, 20 and 22
"19. It is a well-accepted principle that in the case of sale of immovable property, time is
never regarded as the essence of the contract.
@page-SC1207
In fact, there is a presumption against time being the essence of the contract. This
principle is not in any way different from that obtainable in England. Under the law of
equity which governs the rights of the parties in the case of specific performance of
contract to sell real estate, law looks not at the letter but at the substance of the
agreement. It has to be ascertained whether under the terms of the contract the parties
named a specific time within which completion was to take place, really and in substance
it was intended that it should be completed within a reasonable time. An intention to
make time the essence of the contract must be expressed in unequivocal language."

"21. In Govind Prasad Chaturvedi v. Hari Dutt Shastri (1977) 2 SCC 539 following the
above ruling it was held at pages 543-544: (SCC para 5) AIR 1977 SC 1005 at pp.
1007-08

It is settled law that the fixation of the period within which the contract has to be
performed does not make the stipulation as to time the essence of the contract. When a
contract relates to sale of immovable property it will normally be presumed that the time
is not the essence of the contract. [Vide Gomathinayagam Pillai v. Pallaniswami. Nadar (a
p. 233).] It may also be mentioned that the language used in the agreement is not such as
to indicate in unmistakable terms that the time is of the essence of the contract. The
intention to treat time as the essence of the contract may be evidenced by circumstances
which are sufficiently strong to displace the normal presumption that in a contract of sale
of land stipulation as to time is not the essence of the contract." AIR 1967 SC 868 at
p. 871

23. In Indira Kaur (Smt.) v. Sheo Lal Kapoor (1988) 2 SCC 488 in paragraph 6 it was
held as under : AIR 1988 SC 1074, Para 6

"... The law is well-settled that in transactions of sale of immovable properties, time is not
the essence of the contract."
10. It is clear that in the case of sale of immoveable property, there is no presumption as
to time being the essence of the contract. Even where the parties have expressly provided
that time is the essence of the contract, such a stipulation will have to be read along with
other provisions of the contract. For instance, if the contract was to include clauses
providing for extension of time in certain contingencies or for payment of fine or penalty
for every day or week, the work undertaken remains unfinished on the expiry of the time
provided in the contract, such clauses would be construed as rendering ineffective the
express provision relating to the time being of the essence of contract. In the case on
hand, though the parties agreed that the sale deed is to be executed within six months, in
the last paragraph they made it clear that in the event of failure to execute the sale deed,
the earnest money will be forfeited. In such circumstances, the above-mentioned clauses
in the last three paragraphs of the agreement of sale would render ineffective the specific
provision relating to the time being the essence of contract.
11. This Court in Swarnam Ramachandran (Smt.) and Another vs. Aravacode Chakungal
Jayapalan, (2004) 8 SCC 689 has once again reiterated that time is not the essence of
contract relating to immoveable property. The following statement of law in para 12 are
rightly applicable to the case on hand :
"12. That time is presumed not to be of essence of the contract relating to immovable
property, but it is of essence in contracts of reconveyance or renewal of lease. The onus to
plead and prove that time was the essence of the contract is on the person alleging it, thus
giving an opportunity to the other side to adduce rebuttal evidence that time was not of
essence. That when the plaintiff pleads that time was not of essence and the defendant
does not deny it by evidence, the court is bound to accept the plea of the plaintiff. In
cases where notice is given making time of the essence, it is duty of the court to examine
the real intention of the party giving such notice by looking at the facts and circumstances
of each case. That a vendor has no right to make time of the essence, unless he is ready
and willing to proceed to completion and secondly, when the vendor purports to make
time of the essence, the purchaser must be guilty of such gross default as to entitle the
vendor to rescind the contract."
12

. As observed in the said decision, in the case on hand the appellants/plaintiffs clearly
established their claim to secure specific performance of the agreement by leading cogent
evidence whereas the respondent 1997 AIR SCW 956

@page-SC1208
defendant having pleaded that time was the essence of the contract neither entered the
witness box nor led any evidence whatsoever. The High Court lost sight of the above
material aspect and the conduct of the defendant in not strengthening his plea by placing
acceptable evidence. In such circumstances, as rightly argued by learned counsel for the
appellants, the High Court should have confirmed the decree of specific performance
granted by the trial Court. On the other hand, the High Court wrongly placed reliance on
the decision of this Court in K.S. Vidyanadam and Others vs. Vairavan, (1997) 3 SCC 1
as in the facts of that case, this Court found that granting for specific performance was
inequitable, however such aspect of the matter was totally absent in the case on hand.
Even otherwise, para 11 of the judgment shows that the subject-matter of the property
was an urban immoveable property and in such special circumstance relaxed the general
rule that time is not the essence of the contract in the case of immoveable properties. In
the case on hand, the details furnished in the agreement clearly show that the subject-
matter of the property is an agricultural land situated in Kolhapur Dist., Maharashtra. In
such circumstances, the decision in K.S. Vidyanadam and Ors. (supra) is not applicable to
the facts on hand. In the facts of the present case, which we have already adverted to,
neither the terms of agreement nor the intention of the parties indicate that the time is an
essence of the agreement. We have already pointed that having raised such a plea the
respondent even did not bother to lead any evidence.
13
. It is true that the defendant in his written statement has made a bald claim that the time
was the essence of contract. Even if we accept the recital in the agreement of sale (Exh.
18) that the sale deed has to be executed within a period of six months, there is an express
provision in the agreement itself that failure to adhere the time, the earnest money will be
forfeited. In such circumstances and in view of recital pertaining to forfeiture of the
earnest money makes it clear that time was never intended by the parties to be of essence.
The Constitution Bench decision in Chand Rani vs. Kamal Rani (supra) also makes it
clear that mere fixation of time within which contract is to be performed does not make
the stipulation as to the time as the essence of contract. Further, we have already pointed
out that the defendant has not bothered to prove his claim on oath before the Court to the
effect that it was the plaintiffs who avoided performing their part of contract. All the
abovementioned material aspects were correctly appreciated by the trial Court and
unfortunately the High Court failed to adhere to the well known principles and the
conduct of the defendant. When the third plaintiff deposed before the Court explaining
their case with reference to the recitals in the agreement of sale including the reference to
the legal notice to the defendant, in the absence of contra evidence on the side of the
defendant, we are unable to agree with the conclusion arrived at by the High Court in
nonsuiting the plaintiff. The High Court commented the conduct of the plaintiffs in
praying for refund of the earnest money, namely, Rs.20,000/- paid as advance. As rightly
pointed out, the claim for refund of earnest money is only their alternative claim. It is not
in dispute that in all suits for specific performance, the plaintiff is entitled to seek
alternative relief in the event the decree for specific performance cannot be granted for
any reason, hence there is no infirmity in the alternative plea of refund. 1993 AIR
SCW 1371

14. In the light of what has been stated above, we set aside the judgment and decree of
the High Court and confirm the decree granted by the trial Court. In view of the said
conclusion, the appellants/plaintiffs are directed to deposit the balance amount of sale
consideration i.e., Rs. 1,92,500/- in the trial Court within a period of eight weeks
whereupon the respondent/defendant shall execute the sale deed of the suit lands Block
No. 208 admeasuring 0.60 R and Block No. 209 admeasuring 0.40 R of Village Nagaon,
Tahsil Hatkanangale as per the agreement dated 31-07-1985. In case of failure of the
defendant to execute the sale deed, the plaintiffs shall be entitled to get the sale deed
executed through Court.
15. The civil appeal is allowed on the above terms. However, in the facts and
circumstances of the case, there shall be no order as to costs.
Appeal allowed.
@page-SC1209
AIR 2008 SUPREME COURT 1209 "State of Punjab v. Jalour Singh"
(From : Punjab and Haryana)
Coram : 3 K. G. BALAKRISHNAN, C.J.I., G. P. MATHUR AND R. V.
RAVEENDRAN, JJ.
Civil Appeal No. 522 of 2008 (arising out of SLP (C) No. 3847 of 2005), D/- 18 -1 -2008.
State of Punjab and Anr. v. Jalour Singh and Ors.
(A) Legal Services Authorities Act (39 of 1987), S.20 - LEGAL SERVICES
AUTHORITIES - LOK ADALAT - Lok Adalat - Have no adjudicatory or judicial
function - Its role is to guide and persuade parties to reach at a compromise or settlement
- Act of conducting Lok Adalat as Courts, hearing parties and imposing its view on
parties would drive away litigants from Lok Adalats.
Lok Adalats have no adjudicatory or judicial functions. Their functions relate purely to
conciliation. When the Act refers to 'determination' by the Lok Adalat and 'award' by the
Lok Adalat, the said Act does not contemplate nor require an adjudicatory judicial
determination, but a non-adjudicatory determination based on a compromise or
settlement, arrived at by the parties, with guidance and assistance from the Lok Adalat.
The 'award' of the Lok Adalat does not mean any independent verdict or opinion arrived
at by any decision-making process. The making of the award is merely an administrative
act of incorporating the terms of settlement or compromise agreed by parties in the
presence of the Lok Adalat, in the form of an executable order under the signature and
seal of the Lok Adalat. Conducting Lok Adalat like Courts, by hearing parties, and
imposing their views as to what is just and equitable, on the parties, instead of fostering
alternative dispute resolution through Lok Adalats, will drive the litigants away from Lok
Adalats. Lok Adalats should, therefore, resist their temptation to play the part of Judges
and constantly strive to function as conciliators. (Paras 8, 9)
(B) Legal Services Authorities Act (39 of 1987), S.21 - LEGAL SERVICES
AUTHORITIES - LOK ADALAT - WRITS - HIGH COURT - Award passed by Lok
Adalat - Not appealable - Can only be challenged by filing petition under Art.226 or
Art.227 of Constitution. (Para 12)
(C) Legal Services Authorities Act (39 of 1987), S.20 - Motor Vehicles Act (59 of 1988),
S.173 - LEGAL SERVICES AUTHORITIES - MOTOR VEHICLES - APPEAL -
DISMISSAL - Appeal against accident claim award - Transferred to Lok Adalat - Lok
Adalat, in absence of compromise, passing award enhancing compensation - Liberty
given to party to approach High Court for disposal of appeal in case of disagreement -
Application made to High Court for disposal of pending appeal - Dismissal on ground
that award is binding and application made was not tenable - Improper - Dismissal of
petition under Art.227 subsequently filed against award on ground that award has become
final as objections against it have been dismissed - Also improper - Award of Lok Adalat
as also orders of High Court liable to be set aside.
C.R.P. No. 970 of 2004, D/- 26-2-2004 (P and H), Reversed. (Paras 11, 13, 14)

Pahul Malik and Rohit Walcha (for Ajay Pal), Advocates, for Appellants; Neeraj Kumar
Jain and Ugra Shankar Prasad, Advocates, for Respondents.
Judgement
K. G. BALAKRISHNAN, C.J.I. :-Delay condoned. Leave granted. Heard the learned
counsel.
2. Respondents 1 and 2 herein - the husband and son of one Amarjit Kaur who died in a
motor accident involving a Punjab roadways bus, filed a claim petition before the Motor
Accident Claims Tribunal, Faridkot. As against the compensation of Rs. 5 lacs claimed,
the Tribunal, on 1-12-1998 awarded a compensation of Rs. 1,44,000. Not being satisfied
with the quantum of compensation, respondents 1 and 2 filed FAO No. 1549/1999 before
the Punjab and Haryana High Court. The said appeal was referred to Lok Adalat
organised by the High Court, for settlement.
3. The High Court Lok Adalat took up the case on 3-8-2001. The parties were not present.
Their counsel were present. After hearing them the Lok Adalat passed the following order
:
FAO No. 1549 of 1999
"After hearing counsel for the parties, we propose to increase in the amount of
compensation, which is considered just and reasonable in this case.
The accident took place on March 4, 1997. Amarjit Kaur, aged about 32 years, died in the
accident. Her husband and minor son claimed compensation. The Tribunal granted Rs.
1,44,000/- along with 12 per
@page-SC1210
cent per annum interest. Feeling dissatisfied, they are in appeal.
The deceased was doing household work and also looking after some cattle and selling
milk. The tribunal fixed earning capacity at Rs.900/- and dependency at Rs.600/-
Applying multiplier of 15, compensation was worked out at Rs. 1,08,000/-. To this a sum
of Rs.28,253 on account of medical expenses, Rs.2147/- towards incidental charges and
Rs.5600/- towards hospital charges were allowed. We are of the opinion that the earning
capacity of the household wife has been determined on the lower side. An ordinary
labourer gets Rs.1200/- per mensem and at the lowest at least Rs. 1200/-should have been
determined the earning capacity of the deceased and dependency of the claimants at
Rs.800/-. The multiplier of 15 applied in this case is also on the lower side. Since the
deceased was aged 32 years, as per Schedule attached to the Motor Vehicles Act,
multiplier should have been 17. Thus, compensation worked out at Rs.1,63,200/-
(Rs.800/- x 12 x 17). To this a sum of Rs.7,000/- is added i.e. Rs.2,000/ - towards funeral
expenses and Rs.5,000/-towards loss of consortium, payable to the husband, making total
compensation payable at Rs.1,70,200/-. The Tribunal under this head allowed
compensation of Rs. 1,08,000/- i.e. under this head the claimants would get Rs.62,200/-
over and above that amount. The compensation granted under other heads is considered
just and reasonable.
Thus, while allowing the appeal, we grant compensation oJRs.62,200/- over and above
the amount awarded by the Tribunal to the appellants, who would share it equally. On this
amount they will get interest at the rate of 12 per cent per annum from the date of filing
of the claim petition i.e. July 28, 1997, till payment. Two months time is allowed to the
respondents to make the payment.
If the parties object to the proposed order as above, they may move the High Court within
two months for disposal of the appeal on merits according to law.
Copies of the order be supplied to the counsel for the parties."
(Emphasis supplied)
4. Punjab Roadways (second appellant herein) filed an application dated 15-1-2002 (CM
No. 13988-CII of 2002 in FAO No.1549/ 1999) to set aside order dated 3-8-2001 passed
by the Lok Adalat, as it was passed without their consent. The said application was
rejected by a learned Single Judge by a short order dated 11-9-2002 on the ground that
such objections were not maintainable or entertainable, having regard to its decision in
Charanjit Kaur v. Balwant Singh (CM No. 13988-CII of 2002 in FAO No. 1827/1999
decided on 30-7-2002) and other cases. In Charanjit Kaur, the learned single Judge had
held that an order passed by the Lok Adalat can be challenged only by a petition under
Article 227 of the Constitution, as all proceedings before the Lok Adalat are deemed to be
judicial proceedings and Lok Adalat is deemed to be a civil court under section 22(3) of
Legal Services Authorities Act, 1987.
5. The appellants, therefore, filed a petition under Article 227 of the Constitution (Civil
Revision Petition No.970/2004) challenging the order dated 3-8-2001 of the Lok Adalat.
The said petition was rejected by another single Judge of the High Court by the following
order dated 26-2-2003 :
"The instant petition has been filed under Article 227 of the Constitution seeking
necessary directions quashing the order dated 3-8-2001 passed by the Lok Adalat
enhancing the compensation in favour of the claimant-respondents to the tune of
Rs.62,000/-. The order of the Lok Adalat specifically indicated that if the parties were not
satisfied, they could file objections within a period of two months for the disposal of the
appeal on merits in accordance with law. The petitioners-State had filed objections which
were dismissed on 11-9-2002 and the order of the Lok Adalat dated 3-8-2001 had
attained finality.
Now the instant petition has been filed against challenging the order of the Lok Adalat
dated 3-8-2001. Nothing has been pointed out showing that such a petition under Article
227 of the Constitution is maintainable. Apart from the fact that the Lok Adalat has
granted time for filing the objections and the objections have been dismissed, the meagre
increase in the amount of compensation does not warrant any interference.
In view of the above, the petition is dismissed being not maintainable."
(Emphasis supplied)
The said order is under challenge in this appeal by special leave.
6. We are rather dismayed at the manner
@page-SC1211
in which the entire matter has been dealt with, undermining the very purpose and object
of Lok Adalats. At every stage the Lok Adalat and the High Court have acted in a manner
contrary to law.
7. A reference to relevant provisions will be of some assistance, before examination of the
issues involved. Section 19 of the Legal Services Authorities Act, 1987 ('LSA Act' for
short) provides for organisation of Lok Adalats. Section 19(5)(i) of LSA Act provides that
a Lok Adalat shall have jurisdiction to determine and to arrive at a compromise or
settlement between the parties to a dispute in respect of any case pending before any
court for which the Lok Adalat is organised. Section 20 relates to cognizance of cases by
Lok Adalats. Sub-section (1) refers to Lok Adalats taking cognizance of cases referred to
by courts and sub-section (2) refers to Lok Adalats taking cognizance of matters at pre-
litigation stage. The relevant portions of other sub-sections of section 20, relating to cases
referred by courts, are extracted below :
"(3) Where any case is referred to a Lok Adalat under sub-section (1) ...... the Lok Adalat
shall proceed to dispose of the case .......and arrive at a compromise or settlement
between the parties.
(4) Every Lok Adalat shall, while determining any reference before it under this Act, act
with utmost expedition to arrive at a compromise or settlement between the parties and
shall be guided by the principles of justice, equity, fair play and other legal principles.
(5) Where no award is made by the Lok Adalat on the ground that no compromise or
settlement could be arrived at between the parties, the record of the case shall be returned
by it to the court, from which the reference has been received under sub-section (1) for
disposal in accordance with law.
(7) Where the record of the case is returned under sub-section (5) to the court, such court
shall proceed to deal with such case from the stage which was reached before such
reference under sub-section (1)]"
(Emphasis supplied)
8. It is evident from the said provisions that Lok Adalats have no adjudicatory or judicial
functions. Their functions relate purely to conciliation. A Lok Adalat determines a
reference on the basis of a compromise or settlement between the parties at its instance,
and put its seal of confirmation by making an award in terms of the compromise or
settlement. When the Lok Adalat is not able to arrive at a settlement or compromise, no
award is made and the case record is returned to the court from which the reference was
received, for disposal in accordance with law. No Lok Adalat has the power to "hear"
parties to adjudicate cases as a court does. It discusses the subject-matter with the parties
and persuades them to arrive at a just settlement. In their conciliatory role, the Lok
Adalats are guided by principles of justice, equity, fair play. When the LSA Act refers to
'determination' by the Lok Adalat and 'award' by the Lok Adalat, the said Act does not
contemplate nor require an adjudicatory judicial determination, but a non-adjudicatory
determination based on a compromise or settlement, arrived at by the parties, with
guidance and assistance from the Lok Adalat. The 'award' of the Lok Adalat does not
mean any independent verdict or opinion arrived at by any decision making process. The
making of the award is merely an administrative act of incorporating the terms of
settlement or compromise agreed by parties in the presence of the Lok Adalat, in the form
of an executable order under the signature and seal of the Lok Adalat.
9. But we find that many sitting or retired Judges, while participating in Lok Adalats as
members, tend to conduct Lok Adalats like courts, by hearing parties, and imposing their
views as to what is just and equitable, on the parties. Sometimes they get carried away
and proceed to pass orders on merits, as in this case, even though there is no consensus or
settlement. Such acts, instead of fostering alternative dispute resolution through Lok
Adalats, will drive the litigants away from Lok Adalats. Lok Adalats should resist their
temptation to play the part of Judges and constantly strive to function as conciliators. The
endeavour and effort of the Lok Adalats should be to guide and persuade the parties, with
reference to principles of justice, equity and fair play to compromise and settle the
dispute by explaining the pros and cons, strength and weaknesses, advantages and
disadvantages of their respective claims.
10. The order of the Lok Adalat in this case (extracted above), shows that it assumed a
judicial role, heard parties, ignored the absence of consensus, and increased the
@page-SC1212
compensation to an extent it considered just and reasonable, by a reasoned order which is
adjudicatory in nature. It arrogated to itself the appellate powers of the High Court and
'allowed' the appeal and 'directed' the respondents in the appeal to pay the enhanced
compensation of Rs.62,200/- within two months. The order of the Lok Adalat was not
passed by consent of parties or in pursuance of any compromise or settlement between
the parties, is evident from its observation that "if the parties object to the proposed order
they may move the High Court within two months for disposal of the appeal on merits
according to law". Such an order is not an award of the Lok Adalat. Being contrary to law
and beyond the power and jurisdiction of the Lok Adalat, it is void in the eye of law. Such
orders which "impose" the views of the Lok Adalats on the parties, whatever be the good
intention behind them, bring a bad name to Lok Adalats and legal services.
11. The travails of the parties did not end with the Lok Adalat. Because the Lok Adalat
directed the aggrieved party to move the High Court for disposal of appeal on merits if
they had objection to its order, the appellants moved the High Court by an application in
the appeal, stating that they had not agreed to the enhancement proposed by Lok Adalat
and praying that the order of the Lok Adalat increasing the compensation by Rs.62,200
may be set aside as there was no settlement or compromise. The learned single Judge
failed to notice that there was no settlement or compromise between the parties; that the
order made by the Lok Adalat was not an award in terms of any settlement as
contemplated under the LSA Act; that the Lok Adalat had clearly stated that the parties
may either agree to it, or move the High Court for disposal of the appeal on merits in
accordance with law; and that in the absence of any settlement and 'award', the appeal
before the High Court continued to be pending and could not have been treated as finally
disposed of. The learned single Judge instead of perusing the order of the Lok Adalat and
hearing the appeal on merits, proceeded on a baseless assumption that the order dated 3-
8-2001 of the Lok Adalat was a binding award and therefore an application to hear the
appeal, was not maintainable and the only remedy for the appellants was to challenge the
order of the Lok Adalat by filing a writ petition under Article 227 of the Constitution.
12. It is true that where an award is made by Lok Adalat in terms of a settlement arrived
at between the parties, (which is duly signed by parties and annexed to the award of the
Lok Adalat), it becomes final and binding on the parties to the settlement and becomes
executable as if it is a decree of a civil court, and no appeal lies against it to any court. If
any party wants to challenge such an award based on settlement, it can be done only by
filing a petition under Article 226 and/or Article 227 of the Constitution, that too on very
limited grounds. But where no compromise or settlement is signed by the parties and the
order of the Lok Adalat does not refer to any settlement, but directs the respondent to
either make payment if it agrees to the order, or approach the High Court for disposal of
appeal on merits, if it does not agree, is not an award of the Lok Adalat. The question of
challenging such an order in a petition under Article 227 does not arise. As already
noticed, in such a situation, the High Court ought to have heard and disposed of the
appeal on merits.
13. But the travails continued. In view of the order dated 11-9-2002 passed by the learned
single Judge holding that a petition under Article 227 has to be filed to challenge the
order of the Lok Adalat, the appellants filed a petition under Article 2,27. But the said
petition was dismissed by another single Judge on the ground that the order of Lok Adalat
passed on 3-8-2001 had attained finality as the objections to it were dismissed on 11-9-
2002 and a petition under Article 227 was not maintainable to challenge the order of Lok
Adalat. He failed to notice that the order dated 3-8-2001 was neither a decision nor had it
attained finality. He also failed to notice that the objections to the order were not rejected
by the High Court after consideration on merits. He also overlooked the fact that the
learned Judge who decided the appellants' application, had directed that the order of the
Lok Adalat should be challenged by filing a petition under Article 227. Be that as it may.
14. Thus we find that the Lok Adalat exercised a power/jurisdiction not vested in it. On
the other hand, the High Court twice refused to exercise the jurisdiction vested in it,
thereby denying justice and driving the appellants to this Court. In this process, a simple
appeal by the legal heirs of the
@page-SC1213
deceased for enhancement of compensation,has been tossed around and is pending for
more than eight years, putting them to avoidable expense and harassment.
15. We therefore allow this appeal and quash the order dated 3-8-2001 of the Lok Adalat
as also set aside the orders dated 11-9-2002 and 26-2-2003 of the High Court. As a
consequence, the High Court shall hear and dispose of FAO No. 1549/1999 which
continues to be pending on its record, on merits in accordance with law. The High Court
is requested to dispose of the appeal expeditiously. Parties to bear their respective costs.
Appeal allowed.
AIR 2008 SUPREME COURT 1213 "S. K. Sinha, Chief Enforcement Officer v.
Videocon International Ltd"
(From : Bombay)*
Coram : 2 C. K. THAKKER AND PRAKASH PRABHAKAR NAOLEKAR, JJ.
Criminal Appeal No. 175 of 2008 (arising out of SLP (Cri.) No. 5718 of 2006), D/- 25 -1
-2008.
S.K. Sinha, Chief Enforcement Officer v. M/s. Videocon International Ltd. and Ors.
(A) Criminal P.C. (2 of 1974), S.190, S.204 - COGNIZANCE OF OFFENCE - ISSUE
OF PROCESS - FOREIGN EXCHANGE - Cognizance - Means to take notice of
judicially - Taking of cognizance is sine qua non for trial - Cannot be equated with
issuance of process - Complaint for offences under FERA - Cognizance held on facts was
taken before period of limitation stipulated by S.49(3) of FEMA.
Cri. W.P. No. 476 of 2005, D/- 26-4-2006 (Bom), Reversed.
Foreign Exchange Management Act (42 of 1999), S.49(3).
'Cognizance' has no esoteric or mystic significance in criminal law. It merely means
'becomes aware of and when used with reference to a Court or a Judge, it connotes 'to
take notice of judicially'. It indicates the point when a Court or a Magistrate takes judicial
notice of an offence with a view to initiating proceedings in respect of such offence said
to have been committed by someone. Taking cognizance' does not involve any formal
action of any kind. It occurs as soon as a Magistrate applies his mind to the suspected
commission of an offence. Cognizance is taken prior to commencement of criminal
proceedings. Taking of cognizance is thus a sine qua non or condition precedent for
holding a valid trial. Cognizance is taken of an offence and not of an offender. Whether or
not a Magistrate has taken cognizance of an offence depends on the facts and
circumstances of each case and no rule of universal application can be laid down as to
when a Magistrate can be said to have taken cognizance. Chapter XIV (Sections 190-199)
of the Code deals with 'conditions requisite for initiation of proceedings'. Section 190
empowers a Magistrate to take cognizance of an offence in certain circumstances. Then
comes Chapter XVI (Commencement of Proceedings before Magistrates). This Chapter
will apply only after cognizance of an offence has been taken by a Magistrate under
Chapter XIV. Section 204, authorises the Magistrate to issue process. From the scheme of
the Code it is clear that 'Initiation of Proceedings', dealt with in Chapter XIV, is different
from 'Commencement of Proceedings' covered by Chapter XVI. For commencement of
proceedings, there must be initiation of proceedings. Without initiation of proceedings
under Chapter XIV, there cannot be commencement of proceedings before a Magistrate
under Chapter XVI. Initiation of proceedings by issuing process against accused under
Chapter XIV cannot be equated with commencement of proceedings, taking of
cognizance under Chapter XVI. (Paras 12, 14, 15)
In the present case a criminal complaint was filed against respondent-Company for
contravention of Sections 18(2) and 18(3) read with Section 68(1), punishable under
Section 56(l)(ii) of FERA. On the same day, which was well before expiry of period of
limitation provided under S. 49(3) of FEMA after hearing the counsel for the Department,
the Chief Metropolitan Magistrate took cognizance of the offence and issued summons to
the accused. The process requiring the respondent to appear before the Court and answer
the charge under FERA was issued undoubtedly after expiry of period of limitation but
that was in pursuance of cognisance that was taken earlier. Quashing of proceedings by
equating taking cognizance with issuance of process and in holding that the complaint
was barred by law was improper.
Cri. W.P. No. 476 of 2005, D/-26-4-2006 (Bom), Reversed. (Paras 26, 31)
(B) Foreign Exchange Management Act (42 of 1999), S.49(3) - FOREIGN EXCHANGE
- LIMITATION - COGNIZANCE OF OFFENCE -
@page-SC1214
Offence under FERA - Limitation for taking cognizance - Computation - Relevant date is
date of filing complaint or date of taking cognizance - Question left open. (Paras 27, 29)
Cases Referred : Chronological Paras
2007 AIR SCW 4998 : AIR 2007 SC 2762 : 2007 Cri LJ 4068 (Ref.) 27, 28
2003 AIR SCW 5333 : AIR 2003 SC 4560 : 2003 Cri LJ 4543 (Ref.) 27, 28
AIR 1978 SC 1568 : 1978 Cri LJ 1687 (Ref.) 20
AIR 1976 SC 1672 : 1976 Cri LJ 1361 (Ref.) 24
AIR 1972 SC 2639 (Ref.) 22
AIR 1971 SC 2372 : 1971 Cri LJ 1697 (Ref.) 23
AIR 1967 SC 528 : 1967 Cri LJ 528 (Ref.) 24
AIR 1963 SC 765 : 1963 (1) Cri LJ 797 (Ref.) 20
AIR 1961 SC 986 : 1961 (2) Cri LJ 39 (Ref.) 21
AIR 1959 SC 1118 : 1959 Cri LJ 1368 (Ref.) 20
AIR 1951 SC 207 : 1951 (52) Cri LJ 775 (Ref.) 18
AIR 1950 Cal 437 : 1951 (52) Cri LJ 806 (Ref.) 17, 18, 19
B. Dutta, A.S.G., A.K. Srivastava, Naveen Prakash, Vikas Sharma and B. Krishna Prasad
with him, for Appellant; R.F. Nariman, Sr. Advocate, Subrat Birla, S.C. Birla, Shivaji M.
Jadhav, Rahul Joshi, Chinmoy Khaladkar and Ravindra Keshavrao Adsure with him, for
Respondents.
* Cri. W.P. No. 476 of 2005, D/- 26-4-2006 (Bom).
Judgement
C. K. THAKKER, J. :- Leave granted.
2. In the present appeal, we are called upon to decide the correctness or otherwise of the
proposition of law by the High Court of Judicature at Bombay whether issuance of
process in a criminal case is one and the same thing or can be equated with taking
cognizance by a Criminal Court? And if the period of initiation of criminal proceedings
has elapsed at the time of issue of process by a Court, the proceedings should be quashed
as barred by limitation?
3. To appreciate the controversy raised in the appeal instituted by the Chief Enforcement
Officer, Enforcement Directorate, Government of India (appellant herein), few relevant
facts may be noted.
4. Respondent No. 1-M/s. Videocon International Ltd. ('Company' for short) is a 'Public
Limited Company' incorporated under the Companies Act, 1956 having its business at
Mumbai and Aurangabad in the State of Maharashtra. On October 13, 1989, the
Company entered into an agreement with Radio Export (Moscow) for the supply of
colour tubes, electrolytic capacitors, transformers, etc., for Rs. 44,04,00,000/-. The
payment was made by respondent No. 1 Company to Japanese and Korean suppliers. But
before any payment could be received by respondent No. 1 from the USSR Company,
there was political turmoil in the USSR and payment to foreign suppliers was disrupted.
On January 5, 1993, Additional Director General, Directorate of Revenue Intelligence,
Mumbai addressed a letter to the appellant alerting him about the activities of the
Company in connection with the agreement to supply television sets to Radio Export,
Moscow. Based on the information forwarded by the Directorate of Revenue Intelligence,
Bombay, the appellant addressed two letters to the Chief Manager of Indian Bank,
Nariman Point, Bombay requesting the Bank to supply details of the export outstanding
of the Company. Indian Bank supplied necessary information and indicted that the export
outstanding of the Company was Rs. 16,60,00,000/-. The Reserve Bank of India turned
down the request of the Company for reimbursement of differential amount remaining
unpaid on the ground that the exports were effected from Korea and Japan and not from
India and the Company was not entitled to reimbursement. In pursuance of the summons
issued under S. 40 of the Foreign Exchange Regulation Act, 1973 (hereinafter referred to
as 'FERA'), Raj Kumar Dhoot, Director of the Company appeared before the Department
on April 25, 1999 and made a statement that there was an agreement between the
company and M/s. Radio Export, Moscow for supply of two lakh television sets and other
equipments for Rs. 44,04,00,000/-. The amount was received by the company through
State Bank of India, Overseas Branch, Bombay. He further stated that the television sets
had been procured from Korea and Japan who had been paid equivalent to Rs.
19,00,00,000/- in foreign exchange. Export bills raised from the sale to M/s. Radio
Export, Moscow were equivalent to Rs. 16,00,00,000/-. Whereas the contract with the
suppliers in Korea and Japan stipulated
@page-SC1215
payment in US Dollars, the contract with the USSR company required payment in Indian
Rupees. Since the value of rupee against the US Dollar fell down, the company had to
pay more rupees to their foreign suppliers. On June 1, 2000, FERA was replaced by the
Foreign Exchange Management Act, 1999 (hereinafter referred to as 'FEMA').
5. On May 24, 2002, the appellant-complainant in the capacity as Chief Enforcement
Officer, Government of India, filed Criminal Complaint No. 1149/S/2002 against the
company alleging that the company had received an amount of Rs. 44,04,00,000/-
through State Bank of India, Bombay but it failed to take steps to realise export proceeds
amounting to Rs. 16,60,00,000/- within the stipulated period of six months. It thereby
contravened Ss. 18(2) and 18(3) read with S. 68(1), punishable under S. 56(1)(ii) of
FERA. On the same day, which was well before expiry of period of limitation provided
under S. 49(3) of FEMA i.e. on May 24, 2002, after hearing the learned counsel for the
Department, the Chief Metropolitan Magistrate, Esplanade, Mumbai took cognizance of
the offence and issued summons to the accused. On February 3, 2003, the Chief
Metropolitan Magistrate issued process requiring the respondents to appear before the
Court and answer the charge under FERA.
6. In October, 2004, the respondents filed a petition being Criminal Writ Petition No. 476
of 2005 in the High Court of Judicature at Bombay by invoking Art. 227 of the
Constitution as also S. 482 of the Code of Criminal Procedure, 1973 (hereinafter referred
to as 'the Code') seeking quashing of criminal proceedings initiated vide complaint dated
May 24, 2002 on the ground that cognizance was taken by the Court after the period of
limitation and the proceedings were, therefore, liable to be quashed. The High Court, by
the impugned order dated April, 26, 2006, quashed the proceedings initiated against the
respondents on the ground that cognizance could be said to have been taken when process
was issued and since process was issued in February, 2003, the proceedings were time-
barred. The complaint was, therefore, quashed by the High Court. The said order is
challenged by the appellant in the present appeal.
7. Notice was issued by this Court on September 29, 2006. The respondents appeared.
Counter-affidavit and rejoinder affidavit were then filed. The Registry was directed to
place the matter for final hearing on a non-miscellaneous day and that is how the matter
is placed before us.
8. We have heard the learned counsel for the parties.
9. The learned counsel for the appellant contended that the High Court was in clear error
in equating taking cognizance of an offence with issuance of process and in holding that
the cognizance was taken after the period of limitation and hence the proceedings were
time-barred and liable to be quashed. It was submitted that FEMA came into force from
June 1, 2000 and under subsection (3) of S. 49 of FEMA, cognizance of an offence under
FERA could have been taken within a period of two years from the date of
commencement of the new Act. It was submitted that cognizance was taken by the Chief
Metropolitan Magistrate, Mumbai on May 24, 2002, i.e. the day when complaint was
filed which was well within the period of limitation provided by S. 49(3) of FEMA and as
such the Criminal Court was within its power in issuing process and in proceeding with
the matter and the High Court was not justified in quashing the proceedings on the
ground that cognizance was taken by the Court on February 3, 2003 when process was
issued by the Chief Metropolitan Magistrate, Mumbai. It was alternatively submitted by
the learned counsel that the relevant date for counting the period of limitation is not the
date of taking cognizance or issuance of process by the Court but the date of filing
complaint. It was stated that the point has been concluded by various decisions of this
Court. Since the complaint was filed on May 24, 2002, which was within the period of
limitation, the High Court was wrong in treating the criminal complaint as barred by
limitation and in quashing it. The order passed by the High Court, thus, deserves to be set
aside by directing the Chief Metropolitan Magistrate, Mumbai to proceed with the case
and decide it in accordance with law.
10. The learned counsel for the respondents, on the other hand, supported the order
passed by the High Court. It was submitted that the High Court was wholly right in
quashing the proceedings. Admittedly, process was issued in February, 2003 while under
S. 49(3) of FEMA, proceedings under the old Act (FERA) could not have been initiated
@page-SC1216
after the expiry of two years from the commencement of the new Act (FEMA). FEMA
came into force on June 1, 2000 and hence cognizance of an offence under FERA could
have been taken under FEMA latest by June 1, 2002. Issuance of process in February,
2003, therefore, was clearly time-barred and the High Court was right in quashing the
proceedings. It was also submitted that the appellant was not right in submitting that the
relevant date for computing the period of limitation is date of filing of complaint. The
material date is the date of taking cognizance by a competent Criminal Court. Sub-section
(3) of S. 49 of FEMA is a special provision, which must be given effect to and even on
that ground, the complaint was barred by time. Finally, it was submitted that though the
High Court had not considered the merits of the matter, the provisions of FERA had no
application to the facts of the case as it cannot be said that the accused had committed
any offence under FERA. Considering the said fact also, this Court may not interfere with
the order passed by the High Court in exercise of discretionary jurisdiction under Art. 136
of the Constitution. It was, therefore, submitted that the appeal may be dismissed.
11. Having heard learned counsel for the parties and having perused the relevant
provisions of law as also various judicial pronouncements, we are of the view that the
High Court was in error in equating issuance of process with taking cognizance by a
Criminal Court and in quashing the proceedings treating them as time-barred.
12. The expression 'cognizance' has not been defined in the Code. But the word
(cognizance) is of indefinite import. It has no esoteric or mystic significance in criminal
law. It merely means 'become aware of and when used with reference to a Court or a
Judge, it connotes 'to take notice of judicially.' It indicates the point when a Court or a
Magistrate takes judicial notice of an offence with a view to initiating proceedings in
respect of such offence said to have been committed by someone. Taking cognizance'
does not involve any formal action of any kind. It occurs as soon as a Magistrate applies
his mind to the suspected commission of an offence. Cognizance is taken prior to
commencement of criminal proceedings. Taking of cognizance is thus a sine qua non or
condition precedent for holding a valid trial. Cognizance is taken of an offence and not of
an offender. Whether or not a Magistrate has taken cognizance of an offence depends on
the facts and circumstances of each case and no rule of universal application can be laid
down as to when a Magistrate can be said to have taken cognizance. Chapter XIV
(Sections 190-199) of the Code deals with 'conditions requisite for initiation of
proceedings.' Section 190 empowers a Magistrate to take cognizance of an offence in
certain circumstances. Sub-section (1) thereof is material and may be quoted in extenso.
1) Subject to the provisions of this Chapter, any Magistrate of the first class, and any
Magistrate of the second class specially empowered in this behalf under sub-section (2),
may take cognizance of any offence -
(a) upon receiving a complaint of facts which constitute such offence;
(b) upon a police report of such facts;
(c) upon information received from any person other than a police officer, or upon his
own knowledge, that such offence has been committed.
13. Chapter XV (Sections 200-203) relates to 'complaints to Magistrates' and covers cases
before actual commencement of proceedings in a Court or before a Magistrate. Section
200 of the Code requires a Magistrate taking cognizance of an offence to examine the
complainant and his witnesses on oath. Section 202, however, enacts that a Magistrate is
not bound to issue process against the accused as a matter of course. It enables him
before the issue of process either to inquire into the case himself or direct an investigation
to be made by a Police Officer or by such other person as he thinks fit for the purpose of
deciding whether there is sufficient ground for proceeding further. The underlying object
of the inquiry under S. 202 is to ascertain whether there is prima facie case against the
accused. It thus allows a Magistrate to form an opinion whether the process should or
should not be issued. The scope of inquiry under S. 202 is, no doubt, extremely limited.
At that stage, what a Magistrate is called upon to see is whether there is sufficient ground
for proceeding with the matter and not whether there is sufficient ground for conviction
of the accused.
14. Then comes Chapter XVI (Commencement of Proceedings before Magistrates). This
Chapter will apply only after cognizance
@page-SC1217
of an offence has been taken by a Magistrate under Chapter XIV. Section 204,
whereunder process can be issued, is another material provision which reads as under :
204. Issue of process. - (1) If in the opinion of a Magistrate taking cognizance of an
ollence there is sufficient ground for proceeding, and the case appears to be -
(a) a summons case, he shall issue his summons for the attendance of the accused, or
(b) a warrant case, he may issue a warrant, or, if he thinks fit, a summons, for causing the
accused to be brought or to appear at a certain time before such Magistrate or (if he has
no jurisdiction himeslf) some other Magistrate having jurisdiction.
(2) No summons or warrant shall be issued against the accused under sub-section (1) until
a list of the prosecution witnesses has been filed.
(3) In a proceeding instituted upon a complaint made in writing, every summons or
warrant issued under sub-section (1) shall be accompanied by a copy of such complaint.
(4) When by any law for the time being in force any process fees or other fees are
payable, no process shall be issued until the fees are paid and, if such fees are not paid
within a reasonable time, the Magistrate may dismiss the complaint.
(5) Nothing in this section shall be deemed to affect the provisions of S. 87.
15. From the above scheme of the Code, in our judgment, it is clear that 'Initiation of
proceedings,' dealt with in Chapter XIV, is different from 'commencement of proceedings'
covered by Chapter XVI. For commencement of proceedings, there must be initiation of
proceedings. In other words, initiation of proceedings must precede commencement of
proceedings. Without initiation of proceedings under Chapter XIV, there cannot be
commencement of proceedings before a Magistrate under Chapter XVI. The High Court,
in our considered view, was not right in equating initiation of proceedings under Chapter
XIV with commencement of proceedings, under Chapter XVI.
16. Let us now consider the question in the light of judicial pronouncements on the point.
17. In Superintendent and Remembrancer of Legal Affairs v. Abani Kumar Banerjee, AIR
1950 Cal 437, the High Court of Calcutta had an occasion to consider the ambit and
scope of the phrase 'taking cognizance' under S. 190 of the Code of Criminal Procedure,
1898 which was in pari materia to S. 190 of the present Code of 1973. Referring to
various decisions. Das Gupta, J. (as his Lordship then was) stated :
"What is 'taking cognizance' has not been defined in the Criminal Procedure Code, and I
have no desire now to attempt to define it. It seems to me clear, however, that before it
can be said that any Magistrate has taken cognizance of any offence under S. 190 (1) (a),
Criminal P. C., he must not only have applied his mind to the contents of the petition, but
he must have done so for the purpose of proceeding in a particular way as indicated in the
subsequent provisions of this chapter, proceeding under S. 200, and thereafter sending it
for enquiry and report under S. 202. When the Magistrate applies his mind not for the
purpose of proceeding under the subsequent sections of this chapter, but for taking action
of some other kind, e.g., ordering investigation under S. 156(3). or issuing a search
warrant for the purpose of the investigation, he cannot be said to have taken cognizance
of the offence."
18

. R. R. Chan v. State of Uttar Pradesh, 1951 SCR 312 was probably the first leading
decision of this Court on the point. There, the police, having suspected the appellant-
accused to be guilty of offences punishable under Ss. 161 and 165 of the Indian Penal
Code (IPC) as also under the Prevention of Corruption Act, 1947, applied to the District
Magistrate, Kanpur to issue warrant of arrest on October, 22, 1947. Warrant was issued
on the next day and the accused was arrested on October 27,1947. On March, 25, 1949,
the accused was produced before the Magistrate to answer the charge-sheet submitted by
the prosecution. According to the accused, on October 22, 1947, when warrant for his
arrest was issued by the Magistrate, the Magistrate was said to have taken cognizance of
offence and since no sanction of the Government had been obtained before that date,
initiation of proceedings against him was unlawful. The question before the Court was as
to when cognizance of the offence could be said to have been taken by the Magistrate
under S. 190 of the Code. Considering the circumstances under which 'cognizance of
offence' under AIR 1951 SC 207
AIR 1950 Cal 437

@page-SC1218
sub-section (1) of S. 190 of the Code can be taken by a Magistrate and referring to Abani
Kumar Banerjee, the Court, speaking through Kania, C.J. stated :
"II is clear from the wording of the section that the initiation of the proceedings against a
person commences on the cognizance of the offence by the Magistrate under one of the
three contingencies mentioned in the section. The first contingency evidently is in respect
of non-cognizable offences as defined in the Criminal Procedure Code on the complaint
on an aggrieved person. The second is on a police report, which evidently is the case of a
cognizable offence when the police have completed their investigation and come to the
Magistrate for the issue of a process. The third is when the Magistrate himself takes
notice of an offence and issues the process. It is important to remember that in respect of
any cognizable offence, the police, at the initial stage when they are investigating the
matter, can arrest a person without obtaining an order from the Magistrate. Under S.
167(b) of the Criminal Procedure Code the police have of course to put up the person so
arrested before a Magistrate within 24 hours and obtain an order of remand to police
custody for the purpose of further investigation, if they so desire. But they have the
power to arrest a person for the purpose of investigation without approaching the
Magistrate first. Therefore in cases of cognizable offence before proceedings are initiated
and while the matter is under investigation by the police the suspected person is liable to
be arrested by the police without an order by the Magistrate."
19
. Approving the observations of Das Gupta, J. in Abani Kumar Banerjee, this Court held
that it was on March 25, 1949 when the Magistrate issued a notice under S. 190 of the
Code against the accused that he took 'cognizance' of the offence. Since before that day,
sanction had been granted by the Government, the proceedings could not be said to have
been initiated without authority of law. AIR 1950 Cal 437

20

. Again in Narayandas Bhagwandas Madhavdas v. State of West Bengal (1960) 1 SCR 93,
this Court observed that when cognizance is taken of an offence depends upon the facts
and circumstances of each case and it is impossible to attempt to define what is meant by
taking cognizance. Issuance of a search warrant for the purpose of an investigation or a
warrant of arrest of accused cannot by itself be regarded as an act of taking cognizance of
an offence. It is only when a Magistrate applies his mind for proceeding under S. 200 and
subsequent sections of Chapter XV or under S. 204 of Chapter XVI of the Code that it
can be positively stated that he had applied his mind and thereby had taken cognizance of
an offence (see also Ajit Kumar Palit v. State of W.B. and Anr. (1963) Supp (1) SCR 953;
Hareram Satpathy v. Tikaram Agarwala and Anr. (1978) 4 SCC 58). AIR 1959 SC
1118
AIR 1963 SC 765
AIR 1978 SC 1568

21

. In Gopal Das Sindhi and Ors. v. State of Assam and Anr., AIR 1961 SC 986, referring to
earlier judgments, this Court said : At p. 989, Para 7

"We cannot read the provisions of S. 190 to mean that once a complaint is filed, a
Magistrate is bound to take cognizance if the facts stated in the complaint disclose the
commission of any offence. We are unable to construe the word 'may' in S. 190 to mean
'must'. The reason is obvious. A complaint disclosing cognizable offences may well
justify a Magistrate in sending the complaint under S. 156(3) to the police for
investigation. There is no reason why the time of the Magistrate should be wasted when
primarily the duty to investigate in cases involving cognizable offences is with the police.
On the other hand, there may be occasions when the Magistrate may exercise his
discretion and take cognizance of a cognizable offence. If he does so then he would have
to proceed in the manner provided by Chapter XVI of the Code."
22

. In Nirmaljit Singh Hoon v. State of West Bengal and Anr. (1973) 3 SCC 753, the Court
stated that it is well settled that before a Magistrate can be said to have taken cognizance
of an offence under S. 190(1)(a) of the Code, he must have not only applied his mind to
the contents of the complaint presented before him, but must have done so for the
purpose of proceeding under S. 200 and the provisions following that section. Where,
however, he applies his mind only for ordering an investigation under S. 156(3) or issues
a warrant for arrest of accused, he cannot be said to have taken cognizance of the offence.
AIR 1972 SC 2639

@page-SC1219
23

. in Darshan Singh Ram Kishan v. State of Maharashtra (1972) 1 SCR 571, speaking for
the Court, Shelat, J. stated that under S. 190 of the Code, a Magistrate may take
cognizance of an offence either (a) upon receiving a complaint, or (b) upon a police
report, or (c) upon information received from a person other than a police officer or even
upon his own information or suspicion that such an offence has been committed. As has
often been said, taking cognizance does not involve any formal action or indeed action of
any kind. It occurs as soon as a Magistrate applies his mind to the suspected commission
of an offence. Cognizance, thus, takes place at a point when a Magistrate first takes
judicial notice of an offence. AIR 1971 SC 2372

24

. In Devarapalli Lakshminarayana Reddy and Ors. v. V. Narayana Reddy and Ors. (1976)
3 SCC 252, this Court said : AIR 1976 SC 1672, (Paras 13 and 14)

"It is well settled that when a Magistrate receives a complaint, he is not bound to take
cognizance if the facts alleged in the complaint, disclose the commission of an offence.
This is clear from the use of the words "may take cognizance" which in the context in
which they occur cannot be equated with must take cognizance." The word "may" gives a
discretion to the Magistrate in the matter. If on a reading of the complaint he finds that
the allegations therein disclose a cognizable offence and the forwarding of the complaint
to the police for investigation under S. 156(3) will be conducive to justice and save the
valuable time of the Magistrate from, being wasted in enquiring into a matter which was
primarily the duty of the police to investigate, he will be justified in adopting that course
as an alternative to taking cognizance of the offence, himself.

This raises the incidental question : What is meant by "taking cognizance of an offence"
by a Magistrate within the contemplation of S. 190? This expression has not been defined
in the Code. But from the scheme of the Code, the content and marginal heading of S.
190 and the caption of Chapter XIV under which Ss. 190 to 199 occur, it is clear that a
case can be said to be instituted in a Court only when the Court takes cognizance of the
offence alleged therein. The ways in which such cognizance can be taken are set out in
Cls. (a), (b) and (c) of S. 190(1). Whether the Magistrate has or has not taken cognizance
of the offence will depend on the circumstances of the particular case including the mode
in which the case is sought to be instituted and the nature of the preliminary action, if
any, taken by the Magistrate. Broadly speaking, when on receiving a complaint, the
Magistrate applies his mind for the purposes of proceeding under S. 200 and the
succeeding sections in Chapter XV of the Code of 1973, he is said to have taken
cognizance of the offence within the meaning of S. 190(1)(a). If, instead of proceeding
under Chapter XV, he has in the judicial exercise of his discretion taken action of some
other kind, such as issuing a search warrant for the purpose of investigation, or ordering
investigation by the police under S. 156(3), he cannot be said to have taken cognlzane of
any offence." (see also M. L. Sethi v. R. P. Kapur and Anr. (1967) 1 SCR 520). AIR
1967 SC 528

25. In the case on hand, it is amply clear that cognizance of the offence was taken by the
Chief Metropolitan Magistrate, Mumbai on May 24, 2002, i.e., the day on which the
complaint was filed, the Magistrate, after hearing the counsel for the department, took
cognizance of the offence and passed the following order :
"Mr. S. A. A. Naqvi, counsel for the department is present. Complainant is public servant.
Cognizance is taken. Issue summons to accused under S. 18 (2) (3) of FERA, 73 read
with Central Notification and r/w S. 68(1) of the said Act and r/w S. 56(1)(i) and r/w S.
49(3)(4) of FEMA, 1999.
Summons returnable on 7-2-2003 at 3 p.m." (Emphasis supplied)
26. Undoubtedly, the process was issued on February 3, 2003. In our judgment, however,
it was in pursuance of the cognizance taken by the Court on May 24, 2002 that a
subsequent action was taken under S. 204 under Chapter XVI. Taking cognizance of
offence was entirely different from initiating proceedings; rather it was the condition
precedent to the initiation of the proceedings. Order of issuance of process on February 3,
2003 by the Court was in pursuance of and consequent to taking cognizance of an offence
on May 24, 2002. The High Court, in our view, therefore, was not right in equating taking
cognizance with issuance of process and in holding that the complaint was barred by law
and criminal proceedings were liable to be quashed. The order passed
@page-SC1220
by the High Court, thus, deserves to be quashed and set aside.
27

. It was also contended by the learned counsel for the appellant that the relevant date for
considering the question of limitation is the date of filing of complaint and not taking
cognizance or issuance of process by a Court of law. In this connection, our attention was
invited by the counsel to Bharat Damodar Kale and Anr. v. State of A.P. (2003) 8 SCC
559 and a recent decision of this Court in Japani Sahoo v. Chandra Sekhar Mohanty
(2007) 7 SCC 394. In Japani Sahoo, one of us (C. K. Thakker, J.), after considering
decisions of various High Courts as also Bharat Damodar Kale, stated : 2003 AIR
SCW 5333
2007 AIR SCW 4998

"52. The matter can be looked at from different angle also. Once it is accepted (and there
is no dispute about it) that it is not within the domain of the complainant or prosecuting
agency to take cognizance of an offence or to issue process and the only thing the former
can do is to file a complaint or initiate proceedings in accordance with law. If that action
of initiation of proceedings has been taken within the period of limitation, the
complainant is not responsible for any delay on the part of the Court or Magistrate in
issuing process or taking cognizance of an offence. Now, if he is sought to be penalised
because of the omission, default or inaction on the part of the Court or Magistrate, the
provision of law may have to be tested on the touchstone of Art. 14 of the Constitution. It
can possibly be urged that such a provision is totally arbitrary, irrational and
unreasonable. It is settled law that a Court of law would interpret a provision which
would help sustaining the validity of law by applying the doctrine of reasonable
construction rather than making it vulnerable and unconstitutional by adopting rule of
'litera legis.' Connecting the provision of limitation in S. 468 of the Code with issuing of
process or taking of cognizance by the Court may make it unsustainable and ultra vires
Art. 14 of the Constitution."
28

. The learned counsel for the respondent, on the other hand, tried to distinguish Bharat
Damodar Kale and Japani Sahoo submitting that in both the decisions, this Court was
called upon to consider, inter alia, S. 468 of the Code providing for limitation for taking
cognizance of certain offences. According to the counsel, S. 468 of the Code starts with
the expression "Except as provided elsewhere in this Code....." Section 49(3) of FEMA,
on the other hand, starts with a non obstante clause ("Notwithstanding anything contained
in any other law for the time being in force"). It was, therefore, submitted that the ratio
laid down in the above two cases would not be applicable to the instant case. 2003
AIR SCW 5333
2007 AIR SCW 4998

29. In our opinion, it would not be necessary for us to express any opinion one way or the
other on the larger question. We have already held in the earlier part of the judgment that
in the case on hand, cognizance of an offence had already been taken by the Chief
Metropolitan Magistrate, Mumbai on May 24, 2002, well within the period prescribed by
sub-section (3) of S. 49 of FEMA within two years of coming into force of the Act from
June 1, 2000. We, therefore, express no opinion on the question raised by the learned
counsel for the respondent.
30. As regards quashing of proceedings on merits, the learned counsel for the appellant is
right in submitting that the High Court has not at all touched the merits of the case and
proceedings were not quashed on the ground that the provisions of FERA do not apply to
the case before the Court. The High Court dealt with only one point as to whether the
proceedings were liable to be quashed on the ground that they were time-barred and
upholding the contention of the accused, passed the impugned order. As we are of the
view that the High Court was not right in quashing the proceedings on the ground of
limitation, the order deserves to be set aside by remitting the matter to the Chief
Metropolitan Magistrate, Mumbai to be decided in accordance with law. We may,
however, clarify that it is open to the respondents to take all contentions including the
contention as to applicability or otherwise of FERA to the facts of the case. As and when
such question will be raised, the Court will pass an appropriate order in accordance with
law.
31. For the foregoing reasons, the appeal is allowed. The order passed by the High Court
is set aside and it is held that cognizance of the offence had already been taken by the
competent Criminal Court i.e. Chief Metropolitan Magistrate, Mumbai on May 24, 2002
and it could not be said that the proceedings were barred by S. 49(3) of FEMA. The Chief
Metropolitan Magistrate
@page-SC1221
will now proceed to consider the matter in accordance with law. All contentions of all
parties are kept open except the one decided by us in this appeal. Since the matter is very
old, the Court will give priority and will decide it as expeditiously as possible, preferably
before June 30, 2008.
32. Ordered accordingly.
Appeal allowed.
AIR 2008 SUPREME COURT 1221 "A.P.S.R.T.C. v. M. Ramadevi"
(From : Andhra Pradesh)*
Coram : 2 Dr. A. PASAYAT AND P. SATHASIVAM, JJ.
Civil Appeal No. 682 of 2008 (arising out of SLP (C) No. 11647 of 2004, D/- 25 -1
-2008.
A.P.S.R.T.C. and Anr. v. M. Ramadevi and Ors.
(A) Motor Vehicles Act (59 of 1988), S.168 - MOTOR VEHICLES - TRIBUNALS -
Compensation - Enhancement - There is no restriction that Tribunal / Court cannot award
compensation exceeding the amount claimed.
2002 AIR SCW 5348 Foll. (Paras 8, 9)
(B) Motor Vehicles Act (59 of 1988), S.168 - MOTOR VEHICLES - INTEREST -
Accident - Claim for compensation - Deceased, bus driver, was 40 years of age - Monthly
income taken at Rs. 3,000/-, after deducting 1/3rd therefrom, annual contribution is fixed
at Rs. 24,000/- - Mutiplier of 10 adopted - Amount of Rs. 20,000/- fixed for pecuniary
damages and consortium - Claimants thus, would be entitled to Rs. 2,60,000/-- Interest
rate of 9% fixed by High Court, not interfered with. (Para 12)
Cases Referred : Chronological Paras
2002 AIR SCW 5348 : AIR 2003 SC 674 : 2003 AIR - Kant HCR 242 (Foll.) 9
Ms. K. Radha Rani, P. Vijay Kumar, D. Mahesh Babu, for Appellants; G.V.R. Choudhary,
K. Shivaraj Choudhuri, for Respondents.
* Civ. Misc. Appeal No. 784 of 2002, D/- 27-11-2003 (AP)
Judgement
Dr. ARIJIT PASAYAT, J. :- Leave granted.
2. Challenge in this appeal is to the judgment of the learned single Judge of the Andhra
Pradesh High Court. The appellant-Corporation had filed an appeal before the High Court
questioning correctness of the award made by the Motor Accidents Claims Tribunal-cum-
1st Addl. District Judge, R.R. District at Saroornagar, Hyderabad (hereinafter referred to
as the Tribunal).
3. Background facts are as follows :
A claim petition under S. 166 of the Motor Vehicles Act, 1988 was filed by the
respondents claiming compensation of Rs. 5,00,000/- on account of the death of M.
Nageshwar Rao (hereinafter referred to as the 'deceased') in an accident on 18-5-1998.
The deceased was working as a driver of the appellant-Corporation. In the claim petition
it was stated that because of the rash and negligent driving of the bus No. A.P. 10 Z 998
belonging to the Corporation which was being driven in a rash and negligent manner, the
deceased lost his life. It was claimed that the deceased was 38 years of age and was an
employee of the Corporation and was earning salary of Rs. 4,467.50 p.m. The appellant-
Corporation filed its objection before the Tribunal taking the stand that it was not liable to
pay any compensation. The quantum of salary claimed and the age was also disputed.
4. The Tribunal observed that the age of the deceased was 40 years of age and he was
getting a salary of Rs. 4,000/- p.m. and after deduction his take home pay was Rs. 2,367/-
and the total emoluments was Rs. 3,983/-. Applying the multiplier of 12 the entitlement
was fixed at Rs. 2,16,000/-, in addition Rs. 15,000/- for non-pecuniary damages and Rs.
5,000/- as consortium was awarded. Thus the total compensation awarded was fixed at
Rs. 2,46,000/-. The same was directed to be paid with interest @ 12% p.a.
5. The appellant-Corporation filed appeal before the High Court. It is to be noted that the
claimants did not prefer any appeal. The High Court held that the award as made was
inadequate and just compensation was not awarded.
6. The High Court was of the view that the pay of the deceased was Rs. 3,536/- and not
Rs. 2,367/- as noted by the Tribunal. Accordingly, it fixed the basic pay of Rs. 3,500/-
after deducting 1/3rd towards the personal expenses. The monthly contribution was fixed
at Rs. 2,333/- and the annual contribution at Rs. 27,996/-. The multiplier was taken at 12.
Accordingly, entitlement
@page-SC1222
was fixed at Rs. 3,35,952/- to which was added the sum of Rs. 20,000/- additionally
awarded by the Tribunal.
7. In support of the appeal, learned counsel for the appellants submitted that when there
was no appeal by the claimants in the appeal filed by the appellant-Corporation, the High
Court should not have enhanced the amount. It was also submitted that the multiplier as
adopted was high.
8. Learned counsel for the respondents on the other hand submitted that there is no
embargo on the Tribunal or the High Court awarding compensation exceeding the amount
claimed. It was also submitted that the interest was reduced to 9% from 12% as fixed by
the Tribunal. It was, therefore, submitted that there was no infirmity in the High Court's
order.
9

. In Nagappa v. Gurdial Singh and Ors. (2003 (2) SCC 274) para 21 as follows : 2002
AIR SCW 5348, Para 21

"21. For the reasons discussed above, in our view, under the MV Act, there is no
restriction that the Tribunal/Court cannot award compensation amount exceeding the
claimed amount. The function of the Tribunal/Court is to award "just" compensation
which is reasonable on the basis of evidence produced on record. Further, in such cases
there is no question of claim becoming time-barred or it cannot be contended that by
enhancing the claim there should be change of cause of action. It is also to be stated that
as provided under sub-section (4) to S. 166, even the report submitted to the Claims
Tribunal under sub-section (6) of S. 158 can be treated as an application for
compensation under the MV Act. If required, in appropriate cases, the Court may permit
amendment to the claim petition."
10. The other question that remains to be adjudicated is whether the income has been
rightly adopted by the Tribunal and the High Court was correct and whether the correct
multiplier was adopted.
11. Considering the figure in the Ex. A/7 the monthly income taken at Rs. 3,000/-after
deducting 1/3rd therefrom the annual contribution is fixed at Rs. 24,000/-.
12. Adopting the multiplier of 10, the amount payable to the claimants comes to Rs.
2,40,000/- to that shall be added the amount of Rs. 20,000/- fixed by the Tribunal for non-
pecuniary damages and consortium as there was no challenge by the Corporation to the
award of such amounts. Therefore, the entitlement of the claimant comes to Rs.
2,60,000/-. The interest rate of 9% fixed by the High Court does not warrant any
interference. A sum of Rs. 2,00,000/- has been deposited by the Corporation pursuant to
the direction given by this Court on 19-7-2004. Admittedly, the same has been withdrawn
by the claimants. The balance amount shall be deposited by the appellant-Corporation
within six weeks from today. Tribunal shall fix the terms for withdrawal/deposit of the
amount in FDs as deemed appropriate.
13. The appeal is allowed to the aforesaid extent. No costs.
Order accordingly.
AIR 2008 SUPREME COURT 1222 "Abhishek Malviya v. Addl. Welfare Commissioner"
(From : Madhya Pradesh)*
(At Jabalpur).
Coram : 2 R. V. RAVEENDRAN AND P. SATHASIVAM, JJ.
Civil Appeal No. 7093 of 2001, D/- 23 -1 -2008.
Abhishek Malviya v. Addl. Welfare Commissioner and Anr.
Constitution of India, Art.136 - Civil P.C. (5 of 1908), O.23, R.1 - SPECIAL LEAVE
PETITION - WITHDRAWAL OF SUIT - Special leave petition - Withdrawal - Bar to re-
agitate matter - Petition against order of Additional Welfare Commissioner assessing
compensation payable to victim of Bhopal Gas Tragedy - Petition allowed to be
withdrawn for correction of error in order describing claimant as deceased - No liberty
was reserved to file fresh appeal or seek review on merits - Order of Addl. Welfare
Commissioner having attained finality efforts to re-agitate matter is an exercise in futility.
Bhopal Gas Leak Disaster (Processing of Claims) Act (24 of 1985), S.9. (Para 8)

Sushil Kumar Jain, H.D. Thanvi, Puneet Jain, for Appellant; Vikas Singh, ASG (NP), Ms.
Vibha Datta Makhija, S. Wasim A. Qadri, B.K. Prasad , V.V. Balaram Das, for
Respondents.
* W. P. No.2629 of 2000, D/- 5-2-2001 (M.P.) (At Jabalpur).
Judgement
R. V. RAVEENDRAN, J. :-This appeal relates to compensation payable to a Bhopal gas
tragedy victim. The appellant was in the
@page-SC1223
womb of his mother on the fateful day that is, 2/3-12-1984. He was subsequently born on
14-5-1985. An application was made on his behalf for payment of compensation of Rs.
50,000/- under the scheme for payment of compensation to the gas victims alleging that
the appellant's pregnant mother was affected by the leaked gas and consequently, the
appellant, who was in her womb, was also affected.
2. The Bhopal Gas Leak Disaster (Registration and Processing of Claims) Scheme, 1985
was framed by the Central Government in exercise of power under S. 9 of the Bhopal
Gas Leak Disaster (Processing of Claims) Act, 1985. Clause 5 thereof provides for
categorisation and registration of claims. The application on behalf of appellant was
considered by the Deputy Commissioner, for Bhopal gas victims. The appellant had been
examined and a medical dossier prepared at the time of State sponsored medical
examination including X-ray of chest. The reports of the examination on 25-8-1988
showed that the appellant did not suffer from any disease. Therefore, his condition was
recorded as 'normal,' and the appellant was placed in category 'A' under general injuries.
This was contested by the appellant's father on the ground that ever since the time of
birth, the appellant had heart and respiratory problems and he was treated as Chautram
Hospital, Indore. After a detailed examination of the medial and other records, the Deputy
Commissioner passed an order dated 17-6-1996. He held that the appellant had
respiratory problems immediately after birth and for a short time thereafter; and that the
treatment given to appellant was for cough and cold, fever and other normal ailments.
However as the resistance capacity of the appellant was decreased due to ill effect of gas,
he was classified under the category 'temporary partial disability' falling under Para 5(2)
(d) of the Scheme and a compensation of Rs. 45,000/- was awarded.
3. An appeal was filed by the appellant claiming compensation of Rs. 1,00,000/-. The
appeal was allowed in part, by the First Additional Welfare Commissioner for Bhopal by
order dated 13-3-1997. The appellate authority noted that the appellant's heart disease
could not be attributed to MIC gas, as that did not affect the heart directly or indirectly.
However as appellant suffered from pneumonia immediately after his birth and later as he
was suffering from bronchitis in the year 1988 and treatment continued for respiratory
problems, he increased the compensation by Rs. 10,000/- that is in all Rs. 55,000/-.
4. The appellant filed a special leave petition before this Court seeking leave to appeal
against the said order of Additional Welfare Commissioner. One of the grounds of
challenge was that the Additional Welfare Commissioner had referred to the appellant as
'deceased' and that showed non-application of mind. (This however apparently was a
typographical error as in the subsequent part of the judgment, the Appellate
Commissioner had proceeded on the basis that the appellant is alive and discussed the
entire history. Be that as it may). This Court by order dated 4-5-1999 dismissed the
special leave petition as withdrawn, recording the submission on behalf of the appellant
that he wanted to apply to the Additional Welfare Commissioner for correction of the
order.
5. Thereafter, on an application for modification, the matter was again considered by the
First Additional Welfare Commissioner. While seeking correction of the order by deletion
of the expression 'deceased,' the appellant also claimed increased compensation of Rs. 10
lakhs in view of his continued medical treatment. The application was disposed of by
order dated 6-8-1999, correcting the typographical mistake by deleting the word
'deceased' used in Paras 2 and 3 of the earlier order dated 13-3-1997. But the Additional
Welfare Commissioner refused to reconsider the appeal on merits or to increase the
compensation, as there was no direction by this Court to reconsider the claim.The
appellant again approached this Court in S.L.P. (C) No. 19076/1999 challenging the order
dated 6-8-1999. That SLP was also dismissed on 6-1-2000 reserving liberty to the
appellant to approach the High Court under Art. 226 of the Constitution or seek any other
remedy available in law.
6. Thereafter, the appellant again filed an appeal before the First Additional Welfare
Commissioner who found no reason to disturb the earlier order dated 6-8-1999. He,
therefore, dismissed the appeal on 23-2-2000. That order was challenged in W.P. (C) No.
2629/2000. The High Court of Madhya Pradesh found that the compensation awarded did
not warrant interference and accordingly dismissed the writ petition, by
@page-SC1224
the impugned order dated 5-2-2001. The High Court held that it was evident from the
order dated 4-5-1999, that this Court found no ground to interfere with the order of the
Additional Welfare Commissioner on merits and the SLP was dismissed and liberty was
reserved only for the purpose of seeking correction of the typographical error. The
appellant has challenged the order of the High Court in this appeal by special leave.
7. It is the contention of the appellant that while disposing of the SLP by order dated 4-5-
1999. this Court did not examine the order dated 13-3-1997 of the Additional Welfare
Commissioner on merits and the SLP was dismissed as withdrawn to enable him
approach the Additional Welfare Commissioner for rehearing and modification and,
therefore, the Additional Welfare Commissioner was bound to reconsider the matter.
8. We find no merit in appellant's contention. The order dated 4-5-1999 of this Court
specifically refers to the error in the order describing the appellant as 'deceased' and
dismissed the SLP as withdrawn with the following observation : "He wants to apply to
the Additional Welfare Commissioner for correction. We express no opinion in that
behalf." No liberty was reserved to file a fresh appeal or seek review of the order dated
13-3-1997 on merits. The order dated 13-3-1997 having attained finality, his efforts to
reagitate the issue again and again is an exercise in futility. We are therefore of the view
that appeal is liable to be dismissed.
9. The learned counsel for appellant had also made submissions on merits. But he was
unable to demonstrate how the appellant was entitled to be placed in a higher category
under the scheme so as to become entitled for higher compensation. The appellant
produced some documents to show that he has been undergoing treatment for bronchitis
and other problems. His respiratory problem and bronchitis were considered and the
compensation was increased from Rs. 45,000/- to Rs. 55,000/- by the Addl. Welfare
Commissioner. Even on merits, we find no reason to interfere with the categorisation of
appellant for the purpose of compensation. The appeal is accordingly dismissed.
Appeal dismissed.
AIR 2008 SUPREME COURT 1224 "Dayanand Rayu Mandrekar v. Chandrakant Uttarn
Chodankar"
(From : AIR 2006 Bom 16)
Coram : 3 K. G. BALAKRISHNAN, DALVEER BHANDARI AND D. K. JAIN, JJ.
Civil Appeal No. 3578 with 3579 of 2005, D/- 18 -1 -2007.
Dayanand Rayu Mandrekar v. Chandrakant Uttam Chodankar and Ors.
(A) Representation of the People Act (43 of 1951), S.10 - Goa, Daman and Diu Members
of Legislative Assembly (Removal of Disqualifications) Act (1 of 1982), Sch.1, Cl.(9) -
ELECTION - LEGISLATIVE ASSEMBLY - WORDS AND PHRASES - Election -
Disqualification - Holder of "Office of profit" - Exemption of certain offices by State Act
- Limited to offices without remuneration except compensatory allowance - Elected
candidates, Chairman of State Board/Corporation given variety of perquisites for
unlimited use - Cannot be said to be getting only compensatory allowance - Stand
disqualified from election.
Words and Phrases - Compensatory allowance - Does not include perquisites for
unlimited use. (Para 7)
(B) Goa, Daman and Diu Members of Legislative Assembly (Removal of
Disqualifications) Act (1 of 1982), Sch.1, Cl.(9) - LEGISLATIVE ASSEMBLY -
ELECTION - Election - Removal of disqualification of holders of certain offices of profit
under Government - Benefit - Not available when office carries remuneration - Actual
payment of remuneration is not essential. (Para 7)
(C) Goa, Daman and Diu Members of Legislative Assembly (Removal of
Disqualifications) Act (1 of 1982), S.1 - Goa State Re-organisation Act (1987), S.57 -
LEGISLATIVE ASSEMBLY - STATE REORGANISATION - APPLICABILITY OF AN
ACT - Applicability of 1982 Act to State of Goa - In absence of its adoption by 1987 Act
- Question left open. (Para 11)

Dhruv Mehta, Jha and Yashraj Singh Deora (for M/s. K.L. Mehta and Co.), for Appellant;
M.N. Krishnamani, Dr. Abhishek M. Singhvi, Sr. Advocates, Bhavanishankar V. Gadnis,
S.U.K. Sagar, Ms. Bina Madhavan, A. Venayagam, Vinayakam (for M/s. Lawyer's Knit
and Co.), Sriniwas Khalap and Ashok Mathur with them, for Respondents.
Judgement
JUDGMENT :- The appellants, in these
@page-SC1225
two appeals, challenge the judgment in the Election Petition Nos. 1 and 2 of 2002. In
both these cases a common questions of law had arisen and, therefore, we heard the
matter together and are disposing these appeals by way of a common order. The appellant
in C.A. No. 3578/05 was elected to the Legislative Assembly of State of Goa from Siolim
constituency in the election held on 30-5-2002, whereas the appellant in C.A. No.
3579/05 was elected from Vasco-da-Gama Assembly constituency of the State
Legislature. The election petitions were preferred by two unsuccessful candidates in the
elections alleging that these two appellants were holding 'office of profit' at the time when
they contested the elections and, therefore, they were ineligible to be elected to the
legislature. At the time of filing their nominations, the appellant in C.A. No. 3578/05 was
the Chairman of the Goa Khadi and Village Industries Board of the State of Goa, whereas
the appellant in C.A. No. 3579/05 was the Chairman of the Goa State Scheduled Castes
and Other Backward Classes Finance and Development Corporation Ltd. of the State of
Goa. The appellants in these two cases contended before the High Court that they were
not holding an 'office of profit' and were not receiving any salary or allowances for the
said post they held and by virtue of the provision contained in the Goa, Daman and Diu
Members of Legislative Assembly [Removal of Disqualifications] Act, 1982 (for short
'the 1982 Act'), the disqualification, if any, was removed especially by clause (9) of the
Schedule. The pleas set-up by the appellants were rejected and the High Court held that
these appellants were holding the 'office of profit' and that they were not entitled to
contest the election as they were disqualified and the election petitions were allowed and
elections of appellants were set aside.
2. We have heard the counsel for the appellants and counsel for the respondents.
3. It is not disputed that the appellants were holding the office as alleged in the election
petition, but contended that they were not receiving any salary or allowances and were
only receiving some perquisites. It is not disputed that these two appellants, by virtue of
their office, enjoyed the privilege of a chauffeur driven car with unrestricted use of petrol.
The appellants were also given the services of a PA, a clerk and a Peon and they were
provided with a residential telephone with unrestricted number of calls. They were also
provided with a mobile telephone and newspapers were supplied at their residences and
the expenses were paid from the funds of the office.
4. Under Rule 7 of the Goa, Daman and Diu Khadi and Village Industries Board Rules,
1967 (for short the 1967 Rules'), 'The Chairman, the Vice-Chairman and other members
of the Board shall be paid such salary or honorarium and allowances from the funds of
the Boards as the Government may from time to time fix." The appellant in C.A. No.
3578/05 was not receiving any salary or honorarium as, according to him, the
government had not fixed any such salary or honorarium. The question that arises for
consideration is whether the appellants could seek the benefit of the 1982 Act. By virtue
of clause (9) of the Schedule, the appellant contended, that the office of
Chairman/Director or member of the statutory or non-statutory Board are exempted from
any disqualification but the proviso to clause (9) of the Schedule makes it further clear
that this disqualification is circumscribed by a further limitation.
5. Clause 9 of the Schedule reads as follows :
"9. The office of Chairman, Director or member of a statutory or non-statutory body or
committee or corporation constituted by the Government of Goa, Daman and Diu :
Provided that the Chairman, Director or Member of any of the aforesaid committees or
bodies or corporations is not entitled to any remuneration other than compensatory
allowance."
6. An explanation was also added to clause (9). The same reads thus :
"Explanation. - For the purpose of the aforesaid entries -
"Compensatory allowance" means any sum of money payable to the holder of an office
by way of daily allowance [such allowance not exceeding the amount of daily allowance
to which a member of the Legislative Assembly is entitled under the Goa, Daman and
Diu Salary, Allowances and Pension of the Members of the Legislative Assembly Act,
1964 (2 of 1965)], any conveyance allowance, house rent allowance or travelling
allowance for the purpose of enabling him to recoup any expenditure incurred by him in
performing the functions of that office".
@page-SC1226
7. The proviso makes it abundantly clear that the compensatory allowance would only
mean 'any expense which is incurred by the holder of the office in discharge of his
official function to be compensated by claim' and if any other sum of money or other
perquisites are made to the holder of office as compensatory allowance, he would not get
the benefit of clause (9) of the Schedule which was added. In the instant cases, the
appellants were certainly in receipt of variety of perquisites which cannot be said to be
given to them by way of compensatory allowance. The mobile phone, telephone and the
chauffeur driven car were all permitted to be used for unlimited purposes and they were
not restricted to official purposes. Moreover, Rule 7 of the 1967 Rules specifically states
that the Chairman, Vice-Chairman and other members of the Board shall be paid such
salary or other honorarium and allowances from the funds of the Boards as the
Government may from time to time fix. The appellants were entitled to get salary or
honorarium by virtue of this rule. The mere fact that they had not received or they had not
opted to get this salary or honorarium is immaterial. By virtue of the said rule, they are
entitled to get salary or honorarium and that, by itself, would show that they were not
entitled to get the benefit of the Schedule of the 1982 Act.
8. The respondents in these two cases had raised a contention that the 1982 Act itself was
not applicable to the State of Goa, Daman and Diu as the same was not adopted by the
State Legislature. The respondents had contended that in the absence of adoption under
Section 57 of the Goa State Re-organisation Act, 1987, the 1982 Act had no application to
the State of Goa, Daman and Diu. This plea was accepted by the learned Single Judge of
the High Court. The respondents in these cases contended that the finding of the learned
Single Judge in this regard is not correct.
9. In this case, the appellants herein contended that Article 239A of the Constitution
provided for creation of local legislatures or council of ministers or both for certain
Union Territories and the Parliament enacted the Government Territories Act, 1963. As
per Section 3 of the Act of 1963, the Legislative Assembly of the Union Territory of Goa,
Daman and Diu came into existence and sub-section (1) of Section 14 of the Act of 1963
provided that a person shall be disqualified for being chosen as, and for being a Member
of the Legislative Assembly of the Union Territory, inter alia, if he holds any office of
profit under the Government of India, or the Government of any State, or the Government
of the Union Territory, other than the office declared by law made by Parliament, or the
Legislative Assembly of the Union Territory, not to disqualify its holder, i.e., the
Legislative Assembly of the Union Territory of Goa, Daman and Diu. The Act passed in
1982 provided for removal of certain disqualifications for being chosen and for being a
member of the Legislative Assembly of Goa, Daman and Diu. That Act was passed under
Section 14(1) of the Act of 1963.
10. The respondent had contended that when Union Territory of Goa, Daman and Diu
became a State, the Assembly of the State of Goa had not passed any law nor had adopted
the 1982 Act which was in force. The appellants contended that as per Section 66 of the
Goa, Daman and Diu Reorganization Act, 1987, the Act of 1982 continues to be in force
but this plea was rejected by the learned Single Judge.
11. We have examined the claim of the appellants in the light of the 1982 Act and hold
that the appellants are not entitled to get the protection of the Act. Therefore, whether this
Act was adopted by the State Assembly of Goa or not, need not be considered at this
stage and we leave open the question to be considered in other appropriate cases.
12. We find no merit in these appeals and the appeals are dismissed accordingly.
Appeals dismissed.
AIR 2008 SUPREME COURT 1226 "Pogula Komuraiah v. State of A. P."
(From : Andhra Pradesh)*
Coram : 2 Dr. A. PASAYAT AND P. SATHASIVAM, JJ.
Criminal Appeal No. 94 of 2008 (arising out of SLP (Cri.) No. 6190 of 2007), D/- 15 -1
-2008.
Pogula Komuraiah v. State of A.P.
Penal Code (45 of 1860), S.300, S.304, Part I - MURDER - CULPABLE HOMICIDE -
COMPLAINT - Murder - Complaint filed after 5 hours - Motive absent - Overt act
attributed
@page-SC1227
to accused-appellant - Appellant armed only with lathi and not iron rod as alleged -
Medical evidence not corroborating oral evidence as to manner of assault - Conviction of
appellant liable to be altered from S.300 to S.304, Part I. (Para 9)
S. Sadasiv Reddy, Mrs. S. Usha Reddy, for Appellant; Ms. Altaf FaLhima, Mrs. D.
Bharathi Reddy, for Respondent.
* Cri. A. No. 1114 of 2005, D/- 27-7-2006 (A.P.).
Judgement
Dr. ARIJIT PASAYAT, J. :-Leave granted.
2. Challenge in this appeal is to the judgment of a Division Bench of Andhra Pradesh
High Court, disposing of four Criminal Appeals i.e. Criminal Appeal Nos. 1114, 1128,
1130 and 1155 of 2005.
3. 16 accused persons were charged for offence punishable under Sections 147, 148, 448
read with Section 149 and Section 302 read with Section 149, 324 read with Section 149
of the Indian Penal Code, 1860 (in short IPC).
4. The High Court by the impugned order disposed of the appeals with the following
observations:
In the result, Crl.A.No. 1114 of 2005 is allowed in part. Crl. A.No.1128 of 2005 is
allowed. Crl. A.No.1130 of 2005 is dismissed. Crl. A.No. 1155 of 2005 is allowed. The
convictions and sentences imposed by the lower Court on A-1, A-3, A-7 to A-9, A-12 and
A-13 for the offences under Sections 302 read with Sections 149, 148, 448 read with 149,
324 read with 149 of I.P.C., are confirmed. The convictions and the sentences imposed on
A-2, A-4 to A-6, A-10, A-11 and A-14 to A-16 for all offences are set aside and they shall
be set at liberty forthwith, if they are not required in any other crime.
5. It is 10 be noted that the appellant was accused No. 12 (for short A12) before the Trial
Court. It is relevant to note that accused Nos. 1 and 3 filed SLP (Crl.) No.5591 of 2006
before this Court which was subsequently converted into Criminal Appeal No. 222 of
2006. By judgment dated 19th February, 2007 the appeal was partly allowed with the
following findings:
"......If the evidence on record is considered on the touchstone principles set out above the
inevitable conclusion is that the proper conviction would be Section 304 Part I IPC
instead of Section 302 IPC. The conviction of the appellants is accordingly altered from
Section 302 read with Section 149 to Section 304 Part I read with Section 149 IPC.
Custodial sentence of 10 years would meet the ends of justice. The findings of the guilt in
respect of other offences and the sentences imposed do not warrant interference. The
sentence shall run concurrently. The appeal is allowed to the aforesaid extent.
6. Learned counsel for the appellant submitted that the present appellant stands in the
same footing as the appellants in Criminal Appeal No.222 of 2006 and the present appeal
may be disposed of on similar terms.
7. Learned counsel for the respondent-State submitted that the present appelllant was
armed with an iron rod, while the appellants in Criminal Appeal No.222 of 2006 were
armed with lathi. Therefore, the similar treatment cannot be given to the present
appellant.
8. It is to be noted that the High Court with reference to the evidence of PW1 noted as
follows :
"19. Sri C.Padmanabha Reddy, the learned Senior Counsel for the appellants submitted
that there was a delay of five hours in preferring the complaint and there were no specific
overt acts attributed to the accused and omnibus allegations were made. The medical
evidence is not corroborating with the oral evidence and the accused has no
premeditation to kill the deceased and no motive was suggested by the prosecution for the
commission of the offence and it was only in respect of the hiring of the Auto rickshaw
by the deceased. He further submitted that in Ex.P-1 complaint only seven accused were
said to be attacked and the witnesses mentioned in the inquest report were not examined.
PW-1 attributed overt acts only to A-12 and A-13 and the remaining accused were said to
be beaten with sticks which is different from the version given in Ex.P-1. Though PWs. 5
and 6 stated that all the accused attacked, their names were not mentioned in Ex.P-1. The
overt acts attributed to the accused during the course of evidence were not mentioned in
the earlier statements and the whole version is subsequently developed to strengthen the
prosecution. The receipt of injuries by PW-2 was not corroborated with the evidence of
the Doctor who examined PW-2. Though the accused were attributed overt acts of
beating the deceased, there were no corresponding injuries on the deceased
@page-SC1228
and for the alleged recovery of properties, the recovery panch turned hostile and did not
support the prosecution case and as A-13 was implicated in this case, all the accused are
entitled for benefit of doubt and they are entitled for acquittal.
24. Since there is specific mention about A-1, A-3, A-7 to A-9, A-12 and A-13 beating the
deceased with sticks, we are unable to agree with the argument that the witnesses
improved the version by attributing overt acts to the accused in the evidence......"
9. Above being the position, the findings recorded in Criminal Appeal No.222 of 2006 by
this Court are applicable to the present appeal. Accordingly appellant's conviction is
altered to Section 304 Part I read with Section 149 IPC as was done in the case of the
appellants in the aforesaid appeal. Custodial sentence of 10 years would meet the ends of
justice. The findings of the guilt in respect of other offence and the sentences imposed do
not warrant any interference. The sentences shall run concurrently.

10. The appeal is allowed to the aforesaid extent.


Order accordingly.
AIR 2008 SUPREME COURT 1228 "General Manager, Pench Area, Parasia, M. P. v.
Barkan"
(From : Madhya Pradesh)*
Coram : 2 Dr. A. PASAYAT AND P. SATHASIVAM, JJ.
Civil Appeal No. 2711 of 2001, D/- 13 -12 -2007.
General Manager, Pench Area, Parasia, M. P. and Ann v. Barkan @ Kanhaiya.
Constitution of India, Art.133 - APPEAL - CONTRACT - DOCUMENTS - Appeal -
Permission to file documents in evidence - Employment to land oustees - Suit for specific
performance of promise to give employment - State producing evidence to show that
employment has been given as per promise - But no document in that regard filed - S. C.
permitting State to file document - Document filed showing that promise has been
fulfilled - Suit liable to be dismissed. (Paras 5, 6)

Anip Sachthey and Mohit Paul, for Appellants; B.K. Satija, for Respondent.
* S.A. No. 856 of 1996, D/- 21-12-1999 (MP).
Judgement
1. Dr. ARIJIT PASAYAT, J. :- Challenge in this appeal is to the order passed by a learned
Single Judge of the Madhya Pradesh High Court at Jabalpur Bench dismissing the appeal
filed by the appellants.
2. Background facts in a nutshell are as follows:
Respondent filed a suit for specific performance of the contract of employment.
According to the appellants, his lands were acquired for the purpose of construction of
quarters for the employees.
2-A. Sale-deed was executed in respect of the land and there was specific provision in a
preceding agreement that four persons were to be given employment. Allegation was that
only three had been provided employment and in spite of assurance the defendants did
not give the job to the plaintiffs.
3. Stand of the defendants was that the suit was not maintainable. In fact, four persons
have been given employment. The Trial Court and the First Appellate Court accepted the
position that three persons had been given jobs but held that no job was provided to the
appellant. The Trial Court noticed that even though it was contended by the present
appellants that one son of the plaintiff had been given a job, no document in that regard
had been filed. The First Appellate Court and the High Court were of the same view.
4. The High Court held that the stand of the appellant that in view of Section 14 of the
Specific Relief Act, 1963, suit for specific performance is not maintainable and is subject
to certain exceptions. It was held that since there was a solemn promise to employ four
persons the appellants should not be permitted to wriggle out the promise by taking the
plea that Section 14 of the Act bars a suit of the nature filed.
5. By order dated March 31, 2000, this Court had permitted the appellants to file
documents to show that the son of the respondent No. 1 had been given appointment on
his nomination. The same has been filed. Though this document was not part of the
records of the Courts below, but other evidence was available to show that, in fact, son of
respondent No. 1 named Guntoo was appointed at the request of respondent No. 1. The
document placed on record by the appellant pursuant to the order of this Court also
clearly establishes this fact.
6. In that view of the matter the suit filed
@page-SC1229
by respondent No. 1 deserves to be dismissed and the orders of the Trial Court, First
Appellate Court and the High Court in the Second Appeal deserve to be set aside which
we direct.
7. The appeal is allowed to the aforesaid extent. No costs.
Order accordingly.
AIR 2008 SUPREME COURT 1229 "Rakesh v. State of M.P."
(From : Madhya Pradesh)*
Coram : 2 Dr. A. PASAYAT AND P. SATHASIVAM, JJ.
Criminal Appeal No. 287 of 2008 (arising out of SLP (Cri.) No. 6598 of 2007), D/- 11 -2
-2008.
Rakesh v. State of M.P.
(A) Penal Code (45 of 1860), S.300, Exception 1 and Exception 4 - MURDER -
CULPABLE HOMICIDE - Murder or culpable homicide not murder - Exceptions 1 and 4
- Contra distinguished - Incident outcome of sudden fight - Benefit of Exception 4 -
Ingredients to be proved.
The Fourth Exception to S. 300 IPC covers acts done in a sudden fight. The said
Exception deals with a case of prosecution not covered by the First Exception, after
which its place would have been more appropriate. The Exception is founded upon the
same principle, for in both there is absence of premeditation. But, while in the case of
Exception 1 there is total deprivation of self control, in case of Exception 4, there is only
that heat of passion which clouds men's sober reason and urges them to deeds which they
would not otherwise do. There is provocation in Exception 4 as in Exception 1; but the
injury done is not the direct consequence of that provocation. In fact Exception 4 deals
with cases in which notwithstanding that a blow may have been struck, or some
provocation given in the origin of the dispute or in whatever way the quarrel may have
orginated, yet the subsequent conduct of both parties puts them in respect of guilt upon
equal footing. For the application of Exception 4, it is not sufficient to show that there
was a sudden quarrel and that there was no premeditation. It must further be shown that
the offender has not taken undue advantage or acted in cruel or unusual manner. The
expression 'undue advantage' as used in the provision means 'unfair advantage'. (Para
7)
(B) Penal Code (45 of 1860), S.300, Exception 4, S.304, Part 1 - MURDER -
CULPABLE HOMICIDE - Murder - Accused alleged to have assaulted deceased with
knife, kicks, fits and blows - Incident taking place out of a quarrel between accused and
deceased - Accused entitled to benefit of Exception 4 to S.300 - Liable to be convicted
only u/S.304, Part 1. (Paras 3, 9)
Cases Referred : Chronological Paras
2007 AIR SCW 7153 : AIR 2008 SC 406 (Rel. on) 8
2006 AIR SCW 1678 : 2006 Cri LJ 2111 : 2006 (3) AIR Bom R 321 (Rel. on) 8
Rajiv Talwar, Parmanand, for Appellant; Govind Goel, C.D. Singh, M.P. Singh, Vairagya
Vardhan, Sunny Chowdhary, Ram Naresh Yadav, Navin Sharma, for Respondent.
* Cri. A. No. 382 of 2000, D/- 10-7-2006 (M.P.) (Indore Bench)
Judgement
1. Dr. ARIJIT PASAYAT, J. :- Leave granted.
2. Challenge in this appeal is to the judgment of a Division Bench of the Madhya Pradesh
High Court at Indore, upholding conviction of the appellant for offence punishable under
Section 302 of the Indian Penal Code, 1860 (in short the 'IPC') and sentence of
imprisonment for life. Four persons faced trial for committing murder of Kailash
(hereinafter referred to as the 'deceased'). Though the trial court had convicted the
appellant for offence punishable under Section 302 IPC, three persons were convicted for
offences punishable under Section 302 read with Section 34 IPC. By the impugned
judgment, conviction of others was altered and each one of them was convicted for
offences punishable under Section 326 IPC read with 34 IPC and was sentenced to
undergo rigorous imprisonment for three years each and to pay a fine of Rs. 1,000/- with
default stipulation. But the conviction of the appellant as rioted above was maintained.
3. Background facts in a nutshell are as follows :
On 15/11/1998 at about 08.30 pm, in Bhagirathpura, near the house of Sheetal Deen,
Complainant Ramesh and witness Lalchand were standing near the culvert,
@page-SC1230
when Praveen (PW4) came shouting that brother of Ramesh namely Kamlesh was being
assaulted by the appellants. These persons, therefore, rushed to the place and witnessed
that appellant Shailu, Raju, and Ravi had kept Kailash in their grip, while Rakesh was
assaulting him with a knife, and others were administering kicks, fits and blows. When
these persons raised an alarm, the accused persons fled away. Kailash was immediately
taken to M. Y. Hospital. He had number of injuries which had been dressed initially but
when Doctor saw Kailash, he declared him dead. According to Ramesh Prajapat there
was a quarrel between them with regard to peels of eggs and it was on that account the
accused persons had assaulted his brother. Report on this incident (Ex P /18) was lodged
which was recorded in Rojnamcha. On being informed by the operator from M.Y.
Hospital about death of Kailash, Inayat Hussain recorded information as (Ex. P/28m) and
forwarded Raifulla Khan to investigate. Raifulla Khan then recorded Dehati Naish
(Ex.P/W 2) was lodged by Ramesh and after issuing Subpoena, held inquest of which he
prepared report. He also forwarded the dead body under requisition Ex.P2/27 of which
post-mortem report was received from Dr. Raj Kumar Singh.
On the basis of information lodged, investigation was undertaken and charge sheet was
placed. The accused persons abjured guilt and pleaded false implication. The trial court
and the High Court found the evidence of the witnesses to be credible and cogent and as
noted above directed conviction.
4. In support of the appeal, learned counsel for the appellant submitted that the evidence
does not establish guilt of the present appellant. According to him, even if prosecution
version is accepted in toto, offence under Section 302 IPC is not made out. In any event
an offence under Section 302 IPC is not made out. According to him the occurrence took
place in the course of a sudden quarrel and therefore Exception 4 to Section 300 IPC is
attracted.
5. Learned counsel for the State supported the judgments of the Courts below.
6. For bringing in operation of Exception 4 to Section 300 IPC, it has to be established
that the act was committed without premeditation, in a sudden fight in the heat of passion
upon a sudden quarrel without the offender having taken Undue advantage and not
having acted in a cruel or unusual manner.
7. The Fourth Exception to Section 300 IPC covers acts done in a sudden fight. The said
Exception deals with a case of prosecution not covered by the First Exception, after
which its place would have been more appropriate. The Exception is founded upon the
same principle, for in both there is absence of premeditation. But, while in the case of
Exception 1 there is total deprivation of self-control, in case of Exception 4, there is only
that heat of passion which clouds men's sober reason and urges them to deeds which they
would not otherwise do. There is provocation in Exception 4 as in Exception 1; but the
injury done is not the direct consequence of that provocation. In fact Exception 4 deals
with cases in which notwithstanding that a blow may have been struck, or some
provocation given in the origin of the dispute or in whatever way the quarrel may have
originated, yet the subsequent conduct of both parties puts them in respect of guilt upon
equal footing. A "sudden fight" implies mutual provocation and blows on each side. The
homicide committed is then clearly not traceable to unilateral provocation, nor in such
cases could the whole blame be placed on one side. For if it were so, the Exception more
appropriately applicable would be Exception 1. There is no previous deliberation or
determination to fight. A fight suddenly takes place, for which both parties are more or
less to be blamed. It may be that one of them starts it, but if the other had not aggravated
it by his own conduct it would not have taken the serious turn it did. There is then mutual
provocation and aggravation, and it is difficult to apportion the share of blame which
attaches to each fighter. The help of Exception 4 can be invoked if death is caused (a)
without premeditation; (b) in a sudden fight; (c) without the offender having taken undue
advantage or acted in a cruel or unusual manner; and (d) the fight must have been with
the person killed. To bring a case within Exception 4 all the ingredients mentioned in it
must be found. It is to be noted that the "fight" occurring in Exception 4 to Section 300
IPC is not defined in IPC. It takes two to make a fight. Heat of passion requires that there
must be no time for the passions to cool down and in this case, the parties have worked
themselves into a fury on account of the verbal altercation in the beginning.
@page-SC1231
A fight is a combat between two or more persons whether with or without weapons. It is
not possible to enunciate any general rule as to what shall be deemed to be a sudden
quarrel. It is a question of fact and whether a quarrel is sudden or not must necessarily
depend upon the proved facts of each case. For the application of Exception 4, it is not
sufficient to show that there was a sudden quarrel and that there was no premeditation. It
must further be shown that the offender has not taken undue advantage or acted in cruel
or unusual manner. The expression "undue advantage" as used in the provision means
"unfair advantage".
8

. The above position is highlighted in Sandhya Jadhav v. State of Maharashtra (2006) 4


SCC 653), Thankachan and Anr. v. State of Kerala (2007 (11) SCR 1128). 2006 AIR
SCW 1678
2007 AIR SCW 7153

9. In the background of the principles of law indicated above, the appropriate conviction
would be in terms of Section 304 Part I IPC, and custodial sentence of 10 years would
meet the ends of justice.
10. Appeal is allowed to the aforesaid extent.
Order accordingly.
AIR 2008 SUPREME COURT 1231 "Santosh Kumar v. Sarla Devi"
(From : Allahabad)*
Coram : 2 TARUN CHATTERJEE AND H. S. BEDI, JJ.
S. L. P. (C) No. 8146 of 2007, D/- 6 -2 -2008.
Santosh Kumar and Anr. v. Sarla Devi.
U.P. Urban Buildings (Regulation of Letting, Rent and Eviction) Act (13 of 1972), S.21 -
EVICTION - TENANCY - SUPREME COURT - Eviction - Tenant refused offer of two
shops by landlord before High Court - However agreed to accept such offer before
Supreme Court and is ready to vacate premises in question on receiving possession in
respect of two shops - Supreme Court passed order accordingly. (Para 2)

Ajay Choudhary, for Petitioners; Garima Prashad, Ms. Neha Goyal, for Respondent.
* C. M. W. P. No. 65813 of 2006, D/- 7-12-2006, reported in 2007 (66) All LR 463 : 2007
(2) ALJ (NOC) 208.
Judgement
ORDER:-This special leave petition is directed against the judgment and final order dated
7th of December, 2006 passed by the High Court of Judicature at Allahabad in Civil
Misc. Writ Petition No.65813 of 2006. On 17th of May, 2007, a limited notice was
issued. While issuing a limited notice on the special leave petition, the following order
was passed :-
"Learned senior counsel for the petitioners submits that although the respondent-landlord
had offered two shops to the petitioner-tenants, but that offer was refused by them. He
submits that the petitioners are now agreeable to accept such offer and is ready to vacate
the suit premises subject thereto. Accordingly, we order issuance notice limited to the
question that whether the offer of the respondent-landlord, which was not accepted by the
petitioners in the High Court and is now acceptable to them, is still open for acceptance.
There will be ad interim stay of dispossession till 15th July, 2007."
2. A reading of this order would show that in the High Court the petitioners who are the
tenants was offered by the landlord-respondent two shops, but that offer was refused by
them. But in this Court, the petitioners are agreed to accept such offer and is ready to
vacate the premises in question, subject to receiving possession in respect of two shops
which were offered to the petitioners before the High Court. The learned counsel
appearing on behalf of the respondent had submitted before us that they shall hand over
two shop rooms, namely, Shop Nos. 6 and 7 Balaji Market, Main Road, Tundla (U.P.) to
the petitioners within a period of two weeks from this date. If possession of the aforesaid
two shop rooms is delivered, the appellant shall deliver possession of the tenanted suit
premises to the respondent within a month from the date of taking over possession of two
shop rooms from the respondent by them. In the event, possession of two shop rooms are
not delivered or taken possession of, it will be open to the respondent to execute the
decree in accordance with law and this special leave petition shall stand dismissed.
3. With the above direction, the special leave petition is dismissed. No order as to costs.
Petition dismissed.
@page-SC1232
AIR 2008 SUPREME COURT 1232 "State of Orissa v. M/s. Tata Iron and Steel Co. Ltd."
(From : Orissa)*
Coram : 2 Dr. A. PASAYAT AND P. SATHASIVAM, JJ.
Civil Appeal Nos. 653 with 654, 655 and 671 To 673 of 2006, D/- 4 -2 -2008.
State of Orissa and Anr. v. M/s. Tata Iron and Steel Co. Ltd. and Ors.
Constitution of India, Art.226 - WRITS - JUDICIAL REVIEW - CONTRACT -
INDUSTRIAL DEVELOPMENT - Judicial review - Contractual matter - Offers invited
for participation in joint venture - Petition challenging threshold rejection of technical
bids of petitioners - High Court holding that selection of offerer was made without proper
evaluation of bid and not in impartial manner - Conclusion of High Court held were
indefensible being based on extraneous material - Considering passage of time S. C.
instead of remanding matter to H. C. passed necessary directions to State Industrial
Development Corporation. (Paras 10, 11)

G.E. Vahanvati, SG, B.K. Mohanti, AG (Orissa), Arun Jaitley, K.K. Venugopal, Dr. A.M.
Singhvi, Shyam Divan, Shanti Bhushan, Ashok Parija, T.R. Andhiarujina, R.F. Nariman,
Sr. Advocates, Rajat Rath, Ms. Suman Kukrety, Raj Kumar Mehta, Rishi Maheshwari,
P.S. Sudheer, Ms. Aanne Mathew, Mrs. Shally Bhasin Maheshwari, Sanjeev K. Kapoor,
Vishal Gupta, Vikram Bajaj, Kumar Mihir, Avinash Menon (for M/ s. Khaitan and Co.),
R.N. Karnajawala, Gopal Jain, Akhil Sibal, Ms. Nandini Gore, Ms. Prachi Goel. Mrs.
Manik Karanjawala, for appearing parties.
* W.P. (C) No. 6798 of 2004, D/- 18-11-2004 (Orissa).
Judgement
Dr. ARIJIT PASAYAT, J. :-Challenge in these appeals is to the judgment of a Division
Bench of the Orissa High Court allowing the Writ Petitions filed by Visa Industries
Limited (in short the 'VISA') and Another (Writ Petition (C) No. 5128 of 2004) and M/s.
Tata Iron and Steel Co. Ltd. (in short the 'TISCO') (Writ Petition (C) No. 6798 of 2004).
By the impugned Judgment the High Court held that the technical bids of VISA and
TISCO could not have been rejected at the threshold without proper evaluation in terms
of the eligibility condition as set out in the concerned advertisement. It was also held that
Jindal Strip Limited (in short 'Jindal's') bids were never evaluated and assessed in a
dispassionate and impartial manner. There was no attempt to find out as to which of the
bids offered by the three parties would give maximum advantage to the State in terms of
public interest and State exchequer Certain other observations were made questioning
bona fides of the officials of the State and Industrial Development Corporation of Orissa
Limited (in short the 'IDCOL'). It was held that power of judicial review was to be
exercised as the selection of Jindal as a Joint Venture Partner for the project in question
was not properly done. Therefore the IDCOL's decision to select Jindal cannot be
maintained and was set aside.
2. It was noted that the matter could have been remitted to IDCOL for fresh evaluation
and formation of merits on the bids of the respective parties, but it was not thought
proper. It was also noted that Jindal proposed to set up stainless Steel Industry which
could not have been considered as a relevant factor while deciding the question of Joint
Venture Partner. However, IDCOL was given the opportunity to issue a fresh
advertisement for the purpose of setting out in clear terms whether it wants stainless
industries to be set up in the State or other industry where chrome could be used as an
ingredient.
3. The technical bids offered by the various parties are on record. By the last date for
receipt of offers, four parties had submitted their offers but later on Jindal Steel Power
Ltd. did not want to continue.
4. The present appeals arise out of Special Leave Petitions filed by the State of Orissa,
IDCOL and Jindal.
5. Primary stand of Mr. G.E. Vahanvati, Learned Solicitor General is that the High
Court's approach is clearly erroneous. It has taken into account various irrelevant and
extraneous materials without even any pleading in that regard. It has assumed collusion,
loss of revenue if Jindal's bid was to be accepted. It is not fathomable as to on what basis
the conclusions were arrived at, that too without any material foundation. The similar
effect is the submission of Jindal. It is to be noted that Jindal Steels Limited is presently
known as Jindal Stainless Steel Limited. However for the sake of convenience it shall be
described as 'Jindal' in this judgment.
@page-SC1233
6. To similar effect is the submission of learned counsel for IDCOL.
7. Learned counsel for TISCO and VISA submitted that the conclusions of the High
Court are in order. Considering the parameters of judicial review it is clear that the
Government granted approval in the most mechanical manner without application of
mind to the facts of the case. It was submitted that as has been rightly held by the High
Court Jindal did not satisfy the required parameters and, therefore, its bid could not have
been accepted.
8. One of the factors highlighted by learned counsel for TISCO is that the information
brochure and the NIT referred to certain vague expressions like "Value addition". In view
of such an indefinite condition the bids submitted by TISCO and VISA could not have
been rejected at the threshold and therefore the High Court has rightly interfered in the
matter.
9. At this juncture, it would be relevant to quote the Recommendations of the Technical
Committee constituted for evaluation of the offers received for development of
Tangarpada Chromite Deposit in Joint Venture, which reads as follows :
"RECOMMENDATIONS OF THE TECHNICAL COMMITTEE CONSTITUTED FOR
EVALUATING THE OFFERS RECEIVED FOR DEVELOPMENT OF TANGARPADA
CHROMITE DEPOSIT IN JOINT VENTURE
Offers for development of Tangarpada Chromite Deposit in Joint Venture were received
from four parties namely :
1. Tata Iron and Steel Co. Ltd.;
2. Jindal Strips Limited;
3. Jindal Steel and Power Ltd.; and
4. VISA Industries Limited
Before opening of the sealed offers, Jindal Steel and Power (one of the offer) withdrew
its offer. The technical bid of the other three parties were opened by the committee in
presence of the respective parties on 9th December, 2002. Each party presented their case
before the Technical Committee on the same day."
10. It may be stated here that certain conclusions of the High Court are clearly
indefensible. The observations relating to favoritism, so far as Jindal is concerned, are
clearly without any foundation.
11. On the sole ground that the High Court had relied upon extraneous materials and has
arrived at unfounded conclusions, in normal course we would have set aside the order
and asked the High Court to re-consider the matter. But considering the passage of time
and more particularly the fact that the advertisement was issued in 2002 and on the basis
of materials on record, we dispose of the appeals on the following terms :
1. It shall be treated that the technical bids of all the three parties are valid.
2. The financial bids were submitted about five years back it would be appropriate to
permit the parties to submit revised financial bids within three weeks.
3. The appropriate and authorized Committee of IDCOL shall consider the technical bids
and the financial bids, keeping in view the parameters of the advertisement, the NIT and
the best interest of the State.
12. It is needless to say the Committee examining the bids shall take note of all relevant
factors. In case it is considered appropriate and in the interest of the State, it shall be open
to the State Government to negotiate with the parties so that the best interest of the State
including generation of the revenue of the State and overall development of the State in
the relevant fields could be achieved.
13. Since the matter is pending since long it would be desirable for the State Government
to ensure that the technical bids and the revised financial bids to be submitted within
three weeks as directed earlier, be evaluated and informed decision taken by end of June,
2008. The observations and conclusions about mala fides of the officials and their alleged
favouritism stand quashed.
14. The appeals are allowed to the aforesaid extent without any order as to costs.
Order accordingly.
AIR 2008 SUPREME COURT 1233 "Ramesh Singh v. Satbir Singh"
(From : Delhi)*
Coram : 2 S. B. SINHA AND V. S. SIRPURKAR, JJ.
Civil Appeal Nos. 545-546 of 2008 (arising out of SLP (C) Nos. 13019-13020 of 2007),
D/- 21 -1 -2008.
Ramesh Singh and Ann v. Satbir Singh and Anr.
Motor Vehicles Act (59 of 1988), S.168 - MOTOR VEHICLES - Motor accident -
Compensation -
@page-SC1234
Multiplier - Determination - Age of accused or of claimant whichever is higher is relevant
- Deceased aged 22 years - Claimant 55 years old - Selection of multiplier 8 is proper -
Second schedule to act to be used as guide not as mathematical formula. (Paras 4, 5)
Cases Referred : Chronological Paras
2007 AIR SCW 6197 : AIR 2008 SC 103 : 2007 (6) AIR Kar R 481 (Rel. on) 5
2005 AIR SCW 1801 : AIR 2005 SC 2157 (Rel. on) 4
2004 AIR SCW 1864 : AIR 2004 SC 2107 (Ref.) 5
(1996) 4 SCC 362 (Rel. on) 5
Ms. Manjeet Chawla, for Appellants; Praveen Swarup, for Respondents.
MAC. App. Nos. 330-331 of 2006, D/- 31-1-2007(Del)
Judgement
1. V. S. SIRPURKAR, J. :-Leave granted.
2. Not being satisfied with the Judgment of the High Court enhancing the compensation
by a sum of Rs. 50,000/-, the parents of deceased Banu Pratap Singh have filed these
appeals. Deceased Banu Pratap Singh was killed in an accident on 29-3-2004 involving a
truck which was being driven by first respondent, Satbir Singh. The truck belonged to
Municipal Corporation of Delhi. At the time of his death, Bhanu Pratap Singh was about
22 years of age. It was claimed by the first appellant, i.e., the father of the deceased that
he was 41 years old at the time of death of Bhanu Pratap Singh. The Trial Court, on the
basis of the evidence, came to the conclusion that the annual loss of dependency
regarding Bhanu Pratap Singh could be taken at Rs. 28,992/-. It was further held that
Appellant No. 1, the father of the deceased was 55 years of age at the time of accident
and that is how the Trial Court applied the multiplier of 8 years and held that the total loss
of dependency was Rs. 2,31,936/-. Further compensation of Rs. 2,000/- for funeral
expenses and Rs. 2500/ on account of loss of estate was added to the above sum and total
compensation of Rs. 2,36,436/- was awarded with interest at 6% from the date of filing of
the petition till realization. It was held that both respondents, namely, the driver and the
owner, i.e., Municipal Corporation of Delhi were jointly and severally liable to pay the
compensation, however, primary obligation to pay the compensation was fixed against
second respondent. An appeal was filed by the appellants herein before the High Court
wherein three grounds were raised. It was firstly contended that the future prospects were
ignored by the Tribunal; secondly it was contended that the Tribunal was wrong in
adopting the multiplier of 8 as the father of the deceased was only 41 years of age at the
time of death; and the third contention was that no compensation was awarded for the
loss of love and affection of a son to the parents. The High Court disbelieved the theory
that the father was only 41 years of age on the date of the accident or that he was
confused when he mentioned his age to be 55 years at the time of evidence. The High
Court also disbelieved the High School certificate in relation to the father and held the
claim to be absurd. The High Court considered the first and the second contentions
together since they were inter-related and held that increase of Rs. 50,000/- would be
reasonable, taking into account the possibility of increase in minimum wages, due to loss
of love and affection of the child and pain and sufferings which the parents would live all
their life. The High Court passed the order accordingly.
3. Learned counsel appearing on behalf of the appellant very fairly does not argue the
question of the age of the father and accepted the findings that the father was 55 years at
the time of the accident and not 41 years as claimed by him in the appeal filed before the
High Court. However, as regards the application of the multiplier, the learned counsel
heavily relied on the Second Schedule and contends that this was the case under Section
163A of the Motor Vehicles Act and since the age of the deceased was only 22 years, the
multiplier of 16 was liable to be made applicable. Alternatively, the counsel submits that
at least the multiplier of 11 ought to have been made applicable considering the age of the
Appellant No. 2, the mother of the deceased, to be 52 years.
4

. We have given anxious consideration to these contentions and are of the opinion that the
same are devoid of any merits. Considering the law laid down in New India Assurance
Co. Ltd. v. Charlie [(2005) 10 SCC 720], it is clear that the choice of multiplier is
determined by the age of the deceased or claimants whichever is higher. Admittedly, the
age of the father was 55 years. The question of mothers age never cropped up because
2005 AIR SCW 1801

@page-SC1235
that was not the contention raised even before the Trial Court or before us. Taking the age
to be 55 years, in our opinion, the courts below have not committed any illegality in
applying the multiplier of 8 since the father was running 56th year of his life.
5

. The learned counsel relying on the 2nd Schedule of the Act contended that the deceased
being about 16 or 17 years of age, a multiplier of 16 or 17 should have been granted. It is
undoubtedly true that Section 163-A was brought on the Statute book to shorten the
period of litigation. The burden to prove the negligence or fault on the part of driver and
other allied burdens u/S. 140 or 166 were really cumbersome and time consuming.
Therefore as a part of social justice, a system was introduced via Section 163-A wherein
such burden was avoided and thereby a speedy remedy was provided, The relief u/S. 163-
A has been held not to be additional but alternate. The Schedule provided has been
threadbare discussed in various pronouncements including Deepal Girishbhai Soni vs.
United India Insurance Co. Ltd. [(2004) 5 SCC 385]. 2nd Schedule is to be used not only
referring to age of victim but also other factors relevant therefor. Complicated questions
of facts and law arising in accident cases cannot be answered all times by relying on
mathematical equations. In fact in U.P. State Road Transport Corporation vs. Trilok
Chandra [(1996) 4 SCC 362], Ahmedi.J. (As the Chief Justice then was) has pointed out
the shortcomings in the said Schedule and has held that the Schedule can only be used as
a guide. It was also held that the selection of multiplier cannot in all cases be solely
dependent on the age of the deceased. If a youngman is killed in the accident leaving
behind aged parents who may not survive long enough to match with a high multiplier
provided by the 2nd Schedule, then the Court has to offset such high multiplier and
balance the same with the short life expectancy of the claimants. That precisely has
happened in this case. Age of the parents was held as a relevant factor in case of minor's
death in recent decision in Oriental Insurance Co. Ltd. vs. Syed Ibrahim and Ors. [JT
2007 (11) SC 113]. In our considered opinion, the Courts below rightly struck the said
balance. 2004 AIR SCW 1864
2007 AIR SCW 6197

6. With this, we dispose of these appeals. There will be no order as to costs.


Order accordingly.
AIR 2008 SUPREME COURT 1235 "U.P. State Sugar Corpn. Ltd. v. Kamal Swaroop
Tondon"
(From : Allahabad)
Coram : 2 C. K. THAKKER AND P. SATHASIVAM, JJ.
Civil Appeal No. 513 of 2008 (arising out of SLP (C) No. 11599 of 2006), D/- 18 -1
-2008.
U.P. State Sugar Corpn. Ltd. and Ors. v. Kamal Swaroop Tondon.
(A) Constitution of India, Art.311 - TERMINATION OF SERVICE - DEPARTMENTAL
PROCEEDINGS - SERVICE MATTERS - Departmeatal proceedings - Initiation of -
Show cause notice served upon employee on last day of fete service - Proseedings cannot
be said to be initiated against employee after he retired from service.
In the instant case the show cause notice was issued to the respondent employee on
January 13, 2000 when he was very much in service. The respondent submitted his
explanation on January 15, 2000 which was not found to be satisfactory. A regular show
cause notice was, therefore, issued by the Corporation on January 31, 2000 and was
served upon the respondent employee on the same day. The notice was also sent by
registered post which was received by the employee on February 11, 2000. But it is clear
from the documents that show cause notice was issued and replied. A regular show notice
as to departmental inquiry was also served upon the respondent employee on the last day
of his service which was January 31, 2000. Therefore, it could not be said that the
proceedings had been initiated against the respondent employee after he retired from
service. (Paras 10, 31)
(B) Constitution of India, Art.311 - TERMINATION OF SERVICE - DEPARTMENTAL
PROCEEDINGS - NEGLIGENCE - Departmental proceedings - Initiated for recovery of
losses sustained by Corporation due to negligence of delinquent employee - Could be
continued even after retirement of said employee - Said loss could be recovered from his
retiral benefits.
W. P. No. 484 (S/B) of 2000, D/-24-02-2006 (All.), Reversed.
No rule of universal application can be laid down that if there is delay in initiation of
proceedings for a particular period, they must necessarily be quashed. Therefore,
@page-SC1236
when departmental proceedings were initiated against the delinquent on basis of show
cause notice served upon him while he was very much in service it cannot be said that,
the proceedings were initiated after the respondent retired and that there was no power,
authority or jurisdiction with the employer Corporation to take any action against the
employee. The proceedings could have been taken for the recovery of financial loss
suffered by the Corporation due to negligence and carelessness attributable to the
respondent employee. The impugned action, therefore, cannot be said, to be illegal or
without jurisdiction and therefore, not liable to be quashed.
W. P. No. 484 (S/B) of 2000, D/-24-2-2006 (All.), Reversed. (Paras 25, 28, 31, 37)
(C) Constitution of India, Art.311, Art.226 - TERMINATION OF SERVICE - WRITS -
DEPARTMENTAL PROCEEDINGS - Departmental proceedings - No rule of universal
application can be laid down that if there is delay in initiation of proceedings for a
particular period, they must necessarily be quashed. (Para 27)
Cases Referred : Chronological Paras
2005 AIR SCW 2177 : AIR 2005 SC 3039 : 2005 Lab IC 1857 (Ref) 35, 36
2005 AIR SCW 5690 : AIR 2006 SC 207 : 2005 Lab IC 4332 : 2006 (1) AIR Kar R 186
(Disting, Pt B) 26, 27
2004 AIR SCW 2294 : AIR 2004 SC 2135 : 2004 Lab IC 1752 (Ref.) 24
1994 AIR SCW 936 : AIR 1994 SC 1484 (Ref.) 17
1991 AIR SCW 2276 : AIR 1991 SC 2010 : 1991 Lab IC 2045 (Ref.) 22, 23
AIR 1988 SC 842 : 1988 Lab IC 991 (Ref.) 21
AIR 1987 SC 943 : 1987 Lab IC 689 (Ref.) 19
AIR 1983 SC 130 : 1983 Lab IC 1 (Ref.) 20
AIR 1978 SC 1109 : 1978 Lab IC 824 (Ref.) 21
AIR 1973 SC 834 : 1973 Lab IC 440 (Ref.) 20
AIR 1971 SC 1409 : 1971 Lab IC 881 (Ref.) 20
AIR 1967 SC 1286 (Ref.) 14, 15
AIR 1962 SC 673 (Ref.) 12, 13
AIR 1960 SC 247 (Ref.) 16
AIR 1955 SC 425 (Ref.) 34
AIR 1952 SC 192 (Ref.) 33
Rakesh Uttamchandra Upadhyay and Ajay Kumar Rai, for Appellants; Ravi Prakash
Mehrotra and Garvesh Kabra, for Respondent.
Judgement
1. C. K. THAKKER, J. :-Leave granted.
2. The present appeal is filed against the judgment and order passed by the High Court of
Judicature at Allahabad (Lucknow Bench) on February 24, 2006 in Writ Petition No. 484
(S/B) of 2000.
3. Necessary facts giving rise to the appeal are that the respondent herein was serving
with the appellant- U.P. State Sugar Corporation Ltd. ("Corporation" for short) as
Resident Engineer at the Head Office of the Corporation at Lucknow. On January 13,
2000, a show cause notice was issued to him stating therein that a work was allotted to
M/s. Gupta and Co., Dehradoon for construction of residential houses in Saharanpur. The
Contractor had given two Fixed Deposit Receipts (FDRs) towards the security for the
work to be done. The details of FDRs were given in the notice. It was alleged that the
Corporation suffered loss of Rupees one lakh due to lack of precaution, irregularity, gross
negligence and carelessness by the respondent. The respondent was, therefore, called
upon to submit explanation within three days why disciplinary action should not be taken
against him. On January 15, 2000, the respondent submitted his reply denying the
allegations and contending that he had not committed any illegality and there was no
justification to ask for his explanation. The Corporation was not satisfied with the reply
filed by the respondent and decided to hold departmental inquiry against him. On January
31, 2000, therefore, show cause notice was issued to the respondent for the losses caused
to the Corporation due to negligence and carelessness on the part of the respondent. It
may be noted at this stage that the respondent retired on attaining the age of
superannuation (60 years) on the same day, i.e. January 31, 2000. According to the
respondent, since he retired from service on January 31, 2000, no proceedings could have
been initiated against him and issuance of show cause notice which was received by him
after office hours at 6.45 p.m. on January 31, 2000 was illegal as there was no
relationship of employer and employee between the Corporation and him. He, therefore,
filed a writ petition in the High Court of Allahabad at Lucknow Bench on April 11, 2000.
In the petition a prayer was made for quashing charge-sheet and departmental
@page-SC1237
proceedings. During the pendency of the petition, however, two orders came to be passed
against the respondent on March 24, 2001 and April 26, 2005. By the first order of March
24, 2001, an amount of Rupees one lakh was ordered to be recovered from the respondent
as the Corporation suffered loss of the said amount which was ordered to be adjusted
from the gratuity of the respondent. By the second order dated April 26, 2005, an amount
of Rs. 73,235-50ps which was ½ portion of the amount of Rs. 1,46,471.00ps was
directed to be recovered as loss had been caused to the Corporation due to negligence of
the respondent. The respondent sought amendment in the petition and challenged the
above two orders as well.
4. The High Court, by the impugned order, allowed the writ petition holding that the
order dated January 31, 2000 commencing disciplinary inquiry against the writ petitioner
was illegal as he had retired on that day. No proceedings, hence, could have been initiated
against him. Consequently, orders passed in 2001 and 2005 could not have been made
and they were liable to be quashed. The Corporation was directed to pay to the writ
petitioner all the benefits of gratuity, leave encashment and other dues payable to him
with interest @ 8% p.a. from the date of retirement till the date of actual payment. Being
aggrieved by the order passed by the High Court, the Corporation has approached this
Court.
5. On July 28, 2006, notice was issued by this Court. Since contempt proceedings were
initiated by the writ petitioner in the meantime, who succeeded before the High Court,
this Court stayed those proceedings. Counter-affidavit and affidavit-in-rejoinder were
filed thereafter and the matter was ordered to be placed for final hearing. That is how the
matter has been placed before us.
6. We have heard the learned counsel for the parties.
7. The learned counsel for the appellant-Corporation contended that the High Court was
wholly wrong in quashing departmental proceedings and consequential orders passed by
the authorities which were legal, valid and in consonance with law. It was submitted that
show cause notice was issued on January 13, 2000 well in time before the writ petitioner
retired asking for an explanation as to why proceedings should not be initiated against
him. It was further submitted that even charge-sheet was issued on January 31, 2000 and
it was within power of the Corporation to issue such charge-sheet and the High Court
ought not to have set aside the inquiry proceedings and consequential orders. It was urged
that it is settled law that relationship of employer and employee continues to remain so
long as all retiral benefits have not been paid to the employee. Since the amount of
gratuity, leave encashment and other pensionary benefits were yet to be paid to the
employee, the tie continued and proceedings initiated against the writ petitioner were in
accordance with law and should not have been interfered with. That apart, under the U.P.
State Sugar Corporation Ltd. General Service Rules, 1988 (hereinafter called "the
Rules"), such proceedings could have been initiated even after an employee has retired
since they related to the recovery of losses caused to the Corporation by the respondent-
employee. Since the present proceedings were for recovery of loss caused to the
Corporation, such an action could have been taken under the Rules and the High Court
was wrong in holding that the proceedings could not have been held. Finally, it was
submitted that it was the case of the Corporation that because of acts and omissions of the
respondent-employee, loss had been caused to the Corporation. When the amount of loss
was sought to be recovered from the employee, the High Court ought not to have
exercised discretionary and equitable jurisdiction under Article 226 of the Constitution
and on that count also, the impugned action deserves to be set aside. On all these
grounds, it was submitted that the impugned order of the High Court is liable to be set
aside and the writ petition filed by the writ-petitioner should be ordered to be dismissed
by allowing the appeal.
8. The learned counsel for the respondent, on the other hand, supported the order of the
High Court. He submitted that the date of issuance of show cause notice was totally
irrelevant. Charge-sheet was issued only on January 31, 2000 and a finding was recorded
by the High Court that it was received by the respondent after office hours of January 31,
2000. By the time, the tie was broken and there was no relationship of employer and
employee between the Corporation and the writ-petitioner. No departmental proceedings,
therefore, could have been initiated against the writ-petitioner
@page-SC1238
and they were liable to be quashed. When the proceedings were without jurisdiction,
orders passed in 2001 and 2005 which were consequential, were obviously without
power, authority or jurisdiction on the part of the Corporation in passing them. The High
Court was, therefore, fully justified in quashing those orders also. Since the order passed
by the High Court is legal, valid and proper, it calls for no interference by this Court and
the appeal deserves to be dismissed.
9. From the facts noted above, it is amply clear that two orders which were passed against
the respondent-employee related to recovery of certain amount from the respondent-
employee on the ground that there was carelessness, negligence or omission on his part in
the discharge of his duties which resulted in loss to the Corporation. By the order dated
March 24, 2001, an amount of Rupees one lakh which was financial loss suffered by the
Corporation was ordered to be adjusted against the gratuity of the employee. Likewise,
by order dated April 26, 2005, an amount of Rs. 73,235.50 p. [½ of Rs. 1,46,471.00]
was ordered to be adjusted against the amount of gratuity and encashment of earned leave
which was also the financial loss suffered by the Corporation as a result of negligence of
the respondent. It is in the light of the above facts that we have to consider whether such
an action could have been taken against the respondent-employee by the appellant-
Corporation.
10. The learned counsel for the appellant is right when he submitted that show cause
notice was issued to the respondent-employee on January 13, 2000 when he was very
much in service. The respondent submitted his explanation on January 15, 2000 which
was not found to be satisfactory. A regular show cause notice was, therefore, issued by
the Corporation on January 31, 2000 and was served upon the respondent-employee on
the same day. The notice was also sent by registered post which was received by the
employee on February 11, 2000. But it is clear from the documents that show cause
notice was issued and replied. A regular show cause notice as to departmental inquiry was
also served upon the respondent-employee on the last day of his service which was
January 31, 2000. In our opinion, therefore, it could not be said that the proceedings had
been initiated against the respondent-employee after he retired from service.
11. Now it is well settled that retiral benefits are earned by an employee for long and
meritorious services rendered by him/her. They are not paid to the employee gratuitously
or merely as a matter of boon. It is paid to him/her for his/her dedicated and devoted
work.
12. In Garment Cleaning Works, Bombay v. Workmen, AIR 1962 SC 673, the relevant
clause of the Gratuity Scheme provided that if a workman was dismissed or discharged
for misconduct causing financial loss to the employer, gratuity to the extent of loss should
not be paid to the workman concerned. It was contended on behalf of the employer that
the retrenchment benefit and gratuity were payable to the employee for his long and
meritorious services and if he was dismissed by misconduct, he would not be entitled to
claim retrenchment benefits or gratuity and the benefits could be denied to him.
13

. Dealing with the argument and the basis of payment of gratuity, this Court, speaking
through P.B. Gajendragadkar, J. (as His Lordship then was), said : AIR 1962 SC 673 at
p. 675

"5. On principle if gratuity is earned by an employee for long and meritorious service it is
difficult to understand why the benefit thus earned by long and meritorious service should
not be available to the employee even though at the end of such service he may have been
found guilty of misconduct which entails his dismissal. Gratuity is not paid to the
employee gratuitously or merely as a matter of boon. It is paid to him for the service
rendered by him to the employer, and when it is once earned it is difficult to understand
why it should necessarily be denied to him whatever may be the nature of misconduct for
his dismissal. Then, as to the definition of retrenchment in the Industrial Disputes Act, we
are not satisfied that gratuity and retrenchment compensation stand exactly on the same
footing in regard to the effect of misconduct on the rights of workmen. The rule of the
provident fund scheme shows not that the whole provident fund is denied to the employee
even if he is dismissed but it merely authorises certain deductions to be made and then
too the deductions thus made do not revert to the employer either. Therefore we do not
think that it would be possible to accede to the general argument that in all
@page-SC1239
cases where the service of an employee is terminated for misconduct gratuity should not
be paid to him. It appears that in award which framed gratuity schemes sometimes simple
misconduct is distinguished from gross misconduct and a penalty of forfeiture of gratuity
benefit is denied in the latter case but not in the former, but latterly industrial tribunals
appear generally to have adopted the rule which is contained in clause (ii)(b) of the
present scheme. If the misconduct for which the service of an employee is terminated has
caused financial loss to the works, then before gratuity could be paid to the employee he
is called upon to compensate the employer for the whole of the financial loss caused by
his misconduct, and after this compensation is paid to the employer if any balance from
the gratuity claimable by the employee remains that is paid to him..
(Emphasis supplied)
14

. In Calcutta Insurance Co. Ltd. v. Workmen, (1967) 2 SCR 596, this Court considered
the concept of gratuity. It referred to Garment Cleaning Works and other cases. It noted
that the opinion expressed in those cases was that gratuity was earned by an employee for
"long and meritorious service" and consequently it must be given to him even though at
the end of such service, he may have been found guilty of misconduct entailing his
dismissal. AIR 1967 SC 1286

15

. The Court then said; AIR 1967 SC 1286, Para 13

"In principle, it is difficult to concur in the above opinion. Gratuity cannot be put on the
same level as wages. We are inclined to think that it is paid to a workman to ensure good
conduct through out the period he serves the employer. "Long and meritorious service"
must mean long and unbroken period of service meritorious to the end. As the period of
service must be unbroken, so must the continuity of meritorious service be a condition for
entitling the workman to gratuity. If a workman commits such misconduct as causes
financial loss to his employer, the employer would under the general law have a right of
action against the employee for the loss caused and making a provision for withholding
payment of gratuity where such loss caused to the employer does not seem to aid to the
harmonious employment of labourers or workmen. Further, the misconduct may be such
as to undermine the discipline in the workers a case in which it would be extremely
difficult to assess the financial loss to the employer."
(Emphasis supplied)
16. In M. Narasimhachar v. State of Mysore, AIR 1960 SC 247, an amount of Rs.5,215/-
was deducted from pension of the Government servant. The action was challenged by the
employee. Considering the relevant provisions of the Rules, this Court held that the
Government had reserved to itself the right to order the recovery from pension and
compassionate allowances of the Government servant of any amount on account of losses
found to have been caused to Government by negligence or fraud of such officer during
his service.
17
. Again, in Jarnail Singh v. Secretary, Ministry of Home Affairs and Ors., (1993) 1 SCC
47 : JT 1992 Supp SC 489, this Court considered the provisions of Central Civil Services
(Pension) Rules, 1972. The definition of 'pension' included gratuity under Rule 3. Rule 9
conferred on the President power to withhold or withdraw pension in certain
circumstances. An order was passed against an employee withholding pension and the
entire amount of death-cum-retirement gratuity otherwise admissible to him. The
direction was given on account of serious irregularities found to have been committed by
the workman. The workman challenged that order unsuccessfully and thereafter
approached this Court. His contention was that an amount of gratuity could not have been
withheld. 1994 AIR SCW 936

18. Negativing the contention, this Court held that the power to withhold gratuity was
conferred on the President and such action could not be said to be illegal. It was ruled that
the Government could adjust its dues against the amount of death-cum-retirement
gratuity otherwise payable to Government servant.
19

. In State of Uttar Pradesh v. Brahm Datt Sharma and Ann, (1987) 2 SCC 179 : JT 1987
(1) SC 571, this Court held that it was open to Government to reduce, forfeit, withhold or
recover pension, after affording hearing to the affected person, on ground of
unsatisfactory service based on proved findings of serious misconduct or causing
pecuniary loss to the Government. Such proceedings can be initiated even after
retirement for AIR 1987 SC 943

@page-SC1240
misconduct, negligence or financial irregularity. Where Government servant was found
guilty of misconduct or negligence resulting in financial loss to the Government, it was
competent to the Government to direct reduction in pension.
20

. Interpreting Article 470 of U.P. Civil Service Regulations, this Court observed that the
said provision stated that full pension would not be awarded as a matter of course to a
Government servant on his retirement. It was awarded to him if service rendered by him
was satisfactory. In case of absence of 'thoroughly satisfactory' service, the authority was
competent to reduce the amount of pension. Referring to Deokinandan Prasad v. State of
Bihar, 1971 Supp SCR 634, State of Punjab v. K.R. Erry, (1973) 2 SCR 405 and D.S.
Nakara v. Union of India, (1983) 2 SCR 165, the Court held that pension was not a
'bounty' and an employee was entitled to pensionary benefits, but proceeded to state that a
Government employee would earn pension by rendering long and efficient service.
Considering Narasimhachar, the Court held that the employer had right to reduce pension
of an employee if services rendered by him were found to be unsatisfactory. Only thing is
that in such cases before taking any action, principles of natural justice must be observed.
AIR 1971 SC 1409
AIR 1973 SC 834
AIR 1983 SC 130
21

. In State of Maharashtra v. M.H. Mazumdar, (1988) 2 SCC 52 : JT 1988 (1) SC 432, the
Court held that departmental inquiry can be instituted against a Government servant after
superannuation and pension can be reduced on proved charges of misconduct, negligence
or financial irregularity committed during the period of service. Following
Narasimhachar and Brahm Datt Sharma, and distinguishing B.J. Shelat v. State of
Gujarat, (1978) 2 SCC 202, the Court held that when financial loss was caused to the
Government by any act or omission on the part of its employee, the purpose of inquiry
was not to inflict any punishment, but to determine the pension of an employee. Such an
action, in our view, can be taken so that the Government may not have to suffer
financially. AIR 1988 SC 842
AIR 1978 SC 1109

22

. Reference was also made to a leading decision in Union of India and Ors. v. K.V.
Jankiraman and Ors., (1991) 4 SCC 109 : JT 1991 (3) SC 527. In Jankiraman, the
question which came up for consideration before this Court related to promotion of an
officer and adoption of "sealed cover procedure". It was held that consideration of case of
an employee for promotion could not be withheld merely on the ground of pendency of
any departmental inquiry/criminal investigation against him. It could, however be
resorted to once charge memo/charge-sheet is issued. 1991 AIR SCW 2276

23

. It was submitted by the learned counsel for the Corporation that in the case on hand, not
only notice was issued to the respondent-employee on January 13, 2000, but even regular
show cause notice was issued on January 31, 2000 and hence the proceedings could have
been continued on the basis of law laid down in Jankiraman. 1991 AIR SCW 2276

24

. In UCO Bank and Ors. v. Sanwar Mal, (2004) 4 SCC 412 : JT 2004 Supp 2 SC 487, the
Court held that two concepts; (i) resignation; and (ii) retirement were different and
employed for different purposes and in different contexts. Resignation brings about
complete cessation of master and servant relationship, but retirement does not do so. In
case of retirement, master and servant relationship continues for grant of retiral benefits.
2004 AIR SCW 2294

25. If it is so, the appellant-Corporation, in our opinion, is right in submitting that the
proceedings could have been continued after the retirement of the respondent-employee
as far as the financial loss caused to the Corporation because of negligence on the part of
employee and the benefit claimed by the respondent-workman on his terminal benefits.
26
. Strong reliance was placed by the learned counsel for the respondent on P.V. Mahadevan
v. MD, T.N. Housing Board, (2005) 6 SCC 636 : JT 2005 (7) SC 417. In that case, there
was inordinate delay of ten years in initiating departmental proceedings against an
employee. In absence of convincing explanation by the employer for such inordinate
delay, this Court held that the proceedings were liable to be quashed. 2005 AIR
SCW 5690

27. In our opinion, Mahadevan does not help the respondent. No rigid, inflexible or
invariable test can be applied as to when
@page-SC1241
the proceedings should be allowed to be continued and when they should be ordered to be
dropped. In such cases there is neither lower limit nor upper limit. If on the facts and in
the circumstances of the case, the Court is satisfied that there was gross, inordinate and
unexplained delay in initiating departmental proceedings and continuation of such
proceedings would seriously prejudice the employee and would result in miscarriage of
justice, it may quash them. We may, however, hasten to add that it is an exception to the
general rule that once the proceedings are initiated, they must be taken to the logical end.
It, therefore, cannot be laid down as a proposition of law or a rule of universal application
that if there is delay in initiation of proceedings for a particular period, they must
necessarily be quashed.
28. In the present case, the High Court has not quashed the proceedings on the ground
that there was inordinate and unexplained delay on the part of the Corporation in
initiating such proceedings against the respondent. According to the High Court, since the
respondent retired on January 31, 2000, the proceedings could not have been continued
against him. From the case law referred to by us hereinabove, it is clear that such
proceedings could have been continued since they were initiated for the recovery of
losses sustained by the Corporation due to negligence on the part of the respondent-
employee. Such loss caused to the Corporation could be recovered from the respondent
from the retiral benefits of the respondent.
29. The learned counsel for the appellant-Corporation also referred to the Rules. Chapter
IV titles "Fundamental Duties of Service". Rule 31 expressly states that an employee of
the Corporation would be 'whole time employee'. Chapter VIII (Rules 93 to 107) deals
with 'Disciplinary Proceedings'. Rule 93 is material and relevant part thereof reads thus;
93. The following penalties may, for good and sufficient reason and as hereinafter
provides, be imposed on an employee.
A. MINOR PENALTIES
(i) Censure
(ii) With-holding of annual increment(s), including stoppage of an efficiency
bar/assessment stage with or without cumulative effect.
(iii) Recovery from pay or from such other amounts as may be due to the employee of the
whole or part of any pecuniary loss caused to the Corporation by negligence or breach of
orders on his part :
B. MAJOR PANALTIES
(iv) reduction to a lower grade or post or to a lower stage in a time scale;
(v) removal from service which does not disqualify from future employment,
(vi) dismissal from service which ordinarily disqualifies from future employment.
(Emphasis supplied)
30. Rule 102 prescribes procedure before starting enquiry. Rule 103 provides for major
penalties. Rule 109 lays down procedure for imposition of minor penalties and is another
important provision which may be quoted in extenso.
109. (1) Whenever the punishing authority is satisfied that good and sufficient reasons
exist for adopting such a course it may impose the penalty of -
(1) Censure, or
(ii) Stoppage at an efficiency bar.
Provided that it shall not be necessary to frame formal charges against the employee
concerned but his explanation may be called and considered before imposing such a
penalty.
(2) In all cases where the punishing authority imposes the penalty of -
(i) Withholding of increments in the time scale at stages where there is no efficient bar.
(ii) Recovery from pay of the whole or part of any pecuniary loss caused to the
Corporation by negligence or breach of orders.
Formal proceedings embodying statement of the offence or fault, the explanation of the
person concerned, and the reasons for punishment shall be recorded.
Provided that it shall not be necessary to record such proceedings in cases where an
employee's increment in the time scale of his pay at any stage other than an efficiency bar
is stopped due to his integrity remaining uncertified.
(Emphasis supplied)
31. It is, therefore, clear that so far as minor penalty is concerned, it is not necessary for
the Corporation to follow detailed and lengthy procedure laid down for imposition of
major penalties. In the instance
@page-SC1242
case, the proceedings had been initiated by the appellant-Corporation against the
respondent-employee for recovery of pecuniary loss caused to the Corporation by
negligence on his part. The proceedings, hence, could be instituted by issuing notice
which was done on January 13, 2000. The said action, therefore, could not have been held
bad or without power, authority or jurisdiction on the part of the Corporation. As we have
already observed earlier, even regular show cause notice was served on January 31, 2000
which was also during the employment of respondent. The High Court, in our view, was
wrong in quashing the proceedings and setting aside orders dated March 24, 2001 and
April 26, 2005. The impugned order of the High Court, therefore, deserves to be set
aside.
32. Finally, the learned counsel for the appellant-Corporation is right in submitting that
the High Court was exercising discretionary and equitable jurisdiction under Article 226
of the Constitution. It is well-settled that the jurisdiction of the High Court under Article
226 of the Constitution is equitable and discretionary. The power under that Article can
be exercised by the High Court "to reach injustice wherever it is found".
33

. In Veerappa Pillai v. Raman and Raman Ltd. and Ors., 1953 SCR 583, the Constitution
Bench of this Court speaking through Chandrasekhara Aiyar, J. observed that the writs
referred to in Article 226 of the Constitution are obviously intended to enable the High
Court to issue them in grave cases where the subordinate tribunals or bodies or officers
act wholly without jurisdiction, or in excess of it, or in violation of the principles of
natural justice, or refuse to exercise jurisdiction vested in them, or there is an error
apparent on the face of the record, and such act, omission, error, or excess has resulted in
manifest injustice. AIR 1952 SC 192

34

. Again, in leading case of Sangram Singh v. Election Tribunal, Kotah, (1955) 2 SCR 1,
dealing with the ambit and scope of powers of High Courts under Article 226 of the
Constitution, Bose, J. stated; AIR 1955 SC 425, Para 14

"That, however, is not to say that the jurisdiction will be exercised whenever there is an
error of law. The High Courts do not, and should not, act as courts of appeal under Article
226. Their powers are purely discretionary and though no limits can be placed upon that
discretion it must be exercised along recognized lines and not arbitrarily; and one of the
limitations imposed by the courts on themselves is that they will not exercise jurisdiction
in this class of case unless substantial injustice has ensued, or is likely to ensue. They will
not allow themselves to be turned into Courts of appeal or revision to set right mere
errors of law which do not occasion injustice in a broad and general sense, for, though no
legislature can impose limitations on these constitutional powers it is a sound exercise of
discretion to bear in mind the policy of the legislature to have disputes about these special
rights decided as speedily as may be. Therefore, writ petitions should not be lightly
entertained in this class of case."
(Emphasis supplied)
35

. Recently, in Secretary, ONGC Ltd. and Ann v. V.U. Warner, (2005) 5 SCC 245 : JT 2005
(4) SC 489, an employee of Oil and Natural Gas Commission (ONGC) unauthorisedly
retained an official accommodation after his retirement. When penal rent was charged
and sought to be recovered from retiral benefits of the employee, he filed a petition
invoking Article 226 of the Constitution. The High Court allowed the petition and
directed the Corporation to release all the benefits to which the employee was entitled.
The High Court observed that it was open to the Corporation to take appropriate
proceedings for recovery of the dues claimed by the Corporation. Aggrieved ONGC
approached this Court. 2005 AIR SCW 2177

36

. Allowing the appeal, setting aside the order passed by the High Court and considering
the relevant decisions on the point, one of us (C.K. Thakker, J.) observed; 2005 AIR
SCW 2177, Para 34

"As already adverted to by us hereinabove, the facts of the present, case did not deserve
interference by the High Court in exercise of equitable jurisdiction under Article 226 of
the Constitution. The respondent-petitioner before the High Court-, was a responsible
officer holding the post of Additional Director (Finance and Accounts). He was, thus,
gold collar employee of the Commission. In the capacity of employee of the
Commission, he was allotted a residential quarter. He reached the age of superannuation
and retired after office hours of February 28, 1990. He was, therefore, required
@page-SC1243
to vacate the quarter allotted to him by the Commission. The Commission, as per its
policy, granted four months time to vacate. He, however, failed to do so. His prayer for
continuing to occupy the quarter was duly considered and rejected on relevant and
germane grounds. The residential accommodation constructed by him by taking loan at
the concessional rate from the Commission was leased to Commission, but the possession
of that quarter was restored to him taking into account the fact that he had retired and
now he will have to vacate the quarter allotted to him by the Commission. In spite of that,
he continued to occupy the quarter ignoring the warning by the Commission that if he
would not vacate latest by June 30, 1990, penal rent would be charged from him. In our
judgment, considering all these facts, the High Court was wholly unjustified in exercising
extraordinary and equitable jurisdiction in favour of the petitioner respondent herein and
on that ground also, the order passed by the High Court deserves to be set aside".
(Emphasis supplied)
37. Considering the facts and circumstances in their entirety, in our considered opinion,
the High Court was wrong in holding that the proceedings were initiated after the
respondent retired and there was no power, authority or jurisdiction with the Corporation
to take any action against the writ-petitioner and in setting aside the orders passed against
him. In our judgment, proceedings could have been taken for the recovery of financial
loss suffered by the Corporation due to negligence and carelessness attributable to the
respondent-employee. The impugned action, therefore, cannot be said to be illegal or
without jurisdiction and the High Court was not right in quashing the proceedings as also
the orders issued by the Corporation. The appeal, therefore, deserves to be allowed by
setting aside the order of the High Court.
38. For the foregoing reasons, the appeal is allowed and the order passed by the High
Court is set aside. But since the High Court has allowed the petition only on the ground
that the proceedings could not have been instituted against the writ-petitioner, it would be
appropriate if we remit the matter to the High Court so as to enable it to consider the rival
contentions of the parties and take an appropriate decision on merits. We may clarify that
we may not be understood to have expressed any opinion one way or the other on the
controversy involved in the case and as and when the High Court will take up the writ
petition, it will decide the same without being influenced by any observation made in this
judgment. On the facts and in the circumstances of the case, the parties will bear their
own costs.
Appeal allowed.
AIR 2008 SUPREME COURT 1243 "Balwant Singh v. State of Punjab"
(From : 2005 (3) All Cri. LR 794)
(Punj. and Har.)
Coram : 2 PRAKASH PRABHAKAR NAOLEKAR AND MARKANDEY KATJU, JJ.
Criminal Appeal No. 621 of 2006, D/- 6 -2 -2008.
Balwant Singh and Ors. v. State of Punjab.
Penal Code (45 of 1860), S.300 - MURDER - EVIDENCE - Murder - Circumstantial
evidence - Accused armed with deadly weapons present near house of deceased - Mother
of deceased when she returned home saw accused giving gandasa blow to deceased who
had fallen down after receiving fatal injuries - All accused gave thrust and kick blows to
deceased who was already injured - No one else was present on spot except accused -
Circumstances contain all links in chain connecting accused - Accused liable to be
convicted for murder - Evidence of mother corroborated by medical evidence and one
other witness - Fact that mother of deceased had not witnessed fatal injuries that were
caused, immaterial. (Para 11)
Penal Code (45 of 1860), S.34, S.300 - COMMON INTENTION - MURDER - Murder -
Common intention - Accused going together armed with deadly weapons causing fatal
injuries - All accused liable to be convicted - Absence of evidence as to part played by
each accused - Inconsequential. (Para 13)

K.B. Sinha, Sr. Advocate, Mrs. Kawaljit Kochar, Ms. Sweta Rani, Ms. Kusum
Chaudhary, for Appellants; Kuldip Singh, R K. Pandey, for Respondent.
Judgement
1. MARKANDEY KATJU, J.:-This appeal has been filed against the impugned judgment
of the Punjab and Haryana High Court dated 4-5-2005 in Criminal Appeal No. 636-DB of
2002.
@page-SC1244
2. Heard learned counsel for the parties and perused the record.
3. The appellants are one Balwant Singh and his three sons Balwinder Singh, Harbans
Singh and Malkiat Singh. They were convicted under Section 302 and other provisions of
the Indian Penal Code by the Additional Sessions Judge, Bathinda on 30-7-2002. Against
that judgment they filed an appeal in the High Court which was dismissed and hence this
appeal.
4. The prosecution case is that on 17-7-1998 at about 10-10.30 a. m. Bharpur Kaur (PW1)
wife of Jarnail Singh r/o village Khokhar, Police Station Kalawali had along with her
sons Vakil Singh, Gurjant Singh and Nachhattar Singh gone to village Gill Patti, Bathinda
to meet her sister's daughter Amarjit Kaur (PW-3), who was at that time residing in street
No. 6, Janta Nagar, Bathinda. From there along with Amarjit Kaur and her son Kuldeep
Singh alias Gurtej Singh they had gone to the house of Amarjit Kaur in village Gill Patti.
When they alighted from the jeep in front of the house of Amarjit Kaur, they found the
appellant Balwant Singh and his sons Harbans Singh, Malkiat Singh and Balwinder
Singh armed with kirpans and gandasa sitting in the Deodi of the house of Natha Singh
son of Wazir Singh. Amarjit Kaur along with Gurjant Singh and Nachhattar Singh left the
house to bring milk and vegetable while Bharpur Kaur PW1, deceased Kuldeep Singh
and Vakil Singh remained in the house. In the mean time, Balwant Singh and his son
Harbans Singh armed with swords and Malkiat Singh and Balwinder Singh armed with
gandasas and Natha Singh empty handed trespassed into Amrit Kaur's house. Natha
Singh allegedly raised a Lalkara "Inna Noo Jaggar Singh Noo Katai Karan Atte Ghar To
Kabza Karan Da Maja Chakhaounde Haan. Ajj Ahe Bach Ke Na Jaan". Thereupon,
Kuldeep Singh and Vakil Singh sons of Bharpur Kaur tried to go out of the house in order
to save themselves but Harbans Singh gave a blow with the kirpan on the left side of the
forehead and on the head of Kuldeep Singh, as a result of which he fell down on the
ground. Thereafter, Balwant Singh gave a kirpan blow which resulted in infliction of
injury on his right ear. Malkiat Singh gave a gandasa blow from its reverse side on the
right cheek and ear of Kuldeep Singh. When Bharpur Kaur PW1 intervened to save
Kuldeep Singh, Balwinder Singh and Malkiat Singh gave gandasa blow from reverse
side, which resulted in injuries on her left wrist joint, left kneejoint and nose. Vakil Singh
raised a shout "Na Maro Na Maro" whereupon Malkiat Singh and Balwinder Singh gave
gandasa blows on the preson of Kuldeep Singh on his right and left thigh and below the
knee joint when he was lying on the ground. In the meantime, Amarjit Kaur, mother of
the deceased Kuldeep Singh, Gurjant Singh and Nachhattar Singh returned to the scene of
occurrence and they also raised an alarm on hearing which the assailants fled away from
the spot taking their respective weapons with them. Kuldeep Singh succumbed to the
injuries.
5. Bharpur Kaur PW1 proceeded for the Police Station and on the way met SI Chand
Singh SHO of Police Station Sadar, Bathinda, who was present at Aeroplane Chowk,
Bathinda where he had set up a Naka for special checking of vehicles and got reduced her
statement into writing. On the basis of the statement, which was completed at 12.30 p.m.
on 17-7-1998, a formal FIR Ex PA/2 was recorded in Police Station Thermal Bathinda at
12.45 p.m. on the same day. He himself proceeded to the spot where he prepared inquest
report and during the investigation took into possession blood staind earth vide recovery
memo Ex PR, recorded the statements of the witnesses and forwarded the dead body to
Civil Hospital, Bathinda through Constable Joginder Singh along with request for Post
Mortem Examination ex. PE. On 18-7-1998 he took into possession clothes of the
deceased, which were handed over to him by Constable Joginder Singh.
6. On 21-7-1998 he arrested Balwant Singh and Malkiat Singh, who made disclosure
statements pursuant whereof the Kirpan Ex. P12 and gandasa Ex. P13 were taken into
possession through recovery memo Ex. PU/2 and Ex. PU/3. Harbans Singh and
Balwinder Singh were apprehended on 22-7-1998 and they too made disclosure
statements Ex. PU/4 and Ex. PU/ 5 respectively pursuant whereof kirpan Ex. P/14 and
gandasa Ex. P/15 respectively were taken into possession vide recovery memo Ex. P/6
and Ex. PU/7 respectively. Balwant Singh and Malkiat Singh, appellants made separate
disclosure statements Ex. PU/16 and Ex PU/17 pursuant whereof their shirts Ex. P/16 and
Ex. P/17 which were blood stained were taken into possession
@page-SC1245
through recovery memos Ex PU/18 and Ex. PU/19 respectively. The same were
forwarded to the Chemical Examiner for examination and on receipt of the report of
Chemical Examiner, the challan was put in Court against the appellants before the Illaqa
Magistrate, who upon finding that the offences disclosed were exclusively triable by the
Courts of Sessions, committed the same to the Court of Sessions for trial.
7. The incident in question occurred on 17-7-1998 at 10.30 a.m. and the FIR was filed at
12.45 p.m. Hence there was no delay in filing the FIR.
8. Learned counsel for the appellants submitted that PW1 Bharpur Kaur who lodged the
FIR turned hostile as is evident from her statement in Court on 28-2-2000. PW2 Gurjant
Singh also turned hostile. Hence we are left only with the testimony of PW3 Amarjit
Kaur and PW8 Sukhjit Kaur. We have, therefore, to see whether conviction on their
testimonies would be safe.
9. We have carefully gone through the testimony of these two witnesses and we see no
reason to disbelieve the same, especially since they broadly corroborate each other.
Amarjit Kaur stated in her evidence that Bharpur Kaur and her son came to her house in
Janta Nagar, Bathinda in a rented jeep and there she and her son Kuldeep Singh
(deceased) accompanied them in the jeep and they went to village Gill Patti, where also
Amarjit Kaur has a house. The jeep was parked near the house of Amarjit Kaur where
they saw all the accused persons sitting in the Deodi of Natgha Singh. Balwant Singh and
Harbans Singh accused were armed with kirpan and Balwinder Singh and Malkiat Singh
were armed with gandasa. Amarjit Kaur, Gurjant Singh and Nachattar Singh went out to
purchase vegetables while Kuldeep Singh, Bharpur Kaur and her son Vakil Singh
remained present in the house of Amarjit Kaur. When Amarjit Kaur and two others
returned to the house at 10 or 10.30 a.m. they saw her son Kuldeep Singh lying on the
ground. In their view Balwinder Singh gave gandasa blow on the leg of deceased
Kuldeep Singh and Malkiat Singh gave a gandasa blow from its blunt side in his
abdomen. All the accused gave thrust blows as well as kick blows to her son Kuldeep
Singh. The motive behind the occurrence was said to be the desire to usurp the property
of Amarjit Kaur by killing her son.
10. Learned counsel for the appellant submitted that from the evidence of Amarjit Kaur it
appears that she only saw Balwinder Singh giving a gandasa blow on the leg of Kuldeep
Singh and Malkiat Singh giving a gandasa blow from the blunt side in his abdomen.
From this evidence learned counsel for the appellant has tried to infer that Amarjit Kaur
did not see the main incident in which the fatal injuries were caused. These fatal injuries
are mentioned in the evidence of the doctor who conducted the post mortem. The doctor
mentioned the following injuries :
1. Lacerated wound present on frontal region of head 10x5 cm underlying bone was
fractured. Brain matter exposed. On dissection multiple fractures were present pieces of
the bone were present in the brain matter. Haemotoma was present on frontal region of
head.
2. Fracture of mandible evident.
3. Incised wound 4 m x 3 cm on right ear and back of head. Loss of part of right ear. On
dissection underlying bone fractured. Haemotoma was present.
4. Incise wound 4 cm x 2 cm on frontal region of lower leg on right side. Underlying
bone fractured. Clotted blood present. Multiple abrasions on right leg.
5. Multiple abrasion on right arm.
6. Bruise 8 cm x 4 cm lower abdomen below umbilicus.
7. Bruise 4 cm x 3 cm on left wrist joint.
11. Thus it appears that there were injuries on the head of Kuldeep Singh. It may be that
Amarjit Kaur was not present when these injuries were caused to Kuldeep Singh, but in
our opinion there is strong circumstantial evidence that they were caused by the accused.
In our opinion this circumstantial evidence is sufficient to uphold the conviction because
it contains all the links in the chain which connect the accused with the incident. These
links are :
(a) The accused persons were present with deadly weapons near the house of Amarjit
Kaur. When persons come armed with deadly weapons to someone's house, it is a strong
circumstance to indicate that they had come with deadly intentions.
(b) When Amarjit Kaur and others returned to the house at 10 or 10.30 a.m., they saw
Balwinder Singh giving gandasa blow
@page-SC1246
to her son on the right leg and Malkiat Singh giving gandasa blow from the blunt side of
gandasa in his abdomen. These blows were evidently given after the fatal injuries on the
head of Kuldeep Singh.
(c) All the accused thereafter gave thrust blows as well as kick blows to Kuldeep Singh
who was lying on the ground. Hitting and kicking an injured man who is already lying on
the ground shows the deadly intent of the accused.
(d) There was no one else other than the accused who is said to have been on the spot at
the time of the incident.
12. The evidence of Amarjit Kaurh as been corroborated by the evidence of the doctor
and PW8 and we see no reason to disbelieve their evidence also. PW8 Sukhjit Kaur has
stated in her evidence that she saw the accused armed with kirpans and gandasas going
towards the house of her father Jaggar Singh (deceased husband of Amarjit). From this an
inference can be drawn that the accused armed with weapons were going with deadly
intentions towards the house. Sukhjit Kaur PW8 also stated that when they came back
after 30 to 45 minutes they were armed with the same weapons with blood stains and the
clothes also had blood stains which they later changed and ran away. This evidence of
Sukhjit Kaur corroborates the evidence of Amarjit Kaur, though it is true that neither saw
the main incident in which the fatal injuries were caused. However, as already stated
above, there is strong circumstantial evidence to connect the accused with the crime.
13. Learned counsel for the appellant then submitted that it is not clear which of the
accused caused which particular injury. In our opinion that will not matter because
Section 34 IPC is clearly attracted to the facts of the case. When persons go together
armed with deadly weapons and fatal injuires are caused to the deceased, all of them are
equally liable in view of Section 34 IPC.
14. In view of the above, there is no merit in this appeal. The appeal is accordingly
dismissed.
Appeal dismissed.
AIR 2008 SUPREME COURT 1246 "Ramanlal Bhailal Patel v. State of Gujarat"
(From: 2003 AIHC 2552)
Coram : 2 R. V. RAVEENDRAN AND LOKESHWAR SINGH PANTA, JJ.
Civil Appeal No. 4420 of 2004, D/- 5 -2 -2008.
Ramanlal Bhailal Patel and Ors. v. State of Gujarat.
(A) Gujarat Agricultural Lands Celling Act (27 of 1961), S.2(21), S.6 - Bombay General
Clauses Act (1 of 1904), S.3(35) - AGRICULTURAL LAND - LAND CEILING -
GENERAL CLAUSES - INTERPRETATION OF STATUTES - 'Person' for purpose of
land celling - Includes Corporation, association of persons or body of individuals as
included in "person" by General Clauses Act - Not limited to natural person or joint
totally only.
Interpretation of Statutes- Definition Clause - Use of words includes.
The word 'person' in the Celling Act includes natural human being, company or an
association of persons/body of individuals, and a H. U. F. or any other group of persons,
the members of which are joint in residence and estate by custom or usage.(Para 17)
The word 'person' is defined in the Act, but it is an inclusive definition, that is "a person
includes a joint family."Where the definition is an inclusive definition, the use of the
word 'includes' indicates an intention to enlarge the meaning of the word used in the
Statute. Consequently, the word must be construed as comprehending not only such
things which they signify according to their natural import, but also those things which
the interpretation clause declares that they shall include. The ordinary, popular and
natural meaning of the word 'person' is 'a specific individual human being'. But in law the
word 'person' has a slightly different connotation, and refers to any entity that is
recognized by law as having the rights and duties of a human being. The General Clauses
Act defines a 'person' as including a corporation or an association of persons or a body of
individuals whether incorporated or not. The said general legal definition is, however,
either modified or restricted or expanded in different statutes with reference to the object
of the enactment or the context in which it is used. Both definitions of the word 'person',
in General Clauses Act and Ceiling Act, are inclusive definitions. The inclusive definition
of 'person' in General Clauses Act applies to all Gujarat Act unless
@page-SC1247
there is anything repugnant in the subject or the context. The inclusive definition of
'person' in Section 2(21) of the Celling Act, does not indicate anything repugnant to the
definition of 'person' in General Clauses Act but merely adds 'joint family' as defined.
(Paras 16, 17, 18)
(B) Bombay General Clauses Act (1 of 1904), S.3(35) - GENERAL CLAUSES -
WORDS AND PHRASES - Person - Association of persons - Constitution - Essentials -
Coming together of persons must be by their volition and to achieve some common
benefit - Co-owners do not always constitute association of persons.
Words and Phrases - Association of persons.
A mere combination of persons or coming together of persons without anything more,
without any intention to have a joint venture or carry on some common activity with a
common understanding and purpose will not convert two or more persons into a body of
individuals /association of persons. An 'association of persons/body of individuals' is one
in which two or more persons join in a common purpose and common action to achieve
some common benefit. Where there is a combination of individuals by volition of the
parties, engaged together in some joint enterprise or venture, it is known as 'association
of persons/body of individuals'. The common object will have some relevance to
determine whether a group or set of persons is an association of persons or body of
individuals with reference to a particular statute. Normally, where a group of persons
have not become co-owners by their volition with a common purpose, they cannot be
considered as a 'person'. When the children of the owner of a property succeed to his
property by testamentary succession or inherit by operation of law, they become co-
owners, but the co-ownership is not by volition of parties nor do they have any common
purpose. Each can act in regard to his/her share, on his/ her own, without any right or
obligation towards the other owners. The legal heirs though co-owners, do not
automatically become an 'association of persons/body of individuals'. (Paras 20, 21)
Mere purchase under a common deed without anything more, will not convert a co-
ownership into a joint enterprise. Thus, when there are ten co-owners of a property, they
are ten persons and not a 'body of individuals' to be treated as a 'single person'. But, if the
co-owners proceed further and enter into an arrangement or agreement to have a joint
enterprise or venture to produce a common result for their benefit, then the co-owners
may answer the definition of a 'person'. (Para 21)
(C) Gujarat Agricultural Lands Celling Act (27 of 1961), S.2(21), S.6, S.8 -
AGRICULTURAL LAND - 'Person' -Appellants purchasing land in joint names - Such
mode was adopted only to facilitate negotiations - Not for cultivating land jointly or to
have a joint venture - After purchase appellants divided lands and informed land revenue
authorities and each co-owner was registered as owner of the respective land - Appellants
cannot be treated as an 'association of persons/body of individuals' and therefore, a single
'person' - Fact that appellants could be said to have partitioned property in anticipation of
Act nor had applied u/S.8(2) - Is of no consequence.
2003 AIHC 2552 (Guj.), Reversed. (Paras 23, 24, 25, 26)
Cases Referred : Chronological Paras
1997 AIR SCW 1746 : AIR 1997 SC 1973 : 1997 Tax LR 566 (Rel. on, Pnt. B) 20
AIR 1979 SC 734 (Rel. on) 15
AIR 1976 SC 2316 (Rel. on) 18
AIR 1976 SC 2335 (Ref) 19
AIR 1970 SC 1707 (Rel. on, Pnt. B) 20
AIR 1961 SC 1043 (Rel. on, Pnt. B) 20
AIR 1960 SC 1172 (Rel. on, Pnt. B) 20
(1949)1 KB 142 : (1948) 2 All ER 610 15
Ravindra Shrivastava, Sr. Advocate, Ajay Kumar Jha, Mrs. Shakun Tiwari, Ms. Ranjeeta
Rohtagi, Kunal Verma, Arjun Garg, M. Mannan and Rajul Shrivastava (for M/s. P.H.
Parekh and Co.), with him for Appellants; R.P. Bhat, Sr. Advocate, Ms. Hemantika Wahi
and Ms. Pinky Behera, with him for Respondent.
Judgement
R. V. RAVEBNDRAN, J. :- This appeal by special leave, against the judgment dated 4-3-
2003 passed by the Gujarat High Court in L.P.A. No. 123 of 2000, relates to the
interpretation of the word 'person' in the Gujarat Agricultural Lands Ceiling Act, 1960
('Ceiling Act' for short).
@page-SC1248
The Facts :
2. The five appellants along with their respective spouses purchased 172 acres and 36
guntas of agricultural land in Madheli village under four sale deeds dated 14-11-1970
(128A, 26G), 29-4-1971 (26A, 37G), 23-6-1971 (10A, 14G) and 18-12-1971 (6A, 39G)
for a total consideration of Rs. 46,300/-. Before such purchase, the ten purchasers entered
into an agreement dated 15-9-1969 proposing to purchase about 175 acres of land in their
joint names and recording the conditions subject to which they proposed to purchase such
land. The reason stated in the agreement for the joint purchase, was to avoid each of them
having to individually negotiate and enter into separate agreements with several owners
of the lands. They agreed that one of them would be authorized to hold negotiations and
go through the procedural requirements for the purchase; and that after purchasing the
land in their names, they would divide the lands equally as early as possible. They also
agreed that there will be no collective or joint cultivation of the lands proposed to be
purchased by them.
3. After the purchase of the lands, the ten purchasers entered into another agreement on
30-12-1971 recording that the total sale price of Rs. 46,300/- was paid equally by all of
them. The said agreement also recorded the division of 172A, 36G of land purchased by
them into ten portions among them. Under the said agreement, they also agreed to get
their names registered in the Land Revenue Records as per the division. Subsequently, on
their applications, Mutation Entry No. 1371 dated 25-2-1976 (duly verified and approved
on 19-4-1976) was made showing each co-owner as the separate owner of the lands
respectively allotted to him/her, vide Hak Patrak (Village Form No. 6), issued by the
Land Revenue authorities.
The Proceedings :
4. The Mamlatdar issued a notice dated 5-11-1976 under Section 20 of the Celling Act to
the co-owners seeking particulars of the lands held by them, to determine whether they
held any surplus land. After considering their statements, he passed an order dated 30-4-
1983. He held that the total land held by them was 182 A. 23 Guntas; that when the lands
were converted into 'C' Class category, the extent was 181A, 31G; that each couple
(husband and wife) were together entitled to one unit (36 acres of 'C' category land); and
that therefore the five couples (the five appellants with their spouses) were entitled to
hold five units, that is, 180 acres. Consequently, he made a declaration under Section 21
of the Ceiling Act, that the surplus holding was 1 acre, 31 guntas and that such surplus
land (in Sy. No.643/43 in Madheli) shall vest in the Government.
5. The Deputy Collector, Dabhoi, in exercise of suo motu power of revision under section
37 of the Ceiling Act, issued a show cause notice dated 3-5-1984 to the appellants, being
of the view that the determination of surplus land by the Mamlatdar was contrary to the
provisions of the Ceiling Act. After hearing, he passed an order dated 23-8-1984 holding
that a group of persons or association of persons purchasing agricultural land together,
had to be treated as 'a person' under the Act and therefore they could jointly hold only one
unit (36 Acres) in view of the provision of section 6(1) of the Ceiling Act. As a
consequence, he determined the surplus land as 145A, 31G and directed the Mamlatdar to
obtain selection of the land to be surrendered. The said order was challenged by the
appellants before the Gujarat Revenue Tribunal, by invoking its revisional jurisdiction.
The Tribunal, by its judgment dated 29-12-1987, upheld the decision of the Deputy
Collector, by applying the definition of 'person' in the Bombay General Clauses Act,
1904, to the word 'person' in the Ceiling Act.
6. The appellants challenged the order of the Tribunal before the Gujarat High Court. A
learned Single Judge by order dated 30-12-1999 allowed the petition and remanded the
matter to the Revenue Tribunal to decide whether the definition of person' in the Bombay
General Clauses Act, 1904 ('General Clauses Act' for short) could be imported into the
definition of a 'person' under the Ceiling Act. The appellants challenged the order of the
learned Single Judge in appeal. A Division Bench of the Gujarat High Court allowed the
appeal in part, by judgment dated 4-3-2003. It held that there was no need for remand, as
the question whether the definition of 'person' included an association of persons, was a
pure question of law which can be decided by the High Court itself. It held that definition
of 'person' in the General Clauses Act has to be read into the definition of 'person' in the
@page-SC1249
Ceiling Act and therefore, the decision of the Revenue Tribunal treating the ten co-
owners as an association of persons, and consequently, a 'person' for the purpose of the
Ceiling Act, entitling them to hold only one unit (36 acres), did not suffer from any
infirmity. The order of the Tribunal was thus restored. The said order of the Division
Bench of the High Court is challenged in this appeal by special leave.
The Contentions :
7. The appellants contend that the definition of 'person' in General Clauses Act cannot be
read into the definition of 'person' in the Ceiling Act. They submitted that the general
definition in the General Clauses Act would apply only in the absence of a specific
definition in the concerned enactment; that as the Ceiling Act itself defined the word
'person', there was an legislative intention to exclude the general definition of the word
'person' in the GC Act; and that consequently the definition of 'person' in section 2(21)
should be interpreted as referring only to a natural person (a human being) or a joint
family as defined in the Ceiling Act; and that therefore, an association of persons or body
of individuals will not be a 'person' for the purpose of Ceiling Act.
7.1 Alternatively, it is submitted that even if the word 'person' is held to include an
association of persons or body of individuals, a co-ownership cannot be considered as a
body of individuals or association of persons and each co-owner should be considered as
a person for the purposes of the Ceiling Act.
7.2 The appellants contended that section 6(2) of the Ceiling Act provided that while
calculating the holding of an individual who is a member of a family, the land held by the
spouse (as also minor sons and minor unmarried daughters) has to be clubbed with the
individual's holding; and therefore, in this case, the holding of wife of each appellant
should be added to his holding for the purpose of considering whether he held any
surplus land. In other words, each couple (each of the five appellants with his wife) was a
'person' entitled to hold one unit (36 acres); and therefore, the decision of the Mamlatdar
that the appellants were entitled to hold five units (180 acres) and only the land in excess
of five units was surplus land was in accordance with law.
8. On the other hand, the respondent State contended that the definition of 'person' in the
GC Act will have to be read into the definition of 'person' in section 2(21) of the Ceiling
Act and therefore any 'association of persons' or 'body of individuals' will have to be
treated as a person. It is submitted that when the five appellants and (heir respective
spouses joined together to purchase 172 acres 36 guntas of land, they constituted an
'association of persons' or 'body of individuals' and therefore, the purchase by the ten
purchasers was a purchase by a 'person' and the ten co-owners as a 'person' were entitled
to hold only one unit (36 acres of 'C' class land).
8.1 The respondent's next contention is based on section 8 of Ceiling Act which provides
that any transfer or partition effected between 24-1-1971 and the date on which the
Gujarat Agricultural Land Ceiling (Amendment) Act, 1972 came into force shall be
deemed to have been made in anticipation in order to defeat the object of the said
Amending Act unless it is proved to the contrary. It is contended that the partition
effected among the ten co-owners under the unregistered agreement dated 30-12-1971
should be deemed to have been effected with the intention of defeating the object of
Gujarat Agricultural Lands Ceiling (Amendment) Act, 1972, (Amending Act for short)
having regard to the provisions of section 8 of the Ceiling Act. It is pointed out that the
only way to avoid such a presumption was to make an application to the Collector under
sub-section (2) of section 8 in the prescribed form within the prescribed period seeking a
declaration that such partition was not made in anticipation in order to defeat the object
of the Amending Act, 1972. It is contended that as such an application was not made to
the Collector, the partition will have to be deemed as having been made to defeat the
object of the Act and consequently the partition shall have to be ignored in computing the
surplus land under the Act.
8.2 Lastly it is submitted by the respondents that the joint purchase by ten persons was a
ruse to circumvent the bar contained in section 63 of the Bombay Tenancy and
Agricultural Lands Act, 1948 (Tenancy Act' for short) which prohibits sale of agricultural
land in favour of non-agriculturists. It is alleged that most of the ten purchasers were non-
agriculturists who could not have purchased agricultural land, and
@page-SC1250
they have attempted to circumvent the bar contained in Tenancy Act, by nominally
joining with some agriculturists and buying land as co-owners and thereafter effecting a
partition and claiming exclusive ownership. It is contended that if each co-owner is a
distinct 'person', then sale in favour of non-agriculturists will not be valid, even if the
purchase was jointly in the names of agriculturists and non-agriculturist and
consequently, the lands to the extent purchased by the non-agriculturists will vest in the
State Government.
9. The appellants replied that even if the partition was ignored under section 8 of the
Ceiling Act, it would not affect the calculation of surplus land, as each co-owner was a
'person' and each family (husband and wife) will be entitled to one unit. In regard to
contention based on Section 63 of the Tenancy Act, it was submitted that while
determining the surplus land under the provisions of the Ceiling Act, there was no
question of holding any enquiry under section 63 of Tenancy Act. And at all events, even
if the question as to whether the purchasers were agriculturists or not, has to be gone into,
the same being a question of fact that will have to be decided by a separate inquiry under
the Tenancy Act and not in the proceedings under the Ceiling Act.
10. On the contentions raised, the following questions arise for our consideration :
(i) Whether the definition of 'person' in the Gujarat Agricultural Lands Ceiling Act, 1960,
includes a body of individuals/association of persons ?
(ii) Whether co-ownership, per se, is an 'association of persons/body of individuals' and
therefore, constitutes a 'person'?
(iii) Whether the ten purchasers, who became co-owners of the land, together constitute a
'body of individuals/association of persons' and therefore a 'person' within the meaning of
that expression in the Ceiling Act?
(iv) Whether the partition dated 30-12-1971 among the co-owners is 'deemed to have
been made in anticipation to defeat the object of Gujarat Agricultural Lands Ceiling
(Amendment) Act, 1972' under section 8(1) of the Ceiling Act; and if so what is the effect
of failure to make an application under sub-section (2) of section 8 of the Ceiling Act.
(v) What would be the position if some of the co-owners were non- agriculturists at the
time of purchase of the lands? Whether the Mamlatdar can examine this issue when
considering the question of surplus land under the Ceiling Act?
Statutory Provisions :
11. The Ceiling Act was enacted to fix a ceiling on holding of agricultural lands and to
provide for the acquisition and disposal of surplus agricultural land. Section 4 relates to
delimitation of local areas and provides that there shall be different classes of local areas
in the State as specified in Schedule I and the local areas falling in each such class shall
be as respectively specified in Schedule II. Section 5 deals with ceiling areas. Sub-section
(1) thereof provides that subject to the provisions of sub-sections (2) and (3), in relation
to each class of local area as specified in Schedule I, the ceiling area with reference to
each class of land shall be as specified in the said Schedule against the respective class of
local area. Madheli village, Waghodia Taluk (Baroda District) is specified as class C area
under Schedule II. It is not in dispute that the lands in question were found to be dry crop
land, and therefore the unit ceiling area was 36 Acres under Schedule I to the Ceiling Act.
11.1 Section 6 deals with ceiling on holding of land. Sub-sections (1), (2), 3(B), 3(C), and
3(D) which are relevant are extracted below :
"(1) Notwithstanding anything contained in any law for the time being in force or in any
agreement, usage or decree or order of a Court, with effect from the appointed day, no
person shall, subject to the provisions of sub-sections (2), (3), (3B) and (4) be entitled to
hold whether as owner or tenant or partly as owner and partly as tenant land in excess of
the ceiling area.
(2) Where an individual, who holds land, is a member of a family, not being a joint
family which consists of the individual and his spouse (or more than one spouse) and
their minor sons and minor unmarried daughters, irrespective of whether the family also
includes any major son, land is also separately held by such individual's spouse or minor
children, then the lands held by the individual and the said members of the individual's
family excluding the major sons, if any, shall be grouped together for the purposes of the
Act and the provisions of the Act shall apply to the total land so grouped
@page-SC1251
together as if such land had been held by one person.
xxxxxxx
(3B) Where a family or a joint family consist of more than five members comprising a
person and other members belonging to all or any of the following categories, namely :
(i) minor son,
(ii) widow of a. pre-deceased son,
(iii) minor son or unmarried daughter of a pre-deceased son, where his or her mother is
dead,
such family shall be entitled to hold land in excess of the ceiling area to the extent of one
fifth of the ceiling area for each member in excess of five, so however that the total
holding of the family does not exceed twice the ceiling area; and in such a case, in
relation to the holding of such family, such area shall be deemed to be the ceiling area:
xxxxx
(3C) Where a family or a joint family irrespective of the number of members includes a
major son, then such major son shall be deemed to be a separate person for the purposes
of sub-section (1).
xxxxx
11.2 Section 8 deals with transfers and partitions effected to defeat the objects of the
Ceiling Act. Sub-section (1) provides that where after 24-1-1971 but before the stipulated
date (the date on which the Gujarat Agricultural Lands Ceiling (Amendment) Act, 1972 -
Gujarat Act 2 of 1974) came into force), any person has transferred or partitioned any
land held by him, then notwithstanding anything contained in any law for the time being
in force, such transfer or partition, shall, unless it is proved to the contrary, be deemed to
have been made in anticipation in order to defeat the object of the Amending Act of 1972.
Sub-sections (2) and (4) of section 8 read as under :
"(2) Any person effected by the provisions of sub-section (1) may, within the prescribed
period and in the prescribed form, make an application to the Collector for a declaration
that the transfer or partition was not made in anticipation in order to defeat the object of
this Act, or as the case may be, of the Amending Act of 1972.
xxxxx
(4) Where the application is rejected, the transfer or, as the case may be, the partition
shall be ignored in computing under this Act the area of surplus land, if any, held by such
person."
11.3 Section 2 contains the definitions. Clauses (16) and (21) which are relevant are
extracted below :
"2. Definitions - In this Act, unless the context requires otherwise - x x x x x (16) "Joint
family" means a undivided Hindu family and in the case of other persons a group or unit
the members of which by custom or usage are joint in estate or residence;
xxxxx
(21) "person" includes a joint family;"
12. Bombay General Clauses Act, 1904 also defines the word 'person'. The said definition
in section 3(35) is extracted below :
"3. Definitions : In this Act, and in all Bombay and Gujarat Acts made after the
commencement of this Act, unless there is anything repugnant in the subject or
context. ...........(35). "Person" shall include any company or association or body of
individuals, whether incorporated or not;
13. Section 63 of the Bombay Tenancy and Agricultural Lands Act, 1948, as applicable in
Gujarat bars transfer to non-agriculturists. Sub-section (1) of section 63 provides that
save as otherwise provided in the Act, no sale shall be valid in favour of a person - (i)
who is not an agriculturist, or (ii) who being an agriculturist cultivates lands not less than
ceiling area, or (iii) who is not an agricultural labour. The first proviso to sub-section (1)
provides that the Collector (or an officer authorized by the State Government) may grant
permission for such sale, but the second proviso to sub-section (1) provides that no such
permission shall be granted where the land is being sold to a person who is not an
agriculturist for agricultural purpose, if the annual income of such person from other
sources exceeds Rupees five thousand.
Question (i) - who is a 'person' ?
14. The extent of land that could be held by the appellants depends upon the
interpretation of the word 'person' in section 6(1) of the Ceiling Act which provides that
"no person shall be entitled to hold land in excess of the ceiling area". If the ten co-
owners are considered as an 'association of persons' or 'body of individuals', and
consequently as a 'person', then the ten co-owners
@page-SC1252
together as a person, will be entitled to only one unit of land which is the ceiling area per
person. But if 'association of persons' or body of individuals is not a 'person', or if a co-
ownership is not an association of person/body of individuals, then each co-owner or the
family of each co-owner, as the case may be will be a separate 'person' having regard to
the definition of person in section 2(21) of Ceiling Act, in which event, each family will
be entitled to hold one unit of land.
15. The word 'person' is defined in the Act, but it is an inclusive-definition, that is "a
person includes a joint family." Where the definition is an inclusive definition, the use of
the word 'includes' indicates an intention to enlarge the meaning of the word used in the
Statute. Consequently, the word must be construed as comprehending not only such
things which they signify according to their natural import, but also those things which
the interpretation clause declares that they shall include. Thus, where a definition uses the
word 'includes', as contrasted from 'means', the word defined not only bears its ordinary
popular and natural meaning, but in addition also bear the extended statutory meaning
(See S.K. Gupta v. K.P. Jain, AIR 1979 SC 734 following Dilworth vs. Commissioner of
Stamps, 1899 AC 99 and Jobbins vs. Middlesex County Council -1949 (1) KB 142).
16. The ordinary, popular and natural meaning of the word 'person' is 'a specific
individual human being'. But in law the word 'person' has a slightly different connotation,
and refers to any entity that is recognized by law as having the rights and duties of a
human being. Salmond defines 'person' as 'any being whom the law regards as capable of
rights and duties' or as 'a being, whether human or not, of which rights and duties are the
attributes' (Jurisprudence : 12th Edition Page 299]. Thus the word 'person', in law, unless
otherwise intended, refers not only to a natural person (male or female human being), but
also any legal person (that is an entity that is recognized by law as having or capable of
having rights and duties). The General Clauses Act thus defines a 'person' as including a
corporation or an association of persons or a body of individuals whether incorporated or
not. The said general legal definition is, however, either modified or restricted or
expanded in different statutes with reference to the object of the enactment or the context
in which it is used. For instance, the definition of the word 'person' in Income Tax Act, is
very wide and includes an individual, a Hindu Undivided Family, a company, a firm, an
association of persons or body of individuals whether incorporated or not, a local
authority and every other artificial juridical person. At the other extreme is the
Citizenship Act, section 2(f) of which reads thus : "Person does not include any company
or association or body of individuals whether incorporated or not." Similarly, the
definition under Section 2(g) of Representation of the People Act 1950, is "person" does
not include a body of persons.
17. Both definitions of the word 'person', in General Clauses Act and Ceiling Act, are
inclusive definitions. The inclusive definition of 'person' in General Clauses Act applies
to all Gujarat Act unless there is anything repugnant in the subject or the context. The
inclusive definition of 'person' in section 2(21) of the Ceiling Act, does not indicate
anything repugnant to the definition of 'person' in General Clauses Act, but merely adds
'joint family' to the existing definition. Therefore the definition of person in the Ceiling
Act, would include the definition of person in section 3(35) of General Clauses Act. The
resultant position can be stated thus : The definition of person in General Clauses Act,
being an inclusive definition, would include the ordinary, popular and general meaning
and those specifically included in the definition. The inclusive definition of 'person' in the
Ceiling Act, in the absence of any exclusion, would have the same meaning assigned to
the word in the General Clauses Act, and in addition, a joint family' as defined. Thus, the
word 'person' in the Ceiling Act will, unless the context otherwise requires, refer to :
(i) a natural human being,
(ii) any legal entity which is capable of possessing rights and duties, including any
company or association of persons or body of individuals (whether incorporated or not);
and
(iii) a Hindu Undivided Family or any other group or unit of persons, the members of
which by custom or usage, are joint in estate and residence.
18

. We are fortified in this view by the decision of this Court in Hasmukhalal Dahayabhai
vs. State of Gujarat, 1976 (4) SCC 100, wherein thisAIR 1976 SC 2316
Para 16 of AIR

@page-SC1253
Court had occasion to consider the definition of 'person' in the Ceiling Act, in a different
context. It was contended in that case that in view of the definition of 'person' in General
Clauses Act, 1897, a central enactment, that is, 'person' shall include any company or
association or body of individuals, whether incorporated or not, there cannot be a
different definition in the Ceiling Act, and therefore, section 6(2) of the Ceiling Act
treating family as a 'person' was unconstitutional. It was pointed out that section 6(2) had
the effect of making a person who held land within ceiling limit, to lose part of his/ her
holding, on marriage to someone who also held land within ceiling limit. (For example, if
a bachelor who was holding 35 acres of land which is within ceiling limit, married
someone who held 20 acres, they will together lose 19 acres by reason of the fact that
they formed a 'family'). This Court negatived the challenge to the definition of 'person'. In
that context this Court observed that the term 'person' is not, strictly speaking, defined in
the Act, and the definition merely clarified that the term includes a joint family and did
not exclude an individual from being a person in the eye of law. This Court observed that
the term 'person' for the purposes of Ceiling Act would include individuals as natural
persons as well as group or body of individuals as artificial persons, as also a joint family
and a family. This Court proceeded to explain section 6(2) thus:
"We do not find any fixed concept of "person" anywhere. No doubt the concept is wide so
that it could be contended that it should not be narrowed down or confined. But does
Section 6(2) do that? Section 6(2) does not either disable a husband or wife from owning
or holding their separate properties separately. It does not merge or destroy their separate
legal personalities. It requires their separate holdings to be grouped together as though
they were held by one person only for the purpose of determining the ceiling limit for
each member of a family. It may indirectly have the effect of disabling a member of a
family from holding land upto the prescribed ceiling limit for a person holding as an
individual. In other words, the result is that such a member of a family will have to be
content with a holding less than that of an unmarried individual. It has the effect of
making it clear that what have to be grouped together are the separate properties of
individuals belonging to families other than what are "joint families", in law. It takes in
and applies to members of families other than undivided Hindu families. It means that
married persons and their minor children will have to be viewed as though they hold one
lot together even though they retain their separate legal personalities and remain
competent owners of their separate holdings. It does not affect either their legal status or
competence. It does reduce their individual holdings."
Question (ii) - Whether co-owners are together a 'person'?
19
. When several persons acquire undivided shares (as contrasted from defined portions) in
a property, either equal or unequal, they become co-owners of the property; or where an
owner of a property transfers a share in the property to another, the transferee becomes a
co-owner along with the original owner. To be termed as co-owners, the right of each
owner should be co-ordinate with the other 'owners'. If the right of one is higher in degree
than that of the other, there is no co-ownership. For example, a mortgagor and mortgagee
are not co-owners. A lessor and lessee are not co-owners. Whether the shares are equal or
not, each co-owner is entitled to be in possession of every part of the property, jointly
with the other co-owners. In Sri Ram Pasricha v. Jagannath [AIR 1976 SC 2335], this
Court observed : Para 29 of AIR

"Jurisprudentially it is not correct to say that a co-owner of a property is not its owner. He
owns every part of the composite property along with others and it cannot be said that he
is only a part-owner or a fractional owner of the property. The position will change only
when partition takes place. .."

This Court also relied on the following passage from Salmond's 'Jurisprudence' : Para
28 of AIR

"It is an undivided unity, which is vested at the same time in more than one person The
several ownership of a part is a different thing from the co-ownership of the whole. So
soon as each of two co-owners begins to own a part of the thing instead of the whole of it,
the co-ownership has been dissolved into sole ownership by the process known as
partition. Co-ownership involves the undivided integrity of what is owned."
20

. The terms 'association of persons' 1997 AIR SCW 1746

@page-SC1254
and 'body of individuals' (which are interchangeable) have a legal connotation and refer
to an entity having rights and duties. They are not to be understood literally. For example,
if half a dozen people are travelling in a car or a boat, or standing in a bus stop, they may
be a group of persons or a 'body of individuals' in the literal sense. But they are not an
association of persons/body of individuals in the legal sense. When a calamity occurs or a
disaster strikes, and a band of volunteers or doctors meet at the site and associate or co-
operate with each other for providing relief to victims, and not doing anything for their
own benefit, they may literally be an association of persons, but they are not 'an
association of persons/ body of individuals' in the legal sense. A mere combination of
persons or coming together of persons without anything more, without any intention to
have a joint venture or carry on some common activity with a common understanding and
purpose will not convert two or more persons into a body of individuals/association of
persons. An 'association of persons/body of individuals' is one in which two or more
persons join in a common purpose and common action to achieve some common benefit.
Where there is a combination of individuals by volition of the parties, engaged together
in some joint enterprise or venture, it is known as 'association of persons/body of
individuals'. The common object will have some relevance to determine whether a group
or set of persons is an association of persons or body of individuals with reference to a
particular statute. For example, when the said terms 'association of persons' or 'body of
individuals' occur in a section which imposes a tax on income, the association must be
one the object of which is to produce income, profit or gain, [vide : Commissioner of
Income Tax vs. Indira Balkrishna (AIR 1960 SC 1172), Mohammed Noorulla vs.
Commissioner of Income Tax, Madras (AIR 1961 SC 1043), M.V. Shanmugam vs.
Commissioner of Income Tax, Madras (AIR 1970 SC 1707) and Meera and Company vs.
Commissioner of Income Tax, 1997 (4) SCC 677). But the object need not always be to
carry on commercial or business activity. For example, when the word 'person' occurs in
a statute relating to agriculture or ceiling on land holding, the term 'association of
persons/body of individuals' may refer to a combination of individuals who join together
to acquire and own land as co-owners and carry on agricultural operations as a joint
enterprise.
21. Normally, where a group of persons have not become co-owners by their volition
with a common purpose, they cannot be considered as a 'person'. When the children of
the owner of a property succeed to his property by testamentary succession or inherit by
operation of law, they become co-owners, but the co-ownership is not by volition of
parties nor do they have any common purpose. Each can act in regard to his/ her share, on
his/her own, without any right or obligation towards the other owners. The legal heirs
though co-owners, do not automatically become an 'association of persons/ body of
individuals'. When different persons buy undivided shares in a plot of land and engage a
common developer to construct an apartment building, with individual ownership in
regard to respective apartment and joint ownership of common areas, the co-owners of
the plot of land, do not become an 'association of persons/body of individuals', in the
absence of a deeming provision in a statute or an agreement. Similarly, when two or more
persons merely purchase a property, under a common sale deed, without any agreement
to have a common or joint venture, they will not become an 'association of persons/body
of individuals'. Mere purchase under a common deed without anything more, will not
convert a co-ownership into ajoint enterprise. Thus when there are ten co-owners of a
property, they are ten persons and not a 'body of individuals' to be treated as a 'single
person'. But if the co-owners proceed further and enter into an arrangement or agreement
to have a joint enterprise or venture to produce a common result for their benefit, then the
co-owners may answer the definition of a 'person'.
Question (iii) - Whether the ten purchasers constitute a 'person'?
22. We will now examine whether a group of individuals purchasing agricultural land
jointly as co-owners, not with the intention of retaining the property in co-ownership and
carrying on agricultural activities jointly, nor with the intention of managing it as a joint
venture nor with the intention of holding it together to generate income, profit or gain,
but solely with the intention of dividing the land so purchased and hold their respective
shares separately and individually, can be considered as a 'person' for the purposes of the
Ceiling Act. The Tribunal and the High court have proceeded on the
@page-SC1255
basis that the ten purchasers constituted an 'association of persons' and therefore a
separate juristic person. Let us examine whether the said conclusion is correct.
23. Instead of buying the land (172 acres, 36 guntas) jointly under the four sale deeds it
was open to the ten persons to have bought the lands individually, that is each of them
purchasing such extent of land as he or she wanted. If they had registered the sale deeds
individually (subject to each of them being entitled to buy agricultural land, under the
land reforms laws in force) each couple would have been entitled to hold land to the
extent of one unit. Instead of each individual or couple purchasing the land in their
respective names, if for convenience in negotiations, ten individuals buy the land jointly,
the position will be no different. It cannot be said that merely because the sale deed is in
the joint names of ten persons, they purchased the land as "an association of persons" or
as "body of individuals" with the common intention of carrying on agricultural activities
jointly or producing income, profit or gain or carry on some common joint venture. In
fact before purchasing the lands, the ten persons had entered into an agreement placing
on record that the object of purchasing the lands jointly was only to facilitate negotiations
and avoid duplicating the purchase procedures and not to cultivate them jointly. There is
no evidence of any joint cultivation, nor any evidence of any intention to have a joint
venture. On the other hand, after purchase, they divided the lands and informed the land
revenue authorities and each co-owner was registered as the owner of the respective land
allotted to him/her. This is not a case where a body of individuals purchased the land with
the intention of having a continued community of interest by way of a joint venture or as
a business venture. It is therefore not possible to treat the ten purchasers as an 'association
of persons/body of individuals' nor is it permissible to treat them as a single 'person',
thereby restricting their entitlement to hold land to only one unit, even though there are
ten purchasers.
24. The Tribunal and the High Court were right in holding that the word 'person' in the
Ceiling Act includes an 'association of persons/body of individuals'. But they were not
justified in treating the co-owners as an 'association of persons', or in holding that the ten
co-owners will be entitled to own only one unit. Having regard to section 6(2) of the Act,
the share of each couple (husband and wife) in the land, plus any other land individually
held by them will have to be calculated to find out whether they held any land in excess
of the ceiling limit. Therefore the share of each appellant in the lands jointly purchased,
with the addition of the lands held by his spouse, and addition of any other land held by
them, will give the basis for determining the surplus land. For example, if a husband's
share as co-owner is 20 acres and wife's share as co-owner is 20 acres, and their other
individual holding is another 10 acres (all of the same category in C class), the total
holding of the family will be 50 acres (20+20+10 acres) and the surplus will be 14 acres.
Question (iv) - What is the effect of section 8 of Ceiling Act?
25. The fact that the partition among the co-owners on 30-12-1971 was between 24-1-
1971 and the date on which the Gujarat Agricultural Land Ceiling Amendment Act, 1972
came into effect, is not in dispute. Subsection (1) of section 8 makes it clear that where
any person has transferred or partitioned any land held by him between those dates then
notwithstanding anything contained in any law, such partition shall be deemed to have
been made in anticipation in order to defeat the object of the said Amendment Act unless
it is proved to the contrary. Sub-section (2) provides how the affected person may prove
the contrary by filing an application to the collector seeking a declaration to that effect.
When such an application is made, sub-section (3) requires the Collector to hold an
inquiry and make an order either rejecting the application or declaring that the partition
was not made in anticipation in order to defeat the object of the Amending Act, 1972.
Sub-section (4) lays down that where an application for declaration is rejected, the
transfer or partition shall be ignored in computing the area of surplus land held by such
person. The same will be the effect where no application is made at all. It is thus clear
that the statutory presumption that a partition made between those dates was intended to
defeat the object of the Amendment Act cannot be displaced unless an application is
made under sub-section (2) and a declaration is made by the Collector under sub-section
(3). In this case, neither any application was made under sub-section (2) nor any
declaration
@page-SC1256
was made under sub-section (3). Therefore it has to be held that the partition effected on
30-12-1971 was deemed to have been made in order to defeat the object of the
Amendment Act, 1972 and consequently, the partition will have to be ignored while
computing the surplus land.
26. But in this case the exclusion of the partition may not have any significant effect. As
we have held that the ten co-owners together are not to be treated as a person, the share of
the co-owner will have to be calculated and the lands individually held by such co-owner
will have to be added to determine his holding. Thereafter, the holdings of the person
with that of his spouse and minor sons and unmarried daughter will have to be aggregated
to determine the surplus.
Question (v) - What requires to be done?
27. The Mamlatdar will have to decide the matter by holding an enquiry under section 21
of the Ceiling Act keeping in view the principles laid down in sections 6 to 8 of the
Ceiling Act. Further section 63 of the Tenancy Act also has to be kept in view while
examining the claim of co-owners. If the sale was effected jointly in the name often
persons to enable some non-agriculturists who were barred from buying agricultural land,
to buy agricultural land by joining some agriculturists as co-purchasers, the sale to the
extent it is in favour of non-agriculturists will not be valid and the consequences on
account of sale not being valid for violation of section 63 of Tenancy Act will follow as
provided in section 84C of that Act. For example if ten purchasers purchase hundred
acres of land with equal shares, and six of them are non-agriculturists, then the sale in
respect of the six non-agriculturists (to an extent of 60 acres) will not be valid and such
land purchased by non-agriculturists may have to vest in the State Government as
provided in section 84C of the Tenancy Act. In this case no such enquiry has been held to
find out about the validity of the sale. Therefore, the matter will have to be remitted to the
Mamlatdar to hold an enquiry under sections 63 and 84C of Tenancy Act to decide
whether all the purchasers were agriculturists who were entitled to purchase agricultural
land and whether transfer in favour of all of them is valid or invalid and to make
consequential orders.
28. We therefore, allow this appeal, set aside the orders of the High Court as also that of
the authorities below. As a consequence, we direct the Mamlatdar :
(a) to decide whether any of the ten purchasers is a non-agriculturist and if so the extent
of transfer in favour of such non-agriculturist which will be invalid and pass
consequential orders in respect of such land in accordance with law;
(b) to determine whether any of the ten purchasers who are agriculturists, holds excess
land by considering their share in the lands purchased as co-owners, with other lands as
provided in sections 6 to 8 of the Ceiling Act, and pass appropriate orders in accordance
with law.
Appeal allowed.
AIR 2008 SUPREME COURT 1256 "V. Subbulakshmi v. S. Lakshmi"
(From : Madras)*
Coram : 2 S. B. SINHA AND H. S. BEDI, JJ.
Civil Appeal No. 990 of 2008 (arising out of SLP (C) No. 19532 of 2006), D/- 5 -2 -2008.
V. Subbulakshmi and Ors. v. S. Lakshmi and Anr.
(A) Motor Vehicles Act (59 of 1988), S.173, S.149(2) - MOTOR VEHICLES - APPEAL
- INSURANCE - Appeal - Joint appeal by owner and Insurance Company -
Maintainability - Question not decided since owner was "aggrieved person" and
therefore, appeal even in absence of insurer was maintainable. (Paras 13, 14)
(B) Motor Vehicles Act (59 of 1988), S.168 - MOTOR VEHICLES - DOCUMENTS -
Motor accident - Compensation - Income of victim - Claimants producing documents
showing income of deceased as Rs. 12,500/- p. m. - Documents so produced found
unreliable - Court taking it at Rs. 7,000/- p. m. - Though without any basis, cannot be said
to be improper when no reliable evidence was on record and guess work was Inevitable.
(Paras 18)
Cases Referred : Chronological Paras
2002 AIR SCW 3899 : AIR 2002 SC 3350 (Ref.) 15
2000 AIR SCW 1321 : AIR 2000 SC 1565 (Expln.) 11, 12
(1998)9 SCC 202 10
AIR 1993 Del 324 10
@page-SC1257

AIR 1993 JandK 69 10


AIR 1987 MP 244 10
AIR 1979 Cal 152 10
V. Krishnamurthy, Sr. Advocate, V. Ramasubramanian, for Appellants; Ashok Kumar
Sharma, for Respondents.
* C.M.A. No. 222 of 2002, D/- 11-2-2006 (Mad) (Madurai Bench)
Judgement
1. S. B. SINHA, J. :-Leave granted.
2. First respondent is the owner of a bus. Allegedly, owing to rash and negligent driving
by the driver of the said vehicle, an accident took place wherein one Vadivelu, the
predecessor in interest of the appellants died.
3. An application under Section 166 of the Act claiming compensation for a sum of Rs.25
lakhs was filed by the appellants in the Court of Motor Accidents Claims Tribunal
(Additional District Judge-cum-Chief Judicial Magistrate, Karur). A written statement
was filed by the Insurance Company in the said proceedings. The same was adopted by
the owner of the vehicle. Before the Tribunal, the appellants produced some documents to
show that the income of the deceased was about 12,500/- per month. He is said to have
been deriving income both as an agriculturist as also from his business as commission
agent in the business of coconut.
4. The Tribunal, inter alia, keeping in view the fact that the Income Tax Returns were
filed only after the death of the said Vadivellu, estimated at Rs. 9,600/- per month.
The High Court, however, estimated the income of the deceased to be around a sum of
Rs. 4,000/- per month, from his agricultural operation and Rs. 3,000/- from his
commission business, totalling a sum of Rs. 7,000/- per month and upon deducting 1/ 3rd
thereof from the amount towards his personal expenses, the High Court held that his
contribution to his family would come to about of Rs. 4,667/- per month. Applying the
multiplier of 18, the loss of income was assessed at Rs. 10,08,072/-, instead and in place
of Rs. 13,82,400/- as was found by the Tribunal.
5. Appellant is, thus, before us. Despite service of notice, the first respondent has not
appeared.
6. Mr. V. Krishnamurthy, the learned senior counsel appearing on behalf of the appellant,
inter alia, would submit that a joint appeal by the owner of the vehicle and the Insurance
Company was not maintainable. It was furthermore urged that the High Court without
analysing the evidence on records has arbitrarily reduced the amount of income of the
deceased from Rs. 9,600/-as was found by the learned Tribunal, to a sum of Rs. 7,000/-
per month.
7. Mr. Ashok Kumar Sharma, the learned counsel appearing on behalf of the second
respondent, on the other hand, submitted that the appeal before the High Court in terms
of Section 173 of the Act was maintainable. According to the learned counsel keeping in
view the phraseology used in Section 173 of the Act, an appeal subject to the limitation
provided under sub-Section (2) thereof would be maintainable against each and every
award and, thus, if an appeal is maintainable at the instance of the Insurance Company, it
matters little as to whether it was filed with the owner of the vehicle or not. The learned
counsel furthermore urged that the Tribunal has failed to take into consideration the fact
that the documents filed by the claimants/appellants purporting to establish the quantum
of income of the deceased being wholly unreliable, the same could not have been taken
into consideration for the purpose of computation of income.
8. We may at the outset notice that the High Court was although of the opinion that no
appeal would be maintainable at the instance of an insurance company unless permission
of the court was obtained by it in terms of Section 170 of Act, observed that the owner of
the vehicle being an appellant, the appeal would be maintainable at his instance.
9. The relevant statutory provisions, being Sections 149(2), 170 and 173 may be noticed
by us, which are as under :
"149. (2) No sum shall be payable by an insurer under sub-section (1) in respect of any
judgment or award unless, before the commencement of the proceedings in which the
judgment of award is given the insurer had notice through the Court or, as the case may
be, the Claims Tribunal of the bringing of the proceedings, or in respect of such judgment
or award so long as execution is stayed thereon pending an appeal; and an insurer to
whom notice of the bringing of any such proceedings is so given shall be entitled to be
made a party thereto and to defend the action on any of the following grounds, namely :-
@page-SC1258
(a) that there has been a breach of a specified condition of the policy, being one of the
following conditions, namely :-
(i) a condition excluding the use of the vehicle -
(a) for hire or reward, where the vehicle is on the date of the contract of insurance a
vehicle not covered by a permit to ply for hire or reward, or
(b) for organised racing and speed testing, or
(c) for a purpose not allowed by the permit under which the vehicle is used, where the
vehicle is a transport vehicle, or
(d) without side-car being attached where the vehicle is a motor cycle; or
(ii) a condition excluding driving by a named person or persons or by any person who is
not duly licensed, or by any person who has been disqualified for holding or obtaining a
driving licence during the period of disqualification; or
(iii) a condition excluding liability for injury caused or contributed to by conditions of
war, civil war, riot or civil commotion; or
(b) that the policy is void on the ground that it was obtained by the nondisclosure of a
material fact or by a representation of fact which was false in some material particular.
Section 170 - Impleading insurer in certain cases - Where in the course of any inquiry, the
Claims Tribunal is satisfied that -
(a) there is collusion between the person making the claim and the person against whom
the claim is made, or
(b) the person against whom the claim is made has failed to contest the claim, it may, for
reasons to be recorded in writing, direct that the insurer who may be liable in respect of
such claim, shall be impleaded as a party to the proceeding and the insurer so impleaded
shall thereupon have, without prejudice to the provisions contained in sub-section (2) of
section 149, the right to contest the claim on all or any of the grounds that are available to
the person against whom the claim has been made.
Section 173 - Appeals - (1) Subject to the provisions of sub-section (2) any person
aggrieved by an award of a Claims Tribunal may, within ninety days from the date of the
award, prefer an appeal to the High Court:
Provided that no appeal by the person who is required to pay any amount in terms of such
award shall be entertained by the High Court unless he has deposited with it twenty-five
thousand rupees or fifty per cent, of the amount so awarded, whichever is less, in the
manner directed by the High Court:
Provided further that the High Court may entertain the appeal after the expiry of the said
period of ninety days, if it is satisfied that the appellant was prevented by sufficient cause
from preferring the appeal in time.
(2) No appeal shall lie against any award of a Claims Tribunal if the amount in dispute in
the appeal is less than ten thousand rupees."
10. The maintainability of an appeal by the Insurance Company together with the owner
of the vehicle came up for consideration before this Court in Narendra Kumar and
Another Vs. Yarenissa and Others [(1998) 9 SCC 202], wherein it was clearly held that an
appeal by the owner of the vehicle is maintainable despite the fact that in terms of an
Award, he is to be reimbursed by the insurance company, stating;

"6......If the award has gone against the tortfeasors it is difficult to accept the contention
that the tortfeasor is not "an aggrieved person" as has been held by some of the High
Courts vide Kantilal and Bros. v. Ramarani Debi, New India Assurance Co. Ltd. v.
Shakuntla Bai, Nahar Singh v. Manohar Kumar, Radha Kishan Sachdeva v. Fit. Lt. L.D.
Sharma merely because under the scheme of Section 96 if a decree or award has been
made against the tortfeasors the insurer is liable to answer judgment "as if a judgment-
debtor". That does not snatch away the right of the tortfeasors who are jointly and
severally liable to answer judgment from preferring an appeal under Section 110-D of the
Act. If for some reason or the other the claimants desire to execute the award against the
tortfeasors because they are not in a position to recover the money from the insurer the
law does not preclude them from doing so and, therefore, so long as the award or decree
makes them liable to pay the amount of compensation they are aggrieved persons within
the meaning of Section 110-D and would be entitled to prefer an appeal. But merely
because a joint appeal is preferred AIR 1979 Cal 152
AIR 1987 MP 224
AIR 1993 JandK 69
AIR 1993 Del 324

@page-SC1259
and it is found that one of the appellants, namely, the insurer was not competent to prefer
an appeal, we fail to see why the appeal by the tortfeasor, the owner of the vehicle, cannot
be proceeded with after dismissing or rejecting the appeal of the insurer. To take a view
that the owner is not an aggrieved party because the Insurance Company is liable in law
to answer judgment would lead to an anomalous situation in that no appeal would lie by
the tortfeasors against any award because the same logic applies in the case of a driver of
the vehicle. The question can be decided a little differently. Can a claim application be
filed against the Insurance Company alone if the tortfeasors are not the aggrieved parties
under Section 110-D of the Act? The answer would obviously be in the negative. If that is
so, they are persons against whom the claim application must be preferred and an award
sought for otherwise the insurer would not be put to notice and would not be liable to
answer judgment as if a judgment-debtor. Therefore, on first principle it would appear
that the contention that the owner of a vehicle is not an aggrieved party is unsustainable."
It was furthermore held;
"7. For the reasons stated above, we are of the opinion that even in the case of a joint
appeal by insurer and owner of offending vehicle if an award has been made against the
tortfeasors as well as the insurer even though an appeal filed by the insurer is not
competent, it may not be dismissed as such. The tortfeasor can proceed with the appeal
after the cause-title is suitably amended by deleting the name of the insurer."
11

. However, another Bench of this Court in Chinnama George and Others Vs. N.K. Raju
and Another [(2000) 4 SCC 130] opined : 2000 AIR SCW 1321

"6. Admittedly, none of the grounds as given in Sub-section (2) of Section 149 exist for
the insurer to defend the claims petition. That being so, no right existed in the insurer to
file appeal against the award of the Claims Tribunal. However, by adding N.K. Raju, the
owner as co-appellant, an appeal was filed in the High Court which led to the impugned
judgment. None of the grounds on which insurer could defend the claims petition was the
subject-matter of the appeal as far as the insurer is concerned. We have already noticed
above that we have not been able to figure out from the impugned judgment as to how the
owner felt aggrieved by the award of the Claims Tribunal. The impugned judgment does
not reflect any grievance of the owner or even that of the driver of the offending bus
against the award of the Claims Tribunal. The insurer by associating the owner or the
driver in the appeal when the owner or the driver is not an aggrieved person cannot be
allowed to mock at the law which prohibit the insurer from filing any appeal except on
the limited grounds on which it could defend the claims petition. We cannot put our
stamp of approval as to the validity of the appeal by the insurer merely by associating the
insured. Provision of law cannot be undermined in this way. We have to give effect to the
real purpose to the provision of law relating to the award of compensation in respect of
the accident arising out of the use of the motor vehicles and cannot permit the insurer to
give him right to defend or appeal on grounds not permitted by law by a backdoor
method. Any other interpretation will produce unjust results and open gates for the
insurer to challenge any award. We have to adopt purposive approach which would not
defeat the broad purpose of the Act. Court has to give effect to true object of the Act by
adopting purposive approach.
7. Sections 146, 147, 149 and 173 are in the scheme of the Act and when read together
mean : (1) it is legally obligatory to insure the motor vehicle against third party risk.
Driving an uninsured vehicle is an offence punishable with an imprisonment extending
up to three months or the fine which may extend to Rs. 1,000/- or both; (2) Policy of
insurance must comply with the requirements as contained in Section 147 of the Act; (3)
It is obligatory for the insurer to satisfy the judgments and awards against the person
insured in respect of third party risks. These are Sub-sections (1) and (7) of Section 149.
Grounds on which insurer can avoid his liability are given in Sub-section (2) of Section
149.
8. If none of the conditions as contained in Sub-section (2) of Section 149 exist for the
insurer to avoid the policy of insurance he is legally bound to satisfy the award, he cannot
be a person aggrieved by the award. In that case insurer will be barred from filing any
appeal against the award of the Claims Tribunal."
12

. In Chinnamma George, the owner did not challenge the 2000 AIR SCW 1321

@page-SC1260
findings of the Tribunal that the bus was being driven by the driver in a rash and
negligent manner. It was therefore, held that the owner was not an aggrieved person to
maintain an appeal. It was in the aforementioned context this Court observed that none of
the grounds as laid down under sub-Section (2) of Section 149 of the Act having been
satisfied, an appeal by the Insurance Company was not maintainable, observing that an
insurer having a limited area to defend the claim petition, it cannot circumvent the same
by associating itself with the owner/driver in an appeal when the owner/ driver is not an
aggrieved person and, thus, cannot be allowed to mock at the law.
13. In the instant case, the owner of the bus was an aggrieved person. He could maintain
an appeal of his own. Section 173 of the Act confers a right on any aggrieved person to
prefer an appeal from an award.
14. In the present case, it is not necessary for us to go into the larger question as to
whether having regard to the bar contained in sub-Section (2) of Section 149 of the Act,
the second respondent could have preferred an appeal questioning the quantum of
compensation, as the High Court held that the appeal, even after deletion of the second
respondent from the array of the parties, the appeal preferred by the first respondent was
maintainable.
15
. We may only notice that the aforementioned two decisions although have been referred
to by a three Judge Bench of this Court in National Insurance Co. Ltd., Chandigarh Vs.
Nicolletta Rohtagi and Others [(2002) 7 SCC 456], it was not specifically held even
therein that a joint appeal by the owner and the insurer would not be maintainable.2002
AIR SCW 3899

16. However, in this case, the appeal preferred by the Insurance Company has been
dismissed. The High Court has only entertained the appeal of the owner.
17. So far as the question in regard to the quantum of compensation awarded in favour of
the appellants is concerned, we are of the opinion that the High Court has taken into
consideration all the relevant evidences brought on record.
The accident took place on 7-5-1997. Income tax returns were filed on 23-6-1997.
The Income Tax Returns (Exp. P-14), therefore, have rightly not been relied upon.
Ex.P-8 is a deed of lease. It was an unregistered document. Although the document was
purported to have been executed on 10-4-1993, the genuineness thereof was open to
question. The stamp paper was purchased in the year 1983 but an interpolation was made
therein to show that it was purchased in 1993. The purported receipts granted by the
tenant were also unstamped.
18. In the aforementioned fact situation. the High Court has not relied upon all the
aforementioned documents, filed by the appellant. It may be true that there was no basis
for the High Court to arrive at the conclusion that the income of the deceased was Rs.
4,000/- from agricultural operation and Rs. 3,000/- from his commission business, but no
reliable document having been produced to show that the deceased was earning an
income of Rs. 12,500/- per month, as claimed. The High Court, in our opinion, cannot be
held to have, thus, committed any grave error in this behalf. There is no dispute as
regards application of the multiplier. In a case of this nature, some guess work is
inevitable. This Court could have gone into the question provided there was some
materials had been brought on record by the appellants upon which reliance could be
placed. There being no such material available on record, we are not in a position to
interfere with the impugned judgment of the High Court.
19. We, therefore, are of the opinion that it is not a fit case where this Court should
interfere with the judgment of the High Court. Appeal is dismissed. No costs.
Appeal dismissed.
AIR 2008 SUPREME COURT 1260 "Babu Ram v. State of Punjab"
(From : Punjab and Haryana)*
Coram : 2 PRAKASH PRABHAKAR NAOLEKAR AND LOKESHWAR SINGH
PANTA, JJ.
Criminal Appeal No. 654 of 2006, D/- 15 -2 -2008.
Babu Ram and Ors. v. State of Punjab.
(A) Penal Code (45 of 1860), S.300 - MURDER - Murder - Injuries on person of accused
- Non-explanation - Assumed more importance when witnesses examined are either
interested or inimical. (Para 19)
@page-SC1261
(B) Penal Code (45 of 1860), S.300 - MURDER - WITNESS - Murder - Non-
examination of independent witnesses - Casts doubt on prosecution case - Probabilises
defence Version. (Para 13)
(C) Penal Code (45 of 1860), S.300 - MURDER - PRIVATE DEFENCE - PLEA -
Murder - Plea of self-defence - Prosecution case that accused caught hold of deceased in
street and inflicted injury on his chest - Defence plea that deceased came armed to house
of accused, abused and inflicted injuries to accused and his wife - Defence plea
corroborated by medical evidence and evidence of wife of accused - Injuries on accused
not explained by prosecution - Injuries on accused simple but one injury on wife of
accused was grievous - Accused could be said to have apprehended danger to his and
wife's life - Infliction of single injury by accused with cobblers instrument he was
working with - Is in exercise of right of private defence - Accused liable to be acquitted.
(Paras 20, 21, 22)
Cases Referred : Chronological Paras
AIR 1976 SC 2263 : 1976 Cri LJ 1736 18
K.B. Sinha, Sr. Advocate, Mrs. Kawaljit Kochar, Ms. Sweta Rani, R.S. Mahla, Ms.
Kusum Chaudhary, for Appellants; Ranbir Yadav (for Kuldip Singh), for Respondent.
* Cri. A. No. 265-DB of 1997 and Cri. R. No. 306 of 1997, D/- 3-8-2005 (P and H).
Judgement
1. LOKESHWAR SINGH PANTA, J. :-The appellants have filed this appeal against the
Judgment and Order dated August 03, 2005 passed by a Division Bench of the Punjab
and Haryana High Court at Chandigarh, confirming the conviction and sentence in
respect of the offence punishable under Section 302 of the Indian Penal Code (for short
the 'IPC') and a fine of Rs.2000/- with a default clause to undergo R.I. for six months
awarded to Suraj Dev and imposing punishment of life imprisonment upon appellants
Babu Ram and Indraj under Section 302 read with Section 34 of IPC and to pay a fine of
Rs.2000/- each, in default of payment of fine to further suffer R.I. for six months each in
Sessions Case No. 83 of 1993 / Sessions Trial No. 10 of 1997.
2. Briefly put, the facts may be summarized as follows: On 03-03-1993 at about 7.30 PM,
the complainant Sohan Lal (PW-2) on hearing some noise coming from the street near the
house of Ram Pratap (PW-3) came out of his house with a torch in his hand. PWs.2 and 3
saw that Babu Ram (A-1) and Indraj (A-2) had caught hold of Ant Ram while Suraj Dev
(A-3) gave a blow with a 'Rambi' (an instrument for cutting leather by a cobbler) on the
chest of Ant Ram. On receipt of the said blow, Ant Ram fell down. PWs-2 and 3 raised an
alarm which attracted many other persons on the spot. Ant Ram was taken to the Civil
Hospital, Abohar, by PW-2 and PW-3 where he was declared dead. A ruqqa was sent to
Police Station regarding receipt of the dead body of Ant Ram. A.S.I. Hardev Singh (PW-
5) came to the hospital and recorded statement of PW-2 Sohan Lal. On the basis of the
said statement, formal FIR Ex.PH/1 came to be registered at the Police Station. The
Investigating Officer started investigation and prepared an inquest report. He recorded the
statement of PW-3 Ram Pratap on the following day of the incident. He went to the place
of occurrence and lifted the blood-stained earth and prepared a site plan.
3. On 08-03-1993, Indraj (A-2) was discharged from hospital and was arrested by the
Investigating Officer and thereafter investigation of the case was taken over by S.I.
Jagdev Singh. On 10-03-1993, S.I. Jagdev Singh accompanied by A.S.I. Hardev Singh
(PW-5) went to Village Kikker Khera, where Ram Pratap, Ex-Sarpanch, produced Babu
Ram (A-1) and Suraj Dev (A-3) who, later on, were arrested. It was further case of the
prosecution that on 12-03-1993, pursuant to the disclosure statement Ex.PK made by A-3,
the Police recovered one bloodstained 'Rambi' concealed in an iron box meant for
keeping clothes from his house. 'Rambi' (Ex.P1) was taken into possession through
Memo (Ex.PK/2). On completion of the investigation, charge sheet was presented against
the appellants A-1, A-2 and A-3 in the Court of Illaqa Magistrate, who committed the
same to the Court of Sessions. The learned Sessions Judge, Ferozpur, charge-sheeted A-3
Suraj Dev under Section 302 IPC, while A-1 Babu Ram and A-2 Indraj were charged
under Section 302 read with Section 34 of the IPC. The appellants pleaded not guilty to
the offence and claimed to be tried.
4. In support of its case, the prosecution examined Dr. Lal Chand Thakral as PW-1, Sohan
Lal as PW-2, Ram Pratap as PW-3, Het Ram as PW-4 and A.S.I. Hardev Singh
@page-SC1262
as PW-5. In their statements recorded under Section 313 of the Code of Criminal
Procedure, the appellants Babu Ram (A-1) pleaded as under :-
"I am innocent. I have love affairs with Kant daughter of Sohan Lal. I have produced
letters Mark D. 1 to D.8 which are in her own hand-writing and I have been falsely
involved due to this reason."
Indraj (A-2) pleaded as under :-
"I was preparing shoes by cutting leather with the Rambi. Ant Ram came there and
started abusing me for not supporting Sohan Lal. We exchanged abuses and a quarrel
started. In the meanwhile, my wife also came there. Ant Ram had a Kapa with him. He
gave blows to me and my wife with Kapa from its sharp and blunt side. In order to
defend me and my wife, I gave a Rambi blow to him. Police helped the deceased party as
brother of Ant Ram, i.e. Main Pal is working in the Police Department. Babu Ram and
my son Suraj Dev have been falsely implicated. Eye-witnesses Sohan Lal and Ram
Pratap are made up witnesses."
5. A-3 Suraj Dev pleaded that he is innocent and has been falsely implicated in the case
being the son of A-2 Indraj. In their defence, the appellants have examined Dr. Prithvi Raj
(DW-1), who medico legally examined A-2 Indraj and also his injured wife Smt. Maya.
Dr. R.K. Arora (DW-2) radiologically examined Smt. Maya (DW-3).
6. The Trial Court, on appreciation of the evidence, held the appellants guilty of the
murder of Ant Ram and, accordingly, convicted and sentenced them as aforesaid.
7. The appellants preferred an appeal under Section 374 (2) of the Cr.P.C. before the High
Court, whereas PW-2 Sohan Lal filed Criminal Revision No.306 of 1997 claiming
compensation to the heirs of the deceased. The High court, as stated above, confirmed the
conviction and sentence imposed upon the appellants, but dismissed the Criminal
Revision of the complainant.
8. The appellants are, thus, before us by Special Leave to Appeal.
9. Mr. K.B. Sinha, learned counsel for the appellants, assailed the judgment of the High
Court inter alia on two following grounds:-
(a) that the genesis of the occurrence is shrouded in mystery and the prosecution has
failed to explain the injuries on the person of the appellant-Indraj and his wife DW-3 Smt.
Maya, therefore, the conviction of the appellants suffers from infirmity and perversity;
(b) that the injury, which was found fatal to the deceased Ant Ram, was caused by the
appellant-Indraj in his self-defence and also to protect the body of his wife Smt. Maya
from further assault by the deceased. Thus, the appellants are entitled to the benefit of
doubt.
10. Learned counsel for the respondent-State, on the other hand, submitted that the
reasons given by the Trial Court as well as by the High Court regarding the order of
conviction of the appellants are based upon proper appreciation of the evidence led by the
prosecution in the case. According to him, the evidence of PW-2 and PW-3, the eye-
witnesses of the incident, is cogent and satisfactory with the hypothesis of the guilt of the
accused and the accused had no right of private defence to cause the grievous injury on
the vital part of the body of the deceased with sharp edged weapon and, therefore, the
plea of the right of private defence of accused Indraj is not tenable and acceptable.
11. Before adverting to the arguments advanced by the learned counsel, we shall, at the
threshold, point out that the incident, which resulted in the death of Ant Ram, is not in
dispute. We have reappraised and scrutinized the evidence of the eye-witnesses as well as
medical evidence of PW-1 Dr. Lal Chand Thakral who, at the relevant time, was posted
as Medical Officer, Civil Hospital, Abohar. Dr. Lal Chand Thakral conducted the post-
mortem on the dead body of Ant Ram on 04-03-1993 at 11.00 a.m. and found the
following injuries on his body:
1. A stab incised wound 2.5 cm X 1.5 cm present on the front of the chest on left side 4.5
cm below, the overlying shirt and vest.
2. On probing, the direction of wound was downwards medically and upwards. On
dissection clotted blood was present in underlying tissues, 5th rib was cut. On further
dissection the pericardium was out and left ventricle arterially was cut. There was about
700 cc of blood was present on the left side of the chest cavity. Heart was empty. The
injury was antemortem in nature.
12. In the opinion of the Doctor, the cause of death of Ant Ram was haemorrhage,
@page-SC1263
shock and injury to heart which was sufficient to cause death in due course of events.
13. PW-2 Sohan Lal and PW-3 Ram Pratap, the alleged eye- witnesses of the occurrence,
are interested witnesses. They are brother and nephew respectively of the deceased. The
prosecution has not examined any other witness in support of its case though it has come
in the evidence of PW-3 Ram Pratap that large number of persons including Chunni,
Banwari and Chhotu were present at the scene of occurrence, who witnessed the incident.
The defence of the accused Indraj corroborated by the evidence of his wife DW-3 appears
to be more plausible than the rival version of the evidence of PWs-2 and 3.
14. DW-1 Dr. Prithvi Raj was BMO posted at Civil Hospital, Abohar, at the relevant time.
He medico-legally examined the accused-Indraj on 03-03-1993 at 10.45 p.m. in the
hospital and found the following injuries on his person:
"1. There was an incised wound 2 x 0.5 cm bone deep sent on arteries lateral aspect of
left forearm, just above the wrist joint. Fresh bleeding was present. X-ray was advised.
2. There was an incised wound 2 x 0.5 cm bone deep present on back right forearm 5 cm
below electron on process. Fresh bleeding was present. X-ray was advised.
3. There was an unscabbed abrasion 1 x 1 cm over bridge of nose. X-ray was advised.
4. There was an abrasion 1.5 x 1 cm on right side of face below right eye adjacent to
nose.
5. There was an abrasion 3 x 1cm on back of right forearm on its upper part, 3 cm below
injury No.2.
6. There was an abrasion 1.5 x 1 cm on lateral side of right forearm on its lower part.
7. There was an abrasion 4 x 3 cm on front of left knee. Injuries Nos. l, 2 and 3 were kept
under observation and the remaining injuries were declared as simple. The probable
duration of the injuries was within six hours. Injuries Nos. 1 and 2 were caused by sharp
edged weapon while the remaining was caused by blunt weapon. I have brought the
original MLR today in the court. Exh.D is the certified copy of the MLR of Indraj and
Exh.DA/1 is the pictorial diagram showing the seat of injuries."
As per the opinion of Dr. Prithvi Raj, the abovesaid injuries on the person of accused
Indraj were found simple in nature after X-ray examination.
15. On the same day, Dr. Prithvi Raj medico-legally examined DW-3 Smt. Maya, wife of
the accused Indraj, and he found following injuries on her person :-
1. There was a reddish contusion 2 x 1.5 cm with underlying swelling 5 x 3 cm on
dorsum of right hand on medial side. X-ray was advised.
2. There was a reddish contusion 6 x 1.5 cm on lateral side of right arm on its middle.
3. There was an abrasion 3 x 2 cm on medial side of the left forearm on its upper part.
In the opinion of the Doctor, Injury Nos. 2 and 3 were found simple in nature, whereas
Injury No. 1 was declared grievous in nature caused by blunt weapon within the duration
of six hours.
16. DW-2 Mr. R.K. Arora was S.M.O., Civil Hospital, Abohar, at the relevant time. Dr.
Arora conducted X-ray examination of Smt. Maya on 04-03-1993 and found fracture of
the base of proximal phalyn of right little finger.
17. DW-3 Smt. Maya stated in her deposition that on the day of occurrence her husband
Indraj was cutting leather for making shoes with a 'Rambi' when Ant Ram came in front
of their house and without any cause he started abusing her husband. Ant Ram was armed
with 'Kapa' with which he gave 6-7 blows to her husband from sharp and blunt side of the
weapon. She tried to save her husband from further assault, but Ant Ram inflicted
grievous injuries on her arms with 'Kapa'. She stated that her husband Indraj was carrying
a 'Rambi' in his hand with which he inflicted injury to Ant Ram to save himself as well as
her body. She further stated that she went to the hospital along with her injured husband
where Doctor medically examined them. She narrated the entire incident to the police,
but the police did not listen to her version as the nephew of the deceased is posted as a
Constable in the Police Department. Despite searching lengthy cross-examination, the
prosecution has failed to elicit any material to shatter and discredit her testimony. She
categorically denied the suggestion of the prosecution that the occurrence had taken place
in the street in front of the house of
@page-SC1264
PW-3 Ram Pratap. Further suggestion that all the three accused persons committed
murder of Ant Ram was categorically denied by her. The plea of the prosecution that she
and her husband Indraj were given slaps and fist blows by the general public, who
collected at the scene of the occurrence, has been specifically denied by her. Dr. Prithvi
Raj has categorically opined that the injuries on the person of accused Indraj and his wife
Smt. Maya could be caused by sharp and blunt weapon and not by slaps and fist blows.
18. It is a well-settled law that in a murder case, the non-explanation of the injuries
sustained by the accused at about the time of the occurrence or in the course of altercation
is a very important circumstance from which the Court can draw the following
inferences:-
1. that the prosecution has suppressed the genesis and the origin of the occurrence and
has thus not presented the true version;
2. that the witnesses who have denied the presence of the injuries on the person of the
accused are lying on a most material point and therefore their evidence is unreliable;
3. that in case there is a defence version which explains the injuries on the person of the
accused it is rendered probable so as to throw doubt on the prosecution case. [See
Lakshmi Singh v. State of Bihar; AIR 1976 SC 2263]
19. Further, it is important to point out that the omission on the part of the prosecution to
explain the injuries on the person of the accused assumes much greater importance where
the evidence consists of interested or inimical witnesses or where the defence gives a
version which competes in probability with that of the prosecution one.
20. In the present case, the prosecution has not explained the injuries on the person of the
accused Indraj and his wife Smt. Maya Devi. It has come in the evidence of PW-5 S.I.
Hardev Singh, the Investigating Officer, that the accused Indraj was admitted in the
hospital and he was arrested on 08-03-1993 after discharge from the hospital. The
Investigating Officer has also collected the MLRs of the accused Indraj and his wife
Maya on the intervening night of 03rd/04th March 1993 at about 12.30 a.m. The
occurrence had taken place on 03-03-1993 at about 7.30 p.m. as per the prosecution
version itself. The evidence of Dr. Prithvi Raj would show that the duration of the injuries
suffered by the accused Indraj and his wife Smt. Maya was within duration of six hours.
The evidence of DW-Smt. Maya, corroborated by the medical evidence of Dr. Prithvi
Raj, is more probable, satisfactory and convincing than the evidence of PWs. 2 and 3 who
are interested witnesses and who have not given true genesis and origin of the occurrence
in their testimony. Thus, non-explanation of the injuries on the person of the accused
Indraj and his wife Smt. Maya supports the version of the defence that the accused Indraj
inflicted single blow with 'Rambi' on the person of Ant Ram in private defence of his
body and also the person of his wife who had suffered several injuries at the hands of Ant
Ram with a weapon called 'Kapa', though the injury received by the deceased Ant Ram
was turned out to be fatal in nature but it was not inflicted by the accused with the
intention of causing death of the deceased or with the intention or knowledge that the
injury would, in all probability, cause his death.
21. We do not agree with the courts be low that the accused Indraj had no right of private
defence to his body or to the person of his wife. It is established on record by DW-3 that
Ant Ram came in front of their house and for no cause he started abusing her husband.
Ant Ram gave 6-7 blows to her husband with sharp as well as blunt side of 'Kapa' which
he was holding in his hand. Ant Ram also inflicted injuries on the arms of DW-Maya and
it was at that time and in that process that 'Rambi' blow was given to Ant Ram by her
husband to save himself and his wife from further blows, A 'Kapa' is capable of causing
simple as well as fatal injuries to the accused Indraj and his wife Smt. Maya. We are of
the view that in such a situation the accused Indraj could necessarily apprehend danger to
his life and to the life of his wife at the hand of Ant Ram and in that process if one blow
was inflicted by the accused on the person of Ant Ram which has proved fatal, the
accused had the right of private defence to his body as well as to the person of his wife
Smt. Maya. The injuries on the person of the accused Indraj were simple in nature,
whereas one injury found on the person of his wife Smt. Maya by the Doctor was
grievous in nature. The evidence of DW-3, corroborated by the medi
@page-SC1265
medical evidence, is so clear and cogent, so independent and disinterested, so probable,
consistent and credit-worthy that it has to be relied upon and accepted vis-a-vis the
evidence of the prosecution.
22. Having given our careful consideration to the submissions made by the learned
counsel for the parties and in the light of the evidence and in the background of the well-
settled proposition of law and in view of the improbabilities, the serious omissions and
infirmities, the interested nature of the evidence and other circumstances, it was clear that
the prosecution has failed to prove the case against the appellants beyond reasonable
doubt. The High Court was in error in brushing aside serious infirmity in the prosecution
case regarding non-explanation of the injuries sustained by the accused Indraj and his
wife DW-Smt. Maya and also not accepting the plea of the right of defence of the
accused on unconvincing premises.
23. For the reasons given above, we allow the appeal, set aside the conviction of the
appellants-Babu Ram and Indraj under Section 302 read with Section 34 of the IPC and
Suraj Dev under Section 302 of the IPC. The appellants are, accordingly, acquitted.
24. Appellants-Babu Ram, Indraj and Suraj Dev, who are presently in jail, shall be set
free forthwith if they are not required in any other case. Fine imposed by the Trial Court,
if realized from the appellants, shall be paid back to them.
Appeal allowed.
AIR 2008 SUPREME COURT 1265 "Har Prasad v. Ranveer Singh"
(From : Allahabad)*
Coram : 2 Dr. A. PASAYAT AND P. SATHASIVAM, JJ.
Criminal Appeal No. 294 of 2008 (arising out of SLP (Cri.) No. 365 of 2007), D/- 12 -2
-2008.
Har Prasad and Anr v. Ranveer Singh and Anr.
Criminal P.C. (2 of 1974), S.190, S.173 - COGNIZANCE OF OFFENCE -
INVESTIGATION - POLICE OFFICERS - Complaint and police report regarding
offence - Complaint accompanied by false affidavit - That becomes irrelevant when
cognisance is taken on police report. (Para 8)
Cases Referred : Chronological Paras
AIR 1968 SC 117 : 1968 Cri LJ 97 (Ref.) 7
AIR 1955 SC 196 : 1955 Cri LJ 526 7
Shakil Ahmed Syed, for Appellants; Shall Kumar Dwivedi, A.A.G., Debasis Misra,
Anuvrat Sharma, S.N. Pandey, Ms. Vandana Mishra and Ms. Vibha Dwivedi, with him
for Respondents.
* Cri. Revn. No. 147 of 2001, D/- 24-11-2006 (All.)
Judgement
Dr. ARIJIT PASAYAT, J :- Leave granted.
2. Challenge in this appeal is to the order passed by a learned Single Judge of the
Allahabad High Court allowing the revision filed by respondent No. 1. The revision was
filed questioning the legality of the order dated 18-11-2000 passed by XIII Additional
District and Sessions Judge, Aligarh in Criminal Revision No.272 of 2000 accepting the
contention that the informant of the case got a false affidavit filed along with protest
petition, and therefore no action could have been taken.
3. Stand taken before the learned Sessions Judge was that by the time the protest petition
was filed the informant had died and false affidavit with a thumb impression was filed.
Since the informant had already died, the learned Magistrate could not have been
proceeded in the matter. This found acceptance by the learned Sessions Judge. The High
Court by the impugned order had held that the order was not passed on the protest
petition and was in fact passed on consideration of the report submitted in terms of
Section 173 of the Code of Criminal Procedure, 1973 (in short 'Cr.P.C.').
4. Learned counsel for the appellants submitted that the High Court fell in grave error by
holding that the filing of false affidavit, if any, along with protest petition was immaterial.
According to him, when the learned Magistrate acted upon the protest petition, the view
that the affidavit along with the protest petition was not of any consequence, cannot be
maintained.
5. Learned counsel for the respondents on the other hand submitted that a bare reading of
the order passed by learned Magistrate shows that the order did not have its foundation
on the protest petition, but was relatable to the report submitted under Section 173 Cr.P.C.
6. The only question that falls for consideration is whether the order was passed by
learned Magistrate on protest petition or on the police report.
@page-SC1266
7. Reference may be made to a judgment of this Court in Abhinandan Jha and Ors. v.
Dinesh Mishra (AIR 1968 SC 117) where it was held as follows :
"8. It is now only necessary to refer to Section 190, occurring in Chapter XIV, relating to
jurisdiction of Criminal courts in inquiries and trials. That section is to be found under the
heading "Conditions requisite for initiation of proceedings" and sub-section (1) is as
follows :
(1) Except as hereinafter provided, any Presidency Magistrate, District Magistrate or
Sub-divisional Magistrate, and any other Magistrate specially empowered in this behalf,
may take cognizance of any offence
(a) upon receiving a complaint of facts which constitute such offence;
(b) upon a report in writing of such facts made by any police-officer;
(c) upon information received from any person other than a police-officer, or upon his
own knowledge or suspicion, that such offence has been committed."
9. From the foregoing sections, occurring in Chapter XIV, it will be seen that very
elaborate provisions have been made for securing that an investigation does take place
into a reported offence and the investigation is carried out within the limits of the law,
without causing any harassment to the accused and is also completed without
unnecessary or undue delay. But the point to be noted is that the manner and method of
conducting the investigation, are left entirely to the police, and the Magistrate, so far as
we can see, has no power under any of these provisions, to interfere with the same. If, on
investigation, it appears to the officer, in-charge of a police station, or to the officer
making an investigation, that there is no sufficient evidence or reasonable grounds of
suspicion justifying the forwarding of an accused to a Magistrate, S. 169 says that the
officer shall release the accused, if in custody, on his executing a bond to appear before
the Magistrate. Similarly, if, on the other hand, it appears to the officer, in-charge of a
police station, or to the officer making the investigation, under Chapter XIV, that there is
sufficient evidence or reasonable ground to justify the forwarding of an accused to a
Magistrate, such an officer is required, under S. 170, to forward the accused to a
Magistrate or, if the offence is bailable, to take security from him for his appearance
before such Magistrate. But, whether a case comes under S. 169, or under S. 170, of the
Code, on the completion of the investigation, the police officer has to submit a report to
the Magistrate, under S. 173, in the manner indicated therein, containing the various
details. The question as to whether the Magistrate has got power to direct the police to
file a charge - sheet, on receipt of a report under S. 173 really depends upon the nature of
the jurisdiction exercised by a Magistrate, on receiving a report.
xx xx xx
12. Though it may be that a report submitted by the police may have to be dealt with
judicially, by a Magistrate, and although the Magistrate may have certain supervisory
powers, nevertheless, we are not inclined to agree with the further view that from these
considerations alone it can be said that when the police submit a report that no case has
been made out for sending up an accused for trial, it is open to the Magistrate to direct the
police to file a charge-sheet. But, we may make it clear, that this is not to say that the
Magistrate is absolutely powerless, because, as will be indicated later, it is open to him to
take cognizance of an offence and proceed, according to law. We do not also find any
such power, under Section 173(3), as is sought to be inferred, in some of the decisions
cited above. As we have indicated broadly the approach made by the various High Courts
in coming to different conclusions, we do not think it necessary to refer to those decisions
in detail.
13. It will be seen that the Code, as such, does not use the expression 'charge-sheet' or
'final report'. But it is understood, in the Police Manual containing Rules and Regulations,
that a report by the police, filed under Section 170 of the Code, is referred to as a 'charge-
sheet'. But in respect of the reports sent under Section 169 i.e. when there is no sufficient
evidence to justify the forwarding of the accused to a Magistrate, it is termed variously, in
different States, as either 'referred charge', 'final report', or 'summary'.
xx xx xx
17. We have to approach the question, arising for consideration in this case, in the light of
the circumstances pointed out above. We have already referred to the scheme of
@page-SC1267
Chapter XIV, as well as the observations of this Court in Rishbud and Inder Singh's Case
(AIR 1955 SC 196) that the information of the opinion as to whether or not there is a case
to place the accused on trial before a Magistrate, is left to the officer in-charge of the
police station. There is no express power, so far as we can see, which gives jurisdiction to
pass an order of the nature under attack; nor can any such powers be implied. There is
certainly no obligation, on the Magistrate, to accept the report, if he does not agree with
the opinion formed by the police. Under those circumstances, if he still suspects that an
offence has been committed, he is entitled, notwithstanding the opinion of the police, to
take cognizance, under S. 190(1) (c) of the Code. That provision, in our opinion, is
obviously intended to secure that offences may not go unpunished and justice may be
invoked even where persons individually aggrieved are unwilling or unable to prosecute,
or the police, either wantonly or through bona fide error, fail to submit a report, setting
out the facts constituting the offence. Therefore, a very wide power is conferred on the
Magistrate to take cognizance of an offence, not only when he receives information about
the commission of an offence from a third person, but also where he has knowledge or
even suspicion that the offence has been committed. It is open to the Magistrate to take
cognizance of the offence, under S. 190(1) (c), on the ground that, after having due regard
to the final report and the police records placed before him, he has reason to suspect that
an offence has been committed. Therefore, these circumstances will also clearly negative
the power of a Magistrate to call for a charge-sheet from the police, when they have
submitted a final report. The entire scheme of Chapter XIV clearly indicates that the
formation of the opinion, as to whether or not there is a case to place the accused for trial,
is that of the officer incharge of the police station and that opinion determines whether
the report is to be under S. 170, being a 'charge-sheet', or under S. 169, 'a final report'. It
is no doubt open to the Magistrate, as we have already pointed out, to accept or disagree
with the opinion of the police and, if he disagrees, he is entitled to adopt any one of the
courses indicated by us. But he cannot direct the police to submit a charge-sheet, because,
the submission of the report depends upon the opinion formed by the police, and not on
the opinion of the Magistrate. The Magistrate cannot compel the police to form a
particular opinion, on the investigation, and to submit a report, according to such opinion.
That will be really encroaching on the sphere of the police and compelling the police to
form an opinion so as to accord with the decision of the Magistrate and send a report,
either under S. 169, or under S. 170, depending upon the nature of the decision. Such a
function has been left to the police, under the Code."
8. As the factual position goes to show the order passed by learned Magistrate was in
consideration of the police report and was not relatable to the protest petition. That being
so, the view of the High Court does not suffer from any infirmity and no interference is
called for.
9. The appeal is dismissed.
Appeal dismissed.
AIR 2008 SUPREME COURT 1267 "B. K. Sri Harsha v. M/s. Bharath Heavy Electricals
Ltd."
(From : Karnataka)*
Coram : 2 Dr. A. PASAYAT AND P. SATHASIVAM, JJ.
Civil Appeal Nos. 6329-6330 of 2004, D/- 8 -2 -2008.
B.K. Sri Harsha (D) by L.R. and Anr. v. M/s. Bharath Heavy Electricals Ltd.
Civil P.C. (5 of 1908), S.96, O.41, R.1 - APPEAL - CONTRACT - DECREE -
DISMISSAL - Appeal - Manner of disposal - Appeal against decree in suit for specific
performance - Dismissal by High Court in exercise of powers under O.41, R.1, C. P. C. -
High Court's judgment showing that there was no serious effort to analyse various points
raised - High Court has also given finding regarding adverse possession in a suit for
specific performance - Thus, there is total non-application of mind - Therefore, manner in
which appeals were dismissed was not proper - Matter remitted to High Court for fresh
consideration.
R. F. A. No. 608 with 609 of 2002, D/-4-6-2003 (Kant.), Reversed. (Paras 9, 10)
Cases Referred : Chronological Paras
(2005)7 SCC 60 (Ref.) 8
K. Parasaran, Sr. Advocate, S.K. Kulkarni,
@page-SC1268
M. Gireesh Kumar and Vijay Kumar, with him for Appellants; B. Dutta, A.S.G., B.K.
Satija and Banamali Shukla, with him for Respondent.
* R.F.A. No. 608 with 609 of 2002, D/- 4-6-2003 (Kant.)
Judgement
1. Dr. ARIJIT PASAYAT, J. :- Challenge in these appeals is to the judgment of the learned
Single Judge of the Karnataka High Court dismissing the First Appeals filed under
Section 96 of the Code of Civil Procedure, 1908 (in short 'CPC'). The First Appeals were
filed against the judgment and decree passed in OS No.285/1984 and OS No.286/1984 on
the file of XXXI Additional City Civil Judge, Bangalore, decreeing the suit for specific
performance.
2. Background facts in a nutshell are as follows :
Two suits were filed by the respondent, which were consolidated. The respondents as
plaintiff sought specific performance in respect of certain properties which were allegedly
agreed to be sold by the appellants Smt. B. Saroja Devi and her husband Sri B.K. Harsha
under the contract. Since the validity and the genuineness of two agreements Exhibits P-l
and P-2 were not disputed, the Trial Court was of the view that the only question which
arises for consideration was whether the respondent was entitled to the relief of specific
performance. The Trial Court came to hold that the respondent-company was always
ready and willing to perform its part of the contract. It was also found that the two
agreements were never revoked or cancelled by the appellants at any time. Further, it was
held that the suit for specific performance was filed within the period of limitation. The
Trial Court further came to hold that the respondent-company being in possession of the
suit property from 2-5-1974, equality lies in its favour in granting specific performance
and more so, when major portion of the agreement consideration had already been paid.
Therefore, both these suits were decreed. The High Court as noted above, dismissed the
First Appeals.
3. Learned counsel for the appellants submitted that though the judgment and decree
impugned run to several pages, major part of it consists of the averments and
reproduction of the part of the trial Court's judgment.
4. It is submitted that this was not an appropriate way for disposal of the First Appeals. It
was also submitted that some of the conclusions were legally unsupportable. It was
submitted that when the suit is for specific performance, the special features of such a suit
have to be kept in view, which has not been done.
5. Learned counsel for the respondent on the other hand submitted that the High Court
had referred to the factual and legal position in detail and, therefore, the judgment and
decree do not suffer from any infirmity to warrant interference.
6. It is to be noted that pages 4 to 18 of judgment (in the paper book) are quotations from
the Trial Court's judgment. The quotation was made after briefly referring to the major
issues. Up to page 21 contentions were noted. Learned Single Judge dismissed the
appeals in purported exercise of power under Order 41, Rule 1, CPC. Though strictly
speaking, the judgment cannot be said to be in limine dismissal of the appeals, yet the
manner of disposal of the First Appeals leaves much to be desired. When triable issues
are involved, the appeals should not be summarily dismissed or disposed of in the manner
done. The suit was for specific performance and the Trial Court recorded findings about
adverse possession. That being so, according to learned counsel for the appellants, triable
issues are involved. It was pointed out that the stand of the appellants was relatable to
lack of readiness, alleged laches, limited novation.
7. A bare reading of the High Court's judgment shows that there was no serious effort to
analyse the various points raised. It was submitted that there was notice terminating the
agreement. It was categorically stated that plaintiffs were never ready and willing to fufill
their part of contract.
8. The nature of suit for specific performance of contract has been highlighted by this
Court in several cases. In Rajeshwari v. Puran Indoria (2005 (7) SCC 60), it was inter alia
observed as under :
"5. Normally, a suit for specific performance of an agreement for sale of immovable
property involves the question whether the plaintiff was ready and willing to perform his
part of the contract in terms of Section 16 of the Specific Relief Act, whether
@page-SC1269
it was a case for exercise of discretion by the court to decree specific performance in
terms of Section 20 of the Specific Relief Act and whether there were laches on the part
of the plaintiff in approaching the court to enforce specific performance of the contract.
In some cases, a question of limitation may also arise in the context of Article 54 of the
Limitation Act on the terms of the agreement for sale. Other questions like the
genuineness of the agreement, abandoning of the right to specific performance, a
novation and so on, may also arise in some cases. No doubt, a finding on the three
primary aspects indicated earlier would depend upon the appreciation of the pleadings
and the evidence in the case in the light of the surrounding circumstances.
6. The right to specific performance of an agreement for sale of immovable property,
when filed, raises questions of substantial importance between the parties as to whether
the plaintiff has satisfied the requirements of Section 16 of the Specific Relief Act,
whether it is a case in which specific performance of the contract is enforceable in terms
of Section 10, whether in terms of Section 20 of the Act, the discretion to decree specific
performance should be exercised by the court and in some cases, whether the suit was
barred by limitation and even if not, whether the plaintiff has been guilty of negligence or
laches disentitling him to a decree for specific performance. These questions, by and
large, may not be questions of law of general importance. But they cannot also be
considered to be pure questions of fact based on an appreciation of the evidence in the
case. They are questions which have to be adjudicated upon, in the context of the relevant
provisions of the Specific Relief Act and the Limitation Act (if the question of limitation
is involved). Though an order in exercise of discretion may not involve a substantial
question of law, the question whether a court could, in law, exercise a discretion at all for
decreeing specific performance, could be a question of law that substantially affects the
rights of parties in that suit."
9. The High Court has also given a finding regarding adverse possession in a suit for
specific performance. Above being the position, there is total non-application of mind.
The manner in which the appeals were dismissed cannot be said to be proper.
10. Above being the position, the impugned judgment deserves to be set aside. The matter
is remitted to the High Court to consider the matter afresh. The appeals are accordingly
disposed of. There shall be no order as to costs.
Order accordingly.
AIR 2008 SUPREME COURT 1269 "State of U. P. v. Ajai Kumar"
(From : Allahabad)*
Coram : 2 Dr. A. PASAYAT AND P. SATHASIVAM, JJ.
Criminal Appeal No. 277 of 2008 (arising out of SLP (Cri.) No. 2374 of 2007), D/- 7 -2
-2008.
State of U.P. v. Ajai Kumar.
Criminal P.C. (2 of 1974), S.378 - APPEAL - APPELLATE COURT - Appeal against
acquittal - Accused alleged to have snactched away money after firing shots - Injured
witnesses categorically implicating accused - Part of money also recovered from accused
- Accused acquitted on ground that victim could not give number of currency notes -
Money recovered however, bearing stamp of bank from which it was withdrawn -
Appellate Court only on basis of telegram sent by relative of accused that his arrest was
postdated by Police refused to interfere with acquittal - Fact that currency notes had
stamp of bank - Lost sight of - Order liable to be set aside.
G. A. No. 58 of 2003, D/- 4-9-2006 (All). Reversed. (Para 8)
Cases Referred : Chronological Paras
2004 AIR SCW 751 : AIR 2004 SC 1794 : 2004 Cri LJ 1385 5
2004 AIR SCW 4321 : AIR 2004 SC 4520 : 2004 Cri LJ 3842 (Rel. on) 5
S. G. Hussain, Sr. Advocate, Manoj K. Mishra, Anil Kumar Jha, for Appellant; Mrs. K.
Sarda Devi, for Respondent.
* G.A. No. 58 of 2003, D/- 4-9-2006 (All.)
Judgement
1. Dr. ARIJIT PASAYAT, J. :-Leave granted.
2. Challenge in this appeal is to the order passed by a Division Bench of the
@page-SC1270
Allahabad High Court dismissing the appeal filed by the appellant-State questioning
correctness of the order of acquittal recorded by the trial Court. Originally, three persons
apart from respondents were arrayed as accused persons. Two of them expired before trial
was concluded and one had absconded and could not be arrested.
Four persons faced trial for offences punishable under Sections 394, 307, 411 of the
Indian Penal Code, 1860 (in short the 'IPC'). The allegation was that on 15.3.1994 while
the informant along with one Sushil Kumar, son of the owner Shri Gopal was going
towards the shop after withdrawing Rs.1,25,000/- from the State Bank of India, the
accused persons forcibly snatched away the money after firing shots from the pistols held
by them. The informant and aforesaid Sushil Kumar suffered injuries and were taken to
hospital for treatment. The first information report was lodged and investigation was
undertaken and part of the money was recovered from the accused persons. Several
witnesses were examined to further the prosecution version.
PWs 1 and 2 i.e. Bhagwat Narain and Sushil Kumar were stated to have sustained injuries
in the incident. The trial Court directed acquittal primarily on the ground that the
witnesses could not say definitely as regards the numbers on currency notes which were
stated to have been withdrawn from the bank and to have been robbed by the accused
persons. This was highlighted to show the fallacy of the conclusions to direct acquittal.
Several other factors were also indicated questioning correctness of the decision. Appeal
was filed with leave of the High Court and same was dismissed with the following
observations:
"........We have perused the judgment. A perusal of which would indicate that Prem
Narayan the relative of Chandesh Ravat (dead) has made a telegram on 17.3.1994 to the
Senior Superintendent of Police concerned to the effect that Chandesh Ravat was arrested
by the police of Mahurani from his house and the arrest has shown by the police is
20.3.1994, therefore, the arrest as well as the recovery becomes doubtful.
In above view of the matter no interference in the order of acquittal is warranted.
The leave to appeal is hereby rejected."
3. Learned counsel for the appellant submitted that the High Court has not indicated the
basis for coming to the conclusion that the trial Court was right. In fact there was no
analysis of the evidence of the victims who had categorically implicated the accused
persons and had also described in detail the respective role played by each.
4. Learned counsel for the respondent on the other hand submitted that the order of
acquittal was reinforced by the order of dismissal of the appeal by the impugned order
and no interference is therefore called for.
5

. While dealing with leave to appeal against acquittal, this Court in State of Rajasthan v.
Sohan Lal (2004 (5) SCC 573) inter alia observed as under : 2004 AIR SCW 4321

"3. We have carefully considered the submissions of the learned counsel appearing on
either side. This Court in State of Orissa v. Dhaniram Luhar (2004 (5) SCC 568) has
while reiterating the view expressed in the earlier cases for the past two decades
emphasised the necessity, duty and obligation of the High Court to record reasons in
disposing of such cases. The hallmark of a judgment/order and exercise of judicial power
by a judicial forum is to disclose the reasons for its decision and giving of reasons has
been always insisted upon as one of the fundamentals of sound administration justice-
delivery system, to make known that there had been proper and due application of mind
to the issue before the Court and also as an essential requisite of principles of natural
justice. The fact that the entertaining of an appeal at the instance of the State against an
order of acquittal for an effective consideration of the same on merits is made subject to
the preliminary exercise of obtaining of leave to appeal from the High Court, is no reason
to consider it as an appeal of any inferior quality or grade, when it has been specifically
and statutorily provided for, or sufficient to obviate and dispense with the obvious
necessity to record reasons. Any judicial power has to be judiciously exercised and the
mere fact that discretion is vested with the court/forum to exercise the same either way
does not constitute any licence 2004 AIR SCW 751

@page-SC1271
to exercise it at whims or fancies and arbitrarily as used to be conveyed by the well-
known saying: "varying according to the Chancellor's foot". Arbitrariness has been
always held to be the anathema of judicial exercise of any power, all the more so when
such orders are amenable to challenge further before higher forums. The State does not in
pursuing or conducting a criminal case or an appeal espouse any right of its own but
really vindicates the cause of society at large, to prevent recurrence as well as punish
offences and offenders respectively, in order to preserve orderliness in society and avert
anarchy, by upholding the rule of law. The provision for seeking leave to appeal is in
order to ensure that no frivolous appeals are filed against orders of acquittal, as a matter
of course, but that does not enable the High Court to mechanically refuse to grant leave
by mere cryptic or readymade observations, as in this case ("the court does not find any
error"), with no further, on the face of it, indication of any application of mind
whatsoever. All the more so, when the orders of the High Court are amenable to further
challenge before this Court. Such ritualistic observations and summary disposal which
has the effect of, at times, and as in this case, foreclosing statutory right of appeal, though
a regulated one, cannot be said to be a proper and judicial manner disposing of
judiciously the claim before courts. The giving of reasons for a decision is an essential
attribute of judicial and judicious disposal of a matter before courts, and which is the only
indication to know about the manner and quality of exercise undertaken, as also the fact
that the court concerned had really applied its mind. All the more so, when refusal of
leave to appeal has the effect of foreclosing once and for all a scope for scrutiny of the
judgment of the trial court even at the instance and hands of the first appellate court. The
need for recording reasons for the conclusion arrived at by the High Court, to refuse to
grant leave to appeal, in our view, has nothing to do with the fact that the appeal
envisaged under Section 378 Cr.P.C is conditioned upon the seeking for and obtaining of
the leave from the Court. This Court has repeatedly laid down that as the first appellate
court the High Court, even while dealing with an appeal against acquittal, was also
entitled, and obliged as well, to scan through and if need be reappreciate the entire
evidence, though while choosing to interfere only the court should find an absolute
assurance of the guilt on the basis of the evidence on record and not merely because the
High Court could take one more possible or a different view only. Except the above,
where the matter of the extent and depth of consideration of the appeal is concerned, no
distinctions or differences in approach are envisaged in dealing with an appeal as such
merely because one was against conviction or the other against an acquittal."
(Underlined for emphasis)
6. In view of the fact that the High Court's conclusion is clearly presumptuous and the
mere claim that a telegram was sent by a relative of Chandesh Ravat the deceased-
accused, same could not have been a ground to hold that the prosecution version was
unacceptable and the trial Court had rightly directed acquittal.
7. Learned counsel for the respondent on the other hand submitted that not only on the
ground of a telegram but also on other grounds, the High Court upheld the view of the
trial Court.
8. A bare reading of the impugned order which is reproduced above goes to show that the
only ground on which the High Court found that there was no scope for interference was
the telegram sent by a relative. Various other factors which throw light on the controversy
have not been considered in the proper perspective by the High Court. The effect of the
evidence of the two victims and the recovery of part of the recovered amount has been
completely lost sight of. It is to be noted that contrary to what the trial Court and the High
Court noted, the seized recovery notes clearly show the stamp of the bank from where the
money was withdrawn. The relevance of this factor has been completely lost sight of by
the trial Court and the High Court.
9. That being so, we set aside the impugned order of the High Court and remit the matter
to it for consideration in accordance with law.
10. The appeal is allowed.
Appeal allowed.
@page-SC1272
AIR 2008 SUPREME COURT 1272 "Barkat Ali v. Badri Narain"
(From : AIR 2001 Raj 51)
Coram : 2 Dr. A. PASAYAT AND P. SATHASIVAM, JJ.
Civil Appeal No. 1383 of 2002, D/- 6 -2 -2008.
Barkat Ali and Anr. v. Badri Narain (D) by L.Rs.
Civil P.C. (5 of 1908), O.21, R.22, R.23, S.11 - EXECUTION - RES JUDICATA -
DECREE - ATTACHMENT - Execution - Objection at subsequent stage - Res judicata -
Execution of decree - Issuance of warrant of attachment - Objection raised against
execution by judgment debtor - Failure on part of judgment debtor to raise objection
earlier thereby allowing preliminary stage to come to an end - Unless there is
modification of order, objection at subsequent stage after issuance of warrant of
attachment would be barred by principles of constructive res jndicata.
Order XXI, Rule 22, C. P. C. culminates in end of one stage before attachment of the
property can take place in furtherance of execution of decree. The proceedings under
Order XXI, Rule 23 can only be taken if the executing Court either finds that after issuing
notice, under Order XXI Rule 21 the judgment debtor has not raised any objection or if
such objection has been raised, the same has been decided by the executing Court. Sub-
rule (1) as well as sub-rule (2) under Order XXI, Rule 22, operates simultaneously on the
same field. Sub-rule (1) operates when no objection is filed. Then the Court proceeds and
clears the way for going to the next stage of the proceedings, namely, attachment of the
property and if the Court finds objections on record then it decides the objections in the
first instance and thereafter clears the way for taking up the matter for attachment of the
property if the objections have been overruled. Whether the order is made under sub-rule
(1) or sub-rule (2), it has the effect of determining the preliminary stage before the
attachment process is set in motion. In this background, the order of the Court to proceed
with attachment on finding that no objection has been raised also operates as an order
deciding the preliminary stage of the execution proceedings and operates as if the
judgment debtor has no objection to file. If thereafter, the judgment debtor wants to raise
an objection in the same proceedings in the absence of any modification of order passed
under Order XXI, Rule 22, sub-rule (1) or (2), he has to take recourse to get rid of the
order by way of appeal. There is no dispute and it has not been agitated that the order for
proceeding by the judgment under Order XXI, Rule 22 amounts to a decree under Section
47 of C.P.C. and it is appealable as a decree i.e. to say it is not an appeal against the
interim order but an appeal against the decree which is provided against the final order. It
means that at the different stages of the execution, orders passed by the executing Court
have attained finality unless they are set aside by way of appeal before the higher forum.
Otherwise they bind the parties at the subsequent stage of the execution proceedings so
that the smooth progress of execution is not jeopardised and the stage which reached the
finality by dint of various orders of the Order XXI, operates as res judicata for the
subsequent stage of the proceedings, where a judgment debtor has an opportunity to raise
an objection which he could have raised but failed to take and allowed the preliminary
stage to come to an end for taking up the matter to the next stage for attachment of
property and sale of the property under Order XXI Rule 23 which fell within the above
principle, the judgment debtor thereafter cannot raise such objections subsequently and
revert back to earlier stage of proceedings unless the order resulting in termination of
preliminary stage which amounts to a decree is appealed against and order is set aside or
modified. The principles of res judiciata not only apply in respect of separate proceedings
but the general principles also apply at the subsequent- stage of the same proceedings
also.
AIR 1964 SC 993; AIR 1960 SC 941, Relied on. (Paras 7, 8)
Cases Referred : Chronological Paras
AIR 1964 SC 993 (Rel. on) 9
AIR 1960 SC 941 (Rel. on) 10
Puneet Jain, Ms. Christi Jain, Piyush Jain, H. D. Thanvi and Sushil Kumar Jain, for
Appellants.
Judgement
Dr. ARIJIT PASAYAT, J. :- Challenge in this appeal is to the order passed by a Division
Bench of the Rajasthan High Court at Jodhpur dismissing the special appeal filed under
Section 18 of the Rajasthan High Court Ordinance 1949 (in short 'Ordinance') against
judgment of learned Single Judge
@page-SC1273
dated 16.1.1981 in SB Civil Misc. first appeal No.5/75.
2. Background facts in a nutshell are as under :
The respondents are legal representatives of the decree holder Badrinarain and the
appellants are the legal representatives of the judgment-debtor Abdul Ghani. The said
Badrinarain obtained a decree against Abdul Ghani in a mortgage suit on 11.5.1952 in
which an amount of Rs.11,194.25/- was determined as payable by the said Abdul Ghani
from the date of final decree. Successive execution applications were filed for recovering
the said sum. First application for execution was filed on 7.10.1952 in which proceedings
the decree was partially satisfied. The proceedings ended on 21.12.1956. The second
execution resulted in further partial satisfaction. The said execution terminated on
25.9.1957. The third execution application which was filed on 20th May 1958 resulted in
further partial satisfaction of the decree and the said proceedings ended on 6.8.1960. The
present execution application for the recovery of remainder sum was filed on 30th
January, 1971. The notice of the application was issued to all the appellants and another
son who was reported to be dead by the process server. The appellant No. 1 accepted
service on behalf of appellant Nos.2 and 3, who were then minors. The notice was served
on 20.4.1972 for hearing on 3.6.1972. An appearance was filed by the counsel on
3.6.1972, who sought time to file objections which was granted and the proceedings were
adjourned to 5.8.1972. On 5.8.1972, again adjournment was sought which was granted
and the case was adjourned to 12.8.1972. On 12.8.1972 also, the proceedings could not
proceed further because the learned Presiding Judge was on leave and the case was
adjourned to 16.9.1972. On 16.9.1972, the Court finding that no objections have been
filed till then by the judgment-debtors, the decree holder was directed to file expense for
carrying out attachment within five days on the submission of which the warrants of
attachment could be issued and the proceedings were adjourned to 21.9.1972. The
attachment warrant was not issued prior to 21.9.1972. On finding that expenses for
attachment has been filed, the executing Court ordered for the issuance of warrant of
attachment on 21.9.1972. After issuance of warrant of attachment, the objections were
filed by the appellant on 21.9.1972 pleading inter alia that the execution proceedings
were barred by time and that amount for which the execution was sought was also not
correctly stated. The executing Court found that since after completing preliminaries of
issuing notice and finding that no objection has been filed in spite of the service under
Order XXI, Rule 22 of the Code of Civil Procedure, 1908 (in short 'CPC') and the Court
had proceeded to next stage of execution for attaching the property under Order XXI,
Rules 23 and 24 of C.P.C., any objection raised subsequent thereto cannot be entertained
being barred by principles of constructive res judicata. Against the dismissal of the
objections dated 16.11.1972 by order dated 13.7.1974, an appeal was, preferred before
the High Court which has been dismissed by the Learned Single Judge by judgment dated
16.1.1981. The Learned Single Judge found that the objections filed on 16.11.1972, after
the warrant of attachment was issued, could not be entertained by the executing Court as
the same was barred by principles of constructive res judicata. Ancillary issues raised by
the Learned Counsel for the appellant were also found to be not sustainable and the
appeal was dismissed.
3. As noted above, learned Single Judge found that the objection filed after issuance
warrant of attachment could not be entertained by the executing Court as the same was
barred by principles of constructive res judicata.
4. The same contention was raised before the Division Bench which did not find any
substance.
5. Learned counsel for the appellant reiterated the stand taken before the learned Single
Judge and the Division Bench.
6. There is no appearance on behalf of the respondent.
7. Order XXI, Rule 22, CPC culminates in end of one stage before attachment of the
property can take place in furtherance of execution of decree. The proceedings under
Order XXI, Rule 23 can only be taken if the executing Court either finds that after issuing
notice, under Order XXI, Rule 21 the judgment-debtor has not raised any objection or if
such objection has been raised, the same has been decided by the executing Court. Sub-
rule (1) as well as sub-rule (2) under Order XXI, Rule 22, operates simultaneously
@page-SC1274
on the same field. Sub-rule (1) operates when no objection is filed. Then the Court
proceeds and clears the way for going to the next stage of the proceedings namely
attachment of the property and if the Court finds objections on record then it decides the
objections in the first instance and thereafter clears the way for taking up the matter for
attachment of the property if the objections have been overruled. Whether the order is
made under sub-rule (1) or sub-rule (2), it has the effect of determining the preliminary
stage before the attachment process is set in motion. In this background, the order of the
Court to proceed with attachment on finding that no objection has been raised also
operates as an order deciding the preliminary stage of the execution proceedings and
operates as if the judgment-debtor has no objection to file. If thereafter, the judgment-
debtor wants to raise an objection in the same proceedings in the absence of any
modification of order passed under Order XXI, Rule 22, sub-rule (1) or (2), he has to take
recourse to get rid of the order by way of appeal. There is no dispute and it has not been
agitated that the order for proceeding by the judgment under Order XXI, Rule 22
amounts to a decree under Section 47 of CPC and it is appealable as a decree i.e to say it
is not an appeal against the interim order but an appeal against the decree which is
provided against the final order. It means that at the different stages of the execution
orders passed by the executing court have attained finality unless they are set aside by
way of appeal before the higher forum. Otherwise they bind the parties at the subsequent
stage of the execution proceedings so that the smooth progress of execution is not
jeopardised and the stage which reached the finality by dint of various orders of the Order
XXI, operates as res judicata for the subsequent stage of the proceedings. Since the order
passed at different stage itself operates as a decree and is appealable as such, the same
cannot be challenged in appeal against subsequent orders also, because appeal against an
order passed under Order XXI, Rule 22 does not amount to appeal against order at initial
stage, but amounts to a decree finally determining the question. That is why no appeal
against orders made under Order XXI has been provided under Order 43. In this
background, where a judgment-debtor has an opportunity to raise an objection which he
could have raised but failed to take and allowed the preliminary stage to come to an end
for taking up the matter to the next stage for attachment of property and sale of the
property under Order XXI, Rule 23 which fell within the above principle, the judgment-
debtor thereafter cannot raise such objections subsequently and revert back to earlier
stage of proceedings unless the order resulting in termination of preliminary stage which
amounts to a decree is appealed against and order is set aside or modified.
8. The principles of res judicata not only apply in respect of separate proceedings but the
general principles also apply at the subsequent stage of the same proceedings also and the
same Court is precluded to go into that question again which has been decided or deemed
to have been decided by it at an early stage.
9. In Arjun Singh v. Mohindra Kumar and Ors. (AIR 1964 SC 993) it was observed as
follows :
"Scope of principle of res judicata is not confined to what is contained in Section 11 but
is of more general application. Again, res judicata could be as much applicable to
different stages of the same suit as to findings on issues in different suits. Where the
principles of res judicata is invoked in the case of the different stages of proceedings in
the same suit, the nature of the proceedings, the scope of the enquiry which the adjectival
law provides, the decision being reached, as well as the specific provisions made on
matters touching such decision are some of the material and the relevant factors to be
considered before the principle is held applicable."
10

. In Satyadhyan Ghosal and Ors. v. Smt. Deorajin Debi and Ann (AIR 1960 SC 941) it
was observed as follows : Para 8 of AIR

"The principle of res judicata applies also as between two stages in the same litigation to
this extent that a court, whether the Trial Court or a Higher Court having at an earlier
stage decided a matter in one way will not allow the parties to re-agitate the matter again
at a subsequent stage of the same proceedings."
11. Above being the position, the High Court was justified in dismissing the special
appeal and in confirming the order of learned Single Judge. The appeal is without merit,
deserves dismissal, which we direct.
Appeal dismissed.
@page-SC1275
AIR 2008 SUPREME COURT 1275 "Thimmaiah v. Shabira"
(From : Karnataka)*
Coram : 2 Dr. A. PASAYAT AND P. SATHASIVAM, JJ.
Civil Appeal No. 831 of 2002, D/- 6 -2 -2008.
Sri. Thimmaiah v. Shabira and Ors.
Specific Relief Act (47 of 1963), S.38 - INJUNCTION - POSSESSION - Permanent
Injunction - Suit by plaintiff to restrain defendants from Interfering with peaceful
possession and enjoyment of property - Finding by Trial court that plaintiffs failed to
prove their possession - Grant of relief to plaintiff by High Court without considering
effect of these findings and without recording any finding regarding possession - Illegal.
R. P. A. No. 598 of 1998, D/- 18-7-2001 (Kant.), Reversed. (Paras 6, 7)

Vikas Rojipura and F. C. Vidya Sagar, for Appellant; S. N. Bhat, N. P. S. Panwar, D. P.


Chaturvedi and Ms. Kiran Suri, for Respondents.
* R.F.A. No. 598 of 1998, D/- 18-7-2001 (Kant).
Judgement
1. Dr. ARIJIT PASAYAT, J. :-Heard learned counsel for the parties.
2. Challenge in this appeal is to the judgment of a learned Single Judge of the Karnataka
High Court allowing the First Appeal filed by the respondents under Section 96 of the
Code of Civil Procedure, 1908 (in short the 'CPC').
3. The factual background needs to be noted in brief :
The appeal before the High Court was by the plaintiffs who are respondents in the present
appeal.
The plaintiffs 1 and 2 are the wife and husband. According to the plaintiffs, the 1st
plaintiff purchased site No.43 in survey No.37 of Avalahalli Village, Bangalore South
Taluk, measuring East to West 45' and North to South 30' and bounded on East by 5th
Main Road, on the West by Site No.46, on the North by Site No.42 and on the South by
Site No.44. According to them, the 2nd defendant (respondent No.3 herein) sold the
property as power of attorney holder of one Narayana Rao in favour of the 1st plaintiff
under a registered sale deed dated 7.6.1984. At the time of purchase, a temporary
structure was there on the property and with an intention to construct a new building, they
pulled down the temporary structure. When the plaintiffs started demolishing the said
structure, the 1st. defendant (appellant herein) made an attempt to interfere with the
peaceful possession and enjoyment of the property and that under the guise of purchasing
of a site No.42, the 1st defendant also made an attempt to encroach on the plaintiffs
property. Therefore, the plaintiffs filed a suit for judgment and decree for permanent
injunction to restrain the defendants from interfering with the peaceful possession and
enjoyment of the property.
The 1st defendant filed the written statement contending that he has purchased the
property from one Nagaraja who is the 3rd defendant and that the 1st defendant is in
possession of site No.42 which is measuring 45' x 60'. Therefore, he requested to dismiss
the suit of the plaintiffs.
The 2nd defendant has supported the case of the plaintiffs. The 3rd defendant has not
filed any written statement. According to the 1st defendant the 3rd defendant is the owner
of the property. Based on the above pleadings the trial Court framed the following
issues :-
(i) Whether the plaintiff is in lawful possession of the plaint schedule property on the date
of filing the suit?
(ii) Whether the illegal interference is proved?
(iii) Whether the plaintiff is entitled to permanent injunction as prayed?"
The 2nd plaintiff has been examined as PW-1. The 2nd defendant-vendor of the 1st
plaintiff has been examined as PW-2 and got marked Ex.P-1 to P-14. On behalf of the
defendants, the 1st defendant has been examined as DW-1. After appreciating the oral
and documentary evidence the trial Court dismissed the suit of the plaintiffs. Against the
said judgment and decree, the First Appeal is filed by the plaintiffs.
It is to be noted that the High Court formulated the following point for determination in
appeal :
"Whether the 1st plaintiff proved that the 2nd defendant had the power to alienate site
No.43 in her favour, and if so, is she entitled for a decree in her favour?"
The High Court allowed the appeal holding that plaintiff No. 1 had proved her case in
respect of Site No.43 in view of Exs. P-1
@page-SC1276
and P-2. Adverse inference was drawn because the defendant No. 1 failed to produce the
power of Attorney executed by Narayana Rao in favour of 3rd defendant.
4. In support of the appeal, though many points were urged, the primary stand was that in
a suit for permanent injunction, the foundational fact which had to be established was
possession. In the instant case, the trial Court while answering Issue Nos. 1 and 3
categorically held that the plaintiffs had failed to prove their possession. There is no
finding recorded by the High Court regarding possession and even while formulating the
point for determination the question of possession was not considered.
5. Per contra, learned counsel for the respondents submitted that the parties proceeded on
the basis of title and since the trial Court recorded findings regarding possession which
are contrary to the materials on record, the High Court has rightly allowed the appeal.
5A. Undisputedly, the suit was one for permanent injunction and in such a suit the
plaintiff has to establish that he is in possession in order to be entitled to a decree for
permanent injunction. The general proposition is well settled that a plaintiff not in
possession is not entitled to the relief without claiming recovery of possession. Before an
injunction can be granted it has to be shown that the plaintiff was in possession.
6. In the instant case, Issues Nos. 1 and 3 which were framed on 1.10.1988 clearly refer
to this vital aspect. The trial Court while answering the aforesaid issues held in the
negative. Unfortunately, the High Court did not consider the effect of these findings and
even did not record any finding regarding possession. Therefore, as rightly contended by
learned counsel for the appellant, the High Court could not have allowed the appeal. As
noted above, even while formulating the point for determination, the High Court did not
formulate the question relating to possession.
7. In the aforesaid circumstances, we set aside the impugned judgment of the High Court
and remit the matter to the High Court to formulate a definite point relating to possession
and then analyse the evidence on record with reference to that question and decide the
appeal.
8. Since the matter is pending since long, the High Court is requested to dispose of the
appeal as early as practicable preferably by the end of August, 2008.
9. The appeal is allowed to the aforesaid extent without any order as to costs.
Order accordingly.
AIR 2008 SUPREME COURT 1276 "Dharma Naika v. Rama Naika"
(From : Karnataka)*
Coram : 2 TARUN CHATTERJEE AND DALVEER BHANDARI, JJ.
Civil Appeal No. 2802 of 2001, D/- 5 -2 -2008.
Dharma Naika v. Rama Naika and Anr.
(A) Karnataka Scheduled Castes and Scheduled Tribes (Prohibition of Transfer of Certain
Lands) Act (2 of 1979), S.3(e), S.4 - SCHEDULED CASTES AND SCHEDULED
TRIBES - LAND - GOVERNMENT GRANT - SALE DEED - Prohibition on transfer of
granted land - Extends to transfer made before or after enforcement of Act - Agreement
for sale entered after prohibition period stipulated in grant and before Act - Sale deed
executed after Act without permission of State Govt. - Transfer is null and void - Plea that
agreement for sate being also 'transfer' u/S.3(e) it is valid - Not tenable. (Paras 12, 14,
15)
(B) Contract Act (9 of 1872), S.54 - SALE - AGREEMENT TO SELL - CONTRACT -
Sale - Does not include agreement for sale - Agreement to sell does not by itself create
any interest of proposed vendee in immovable property - Agreement for sale is an
executory contract whereas sate is an executed contract. (Para 6)
Cases Referred : Chronological Paras
2004 AIR SCW 5110 : AIR 2004 SC 4342 (Rel. on) 6
AIR 1984 SC 1151 (Disting.) 17
R.S. Hegde (for P.P. Singh), for Appellant; Ms. Vaijayanthi Girish (for T.N. Rao), for
Respondents.
* W.A. No. 3542 of 1998, D/- 12-7-2000 (Kant.)
Judgement
1. TARUN CHATTERJEE, J. :- The subject-matter of this appeal relates to the
Government granted land measuring 2 acres 20 guntas in Sy.No.365/1 (New No.685)
situated at Nayakanahatti Village, Challakera Taluk, District Chitradurga in the State of
Karnataka (hereinafter referred to as the 'scheduled land'). The certificate of grant was
issued on 10th of September, 1955 in favour
@page-SC1277
of one Shri Tejyanaika in which one of the conditions stipulated was that the grantee shall
not alienate the granted land for a period of 15 years.
2. On 10th of September, 1970, the abovementioned period of 15 years during which the
land could not be alienated as per the conditions of the certificate of grant had expired.
After the expiry of the aforesaid period of 15 years, more precisely on 6th of July, 1976,
an agreement to sell was executed by the legal heirs of the original grantee who, in the
meantime, had expired. Under this agreement for sale, the vendors, namely, the legal
heirs of the original grantee had agreed to sell the scheduled land to the father of the
appellant. Subsequent to the agreement for sale and the commencement of the Act, more
precisely on 13th of October, 1986, a deed of sale was executed and registered by the
vendors in respect of the scheduled land. After the registration of the sale deed, the
authority under the Act initiated a proceeding for resumption of the scheduled land as,
according to the authorities, the sale was in violation of the provisions of the Act. This is
because the sale was effected after the commencement of the Act without previous
permission of the Government. An order was passed by the Assistant Commissioner who
had the authority to direct resumption of the land holding that the sale having taken place
on 13th of October, 1986, when the Act had already come into force, the sale deed dated
13th of October, 1986 must be found to be null and void in view of the prohibition
contained in Section 4 of the Act. An appeal was carried against the aforesaid order
before the appellate authority which was also dismissed, inter alia, on the finding that the
title of the scheduled land could not pass under the registered sale deed dated 13th of
October, 1986 as, admittedly, the same was executed and registered after the coming into
force of the Act and therefore, in view of Section 4(2) of the Act, the sale made without
obtaining prior permission of the Government was found to be null and void. Feeling
aggrieved by the concurrent orders of the Assistant Commissioner as well as the appellate
authority, a writ petition was filed before the High Court of Karnataka at Bangalore,
which was dismissed by a learned Judge of the High Court and against that order of the
learned Single Judge, a writ appeal was filed, which was also dismissed. Feeling
aggrieved, this special leave petition was filed in respect of which leave has already been
granted.
3. We have heard the learned counsel for the parties and examined the orders passed by
the High Court as well as the authorities below. On a careful examination of the aforesaid
orders and the materials on record and also the provisions of the Act, including the
objects and reasons for which this Act was introduced, we are of the view that this appeal
is liable to be dismissed for the reasons given hereunder, but before we proceed further,
we may keep it on record that when the agreement for sale dated 6th of July, 1976 was
subsisting, the Karnataka Scheduled Castes and Scheduled Tribes (Prohibition of Transfer
of Certain Lands) Act, 1978 (in short 'the Act') came into force with effect from 1st of
January, 1979.
4. The only question that needs to be decided in this appeal is whether the sale deed,
which was executed and registered after the commencement of the Act but in respect of
which, the agreement for sale was executed before the commencement of the Act, would
be hit by the provisions of Section 4 of the Act. In order to decide this question and
before considering the relevant provisions of the Act with which we would be associated
for a proper decision in this case later, it would be appropriate to reproduce the statement
of objects and reasons of the Act which was introduced by the Karnataka Gazette,
Extraordinary, dated 30th of June, 1978, which reads as under :-
"The non-alienation clause contained in the existing Land Grant Rules and the provisions
for cancellation of grants where the land is alienated in contravention of the abovesaid
provision are found not sufficient to help the Scheduled Castes and Scheduled Tribes
grantees whose ignorance and poverty have been exploited by persons belonging to the
affluent and powerful sections to obtain sales or mortgages either for a nominal
consideration or for no consideration at all and they have become the victims of
circumstances. To fulfill the purposes of the grant, the land even if it has been alienated,
should be restored to the original grantee or his heirs.
The Government of India has also been urging the State Government for enacting a
legislation to prevent alienation of lands granted to Scheduled Castes and Scheduled
Tribes by Government on the lines of the
@page-SC1278
model legislation prepared by it and circulated to the State Government."
A plain reading of the statement of objects and reasons, for which the legislature has
introduced this Act, would show that the non-alienation clause contained in the existing
land grant rules and the provisions for cancellation of grants where the land was alienated
in contravention of the abovesaid provisions were found insufficient to help the
Scheduled Castes and Scheduled Tribes grantees. From the objects and reasons of the
Act, it is evident that ignorance and poverty of the scheduled castes and scheduled tribes
were exploited by persons belonging to the affluent and powerful sections to get sales or
mortgages, either for a nominal consideration or for no consideration at all and on
account of this, the scheduled castes and scheduled tribes had become the victims of
circumstances. It is for this reason and to fulfill the purposes of the grant, it was thought
fit by the legislature that the land, even if it has been alienated, must be restored to the
original grantee or his heirs and legal representatives who are admittedly scheduled castes
and scheduled tribes. It is also evident from the objects and reasons of the Act that the
Central Government was also urging the State Government to enact a legislation to
prevent alienation of lands granted to the scheduled castes and scheduled tribes by the
State Government on the lines of the model legislation prepared by it and circulated to
the State Government. It is in that background, the Act was introduced providing for
prohibition of transfer and restoration of lands granted by the Government to persons
belonging to scheduled castes and scheduled tribes in the State. However, it is also
evident from the relevant provisions of the Act with which we would be dealing with
later that total prohibition of transfer by scheduled castes and scheduled tribes was also
not intended by the legislature. It is provided that in respect of transfers after the
commencement of the Act, it would be open to transfer the land granted to scheduled
castes and scheduled tribes if prior permission is obtained from the State Government.
5. Keeping the objects and reasons for which the Act was introduced in mind for the
purpose of deciding the present appeal and also for the purpose of giving proper
Interpretation to the relevant provisions of the Act with which we are concerned, let us
now deal with certain relevant provisions of the Act. Section 3 of the Act is a defining
section. Section 3(b) defines "Granted Land" which means any land granted by the
Government to a person belonging to any of the Scheduled Castes or the Scheduled
Tribes and includes land allotted or granted to such person under the relevant law for the
time being in force relating to agrarian reforms or land ceilings or abolition of imams,
other than that relating to hereditary offices or rights and the word "granted" shall be
construed accordingly.
6. Next is the definition of "Transfer" under Section 3(e) of the Act. "Transfer" means a
sale, gift, exchange, mortgage with or without possession, lease or any other transaction
not being a partition among members of a family or a testamentary disposition and
includes the creation of a charge or an agreement to sell, exchange, mortgage or lease or
enter into any other transaction.

A bare reading of the definition of "Transfer" as defined in Section 3(e) of the Act would
show that an "agreement for sale" of any "granted land" is included within the meaning of
"Transfer". That being the position, the word "Transfer" as defined under the Act is an
inclusive definition. That is to say, it includes "sale" as well as "agreement for sale",
although an agreement for sale under the Transfer of Property Act is not a transfer and the
right, title or interest in the land does not pass until the sale deed is executed and
registered. "Sale" has been defined in Section 54 of the Transfer of Property Act which
means transfer of ownership in exchange for a price paid or promised or part-paid and
partpromised. As noted herein-earlier, an agreement to sell does not by itself create any
interest of the proposed vendee in the immovable property but only creates an
enforceable right in the parties. (See : Rambhau Namdeo Gajre vs. Narayan Bapuji
Dhotra (Dead) through LRs. [(2004) 8 SCC 614]). Therefore, it is clear that under the
general law, that is, under the Transfer of Property Act, an 'agreement for sale' is not the
same as 'sale' and in the case of an agreement for sale, the title of the property agreed to
be sold still remains with the vendor but in the case of 'sale', title of the property is vested
with the vendee. Therefore, an agreement for sale is an executory contract whereas sale is
an executed contract. 2004 AIR SCW 5110

@page-SC1279
7. Let us now consider Section 4 of the Act, which is the most relevant provision for the
purpose of deciding this appeal. Section 4 deals with prohibition of transfer of granted
lands and reads as under :-
"(1) Notwithstanding anything in any law, agreement, contract or instrument, any transfer
of granted land made either before or after the commencement of this Act, in
contravention of the terms of the grant of such land or the law providing for such grant,
or sub-section (2) shall be null and void and no right, title or interest in such land shall be
conveyed or be deemed ever to have conveyed by such transfer.
(2) No person shall, after the commencement of this Act, transfer or acquire by transfer
any granted land without the previous permission of the Government.
(3) The provisions of sub-sections (1) and (2) shall apply also to the sale of any land in
execution of a decree or order of a civil court or of any award or order of any other
authority."
8. Section 5 deals with resumption and restitution of granted lands. Under this provision,
power has been conferred on the Assistant Commissioner to initiate a proceeding and
hold that the transfer of any granted land is null and void under subsection (1) of Section
4 and if the transfer of such granted land is found to be null and void, he is empowered to
take possession of such land after evicting all such persons in possession thereof in such
manner as may be prescribed under the Act. This power can be exercised by the Assistant
Commissioner on an application by any interested person or on information given in
writing by any person or even suo motu.
9. Section 6 of the Act prohibits registration of transfer of granted lands. It says that
notwithstanding anything in the Registration Act, 1908, on or after the commencement of
this Act, no registering officer shall accept for registration any document relating to the
transfer of, or to the creation of any interest in, any granted land included in a list of
granted lands furnished to the registering officer except where such transfer is in
accordance with this Act or the terms of the grant of such land or the law providing for
such grant. From a bare reading of this provision, it is clear that an embargo has been
placed on the registering officer to accept any document for registration relating to the
transfer of, or to the creation of any interest in, any granted land except where the transfer
is in accordance with the grounds mentioned in the said section.
10. The only other relevant provision to be considered for the purpose of this appeal is
Section 11 of the Act, which reads as under :
"The provision of this Act shall have effect notwithstanding anything inconsistent
therewith contained in any other law for the time being in force or any custom, usage or
contract or any decree or order of a Court, Tribunal or other Authority."
Section 11 of the Act, therefore, provides that the provisions of this Act shall override all
other laws inconsistent with this Act. Since, in this case, we are not concerned with any
inconsistency with any other law for the time being in force or any custom, usage or
contract or any decree or order of a Court, Tribunal or other Authority, it is needless to
deal with this provision in the present case.
11. Keeping these provisions and the objects and reasons of the Act in mind, let us now
deal with the submissions advanced by the learned counsel appearing on behalf of the
appellant. According to the learned counsel for the appellant, having regard to the fact
that the transfer of the granted land was made after the expiry of the prohibited period
and before the coming into force of the Act, such transfer could not be hit by the
provisions contained in Section 4(2) of the Act. In this connection, the learned counsel for
the appellant had drawn our attention to Section 3(e) of the Act, which defines "Transfer".
We have already dealt with the definition of "Transfer" herein earlier. According to the
learned counsel for the appellant, the prohibition imposed under Section 4 of the Act
would not be applicable to the facts of the present case. As noted herein-earlier, the
learned counsel, therefore, submitted that in view of the above, the High Court as well as
the authorities below had committed an error in holding that the sale deed, having been
executed and registered after the commencement of the Act, must be found to be null and
void and that by the said sale deed, the right, title or interest in the granted land must be
restored by the Assistant Commissioner, in the exercise of his power under Section 5 of
the Act, to the respondents.
@page-SC1280
This submission of the learned counsel for the appellant was contested by the learned
counsel appearing for the respondents. According to the learned counsel for the
respondents, the transfer of the granted land must be hit by Section 4 of the Act as,
admittedly, the sale deed was executed and registered after the commencement of the Act.
The learned counsel for the respondents also contended that in view of the prohibition
contained in Section 4 of the Act, even if the transfer was made before the
commencement of the Act in view of the agreement for sale, still since the sale deed was
executed and registered after the commencement of the Act, the same must be hit by
Section 4 of the Act and therefore, no right, title or interest in such granted land shall be
conveyed or be deemed ever to have conveyed by such transfer and that being the
position, no interference could be made with the impugned judgment as well as with the
orders of the authorities.
12. Having heard the learned counsel for the parties and after examining the objects and
reasons and the relevant provisions of the Act, as noted herein-earlier, in depth and in
detail, we have no hesitation to hold that the submissions of the learned counsel for the
appellant cannot at all be accepted. It is true that the agreement for sale in respect of the
granted land was executed before the commencement of the Act. It is also an admitted
position that "Transfer" under the Act includes an agreement to sell as well. Keeping this
fact in mind, let us now see whether in view of Section 4 of the Act, the transfer of the
land, in respect of which the agreement for sale was executed before the commencement
of the Act but which was effected after the commencement of the Act by execution and
registration of the sale deed, could be said to be null and void. Section 4 (1) of the Act in
clear terms provides that notwithstanding anything contained in any law, agreement,
contract or instrument, any transfer of granted land made either before or after the
commencement of the Act in contravention of either (a) the terms of grant of such land;
or (b) the provisions of the law providing for such grant; or (c) sub-section (2) of Section
4 of the Act, it shall be null and void and no right, title or interest in such land shall be
conveyed or be deemed ever to have conveyed by such transfer. Therefore, under Section
4 (1) of the Act, it can be safely concluded that this provision declares any transfer of
granted land made either before or after the coming into force of the Act, to be null and
void if it is in contravention of the conditions specified therein.
Section 4(2) of the Act, as noted herein-earlier, deals with the transfer of granted land
after the commencement of the Act i.e. after 1st of January, 1979. For the purpose of
Section 4(2), the court must be satisfied that 1) the sale deed was executed and registered
after the commencement of the Act, and 2) the same was executed and registered without
seeking prior permission of the State Government. Therefore, Section 4(2) clearly
postulates that a transferee cannot acquire the granted land from the grantee without
seeking the permission of the Government nor can the grantee transfer it without seeking
prior permission from the Government. We have already considered the scheme of the
Act as also the objects and reasons for which it was introduced. It is an admitted position
that the Act was introduced to help and protect the right, title and interest of the
scheduled castes and scheduled tribes, in respect of the granted lands, whose poverty and
status in the society was taken advantage of by some rich and affluent persons who took
their lands either by paying a paltry sum or even without paying anything to them.
13. As noted herein-earlier, it is true that in this case, admittedly, the parties had entered
into an agreement for sale in respect of the granted land before the commencement of the
Act. It is also an admitted position that the respondents belong/belonged to the scheduled
caste community. As already noted herein-earlier, for the purposes of this Act, "Transfer"
has been defined to include an "agreement for sale" although under the general law, an
"agreement for sale" will not by itself transfer the granted land automatically to the
purchaser/appellant. From an overall consideration of the objects and reasons for which
this Act was introduced viz., to protect the right and interest of the scheduled castes and
scheduled tribes in respect of the granted lands and the relevant provisions of the Act, it is
pellucid that the definition of "Transfer" under Section 3(e) of the Act includes an
agreement for sale also and "Transfer" has been so defined to protect the right, title and
interest of the scheduled castes and scheduled tribes so that possession of the lands
@page-SC1281
could be restored to them even if they had entered into an agreement for sale. It would be
necessary for us at this stage to examine Section 4 of the Act in depth and in detail. As
noted herein-earlier, Section 4 deals with prohibition of transfer of granted lands.
Subsection (1) of Section 4 starts with the non obstinate clause and provides that any
transfer of granted land, either before or after the commencement of the Act in
contravention of the terms of the grant of such land or the law providing for such grant or
sub-section (2) shall be null and void and no right, title or interest in such land shall be
conveyed or be deemed ever to have conveyed by such transfer. In our view, therefore, it
is clear from a plain reading of Section 4(1) that if any one of the conditions is satisfied,
it would render the transfer null and void.
14. Let us, therefore, consider whether any of the conditions is satisfied in the present
case and thereby, whether, the transfer shall be null and void conveying or deeming ever
to have conveyed no right, title or interest of such land by such transfer. So far as the first
condition, namely, transfer in contravention of the terms of the grant of such land is
concerned, it cannot be disputed in the facts of this case that there was no contravention
of the terms of the grant of such land as the transfer was admittedly made after 15 years
of the date of certificate, which was the only condition regarding prohibition of transfer
in the grant. It is also not in dispute that there is no contravention of any law providing
for such grant. Therefore, so far as these two conditions are concerned, it cannot be
disputed that they are not satisfied. Now, let us take into consideration the third condition
i.e. transfer made in contravention of sub-section (2) of Section 4 of the Act. In respect of
this condition, a transfer of any granted land made after the commencement of the Act in
contravention of sub-section (2) shall be null and void and no right, title or interest in
such land shall be conveyed or be deemed ever to have conveyed by such transfer. Sub-
Section (2) of Section 4 clearly says that no person shall, after the commencement of this
Act, transfer or acquire by transfer any granted land without the previous permission of
the Government. Therefore, sub-section (2) of Section 4 prohibits transfer or acquisition
by transfer, either by the transferor or by the transferee of any granted land without the
previous permission of the Government. Therefore, after the commencement of this Act,
if any transfer is effected or any person acquires any granted land by transfer, without the
previous permission of the Government, such transfer shall be null and void and no right,
title or interest in such land shall be conveyed or be deemed ever to have conveyed by
such transfer.
15. So far as the facts of the present case are concerned, admittedly, the transfer was
effected after the commencement of the Act by a deed of sale dated 13th of October, 1986
without the previous permission of the Government. That being the position, we have no
hesitation to hold that such transfer must be held to be null and void and no right, title or
interest in such land shall be conveyed or be deemed ever to have conveyed by such
transfer.
16. As argued and noted herein-earlier, the learned counsel for the appellant submitted
that in view of the admitted fact that the agreement for sale, which is also a transfer
within the meaning of Section 3(e) of the Act, was made before the commencement of the
Act, it cannot be held that such transfer was null and void and that no right, title or
interest shall be conveyed by such transfer. We are unable to agree with this submission
of the learned counsel for the appellant. As noted herein-earlier, it is true that by virtue of
Section 3(e) of the Act, Transfer" includes an agreement for sale. We have to keep in
mind that in order to protect the right, title and interest of the scheduled castes and
scheduled tribes, this Act was promulgated as it was found that some affluent and
influential persons of the society, either by payment of a mere consideration or by no
payment at all, sought to get the property transferred in their favour from the scheduled
castes and scheduled tribes community. In our view, it is for this reason and in order to
protect the scheduled castes and scheduled tribes community, transfer has been defined to
include an agreement for sale under the Act so that even if an agreement for sale is
executed by a scheduled caste or scheduled tribe and possession is delivered to the
vendee, it would be open to the authority under Section 5 of the Act to take steps for
resumption of the land from the vendee. Otherwise, there can be
@page-SC1282
cases where merely by entering into an agreement for sale, possession of the land is
delivered to the vendee and supposing transfer as defined in this Act does not include an
agreement for sale, then in such cases, it would not be open to the authority under the Act
to take steps for the resumption of the granted land as, merely by executing an agreement
for sale, no transfer is effected.
17

. Before parting with this judgment, we may note that the learned counsel for the
appellant in support of his contention, as noted hereinabove, relied on a decision of this
Court in the case of Manchegowda and Ors. vs. State of Karnataka and Ors. [(1984) 3
SCC 301]. This decision was also relied by the learned counsel who appeared for the
appellant before the learned single Judge of the Karnataka High Court. In our view, the
decision of this Court in Manchegowda's case was rightly distinguished by the learned
single Judge. We are in agreement with the decision of this Court in Manchegowda's case
but the scope of challenge by the petitioners in that decision was limited which was stated
at paragraph 7 of the said judgment, as follows :- AIR 1984 SC 1151, Para 7

"It may be noted that the validity of the Act in so far as it imposes prohibition on transfer
of granted land after the commencement of the Act has not been challenged and the
principal objection to the validity of the Act is taken because of the provisions in the Act
seeking to nullify the transfers of granted lands effected before the commencement of the
Act."
Therefore, we are in full agreement with the views expressed by the learned single Judge
of the High Court that the scope of challenge by the petitioners in the aforesaid decision
of this Court was limited and, therefore, that decision cannot be of any help to the
appellant in the present case.
18. That being the position, we do not find any substance in the arguments of the learned
counsel for the appellant and accordingly, this appeal fails. The appeal is, therefore,
dismissed. There will be no order as to costs.
Appeal dismissed.
AIR 2008 SUPREME COURT 1282 "Bachchan Devi v. Nagar Nigam, Gorakhpur"
(From : Allahabad)*
Coram : 2 Dr. A. PASAYAT AND LOKESHWAR SINGH PANTA, JJ.
Civil Appeal No. 992 of 2008 (arising out of SLP (C) No. 24576 of 2004), D/- 5 -2 -2008.
Smt. Bachchan Devi and Anr. v. Nagar Nigam, Gorakhpur and Anr.
(A) Civil P.C. (5 of 1908), O.41, R.25 - APPEAL - APPELLATE COURT - Framing
additional issues - Powers of appellate Court.
The provision under O. 41, R. 25 becomes operative when the appellate Court comes to
the conclusion about the omission on the part of the lower Court to frame or try any issue.
Once the appellate Court directs the lower Court to do so, it is incubment upon the trial
Court to take additional evidence required. There may be cases where additional evidence
may not be required. But where the additional evidence is required, then the lower Court
has to return the evidence so recorded to the appellate Court together with the findings
thereon and the reasons therefor. Requirement for recording the finding of facts and the
reasons disclosed from the facts is because the appellate Court at the first instance has
come to the conclusion that the lower Court has omitted to frame or try any issue or to
determine any question of fact material for the right decision of the suit on merits. Where
a finding is called for on the basis of certain issues framed by the appellate Court, the
appeal is not disposed of either in whole or in part. Therefore, the parties cannot be
barred from arguing the whole appeal after the findings are received from the Court of the
first instance.
AIR 1974 SC 1702, Rel. on. (Para 10)
(B) Civil P.C. (5 of 1908), O.41, R.25 - APPEAL - INTERPRETATION OF STATUTES -
APPELLATE COURT - Use of words 'may' and 'shall' in same provision - Manifest
intention of legislature to make one part directory and another mandatory - But that by
itself not decisive - Power of Court to find out whether provision is directory or
mandatory - Not impaired.
Interpretation of Statutes - Words 'may' and 'shall' - Use in same provision - Effect.
@page-SC1283

Where the legislature uses two words "may" and "shall" in two different parts of the same
provision prima facie it would appear that the legislature manifested its intent on to make
one part directory and another mandatory. But that by itself is not decisive. The power of
Court to find out whether the provision is directory or mandatory remains unimpaired.
(Para 34)
The ultimate rule in construing auxiliary verbs like 'may' and 'shall' is to discover the
legislative intent; and the use of words 'may' and 'shall' is not decisive of its discretion or
mandates. The use of the words 'may' and 'shall' may help the Courts in ascertaining the
legislative invent without giving to either a controlling or a determining effect. The
Courts have further to consider the subject-matter, the purpose of the provisions, the
object intended to be secured by the statute which is of prime importance, as also the
actual words employed. (Para 33)
The use of the words 'shall' in a statute, though generally taken in a mandatory sense,
does not necessarily mean that in every case it shall have that effect, that is to say, that
unless the words of the statute are punctiliously followed, the proceeding or the outcome
of the proceeding would be invalid. On the other hand, it is not always correct to say that
when the word 'may' has been used, the statute is only permissible or directory in the
sense that non-compliance with those provisions will not render the proceeding invalid.
(Para 28)
(C) Civil P.C. (5 of 1908), O.41, R.25 - APPEAL - APPELLATE COURT - REMAND
OF MATTER - Framing of fresh issues - Powers of appellate Court to direct - Defendant
amended written statement during pendency of appeal - Additional issues framed -
Appellate Court in circumstances could exercise discretion under O.41, R.25 and it could
record evidence itself - Order remanding matter to trial court for fresh decision - Illegal.
(Para 37)
Cases Referred : Chronological Paras
2007 AIR SCW 6072 : AIR 2008 SC 48 : 2007 (6) ALJ 392 (Ref.) 27
AIR 1980 SC 1622 : 1980 Cri LJ 1075 (Ref.) 24
AIR 1978 SC 955 (Ref.) 20
AIR 1975 SC 1409 (Ref.) 6
AIR 1974 SC 1702 (Rel. on) 10
1968 AC 997 : (1968) 1 All ER 694 : (1968) 2 WLR 924 (HL) 19
AIR 1952 SC 16 22
AIR 1923 PC 138 21
AIR 1923 All 384 10
(1911) 2 QB 1131 20
(1890) 44 Ch D 262 14
(1889) 60 LT 963 20
(1880) 5 AC 214 : 49 LJQB 580 : (1874-80) All ER (Rep) 43 (HL) 15
Jaideep Gupta, Sr. Advocate, Praveen Kumar, for Appellants; V.J. Francis, for
Respondents.
* F.A.F.O. No. 2264 of 2004, D/- 1-9-2004 (All.)
Judgement
1. Dr. ARIJIT PASAYAT, J. :- Leave granted.
2. Challenge in this appeal is to the order passed by learned Single Judge of the
Allahabad High Court setting aside the order passed by the First Appellate Court, and
directing it to take decision on merit.
3. Essential facts are as follows : Respondent No. 1 as plaintiff No. 1 along with one
Gabbu filed Suit No. 23 of 1960 for declaration that the land in dispute belongs to it and
the defendants have no concern with the property. Assertion was that the property in
question had vested in it in view of the notification issued by the State Government after
abolition of Zamindari. The suit was decreed on 17-1-1972. The trial court granted relief
of permanent injunction in respect of suit property as described in the Schedule and also
declared that plaintiff No. 1 is the owner of the said plot. The decree was challenged by
way of appeal by the appellants. During the pendency of the appeal, an application to
amend the written statement was allowed by the Appellate Court. Thereafter certain
additional issues were framed. The Appellate Court was of the view since the written
statement had been amended during the pendency of the appeal, the matter should be
remanded to the trial court for fresh decision. Challenging the order passed, an appeal
was filed by respondent No. 1 before the High Court. Stand of the plaintiff No. 1 before
the High Court was that the Appellate Court committed an illegality in remanding the
matter for fresh consideration. It was submitted that the Appellate Court could have
exercised its discretion under Order XLI, Rule 25 of the Code of Civil Procedure, 1908
(in short 'the Code') and it could have recorded evidence itself. It was the opinion that the
same was
@page-SC1284
necessary for disposal of the appeal.
4. Stand of the defendants on the contrary was that two courses were available to the
Appellate Court. First was to pass the remand order after setting aside the findings. The
said course has been adopted. The other course was to call for findings on the issue by
remitting it to the trial court. The High Court was of the view that the order of remand
should be passed rarely and in the instant case that was not the case. That being so, the
High Court set aside the order of the First Appellant Court and the matter was remanded
to it for decision of the appeal on merit.
5. Learned counsel for the appellant submitted that the true scope and ambit of Order
XLI, Rule 25 has been improperly pressed into service. In the background of the factual
position, the order of the High Court cannot be maintained. The High Court, however,
noted that if the Appellate Court is of the opinion that the evidence is insufficient, the
matter may be remanded to the trial court for recording evidence in terms of Order XLI,
Rule 25 of the Code.
6
. In response, learned counsel for the respondent submitted that it will not be in the
interest of the parties to go on litigating and for that purpose the only course which was
available has been adopted. Strong reliance was placed on a decision of this Court in
Pasupuleti Venkateswarlu v. The Motor and General Traders (1975 (1) SCC 770). AIR
1975 SC 1409

7. Order XLI, Rule 25 of the Code reads as follows :


"ORDER XLI : APPEALS FROM ORIGINAL DECREES
25. Where Appellate Court may frame issues and refer them for trial to Court whose
decree appealed from. - Where the Court from whose decree the appeal is preferred has
omitted to frame or try any issue, or to determine any question of fact, which appears to
the Appellate Court essential to the right decision of the suit upon the merits, the
Appellate Court may, if necessary, frame issues, and refer the same for trial to the Court
from whose decree the appeal is preferred, and in such case shall direct such Court to
take the additional evidence required; and such Court shall proceed to try such issues, and
shall return the evidence to the Appellate Court together with its findings thereon and the
reasons therefor [within such time as may be fixed by the Appellate Court or extended by
it from time to time]."
8. There is ho scope for any doubt that in a suit as well as the first appeal all disputed
facts are open for decision. A point of fact is not to be decided in second appeal where
only a substantial question of law is to be looked into. There is some amount of
controversy as to whether the provisions are mandatory, notwithstanding the fact that the
word 'may' has been used. The First Appellate Court is the last Court of facts.
9. Under Order XLI, Rule 25, if it appears to the Appellate Court that any fact essential
for the decision in the suit was to be determined, it could frame an issue on the point and
refer the same for trial, to the Court from whose decree the appeal is preferred and in
such case, shall direct such court to take additional evidence required. The order of
remand should not be passed as a matter of routine The First Appellate Court which has
the power to analyse the factual position can decide the issue and the additional issues. In
the instant case the First Appellate Court, inter alia, observed as follows :
"As such, it would not be proper for the first Appellate Court in such matter to itself
record the evidence and to give its findings in regard to newly created issues. The Hon'ble
High Court has also held that in the present matter under the provision of Order 41 Rule
25 of Civil Procedure Code, becomes mandatory (shall) though in this provision, the
word 'may' has been used. No doubt in the present matter also the Appellate Court has
framed 6 additional issues which are legal in nature and also factual, with the result if the
Appellate Court gives its findings relating to said legal and factual issues after itself
recording (receiving) evidence then the aggrieved party would be prevented from his
right of filing first appeal. Accordingly, the aforesaid ratio laid down by the Hon'ble High
Court is fully applicable in the present matter."
10

. A bare reading of the provision makes it clear that the same comes into operation when
the Court, from whose decree the appeal is preferred, has omitted to frame or try and
issue, or to determine any question of fact which appears to the appellate court essential
for the right decision of the suit upon the merits. In order to bring in application of Order
XLI, Rule 25 the appellate court must come to a conclusion that the AIR 1974 SC
1702

@page-SC1285
lower court has omitted to frame issues and/ or has failed to determine any question of
fact which in the opinion of the appellate court are essential for the right decision of the
suit on merits. Once the appellate court comes to such a conclusion it may, if necessary,
frame the issues and refer the same to the trial court. In other words there is no
compulsion on the part of the appellate Court to do so. This is clear from the use of the
expression 'may'. But the further question that arises is whether in such a case the
appellate court is bound to direct the trial court to take additional evidence required. This
is a mandatory requirement as is evident from the provision itself because it provides that
the lower court shall proceed to try such case and shall return the evidence to the
appellate court together with findings therein and the reasons therefor. As noted above,
the provision becomes operative when the appellate court comes to the conclusion about
the omission on the part of the lower court to frame or try any issue. Once the appellate
court directs the lower court to do so, it is incumbent upon the trial court to take
additional evidence required. As has been rightly contended by learned counsel for the
appellant, there may be cases where additional evidence may not be required. But where
the additional evidence is required, then the lower court has to return the evidence so
recorded to the appellate court together with the findings thereon and the reasons therefor.
Requirement for recording the finding of facts and the reasons disclosed from the facts is
because the appellate court at the first instance has come to the conclusion that the lower
court has omitted to frame or try any issue or to determine any question of fact material
for the right decision of the suit on merits. It has to be noted that where a finding is called
for on the basis of certain issues framed by the appellate court, the appeal is not disposed
of either in whole or in part. Therefore the parties cannot be barred from arguing the
whole appeal after the findings are received from the court of the first instance. This
position was highlighted in Gogula Gurumurthy and Others v. Kurimeti Ayyappa (1975
(4) SCC 458), where it was inter-alia observed in para 5 as follows :
"We consider that when a finding is called for on the basis of certain issues framed by the
appellate Court the appeal is not disposed of either in whole or in part. Therefore the
parties cannot be barred from arguing the whole appeal after the findings are received
from the court of first instance. We find the same view taken in Gopi Nath Shukul v. Sat
Narain Shukul (AIR 1923 All 384)."
11. The delicate question that remains to be examined is what is the position in law when
both the expression "shall" and "may" are used in the same provision.
12. Mere use of word 'may' or 'shall' is not conclusive. The question whether a particular
provision of a statute is directory or mandatory cannot be resolved by laying down any
general rule of universal application. Such controversy has to be decided by ascertaining
the intention of the Legislature and not by looking at the language in which the provision
is clothed. And for finding out the legislative intent, the Court must examine the scheme
of the Act, purpose and object underlying the provision, consequences likely to ensue or
inconvenience likely to result if the provision is read one way or the other and many more
considerations relevant to the issue.
13. Several statutes confer power on authorities and officers to be exercised by them at
their discretion. The power is in permissive language, such as, 'it may be lawful', 'it may
be permissible', 'it may be open to do', etc. In certain circumstances, however, such power
is 'coupled with duty' and must be exercised.
14. More than a century ago, in Baker, Re, (1890) 44 Ch D 262, Cotton, L.J. stated;
I think that great misconception is caused by saying that in some cases 'may' means
'must'. It never can mean 'must', so long as the English language retains its meaning; but
it gives a power, and then it may be question in what cases, where a Judge has a power
given by him by the word 'may', it becomes his duty to exercise it.
(Emphasis supplied)
15. In the leading case of Julius v. Lord Bishop of Oxford (1880) 5 AC 214 : 49 LJ QB
580 : (1874-80) All ER Rep 43 (HL), the Bishop was empowered to issue a commission
of inquiry in case of alleged misconduct by a clergyman, either on an application by
someone or suo motu. The question was whether the Bishop had right to refuse
commission when an application was made. The House of Lords held that the Bishop had
@page-SC1286
discretion to act pursuant to the complaint and no mandatory duty was imposed on him.
16. Earl Cairns, L.C., however, made the following remarkable and oft-quoted
observations :
"The words 'it shall be lawful' are not equivocal. They are plain and unambiguous. They
are words merely making that legal and possible which there would otherwise be no right
or authority to do. They confer a faculty or power and they do not of themselves do more
than confer a faculty or power. But there may be something in the nature of the thing
empowered to be done, something in the object for which it is to be done, something in
the title of the person or persons for whose benefit the power is to be exercised, which
may couple the power with aduty, and make it the duty of the person in whom the power
is reposed, to exercise that power when called upon to do so."
17. Explaining the doctrine of power coupled with duty, de Smith, ('Judicial Review of
Administrative Action', 1995; pp.300-01) states :
"Sometimes the question before a court is whether words which apparently confer a
discretion are instead to be interpreted as imposing duty. Such words as 'may' and 'it shall
be lawful' are prima facie to be construed as permissive, not imperative. Exceptionally,
however, they may be construed as imposing a duty to act, and even a duty to act in one
particular manner."
(Emphasis supplied)
18. Wade also says (Wade and Forsyth; 'Administrative Law: 9th Edn.) : p.233) :
"The hallmark of discretionary power is permissive language using words such as 'may'
or 'it shall be lawful', as opposed to obligatory language such as 'shall'. But this simple
distinction is not always a sure guide, for there have been many decisions in which
permissive language has been construed as obligatory. This is not so much because one
form of words is interpreted to mean its opposite, as because the power conferred is, in
the circumstances, prescribed by the Act, coupled with a duty to exercise it in a proper
case."
(Emphasis supplied)
19. In the leading case of Padfleld v. Minister of Agriculture, Fisheries and Food 1968
AC 997 : (1968) 1 All ER 694 : (1968) 2 WLR 924 (HL), the relevant Act provided for
the reference of a complaint to a committee of investigation 'if the Minister so directs'.
The Minister refused to act on a complaint. It was held that the Minister was required to
act on a complaint in absence of good and relevant reasons to the contrary.
20

. Likewise, it was held that the licensing authorities were bound to renew licences of cab
drivers if the prescribed procedural requirements had been complied with [R. v.
Metropolitan Police Commissioner (1911) 2 QB 1131). Similarly, local authorities were
held bound to approve building plans if they were in conformity with bye-laws [R.V.
Nescastle-upon-Tyne Corporation (1889) 60 LT 963]. Again, the court was required to
pass a decree for possession in favour of a landlord, if the relevant grounds existed
[Ganpat Ladha v. Shashikant (1978 (2) SCC 573). AIR 1978 SC 955

21. In Alcock v. Chief Revenue Authority 50 IA 227 : AIR 1923 PC 138, the relevant
statute provided that if in the course of any assessment a question arises as to the
interpretation of the Act, the Chief Revenue Authority 'may' draw up a statement of the
case and refer it to the High Court. Holding the provision to be mandatory and following
Julius, Lord Phillimore observed :
"When a capacity or power is given to a public authority, there may be circumstance
which couple with the power of duty to exercise it."
22

. In Commissioner of Police v. Gordhandas Bhanji (1952 (1) SCR 135), Rule 250 of the
Rules for Licensing and Controlling Theatres and Other Places of Public Amusement in
Bombay City, 1884 read as under : AIR 1952 SC 16

"The Commissioner shall have power in his absolute discretion at any time to cancel or
suspend any licence granted under these Rules."
23. It was contended that there was no specific legal duty compelling the Commissioner
to exercise the discretion. Rule 250 merely vested discretion in him but it did not require
him to exercise the power. Relying upon the observations of Earl Cairns, L.C., the Court
observed :
"The discretion vested in the Commissioner of Police under Rule 250 has been conferred
upon him for public reasons involving the convenience, safety, morality and the welfare
of the public at large. An enabling power of this kind conferred for public
@page-SC1287
reasons and for the public benefit is in our opinion, coupled with a duty to exercise it
when the circumstances so demand. It is a duty which cannot be shirked or shelved nor
can it be evaded.... "
(Emphasis supplied)
24. In Ratlam Municipality v. Vardichan (1980 (4) SCC 162), some residents of Ratlam
Municipality moved the Sub-Divisional Magistrate under Section 133 of the Code of
Criminal Procedure, 1973 for abatement of nuisance by directing the municipality to
construct drain pipes with flow of water to wash the filth and stop the stench. The
Magistrate found the facts proved and issued necessary directions. The Sessions Court, in
appeal, reversed the order. The High Court, in revision, restored the judgment of the
Magistrate and the matter was carried to the Supreme Court.
25. This Court summarized the principle thus :
"The key question we have to answer is whether by affirmative action a court can compel
a statutory body to carry out its duty to the community by constructing sanitation
facilities at great cost and on a time-bound basis. At issue is the coming of age of that
branch of public law bearing on community actions and the court's power to force public
bodies under public duties to implement specific plans in response to public grievances."
26. Holding the provision obligatory, the Court observed :
"Judicial discretion when facts for its exercise are present, has a mandatory import.
Therefore, when the Sub-Divisional Magistrate, Ratlam, has, before him, information and
evidence, which disclose the existence of a public nuisance and, on the materials placed,
he considers that such unlawful obstruction or nuisance should be removed from any
public place which may be lawfully used by the public, he shall act.... This is a public
duty implicit in the public power to be exercised on behalf of the public and pursuant to a
public proceeding.
(Emphasis supplied)
27. We do not wish to refer to other cases on the point. We are, however, in agreement
with the observations of Earl Cairns, L.J. in Julius referred to above wherein His
Lordship stated :

"Where a power is deposited with a public officer for the purpose of being used for the
benefit of persons who are specifically pointed out, and with regard to whom a definition
is supplied by the Legislature of the conditions upon which they are entitled to call for its
exercise, that power ought to be exercised, and the Court will require it to be exercised."
(See M/s. Dhampur Sugar Mills Ltd. v. State of U.P. 2007 (10) SCR 245). 2007 AIR
SCW 6072

28. The use of the words 'shall' in a statute, though generally taken in a mandatory sense,
does not necessarily mean that in every case it shall have that effect, that is to say, that
unless the words of the statute are punctiliously followed, the proceeding or the outcome
of the proceeding would be invalid. On the other hand, it is not always correct to say that
when the word 'may' has been used, the statute is only permissible or directory in the
sense that non-compliance with those provisions will not render the proceeding invalid.
29. Words are the skin of the language. The language is the medium of expressing the
intention and the object that particular provision or the Act seeks to achieve. Therefore, it
is necessary to ascertain the intention. The word 'shall' is not always decisive. Regard
must be had to the context, subject matter and object of the statutory provision in
question in determining whether the same is mandatory or directory. No universal
principle of law could be laid in that behalf as to whether a particular provision or
enactment shall be considered mandatory or directory. It is the duty of the court to try to
get at the real intention of the legislature by carefully analysing the whole scope of the
statute or section or a phrase under consideration. The word 'shall', though prima facie
gives impression of being of mandatory character, it requires to be considered in the light
of the intention of the legislature by carefully attending to the scope of the statute, its
nature and design and the consequences that would flow from the construction thereof
one way or the other. In that behalf, the court is required to keep in view the impact on
the profession, necessity of its compliance; whether the statute, if it is avoided, provides
for any contingency for non-compliance; if the word 'shall' is construed as having
mandatory character, the mischief that would ensure by such construction; whether the
public convenience would be subserved or public inconvenience or the general
inconvenience that may ensue if it is held
@page-SC1288
mandatory and all other relevant circumstances are required to be taken into
consideration in construing whether the provision would be mandatory or directory.
30. The question, whether a particular provision of a statute, which, on the face of it,
appears mandatory inasmuch as it used the word 'shall', or is merely directory, cannot be
resolved by laying down any general rule, but depends upon the facts of each case
particularly on a consideration of the purpose and object of the enactment in making the
provision. To ascertain the intention, the court has to examine carefully the object of the
statute, consequence that may follow from insisting on a strict observance of the
particular provision and, above all, the general scheme of the other provisions of which it
forms a part. The purpose for which the provision has been made, the object to be
attained, the intention of the legislature in making the provision, the serious
inconvenience or injustice which may result in treating the provision one way or the
other, the relation of the provision to other consideration which may arise on the facts of
any particular case, have all to be taken into account in arriving at the conclusion whether
the provision is mandatory or directory. Two main considerations for regarding a rule as
directory are: (i) absence of any provision for the contingency of any particular rule not
being complied with or followed, and (ii) serious general inconvenience and prejudice to
the general public would result if the act in question is declared invalid for non-
compliance with the particular rule.
31. It is well-settled that the use of word 'may' in a statutory provision would not by itself
show that the provision is directory in nature. In some cases, the legislature may use the
word 'may' as a matter of pure conventional courtesy and yet intend a mandatory force. In
order, therefore, to interpret the legal import of the word 'may', the court has to consider
various factors, namely, the object and the scheme of the Act, the context and the
background against which the words have been used, the purpose and the advantages
sought to be achieved by the use of this word, and the like. It is equally well-settled that
where the word 'may' involves a discretion coupled with an obligation or where it confers
a positive benefit to a general class of subjects in a utility Act, or where the court
advances a remedy and suppresses the mischief, or where giving the words directory
significance would defeat the very object of the Act, the word 'may' should be interpreted
to convey a mandatory force. As a general rule, the word 'may' is permissive and
operative to confer discretion and especially so, where it is used in juxtaposition to the
word 'shall', which ordinarily is imperative as it imposes a duty. Cases however, are not
wanting where the words 'may' 'shall', and 'must' are used interchangeably. In order to
find out whether these words are being used in a directory or in a mandatory sense, the
intent of the legislature should be looked into along with the pertinent circumstances. The
distinction of mandatory compliance or directory effect of the language depends upon the
language couched in the statute under consideration and its object, purpose and effect.
The distinction reflected in the use of the word 'shall' or 'may' depends on conferment of
power. Depending upon the context, 'may' does not always mean may. 'May' is a must for
enabling compliance of provision but there are cases in which, for various reasons, as
soon as a person who is within the statute is entrusted with the power, it becomes his duty
to exercise that power. Where the language of statute creates a duty, the special remedy is
prescribed for non-performance of the duty.
32. If it appears to be the settled intention of the legislature to convey the sense of
compulsion, as where an obligation is created, the use of the word 'may' will not prevent
the court from giving it the effect of Compulsion or obligation. Where the statute was
passed purely in public interest and that rights of private citizens have been considerably
modified and curtailed in the interests of the general development of an area or in the
interests or removal of slums and unsanitary areas. Though the power is conferred upon
the statutory body by the use of the word 'may' that power must be construed as a
statutory duty. Conversely, the use of the term 'shall' may indicate the use in optional or
permissive sense. Although in general sense 'may' is enabling or discretional and 'shall' is
obligatory, the connotation is not inelastic and inviolate." Where to interpret the word
'may' as directory would render the very object of the Act as nugatory, the word 'may'
must mean 'shall'.
33. The ultimate rule in construing auxiliary verbs like 'may' and 'shall' is to discover the
legislative intent; and the use of words 'may' and 'shall' is not decisive of its
@page-SC1289
discretion or mandates. The use of the words 'may' and 'shall' may help the courts in
ascertaining the legislative intent without giving to either a controlling or a determinating
effect. The courts have further to consider the subject matter, the purpose of the
provisions, the object intended to be secured by the statute which is of prime importance,
as also the actual words employed.
34. Obviously where the legislature uses two words may and shall in two different parts
of the same provision prima facie it would appear that the legislature manifested its intent
on to make one part directory and another mandatory. But that by itself is not decisive.
The power of court to find out whether the provision is directory or mandatory remains
unimpaired.
35. One additional factor, which may not have an effect on the appeal is to be noted. The
First Appellate Court after judgment of the High Court dated 4-2-2005 disposed of the
appeal and remitted the matter to the trial Court. The stay order of this Court was passed
on 7-2-2005.
36. It is to be noted that the High Court in the impugned judgment has noted that if the
Appellate Court is of the opinion that if the evidence is insufficient to decide the issue,
only then the matter may be remitted to the trial Court.
37. Above being the position, the appeal by the respondents before the Allahabad High
Court has been rightly allowed. In any event, the order does not suffer from any infirmity
to warrant interference. The appeal is dismissed.
Appeal dismissed.
AIR 2008 SUPREME COURT 1289 "Anand Sharadchandra Oka v. University of
Mumbai"
(From : Bombay)
Coram : 2 C. K. THAKKER AND ALTAMAS KABIR, JJ.
Civil Appeal No. 967 of 2008 (arising out of SLP (C) No. 4590 of 2006), D/- 4 -2 -2008.
Anand Sharadchandra Oka v. University of Mumbai and Ors.
(A) Constitution of India, Art.226 - WRITS - UNIVERSITY - ELECTION - Aggrieved
party - University senate election - Inclusion of names of registered graduates in electrol
roll - Term 'graduate' interpreted to be graduate of university only - Petitioner obtained
graduate degree from university and post graduate degree from other university - Entitled
to be registered in electrol roll - Writ petition by him against interpretation of term
'graduate' given by university - Not maintainable, he being not a 'aggrieved party' -
Petition was not in form of PIL thus High Court ought not to have decided question
raised.
Maharashtra State Universities Act (35 of 1994), S.99. (Para 11)
(B) Maharashtra State Universities Act (35 of 1994), S.99, S.2(36) - UNIVERSITY -
Register of registered graduates - Term 'graduate' - Registration was restricted by
university to those who have graduated from university - Said interpretation of university
cannot be said to be unwarranted, illegal or contrary to statutory provisions - Impugned
action of university cannot be said to be illegal or improprer in absence of any challenge
to constitutional validity of statutory provision. (Paras 16, 17, 18)

Vinay Navare, Naresh Kumar, for Appellant; Ravindra Keshavrao Adsure, S. S. Shinde,
for Respondents.
Judgement
C. K. THAKKER, J. :- Leave granted.
2. The present appeal is filed against final judgment and order dated August 8, 2005
passed by the High Court of Judicature at Bombay in Writ Petition No. 1513 of 2005. By
the impugned order, the High Court dismissed the petition on the ground that the writ
petitioner could not be said to be 'aggrieved party'. In view of the said finding, the High
Court did not consider it appropriate to express any opinion on the question raised in the
petition.
3. Shortly stated the facts of the case are that the first respondent is University of
Mumbai. Respondent Nos. 2 and 3 are Vice Chancellor and Registrar respectively of
respondent No. 1, whereas Respondent No. 4 is the State of Maharashtra. The University
is governed by the provisions of the Maharashtra Universities Act, 1994 (hereinafter
referred to as 'the Act').
4. On August 2, 1999, the respondent University issued a notification calling for
applications from registered graduates in the prescribed form for getting their names
registered in the electoral roll for electing ten members in the Senate of the University.
The writ-petitioner who holds LL.M. degree of the University applied for registering his
name
@page-SC1290
in the said roll. The respondent-University, however, addressed a letter to the writ
petitioner, calling upon him to submit his Bachelor Degree Certificate to ascertain
whether he had obtained Graduate Degree from the said University. According to the writ
petitioner, if a person has obtained Master Degree or Doctoral Degree from the
University, his name also should be included in the electoral roll and he cannot be denied
registration only on the ground that he had not obtained Graduate Degree from the
University. The writ petitioner, in the circumstances, approached the High Court by filing
Writ Petition No. 436 of 2000 challenging the interpretation placed by the respondent-
University on the term 'Graduate'. The High Court found prima facie substance in
argument of the writ petitioner and admitted the petition by issuing Rule nisi. But, by the
time the writ petition came up for final hearing, elections were over and the High Court
did not think it fit to express any opinion on the question of law raised by the writ
petitioner and disposed it of observing that the petition had become 'infructuous'. The
question of law, however, was kept open.
5. Once again when the elections were scheduled to be held, the question of interpretation
of the word 'Graduate' came up for consideration. The writ-petitioner addressed a letter to
the University on October 25, 2004 to re-consider the legal issue. The respondent-
University, however, disregarded the writ petitioner's request and issued a notification on
April 22, 2005 for election of Senate. It insisted to register names of those persons who
had obtained Graduate Degree from the University. The writ petitioner, therefore, was
constrained to approach the High Court again by filing the present petition, i.e. Writ
Petition 1513 of 2005. Notice was issued by the Court and the respondents appeared. An
affidavit was filed on behalf of the respondents wherein it was contended that the writ
petitioner could not be said to be 'aggrieved party' in view of the fact that he was
graduated from Bombay University and his name could be registered in the electoral roll.
No other person had made any grievance who was graduated from other University and
obtained Master Degree or Doctoral Degree from Bombay University and was denied
enrolment of his name in the electoral roll. The petition filed by the writ petitioner,
therefore, was not maintainable.
6. The High Court in the impugned order observed that the writ-petitioner himself was a
graduate who obtained B.A. Degree from the respondent-University. He could not,
therefore, have any grievance in the matter. The contention of the writ petitioner was that
the respondent-University was wrongly interpreting the word 'Graduate' in a restricted
manner and several other persons who were not graduated from respondent-University,
but obtained Master or Doctoral Degree from the University were not enrolled in the
electoral roll. According to the High Court, since the writ-petitioner was not 'aggrieved
party', the petition was liable to be dismissed and accordingly, it was dismissed. The said
order is challenged by the writ petitioner in the present appeal.
7. Notice was issued on February 27, 2006 by this Court and on August 27, 2006, the
Registry was directed to place the matter for hearing on a non-miscellaneous day. That is
how the matter has been placed before us.
8. We have heard the learned counsel for the parties.
9. The learned counsel for the appellant submitted that the High Court was wrong in
dismissing the petition on the ground of locus standi. The Court ought to have
appreciated that the question was of interpretation of law and it ought to have decided the
issue one way or the other. According to the appellant, even in past, the High Court did
not decide the matter on merits and disposed of his writ petition as 'infructuous'. Again
the question has come up and even in future, at every election, such question will arise. It
was, therefore, submitted that the High Court was wrong in not deciding the controversy.
10. The learned counsel for the respondents, on the other hand, submitted that the High
Court was justified in dismissing the writ petition on the ground that the petitioner was
not aggrieved person. The writ petition was not in the nature of Public Interest Litigation
(PIL) and when the writ-petitioner himself was graduated from the respondent-
University, his name could be there in the electoral roll. The High Court, hence, refused
to enter into larger question. The counsel, however, admitted that there may be certain
persons who might have been graduated from other Universities and obtained Master
Degree or Doctoral Degree from Bombay University and whose names
@page-SC1291
on that ground might not have been registered in the electoral roll. But it was submitted
that this is the provision of law, the University has rightly interpreted it and refused to
register their names. He further submitted that the constitutional validity or vires of the
provision had not been challenged by the writ-petitioner. In the light of the statutory
provisions, the University decided not to register names of persons who were graduated
from other University and no fault can be found against such action. He, therefore,
submitted that the appeal deserves to be dismissed.
11. Having heard the rival contentions of the parties, in our opinion, it cannot be said that
the High Court was wrong in dismissing the writ petition filed by the writ-petitioner-
appellant herein. It is expressly stated by the High Court that the writ-petitioner obtained
B.A. Degree from Bombay University. Thus, the writ-petitioner was graduated from the
respondent-University. His name, therefore, can be registered in the electoral roll for
electing members of Senate. He was not, therefore, an 'aggrieved party'. The writ petition
was not in the form of PIL and it cannot be said that the High Court ought to have
decided the question. To that extent, therefore, the grievance voiced by the wilt-petitioner
is not justifiable.
12. It is, no doubt, equally true that there may be some persons who might have obtained
Graduate Degree from Universities other than the respondent-University and Master
Degree or Doctoral Degree from Bombay University. According to the interpretation
adopted by the respondent-University, their names cannot be registered under the Act. We
have, therefore, to consider whether the action of the University is illegal, contrary to law
or otherwise objectionable. The learned counsel for the respondents, in this connection,
referred to the relevant provisions of the Act. Section 2 defines certain terms and the
word 'University' is defined in Clause (36) of Section 2 which reads thus;
"University" means any of the universities mentioned in the Schedule.
13. The Schedule to the Act specifies Universities. The term 'Graduate' is not defined in
the Act. Section 3 provides for "Incorporation of Universities". Section 6 deals with
"Jurisdiction and Admission to Privileges of University". Section 24 enumerates
Authorities of the University. One of the Authorities of the University is "Senate".
Section 25 declares that Senate shall be the Principal Authority for all financial estimates
and budgetary appropriations and for providing social feedback to the University on
current and future academic programmes and also provides for its constitution. Section
26 lays down functions and duties of Senate. Chapter XI relates to Enrolment, Degrees
and Convocation. Section 99 is a material provision and provides for Registered
Graduates. Sub-section (1) of the said section is material and reads thus;
"(1) Subject to the provisions of sub-section (2), the following persons shall be entitled to
have their names entered in the register of registered graduates or deemed to be registered
graduates, maintained by the university, namely :-
(a) who are graduates of the university :
(b) who are graduates of the present university from which corresponding new university
is established; Provided. . .
(2) ..........."
(Emphasis supplied)
14. Section 100 enables the Chancellor to remove name of any person from register of
graduates.
15. Clause (a) of sub-section (1) of Section 99, in our opinion, is clear and unambiguous.
It specifically and unequivocally declares that only those persons who are 'Graduates of
the University' are entitled to have their names entered in the register of registered
graduates. As already observed earlier, University means any university mentioned in the
Schedule. It is not even the case of the writ-petitioner either before the High Court or
before us that name of any person who has graduated from the University as defined in
Section 2(36) of the Act has not been included in the register. It, therefore, cannot be said
that the interpretation of the respondent-University is unwarranted, illegal or contrary to
statutory provisions. In our opinion, the learned counsel for the respondent-University is
also right in contending that the constitutional validity of statutory provision has not been
challenged by the writ-petitioner and, as such, the Court is called upon only to interpret
the provision as it stands treating it to be valid and intra vires. If it is so, the limited
controversy before the Court is whether the University is right in interpreting the relevant
@page-SC1292
provision of law in Section 99 read with Section 2 of the Act.
16. Learned counsel for the appellant, however, submitted that the repealed statute,
namely, the Bombay Universities Act, 1974, treated persons who had obtained Graduate
Degree from other university, but Master Degree or Doctoral Degree from Bombay
University as eligible and qualified to be included in the register of registered graduates
of the University. In our opinion, however, the above circumstance, instead of supporting
the writ-petitioner may support the respondents as it can be said that though there was
such provision in the previous Act, the Legislature consciously and deliberately departed
from it and the registration was restricted to those who must have graduated from the
University. The said contention, therefore, cannot take the case of the writ-petitioner
further.
17. It was then contended that in several other Universities, such persons who had
obtained Graduate Degree from other universities, but obtained Master degree or
Doctoral Degree from those Universities have been treated as eligible to get their names
registered in the register of graduates. Even on that ground, the impugned action of the
respondent-University cannot be said to be legal or proper. We are afraid, we cannot
uphold the contention of the writ-petitioner. It is for the Legislature to provide for
registration of graduates and in absence of any challenge to the constitutional validity, we
have to give literal interpretation to the expressions used and terms defined in the statute
book.
18. As already noted by us, considering Section 99 in the light of Clause (36) of Section 2
of the Act, it cannot be said that the University was wrong or had committed any error in
interpreting the provision and in depriving any person of his right. If it is so, no grievance
can be made against such interpretation. We, therefore, see no substance in the argument
raised by the writ petitioner.
19. For the foregoing reasons, the appeal deserves to be dismissed and is, accordingly,
dismissed. On the facts and in the circumstances of the case, however, there shall be no
order as to costs.
Appeal dismissed.
AIR 2008 SUPREME COURT 1292 "State of Rajasthan v. Madan Singh"
(From : 2005 (2) Cri LR (Raj) 1662)
Coram : 2 Dr. A. PASAYAT AND P. SATHASIVAM, JJ.
Criminal Appeal No. 234 of 2008 (arising out of SLP (Cri.) No. 3629 of 2006), D/- 1 -2
-2008.
State of Rajasthan v. Madan Singh.
Penal Code (45 of 1860), S.376(2)(f) - RAPE - SENTENCE IMPOSITION -
SENTENCE REDUCTION - Rape on girl below 12 years of age - Sentence - Imposition
of sentence below minimum prescribed - Accused was young, is only bread-earner of his
family - Not adequate and special reasons for imposing less sentence - Order reducing
sentence to 7 years R. I. liable to be set aside.
2005 (2) Cri LR (Raj) 1662. Reversed. (Para 8)
(B) Penal Code (45 of 1860), S.376 - RAPE - Rape - Sentence - Considerations.
The measure of punishment in a case of rape cannot depend upon the social status of the
victim or the accused. It must depend upon the conduct of the accused, the state and age
of the sexually assaulted female and the gravity of the criminal act. Crimes of violence
upon women need to be severely dealt with. The socio-economic status, religion, race,
caste or creed of the accused or the victim are irrelevant considerations in sentencing
policy. Protection of society and deterring the criminal is the avowed object of law and
that is required to be achieved by imposing an appropriate sentence. (Para 8)

Milind Kumar and Aruneshwar Gupta, for Appellant.


Judgement
Dr. ARIJIT PASAYAT. J. :- Leave granted.
2. Challenge in this appeal is to the judgment of a learned Single Judge of the Rajasthan
High Court at Jodhpur. A learned Single Judge by the impugned judgment while
upholding the conviction for offence punishable under Section 376 (2)(f) of the Indian
Penal Code, 1860 (in short the IPC), reduced the sentence from 10 years to 7 years.
3. The respondent allegedly committed rape on a minor girl aged about 10 years on 29-8-
1999. There is no need to refer to the factual position in detail as the High Court has
upheld the conviction. It only needs to be noted that on the basis of the evidence
@page-SC1293
adduced, the trial Court found that the victim was aged about 10 years. The only point
which was urged before the High Court in addition to the question of sentence was that
the offence at best was one under Section 376 read with Section 511, IPC. It was
submitted that the accused had suffered custody of about 6 years and, therefore, he being
only bread earner of the family and being of young age, the sentence should be reduced
to the period already undergone. The plea was opposed by the State stating that in view of
the statutory minimum sentence provided, no leniency was called for. The High Court
found that the trial Court was justified in holding the appellant guilty of offence
punishable under Section 376 (2)(f) of IPC. As the victim was aged about 10 years, it
held that considering the factual position after assigning reason the minimum sentence
can be reduced. Having so observed, the High Court reduced the sentence to seven years
and a fine of Rs.5,000/ - with default stipulation with the following conclusions was
imposed :
"After having considered the entire matter and also taking into consideration the
submission of learned counsel that the accused is a young person who is the only bread
earner of his family and his kids who have now grown up need his supervision, I deem it
proper to reduce his sentence under Section 376(2)(f) to a term of 7 years with fine of
Rs.5,000/- in default, to further suffer one years simple imprisonment and modify the
order of learned trial Court to that extent."
4. Learned counsel for the appellant submitted that when minimum sentence is
prescribed, only for adequate and special reasons the sentence less than minimum
provided for can be imposed. In the instant case the reasons indicated did not meet the
requirement of law.
5. The respondent has not entered appearance in spite of service of notice.
6. Both in cases of sub-sections (1) and (2) of Section 376 the court has the discretion to
impose a sentence of imprisonment less than the prescribed minimum for "adequate and
special reasons". If the court does not mention such reasons in the judgment, there is no
scope for awarding a sentence lesser than the prescribed minimum.
7. It is to be noted that in sub-section (2) of Section 376 I.P.C. more stringent punishment
can be awarded taking into account the special features indicated in the said sub-section.
The present case is covered by Section 376(2)(f) IPC i.e. when rape is committed on a
woman when she is under 12 years of age. Admittedly, in the case at hand the victim was
10 years of age at the time of commission of offence.
8. The measure of punishment in a case of rape cannot depend upon the social status of
the victim or the accused. It must depend upon the conduct of the accused, the state and
age of the sexually assaulted female and the gravity of the criminal act. Crimes of
violence upon women need to be severely dealt with. The socio-economic status, religion,
race, caste or creed of the accused or the victim are irrelevant considerations in
sentencing policy. Protection of society and deterring the criminal is the avowed object of
law and that is required to be achieved by imposing an appropriate sentence. The
sentencing Courts are expected to consider all relevant facts and circumstances bearing
on the question of sentence and proceed to impose a sentence commensurate with the
gravity of the offence. Courts must hear the loud cry for justice by the society in cases of
the heinous crime of rape on innocent helpless girls of tender years, as in this case, and
respond by imposition of proper sentence. Public abhorrence of the crime needs
reflection through imposition of appropriate sentence by the Court. There are no
extenuating or mitigating circumstances available on the record which may justify
imposition of any sentence less than the prescribed minimum on the respondent. To show
mercy in the case of such a heinous crime would be a travesty of justice and the plea for
leniency is wholly misplaced.
9. The legislative mandate to impose a sentence for the offence of rape on a girl under 12
years of age, for a term which shall not be less than 10 years, but which may extend to
life and also to fine reflects the intent of stringency in sentence. The proviso to Section
376(2) IPC, of course, lays down that the court may, for adequate and special reasons to
be mentioned in the judgment, impose sentence of imprisonment of either description for
a term of less than 10 years. Thus, the normal sentence in a case where rape is committed
on a child below 12 years of age is not less than 10 years' RI, though in exceptional cases
"for special and adequate reasons" sentence of less than 10
@page-SC1294
years' RI can also be awarded. It is a fundamental rule of construction that a proviso must
be considered with relation to the principal matter to which it stands as a proviso
particularly in such like penal provisions. The courts are obliged to respect the legislative
mandate in the matter of awarding of sentence in all such cases. Recourse to the proviso
can be had only for "special and adequate reasons" and not in a casual manner. Whether
there exist any "special and adequate reasons" would depend upon a variety of factors
and the peculiar facts and circumstances of each case. No hard and fast rule of universal
application can be laid down in that behalf.
10. In view of the position in law indicated above, the judgment of the High Court
reducing the sentence to 7 years is clearly unsustainable and is set aside. The sentence of
10 years as imposed by the trial Court is restored.
11. The appeal is allowed.
Appeal allowed.
AIR 2008 SUPREME COURT 1294 "Ran Singh v. State of Haryana"
(From : Punjab and Haryana)*
Coram : 2 Dr. A. PASAYAT AND P. SATHASIVAM, JJ.
Criminal Appeal No. 222 of 2008 (arising out of SLP (Cri.) No. 3089 of 2006), D/- 30 -1
-2008.
Ran Singh and Anr. v. State of Haryana and Anr.
Penal Code (45 of 1860), S.498A - CRUELTY BY HUSBAND OR HIS RELATIVE -
COURT OF SESSIONS - REVISION - DISCHARGE - Cruelty to woman - Finding of
fact - Interference with - Allegations of harassment of daughter of complainant by
husband and in-laws for non-fulfillment of dowry demand - Discharge of in-laws by
Sessions Court as no case was made against them - Reversal of finding by High Court by
holding that mother-in-law and father-in-law are persons "who could misappropriate" and
"who could practice cruelty", without assigning reasons - Is improper.
Crl. R. No. 2468 of 2003. D/-29-11-2005 (PandH). Reversed.
Criminal P.C. (2 of 1974), S.401.
Right to reason is an indispensable part of a sound judicial system, reasons at least
sufficient to indicate an application of mind to the matter before Court. Another rationale
is that the affected party can know why the decision has gone against him. One of the
salutary requirements of natural justice is spelling out reasons for the order made, in other
words, a speaking out.(Para 10)
In the instant case order of High Court that father-in-law and mother-in-law 'could
misappropriate' and 'who can practice cruelty' and thus they are liable for prosecution is
invalid. The conclusions are presumptuous. Learned Additional Sessions Judge by a well
reasoned order had held that there was no material to show that demand for any dowry
was made and an attempt was made to rope in many persons. When the High Court was
interfering with such conclusions arrived at on facts it ought to have indicated the reasons
necessitating such interference. That has not been done and on the contrary on
presumptuous conclusions the order of learned additional Sessions Judge has been set
aside. Thus, order of High Court was liable to be set aside.
Crl. R. No. 2468 of 2003, D/-29-11-2005 (PandH), Reversed. (Paras 8, 12)
Cases Referred : Chronological Paras
2001 AIR SCW 3793 : AIR 2001 SC 2828 : 2001 Cri LJ 4625 (Ref) 7
1974 ICR 120 (NIRC) (Ref) 10
(1971)1 All ER 1148 : (1971) 2 WLR 742 (Ref) 10
Rishi Malhotra, Prem Malhotra, for Appellants; Rajeev Gaur 'Naseem', Rajesh Ranjan,
T.V. George, Chander Shekhar Ashri, for Respondents.
* Cri. R. No. 2468 of 2003, D/- 29-11-2005 (PandH).
Judgement
Dr. ARIJIT PASAYAT, J. :- Leave granted.
2. Challenge in this appeal is to the order passed by a learned Single Judge of the Punjab
and Haryana High Court allowing the Revision Petition filed under Section 401 of the
Code of Criminal Procedure, 1973 (in short the 'Code') which was filed before it by Kurra
Ram since deceased and represented by his daughter i.e. respondent No.2 in the present
appeal.
3. Background facts in a nutshell are as follows :
A complaint was filed by the aforesaid Kurra Ram alleging commission of offences
@page-SC1295
punishable under Sections 498-A, 406, 323, 506, 148 and 149 of the Indian Penal Code,
1860 (in short the 'IPC') by Jaswant-son in law and husband of his daughter-Saroj, Ran
Singh and Raj Bala, the present appellants who were father and mother of Jaswant and
two others namely, Jai Singh and Suman, the brother and married sister of Jaswant.
It was stated in the complaint that Saroj got married to Jaswant on 14-4-1994 and that she
was harassed for dowry by the aforesaid accused persons. Learned Additional Chief
Judicial Magistrate, Hissar, after recording preliminary evidence of the complainant,
decided to proceed against all the accused persons for the alleged offences. Separate
Revision Petitions were filed by Jai Singh, Ran Singh and Suman taking the stand that
there is no offence made out so far as they are concerned. Learned Additional Sessions
Judge found that no case was made out against aforesaid accused persons and directed
that proceedings would continue only against Jaswant. The order dated 4-11-2003
disposing of the revisions in the aforesaid manner was challenged by Kurra Ram in the
Revision Petition before the High Court. It was held by High Court that there is no
ground to proceed against Jai Singh and Suman who may just be living in the house, but
may not be interfering in matrimonial problems of Saroj and Jaswant. Therefore, the
order of the Additional Sessions Judge was upheld to that extent. But so far as the present
appellants are concerned the High Court inter alia observed as follows :
"However, when articles of dowry are handed over to elder members in the family that
will mean that those were handed over to Ran Singh and Raj Bala i.e. father and mother
of the husband who could misappropriate. It is they who can practice cruelty for less
dowry or otherwise."
(Underlined for emphasis)
The High Court noted that police had earlier registered a case and had sent cancellation
report and thereafter the complaint was filed by Kurra Ram who appeared as PW-1, as his
son Rajesh appeared as PW-2 and Saroj as PW-3.
4. Learned counsel for the appellants submitted that the High Court failed to notice that
some customary articles were given to relatives of the bridegroom. That cannot be
covered by the expression 'dowry'. High Court noticed the fact that the complainant tried
to rope even a married sister who was living far away and the brother, which shows the
tendency to falsely implicate them. Reference is also made to the following obserr
vations of the High Court :
"..They are close relatives but the fact remains that an effort is made by the complainant
to implicate as many persons as possible, in such matters."
5. Learned counsel for the respondent-State and the complainant submitted that it is not a
case where the Additional Sessions Judge should have interfered and the High Court has
therefore rightly set aside the order dated 4-11-2003 which was impugned before it.
6. Section 2 of the Dowry Prohibition Act, 1961 (in short 'Dowry Act') defines "dowry"
as under :-
Section 2. Definition of 'dowry' - In this Act, 'dowry' means any property or valuable
security given or agreed to be given either directly or indirectly -
(a) by one party to a marriage to the other party to the marriage; or
(b) by the parents of either party to a marriage or by any other person, to either party to
the marriage or to any other person,
at or before or any time after the marriage in connection with the marriage of the said
parties, but does not include dower or mehr in the case of persons to whom the Muslim
Personal Law (Shariat) applies.
Explanation I - For the removal of doubts, it is hereby declared that any presents made at
the time of a marriage to either party to the marriage in the form of cash, ornaments,
clothes or other articles, shall not be deemed to be dowry within the meaning of this
section, unless they are made as consideration for the marriage of the said parties.
Explanation II - The expression 'valuable security' has the same meaning in Section 30 of
the Indian Penal Code (45 of 1860)."
7

. The word "dowry" is defined in Section 2 of the Dowry Act. Thus, there are three
occasions related to dowry. One is before the marriage, second is at the time of marriage
and the third "at any time" after the marriage. The third occasion may appear to be
unending period. But the crucial words are "in connection with the marriage of the
2001 AIR SCW 3793

@page-SC1296
said parties". Other payments which are customary payments e.g. given at the time of
birth of a child or other ceremonies as are prevalent in different societies are not covered
by the expression "dowry". (See Satvir Singh v. State of Punjab (2001 (8) SCC 633)).
8. The High Court has fallen in grave error while observing that present appellants "could
misappropriate" and "who can practice cruelty". The conclusions to say the least are
presumptuous. Learned Additional Sessions Judge by a well reasoned order had held that
there was no material to show that demand for any dowry was made and an attempt was
made to rope in many persons. When the High Court was interfering with such
conclusions arrived at on facts it ought to have indicated the reasons necessitating such
interference. That has not been done and on the contrary on presumptuous conclusions
the order of learned Additional Sessions Judge has been set aside.
9. Reasons introduce clarity in an order. On plainest consideration of justice, the High
Court ought to have set forth its reasons, howsoever brief, in its order indicative of an
application of its mind. The absence of reasons has rendered the High Court's judgment
not sustainable.
10. Even in respect of administrative orders Lord Denning M. R. in Breen v.
Amalgamated Engineering Union (1971 (1) All ER 1148) observed "The giving of
reasons is one of the fundamentals of good administration". In Alexander Machinery
(Dudley) Ltd. v. Crabtree (1974 LCR 120) it was observed : "Failure to give reasons
amounts to denial of justice". Reasons are live links between the mind of the decision
taker to the controversy in question and the decision or conclusion arrived at". Reasons
substitute subjectivity by objectivity. The emphasis on recording reasons is that if the
decision reveals the "inscrutable face of the sphinx", it can, by its silence, render it
virtually impossible for the Courts to perform their appellate function or exercise the
power of judicial review in adjudging the validity of the decision. Right to reason is an
indispensable part of a sound judicial system, reasons at least sufficient to indicate an
application of mind to the matter before Court. Another rationale is that the affected party
can know why the decision has gone against him. One of the salutary requirements of
natural justice is spelling out reasons for the order made, in other words, a speaking out.
The "inscrutable face of a sphinx" is ordinarily incongruous with a judicial or quasi-
judicial performance.
11. It is to be noted that the High Court itself has held that there was an attempt to rope in
many persons and it did not find any merit or challenge to the discharge of the married
sister and the brother.
12. Above being the position, the impugned order of the High Court cannot be maintained
and is set aside. We make it clear that we have not expressed any opinion on merits so far
as husband Jaswant is concerned.
13. The appeal is allowed to the aforesaid extent.
Appeal allowed.
AIR 2008 SUPREME COURT 1296 "S. Anand v. Vasumathi Chandrasekar"
(From : Madras)*
Coram : 2 S. B. SINHA AND V. S. SIRPURKAR, JJ.
Criminal Appeal No. 311 of 2008 (arising out of SLP (Cri.) No. 3772 of 2007), D/- 14 -2
-2008.
S. Anand v. Vasumathi Chandrasekar.
(A) Criminal P.C. (2 of 1974), S.256 - SUMMONS CASE - DISHONOUR OF CHEQUE
- COMPLAINT - PERSONAL APPEARANCE - Complaint for dishonour of cheque -
Dismissal in default - Witnesses on behalf of complainant examined - Complainant
closed his case - Date fixed for examination of defence witnesses - Personal attendance of
complainant not essential for progress of case - Dismissal of complaint for non-
appearance of complainant - Not proper. (Paras 11, 13)
(B) Criminal P.C. (2 of 1974), S.385 - APPEAL - PRACTICE AND PROCEDURE -
DISMISSAL - APPELLATE COURT - Appeal - Procedure - Dismissal of criminal
complaint for dishonour of cheque - Appeal against - Even though, legal aid counsel
appointed, appellate Court deciding appeal without notice to accused, not approved.
(Paras 6, 16)
Cases Referred : Chronological Paras
2004 AIR SCW 6248 : AIR 2005 SC 48 : 2005 Cri LJ 112 (Ref.) 14
1998 AIR SCW 192 : AIR 1998 SC 596 : 1998 Cri LJ 856 (Ref.) 5
@page-SC1297
Appellant in person; A. Regunathan, Sr. Adv., P. Vinay Kumar, for Respondent.
* Cri. Appeal No. 537 of 2006, D/- 15-9-2006 (Mad).
Judgement
1. S. B. SINHA, J :- Leave granted.
2. Appellant was being prosecuted in the Court of Metropolitan Magistrate, Saidapet,
Chennai for alleged commission of an offence under Section 138 of the Negotiable
Instruments Act (for short "the Act") on the basis of a complaint petition filed by the
respondent herein.
3. In the said proceedings, witnesses on behalf of the prosecution had been examined.
Complainant closed her case. A date was fixed for examination of the defence witness
and argument on 10.04.2006.
However, the appellant filed an application for cross-examination of the complainant
herself which was rejected. A revision application was filed thereagainst in the Court of
the Sessions Judge.
In the said revision application, no order of stay was passed. Whereas the appellant had
continuously remained present before the Trial Judge, the complainant remained absent.
4. On or about 18.04.2006, the appellant filed an application for his acquittal on the
ground of absence of the complainant. By an order dated 24.04.2006, the learned
Metropolitan Magistrate acquitted the accused under Section 256(1) of the Code of
Criminal Procedure stating :
"Complainant absent. No representation for several hearings. Accused present. Petition
u/s. 256(1) Cr. P.C. is allowed. Complainant continuously absent from the hearing date
3.3.05. Hence, Complainant called three times. Neither the complainant nor his counsel
represent before the Court till 5.30 p.m. CW1 examined. Hence Accused is acquitted u/s.
256(1) of Cr.P.C."
5

. An appeal was preferred thereagainst before the High Court. The same was allowed
relying on or on the basis of a decision of this Court in Associated Cement Co. Ltd. v.
Keshvanand [(1998) 1 SCC 687 : AIR 1998 SC 596]. 1998 AIR SCW 192

6. We may, at the outset, notice that before passing the impugned order, the High Court
did not choose to serve notice upon the appellant opining that no useful purpose would be
served in keeping the appeal pending and one G. Vinodkumar was appointed as a legal
aid counsel. Aggrieved thereby, the appellant is before us.
7. It was submitted by Mr. Anand, appearing in person, that the complainant having
remained absent for more than one year, the High Court ought not to have interfered with
the discretionary jurisdiction exercised by the learned Metropolitan Magistrate,
particularly when he had been appearing in person and the complainant not only executed
a power of attorney in favour of another, a lawyer was also appointed.
Mr. Anand would submit that it was obligatory on the part of the advocate who is an
agent of his client to appear on the dates of hearing, more so when an accused had been
appearing in person and remained present in court for all the days of hearing. In any
event, it was urged, the High Court committed a serious error in disposing of the appeal
only upon hearing a legal aid counsel and even the submissions made by him had not
been noticed.
8. Mr. A. Regunathan, learned senior counsel appearing on behalf of the respondent,
however, submitted that in view of the fact that the matter was adjourned for examination
of DWs, the learned Magistrate could not have exercised its jurisdiction under Section
256 of the Code of Criminal Procedure.
9. Chapter XX of the Code of Criminal Procedure deals with trial of summons cases by
Magistrates.
Section 256 of the Code reads as under :
"256. Non-appearance or death of complainant. - (1) If the summons has been issued on
complainant, and on the day appointed for the appearance of the accused, or any day
subsequent thereto to which the hearing may be adjourned, the complainant does not
appear, the Magistrate shall, notwithstanding anything hereinbefore contained, acquit the
accused, unless for some reason he thinks it proper to adjourn the hearing of the case to
some other day :
Provided that where the complainant is represented by a pleader or by the officer
conducting the prosecution or where the Magistrate is of opinion that the personal
attendance of the complainant is not necessary, the Magistrate may dispense with his
attendance and proceed with the case.
(2) The provisions of sub-section (1) shall, so far as may be, apply also to cases where the
non-appearance of the complainant is due to his death.
10. Section 256 of the Code provides
@page-SC1298
for disposal of a complaint in default. It entails in acquittal. But, the question which arises
for consideration is as to whether the said provision could have been resorted to in the
facts of the case as the witnesses on behalf of complainant have already been examined.
11. The date was fixed for examining the defence witnesses. Appellant could have
examined witnesses, if he wanted to do the same. In that case, the appearance of the
complainant was not necessary. It was for her to cross-examine the witnesses examined
on behalf of the defence.
12. The accused was entitled to file an application under Section 311 of the Code of
Criminal Procedure. Such an application was required to be considered and disposed of
by the learned Magistrate. We have noticed hereinbefore that the complainant did not
examine herself as a witness. She was sought to be summoned again for cross-
examination. The said prayer has not yet been allowed. But, that would not mean that on
that ground the court would exercise its discretionary jurisdiction under Section 256 of
the Code of Criminal Procedure at that stage or the defence would not examine his
witnesses.
13. Presence of the complainant or her lawyer would have been necessary, as indicated
hereinbefore, only for the purpose of cross-examination of the witnesses examined on
behalf of the defence. If she did not intend to do so, she would do so at her peril but it
cannot be said that her presence was absolutely necessary. Furthermore, when the
prosecution has closed its case and the accused has been examined under Section 311 of
the Code of Criminal Procedure, the court was required to pass a judgment on merit of
the matter.
14. We are not concerned herein as to whether the constituted attorney of the complainant
could represent the complainant.
Reliance in this behalf having placed on Jimmy Jahangir Madan v. Bolly Cariyappa
Hindley (Dead) By LRs. [(2004) 12 SCC 509] need not, thus, be considered by us.
2004 AIR SCW 6248

15. Similar contention of the complainant that the advocate is an agent of his client and it
is his duty to appear on behalf of his client, in our opinion, is beyond the scope of this
appeal.
16. We, therefore, although do not approve the manner in which the appeal has been
disposed of by the High Court, are of the opinion that it is not a fit case where we should
exercise our jurisdiction under Article 136 of the Constitution of India.
17. However, keeping in view of the fact that the complaint petition was filed as far back
on 10.01.2002, the learned Trial Judge should proceed with the matter in accordance with
law and dispose of the case as expeditiously as possible. On the date(s) on which the
accused remains present, the complainant would not take any adjournment and in the
event she does not choose to be represented in the court, the court shall proceed in the
matter in accordance with law. Both the accused and complainant are directed to appear
in the Trial Court within two weeks from date.
17A. The appeal is dismissed with the aforementioned observations.
Appeal dismissed.
AIR 2008 SUPREME COURT 1298 "Ramesh Dass v. Raghu Nath"
(From : Punjab and Haryana)*
Coram : 2 Dr. A. PASAYAT AND P. SATHASIVAM, JJ.
Criminal Appeal No. 313 with 314 of 2008 (arising out of SLP (Cri.) No. 4646 with 5321
of 2006), D/- 14 -2 -2008.
Ramesh Dass v. Raghu Nath and Ors.
Criminal P.C. (2 of 1974), S.360 - Probation of Offenders Act (20 of 1958), S.4 -
PROBATION OF OFFENDERS - GRIEVOUS HURT - GENERAL CLAUSES - LIFE
IMPRISONMENT - Release of accused on probation - Accused charged with
commission of offence u/S.326 of Penal Code - S.360 has no application as offence is
punishable with life imprisonment - Provisions of Probation of Offenders Act would
apply - Release of accused u/S.360 of Criminal P.C. not proper - Matter remitted back to
High Court for fresh consideration regarding non-applicability of S.360. Criminal P.C.
Cri. R. No. 782 of 1991. D/- 8-5-2006 (P and H), Reversed.
Penal Code (45 of 1860), S.326.
General Clauses Act (10 of 1897), S.8(1). (Paras 6, 11, 12)
@page-SC1299
Cases Referred : Chronological Paras
2006 AIR SCW 4062 : AIR 2006 SC 3051 : 2006 (5) ALJ 694 : 2006 Cri LJ 4068 (Ref.)
8
(2006) 6 SCC 159 (Ref.) 8
AIR 1973 SC 2427 : 1973 Cri LJ 1614 (Ref.) 10
Dr. Kanwal Sapra, Gyaneshwar Bhat, B. B. Sinha, A.L. Trehan, T.V. George, for
Appellant; Ms. Lily Thomas, A.K. Panigrahi, Ms. A. Rama Devi, for Respondents.
* Cri. R. No. 782 of 1991, D/- 8-5-2006 (P and H).
Judgement
Dr. ARIJIT PASAYAT, J. :- Leave granted.
2. These two appeals have their matrix on an order passed by a learned Single Judge of
the Punjab and Haryana High Court. By the impugned judgment a Criminal Revision was
filed by the informant and Criminal Appeal filed by the State of Haryana seeking
enhancement of the sentence were disposed of.
Respondents faced trial for alleged commission of offences punishable under Sections
148 and 326 read with Section 149, Section 325 read with Section 149, Section 324 read
with Section 149 and Section 323 read with Section 149 of the Indian Penal Code, 1860
(in short the 'IPC') and Chief Judicial Magistrate, Karnal convicted and sentenced five of
the accused persons in the following manner :

Sections Imprisonment Fine


326/149 IPC Five years RI Rs. 5,000/- each. In default of payment of fine R.I. for one
year.
325/149 IPC Two years R.I. Rs.500/- each. In default of payment of fine R.I. for six
months
148 I.P.C. Six months R.I.
323/149 IPC Three months R.I.

All the sentences were directed to run concurrently. It was directed that in case the fine
amount imposed was realized, an amount of Rs. 20,000/- was to be paid as compensation
to the informant Pindi Dass who was injured and also a sum of Rs. 5,000/- was payable to
Ramesh Dass another injured victim.
The accused persons filed an appeal and learned Additional Sessions Judge, Karnal
reduced the sentence under Section 326 read with section 149 IPC to three years but
upheld all other sentences and the fine imposed by the trial Court. The accused persons
filed criminal revision before the High Court for quashing the judgment and order of
conviction of the trial Court. As noted above, the State of Haryana filed an appeal for
enhancement of the sentence and informant also filed a revision application. The High
Court dismissed the Criminal Appeal of the State and Criminal Revision of the injured for
enhancement of sentence but modified the judgment of courts below and directed release
of the accused persons on probation under Section 360 of the Code of Criminal
Procedure, 1973 (in short the 'Code') and enhanced the amount of fine to Rs. 15,000/-
each and directed 50% shall be payable to the injured Pindi Dass. It is to be noted that in
the meantime Pindi Dass has died.
3. Learned counsel for the informant submitted that the direction given by the High Court
cannot be maintained. Firstly, the Probation of Offenders Act, 1958 (in short the
'Probation Act') is applicable to the State of Haryana and therefore, Section 360 is not
applicable. In any event, life imprisonment is provided for an offence punishable under
Section 326, IPC. So, Section 360 of the Code also has no application.
4. Learned counsel for the accused respondents supported the judgment of the High
Court. It further submitted that considering the fact that incident took place long back,
this Court should not interfere by exercising jurisdiction under Article 136 of the
Constitution, 1950 (in short the 'Constitution'). The State of Haryana-appellant in
Criminal Appeal arising out of SLP (Cri.) No.5321 of 2006 supported the stand of the
informant who is appellant in Criminal appeal relatable to SLP (Crl.) No. 4646/2006.
5. Where the provisions of the Probation Act are applicable the employment of Section
360 of the Code is not to be made. In cases of such application, it would be an illegality
resulting in highly undesirable consequences, which the legislature, who gave birth to the
Probation Act and the Code wanted to obviate. Yet the legislature in its wisdom has
obliged the Court under Section 361 of the Code to apply one of the other beneficial
provisions; be it Section 360 of the Code or the provisions of the Probation Act. It is only
by providing special reasons
@page-SC1300
that their applicability can be withheld by the Court. The comparative elevation of the
provisions of the Probation Act are further noticed in sub-section (10) of Section 360 of
the Code which makes it clear that nothing in the said Section shall affect the provisions
of the Probation Act. Those provisions have a paramountcy of their own in the respective
areas where they are applicable.
6. Section 360 of the Code relates only to persons not under 21 years of age convicted for
an offence punishable with fine only or with imprisonment for a term of seven years or
less, to any person under 21 years of age or any woman convicted of an offence not
punishable with sentence of death or imprisonment for life. The scope of Section 4 of the
Probation Act is much wider. It applies to any person found guilty of having committed
an offence not punishable with death or imprisonment for life. Section 360 of the Code
does not provide for any role for Probation Officers in assisting the Courts in relation to
supervision and other matters while Probation Act does make such a provision. While
Section 12 of the Probation Act states that the person found guilty of an offence and dealt
with under Section 3 or 4 of the Probation Act shall not suffer disqualification, if any,
attached to conviction of an offence under any law, the Code does not contain parallel
provision. Two statutes with such significant differences could not be intended to co-exist
at the same time in the same area. Such co-existence would lead to anomalous results.
The intention to retain the provisions of Section 360 of the Code and the provisions of the
Probation Act as applicable at the same time in a given area cannot be gathered from the
provisions of Section 360 or any other provision of the Code. Therefore, by virtue of
Section 8(1) of the General Clauses Act, where the provisions of the Act have been
brought into force, the provisions of Section 360 of the Code are wholly inapplicable.
7. Enforcement of Probation Act in some particular area excludes the applicability of the
provisions of Sections 360, 361 of the Code in that area.
8

. The aforesaid position was highlighted in Chhanni v. State of U.P. (2006 (5) SCC 396)
and Daljit Singh and Ors. v. State of Punjab (2006 (6) SCC 159). 2006 AIR SCW 4062

9. Further, Section 360(1) of the Code itself provides that if for any offence life sentence
is provided for, section 360 of the Code would have no application.
10

. Undisputedly, in Jagdev Singh v. State of Punjab (1973 SCC (Crl.) 977) it was held that
Section 360 has no application since under Section 326 IPC the said offence carries life
imprisonment. In any event, that question is academic. AIR 1973 SC 2427
11. In view of what has been stated above, Section 360 of the Code has no application to
the facts of the present case. The High Court seems to have lost sight of the correct
position in law.
12. Above being the position, the matter is remitted to the High Court to consider about
the non-applicability of Section 360 of the Code both on the ground that the Probation
Act has application and Section 326 carries life imprisonment.
13. The appeals are allowed with no order as to costs.
Appeal allowed.
AIR 2008 SUPREME COURT 1300 "State of M. P. v. Hazarilal"
(From : Madhya Pradesh)*
Coram : 2 S. B. SINHA AND H. S. BEDI, JJ.
Civil Appeal No. 6498 of 2005, D/- 12 -2 -2008.
State of M.P. and Ors. v. Hazarilal.
(A) Constitution of India, Art.311 - M.P. Civil Services (Classification, Control and
Appeal) Rules (1966), R.19 - TERMINATION OF SERVICE - CIVIL SERVICE -
DISCIPLINARY AUTHORITY - HURT - Disciplinary Authority - Power to impose
punishment without inquiry in cases of conviction for criminal offence - Not to be
exercised irrespective of nature of offence or punishment which has been imposed -
Contingency peon - Conviction for offence of assault u/S.323. I.P.C. - Sentence of fine
only passed - Order of dismissal from service - Neither proper - Nor could be said to be
proportionate. (Paras 7, 8, 12)
(B) Constitution of India, Art.226 - WRITS - JUDICIAL REVIEW - DOCTRINES -
SERVICE MATTERS - Judicial review - Doctrine of proportionality - Has replaced
Wednesbury principle of unreasonableness. (Para 12)
@page-SC1301

Cases Referred : Chronological Paras


2007 AIR SCW 103 : AIR 2007 SC 548 (Ref.) 12
2007 AIR SCW 2688 : 2007 Lab IC 2021 (Ref.) 16
(2007) 2 All ER 273 (Ref.) 13
(2007) 3 All ER 1007 (Ref.) 13
(2007) 4 All ER 15 (Ref.) 13
(2007) 4 All ER 177 (Ref.) 13
(2007) 3 WLR 922 (Ref.) 13
2006 AIR SCW 1149 : AIR 2006 SC 3548 : 2006 Lab IC 1326 : 2006 (2) ALJ 803 (Ref.)
12
2006 AIR SCW 3120 : AIR 2006 SC 2360 (Ref.) 12, 16
(1996) QB 517 14
AIR 1987 SC 2386 : 1988 Cri LJ 158 (Ref.) 16
AIR 1985 SC 772 : 1985 Lab IC 590 (Rel. on) 3, 9, 10
Siddhartha Dave, Ms. Vibha Datta Makhija and Ms. Nemtiben Ao, for Appellants; Nitin
S. Tambwekar, B.S. Sai and K. Rajeev, for Respondent.
* W.P. No. 375 of 2003, D/- 28-4-2004 (MP).
Judgement
1. S. B. SINHA, J. :-Respondent was a Peon appointed in a Middle School. He is said to
have assaulted one Ram Singh on 5th October, 1989. He was prosecuted for commission
of the said offence and was convicted by a Court of Magistrate by a judgment dated 22nd
July, 1992 under Section 323 read with Section 34 of the Indian Penal Code and
sentenced to undergo one month's simple imprisonment. On an appeal preferred by him,
the sentence was reduced to a fine of Rs.500/- only. A revision there against was filed by
the respondent herein before the High Court.
2. A show cause notice was issued to the respondent as to why disciplinary action shall
not be taken against him in view of the judgment of conviction passed against him in the
said criminal case. By an order dated 25th November, 1993 his services were terminated
by the Deputy Director, Vidisha. An appeal thereagainst was preferred by the respondent
in terms of Madhya Pradesh State Services Act. However, no order was passed therein. A
revision was filed by him before the Deputy Director, Public Education. During the
pendency of the said revision application, his criminal revision petition filed before the
High Court was dismissed. The prayer of the respondent that he be reinstated in service
was rejected in terms of the order dated 11th January, 1994 passed by the Deputy
Director, Public Education, Vidisha.
3. Respondent thereafter filed an Original Application before the State Administrative
Tribunal, Gwalior. The Tribunal by an order dated 25th November, 2002 allowed the said
application holding. -

"However, the applicant succeeds on the ground that the punishment of removal from
service is grossly excessive because the punishment was only under section 323 IPC and
the High Court has clarified that the punishment does not involve any moral turpitude
every power vested in a public authority has to be exercised fairly, justly and reasonably.
Respondents should have applied their mind to the penalty which should be appropriately
be imposed in the circumstances of the case. Please see Shankar Das v. Union of India
(1985 (2) SCC 358). This does not seem to have been done." AIR 1985 SC 772

A writ petition filed thereagainst by the appellants before the High Court has been
dismissed by reason of the impugned judgment.
4. Mr. Siddhartha Dave, learned counsel appearing on behalf of the appellants, submitted
that the High Court committed a manifest error in passing the impugned judgment in so
far as it failed to take into consideration that the Tribunal or the High Court could not
have interfered with the quantum of punishment.
5. The case in hand appears to be a gross one. This Court is unable to appreciate the
attitude on the part of the appellant herein which ex-facie appears to be wholly
unreasonable. Respondent had not committed any misconduct within the meaning of the
provisions of the Service Rules. He was involved in a matter for causing simple injury to
another person. He was not even sent to prison. Only a sum of Rs.500/- was imposed
upon him as fine.
6. Rule 19 of M.P. Civil Services (Classification, Control and Appeal) Rules, 1966, which
provides for special procedure in certain cases, to which reliance has been placed by the
appellants does not appear to be applicable in the instant case. The said Rule reads thus :-
"19. Special procedure in certain cases.
Notwithstanding anything contained in Rule 14 to Rule 18
@page-SC1302
(i) where any penalty is imposed on a Government servant on the ground of conduct
which has led to his conviction on a criminal charge, or
(ii) where the disciplinary authority is satisfied for reasons to be recorded by it in writing
that it is not reasonably practicable to hold an inquiry in the manner provided in these
rules, or
(iii) where the Governor is satisfied that in the interest of the security of the State, it is not
expedient to hold any inquiry in the manner provided in these rules, the disciplinary
authority may consider the circumstances of the case and make such orders thereon as it
deems fit.
Provided that the Commission shall be consulted where such consultation is necessary,
before any orders are made in any case under this rule."
7. By reason of the said provision, thus, "the disciplinary authority has been empowered
to consider the circumstances of the case where any penalty is imposed on a Government
servant on the ground of conduct which has led to his conviction on a criminal charge",
but the same would not mean that irrespective of the nature of the case in which he was
involved or the punishment which has been imposed upon him, an order of dismissal
must be passed. Such a construction, in our opinion, is not warranted.
8. An authority which is conferred with a statutory discretionary power is bound to take
into consideration all the attending facts and circumstances of the case before imposing
an order of punishment. While exercising such power, the disciplinary authority must act
reasonably and fairly. Respondent occupied the lowest rank of the cadre. He was merely a
contingency peon. Continuation of his service in the department would not bring a bad
name to the State. He was not convicted for any act involving moral turpitude. He was
not punished for any heinous offence.
9

. The Tribunal, in our opinion, rightly placed reliance upon the decision of this Court in
Shankar Das v. Union of India, (1985) 2 SCC 358 wherein this Court commended the
judgment of a Magistrate of Delhi as he had let off the appellant therein under Section 12
of the Probation of Offenders Act stating :- AIR 1985 SC 772

"Misfortune dogged the accused for about a year........... and it seems that it was under the
force of adverse circumstances that he held back the money in question. Shankar Dass is
a middle-aged man and it is obvious that it was under compelling circumstances that he
could not deposit the money in question in time. He is not a previous convict. Having
regard to the circumstances of the case, I am of the opinion that he should be dealt with
under the Probation of Offenders Act, 1958."
10

. Despite the said observation Shankar Das was dismissed from service. This Court
held :- AIR 1985 SC 772

"7. It is to be lamented that despite these observations of the learned Magistrate, the
Government chose to dismiss the appellant in a huff, without applying its mind to the
penalty which could appropriately be imposed upon him insofar as his service career was
concerned. Clause (a) of the second proviso to Article 311 (2) of the Constitution confers
on the Government the power to dismiss a person from service "on the ground of conduct
which has led to his conviction on a criminal charge". But, that power, like every other
power, has to be exercised fairly, justly and reasonably. Surely, the Constitution does not
contemplate that a government servant who is convicted for parking his scooter in a no-
parking area should be dismissed from service. He may, perhaps, not be entitled to be
heard on the question of penalty since clause (a) of the second proviso to Article 311 (2)
makes the provisions of that article inapplicable when a penalty is to be imposed on a
government servant on the ground of conduct which has led to his conviction on a
criminal charge. But the right to impose a penalty carries with it the duty to act justly.
Considering the facts of this case, there can be no two opinions that the penalty of
dismissal from service imposed upon the appellant is whimsical."
11. We express similar dis-satisfaction in this case.
12

. Furthermore the legal parameters of judicial review has undergone a change.


Wednesbury principle of unreasonableness has been replaced by the doctrine of
proportionality. [See : Indian Airlines Ltd. vs. Prabha D.Kumari : (2006) 11 SCC 67;
State of U.P. vs. Sheo Shanker Lal Srivastava : (2006) 3 SCC 276 and M.P. 2007
AIR SCW 103
2006 AIR SCW 1149
2006 AIR SCW 3120

@page-SC1303
Gangadharan and another vs. State of Kerala and others : AIR 2006 SC 2360.)
13. At this stage we may also notice the application of the Doctrine by the United
Kingdom House of Lords in Seal (FC) (Appellant) vs. Chief Constable of South Wales
Police (Respondent) : [2007] 4 All ER 177; Huang (FC) (Respondent) v. Secretary of
State for the Home Department (Appellant) and Kashmiri (FC) (Appellant) vs. Secretary
of State for the Home Department (Respondent) (Conjoined Appeals) : [2007] 4 All ER
15; Tweed (Appellant) vs. Parades Commission for Northern Ireland (Respondents)
(Northern Ireland) [2007] 2 All ER 273; Belfast City Council (Appellants) vs. Miss
Behavin' Limited (Respondents) (Northern Ireland) [2007] 3 All ER 1007 and R (on the
application of Countryside Alliance and others) vs. Her Majesty's Attorney General and
another [2007] 3 WLR 922.
14. It is interesting to note that distinguishing between the traditional grounds of judicial
review and the doctrine of proportionality, Lord Carswell in Tweed (Supra) after referring
to previous decisions and authorities, observed :
"The starting point is that there is an overlap between the traditional grounds of review
and the approach of proportionality. Most cases would be decided in the same way
whichever approach is adopted. But the intensity of review is somewhat greater under the
proportionality approach. Making due allowance for important structural differences
between various convention rights, which I do not propose to discuss, a few
generalisations are perhaps permissible. I would mention three concrete differences
without suggesting that my statement is exhaustive. First, the doctrine of proportionality
may require the reviewing court to assess the balance which the decision maker has
struck, not merely whether it is within the range of rational or reasonable decisions.
Secondly, the proportionality test may go further than the traditional grounds of review
inasmuch as it may require attention to be directed to the relative weight accorded to
interests and considerations. Thirdly, even the heightened scrutiny test developed in R v.
Ministry of Defence, Ex p Smith [1996] QB 517, 554 is not necessarily appropriate to the
protection of human rights."
15. Applying the said principle also, in our opinion, no interference with the impugned
judgment is called for.
16

. Reliance has been placed by the learned counsel on Coimbatore District Central
Cooperative Bank vs. Coimbatore District Central Cooperative Bank Employees
Association and another, (2007) 4 SCC 669 wherein also this Court accepted the
applicability of the doctrine of proportionality. Therein this Court has quoted with
approval the decision of this Court in Ranjit Thakur vs. Union of India and others :
AIR 1987 SC 2386
2006 AIR SCW 3120

(1987) 4 SCC 611 as also M.P. Gangadharan and another vs. State of Kerala and others :
(2006) 6 SCC 162, which had applied the doctrine of proportionality.
17. For the reasons aforementioned there is no merit in this appeal which is dismissed
with costs. Counsel fee is quantified at Rs. 25,000/-.
Appeal dismissed.
AIR 2008 SUPREME COURT 1303 "Sirisia Sthal, Imli Chati, Muzaffarpur v. State of
Bihar"
(From : Patna)*
Coram : 2 Dr. A. PASAYAT AND P. SATHASIVAM, JJ.
Civil Appeal No. 1001 of 2002, D/- 11 -2 -2008.
Sirisia Sthal, Imli Chati, Muzaffarpur and Ors. v. State of Bihar and Ors.
Constitution of India, Art.226 - WRITS - HIGH COURT - LAND CEILING - LAND
REFORMS - Powers of High Court - Petition filed before High Court challenging vires
of certain provisions of Bihar Land Reforms (Fixation of Ceiling Area and Acquisition of
Surplus Land) Act - High Court observed that enactment/amendment was included in 9th
Schedule of Constitution though it was not - Order of High Court liable to be set aside -
Matter remitted for fresh consideration.
C.W.J.C. No. 10233 of 1995, D/-13-03-2001 (Pat), Reversed. (Para 6)

S.B. Sanyal, Sr. Adv., Ranjan Mukherjee, for Appellants; Saurabh Kirpal, Gopal Singh,
for Respondents.
* C.W.J.C. No. 10233 of 1995, D/- 13-3-2001 (Pat).
Judgement
Dr. ARIJIT PASAYAT, J. :- Challenge in this appeal is to the order passed by a
@page-SC1304
division Bench of the Patna High Court dismissing the writ petition filed by the
appellants. The writ petition was filed challenging vires of certain provisions of Bihar
Land Reforms (Fixation of Ceiling Area and Acquisition of Surplus Land) Act, 1961 (in
short 'the Act'). The writ petition filed by the appellants was dismissed on the ground that
no return was filed, and after preparation of draft statements they could have got
opportunity to file objection. It was held that the writ petition was filed challenging vires
of an enactment which was included in 9th Schedule of the Constitution of India, 1950
(in short the 'Constitution').
2. In support of the appeal learned counsel for the appellants submitted that since the
vires of certain provision were being challenged and the amendment to Section 29 of the
Act was under challenge, the question of filing return did not arise. Further it was
submitted that the amendment was not included in the 9th Schedule as was observed by
the High Court. Earlier, all the writ petitioners were granted exemption under Section
29(2) (a)(ii) of the Act to hold an extra unit required for the purpose of performing
religious rites and its maintenance but by the amendment the same was taken away.
3. Learned counsel for the respondent-State on the other hand submitted that though the
amendment was not part of the 9th Schedule to the Constitution, yet the effect of the
amendment is that the power to exempt stood deleted with retrospective effect.
4. Prayers in the writ petition were to the following effect :
"It is therefore, prayed that your Lordships may graciously be pleased to admit this
application, issue Rule NISI against the respondents calling upon them to show-cause as
to why the Section 2 of the impugned ordinance (Annexure 1) and the directions
contained in Annexure 2 be declared ultra vires of the Constitution of India and quashed
after hearing the party or parties, rule may be made absolute;
And/or
ii) That such order, writ, direction or order may be passed to your Lordships as may deem
fit and proper in the facts and circumstances of the case."
5. Subsequently, the prayers were amended in the following terms :
"It is, therefore, prayed that the prayer portion of the writ application be kindly permitted
to be amended as followed in the light of the facts stated above :-
"That after first prayer in the writ petition, the following be added :-
RULE NISI be also issued against the respondents calling upon them to show cause as to
why Section 2 of the impugned Bihar Act 8 of 1997 (Annexure 3 and the directions
contained in para 5 (Gha)(vi) of the Annexure 4 be not declared ultra vires the
Constitution of India and quashed and after hearing the parties RULE NISI be made
absolute."
6. Since the High Court has not applied its mind to the challenge raised and has
erroneously referred to the 9th Schedule to the Constitution, it would be appropriate to set
aside the impugned order of the High Court and remit the matter to it for fresh
consideration in accordance with law. Since the writ petition is of the year 1995, the High
Court is requested to take up the matter early and decide the writ petition as early as
practicable, preferably by the end of October, 2008.
7. The appeal is allowed to the extent indicated without any order as to costs.
Order accordingly.
AIR 2008 SUPREME COURT 1304 "Jagadeesh v. State of Karnataka"
(From : Karnataka)*
Coram : 2 TARUN CHATTERJEE AND AFTAB ALAM, JJ.
Civil Appeal No. 3377 of 2001, D/- 12 -2 -2008.
Jagadeesh and Anr. v. State of Karnataka and Ors.
Karnataka Land Reforms Act (10 of 1962), S.121A - LAND REFORMS - REVISION -
CONCURRENT FINDINGS - TRIBUNALS - SPECIAL LEAVE APPEAL - Revision -
Power exercised by High Court - Is wider than one exercised u/S.115, CPC - High Court
setting aside concurrent findings of fact recorded by Tribunal - Findings of High Court
based on material evidence on record - Not liable to be interfered with under Art.136 of
Constitution.
Constitution of India, Art.136.
The revisional power exercised by the High Court under S. 121-A is wider than the one
exercised by the High Court in its
@page-SC1305
revisional jurisdiction under S. 115 of CPC. Since S. 121-A of the Act clearly empowers
the High Court to look into the legality of the orders impugned, therefore, it would be
open to the High Court to consider the material evidence on record when it finds that
such evidence was not at all considered by the Tribunals below or when the conclusion
arrived at by the Tribunals below run contrary to the materials on record or when it finds
that there is no evidence to support the conclusion of the Tribunals below or that the
reasons given by the Tribunals below are absolutely perverse or a finding was such that
no Court would come to such a conclusion or that the decisions of the Tribunals below
were manifestly unjust. Therefore, under S. 121-A of Act, in the presence of any of the
above-mentioned circumstances, the High Court is empowered to look into the legality of
the orders impugned in deciding the question whether the appellants could be held to be
the tenants. The findings of the High Court, while setting aside the concurrent findings of
fact of the Tribunals, which were based on consideration of the material evidence on
record, it cannot be said that the High Court was not justified in setting aside the said
concurrent findings of fact in the exercise of its jurisdiction under S. 121-A. (Paras
1,5, 6, 7, 8, 9, 10, 12, 13, 17, 18)
Since the material evidence on record was not considered at all by the Tribunal and non-
consideration of the material evidence on record is a question of law, the High Court was
entitled to interfere.
(1999) 1 SCC 74, Disting. (Para 14)
Furthermore the view taken by the High Court was plausible. Therefore, the question of
interference by Supreme Court under Art. 136 of Constitution is not warranted. (Para
16)
Cases Referred : Chronological Paras
2000 AIR SCW 2065 : AIR 2000 SC 2276 (Ref.) 17
1999 AIR SCW 755 : AIR 1999 SC 1104 (Ref.) 17
1999 AIR SCW 766 : AIR 1999 SC 1114 (Ref.) 14
1999 AIR SCW 2306 (Ref.) 14
(1997) 10 SCC 305 (Ref.) 16
AIR 1964 SC 1320 (Ref.) 14
N.D.B. Raju, Mrs. Bharathe Raju and Guntur Prabhakar, for Appellants; S.N.Bhat,
Sanjay R. Hegde, Vikrant Yadav and Amit Kr. Chawla, for Respondents.
* L.R.R.P. No. 2420 of 1990, D/- 20-7-2000 (Kant).
Judgement
TARUN CHATTERJEE, J. :- In our view, although the High Court had set aside the
concurrent findings of fact arrived at by the Tribunals below under the Karnataka Land
Reforms Act, 1974 (in short 'the Act') in the exercise of its revisional jurisdiction under
Section 121A of the Act, even then, this is not a fit case where this Court, in the exercise
of its power under Article 136 of the Constitution would interfere with such an order of
the High Court.
2. The appellants in this appeal, claiming to be the tenants of agricultural land, bearing
Survey No. 125/1, measuring 3 acres 11 Gunthas (hereinafter called as the "scheduled
land") situated in Lingabahalli Village, Madhugiri Taluk in the State of Karnataka, filed
Form No. 7 before the Land Tribunal praying for a declaration that they had acquired
occupancy rights in respect of the scheduled land. They alleged that they were cultivating
the scheduled land from 1968 till the notified date under the Act on Wara basis giving
1/3rd of the share in the foodgrains to respondent No.4. Accordingly, the appellants
prayed for an order of occupancy right in respect of the scheduled land alleging that they
and their father were cultivating the scheduled land as occupancy right holders relying,
inter alia, on the entries under the RTC record.
3. The case of the appellants, as made out, was disputed by the respondent No. 4. The
case of respondent No. 4 was that the scheduled land was mortgaged to the 3rd
respondent, Rajashankar, in the year 1968 and after the expiry of the said mortgage, the
mortgagee was liable to deliver possession of the same. The case of tenancy as made out
by the appellants or their father was denied. It was alleged by the respondent No. 4 that
since the respondent No.3 was a film actor and had settled in Madras (now Chennai),
with the consent of the respondent No. 3, the scheduled land was given to the father of
the appellants and the father of the appellants was cultivating the same from the year
1968 but not as a tenant. Accordingly, they prayed for rejection of the application filed by
the father of the appellants claiming occupancy rights under the Act. Initially, the Land
Tribunal allowed the application of the father of the appellants and feeling aggrieved, a
writ petition was filed against the said order. The High
@page-SC1306
Court had set aside the order of the Land Tribunal and remanded the case back to the
Tribunal for a fresh decision. The Land Tribunal, after remand, relying on the entries in
the RTC record and some other materials on record, granted occupancy rights in favour
of the appellants.
4. Feeling aggrieved, the respondent No. 4 filed an appeal before the Appellate Authority,
which was also dismissed. A revision petition, thereafter, was moved before the High
Court and the High Court, by the impugned judgment, had set aside the concurrent
findings of fact and rejected the application filed by the father, since deceased, of the
appellants holding, inter alia, that the appellants or their father had failed to prove the
tenancy in respect of the scheduled land. A special leave petition was filed against the
judgment of the High Court, setting aside the concurrent orders allowing the application,
in respect of which leave has already been granted.
5. We have heard Mr. Raju, learned counsel appearing on behalf of the appellants and Mr.
S. N. Bhat, learned counsel appearing on behalf of the respondents. We have examined
the impugned judgment of the High Court as well as the orders of the Tribunals below. It
is true that the High Court, while exercising its revisional power under Section 121A of
the Act, had set aside the concurrent findings of fact of the Land Tribunal as well as of
the appellate authority, even then, examining the findings of the High Court and
considering the power conferred on it in the revisional jurisdiction under Section 121A of
the Act, we do not find any reason to interfere with the impugned order of the High Court
in the exercise of our power under Article 136 of the Constitution. While setting aside the
findings of the Tribunal, the High Court, at paragraph 7 of the impugned judgment made
the following findings :-
"It is an undisputed fact that the revision petitioner has mortgaged the land in dispute in
favour of the 5th respondent, Rajashankar in the year 1968 and after the expiry of the
mortgage period, since the 5th respondent failed to deliver back the possession of the
land in dispute to him, he filed the suit for redemption and obtained a decree for
redemption. When the matter stood thus, the father of the respondent Nos. 3 and 4
Gondappa, who is the uncle of the 5th respondent, Rajashankar, filed Form No. 7 before
the Land Tribunal claiming occupancy rights in respect of the land in dispute contending
that he is the tenant of the said land, under the 5th respondent from the year 1968, i.e.
subsequent to the date of mortgage. To prove this fact, he relied upon the entries in the
R.T.C. extract for the years 1968 to 1974 wherein his name is shown as the person in
cultivation of the land in dispute. But, it is significant to note that the nature of cultivation
of the land is not shown as that of a tenant in the said R.T.C. extracts. In one year, the
nature of cultivation is described as "Swantha" and in the years, the column is left blank.
Thus the R.T.C. extracts produced by him do not support his contention that he was
cultivating the land in dispute as a tenant. He has not produced any Geni receipts or any
Lease Agreement to show that the 5th respondent has leased out the land in dispute in his
favour on crop share basis and that he paid the Geni to the 5th respondent. Thus, he has
no documentary evidence in respect of his claim that he came in possession of the land in
dispute as a tenant under the 5th respondent and that he was cultivating the land in
dispute as a tenant. It is further significant to note that in the evidence given by the
respondent No. 3 before the Land Tribunal, he claimed that his father has taken the land
in dispute on lease in the year 1962, from the father of the petitioners, Gundu Rao. Even
in respect of the said claim, he failed to produce any documentary evidence evidencing
the said lease of land in dispute from Gundu Rao. On the other hand, in Form No. 7 filed
by Gondappa, the father of the respondents 3 and 4, he alleged that he was the tenant
under the 5th respondent in respect of the land in dispute from the year 1968. Thus, there
is no consistent stand regarding the year of commencement of tenancy or under whom,
Gondappa, the father of the respondents 3 and 4 became the tenant. So, the only question
which arises for consideration is whether the said cultivation of the land in dispute by the
father of the respondents 3 and 4 during the years 1968 to 1974 can be presumed to be
that of a tenant under the provisions of Section 4 of the Karnataka Land Reforms Act.
Section 4 of the Act makes it clear that a member of the owner's family cannot be
considered as a deemed tenant, even if he is lawfully cultivating the land belonging to
owner. In the present case, since the father of the respondents 3 and 4,
@page-SC1307
is the uncle of the respondent No. 5, it cannot be said that he is not a member of the
family of the respondent No. 5. Though there is no evidence on record to show that there
are any joint family properties belonging to the joint family of respondent No. 5 and his
uncle, there is nothing on record to show that they are not living as members of the joint
family. So, it is not possible to presume that the father of the respondent Nos. 3 and 4
Gondappa, who is the uncle of respondent No. 5 was not the member of the family of the
mortgagee, respondent No. 5."
6. Again, the High Court, while setting aside the findings of fact also made the following
findings :-
"But in the instant case, since the respondents 3 and 4 failed to produce any evidence to
show that their father was cultivating the land in dispute as a tenant under the 5th
respondent mortgagee and even when the entries in the R.T.C. extract produced do not
support the claim of tenancy set up by the father of the respondents 3 and 4, the question
of drawing presumption of deemed tenancy in his favour under Section 4 of the Act does
not arise. The father of the respondents 3 and 4, being the uncle of respondent No. 5-
Mortgagee, it is also quite possible that he might have been allowed to cultivate the land
in dispute under the personal supervision of respondent No. 5 by assisting him in
cultivation of the said land. . . . . . . . . . . . . . . .
In the present case also, the respondents 3 and 4 failed to prove that their father was
cultivating the land in dispute from the year 1968 as a tenant under the respondent No. 5
and that after the death of their father, they continued as tenants in respect of the land in
dispute............... It is also significant to note that the respondent No. 5, who was alive
when the enquiry was pending before the Land Tribunal has not given evidence in favour
of the respondents 3 and 4 stating that he has leased out the land in dispute in favour of
the respondents 3 and 4. Except the interested testimony of respondents 3 and 4, there is
nothing else on record to show that their father was inducted as a tenant to cultivate the
land in dispute by the 5th respondent after the land is dispute was taken on mortgage by
him. So, it is not possible to presume that the father of the respondents 3 and 4 was
inducted as a tenant by the mortgagee, the 5th respondent, in respect of the land in
dispute...........Since the respondents 3 and 4 failed to produce any documentary evidence
to show that their father was put in possession of the land in dispute by the 5th
respondent, mortgagee as a 'tenant' and that they are continuing as tenants in respect of
the said land after the death of their father, I find that they are not entitled to grant of
occupational rights ........... The earlier decision of this Court reported in ILR 1996 Kar
page 2340 that when a person fails to prove that he is cultivating the land as tenant, he
cannot be granted occupational right notwithstanding the fact that he might be in
possession of the land and cultivating the same, is applicable to the facts of the present
case on all fours."
7. From a careful examination of the findings given by the High Court, as quoted
hereinabove, in upsetting the concurrent findings of fact arrived at by the Tribunals
below, we are not in a position to hold that the High Court was not justified in setting
aside the concurrent orders of the Tribunals below in the exercise of its revisional power
under Section 121A of the Act. The power conferred on the High Court to revise the
orders of the Tribunals below has been provided in Section 121A of the Act, which runs
as under :-
"The High Court may at any time call for the records of any order or proceeding recorded
by the Appellate Authority under this Act or any other law for the purpose of satisfying
itself as to the legality of such order or as to the regularity of such proceeding and may
pass such order with respect thereto as it thinks fit".
8. From a plain reading of Section 121A of the Act, under which revisional jurisdiction
can be exercised, it would be clear that the High Court, while exercising such power is
entitled to re-appreciate the evidence when it finds that the conclusion arrived at by the
appellate authority runs contrary to the materials on record and when it finds that there is
no evidence to support the conclusion of the appellate authority or when it finds that the
reasons given by the appellate authority are absolutely perverse and cannot be supported
by the evidence on record. It would also be clear from a plain reading of Section 121A of
the Act that the High Court is also entitled to interfere with the orders of the Tribunals
below when the material evidence on record was ignored or a finding was such that no
court would come
@page-SC1308
to such conclusion or that the decision of the Tribunals below was manifestly unjust.
9. We have carefully examined the provisions under Section 121A of the Act, which is the
revisional power under the Act, and also the provisions under Section 115 of the Code of
Civil Procedure (for short "the Code"). So far as Section 115 of the Code is concerned, it
has been made clear that it is only in case of a jurisdictional error or when the courts
below had acted with material irregularity in the exercise of their jurisdiction that the
question of interfering with such an order can arise, otherwise, the High Court is not
entitled to interfere with any other order which does not satisfy the conditions laid down
for interference under Section 115 of the Code. On the other hand, in our view, under
Section 121A of the Act, it would be open to the High Court to interfere with the orders
of the Tribunals below as the High Court is empowered to look into the legality of the
order or regularity of the proceedings although, in the exercise of revisional jurisdiction
under section 115 of the Code, the High Court is not entitled to look into the legality of
the order or the regularity of the proceedings but only entitled to interfere with the orders
of the Tribunals or the courts below when it finds that they have (a) exercised a
jurisdiction not vested in them by law, or (b) failed to exercise a jurisdiction so vested, or
(c) acted in the exercise of their jurisdiction illegally or with material irregularity.
Reading the aforesaid provisions viz., Section 121A of the Act and Section 115 of the
Code, we have no hesitation in our mind to hold that the revisional power exercised by
the High Court under section 121A of the Act is wider than the one exercised by the High
Court in its revisional jurisdiction under Section 115 of the Code. As noted hereinearlier,
since section 121A of the Act clearly empowers the High Court to look into the legality of
the orders impugned, therefore, it would be open to the High Court to consider the
material evidence on record, when it finds that such evidence was not at all considered by
the Tribunals below or when the conclusion arrived at by the Tribunals below run
contrary to the materials on record or when it finds that there is no evidence to support
the conclusion of the Tribunals below or that the reasons given by the Tribunals below
are absolutely perverse or a finding was such that no court would come to such a
conclusion or that the decisions of the Tribunals below were manifestly unjust. Therefore,
under section 121A of the Act, in the presence of any of the abovementioned
circumstances, the High Court is empowered to look into the legality of the orders
impugned in deciding the question whether the appellants could be held to be the tenants
under the respondent Nos. 3 or 4.
10. Keeping the aforesaid principles in mind as to when the High Court would be
justified, in the exercise of its power under Section 121A of the Act, to examine the
legality of the orders of the Tribunals below in an appropriate case, let us now examine
the findings of the High Court, while setting aside the concurrent findings of fact of the
Tribunals below. In our view, on a careful examination of the findings of the High Court,
which were based on consideration of the material evidence on record, it is difficult for us
to hold that the High Court was not justified in setting aside the concurrent findings of
fact of the Tribunals below in the exercise of its jurisdiction under Section 121A of the
Act.
11. We have already noted the findings made by the High Court in the impugned
judgment on the question whether the appellants could be held to be the tenants on the
evidence and materials on record. While doing so, in our view, the High Court was
justified in coming to the conclusion that the evidence and material on record would
clearly establish that the appellants were not able to prove that they were the tenants in
respect of the scheduled land under the respondents. One of the main criteria for deciding
whether a particular person is a tenant or not is to see whether there was payment of rent,
either in cash or in kind. In this case, while rejecting the claim of the appellants, the High
Court had considered that the appellants had failed to satisfy the court that any payment
of rent was made either by the father of the appellants or by the appellants themselves.
12. The Tribunals below, while accepting the case of the appellants, had relied on the
entries made in the RTC record in respect of certain period. While considering such
entries, the High Court had rightly held that from the entries in the RTC record for the
years 1968 to 1974, the name of the appellants was not shown as the person in cultivation
of the land in dispute and also the
@page-SC1309
nature of cultivation of the scheduled land was not shown as that of the tenants in the said
RTC record. That being the position, the High Court had come to a proper conclusion that
the entries in the RTC extracts produced by the appellants could not support the
contention that they were cultivating the land in dispute as the tenants. In our view also,
the High Court was fully justified in drawing an adverse inference against the appellants
for not producing any Geni receipts or any lease agreement to show that the 5th
respondent before the High Court (respondent No. 3 herein) had, in fact, leased out the
scheduled land in favour of the appellants or their father, since deceased, on crop share
basis and that the appellants had paid the Geni to the 5th respondent. Such being the
findings arrived at by the High Court with which we are in concurrence, it is difficult to
hold that the tenancy claimed by the appellants in respect of the scheduled land could be
established.
13. Considering the above aspect of the matter and after considering the scope of Section
121A of the Act, we are, therefore, unable to agree with the learned counsel for the
appellants that in the exercise of revisional jurisdiction under Section 121A of the Act,
the High Court was not entitled to set aside the concurrent findings of fact arrived at by
the appellate authority and the land Tribunal. Such being the position, we do not find any
reason to interfere with the judgment of the High Court, although the High Court, in the
exercise of its power under Section 121A of the Act, had set aside the concurrent orders
of the appellate authority as well as the Land Tribunal.
14

. Mr. Raju, the learned counsel appearing on behalf of the appellants, however, contended
before us that it was not open to the High Court, in the exercise of its revisional
jurisdiction under Section 121A of the Act, to interfere with the concurrent findings of
fact arrived at by the appellate authority and the Land Tribunal. In support of his
contention, he had relied on a decision of this Court in the case of Dahya Lal and Ors. vs.
Rasul Mohammed Abdul Rahim [1963 (3) SCR 1]. He also relied on a decision of this
Court in the case of Mohan Balaku Patil and Ors. vs. Krishnoji Bhaurao Hundre (Dead)
by L. Rs. [(2000) 1 SCC 518] and Krishtappa Yellappa Pujar and Ors. vs. Ram Samsthan
Beladhadi [(1999) 1 SCC 74]. In our view, so far as the decision in the case of Mohan
Balaku Patil and Ors. vs. Krishnoji Bhaurao Hundre (Dead) by LRs. [(2000) 1 SCC 518]
is concerned, it is difficult to conceive how this decision could be of any help to the
appellants. In that case, the findings recorded by the appellate authority as affirmed by
the High Court by placing reliance on the entries made in the record of rights to the effect
that the appellants were not in possession of the land on the relevant date nor were they
cultivating the same, were not accepted by this court. In any view of the matter, in that
decision, relying on the aforesaid findings, this Court also had set aside the order made
by the appellate authority as affirmed by the High Court in revision and restored the order
made by the Land Tribunal. If that case is of any help to the facts of the present case, it
would be in favour of the respondents. So far as Krishtappa Yellapa Pujar and
Ors. vs. Ram Samsthan Beladhadi [(1999) 1 SCC 74] is concerned, we again fail to
understand that how this could be of any help to the appellants. In that decision, it has
been made clear that the High Court was entitled to interfere with the orders of the
appellate authority only on question of law or irregularity in procedure and on no other
aspect. In our view, we have already held that the High Court was entitled to interfere
with the concurrent orders of the tribunals below as material evidence on record was not
considered at all and non consideration of the material evidence on record is a question of
law and, therefore, the High Court was entitled to interfere. Accordingly, this decision is
of no help to the appellants. Lastly, in our view, in view of the discussion made herein
above, the decision relied on by the learned counsel for the appellant in the case of Dahya
Lal and Ors. vs. Rasul Mohammed Abdul Rahim [1963 (3) SCR 1] need not be discussed.
AIR 1964 SC 1320
1999 AIR SCW 766
1999 AIR SCW 2306
1999 AIR SCW 766
1999 AIR SCW 2306
AIR 1964 SC 1320

15. There is another aspect of this matter. Even assuming that the High Court was not
justified in setting aside the concurrent findings of fact in the exercise of its revisional
jurisdiction under Section 121A of the Act, then also, we are of the view that it is not a fit
case where this Court should interfere with the impugned judgment of the High Court in
the exercise of our power under
@page-SC1310
Article 136 of the Constitution.
16. In Union of India and Ors. vs. Gangadhar Narsingdas Aggarwal and Anr. [(1997) 10
SCC 305], this Court, while declining to interfere with the order of the High Court in the
exercise of its power under Article 136 of the Constitution, held that even if two views
are possible, the view taken by the High Court being a plausible one, it would not call for
intervention by this Court under Article 136 of the Constitution. Considering the
concurrent orders of the appellate authority and the land tribunal and the impugned order
of the High Court, we are in agreement with the High Court because the view taken by it
was plausible and therefore, the question of interference by us under Article 136 of the
Constitution is not warranted.
17

. Again in Jai Mangal Oraon v. Mira Nayak (Smt.) and Ors. [(2000) 5 SCC 141], this
Court had laid down that when there was nothing illegal and wrong in the reasoning and
conclusions arrived at by the High Court and the same appeared to be well merited and in
accordance with the interpretation of statutory provisions, this Court would not interfere
with the order of the High Court under Article 136 of the Constitution. We have already
considered the findings made by the High Court while setting aside the concurrent orders
of the tribunals below and found that the same appear to be well merited and in
accordance with the material evidence on record, therefore, this Court would not interfere
with the order of the High Court under Article 136 of the Constitution. Finally in
Taherakhatoon (D) By LRs. v. Salambin Mohammad [(1999) 2 SCC 635], this Court at
paragraph 20 has observed as follows : 2000 AIR SCW 2065
1999 AIR SCW 755, Para 20

"In view of the above decisions, even though we are now dealing with the appeal after
grant of special leave, we are not bound to go into merits and even if we do so and
declare the law or point out the error - still we may not interfere if the justice of the case
on facts does not require interference or if we feel that the relief could be moulded in a
different fashion ............."
18. In view of the aforesaid, we are, therefore, of the view that this is not a fit case where
this Court shall interfere with the order passed by the High Court under Section 121A of
the Act.
19. For the reasons aforesaid, this appeal fails and is dismissed without any order as to
costs.
Appeal dismissed.
AIR 2008 SUPREME COURT 1310 "Life Insurance Corporation of India v. Jaya
Chandel"
Coram : 2 Dr. A. PASAYAT AND P. SATHASIVAM, JJ.
Civil Appeal No. 1089 of 2008 (arising out of SLP (C) No. 12792 of 2005), D/- 7 -2
-2008.*
Life Insurance Corporation of India v. Jaya Chandel.
Life Insurance Corporation Act (31 of 1956), S.43 - Insurance Act (4 of 1938), S.64VB -
LIFE INSURANCE CORPORATION - INSURANCE - Life policy - Lapsed for
nonpayment of premium - Revival - Amount of unpaid premium sent before death of
assured but received after death - Date of sending of cheque - Cannot be taken as date of
receipt of premium for purpose of revival - S.64-VB of Insurance Act does not apply to
life policy - Repudiation of claim proper. (Para 7)

P.S. Patwalia, Sr. Advocate, Ms. Indra Sawhney, for Appellant; Ms. Pragati Neekhra,
Suryanarayana Singh, Dharmendra Kumar Sinha, for Respondent.
* From order of the National Consumer Disputes Readressal Commission, New Delhi in
R.P. No. 2068 of 2001, D/- 17-1-2005.
Judgement
Dr. ARIJIT PASAYAT, J. :-Leave granted.
2. Challenge in this appeal is to the order passed by the National Consumer Disputes
Redressal Commission (in short the "National Commission") dismissing the Revision
Petition filed by the appellant. Challenge before the Commission was to the order passed
in appeal by the Himachal Pradesh State Consumer Disputes Redressal Commission,
Shimla (in short the 'State Commission') which in turn had upheld the order passed by the
District Forum, Shimla (in short the 'District Forum').
Background facts in a nutshell are as follows :
One Karan Singh Chandel (hereinafter referred to as the 'deceased') had taken a Life
Insurance Policy and was insured for a sum of Rs. 1,50,000/-. The annual premium
payable was Rs. 12,821/-. The policy was taken on 28.3.1994. The annual premium
@page-SC1311
which was to be paid on or before 28.3.1995 was not paid. In terms of the policy, the
same became inoperative after one month. The insured died on 1.7.1995. A cheque drawn
on Jogindra Co-operative Bank Ltd. for an amount of Rs. 12,821/- purportedly on
account of premium along with late fee of Rs. 189/- was issued by one Prakash Chand
Thakur on 27.6.1995. The same was received on 12.7.1995. According to the claimant i.e.
widow of the deceased, the cheque was issued before the death of the insured and
therefore, the appellant could not have repudiated the claim.
3. The stand of the present appellant was that the policy had lapsed due to non-payment
of premium in time. This plea was not accepted by the District Forum on the ground that
the cheque was claimed to have been issued on 12.7.1995, but is presumed to have been
received earlier than that date. The State Commission held that in any event the amount
was received within the grace period and therefore, the claim could not have been
repudiated. Accordingly the appeal filed by the appellant was dismissed. The National
Forum dismissed the Revision holding that Section 64-VB of the Insurance Act, 1938 (in
short the 'Insurance Act') was applicable where the premium is tendered by postal money
order or cheque sent by post and the risk may be assumed on the date on which the
money order is booked or the cheque is posted, as the case may be. Therefore, it was held
that there was revival. It did not accept the stand of the appellant that the revival was not
a matter of right.
4. In support of the appeal, learned counsel for the appellant submitted that the District
Forum, the State Commission and the National Commission failed to notice certain
relevant factors. It was not explained as to why the cheque was issued by Prakash Chand
Thakur and not by the insured. This is sufficient to show that subsequently a cheque was
issued to regularize the policy. Further the cheque was received on 12.7.1995 much after
the death and this itself is sufficient to show that the cheque was not issued prior to the
death of the insured. The extract of the receipt register has been filed which shows that
the cheque was received on 12.7.1995. The State Commission came to the conclusion
that the cheque was issued during the grace period. This is also factually incorrect
because the grace period is 30 days, the premium was due on 28.3.1995 and the cheque
was issued much beyond the grace period. Additionally, Section 64-VB does not apply to
the appellant. In this context Section 43 of the Life Insurance Corporation Act, 1956 (in
short the 'Act') has relevance. Reference is also made to Condition 2 of the policy.
5. In reply learned counsel for the claimant submitted that it is not Condition 2 of the
policy which is applicable, but Condition No. 3 which is applicable. It is stated that no
adverse inference can be drawn because the insured had not signed the cheque and
merely because the cheque was received after the death of the deceased that does not
entitle the appellant to refuse a genuine claim.
Conditions 2 and 3 of the policy read as follows :
"2. Payment of premium : A grace period of one month but not less than 30 days will be
allowed for payment of yearly, half-yearly or quarterly premiums and 15 days for
monthly premiums. If death occurs within this period and before the payment of the
premium then due, the Policy will still be valid and the sum assured paid after deduction
of the said premium as also the unpaid premiums falling due before the next anniversary
of the Policy. If premium is not paid before the expiry of the days of grace the Policy
lapses. If the Policy has not lapsed and the claim is admitted in case of death under a
Policy where the mode of payment of premium is other than yearly, unpaid premiums if
any falling due before the next Policy anniversary shall be deducted from the claim
amount."
"3. Revival of discontinued Policies : If the Policy has lapsed it may be revived during
the lifetime of the Life Assured, but within a period of 5 years from the date of the first
unpaid premium and before the date of maturity, on submission of proof of continued
insurability to the satisfaction of the Corporation and the payment of all the arrears of
premium together with interest at such rate as may be fixed by the Corporation from time
to time compounding half yearly. The Corporation reserves the right to accept or decline
the revival of discontinued policy. The revival of a discontinued policy shall take effect
only after the same is approved by the Corporation and is specifically communicated to
the life assured."
6. The grace period is one month and
@page-SC1312
therefore the State Commission was not justified in holding that the payment was made
within the grace period. Condition 3 relates to revival of discontinued policy. A bare
reading of the condition shows that it can be revived during the lifetime of the assured. In
the instant case the cheque was admittedly received after the death of the assured. Further
the revival takes effect only after the same is approved by the Corporation and is
specifically communicated to the life insured. In the present case this is not the situation.
Further Section 43 of the Act reads as follows :
43. Application of the Insurance Act.
(1) The following section of the Insurance Act shall, so far as may be, apply to the
Corporation as they apply to any other insurer, namely :-Sections 2, 2B, 3, 18, 26, 33, 38,
39, 31, 45, 46, 47A, 50, 51, 52, 110A, 110B, 110C, 119, 121, 122 and 123.
(2) The Central Government shall as soon as may be after the commencement of this Act,
by notification in the Official Gazette, direct that the following sections of the Insurance
Act shall apply to the Corporation subject to such conditions and modifications as may be
specified in the notification, namely :-Sections 2D, 10, 11, 13, 14, 15, 20, 21, 22, 23, 25,
27A, 28A, 35, 36, 37, 40, 40A, 43, 44,102 to 106, 107 to 110, 111, 113, 114 and 116A.
1[(2A)] Section 42 of the Insurance Act shall have effect in relation to the issue to any
individual of a licence to act as an agent for the purpose of soliciting or procuring life
insurance business for the Corporation as if the reference to an officer authorised by the
Controller in this behalf in sub-section (1) thereof included a reference to an officer of the
Corporation authorised by the Controller in this behalf.]
(3) The Central Government may, by notification in the Official Gazette, direct that all or
any of the Insurance Act other than those specified in sub-section (1) or sub-section (2)
shall apply to the Corporation subject to such conditions and modifications as may be
specified in the notification.
(4) Every notification issued under subsection (2) or sub-section (3) shall be laid for not
less than thirty days before both Houses of Parliament as soon as possible after it is
issued, and shall be subject to such modifications as Parliament may make during the
session in which it is so laid or the session immediately following.
(5) Save as provided in this section, nothing contained in the Insurance Act shall apply to
the Corporation."
7. Section 43 of the Act enumerates the various Sections of Insurance Act which have
application to the Act and Section 64-VB is not one of them. That being so also the
National Commission was not justified in its conclusion about the applicability of that
provision.
8. Looked at from any angle the orders passed by the District Forum, the State Forum and
National Commission cannot be maintained and are set aside.
9. Appeal is allowed. There shall be no order as to costs.
Appeal allowed.
AIR 2008 SUPREME COURT 1312 "Baboolal Sharda v. Savitribai"
(From : Madhya Pradesh)*
Coram : 2 Dr. A. PASAYAT AND P. SATHASIVAM, JJ.
Civil Appeal No. 1669 of 2002, D/- 5 -2 -2008.
Baboolal Sharda and Anr. v. Smt. Savitribai and Ors.
M.P. Samaj Ke Kamjor Vargon Ke Krishi Bhumi Hadapne Sambandhi Knchakron Se
Paritran Tatha Mokti Adhiniyam (3 of 1977), S.5, S.6, S.15 - LAND - RESTORATION -
APPLICABILITY OF AN ACT - AUCTION SALE - Restoration of Land - Applicability
of Adhiniyam - Purchase of land made by appellant at Court auction - No material placed
before authorities to show that lands in question had any relation to any transaction of
loan - On the contrary Court auctions were for non-payment of revenue in respect of
lands - No material before SDO or Collector or High Court to show that appellants had
any role to play in Court auction or that they were responsible for nonpayment of revenue
for which Court auctions were held - Adhiniyam had thus, no application - Claim by
respondent for restoration of lands to her on ground that transfer to appellants was in
violation of stipulations under Adhiniyam - Deserves to be dismissed.
W.P. No. 851 of 1991, D/- 14-11-2000 (MP), Reversed. (Paras 4, 5, 6)
@page-SC1313

A.K. Chitale, Sr. Advocate, Niraj Sharma, for Appellants; C.D. Singh, Merusagar
Samantaray, Vairagya Vardhan. Sunny Chowdhary, Ms. Prema Kumari, for Respondents.
* W.P. No. 851 of 1991, D/- 14-11-2000 (MP).
Judgement
Dr. ARIJIT PASATAT, J. :- Challenge in this appeal is to the order passed by a learned
Single Judge of the Madhya Pradesh High Court, Indore Bench dismissing the writ
petition filed by the appellants under Article 227 of the Constitution of India, 1950 (in
short the 'Constitution'). The appellants had challenged the appellate order passed by the
Collector in exercise of appellate powers conferred by the Madhya Pradesh Samaj Ke
Kamjor Vargon Ke Krishi Bhumi Hadapane Sambandhi Kuchakron Se Paritran Tatha
Mukti Adhiniyam, 1976 (in short the '1976 Adhiniyam'). The original order was passed
by the SDO on 20-11-1990. The complaint was filed by respondent No. 1 stating that
under Section 5 of the Adhiniyam, the land purchased by the appellants needs to be
restored to her as transfer to appellants was in violation of the stipulations contained in
the Adhiniyam. It was alleged that Ram Prasad Sharda, father of the appellant No. 1 had
grabbed the land and after his death the land was in possession of his successor the
appellant No. 1. The appellants took the stand that the purchases in question were in court
auctions and therefore the Adhiniyam has no application. The SDO did not find any
substance in it. According to him, Section 15 of the Adhiniyam clearly applied to the
facts of the case. It was also held that Section 6 is also relevant. The SDO did not accept
the stand that the purchase being under court auctions, the Adhiniyam had no application
holding that under the definition of "lender of money" and "prohibited transaction of
loan" the appellants were required to restore the possession of the land to the applicant-
respondent No. 1. Appeal filed before the Collector as noted above did not bring any
relief.
2. Before the High Court the stands taken before the SDO and the Collector were
reiterated. But the High Court had abruptly concluded that the appellate order clearly
established that the Act was applicable. It did not examine the various points raised.
3. It may be noted at this juncture that there was a period of limitation fixed for filing the
claim after enactment of the Adhiniyam. Appellants' specific stand was that the
application was filed much beyond the prescribed time. The High Court merely noted that
the time for filing the claims was extended. It did not record any positive finding that the
application was filed within the extended time.
Section 2 of the Adhiniyam so far as relevant reads as follows:
"2. Definitions - In this Act, unless the context otherwise requires;
xx xx xx
(c) "holder of agricultural land" in the weaker sections of the people means a holder of
land used for purposes of Agriculture not exceeding eight hectares of unirrigated land
within the State whether as a Bhumiswami or an occupancy tenant or a Government
lessee either in any one or all of the capacities together within the meaning of the Code.
Explanation. - One hectare of irrigated land shall be equal to two hectares of unirrigated
land and vice versa.
(d) "lender of money" means a person advancing loan to a holder of agricultural land,
whether registered under the Madhya Pradesh Money Lenders Act, 1934 (No. 13 of
1934) or not;
(f) "prohibited transaction of loan" means a transaction in which a lender of money
advances loan to a holder of agricultural land against security of his interest in land,
whether at the time of advancing the loan or at any time thereafter during the currency of
the loan in any of the following modes, namely;
(i) agreement to sell land with or without delivery of possession;
(ii) outright sale of land with or without delivery of possession accompanied by separate
agreement to re-sell it.
(iii) outright sale of land with or without delivery of possession with a distinct oral
understanding that the sale shall not be acted upon if the loan is re-paid;
(iv) outright sale of land with or without delivery of possession with a condition
incorporated in the sale deed to re-sell it on re-payment of the Loan;
(v) transaction in any modes other than those specified in clauses (i) to (iv) affecting
interest in land including a fraudulent transaction designed to defeat the provisions of any
law regulating money lending or interest. for the time being in force, and includes all
those transactions in which a lender of money has, after the appointed day but on
@page-SC1314
or before the date of publication of this Act in the Gazette, obtained possession of land of
the holder of agricultural land through court or, by force or otherwise or obtained a
decree for such possession towards satisfaction of loan;"
Section 6 of the Adhiniyam reads as follows :
"(1) The Sub-Divisional Officer may, on his own motion in any transaction of loan and
shall on receipt of an application under Section 5 in the transaction of loan referred to
therein, make preliminary enquiry as he may in the circumstances of the case deem fit, to
ascertain whether the transaction of loan is a prohibited transaction of loan and notice in
Form II to furnish information in the form enclosed with the notice in respect of the land
within such time, not exceeding 1 days as may be specified in the notice.
as per sub-sections (2), (3) and (4) of this section which are reproduced below :
(2) The Sub-Divisional Officer shall by a notice served on the parties to the prohibited
transaction of loan call upon them to place all relevant facts and documents before him at
such place, on such date and at such time as may be specified in the notice.
(3) The Sub-Divisional Officer shall at the place and on the date and time specified in the
notice, afford an opportunity to the parties of being heard in person and may, if necessary,
examine all of the parties interested in land to elucidate information relevant to the
transaction of loan.
(4) During the enquiry the sub-divisional Officer shall, for the purpose of ascertaining the
true nature of transaction' of loan, try to collect, as far as, may be, information with
respect to the following facts, namely :
(i) the amount of principal money;
(ii) the market value of the land at the time of transaction;
(iii) adequacy of the amount of principal money as consideration for sale in the context of
the market value under clause (ii);
(iv) whether the consideration shown in the document was paid whole or in part privately
or before the Sub-Registrar;
(v) whether possession of the land was actually delivered to the lender of money as per
recitals in the said document. If not, when and in what manner the lender of money
obtained possession of the land;
(vi) What were the terms of the actual agreement between the lender of money and the
holder of agricultural and including the rate of interest;
(vii) the extent of urgency for the loan and availability of other sources to the holder of
agricultural land to obtain the same;
(viii) payment, if any, made by the holder of agricultural land to the lender of money
towards the loan;
(ix) whether the lender of money is registered money lender or not;
(x) any other surrounding circumstances which the Sub-Divisional Officer may deem fit
to consider."
Section 15 of the Adhiniyam reads as follows :
"15. Transfer of land which is subject matter of prohibited transaction of loan to be null
and void - Notwithstanding anything contained in any law for the time being in force
where a lender of money transfers any land, which may be a subject matter of a
prohibited transaction of loan, by way of sale, gift, exchange, lease or otherwise, such
transfer shall be deemed to have been made to defeat the provisions of this Act and be
null and void."
Section 5 deals with the application for protection and seeking relief under the Act and
the same reads as follows :
"5. Application for protection and seeking relief under this Act - A holder of agricultural
land who is a party to any transaction of loan subsisting on the appointed day or entered
into thereafter may apply to the Sub-Divisional Officer within such time; and in such
form and manner as may be prescribed for protection and relief under this Act."
4. A bare reading of the various provisions makes the position clear that a holder of
agricultural land who is a party to any transaction of loan subsisting on the appointing
date or entered into thereafter can apply to the SDO in the prescribed form and manner
for protection and relief under the Statute. Undisputedly the purchases made by the
appellants were at Court auctions. There was no material whatsoever placed before the
authorities to show that the lands in question had any relation to any transaction of loan.
On the contrary the Court auctions were for non-payment of revenue in respect of the
lands.
5. Learned counsel for the respondent-
@page-SC1315
State vehemently submitted that the Adhiniyam specifically referred to "Kuchakron" that
means manipulation and ill-design. There was no material before the SDO or the
Collector or the High Court to show that the appellants had any role to play in the Court
auction or that they were responsible for non-payment of revenue for which Court
auctions were held.
6. Above being the position, the Adhiniyam had clearly no application to the facts of the
case and therefore the impugned orders passed by the SDO, the Collector and the High
Court cannot be maintained and are set aside. The claim made by respondent No. l
deserves to be dismissed.
7. The appeal is allowed but without any order as to costs.
Appeal allowed.
AIR 2008 SUPREME COURT 1315 "Chief Engineer, Hydel Project v. Ravinder Nath"
(From : Punjab and Haryana)*
Coram : 2 S. B. SINHA AND V. S. SIRPURKAR, JJ.
Civil Appeal No. 658 of 2008 (arising out of SLP (C) No. 18774 of 2005) D/- 24 -1
-2008.
Chief Engineer, Hydel Project and Ors. v. Ravinder Nath and Ors.
(A) Civil P.C. (5 of 1908), S.9 - CIVIL COURT - TERMINATION OF SERVICE -
INDUSTRIAL DISPUTE - SERVICE MATTERS - Bar to jurisdiction of civil Court -
Plaintiff employed on work charged basis till completion of project - Termination of
services - Suit for declaration that orders of termination were null and void etc. - Claim
by employer that termination simpliciter was effected in light of Rules under Certified
Standing Orders - Plaintiffs on other hand alleging that principles under provisions of
Certified Standing Orders were completely ignored and highly arbitrary approach was
adopted by employer by picking and choosing plaintiffs for purpose of termination -
Dispute fell under premise of Industrial Disputes Act - Civil Court's jurisdiction barred -
Suit liable to be dismissed.
Industrial Disputes Act (14 of 1947), S.11A.
Regular Second Appeal No. 2286 of 1991, D/- 27-4-2005 (P and H), Reversed. (Paras
14, 15)
(B) Constitution of India, Art.133 - APPEAL - DECREE - DOCTRINES - Objection to
jurisdiction - Decree itself held to be without jurisdiction and hit by doctrine of 'coram
non judice' - Decree cannot be upheld on ground that objection to jurisdiction was not
taken at initial first appellate or second appellate stage. (Para 19)
Cases Referred : Chronological Paras
2007 AIR SCW 1011 : AIR 2007 SC 1077 : 2007 (2) AIR Bom R 611 (Foll) (Pt B)
18
2006 AIR SCW 5432 : 2006 Lab IC 4281 : 2007 (1) AIR Jhar R 598 (Ref) (Pt. B) 18
2006 AIR SCW 6167 : AIR 2007 SC 340 (Ref) (Pt B) 18
2005 AIR SCW 5369 : AIR 2005 SC 4446 (Foll) (Pt B) 17
(2005) 7 SCC 447 (Foll) 8, 12
2004 AIR SCW 5024 : AIR 2004 SC 4355 : 2004 Lab IC 3685 : 2004 AIR Jhar HCR
2748 (Foll) (Pt A) 12, 14
2002 AIR SCW 710 : AIR 2002 SC 997 : 2002 Lab IC 794 (Foll) (Pt A) 12
1995 AIR SCW 2683 : AIR 1995 SC 1715 : 1995 Lab IC 2241 (Foll) (Pt A) 8, 11,
12
(1991) 1 RSJ 770 16
AIR 1990 SC 255 : 1990 Lab IC 308 (Foll) (Pt A) 8, 10, 11
1989 Lab IC 807 (P and H) (Ref) (Pt A) 5, 6
AIR 1987 SC 1801 : 1988 Lab IC 110 (Ref) 6
AIR 1979 SC 193 : 1978 Lab IC 1672 (Ref) (Pt B) 18
AIR 1975 SC 2238 : 1975 Lab IC 1651 (Foll) (Pt A)8, 9, 11
AIR 1969 SC 78 (Foll) (Pt A) 11
AIR 1966 SC 634 (Foll) (Pt. B) 17
AIR 1954 SC 340 (Foll) (Pt. B) 17
Harinder Mohan Singh, Kaushal Yadav, for Appellants; Prem Malhotra, for Respondents.
* R. S. A. No. 2286 of 1991, D/- 27-4-2005 (P and H).
Judgement
1. V. S. SIRPURKAR, J.Leave granted.
2. Chief Engineer, Superintending Engineer (Construction Circle) and Personnel Officer,
Anandpur Sahib Hydel Project have filed this appeal to question the correctness of the
judgment of the Punjab and Haryana High Court in Regular Second Appeal confirming
the judgment passed by the Additional District Judge, Ropar and Senior Sub Judge,
Ropar, basically on the ground that there was a complete lack of jurisdiction in the above
three Civil Courts since the issues squarely fall within the ambit of the Industrial Disputes
Act, 1947 and as such
@page-SC1316
the remedy for the 9 respondents-workmen, who are workmen under the Industrial
Disputes Act, lies with the authorities thereunder and not with the Civil Court.
BASIC FACTS
3. Nine respondents herein filed a Civil Suit before the Senior Sub Judge, Ropar for the
relief of (i) declaration to the effect that the orders of their termination/retrenchment from
service were illegal and (ii) that they were entitled to reinstatement in service with back-
wages. It was pleaded that the plaintiffs-respondents were skilled workers and were
working on the Anandpur Sahib Hydel Project (hereinafter called "the Project") in
various capacities such as T. Mate, Mixer Operator, Beldar, etc. for more than 5 years
and, therefore, as per the Standing Orders and Rules they were regular employees of the
defendants. It was alleged that the defendants did not maintain any seniority-list of the
workers and various categories of services on the said Project and they arbitrarily
removed the plaintiffs-respondents from service on the dates mentioned in Annexure A to
the plaint by obtaining their signatures on papers under coercion and force and also
forced them to accept payments. It was further alleged that while removing the plaintiffs-
respondent, the defendants-appellants did not observe the seniority, meaning thereby
while the juniors were retained in service, the seniors were retrenched. It was alleged that
action was based on pick and choose policy and was discriminatory and amounted to
victimization. It was also alleged that those workers who had completed service for 1000
days, could not have been retrenched (as was held by the Punjab and Haryana High Court
in Mehanga Ram v. Punjab State, Civil Writ No. 718 of 1986).
4. This claim was contested by the State of Punjab. It was firstly urged that the suit was
bad as common suit could not have been filed since the cause of action of each defendant
was distinct and separate. It was urged that the plaintiffs-respondents were appointed on
purely temporary basis as work-charged employees and after the completion of the
project, their services were validly terminated as per Rule 20(1) read with Rule 3(a) of
the Certified Standing Orders for the work-charged staff on the said Project. Since the
termination was complete on payment of necessary gratuity etc., there can be no cause of
action and as such the present suit was not maintainable in the present form. It was also
urged that notice under Section 80 CPC was not given and the suit was also barred by
limitation. It was reiterated that the principle of first come last go was strictly observed
since there was a regular seniority-list maintained for the Project as a whole and that
there was no discrimination or victimization. On merits also the suit was opposed on the
ground that since the plaintiffs-respondent were work-charged employees for a work of
temporary nature, on completion of the project their services were terminated as per the
Rules which governed their service conditions (the Certified Standing Orders). The
following issues were framed in between the parties by the Court :
"1. Whether the impugned orders of termination, retrenchment of the plaintiffs are illegal,
unauthorized, ultra vires and ineffective as alleged? OPP
2. Whether the suit is bad for misjoinder of parties? OPP
3. Whether the suit is maintainable in the present form? OPP
4. Whether no valid notice u/S. 80 CPC has been served by the plaintiff on the
defendants? OPP
5. Whether the suit is within limitation?
6. Whether the plaintiffs are entitled to the declaration and injunction prayed for? OPP
7. Relief."
5
. The Trial Court, on the basis of the evidence, came to the conclusion that the
defendants-appellants had not observed the principle of last come first go in making the
retrenchments. The Trial Court also relied on the judgment of the Punjab and Haryana
High Court in Piara Singh and Ors. v. State of Haryana [1989 PLR 396] and one another
judgment, the copy of which was filed Vide Exhibit D-13 wherein the High Court had
given directions that the workers so retrenched should be accommodated somewhere-else
in some other projects and such appointments in the new projects would be treated as
new appointments for the purpose of seniority and that the relief given to such workers
would be without prejudice to the retrenchment and any other compensation that such
workers would be entitled to under the provisions of the Industrial Disputes Act, 1947.
Relying 1989 Lab IC 807

@page-SC1317
on these observations, the impugned orders of termination were held illegal. As regards
issue regarding tenability of the suit, all that was said by the Trial Court was that the
Government Pleader could not point out any defect in the form of the suit except that the
plaintiffs had not challenged any specified orders regarding the termination of their
services. (That is the only discussion in respect of the tenability). Though it was held that
there was no evidence to hold that the persons junior to the plaintiffs-respondents were
retained in service, there was no evidence on record to show that as to what would be the
position of the plaintiffs in overall seniority-list when finalized vis-a-vis the other
employees who have been retained or retrenched and, therefore, it was held that the
plaintiffs were entitled to declaration and mandatory injunction only to the extent that
they had right to be taken back in service and in case it was found that they were entitled
to be retained on the project on the basis of the seniority, they would be entitled to be
absorbed on other projects of the defendants according to their qualifications and fitness
within a period of six months from the date of the judgment. A curious relief was granted
in the following terms :
"17. In view of my foregoing findings, the suit of the plaintiffs partly succeeds.
Accordingly, I pass a decree in favour of the plaintiffs and against defendants No. 1, 3 to
5 for declarations to the effect that the plaintiffs are entitled to be taken back in service.
However, in case it is found that by virtue of their overall seniority in their respective
categories of workers at the time of their retrenchment, they were not entitled to be
retained on the APS Project, then they shall be absorbed in other projects under the
defendants within a period of six months from the date of this judgment.."
6

. This order of the Trial Court was appealed against by the defendants-appellants before
the Additional District Judge, Ropar which appeal was dismissed. In its judgment the
Appellate Court has referred to the arguments advanced by the appellants relying on Rule
20(1) of the Standing Orders governing the work-charged staff of the Project as also to
the contention raised on behalf of the plaintiffs-respondent that the defendants-appellants
had not violated principle of last come first go. The Appellate Court accepted that such
principle was not strictly adhered to and further held that the Anandpur Hydel Project was
a "State" and the plaintiffs-respondents were entitled to the protection contained under
the Constitution of India and CSR which provided that the work-charged employees
could not be allowed to remain as such for more than six months. Relying on the decision
of this Court reported in Supreme Court of India v. Cynamide India Ltd. (AIR 1987 SC
1801) and Piara Singh v. State of Haryana (1989 PLR (1) 396), the Appellate Court
confirmed the findings of the Trial Court and dismissed the appeal. 1989 Lab IC 807

7. The matter was taken before the High Court by way of a Second Appeal on various
grounds. To begin with the High Court granted stay of the operation of the orders passed
by the courts below. However, by the subsequent order, the stay application was
dismissed and the said order granted on 20-12-1991 was vacated. This came to be
challenged by way of a Special Leave Petition before this Court. In the Special Leave
Petition a contention was raised by way of Ground (8) that the Civil Court had no
jurisdiction to entertain the suit since the relief of reinstatement in the present case was
available only under the Industrial Disputes Act. This Court, however, did not interfere at
that stage and directed the High Court to dispose of the Second Appeal as expeditiously
as possible. The High Court dismissed the Second Appeal necessitating the present appeal
before us.
CONTENTIONS
8

. Learned counsel appearing on behalf of the appellants urged that since the issues
squarely fell within the ambit of the Industrial Disputes Act, 1947 and since there is a
specific remedy available to the plaintiffs-respondents under that Act, the jurisdiction of
the Civil Court was impliedly excluded and all the courts below erred in entertaining and
deciding upon the issues much less adverse to the appellants. Learned counsel, relying on
this Court's judgments in The Premier Automobiles Ltd. and Ors. v. Kamlekar Shantaram
Wadke of Bombay and Ors. [(1976) 1 SCC 496], Jitendra Nath Biswas v. M/s. Empire of
India and Ceylon Tea Co. and Anr. [(1989) 3 SCC 582]; Raj as than State Road Transport
Corporation and Anr. v. Krishna Kant and Ors. [(1995) 5 SCC 74]; and Rajasthan State
AIR 1975 SC 2238
AIR 1990 SC 255
1995 AIR SCW 2683

@page-SC1318
Road Transport Corporation and Ors. v. Zakir Hussain [(2005) 7 SCC 447] urged that the
legal position in this behalf was settled. On the other hand the learned counsel on behalf
of the respondents urged that firstly this issue relating to jurisdiction was not raised by the
respondents before any courts below and it is only for the first time that the objection to
the jurisdiction has been raised before this Court. Learned counsel for the respondents
also urged that the issue was not covered under the labour jurisprudence and under the
provisions of the Industrial Disputes Act, 1947 and the jurisdiction of the Civil Court
could not be said to be barred.
DISCUSSIONS
9. We would take into account the objection to the effect that the contention regarding the
jurisdiction was not raised and, therefore, it could not be allowed to be raised at this late
stage, for that it will have to be found as to whether the issue regarding the reinstatement
and the payment of back-wages could be said to be covered under the provisions of
Industrial Disputes Act. The question of the Civil Court's jurisdiction being excluded
came, for the first time, before this Court in The Premier Automobiles's case (supra). In
that case the Court culled out following four principles :
(1) If the dispute is not an industrial dispute, nor does it relate to enforcement of any
other right under the Act the remedy lies only in the civil court.
(2) If the dispute is an industrial dispute arising out of a right or liability under the
general or common law and not under the Act, the jurisdiction of the civil court is
alternative, leaving it to the election of the senior concerned to choose his remedy for the
relief which is competent to be granted in a particular remedy.
(3) If the industrial dispute relates to the enforcement of a right or an obligation created
under the Act, then the only remedy available to the suitor is to get an adjudication under
the Act.
(4) If the right which is sought to be enforced is a right created under the Act such as
Chapter VA then the remedy for its enforcement is either Section 33C or the raising of an
industrial dispute, as the case may be."
10. The second decision came in Jitendra Nath Biswas's case (supra), wherein this Court
specifically held, interpreting Section 9 of the CPC that the Civil Court shall have no
jurisdiction where its jurisdiction is expressly or impliedly barred. The Court held: "It
could not be disputed that a contract of employment for personal service could not be
specifically enforced and it is also clear that except the industrial law, under the law of
contract and the civil law, an employee whose services are terminated could not seek the
relief of reinstatement with back wages. At best he could seek the relief of damages for
breach of contract. The manner in which the relief has been framed by the appellant
plaintiff in this case, although he seeks a declaration and injunction but in substance it is
nothing but the relief of reinstatement and back wages. The relief could only be available
to a workman under the Industrial Disputes Act."
The Court, therefore, proceeded to hold that the civil courts jurisdiction was barred. In
this case very peculiarly it was not disputed that the Industrial Employment (Standing
Orders) Act was also applicable to the workman and an inquiry for misconduct was
conducted against the appellant in accordance with the standing orders. It was argued
before the court, however, that since it was solely the discretion of the Conciliation
Officer to proceed with the conciliation proceedings and since even after the report given
by the Conciliation Officer it was the discretion of the State Government to make a
Reference or not, the civil courts jurisdiction was not barred. This Court repelled that
contention after discussing the duties of the Conciliation Officer and held that the civil
courts jurisdiction was barred.
11

. On its heels came the case of Krishana Kant (supra). This was a case where, pursuant to
the disciplinary inquiry held against some of the workers on charges of misconduct, their
services were terminated. The suits were filed for a declaration that the orders terminating
their services were illegal and invalid and for further declaration that they must be
deemed to have continued and still continuing in the service of the Corporation with all
consequential benefits. This Court elaborately considered the law laid down earlier in the
cases of Premier Automobiles's case and Jitendra Nath Biswas's (supra) and after
considering the concept of "industrial dispute" as covered under Sections 2(k) and 2-A of
Industrial Disputes Act, 1947 came 1995 AIR SCW 2683

@page-SC1319
to the conclusion that the disputes not covered under Section 2(k) or 2-A could be
determined by Civil Court or by arbitration but disputes relating to right or obligation
created by the Industrial Disputes Act can be adjudicated only by the forum created by
the Industrial Disputes Act. This was a case where the Corporation was armed with the
Certified Standing Orders. The Court held that the Certified Standing Orders are not in
the nature of delegated or subordinate legislation. It was held that the Certified Standing
Orders were statutorily imposed conditions of service and the complaint made by the
workman relating to breach thereof could only be tried under the machinery and the
procedure provided by the Industrial Disputes Act and the civil courts jurisdiction was
impliedly barred to that extent. The Court while referring to the seven principles culled
out by this Court in Dhulabhai v. State of M.P. [(1968) 3 SCR 662 : AIR 1969 SC 78]
further explained the decision in Premier Automobile (supra) to hold that not only the
disputes under the Industrial Disputes Act were barred but the disputes arising out of the
sister enactments like Industrial Employment (Standing Orders) Act also stood outside
the jurisdiction of the civil court since they did not provide a special forum of their own
for enforcement of the rights and liabilities created by them. The Court, therefore, held :
"Thus a dispute involving the enforcement of the rights and liabilities created by the
certified standing orders has necessarily got to be adjudicated only in the forums created
by the Industrial Disputes Act within the meaning of Sections 2(k) and 2-A of Industrial
Disputes Act or such enactment says that such dispute shall be either treated as an
industrial dispute or shall be adjudicated by any of the forums created by the Industrial
Disputes Act. The civil court have no jurisdiction to entertain such suits." The Court
further went on to say that the enforcement of the Industrial Employment Standing
Orders is an industrial dispute and if it satisfies the requirement of Section 2(k) and/or
Section 2-A of the Industries Disputes Act, it must be adjudicated in the forums created
by the Industrial Disputes Act alone.
12

. Though there are number of other cases followed, we would choose to consider the
decision in Zakir Hussain's case (supra). This case also arose out of the termination
simpliciter effected by the Corporation of the conductor who was appointed on probation
basis for a period of two years and since his services were not found satisfactory, the
same were terminated, of course with necessary compensation prescribed as per the Rules
of the Corporation. The court after considering all the earlier cases cited above and
referring to the seven principles culled out in the case of Krishan Kant, came to the
conclusion as arrived at in Krishan Kant's case. Two other cases were referred to, they
being B.S. Bharti v. IBP Co. Ltd. [(2004) and SCC 550] and Chandrakant Tukaram
Nikam v. Municipal Corporation of Ahmedabad [(2002) 2 SCC 542]. It was held by the
court that the reliefs craved in the said cases squarely fell within the arena of Industrial
Disputes Act and, therefore, civil courts jurisdiction was clearly barred. On the question
of the ad hoc appointment of the employee, the court came to the conclusion that the
respondent was a probationer and did not have any substantive right to hold the post and
was not entitled to a decree of declaration which was erroneously granted by the lower
courts. 2004 AIR SCW 5024
2002 AIR SCW 710

13. Now coming to the facts of the present case, there is no dispute that there are
Certified Standing Orders in vogue. The nine plaintiffs-respondents were engaged on
work-charged basis till the completion of the Project. Their services came to be
terminated after the completion of the Project in January, 1985 as they were not required
due to the completion of the Project and since they were engaged temporarily. It was
pointed out before us and not disputed that the services were terminated vide order dated
6-7-1985 under Rule 20(1) read with Rule 3-A of the Certified Standing Orders for work-
charged staff. It is also not disputed that they were paid gratuity, retrenchment
compensation as also the compensation for notice and that they had duly accepted the
order. In the civil suit it was prayed that a decree be passed for declaration to the effect
that the orders of termination/retrenchment of their service were null and void and that
they should be reinstated with back-wages. A mandatory injunction to that effect was
sought for. It was urged before the trial court that the defendants-appellants have not
maintained a proper seniority list and that had resulted in the breach of the principle of
last come first go and, therefore, their termination was
@page-SC1320
bad in law. In short, the original plaintiffs-respondents had averred the breach of Section
25-G of the Industrial Disputes Act, in that, they had alleged that the employer had shown
discriminatory attitude and the plaintiffs-respondents were picked and chosen for being
terminated and thus were victimized. On the other hand defence raised was that there
were certain cut-off dates fixed for the retention of the employees and all the plaintiffs-
respondent had actually joined the service after that cut-off date and, therefore, they were
terminated in terms of Rule 20(1) read with Rule 3-A of the Certified Standing Orders
relating to work-charged staff.
14. From the above discussion there is no doubt that the dispute and the main issue fell
squarely under the premise of Industrial Disputes Act. Further as specifically held in
Krishna Kant's case that where the Certified Standing Orders were applicable and where
the breach thereof was complained of, such issues fell in the exclusive area of the
machinery provided by the Industrial Disputes Act and as such the civil court's
jurisdiction was specifically barred. We are left with no doubt that the situation is
identical in the present case.
15. In the present case while the employers-appellants claimed that the termination
simpliciter was effected in the light of the Rules under the Certified Standing Orders, the
plaintiffs-respondents alleged that the principles under the provisions of the Certified
Standing Orders were completely ignored and a highly arbitrary, discriminatory approach
was adopted by the employer by picking and choosing the plaintiffs for the purposes of
termination. The dispute, therefore, clearly fell outside the civil court's jurisdiction as per
the decisions of this Court relied upon earlier.
16. However, the question is that this issue of jurisdiction was not raised either before the
First Appellate Court or the Second Appellate Court. Learned counsel for the respondents
very vociferously argued before us that for the reasons best known to the appellants, this
objection regarding the jurisdiction was never raised specifically. We have seen the
written statement. In the written statement the defendants-appellants have raised a plea
though not specifically but there is a clear reference to Rule 20(1) read with Rule 3-A of
the Certified Standing Orders for the work-charged staff on the Project. It is stated, which
is apparent from the judgment of the trial court that since the services of plaintiffs have
already been terminated on payment of necessary gratuity etc., they have no cause of
action and that the present suit is not maintainable in the present form and is also not
competent without notice under Section 80 CPC besides being barred by limitation". The
tenability of the suit was, therefore, raised and vide Issue No. 3, the trial court also
considered the tenability of the suit in the present form. The trial court has not, however,
adverted to the jurisdiction aspect as is being presently highlighted before us. Same is the
story about the First Appellate Court and the Second Appellate Court. However, it is not
as if this issue was not raised altogether. At least a notice of this issue was given to the
respondents in SLP (C) 11086 of 1992 which was filed on behalf of the appellants to
challenge the dismissal of the said application by the High Court. It is very specifically
raised therein in Ground No. 8, which is reproduced as under :
"Because the civil court had no jurisdiction to entertain the suit. The relief of
reinstatement in the present case was available only under the Industrial Disputes Act and
therefore the jurisdiction of the Civil Court was expressly barred. [(1991) 1 RSJ 770].
The declaration claimed by the plaintiffs in his suit could be granted by the Labour Court
under the Industrial Disputes Act and consequential relief was also exclusively outside
the jurisdiction of the civil court. The plaintiff respondents are basing their case mainly
on the provision of Industrial Disputes Act, 1947 and thus the claim of the
plaintiffs/respondents could only be adjudicated by the Labour Court. The proposition of
law is now well settled by repeated pronouncement made by the Apex Court." Therefore,
it is not that the respondents herein had no notice of such an objection. This Court only
directed the High Court to dispose of the appeal before it expeditiously. However, it does
not seem that the question was raised by the counsel of the appellants before the High
Court in the present form. On the other hand the High Court has very specifically held
that there was no substantial question of law involved in the matter.
17

. In our considered opinion, it cannot be said that there was no question of law involved
as we have pointed out that the issues squarely fell in the area covered by the 2005
AIR SCW 5369

@page-SC1321
Industrial Disputes Act and was, therefore, specifically barred. The question is whether
this issue regarding the jurisdiction could be allowed to be raised before us. The question
of jurisdiction came up before this Court in Harshad Chiman Lal Modi v. DLF Universal
Ltd. and Anr. [(2005) 7 SCC 791]. The Court therein was considering the question raised
whether the court had jurisdiction under Section 16(d) CPC to deal with the matter in
question. In short the court was considering whether the amendment could have been
allowed raising objection to the territorial jurisdiction. This Court in para 30 observed as
under :
"We are unable to uphold the contention. The jurisdiction of a court may be classified
into several categories. The important categories are (i) territorial or local jurisdiction; (ii)
pecuniary jurisdiction; and (iii) jurisdiction over the subject-matter. So far as territorial
and pecuniary jurisdiction are concerned, objection to such jurisdiction has to be taken at
the earliest possible opportunity and in any case at or before settlement of issues. The law
is well settled on the point that if such objection is not taken at the earliest, it cannot be
allowed to be taken at a subsequent stage. Jurisdiction as to subject-matter, however, is
totally distinct and stands on a different footing. Where a court has no jurisdiction over
the subject-matter of the suit by reason of any limitation imposed by statute, charter or
commission, it cannot take up the cause or matter. An order passed by a court having no
jurisdiction is a nullity."
The Court then proceeded to rely on the case in Bahrein Petroleum Co. Ltd. v. P.J. Pappu
[(1966) 1 SCR 461 : AIR1966 SC 634] and observed in para 32 that neither consent nor
waiver nor acquiescence can confer jurisdiction upon a court, otherwise incompetent to
try the suit. The Court further observed that"
"It is well settled and needs no authority that where a court takes upon itself to exercise a
jurisdiction it does not possess, its decision amounts to nothing. A decree passed by a
court having no jurisdiction is non est and its invalidity can be set up whenever it is
sought to be enforced as a foundation for a right, even at the stage of execution or in
collateral proceedings. A decree passed by a court without jurisdiction is a coram non
judice."
The Court also relied upon the decision in Kiran Singh v. Chaman Paswan [(1955) 1 SCR
117 : AIR 1954 SC 340] and quoted therefrom :
"It is a fundamental principle well established that a decree passed by a court without
jurisdiction is a nullity, and that its invalidity could be set up whenever and wherever it is
sought to be enforced or relied upon, even at the stage of execution and even in collateral
proceedings. A defect of jurisdiction strikes at the very authority of the court to pass any
decree, and such a defect cannot be cured even by consent of parties."
Though in the aforementioned decision these observations were made since the
defendants before raising the objection to the territorial jurisdiction had admitted that the
court had the jurisdiction, the force of this decision cannot be ignored and it has to be
held that such a decree would continue to be a nullity.
18

. The aforementioned decision was followed again in Hasham Abbas Sayyad v. Usman
Abbas Sayyad and Ors. [(2007) 2 SCC 355] where one of us, Sinha, J. was a party. Of
course while following this decision the Court referred to the decisions in Chief Justice of
A.P. v. L.V.A. Dixitulu [(1979) 2 SCC 34]; Zila Sahakari Kendrya Bank Maryadit v.
Shahjadi Begum [(2006) 11 SCC 692] as also Shahabad Cooperative Sugar Mills Ltd. v.
Special Secretary to Govt. of Haryana [(2006) 12 SCC 404]. 2007 AIR SCW 1011
AIR 1979 SC 193
2006 AIR SCW 5432
2006 AIR SCW 6167

CONCLUSION
19. Once the original decree itself has been held to be without jurisdiction and hit by the
doctrine of coram non judice, there would be no question of upholding the same merely
on the ground that the objection to the jurisdiction was not taken at the initial, First
Appellate or the Second Appellate stage. It must, therefore, be held that the civil court in
this case had no jurisdiction to deal with the suit and resultantly the judgments of the
Trial Court, First Appellate Court and the Second Appellate Court are liable to be set
aside for that reason alone and the appeal is liable to be allowed. In view of this verdict of
ours, we have deliberately not chosen to go into the other contentions raised on merits.
We, however, make
@page-SC1322
it clear that we have not, in any manner, commented upon the rights of the plaintiffs-
respondents, if any, arising out of the Labour Jurisprudence.
20. In the result the appeal is allowed but without any order as to costs.
Appeal allowed.
AIR 2008 SUPREME COURT 1322 "Emp. State Insurance Corporation v. H. M. T. Ltd."
(From : Karnataka)
Coram : 2 S. B. SINHA AND J. M. PANCHAL, JJ.
Civil Appeal No. 340 of 2008 (arising out of SLP (C) No. 4827 of 2006), D/- 11 -1 -2008.
Emp. State Insurance Corporation v. H.M.T. Ltd. and Anr.
(A) Employees' State Insurance Act (34 of 1948), S.85B - Employees' State Insurance
(General) Regulations (1950), Regn.31C - EMPLOYEES STATE INSURANCE - E.S.I.
Contribution - Failure to make - Levy of damages is by way of penalty - S.85-B provides
for an enabling provision - It does not however envisage mandatory levy of damages in
all situations - Such an intention is not decipherable from provision.
1993 AIR SCW 3962 and 2007 AIR SCW 4323, Relied on.
1998 AIR SCW 352, Disting.
Writ App. No. 2587 of 2004 (L-ESI), D/-12-09-2005 (Kar). Reversed. (Paras 12, 13,
14, 17)
(B) Employees' State Insurance Act (34 of 1948), S.85B, Proviso - EMPLOYEES STATE
INSURANCE - NATURAL JUSTICE - PRINCIPLES - EQUALITY - E.S.I. contribution
- Failure to make - Levy of damages - Proviso incorporates principles of natural justice.
Constitution of India, Art.14. (Para 11)
(C) Employees' State Insurance Act (34 of 1948), S.85B - Employees' State Insurance
(General) Regulations (1950), Regn.31C - EMPLOYEES STATE INSURANCE - Failure
to make E.S.I. contribution - Levy of damages - Existence of mens rea or actus reus to
contravene a statutory provision - Is necessary ingredient for levy of damages and/or
quantum thereof. (Para 21)
Cases Referred : Chronological Paras
2007 AIR SCW 4323 : 2007 Tax LR 681 (Rel. on) 19
1998 AIR SCW 352 : AIR 1998 SC 688 : 1998 Lab IC 483 (Disting) 15
1993 AIR SCW 3962 : AIR 1994 SC 521 (Rel. on) 18
C.S. Rajan, Sr. Advocate, V.J. Francis and Anupam Mishra, for Appellant; C.V. Francis,
G. Prakash and Sanjay R. Hegde, for Respondents.
Judgement
S. B. SINHA, J. :- Leave granted.
Interpretation and application of Section 85-B of the Employees State Insurance Act (The
Act) and Regulation 32C of the Employees State Insurance General Regulations, 1950
(The Regulations) is in question in this appeal which arises out of a judgment and order
dated 12-9-2005 passed by a Division Bench of the Karnataka High Court in Writ Appeal
No. 2587 of 2004 allowing the appeal in part preferred from the judgment and order
dated 25-3-2000 passed by a learned Single Judge of the said Court in Writ Petition No.
38753 of 1998.
2. Respondent is an 'employer' within the meaning of the provisions of Section 2(17) of
the Act. Indisputably, prior to issuance of the notification dated 27-3-1992, the wage
ceiling of the employees was restricted to Rs. l,600/-per month. The same was increased
to Rs. 3000/ per month with a view to bring them within the purview of the Act.
3. Validity of the said notification was challenged in a large number of writ petitions by
the employees. By an interim order passed by the High Court, the operation of the
notification was directed to be stayed. The said writ petitions were dismissed by an order
dated 5-8-1992.
4. Writ appeals were filed by the 'employees' through their respective Trade Unions.
While admitting the said appeals, the interim order operating during the pendency of the
writ petition was allowed to continue.
The said writ appeals were also dismissed by the Division Bench of the High Court by
reason of a judgment and order dated 11-7-1995, inter alia, on the premises that there was
no impediment for the respondent herein to deposit the contributions of the employees
concerned.
5. On and from the said date interest was claimed till the date of actual payment.
Appellant thereafter also raised a claim for payment of interest for delayed payment and
furthermore levied damages in terms of Section 85B of the Act.
6. A writ petition filed by the respondent
@page-SC1323
herein questioning the validity of the said notice dated 9-6-1998 was dismissed by an
order dated 25-3-2000. A review petition was filed by the appellant herein which was also
dismissed on merits.
An intra Court appeal was preferred there against and by reason of the impugned
judgment, it was allowed in part opining :
1. that although period of delay is slightly more than two years, some reasonable time
should be allowed for deposit of contributions and, thus, restricting the period of payment
of interest to two years only.
2. No damage should be directed to be levied in the facts and circumstances of the case as
Section 85B of the Act provides for an enabling provision and does not make it
mandatory to levy damages in every case.
7. Mr. C. S. Rajan, learned senior Counsel appearing for the appellant, submitted that the
High Court committed a serious error in passing the impugned judgment in so far as it
failed to take into consideration the purported effect of Regulation 31C of the Regulations
which provides for levy of interest as also damages.
8. Mr. C. V. Francis, learned counsel appearing on behalf of the respondent, on the other
hand, would support the impugned judgment.
9. The said Act was enacted to provide for certain benefits to the employees of an
establishment in case of sickness, maternity and employment injury and to make
provisions for certain other matters in relation thereto.
10. Chapter IV of the Act provides for payment of contributions. Section 39 of the Act
postulates payment of contributions thereunder both by the 'employer' as also the
'employee'.
11. Section 85B of the Act empowers the Corporation to recover damages in the event an
employer fails to make the payment of the amount due in respect of contribution; subject,
however, to the condition that the amount thereof would not exceed the amount of arrears
as may be specified in the Regulations. Proviso appended thereto incorporates the
principles of 'Natural Justice'.
12. Obligation on the part of the employer to deposit the contributions of both the
'employer' and the 'employee' is not in dispute.
What is in dispute is as to whether the amount of damages specified in Regulation 31C of
the Regulation is imperative in character or not.
It is a well known principle of law that a subordinate legislation must conform to the
provisions of the Legislative Act. Section 85B of the Act provides for an enabling
provision. It does not envisage mandatory levy of damages. It does not contemplate
computation of quantum of damages in the manner prescribed under the regulations.
13. The statutory liability of the employer is not in dispute. An employee being required
to be compulsorily insured, the employer is bound to make his part of the contribution.
An employee is also bound to make his contribution under the Act. But the same does not
mean that levy of damages in all situations would be imperative.
14. Section 85B of the Act uses the words 'may recover'. Levy of damages thereunder is
by way of penalty. The Legislature limited the jurisdiction of the authority to levy
penalty, i.e., not exceeding the amount of arrears. Regulation 31C of the Regulations,
therefore, in our opinion, must be construed keeping in view the language used in the
Legislative Act and not de hors the same.
15

. Our attention, however, has been drawn to a decision of this Court in Hindustan Times
Ltd. v. Union of India (1998) 2 SCC 242 wherein it has been laid down : 1998 AIR
SCW 352
(Para 28)

"From the aforesaid decisions, the following principles can be summarized :


The authority under Section 14-B has to apply his mind to the facts of the case and the
reply to the show cause notice and pass a reasoned order after following principles of
natural justice and giving a reasonable opportunity of being heard; the Regional
Provident Fund Commissioner usually takes into consideration the number of defaults,
the period of delay, the frequency of default and the amounts involved; default on the part
of the employer based on plea of power-cut, financial problems relating to other
indebtedness or the delay in realization of amounts paid by the cheques or drafts, cannot
be justifiable grounds for the employer to escape liability; there is no period of limitation
prescribed by the legislature for initiating action for recovery of damages under Section
14B."
16. It was, however, opined that in certain
@page-SC1324
situations, the employer can claim the benefit of "irretrievable prejudice' in case a
demand for damages is made after several years. In that case, this Court was concerned,
inter alia, with a question in regard to the effect of levy of damages after a long time. The
question which, inter alia, arose for consideration therein was as to whether suo motu
revisional jurisdiction could be exercised by the revisional authority at any time it desires.
The Court made a distinction between the cases involving 'recovery of money' from an
employer who had withheld the contribution made by the workmen in trust and other
cases. It was in that situation opined supra. We are not concerned with such a situation
herein.
17. A penal provision should be construed strictly. Only because a provision has been
made for levy of penalty, the same by itself would not lead to the conclusion that penalty
must be levied in all situations. Such an intention on the part of the legislature is not
decipherable from Section 85B of the Act. When a discretionary jurisdiction has been
conferred on a statutory authority to levy penal damages by reason of an enabling
provision, the same cannot be construed as imperative. Even otherwise, an endeavour
should be made to construe such penal provisions as discretionary, under the statute is
held to be mandatory in character.
18

. In Prestolite (India) Ltd. v. Regional Director and Anr. (1994 Supp. (3) SCC 690), this
Court rejected a contention raised by the Regional Director of Employees Insurance that
under the Employee's State Insurance General Regulations guidelines have been indicated
showing as to how damages for delayed payment are to be imposed and since such
guidelines have been followed, no exception should be taken thereto made to the
impugned adjudication, stating : 1993 AIR SCW 3962
(Para 4)

"Even if the regulations have prescribed general guidelines and the upper limits at which
the imposition of damages can be made, it cannot be contended that in no case, the
mitigating circumstances can be taken into consideration by the adjudicating authority in
finally deciding the matter and it is bound to act mechanically in applying the uppermost
limit of the table. In the instant case, it appears to us that the order has been passed
without indicating any reason whatsoever as to why grounds for delayed payment were
not to be accepted. There is no indication as to why the imposition of damages at the rate
specified in the order was required to be made. Simply because the appellant did not
appear in person and produce materials to support the objections, the employee's case
could not be discarded in limine. On the contrary, the objection ought to have been
considered on merits."
19

. In Dilip N. Shroff v. Joint Commissioner of Income-tax, Mumbai and Anr. ((2007 6


SCC 329), this Court stated : 2007 AIR SCW 4323

"40. Thus, it appears that there is distinct line of authorities which clearly lays down that
in considering a question of penalty, means rea is not a relevant consideration. Even
assuming that when the statute says that one is liable for penalty if one furnishes
inaccurate particulars, it may or may not by itself be held to be enough if the particulars
furnished are found to be inaccurate is anything more needed but the question would still
be as to whether reliance placed on some valuation of an approved valuer and, therefore,
the furnishing of inaccurate particulars was of deliberate, meaning thereby that an
element of mens rea is needed before penalty can be imposed, should have received
serious consideration in the light of a large number of decisions of this Court."
20. We agree with the said view as also for the additional reason that the subordinate
legislation cannot override the principal legislative provisions.
The statute itself does not say that a penalty has to be levied only in the manner
prescribed. It is also not a case where the authority is left with no discretion. The
legislation does not provide that adjudication for the purpose of levy of penalty
proceeding would be a mere formality or imposition of penalty as also computation of the
quantum thereof became a foregone conclusion. Ordinarily, even such a provision would
not be held to providing for mandatory imposition of penalty, if the proceeding is an
adjudicatory one or compliance of the principles of natural justice is necessary
thereunder.
21. Existence of mens rea or actus reus to contravene a statutory provision must also be
held to be a necessary ingredient for levy of damages and/or the quantum thereof.
@page-SC1325
22. The Division Bench of the High Court, therefore, in our opinion, was not wrong in
opining that Section 85B provides for an enabling provision. What, however, cannot be
appreciated that is such a construction itself would lead to the conclusion that the High
Court is entitled to substitute its view in place of the statutory authority. In our considered
view, therefore, the matter should be considered afresh for determination of quantum of
damages etc. in the light of the observations made hereinbefore.
23. We are, therefore, of the opinion that the impugned judgments cannot be sustained. It
is set aside accordingly and the matter is remitted to the High Court for consideration of
the matter afresh in the light of the observations made herein. The appeal is allowed to
the aforementioned extent. In the facts and circumstances of this case, there shall be no
order as to costs.
Appeal allowed.
AIR 2008 SUPREME COURT 1325 "Krishna Janardhan Bhat v. Dattatraya G. Hegde"
(From : Karnataka)*
Coram : 2 S. B. SINHA AND H. S. BEDI, JJ.
Criminal Appeal No. 518 of 2006, D/- 11 -1 -2008.
Krishna Janardhan Bhat v. Dattatraya G. Hegde.
(A) Negotiable Instruments Act (26 of 1881), S.139 - DISHONOUR OF CHEQUE -
Presumption under - S.139 merely raises presumption in favour of holder of cheque that
same has been issued for discharge of any debt or other liability - Existence of legally
recoverable debt - Is not a matter of presumption u/S.139. (Para 21)
(B) Negotiable Instruments Act (26 of 1881), S.138, S.139 - DISHONOUR OF CHEQUE
- Dishonour of cheque - Defence - Proof - Accused not required to step into witness-box -
He may discharge his burden on basis of materials already brought on record - Question
whether statutory presumption rebutted or not - Must be determined in view of other
evidences on record.
Cri. R.P. No. 1470 of 2004, D/- 22-6-2005 (Kar), Reversed.
In case of dishonour of cheque it cannot be said that for proving the defence the accused
is required to step into the witness-box and unless he does so he would not be discharging
his burden. (Paras 22, 29, 35)
An accused for discharging the burden of proof placed upon him under a statute need not
examine himself. He may discharge his burden on the basis of the materials already
brought on record. An accused has a constitutional right to maintain silence. Standard of
proof on the part of an accused and that of the prosecution in a criminal case is different.
(Para 23)
Furthermore, whereas prosecution must prove the guilt of an accused beyond all
reasonable doubt, the standard of proof so as to prove a defence on the part of an accused
is 'preponderance of probabilities.' Inference of preponderance of probabilities can be
drawn not only from the materials brought on record by the parties but also by reference
to the circumstances upon which he relies. (Para 25)
A statutory presumption has an evidentiary value. The question as to whether the
presumption stood rebutted or not, must, therefore, be determined keeping in view the
other evidence on record. For the said purpose, stepping into the witness-box by the
appellant is not imperative. In a case of this nature, where the chances of false
implication cannot be ruled out, the background fact and the conduct of the parties
together with their legal requirements are required to be taken into consideration. (Para
26)

(C) Negotiable Instruments Act (26 of 1881), S.139 - DISHONOUR OF CHEQUE -


Presumption under - Rebuttal - Duty of Court - Presumption of innocence as human
rights and doctrine of reverse burden introduced by S. 139 - Should be delicately
balanced - It largely depends on factual matrix of each case.
The provision under S. 139 has been inserted to regulate the growing business, trade,
commerce and industrial activities of the country and the strict liability to promote
greater vigilance in financial matters and to safeguard the faith of the creditor in the
drawer of the cheque which is essential to the economic life of a developing country
@page-SC1326
like India. This however, shall not mean that the Courts shall put a blind eye to the
ground realities. Statute mandates raising of presumption but it stops at that. It does not
say how presumption drawn should be held to have rebutted. Other important principles
of legal jurisprudence, namely, presumption of innocence as human rights and the
doctrine of reverse burden introduced by S. 139 should be delicately balanced. Such
balancing acts, indisputably would largely depend upon the factual matrix of each case,
the materials brought on record and having regard to legal principles governing the same.
(Para 34)
The Courts must be on guard to see that merely on the application of presumption as
contemplated under S. 139, the same may not lead to injustice or mistaken conviction.
(Para 33)
Cases Referred : Chronological Paras
2007 AIR SW 6736 : 2008 (1) AIR Kar R 129 (Ref.) 30
2007 (12) Scale 96 (Ref.) 30
2006 AIR SCW 4652 : AIR 2006 SC 3366 : 2006 Cri LJ 4607 : 2006 CLC 1533 : 2006
(6) AIR Kar R 84 (Ref.) 27, 30, 35
2006 AIR SCW 5853 : AIR 2007 SC 451 : 2007 Cri LJ 304 : 2007 (1) AIR Jhar R 761
(Ref.) 33
2005 AIR SCW 2215 : AIR 2005 SC 2277 : 2005 Cri LJ 2533 (Ref.) 33
2004 AIR SCW 3094 : AIR 2004 SC 3249 : 2004 Cri LJ 2842 (Ref.) 33
2001 AIR SCW 3861 : AIR 2001 SC 3897 : 2001 Cri LJ 4647 : 2001 CLC 1646 (Ref.)
31
2001 AIR SCW 4344 : AIR 2001 SC 2895 : 2001 Cri LJ 4745 : 2001 CLC 1754 (Ref.)
31
2001 AIR - Kant HCR 2154 14
ILR 2001 Kar 4127 12
1999 AIR SCW 636 : AIR 1999 SC 1008 (Ref.) 24
1999 AIR SCW 3809 : AIR 1999 SC 3762 : 1999 Cri LJ 4606 (Ref.) 13
S. Balakrishnan, Sr. Advocate. (AC) and Krishna Janardhan Bhat, Appellant-in-Person;
S.N. Bhat, N.P.S. Pawar and D.P. Chaturvedi, for Respondent.
Judgement
1. S. B. SINHA, J. :-Appellant and one R.G. Bhat were jointly running a business in the
name and style of Vinaya Enterprises at Hubli together. Appellant executed a Power of
Attorney in his favour.
2. Allegedly, he had handed over four blank cheques to the said constituted attorney for
meeting the expenses of the business. The counterfoil of the cheque books was also
allegedly filled in by Shri R.G. Bhat.
The cheque bearing No. 044483 was shown to have been a self-drawn one for a sum of
Rs. 1500/-.
3. Disputes and differences having arisen between the appellant and the said R.G. Bhat in
connection with running of the said business, the power of attorney granted in his favour
was cancelled by the appellant. Disputes and differences between the parties were
referred to the Panchayat. In the meeting of the Panchayat held on 02-10-1996,
complainant/respondent who is the brother-in-law of the said R.G. Bhat was admittedly
present. He participated therein. The result of the said meeting of the Panchayat is not
known but it is not in dispute that the appellant herein issued a public notice through his
advocate in a local newspaper on 3-10-1996 to the following effect :
"My client Sh. Krishna Janardhana Bhat, Proprietor of Vinaya Enterprises, Tarihal Hubli
has given authority to give notice as follows :
My client appointed Shri Raghavendra Ganapati Bhat as his Power of Attorney Holder on
21-8-1993 to run Vinay Enterprises as agent. He has started misusing the terms and
conditions of the Power of Attorney. Hence my client cancelled the Power of Attorney on
21-8-96 by giving notice. If at all anybody deals with him on the Power of Attorney my
client is not responsible in future."
4-5. On the premise that the respondent advanced a sum of Rs. 1,50,000/- to the appellant
on 14-6-1998 and the latter on his own went to his house on 20-7-1998 to return the loan
by an account payee cheque which having been dishonoured when presented; a complaint
petition was filed.
6. Prior thereto, a notice was sent on 27-8-1998 which was allegedly served on the
appellant on 5-9-1998. He on that day itself sent a reply alleging in substance that the
complainant had been colluding with R.G. Bhat in regard thereto, stating :
"Your client D.G. Hegde Goddalamane is husband of sister of my power of attorney
holder R.G. Bhat (Proprietor Prasad Enterprises Tarihal Industrial Estate) of Hubli. I
@page-SC1327
do not have any dealing with him as alleged in your letter.
Knowing that the power of attorney holder R.G. Bhat has lost faith and having acted
illegally and in anticipation of his committing further illegal acts I have legally cancelled
my power of attorney and published the notice in a famous Kannada daily Samyukta
Karnataka on 3-10-96. From that date I do not have any relation with him or any of his
relatives including your client.
Please verify the handwriting and signature on the cheque and advice your client not to
do such (illegalities) colluding with his brother-in-law."
7. The learned Trial Judge convicted the appellant and sentenced him to undergo
imprisonment for six months and further directed payment of compensation for a sum of
Rs. 1,50,000/-. An appeal preferred thereagainst was dismissed by the Sessions Judge by
a judgment and order dated 28-7-2004.
8. The High Court in exercise of its revisional jurisdiction, however, on a revision
petition filed by the appellant, partly allowed the same by reducing the substantive
sentence to one week.
9. The Special Leave Petition was filed by the appellant in person. As it was noticed by a
Bench of this Court that some question of law arises for its consideration, Mr. S.
Balakrishnan, learned senior counsel was requested to assist the Court.
10. Mr. Balakrishnan urged that the learned Trial Judge, the Sessions Court as also the
High Court committed a serious illegality insofar as it misread and misapplied the
provisions of Section 139 of the Negotiable Instruments Act (for short "the Act").
It was contended that the procedural requirements of Section 138 are :
(i) There is a legally enforceable debt.
(ii) The drawer of the cheque issued the cheque to satisfy part or whole of the debt.
(iii) The cheque so issued has been returned due to insufficiency of funds.
It was urged that only ingredient No. 2 is a subject-matter of presumption under Section
139 of the Act and not the first one. It was argued that except the word of mouth of the
complainant nothing has been brought on record to prove the offence as against the
appellant.
11. Mr. S.N. Bhat, learned counsel appearing on behalf of the respondent, on the other
hand, submitted that the appellant has rightly been found guilty of commission of an
offence under Section 138 of the Act as bouncing of the cheque issued by him carries a
mandatory presumption in terms of Section 139 read with Section 118 (a) of the Act.
It was urged that it is not believable that the appellant despite referring the dispute to the
Panchayat and issuing a paper publication on 3-10-1996 would not insist on taking back
the cheque book from his erstwhile constituted attorney or would not inform the bank
thereabout. Moreover, he having come out with a positive defence, it was for him to
prove the same.
12. Before we embark upon the factual issue involved herein, we would notice the
manner in which the court proceeded to determine the case.
The learned Trial Judge framed the following points for its determination :
"(1) Whether the complainant proves the hilt that the accused to discharge earlier debt of
Rs.1,50,000/-, has got issued a cheque on 20-7-1998 for Rs.1,50,000/- drawn at Vijay
Bank, Tarahal Branch, Hubli?
(2) If so, whether the said cheque came to be dishonoured as "funds insufficient" after its
presentation and despite of issuance of notice, the accused did not pay the due amount
within stipulated time without any cause, thereby Negotiable Instruments Act?"
The learned Trial Judge noticed the contents of the claim petition as also the evidence of
PW-1. It also noticed the suggestions given to the said PW-1 by the appellant herein.
Upon taking into consideration the same as also the statement of the appellant under
Section 313 of the Code of Criminal Procedure, it posed a question as to whether there
was no debt payable by the accused to the complainant and if so, whether the
complainant colluding with R.G. Bhat had created the cheque with an intention to cause
loss to the appellant. It, however, without making any further discussion, answered the
said question directly on the material brought on record referring to a decision of the
Karnataka High Court in S.R. Muralidar v. Ashok G.Y. [ILR 2001 Karnataka 4127] in
extenso and opining that his decision in the case is similar to that of
@page-SC1328
the Karnataka High Court, stating :
"Considering the proposition of law, in the present case also the accused admitted the
signature on Ex. P. 1. But, the contention is that his P.A. Holder mis-utilized his signed
blank cheques through his relative complainant and the fact of the present case and fact
of the decision mentioned by me are similar one and the observation made by the Hon'ble
High Court in the above decision and principle laid down therein are clearly applicable to
the case in hand. Therefore, the defence taken by the accused herein without stepping into
the witness-box, is not acceptable one and there is no cogent evidence produced by the
accused to prove his special reasons for issuance of the cheque in question."
13

. It again referred to a decision of this Court in K. Bhaskaran v. Sankaran Vaidhyan Balan


and others [AIR 1999 SC 3762] and made almost a similar observation holding that as
the complainant has discharged his initial burden, the onus shifted on the accused to
produce rebuttal evidence against the presumption laid down in favour of the complainant
stating : 1999 AIR SCW 3809

"Here, the accused has not produced any evidence to discard the testimony of PW-1.
Therefore, the presumption is to be drawn in favour of the holder of the cheque, who has
received it for discharge of liability in view of the decision of the Hon'ble Supreme
Court."
14. Yet again, it relied upon a decision of the Karnataka High Court in M/s. Devi Tyres v.
Nawab Jan [AIR 2001 Karnataka HCR 2154], wherein it was opined :
"There is issued (sic) that the amount is payable and no criminal court is required to
embark upon any enquiry that goes behind the Act of issuance of the cheque. If the
drawer contends that there were certain special reasons whereby a cheque was issued and
that the cheque was not intended to be encashed or honoured, the onus of establishing this
shifts squarely to the accused."
15. The complainant's case was, thus, primarily accepted for the reason that the appellant
did not step into the witness box.
16. The appellate court took an identical stand. It proceeded on the premise that the
statement of accused under Section 313 of the Code of Criminal Procedure regarding
misuse of blank cheque by the complainant and filling up Rs. 1,50,000/- instead of Rs.
1500/- is contradictory to his own admission in the reply to the notice issued to him.
On what basis the said opinion was formed is not known. The appellate court refused to
enter into the question as to whether the prosecution case is wholly unreliable, as the
complainant had not been able to show his source of income so as to enable him to
advance a huge loan of Rs. 1,50,000/-, holding :
"Now as far as the financial ability of the complainant to issue cheque for such huge
amount to the accused is not a matter to be considered by the trial court or by me also
since issue of Ex. P. 1 and its dishonour is proved by the complainant beyond reasonable
doubt."
17. The High Court in exercise of its revisional jurisdiction although accepted the
contention of the appellant that the presumption under Section 139 of the Act extends
"only to the extent that the holder of the cheque received the cheque for the discharge in
whole or in part of any debt or other liability" and the same only means "that cheque was
issued for consideration, but does not extend to the extent that the cheque was issued for
the discharge of the debt or liability as pleaded by the accused", opined that the
complainant had discharged that onus by adducing his own evidence. Observing that the
appellant did not step into the witness box, it was opined that although the relationship
between the appellant and Shri R.G. Bhat was strained, there was nothing to show that
the relationship between the appellant and the complainant became strained despite the
fact that a panchayat meeting was held in regard to the said dispute in 1996. The High
Court, however, refused to go into the factual aspect of the matter stating that it was
exercising a revisional jurisdiction, stating :
"Since the burden of proving that the cheque had been misused is on the accused-
petitioner, and there being a concurrent finding of the Trial Court and the Appellant Court
with regard to that holding that the petitioner had failed to discharge that burden, I do not
find any ground to interfere in the order of the Trial Court and that the Appellate Court,
so far as they hold the petitioner guilty of an offence punishable under Section 138 of the
Negotiable Instruments Act."
@page-SC1329
18. Before embarking upon the legal issues, we may analyse the deposition of PW-1
Complainant. He was a resident of village Goddalmane. Appellant is a resident of village
Kekkar. As he was running an industry at Hubli, he sometimes resided in Hubli also.
They were said to be friends. He asked him to give a loan of Rs. 1.5 lakhs in the first
week of June, 1998 and the amount was handed over to him on 14th June, 1998. It was
allegedly agreed that on the appellant's failure to repay the said loan within one month,
15% interest would be charged. No document was executed; no pronote was executed; no
receipt was obtained. Appellant is said to have come to his house suo motu on 20-07-
1998 and handed over the cheque which was sent to Varada Grameen Bank for collection
whereupon notice had been issued. Despite the fact that he was aware that a dispute had
been raised in regard to the writings in the cheque, the same was not proved. Merely, the
cheque was tendered and it was marked as an exhibit. The cheque appears to have been
issued as a proprietor of a business concern.
Despite the fact that R.G. Bhat was his brother-in-law, he denied that he was running the
said business. He also feigned his ignorance as to whether the said industry was being run
by R.G. Bhat on the basis of the Power of Attorney executed by the appellant. He,
however, accepted that they had been running it together. He also accepted the
relationship between him and R.G. Bhat. He knew about the dispute. He accepted that a
panchayat meeting was held in regard thereto. Surprisingly, he denied his knowledge in
regard to the existence of the power of attorney stating that the same was not made in his
presence. He admitted that he was present on 2-10-1996 in the panchayat meeting to
resolve the problem arising out of the dispute between R.G. Bhat and the appellant. He
accepted that wooden and steel materials were placed in Vinay Enterprises and R.G. Bhat
had been running the same type of industry in Tarihal Industrial Estate. According to him,
he had been running such an industry in the name of Prasad Enterprises even prior to
1996. His acquaintance, according to him, with the appellant was only through his
brother-in-law. He did not say that he had friendship with the appellant. There also does
not appear to be any business transactions between them. He could not state about the
denomination of the notes although according to him he had drawn the amount from the
society.
He did not produce any books of accounts or any other proof to show that he got so much
money from the Bank. He admittedly did not have any written document pertaining to the
accused. He accepted that there was no witness to the transaction. He, of course, denied
certain suggestions, but the suggestions put to him were required to be considered by the
court below in the backdrop of the facts and circumstances of the case.
19. The courts below failed to notice that ordinarily in terms of Section 269-SS of the
Income-tax Act, any advance taken by way of any loan of more than Rs. 20,000/- was to
be made by way of an account payee cheque only.
Section 271D of the Income-tax Act reads as under :
"271D. Penalty for failure to comply with the provisions of section 269-SS.- (1) If a
person takes or accepts any loan or deposit in contravention of the provisions of section
269-SS, he shall be liable to pay, by way of penalty, a sum equal to the amount of the
loan or deposit so taken or accepted.
(2) Any penalty impossible under sub-section (1) shall be imposed by the Joint
Commissioner."
20. Indisputably, a mandatory presumption is required to be raised in terms of Section
118(b) and Section 139 of the Act. Section 13(1) of the Act defines "negotiable
instrument" to mean "a promissory note, bill of exchange or cheque payable either to
order or to bearer."
Section 138 of the Act has three ingredients, viz. :
(i) that there is a legally enforceable debt;
(ii) that the cheque was drawn from the account of bank for discharge in whole or in part
of any debt or other liability which presupposes a legally enforceable debt; and
(iii) that the cheque so issued had been returned due to insufficiency of funds.
21. The proviso appended to the said section provides for compliance of legal
requirements before a complaint petition can be acted upon by a court of law. Section 139
of the Act merely raises a presumption in regard to the second aspect of the matter
@page-SC1330
Existence of legally recoverable debt is not a matter of presumption under Section 139 of
the Act. It merely raises a presumption in favour of a holder of the cheque that the same
has been issued for discharge of any debt or other liability.
22. The courts below, as noticed hereinbefore, proceeded on the basis that Section 139
raises a presumption in regard to existence of a debt also. The courts below, in our
opinion, committed a serious error in proceeding on the basis that for proving the defence
the accused is required to step into the witness box and unless he does so he would not be
discharging his burden. Such an approach on the part of the courts, we feel, is not correct.
23. An accused for discharging the burden of proof placed upon him under a statute need
not examine himself. He may discharge his burden on the basis of the materials already
brought on records. An accused has a constitutional right to maintain silence. Standard of
proof on the part of an accused and that of the prosecution in a criminal case is different.
24

. In Bharat Barrel and Drum Manufacturing Company v. Amin Chand Pyarelal [(1999) 3
SCC 35] interpreting Section 118(a) of the Act, this Court opined : "Upon consideration
of various judgments as noted hereinabove, the position of law which emerges is that
once execution of the promissory note is admitted, the presumption under Section 118(a)
would arise that it is supported by a consideration. Such a presumption is rebuttable. The
defendant can prove the non-existence of a consideration by raising a probable defence. If
the defendant is proved to have discharged the initial onus of proof showing that the
existence of consideration was improbable or doubtful or the same was illegal, the onus
would shift to the plaintiff who will be obliged to prove it as a matter of fact and upon its
failure to prove would disentitle him to the grant of relief on the basis of the negotiable
instrument. The burden upon the defendant of proving the non-existence of the
consideration can be either direct or by bringing on record the preponderance of
probabilities by reference to the circumstances upon which he relies. In such an event, the
plaintiff is entitled under law to rely upon all the evidence led in the case including that of
the plaintiff as well. In case, where the defendant fails to discharge the initial onus of
proof by showing the non-existence of the consideration, the plaintiff would invariably be
held entitled to the benefit of presumption arising under Section 118(a) in his favour. The
court may not insist upon the defendant to disprove the existence of consideration by
leading direct evidence as the existence of negative evidence is neither possible nor
contemplated and even if led. is to be seen with a doubt..." 1999 AIR SCW 636, Para 12

[Emphasis supplied]
25. Furthermore, whereas prosecution must prove the guilt of an accused beyond all
reasonable doubt, the standard of proof so as to prove a defence on the part of an accused
is "preponderance of probabilities". Inference of preponderance of probabilities can be
drawn not only from the materials brought on records by the parties but also by reference
to the circumstances upon which he relies.
26. A statutory presumption has an evidentiary value. The question as to whether the
presumption whether stood rebutted or not, must, therefore, be determined keeping in
view the other evidences on record. For the said purpose, stepping into the witness box
by the appellant is not imperative. In a case of this nature, where the chances of false
implication cannot be ruled out, the background fact and the conduct of the parties
together with their legal requirements are required to be taken into consideration.
27

. In M.S. Narayana Menon alias Mani v. State of Kerala and Another [(2006) 6 SCC 39],
it was held that once the accused is found to discharge his initial burden, it shifts to the
complainant. 2006 AIR SCW 4652
28. Four cheques, according to the accused, appear to have been drawn on the same day.
The counterfoil of the cheque book, according to the appellant, was in the handwriting of
R.G. Bhat wherein it was shown that apart from other payments, a sum of Rs. 1500/- was
withdrawn on a self-drawn cheque. The courts below proceeded to hold that the defence
raised by the appellant has not been proved, which, in our opinion, is not correct. He did
not know that the said cheque had not been encashed. He replied to the notice thinking
that one of the cheques has been misused. There is nothing on record to show that he
knew that one of the cheques was still with R.G. Bhat.
@page-SC1331
29. Disputes and differences between him and R.G. Bhat stood established by admission
of the respondent himself. Similar industry was being run by R.G. Bhat although he was
acting as the constituted attorney of the appellant. According to the appellant, R.G. Bhat
had cheated him. The counterfoil showed that not more than Rs. 20,000/- had ever been
withdrawn from that Bank at a time. The courts were required to draw an inference as to
the probability of the complainant's advancing a sum of Rs. 1.5 lakhs on mere asking and
that too without keeping any documentary proof. Even there was no witness. The
purported story that the appellant would himself come forward to return the amount by a
cheque knowing fully well that he did not have any sufficient funds is difficult to believe.
30

. In K. Prakashan v. P.K. Surenderan [2007 (12) SCALE 96], this Court following M.S.
Narayana Menon (supra) opined : 2006 AIR SCW 4652

"12. The Act raises two presumptions; firstly, in regard to the passing of consideration as
contained in Section 118 (a) therein and, secondly, a presumption that the holder of
cheque receiving the same of the nature referred to in Section 139 discharged in whole or
in part any debt or other liability. Presumptions both under Sections 118 (a) and 139 are
rebuttable in nature. Having regard to the definition of terms "proved" and "disproved" as
contained in Section 3 of the Evidence Act as also the nature of the said burden upon the
prosecution vis-a-vis an accused it is not necessary that the accused must step into the
witness box to discharge the burden of proof in terms of the aforementioned provision.
13. It is furthermore not in doubt or dispute that whereas the standard of proof so far as
the prosecution is concerned is proof of guilt beyond all reasonable doubt; the one on the
accused is only mere preponderance of probability."

In John K. John v. Tom Varghese and Anr. [JT 2007 (13) SC 222], this Court held :
2007 AIR SCW 6736

" 10...The High Court was entitled to take notice of the conduct of the parties. It has been
found by the High Court as of fact that the complainant did not approach the court with
clean hands. His conduct was not that of a prudent man. Why no instrument was executed
although a huge sum of money was
allegedly paid to the respondent was a relevant question which could be posed in the
matter. It was open to the High Court to draw its own conclusion therein. Not only no
document had been executed, even no interest had been charged. It would be absurd to
form an opinion that despite knowing that the respondent even was not in a position to
discharge his burden to pay instalments in respect of the prized amount, an advance
would be made to him and that too even after institution of three civil suits. The amount
advanced even did not carry any interest. If in a situation of this nature, the High Court
has arrived at a finding that the respondent has discharged his burden of proof cast on
him under Section 139 of the Act, no exception thereto can be taken."
31

. Mr. Bhat relied upon a decision of this Court in Hiten P. Dalal v. Bratindranath Banerjee
[(2001) 6 SCC 16] wherein this Court held : 2001 AIR SCW 3861

"22...Presumptions are rules of evidence and do not conflict with the presumption of
innocence, because by the latter, all that is meant is that the prosecution is obliged to
prove the case against the accused beyond reasonable doubt. The obligation on the
prosecution may be discharged with the help of presumptions of law or fact unless the
accused adduces evidence showing the reasonable possibility of the non-existence of the
presumed fact.
23 . In other words, provided the facts required to form the basis of a presumption of law
exist, no discretion is left with the court but to draw the statutory conclusion, but this
does not preclude the person against whom the presumption is drawn from rebutting it
and proving the contrary. A fact is said to be proved when,
"after considering the matters before it, the court either believes it to exist, or considers
its existence so probable that a prudent man ought, under the circumstances of the
particular case, to act upon the supposition that it exists".
Therefore, the rebuttal does not have to be conclusively established but such evidence
must be adduced before the court in support of the defence that the court must either
believe the defence to exist or consider its existence to be reasonably probable, the
standard of reasonability being that of the "prudent man."."
@page-SC1332

[See also K.N. Beena v. Muniyappan and another (2001) 8 SCC 458] 2001 AIR
SCW 4344

32. We assume that the law laid down therein is correct. The views we have taken are not
inconsistent therewith.
33. But, we may at the same time notice the development of law in this area in some
jurisdictions.

The presumption of innocence is a human right. [See Narender Singh and Ann v. State of
M.P. (2004) 10 SCC 699; Ranjitsing Brahmajeetsing Sharma v. State of Maharashtra and
Anr. (2005) 5 SCC 294 and Rajesh Ranjan Yadav @ Pappu Yadav v. CBI through its
Director (2007) 1 SCC 70] Article 6(2) of he European Convention on Human Rights
provides : "Everyone charged with a criminal offence shall be presumed innocent until
proved guilty according to law". Although India is not bound by the aforementioned
Convention and as such it may not be necessary like the countries forming European
countries to bring common law into land with the Convention, a balancing of the accused
rights and the interest of the society is required to be taken into consideration. In India,
however, subject to the statutory interdicts, the said principle forms the basis of criminal
jurisprudence. For the aforementioned purpose the nature of the offence, seriousness as
also gravity thereof may be taken into consideration. The courts must be on guard to see
that merely on the application of presumption as contemplated under Section 139 of the
Negotiable Instruments Act, the same may not lead to injustice or mistaken conviction. It
is for the aforementioned reasons that we have taken into consideration the decisions
operating in the field where the difficulty of proving a negative has been emphasized. It is
not suggested that a negative can never be proved but there are cases where such
difficulties are faced by the accused e.g., honest and reasonable mistake of fact. In a
recent Article "The Presumption of Innocence and Reverse Burdens : A Balancing Duty"
published in [2007] CLJ (March Part) 142 it has been stated :- 2004 AIR SCW 3094
2005 AIR SCW 2215
2006 AIR SCW 5853

"In determining whether a reverse burden is compatible with the presumption of


innocence regard should also be had to the pragmatics of proof. How difficult would it be
for the prosecution to prove guilt without the reverse burden? How easily could an
innocent defendant discharge the reverse burden? But courts will not allow these
pragmatic considerations to override the legitimate rights of the defendant. Pragmatism
will have greater sway where the reverse burden would not pose the risk of great injustice
- where the offence is not too serious or the reverse burden only concerns a matter
incidental to guilt. And greater weight will be given to prosecutorial efficiency in the
regulatory environment."
34. We are not oblivious of the fact that the said provision has been inserted to regulate
the growing business, trade, commerce and industrial activities of the country and the
strict liability to promote greater vigilance in financial matters and to safeguard the faith
of the creditor in the drawer of the cheque which is essential to the economic life of a
developing country like India. This, however, shall not mean that the courts shall put a
blind eye to the ground realities. Statute mandates raising of presumption but it stops at
that. It does not say how presumption drawn should be held to have rebutted. Other
important principles of legal jurisprudence, namely presumption of innocence as human
rights and the doctrine of reverse burden introduced by Section 139 should be delicately
balanced. Such balancing acts, indisputably would largely depend upon the factual matrix
of each case, the materials brought on record and having regard to legal principles
governing the same.
35

. Keeping in view the peculiar facts and circumstances of this case, we are of the opinion
that the courts below approached the case from a wholly wrong angle, viz., wrong
application of the legal principles in the fact-situation of the case. In view of the legal
position as has been enunciated by this Court in M.S. Narayana Menon (supra) and later
cases, we are of the opinion that the High Court should have entertained the revision
application. 2006 AIR SCW 4652
36. For the reasons aforementioned, the appeal is allowed. The judgments of conviction
and sentence passed against the appellant are set aside.
Appeal allowed.
@page-SC1333
AIR 2008 SUPREME COURT 1333 "Kulwinder Kaur v. Kandi Friends Education Trust"
(From : Punjab and Haryana)
Coram : 2 C. K. THAKKER AND MARKANDEY KATJU, JJ.
Civil Appeal No. 338 of 2008 (arising out of S.L.P. (C) No. 21147 of 2006), D/- 11 -1
-2008.
Kulwinder Kaur alias Kulwinder Gurcharan Singh v. Kandi Friends Education Trust and
Ors.
Civil P.C. (5 of 1908), S.24 - TRANSFER OF CASE - HIGH COURT - Transfer of suit -
Powers u/S.24 cannot be exercised ipse dixit - Application by plaintiff - Assertion made
by plaintiff contradicted by defendants - High Court without applying mind to those
aspects and without recording any reason/ground allowed application by observing that it
would be 'appropriate' to transfer the suit pending in Court of "A," Civil Judge, at 'R' to
Court of 'Y' Additional Civil Judge at 'C' - Order passed by High Court liable to be set
aside - Matter remitted for fresh disposal.
C.M. No. 22108-CII of 2006. D/- 17-11-2006 (PandH), Reversed. (Paras 13, 17, 18)
Cases Referred : Chronological Paras
AIR 1990 SC 113 (Ref.) 16
AIR 1979 SC 468 : 1979 Cri LJ 458 (Ref.) 15
Puneet Bali, Hittan Nehra and S.S. Jauhar, for Appellant; Sridhar, Sr. Advocate, Rishi
Malhotra, R. Singh and Prem Malhotra, for Respondents.
Judgement
1. C. K. THAKKER. J, :-Leave granted.
2. This appeal is directed against the judgment and order dated November 17, 2006 in
Civil Miscellaneous No. 22108-CII of 2006. By the said order, the High Court of Punjab
and Haryana transferred Civil Suit No. 506 of 2003 from the Court of Smt. Asha Kondal,
Civil Judge (Sr. Divn.), Ropar to the Court of Sh. Y. S. Rathore, Additional Civil Judge
(Sr. Divn.), Chandigarh.
3. Short facts giving rise to present appeal are that Kandi Friends Educational Trust
("Trust" for short) was set up for establishing professional educational institutions with
prominent educationists and industrialists of the State of Punjab on September 24, 1997.
It is the case of the appellant that Gurcharan Singh, her husband was the Founder
Chairman of the Trust whereas the appellant was a Trustee along with the Founder
Chairman. In 1998, certain new trustees were inducted including one B. S. Randhawa. In
September, 2002, elections were held and Gurcharan Singh was again elected as the
Chairman of the Trust. B. S. Randhawa and his wife Hardev Kaur raised protest against
the said election. In December, 2002, Gurcharan Singh, Chairman of the Trust sought
certain amendments in the constitution of the Trust which were approved by majority
though B. S. Randhawa and Hardev Kaur opposed to such amendments. On June 21,
2003, Gurcharan Singh, Chairman of the Trust was murdered while he was taking stroll
in a park along with the appellant. B. S. Randhawa, who was one of the Trustees, was
arrested as the main accused and was charged for committing murder of Gurcharan
Singh. F.I.R. No. 271 of 2003 was registered on the same day at Mohali Police Station. In
view of death of Gurcharan Singh, election of the Chairman was again held on July 23,
2003 and the appellant was unanimously elected as the Chairperson. Ms. Japneet Kaur
was nominated as trustee being daughter of late Gurcharan Singh and she also started
attending meetings of the Trust. B. S. Randhawa and Hardev Kaur were obviously
unhappy with the development. Hardev Kaur, hence, filed a suit on July 25, 2003 for a
declaration that all proceedings conducted by the defendants in the meeting dated July 23,
2003 in which the appellant was elected as the Chairperson were illegal, null and void
and liable to be set aside. Certain other reliefs were also claimed. In the suit, the appellant
herein was impleaded as defendant No. 4. Along with the plaint, the plaintiffs filed an
application under O. XXXIX, Rr. 1 and 2 of the Code of Civil Procedure, 1908
(hereinafter referred to as "the Code") for interim relief, but only limited interim relief
was granted and the plaintiff-Smt. Hardev Kaur was allowed to attend meetings of the
Trust. Other interim reliefs were expressly refused. Having failed to get relief sought in
interim application, Hardev Kaur and B. S. Randhawa filed another suit, i.e. the present
suit in October, 2003 in the name of Kandi Friends Education Trust through its so-called
General Secretary Jaspal Singh. Though the appellant was duly elected as Chairperson of
the Trust, she was wrongly described as Trustee and it was stated by the plaintiff that they
were
@page-SC1334
in-charge and in management of the Trust. A declaration was sought that the resolution
dated October 14, 2003 adopted by the defendants was illegal, null and void.
Consequential reliefs were also prayed.
4. On June 4, 2005, one more suit was filed by Jaspal Singh for declaration and
permanent injunction against the appellant. In interim injunction, only status quo was
ordered to be maintained by the Court. Jaspal Singh also filed a transfer application No.
14 of 2006 in the District Court, Ropar for transfer of suit from the Court of Smt. Asha
Kondal to the Court of Shri A. S. Garewal, which was, however, dismissed as withdrawn.
5. On November 2, 2006, the Trust filed an application under S. 24 of the Code in the
High Court of Punjab and Haryana being Civil Miscellaneous No. 22108-CII of 2006 for
transfer of Suit No. 506 of 2003 pending in the Court of Smt. Asha Kondal, Sub-Judge,
(Sr. Divn.), Ropar "to any other Court of competent jurisdiction in Chandigarh or in the
State of Haryana" in view of "peculiar facts and circumstances of the case in the interest
of justice." It was inter alia alleged in the transfer application that though the suit was
instituted in 2003 seeking injunction against the defendant-appellant herein and others, it
was pending even in November, 2006. More than three years had passed and yet there
was no 'much progress' in the case. It was further alleged that there was lot of local
pressure which had led to delay and it had given advantage to the defendants as they were
in power and were trying to protract the proceedings. It was asserted that the Institution
was one of the most prestigious institutions in the area and lots of funds were generated
as there were several students. Hence, the Committee members who were in office were
trying their level best to stall the proceedings by using various tactics. It was also stated
that though the term of the appellant expired on August 31, 2005, she continued to be in
power simply because no case filed against her was decided either way. A prayer was,
therefore, made to transfer the case.
6. The appellant herein filed detailed reply to the application contending that false and
scandalous allegations have been levelled by the plaintiff against the defendants which
were not correct. It was submitted that suit filed by the plaintiff was not maintainable as
there was no proper resolution and no authority had been given by the appellant-
Chairperson to file such suit. It was further stated that no one could have a Court of one's
own choice and on the facts and in the circumstances, no case was made out for transfer
of suit. It was stated that Zimni orders clearly revealed that there was no delay on the part
of the defendants. The delay was largely attributable to the plaintiff-trust.
7. In this connection, it was stated in the reply as under :

Date of filing : 11-11-2003


Issues framed : 24-12-2003
For PWs. : 30-3-2004
No PWs. Produced : Till date 29-11 -2006.

It was, therefore, submitted that the application was devoid of merit, laced with malice
and was liable to be dismissed.
8. The learned single Judge of the High Court observed that it was alleged by the plaintiff
that though the suit was filed in 2003, there was 'no substantive progress' in the suit. The
learned Judge no doubt observed that certain allegations levelled by the plaintiff had been
controverted and counter-allegations had been made by the defendants. But without going
into the allegations and counter-allegations, it would be appropriate to transfer the suit to
Chandigarh. Accordingly, by the impugned order, the suit was transferred and a direction
was issued to decide the matter expeditiously by giving two opportunities to each of the
parties spread over a period of six months.
9. The aforesaid order is challenged by the appellant in the present appeal. On January 5,
2007, notice was issued by this Court and further proceedings in the suit were stayed.
Counter-affidavit was thereafter filed. The matter then was ordered to be posted for final
hearing.
10. We have heard the learned counsel for the parties.
11. The learned counsel for the appellant contended that the High Court committed an
error of law and of jurisdiction in transferring the case from Ropar to Chandigarh. It was
submitted that no reasons/grounds have been disclosed for taking such action of
transferring the suit. The counsel submitted that all the allegations levelled by the
plaintiff had been controverted
@page-SC1335
by the defendants and even the learned Judge of the High Court had observed in the order
that there were allegations and counter-allegations by the parties. In spite of such
situation, the Court passed the impugned order of transfer which is not in consonance
with law. On merits, it was submitted that it was factually incorrect to allege that there
was delay on the part of the defendants. The defendants had produced Zimni which went
to show that it was the plaintiff and not the defendants who was responsible for the delay.
If it is so, the High Court was wrong in passing the impugned order. Finally, it was
submitted that the High Court was not justified in transferring the case in the Court of a
particular named Judge. Normally, no such order is passed. Even in the transfer
application, no such prayer was made by the plaintiff. The order to that extent, therefore,
deserves to be set aside.
12. The learned counsel for the respondent, on the other hand, supported the impugned
order. It was submitted that the High Court was satisfied that S. 24 of the Code confers
discretionary power on the Court to transfer a case from one Court to any other Court
subordinate to it. In exercise of the said power, an action has been taken which cannot be
challenged under Art. 136 of the Constitution. It was submitted that the High Court took
into account 'ground reality' that a suit of 2003 which was of an urgent nature was not
disposed of even in 2006. If, in the light of the above fact, the case is transferred, it could
not be said that the order deserves interference in exercise of discretionary jurisdiction by
this Court. It was, therefore, submitted that the appeal deserves to be dismissed.
13. Having considered rival contentions of the parties and having gone through the
proceedings of the case, we are of the view that the impugned order deserves to be set
aside. So far as the power of transfer is concerned, S. 24 of the Code empowers a High
Court or a District Court to transfer inter alia any suit, appeal or other proceeding pending
before it or in any Court subordinate to it to any other Court for trial and disposal. The
said provision confers comprehensive power on the Court to transfer suits, appeals or
other proceedings 'at any stage' either on an application by any party or suo motu.
14. Although the discretionary power of transfer of cases cannot be imprisoned within a
strait-jacket of any cast-iron formula unanimously applicable to all situations, it cannot be
gainsaid that the power to transfer a case must be exercised with due care, caution and
circumspection. Reading Ss. 24 and 25 of the Code together and keeping in view various
judicial pronouncements, certain broad propositions as to what may constitute a ground
for transfer have been laid down by Courts. They are balance of convenience or
inconvenience to plaintiff or defendant or witnesses; convenience or inconvenience of a
particular place of trial having regard to the nature of evidence on the points involved in
the suit; issues raised by the parties; reasonable apprehension in the mind of the litigant
that he might not get justice in the Court in which the suit is pending; important questions
of law involved or a considerable section of public interested in the litigation; 'interest of
justice' demanding for transfer of suit, appeal or other proceeding, etc. Above are some of
the instances which are germane in considering the question of transfer of a suit, appeal
or other proceeding. They are, however, illustrative in nature and by no means be treated
as exhaustive. If on the above or other relevant considerations, the Court feels that the
plaintiff or the defendant is not likely to have a 'fair trial' in the Court from which he
seeks to transfer a case, it is not only the power, but the duty of the Court to make such
order.
15

. In Maneka Sanjay Gandhi v. Rani Jethmalani, (1979) 2 SCR 378, this Court stated :
AIR 1979 SC 468, (Para 2)

"Assurance of a fair trial is the first imperative of the dispensation of justice and the
central criterion for the Court to consider when a motion for transfer is made is not the
hypersensitivity or relative convenience of a party or easy availability of legal services or
like mini grievances. Something more substantial, more compelling, more imperiling,
from the point of view of public justice and its attendant environment, is necessitous if
the Court is to exercise its power of transfer. This is the cardinal principle although the
circumstances may be myriad and vary from case to case."
(Emphasis supplied)
16

. Similarly in Subramaniam Swamy v. Ramakrishna Hegde, (1990)1 SCC 4, dealing with


power of this Court to transfer a case under AIR 1990 SC 113
(Para 8)

@page-SC1336
Section 25 of the Code, A. M. Ahmadi, J. (as His Lordship then was) stated :
"Under the old section the State Government was empowered to transfer a suit, appeal or
other proceeding pending in the High Court of that State to any other High Court on
receipt of a report from the Judge trying or hearing the suit that there existed reasonable
grounds for such transfer provided the State Government of the State in which the other
High Court had its principal seat consented to the transfer. The present Section 25 confers
the power of transfer on the Supreme Court and is of wide amplitude. Under the present
provision the Supreme Court is empowered at any stage to transfer any suit, appeal or
other proceeding from a High Court or other Civil Court in one State to a High Court or
other Civil Court of another State if it is satisfied that such an order is expedient for the
ends of justice. The cardinal principle for the exercise of power under this section is that
the ends of justice demand the transfer of the suit, appeal or other proceeding. The
question of expediency would depend on the facts and circumstances of each case but the
paramount consideration for the exercise of power must be to meet the ends of justice. It
is true that if more than one Court has jurisdiction under the Code to try the suit, the
plaintiff as dominus litis has a right to choose the Court and the defendant cannot demand
that the suit be tried in any particular Court convenient to him. The mere convenience of
the parties or any one of them may not be enough for the exercise of power but it must
also be shown that trial in the chosen forum will result in denial of justice. Cases are not
unknown where a party seeking justice chooses a forum most inconvenient to the
adversary with a view to depriving that party of a fair trial. The Parliament has, therefore,
invested this Court with the discretion to transfer the case from one Court to another if
that is considered expedient to meet the ends of justice. Words of wide amplitude for the
ends of justice have been advisedly used to leave the matter to the discretion of the Apex
Court as it is not possible to conceive of all situations requiring or justifying the exercise
of power. But the paramount consideration must be to see that Justice according to law is
done: If for achieving that objective the transfer of the case is imperative, there should be
no hesitation to transfer the case even if it is likely to cause some inconvenience to the
plaintiff. The petitioner's plea for the transfer of the case must be tested on this
touchstone."
(Emphasis supplied)
17. In the case on hand, the High Court without stating anything whatsoever as to
allegations and counter-allegations, without considering the reply submitted by the
appellant herein and without recording any reason/ground passed the impugned order
transferring the case. The learned counsel for the contesting respondent no doubt
submitted that the Court has not observed anything since observations by a High Court
one way or the other might prejudice one of the parties to the suit. It is true that normally
while making an order of transfer, the Court may not enter into merits of the matter as it
may affect the final outcome of the proceedings or cause prejudice to one or the other
side. At the same time, however, an order of transfer must reflect application of mind by
the Court and the circumstances which weighed in taking the action. In the instant case, it
was alleged by the plaintiff that though more than three years had passed from instituting
the suit, it was not disposed of and delay had been caused by the defendants as they were
in office and they wanted to prolong the proceedings so that they may take undue benefit
of their status. The defendants, in the reply filed by them, contended that delay had not
been caused by them, but it was the plaintiff who was responsible for not proceeding with
the suit and was to be blamed for creation of such situation. In support of the contention,
Zimni proceedings were relied upon. It was also urged that the plaintiff-side could not get
favourable order on applications under Order XXXIX, Rules 1 and 2 of the Code and,
hence, it wanted to get the case transferred. In view of the assertion and retraction by the
plaintiff and the defendants, in our considered opinion, the High Court ought to have
applied its mind to those aspects and prima facie satisfied as to the grounds put forward
by the plaintiff in the transfer application and ought to have passed an order one way or
the other without entering into the controversy in the suit. Unfortunately, the High Court
allowed the application observing that it would be 'appropriate' to transfer the suit
pending in the Court of Smt. Asha Konal, Civil Judge, (Sr. Divn.), Ropar to the Court of
Sh. Y. S. Rathore, Additional Civil Judge (Sr. Divn.),
@page-SC1337
Chandigarh. In our opinion, powers under Section 24 of the Code cannot be exercised
ipse dixit in the manner in which it has been done. Only on that ground and without
entering into larger issue, the appeal deserves to be allowed and is accordingly, allowed.
18. For the foregoing reasons, the appeal is allowed. The order passed by the High Court
is set aside and the matter is remitted to the High Court for fresh disposal in accordance
with law after hearing the parties. On the facts and in circumstances of the case, however,
there shall be no order as to costs.
19. Before parting with the matter, we make it clear that we have not entered into
correctness or otherwise of what is stated by the plaintiff or by the defendants and we
may not be understood to have expressed any opinion on allegations and counter-
allegations. As and when the matter will be placed before the High Court, the Court will
take an appropriate decision on its own merits without being inhibited or influenced by
the observations made by us in this judgment.
Appeal allowed.
AIR 2008 SUPREME COURT 1337 "K. S. Krishna Sarma v. Kifayat Ali"
(From : Andhra Pradesh)
Coram : 2 Dr. A. PASAYAT AND P. SATHASIVAM, JJ.
Civil Appeal No. 187 of 2008 (arising out of S.L.P. (C) No. 24776 of 2005), D/- 9 -1
-2008.
K.S. Krishna Sarma v. Kifayat Ali.
Civil P.C. (5 of 1908), O.41, R.27 - APPEAL - EVIDENCE - DECLARATION OF
TITLE - REMAND OF MATTER - Additional evidence - Suit for declaration of title -
Defendant subsequently impleaded - Case remanded with direction to consider matter in
so far as interest of added defendant is concerned - Additional issues framed - Parties
cannot lead evidence afresh on other issues - Evidence to be restricted in respect of
defence taken by the newly impleaded defendant in written statement. (Para 6)
Sridhar Potaraju, D. Julius Riamei, John Mathew, for Appellant; Anil Kumar Tandale, for
Respondent.
Judgement
P. SATHASIVAM, J. :-Leave granted.
2. This appeal is directed against the order dated 13-9-2005 passed by the learned
single Judge of the High Court of Andhra Pradesh in C.R.P. No. 3360 of 2005 in and by
which the learned Judge upheld the order dated 24-2-2005 of the Xth Additional Chief
Judge (Fast Track Court), City Civil Court, Hyderabad in O.S. No. 296 of 1982.
3. Brief facts in nutshell are :
The first defendant in O.S. No. 296 of 1982 on the file of the Xth Additional Chief Judge
(Fast Track Court), City Civil Court, Hyderabad is the appellant in the present appeal.
The respondent herein was the plaintiff in that suit. In respect of the agricultural land
measuring Acs. 32.00 covering Survey Nos. 141, 142 and 143 and buildings belonging to
one late Salarjung, the plaintiff filed the said suit for declaration of title and for
consequential possession. The suit was filed originally against K. S. Krishna Sarma, the
appellant herein, and one Seshachalapathi as defendants. During the pendency of the suit,
Seshachalapathi died and his legal representatives were sought to be brought on record in
LA. No. 189 of 1983. Among the legal representatives, one Smt. A. Annapurna, daughter
of late Seshachalapathi, was not brought on record since the application to bring her on
record came to be dismissed due to non-payment of process fee. Other legal
representatives were brought on record. The suit was resisted by filing written statements
by 1st and 4th defendants. Finally, the suit was decreed in favour of the plaintiff. The
appeal was filed before the High Court at the instance of defendant Nos. 1, 2 and 4.
Learned single Judge of the High Court, after finding that in the absence of Smt. A.
Annapurna, one of the legal representatives, the decree was defective, allowed the appeal
and remanded the matter to the trial Court with a direction to permit Smt. A. Annapurna
to come on record. The said order of the learned single Judge was challenged by the
plaintiff by filing L.P.A. No. 27 of 1997 before the Division Bench of the High Court.
The Division Bench set aside the order of the learned single Judge and remitted the
matter to the learned single Judge with a direction to rehear the matter insofar as
respondent No. 8 is concerned who was transposed as appellant No. 3 and consider the
validity of the decree passed during the absence of respondent No. 8 amongst other
matters on merits. Thereafter, the matter was heard by learned single Judge and by order
dated 7-3-2000, the learned single Judge set aside
@page-SC1338
the judgment and decree of the trial Court and remanded the matter for de novo enquiry
with a direction to permit Smt. A. Annapurna to come on record and to consider her
written statement. In an application for clarification, i.e. C.M.P. No. 22134 of 2000, it
was clarified that there is no need to record the entire evidence afresh, but Smt. A.
Annapurna should be permitted to come on record and file her written statement and
decide the matter insofar as her interest is concerned. After the said clarification, Smt. A.
Annapurna was added as a party and she also filed her written statement. P.W. 1 was
recalled and re-examined. D.W. 1 sought to file an additional affidavit in lieu of chief
examination introducing many documents. The learned trial Judge directed the
defendants to restrict themselves relating to the right of Smt. A. Annapurna over the suit
scheduled property for the purpose of leading evidence and saying so returned the
additional affidavit filed by D.W. 1 with a direction to file a fresh affidavit confining to
the right of the 8th defendant as per the direction of the High Court. The said order dated
24-2-2005 of the Xth Additional Chief Judge was challenged by way of C.R.P. No. 3360
of 2005 before the High Court under Art. 227 of the Constitution of India. The learned
single Judge, in the light of the earlier orders, particularly, order dated 5-7-2001
clarifying earlier order dated 7-3-2000, dismissed the revision in limine and upheld the
order of the trial Judge. Questioning the said order, the 1st defendant has filed the present
appeal after getting leave from this Court.
4. Heard learned counsel for both the parties.
5. The only point for consideration in this appeal is whether the appellant 1st-defendant is
entitled to lead evidence in respect of all issues including additional issues afresh or to be
confined only in respect of 8th defendant who was subsequently impleaded on the orders
of the High Court?
6. Though learned counsel for the appellant strenuously contended that after remand and
after framing additional issues, the appellant is entitled to lead fresh evidence, in view of
clarificatory order dated 5-7-2001 in Civil Misc. Petition No. 22134 of 2000 in C.C.C.A.
No. 94 of 1987, it is open to the parties to lead evidence only in respect of the defence
taken in the written statement of newly impleaded defendant. After allowing Smt. A.
Annapurna to come on record and to file her written statement, it is but proper for the
parties to lead evidence only in respect of the stand taken in the written statement filed by
her. It is worthwhile to refer the clarificatory order dated 5-7-2001 of the learned single
Judge which reads as under :
"It is brought to my notice by Sri B. Ramamohan Reddy, learned counsel that the trial
Court is under the impression that the entire evidence has to be recorded afresh. It is
clarified that the trial Court need not record the entire evidence afresh but permit the said
Annapurna to come on record and file her written statement and decide the matter in so
far as her interests are concerned.
The petition is accordingly disposed of."
It is clear that there is no need to record the evidence afresh in respect of all issues and
the direction was to permit Smt. A. Annapurna to come on record, file her written
statement and decide the matter based on her claim as well as other materials which were
on record. As a matter of fact, after remand and after impleadment of 8th defendant, P.W.
1 confined himself to the case as against the 8th defendant. In view of the same, as rightly
observed by the learned single Judge of the High Court, the 1st defendant cannot be
permitted to lead evidence afresh on other issues. We are satisfied that the learned trial
Judge as well as the learned single Judge of the High Court correctly understood the
earlier orders including the clarificatory order dated 5-7-2001 and rightly issued direction
to the 1st defendant to confine himself to the defence taken in the written statement of the
8th defendant. We reiterate and clarify that the parties are at liberty to lead fresh evidence
only in respect of defence/stand taken by the newly impleaded 8th defendant (Smt. A.
Annapurna) in her written statement.
7. With the above clarification, the appeal is disposed of. No costs.
Order accordingly.
@page-SC1339
AIR 2008 SUPREME COURT 1339 "Oriental Bank of Commerce v. Sunder Lal Jain"
(From : Delhi)*
Coram : 2 G. P. MATHUR AND AFTAB ALAM, JJ.
Civil Appeal No. 82 of 2008 (arising out of S.L.P. (C) No. 882 of 2006), D/- 8 -1 -2008.
Oriental Bank of Commerce v. Sunder Lal Jain and Anr.
(A) Constitution of India, Art.226 - WRITS - COMPROMISE - DECREE - Order by
consent - Statement by counsel for party that his client will consider particular suggestion
given by other side - Would not amount to consent by concerned party - Order passed on
such statement of the counsel - Cannot be said to be order passed on consent.
Civil P.C. (5 of 1908), O.23, R.3. (Para 4)
(B) Constitution of India, Art.226 - WRITS - RESERVE BANK OF INDIA - Mandamus
- Issuance of - Respondents seeking direction to Bank to declare their account as Non-
Performing Asset, NPA and to apply revised guidelines issued by Reserve Bank to their
case - Said guidelines are purely executive instructions and having no statutory force -
Not creating any right in favour of borrowers - Issuance of mandamus by High Court
directing bank to declare respondents account as NPA and to apply said RBI guidelines -
Not proper - Further direction that outstanding shall be recoverable by quarterly
instalments over period of two years - Illegal, being contrary to guidelines.
W.P. (C) No. 559 of 2004, D/-17-8-2005 (Del), Reversed.
Reserve Bank of India Act (2 of 1934), S.45JA. (Paras 6, 9, 10)
(C) Constitution of India, Art.226 - WRITS - DEBT RECOVERY TRIBUNAL -
RECOVERY OF DEBT - Stay of recovery proceedings - Decree for recoverable amount
passed against respondents, borrower by Debt Recovery Tribunal - Attained finality -
Could not be stayed in independent writ petition when respondents had not chosen to
assail decree by filing appeal, a statutory remedy.
Recovery of Debts Due to Banks and Financial Institutions Act (51 of 1993), S.20.
(Para 10)
Cases Referred : Chronological Paras
AIR 1977 SC 2149 (Ref.) 9
AIR 1966 SC 334 (Ref.) 9
AIR 1962 SC 1210 (Ref.) 9
AIR 1973 SC 964 : 1973 Lab IC 1212 (Ref.) 9
B. Sandhya Goswami, for Appellant: Suresh Chandra Tripathy, for Respondents.
* W.P. (C) No. 559 of 2004, D/- 17-8-2005 (Delhi).
Judgement
1. G. P. MATHUR, J. :-Leave granted.
2. This appeal, by special leave, has been preferred against the judgment and order dated
17-8-2005 of Delhi High Court, by which a direction was issued to the appellant Oriental
Bank of Commerce to declare the respondents' account as Non-Performing Asset (NPA)
from 31st March, 2000 and to apply the Reserve Bank of India Guidelines to their case
and communicate the outstandings which shall be recoverable by quarterly instalments
over a period of two years.
3. The respondents Sunder Lal Jain and another were sanctioned credit facility for Rs.20
lakhs on 12-12-1996. The respondent defaulted in repayment of the amount and their
account was declared as NPA on 31-3-2001. On 21-2-2002, the appellant Oriental Bank
of Commerce filed a petition against the respondents being O.A. No.21 of 2002 before
the Debt Recovery Tribunal-III, Delhi (for short the DRT). The DRT passed a decree in
favour of the appellant for recovery of Rs.20,27,862/- along with interest on 14-11-2003.
The appellant initiated execution proceedings for recovery of the amount from the
respondents and a recovery certificate was issued on 8-12-2003. The respondents did not
file any appeal challenging the decree passed by the DRT. Instead, the respondents filed
WP(C) No.559 of 2005 and WP(C) No.560 of 2004 before Delhi High Court praying that
a direction be issued to the appellant, Oriental Bank of Commerce, to declare their
account as NPA from 31-3-2000 and apply RBI Guidelines for reconciliation and settling
the accounts with them. The petitions were disposed of by a short order on 17-8-2005
which reads as under :-
"The petitioners have a remedy available to them of filing an appeal against the recovery
proceedings to the DRT which remedy has not been taken and for which this petition is
liable to be rejected. However, since this petition has been pending for quite some time
before this Court and the first
@page-SC1340
Respondent has agreed to consider declaring the account as NPA from 31st March, 2000,
there is no impediment in disposing of this petition by the following order :
First Respondent-Bank is directed to declare the Petitioners account as NPA from 31st
March, 2000 and apply the RBI guidelines to their case and then to communicate the
outstandings, which shall be recoverable by quarterly instalments over a period of two
years.
In case, this course works out and Petitioners after reconciling their accounts do not
commit any default, the execution proceedings against them pending before the Recovery
Officer shall be stayed and the recovery certificate passed against them shall stand
cancelled. However, in case of default it shall be open to the first Respondent-Bank to
activate the recovery proceedings against them. In that event, it shall be open to the
Petitioners to take any appropriate remedy, which they may have against the recovery
certificate or its execution in law."
Feeling aggrieved by the above noted order, the appellant, Oriental Bank of Commerce,
has preferred this appeal.
4. Learned counsel for the appellant has submitted that the bank had filed a suit for
recovery of the amount and the same had been decreed on 14-11-2003 and thereafter a
recovery certificate had also been issued on 8-12-2003 in the execution proceedings
initiated by the appellant. The respondents did not file any appeal to challenge the decree
passed by the DRT and, therefore, the same attained finality. In these circumstances, the
writ petition filed by the respondents to declare their account as NPA from 31-3-2000 and
to apply the RBI guidelines to their case was not at all maintainable and the order passed
by the High Court is clearly erroneous in law. Learned counsel for the respondents has,
on the other hand, tried to support the order passed by the High Court and has submitted
that the same had been passed on consent of the parties and, therefore, it is not open to
the appellant to challenge the same.
Regarding the submission of learned counsel for the respondents that the order under
challenge has been passed on consent of the parties, it may be noted that the High Court
has recorded that the first respondent has agreed to consider declaring the account as NPA
from 31st March, 2000. Learned counsel for the appellant bank has vehemently submitted
that no consent had been given by the counsel for the bank to declare the account as NPA
from 31st March, 2000, nor any such instructions had been given to the counsel by the
appellant bank. That apart, the order of the High Court mentions "has agreed to consider".
It only means that the bank will examine and consider. "Consider" means to look at
closely and carefully; to think or deliberate on; to take into account. There was thus no
consent on the part of the appellant bank to declare the account as NPA from 31st March,
2000. A statement by a counsel for a party that his client will consider a particular
suggestion given by the other side would not amount to a consent by the concerned party
and an order passed on such a statement of the counsel cannot be said to be an order
passed on consent. It is, therefore, not possible to accept the contention raised by learned
counsel for the respondents that the impugned order of the High Court has been passed
on the consent of the appellant bank and consequently the present appeal is not
maintainable.
5. Before considering the submission made by learned counsel for the parties on merits of
the case, it is necessary to take note of the essential features of the revised guidelines
issued by the Reserve Bank of India on January 29, 2003 regarding Non-Performing
Assets of public sector banks which read as under :-
"Revised guidelines for compromise settlement of chronic Non-Performing Assets
(NPAs) of public sector banks
DBOD.BP.BC.65/21.04.117/2002-2003
January 29, 2003
Chairman and Managing Directors of all Public Sector Banks.
Dear Sir.
Please refer to our circular DBOD.BP. BC.11/21.01.040/99-00 dated 27th July, 2000,
setting out the guidelines for compromise settlements of chronic NPAs up to Rs.5.00
crore.
2. A review of compromise settlements of NPAs through the above scheme has revealed
that the progress of recovery of NPAs through this mechanism has been moderate. In
consultation with Government of India, it has been decided to give one more opportunity
to the borrowers to come forward for settlement of their outstanding
@page-SC1341
dues. Hence fresh guidelines are now issued, which will provide a simplified, non-
discretionary and non-discriminatory mechanism for compromise settlement of chronic
NPAs below the prescribed value ceiling. All public sector banks should uniformly
implement these guidelines, so that maximum realization of dues is achieved from the
stock of NPAs within the stipulated time.
3. .....................Â…Â…Â…Â…Â…Â…Â…Â…Â…Â…Â…Â…Â…Â…Â…Â…Â…
Â….
(A) Guidelines for compromise settlement of chronic NPAs upto Rs.10.00 crore [i]
Coverage
a) The revised guidelines will cover all NPAs in all sectors irrespective of the nature of
business which have become doubtful or loss as on 31st March 2000 with outstanding
balance of Rs. 10.00 crore and below on the cut off date.
b) The guidelines will also cover NPAs classified as sub-standard as on 31st March, 2000,
which have subsequently become doubtful or loss.
c) These guidelines will cover cases on which the banks have initiated action under the
Securitisation and Reconstruction of Financial Assets and Enforcement of Security
Interest Act, 2002 and also cases pending before Courts/DRTs/BIFR, subject to consent
decree being obtained from the Courts/DRTs/BIFR
d) Cases of wilful default, fraud and malfeasance will not be covered.
e) The last date for receipt of applications from borrowers would be as at the close of
business on 30th April, 2003. The processing under the revised guidelines should be
completed by 31st October, 2003.
(ii) Settlement Formula - amount and cut off date
a)NPAs classified as Doubtful or Loss as on 31st March, 2000
The minimum amount that should be recovered under the revised guidelines in respect of
compromise settlement of NPAs classified as doubtful or loss as on 31st March, 2000
would be 100% of the outstanding balance in the account as on the date of transfer to the
protested bills account or the amount outstanding as on the date on which the account
was categorized as doubtful NPAs, whichever happened earlier, as the case may be;
b)NPAs classified as sub-standard as on 31st March, 2000 which became doubtful
or loss subsequently.
The minimum amount that should be recovered in respect of NPAs classified as sub-
standard as on 31st March, 2000 which became doubtful or loss subsequently would be
100% of the outstanding balance in the account as on the date of transfer to the protested
bills account or the amount as on the date on which the account was categorized as
doubtful NPAs, whichever happened earlier, as the case may be, plus interest at existing
Prime Lending Rate from 1st April, 2000 till the date of final payment.
(iii) Payment
The amount of settlement arrived at in both the above cases, should preferably be paid in
one lump sum. In cases where the borrowers are unable to pay the entire amount in one
lump sum, at least 25% of the amount of settlement should be paid upfront and the
balance amount of 75% should be recovered in instalments within a period of one year
together with interest at the existing Prime Lending Rate from the date of settlement up to
the date of final payment.
............................................................"
6. A perusal of the aforesaid revised guidelines issued by the Reserve Bank of India on
January 29, 2003 for compromise settlement of chronic Non-Performing Assets (NPAs)
of public sector banks will show that the same will be applicable and will cover NPAs
classified as sub-standard as on 31st March, 2000 which have subsequently become
doubtful or loss. The revised guidelines have no application where the NPAs have not
been classified as sub-standard as on 31st March, 2000. It is not in dispute that the
account of the respondents was a performing account between 1-4-2000 and 31-3-2001.
According to the records of the bank, the account was consigned to Protest Bill Account
on 15-10-2001 and was declared as NPA as per prudential norms of RBI on 31-3-2001.
The respondents contested the case before the DRT and did not admit their liability. No
such plea was raised that their account had become NPA as on 31-3-2000 before DRT.
Therefore, the revised guidelines issued by Reserve Bank of India on January 29, 2003
for compromise settlement of chronic Non-Performing Assets (NPAs) of public sector
banks were not at all applicable to the facts and circumstances of the case and no
direction could be issued
@page-SC1342
to declare the respondents, account as NPA from 31st March, 2000. The guidelines
further provide that in case where borrowers are unable to pay the entire amount in lump
sum, at least 25% of the amount of settlement should be paid upfront and the balance
amount of 75% should be recovered in instalments within a period of one year together
with interest. The High Court, in the impugned order, has directed that the amount should
be recovered by the appellant bank in quarterly instalments over a period of two years.
This is again contrary to the revised guidelines, which provide a period of one year only
for recovery of the entire amount.
7. It is important to note that the revised guidelines issued by the Reserve Bank of India
on January 29, 2003 are only in the nature of internal guidelines for the banks and
financial institutions. They are purely executive instructions and have no statutory force.
They do not create any right in favour of the borrowers. In order to avail relief under the
guidelines, the eligibility criteria must be strictly fulfilled and one of them is that the
account must be an NPA as on 31st March, 2000. What the respondents want is that a writ
of mandamus be issued commanding the appellant bank to declare the respondents,
account as NPA from 31st March, 2000 and apply the RBI Guidelines to their case
whereby their liability towards the appellant bank will be considerably reduced by way of
one time settlement.
8. The principles on which a writ of mandamus can be issued have been stated as under in
The Law of Extraordinary Legal Remedies by F.G. Ferris and F.G. Ferris, Jr. :
Note 187- Mandamus, at common law, is a highly prerogative writ, usually issuing out of
the highest court of general Jurisdiction, in the name of the sovereignty, directed to any
natural person, corporation or inferior court within the jurisdiction, requiring them to do
some particular thing therein specified, and which appertains to their office or duty.
Generally speaking, it may be said that mandamus is a summary writ, issuing from the
proper court, commanding the official or board to which it is addressed to perform some
specific legal duty to which the party applying for the writ is entitled of legal right to
have performed.
Note 192 -Mandamus is, subject to the exercise of a sound judicial discretion, the
appropriate remedy to enforce a plain, positive, specific and ministerial duty presently
existing and imposed by law upon officers and others who refuse or neglect to perform
such duty, when there is no other adequate and specific legal remedy and without which
there would be a failure of justice. The chief function of the writ is to compel the
performance of public duties prescribed by statute, and to keep subordinate and inferior
bodies and Tribunals exercising public functions within their jurisdictions. It is not
necessary, however, that the duty be imposed by statute; mandamus lies as well for the
enforcement of a common law duty.
Note 196- Mandamus is not a writ of right. Its issuance unquestionably lies in the sound
judicial discretion of the Court, subject always to the well-settled principles which have
been established by the Courts. An action in mandamus is not governed by the principles
of ordinary litigation where the matters alleged on one side and not denied on the other
are taken as true, and Judgment pronounced thereon as of course. While mandamus is
classed as a legal remedy, its issuance is largely controlled by equitable principles. Before
granting the writ the Court may, and should, look to the larger public interest which may
be concerned - an interest which private litigants are apt to over-look when striving for
private ends. The Court should act in view of all the existing facts, and with due regard to
the consequences which will result. It is in every case a discretion dependent upon all the
surrounding facts and circumstances.
Note 206 - .......The correct rule is that mandamus will not lie where the duty is clearly
discretionary and the party upon whom the duty rests has exercised his discretion
reasonably and within his jurisdiction, that is, upon facts sufficient to support his action.
9. These very principles have been adopted in our country. In Bihar Eastern Gangetic
Fishermen Cooperative Society Ltd. v. Sipahi Singh and others, AIR 1977 SC 2149, after
referring to the earlier decisions in Lekhraj Satramdas Lalvani v. Deputy Custodian-cum-
Managing Officer, AIR 1966 SC 334; Dr. Rai Shivendra Bahadur v. The Governing Body
of the Nalanda College, AIR 1962 SC 1210 and Dr. Umakant Saran v. State of Bihar, AIR
1973 SC 964, this Court observed as follows in paragraph 15 of the reports :
@page-SC1343
"...... There is abundant authority in favour of the proposition that a writ of mandamus
can be granted only in a case where there is a statutory duty imposed upon the officer
concerned and there is a failure on the part of the officer to discharge the statutory
obligation. The chief function of a writ is to compel performance of public duties
prescribed by statute and to keep subordinate Tribunals and officers exercising public
functions within the limit of their jurisdiction. It follows, therefore, that in order that
mandamus may issue to compel the authorities to do something, it must be shown that
there is a statute which imposes a legal duty and the aggrieved party has a legal right
under the statute to enforce its performance..........In the instant case, it has not been
shown by respondent No. 1 that there is any statute or rule having the force of law which
casts a duty on respondents 2 to 4 which they failed to perform. All that is sought to be
enforced is an obligation flowing from a contract which, as already indicated, is also not
binding and enforceable. Accordingly, we are clearly of the opinion that respondent No. 1
was not entitled to apply for grant of a writ of mandamus under Article 226 of the
Constitution and the High Court was not competent to issue the same."
Therefore, in order that a writ of mandamus may be issued, there must be a legal right
with the parry asking for the writ to compel the performance of some statutory duty cast
upon the authorities. The respondents have not been able to show that there is any statute
or rule having the force of law which casts a duty on the appellant bank to declare their
account as NPA from 31st March, 2000 and apply R.B.I. guidelines to their case.
10. The High Court, therefore, erred in issuing a writ of mandamus directing the appellant
bank to declare the respondents' account as NPA from 31st March, 2000 and to apply the
RBI Guidelines to their case and communicate the outstandings which shall be
recoverable by quarterly instalments over a period of two years.The later part of the order
passed by the High Court wherein a direction has been issued to stay the recovery
proceedings and the recovery certificate issued against the respondents has been
cancelled is also wholly illegal as the decree passed by the DRT had attained finality and
proceedings for execution of decree could not be stayed in an independent writ petition
when the respondents had not chosen to assail the decree by filing an appeal, which is a
statutory remedy provided under Section 20 of Recovery of Debts Due to Banks and
Financial Institutions Act, 1993.
11. In view of the discussions made above, the appeal is allowed and the impugned
judgment and order dated 17-8-2005 passed by the High Court is set aside. The appellant
will be entitled to its costs.
Appeal allowed.
AIR 2008 SUPREME COURT 1343 "Delhi Development Authority, N. D. v. Joint Action
Committee, Allottee of SFS Flats"
(From : Delhi)
Coram : 2 S. B. SINHA AND H. S. BEDI, JJ.
Civil Appeal Nos. 6668-6698 and 6700-6732 of 2000 with C. A. Nos. 6666, 6667 and
6733 of 2000 And C. A. Nos. 5881, 5869, 5882, 5880 and 5872 of 2007, (SLP (C) Nos.
25385 of 2005, 1003,8033, 8262 and 13512 of 2006), D/- 13 -12 -2007.
Delhi Development Authority, N.D. and Anr. v. Joint Action Committee, Allottee of SFS
Flats and Ors.
(A) Evidence Act (1 of 1872), S.115 - ESTOPPEL - DEVELOPMENT AUTHORITY -
SURCHARGE - RESTORATION - Estoppel - Homing scheme by Development
Authority - Allotment letter stipulating norms for fixing price, payment schedule of
installments and interest chargeable for default - Office order issued subsequently to levy
20% surcharge for restoration - Authority despite office order condoning default of
allottee as per terms of allotment letter and restoring his allotment - Conduct of
Development Authority estoppes it from raising a plea as regards application of office
order. (Paras 55, 56)
(B) Contract Act (9 of 1872), S.60 - CONTRACT - Novation of contract - Cannot be
made by unilateral act.
It is well known principle of law that a person would be bound by the terms of the
contract subject of course to its validity. A contract in certain situations may also be
avoided. With a view to make novation of a contract binding and in particular some of the
terms and conditions thereof, the offeree must be made known thereabout. A party to the
contract cannot at a later stage, while the contract was being performed, impose terms
and conditions which were not part of the offer and which were based upon unilateral
issuance of office orders, but not communicated
@page-SC1344
to the other party to the contract and which were not even the subject matter of a public
notice. (Para 57)
Terms and conditions of the contract can indisputably be altered or modified. They
cannot, however, be done unilaterally unless there exists any provision either in contract
itself or in law. Novation of contract must precede the contract making process. The
parties thereto must be ad idem so far as the terms and conditions are concerned. If a
contracting party, intended to alter or modify the terms of contract, it was obligatory on
its part to bring the same to the notice of the allocatee. Having not done so, it, relying on
or on the basis of the purported office orders which is not backed by any statute, new
terms of contract could not be thrust upon the other party to the contract. (Para 61)
(C) Constitution of India, Art.32, Art.226 - WRITS - JUDICIAL REVIEW - POLICY
DECISION - Judicial review - Policy decisions - Not beyond pale of judicial review -
Grounds available for review enumerated. (Para 59)
(D) Constitution of India, Art.73 - POLICY DECISION - CONTRACT - Policy decision
- Decision to change/alter terms of contract - Is in realm of contract - Not a policy
decision. (Para 61)
(E) Constitution of India, Art.32, Art.226 - WRITS - JUDICIAL REVIEW - Judicial
review - Price fixation - Superior Courts ordinarily would not interfere - But there does
not exist any absolute ban. (Para 64)
(F) Delhi Development Act (61 of 1957), S.57 - Delhi Development Authority
(Management and Disposal of Housing Estates) Regulations (1968), Regn.6, Regn.5 -
DEVELOPMENT AUTHORITY - ALLOTMENT OF PREMISES - EQUALITY -
WRITS - Housing scheme - Allotment - Contract entered into with allottees - Emanates
from a statute or is otherwise governed by it - Such contracts can be judicially reviewed
not only on touchstone of Art.14 but also on touchstone of source of power under statute.
Constitution of India, Art.226. (Para 62)
(G) Delhi Development Act (61 of 1957), S.57 - Delhi Development Authority
(Management and Disposal of Housing Estates) Regulations (1968), Regn.6 -
DEVELOPMENT AUTHORITY - PLANNING AND DEVELOPMENT - HOUSING
BOARD - Housing scheme - Cost of construction - Fixation - Cannot be done dehors
factors mentioned in tender notice - Not so even in exercise of residuary power.
When fixation of cost of accommodation is done having regard to the relevant factors on
the basis of which brochure as well as the notice inviting tender was issued, the superior
Courts may not interfere; but the same must be done in terms of the original contract and
not dehors the same. The authority, even while exercising its residuary power, is required
to act within the Four Corners of the contract. While doing so, the terms of the contract
cannot be altered to include any other factors, which were not contemplated thereunder.
While computing the extra cost, no additional factors, thus, can be taken into
consideration. If such a power were conceded in the authority, the same would give rise
to exercise of arbitrary power. It is not contemplated in law. (Para 66)
(H) Delhi Development Act (61 of 1957), S.57 - Delhi Development Authority
(Management and Disposal of Housing Estates) Regulations (1968), Regn.6 -
DEVELOPMENT AUTHORITY - PLANNING AND DEVELOPMENT - Self financing
housing scheme - Cost of construction - Fixation - Conversion charges - Though could be
levied by DDA - Cannot go in computation of construction cost - Levy by DDA of
additional amount of 20% and surcharge of 20% over ordinary cost of construction by
issuing office order - Improper.
C. W. P. Nos. 793, 1118, 2209 and etc. of 1995, D/-23-07-1999 and L. P. A. No. 844 of
2003. D/-22-7-2005 (KB) (Del). Reversed.
The directions issued by the Central Govt. to levy Conversion charges is binding on DDA
for converting the leasehold into freehold. However, such a power also must be exercised
reasonably and fairly. Conversion of the property from leasehold to freehold is a separate
transaction. The same has nothing to do with the actions, qua contract. Imposition of
conversion charges, therefore, even if, per se, may not be held to be bad, the said factor
cannot be taken into consideration for the purpose of computing construction costs. After
1996, the ordinary cost of construction of a flat was Rs. 2,00,000/-in South Delhi but not
only it formed the basis for computing the final cost but also 20% additional amount as
also 20%
@page-SC1345
charge were claimed thereupon. Sometimes interest also was charged as and when
applicable. Thus, so long the conversion charge is charged by way of a separate
transaction, no exception can be taken. But, purported price fixation as has been done in
the instant case cannot be approved. (Para 68)
An executive officer, in absence of any provision of a statute, cannot apply his own
decision with a retrospective effect. A delegatee is bound to act within the four corners of
the delegation and not beyond the same. Current cost has been calculated upon
computing 20% over and above the actual cost. A provision for surcharge had also been
made in terms whereof a premium of 2% over the disposal cost was worked out on
current cost for the SFS flats in South Delhi. Imposition of surcharge is subject to the
condition that the real value in the market of DDA flats would be much more than it had
been charging as per the cost formula. Parameters of computation of disposal price have
been laid down. The authority having itself adopted a formula for computing the disposal
cost, the same was binding upon the delegatees. A delegatee cannot take any action
contrary to or inconsistent with the factors laid down for computation of disposal cost as
defined in S. 2 (30) of the Act. Regulations 5 and 6 do not authorize the delegatee to
apply a formula which was not contemplated by the Authority itself. If an Executive
Authority in absence of any statutory provision cannot apply a decision with retrospective
effect, the same would be ultra vires.
C. W. P. Nos. 793, 1118, 2209 and etc. etc. of 1995, D/-23-7-1999 and L. P. A. No. 844 of
2003, D/-22-7-2005 (FB) (Del), Reversed.
(I) Contract Act (9 of 1872), S.29 - CONTRACT - Vagueness - Definite price - Essential
element for binding contract.
It is well settled that a definite price is an essential element of a binding agreement.
Although a definite price need not be stated in the contract, but assertion thereof either
expressly or impliedly is imperative. (Para 78)
Cases Referred : Chronological Paras
2007 AIR SCW 2111 : AIR 2007 SC (Supp 182 (Pt H) (Rel. on) 75
2007 AIR SCW 4387 : AIR 2007 SC 2458 35
2005 AIR SCW 2676 : AIR 2005 SC 2821 (Ref) 75
(2003) LPA No. 727 of 2002, D/-15-12-2003 (Del) 25, 80, 84
2002 AIR SCW 3436 : AIR 2002 SC 2940 (Ref) 24, 26
AIR 2001 Del 39 86
1999 AIHC 3748 (Del) 86
AIR 1995 Del 212 (FB) 22, 24, 26
1994 AIR SCW 3985 : AIR 1995 SC 1 (Ref) 22, 26
AIR 1983 SC 848 (Pt. D) (Rel. on) 63
AIR 1980 SC 738 (Ref) 22, 25, 26, 80, 82, 84
Sunil Gupta, V. Shekar, Sr. Advocates, Gopal Shankaranarayanana, Ms. Indu Malhotra,
S.K. Rungta, A.P. Dhamija, Ms. Madhu Sudan Bhayana, Mrs. Pratibha Jain, C.S. Ashri,
S. Ganesh, Pradeep K. Dubey, Sudhir K. Sajwan, Zangpo Sherpa, Abhigya, Ashok Kumar
Sharma, Ms. Madhu Moolchandani, Maninder Singh, Ms. Pratibha M. Singh, Ms. Surbhi
Mehta, Gaurav Sharma, Sumeet Bhatia, Debasis Misra, Kuldip Singh, Ms. Anysuya
Selvan, Ms. S. Janani, Rajeev Sharma, Sarad Kumar Singhania, E.C. Agrawala, Mahesh
Agarwal, Rishi Agrawala, Gaurav Goel, Amit Sharma, Ms. Neha Aggarwal, T.L. Garg,
J.S. Attri, Surya Kant, Subramanyam Prasad, Kamlendra Mishra, Ms. Kum Kum Sen, for
appearing parties. Jayabrata Bhattacharjee, Respondent in person.
Judgement
S. B. SINHA, J.:-Leave granted in all the Special Leave Petitions.
CIVIL APPEAL Nos. 6666, 6667, 6668-6698, 6799-6732 and 6733 OF 2000 CIVIL
APPEALS @ SLP (CIVIL) Nos. 25385 of 2005, 1003, 8033 of 2006 and 13512 of 2006 :
2. This batch of appeals arising out of a judgment and order dated 23-7-1999 passed by a
Division Bench of the Delhi High Court, inter alia, in Writ Petitions No. 793 of 1993 as
also a judgment and order dated 22-7-2005 passed by a Full Bench of the Delhi High
Court in Letters Patent Appeal Nos. 844 of 2003 etc. were heard together and are being
disposed of by this common judgment.
FACTS :
3. Delhi Development Authority (for short, 'the Authority') has been constituted under the
Delhi Development Act, 1957 (for short, 'the Act'). Indisputably, it develops different
areas in the town of Delhi and constructs houses for all groups of people.
4. Principally it allocates flats under six different schemes viz : (i) Self Financing
@page-SC1346
Scheme (SFS); (ii) Higher Income Group Scheme (HIG Scheme); (iii) Middle Income
Group Scheme (MIG Scheme); (iv) Lower Income Group Scheme (LIG Scheme), (v)
Janata Scheme; and (vi) Expandable Housing Scheme.
5. The flats constructed and allocated under the SFS Scheme are distinct and different
from the other five schemes launched by the Authority. We shall advert to the said
distinction a little later.
6. Suffice, however, it to say that not only costs of such schemes are calculated on
different basis but the rights and stipulated liabilities thereunder are also different. Cost of
flats vary from scheme to scheme. Under one of the schemes, applications were invited
by the Authority from 22-12-1992 to 11-1-1993.
7. We may notice some of the provisions containing the terms and conditions on the basis
whereof such an offer was made.
"5.10 The details of the flats and tentative cost etc. are available in Annexure-B. The cost
of the flats mentioned there is tentative and subject to revision on account of escalation in
the value of land and cost of construction. Please note that there is a possibility of upward
revision of the tentative cost.
5.11 Those who are successful for a ready built flat will be called upon to make the
payment in lump sum within 60 days. Others who are successful for a flat where the work
is already in progress will be asked to deposit within 30 days a specified percentage
anything upto 90% of the estimated disposal price representing the expenditure already
made and the amount required for construction of flats in next 3 to 4 months. Applicants
successful for new allocations are asked to pay 25% the estimated cost of the flat by way
of 1st instalment payable within 30 days. In each of the case 60 days time is further given
to remit the amount with prescribed interest.
11.2. The demand-cum-allocation letters issued will indicate the prescribed dates by
which the payments will be required to be made. The demand letter for final instalment
for the flats in progress and new allocations will be issued separately and this may also
include the possible increase in the cost of the flat. No separate letters will be issued for
any of the subsequent instalments. It will be obligatory on the part of the allocatees to
make the payments and deposit the site documents before the due dates indicated. In the
event of default the allocation/ allotment of the flat in the scheme will be liable to be
cancelled. If submission of documents as demanded are delayed, maintenance charges
will be leviable provided the delay in submission of documents is regularized.
12.2. If the allowtment of flat is cancelled (either on the allottee's own request or due to
the non fulfilment of the terms and conditions of allocation by the allottee) after the
expiry of 1, 2, 3 and 4 months from the date of issue of demand-cum-allocation letter,
interest calculated @ 12% p. a. for the 1st month and 18% p.a. for the 2nd, 3rd and 4th
month on the amount demanded in the demand letter shall be charged in addition to the
amount of penalty specified above.
14.1. The allottee shall be entitled to take delivery of the possession only after he has
completed all the formalities, paid all dues and furnished/executed all the documents as
required in the allotment-cum-demand letter of the Delhi Development Authority."
8. Appellants in the first batch of cases and the respondents in the second batch
(hereinafter referred to as 'the registrants') applied for allocation of flats under the SFS
Scheme. The said scheme was floated in terms of Item No. 112 of 1992. For the said
purpose, brochures are issued. Those who desired to have allocation of such flats were
asked to opt therefor at three different places. Allocation of flats under the said Scheme is
made upon 90% payment of the estimated costs. However, allotment is made on draw of
specific number of flats. Allocation of flats may be made in respect of areas, floors and/or
the pockets. On receipt of the letter of demand-cum-allocation by the registrant, the
schedule of payment commences. Estimated cost for construction is calculated on the
basis of the value of the land and likely cost of constructions.
9. The letter of demand-cum-allocation issued to successful registrants contains a
condition which may be noticed for the purpose of these cases and read as under :
"4. The amount demanded should be paid on or before the due date mentioned in paras 2
and 3 above. Extension of time for making payment of the amount demanded in column 7
of para 3 above up to a maximum period of 90 days from the due date is admissible.
@page-SC1347
An allottee need not apply for extension but he will have to pay interest @ 12% p.a. for
the first month and @ 18% p.a. for the subsequent period. In case payment of the amount
asked for in the demand letter is not made within 90 days of the due date, the allotment
shall stand cancelled automatically. However, cancellation due to non-payment of first 4
instalments during the stipulated period can be got restored on payment of dues with
interest along with cancellation and restoration charges for each cancellation due to non-
payment, subject to availability of the allocated flat. The cancellation due to non-
payment of final instalment within 120 days of the date of issue (letter of the block date)
of the demand letter for 5th instalment shall not be restored under any circumstances."
10. It may be useful to notice some other clauses containing terms and conditions of
allocation, which are as under :
"1. No separate demand letters will be issued for 2nd, 3rd and 4th instalments. It will be
obligatory on your part to make the payment before the due date indicated at page-1;
failing which the allocation is liable to be cancelled.
2. The estimated cost of the flats as given in this letter is provisional and subject to
revision on the completion of the flat. Any price difference between the estimated cost
and the cost as it comes out on completion as per costing formula then in vogue would
have to be paid along with the 5th and final instalment. There would be no review of the
cost of the flat in the intervening period. Interest @ 7% on the amount deposited will be
payable for the period beyond 3½ years to the date of issue of possession letter if the
construction of the houses is not completed by then.
3. The amount demanded should be paid on or before the stipulated due date failing
which the allotment shall be liable to be cancelled without notice. In case, due to
unavoidable reasons, the allottee is not able to make the payment within the due time,
then he must ensure that his acceptance of the allotment reaches the Housing Department
before the due date of payment with a request for extension."
11. Registrants are said to have defaulted resulting in purported automatic cancellation of
their allotments. Show cause notices were issued to them. Some of them allegedly
expressed their difficulties in regard thereto.
Decisions impugned
12. The Vice-Chairman of the appellant who is said to be delegated with the power of the
Authority to which reference would be made hereinafter took a policy decision which is
reflected from Office Order issued on 16-8-1996, the relevant clauses whereof are :
"2. With the approval of L.G. a decision was taken that the current price for South Delhi
flats will be worked out by adding a surcharge of 20% from the price worked out as per
old formula. The approval of L. G. to this decision was granted on 12-7-1996.
3. There are presently cases in the Housing Department where there have been delays in
the making of the payments of the flats allocated/allotted in South Delhi under SFS.
Before the aforesaid revision took place, delays of one year or so were being regularised
with usual charges, i.e. on payment of 18% interest per annum and restoration charges,
etc. in few cases where delays are unusually long, current price has also been demanded."
13. The aforementioned Office Order dated 16-8-1996 was reiterated in a resolution dated
27-8-1996 wherein a further decision was taken to impose a surcharge of 20% "over and
above" disposal price only in respect of registrants who had been allotted flats in South
Delhi. The said decision was taken to balance the reduced cash flow of the Authority. It
reads as under :
"1. On the basis of the aforesaid resolutions of the Authority, 50% flats are proposed to be
offered to the public. It is also being proposed to offer to public the unavailed flats if any
out of the 50% flats being reserved for Govt. organisations / PSUs. In this manner,
number of flats to be offered to the public can be beyond 50%.
xxxx xxxx xxxx xxxx xxxx xxxx
4. To balance the reduced cash in flow because of the proposed discount it will be
necessary to charge premium in the areas where the real value in the market of DDA flats
is much more than what DDA is charging as per its costing formula in the demand letters.
It would be in the fitness of thing to charge premium of 20% over the disposal cost
worked out for the flats in South Delhi SFS."
@page-SC1348
14. Yet again, on 5-11-1998, a purported clarification was issued in regard to
regularization of flats where there had been delay in payment of first four instalments,
stating :
"While issuing allocation/allotment letters to the registrants of various schemes
announced by DDA, a demand is raised from the concerned allottees specifying the
amounts to be paid with due date of payment. However, sometimes on account of the
problem faced by the concerned allottees the payment received by the DDA are later than
the scheduled date. Such cases are usually examined on merits and the delay is
regularised if there is merit in the case as non-regularisation of delays in deserving cases
may be resulted by the allottees. The following shall be the rules applicable to the
allottees of all category of flats in case there payments are delayed and are regularised by
the competent authority.
A. Competent Authority to regularise the delay :

Period of delay
Designation
(i) Upto 3 days Jt. Dy. Directors
(ii) Beyond 3 days but upto 90 days Director (Housing)
(iii) Beyond 90 days but upto 1 year Commissioner (Housing)
(iv) Beyond 1 year but upto 1 year 6 months Principal Commissioner
(v) More than 1 year 6 months Vice-Chairman

B. Price of the Flat :


(i) If the allocated/allotted flat is in South Delhi where the construction has been
undertaken by the South East Zone and South West Zone of the Engineering Wing Except
Dwarka (being in West Delhi) the Price of the flat if restored, would be "Old Cost"
interest or current cost whichever is higher.
(ii) In case where allottees of the localities mentioned (i) above default a small percentage
of demand amount upto 10% beyond the due date, this delay, if regularised would be on
"Old Cost-interest"
(iii) In other cases of all category flats i.e. where the construction of flats has been
undertaken by other zones of Engineering Wing the restoration shall be at "Old Cost-
interest".
2. A decision exists that while working out the current cost for flats in South Delhi, a
surcharge of 20% from the price worked out as per old formula, will be added. This
surcharge will continue to be added for South Delhi flats. The interest rates in the above
case shall be @ 18% per annum on the default amount.
C. RESTORATION CHARGES : In addition to the above, the allottees/ allocatees whose
allotment is restored by the competent authority, shall be liable to pay Restoration
Charges @ 2.5% of the registration money of the respective scheme."
15. Another office order dated 31-3-1999 was issued in regard to imposition of 20%
surcharge over and above disposal price only in respect of registrants in South Delhi,
relevant portion whereof reads as under :
"2. Price of the flat
(i) In cases (pertaining to any locality) where demanded amounts were received prior to
22-8-1996 by DDA, the restoration of allotment/regularisation of delay, if considered
would be on "old cost interest".
(ii) In case where allottees default a small percentage of total demanded amount upto
10% beyond the due date, the delay if regularised, would be on "old cost + interest."
(iii) If the allocated/allotted flat is in South Delhi i.e. where the construction has been
undertaken by the South East Zone and South West Zone of the Engineering Wing except
Dwarka (being in West Delhi), the price of the flat, if restored, would be "old cost +
interest or current cost", whichever is higher. This clause will be applicable in cases for
which demand amount by DDA is received after 22-8-1996 and the delay is regularised.
(iv) In other cases of all category flats i.e. where the construction of flats has been
undertaken by other zones of Engineering Wing, the restoration shall be at "old cost +
interest".
3. Sur Charge
The premium of 20% over the disposal cost worked out on current cost or old cost for the
SFS flats in South Delhi, where the real value in the market of DDA flats is much more
than DDA is charging as per its costing formula, shall be charged."
16. The said orders were issued in purported supersession of the previous orders on the
subject and were to come into force
@page-SC1349
with immediate effect. Some of the registrants made payments pursuant thereto or in
furtherance thereof. Some had made payments although, according to them, afore
mentioned resolution will have no application to their case.
17. The said resolution was given a retrospective effect and retroactive operation.
Proceedings :
18. Several writ petitions were filed at that stage questioning the legality and/or validity
of the said purported resolutions.
19. A learned single Judge of the Delhi High Court allowed the said writ petitions in part
quashing the policy of charging current cost and upholding the policy of charging 20%
surcharge. Letters Patent Appeals were preferred there against. A Division Bench of the
High Court, having regard to conflict in decisions operating i the field referred the matter
to a larger Bench.
20. The Full Bench of the said Court by reason of the impugned judgment modified the
judgment and order of the learned single Judge in respect of 'current cost' holding that the
Authority had the requisite jurisdiction also in respect thereof. The validity of levy of
20% surcharge was also upheld.
Contention before the Full Bench :
21. Before the Full Bench, the registrants, inter alia, raised a contention that levy of an
additional amount over and above the disposal price on the allocatees of flats in South
Delhi was wholly unjustified. It was also urged that adoption of current cost formula
being contrary to the regulations was also not sustainable in law inasmuch as rights of the
writ petitioners crystallized on issuance of the allocation letter and not when the actual
allotment of flat took place. Levy of surcharge amounts to a levy of tax or cess, wherefor
there is no authority in law.
22

. Relying on or on the basis of a decision of this Court in Premji Bhai Parmar and Others
v. Delhi Development Authority (AIR 1980 SC 738) as also on Delhi Development
Authority v. Pushpendra Kumar Jain (AIR 1995 SC 1), the respondents, on the other
hand, contended that the right of registrants gets crystallized only upon final allotment
and not at the stage of issuance of allocation letter. Relying furthermore upon a Full
Bench decision of the High Court in Smt. Sheelawant v. Delhi Development Authority
(1995 (1) AD (Delhi) 725), its jurisdiction to delve into the price fixation policy was also
questioned. 1994 AIR SCW 3985
AIR 1995 Del 212

Issues raised before the Full Bench :


23. Two principal issues which were raised before the High Court are :
(i) Whether the action of the Development Authority in levying 20% surcharge from the
registrants of the South Delhi is justified?
(ii) Whether demand for payment of current cost as calculated by the Delhi Development
Authority from the defaulter registrants could be said to be justified?"
Findings of the Full Bench :
24. (i) Levy of 20% surcharge is within the competence of the Authority in view of the
definition of the 'disposal price' as contained in Regulation 2(13) of the Delhi
Development Authority (Management and Disposal of Housing Estates) Regulations,
1968 (for short, 'the Regulations)', in respect whereof a decision was taken by the Vice
Chairman on 22-8-1996.
(ii) The said Regulations governing the field exclusively permits the Authority to decide
and fix the price which would include surcharge being in the realm of contract, the
relationship between the parties was clearly contractual; surcharge being a component of
the price of the flat.
(iii) It is also to be noted that most of the Appellants were defaulter who had defaulted in
making payments of the first four instalments on time and therefore there was delay in
making payments in their cases. Since there was an automatic cancellation clause in the
agreement the original contracts stood cancelled.
(iv) Regulations 2 (13) of the DDA Regulations defined disposal price or hire purchase
price in relation to a property to mean such price as may be fixed by the authority. Hence
the levy of surcharge was within the competence of the authority.
(v) DDA could include surcharge in its pricing formula which would be in the realm of
contract as the relationship was purely contractual.

(vi) The meaning of the word "cost and price" has been settled by the Full Bench of
AIR 1995 Del 212
2002 AIR SCW 3436

@page-SC1350
the Delhi High Court in Sheelawant v. DDA which was upheld by the Supreme Court and
reaffirmed in DDA v. Ashok Kumar Behl ((2002) 7 SCC 135).
(vii) The present case was different from the case of P.N. Verma v. Union of India (AIR
1985 Delhi 417).
Submissions of the learned counsel on behalf of the registrants :
25. Mr. Rungta, Mr. Maninder Singh and Mr. Shekhar, learned counsel, submitted :
(i) Having regard to the admitted fact that all allocations were made during the period
1991 and 1994 and all of them having paid the instalments except the fourth one, prior to
taking of the purported policy decision dated 22-8-1996, the Authority had no jurisdiction
either to recalculate the current cost or impose a levy of 20% surcharge.
(ii) As the impugned 'Levy' comprises of three elements, namely, (i) the current cost
which is determined; where over (ii) 20% over the actual cost is taken into consideration,
and again (iii) 20% surcharge is required to be paid, the same is unreasonable.
(iii) The Full Bench wrongly applied P.N. Verma (supra), R.K. Sacher (supra) and Premji
Bhai Parmar (supra), which were decided on wholly different set of facts as the scheme(s)
involved therein was for low income group of people in terms whereof payments were to
be made on completion of construction; whereas in the case of SFS, instalments of
payment is sought for immediately after the allocation.
(iv) Restoration of allotment in the case of defaulter having been the subject matter of
contract, the terms and conditions thereof could not have been altered or modified by the
Authority unilaterally.
(v) In any event, the impugned policy decision cannot be given retrospective effect or
retrospective operation.
(vi) Classification for value of flats being income-wise, area-wise, time-wise and scheme-
wise as has been laid down in Premji Bhai Parmar (supra), any micro classification
introduced by the impugned policy decision insofar as the registrants of South Delhi
alone had been asked to pay a higher amount must be held to be discriminatory in nature.
(vii) The purported cut-off date being 22-8-1996 creates a class between those whose
allotments, although cancelled, had been restored on the terms of the contract and those
who had applied for restoration thereafter.
(viii) As policy did not prescribe any rational basis and the same having wrongly been
applied, the same is wholly without jurisdiction and, thus, a nullity.
(ix) Clause 5.10 of the brochure has wrongly been applied by the Full Bench insofar as
value of the land and cost of construction were also existing in P.N. Verma (supra), R.K.
Sacher (supra) and Premji Bhai Parmar (supra).
(x) The policy decision having been taken on the basis of 'lack of cash flow', the stand
taken by the Authority in its counter affidavit, that a large sum of money was invested for
rehabilitation of the migrants from Jammu and Kashmir and Punjab is wholly untenable.
(xi) As the current cost of the flat was required to be determined in terms of the definition
contained in Regulation 2(13) of the Regulations, any other mode or method adopted in
respect thereof was wholly illegal.
(xii) Had such value been required to be arrived at having regard to the nature and time of
the project when it was launched, the impugned policy decision is barred under the
principle of promissory estoppel.
(xiii) By reason of an Executive Order, levy cannot be imposed with retrospective effect.
(xiv) The Full Bench of the High Court misconstrued and misinterpreted the decision of
the Division Bench of the Delhi High Court in P.N. Verma (supra), which covered the
case of registrants.
26. Mr. Arun Jaitley and Mr. Sunil Gupta, learned senior counsel, appearing on behalf of
the respondents, on the other hand, submitted :
(i) The relationship between the parties being contractual, the writ petitions were not
maintainable.
(ii) Price fixation of flats being within the exclusive domain of the Authority, the High
Court has rightly refused to interfere therewith.
(iii) The findings arrived at in P.N. Verma (supra) falling in the line of Pushpendra Kumar
Jain (supra), on the one hand, and
@page-SC1351
Premji Bhai Parmar (supra), on the other, are strictly not applicable in this case as the
policy decision adopted by the Authority on 22-8-1996 was a new one in terms whereof
having regard to the equitable principle in mind, the Authority adopted a policy dehors
clause (4) of the letter of allotment.

(iv) This Court not only in Premji Bhai Parmar (supra) but also subsequently having
upheld the decisions of the Delhi High Court in P.N. Verma (Supra) and Sheelawanti
(supra) and in D.D.A. v. Ashok Kumar Bhel ((2002 (7) SCC 135), the same constituted a
binding precedent. 2002 AIR SCW 3436

(v) Sections 5 and 6 of the Act read with Regulation 5 having authorised the Authority to
fix the price of the flats, the validity of impugned restoration policy comprising payment
of surcharge of 20% and the interest on the said rate or current cost whichever is higher,
could not have been questioned in writ proceedings.
(vi) As the policy was formulated on the representations made by the registrants, the
registrants are estopped and precluded from questioning the validity of the same.
(vii) In a case of this nature where interpretation of clause of contract is involved, judicial
review is not permissible.
(viii) Levy of surcharge for the subsidised schemes for weaker sections of the society
having, inter alia, been upheld in Premji Bhai Parmar (supra), the impugned policy
decision must be held to be reasonable so as to satisfy the test of Article 14 of the
Constitution of India.
(ix) Cost of the land even in South Delhi having been worked out on the valuation of the
land situated in Dwarka, classification made in respect of the allocation at South Delhi
where market rates were much higher from the actual cost, the scheme for restoration of
flats in that area cannot be held to be arbitrary or discriminatory.
Statutory provisions :
27. The Act was enacted to provide for the development of Delhi according to plan and
for matters connected therewith or ancillary thereto.
Interpretation clause is contained in Section 2 of the Act, Section 2(h) defines the term
'Regulation' to mean a Regulation made under this Act.
Section 3 provides for constitution of the Authority consisting of a Chairman, Who shall
be the Lieutenant Governor of the National Capital Territory of Delhi, ex-officio, a Vice-
Chairman to be appointed by the Central Government, amongst others. Section 4
provides for staff of the Authority. Section 5-A provides for constitution of Committee in
the following terms :
"5-A. Constitution of Committee.- (1) The Authority may constitute as many Committees
consisting wholly of members or wholly of other persons or partly of members and partly
of other persons and for such purpose or purposes as it may think fit.
(2) A Committee constituted under this Section shall meet at such time and place and
shall observe such Rules of procedure in regard to the transaction of business at its
meetings as may be determined by Regulations made in this behalf.
(3) The members of a Committee (other than the members of the Authority) shall be paid
such fees and allowances for attending its meetings and for attending to any other work
of the Authority, as may be determined by Regulations made in this behalf."
Section 6 provides for objects of the Authority, in the following terms :
"6. Objects of the Authority.- The objects of the Authority shall be to promote and secure
the development of Delhi according to plan and for that purpose the Authority shall have
the power to acquire hold, manage and dispose of land and other property, to carry out
building, engineering, and manage and dispose of land and property, to execute works in
connection with supply of water and electricity, disposal of sewage and other services and
amenities and generally to do anything necessary or expedient for purposes of such
development or purposes incidental thereto :
Provided that save as provided in this Act, nothing contained in this act shall be construed
as authorizing the disregard by the Authority of any law for the time being in force."
Section 52 of the Act reads as under :
"52. Power to delegate. - The Authority may, by notification in the Official Gazette, direct
that any power exercisable by it under this Act except the power to make Regulations
may also be exercised by such officer or local Authority or committee constituted under
Section 5-A as may be mentioned
@page-SC1352
therein, in such cases and subject to such conditions, if any, as may be specified therein.
(2) The Central Government may, by notification in the Official Gazette, direct that any
power exercisable by it under this Act, except the power to make Rules may also be
exercised by such officer as may be mentioned therein, in such cases and subject to such
conditions, if any, as may be specified therein.
(3) The Lieutenant Governor of the National Capital Territory of Delhi may, by
notification in the Official Gazette, direct that any power exercisable by him under this
Act, except the power to hear appeals, may also be exercised by such officer as may be
mentioned therein, in such cases and subject to such conditions, if any, as may be
specified therein."
Section 57(1)(f) of the Act reads as under :
"57. Power to make Regulations. (1) The Authority, with the previous approval of the
Central Government, may, by notification in the Official Gazette, make Regulations
consistent with this Act and the Rules made thereunder, to carry out the purposes of this
Act, and without prejudice to the generality of this power, such Regulations may provide
for -
(f) The terms and conditions subject to which user of lands and buildings in contravention
of plans may be continued."
28. Indisputably, in exercise of its regulation making power contained in Section 57 of
the Act, the Central Government made regulations known as 'Delhi Development
Authority (Management and Disposal of Housing Estates) Regulations, 1968'.
Regulation 2 (13) thereof defines 'disposal price' in the following terms :
"2(13) "disposal price" or "hire purchase price" in relation to property means such price
as may be fixed by the Authority for such property."
Sub-sections (25) and (30) of Section 2 of the Act define "penalty" and "Scheme"
respectively, as under :
"2(25) "penalty" means a additional amount as laid down in the relevant agreement
payable by the allottee or hirer as a consequence of his default in the payment of
prescribed dues,"
"2(30) "scheme" means a scheme prepared by the Authority for the creation of one or
more housing estates;"
Regulation 3 empowers the Vice-Chairman to administer the Act subject to general
guidance and resolutions of the Authority, who may delegate his powers to any officer of
the Authority. Regulatin 5 provides for disposal of the property which may be effected by
either hire-purchase or sale or in such other manner and subject to such terms and
conditions as may be decided by the Authority from time to time.
Regulation 6 provides for fixation of price to be one which may be determined by the
Authority.
Regulation 8 provides for the manner of payment of disposal price.
Chapter III of the Regulations provides for the procedure for disposal of property. In
terms of Regulation 30 of the Regulations, the Authority is mandated to prepare an
allotment register in which names and other particulars of the registrants are to be
entered. The names of the persons on the waiting list should also be entered in a separate
section of the same register in the order in which their names appear in the draw of lots.
Power to decide representations has been conferred upon the Committee in regard to the
selection of applicants for allotments of property.
Regulation 37 provides for handing over of possession of the property.
Regulation 59 provides for delegation of all or any of the powers of the authority under
the Regulations to the Vice-Chairman or to a whole time member.
Function of DDA :
29. In Premji Bhai Parmar (supra), a Bench of this Court noticed that the Authority had
adopted a resolution delegating its power to the Vice-Chairman and the power to fix
disposal price was said to have been delegated to the Vice-Chairman.
30. At the outset, we may notice that the stand taken before us by Mr. Jaitely that the
scheme in question was a new one which had to be framed keeping in view the
exigencies of the situation arising out of the representations made by the registrants,
whose allotment had been cancelled, had not been raised before the High Court. Such a
stand, in fact, had never been taken.
31. The Authority issues an invitation
@page-SC1353
through its brochure.
32. The 5th SFS Scheme was announced in the year 1982. Any person could himself get
registered upon payment of fees prescribed therefor. The brochure published by the
authority indicated the tentative cost of different flats in different scheme category wise.
Pursuant thereto, an intending allocattee applies thereunder, and if in the draw of lots he
is declared successful, an allocation-cum-demand letter specifying the locality and the
floor is communicated to him. It is not the case of the parties that the estimated cost or
the tentative cost was the final one. The authority indisputably has the jurisdiction to fix
the disposal price finally upon taking into consideration all relevant factors. In terms of
the letter of allotment the estimated price is to be paid in four equal instalments.
Admittedly, question of the final cost is communicated to the registrants who is asked to
pay the balance amount after deducting the payment made in four instalments together
with the 5th instalment. The estimated cost, according to the authority, itself should be
worked out having regard to the following parameters.
1. Cost of construction
2. Community facility charges
3. Floor equilization for ground floor and subsidised for upper floors
4. Departmental charges
5. Administrative charges
6. Cost of land (Land Rate on the date of allocation letter is taken).
The land rates of the project area are determined on the basis of break even rates arrived
on cost benefit analysis which includes cost of acquisition, enhanced compensation and
future realisations. This rate is approved by the Government of India. The land rates
approved by the Government of India for Dwarka is also applied on flats in South Delhi.
They are predetermined rates on actual break even basis.
7. Surcharge @ 20% w.e.f. 16-8-1996."
33. Indisputably again, other components for determining the cost remains the same
every year. The only change which was effected, is the change in the land rate, which is
approved by the Government. Other parameters for calculating the material cost were
approved by the Vice-Chairman of the authority.
34. Delhi Development Authority has been created under a Parliamentary Act. It,
indisputedly, is a State within the meaning of Art. 12 of the Constitution of India. Being
so, the provisions of Part III of the Constitution of India must be applied by it.
Undisputedly, again, it has also the duty to strive hard for giving effect to the Directive
Principles of State Policy as contained in Part IV of the Constitution of India.
35

. Objects of the DDA is stated in S. 6 of the Act. We may notice that although the heading
of S. 6 states about the object of the Act, the main provision contain both its objects and
powers. It is also curious to notice that its power to constitute a scheme so as to provide
housing facilities to the citizens of India and, in particular, those belonging to lower
income group as also coming from lower strata of the society has not been mentioned
which we would find pari materia in statutes framed by other States. (See Chairman,
Indore Vikas Pradhikaran v. M/s. Pure Industrial Cock and Chem. Ltd. and Ors, 2007 (8)
Scale 110). 2007 AIR SCW 4387

36. It is accepted that although the Act was enacted in the year 1957, the idea of
providing for implementation of such housing scheme started much later. The Rules, as
noticed hereinabove, were framed only in the year 1968 and implementation of actual
housing schemes are said to have been started in late seventies or early eighties.
37. While acting as a 'State' within the meaning of Art. 12 of the Constitution of India, it
is imperative that D.D.A., while implementing its statutory power, upholds the
fundamental rights of the citizens and strive hard to give effect to their Directive
Principles of the State Policy. We, however, cannot also shut our eyes to the fact that in
terms of Art. 37 of the Constitution of India whereas the provisions of Part III are
justiciable, the provisions of Part IV are not. Only when an action of the State is taken to
give effect to any of the provision of Part IV of the Constitution of India which is not
otherwise ultra vires the Constitution or offends the principles embodied in Part III of the
Constitution of India, the same may be upheld, having regard to the provisions contained
in Part III thereof. The action of the State, therefore, must at the first instance be adjudged
on the touchstone of the principles of Fundamental Rights and then the provisions
contained in the Parliamentary Act, the regulations framed thereunder as
@page-SC1354
also the terms of the contract entered into by and between the parties.
38. We mayor may not agree with the submission of learned counsel for the appellants
that the right of housing arising out of such a scheme is a fundamental right within the
meaning of Arts. 19(1)(e) and 21 of the Constitution of India, but there cannot be any
doubt whatsoever that the action of a State must satisfy the principal requirements of Art.
14, viz., treating persons similarly situated equally and grant of equal protection to them.
Reasonableness and fairness is the heart and soul of Art. 14 of the Constitution of India.
Keeping the aforementioned principles in mind, we may consider the points involved
herein.
39. The basic fact that the scheme was floated in the year 1991 being SFS is not in
dispute. It has also not been denied or disputed before us that the said scheme in its
application is fundamentally different from those of the others schemes, viz., MIG, LIG,
Janta and Expandable Housing Scheme. There is also not much dispute as regards the fact
that in terms of the said scheme, estimated costs as well as rights and liabilities of the
parties are laid down in the invitation to offer. Allocation of the area, floor etc. ought to
be notified on acceptance of the offer by the registrants. Such allocation again
undisputedly is made on the basis of draw of lot having regard to the specific number of
flats available. The registrants have no choice in that behalf. Although he might have
exercised his right of option in relation to the area or the floor but then he, in fact, has no
hands thereover. The letter of allotment contains the schedule of payment as also other
terms and conditions in support thereof.
40. We, in this batch of appeals, are principally concerned, inter alia, with the
interpretation of Cl. 4 of the letters of allotment.
41. This scheme, ordinarily, was to operate within a time frame. DDA was expected to
complete the constructions within a period of 2½ years. Four six monthly instalments
were required to be paid by the registrants within the aforementioned period which would
include the grace period.
42. Whereas ninety days' time is not taken into consideration for the purpose of
computing the default clause, indisputably, again the power of the authority to regularize
the default on receipt of interest @ 18% per annum on the amount due to and owing to an
allottee is specifically provided for. The period of 2½ years vis-a-vis the six monthly
interest must have been laid down keeping in view the fact that whereas the amount
deposited by the registrants by instalments would be invested for construction of the flats.
18% interest is prescribed for default both on the part of the DDA as also on the part of
the registrants. No time limit is fixed for completion of the scheme. The only penalty
which the scheme prescribes is payment of interest. So far as area of SFS is concerned, a
registrant has no role to play. Admittedly, constructions have been completed in the year
2000.
43. For interpretation of Cl. 4 of the scheme, the aforementioned background is required
to be borne in mind.
Ingredients of Cl. 4 are :
"(a) The allotment letter would indicate the four due dates on which the first four
installments are to be paid. The fifth installment would be paid on demand.
(b) 90 days delay in the first four installment is condonable subject to payment of
prescribed interest.
(c) After the expiry of 90 days if payment is not made for first four installment there
would be automatic cancellation.
(d) This automatic cancellation can be restored on payment of interest and other charges
subject to availability of the flat.
(e) The cancellation due to non-payment of final installment will be made if the payment
is not made within 120 days with no provision of any further extension."
44. It has not been denied or disputed that although the registrants defaulted in making
payments but the flats were available. In fact, when the default took place, the flats were
not constructed. They have, thus, not been allotted to the persons on the wait list.
45. It may be true that recourse to Cl. (d) should be undertaken by the allocattees within a
reasonable time. What would be a reasonable time would, however, depend on the facts
and circumstances of each case. No hard and fast rule can be laid down therefor.
46. In a given case, it may be a few months but in another having regard to the conduct of
DDA, it may be one year or more.
@page-SC1355
What would constitute a reasonable period must also be considered keeping in view the
rights of the parties as also the fact that in terms of Cl. 4 of the offer of allotment there
does not exist any prohibition to pray for regularisation upon default, even after a period
of 120 days. In a situation of this nature, it may not be unjustified to arrive at the
conclusion that such a right can be exericsed, if not, when the flats were ready for
handing over actual possession, but at least when there has been a substantial progress.
We must also take into consideration that the scheme, the letter of allotment, the contract
between the parties to pay interest in case of default to each other leads to a conclusion
that DDA in its wisdom thought that payment of 18% interest shall subserve the purpose.
We, however, hasten to add that it does not mean that DDA must entertain such an
application for restoration and/or condonation of default despite lapse of time. It has its
own right in relation thereto which may be invoked. The right to allot the flat to a person
who is on the waitlist as a result whereof a seal of finality can be put, the right of the
registrants or registrants even the whole or a part of the advance or other amounts
deposited by him stand forfeited.
Interpretation of the Act :
47. DDA functions through the Committees constituted in terms of S. 5-A of the Act.
Power has been delegated in favour of its Chairman and the Vice-Chairman. Such
delegation of power, however, as provided for under S. 52 of the Act does not extend to
make regulations. What, therefore, cannot be done by way of regulation, cannot be done
by executive order.
48. It has not been denied or disputed that having regard to Cl. 5.10 of the brochure and
Cl. 4 of the letter of allotment, the term "tentative cost" must be given its natural
meaning.
49. It varies from time to time as it is not a one time process. The price difference
between the estimated cost and the initial cost "as it worked out on the completion as per
costing formula in vogue" would have to be paid along with the 5th and final instalment.
Notifications :
50. The impugned circulars have three distinct elements :
1. Price of South Delhi flats would be worked out by adding 20% surcharge in terms of
the office order dated 16-8-1996 duly approved on 22-8-1996.
2. 20% surcharge will have to be paid in case where there is a small delay, in which case
only interest has to be paid.
3. In all other cases original cost + 18% or the current cost whichever is higher would be
payable.
51. Legality and/or validity of the said circulars is in question. We may, at the outset,
notice that there is nothing on record to show that the office orders dated 16-8-1996, 27-
8-1996 and 21-3-1999 were published in the Gazette or were otherwise brought to the
notice of the registrants.
Conduct :
52. Conduct of the parties may be noticed from the case of Manjit Singh. Admittedly, the
registrants failed to pay instalments within the stipulated period. A notice dated 23-4-
1997 was served on him which is in the following terms :
"WHEREAS you had been allocated a second floor category-II, SFS flat in Pkt. F, Sheikh
Saria Residential Scheme vide allocation-cum-demand letter dated 31 -3-1993 under the
DDA (Management and Disposal of Housing Estates) Regulations, 1968 against your
registration under the 5th SFS - 1982.
AND WHEREAS as per the allocation-cum-demand letter the following installments
were to be paid as per the following schedule :-

Installments Amount Due Date Installment paid on Challan No.

Ist Rs. 1,35,867/- 30-4-93 24-5-93 032601


(Rs. 1, 35, 867)
IInd Rs. 1,25,060/- 30-10-93 30-10-93
31-5-94
(Rs. 50,000) 112025
(Rs. 43,675)
IIIrd Rs. 1,56,325/- 30-4-94 24-11-94 010243
(Rs. 50,000)
IVth Rs. 1,25,060 30-10-96 25-7-96 010245
(Rs. 50,000)

From the perusal of the above chart, it can be seen that you have not deposited the
installment as per schedule indicated in the allocation-cum-demand letter.
AND WHEREAS as per the terms and conditions of the allocation-cum-demand letter as
contained in Cl. 4 the allocation was liable to be cancelled if the installments are not
made as per the schedule.
AND WHEREAS it is evident that you failed to deposit the installments as per the
schedule and thus have committed the breach of the terms and conditions of the
allocation-cum-demand letter dated 31-3-1993.
Therefore, I, Dy. Director (SFS) hereby inform you that the allocation made to you vide
letter dated 26-3-93-31-3-1993 stands automatically cancelled due to the breach of the
terms and conditions of allocation.
@page-SC1356
It is further inform (sic) that the 5th SFS Scheme, 1982 stands already closed, therefore,
you are advised to return all the original FDR, Demand-cum-allocation letter and third
copy of challan so that your case can be processed for the refund of the amount deposited
by you."
53. He submitted a representation praying for condonation of default. In his
representation, he stated :
"Immediately on receipt of the DDA letter I ran hither and thither for financial loans from
relatives, friends, and well wishers and was able to deposit Rs. 2,12,770/- vide challan
No. 039576 dated 12-5-97 (Annex-ure-F) bringing the total payment to DDA to 90% of
the cost of flat. The flats are yet to be completed.
I now, therefore, humbly APPEAL to you to condone the delay in the payment of
installments for reasons stated above, on humanitarian and compassionate grounds and to
regularise the allocation of the flat for which act of kindness I shall ever pray. I am also
ready to pay any amount yet due in the form of interest."
54. The said representation was accepted by DDA in terms of a letter dated 10-12-1998
which reads as under :
"It is our pleasure to inform that you have been allotted SFS flat with the following
details through a computerised draw held on -/10/1998 :-

Flat No. 61
Floor SEC
Category II
Sector
Block
Pocket
Type
Locality SHEIKH SARAI

We are advising the Housing Acts Department to issue the 5th and Final Demand Letter
to you at the earliest."
55. It allotted the flat. No condition therefor was put. It did not ask for payment of any
surcharge. The original terms were not deviated from. It did not ask for any extra cost.
The delay in payment of installment was condoned and the allocation of flat which stood
cancelled in terms of the aforementioned notice dated 23-4-1997 stood restored. DDA,
thus, acted strictly in terms of the original scheme. The offer of the allocattee was
accepted relying on or on the basis of Cl. 4, viz., the amount of default and an interest
charged thereupon @ 18% per annum. It is only when the 5th and final instalment was
directed to be paid in terms of demand letter dated 15-2-1999, the current cost computed
in terms of the office order dated 16-8-1996 and subsequent office orders was included.
The allocattee at that stage might not have any other option but to pay the same for
obtaining possession. But by reason thereof, he never gave up his right to question the
action on the part of DDA. Rule of estoppel, therefore, has no application.
56. It was on the other hand, DDA who having accepted the offer of the allocatte by
restoring the allotment, in our opinion, is estopped and precluded from raising a plea as
regards application of office order dated 16-8-1996. It may be noticed that even contents
of those letters were not disclosed to the allocattee.
Was it a Restoration Scheme?
57. The office orders, on the basis whereof the purported impugned policy had been
taken, do not refer to the scheme as a restoration scheme. The resolutions do not say so.
Had it been so, DDA would have issued a fresh notification or at least made its stand
@page-SC1357
lear to the allocattees either by way of public notice or by informing each of such
defaulters individually. Had such conditions for the purpose of restoration being made
known, the allocattees would have accepted it or rejected it. Evidently, it is a part of the
original scheme. It is not a new one. It is well known principle of law that a person would
be bound by the terms of the contract subject of course to its validity. A contract in certain
situations may also be avoided. With a view to make novation of a contract binding and
in particular some of the terms and conditions thereof, the offeree must be made known
thereabout. A party to the contract cannot at a later stage, while the contract was being
performed, impose terms and conditions which were not part of the offer and which were
based upon unilateral issuance of office orders, but not communicated to the other party
to the contract and which were not even the subject-matter of a public notice. Apart from
the fact that the parties rightly or wrongly proceeded on the basis that the demand by way
of 5th instalment was a part of the original scheme, DDA in its counter-affidavit either
before the High Court or before us did not raise any contra plea. Submissions of Mr.
Jaitley in this behalf could have been taken into consideration only if they were pleaded
in the counter-affidavit filed by DDA before the High Court.
58. We have also reservations that in absence of any provision contained in the Act or the
Regulations, the delegatee on its own could frame such a new scheme. If not, the
purported restoration policy would be ultra vires. We, therefore, cannot persuade
ourselves to agree with the contention of DDA that the restoration policy cannot be tested
on grounds of terms and conditions of the original scheme. If it is a new scheme, it would
bear repetition to state, the terms contained therein should have been known to the
allocattees. We, thus, have no other option but to proceed to consider the legality and/or
validity of such imposition on the premise that the impugned policy decision is a part of
the original scheme and does not contain any new policy.
Policy decision :
59. An executive order termed as a policy decision is not beyond the pale of judicial
review. Whereas the superior Courts may not interfere with the nitty gritties of the policy,
or substitute one by the other but it will not be correct to contend that the Court shall like
its judicial hands off, when a plea is raised that the impugned decision is a policy
decision. Interference therewith on the part of the superior Court would not be without
jurisdiction as it is subject to judicial review.
60. Broadly, a policy decision is subject to judicial review on the following grounds :
(a) if it is unconstitutional;
(b) if it is dehors the provisions of the Act and the Regulations;
(c) if the delegatee has acted beyond its power of delegation;
(d) if the executive policy is contrary to the statutory or a larger policy.
61. The stand taken by DDA itself is that the relationship between the parties arises out of
the contract. The terms and conditions therefor were, therefore, required to be complied
with by both the parties. Terms and conditions of the contract can indisputably be altered
or modified. They cannot, however, be done unilaterally unless there exists any provision
either in contract itself or in law. Novation of contract in terms of S. 60 of the Contract
Act must precede the contract making process. The parties thereto must be ad idem so far
as the terms and conditions are concerned. If DDA, a contracting party, intended to alter
or modify the terms of contract, it was obligatory on its part to bring the same to the
notice of the allocatee. Having not done so, it, relying on or on the basis of the purported
office orders which is not backed by any statute, new terms of contract could thrust upon
the other parry to the contract. The said purported policy is, therefore, not beyond the
pale of judicial review. In fact, being in the realm of contract, it cannot be stated to be a
policy decision as such.
Price Fixation
62. We would assume that the office orders were issued by DDA keeping in view the
representations made by a large number of defaulters. The plea taken by DDA gives rise
to a dichotomy. If it is a case of contract qua contract, the provisions of the Contract Act
must be taken recourse to. If DDA was exercising a statutory power, the same must be
tested on application of doctrine of ultra vires. Floating a scheme for providing housing
facilities to a group of people, although is governed by statute, power under the statute by
an executive not
@page-SC1358
only can be tested on the touchstone of Art. 14 of the Constitution of India, but can also
be tested on the touchstone of source of the power under the statute. No provision either
in the Act or the Regulations was brought to our notice which makes the allocatte bound
by the purported policy decision taken by DDA. Even if it is so, the superior Courts may
exercise its power of judicial review as the power which is sought to be exercised by a
statutory authority is not under the contract but under a statute. When a contract emanates
from a statute or is otherwise governed by the provisions thereof, the superior Court can
also exercise the power of judicial review.
63. In Gujarat State Financial Corporation v. M/s. Lotus Hotels Pvt. Ltd. ((1983) 3 SCC
379 : AIR 1983 SC 848), it is stated that such contracts can be subject to judicial review.
64. Regulations 5 and 6 should not be very liberally construed. Concededly, the manner
in which power is to be exercised is governed by the past practice. Norms have been
fixed for computing the disposal cost. Although, the superior Courts ordinarily would not
interfere in the price fixation, but there does not exist any absolute ban In a case where
fixation of price is required to be made in a particular manner and upon taking into
consideration the factors prescribed and if price is fixed dehors the statutory provisions,
judicial review would be permissible.
65. Strong reliance has been placed upon the residuary power conferred upon the
authority to determine the cost of construction.
66. When the same is done having regard to the relevant factors on the basis of which
brochure as well as the notice inviting tender was issued, the superior Courts may not
interfere; but the same must be done in terms of the original contract and not dehors the
same. The authority, even while exercising its residuary power, is required to act within
the four corners of the contract. While doing so, the terms of the contract cannot be
altered to include any other factors which were contemplated thereunder. While
computing the extra cost, no additional factor, thus, can be taken into consideration. If
such a power is conceded in the authority, the same would give rise to exercise of
arbitrary power. It is not contemplated in law. When construing a provision delegates a
power on an authority under a statute, the constitutional provisions must be kept in mind.
67. At the time of calculation of the amount which would be the subject-matter of
demand of 5th and final instalment, the jurisdiction of DDA is to keep itself confined
only to the factors on the basis whereof, the brochure has been issued and offer was
made. No additional factor, thus, could be taken into consideration at the time of issuing
notice other than the ones on the basis whereof offer was made by the registrants.
Imposition of equalisation charge also fails within the said purview.
68. There may be some charges like conversion charges which per se may or may not be
bad. Conversion charges levied by the DDA pursuant to the directions issued by the
Central Government which in terms of S. 41 of the Act is binding on DDA for converting
the leasehold into freehold. However, such a power also must be exercised reasonably
and fairly. Conversion of the property from leasehold to freehold is a separate transaction.
The same has nothing to do with the actions, qua contract. Imposition of conversion
charges, therefore, even if, per se, may not be held to be bad, the said factor cannot be
taken into consideration for the purpose of computing construction costs. The High Court
has struck down the inclusion of such conversion charges in the costing of the flats. After
1996, the ordinary cost of construction of a flat was Rs. 2,00,000/-, in South Delhi but not
only it framed the basis for computing the final cost but also 20% additional amount as
also 20% surcharge were claimed thereupon. Sometimes interest also was charged as and
when applicable. Thus, so long the conversion charge is charged by way of a separate
transaction, no exception can be taken. But, purported price fixation as has been done in
the instant case cannot receive our approval. The same is, thus, in our opinion, bad in law.
Applicability :
60. The scheme in question was floated in 1992-93. The purported default on the part of
the registrants took place prior to the purported adoption of the policy decision. The
purported office order dated 16-8-1996 discloses/shows that an internal decision had been
taken to condone the delay in making payments of first four instalments. The authorities
mentioned
@page-SC1359
therein had been delegated with the power for the period(s) mentioned as under :

Director (H)-1 Up to 3 months


Commissioner (Housing) Up to 1 year
Principal Commissioner From 1 year to 1 % year
Vice-Chairman Full powers

Clause 3 of the said office order reads as under :


"There are presently cases in the Housing Department where there have been delays in
the making of the payments of the flats allocated/allotted in South Delhi under SFS.
Before the aforesaid revision took place, delays of one year or so were being regularised
with usual charges, i.e., on payment of 18% interest per annum and restoration charges,
etc. in few cases where delays are unusually long, current price has also been demanded."
70. Thus, a decision in that behalf had not only been taken but also was made applicable
both in the case where the delay is of one year or so and the delay which was unusually
long. By reason of the said circular, delay in making payments of instalment was to be
condoned on payment of either current price or old price whichever is higher. From a
perusal of the Resolution dated 27-8-1996, it appears that 20% surcharge was levied over
the disposal cost worked out for the flats in South Delhi SFS. It does not show that any
subsidy was proposed to be granted for the migrants from Jammu and Kashmir or Punjab.
The policy was taken only with a view to balance the reduced cash in-flow. DDA, thus,
had in view commercial aspect of the matter and not the social justice aspect.
71. Again, by reason of the office order dated 31-3-1999, the delegation of power in
favour of various authorities was redefined. The Vice-Chairman could deal with delay or
default even if it exceeds one year and six months. 22-8-1996 was prescribed as the cut
off date for the purpose thereof. Price of the flat was to be calculated on the basis of
either current price or old price whichever is higher. It was sought to be applied
irrespective of the extent of delay. On what basis 22-8-1996 was taken to be the cut off
date has not been disclosed. We would, however, assume that the said date was taken into
consideration in view of the Resolution dated 27-8-1996.
72. An executive officer, in absence of any provision of a statute, cannot apply his, own
decision with a retrospective effect. A delegatee is bound to act within the four corners of
the delegation and not beyond the same.
73. Delegation of power in favour of an authority under a statute must also be tested in
terms of the statutory provisions. No provision under the Act or the Regulations has been
brought to our notice which empowers the delegatee to alter the terms and conditions of
the contract with retrospective effect. The purported policy decision must, therefore, be
tested not only having regard to the provisions of the statute but also having regard to
Clause 4 of the offer.
74. Current cost has been calculated upon computing 20% over and above the actual cost.
A provision for surcharge had also been made in terms whereof a premium of 20% over
the disposal cost was worked out on current cost for the SFS flats in South Delhi.
Imposition of surcharge is subject to the condition that the real value in the market of
DDA flats would be much more than it had been charging as per the cost formula.
Parameters of computation of disposal price have been laid down which we have noticed
supra. The authority having itself adopted a formula for computing the disposal cost, the
same was binding upon the delegatees. A delegatee cannot take any action contrary to or
inconsistent with the factors laid down for computation of disposal cost as defined in
Section 2(30) of the Act. Regulations 5 and 6 do not authorize the delegatee to apply a
formula which was not contemplated by the Authority itself. If an Executive Authority in
absence of any statutory provision cannot apply a decision with retrospective effect, the
same would be ultra vires.
75

. In Vice Chancellor, M.D. University, Rohtak v. Jahan Singh, (2007 (4) Scale 226), this
Court observed : 2007 AIR SCW 2111

The Act does not confer any power on the Executive Council to make a regulation with
retrospective effect. The purported regulations, thus, could not have been given
retrospective effect or retro-active operation as it is now well-settled that in absence of
any provision contained in the legislative Act, a delegatee cannot make a delegated
legislation with retrospective effect."
[See also Ashok Lanka and Anr. v. Rishi Dixit and Ors., (2005) 5 SCC 598] 2005
AIR SCW 2676

@page-SC1360
76. A definite price is an essential element of binding agreement. A definite price
although need not be stated in the contract but it must be worked out on some premise as
was laid down in the contract. A contract cannot be uncertain. It must not be vague.
Section 29 of the Indian Contract Act reads as under :
"Section 29 - Agreement void for uncertainty. Agreements, the meaning of which is not
certain, or capable of being made certain, are void."
77. A contract, therefore, must be construed so as to lead to a conclusion that the parties
understood the meaning thereof. The terms of agreement cannot be vague or indefinite.
No mechanism has been provided for interpretation of the terms of the contract. When a
contract has been worked out, a fresh liability cannot thrust upon a contracting party.
78. It is settled that a definite price is an essential element of a binding agreement.
Although a definite price need not be stated in the contract, but assertion thereof either
expressly or impliedly is imperative.
Impugned Judgment :
79

. The Full Bench of the Delhi High Court has placed strong reliance on P.N. Verma
(supra). One of the foundamental errors which has been committed by the Full Bench,
with respect, is appying P.N. Verma (supra) without noticing the distinction between the
provisions contained in the clauses of the Brochure in the present case and those
obtaining therein. In the present case Clause 5.10 of the DDA Brochure stipulates that the
price mentioned in the allocation letter is only the estimated price and it could be changed
only on the basis of escalations in the price to be determined by DDA on the completion
of flats. In P.N. Verma (supra), however, the price was to be fixed on allotment of flats. It
gives rise to a lot of difference in determining the issue. AIR 1985 Del 417

In P.N. Verma (supra), the High Court observed :


"24. . . If the stand of the DDA is that the price they demand is only on the same formula
as was announced and that the increased price demanded is only due to escalation in cost
of construction and fluctuation in other cost factors, then the issue will only be whether
the fixation of price is in accordance with the contract and that can be gone into, both by
reason of principle and because it will involve complicated factual investigations, only in
a suit. This is the alternative stand of the DDA which we will discuss later. But if the
DDA says, whatever we may have said earlier, we can fix the price on any basis and that
cannot be questioned at all because it is a contractual matter, the argument is fallacious
because this stand of the DDA means going behind the contract and revising the earlier
formula of price fixation which means travelling back to the precontractual statutory
stage..."
It was also stated :
"46. . . That actual cost was tentatively fixed at a figure 'which was announced subject,
however, to variations in cost factors. It is not open to the DDA to alter this basis for the
determination of the disposal cost. So long as the DDA adheres to this basis in fixing the
disposal cost, Court will not interfere. It will not interfere even if there should be some
mistakes or exaggerations in the calculation of the components of the figure arrived at, as
that would be a matter for determination after evidence and investigation in a suit. Even if
an item should be included in the cost components, about the inclusion of which as one of
the cost factors there could be some doubt, the writ Court may not interfere and may
leave the parties to fight out their battle in a regular suit. But where the disposal cost is
fixed on a basis totally different from that announced earlier or where the components
taken into account cannot be described by any stretch of imagination as cost factors or
where a component of the cost is shown to have been fixed arbitrarily and without any
basis whatsoever, the Court has no option but to quash the determination of the disposal
cost so fixed and direct the DDA to undertake afresh a proper determination thereof in
accordance with the terms of the original contract or after excluding the items
unwarrantedly included therein or after re-determining the value of any component on a
proper and reasoned basis..."
On legal principle, therefore, ratio of P. N. Verma (supra) is not very different from what
has been held herein.
80

. Another fundamental error committed by the Full Bench was to approve para 29 of the
judgment of the Division Bench of the Delhi High Court in R.K. Sachar v. DDA, (LPA
No. 727 of 2002 decided on 15-12-2003) opining that this Court had already AIR
1980 SC 738

@page-SC1361
approved the contention of the DDA that it was entitled to recover such a charge in the
light of the decision in Premji Bhai Parmar (supra).
81. The Full Bench failed to notice that in P. N. Verma (supra) surcharge over and above
the disposal price titled "equalization charge" purporting to provide subsidy for
construction of flats for weaker sections of the society in trans Yamuna area was held to
be ultra vires and that the said decision has been approved by this Court. P. N. Verma
(supra), therefore, was misread and misconstrued.
82

. In Premji Bhai Parmar (supra), surcharge was a component of the disposal price. The
disposal price was known to the registrants at the time of conclusion of contract. As the
contract was found to be binding on the parties, levy of surcharge, thus, was held to be
vitiated in law. AIR 1980 SC 738

83. In this case, the case of the appellant is not that they are not bound to pay the binding
contractual stipulation as contained in Clause 4 of the letter of allotment. They are and
they must. But what cannot be thrust on them is the price determined on the basis of
factors which were not contemplated in the original contract.
84. In Premji Bhai Parmar (supra), this Court did not record a finding that even if a
surcharge was not a part of Brochure, still the same could be imposed without any
sanction in law so as to bind the allocatees to pay the same although neither they were
made aware thereof nor did they give their consent for payment of surcharge as a part of
the contract. The Full Bench of the High Court wrongly relied upon R. K. Sachar (supra).
In the light of the decision of this Court in Premji Bhai Parmar (supra), the nature of levy
should have been held to be completely distinct and, thus, Premji Bhai Parmar (supra)
had no application to the fact of the present case.
85. The Full Bench also misdirected itself insofar as it failed to take into consideration
that levy of 20% surcharge was in effect and substance a compulsory exaction to augment
the revenue requirements of the DDA and, thus, could not have been a part of the
contract. Any compulsory exaction should be viewed in the light of Article 265 of the
Constitution of India, unless it comes within the sphere of contract.
86

. It may be reiterated that it is only those components which fall within the Brochure of
the DDA or within the purview of the statutory requirements can be included in the
exercise of price fixation. To the said extent are the decision of the Delhi High Court in P.
N. Verma (supra), Narsingh Jain v. Union of India, [(80) 1999 DLT 742] and DDA SFS
Flat Owners Society v. UOI, [AIR 2001 Del 39]. 1999 AIHC 3748

87. Against the said judgment of the Division Bench of the High Court in P. N. Verma
(supra), an appeal was preferred by the DDA before this Court. This Court in the said
appeal titled as DDA v. SFS Assn. and Ors. (Civil Appeal No. 4402 of 1985] rejected the
contention of the DDA that under the terms of the Brochure related to the said scheme it
was empowered to recover from the registrants an additional amount over and above the
disposal price by way of equalization charges in the following terms :
"The lengthy and elaborate judgment of the High Court under appeal makes instructive
reading in prohibiting the DDA from adding to the prices of the named flats on escalation
termed as 'equalisation and ad hoc charges'. From the terms of the model contract entered
into by the DDA with the people who opted for the self-financing scheme, charging of the
said equalization and ad hoc charges is evidently totally missing.
The DDA in support thereof has banked upon the justness of its cause and demand, and
has nowhere been able to project that to begin with, it was part and parcel of the cost
factor. The High Court has seen through its design and has termed the venture as a
camouflage. We see no reason to take a different view than the one taken by the High
Court."
The said decision of this Court is binding upon the DDA.
Conclusion :
88. For the reasons aforementioned, the impugned judgment cannot be sustained which is
set aside accordingly. The appeals are allowed with costs. Counsel's fee assessed at Rs.
25,000/- in each case. Appeals filed by the DDA are dismissed.
Appeals allowed.
@page-SC1362
AIR 2008 SUPREME COURT 1362 "Shaikh Rafiq v. State of Maharashtra"
(From : Bombay)*
Coram : 2 PRAKASH PRABHAKAR NAOLEKAR AND MARKANDEY KATJU, JJ.
Criminal Appeal No. 169 of 2006, D/- 22 -1 -2008.
Shaikh Rafiq and Anr. v. State of Maharashtra.
(A) Evidence Act (1 of 1872), S.32 - DYING DECLARATION - POLICE OFFICERS -
MAGISTRATE - Dying declaration - Recorded by Police Officer - Special Executive
Magistrate though available not summoned - Certificate from Medical Officer as to
fitness and consciousness of declarant not taken - Time of recording of declaration also
not recorded - Declaration cannot be relied upon. (Paras 2, 3)
(B) Penal Code (45 of 1860), S.300 - MURDER - Murder - Improbability of prosecution
case - Allegation that accused along with daughter in law of deceased went to house of
deceased - Altercations took place as accused insisted deceased to take back his daughter
in law - Thereupon one of deceased poured kerosene and other ignited deceased - Story
improbable as accused had no direct relation with daughter in law of deceased and had
only gone to patch up disputes - Accused entitled to be acquitted.
Cri. A. No. 509 of 2004, D/-19-09-2005 (Bom), Reversed. (Paras 4, 5)

Ms. Bina Madhvan (for Lawyers Knit and Co.) for Appellants; Sushil Karanjkar (for
Ravindra K. Adsure) for Respondent.
* Cri. Appeal No. 509 of 2004, D/- 19-9-2005 (Bom) (Aurangabad Bench)
Judgement
JUDGMENT :-Appellants-accused No. 1 Shaikh Rafiq, No. 2 Fatimbee along with
accused No. 3 Jaibunisa were prosecuted for committing the murder of Noor Miya Mohd.
Hussain. Appellant-accused Nos. 1 and 2 were convicted by the Sessions Court under
Section 302 read with Section 34 of the Indian Penal Code and sentenced to suffer
imprisonment for life and to pay a fine of Rs. 5000/- each; in default, to suffer R.I. for six
months. Accused No. 3 Jaibunisa was acquitted by the Sessions Court. The order of the
Sessions Court was affirmed by the High Court in appeal. Aggrieved by the judgment
passed by the High Court, the present appeal by way of special leave petition, has been
preferred by the appellants.
2. As per the prosecution case, on 3-11-2002 at about 12 noon, on receipt of intimation
from the Medical Officer of Civil Hospital, P.W. 1 ASI Maroti proceeded to Burn Ward of
hospital where he found that Noor Miya Mohd. Hussain had suffered burn injuries. PW. 1
enquired from Noor Miya Mohd. Hussain (since deceased) about the incident to which he
narrated that the appellant-accused Nos. 1 and 2 along with accused No. 3 Jaibunisa
(daughter-in-law of deceased) had come to his house where appellant Nos. 1 and 2
insisted upon him to keep accused No. 3 with him which was refused by him. Some
altercations took place between Noor Miya Mohd. Hussain and the appellants and,
thereafter, appellant No. 2 poured kerosene on the person of Noor Miya Mohd. Hussain
and the appellant No. 1 set Noor Miya Mohd. Hussain on fire by igniting a match stick.
The conviction of the appellants was solely based on the dying declaration recorded by
P.W. 1 who deposed in his examination-in-chief that he recorded the dying declaration of
Noor Miya Mohd. Hussain (since deceased) wherein the deceased told him that appellant
Nos. 1 and 2 had come to his house along with his daughter-in-law accused No. 3
Jaibunisa and insisted upon him to keep accused No. 3 in his house. Upon his refusal,
some altercations took place between the appellants and deceased and after that appellant
No. 2 poured kerosene on the person of Noor Miya Mohd. Hussain (since deceased) and
appellant No. 1 set him ablaze. He obtained the signature of Noor Miya Mohd. Hussain
(since deceased) on the dying declaration after reading over the statement to him. The
dying declaration also bears the signature of Medical Officer. In his cross-examination,
he stated that he was serving in the Department for the last 30 years and was aware about
the procedure of recording dying declaration and was aware of the fact that Special
Executive Magistrates were also available for recording the dying declaration but he did
not call any of them. It was further admitted by him that he was accompanied by the
Medical Officer to the Burn Ward to identify Noor Miya Mohd. Hussain (since deceased)
and he did not take the certificate of fitness from the Doctor whether Noor Miya Mohd.
Hussain (since deceased) was in a position to give statement or not. He further admitted
that he did not obtain endorsement of the Medical Officer about consciousness of Noor
Miya Mohd. Hussain (since deceased) and he did not record the time of the dying
declaration being recorded.
3. Considering the dying declaration and the manner in which it was recorded, we cannot
rely upon the dying declaration recorded
@page-SC1363
by PW 1. Apart from this fact, there is no other evidence on record to implicate the
appellants in the incident.
4. Further, the story of the prosecution appears to be improbable. It is the case of the
prosecution that the daughter-in-law of the deceased accompanied by the appellants came
to his house and some altercations took place and thereafter appellant No. 2 poured
kerosene on his person and appellant No. 1 set him ablaze by igniting match stick. We fail
to understand as to why the appellants will carry kerosene to the house of the deceased
when they had gone there to patch up the quarrel. We also fail to understand as to why the
extreme step of setting Noor Miya Mohd. Hussain (since deceased) on fire would be
taken by the appellants who had no direct relations with the daughter-in-law of Noor
Miya Mohd. Hussain (since deceased). From the story put up by the prosecution, the
whole incident, as is being alleged to have happened, is wholly improbable and cannot be
relied upon.
5. For the aforesaid reasons, we are of the view that no case is made out by the
prosecution and the appellants-accused are entitled for acquittal. The appeal is,
accordingly, allowed. The orders of the courts below i.e. High Court and Sessions Court
are set aside. The appellants-accused shall be set at liberty if they are not required in any
other case.
Appeal allowed.
AIR 2008 SUPREME COURT 1363 "J. C. Budhraja v. Chairman, Orissa Mining
Corporation Ltd."
(From : 2000 (89) Cut LT 471)
Coram : 3 H. K. SEMA, G. P. MATHUR AND R. V. RAVEENDRAN, JJ.
Civil Appeal Nos. 1971-1973 of 2000, D/- 18 -1 -2008.
J.C. Budhraja v. Chairman, Orissa Mining Corpn. Ltd. and Anr.
(A) Limitation Act (36 of 1963), S.18 - LIMITATION - ARBITRATION -
Acknowledgment - Extension of limitation - Corporation acknowledged pending claims
of contractor and paid certain amount thereafter - Thus limitation would get extended by
three years from date of payment and at all by three years from date of acknowledgment -
Claims made within said three years before arbitrator - Not barred.
Arbitration Act (10 of 1940), S.8(2).
What can be acknowledged is a present subsisting liability. An acknowledgment made
with reference to a liability, cannot extend limitation for a time barred liability or a claim
that was not made at the time of acknowledgment or some other liability relating to other
transactions. Any admission of jural relationship in regard to the ascertained sum due or a
pending claim, cannot be an acknowledgment for a new additional claim for damages.
(Para 15)
In the instant case certain payments were made to contractor by the Corporation, OMC.
Contractor had made some claims and OMC wrote a letter dated 28-10-1978 in regard to
the pending claims of the contractor. OMC informed the contractor that it has been
decided to constitute a Committee which will go into the claims of the contractor so that
the dues, if any, could be ascertained. It further stated that on the details of the claims and
payments received being given to the contractor, OMC will settle up the pending matters
at the earliest. This clearly showed an intention on the part of OMC to admit the jural
relationship of contractor and employer and an intention to settle the pending claims after
being satisfied about them. Therefore, the letter dated 28-10-1978 was clearly an
acknowledgment in writing in so far as the 'pending claims' of the contractor. The
Committee constituted by the OMC examined these claims and admitted the claims only
to an extent of Rs. 3,52,916/- as per its final report dated 7-12-1979. OMC paid Rs.
3,50,000/- on 4-3-1980. In view of the acknowledgment in writing on 28-10-1978 and
payment of the Rs. 3,50,000/- on 4-3-1980, it can be said that in regard to the pending
claims of the contractor, the limitation stood extended by three years from 4-3-1980 and
at all events by three years from 28-10-1978. It is not in dispute that the contractor issued
the notice invoking arbitration on 4-6-1980 and immediately filed a petition under S. 8(2)
of the Arbitration Act for appointment of arbitrator which was allowed on 6-10-1980.
Therefore, whatever claims were made before the Arbitrator which was part of the
pending claim, was within time, having been made within three years from 28-10-1978
and 4-6-1980. (Para 16)
(B) Arbitration Act (10 of 1940), S.8(2), S.30 - ARBITRATION - LIMITATION - Award
- Setting aside of - Error apparent on face of record - Period of limitation for filing
petition seeking appointment of arbitrator - Cannot be confused with period of limitation
for making claim - Acknowledgment of pending claims of contractor by corporation -
Limitation gets extended only in
@page-SC1364
regard to liability which was acknowledged - And not regarding future claims - Fresh
claim made for first time before arbitrator - Finding by arbitrator that entire claim was
within limitation - Amounts to error apparent on face of record and legal misconduct -
Award made in respect of fresh claims, liable to be set aside.
2000 (89) Cut LT 4712, Reversed. (Paras 16, 19, 20, 21, 26, 28)
(C) Arbitration Act (10 of 1940), S.30 - ARBITRATION - MISCONDUCT - Award -
Setting aside of - Misconduct - Arbitrator awarding more than what was claimed in the
claim statement (by showing a lessor amount as having been paid by Corporation though
claim statement by contractor showed a higher amount) - Can be said to have committed
legal misconduct - Award in excess of claim, liable to be set aside. (Paras 22, 23, 26, 28)
(D) Arbitration Act (10 of 1940), S.30 - ARBITRATION - Award - Setting aside of -
Claim by contractor in regard to escalation in cost at the rate of 15%, 28.5% and 32% in
respect of value of work done in three years respectively - But arbitrator awarded
escalation at a flat rate of 32.6% on entire cost of work done from beginning - It amounts
to exceeding jurisdiction - Award in excess of what was claimed was invalid. (Paras
24, 26, 28)
Cases Referred : Chronological Paras
1999 AIR SCW 421 : AIR 1999 SC 801 (Rel. on) 18
1995 AIR SCW 1575 : AIR 1995 SC 2189 (Ref.) 7
1994 AIR SCW 1335 : AIR 1994 SC 1615 (Rel. on) 18
AIR 1988 SC 1007 (Rel. on) 18
AIR 1987 SC 81 27
AIR 1961 SC 1236 (Ref.) 14
A.K. Panda, T.S. Doabia, Sr. Advocates, Rutwik Panda, Ms. Mridual Aggrawal, for
Appellant; C.S. Vaidyanathan, Sr. Advocate, K.V. Viswanathan, Rajeev Singh, Anup
Kumar Singh, for Respondents.
Judgement
RAVEENDRAN, J. :- These appeals are filed against the common judgment dated
15.10.1999 passed by the High Court of Orissa in Misc. Appeal No.296/1998 filed by the
respondents and Misc. Appeal No. 198/1998 and Civil Revision No.109/ 1998 filed by
the appellant.
2. The appellant is stated to be legal heir and successor in interest of N.C. Budhraja
(hereinafter referred to as the contractor). M/s. Orissa Mining Corporation Ltd. (for short
OMC or respondent) entered into an agreement dated 16-9-1967 (Agreement No. 30/F-2)
for removal of over-burden at Kaliapani (Cuttack District) by excavation in all kinds of
soil (including stoney earth and gravel mixed with boulders, and depositing/disposing of
the same, as directed. The maximum lift was 6m including initial lift of one metre. The
order to commence work was issued on 23-9-1967. Parties also entered into three
supplementary agreements in regard to the said contract No. 30/F-2, on 2-8-1969, 7-3-
1970 and 10-2-1972. [Note : OMC had also entered into other contracts with the
contractor including contract dated 22-2-1968 (Contract No.2/F/2) for raising Chrome
Ore by open excavation from the said mining area. We are not concerned with those
contracts in these appeals].
3. The main agreement enumerated two items of work in its schedule. The first, second,
and third supplementary agreements enumerated respectively eight items, one item and
four items in their respective schedules. The work was completed by the contractor on
15-6-1975. The final bill in respect of the work was prepared by OMC on 21-10-1976. It
was revised in March-April 1977 by OMC. The final bill it showed the total value of the
work done (under several items in the schedule to main and three supplementary
agreements) as Rs. 1,49,190,76.74. The contractor countersigned the said bill on 14-4-
1977 under protest, but, however, certified and confirmed that the measurements shown
therein were correct.
4. According to the contractor, having regard to the zig-zag route by which the over
burden had to be carried, the actual lead was much longer and actual lift was much higher
than what were stipulated in the agreement. He contended that the amounts shown as due
for the work done was as per contract rates which was for removing overburden to the
extent of lift and lead provided in the contract schedules; and at several places, he had to
cut and remove the over-burden beyond the extent of lift and lead provided in the
contract, and he should be paid for such extra leads and lifts. He claimed to have
executed certain additional works not provided in the contract schedules, on the
directions of OMC. He therefore represented that the matter may be examined and
enquired into for determination of proper amounts due. In view of the several
representations made by the contractor in respect of the contract no. 30/F-2 as also other
contracts, OMC sent the following
@page-SC1365
letter dated 28.10.1978 to the contractor :-
"Re : Settlement of pending claims.
You had called on Chairman, OMC, recently and apprised him of the dues receivable by
you in respect of certain long pending matters such as mine benches work and raising at
Kaliapani Quarry-I. In the matter of Kaliapani it has been decided to constitute a
committee which will go separately into your claims and other facts, in which connection
you are requested to give all possible help and assistance, so that your dues, if any, will
be ascertainable.
In regard to other pending matters, you had indicated yourself that you will give the
details of claims and payment received by you. This may be given within a day or two so
as to enable OMC to settle up the above at the earliest. "
5. The contractor sent a reply dated 16-11-1978 enclosing therewith a statement
quantifying his claims relating to contract No. 30F-2 (subject matter of these appeals) as
also another contract (No. 2F-2). A Committee was constituted by OMC to scrutinize and
recommend on the admissibility of the claims made by the contractor in regard to
Agreement No. 30/F-2 and Agreement No.2/F-2. Several meetings were held by the said
Committee and the claims of the contractor aggregating to Rs. 50,15,820 in regard to
contract No. 30.F2 were considered. Ultimately the Committee submitted a final report
dated 7-12-1979 expressing the view that the contractor could be paid only a sum of Rs.
3,52,916/- in regard to his claims in respect of the two contracts. The contractor,
thereafter, wrote a letter dated 29-2-1980 stating that he had come to know that the
Committee had submitted its final report and requested for a copy of the report and for
payments of the amounts due. OMC sent a reply dated 4-3-1980 stating that the claims
were not accepted yet but however agreed to release a sum of Rs. 3.5 lakhs and released
the said sum on that day.
6. The contractor sent a notice dated 4-6-1980 invoking the Arbitration Agreement
(Clause 23) in respect of pending claims relating to Contract No. 30F-2 and two other
contracts. He suggested a panel of names and requested OMC to appoint one of them as
Arbitrator. Immediately, thereafter, the contractor filed Misc. Case No.306/80 in regard to
the contract in the Court of the Sub-Judge, Bhubaneswar, under section 8(2) of
Arbitration Act, 1940 (Act for short) for appointment of an Arbitrator. The court allowed
the said petition by order dated 6-10-1980 appointing Mr. Justice Balakrishna Patro, a
retired Judge of the Orissa High Court as Arbitrator by consent. On 16-12-1982, an
application was made by the present appellant under Order 22 Rule 3 CPC claiming to be
the son of legatee of the contractor and for substituting him in place of the deceased N.C.
Budhraja, as his legal heir. The said application was allowed by the court on 15-11-1985.
In the meanwhile, Arbitration Act, 1940 ('Act' for short) was amended by the Arbitration
(Orissa Amendment) Act, 1984, inserting section 41A providing for constitution of and
reference to the Arbitration Tribunal. By Notification dated 3-5-1986, (amended by
Memo dated 23-6-1986) the State Government constituted a one Member Special Arbitral
Tribunal with Justice N. K. Das as Arbitral Tribunal to settle the disputes between the
contractor and OMC in regard to contract No. 30/F-2.
7. The contractor filed a claim statement dated 27-6-1986 before the arbitrator praying for
an award of Rs. 3,41,42,040 with interest from 1-6-1986, as detailed below :

Value of work done by the contractor : Rs. 2,45,85,183.89


Less : Amounts paid by OMC to the Contractor
Rs. 1.49.88.566.90
Balance due : (Rs.95,96,616.99) rounded off as : Rs. 95,96,616.00
Add : Interest on the said amounts from the respective dates: Rs.2,40,09,948.00
Add : Interest on belated Payments :
Rs. 5,35,476.00 Rs. 2,45,45,424.00
________________
Total Rs. 3,41,42,040.00
________________

@page-SC1366
In the claim statement filed before the arbitrator, the nature and quantum of claim made
was different from what was claimed in the letter dated 16-11-1978 which was
considered by the Committee. In the claim statement the contractor abandoned claims to
an extent of Rs. 21,83,692 out of the claim of Rs. 50,15,820/- made on 16-11-1978 and
claimed only Rs. 26,32,128 from the original claim. The balance of the claim was fresh
claims, not made earlier. The claim of Rs. 95,96,616 made before the arbitrator was made
up of two parts, first being a part of the original claim made in the letter dated 16-11-
1978 and the second being completely fresh claims made for the first time in the claim
statement, as detailed below :

(i) Out of the original claim of Rs. 50,15,820 made in the letter dated 16-11-1978
(The claim for balance of Rs. 21,83,692 not pursued in arbitration) Rs. 28,32,128
(ii) Fresh claims not made earlier. Rs. 67,64,488

(Note : As per actual calculations, the total of the claims made by the appellant was Rs.
96,66,107 and the new claims were Rs. 68,33,979).
8. The Arbitrator made a reasoned award dated 28.11.1986 holding that the appellant was
entitled to a sum of Rs. 1,02,66,901.36 (which was more than the claim of Rs. 95,96,616)
with interest at 12% per annum from 1-8-1977 till date of award, and future interest at the
rate of 6% P.A. from the expiry of one month from the date of the award till date of
decree. The award is in respect of 35 claims. Out of 35 claims, Items 1 to 16 related to the
schedule items of work under the contract (main agreement and the supplementary
agreement 1 to 3). Items 17 to 34 were in respect of work which did not form part of the
contract schedule. Claim 35 related to escalation in cost of labour and material on
account of delay in execution.
8.1 The details of the items 1 to 16, (that is description of work, total amount claimed,
amount admitted, difference in dispute and amount awarded) are as under :
Sl.No.
Description of item
Claim of Contractor
Amount admitted by OMC
Amount in dispute
Award by Arbitrator
1 Removal of overburden in all kinds of soil etc. within a lead of 100 m. (Maximum
lift 6M) 50802.98 45040.32 5762.66 5762.66
2 Removal of overburden etc. within a lead of one km beyond initial lead of 30m
(maximum lift 6m) 406881.20 406581.20 300.00 300.00
3 Transportation of excavated overburdened etc., within a lead of 1 km beyond
initial lead of 6m 676228.94 616245.60 59983.34 59983.34
4 Transportation of excavated over burdened etc., within a lead of 2km beyond one
km. 5361.09 5361.09 0 0
5 Clearing heavy jungle etc., 6201.72 3303.72 2898.00 0
6 Cutting and uprooting trees etc., 5' grith 29800.80 14205.60
15595.20 0
7 Cutting and uprooting trees etc., 5' to 10' grith 11352.00 3360.00
7992.00 0
8 Excavation of overburden in all kinds of rocks etc., upto 60m lead 3689850.00
3390979.12 298870.88 298870.88
@page-SC1367
9 Excavation of overbur- den in all kinds of rock etc., upto 2km distance and within
lifts of 35m. 10379041.20 10379041.20 0 0
10 Lift beyond 15m upto 16m depth. 5066.85 5054.84 12.01 12.01
11 Lift beyond 16m upto 17m. 9858.70 9785.55 73.15 73.15
12 Lift beyond 17m upto 18m. 12647.79 12373.84 273.95 273.95
13 Lift beyond 18m upto 19m. 13358.54 13037.07 321.47 321.47
14 Lift beyond 19m upto 20m. 9812.25 9447.79 364.46 364.46
15 Lift beyond 20 upto 21m. 5070.33 4882.89 187.44 187.44
16 Lift beyond 21m upto 22m. 89.71 76.89 12.82 12.82

1,53,16,507.00
149,19,076/72
397,430/28
365,862/18

Though in the claim statement, the appellant had clearly stated that he had in all received
Rs. 149,88,566.90, and given credit for the said sum, during the hearing, the appellant
contended that instead of Rs. 149,88,566.90, he had appropriated only Rs. 120,01,695,90
towards this contract and that the balance of Rs. 29,86,871/- had been adjusted towards
some other contracts. Even though the claim statement was not amended, the Arbitrator
proceeded on that basis and awarded Rs. 32,83,243 in respect of items 1 to 16 as under :

A. Total amount claimed for Items 1 to 16 Rs. 153,16,507.00


B. Total of Items 1 to 16 admitted by OMC Rs. 149,19,076.72
C. Total of claims admitted by Arbitrator Rs. 3,65,862.18
(B+C) Total Rs. 152,84,938.90
Amount shown as received by contractor towards Items 1 to 16 (as against Rs.
149,88,566.90 shown as received from OMC in the claim statement) Rs.
1,20,01,695.90
BALANCE arrived at Arbitrator as due to contractor in respect of items 1 to 16 Rs.
32,83,243.00

8.2 Claims of contractor at SI. No. 17 to 34 related to items of work not covered in the
schedule to the contract, for which claim was made on the basis of damages/quantum
meruit. As against the total of Rs. 70,56,573,55 claimed in regard to these 18 items (Items
17 to 34), the Arbitrator awarded in all Rs. 52,56,847,36. The details of the claims made
by the appellant and the amount awarded in respect of each of them are as under :

S.No. Description of item Amount Claimed Amount Awarded


17 Extra Head Lead for 90 m 2810144.10 2450042.88
18 Removal of excavated materials from the edge of the quarry 54888.60
50858.60
19 Unmeasured quantity of excavation 848372.64 664720.00
20 Catch Water Drain 278842.50 27842.50
21 Removal of slipped earth from side slopes 143646.00 Nil
22 Restoration of benches to proper shape 186761.16 140070.87
23 Bullah Pilling to prevent slipping of benches 15722.70 Nil
24 Dry rubble packing 202499.00 122856.40
25 Extra lift during construction of Haul Road 262837.73 262837.73
26 Extra lift for excavated materials dumped at quarry edge 360690.49
270517.88
@page-SC1368
27 Extra lift measured by Surveyor but not paid. 1396128.63 1047096.50
28 Idle labour due to non-supply of working plan. 145577.00 Nil
29 Idle labour due to want of working site. 76850.00 Nil
30 Idle labour due to stoppage of work by the respondent and restriction of working
area. 389288.00 194644.00
31 Repairing of Haul road damaged by cyclone.10640.00 Nil
32 Reconstruction of Damsala Embankment. 25370.00 25370.00
33 Barbed wire fencing 27315.00 Nil
34 Supply of electricity to work site and respondent's colony. 72000.00 Nil

Rs. 70,56,573.55
Rs. 52,56,847.36

8.3 The last item of claim of the appellant, namely item No. 35 was for Rs. 22,17,188.34
as escalation in cost between 1972 and 1975 on account of increase in cost of labour and
material, based on the General Price Index. The Arbitrator determined the value of work
executed after 1-4-1973 as Rs. 52,96,967/-. He awarded an escalation of 32.6% on the
said value of work and awarded Rs. 17,26,811.00 as escalation in cost of labour and
material.
8.4 Thus the Arbitrator awarded Rs. 102,66,901.66 to the appellant as detailed below
(exclusive of interest), as against the claim of Rs. 95,96,616/- (exclusive of interest) made
by the appellant :

(i) Amounts award in respect of claims 1 to 16 Rs. 32,83,243.00


(ii) Amounts awarded in respect of claims 17 to 34 (as against claim of Rs. 70,56,
573.55) Rs. 52,56,847.36
(iii) Amount awarded in respect of claim 35 as escalation (as against claim of Rs.
22,17,188.34) Rs. 17,26,811.00
________________
Total award Rs. 102,66,901.36
________________

9. The contractor filed OS No. 224/1986 for making the award rule of the court, on the
file of the Civil Judge, Sr. Division, Bhubaneswar. The objections to the said award filed
by OMC were registered as Misc. Case No.5/1987. The said court, by common judgment
dated 21-3-1998, overruled the objections and directed that the award of the arbitrator be
made the rule of the court and a decree be drawn in terms of the award.
10. Feeling aggrieved, OMC filed Misc. Appeal No. 296/1998, challenging the decision
of the Civil Judge refusing to set aside the award, directing a decree in terms of the
award. The contractor filed Misc. Appeal No. 198/1998 and Civil Revision No. 109/1998
claiming future interest from the date of decree as the judgment of the Civil Judge was
silent on that aspect. The High Court heard and disposed of the said appeals and revision
petition by common judgment 15-10-1999. It allowed Misc. Appeal No.296/1998 filed by
OMC and dismissed M.A. No. 198/1998 and C.R. No. 109/ 1998 filed by the contractor.
The High Court held :
(i) The claim of the contractor was barred by limitation and therefore the award was
liable to be set aside.
(ii) The arbitrator acted beyond his jurisdiction in awarding huge amounts towards
alleged extra work, even though there was nothing to indicate that conditions
contemplated in proviso to Clause 11 (relating to additional work) were satisfied.
(iii) Though the award purported to be a reasoned award, the award in regard to Items 17,
18, 19 and 25 to 27 was not supported
@page-SC1369
by any reason and therefore, the award was liable to be set aside.
(iv) The award in respect of escalation in cost (item 35) at the rate of 32.6% of the value
of work was without basis, (when the claim itself was for a lesser rate), in the absence of
any provision in the contract for escalation, amounted to legal misconduct.
(v) The award being in excess of the claim made by the contractor shocked the judicial
conscience of the court.
(vi) Interest could have been awarded by the arbitrator only from the date of reference
(6.10.1980) and could not be awarded in regard to any pre-reference period.
(vii) Though in the normal course, some of the issues would have necessitated remitting
the matter to the arbitrator for fresh consideration, it was not necessary to remit the matter
as the entire award was being set aside on the ground of limitation.
The said decision of the High Court is challenged by the appellant in this appeal by
special leave.
11. On the contentions urged, the following questions arise for consideration :
(i) Whether the claim made before the arbitrator or any part thereof was barred by
limitation?
(ii) Whether the award is liable to be set aside on the ground of legal misconduct and the
error apparent on the face of the award?
(iii) Whether the award is liable to be set aside on the ground that the arbitrator exceeded
his jurisdiction?
(iv) To what relief the parties are entitled?
Questions (i) and (ii) :
12. The Arbitrator held that the claims were not barred. He held :
"In the case of a suit, the date on which the cause of action arises is the date from which
the limitation period starts. Under section 20, it is the date on which the right to apply
accrues that determines the starting point. That starting point does not coincide with the
date on which the cause of action for filing a suit arises. The same principle would apply
to an application under section 8 of the Act......The claimant signed the final bill on 14-4-
1977 under protest. It is not correct to say that the claimant accepted the final bill. All
these factors show that negotiation was going on and the matter was in a nebulous and
fluid stage. The committee gave its report in December, 1979. In March, 1980 some
portion out of the money said to have been found due by the committee was paid on ad
hoc basis. Notice was given by the contractor on 14-6-80. So, the dispute as to final bill
still continues. Till the final bill is prepared and accepted by the contractor, limitation
would not accrue. When the matter went to court in 1980, it was not barred by
limitation ..........."
13. The High Court found that the work was completed on 15-6-1975, final measurement
was taken on 16-6-1975 and the final bill was signed by the contractor under protest on
14-4-1977 and therefore held that the cause of action for the contractor to make a claim
arose on 14-4-1977. According to the High Court, as the notice invoking arbitration was
issued on 4-6-1980 and the petition under section 8(2) of the Act was filed thereafter,
beyond three years from 14-4-1977, the entire claim was barred by limitation. The High
Court further held that as the final bill was signed under protest by the contractor, it could
be said that the cause of action arose on a date subsequent to the date of signing of the
final bill. It further held that the fact that the Departmental Committee considered the
claims in 1979, subsequent to the signing of the final bill under protest, did not have the
effect of saving/ extending limitation in the absence of any acknowledgement in writing
as required under section 18 of the Act.
14. Section 18 of the Limitation Act, 1963 deals with effect of acknowledgement in
writing. Sub-section (1) thereof provides that where, before the expiration of the
prescribed period for a suit or application in respect of any right, an acknowledgement of
liability in respect of such right has been made in writing signed by the party against
whom such right is claimed, a fresh period of limitation shall be computed from the time
when the acknowledgement was so signed. The explanation to the section provides that
an acknowledgement may be sufficient though it omits to specify the exact nature of the
right or avers that the time for payment has not yet come or is accompanied by a refusal
to pay, or is coupled with a claim to set off, or is addressed to a person other than a
person entitled to the right. Interpreting section 19 of the Limitation Act, 1908
(corresponding to section 18 of the Limitation Act, 1963) this Court in Shapur
@page-SC1370
3Fredoom Mazda v. Durga Prosad Chamaria (AIR 1961 SC 1236), held :
".......... acknowledgement as prescribed by section 19 merely renews debt; it does not
create a new right of action. It is a mere acknowledgement of the liability in respect of the
right in question; it need not be accompanied by a promise to pay either expressly or even
by implication.
The statement on which a plea of acknowledgement is based must relate to a present
subsisting liability though the exact nature or the specific character of the said liability
may not be indicated in words. Words used in the acknowledgement must, however,
indicate the existence of jural relationship between the parties such as that of debtor and
creditor, and it must appear that the statement is made with the intention to admit such
jural relationship. Such intention can be inferred by implication from the nature of the
admission, and need not be expressed in words. If the statement is fairly clear, then the
intention to admit jural relationship may be implied from it. The admission in question
need not be express but must be made in circumstances and in words from which the
court can reasonably infer that the person making the admission intended to refer to a
subsisting liability as at the date of the statement. Stated generally, courts lean in favour
of a liberal construction of such statements though it does not mean that where no
admission is made one should be inferred, or where a statement was made clearly without
intending to admit the existence of jural relationship such intention could be fastened on
the maker of the statement by an involved or far-fetched process of reasoning."
In construing words used in the statements made in writing on which a plea of
acknowledgment rests oral evidence has been expressly excluded but surrounding
circumstances can always be considered. The effect of the words used in a particular
document must inevitably depend upon the context in which the words are used and
would always be conditioned by the tenor of the said document...........
15. It is now well settled that a writing to be an acknowledgment of liability must involve
an admission of a subsisting jural relationship between the parties and a conscious
affirmation of an intention of continuing such relationship in regard to an existing
liability. The admission need not be in regard to any precise amount nor by expressed
words. If a defendant writes to the plaintiff requesting him to send his claim for
verification and payment, it amounts to an acknowledgment. But, if the defendant merely
says, without admitting liability, it would like to examine the claim or the accounts, it
may not amount to acknowledgment. In other words, a writing, to be treated as an
acknowledgment of liability should consciously admit his liability to pay or admit his
intention to pay the debt. Let us illustrate. If a creditor sends a demand notice demanding
payment of Rs. 1 lakh due under a promissory note executed by the debtor and the debtor
sends a reply stating that he would pay the amount due, without mentioning the amount,
it will still be an acknowledgment of liability. If a writing is relied on as an
acknowledgment for extending the period of limitation in respect of the amount or right
claimed in the suit, the acknowledgment should necessarily be in respect of the subject-
matter of the suit. If a person executes a work and issues a demand letter making a claim
for the amount due as per the final bill and the defendant agrees to verify the bill and pay
the amount, the acknowledgment will save limitation for a suit for recovery of only such
bill amount, but will not extend the limitation in regard to any fresh or additional claim
for damages made in the suit, which was not a part of the bill or the demand letter. Again
we may illustrate. If a house is constructed under the item rate contract and the amount
due in regard to work executed is Rs. two lakhs and certain part payments say
aggregating to Rs. 1,25,000/- have been made and the contractor demands payment of the
balance of Rs. 75,000/- due towards the bill and the employer acknowledges liability, that
acknowledgment will be only in regard to the sum of Rs.75,000/- which is due. If the
contractor files a suit for recovery of the said Rs.75,000/- due in regard to work done and
also for recovery of Rs. 50,000/- as damages for breach by the employer and the said suit
is filed beyond three years from completion of work and submission of the bill but within
three years from the date of acknowledgment, the suit will be saved from bar of limitation
only in regard to the liability that was acknowledged namely Rs. 75,000/- and not in
regard to the fresh or additional claim of Rs. 50,000/- which was not the subject-
@page-SC1371
matter of acknowledgment. What can be acknowledged is a present subsisting liability.
An acknowledgment made with reference to a liability, cannot extend limitation for a
time- barred liability or a claim that was not made at the time of acknowledgment or
some other liability relating to other transactions. Any admission of jural relationship in
regard to the ascertained sum due or a pending claim, cannot be an acknowledgment for a
new additional claim for damages.
16. We will now examine this case with reference to the said principles. In this case, the
cause of action accrued on 14-4-1977 when the final bill was signed by the contractor. It
is not in dispute that the final bill showed that a sum of Rs. 17,69,608.73 was payable to
the contractor (after giving credit to the payments made and after withholding a sum of
Rs. 7,45,953.83 as 5% security deposit). Towards the said sum of Rs. 17,69,608.73, Rs.
17 lacs was paid on 25-2-1976 and Rs.70,000/- was paid on 6-8-1977. The contractor had
made some claims and OMC wrote a letter dated 28-10-1978 in regard to the pending
claims of the contractor. In regard to Kaliapani matters, OMC informed the contractor
that it has been decided to constitute a Committee which will go into the claims of the
contractor so that the dues, if any, could be ascertained. It further stated that on the details
of the claims and payments received being given to the contractor, OMC will settle up the
pending matters at the earliest. This clearly showed an intention on the part of OMC to
admit the jural relationship of contractor and employer and an intention to settle the
pending claims after being satisfied about them. Therefore, the letter dated 28-10-1978
was clearly an acknowledgment in writing insofar as the "pending claims" of the
contractor. What were the pending claims is made clear in the letter dated 16-11-1978
written by the contractor enclosing a statement showing that in all, a sum of Rs.
50,15,820/- was due. The Committee constituted by the OMC examined these claims and
admitted the claims only to an extent of Rs. 3,52,916/-as per its final report dated 7-12-
1979. OMC paid Rs. 3,50,000/- on 4-3-1980. In view of the acknowledgment in writing
on 28-10-1978 and payment of the Rs. 3,50,000/- on 4-3-1980, it can be said that in
regard to the pending claims of the contractor, the limitation stood extended by 3three
years from 4-3-1980 and at all event by three years from 28-10-1978. It is not in dispute
that the contractor issued the notice invoking arbitration on 4-6-1980 and immediately
filed a petition under section 8(2) of the Act for appointment of Arbitrator which was
allowed on 6-10-1980. Therefore, whatever claims were made before the Arbitrator
which was part of the claim of Rs. 50,15,820, was within time, having been made within
three years from 28-10-1978 and 4-6-1980.
17. In regard to the claims aggregating to Rs. 95,96,616/- made in the claim statement
filed before the Arbitrator, only claims aggregating to Rs. 28,32,138 related to and
formed part of the said pending claim of Rs. 50,15,820. The appellant did not make a
claim in regard to the remaining Rs. 21,83,692. Therefore, out of the claim of Rs.
95,96,616 made by the appellant before the Arbitrator, the claim for only Rs. 28,32,138/-
was not barred by limitation. The remaining claims of the appellant aggregating to Rs.
67,64,488/- out of the total of Rs. 95,96,616/- being fresh claims, were not "pending
claims" in respect of which the acknowledgment was made. Therefore the said fresh
claims aggregating to Rs. 67,64,488 made for the first time in the claims statement filed
on 27-6-1986 were clearly barred by limitation.
18

. The learned counsel for the appellant submitted that the limitation would begin to run
from the date on which a difference arose between the parties, and in this case the
difference arose only when OMC refused to comply with the notice dated 4-6-1980
seeking reference to arbitration. We are afraid, the contention is without merit. The
appellant is obviously confusing the limitation for a petition under section 8(2) of the
Arbitration Act, 1940 with the limitation for the claim itself. The limitation for a suit is
calculated as on the date of filing of the suit. In the case of arbitration, limitation for the
claim is to be calculated on the date on which the arbitration is deemed to have
commenced. Section 37(3) of the Act provides that for the purpose of Limitation Act, an
arbitration is deemed to have been commenced when one party to the arbitration
agreement serves on the other party thereto, a notice requiring the appointment of an
arbitrator. Such a notice having been served on 4-6-1980, it has to be seen whether the
claims were in time as on thatAIR 1988 SC 1007
1994 AIR SCW 1335
1999 AIR SCW 421

@page-SC1372
date. If the claims were barred on 4.6.1980, it follows that the claims had to be rejected
by the arbitrator on the ground that the claims were barred by limitation. The said period
has nothing to do with the period of limitation for filing a petition under section 8(2) of
the Act. Insofar as a petition under section 8(2), the cause of action would arise when the
other party fails to comply with the notice invoking arbitration. Therefore, the period of
limitation for filing a petition under section 8(2) seeking appointment of an arbitrator
cannot be confused with the period of limitation for making a claim. The decisions of this
Court in Inder Singh Rekhi v. Delhi Development Authority, (1988) 2 SCC 338; Panchu
Gopal Bose v. Board of Trustees for Port of Calcutta, (1993) 4 SCC 338 and Utkal
Commercial Corporation v. Central Coal Fields, (1999) 2 SCC 571 also make this
position clear.
19. The appellant next contended, relying on section 18 of the Limitation Act, that as
there was acknowledgment of liability in regard to Contract No.30/F-2 in the letter dated
28-10-1978, and the notice invoking arbitration was issued on 4-6-1980 within 3 years
from 28-10-1978, he was at liberty to make any claim in regard to the contract before the
arbitrator, (even though such claims had not been earlier made) and all such claims shall
have to be treated as being within the period of limitation. Such a contention cannot be
countenanced. As noticed above, the cause of action arose on 14-4-1977. But for the
acknowledgment on 28-10-1978, on the date of invoking arbitration (4-6-1980), the
claims would have been barred by time as being beyond the period of limitation. The
limitation is extended only in regard to the liability which was acknowledged in the letter
dated 28-10-1978. It is not in dispute that either on 28-10-1978 or on 4-3-1980, the
contractor had not made the fresh claims aggregating to Rs. 67,64,488 and the question of
such claims made in future for the first time on 27-6-1986, being acknowledged by OMC
on 28-10-1998 did not arise.
20. Another aspect requires to be noticed. The contractor was N.C. Budhraja. The original
claim (which was the subject-matter of letter dated 28-10-1978, subjected to examination
by the Committee as per report dated 7-12-1979, and towards which Rs. 3,50,000/- was
paid) made by the contractor N.C. Budhraja aggregated to Rs. 50,15,820. The appellant
who is his LR cannot for the first time make a fresh claim before the Arbitrator, which
was never made by N.C. Budhraja. The appellant could only pursue the claim made by
N.C. Budhraja, which were pending or subsisting when N.C. Budhraja issued the notice
dated 4-6-1980
21. The arbitrator committed an error apparent on the face of the record and a legal
misconduct in holding that the entire claim was within time. His assumption that if the
application filed by the contractor in 1980 under Section 8(2) of Arbitration Act for
appointment of an Arbitrator was in time, all claims made in the claim statement filed
before the Arbitrator appointed in such proceeding under Section 8(2) are also in time, is
patently erroneous and is an error apparent on the face of the record. The reasoning of the
arbitrator that on account of the formation of the Committee by OMC to scrutinize the
pending claims in pursuance of the CMC's letter dated 28-10-1978, and the payment of
Rs. 3,50,000/- on 4-3-1980 in pursuance of the Committee giving its final report on 7-12-
1979, every claim of the contract including new claims which were made for the first
time in the claim statement filed in 1986 (as contrasted with 'pending claims' considered
by OMC), are not barred by limitation, is also an error apparent in the face of the award.
Under section 18 an acknowledgment in writing extends the limitation. Under section 19
a payment made on account of a debt, enables a fresh period of limitation being
computed. Therefore, the letter of OMC dated 28-10-1978 and the payment of Rs.
3,50,000/- by OMC, would result in a fresh period of limitation being computed only in
regard to the 'existing debt' in respect of which acknowledgment and payment was made.
Admittedly, as at that time, the claim of the contractor was only for a sum of Rs.
50,15,820. Therefore, the letter dated 28-10-1978 and payment on 4-3-1980 extended the
limitation only in respect of the claims which were part of the said claim of Rs.
50,15,820. Therefore, the fresh claims of Rs. 67,64,488/- (out of the total claim of Rs.
95,96,616) is barred by limitation and the award made in that behalf is liable to be set
aside. Consequently, we hold that only that part of the claim before the Arbitrator which
was part of the claim of Rs. 50,15,820/- made by the contractor, that was existing or
pending as on 28-10-1978 and 4-3-1980, namely Rs. 28,32,128 (out of Rs. 95,96,616)
could have been considered
@page-SC1373
by the Arbitrator.
Question (iii) and (iv) :
22. In the claim statement filed before the arbitrator the appellant showed the value of
work done as Rs. 2,45,85,183.89 and the total payments made by OMC as Rs.
1,49,88,566.90. Thus he claimed the balance due as Rs. 95,96,616. Even while
calculating the interest on the amount outstanding, the claimant proceeded on the basis
that he has received in all, Rs. 1,49,88,566.90 from OMC. The prayer before the
arbitrator in the claim statement was for the award of Rs.95,96,616 in regard to the work
done after giving credit of Rs. 1,49,88,566.90. The categorical stand of the contractor and
the appellant all along has been that OMC had paid in all a sum of Rs. 1,49,88,566.90.
But during the course of the arbitration proceedings, the appellant contended that out of
Rs. 1,49,88,566.90 received from OMC and taken into credit towards this contract, a sum
of Rs. 29,86,871/-was being appropriated towards other contracts and therefore the
payments made by OMC towards this contract should be taken as Rs. 120,01,695.90. The
arbitrator has mechanically accepted the said altered stand contrary to the claim statement
and proceeded to determine the amount payable by OMC, by taking the amount paid by
OMC as Rs. 120,01,659.90 towards this contract, even though the claim statement
showing that OMC had paid Rs. 149,88,566.90 remained unaltered. The claim statement
was not amended to show that only Rs. 120,01,659.90 had been received from OMC in
regard to the contract. When the claim made in the claim statement is after adjusting Rs.
149,88,566.90 paid by OMC towards the work, the arbitrator cannot proceed on the basis
that only Rs. 120,01,659.90 was paid towards the work. As a result though the Arbitrator
found that the amount payable towards claims at Items 1 to 16 was only Rs. 365,862.18,
he awarded Rs. 32,83,243/- to the appellant, thereby increasing the liability of OMC by
Rs. 29,86,871/-. By awarding more than what was claimed in the claim statement (by
showing a lesser amount as having been paid by OMC though claim statement showed a
higher amount), the Arbitrator clearly exceeded his jurisdiction. The Arbitrator thus
committed a legal misconduct and the award to that extent is liable to be set aside.
Therefore the amount awarded in respect of claims at items 1 to 16 by the Arbitrator is to
be reduced by Rs. 29,86,871/-.
23. The Arbitrator has exceeded his jurisdiction in another respect. The total claim made
by the contractor before the Arbitration was Rs. 95,96,616/- (excluding interest). But the
amount awarded by the Arbitrator towards the said claim was Rs. 1,02,66,901/36
(excluding interest). Making an award in excess of the claim itself by Rs. 6,70,285 is a
clear act of exceeding the jurisdiction and amounts to a legal misconduct and to that
extent of Rs. 6,70,285/- the award is invalid.
24. In regard to item 35, that is escalation in cost, the claim in the claim statement was at
the rate of 15% for the value of work done in 1972-73, 28.5% in respect of value of work
done in 1973-74 and 32% in respect of work done in 1974-75. But the Arbitrator has
awarded escalation at a flat rate of 32.6% on the entire cost of work done from 1-4-1973
and thereby awarded an escalation in excess of what was claimed. This also amounts to
exceeding the jurisdiction and therefore legal misconduct. The award in excess of what
was claimed was invalid.
25. The award of the Arbitrator in respect of time-barred claim of Rs. 67,64,488 is an
error apparent on the face of the award. Award of amounts in excess of claim (referred to
in paras 22, 23 and 24) clearly amount to exceeding the jurisdiction. All these, that is
awarding amount towards time-barred part of the claim of Rs. 67,64,488, and awarding
amounts of Rs. 29,86,871, Rs.6,70,285 and escalation in cost at a rate more than what is
claimed, are all legal misconducts and the award in regard to those amounts are null and
void. There is, however, some overlapping of the aforesaid amounts.
26. Does it mean that the entire award should be set aside? The answer is no. That part of
the award which is valid and separable can be upheld. That part relates to the claims
which were validly before the Arbitrator, which were part of the existing or pending
claims of Rs. 50,15,820 and which were not barred by limitation. As stated above they
were the claims which were existing or pending in 1978, 1979 and 1980 (considered by
the Committee and payment made by OMC) which were carried before the Arbitrator to
an extent of Rs. 28,32,128. Only the amounts awarded by the Arbitrator against those
claims can be considered as award validly made in Arbitration, falling within jurisdiction.
They are clearly severable from the other portions of the award. The particulars of the
claims and corresponding awards are as follows :
@page-SC1374

Item No. Description Item No. in letter dated 16.11.78 Contractor's claim
originally made 16.11.78 Contractor's claim before Arbitrator Award by Arbitrator
7 Cutting and uprooting trees 5 20,869.32 11,352.00 Nil
17 Extra head lead for 90 M 7(a) 2,61,926.40 28,10,144.00 24,50,042.88
(261,926.40)*
18 Removal of excavated material from edge of quarry 3 3,43,360.58
54,888.60 50,858.60
19 Unmeasured quantity of excavation 4 8,44,360.00 8,48,372.64
6,64,720.00
20 Catch water drain 12 1,55,400.00 27,842.50 27,842.50
21 Removal of slipped earth from side slopes 15 3,42,960.00 1,43,646.00
Nil
26 Extra lift for excavated material dumped at quarry edge 7(e) 42,370.00
3,60,690.49 270,517.88 (42,370.00)*
27 Extra lift measured by surveyor 7(b) 1,25,642.00 13,96,128.67
10,47,096.50 (125,642.00)*
28 Idle labour (non supply of plans) 9(a) 1,45,575.00 1,45,575.00 Nil
29 Idle labour (for want of site) 9(c) 75,450.00 76,850.00 Nil
30 Idle labour (due to stoppage of work and restriction of working area) 9(b)
and (d) 2,61,205.00 3,89,288.00 194,644.00
31 Repairing road damaged by cyclone 10(iii) 10,640.00 10,640.00 Nil
32 Reconstruction of Embasskent 10(vii) 25,370.00 25,370.00
25,370.00
33 Wire fencing 13 1,05,000.00 27,315.00 Nil
34 Electricity supply to work site and colony 6(b)
72,000.00 72,000.00 Nil
28,32,128.30
13,93,373.50

[Note : The figures shown by (*) in the column 'Award by Arbitrator', refer to the
maximum that could have been awarded by the Arbitrator having regard to claim that was
not barred by limitation].
Thus the total amount awarded by the Arbitrator against claims which were not barred by
limitation was only Rs. 13,93,373.50. The award to this extent is not open to challenge.
This part of the award does not suffer from any legal misconduct. There is also no error
apparent on the face of the award in respect of the amount. It is not open to challenge.
27

. The scope of interference is limited. In Hindustan Construction Co. Ltd. vs. Governor of
Orissa, (1995) 3 SCC 8, this Court held : 1995 AIR SCW 1575, (Para 10)

"It is well known that the court while considering the question whether the award should
be set aside, does not examine that question as an appellate court. While exercising the
said power, the court cannot reappreciate all the materials on the record for the purpose of
recording a finding whether in the facts and circumstances of a
@page-SC1375
particular case the award in question could have been made. Such award can be set aside
on any of the grounds specified in section 30 of the Act."

In Hindustan Tea Co. vs. M/s. K. Sashikant and Co., 1986 (Supp) SCC 506, this Court
observed thus : AIR 1987 SC 81, (Para 2)

"The Award is reasoned one. The objections which have been raised against the Award
are such that they cannot indeed be taken into consideration within the limited ambit of
challenge admissible under the scheme of the Arbitration Act. Under the law, the
Arbitrator is made the final arbiter of the dispute between the parties. The award is not
open to challenge on the ground that the Arbitrator has reached a wrong conclusion or has
failed to appreciate facts."
Therefore, the Award of the Arbitrator has to be upheld to an extent of Rs. 13,93,373.50.
28. In view of the foregoing, we allow these appeals in part, set aside the judgment of the
High Court and direct a decree in terms of the award for a sum of Rs. 13,93,373.50 with
interest at the rate of 12% p.a. from 1-8-1977 to date of award (28-11-1986) and at the
rate of 6% p.a. thereafter, that is from 29-11-1986 till date of payment. Parties to bear
their respective costs.
Appeal partly allowed.
AIR 2008 SUPREME COURT 1375 "State of Maharashtra v. Devahari Devasingh
Pawar"
(From : Bombay)
Coram : 2 G. P. MATHUR AND AFTAB ALAM, JJ.
Criminal Appeal No. 122 of 2008 (arising out of SLP (Cri.) No. 1268 of 2006), D/- 18 -1
-2008.
State of Maharashtra v. Devahari Devasingh Pawar and Ors.
Criminal P.C. (2 of 1974), S.197 - SANCTION FOR PROSECUTION - SCREENING
OF OFFENDER - DOCUMENTS - NEGLIGENCE - DRUGS, COSMETICS AND
MAGIC REMEDIES - Sanction for prosecution - When necessary - Supply of HIV
contaminated blood by blood bank to Govt. Hospital - Charge u/S.201, u/S.204, u/S.269,
Penal Code - Allegations against accused- doctor about tampering with entries made in
official registers, tearing of pages from different official registers and stowing them away
in his house - Such acts had no nexus or connection to discharge of his official duties -
Sanction for prosecution on basis of such allegations - Not required - Question whether
sanction was required for prosecution of other offences under Drugs Act left open to be
decided by trial Court.
Cri. Revn. Appln. Nos. 50 and 87 of 2004, D/-20-04-2005 (Bom.), Reversed.
Penal Code (45 of 1860), S.201, S.204, S.269. (Paras 13, 15)
Cases Referred : Chronological Paras
2005 AIR SCW 5875 : AIR 2006 SC 336 : 2005 Cri LJ 5068 (Foll.) 12
2000 AIR SCW 3725 : AIR 2000 SC 3187 : 2000 Cri LJ 4631 (Disting.) 8, 11
Shekhar Naphade, Sr. Advocate, Ravindra Keshavrao Adsure, for Appellant; Vivek K.
Tankha, Sr. Advocate, Subramonium Prasad, Jay Kishore Singh, Vivek Gupta, Anurag
Sharma, Prashant Kumar, and Arjun Bansal, for Respondents.
Judgement
1. ORDER :- Leave granted.
2. Heard Mr. Shekhar Naphade, learned senior counsel for the appellant, Mr.
Subramonium Prasad, learned counsel for Ku. Pradnya Sudhakar Phadnavis, respondent
No. 3 and Mr. Vivek Tankha, learned senior counsel for Dr. Prakashchandra, respondent
No. 7. None of the other respondents are represented before us despite service.
3. This appeal is directed against the order dated April 20, 2005 passed by the High Court
of Bombay, Nagpur Bench in Criminal Revision Application No. 50 of 2004 and
Criminal Application No. 87 of 2004 by which the High Court quashed the proceedings
of Criminal Case No. 48 of 1994 pending before the Additional Chief Judicial Magistrate,
Nagpur on the ground that there was no sanction for prosecution of the accused
(respondents before this Court) as required under Section 197 of the Criminal Procedure
Code (hereinafter referred to as "the Code").
4. It came to light that HIV contaminated blood was supplied to the Government Medical
College and Hospital, Nagpur by its blood bank and as a result, some patients who were
given blood transfusion had tested HIV positive. After making some preliminary inquiry,
the Drugs Inspector, Nagpur lodged a first information report with the police. The police
investigation led to further charges of a serious nature coming to light. It is stated on
behalf of the appellant that in the course of investigation it was found that entries in
@page-SC1376
the registers maintained at the blood bank were tampered with and "corrections" were
made without any initials to certify those corrections. More seriously some pages were
missing from the Donor Register and likewise some pages were torn off from the Issue
Register for dates 10-4-1993 to 12-5-1993. Further investigation revealed that the pages
from the official registers were torn by Dr. P.P. Sancheti (accused No. 1-respondent No.
7) who carried away those pages with him on his transfer on 2-6-1993. He did not return
the torn pages despite a number of letters sent by the departmental authorities. After some
effort, the police was able to find out accused No. 1-respondent No. 7 and in course of
search of his house the documents, namely; pages from the Issue Register for dates 10-4-
1993 to 12-5-1993 and pages from the Donor Register relating to blood units 2478 to
2510 were recovered.
5. Apart from the above allegations that prima facie constitute different offences
including forgery, causing disappearance of evidence of offence, destruction of
documents to prevent its production as evidence etc. punishable under the Indian Penal
Code (hereinafter referred to as "I.P.C."), several acts of omission and commission of
culpable nature also came to light in the working of the blood bank. On conclusion of
investigation the police submitted charge-sheet against the accused under Sections 201,
204 and 269/34 of I.P.C. Though the investigation also revealed interpolations in the
official records, no charge-sheet was submitted for the offence of forgery.
6. It appears that the Drugs Inspector took the view that the acts of omission and
commission in the working of the blood bank also gave rise to offences under the Drugs
and Cosmetics Act, 1940 and the Rules framed thereunder (hereinafter referred to as "The
Drugs Act"). He, accordingly, filed a complaint under Section 21 of the Drugs Act for
prosecuting the accused 1 to 7 (respondent Nos. 1 to 7) for offences punishable under
Section 18(a)(i) read with Sections 27 and 17-A and C of the Drugs Act.
7. Here, it needs to be stated that accused Nos. 1 and 2 in the complaint case (respondent
Nos. 6 and 7) were doctors; accused No. 1-respondent No. 7 being the Blood Transfusion
Officer and accused Nos. 3 to 7 (respondent Nos. 1 to 5) were Technicians in the
Government Medical College and Hospital.
8

. On an application made by the Drugs Inspector, the complaint filed by him was
amalgamated with the earlier police case and resultantly the learned Additional Chief
Judicial Magistrate took cognizance under Sections 269, 201 and 204 read with Section
34, I.P.C. and Section 18(a)(i) read with Sections 27 and 17-A and C of the Drugs Act and
summoned the seven accused to face trial. The trial did not make any progress for
sometime and on 31-1-2001, the accused filed a petition for quashing the proceedings as
the prosecution had not produced sanction from the State Government. The learned
Additional Chief Judicial Magistrate allowed the petition and by order dated 10-4-2001
quashed the proceedings of the case. Against the order passed by the learned Additional
Chief Judicial Magistrate, the State preferred Criminal Revision No. 445 of 2001 before
the Sessions Judge, Nagpur. The learned Sessions Judge, on hearing the parties, allowed
the revision and set aside the order of the trial court. He directed the trial court to proceed
with the trial leaving the question of sanction open to be adjudicated at the time of
conclusion of trial. Against the order of the Sessions Court, the respondents moved the
Nagpur Bench of the High Court in Criminal Revision Application No. 50 of 2004. The
High Court as noted above set aside the order of the Sessions Judge and quashed the
proceedings on the ground that the prosecution had not produced the order of sanction
from the State Government before the Trial Court. In taking the view that the prosecution
could not proceed against the accused for want of Government sanction, the High Court
mainly relied upon the decision of this Court in Abdul Wahab Ansari v. State of Bihar,
2000 (8) SCC 500. 2000 AIR SCW 3725
9. Mr. Shekhar Naphade, learned senior counsel for the appellant submitted that the order
of the High Court was not sustainable in law for more reasons than one. He stated that
respondents 1 to 5 (accused Nos. 3 to 7) before the trial court were technicians and for
their removal from service, there was no requirement of sanction of the State
Government. They could simply be removed by the Dean of the Medical College and
Hospital who was their appointing authority. The High Court, thus, overlooked that
insofar as respondent Nos. 1 to 5 are concerned, there was no application of Section 197
of the Code.
@page-SC1377
10. Mr. Subramonium Prasad, learned counsel appearing for respondent No. 3 submitted
that his client though a technician was nevertheless entitled to the protection of Section
197 of the Code if that protection was extended to the two doctors, accused in the case.
Learned counsel also submitted that insofar as respondent No. 3 was concerned, there
was no allegation against her in regard to any offence under the Penal Code and as a
matter of fact, she was not even an accused in the police case. She was named as one of
the accused only in the complaint filed by the Drugs Inspector relating to the offences
under the Drugs Act. In that regard, learned counsel submitted that she was not acting
individually on her own but she was part of a team head by Dr. P.P. Sancheti, accused No.
1 respondent No. 7 and in case the protection of Section 197 of the Code was given to Dr.
P.P. Sancheti, having regard to the object and purpose of the provision, there was no
reason why the same protection should not be made available to her and to other
technicians who were simply members of the team.
11

. We do not wish to make any comment on the submissions made by Mr. Subramonium
Prasad as in our considered opinion, the provisions of Section 197 of the Code had no
application even in regard to the two accused doctors (respondent Nos. 6 and 7) at least
insofar as the offences under the Penal Code are concerned. As noted above, the High
Court has primarily relied upon the decision of this Court in the case of Abdul Ansari
(supra). In that case, in the course of removal of encroachments, the Duty Magistrate had
given orders for opening fire in order to disperse a fully armed mob threatening to
overrun the police party. In the police firing some causalities had taken place and
prosecution was initiated under different sections of Penal Code including Section 302 of
I.P.C. and Section 27 of the Arms Act. It was in those facts that this Court held that the
occurrence had taken place in the discharge of official duties of the accused and hence,
the prosecution could not proceed for want of sanction by the State Government. Here the
facts are entirely different and we see no application of the decision in the case of Abdul
Ansari (supra). 2000 AIR SCW 3725

12

. In Romesh Lal Jain vs. Naginder Singh Rana and others (2006) 1 SCC 294, this Court
held and observed as under : 2005 AIR SCW 5875, (Para 37)
"33. The upshot of the aforementioned discussions is that whereas an order of sanction in
terms of Section 197 Cr.P.C. is required to be obtained when the offence complained of
against the public servant is attributable to the discharge of his public duty or has a direct
nexus therewith, but the same would not be necessary when the offence complained of
has nothing to do with the same. A plea relating to want of sanction although desirably
should be considered at an early stage of the proceedings, but the same would not mean
that the accused cannot take the said plea or the court cannot consider the same at a later
stage. Each case has to be considered on its own facts. Furthermore, there may be cases
where the question as to whether the sanction was required to be obtained or not would
not be possible to be determined unless some evidence is taken, and in such an event, the
said question may have to be considered even after the witnesses are examined."
13. In light of the above passage, we fail to see how tampering with the entries made in
official registers, tearing of pages from different official registers and stowing them away
in one's house can be related to the discharge of official duties. We do not have the
slightest doubt that the allegations made against the accused related to acts that had no
nexus or connection to the discharge of their official duties and, therefore, their
prosecution on those allegations had no need of any sanction under Section 197 of the
Code.
14. Mr. Tankha, learned senior counsel, however, submitted that other alleged offences
under the Drugs Act undoubtedly related to the discharge of official duties by accused
No. 1-respondent No. 7 and, therefore, the prosecution for those offences was not
permissible in the absence of sanction under Section 197 of the Code.
15. As shown above, a substantial part of cases against the accused does not require any
sanction for their prosecution. The facts of the case do not warrant any piecemeal
quashing or discharge of the accused. We, therefore, consider it appropriate and just that
the trial of the accused should be allowed to proceed without any hindrance. After the
evidence of two the sides are led, the trial court will be in a better position to judge
whether or not any offences are made
@page-SC1378
out under the Drugs Act and; whether or not any offences, if are made out, could be said
to have been committed by the accused in discharge of their official duties and whether or
not any sanction of the State Government was required for their prosecution for those
offences and what would be the effect of non-production of sanction by the prosecution.
The question of sanction for prosecution under the Drugs Act is thus left open to be
decided by the trial court at the end of the trial. In the result, the order of the High Court
coming under appeal is set aside and the matter is remitted to the trial court with the
aforesaid directions and observations. In the result, this appeal stands allowed.
Appeal allowed.
AIR 2008 SUPREME COURT 1378 "Pratap Lakshman Muchandi v. Shamlal Uddavadas
Wadhwa"
(From : Karnataka)
Coram : 2 A. K. MATHUR AND MARKANDEY KATJU, JJ.
Civil Appeal No. 666 with 728 of 2002, C.P. (C) Nos. 52 of 2006 in CA No. 728 of 2002
and 58 of 2006 in CA. No. 666 of 2002, D/- 18 -1 -2008.
Pratap Lakshman Muchandi and Ors. v. Shamlal Uddavadas Wadhwa and Ors.
(A) Specific Relief Act (47 of 1963), S.20 - CONTRACT - DECREE - AGREEMENT
TO SELL - Decree for specific performance - Discretionary relief - Unfair advantage -
Moulding of relief - Agreement to sell joint family property for necessity - Litigation
pending for years - Property prices soaring - To settle equities Court directed purchaser to
pay higher price to seller for executing decree. (Para 8)
(B) Specific Relief Act (47 of 1963), S.21 - AGREEMENT TO SELL - ADVERSE
POSSESSION - Suit for specific performance of agreement to sell - Property in question
in possession of third party - Third party claiming title by adverse possession unable to
prove it - Court to prevent another round of litigation directed third party to handover
possession to seller. (Para 9)
Cases Referred : Chronological Paras
2005 AIR SCW 5622 : AIR 2006 SC 145 : 2006(1) AIR Jhar R 121 : 2006 (1) AIR Bom
R 151 : 2006 (1) ALJ 284 : 2006 (1) AIR Kar R 74 5
(2004) 8 SCC 689 5
2001 AIR SCW 2731 : AIR 2001 SC 2446 5
1995 AIR SCW 2528 : AIR 1995 SC 1607 5
AIR 1986 Bom 15 5, 9
(1824-34) All ER 317 5, 9
K. Ramamoorthy and Raju Ramachandran, Sr. Advocates, R.S. Hegde, Chandra Prakash,
Rahul Tyagi, J.K. Nayyar, K.K. Mani and P.P. Singh, for Appellants; Mathai M. Paikeday,
Sr. Advocate, Shishir Pinaki, Sanjay Jain and P. Narasimhan, for Respondents.
Judgement
A. K. MATHUR, J. :- Both the Civil Appeals arise against the order passed by the
Karnataka High Court in RFA Nos.290 and 311 of 1993 dated 17.12.1999. Therefore,
both the appeals are disposed of by a common order.
2. The brief facts which are necessary for the disposal of these appeals are that a suit was
filed on the basis of an agreement to sell dated 24-4-1982 for a consideration of Rs.
1,20,000/- for property, namely, open space with some dilapidated room bearing CTS No.
4094/18/2 admeasuring 472 square yards, College Road, Belgaum. The agreement was
executed by the first defendant as the 'Kartha' of Hindu joint family along with other
defendant Nos.2 to 4. A sum of Rs. 10,000/- was paid as advance and the agreement was
to be concluded within six months. As the defendants did not execute the sale deed within
the stipulated time, a suit was filed by the plaintiff after giving notice dated 10.5.1983 for
enforcement of the agreement to sell. The defendant Nos. 1 to 5 also filed a suit being
O.S.No.236 of 1982 for injunction against defendant Nos.6 to 15 and took a plea that
because of the pendency of their suit, they could not execute the sale deed and they
would execute the sale deed after decree in their favour was passed. The plaintiff
suspected their movements and, therefore, he filed the present suit. Defendant Nos. 1 to 3
filed a common written statement admitting the joint Hindu family consisting of
defendant Nos. 1 to 4. But they denied that the 1st defendant was the Kartha of the
family. They admitted that the suit property was an ancestral property and they were the
absolute owners. They also denied the agreement to sell and receipt of the advance. They
further took a plea that they agreed to sell the property for a sum of Rs. 1,70,000/- at the
first instance and the deed of the agreement was typed and signed by the parties and the
earnest money in sum of Rs. 10,000/- was paid and
@page-SC1379
they were willing to sell the property for a sum of Rs. 1,70,000/- and as the plaintiff did
not pay the balance sum, therefore, the sale deed could not be executed. The defendant
No.4 was a minor when the suit was instituted, but became major during the pendency of
the suit and he denied that the defendant No.1 was his natural guardian. The defendant
No. 5 also claimed 1/5th share in the property. The defendant No. 1 died during the
pendency of the suit and his other daughter was brought on record as defendant No. 1(a).
She also filed a written statement denying the agreement of sale. Defendant No.6
contended that there was no collusion between the defendant Nos. 6 to 15 and defendant
Nos. 1 to 4. They also contended that the agreement cannot be enforced as against them
as defendant Nos. 1 to 5 were never in possession of the suit property. Defendant Nos.6
to 15 claimed the ownership by way of adverse possession and claimed to be in such
exclusive possession from the year 1957 onwards with the knowledge of defendant Nos.
1 to 5. Therefore, it was contended that the agreement of sale was not enforceable
because of the laches on the part of the plaintiff. On the basis of these pleadings, nine
issues were framed and then three more additional issues were framed. The Trial Court
after analyzing the evidence decreed the suit and directed the defendant Nos. 1 (a) to 5 to
execute the sale deed in favour of plaintiff by receiving the balance consideration of Rs.
1,10,000/- and hand over possession, at the same time, a decree was passed evicting the
defendant Nos.6 to 15 from the premises in question. The Trial Court further directed
defendant Nos. 6 to 15 to hand over the possession to the plaintiff. Aggrieved against this
judgment and decree passed by the Trial Court, two appeals were preferred before the
High Court. Both the appeals were taken up together. The grievance of defendant Nos. 1
to 5 was that the agreement of sale was not proved and appeal by another batch of
persons who were directed to be evicted from the premises in question and to hand over
the possession, was filed, i.e. Appeal No. 311 of 1983 and Appeal No.290 of 1993. Both
these appeals were tagged together.
3. The High Court again reviewed the evidence and while hearing the appeals, it felt that
document executed by P.W. 1 contained some corrections or erasure. Consequently, the
document was sent for the expert opinion and after receipt of the report of the Assistant
Director (questioned document), Forensic Science Laboratory, Bangalore, evidence of
erasure was found and subsequent typing of figures of Rs. 1,20,000/-was detected. Both
the parties were directed to file their objection to the report of the Handwriting Expert.
The High Court framed following two questions, viz.;
"(i) Whether the agreement of sale is true and binding on all the defendants ?
(ii) Whether the defendants 6 to 15 perfected their title over suit property by way of
adverse possession?"
4. The High Court, after review of the evidence came to the conclusion that because of
the legal necessity as admitted by the defendants, an agreement of sale was executed for
the aforesaid property and a sum of Rs. 10,000/- was taken as advance. The High Court
also observed that defendant No. 1 was the Kartha of the family, who died and it was not
open to his sons to challenge that there was no family necessity for sale of the property.
So far as the agreement to sell was concerned, the High Court also affirmed the finding of
the trial court and did not find any reason to take a different view of the matter. The High
Court also affirmed that in fact, the agreement of sale was for a sum of Rs. 1,20,000/- and
not for Rs. 1,70,000/- as alleged. So far as the possession by the defendant Nos. 6 to 15
was concerned, the Trial Court as well as the High Court affirmed that the plea of adverse
possession was very vague and these persons were carrying on timber business in suit
property and it was very difficult to hold that they perfected their title by way of adverse
possession. It was also observed that these persons were in permissive possession. It was
also found by both the Courts below that there was no evidence to show that the title was
perfected by way of adverse possession. Consequently, the High Court confirmed the
finding of the Trial Court. Aggrieved against this judgment, two appeals were filed and
they were tagged together, and are being disposed of by this common order.
5

. Learned counsel for the appellants submitted that the findings given by both the Courts
below cannot be accepted and in support thereof, learned counsel invited our attention to
a number of decisions of this Court i.e. V. Pechimuthu v. Gowrammal 2001 AIR
SCW 2731
1995 AIR SCW 2528
2005 AIR SCW 5622

@page-SC1380
[(2001) 7 SCC 617], Swarnam Ramachandran (Smt.) and Anr. v. Aravacode Chakungal
Jayapalan [(2004) 8 SCC 689], S.V.R. Mudaliar (Dead) by L.Rs. and Ors. v. Rajabu F.
Buhari (Mrs.) (Dead) by L.Rs. and Ors. [(1995) 4 SCC 15] and P. C.Varghese v. Devaki
Amma Balambika Devi and Ors. [(2005) 8 SCC 486]. Mr. K.Ramamoorthy, learned
senior counsel appearing for the appellants in Civil Appeal No. 666 of 2002 submitted
that both the Courts below could not have passed an eviction decree against the
appellants in these very proceedings as they were claiming the property by way of
adverse possession, and in support thereof, he has invited our attention to a decision of
Bombay High Court in Mohd. Hanif (deceased by L.Rs.) and Ors. v. Mariam Begum and
Ors. [AIR 1986 Bom 15] and an English decision in Tasker v. Small (1824-34 All ER
317].
6. We have heard learned counsel for the parties and perused the record. As per the
findings given by both the Courts below it is clear that the agreement to sell was entered
into for family necessity and the same was agreed by the father of the defendant though
the father died during the course of the pendency of the suit. Therefore, he could not be
examined. Learned counsel has submitted that the appreciation done by both the Courts
below is not correct and in fact the property was not agreed to be sold for Rs. 1,20,000/-
but the consideration money was Rs. 1,70,000/- and the appellants themselves were not
willing to pay the remaining amount. Hence he submitted that the agreement to sell
cannot be executed.
7. We have examined the record and found that as per the evidence on record what is
apparent is that the agreement to sell in question was for the purpose of family necessity
only and it does not lie in the mouth of the sons to deny the agreement to sell for which a
sum of Rs. 10,000/- was already received. After going through the evidence also we are
of opinion that the Courts below have correctly appreciated the testimony and rightly
reached the conclusion that the agreement to sell was for Rs. 1,20,000/- only. So far as
the allegation of interpolation in the document in question i.e. agreement to sell was
concerned, it was sent for examination by the Handwriting expert, and the report of the
expert was received and the same was accepted. The opinion of expert was that there is
erasure but not tampering with the document. The document in question is genuine and
has been rightly acted upon by both the Courts below. In this connection, learned counsel
invited our attention to various decisions referred to above but that does not make any
difference in the matter because factually we are satisfied that the agreement to sell was
executed for family necessity. Therefore, the various decisions referred to by learned
counsel for the appellants do not take the case of the appellants any far. Hence we are of
opinion that the agreement to sell was executed for family necessity and the appellants
cannot get out of it.
8. But at the same time it is also true that the agreement to sell was executed way back in
the year 1982. Since after 1982 much water has flown under the bridge, the value of the
real estate has shot up very high, therefore, while exercising our jurisdiction under
Section 20 of the Specific Relief Act, 1963 we would like to be equitable and would not
allow the sale of property to be executed for a sum of Rs. 1,20,000/-. The litigation has
prolonged for almost 25 years and now at last reached at the end of the journey.
Therefore, we have to settle the equity between the parties. We hold that the agreement to
sell was genuine and it was executed for bona fide necessity but because of passage of
time we direct that the respondents shall pay a sum of Rs. 5 lacs in addition to Rs.
1,10,000/- as out of Rs. 1,20,000/-, Rs. 10,000/- has already been paid as advance. On
receipt of Rs. 1,10,000/- and Rs.5 lacs [Rs. 6,10,000/-] the appellants shall execute the
agreement to sell for the property in question.
9. Mr. Ramamoorthy, learned senior counsel for the appellants in C.A.No. 666 of 2002
submitted that in this appeal an order of eviction cannot be passed and in support of that
invited our attention to a decision of Bombay High Court in Mohd. Hanif (deceased by
L.Rs.) and Ors. v. Mariam Begum and Ors. [AIR 1986 Bom 15] and English decision in
Tasker v. Small [1824-34 All ER 317]. It is true that the appellants in this appeal claimed
the property in question by way of adverse possession but neither before the trial court
nor before the High Court the appellants could show any justification for the possession
of the property in question. We also asked Mr. Ramamoorthy under what
@page-SC1381
legal sanction the appellants are in possession of the premises in question. He has failed
to point out anything except by way of permissible possession by the appellants in
C.A.No.666 of 2002. Therefore, the occupation of these appellants in C.A.No.666 of
2002 was at best a permissible possession and now that we are enforcing the agreement
to sell and direct the appellants in C.A.No. 728 of 2002 to execute the sale deed in respect
of the property in question in favour of the respondent-plaintiff, we cannot permit the
appellants to continue in possession of the property in question. Apart from this in order
to put quietus to the whole litigation it would be just and proper that the appellants in
C.A.No.728 of 2002 should be directed to hand over the vacant possession of the
property in question to the respondent-plaintiffs on payment of a sum of Rs. 6,10,000/-
[Rs. 5,00,000/- + Rs. 1,10,000/-] to the appellants. We cannot leave the matter again for
another round of litigation as otherwise the respondent-plaintiff will have to file another
case for taking possession of the property in question and it will take another decade or
so. Therefore, in order to do complete justice, it is directed that the appellants in
C.A.No.728 of 2002 shall hand over the possession of the property in question to the
respondent-plaintiffs in the event of the respondent-plaintiffs paying a sum of Rs.
1,10,000/-, the original amount agreed in the agreement for sale and over and above a
sum of Rs. 5,00,000/- i.e. Rs. 6,10,000/- within a period of three months from today and
on receipt of the aforesaid amount, the appellants in C.A.No.728 of 2002 shall hand over
the possession of the premises in question. In case the appellants fail to hand over the
possession of the property in question, the respondent-plaintiff may resort to the help of
the police authorities for taking vacant possession of the property in question.
10. As a result of our above discussion, both the appeals are disposed of with no order as
to costs.
11. Since we have disposed of the civil appeals as indicated above, the contempt petitions
are also disposed of in the light of the above order.
Order accordingly.
AIR 2008 SUPREME COURT 1381 "Kunju v. State of Tamil Nadu"
(From : Madras)*
Coram : 2 Dr. A. PASAYAT AND AFTAB ALAM, JJ.
Criminal Appeal No. 112 of 2008 (arising out of SLP (Cri.) No. 3221 of 2006), D/- 16 -1
-2008.
Kunju alias Balachandran v. State of T.N.
Evidence Act (1 of 1872), S.134 - WITNESS - MURDER - EVIDENCE - Murder case -
Sole eye-witness - Testimony of - Conviction can be based on it - Test is whether
evidence has a ring of truth, is cogent, credible and trustworthy or otherwise -
Corroboration would be required only if witness is neither wholly reliable nor wholly
unreliable.
AIR 1957 SC 614 and AIR 1994 SC 1251, Relied on. (Para 11)
Cases Referred : Chronological Paras
2003 AIR SCW 6026 : AIR 2004 SC 552 : 2004 Cri LJ 819 10
1994 AIR SCW 564 : AIR 1994 SC 1251 : 1994 Cri LJ 1106 (Rel. on) 9
AIR 1957 SC 614 : 1957 Cri LJ 1000 (Rel. on) 8, 9
Shashi Bhushan Kumar, for Appellant; V. Kanakaraj, Sr. Advocate, S. Joseph Aristotle, S.
Prabhu Ramasubramanian and V.G. Pragasam, for Respondent.
* Cri. Appeal No. 28 of 2003, D/- 30-6-2005 (Mad.)
Judgement
Dr. ARIJIT PASAYAT, J. :- Leave granted.
2. Challenge in this appeal is to the judgment of a Division Bench of the Madras High
Court dismissing the appeal filed by the appellant who was convicted for offence
punishable under Section 302 of the Indian Penal Code, 1860 (in short the 'IPC') and
sentenced to undergo imprisonment for life and to pay a fine of Rs.200/- with default
stipulation. The conviction was recorded and sentence imposed by learned Additional
Sessions Judge Gobichettipalayam in Sessions Case No. 59 of 2002 dated 03-09-2002.
3. Factual background in a nutshell is as follows :
The accused Kunju alias Balachandran is the resident of Ceylon Refugee Camp at
Bhavanisagar. Sudhakaran (hereinafter referred to as the 'deceased') also was residing
@page-SC1382
in the same Refugee Camp. Prior to the date of occurrence, the parents of the accused
arranged to get his marriage with Selvi (PW-5). Betrothal ceremony was also over. The
deceased fell in love with Selvi (PW-5). Two days prior to the date of occurrence, the
deceased met PW-5 and offered flower to her. PW-5 refused to receive the flower and
told him that already her betrothal was held with the accused. This incident was informed
to the accused.
On the date of occurrence i.e. on 28.2.2001, at about 6.50 p.m., the deceased along with
two other friends, Stephen (PW-1) and Siva (PW-2) was proceeding to take bath at A.R.S.
Canal. The accused came there and restrained the deceased by catching hold of his arm
and dragged and assaulted him while abusing him in filthy language. He took out a Vettu
Aruval (M.O.I.) from his hip and gave cuts on various parts of the body of the deceased.
The deceased fell down, but the accused continued to inflict injuries all over the body.
PWs 1 and 2, the other witnesses made a hue and cry. The people also gathered there.
Therefore, the accused ran away from the scene place.
On witnessing this incident, P.W.2 immediately went to the house of the deceased and
informed P.W.3, the brother of the deceased. P.W.3 came to the scene and found that his
brother was gasping for his life.
Thereafter, PW-3 arranged for taking the injured to the Bhavanisagar Government
Hospital, where first aid was given. Then, on receipt of the message from hospital, P.W. 7
sub-Inspector of Police came to the hospital, recorded the statement (Ex.P21) from PW-1.
The case was registered for the offences punishable under Sections 341 and 307, IPC.
Since the injured was in a serious condition, he was taken to Coimbatore Government
Hospital by PW.3. On the way, the injured died. On receipt of the death information
Ex.P26, the Inspector of Police P.W. 18 took up investigation and altered the case into
one under Sections 341 and 302, IPC.
4. After that investigation charge sheet was filed. Since the accused pleaded innocence,
trial was conducted. Learned trial Court considered the evidence on record and placing
reliance on the evidence of PW2 recorded the conviction and imposed sentence as noted
above. It is relevant to note that PW 1 who was the author of the First Information Report
(in short the 'FIR') resiled from his statement recorded during investigation. The trial
Court noted that though to certain extent PW 1 departed from his statement during
investigation, he accepted that three persons including the deceased and PW2 had gone to
take bath but at that time the accused also came bare. Before the High Court, the stand
taken before the trial Court was reiterated. But the High Court did not find any substance
and dismissed the appeal.
5. In support of the appeal learned counsel for the appellant submitted that the motive for
the crime has not been established as the evidence of the girl does not show that she was
being harassed by the deceased. Additionally, it is submitted that after PW 1 did not fully
support the prosecution version and on the testimony of a single witness i.e. PW 2, the
conviction should not have been recorded.
6. Learned counsel for the respondent supported the impugned judgment.
7. As rightly noted by the trial Court and the High Court even though PW 1 did not
support the prosecution version in toto, yet his evidence lent corroboration to the
evidence of PW2 that deceased, PW 2 and another had gone to take bath and at that time
the accused came there. The evidence of PW 2 has not been shaken although he was cross
examined at length.
7a. It is necessary to refer to the pivotal argument of the appellant's learned counsel that
PW-2 is the sole eyewitness in the present case and no conviction should be based on the
testimony of such an eyewitness who cannot be described as wholly reliable.
8. In Vadivelu Thevar v. State of Madras (AIR 1957 SC 614) this Court had gone into this
controversy and divided the nature of witnesses in three categories, namely, wholly
reliable, wholly unreliable and lastly, neither wholly reliable nor wholly unreliable. In the
case of the first two categories this Court said that they pose little difficulty but in the
case of the third category of witnesses, corroboration would be required. The relevant
portion is quoted as under: (AIR p. 619, paras 11-12)
"Hence, in our opinion, it is a sound and well-established rule of law that the court is
concerned with the quality and not with the quantity of the evidence necessary for
proving or disproving a fact. Generally speaking
@page-SC1383
oral testimony in this context may be classified into three categories, namely :
(1) Wholly reliable.
(2) Wholly unreliable.
(3) Neither wholly reliable nor wholly unreliable.
In the first category of proof, the court should have no difficulty in coming to its
conclusion either way it may convict or may acquit on the testimony of a single witness,
if it is found to be above reproach or suspicion of interestedness, incompetence or
subornation. In the second category, the court equally has no difficulty in coming to its
conclusion. It is in the third category of cases, that the court has to be circumspect and
has to look for corroboration in material particulars by reliable testimony, direct or
circumstantial. There is another danger in insisting on plurality of witnesses. Irrespective
of the quality of the oral evidence of a single witness, if courts were to insist on plurality
of witnesses in proof of any fact, they will be indirectly encouraging subornation of
witnesses.
9

. Vadivelu Thevar case (supra) was referred to with approval in the case of Jagdish Prasad
v. State of M.P. (AIR 1994 SC 1251). This Court held that as a general rule the court can
and may act on the testimony of a single witness provided he is wholly reliable. There is
no legal impediment in convicting a person on the sole testimony of a single witness.
That is the logic of Section 134 of the Indian Evidence Act, 1872 (in short "the Evidence
Act"). But, if there are doubts about the testimony the courts will insist on corroboration.
It is for the court to act upon the testimony of witnesses. It is not the number, the quantity,
but the quality that is material. The time-honoured principle is that evidence has to be
weighed and not counted. On this principle stands the edifice of Section 134 of the
Evidence Act. The test is whether the evidence has a ring of truth, is cogent, credible and
trustworthy, or otherwise. 1994 AIR SCW 564

10

. The above position was highlighted in Sunil Kumar v. State Govt. of NCT of Delhi
[(2003) 11 SCC 367]. 2003 AIR SCW 6026

11. On analysis of the factual scenario and on applying the principles of law stated above,
the inevitable conclusion is that the appeal is without merit, deserves dismissal, which we
direct.
Appeal dismissed.
AIR 2008 SUPREME COURT 1383 "Benjamin v. State"
(From : Madras)
Coram : 2 S. B. SINHA AND H. S. BEDI, JJ.
Criminal Appeal No. 76 of 2006 (arising out of SLP (C) No. 863 of 2007), D/- 11 -1
-2008.
Benjamin v. State.
Penal Code (45 of 1860), S.300 - MURDER - EVIDENCE - Intention to kill -
Determination of - Number of injuries - Not by itself sufficient - Considering number of
injuries, nature of such injuries and medical evidence corroborating ocular evidence,
held, that there could not be any doubt that accused had intention to kill.
Evidence Act (1 of 1872), S.114. (Paras 17, 18)

T. Raja, for Appellant; V. Kanakaraj, Sr. Advocate, V.G. Pragasam, S. Joseph Aristotle, S.
Prabhu Ramasubramanian, for Respondent.
Judgement
S. B. SINHA, J. :-Leave granted.
2. This appeal is directed against the judgment and order dated 18th January, 2006 passed
by a Division Bench of the Madras High Court in Criminal Appeal No. 142 of 1997
whereby and whereunder the appeal filed by the appellant against an order of conviction
dated 30th January, 1997 passed by the Principal Sessions Judge, Dindigul Anna District,
was dismissed.
3. Appellant with Jesu Raj and Arokiyam were proceeded against for commission of an
offence under Section 302 of the Indian Penal Code. The first and second accused were
charged for commission of an offence punishable under Section 302 of the Indian Penal
Code whereas the third accused was charged for commission of an offence under Section
302 of the Indian Penal Code read with Section 34 thereof.
4. Enmity between the parties is admitted. There were instances to show that the accused
had been causing various kinds of mischief. The incident in question occurred on 26th
November, 1994. Two years prior thereto, accused Nos. 1 and 3 assaulted the deceased
and PW-4, Viyakula Mary. Four months thereafter, the appellant (accused No. 1) is said
to have caused damage to the pipeline of the water of the house of the deceased.
@page-SC1384
A few months before the incident in question, allegedly the haystack of the deceased was
put on fire.
Two weeks prior to the incident again, the deceased was assaulted resulting in initiation
of a criminal proceeding against the accused.
5. On 26th November, 1994 at about 9.00 p. m. while the deceased and the informant-
Kolandaisamy (PW-1) were proceeding towards their house from Nilakkottai Market on
their bicycles, somebody had flashed torch light on the deceased. Deceased questioned
thereabout. At that time PW-1 also flashed light from his torch towards the opposite
direction and saw the appellant. Appellant assaulted the deceased with a wood log which
was marked as M. O. 1. He saw the accused No. 3 catching hold of the deceased. He was
threatened with dire consequences, if he intervened, by the accused No. 2 with a knife.
The deceased fell down but still was repeatedly assaulted. PW-1 ran from the spot. He
was chased. He allegedly hid himself in a bush.
6. PW-1 met PW-2 Saveriar and informed him about the incident that had taken place.
They rushed to their village. They came back to the scene of occurrence and found that
the deceased had expired.
7. PW-1 went to Nilakottai police station. A First Information Report was lodged at about
1.00 A. M. on 27th November, 1994. The Investigating officer reached the place of
occurrence at about 2.00 A. M. He recovered blood stained earth and a blue shawl.
Inquest of the dead body was conducted between 3.00 a. m. and 5.00 a.m. He had also
examined some witnesses being PWs. 1, 2, 5 and 6. The body was sent for postmortem
examination. Autopsy was conducted at about 11.15a.m. on 27th November, 1994. The
following anti mortem injuries were found on the person of the deceased :-
"1. On the back side of the head a bruise of the size 3 cm x 2 cm was found.
2. On the back of the left side of the head a bruise of size 3 cm x 2 cm was found.
3. On the upper portion of the head a swelling of the size 3 cm x 3 cm was found.
4. On the front side of the head a swelling of size 3 cm x 3 cm was found.
5. On the right side of the eye brow a bruise of size 2 cm x 1 cm was found.
6. Below the right eye a bruise of size 2 cm x 2 cm was found. Right eye was closed.
Outside the right was totally blackened.
7. A swelling of the size 6 cm x 4 cm was found on the right cheek."
8. The learned Sessions Judge relying on or on the basis of the material brought on record
accepted the deposition of PW-1, Kolandaisamy. All the other accused were held guilty of
the charges leveled against them. On an appeal preferred by the accused before the High
Court, it while upholding the conviction and sentence of the appellant herein, it recorded
a judgment of acquittal so far as accused Nos. 2 and 3 are concerned. It was opined that
accused No. 2 being father in law of the appellant, might not have any motive to cause
the murder of the deceased. Besides, no incised injury has been found on the deceased.
As regards accused No. 3, it was held that there was no cogent material to connect him
with the crime.
9. Mr. T. Raja, learned counsel appearing on behalf of the appellant in support of the
appeal, inter alia would submit :-
(1) That the High Court committed a serious error in drawing presumption on the basis of
the appellant's conduct, to hold that he had the requisite intention to kill the deceased.
Such a presumption, learned counsel would contend, is not available in law.

(2) The cycle and torch used by PW-1 having not been seized, which were material for
the purpose of corroborating his statement, the impugned judgment is liable to be set
aside.
(3) If the articles which PW-1 had bought, could be seized, there was absolutely no
reason as to why the cycle and the torch used by PW-1 could not have also been seized.
10. Mr. V. Kanakaraj, learned senior counsel appearing on behalf of the State, on the
other hand, supported the impugned judgment,
11. PW-1 and PW-4 have proved enmity between the parties. Veracity of statements of
the said witnesses to that effect was not tested in the cross examination. The fact that an
incident had occurred a few days prior to the incident in question is also not in dispute.
12. The First Information Report was received by the Investigating officer, PW-10,
@page-SC1385
at 1.00 a.m. on 27th November, 1994. He immediately came to the site. The distance
between the place of occurrence and Nilakkottai Police Station is about 3 kms. The
Investigating Officer reached the place of occurrence at about 2.00 a. m. Thus, the
investigation commenced immediately after the lodging of the First Information Report.
It is not a case where an undue delay in lodging the first Information Report took place.
13. Homicidal death of Maria Michel stands undisputed. It is also evident from the
postmortem examination report that the injuries on the deceased could have been caused
by a log of wood which was marked as M. O. 1. The deceased suffered atleast three
fractures on his head.
14. PW-1, although may be a son of the deceased, but it is difficult to disbelieve his
statement that he had witnessed the occurrence.
15. We have been taken through the entirety of his deposition and do not find any reason
to differ with the views of the learned Sessions Judge as also the High Court. The cycle
and the torch used by PW-1 had no connection with the commission of the offence. If
they had not been seized by the police, for one reason or the other, the same by itself
would not be a ground to disbelieve the statement of P. W. 1.
16. The following finding of the High Court had been commented upon by Mr. Raja.
"From the conduct of the first accused, it can be presumed that the first accused had the
intention to kill the deceased and therefore, the prosecution has proved its case in so far
as the first accused is concerned."
17. The High Court while mentioning about the conduct of the appellant, meant overt acts
attributed on the part of the appellant. The High Court merely opined that in view of the
number of injuries inflicted upon the deceased, he had an intention to kill him. Intention
to kill a person must be determined having regard to the factual scenario involved in each
case. The doctor PW-3, K. Subramaniam, has clearly stated that the injuries suffered by
the deceased could have been caused by the log of wood, which was marked as M. O. 1.
Medical evidence, thus, corroborated the ocular evidence.
18. Keeping in view the number of injuries inflicted on the deceased as also the nature
thereof and furthermore in view of the opinion expressed by the doctor, there cannot be
any doubt whatsoever that the appellant had the intention to kill the deceased.

19. For the reasons abovementioned we do not find any merit in this appeal which is
accordingly dismissed.
Appeal dismissed.
AIR 2008 SUPREME COURT 1385 "Samira Kohli v. Prabha Manchanda"
Coram : 3 B. N. AGRAWAL, PRAKASH PRABHAKAR NAOLEKAR AND R. V.
RAVEENDRAN, JJ.
Civil Appeal No. 1949 of 2004, D/- 16 -1 -2008.
Samira Kohli v. Prabha Manchanda and Anr.
(A) Medical Council Act (102 of 1956), S.20A, S.33 - MEDICAL COUNCIL -
MEDICAL NEGLIGENCE - Medical negligence - Patient's consent to undergo treatment
- Nature of express consent - Forms prevalent in U.K. (real consent) and U.S. (informed
consent) - Difference in element involved, explained. (Para 14)
(B) Consumer Protection Act (68 of 1986), S.21, S.2(o) - Medical Council Act (102 of
1956), S.20A - CONSUMER PROTECTION - MEDICAL COUNCIL - MEDICAL
NEGLIGENCE - Medical negligence - Patient's consent as defence - Limited only to
procedures for which consent is given - Additional procedure (outside consent) can be
carried out by doctor only when delay would cause imminent danger to life/health of
patient.
The consent given by the patient authorises the doctor to carry out only the procedure for
which express consent is given. The only exception to this rule is the principle of
necessity by which the doctor is permitted to perform further or additional procedure. The
exception is restricted to cases where the patient is temporarily incompetent (being
unconscious), and permits the procedure delaying of which would be unreasonable
because of the imminent danger to the life or health of the patient. Reasons such as it is
quite possible that if the patient been conscious, and informed about the need for the
additional procedure, the patient might have agreed to it, that the additional procedure is
beneficial and in the interests of the patient, that postponement of the additional
procedure (say removal of an organ) may require another surgery, whereas removal of the
affected organ during
@page-SC1386
the initial diagnostic or exploratory surgery, would save the patient from the pain and cost
of a second operation, may be practical or convenient but they are not relevant. What is
relevant and of importance is the inviolable nature of the patient's right in regard to his
body and his right to decide whether he should undergo the particular treatment or
surgery or not. (Paras 16, 17)
(C) Consumer Protection Act (68 of 1986), S.2(o), S.21 - Medical Council Act (102 of
1956), S.33 - CONSUMER PROTECTION - MEDICAL COUNCIL - CONTRACT -
Medical services - Patient's consent to undergo treatment - Legal requirements for 'real
consent' - Extent of information that may be given by Doctor before obtaining consent -
Limits to which doctor is bound by patient's consent.
Contract Act (9 of 1872), S.13.
Considering the Indian panorama where majority of citizens requiring medical care and
treatment fall below the poverty line. Most of them are illiterate or semi-literate. They
cannot comprehend medical terms, concepts, and treatment procedures. They cannot
understand the functions of various organs or the effect of removal of such organs. They
do not have access to effective but costly diagnostic procedures. There is a need to keep
the cost of treatment within affordable limits. Bringing in the American concepts and
standards of treatment procedures and disclosures of risks, consequences and choice will
inevitably bring in higher cost-structure of American medical care. Patients in India
cannot afford them. The following principles would presently govern patient's consent :
i) A doctor has to seek and secure the consent of the patient before commencing a
'treatment' (the term 'treatment' includes surgery also). The consent so obtained should be
real and valid, which means that : the patient should have the capacity and competence to
consent; his consent should be voluntary; and his consent should be on the basis of
adequate information concerning the nature of the treatment procedure, so that he knows
what he is consenting to.
ii) The 'adequate information' to be furnished by the doctor (or a member of his team)
who treats the patient, should enable the patient to make a balanced judgment as to
whether he should submit himself to the particular treatment or not. This means that the
Doctor should disclose - (a) nature and procedure of the treatment and its purpose,
benefits and effect; (b) alternatives if any, available; (c) an outline of the substantial risks;
and (d) adverse consequences of refusing treatment. But there is no need to explain
remote or theoretical risks involved, which may frighten or confuse a patient and result in
refusal of consent for the necessary treatment. Similarly, there is no need to explain the
remote or theoretical risks of refusal to take treatment which may persuade a patient to
undergo a fanciful or unnecessary treatment. A balance should be achieved between the
need for disclosing necessary and adequate information and at the same time avoid the
possibility of the patient being deterred from agreeing to a necessary treatment or offering
to undergo an unnecessary treatment.
iii) Consent given only for a diagnostic procedure, cannot be considered as consent for
therapeutic treatment. Consent given for a specific treatment procedure will not be valid
for conducting some other treatment procedure. The fact that the unauthorized additional
surgery is beneficial to the patient, or that it would save considerable time and expense to
the patient, or would relieve the patient from pain and suffering in future, are not grounds
of defence in an action in tort for negligence or assault and battery. The only exception to
this rule is where the additional procedure though unauthorized, is necessary in order to
save the life or preserve the health of the patient and it would be unreasonable to delay
such unauthorized procedure until patient regains consciousness and takes a decision.
iv) There can be a common consent for diagnostic and operative procedures where they
are contemplated. There can also be a common consent for a particular surgical procedure
and an additional or further procedure that may become necessary during the course of
surgery.
v) The nature and extent of information to be furnished by the doctor to the patient to
secure the consent need not be of the stringent and high degree mentioned in Canterbury
1972 (464) Fed Rep 2d 772 but should be of the extent which is accepted as normal and
proper by a body of medical men skilled and experienced in the particular field. It will
depend upon the physical and mental condition of the patient, the nature of treatment, and
the risk and consequences
@page-SC1387
attached to the treatment.
The 'real consent' concept evolved in England has been preferred to 'reasonably prudent
patient test' evolved in America having regard to the ground realities in medical and
health-care in India. But if medical practitioners and private hospitals become more and
more commercialized, and if there is a corresponding increase in the awareness of
patient's rights among the public, inevitably, a day may come when Courts in India may
have to move towards American concept.
(Paras 31, 32)
(D) Consumer Protection Act (68 of 1986), S.21 - Medical Council Act (102 of 1956),
S.33 - CONSUMER PROTECTION - MEDICAL COUNCIL - MEDICAL
NEGLIGENCE - Medical negligence - Consent given for diagnostic surgery - Removal
of uterus and ovaries on such consent - Is unauthorised and unwarranted.
The appellant, an unmarried woman aged 44 years, visited the clinic of the respondent
complaining of prolonged menstrual bleeding for nine days. The respondent examined
and advised her to undergo an ultrasound test on the same day. After examining the
report, the appellant was advised laparoscopy test under general anesthesia, for making
an affirmative diagnosis. Accordingly, the appellant went to the respondent's clinic with
her mother. On admission, the appellant's signatures were taken on - 1) admission and
discharge card; 2) consent form for hospital admission and medical treatment; and 3)
consent form for surgery. The Admission Card showed that admission was "for diagnostic
and operative laparoscopy". The consent form for surgery described the procedure to be
undergone by the appellant as "diagnostic and operative laparoscopy, Laparotomy may be
needed". Thereafter, appellant was put under general anesthesia and subjected to a
laparoscopic examination. When the appellant was still unconscious, the doctor, who was
assisting the respondent, came out of the Operation Theatre and took the consent of
appellant's mother, who was waiting outside, for performing hysterectomy under general
anesthesia. Thereafter, the respondent performed abdominal hysterectomy (removal of
uterus) and bilateral salpingo-oophorectomy (removal of ovaries and fallopian tubes).
Held, the removal of uterus and ovaries of the appellant on consent given for diagnostic
surgery was unauthorised and unwarranted. (Para 54)
It cannot be accepted that, term 'laparotomy' used in the consent form is equal to or same
as hysterectomy. Medical texts and authorities clearly spell out that Laparotomy is at best
the initial step that is necessary for performing hysterectomy or salpingo-oophorectomy.
Laparotomy by itself is not hysterectomy or salpingo-copherectomy. Nor does
'hysterectomy' include salpingo-oophorectomy, in the case of woman who has not
attained menopause. Laparotomy does not refer to surgical removal of any vital or
reproductive organs. (Paras 42, 43)
The words "such medical treatment as is considered necessary for me for............ ..."in the
consent form cannot be construed as consent for surgical treatment by way of removal of
uterus and ovaries. The view that "the informed choice of surgical procedures has to be
left to the operating surgeon depending on his/her discretion, after assessing the damage
to the internal organs, but subject to his/her exercising care and caution" proceeds on the
erroneous assumption that where the surgeon has shown adequate care and caution in
performing the surgery, the consent of the patient for removal of an organ is unnecessary.
(Para 48)
(E) Consumer Protection Act (68 of 1986), S.21 - Medical Council Act (102 of 1956),
S.33 - CONSUMER PROTECTION - MEDICAL COUNCIL - MEDICAL
NEGLIGENCE - Medical negligence - Consent to undergo treatment - Consent given by
patient's relative - Not valid and real consent when patient is a competent adult, there was
no medical emergency and matter was only at stage of diagnosis. (Para 45)
(F) Consumer Protection Act (68 of 1986), S.21 - CONSUMER PROTECTION -
MEDICAL NEGLIGENCE - Medical negligence - Consent for treatment - Absence -
Correctness or appropriateness of the treatment procedures, does not make treatment
legal. (Para 52)
(G) Consumer Protection Act (68 of 1986), S.2(o), S.21 - CONSUMER PROTECTION -
MEDICAL NEGLIGENCE - Medical negligence - Treatment beyond consent given by
patient - Damages - Patient admitted/ consented for diagnostic laparoscopy - Doctor
performing radical surgery for removal of uterus and ovaries - Surgery
@page-SC1388
done in interest of patient as permanent relief against menstrual problem - Patient aged 44
years and reaching age of menopause - No serious health repercussions faced -
Considering mitigating circumstances doctor denied fees charged and directed to pay Rs.
25,000/- as damages. (Para 54)
Cases Referred: Chronological Paras
2001 AIR SCW 3881 : AIR 2001 SC 3914 25
1998 (1) AC 232 33
1998 (48) BMLR 118 33
1996 AIR SCW 919 : AIR 1996 SC 2377 25
1995 AIR SCW 4463 : AIR 1996 SC 550 25
1992 (109) ALR 625 (Canadian SC) 33
1989 (2) All ER 545 15, 16
(1985) 1 All ER 643 24, 33
(1980) 114 DLR (3d) 1 33
1972 (464) Fed Rep 2d 772 14, 21, 22, 33
(1957) 2 All ER 118 : (1957) 1 WLR 582 22, 33
1957 (154) Cal App 2d 560 20
1950 SC 200 (Scottish case) 23
1949 (2) DLR 442 16
(1944) 1 KB 476 20
1933 (3) DLR 260 16
(1914) 211 NY 125 15
Prashant Bhushan, Ms. Rukhsana Choudhary and Ms. Sumita Hazarika, for Appellant;
Aman Lekhi, Sr. Advocate, Meenakshi Lekhi, Rajan Chourasia, Jaspreet S. Rai, Rakesh
Kumar, Rohit Nagpal, Harish Pandey, Abhijit Das, Rameshwar Prasad Goyal, Rajesh
Kumar, Ajay Majithia and Dr. Kailash Chand with him, for Respondents.
Judgement
RAVEENDRAN, J. :- This appeal is filed against the order dated 19-11-2003 passed by
the National Consumer Disputes Redressal Commission (for short 'Commission')
rejecting the appellants complaint (O.P. No. 12/1996) under Section 21 of the Consumer
Protection Act, 1986 (Act for short).
Undisputed facts
2. On 9-5-1995, the appellant, an unmarried woman aged 44 years, visited the clinic of
the first respondent (for short the respondent) complaining of prolonged menstrual
bleeding for nine days. The respondent examined and advised her to undergo an
ultrasound test on the same day. After examining the report, the respondent had a
discussion with appellant and advised her to come on the next day (10-5-1995) for a
laparoscopy test under general anesthesia, for making an affirmative diagnosis.
3. Accordingly, on 10-5-1995, the appellant went to the respondent's clinic with her
mother. On admission, the appellant's signatures were taken on (i) admission and
discharge card; (ii) consent form for hospital admission and medical treatment; and (iii)
consent form for surgery. The Admission Card showed that admission was "for diagnostic
and operative laparoscopy on 10-5-1995". The consent form for surgery filled by Dr. Lata
Rangan (respondent's assistant) described the procedure to be undergone by the appellant
as "diagnostic and operative laparoscopy. Laparotomy may be needed". Thereafter,
appellant was put under general anesthesia and subjected to a laparoscopic examination.
When the appellant was still unconscious, Dr. Lata Rangan, who was assisting the
respondent, came out of the Operation Theatre and took the consent of appellant's mother,
who was waiting outside, for performing hysterectomy under general anesthesia.
Thereafter, the Respondent performed a abdominal hysterectomy (removal of uterus) and
bilateral salpingooophorectomy (removal of ovaries and fallopian tubes). The appellant
left the respondents clinic on 15-5-1995 without settling the bill.
4. On 23-5-1995, the respondent lodged a complaint with the Police alleging that on 15-
5-1995, the Appellant's friend (Commander Zutshi) had abused and threatened her
(respondent) and that against medical advice, he got the appellant discharged without
clearing the bill. The appellant also lodged a complaint against the respondent on 31-5-
1995, alleging negligence and unauthorized removal of her reproductive organs. The first
respondent issued a legal notice dated 5-6-1995 demanding Rs. 39,325/- for professional
services. The appellant sent a reply dated 12-7-1995. There was a rejoinder dated 18-7-
1995 from the respondent and a further reply dated 11-9-1995 from the appellant. On 19-
1-1996 the appellant filed a complaint before the Commission claiming a compensation
of Rs.25 lakhs from the Respondent. The appellant alleged that respondent was negligent
in treating her; that the radical surgery
@page-SC1389
by which her uterus, ovaries and fallopian tubes were removed without her consent, when
she was under general anesthesia for a Laparoscopic test, was unlawful, unauthorized and
unwarranted; that on account of the removal of her reproductive organs, she had suffered
premature menopause necessitating a prolonged medical treatment and a Harmone
Replacement Therapy (HRT) course, apart from making her vulnerable to health
problems by way of side effects. The compensation claimed was for the loss of
reproductive organs and consequential loss of opportunity to become a mother, for
diminished matrimonial prospects, for physical injury resulting in the loss of vital body
organs and irreversible permanent damage, for pain, suffering emotional stress and
trauma, and for decline in the health and increasing vulnerability to health hazards.
5. During the pendency of the complaint, at the instance of the respondent, her insurer-
New India Assurance Co. Ltd, was impleaded as the second respondent. Parties led
evidence - both oral and documentary, Appellant examined an expert witness (Dr. Puneet
Bedi, Obstetrician and Gynaecologist), her mother (Sumi Kohli) and herself. The
respondent examined herself, an expert witness (Dr. Sudha Salhan, Professor of
Obstetrics and Gynaecology and President of Association of Obstetricians and
Gynaecologists of Delhi), Dr. Latha Rangan (Doctor who assisted the Respondent) and
Dr. Shiela Mehra (Anaesthetist for the surgery). The medical records and notices
exchanged were produced as evidence. After hearing arguments, the Commission
dismissed the complaint by order dated 19-11-2003. The Commission held : (a) the
appellant voluntarily visited the respondents clinic for treatment and consented for
diagnostic procedures and operative surgery; (ii) the hysterectomy and other surgical
procedures were done with adequate care and caution; and (iii) the surgical removal of
uterus, ovaries etc. was necessitated as the appellant was found to be suffering from
endometriosis (Grade IV), and if they had not been removed, there was likelihood of the
lesion extending to the intestines and bladder and damaging them. Feeling aggrieved, the
appellant has filed this appeal.
The appellant's version :
6. The appellant consulted respondent on 9-5-1995. Respondent wanted an ultra-sound
test to be done on (he same day. In the evening, after seeing the ultrasound report, the
respondent informed her that she was suffering from fibroids and that to make a firm
diagnosis, she had to undergo a laparoscopic test the next day. The respondent informed
her that the test was a minor procedure involving a small puncture for examination under
general anesthesia. The respondent informed her that the costs of laparoscopic test,
hospitalization, and anesthetist's charges would be around Rs. 8000 to 9,000. Respondent
spent hardly 4 to 5 minutes with her and there was no discussion about the nature of
treatment. Respondent merely told her that she will discuss the line of treatment, after the
laparoscopic test. On 10-5-1995, she went to the clinic only for a diagnostic laparoscopy.
Her signature was taken on some blank printed forms without giving her an opportunity
to read the contents. As only a diagnostic procedure by way of a laparoscopic test was to
be conducted, there was no discussion, even on 10-5-995, with regard to any proposed
treatment. As she was intending to marry within a month and start a family, she would
have refused consent for removal of her reproductive organs and would have opted for
conservative treatment, had she been informed about any proposed surgery for removal of
her reproductive organs.
7. When the appellant was under general anaesthesia, respondent rushed out of the
operation theatre and told appellant's mother that she had started bleeding profusely and
gave an impression that the only way to save her life was by performing an extensive
surgery. Appellant's aged mother was made to believe that there was a life threatening
situation, and her signature was taken to some paper. Respondent did not choose to wait
till appellant regained consciousness, to discuss about the findings of the laparoscopic
test and take her consent for treatment. The appellant was kept in the dark about the
radical surgery performed on her. She came to know about it, only on 14-5-1995 when
respondent's son casually informed her about the removal of her reproductive organs.
When she asked the respondent as to why there should be profuse bleeding during a
Laparoscopic test (as informed to appellant's mother) and why her reproductive organs
were removed in such haste without informing her, without her
@page-SC1390
consent, and without affording her an opportunity to consider other options or seek other
opinion, the respondent answered rudely that due to her age, conception was not possible,
and therefore, the removal of her reproductive organs did not make any difference.
8. As she was admitted only for a diagnostic procedure, namely a laparoscopy test, and as
she had given consent only for a laparoscopy test and as her mother's consent for
conducting hysterectomy had been obtained by misrepresentation, there was no valid
consent for the radical surgery. The respondent also tried to cover up her
unwarranted/negligent act by falsely alleging that the appellant was suffering from
endometriosis. The respondent was guilty of two distinct acts of negligence : the first was
the failure to take her consent, much less an informed consent, for the radical surgery
involving removal of reproductive organs; and the second was the failure to exhaust
conservative treatment before resorting to radical surgery, particularly when such drastic
irreversible surgical procedure was not warranted in her case. The respondent did not
inform the appellant, of the possible risks, side effects and complications associated with
such surgery, before undertaking the surgical procedure. Such surgery without her
consent was also in violation of medical Rules and ethics. Removal of her reproductive
organs also resulted in a severe physical impairment, and necessitated prolonged further
treatment. The respondent was also not qualified to claim to be a specialist in Obstetrics
and Gynaecology and therefore could not have performed the surgery which only a
qualified Gynaecologist could perform.
The respondent's version
9. The appellant had an emergency consultation with the respondent on 9-5-1995,
complaining that she had heavy vaginal bleeding from 30-4-1995, that her periods were
irregular, and that she was suffering from excessive, irregular and painful menstruation
(menorrhagia and dysmenorrhea) for a few months. On a clinical examination, the
respondent found a huge mass in the pelvic region and tenderness in the whole area. In
view of the severe condition, Respondent advised an ultrasound examination on the same
evening. Such examination showed fibroids in the uterus, a large chocolate cyst (also
known as endometrical cyst) on the right side and small cysts on the left side. On the
basis of clinical and ultra sound examination, she made a provisional diagnosis of
endometriosis and informed the appellant about the nature of the ailment, the anticipated
extent of severity, and the modality of treatment. She further informed the appellant that a
laparoscopic examination was needed to confirm the diagnosis; that if on such
examination, she found that the condition was manageable with conservative surgery, she
would only remove the chocolate cyst and fulgurate the endometric areas and follow it by
medical therapy; and that if the lesion was extensive, then considering her age and
likelihood of destruction of the function of the tubes, she will perform hysterectomy. She
also explained the surgical procedure involved, and answered appellant's queries. The
appellant stated that she was in acute discomfort and wanted a permanent cure and,
therefore whatever was considered necessary, including a hysterectomy may be
performed. When appellant's mother called on her on the same evening, the respondent
explained to her also about the nature of disease and the proposed treatment, and
appellant's mother stated that she may do whatever was best for her daughter. According
to the accepted medical practice, if endometriosis is widespread in the pelvis causing
adhesions, and if the woman is over 40 years of age, the best and safest form of cure was
to remove the uterus and the ovaries. As there is a decline in fecundity for most women in
the fourth decade and a further decline in women in their forties, hysterectomy is always
considered as a reasonable and favoured option. Further, endometriosis itself affected
fertility adversely. All these were made known to the appellant before she authorised the
removal of uterus and ovaries, if found necessary on laparoscopic examination.
10. On 10-5-1995, the appellant's consent was formally recorded in the consent form by
Dr. Lata Rangan - respondent's assistant. Dr. Lata Rangan informed the appellant about
the consequences of such consent and explained the procedure that was proposed. The
appellant signed the consent forms only after she read the duly filled up forms and
understood their contents. All the requisite tests to be conducted mandatorily before the
surgery were performed including Blood Grouping, HIV, Hemoglobin, PCV,
@page-SC1391
BT, CT and ECG. The laparoscopic examination of the uterus surface confirmed the
provisional diagnosis of endometriosis. The right ovary was enlarged and showed a
chocolate cyst stuck to the bowel. Right tube was also involved in the lesion. The left
ovary and tube were also stuck to the bowel near the cervix. A few small cysts were seen
on the left ovary. The pelvic organs were thick and difficult to mobilize. Having regard to
the extent of the lesion and the condition of appellant's uterus and ovaries, she decided
that conservative surgery would not be sufficient and the appellants problem required
removal of uterus and ovaries. The respondent sent her assistant, Dr. Lata Rangan to
explain to appellants mother that the lesion would not respond to conservative surgery
and a hysterectomy had to be performed and took her consent. The surgery was extremely
difficult due to adhesions and vascularity of surface. A sub-total hysterectomy was done
followed by the removal of rest of the stump of cervix. As the right ovary was completely
stuck down to bowel, pouch of douglas, post surface and tube, it had to be removed
piecemeal. When appellant regained consciousness, she was informed about the surgery.
The appellant felt assured that heavy bleeding and pain would not recur. There was no
protest either from the appellant or her mother, in regard to the removal of the ovaries and
uterus.
11. However, on 15-5-1995, Commander Zutshi to whom appellant was said to have been
engaged, created a scene and got her discharged. At the time of discharge, the summary
of procedure and prescription of medicines were given to her. As the bill was not paid, the
respondent filed Suit No.469/ 1995 for recovery of the bill amount and the said suit was
decreed in due course.
12. Respondent performed the proper surgical procedure in pursuance of the consent
given by the appellant and there was no negligence, illegality, impropriety or professional
misconduct. There was real and informed consent by the appellant for the removal of her
reproductive organs. The surgery (removal of uterus and ovaries), not only cured the
appellant of her disease but also saved her intestines, bladder and ureter from possible
damage. But for the surgical removal, there was likelihood of the intestines being
damaged due to extension of lesion thereby causing bleeding, fibrosis and narrowing of
the gut; there was also likelihood of the lesion going to the surface of the bladder
penetrating the wall and causing haematuria and the ureter being damaged due to fibrosis
and leading to damage of the kidney, with a reasonable real chance of developing cancer.
As the complainant was already on the wrong side of 40 years which is a peri-
menopausal age and as the appellant had menorrhagia which prevented her from
ovulating regularly and giving her regular cycle necessary for pregnancy and as
endometriosis prevented fertilization and also produced reaction in the pelvis which
increased the lymphocytes and macrophages which destroyed the ova and sperm, there
was no chance of appellant conceiving, even if the surgery had not been performed. The
removal of her uterus and ovaries was proper and necessary and there was no negligence
on the part of the respondent in performing the surgery. A Doctor who has acted in
accordance with a practice accepted as proper by medical fraternity cannot be said to
have acted negligently. In the realm of diagnosis and treatment there is ample scope for
genuine differences of opinion and no Doctor can be said to have acted negligently
merely because his or her opinion differs from that of other Doctors or because he or she
has displayed lesser skill or knowledge when compared to others. There was thus no
negligence on her part.
Questions for consideration :
13. On the contentions raised, the following questions arise for our consideration :
(i) Whether informed consent of a patient is necessary for surgical procedure involving
removal of reproductive organs? If so what is the nature of such consent ?
(ii) When a patient consults a medical practitioner, whether consent given for diagnostic
surgery, can be construed as consent for performing additional or further surgical
procedure - either as conservative treatment or as radical treatment - without the specific
consent for such additional or further surgery.
(iii) Whether there was consent by the appellant, for the abdominal hysterectomy and
Bilateral Salpingo-oophorectomy (for short AH-BSO) performed by the respondent?
(iv) Whether the respondent had falsely invented a case that appellant was suffering from
endometriosis to explain the unauthorized and unwarranted removal of
@page-SC1392
uterus and ovaries, and whether such radical surgery was either to cover-up negligence in
conducting diagnostic laparoscopy or to claim a higher fee ?
(v) Even if appellant was suffering from endometriosis, the respondent ought to have
resorted to conservative treatment/surgery instead of performing radical surgery ?
(vi) Whether the Respondent is guilty of the tortious act of negligence/battery amounting
to deficiency in service, and consequently liable to pay damages to the appellant.
Re : Question No. (i) and (ii)
14. Consent in the context of a doctor-patient relationship, means the grant of permission
by the patient for an act to be carried out by the doctor, such as a diagnostic, surgical or
therapeutic procedure. Consent can be implied in some circumstances from the action of
the patient. For example, when a patient enters a Dentist's clinic and sits in the Dental
chair, his consent is implied for examination, diagnosis and consultation. Except where
consent can be clearly and obviously implied, there should be express consent. There is,
however, a significant difference in the nature of express consent of the patient, known as
'real consent' in UK and as 'informed consent' in America. In UK, the elements of consent
are defined with reference to the patient and a consent is considered to be valid and 'real'
when (i) the patient gives it voluntarily without any coercion; (ii) the patient has the
capacity and competence to give consent; and (iii) the patient has the minimum of
adequate level of information about the nature of the procedure to which he is consenting
to. On the other hand, the concept of 'informed consent' developed by American courts,
while retaining the basic requirements consent, shifts the emphasis to the doctor's duty to
disclose the necessary information to the patient to secure his consent. 'Informed consent'
is defined in Taber's Cyclopedic Medical Dictionary thus :
"Consent that is given by a person after receipt of the following information : the nature
and purpose of the proposed procedure or treatment; the expected outcome and the
likelihood of success; the risks; the alternatives to the procedure and supporting
information regarding those alternatives; and the effect of no treatment or procedure,
including the effect on the prognosis and the material risks associated with no treatment.
Also included are instructions concerning what should be done if the procedure turns out
to be harmful or unsuccessful."
In Canterbury v. Spence, 1972 [464] Federal Reporter 2d. 772, the United States Courts
of appeals, District of Columbia Circuit, emphasized the element of Doctor's duty in
'informed consent' thus :
"It is well established that the physician must seek and secure his patient's consent before
commencing an operation or other course of treatment. It is also clear that the consent, to
be efficacious, must be free from imposition upon the patient. It is the settled rule that
therapy not authorized by the patient may amount to a tort - a common law battery - by
the physician. And it is evident that it is normally impossible to obtain a consent worthy
of the name unless the physician first elucidates the options and the perils for the patient's
edification. Thus the physician has long borne a duty, on pain of liability for unauthorized
treatment, to make adequate disclosure to the patient."
[Emphasis supplied]
15. The basic principle in regard to patient's consent may be traced to the following
classic statement by Justice Cardozo in Schoendorff vs. Society of New York Hospital -
(1914) 211 NY 125 :
"Every human being of adult years and sound mind has a right to determine what should
be done with his body; and a surgeon who performs the operation without his patient's
consent, commits an assault for which he is liable in damages."
This principle has been accepted by English court also. In Re : F. 1989(2) All ER 545, the
House of Lords while dealing with a case of sterilization of a mental patient reiterated the
fundamental principle that every person's body is inviolate and performance of a medical
operation on a person without his or her consent is unlawful. The English law on this
aspect is summarised thus in Principles of Medical Law (published by Oxford University
Press - Second Edition, edited by Andrew Grubb, Para 3.04, Page 133) :
"Any intentional touching of a person is unlawful and amounts to the tort of battery
unless it is justified by consent or other lawful authority. In medical law, this means that a
doctor may only carry out a medical treatment
@page-SC1393
or procedure which involves contact with a patient if there exists a valid consent by the
patient (or another person authorized by law to consent on his behalf) or if the touching is
permitted notwithstanding the absence of consent."
16. The next question is whether in an action for negligence/battery for performance of an
unauthorized surgical procedure, the Doctor can put forth as defence the consent given
for a particular operative procedure, as consent for any additional or further operative
procedures performed in the interests of the patient. In Murray vs. McMurchy - 1949 (2)
DLR 442, the Supreme Court of BC, Canada, was considering a claim for battery by a
patient who underwent a caesarian section. During the course of caesarian section, the
doctor found fibroid tumors in the patient's uterus. Being of the view that such tumours
would be a danger in case of future pregnancy, he performed a sterilization operation.
The court upheld the claim for damages for battery. It held that sterilization could not be
justified under the principle of necessity, as there was no immediate threat or danger to
the patient's health or life and it would not have been unreasonable to postpone the
operation to secure the patient's consent. The fact that the doctor found it convenient to
perform the sterilization operation without consent as the patient was already under
general anaesthetic, was held to be not a valid defence. A somewhat similar view was
expressed by Courts of Appeal in England in Re : F. (supra). It was held that the
additional or further treatment which can be given (outside the consented procedure)
should be confined to only such treatment as is necessary to meet the emergency, and as
such needs to be carried out at once and before the patient is likely to be in a position to
make a decision for himself. Lord Goff observed :
"Where, for example, a surgeon performs an operation without his consent on a patient
temporarily rendered unconscious in an accident, he should do no more than is
reasonably required, in the best interests of the patient, before he recovers consciousness.
I can see no practical difficulty arising from this requirement, which derives from the fact
that the patient is expected before long to regain consciousness and can then be consulted
about longer term measures."
The decision in Marshell vs. Curry - 1933 (3) DLR 260 decided by the Supreme Court of
NS, Canada, illustrates the exception to the rule, that an unauthorized procedure may be
justified if the patient's medical condition brooks no delay and warrants immediate action
without waiting for the patient to regain consciousness and take a decision for himself. In
that case the doctor discovered a grossly diseased testicle while performing a hernia
operation. As the doctor considered it to be gangrenous, posing a threat to patient's life
and health, the doctor removed it without consent, as a part of the hernia operation. An
action for battery was brought on the ground that the consent was for a hernia operation
and removal of testicle was not consent. The claim was dismissed. The court was of the
view that the doctor can act without the consent of the patient where it is necessary to
save the life or preserve the health of the patient. Thus, the principle of necessity by
which the doctor is permitted to perform further or additional procedure (unauthorized) is
restricted to cases where the patient is temporarily incompetent (being unconscious), to
permit the procedure delaying of which would be unreasonable because of the imminent
danger to the life or health of the patient.
17. It is quite possible that if the patient been conscious, and informed about the need for
the additional procedure, the patient might have agreed to it. It may be that the additional
procedure is beneficial and in the interests of the patient. It may be that postponement of
the additional procedure (say removal of an organ) may require another surgery, whereas
removal of the affected organ during the initial diagnostic or exploratory surgery, would
save the patient from the pain and cost of a second operation. Howsoever practical or
convenient the reasons may be, they are not relevant. What is relevant and of importance
is the inviolable nature of the patient's right in regard to his body and his right to decide
whether he should undergo the particular treatment or surgery or not. Therefore at the risk
of repetition, we may add that unless the unauthorized additional or further procedure is
necessary in order to save the life or preserve the health of the patient and it would be
unreasonable (as contrasted from being merely inconvenient) to delay the further
procedure until the patient regains consciousness
@page-SC1394
and takes a decision, a doctor cannot perform such procedure without the consent of the
patient.
18. We may also refer to the code of medical ethics laid down by the Medical Council of
India (approved by the Central Government under section 33 of Indian Medical Council
Act, 1956). It contains a chapter relating to disciplinary action which enumerates a list of
responsibilities, violation of which will be professional misconduct. Clause 13 of the said
chapter places the following responsibility on a doctor :
"13. Before performing an operation the physician should obtain in writing the consent
from the husband or wife, parent or guardian in the case of a minor, or the patient himself
as the case may be. In an operation which may result in sterility the consent of both
husband and wife is needed."
We may also refer to the following guidelines to doctors, issued by the General Medical
Council of U.K. in seeking consent of the patient for investigation and treatment :
"Patients have a right to information about their condition and the treatment options
available to them. The amount of information you give each patient will vary, according
to factors such as the nature of the condition, the complexity of the treatment, the risks
associated with the treatment or procedure, and the patient's own wishes. For example,
patients may need more information to make an informed decision about the procedure
which carries a high risk of failure or adverse side effects; or about an investigation for a
condition which, if present, could have serious implications for the patient's employment,
social or personal life.
xxxxxx
You should raise with patients the possibility of additional problems coming to light
during a procedure when the patient is unconscious or otherwise unable to make a
decision. You should seek consent to treat any problems which you think may arise and
ascertain whether there are any procedures to which the patient would object, or prefer to
give further thought before you proceed."
The Consent form for Hospital admission and medical treatment, to which appellant's
signature was obtained by the respondent on 10.5.1995, which can safely be presumed to
constitute the contract between the parties, specifically states :
"(A) It is customary, except in emergency or extraordinary circumstances, that no
substantial procedures are performed upon a patient unless and until he or she has had an
opportunity to discuss them with the physician or other health professional to the patient's
satisfaction.
(B) Each patient has right to consent, or to refuse consent, to any proposed procedure of
therapeutic course."
19. We therefore hold that in Medical Law, where a surgeon is consulted by a patient, and
consent of the patient is taken for diagnostic procedure/surgery, such consent cannot be
considered as authorisation or permission to perform therapeutic surgery either
conservative or radical (except in life threatening or emergent situations). Similarly
where the consent by the patient is for a particular operative surgery, it cannot be treated
as consent for an unauthorized additional procedure involving removal of an organ, only
on the ground that such removal is beneficial to the patient or is likely to prevent some
danger developing in future, where there is no imminent danger to the life or health of the
patient.
20. We may next consider the nature of information that is required to be furnished by a
Doctor to secure a valid or real consent. In Bowater v. Rowley Regis Corporation -
[1944] 1 KB 476, Scott L.J. observed :
"A man cannot be said to be truly 'willing' unless he is in a position to choose freely, and
freedom of choice predicates, not only full knowledge of the circumstances on which the
exercise of choice is conditioned, so that he may be able to choose wisely, but the
absence from his mind of any feeling of constraint so that nothing shall interfere with the
freedom of his will."
In Salgo vs. Leland Stanford [154 Cal. App. 2d.560 (1957)], it was held that a physician
violates his duty to his patient and subjects himself to liability if he withholds any facts
which are necessary to form the basis of an intelligent consent by the patient to the
proposed treatment.
21. Canterbury (supra) explored the rationale of a Doctor's duty to reasonably inform a
patient as to the treatment alternatives available and the risk incidental to them, as also
the scope of the disclosure requirement and the physician's privileges not to disclose. It
laid down the 'reasonably prudent patient test' which required the
@page-SC1395
doctor to disclose all material risks to a patient, to show an 'informed consent'. It was held
:
"True consent to what happens to one's self is the informed exercise of a choice, and that
entails an opportunity to evaluate knowledgeably the options available and the risks
attendant upon each. The average patient has little or no understanding of the medical
arts, and ordinarily has only his physician to whom he can look for enlightenment with
which to reach an intelligent decision. From these almost axiomatic considerations
springs the need, and in turn the requirement, of a reasonable divulgence by physician to
patient to make such a decision possible.
.........Just as plainly, due care normally demands that the physician warn the patient of
any risks to his well being which contemplated therapy may involve.
The context in which the duty of risk-disclosure arises is invariably the occasion for
decision as to whether a particular treatment procedure is to be undertaken. To the
physician, whose training enables a self-satisfying evaluation, the answer may seem clear,
but it is the prerogative of the patient, not the physician, to determine for himself the
direction in which his interests seem to lie. To enable the patient to chart his course
understandably, some familiarity with the therapeutic alternatives and their hazards
becomes essential..........
A reasonable revelation in these respects is not only a necessity but, as we see it, is as
much a matter of the physician's duty. It is a duty to warn of the dangers lurking in the
proposed treatment, and that is surely a facet of due care. It is, too, a duty to in part
information which the patient has every right to expect. The patient's reliance upon the
physician is a trust of the kind which traditionally has exacted obligations beyond those
associated with arms length transactions. His dependence upon the physician for
information affecting his well-being, in terms of contemplated treatment, is well-nigh
abject.......... we ourselves have found "in the fiducial qualities of (the physician-patient)
relationship the physician's duty to reveal to the patient that which in his best interests it
is important that he should know." We now find, as a part of the physician's overall
obligation to the patient, a similar duty of reasonable disclosure of the choices with
respect to proposed therapy and the dangers inherently and potentially involve.
In our view, the patient's right of self-decision shapes the boundaries of the duty to reveal.
That right can be effectively exercised only if the patient possesses enough information to
enable an intelligent choice. The scope of the physician's communications to the patient,
then, must be measured by the patient's need, and that need is the information material to
the decision. Thus the test for determining whether a particular peril must be divulged is
its materially to the patient's decision : all risks potentially affecting the decision must be
unmasked."
It was further held that a risk is material 'when a reasonable person, in what the physician
knows or should know to be the patient's position, would be likely to attach significance
to the risk or cluster of risks in deciding whether or not to forego the proposed therapy'.
The doctor, therefore, is required to communicate all inherent and potential hazards of the
proposed treatment, the alternatives to that treatment, if any, and the likely effect if the
patient remained untreated. This stringent standard of disclosure was subjected to only
two exceptions : (i) where there was a genuine emergency, e.g. the patient was
unconscious; and (ii) where the information would be harmful to the patient, e.g. where it
might cause psychological damage, or where the patient would become so emotionally
distraught as to prevent a rational decision. It, however, appears that several States in
USA have chosen to avoid the decision in Canterbury by enacting legislation which
severely curtails operation of the doctrine of informed consent.
22. The stringent standards regarding disclosure laid down in Canterbury, as necessary to
secure an informed consent of the patient, was not accepted in the English courts. In
England, standard applicable is popularly known as the Bolam Test, first laid down in
Bolam v. Friern Hospital Management Committee - [1957] 2 All ER 118. McNair J., in a
trial relating to negligence of a medical practitioner, while instructing the Jury, stated thus
:
"(i) A doctor is not negligent, if he has acted in accordance with a practice accepted as
proper by a responsible body of medical men skilled in that particular art...... Putting it
the other way round, a doctor is not negligent, if he is acting in accordance with
@page-SC1396
such a practice, merely because there is a body of opinion that takes a contrary view. At
the same time, that does not mean that a medical man can obstinately and pigheadedly
carry on with some old technique if it has been proved to be contrary to what is really
substantially the whole of informed medical opinion.
(ii) When a doctor dealing with a sick man strongly believed that the only hope of cure
was submission to a particular therapy, he could not be criticized if, believing the danger
involved in the treatment to be minimal, did not stress them to the patient.
(iii) In order to recover damages for failure to give warning the plaintiff must show not
only that the failure was negligent but also that if he had been warned he would not have
consented to the treatment.
23. Hunter v. Hanley (1955 SC 200), a Scottish case is also worth noticing. In that
decision, Lord President Clyde held :
"In the realm of diagnosis and treatment there is ample scope for genuine difference of
opinion and one man clearly is not negligent merely because his conclusion differs from
that of other professional men, nor because he has displayed less skill or knowledge than
others would have shown. The true test for establishing negligence in diagnosis or
treatment on the part of a doctor is whether he has been proved to be guilty of such
failure as no doctor of ordinary skill would be guilty of if acting with ordinary care."
He also laid down the following requirements to be established by a patient to fasten
liability on the ground of want of care or negligence on the part of the doctor :
'To establish liability by a doctor where deviation from normal practice is alleged, three
facts require to be established. First of all it must be proved that there is a usual and
normal practice; secondly it must be proved that the defender has not adopted that
practice; and thirdly (and this is of crucial importance) it must be established that the
course the doctor adopted is one which no professional man of ordinary skill would have
taken if he had been acting with ordinary care."
24. In Sidaway v. Bethlem Royal Hospital Governors and Ors. [1985] 1 All ER 643, the
House of Lords, per majority, adopted the Bolam test, as the measure of doctor's duty to
disclose information about the potential consequences and risks of proposed medical
treatment. In that case the defendant, a surgeon, warned the plaintiff of the possibility of
disturbing a nerve root while advising an operation on the spinal column to relieve
shoulder and neck pain. He did not however mention the possibility of damage to the
spinal cord. Though the operation was performed without negligence, the plaintiff
sustained damage to spinal cord resulting in partial paralysis. The plaintiff alleged that
defendant was negligent in falling to inform her about the said risk and that had she
known the true position, she would not have accepted the treatment. The trial Judge and
Court of Appeal applied the Bolam test and concluded that the defendant had acted in
accordance with a practice accepted as proper by a responsible body of medical opinion,
in not informing the plaintiff of the risk of damage to spinal cord. Consequently, the
claim for damages was rejected. The House of Lords upheld the decision of the Court of
Appeal that the doctrine of informed consent based on full disclosure of all the facts to
the patient, was not the appropriate test of liability for negligence, under English law. The
majority were of the view that the test of liability in respect of a doctor's duty to warn his
patient of risks inherent in treatment recommended by him was the same as the test
applicable to diagnosis and treatment, namely, that the doctor was required to act in
accordance with the practice accepted at the time as proper by a responsible body of
medical opinion. Lord Diplock stated :
"In English jurisprudence the doctor's relationship with his patient which gives rise to the
normal duty of care to exercise his skill and judgment to improve the patient's health in
any particular respect in which the patient has sought his aid has hitherto been treated as a
single comprehensive duty covering all the ways in which a doctor is called on to
exercise his skill and judgment in the improvement of the physical or mental condition of
the patient for which his services either as a general practitioner or as a specialist have
been engaged. This general duty is not subject to dissection into a number of component
parts to which different criteria of what satisfy the duty of care apply, such as diagnosis,
treatment and advice (including warning of any risks of something going wrong however
skillfully the treatment advised is carried out). The Bolam case itself
@page-SC1397
embraced failure to advise the patient of the risk involved in the electric shock treatment
as one of the allegations of negligence against the surgeon as well as negligence in the
actual carrying out of treatment in which that risk did result in injury to the patient. The
same criteria were applied to both these aspects of the surgeon's duty of care. In modern
medicine and surgery such dissection of the various things a doctor has to do in the
exercise of his whole duty of care owed to his patient is neither legally meaningful nor
medically practicable....... To decide what risks the existence of which a patient should be
voluntarily warned and the terms in which such warning, if any, should be given, having
regard to the effect that the warning may have, is as much an exercise of professional
skill and judgment as any other part of the doctor's comprehensive duty of care to the
individual patient, and expert medical evidence on this matter should be treated in just the
same way. The Bolam test should be applied."
Lord Bridge stated :
"I recognize the logical force of the Canterbury doctrine, proceeding from the premise
that the patient's right to make his own decision must at all costs be safeguarded against
the kind of medical paternalism which assumes that 'doctor knows best'. But, with all
respect, I regard the doctrine as quite impractical in application for three principal
reasons. First, it gives insufficient weight to the realities of the doctor/ patient
relationship. A very wide variety of factors must enter into a doctor's clinical judgment
not only as to what treatment is appropriate for a particular patient, but also as to how
best to communicate to the patient the significant factors necessary to enable the patient
to make an informed decision whether to undergo the treatment. The doctor cannot set
out to educate the patient to his own standard of medical knowledge of all the relevant
factors involved. He may take the view, certainly with some patients, that the very fact of
his volunteering, without being asked, information of some remote risk involved in the
treatment proposed, even though he described it as remote, may lead to that risk assuming
an undue significance in the patient's calculations. Second, it would seem to me quite
unrealistic in any medical negligence action to confine the expert medical evidence to an
explanation of the primary medical factors involved and to deny the court the benefit of
evidence of medical opinion and practice on the particular issue of disclosure which is
under consideration. Third, the objective test which Canterbury propounds seems to me
to be so imprecise as to be almost meaningless. If it is to be left to individual judges to
decide for themselves what "a reasonable person in the patient's position' would consider
a risk of sufficient significance that he should be told about it, the outcome of litigation in
this field is likely to be quite unpredictable."
Lord Bridge however made it clear that when questioned specifically by the patient about
the risks involved in a particular treatment proposed, the doctor's duty is to answer
truthfully and as fully as the questioner requires. He further held that remote risk of
damage (referred to as risk at 1 or 2%) need not be disclosed but if the risk of damage is
substantial (referred to as 10% risk), it may have to be disclosed. Lord Scarman, in
minority, was inclined to adopt the more stringent test laid down in Canterbury.
25

. In India, Bolam test has broadly been accepted as the general rule. We may refer three
cases of this Court. In Achutrao Haribhau Khodwa vs. State of Maharashtra - 1996 (2)
SCC 634, this Court held : 1996 AIR SCW 919, Paras 15 and 16

"The skill of medical practitioners differs from doctor to doctor. The nature of the
profession is such that there may be more than one course of treatment which may be
advisable for treating a patient. Courts would indeed be slow in attributing negligence on
the part of a doctor if he has performed his duties to the best of his ability and with due
care and caution. Medical opinion may differ with regard to the course of action to be
taken by a doctor treating a patient, but as long as a doctor acts in a manner which is
acceptable to the medical profession and the Court finds that he has attended on the
patient with due care skill and diligence and if the patient still does not survive or suffers
a permanent ailment, it would be difficult to hold the doctor to be guilty of
negligence.........In cases where the doctors act carelessly and in a manner which is not
expected of a medical practitioner, then in such a case an action in torts would be
maintainable."

In Vinitha Ashok vs. Lakshmi Hospital-2001 (8) SCC 731, this 2001 AIR SCW 3881,
Para 28

@page-SC1398
Court after referring to Bolam, Sidaway and Achutrao, clarified :
"A doctor will be liable for negligence in respect of diagnosis and treatment in spite of a
body of professional opinion approving his conduct where it has not been established to
the court's satisfaction that such opinion relied on is reasonable or responsible. If it can be
demonstrated that the professional opinion is not capable of withstanding the logical
analysis, the court would be entitled to hold that the body of opinion is not reasonable or
responsible.

In Indian Medical Association v. V. P. Shantha - 1995 (6) SCC 651, this Court held :
1995 AIR SCW 4463, Para 23

The approach of the courts is to require that professional men should possess a certain
minimum degree of competence and that they should exercise reasonable care in the
discharge of their duties. In general, a professional man owes to his client a duty in tort as
well as in contract to exercise reasonable care in giving advice or performing services".
Neither Achutrao nor Vinitha Ashok referred to the American view expressed in
Canterbury.
26. In India, majority of citizens requiring medical care and treatment fall below the
poverty line. Most of them are illiterate or semi-literate. They cannot comprehend
medical terms, concepts, and treatment procedures. They cannot understand the functions
of various organs or the effect of removal of such organs. They do not have access to
effective but costly diagnostic procedures. Poor patients lying in the corridors of hospitals
after admission for want of beds or patients waiting for days on the roadside for an
admission or a mere examination, is a common sight. For them, any treatment with
reference to rough and ready diagnosis based on their outward symptoms and doctor's
experience or intuition is acceptable and welcome so long as it is free or cheap; and
whatever the doctor decides as being in their interest, is usually unquestioningly
accepted. They are a passive, ignorant and uninvolved in treatment procedures. The poor
and needy face a hostile medical environment - inadequacy in the number of hospitals
and beds, non-availability of adequate treatment facilities, utter lack of qualitative
treatment, corruption, callousness and apathy. Many poor patients with serious ailments
(eg. heart patients and cancer patients) have to wait for months for their turn even for
diagnosis, and due to limited treatment facilities, many die even before their turn comes
for treatment. What choice do these poor patients have? Any treatment of whatever
degree, is a boon or a favour, for them. The stark reality is that for a vast majority in the
country, the concepts of informed consent or any form of consent, and choice in
treatment, have no meaning or relevance.
The position of doctors in Government and charitable hospitals, who treat them, is also
unenviable. They are overworked, understaffed, with little or no diagnostic or surgical
facilities and limited choice of medicines and treatment procedures. They have to
improvise with virtual non-existent facilities and limited dubious medicines. They are
required to be committed, service oriented and non-commercial in outlook. What choice
of treatment can these doctors give to the poor patients? What informed consent they can
take from them?
27. On the other hand, we have the Doctors, hospitals, nursing homes and clinics in the
private commercial sector. There is a general perception among the middle class public
that these private hospitals and doctors prescribe avoidable costly diagnostic procedures
and medicines, and subject them to unwanted surgical procedures, for financial gain. The
public feel that many doctors who have spent a crore or more for becoming a specialist,
or nursing homes which have invested several crores on diagnostic and infrastructure
facilities, would necessarily operate with a purely commercial and not service motive;
that such doctors and hospitals would advise extensive costly treatment procedures and
surgeries, where conservative or simple treatment may meet the need; and that what used
to be a noble service oriented profession is slowly but steadily converting into a purely
business.
28. But unfortunately not all doctors in government hospitals are paragons of service, nor
fortunately, all private hospitals/ doctors are commercial minded. There are many a
doctor in government hospitals who do not care about patients and unscrupulously insist
upon 'unofficial' payment for free treatment or insist upon private consultations. On the
other hand, many private hospitals and Doctors give the best of treatment without
exploitation, at a reasonable
@page-SC1399
cost, charging a fee, which is resonable recompense for the service rendered. Of course,
some doctors, both in private practice or in government service, look at patients not as
persons who should be relieved from pain and suffering by prompt and proper treatment
at an affordable cost, but as potential income-providers / customers who can be exploited
by prolonged or radical diagnostic and treatment procedures. It is this minority who bring
a bad name to the entire profession.
29. Health care (like education) can thrive in the hands of charitable institutions. It also
requires more serious attention from the State. In a developing country like ours where
teeming millions of poor, downtrodden and illiterate cry out for health-care, there is a
desperate need for making healthcare easily accessible and affordable. Remarkable
developments in the field of medicine might have revolutionalized health care. But they
cannot be afforded by the common man. The woes of non-affording patients have in no
way decreased. Gone are the days when any patient could go to a neighbourhood general
practitioner or a family doctor and get affordable treatment at a very reasonable cost, with
affection, care and concern. Their noble tribe is dwindling. Every Doctor wants to be a
specialist. The proliferation of specialists and super specialists, have exhausted many a
patient both financially and physically, by having to move from doctor to doctor, in
search of the appropriate specialist who can identify the problem and provide treatment.
What used to be competent treatment by one General Practitioner has now become multi-
pronged treatment by several specialists. Law stepping in to provide remedy for
negligence or deficiency in service by medical practitioners, has its own twin adverse
effects. More and more private doctors and hospitals have, of necessity, started playing it
safe, by subjecting or requiring the patients to undergo various costly diagnostic
procedures and tests to avoid any allegations of negligence, even though they might have
already identified the ailment with reference to the symptoms and medical history with
90% certainly, by their knowledge and experience. Secondly more and more doctors
particularly surgeons in private practice are forced to cover themselves by taking out
insurance, the cost of which is also ultimately passed on to the patient, by way of a higher
fee. As a consequence, it is now common that a comparatively simple ailment, which
earlier used to be treated at the cost of a few rupees by consulting a single doctor,
requires an expense of several hundred or thousands on account of four factors : (i)
commercialization of medical treatment; (ii) increase in specialists as contrasted from
general practitioners and the need for consulting more than one doctor; (iii) varied
diagnostic and treatment procedures at high cost; and (iv) need for doctors to have
insurance cover. The obvious, may be naive, answer to unwarranted diagnostic
procedures and treatment and prohibitive cost of treatment, is an increase in the
participation of health care by the State and charitable institutions. An enlightened and
committed medical profession can also provide a better alternative. Be that as it may. We
are not trying to intrude on matters of policy, nor are we against proper diagnosis or
specialisation. We are only worried about the enormous hardship and expense to which
the common man is subjected, and are merely voicing the concern of those who are not
able to fend for themselves. We will be too happy if what we have observed is an
overstatement, but our intuition tells us that it is an understatement.
30. What we are considering in this case, is not the duties or obligations of doctors in
government charitable hospitals where treatment is free or on actual cost basis. We are
concerned with doctors in private practice and hospitals and nursing homes run
commercially, where the relationship of doctors and patients are contractual in origin, the
service is in consideration of a fee paid by the patient, where the contract implies that the
professional men possessing a minimum degree of competence would exercise
reasonable care in the discharge of their duties while giving advice or treatment.
31. There is a need to keep the cost of treatment within affordable limits. Bringing in the
American concepts and standards of treatment procedures and disclosure of risks,
consequences and choices will inevitably bring in higher cost-structure of American
medical care. Patients in India cannot afford them. People in India still have great regard
and respect for Doctors. The Members of medical profession have also, by and large,
shown care and concern for the patients. There is an atmosphere of trust and implicit faith
in the advice given by the Doctor.
@page-SC1400
The India psyche rarely questions or challenges the medical advice. Having regard to the
conditions obtaining in India, as also the settled and recognized practices of medical
fraternity in India, we are of the view that to nurture the doctor -patient relationship on
the basis of trust, the extent and nature of information required to be given by doctors
should continue to be governed by the Bolam test rather than the 'reasonably prudential
patient' test evolved in Canterbury. It is for the doctor to decide, with reference to the
condition of the patient, nature of illness, and the prevailing established practices, how
much information regarding risks and consequences should be given to the patients, and
how they should be couched, having the best interests of the patient. A doctor cannot be
held negligent either in regard to diagnosis or treatment or in disclosing the risks involved
in a particular surgical procedure or treatment, if the doctor has acted with normal care, in
accordance with a recognised practices accepted as proper by a responsible body of
medical men skilled in that particular field, even though there may be a body of opinion
that takes a contrary view. Where there are more than one recognized school of
established medical practice, it is not negligence for a doctor to follow any one of those
practices, in preference to the others.
32. We may now summarize principles' relating to consent as follows :
(i) A doctor has to seek and secure the consent of the patient before commencing a
'treatment' (the term 'treatment' includes surgery also). The consent so obtained should be
real and valid, which means that : the patient should have the capacity and competence to
consent; his consent should be voluntary; and his consent should be on the basis of
adequate information concerning the nature of the treatment procedure, so that he knows
what is consenting to.
(ii) The 'adequate information' to be furnished by the doctor (or a member of his team)
who treats the patient, should enable the patient to make a balanced judgment as to
whether he should submit himself to the particular treatment or not. This means that the
Doctor should disclose (a) nature and procedure of the treatment and its purpose, benefits
and effect; (b) alternatives if any available; (c) an outline of the substantial risks; and (d)
adverse consequences of refusing treatment. But there is no need to explain remote or
theoretical risks involved, which may frighten or confuse a patient and result in refusal of
consent for the necessary treatment. Similarly, there is no need to explain the remote or
theoretical risks of refusal to take treatment which may persuade a patient to undergo a
fanciful or unnecessary treatment. A balance should be achieved between the need for
disclosing necessary and adequate information and at the same time avoid the possibility
of the patient being deterred from agreeing to a necessary treatment or offering to
undergo an unnecessary treatment.
(iii) Consent given only for a diagnostic procedure, cannot be considered as consent for
therapeutic treatment. Consent given for a specific treatment procedure will not be valid
for conducting some other treatment procedure. The fact that the unauthorized additional
surgery is beneficial to the patient, or that it would save considerable time and expense to
the patient, or would relieve the patient from pain and suffering in future, are not grounds
of defence in an action in tort for negligence or assault and battery. The only exception to
this rule is where the additional procedure though unauthorized, is necessary in order to
save the life or preserve the health of the patient and it would be unreasonable to delay
such unauthorized procedure until patient regains consciousness and takes a decision.
(iv) There can be a common consent for diagnostic and operative procedures where they
are contemplated. There can also be a common consent for a particular surgical procedure
and an additional or further procedure that may become necessary during the course of
surgery.
(v) The nature and extent of information to be furnished by the doctor to the patient to
secure the consent need not be of the stringent and high degree mentioned in Canterbury
but should be of the extent which is accepted as normal and proper by a body of medical
men skilled and experienced in the particular field. It will depend upon the physical and
mental condition of the patient, the nature of treatment, and the risk and consequences
attached to the treatment.
33. We may note here that courts in Canada and Australia have moved towards
Canterbury standard of disclosure and informed consent - vide Reibl v. Hughes (1980)
114 DLR (3d.) 1 decided by the Canadian Supreme Court and Rogers v. Whittaker -
@page-SC1401
1992 (109) ALR 625 decided by the High Court of Australia. Even in England there is a
tendency to make the doctor's duty to inform more stringent than Bolam's test adopted in
Sidaway. Lord Scarman's minority view in Sidaway favouring Canterbury, in course of
time, may ultimately become the law in England. A beginning has been made in Bolitho
v. City and Hackney HA -1998 1 AC 232 and Pearce v. United Bristol Healthcare NHS
Trust 1998 (48) BMLR 118. We have however, consciously preferred the 'real consent'
concept evolved in Bolam and Sidaway in preference to the 'reasonably prudent patient
test' in Canterbury, having regard to the ground realities in medical and health-care in
India. But if medical practitioners and private hospitals become more and more
commercialized, and if there is a corresponding increase in the awareness of patient's
rights among the public, inevitably, a day may come when we may have to move towards
Canterbury. But not for the present.
Re : Question No. (iii)
34. 'Gynaecology' (second edition) edited by Robert W. Shah, describes 'real consent'
with reference to Gynaecologists (page 867 et seq) as follows :
"An increasingly important risk area for all doctors is the question of consent. No-one
may lay hands on another against their will without running the risk of criminal
prosecution for assault and, if injury results, a civil action for damages for trespass or
negligence. In the case of a doctor, consent to any physical interference will readily be
implied; a woman must be assumed to consent to a normal physical examination if she
consults a gynaecologist, in the absence of clear evidence of her refusal or restriction of
such examination. The problems arise when the gynaecologist's intervention results in
unfortunate side effects or permanent interference with a function, whether or not any
part of the body is removed. For example, if the gynaecologist agrees with the patient to
perform a hysterectomy and removes the ovaries without her specific consent, that will be
a trespass and an act of negligence. The only available defence will be that it was
necessary for the life of the patient to proceed at once to remove the ovaries because of
some perceived pathology in them.
What is meant by consent? The term 'informed consent' is often used, but there is no such
concept in English law. The consent must be real : that is to say, the patient must have
been given sufficient information for her to understand the nature of the operation, its
likely effects, and any complications which may arise and which the surgeon in the
exercise of his duty to the patient considers she should be made aware of; only then can
she reach a proper decision. But the surgeon need not warn the patient of remote risks,
any more than an anaesthetist need warn the patient that a certain small number of those
anaesthetized will suffer cardiac arrest or never recover consciousness. Only where there
is a recognized risk, rather than a rare complication, is the surgeon under an obligation to
warn the patient of that risk. He is not under a duty to warn the patient of the possible
results of hypothetical negligent surgery.........
In advising an operation, therefore, the doctor must do so in the way in which a
competent gynaecologist exercising reasonable skill and care in similar circumstances
would have done. In doing this he will take into account the personality of the patient and
the importance of the operation to her future well being. It may be good practice not to
warn a very nervous patient of any possible complications if she requires immediate
surgery for, say, a malignant condition. The doctor must decide how much to say to her
taking into account his assessment of her personality, the questions she asks and his view
of how much she understands. If the patient asks a direct question, she must be given a
truthful answer...........
To take the example of hysterectomy : although the surgeon will tell the patient that it is
proposed to remove her uterus and perhaps her ovaries, and describe what that will mean
for her future well being (sterility, premature menopause), she will not be warned of the
possibility of damage to the ureter, vesicovaginal fistula, fatal haemorrhage or anaesthetic
death."
35. The specific case of the appellant was that she got herself admitted on 10.5.1995 only
for a diagnostic laparoscopy; that she was not informed either on 9th or 10th that she was
suffering from endometriosis or that her reproductive organs had to be removed to cure
her from the said disease; that her consent was not obtained for the removal of her
reproductive organs; and that when she was under general anaesthesia for diagnostic
@page-SC1402
laparoscopy, respondent came out of the operation theatre and informed her aged mother
that the patient was bleeding profusely which might endanger her life and hysterectomy
was the only option to save her life, and took her consent.
36. The respondent on the other hand contends that on the basis of clinical and ultra
sound examination on 9.5.1995, she had made a provisional diagnosis of endometriosis;
that on same day, she informed the complainant and her mother separately, that she would
do a diagnostic laparoscopy on the next day and if the endometric lesion was found to be
mild or moderate, she will adopt a conservative treatment by operative laparoscopy, but if
the lesion was extensive then considering her age and extent of lesion and likelihood of
destruction of the functions of the tube, a laparotomy would be done; that the appellant
was admitted to the hospital for diagnostic and operative laparoscopy and laparotomy and
appellant's consent was obtained for such procedures; that the decision to operate and
remove the uterus and ovaries was not sudden, nor on account of any emergent situation
developing during laproscopy; and that the radical surgery was authorized, as it was
preceded by a valid consent. She also contends that as the appellant wanted a permanent
cure, the decision to conduct a hysterectomy was medically correct and the surgical
procedure in fact cured the appellant and saved her intestines, bladder and ureter being
damaged due to extension of the lesion. She had also tried to justify the surgical removal
of the uterus and ovaries, with reference to the age and medical condition of the
complainant.
37. The summary of the surgical procedure (dictated by respondent and handwritten by
her assistant Dr. Lata Rangan) furnished to the appellant also confirms that no emergency
or life threatening situation developed during laparoscopy. This is reiterated in the
evidence of respondent and Dr. Lata Rangan. In her affidavit dated 16.2.2002 filed by
way of examination-in-chief, the respondent stated :
"15. The laparoscopic examination revealed a frozen pelvis and considering the extent of
the lesion it was decided that conservative surgery was not advisable and the nature of the
problem required for its cure hysterectomy.
16. When the Deponent decided to perform hysterectomy she told Dr. Lata to intimate the
mother of Ms. Samira Kohli of the fact that hysterectomy was going to be performed on
her. No complications had arisen in the operation theatre and the procedure being
performed was in terms of the consent given by Ms. Samira Kohli herself."
In her affidavit dated 16-2-2002 filed by way of examination-in-chief, Dr. Lata Rangan
stated :
"14.1 was in the Operation Theatre along with Dr. Prabha Manchanda. The laparoscopic
examination revealed a frozen pelvis and considering the extent of the lesion it was
decided that conservative surgery was not possible and that the nature of the problem
required performance of hysterectomy.
15. When it was decided to perform hysterectomy the deponent was told by Dr. Prabha
Manchanda to intimate the mother of Ms. Samira Kohli of the fact that hysterectomy was
now going to be performed on her. No complications had arisen in the Operation Theatre
and the procedure conducted therein was in terms of the consent given by Ms. Samira
Kohli herself. I got the mother to sign the Form too so that the factum of intimation was
duly documented."
Thus, the respondent's definite case is that on 9-5-1995, the respondent had provisionally
diagnosed endometriosis and informed the appellant; that appellant had agreed that
hysterectomy may be performed if the lesion was extensive; and that in pursuance of such
consent, reiterated in writing by the appellant in the consent form on 10-5-1995, she
performed the AH-BSO removing the uterus and ovaries on finding extensive
endometriosis. In other words, according to respondent, the abdominal hysterectomy and
bilateral salpingo-oopherectomy (AH-BSO) was not necessitated on account of any
emergency or life threatening situation developing or being discovered when
laparoscopic test was conducted, but according to an agreed plan, consented by the
appellant and her mother on 9-5-1995 itself, reiterated in writing on 10-5-1995. Therefore
the defence of respondent is one based on specific consent. Let us therefore examine
whether there was consent.
38. The Admission and Discharge card maintained and produced by the respondent
showed that the appellant was admitted "for
@page-SC1403
diagnostic and (?)operative laparoscopy on 10-5-1995". The OPD card dated 9-5-1995
does not refer to endometriosis, which is also admitted by the respondent in her cross-
examination. In fact, the respondent also admitted that the confirmation of diagnosis is
possible only after laparoscopy test :
"On clinical and ultrasound examination a diagnosis can be made to some extent. But
precise diagnosis will have to be on laparoscopy."
The consent form dated 10-5-1995 signed by the appellant states that appellant has been
informed that the treatment to be undertaken is "diagnostic and operative laparoscopy.
Laparotomy may be needed." The case summary dictated by respondent and written by
Dr. Lata Rangan also pearly says "admitted for Hysteroscopy, diagnostic laparoscopy and
operative laparoscopy on 10-5-1995." (Note : Hysteroscopy is inspection of uterus by
special endoscope and laparoscopy is abdominal exploration by special endoscope.)

39. In this context, we may also refer to a notice dated 5.6.1995 issued by respondent to
the appellant through counsel, demanding payment of Rs.39,325/- towards the bill
amount. Paras 1, 3, and 4 are relevant which are extracted below :
"1. You were admitted to our clinic Dr. Manchanda, No.7, Ring Road, Lajpat Nagar, New
Delhi for diagnostic and operative laparoscopy and Endometrial biopsy on 10-5-
1995."...............
"3. The findings of laparoscopy were : a very extensive lesion of the endometriosis with
pools of blood, extensive adherence involving the tubes of the uterus and ovaries, a
chocolate cyst in the right ovary and areas of endometriosis on the surface of the left
ovary but no cyst."
"4. The findings were duly conveyed to Ms. Somi Kohli who was also shown a video
recording of the lesion. You and Mrs. Somi Kohli were informed that conservative
surgery would be futile and removal of the uterus and more extensive surgery,
considering your age and extensive lesion and destruction of the functions of the tubes,
was preferable."
This also makes it clear that the appellant was not admitted for conducting hysterectomy
or bilateral salpingo-oopherectomy, but only for diagnostic purposes. We may, however,
refer to a wrong statement of fact made in the said notice. It states that on 10-5-1995 after
conducting a laparoscopic examination, the video-recording of the lesion was shown to
appellant's mother, and the respondent informed the appellant and her mother that
conservative surgery would be futile and removal of uterus and more extensive surgery
was preferable having regard to the more extensive lesion and destruction of the function
of the tubes. But this statement cannot be true. The extensive nature of lesion and
destruction of the functions obviously became evident only after diagnostic laparoscopy.
But after diagnostic laparoscopy and the video recording of the Lesion, there was no
occasion for respondent to inform anything to appellant. When the laparoscopy and video
recording was made, the appellant was already unconscious. Before she regained
consciousness, AH-BSO was performed removing her uterus and ovaries. Therefore, the
appellant could not have been informed on 10-5-1995 that conservative surgery would be
futile and removal of uterus and extensive surgery was preferable in view of the extensive
lesion and destruction of the function of the tubes did not arise.
40. The admission card makes it clear that the appellant was admitted only for diagnostic
and operative laparoscopy. It does not refer to laparotomy. The consent form shows that
the appellant gave consent only for diagnostic operative laparoscopy, and laparotomy if
needed. Laparotomy is a surgical procedure to open up the abdomen or an abdominal
operation. It refers to the operation performed to examine the abdominal organs and aid
diagnosis. Many a time, after the diagnosis is made and the problem is identified it may
be fixed during the laparotomy itself. In other cases, a subsequent surgery may be
required. Laparotomy can no doubt be either a diagnostic or therapeutic. In the former,
more often referred to as the exploratory laparotomy, an exercise is undertaken to identify
the nature of the disease. In the latter, a therapeutic laparotomy is conducted after the
cause has been identified. When a specific operation say hysterectomy or salpingo-
oopherectomy is planned, laparotomy is merely the first step of the procedure, followed
by the actual specific operation, namely hysterectomy or salpingo-oopherectomy.
Depending upon the incision placement, laparotomy gives access to any abdominal organ
or space and
@page-SC1404
is the first step in any major diagnostic or therapeutic surgical procedure involving a) the
lower part of the digestive tract, b) liver, pancreas and spine, c) bladder, d) female
reproductive organs and e) retroperitonium. On the other hand, hysterectomy and
slapingo-oopherectomy follow laparotomy and are not themselves referred to as
laparotomy. Therefore, when the consent form refers to diagnostic and operative
laparoscopy and "laparotomy if needed", it refers to a consent for a definite laparoscopy
with a contingent laparotomy if needed. It does not amount to consent for OH-BSO
surgery removing the uterus and ovaries/ fallopian tubes. If the appellant had consented
for a OH-BSO then the consent form would have given consent for "diagnostic and
operative laparoscopy. Laparotomy, hysterectomy and bilateral salpingo-oopherectomy, if
needed."
41. On the documentary evidence and the histopathology report the appellant also raised
an issue as to whether appellant was suffering from endometriosis at all. She points out
that ultra-sound did not disclose endometriosis and the histopathology report does not
confirm endometriosis. The respective experts examined on either side have expressed
divergent views as to whether appellant was suffering from endometriosis. It may not be
necessary to give a definite finding on this aspect, as the real question for consideration is
whether appellant gave consent for hysterectomy and bilateral salpingo-oopherectomy
and not whether appellant was suffering from endometriosis. Similarly there is
divergence of expert opinion as to whether removal of uterus and ovaries was the
standard or recognized remedy even if there was endometriosis and whether conservative
treatment was an alternative. Here again it is not necessary to record any finding as to
which is the proper remedy. It is sufficient to note that there are different modes of
treatment favoured by different schools of thought among Gynaecologists.
42. Respondent contended that the term 'laparotomy' is used in the consent form (by her
assistant Dr. Lata Rangan) is equal to or same as hysterectomy. The respondent's
contention that 'Laparotomy' refers to and includes hysterectomy and bilateral salpingo-
oopherectomy cannot be accepted. The following clear evidence of appellant's expert
witness - Dr. Puneet Bedi (CW 1) is not challenged in cross examination :
"Laparotomy is opening up of the abdomen which is quite different from hysterectomy.
Hysterectomy is a procedure which involves surgical removal of uterus. The two
procedures are totally different and consent for each procedure has to be obtained
separately."
On the other hand, the evidence of respondent's expert witness (Dr. Sudha Salhan) on this
question is evasive and clearly implies laparotomy is not the same as hysterectomy. The
relevant portion of her evidence is extracted below :
"Q. As per which medical authority, laparotomy is equal to hysterectomy?
Ans. Consent for laparotomy permits undertaking for such surgical procedure necessary
to treat medical conditions including hysterectomy.
Q. I put it to you that the medical practice is to take specific consent for hysterectomy.
Ans. Whenever we do hysterectomy only, specific consent is obtained."
43. Medical texts and authorities clearly spell out that Laparotomy is at best the initial
step that is necessary for performing hysterectomy or salpingo-oopherectomy.
Laparotomy by itself is not hysterectomy or salpingo-oopherectomy. Nor does
'hysterectomy' include salpingo-oopherectomy, in the case of woman who has not
attained menopause. Laparotomy does not refer to surgical removal of any vital or
reproductive organs. Laparotomy is usually exploratory and once the internal organs are
exposed and examined and the disease or ailment is diagnosed, the problem may be
addressed and fixed during the course of such laparotomy (as for example, removal of
cysts and fulguration of endometric area as stated by respondent herself as a conservative
form of treatment). But Laparotomy is never understood as referring to removal of any
organ. In medical circles, it is well recognized that a catch all clause giving the surgeon
permission to do anything necessary does not give roving authority to remove whatever
he fancies may be for the good of the patient. For example, a surgeon cannot construe a
consent to termination of pregnancy as a consent to sterilize the patient.
44. When the oral and documentary evidence is considered in the light of the legal
position discussed above while answering
@page-SC1405
questions (i) and (ii), it is clear that there was no consent by the appellant for conducting
hysterectomy and bilateral salpingo-oopherectomy.
45. The Respondent next contended that the consent given by the appellant's mother for
performing hysterectomy should be considered as valid consent for performing
hysterectomy and salpingo-oopherectomy. The appellant was neither a minor, nor
mentally challenged, nor incapacitated. When a patient is a competent adult, there is no
question of someone else giving consent on her behalf. There was no medical emergency
during surgery. The appellant was only temporarily unconscious, undergoing only a
diagnostic procedure by way of laparoscopy. The respondent ought to have waited till the
appellant regained consciousness, discussed the result of the laparoscopic examination
and then taken her consent for the removal of her uterus and ovaries. In the absence of an
emergency and as the matter was still at the stage of diagnosis, the question of taking her
mother's consent for radical surgery did not arise. Therefore, such consent by mother
cannot be treated as valid or real consent. Further a consent for hysterectomy, is not a
consent for bilateral salpingo-oophorectomy.
46. There is another facet of the consent given by the appellant's mother which requires to
be noticed. The respondent's specific case is that the appellant had agreed for the surgical
removal of uterus and ovaries depending upon the extent of the lesion. It is also her
specific case that the consent by signing the consent form on 10-5-1995 wherein the
treatment is mentioned as "diagnostic and operative laparoscopy. Laparotomy may be
needed," includes the AH-BSO surgery for removal of uterus and ovaries. If the term
'laparotomy' is to include hysterectomy and salpingo-oophorectomy as contended by the
respondent and there was a specific consent by the appellant in the consent form signed
by her on 10-5-1995, there was absolutely no need for the respondent to send word
through her assistant Dr. Lata Rangan to get the consent of appellant's mother for
performing hysterectomy under general anesthesia. The very fact that such consent was
sought from appellant's mother for conducting hysterectomy is a clear indication that
there was no prior consent for hysterectomy by the appellant.
47. We may, therefore, summarize the factual position thus :
(i) On 9-5-1995 there was no confirmed diagnosis of endometriosis. The OPD slip does
not refer to a provisional diagnosis of endometriosis on the basis of personal examination.
Though there is a detailed reference to the findings of ultrasound in the entry relating to
9-5-1995 in the OPD slip, there is no reference to endometriosis which shows that
ultrasound report did not show endometriosis. In fact, ultra-sound may disclose fibroids,
chocolate cyst or other abnormality which may indicate endometriosis, but cannot by
itself lead to a diagnosis of endometriosis. This is evident from the evidence of CW1,
RW1 and RW2 and recognized text books. In fact respondent's expert Dr. Sudha Salhan
admits in her cross-examination that endometriosis can only be suspected but not
diagnosed by ultrasound and it can be confirmed only by laparoscopy. Even according to
respondent, endometriosis was confirmed only by laparoscopy. [Books on "Gynaecology"
clearly state : "The best means to diagnose endometriosis is by direct visualization at
laparoscopy or laparotomy, with histological confirmation where uncertainty persists."]
Therefore the claim of respondent that she had discussed in detail about endometriosis
and the treatment on 9-5-1995 on the basis of her personal examination and ultra-sound
report appears to be doubtful.
(ii) The appellant was admitted only for diagnostic laparoscopy (and at best for limited
surgical treatment that could be made by laparoscopy). She was not admitted for
hysterectomy or bilateral salpingo-oophorectomy.
(iii) There was no consent by appellant for hysterectomy or bilateral salpingo-
oopherectomy. The words "Laparotomy may be needed" in the consent form dated
10.5.1995 can only refer to therapeutic procedures which are conservative in nature (as
for example removal of chocolate cyst and fulguration of endometric areas, as stated by
respondent herself as a choice of treatment), and not radical surgery involving removal of
important organs.
48. We find that the Commission has, without any legal basis, concluded that "the
informed choice has to be left to the operating
@page-SC1406
Surgeon depending on his/her discretion, after assessing the damage to the internal
organs, but subject to his/her exercising care and caution". It also erred in construing the
words "such medical treatment as is considered necessary for me for......" in the consent
form as including surgical treatment by way of removal of uterus and ovaries. The
Commission has also observed : "whether the uterus should have been removed or not or
some other surgical procedure should have been followed are matters to be left to the
discretion of the performing Surgeon, as long as the Surgeon does the work with adequate
care and caution". This proceeds on the erroneous assumption that where the Surgeon has
shown adequate care and caution in performing the surgery, the consent of the patient for
removal of an organ is unnecessary. The Commission failed to notice that the question
was not about the correctness of the decision to remove the uterus and ovaries, but the
failure to obtain the consent for removal of those important organs. There was a also faint
attempt on the part of the respondent's counsel to contend that what were removed were
not 'vital' organs and having regard to the advanced age of the appellant, as procreation
was not possible, uterus and ovaries were virtually redundant organs. The appellant's
counsel seriously disputes the position and contends that procreation was possible even at
the age of 44 years. Suffice it to say that for a woman who has not married and not yet
reached menopause, the reproductive organs are certainly important organs. There is also
no dispute that removal of ovaries leads to abrupt menopause causing hormonal
imbalance and consequential adverse effects.
Re : Question Nos.(iv) and (v) :
49. The case of the appellant is that she was not suffering from endometriosis, and
therefore, there was no need to remove the uterus and ovaries. In this behalf, she
examined Dr. Puneet Bedi (Obstetrician and Gynaecologist) who gave hormone therapy
to appellant for about two years prior to his examination in 2002. He stated that the best
method to diagnose endometriosis is diagnostic laparoscopy; that the presence of
endometrial tissue anywhere outside the uterus is called Endometriosis; that the
Histopathology report did not confirm endometriosis in the case of appellant; and that the
mode of treatment for endometriosis would depend on the existing extent of the disease.
He also stated that removal of uterus results in abrupt menopause. In natural menopause,
which is a slow process, the body gets time to acclimatize to the low level of hormones
gradually. On the other hand when the ovaries are removed, there is an abrupt stoppage of
natural hormones and therefore Hormone Replacement Therapy is necessary to make up
the loss of natural hormones. Hormone Replacement Therapy is also given even when
there is a natural menopause. But hormone replacement therapy has side effects and
complications. He also stated that on the basis of materials available on the file, he was of
the view that Hysterectomy was not called for immediately. But if endometriosis had
been proven from history and following diagnostic laparoscopy, hysterectomy could be
considered as a last resort if all other medical methods failed. What is relevant from the
evidence of Dr. Puneet Bedi, is that he does not say that hysterectomy is not the remedy
for endometriosis, but only that it is a procedure that has to be considered as a last resort.
50. On the other hand, the respondent who is herself a experienced Obstetrician and
Gynaecologist has given detailed evidence, giving the reasons for diagnosing the problem
of appellant as endometriosis and has referred to in detail, the need for the surgery. She
stated that having regard to the medical condition of complainant, her decision to perform
hysterectomy was medically correct. The complainant wanted a cure for her problem and
the AH-BSO surgery provided her such cure, apart from protecting her against any future
damage to intestines, bladder and ureter. She explained that if the uterus and ovaries had
not been removed there was a likelihood of lesion extending to the intestines causing
bleedings, fibrosis and narrowing of the gut; the lesion could also go to the surface of the
bladder penetrating the wall and causing haematuria and the ureter could be damaged due
to fibrosis leading to damage of the kidney; there was also a chance of development of
cancer also. She also pointed out that the complainant being 44 years of age, was in the
premenopausal period and had menorrhagia which prevented regular ovulation which
@page-SC1407
was necessary for pregnancy; that endometriosls also prevented fertilization and
produced reaction in the pelvis which increased lymphocytes and macrophages which
destroy the ova and sperm; and that the state of bodily health did not depend upon the
existence of uterus and ovaries.
51. The respondent also examined Dr. Sudha Salhan, Professor and Head of Department
(Obstetrics and Gynaecology) and President of the Association of Obstetricians and
Gynaecologists of Delhi. Having seen the records relating to appellant including the
record pertaining to clinical and ultra-sound examinations, she was of the view that the
treatment given to appellant was correct and appropriate to appellant's medical condition.
She stated that the treatment is determined by severity of the disease and hysterectomy
was not an unreasonable option as there was no scope left for fecundability in a woman
aged 44 years suffering from endometriosis. She also stated that the histopathology report
dated 15.5.1995 confirmed the diagnosis of endometriosis made by respondent. She also
stated that she saw video-tape of the laparoscopic examination and concurred that the
opinion of respondent that the lesion being extensive conservation surgery was not
possible and the problem could effectively be addressed only by more extensive surgery
that is removal of the uterus and ovaries. She also stated that the presence of chocolate
cyst was indicative of endometriosis. She also stated that medication merely suppresses
endometriosis and the definitive treatment was surgical removal of the uterus and both
the ovaries. She also stated that hysterectomy is done when uterus comes out from a
prolapse and the woman is elderly, or when there is a cancer of the uterus, or when there
are massive fibroids or when a severe grade of endometriosis along with ovaries or in
cases of malignancy or the cancer of the ovaries.
52. The evidence therefore demonstrates that on laparoscopic examination, respondent
was satisfied that appellant was suffering from endometriosis. The evidence also
demonstrates that there is more than one way of treating endometriosis. While one view
favours conservative treatment with hysterectomy as a last resort, the other favours
hysterectomy as a complete and immediate cure. The age of the patient, the stage of
endometriosis among others will be determining factors for choosing the method of
treatment. The very suggestion made by appellant's counsel to the expert witness Dr.
Sudha Salhan that worldwide studies show that most hysterectomies are conducted
unnecessarily by Gynaecologists demonstrates that it is considered as a favoured
treatment procedure among medical fraternity, offering a permanent cure. Therefore
respondent cannot be held to be negligent, merely because she chose to perform radical
surgery in preference to conservative treatment. This finding, however, has no bearing on
the issue of consent which has been held against the respondent. The correctness or
appropriateness of the treatment procedure, does not make the treatment legal, in the
absence of consent for the treatment.
53. It is true that the appellant has disputed the respondent's finding that she was suffering
from endometriosis. The histopathology report also does not diagnose any endometriosis.
The expert witness examined on behalf of the appellant has also stated that there was no
evidence that the appellant was suffering from endometriosis. On the other hand the
respondent has relied on some observations of the histopathology report and on her own
observations which has been recorded in the case summary to conclude that the appellant
was suffering from endometriosis. The evidence shows that the respondent having found
evidence of endometriosis, proceeded on the basis that removal of uterus and ovaries was
beneficial to the health of the appellant having regard to the age of the appellant and
condition of the appellant to provide a permanent cure to her ailment, though not
authorized to do so. On a overall consideration of the evidence, we are not prepared to
accept the claim of appellant that the respondent falsely invented a case that the appellant
was suffering from endometriosis to cover up some negligence on her part in conducting
the diagnostic/operative laparoscopy or to explain the unauthorized and unwarranted
removal of uterus and ovaries.
Re : Question No. (vi) :
54. In view of our finding that there was no consent by the appellant for performing
hysterectomy and salpingo-oophorectomy, performance of such surgery was an
unauthorized invasion and interference with appellant's body which amounted to a
tortuous
@page-SC1408
act of assault and battery and therefore a deficiency in service. But as noticed above,
there are several mitigating circumstances. The respondent did it in the interest of the
appellant. As the appellant was already 44 years old and was having serious menstrual
problems, the respondent thought that by surgical removal of uterus and ovaries she was
providing permanent relief. It is also possible that the respondent thought that the
appellant may approve the additional surgical procedure when she regained
consciousness and the consent by appellant's mother gave her authority. This is a case of
respondent acting in excess of consent but in good faith and for the benefit of the
appellant. Though the appellant has alleged that she had to undergo Hormone Therapy, no
other serious repercussions is made out as a result of the removal. The appellant was
already fast approaching the age of menopause and in all probability required such
Hormone Therapy. Even assuming that AH-BSO surgery was not immediately required,
there was a reasonable certainty that she would have ultimately required the said
treatment for a complete cure. On the facts and circumstances, we consider that interests
of justice would be served if the respondent is denied the entire fee charged for the
surgery and in addition, directed to pay Rs. 25,000 as compensation for the unauthorized
AH-BSO surgery to the appellant.
55. We accordingly allow this appeal and set aside the order of the Commission and allow
the appellant's claim in part. If the respondent has already received the bill amount or any
part thereof from the appellant (either by executing the decree said to have been obtained
by her or otherwise), the respondent shall refund the same to the appellant with interest at
the rate of 10% per annum from the date of payment till the date of re-payment. The
Respondent shall pay to the appellant a sum of Rs. 25,000/-as compensation with interest
thereon at the rate of 10% per annum from 19-11-2003 (the date of the order of
Commission) till date of payment. The appellant will also be entitled to costs of Rs. 5,000
from the respondent.
Appeal allowed.
AIR 2008 SUPREME COURT 1408 "Oriental Insurance Co. Ltd. v. Prithvi Raj"
Coram : 2 Dr. A. PASAYAT AND P. SATHASIVAM, JJ.
Civil Appeal No. 648 of 2008 (arising out of SLP (C) No. 12607 of 2005), D/- 24 -1
-2008*.
Oriental Insurance Co. Limited v. Prithvi Raj.
Motor Vehicles Act (59 of 1988), S.149 - MOTOR VEHICLES - INSURANCE -
WITNESS - Liability of Insurance Company - Accident - Driver of offending vehicle,
alleged to be not having valid and operating driving licence - Evidence of witness who
produced official records clearly established that no driving licence was issued to said
driver in order to enable and legally permit him to drive motor vehicle - There was no
cross-examination of said witness - Liability could not be saddled on Insurance
Company.
2007 AIR SCW 2279, Followed. (Paras 10, 11)
Cases Referred : Chronological Paras
2007 AIR SCW 2279 : AIR 2007 SC 1563 (Foll.) 8, 9
2006 AIR SCW 4832 (Ref.) 6
2004 AIR SCW 663 : AIR 2004 SC 1531 (Ref.) 6, 8
2004 AIR SCW 5481 : AIR 2004 SC 4794 : 2004 All LJ 3628 (Ref.) 8
2004 AIR SCW 6924 : AIR 2005 SC 286 : 2005 All LJ 206 (Ref.) 8
2003 AIR SCW 1695 : AIR 2003 SC 1292 (Ref.) 4, 5
2001 AIR SCW 1340 : AIR 2001 SC 1419 (Ref.) 8
1995 AIR SCW 4520 : AIR 1996 SC 586 (Ref.) 8
Kishore Rawat and M.K. Dua, for Appellant; J.S. Attri, for Respondent.
* From order of the National Consumer Disputes Redressal Commission, New Delhi in
F.A. No. 823 of 2003, D/- 9-3-2005.
Judgement
1. Dr. ARIJIT PASAYAT, J. :-Heard learned counsel for the parties.
2. Leave granted.
3. Challenge in this appeal is to the order passed by the National Consumer Disputes
Redressal Commission, New Delhi (in short 'the Commission') allowing the First Appeal
filed by the appellant before it (the respondent herein). He is hereinafter referred to as the
complainant. Before the Himachal Pradesh State Consumer Disputes
@page-SC1409
Redressal Commission (in short the "State Commission"), the complainant had filed a
complaint alleging that a Mini Bus owned by the complainant met with an accident
during the period when the Insurance Cover/policy issued by the appellant-Insurance
Company was in currency. The incident was reported to the Insurance Company but the
claim was not settled on the ground that the Driver of the offending vehicle did not have a
valid and operating driving license. The complainant took the stand that there was a
renewal of the driving license which was valid and legal and, therefore, the claim could
not have been repudiated by the Insurance Company. The State Commission rejected the
plea, categorically holding that there was no valid license issued by the R.T.A,
Hyderabad, as claimed by the Driver.
4

. In appeal by the impugned order, a contrary view was taken and it was held that it was
accepted, as rightly noted by the State Commission, that the licensing authority at
Hyderabad had not issued any license as claimed. Yet, in view of the fact that there was a
renewal at Tinsukhia, the claim could not have been refused by the insurance company.
Reliance was placed on a decision of this Court in United India Insurance Co. Limited v.
Lehru and Ors. (2003 (3) SCC 338) in support of the view. 2003 AIR SCW 1695

5. Learned counsel for the appellant-insurance company submitted that Lehru's case
(supra) related to a third party claim and not an own damage claim.
6

. Learned counsel for the respondent, on the other hand, relied on a decision of this Court
in Lal Chand v. Oriental Insurance Co. Ltd. (2006 (8) SCALE 531) to contend that the
view taken by the National Commission was correct. Reliance has also been placed on a
decision of this Court in National Insurance Co. Ltd. v. Swaran Singh and Ors. (2004 (3)
SCC 297). 2006 AIR SCW 4832
2004 AIR SCW 663

7. It is to be noted that Swaran Singh's case (supra) was rendered in the background of
Section 149 of the Motor Vehicles Act, 1988 (in short the 'Act').
8

. This Court had occasion to deal with a similar issue in National Insurance Co. Ltd. v.
Laxmi Narain Dhut (2007 (3) SCR 579). It was inter alia held as follows : 2007 AIR
SCW 2279

"8. Section 149 of the Act relates to duty of insurers to satisfy judgments and awards
against persons insured in respect of third party risks. The language of the provision is
clear that it only relates to third party risk. The corresponding provision in the old Act is
Section 96. Section 166 of the Act relates to application for compensation. The same
corresponds to Section 110-A of the old Act. Section 168 of the Act relates to award of
the Claims Tribunal which corresponds to Section 110-B of the old Act. Section 170 deals
with impleadment of the insurer in certain cases. Section 149 of the Act needs to be noted
in full. The same reads as follows :
"149. Duty of Insurers to satisfy judgments and awards against persons insured in respect
of third party risks- (1) If, after a certificate of insurance has been issued under sub-
section (3) of Section 147, in favour of the person by whom a policy has been effected,
judgment or award in respect of any such liability as is required to be covered by a policy
under clause (b) of sub-section (1) of Section 147 (being a liability covered by the terms
of the policy) or under the provisions of Section 163-A) is obtained against any person
insured by the policy then, notwithstanding that the insurer may be entitled to avoid or
cancel or may have avoid or cancelled the policy, the insurer shall, subject to the
provisions of this section, pay to the person entitled to the benefit of the decree any sum
not exceeding the sum assured payable thereunder, as if were the judgment-debtor, in
respect of the liability, together with any amount payable in respect of costs and any sum
payable in respect of interest on that sum by virtue of any enactment relating to interest
on judgments. (2) No sum shall be payable by an insurer under sub-section (1) in respect
of any judgment or award unless, before the commencement of the proceedings in which
the judgment or award is given the insurer had notice through the Court or, as the case
may be, the Claims Tribunal of the bringing of the proceedings, or in respect of such
judgment or award so long as execution is stayed thereon pending an appeal; and an
insurer to whom notice of the bringing of any such proceedings is so given shall be
entitled to be made a party thereto and to defend the action on any of the following
grounds, namely :-
(a) that there has been a breach of a specified
@page-SC1410
condition of the policy, being one of the following conditions, namely :-
(i) a condition excluding the use of the vehicle -
(a) for hire or reward, where the vehicle is on the date of the contract of insurance a
vehicle not covered by a permit to ply for hire or reward, or
(b) for organized racing and speed testing, or
(c) for a purpose not allowed by the permit under which the vehicle is used, where the
vehicle is a transport vehicle, or
(d) without side-car being attached where the vehicle is a motor cycle; or
(ii) a condition excluding driving by a named person or persons or by any person who is
not duly licensed, or by any person who has been disqualified for holding or obtaining a
driving licence during the period of disqualification; or
(iii) a condition excluding liability for injury caused or contributed to by conditions of
war, civil war, riot or civil commotion; or
(b) that the policy is void on the ground that it was obtained by the non-disclosure of a
material fact or by a representation of fact which was false in some material particular.
(3) Where any such judgment as is referred to in sub-section (1) is obtained from a Court
in a reciprocating country and in the case of a foreign Judgment is, by virtue of the
provisions of section 13 of the Code of Civil Procedure, 1908 (5 of 1908) conclusive as
to any matter adjudicated upon by it, the insurer (being an insurer registered under the
Insurance Act, 1938 (4 of 1938) and whether or not he is registered under the
corresponding law of the reciprocating country) shall be liable to the person entitled to
the benefit of the decree in the manner and to the extent specified in sub-section (1), as if
the judgment were given by a Court in India :
Provided that no sum shall be payable by the insurer in respect of any such judgment
unless, before the commencement of the proceedings in which the judgment is given, the
insurer had notice through the Court concerned of the bringing of the proceedings and the
insurer to whom notice is so given is entitled under the corresponding law of the
reciprocating country, to be made a party to the proceedings and to defend the action on
grounds similar to those specified in sub-section (2).
(4) Where a certificate of insurance has been issued under sub-section (3) of section 147
to the person by whom a policy has been effected, so much of the policy as purports to
restrict the insurance of the persons insured thereby by reference to any condition other
than those in clause (b) of sub-section (2) shall, as respects such liabilities as are required
to be covered by a policy under clause (b) of sub-section (1) of section 147, be of no
effect :
Provided that any sum paid by the insurer in or towards the discharge of any liability of
any person which is covered by the policy by virtue only of this sub-section shall be
recoverable by the insurer from that person.
(5) If the amount which an insurer becomes liable under this section to pay in respect of a
liability incurred by a person insured by a policy exceeds the amount for which the
insurer would apart from the provisions of this section be liable under the policy in
respect of that liability, the insurer shall be entitled to recover the excess from that
person.
(6) In this section the expression "material fact" and "material particular" means,
respectively a fact or particular of such a nature as to influence the judgment of a prudent
insurer in determining whether he will take the risk and, if so, at what premium and on
what conditions, and the expression "liability covered by the terms of the policy" means a
liability which is covered by the policy or which would be so covered but for the fact that
the insurer is entitled to avoid or cancel or has avoided or cancelled the policy.
(7) No insurer to whom the notice referred to in sub-section (2) or sub-section (3) has
been given shall be entitled to avoid his liability to any person entitled to the benefit of
any such judgment or award as is referred to in sub-section (1) or in such judgment as is
referred to in sub-section (3) otherwise than in the manner provided for in sub-section (2)
or in the corresponding law of the reciprocating country, as the case may be.
Explanation: For the purposes of this section, "Claims Tribunal" means a Claims Tribunal
constituted under Section 165 and "award" means an award made by that Tribunal under
Section 168."
@page-SC1411

"9. In Swaran Singh's case (supra) on which learned counsel for the parties have placed
reliance undisputedly related to a case under Section 149 of the Act. This Court
elaborately dealt with the scope and ambit of Sections 147 and 149 of the Act and after
tracing the history of compulsory insurance and the rights of the third parties, held that
the concerned cases were mainly concerned with third party rights under the policy. It
was held in that context that any condition in the policy whereby the right of the third
party is taken away would be void, as noted in para 23 of the judgment. 2004 AIR
SCW 663
10. In paras 69 and 70 the principles were culled out in the following terms :
"The Insurance Company is required to prove the breach of the condition of the contract
of insurance by cogent evidence. In the event the Insurance Company falls to prove that
there has been breach of conditions of the policy on the part of the insured, the Insurance
Company cannot be absolved of its liability. This Court did not lay down a degree of
proof, but held that the parties alleging the breach must be held to have succeeded in
establishing the breach of the condition of the contract of insurance, on the part of the
Insurance Company by discharging its burden of proof. The Tribunal, must arrive at a
finding on the basis of the materials available on the records."
11. In para 110 also the summary of the findings were recorded which reads as follows :
(i) Chapter XI of the Motor Vehicles Act, 1988 providing compulsory insurance of
vehicles against third-party risks is a social welfare legislation to extend relief by
compensation to victims of accidents caused by use of motor vehicles. The provisions of
compulsory insurance coverage of all vehicles are with this paramount object and the
provisions of the Act have to be so interpreted as to effectuate the said object.
(ii) An insurer is entitled to raise a defence in a claim petition filed under Section 163-A
or Section 166 of the Motor Vehicles Act, 1988, inter alia, in terms of Section 149(2)(a)
(ii) of the said Act.
(iii) The breach of policy condition e.g. disqualification of the driver or invalid driving
licence of the driver, as contained in subsection (2)(a)(ii) of Section 149, has to be proved
to have been committed by the insured for avoiding liability by the insurer. Mere absence,
fake or invalid driving licence or disqualification of the driver for driving at the relevant
time, are not in themselves defences available to the insurer against either the insured or
the third parties. To avoid its liability towards the insured, the insurer has to prove that
the insured was guilty of negligence and failed to exercise reasonable care in the matter
of fulfilling the condition of the policy regarding use of vehicles by a duly licensed driver
or one who was not disqualified to drive at the relevant time.
(iv) Insurance companies, however, with a view to avoid their liability must not only
establish the available defence(s) raised in the said proceedings but must also establish
"breach" on the part of the owner of the vehicle; the burden of proof wherefore would be
on them.
(v) The court cannot lay down any criteria as to how the said burden would be
discharged, Inasmuch as the same would depend upon the facts and circumstances of
each case.
(vi) Even where the insurer is able to prove breach on the part of the insured concerning
the policy condition regarding holding of a valid licence by the driver or his qualification
to drive during the relevant period, the insurer would not be allowed to avoid its liability
towards the insured unless the said breach or breaches on the condition of driving licence
is/are so fundamental as are found to have contributed to the cause of the accident. The
Tribunals in interpreting the policy conditions would apply "the rule of main purpose"
and the concept of "fundamental breach" to allow defences available to the insurer under
Section 149(2) of the Act.
(vii) The question, as to whether the owner has taken reasonable care to find out as to
whether the driving licence produced by the driver (a fake one or otherwise), does not
fulfil the requirements of law or not will have to be determined in each case.
(viii) If a vehicle at the time of accident was driven by a person having a learner's licence,
the insurance companies would be liable to satisfy the decree.
(ix) The Claims Tribunal constituted under Section 165 read with Section 168 is
empowered to adjudicate all claims in respect of the accidents involving death or of
bodily injury or damage to property of third
@page-SC1412
party arising in use of motor vehicle. The said power of the Tribunal is not restricted to
decide the claims inter se between claimant or claimants on one side and insured, insurer
and driver on the other. In the course of adjudicating the claim for compensation and to
decide the availability of defence or defences to the insurer, the Tribunal has necessarily
the power and jurisdiction to decide disputes inter se between the insurer and the insured.
(x) The decision rendered on the claims and disputes inter se between the insurer and
insured in the course of adjudication of claim for compensation by the claimants and the
award made thereon is enforceable and executable in the same manner as provided in
Section 174 of the Act for enforcement and execution of the award in favour of the
claimants.
(x) Where on adjudication of the claim under the Act the Tribunal arrives at a conclusion
that the insurer has satisfactorily proved its defence in accordance with the provisions of
Section 149(2) read with subsection (7), as interpreted by this Court above, the Tribunal
can direct that the insurer is liable to be reimbursed by the insured for the compensation
and other amounts which it has been compelled to pay to the third party under the award
of the Tribunal. Such determination of claim by the Tribunal will be enforceable and the
money found due to the insurer from the insured will be recoverable on a certificate
issued by the Tribunal to the Collector in the same manner under Section 174 of the Act
as arrears of land revenue. The certificate will be issued for the recovery as arrears of
land revenue only if, as required by sub-section (3) of Section 168 of the Act the insured
fails to deposit the amount awarded in favour of the insurer within thirty days from the
date of announcement of the award by the Tribunal.
(xi) The provisions contained in subsection (4) with the proviso thereunder and sub-
section (5) which are intended to cover specified contingencies mentioned therein to
enable the insurer to recover the amount paid under the contract of insurance on behalf of
the insured can be taken recourse to by the Tribunal and be extended to claims and
defences of the insurer against the insured by relegating them to the remedy before
regular court in cases where on given facts and circumstances adjudication of their claims
inter se might delay the adjudication of the claims of the victims."
12. At this juncture, it would be necessary to test the logic behind Section 149 of the Act.
The conditions under the said provision relate only to third party risks and claims.
17. Section 149 is part of Chapter XI which is titled "Insurance of Motor Vehicles against
Third Parties". A significant factor which needs to be noticed is that there is no
contractual relation between the insurance company and the third party. The liabilities
and the obligations relatable to third parties are created only by fiction of Sections 147
and 149 of the Act.

18. It is also to be noted that the terms of the policy have to be construed as it is and there
is no scope for adding or subtracting something. However liberally the policy may be
construed, such liberalism cannot be extended to permit substitution of words which are
not intended. (See United India Insurance Co. Ltd. v. Harchand Rai 1 Chandan Lal (2004
(8) SCC 644 and Polymat India (P) Ltd. V. National Insurance Company Ltd. and Ors.
(2005 (9) SCC 174). 2004 AIR SCW 5481
2004 AIR SCW 6924

19. The primary stand of the insurance company is that the person driving the vehicle did
not have a valid driving license. In Swaran Singh's case (supra) the following situations
were noted : 2004 AIR SCW 663

(i) the driver had a license but it was fake;


(ii) the driver had no license at all;
(iii) the driver originally had a valid license but it had expired as on the date of the
accident and had not been renewed;
(iv) the license was for a class of vehicles other than that which was the insured vehicle;
(v) the license was a learner's license.
Category (i) may cover two types of situations. First, the license itself was fake and the
second is where originally that license is fake but there has been a renewal subsequently
in accordance with law.
20. Chapter II contains Sections 3, 4 and 5 of the Act relating to licensing of drivers
driving the motor vehicles.
24. In the background of the statutory provisions, one thing is crystal clear i.e. the statute
is beneficial one qua the third party. But that benefit cannot be extended to the
@page-SC1413
owner of the offending vehicle. The logic of fake license has to be considered differently
in respect of third party and in respect of own damage claims.

25. It would be appropriate to take note of what was stated in Complete Insulations (P)
Ltd. v. New India Assurance Co. Ltd. (1996 (1) SCC 221). In paras 9 and 10, it was
observed as follows : 1995 AIR SCW 4520

"9. Section 157 appears in Chapter XI entitled "Insurance of Motor Vehicles against Third
Party Risks" and comprises Sections 145 to 164. Section 145 defines certain expressions
used in the various provisions of that Chapter. The expression "Certificate of Insurance"
means a certificate issued by the authorised insurer under Section 147(3). "Policy of
Insurance" includes a certificate of insurance. Section 146(1) posits that "no person shall
use except as a passenger, or cause or allow any other person to use, a motor vehicle in a
public place, unless there is in force in relation to the use of the vehicle by that person or
that other person, as the case may be, a policy of insurance complying with the
requirements of this chapter". Of course this provision does not apply to vehicles owned
by the Central or State Government and used for Government purposes not connected
with any commercial enterprise. This provision corresponds to Section 94 of the old Act.
Section 147 provides that the policy of insurance to be issued by the authorized insurer
must insure the specified person or classes of persons against any liability incurred in
respect of death of or bodily injury to any person or damage to any property of a third
party as well as against the death of or bodily injury caused to any passenger of a public
service vehicle caused by or arising out of the use of the vehicle in a public place. This
provision is akin to Section 95 of the old Act. It will be seen that the liability extends to
damage to any property of a third party and not damage to the property of the owner of
the vehicle, i.e., the insured. Sub-section (2) stipulates the extent of liability and in the
case of property of a third party the limit of liability is Rupees six thousand only. The
proviso to that sub-section continues the liability fixed under the policy for four months
or till the date of its actual expiry, whichever is earlier, sub-section (3) next provides that
the policy of insurance shall be of no effect unless and until the insurer has issued a
certificate of insurance in the prescribed form. The next important provision which we
may notice is Section 156 which sets out the effect of the certificate of insurance. It says
that when the insurer issues the certificate of insurance, then even if the policy of
insurance has not as yet been issued the insurer shall, as between himself and any other
person except the insured be deemed to have issued to the insured a policy of insurance
conforming in all respects with the description and particulars stated in the certificate. It
is obvious on a plain reading of this provision that the legislature was anxious to protect
third-party interest. Then comes Section 157 which we extracted earlier. This provision
lays down that when the owner of vehicle in relation whereto a certificate of insurance is
issued transfers to another person the ownership of the motor vehicle, the certificate of
insurance together with the policy described therein shall be deemed to have been
transferred in favour of the new owner of the vehicle with effect from the date of transfer.
Sub-section (2) requires the transferee to apply within fourteen days from the date of
transfer to the insurer for making necessary changes in the certificate of insurance and the
policy described therein in his favour. These are the relevant provisions of Chapter XI
which have a bearing on the question of insurer's liability in the present case.
10. There can be no doubt that the said chapter provides for compulsory insurance of
vehicles to cover third-party risks. Section 146 forbids the use of a vehicle in a public
place unless there is in force in relation to the use of that vehicle a policy of insurance
complying with the requirements of that chapter. Any breach of this provision may attract
penal action. In the case of property, the coverage extends to property of a third parry i.e.
a person other than the insured. This is clear from Section 147(l)(b)(i) which clearly
refers to "damage to any property of a third party" and not damage to the property of the
'insured' himself. And the limit of liability fixed for damage to property of a third party is
Rupees six thousand only as pointed out earlier. That is why even the Claims Tribunal
constituted under Section 165 is invested with jurisdiction to adjudicate upon claims for
compensation in respect of accidents involving death of or bodily injury to persons
arising out of the use of motor vehicles, or damage
@page-SC1414
to any property of a third party so arising, or both. Here also it is restricted to damage to
third-party property and not the property of the insured."
26. The restrictions relating to appeal in terms of Section 173 (2) does not apply to own
damage cases.

38. The inevitable conclusion therefore is that the decision in Swaran Singh's case (supra)
has no application to own damage cases. The effect of fake license has to be considered
in the light of what has been stated by this Court in New India Assurance Co., Shimla v.
Kamla and Ors. (2001 (4) SCC 342). Once the license is a fake one the renewal cannot
take away the effect of fake license. It was observed in Kamla's case (supra) as follows :
2004 AIR SCW 663
2001 AIR SCW 1340

"12. As a point of law we have no manner of doubt that a fake licence cannot get its
forgery outfit stripped off merely on account of some officer renewing the same with or
without knowing it to be forged. Section 15 of the Act only empowers any Licensing
Authority to "renew a driving licence issued under the provisions of this Act with effect
from the date of its expiry". No Licensing Authority has the power to renew a fake
licence and, therefore, a renewal if at all made cannot transform a fake licence as
genuine. Any counterfeit document showing that it contains a purported order of a
statutory authority would ever remain counterfeit albeit the fact that other persons
including some statutory authorities would have acted on the document unwittingly on
the assumption that it is genuine".
39. As noted above, the conceptual difference between third party right and own damage
cases has to be kept in view. Initially, the burden is on the insurer to prove that the license
was a fake one. Once it is established the natural consequences have to flow.
9

. The above aspects were highlighted recently in Laxmi Narain Dhut case (supra). 2007
AIR SCW 2279

10. In the instant case, the State Commission has categorically found that the evidence on
record clearly established that the licensing authority had not issued any license, as was
claimed by the Driver and the respondent. The evidence of Shri A.V.V. Rajan. Junior
Assistant of the Office of the Jt. Commissioner and Secretary, RTA, Hyderabad who
produced the official records clearly established that no driving license was issued to Shri
Ravinder Kumar or Ravinder Singh in order to enable and legally permit him to drive a
motor vehicle. There was no cross-examination of the said witness. The National
Commission also found that there was no defect in the finding recorded by the State
Commission in this regard.
11. It appears that pursuant to the orders dated 14-07-2005 passed by this Court, the
entire amount awarded was deposited in this Court. Since, we have held that the
appellant-Insurance Company has no liability, the amount deposited be returned to the
appellant-Insurance Company with accrued interest, if any.
12. The appeal is allowed. No costs.
Appeal allowed.
AIR 2008 SUPREME COURT 1414 "Suresh Nanda v. C.B.I."
(From : 2007 Cri LJ (NOC) 226 : 2007 (138) DLT 123)
Coram : 2 PRAKASH PRABHAKAR NAOLEKAR AND MARKANDEY KATJU, JJ.
Criminal Appeal No. 179 of 2008 (arising out of SLP (Cri.) No. 3408 of 2007), D/- 24 -1
-2008.
Suresh Nanda v. C.B.I.
Criminal P.C. (2 of 1974), S.102, S.104, S.165 - Passports Act (15 of 1967), S.10 -
SEARCH AND SEIZURE - POLICE OFFICERS - PASSPORT - INTERPRETATION
OF STATUTES - WORDS AND PHRASES - Power of Police to seize property - Does
not extend to impounding of passport - Even Court cannot impound passport - Provisions
of Passports Act prevail over Criminal P.C. - 'Seize' and 'impounding' - Convey different
meaning.
2007 Cri LJ (NOC) 226 : 2007 (138) DLT 123, Reversed.
Interpretation of Statutes - Special Act excludes general Act.
Words and Phrases - Seize and Impound - Difference.
While the police may have the power to seize a passport under Section 102(1), Cr.P.C., it
does not have the power to impound the same. Impounding of a passport can only be
done by the passport authority under Section 10(3) of the Passports Act, 1967. Even the
Court cannot impound a passport. Though, Section 104, Cr.P.C. states that the Court may,
if it thinks fit, impound any document or thing produced
@page-SC1415
before it, this provision will only enable the Court to impound any document or thing
Other than a passport. This is because impounding passport is provided for in Section
10(3) of the Passports Act. The Passports Act is a special law while the Cr.P.C. is a
general law. It is well settled that the special law prevails over the general law.
2007 Cri LJ (NOC) 226 : 2007 (138) DLT 123, Reversed. (Paras 9, 11, 15)
There is a difference between seizing of a document and impounding a document. A
seizure is made at a particular moment when a person or authority takes into his
possession some property which was earlier not in his possession. Thus, seizure is done at
a particular moment of time. However, if after seizing of a property or document the said
property or document is retained for some period of time, then such retention amounts to
impounding of the property/or document. (Para 12)
Cases Referred : Chronological Paras
1999 AIR SCW 3041 : AIR 1999 SC 3097 (Rel. on.) 8
1999 AIR SCW 3074 : AIR 1999 SC 3125 (Rel. on.) 8
AIR 1978 SC 597 (Ref.) 5
AIR 1967 SC 1269 (Ref.) 13
AIR 1967 SC 1836 (Ref.) 4, 5
AIR 1966 SC 135 (Rel. on) 8
AIR 1950 SC 27 5
Harish N. Salve, Sidharth Luthra, Mukul Rohtagi, Sr. Advocates, Sandeep Kapur, Ruchin
Midha, R.N. Karanjawala and Mrs. Manil Karanjawala, for Appellant; A. Sharan, A.S.G.,
A. Mariarputham and B. Krishna Prasad, for Respondent.
Judgement
JUDGMENT :- Leave granted.
2

. The appellant claims to be a non-resident Indian settled in United Kingdom for the last
23 years. The passport of the appellant as well as other documents were seized by the
respondent from 4, Prithviraj Road, New Delhi in a search conducted on 10-10-2006
when the appellant was on a visit to India. The said search and seizure was pursuant to an
F.I.R. dated 9-10-2006 registered on the basis of a sting operation carried out by a news
portal in the year 2001. The passport seized during the search was retained by the C.B.I.
officials. An application was moved by the appellant before the Special Judge, C.B.I.,
Patiala House Courts, New Delhi praying for release of his passport so that he can
travelabroad to London and Dubai for a period of 15 days. The learned Special Judge, by
order dated 15-1-2007, directed the release of the passport to the appellant by imposing
upon him certain conditions. Aggrieved against the order passed by the learned Special
Judge, C.B.I., the respondent preferred Criminal Revision before the High Court. The
High Court, by order dated 5-2-2007, reversed the order of the learned Special Judge and
refused to release the passport to the appellant. Aggrieved against the order of the High
Court, present appeal, by special leave, has been preferred by the appellant.
Reported in 2007 Cri LJ (NOC) 226 (Del)

3. Learned senior counsel appearing for the appellant submitted that the power and
jurisdiction to impound the passport of any individual has to be exercised under the
Passports Act, 1967 (hereinafter referred to as "The Act"). He specifically referred to
subsection (3)(e) of Section 10 of the Act which reads as under :
"(3) The passport authority may impound or cause to be impounded or revoke a passport
or travel document -
(e) if proceedings in respect of an offence alleged to have been committed by the holder
of the passport or travel document are pending before a Criminal Court in India :"
Reference was also made to Section 10A of the Act which has been introduced by Act
17/2002 w.e.f. 17-10-2001.
4

. Learned senior counsel for the appellant also placed reliance on the decision of 5-Judge
Bench of this Court in Satwant Singh Sawhney v. D. Ramarathnam, Asstt. Passport
Officer, (1967) 3 SCR 525 wherein in para 31, it was held as under : AIR 1967 SC
1836, Para 32

"31 : For the reasons mentioned above, we would accept the view of Kerala, Bombay and
Mysore High Courts in preference to that expressed by the Delhi High Court. It follows
that under Article 21 of the Constitution no person can be deprived of his right to travel
except according to procedure established by law. It is not disputed that no law was made
by the State regulating or depriving persons of such a right."
5

. A similar view is registered in the decision rendered by 7-Judge Bench of this Court in
Maneka Gandhi v. Union of India and another, (1978) 1 SCC 248 wherein AIR 1978 SC
597, Para 54

@page-SC1416
at page 280, it was held as under :

".........Now, it has been held by this Court in Satwant Singh's case (supra) that 'personal
liberty' within the meaning of Article 21 includes within its ambit the right to go abroad
and consequently no person can be deprived of this right except according to procedure
prescribed by law. Prior to the enactment of the Passport Act, 1967, there was no law
regulating the right of a person to go abroad and that was the reason why the order of the
Passport Officer refusing to issue passport to the petitioner in Satwant Singh's case
(supra) was struck down as invalid. It will be seen at once from the language of Article 21
that the protection it secures is a limited one. It safeguards the right to go abroad against
executive interference which is not supported by law; and law here means 'enacted law'
or 'State law' (vide A. K. Gopalan case). Thus, no person can be deprived of his right to
go abroad unless there is a law made by the State prescribing the procedure for so
depriving him and the deprivation is effected strictly in accordance with such
procedure.........." AIR 1967 SC 1836
AIR 1950 SC 27

6. On the other hand, learned Additional Solicitor General appearing for the respondent
submitted that the passport was seized and impounded by exercising the powers under
Section 102 read with Sections 165 and 104 of Code of Criminal Procedure (hereinafter
referred to as "the Cr.P.C."). He further contended that the power to retain and impound
the passport has been rightly exercised by the respondent as there is an order dated 3-11-
2006 passed by the learned Special Judge for C.B.I. exercising the power under Section
104 of Cr.P.C.
7. Sub-section (3)(e) of Section 10 of the Act provides for impounding of a passport if
proceedings in respect of an offence alleged to have been committed by the holder of the
passport or travel document are pending before a Criminal Court in India. Thus, the
Passport Authority has the power to impound the passport under the Act. Section 102 of
Cr.P.C. gives powers to the police officer to seize any property which may be alleged or
suspended to have been stolen or which may be found under circumstances which create
suspicion of the commission of any offence. Sub-section (5) of Section 165 of Cr.P.C.
provides that the copies of record made under sub-section (1) or sub-section (3) shall
forthwith be sent to the nearest Magistrate empowered to take cognizance to the offence
whereas Section 104 of Cr.P.C. authorizes the Court to impound any document or thing
produced before it under the Code. Section 165 of Cr.P.C. does not speak about the
passport which has been searched and seized as in the present case. It does not speak
about the documents found in search, but copies of records prepared under sub-section
(1) and sub-section (3). "Impound" means to keep in custody of the law. There must be
some distinct action which will show that documents or things have been impounded.
According to the Oxford Dictionary "impound" means to" take legal or formal
possession. In the present case, the passport of the appellant is in possession of CBI right
from the date it has been seized by the CBI. When we read Section 104 of Cr.P.C. and
Section 10 of the Act together, under Cr.P.C., the Court is empowered to impound any
document or thing produced before it whereas the Act speaks specifically of impounding
of the passport.
8

. Thus, the Act is a special Act relating to a matter of passport, whereas Section 104 of the
Cr.P.C. authorizes the Court to impound document or thing produced before it. Where
there is a special Act dealing with specific subject, resort should be had to that Act
instead of general Act providing for the matter connected with the specific Act. As the
Passports Act is a special Act, the rule that "general provision should yield to the specific
provision" is to be applied. See : Dam Valaji Shah and another v. Life Corporation of
India and others (AIR 1966 SC 135). Gobind Sugar Mills Ltd. v. State of Bihar and
others (1999 (7) SCC 76) and Belsund Sugar Co. Ltd. v. State of Bihar and others (AIR
1999 SC 3125). 1999 AIR SCW 3041
1999 AIR SCW 3074

9. The Act being a specific Act whereas Section 104 of Cr. P. C. is a general provision for
impounding any document or thing, it shall prevail over that section in the Cr. P. C. as
regards the passport. Thus, by necessary implication, the power of Court to impound any
document or thing produced before it would exclude passport.
10. In the instant case, no steps have been taken under Section 10 of the Act which
provides for variation, impounding and revocation of the passports and travel documents.
Section 10A of the Act which provides for an order to suspend with immediate
@page-SC1417
effect any passport or travel document; such other appropriate order which may have the
effect of rendering any passport or travel document invalid, for a period not exceeding
four weeks, if the Central Government or any designated officer on its satisfaction holds
that it is necessary in public interest to do without prejudice to the generality of the
provisions contained in Section 10 by approaching the Central Government or any
designated officer. Therefore, it appears that the passport of the appellant cannot be
impounded except by the Passport Authority in accordance with law. The retention of the
passport by the respondent (CBI) has not been done in conformity with the provisions of
law as there is no order of the passport authorities under Section 10(3)(e) or by the
Central Government or any designated officer under Section 10A of the Act to impound
the passport by the respondent exercising the powers vested under the Act.
11. Learned Additional Solicitor General has submitted that the police has power to seize
a passport in view of Section 102(1) of the Cr.P.C. which states :
"Power of police officer to seize certain property : (1) Any police officer may seize any
property which may be alleged or suspected to have been stolen, or which may be found
under circumstances which create suspicion of the commission of any offence".
In our opinion, while the police may have the power to seize a passport under Section
102(1), Cr. P. C., it does not have the power to impound the same. Impounding of a
passport can only be done by the passport authority under Section 10(3) of the Passports
Act, 1967.
12. It may be mentioned that there is a difference between seizing of a document and
impounding a document. A seizure is made at a particular moment when a person or
authority takes into his possession some property which was earlier not in his possession.
Thus, seizure is done at a particular moment of time. However, if after seizing of a
property or document the said property or document is retained for some period of time,
when such retention amounts to impounding of the property/or document. In the Law
Lexicon by P. Ramanatha Aiyar (2nd Edition), the word "impound" has been defined to
mean "to take possession of a document or thing for being held in custody in accordance
with law". Thus, the word "impounding" really means retention of possession of a good
or a document which has been seized.
13. Hence, while the police may have power to seize a passport under Section 102,
Cr.P.C. if it is permissible within the authority given under Section 102 of Cr.P.C., it does
not have power to retain or impound the same, because that can only be done by the
passport authority under Section 10(3) of the Passports Act. Hence, if the police seizes a
passport (which it has power to do under Section 102, Cr.P.C.), thereafter the police must
send it along with a letter to the passport authority clearly stating that the seized passport
deserves to be impounded for one of the reasons mentioned in Section 10(3) of the Act. It
is thereafter the passport authority to decide whether to impound the passport or not.
Since impounding of a passport has civil consequences, the passport authority must give
an opportunity of hearing to the person concerned before impounding his passport. It is
well settled that any order which has civil consequences must be passed after giving
opportunity of hearing to a party vide State of Orissa v. Binapani Dei (AIR 1967 SC
1269).
14. In the present case, neither the passport authority passed any order of impounding nor
was any opportunity of hearing given to the appellant by the passport authority for
impounding the document. It was only the CBI authority which has retained possession
of the passport (which in substance amounts to impounding it) from October, 2006. In
our opinion, this was clearly illegal. Under Section 10A of the Act retention by the
Central Government can only be for four weeks. Thereafter it can only be retained by an
order of the Passport Authority under Section 10(3).
15. In our opinion, even the Court cannot impound a passport. Though, no doubt, Section
104, Cr.P.C. states that the Court may, if it thinks fit, impound any document or thing
produced before it, in our opinion, this provision will only enable the Court to impound
any document or thing other than a passport. This is because impounding a "passport" is
provided for in Section 10(3) of the Passports Act. The Passports Act is a special law
while the Cr. P. C. is a general law. It is well settled that the special law prevails over the
general law vide G. P. Singh's Principles of Statutory Interpretation
@page-SC1418
(9th Edition pg. 133). This principle is expressed in the maxim "Generalia special bus
non derogant". Hence, impounding of a passport cannot be done by the Court under
Section 104, Cr.P.C. though it can impound any other document or thing.
16. For the aforesaid reasons, we set aside the impugned order of the High Court and
direct the respondent to hand over the passport to the appellant within a week from today.
However, it shall be open to the respondent to approach the Passport Authorities under
Section 10 or the authorities under Section 10A of the Act for impounding the passport of
the appellant in accordance with law.
17. We, however, make it clear that we are not expressing any opinion on the merits of
the case and are not deciding whether the passport can be impounded as a condition for
grant of bail.
18. The appeal stands disposed of accordingly.
Order accordingly.
AIR 2008 SUPREME COURT 1418 "National Insurance Company Ltd. v. Annappa
Irappa Nesaria"
(From : Karnataka)
Coram : 2 S. B. SINHA AND H. S. BEDI, JJ.
Civil Appeal No. 574 of 2008 (arising out of SLP (C) No. 21632 of 2003), D/- 22 -1
-2008.
National Insurance Company Ltd. v. Annappa Irappa Nesaria and Ors.
Motor Vehicles Act (59 of 1988), S.2(21), S.149 - Central Motor Vehicles Rules (1989),
Form 4 (Before its amendment in 2001) - MOTOR VEHICLES - LICENSE -
INSURANCE - Light motor vehicle - Covers "light passenger carriage vehicle" and
"light goods carriage vehicle" - Driver possessing LMV licence - Cannot be said to not
possess effective licence to drive Matador van having Goods Carriage permit - Insurance
company cannot shirk its liability to pay compensation. (Paras 16, 8, 17)
Cases Referred : Chronological Paras
2000 AIHC 2695 (Kant) 5
1999 AIR SCW 3142 : AIR 1999 SC 3181 (Ref.) 8
Atul Nanda, Ms. Rameeza Hakeem, Rajesh Kumar, P.N. Puri, Parmanand Gaur, Ms.
Neha Gaur, Vishnu Mehra and B.K. Satija, for Appellant; J.P. Dhanda, Ms. Raj Rani
Dhanda, Dr. Sushil, Balwada, S.N. Bhat, Neraj Kumar Jain, Bharat Singh, Sanjay Singh,
Sandeep Chaturvedi, Ugra Shankar Prasad, Rajesh Tyagi, Dr. Aparna Bhardwaj, Atishi
Dipankar and Ashwani Kumar for Respondents.
Judgement
1. S. B. SINHA, J. :-Leave granted.
2. Appellant National Insurance Company is before us, aggrieved by and dissatisfied with
the judgment and order dated 14th July, 2003, passed by a learned Single Judge of the
Karnataka High Court in M.F.A. No. 7788/2002 dismissing the appeal preferred against
an award dated 17-9-2002 passed by the Motor Accident Claims Tribunal ("Tribunal" for
short) in M.V.C No. 124/2000.
3. The fact of the matter relevant for the purpose of disposal of this appeal is as under :
The vehicle involved in the accident was a Matador Van bearing registration No. KA-
23/2890. It had a Goods Carriage permit granted in terms of Form No. 7 of the Motor
Vehicles Act. It was insured with the appellant. The said vehicle met with an accident on
9-12-1999 causing death of one Gangawwa wife of the respondent No.1 herein. A
contention was raised on behalf of the appellant that the driver of the said vehicle did not
possess an effective licence to drive a transport vehicle.
4. An issue (being Issue No. 3), inter alia, was framed in that behalf before the Tribunal,
which reads as under :
"Whether the R-3 proves that driver of offending vehicle was not an authorised person to
drive the same?"
5. The learned Tribunal in support of its award on Issue No.3, held :

"DW-1 in this cross examination admitted that offending vehicle is authorised to transport
3500 kgs. goods. Further, in this cross-examination stated that LMV means transport
vehicle which unladen weight does not exceed 7500 kgs. Ex.D-2 at Column No. 12
clearly shows that the unladen weight of the offending vehicle is 3500 kgs. Thereby it is
very much clear that said unladen weight of the offending vehicle is much less than 7500
kgs. as admitted by DW-1 in his cross-examination which is also the effect 2000 AIHC
2695

@page-SC1419
as defined in the MV Act. So when the unladen weight of the offending vehicle is less
than 7500 kgs. the RW-1 driver who is having DL as per Ex.D-3 is certainly authorised to
drive the offending vehicle. The decision relied upon by Adv. for petitioners at serial No.
2 ruling reported in 2000(5) KLJ 473 (DB) our own Hon'ble High Court had clearly held
that where offending vehicle is weighing 4960 kgs. driven by a person having DL to drive
the LMV, there is breach of insurance policy, as statute classifies vehicle weighing below
7500 kgs. as LMVs and Insurer is liable to satisfy award in respect of accident that
occurred when the vehicle was being driven by driver holding such licence. So in view of
the decision of Division Bench of our own Hon'ble High Court and also the decision of
Supreme Court referred at Sl. No. 1 Adv. for petitioners it is very much clear that RW1 is
having valid DL as per Exh.D3 and the offending vehicle unladen weight is 3500 kgs. is
certainly was having effective and valid DL and so R3 has failed to prove the issue No. 3
and accordingly I answer the same in the negative."
6. The High Court on an appeal preferred by the appellant herein opined :
"Counsel for the appellant-Insurance Company, questioning the liability, contended that
the driver did not possess a valid licence to drive LMV. According to the respondents, the
driver had driving licence to drive LMV, a transport vehicle. According to the appellant,
the vehicle in question involved in the accident is a transport vehicle. The said contention
cannot be accepted by this Court, in view of the fact that claimants are third parties even
on the ground that there is violation of terms and conditions of policy, the insurance
company cannot be permitted to contend that it has no liability. Accordingly, I do not see
any merit in this appeal."
7. Mr. Vishnu Mehra, learned counsel appearing on behalf of the appellant would submit
that the High Court committed a serious error in passing the impugned judgment in so far
as it failed to take into consideration that a light motor vehicle cannot be a transport
vehicle within the meaning of the provisions of the Act. It was submitted that for the
purpose of grant of licence for driving a vehicle, an application has to be filed in Form
No.4, whereafter only a licence is granted in Form No.6. Learned counsel contended that
the said forms have been prescribed in terms of Rules 14 and 16 of the Central Motor
Vehicles Rules, 1989, and on a perusal thereof as also the aforementioned Forms, it
would be clear that a light motor vehicle does not answer the description of a transport
vehicle.
8. Mr. S.N. Bhat, learned counsel appearing on behalf of the respondents, on the other
hand, submitted that the contention raised herein by the appellant has neither been raised
before the Tribunal nor before the High Court. In any event, it was urged, that keeping in
view the definition of the light motor vehicle as contained in Section 2(21) of the Motor
Vehicles Act, 1988 (Act for short), a light goods carriage would come within the purview
thereof.
A 'light goods carriage' having not been defined in the Act, the definition of the light
motor vehicle clearly indicates that it takes within its umbrage, both a transport vehicle
and a non-transport vehicle.

Strong reliance has been placed in this behalf by the learned counsel in Ashok Gangadhar
Maratha v. Oriental Insurance Company Ltd., (1999 (6) SCC 620). 1999 AIR SCW 3142

9. The Motor Vehicles Act, 1988, which was enacted to consolidate and amend the law
relating to motor vehicles, is a complete code.
10. Section 2 of the Act provides for interpretation of the terms contained herein. It
employs the words unless the context otherwise requires. Section 2(16) of the Act defines
heavy goods vehicle to mean any goods carriage the gross vehicle weight of which, or a
tractor or a road-roller the unladen weight of either of which, exceeds 12,000 kilograms.
11. Section 2(21) defines light motor vehicle and Section 2(23) defines medium goods
vehicle as under :
"Light motor vehicle means a transport vehicle or omnibus the gross vehicle weight of
either of which or a motor car or tractor or road-roller the unladen weight of any of
which, does not exceed 7500 kilograms."
"Medium goods vehicle means any goods
@page-SC1420
carriage other than a light motor vehicle or a heavy goods vehicle."
Section 3 of the Act is in the following terms :
"3. Necessity for driving licence.- (1) No person shall drive a motor vehicle in any public
place unless he holds an effective driving licence issued to him authorising him to drive
the vehicle; and no person shall so drive a transport vehicle other than a motorcab or
motor cycle hired for his own use or rented under any scheme made under sub-section (2)
of section 75 unless his driving licence specifically entitles him so to do."
12. The Central Government has framed Rules known as The Central Motor Vehicles
Rules, 1989.
13. The word Form has been defined in Rule 2(e) to mean a Form appended to the rules.
"I apply for a licence to enable me to drive vehicles of the following description :
(d) Light motor vehicle
(e) Medium goods vehicle
(g) Heavy goods vehicle
(j) Motor vehicles of the following description :...."
After amendment the relevant portion of Form 4 reads as under :
I apply for a licence to enable me to drive vehicles of the following description :
(d) Light motor vehicle
(e) Transport vehicle
(j) Motor vehicles of the following description :...."
14. Rule 14 prescribes for filing of an application in Form 4, for a licence to drive a
motor vehicle, categorizing the same in nine types of vehicles.
Clause (e) provides for transport vehicle which has been substituted by G.S.R 221 (E)
with effect from 28-3-2001. Before the amendment in 2001, the entries medium goods
vehicle and heavy goods vehicle existed which have been substituted by transport
vehicle. As noticed hereinbefore, Light Motor Vehicles also found place therein.
15. Light Motor Vehicle is defined in Section 2(21) and, therefore, in view of the
provision, as then existed, it included a light transport vehicle. Form 6 provides for the
manner in which the licence is to be granted, the relevant portion whereof read as under :
"Authorisation to drive transport vehicle
Number...................................
Date.......................................
Authorised to drive transport vehicle with effect from.....................................
Badge number...............................
Signature.....................................
Designation of the licensing authority
Name and designation of their authority who conducted the driving test."
16. From what has been noticed hereinbefore, it is evident that transport vehicle has now
been substituted for 'medium goods vehicle' and 'heavy goods vehicle'. The light motor
vehicle continued, at the relevant point of time, to cover both, 'light passenger carriage
vehicle' and 'light goods carriage vehicle'.
A driver who had a valid licence to drive a light motor vehicle, therefore, was authorised
to drive a light goods vehicle as well.
17. The amendments carried out in the Rules having a prospective operation, the licence
held by the driver of the vehicle in question cannot be said to be invalid in law.
18. For the reasons aforementioned there is no merit in this appeal and it is dismissed
with costs which we quantify at Rs.25,000/-(Rupees Twenty Five Thousand only).
Appeal dismissed.
AIR 2008 SUPREME COURT 1420 "Vidyadhari v. Sukhrana Bai"
(From : Madhya Pradesh)*
Coram : 2 S. B. SINHA AND V. S. SIRPURKAR, JJ.
Civil Appeal No. 575 of 2008 (arising out of SLP (C) No. 6758 of 2007), D/- 22 -1 -2008.
Vidyadhari and Ors. v. Sukhrana Bai and Ors.
Succession Act (39 of 1925), S.372 - SUCCESSION - Succession certificate - Grant of -
Claimant though not regularly wedded stayed for long time as wife of deceased husband -
Not only made nominee by deceased but also was mother of four children of deceased -
Entitled to grant of
@page-SC1421
succession certificate.
Misc. Appeal Nos. 33 and 43 of 1998, D/-20-09-2006 (MP), Reversed.
The claimant, nominee of deceased, who married with him during subsistence of earlier
marriage, could not claim status of legally wedded wife. The claimant continued to stay
with deceased as his wife for long time and was a person of confidence for deceased who
had nominated her for his Provident Fund, Life Cover Scheme, Pension and amount of
Life Insurance and amount of other dues. She was not only a nominee but also was the
mother of four children of deceased who were the legal heirs of deceased and whose
names were also found in the declaration form of deceased during his lifetime. The
claimant nominee made claim for succession certificate in regard to death benefits arising
out of employment of deceased. Thus she can always file an application for succession
certificate under S. 372 of Indian Succession Act as there is nothing in that section to
prevent such a nominee from claiming the certificate on the basis of nomination. In the
circumstances she was always preferable even to the legally wedded wife who had never
stayed with deceased as his wife and who had gone to the extent of claiming the
succession certificate to the exclusion of legal heirs of deceased. In the grant of
succession certificate the Court has to use its discretion where the rival claims, as in this
case, are made for the succession certificate for the properties of the deceased. She was
the legally wedded wife merely because of her legal status would not be entitled to a
succession certificate in comparison to claimant nominee who all through had stayed as
the wife of deceased, had born his four children and had claimed a succession certificate
on behalf children also. Therefore grant of claim of legally wedded wife to the exclusion
not only of the nominee of deceased but also to the exclusion of his legitimate legal heirs
would be illegal. However, legally wedded wife is also being one of the legal heirs and
besides the four children she would have the equal share in deceased's estate which would
be 1/5th. To balance of equities, therefore, Court granted succession certificate to
nominee of deceased but with a rider that she would protect the 1/5th share of legally
wedded wife in deceased's properties and would hand over the same to her.
Misc. Appeal Nos. 33 and 43 of 1998, D/-20-09-2006 (MP), Reversed. (Paras 9, 10,
11)
Cases Referred : Chronological Paras
2002 AIR SCW 674 : AIR 2002 SC 971 : 2002 AIR Kant HCR 748 (Disting) 7, 9
2002 AIR SCW 273 : AIR 2000 SC 375 : 2000 Lab IC 976 (Ref) 7, 10
AIR 1998 MP 114 6
1996 AIR SCW 4157 : AIR 1997 SC 10 (Disting) 5, 9
Mrs. Anuradha Mutatkar, Prakash Shrivastava, for Appellants; Ms. Sunita Sharma, Ms.
Sudha Pal, Subramonium Prasad, Ms. Varuna Bhandari Gugnani, Rameshwar Prasad
Goyal, Sunil Roy, for Respondents.
Judgement
V. S. SIRPURKAR, J. :- Leave granted.
2. A common judgment of the High Court of Madhya Pradesh at Jabalpur, disposing of
two Miscellaneous Appeals is in challenge before us. The appeals were filed by one Smt.
Sukhrana Bai claiming herself to be the widow of one Sheetaldeen. Sheetaldeen was
working as a CCM Helper in Mines P. K. 1 of the Western Coalfields at Pathakheda and
died on 9-5-1993 while in service. Two separate applications came to be filed under
Section 372 of the Indian Succession Act for obtaining succession certificate with respect
to the movable properties of deceased Sheetaldeen, one of them was filed by Vidhyadhari
registered as Succession Case No. 3/96 while the other came to be filed by Sukhrana Bai
which was registered as Succession Case No. 10/95. Both the cases were joined and tried
together by the Trial Court which allowed the application filed by Vidyadhari (SC No.
3/96) and dismissed the one filed by Sukharana Bai (SC No. 10/95). Sukhrana Bai,
therefore, filed two Miscellaneous Appeals being MA 33/1998 and MA 43/1998 which
came to be allowed by the High Court in favour of Sukhrana Bai. Vidyadhari, therefore,
is before us in this appeal. Before we proceed with the matter, a factual background
would be necessary.
3. Admittedly, Sukhrana Bai was the first wife of Sheetaldeen, while during the
subsistence of this marriage, Sheetaldeen got married with Vidhyadhari. Two sons and
two daughters were born to Vidhyadhari, they being Smt. Savitri, Naresh @ Ramesh, Ms.
Chanda @ Durga and Baliram, while Sukhrana Bai does not have any children.
4. Vidhyadhari in her application before the Trial Court (SC No. 3/96), besides herself,
@page-SC1422
disclosed the names of her children as the legal heirs of Sheetaldeen. It was also revealed
that deceased Sheetaldeen had nominated her for receiving amounts under the Provident
Fund, Family Pension Scheme and Coal Mines Deposits Life Scheme. She also disclosed
that she has received a sum of Rs. 45036/- towards gratuity amount of the deceased from
the employer of Sheetaldeen, i.e., Western Coalfields Ltd. She, therefore, claimed the
Succession Certificate on the basis of the nominations besides her marriage with
Sheetaldeen.
5. As stated above, both the Succession Cases came to be consolidated and tried together.
In SC No. 10/95, filed by Sukhrana Bai, Vidyadhari raised a objection that Sukhrana Bai
was not the heir of deceased Sheetaldeen and though Sheetaldeen initially nominated
Vidyadhari to receive the dues after his death as per Form A, subsequently he cancelled
that nomination and filled in a second Form A in which he had nominated Smt.
Vidyadhari and in description of his family members he had indicated her to be the wife,
one Naresh as his son and Ms. Chanda @ Durga as his daughter. It was also pointed out
that Sukhrana Bai had not claimed any dues from the office of Sheetaldeen. WCL which
is a party, contended that the non-applicant had no knowledge about the valid marriage
between the deceased and Sukhrana Bai and it was also admitted that Sheetaldeen had
nominated Vidyadhari to receive the total amount and had registered her as his nominee.
Following issues came to be framed by the Trial Court.
"(1) Whether the legal widow of the deceased Sheetaldeen is the applicant Smt. Sukhrana
of Case No. 10/95 or Vidhyadhari of Case No. 3/96?
(2) Whether Smt. Savitri, Naresh alias Ramesh, Ms. Chanda alias Durga and Baliram, as
mentioned in the application of Case No. 3/96 are the children of applicant Vidhyadhari,
sired by deceased Sheetaldeen?
(3) If yes, whether they are the heirs of deceased Sheetaldeen?
(4) For receiving the amount due to deceased Sheetaldeen, issuance of Succession
Certificate in whose favour would be just and proper?
(B) Relief and expenses?
Both oral and documentary evidence was led by both the parties. Sukhrana Bai examined
herself as AW1 along with three other witnesses, namely, Kanhaiyalal (AW2), Ram
Prasad (AW3) and Shivnath (AW4). On the basis of the evidence led, the Trial Court held
Vidyadhari to be the legal widow of deceased Sheetaldeen. It was also held that the
children Smt. Savitri, Naresh @ Ramesh, Ms. Chanda @ Durga and Baliram mentioned
in SC No. 3/96 were sired by deceased Sheetaldeen and were his children. They were also
held to be heirs of deceased Sheetaldeen. The Trial Court also held that the Succession
Certificate was liable to be issued in favour of Vidhyadhari and not in favour of Sukhrana
Bai. In its judgment the Trial Court referred to an admission made by Vidyadhari in her
affidavit Exhibit C-7 wherein she had stated on oath that she is the second wife of
Sheetaldeen and Sukhrana Bai was the first wife. The Trial Court also referred to the
proved fact that Sheetaldeen initially had nominated Sukharana Bai as a nominee
indicating her to be his wife in Form A. After discussing the voluminous oral evidence
led by the parties, the Trial Court held that Sukhrana Bai was earlier married to
Sheetaldeen and there were no issues out of this wedlock and thereafter Sheetaldeen
married Vidyadhari and for about 20 to 25 years he lived with Vidyadhari till his death
while Sukhrana Bai never came to stay with him. The observation of the Trial Court in
para 18 of its judgment is as under :
".......which means that either Sukhrana Devi deserted him or Sheetaldeen left her."

The Trial Court then proceeded to hold in Para 19 that Sheetaldeen belonged to the
'Shudra' community and in Shudra community if the wife deserts her husband and no
effort is made by the husband to take her back as his wife then under Hindu Law it is
presumed that divorce has taken place between the two, as has been held by the Supreme
Court in Govind Raju v. K. Muni Swami Gonder and Ors. (AIR 1997 SC 10). A finding
was given that Sheetaldeen had divorced Sukhrana Bai and solemnized second marriage
with Vidhyadhari and, therefore, the marriage of Vidhyadhari could not be said to be
illegal. On that basis the Trial Court excluded the claim of Sukhrana Bai and granted the
claim of Vidhyadhari holding that she was entitled to receive the amount of Rs.
1,30,000/- from 1996 AIR SCW 4157

@page-SC1423
WCL towards Sheetaldeen's Provident Fund, Life Cover Scheme, Pension and amount of
Life Insurance and amount of other dues payable to the successor of Sheetaldeen on his
death. It was also observed in para 23 as under :
"....In that amount, applicant Vidhyadhari and her sons and daughters will have equal
share. On receipt of the said amount, applicant Vidhyadhari shall distribute the amount to
her sons and daughters as per their share....."
Resultantly the Trial Court dismissed Sukhrana Bai's application.
6. The High Court, however, concluded that the theory of customary divorce between
Sukhrana Bai and Sheetaldeen was a myth. It was noted that there was no evidence on
record to hold that customary divorce had taken place between Sukhrana Bai and
Sheetaldeen nor was there any pleading about the factum of any customary divorce or
existence of any custom. Relying on a reported decision in Smt. Savitri Devi v.
Manorama Bai (AIR 1998 MP 114), the High Court came to the conclusion that the
alleged customary divorce between Sukhrana Bai and deceased Sheetaldeen was not
established. Stopping here itself, the High Court allowed both the appeals and directed
that the Succession Certificate should be granted in favour of Sukharana Bai.
7

. Learned counsel appearing for the appellant Vidhyadhari strenuously urged that the
High Court could not have straightway granted the claim of Sukharana Bai. Learned
counsel pointed out that in grant of certificate in favour of Sukhrana Bai, the claim of
four children was altogether ignored as, admittedly, Sukhrana Bai had sought the
certificate for herself alone. Learned counsel points out that even if the theory of divorce
between Sukhrana Bai and Sheetaldeen is described and even if Vidhyadhari is not held
to be his legal wife since the children admittedly were sired by Sheetaldeen, they were
legitimate children entitled to inherit Sheetaldeen. On this point, learned counsel relied on
Rameshwari Devi v. State of Bihar and Ors. ((2000) 2 SCC 431). Learned counsel
pointed out that in her application Vidhyadhari had specifically mentioned the names of
four children as the legal heirs besides herself, while Sukhrana Bai had claimed that she
was the only legal heir of Sheetaldeen. Learned counsel tried to urge, relying on a
reported decision in Yamanji H. Jadhav v. Nirmala ((2002) 2 SCC 637), that in this case
the customary divorce should have been held to be proved. 2000 AIR SCW 273
2002 AIR SCW 674

8. As against this, learned counsel appearing for respondent Sukhrana Bai supported the
judgment of the High Court and contended that she being the only legal heir of deceased
Sheetaldeen, she alone was entitled to the grant of Succession Certificate as ordered by
the High Court.
9

. There can be no dispute that Vidhyadhari had never pleaded any divorce, much less
customary divorce between Sukhrana Bai and Sheetaldeen. There were no pleadings and
hence no issue arose on that count. In our opinion, therefore, the High Court was right in
holding that marriage between Sukhrana Bai and Sheetaldeen was very much subsisting
when Sheetaldeen got married to Vidhyadhari. Learned counsel tried to rely on the
reported decision in Govind Raju's case (supra). We are afraid the decision is of no help
to the respondent as basically the issue in that decision was about the legitimacy of the
children born to a mother whose first marriage was not dissolved and yet she had
contracted the second marriage. This is apart from the fact that in the present case there
were no pleadings about the existence of custom and alleged divorce thereunder.
Therefore, there was no evidence led on that issue. In our opinion the decision in Govind
Raju's case is not applicable. Even the other decision in Yamanaji's case is not applicable
as the facts are entirely different. In Yamanaji's case there was a Deed of divorce
executed by the wife. The question was whether there was a customary divorce. There
was a custom permitting divorce by executing deed existing in the community to which
the parties belonged. Such is not the situation here. There is neither any Divorce Deed nor
even the assertion on the part of Vidhyadhari that Sheetaldeen had divorced Sukhrana
Bai. We, therefore, accept the finding of the High Court that Sukhrana Bai was the legally
wedded wife while Vidhyadhar could not claim that status. 1996 AIR SCW 4157
2002 AIR SCW 674

10

. However, unfortunately, the High Court stopped there only and did not consider the
question as to whether in spite of this factual scenario Vidhyadhari could be 2000
AIR SCW 273

@page-SC1424
rendered the Succession Certificate. The High Court almost presumed that Succession
Certificate can be applied for only by the legally wedded wife to the exclusion of
anybody else. The High Court completely ignored the admitted situation that this
Succession Certificate was for the purposes of collecting the provident Fund, Life Cover
Scheme, Pension and amount of Life Insurance and amount of other dues in the nature of
death benefits of Sheetaldeen. That Vidhyadhari was a nominee is not disputed by anyone
and is, therefore, proved. Vidhyadhari had claimed the Succession Certificate mentioning
therein the names of four children whose status as legitimate children of Sheetaldeen
could not and cannot be disputed. This Court in a reported decision in Rameshwari Devi's
case (supra) has held that even if a Government Servant had contracted second marriage
during the subsistence of his first marriage, children born out of such second marriage
would still be legitimate though the second marriage itself would be void. The Court,
therefore, went on to hold that such children would be entitled to the pension but not the
second wife. It was, therefore, bound to be considered by the High Court as to whether
Vidhyadhari being the nominee of Sheetaldeen could legitimately file an application for
Succession Certificate and could be granted the same. The law is clear on this issue that a
nominee like Vidhyadhari who was claiming the death benefits arising out of the
employment can always file an application under Section 372 of the Indian Succession
Act as there is nothing in that Section to prevent such a nominee from claiming the
certificate on the basis of nomination. The High Court should have realised that
Vidhyadhari was not only a nominee but also was the mother of four children of
Sheetaldeen who were the legal heirs of Sheetaldeen and whose names were also found in
Form A which was the declaration of Sheetaldeen during his life time. In her application
Vidhyadhari candidly pointed out the names of the four children as the legal heirs of
Sheetaldeen. No doubt that she herself has claimed to be legal heir which status she could
not claim but besides that she had the status of a nominee of Sheetaldeen. She continued
to stay with Sheetaldeen as his wife for long time and was a person of confidence for
Sheetaldeen who had nominated her for his Provident Fund, Life Cover Scheme, Pension
and amount of Life Insurance and amount of other dues. Under such circumstances she
always preferable even to the legally weded wife like Sukhrana Bai who had never stayed
with Sheetaldeen as his wife and who had gone to the extent of claiming the Succession
Certificate to the exclusion of legal heirs of Sheetaldeen. In the grant of Succession
Certificate the Court has to use its discretion where the rival claims, as in this case, are
made for the Succession Certificate for the properties of the deceased. The High Court
should have taken into consideration these crucial circumstances. Merely because
Sukhrana Bai was the legally wedded wife that by itself did not entitle her to a
Succession Certificate in comparison to Vidhyadhari who all through had stayed as the
wife of Sheetaldeen, had born his four children and had claimed a Succession Certificate
on behalf of children also. In our opinion, the High Court was not justified in granting the
claim of Sukhrana Bai to the exclusion not only of the nominee of Sheetaldeen but also to
the exclusion of his legitimate legal heirs.
11. Therefore, though we agree with the High Court that Sukhrana Bai was the only
legitimate wife yet, we would chose to grant the certificate in favour of Vidhyadhari who
was his nominee and the mother of his four children. However, we must balance the
equities as Sukhrana Bai is also one of the legal heirs and besides the four children she
would have the equal share in Sheetaldeen's estate which would be 1/5th. To balance the
equities we would, therefore, chose to grant Succession Certificate to Vidhyadhari but
with a rider that she would protect the 1/5th share of Sukhrana Bai in Sheetaldeen's
properties and would hand over the same to her. As the nominee she would hold the 1/5th
share of Sukhrana Bai in trust and would be responsible to pay the same to Sukhrana Bai.
We direct that for this purpose she would give a security in the Trial Court to the
satisfaction of the trial Court.
12-13. It should not be understood by the above that we are, in any way, deciding the
status of Vidhadhari finally, she may still prosecute her own remedies for establishing her
own status independently of these proceedings.
14. In the result the appeal is allowed. In the facts and circumstances of the case, there
will be no order as to costs.
Appeal allowed.
@page-SC1425
AIR 2008 SUPREME COURT 1425 "Ghasita Sahu v. State of Madhya Pradesh"
(From : Madhya Pradesh)*
Coram : 2 S. B. SINHA AND V. S. SIRPURKAR, JJ.
Criminal Appeal No. 184 of 2008 (arising out of SLP (Cri.) No. 1743 of 2006), D/- 28 -1
-2008.
Ghasita Sahu v. State of M.P.
(A) Narcotic Drugs and Psychotropic Substances Act (61 of 1985),S.51 - Criminal P.C. (2
of 1974), S.100 - NARCOTIC DRUGS - SEARCH AND SEIZURE - Search - Validity -
Plea that local Panchas not added to search party - Panchas though not supporting
prosecution not stating that they were not localites - No question/suggestion put to them
or to I. O. that they were stock witnesses - Plea held was not tenable. (Para 4)
(B) Narcotic Drugs and Psychotropic Substances Act (61 of 1985), S.50, S.42 -
NARCOTIC DRUGS - SEARCH AND SEIZURE - Search - Informing accused of his
right to be searched before Magistrate or Gazetted Officer - Requirement restricted to
search of person of accused - Does not apply to search of house of accused - Question as
to manner of informing accused of his right does not arise. (Paras 6, 7)
(C) Narcotic Drugs and Psychotropic Substances Act (61 of 1985), S.8, S.20 -
NARCOTIC DRUGS - SEARCH AND SEIZURE - SENTENCE REDUCTION -
Punishment - Accused convicted for possession of Ganja - Quantity seized was less than
commercial quantity - Accused middle aged and has poor back-ground - Had been in jail
for four years since arrest - Sentence reduced from five years to period already
undergone. (Para 8)
Cases Referred : Chronological Paras
2005 AIR SCW 2154 : AIR 2005 SC 2265 : 2005 Cri LJ 2208 (Rel. on) 7
Ms. Sangeeta Kumar and Ms. Shivangi Thagale, for Appellant; Govind Goel, C.D. Singh,
Maru Sagar Samanta Ray, Vairagya Vardhan, Sunny Choudhary and Ram Naresh Yadav,
for Respondent.
* Cri. A. No. 1344 of 2004, D/- 22-7-2005 (MP).
Judgement
1. V. S. SIRPURKAR, J. :- Leave granted.
2. The appellant herein challenges his conviction for the offence under Section 8 read
with Section 20(b)(ii) of the Narcotic Drugs and Psychotropic Substances Act, 1985
(hereinafter referred to as "NDPS Act") recorded by the Trial Court and confirmed by the
High Court.
3. On the prior information Arun Pandey (PW6) searched the house of the appellant and
found 17.750 Kgs. of Ganja kept in a gunny bag from one room. Before the search was
taken, the Investigation Officer had completed all the formalities as per Section 42 of
NDPS Act. At the time of search, the appellant was apprised of the information as also
the proposed search and was also given an option to have a search in presence of a
Gazetted Officer. However, the appellant had not opted such an option and consented to
the search by the search party led by Investigating Officer Arun Pandey (PW6). The
Ganja (17.750 Kg.) was seized from one of the rooms and after samples were drawn, rest
of it was sent to Malkhana for the safe custody. The sample packages were sent to
Forensic Science Laboratory wherein it was confirmed that it was Ganja. The
investigation having been completed, the appellant was charge-sheeted. The appellant
pleaded not guilty. However, relying on the statement of Arun Pandey (PW6) and Shiv
Kumar (PW1) as also the documents including the Panchanama, the appellant was found
guilty and was convicted of the offences charged. He was directed to undergo Rigorous
Imprisonment for a period of five years. He was also directed to pay a fine of Rs.
20,000/-. in default rigorous imprisonment of one year. This conviction was challenged
before the High Court. However, the High Court, after going through the evidence
confirmed the conviction and the sentence, necessitating the present appeal.
4. It was firstly contended by the counsel for the appellant that the search itself was
illegal as the Panchas for the search firstly had not supported the same and secondly they
were not the local panchas. We were, therefore, taken through the evidence of the two
Panchas Raju (PW4) and Sanju Tiwari (PW5). We have carefully gone through their
evidence. Both of them have not supported the prosecution inasmuch as they have even
refused to identify the accused. There is nothing in their evidence to suggest that they
were not local panchas. They have not even been distantly suggested that they were the
visual panchas and stock witnesses of local police and were not residents of the area
wherefrom the Ganja was recovered. Learned counsel tried to rely on
@page-SC1426
the evidence of Arun Pandey (PW 6). However nothing has been suggested to him in
respect of panchas not being local panchas. The Investigating Officer seems to have taken
all precautions as per Section 100 of Criminal Procedure Code. Hence the contention is
rejected.
5. Learned counsel secondly suggested that in fact this accused had met with an accident
with the police jeep driven by Arun Kumar (PW6) and, therefore, he was falsely
implicated at the instance of the police. Very curiously this is not suggested to the witness
at all. In the absence of any suggestion or material in cross-examination such lame plea
cannot be accepted.
6. Lastly, the learned counsel tried to suggest that the appellant was not given any idea
about his right to have the search taken in presence of a Gazetted Officer in terms of
Section 42 of the Act. We have carefully seen the evidence. To a specific question: what
did you say to the accused at the place of occurrence?, the answer by the witness is: I told
him that we have an information from Mukhbir that there is some Ganja hidden in your
house and I have to take your search. If you want the search to be conducted in the
presence of some Gazetted Officer or in the presence of a Magistrate or you had no
objection if I conduct the search myself. Before that even in the examination-in-chief the
witness had very specifically stated "Ghasita Sahu was informed about the information
received from the informant and it was asked from him if he wanted any Magistrate to
conduct the search or the police themselves could have done that". Learned counsel
wanted to suggest that this was not the way to inform the accused of his right.
Unfortunately, no such specific question was put to the witness and in our opinion
considering the language, the search of the house cannot be said to be illegal in any
manner.
7

. In the first place, there is no question in this case, of any such right of the accused.
Section 51 of the Act specifically provides that the provisions of Criminal Procedure
Code shall apply in so far as they are inconsistent to the provisions of the Act to all
warrants, arrest, searches and seizures made under this Act. The right of the search being
taken only in presence of a Magistrate or a Gazetted Officer is restricted where the search
is to be taken of a "person" of the accused. In this case the search was of a house and,
therefore, all that the Investigating Officer had to follow was the conditions under Section
42 of the Act read with Section 100, Cr.P.C. Therefore, the argument that the accused had
any right in respect of the aforementioned search and that right has been breached is
wholly incorrect. The law is now settled that this condition under Section 50 applies only
where the search is of a "person" of accused [See State of H.P. v. Pawan Kumar [(2005) 4
SCC 350]]. In this case the search was not of the person but of his house. 2005 AIR
SCW 2154

8. However, it is pointed out by the learned counsel that the quantity of Ganja was less
than the commercial quantity though more than the small quantity and that the accused
has all through been behind the bars after his arrest and he has almost completed four
years in Jail. Considering that the accused is a middle-aged man and comes from the poor
background as claimed by the counsel, we would chose to modify his punishment of five
years to the sentence already undergone. We also reduce the amount of fine from Rs.
20,000/- to Rs.10,000/- and in default of payment of fine the accused would undergo
further period of Rigorous Imprisonment for six months. Barring this modification, the
appeal is dismissed.
Appeal dismissed.
AIR 2008 SUPREME COURT 1426 "Sher Singh v. State of Punjab"
(From : 2005 (1) Chand Cri C 338 (Punj And Har))
Coram : 2 PRAKASH PRABHAKAR NAOLEKAR AND MARKANDEY KATJU, JJ.
Criminal Appeal No. 646 of 2006, D/- 15 -2 -2008.
Sher Singh and Ann v. State of Punjab.
(A) Evidence Act (1 of 1872), S.32 - DYING DECLARATION - Dying declaration -
Reliability - Certificate of doctor as to fitness of declarant - Not always essential -
Requirement of certificate of doctor is only a rule of caution.
In case of dying declaration since the accused has no power of cross-examination, the
Court would insist that the dying declaration should be of such a nature as to inspire full
confidence of the Court in its truthfulness and correctness. The Court should a
@page-SC1427
ensure that the statement was not as a result of tutoring or prompting or a product of
imagination. It is for the Court to ascertain from the evidence placed on record that the
deceased was in a fit state of mind and had ample opportunity to observe and identify the
culprit. Normally, the Court places reliance on the medical evidence for reaching the
conclusion whether the person making a dying declaration was in a fit state of mind, but
where the person recording the statement states that the deceased was in a fit and
conscious state, the medical opinion will not prevail, nor can it be said that since there is
no certification of the doctor as to the fitness of mind of the declarant, the dying
declaration is not acceptable. What is essential is that the person recording the dying
declaration must be satisfied that the deceased was in a fit state of mind. Where it is
proved by the testimony of the Magistrate that the declarant was fit to make the statement
without there being the doctor's opinion to that effect, it can be acted upon provided the
Court ultimately holds the same to be voluntary and truthful. A certificate by the doctor is
essentially a rule of caution and, therefore, the voluntary and truthful nature of a
statement can be established otherwise. (Para 14)
(B) Penal Code (45 of 1860), S.300 - MURDER - DYING DECLARATION -
MAGISTRATE - Murder - Dying declaration - Deceased receiving 80% burn injuries -
Multiple dying declarations - First declaration recorded by police immediately after
incident giving story of accidental burn - Declaration recorded in presence of mother-in-
law - Officer recording statement deposing that declarant was under pressure - Second
dying declaration recorded by Magistrate inculpating all her in-laws and husband -
Declarant specifically stating that she was hospitalised on condition of giving wrong
statement - Subsequent declaration consistent with 2nd declaration - Second dying
declaration was more probable and looks natural - Although certificate of doctor was
absent - Can be relied upon when Magistrate who recorded it had certified fitness of
declarant - Conviction of accused, proper. (Para 15)
Cases Referred : Chronological Paras
2002 AIR SCW 3479 : AIR 2002 SC 2973 : 2002 Cri LJ 4095 (Ref.) 13
1999 AIR SCW 3440 : AIR 1999 SC 3455 : 1999 Cri LJ 4321 (Ref.) 10, 13
1999 AIR SCW 3727 : AIR 1999 SC 3695 : 1999 Cri LJ 4582 (Ref.) 12
(1998) 9 SCC 691 (Ref.) 11
Chander Shekhar Ashri (A.C.) and Ms. Rekha Aggarwal, for Appellants; Rahul Malik
(for Kuldip Singh), for Respondent.
Judgement
1. P. P. NAOLEKAR, J. :-Three accused have been convicted and sentenced with
rigorous imprisonment for life under Section 302/34 of the Indian Penal Code (IPC) and a
fine of Rs.1,000/- each and in default of which to undergo further rigorous imprisonment
of three months. The fourth accused has been tried in the Juvenile Court and hence no
order was passed by the Court regarding her.
2. The brief facts are that Jaspal Kaur (deceased) married the accused-appellant Sher
Singh in 1993. She was living in her matrimonial home for one-and-a-half years with the
accused her husband Sher Singh, father-in-law Attar Singh, mother-in-law Kailash Kaur
and sister-in-law Lakhwinder Kaur alias Rani who has been tried in the Juvenile Court.
On 18-7-1994 at about 12.00 noon, the deceased received serious burn injuries and was
taken to the Civil Hospital, Ludhiana. ASI Hakim Singh was informed and told to record
her statement. Hakim Singh (DW 1) recorded her statement at 9.00 p.m. in which she said
that the fire was accidental, caught while preparing tea. When her uncle Harbhajan Singh
(PW 4) met her on 19-7-1994, the deceased informed him that she was burnt by the
accused. On 20-7-1994, he moved an application before the District Magistrate to record
her statement. The ADM directed the Executive Magistrate, Rajiv Prashar (PW 7) to
record her statement and on 20-7-1994 he recorded her statement. Her uncle moved
another application this time before the DSP(Rural) Kanwarjit Singh (PW 1) requesting
him to re-examine the matter as according to him she was forced to make a wrong
statement before Hakim Singh. On 22-7-1994 the S.I. recorded her statement (Exh.PJ) at
about 8.05 p.m. after taking the doctor's opinion. He stated that she was fit to make a
statement. On 23-7-1994 Jaspal Kaur died due to burn injuries. Hence the offence was
converted into that of Section 302 read with Section 34, IPC which resulted in trial and
conviction.
3. It is submitted by the learned counsel for the appellant before us that while
appreciating
@page-SC1428
the evidence, reliance should have been placed upon the first dying declaration made on
18-7-1994, which was first in time immediately after the incident wherein she stated that
the fire was accidental and no one was responsible for the same, particularly when there
are 6 dying declarations in total (3 written and 3 oral) wherein the statement has been
improved from time to time. Submission of the learned counsel for the appellants is that it
is only when the uncle of the deceased met her in the hospital that she changed her first
dying declaration and implicated the accused-appellants for commission of crime. When
the dying declaration was recorded by the Executive Magistrate on 20-7-1994, there is no
certification of the doctor that she was in a fit state of mind to give the dying declaration
even though she had received 80% burns. It is urged that one local congress worker
Nirmala Sharma was present at the bedside of the deceased when the dying declaration
was made by her on 20-7-1994 and possibility of her being tutored could not be ruled out.
4. To appreciate argument of the learned counsel for the appellants, it would be necessary
to scrutinize the written dying declarations made by the deceased to Hakim Singh, Rajiv
Prashar and Arvind Puri and oral dying declarations made before her uncle and father. On
18-7-1994, ASI Hakim Singh recorded her statement when the doctor endorsed that she
was fit to make a statement. However, the doctor was not present when the dying
declaration was made, though her mother-in-law, one of the accused was present. Hakim
Singh in his statement before the Court stated that while recording the dying declaration
he felt that Jaspal Kaur was under pressure. In this statement, she said that when the
incident took place her husband and father-in-law were not present in the house and her
mother-in-law was standing outside the house, in front of the gate, while she was
preparing tea. The stove suddenly burst and she was soaked with oil and her clothes
caught fire. She shouted "bachao, bachao", her mother-in-law heard her cries for help and
doused her with water from the bathroom. Thereafter, she was taken to the hospital by her
mother-in-law with the help of her neighbours and she said that no one was at fault for
the accident.
5. Harbhajan Singh (PW 4) in his deposition said that after 6 to 7 months of the marriage
of the deceased with one of the accused, Sher Singh, all accused persons were demanding
money to the tune of Rs.10,000/-. On 15-7-1994, the deceased had gone to his house at
Jullandhar and told him that she has to take Rs.10,000/-from her father otherwise she
would have to face dire consequences. At that time, the father of the deceased, Balkar
Singh, was in Thailand and, therefore, Harbhajan Singh sent her back. On 19-7-1994, he
went to Ludhiana to see Jaspal Kaur. He saw that the house was locked and on enquiring
from neighbours he came to know that she was admitted in the hospital. He met her in the
Medical Hospital where she was admitted. She told him that her husband, mother-in-law,
father-in-law and sister-in-law set her ablaze and that her mother-in-law held her by her
hair and threw her on the ground, Attar Singh poured kerosene oil from a 'peepi' lying
closeby on her and Sher Singh set her on fire with a match-box and her sister-in-law
exhorted them that she should be burnt so that she does not survive. She requested them
to take her to the hospital and the accused persons said that if she would make a
statement in their favour then alone she would be taken to the hospital. Consequently,
since she was under fear, she made a statement in their favour to the police. Hearing this,
PW-4 moved an application before the DM for re-recording of her statement. On 20-7-
1994. the deceased's statement was recorded by the Executive Magistrate Rajiv Prashar
(PW 7) (Exh.PG) in the presence of Dr. Rajinder Kumar and Nirmala Sharma. In the
dying declaration, it was stated that she was burnt by her in-laws, when her mother-in-
law asked her to prepare tea. Her father-in-law, mother-in-law and sister-in-law poured
oil on her and burnt her. She said that her husband was not with her but thereafter in the
next sentence she said that they were four - father-in-law, mother-in-law, sister-in-law and
husband. She said further that they had stated that unless she would make a wrong
statement they would not take her to the hospital and on agreeing to it they had taken her
to the hospital. It is stated by her that her sister-in-law lit the fire by match-stick. She
went to the bathroom where the bucket of water was kept and poured the same upon
herself.
6. Dr. Rajinder Kumar, Registrar, Plastic
@page-SC1429
Surgery, New Daya Nand Hospital, Ludhiana issued a certificate to the effect that Jaspal
Kaur, aged 19 years, was admitted in the hospital on 19-7-1994 at 4.10 p.m. and
according to the record, the patient had got burn injuries upto 80 per cent. There is no
certificate of the doctor that the patient was in a condition to make a dying declaration but
it is apparent from the dying declaration that the doctor was present when it was
recorded.
7. Shri Rajiv Prashar, District Transport Officer, Gurdaspur (PW-7), who was posted as
Executive Magistrate on 20-7-1994, recorded the statement. He deposed that he reached
the hospital and enquired from Dr. Rajinder Kumar who was standing near the deceased
whether she was in a condition to make the statement and then the statement was
recorded. He deposed that the statement was read over to her and he obtained her right
hand thumb impression and the thumb impression of her right foot. He stated that the
statement is in his hand and bears his signature.
8. The oral dying declaration made before the father Balkar Singh (PW-6) on 22-7-1994
cannot be relied upon. It is admitted by him in the cross-examination that when he went
to the hospital he did not have any talk with his daughter as she was not in a position to
speak at that time. He touched her but she was unable to speak.
9. DSP (Rural), Ludhiana instructed SI Arvind Puri (PW 8) to record the statement of
injured Jaspal Kaur. Accordingly, he went to the hospital and moved an application
before the doctor whether the injured was in a fit condition to make the statement or not.
On 22-7-1994 at 6.45 p.m., the doctor certified that she was fit to make a statement and
accordingly her statement was recorded on 22-7-1994. In her statement, she said that her
father-in-law Attar Singh and husband Sher Singh often used to give beatings to her and
they used to ask her to bring Rs. 10,000/- from her parents. When she was preparing tea,
her mother-in-law caught hold of her hair and pressed the same towards the ground and
her father-in-law picked up a kerosene oil can and poured the same on her body and her
husband Sher Singh lit the match box. She raised alarm. Her sister-in-law Rani exhorted
that she should be burnt in such a manner that she might not escape. Her husband asked
all other persons to move out and thereafter closed the room from outside. After some
time, her husband opened the door of the room, but by that time she was badly burnt and
was speaking at a very low speech. When she pleaded them to take her to the hospital, her
father-in-law, husband and sister-in-law said that they would take her to the hospital only
if she would make the statement in their favour. On that she told them that she would do
so. She got her statement recorded on 18-7-1994 under duress of her in-laws. On 22-7-
1994, she stated that the offence had been committed by her mother-in-law, father-in-law,
husband and sister-in-law for not bringing Rs.10,000/-from her parents, with an intention
to kill her by pouring kerosene oil upon her. She stated that she had no fear of anyone
then and thereafter she got her above mentioned statement recorded. According to her, the
statement heard by her was correct. She put her right thumb impression on the dying
declaration. This statement is proved by SI Arvind Puri (PW-8).
10

. In Paparambaka Rosamma and Ors. v. State of Andhra Pradesh. AIR 1999 SC 3455, it
was held by this Court that although the doctor had appended a certificate to the dying d
preciating eclaration to the effect that the patient was conscious while recording the
statement, yet it would not be safe to accept the dying declaration as true and genuine,
since the certificate of the doctor was only to the effect that the patient was conscious
while recording the statement. It is necessary for the prosecution to prove, that the dying
declaration is true, voluntary and free from all doubts. But the doctor's certificate only
said that the patient was conscious, it did not say that the patient was in a fit state of
mind. In medical science there are two stages - one of consciousness and the other of a fit
state of mind, but they are not synonyms, and one may be conscious without being in a fit
state of mind. The Court did not rely upon the dying declaration as the court had also
found serious lacunae in other material particulars. 1999 AIR SCW 3440

11. But in Harjit Kaur and Others v. State of Punjab and others, (1998) 9 SCC 691, it is
held that even if the dying declaration is not certified by the doctor, it will still have to be
accepted because the person recording it had stated that the victim was fit to make the
statement and had said that he took the doctor's opinion regarding the same.
@page-SC1430
12

. In Koli Chunilal Savji and Anr. v. State of Gujarat, AIR 1999 SC 3695, the question
again was whether in the absence of a doctor's certificate as regards the mental fitness of
the person to make a statement, would it not be reliable? This Court held that the
requirement of such endorsement is only a matter of prudence and the ultimate test is
whether the dying declaration is voluntary and truthful. Before recording the dying
declaration, the officer concerned must find that the declarant was in a fit condition to
make the statement and if the Magistrate is satisfied about the condition of the patient to
make the statement, such statement can be relied upon. 1999 AIR SCW 3727

13

. In Laxman v. State of Maharashtra, AIR 2002 SC 2973,a Constitution Bench of this


Court had an occasion to consider similar aspects regarding veracity of dying declaration
where the doctor's certificate regarding the fitness of a person had not been taken. This
Court held that if the person recording the statement is satisfied that the person was fit
then the veracity of the declaration will not be questioned. The Court said that the view
taken in Paparambaka case (supra) of getting the doctor's certificate on the state of mind
of the patient to make the statement would be a hyper-technical view, particularly when
the Magistrate stated that the patient was in a fit state of mind and whereafter he recorded
the dying declaration. The Court further held that where the Magistrate had ascertained
from the doctor whether the victim was in a fit condition to make the statement and
obtained an endorsement to that effect, merely because the endorsement was not made on
the dying declaration but on the application, it would not render the dying declaration
suspicious in any manner. 2002 AIR SCW 3479
1999 AIR SCW 3440

14. Acceptability of a dying declaration is greater because the declaration is made in


extremity. When the party is at the verge of death, one rarely finds any motive to tell
falsehood and it is for this reason that the requirements of oath and cross examination are
dispensed with in case of a dying declaration. Since the accused has no power of cross-
examination, the court would insist that the dying declaration should be of such a nature
as to inspire full confidence of the court in its truthfulness and correctness. The court
should ensure that the statement was not as a result of tutoring or prompting or a product
of imagination. It is for the court to ascertain from the evidence placed on record that the
deceased was in a fit state of mind and had ample opportunity to observe and identify the
culprit. Normally, the court places reliance on the medical evidence for reaching the
conclusion whether the person making a dying declaration was in a fit state of mind, but
where the person recording the statement states that the deceased was in a fit and
conscious state, the medical opinion will not prevail, nor can it be said that since there is
no certification of the doctor as to the fitness of mind of the declarant, the dying
declaration is not acceptable. What is essential is that the person recording the dying
declaration must be satisfied that the deceased was in a fit state of mind. Where it is
proved by the testimony of the Magistrate that the declarant was fit to make the statement
without there being the doctor's opinion to that effect, it can be acted upon provided the
court ultimately holds the same to be voluntary and truthful. A certificate by the doctor is
essentially a rule of caution and, therefore, the voluntary and truthful nature of a
statement can be established otherwise.
15. In the present case, the first dying declaration was recorded on 18-7-1994 by ASI
Hakim Singh (DW-1). The victim did not name any of the accused persons and said that
it was a case of an accident. However, in the statement before the court, Hakim Singh
(DW.-l) specifically deposed that he noted that the declarant was under pressure and at
the time of recording of the dying declaration, her mother-in-law was present with her. In
the subsequent dying declaration recorded by the Executive Magistrate Rajiv Prashar
(PW 7) on 20-7-1994, she stated that she was taken to the hospital by the accused only on
the condition that she would make a wrong statement. This was reiterated by her in her
oral dying declaration and also in the written dying declaration recorded by SI Arvind
Puri (PW 8) on 22-7-1994. The first dying declaration exonerating the accused persons
made immediately after she was admitted in the hospital was under threat and duress that
she would be admitted in the hospital only if she would give a statement in favour of the
accused persons in order to save her in-laws and husband. The first dying declaration
@page-SC1431
does not appear to be coming from a person with free mind without there being any
threat. The second dying declaration was more probable and looks natural to us. Although
it does not contain the certificate of the doctor that she was in a fit state of mind to give
the dying declaration but the Magistrate who recorded the statement had certified that she
was in a conscious state of mind and in a position to make the statement to him. Mere
fact that it was contrary to the first declaration would not make it untrue. The oral dying
declaration made to the uncle is consistent with the second dying declaration implicating
the accused persons stating about their involvement in the commission of crime. The
third dying declaration recorded by the SI on the direction of his superior officer is
consistent with the second dying declaration and the oral dying declaration made to her
uncle though with some minor inconsistencies. The third dying declaration was recorded
after the doctor certified that she was in a fit state of mind to give the statement.
16. On overall consideration of the entire evidence, we find no infirmity in the judgment
of the High Court which has considered all material evidence placed by the prosecution
while arriving at the conclusion of finding the accused guilty of an offence they were
charged with. The appeal is, accordingly, dismissed.
Appeal dismissed.
AIR 2008 SUPREME COURT 1431 "B. P. Agarwal v. Dhanalakshmi Bank Ltd."
(From : Kerala)*
Coram : 2 Dr. A. PASAYAT AND P. SATHASIVAM, JJ.
Civil Appeal No. 922 of 2002, D/- 25 -1 -2008.
B.P. Agarwal and Anr. v. Dhanalakshmi Bank Ltd. and Ors.
Civil P.C. (5 of 1908), O.41, R.1(3) - APPEAL - Appeal - Powers of Court - Direction to
appellant to deposit certain sum in trial Court within particular time - Could not be passed
by High Court in absence of application for stay.
A. S. No. 48 of 2001, D/-07-06-2001 (Ker.), Reversed. (Para 6)
Cases Referred : Chronological Paras
2004 AIR SCW 3102 : AIR 2004 SC 3325 (Ref.) 5
(1998) 8 SCC 676 (Ref.) 4
B.V. Deepak for M/s. T.T.K. Deepak and Co., for Appellants; K.V. Mohan for K.R.
Nambiar, for Respondents.
* A.S. No. 48 of 2001, D/- 7-6-2001 (Ker.)
Judgement
1. Dr. ARIJIT PASAYAT, J. :- Challenge in this appeal is to the order passed by the
Division Bench of the Kerala High Court. By the impugned order the High Court in
exercise of jurisdiction under order XLI Rule 1 (3) of the Code of Civil Procedure, 1908
(in short the 'CPC') directed the appellant to deposit a sum of Rs.5,00,000/- in trial court
within a particular time. Appellants question the correctness of the order on the ground
that the High Court could not have referred to Order XLI Rule 1 (3) in the absence of any
application for stay.
2. Learned counsel for the respondents on the other hand supported the order of the High
Court.
3. Undisputedly, in the present case there was no application for stay filed. A few
decisions of this Court being relevant need to be noted.
4. In Kayamuddin Shamsuddin Khan v. State Bank of India [1998 (8) SCC 676] the
dispute related to Order XLI, Rule 1 (3) it was held that if the amount is not deposited,
the appeal could be directed to be dismissed. Obviously reference was to Order XLIII,
Rule 5(5). In paragraphs 6 and 8 this Court observed as follows :
"6. The learned counsel for the respondent has invited our attention to sub-rule (3) of
Rule 1 of Order XLI in the Code of Civil Procedure, as amended in the State of
Maharashtra, which reads as under :
"(3) Where the appeal is against a decree for payment of money, the appellant shall,
within such time as the Appellate Court may allow, deposit the amount disputed in the
appeal or furnish such security in respect thereof as the Court may think fit:
Provided that the Court may dispense with the deposit or security where it deems fit to do
so for sufficient cause."
8. This would mean that non-compliance with the direction given regarding deposit under
sub-rule (3) of Rule 1 of Order XLI would result in the Court refusing to stay the
execution of the decree. In other words, the application for stay of the execution of
@page-SC1432
the decree could be dismissed for such non-compliance but the Court could not give a
direction for the dismissal of the appeal itself for such non-compliance."
5

. Similarly, in Devi Theatre v. Vishwanath Raju (2004 (7) SCC 337] it was inter alia
observed as follows : 2004 AIR SCW 3102

"5. The learned counsel for the appellant submits that appeal lies from every decree
passed by any court exercising original jurisdiction. The jurisdiction of the court in first
appeal extends to examine the questions of facts as well as that of law. It is though true as
pointed out by the learned counsel for the respondent that under Order 41, Rule 11. CPC
it would be open for the court to dismiss the appeal in limine at the time of admission but
even examining the matter from that point of view we find that the court while
considering the question of admission of appeal filed under Section 96, CPC, may admit
the appeal if considered fit for full hearing having prima facie merit. Otherwise, if it finds
that the appeal lacks merits, it may be dismissed at the initial stage itself. But admission
of the appeal, subject to condition of deposit of some given amount, is not envisaged in
the provision as contained under Section 96 read with Order 41. Rule 11, CPC. The
deposit of the money would obviously have no connection with the merits of the case,
which alone would be the basis for admitting or not admitting an appeal filed under
Section 96, CPC. Further, imposition of condition that failure to deposit the amount
would result in dismissal of the appeal compounds the infirmity in the order of
conditional admission.
6. It is a different matter, in case the appellant prays for stay of the execution of the
decree or for any order by way of an interim relief during the pendency of the appeal; it is
open for the court to impose any condition as it may think fit and proper in the facts and
circumstances of the case. Otherwise imposing a condition of deposit of money subject to
which an appeal may be admitted for hearing on merits, is not legally justified and such
order cannot be sustained."
6. In the instant case there is no direction that in case of non-payment, the appeal is to be
dismissed. In the absence of any application for stay the High Court could not have
passed the order impugned. The direction for deposit as given accordingly stands vacated.
7. The appeal is allowed but without any order as to costs.
Appeal allowed.
AIR 2008 SUPREME COURT 1432 "Labh Singh v. Bachan Singh"
(From : 2005 (2) Land LR 73 (Punj. And Har.))
Coram : 2 Dr. A. PASAYAT AND AFTAB ALAM, JJ.
Civil Appeal No. 342 of 2008 (arising out of SLP (C) No. 3100 of 2005), D/- 11 -1 -2008.
Labh Singh and Ors. v. Bachan Singh.
Punjab Pre-emption Act (1 of 1913), S.15 - PRE-EMPTION - POSSESSION - POWER
OF ATTORNEY - Pre-emption - Plaintiff relative of vendor claiming superior right of
preemption as co-sharer of vendor - Vendor selling his share and his sister's share in land
to defendant as power of attorney holder of his sister - Sale to extent of share of sister is
not pre-emptible - Defendant thus have improved his status as that of a co-sharer -
Plaintiff does not have superior right of pre-emption - Suit for possession liable to be
dismissed. (Para 5)
Cases Referred : Chronological Paras
(1996) 7 SCC 97 (Rel. on) 9, 11, 13, 15
1991 AIR SCW 2926 : AIR 1992 SC 207 11
AIR 1986 SC 859 (Rel. on) 8, 10, 11, 13, 15
Pardeep Gupta, K.K. Mohan, Suresh Bharati, Ms. Vijay Laxmi Lithanthem, for
Appellants; Kuldip Singh, for Respondent.
Judgement
Dr. ARIJIT PASAYAT, J. :- Heard learned counsel for the parties.
2. Leave granted.
3. Challenge in this appeal is to the judgment dated 29-10-2004 passed by a learned
Single Judge of the Punjab and Haryana High Court in a Second Appeal filed under
Section 100 of the Code of Civil Procedure, 1908 (in short 'CPC'). The Second Appeal
before the High Court was filed by the plaintiff who had succeeded before the Trial
Court; but the First Appellate Court set aside the judgment and decree passed. In the
Second Appeal, the judgment and decree of the
@page-SC1433
Trial Court was restored and those of the First Appellate Court were set aside. The
respondent as plaintiff filed a suit for preemption.
4. Background facts in a nutshell are as follows :
The plaintiff filed suit for possession alleging therein that the vendor Singh Ram is jointly
recorded as owner of half share of land measuring 24 kanals situated in village Fatehbad
Tehsil Naraingarh. The plaintiff and vendor Singh Ram are related to each other as the
plaintiff is fourth degree collector of the vendor. Singh Ram had sold half of 24 kanal of
land by way of registered sale deed dated 2-6-1979 registered on 29-6-1979 for an
ostensible consideration of Rs. 30,000/-.
The plaintiff inter alia claimed superior right of pre-emption as a co-sharer with the
vendor in the land in dispute under Section 15 (1) of Punjab Pre-emption Act, 1913
(hereinafter referred to as the 'Act'). It was the case of the defendant that Singh Ram was
owner of only 3/4th share and his sister was owner of 1/4th share and both of them were
jointly owners of half of the land. Singh Ram alone has half share of land measuring 24
kanals, but it was asserted that the sale deed was by Singh Ram and Angrezo who are
owners of the land. In replication, it was pointed out that the sale is by Singh Ram for
himself and as Mukhtiar of Smt. Angrezo. Therefore, the sale is pre-emptible.
5. The Trial Court decreed the suit on the ground that the plaintiff is a co-sharer and has
thus superior right of pre-emption. The Trial Court negatived the argument raised by the
defendant that the sale is by a female and thus governed by the provisions of sub-section
(2) of Section 15 of the Act. However, in the appeal filed by defendant, the judgment and
the decree passed by Trial Court were set aside and it was held that vendee has improved
his status as that of a co-sharer in view of the fact that the sale to the extent of share of
Angrezo is not preemptible being not governed by the provisions of Section 15(2) of the
Act and thus the plaintiff does not have superior right of pre-emption.
6. In Second Appeal, the following questions were formulated for consideration :
1. Whether the plaintiff has superior right of pre-emption as co-sharer?
2. Whether the suit for pre-emption can be dismissed for not disclosing the complete fact
regarding the sale by Angrezo, a female vendor?
7. The High Court was of the view that right of pre-emptor cannot be defeated by virtue
of amendment in Section 15 of the Act taking away right on the basis of co-sharer.
8

. In support of the appeal, learned counsel for the appellants submitted that the High
Court misconstrued the decision of this Court in Atam Prakash v. State of Haryana and
Ors. (1986 (2) SCC 249). AIR 1986 SC 859

9. It was further submitted that the view was re-iterated in Mahant Braham Dass Singh
Pannu v. Om Prakash Chaudhary (1996 (7) SCC 97).
10

. In Atam Prakash case (supra) it was inter alia observed as follows :- AIR 1986 SC
859, Para 13

"We are thus unable to find any justification for the classification contained in Section 15
of the Punjab Pre-emption Act of the Kinsfolk entitled to preemption. The right of pre-
emption based on consanguinity is a relic of the feudal past. It is totally inconsistent with
the constitutional scheme. It is inconsistent with modern ideas. The reason which justified
its recognition quarter of a century ago namely, the preservation of the integrity of rural
society, the unity of family life and the agnatic theory of succession are today irrelevant.
The list of kinsfolk mentioned as entitled to pre-emption is intrinsically defective and
self-contradictory. There is therefore no reasonable classification and clauses "First",
"Secondly" and "thirdly" of Section 15(1)(a). First, secondly, and thirdly of Section 15(1)
(b), clauses First, secondly and thirdly of Section 15 (1)(c) and the whole of Section
15(2) are, therefore, declared ultra-vires the Constitution.
We are told that in some cases suits are pending in various courts and, where decrees
have been passed, appeals are pending in appellate courts. Such suits and appeals will
now be disposed of in accordance with the declaration granted by us. We are told that
there are a few cases where suits have been decreed and the decrees have become final,
no appeals having been filed against those decrees. The decrees will be binding inter-
parties and the declaration granted by us will be of no avail to the parties
@page-SC1434
thereto."
11. In Mahant Braham Dass's case (supra) it was noted as follows :

"The question then is whether he is a co-sharer. It is seen that at one time he was co-
sharer but subsequently, brothers effected by mutual consent partition and the vendee/
appellants vendor Jai Singh was in separate possession and enjoyment of the property.
Therefore, the mere mention in para 3 that he is a co-sharer is not independent of the right
to vicinage. It would appear that the pleading was made on the basis that the respondent
is not the real brother of the vendor of the appellant and on the basis thereof he claimed to
be the co-sharer. Therefore, Mr. G.K. Bansal, learned counsel for the respondent, sought
to place reliance on the judgment of this Court in Bhikha Ram v. Ram Sarup (1992 (1)
SCC 319) where a Bench of three Judges of this Court held that a co-sharer has a right of
pre-emption under clause Fourthly' of Section 15(1)(b) which was not declared ultra vires
in Atam Prakash v. State of Haryana (1986 (2) SCC 249) and, therefore, he was entitled
to seek pre-emption. It is true that independent of right of kinship, if there is any right as
co-sharer, in other words, on the date when the alienation was made if the vendor of the
appellant had remained in joint possession and enjoyment without any partition, he would
become a co-sharer with the respondent independent of the right of kinship. But if the
joint enjoyment is by virtue of the unity in possession and enjoyment as members of the
joint family property then it is not an independent right of co-sharer but as a member of
the joint family or coparcener." 1991 AIR SCW 2926
AIR 1986 SC 859

12. Learned counsel for the respondent on the other hand supported the judgment of the
High Court.
13

. As was noted in Atam Prakash Case (supra), the decision was applicable to pending
suits and appeals. As noted above, the view was re-iterated in Mahant Braham Dass case
(supra). AIR 1986 SC 859

14. A few factual aspects as evident from the order of the trial Court which projects the
case of the parties need to be noted :
"But learned counsel for the defendants has argued that since it has been stated by the
plaintiff that he is cultivating the land separately, so the plaintiff is not a co-sharer in the
suit land. But this arguments of the learned counsel for the defendants, is not
maintainable because the plaintiff has stated that the property in dispute was a joint
property with the vendors and himself. So, on this ground the plaintiff has superior right
of pre-emption over the suit land.
15

. In view of what has been stated by this Court in Atam Prakash case (supra) and Mahant
Braham Dass case (supra) the inevitable result is that the appeal deserves to succeed
which we direct. The judgment of the High Court restoring the judgment and decree of
the trial Court is set aside. The first Appellate Court had taken the correct view. It is
stated that certain amounts have been deposited by the respondent with the trial Court.
The said Court shall permit withdrawal of the amount deposited by the respondent on a
proper application being made. AIR 1986 SC 859

16. The appeal is allowed. There will be no order as to costs.


Appeal allowed.
AIR 2008 SUPREME COURT 1434 "Balu v. State of Tamil Nadu"
(From : Madras)*
Coram : 2 S. B. SINHA AND V. S. SIRPURKAR, JJ.
Criminal Appeal No. 295 of 2008 (arising out of SLP (Cri.) No. 4905 of 2007), D/- 12 -2
-2008.
Balu alias Bakthavatchalu v. State of T.N.
Juvenile Justice (Care and Protection of Children) Act (56 of 2000), S.2(1) (as substituted
by 2006 Act) - JUVENILE JUSTICE - TRIAL COURT - "Juvenile in conflict with law" -
Determination of age of offender - Trial Court directed to make enquiry as to age of
appellant on date of commission of offence - Further directed that if he is found to be
juvenile within meaning of Act trial Court would proceed in accordance with law.
Cri. A. No. 724 of 2001, D/-16-11-2005 (Mad), Reversed. (Para 16)
Cases Referred : Chronological Paras
2006 AIR SCW 2319 : AIR 2006 SC 1933 : 2006 Cri LJ 2464 (Foll.) 15
@page-SC1435

2006 AIR SCW 2648 : AIR 2006 SC 2157 : 2006 Cri LJ 2791 : 2006 (4) ALJ 269 (Foll.)
14
2005 AIR SCW 3088 : AIR 2005 SC 2731 : 2005 Cri LJ 3091 (Foll.) 8
2005 AIR SCW 5700 : AIR 2006 SC 191 : 2006 Cri LJ 126 (Foll.) 13
2000 AIR SCW 2037 : AIR 2000 SC 2264 : 2000 Cri LJ 2971 (Foll.) 8
1997 AIR SCW 4205 : AIR 1998 SC 236 : 1998 Cri LJ 390 15
AIR 1989 SC 1329 : 1990 Cri LJ 2671 : 1990 All LJ 65 13
Rana Mukherjee, Anand, Ms. Indrani, Ishit Saharia, Abhijit Sengupta, for Appellant; V.G.
Pragasam, S. Joseph Aristotle, S. Prabu Ramasubramanian, for Respondent.
* Cri. A. No. 724 of 2001, D/- 16-11-2005 (Mad).
Judgement
1. S. B. SINHA, J. :-Leave granted.
2. Appellant was prosecuted for commission of an offence under Section 302 of the
Indian Penal Code. The occurrence took place on 20th April, 1998. He was arrested on
the charge of murder of one Ramu Maistry on 8th May, 1998. Upon completion of
investigation a charge sheet was filed against him on 30th November, 1998. The learned
trial Court delivered a judgment on 28th April, 2000. In the said judgment his age was
shown to be '18'. An application was filed for sending him to Borstal School in terms of
Section 10-A of the Tamil Nadu Borstal Schools Act, which was refused. An appeal
preferred by the appellant before the High Court has been dismissed by reason of the
impugned judgment.
This Court issued a limited notice as to whether the appellant was a juvenile on the date
of occurrence of the incident.
3. Mr. Mukherjee, the learned counsel appearing on behalf of the appellant, submitted
that in view of the materials placed on records, an inquiry should have been initiated as
regards the age of the appellant.
4. The Juvenile Justice Act, 1986 (hereinafter referred to as "the Act" was applicable
when the incident took place. In terms whereof, a juvenile, under Section 2(h) was
defined as a boy who has not attained the age of 16 years.
5. The Parliament, however, enacted, the Juvenile Justice (Care and Protection of
Children) Act, 2000. It came into force with effect from 1st April, 2001.
6. Section 2(k) defines 'juvenile' to mean a person who has not completed eighteen years
of age.
7. Section 20 of the Act reads as under :-
"20. Special provision in respect of pending cases.-
Notwithstanding anything contained in this Act, all proceedings in respect of a juvenile
pending in any court in any area on the date on which this Act comes into force in that
area, shall be continued in that court as if this Act had not been passed and if the court
finds that the juvenile has committed an offence, it shall record such finding and instead
of passing any sentence in respect of the juvenile, forward the juvenile to the Board
which shall pass orders in respect of that juvenile in accordance with the provisions of
this Act as if it had been satisfied on inquiry under this Act that a juvenile has committed
the offence."
8

. A question was raised as to whether the date on which the incident took place or the date
on which the accused was produced before the Court would be the relevant date for
computing the age of juvenile in view of the decision of this Court in Arnit Das v. State
of Bihar : (2000) 5 SCC 488. The correctness of the said decision came up for
consideration before a Constitution Bench of this Court in Pratap Singh vs. State of
Jharkhand : (2005) 3 SCC 551. The Constitution Bench held 2000 AIR SCW 2037
2005 AIR SCW 3088

"31. Section 20 of the Act as quoted above deals with the special provision in respect of
pending cases and begins with a non obstante clause. The sentence "notwithstanding
anything contained in this Act, all proceedings in respect of a juvenile pending in any
court in any area on the date on which this Act came into force" has great significance.
The proceedings in respect of a juvenile pending in any court referred to in Section 20 of
the Act are relatable to proceedings initiated before the 2000 Act came into force and
which are pending when the 2000 Act came into force. The term "any court" would
include even ordinary criminal courts. If the person was a "juvenile" under the 1986 Act
the proceedings would not be pending in criminal courts. They would be pending in
criminal courts only if the boy had crossed 16 years or the girl had crossed 18 years. This
shows that Section 20 refers to cases where a person had ceased to be a juvenile under the
1986 Act but had not yet crossed the age of 18 years then the pending case shall continue
in that court as if the 2000
@page-SC1436
Act has not been passed and if the court finds that the juvenile has committed an offence,
it shall record such finding and instead of passing any sentence in respect of the juvenile,
shall forward the juvenile to the Board which shall pass orders in respect of that
juvenile."
It concluded :-
"37 . The net result is :
(b) The 2000 Act would be applicable in a pending proceeding in any court/authority
initiated under the 1986 Act and is pending when the 2000 Act came into force and the
person had not completed 18 years of age as on 1-4-2001." In a separate judgment, one of
us (S.B. Sinha, J.) stated :-
"95. Section 20 of the Act of 2000 would, therefore, be applicable when a person is below
the age of 18 years as on 1-4-2001. For the purpose of attracting Section 20 of the Act, it
must be established that: (i) on the date of coming into force the proceedings in which the
petitioner was accused were pending; and (ii) on that day he was below the age of 18
years. For the purpose of the said Act, both the aforementioned conditions are required to
be fulfilled. By reason of the provisions of the said Act of 2000, the protection granted to
a juvenile has only been extended but such extension is not absolute but only a limited
one. It would apply strictly when the conditions precedent therefor as contained in
Section 20 or Section 64 are fulfilled. The said provisions repeatedly refer to the words
"juvenile" or "delinquent juveniles" specifically. This appears to be the object of the Act
and for ascertaining the true intent of Parliament, the rule of purposive construction must
be adopted. The purpose of the Act would stand defeated if a child continues to be in the
company of an adult. Thus, the Act of 2000 intends to give the protection only to a
juvenile within the meaning of the said Act and not an adult. In other words, although it
would apply to a person who is still a juvenile having not attained the age of 18 years but
shall not apply to a person who has already attained the age of 18 years on the date of
coming into force thereof or who had not attained the age of 18 years on the date of
commission of the offence but has since ceased to be a juvenile."
9. Recently the Parliament has introduced Juvenile Justice (Care and Protection of
Children) Amendment Act, 2006 (which came into force with effect from 23-8-2006), in
terms whereof retrospective and restorative meaning was given to the definition of
'juvenile' stating :-
"4. In section 2 of the principal Act, -
(iv) for clause (1), the following clause shall be substituted, namely :-
(1) "juvenile in conflict with law" means a juvenile who is alleged to have committed an
offence and has not completed eighteenth year of age as on the date of commission of
such offence;"
10. In view of the decision of the Constitution Bench of this Court as also the
amendments carried out by the Parliament, evidently the question as to whether the
appellant was aged '18' as on 1st April, 2001 requires consideration.
11. In a situation of this nature, where despite the possibility of a juvenile having been
tried and convicted for rigorous imprisonment for life by the trial court or the High Court,
this Court has in a large number of decisions directed an enquiry to be made as regards
the age of the juvenile.
12. We shall refer to a few of them.
13

. In Gurpreet Singh vs. State of Punjab : (2005) 12 SCC 615 a Bench of this Court opined
:- 2005 AIR SCW 5700

"18. Shri Prabha Shanker Misra, learned Senior Counsel appearing in support of Criminal
Appeal No. 710 of 1995 apart from challenging the conviction of the appellant Mohinder
Pal Singh on merits, which we have already dealt with, submitted that on the date of the
alleged occurrence, he was a juvenile within the meaning of Section 2(h) of the Juvenile
Justice Act, 1986 (hereinafter referred to as "the Act") as on that date he had not attained
the age of 16 years. It appears that this point was not raised either before the trial court or
the High Court. But it is well settled that in such an eventuality, this Court should first
consider the legality or otherwise of conviction of the accused and in case the conviction
is upheld, a report should be called for from the trial court on the point as to whether the
accused was juvenile on the date of occurrence and upon receipt of the report, if it is
found that the accused was juvenile on such date and continues to be so, he shall be sent
to juvenile home. But in case it finds that on the AIR 1989 SC 1329

@page-SC1437
date of the occurrence, he was juvenile but on the date this Court is passing final order
upon the report received from the trial court, he no longer continues to be juvenile, the
sentence imposed against him would be liable to be set aside. Reference in this
connection may be made to a decision of this Court in Bhoop Ram v. State of U.P. 7 in
which case at the time of grant of special leave to appeal, report was called for from the
trial court as to whether the accused was juvenile or not which reported that the accused
was not a juvenile on the date of the occurrence but this Court, differing with the report
of the trial court, came to the conclusion that the accused was juvenile on the date the
offence was committed and as he was no longer a juvenile on the day of judgment of this
Court, sentence awarded against him was set aside, though the conviction was upheld. In
the present case, we have already upheld the conviction of the appellant Mohinder Pal
Singh as well but it would be just and expedient to call for a report from the trial court in
relation to his age on the date of the occurrence." It was directed :-
"20. In Criminal Appeal No. 710 of 1995 filed by appellant Mohinder Pal Singh, call for
a report from the trial court as to whether on the date of occurrence this appellant was
juvenile within the meaning of Section 2(h) of the Juvenile Justice Act, 1986? The trial
court shall give opportunity to both the parties to adduce evidence on this point. Let the
entire original records of the trial court be returned to it. Report as well as records must
be sent to this Court within a period of three months from the receipt of this order. Upon
receipt of report from the trial court, final order shall be passed in this appeal."
14

. In Ravinder Singh Gorkhi vs. State of U.P : (2006) 5 SCC 584 this Court held :- 2006
AIR SCW 2648

"21. Determination of the date of birth of a person before a court of law, whether in a
civil proceeding or a criminal proceeding, would depend upon the facts and
circumstances of each case. Such a date of birth has to be determined on the basis of the
materials on records. It will be a matter of appreciation of evidence adduced by the
parties. Different standards having regard to the provision of Section 35 of the Evidence
Act cannot be applied in a civil case or a criminal case.
It was furthermore held :-
"38. The age of a person as recorded in the school register or otherwise may be used for
various purposes, namely, for obtaining admission; for obtaining an appointment; for
contesting election; registration of marriage; obtaining a separate unit under the ceiling
laws; and even for the purpose of litigating before a civil forum e.g. necessity of being
represented in a court of law by a guardian or where a suit is filed on the ground that the
plaintiff being a minor he was not appropriately represented therein or any transaction
made on his behalf was void as he was a minor. A court of law for the purpose of
determining the age of a party to the lis, having regard to the provisions of Section 35 of
the Evidence Act will have to apply the same standard. No different standard can be
applied in case of an accused as in a case of abduction or rape, or similar offence where
the victim or the prosecutrix although might have consented with the accused, if on the
basis of the entries made in the register maintained by the school, a judgment of
conviction is recorded, the accused would be deprived of his constitutional right under
Article 21 of the Constitution, as in that case the accused may unjustly be convicted.
39. We are, therefore, of the opinion that until the age of a person is required to be
determined in a manner laid down under a statute, different standard of proof should not
be adopted. It is no doubt true that the court must strike a balance. In case of a dispute,
the court may appreciate the evidence having regard to the facts and circumstances of the
case. It would be a duty of the Court of law to accord the benefit to a juvenile, provided
he is one. To give the same benefit to a person who in fact is not a juvenile may cause
injustice to the victim. In this case, the appellant had never been serious in projecting his
plea that he on the date of commission of the offence was a minor. He made such
statement for the first time while he was examined under Section 313 of the Code of
Criminal Procedure.
40. The family background of the appellant is also a relevant fact. His father was a
"Pradhan" of the village. He was found to be in possession of an unlicensed firearm. He
was all along represented by a lawyer. The
@page-SC1438
court estimated his age to be 18 years. He was tried jointly with the other accused. He
had been treated alike with the other accused. On merit of the matter also the appellant
stands on the same footing as the other accused. The prosecution has proved its case. In
fact no such plea could be raised as the special leave petition of the persons similarly
situated was dismissed when the Court issued notice having regard to the contention
raised by him for the first time that he was a minor on the date of occurrence."
15

. However, in Jitendra Ram VB. State of Jharkhand : (2006) 9 SCC 428 this Court
noticed that in a similar situation it would be necessary to make an enquiry. It was
stated :- 2006 AIR SCW 2319

"20. We are, however, not oblivious of the decision of this Court in Bhola Bhagat v. State
of Bihar wherein an obligation has been cast on the court that where such a plea is raised
having regard to the beneficial nature of the socially oriented legislation, the same should
be examined with great care. We are, however, of the opinion that the same would not
mean that a person who is not entitled to the benefit of the said Act would be dealt with
leniently only because such a plea is raised. Each plea must be judged on its own merit.
Each case has to be considered on the basis of the materials brought on records." 1997
AIR SCW 4205

It was furthermore held :-


22. We, therefore, are of the opinion that the determination of the age of the appellant as
on the date of the commission of the offence should be done afresh by the learned
Sessions Judge."
16. We are, therefore, of the view that in this case the trial judge should be directed to
hold the enquiry in regard to the age of the appellant on the date of commission of the
offence and in the event it is found that the appellant was a juvenile within the meaning
of the provisions of the said Act, he should proceed with the matter in accordance with
law. It is directed accordingly.
17. The appeal is allowed on the aforesaid terms.
Appeal allowed.
AIR 2008 SUPREME COURT 1438 "B.K. Muniraju v. State of Karnataka"
(From : Karnataka)
Coram : 2 TARUN CHATTERJEE AND P. SATHASIVAM, JJ.
Civil Appeal No. 1320 of 2008 (arising out of SLP (C) No. 21914 of 2004), D/- 15 -2
-2008.
B.K. Muniraju v. State of Karnataka and Ors.
(A) Karnataka Scheduled Castes and Scheduled Tribes (Prohibition of Transfer of Certain
Lands) Act (2 of 1979), S.4(1) - SCHEDULED CASTES AND SCHEDULED TRIBES -
LAND - DOCUMENTS - Grant of land - What amounts to - Document showing land in
question was purchased in public auction - Merely because said document styled or titled
as 'Certificate of Grant' - It cannot be construed that land was 'granted land'.
Document - Interpretation - Recitals to be looked into and not only the title. (Paras
12, 13)
(B) Constitution of India, Art.226 - WRITS - Certiorari - Finding of fact - Interference -
Authorities considered revenue extract and found that land was not 'granted land' but was
purchased in public auction - Cannot be interfered with under Art.226. (Para 16)
Cases Referred : Chronological Paras
2006 AIR SCW 4235 : AIR 2006 SC 2965 (Ref.) 12
2005 AIR SCW 3657 : AIR 2005 SC 4013 (Dist.) 17
2004 AIR SCW 2888 : AIR 2004 SC 2428 (Ref.) 12
(2004) 13 SCC 90 (Dist.) 17
2003 AIR SCW 1693 : AIR 2003 SC 1290 (Dist.) 17
2003 AIR SCW 3872 : AIR 2003 SC 3044 (Ref.) 14, 15
1999 AIR SCW 1129 : AIR 1999 SC 1441 (Ref.) 12
AIR 1984 SC 1151 (Dist.) 17
Altaf Ahmed, Sr. Adv., S.K. Kulkarni, M. Greesh Kumar, Vijay Kumar, Gd.
Aswadhanarayana, for Appellant; S.N. Bhat, D.P. Chaturvedi, Amit Kr. Chawla, Sanjay
R. Hegde, for Respondents.
Judgement
1. P. SATHASIVAM, J.Leave granted.
2. This appeal is directed against the order dated 12-7-2004 in Writ Appeal No. 795 of
2002 of the High Court of Karnataka at Bangalore dismissing the appeal of the
@page-SC1439
appellant confirming the order dated 7-12-2001 in Writ Petition No. 809 of 2000 of the
learned single Judge.
3. Brief facts :
The land in question bearing Survey No. 72 of Bommanahalli Village, Begur Hobli,
Bangalore South Taluk was allotted to the grand-father of the appellant, namely,
Motappa, who belongs to the Bhovi Community under the Mysore Land Grant Rules.
The total extent of mortgaged land was 2 acres 29 guntas. Later in 1956, Motappa
mortgaged 1 acre of the said land to one Munichennamma. Subsequently in 1959, he
mortgaged another portion of the land in favour of one Ramachandra Reddy, who was the
father of respondent Nos. 4 and 5 herein by raising a loan of Rs. 1,000/- to discharge his
previous mortgage amount.
4. After the demise of Motappa and Ramachandra Reddy, B. K. Muniraju, the appellant
herein and one M. Gopal paid the mortgaged amount of Rs. 1,000/- to respondent Nos. 4
and 5 and requested them to discharge the mortgage. Respondent Nos. 4 and 5 refused to
discharge the mortgage on the ground that it was not a mortgage deed but an absolute sale
deed. On coming into force of The Karnataka Scheduled Castes and Scheduled Tribes
(Prohibition of Transfer of Certain Lands) Act, 1978 (hereinafter referred to as "the Act"),
in the year 1981, the appellant herein along with M. Gopal, who died during the
pendency of writ appeal, and also a legal heir of deceased Motappa filed an application
before the Assistant Commissioner for cancellation of the sale deed and restoration of the
land under the provisions of the said Act. The Assistant Commissioner dropped the
proceedings on the ground that the alienation is after the expiry period of ten years of
non-alienation clause and hence the sale transaction is not in violation of the condition
governing grant and, therefore, it does not attract the provisions of Section 4(1) of the
Act. Against the said order, the appellant preferred an appeal before the Deputy
Commissioner which was also dismissed. Thereafter, the appellant approached the High
Court by filing Writ Petition No. 11821 of 1987 and the High Court allowed the said
petition and remanded the matter to the Assistant Commissioner for fresh disposal. The
High Court, in its order, directed the Assistant Commissioner to examine the original
Saguvali chit besides the original records relating to grant proceedings to the land in
question and then decide whether or not the provisions of the Act are attracted to the facts
of the case and whether the prohibition or alienation has been violated by the grantee.
Based on the said direction of the High Court, the Assistant Commissioner heard the
matter afresh. Finally, the Assistant Commissioner dismissed the matter holding that the
land in question is not a "granted land" but was purchased by Motappa in public auction
and hence, the Act is not applicable. It was also held that the auction purchaser does not
come under the Rules relating to the grant and there is no violation of the conditions of
the grant. Questioning the said order of the Assistant Commissioner, the appellant herein
preferred an appeal before the Deputy Commissioner who also dismissed the appeal
holding that the land granted at an upset price is alienated after a period of ten years of
non-alienation condition period, that the land granted at an upset price, did not attract the
provisions of Section 4(1) of the Act. The Deputy Commissioner also held that the
Saguvali chit issued in Form No. 1 indicates that the land was allotted to Motappa in
public auction at an upset price of Rs. 408.12.
5. Aggrieved by the aforesaid order of the Deputy Commissioner, the appellant filed Writ
Petition No. 809 of 2000 before the High Court. The main grievance of the appellant
before the learned single Judge was that the land was granted under the Mysore Land
Revenue Rules and not a land sold at public auction. It was contended that Insofar as the
"granted lands" are concerned, there was a total prohibition from alienating the land as
per the Rules that were prevailing at the time of grant in question. However, the learned
single Judge dismissed the writ petition and the writ appeal filed by the appellant herein
was also dismissed by the Division Bench which is impugned in this appeal.
6. Heard Mr. Altaf Ahmed, learned senior counsel appearing for the appellant and Mr. S.
N. Bhat, learned counsel appearing for respondent Nos. 4 and 5 and Mr. Amit Kumar
Chawla, learned counsel appearing for respondent Nos. 1-3.
7. Mr. Altaf Ahmed, learned senior counsel appearing for the appellant, by taking us
through the provisions of the Karnataka Scheduled Castes and Scheduled Tribes
(Prohibition of Transfer of Certain Lands)
@page-SC1440
Act, 1978 as well as the Rules under the Mysore Land Revenue Code, contended that the
authorities as well as the High Court committed an error in rejecting the claim of the
appellant inasmuch as it controverts the said Act and the Rules. He further submitted that
in view of the earlier order of the High Court directing the Assistant Commissioner to
verify the original record and arrive a fresh conclusion, the contrary decision by the said
officer is liable to be interfered with. On the other hand, Mr. S. N. Bhat, learned counsel
appearing for the contesting respondents 4 and 5, submitted that in view of the specific
factual finding by both the authorities namely, that Motappa purchased the land in
question by offering "price" in a public auction and it was not a "granted land", and the
same was duly considered and affirmed by the High Court, there is no valid ground for
interference.
8. We have perused all the relevant materials, Annexures and considered the rival
contentions. The only question to be considered in this appeal is whether the land in
question was "granted land" or "purchased for a price" by Motappa ? If it is found that the
subject-matter of a land was not a "granted land", it does not attract the vice of Section 4
of the Act.
9. The Statement of Objects and Reasons of the Karnataka Scheduled Castes and
Scheduled Tribes (Prohibition of Transfer of Certain Lands) Act, 1978 shows that the
non-alienation clause contained in the existing Land Grant Rules and the provisions for
cancellation of grants where the land is alienated in contravention of the above said
provision are found not sufficient to help the Scheduled Castes and Scheduled Tribes
grantees whose ignorance and poverty have been exploited by persons belonging to the
affluent and powerful sections to obtain sales or mortgages either for a nominal
consideration or for no consideration at all and they have become the victims of
circumstances. To fulfil the purposes of the grant, the land even if it has been alienated,
should be restored to the original grantee or his heirs. It is clear that in order to provide
for the prohibition of transfer of certain lands granted by Government to persons
belonging to the Scheduled Castes and Scheduled Tribes in the State of Karnataka, the
above said Act was enacted. In order to implement the provisions of the Act, the Rules
were framed. Among the provisions, we are concerned with Section 4 which prohibits
transfer of "granted land". It makes it clear that notwithstanding anything in any law,
agreement, contract on instrument, any transfer of granted land made either before or
after the commencement of the Act, in contravention of the terms of the grant of such
land or the law providing for such grant, or subsection (2) shall be null and void and no
right, title or interest in such land shall be conveyed or be deemed ever to have conveyed
by such transfer. Sub-section (2) makes it clear that no person shall, after the
commencement of the Act, transfer or acquire by transfer any granted land without the
previous permission of the Government. As per sub-section (3), the provisions of sub-
sections (1) and (2) also apply to the sale of any land in execution of a decree or order of
a Civil Court or of any award or order of any other authority.
10. Among the Rules, Rule 43 is relevant which speaks about the grant of occupancies.
Sub-rule (1) of Rule 43 mandates that all the lands shall ordinarily be sold by public
auction. Sub-rule (5) mandates Grant of occupancies to members of depressed classes.
Sub-rule (8) makes it clear that lands granted free or at upset price shall not be alienated
but may be accepted as security for loans. The note appended to the above provisions
makes it clear that depressed classes occurring in these rules will have to be constructed
as equivalent to the words "Scheduled Caste and Scheduled Tribes" occurring in the
Constitution of India.
11. From the materials, now we have to see whether the land purchased by Motappa was
a "granted land" as claimed by the appellant herein and one M. Gopal or purchased by
public auction for a price as claimed by contesting respondents 4 and 5 herein ? In order
to understand whether the land in question was a "granted land" or "land purchased for a
price at a public auction", it is incumbent on the part of the authorities to look into the
relevant records and decide the same. In view of the controversy in question, we verified
the document and the orders passed by the Assistant and the Deputy Commissioner and
the factual findings recorded by them. It reveals that the land in question was granted in
1948 and the Certificate of Grant/Saguvali Chit which was filed as Annexure R-1 before
the High Court (Annexure P-3 before us) shows that
@page-SC1441
the same was sold in public auction for a price. In other words, the land was purchased by
Motappa at a public auction and it was not a "granted land" within the meaning of Rule
43(8) of the Rules. It was contended that the finding recorded by both the authorities is
essentially a factual finding based on the relevant materials and the same cannot be
interfered with by the writ Court.
12

. The document in question which is filed as Annexure P-3, has been styled or titled as
"Certificate of Grant". In order to know the real nature of the document, one has to look
into the recitals of the document and not the title of the document. The intention is to be
gathered from the recitals in the deed, the conduct of the parties and the evidence on
record. It is settled law that the question of construction of a document is to be decided by
finding out the intention of the executant, firstly, from a comprehensive reading of the
terms of the document itself, and then, by looking into to the extent permissible - the
prevailing circumstances which persuaded the author of the document to execute it. With
a view to ascertain the nature of a transaction, the document has to be read as a whole. A
sentence or term used may not be determinative of the real nature of transaction.
Reference in this regard can be made to the following cases i.e. Vidhyadhar v. Manikrao
and Anr., (1999) 3 SCC 573; Subbegowda (Dead) by LR. v. Thimmegowda (Dead) by
LRs., (2004) 9 SCC 734 and Bishwanath Prasad Singh v. Rajendra Prasad and Anr.,
(2006) 4 SCC 432. 1999 AIR SCW 1129
2004 AIR SCW 2888
2006 AIR SCW 4235
13. The above principles make it clear that we have to see terms and conditions and
recitals in the document and not the title alone. Though the document, according to the
appellant, "Certificate of Grant", perusal of the clauses therein, clearly shows that the
land was sold on 4-3-1948 in a public auction and Motappa purchased the same for a
price of Rs. 408.12. In addition to the recitals, the "darkhast registered extract" produced
as Annexure "C" before the High Court also shows that the land in question was sold for
a "price". Form I also indicates that the land in question was purchased and what was
paid by the purchaser under the said document was the purchase price. In the light of the
principles mentionedabove and the terms and conditions in the recital clearly show that
the land was purchased by Motappa in a public auction for a price. Merely because the
document has been styled or titled as "Certificate of Grant", it cannot be construed that
the land was a "granted land" attracting the provisions of the Act and the Rules. The
Assistant and the Deputy Commissioner, the authorized authorities under the Act and
Rules, on verification of the contents of the document coupled with Revenue extract
rightly concluded that the land was purchased by Motappa in a public auction for a price
even in the year 1948.
14

. Now let us consider the jurisdiction of the High Court for interference in a factual
decision arrived at by the authorities. It is relevant to point out that the appellant and Mr.
Gopal filed Writ Petition No. 809 of 2000 before the High Court praying to quash the
orders dated 11-3-1999 and 20-9-1999 issued by the Assistant and Deputy
Commissioner-respondent Nos. 2 and 3 herein, by issuance of Certiorari. It is settled law
that a writ of Certiorari can only be issued in exercise of extraordinary jurisdiction which
is different from appellate jurisdiction. The writ jurisdiction extends only to cases where
orders are passed by inferior Courts or tribunals or authorities in excess of their
jurisdiction or as a result of their refusal to exercise jurisdiction vested in them or they act
illegally or improperly in the exercise of their jurisdiction causing grave mis-carriage of
justice. In regard to a finding of fact recorded by an inferior tribunal or authority, a writ
of Certiorari can be issued only if in recording such a finding, the tribunal/authority has
acted on evidence which is legally inadmissible, or has refused to admit an admissible
evidence, or if the finding is not supported by any evidence at all, because in such cases
the error amounts to an error of law. It is needless to mention that a pure error of fact,
however grave, cannot be corrected by a writ. It is useful to refer the decision of this
Court in Surya Dev Rai v. Ram Chander Rai and Others, (2003) 6 SCC 675 wherein, in
para 38, held as under : 2003 AIR SCW 3872

"........(3) Certiorari, under Article 226 of the Constitution, is issued for correcting gross
errors of jurisdiction i.e. when a subordinate Court is found to have acted (i) without
jurisdiction - by assuming jurisdiction where there exists none, or (ii) in excess of
@page-SC1442
its jurisdiction - by overstepping or crossing the limits of jurisdiction, or (iii) acting in
flagrant disregard of law or the rules of procedure or acting in violation of principles of
natural justice where there is no procedure specified, and thereby occasioning failure of
justice."
15

. It is clear that whether it is a writ of Certiorari or the exercise of supervisory


jurisdiction, none is available to correct mere errors of fact or of law unless the following
requirements are satisfied : (i) the error is manifest and apparent on the face of the
proceedings such as when it is based on clear ignorance or utter disregard of the
provisions of law, and (ii) a grave injustice or gross failure of justice has occasioned
thereby. It is also clear that the High Court in exercise of certiorari or supervisory
jurisdiction will not convert itself into a Court of appeal and indulge in reappreciation or
evaluation of evidence or correct errors in drawing inferences or correct errors of mere
formal or technical character. As observed in Surya Dev Rai v. Ram Chander Rai (supra),
the exercise of jurisdiction under Article 226 or 227 of the Constitution cannot be tied
down in a straight jacket formula or rigid rules. To put it clear though the power is there
but the exercise is discretionary which will be governed solely by the dictates of judicial
conscience enriched by judicial experience and practical wisdom of the Judge. 2003
AIR SCW 3872

16. In the light of the above principles, let us consider whether the High Court committed
any error in dismissing the writ petition. We have already referred to the recitals in the
document produced before the High Court which though titled as certificate of
grant/Saguvali chit, various terms and conditions make it clear that the land was
purchased by Motappa in a public auction on payment of a price for Rs. 408.12. In
addition, the two authorities as well as the High Court adverted to the revenue extract and
concluded that it was not a "granted land" and it was purchased in a public auction on
payment of a price. In the light of the factual conclusion, we are satisfied that the High
Court has rightly refused to quash the orders of the said authorities and dismissed the writ
petition. If the factual finding that the subject-matter of the land was a "granted land"
undoubtedly it attracts bar under Section 4 of the Act and follow the conditions as stated
in Rule 43(1)(5) and (8).
17

. In the light of the above conclusion, though Mr. Altaf Ahmed, learned senior counsel
heavily relied on the decisions of this Court, namely, Manchegowda and Others v. State
of Karnataka and Others, (1984) 3 SCC 301; Siddegowda v. Assistant Commissioner and
Others, (2003) 10 SCC 675 : Guntaiah and Others v. Hambamma and Others, (2008) 6
SCC 228 and T. Mohd. Haneef v. Dy. Commissioner and Others, (2004) 13 SCC 90, in
our opinion, it would be of no help to the stand taken by him. Hence, we refrain from
elaborating the ratio laid down therein. AIR 1984 SC 1151
2003 AIR SCW 1693
2005 AIR SCW 3657

18. In the result, there is no merit in the appeal and the same is dismissed. However, in
the facts and circumstances of the case, there is no order as to costs.
Appeal dismissed.
AIR 2008 SUPREME COURT 1442 "Progressive Education Society v. Rajendra"
(From : Bombay)*
Coram : 2 A. K. MATHUR AND ALTAMAS KABIR, JJ.
Civil Appeal No. 1318 of 2008 (arising out of SLP (C) No. 7006 of 2007). D/- 15 -2
-2008.
Progressive Education Society and Anr v. Rajendra and Anr
Maharashtra Employees of Private Schools (Conditions of Service) Regulation Act (3 of
1978), S.5(3) - Maharashtra Employees of Private Schools (Conditions of Service) Rules
(1981), R.14, R.15(6) - SERVICE MATTERS - TERMINATION OF SERVICE -
Probationer - Termination of service - R.14 and R.15 does not override provisions of
S.5(3) - Performance of probationer have to be taken into consideration before
terminating his services - Documents produced by School Management, not above
suspicion - Requirements of R.15(6) and R.14 thus not complied with - Order terminating
services of probationer in exercise of powers u/S.5(3) - Liable to be set aside.
If the provisions under S. 5(3) and R. 15(6) are read together, it would mean that before
taking recourse to the powers vested under sub-section (3) of S. 5, the performance of
@page-SC1443
an employee appointed on probation would have to be taken into consideration by the
School Management before terminating his services. Accordingly, while Rules 14 and 15
of the Rules, 1981 cannot override the provisions of sub-sec. (3) of S. 5, it has to be said
that the requirements of sub-rule (6) of R. 15 would be a factor which the School
Management has to take into consideration while exercising the powers which it
undoubtedly has and is recognised under sub-sec. (3) of S. 5. (Paras 14, 15)
In the instant case the school Management relied upon the confidential report of the
Probationer while purporting to pass the order terminating his services on 1st August,
1994. The said Confidential Report produced on behalf of the School Management does
not inspire confidence on account of the different dates which appear both on Part I and
Part II of the said Report. Part I of the Self Assessment Form gives the particulars of the
concerned teacher and the remarks of the Reporting authority, namely, the Head Mistress
of the School. The date in the said Part is shown as 4th July, 1994, whereas the date at the
end of Part II, which is the form of the Confidential Report giving details of the teacher's
performance, is dated 24th June, 1994, which appears to be in line with the date given of
the forwarding letter written by the Head Mistress to the Secretary of the Society. In
addition to the confusion created by the different dates on the form, there is a third date
which appears on Part I of the Self Assessment Form which shows that the documents
were presumably forwarded to the Management of the School on 6th August, 1994,
which is a date which is prior to the date of termination of the services of the respondent
probationer namely, 1st August, 1994. This merely goes to show that the said documents
are not above suspicion and that the requirements of R. 15(6) and R. 14 had not been
complied with prior to invocation by the School Management of the powers under sub-
sec. (3) of S. 5 of the Act. Thus the order terminating services of the probationer, liable to
be set aside. (Paras 17, 18)

Shivaji M. Jadhav, for Appellants; S.S. Shinde, Ms. Asha G. Nair, P.C. Madkholkar,
Manish Pitale, Deepak Gupta, Chander Shekhar Ashri, for Respondents.
* W.P. No. 939 of 1997, D/- 15-12-2006 (Bom) (at Nagpur).
Judgement
1. ALTAMAS KABIR, J. :-Leave granted.
2. The Appellant No. 1 herein is a Society which runs a school wherein the Respondent
No. 1 herein was appointed on probation on 4th August, 1992. Pursuant to the order of
appointment, the Respondent No. 1 joined his duties in the school on 8th August, 1992
and his appointment was approved by the Respondent No.2, The Education Officer
(Secondary) Zilla Parishad, District Wardha, Bombay, on probation for a period of two
years from 8th August, 1992. Ordinarily, the period of probation would have come to an
end on 7th August, 1994, but before completion of the said period, the service of the
Respondent No. 1 was terminated by the Management of the Appellant-Society with
effect from 31st July, 1994, although the order of termination was dated 1st August, 1994,
on the ground that his work was found to be unsatisfactory during the period of
probation. While terminating his services, the Appellant-Society also paid a sum of
Rs.3076/- to the Respondent No.1 as notice pay.
3. The Respondent No. 1 challenged the order of termination of his service before the
School Tribunal under Section 9 of The Maharashtra Employees of Private Schools
(Conditions of Service) Regulation Act, 1977 (hereinafter referred to as the "MEPS,
Act"). The basic ground of challenge taken by the Respondent No. 1 was that there was
nothing wrong with his performance or conduct and that the results in Mathematics,
which was his subject, was cent per cent. The Respondent No. 1 also contended that his
termination was in contravention of Section 5(3) of the MEPS Act and the Management
did not have any material before it to justify the termination order.
4. The aforesaid appeal preferred by the Respondent No. 1 was strongly opposed by the
Management and it was reiterated that the services of the Respondent No. 1 had to be
terminated on account of the fact that his performance was not satisfactory.
5. The School Tribunal, however, found in favour of the Respondent No. 1 mainly on two
grounds. It came to a finding that as required under Rules 14 and 15 of The Maharashtra
Employees of Private Schools (Conditions of Service) Rules, 1981 (hereinafter referred
to as the "MEPS Rules, 1981"), no assessment of the work of Respondent No. 1 had been
done by the Appellant-Society and that what had been produced on
@page-SC1444
behalf of the Management had been prepared later on. The Tribunal also found that the
Society had not taken any resolution to terminate the services of the Respondent No. 1
and that the document on which the Management is said to have relied, reached the
Management only on 6th August, 1994 when the services of the Respondent No. 1 had
already been terminated. On account of the above, and in particular violation of Rule
15(6) of the MEPS Rules, 1981, the Tribunal allowed the appeal and set aside the order of
termination with a direction on the Appellant-Society to reinstate the Respondent No. 1 in
the same post from 1st April, 1997 and to pay him his arrears of salary from 1st August,
1994 till the date of re-joining his duties in the school.
6. The said order of the School Tribunal was challenged by the Society before the Nagpur
Bench of the Bombay High Court by way of Writ Petition No.939 of 1997. Affirming the
view taken by the School Tribunal, the High Court dismissed the Writ Petition against
which the instant appeal has been filed.
7. Notwithstanding the findings of the School Tribunal, the High Court also went into the
matter in some detail and had occasion to consider the effect of the provisions of Section
5(3) of the MEPS Act, Rules 14 and 15 and in particular sub-rule (6) of Rule 15 of the
MEPS Rules, 1981. The High Court found that the power to terminate the services of a
Probationer was available to the Management under sub-section (3) of Section 5, but that
sub-rule (6) of Rule 15 had also to be taken into consideration while exercising power
under sub-section (3) of Section 5 of the MEPS Act.
8. Basing its decision on the manner in which the services of Respondent No. 1 had been
terminated without a proper assessment of his work during the probation period and also
in view of the fact that the Management did not, in fact, have any occasion to consider
the documents which were alleged to have been prepared by the Head Mistress of the
School, the High Court affirmed the findings of the School Tribunal and dismissed the
Writ Petition.
9. On behalf of the Management of the Society, which is in appeal before us, it has been
urged that both the School Tribunal as well as the High Court had misconstrued the
materials which had been prepared by the Head Mistress and produced on behalf of the
School and had been relied upon by the Management of the Appellant-Society to
terminate the services of the Respondent No. 1. It was submitted that the Annual
Confidential Report along with all its Annexures had been duly shown to the Respondent
No. 1 on 7th July, 1994 which would be evident from his signature and the date against it
on the form itself. It was submitted that the Respondent No. 1 had been duly informed of
his performance and the assessment made on the basis thereof which would clearly
disprove the case of the Respondent No. 1 that no assessment had been made of his
performance during his period of probation or that he was not informed of the same
before his services were terminated. It was urged that the requirement of Rules 14 and 15,
and, in particular 15(6) of the MEPS Rules, had been strictly complied with, which
enabled the Society, which was in Management of the School, to take a decision to
terminate the services of the Respondent No. 1. It was also submitted that both the
Tribunal and the High Court had erred in holding otherwise and that if the interpretation
sought to be given both by the Tribunal and the High Court is to be accepted, it would
result in the Rules having an overriding effect over the statute itself which vested the
authority with powers to terminate the services of a Probationer if in its opinion the
performance of the Probationer during the period of probation was found to be
unsatisfactory.
10. Opposing the submissions made on behalf of the Appellant-Society, the learned
counsel for both the Respondent No. 1 and the Respondent No.2 contended that no
interference was called for with the judgment both of the School Tribunal as also the
High Court on account of the suspicious nature of the documents which had been
produced before the Tribunal and the High Court on behalf of the School Management
and in particular the Annual Confidential Report for teaching staff, which under sub-rule
(6) of Rule 15, the Management was under an obligation to maintain. It was reiterated by
learned counsel appearing for the Respondent No. 1 that the said Report itself, as has
been discussed both by the School Tribunal as well as the High Court, would go to show
that the same had been prepared only for the purposes of documentation and that the
same had not been considered by the Management when the order of termination
@page-SC1445
of the services of the Respondent No. 1 was passed. It was pointed out that the letter
addressed by the Head Mistress of the School to the Secretary of the Progressive
Education Society, the Appellant herein, enclosing a copy of the Confidential Report, is
dated 24th June, 1994, whereas the Report itself is dated 4th July, 1994, which, in no
uncertain terms, established that the forwarding letter of the Head Mistress alleged to
have been sent on 24th June, 1994 was an afterthought or had been prepared when the
Report itself was not ready. In addition to the above, it was also pointed out that at the
end of the Assessment Form the signature of the Reviewing Authority did not indicate
any date on which it had been signed, once again giving rise to the suspicion that the
document had been prepared only for the purposes of the record but not for the purpose
indicated in sub-rule (6) of Rule 15 of MEPS Rules, 1981 read with subsection (3) of
Section 5 of the MEPS Act.
11. It was lastly pointed out that the first page of the Confidential Report bears a date on
the right top hand corner, namely, 6th August, 1994 which has been tried to be explained
as being the date of official dispatch of the records of the School Management. It has
been submitted that the said date could also indicate that the said document was not
before the Management of the School when the order of termination had been passed
earlier on 1st August, 1994.
12. Similar submissions were advanced on behalf of Respondent No.2 and it was urged
that neither the order of the School Tribunal nor the order of the High Court warranted
any interference.
13. On a consideration of the submissions made on behalf of the respective parties, the
main issue which, in our view, requires determination in this appeal is whether the
provisions of Rules 14 and 15, and, in particular sub-rule (6) of Rule 15 of the MEPS
Rules, 1981, would control the powers vested in the Management of the School under
sub-section (3) of Section 5 of the MEPS Act. The law with regard to termination of the
services of a Probationer is well established and it has been repeatedly held that such a
power lies with the Appointing Authority which is at liberty to terminate the services of a
Probationer if it finds the performance of the Probationer to be unsatisfactory during the
period of probation. The assessment has to be made by the Appointing Authority itself
and the satisfaction is that of the Appointing Authority as well. Unless a stigma is
attached to the termination or the Probationer is called upon to show cause for any
shortcoming which may subsequently be the cause for termination of the Probationer's
service, the Management or the Appointing Authority is not required to give any
explanation or reason for terminating the services except informing him that his services
have been found to be unsatisfactory.
14. The facts of this case are a little different from the normal cases relating to probation
and the termination of the services of a Probationer in that the satisfaction required to be
arrived at under sub-section (3) of Section 5 of the MEPS Act has to be read along with
Rule 15 of the MEPS Rules, 1981 with particular reference to sub-rule (6) which
provides that the performance of an employee appointed on probation is to be objectively
assessed by the Head during the period of his probation and a record of such assessment
is to be maintained. If the two provisions are read together, it would mean that before
taking recourse to the powers vested under sub-section (3) of Section 5 of the MEPS Act,
the performance of an employee appointed on probation would have to be taken into
consideration by the School Management before terminating his services.
15. Accordingly, while Rules 14 and 15 of the MEPS Rules, 1981 cannot override the
provisions of sub-section (3) of Section 5 of the MEPS Act, it has to be said that the
requirements of sub-rule (6) of Rule 15 would be a factor which the School Management
has to take into consideration while exercising the powers which it undoubtedly has and
is recognised under sub-section (3) of Section 5 of the Act.
16. This brings us to the next question regarding the sufficiency of the materials before
the School Management while purporting to pass the order of termination on 1st August,
1994. As has been discussed, both by the School Tribunal and the High Court, the
Confidential Report which has been produced on behalf of the School Management does
not inspire confidence on account of the different dates which appear both on Part-I and
Part-II of the said Report. Part-I of the Self-Assessment Form gives the particulars of the
concerned teacher and the remarks of the Reporting
@page-SC1446
Authority, namely, the Head Mistress of the School. The date in the said Part is shown as
4th July, 1994, whereas the date at the end of Part-II, which is the form of the
Confidential Report giving details of the teacher's performance, is dated 24th June, 1994,
which appears to be in line with the date given of the forwarding letter written by the
Head Mistress to the Secretary of the Society. To add to the confusion created by the
different dates on the form, there is a third date which appears on Part-I of the Self-
Assessment Form which shows that the documents were presumably forwarded to the
Management of the School on 6th August, 1994, which is a date which is prior to the date
of termination of the services of the Respondent No. 1, namely, 1st August, 1994.
17. This merely goes to show that the said documents are not above suspicion and that the
requirements of Rule 15(6) and Rule 14 had not been complied with prior to invocation
by the School Management of the powers under sub-section (3) of Section 5 of the MEPS
Act.
18. In such circumstances, we are inclined to agree with the views expressed by the
School Tribunal as well as the High Court and we see no grounds to interfere with the
order impugned in this appeal.
19. The appeal, therefore, stands rejected. There will be no order as to costs.
Appeal dismissed.
AIR 2008 SUPREME COURT 1446 "State of Jharkhand v. Bijay Kumar"
(From : Jharkhand)*
Coram : 2 S. B. SINHA AND V. S. SIRPURKAR, JJ.
Civil Appeal No. 1270 of 2008 (arising out of SLP (C) No. 16466 of 2006), D/- 14 -2
-2008.
State of Jharkhand and Ors. v. Bijay Kumar and Ors.
(A) Constitution of India, Art.142, Art.226 - SUPREME COURT - WRITS - Power to do
justice - Order passed by S.C. granting special benefits to parties before it - Benefits
specifically confined by order only to parties of that case - Similar benefit cannot be
extended by High Court to others by relying upon order of S.C. - Power as under Art.152
is not available to High Court.
W. P. (S) No. 1599 of 2005, D/-30-06-2006 (Jhar), Reversed. (Para 11)
(B) Constitution of India, Art.16 - EQUALITY IN PUBLIC EMPLOYMENT -
APPOINTMENT - Equality in appointment - Has to be protected - Weightage in services
over new applicants granted to some ad hoc appointees whose service were terminated -
Cannot be extended to others.(Para 14)
Cases Referred : Chronological Paras
(2006) W. P. (S) No. 156 of 2005, D/- 23-6-2006 (reported in 2007 (1) JLJR 203) (Jhar)
5
2006 AIR SCW 5632 (Ref.) 12
AIR 1975 SC 538 : 1975 Lab IC 363 (Disting.) 7
AIR 1966 SC 1942 (Ref.) 7
B.B. Singh, Kumar Rajesh and Rajesh Kr. Pathak, for Appellants; P.P. Rao, Sr. Advocate,
Guntur Prabhakar, for Respondents.
* W.P. (S) No. 1599 of 2005, D/- 30-6-2006 (Jhar).
Judgement
S. B. SINHA, J. :- Leave granted.
1. Respondents herein were appointed on ad hoc basis in Frozen Semen Bank Project as
Technical Assistants in the Department of Animal Husbandry of the Government of
Bihar. The said appointments were made by the Regional Director, Animal Husbandry,
Ranchi.

2. In the year 1996, a scam known as 'Fodder Scam' came to light and all unauthorized
appointments were declared illegal by the State, inter alia, on the premise :
"1. The Regional Director, Animal Husbandry, Ranchi was not competent to make any
such appointment.
2. No advertisement was issued before making such appointment.
3. No reservation policy was followed.
4. The number of persons appointed, were much more than the sanctioned post and
5. Besides these many other rules and norms of appointment were also flouted.
Pursuant thereto or in furtherance thereof, the services of the respondents were
terminated in 1997. Services of some other employees were also terminated on 23-10-
1998. They filed writ petitions before the then Ranchi Bench of the Patna High Court,
which were dismissed.
Letters Patent Appeals preferred there against were also dismissed observing that in the
event their cases are considered for appointment, relaxation in respect of age
@page-SC1447
and weightage in the future appointment should be given.
3. One Umakant Sinha and others approached this Court. By a judgment and order dated
23-7-2003, this Court while opining that ad hoc appointees have no right to claim
regularization in the service, directed :
"In this view of the matter, we direct the State of Jharkhand to consider at the earliest for
recruiting Technical Assistant for the Semen Bank Project and to fill up the existing
vacancies within a period of three months from today. For that purpose the Respondent-
State is directed to constitute a Selection Committee as per the existing Rules within a
period of three months from today.
(ii) The appellants whose services are terminated may apply to the Secretary of the
Animal Husbandry Department within a period of one month for being re-appointed or
for regularization of their services. The Committee shall consider the eligibility,
suitability, past record as well as the educational qualifications of the appellants as per the
Rules as on today.
(iii) The Committee shall give relaxation of age and weightage over outsiders as directed
by the High Court. However, if the appellants are found unsuitable for some reasons, it
would be open to the Committee to reject their applications.
Appropriate authority shall issue orders for appointment after considering the roster and
the merit list, on available vacancies.
The appeals stand disposed of accordingly. There shall be no order as to costs. We,
however, make it clear that this case shall not be treated as precedent as we have decided
it purely on the facts and in the peculiar circumstances of this case."
(Emphasis supplied)
4. An interlocutory application was filed for extension of time to comply with the said
order and by an order dated 2-12-2003, three months' time was granted for constitution of
the Committee. It was clarified that directions by this Court were in relation to the
appellants in the said appeals.
Another writ petition, however, was filed in the High Court of Jharkhand which came
into being in November 2000 by Bijay Kumar and others. The said writ petition was also
dismissed by an order dated 6-2-2004. Special Leave Petitions were also filed there-
against and by a judgment and order dated 10-4-2006, a Bench of this Court directed :
"We are of the view that the issue should be resolved finally and should not be kept
pending. The earlier order of this Court was passed on 23-7-2003. The right to be selected
thereunder cannot continue indefinitely. Therefore, we direct : (1) that the Respondent-
Authority shall advertise in the local newspapers having wide circulation of the holding
of selections for the purpose of filling of those vacancies which according to the Rules,
are to be filled by direct appointment. (2) Applications should be asked for from these ad
hoc employees who were appointed in the Frozen Semen Bank Project between 1988 to
1992 and whose services were terminated in 1998. No such employee who has already
been considered by the Selection Committee under the order dated 23-7-2003 shall apply.
(3) No appointment already made either in the normal course or pursuant to the earlier
order of this Court shall be disturbed. (4)The advertisement shall specify the last date
with which the candidate concerned shall apply for consideration including thereon proof
that the candidate had served with the Respondents. (5) The selection shall be made by
the Selection Committee only against the available vacancies. Preference shall be given
to those who have longer tenure of service over those who have shorter tenure of service,
other things being equal. (6) All other conditions in the order dated 23-7-2003 shall be
operative as far as the procedure to be followed by the Selection Committee to be set up
pursuant to this order is concerned. (7) This shall be a one time exercise. No further
application shall be entertained by such ad hoc employees. This fact should also be
mentioned in the advertisement."
(Emphasis supplied)
5. An advertisement by the appellant was issued. Questioning the legality of the said
advertisement, a writ petition was filed before the Jharkhand High Court which was
marked as Writ Petition (S) No. 156 of 2005. A direction was issued therein that the
process of selection should be completed within four months.
Respondents, however, filed another writ petition before the High Court which by reason
of the impugned judgment dated 30-6-2006 was disposed of, directing :
"In view of the judgment rendered by the
@page-SC1448
Division Bench on 23rd June, 2006 in W.P.(S) No. 156 of 2005 (Chandra Kant and Ors.
vs. State of Jharkhand and Ors.) Supreme Court decision dated 10th April, 2006 and
earlier decision dated 23rd July, 2003 passed in Civil Appeal Nos. 5342-5343 of 2003 and
analogous cases, this application is disposed of with liberty to the petitioners to apply
(those who have not applied in pursuance of the advertisement to the post of Technical
Assistant), if any advertisement is issued in future."
6. Mr. B. B. Singh, learned counsel appearing on behalf of the appellant, would submit
that the aforementioned direction of the High Court, if implemented, would run contrary
to and inconsistent with the directions contained in this Court's orders dated 23-7-2003,
2-12-2003 and 10-4-2006. It was urged that this Court, in the aforementioned orders,
having directed not to treat the same as precedent as also one time exercise, the directions
issued by the High Court are clearly violative thereof.
7

. Mr. P.P. Rao, learned senior counsel appearing on behalf of the respondents, on the other
hand, urged that one time exercise directed to be carried out by this Court had not been
fully complied with and, thus, the High Court was entitled to pass the impugned order. It
was submitted that this Court, in any event, in exercise of its jurisdiction under Article
142 of the Constitution of India, can issue necessary directions so as to extend the
benefits also to those who had not approached this Court earlier. Strong reliance in this
behalf has been placed on Amrit Lal Berry v. Collector of Central Excise, New Delhi and
Ors. [(1975) 3 SCR 960] and B. N. Nagarajan and Ors. v. State of Mysore and Ors.
[(1966) 3 SCR 682]. It was contended that advertisement having been issued in July 2006
and the impugned order having been passed by the High Court in June 2006 and
furthermore in view of the fact that the respondents have appeared in the examination,
this Court should not exercise its discretionary jurisdiction under Article 136 of the
Constitution of India. AIR 1975 SC 538
AIR 1966 SC 1942

8. Respondents herein were appointed in 1992. They were removed from service in 1997.
They did not question the said orders of termination. We have noticed hereinbefore that
Abhay Kumar, Uma Kant Sinha and others had questioned the orders of their termination
from service before the Patna High Court.
9. This Court in its order dated 23-7-2003, although opined that they had no right to
continue in service, issued some directions evidently in exercise of its power under
Article 142 of the Constitution of India.
The said order was to be kept confined only to the appellants thereof, was reiterated by
this Court in its order dated 2-12-2003.
It may be true that while Abhay Kumar and others approached this Court aggrieved by
and dissatisfied with the judgment and order dated 16-2-2004 of the learned Single Judge
of the High Court, this Court, in its order dated 10-4-2006 intended to resolve the issue
finally. It was in that situation opined that the right to be selected cannot continue
indefinitely. This Court dealt with the grievances of those employees whose services have
been terminated in the year 1998.
10. The core question, therefore, which arises for our consideration is as to whether the
High Court had any jurisdiction to issue a similar direction relying on or on the basis of
the orders of this Court which had been passed in terms of Article 142 of the Constitution
and confined only to the appellants of those cases.
Indisputably, Respondents were not parties to the said orders. No benefit thereof, thus,
could be extended in their favour. The High Court, therefore, in our opinion, could not
have issued the aforementioned directions.
11. Constitution of India conferred a special jurisdiction in this Court only. Although
power of judicial review has been conferred on the High Courts, it had not been given
any special jurisdiction as has been done on the Supreme Court in terms of Article 142 of
the Constitution of India. It is, therefore, very difficult to comprehend that the High Court
could issue the impugned direction which, in effect and substance, would be violative
thereof.

Reliance placed by Mr. Rao on Amrit Lal Berry (supra) is not apposite. When a law is
declared by this Court, it becomes the law of land in terms of Article 141 of the
Constitution of India. Indisputably, therefore, such law would enure to the benefit of the
persons AIR 1975 SC 538

@page-SC1449
similarly situated but it is incomprehensible that when this Court grants some special
benefits to the parties who are before it and confined the relief only to them, such a
benefit can be extended by the High Court relying on or on the basis thereof. The matter
might have been different if this Court had not issued such directions, namely, the said
order should be kept confined to the parties before it. When this Court declared that no
further application should be entertained at the instance of other ad hoc employees in the
name of parity or otherwise, the High Court could not have issued a direction which
would run counter to the spirit of the said order.
12. Submission of Mr. Rao that notwithstanding the orders dated 23-7-2003 and 10-4-
2006, the High Court have committed no mistake in issuing the impugned directions, is
stated to be rejected. This Court intended to pass a final order. The said order should have
been allowed to attain finality. If a third order can be issued, others who are waiting on
the fence may be held to be entitled to file a fourth application. The process, thus, would
go on unendingly. Relaxation given for a limited purpose would become a rule and not
the exception.

In Kendriya Vidyalaya Sangathan and Ors. v. Sajal Kumar Roy and Ors. [(2006) 8 SCC
671], dealing with the question of age relaxation, it was held : 2006 AIR SCW 5632,
Para 11

"The appointing authorities are required to apply their mind while exercising their
discretionary jurisdiction to relax the age-limits. Discretion of the authorities is required
to be exercised only for deserving candidates and upon recommendations of the
Appointing Committee/Selection Committee. The requirements to comply with the rules,
it is trite, were required to be complied with fairly and reasonably. They were bound by
the rules. The discretionary jurisdiction could be exercised for relaxation of age provided
for in the rules and within the four corners thereof. As the respondents do not come
within the purview of the exception contained in Article 45 of the Education Code, in our
opinion, the Tribunal and consequently, the High Court committed a manifest error in
issuing the aforementioned directions."
13. There is a sea change in the situation in the field of public employment. Ten years
have passed. A new State has come into being. Thousands of persons have acquired
similar or higher qualifications. They have got their names registered in the Employment
Exchanges. The job opportunities in a specialized field being limited, those who are now
entitled to be considered, may not be considered at all, if the order of the High Court is
allowed to operate.
14. Constitutional guarantee of equality as envisaged under Articles 14 and 16 of the
Constitution of India must be protected. While passing one order or the other, we should
not forget the interest of those who are not before us. Citizens have human right of
development and offer of appointment on such posts should be directed to be made only
on merit. Only because some persons had approached this Court and obtained an order,
the same should not be extended to others to which they are not otherwise entitled to,
namely, weightage in service over the new applicants.
15. For the reasons aforementioned, the impugned judgment cannot be sustained. It is set
aside accordingly. The appeal is allowed with no order as to costs.
Appeal allowed.
AIR 2008 SUPREME COURT 1449 "Bharat Heavy Electricals Ltd. v. ESI Corporation"
(From : 2006 (4) Mad LJ 1447)
Coram : 2 S. B. SINHA AND V. S. SIRPURKAR, JJ.
Civil Appeal No. 1271 of 2008 (arising out of SLP (C) No. 2082 of 2007), D/- 14 -2
-2008.
Bharat Heavy Electricals Ltd. v. ESI Corporation.
(A) Employees' State Insurance Act (34 of 1948), S.45A - EMPLOYEES STATE
INSURANCE - CONTRACT - Determination of contribution - Proceedings for - Act
recognises existence of immediate employer - Opportunity of hearing is to be given to
both principal employer and immediate employer - Determination of exact liability of
contractor is necessary when contract came to end - Order of determining authority
without giving opportunity of hearing to principal employer in regard to names and
particulars of contractors - Illegal.
2006 (4) Mad LJ 1447, Reversed.
@page-SC1450

The proviso appended to S. 45A of the E.S.I. Act provides for a statutory mandate of
giving a reasonable opportunity of being heard. The quantum of amount due has to be
determined in respect of all contract workers engaged by the contractors. The principal
employer would be entitled to recover the contributions from the contractor; they being
the immediate employers. Whereas under the Employees Provident Fund Act, the
principal employer is statutorily liable in terms of the provisions of the Act to comply
with the provisions therein; in terms of the Act, the principal employer is entitled to
recover the amount of contribution payable by the immediate employer for them. S. 45A
of the E.S.I. Act enables the appropriate authority to recover such dues both from the
principal as also the immediate employer. It provides for an opportunity of hearing to
both of them. (Paras 12, 13)
Sub-section (3) of S. 44 enjoins upon the principal as also the immediate employers to
maintain registers or records as may be required by regulations. S. 45 also empowers the
Inspector of Corporation to require an immediate or principal employer to furnish to him
such information as he may consider necessary in regard to the compliance of the
provisions of the Act by them. The Act, therefore, recognizes the existence of an
immediate employer. (Para 13)
The principal employers have a statutory right to recover the dues from the
contractors/immediate employers. Thus, determination of the exact liability on the part of
the contractors is necessary in the instant case, keeping in view the fact that they or some
of them may not be under the control of the principal employer having regard to the fact
that the contract has come to an end. Therefore, the order passed by the determining
authority without giving an opportunity of hearing to the petitioner-principal employer in
regard to the names and other particulars of the contractors, would be illegal.
2006 (4) Mad LJ 1447, Reversed. (Para 17)
(B) Employees' State Insurance Act (34 of 1948), S.45A - Employees' Provident Funds
and Miscellaneous Provisions Act (19 of 1952), S.7A - EMPLOYEES STATE
INSURANCE - PROVIDENT FUND - OBJECT OF AN ACT - NATURAL JUSTICE -
Scope - Contribution - Determination of liability of employer - Purport and object of both
Statutes is same - Both Acts envisage compliance of natural justice. (Para 12)
Cases Referred : Chronological Paras
2000 (2) Lab LJ 593 (Mad) 7, 16
(1998) 9 SCC 74 (Ref.) 16
1993 AIR SCW 3492 : AIR 1993 SC 2655 : 1994 Lab IC 24 (Ref.) 5, 14
1990 (2) Lab LN 777 (Mad) 5, 6, 16
1989 (4) JT (SC) 380 : (1990) 1 SCC 68 (Ref.) 12, 14, 16
Milon K. Banerji, A.G., Gaurab Banerji, Sr. Adv., Saurav Agrawal and Ms. Ruby Singh
Ahuja with them, for Appellant; V.J. Francis, Anupam Mishra and Jenis, for Respondent.
Judgement
S. B. SINHA, J. :- Leave granted.
Background facts :
2. Appellant herein is a Public Sector Undertaking. It used to engage contractors for
various purposes. It received a notice on or about 3-9-1992 purported to have been issued
under Section 45A of the Employees' State Insurance Act, 1948 (for short "the Act") on
the premise that they had not deposited the Employees' State Insurance contribution for
the period 19-7-1981 to 30-9-1991.
3. In its show cause, in response to the notice issued by the respondents, the appellant
stated that the workmen concerned had been engaged by the contractors who would be in
possession of the relevant records to show as to whether or not any contribution was
payable or whether the Act was applicable in respect of the concerned workmen. A list of
contractors along with their addresses who were involved during the period in question
was annexed to the said show-cause. A prayer was made before the Authority to implead
the said contractors as parties in the said proceedings under Section 45A of the Act as
immediate employers.
4. By a letter dated 8-3-1993 the said prayer was rejected by the competent authority of
the respondent, stating :
"With reference to the above, I have to invite your kind attention on the above subject and
inform you that engaging the contractors
@page-SC1451
for BHEL works is an internal affair of the factory and our Corporation is not preventing
you in any manner in bringing along with you those contractors to explain the nature of
expenditure incurred by you through the contractors. You are not denied any opportunity
to represent your case properly. You may recover the ESI contribution along with
employers share from your contractors (i.e. immediate employer) under Sections 40 and
41 of the ESI Act. As per Section 41(1) of the ESI Act, the principal employer can
recover the contributions from the immediate employer even as deduction from any
amount payable by them under any contract or even as a debt payable by the contractors.
So, it is not necessary for the Employees State Insurance Corporation to implead the
contractors to enable you (principal employer) to invoke your right of recovery. I am
therefore, to intimate you that your request as communicated in the affidavit cannot be
acceded to."
Proceedings :
5. A Writ Petition was preferred there-against before the Madras High Court wherein a
decision of the said court in Madras Gymkhana (represented by its Honourary Secretary),
Madras v. Employees' State Insurance Corporation (represented by its Regional Director),
Madras [1990 (2) Labour Law Notes 777] was relied upon. By an order dated 11-4-2000,
a learned Single Judge, doubting the correctness of the said decision, referred the matter
to a Division Bench opining :
"In view of the judgment of the Hon'ble Supreme Court of India referred to above,
namely, AIR 1993 SC pg.2655 and the other judgment namely, 1989 (4) JT (SC) 380, I
am of the respectful opinion that the judgment of this Court reported in 1990-2 L.L.N 777
does not appear to have decided the issue correctly and, therefore, it definitely calls for a
reconsideration by a larger Bench. The Registry is, therefore, directed to place this order
of reference, my judgment containing reasons and the material papers before My Lord the
Honble Chief Justice for referring the issue involved in this case for consideration by a
larger Bench."
6. By reason of the impugned judgment, a Division Bench of the Madras High Court,
while overruling the said decision in Madras Gymkhana (supra) held :
"The scheme of the ESI Act does not envisage separate and independent determination of
contribution payable by the principal employer and the immediate employer in respect of
employees directly employed by the principal employer and the contract employees
respectively. When once the authority is satisfied that persons were employed by or
through an immediate on the premises of the factory or establishment or under the
supervision of the principal employer and if for any reason the principal employer fails to
submit, furnish or maintain the records and registers in accordance with the provisions of
Sec.44, the Corporation is within their powers to determine the contribution payable in
respect of contract employees against the principal employer without looking for the
immediate employer. As already stated, in an enquiry under Section 45-A of the ESI Act
all that is required is the authority must give a reasonable opportunity of being heard to
the employer concerned. That has been complied with by the respondent in the present
case by issuing the show cause notice dated 3-9-1991, wherein the Corporation has also
afforded a personal hearing to the petitioner. The decisions relied on by the petitioner, viz.
Food Corporation of India, Ashok Leyland Limited and Chennai Petroleum Corporation
Ltd., cited supra, are of no assistance to them."
Contentions :
7. Mr. Milon K. Banerjee, learned Attorney General for India appearing for the appellant,
submitted that the High Court committed a serious error in passing the impugned
judgment in so far as it failed to construe the provisions of the Act in their proper
perspective. Learned Attorney General has placed strong reliance upon a decision of P.
Sathasivam, J., (as His Lordship then was) in Ashok Leyland Limited v. Employees' State
Insurance Corporation [(2000) 2 LLJ 593].
8. Mr. Francis, learned counsel appearing on behalf of the respondent would, however,
support the impugned judgment.
The Act :
9. The Act was enacted to provide for certain benefits to the employees in cases of
@page-SC1452
sickness, maternity and employment injury and to make provisions for certain other
matters in relation thereto.
The term 'employee' has been given a wide definition. In terms of sub-section (9) of
Section 2 of the Act, it includes a person employed directly by the principal employer or
by or through an immediate employer.
'Immediate employer' has been defined in Section 2(13) to mean :
2(13) 'immediate employer', in relation to employees employed by or through him, means
a person who has undertaken the execution, on the premises of a factory or an
establishment to which this Act applies or under the supervision of the principal employer
or his agent, of the whole or any part of any work which is ordinarily part of the work of
the factory or establishment of the principal employer or is preliminary to the work
carried on in, or incidental to the purpose of, any such factory or establishment, and
includes a person by whom the services of an employee who has entered into a contract
of service with him are temporarily lent or let on hire to the principal employer and
includes a contractor;"
Section 2(17) defines 'principal employer' in the following terms :
"2(17) principal employer means -
(i) in a factory, the owner or occupier of the factory and includes the managing agent of
such owner or occupier, and where a person has been named as the manager of the
factory under the Factories Act, 1948 (63 of 1948), the person so named;
(ii) in any establishment under the control of any department of any Government in India,
the authority appointed by such Government in this behalf or where no authority is so
appointed, the head of the Department;
(iii) in any other establishment, any person responsible for the supervision and control of
the establishment;"
Chapter IV of the Act provides for mandatory insurance of all the employees in the
manner provided for therein. Section 39 provides for payment of contribution. Section 40
provides for the principal employer to pay contribution in the first instance, whereas an
enabling provision has been enacted for recovery of the contribution from the employee
directly if he is employed by the principal employer directly. Section 41 empowers the
principal employer to recover the amount of the contribution so paid from the immediate
employer either by deduction from any amount payable to him by the principal employer
under any contract or as a debt payable by the immediate employer. Sub-section (1A) of
Section 41 mandates that the immediate employer shall maintain a register of employees
employed by or through him as provided for in the regulations and submit the same to the
principal employer before the settlement of any amount payable under sub-section (1).
We may, however, notice that the said provision was introduced by Act No.29 of 1989
w.e.f 1-2-1991. Section 45A lays down the manner in which the contributions payable in
certain cases shall be determined, sub-section (1) whereof reads, thus :
"45A(1) Where in respect of a factory or establishment no returns, particulars, registers or
records are submitted, furnished or maintained in accordance with the provisions of
Section 44 or any Inspector or other official of the Corporation referred to in subsection
(2) of Section 45 is prevented in any manner by the principal or immediate employer or
any other person, in exercising his functions or discharging his duties under Section 45,
the Corporation may, on the basis of information available to it, by order, determine the
amount of contributions payable in respect of the employees of that factory or
establishment.
Provided that no such order shall be passed by the Corporation unless the principal or
immediate employer or the person in charge of the factory or establishment has been
given a reasonable opportunity of being heard."
Application of the Act :
10. The period in question is 19-7-1981 to 30-9-1991. No return of contribution is also
said to have been filed for the said period.
Clause (4) of the show-cause notice dated 3-9-1992 reads as under :
"And whereas it is proposed to determine and recover the amount of contribution payable
in respect of the employees of your factory establishment under Section 45A of the Act,
as under :
@page-SC1453

S.No. Nature of Dues Period Amount of contribution payable Basis for


calculation
From To
1 2 3a 3b 4 5
Contribution due on the wages paid through immediate employer (contractors)
July 1981 Sept. 1991 Rs. 3,32,45,042.95 As shown in the appendices

And whereas, it is proposed to afford M/ s. an opportunity as required under Section


45A(1)(b) to show cause against the said determination and recovery.
Please show cause within 15 days hereof as to why assessment should not be made as
proposed above. In case you have any objections you are hereby given an opportunity to
explain the same and or to file a statement giving full particulars of the contributions
actually due as per your records for the above said period within the time specified above.
In case you desire to represent your case personally you may appear before the
undersigned in person or through an authorized representative on 24-09-1992 at 10.00 am
with necessary document to explain your case."
11. Appellant herein affirmed an affidavit in support of its application in implead the third
parties/contractor, stating :
"For the period in question, third parties/ contractors are involved and only they would be
in possession of records relevant to determine whether or not contributions at all are
payable or as to whether at all the Employees' State Insurance Act, 1948 is applicable in
the first place.
This Management viz., Bharat Heavy Electricals Ltd., do not have details with regards to
the work of wages, if any, paid by the third parties/contractors. The relevant information,
materials and such like would be available only with said third parties/contractors whose
names and addresses in so far as they are available at present are enclosed as annexures to
this petition. The names and addresses of the rest of the third parties/contractors who
were involved for the period in question would be furnished as and when the same are
available."
A prayer was made to implead the contractors mentioned in the annexures to the said
affidavit as parties.
Precedent :
11A. In Food Corporation of India v. Provident Fund Commissioner and Ors. [(1990) 1
SCC 68], this Court while considering the provisions of Section 7A of the Employees'
Provident Funds and Miscellaneous Provisions Act, 1952 held :
"It will be seen from the above provisions that the Commissioner is authorised to enforce
attendance in person and also to examine any person on oath. He has the power requiring
the discovery and production of documents. This power was given to the Commissioner
to decide not abstract questions of law, but only to determine actual concrete differences
in payment of contribution and other dues by identifying the workmen. The
Commissioner should exercise all his powers to collect all evidence and collate all
material before coming to proper conclusion. That is the legal duty of the Commissioner.
It would be failure to exercise the jurisdiction particularly when a party to the
proceedings requests for summoning evidence from a particular person."
The Division Bench of the High Court distinguished the said decision holding that the
provisions of Section 7A of the Employees Provident Fund and Miscellaneous Provisions
Act, 1952 are not in pari materia with the provisions of the Act stating :
"An inquiry under sub-section (1) of Section 7-A can be initiated to decide the dispute
regarding the applicability of the Act to an establishment and to determine the amount
due from any employer under any provisions of the Act, the Scheme or the Pension
Scheme or the Insurance Scheme, as the case may be. For the purposes of such inquiry,
the authorities under the Act are vested with the same powers as are vested
@page-SC1454
in a civil court for trying a suit though such powers are restricted to certain specified
matters, viz. to enforce the attendance of any person or examining him on oath, requiring
the discovery and production of documents, receiving evidence on affidavit, issuing
commission for the examination of witnesses. A fiction is created under Section 7-A that
an inquiry thereunder is deemed to be a Judicial proceeding. The observance of principles
of natural justice is also mandated vide subsection (3) which says that no order under
sub-section (1) shall be made unless the employer concerned is given a reasonable
opportunity of representing his case. Thus, it is obvious that such specific powers are
given to the authorities concerned to decide not abstract question of law, but to determine
actual concrete differences in payment of contribution and other dues by identifying the
workmen and the authorities should exercise all their powers to collect all evidence and
collate all material before coming to proper conclusion and as such an inquiry under
Section 7-A is more or the less a trial of a suit before a civil court and Judicial in nature.
The powers so conferred on the authorities concerned are being statutory powers, a legal
duty is cast on such authorities to exercise the same when situation arises and failure to
exercise the jurisdiction, especially when a party to the proceedings requests for such
exercise, would lead to nullification of the order passed in the inquiry."
Analysis :
12. We, with respect to the learned Judges, fail to notice any significant difference in the
purport and object of both the provisions. The purport and object of both the statutes, for
all intent and purport, in our opinion, is the same. In the proceedings initiated under
Section 45A of the Act, an immediate employer or principal employer may also show that
they are not liable to deposit any contribution on behalf of the employees as the
establishment in question did not come within the purview thereof. The purpose of the
proceedings, both under the Act as also the Employees Provident Funds Act, is to
determine the amount due from any employer in respect of the employees under the
statutory schemes. Both the Acts envisage compliance of principles of natural justice. The
proviso appended to Section 45A of the Act provides for a statutory mandate of giving a
reasonable opportunity of being heard.
13. The quantum of amount due has to be determined in respect of all contract workers
engaged by the contractors. The principal employer would be entitled to recover the
contributions from the contractor; they being the immediate employers. Whereas under
the Provident Funds Act, the principal employer is statutorily liable in terms of the
provisions of the Act to comply with the provisions therein; in terms of the Act, the
principal employer is entitled to recover the amount of contribution payable by the
immediate employer for them.
Section 45A of the Act enables the appropriate authority to recover such dues both from
the principal as also the immediate employer. It provides for an opportunity of hearing to
both of them.
Apart from Section 41 (1A), Regulation 32 of the Employees' State Insurance (General)
Regulations, 1950 mandates an immediate employers to maintain registers in the
prescribed form(s). An order passed under Section 45A of the Act has a serious civil and/
or financial consequence as the amount so determined is liable to be recovered as arrears
of land revenue. Section 44 of the Act, not only mandates the principal employer, but also
the immediate employer to file its reports and maintain registers. Under subsection (2) of
Section 44, when such reports are not submitted either by the principal employer or by
the immediate employers, the Corporation may require the person in charge of the factory
or establishment to furnish such particulars as it may consider necessary for the purpose
of enabling the Corporation to decide whether the factory or establishment is a factory or
establishment to which this Act applies. Sub-section (3) of Section 44 of the Act enjoins
upon the principal as also the immediate employers to maintain registers or records as
may be required by regulations. Section 45 also empowers the Inspector of Corporation
to require an immediate or principal employer to furnish to him such information as he
may consider necessary in regard to the compliance of the provisions of the Act by them.
The Act, therefore, recognizes the existence of an immediate employer.
14. We may also notice that in terms of the provisions of the Contract Labour (Regulation
and Abolition) Act, 1970 and the Rules framed thereunder, a contractor is required to
maintain a register of the workmen employed
@page-SC1455
by him. The contractor is also required to issue an employment card to the said workers.
Muster rolls, wages registers and other records in respect of each worker engaged by the
contractor are also required to be maintained.

Reliance has been placed by the Division Bench as also by Mr. Francis on Employees
State Insurance Corporation v. Harrison Malayalara Pvt. Ltd. [(1993) 4 SCC 361).
Unfortunately, therein attention of this Court was not drawn to the case of Food
Corporation of India (supra). Even otherwise, the said decision has no application to the
fact of the present case. The question therein which arose for consideration was as to
whether the employees of the contractor who were casual employees were identifiable or
not. It is in that context, this Court opined : 1993 AIR SCW 3492, Para 3

"Under the Act, the scheme is more akin to group insurance. The contribution paid
entitles the workman Insured to the benefit under the Act. However, he does not get any
part of the contribution back if during the benefit period, he does not qualify for any of
the benefits. The contribution made by him and by his employer is credited to the
insurance fund created under the Act and it becomes available for others or for himself,
during other benefit periods, if he continues in employment. What is more, there is no
relation between contribution made and the benefit availed of. The contribution is
uniform for all workmen and is a percentage of the wages earned by them. It has no
relation to the risks against which the workman stands statutorily insured. It is for this
reason that the Act envisages automatic obligation to pay the contribution once the
factory or the establishment is covered by the Act, and the obligation to pay the
contribution commences from the date of the application of the Act to such factory or
establishment. The obligation ceases only when the Act ceases to apply to the factory/
establishment. The obligation to make contribution does not depend upon whether the
particular employee or employees cease to be employee/employees after the contribution
period and the benefit period expire."
15. In that case, it was not disputed that the Act applied to casual workmen. Here,
however, the applicability of the Act itself is in question. In proceedings under Section
45A, not only the applicability of the Act but also the quantum thereof which may be held
to be payable may be the subject matter of determination.
16. Reliance has also been placed on a decision of this Court in Employees' State
Insurance Corporation v. Harrisons Malayalam Ltd. (2nd case) [(1998) 9 SCC 74,
wherein this Court referring to the first case opined that the liability of the employer to
contribute arose from the very first day of employment. There is no dispute with regard to
the aforementioned proposition of law but the dispute being both in regard to the
applicability as also the quantum, in our opinion, the respondent authority had the
requisite jurisdiction to implead the third party or summon them before it to produce all
relevant documents.
In Ashok Leyland, P. Sathasivam, J. following the Food Corporation (supra) and Madras
Gymkhana (supra) held :
"13..........The respondent is also directed to implead the contractors/sub-contractors if it
(respondent) feels that they are necessary and proper parties on the basis of the
Information furnished by the petitioner, for adjudication of the matter in controversy and
to proceed further."
Conclusion :
17. Determination of the exact liability on the part of the contractors is necessary keeping
in view the fact that they or some of them may not be under the control of the principal
employer having regard to the fact that the contract has come to an end. It will bear
repetition to state that the principal employers have a statutory right to recover the dues
from the contractors/immediate employers.
18. It appears that the determining authority did not give an opportunity of hearing to the
petitioner in regard to the names and other particulars of the contractors. The impugned
judgment, therefore, cannot be sustained. It is set aside accordingly. The appeal is
allowed and the matter is remitted to the ESI Corporation/determination authority for
considering the matter afresh. The authority shall either implead the contractors as parties
and/or summon them for producing necessary records for the said purpose. In the facts
and circumstances of the case, there shall be no order as to costs.
Appeal allowed.
@page-SC1456
AIR 2008 SUPREME COURT 1456 "Patinhare Purayil Nabeesumma v. Miniyatan
Zacharias"
(From : Kerala)
Coram : 2 S. B. SINHA AND V. S. SIRPURKAR, JJ.
Civil Appeal No. 1231 of 2008 (arising out of SLP (C) No. 7573 of 2007), D/- 12 -2
-2008.
Patinhare Purayil Nabeesumma v. Miniyatan Zacharias and Anr.
(A) Evidence Act (1 of 1872), S.110 - EVIDENCE - POSSESSION - TENANCY -
Presumption as to possession - Appellant claiming that her husband was cultivating
tenant in respect of suit property - Tax receipts filed by her also showing that entire suit
land was subject-matter of grant of tenancy by landlord and tax has been paid by her at
least from 1955 onwards - Thus, presumption can be raised in regard her possession both
backward and forward. (Para 24)
(B) Kerala Land Reforms Act (1 of 1964), S.72K, S.72B, S.13 - LAND REFORMS -
TRIBUNALS - Certificate of assignment - Cannot be granted by Land Tribunal to both
parties - Earlier certificate granted in favour of appellant - Not set aside on ground of
fraud or illegality - Was conclusive against Tribunal - Land Tribunal, therefore, had no
jurisdiction to issue second certificate.
A certificate issued under S. 72-K of the Act is conclusive. Once the same is found to be
conclusive, the same cannot be refused to be taken into consideration for any purpose
whatsoever. In view of the statutory scheme, both the parties could not have been given
the certificates of assignment. The certificate in favour of the appellant, even otherwise,
having been granted earlier and the same having not been set aside on the ground of fraud
or illegality, it was conclusive even as against the Land Tribunal. The Land Tribunal,
therefore, had no jurisdiction to issue a second certificate. (Paras 32, 27)
(C) Kerala Land Reforms Act (1 of 1964), S.72B, S.13 - LAND REFORMS -
TRIBUNALS - Certificate of assignment - Grant of, by Land Tribunal - Tribunal could
not have, on basis of stray statement made by co-villager, arrived at finding that
defendant had been in possession - Said purported finding on basis of statement made
before Revenue Inspector could not have been acted upon. (Paras 28, 29)
(D) Kerala Land Reforms Act (1 of 1964), S.72B, S.13 - LAND REFORMS -
POSSESSION - TRIBUNALS - HIGH COURT - Assignment of land - Suit for
possession - Claim on basis of certificate issued by Land Tribunal - High Court
proceeded only on basis of findings of Tribunal - Failed to notice that said findings were
overturned by Civil Court - Findings of High Court liable to be set aside.
S. A. No. 643 of 1994, D/-25-01-2007 (Ker), Reversed. (Para 33)
Cases Referred : Chronological Paras
2007 AIR SCW 5741 : AIR 2007 SC 3162 (Ref.) 27
2006 AIR SCW 4168 : AIR 2006 SC 3028 (Ref.) 27
T.V. George, R. Marar and Vikram S. Mawari, for Appellant; H.V. Hameed, Ranjit K.C.,
Alex Thomas and K. Rajeev, for Respondents.
Judgement
S. B. SINHA, J. :- Leave granted.
2. Plaintiff in a suit for declaration of title and possession is before us, aggrieved by and
dissatisfied with the judgment and decree dated 25th January, 2007 passed by a learned
Single Judge of the Kerala High Court in Second Appeal No. 643 of 1994, whereby and
whereunder the judgment and order of the first appellate court dated 13th January, 1994
as also that of the trial court dated 27th February, 1993 respectively, decreeing the suit of
the appellant, were set aside.
3. In this appeal we are concerned only with Item No. 2 of the properties described in
Schedule 'B' of the plaint, which reads as under :-
Schedule B

Item No. Revised Survey No. Measurement


1. 90/7 21 cents B
2. 90/3 26 cents A
3. 90/8 13.5 cents F

4. Husband of the appellant was an agriculturist. He was in cultivating possession of five


items of properties as a tenant under one Jenmi Palkodan Kunhmina Ayissa. Appellant
contended that her husband had been in possession of the suit land from the date of
settlement which took place in the year 1943; and in support thereof, Revenue Tax
Receipts since 1955 were filed.
@page-SC1457
5. The Legislature of the State of Kerala enacted Kerala Land Reforms Act, 1963 (the Act
for short) with a view to enact a comprehensive legislation relating to land reforms in the
State.
6. Section 13 of the Act provides for rights of tenants for fixity of tenure in respect of the
land holding. Section 27 of the Act provides for determination of a fair rent which a
tenant is liable to pay to the landlord. Section 53 confers upon a cultivating tenant who is
entitled to fixity of tenure, a right to purchase the right, title, interest and ownership of the
land by moving an application before the Land Tribunal constituted under the Act upon
payment of purchase price as may be determined in the manner laid down under Section
55 thereof. Section 72 of the Act, which was inserted in the year 1969, provides for
vesting of the landlords' rights in the State as regards holdings held by cultivating tenants
entitled to fixity of tenure and in respect of which certificate of purchase has not been
issued under subsection (2) of Section 59.
7. Whereas Section 72 provides for cultivating tenant's right to assignment of the land
which he had been holding, Section 72-F provides that where an application under
Section 72-B is moved before the Land Tribunal, it upon issuing a notice to the landlord
and other intermediaries, if any, may determine the quantum of compensation and
purchase price payable therefor.
8. A certificate of purchase issued by the Land Tribunal to the cultivating tenant under
sub-clause (2) of Section 72-K is conclusive proof of assignment to the tenant of the
right, title and interest of the landowner and the intermediaries, if any, over the holding or
portion thereof to which the assignment relates.
9. Indisputably the husband of the appellant applied for and has been granted a certificate
of purchase by the Land Tribunal in the year 1976.
10. Respondents also applied for and were granted such a certificate by the Land Tribunal
of the same land in 1977. Two certificates, therefore, came to be issued in respect of the
same land.
11. Inter alia on the premise that she may be dispossessed the appellant filed a suit for
permanent injunction and for recovery of possession of the immovable properties
described in Schedules A and B of the plaint. In their written statement, the defendants
did not raise any contention with regard to the plots of land described in Schedule A of
the plaint. However, the right, title and possession of the appellant in regard to the plots
of land mentioned in Schedule B were questioned.
12. Several issues were framed by the learned trial Judge, the relevant one being Issue
Nos. 1, 2, 4 and 5, which read as under :-
" 1. Whether the plaintiff is in possession of the plaint properties?
2. Whether the plaintiff is entitled to the injunction prayed for?
4 Whether the plaintiff has title to the plaint schedule property?
5. Whether the plaintiff is entitled to recovery of possession of any portion of the plaint
schedule property?"
13. On 25th March, 1988 the learned trial Judge decreed the suit of the plaintiff appellant
for all the items of the suit land, except for Item No. 1 of Schedule B.
14. On appeals preferred by the appellant as well as the respondents, the learned
Subordinate Judge, Thalassery, passed the following order on 18th December, 1990.
"10. In view of the foregoing discussions, I hold that the finding of the trial Court with
regard to items 2 and 3 in the plaint A schedule is liable to be confirmed. So also the
finding of the trial Court in favour to the plaintiff for recovery of possession of item No.
3 in the plaint B schedule is also to be confirmed. The finding of the lower Court with
regard to item No. 1 in the plaint B schedule that the same belongs to the defendants is
also to be confirmed. But, the finding on item No. 2 in the plaint B schedule is liable to
be set aside and the question of tenancy over this item (R.S. 90/3) has to be remanded to
the lower court for referring the matter to the Land Tribunal for a fresh adjudication. The
point is answered accordingly."
15. The learned Munsif, in view of the aforementioned, as also the two conflicting
certificates, referred the matter to the Land Tribunal.
16. In its order dated 15th October, 1992, the Land Tribunal, in regard to the possession
of the parties, opined :-
"Except the receipts referred to above and Ex.A2 purchase certificate, the plaintiff has not
produced any other title deeds evidencing
@page-SC1458
tenancy or of creation of tenancy in favour of plaintiffs predecessors by the land owner."
It was held :-
"The respondent did not produce any jenmam deed to prove conclusive that Shri
Palakodan Moideen is the actual jenmi of the suit property.
The case is that the 1st respondent (defendant) does not possess any title deeds or valid
documents evidencing tenancy except some land revenue receipts towards payment of
assessment since the year 1976-77, the manuscript rent receipts available in the case
records as Ex.B1 to B1(f) have not been proved and as such the receipts have no
evidentiary value. The mere production of land tax receipts cannot be taken as a
conclusive evidence to prove the title to the property. The 1st respondent (defendant) has
no title deeds, whatsoever, to establish creation of tenancy in his favour on or before 1-4-
1964. But the 1st respondent has obtained purchase certificate in SM 6343/ 77 dt. 31-10-
1977 (Ex.B3) by filing J form statement whereas the plaintiffs predecessor had obtained
purchase certificate previously for the same property as per SM 6324/ 75 dated 7-6-76 by
filing J form statement. It is evident that one of the purchase certificate has been obtained
by fraud or collusion, but none of the aggrieved parties did not approach this tribunal for
remedy under section 72MM (7) of the KLR Act.
On a consideration of the facts and circumstances of the case and the report of the
authorized officer I am satisfied that the respondents (A party in SM 6343/77) is in
possession of the property but he does not possess any valid title deeds evidencing
tenancy. No valid records have been produced by the respondent (defendant) (A party in
SM 6343/77) to establish creation of tenancy in his favour by the land owner on or before
1-4-1964. As a result, I hold that the respondents/defendant in OS 105/85 are having no
tenancy right over the said property."
17. Before the trial court, the parties adduced oral as well as documentary evidence.
Keeping in view the fact that the plaintiff has been able to prove his possession by
producing tax receipts from 1955 onwards; whereas the defendants were paying tax from
1977 onwards, the learned trial Judge in his judgment dated 27th February, 1993 held :-
"On a proper appraisal of the evidence adduced in this case it may be safely concluded
that the property comprised in R.S. No. 90/3 the predecessor in interest of the plaintiff,
Bavu Valappil Mammad had title over these property. This right has been subsequently
developed on the plaintiff. In the light of discussions made this court is satisfied that the
plaintiff has title to item No. 2 in the B schedule property and she is entitled to recover
possession of the same as it is evident that the defendant is in possession of the same."
As regards the contention of the defendants that they had acquired title by adverse
possession, it was held that the same has not been proved.
18. The first appellate court, by its Judgment dated 13th January, 1994, affirmed the said
findings of the learned trial court holding that the purchase certificate was granted by the
Tribunal upon service of notice upon Ayisumma, who was the original landlord. It was
held :-
"There is also no satisfactory or reliable evidence to prove that the appellants have been
in possession of item No.2 of B schedule at any point of time. If really the first appellant
had been in possession of the property ever since 1959 at least he would have paid the
assessment in respect of the plaint schedule property. But for the first time the assessment
was paid by the respondent in 1974. This is just prior to the initiation of proceedings
before the Land Tribunal. From the evidence available in this case I find that the Land
Tribunal and the lower Court correctly came to the conclusion that the respondent is the
tenant of the disputed property and item No.2 of B schedule belongs to her. So this point
is answered in favour of the respondent."
Plea of the respondents in regard to his claim of adverse possession was also negated.
19. The High Court in the second appeal filed by the respondents framed the following
substantial question of law :-
"Whether the courts below were justified in holding that the plaintiff has title to be
granted a decree for recovery of possession of B schedule item No.2 from the defendants,
after the Land Tribunal had found that the plaintiff does not have valid title deeds
evidencing tenancy?"
@page-SC1459
20. The High Court proceeded on the basis that as both the parties failed to prove their
title over the property, the defendant-respondent being in prior possession over item No.2
of Schedule B of the suit property, the suit should have been dismissed to that extent.
21. Mr. T.V. George, learned counsel appearing on behalf of the appellant in support of
the appeal would submit :-
1) The High Court committed a serious error in interfering with the concurrent finding of
fact arrived at by the courts below in regard to possession of the appellant.
2) Keeping in view the underlying principle governing the Act, the fact that was required
to be determined was whether respondent No. 1, having failed to prove that he was a
tenant within the meaning of Section 13 of the Act, could have been granted a certificate
of assignment.
22. Mr. H.V. Hameed, learned counsel appearing on behalf of the first respondent, on the
other hand, urged that the learned trial court as also the court of appeal proceeded to
determine the issue only on the premise that the defendant being a sub-tenant under the
plaintiff-appellant, the High Court cannot be said to have committed any error in
applying the correct legal principle, namely that if the parties have not been able to prove
their title, the respondent who was in prior possession, should be allowed to continue to
do so.
23. We have noticed hereinbefore the relevant provisions of the Act.
24. In respect of four items of the properties, the plaintiff-appellant has been able to prove
her title as also the possession. The fact that her husband was the cultivating tenant in
respect of the suit property is not in dispute. The tax receipts filed by her also go to show
that the entire suit land was the subject-matter of grant of tenancy by the landlord and if
not from a date, tax has been paid by the appellant-plaintiff at least from 1955 onwards.
In view of Section 110 of the Indian Evidence Act a presumption can be raised in regard
possession both backward and forward.
25. Section 72 of the Act was inserted in the year 1969. If prior to coming into force of
1969 Amendment Act, a tenant had purchased the right, title and interest of the landlord,
the matter might have been different. Section 72-B of the Act empowers the Tribunal to
entertain an application for assignment of the right, title and interest of the landlord in
favour of the tenant on payment of a price to be determined in the manner envisaged
thereunder.
26. The jurisdiction of the Tribunal, therefore, was restricted. Before arriving at a
conclusion that the applicant was entitled to a certificate of assignment, a finding was
required to be arrived at that he was a cultivating tenant within the meaning of Section 13
of the Act. The properties of the erstwhile landlord or intermediates having vested in the
State, they were conferred a limited right, namely, the right to receive the sale proceeds.
27

. Once a certificate of assignment had been granted in favour of the husband of the
appellant, no other certificate could have been issued unless a finding of fact was arrived
at that the first certificate was obtained by fraud as was the case in Hamza Haji v. State of
Kerala and another, 2006 (8) Scale 75 and A. A. Gopalakrishnan v. Cochin Devaswom
Board and others, 2007 (10) Scale 572. 2006 AIR SCW 4168
2007 AIR SCW 5741

28. The Land Tribunal on the reference made by a Civil Court was required to arrive at
the conclusion one way or the other as to whether the plaintiff or the defendant was the
cultivating tenant within the meaning of Section 13 of the Act. It could not have, on the
basis of a stray statement made by a co-villager, arrived at a finding that the defendant
had been in possession. The said purported finding on the basis of a statement made
before the Revenue Inspector, CR, Payyannur could not have been acted upon.
29. In any event, the Civil Court was the final court of fact. Before it oral or documentary
evidence had been adduced. It was not bound by the observations made by the Tribunal
either in regard to title or possession of the property. The Tribunal moreover failed to
consider that a tenancy can be created orally upon delivery of possession and/ or upon
grant of rent receipt. Rent receipt indisputably evidences possession. What was relevant
for the purpose of determination of the issue was who was in possession of the properties
in question when Section 72-B of the Act came into force. As the Tribunal itself had
issued two certificates, the jurisdiction to determine the right, title and interest as also
possession of the suit
@page-SC1460
properties was only with the Civil Courts. It is, therefore, not correct to contend that as on
the basis of the purported report, the Tribunal had found possession over the plot in
question.
30. The approach of the High Court, with respect, was not correct. The right to obtain a
certificate of assignment is dependent upon one's right as a tenant in terms of Section 13
of the Act and not otherwise. The High Court could not have held that the appellant was
an intermediatory and as such his estate had also vested with the respondents. No
contention was raised by the appellant that the respondent was the sub-tenant of the
appellant. The same, in our opinion, was irrelevant.
31. The High Court proceeded only on the basis of the findings of the Tribunal. It failed
to notice that for all intent and purport the said findings was overturned by the Civil
Court, wherefor it had the requisite jurisdiction.
32. A certificate issued under Section 72-K of the Act is conclusive. Once the same is
found to be conclusive, the same cannot be refused to be taken into consideration for any
purpose whatsoever. The only issue which, therefore, should have been raised by the
High Court was as to who was entitled thereto, keeping in view the fact that the Land
Tribunal had granted certificates of assignment to both the parties. In view of the
statutory scheme, both the parties could not have been given the certificates of
assignment. The certificate in favour of the appellant, even otherwise, having been
granted earlier and the same having not been set aside on the ground of fraud or illegality,
it was conclusive even as against the Land Tribunal. The Land Tribunal, therefore, had no
jurisdiction to issue a second certificate.
33. For the reasons aforementioned the impugned judgment of the High Court is
unsustainable which is set aside accordingly. The appeal is allowed. However, in the facts
and circumstances of the case, there shall be no order as to costs.
Appeal allowed.
AIR 2008 SUPREME COURT 1460 "Nivrutti Pandurang Kokate v. State of
Maharashtra"
(From : Bombay)*
Coram : 2 Dr. A. PASAYAT AND P. SATHASIVAM, JJ.
Criminal Appeal No. 345 of 2008 (arising out of SLP (Cri.) No. 5059 of 2007), D/- 19 -2
-2008.
Nivrutti Pandurang Kokate and Ors. v. State of Maharashtra.
Penal Code (45 of 1860), S.300 - Evidence Act (1 of 1872), S.110 - MURDER -
EVIDENCE - WITNESS - Murder - Child witness - Conviction on basis of - Murder of
husband by his wife and paramours - Daughter of accused giving concise, specific and
vivid testimony of incident - Evidence neither embellished nor embroidered - Conviction
of accused can be based on it.(Para 10)
Cases Referred : Chronological Paras
2003 AIR SCW 5547 : AIR 2004 SC 23 : 2004 Cri LJ 19 (Rel. on) 10
2001 AIR SCW 81 : AIR 2001 SC 482 : 2001 Cri LJ 705 (Rel. on) 8
(1997) 5 SCC 341 (Rel. on) 9
(1895) 159 US 523 : 40 Law Ed 244 8
Kanhaiya Priyadarshi, AC, for Appellants; Chinmoy Khaladkar, Ravindra Keshavrao
Adsure, for Respondent.
* Cri.A. No. 621 of 2002, D/- 22-11-2006 (Bom).
Judgement
1. Dr. ARIJIT PASAYAT, J. :-Leave granted.
2. Challenge in this appeal is to the judgment rendered by a Division Bench of the
Bombay High Court. Each of the appellants was convicted for offence punishable under
Sections 302 and 201 read with Section 34 of the Indian Penal Code, 1860 (in short the
'IPC') for allegedly committing murder of one Baban Misal (hereinafter referred to as the
'deceased') in the night between 9th July, 1998 and 10th July 1998. It was further alleged
that they had buried him in his agricultural land, a short distance from his house. Ranjana
Baban Misal who was the accused No. 1 and the appellant No. 1 before the High Court,
had expired and therefore, the appeal was held to have abetted so far as she is concerned.
Appellants 2 and 3 were claimed to be her paramours and appellant No. 4 is the son of
appellant No. 1 and the deceased. He had other siblings one of which was examined as an
eye witness to
@page-SC1461
the incident.
3. The prosecution version in a nutshell was that deceased appellant Ranjana had extra
marital affairs with appellants 2 and 3. Since the deceased objected to such activities,
they together with her son committed the murder of the deceased and disposed of the
dead body by burying it in his own agricultural land near his house and by disposing of
the blood, blood stained clothes and other articles.
4. The case of the accused persons was one of denial. The trial court placing reliance on
the evidence of the daughter of the deceased PW 13, who was aged about 12 or 13 years
at the time of the incident, found the accused persons guilty.
5. In support of the appeal learned counsel for the appellants submitted that no credence
should have been attached to the evidence of PW 13. It was submitted that unexplained
delay in making search for the deceased and ultimately missing report was given.
Learned counsel for the State on the other hand supported the judgment.
6. We shall deal with the acceptability of child witness PW 13. There are certain other
factors which also have relevance. The recovery of the weapon of the assault led to
further investigation. PW 9 is shop keeper who sold the said weapon to the appellant
No.3 on the date of incident. This was followed by another purchase by appellant No. 4
from PW 11 of 9 kgs of salt. The trial Court and High Court noted that salt acts as a
preservative. So far as evidence of PW 13 is concerned it goes to show that the deceased
was sleeping alone in his hut and eating in his brother's house. There was an extremely
estranged relationship of the deceased with his wife and it was known to the relatives.
The recovery of the dead body from the pit in the agricultural land at a short distance also
has relevance.
7. PW 13 has deposed that her mother of the deceased appellant No. 1 washed the blood
of the father with a bucket of water and cloth. She poured it outside the house. The
appellants spread shawl on tiles. They put the dead body on the shawl and put gunny bag
on the dead body. They lifted it by holding the shawl. They carried the body to their field.
They buried it in the pit. Thereafter they returned home. Appellant Nos. 2 and 3 went to
their respective houses. The appellant No. 1 locked the house where the deceased was
killed and she went to the hut to sleep. She went near her brother who had continued to
sleep through the incident and slept. Her evidence is as concise and precise and as it is
specific and vivid. It is neither embellished nor embroidered. It is the evidence of a child
who has seen through the unusual and cruel incidence. She was a girl of tender age who
saw the killing of her father by her mother and others.
8

. The age of the witness during examination was taken to be about 12 years. The Indian
Evidence Act, 1872 (in short "the Evidence Act") does not prescribe any particular age as
a determinative factor to treat a witness to be a competent one. On the contrary, Section
118 of the Evidence Act envisages that all persons shall be competent to testify, unless the
court considers that they are prevented from understanding the questions put to them or
from giving rational answers to these questions, because of tender years, extreme old age,
disease whether of mind, or any other cause of the same kind. A child of tender age can
be allowed to testify if he has intellectual capacity to understand questions and give
rational answers thereto. This position was concisely stated by Brewer, J. in Wheeler v.
United States (159 US 523). The evidence of a child witness is not required to be rejected
per se, but the Court as a rule of prudence considers such evidence with close scrutiny
and only on being convinced about the quality thereof and reliability can record
conviction, based thereon. [See Suryanarayana v. State of Karnataka (2001 (9) SCC 129)]
2001 AIR SCW 81

9. In Dattu Ramrao Sakhare v. State of Maharashtra [(1997) 5 SCC 341] it was held as
follows: (SCC p. 343, para 5):
"A child witness if found competent to depose to the facts and reliable one such evidence
could be the basis of conviction. In other words even in the absence of oath the evidence
of a child witness can be considered under Section 118 of the Evidence Act provided that
such witness is able to understand the questions and able to give rational answers thereof.
The evidence of a child witness and credibility thereof would depend upon the
circumstances of each case. The only precaution which the court should bear in mind
while assessing the evidence of a child witness is that the witness must be a reliable one
and his/her
@page-SC1462
demeanour must be like any other competent witness and there is no likelihood of being
tutored."
The decision on the question whether the child witness has sufficient intelligence
primarily rests with the trial Judge who notices his manners, his apparent possession or
lack of intelligence, and the said Judge may resort to any examination which will tend to
disclose his capacity and intelligence as well as his understanding of the obligation of an
oath. The decision of the trial court may, however, be disturbed by the higher court if
from what is preserved in the records, it is clear that his conclusion was erroneous. This
precaution is necessary because child witnesses are amenable to tutoring and often live in
a world of make-believe. Though it is an established principle that child witnesses are
dangerous witnesses as they are pliable and liable to be influenced easily, shaken and
moulded, but it is also an accepted norm that if after careful scrutiny of their evidence the
court comes to the conclusion that there is an impress of truth in it, there is no obstacle in
the way of accepting the evidence of a child witness.
10

. The above position was highlighted in Ratansingh Dalsukhbhai Nayak v. State of


Gujarat (2004 (1) SCC 64). Looked at from any angle the judgments of the trial court and
the High Court do not suffer from any infirmity to warrant interference. 2003 AIR
SCW 5547

11. Appeal is accordingly dismissed.


Appeal dismissed.
AIR 2008 SUPREME COURT 1462 "Mohd. Hussain v. Occhavlal"
(From : Madhya Pradesh)
Coram : 2 A. K. MATHUR AND TARUN CHATTERJEE, JJ.
Civil Appeal No. 912 of 1999, D/- 19 -2 -2008.
Mohd. Hussain (Dead) by L.Rs. and Ors. v. Occhavlal and Ors.
(A) Civil P.C. (5 of 1908), O.22, R.4, S.100 - LEGAL REPRESENTATIVES - APPEAL -
ABATEMENT - Abatement of appeal - Death of one of respondents - Judgment delivered
thereafter - Some of heirs and legal representatives of deceased were already on record -
Setting aside entire judgment for not bringing entire body of heirs and legal
representatives of deceased on a record - Would be too technical.
S. A. No. 27/1978, D/- 28-02-1992 (MP), Reversed. (Para 3)
(B) Civil P.C. (5 of 1908), O.1, R.10, S.100 - CIVIL PROCEDURE - APPEAL -
MORTGAGE - Second appeal - Necessary parties - Suit for redemption - One of
mortgagees already dead - Impleadment of his two married daughters - One of the
daughters already dead and another was neither in occupation of suit premises nor
residing with mortgagee at time of his death - Suit could not be said to be not
maintainable for not impleading said two married daughters of mortgagee - Moreso when
two sons of mortgagee were already on record and representing interest in estate - No
fraud or collusion shown between plaintiffs and said two sons of mortgagee - Dismissal
of suit at second appellate stage for non joinder of parties i.e. said two married daughters
of mortgagee, liable to be set aside.
S. A. No. 27 of 1978, D/-28-02-1992 (UP), Reversed. (Paras 9 , 10, 11, 12)
(C) Civil P.C. (5 of 1908), S.100 - APPEAL - HIGH COURT - CONCURRENT
FINDINGS - Second appeal - Powers of High Court - Concurrent findings by Courts that
two married daughters of one of deceased mortgagee were not necessary parties in suit
for redemption - Finding by High Court in second appeal that suit was not maintainable
in law as said two married daughters pf mortgagee were not made parties to suit - Not
permissible.
S. A. No. 27 of 1978, D/-28-02-1992 (MP), Reversed. (Para 12)
Cases Referred : Chronological Paras
(1991) 3 SCC 114 (Ref.) 10
AIR 1965 SC 792 (Ref.) 9
AIR 1965 SC 271 (Ref.) 8, 11
AIR 1941 Nagpur 5 8
AIR 1938 Nagpur 32 8
AIR 1919 PC 24 8
AIR 1915 Oudh 29 8
AIR 1914 All 109 8
S.K. Gambhir, Sr. Adv., Anil Sharma, B.K. Sharma, H.K. Puri, for Appellants; Alok
Bachawat, Ms. Sameena Ahmed, Harinder Mohan Singh, for Respondents.
Judgement
1. TARUN CHATTERJEE, J. :-This appeal is directed against the judgment dated 28th of
February, 1992, which was delivered on 20th of March, 1992 by a learned
@page-SC1463
Judge of the High Court of Madhya Pradesh at Indore in Second Appeal No. 27/1978
whereby the concurrent judgments of the courts below decreeing the suit for redemption
of mortgage filed by the appellants against the respondents were set aside practically on
the ground that the suit for redemption could not be held to be maintainable in law in the
absence of the two married daughters of one of the mortgagees.
2. Before we narrate the facts leading to the filing of this appeal, we may note the two
questions which were posed by the learned counsel for the parties and need to be decided
in this appeal, which are as follows:
i) Whether the second appeal of the respondents 1 to 4 herein, who were the appellants in
the High Court, had abated as they had failed to make an application to bring the legal
heirs and representatives of Mohd. Hussain, one of the respondents in the High Court
who had died during the pendency of that second appeal?
ii) Whether in the absence of the two married daughters of one of the mortgagees, it could
be held that the suit for redemption of mortgage was not maintainable in law, that is to
say the suit for redemption could be dismissed on account of their non-impleadment?
3. Let us, therefore, take up the first question for our decision. The question is whether
the second appeal, which was filed by the respondents 1 to 4, had abated in its entirety on
the death of Mohd. Hussain. Mr. Gambhir, the learned senior counsel appearing for the
appellants contended that in view of the finding that one of the respondents in the second
appeal viz., Mohd. Hussain had died, and no application for substitution of his heirs and
legal representatives was made even till the signing of the judgment, the second appeal
had abated in its entirety and therefore, until and unless the abatement caused on the
death of Mohd. Hussain was set aside, the judgment in the second appeal is liable to be
set aside without going into the merits of the same. From the record, it appears that
Mohd. Hussain had died on 19th of November, 1991. It is true that the application for
substitution after setting aside abatement was filed by the appellants in the second appeal
to bring on record the heirs and legal representatives of the deceased Mohd. Hussain on
3rd of March, 1992 after the judgment was already signed by the learned Judge. It is an
admitted position that some of the heirs and legal representatives of Mohd. Hussain were
already on record in the file of the second appeal. Such being the position, in our view,
the question of abatement of the second appeal on the death of Mohd. Hussain could not
arise at all as some of his heirs and legal representatives were admittedly on record. Only
the question of noting the death of Mohd. Hussain could arise and his name could be
deleted from the array of respondents in the second appeal. That being the position, even
if the judgment was delivered after the death of Mohd. Hussain whose entire body of
heirs and legal representatives were not brought on record, even then the only
requirement under the law was to take note of the death of Mohd. Hussain and delete his
name from the array of respondents in the second appeal and the rest of the heirs and
legal representatives who were not brought on record could be added in the cause title of
the memorandum of appeal. Therefore, in our view, it would be considered too technical
to set aside the entire judgment of the High Court on the ground of not bringing the entire
body of heirs and legal representatives of Mohd. Hussain because some of his heirs and
legal representatives were on record and the left out heirs and legal representatives were
sufficiently represented by the other heirs on record. Accordingly, the first question, as
posed hereinabove, is decided in favour of the present respondents.
4. We may now narrate the relevant facts leading to the filing of this appeal. On 24th of
April, 1932, late Hasan All entered into a mortgage with possession of the suit premises
with late Nandram and his two sons, Manaklal and Motilal for Rs. 300/-. On or about
17th of July, 1967, a suit was brought by Hussainabai, Sugrabai and Mohd. Hussain,
being heirs of Hasan Ali, (appellants herein) against Manaklal and Motilal (defendant
Nos. 1 and 2) and their sons (proforma defendant Nos. 3 and 7) for redemption of
mortgage of the suit premises, as fully described in the schedule of the plaint. At the time
of filing of the suit for redemption of mortgage by the plaintiffs/appellants, Nandram was
already dead leaving behind his two sons viz., Manaklal and Motilal and two married
daughters viz., Annapurna and Pyaribai. It was the case of the plaintiffs/
@page-SC1464
appellants that the respondents were avoiding to let the appellants have the suit premises
redeemed and that the respondents had the intention to deprive them of the suit premises.
Accordingly, on the allegations made in the plaint, the plaintiffs/appellants sought for a
decree in the suit for redemption in respect of the suit premises. The suit was contested by
the respondents in which it was, inter alia, alleged that the suit premises was in fact sold
by Hasan Ali, since deceased, to them and accordingly, the appellants could not demand
account from them. It was further alleged that the suit was bad on account of non-joinder
of parties as all the legal heirs of Nandram, namely the two married daughters Annapurna
and Pyaribai were not made parties although they were necessary parties. A case of
adverse possession was also pleaded by the respondents in respect of the suit premises.
Accordingly, the respondents pleaded that the suit must be dismissed not only on merits
but also on the ground of non-joinder of parties.
5. The suit of the appellants was decreed in which the trial court found that the appellants
were the legal heirs of Hasan Ali and had the right to redeem the mortgage and to recover
the suit premises from the respondents. The plea of adverse possession raised by the
respondents was rejected and the plea of respondents that the suit was not maintainable in
law in the absence of the two married daughters of Nandram, one of the mortgagees, was
also rejected.
6. Feeling aggrieved, an appeal was carried to the appellate court, which was also
dismissed. The first appellate court held that since the two married daughters were not
residing with Nandram at the time of his death, they were not necessary parties in the suit
for redemption. It was also the finding of the first appellate court that out of the two
married daughters of Nandram, Annapurna was not alive. So far as the other daughter
was concerned, the appellate court held that at the time of the death of Nandram, she was
not residing with him and, therefore, she was also not a necessary party in the suit. It was
further found that the married daughters of Nandram were not in possession of the suit
premises and that since the suit was not for partition of the suit premises in which the
interest of the married daughters could be considered, they were not necessary parties.
Finally, it was held that since Ochavlal-D.W. 1 had clearly deposed that the partition of
the suit premises was already done and after partition, the suit premises had come to his
share and therefore, the married daughters of Nandram had no interest in the same and
accordingly, they were not necessary parties.
7. Aggrieved by the decision of the First Appellate court, which affirmed the judgment of
the Trial Court, the respondents preferred a second appeal in the High Court. The High
Court, as noted herein earlier, had set aside the concurrent judgments of the courts below
and held that the suit was bad and liable to be dismissed because the two married
daughters of Nandram, who were necessary parties to the suit for redemption, had not
been made parties. However, the findings of the courts below to the extent that the two
married daughters were not necessary parties on the death of Nandram, one of the
mortgagees, for the reasons that at the time of his death, they were neither living with him
nor were in occupation of the suit premises and that one of the daughters viz., Annapurna
was already dead, were not considered by the High Court. Therefore, so far as the merits
of the second appeal were concerned, the High Court had not considered the same and
allowed the second appeal on the ground of non-joinder of necessary parties. On the
question of theory of substantial representation of the two married daughters of Late
Nandram by his two sons, it was held that the same would not salvage the case of the
plaintiffs/appellants in the facts and circumstances of the case. It is this judgment of the
High Court, which is impugned in this appeal.
8. As noted herein earlier, the second question, which needs to be looked into and decided
in this appeal is whether the two married daughters of Nandram viz., Annapurna and
Pyaribai were necessary parties to the suit for redemption of mortgage, that is to say
whether in their absence, the suit was maintainable in law. The High Court in the
impugned judgment had relied on Section 19 of the Hindu Succession Act, 1956 and held
that since the two sons and the two married daughters of late Nandram had succeeded to
his estate as tenants-in-common and not as joint tenants, the suit was not maintainable in
law in the absence of the two married daughters. In support of its conclusion that the suit
was not maintainable
@page-SC1465
in the absence of the two married daughters, reliance was placed by the High Court on
the following cases :-
(a) Girdhar Parashram Kirad v. Firm Motilal Champalal, Owners, Hiralal Champalal and
others [AIR 1941 Nagpur 5] (DB)
(b) Ghanaram and others v. Balbhadra Sai and others [AIR 1938 Nagpur 32]
(c) Sunitibala Debi v. Dhara Sundari Debi and another [AIR 1919 PC 24]
(d) Rudra Singh v. Jangi Singh and others [AIR 1915 Oudh 29]

(e) Saeed-ud-din Khan v. Hiralal [1914 24 IC 25]. AIR 1914 All 109

Accordingly, the High Court had negatived the contention of the present appellants that
the doctrine of substantial representation would come to their aid in the facts and
circumstances of the case and held that the defendants/respondents did not represent the
interest of the two married daughters and therefore, in their absence, the respondents
could not have given a valid discharge to the appellants. Another ground on which the
High Court had set aside the judgments of the courts below was that since the objection
as to non-joinder was taken at the earliest opportunity by the respondents and the
appellants without rectifying the said defect had proceeded with the hearing of the said
suit, the question of making good the defect, which was fatal, could not be corrected at
the second appellate stage. It was also held by the High Court that if the appellants were
afforded an opportunity of rectifying the defect as to the non-joinder of parties at that
belated stage, the suit must fail on the ground of limitation. Reliance in this regard was
placed by the High Court in the case of Kanakarathanammal v. Loganatha Mudaliar and
another [AIR 1965 SC 271].
9. Keeping the aforesaid findings of the High Court as well as the courts below in mind,
let us now examine whether the High Court was justified in dismissing the suit of the
plaintiffs/appellants at the second appellate stage on the ground of non-joinder of
necessary parties when, admittedly, the two sons of the deceased mortgagee, who were
also mortgagees in respect of the suit premises, were already representing the estate of
the deceased mortgagee. The High Court, as noted herein earlier, held that the two
married daughters of Nandram, one of the mortgagees, were necessary parties in the suit
for redemption of mortgage and in their absence, the suit was not maintainable in law. We
are unable to endorse the views expressed by the High Court. It is true that in a suit for
redemption of mortgage, all the heirs and legal representatives of the deceased mortgagee
are necessary parties but, in the facts and circumstances of the present case, we do not
find any reason to agree that in the absence of the two married daughters, the suit could
not be maintainable in law, for at least two reasons:-
i) It was the finding of the first appellate court that at the time of filing of the suit for
redemption, one of the mortgagees viz., Nandram was already dead. A finding was also
made that one of the married daughters viz., Annapurna was dead. If this finding is
accepted, then Annapurna cannot be said to be a necessary party at the time of filing of
the suit. So far as the other married daughter viz., Pyaribai is concerned, the finding of
the appellate court was to the effect that she was not in occupation of the suit premises
nor was she staying with the mortgagee viz., Nandram at the time of his death. Again, if
this finding is also accepted, we are not in a position to hold that the suit could not be
held to be not maintainable in law in the absence of the two married daughters.
ii) Even assuming that the two married daughters of Nandram were necessary parties,
then also, we must hold that the interest of the two married daughters in the estate of
Nandram was sufficiently represented by their two brothers viz., Manaklal and Motilal. In
the case of N.K. Mohd. Sulaiman Sahib v. N.C. Mohd. Ismail Saheb and others [AIR
1966 SC 792], this court in paragraph 14 observed as follows: -
"14. Ordinarily the Court does not regard a decree binding upon a person who was not
impleaded eo nomine in the action. But to that rule there are certain recognized
exceptions. Where by the personal law governing the absent heir the heir impleaded
represents his interest in the estate of the deceased, there is vet another exception which is
evolved in the larger interest of administration of justice. If there be a debt justly due and
no prejudice is shown to the absent heir, the decree in an action where the plaintiff has
after bona fide enquiry impleaded
@page-SC1466
all the heirs known to him will ordinarily be held binding upon all persons interested in
the estate. The Court will undoubtedly investigate, if invited, whether the decree was
obtained by fraud, collusion or other means intended to overreach the Court. The Court
will also enquire whether there was a real contest in the suit, and may for that purpose
ascertain whether there was any special defence which the absent defendant could put
forward, but which was not put forward. Where however on account of a bona fide error,
the plaintiff seeking relief institutes his suit against a person who is not representing the
estate of a deceased person against whom the plaintiff has a claim either at all or even
partially, in the absence of fraud or collusion or other ground which taint the decree, a
decree passed against the persons impleaded as heirs binds the estate, even though other
persons interested in the estate are not brought on the record. This principle applies to all
parties irrespective of their religious persuasion.
(Emphasis supplied)
From a bare reading of the aforesaid observation of this court in the abovementioned
decision, it is clear that ordinarily the court does not regard a decree binding upon a
person who was not impleaded in the action. While making this observation, this court
culled out some important exceptions :-
(i) Where by the personal law governing the absent heir, the heir impleaded represents his
interest in the estate of the deceased, the decree would be binding on all the persons
interested in the estate.
(ii) If there be a debt justly due and no prejudice is shown to the absent heir, the decree in
an action where the plaintiff has after bona fide enquiry impleaded all the heirs known to
him will ordinarily be held binding upon all persons interested in the estate.
(iii) The court will also investigate, if invited, whether the decree was obtained by fraud,
collusion or other means intended to overreach the court. Therefore, in the absence of
fraud, collusion or other similar grounds, which taint the decree, a decree passed against
the heirs impleaded binds the other heirs as well even though the other persons interested
are not brought on record.
10. We find no difficulty in following the principle laid down by this court in the
aforesaid decision. The two sons viz., Manaklal and Motilal, who were also the original
mortgagees along with Nandram, being the sons of Nandram, duly represented the estate
of the deceased. It was not the case of the defendants/respondents either in the written
statement or in evidence that the two married daughters were not made parties collusively
or fraudulently. The suit filed by the appellants only against the two sons of late Nandram
and their sons was not out of fraud or collusion between them. It is also clear from the
record that the two sons of Nandram seriously contested the suit and also the appeal filed
against the judgment of the trial court before the first appellate court and finally the
second appeal in the High Court. Therefore, by no stretch of imagination, it can be said
that the suit was filed by the plaintiffs/appellants in collusion or fraud with the two sons
of Nandram. Therefore, in the absence of such a defence, it must be held that the estate of
late Nandram, one of the mortgagees, was sufficiently and in a bona fide manner
represented by Manaklal and Motilal and there was no fraud or collusion between them
and the plaintiffs/appellants and accordingly, the decree that would be passed against
Manaklal and Motilal as heirs and legal representatives of late Nandram also binds the
estate even though the two married daughters, who may be interested in the estate, were
not brought on record. This view is also supported by the decision of this court in
Surayya Begum (Mst.) v. Mohd. Usman and others [(1991) 3 SCC 114]. In that case, this
court in paragraph 9 has observed as follows:-
"..........This of course, is subject to the essential condition that the interest of a person
concerned has really been represented by the others; in other words, his interest has been
looked after in a bona fide manner. If there be any clash of interests between the person
concerned and his assumed representative or if the latter due to collusion or for any other
reason, mala fide neglects to defend the case, he cannot be considered to be a
representative.."
11. In view of our discussions made here-inabove and following the principles laid down
in the aforesaid two decisions of this court, we are, therefore, of the view that the two
sons had sufficiently and in a bona fide
@page-SC1467
manner represented the estate of the deceased Nandram and therefore, the suit could not
be dismissed on that ground. It is true that the objection as to maintainability of the suit in
the absence of the two married daughters was taken in the suit itself but we should not
forget that in view of the findings arrived at by the trial court as well as by the appellate
court, the suit of the appellants was decreed which was affirmed at the first appellate
stage. In view of the discussions made hereinabove that the two sons of late Nandram had
substantially represented the estate of the deceased which binds the married daughters of
late Nandram, it is not necessary for us to go into the question of limitation if the
daughters are now allowed to be impleaded in the suit. Accordingly, it is not necessary
for us to deal with the decision of this court in Kanakarathanammal v. Loganatha
Mudaliar and another [AIR 1965 SC 271] in the facts and circumstances of the case and
in view of the discussions made hereinabove.
12. For the reasons aforesaid, we are, therefore, of the view that the High Court had failed
at the second appellate stage by dismissing the suit of the plaintiffs/appellants on the
ground of non-joinder of parties because, in our view, the two sons of late Nandram duly,
substantially and in a bona fide manner represented the interest in the estate, if there be
any, of the two married daughters, in the absence of any case made out of fraud or
collusion between the plaintiffs/appellants and the two sons of late Nandram. The
defendants/respondents all throughout denied the claim of the plaintiffs/appellants made
in the suit and contended, inter alia, that the suit premises was sold to them and it was not
a case of mortgage. In fact, a case of adverse possession was made out by them i.e. it was
contended that the defendants/respondents had acquired title to the suit premises by virtue
of adverse possession. That apart, from the findings arrived at by the appellate court, as
noted herein earlier, which were not challenged before us by the learned counsel for the
respondents, it is clear that i) one of the daughters viz., Annapurna was already dead; ii)
the other daughter viz., Pyaribai had no interest in the suit premises as she was not
residing with late Nandram at the time of his death and iii) reliance was placed on the
deposition of D.W.1-Occhavlal who deposed that there was a partition of the suit
premises which fell in his share and therefore, it was concluded that the two married
daughters were not necessary parties. That being the concurrent findings of fact arrived at
by the courts below, it was not open to the High Court at the second appellate stage to
hold that the suit was not maintainable in law as the two married daughters of Nandram
were not made parties to the suit for redemption.
13. Before we conclude, we may note that while allowing the second appeal, the High
Court had not considered the same on merits but in view of the stand taken by the learned
counsel for the respondents before us, we do not find any reason to upset the findings of
the courts below on merits viz., the suit premises was mortgaged with the respondents at
a sum of Rs. 300/- and therefore, the appellants were entitled to a decree in the suit for
redemption. Since, this finding was not challenged before us by the learned counsel for
the respondents, it is not necessary for us to remit the case back to the High Court for a
decision on merits. Accordingly, the appeal is bound to succeed and is, therefore,
allowed. The judgment and decree of the High Court is set aside and that of the courts
below are restored. There will be no order as to costs.
Appeal allowed.
AIR 2008 SUPREME COURT 1467 "Cherotte Sugathan v. Cherotte Bharathi"
(From : Kerala)*
Coram : 2 S. B. SINHA AND V. S. SIRPURKAR, JJ.
Civil Appeal No. 1323 of 2008 (arising out of SLP (C) No. 236 of 2004), D/- 15 -2 -2008.
Cherotte Sugathan (D) by L.Rs. and Ors. v. Cherotte Bharathi and Ors.
Hindu Succession Act (30 of 1956), S.24, S.14, S.4 - Hindu Widow's Remarriage Act (15
of 1856), S.2 (since repealed) - SUCCESSION - MARRIAGE - Remarrying widows -
Restriction on succession - Widow inheriting property of her husband on his death -
Becomes its absolute owner - Subsequent remarriage does not divest her of property in
view of S.24, S.14 - Succession Act overrides provisions of 1856 Act. (Para 13)
@page-SC1468
Cases Referred : Chronological Paras
1999 AIR SCW 4583 : AIR 2000 SC 434 (Approved) 15
AIR 1999 Kerala 62 (Approved) 14
AIR 1983 Patna 33 (Approved) 14
AIR 1976 SC 2595 (Approved) 14
K. Rajeev, for Appellants; A. Raghunath, for Respondents.
* A.S. No. 645 of 1992 (N), D/- 7-7-2003 (Ker).
Judgement
1. S. B. SINHA, J. :- Leave granted.
2. Whether Section 2 of the Hindu Widows Re-Marriage Act, 1856 would apply to the
facts of the present case is the question in this appeal.
3. The fact involved herein is as under :
The properties in dispute belonged to one Sri Pervakutty. He had three sons and two
daughters, namely, Sugathan, Surendran, Sukumaran @ Soman, Soumini and Karhiayani.
He allegedly executed a Will on 11-10-1975 bequeathing the said properties in favour of
his sons. In the said Will, provisions were allegedly made for payment of monthly
allowance to the wife of Sri Pervakutty, defendant No. 3 (since deceased) as also right of
residence in the house situated therein. Sri Pervakutty died on 20-10-1975. Sukumaran
died on 2-8-1976.
4. First respondent is his widow. First respondent remarried one Elambilakkat
Sudhakaran. Sudhakaran died on 12-9-1979.
She filed a suit on 31-12-1985 for partition claiming 1/3rd share in the suit property.
Appellant herein, inter alia, contended that she, in terms of Section 2 of the Hindu
Widow's Re-marriage Act, 1856. having ceased to have any right in the properties
inherited by her from her husband Sukumaran, the suit was not maintainable.
Respondent Nos. 2 and 3, the daughter of Sri Pervakutty, inter alia, raised a contention
that the purported Will dated 11-10-1975 was not a valid one.
5. By a judgment and order dated 31-3-1992, the said suit for partition was decreed
declaring 1/3rd share in the suit properties in favour of the first respondent. It was opined
that since the testator bequeathed the tenancy right as contained in item No. 2 of the
schedule, the same was available for partition.
Appellants preferred an appeal there-against. Respondent Nos.2 and 3 (defendant Nos. 4
and 5) also preferred separate appeals.
6. By reason of the impugned judgment, the High Court allowed the appeals preferred by
respondent Nos. 2 and 3 holding :
"In this case, the plaintiff has claimed succession on the basis of Will. If that be so, the
lower court was correct in holding that Section 23 of the Hindu Succession Act is not
applicable to defendants 1 and 2. But if the succession is not on the basis of Will, then
defendants 1 and 2 will be entitled to the benefit of Section 23 of the Hindu Succession
Act."
In regard to the applicability of the 1856 Act, it was held :
"So far this case is concerned, according to us, Section 24 of the Hindu Succession Act
applies and the plaintiff is entitled to succeed."
It was directed :
"In the above view of the matter, the appeals are disposed of as follows :
The case is remanded to the lower court to frame issue regarding the validity of the Will
and to give an opportunity to the parties to adduce evidence regarding the same and
decide the issue whether the Will is valid of not. The other findings in the judgment are
upheld except the finding regarding the building house in Item No.1 of A schedule. If the
court below takes the view that the Will is not valid, then the contention of defendants 1
and 2 regarding residence in the building house should be considered again."
7. Mr. K. Rajeev, learned counsel appearing on behalf of the appellant, in support of the
appeals, would submit that keeping in view the provisions of Section 2 of the 1856 Act,
Respondent No. 1 could not have been held to have any right in the properties inherited
by her from her husband as she remarried on 12-2-1979.
8. Mr. Raghunath, learned counsel appearing on behalf of the respondent, however,
would support the judgment.
9. Hindu Widow's Remarriage Act was enacted to remove all legal obstacles to the
marriage of Hindu widows.
Section 1 of the said Act encompasses within its fold the said legal policy. Section 2 reads
as under :
"2. Rights of widow in deceased husbands property to cease on her re-marriage.- All
rights and interests which any widow may have in her deceased husband's property by
way of maintenance, or by inheritance to her
@page-SC1469
husband to his lineal successors, or by virtue of any Will or testamentary disposition
conferring upon her, without express permission to re-marry, only a limited interest in
such property, with no power of alienating the same, shall upon her re-marriage cease and
determine as if she had then died; and the next heirs of her deceased husband, or other
persons entitled to the property on her death, shall thereupon succeed to the same."
10. Applicability of the said provision must be tested having regard to the provisions
contained in Hindu Succession Act, 1956. Section 4 of the Act provides for the overriding
effect of the Act stating :
"4. Overriding effect of Act.- (1) Save as otherwise expressly provided in this Act,-
(a) any text, rule or interpretation of Hindu Law or any custom or usage as part of that
law in force immediately before the commencement of this Act, shall cease to have effect
with respect to any matter for which provision is made in this Act;
(b) any other law in force immediately before the commencement of this Act shall cease
to apply to Hindus in so far as it is inconsistent with any of the provisions contained in
this Act."
11. The Act brought about a sea change in Shastric Hindu Law. Hindu widows were
brought on equal footing in the matter of inheritance and succession along with the male
heirs. Section 14(1) stipulates that any property possessed by a female Hindu, whether
acquired before or after the commencement of the Act, will be held by her as a full owner
thereof.
Section 24, as it then stood, reads as under :
"24. Certain widows remarrying may not inherit as widows.- Any heir who is related to
an intestate as the widow of a pre-deceased son, the widow of a pre-deceased son of a
predeceased son or the widow of a brother shall not be entitled to succeed to the property
of the intestate as such widow, if on the date the succession opens, she has remarried."
12. Upon the death of Sukumaran, his share vested in the first respondent absolutely.
Such absolute vesting of property in her could not be subjected to divestment save and
except by reason of a statute.
13. Succession had not opened in this case when the 1956 Act came into force. Section 2
of the 1856 Act speaks about a limited right but when succession opened on 2-8-1976.
first respondent became an absolute owner of the property by reason of inheritance from
her husband in terms of sub-section (1) of Section 14 of the 1956 Act.
Section 4 of the 1956 Act has an overriding effect. The provisions of 1956 Act, thus, shall
prevail over the text of any Hindu Law or the provisions of 1856 Act. Section 2 of the
1856 Act would not prevail over the provisions of the 1956 Act having regard to Sections
4 and 24 thereof.
14. The question posed before us is no longer res integra.

In Chando Mehtain and Ors. v. Khublal Mahto and Ors. [AIR 1983 Patna 33], the Patna
High Court opined : Para 6 of AIR

"The Hindu Widow's Remarriage Act, 1856 has not been repealed by the Hindu
Succession Act, 1956 but Section 4 of the latter Act has an overriding effect and in effect
abrogates the operation of the Hindu Widows Remarriage Act, 1856. According to
Section 4 of the Hindu Succession Act all existing laws whether in the shape of
enactments or otherwise shall cease to apply to Hindus in so far as they are inconsistent
with any of the provisions contained in this Act."
In Kasturi Devi v. Deputy Director of Consolidation [AIR 1976 SC 2595], this Court
categorically held that a mother cannot be divested of her interest in the deceased son's
property either on the ground of unchastity or remarriage.
Kerala High Court, in Thankam v. Rajan [AIR 1999 Kerala 62], held that remarriage of
the wife cannot be a ground for her losing right to succeed to her deceased husband's
property.
15

. Yet again this Court, in Velamuri Venkata Sivaprasad (Dead) by L.Rs. v. Kothuri
Venkateswarlu (Dead) by L.Rs. and Ors. [(2000) 2 SCC 139], held : 1999 AIR
SCW 4583, Para 54

"52. incidentally. Section 24 of the Succession Act of 1956 placed certain restrictions on
certain specified widows in the event of there being a re-marriage; while it is true that the
section speaks of a pre-deceased son or son of a pre-deceased son but this in our view is a
reflection of the Shastric Law
@page-SC1470
on to the statute. The Act of 1956 in terms of Section 8 permits the widow of a Hindu
male to inherit simultaneously with the son, daughter and other heirs specified in Class I
of the Schedule. As a matter of fact she takes her share absolutely and not the widow's
estate only in terms of Section 14. Re-marriage of a widow stands legalised by reason of
the incorporation of the Act of 1956 but on her re-marriage she forfeits the right to obtain
any benefit from out of her deceased husband's estate and Section 2 of the Act of 1856 as
noticed above is very specific that the estate in that event would pass on to the next heir
of her deceased husband as if she were dead. Incidentally, the Act of 1856 does not stand
abrogated or repealed by the Succession Act of 1956 and it is only by Act 24 of 1983 that
the Act stands repealed. As such the Act of 1856 had its fullest application in the
contextual facts in 1956 when Section 14(1) of the Hindu Succession Act was relied upon
by Defendant 1."
We respectfully agree with the said view.
16. For the reasons aforementioned, we do not find any infirmity in the judgment of the
High Court. The appeal, therefore, is dismissed without any order as to costs.
Appeal dismissed.
AIR 2008 SUPREME COURT 1470 "K. Manjusree v. State of A. P."
(From : Andhra Pradesh)*
Coram : 3 K. G. BALAKRISHNAN, R. V. RAVEENDRAN AND J. M. PANCHAL,
JJ.
Civil Appeal No. 1313 of 2008 (arising out of SLP (C) No. 18330 of 2006) with W. P. (C)
Nos. 51 and 97 of 2007 and etc., D/- 15 -2 -2008.
K. Manjusree, etc. v. State of A.P. and Ann
(A) Constitution of India, Art.233 - APPOINTMENT - JUDICIAL SERVICE - Selection
to post of District and Sessions Judge - Criteria decided by Administrative Committee of
High Court - was only marks for written examination and not for interviews -
Introduction of requirement of minimum for interview, after entire selection process was
completed - Impermissible. (Paras 24, 28, 29, 32)
(B) Constitution of India, Art.233 - APPOINTMENT - JUDICIAL SERVICE - Post of
District and Sessions Judge - Selection for - Full Court of High Court approved resolution
of Administrative Committee and retained entire process of selection, written
examination and interviews - However, on assumption that resolution of Administrative
Committee prescribed minimum marks for interviews, directed revision merit list by
adopting minimum percentage of marks for interviews - Action of Full Court, held, was
not proper.
W. P. Nos. 10061 and 10062 of 2006, D/- 30-10-2006 (A. P.), Reversed. (Paras 30, 31)
(C) Constitution of India, Art.136 - SPECIAL LEAVE PETITION - Petition under -
Against selection for post of District and Sessions Judge - Petitioner was not a selected
candidate, either in first list or second list - Did not challenge process of selection by
filing writ petition - Was not party to writ petitions - Would in no way aggrieved as she
will not be selected, by adopting either method - Further there was delay of 190 days -
Thus SLP by her liable to be dismissed being not (Para 34)
Cases Referred : Chronological Paras
2001 AIR SCW 4885 : AIR 2002 SC 224 : 2002 Lab IC 128 (Rel. on, Pnt. A) 28
AIR 1987 SC 2267 : 1987 Lab IC 1914 (Foll., Pnt. A) 24, 27
AIR 1985 SC 1351 : 1985 Lab IC 1625 (Foll., Pnt. A) 24, 26
AIR 1984 SC 541 : 1984 Lab IC 301 (Foll., Pnt. A) 24, 25
V. KanagaraJ, Sr. Advocate, Mrs. Anjani Aiyagari, V. Vivekananda, Ms. I. Madhavi, P.
Venkat Reddy, B. Ramana Murthy, Anil Kumar Tandale, V. Sridhar Reddy, K. Santhi
Kumar and R.V. Kameshwaran, for Appellants; P.P. Rao, P.S. Patwalia, Sr. Advocates,
T.V. Ratnam, Mrs. D. Bharathi Reddy, Y. Prabhakara Rao, A.V. Rao, Ugra Narasimha and
Prabhakar Parnam, for Respondents; T.L.V. Iyer, Ranjit Kumar, Sr. Advocates, B. Sridhar,
D. Bharat Kumar, Ms. M. Indrani, Anand, Abhijit Sengupta, Satyajit A. Desai, Ms.
Anagha S. Desai, Amol N. Suryawanshi, for Impleading Party.
* W. P. Nos. 10061 and 10062 of 2006, D/- 30-10-2006 (A.P.).
Judgement
R. V. RAVEENDRAN, J. :- Leave granted in the Special Leave Petition by K. Manjusree.
The selection to ten posts of District and Sessions Judges (Grade-II) in the Andhra
Pradesh State Higher Judicial Service in pursuance of the advertisement dated
@page-SC1471
28-5-2004 is the subject-matter of this appeal by special leave and writ petitions.
2. Selection and appointments to the post of District and Sessions Judges (Grade II) are
governed by the Andhra Pradesh State Higher Judicial Service Rules, 1958 (Rules for
short). The said Rules provide that one-third of the total number of permanent posts of
District and Sessions Judges (Grade II) should be filled by direct recruitment. It also
prescribes the qualifications for appointment, but does not prescribe any procedure for
selection. As the Rules only prescribe the qualifications for appointment but did not lay
down the selection procedure, the manner and method of selection is decided by the High
Court, for every selection, as and when the vacancies are notified for selection.
3. The Government of Andhra Pradesh issued an advertisement dated 28-5-2004 inviting
applications for appointment to the following ten posts of District and Sessions Judges
(Grade II) in the A. P. State Higher Judicial Service by direct recruitment :

Open category : 4 (1 Woman)


Backward Class-Group A : 1 (Woman)
Backward Class Group B : 1 (Woman)
Scheduled Caste : 2 (1 Woman)
Scheduled Tribe : 1

The advertisement stated that a written examination followed by an interview will be held
for selection to the above posts. The last date for receipt of applications was 15-6-2004.
In all 1637 applications were received. On scrutiny 1516 applicants were eligible to take
the written examination.
4. The Pull Court of Andhra Pradesh High Court has authorized its Chief Justice to
constitute Committees for the convenience of administration. The resolutions of the Full
Court containing the guidelines relating to the functioning of the High Court have been
compiled in the form of Standing Orders. SO 2.13 enumerates the matters to be dealt with
by the Full Court. SO 2.14 enumerates the matters to be dealt with by the Administrative
Committee. Recruitment of District Judges is a matter to be dealt with by the
Administrative Committee under SO 2.14. The decision/minutes of the Administrative
Committee in regard to recruitment of District Judges are thereafter placed before the
Full Court for its consideration under SO 2.13.
5. The Administrative Committee by its resolution dated 30-11-2004 decided the method
and manner of selection. It resolved to conduct the written examination for the candidates
for 75 marks and oral examination for 25 marks. It also resolved that the minimum
qualifying marks for the OC, BC, SC and ST candidates shall be as prescribed earlier. As
per its direction, the written examination was held on 30-1-2005 and 1026 candidates
appeared for the examination. The results were declared on 24-2-2005 and 83 candidates
were successful in the written examination. Due to the pendency of some litigation,
interviews could not be held immediately. A Committee of five Judges was constituted
for interviewing the candidates and interviews were held in March 2006. Thereafter, the
marks obtained by the 83 candidates in the written examination and in the interview were
aggregated and a consolidated merit list of the 83 candidates was prepared in the order of
merit on the basis of the aggregate marks. It contained (i) the registration number, (ii) the
names of the candidates, (iii) reservation category, (iv) marks secured in the written
examination out of 100 marks, (v) marks secured in the interview out of 25 marks, and
(vi) the total marks secured in the written examination and interview out of 125.
Thereafter, five more merit lists in respect of categories BC-A, BC-B, BC-D, SC and ST
were prepared. On the basis of the said lists, the Interview Committee finalized the
following list of candidates to be recommended for appointment as per merit and
reservation, and submitted to the Administrative Committee with a report dated 3-4-
2006 :
@page-SC1472
DISTRICT AND SESSIONS JUDGES GRADE-II EXAMINATION
(LIST OF CANDIDATES TO BE RECOMMENDED FOR APPOINTMENT)

S1. No. Regn. Number Name of the candidate Category Written Exam.
(out of 100) Interview (out of 25) Total marks (125)
01 1859 Smt. Venkata Jyothirmayee OC 72 9.6 81.6
02 1775 Smt. C. Sumalatha OC 61 19.4 80.4
03 1073 Smt. K. Manju Sree OC 68.5 10.6 79.1
04 1694 A. Hari Haranatha Sarma OC 64.5 14.4 78.9
05 1009 Smt. G. Anupama Chakravarthy BC (A)51 8.6 59.6
06 1590 Smt. V. B. Nirmala Geethamba BC (B)59.5 16.4 75.9
07 1059 M. Lakshman BC (D)59 8.2 67.2
08 1176 BSV. Prakash Kumar SC 49 10 59
09 2336 Smt. Girija M. Priyadarshani SC 48 8.4 56.4
10 1220 N. Tukaramji ST 36.5 11.4 47.9

6. The Administrative Committee considered the report, the merit list and list of
recommended candidates proposed by the interview and by resolution dated 4-4-2006
approved the selection of the said ten candidates and directed the said 'select list' be
placed before the Full Court on 6-4-2006 for its consideration.
7. The Full Court considered the resolutions of the Administrative Committee dated 30-
11-2004 and 4-4-2006 and the record of selection. The Full Court impliedly approved the
resolution dated 30-11-2004. But it did not agree with the selection list prepared by the
Interview Committee and approved by the Administrative Committee by resolution dated
4-4-2006. The Full Court authorized the Chief Justice to constitute a Committee of
Judges for preparing a fresh list of candidates to be recommended for appointment of
District and Sessions Judges (Grade II). Accordingly, the Chief Justice appointed a Sub-
Committee of two Judges on 7-4-2006. The said Sub-Committee was of the view that the
candidates should be evaluated with reference to written examination marks of 75 and
interview marks of 25 as per the resolution dated 30-11-2004, instead of being evaluated
with reference to written examination marks of 100 and interview of 25, thereby varying
the prescribed ratio between written examination marks and interview marks from 3:1 to
4:1. Therefore, it scaled down the marks obtained by the candidates in the written
examination with reference to a total of 100, in proportion to a maximum marks of 75. By
adding the interview marks of 25, the total marks obtained by the candidates with
reference to a total of 100 (as against 125) were re-calculated. The Sub-Committee was
also of the view that apart from applying the minimum marks for the written examination
for determining the eligibility of the candidates to appear in the interview the same cut off
percentage should be applied for interview marks, and those who fail to secure such
minimum marks in the interview should be considered as having failed. As the minimum
percentage for passing the written examination was 50% for open category, 40% for
backward classes and 35% for SC and ST, only those candidates who secured the
minimum of 12.5 marks (open category), 10 marks (BC candidates) and 8.75 marks (SC
and ST candidates) were considered as having succeeded in the interview. Consequently,
only 31 candidates were found to have qualified both in the written examination and
interview and a revised merit list was prepared pruned down to 31 successful candidates.
On that basis, nine candidates were recommended for appointment as follows :
@page-SC1473
DISTRICT AND SESSIONS JUDGES GRADE-II EXAMINATION, 2005
(LIST OF CANDIDATES TO BE RECOMMENDED FOR APPOINTMENT)

Sl. No. Regn. Number Name of the candidate Category Marks in Written
Exam.(out of 75) Interview (out of 25) Total Marks (out of 100)
1 1775 Smt. C. Sumalatha OC 45.75 19.4 65.15
2 1117 Smt. G. Radharani OC 46.87 16 62.87
3 1694 A. Hari Haranatha Sarma OC 48.37 14.4 62.77
4 1590 Smt. V. B. Nirmala Geethamba (BC.B) OC (W) 44.62 16.4
61.02
5 1186 K. Sreenivas BC.D 38.25 12.6 50.85
6 1072 Smt. P. Manjula Devi BC.B(W) 33.75 13.2 46.95
7 1176 BSV. Prakash Kumar SC 36.75 10 46.75
8 1151 Smt. M. Renuka >BC.A(W) 30 14 44
9 1220 N. Tukaramji ST 27.37 11.4 38.77

One vacancy relating to 'Scheduled Caste (Women)' was left unfilled as there was no
qualified candidate.
8. The said report and the selection list were considered by the Full Court on 28-4-2006
and it was resolved to accept the names of the aforesaid nine candidates in the said list to
the State Government for appointment. The second list contained the names of 5 out of
10 candidates recommended in the first list (SI. Nos. 2, 4, 6, 8 and 10 in the first list were
SI. Nos. 1, 3, 4, 7 and 9 in the second list). Five candidates in the first list (SI. Nos. 1, 3,
5, 7 and 9 in the first list) got eliminated as they failed to secure the minimum marks in
the interview and four fresh candidates entered the second list (SI. Nos. 2, 5, 6 and 8 in
the second list). No candidate was selected under the category SC (Woman) as no
candidate of that category secured the minimum marks in the interview.
9. Two of the candidates whose names were found in the first list and who got excluded
in the second list namely K. Manjusri (SI. No. 3) and M. Lakshman (SI. No. 7) filed W.P.
Nos. 10061/2006 and 10062/2006 in the High Court praying for a declaration that the
action of the High Court in preparing the selection list by prescribing minimum
qualifying marks for the interview was arbitrary and illegal and seeking a direction to the
High Court to redraw the selection list without adopting minimum qualifying marks for
the interview. The said writ petitions were dismissed by the High Court by a common
judgment dated 30-10-2006.
Civil Appeal arising from SLP [C] No. 18330/2006
10. This appeal is by K. Manjusri whose name was found in the first list. She contended
that the minimum marks for interview not having been prescribed either under the rules
or by the resolution dated 30-11-2004 by the Administrative Committee, the action of the
Full Court altering the norms for selection by introducing minimum marks for interview,
after completion of the selection process, would amount to changing the rules of the
game, not only after the game was started but after the game was played.
11. Several applications for impleadment filed by the selected/non-selected candidates
have been ordered to be heard along with the main matter. IA No. 2 was filed by A.
Hariharanatha Sarma, N. Thukaramji, V. B. Nirmala Geethamba and BSV Prakash Kumar
whose names were found in both the first and second lists. IA Nos. 3 and 5 are filed by G.
Anupama Chakravarthy and P. Venkata Jyothirmal who were at SI. Nos. 5 and 1 in the
first list (whose names were omitted in the second list). IA No. 4 is filed by G. Radha
Rani, K. Sreenivas and M. Renuka whose names are found in the second list at SI. Nos.
2, 5, 6 and 8. They were also heard. While the applicants in IA Nos.
@page-SC1474
2, 3 and 5 have supported the contentions urged by the appellant, the applicants in IA No.
4 have contended to the contrary.
SLP [C] No. [CC Nos. 7188-89/2007] K. Manjusree v. State of A. P.
12. One E. Thirumala Devi whose name is found neither in the first list nor in the second
list has filed this SLP. She was not a party in the writ petition before the High Court and
has filed this SLP with an application seeking permission to file the SLP and for
condoning the delay of 192 days in filing the SLP. She has contended that applying the
criterion of minimum qualifying marks in the interview, without notifying the same to the
candidates was violative of principles of natural justice. She has contended that the
selection procedure was illegal and, therefore, the entire selection process should be
scrapped and High Court, should be directed to hold fresh selections.
Writ Petition [C] No. 51/2007
13. The petitioner Girija M. Priyadarsini, (whose name was in the first list of selected
candidates, under the category 'SC- Woman') has contended that minimum qualifying
marks could not be applied for interviews. She further contended that even if resolution
dated 30-11-2004 of the Administrative Committee is construed as prescribing minimum
marks for interview, such minimum marks would be applicable only in regard to open
category, backward classes and scheduled Tribes, but not to Scheduled Castes. She
submits that the resolution dated 30-11-2004 merely adopts what was prescribed earlier,
that is what was resolved earlier on 24-7-2001 and 21-2-2002. She points out the said
resolutions did not prescribe any minimum marks in respect of Scheduled Caste
candidates; and that therefore, she was entitled to be selected, to the post reserved for
'Scheduled Caste (Woman)'.
Writ Petition [C] No. 97/2007
14. The petitioner Kaki Shanti Kumar is a Scheduled Caste candidate. He was not one of
the selected candidates either in the first list or in the second list. According to him,
having regard to the policy of the State Government contained in the notifications dated
9-1-2004 and 17-2-2005, if any post earmarked for 'Scheduled Caste-Woman' cannot be
filled for want of suitable candidate, such post should be filled by a Scheduled Caste-
male candidates, by transferring the post to SC (General). He claims that the post left
unfilled earmarked form 'SC-Woman' should have been treated as 'SC-General' vacancy
and he ought to have been selected for that post.
Questions for consideration
15. On the contentions urged, the following questions arise for consideration :
(i) What was the procedure (method and manner of selection) prescribed by the
Administrative Committee for filling the posts advertised on 28.5.2004?
(ii)Whether the list prepared by the Interview Committee and approved by the
Administrative Committee suffered from any error, irregularity or illegality?
(iii)Whether the procedure adopted by the Full Court in preparing the fresh selection list
by applying the requirement of minimum marks for interview also, is legal and valid?
Re : Question (i)
16. The Rules did not prescribe any procedure for selection. When the posts were
advertised, the only criterion for selection that was mentioned was that the selection will
be by holding a written examination followed by an interview. The manner of holding
written examinations and interviews, the marks for written examination and interview,
whether the candidates should secure any minimum marks in the written examination
and/or interview, were all yet to be decided.
17. As per the practice followed by the High Court (Standing Orders referred to above)
the entire process of recruitment of District Judges was to be dealt with by the
Administrative Committee and the decisions of the Administrative Committee were
placed before the Full Court for its consideration and approval. The Administrative
Committee at its meeting held on 30-11-2004 considered the method and manner of
recruitment to be adopted in regard to the said recruitment and took the following three
decisions : (i) that the written examination will be held on 30-1-2005 simultaneously at
four centres; (ii) that the marks for the written examination shall be 75 and for oral
examination 25; and (iii) that the "minimum qualifying marks for OC/BC/SC/ST shall be
as prescribed earlier". The first two decisions are self-contained and clear. In regard to the
third decision, it becomes necessary to ascertain what was the minimum qualifying marks
for OC/BC/SC/ST which had been
@page-SC1475
prescribed earlier. There was no general prescription of guidelines or norms or criteria for
holding the written examination and interview marks therefor. The procedure to be
applied in regard to each recruitment was laid down separately by the Administrative
Committee as and when the recruitment was done. When the Administrative Committee
decided on 30-11-2004 that the minimum qualifying marks for OC/BC/SC/ST shall be as
prescribed earlier it obviously referred to what was prescribed when the previous
recruitment was made in 2001-2002. The High Court has produced the relevant minutes
relating to such earlier recruitment. It is seen that the Administrative Committee had laid
down the following method and manner for the recruitment of six posts of District and
Sessions Judges (Grade II) by its resolution dated 24-7-2001 (approved by the Full Court
on 16-8-2001) :
"Considered and resolved that the mode of examination be by way of written test for 75
marks and oral interview for 25 marks and the minimum qualifying marks for open
category is 50 marks, for Backward Classes (B.Cs) 40 marks and Scheduled Tribes
(S.Ts.) 35 marks in the written examination and the same ratio will apply for oral
interview also."
The minimum qualifying marks for the written examination was subsequently
amended/corrected by Administrative Committee at its meeting held on 21-2-2002
(approved by Full Court on 6-3-2002) as follows :
"Considered and resolved to correct the typographical error occurred in the resolution of
the Administrative Committee Meeting held on 24-7-2001 mentioning 50 marks, 40
marks and 35 marks instead of 50 per cent, 40 per cent. and 35 per cent i.e. the minimum
qualifying marks for Open Category is 50 percent., for Backward Classes (B.Cs.) 40 per
cent and Scheduled Tribes (S.Ts.) 35 per cent in the written examination."
18. Let us try to analyse and find out the combined effect of the resolutions dated 24-7-
2001 and 21-2-2002. The resolution dated 24-7-2001 prescribed the following marks for
the written examination and the interview :
(a) The marks for written examination was 75 marks and the minimum qualifying marks
was 50 marks for open category, 40 marks for backward classes and 35 marks for
Scheduled Tribes;
(b) The marks prescribed for interview was 25 marks and the minimum qualifying marks
for interview was 16.67 marks for open category, 13.33 marks for Backward Classes, and
11.67 marks for Scheduled Tribes (by applying the ratio that was prescribed for written
examination).
The resolution dated 24-7-2001 was amended on 21-2-2002 and it was decided to have
only minimum qualifying marks in the written test and not for the oral examination. This
is evident from the subject placed for consideration on 21-2-2002 and the resolution on
the subject. The subject for consideration was : "Minimum qualifying marks in the
written examination". The resolution stated that the minimum qualifying marks was 50%
for open category, 40% for Backward Classes and 35% for Scheduled Tribes in the
written examination. It did not prescribe any minimum for the interviews. Nor was it
understood as prescribing any minimum marks for the interview. That the Administrative
Committee and Full Court intended and in fact proceeded on the basis that there would be
no minimum marks for the interview is evident from the fact that in regard to recruitment
of 6 posts in 2001-2002, the minimum qualifying marks of 50%, 40% and 35% were
applied only for the written examination and no minimum qualifying marks were applied
in respect of interviews. We are informed that for the 2001-2002 selections, the procedure
adopted was that all candidates who passed the written examination by securing the
minimum marks were called for interview and the interview marks were added to the
written examination marks for the purpose of preparing the merit list and for the purpose
of selection. No minimum marks were applied for interview and no candidate was
excluded on the ground of not securing any minimum marks in the interview. It is also
not in dispute that even in the earlier selections (held prior to 2001-2002) the High Court
had applied minimum marks for interviews. Therefore the only inference is that when the
Administrative Committee resolved on 30-11-2004 that the minimum qualifying marks
for OC/BC/SC/ST shall be as prescribed earlier what it meant and provided was that there
will be minimum qualifying marks for the written examination only, that is 50% for OC,
40% for BC and 35% for
@page-SC1476
ST. It may however be mentioned that though minimum of 35% was prescribed only for
ST candidates in regard to 2001-2002 selections, that percentage was adopted and applied
in the written examination for both SC and ST candidates by the resolution dated 30-11-
2004.
19. The Administrative Committee of the High Court (Chief Justice and five Senior
Judges) as also the Interview Committee consisting of five Judges (the Chief Justice and
four other Judges) all along intended, understood and proceeded on the basis with
reference to the current selection that minimum percentage was applicable only to written
examination and not for interviews. This is evident from the manner in which interviews
were conducted and merit list and selection list were prepared by the Interview
Committee and approved by the Administrative Committee. This shows that the
Interview Committee conducted the interviews on 13th, 14th, 16th, 17th, 18th, 20th, 24th
and 31st of March, 2006 on the understanding that there were no minimum marks for
interviews, that the marks awarded by them in the interview will not by itself have the
effect of excluding or ousting any candidate from being selected, and that marks awarded
by them in the interviews will merely be added to the written examination marks, for
preparation of the merit list and selection. We are referring to this aspect, as the manner
of conducting interviews and awarding marks in interviews, by the five members of the
Interviewing Committee would have been markedly different if they had to proceed on
the basis that there were minimum marks to be secured in the interview for being
considered for selection and that the marks awarded by them would have the effect of
barring or ousting any candidate from being considered for selection. Thus, the entire
process of selection - from the stage of holding the examination, holding interviews and
finalizing the list of candidates to be selected - was done by the Selection Committee on
the basis that there was no minimum marks for interview. To put it differently the game
was played under the rule that there was no minimum marks for the interview.
20. Shri P. P. Rao, learned senior counsel appearing on behalf of the High Court
submitted that the Resolution dated 21-2-2002 merely corrected a typographical error in
the Resolution dated 24-7-2001, regarding minimum marks relating to written
examination, and the last portion of the Resolution dated 24-7-2001, relating to
interviews, (that is, the portion reading "and the same ratio will apply for oral interview
also") remained unaltered. According to him, when the Administrative Committee passed
the Resolution dated 21-2-2002 in regard to the earlier selection and again passed the
resolution dated 30-11-2004 in regard to the current selection, to conduct the examination
with minimum qualifying marks as prescribed earlier, the intention was to have minimum
marks both for written examination and the interview. We have already examined the
resolutions dated 24-7-2001 and 21-2-2002 and held that the combined effect was to
apply minimum percentage to only written examination and not for the interview.
However, to test the correctness of his contention, we asked the learned counsel for the
High Court to explain why the 2001-2002 selections were done without applying
minimum marks for interview. He was not in a position to explain why the 2001-2002
selections were made without applying any minimum marks for the interviews, if the
resolutions dated 24-7-2001 and 21-2-2002 had really provided that there should be a
minimum marks for the interview. The only explanation was that it was due to some
oversight or mistake. The said explanation is neither satisfactory nor valid.
Re : Question (ii)
21. The merit list and selection list prepared by the Interview Committee and approved by
the Administrative Committee, on the basis that there was no minimum marks for
interview, however, contained one error. The inter se merit of the candidates were
prepared with reference to a total of 125 marks, comprising 100 for the written
examination and 25 for the interview. But the Administrative Committee had clearly
resolved on 30-11-2004 that evaluation of performance should be with reference to a
maximum marks of 75 for written examination and 25 for interview. The written
examination was however, conducted with reference to a question paper set for a
maximum of 100 marks. The interviews, of course, were held with reference to maximum
of 25 marks. Therefore, it was necessary to scale down the marks secured by the
candidates in the written examination (with reference to a maximum of 100 marks)
proportionately to arrive at the marks with reference
@page-SC1477
to a maximum of 75 marks so that the ratio of maximum marks in written examination
and interview would be 3 : 1. If the maximum marks for the written examination was 100
and for the interview was 25, then the ratio between the marks for written examination
and interview would be 4 : 1, thereby altering the prescribed marks, after the selection
process had begun. We are, therefore, of the view that the first list requested an
arithmetical correction, that is, scaling down of the written examination marks to three-
fourth of what was secured by them with reference to a maximum of 100 marks, so that
the ratio of 3 : 1 could be maintained in respect of the marks for written examination and
interviews.
Re : Question (iii)
22. When the Administrative Committee placed the merit lists and Selection List before
Full Court, apparently objections were raised on two grounds. One related to the failure
to provide the minimum of 50%, 40% and 35% marks for interviews, on the
interpretation of resolution dated 30-11-2004 read with earlier resolutions dated 24-7-
2001 and 21-2-2002. The second objection was that even though the Administrative
Committee had resolved that the marks for written examination would be 75 and
interview would be 25, at the time of tabulating the marks, the marks secured (out of 100
marks) in the written examination had been taken into account without scaling it down
with reference to a maximum of 75 marks. The Full Court therefore, appointed a Sub-
Committee of two Judges to examine the matter and prepare a fresh merit list and
selection list. The Sub-Committee examined the matter and submitted a revised merit list
by incorporating two changes. Firstly, while tabulating the marks, it scaled down the
marks secured by the candidates in the written examination with reference to a maximum
of 100 marks, in proportion to a maximum of 75 marks so that the final marks were with
reference to a base of 75 marks for written examination and 25 marks for interview as
resolved on 30-11-2004. Secondly, it applied the minimum percentage of 50%, 40% and
35% for OC, BC, SC/ST even in regard to interviews and consequently, eliminated those
who secured less than the minimum in the interview from the process of selection. The
final selection list was prepared with reference to the fresh merit list prepared by
incorporating the said two changes.
23. As far as the first change is concerned, we have already held that scaling down is
unexceptional as it is in consonance with the criteria decided by the Administrative
Committee on 30-11-2004 before commencing the selection process.
24

. But what could not have been done was the second change, by introduction of the
criterion of minimum marks for the interview. The minimum marks for interview had
never been adopted by the Andhra Pradesh High Court earlier for selection of District and
Sessions Judges, (Grade II). In regard to the present selection, the Administrative
Committee merely adopted the previous procedure in vogue. The previous procedure as
stated above was to apply minimum marks only for written examination and not for the
oral examination. We have referred to the proper interpretation of the earlier resolutions
dated 24-7-2001 and 21-2-2002 and held that what was adopted on 30-11-2004 was only
minimum marks for written examination and not for the interviews. Therefore,
introduction of the requirement of minimum marks for interview, after the entire selection
process (consisting of written examination and interview) was completed, would amount
to changing the rules of the game after the game was played which is clearly
impermissible. We are fortified in this view by several decisions of this Court. It is
sufficient to refer to three of them P. K. Ramachandra Iyer v. Union of India 1984 (2)
SCC 141, Umesh Chandra Shukla v. Union of India 1985 (3) SCC 721, and Durgacharan
Misra v. State of Orissa 1987 (4) SCC 646. AIR 1984 SC 541
AIR 1985 SC 1351
AIR 1987 SC 2267

25

. In Ramachandra Iyer (supra), this Court was considering the validity of a selection
process under the ICAR Rules, 1977 which provided for minimum marks only in the
written examination and did not envisage obtaining minimum marks in the interview. But
the Recruitment Board (ASRB) prescribed a further qualification of obtaining minimum
marks in the interview also. This Court observed that the power to prescribe minimum
marks in the interview should be explicit and cannot be read by implication for the
obvious reason that such deviation from the rules is likely to cause irreparable and
irreversible harm. This Court held that as there was no power under the rules for the
Selection Board to prescribe the additional AIR 1984 SC 541, Para 44

@page-SC1478
qualification of securing minimum marks in the interview, the restriction was
impermissible and had a direct impact on the merit list because the merit list was to be
prepared according to the aggregate marks obtained by the candidates at written test and
interview. This Court observed :
"Once an additional qualification of obtaining minimum marks at the viva voce test is
adhered to, a candidate who may figure high up in the merit list was likely to be rejected
on the ground that he has not obtaining minimum qualifying marks at viva voce test. To
illustrate, a candidate who has obtained 400 marks at the written test and obtained 38
marks at the viva voce test, if considered on the aggregate of marks being 438 was likely
to come within the zone of selection, but would be eliminated by the ASRB on the
ground that he has not obtained qualifying marks at viva voce test. This was
impermissible and contrary to rules and the merit list prepared in contravention of rules
cannot be sustained.
26

. In Umesh Chandra (supra), the scope of the Delhi Judicial Service Rules, 1970 came up
for consideration. The rules provided that those who secured the prescribed minimum
qualifying marks in the written examination will be called for viva voce; and that the
marks obtained in the viva voce shall be added to the marks obtained in the written test
and the candidates ranking shall depend on the aggregate of both 27 candidates were
found eligible to appear for viva voce on the basis of their having secured the minimum
prescribed marks in the written examination. The final list was therefore, expected to be
prepared by merely adding the viva voce marks to the written examination marks in
regard to those 27 candidates. But the final list that was prepared contained some new
names which were not in the list of 27 candidates who passed the written examination.
Some names were omitted from the list of 27 candidates who passed the written
examination. It was found that the Selection Committee had moderated the written
examination marks by an addition of 2% for all the candidates, as a result of which some
candidates who did not get through the written examination, became eligible for viva
voce and came into the list. Secondly, the Selection Committee prescribed for selection, a
minimum aggregate of 600 marks in the written examination and viva voce which was
not provided in the Rules and that resulted in some of the names in the list of 27 being
omitted. This Court held neither was permissible. Dealing with the prescription of
minimum 600 marks in the aggregate this Court observed : AIR 1985 SC 1351

There is no power reserved under Rule 18 of the Rules for the High Court to fix its own
minimum marks in order to include candidates in the final list. It is stated in paragraph 7
of the counter-affidavit filed in Writ Petition 4363 of 1985 that the Selection Committee
has inherent power to select candidates who according to it are suitable for appointment
by prescribing the minimum marks which a candidate should obtain in the aggregate in
order to get into the Delhi Judicial Service . . . . But on going through the Rules, we are
of the view that no fresh disqualification or bar may be created by the High Court or the
Selection Committee merely on the basis of the marks obtained at the examination
because clause (6) of the Appendix itself has laid down the minimum marks which a
candidate should obtain in the written papers or in the aggregate in order to qualify
himself to become a member of the Judicial Service. The prescription of the minimum of
600 marks in the aggregate by the Selection Committee as an addition requirement which
the candidate has to satisfy amounts to an amendment of what is prescribed by clause (6)
of the Appendix ...... We are of the view that the Selection Committee has no power to
prescribe the minimum marks which a candidate should obtain in the aggregate different
from the minimum already prescribed by the Rules in its Appendix. We are, therefore, of
the view that the exclusion of the names of certain candidates, who had not secured 600
marks in the aggregate including marks obtained at the viva voce test from the list
prepared under Rule 18 of the Rules is not legal."
27

. In Durgacharan Misra (supra), this Court was considering the selection under the Orissa
Service Rules which did not prescribe any minimum qualifying marks to be secured in
viva voce for selection of Munsifs. The rules merely required that after the viva voce test
the State Public Service Commission shall add the marks of the viva voce test to the
marks in the written test. But the State Public Service Commission which AIR 1987 SC
2267

@page-SC1479
was the selecting authority prescribed minimum qualifying marks for the viva voce test
also. This Court held that the Commission had no power to prescribe the minimum
standard at viva voce test for determining the suitability of candidates for appointment of
Munsifs.
28

. In Maharashtra State Road Transport Corporation v. Rajendra Bhimrao Mandve 2001


(10) SCC 51, this Court observed that the rules of the game, meaning thereby, that the
criteria for selection cannot be altered by the authorities concerned in the middle or after
the process of selection has commenced. In this case the position is much more serious.
Here, not only the rules of the game were changed, but they were changed after the game
has been played and the results of the game were being awaited. That is unacceptable and
impermissible.2001 AIR SCW 4885

29. The resolution dated 30-11-2004 merely adopted the procedure prescribed earlier. The
previous procedure was not to have any minimum marks for interview. Therefore,
extending the minimum marks prescribed for written examination, to interviews, in the
selection process is impermissible. We may clarify that prescription of minimum marks
for any interview is not illegal. We have no doubt that the authority making rules
regulating the selection, can prescribe by rules, the minimum marks both for written
examination and interviews, or prescribe minimum marks for written examination but not
for interview, or may not prescribe any minimum marks for either written examination or
interview. Where the rules do not prescribe any procedure, the Selection Committee may
also prescribe the minimum marks, as stated above. But if the Selection Committee want
to prescribe minimum marks for interview, it should do so before the commencement of
selection process. If the selection committee prescribed minimum marks only for the
written examination, before the commencement of selection process, it cannot either
during the selection process or after the selection process, add an additional requirement
that the candidates should also secure minimum marks in the interview. What we have
found to be illegal, is changing the criteria after completion of the selection process,
when the entire selection proceeded on the basis that there will be no minimum marks for
the interview.
30. It was submitted that Administrative Committee and Interview Committee were only
delegates of the Full Court and the Full Court has the absolute power to determine or
regulate the process of selection and it has also the power and authority to modify the
decisions of the Administrative Committee. There can be no doubt about the proposition.
The Administrative Committee being only a delegate of the Full Court, all decisions and
resolutions of Administrative Committee are placed before the Full Court for its approval
and the Full Court may approve, modify or reverse any decision of the Administrative
Committee. For example when the resolution dated 30-11-2004 was passed it was open to
the Full Court, before the process of selection began, to either specifically introduce a
provision that there should be minimum marks for interviews, or prescribe a different
ratio of marks instead of 75 for written examination and 25 for interview, or even delete
the entire requirement of minimum marks even for the written examination. But that was
not done. The Full Court allowed the Administrative Committee to determine the method
and manner of selection and also allowed it to conduct the examination and interviews
with reference to the method and manner determined by the Administrative Committee.
Once the selection process was completed with reference to the criteria adopted by the
Administrative Committee and the results were placed before it, the Full Court did not
find fault with the criteria decided by the Administrative Committee (as per resolution
dated 30-11-2004) or the process of examinations and interviews conducted by the
Administrative Committee and Interview Committee. If the Full Court had found that the
procedure adopted in the examinations or interviews was contrary to the procedure
prescribed, the Full Court could have set aside the entire process of selection and directed
the Administrative Committee to conduct a fresh selection. The resolution dated 30-11-
2004 was approved. It did not find any irregularity in the examination conducted by the
Administrative Committee or the interviews held by the Selection Committee. The
assessment of performance in the written test by the candidates was not disturbed. The
assessment of performance in the interview by the Selection Committee was not
disturbed. The Full Court however, introduced a new requirement as to minimum
@page-SC1480
marks in the Interview by an interpretative process which is not warranted and which is at
variance with the interpretation adopted while implementing the current selection process
and the earlier selections. As the Full Court approved the resolution dated 30-11-2004 of
the Administrative Committee and also decided to retain the entire process of selection
consisting of written examination and interviews it could not have introduced a new
requirement of minimum marks in interviews, which had the effect of eliminating
candidates, who would otherwise be eligible and suitable for selection. Therefore, we
hold that the action of Full Court in revising the merit list by adopting a minimum
percentage of marks for interviews was impermissible.
31. The Division Bench of the High Court while considering the validity of the second
list, has completely missed this aspect of the matter. It has proceeded on an erroneous
assumption that the resolution dated 30-11-2004 of the Administrative Committee
prescribed minimum marks for interviews. Consequently, it erroneously held that the
Administrative Committee had acted contrary to its own resolution dated 30-11-2004 in
not excluding candidates who had not secured the minimum marks in the interview and
that the Full Court had merely corrected the wrong action of the Administrative
Committee by drawing up the revised merit list by applying marks for interview also. The
decision of the Division Bench therefore, cannot be sustained.
CONCLUSION
32. We therefore, find that the judgment of the Division Bench of the High Court has to
be set aside with a direction to the AP High Court to redraw the merit list without
applying any minimum marks for interview. The merit list will have to be prepared in
regard to 83 candidates by adding the marks secured in written examination and the
marks secured in the interview. Thereafter, separate lists have to be prepared for each
reservation category and then the final selection of 10 candidates will have to be made.
The scaling down of the written examination marks with reference to 75 instead of 100 is
however, proper.
33. In view of our said decision, WP(C) No. 51/2007 and WP(C) No. 97/2007 do not
survive for consideration. As a candidate is available under the category of SC (Woman)
and she will be selected, the question of considering whether that post should be
transferred to SC (General) does not arise.
34. The SLP by Thirumala Devi is not maintained. She was not a selected candidate,
either in the first list or second list. She did not challenge the process of selection by
filing a writ petition. She was not a party to the writ petitions. She is in no way aggrieved
as she will not be selected, by adopting either method. There is also a delay of 190 days.
Therefore, the said SLP is liable to be rejected on the ground of delay and on the ground
it is not maintainable.
35. In view of the above, we dispose of the matter as follows :
(i) The application for impleadment (IAs 2, 3, 4 and 5 filed in SLP(C) No. 18330/2006)
are allowed.
(ii) The civil appeal filed by K. Manjusree is allowed and the judgment of the High Court
is set aside. The High Court is directed to prepare a fresh merit list in regard to 83
candidates with reference to their marks in written test and interview without applying
any minimum marks for interviews and thereafter finalise the selections in accordance
with law.
(iii) The appointments of five candidates in pursuance of our interim order need not be
disturbed. The said five candidates will find a place in the selection list even when it is
redone, though their ranks/reservation category may vary. Their rank and seniority will
depend upon the fresh selection list of ten candidates to be drawn and not on the
appointment made in pursuance of the interim order.
(iv) WP(C) No. 51/2007 and WP(C) No. 97/ 2007 are dismissed.
(v) The application for permission to file SLP by Thirumala Devi is rejected. As a
consequence SLP (CC) No. 7188-79/2007 is rejected.
Order accordingly.
AIR 2008 SUPREME COURT 1480 "Gowdara Nanjappa v. Matada Basaiah"
(From : 2000 (4) Kant LJ 357)
Coram : 2 Dr. A. PASAYAT AND P. SATHASIVAM, JJ.
Civil Appeal No. 8060 of 2001, D/- 15 -2 -2008.
Gowdara Nanjappa v. Matada Basaiah and Ors.
Karnataka Land Reforms Act (10 of 1962), S.4 - LAND REFORMS - TRIBUNALS -
RECORD-OF-RIGHTS -
@page-SC1481
Occupancy rights - Grant of - Revenue (Pahni) extract proved tenancy of appellant and
possession as on relevant date - Presumption to be drawn from such record of rights
cannot be bypassed in absence of any material - Spot inspection made by Land Tribunal
having no relevance to decide basic issues - Cannot be relied upon - Grant of occupancy
right in favour of appellant - Was proper.
2000 (4) Kant LJ 357, Reversed. (Paras 5, 7)
Cases Referred : Chronological Paras
ILR ( 1992) Kant 1827 2
R.S. Hegde, Chandra Prakash, P.P. Singh, for Appellant; M. Gireesh Kumar, S. K.
Kulkarni, Kh. Nobin Singh, Vikrant Yadav, Amit Kr. Chawla, Sanjay R. Hegde, for
Respondents.
Judgement
1. Dr. ARIJIT PASAYAT, J. :- Challenge in this appeal is to the judgment of a learned
Single Judge of the Karnataka High Court allowing the revision filed by respondent No. 1
under Section 121(A) of the Karnataka Land Reforms Act, 1961 (in short the 'Act'). In the
revision petition challenge was to the order passed by the Land Reforms Appellate
Tribunal, Shimoga (in short the 'Appellate Tribunal'). By the impugned order before the
High Court the Appellate Tribunal had set aside the order passed by the Land Tribunal,
Shimoga Taluk (in short the 'Tribunal').
Background facts in a nutshell are as follows :
The lands in Sy. Nos. 3, 6/2, 20 and 41/ 2 situated at Venkatapura village are the Inam
lands endowed to Sri Kudli Rameshwara Devaru. In respect of the said lands, the
applicants-Sri Subbaraya, Gowdara Nanjappa, Matada Basaiah and Smt. Vrundamma
filed applications for grant of occupancy rights. Subbaraya filed Form No. 1 claiming to
be an Inamdar to the entire extent of 27 acres 29 guntas in the above survey-numbers.
Gowdara Nanjappa also filed Form No. 1 claiming 2 acres 6 guntas in Sy. No. 41/2 as a
tenant under the temple. The present petitioner Matada Basaiah also filed application for
grant of occupancy right as tenant under Subbaraya in respect of the land measuring 2
acre 6 guntas. Another person Manjappa, husband of Vrundamma also filed an
application for grant of occupancy right in Sy. No. 41/2 to the extent of 1 acre 20 guntas.
The Land Tribunal by its order dated 11-9-1981 granted occupancy right in favour of
Gowdara Nanjappa to the extent of 2 acres 6 guntas and in respect of other applicants
who are not parties in this petition. The said order was questioned by the present
petitioner in W.P. No. 17043/83 before this Court. This Court, in so far it relates to Sy.
No. 41/2 quashed the order of the Land Tribunal and remitted back the matter for fresh
disposal in accordance with law. The Land Tribunal took up the matter for consideration
by permitting the parties to lead evidence, recorded the evidence of Manjunatha,
Subbaraya, Gowdara Nanjappa, Vrundamna and the evidence of the petitioner Matada
Basaiah. After considering the report and the entries made in the property and income of
Muzrai Institutions maintained in the Taluk Office and also the entries found in the quit
rent register, the Land Tribunal by its order dated 17-4-1986 granted occupancy rights in
favour of Matada Basaiah to an extent of 2 acres 26 guntas and an extent of 1 acre 20
guntas in favour of Smt. Vrundamma. The order of the Land Tribunal was questioned by
Gowdara Nanjappa, who is respondent No. 1 in W.P. No. 9587/86 before the High Court.
The High Court by an order dated 29-9-1986 transmitted the records to the Appellate
Authority, Shimoga in view of the amendment and the same was registered as LRA (W)
No. 749/86 before the Land Reforms Appellate Authority. The Appellate Authority, by its
order dated 27-10-1988 allowed the appeal of Gowdara Nanjappa setting aside the order
of the Land Tribunal dated 17-4-1986 in so far as it relates to an extent of 2 acres 26
guntas which was conferred, in favour of Matada Basaiah. Being aggrieved of the order
of the Appellate Authority, the petitioner who is a rival tenant has come up with this
revision.
2. Respondent No. 1's stand before the High Court was that Inamdar Subbaraya,
respondent No. 1 was a tenant in respect of land measuring 2 acres 26 guntas in Survey
No. 41/2A. It was held that the Tribunal had rightly granted occupancy right in his favour.
Stand of the present appellant before the High Court was that occupancy has been granted
based on the entries in the R.T.C. extract and presumption arises regarding the possession
and cultivation. Therefore, the Appellate Tribunal had rightly interfered with the order of
the Tribunal. The
@page-SC1482
High Court formulated two issues for consideration :
1. Whether the jurisdiction exercised by the Land Tribunal in so far as the adjudication of
the matter involving the Inam lands prior to rendering of the judgment in Shri Kudli
Sringeri Maha Samsthanam v. State of Karnataka reported in ILR 1992 Kar 1827 dated
24-4-1992 is bad and thereby the matter requires to be remanded to the Special Deputy
Commissioner for adjudication?
2. Whether the Appellate Authority is justified in interfering with the finding of the Land
Tribunal setting aside the grant of occupancy right in favour of the petitioner Matada
Basaiah?
3. It appears that the High Court did not accept the presumptive value of the entries made
in the R.T.C. extract but relied upon certain spot inspection made by the Tribunal.
4. Learned counsel for the appellant submits that the approach is clearly erroneous. It is
pointed out that the appellant appears as a tenant in the RTC record.
5. Learned counsel for the respondents supported the High Court's order. Initially claim
of respondent No. 1 was to be a lessee and an application was filed which was rejected.
Subsequently he claimed that he was a tenant. The High Court relied only on Subbaraya's
evidence. It failed to notice that initially the name of respondents and/or Subbaraya
appeared in the R.T.C. There was no material before the High Court to by-pass the
presumption to be drawn from the record of rights. The High Court appears to have
placed emphasis on the spot inspection made by the Tribunal. Unfortunately the High
Court overlooked the fact that the spot inspection was made in 1986 and such spot
inspection did not have a relevance to decide the basic issues. The revenue entries related
to past period and were spread over a number of years and continued till appointed date.
A finding recorded by the appellate authority has also relevance :
". . It is significant to note that the name of Subbaraya who claims himself to be the
landlord of the suit land does not appear in the pahanies and RTC extracts of the suit land
at any time either as owner or as a tenant. As discussed above, Rameswara Devaru Deity
is admittedly Khatedar of the suit land and there is no material on record to show that the
4th respondent Subbaraya was authorized to lease the suit land on behalf of Rameswara
Devaru Deity. Therefore the Land Tribunal was not justified in conferring the occupancy
rights in favour of 3rd respondent Matala Basaiah to an extent of 2 acres 26 guntas on the
version of Subbaraya who himself had claimed occupancy rights of the suit land, and
who had no authority to lease the lands on behalf of Rameswara Devaru Deity."
6. The High Court also failed to notice that Subbaraya himself has no right or title or
interest in the land and his application for grant of occupancy right was rejected. There
was no evidence of the respondent No. 1 being a tenant in lawful possession of the land.
The Pahani Extract proved tenancy of the appellant and possession as 1-3-1974 which is
the relevant date for consideration.
7. Above being the position, the impugned order of the High Court is clearly
unsustainable and is set aside, and the order passed by the Appellate Tribunal stands
restored.
8. Appeal is allowed without any order as to costs.
Appeal allowed.
AIR 2008 SUPREME COURT 1482 "State Bank of Patiala v. Manjeet"
(From : Punjab and Haryana)*
Coram : 2 Dr. A. PASAYAT AND P. SATHASIVAM, JJ.
Civil Appeal No. 1319 of 2008 (arising out of SLP (C) No. 775 of 2006), D/- 15 -2 -2008.
State Bank of Patiala v. Manjeet.
State Bank of India (Subsidiary Banks) Act (38 of 1959), S.63(2) - State Bank of Patiala
(Employees) Pension Regulations (1995), Regn.3, Regn.40(3) - BANKING - FAMILY
PENSION SCHEME - Family pension - Option to be exercised within 120 days from
notified date - Respondent, dependent of deceased, failed to exercise option within time -
It cannot be said that there was no intimation to respondent about exercise of option as
her mother was serving in bank and in fact she had exercised option for her own pension
and not for family pension - In facts and circumstances order directing
@page-SC1483
grant of pension to respondent - Was illegal.
2006 Lab IC (NOC) 11 (Punj and Har), Reversed. (Para 10)
Cases Referred : Chronological Paras
2005 AIR SCW 3664 : AIR 2005 SC 3134 : 2005 Lab IC 3097 (Ref.) 8
1994 AIR SCW 4552 : AIR 1995 SC 360 (Ref.) 8
AIR 1987 SC 1059 (Ref.) 8
Vishnu Mehra, B.K. Satija, for Appellant; Sunil Atri, Chander Shekhar Ashri, for
Respondent.
* C. W. P. No. 19475 of 2003, D/- 15-9-2005, reported in 2006 (1) Cur LR 91 : 2006 Lab
IC (NOC) 11 (P and H).
Judgement
1. Dr. ARIJIT PASAYAT, J. :- Leave granted.
2. Challenge in this appeal is to the judgment of a Division Bench of the Punjab and
Haryana High Court allowing the writ petition filed by the respondent and holding her to
be entitled to grant of family pension as per the provisions of State Bank of Patiala
(Employees) Pension Regulations, 1995 (in short the 'Regulations').
Background facts in a nutshell are as follows :
Late Jai Singh, father of the respondent joined service in the appellant-Bank on 19-11-
1985. Few months after i.e. on 4-6-1986 he was killed while on duty. On 11-9-1986 and
1-10-1986 Jai Singh's widow Smt. Birmati-mother of the respondent was paid gratuity
and provident fund of late Jai Singh. On 29-12-1986 the aforesaid Smt. Birmati was
given appointment in the appellant-Bank as Record Keeper-cum-Godown Keeper on
compassionate grounds. On 23-3-1996 in exercise of power conferred by sub-section (1)
of Clause (O) of sub-section (2) of Section 63 of the State Bank of India (Subsidiary
Banks) Act, 1959 (in short the 'Act') the Regulations were framed. The Regulations
provide for establishment and maintenance of pension funds for the benefit of the
employees of the State Bank of Patiala. The regulations were published in the Official
Gazette on 23-3-1996 and were operative w.e.f. 29-9-1995.
On 12-8-2003 respondent attained majority. On 16-9-2003 she applied for family pension
of late Jai Singh. On 1-10-2003 the appellant-Bank rejected the claim for family pension
on several grounds; (i) the family pension was payable to the widow till the death or her
remarriage and (ii) the option for pension was required to be made by eligible dependent
of the deceased employee within 120 days from the notified date i.e. on or before 20-7-
1996. Another representation was made on 28-10-2003. Again on 11-11-2003 the claim
was rejected stating that since her mother was alive only she was eligible for grant of
family pension provided she had completed the required formalities within the prescribed
period.
3. A writ petition was filed for a direction to the appellant-Bank to give family pension to
the respondent. The Division Bench, as noted above, allowed the writ petition holding
that family pension was illegally denied to her.
4. In support of the appeal, learned counsel for the appellant-Bank submitted that
Regulation 3 of the Regulations deals with cases of employees who had already died. It
was also submitted that in terms of Regulation 40 (3) where family pension is granted
under the regulation to a minor, it shall be payable to the guardian of the minor. At no
point of time, not even in the representation, respondent had indicated about the alleged
re-marriage of Birmati. Only for the first time such a stand had been taken in the writ
petition. Furthermore, the respondent admitted that she was living with Smt. Birmati.
5. Learned counsel for the respondent on the other hand supported the judgment of the
High Court.
Regulation 3 reads as follows : "(1) To exercise an option in writing within one hundred
and twenty days from the notified date to become member of the Fund; and
(2) To refund within sixty days after the expiry of the said period of one hundred and
twenty days specified in Clause (B) the entire amount of the Bank's contribution to the
Provident Fund including interest accrued thereon together with a further simple interest
at the rate of six per cent., per annum on the said amount from the date of settlement of
the Provident Fund account till the date of refund of the aforesaid amount to the Bank."
6. Regulation 40(3) is also relevant and reference has already been made to that
provision. In terms of Regulation 3 the option was required to be exercised within a
period of 120 days from the notified date and there was a requirement of refunding
@page-SC1484
the contribution within 60 days after the aforesaid 120 days period.
7. Respondent's mother opted for her own pension and not for family pension.
Regulation 40(3) reads as follows :
"Where family pension is granted under this regulation to a minor, it shall be payable to
the guardian on behalf of the minor"
8
. In Jai Singh B. Chauhan and Ors. Punjab National Bank and Ors. (2005 (6) SCC 262), it
was observed as follows : 2005 AIR SCW 3664

"6. For the purpose of adjudicating the dispute few provisions in the Regulations need to
be noted.
7. "Notified Date" is defined in Regulation 2 as follows :
"notified date" means the date on which these regulations are published in the official
Gazette;"
8. In terms of Regulation 1, the Regulations were deemed to have come into force on the
date of their publication in the Official Gazette.
9. Regulation 3, so far as relevant reads as follows :
"3. These regulations shall apply to employees who, -
xxx xxx xxx
(3) (a) are in the service of the Bank before the notified date and continue to be in the
service of the Bank on or after the notified date; and
(b) exercise an option in writing within one hundred and twenty days from the notified
date to become member of the Fund; and
(c) authorize the trust of the Provident Fund of the Bank to transfer the entire contribution
of the Bank along with the interest accrued thereon to the credit of the Fund constituted
for the purpose under Regulation 5."
10. As per Regulation 3 (3)(b) option was to be exercised in writing within one hundred
and twenty days from the notified date to become member of the fund.
11. Regulation 3 (3)(c) is also of considerable importance. It required transfer of the
entire contribution of the Bank along with interest accrued thereon to the credit of the
fund constituted for the purpose under Regulation 5, and authorized trust of from the
amount of the Provident Fund of the Bank to effect the transfer.
xx xx xx

14. In M/s. Pankaj Jain Agencies v. Union of India and others (1994 (5) SCC 198) a
three-Judge Bench of this Court held as follows : 1994 AIR SCW 4552, Para 9

"17. In the present case indisputably the mode of publication prescribed by Section 25(1)
was complied with. The notification was published in the official Gazette on the 13-2-
1986. As to the effect of the publication in the official Gazette, this Court held
(Srinivasan case 1987 (1) SCC 658, 672 : AIR 1987 SC 1059, 1067) :
"Where the parent statute is silent, but the subordinate legislation itself prescribes the
manner of publication, such a mode of publication may be sufficient, if reasonable. If the
subordinate legislation does not prescribe the mode of publication or if the subordinate
legislation prescribes a plainly unreasonable mode of publication, it will take effect only
when it is published through the customarily recognized official channel, namely, the
Official Gazette or some other reasonable mode of publication.

18. We, therefore, see no substance in the contention that notwithstanding the publication
in the Official Gazette there was yet a failure to make the law known and that, therefore,
the notification did not acquire the elements of operativeness and enforceability. This
contention of Shri Ganesh is unacceptable."
9. The High Court was not justified in saying that there was no intimation to the
respondent about the exercise of option. Factually also it is not correct. Respondent's
mother was serving in the bank and in fact had exercised the option for her own pension
and not for family pension.
10. Above being the position, the High Court was not justified in directing grant of family
pension to the respondent. In view of above, the impugned judgment of the High Court is
set aside. The appeal is allowed. There will be no order as to costs.
Appeal allowed.
@page-SC1485
AIR 2008 SUPREME COURT 1485 "Life Insurance Corpn. of India v. Retired L.I.C.
Officers Assocn."
(From : Kerala)
Coram : 2 S. B. SINHA AND H. S. BEDI, JJ.
Civil Appeal No. 1289 of 2007, D/- 12 -2 -2008.
Life Insurance Corporation of India and Ors. v. Retired L.I.C. Officers Association and
Ors.
(A) Constitution of India, Art.245 - Life Insurance Corporation Act (31 of 1956), S.49 -
Life Insurance Corporation of India (Staff) Regulations (1960), Regn.51, Regn.77 -
LEGISLATION - LIFE INSURANCE CORPORATION - DELEGATION OF POWER -
PAY SCALE - Delegation of powers - Delegatee - Extent of powers - Revised pay scales
of Corporation employees - Powers of Chairman to fix method of fixation, eligibility and
cut off date for revised pay - Cannot and does not extend to fixation of gratuity - Words
'and other matters connected therewith or incidental thereto' - Connotation.
Chap. IV of the Regulations deal with 'pay and allowances'. Regn. 51 in Chap. IV confers
power upon the Chairman to fix a date from which the revision in pay shall apply. It
applies to pay, dearness allowance and other allowances applicable to the employees of
the Corporation. Regn. 51(1) postulates grant of pay, dearness allowance and other
allowances in the manner as prescribed in the IInd Schedule. The basic pay and other
allowances to Class II employees are regulated under the provisions contained in
Schedule III thereof. Revision of pay, dearness allowance and other allowances
applicable to the employees of the Corporation stricto sensu are not covered by Cl. (2) of
Regn. 51. It merely states that when a revision takes place pursuant to or in furtherance of
any award, agreement or settlement or otherwise, the Chairman of the Corporation will
have the jurisdiction in regard to :- (a) The method of fixation of pay in the new scales;
(b) The eligibility for the benefit of revision; and (c) the date from which the revision
shall apply. Regn. 55(2) has to be strictly construed method of fixation, eligibility for the
benefit of revision and the date from which the revisions shall apply are thus, the only
areas within which the Chairman can exercise jurisdiction. The effect of revision of pay
scales on other spheres and which are otherwise governed by another statute or other
provisions of the said Regulations would not come within the purview thereof. The
terminology in Regn. 55(2) 'and other matters connected therewith or incidental thereto'
must, therefore, be held to have a direct nexus with any one of the aforementioned three
elements. The same has nothing to do with the construction of any other provision of the
Regulations. The words 'incidental to' cannot be interpreted too broadly. (Paras 17, 18,
19)
The power of Chairman under Regn. 51(2) cannot therefore, extend to reduction of
gratuity payable to employees as per Regn. 77 in Chap. VII. (Paras 24, 26)
A delegatee must exercise its powers within the Four Corners of the statute. The power of
a sub-delegatee is more restricted. A delegatee cannot act in violation of a statute. A sub-
delegatee cannot exercise any power, which is not meant to be conferred upon him by
reason of statutory provisions. (Para 26)
(B) WORDS AND PHRASES - Words and Phrases - 'Incidental or connected' - Do not
mean matter which are of casual nature - But same must have something to do with the
nature of power granted to the authority concerned. (Para 27)
Cases Referred : Chronological Paras
2007 AIR SCW 531 : AIR 2007 SC 819 (Rel. on) 26
2007 AIR SCW 3931 : AIR 2007 SC 2253 : 2007 (4) ALJ 711 (Rel. on) 26
2007 AIR SCW 5722 : AIR 2007 SC 3159 : 2007 Lab IC 4452 (Ref.) 25
2007 AIR SCW 7750 (Ref.) 13
2006 AIR SCW 1361 : AIR 2006 SC 1420 (Ref.) 12
2006 AIR SCW 1392 : AIR 2006 SC 1489 : 2006 (3) AIR Bom R 164 (Rel. on) 26
2006 AIR SCW 2676 : AIR 2006 SC 2145 : 2006 (4) AIR Kar R 194 (Ref.) 12
2005 AIR SCW 6045 : AIR 2006 SC 407 : 2006 Lab IC 137 (Rel. on) 12
1989 Supp (1) SCC 347 (Rel. on) 26
AIR 1961 SC 284 (Rel. on) 26
AIR 1958 Cal 68 21
AIR 1947 Lahore 355 (FB) 21
P.S. Patwalia, Sr. Adv., S. Rajappa, H. Jairaman, Ms. Tania Walia, Devish Tripathi, for
Appellants; P.S. Narasimha, Sridhar Potaraju, D. Julius Diamei, Ms. Mandakani, for
Respondent.
Judgement
1. S. B. SINHA, J. :-Jurisdiction of the Chairman of the Life Insurance Corporation of
India (Corporation) to issue instructions
@page-SC1486
in terms of Regulation 51 of the Life Insurance Corporation of India Class-I Officers
(Revision of Terms and Conditions of Service) Instructions, 1996 is in question in this
appeal which arises out of a judgment and order dated 29th September, 2005 passed by a
Division Bench of the Kerala High Court in Writ Appeal No. 32 of 2004.
2. We may notice only the admitted facts herein.
Respondent No. 1 is an Association of officers who have retired from the services of the
appellant-Corporation which is a statutory authority constituted and incorporated under
the Life Insurance Corporation Act, 1956.
During the period of 1st August, 1992 and 31st July, 1994 a revision of scales of pay of
the officers and employees of the Corporation took place. Different cut off dates were
fixed for grant of different nature of allowances as also pay by the Chairman of the
Corporation in purported exercise of his power under Regulation 51 of the Regulations.
Whereas 1st April, 1993 was the cut off date for revision of pay; 1st August, 1994 was
fixed as the cut off date for the purpose of payment of gratuity on the basis of revised pay.
However, so far as those employees who had retired prior to 1st August, 1994 are
concerned, they were directed to be entitled to reduce gratuity based on the reduced scale
of pay with effect from 1st April, 1993 only. The arrears of pay were directed to be paid
only w.e.f. 1st April, 1993.
3. Indisputably, whereas the Gujarat and Kerala High Court upheld the validity of the
instructions issued by the Chairman of the appellant-Corporation, the Karnataka High
Court took a different view.
4. The claim of Respondent No. 1 was allowed in part by a learned Single Judge of the
High Court by his order dated 8th July, 2003 holding :-
"A reading of Ext.P.3 (instructions issued by the Chairman for supplementary of
Revisionist in respect of class I officers and claimed IV will definitely go to show that it
cannot operate as far as the claims for gratuity is concerned. It is admitted that at least
certain officers, represented by the petitioner Association were deemed as having revised
salary from April, 1993 onwards. In that view, at the time of retirement, they were
deemed as getting a salary which alone could have been taken notice of for computing
gratuity, if Regulation No. 77 has any application. It is definite that the restriction in Ext.
P. 3 and benevolence in Regulation No. 77 could not have co-existed because the
Corporation is offering gratuity at the rate less than the amount an employee had
notionally drawn at the time of their respective retirement. It is also pertinent to note that
when powers were conferred on the Chairman under Regulation No. 51(2), specific
reference was there about the incidents of DA and other allowances. There is no reference
to any alteration permissible in respect of gratuity. It leads to the position that the
regulation did not permit the Chairman to disturb criterion for gratuity payment by
exercise of powers under Regulation No. 51(2)."
It was further held :-
"There was no power on the part of the Bank Management in that case to disturb the
settlement, and the gratuity was to be paid on the basis of last drawn pay. Likewise, in the
present case, it would not have been permissible for the Chairman to unsettle the benefits
that had been spoken to by Regulation No.77 while issuing Ext.P.3 order."
5. A Division Bench of the said High Court on an intra court appeal preferred by the
appellants herein upheld the said findings.
6. Mr. Patwalia, learned senior counsel appearing on behalf of the appellants, in support
of this appeal, submitted :-
i) Pension and Gratuity having two different concepts, the High Court committed a
serious error in holding that the Chairman of the Corporation had no jurisdiction to issue
the instructions.
ii) Sub-regulation (2) of Regulation 51 being of wide amplitude, the jurisdiction of the
Chairman to fix cut off dates was not only applicable in respect of pay and allowances
covered by Schedule II of the Regulations but also included "gratuity" as envisaged under
Regulation 77, as the quantum thereof has a direct nexus with the payment of salary.
iii) An employer, subject to the applicability of the doctrine of reasonableness and non-
arbitrariness, can fix a cut off date for the implementation of the revised pay and
allowances.
iv) The amount of gratuity payable has to be calculated upon the permanent pay and
@page-SC1487
once the gratuity has been paid, no further amount is payable only because the salary has
been revised.
7. Mr. P.S. Narasimha, learned counsel appearing on behalf of the respondents, on the
other hand, contended that the power of the Chairman of the Corporation to issue
instructions being limited to Chapter IV of the Regulations, it has no application in
relation to the payment of gratuity as provided for in Regulation 77 thereof.
8. Appellant-Corporation in exercise of its powers conferred upon it by clauses (b) and
(bb) of sub-section (2) of Section 49 of the Life Insurance Corporation Act, 1956, with
the previous approval of the Central Government, made Regulations known as "Life
Insurance Corporation of India (Staff) Regulations, 1960" (in short 'the Regulations').
Chapter IV of the said Regulations deal with "Pay and Allowances". Regulation 51
thereof reads as under :-
"Scales of Pay :
51. (1) The scales of pay, dearness allowance and other allowances (wherever payable)
applicable to the employees of the Corporation in India shall be as prescribed in Schedule
II hereto.
(1A) The basic pay and other allowance admissible from time to time to an employee
belonging to Class II shall be regulated in accordance with the provisions contained in
Schedule III.
(2) Whereas the scales of pay, dearness allowance or other allowances applicable to the
employees of the Corporation or any class of them are revised in pursuance of any award,
agreement or settlement, or otherwise, the method of fixation of pay in the new scales,
the eligibility for the benefit of revision, the date from which the revision shall apply, and
other matters connected therewith or incidental thereto shall be regulated by instructions
issued by the Chairman in this behalf."
(Emphasis supplied)
9. Chapter VII of the said Regulations deals with Miscellaneous Matters. Regulation 76
deals with Provident Fund. Regulation 77 deals with Gratuity. Regulation 78 deals with
Superannuation Fund. Regulation 79 deals with Travelling Allowance Rules. There are
other provisions also dealing with some other benefits which are to be granted to the
employees of the Corporation.
10. Regulation 51 indisputably confers power upon the Chairman to fix a date from
which the revision in pay shall apply. It applies to pay, dearness allowance and other
allowances applicable to the employees of the Corporation. The question, as would
appear from the discussions made hereinafter, is as to whether the expression "the date
from which the revisions shall apply, and other matters connected therewith or incidental
thereto", would also include the matter relating to payment of gratuity which is otherwise
covered by Regulation 77 thereof.
11. Although Mr. Patwalia has relied upon a large number of decisions of this Court for
the purpose of making a distinction between the terms "pension" and "gratuity" as also
the Jurisdiction of the employer to fix a cut off date, it may not be necessary to deal with
all of them.
12

. We may, however, note some precedents operating in the field. Recently in H.E.C.
Voluntary Retired Employees Welfare Society and another vs. Heavy Engineering
Corporation Ltd. and others : (2006) 3 SCC 708 this Court observed :- 2006 AIR
SCW 1361
"24. In State of A.P. v. A.P. Pensioners Assn. this Court categorically held that the
financial implication is a relevant criterion for the State Government to determine as to
what benefits can be granted pursuant to or in furtherance of the recommendations of a
Pay Revision Committee. A' fortiori while taking that factor into account, an employer
indisputably would also take into consideration the number of employees to whom such
benefit can be extended." 2005 AIR SCW 6045

{See also U.P. Rahavendra Acharya and others vs. State of Karnataka and others [(2006)
9 SCC 630]} 2006 AIR SCW 2676

13

. It is also interesting to notice a decision of this Court in State of Andhra Pradesh and
another vs. A.P. Pensioners' Association and others : (2005) 13 SCC 161 wherein it was
opined :- 2005 AIR SCW 6045

"28. Computation of retirement gratuity payable to a government servant is, therefore,


required to be done on the basis of the formula laid down therein. A bare perusal of the
aforementioned Rule clearly shows that for the purpose of computation either ¼th of
the emoluments for each completed
@page-SC1488
six-monthly period of service, or 3/16th of emoluments for each completed six-monthly
period of service, is to be taken into consideration. Such emoluments necessarily were
payable either immediately before the date of retirement or the date of death. On 1-4-
1999, in view of the clear expressions contained in the aforementioned GO No. 114,
those employees who retired between the period 1-7-1998 and 1-4-1999 would have
received the actual benefit calculated in terms of the said Rule. The submission of Mr.
Lalit to the effect that they became entitled to enhanced pay and, therefore, to enhanced
gratuity from 1-7-1998 is not wholly correct. They became entitled thereto but only
notionally for the purpose of calculation of such recurring liability of the State which
became payable with effect from 1-4-1999. The High Court has heavily relied upon the
purported legal fiction created in the said Rule to the effect that the same would come
into force with effect from 1-7-1998. The legal fiction undoubtedly is to be construed in
such a manner so as to enable a person, for whose benefit such legal fiction has been
created, to obtain all consequences flowing therefrom."
It was further observed :-
"30. The case at hand indeed poses a different problem. Although like Gurupad
Khandappa Magdum a notional revision of pay was to be considered as if the same took
effect from 1-7-1998, but the Rules went further and stated that the actual monetary
benefit thereof shall be given with effect from 1-4-1999. The Rules, therefore, not only
create a legal fiction but also provide the limitations in operation thereof. If the effect of
the legal fiction is extended in the manner suggested by Mr. Lalit, clause (4) (sic Rule 4)
of the Rules will become otiose. In other words, all the consequences ordinarily flowing
from a rule would be given effect to if the rule otherwise does not limit the operation
thereof. If the rule itself provides a limitation on its operation, the consequences flowing
from the legal fiction have to be understood in the light of the limitations prescribed.
Thus, it is not possible to construe the legal fiction as simply as suggested by Mr. Lalit."

[See also State of Tamil Nadu v. Seshachalam, 2007 (11) SCALE 239]. 2007 AIR
SCW 7750

14. The Regulations are subordinate legislation. Chairman of the Corporation is a


statutory authority. Power to fix a cut off date has been conferred upon him by way of
statutory provision. The same requires a strict interpretation. Chapter IV of Regulations
envisages scales of pay. It also talks of dearness allowance and other allowances as
envisaged under the IInd Schedule thereof. Clause (2) of the said Regulation, as indicated
hereinbefore, confers jurisdiction on the Chairman of the Corporation to regulate the
same as also other matters connected therewith or incidental thereof by issuance of
instructions.
15. It may be true, as was contended by Mr. Patwalia, that the cut off dates were fixed
upon holding negotiations with the Unions.
However, the jurisdiction of the Chairman to fix a cut off date is in question in terms of
sub-regulation (2) of Regulation 51. Instructions have been issued under the said
provision alone. Instructions not only cover the scales of pay from a particular date but
different dates have been fixed for different types of allowances. We have noticed
hereinbefore that whereas dearness allowance and some other allowances, as for instance
'house rent allowance' and 'city compensatory allowance' are envisaged by IInd Schedule
appended to the said Regulations, the other allowances, and for instance, the 'Provident
Fund' and 'Gratuity' have nothing to do therewith. Provident Fund and Gratuity are
ordinarily governed by the Acts enacted by the Parliament, subject to the conditions
contained therein.
16. Regulation 77 of the Regulations, specifies the employees who would be entitled to
payment of gratuity. Clause (2) of Regulation 77 provides for the manner in which the
amount of gratuity shall be payable. Neither the payment of Provident Fund nor the
payment of Gratuity is thus covered by the provisions contained in Chapter IV of the
Regulations.
17. Clause (1) of Regulation 51 postulates grant of pay, dearness allowance and other
allowances in the manner as prescribed in the IInd Schedule. The basic pay and other
allowances to Class II employees are regulated under the provisions contained in
Schedule III thereof. Revision of pay, dearness allowance and other allowances
applicable to the employees of the Corporation stricto sensu are not covered by clause (2)
of Regulation 51. It merely states that when a revision takes place pursuant to or in
furtherance of any award, agreement or
@page-SC1489
settlement or otherwise, the Chairman of the Corporation will have the jurisdiction in
regard to :-
a) the method of fixation of pay in the new scales;
b) the eligibility for the benefit of revision; and
c) the date from which the revision shall apply.
18. Method of fixation, eligibility for the benefit of revision and the date from which the
revisions shall apply are thus, the only areas within which the Chairman can exercise
jurisdiction. The effect of revision of pay scales on other spheres and which are otherwise
governed by another statute or other provisions of the said Regulations would not come
within the purview thereof.
19. The terminology used "and other matters connected therewith or incidental thereto"
must, therefore, be held to have a direct nexus with any one of the aforementioned three
elements. The same has nothing to do with the construction of any other provision of the
Regulations. The words "incidental to" cannot be interpreted too broadly. It cannot be
read independently of the main provision. It cannot serve some other purpose which is
not covered by Regulation 51 of the Regulations. It cannot be permitted to encroach upon
an area which is not within the jurisdiction of the Chairman of the Corporation.
20. It is one thing to say that the court while exercising its jurisdiction would be entitled
to exercise such incidental power for determination of the principal issue but it is another
thing to say that a statutory authority in such matters would be held to have such power
which is beyond the scope and purport of the principal provisions.
21. The word "Incidental" has been defined in Advanced Law Lexicon 3rd (2005)
Edition, Book 2 at 2275 to mean :-
"According to Stroud's Judicial Dictionary, a thing is said to be incidental to another
when it appertains to the principal thing. According to the ordinary Dictionary meaning,
it signifies a subordinate action. Hukumchand Jute Mills Ltd. vs. Labour Appellate
Tribunal, AIR 1958 Cal. 68, 70. (Industrial Disputes Act (14 of 1917), S. 10(4)).

The word "incidental" does not imply any casual or fortuitous connection. In a legal sense
as applied to powers, it means a power which is subsidiary to that which has been
expressed, and of an instrumental nature in relation thereto, which is both necessary and
proper for the carrying into execution of the main power which has been expressly
conferred. (Dunichand and Co. vs. Narain Das and Co. (1947) 17 Comp. Cas. 195 (FB))."
AIR 1947 Lahore 355

22. Each word employed in a statute must take colour from the purport and object for
which it is used. The principle of purposive interpretation, therefore, should be taken
recourse to.
23. Revision of scales of pay as also other allowances is technical in nature. When a
benefit is extended to a group of employees the effect of such benefit, if otherwise comes
within the purview thereof must be held to be applicable to other groups of employees
also. An employee is entitled to gratuity. It is not a bounty. It is payable on successful
tenure of service. Regulation 77 provides as to how the amount of gratuity is to be
calculated. Regulation 51 provides for a rule of measurement. Only because it employed
the word "permanent basic pay", the same will not itself lead to the conclusion that once
an employee has retired, he would not be entitled to any revision of the amount of
gratuity.
24. The Chairman of the Corporation has himself given a retrospective effect to revision
in scales of pay. Such a retrospective effect has also been given so as to benefit a class of
employees. The employees, irrespective of the fact whether they had superannuated or
not, were given the benefit of arrears of pay from 1st August, 1993. By reason of grant of
such benefit both to serving employees as also the superannuated employees, both the
class of employees became entitled thereto as of right. If by reason thereof, even a retired
employee, as on the date of retirement, became entitled to the benefit of the revised scale
of pay, the same for all intent and purpose must be taken to be the permanent basic pay,
apart from other allowances, if any, which are required to be taken into consideration for
the purpose of computation of the amount of gratuity.
25

. In Indian Bank and another vs. N. Venkatramani : 2007 (10) SCALE 475 : this Court
gave effect to the beneficial provision in the light of the rule of measurement, stating :-
2007 AIR SCW 5722

@page-SC1490
"13. It may be true that various provisions of the Regulations as for example Regulations
16, 17, 19, 23, etc. provided for qualifying service. Regulation 18 is not controlled by any
of the said provisions. It does not brook any restrictive interpretation. It only provides for
a rule of measurement. An employee, as noticed hereinbefore, was entitled to pension
provided he has completed the specified period of service. How such a period of service
would be computed is a matter which is governed by the statute. It is one thing to say that
a statute provides for completion of fifteen years of minimum service, but if a provision
provides for measurement of the period, the same cannot be lost sight of. Provision of the
Regulations which are beneficial in nature, in our opinion, should be construed liberally."
26

. Contention of Mr. Patwalia that the Chairman of the Corporation having power even to
fix the cut off dates for different purposes, the jurisdiction exercised by him to do so for
payment of gratuity, which has a direct nexus with the revised pay of scale cannot be
accepted. Once he fixes a cut off date for the purpose of giving effect to the agreement
vis-a-vis the payment of arrears in terms thereof, he cannot exercise further jurisdiction in
respect of a matter which is not controlled by Chapter IV but is controlled by other
provisions of statutes and Parliament Acts governing the field. A delegatee must exercise
its powers within the four-corners of the statute. The power of a sub-delegatee is more
restricted. A delegatee cannot act in violation of a statute. A sub-delegatee cannot exercise
any power which is not meant to be conferred upon him by reason of statutory provisions.
It must conform not only to the provisions of the Regulations and the Act but also other
Parliamentary Acts. [See Kurmanchal Inst. of Degree and Diploma and Ors. vs.
Chancellor, M.J.P. Rohilkhand Univ. and Ors. (2007) 6 SCC 35, Kerala Samsthana
Chethu Thozhilali Union vs. State of Kerala and Ors. (2006) 4 SCC 327 Bombay Dyeing
and Mfg. Co. Ltd. vs. Bombay Environmental Action Group and Ors. (2006) 3 SCC 434,
State of Kerala and Ors. vs. Unni and Anr (2007) 2 SCC 365, State of Orissa and another
vs. M/s. Chakobhai Ghelabhai and Company : 1961 (1) SCR 719 and M/s. Shroff and Co.
vs. Municipal Corporation of Greater Bombay and another : (1989) Supp. 1 SCC 347].
2007 AIR SCW 3931
2006 AIR SCW 1869
2006 AIR SCW 1392
2007 AIR SCW 531
AIR 1961 SC 284

27. We, however, do not intend to lay down the law that the expression "incidental" or
"connected" would be matters which are of a casual nature only, but, we reiterate that the
same must have something to do with the nature of power granted to the authority
concerned.
28. Unfortunately before the Gujarat High Court and the Karnataka High Court, the
counsel re-missed in bringing to the Court's notice this aspect of the matter.
29. We, therefore, do not find any merit in this appeal which is accordingly dismissed
with costs. Counsel's fee assessed at Rs. 25,000/-.
Appeals dismissed.
AIR 2008 SUPREME COURT 1490 "Bhanwar Singh v. Puran"
(From : Punjab and Haryana)*
Coram : 2 S. B. SINHA AND V. S. SIRPURKAR, JJ.
Civil Appeal No. 1233 of 2006 (arising out of SLP (C) No. 9503 of 2007), D/- 12 -2
-2008.
Bhanwar Singh v. Puran and Ors.
Hindu Succession Act (30 of 1956), S.8, S.6 - SUCCESSION - SALE - PARTITION -
Joint family property - Suit to set aside sale - Ground that it was not for legal necessity -
Sale in question by plaintiff's father - Plaintiff's father inheriting property along with his
sister - Land partitioned and recorded in name of each - Loses character of joint family
property - S.6 has not application - Plaintiff's father and his sisters were tenants in
common - Each had right to transfer his share of land - Suit liable to be dismissed. (Paras
13, 17, 18)
Cases Referred : Chronological Paras
(2006) 8 SCC 581 (Disting.) 16
1994 Supp (2) SCC 112 (Ref.) 15
1993 Supp (1) SCC 580 (Ref.) 15
AIR 1987 SC 558 (Ref.) 15
AIR 1986 SC 1753 : 1986 Tax LR 1328 (Rel. on) 14, 15
1982 Tax LR 483 (Cal) 15
(1977) 108 ITR 417 (Guj) (Not good law in view of AIR 1986 SC 1753) 15
AIR 1975 SC 495 16
@page-SC1491

Gagan Gupta, for Appellant; Manoj Swarup and Rohit Sohgaura, for Respondents.
* R. S. A. No. 3924 of 2006, D/- 14-11-2006 (P and H).
Judgement
1. S. B. SINHA, J. :-Leave granted.
2. Applicability of Section 8 of the Hindu Succession Act, 1956 (the Act) to the facts of
the present case is in question in this appeal which arises out of a judgment and order
dated 14.11.2006 passed by a learned Single Judge of the Punjab and Haryana High
Court whereby and whereunder the second appeal preferred by the appellant herein was
dismissed.
3. One Bhima was the owner of the property. He died in the year 1972 leaving behind his
son, Sant Ram and three daughters, Shanti, Manti and Shakuntala. Appellant, who is son
of Sant Ram was born in the year 1977. He attained majority in the year 1995. The
properties in suit were partitioned between Sant Ram and his sisters. Their names were
mutated in the revenue records of rights. Their shares in the properties of the deceased
Bhima were shown to be 1/4th each in the revenue records of 1973-74.
4. Inter alia, on the premise that the properties of Bhima were joint family properties and
the same were transferred by Sant Ram, firstly by way of mortgage and thereafter by sale
in favour of the respondents herein in the year 1985, the appellant filed a suit for setting
aside the said alienations. It was contended that the consideration for the said transaction
being a meagre sum of Rs. 12,000/- and furthermore being not for legal necessity, the
same should be set aside.
5. The said suit was decreed by the learned Trial Judge holding that the property was joint
family one and Sant Ram being the 'Karta', could not have transferred the same, save and
except by way of legal necessity. The learned First Appellate Court, however, reversed
the same findings, inter alia, holding that upon the death of Bhima, Sant Ram became a
co-sharer of the property and having regard to the entries of the jamabandi for the year
1973-74, it had been established that he, along with his sisters, having inherited the same
in equal shares, the property lost the character of ancestral property in terms of Section 8
of the Hindu Succession Act.
6. It was furthermore opined that even if the property was a joint property, the interest of
Sant Ram being 1/4th in the half share therein and the other half of Bhima having been
inherited by Sant Ram and his sisters, the disputed property ceased to be a Hindu
Undivided Family Property. In any event, the Deed of Sale executed by Sant Ram having
been executed for legal necessity as the suit property had already been mortgaged, the
deeds of sale could not have been cancelled.
A limited notice was issued by this Court as to whether the father of the petitioner had
inherited the property from his forefathers.
7. Mr. Gagan Gupta, learned counsel appearing on behalf of the appellant, would submit
that the Appellate Court as also the High Court committed a serious error in so far as they
failed to take into consideration the well settled principles of Hindu Law that transfer
made by the father after the birth of the son would be held to be illegal unless legal
necessity therefor is proved, as such transactions could be entered into by the manager or
karta of the family only for legal necessity and for no other. The Appellate Court, it was
contended, committed a serious error in so far as it proceeded to hold that the property in
question became separate property at the hands of Sant Ram, but, despite the same, it
proceeded to determine the question of legal necessity also. It was furthermore submitted
that only because some entries have been made in the record of rights, the same by itself
would not lead to deprivation of the title in the property in the appellant.
8. Mr. Manoj Swarup, learned counsel appearing on behalf of the respondents, on the
other hand, would submit that in view of Section 8 of the Hindu Succession Act, as the
son of Bhima and his daughters inherited his property and not the appellant as a
grandson, the impugned judgment is unassailable.
9. The fact that the property at one point of time was a joint family property stands
admitted.
10. The only question arises for consideration is as to whether the appellant had acquired
any interest therein by his birth in the year 1977; Bhima having died in 1972.
11. The Act brought about a sea change in the matter of inheritance and succession
amongst Hindus. Section 4 of the Act contains a non obstante provision in terms whereof
any text, rule or interpretation of Hindu Law or any custom or usage as part
@page-SC1492
of that law in force immediately before the commencement of the Act, ceased to have
effect with respect to any matter for which provision is made therein save as otherwise
expressly provided.
Section 6 of the Act, as it stood at the relevant time, provided for devolution of interest in
the coparcenary property. Section 8 lays down the general rules of succession that the
property of a male dying intestate devolve according to the provisions of the Chapter as
specified in clause (1) of the Schedule. In the Schedule appended to the Act, natural sons
and daughters are placed in Class-I heirs but a grandson, so long as father is alive, has not
been included. Section 19 of the Act provides that in the event of succession by two or
more heirs, they will take the property per capita and not per stripes, as also tenants-in-
common and not as joint tenants.
12. Indisputably, Bhima left behind Sant Ram and three daughters. In terms of Section 8
of the Act, therefore, the properties of Bhima devolved upon Sant Ram and his three
sisters. Each had 1/4th share in the property. Apart from the legal position, factually the
same was also reflected in the record of rights. A partition had taken place amongst the
heirs of Bhima.
13. Although the learned First Appellate Court proceeded to consider the effect of Section
6 of the Act, in our opinion, the same was not applicable in the facts and circumstances of
the case. In any event, it had rightly been held that even in such a case, having regard to
Section 8 as also Section 19 of the Act, the properties ceased to be joint family property
and all the heirs and legal representatives of Bhima would succeed to his interest as
tenants in common and not as joint tenants. In a case of this nature, the joint coparcenary
did not continue.
14

. Interpretation of Section 8 of the Hindu Succession Act came up for consideration


before this Court in Commissioner of Wealth Tax, Kanpur and Ors. v. Chander Sen and
Ors. [(1986) 3 SCR 254]. Mukherjee, J. (as the learned Chief Justice then was) upon
considering the changes effected by the Hindu Succession Act as also the implication
thereof and upon taking into consideration the decisions of Calcutta High Court, Madhya
Pradesh High Court, Andhra Pradesh High Court as also Madras High Court on the one
hand and the Gujarat High Court on the other, opined : AIR 1986 SC 1753, Paras 20,
21, 22

"In view of the preamble to the Act, i.e., that to modify where necessary and to codify the
law, in our opinion it is not possible when Schedule indicates heirs in class I and only
includes son and does not include son's son but does include son of a predeceased son, to
say that when son inherits the property in the situation contemplated by Section 8 he
takes it as karta of his own undivided family. The Gujarat High Court's view noted above,
if accepted, would mean that though the son of a predeceased son and not the son of a son
who is intended to be excluded under Section 8 to inherit, the latter would by applying
the old Hindu law get a right by birth of the said property contrary to the scheme outlined
in Section 8. Furthermore as noted by the Andhra Pradesh High Court that the Act makes
it clear by Section 4 that one should look to the Act in case of doubt and not to the
preexisting Hindu law. It would be difficult to hold today the property which devolved on
a Hindu under Section 8 of the Hindu Succession Act would be HUF in his hand vis-a-vis
his own son; that would amount to creating two classes among the heirs mentioned in
class I, the male heirs in whose hands it will be joint Hindu family property and vis-a-vis
son and female heirs with respect to whom no such concept could be applied or
contemplated. It may be mentioned that heirs in class I of Schedule under Section 8 of the
Act included widow, mother, daughter of predeceased son etc.
Before we conclude we may state that we have noted the observations of Mulla's
Commentary on Hindu law 15th Edn. dealing with Section 6 of the Hindu Succession Act
at pages 924-26 as well as Mayne's on Hindu Law, 12th Edition pages 918-919.
The express words of Section 8 of The Hindu Succession Act, 1956 cannot be ignored
and must prevail. The preamble to the Act reiterates that the Act is, inter alia, to 'amend'
the law, with that background the express language which excludes son's son but included
son of a predeceased son cannot be ignored."
15. The Gujarat High Court in Commissioner of Income-tax, Gujarat-I v. Babubhai
Manshkhbhai (Deceased) [108 ITR 417], however, it may be noticed, had taken the view
that in the case of the Hindus governed by Mitakshara law, where a son inherited
@page-SC1493
the self- acquired property of his father, he took it as a joint family property of himself
and his son and not as his separate property. The said view, as indicated hereinbefore was
not accepted by this Court.

The principle evolved in Chander Sen (supra) was reiterated by this Court in Yodhishter
v. Ashok Kumar [(1987) 1 SCR 516 at 523]; Sunderdas Thackersay and Bros. v.
Commissioner of Income-tax [1982 (137) ITR 646]; Commissioner of Income Tax v. P.L.
Karuppan Chettiar [1993 Supp (1) SCC 580]; and Additional Commissioner of Income-
tax v. M. Karthikeyan [1994 Supp (2) SCC 112]. AIR 1986 SC 1753
AIR 1987 SC 558
1982 Tax LR 483

In Yodhishter (supra), this Court observed : AIR 1987 SC 558, Para 10

"This question has been considered by this Court in Commissioner of Wealth Tax,
Kanpur and Ors. v. Chander Sen and Ors. [(1987) 1 SCR 516] where one of us
(Sabyasachi Mukharji, J.) observed that under the Hindu Law, the moment a son is born,
he gets a share in father's property and become part of the coparcenary. His right accrues
to him not on the death of the father or inheritance from the father but with the very fact
of his birth. Normally, therefore, whenever the father gets a property from whatever
source, from the grandfather or from any other source, be it separated property or not, his
son should have a share in that and it will become part of the joint Hindu family of his
son and grandson and other members who form joint Hindu family with him." AIR
1986 SC 1753
16. Moreover, recently in Sheela Devi and Ors. v. Lal Chand and Anr. [(2006 (8) SCC
581], a Bench of this Court of which one of us was a member, held :
"21. The Act indisputably would prevail over the old Hindu Law. We may notice that
Parliament, with a view to confer right upon the female heirs, even in relation to the joint
family property, enacted the Hindu Succession Act, 2005. Such a provision was enacted
as far back in 1987 by the State of Andhra Pradesh. The succession having opened in
1989, evidently, the provisions of the Amendment Act, 2005 would have no application.
Sub-section (1) of Section 6 of the Act governs the law relating to succession on the death
of a coparcener in the event the heirs are only male descendants. But, the proviso
appended to sub-section (1) of Section 6 of the Act creates an exception. First son of
Babu Lal viz. Lal Chand, was, thus, a coparcener. Section 6 is an exception to the general
rules. It was, therefore, obligatory on the part of the respondent-plaintiffs to show that
apart from Lal Chand, Sohan Lal will also derive the benefit thereof. So far as the second
son, Sohan Lal is concerned, no evidence has been brought on record to show that he was
born prior to coming into force of the Hindu Succession Act, 1956."
In that case, the properties in question were joint family properties. They were
coperceners. After the death of Tulsi Ram, Babu Ram, whose heirs were the appellants
therein, inherited 1/5th share in the property. The relationship between the parties was not
in dispute. Tulsi Ram was the owner of the property. He died in the year 1889 leaving
behind five sons, namely, Waliwati, Babu Ram, Charanji Lal, Hukam Chand and Uggar
Sain. On the death of Uggar Sain 1/20th share of Tulsi Ram was also devolved on him.
The High Court arrived at a finding of fact that the properties were coparcenary and
ancestral property. It was held that the law which was applicable in the case would be the
one which was prevailing before corning into force of the Hindu Succession Act and the
parties would be governed thereby under the provisions thereof. It was in the
aforementioned situation and having regard to the fact that the succession of the property
was governed in terms of Section 6 of the Act, it was held :

"12. The principle of law applicable in this case is that so long a property remains in the
hands of a single person, the same was to be treated as a separate property, and thus such
a person would be entitled to dispose of the coparcenary property as the same were his
separate property, but, if a son is subsequently born to him or adopted by him, the
alienation whether it is by way of sale, mortgage or gift, will nevertheless stand, for a son
cannot object to alienations so made by his father before he was born or begotten (See C.
Krishna Prasad v. CIT [(1975 1 SCC 160]). But once a son is born, it becomes a
coparcenary property and he would acquire an interest therein." AIR 1975 SC 498

In that case, as noticed hereinbefore, Babu Ram had no son in the year 1922 but
@page-SC1494
a son, Lal Chand, was born to him in the year 1938 and another son, Sohan Lal, was born
in 1956. It was in the aforementioned situation, this Court held that a joint family revived
on the birth of Lal Chand. This Court, in that view of the matter also opined that as there
was no proof as to whether the second son was born after the corning into force of the
Hindu Succession Act, it was held that his heirs were not entitled to take the benefit of the
coparcenary interest.
Sheela Devi, therefore, is not applicable to the fact of the present case.
17. It is true that the first Court of Appeal also entered into the question of legal necessity
for Sant Ram to alienate the property in favour of the contesting respondents but the said
issue was considered in the alternative to the principal issue. If the First Appellate Court
was correct in its opinion and we do not see any reason to differ therewith that Section 6
of the Hindu Succession Act was not attracted to the facts of this case in view of the fact
that Sant Ram and his sisters having partitioned their properties became owners to the
extent of ¼th share each, he had the requisite right to transfer the lands falling within
his share.
18. Furthermore, in terms of Section 19 of the Act, as Sant Ram and his sisters became
tenants-in-common and took the properties devolved upon them per capita and not per
stirpes, each one of them was entitled to alienate their share, particularly when different
properties were allotted in their favour. It is, therefore, not correct to contend that the
Court of First Appeal arrived at a self-contradictory or inconsistent finding, as was
submitted by Mr. Gupta.
19. For the reasons aforementioned, there is no infirmity in the impugned judgment.
There is no merit in the case. It is dismissed accordingly. In the facts and circumstances
of the case, however, there shall be no order as to costs.
Appeal dismissed.
AIR 2008 SUPREME COURT 1494 "Swaika Properties Pvt. Ltd. v. State of Rajasthan"
(From : 2006 AIHC 3465 (Rajasthan))
Coram : 2 ASHOK BHAN AND DALVEER BHANDARI, JJ.
Civil Appeal No. 1081 of 2008 (arising out of SLP (C) No. 16910 of 2006), D/- 7 -2
-2008.
Swaika Properties Pvt. Ltd. and Anr. v. State of Rajasthan and Ors.
Rajasthan Urban Improvement Act (35 of 1959), S.52 - Constitution of India, Art.226 -
TOWN PLANNING - PLANNING AND DEVELOPMENT - ACQUISITION OF
LAND - WRITS - Acquisition of land - Writ petition against - Filed after possession has
been taken and award has become final - Liable to be dismissed for delay and laches.
(Paras 13, 17)
Cases Referred : Chronological Paras
2000 AIR SCW 197 : AIR 2000 SC 671 (Rel. on) 16
(1997) 2 SCC 627 (Rel. on) 16
1996 AIR SCW 3871 : AIR 1997 SC 482 (Rel. on) 13
1996 AIR SCW 3970 (Rel. on) 15
Dr. A.M. Singhvi, Sr. Advocate, Dr. Manish Singhvi and P.V. Yogeswaran, for Appellants;
Bharat Vyas, AAG, C.S. Vaidyanathan, Sr. Advocate, Manish Kumar and Ansar Ahmad
Chaudhary, for Respondents.
Judgement
1. ASHOK BHAN, J. :- Leave granted.
2. Challenge in the present appeal is to the order passed on 04th September, 2006 in D.B.
Special Appeal (W) No. 134 of 2006 by a Division Bench of the High Court of Rajasthan
dismissing the appeal filed by the appellants whereby the Division Bench has upheld the
judgment and order passed by a learned Single Judge of the same High Court dismissing
the writ petition filed by the appellants thereby upholding the acquisition proceedings in
respect of the land of the appellants.
3. In order to appreciate the grievance of the appellants, facts leading to the filing of the
appeal are necessary to be indicated.
4. Notice under Section 52(2) [which is equivalent to Section 4 of the Land Acquisition
Act, 1894] of the Rajasthan Urban Improvement Act, 1959 (for short, 'the Act') was
issued on 25th June, 1975 for acquiring the land of the appellants bearing Khasra No. 383
measuring 14 bighas and 16 biswas situated at Madrampura, Jaipur, Rajasthan. As per the
said notice, land was sought to be acquired for improvement and purposes of Jaipur Town
- extension of Civil Lines Area for construction of buildings. On 23rd August, 1975,
another notice was issued by the State under Section 52(2) of the Act indicating the
purpose of acquisition of land for extension of civil lines and planning of housing
scheme. Appellants, on 08th September 1975, filed objections to the acquisition of their
land. The appellants also
@page-SC1495
submitted their representation from time to time. The Land Acquisition Officer, however,
rejected the objections. On 08th February 1984, the Government issued declaration under
Section 52(1) of the Act [which is equivalent to Section 6 of the Land Acquisition Act,
1894]. Pursuant to the said declaration, notice under Section 52(5) of the Act was issued
asking the appellants to hand over possession of the land.
5. After these notices were issued, the appellants filed Writ Petition No. 5972 of 1984
before the High Court of Calcutta. A learned Single Judge of the High Court of Calcutta,
while issuing rule nisi, granted stay of the acquisition proceedings and restrained the
respondents from taking possession of the land. Aggrieved against the said order the
respondents filed Special Leave Petition before this Court, inter alia, challenging the
jurisdiction of the Calcutta High Court to entertain the writ petition. Since the Special
Leave Petition was filed against an interim order, this Court did not go into the merits of
the case and while granting leave, by order dated 08th April, 1985, set aside the order of
the Calcutta High Court and held that the Calcutta High Court did not have the territorial
jurisdiction to entertain the writ petition in respect of a land situated in the State of
Rajasthan. Thus, the proceedings before the Calcutta High Court came to an end.
6. On 17.02.1987, possession of the land in question is alleged to have been taken by the
respondents and the same was handed over to Jaipur Development Authority. However,
according to the the appellants, possession was not taken.
7. The appellants filed Writ Petition No. 1507 of 1987 in the High Court of Rajasthan
which was withdrawn by them on 10th March, 1989 with liberty to file a fresh writ
petition.
8. On 26th June, 1989, the Land Acquisition Officer passed the award and forwarded the
same for approval to the State Government. According to the respondents, the State
Government accorded its approval on 29th July, 1989 which was declared by the Land
Acquisition Officer on 30th July, 1989 in accordance with law.
9. In the meantime, the appellants filed S.B. Civil Writ Petition No.2911 of 1989 seeking
quashing of Notification dated 08th February, 1984 and also notice dated 7th/18th
February, 2007 by which the possession is alleged to have been taken. One of the
objections taken by the respondents before the High Court was that the writ petition could
not be entertained after taking over of the possession of the land and handing over the
same to Jaipur Development Authority and the award having been passed in respect of
the said land in accordance with law. It was also alleged that the award was not the
subject-matter of the writ petition. It was also pointed out that the appellants were
simultaneously pursuing the Reference Application for enhancement of compensation.
Ultimately, a learned Single Judge of the High Court of Rajasthan, accepting the
submissions of the respondents, dismissed the writ petition holding that there was a
genuine public purpose behind initiating the acquisition proceedings.
10. Being aggrieved, the appellants carried the matter in appeal before the Division
Bench of the High Court of Rajasthan. The Division Bench, agreeing with the view taken
by the learned Single Judge, dismissed the appeal and held that since the appellants had
filed an application under Section 18 of the Land Acquisition Act, 1894 for enhancement
of the compensation they are not entitled to the relief sought for in the writ petition. The
said order is under challenge before us.
11. A preliminary objection has been taken by the respondents to the effect that the appeal
is liable to be dismissed on the ground of the delay on the part of the appellants to
challenge the acquisition proceedings. It is also submitted that the acquisition of the land
cannot be challenged after taking over of the possession and after the award having
become final. In support of this submission, the respondents have relied upon a number
of judgments of this Court.
12. Counsel for the appellants, however, strenuously contended that there was no delay
on the part of the appellants in filing the writ petition challenging the acquisition
proceedings. He submitted that soon after the notice under Section 52(5) was issued by
the respondents, the appellants filed writ petition before the High Court of Calcutta which
stood dismissed pursuant to an order of this Court, as noted above. Thereafter the
appellants filed another writ petition before the High Court of Rajasthan which was
withdrawn. Within a period of four
@page-SC1496
months thereof, i.e., on 05th July 1989, another writ petition bearing No. 2911 of 1989
was filed. Counsel also submitted that these acts of the appellants demonstrate that there
was no delay on the part of the appellants to seek redressal of their grievance. It was
contended that actual possession of the land was never taken by the respondents on
17th/18th February, 1987.
13

. We do not find any substance in the submissions of the counsel for the appellants. No
doubt, the appellants had filed a writ petition before the Calcutta High Court challenging
the acquisition proceedings, but the said writ petition was dismissed by this Court on 08th
April, 1985 holding that the Calcutta High Court did not have the territorial jurisdiction
to entertain the writ petition. Thereafter, till 1987 the appellants did not challenge the
acquisition proceedings and the writ petition was filed by it before the Rajasthan High
Court which had the territorial jurisdiction in the matter and the same was withdrawn
which was again filed within the next four months thereof, meaning thereby, during the
interregnum the appellants slept over the matter. However, the appellants have not been
able to give any explanation for the same. Insofar as the contention regarding the
possession having not been taken is concerned, the respondents submit that the
possession of the land in dispute has already been taken. Be that as it may, the award in
respect of the land having become final, the State Government is vested with the powers
to take possession of the land concerned and, therefore, there is no reason to disbelieve
the claim of the State Government that the possession had been taken before the filing of
the writ petition. Moreover, the appellants sought enhancement of compensation by filing
reference application under Section 18 of the Land Acquisition Act, 1894.
Simultaneously, the appellants filed writ petition before the High Court of Rajasthan after
passing of the award. This Court has repeatedly held that a writ petition challenging the
notification for acquisition of land, if filed after the possession having been taken, is not
maintainable. In the case of Municipal Corporation of Greater Bombay v. Industrial
Development Investment Co. Pvt. Ltd. and Ors. (1996) 11 SCC 501, where K.
Ramaswamy, J. speaking for a Bench consisting of His Lordship and S.B. Majmudar, J.
held : 1996 AIR SCW 3871, para 29

"It is thus well-settled law that when there is inordinate delay in filing the writ petition
and when all steps taken in the acquisition proceedings have become final, the Court
should be loath to quash the notifications. The High Court has, no doubt, discretionary
powers under Article 226 of the Constitution to quash the notification under Section 4(1)
and declaration under Section 6. But it should be exercised taking all relevant factors into
pragmatic consideration. When the award was passed and possession was taken, the
Court should not have exercised its power to quash the award which is a material factor
to be taken into consideration before exercising the power under Article 226. The fact that
no third party rights were created in the case is hardly a ground for interference. The
Division Bench of the High Court was not right in interfering with the discretion
exercised by the learned Single Judge dismissing the writ petition on the ground of
laches."
14. In the concurring judgment, S.B. Majmudar, J. held as under :
".....Such a belated writ petition, therefore, was rightly rejected by the learned Single
Judge on the ground of gross delay and laches. The respondent-writ petitioners can be
said to have waived their objections to the acquisition on the ground of extinction of
public purpose by their own inaction, lethargy and indolent conduct. The Division Bench
of the High Court had taken the view that because of their inaction no vested rights of
third parties are created. That finding is obviously incorrect for the simple reason that
because of the indolent conduct of the writ petitioners land got acquired, award was
passed, compensation was handed over to various claimants including the landlord.
Reference applications came to be filed for larger compensation by claimants including
writ petitioners themselves. The acquired land got vested in the State Government and the
Municipal Corporation free from all encumbrances as enjoined by Section 16 of the Land
Acquisition Act. Thus right to get more compensation got vested in diverse claimants by
passing of the award, as well as vested right was created in favour of the Bombay
Municipal Corporation by virtue of the vesting of the land in the State Government for
being handed over to the Corporation. All these events could not be wished away by
observing that no third party rights were created
@page-SC1497
by them. The writ petition came to be filed after all these events had taken place. Such a
writ petition was clearly stillborn due to gross delay and laches....."
15
. Similarly, in the case of State of Rajasthan and Ors. v. D.R. Laxmi and Ors. (1996) 6
SCC 445, following the decision of this Court in the case of Municipal Corporation of
Greater Bombay (supra) it was held : 1996 AIR SCW 3970, Para 9

".... When the award was passed and possession was taken, the Court should not have
exercised its power to quash the award which is a material factor to be taken into
consideration before exercising the power under Article 226. The fact that no third party
rights were created in the case, is hardly a ground for interference. The Division Bench of
the High Court was not right in interfering with the discretion exercised by the learned
Single Judge dismissing the writ petition on the ground of laches....."
16

. To the similar effect is the judgment of this Court in the case of Municipal Council,
Ahmednagar and Ann v. Shah Hyder Beig and Ors. (2000) 2 SCC 48, this Court,
following the decision of this Court in the case of C. Padma v. Dy. Secy. to the Govt. of
T.N. (1997) 2 SCC 627, held : 2000 AIR SCW 197, Para 17

"In any event, after the award is passed no writ petition can be filed challenging the
acquisition notice or against any proceeding thereunder. This has been the consistent view
taken by this Court and in one of the recent cases (C. Padma v. Dy. Secy. to the Govt. of
T.N.) ...."
17. In the present case also, the writ petition having been filed after taking over the
possession and the award having become final, the same deserves to be dismissed on the
ground of delay and laches. Accordingly, the order of the learned Single Judge and that of
the Division Bench are affirmed to the extent of dismissal of the writ petition and the
special appeal without going into the merits thereof. This appeal also deserves to be
dismissed without going into the merits of the case and is dismissed as such. No costs.
Appeal dismissed.
AIR 2008 SUPREME COURT 1497 "State of Rajasthan v. Lala"
(From : Rajasthan)*
Coram : 2 S. B. SINHA AND H. S. BEDI, JJ.
Criminal Appeal No. 518 with 519 of 2001, D/- 21 -2 -2008.
State of Rajasthan v. Lala alias Abdul Salam.
Criminal P.C. (2 of 1974), S.386 - APPEAL - MURDER - EVIDENCE - ARREST -
Appeal against conviction - Interference - Murder case - Record showing gross attempt
by prosecution to create false evidence - Place of incident changed to justify presence of
eye-witnesses - Roznamacha interpolated to show presence of police officers who
allegedly arrested accused - Memo of arrest prepared by police falsified by evidence of
Traffic Police as to arrest - Arrest of accused thus also shrouded in mystery - Acquittal of
accused proper. (Para 9)

Naveen Kumar Singh, Shashwat Gupta, (for Aruneshwar Gupta), Srilok Nath Rath,
Manish Garg, Ms. Padmini Mehra, Chanchal Kumar Ganguli, for Appellant; B.B. Singh,
AC, Kumar Rajesh Singh, M.M. Kashyap, for Respondent.
* S. B. Cri. Appeal No. 243 of 1998, D/- 19-5-2000 (Raj) (Jaipur Bench).
Judgement
HARJIT SINGH BEDI, J. :-This appeal by way of special leave at the instance of the
State of Rajasthan has been filed challenging the judgment of the Rajasthan High Court
whereby the appeal filed by the accused/respondent Lala @ Abdul Salam against his
conviction and sentence for an offence under Section 304 Part-II I. P. C. has been allowed
leading to his acquittal. The facts are as under :
2. At about 3.00 p.m. on 3rd August, 1996, police station Bani Park, Jaipur received
information that some person had been caused injuries with a knife in front of Dilbahar
Wine Store and had been removed to the SMS hospital. This report was immediately
entered in the roznamacha and as a consequence thereof ASI Banwari Lal PW 14
accompanied by Constables Khem Chand and Jai Kishan went to the hospital and found
Satish Kumar PW3 present there. Satish Kumar gave a written report Ex. P-4 to ASI
Banwari Lal in which he stated that at about 2/2.30 p.m. while at home he had received
information from some person that his brother Om Prakash @ Omi had been
@page-SC1498
caused injuries with a knife near the Dilbahar Wine Store and was lying there in an
injured condition. He further stated that he rushed to the Wine Store which was near the
Khasa Kothi Circle and found Om Prakash screaming in pain with blood oozing from his
stomach. On enquiry, Om Prakash told him that Lala @ Abdul Salam s/o. Mohd. Yusuf
who was a pick pocket by profession had stabbed him after a quarrel over some money
matter. Satish Kumar further stated that he had removed Om Prakash to the hospital in an
Auto rikshaw and had got him admitted in Ward No. 2. As Om Prakash was not in a
position to give any statement, an FIR was registered under Section 307 IPC on the basis
of the written information provided by Satish Kumar. ASI Banwari Lal then started with
the investigation and also made a search for the accused and he was finally arrested from
near the Maharani Hotel later the same day. Om Prakash died at 10.30 p.m. and the case
was converted to one under Section 302 IPC. During the course of the investigation a
knife, the alleged murder weapon, and the clothes of the accused were recovered at his
instance and were sent to the Forensic Science Laboratory for examination. On the
completion of the investigation, he was charged for an offence punishable under Section
302 of the IPC and Section 4/25 of the Arms Act to which he pleaded not guilty and
claimed trial.
3. The prosecution in support of its case relied inter alia on the statements of the brothers
of the deceased, PW2 Bhagwan Sahai and PW 3 Satish respectively, PW5 Bhagirath, a
police constable posted at Bani Park Police Station and who was at the relevant time on
duty on the Sawai Jai Singh Highway, two recovery witnesses PW6 Ram Singh, who
were declared hostile, PW8 Jagdish Prasad Head Constable Traffic Police who was on
duty at the Khasa Kothi round about along with Deepak Kumar and Jagdish Prasad
constables and had reached Dilbhar Wine Store on hearing a commotion. PW 14 Banwari
Lal ASI, and PW 15 Dinesh Kumar Sharma DCP both Investigating Officers in this case
and PW 17 Dr. O. P. Saini who had conducted the post mortem examination and prepared
the report.
4. The prosecution case was then put to the accused and his statement was recorded under
Section 313 Cr. P. C. He denied the allegations levelled against him and pleaded that he
had been caught on 1st of August, 1996 by some persons while he was selling lottery
tickets and handed over to two police constables who had demanded money from him
and on his refusal to oblige them he had been taken to the police station and a false case
made out against him. He also examined a DW, one Bhasin in his defence.
5. The trial Court held that the witnesses were reliable and that the injuries that had been
caused by the accused had led to the death of Om Prakash but a case of murder was not
spelt out on the facts and that the accused was liable for conviction and sentence under
Section 304 Part-II of the IPC and that he was also entitled to the benefit of doubt and
thereby to an acquittal for the offence under the Arms Act. The accused was thus
sentenced to undergo RI for seven years under Section 304 Part-II IPC. An appeal was
thereafter taken by accused in the High Court, which as already mentioned above has
been allowed leading to the present appeal at the instance of the State. In arriving at its
conclusions the High Court observed that it was clear that the Roznamacha entry Ex. D-4
had been tampered with not only as to the time at which it had been recorded but also the
material facts had been obviously over written. The Court also held that ASI Banwari Lal
had deposed that Satish PW3 had given a written report at about 5 P. M. in the hospital
and that on its basis an FIR had been registered in the police station at 5.30 p.m. under
Section 307 IPC but as Satish had stated that the report had been given in the police
Station at 8.30 P. M., the story given by the police officer could not be believed. The
Court also observed that the initial prosecution story was that the incident had happened
in front of Dilbhar Wine Store but the venue had subsequently been changed with the
intention of shielding the employees of the liquor vend. The High Court then referred to
the Roznamacha Ex. D-4 and the written report Ex. P-4 given by Satish as also the
statement of Radhey Shyam PW-1 maternal uncle of the deceased and a witness to the
site plan Ex. P-2 to highlight that the incident had happened opposite Dilbagh Wine
Store. Likewise the Court referred to the statements of PW3 Satish, PW-4 Constable
Inder Raj, PW-5 Constable Bhagirath that the incident had happened near the liquor shop
and not across the road.
6. The High Court also found that as per
@page-SC1499
the memo Ex. P-9, the accused had been arrested by ASI Banwari Lal at 8.30 p. m. on the
day of incident after he had received information from an informer that he was sitting at
the C. T. S. bus station and that he had been found at that place and had attempted to run
away but had been caught near Maharani Hotel and a memo of arrest Ex. P-9 had been
prepared. The Court, however, observed that the statement of PW8 HC Jagdish Prasad of
the traffic police completely demolished the story of arrest as he had deposed that on
hearing a noise he along with others had rushed to Dilbagh hotel and seeing some one
inflicting injuries on the deceased and after causing the injuries had attempted to run
away and that he had been chased by several policemen and arrested near the Maharani
Hotel. The Court remarked that from a perusal of Ex. D-6, the report No. 263 in the
Roznamacha of the traffic police which had been recorded at 9.30 p.m. wherefrom it was
clear that the above facts leading to the arrest of the accused were not in consonance with
the prosecution version. The Court thus concluded that the story of arrest was a complete
concoction and could not be believed. The Court then went specifically in the evidence of
the eye-witnesses and held that from a perusal of Ex. P-13, the Roznamacha of the traffic
police station it was clear that Inder Raj Sharma and Bhagirath constable had been on
duty at the relevant time and that their presence had been interpolated in the record by
Hari Narain PW to show their presence. The High Court accordingly acquitted the
accused.
7. Learned Counsel for the State has argued that the High Court had not adequately met
the reasons given by the trial Court in making the order of conviction. He has pointed out
that the incident had happened in the afternoon and there was no reasons at all in
disbelieving the statement of PW 3 Satish to whom the deceased had made a dying
declaration as to his assailants. It has also been pointed out that the accused had been
apprehended soon after the incident and the fact that he admitted that he had attempted to
run away pointed to his guilt.
8. Learned Counsel for the accused however pointed out that the High Court had given
very good reasons for discarding the prosecution story and that no interference was
therefore called for at this stage.
9. We have considered the arguments of both the learned counsel. We note from the
record that there has been gross attempt on the part of the prosecution to create false
evidence in this case not only as the site of the incident (being changed from being
opposite the Dilbahar Wine Store to a place across the road) but on several other aspects
as well and with an attempt to justify the presence of the eye-witnesses. Likewise we
observe that the arrest of the accused is shrouded in suspicious circumstances. Here again
we find that the Roznamacha had been interpolated to show the presence of the police
officers who are alleged to have arrested him and the story projected by ASI Banwari Lal
that the accused had been arrested from near Maharani Hotel at about 2.30 p. m. has been
completely falsified by P-263 in Ex. D-13 which shows that the accused had in fact been
arrested by Constable Jagdish Prasad and some others in different circumstances and
from a different place. It was in this background that the High Court was compelled to
order administrative action against the police officers who had created false evidence in
the case and DSP Dinesh Kumar the indicted police officer is before us in Criminal
Appeal No. 569 of 2001. It has been pointed out on his behalf that he had taken over the
investigation on 4th August, 1996, only as an accused belonging to a Scheduled Caste/
Scheduled Tribe was involved that the only investigation that he made was with regard to
the recovery of the murder weapon. In this view of matter, we are of the opinion that no
action is called for against this officer and the observations made against him by the High
Court need to be expunged.
10. For the reasons recorded above. Criminal Appeal No. 518 of 2001 is dismissed.
Criminal Appeal No. 519 of 2001 is, however, allowed and all adverse remarks made
against DSP Dinesh Kumar are deemed to be expunged.
Order accordingly.
AIR 2008 SUPREME COURT 1499 "Regional P. F. Commr. v. Madathupatti Weavers
Co-op. Production and Sales Soc. Ltd."
(From : 2003 (3) Lab LJ 795 (Madras))
Coram : 2 TARUN CHATTERJEE AND HARJIT SINGH BEDI, JJ.
Civil Appeal No. 8598 of 2001, D/- 20 -2 -2008.
Regional Provident Fund Commr. and Ors. v. Madathupatti Weavers Co-op. Production
and Sales Soc. Ltd. @page-SC1500
Employees' Provident Funds and Miscellaneous Provisions Act (19 of 1952), S.2(g), S.17
- PROVIDENT FUND - APPLICABILITY OF AN ACT - SUPREME COURT -
APPEAL - Applicability of Act - Employer-Employee relationship - Weaver's Co-
operative Society and its members - Finding that no employer-Employee relationship
exists between two - Not interfered by S.C. in appeal - Enquiry conducted u/S.17 also
held improper. (Para 2)

Ashok Bhan, Mrs. Asha G. Nair, D.S. Mahra and B.V. Balram Das, for Appellants; R.
Nedumaran, for Respondent.
Judgement
JUDGMENT :- Heard learned counsel for the parties.
2. We do not find any ground to interfere with the impugned order of the High Court.
3. The Civil Appeal is dismissed with no order as to costs.
Appeal dismissed.
AIR 2008 SUPREME COURT 1500 "Shaik Nagoor v. State of A. P."
(From : Andhra Pradesh)*
Coram : 2 Dr. A. PASAYAT AND P. SATHASIVAM, JJ.
Criminal Appeal No. 346 of 2008 (arising out of SLP (Cri.) No. 3019 of 2007), D/- 20 -2
-2008.
Shaik Nagoor v. State of A.P.
Evidence Act (1 of 1872), S.32 - DYING DECLARATION - MAGISTRATE - Dying
declaration - Reliability - Deceased had suffered extensive burn injuries - Dying
declarations made to Magistrate and Head Constable - Not suffering from contradictions -
No suggestion put either to Magistrate or to Head-Constable about condition of deceased
to give statement - Plea that accused was not fit to give statement - Not tenable. (Para
12)
Cases Referred : Chronological Paras
2004 AIR SCW 902 : AIR 2004 SC 1616 : 2004 Cri LJ 1409 (Ref.) 8
2004 AIR SCW 7396 : AIR 2005 SC 1473 (Ref.) 11
(2004) 10 SCC 776 (Ref.) 10
2003 AIR SCW 7074 : AIR 2004 SC 846 (Ref.) 9
1992 AIR SCW 2050 : AIR 1992 SC 1817 : 1992 Cri LJ 2919 (Ref.) 11
AIR 1989 SC 1519 : 1989 Cri LJ 1485 11
AIR 1988 SC 912 : 1988 Cri LJ 936 (Ref.) 11
AIR 1985 SC 416 (Ref.) 11
AIR 1983 SC 164 : 1983 Cri LJ 221 (Ref.) 11
AIR 1982 SC 839 : 1982 Cri LJ 630 (2) 11
AIR 1982 SC 1021 : 1982 Cri LJ 986 (Ref.) 11
AIR 1981 SC 617 : 1981 Cri LJ 9 (Ref.) 11
(1981) 2 SCC 654 (Ref.) 11
AIR 1979 SC 1505 : 1979 Cri LJ 1122 (Ref.) 11
AIR 1976 SC 1994 : 1976 Cri LJ 1548 (Ref.) 11
AIR 1976 SC 2199 : 1976 Cri LJ 1718 (Ref.) 11
AIR 1974 SC 332 : 1974 Cri LJ 361 (Ref.) 11
Ananga Bhattacharya, D. Julius Riamei and Sridhar Potaraju, for Appellant; D. Bharathi
Reddy, for Respondent.
* Cri. A. No. 472 of 2004, D/- 7-3-2007 (AP)
Judgement
1. Dr. ARIJIT PASAYAT, J. :- Leave granted.
2. Challenge in this appeal is to the judgment of a learned Single Judge of the Andhra
Pradesh High Court. By the impugned judgment conviction of the appellant for offences
punishable under Sections 354 and 448 of the Indian Penal Code, 1860 (in short the 'IPC')
was upheld, but the conviction for offense punishable under Section 306 IPC was set
aside. However the sentence of three years imprisonment for offence punishable under
Section 354 IPC which was imposed by the trial court was reduced to two years. The
sentence of six months imprisonment and fine for offences relatable to Section 448 IPC
were maintained by the High Court.
3. Prosecution version in a nutshell is as follows :
Shaik Khasim Bee (hereinafter referred to as the 'deceased') is daughter of Shaik Nagoor
(PW5) and Shaik Nazer Bee (PW1). Accused, Shaik Nagoor was at the relevant point of
time the tenant in their house at Singhnagar, Vijayawada. Accused as a tenant in a small
hut in the same compound of the house of PW 1. It appears that accused was soliciting
the deceased for sexual intercourse. On 12.11.1999 around 1.00 pm. PW 1 and the
deceased went for Namaz and thereafter deceased returned home while PW-1 was
coming behind after talking to one Kursheed Begum for some time. When the deceased
came home and went into middle portion of the house, which was vacant for collecting
dried clothes, accused allegedly came behind, caught hold of her, and when
@page-SC1501
she threatened him saying that she would complain to her mother about the acts of the
accused, he in turn replied that he himself, would complain to her mother saying that she
herself called him and thereby, would defame her and her family. Feeling disturbed and
suffering from emotional turmoil, deceased went into room, poured kerosene and set fire
to herself. On receipt of intimation from the Hospital, police of Nunna Rural Police
Station, Vijayawada City, registered a case against the accused in Crime No. 258 of 1999
for the offences punishable under Sections 448, 354 and 306 IPC and after investigation
filed charge sheet and the same was taken on file in S.C. No. 181 of 2001. Accused
pleaded innocence and false implication.
4. In order to further its version prosecution examined 12 witnesses and marked several
documents. The trial court placed reliance on the dying declaration (Exh. P4-P9) recorded
by the learned 7th Additional Senior Civil Judge, City Civil Court, Hyderabad and the
Head Constable respectively on 12.11.1999. The High Court found that offence under
Section 306 IPC as noted above was not made out. However, concurred with the learned
trial judge that the offences punishable under Sections 354 and 448 IPC were clearly
made out. Accordingly the impugned judgment was passed.
5. In support of the appeal, learned counsel for the appellant submitted that the dying
declarations should not have been relied upon by the trial court and the High Court. It
was his case that considering the extent of burns sustained by the deceased it was
impossible on her part to give any dying declaration.
6. Learned counsel for the respondent on the other hand supported the impugned
judgment of the High Court.
7. We see no reason to doubt the veracity of the dying declarations especially since there
is consistency between them. We see no reason why the judicial officer should make a
false statement about the dying declaration.
8
. As observed by this Court in Narain Singh v. State of Haryana AIR vide para 7 : (SCC p.
267, para 7) 2004 AIR SCW 902

"A dying declaration made by a person on the verge of his death has a special sanctity as
at that solemn moment a person is
most unlikely to make any untrue statement. The shadow of impending death is by itself
guarantee of the truth of the statement of the deceased regarding the circumstances
leading to his death. But at the same time the dying declaration like any other evidence
has to be tested on the touchstone of credibility to be acceptable. It is more so, as the
accused does not get an opportunity of questioning veracity of the statement by cross-
examination. The dying declaration if found reliable can form the base of conviction."
9

. In Babulal v. State of M.P. (2003 (12) SCC 490) this Court observed vide in para 7 of
the said decision as under : (SCC p. 494) 2003 AIR SCW 7074, Para 7

"A person who is facing imminent death, with even a shadow of continuing in this world
practically non-existent, every motive of falsehood is obliterated. The mind gets altered
by most powerful ethical reasons to speak only the truth. Great solemnity and sanctity is
attached to the words of a dying person because a person on the verge of death is not
likely to tell lies or to concoct a case so as to implicate an innocent person. The maxim is
'a man will not meet his Maker with a lie in his mouth' (nemo moriturus praesumitur
mentiri). Mathew Arnold said, 'truth sits on the lips of a dying man'. The general principle
on which the species of evidence is admitted is that they are declarations made in
extremity, when the party is at the point of death, and when every hope of this world is
gone, when every motive to falsehood is silenced and mind induced by the most powerful
consideration to speak the truth; situation so solemn that law considers the same as
creating an obligation equal to that which is imposed by a positive oath administered in a
court of justice."
10. In Ravi v. State of T.N. ((2004 (10) SCC 776) this Court observed that : (SCC p. 777,
para 3)
"If the truthfulness of the dying declaration cannot be doubted, the same alone can form
the basis of conviction of an accused and the same does not require any corroboration,
whatsoever, in law."
11

. In Muthu Kutty v. State (2005 (9) SCC 113) vide para 15 this Court observed as under :
(SCC pp. 120-21) 2004 AIR SCW 7396, Para 15

"15. Though a dying declaration is entitled to great weight, it is worthwhile to note that
the accused has no power of cross-examination 1992 AIR SCW 2050

@page-SC1502
Such a power is essential for eliciting the truth as an obligation of oath could be. This is
the reason the court also insists that the dying declaration should be of such a nature as to
inspire full confidence of the court in its correctness. The court has to be on guard that the
statement of the deceased was not as a result of either tutoring, or prompting or a product
of imagination. The court must be further satisfied that the deceased was in a fit state of
mind after a clear opportunity to observe and identify the assailant. Once the court is
satisfied that the declaration was true and voluntary, undoubtedly, it can base its
conviction without any further corroboration. It cannot be laid down as an absolute rule
of law that the dying declaration cannot form the sole basis of conviction unless it is
corroborated. The rule requiring corroboration is merely a rule of prudence. This Court
has laid down in several judgments the principles governing dying declaration, which
could be summed up as under as indicated in Paniben v. State of Gujarat (1992 (2) SCC
474) : (SCC pp. 480-81, paras 18-19)
(Emphasis supplied)

(i) There is neither rule of law nor of prudence that dying declaration cannot be acted
upon without corroboration. (See Munnu Raja v. State of M.P. (1976 (3) SCC 104)
AIR 1976 SC 2199

(ii) If the Court is satisfied that the dying declaration is true and voluntary it can base
conviction on it, without corroboration. (See State of U.P. v. Ram Sagar Yadav and
Ramawati Devi v. State of Bihar (1985 (1) SCC 552) AIR 1985 SC 416
AIR 1983 SC 164

(iii) The court has to scrutinise the dying declaration carefully and must ensure that the
declaration is not the result of tutoring, prompting or imagination. The deceased had an
opportunity to observe and identify the assailants and was in a fit state to make the
declaration. (See K. Ramachandra Reddy v. Public Prosecutor (1976 (3) SCC 618)
AIR 1976 SC 1994

(iv) Where dying declaration is suspicious, it should not be acted upon without
corroborative evidence. (See Rasheed Beg v. State of M.P. (1974 (4) SCC 264) AIR
1974 SC 332

(v) Where the deceased was unconscious and could never make any dying declaration the
evidence with regard to it is to be rejected. (See Kake Singh v. State of M.P. (1981
Supp.SCC 25) AIR 1982 SC 1021

(vi) A dying declaration which suffers from infirmity cannot form the basis of conviction.
(See Ram Manorath v. State of U.P.(1981 (2) SCC 654)

(vii) Merely because a dying declaration does not contain the details as to the occurrence,
it is not to be rejected. (See State of Maharashtra v. Krishnamurti Laxmipati Naidu (1980
Supp. SCC 455) AIR 1981 SC 617
(viii) Equally, merely because it is a brief statement, it is not to be discarded. On the
contrary, the shortness of the statement itself guarantees truth. (See Surajdeo Ojha v. State
of Bihar (1980 Supp. SCC 769)) AIR 1979 SC 1505

(ix) Normally the court in order to satisfy whether deceased was in a fit mental condition
to make the dying declaration look up to the medical opinion. But where the eye-witness
said that the deceased was in a fit and conscious state to make the dying declaration, the
medical opinion cannot prevail. (See Nanhau Ram v. State of M.P. (1988 Supp. SCC
152)) AIR 1988 SC 912

(x) Where the prosecution version differs from the version as given in the dying
declaration, the said declaration cannot be acted upon. (See State of U.P. v. Madan
Mohan (1989 (3) SCC 390)) AIR 1982 SC 1519

(xi) Where there are more than one statement in the nature of dying declaration, one first
in point of time must be preferred. Of course, if the plurality of dying declaration could
be held to be trustworthy and reliable, it has to be accepted. (See Mohanlal Gangaram
Gehani v. State of Maharashtra (1982 (1) SCC 700))". AIR 1982 SC 839

12. So far as the practicability of the deceased giving dying declaration is concerned it is
significant that the learned Additional Senior Civil Judge who has examined PW 7 and
the constable PW 10 have described in detail as to what the deceased has stated to each
one of them. There was not even any suggestion to either of the witnesses that the
deceased was not in a fit condition to give any statement as claimed. That being so, there
is no substance in the plea of learned counsel for the appellant that the deceased was not
in a physical condition to
@page-SC1503
give a statement.
13. The trial Court and the High Court have analysed the evidence of these witnesses and
the statements made in the dying declaration referred to above to hold the accused guilty.
14. That being so, no interference is called for. The appeal fails and is dismissed.
Appeal dismissed.
AIR 2008 SUPREME COURT 1503 "V. J. Thomas v. Pathrose Abraham"
(From : AIR 2005 Ker 252)
Coram : 2 S. B. SINHA AND H. S. BEDI, JJ.
Civil Appeal No. 989 of 2008 (arising out of SLP (C) No. 20338 of 2005), D/- 5 -2 -2008.
V.J. Thomas v. Pathrose Abraham and Ors.
Civil P.C. (5 of 1908), O.1, R.8 and O.1, R.10 - CIVIL PROCEDURE - DECLARATION
OF TITLE - POSSESSION - SPECIAL LEAVE APPEAL - Representative suit -
Impleadment of third party - Suit for declaration of title and possession - Prayer for
restraining defendants from using property as if it is public pathway - Impleadment of
users of pathway for purpose of examination of question as to whether notice under O.1,
R.10 was published in newspaper having wide circulation in locality - No interference
under Art.136.
Constitution of India, Art.136.
A litigant may execute a decree, which was obtained for the benefit of the people of the
locality, but if he intends to execute a decree which was obtained for his own benefit,
those who would be affected thereby should ordinarily be made parties to the suit.
Similarly, if a village pathway is the subject-matter of the suit on the premise that it is the
personal property of the plaintiff, those who use the said pathway or at least have lands
adjacent thereto should ordinarily be impleaded as parties. In the latter case, a decree,
which has been obtained by suppression of fact or collusively would not be executable
against those who were not parties to the suit. (Para 14)
Where in a suit for declaration of title and possession of property, a prayer was made for
restraining the defendants from using the property as a pathway as if it was a pathway, if
for the purpose of examination of question whether notice in terms of O. 1 R. 10 was
published in newspaper having wide circulation in the locality, the persons, users of
pathway, were impleaded by the Court so as to enable them to press their applications for
setting aside the ex parte decree upon condonation of delay, the order cannot be interfered
with under Art. 136. (Para 18)
Cases Referred : Chronological Paras
2005 AIR SCW 2738 : AIR 2005 SC 2544 (Ref.) 17
1998 AIR SCW 2772 : AIR 1998 SC 2722 : 1998 Lab IC 3021 (Ref.) 17
AIR 1990 SC 642 (Ref.) 15
AIR 1923 Mad 472 (2) (Ref.) 11
V.J. Francis, P.I. Jose, Anupam Mishra, for Appellant; M.T. George, for Respondents.
Judgement
1. S. B. SINHA, J. :- Leave granted.
2

. A question of some importance as to whether a third party can be impleaded in a suit


filed under Order 1 Rule 8 of the Code of Civil Procedure is involved in this appeal
which arises out of the judgment and order dated 30th June, 2005 passed by a Learned
Single Judge of the Kerala High Court in CRP No. 272 of 2004 dismissing the revision
application filed by the appellant herein challenging the order dated 17-10-2003 passed
by Additional Sub Court, Kottayam. Reported in AIR 2005 Ker 252

3. The basic fact of the matter is not in dispute.


4. Appellant herein filed Original Suit No. 364 of 1997 in the Court of Additional Sub
Court, Kottayam, for declaration of his title and for possession of the suit property. A
prayer therein was made for restraining the defendants who were three in number
(Respondent Nos. 3 to 5 herein) from interfering with his right to enjoy the property by
entering upon it or using it as a pathway as if it was a public pathway. A purported
publication was also made in a newspaper purported to be in terms of Order 1 Rule 10 of
the Code of Civil Procedure.
5. Respondent Nos. 3 to 5 herein did not appear in the suit as a result whereof, an ex parte
decree was passed on 26.3.1998. An application for execution of the said decree was
filed. Respondent Nos. 1 and 2 herein in the said execution case filed five applications,
the details whereof are as under :
I.A.No.965/2002- Under Order 9 Rule 13, for setting aside the ex parte decree.
I.A.No.966/2002- For Condonation of Delay.
I.A.No.967/2002- Application for Stay of
@page-SC1504
all the proceedings in the execution.
I.A.No.968/2002- Application filed by the respondent No. 1 for permission to contest the
suit and to add himself in the array of parties as additional defendant No. 1.
I.A.No. 969/2002- Application filed by respondent No.2 for permission to contest the suit
and for making her as additional defendant No. 5.
6. The Executing Court allowed I.A. No. 968 of 2002 and 969 of 2002 impleading
respondent Nos. land 2 as defendant Nos. 5 and 7 in the suit. The Revision Application
filed thereagainst has been dismissed by a single Judge of the High Court by reason of the
impugned judgment.
7. Mr. V. J. Francis, learned counsel appearing on behalf of the appellant, submitted that a
suit, wherein notice under Order 1 Rule 8 of Code of Civil Procedure has been issued,
could not have been reopened at the instance of the respondent Nos. 1 and 2 herein
without allowing their application for condonation of delay and for setting aside the ex
parte decree. It was urged that no case has been made out for condonation of delay as the
applications for setting aside the ex parte decree was filed after more than four years from
the passing thereof.
Respondent Nos. 1 and 2, in any event, it was contended, could not have represented the
panchayat and, thus, the impugned order is unsustainable.
8. Mr. M.T. George, learned counsel appearing on behalf of the respondent, however,
would support the impugned judgment.
9. The suit land measuring 450 ft. x 4 ft. starts from Vattachalpady junction of
Manarcadu-Thengana PWD road and ends at Kuttiyilpadyperumpanachi Panchayat road
on the east.
Respondent Nos. 1 and 2 were not parties to the suit. They, in their applications, inter
alia, contended that the appellant and the respondent Nos. 3 to 5 herein are neighbours
and close associates. The suit was a collusive one. Leave of the Court under Order 1 Rule
8 was obtained on a misrepresentation.
10. A specific contention was also raised that plaintiff-petitioner deliberately and
intentionally had not impleaded the users of the pathway in the said suit. It was stated that
publication of the notice purported to be under Order 1 Rule 8 CPC was made in the
newspapers which did not have wide circulation in the locality.
11

. The High Court has, inter alia, relying upon the decision of the Madras High Court in
Swaminatha Mudaliar vs. Kumaraswami Chettiar and others [(1923) 44 MLJ 282]
accepted the said contentions of the respondent, holding : AIR 1923 SC 472(2)

"It may be unusual to bring fresh plaintiffs on the record after a decree has been passed;
but there is authority for doing so under Order 1 Rule 10.
Order 1 Rule 8 expressly permits any person on whose behalf a representative suit is
instituted to apply to the Court to be brought on the record, and the words of this rule are
not limited, as they are by Order 1 Rule 10 by the purpose being expressed as that of
adjudication on the questions arising in the suit."
12. The plaint is not before us. The application purported to have been filed by the
applicant under Order 1 Rule 8 is also not before us. On what basis, the respondent Nos.
3 to 5 were impleaded in the suit and in which capacity, thus, is not known.
13. A suit filed in terms of Order 1 Rule 8 should ordinarily be premised on the ground
that the defendants represent the parties interested in the suit. Defendants in such a suit,
although, must be able to represent the public in general, but no personal decree can be
passed against them. To what extent the original defendants were interested in the suit
property at least in respect of the portion thereof is not known.
14. A litigant may execute a decree which was obtained for the benefit of the people of
the locality but if he intends to execute a decree which was obtained for his own benefit,
those who would be affected thereby should ordinarily be made parties to the suit.
Similarly, if a village pathway is the subject matter of the suit on the premise that it is the
personal property of the plaintiff, those who use the said pathway or at least have lands
adjacent thereto should ordinarily be impleaded as parties. In the latter case, like the
present one, applying the legal principles, as noticed hereinbefore, we are of the opinion
that a decree which has been obtained by suppression of fact or collusively would not be
executable against those who were not parties to the suit.
15

. Reliance has been placed by Mr. AIR 1990 SC 642, Para 8

@page-SC1505
Francis on Chairman, Tamil Nadu
Housing Board, Madras v. T.N. Ganapathy [(1990) 1 SCC wherein the question which
arose for consideration centered around the maintainability of the suit at the instance of
the plaintiffs who belonged to the category of low income group against Housing Board
for a permanent injunction from demanding and calculating from the allottees any
additional price, was answered in the following terms :
"Coming to the relevant circumstances in the present case, it will be seen that all the
allotments in Ashok Nagar were made under the same scheme and all the relevant facts
are common. The basis of the impugned demand of the appellant is equally applicable to
all the allottees and the plea of the plaintiff is available to all of them. The trial court was,
therefore, perfectly right in permitting the plaintiff to proceed under Order 1, Rule 8 of
the Code of Civil Procedure. Nobody in this situation can complain of any inconvenience
or injustice. On the other hand, the appellant is being saved from being involved in
unnecessary repeated litigation."
16. As indicated hereinbefore, we have no idea as to what was the nature of interest was
claimed by the Original Defendant Nos. 1 to 3 (Respondent Nos. 3 to 5) in the suit land.
In any event, whether the service of notice was proper would also be the subject matter of
an enquiry by the learned trial court. It has also to be seen as to whether the notice in
terms of Order 1 Rule 10 of the Code was published in a newspaper having a wide
circulation in the locality.
17

. In Union of India and Ors. v. Dinanath Shantaram Karekar and Ors. [1998 (4) SCALE
659], this Court held : 1998 AIR SCW 2772, Para 4

"So far as the service of show cause notice is concerned, it also cannot be treated to have
been served. Service of this notice was sought to be effected on the respondent by
publication in a newspaper without making any earlier effort to serve him personally by
tendering the show cause notice either through the office peon or by registered post.
There is nothing on record to indicate that the newspaper in which the show-cause notice
was published was a popular newspaper which was expected to be read by the public in
general or that it had wide circulation in the area or locality where the respondent lived.
The show-cause notice cannot, therefore, in these circumstances, be held to have been
served on the respondent. In any case, since the very initiation of the disciplinary
proceedings was bad for the reason that the charge sheet was not served, all subsequent
steps and stages, including the issuance of the show-cause notice would be bad."

In Church of North India v. Lavajibhai Ratanjibhai and Ors. [(2005) 10 SCC 760], it was
observed 2005 AIR SCW 2738, Para 76

"71... bars a suit to enforce a right on behalf of a public trust. C.N.I. got itself registered
as a public trust in the year 1981. A suit evidently was filed by the plaintiffs in the year
1980 because C.N.I. was not then entitled to file a suit. It may be true that the suit was
filed under Order 1, Rule 8 of the Code of Civil Procedure but therein the question as to
whether the Appellant herein, being a registered trust became entitled to the properties of
Brethren Church could not have been gone into. What is prohibited is to enforce a right
on behalf of a public trust. When the plaintiffs intended to enforce a right on behalf of the
Appellant, the suit was evidently not maintainable."
18. If for the purpose of examination of the said question, amongst others, the executing
court has allowed the applications for impleadment of the respondent Nos. 1 and 3 herein
as defendant Nos. 4 and 5, so as to enable them to press their applications for setting
aside the ex parte decree upon condonation of delay; we do not see any reason to interfere
therewith in exercise of our discretionary jurisdiction under Article 136 of the
Constitution of India.
19. For the reasons above mentioned, there is no merit in this appeal which is accordingly
dismissed with costs. Counsel's fee assessed at Rs. 10,000/- (Rupees ten thousand only).
Appeal dismissed.
AIR 2008 SUPREME COURT 1505 "J. Ramulu v. State of A. P."
(From : Andhra Pradesh)
Coram : 2 PRAKASH PRABHAKAR NAOLEKAR AND LOKESHWAR SINGH
PANTA, JJ.
Criminal Appeal No. 758 with 763 of 2006, D/- 26 -2 -2008.
J. Ramulu v. State of A.P.

WITH
G. Venkatesh v. State of A.P. @page-SC1506
Evidence Act (1 of 1872), S.32 - Penal Code (45 of 1860), S.300 - DYING
DECLARATION - MURDER - Dying declaration - Reliability - Deceased suffering
multiple burn injuries on face, neck and chest due to acid allegedly thrown by accused -
Unable to see or speak - Declaration recorded by Police Officer immediately after
incident not adduced in evidence - Second declaration naming accused recorded by
Magistrate days after hospitalisation - Declaration though recorded only on basis of signs
and gestures was detailed and lengthy - Doctor present at time of recording neither named
nor examined - Eye witnesses who were close relations of deceased stating that accused
were named by deceased because of tutoring - Declaration is doubtful - Conviction
cannot be rested on it.
Crl. A. No. 2290 of 2004, D/-22-12-2005 (AP), Reversed. (Paras 12, 13, 15)
Cases Referred : Chronological Paras
2006 AIR SCW 1053 : AIR 2006 SC 1319 : 2006 Cri LJ 1629 (Rel. on) 14
Sushil Kumar, Sr. Advocate, A. Dasharatha, Aditya Kumar, Naveen R. Nath, U.U. Lalit,
Sr. Advocate, Guntur Prabhakar, for Appellants; Ms. Altaf Fathima, (for Mrs. D. Bharathi
Reddy) for Respondent.
Judgement
1. LOKESHWAR SINGH PANTA, J. :-The above-said two appeals relate to single
incident and are directed against common Judgment dated 22nd December, 2005 passed
by the High Court of Judicature, Andhra Pradesh at Hyderabad in Criminal Appeal No.
2290 of 2004, they are heard together and shall stand disposed of by this common
judgment.
2. Criminal Appeal No. 763 of 2006 has been filed by G. Venkatesh (A-1) against his
conviction under Section 302 of the Indian Penal Code [for short 'IPC'] and sentence to
undergo imprisonment for life and to pay a fine of Rs.2000/-, in default thereof to suffer
simple imprisonment for two months. Criminal Appeal No. 758 of 2006 has been filed by
J. Ramulu (A-2) against his conviction under Section 302 read with Section 34 of IPC
and sentence to undergo imprisonment for life and to pay a fine of Rs.2000/-, in default
thereof to suffer simple imprisonment for two months, passed by II Additional
Metropolitan Sessions Judge, Hyderabad and confirmed by the High Court in Criminal
Appeal No. 2290 of 2004.
3. In all, nine accused persons were tried by the II Additional Metropolitan Sessions
Judge, Hyderabad, in Sessions Case No.352 of 2001 for the offences under Section 302,
IPC, and Section 302, IPC, read with Section 34 of IPC and Section 109 of IPC. A-1 and
A-2 were found guilty of the murder of G. Janardhan, while other seven accused were
acquitted of the charges.
4. Briefly stated, the case of the prosecution against A-1, A-2 and other accused persons
is that G. Janardhan was resident of Anandnagar Colony, Malakpet. A-1 is resident of
R.K. Puram, whereas A-2 and A-7 are residents of Dilshuknagar and A-3 and A-6 are
residents of Chaitanyapuri. A-4 is resident of Vanaparthy, Mahaboobnagar District, A-5 is
resident of Kothakota, Mahaboobnagar District, and A-8 and A-9 are residents of
Jadcherla, Mahaboobnagar District. A-1, A-2 and G. Janardhan were partners in
Manjunadha Rice Mill and they had some disputes between themselves and other
partners regarding the mill transactions. G. Janardhan filed O.S. No.92 of 1999 on the file
of the I Additional Senior Civil Judge, Ranga Reddy, against A-1 and A-2 for dissolution
of the partnership of the mill. Second suit O.S.No.579 of 1994 on the file of the II
Additional Senior Civil Judge, Ranga Reddy, is also pending between the parties. G.
Janardhan also filed C.C. No. 114 of 1998 on the file of the Additional Judicial First
Class Magistrate, Hyderabad, inter alia alleging that the accused persons cheated him in
the transactions of the rice mill.
5. On 08.07.2000 at about 4.00 P.M., G. Janardhan along with his eldest paternal uncle G.
Satyanarayana went to Ramkrishna Muth and after completing of their prayers, they
came back to their residence by city bus. After getting down from the bus, G.
Satyanarayana went to his house, while G. Janardhan proceeded to go to his house, A-1
and A-2 stated to have come from the rice mill road on a Scooter. A-1 was pillion rider,
he sprinkled acid on the face of G. Janardhan with a Mug. G. Jandardhan shouted for help
which attracted the attention of G. Raju, son (PW-1), Jamuna Rani, daughter (PW-2), G.
Savithri, wife of G. Janardhan (PW-3), who were sitting in the verandah of their house.
They immediately came to the spot and took G. Janardhan to Yashoda Hospital for
medical treatment. G. Janardhan received burn injuries on his
@page-SC1507
face, chest and neck. PW-1 went to the Police Station Chaderghat, Hyderabad and handed
over complaint (Ex.P1) to Shri B. Sivaranireddy (PW-9), who, at the relevant time, was
Sub-Inspector of the Police Station, Chaderghat. On the basis of the said complaint, FIR
(Ex.P26) was registered under Section 307 read with Section 34 of IPC. G. Guravaiah
(PW-13), Sub-Inspector of Police took up investigation of the crime at 10.20 P.M. on the
same day. He went to Yashoda Hospital, Malakpet, where injured G. Janardhan was
admitted in emergency ward. He recorded the statements of PWs-1, 2, 3 and G. Anuradha
(PW-4) wife of PW-1 and daughter-in-law of injured G. Janardhan. On the next day, i.e.
09-07-2000, at about 7.00 A.M., the Investigating Officer recovered burnt leaves of small
plants and acid-mixed earth and control earth from the place of occurrence. He again
went to Yashoda Hospital where the injured G. Janardhan gave his statement by gestures
and writing on small chits to him. The statement of injured G. Janardhan was recorded
after obtaining permission from the Doctor. On the same day at about 8.00 P.M., the
Investigating Officer apprehended A-1 and A-2 on the road in front of the house of A-1.
He recorded the disclosure statement of A-1 which is marked as Ex.P24 and on the basis
of the said statement, Scooter bearing No. AP 28 L 2745 was recovered and one green
colour mug was taken into possession, from inside the dicky of the Scooter. On
requisition Ex.P27 sent by the Inspector of Police, B. Gyaneshwar Rao (PW-11) XIV
Metropolitan Magistrate, Hyderabad, at midnight of 11/12-07-2000, went to Yashoda
Hospital and recorded statement made by G. Janardhan to him by gestures and signs as
the injured was unable to see and talk due to burn injuries. On 25.07.2000, P. William
Caruy (PW-12) received information in regard to the death of injured G.Janardhan at
about 12.30 A.M. He converted the offence in the FIR from Section 307 of IPC to
Section 302 of IPC. After receipt of the dead body and completion of the investigation,
charge-sheet was filed against the above said nine accused persons.
6. The accused pleaded not guilty to the charges and claimed to be tried. The prosecution
examined as many as 13 witnesses and produced on record 32 documents in support of its
case. The accused persons in their statements recorded under Section 313, Cr.P.C. pleaded
not guilty to the charges and claimed to be tried. No defence witness has been examined
by them. The trial court, on appreciation of the oral and documentary evidence, found A-
1 and A-2 guilty of the charge of murder of G. Janardhan and convicted and sentenced
them as aforesaid, while no case has been found against A-3, A-4 and A-6 to A-9,
therefore they are acquitted of the charge. During the pendency of the trial, A-5 had
expired, therefore, the trial stood abated against him.
7. Being aggrieved against the Judgment and Order of the learned Trial Judge, A-1 and A-
2 filed appeal under Section 374 (2) of the Cr.P.C. before the High Court. The High Court
dismissed their appeal and confirmed their sentence. Hence, A-1 and A-2 have filed these
appeals by special leave.
8. We have heard Mr. U. U. Lalit, learned senior Advocate appearing on behalf of A-1,
Mr. Sushil Kumar, learned senior Advocate appearing on behalf of A-2, and Ms. Altaf
Fathima, Advocate for respondent-State, and with their assistance, we have examined the
entire oral and documentary evidence on record. The learned counsel for A-1 and A-2,
Inter alia, contended : (a) that in the Yashoda Hospital record where injured G. Janardhan
was admitted on 08-07-2000, it was specifically stated therein that some unknown
offenders had thrown acid on the face of the injured G. Janardhan, the general diary
number column in the FIR has been left blank, which would suggest that the first
recorded information, being the intimation by the Hospital authorities referring to
unknown persons as the culprits, has been suppressed; (b) PW-1, the son of G. Janardhan,
clearly stated that on the evening of 09-07-2000, Police got complaint (Ex.P1) made from
him in which the names of A-1 and A-2 were disclosed at the instance of their relatives,
on the basis of which tutored FIR (Ex.P25) came to be registered; (c) PW-4, daughter-in-
law of G. Janardhan, also clearly stated that Police got complaint (Ex.P1) recorded from
her husband (PW-1) at the instance of their relatives; and (d) the so-called dying
declaration (Ex.P28) recorded by PW-11- the Magistrate, on 11/12-07-2000 at midnight
mentioning the names of A-1 and A-2 along with the names of A-3 to A-9, was as a result
of due deliberation and tutoring of G. Janardhan by his relatives as per the admission of
PW-1 and the statement allegedly made by G.
@page-SC1508
Janardhan before the Magistrate was vague and there is no specific reference to A-1 and
A-2 implicating them in the commission of the offence. Lastly, the learned counsel
submitted that the trial court and the High Court have considered the alleged dying
declaration as partly untrue in respect of accused A-3 to A-9, who were acquitted of the
charges and partly true against A-1 and A-2 without any corroboration from independent
evidence and, therefore, no implicit reliance could have been placed on tutored dying
declaration.
9. Per contra, the learned counsel appearing on behalf of the respondent-State contended
that the evidence of PW-11 is very clear, who recorded dying declaration (Ex.P28) of G.
Janardhan in which the names of A-1 and A-2, who poured acid on his face which caused
his death, were mentioned by the deceased. The learned counsel also contended that the
trial court as well as the High Court have appreciated the entire evidence in its right
perspective and this Court shall be slow to interfere in the well-reasoned and well-
merited judgments of the Courts below.
10. We have given our anxious and thoughtful consideration to the respective contentions
of the learned counsel for the parties. We may, at the outset, record that PW-1, son, PW-2,
daughter, PW-3, wife, and PW-4, daughter-in-law of G. Janardhan, the alleged witnesses
of the occurrence, have not supported the case of the prosecution at all and despite
searching cross-examination by the learned Public Prosecutor, no material evidence is
elicited from their testimony to implicate A-1 and A-2 in the commission of the crime.
PW-1, the son of the deceased, on 08-07-2000 submitted report (Ex.P1) to the police at
Police Station, Chaderghat, on the basis of which FIR (Ex.P25) was registered at the
Police Station. He deposed before the Court that he gave complaint (Ex.P1) to the Police
at the instance of their relatives. Even this witness did not say in his statement under
Section 164 of the Code of Criminal Procedure that he and his sister (PW-2), his mother
(PW-3) and his wife (PW-4) witnessed the incident and/or his father disclosed the names
of A-1 and A-2 who sprinkled acid on his face. It is his evidence that on the day of the
occurrence they heard the sound of cry of his father near the gate of their house and he
along with PWs-2 and 3 rushed to the spot and found acid burns on the face and neck of
his father who was not in a position to speak nor he could open his eyes. The incident
took place in front of the house of injured G. Janardhan at around 8 P.M. PW-1
categorically stated that he noticed two unknown persons going on Scooter at the place of
occurrence. PW-1 deposed that his father had given the names of A-1 and A-2 and other
seven accused persons to PW-11 the Magistrate, on having tutored by their relatives. This
witness in cross-examination conducted by the learned counsel for A-1 and A-2
categorically stated that he did not know the persons who poured acid on the face of his
father. It is his evidence that their relatives came to Yashoda Hospital where his father
was taken immediately after the incident. The Police also reached at the Hospital and
wanted to record his father's statement but his father was not in a position to speak, so his
statement could not be recorded. He stated that his father before the incident had been
telling him that naxalites were threatening him with dire consequences, if money was not
paid to them.
11. PW-2 daughter of G. Janardhan, deposed that on the day of incident, i.e. 08-07-2000,
she was not at the house of her parents, but she was at a house in Warasiguda when at
about 7.00 or 7.30 P.M. she received telephone call that her father had been taken to
Yashoda Hospital for medical treatment for acid burns. She deposed that she did not
know who caused acid burns injuries to her father. She denied the suggestion of the
prosecution that she deposed against the prosecution in order to help the accused. It is the
evidence of PW3 wife of the deceased that on 08-07-2000 at about 7.30 or 8.00 P.M.
when she was in her house, she heard some sound of cry. She came out of her house and
saw her husband coming inside the gate of their house, who was unable to speak. He was
shifted to Yashoda Hospital as he received acid burns. She did not see the persons who
caused acid burns to him. The suggestion of the Public Prosecutor that she has resiled
from her earlier statement in order to help A-1 and A-2, is denied by her. Similarly, PW4
wife of PW-1 has not supported the prosecution version. According to this witness, her
father-in-law received acid burns at 7.30 or 8.00 P.M. near their house when she along
with her husband and mother-in-law was present in the house. They took injured
@page-SC1509
G.Janardhan to Yashoda Hospital where he was admitted in emergency ward. She was not
allowed to go inside the ward while PWs 1, 2 and 3 were allowed to go inside the ward.
Their relatives were also present in the hospital. In cross-examination by the learned
Public Prosecutor, she stated that her father-in-law was in Intensive Care Unit for 15 days
and thereafter he was shifted to Apollo Hospital. Their relatives were regularly visiting
her father-in-law in the unit. She stated that on 09-07-2000, their relatives and police got
a report drafted from her husband against A-1 and A-2.
12. The oral evidence of the eye-witnesses, discussed above, who are none else than the
son, daughter, wife and daughter-in-law of the deceased, have not supported the
prosecution version to prove that it were A-1 and A-2 who poured acid on the face, neck,
etc. of the deceased G.Janardhan on the day of occurrence. PWs 1, 2, 3 and 4 are the
truthful witnesses whose testimony has to be accepted without any embellishment. The
family members of G. Janardhan would have not spared A-1 and A-2, if they in fact were
the real culprits who caused acid burns injuries on his person. The evidence of the eye-
witnesses clinches that the deceased had named A-1 and A-2 as assailants in dying
declaration made to PW-11, the Magistrate, on the intervening night of 11/ 12-07-2000 on
being tutored by his relatives during the period 07/08.07.2000 to 11/ 12-07-2000, when
they had gone to visit him in the ward of the hospital. PW-11 recorded original dying
declaration of the deceased G.Janardhan in Telugu. The evidence of PW-11 would show
that on 11/12-07-2000 at 12 O' Clock mid-night he received requisition (Ex.P27) from the
Inspector of Police, Police Station Chaderghat requesting him to record dying declaration
of G.Janardhan at Yashoda Hospital. He stated that the declarant was not able to speak
and see due to burn injuries but he was responding by gestures to the questions put to
him. He gave him pen and a paper to write his statement who wrote the answers on the
proceedings of the dying declaration. On perusal of the dying declaration, it reveals that
the deceased had given the names of A-1 and A-2 and other accused persons who were
acquitted by the trial court. It has come in the cross-examination of PW-11 that he did not
make any endorsement on the proceedings of the dying declaration that the declarant was
physically and mentally fit throughout the proceedings. Similarly, the Doctor on duty also
did not specifically state in his endorsement that the declarant was physically and
mentally in a fit state to make the statement. PW-11 also admitted suggestion of the
defence that in Ex.P28 he did not mention that he disclosed his identity to the declarant
before recording dying declaration.
13. We have carefully examined the reasons recorded by the learned Sessions Judge and
the High Court for accepting the dying declaration as credible and for accepting the
evidence of Magistrate who recorded the alleged dying declaration (Ex.P28) as bringing
home the charge of murder against A-1 and A-2 beyond shadow of reasonable doubts.
The dying declaration allegedly made by the deceased, in our view, is not free from doubt
and embellishment. It is the specific case of PW-13 G. Guravaiah, Sub-Inspector, who
went to the hospital immediately on receipt of the complaint (Ex.P1) and found injured
G.Janardhan admitted in the Yashoda Hospital on 09-07-2000 that he took the permission
of the duty Doctor in regard to the physical and mental condition of the deceased. As per
his version, the Doctor certified that injured G.Janardhan was fit to make statement. He
recorded the statement of injured G.Janardhan on small chits which he did not place on
record of the case. This version of the Investigating Officer clearly and plainly shows that
on 09-07-2000 G.Janardhan made some statement to the Investigating Officer which in
all probability did not contain the names of A-1 and A-2 who were responsible for
throwing acid on his face, neck and chest. Those chits were important documentary
evidence which was deliberately withheld by the prosecution from the Court with clear
intention of suppressing the true version of G. Janardhan subscribed by him on some
paper chits at the first available opportunity. G.Janardhan remained in Yashoda Hospital
from 08-07-2000 till 24-07-2000 on which date he left the hospital and got himself
admitted in Apollo Hospital where he died on the next day. The medical report of
Yashoda Hospital reveals that injured G.Janardhan had refused to undergo surgery and
got discharged from the hospital against medical advice. PWs-2 and 4 clearly and plainly
deposed that the deceased made tutored statement to PW-11 at the behest of their
relatives who had been the regular visitors of
@page-SC1510
the ward where the deceased before death was lying and they had compelled the deceased
to mention the names of A-1 and A-2 along with other accused. The suppression and
withholding of the first dying declaration of the deceased recorded by PW-13 on 09-07-
2000, by itself creates suspicion and reasonable doubt as to the correctness and
truthfulness of the dying declaration allegedly made by the deceased to PW-11, XIV
Metropolitan Magistrate.
14

. This Court in P. Mani v. State of T.N. [(2006) 3 SCC 161], while dealing with the
question of dying declaration, held that conviction can be recorded on the basis of the
dying declaration alone but the same must be wholly reliable. In a case where suspicion
can be raised as regards the correctness of the dying declaration, the Court before
convicting an accused on the basis thereof would look for some corroborative evidence.
Suspicion is no substitute for proof. If evidence brought on records suggests that such
dying declaration does not reveal the entire truth, it may be considered only as a piece of
evidence in which event conviction may not be rested only on the basis thereof. The
question as to whether a dying declaration is of impeccable character would depend upon
several factors; physical and mental condition of the deceased is one of them. 2006
AIR SCW 1053

15. As noticed above, the medical report raised a number of questions which have not
been satisfactorily answered, which precluded implicit acceptance of the dying
declaration (Ex.P28). First, PWs 1 and 3, who took injured G.Janardhan to Apollo
Hospital and got him admitted there, have deposed that at that time G.Janardhan was not
physically and mentally fit to make the statement. Second, who was the doctor on duty at
the time of admission? PWs-1 and 2 did not say about it and the history sheet reveals that
the injured was alleged to have sustained multiple burns over face and upper part of chest
and neck when attacked with acid by unknown persons near his residence at about 8.15
P.M. PW-13 recorded the first dying declaration of the injured G.Janardhan on 09-07-
2000 in the presence of some doctor, but the name of the Doctor has not been mentioned
by him. The Doctor, in whose presence PW-11 recorded dying declaration (Ex.P28), has
not been examined by the prosecution to corroborate the correctness and truthfulness of
the dying declaration on which conviction of A-1 and A-2 has been recorded by the Trial
Court and confirmed by the High Court, while the same document was not accepted by
the courts below in respect of A-3 to A-9 whose names were also mentioned in the dying
declaration. The dying declaration shows that the deceased was not in a position to speak
and see and in such state of mind, it is highly doubtful and unbelievable that the deceased
had written such a lengthy statement running in more than 3 pages containing various
details by sign and gestures. The contents of the dying declaration are shrouded by doubts
and suspicion and the entire evidence, discussed above, suggests that the dying
declaration does not reveal the entire truth, it has to be considered only as a piece of
evidence on which no implicit reliance can be placed and in which event conviction
cannot be rested solely on the basis of such doubtful dying declaration. In the facts and
circumstances, we are not satisfied with the findings recorded by the Trial Judge and the
High Court holding A-1 and A-2 guilty of the offence on the basis of weak and slender
evidence led on record by the prosecution.
16. In our view, A-1 and A-2 are entitled to benefit of doubt. In the result, we allow the
appeals and set aside the judgments of the High Court and the Trial Court. A-1 and A-2
are acquitted of the charges against them.
17. G. Venkatesh and J. Ramulu shall be set at liberty forthwith, unless required to be
detained in connection with any other case.
Appeal allowed.
AIR 2008 SUPREME COURT 1510 "Goyal Enterprises, M/s. v. State of Jharkhand"
(From : Jharkhand)*
Coram : 2 Dr. A. PASAYAT AND J. M. PANCHAL, JJ.
Criminal Appeal No. 377 of 2008 (arising out of SLP (Cri.) No. 4710 of 2006), D/- 25 -2
-2008.
M/s. Goyal Enterprises v. State of Jharkhand and Anr.
Criminal P.C. (2 of 1974), S.378(4) - APPEAL - NATURAL JUSTICE - Leave to file
appeal against acquittal - Order granting/refusing leave has to be
@page-SC1511
reasoned - Summary disposal of application without reasons - Improper - Giving reasons
is a salutary requirement of natural justice Rules.
Crl. M. P. No. 619 of 2006, D/-26-06-2006 (Jhar), Reversed. (Paras 6, 8)
Cases Referred : Chronological Paras
2004 AIR SCW 102 : AIR 2004 SC 1203 : 2004 Cri LJ 916 (Ref.) 9
2003 AIR SCW 5095 : AIR 2003 SC 4664 : 2003 Cri LJ 5040 (Ref.) 7
(2001) 10 SCC 607 (Rel. on) 6
AIR 1987 SC 724 : 1987 Cri LJ 698 (Ref.) 6
AIR 1982 SC 1215 (Rel. on) 6
1974 ICR 120 (NIRC)8
(1971) 1 All ER 1148 8
Barun Kumar Sinha, Mrs. Pratibha Sinha, B.K. Satija, for Appellant; P.S. Mishra, Sr.
Advocate, Ravi C. Prakash, Tathagat H. Vardhan, Upendra Mishra, Dhruv Kumar Jha,
Manu Shankar Mishra, Ajit Kumar Sinha, for Respondents.
* Cri. M. P. No. 619 of 2006, D/- 26-6-2006 (Jhar).
Judgement
1. Dr. ARIJIT PASAYAT, J. :-Leave granted.
2. Challenge in this appeal is to the order passed by a Division Bench of the Jharkhand
High Court refusing to grant leave to appeal.
3. Stand of the appellant is that the order of the Division Bench summarily dismissing the
application cannot be sustained. Learned counsel for respondent No.2, on the other hand,
supported the order stating that though the order is non-reasoned, yet this is not a fit case
for exercise of power under Article 136 of the Constitution of India, 1950 (for short 'The
Constitution').
4. The application before the High Court for grant of leave was filed under Section
378(4) of the Code of Criminal Procedure, 1973 (for short The Cr.P.C.').
5. In the instant case proceeding was initiated on the basis of a complaint filed before the
learned Judicial Magistrate, Ist Class, Jamshedpur alleging commission of offence
punishable under Section 138 of the Negotiable Instruments Act, 1881 (for short 'The
Act'). The accused who is respondent No.2 in the petition was found guilty, and was
accordingly, convicted and sentenced to undergo simple imprisonment for six months. He
was also directed to pay by way of compensation the cheque amount of Rs.61,860/- and
Rs.62,860/- to the complainant within one month from the passing of the order. The
accused filed a petition for revision before the Sessions Court. Learned Additional
Sessions Judge, Fast Track Court No.2, Jamshedpur, by order dated 2.3.2006 set aside the
judgment of conviction and sentence as passed by the learned Judicial Magistrate.
Thereafter, as noted above, application in terms of Section 378(4), Cr.P.C, was filed. The
same has been dismissed summarily by a Division Bench of the High Court.
6

. The High Court has not given any reasons for refusing to grant leave to file appeal
against acquittal, and seems to have been completely oblivious to the fact that by such
refusal, a close scrutiny of the order of acquittal, by the appellate forum, has been lost
once and for all. The manner in which appeal against acquittal has been dealt with by the
High Court leaves much to be desired. Reasons introduce clarity in an order. On plainest
consideration of justice, the High Court ought to have set forth its reasons, howsoever
brief in its order, indicative of an application of its mind; all the more when its order is
amenable to further avenue of challenge. The absence of reasons has rendered the High
Court order not sustainable. Similar view was expressed in State of U.P. v. Battan (2001
(10) SCC 607). About two decades back in State of Maharashtra v. Vithal Rao Pritirao
Chawan (1981 (4) SCC 129), the desirability of a speaking order while dealing with an
application for grant of leave was highlighted. The requirement of indicating reasons in
such cases has been judicially recognised as imperative. The view was reiterated in
Jawahar Lal Singh v. Naresh Singh (1987 (2) SCC 222). Judicial discipline to abide by
declaration of law by this Court, cannot be forsaken, under any pretext by any authority
or court, be it even the highest court in a State, oblivious to Article 141 of the
Constitution. 1982 AIR SCW 1215
AIR 1987 SC 724

. Reason is the heartbeat of every conclusion, and without the same it becomes lifeless.
(See Raj Kishore Jha v. State of Bihar, 2003 (11) SCC 519).2003 AIR SCW 5095

8. Even in respect of administrative orders Lord Denning, M.R. in Breen v. Amalgamated


Engg. Union (1971) 1 All ER 1148, observed: "The giving of reasons is one of the
fundamentals of good administration."
@page-SC1512
In Alexander Machinery (Dudley) Ltd. v. Crabtree, 1974 ICR 120 (NIRC) it was
observed : "Failure to give reasons amounts to denial of justice." "Reasons are live links
between the mind of the decision-taker to the controversy in question and the decision or
conclusion arrived at." Reasons substitute subjectivity by objectivity. The emphasis on
recording reasons is that if the decision reveals the "inscrutable face of the sphinx", it can,
by its silence, render it virtually impossible for the courts to perform their appellate
function or exercise the power of judicial review in adjudging the validity of the decision.
Right to reason is an indispensable part of a sound judicial system; reasons at least
sufficient to indicate an application of mind to the matter before court. Another rationale
is that the affected party can know why the decision has gone against him. One of the
salutary requirements of natural justice is spelling out reasons for the order made; in other
words, a speaking-out. The "inscrutable face of the sphinx" is ordinarily incongruous
with a judicial or quasi-judicial performance.
9

. The above position was highlighted by this Court in State of Punjab v. Bhag Singh
(2004 (1) SCC 547). 2004 AIR SCW 102

10. In view of the aforesaid legal position, the impugned judgment of the High Court is
unsustainable and is set aside. We grant leave to the State to file the appeal. The High
Court shall entertain the appeal and after formal notice to the respondents hear the appeal
and dispose of it in accordance with law, uninfluenced by any observation made in the
present appeal. The appeal is allowed to the extent indicated.
Order accordingly.
AIR 2008 SUPREME COURT 1512 "Bant Singh v. Niranjan Singh"
(From : AIR 2004 Punj. and Har. 334)
Coram : 2 S. B. SINHA AND H. S. BEDI, JJ.
Civil Appeal No. 7162 of 2005, D/- 15 -2 -2008.
Bant Singh and Anr v. Niranjan Singh (D) by LRs and Anr.
(A) Evidence Act (1 of 1872), S.50 - EVIDENCE - WITNESS - Relationship - Relevancy
of opinion of witness - Suit for property - Plaintiff disputing that mother of defendants,
was his sister - Witness brother-in-law of defendants mother - Aged 80 years - Having
knowledge about family affairs of plaintiff - He, even had attended marriage of mother of
defendants - Proving genealogy of family - Can be said to have special means of
knowledge - Evidence admissible.
AIR 2004 P and H 334, Reversed.
(1996) 11 SCC 88, (1977) 4 SCC 65, Ref.
AIR 1959 SC 914, Rel. on (Paras 17, 21)
(B) Constitution of India, Art.133 - APPEAL - HIGH COURT - SUPREME COURT -
Appeal - Question whether there was severance of joint family property - Question of fact
- Cannot be raised in appeal - Failure to raise such question before High Court - Supreme
Court cannot be asked to formulate such question and remit case to High Court. (Paras
22, 23)
Cases Referred : Chronological Paras
(1996) 11 SCC 88 (Ref.) 20
AIR 1977 SC 2002 (Ref.) 20
AIR 1959 SC 914 (Rel. on) 10, 19
P.N. Puri, for Appellants; Manoj Swarup, Rohit Sohagura, for Respondents.
Judgement
S. B. SINHA, J. :- Application of Section 50 of the Indian Evidence Act, 1872 is involved
in this appeal which arises out a judgment and order dated 27-8-2003 passed by a learned
Single Judge of the Punjab and Haryana High Court in Regular Second Appeal No. 1290
of 1982 allowing the appeal from a judgment and order dated 12-3-1982 passed by the
Additional District Judge, Patiala setting aside the judgment and decree dated 31-3-1981
passed by the Subordinate Judge, First Class, Patiala decreeing the suit of the
respondents. Before embarking upon the said question, we may notice the genealogical
tree of the parties.

2. Chartu died in 1935. According to the appellants upon death of Chartu, the properties
devolved upon his sons Munshi and Bakhtawar in equal shares. Entries in that behalf in
the revenue records were made showing interest of Munshi and Bakhtawar in equal
shares.
3. On or about 16-6-1956, Munshi Singh died. His share in the property was inherited by
his son Niranjan Singh. To the same effect allegedly mutation in the revenue records was
carried out. Bakhtawar, the other son of Chartu died on 25-10-1972. The share of
Bakhtawar Singh in the said property
@page-SC1513
was mutated in favour of Niranjan Singh, son of Munshi Singh and Bant Singh and
Bachan Kaur, son and daughter of Nandi, being his class two heirs. On or about 28-7-
1978, the respondent herein filed a suit challenging the said order passed by the Revenue
Officer in the mutation proceedings, inter alia, claiming that the property was a joint
coparcenary property and, thus, Nandi did not derive any interest therein. It was
furthermore contended that Nandi was not the daughter of Chartu. The learned Trial
Judge in view of the pleadings of the parties framed the following issues :
"i) Whether the disputed property is joint Hindu Family coparcenary property of the
plaintiff and the deceased Bakhtawar Singh and his ancestors? OPP
ii) Whether deceased Nandi was the daughter of Chartu as alleged? OPD
iii) Whether defendant Nos. 1 and 2 are the son and daughter of aforesaid Nandi and are
heirs of deceased Bakhtawar Singh? OPD
iv) Whether the plaintiff is in possession of the suit land? If not its effect? OPD v) Relief.
4. In its judgment and order dated 31-3-1981, the learned Trial Judge held that the
property in suit being joint coparcenary property, Nandi did not inherit any interest
therein. However, in regard to the status of the appellants, it was held that Nandi was the
daughter of Chartu and sister of Bakhtawar Singh.
5. Appellants preferred an appeal thereagainst which, by a judgment and order dated 12-
3-1981, was allowed.
6. While upholding the findings of the learned Trial Judge in regard to the relationship of
the appellants vis-a-vis the said Bakhtawar Singh holding that Nandi, was the mother of
the appellants was the daughter of Chartu and the sister of Bakhtawar Singh. It was also
held that the property in suit was not a coparcenary property and, thus, Niranjan Singh
and Bakhtawar Singh had half shares each therein.
7. Cross objection of the respondents in regard to the finding of the learned Trial Judge
on issue Nos. 2 and 3 was also dismissed.
A second appeal was preferred thereagainst.
The following substantial questions of law were formulated by the High Court :-
"a) Whether the evidence led by the defendants conforms to the requirements of Section
50 of the Indian Evidence Act, 1872 and as such can be taken to have proved the
relationship of Smt. Nandi, mother of defendants No. 1 and 2 with Bakhtawar Singh
deceased?
b) As to whether the learned courts below have returned their findings on the basis of
such evidence which is not admissible in view of Section 50 of the Indian Evidence Act,
1872 and are also based upon the misreading of the evidence led by the parties?
c) Whether the courts below having not dealt with the matter in controversy in correct
perspective, the judgments are liable to be set aside being judicially perverse?
8. The High Court, in its judgment, entered into the question of fact and sought to
appreciate the evidence of D.W.-1, Jhaggar Singh and DW.2 Hajura Singh and opined
that the evidence led by them was not in conformity with Section 50 of the Indian
Evidence Act. On the said finding, the Second Appeal was allowed.
9. Mr. P.N. Puri, the learned counsel appearing on behalf of the appellants, submitted that
the High Court committed a serious error of law in interfering with the findings of fact
arrived at by the courts below.
10. Mr. Manoj Swarup, learned counsel appearing on behalf of the respondent, on the
other hand, took us through the depositions of DW-1 and DW-2 as also the decision of
this Court in Dolgobinda Paricha vs. Nimai Charan Misra [AIR 1959 SC 914] to contend
that the evidence of the said witness
@page-SC1514
was rightly held to be not conforming to the requirements of Section 50 of the Evidence
Act.
The learned counsel also made an endeavour to raise a contention that the property, in
question, being a coparcenary property, Nandi, in any event, did not inherit any right, title
or interest upon the death of Bakhtawar Singh.
11. Relationship of Nandi as sister of Munshi Singh and Bakhtawar Singh was sought to
be proved by Jhaggar Singh, DW-1, Hajura Singh, DW-2, Bachan Kaur, DW-3 and Bant
Singh, DW-4.
12. The learned Trial Court in its judgment on issue Nos. 2 and 3 analysed the evidences
of the said witnesses in great details.
13. It does not appear from the judgments of the learned Subordinate Judge as also the
learned Additional District Judge that any evidence was adduced on behalf of the
respondent to establish as to whose daughter Nandi was. Plaintiffs/Respondents,
furthermore failed to establish as to how the order of the revenue authorities directing
mutation of the name of the appellants herein was illegal.
14. An entry made in the revenue records may not be decisive to as regards the status of
the parties but a presumption in regard to possession can be raised on the basis thereof.
15. The High Court, unfortunately did not refer to the depositions of the witnesses
examined on behalf of the appellants at all. It proceeded only on the basis that the oral
evidence of DW-1 and DW-2 do not pass the tests of Section 50 of the Indian Evidence
Act.
16. DW-1, at the time of his deposition, was aged 80 years. He is the brother of Baksha
Singh, husband of Nandi. He proved the genealogy of the family of Chartu. Nandi, was
the wife of his brother. He, therefore, had special knowledge in regard to relationship
between her, on the one hand, and Munshi and Bakhtawar Singh, on the other.
17. He attended the marriage of Nandi. Bakhtawar and Munshi, according to him, used to
come to their house. The relationship between the appellants and the said Munshi Singh
and Bakhtawar Singh was, thus, within his special knowledge. He categorically stated the
manner in which the appellants used to be treated by Munshi Singh and Bakhtawar
Singh.
In answer to a question put to him in cross-examination, he, in no uncertain terms,
categorically stated that he had heard as well as saw Nandi calling Munshi Singh and
Bakhtawar Singh as brother. He attended the marriage of Nandi when he was only 15
years old.
18. To the similar effect is the evidence of DW-2, Hajura Singh who was another brother
of Baksha Singh. He was aged 75 years on the date of his deposition.
DW-2 is a resident of the same village. He had also watched the parties from a close
quarter and, thus, could depose about the conduct of Nandi vis-a-vis Munshi Singh and
Bakhtawar Singh.
He apart from other things testified that Bakhtawar Singh had come to attend the
marriage of Bachan Kaur. He stated in detail about the family of both the parties. He
deposed that he had old dealings with Baksha Singh, being of the same village. He also
attended the marriage of the brother of Baksha Singh, being a close relative.
19

. In Dolgobinda Paricha (supra), this Court relied upon the evidence of two witnesses
who had attended the marriage of Haripriya and the Lokanath which was in dispute. In
that case, the relationship of Ahalya and Malabati as the daughter of Lokanath was in
dispute. AIR 1959 SC 914, at p. 918

Interpreting Section 50 of the Evidence Act, this Court held :


".........On a plain reading of the section it is quite clear that it deals with relevancy of a
particular fact. It states in effect that when the court has to form an opinion as to the
relationship of one person to another the opinion expressed by conduct as to the existence
of such relationship of any person who has special means of knowledge on the subject of
that relationship is a relevant fact. The two illustrations appended to the section clearly
bring out the true scope and effect of the section. It appears to us that the essential
requirements of the section are-
(1) there must be a case where the court has to form an opinion as to the relationship of
one person to another;
(2) in such a case, the opinion expressed by conduct as to the existence of such
relationship is a relevant fact; (3) but the person whose opinion expressed by conduct is
relevant must be a person who as a member of the family or otherwise has special means
@page-SC1515
of knowledge on the particular subject of relationship; in other words, the person must
fulfill the condition laid down in the latter part of the section. If the person fulfils that
condition, then what is relevant is his opinion expressed by conduct. Opinion means
something more than mere retailing of gossip or of hearsay; it means judgment or belief,
that is, a belief or a conviction resulting from what one thinks on a particular question.
Now, the belief or conviction may manifest itself in conduct or behaviour which indicates
the existence of the belief or opinion. What the section says is that such conduct or
outward behaviour as evidence of the opinion held is relevant and may, therefore, be
proved."
It was furthermore held :
7.........If we remember that the offered item of evidence under Section 50 is conduct in
the sense explained above, then there is no difficulty in holding that such conduct or
outward behaviour must be proved in the manner laid down in Section 60; if the conduct
relates to something which can be seen, it must be proved by the person who saw it; if it
is something which can be heard, then it must be proved by the person who heard it; and
so on. The conduct must be of the person who fulfils the essential conditions of Section
50, and it must be proved in the manner laid down in the provisions relating to proof. It
appears to us that that portion of Section 60 which provides that the person who holds an
opinion must be called to prove his opinion does not necessarily delimit the scope of
Section 50 in the sense that opinion expressed by conduct must be proved only by the
person whose conduct expresses the opinion. Conduct, as an external perceptible fact,
may be proved either by the testimony of the person himself whose opinion is evidence
under Section 50 or by some other person acquainted with the fact which express such
opinion, and as the testimony must relate to external facts which constitute conduct and is
given by persons personally acquainted with such facts, the testimony is in each case
direct within the meaning of Section 60. This, in our opinion, is the true inter-relation
between Section 50 and Section 60 of the Evidence Act.
20. Applying the aforementioned tests in regard to the evidences of Janardan Misra and
Dharanidhar Misra, the two witnesses who were examined to prove the relationship
between Ahalya and Malabandi, it was opined;
"9. The first question which we must consider is if Janardan Misra and Dharanidhar
Misra had special means of knowing the disputed relationship. Janardan Misra was aged
about 62 in 1946, and he was related to the family of Baidyanath Misra. Kashi Nath
Misra was his grandfather and was a brother of Baidyanath Misra. Obviously, therefore
Janardan Misra had special means of knowing the disputed relationship, being related to
Baidyanath and therefore to Haripriya, who was the second wife of Lokenath. He said in
his evidence that he knew Lokenath Parichha, had seen his first wife Satyabhama and
remembered the marriage of Haripriya with Lokenath Parichha. Obviously, therefore, he
fulfilled the condition of special knowledge. He further said that he attended the marriage
of Malabati, daughter of Lokenath, when Lokenath was living. That marriage took place
in the house of Lokenath. He also said that he was present when the first two daughters of
Malabati were married and also at the time of the upanayan ceremonies of Plaintiffs 1 and
2. According to the witness, Shyam Sundar Pujari, a son of a sister of Lokenath, acted as
a maternal uncle at the time of the marriage of the eldest daughter of Malabati and
Dayasgar Misra carried Radhika, second daughter of Malabati, at the time of her
marriage."

See also Shantinath Ramu Danole and Anr. vs. Jambu Ramu Danole and Ors. [(1996) 11
SCC 88] and Munshi Singh Vs. Mal Dass [(1977) 4 SCC 65]. AIR 1977 SC 2002
21. Applying the same tests, we have not doubt that the evidence of DW-1 and DW-2 are
admissible in evidence being in conformity with the provisions of Section 50 of the
Indian Evidence Act. It will bear repetition to state that the High Court, for the reasons
best known to it, did not advert to the depositions of the witnesses examined on behalf of
the appellants at all. The High Court could have interfered with the finding of fact in a
second appeal provided it applied the right tests, but it failed to do so.
22. The submission of Mr. Manoj Swarup, learned counsel for the respondent that this
Court should enter into the question as to whether the suit property was an ancestral
property or not, in our opinion cannot be accepted. No substantial question
@page-SC1516
of law in that behalf was raised. Out of the three substantial questions of law, as referred
to hereinbefore, only the first two questions related to application of Section 50 of the
Indian Evidence Act. The third question formulated was not a substantial question of law
at all.
As the respondent failed to persuade the High Court to formulate any substantial question
of law on that point viz. that the property, in question, was a coparcenary property, it is
too late in the day to ask this Court to formulate such a question and remit the matter to
the High Court.
23. Whether there has been a severance of the joint family property between Munshi
Singh and Bakhtawar Singh is essentially a question of fact and, thus, in our opinion, the
said question cannot be permitted to be reopened before us.
24. For the reasons aforementioned, the impugned judgment cannot be sustained. It is set
aside accordingly. The appeal is allowed. However, in the facts and circumstances of this
case, there shall be no order as to costs.
Appeal allowed.
AIR 2008 SUPREME COURT 1516 "Arun Kumar v. State of Bihar"
(From : Patna)*
Coram : 2 Dr. A. PASAYAT AND P. SATHASIVAM, JJ.
Criminal Appeal No. 347 of 2008 (arising out of SLP (Cri.) No. 2892 of 2007), D/- 19 -2
-2008.
Arun Kumar v. State of Bihar and Anr.
Juvenile Justice Act (53 of 1986), S.8, S.38 - Criminal P.C. (2 of 1974), S.401 -
JUVENILE JUSTICE - REVISION - HIGH COURT - TRIAL COURT - Juvenile -
Determination of age of accused by Juvenile Justice Board - Trial Court refusing to refer
accused to Board on finding that accused was not juvenile - Finding based on analysis of
documents and observation of accused - Interference therewith by High Court without
considering fact that accused was shown as major in Charge Sheet and his School
Leaving Certificate mentions different name - Any infirmity in conclusions of Trial Court
also not shown - Order of High Court liable to be set aside.
Cri. Misc. C. No. 2852 of 2007, D/- 19-01-2007, (Pat), Reversed. (Paras 5, 6)

Ranjan Mukherjee and S.C. Ghosh, for Appellant; Manish Kumar, Gopal Singh and
Lakshmi Raman Singh, for Respondents.
* Cri. Misc. No. 2852 of 2007, D/- 19-1-2007 (Pat).
Judgement
Dr. ARIJIT PASAYAT, J. :-Leave granted.
2. Challenge in this appeal is to the order passed by a learned single Judge of the Patna
High Court quashing the order passed by learned Additional District Judge, Fast Track
Court Vth, Shekhpura. By the said order the learned Additional Sessions Judge held that
respondent No. 2 Munna Kumar was not juvenile and, therefore, there was no need to
refer his case to the Juvenile Justice Board for ascertaining of his age and, then for trial. It
was observed by the High Court that the prayer was rejected only on the ground that two
or three witnesses were examined and though the accused was in possession of School
Leaving Certificate, mark sheet etc. to show that he was a juvenile, the prayer could not
have been rejected. The High Court in a very cryptic manner observed that the
application of the accused deserved to be allowed and directed the Court below to
consider the accused as a juvenile and to proceed accordingly.
3. Learned counsel for the informant submitted that the documents produced had been
analysed by the trial Court and it was categorically held that at the time of framing charge
on observation it was noticed that he was major without any doubt. In the certificate filed
his name was disclosed to be Priyatam Bihari though all through his name was stated to
be Munna Kumar. Learned single Judge of the High Court did not even consider as to
how the conclusions of the trial Court suffered from any infirmity. Merely referring to the
stand of the accused and even without analyzing the correctness or otherwise of the
observations and conclusions made by the trial Court he came to hold that the accused
was a juvenile. Additionally, the complainant was a party before the High Court but no
notice was issued. There is no appearance on behalf of respondent No. 2 accused.
4. Learned counsel for the State supported the stand of the informant.
5. The High Court has failed to notice several relevant factors. Firstly, at the time of
framing charges, the age of the accused
@page-SC1517
was recorded as major. Similarly, the difference in names in the documents has not been
explained by the accused.
6. Further, as rightly contended by learned counsel for appellant, no discussion has been
made as to how the conclusions of the trial Court suffered from any infirmity.
7. Finally, no notice was issued to the appellant before the matter was disposed of.
8. Above being the position, the impugned order of the High Court is set aside and the
matter is remitted to it to consider the matter afresh and pass a reasoned order in
accordance with law.
9. The appeal is allowed.
Appeal allowed.
AIR 2008 SUPREME COURT 1517 "B. L. Gupta Construction Ltd., M/s. v. M/s. Bharat
Co-op. Group Hsg. Sty. Ltd."
(From : Delhi)*
Coram : 2 S. B. SINHA AND H. S. BEDI, JJ.
Civil Appeal No. 1511 of 2008 (arising out of SLP (C) No. 4654 of 2007), D/- 22 -2
-2008.
M/s. B.L. Gupta Construction Ltd. v. M/s. Bharat Co-op Group Hsg. Sty. Ltd.
Arbitration Act (10 of 1940), S.30 - ARBITRATION - INTEREST - SUPREME COURT
- Award - Challenge as to - Interest awarded only against some claims - Calculations
made at stage of arbitrator and at stage of execution in High Court - Directions also
issued by Supreme Court to make payment of interest on specific sums from particular
dates - Appellant. claimant dragged on proceedings for years together despite orders
clarifying all issues from time to time - Misused legal process - Raising several issues by
him as to method to be adopted for calculation and as to how payments made from time
to time had to be adjusted - Not permissible - Costs imposed on appellant. (Para 7)

Jayant Das, Sr. Advocate, O.P. Khadaria, Deepak Khadaria, K.M.M. Khan, C.S.N. Mohan
Rao, for Appellant; R.K. Watel, Ms. Praveena Gautam, Pramod B. Agarwala, for
Respondent.
* R. A. No. 382 of 2006 and EFA (OS) No. 12 of 2005, D/- 7-11-2006 and 21-9-2006
(Delhi) respectively.
Judgement
1. HARJIT SINGH BEDI, J.:-Special leave granted.
2. The facts leading to the filing of this appeal are as under :
3. The appellant B.L.Gupta Construction Ltd. was awarded a contract by the respondent
for construction of 308 dwelling units. The contract also provided for resolution of
disputes through an arbitrator. A dispute having arisen, the High Court vide order of 5-12-
1991 ordered the appointment of an arbitrator. The arbitrator rendered his award on 18-
12-1992 for a sum of Rs.25,14,424/- in favour of the appellant and a counter claim for
Rs. 1,18,393.32p. in favour of the respondent. The principal amount thus determined to
be paid to the appellant was Rs.23,96,031/-. The arbitrator also granted interest in the
following terms:
"I award the following interest to be paid by the respondents to the claimants as under:-
(a) Interest at the rate of 18 (eighteen) per cent per annum on the amount awarded under
claim No. 1 after expiry of 50 days from the date of submission of Final bill dated 18-4-
1990 i.e. 8-6-1990 to the date prior to the arbitration proceedings i.e. 5-12-1991.
(b) Pendente-lite interest at the rate of 18 (eighteen) per cent per annum from 6-12-1991
to the date of publication of award 18-12-1992.
(c) If the respondent does not pay the awarded amount within a period of 60 days from
the date of publication of the award, the respondents shall also pay interest at the rate of
18 (eighteen) per cent per annum on the gross amount of award for all the claims from
19-12-1992 to the date of decree or date of payment whichever is earlier."
4. The respondent society thereafter filed objections under Sections 30 and 33 of the
Arbitration Act, 1940. These objections were dismissed and the award was made a rule of
the Court and a decree Annexure P-2 was accordingly passed on 5-2-1996. An appeal
[FAO(OS) No. 78/96] was filed in the High Court against the aforesaid order and on
admission on 28-9-1996 a stay of execution was granted subject to deposit of the
principal amount in Court. Concededly this amount has been released to the decree-
holder appellant on furnishing a bank guarantee on 27-10-1998. The Division Bench
while disposing of the aforesaid FAO in its order dated 1-8-2001 modified the decree to
the extent that the interest prior to
@page-SC1518
18-12-1998 awarded by the Arbitrator was declined and future interest reduced from 18%
to 10% per annum and as the respondent had deposited the amount in the High Court on
28-9-1996 it was directed that the interest would cease to be payable on and from the said
date. The appellant thereupon filed a Special Leave Petition in this Court in which leave
was granted. The respondent also made a part payment of Rs. 8,39,791/- as interest on 8-
12-2001 to the appellant. The aforesaid appeal was allowed on 5-11-2003 and it was held
that the respondent would pay pre-reference and pendente-lite interest at 10% per annum
on the amount of the award. The respondent thereafter made another payment of Rs.
6,14,911/- to the appellant. The appellant also filed an execution application
(E.A.No.282/04) submitting its calculations and claimed a further sum of Rs. 19,07,872/-
as on 19-7-2004. A copy of this application has been filed as Annexure P-5 to the grounds
in the Special Leave Petition. The learned Single Judge in his order dated 21-7-2005
ordered the Registry to calculate the pre-reference interest at 10% per annum on the sum
of Rs.23,96,030-38p. for a period from 8-6-1990 to 18-12-1992 and likewise the interest
on the aforesaid amount for the period from 18-12-92 to 28-9-96 i.e. date on which the
amount had been deposited by the respondent in Court and the matter was adjourned for
further hearing to 16-8-2005. On 16-8-2005 the learned Single Judge held that the
question of payment of interest both pendente-lite and future interest had been settled and
accordingly dropped the proceedings. The appellant thereupon filed an appeal before the
Division Bench challenging the orders dated 21-7-2005 and 16-8-2005 in Appeal (EFA
(OS)No. 12/05). By order dated 21.9.2006 the Division Bench dismissed, the appeal
holding that the learned Single Judge had rightly directed the future interest be calculated
only on the principal amount of Rs.23,96,031/-. A review application filed by the
appellant against the aforesaid order was also dismissed on 18-10-2006. Both these
matters have been impugned in the present appeal.
5. A reply has been filed by the respondent in response to the notice. It has been
highlighted that 14 claims had been raised before the Arbitrator and whereas Claim Nos.
1 to 12 were under several heads including the payment for the work done, Claim No. 13
was with regard to interest simplicitor both pre-reference and pendente-lite. It has been
pointed out that the Arbitrator had in his award dated 18-12-1992 awarded a total sum of
Rs.25,14,424/- to the appellant partly allowing his claim Nos. 1, 2 and 5 in the following
terms - Claim No. 1 Rs.24,64,424/-, Claim No.2 Rs.10,000/-and Claim No.5 Rs.40,000/-
and that the Arbitrator had awarded interest only against Claim No. 1 at 18% per annum
from 8-6-1990 to 5-12-1991 (Pre-reference) and pendente-lite interest at 18% per annum
from 6-12-1991 to the date of the publication of the award i.e. 18-12-1992 and that no
interest had been awarded with respect to the claim Nos.2 to 5 and that the award had
clarified that if the amount awarded was not paid within the period of 60 days from the
date of the publication of the award, the respondent would be liable to pay future interest
at 18% on the gross amount of the award for all claims allowed from 19-12-1992 to the
date of payment. It has further been highlighted that the respondent had filed an appeal
before the Division Bench being FAO(OS) 78/96 and a direction had been issued to
deposit the principal amount in terms of the award i.e. Rs.23,96,031/-which in fact had
been withdrawn by the petitioner on 28-9-1996 without any objection and that the
Division Bench had finally disposed of the appeal observing as under :
"Consequently, the appeal is partly allowed to the extent aforementioned, the judgment of
the learned Single Judge as also the award is set aside to that extent. Consequently,
decree shall stand modified to the extent that the Claims of the respondent (the petitioner
herein) to interest prior to 18-12-92 stands rejected. The respondent shall be entitled to
future interest @ 10% per annum on the amount of award under Claim No. 1 from 18th
December 1992 till the date of decree and from the date of decree till date of payment on
the amount payable by the Society (respondent herein) to the respondent (petitioner
herein) in terms of Award.
Since the appellant had deposited the amount in the Court on 28th September 1996,
interest will cease to be payable on and from that date on the deposited amount."
6. The appellant aggrieved by the aforesaid order of the Division Bench, filed SLP in this
Court against the judgment dated
@page-SC1519
1-8-2001 and this Court vide order dated 5-11-2003 finally disposed of the matter by
partly allowing the appeal directing the respondent to pay pre-reference and pendente-lite
interest at 10% per annum on Claim No. 1 and affirmed the other parts of the judgment of
the Division Bench which included the cessation of interest on the principal amount
deposited as well as the future interest on Claim Nos. 2 to 5. It has accordingly been
pleaded that as the respondent had already paid the entire amount in terms of the
judgment of the Division Bench in FAO (OS) 78/ 96 on 28-9-1996 only pendente-lite
interest for the period from 8-6-1990 to 18-12-1992 at 10% per annum on the principal
amount under Claim No. 1 i.e. a sum of Rs. 6,14,911/- was payable and that the aforesaid
amount had already been paid. The appellant thereafter filed yet another execution
application in the High Court claiming that the entire amount in terms of the orders of the
Court had not been paid on which the respondent in terms of the order dated 3-2-2005
deposited a further sum of Rs.63,658/- on 9-2-2005. It is this execution application which
has now been disposed of by the impugned orders.
7. We have heard the learned counsel for the parties and have gone through the record.
Several issues have been sought to be raised by the learned counsel for the appellant as to
the method to be adopted for the calculation of and as to how the payments made from
time to time had to be adjusted. We are, however, of the opinion that no such argument is
open to the appellant as of now for the reason that interest had been awarded only against
some claims and that the calculations have been made both at the stage of the Arbitrator
and at the stage of the execution in the High Court and directions have been issued by
this Court to make payment on interest on specific sums from particular dates. It is also to
be noticed that the appellant has dragged on the proceedings for years together despite the
orders clarifying all issues from time to time. We have given full details of the
proceedings from the start of the arbitration till their culmination in the Supreme Court
advisedly to bring home the point that the legal process has been misused by the
appellant. We have therefore no hesitation in dismissing the appeal and while doing so
impose Rs.50,000/- as costs on the appellant.
Order accordingly.
AIR 2008 SUPREME COURT 1519 "Sobhagyamal v. Gopal Das Nikhra"
(From : Madhya Pradesh)*
Coram : 2 PRAKASH PRABHAKAR NAOLEKAR AND LOKESHWAR SINGH
PANTA, JJ.
Civil Appeal Nos. 1839-1840 of 2004, D/- 22 -2 -2008.
Sobhagyamal and Anr. v. Gopal Das Nikhra.
M.P. Accommodation Control Act (41 of 1961), S.12(1)(a), S.12(1)(f), S.12(3), S.13(5),
S.13(6) - ACCOMMODATION CONTROL - EVICTION - TENANCY - Eviction -
Default in payment of rent - Tenant granted benefit of S.12(3) in suit for eviction on
grounds of default, bona fide need etc. - Making second default during pendency of
appeal - Subsequent suit for eviction for default would be maintainable.
S. 12(3) of the Act provides for an exception to the general rule contained in S. 12(1) (a)
that in the event tenant becomes a defaulter, he is liable to be evicted. From the proviso to
S. 12(3) it is clear that the protection given to the tenant is only one time protection.
Proviso appended to S. 12(3) controls the main provisions. The exemption contained in S.
12(3), thus, is not extended to the tenant who becomes a defaulter for more than once. In
the present case suit for eviction on ground of default, sub-letting and bona fide
requirement was filed against the tenant. On institution of the suit, the tenant deposited
the rent within one month of the service of writ of summon of the Court on him. The
tenant was therefore granted benefit of S. 12(3) and eviction on ground of default was
refused. The suit was however decreed by Trial Court on ground of bona fide need.
Appeal was filed against the decree. During the pendency of appeal before Supreme
Court the tenant neither deposited nor paid rent for three consecutive months. Landlord
therefore filed second suit for eviction on ground of default. The dismissal of second suit
on ground that non-deposit of rent in Court in the previous proceedings or tendering rent
to the landlord could not be considered as arrears of rent; and that at best his defence
against eviction could have been struck out under S. 13(6) of the Act is improper. By S.
13(5) the tenant is protected from eviction on ground of default if he makes and continues
to make deposit or pay month by month by 15th of each succeeding month a sum
equivalent
@page-SC1520
to the rent at that rate till the decision of the suit, appeal or proceeding, as the case may
be. If there is a default for three consecutive months in the payment of rent and the rent
has not been tendered within two months of the service of notice by the landlord for
payment of arrears, a cause of action accrued in favour of the landlord to initiate
proceedings for ejectment of the tenant by filing a suit under S. 12(1)(a) of the Act and
thereafter S. 12(3) or S. 13(5) would not be attracted. (Para 10)

Sushil Kumar Jain, R.K. Patni, Puneet Jain, Ms. Christi Jain, H.D. Thanvi, Ms. Pratibha
Jain, for Appellants; Rajesh for Respondent.
* L. P. A. No. 25 of 2001, D/- 16-8-2002 and F. A. No. 86 of 2000, D/- 21-12-2000 (M.P.)
(Gwalior Bench).
Judgement
1. P.P. NAOLEKAR, J.The brief facts material for the decision of this case are that Gopal
Das (respondent herein) was inducted as a tenant in the suit premises on the monthly rent
of Rs.350/- w.e.f. 14-12-1973. As the respondent was not paying the rent of the suit
premises regularly and was defaulter, a notice demanding arrears of rent was issued by
the landlord on 28-6-1975. Despite the service of notice, the respondent did not pay the
rent within two months from the service of notice and, therefore, the landlord filed a civil
suit being Suit No. 75A/ 1979 for eviction of the respondent from the suit premises on the
ground of Section 12(1)(a) of the Madhya Pradesh Accommodation Control Act, 1961
(hereinafter referred to as "the Act") (i.e. default in payment of rent) and on the ground of
Section 12(1)(b) (i.e. sub-letting) and later on, by amendment, on the ground of Section
12(1)(f) (i.e. bona fide necessity of the accommodation for non-residential purposes). On
institution of the suit, the respondent deposited the rent within one month of the service
of writ of summon of the Court on him. The suit was decreed by the trial Court on the
ground of bona fide requirement under Section 12(1)(f) of the Act. With regard to the
default, the trial Court found as under:
"14. Issue No. 9 is decided against defendant, hence rent of Rs.2800/- from 13-12-74 to
12-8-75, mesne profits 237/- and interest 161 /- as per agreement total Rs.3198.00 are due
of the plaintiff on the defendant and he is entitled to get the above amount, I give such
finding.
15. Though, the defendant did not pay rent within two months after receipt of notice, but
he raised dispute of rent under Section 13(2) of the Act which was not decided. As such
the defendant has deposited all the upto date amount, hence I give finding that defendant
has not paid or deposited all the arrears of rent within two months from receipt of notice
but deposited during pendency of suit. Therefore, the defendant will get benefit of
Section 13(5) and Section 12(3) and the plaintiff is not entitled to get decree under
Section 12(1)(a) of the Act."
Thus, the respondent was given benefit under Section 12(3) of the Act. The respondent
preferred an appeal being Appeal No. 27A/1980 challenging the decree for ejectment on
the ground of bona fide need. It would be pertinent to note that the landlord did not prefer
any appeal or file any cross-objection challenging the refusal of decree on the ground of
arrears of rent. The first appellate Court confirmed the decree passed by the trial Court.
The respondent preferred a second appeal being Second Appeal No.47/ 1982. The second
appeal preferred by the respondent was allowed by the High Court and the suit of the
landlord on the ground of bona fide need was dismissed as premature. While allowing the
appeal of the respondent, the High Court observed: "No decree for ejectment on the
ground under Section 12(1)(a) of the Act could have been passed against the appellant,
because he had, admittedly, complied with the provisions of Section 13(1) of the Act."
Against the said judgment and decree of the High Court, the landlord filed a special leave
petition in this Court. During the pendency of the proceedings, the then landlord Lakshmi
Chand expired and his legal representatives (appellants herein) were brought on record.
The special leave petition was converted into Civil Appeal No. 3931/1986. During the
pendency of the aforementioned proceedings, since the respondent had not deposited the
rent or paid it to the landlord, the appellants served a notice dated 27-2-1991 upon the
respondent demanding arrears of rent intimating him that he had committed a default in
payment of rent due from 13-12-1984 to 13-2-1991. The said notice was served on the
respondent on 5-3-1991. Despite the service of notice of demand for arrears of rent, the
respondent did not pay the rent within two months of the service. The appellants/landlord
filed a second suit being Suit No. 78A/1992 against the respondent
@page-SC1521
under Section 12(1) (a) of the Act on the ground of default in payment of rent. During the
pendency of these proceedings, matter was taken up by the Supreme Court in Civil
Appeal No 3931/1986 and was disposed of on 24-8-1994. The Supreme Court held that
insofar as the finding recorded by the High Court on the question of bona fide necessity is
concerned, the Court does not find any ground to interfere with the same. The Court
further held :
"It appears that a second suit for eviction has been filed on the ground of default in
payment of rent and the same is pending. The submission of Shri Jain is that the said suit
may be prejudiced by the following observations contained in the judgment of the High
Court :
"It is manifest that the suit for ejectment was not maintainable, because on the date of suit
the period of lease as per clause (1) of the lease deed (Ext. P. 1) had not come to an end."
Shri Jain has contended that the said observations can be construed to mean that the High
Court has found that the earlier suit for eviction on the ground of default in payment of
rent was premature. We are unable to construe the said observations in this light. In our
view the said observations only refer to the suit insofar as it relates to eviction on the
ground of bona fide personal necessity."
Thus, the first proceeding came to an end.
2. In the second proceeding, by judgment and decree dated 8-3-2000, the trial Court
decreed the suit holding that the rent from 13-12-1984 was due from the respondent and
thus he had committed default in payment of rent and was liable to be ejected on the
ground of arrears of rent. The respondent preferred an appeal in the High Court being
F.A. No.86/2000. On 21-12-2000, the appeal of the respondent was allowed and the
judgment and decree of the trial Court was set aside. The High Court held that since the
appellants herein did not take any steps to get the defence of the respondent struck out
while the case was pending before the Supreme Court the appellants were precluded from
getting a decree for default of the period when the case was pending before the Supreme
Court. The appellants herein preferred a letters patent appeal before the Division Bench.
LPA was dismissed as not maintainable and thus the appellants are before us.
3. From the aforesaid facts, it is apparent that the respondent/tenant got benefit of Section
12(3) of the Act in the previous proceedings and thus no decree for ejectment was passed
against him on the ground of arrears of rent under Section 12(1)(a) of the Act. The second
proceeding of filing a suit for ejectment under Section 12(1)(a) of the Act was initiated by
the appellants herein/landlord after service of notice demanding arrears of rent due during
the pendency of the previous proceedings. It was contended by the respondent that non-
deposit of rent in Court in the previous proceedings or tendering rent to the landlord
could not be considered as arrears of rent; and that at best his defence against eviction
could have been struck out under Section 13(6) of the Act. Non-payment of rent during
the pendency of the previous proceedings would not be treated as arrears of rent to give a
cause of action to the landlord to file a suit on the ground of arrears of rent. The High
Court upheld this contention and found that no ground under Section 12(1)(a) of the Act
was available to the appellants herein/landlord for non-payment of rent by the
respondent/tenant during the pendency of the previous proceedings before the Court.
4. It is contended by the learned counsel for the appellants that during the pendency of
the proceedings in the Court after taking benefit under Section 12(3) of the Act, a tenant
is liable to deposit the rent or to tender it to the landlord to avoid any decree under
Section 12(1)(a) of the Act on the ground of arrears of rent. A tenant having taken benefit
under Section 12(3) of the Act, is bound to pay the rent to the landlord or deposit it in the
Court to avoid the decree for ejectment under Section 12(1)(a) of the Act. Since the
respondent/tenant had committed default in payment of rent for three consecutive
months, the High Court should have confirmed the decree passed by the trial Court on the
ground under Section 12(1)(a) of the Act, the tenant having failed to pay the rent to the
landlord even on service of notice of demand on him. It is further urged that Section
13(6) of the Act does not give a protection to the tenant from the ejectment on non-
payment of rent to the landlord for three consecutive months.
5. It is urged by the learned counsel for the respondent/tenant that since the matter was
pending consideration before the
@page-SC1522
Court the appellants/landlord at best could have moved an application under Section
13(6) for striking out defence if the respondent/tenant had not deposited the rent as
required under Section 13(1) of the Act, but the said default in payment of rent would not
be treated as arrears of rent giving cause of action to the appellants/landlord to institute a
suit for ejectment on the ground of arrears of rent.
6. To appreciate the submissions made by the learned counsel appearing for the
respective parties, it will be necessary to refer the relevant previsions of the Act, which
read as under :
"12, Restriction on eviction of tenants. - (1) Notwithstanding anything to the contrary
contained in any other law or contract, no suit shall be filed in any civil Court against a
tenant for his eviction from any accommodation except on one or more of the following
grounds only, namely
(a) that the tenant has neither paid nor tendered the whole of the arrears of the rent legally
recoverable from him within two months of the date on which a notice of demand for the
arrears of rent has been served on him by the landlord in the prescribed manner;
XXX XXX XXX
(3) No order for the eviction of a tenant shall be made on the ground specified in clause
(a) of sub-section (1), if the tenant makes payment or deposit as required by Section 13 :
Provided that no tenant shall be entitled to the benefit under this sub-section, if, having
obtained such benefit once in respect of any accommodation, he again makes a default in
the payment of rent of that accommodation for three consecutive months.
xxx xxx xxx"
"13. When tenant can get benefit of protection against eviction. - (1) On a suit or any
other proceeding being instituted by a landlord on any of the grounds referred to in
Section 12 or in any appeal or any other proceeding by a tenant against any decree or
order for his eviction, the tenant shall, within one month of the service of writ of
summons or notice of appeal or of any other proceeding, or within one month of
institution of appeal or any other proceeding by the tenant, as the case may be, or within
such further time as the Court may on an application made to it allow in this behalf,
deposit in the Court or pay to the landlord, an amount calculated at the rate of rent at
which it was paid, for the period for which the tenant may have made default including
the period subsequent thereto up to the end of the month previous to that in which the
deposit or payment is made; and shall thereafter continue to deposit or pay, month by
month by the 15th of each succeeding month a sum equivalent to the rent at that rate till
the decision of the suit, appeal or proceeding, as the case may be.
xxx xxx xxx
(5) If a tenant makes deposit or payment as required by subsection (1) or sub-section (2),
no decree or order shall be made by the Court for the recovery of possession of the
accommodation on the ground of default in the payment of rent by the tenant, but the
Court may allow such cost as it may deem fit to the landlord.
(6) If a tenant fails to deposit or pay any amount as required by this Section, the Court
may order the defence against eviction to be struck out and shall proceed with the hearing
of the suit, appeal or proceeding, as the case may be."
7. A landlord can seek ejectment of his tenant from the premises let out to him only on
the ground/s enumerated in Section 12 of the Act. Clause (a) of sub-section (1) of Section
12 of the Act authorises the landlord to seek ejectment of his tenant if he has neither paid
nor tendered the whole of the arrears of rent legally recoverable from him within two
months of the service of notice demanding the arrears of rent. Sub-section (3) of Section
12 puts a caveat on the right of the landlord to get ejectment on the ground of arrears of
rent if the tenant makes payment or deposit as required by Section 13. However, by virtue
of the proviso to sub-section (3), the benefit given to the tenant, on compliance of the
payment of rent as provided under Section 13, would be available to him only once in
respect of that accommodation, but on default in the payment of rent in respect of same
accommodation for three consecutive months he would not be entitled for protection by
depositing the rent as provided under Section 13 in the subsequent proceedings initiated
by the landlord for ejectment of the tenant on the ground of arrears of rent.
8. Section 13 of the Act requires that the
@page-SC1523
tenant shall within one month of the service of writ of summons or notice of appeal or of
any other proceeding deposit the rent when the proceedings are initiated by the landlord
on any of the grounds referred to in Section 12 or within one month of institution of
appeal or any other proceeding when taken by the tenant against any decree or order for
his eviction. The period of one month given to the tenant for depositing the rent from the
date of the summons or the notice of appeal or of any other proceeding could be extended
by the Court on an application made to it. The rent which is required to be deposited
under the Section can be in the Court or it may be made over to the landlord. The Section
further requires that after the deposit of the arrears of rent the tenant shall continue to
make deposit or pay month by month by 15th of each succeeding month a sum equivalent
to the rent at that rate till the decision of the suit, appeal or proceeding, as the case may
be. Sub-section (5) of Section 13 provides that if the tenant makes deposit or payment as
required by sub-section (1) or sub-section (2) no decree or order shall be made by the
Court for recovery of possession on the ground of default in the payment of rent by the
tenant. Sub-section (6) gives an option to the landlord if the tenant does not deposit the
rent or pay it to the landlord as required under Section 13 to move an application for the
defence against eviction to be struck out. Sub-section (5) of Section 13 has no application
in a case when the ejectment is not sought by the landlord on the ground of arrears of
rent, but the suit is instituted by the landlord on any other ground/s of Section 12 of the
Act. Striking out of the defence of the tenant on an application moved by the landlord, is
a provision applicable in the suit for ejectment on any of the grounds mentioned under
Section 12 inclusive of under Section 12(1)(a) of the Act, whereas sub-section (5) of
Section 13 would apply only when the suit is instituted for ejectment on the ground of
arrears of rent under Section 12(1)(a) of the Act.
9. From the aforesaid, it is clear that Section 12(3) of the Act provides for an exception to
the general rule contained in Section 12(1)(a) that in the event tenant becomes a defaulter,
he is liable to be evicted. From the proviso to Section 12(3) of the Act, it is clear that the
protection given to the tenant is only one time protection. Proviso appended to Section
12(3) controls the main provisions. The exemption contained in Section 12(3), thus, is not
extended to the tenant who becomes a defaulter for more than once. In view of the
aforesaid, we are of the opinion that once the tenant had availed the benefit of the proviso
to Section 12(3) of the Act, the said benefit was not available to the tenant in committing
a further default in payment of rent for three consecutive months.
10. The tenant can only be protected against ejectment on the ground of arrears of rent in
the subsequent proceedings if he deposits the rent in the Court or pay it to the landlord
during the pendency of the proceedings in the Court or pay it to the landlord after the suit
is decided by the Court. If there is a default for three consecutive months in the payment
of rent and the rent has not been tendered within two months of the service of notice by
the landlord for payment of arrears, a cause of action accrued in favour of the landlord to
initiate proceedings for ejectment of the tenant by filing a suit under Section 12(1)(a) of
the Act and thereafter Section 12(3) or Section 13(5) would not be attracted.
11. The High Court has committed an error in applying the provisions of sub-section (6)
of Section 13 to the second suit initiated by the landlord under Section 12(1)(a) on the
ground of arrears of rent. That provision is only for the purpose of striking out of the
defence of a tenant if the rent is not deposited as required under Section 13 which has
nothing to do with the provisions of sub-section (3) of Section 12 or sub-section (5) of
Section 13.
12. In the present case, the trial Court gave benefit to the tenant of Section 12(3) of the
Act in the previous proceedings. The tenant by not depositing the rent either in the Court
or paying it to the landlord, has committed a default and there being three consecutive
defaults in the payment of rent as referred in proviso to sub-section (3) of Section 12 of
the Act and on non-payment of arrears of rent within two months of the service of notice
of demand, the landlord would be entitled to file a second suit for ejectment on the
ground of arrears of rent and the Court has to pass a decree for ejectment under Section
12(l)(a) of the Act.
13. For the aforesaid reasons, Civil Appeal No. 1839/2004 is allowed. The impugned
judgment dated 21-12-2000 passed
@page-SC1524
by the High Court in F.A. No. 86/2000 is set aside. Plaintiffs' (appellants herein) suit is
decreed for ejectment under Section 12(1)(a) of the Act and trial Court's judgment and
decree is confirmed. Since the matter is pending consideration for ejectment of the tenant
since 1992, we direct the executing Court to execute the decree within the period of three
months from the date of filing of the execution application by the landlord.
14. In view of our order in Civil Appeal No. 1839/2004, no orders are required to be
passed in Civil Appeal No. 1840/2004. Civil Appeal No. 1840/2004 stands disposed of
accordingly.
Order accordingly.
AIR 2008 SUPREME COURT 1524 "State of Rajasthan v. Rohitas"
(From :Rajasthan)*
Coram : 2 Dr. A. PASAYAT AND P. SATHASIVAM, JJ.
Criminal Appeal No. 361 of 2008 (arising out of SLP (Cri.) No. 5125 of 2007), D/- 22 -2
-2008.
State of Rajasthan v. Rohitas and Ors.
Criminal P.C. (2 of 1974), S.378(3) - APPEAL - NATURAL JUSTICE - PRINCIPLES -
Appeal against acquittal - Grant of leave to file appeal - Order granting / refusing leave
ought to be reasoned - Giving of reasons is a statutory requirement of principles of
natural justice.
S.B. Cri. Leave to Appeal No. 193 of 2006, D/-31-07-2006 (Raj), Reversed. (Paras
6, 8)
Cases Referred : Chronological Paras
2004 AIR SCW 751 : AIR 2004 SC 1794 : 2004 Cri LJ 1385 (Ref.) 9
2003 AIR SCW 5095 : AIR 2003 SC 4664 : 2003 Cri LJ 5040 (Ref.) 7
(2001) 10 SCC 607 (Rel. on) 6
AIR 1987 SC 724 (Ref.) 6
AIR 1982 SC 1215 (Rel. on) 6
1974 ICR 120 (NIRC)8
(1971)1 All ER 1148 8
Aruneshwar Gupta, for Appellant.
* S. B. Cri. Leave to Appeal No. 193 of 2006, D/- 31-7-2006 (Raj) (Jaipur Bench)
Judgement
1. Dr. ARIJIT PASAYAT, J.:- Leave granted.
2. Challenge in this appeal is to the order passed by a learned Single Judge of the
Rajasthan High Court, Jaipur Bench, dismissing the application filed for grant of leave to
prefer an appeal in terms of Section 378 (1) of the Code of Criminal Procedure, 1973 (in
short the 'Cr.P.C.').
3. Background facts in a nutshell are as follows :
Respondent faced trial for alleged commission of offences punishable under Sections 498
(A) and 304 (B) of the Indian Penal Code, 1860 (in short the 'IPC'). It was the case of the
prosecution that because of the torture meted out for bringing less dowry, she was
murdered. Her dead body was found in the well of the accused persons. It was the case of
the complainant that after killing her for dowry, she was thrown into the well. Charges
were framed and the accused persons faced trial.
The trial Court directed acquittal. Thereafter, as noted above, the appellant-State filed an
application for grant of leave, which was rejected. Stand of the appellant was that the
summary dismissal is not sustainable in law. There is no appearance on behalf of the
respondent-accused.
4. Section 378 of the Code deals with the power of the High Court to grant leave in case
of acquittal. Sub-sections (1) and (3) of Section 378 read as follows :
"378(1) Save as otherwise provided in sub-section (2) and subject to the provisions of
sub-section (3) and (5), the State Government may, in any case, direct the Public
Prosecutor to present an appeal to the High Court from an original or appellate order of
acquittal passed by any Court other than a High Court or an order of acquittal passed by
the Court of Session in revision.
(3) No appeal under sub-section (1) or sub-section (2) shall be entertained except with the
leave of the High Court".
5. To say the least the order is practicably unreasoned.
6
. The effect of the admission of the accused in the background of testimony of official
witnesses and the documents exhibited needed adjudication in appeal. The High Court
has not given any reasons for refusing to grant leave to file appeal against acquittal, and
seems to have been completely oblivious to the fact that by such refusal, a close scrutiny
of the order of acquittal, by the appellate forum, has been lost once and for all. The
manner in which appeal against acquittal has been dealt with by the High Court leaves
much to be desired. Reasons introduce clarity in an order. On plainest AIR 1987 SC
724

@page-SC1525
consideration of justice, the High Court ought to have set forth its reasons, howsoever
brief in its order, indicative of an application of its mind; all the more when its order is
amenable to further avenue of challenge. The absence of reasons has rendered the High
Court order not sustainable. Similar view was expressed in State of U.P. v. Battan and
Ors. (2001 (10) SCC 607). About two decades back in State of Maharashtra v. Vithal Rao
Pritirao Chawan (AIR 1982 SC 1215) the desirability of a speaking order while dealing
with an application for grant of leave was highlighted. The requirement of indicating
reasons in such cases has been judicially recognized as imperative. The view was re-
iterated in Jawahar Lal Singh v. Naresh Singh and Ors. (1987 (2) SCC 222). Judicial
discipline to abide by declaration of law by this Court, cannot be forsaken, under any
pretext by any authority or Court, be it even the highest Court in a State, oblivious to
Article 141 of the Constitution of India, 1950 (in short the 'Constitution').
7

. Reason is the heartbeat of every conclusion, and without the same it becomes lifeless.
(See Raj Kishore Jha v. State of Bihar and Ors. (2003 (7) Supreme 152). 2003 AIR
SCW 5095

8. Even in respect of administrative orders, Lord Denning M.R. in Breen v. Amalgamated


Engineering Union (1971 (1) All E.R. 1148) observed "The giving of reasons is one of
the fundamentals of good administration". In Alexander Machinery (Dudley) Ltd. v.
Crabtree (1974 ICR 120)(NIRC) it was observed : "Failure to give reasons amounts to
denial of justice". Reasons are live links between the mind of the decision-taker to the
controversy in question and the decision or conclusion arrived at". Reasons substitute
subjectivity by objectivity. The emphasis on recording reasons is that if the decision
reveals the "inscrutable face of the sphinx", it can, by its silence, render it virtually
impossible for the Courts to perform their appellate function or exercise the power of
judicial review in adjudging the validity of the decision. Right to reason is an
indispensable part of a sound judicial system; reasons at least sufficient to indicate an
application of mind to the matter before Court. Another rationale is that the affected party
can know why the decision has gone against him. One of the salutary requirements of
natural justice is spelling out reasons for the order made; in other words, a speaking out.
The "inscrutable face of a sphinx" is ordinarily incongruous with a judicial or quasi-
judicial performance.
9
. The above position was highlighted in State of Orissa v. Dhaniram Lunar (2004(5) SCC
568). 2004 AIR SCW 751

10. Therefore, the impugned order of the High Court cannot be sustained and is set aside,
and matter is remitted to it. The High Court shall take up the matter afresh and dispose of
the same in accordance with law. The appeal is allowed without any order as to costs.
Appeal allowed.
AIR 2008 SUPREME COURT 1525 "New India Assurance Co. Ltd. v. Vipin Behari Lal
Srivastava"
(From : Allahabad)*
Coram : 2 Dr. A. PASAYAT AND S. H. KAPADIA, JJ.
Civil Appeal No. 5213 of 2006, D/- 21 -2 -2008.
New India Assurance Co. Ltd. v. Vipin Behari Lal Srivastava
General Insurance Business (Nationalisation) Act (57 of 1972), S.16 - INSURANCE -
TERMINATION OF SERVICE - INDUSTRIAL DISPUTE - SERVICE MATTERS -
Termination of service - Employee, respondent unanthorisedly remained absent for more
than 600 days - Respondent alleged that he was suffering from Tuberculosis - Applied for
medical leave but management did not pass nay order on his application - However
medical certificate from. Registered Medical Practitioner stating diagnosis and probable
duration of treatment as required under Rules not produced - Management by its letter
directed him to join back immediately failing which presumption of abandonment of job
etc. was to be drawn - Respondent did not join even after receipt of said letter - Thus
there was no condonation of absence without leave - Order terminating his services - Was
proper.
Industrial Disputes Act (14 of 1947), Sch.2, Item 3.
S. A. No. 125 of 2006 and CWP No. 17204 of 1998, D/-10-02-2006 and 20-01-2006
(All.), Reversed. (Para 12)
@page-SC1526
Cases Referred : Chronological Paras
(2005)5 SCC 337 (Ref.) 11
Dinesh Mathur, Sunil Murarka, Saurabh Jain, K. Rajeev, for Appellant; Shrish Kumar
Misra, Ajay Kr. Singh, for Respondent.
* S. A. No. 125 of 2006, D/- 10-2-2006 and C. W. P. No. 17204 of 1998, D/- 20-1-2006
(All.)
Judgement
1. Dr. ARIJIT PASAYAT, J.:-Challenge in this appeal is to the order passed by a learned
Single Judge of the Allahabad High Court dismissing the writ petition filed by the
appellant questioning the correctness of the Award dated 28-1-1998 passed in Industrial
Dispute No. 111 of 1987 passed by the Presiding Officer, Central Government Industrial
Tribunal-cum-Labour Court, Kanpur, Uttar Pradesh (in short, 'the Tribunal'). The award
was passed in the reference made by the Central Government, Ministry of Labour,
referring the following dispute for adjudication of the Tribunal :
"Whether the action of the management of New India Assurance Company Limited in
removing Sri Vipin Behari Lal Srivastava, typist, Allahabad from service w.e.f. 15-6-
1985, is legal and justified? If not to what relief the concerned workman is entitled?"
2. The controversy lies within a very narrow compass. The respondent was working as a
permanent typist at the Allahabad branch of the appellant-New India Assurance Co. Ltd.
alleging that he had unauthorizedly remained absent for more than 600 days, a charge
sheet was issued. An Enquiry Officer was appointed and after completion of enquiry and
on consideration of the enquiry report, the respondent was removed from service by order
dated 15-6-1985. Thereafter, a dispute was raised and the reference was made, as noted
above. The Tribunal came to hold that during the period in question, i.e., 25-9-1982 to 5-
6-1984, the respondent was suffering from Tuberculosis and he had applied for medical
leave and since the management did not pass any order on his leave applications, the
concerned workman cannot be held responsible and, therefore, he was not absent
unauthorizedly from duty. Accordingly, the order of removal was set aside and order was
passed directing reinstatement with full back wages and consequential benefits including
continuity of service. The same was challenged before the High Court. By the impugned
order, the High Court observed that though the respondent had remained absent, his
absence with leave stood condoned by virtue of the letter dated 3.8.1984 issued by the
Branch Manager of the appellant Company by which the respondent was called back to
work. It was further observed that the Tribunal had also recorded that the management
did not pass any order on the leave application and, therefore, it had to be implied that
leave had been sanctioned. But it was noted that by virtue of a stay order passed in a writ
petition, the proceedings before the Tribunal had remained stayed for about six years and,
therefore, the respondent was not entitled to back wages for the whole period, but was
entitled from 28-1-1998 i.e. from the date of the award. A Letters Patent Appeal was filed
before the Division Bench of the High Court which dismissed holding the same to be not
maintainable.
3. In the present appeal, the order passed by the learned Single Judge has been
questioned.
4. Learned counsel for the appellant submitted that there was no condonation of the
absence of the leave as has been noted by the Tribunal and the High Court; on the
contrary, in the letter in question it was categorically stated that the prayer for leave even
without pay cannot be granted. Therefore, he was directed to join the duty immediately
and failing which it was to be presumed that he was not interested in the job and it shall
also be presumed that he had abandoned the job. It was also pointed out that with a view
to test the correctness of the stand that respondent was ailing, the Deputy Medical Officer
was sent to the house of the respondent along with a senior officer but the respondent was
found absent and it was gathered that he was hale and healthy. With reference to the
relevant Rules, it is submitted that there was no scope for claiming leave as a matter of
right and sick leave can only be granted on certain conditions being fulfilled which were
not fulfilled by the respondent.
5. In response, learned counsel for the respondent submitted that the respondent was
suffering from Tuberculosis for which there is ample material. The authorities insisted on
a certificate from the Chief Medical Officer but did not write directly to the said Officer
though requested by the respondent. Several applications for leave were made but they
were not dealt with by the appellant and, therefore, the Tribunal and the High Court were
justified in directing
@page-SC1527
reinstatement.
6. The main basis for conclusion of the High Court for assuming condonation of the
absence is the letter dated 3-8-1984. The same needs to be quoted in full. It reads as
follows :
THE NEW INDIA ASSURANCE CO. LTD. REGISTERED
3rd August 84
Mr. V.B.L. Srivastava Sr.
No. 6074 51,
Talab Nawal Rai
New Bairadhana
Allahabad
Dear Sir,
This is with reference to your letter of 31st ultimo. You are aware that no leave is due and
we cannot grant you any further leave even without pay. You are, therefore, required to
join your duty immediately, failing which we shall presume that you are no more
interested in the job and we shall also presume that you have abandoned the job.
Thanking you,
Yours faithfully,
Sd/-
Sr. Divisional Manager"
7. A bare look at it shows that there was no condonation of the absence without leave as
held by the High Court. On the contrary, it was clearly indicated that no leave was due
and even leave without pay cannot be granted. Therefore, direction was given to join
back immediately failing which certain presumptions were to be drawn as noted above.
8. The case of the appellant was really not of abandonment but of an unauthorized
absence.
9. The Rules governing "leave" read as follows
"(1) General Principles Governing Grant of Leave: The following general principle shall
govern the grant of leave to the employees :
(a) Leave cannot be claimed as a matter of right.
(b) Leave shall be availed of only after sanction by the competent authority, but one day's
casual leave may be availed of without prior sanction in case of unforeseen emergency,
provided the head of the office is promptly advised of the circumstances under which
prior sanction could not be obtained"
(4) Sick Leave :
(c) Sick Leave can be granted to an employee only on production of a medical certificate
from a Registered Medical Practitioner, which term would include Homeopathic,
Ayurvedic and Unani doctor also provided they are registered medical practitioners.
(d) The certificate should state as clearly as possible the diagnosis and probable duration
of treatment...................."
10. As noted above, sick leave can be granted only on the production of a medical
certificate from a Registered Medical Practitioner clearly stating as far as possible the
diagnosis and probable duration of treatment. There was no such indication in the
certificates purported to have been furnished by the respondent. It is to be noted that the
respondent even did not join after receipt of the letter dated 3-8-1994. The charges
against the respondent, inter alia, were as follows :
"(i) willful insubordination and disobedience of lawful and reasonable orders of his
superiors
(ii) absence without leave, without sufficient grounds or proper or satisfactory
explanation
(iii) absence from his appointed place of work without permission or sufficient cause"
11. In Viveka Nand Sethi vs. Chairman, JandK Bank Ltd. and Ors. [(2005) 5 SCC 337]
this Court, inter alia, observed as follows :
"14. What fell for consideration before the Industrial Tribunal was the interpretation
and/or applicability of the said settlement. The Industrial Tribunal committed an error of
record insofar as it proceeded on the basis that the said settlement had not been proved.
The settlement being an admitted document should have been considered in its proper
perspective by the Industrial Tribunal. Clause (2) of the said settlement is a complete
code by itself. It lays down a complete machinery as to how and in what manner the
employer can arrive at a satisfaction that the workman has no intention to join his duties.
A bare perusal of the said settlement clearly shows that it is for the employee concerned
to submit a proper application for leave. It is not in dispute that after the period of leave
came to an end in
@page-SC1528
June 1983, the workman did not report back for duties. He also did not submit any
application for grant of further leave on medical ground or otherwise. It is in that
situation the memorandum dated 2-11-1983 was issued and he was asked to join his
duties. It is furthermore not in dispute that despite receipt of the said memorandum, the
workman did not join duties pursuant whereto he was served with a notice to show cause
dated 31-12-1982. He was required to resume his duties by 15-1-1984. The Bank
received a telegram on 17-1-1984 and only about a month thereafter he filed an
application for grant of leave on medical ground. It is not the case of the workman that
any leave on medical ground or otherwise was due to him. Opportunities after
opportunities indisputably had been granted to the workman to explain his position but he
chose not to do so except filing applications for grant of medical leave and that too
without annexing proper medical certificates.
18. Mere sending of an application for grant of leave much after the period of leave was
over as also the date of resuming duties cannot be said to be a bona fide act on the part of
the workman. The Bank, as noticed hereinbefore, in response to the lawyer's notice
categorically stated that the workman had been carrying on some business elsewhere.
19. We cannot accept the submission of Mr. Mathur that only because on a later date an
application for grant of medical leave was filed, the same ipso facto would put an
embargo on the exercise of the jurisdiction of the Bank from invoking clause 2 of the
bipartite settlement.
20. It may be true that in a case of this nature, the principles of natural justice were
required to be complied with the same would not mean that a full-fledged departmental
proceeding was required to be initiated. A limited enquiry as to whether the employee
concerned had sufficient explanation for not reporting to duties after the period of leave
had expired or failure on his part on being asked so to do, in our considered view,
amounts to sufficient compliance of the requirements of the principles of natural justice."
12. In view of the factual position, when tested on the touchstone of the principles of law
and governing rules, the inevitable conclusion is that the impugned order of the High
Court passed by the learned Single Judge dismissing the writ petition, i.e. C.W.P. No.
1720/1998, by order dated 20-1-2006 cannot be sustained and is set aside. The order
passed by the departmental authorities directing removal of the respondent from service
is maintained.
13. The Appeal is allowed without any order as to costs.
Appeal allowed.
AIR 2008 SUPREME COURT 1528 "Som Mittal v. Government of Karnataka"
(From : Karnataka)*
Coram : 3 K. G. BALAKRISHNAN, R. V. RAVEENDRAN AND J. M. PANCHAL,
JJ.
Criminal Appeal No. 206 of 2008, D/- 21 -2 -2008.
Som Mittal v. Government of Karnataka.
(A) Criminal P.C. (2 of 1974), S.482 - INHERENT POWERS - WORDS AND
PHRASES - MURDER - Inherent powers - To be used "sparingly, with circumspection
and in rarest of rare cases" - Words "rarest of rare cases" merely emphasize what is
intended to be conveyed by the words "sparingly and with circumspection" - Expression
"rarest of rare cases" is not used in the sense in which it is used with reference to
punishment for offence u/S.302, IPC.
AIR 1989 SC 2222, 1995 AIR SCW 4100, 1997 AIR SCW 4084, 2004 AIR SCW 6185,
2006 AIR SCW 3830, AIR 1977 SC 2229, 2008 AIR SCW 11, AIR 1960 SC 866, AIR
1992 SC 604, AIR 1980 SC 898, 1994 AIR SCW 1886, Ref. (Para 9)
(B) Constitution of India, Art.133, Art.134 - APPEAL - APPELLATE COURT - Powers
of appellate Court - Court should desist from issuing direction affecting executive or
legislative policy, or general directions unconnected with subject-matter of case.
The subject-matter of an appeal whether civil or criminal is the corrections of the
decision of the lower Court. While rendering judgment, Courts should only deal with the
subject-matter of the case and issues involved therein. There is no question of appellate
Court travelling beyond and making observations alien to the case. Any opinion,
observation, comment or recommendation dehors the subject of the appeal, may lead to
confusion in the minds of litigants, members
@page-SC1529
of public and authorities as they will not know-how to regulate their affairs, or whether to
act upon it. Another aspect that requires to be kept in view is the fact that even when it
becomes necessary for a Court for whatsoever reason, to decide or comment upon an
issue not raised by the parties, it may do so only after notifying the parties concerned so
that they can put forth their views on such issue. When Supreme Court render judgments,
it does so with great care and responsibility. The wider the power, more onerous is the
responsibility to ensure that nothing is stated or directed in excess of what is required or
relevant for the case, and to ensure that the Court's orders and decisions do not create any
doubt or confusion in regard to a legal position in the minds of any authority or citizen,
and also to ensure that they do not conflict with any other decision or existing law.
(Paras 10, 11, 12)
(C) Constitution of India, Art.141, Art.134 - PRECEDENT - APPEAL - Binding
precedent - Criminal appeal - Observation, recommendation and directions made by one
of Judges of the Bench - Said observation, recommendations and directions did not relate
to subject-matter - Same are not binding - Directions issued by M. Katju, J. in 2008 AIR
SCW 1003 are not binding. (Para 13)
Cases Referred : Chronological Paras
2008 AIR SCW 11 : AIR 2008 SC 787 (Rel. on) 8
2006 AIR SCW 3830 : AIR 2006 SC 2780 (Rel. on) 8
2004 AIR SCW 6185 : AIR 2005 SC 9 : 2005 Cri LJ 92 (Rel. on) 8
1997 AIR SCW 4084 : AIR 1998 SC 128 : 1998 Cri LJ 1 : 1997 All LJ 2406 (Rel. on)
8
1995 AIR SCW 4100 : AIR 1996 SC 309 : 1996 Cri LJ 381 (Rel. on) 8
1994 AIR SCW 1886 : AIR 1994 SC 1349 : 1994 Cri LJ 1981 (Ref.) 3
AIR 1992 SC 604 : 1992 Cri LJ 527 (Rel. on) 7
AIR 1989 SC 2222 : 1989 Cri LJ 2301 : 1990 All LJ 62 (Rel. on) 8
AIR 1980 SC 898 : 1980 Cri LJ 636 (Ref.) 3
AIR 1977 SC 2229 : 1977 Cri LJ 1900 (Rel. on) 7
AIR 1960 SC 866 (Rel. on) 8
K.K. Venugopal, K.G. Raghavan, Sr. Advocates, Amit Dhingra and Aman Leekha (for
M/s. Dua Associates), for Appellant;
Sanjay R. Hegde, Vikrant Yadav, Amit Kr. Chawla, Arul Varma, for Respondent.
* Cri. P. No. 1535 of 2006, D/- 28-3-2006 (Kant).
Judgement
K.G. BALAKRISHNAN, C.J.I. :- Government of Karnataka represented by Senior
Labour Inspector, 8th Circle, Bangalore, lodged a complaint under Section 200 of the
Code of Criminal Procedure against the appellant, who was the Managing Director of
M/s. Hewlett Packard Global Soft Ltd., in the Court of the Metropolitan Magistrate (TC-
3), Bangalore, for taking cognizance of an offence punishable under Section 30(3) of the
Karnataka Shops and Commercial Establishments Act, 1961 ('Act' for short) for violation
of Section 25 of the said Act. Learned Magistrate took cognizance by order dated
30.12.2005 and directed issue of notice to the appellant. The appellant filed a petition
under Section 482 of Cr.P.C. for setting aside the said order dated 30-12-2005 and for
quashing the complaint. The High Court, by order dated 28-3-2006, rejected the prayer
for quashing the complaint, but altered the offence in respect of which cognizance was
taken as one under Section 30(1) read with Section 25 of the said Act. The said order of
the High Court was challenged by the appellant in this appeal.
2

. The appeal was heard by a Bench consisting of H.K. Sema and Markandey Katju, JJ. By
the main judgment dated 29-1-2008 Sema, J. dismissed the appeal, making it clear that
the court was not expressing any opinion on the merits of the case and the learned
Magistrate shall decide the maintainability of the complaint at the time of framing of the
charge uninfluenced by any observations made by this Court or the High Court. In the
course of his judgment, Sema, J. observed : Reported in 2008 AIR SCW 1003

"In a catena of decisions this Court has deprecated the interference by the High Court in
exercise of its inherent powers under Section 482 of the Code in a routine manner. It has
been consistently held that the power under Section 482 must be exercised sparingly, with
circumspection and in rarest of rare cases. Exercise of inherent power under Section 482
of the Code of Criminal Procedure is not the rule but it is an exception. The exception is
applied only when it brought to the notice of the Court that grave miscarriage of justice
would be committed if the trial is allowed to proceed where the accused would be
harassed unnecessarily
@page-SC1530
if the trial is allowed to linger
3

. In his concurring judgment, Katju, J. agreed that the appeal should be dismissed without
expressing any opinion on merits. He stated that he was rendering a separate opinion as
he was not in agreement with the view expressed by Sema, J. that the power under
Section 482 of Cr.P.C. should be used only in the "rarest of rare cases", though he agreed
with the observation that the said power should be used sparingly. He was of the view
that the words 'rarest of rare cases' are used only with reference to the death penalty for
an offence under Section 302, IPC (See Bachan Singh v. State of Punjab, AIR 1980 SC
898) and the use of the said words was inappropriate while referring to the scope of
exercise of power under Section 482. Paras 1 to 16 of his judgment related to the criminal
appeal. However in paras 17 to 39 of his judgment, the learned Judge expressed concern
over the situation prevailing in Uttar Pradesh on account of omission of Section 438,
Cr.P.C. relating to anticipatory bail by an amendment to the Code by Section 9 of U.P.
Act 16 of 1976 and the consequential hardship created for the public and difficulties
caused to the Allahabad High Court. He made a recommendation to the U.P. Government
to immediately issue an ordinance repealing Section 9 of U.P. Act 16 of 1976 so as to
restore Section 438, Cr.P.C. in Uttar Pradesh empowering the High Court and Sessions
Courts to grant anticipatory bail. He directed the Registry of this Court to send a copy of
his judgment to the Chief Secretary, Home Secretary and Law Secretary of State of U.P.
and also to the Registrar General of the Allahabad High Court and the President/
Secretary of Allahabad Bar Association, Allahabad High Court Advocates' Association
and Oudh Bar Association forthwith. He also referred to the prevailing practice
of police arresting those suspected of involvement in a crime and the directions issued by
this Court in Joginder Kumar v. State of U.P., 1994 (4) SCC 260, in regard to the
procedure to be followed when arresting a person, and directed that copies of his
judgment be sent to the Chief Secretaries, Home Secretaries and Law Secretaries of all
State Governments and Union Territories with a direction to ensure strict compliance
with said decision. 1994 AIR SCW 1886

4. In view of the difference of opinion on legal issues, the appeal was directed to be
placed before the Chief Justice of India for appropriate orders, though both learned
Judges concurred that the appeal should be dismissed. The matter is accordingly placed
before the Bench of three-Judges.
5. When the matter came up, Mr. K. K.Venugopal, learned senior counsel for the
appellant submitted that having regard to the exemption under Section 3(h) of the Act in
respect of persons in management of an establishment, the Act in entirety was
inapplicable to the appellant who was the Managing Director of the establishment. He
also submitted that the question of violation of Section 25 of the Act did not arise as
Appellant's establishment was exempted from the provisions of Section 25 of the Act by
Government Order dated 9-2-2005 and therefore there was no question of violation of
Section 25 or commission of an offence punishable under Section 31(1) of the Act by his
establishment. He therefore submitted that the complaint ought to have been quashed
when its establishment invoked the High Court to exercise its power under Section 482,
Cr.P.C. On the other hand the learned counsel for the respondent-State submitted that the
object of Section 3(h) of the Act was to exclude persons in management from being
considered as employees entitled to seek benefits and reliefs under the Act. He submitted
that the intention of Section 3(h) was not to exempt 'persons in management' from
incurring liability under the Act. He also submitted that the complaint disclosed violation
of the provisions of the proviso to Section 25 of the Act and therefore the learned
Magistrate rightly took cognizance. It is unnecessary to examine these contentions urged
by the parties, on merits. As already noticed, both the learned Judges have concurred and
dismissed the appeal. What is referred is only the legal issues which did not affect the
final decision of the learned Judges that the appeal should be dismissed.
6. Though the learned Judges did not set down the legal issues, we discern the following
two issues from their opinions :
(i) Whether the power under Section 482, Cr.P.C. should be exercised 'sparingly' or
'sparingly with circumspection and in the rarest of rare cases'?
(ii) Whether the recommendations and directions relating to anticipatory bail and
enforcement
@page-SC1531
of the directions relating to arrest laid down in Joginder Kumar were warranted in this
case?
7

. When Sema, J. observed that the power under Section 482, Cr.P.C. was to be used
'sparingly, with circumspection and in rarest of rare cases', he did not lay down any new
proposition of law, but was merely reiterating what was stated by this Court in several
cases, including Kurukshetra University v. State of Haryana, 1977 (4) SCC 451 and State
of Haryana v. Bhajan Lal [1992 Supp(1) SCC 335]. In Kurukshetra University (supra),
this Court observed "that the statutory power under Section 482 has to be exercised
sparingly with circumspection and "in rarest of rare cases". In Bhajan Lal, this Court
reiterated the word of caution that the power of quashing a criminal proceeding should be
exercised "very sparingly and with circumspection and that too in the rarest of rare
cases". It may not therefore be correct to say that the words 'rarest of rare cases' are
appropriate only when considering death sentence for an offence under Section 302, IPC
or that those words are inappropriate when referring to the ambit of the power to be
exercised under Section 482, Cr.P.C. AIR 1977 SC 2229
AIR 1992 SC 604

. Quashing of a complaint or criminal proceedings under Section 482, Cr.P.C. depends on


the facts and circumstances of each case. The scope and ambit of the power under Section
482 has been explained by this Court in a series of decisions - R.P.Kapur v. State of
Punjab, AIR 1960 SC 866; State of Uttar Pradesh v. R. K. Srivastava, 1989 (4) SCC 59;
State of Haryana v. Bhajan Lal, 1992 Supp (1) SCC 335; Mrs. Rupan Deol Bajaj v.
Kanwar Pal Singh Gill, 1995 (6) SCC 194; Pepsi Foods Ltd. v. Special Judicial
Magistrate, 1998 (5) SCC 749; Zandu Pharmaceutical Works v. Mohd. Shara-ful Haque,
2005 (1) SCC 122 : Indian Oil Corporation v. NEPC India Ltd., 2006 (6) SCC 736, and
sonapareddy Maheedhar v. State of Andhra Pradesh, 2007 (14) Scale 321. This Court in
Bhajan Lal (supra) listed the following categories of cases where power under Section
482 could be exercised either to prevent abuse of the process of any Court or otherwise to
secure the ends of justice : AIR 1989 SC 2222
AIR 1992 SC 604
1995 AIR SCW 4100
1997 AIR SCW 4084
2004 AIR SCW 6185
2006 AIR SCW 3830
2008 AIR SCW 11

"(1) Where the allegations made in the first information report or the complaint, even if
they are taken at their face value and accepted in their entirety do not prima facie
constitute any offence or make out a case against the accused.
(2) Where the allegations in the first information report and other materials, if any,
accompanying the FIR do not disclose a cognizable offence, justifying an investigation
by police officers under Section 156(1) of the Code except under an order of a Magistrate
within the purview of Section 155(2) of the Code.
(3) Where the uncontroverted allegations made in the FIR or complaint and the evidence
collected in support of the same do not disclose the commission of any offence and make
out a case against the accused.
(4) Where, the allegations in the FIR do not constitute a cognizable offence but constitute
only a non-cognizable offence, no investigation is permitted by a police officer without
an order of a Magistrate as contemplated under Section 155(2) of the Code.

(5) Where the allegations made in the FIR or complaint are so absurd and inherently
improbable on the basis of which no prudent person can ever reach a just conclusion that
there is sufficient ground for proceeding against the accused.
(6) Where there is an express legal bar engrafted in any of the provisions of the Code or
the concerned Act (under which a criminal proceeding is instituted) to the institution and
continuance of the proceedings and/or where there is a specific provision in the Code or
the concerned Act, providing efficacious redress for the grievance of the aggrieved party.
(7) Where a criminal proceeding is manifestly attended with mala fide and/or where the
proceeding is maliciously instituted with an ulterior motive for wreaking vengeance on
the accused and with a view to spite him due to private and personal grudge."
It was also made clear that it was not possible to lay down precise and inflexible
guidelines or any rigid formula or to give an exhaustive list of the circumstances in which
such power could be exercised.
9. When the words 'rarest of rare cases' are used after the words 'sparingly and with
circumspection' while describing the scope of Section 482, those words merely
emphasize and reiterate what is intended to be conveyed
@page-SC1532
by the words 'sparingly and with circumspection'. They mean that the power under
Section 482 to quash proceedings should not be used mechanically or routinely, but with
care and caution, only when a clear case for quashing is made out and failure to interfere
would lead to a miscarriage of justice. The expression "rarest of rare cases" is not used in
the sense in which it is used with reference to punishment for offences under Section 302,
IPC, but to emphasize that the power under Section 482, Cr.P.C. to quash the FIR or
criminal proceedings should be used sparingly and with circumspection. Judgments are
not to be construed as statutes. Nor words or phrases in judgments to be interpreted like
provisions of a statute. Some words used in a judgment should be read and understood
contextually and are not intended to be taken literally. Many a time a Judge uses a phrase
or expression with the intention of emphasizing a point or accentuating a principle or
even by way of a flourish of writing style. Ratio decidendi of a judgment is not to be
discerned from a stray word or phrase read in isolation.
10. The second issue involves the recommendations made to the Government of U.P. and
directions issued to all States and Union Territories in paras 17 to 39 of the concurring
judgment. The appeal related to the question whether the complaint against the appellant
disclosed the ingredients of an offence under Section 25 of the Karnataka Shops and
Commercial Establishments Act, 1961. The appeal did not relate to grant of anticipatory
bail nor did it relate to rights of arrested persons. This Court has repeatedly cautioned that
while rendering judgments, courts should only deal with the subject-matter of the case
and issues involved therein. Courts should desist from issuing directions affecting
executive or legislative policy, or general directions unconnected with the subject-matter
of the case. A court may express its views on a particular issue in appropriate eases only
where it is relevant to the subject-matter of the case.
11. The subject-matter of an appeal, whether civil or criminal, is the correctness of the
decision of the court below. There is no question of appellate court travelling beyond and
making observations alien to the case. Any opinion, observation, comment or
recommendation dehors the subject of the appeal, may lead to confusion in the minds of
litigants, members of public and authorities as they will not know-how to regulate their
affairs, or whether to act upon it. Another aspect that requires to be kept in view is the
fact that even when it becomes necessary for a court for whatsoever reason, to decide or
comment upon an issue not raised by the parties, it may do so only after notifying the
parties concerned so that they can put forth their views on such issue.
12. When this Court renders judgments, it does so with great care and responsibility. The
law declared by this Court is binding on all courts. All authorities in the territory of India
are required to act in aid of it. Any interpretation of a law or a judgment, by this Court, is
a law declared by this Court. The wider the power, more onerous is the responsibility to
ensure that nothing is stated or directed in excess of what is required or relevant for the
case, and to ensure that the Court's orders and decisions do not create any doubt or
confusion in regard to a legal position in the minds of any authority or citizen, and also to
ensure that they do not conflict with any other decision or existing law. Be that as it may.
13. Insofar as the observations, recommendations, and directions in paras 17 to 39 of the
concurring judgment, suffice it to say that they do not relate to the subject-matter of the
criminal appeal and being the expression of an expectation or hope by only one of the
learned Judges constituting the Bench and not agreed to by the other, is not a decision,
order or direction of the Court. That being so, the directions issued to the Secretary
General of the Supreme Court, State Governments and Union Territories, and
recommendations to the Government of U.P. in the "aside" contained in Paras 17 to 39 of
the concurring judgment are not directions to be complied with.
14. The two questions are answered accordingly.
Order accordingly.
@page-SC1533
AIR 2008 SUPREME COURT 1533 "Mundrika Dubey v. State of Bihar"
(From : Patna)*
Coram : 2 TARUN CHATTERJEE AND H. S. BEDI, JJ.
Civil Appeal No. 1468 of 2008 (arising out of SLP (C) No. 7972 of 2006), D/- 21 -2
-2008.
Mundrika Dubey and Ors. v. State of Bihar and Ors.
(A) Bihar and Orissa Co-operative Societies Act (6 of 1935), S.66 - Bihar Rajya Sahkari
Bhumi Vikas Bank Samiti Rules, R.235, R.232 - CO-OPERATIVE SOCIETIES -
RETIREMENT - Bank employee - Compulsory retirement - Bank getting over-staffed -
Suffering losses - Decision to compulsory retire Class IV employees who have reached
age of 50 years and completed 30 years' service - Falls within powers conferred by
R.235. (Para 8)
(B) Constitution of India, Art.226, Art.32 - Bihar and Orissa Co-operative Societies Act
(6 of 1935), S.66 - WRITS - CO-OPERATIVE SOCIETIES - RETIREMENT - Co-
operative Bank employee - Compulsory retirement in interest of Bank - Power to choose
employee for compulsory retirement - within exclusive discretion of Bank - Court would
not interfere. (Para 9)

Ms. Priya Hingorani, (for M/s. Hingorani and Associates), for Appellants; Shravan
Kumar, Sr. Advocate, B. P. Yadav, Sanjeev Malhotra, Gopal Singh and Manish Kumar,
for Respondents.
* L.P.A. No. 1184 of 2004, D/- 7-12-2005 (Pat).
Judgement
1. HARJIT SINGH BEDI, J. :-Leave granted.
2. This appeal is directed against the order dated December 7, 2005 of the Division Bench
of the Patna High Court whereby the judgment of the learned Single Judge dated October
12, 2004 dismissing the Writ Petition has been confirmed. The facts of the case are as
under:
3. The appellants were appointed as Class IV employees i.e. Peons in the respondent-
Bank in the year 1971. They have been compulsorily retired by the Bank vide order dated
June 5, 2004 made purportedly under Rules 232 and 235 of the Bihar Rajya Shakari
Bhumi Vikas Bank Samiti (hereinafter referred to as the "Rules"). Aggrieved against the
order dated June 5, 2004, the appellants filed several writ petitions in the Patna High
Court raising pleas, inter alia, that the action taken by the Bank was not justified under
Rule 232 as they had not been retired compulsorily on the ground of inefficiency and that
Rule 235 was not a source of power so as to justify an order of compulsory retirement as
it only dealt with the grant of contributory provident fund and gratuity to those employees
who had reached the age of 50 years and had completed 30 years of service at the time of
retirement. In response to the notice issued by the High Court, the respondent-Bank filed
its reply. It was, inter alia, pointed out that the Bank was grossly over-staffed and being
inefficiently run with the result that it had sustained huge losses that had brought it to a
precarious financial position, and that before the action had actually been taken against
the appellants, a committee had been set-up which had examined the entire structure of
the Bank and as a follow-up a large number of offices and Branch Offices had been
closed and a consequent re-structuring made of those which still continued to operate. It
was further highlighted that the over-staffing of the Bank was evident as against the total
requirement of 166 Peons, 507 had in fact been appointed and that the decision to
compulsorily retire the appellants had been taken with hesitation and as one of the
measures necessary to ensure the survival of the Bank. It was also pointed out that the
Board of Directors in its meeting held on December 24, 2003 had examined the relevant
facts and concluded that in the first phase, compulsory retirement should be ordered of
lower grade employees who had completed 30 years of service and 50 years of age. A
copy of these proceedings have been appended as Annexure P-1 to the Paper Book.
4. The learned Single Judge in his judgment dated October 12, 2004 accepted the
explanation tendered by the respondent insofar as the factual aspect was concerned and
also observed that Rule 232 was not applicable in such a case whereas Rule 235 was in
fact applicable and accordingly dismissed the Writ Petition, as already mentioned above.
The appeal filed before the Division Bench of the High Court was also dismissed.
5. Ms. Priya Hingorani, the learned counsel for the appellants has forcefully argued that
Rule 235 on which the respondents had placed reliance for dispensing with the services
@page-SC1534
of the appellants by way of compulsory retirement was not in fact a source of power as it
only dealt with the payment of gratuity and provident fund to a certain category of
employees and that it was only under Rule 232 that an employee could be compulsorily
retired and that too on the ground of inefficiency and as it was not the case of the
respondent-Bank that the appellants were inefficient, the impugned action was
unjustified. It has also been urged that the appellants had put in more than 30 years of
service with the Bank and if any re-structuring was to be made so as to make Bank's
working more efficient, it would have been appropriate to dispense with the staff at the
top i.e. senior officers of the Bank rather than the low paid Class IV employees such as
the appellants.
6. These arguments have been controverted by Shri Shravan Kumar, the learned senior
counsel for the respondents who has pointed out that Rule 235 was Itself the source of
power and operated in a field different from Rule 232 and that it was not for this Court to
interfere so as to determine as to which employee should be retrenched first, and who
later, as this was a matter for the internal administration of the Bank.
7. We have heard the learned counsel for the parties and gone through the record. Rules
232 and 235 are reproduced herein-below :
"Rule 232 : The Bank may, any Bank employee who has completed 21 years of duty and
25 years of total service calculated from the date of his first appointment to retire from
the Bank's services if it is considered that the efficiency or conduct of the employee is not
such as to justify his retention in service. Where any Bank employee is so required to
retire, no claim to any special compensation shall be entertained.
"Rule 235 : A person who retires voluntarily or is required to be retired compulsorily in
Bank's interest on reaching the age of 50 years completing 30 years of service shall be
entitled to contributory provident fund and gratuity as admissible."
8. Concededly the action against the appellants has not been taken under Rule 232 which
deals with the compulsory retirement of an employee who has put in 21 years of duty and
25 years of total service if it is considered that the efficiency or the conduct of the
employee does not justify his retention in service. Rule 235 however talks about
compulsory retirement in the Bank's interest of those who have reached the age of 50
years and have completed 30 years of service and also talks about the benefit of
contributory provident fund and gratuity, as admissible to such employees. Undoubtedly,
action under Rule 232 can only be taken if the employee concerned is inefficient or is
guilty of misconduct whereas the scope of Rule 235 is much wider and compulsory
retirement can be ordered in the Bank's interest. The fact that the two Rules operate in
different fields is also clear from the varying qualifying servoce and that those who retire
under Rule 235 are given some additional financial benefits as a solatium for having to
go despite the fact that their efficiency has in no manner been impaired and merely
because Rule 235 also talks about the payment of contributory provident fund and
gratuity it does not take away the right to retire compulsorily those who have reached the
age of 50 years and have completed 30 years of service and whose retirement is in the
Bank's interest. We therefore endorse the observations of the High Court that the action
taken was Justified under Rule 235, as correct.
9. We are equally of the opinion that it is not for this Court to opine as to who should be
retained in service and who should be retired and at what stage and situation as this is a
matter to be left to the exclusive discretion of the employer. The facts of the case show
that the Bank was not only heavily over-staffed but was also running into huge losses and
substantial pruning which would undoubtedly be hurtful, was required for its survival.
10. We accordingly find no merit in the appeal. Dismissed with no order as to costs.
Appeal dismissed.
AIR 2008 SUPREME COURT 1534 "K. V. Rami Reddi v. Prema"
(From : 2000 (2) Mad LW 826)
Coram : 2 Dr. A. PASAYAT AND P. SATHASIVAM, JJ.
Civil Appeal No. 2551 of 2001, D/- 20 -2 -2008.
K.V. Rami Reddi v. Prema.
Civil P.C. (5 of 1908), O.20, R.1, R.3 - JUDGMENT - Judgment - Pronouncement -
Mode - Declaration of final result orally - Before concise statement of case, points for
@page-SC1535
determination, the decision therefor and reasons therefor have been finalised - Improper.
The declaration by a Judge of his intention of what his 'judgment' is going to be, or a
declaration of his intention of what final result it is going to embody, is not a judgment
until he had crystallized his intentions into a formal shape and pronounced it in open
Court as the final expression of his mind. Civil P. C. does not envisage the writing of a
judgment after deciding the case by an oral judgment and it must not be resorted to and it
would be against public policy to ascertain by evidence along what the 'judgment' of the
Court wag, where the final result was announced orally but the 'judgment', as defined in
the CPC embodying a concise statement of the case, the points for determination, the
decision thereon and the reasons for such decision, was finalized later on. The mere fact
that a major portion of the judgment has been already dictated, will not, by itself, lead to
the conclusion that the Judgment had been delivered. (Paras 9, 11, 12)
Cases Referred : Chronological Paras
1999 AIR SCW 3345 : AIR 1999 SC 3381 (Rel. on) 14
AIR 1969 SC 1167 (Ref.) 10
V. Balachandran, for Appellant; V. Ramasubramanian, for Respondent.
Judgement
Dr. ARIJIT PASATAT, J. :- Heard learned counsel for the parties.
2. Challenge in this appeal is to the judgment of a learned single Judge of the Madras
High Court allowing the Civil Revision petition filed highlighting the irregularities
committed by the learned Seventh Assistant City Civil Judge, Chennai while pronouncing
the judgment in O.S. No. 584 of 1996. The controversy in the suit need not be detailed, as
the points in issue in the present appeal lie within a very narrow compass.
3. The Suit was filed by the present respondent for specific performance to enforce a sale
agreement dated 20-10-1988. The suit is stated to have been decided on 24-03-1999.
According to the present respondent, who was the petitioner in the Civil Revision
petition, even without dictating the judgment to the Stenographer, transcribing and
signing the same, simply an endorsement in the plaint docket sheet was made to the effect
that the plaintiff in the suit was not entitled to the relief of specific performance to
enforce a sale agreement but was entitled to refund of Rs. 2,00,000/-. Stand in the
revision petition was that there was no judgment in the eye of law. It was pointed out that
only the operative portion was dictated on 25-03-1999 during lunch time and, therefore,
the decision rendered on 24.03.1999 was non est in the eye of law and a nullity. Learned
counsel appearing for the respondent in the Civil Revision petition i.e. the present
appellant took the stand that four issues and an additional issue had been framed. The
entire judgment had been dictated by learned Single Judge and the transcribed part
covered the vital issues 1 to 3 and the Stenographer was half way through the fourth issue
and the additional issue. Therefore, it was submitted that a reasonable inference should be
drawn that all the issues had been dictated to the stenographer and on the date the
judgment was pronounced, i.e. 24-03-1999, the judgment must be deemed to have been
completed. Learned Single Judge did not find substance in the stand taken by the present
appellant. It was held that since the learned Trial Judge had not completed the judgment
before he delivered his decision, it has to be held that there was no judgment in the eye of
law. Accordingly, the Civil Revision petition was allowed and Judgment dated 24-03-
1999 was set aside and the matter was remitted to the present Seventh Assistant City
Civil Judge, Chennai who was to hear the arguments afresh and render a decision.
4. Learned counsel for the appellant submitted that the course adopted by learned City
Civil Judge is permissible in law in the background of Order XX, Rule 5 of the Code of
Civil Procedure, 1908 (in short 'the CPC').
5. Learned counsel for the respondent, on the other hand, submitted that the Trial Judge
has not decided the matter in the background of Order XX, Rule 5, CPC. On the contrary,
the provisions of Order XX, Rules 1 and 3 apply to the facts of the case.
6. Order XX, Rule 1 (1) of the CPC (Madras Amendment) reads as follows :
"(1) The Court, after the case has been heard, shall pronounce judgment in open Court,
either at once or on some future day, of which due notice shall be given to the parties or
their pleaders.
(2) The judgment may be pronounced by dictation to a shorthand-writer in open court,
where the presiding Judge has been
@page-SC1536
specially empowered in that behalf by the High Court."
Similarly, Order XX, Rule 3 reads as follows :
"The judgment shall be dated and signed by the Judge in open Court at the time of
pronouncing it and when once signed, shall not afterwards be altered or added to save as
provided by Section 152 or on review."
7. Order XX, Rule 5 on which great emphasis was laid by learned counsel for the
appellant says that in Suits in which issues have been framed, the Court shall state its
finding or decision with the reason therefor, upon each separate issue, unless the finding
upon any one or more of the issues is sufficient for the decision of the Suit.
8. As rightly submitted by learned counsel for the respondent, this was not the view
expressed by the learned Trial Judge.
9. The ultimate question is whether in the instant case the judgment has been validly
delivered? If it is a mere procedural irregularity and the Judge concerned had not signed
the judgment, then the judgment thus rendered cannot be invalidated. Order XX Rule 1
CPC postulates that after the case has been heard, the court hearing the same shall
pronounce the judgment in open court by dictation to the shorthand writer, wherever it is
permissible. It bears the date on which it is pronounced. The date of the judgment is
never altered by the date on which the signature has been put subsequently. The mere fact
that a major portion has been dictated by the learned Judge in the judgment already
dictated, will not, by itself, lead to the conclusion that the judgment had been delivered.
10. In Smt. Swaran Lata Ghosh v. Harendra Kumar Banerjee and Anr. (AIR 1969 SC
1167), it was inter alia held as follows (at Para 6) :
"Trial of a civil dispute in Court is intended to achieve, according to law and the
procedure of the Court, a judicial determination between the contesting parties of the
matter in controversy. Opportunity to the parties interested in the dispute to present their
respective cases on question of law as well as fact, ascertainment of facts by means of
evidence tendered by the parties and adjudication by a reasoned judgment of the dispute
upon a finding on the facts in controversy and application of the law to the facts found,
are essential attributes of a judicial trial. In a judicial trial, the judge not only must reach a
conclusion which he regards as just, but, unless otherwise permitted, by the practice of
the Court or by law, he must record the ultimate mental process leading from the dispute
to its solution. A judicial determination of a disputed claim where substantial questions of
law or fact arise is satisfactorily reached, only if it be supported by the most cogent
reasons that suggest themselves to the Judge; a mere order deciding the matter in dispute
not supported by reasons is no judgment at all. Recording of reasons in support of a
decision of a disputed claim serves more purposes than one. It is intended to ensure that
the decision is not the result of whim or fancy, but of a judicial approach to the matter in
contest; it is also intended to ensure adjudication of the matter according to law and the
procedure established by law. A party to the dispute is ordinarily entitled to know the
grounds on which the Court has decided against him, and more so, when the judgment is
subject to appeal. The Appellate Court will then have adequate material on which it may
determine whether the facts are properly ascertained, the law has been correctly applied
and the resultant decision is just. It is unfortunate that the learned Trial Judge has
recorded no reasons in support of his conclusion, and the High Court in appeal merely
recorded that they thought that the plaintiff had sufficiently proved the case in the plaint."
11. The declaration by a Judge of his intention of what his 'judgment' is going to be, or a
declaration of his intention of what final result it is going to embody, is not a judgment
until he had crystallized his intentions into a formal shape and pronounced it in open
court as the final expression of his mind.
12. The CPC does not envisage the writing of a judgment after deciding the case by an
oral judgment and it must not be resorted to and it would be against public policy to
ascertain by evidence alone what the 'judgment' of the Court was, where the final result
was announced orally but the 'judgment', as defined in the CPC embodying a concise
statement of the case, the points for determination, the decision thereon and the reasons
for such decision, was finalized later on.
13. Section 2(9) of the CPC defines a "judgment" to mean the statement given by
@page-SC1537
the Judge of the grounds for a decree or order.
14

. In Balraj Taneja and Anr. Vs. Sunil Madan and Anr. (1999 (8) SCC 396), it was inter
alia heldas follows : 1999 AIR SCW 3345, Paras 40 and 40A

"There is yet another infirmity in the case which relates to the "judgment" passed by the
single Judge and upheld by the Division Bench.
"Judgment" as defined in Section 2(9) of the Code of Civil Procedure means the
statement given by the Judge of the grounds for a decree or order. What a judgment
should contain is indicated in Order 20 Rule 4(2) which says that a judgment "shall
contain a concise statement of the case, the points for determination, the decision thereon,
and the reasons for such decision". It should be a self-contained document from which it
should appear as to what were the facts of the case and what was the controversy which
was tried to be settled by the Court and in what manner. The process of reasoning by
which the Court came to the ultimate conclusion and decreed the suit should be reflected
clearly in the judgment."
15. Undisputedly, the Trial Judge had not completed the judgment before he delivered his
decision. That being so, the impugned judgment does not suffer from any infirmity to
warrant interference. What the High Court has directed is to hear only the arguments
afresh. While dismissing the appeal, we direct that the arguments shall be heard afresh
and the Trial Court shall deliver its judgment as early as practicable, preferably within
three months from today. To avoid unnecessary delay, let the parties appear before the
Trial Court on 05.03.2008 so that the date for arguments can be fixed.
Appeal dismissed.
AIR 2008 SUPREME COURT 1537 "Liyakat v. State of Uttaranchal"
(From : Uttaranchal)*
Coram : 2 Dr. A. PASAYAT AND P. SATHASIVAM, JJ.
Criminal Appeal Nos. 378, 379 of 2008 (arising out of SLP (Cri.) No. 3314 with 3316 of
2006), D/- 25 -2 -2008.
Liyakat v. State of Uttaranchal.
(A) Evidence Act (1 of 1872), S.3 - EVIDENCE - Circumstantial evidence - An indirect
mode of proof - By drawing inference from facts closely connected to fact in issue.
For a crime to be proved it is not necessary that the crime must be seen to have been
committed and must, in all circumstances be proved by direct ocular evidence by
examining before the Court those persons who had seen its commission. The offence can
be proved by circumstantial evidence also. The principal fact or factum probandum may
be proved indirectly by means of certain inferences drawn from factum probans, that is,
the evidentiary facts. To put it differently, circumstantial evidence is not direct to the
point in issue but consists of evidence of various other facts which are so closely
associated with the fact in issue that taken together they form a chain of circumstances
from which the existence of the principal fact can be legally inferred or presumed. When
a case rests squarely on circumstantial evidence, the inference of guilt can be justified
only when all the incriminating facts and circumstances are found to be incompatible
with the innocence of the accused or the guilt of any other person. (Paras 13, 14)
(B) Penal Code (45 of 1860), S.300 - MURDER - EVIDENCE - Murder - Circumstantial
evidence - Deceased, child of one and half year, placed in custody of accused by its
parents - Parents who enquired about child on their return from field were offered no
explanation as to missing of child by accused - Dead body of child found buried in hut
exclusively possess by accused - No explanation by accused as to how body was found
buried in his hut - Silence of accused is a vital circumstance which adds to chain of
circumstances - Accused liable to be convicted for murder. (Paras 23, 24, 25)
Cases Referred : Chronological Paras
2003 AIR SCW 4097 : AIR 2003 SC 3601 : 2003 Cri LJ 3901 (Ref.) 22
1996 AIR SCW 2903 : AIR 1996 SC 3390 : 1996 Cri LJ 3461 (Ref.) 15
1992 AIR SCW 640 : AIR 1992 SC 840 : 1992 Cri LJ 1104 : 1992 All LJ 1115 (Ref.)
17
AIR 1990 SC 79 : 1990 Cri LJ 605 (Ref.) 16
AIR 1989 SC 1890 : 1989 Cri LJ 2124 (Rel. on) 14
@page-SC1538

AIR 1987 SC 350 : 1987 Cri LJ 330 (Rel. on) 14


AIR 1985 SC 1224 : 1985 Cri LJ 1479 (Rel. on) 14
AIR 1984 SC 1622 : 1984 Cri LJ 1738 21
AIR 1983 SC 446 : 1983 Cri LJ 846 (Rel. on) 14
AIR 1977 SC 1063 : 1977 Cri LJ 639 (Rel. on) 14
AIR 1956 SC 316 : 1956 Cri LJ 559 (Rel. on) 14
AIR 1954 SC 621 : 1954 Cri LJ 1645 (Rel. on) 14
AIR 1952 SC 343 : 1953 Cri LJ 129 (Ref.) 20
Y.P. Singh, (AC), C. Siddharth, Mrs. Prakriti Purnima, for Appellant; Ms. Rachna
Srivastava, for Respondent.
* Cri. Appeal No. 45 of 2004, D/- 6-10-2004 (Utr)
Judgement
Dr. ARIJIT PASAYAT, J. :- Leave granted.
2. These two appeals are directed against the common judgment of the Uttaranchal High
Court.
3. Challenge in this appeal is to judgment of a Division Bench of the Uttaranchal High
Court which disposed of Criminal Reference No. 2 of 2004 and two Criminal Appeal
Nos. 45 and 46 of 2002. The two criminal appeals were filed by Liyakat the present
appellant and co-accused Smt. Zahira. The reference was necessitated as the Learned
Additional District and Sessions Judge, Ist Fast Track Court, Hardwar has awarded death
sentence to the accused Liyakat. He had also awarded sentences of imprisonment for life
to Zahira and the accused No. 3. Both of them were convicted for offences punishable
under Sections 302 and 201 of the Indian Penal Code, 1860 (in short the 'IPC'). The trial
court, however had acquitted Accused Nos. 2 and 4 namely Riyasat and Jeewani.
4. Noor Alam was child of PW 1 Rashid and his wife PW 2 Nasreen. PW 1 Rashid and
PW2 Nasreen used to reside in the neighbourhood of Jeewani, original accused No. 4. On
the fateful day i.e. on 12th day of January, 1999, Nasreen was sitting along with her child
in the courtyard of Gulami's house, who is husband of original accused No. 4 Jeewani. At
that time Zaheera, Jiwani, Liyakat and Riyasat were also there. Her husband Rashid, P.W.
1 came there and asked his wife to go with him to feed fertilizer to the standing crop in
the field. However, Nasreen declined to go with him as there was nobody to look after
Noor Alam. Hearing this, Jeewani and other accused persons told that they will look after
the child and she could leave the child with them. P.W. 3 Brahm Pal and one Rishipal
were also there at that time. The child was left by Nasreen with the accused persons and
she left the place along with her husband. When both of them returned at about 4 O'clock,
they straightway went to the house of Gulami and enquired about Noor Alam. Accused
told them that Noor Alam was playing in the vicinity only. However, they could not find
the child. They searched for the child for the rest of the day and night and even on 13th of
January, 1999, but to no effect. However, in the morning of 14th, when Rashid was
searching for child along with Brahmpal, Bhagwan and Yasin, they searched the house or
hut of Liyakat and saw that in the northern corner of that hut, foot of small child was
protruding out of the ground. Seeing this Rashid reached to the Police Station, Laksar and
reported the matter.
5. A case was registered on that basis and the Incharge of the Police out-post (Chauki)
was informed on wireless and received the message. O.P. Sisodia PW 8 with other
officials reached the house of the accused and in presence of the witnesses, body of Noor
Alam, which was buried in the pit in the northern corner of the hut was recovered.
Panchnama and other formalities were completed and on that very day all the accused
persons were arrested.
6. The prosecution relied on eight witnesses. They being the parents of the child PW 1
Rashid and PW 2 Nasreen and two other witnesses PW 3 Brahm Pal and PW 4 Rishipal.
They all supported the prosecution story. Besides them, PW 6 Dr. R.K. Pande is the
medical officer, who had conducted post mortem on the dead body and PW 7 Rishipal
and PW 8 O.P. Sisodia are the police witnesses.
7. Case of the prosecution in short was that appellant Liyakat, Riyasat, Zahira and
Jeewani committed murder of Noor Alam a child aged about 1½ years, the deceased
and they buried the body of the child with an idea of screening the act. The trial court
accepted the prosecution version in part so far as the appellant and Zahira are concerned
but directed acquittal of the co-accused. The trial court found that the evidence was clear
and cogent and therefore
@page-SC1539
the appellant Liyakat was given life sentence and Zahira was given death sentence. Since
the sentence of death was awarded, the reference was made to the High Court for
confirmation in terms of Section 367 of the Code of Criminal Procedure, 1973 (in short
the 'Cr. P.C.').
8. The High Court by the impugned judgment altered the death sentence awarded to
appellant Liyakat to life sentence. It however upheld the sentence of life imprisonment
awarded to Zahira.
9. In support of the appeal learned counsel for the appellant submitted that the case based
on circumstantial evidence and the circumstances highlighted by the trial court and the
High Court do not warrant a conclusion that the appellants were responsible for the
murder of the child.
10. With reference to the medical evidence, it was submitted that this was not a case of
strangulation and therefore the conclusion of killing the child of suffocation cannot be
maintained.
11. The circumstances which were highlighted against the appellant was the fact that the
child was left in the custody of the appellants, the dead body was found buried in the
premises of the appellants. No explanation by way of suggestion in cross examination or
in the examination under Section 313 Cr.P.C. was offered as to how the dead body was
found buried in the hut of the accused which was in his exclusive use.
12. Following circumstances were highlighted by the prosecution to substantiate its
accusations :
(1) That the child Noor Alam was handed over in the custody of the accused persons at
about 1 O'clock by the parents;
(1A) That the child was either a toddler or a crawling boy;
(2) That the child was missing barely within three hours after he was given in the custody
of the accused persons;
(3) That there was no explanation given by the accused for the missing of the boy to the
parents and they only casually replied that the child must have been playing somewhere
else;
(4) That the dead-body of the child was found buried in the hut, which was in the use and
occupation of the accused persons;
(5) That there is no explanation whatsoever as to how his body came to be buried in the
hut of these accused persons;
(6) That the unsubstantiated defence raised by the appellant No.2 Zaheera suggesting the
alibi, which could not be proved at all and has been rightly disbelieved by the trial Court;
(7) That the child died unnatural and homicidal death due to suffocation and that the child
had died even before it was buried.
13. Before analysing the factual aspects it may be stated that for a crime to be proved it is
not necessary that the crime must be seen to have been committed and must, in all
circumstances be proved by direct ocular evidence by examining before the court those
persons who had seen its commission. The offence can be proved by circumstantial
evidence also. The principal fact or factum probandum may be proved indirectly by
means of certain Inferences drawn from factum probans, that is, the evidentiary facts. To
put it differently, circumstantial evidence is not direct to the point in issue but consists of
evidence of various other facts which are so closely associated with the fact in issue that
taken together they form a chain of circumstances from which the existence of the
principal fact can be legally inferred or presumed.
14
. It has been consistently laid down by this Court that where a case rests squarely on
circumstantial evidence, the inference of guilt can be justified only when all the
incriminating facts and circumstances are found to be incompatible with the innocence of
the accused or the guilt of any other person. (See Hukam Singh v. State of Rajasthan
(1977)2 SCC 99), Eradu v. State of Hyderabad (AIR 1956 SC 316), Earabhadrappa v.
State of Karnataka [(1983) 2 SCC 330], State of U.P. v. Sukhbasi [AIR 1985 SC 1224],
Balwinder Singh v. State of Punjab [AIR 1987 SC 350) and Ashok Kumar Chatterjee v.
State of M.P. [AIR 1989 SC 1890). The circumstances from which an inference as to the
guilt of the accused is drawn have to be proved beyond reasonable doubt and have to be
shown to be closely connected with the principal fact sought to be inferred from those
circumstances. In Bhagat Ram v. State of Punjab [AIR 1954 SC 621] it was laid down
that where the case depends upon the conclusion drawn from circumstances the
cumulative effect of the circumstances must be as to negative the AIR 1977 SC 1063
AIR 1983 SC 446

@page-SC1540
innocence of the accused and bring the offences home beyond any reasonable doubt.
15

. We may also make a reference to a decision of this Court in C. Chenga Reddy v. State of
A. P. [1996 (10) SCC 193] wherein it has been observed thus : (SCC pp. 206-07, para 21)
1996 AIR SCW 2903, (Para 20A)

"21. In a case based on circumstantial evidence, the settled law is that the circumstances
from which the conclusion of guilt is drawn should be fully proved and such
circumstances must be conclusive in nature. Moreover, all the circumstances should be
complete and there should be no gap left in the chain of evidence. Further, the proved
circumstances must be consistent only with the hypothesis of the guilt of the accused and
totally inconsistent with his innocence."
16. In Padala Veera Reddy v. State of A.P. (AIR 1990 SC 79] it was laid down that when
a case rests upon circumstantial evidence, such evidence must satisfy the following tests :
(SCC pp. 710-11, para 10)
"(1) the circumstances from which an inference of guilt is sought to be drawn, must be
cogently and firmly established;
(2) those circumstances should be of a definite tendency unerringly pointing towards guilt
of the accused;
(3) the circumstances, taken cumulatively, should form a chain so complete that there is
no escape from the conclusion that within all human probability the crime was committed
by the accused and none else; and
(4) the circumstantial evidence in order to sustain conviction must be complete and
incapable of explanation of any other hypothesis than that of the guilt of the accused and
such evidence should not only be consistent with the guilt of the accused but should be
inconsistent with his innocence."
17
. In State of U.P. v. Ashok Kumar Srivastava [1992(2) SCC 86] it was pointed out that
great care must be taken in evaluating circumstantial evidence and if the evidence relied
on is reasonably capable of two inferences, the one in favour of the accused must be
accepted. It was also pointed out that the circumstances relied upon must be found to
have been fully established and the cumulative effect of all the facts so established must
be consistent only with the hypothesis of guilt. 1992 AIR SCW 640

18. Sir Alfred Wills in his admirable book Wills' Circumstantial Evidence (Chapter VI)
lays down the following rules specially to be observed in the case of circumstantial
evidence :
"(1) the facts alleged as the basis of any legal inference must be clearly proved and
beyond reasonable doubt connected with the factum probandum; (2) the burden of proof
is always on the party who asserts the existence of any fact, which infers legal
accountability; (3) in all cases, whether of direct or circumstantial evidence the best
evidence must be adduced which the nature of the case admits; (4) in order to justify the
inference of guilt, the inculpatory facts must be incompatible with the innocence of the
accused and incapable of explanation, upon any other reasonable hypothesis than that of
his guilt; and (5) if there be any reasonable doubt of the guilt of the accused, he is entitled
as of right to be acquitted."
19. There is no doubt that conviction can be based solely on circumstantial evidence but
it should be tested on the touchstone of law relating to circumstantial evidence laid down
by this Court as far back as in 1952.
20. In Hanumant Govind Nargundkar v. State of M.P. [AIR 1952 SC 343] it was observed
thus : (AIR pp. 345-46, para 10)
"It is well to remember that in cases where the evidence is of a circumstantial nature, the
circumstances from which the conclusion of guilt is to be drawn should in the first
instance be fully established, and all the facts so established should be consistent only
with the hypothesis of the guilt of the accused. Again, the circumstances should be of a
conclusive nature and tendency and they should be such as to exclude every hypothesis
but the one proposed to be proved. In other words, there must be a chain of evidence so
far complete as not to leave any reasonable ground for a conclusion consistent with the
innocence of the accused and it must be such as to show that within all human probability
the act must have been done by the accused."
21. A reference may be made to a later decision in Sharad Birdhichand Sarda v. State of
Maharashtra [AIR 1984 SC 1622]. Therein, while dealing with circumstantial evidence, it
has been held that the onus was on the prosecution to prove that the chain is complete and
the infirmity of lacuna in prosecution cannot be cured by false defence
@page-SC1541
or plea. The conditions precedent in the words of this Court, before conviction could be
based on circumstantial evidence, must be fully established. They are: (SCC p. 185. para
153)
(1) the circumstances from which the conclusion of guilt is to be drawn should be fully
established. The circumstances concerned must or should and not may be established;
(2) the facts so established should be consistent only with the hypothesis of the guilt of
the accused, that is to say, they should not be explainable on any other hypothesis except
that the accused is guilty;
(3) the circumstances should be of a conclusive nature and tendency;
(4) they should exclude every possible hypothesis except the one to be proved; and
(5) there must be a chain of evidence so complete as not to leave any reasonable ground
for the conclusion consistent with the innocence of the accused and must show that in all
human probability the act must have been done by the accused.
22

. The above position was highlighted in State of Rajasthan v. Raja Ram [2003 (8) SCC
180]. 2003 AIR SCW 4097

23. It has been rightly noted by the trial court and the High Court that the accused persons
were absolutely silent and no explanation was offered as to how the body came to be
buried in their hut which was in their exclusive user.
24. Similarly the non-explanation of this vital circumstance adds to the chain of
circumstances. It is now settled law that if the deceased was in the custody or in the
company of the accused, then the accused must supply some explanation regarding the
disappearance of the deceased.
25. In the factual background, it is considered in the light of the decisions referred to
above, the inevitable conclusion is that the appeals are sans merit, deserve dismissal
which we direct. We record our appreciation for the able manner in which Mr. Y.P. Singh,
learned Amicus Curiae assisted the case.
Appealsdismissed
AIR 2008 SUPREME COURT 1541 "Thiruvengada Pillai v. Navaneethammal"
(From : Madras)
Coram : 2 R. V. RAVEENDRAN AND P. SATHASIVAM, JJ.
Civil Appeal No. 290 of 2001, D/- 19 -2 -2008.
Thiruvengada Pillai v. Navaneethammal and Anr.
(A) Stamp Act (2 of 1899), S.54 - STAMP - EXECUTION - DOCUMENTS - Execution
of document - Use of Stamp Paper - No expiry date - S.54 does not require purchaser of
Stamp Paper should use it within six months.
Stamp Act, 1899 nowhere prescribes any expiry date for use of a stamp paper. S. 54
merely provides that a person possessing a stamp paper for which he has no immediate
use (which is not spoiled or rendered unfit or useless), can seek refund of the value
thereof by surrendering such stamp paper to the Collector provided it was purchased
within the period of six months next preceding the date on which it was so surrendered.
The stipulation of the period of six months prescribed in S. 54 is only for the purpose of
seeking refund of the value of the unused stamp paper, and not for use of the stamp paper.
S. 54 does not require the person who has purchased a stamp paper, to use it within six
months. Therefore, there is no impediment for a stamp paper purchased more than six
months prior to the proposed date of execution, being used for a document. (Para
11)
(B) Stamp Act (2 of 1899), S.35, S.37 - Stamp Rules (1925), R.2 - STAMP -
EXECUTION - DOCUMENTS - Execution of document - Instrument written on two
Stamp papers purchased by same person on different dates - Not invalid.
The Stamp Act is a fiscal enactment intended to secure revenue for the State. In the
absence of any Rule requiring consecutively numbered stamp papers purchased on the
same day, being used for an instrument which is not intended to be registered, a
document cannot be termed as invalid merely because it is written on two stamp papers
purchased by the same person on different dates. Even assuming that use of such stamp
papers is an irregularity, the Court can only deem the document to be not properly
stamped, but cannot, only on that ground, hold the document to be invalid. Even if an
agreement is not executed on requisite stamp paper, it is admissible in
@page-SC1542
evidence on payment of duty and penalty under Section 35 or 37 of the Stamp Act, 1899.
If an agreement executed on a plain paper could be admitted in evidence by paying duty
and penalty, there is no reason why an agreement executed on two stamp papers, even
assuming that they were defective, cannot be accepted on payment of duty and penalty.
But admissibility of a document into evidence and proof of genuineness of such
document are different issues.(Para 12)
(C) Evidence Act (1 of 1872), S.45 - EVIDENCE - Expert evidence - Thumb impression
on documents - Dispute as to - Recording finding about its authenticity by Court without
benefit of any expert opinion, merely on casual perusal - Illegal.
When there is a positive denial by the person who is said to have affixed his finger
impression and where the finger impression in the disputed document is vague or smudgy
or not clear, making it difficult for comparison, the Court should hesitate to venture a
decision based on its own comparison of the disputed and admitted finger impressions.
(Para 15)
In the instant case the first defendant had denied having put her finger impression on
agreement of sale. She died during the pendency of the suit before her turn came for
giving evidence. The document though dated 1980, was executed on two stamp papers
which were purchased in 1973 and 1978. Contrary to the recital in the agreement that
possession had been delivered to the plaintiff, the possession was not in fact delivered to
plaintiff, but continued with the first defendant and she delivered the possession to the
second defendant. The title deeds were not delivered to plaintiff. The attesting witnesses
were close relatives of plaintiff and one of them was not examined. The scribe's evidence
was unsatisfactory. It was also difficult to believe that the first defendant, an illiterate old
woman from a village, would enter into an agreement of sale on 5-1-1980 with plaintiff,
and even when he is ready to complete the sale, sell the property to someone else hardly a
month thereafter, on 11-2-1980. In this background, the finding by the first appellate
Court, recorded without the benefit of any expert opinion, merely on a casual perusal,
that there appeared to be no marked differences between the two thumb impressions, and
therefore, sale agreement must have been executed by first defendant, was unsound.
(Para 16)
(D) Specific Relief Act (47 of 1963), S.20 - CONTRACT - AGREEMENT TO SELL -
DISMISSAL - Agreement of sale - Specific performance - Plaintiffs alleging that first
defendant executed agreement of sale in their favour - Denied by first defendant - Burden
to prove that first defendant had executed agreement would be on plaintiff and not on first
defendant to prove the negative - Various circumstances when taken together, creating
doubt about the genuineness of agreement and dislodge effect of evidence adduced by
plaintiff - Dismissal of suit - Was proper. (Para 17)
Cases Referred : Chronological Paras
2003 AIR SCW 1238 : AIR 2003 SC 1795 : 2003 Lab IC 1170 (Ref.) 14.2, 15
1997 AIR SCW 3315 : AIR 1997 SC 3255 : 1997 Cri LJ 3964 (Ref.) 14.1
1996 AIR SCW 685 : AIR 1996 SC 1140 (Ref.) 14.1
AIR 1980 SC 531 : 1980 Cri LJ 396 (Ref.) 14.2, 15
AIR 1979 SC 14 : 1979 Cri LJ 17 (Ref.) 14.1
Ms. Sunita Sharma, for Appellant; K.K. Mani, C.K.R. Lenin Sekar and Mayur R. Shah,
for Respondents.
Judgement
R. V. RAVEENDRAN, J. :- This appeal by special leave is by the plaintiff in a suit for
specific performance OS No. 290/1980 on the file of District Munsiff, Tindivanam.
Pleadings
2. In the plaint, the plaintiff (appellant) alleged that the first defendant (Adilakshmi)
agreed to sell the suit schedule property to him under an agreement of sale dated 5-1-
1980 for a consideration of Rs. 3,000/-, and received Rs. 2,000/- as advance. She agreed
to execute a sale deed by receiving the balance consideration of Rs. 1,000/- within three
months. Possession of the suit property was delivered to him, under the said agreement.
He issued a notice dated 14-2-1980 calling upon the first defendant to receive the balance
price and execute the sale deed. The first defendant sent a reply denying the agreement.
To avoid performing the agreement of sale, the first defendant executed a nominal sale
deed in regard to the suit property in favour of the second defendant (first respondent
herein), who was her
@page-SC1543
close relative. The said sale was neither valid nor binding on him. On the said averments,
he sought specific performance of the agreement of sale, against the defendant, alleging
that he was ready and willing to perform his part of the contract.
3. The defendants denied the allegation that the first defendant had executed an
agreement of sale dated 5-1-1980 in favour of the plaintiff or that she had delivered
possession of the suit property to him. They contended that plaintiff had concocted and
forged the document with the help of his henchmen to defraud the defendants. They
claimed that the first defendant had executed a valid sale deed dated 11-2-1980 in favour
of the second defendant and had delivered possession of the suit property to her; and that
the second defendant had put up a hut in the schedule property and was actually residing
therein. The second defendant raised an additional contention that she was a bona fide
purchaser for value and therefore, the sale in her favour was valid.
4. During the pendency of the suit first defendant died, and the third defendant (second
respondent herein) was impleaded as her legal representative, who adopted the written
statement of the second defendant.
Issues and the Judgment
5. On the said pleadings, three issues were framed by the trial Court : (i) whether the
agreement put forth by the plaintiff was true or concocted ? (ii) whether the second
defendant had purchased the suit property for valid consideration? and (iii) whether the
plaintiff was entitled to the relief of specific performance? The plaintiff examined himself
as PW-1 and the scribe of the agreement (Ramaswami Pillai) as PW-2 and an attesting
witness to the sale agreement (Venkatesha Pillai) as PW-3. The agreement of sale was
exhibited as Ex. A-1. The notice and reply were marked as Ex. A2 and A4. The second
defendant, (purchaser of the site), gave evidence as DW-1 and the third defendant, who
was also a witness to the sale deed dated 11-2-1980, was examined as DW-2. The sale
deed dated 11-2-1980 executed by first defendant in favour of second defendant was
marked as Ex. B2 and previous title deed was exhibited as Ex. B4. The plaintiff and his
witnesses gave evidence that the sale agreement was duly executed by first defendant in
favour of plaintiff. The defendants gave evidence about the sale in favour of second
defendant and denied execution of any agreement of sale in favour of plaintiff.
6. The trial Court after appreciating the evidence, dismissed the suit by judgment and
decree dated 28-2-1984. It held that the agreement of sale put forth by plaintiff was false
and must have been created after the sale on 11-2-1980 in favour of second defendant, by
using some old stamp papers in his possession. The said finding was based on the
following facts and circumstances :
(a) The sale agreement (A-1) was not executed on currently purchased stamp paper, but
was written on two stamp papers, one purchased on 25-8-1973 in the name of
Thiruvengadam and another purchased on 7-8-1978 in the name of Thiruvengadam Pillai.
(b) The two attestors to the agreement were close relatives of plaintiff. One of them was
Kannan, brother of the plaintiff and he was not examined. The other was Venkatesa Pillai,
uncle of plaintiff examined as PW3. The scribe (PW-2) was a caste- man of plaintiff.
Their evidence was not trustworthy.
(c) Though the agreement of sale recited that the possession of the suit property was
delivered to plaintiff, no such possession was delivered. On the other hand, the second
defendant was put in possession on execution of the sale deed and she put up a thatched
hut in the schedule property and was in actual physical possession. This falsified the
agreement.
(d) If really there was an agreement of sale, in the normal course, the plaintiff would have
obtained the title deeds from the first defendant. But the earlier title deeds were not
delivered to him. On the other hand, they were delivered to the second defendant who
produced them as Ex. B3 and Ex. B4.
(e) In spite of defendants denying the agreement (Ex. A1), the plaintiff failed to discharge
his onus to prove that execution of the agreement as he did not seek reference to a
fingerprint expert to establish that the thumb impression on the agreement was that of the
first defendant.
The first and second appeals
7. Feeling aggrieved, the plaintiff filed an appeal before the Sub-Court, Tindivanam. The
first appellate Court allowed the plaintiff's appeal by judgment dated 12-1-1987, held that
the agreement of sale was proved and decreed the suit granting specific
@page-SC1544
performance. The following reasons were given by the first appellate Court in support of
its finding :
(a) The evidence of PW1 (plaintiff), the scribe (PW-2) and the attestor (PW3) proved the
due execution of the agreement by the first defendant. As the scribe (PW2) was not
related to plaintiff and as PW3 was not a close relative of plaintiff, their evidence could
not have been rejected.
(b) The burden of proving that the agreement of sale was concocted and forged was on
the defendants and they ought to have taken steps to have the document examined by a
Finger Print expert, to establish that the disputed thumb mark in the agreement of sale
(Ex. A1), was different from the admitted thumb mark of the first defendant in the sale
deed (Ex. B2). They failed to do so.
(c) There appeared to be no marked difference between the finger impression in the
agreement of sale (Ex. A1) and the finger impression in the sale deed in favour of the
second defendant (Ex. B2), on a perusal of the said two documents. Therefore, it could be
inferred that first defendant had executed the agreement.
(d) Execution of the agreement of sale on two stamp papers purchased on different dates,
did not invalidate the agreement.
8. Being aggrieved, the second defendant filed a second appeal. The High Court allowed
the second appeal and dismissed the suit, by judgment dated 17-2-1999. The High Court
while restoring the decision of the trial Court held that the agreement of sale was not
genuine for the following reasons.
(i) The first appellate Court had placed the onus wrongly on the defendants to prove the
negative. As the first defendant denied execution of the agreement, the burden of
establishing the execution of document, was on the plaintiff. The plaintiff had failed to
establish by acceptable evidence that Ex. A-1 was a true and valid agreement of sale. The
evidence, examined as a whole, threw considerable doubt as to whether it was truly and
validly executed.
(ii) A perusal of the agreement (Ex. A1) showed that the thumb impression was very pale
and not clear. The first appellate Court could not by a casual comparison of the disputed
thumb impression in the agreement with the admitted thumb impression in the sale deed,
record a finding that there were no marked differences in the thumb impressions in the
two documents (Ex. A1 and Ex. B2). In the absence of an expert's opinion that the thumb
impression on the agreement of the sale was that of the first defendant, the first appellate
Court ought not to have concluded that the agreement of sale was executed by the first
defendant.
(iii) In the normal course, an agreement would be executed on stamp papers purchased
immediately prior to the execution of the agreement. The fact that the agreement was
written on two stamp papers bearing the dates 25-8-1973 and 7-8-1978 purchased in two
different names showed that it was not genuine, but was anti dated and forged.
(iv) The attesting witnesses to the agreement of sale were close relatives of plaintiff.
Their evidence was not trustworthy.
Points for consideration
9. The said judgment of the High Court is challenged in this appeal by special leave. The
appellant contended that having regard to the provisions of Evidence Act, 1872, there was
nothing improper in the first appellate Court comparing the disputed thumb impression in
Ex. A-1 with the admitted thumb impression of first defendant in Ex. B-2; and the finding
of the first appellate Court on such comparison, that there were no marked differences
between the two thumb impressions, being a finding of fact, was not open to interference
in second appeal. It was next contended that the execution of the agreement of sale was
duly proved by the evidence of plaintiff (PW1). the scribe (PW-2) and one of the attesting
witnesses (PW3). It was pointed out there was no evidence to rebut the evidence of PW1,
PW2 and PW3 regarding due execution as first defendant died without giving evidence,
and as the defendants did not seek reference to a finger print expert to prove that the
thumb impression on the agreement of sale was not that of first defendant. It was
submitted that an agreement cannot be doubted or invalidated merely on account of the
fact that the two stamp papers used for the agreement were purchased on different dates.
The Appellant therefore submitted that the sale agreement was duly proved.
10. On the contentions urged, the following questions arise for consideration :
(i) Whether the agreement of sale executed on two stamp papers purchased on
@page-SC1545
different dates and more than six months prior to date of execution is not valid?
(ii) Whether the first appellate Court was justified in comparing the disputed thumb
impression with the admitted thumb impression and recording a finding about the
authenticity of the thumb impression, without the benefit of any opinion of an expert?
(iii) Whether the High Court erred in reversing the judgment of the first appellate Court
in second appeal?
Re : Question (i)
11. The Trial Court and the High Court have doubted the genuineness of the agreement
dated 5-1-1980 because it was written on two stamp papers purchased on 25-8-1973 and
7-8-1978. The learned counsel for first respondent submitted that apart from raising a
doubt about the authenticity of the document, the use of such old stamp papers
invalidated the agreement itself for two reasons. Firstly, it was illegal to use stamp papers
purchased on different dates for execution of a document. Secondly, as the stamp papers
used in the agreement of sale were more than six months old, they were not valid stamp
papers and consequently, the agreement prepared on such 'expired' papers was also not
valid. We will deal with the second contention first. The Indian Stamp Act, 1899 nowhere
prescribes any expiry date for use of a stamp paper. Section 54 merely provides that a
person possessing a stamp paper for which he has no immediate use (which is not spoiled
or rendered unfit or useless), can seek refund of the value thereof by surrendering such
stamp paper to the Collector provided it was purchased within the period of six months
next preceding the date on which it was so surrendered. The stipulation of the period of
six months prescribed in section 54 is only for the purpose of seeking refund of the value
of the unused stamp paper, and not for use of the stamp paper. Section 54 does not require
the person who has purchased a stamp paper, to use it within six months. Therefore, there
is no impediment for a stamp paper purchased more than six months prior to the proposed
date of execution, being used for a document.
12. The Stamp Rules in many States provide that when a person wants to purchase stamp
papers of a specified value and a single stamp paper of such value is not available, the
stamp vendor can supply appropriate number of stamp papers required to make up the
specified value; and that when more than one stamp paper is issued in regard to a single
transaction, the stamp vendor is required to give consecutive numbers. In some States, the
rules further require an endorsement by the stamp vendor on the stamp paper certifying
that a single sheet of required value was not available and therefore more than one sheet
(specifying the number of sheets) have been issued to make up the requisite stamp value.
But the Indian Stamp Rules, 1925 applicable to Tamil Nadu, do not contain any provision
that the stamp papers of required value should be purchased together from the same
vendor with consecutive serial numbers. The Rules merely provide that where two or
more sheets of paper on which stamps are engraved or embossed are used to make up the
amount of duty chargeable in respect of any instrument, a portion of such instrument shall
be written on each sheet so used. No other Rule was brought to our notice which required
use of consecutively numbered stamp papers in the State of Tamil Nadu. The Stamp Act
is a fiscal enactment intended to secure revenue for the State. In the absence of any Rule
requiring consecutively numbered stamp papers purchased on the same day, being used
for an instrument which is not intended to be registered, a document cannot be termed as
invalid merely because it is written on two stamp papers purchased by the same person
on different dates. Even assuming that use of such stamp papers is an irregularity, the
Court can only deem the document to be not properly stamped, but cannot, only on that
ground, hold the document to be invalid. Even if an agreement is not executed on
requisite stamp paper, it is admissible in evidence on payment of duty and penalty under
Section 35 or 37 of the Indian Stamp Act, 1899. If an agreement executed on a plain
paper could be admitted in evidence by paying duty and penalty, there is no reason why
an agreement executed on two stamp papers, even assuming that they were defective,
cannot be accepted on payment of duty and penalty. But admissibility of a document into
evidence and proof of genuineness of such document are different issues.
13. If a person wants to create a back dated agreement, the first hurdle he faces is the
non-availability of stamp paper of such
@page-SC1546
old date. Therefore tampering of the date of issue and seal affixed by the stamp vendor, as
also the entries made by the stamp vendor, are quite common in a forged document.
When the agreement is dated 5-1-1980, and the stamp papers used are purchased in the
years 1973 and 1978, one of the possible inferences is that the plaintiff not being able to
secure an anti-dated stamp paper for creating the agreement (bearing a date prior to the
date of sale in favour of second defendant), made use of some old stamp papers that were
available with him, to fabricate the document. The fact that very old stamp papers of
different dates have been used, may certainly be a circumstance that can be used as a
piece of evidence to cast doubt on the authenticity of the agreement. But that cannot be a
clinching evidence. There is also a possibility that a lay man unfamiliar with legal
provisions relating to stamps, may bona fide think that he could use the old unused stamp
papers lying with him for preparation of the document and accordingly use the old stamp
papers.
Re : Point No. (ii)
14. Section 45 of the Indian Evidence Act, 1872 relates to 'opinion of experts'. It provides
inter alia that when the Court has to form an opinion as to identity of handwriting or
finger Impressions, the opinion upon that point of persons specially skilled in questions
as to identity or handwriting or finger impressions are relevant facts. Section 73 provides
that in order to ascertain whether a finger impression is that of the person by whom it
purports to have been made, any finger impression admitted to have been made by that
person, may be compared with the one which is to be proved. These provisions have been
the subject matter of several decisions of this Court.
14.1

. In The State (Delhi Administration) v. Pali Ram (1979 (2) SCC 158) this Court held that
a Court does not exceed its power under Section 73 if it compares the disputed writing
with the admitted writing of the party so as to reach its own conclusion. But this Court
cautioned : AIR 1979 SC 14, (Para 29)

"Although there is no legal bar to the Judge using his own eyes to compare the disputed
writing with the admitted writing, even without the aid of the evidence of any
handwriting expert, the Judge should, as a matter of prudence and caution, hesitate to
base his finding with regard to the identity of a handwriting which forms the sheet-anchor
of the prosecution case against a person accused of an offence, solely on comparison
made by himself. It is therefore, not advisable that a Judge should take upon himself the
task of comparing the admitted writing with the disputed one to find out whether the two
agree with each other; and the prudent course is to be obtain the opinion and assistance of
an expert."

The caution was reiterated in O. Bharathan v. K. Sudhakaran, 1996 (2) SCC 704. Again
in Ajit Savant Majagvai v. State of Karnataka (1997 (7) SCC 110) referring to Section 73
of the Evidence Act, this Court held : 1996 AIR SCW 685
1997 AIR SCW 3315

"The section does not specify by whom the comparison shall be made. However, looking
to the other provisions of the Act, it is clear that such comparison may either be made by
a handwriting expert under Section 45 or by anyone familiar with the handwriting of the
person concerned as provided by Section 47 or by the Court itself.
As a matter of extreme caution and judicial sobriety, the Court should not normally take
upon itself the responsibility of comparing the disputed signature with that of the
admitted signature or handwriting and in the event of the slightest doubt, leave the matter
to the wisdom of experts. But this does not mean that the Court has not the power to
compare the disputed signature with the admitted signature as this power is clearly
available under Section 73 of the Act."
14.2

. In Murari Lal v. State of Madhya Pradesh 1980 (1) SCC 704, this Court indicated the
circumstances in which the Court may itself compare disputed and admitted writings,
thus : AIR 1980 SC 531
(Para 12)

"The argument that the Court should not venture to compare writing itself, as it would
thereby assume to itself the role of an expert is entirely without force. Section 73 of the
Evidence Act expressly enables the Court to compare disputed writings with admitted or
proved writings to ascertain whether a writing is that of the person by whom it purports to
have been written. If it is hazardous to do so, as sometimes said, we are afraid it is one of
the hazards to which Judge and litigant must expose themselves whenever 2003 AIR
SCW 1238

@page-SC1547
ever it becomes necessary. There may be cases where both sides call experts and the
voices of science are heard. There may be cases where neither side calls an expert, being
ill able to afford him. In all such cases, it becomes the plain duty of the Court to compare
the writings and come to its own conclusions. The duty cannot be avoided by recourse to
the statement that the Court is no expert. Where there are expert opinions, they will aid
the Court. Where there is none, the Court will have to seek guidance from some
authoritative textbook and the Court's own experience and knowledge. But discharge it
must, its plain duty, with or without expert, with or without other evidence." The decision
in Murari Lal (supra) was followed in Lalit Popli v. Canara Bank and Ors. (2003 (3) SCC
583).
15. While there is no doubt that Court can compare the disputed
handwriting/signature/finger impression with the admitted handwriting/signature/finger
impression, such comparison by Court without the assistance of any expert, has always
been considered to be hazardous and risky. When it is said that there is no bar to a Court
to compare the disputed finger impression with the admitted finger impression, it goes
without saying that it can record an opinion or finding on such comparison, only after an
analysis of the characteristics of the admitted finger impression and after verifying
whether the same characteristics are found in the disputed finger impression. The
comparison of the two thumb impressions cannot be casual or by a mere glance. Further,
a finding in the judgment that there appeared to be no marked differences between the
admitted thumb impression and disputed thumb impression, without anything more,
cannot be accepted as a valid finding that the disputed signature is of the person who has
put the admitted thumb impression. Where the Court finds that the disputed finger
impression and admitted thumb impression are clear where the Court is in a position to
identify the characteristics of finger prints, the Court may record a finding on
comparison, even in the absence of an expert's opinion. But where the disputed thumb
impression is smudgy, vague or very light, the Court should not hazard a guess by a
casual perusal. The decision in Muralilal (supra) and Lalit Popli (supra) should not be
construed as laying a proposition that the court is bound to compare the disputed and
admitted finger impressions and record a finding thereon, irrespective of the condition of
the disputed finger impression. When there is a positive denial by the person who is said
to have affixed his finger impression and where the finger impression in the disputed
document is vague or smudgy or not clear, making it difficult for comparison, the Court
should hesitate to venture a decision based on its own comparison of the disputed and
admitted finger impressions. Further even in cases where the Court is constrained to take
up such comparison, it should make a thorough study, if necessary with the assistance of
counsel, to ascertain the characteristics, similarities and dissimilarities. Necessarily, the
judgment should contain the reasons for any conclusion based on comparison of the
thumb impression, if it chooses to record a finding thereon. The Court should avoid
reaching conclusions based on a mere casual or routine glance, or perusal.
16. In this case the first defendant had denied having put her finger impression on Ex. A-
1. She died during the pendency of the suit before her turn came for giving evidence. The
High Court having examined the document has clearly recorded the finding that the
thumb mark in Ex. A-1 was pale (that is light) and not clear. The document though dated
1980, was executed on two stamp papers which were purchased in 1973 and 1978.
Contrary to the recital in the agreement that possession had been delivered to the plaintiff,
the possession was not in fact delivered to plaintiff, but continued with the first defendant
and she delivered the possession to the second defendant. The title deeds were not
delivered to plaintiff. The attesting witnesses were close relatives of plaintiff and one of
them was not examined. The scribe's evidence was unsatisfactory. It was also difficult to
believe that the first defendant, an illiterate old woman from a village, would enter into
an agreement of sale on 5-1-1980 with plaintiff, and even when he is ready to complete
the sale, sell the property to someone else hardly a month thereafter, on 11-2-1980. In this
background, the finding by the first appellate Court, recorded without the benefit of any
expert opinion, merely on a casual perusal, that there appeared to be no marked
differences between the two thumb impressions, and therefore Ex. A-1 (sale agreement)
must have been executed by first defendant, was
@page-SC1548
unsound. The High Court was justified in interfering, with the finding of the first
appellate Court that the Ex. A1 was executed by first defendant.
Re : Point No. (iii)
17. The trial Court had analyzed the evidence properly and had dismissed the suit by
giving cogent reasons. The first appellate Court reversed it by wrongly placing onus on
the defendants. Its observation that when the execution of an unregistered document put
forth by the plaintiff was denied by the defendants, it was for the defendants to establish
that the document was forged or concocted, is not sound proposition. The first appellate
Court proceeded on the basis that it is for the party who asserts something to prove that
thing; and as the defendants alleged that the agreement was forged, it was for them to
prove it. But the first appellate Court lost sight of the fact that the party who propounds
the document will have to prove it. In this case plaintiffs came to Court alleging that the
first defendant had executed an agreement of sale in favour. The first defendant having
denied it, the burden was on the plaintiff to prove that the first defendant had executed the
agreement and not on the first defendant to prove the negative. The issues also placed the
burden on the plaintiff to prove the document to be true. No doubt, the plaintiff attempted
to discharge his burden by examining himself as also scribe and one of the attesting
witnesses. But the various circumstances enumerated by the trial Court and High Court
referred to earlier, when taken together, rightly create a doubt about the genuineness of
the agreement and dislodge the effect of the evidence of PW 1 to 3. We are therefore of
the view that the decision of the High Court, reversing the decision of the first appellate
Court, does not call for interference.
18. We, therefore, find no merit in this appeal and the same is accordingly dismissed.
Parties to bear their respective costs.
Appeal dismissed.
AIR 2008 SUPREME COURT 1548 "N. Srihari v. N. Prakash"
(From : 2005 (4) Andh LD 370)
Coram : 2 Dr. A. PASAYAT AND P. SATHASIVAM, JJ.
Civil Appeal Nos. 1420-1421 with 1422-1429 of 2008 (arising out of SLP (C) Nos.
17808-17809 of 2005 with 18481 of 2007 and etc.), D/- 19 -2 -2008.
N. Srihari (D) by L.Rs. and Ors. v. N. Prakash and Ors.
Civil P.C. (5 of 1908), S.96 - APPEAL - PARTITION - WILL - EXECUTION - Appeal -
Against suit for partition - Claim based on Will executed by plaintiff's grandfather - Only
certified copy of Will adduced by plaintiff - Defendants also referring to Will in their
written statement - Neither party however brought original Will on record - Trial Court
and High Court without giving any specific finding on Will decreeing suit - Matter liable
to be remitted to High Court for fresh consideration. (Para 14)

Harish N. Salve, L.N. Rao, P.S. Mishra, Mukul Rohtagi, Dushyant A. Dave,
Vishwanathan Shetty, T.L. Vishwanatha Iyer, Sr. Advocate, Suyodhan Byrapaneni,
Siddharth Patnaik, G. Arun, Govind Reddy, G. Ramakrishna Prasad, D.V. Shiva Prasad,
Pramod Dayal, Nikunj Dayal, Tathagat H. Vardhan, Dhruv Kumar Jha, Ravi C. Prakash,
O.P. Singh, Upendra Mishra, Manu Shanker Mishra, K. Raghavacharayulu, Sridhar
Potaraju, D. Julius Riamei, S. Udaya Kumar Sagar, Ms. Bina Madhavan, B.D.L. Nirmal
Kumar, Ms. Nupur Kanungo, Hemal K. Sheth, M/s. Lawyer's Knit and Co., Aribam
Guneshwar Sharma, for appearing parties.
Judgement
1. P. SATHASIVAM, J. :- Leave granted.
2. The appellants herein are the unsuccessful defendants in O.S. No. 9 of 1993 on the file
of the trial Court as well as the High Court. The LRs of the deceased parties as well as the
purchasers and third parties who were not parties before the trial Court and the High
Court also filed appeals. The respective claim/stand of the parties is being explained
hereunder. In order to understand their claim, entitlement etc., let us refer to the
geneology table of the family of N. Saya Goud.
@page-SC1549

3. One Shri N. Saya Goud had a wife by name, Smt. Chandramma and two sons, namely.
Balarajiah Goud and Sathaiah Goud. Balarajiah Goud had two wives, Pentamma (first
wife) and Kausalya (second wife), five sons and three daughters through his first wife
and the second wife was issueless. Sathaiah Goud had a wife, Sulochana and six sons. On
2.1.1956, Shri Saya Goud executed a Will under which he mentioned that the lands
bearing Survey Nos. 284, 285, 290, 292 and 293 admeasuring 19 acres and 15 guntas
situated in Lothukanta, Alwal, Ranga Reddy District were in his protected tenancy and
that the other movable properties mentioned therein, were acquired by himself and his
wife Chandramma and bequeathed all the movable and immovable properties jointly held
by him and his wife in favour of his wife, Smt. Chandramma and his eldest daughter-in-
law Pentamma. The beneficiaries of the Will were to enjoy the properties jointly. Smt.
Chandramma was given life time interest under the said Will. Sathaiah Goud, second son
of Shri Saya Goud, was an attestator of the Will dated 2.1.1956 executed by his father.
After the demise of Shri Saya Goud, the pattadars of the land for which Shri Saya Goud
acquired the right of protected tenancy had transferred their pattadars right and interest in
favour of Smt. Chandramma and Smt. Pentamma jointly as the heirs of Shri Saya Goud
upon payment of the required consideration in respect of the land of an extent of 19 acres
15 guntas and consequently the Deputy District Collector passed an award dated
17.4.1956 in favour of Smt. Chandramma and Smt. Pentamma as per his award No.
T/85/1954. Thereafter in 1959, Smt. Chandramma and Smt. Pentamma jointly purchased
an extent of 1 acre 30 guntas of land, therefore, their joint holding had risen to 21 acres
05 guntas. On 6.3.1969, one Registered Settlement Deed was executed by Smt.
Chandramma transferring an extent of 2982 sq. yards of land from the joint holding in
favour of Smt. Sulochana and one Registered Release Deed transferring her undivided
share in favour of Pentamma. On the same day, Smt. Sulochana executed a registered
Disclaimer Deed claiming any right over the property that is vested with Smt. Pentamma
and Smt. Chandramma. In the year 1970, the second son of Shri Balarajiah Goud and
Smt. Pentamma pledged the documents concerning the houses and the land in an extent
of 21 acres with Andhra Bank and obtained loan for business purposes. In failure of
payment
@page-SC1550
of outstanding dues of the loan amount, the Andhra Bank filed O.S. No. 403 of 1976 for
recovery of the amount and thereby obtained a decree in the year 1977 wherein both the
registered Release Deed executed on 6.3.1969 and the Will of Shri Saya Goud dated
2.1.1956 were marked as exhibits. In the year 1977, eldest son of Smt. Pentamma, Shri N.
Srihari, filed a suit for partition against the other defendants/petitioners herein and Smt.
Chandramma. In the year 1981, the suit for partition was compromised. Shri Balarajiah
Goud expired on 24.5.1981.
4. After the demise of Smt. Chandramma, i.e. on 23.4.1984, the sons of Shri Sathaiah
Goud claimed the entire share of Smt. Chandramma through a Will dated 28.9.1979
purported to have been executed in their favour which is supposed to have been found in
a box. They filed O.S. No. 456 of 1984 on the file of the Principal Sub-Judge, Ranga
Reddy District. In the year 1993, the said suit was transferred to the District Judge, Ranga
Reddy District and renumbered as O.S. No.9 of 1993. By judgment and decree dated
8.9.1993, the District Judge allowed the suit in favour of the plaintiffs/respondents herein
and passed a decree in their favour. Aggrieved by the judgment and decree passed by the
learned District Judge, the defendants/appellants herein preferred an appeal bearing No.
78 of 1994 and CMP No. 17581 of 2001 before the High Court of Judicature Andhra
Pradesh at Hyderabad. By the impugned judgment dated 17.2.2005, the learned single
Judge of the High Court dismissed the appeal and ordered the Civil Miscellaneous
Petition.
5. Aggrieved by the judgment in A. S. No. 78 of 1994, N. Srihari (since deceased), N.
Laxminarayana and N. Dayanand (defendant Nos. 2, 5 and 6) filed SLP (C) No. 17808 of
2005. The very same parties aggrieved by the decision in CMP No. 17581 of 2001
preferred SLP (C) No. 17809 of 2005. While ordering notice in the above SLPs, this
Court passed an interim order to the effect that "final decree proceedings may go on, but
the final decree as such shall not be signed unless permitted by this Court". Based on the
said interim order, when the final decree proceedings were in progress Ms. N. Anuradha,
N. Saivenkataramana and B. Sai Nagraj, defendant Nos. 18-20 filed I.A. No. 2017 of
2006 under Section 151, CPC and requested the trial Court not to proceed with the
enquiry in relation to the suit schedule property on the ground that the "Occupancy
Rights Certificate" was issued in favour of the first defendant in respect of the entire suit
schedule property and that the land covered by the provisions of Andhra Pradesh
(Telangana) Area Abolition of Enams Act, 1955 cannot constitute the subject-matter of a
partition suit. The trial Court dismissed the application by order dated 13.07.2006.
Questioning the said order, the petitioners filed CRP No. 3726 of 2006 before the High
Court. By order dated 30.08.2006, learned Single Judge of the High Court confirming the
order of the trial Court dismissed the revision. Against that order of the High Court, the
petitioners therein (defendant Nos. 18-20) filed SLP (C) No. 18481 of 2007.
6. Against the judgment dated 17.02.2005 in A.S. No. 72 of 1994 and CMP No. 17581 of
2001 Mr. Sai Venkataramana, Ms. N. Anuradha and B. Sai Nagraj (appellant Nos. 7, 6
and 8 in the High Court) preferred SLP (C) Nos. 24682 and 24683 of 2005.
7. Against the very same judgment dated 17.02.2005 in A.S. No. 78 of 1994 and CMP
No. 17581 of 2001. One Ramesh Chawla S/ o Manohar not a party before the High Court
has filed SLP (C) No. 26425-26426.
8. Assailing the order in A.S.M.P.14246 of 2004 in A.S. No. 78/1994 filed for impleading
him as respondent No. 23 in A.S. 78 of 1994 which was dismissed by the High Court on
17.02.2005 the said Ramesh Chawla a third party has filed SLP (C) No. 26429 of 2005.
9. Against the same judgment dated 17.02.2005 in A.A. No. 78 of 1994 in CMP No.
17581 of 2001 defendants 2, 4 to 6 and LRs of the third defendant filed SLP (C) No.
23029-23030 of 2007.
10. We heard Mr. Harish N. Salve, Mr. L.N. Rao, Mr. P.S. Mishra and Mr. Mukul
Rohtagi, learned senior counsel for the appellants and Mr. Dushyant A. Dave, Mr.
Vishwanathan Shetty and Mr. T. L. Vishwanatha Iyer, learned senior counsel for the
contesting respondents.
11. Mr. Harish N. Salve, learned senior counsel appearing for the main appellants, after
taking us through the pleadings of all the parties, the judgment of the trial Court
@page-SC1551
and the High Court contended that in spite of the fact that the plaintiffs themselves
referred to the earlier Will dated 02.01.1956 executed by Saya Goud, merely because the
original of the same was not placed before the Court by the contesting defendants, the
same was not considered, hence the decision arrived by the trial Court as well as the High
Court cannot be sustained. He very much commented the impugned judgment of the High
Court which, according to him, failed to take note of the earliest Will dated 02.01.1956.
While elaborating the above point, he submitted that out of 92 page judgment, the High
Court referred to pleadings of the parties and arguments up to page 85 and from page 86-
92 alone discussed the Will (Exh.A-1) dated 28.09.1979 and Release Deed (Exh.A-7)
dated 06.03.1969 and dismissed the appeal and confirmed the judgment and decree of the
trial Court. In the same order, the High Court has also disposed of CMP No. 17581 of
2001 by marking original of Exh.A-7 as Exh.B-64. We heard the submissions of other
counsel.
12. In the light of the submissions made, we have gone through the judgments of the trial
Court, High Court as well as the material documents, namely, (i) Will dated 02.01.1956
(ii) Release Deed (Exh.A-7) dated 06.03.1969 (iii) Settlement Deed (Exh.B-6) dated
06.03.1969 and (iv) Will (Exh.A-1) dated 28.09.1979. As rightly pointed out by Mr.
Salve, though the contesting defendants were not in a position to place the original Will
dated 02.01.1956, admittedly, the plaintiffs have made a reference to the same Will in
their plaint. In O.S. No. 456 of 1984 filed by N. Prakash and 5 of his brothers against N.
Pentamma (Defendant No. 1) and her sons in more than one place referred the Will dated
02.01.1956. In para 4 it was submitted before the trial Court :
"4. During his lifetime, the late Nemuri Saya Goud executed a Will on 02.01.1956 under
which he mentioned that the lands bearing Survey Nos. 284, 285, 290, 292 and 293,
admeasuring 19 Acres and 15 Guntas, situate in Lothukanta, Alwal, Ranga Reddy District
were in his protected tenancy and that the other movable properties mentioned therein,
were acquired by himself and his wife Nemuri Chandramma by their joint exertions and
hard labour. Under the said Will, he bequeathed all the properties movable and
immovable jointly held by him and his wife, jointly to Nemuri Pentamma, the defendant
No. 1 herein and his wife Nemuri Chandramma. Nemuri Saya Goud died a few months
later and the immovable properties standing in the name of Nemuri Saya Goud came into
the joint possession and enjoyment of Nemuri Chandramma and Defendant No. 1 herein.
Subsequently, Nemuri Chandramma and the Defendant No. 1 purchased the rights of the
pattedar under the registered sale deed and thus they become the absolute owners of the
Agricultural Lands mentioned in the Will of late Nemuri Saya Goud. Later, they also
jointly purchased Agricultural Lands bearing Survey No. 291 and 602, situated at
plaintiffs are herewith filing a Geneological Tahsil showing the relationship of parties.
" Apart from the above, specific reference, the plaintiffs have also enclosed a copy of the
Will dated 02.01.1956 executed by N. Saya Goud along with list of documents filed
along with the plaint.

S. No. Date of Document Parties to Description of Document


1 to 6. Xxxx Xxxx Xxx
7. 02.01.1956 N. Saya Goud and Defendants Copy of Will executed by
N.Saya Goud

In the written statement filed by the 3rd defendant, there is a reference to the Will dated
02.01.1956 in para 5, which reads thus :
"5. xxx Therefore, Saya Goud executed a Will dated 02.01.1956. Under the said Will, he
intended to bequeath the said property to the wife of Balaraj Goud, Pentamma so that her
children (sons and the wife of Balaraj Goud) would benefit from his estate. He, however,
made specific mention in the effective part of the Will to the effect that during the
lifetime of Chandramma, Pentamma should look after the welfare of Chandramma and
that Pentamma should spend the income from the properties for the welfare and
maintenance of both, thereby he had safeguarded the well being and maintenance of
Chandramma for the rest of her lifetime by making Pentamma responsible for the same."
It is also useful to refer the stand taken in the written statement filed by the first
defendant. Paras 4 and 5 reads :
"4. As regards para 4, it is submitted that the late Nemuri Saya Goud bequeathed his
@page-SC1552
properties, movable and immovable held by him and his wife, Nemuri Chandramma,
jointly to the defendant No. 1 herein and his wife late Nemuri Chandramma. It is
submitted that Namuri Saya Goud died in the year 1956. After his death, all the properties
devolved upon his wife Chandramma and his daughter-in-law. Pentamma, who is the
defendant No. 1 herein and arms into the joint possession and enjoyment of Nemuri
Chandramma and the defendant No. 1 herein. Thus, Nemuri Chandramma and defendant
No. 1 herein become the absolute owners of the agricultural lands as mentioned in the
Will executed by late Saya Goud. Thereafter, the said Nemuri Chandramma and
defendant No. 1 herein purchased agricultural lands bearing S.No. 284, 285, 290, 291,
292, 293 and 602, situate at Lothucunta, Alwal, R.R. District. It is submitted that
thereafter the late Nemuri Chandramma executed a release deed in favour of the
defendant No. 1 herein. As such the defendant No.1 herein alone has been in continuous
possession and enjoyment of the agricultural lands as absolute owner.
5. xxx The Will executed by late Saya Goud clearly shows that late Nemuri Chandramma
had only life interest in the properties and thereafter all the rights in her favour have been
relinquished in favour of the defendant No. 1 herein."
13. While considering the claim of the parties, learned trial Judge has also adverted to the
Will dated 02.01.1956.
12. Since the common question of law and facts arise in deciding these three issues, these
three issues are being dealt together. For deciding these issues, it is necessary to advert to
some admitted facts and conditions of parties. The plaintiffs, defendants 2 to 6 and 15 to
17, are the grand children of Saya Goud. Plaintiffs are children of Sathaiah Goud,
Defendant No. 1 is the wife of Balarajaiah Goud, Defendants 2 to 6 and 15 to 17 are the
children of Balarajaiah Goud and the 1st defendants, 2 to 5 and 15 to 17, executed a Will
dated 2.01.1956 (the execution of the said Will by Saya Goud is not denied or disputed
but the contents of the Will are under dispute).
"13. Since the Will executed by Saya Goud is not brought on record by either of the
contesting parties, evidence available on record has to be considered to find the contents
of the Will of Saya Goud. In fact, plaintiffs filed a copy of the Will dated 02.01.1956,
executed by Saya Goud along with other documents as document No. 7 with the plaint.
But neither side brought that document on record as an exhibit. Neither side took steps to
send for the original of the Will, which as per the evidence of DW-1 is in the custody of
Andhra Bank."
The above pleadings as well as the discussion by the trial Court clearly show about the
existence of earliest Will dated 02.01.1956 executed by Saya Goud. The fact remains that
though the plaintiffs themselves placed a certified copy of the said Will, original of the
same has not been produced by the defendants. It is their case that the original had been
filed in a connected suit being O.S. No. 403 of 1976 filed by a Bank, Andhra Bank.
Though steps had been taken by the High Court for transfer of the said document, till its
final decision, the same was not reached and ultimately the High Court disposed of the
appeal on the basis of the available materials and confirmed the judgment and decree of
the trial Court.
14. In view of the fact that the plaintiff themselves referred to the Will dated 02.01.1956
in their plaint, asserted the same by the contesting defendant in their written statement
and specific reference to the same by the trial Court as well as the High Court, in the
absence of specific finding as to the Will dated 02.01.1956, we are of the considered view
that in the interest of justice, the matter has to go back to the High Court for fresh
consideration in respect of the earliest Will dated 02.01.1956.

15. In view of the above conclusion, without going into the merits of the claim made by
the parties and without expressing any opinion, we remit the matter to the High Court for
fresh disposal. We permit the appellants/contesting defendants to place the original Will
dated 02.01.1956 for consideration of the High Court. In case, if the original is not
available in view of the earlier proceedings, they are free to place the certified copy of the
Will dated 02.01.1956 and in that event, it is for the High Court to consider the same
including valid objections to be raised by the other parties in accordance with law.
Inasmuch as the appeal is of the year 1994, we request the High Court to dispose of the
same afresh in the light of the observations made above by giving priority not later than
30.09.2008. All the parties to
@page-SC1553
the proceedings including the subsequent purchasers are free to place their respective
stand and it is for the High Court to consider uninfluenced by any of the observations
made above. Till such final decision being taken by the High Court, parties are directed to
maintain status quo prevailing as on date. All the appeals are disposed of accordingly. No
costs.
Order accordingly.
AIR 2008 SUPREME COURT 1553 "Brij Narain Singh v. Adya Prasad"
(From : 2000 All LJ 2003)
Coram : 2 Dr. A. PASAYAT AND R. V. RAVEENDRAN, JJ.
Civil Appeal No. 5689 of 2000, D/- 18 -2 -2008.
Brij Narain Singh v. Adya Prasad (Dead) and Ors.
Civil P.C. (5 of 1908), S.11, O.41, R.33 - RES JUDICATA - APPEAL - Res judicata -
There were four suits - Plaintiffs in all four suits were common but defendants were not
common - Properties were situated in different villages - Decision in one of relevant suit
attained finality - Thus decision in appeals relating to other two suits, would not affect
decision in relevant suit - Would not operate as res judicata in subsequent proceedings as
parties were not same - Further provisions under O.41, R.33, would not be attracted as
emphasis thereunder is on same suit. (Paras 17, 18)
Cases Referred : Chronological Paras
AIR 1977 SC 789 (Ref.) 15
AIR 1974 SC 749 (Ref.) 14
AIR 1953 SC 419 (Dist.) 8, 10
AIR 1931 PC 114 16
AIR 1930 PC 22 12, 13
AIR 1927 Lahore 289 10
AIR 1916 PC 78 13, 14
17 Deccan LR 322 10
33 Deccan LR 603 10
Rameshwar Prasad Goyal, for Appellant; Shashindra Tripathi, Chitranjan Mishra, Ms.
Mridula Ray Bharadwaj, for Respondents.
Judgement
Dr. ARIJIT PASAYAT, J. :- Challenge in this appeal is to the order passed by a learned
Single Judge of the Allahabad High Court allowing the writ petition filed by the
respondents. The writ petitioners had questioned order dated 24-2-1973 passed by the
Assistant Settlement Officer, Consolidation, Jaunpur and the order dated 28-2-1978
passed by the Deputy Director, Consolidation Jaunpur who were the respondents 1 and 2
in the writ petition.
2. The factual position needs to be noted in brief as essentially the pivotal question relates
to the applicability of the principle of res judicata.
2.1 One Gajadhar owned several lands situate in the villages of Kurthuwa, Meerapur
Siroman, Manapur and Ghuskhuri, as fixed rate tenant, including the suit lands. The fixed
rate tenancy of the lands in those villages was mortgaged by Gajadhar. Gajadhar died
leaving behind him his widow Sirtaji, who through registered sale deed dated 8-6-1885
sold her right of redemption in regard to those lands to her relative Mata Badal.
2.2 On the death of Mata Badal, his wife Sheorani, sold the right of redemption in regard
to some of the lands to third parties. After the death of Sheorani, the nephews of Mata
Badal, namely Muneshwar, Bindeshwari and Bal Karan, sold the right of redemption in
respect of the suit properties in Kurthuwa in favour of Bhagwan Din Singh (grandfather
of appellant) under registered sale deed dated 19-6-1911. It would appear that after the
purchase of equity of redemption, the said Bhagwan Din Singh cleared mortgage and was
in possession of the suit lands. Bhagwan Din Singh died leaving him surviving his son
Bhagwati Din Singh (father of appellant - respondent No. 3 in the writ petition from
which this appeal arises).
2.3 Sirtaji who executed the sale deed on 8-6-1885 in favour of Mata Badal died in the
year 1940. On her death, Ganga Prasad and Bhagwati Din (ancestors of Respondents 1 to
6 herein) filed four suits 97 to 100 for partition before the SDC. Machhli Shahar, Jaunpur,
claiming that Gajadhar died issueless, that his wife Sirtaji had inherited only a life
interest in the lands of her husband Gajadhar in the four villages, and that on her death,
the lands of Gajadhar devolved on the near relatives of Gajadhar, namely plaintiffs 1 and
2 and Defendants 1 and 2 in the four suits, who were reversioners in regard to estate of
Gajadhar. Suits 97, 98, 99 and 100 respectively related to the lands in the villages of
Meerapur Siroman, Kurthuwa, Ghuskhuri and Manapur. Bhagwati Din Singh (father of
Appellant) was impleaded as Defendant No.3 in suit No.98,
@page-SC1554
as his father, Bhagwan Din Singh had purchased the right of redemption in respect of the
Kurthuwa lands.
2.4 The following genealogical tree accepted in the earlier proceedings, traces Gajadhar's
relationship with the plaintiffs (Ganga Prasad and Bhagwati Din Singh) and defendants 1
and 2 (Raj Narain and Chandra Bali), in the four suits as also with Mata Badal :

2.5 The four suits were decreed by Sri Ishwar Sahal, SDC Machhall Shahar, Jounpur, by
a common judgment dated 20-3-1944. He held that the sale by Sirtaji under deed dated 8-
6-1885 was not for legal necessity. Bhagwati Din Singh challenged the judgment in Suit
No.98. The first appellate court (Additional Commissioner, Varanasl) dismissed the
appeal (Appeal No.4/327) filed by Bhagwati Din Singh on 2-1-1945 on the ground of
delay. No further appeal was filed and the decision in Suit No.98 attained finality insofar
as Kurthuwa lands claimed by Bhagwati Din Singh. After dismissal of the appeal on 2-1-
1945, on an application by the plaintiffs in Suit No.98, a final decree was passed on 3-4-
1945 and possession was taken by plaintiffs in terms of the decree.
2.6 Two other appeals filed by the purchasers of lands at Ghuskhuri and Manapur
villages, against the common judgment dated 20-3-1944 in Suit Nos. 99 and 1000
travelled up to Board of Revenue and were remanded to the first appellate court. The said
two appeals arising out of suit Nos.99 and 100 were heard by Additional Commissioner,
Varanasi Division, He held that the sale deed dated 8-6-1885 executed by Smt. Sirtaji in
favour of Mata Badal was for legal necessity, that Mata Badal got valid title, and that the
sale deeds executed by Sheorani and others as legal heirs of Mata Badal were valid. He,
therefore, dismissed the two suits (Suit Nos. 99 and 100). That decision was upheld by
the Board of Revenue on 26-12-1967 and judgment which ended in dismissal of suit Nos.
99 and 100 also attained finality.
3. The resultant position was that there was two diverse decisions in regard to the same
sale deed dated 8-6-1885. The first in regard to Kurthuwa village lands in Suit No.98
(purchased by Bhagwan Din Singh) where it was held that the sale by Sirtaji in favour of
Mata Badal on 8-6-1885 was not for legal necessity, that Mata Badal, a relative of her late
husband by taking undue advantage of her young age had obtained the said sale deed
from Sirtaji, and therefore, on her death, the reversioners of her husband's estate namely
plaintiffs 1 and 2 (Bhagwan Din Singh and Ganga Prasad) and defendants 1 and 2 (Raj
Narain and Chandar Bata) were entitled to the lands. Consequently, sales by persons
claiming through Mata Badal did not have any title after the
@page-SC1555
death of Sirtaji in the year 1940. On the other hand, the second decision, relating to
Ghuskhuri and Manapur villages, in suit Nos. 99 and 100, it was held that the sale by
Sirtaji under deed dated 8-6-1885 in favour of Mata Badal was for legal necessity and
therefore, Mata Badal got valid title and consequently, the sale deeds executed by persons
claiming through Mata Badal were valid, and the suits filed by persons claiming to be
reversioners in respect of the estate of Gajadhar did not have any right, title or interests in
the lands sold by Sirtaji.
4. When matters stood thus, in the consolidation proceedings, the Bhagwati Din Singh
(son of Bhagwan Din Singh and father of appellant) filed an objection under section 9 of
UP Consolidation of Holdings Act, 1954 (in short 'Act') contending that the finding
recorded by the court in Suit Nos.97 to 100 under section 49 of the UP Tenancy Act, 1939
(in short 'Tenancy Act') that the sale deed dated 8-6-1885 by Smt. Sirtaji was not for legal
necessity was the subject matter of appeals before the Addl. Commissioner, Varanasi on
5-9-1966 in Appeal No.231/22 and Appeal No.232/23 who held that the sale deed dated
8-6-1885 executed by Smt. Sirtaji in favour of Mata Badal was for discharging the debts
incurred by Gajadhar, and therefore, was for legal necessity. He contended that judgment
dated 20-3-1944 in suit No.98 to the effect that the sale was not for legal necessity should
be deemed to have been set aside or superseded by the subsequent appellate judgment in
the appeals arising from the suit Nos. 99 and 100 which involved an identical issue and
that the decision dated 5-9-1966 would operate as res judicata, in any subsequent
proceedings relating to the lands which were the subject matter of Suit No.98 even
though the decision dated 5-9-1966 did not relate to Suit No.98.
5. The Consolidation Officer held that the order dated 5-9-1966 was in respect of other
village; and was not concerned with the property in question. He ordered for expunging
the name of Bhagwati Din Singh (the original respondent No.3) from basic year entry.
Bhagwati Din Singh filed an appeal before the Settlement Officer (Consolidation) who
allowed the appeal and held that though the writ petitioners had taken possession on the
basis of decree dated 21-6-1945 arising out of Suit No.98, but appeals were filed relating
to arising out of Suit Nos.99 and 100 against the judgment dated 20-3-1944 and in those
appeals the Additional Commissioner had decided against the writ petitioners on 5-9-
1966 and the judgment passed by the trial Court on 20-3-1944 against Bhagwati Din
Singh in suit No. 98 shall be deemed to have been set aside and the judgment dated 5-9-
1936 passed by the Commissioner shall be deemed to be final. It was held that since the
order dated 20-3-1944 was a common judgment, therefore, it shall be deemed to have
been set aside in all the suits. He further held that though the possession was delivered on
the basis of the order dated 21-6-1945 to the writ petitioners, after the decision dated 5-9-
1966, Bhagwati Din Singh had the right to get possession under Section 144 of the Code
of Civil Procedure, 1908 (in short 'CPC'). But since the possession is joint, therefore,
possession shall not be deemed to have come to an end. A revision petition was filed by
the writ petitioners against the judgment before the Deputy Director of Consolidation,
who dismissed the same affirming the findings of the Settlement Officer by order dated
28-2-1978.
6. Before the High Court the stand of the present appellant further was that what was
necessary to be determined was the effect of the judgment dated 5-9-1966. It was pointed
out that since the appeal filed by the writ petitioners (respondents herein) has been
dismissed, holding that the sale deed dated 8-6-1885 was valid, they were not entitled to
the benefit of the judgment dated 20-3-1944.
7. Stand of the present appellant was that when the trial Court's common judgment dated
20-3-1944 that the sale was not for legal necessity, was set aside by the judgment of
appellate authority dated 5-9-1966 in the other appeals arising from Suit Nos. 99 and 100,
it would have binding effect on the parties in O.S.No.98 also. The High Court was of the
view that the sole controversy was as to whether judgment dated 20-3-1944 affirmed by
the appellate Court in the appeal in 1945 relating to Suit No.98 in the case of appellant's
predecessor will operate as res judicata between the writ petitioners and Bhagwati Din
Singh or the judgment which was delivered on 5-9-1966 in the appeals arising from Suit
Nos.99 and 100 will have the effect of res judicata and the judgment dated 20-3-1944
shall be deemed to have been set aside. The High
@page-SC1556
Court considered the effect of the principles of res judicata and held that the judgment
dated 5-9-1966 will not operate as res judicata between the writ petitioners and Bhagwati
Din Singh (respondent no.3) as that judgment was not between the same parties.
Therefore, it was held that the order of the Consolidation Officer was correct and the
orders of the Settlement Officer and the Deputy Director Consolidation were not legally
sustainable. The writ petition was accordingly allowed.
8. In support of the appeal, learned counsel for the appellant submitted that the judgment
dated 5-9-1966 in the two connected appeals was in respect of a common judgment dated
20-3-1944. It was held that the sale was for legal necessity and that will have effect
notwithstanding the fact that the appeal filed by the appellant was dismissed. He placed
strong reliance on a decision of this Court in Narhari and Ors. v. Shanker and Ors. (AIR
1953 SC 419).
9. On the other hand, learned counsel for the respondents submitted that the appeal filed
by the appellant was dismissed and there was no further challenge. In the circumstances,
the benefit of the findings recorded in the other appeals cannot be extended to the
appellant.
10

. The submission needs careful consideration. At the threshold it must be stated that the
decision in Narhari's case (supra) is clearly distinguishable. The relevant portion of the
judgment in question relied on by the appellant reads as follows : AIR 1953 SC 419

"4. In the judgment of the High Court, though reference is given to some of these
decisions, it is merely mentioned that the appellant relies on these decisions. The learned
Judges perhaps thought that in the presence of the Hyderabad Judicial Committee
decision in Jethmal v. Ranglal they need not comment on these decisions at all. There is
also a later decision of the Judicial Committee of the State in Bansilal v. Mohanlal where
the well known and exhaustive authority of the Lahore High Court in Mst. Lachmi v.
Mst. Bhuli was followed. In the Lahore case, there were two cross suits about the same
subject-matter, filed simultaneously between the same parties, whereas in the present
case, there was only one suit and one judgment was given by the trial court and even in
the first appeal to the Sadar Adalat, there was only one judgment, in spite of there being
two appeals by the two sets of defendants. The plaintiffs in their appeal to the High Court
have impleaded all the defendants as respondents and their prayer covers both the appeals
and they have paid consolidated court-fee for the whole suit. It is now well settled that
where there has been one trial, one finding, and one decision, there need not be two
appeals even though two decrees may have been drawn up. As has been observed by Tek
Chand, J. in his learned judgment in Mst Lachmi v. Mst Bhuli mentioned above, the
determining factor is not the decree but the matter in controversy. As he puts it later in his
judgment, the estoppel is not created by the decree but it can only be created by the
judgment. The question of res judicata arises only when there are two suits. Even when
there are two suits, it has been held that a decision given simultaneously cannot be a
decision in the former suit. When there is only one suit, the question of res judicata does
not arise at all and in the present case, both the decrees are in the same case and based on
the same judgment, and the matter decided concerns the entire suit. As such, there is no
question of the application of the principle of res judicata. The same judgment cannot
remain effective just because it was appealed against with a different number or a copy of
it was attached to a different appeal. The two decrees in substance are one. Besides, the
High Court was wrong in not giving to the appellants the benefit of Section 5 of the
Limitation Act because there was conflict of decisions regarding this question not only in
the High Court of the State but also among the different High Courts in India." AIR
1927 Lahore 289

11. Res Judicata is a principle of judicial administration and is based on the common law
maxim of public policy aiming at finality of litigation and preventing a litigant from
being tried twice over on the same issue.
12

. The Privy Council in a series of judgments explained this doctrine. In Kalipada De v.


Dwijapada Das reported in 57 IA 24, the Privy Council held : AIR 1930 PC 22

"The question as to what is to be considered to be res judicata is dealt with by Section 11


of the Code of Civil Procedure, 1908. In that Section are given many examples of
@page-SC1557
circumstances in which the rule concerning res judicata applies; but it has often been
explained by this Board that the terms of Section 11 are not be regarded as exhaustive."
13

. In Kalipada's case (supra), Lord Justice Darling, speaking for the Bench, quoted with
approval the observations of Sir Lowrence Jenkins on Res Judicata in Sheoparsan Singh
and Ors. v. Ramnandan Singh reported in 43 LA. 91. Those observations are oft quoted
and read as follows : AIR 1916 PC 78

"..their Lordships desire to emphasise that the rule of res-judicata, while-founded on


ancient precedent, is dictated by a wisdom which is for all time. 'It hath been well said,'
declared Lord Coke, 'interest reipublicae ut sit finis litium-otherwise, great oppression
might be done under colour and pretence of law' (6 Coke, 9a). Though the rule of the
Code may be traced to an English source, it embodies a doctrine in no way opposed to the
spirit of the law as expounded by the Hindu commentators. Vijnanesvara and Nilakantha
include the plea of a former Judgment among those allowed by law, each citing for this
purpose the text of Katyayana, who describes the plea thus: 'If a person, though defeated
at law, sue again, he should be answered, "You were defeated formerly." This is called the
plea of former Judgment. (See the Mitakshara (Vyavaharaj, bk. II., ch. I., edited by J.R.
Gharpure, p. 14, and the Mayuka, ch.I., s.I, p.II, of Mandlik's edition.) And so the
application of the rule by the Courts in India should be influenced by no technical
considerations of form, but by matter of substance within the limits allowed by law."
14

. This statement of law in Sheoparsan's case (supra) has been approved by this Court in
the case of Iftikhar Ahmed and Ors. v. Syed Meharban Ali and Ors. (1974 (2) SCC 151)
AIR 1916 PC 78
AIR 1974 SC 749

15

. This Court in Lal Chand v. Radha Kishan (1977 (2) SCC 88) also held : AIR 1977 SC
789, Para 19

The principle of Res Judicata is conceived in the larger public interest which requires that
all litigation must, sooner than later, come to an end. The principle is also founded on
equity, justice and good conscience which require that a party which has once succeeded
on an issue should not be permitted to be harassed by a multiplicity of proceedings
involving determination of the same issue.
16. Apart from following those principles, this Court in order to apply the bar of res
judicata among co-defendants must consider several criteria pointed out in the case of Mt.
Munni Bibi and Anr. v. Tirloki Nath and Ors. (AIR 1931 PC 114). In the said case three
tests have been laid down to find out whether the decision in the former suit will operate
as Res Judicata between co-defendants. Those tests are :
(i) There must be a conflict of interest between the co-defendants.
(ii) It must be necessary to decide this conflict in order to give relief to the petitioner.
(iii) The question between the co-respondent must be finally decided.
17. It is to be noted that the factual scenario was entirely different in the said case. It
related to two separate decrees in one suit and therefore it was held that the principle of
res judicata did not apply. Admittedly, in the instant case there were four suits. The
decision that was relevant was in suit No.98 which attained finality. The decision in the
appeals relating to Suit Nos. 99 and 100 does not affect the decision in Suit No.98 which
had attained finality. On a closer reading of the decisions it is clear that it does not help
the appellant, it goes against the submissions made. It also needs to be noted that the
plaintiffs in all the four suits were common but the defendants in the suit were not
common, and the properties were situated in different villages.
18. At this juncture, the provisions of the Order 41, Rule 33 CPC also need to be noted.
By the said provision benefit is available to a party not appealing. But the emphasis is on
the same suit. Therefore, the view of the High Court is irreversible.
19. The appeal is without merit and deserves dismissal which we direct. There shall be no
order as to costs.
Appeal dismissed.
AIR 2008 SUPREME COURT 1557 "Government of A. P. v. R. V. Raju"
(From : Andhra Pradesh)
Coram : 2 TARUN CHATTERJEE AND H. S. BEDI, JJ.
Civil Appeal No. 4258 of 2001, D/- 19 -2 -2008.
Government of A. P. and Anr. v. R. V. Raju and Ors.
A.P. Land Grabbing (Prohibition) Act (12 of 1982), S.4 - LAND - EVICTION - APPEAL
- SPECIAL COURT - Land grabbers - Eviction
@page-SC1558
Appeal against order setting aside finding of special Court that respondent were land
grabbers - Supreme Court directed respondents to get their possession regularised by
depositing certain sum within fixed time - Failure to lead to eviction.
Constitution of India, Art.133. (Para 2)

R. Sundarvardhan, Sr. Advocate, Manoj Saxena, Rajneesh Kr. Singh, Rahul Shukla, T.V.
George, for Appellants; M.N. Rao, Sr. Advocate, A.D.N. Rao, Ms. Rita Ku. Gupta. K.S.
Murthi, for Respondents.
Judgement
1. JUDGMENT:-This appeal is directed against the final judgment and order dated 9th of
February, 2001 passed by the High Court of Judicature at Andhra Pradesh, Hyderabad in
Writ Petition No. 27838 of 1995 by which the High Court had set aside the judgment of
the Special Court dated 29th of September, 1995 in L. G. C. No. 72 of 1991.
2. We have heard the learned counsel for the parties and examined the materials on
record. We are of the view that if an amount of Rs. 8,83,000/- is deposited with the
appellants by the respondents within 4 weeks from this date, the land of the respondents
in respect of which eviction has been sought for by the appellants under the A. P. Land
Grabbing (Prohibition) Act shall stand regularised and the judgment of the Special Court
against which the writ petition was preferred before the High Court shall stand set aside
and the judgment of the High Court passed in Writ Petition No. 27838 of 1995, which is
now challenged before us shall stand confirmed. In the event, the amount, as noted above,
is not deposited within 4 weeks from this date, the order of the High Court shall stand set
aside and this appeal shall stand allowed and the order of the special Court dated 29th of
September, 1995 in LGC No. 72 of 1991 shall stand confirmed.
3. The appeal is thus disposed of with the aforesaid direction. There shall be no order as
to costs.
Order accordingly.
AIR 2008 SUPREME COURT 1558 "Vinay D. Nagar v. State of Rajasthan"
(From : 2005 (2) Rajasthan LR 163)
Coram : 2 PRAKASH PRABHAKAR NAOLEKAR AND LOKESHWAR SINGH
PANTA, JJ.
Criminal Appeal No. 210 of 2007, D/- 3 -3 -2008.
Vinay D. Nagar v. State of Rajasthan.
(A) Criminal P.C. (2 of 1974), S.161, S.162 - INVESTIGATION - POLICE OFFICERS -
OBJECT OF AN ACT - EVIDENCE - Statement recorded by police during investigation
- Admissibility in evidence - Bar u/S.162 - Scope - Limited to use of statement in case
that was under investigation - Use in different case not barred.
Bar of S. 162 Cr. P. C. is in regard to the admissibility of the statement recorded of a
person by the police officer under S. 161 Cr. P. C. and by virtue of S. 162 Cr. P. C. would
be applicable only where such statement is sought to be used at any inquiry or trial in
respect of any offence under investigation at the time when such statement was made. If
the statement made before a police officer in the course of an investigation under Chap.
XII is sought to be used in any proceeding, inquiry or trial in respect of an offence other
than which was under investigation at the time when such statement was made, the bar of
S. 162 will not be attracted. Thus when the statement of office chowkidar was recorded
by the police officers under S. 161 Cr. P. C. during the investigation of abduction case of
a boy, that statement could be used in the subsequent investigation that was being made
with respect to the alleged murder of chowkidar. (Paras 9, 10, 11)
(B) Evidence Act (1 of 1872), S.32 - DYING DECLARATION - Statement of person
who is dead - Admissibility - 'Any circumstances of transaction leading to his death' -
Case of murder of star witness in case of abduction of child - Statement of star witness
recorded by police only showing involvement of accused in abduction case - Had no
remote connection or reference to death of deceased - Would not be admissible u/S.32.
(Para 15)
(C) Penal Code (45 of 1860), S.300 - MURDER - EVIDENCE - Murder - Circumstantial
evidence - Deceased star witness in child abduction case - Alleged to be murdered by
accused to prevent him from giving evidence - Accused absconding after incident - False
statement made by accused that on
@page-SC1559
date of incident he had gone at far of place - No evidence however produced to show that
accused was at place of incident - No proof of accused being last seen with deceased -
Circumstances do not exclude every hypothesis other than guilt of accused - Accused not
liable to be convicted.
2005 (2) Rajasthan LR 163, Reversed. (Para 16)
Cases Referred : Chronological Paras
2000 AIR SCW 2093 : AIR 2000 SC 2324 : 2000 Cri LJ 2993 (Rel. on Pnt. B) 14
1997 AIR SCW 587 : AIR 1997 SC 768 : 1997 Cri LJ 833 (Rel. on Pnt. B) 14
1996 AIR SCW 2903 : AIR 1996 SC 3390 (Ref.) 5
AIR 1990 SC 79 : 1990 Cri LJ 605 (Ref.) 5
AIR 1984 SC 1622 : 1984 Cri LJ 1738 (Rel. on) 14
AIR 1981 SC 1068 : 1981 Cri LJ 597 (Rel. on) 10
AIR 1939 PC 47 : 1939 (40) Cri LJ 364 (Ref) 14
U.U. Lalit, Sr. Advocate, Sanjay Sharawat, Rajesh Sharma, Nitin Sangra, for Appellant;
V. Madhukar, Sunit Ghosh, (for Aruneshwar Gupta) for Respondent.
Judgement
1. P. P. NAOLEKAR, J. :-This criminal appeal by special leave is directed against the
judgment and order dated 23-11-2004 passed by the High Court of Rajasthan, Jaipur
Bench in Criminal Appeal (DB) No. 990/2002, which upheld the conviction and sentence
of the appellant under Sections 364, 450, 302, 201 of the Indian Penal Code, 1860 (for
short "I.P.C.").
2. The relevant facts of the case as per the prosecution are that Kalu (the deceased) was
Chowkidar in the office of Agriculture Extension Bundi and his duty was during the night
in the office premises. He was found missing under mysterious circumstances on 15-07-
2000, hence informant Ramesh Chand Jain, Assistant Director lodged a written report on
15-07-2000 at 7:30 a.m. in the Police Station, Bundi. On the basis of the report, a case
under Section 456/364 IPC was registered and investigation commenced. During the
investigation, it was revealed that Kalu was a star witness in a criminal case registered
against the appellant-Vinay D. Nagar and others under Sections 365, 364, 328, 342, 323,
IPC. As per the prosecution case, the accused along with his other companions had
abducted a child Sonu on 07-07-2000 and had brought Sonu in the office where Kalu was
Chowkidar and kept him in the office for some time. The activities of the accused and his
companions made Kalu suspicious. Since Kalu had seen the accused with Sonu and as the
accused was a Clerk in the same office where Kalu was posted as a Chowkidar, the
statement of Kalu was recorded by the Investigating Officer under Section 161 of the
Code of Criminal Procedure, 1973 (for short "Cr.P.C."). The Investigating Officer moved
an application before the Magistrate on 12-07-2000 to record the statement of Kalu under
Section 164 Cr.P.C. and Kalu was to be produced before the Magistrate on 17-07-2000,
the date given by the Magistrate. But in the intervening period, his dead body was found
in a tank on 19-07-2000. The post mortem report indicated that the death of the deceased
was homicidal. The appellant-accused was arrested and put to trial. In his statement under
Section 313 Cr.P.C., he stated that on the relevant date he had gone to Bombay, but the
explanation was found false in view of the evidence led by the prosecution whereby it
was found that he had gone to Ahmedabad and not to Bombay. The Session Court found
the accused guilty and convicted him.
3. The accused preferred an appeal before the High Court which was dismissed holding
that in the fact situation the deceased had seen the accused with Sonu and had named the
accused as the main culprit. The statement of Kalu was recorded under Section 161
Cr.P.C. on 10-07-2000. On 09-07-2000, the accused absented himself from the office and
disappeared without submitting any leave application. Later on, Kalu was found dead on
19-07-2000. It was held by the High Court that the evidence collected by the prosecution
shows that the accused had a strong motive and the opportunity for committing a crime.
The High Court further held that the accused was absconding and hence the
disappearance of the accused after the occurrence was a relevant circumstance which in
the absence of plausible rebuttal evidence can be taken into consideration. The High
Court was of the view that from the aforesaid circumstances an inference can be drawn
towards the appellant's guilt. All the aforementioned circumstances taken cumulatively
have formed a chain so complete that there is no escape
@page-SC1560
from the conclusion that within all human probability the crime was committed by the
appellant and none else. Hence, the appeal was dismissed by the High Court.
4. It is urged by Shri U.U. Lalit, learned senior counsel for the appellant that the
appellant's conviction is based on circumstantial evidence. The prosecution has relied
upon the statement made by the deceased Kalu under Section 161 Cr.P.C. to prove the
motive for commission of the crime. As per the learned senior counsel, the statement of
Kalu under Section 161 Cr.P.C. read with Section 32 of the Indian Evidence Act, 1872, is
not admissible and thus the courts below have committed an error in relying on the
statement made by the deceased Kalu under Section 161 Cr.P.C. for the alleged motive of
the appellant to commit the crime. It is further urged that the prosecution has completely
failed to prove the chain of circumstances which should point to the guilt of the accused
and none else.
5

. This Court in several cases has expounded principles for cases based on circumstantial
evidence. In the case of C. Chenga Reddy and Ors. v. State of Andhra Pradesh, AIR 1996
SC 3390, this Court in para 20-A observed thus : 1996 AIR SCW 2903

"In a case based on circumstantial evidence, the settled law is that the circumstances from
which the conclusion of guilt is drawn should be fully proved and such circumstances
must be conclusive in nature. Moreover, all the circumstances should be complete and
there should be no gap left in the chain of evidence. Further, the proved circumstances
must be consistent only with the hypothesis of the guilt of the accused and totally
inconsistent with his innocence......"

Further, in Padala Veera Reddy State of Andhra Pradesh and Ors., 1989 (Supp) 2 SCC
706, it was laid down that when a case rests upon circumstantial evidence, such evidence
must satisfy the following tests : AIR 1990 SC 79

(1) the circumstances from which an inference of guilt is sought to be drawn, must be
cogently and firmly established;
(2) those circumstances should be of a definite tendency unerringly pointing towards guilt
of the accused;
(3) the circumstances, taken cumulatively, should form a chain so complete that there is
no escape from the conclusion that within all human probability the crime was committed
by the accused and none else; and
(4) the circumstantial evidence in order to sustain conviction must be complete and
incapable of explanation of any other hypothesis than that of guilt of the accused and
such evidence should not only be consistent with the guilt of the accused but should be
inconsistent with his innocence.
The principle of law is well established that where the evidence is of a circumstantial
nature, circumstances from which the conclusion of guilt is to be drawn should in the first
instance be fully established, and the facts, so established, should be consistent only with
the hypothesis of the guilt of the accused. The circumstances should be of a conclusive
nature and they should be such as to exclude hypothesis than the one proposed to be
proved. In other words, there must be chain of evidence so complete as not to leave any
reasonable ground for a conclusion consistent with the innocence of the accused and it
must be such as to show that within all human probability the act must have been done by
the accused.
6. The circumstances on which the reliance has been placed by the prosecution are that
Kalu was the prime witness in Sonu's kidnapping case and had made Section 161 Cr.P.C.
statement alleging that the accused was responsible for abduction of Sonu; that it was
apprehended by the appellant that Kalu would make a statement before the Magistrate
under Section 164 Cr.P.C. on 17-07-2000 indicting the appellant, hence the appellant-
accused had strong motive to eliminate Kalu; that the accused absented himself from the
office from 10-07-2000 without taking leave; and that in his Section 313 Cr.P.C.
statement he stated that he had gone to Bombay but it was found out that he actually
stayed in Ahmedabad under the fictitious name in a hotel from 11-07-2000 to 12-07-2000
and thereafter in another hotel till 14-07-2000; and that there was a possibility of the
accused reaching Bundi from Ahmedabad on the date of the incident. The fact that he
stayed in Ahmedabad under fictitious name has been relied upon by the prosecution to
show that his conduct was suspicious. On 15-07-2000, Kalu was found missing under
suspicious circumstances and his dead body was found
@page-SC1561
on 19-07-2000.
7. In the statement recorded by the police under Section 161 Cr.P.C. in a case registered
under FIR No. 290/2000, the deceased Kalu had stated that he was on duty from 5.00 in
the evening till 10.00 in the morning of 8-7-2000. At about 8.30 p.m., two men came in a
Maruti car and Vinay D. Nagar, Cashier in his Department came on a motorcycle. They
sat in the officer's room and started making phone calls. On enquiry being made by him,
the accused told him that he was calling some acquaintance. Thereafter, he went to take
meals and when he returned after half an hour, all the three persons were still there and
they left in the same car at 9.00 p.m. The motorcycle of the accused was left there. At
5.00 in the morning, Vinay climbed over the gate and entered the office. Vinay woke him
up and took out the keys from the bag. He opened the main gate and brought the car
inside. He opened the shutter in the verandah and the room. First he took keys and
opened the computer room and then brought out one child from the rear seat of the
Maruti van and put him in the computer room. That child was kept lying in the computer
room for 10-15 minutes. Then after 10-15 minutes they came out of that room and all
three of them put the child in the Maruti van and left. He stated that he had read the
newspaper and learnt from others that last night one boy had been abducted. He stated
that he could identify all four persons who had come to him.
8. It is urged by the learned senior counsel that the statement recorded by the police under
Section 161 Cr.P.C. of the deceased in abduction case is not admissible under Section 162
Cr.P.C. and, therefore, the prosecution could not have relied upon the statement of the
deceased Kalu recorded by the police.
9. The question is whether the statement recorded under Section 161 Cr.P.C. of the
deceased Kalu in a case registered under FIR No. 290/2000 (abduction case) is
admissible in the case registered under FIR No. 301/2000 (murder trial) in view of the
provisions of Section 162 Cr.P.C. Section 162 Cr.P.C. reads as under :
"162. Statements to police not to be signed: Use of statements in evidence. (1) No
statement made by any person to a police officer in the course of an investigation under
this chapter, shall, if reduced to writing, be signed by the person making it; nor shall any
such statement or any record thereof, whether in a police diary or otherwise, or any part
of such statement or record, be used for any purpose, save as hereinafter provided, at any
inquiry or trial in respect of any offence under investigation at the time when such
statement was made :
Provided that when any witness is called for the prosecution in such inquiry or trial
whose statement has been reduced into writing as aforesaid, any part of his statement, if
duly proved, may be used by the accused, and with the permission of the Court, by the
prosecution, to contradict such witness in the manner provided by section 145 of the
Indian Evidence Act, 1872; and when any part of such statement is so used, any part
thereof may also be used in the re-examination of such witness, but for the purpose only
of explaining any matter referred to in his cross-examination.
(2) Nothing in this section shall be deemed to apply to any statement falling within the
provisions of clause (1) of section 32 of the Indian Evidence Act, 1872, or to affect the
provisions of section 27 of that Act.
Explanation.- An omission to state a fact or circumstance in the statement referred to in
sub-section (1) may amount to contradiction if the same appears to be significant and
otherwise relevant having regard to the context in which such omission occurs and
whether any omission amounts to a contradiction in the particular context shall be a
question of fact."
On account of Section 162 Cr.P.C., a statement made by any person to a police officer in
the course of investigation under Chapter XII, if reduced into writing, will not be signed
by the person making it, nor such statement recorded or any part thereof be used for any
purpose at any inquiry or trial in respect of any offence under investigation at the time
when such statement was made. Such statement may be used by an accused and with the
permission of the Court by the prosecution to contradict the witness whose statement was
recorded by the police in the manner provided under Section 145 of the Indian Evidence
Act and can also be used for re-examination of such witness for the purpose only of
explaining any matter referred to in his cross-examination . Bar of Section 162 Cr. P. C.
of proving the statement recorded by the police officer
@page-SC1562
of any person during investigation however shall not apply to any statement falling
within the provision of clause (1) of Section 32 of the Evidence Act, nor it shall affect
Section 27 of the Evidence Act. Bar of Section 162 Cr.P.C. is in regard to the
admissibility of the statement recorded of a person by the police officer under Section
161 Cr.P.C. and by virtue of Section 162 Cr.P.C. would be applicable only where such
statement is sought to be used at any inquiry or trial in respect of any offence under
investigation at the time when such statement was made.
10. In the case of Khatri and others v. State of Bihar and Ors., AIR 1981 SC 1068, this
Court has held that Section 162 Cr.P.C. bars the use of any statement made before the
police officer in the course of an investigation under Chapter XII, whether recorded in the
police diary or otherwise. However, by the express terms of Section, this bar is applicable
only where such statement is sought to be used 'at any inquiry or trial' in respect of any
offence under investigation at the time when such statement was made. If the statement
made before a police officer in the course of an investigation under Chapter XII is sought
to be used in any proceeding, inquiry or trial in respect of an offence other than which
was under investigation at the time when such statement was made, the bar of Section
162 will not be attracted.
11. When the statement of Kalu was recorded by the police officers under Section 161
Cr.P.C. during the investigation of abduction case of a boy, Kalu was alive and thus that
statement could be used in the subsequent investigation that was being made with respect
to the alleged murder of Kalu.
12. It is then urged by the learned senior counsel that even on lifting of Section 162 bar, it
by itself will not make the statement of Kalu recorded by the police admissible in
evidence. Statement can be admitted in evidence only by virtue of any of the provisions
contained in the Evidence Act. Therefore, even if the Section 162 bar would not apply to
Kalu's 161 statement, would it be admissible in evidence. Then the next step would be to
see as to under which provision of the Evidence Act, the same shall be admissible.
According to the learned senior counsel, the statement of a person who is not alive for the
purpose of cross-examination in case need arises, would be admissible only if it falls
within the four corners of Section 32 of the Indian Evidence Act.
13. Section 32 of the Indian Evidence Act enumerates eight clauses in which the
statements made by a dead person or a person who cannot be found or who has become
incapable of giving evidence or whose attendance cannot be procured in Court can be
admitted in evidence. Clauses (2) to (8) of Section 32 are not material for the purpose of
deciding the present case. The relevant provision reads as under :
"32. Cases in which statement of relevant fact by person who is dead or cannot be found,
etc., is relevant. - Statements, written or verbal, of relevant facts made by a person who is
dead, or who cannot be found, or who has become incapable of giving evidence, or
whose attendance cannot be procured, without an amount of delay or expense which
under the circumstances of the case appears to the Court unreasonable, are themselves
relevant facts in the following cases :-
(1) When it relates to cause of death. - When the statement is made by a person as to the
cause of his death, or as to any of the circumstances of the transaction which resulted in
his death, in cases in which the cause of that person's death comes into question.
Such statements are relevant whether the person who made them was or was not, at the
time when they were made, under expectation of death, and whatever may be the nature
of the proceeding in which the cause of his death comes into question." Clause (1) says
that when a statement is made by a person as to the cause of his death or as to any of the
circumstances of the transaction which resulted in his death, such statement would be
relevant. So the question is whether the statement made by deceased Kalu under Section
161 Cr.P.C. in previous investigation would be admissible as per the second part of
Section 32(1) of the Evidence Act which says that the statement made by a person as to
the 'circumstances of the transaction which resulted in his death' would be admissible and
whether the deceased's statement under Section 161 Cr.P.C. falls under 'circumstances of
the transaction which resulted
@page-SC1563
in his death'.
14. In the case of Pakala Narayana Swami v. Emperor, AIR 1939 PC 47, it was held that a
statement merely suggesting motive for a crime cannot be admitted in evidence unless it
is so intimately connected with the transaction itself as to be a circumstance of the
transaction.
In the case of Sharad Birdhichand Sarda v. State of Maharashtra, AIR 1984 SC 1622, this
Court in para 21 held as under :
"Thus, from a review of the authorities mentioned above and the clear language of
Section 32(1) of the Evidence Act, the following propositions emerge :
(1) Section 32 is an exception to the rule of hearsay and makes admissible the statement
of a person who dies, whether the death is a homicide or a suicide, provided the statement
relates to the cause of death, or exhibits circumstances leading to the death. In this
respect, as indicated above, the Indian Evidence Act, in view of the peculiar, conditions
of our society and the diverse nature and character of our people, has thought it necessary
to widen the sphere of S. 32 to avoid injustice.
(2) The test of proximity cannot be too literally construed and practically reduced to a
cut-and-dried formula of universal application so as to be confined in a strait-jacket.
Distance of time would depend or vary with the circumstances of each case. For instance,
where death is a logical culmination of a continuous drama long in process and is, as it
were, a finale of the story, the statement regarding each step directly connected with the
end of the drama would be admissible because the entire statement would have to be read
as an organic whole and not torn from the context. Sometimes statements relevant to or
furnishing an immediate motive may also be admissible as being a part of the transaction
of death. It is manifest that all these statements come to light only after the death of the
deceased who speaks from death. For instance, where the death takes place within a very
short time of the marriage or the distance of time is not spread over more than 3-4 months
the statement may be admissible under S. 32.
(3) The second part of Cl. (1) of S. 32 is yet another exception to the rule that in criminal
law the evidence of a person who was not being subjected to or given an opportunity of
being cross-examined by the accused, would be valueless because the place of cross-
examination is taken by the solemnity and sanctity of oath for the simple reason that a
person on the verge of death is not likely to make a false statement unless there is strong
evidence to show that the statement was secured either by prompting or tutoring.
(4) It may be important to note that Section 32 does not speak of homicide alone but
includes suicide also, hence all the circumstances which may be relevant to prove a case
of homicide would be equally relevant to prove a case of suicide.
(5) Where the main evidence consists of statements and letters written by the deceased
which are directly connected with or related to her death and which reveal a telltale story,
the said statement would clearly fall within the four corners of Section 32 and, therefore,
admissible. The distance of time alone in such cases would not make the statement
irrelevant."

Further, in the case of Rattan Singh v. State of Himachal Pradesh, AIR 1997 SC 768, this
Court has held as under : 1997 AIR SCW 587

"Section 32(1) of the Evidence Act renders a statement relevant which was made by a
person who is dead in cases in which cause of his death comes into question, but its
admissibility depends upon one of the two conditions: Either such statement should relate
to the cause of his death or it should relate to any of the circumstances of transaction
which resulted in his death. The collocation of the words in Section 32(1) "circumstances
of the transaction which resulted in his death" is apparently of wider amplitude than
saying "circumstances which caused his death". There need not necessarily be a direct
nexus between "circumstances" and "death". It is enough if the words spoken by the
deceased have reference to any of the transactions which ended up in the death of the
deceased. Such statement would also fall within the purview of Section 32(1) of the
Evidence Act. In other words, it is not necessary that such circumstance should be
proximate, for, even distant circumstances can also become admissible under the sub-
section, provided it has nexus with the transaction which resulted in the death."
(Headnote-B)
@page-SC1564

In the case of Kans Raj v. State of Punjab and Ors., AIR 2000 SC 2324, a 3-Judge Bench
of this Court dealt with Section 32(1) statement made by the deceased who had allegedly
died due to dowry harassment and in para 10 held as under : 2000 AIR SCW 2093

"Section 32 of the Evidence Act is an exception to the general rule of exclusion of


hearsay evidence and the statements of a person, written or verbal, of relevant facts, after
his death are admissible in evidence if they refer to the cause of his death or to any
circumstances of the transaction which resulted in his death. To attract the provisions of
Section 32, for the purposes of admissibility of the statement of a deceased the
prosecution is required to prove that the statement was made by a person who is dead or
who cannot be found or whose attendance cannot be procured without an amount of delay
or expense or he is incapable of giving evidence and that such statement had been made
under any of the circumstances specified in sub-sections (1) to (8) of Section 32 of the
Act. Section 32 does not require that the statement sought to be admitted in evidence
should have been made in imminent expectation of death. The words "as to any of the
circumstances of the transaction which resulted in his death" appearing in Section 32
must have some proximate relation to the actual occurrence. In other words, the statement
of the deceased relating to the cause of death or the circumstances of the transaction
which resulted in his death must be sufficiently or closely connected with the actual
transaction. To make such statement as substantive evidence, the person or the agency
relying upon it is under a legal obligation to prove the making of such statement as a fact.
If it is in writing, the scribe must be produced in the Court and if it is verbal, it should be
proved by examining the person who heard the deceased making the statement."
15. We have analysed the statement of the deceased Kalu made to the police under
Section 161 Cr.P.C. We do not find that the statement of the deceased was in regard to the
cause of his death, or as to any of the circumstances of the transaction which resulted in
his death. The statement is in regard to the accused's involvement in the abduction of a
boy and has no remote connection or reference to the death of the deceased and thus
would not be admissible under Section 32 of the Evidence Act. The statement recorded
by the police although could be proved as there would not be any bar under Section 162
Cr.P.C. for proof of such statement, but it would not be admissible under Section 32 of
the Evidence Act, and thus it could not have been relied upon by the prosecution to prove
the motive for commission of the crime by the accused appellant.
16. We have gone through the evidence placed on record by the prosecution. None of the
witnesses stated that at the relevant time and/or relevant date they had seen the accused at
Bundi. The witnesses examined by the prosecution have proved the fact that the accused
stayed at some of the hotels in Ahmedabad, but there is no proof of the fact that he
checked in the hotel(s) giving the fictitious name. There is no proof of the accused being
last seen with the deceased. The prosecution has failed to prove the accused's presence on
the date of the incident at Bundi. The evidence adduced by the prosecution does not point
to the guilt of the accused. The circumstances on which the High Court has placed
reliance do not establish the guilt of the accused, nor does it exclude every hypothesis but
the one proposed to be proved by the prosecution. The prosecution has failed to prove the
chain of evidence by which one could clearly and unequivocally reach to the conclusion
of pointing the guilt of the accused-appellant for commission of the crime.
17. For the aforesaid reasons, the appeal is allowed. The judgment of the High Court and
that of the Session Court are set aside. The accused-appellant is directed to be set at
liberty if he is not required in any other case.
Appeal allowed.
AIR 2008 SUPREME COURT 1564 "Kailash v. State of Rajasthan"
(From : Rajasthan)*
Coram : 2 S. B. SINHA AND V. S. SIRPURKAR, JJ.
Criminal Appeal No. 416 of 2008 (arising out of SLP (Cri.) NO. 647 of 2006), D/- 3 -3
-2008.
Kailash v. State of Rajasthan and Anr.
(A) Criminal P.C. (2 of 1974), S.319 - SUMMONS - ADDITIONAL ACCUSED -
Summoning of additional accused
@page-SC1565
Power of Court - Court has to be circumspect while exercising power.
Provisions of S. 319 Cr. P. C. would suggest that during the trial it has to appear from the
evidence that a person not being an accused has committed any offence for which such
person could be tried together with the accused who are also being tried. The key words
in this Section are 'it appears from the evidence', 'any person', 'has committed any
offence. It is not, therefore, that merely because some witnesses have mentioned the
name of such person or that there is some material against that person, the discretion
under S. 319 would be used by the Court. This is apart from the fact that such person
against whom such discretion is used, should be a person who could be tried together
with the accused against whom the trial is already going on. The discretion under S. 319
has to be exercised very sparingly and with caution and only when the concerned Court is
satisfied that some offence has been committed by such person. This power has to be
essentially exercised only on the basis of the evidence. It could, therefore, be used only
after the legal evidence comes on record and from that evidence it appears that the
concerned person has committed an offence. The words 'it appears' are not to be read
lightly.
2007 AIR SCW 3399, AIR 1983 SC 67, 2004 AIR SCW 4809, 2000 AIR SCW 734,
Relied on. (Para 9)
(B) Criminal P.C. (2 of 1974), S.319 - SUMMONS - ADDITIONAL ACCUSED -
Summoning of additional accused - Accused persons allegedly assaulted complainant -
Prayer for arraying appellant - None of witnesses stated that appellant had axe in hand - It
was only during their evidence in Court that they gave improved version by stating that
appellant inflicted axe blow - Conclusion of trial by itself was no reason to try him with
aid of S.319 - High Court did not apply itself independently to conclude that there was
evidence to add him as accused - Rejection of prayer for arraying appellant as accused -
Proper.
2002 AIR SCW 2079, Disting.
S.B. Cri. Revn. Petn. No.416 of 2004, D/-14-11-2005 (Raj.), Reversed. (Paras 12, 13)
Cases Referred : Chronological Paras
2007 AIR SCW 3399 : AIR 2007 SC 1899 : 2007 Cri LJ 3198 : 2007 (4) ALJ 317 (Rel.
on) 10
2004 AIR SCW 4809 : AIR 2004 SC 4298 : 2004 Cri LJ 4185 : 2004 AIR-Kant HCR
2766 (Rel. on) 11
2002 AIR SCW 2079 : AIR 2002 SC 2031 : 2002 Cri LJ 2806 (Disting.) 13
2000 AIR SCW 734 : AIR 2000 SC 1127 : 2000 Cri LJ 1706 (Rel. on) 11
AIR 1983 SC 67 : 1983 Cri LJ 159 (Rel. on) 10
Mrs. Nilofar Qureshi, Vipin Kumar, Ms. Mumtaz Ahmad, Shankar Divate, for Appellant;
Milind Kumar, Aruneshwar Gupta, Ms. Pratibha Jain, for Respondents.
* S. B. Cri. Revn. Petn. No. 416 of 2004, D/- 14-11-2005 (Raj).
Judgement
V. S. SIRPURKAR, J. :-Leave granted.
2. Aggrieved by the order passed by the High Court in Criminal Revision, the accused
comes up before this Court by way of this appeal.
3. By its impugned order, the High Court allowed the Revision and directed the Trial
Court to re-hear the application filed under Section 319 of the Code of Criminal
Procedure by applying its judicious mind and to pass the appropriate order according to
law.
4. The following facts will be necessary for our purpose. On 12-11-2003 a written report
came to be made at Police Station Neem-ka-Thana, District Sikar, Rajasthan by one
Rohitas Kumar, contending therein that while he was having his dinner at his home, he
was attacked by Ram Prasad Kailash, Pawan Kumar, Krishan Kumar, Chameli and
Manju. It was asserted therein that Chameli and Manju also assaulted his wife Maya
Devi. On the basis of this report offences were registered under Sections 147, 148, 341,
452, 24/149, 323, 324/149 and 308/149 IPC. The investigation proceeded and a charge-
sheet came to be filed. However, in that charge-sheet the present appellant Kailash was
not arrayed as an accused. During the course of trial when the prosecution witnesses were
examined, the complainant moved an application under Section 319 Cr.P.C. However,
that application came to be rejected. In that application, the complainant alleged that the
present appellant Kailash was bound to be joined as an accused as it was clear from the
records and the evidence that there was enough material against him.
5. This application was opposed by the other accused persons on the ground that there
was no material against Kailash and,
@page-SC1566
therefore, there was no basis for taking cognizance on the basis of the application made
by the complainant. The State supported the application contending that the witnesses had
stated that Kailash had held an axe in his hand that he had hit on the hand and head of
Rohitas. The injury on the head of Rohitas was proved from the medical report and that
Rohitas, Ram Singh Ramavtar and Maya Devi had also involved Kailash in their
statements before the police under Section 161 Cr.P.C.
6. The learned Sessions Judge took stock of the evidence which was led during the trial
and came to the conclusion that there was nothing in the First Information Report (Exh.P-
1) to suggest that Kailash was having an axe in his hand and that he had caused the injury
on the head or finger of the injured Rohitas. He also found that in the statements of the
witnesses including injured Rohitas, Ram Singh, Ramavtar and Maya Devi, nobody had
stated that Kailash was having an axe in his hand and he caused any injury to Rohitas
with axe. He found that Rohitas, in his police statement, has stated to have been hit with a
lathi. The Sessions Judge also observed that even if Kailash was presumed to be present
at the place of occurrence, it was not proved that he took part in the assault and there was
no justification for taking any cognizance against him. He found that Rohitas (PW1) in
his statement had improved upon his evidence before the Court and had added that
Kailash inflicted axe blow on his hand and head, however, his injuries suggested that they
were caused by a blunt weapon. It was also found by the learned Sessions Judge that even
Ramavtar (PW3) had deposed that nobody struck Rohitas with an axe before him. Even
Maya (PW-4) had also not stated in her statement about Kailash to have inflicted injuries
to Rohitas with an axe. Although these witnesses had stated in their evidence that Kailash
was having an axe in his hand, the Sessions Judge found that they have made
improvement in their evidence. He accordingly dismissed the application by his order
dated 24-4-2004.
7. It was this order of the Sessions Judge which was challenged by way of a Revision
Petition. The Revision remained on pending and in the meantime, however, the other five
accused, against whom the prosecution was going on, were acquitted of the charges under
Sections 147, 148, 452, 324 or 324/149, 325 or 325/149, 308/149 and 341 of the Indian
Penal Code. Only three accused came to be convicted for offences under Section 323
IPC, they were accused Krishna Kumar, Smt. Manju Devi and Chameli. They were,
however, not awarded with any punishment and were given the benefit of Section 4 of the
Parole Act. Very strangely, the trial was not stayed during the pendency of the Revision
Petition before the High Court.
8. Learned counsel appearing on behalf of the appellant pointed out that the High Court,
while exercising its revisional jurisdiction, has patently erred in relying on the
observations made by the Sessions Judge in his acquittal judgment. According to the
learned counsel, the High Court has not applied itself as to the correctness of the
discretion exercised by the Trial Court in not summoning the accused. It was pointed out
that merely because some witnesses in their evidence had involved Kailash, that by itself
would not be sufficient to exercise the powers under Section 319 Cr.P.C. As against this
the learned counsel for the respondents supported the order and pointed out that there
were some observations made by the learned Sessions Judge in his judgment while
acquitting the other accused persons. On these rival submissions it is to be seen as to
whether the High Court was right in allowing the Revision and directing the Sessions
Judge to reconsider the application under Section 319 Cr.P.C. afresh.
9. The powers under Section 319 Cr.P.C. to proceed against any person who is not the
accused are couched in the following words :
"319. Power to proceed against other persons appearing to be guilty of offence. (1)
Where, in the course of any inquiry into, or trial of, an offence, it appears from the
evidence that any person not being the accused has committed any offence for which
such person could be tried together with the accused, the Court may proceed against such
person for the offence which he appears to have committed.
(2) Where such person is not attending the court he may be arrested or summoned, as the
circumstances of the case may require, for the purpose aforesaid.
(3) Any person attending the court although not under arrest or upon a summons, may be
detained by such court for the purpose of the inquiry into, or trial of
@page-SC1567
the offence which he appears to have committed.
(4) Where the court proceeds against any person under sub-section (1) then -
(a) the proceedings in respect of such person shall be commenced afresh, and witnesses
reheard;
(b) subject to the provisions of clause (a), the case may proceed as if such person had
been an accused person when the court took cognizance of the offence upon which the
inquiry or trial was commenced."
A glance at these provisions would suggest that during the trial it has to appear from the
evidence that a person not being an accused has committed any offence for which such
person could be tried together with the accused who are also being tried. The key words
in this Section are "it appears from the evidence".."any person"...... "has committed any
offence". It is not, therefore, that merely because some witnesses have mentioned the
name of such person or that there is some material against that person, the discretion
under Section 319 Cr.P.C. would be used by the court. This is apart from the fact that
such person against whom such discretion is used, should be a person who could be tried
together with the accused against whom the trial is already going on. This Court has, time
and again, declared that the discretion under Section 319 Cr.P.C. has to be exercised very
sparingly and with caution and only when the concerned court is satisfied that some
offence has been committed by such person. This power has to be essentially exercised
only on the basis of the evidence. It could, therefore, be used only after the legal evidence
comes on record and from that evidence it appears that the concerned person has
committed an offence. The words "it appears" are not to be read lightly. In that the court
would have to be circumspect while exercising this power and would have to apply the
caution which the language of the Section demands.
10

. In a reported decision in Mohd. Shafi v. Mohd. Rafiq and Anr. [JT 2007 (5) SC 562], to
which one of us (Sinha, J.) was a party, this Court had observed in para 7 as under :
2007 AIR SCW 3399

"Before, thus, a trial court seeks to take recourse to the said provision, the requisite
ingredients therefore must be fulfilled. Commission of an offence by a person not facing
trial, must, therefore, appear to the court concerned. It cannot be ipse dixit on the part of
the court. Discretion in this behalf must be judicially exercised. It is incumbent that the
court must arrive at its satisfaction in this behalf."

In the above case this Court referred to the decision reported in Municipal Corporation of
Delhi v. Ram Krishan Rohtagi and Ors. [(1983) 1 SCC 1] and highlighted the following
remarks made in para 19 therein which are to the following effect : AIR 1983 SC 67

"19.........But, we would hasten to add that this is really an extraordinary power which is
conferred on the court and should be used very sparingly and only if compelling reasons
exist for taking cognizance against the other person against whom action has not been
taken......."
It was further stated in para 13 :
".......it is evident that before a court exercises its discretionary jurisdiction in terms of
Section 319 of the Code of Criminal Procedure, it must arrive at the satisfaction that there
exists a possibility that the accused so summoned in all likelihood would be convicted.
Such satisfaction can be arrived at inter alia upon completion of the cross-examination of
the said witness. For the said purpose, the court concerned may also like to consider other
evidence." (Emphasis supplied).
11

. In Krishnappa v. State of Karnataka [(2004) 7 SCC 792] this Court, while relying on
another reported decision in Michael Machado v. Central Bureau of Investigation [(2000)
3 SCC 262] went on to hold that the power under Section 319, Cr.P.C. is discretionary
and should be exercised only to achieve criminal Justice and that the court should not
turn against another person whenever it comes across evidence connecting that other
person also with the offence. The Court further observed : 2004 AIR SCW 4809
2000 AIR SCW 734

"..........a judicial exercise is called for, keeping in conspectus of the case, including the
stage at which the trial has already proceeded with the quantum of evidence collected till
then, and also the amount of time which the court had spent for collecting such
evidence......"
The Court further observed :
@page-SC1568
"The Court, while examining an application under Section 319 Cr.P.C., has also to bear in
mind that there is no compelling duty on the court to proceed against other persons. In a
nutshell, it means that for exercise of discretion under Section 319 Cr.P.C., all relevant
factors, including the one noticed above, have to be kept in view and an order is not
required to be made mechanically merely on the ground that some evidence had come on
record implicating the person sought to be added as an accused."
12. Turning to the present case, we find that the Trial Court had properly considered the
evidence of injured Rohitas, Ram Singh, Ramavtar and Maya Devi and had found that
none of the witnesses had stated that Kailash was having an axe in his hand and that he
caused any injury to Rohitas with the axe. In that the court found that the witnesses had
improved their version only at the stage of trial. The court even went to the extent of
saying that even if Kailash was presumed to be present at the spot, that by itself could not
prove that he took part in the assault. The Trial Court had also very specifically noted the
improvement made by Rohitas (PW-1) in stating that Kailash inflicted axe blow on his
hand and head which claim was belied by the medical report recording his injuries. Same
was the situation regarding the evidence of Ramavatar (PW3) as also Maya (PW4). The
trial court found that all these witnesses were giving improved versions during their
evidence in the court. Thus, it was clear that the Trial Court had come to a conclusion that
there was no possibility of convicting Kailash, the present appellant on the basis of the
evidence led before it.
13

. On this backdrop when we see the order passed by the High Court, there does not
appear to be any such effort on the part of the High Court. Basically, the High Court
merely relied on the reported decision in Shashi Kant Singh v. Tarkeshwar Singh and Anr.
[JT 2002 (4) SC 386] where the question was entirely different. There the question was as
to whether if the trial itself was over, could the revisional court direct the said person
against whom the Trial Court had refused to exercise discretion under Section 319 Cr.P.C.
to be tried afresh. In the present case also the conclusion of the trial was irrelevant in so
far as the trial of the appellant is concerned. That by itself was no reason to try him with
the aid of Section 319 Cr.P.C. The High Court should have applied itself independently to
the question as to whether there was any material in the evidence not only to connect the
appellant but whether it was sufficient to justify the words "it appears that such person
has committed the crime". We do not see any such effort in the judgment of the High
Court. On the other hand, the High Court has commented on the language of the
judgment by the Trial Court while acquitting the other accused. That is an irrelevant
consideration. Merely because the Sessions Judge commented upon the present appellant
not being a party accused, that by itself did not justify the interference that there was
evidence against him and the evidence was of such nature as would justify his being
added as an accused much less under Section 319 Cr.P.C. We are, therefore, quite
convinced that the judgment of the High Court is erroneous and must be set aside. 2002
AIR SCW 2079

14. In the result the appeal is allowed, the judgment of the High Court is set aside and the
judgment of the Trial Court is restored.
Appeal allowed.
AIR 2008 SUPREME COURT 1568 "Silvey v. Arun Varghese"
(From : Kerala)
Coram : 2 Dr. A. PASAYAT AND P. SATHASIVAM, JJ.
Civil Appeal No. 830 of 2002, D/- 26 -2 -2008.
Silvey and Ors. v. Arun Varghese and Anr.
Specific Relief Act (47 of 1963), S.16 - CONTRACT - AGREEMENT TO SELL -
DECREE - Suit for specific performance - Agreement to sell - Suit by purchaser -
Agreement casting obligation on vendors to get property registered as Rubber Estate and
get licence for planting rubber plants - Plaintiff always ready and willing to perform his
part - Vendors-defendants, however, have failed to obtain registration and licence from
Rubber Board - False plea also raised by defendants in written statement - Plaintiff
entitled to decree for specific performance. (Paras 11, 12, 13)
Cases Referred : Chronological Paras
1996 AIR SCW 3603 : AIR 1996 SC 2814 (Rel. on) 13
(1980) 2 All ER 145 9
AIR 1928 PC 208 8
@page-SC1569

T.L. Vishwanatha Iyer, Sr. Advocate and T.G. Narayanan Nair, for Appellants; P.
Krishnamoorthy, Sr. Advocate and Romy Chacko, for Respondents.
Judgement
1. Dr. ARIJIT PASAYAT, J. :-Challenge in this appeal is to the judgment of a Division
Bench of the Kerala High Court allowing the appeal filed by the respondents who were
the plaintiffs in a suit for specific performance of an agreement to sell immovable
properties.
2. The plaintiffs in a suit for specific performance of an agreement to sell immovable
properties are the appellants. The defendants, four in number entered into an agreement
for sale of the respective portions held by them under Exhibit A 1 dated 23-2-1986.
Thereunder, they agreed to convey to the plaintiffs an extent of 10 acres of property held
by the four of them at a price of Rs. 19,750/- per acre. They received an advance of Rs.
50,000/-. The agreement provided that the sale deed was to be executed by 17-4-1986.
The defendants were to furnish the plaintiffs with all documents in their possession and
power and also furnish tax receipts for taxes paid up-to-date, the registration book for
registration as a Rubber Estate with the Rubber Board and the licence for planting rubber
plants, get the properties measured by competent persons at the expense of the vendors-
defendants in the presence of the plaintiffs-purchasers or their agents. According to the
plaintiffs, the defendants were never ready with the requisite documents for executing the
sale deed in favour of the plaintiffs and repeated demands by the plaintiffs for execution
of the document did not meet with the proper response and, therefore, they had ultimately
to send a notice through a lawyer on 16-3-1988 demanding performance and the
defendants having failed to respond to that notice, the suit was being filed on 5-7-1988
for specific performance of the agreement for sale. The plaintiffs pleaded that they were
and they have always been ready and willing to perform their part of the contract. The
defendants filed a written statement contending that the plaintiffs were in default. The
plaintiffs never demanded performance of the agreement. The defendants, therefore, were
under the impression that the plaintiffs have abandoned the agreement. The defendants
have, therefore, effected improvements in the property and the value of the property has
increased. It was, therefore, not a fit case where a decree for specific performance should
be granted to the plaintiffs. It was pleaded that the plaintiffs were never ready and willing
to perform their part of the contract.
3. The trial Court held on the materials that time was not the essence of the contract and
that the plaintiffs had the requisite capacity to raise funds for the purchase of the property
covered by Exhibit A 1. But, the trial Court held that plaintiffs had not taken prompt steps
for enforcement of the obligations under Exhibit A 1 agreement for sale. But, it did find
that the defendants had not yet obtained the certificates of registration from the Rubber
Board as envisaged by the agreement for sale and they had not even obtained possession
certificates, since obviously there was considerable dispute about the properties covered
in the survey number, of which the plaint schedule properties formed a part. The trial
Court stating that in view of the delay in the plaintiffs approaching the Court, the
plaintiffs have not shown themselves to be ready and willing to perform their part of the
contract and in the matter of exercise of discretion, specific performance should be
refused to the plaintiffs since the defendants have, on the basis that the plaintiffs were not
any more interested purchasing the property, expended amounts for improvement of the
property. Thus staing that the discretion has to be exercised against the plaintiffs, the trial
Court dismissed the suit for specific performance. But the trial Court granted a decree for
recovery of the advance of Rs. 50,000/-paid by the plaintiffs to the defendants at the time
of entering into Exh. A1 agreement along with interest at the rate of 6% per annum
thereon from the date of suit till date of realization. It is feeling aggrieved by the refusal
to grant the plaintiffs a decree for specific performance that the plaintiffs have filed
appeal before the High Court.
4. In appeal, the High Court reversed the judgment and decree of the trial Court and held
that the suit was to be decreed in favour of the plaintiffs for specific performance as
prayed for.
5. Learned counsel for the appellants submitted that though time may not be the essence
of an agreement but circumstances can show that it was really so. Though extension was
granted, that was not for very long period. Though originally it was stipulated in the
agreement that the sale was to
@page-SC1570
be completed before off set of monsoon, the last extension granted was for a short period
thereafter. There was no extension after 17-4-1986. For long time no demand was made
by the plaintiffs. The plaintiffs were really not interested for executing the sale deed as
they were waiting to see whether the intended purchases of the neighbouring land would
be completed. There was no material to show that at all relevant points of time, the
plaintiffs were ready and willing to perform their part of the agreement. Reference is
made to the evidence of PW 1 to show that the purchase of the agreed land depended
upon acquisition of the neighbouring land.
6. It is submitted that there must be material to show about the readiness and willingness
throughout, even though a person may have funds or is capable of raising funds. These
aspects have been lost sight of by the High Court.
7. Learned counsel for the respondents on the other hand submitted that though in a given
case even in respect of an agreement for sale of immovable property, time may be the
essence of agreement yet it would depend upon several factors. If the circumstances show
that the time was the essence of the agreement that fact can also be taken note of. In the
instant case, it is submitted that the defendants themselves have accepted that the time
was extended and the agreement was to be given effect to before monsoon set in. But the
period extend itself to a period which was admitted after monsoon had set in. A false plea
was taken by the defendants about the plaintiffs having told them to have abandoned the
agreement. This conduct itself disentitled the defendants from opposing the suit for
specific performance of contract.
8. The High Court has found that plaintiffs can be said to have been always ready and
willing to perform their part of the contract from the inception of the contract to the date
of the decree of the trial Court, relying on the principles highlighted in Ardeshir H. Mama
v. Flora Sassoon (AIR 1928 PC 208).
9. In Raineri v. Miles and Anr. (1980 (2) All ER 145 at page 155) it was held as follows :
"In the instant case the date for completion was not expressed to be of the essence, and it
has not been suggested (though I think it might possibly have been) that the surrounding
circumstances nevertheless so rendered it. In that state of affairs the appellants submit
that the law as it has stood ever since 1875 exculpated them from all liability for the
foreseeable damage sustained by the respondents as a direct result of their failure to keep
their word. My Lords, were this indeed right the respondents would suffer a substantial
injustice. The fact that time had not been declared to be of the essence does not mean that
the express date for completion could be supplanted by the Court's treating it as a mere
'target' date and, in effect, enabling the defaulting party to insert into the contractual
provision some such words as'.....or within a reasonable time thereafter."
10. As rightly noted by the High Court, the plaintiffs have pleaded in terms of Section 16-
C of the Specific Relief Act, 1963 (in short the 'Act') that they have alsways been and are
ready and willing to perform their part of the contract. Plaintiff No. 2 as PW 1 has also
spoken about this fact. The case of the plaintiffs was that the defendants were not ready
with the documents as contemplated in clause 2 of Exh. A1 which resulted in the delay in
the performance of the contract and in the plaintiffs seeking the performance of the
contract by the defendants.
11. The High Court has noticed that the agreement in respect of the adjacent land was
entered into much before the agreement in question. It has also been noticed by the High
Court that there was no impediment on the plaintiffs obtaining a sale deed in respect of
adjacent land or that they apprehended it at any point of time that they were not going to
get an assignment to that extent. The assignment in fact was obtained in respect of the
adjacent land. The High Court also higlighted, in our opinion rightly, that the defendants
had not performed their part of the contract under Exh. A1 as they had not obtained
requisite licence for planting rubber plant, except in case of defendant No. 3. None of the
other defendants had obtained the registration book for registration as a rubber estate with
the Rubber Board as envisagd by Clause 7 of the Agreement for sale.
12. DW1 accepted that possession certificates could not be obtained by the defendants in
view of the nature of the property involved in the context of Kerala Land Reforms Act,
and the Kerala Private Forest
@page-SC1571
(Vesting and Assignment) Act. The defendants never responded to the letter Exh. A2
issued by the plaintiffs seeking performance of the contract. No response was also sent to
the letters Exh. A2 to A10. Exh. A6 was a letter sent through registered post which was
refused. The lawyer's notice Exh. All was also not responded to.
13

. As regards the false plea of the defendants, the effect needs to be noted. It was pleaded
that defendant No. 3 had gone to the house of plaintiff No. 2 in Alleppey prior to the
receiving any letter from the plaintiffs and had spoken that they had told him that they
were not keen in enforcing the application under Exh. Al. But when examined as DW1,
the said defendant No. 3 admitted that he had never met the plaintiff as pleaded in the
written statement and that he or any other defendant had never gone to Alleppey to meet
plaintiff No, 2 at his residence to speak about the performance of the contract. The plea
stated in the written statement was abandoned in evidence. In Lourdu Mari David and
Ors. v. Louis Chinnaya Arogiaswamy and Ors. (1996 (5) SCC 589), it was noted that the
conduct of the defendant cannot be ignored while weighing the question of exercise of
discretion for decreeing or denying a decree for specific performance. The High Court
has, after analyzing the factual position, come to the conclusion that the defendants were
really not ready to perform their obligation in terms of the contract and had taken a false
plea in the written statement. 1996 AIR SCW 3603

14. The appeal is without merit, deserves dismissal, which we direct, but in the
circumstance without any order as to costs.
Appeal dismissed.
AIR 2008 SUPREME COURT 1571 "Shantabai v. State of Maharashtra"
(From : Bombay)
Coram : 2 PRAKASH PRABHAKAR NAOLEKAR AND LOKESHWAR SINGH
PANTA, JJ.
Criminal Appeal No. 372 of 2006, D/- 3 -3 -2008.
Shantabai and Ors. v. State of Maharashtra.
Penal Code (45 of 1860), S.300 - MURDER - EVIDENCE - WITNESS - Murder -
Circumstantial evidence - Accused, a husband, wife and their major son - Wife alleged to
be having illicit relations with deceased from 10-15 years - Evidence of selective and
interrogated witnesses, not reliable as other residents of same village have not uttered a
word in that regard - Fact that dead body of deceased found in open space in front of their
house, which is public road - Not sufficient to connect accused with commission of death
of deceased - Evidence to prove prosecution version, that accused left their house after
committing death of deceased, not believable - Prosecution failed to establish that
accused had used recovered weapons of offence in commission of crime - Blood group
on seized clothes of accused did not tally with blood, which was found on clothes of
deceased and on sample of soil, axe. stones, handles etc. - Held, chain of circumstances
against accused not made complete by prosecution - Accused entitled to acquittal.
Cri. Appeal No. 58 of 1995, D/-27-06-2005 (Bom), Reversed. (Paras 19, 20, 21, 22,
23)
Cases Referred : Chronological Paras
2004 AIR SCW 819 : AIR 2004 SC 1492 : 2004 Cri LJ 1394 (Ref.) 15
1996 AIR SCW 2903 : AIR 1996 SC 3390 : 1996 Cri LJ 3461 (Ref.) 14
AIR 1987 SC 350 : 1987 Cri LJ 330 (Ref.) 11
AIR 1987 SC 1921 : 1987 Cri LJ 1918 (Ref.) 11
AIR 1984 SC 1622 : 1984 Cri LJ 1738 (Ref.) 13
AIR 1983 SC 61 : 1983 Cri LJ 155 (Ref.) 11
AIR 1983 SC 446 : 1983 Cri LJ 846 (Ref.) 11
AIR 1982 SC 1157 : 1982 Cri LJ 1243 (Ref.) 11
AIR 1981 SC 738 : 1981 Cri LJ 298 (Ref.) 11
AIR 1952 SC 343 : 1953 Cri LJ 129 (Ref.) 12
Sudhanshu S. Choudhari (for Naresh Kumar), for Appellants; Manish Pitale (for
Ravindra Keshavrao Adsure), for Respondent.
Judgement
1. LOKESHWAR SINGH PANTA, J. :-The appellants Shantabai (A-l), Sajan (A-2) and
Govind (A-3) have filed this appeal against the judgment and order dated 27-06-2005
passed by the Division Bench of the High Court of Judicature at Bombay, Bench at
Aurangabad, in Criminal Appeal
@page-SC1572
No. 58 of 1995 confirming the conviction and sentence for life in respect of the offence
punishable under Section 302 of the Indian Penal Code read with Section 34 of the Indian
Penal Code [for short "the IPC"] and a fine of Rs. 2,000/- each with default clause to
undergo R.I. for six months awarded by the learned Additional Sessions Judge, Biloli, in
Sessions Case No. 160/1993.
2. In all five accused persons were tried by the learned Additional Sessions Judge, Biloli,
under Sections 147, 148 and 302, IPC, read with Section 149, IPC. A-1, A-2 and A-3
were found guilty of the murder of Gunwant Nivrati Dhumale, while Venkar (A-4) and
Anshabai (A-5) were acquitted of the charges framed against them.
3. Briefly stated, the case of the prosecution against the accused persons was that A-1, A-
2 and A-3 are residents of village Loni and A-4 and A-5 are residents of village Shilvani.
A-l is the wife of A-2 and A

Vous aimerez peut-être aussi